general studies (paper i) · test is part of rau’s ias test series for preliminary exam 2019...

49
Test is part of Rau’s IAS Test series for Preliminary Exam 2019 FOUNDATION + CURRENT AFFAIRS GENERAL STUDIES (PAPER –I) FOUNDATION TEST –III TOPIC: NCERT History Class VI-X + Current Affairs Time Allowed: 1½ Hours Maximum Marks : 150 vuqns'k 1- Ikjh{kk Ikz kjEHk gksus ds rqjUr ckn] vki bl ijh{k.k iqfLrdk dh iM+rky vo'; dj ysa fd bles a dks bZ fcuk Nik] iQVk ;k NwVk gqvk i` "B vFkok Ikz 'uka'k] vkfn u gksA ;fn ,slk gS ] rks bls lgh ijh{k.k iqfLrdk ls cny yhft,A 2- bl ijh{k.k iqfLrdk esa 75 iz 'uka 'k (iz'u) fn, x, gS aA iz R;sd iz 'uka 'k fgUnh vkS j vaxz sth nks uks a es a Nik gS iz R;sd iz 'uka'k esa pkj iz R;qÙkj (mÙkj) fn, x, gS aA buesa ls ,d izR;qÙkj dks pq u ys a] ftls vki mÙkj&i=kd ij vafdr djuk pkgrs gS aA ;fn vkidks ,slk yxs fd ,d ls vf/d iz R;qÙkj lgh gS a] rks ml iz R;qÙkj dks vafdr djsa tks vkidks loksZRre yxsA iz R;sd iz 'uka 'k ds fy, dsoy ,d gh iz R;qÙkj pq uuk gS A 3- vkidks vius lHkh iz R;qÙkj vyx ls fn, x, mÙkj&i=kd ij gh vafdr djus gS aA mÙkj&i=kd esa fn, x, funsZ 'k nsf[k,A 4- lHkh iz 'uka'kks a ds vad leku gSaA 5- blls igys fd vki ijh{k.k iqfLrdk ds fofHkUu iz 'uka 'kks a ds iz R;qÙkj mÙkj&i=kd ij vafdr djuk 'kq: djs a] vkidks iz os'k iz ek.k&i=k ds lkFk iz sf"kr vuqns'kksa ds vuqlkj dqN fooj.k mÙkj&i=kd esa nsus gS aA 6- vki vius lHkh iz R;qÙkjksa dks mÙkj&i=kd esa Hkjus ds ckn rFkk ijh{kk ds lekiu Ikj dsoy mÙkj&i=kd v/h{kd dks lkS ai nsaA vkidks vius lkFk ijh{k.k iqfLrdk ys tkus dh vuqefr gS A 7- xyr mÙkjksa ds fy, n.M % Ikz 'u Ik=k esa mEehnokj }kjk fn, x, xyr mÙkjksa ds fy, naM fn;k tk,xkA (i) Ikz R;sd iz 'u ds fy, pkj oS dfYid mÙkj gS aA mEehnokj }kjk iz R;sd ml iz 'u ds fy,] ftlds fy, naM gS ] fn, x, ,d xyr mÙkj ds fy, iz 'u gsrq fu;r fd, x, vadks a dk ,d&frgkbZ naM ds :i esa dkVk tk,xkA (ii) ;fn dks bZ mEehnokj ,d ls vf/d mÙkj nsrk gS ] rks bls xyr mÙkj ekuk tk,xk] ;|fi fn, x, mÙkjks a ls ,d mÙkj lgh gksrk gS ] fiQj Hkh ml iz 'u ds fy, mi;q Z Drkuqlkj gh] mlh rjg dk naM fn;k tk,xk] ;fn mlds fy, naM gS A (iii) ;fn mEehnokj }kjk dksbZ iz 'u gy ugha fd;k tkrk gS ] vFkkZ r~ mEehnokj }kjk mÙkj ugha fn;k tkrk gS ] rks ml iz 'u ds fy, dks bZ naM ugha fn;k tk,xkA This test is part of Rau’s IAS Test series for Preliminary Exam 2019 Test Code FC19E1003

Upload: others

Post on 29-Jan-2020

0 views

Category:

Documents


0 download

TRANSCRIPT

Page 1: GENERAL STUDIES (PAPER I) · Test is part of Rau’s IAS Test series for Preliminary Exam 2019 FOUNDATION + CURRENT AFFAIRS GENERAL STUDIES (PAPER –I) FOUNDATION TEST –III TOPIC:

T e s t i s p a r t o f R a u rsquo s I A S T e s t s e r i e s f o r P r e l i m i n a r y E x a m 2 0 1 9

FOUNDATION + CURRENT AFFAIRS

GENERAL STUDIES (PAPER ndashI)

FOUNDATION TEST ndashIII

TOPIC NCERT History Class VI-X + Current Affairs

Time Allowed 1frac12 Hours Maximum Marks 150

vuqnsk

1- Ikjhkk IkzkjEHk gksus ds rqjUr ckn] vki bl ijhkk iqfLrdk dh iM+rky vo dj ysa fd blesa dksbZ fcuk Nik]

iQVk k NwVk gqvk iB vFkok Ikzukak] vkfn u gksA fn slk gS] rks bls lgh ijhkk iqfLrdk ls cny yhftA

2- bl ijhkk iqfLrdk esa 75 izukak (izu) fn x gSaA izRsd izukak fgUnh vkSj vaxzsth nksuksa esa Nik gS izRsd

izukak esa pkj izRqUgravekj (mUgravekj) fn x gSaA buesa ls d izRqUgravekj dks pqu ysa] ftls vki mUgravekjampi=kd ij vafdr djuk

pkgrs gSaA fn vkidks slk yxs fd d ls vfd izRqUgravekj lgh gSa] rks ml izRqUgravekj dks vafdr djsa tks vkidks

loksZRre yxsA izRsd izukak ds fy dsoy d gh izRqUgravekj pquuk gSA

3- vkidks vius lHkh izRqUgravekj vyx ls fn x mUgravekjampi=kd ij gh vafdr djus gSaA mUgravekjampi=kd esa fn x funsZk

nsf[kA

4- lHkh izukakksa ds vad leku gSaA

5- blls igys fd vki ijhkk iqfLrdk ds fofHkUu izukakksa ds izRqUgravekj mUgravekjampi=kd ij vafdr djuk kq djsa] vkidks

izosk izekkampi=k ds lkFk izsfkr vuqnskksa ds vuqlkj dqN foojk mUgravekjampi=kd esa nsus gSaA

6- vki vius lHkh izRqUgravekjksa dks mUgravekjampi=kd esa Hkjus ds ckn rFkk ijhkk ds lekiu Ikj dsoy mUgravekjampi=kd vhkd dks

lkSai nsaA vkidks vius lkFk ijhkk iqfLrdk ys tkus dh vuqefr gSA

7- xyr mUgravekjksa ds fy nM

Ikzu Ik=k esa mEehnokj kjk fn x xyr mUgravekjksa ds fy naM fnk tkxkA

(i) IkzRsd izu ds fy pkj oSdfYid mUgravekj gSaA mEehnokj kjk izRsd ml izu ds fy] ftlds fy naM gS]

fn x d xyr mUgravekj ds fy izu gsrq fur fd x vadksa dk dampfrgkbZ naM ds i esa dkVk tkxkA

(ii) fn dksbZ mEehnokj d ls vfd mUgravekj nsrk gS] rks bls xyr mUgravekj ekuk tkxk] |fi fn x mUgravekjksa ls d

mUgravekj lgh gksrk gS] fiQj Hkh ml izu ds fy miqZDrkuqlkj gh] mlh rjg dk naM fnk tkxk] fn mlds

fy naM gSA

(iii) fn mEehnokj kjk dksbZ izu gy ugha fdk tkrk gS] vFkkZr~ mEehnokj kjk mUgravekj ugha fnk tkrk gS] rks ml

izu ds fy dksbZ naM ugha fnk tkxkA

T h i s t e s t i s p a r t o f R a u rsquo s I A S T e s t s e r i e s f o r P r e l i m i n a r y E x a m 2 0 1 9

Test Code

FC19E1003

RAUSIAS-FC19E1003 2

Q1) निमननिखित कथि ो म स कौि-सास सही हह

1 ऋगवद म कछ छो द मनहिाओो क दवारा रच गए

2 निशवानमतर िनदक काि म एक महतवपरण ऋनि

िीच नदए गए कट का परय ग कर सही उततर चनिए

(a) किि 1

(b) किि 2

(c) 1 और 2 द ि ो

(d) ि त 1 ि ही 2

Q2) निमननिखित म स कौि-स सथि स अनि क उपय ग क

साकषय परापत हए ह

(a) भीमबटका गफाएो

(b) करिि गफाएो

(c) एनिफ टा गफाएो

(d) किहरी गफाएो

Q3) निमननिखित म स नकस सथि स भनमगत गडढ क घर ो

(Pit Houses) क साकषय पाए गए ह

(a) नचराोद

(b) हललर

(c) बरणह म

(d) महागारा

Q4) पराचीि ििि पररानिय ो का अधययि और ऐनतहानसक

पाोडनिनपय ो क समझि तथा नतनथ निराणरर क

निमननिखित म स कया कहा जाता ह

(a) परािि-निदया (Epigraphy)

(b) पाोडनिनप निजञाि (Manuscriptology)

(c) पिीओगराफी (Palaeography)

(d) नयनमजमनटकस (Numismatics)

Q5) निमननिखित यग ो पर निचार कीनजए

1 िौह उपकरर महापािानरक कबरगाह

2 चरक सोनहता गनरत की पसतक

उपयणकततत यग ो म स कौि-सास सही समनित हह

(a) किि 1

(b) किि 2

(c) 1 और 2 द ि ो

(d) ि त 1 ि ही 2

Q6) निमननिखित यग ो पर निचार कीनजए

1 भाग भनम पर कर

2 कममकार भनमहीि कनि मजदर

3 अशवमर अनय शासक ो क कषतर ो म घमि क

निए छ ड़ा गया एक घ ड़ा (अशव)

उपयणकत यग ो म स कौि-स सही समनित ह

(a) किि 1 और 2

(b) किि 2 और 3

(c) किि 1 और 3

(d) 1 2 और 3

Q7) निमननिखित कथि ो पर निचार कीनजए

1 घ ड़ ो क दवारा िी ोच जाि िाि रथ ो का उपय ग

महाजिपद काि क दौराि शर हआ था

2 नियनमत सथायी सिाए रिि की शरआत

ऋगवनदक शासक ो क दवारा की गई थी

उपयणकत कथि ो म स कौि-सास सही हह

(a) किि 1

(b) किि 2

(c) 1 और 2 द ि ो

(d) ि त 1 ि ही 2

RAUSIAS-FC19E1003 3

Q1) Which of the following statements

isare correct

1 Some verses in Rigveda were

composed by women

2 Vishwamitra was an important

sage in the Vedic period

Select the correct answer using the code

given below

(a) 1 only

(b) 2 only

(c) Both 1 and 2

(d) Neither 1 nor 2

Q2) Which of the following sites yields traces

of the use of fire

(a) Bhimbetka Caves

(b) Kurnool Caves

(c) Elephanta Caves

(d) Kanheri Caves

Q3) Which of the following sites has

evidences of underground pit houses

(a) Chirand

(b) Hallur

(c) Burzahom

(d) Mahagara

Q4) The study of ancient writing systems

and the deciphering and dating of

historical manuscripts is known as-

(a) Epigraphy

(b) Manuscriptology

(c) Palaeography

(d) Numismatics

Q5) Consider the following pairs

1 Iron implements Megalithic

burials

2 Charaka Samhita Book on

Mathematics

Which of the pairs given above isare

correct

(a) 1 only

(b) 2 only

(c) Both 1 and 2

(d) Neither 1 nor 2

Q6) Consider the following pairs

1 Bhaga Tax on land

2 Kammakaras Landless

Agricultural Labourers

3 Ashvamedha A horse is let loose

to wander into the territories of

other rulers

Which of the pairs given above isare

correct

(a) 1 and 2

(b) 2 and 3

(c) 1 and 3

(d) 1 2 and 3

Q7) Consider the following statements

1 The use of Horse drawn chariots

began during the Mahajanapada

period

2 Regular standing armies were

introduced by Rigvedic rulers

Which of the statements given above

isare correct

(a) 1 only

(b) 2 only

(c) Both 1 and 2

(d) Neither 1 nor 2

RAUSIAS-FC19E1003 4

Q8) बदध क जीिि क सोदभण म निमननिखित कथि ो पर

निचार कीनजए

1 बदध ि िारारसी क पास सारिाथ म अपिा

पहिा उपदश नदया था

2 ि निचछिी कि स सोबोनरत थ और कशीिारा म

उिका निरि हआ था

उपयणकत कथि ो म स कौि-सास सही हह

(a) किि 1

(b) किि 2

(c) 1 और 2 द ि ो

(d) ि त 1 ि ही 2

Q9) निमननिखित यग ो पर निचार कीनजए

1 समखया कनपि

2 िशनिक जानमिी

3 नयाय गौतम

उपयणकत यग ो म स कौि-स सही समनित ह

(a) किि 1 और 2

(b) किि 2 और 3

(c) किि 1 और 3

(d) 1 2 और 3

Q10) निमननिखित कथि ो पर निचार कीनजए

1 महािीर की नशकषाऐो िशािी म सोकनित की गई

थी ो

2 नििय नपटक बौदध सोघ क बार म ह

उपयणकत कथि ो म स कौि-सास सही हह

(a) किि 1

(b) किि 2

(c) 1 और 2 द ि ो

(d) ि त 1 ि ही 2

Q11) निमननिखित कथि ो पर निचार कीनजए

1 अथणशासतर का सोकिि निषणगपत ि नकया था

2 मगसथिीस ज चनदरगपत मौयण क दरबार म

आया था सलयकस का राजदत था

उपयणकत कथि ो म स कौि-सास सही हह

(a) किि 1

(b) किि 2

(c) 1 और 2 द ि ो

(d) ि त 1 ि ही 2

Q12) निमननिखित यग ो पर निचार कीनजए

सतभ कपिटल सथान

1 नसोह कनपटि सारिाथ

2 बि कनपटि रामपिण

3 हाथी कनपटि सोनकससा

उपयणकत यग ो म स कौि-सास सही समनित हह

(a) किि 1 और 2

(b) किि 3

(c) किि 1 और 3

(d) 1 2 और 3

Q13) 1857 क नसपाही निदर ह क सोदभण म निमननिखित

कथि ो म स कौि-सास सही हह

1 अहमदललाह शाह फजाबाद क मौििी ि

अोगरज ो क निरदध ि ग ो क एकनतरत नकया

2 किर नसोह ज एक महाि य दधा थ उततर परदश

स सोबोनरत थ

िीच नदए गए कट का परय ग कर सही उततर चनिए

(a) किि 1

(b) किि 2

(c) 1 और 2 द ि ो

(d) ि त 1 ि ही 2

RAUSIAS-FC19E1003 5

Q8) In the context of the life of Buddha

consider the following statements

1 Buddha gave his first sermon at

Sarnath near Varanasi

2 He belonged to the Licchavi clan

and passed away at Kusinara

Which of the above statements isare

correct

(a) 1 only

(b) 2 only

(c) Both 1 and 2

(d) Neither 1 nor 2

Q9) Consider the following pairs

1 Samkhya Kapila

2 Vaisheshika Jamini

3 Nyaya Gautama

Which of the pairs given above isare

correctly matched

(a) 1 and 2 only

(b) 2 and 3 only

(c) 1 and 3 only

(d) 1 2 and 3

Q10) Consider the following statements

1 The teachings of Mahavira were

compiled at Vaishali

2 Vinaya Pitaka talks about the

Buddhist Sangha

Which of the statements given above

isare correct

(a) 1 only

(b) 2 only

(c) Both 1 and 2

(d) Neither 1and 2

Q11) Consider the following statements

1 Arthashastra was compiled by

Vishnugupta

2 Magasthenes who came to the

court of Chandragupta Maurya

was the ambassador of Seleucus

Which of the statements given above

isare correct

(a) 1 only

(b) 2 only

(c) Both 1 and 2

(d) Neither 1 nor 2

Q12) Consider the following pairs

Pillar Capital Place

1 Lion Capital Sarnath

2 Bull Capital Rampurva

3 Elephant Capital Sankissa

Which of the pairs given above isare

matched correctly

(a) 1 and 2

(b) 3 only

(c) 1 and 3

(d) 1 2 and 3

Q13) Consider the following statements

regarding the Revolt of 1857

1 Ahmadullah Shah a maulvi from

Faizabad caught the imagination

of the people and raised a huge

force of supporters against the

Britishers

2 Kunwar Singh was a great fighter

from Uttar Pradesh

Which of the statements given above

isare correct

(a) 1 only

(b) 2 only

(c) Both 1 and 2

(d) Neither 1 nor 2

RAUSIAS-FC19E1003 6

Q14) निमननिखित यग ो पर निचार कीनजए

1 िललािर सनिक

2 आनदमाई दास (गिाम)

3 कदनसयर भनमहीि मजदर

उपयणकत यग ो म स कौि-स सही समनित ह

(a) किि 1 और 2

(b) किि 2 और 3

(c) किि 1 और 3

(d) 1 2 और 3

Q15) निमननिखित सथि ो म स उस सथि की पहचाि कीनजए

ज एक परनसदध तटीय बसती थी और जहा एरटाइि

(Arretine) मदभाोड पाया गया था

(a) महाबिीपरम

(b) तामरनिखपत

(c) अररकमड

(d) अतरोजीिरा

Q16) सोगम सानहतय क सोदभण म निमननिखित कथि ो पर

निचार कीनजए

1 मदरई म सोगमसभाए आय नजत की जाती ो

थी ो

2 मिनदर शबद का परय ग सोगम कनिताओो म

नकया जाता था नजसका अथण ह द राजाओो

का एक समह

उपयणकत कथि ो म स कौि-सास सही हह

(a) किि 1

(b) किि 2

(c) 1 और 2 द ि ो

(d) ि त 1 ि ही 2

Q17) निमननिखित कथि ो पर निचार कीनजए

1 आरखिक िनदक काि म मनहिाऐो सभाओो

और निदाथाओो म भाग िती थी ो

2 आरखिक िनदक काि म िनशषठ ज एक

पजारी ह ता था की एक महतवपरण भनमका

ह ती थी

उपयणकत कथि ो म स कौि-सास सही हह

(a) किि 1

(b) किि 2

(c) 1 और 2 द ि ो

(d) ि त 1 ि ही 2

Q18) निमननिखित कथि ो पर निचार कीनजए

1 किाणटक म जि रमण क निसतार का शरय

चनदरगपत मौयण क जाता ह

2 जि रमण ि ईशवर क अखसततव क मानयता दी ह

तथा िरण पररािी की भरतणिा की ह

उपयणकत कथि ो म स कौि-सास सही हह

(a) किि 1

(b) किि 2

(c) 1 और 2 द ि ो

(d) ि त 1 ि ही 2

Q19) निमननिखित यग ो पर निचार कीनजए

1 पहार पाणडय

2 मदरई च ि

उपयणकत यग ो म स कौि-सास सही समनित हह

(a) किि 1

(b) किि 2

(c) 1 और 2 द ि ो

(d) ि त 1 ि ही 2

RAUSIAS-FC19E1003 7

Q14) Consider the following pairs

1 Vellalar Soldier

2 Adimai Slaves

3 Kadaisiyar Landless labourers

Which of the pairs given above isare

correctly matched

(a) 1 and 2 only

(b) 2 and 3 only

(c) 1 and 3 only

(d) 1 2 and 3

Q15) Identify the site which was a famous

coastal settlement from which Arretine

ware has been found

(a) Mahabalipuram

(b) Tamralipti

(c) Arikamedu

(d) Atranjikhera

Q16) With respect to the Sangam literature

consider the following statements

1 The Sangamsassemblies were

held at Madurai

2 The term muvendar was used in

sangam poems which mean a

group of two kings

Which of the statements given above

isare correct

(a) 1 only

(b) 2 only

(c) Both 1 and 2

(d) Neither 1 nor 2

Q17) Consider the following statements

1 During Early Vedic Period women

attended the Sabha and Vidatha

2 The priest ndash Vasishtha played

important role in Early Vedic

Period

Which of the statements given above

isare correct

(a) 1 only

(b) 2 only

(c) Both 1 and 2

(d) Neither 1 nor 2

Q18) Consider the following statements

1 The spread of Jainism in

Karnataka is attributed to

Chandragupta Maurya

2 Jainism recognised the existence

of the gods and condemns the

Varna System

Which of the statements given above

isare correct

(a) 1 only

(b) 2 only

(c) Both 1 and 2

(d) Neither 1 nor 2

Q19) Consider the following pairs

1 Puhar Pandyas

2 Madurai Cholas

Which of the pairs given above isare

correct

(a) 1 only

(b) 2 only

(c) Both 1 and 2

(d) Neither 1 nor 2

RAUSIAS-FC19E1003 8

Q20) निमननिखित कथि ो म स कौि-सास सही हह

1 बदधचररत का ििक िागसि ह

2 ब नरसतव की पजा महायाि बौदधमत का एक

महतवपरण भाग थी

िीच नदए गए कट का परय ग कर सही उततर चनिए

(a) किि 1

(b) किि 2

(c) 1 और 2 द ि ो

(d) ि त 1 ि ही 2

Q21) निमननिखित कथि ो म स कौि-सास सही हह

1 भखकत क निचार क भागित गीता म सपषट

नकया गया ह

2 तनमि कनि अपपर एक अििर सोत थ

िीच नदए गए कट का परय ग कर सही उततर चनिएः

(a) किि 1

(b) किि 2

(c) 1 और 2 द ि ो

(d) ि त 1 ि ही 2

Q22) निमननिखित म स नकस शासक ि िीरा बजात हए

अपिी छनि नसक ो पर अोनकत करिाई थी

(a) नमिाोदर

(b) चनदरगपत मौयण

(c) गौतमीपतर सतकरी

(d) समदरगपत

Q23) निमननिखित कथि ो म स कौि-सास सही हह

1 सवतोतर भारत ि निकरम सोित क राषटर ीय

किनडर क रप म अपिाया और यह 68 ईसा

पिण म आरि हआ था

2 बािभटट हिणिरणि क दरबार का एक कनि था

िीच नदए गए कट का परय ग कर सही उततर चनिएः

(a) किि 1

(b) किि 2

(c) 1 और 2 द ि ो

(d) ि त 1 ि ही 2

Q24) निमननिखित यग ो पर निचार कीनजएः

1 सोनर-निगरानहका वयापार मोतरी

2 परथम-कनिका परमि नशलपकार

3 साथणिाह परमि बकर

उपयणकत यग ो म स कौि-सास सही समनित हह

(a) किि 1

(b) किि 1 और 3

(c) किि 2

(d) किि 2 और 3

Q25) निमननिखित कथि ो म स कौि-सास सही हह

1 lsquoएह ि नशिाििrsquo पिकनशि नदवतीय स

सोबोनरत ह और रनिकनत क दवारा इसकी रचिा

की गई थी

2 जआि झाोग समदरगपत नदवतीय क शासिकाि

म भारत आया था

िीच नदए गए कट का परय ग कर सही उततर चनिएः

(a) किि 1

(b) किि 2

(c) 1 और 2 द ि ो

(d) ि त 1 ि ही 2

RAUSIAS-FC19E1003 9

Q20) Which of the following statements

isare correct

1 Buddhacharita is authored by

Nagasena

2 The worship of Bodhisattvas was

an important part of Mahayana

Buddhsim

Select the correct answer using the code

given below

(a) 1 only

(b) 2 only

(c) Both 1 and 2

(d) Neither 1 nor 2

Q21) Which of the following statements

isare correct

1 The idea of Bhakti is elucidated in

Bhagavata Gita

2 Tamil poet Appar was an Alvar

saint

Select the correct answer using the code

given below

(a) 1 only

(b) 2 only

(c) Both 1 and 2

(d) Neither 1 nor 2

Q22) Which of the following rulers had his

image inscribed in the coins while

playing a veena

(a) Meander

(b) Chandragupta Maurya

(c) Gautamiputra Satkarni

(d) Samudragupta

Q23) Which of the following statements

isare correct

1 Vikrama Samvat is adopted as the

national calendar by independent

India and it began in 68 BC

2 Banabhatta was a court poet of

Harshavardhana

Select the correct answer using the code

given below

(a) 1 only

(b) 2 only

(c) Both 1 and 2

(d) Neither 1 nor 2

Q24) Consider the following pairs

1 Sandhi-vigrahika Minister of trade

2 Prathama-kulika Chief craftsman

3 Sarthavaha Chief banker

Which of the pairs given above isare

correct

(a) 1 only

(b) 1 and 3 only

(c) 2 only

(d) 2 and 3 only

Q25) Which of the following statements

isare correct

1 Aihole inscription belongs to

Pulakeshin II and was composed

by Ravikriti

2 Xuan Zang came to India during

the reign of Chandragupta II

Select the correct answer using the code

given below

(a) 1 only

(b) 2 only

(c) Both 1 and 2

(d) Neither 1 nor 2

RAUSIAS-FC19E1003 10

Q26) िासतकिा स सोबोनरत निमननिखित ततव ो म स कौि-स

ततव किि नहोद मखनदर ो की िासतकिा क भाग ह

1 नशिर

2 मणडप

3 परदनकषरा पथ

4 गभणगह

िीच नदए गए कट का परय ग कर सही उततर चनिएः

(a) किि 1 3 और 4

(b) किि 2 3 और 4

(c) किि 1 2 और 4

(d) 1 2 3 और 4

Q27) निमननिखित मोनदर ो म स कौि-स मोनदर ईोट ो स बि ह

1 दिगढ़ मोनदर

2 भीतरगाि मोनदर

3 िकषमर मोनदर नसरपर

4 बहदशवर मोनदर

िीच नदए गए कट का परय ग कर सही उततर चनिएः

(a) किि 1 2 और 3

(b) किि 2 3 और 4

(c) किि 1 3 और 4

(d) 1 2 3 और 4

Q28) निमननिखित कथि ो म स कौि-सास सही हह

1 सलताि महममद किी कतब शाह अकबर का

समकािीि था

2 िासतकिा क कषतर म महममद किी कतब शाह

ि कई ईमारत ो का निमाणर करिाया था नजिम

स चार मीिार सिाणनरक परनसदध ह

िीच नदए गए कट का परय ग कर सही उततर चनिएः

(a) किि 1

(b) किि 2

(c) 1 और 2 द ि ो

(d) ि त 1 ि ही 2

Q29) निमननिखित यग ो पर निचार कीनजएः

1 मनिमकिाई सततिार

2 अनभजञाि शाको तिम कानिदास

3 नसिपपानदकारम क ििि

उपयणकत यग ो म स कौि-सास सही समनित हह

(a) किि 1 और 2

(b) किि 2

(c) किि 1 और 3

(d) 1 2 और 3

Q30) निमननिखित कथि ो म स कौि-सास सही हह

1 ldquoसशरत सोनहताrdquo नचनकरता पर एक महतवपरण

रचिा ह

2 बरहमगपत और चरक महतवपरण गनरतजञ थ

िीच नदए गए कट का परय ग कर सही उततर चनिए

(a) किि 1

(b) किि 2

(c) 1 और 2 द ि ो

(d) ि त 1 ि ही 2

Q31) अमीर िसर क सनदभण म निमननिखित कथि ो म स

कौि-सास सही हह

1 अमीर िसर ि अपिी रचिाओो म नििा ह

नक सोसकत नकसी भी कषतर स सोबोनरत िही ो थी

और किि बराहमर ही इस भािा का जञाि रित

2 उन ोि नहोदिी और अिरी क अखसततव का

उललि नकया था

िीच नदए गए कट का परय ग कर सही उततर चनिए

(a) किि 1

(b) किि 2

(c) 1 और 2 द ि ो

(d) ि त 1 ि ही 2

RAUSIAS-FC19E1003 11

Q26) Which of the following architectural

elements were only part of Hindu

temple architecture

1 Shikhara

2 Mandapa

3 Pradakshina patha

4 Garbhagriha

Select the correct answer using the code

given below

(a) 1 3 and 4 only

(b) 2 3 and 4 only

(c) 1 2 and 4 only

(d) 1 2 3 and 4

Q27) Which of the following temples isare

made of bricks

1 Deogarh Temple

2 Bhitargaon Temple

3 Lakshmana temple Sirpur

4 Brihadeshvara Temple

Select the correct answer using the code

given below

(a) 1 2 and 3 only

(b) 2 3 and 4 only

(c) 1 3 and 4 only

(d) 1 2 3 and 4

Q28) Which of the following statements

isare correct

1 Sultan Muhammad Quli Qutab

Shah was a contemporary of

Akbar

2 In the field of architecture

Muhammad Quli Qutab Shah

constructed many buildings the

most famous of which is the Char

Minar

Select the correct answer using the code

given below

(a) 1 only

(b) 2 only

(c) Both 1 and 2

(d) Neither 1 nor 2

Q29) Consider the following pairs

1 Manimekalai Sattanar

2 Abhijnana Shakuntalam Kalidasa

3 Silappadikaram Kovalan

Which of the pairs given above isare

correct

(a) 1 and 2 only

(b) 2 only

(c) 1 and 3 only

(d) 1 2 and 3

Q30) Which of the following statements

isare correct

1 Sushruta Samhita is an important

work on medicine

2 Brahmagupta and Charaka were

important mathematicians

Select the correct answer using the code

given below

(a) 1 only

(b) 2 only

(c) Both 1 and 2

(d) Neither 1 nor 2

Q31) Which of the following statements

isare correct about Amir Khusrau

1 Amir Khusrau records in his works

that Sanskrit did not belong to any

region and only the Brahmans

knew it

2 He recorded the existence of

Hindawi and Awadhi

Select the correct answer using the code

given below

(a) 1 only

(b) 2 only

(c) Both 1 and 2

(d) Neither 1 nor 2

RAUSIAS-FC19E1003 12

Q32) निमननिखित कथि ो पर निचार कीनजए

1 नहरणय-गभण अिषठाि क बार म ऐसा स चा जाता

था नक बनि दि िाि का एक कषनतरय क रप म

पिजणनम ह गा

2 मयरशमणि कदोब िोश का सोसथापक था

उपयणकत कथि ो म स कौि-सास सही हह

(a) किि 1

(b) किि 2

(c) 1 और 2 द ि ो

(d) ि त 1 ि ही 2

Q33) निमननिखित कथि ो म स कौि-सास सही हह

1 कदमई बगार (बिपिणक शरम) क रप म

निया जाि िािा कर था

2 गवानियर परशखसत म िागभट (ज एक चोदि

राजा था) क दवारा नकय गए श िर का िरणि

नकया गया ह

िीच नदए गए कट का परय ग कर सही उततर चनिए

(a) किि 1

(b) किि 2

(c) 1 और 2 द ि ो

(d) ि त 1 ि ही 2

Q34) निमननिखित कथि ो म स कौि-सास सही हह

1 राजतरो नगिी 11िी ो शताबदी म कलहि क दवारा

रनचत एक सोसकत पसतक (टकसट) ह

2 कननौज क निए नतरपकषीय सोघिण म पाि राजिोश

शानमि था

िीच नदए गए कट का परय ग कर सही उततर चनिए

(a) किि 1

(b) किि 2

(c) 1 और 2 द ि ो

(d) ि त 1 ि ही 2

Q35) निमननिखित यग ो पर निचार कीनजए

1 बरहदशवर मोनदर राजराजा च ि

2 उर मापि की इकाई

3 दिदाि मोनदर ो क भनम अिदाि

उपयणकत यग ो म स कौि-स सही समनित ह

(a) किि 1 और 2

(b) किि 2 और 3

(c) किि 1 और 3

(d) 1 2 और 3

Q36) निमननिखित कथि ो म स कौि-सास सही हह

1 नदलली क सलताि ो क अरीि परशासि की भािा

फारसी थी

2 नदलली सलतित म ldquoतारीितािरीिrdquo कनिता

का एक रप था

िीच नदए गए कट का परय ग कर सही उततर चनिए

(a) किि 1

(b) किि 2

(c) 1 और 2 द ि ो

(d) ि त 1 ि ही 2

Q37) निमननिखित कथि ो म स कौि-सास सही हह

1 अिाउददीि खििजी ि अपि सनिक ो क निए

नसरी िाम का एक िया दगणरकषक शहर

बिािाया था

2 िह अपि सनिक ो क िति का भगताि इकता

क रप म करता था

िीच नदए गए कट का परय ग कर सही उततर चनिए

(a) किि 1

(b) किि 2

(c) 1 और 2 द ि ो

(d) ि त 1 ि ही 2

RAUSIAS-FC19E1003 13

Q32) Consider the following statements

1 Hiranya-garbha ritual was thought

to lead to the rebirth of the

sacrificer as a Kshatriya

2 Mayurasharman was the founder

of the Kadamba dynasty

Which of the statements given above

isare correct

(a) 1 only

(b) 2 only

(c) Both 1 and 2

(d) Neither 1 nor 2

Q33) Which of the following statements

isare correct

1 Kadamai was tax taken in form of

forced labour

2 Gwalior Prashasti describes the

exploits of Nagabhata who was a

Chandella king

Select the correct answer using the code

given below

(a) 1 only

(b) 2 only

(c) Both 1 and 2

(d) Neither 1 nor 2

Q34) Which of the following statements

isare correct

1 Rajatarangini is a Sanskrit text

written by Kalhana in the 11th

century

2 Pala dynasty was included in the

tripartite struggle for Kannauj

Select the correct answer using the code

given below

(a) 1 only

(b) 2 only

(c) Both 1 and 2

(d) Neither 1 nor 2

Q35) Consider the following pairs

1 Brihadeshvara temple Rajaraja

Chola

2 ldquoUrrdquo Unit of measurement

3 Devadana Land grants made to

temples

Which of the pairs given above isare

correct

(a) 1 and 2 only

(b) 2 and 3 only

(c) 1 and 3 only

(d) 1 2 and 3

Q36) Which of the following statements

isare correct

1 The language of administration

under the Delhi Sultans was

Persian

2 Tarikhtawarikh was a form of

poetry in the Delhi Sultanate

Select the correct answer using the code

given below

(a) 1 only

(b) 2 only

(c) Both 1 and 2

(d) Neither 1 nor 2

Q37) Which of the following statements

isare correct

1 Alauddin Khilji constructed a new

garrison town named Siri for his

soldiers

2 He paid his soldiers their salaries

in the form of Iqta

Select the correct answer using the code

given below

(a) 1 only

(b) 2 only

(c) Both 1 and 2

(d) Neither 1 nor 2

RAUSIAS-FC19E1003 14

Q38) निमननिखित कथि ो म स कौि-सास सही हह

1 नदलली कतबददीि एबक क अरीि पहिी बार

नकसी सामराजय की राजरािी बिी थी

2 दहिीिाि नसक ो का मदरर मग़ि ो क दवारा

नकया गया था

िीच नदए गए कट का परय ग कर सही उततर चनिए

(a) किि 1

(b) किि 2

(c) 1 और 2 द ि ो

(d) ि त 1 ि ही 2

Q39) निमननिखित यग ो पर निचार कीनजए

1 म ठ की मखिद नसको दर ि दी

2 बगमपरी मखिद नफर ज शाह तगिक

3 कववत- अि - इसलाम कतबददीि ऐबक

उपयणकत यग ो म स कौि-स सही समनित ह

(a) किि 1 और 2

(b) किि 2 और 3

(c) किि 1 और 3

(d) 1 2 और 3

Q40) निमननिखित कथि ो म स कौि-सास सही हह

1 मिसबदार ो क अपिा िति राजसव कायो

नजन जागीर कहत थ क रप म परापत ह ता

था

2 मिसबदार क ज सनय उततरदानयतव सौोप जात

थ उसक अनतगणत उस एक निराणररत सखया म

सिार अथिा घड़सिार ो का रि-रिाि करिा

पड़ता था

िीच नदए गए कट का परय ग कर सही उततर चनिए

(a) किि 1

(b) किि 2

(c) 1 और 2 द ि ो

(d) ि त 1 ि ही 2

Q41) ldquo1942 क भारत छ ड़ आोद ििrdquo क बार म

निमननिखित अिि कि ो म स कौि-सा सतय िही ो ह

(a) यह एक अनहोसक आोद िि था

(b) इसका िततव महातमा गाोरी क दवारा नकया गया

था

(c) यह एक सवाभानिक आोद िि था

(d) इसि सामानयतया शरनमक िगण क आकनिणत

िही ो नकया था

Q42) भारत क ि ग ो ि ldquoसाइमि कमीशिrdquo क आगमि क

निरदध आोद िि नकया था कय ोनक

(a) भारतीय कभी भी 1919 क अनरनियम (The

Act of 1919) क काम की समीकषा िही ो करिा

चाहत थ

(b) साइमि कमीशि ि पराोत ो म दवर (द हर) शासि

क समापत करि की नसफाररश की थी

(c) साइमि कमीशि म क ई भारतीय सदसय िही ो

था

(d) साइमि कमीशि ि दश क निभाजि का

सझाि नदया था

Q43) निमननिखित कथि ो पर निचार कीनजए

भारतीय राषटर ीय आोद िि म दादाभाई िौर जी क दवारा

नकया गया सबस परभािी य गदाि यह था नक उन ोि

1 अोगरज ो क दवारा भारत क आनथणक श िर का

ििासा नकया था

2 पराचीि भारतीय गरोथ ो की वयाखया की थी और

भारतीय ो क आतमनिशवास क पिःसथानपत नकया

था

3 अनय नकसी भी बात स पहि सभी सामानजक

बराइय ो क उनमिि की आिशयकता पर बि

नदया था

उपयणकत कथि ो म स कौि-सास सही हह

(a) किि 1

(b) किि 2 और 3

(c) किि 1 और 3

(d) 1 2 और 3

RAUSIAS-FC19E1003 15

Q38) Which of the following statements

isare correct

1 Delhi first became the capital of a

kingdom under Qutubuddin

Aibak

2 Dehliwal coins were minted by the

Mughals

Select the correct answer using the code

given below

(a) 1 only

(b) 2 only

(c) Both 1 and 2

(d) Neither 1 nor 2

Q39) Consider the following pairs

1 Moth ki Masjid- Sikander Lodi

2 Begumpuri mosque- Firuz Shah

Tughluq

3 Quwwat al ndash Islam- Qutubuddin

Aibak

Which of the above pairs isare correct

(a) 1 and 2 only

(b) 2 and 3 only

(c) 1 and 3 only

(d) 1 2 and 3

Q40) Which of the following statements

isare correct

1 Mansabdars received their salaries

as revenue assignments called

jagirs

2 The mansabdarrsquos military

responsibilities required him to

maintain a specified number of

sawar or cavalrymen

Select the correct answer using the code

given below

(a) 1 only

(b) 2 only

(c) Both 1 and 2

(d) Neither 1 nor 2

Q41) Which one of the following observations

is not true about the Quit India

Movement of 1942

(a) It was a non-violent movement

(b) It was led by Mahatma Gandhi

(c) It was a spontaneous movement

(d) It did not attract the labour class

in general

Q42) The people of India agitated against the

arrival of the Simon Commission

because

(a) Indians never wanted the review of

the working of the Act of 1919

(b) Simon Commission recommended

the abolition of dyarchy in the

Provinces

(c) there was no Indian member in the

Simon Commission

(d) the Simon Commission suggested

the partition of the country

Q43) Consider the following statements

The most effective contribution made by

Dadabhai Naoroji to the cause of Indian

National Movement was that he-

1 exposed the economic exploitation

of India by the British

2 interpreted the ancient Indian

texts and restored the self-

confidence of Indians

3 stressed the need for eradication of

all the social evils before anything

else

Which of the statements given above

isare correct

(a) 1 only

(b) 2 and 3 only

(c) 1 and 3 only

(d) 1 2 and 3

RAUSIAS-FC19E1003 16

Q44) महातमा गाोरी ि 1932 म आमरर अिशि नकया था

कय ोनक

(a) ldquoग िमज सममििrdquo (The Round Table

Conference) भारतीय राजिीनतक

आकाोकषाओो क परा करि म असफि रहा था

(b) काोगरस और मखसलम िीग म मतभद थ

(c) रामस मकड िालड (Ramsay Macdonald)

ि ldquoसाोपरदानयक परसकारrdquo (The Communal

Award) की घ िरा की थी

(d) ldquoसनििय अिजञा आोद ििrdquo (The Civil

Disobedience Movement) असफि रहा

था

Q45) भारत म औपनििनशक शासि की अिनर क सोदभण म

भारत स रि क बनहगणमि का एक महतवपरण भाग गह

शलक (Home Charges) था निमननिखित म स

कौि-सास क ि गह शलक म सखममनित नकया गया

थानकय गए थ

1 िोदि म भारत कायाणिय क निए उपय ग नकय

जाि िािा क ि

2 भारत म नियकत नबरनटश कनमणय ो क िति और

पशि का भगताि करि क निए उपय ग नकय

जाि िािा क ि

3 अोगरज ो क दवारा भारत क बाहर यदध ो क निए

उपय ग नकय जाि िािा क ि

िीच नदए गए कट का परय ग कर सही उततर चनिए

(a) किि 1

(b) किि 1 और 2

(c) किि 2 और 3

(d) 1 2 और 3

Q46) सवतोतरता आोद िि क इनतहास म भारतीय राषटर ीय

काोगरस का 1929 का सतर महतवपरण ह कय ोनक इसम

(a) काोगरस क उददशय क रप म सथािीय सरकार

की पराखपत की घ िरा की गई थी

(b) परण सवराज की पराखपत क काोगरस क िकषय क

रप म अपिाया गया था

(c) असहय ग आोद िि शर नकया गया था

(d) िोदि म ldquoग ि मर सममििrdquo (The Round

Table Conference) म भाग िि का निरणय

निया गया था

Q47) भारतीय सवतोतरता सोगराम क सोदभण म िहर ररप टण

क दवारा निमननिखित म स नकसकी नसफाररश की गई

थीनकिकी नसफाररश की गई थी ो

1 भारत क निए परण सवतोतरता

2 अलपसोखयक ो क निए सीट ो क आरकषर क

निए सोयकत नििाणचक मोडि

3 सोनिराि म भारत क ि ग ो क निए मौनिक

अनरकार ो का परािराि

िीच नदए गए कट का परय ग कर सही उततर चनिए

(a) किि 1

(b) किि 2 और 3

(c) किि 1 और 3

(d) 1 2 और 3

Q48) आरो नभक िनदक आयो का रमण मखय रप स था

(a) भखकत

(b) मनतण पजा और यजञ

(c) परकनत की पजा और यजञ

(d) परकनत की पजा और भखकत

RAUSIAS-FC19E1003 17

Q44) Mahatma Gandhi undertook fast unto

death in 1932 mainly because

(a) The Round Table Conference failed

to satisfy Indian political

aspirations

(b) The Congress and Muslim League

had differences of opinion

(c) Ramsay Macdonald announced the

Communal Award

(d) The Civil Disobedience Movement

failed

Q45) With reference to the period of colonial

rule in India ldquoHome Chargesrdquo formed

an important part of drain of wealth

from India Which of the following funds

constituted ldquoHome Chargesrdquo

1 Funds used to support the India

Office in London

2 Funds used to pay salaries and

pensions of British personnel

engaged in India

3 Funds used for waging wars

outside India by the British

Select the correct answer using the code

given below

(a) 1 only

(b) 1 and 2 only

(c) 2 and 3 only

(d) 1 2 and 3

Q46) The 1929- Session of Indian National

Congress is of significance in the history

of the Freedom Movement because the-

(a) attainment of Self-Government

was declared as the objective of

the Congress

(b) attainment of Poorna Swaraj was

adopted as the goal of the

Congress

(c) Non-Cooperation Movement was

launched

(d) decision to participate in the

Round Table Conference in

London was taken

Q47) With reference to the period of Indian

freedom struggle which of the following

waswere recommended by the Nehru

report

1 Complete Independence for India

2 Joint electorates for reservation of

seats for minorities

3 Provision of fundamental rights for

the people of India in the

Constitution

Select the correct answer using the code

given below

(a) 1 only

(b) 2 and 3 only

(c) 1 and 3 only

(d) 1 2 and 3

Q48) The religion of the early Vedic Aryans was primarily of

(a) Bhakti

(b) image worship and Yajnas

(c) worship of nature and Yajnas

(d) worship of nature and Bhakti

RAUSIAS-FC19E1003 18

Q49) भारत की यातरा करि िाि चीिी यातरी यआि चिाोग

(हयएि साोग) ि समकािीि भारत की सामानय

खसथनतय ो और सोसकनत क दजण नकया था इस सोदभण म

निमननिखित कथि ो म स कौि-सास सही हह

1 सड़क और िदी-मागण (जि-मागण) डकती स

परण रप स सरनकषत थ

2 जहा तक अपरार ो क निए दणड की बात ह

उसक निए नकसी भी वयखकत की निदोिता

अथिा उसक अपरार क निराणररत करि क

निए अनि जि और निि परि क माधयम क

सारि थ

3 वयापाररय ो क घाट ो और परनतबोर सटशि ो पर

शलक ो का भगताि करिा पड़ता था

िीच नदए गए कट का परय ग कर सही उततर चनिए

(a) किि 1

(b) किि 2 और 3

(c) किि 1 और 3

(d) 1 2 और 3

Q50) नसोर घाटी सभयता क सोदभण म निमननिखित कथि ो पर

निचार कीनजए

1 यह मखय रप स एक रमणनिरपकष सभयता थी

तथा हािाोनक इसम रानमणक ततव मौजद था

िनकि िह परनतिश पर हािी िही ो था

2 इस काि क दौराि भारत म कपास का परय ग

कपड़ा बिाि क निए नकया जाता था

उपयणकत कथि ो म स कौि-सास सही हह

(a) किि 1

(b) किि 2

(c) 1 और 2 द ि ो

(d) ि त 1 ि ही 2

Q51) परोदर दास क सोदभण म निमननिखित कथि ो पर निचार

कीनजए

1 परोदर दास एक सोत और भगिाि नशि क एक

महाि भकत थ

2 ि एक सोगीतकार गायक और किाणटक सोगीत

क मखय सोसथापक-परसतािक ो म स एक थ

उपयणकत कथि ो म स कौि-सास सही हह

(a) किि 1

(b) किि 2

(c) 1 और 2 द ि ो

(d) ि त 1 ि ही 2

Q52) निमननिखित म स कौि-सास वयखकत किाणटक सोगीत

की नतरमनतण म शानमि हह

1 बािामरिी कषणा

2 शरी शयाम शासतरी

3 शरी मथसवामी दीनकषतर

िीच नदए गए कट का परय ग कर सही उततर चनिए

(a) किि 1

(b) किि 2

(c) किि 2 और 3

(d) 1 2 और 3

Q53) चियर (Chevayur) और अथ िी (Atholi) म खसथत

महापािार सथि निमननिखित म स नकस राजय म खसथत

(a) तनमििाड

(b) किाणटक

(c) पनिम बोगाि

(d) करि

RAUSIAS-FC19E1003 19

Q49) The Chinese traveller Yuan Chwang

(Hiuen Tsang) who visited India

recorded the general conditions and

culture of India at that time In this

context which of the following

statements isare correct

1 The roads and river-routes were

completely immune from robbery

2 As regards punishment for

offences ordeals by fire water and

poison were the instruments for

determining the innocence or guilt

of a person

3 The tradesmen had to pay duties

at ferries and barrier stations

Select the correct answer using the code

given below

(a) 1 only

(b) 2 and 3 only

(c) 1 and 3 only

(d) 1 2 and 3

Q50) Regarding the Indus Valley Civilization

consider the following statements

1 It was predominantly a secular

civilization and the religious

element though present did not

dominate the scene

2 During this period cotton was

used for manufacturing textiles in

India

Which of the statements given above

isare correct

(a) 1 only

(b) 2 only

(c) Both 1 and 2

(d) Neither 1 nor 2

Q51) Consider the following statements

regarding Purandara Dasa

1 Purandara Dasa was a saint and

great devotee of Lord Shiva

2 He was a composer singer and

one of the chief founding-

proponents of the Carnatic music

Which of the statements given above

isare correct

(a) 1 only

(b) 2 only

(c) Both 1 and 2

(d) Neither 1 nor 2

Q52) Which of the following persons isare

included in the trinity of Carnatic

music

1 Balamurali Krishna

2 Sri Shyama Shastry

3 Sri Muthuswami Dikshitar

Select the correct answer using the code

given below

(a) 1 only

(b) 2 only

(c) 2 and 3 only

(d) 1 2 and 3

Q53) Megalithic sites at Chevayur and Atholi

are located in which of the following

states

(a) Tamil Nadu

(b) Karnataka

(c) West Bengal

(d) Kerala

RAUSIAS-FC19E1003 20

Q54) निमननिखित कथि ो पर निचार कीनजए

1 महापािानरक ि ग कबर ो म िसतएो दफिात थ

2 दनकषर भारत म महापािार सोसकनत एक परण

निकनसत तामर यगीि सोसकनत थी

उपयणकत कथि ो म स कौि-सास सही हह

(a) किि 1

(b) किि 2

(c) 1 और 2 द ि ो

(d) ि त 1 ि ही 2

Q55) निमननिखित म स कौि-स सामराजयसामराजय ो का

अश क क अनभिि ो म उललि नकया गया ह

1 च ि

2 पाणडय

3 करिपतर (चर)

िीच नदए गए कट का परय ग कर सही उततर चनिए

(a) किि 1

(b) किि 1 और 2

(c) किि 3

(d) 1 2 और 3

Q56) भीमा-क रगाोि का यदध को पिी क सनिक ो और

बाजीराि नदवतीय क िततव म एक शखकतशािी पशिा

सिा (मराठ ो) क मधय िड़ा गया था यह यदध

निमननिखित म स नकसका नहससा था

(a) परथम आोगल-मराठा यदध का

(b) नदवतीय आोगल-मराठा यदध का

(c) ततीय आोगल-मसर यदध का

(d) ततीय आोगल-मराठा यदध का

Q57) निमननिखित कथि ो पर निचार कीनजए

1 महादि दसाई ि गाोरीजी क चोपारर आि तथा

नतिकनथया पररािी स जड़ी समसया की जाोच

क निए रारी करि क निए दश भर म उिका

अिसरर नकया था

2 िरहरी पाररि चोपारर सतयागरह क दौराि

गाोरीजी क साथ थ

उपयणकत कथि ो म स कौि-सास सही हह

(a) किि 1

(b) किि 2

(c) 1 और 2 द ि ो

(d) ि त 1 ि ही 2

Q58) निमननिखित कथि ो पर निचार कीनजए

1 िनद राज-िोश ि बराहमर ो और बौदध मठराररय ो

क कर-मकत गाि अिदाि म दि की परथा

आरि की थी

2 सतिाहि ो की आनरकाररक भािा पराकत थी

उपयणकत कथि ो म स कौि-सास सही हह

(a) किि 1

(b) किि 2

(c) 1 और 2 द ि ो

(d) ि त 1 ि ही 2

Q59) एक निरासत क अपिाइए (अडॉपट ए हररटज ndash

Adopt a Heritage) पररय जिा क उददशय ो क

सनदभण म निमननिखित कथि ो पर निचार कीनजए

1 यह पररय जिा र रगार उतपादि और आनथणक

निकास क निए पयणटि कषमता का उि पर

परभाि का उपय ग करगी

2 यह पररय जिा निरासत सथि ो पर निशव सतरीय

आराररक सोरचिा निकनसत करक एक सतत

तरीक स पयणटक आकिणर म िखदध करगी

उपयणकत कथि ो म स कौि-सास सही हह

(a) किि 1

(b) किि 2

(c) 1 और 2 द ि ो

(d) ि त 1 ि ही 2

RAUSIAS-FC19E1003 21

Q54) Consider the following statements

1 Megalithic people buried goods in

graves

2 The megalithic culture in South

India was a full-fledged Copper

Age culture

Which of the statements given above

isare correct

(a) 1 only

(b) 2 only

(c) Both 1 and 2

(d) Neither 1 nor 2

Q55) Which of the following kingdoms isare

mentioned in the Ashokan inscriptions

1 Cholas

2 Pandyas

3 Keralaputras (Cheras)

Select the correct answer using the code

given below

(a) 1 only

(b) 1 and 2 only

(c) 3 only

(d) 1 2 and 3

Q56) The Battle of Bhima-Koregaon was

fought between the soldiers of the

Company and the strong Peshwa army

(Marathas) under Bajirao II This war

was a part of the

(a) First Anglo-Maratha war

(b) Second Anglo-Maratha war

(c) Third Anglo- Mysore war

(d) Third Anglo-Maratha war

Q57) Consider the following statements

1 Mahadev Desai followed Gandhiji all over the country to persuade him to come to Champaran to investigate the problem associated

with tinkathia system

2 Narhari Parikh accompanied Gandhi ji during the Champaran

Satyagraha

Which of the statements given above isare correct

(a) 1 only

(b) 2 only

(c) Both 1 and 2

(d) Neither 1 nor 2

Q58) Consider the following statements

1 The Nanda Dynasty started the practice of granting tax-free villages to brahmanas and

Buddhist monks

2 The official language of the Satavahanas was Prakrit

Which of the statements given above

isare correct

(a) 1 only

(b) 2 only

(c) Both 1 and 2

(d) Neither 1 nor 2

Q59) Consider the following statements about the objectives of the lsquoadopt a heritagersquo

project

1 It will harness tourism potential for its effects on employment generation and economic

development

2 It will enhance the tourist attractiveness in a sustainable manner by developing world class infrastructure at heritage sites

Which of the statements given above

isare correct

(a) 1 only

(b) 2 only

(c) Both 1 and 2

(d) Neither 1 nor 2

RAUSIAS-FC19E1003 22

Q60) ldquoभारतीय जिजातीय सहकारी निपरि निकास सोघrdquo

(The Tribal Co-operative Marketing

Development Federation of India - TRIFED)

क सोदभण म निमननिखित कथि ो पर निचार कीनजए

1 यह एक राषटर ीय सतर का शीिण सोगठि ह ज

भारत सरकार क गह मोतरािय क परशासनिक

नियोतरर क अरीि काम कर रहा ह

2 इसका मखय उददशय दश म जिजातीय ि ग ो

का सामानजक-आनथणक निकास करिा ह

उपयणकत कथि ो म स कौि-सास सही हह

(a) किि 1

(b) किि 2

(c) 1 और 2 द ि ो

(d) ि त 1 ि ही 2

Q61) निमननिखित म स कौि-सास उपनयास परमचोद क

दवारा नििा गया हनिि गए ह

1 रोगभनम

2 ग दाि

3 ग रा

िीच नदए गए कट का परय ग कर सही उततर चनिए

(a) किि 1

(b) किि 2

(c) किि 1 और 2

(d) 1 2 और 3

Q62) नगदधा ितय क सोदभण म निमननिखित कथि ो पर निचार

कीनजए

1 नगदधा नबहार की मनहिाओो क दवारा तयौहार क

समय और फसि की बिाई तथा कटाई क

अिसर पर नकया जाि िािा एक पारोपररक

दहाती ितय ह

2 इस ितय क दवारा मनहिाऐो अपिी परसननता

परकट करती ह तथा नगदधा क परदशणि क

माधयम स परि िचणसव िाि समाज म

मनहिाओो की दबी हई भाििाओो क परकट

करती ह

उपयणकत कथि ो म स कौि-सास सही हह

(a) किि 1

(b) किि 2

(c) 1 और 2 द ि ो

(d) ि त 1 ि ही 2

Q63) निमननिखित कथि ो पर निचार कीनजए

1 मलला शाह बदखशी दारा नशक ह क

आधयाखतमक गर थ

2 औरोगरब ि मजम-उि-बहरीि या द समदर ो

का सोगम िामक उललििीय रचिा नििी थी

3 दारा नशक ह क अपि पिणज अकबर क गर ो

क उततरानरकारी क रप म दिा गया था

नजसम उसि रानमणक बहििाद और समनवयता

क बढ़ािा नदया था

उपयणकत कथि ो म स कौि-सास सही हह

(a) किि 1 और 3

(b) किि 2

(c) किि 1 और 2

(d) 1 2 और 3

RAUSIAS-FC19E1003 23

Q60) Consider the following statements about

the Tribal Cooperative Marketing

Development Federation of India

(TRIFED)

1 It is a national-level apex

organization functioning under the

administrative control of Ministry

of Home Affairs Government of

India

2 The main objective of TRIFED is

socio-economic development of

tribal people in the country

Which of the statements given above

isare correct

(a) 1 only

(b) 2 only

(c) Both 1 and 2

(d) Neither 1 nor 2

Q61) Which of the following novels isare

written by Premchand

1 Rangabhumi

2 Godan

3 Gora

Select the correct answer using the code

given below

(a) 1 only

(b) 2 only

(c) 1 and 2 only

(d) 1 2 and 3

Q62) Consider the following statements about

Giddha dance

1 Giddha is a traditional pastoral

dance performed by the women of

Bihar at festival times and at the

sowing and reaping of the harvest

2 By this dance the women reveal

their joy expel their suppressed

feelings in a male dominated

society through the performance of

Giddha

Which of the statements given above

isare correct

(a) 1 only

(b) 2 only

(c) Both 1 and 2

(d) Neither 1 nor 2

Q63) Consider the following statements

1 Mullah Shah Badakhshi was the

spiritual mentor of Dara Shukoh

2 Aurangzeb wrote the remarkable

work called ldquoMajma-ul-Bahrainrdquo or

the ldquoThe confluence of two seasrdquo

3 Dara Shukoh was seen as

inheriting the qualities of his

ancestor Akbar in that he

promoted religious pluralism and

syncretism

Which of the statements given above

isare correct

(a) 1 and 3 only

(b) 2 only

(c) 1 and 2 only

(d) 1 2 and 3

RAUSIAS-FC19E1003 24

Q64) निमननिखित कथि ो पर निचार कीनजए

1 ग मतशवर परनतमा निोधयनगरी पहाड़ी पर खसथत ह

2 शरिरबिग िा िह सथाि ह जहाो मौयण िोश क

सोसथापक चोदरगपत मौयण अपि नसोहासि क

तयागि क बाद जि तपसवी बि गए थ

उपयणकत कथि ो म स कौि-सास सही हह

(a) किि 1

(b) किि 2

(c) 1 और 2 द ि ो

(d) ि त 1 ि ही 2

Q65) निमननिखित कथि ो पर निचार कीनजए

1 पराताखतवक साकषय स पता चिता ह नक पराची

घाटी सभयता हड़पपा और म हिज दाड़ द ि ो

की पिणिती ह

2 पराची िदी भििशवर स निकिती ह

उपयणकत कथि ो म स कौि-सास सही हह

(a) किि 1

(b) किि 2

(c) 1 और 2 द ि ो

(d) ि त 1 ि ही 2

Q66) निमननिखित कथि ो म स कौि-सास सही हह

1 िजराह क समारक ो क समह का निमाणर

चोदि राजिोश क शासिकाि क दौराि हआ

था

2 य समारक हररिोदर पिणत शरोििा म खसथत ह

3 म रक क यातरी इबन बतता ि अपि सोसमरर ो

म िजराह क मोनदर ो की यातरा का उललि

नकया था तथा इन काजराण िाम स समब नरत

नकया था

िीच नदए गए कट का परय ग कर सही उततर चनिए

(a) किि 1

(b) किि 1 और 2

(c) किि 2 और 3

(d) किि 1 और 3

Q67) निमननिखित कथि ो म स कौि-सास सही हह

1 डॉ बी आर अमबडकर ि दी एनिनहिशि

ऑफ़ कासट (The Annihilation of Caste)

नििी थी नजसम उन ोि नहोद रमण म िोशािगत

पजारी की परथा क उनमिि की आिशयकता

पर बि नदया था

2 डॉ राजदर परसाद ि थॉटस ऑि पानकसताि

(Thoughts on Pakistan) िामक पसतक

नििी थी

िीच नदए गए कट का परय ग कर सही उततर चनिए

(a) किि 1

(b) किि 2

(c) 1 और 2 द ि ो

(d) ि त 1 ि ही 2

Q68) निमननिखित कथि ो म स कौि-सास सही हह

1 महरगढ़ भारतीय उपमहादवीप म एक परनसदध

ििपािार बसती ह ज नसोर पराोत पानकसताि म

खसथत ह

2 बरणह म म कतत ो क उिक सवामी क साथ कबर ो

म दफिाया जाता था

िीच नदए गए कट का परय ग कर सही उततर चनिए

(a) किि 1

(b) किि 2

(c) 1 और 2 द ि ो

(d) ि त 1 ि ही 2

Q69) निमननिखित कथि ो म स कौि-सास सही हह

1 काकानटय मोनदर अनरकतर नशि क समनपणत

2 हिमक ोडा म हजार-सतोभ िाि मोनदर (The

Thousand-Pillared Temple) का निमाणर

काकानटय समराट रदर ि करिाया था

िीच नदए गए कट का परय ग कर सही उततर चनिए

(a) किि 1

(b) किि 2

(c) 1 और 2 द ि ो

(d) ि त 1 ि ही 2

RAUSIAS-FC19E1003 25

Q64) Consider the following statements

1 Gommateshwara Statue is located

on the Vindyagiri Hill

2 Shravanabelagola is the place

where Chandragupta Maurya the

founder of the Mauryan dynasty

became a Jain ascetic after

relinquishing his throne

Which of the statements given above

isare correct

(a) 1 only

(b) 2 only

(c) Both 1 and 2

(d) Neither 1 nor 2

Q65) Consider the following statements

1 Archaeological evidence shows

that the Prachi Valley Civilisation

predates both Harappa and

Mohenjo-Daro

2 The Prachi river originates from

Bhubaneswar

Which of the statements given above

isare correct

(a) 1 only

(b) 2 only

(c) Both 1 and 2

(d) Neither 1 nor 2

Q66) Which of the following statements

isare correct

1 The Khajuraho group of

monuments was built during the

rule of the Chandela dynasty

2 These monuments are located in

Harischandra mountain range

3 Ibn Battuta the Moroccan

traveller in his memoirs mentioned

visiting Khajuraho temples and

called them Kajarra

Select the correct answer using the code

given below

(a) 1 only

(b) 1 and 2

(c) 2 and 3

(d) 1 and 3

Q67) Which of the following statements

isare correct

1 Dr BR Ambedkar wrote the

Annihilation of Caste emphasising

the need to do away with the

practice of hereditary priesthood in

Hinduism

2 The book lsquoThoughts on Pakistanrsquo

was written by Dr Rajendra

Prasad

Select the correct answer using the code

given below

(a) 1 only

(b) 2 only

(c) Both 1 and 2

(d) Neither 1 nor 2

Q68) Which of the following statements

isare correct

1 Mehrgarh is a famous Neolithic

settlement in the Indian

subcontinent which is situated in

Sindh province Pakistan

2 At Burzahom dogs were buried

with their masters in their graves

Select the correct answer using the code

given below

(a) 1 only

(b) 2 only

(c) Both 1 and 2

(d) Neither 1 nor 2

Q69) Which of the following statements

isare correct

1 The Kakatiya temples are

dedicated mostly to Siva

2 The Thousand-Pillared Temple at

Hanamkonda was built by the

Kakatiya king Rudra

Select the correct answer using the code

given below

(a) 1 only

(b) 2 only

(c) Both 1 and 2

(d) Neither 1 nor 2

RAUSIAS-FC19E1003 26

Q70) निमननिखित कथि ो म स कौि-सास सही हह

1 अहमदाबाद नमि हड़ताि क दौराि महातमा

गाोरी ि शरनमक ो क पकष क मजबत करि क

निए आमरर अिशि नकया था

2 अिशि स नमि मानिक ो पर दबाि पड़ा था ज

अोततः शरनमक ो क िति म 15 परनतशत की िखदध

करि क निए सहमत हए थ

िीच नदए गए कट का परय ग कर सही उततर चनिए

(a) किि 1

(b) किि 2

(c) 1 और 2 द ि ो

(d) ि त 1 ि ही 2

Q71) निमननिखित म स नकसक नकिक भारत स यिसक

की माििता की अमतण साोसकनतक निरासत की

परनतनिनर सची (The UNESCOrsquos List of the

Representative List of the Intangible

Cultural Heritage of Humanity) म शानमि

नकया गया ह

1 मनडयटट

2 सोकीतणि

3 को भ मिा

िीच नदए गए कट का परय ग कर सही उततर चनिए

(a) किि 1 और 2

(b) किि 2 और 3

(c) किि 3

(d) 1 2 और 3

Q72) निमननिखित जिजानतय ो म स कौि-सीसी ो

जिजानतजिजानतया िागािड स सोबोनरत हह

1 अोगामी

2 ककी

3 जारिा

िीच नदए गए कट का परय ग कर सही उततर चनिए

(a) किि 1

(b) किि 1 औऔ 2

(c) किि 2

(d) 1 2 और 3

Q73) निमननिखित कथि ो म स कौि-सास सही हह

1 राषटर कट सामराजय की सथापिा दोनतदगण ि की थी

नजसि मानयाित म अपिी राजरािी की

सथापिा की थी

2 राषटर कट समराट अम घििण एक ििक था और

उस कनिताओो पर पहिी कननड़ पसतक नििि

का शरय नदया जाता ह

िीच नदए गए कट का परय ग कर सही उततर चनिए

(a) किि 1

(b) किि 2

(c) 1 और 2 द ि ो

(d) ि त 1 ि ही 2

Q74) निमननिखित कथि ो म स कौि-सास सही हह

1 कशब चोदर सि ि ततवब नरिी सभा की

अधयकषता की थी ज आधयाखतमक सतय की

ि ज म सोिि थी

2 बरहम समाज ि मािि गररमा पर बि नदया

मनतणपजा का निर र नकया और सती परथा जसी

सामानजक बराइय ो की आि चिा की

िीच नदए गए कट का परय ग कर सही उततर चनिए

(a) किि 1

(b) किि 2

(c) 1 और 2 द ि ो

(d) ि त 1 ि ही 2

Q75) निमननिखित कथि ो म स कौि-सास सही हह

1 भारत म नचशती नसिनसिा खवाजा म इिददीि

नचशती क दवारा सथानपत नकया गया था

2 नचशती परोपरा की एक परमि निशिता

आतमसोयम थी नजसम साोसाररक म ह स दरी

बिाए रििा शानमि था

िीच नदए गए कट का परय ग कर सही उततर चनिए

(a) किि 1

(b) किि 2

(c) 1 और 2 द ि ो

(d) ि त 1 ि ही 2

RAUSIAS-FC19E1003 27

Q70) Which of the following statements

isare correct

1 During the Ahmedabad Mill Strike

Mahatma Gandhi undertook a fast

unto death to strengthen the

workersrsquo resolve

2 The fast had effect of putting

pressure on mill owners who

finally agreed to give the workers a

15 per cent increase in wages

Select the correct answer using the code

given below

(a) 1 only

(b) 2 only

(c) Both 1 and 2

(d) Neither 1 nor 2

Q71) Which of the following are included in

the UNESCOrsquos list of the representative

list of the intangible cultural heritage of

humanity from India

1 Mudiyettu

2 Sankirtana

3 Kumbh Mela

Select the correct answer using the code

given below

(a) 1 and 2 only

(b) 2 and 3 only

(c) 3 only

(d) 1 2 and 3

Q72) Which of the following tribes isare

related to Nagaland

1 Angami

2 Kuki

3 Jarawa

Select the correct answer using the code

given below

(a) 1 only

(b) 1 and 2 only

(c) 2 only

(d) 1 2 and 3

Q73) Which of the following statements

isare correct

1 Rashtrakuta kingdom was founded by Dantidurga who established his capital at Manyakhet

2 Amoghavarsha a Rashtrakuta king was an author and is credited with writing the first

Kannada book on poetics

Select the correct answer using the code given below

(a) 1 only

(b) 2 only

(c) Both 1 and 2

(d) Neither 1 nor 2

Q74) Which of the following statements isare correct

1 Keshab Chandra Sen headed the Tattvabodhini Sabha which was engaged in search of spiritual truth

2 The Brahmo Samaj laid emphasis on human dignity opposed idolatry and criticized such social

evils as the practice of Sati

Select the correct answer using the code given below

(a) 1 only

(b) 2 only

(c) Both 1 and 2

(d) Neither 1 nor 2

Q75) Which of the following statements isare correct

1 The Chishti order was established in India by Khwaja Moinuddin

Chishti

2 A major feature of the Chishti tradition was austerity including maintaining a distance from the

worldly power

Select the correct answer using the code

given below

(a) 1 only

(b) 2 only

(c) Both 1 and 2

(d) Neither 1 nor 2

T e s t i s p a r t o f R a u rsquo s I A S T e s t s e r i e s f o r P r e l i m i n a r y E x a m 2 0 1 9

FOUNDATION + CURRENT AFFAIRS

GENERAL STUDIES (PAPER ndashI)

FOUNDATION TEST ndashIII

SUBJECT NCERT History Class VI-X + Current Affairs

Time Allowed 1frac12 Hours Maximum Marks 150

I NSTRUCT IONS

1 IMMEDIATELY AFTER THE COMMENCEMENT OF THE EXAMINATION YOU SHOULD CHECK

THAT THIS TEST BOOKLET DOES NOT HAVE ANY UNPRINTED OR TORN or MISSING PAGES OR

ITEMS ETC IF SO GET IT REPLACED BY A COMPLETE TEST BOOKLET

2 This Test Booklet contains 75 items (questions) Each item is printed both in Hindi and English

Each item comprises four responses (answers) You will select the response which you want to mark

on the Answer Sheet In case you feel that there is more than one correct response mark the

response which you consider the best In any case choose ONLY ONE response for each item

3 You have to mark all your responses ONLY on the separate Answer Sheet (OMR sheet) provided

Read the directions in the Answer Sheet

4 All items carry equal marks

5 Before you proceed to mark in the Answer Sheet the response to various items in the Test booklet

you have to fill in some particulars in the Answer Sheet as per instructions contained therein

6 After you have completed filling in all your responses on the Answer Sheet and the examination has

concluded you should hand over to the Invigilator only the Answer Sheet You are permitted to

take away with you the Test Booklet

7 Penalty for wrong answers

THERE WILL BE PENALTY FOR WRONG ANSWERS MARKED BY A CANDIDATE IN THE

OBJECTIVE TYPE QUESTION PAPERS

(i) There are four alternatives for the answer to every question For each question for which a

wrong answer has been given by the candidate one-third of the marks assigned to that

question will be deducted as penalty

(ii) If a candidate gives more than one answer it will be treated as a wrong answer even if one of

the given answers happens to be correct and there will be same penalty as above to that

question

(iii) If a question is left blank ie no answer is given by the candidate there will be no penalty for

that question

T h i s t e s t i s p a r t o f R a u rsquo s I A S T e s t s e r i e s f o r P r e l i m i n a r y E x a m 2 0 1 9

Test Code

FC19E1003

FC19H1003 29

Answers and Explanations of

NCERT History Class VI-X + Current Affairs (FC19E1003)

Q1) उततर (c)

सपषटीकरण

- ऋगवद म दविय ो और दिताओो क समवपित एक

हजार स अविक सत तर (शल क) ह

- य शल क ऋविय ो क दवारा रच गए थ और परि ो

दवारा सीख जात थ

- हालाोवक कछ शल क मवहलाओो (जस वक अपाला

घ सा ल पामदरा मतरयी और गागी) क दवारा भी रच

गए थ

- ऋगवद म सोिाद क रप म कई शल क मौजद ह

- हम विशवावमतर नामक एक ऋवि और दविय ो क

रप म पजी जान िाली द नवदय ो (वयास और

सतलज) क बीच िाताि का उदाहरण वमलता ह

- इसस पता चलता ह वक विशवावमतर िवदक काल स

सोबोवित थ

Q2) उततर (b)

सपषटीकरण

- करनल गफाओो स राख क अिशि परापत हए ह

ज इस ओर सोकत करत ह वक ततकालीन ल ग

अवि क उपय ग स पररवचत थ

- य गफाएो आोधर परदश म सथथत ह

Q3) उततर (c)

सपषटीकरण

bull बरािह म ितिमान कशमीर म सथथत एक

परागवतहावसक थथल ह जहाो ल ग गडढ क घर ो का

वनमािण करत थ

bull य घर जमीन क ख द कर बनाए जात थ तथा नीच

जान क वलए सीवियाा ह ती थी

bull ऐसा अनमान लगाया जाता ह वक य घर ठो ड क

मौसम म आशरय परदान करत थ

Q4) उततर (c)

सपषटीकरण

bull परालख-विदया (Epigraphy) क वशलालख ो क

अधययन क रप म पररभावित वकया जाता ह

bull हसतवलसखत दसतािज ो क माधयम स इवतहास

और सावहतय क अधययन क पाोडवलवप विजञान

(Manuscriptology) कहत ह

bull पराचीन लखन परणावलय ो क अधययन और

ऐवतहावसक पाोडवलवपय ो क समझन तथा वतवथ

वनिािरण क पलीओगराफी (Palaeography) कहा

जाता ह

bull नयवमजमविकस (Numismatics) वसक ो क

अधययन क सोदवभित करता ह

Q5) उततर (a)

सपषटीकरण

- चरक सोवहता चरक क दवारा वलखी गई आयिद

और िदयक-शासर पर एक महतवपणि पसतक ह

- ि भारतीय िदयक-शासर की पारमपररक परणाली

वजस आयिद क नाम स जाना जाता ह क

अभयासकताि थ

- ऐसा माना जाता ह वक चरक का विकास दसरी

शताबदी (ईसा पिि) और दसरी शताबदी (ईसवी) क

मधय हआ था

Q6) उततर (b)

सपषटीकरण

- भाग फसल ो पर वलए जान िाल कर क सोदवभित

करता ह ज कल फसल उतपादन का 16 िाो भाग

था

- ldquoकममकारrdquo शबद भवमहीन कवि शरवमक िगि क

वलए परय ग वकया जाता था

- ldquoअशवमिrdquo (वजस घ ड क बवलदान क रप म भी

जाना जाता ह) एक अनषठान ह ता था वजसम एक

घ ड क सवतोतर रप स घमन क वलए छ ड वदया

FC19H1003 30

जाता ह और राजा क सवनक उसकी रखिाली

करत थ

Q7) उततर (d)

सपषटीकरण

- ऋगववदक काल म घ ड ो क रथ ो म ज ता जाता था

ज (रथ) भवम मिवशय ो आवद पर कबजा करन क

वलए लड गए यद ो म उपय ग वकए जात थ

- इसस यह पता चलता ह वक घ ड ो यकत रथ ो का

उपय ग महाजनपद काल स काफी पहल आरमभ

हआ था

- ऋगववदक काल म मिवशय ो भवम जल आवद पर

कबजा करन क वलए तथा ल ग ो क पकडन क

वलए यद वकय जात थ

- अविकाोश परि इन यद ो म भाग वलया करत थ

- हालाोवक उस समय क ई वनयवमत सना नही ो ह ती

थी लवकन उस काल म सभाऐो ह ती थी ो वजनम

ल ग यद क मामल ो पर चचाि करत थ

- वनयवमत सनाएा महाजनपद काल का िवशषटय थी

वजनम पदल सवनक ो की विशाल सनाएा रथ तथा

हाथी शावमल ह त थ

Q8) उततर (a)

सपषटीकरण

- बद शाकय कल स सोबोवित थ और कशीनारा म

उनका वनिन हआ था

- बद न अपनी वशकषाएा पराकत भािा म दी थी ो ज

आम ल ग ो की भािा थी

Q9) उततर (c)

सपषटीकरण

- पराचीन भारत म दशिनशासर की छह शाखाएा थी ो

िशविक नयाय समखया य ग पिि वममाोसा और

िदाोत या उततर वममाोसा

- इनकी थथापना करमश कनाद गौतम कवपल

पतोजवल जावमनी और वयास ऋविय ो न की थी

Q10) उततर (b)

सपषटीकरण

महािीर की वशकषाऐो छठी शताबदी म िललभी म

सोकवलत की गई थी ो

Q11) उततर (c)

सपषटीकरण

- पारमपररक रप स चाणकय क कौविलय अथिा

विषणगपत क नाम स जाना जाता ह

- उसन अथिशासतर ज एक पराचीन भारतीय

राजनवतक आलख ह वलखा था

Q12) उततर (d)

सपषटीकरण

- भारत का राषटर ीय वचनह सारनाथ (उततर परदश) क

अश क सतमभ क ऊपर (शीिि पर) वसोह कवपिल

का एक अनरपण ह

- इस राषटर ीय वसदाोत सतयमि जयत क साथ

सोय वजत वकया गया ह

- रामपिि बल का नाम रामपिि (वबहार) क नाम पर

पडा जहाा इसकी ख ज हई थी

- यह अपन नाजक नकाशी मॉडल क वलए परवसदद

ह वजसम क मल तवचा सोिदनशील नथन ो सतकि

कान और मरबत िााग ो क शरषठतर परवतरप क

परदवशित वकया गया ह

- यह भारतीय और फारसी ततव ो का एक ससममशरण

- सोवकससा उततर परदश म सथथत ह

Q13) उततर (a)

सपषटीकरण

का िर वसोह ज एक महान य दा थ वबहार स

सोबोवित थ

Q14) उततर (b)

सपषटीकरण

िललालर शबद बड भ-सवावमय ो क वलए परय ग

वकया जाता था

FC19H1003 31

Q15) उततर (c)

सपषटीकरण

- अररकमड एक तिीय बसती थी जहाो दर दश ो स

आन िाल जहाज ो का माल उतारा जाता था

- यहाो पर ईोि ो का एक विशाल ग दाम वमटटी क

बतिन (वजनम एमफ रा - द हरी मवठय ो का लोबा

घडा - शावमल ह) और एरिाइन (Arretine)

मदभाोड पाए गए थ

- इस थथान पर र मन दीपक काोच क बन पातर और

रतन भी पाए गए थ

Q16) उततर (a)

सपषटीकरण

- मिनदर सोगम कविताओो म उसललसखत एक

तवमल शबद ह वजसका अथि ह ldquoतीन परमखrdquo

- यह तीन सततारि पररिार ो क मसखयाओो क वलए

परय ग वकया जाता ह च ल चर और पाणडय

Q17) उततर (c)

सपषटीकरण

- ऋग िद म सभा विदाथा तथा गण जसी

जनजावतय ो पर अथिा किोब पर आिाररत

सभाओो का उललख ह

- आरसमभक िवदक काल म सभाओो और सवमवतय ो

का विशि महतव ह ता था

- यहाा तक की मसखया अथिा राजा भी उनका

समथिन परापत करन क वलए आतर रहत थ

Q18) उततर (a)

सपषटीकरण

- जन िमि न ईशवर क अससततव क मानयता त दी ह

वकनत उसन ईशवर क वजना क पद स नीच रखा

- जन िमि न बौद िमि की तरह िणि परणाली की

भरतिना नही ो की थी

Q19) उततर (d)

सपषटीकरण

- च ल ो और पाणडय ो न शसकतशाली तिीय शहर ो का

विकास वकया था

- च ल ो का सबस महतवपणि शहर पहार (या

कािरीपटटीनम) था |

- मदरई पाणडय ो की राजिानी थी

Q20) उततर (b)

सपषटीकरण

- ldquoबदचररतrdquo बद का जीिन-ितताोत ह

- इस अशवघ ि क दवारा वलखा गया था

Q21) उततर (a)

सपषटीकरणः

- तवमल कवि अपपर भगिान वशि क भकत थ

- इस परकार ि एक नयनार सोत थ

Q22) उततर (d)

सपषटीकरणः

- समदरगपत एक परवसद गपत शासक था

- उसन वसक ो पर िीणा बजात हए अपनी छवि

अोवकत करिाई थी

- यह सोगीत क परवत उसक परम क दशािता ह

- हम उसकी इलाहाबाद परशससत स महतवपणि

ऐवतहावसक जानकारी वमलती ह वजसकी रचना

उसक दरबार क कवि हररसन न की थी

Q23) उततर (b)

सपषटीकरणः

- विकरम सोित की शरआत ििि 58 ईसा पिि म

चनदरगपत वदवतीय न की थी

- यह शक ो पर उसकी जीत और उस विकरमावदतय

की पदिी वमलन क उपलकषय म आरमभ वकया गया

था

FC19H1003 32

- बानभटट न हिििििन का जीिन-ितताोत हििचररत

(ज सोसकत म थी) वलखी थी

Q24) उततर (c)

सपषटीकरणः

- सोवि-विगरावहका यद एिो शाोवत का मोतरी

- साथििाह वयापाररय ो क कावफल ो का नता

Q25) उततर (a)

सपषटीकरणः

- जआन झाोग (हसआन रताोग ndash Hsuang Tsang)

एक चीनी यातरी था ज हिििििन क शासनकाल म

भारत आया था

- ििि 630 ईसवी स ज दशक आरमभ हआ था उसम

जआन झाोग मधय एवशया ईरान और

अफग़ावनसतान की यातरा करन क पशचात कशमीर

क रासत स भारत आया था

- उसन उततर स पिि तक की यातरा की और िह

लगभग 2 ििि वबहार म रहा

- जआन झाोग न नालनदा विशवविदयालय म विदयावथिय ो

और विदवान ो क साथ पारसपररक विचार-विमशि

वकया थथानीय भािाओ ा म वनपणता परापत की तथा

बौद सतप ो की ख ज की

Q26) उततर (c)

सपषटीकरणः

- परदवकषणा पथ बौद िासतकला म सतप क चार ो

ओर बनाया जान िाला एक घमािदार पथ ह ता

- परशन म वदए गए बाकी क तीन ो ततव वहोद मसनदर ो की

िासतकला क भाग ह

Q27) उततर (d)

सपषटीकरणः

परशन म वदए गए सभी मोवदर ो म वयापक रप स

ईोि ो (पकी ईोि ो) का परय ग पतथर ो क साथ हआ

Q28) उततर (c)

सपषटीकरण

- महममद कली कतब शाह ग लकणडा का सलतान

था

- िह अकबर का समकालीन था

- सावहतय और िासतकला म उसकी अतयाविक

रवच थी

- िह एक महान कवि था

- िह दसखनी उदि फारसी और तलग म वलखता था

- उसन अपन पीछ एक विसतत वदिान (सोगरह)

छ डा ह

- अभी हाल ही म तलोगाना म ग लकणडा क वकल

क अनदर खदाई वकय गए बाग-ए-नाया वकला

बाग क चार ो ओर रप-रखा क मानवचतरण क

वलए भारतीय परातासतवक सिकषण (The

Archaeological Survey of India ndash ASI)

गराउणड पनीिर विोग रडार (Ground Penetrating

Radar) का परय ग करगा

Q29) उततर (a)

सपषटीकरणः

- वसलपपावदकारम एक तवमल महाकावय ह वजसकी

रचना इलाोग क दवारा लगभग 1800 ििि पिि की

गई थी

- यह क िलन नामक एक वयापारी की कहानी ह

ज माििी नामक एक गवणका (िशया) स परम

करन लगा था

- मवनमकलाई क िलन और माििी की पतरी की

कहानी ह

Q30) उततर (a)

सपषटीकरण

- चरक आयिद और वचवकरता की एक महतवपणि

रचना चरक सोवहता क लखक ह

- बरहमगपत क अपनी रचना बरहम-सफि-वसदानत

(ज एक खग लीय रचना ह) क कारण परवससद

वमली

FC19H1003 33

- बगदाद म इसका अनिाद अरबी भािा म वकया

गया था

- इसका इसलावमक गवणत और खग ल-विजञान पर

महतवपणि परभाि पडा था

- बाद म अपन जीिनकाल म बरहमगपत न

ldquoखोडखयाकrdquo वलखी ज एक खग लीय पससतका

(एक छ िी पसतक) थी

- इसम आयिभटट की अिि-रावतर क परतयक वदन की

शरआत परणाली का परय ग वकया गया था

Q31) उततर (c)

सपषटीकरण

- अमीर खसर एक परवसद सफी सोगीतकार कवि

और विदवान थ

- 1318 म उनह ोन पाया वक इस भवम (वहोदसतान) क

हर कषतर म अलग-अलग भािा थी लाहौरी

कशमीरी दवारसमदरी (दवकषणी कनाििक म)

तलोगाना (आोधर परदश म) गजरी (गजरात म)

माबारी (तवमलनाड म ) अििी (पिी उततर परदश

म) और वहोदिी (वदलली क आस-पास क कषतर म)

आवद

- उनह न यह बताया वक सोसकत वकसी भी कषतर स

सोबोवित नही ो थी और किल बराहमण ही इस भािा

का जञान रखत थ

Q32) उततर (c)

सपषटीकरण

- वहरणय-गभि सववणिम गभि क सोदवभित करता ह

- जब बराहमण ो की सहायता स यह अनषठान वकया

जाता था त यह माना जाता था वक बवल दन िाल

का कषवतरय क रप म पनजिनम ह गा

Q33) उततर (d)

सपषटीकरण

- कदमई भवम राजसव पर कर क सोदवभित करता

- गवावलयर परशससत म नागभि क दवारा वकय गए

श िण का िणिन वकया गया ह |

- नागभि एक परवतहार राजा था

Q34) उततर (b)

सपषटीकरण

- राजतरो वगनी 12िी ो शताबदी म कलहन क दवारा

रवचत एक सोसकत पसतक (िकसट) ह

- यह परारसमभक भारत की ऐवतहावसक इवतितत थी

- तकि सोगत रप स इस अपन परकार की सिोततम

और सिािविक विशवसनीय कवत माना जाता ह

- यह कशमीर कषतर क पराचीनतम समय स लकर

उसकी रचना की तारीख तक क समपणि इवतहास

का आचछादन करती ह

Q35) उततर (c)

सपषटीकरण

- गााि की आम सभा क ldquoउरrdquo कहा जाता था

- ldquoउरrdquo म गााि क सभी कर दन िाल वनिासी

शावमल ह त थ

Q36) उततर (a)

सपषटीकरण

- वदलली सलतनत म ldquoतारीखrdquo इवतहास लखन का

एक रप था

- ldquoतािरीखrdquo क लखक विदवान परि ह त थ वजनम

सवचि परशासक इतयावद शावमल थ

Q37) उततर (a)

सपषटीकरण

- अलाउददीन सखलजी अपन सवनक ो क ितन का

भगतान नकद म करता था न वक इकता क रप

- सवनक अपना सामान वदलली म वयापाररय ो स

खरीदत थ अतः इस बात का भय था वक वयापारी

कही ो िसतओो का मलय न बिा द

- इसकी र कथाम क वलए अलाउददीन सखलजी न

वदलली म कीमत ो क वनयसित वकया

FC19H1003 34

- अविकारीगण धयानपििक मलय ो का सिकषण करत

थ तथा ज वयापारी वनिािररत मलय पर माल नही ो

बचत थ उनक दसणडत वकया जाता था

Q38) उततर (d)

सपषटीकरण

- वदलली सििपरथम त मर राजपत ो क अिीन उनक

सामराजय की राजिानी बनी थी

- 12िी ो शताबदी क मधय म अजमर क चौहान ो

(वजनह चाहमान ो क नाम स भी जाना जाता ह) न

त मर राजपत ो क परावजत वकया था

- त मर ो और चौहान ो क अिीन वदलली एक

महतवपणि िावणसजयक क दर बन गया था

- कई जन वयापारी यहाा रहन लग थ और उनह ोन

कई मोवदर भी बनिाए

- यहाा पर मवदरत वसक वजनह ldquoदहलीिालrdquo क नाम

स जाना जाता था वयापक रप स परचलन म थ

Q39) उततर (c)

सपषटीकरण

- म ठ की मसिद का वनमािण वसको दर ल दी क

राजयकाल म उसक मिी क दवारा करिाया गया

था

- बगमपरी मसिद का वनमािण महममद तगलक क

शासनकाल म हआ था

- यह मसिद विशव का पणयथथान (The

Sanctuary of the World) और वदलली म महममद

तगलक की नई राजिानी जहाोपनाह की मखय

मसिद थी

- कववत- अल - इसलाम मसिद का विसतार

इलतसिश और अलाउददीन सखलजी न वकया था

- मीनार का वनमािण तीन सलतान ो कतबददीन ऐबक

इलतसिश और वफर ज शाह तगलक क दवारा

करिाया गया था

Q40) उततर (c)

सपषटीकरण

- मगल ो क अिीन मनसबदार शबद उस वयसकत क

वलए सोदवभित वकया जाता था वजसक पास मनसब

(अथाित पद) ह ता था

- उस अपना ितन राजसव कायो वजनह जागीर कहत

थ क रप म परापत ह ता था

Q41) उततर (b)

सपषटीकरण

- ldquoभारत छ ड आोद लनrdquo वबरविश शासन क

सखलाफ ल ग ो का एक सवाभाविक विदर ह था

- असखल भारतीय काोगरस सवमवत न 8 अगसत 1942

क बमबई म एक बठक का आय जन वकया था

- इस बठक म परवसद सोकलप ldquoभारत छ ड rdquo क

पाररत वकया गया और इस उददशय क परापत करन

क वलए गाोिी क नततव म एक अवहोसक जन सोघिि

आोद लन की शरआत का परसताि वदया गया

- लवकन अगल ही वदन गाोिी और काोगरस क अनय

परमख नताओो क वगरफतार कर वलया गया

- काोगरस क एक बार वफर अिि घ वित वकया गया

था

Q42) उततर (c)

सपषटीकरण

- साइमन कमीशन यनाइविड वको गडम क सात

साोसद ो का एक समह था

- इस वबरविश भारत क वलए सोििावनक सिार ो का

सझाि दन क वलए गवठत वकया गया था

- इस आय ग म िररषठ वबरविश राजनता सर जॉन

साइमन क नततव म किल वबरविश सदसय ही

शावमल थ

- इसवलए भारत क ल ग ो न साइमन कमीशन क

आगमन क विरद आोद लन वकया था

Q43) उततर (a)

सपषटीकरण

bull दादा भाई नौर जी भारत म वबरविश शासन क

आवथिक पररणाम ो क बार म अपनी विर िी

(परवतकल) राय क वलए जान जात थ

FC19H1003 35

bull अपन कई लख ो और भािण ो म विशि रप स

ldquoपाििी एो ड अन-वबरविश रल इन इसणडया

(Poverty and Un-British Rule in India) म

नौर जी न यह तकि वदया वक भारत पर अतयविक

कर लगाया गया था और इसकी सोपवतत इोगलड की

ओर परिावहत की जा रही थी

bull उनह ोन पराचीन भारतीय गरोथ ो की वयाखया करन

का और भारतीय ो क आिविशवास क बहाल

करन पर कायि नही ो वकया था

उनह ोन वकसी और बात स पहल सभी सामावजक

बराइय ो क उनमलन की आिशयकता पर भी बल

नही ो वदया था

Q44) उततर (c)

सपषटीकरण

bull अगसत 1932 म वबरविश परिानमोतरी मकड नालड न

अपन साोपरदावयक परसकार (The Communal

Award) की घ िणा की थी

bull यह भारत क कई साोपरदावयक वहत ो क बीच विवभनन

सोघिो क हल करन क वलए वबरिन का एकतरफा

परयास था

bull यह परसकार (Award) बाद म 1935 क

अविवनयम (The Act of 1935) म शावमल वकया

गया था

bull इस साोपरदावयक परसकार न मससलम ो क वलए

आरवकषत एक अलग वनिािचक मणडल फॉमिल का

विसतार अनय अलपसोखयक ो क वलए वकया था

वजसम वसख ो भारतीय ईसाइय ो आोगल-भारतीय

समदाय यर पीय समदाय तथा विवशषट कषतरीय

समह ो क शावमल वकया गया था

bull गाोिी न इस परसताि क भारतीय समाज क

विभावजत करन क वलए एक घवणत वबरविश

सावजश क रप म दखा और उसक सखलाफ

आमरण अनशन वकया

Q45) उततर (b)

सपषटीकरण

मौजदा आयात और वनयाित क अवतररक़त

औपवनिवशक भारत क वनमनवलसखत खचो क

वलए एक विशिवनवशचत िन रावश भी दनी पडती

थी

(i) परशासन क वयय

(ii) सना क रख-रखाि क वयय

(iii) यद क वयय

(iv) सिावनितत अविकाररय ो की पशन तथा

(v) वबरिन दवारा अपनी उपवनिश बसती

(कॉल नी) क रख-रखाि क वयय

इनह गह शलक (Home Charges) क रप म

जाना जाता था और लगभग परी तरह स भारत क

दवारा इनका भगतान वकया जाता था

bull गह शलक म वनमनवलसखत घिक शावमल थ

(i) भारतीय ऋण पर दय बयाज

(ii) ईसट इोवडया को पनी क शयरिारक ो क

लाभाोश

(iii) लोदन म भारत कायािलय चलान क वलए िन

(iv) भारत म वनयकत वबरविश कवमिय ो क ितन

और पशन का भगतान करन क वलए िन

(v) रलि पर बयाज

(vi) नागररक और सनय शलक

(vii) इोगलड म सट र (सामगरी) की खरीद

Q46) उततर (b)

सपषटीकरण

bull भारतीय राषटर ीय काोगरस का लाहौर सतर 1929 म

जिाहरलाल नहर की अधयकषता म आय वजत

वकया गया था

bull इस सतर म भारतीय राषटर ीय आोद लन स समबसित

कई महतवपणि पररणाम सामन आय थ

(i) सििपरथम इस सतर म काोगरस क अधयकष पद

पर जिाहरलाल नहर क चना गया था ज

काोगरस म िामपोवथय ो की बिती हई ताकत

का सपषट सोकत था

(ii) दसरा इस सतर म पहली बार काोगरस न पणि

सवतोतरता की माोग क उठाया था

इस परकार की माोग काोगरस मोच स पहल कभी भी

नही ो उठाई गई थी

Q47) उततर (b)

सपषटीकरण

FC19H1003 36

bull इस ररप िि न वकसी भी समदाय क वलए पथक

वनिािचक मोडल अथिा अलपसोखयक ो क वलए

भाराोश की वसफाररश नही ो की थी

bull तथावप इस ररप िि न उन पराोत ो म अलपसोखयक

सीि ो क आरकषण की अनमवत दी थी जहाा पर कम

स कम दस परवतशत अलपसोखयक ह

bull लवकन यह समदाय क आकार क अनपात म ह ना

चावहए था

bull इस ररप िि म भारत क वलए पणि सवतोतरता क

वलए क ई पराििान नही ो था

Q48) उततर (c)

सपषटीकरण

bull आरो वभक िवदक आयो का िमि मखय रप स

परकवत की पजा और यजञ था

bull परारो वभक आयि िमि परकवत की पजा क समान था

bull िासति म उनक चार ो ओर की शसकतयाा वजनह न

त ि वनयोवतरत कर सकत थ और न ही समझ पाए

थ उनह वदवयता क साथ वनिवशत वकया गया तथा

उनह मादा या नर दिीदिताओो क रप म

परतीकतव वकया गया था

bull उनह ोन कछ यजञ ो का भी वनषपादन वकया था

Q49) उततर (b)

सपषटीकरण

bull सडक और नदी-मागि (जल-मागि) डकती स

सरवकषत नही ो थ

bull उललखनीय ह वक हिििििन क शासनकाल क

दौरान यआन चिाोग (हयएन साोग) का सारा

सामान लि वलया गया था

Q50) उततर (c)

सपषटीकरण

परशन म वदए गए द न ो कथन सही ह

Q51) उततर (b)

सपषटीकरण

bull परोदर दास एक सोत और भगिान कषण क एक

महान भकत थ

bull परोदर दास क कनाििक सोगीत क वपतामह क

रप म जाना जाता ह

bull यदयवप उनक जनम-थथान क बार म काफी

अिकल लगाई जाती रही ह

bull तथावप अब कननड विशवविदयालय हमपी क दवारा

गवठत एक विशिजञ सवमवत इस वनषकिि पर पहोची

ह वक उनका जनम थथान सोभितया कनाििक का

एक छ िा-सा गााि कषमपरा (वशिम गगा वजला)

था

Q52) उततर (c)

सपषटीकरण

bull शरी तयागराज शरी शयाम शासतरी और शरी मथसवामी

दीवकषतर क कनाििक सोगीत की वतरमवति माना

जाता ह

bull उनक कारण ही 18िी ो-19िी ो शताबदी म कनाििक

सोगीत का सववणिम यग आया था

Q53) उततर (d)

सपषटीकरण

bull अभी हाल ही म लौह यगीन-महापािावणक काल

का 2000 ििि पराना एक दलिभ सारक फगस

(Sarcophagus) (पतथर का ताबत) क ललम क

वियर गाोि (क वयलडी क पास वजला क वझक ड

करल राजय) की एक रॉक-कि गफा स ख जा गया

bull यह ताबत वजसम हविय ो क िकड थ खदाई क

दौरान वमला

bull अभी तक इस परकार की दलिभ ख ज करल क

मातर द ही थथान ो स हई ह

bull य द न ो सारक फगी (Sarcophagi) (पतथर क

ताबत) चियर और अथ ली (वजला क वझक ड) क

महापािाण थथल ो स वमल ह

Q54) उततर (a)

सपषटीकरण

FC19H1003 37

दवकषण भारत म महापािाण सोसकवत एक पणि

विकवसत लौह यगीन सोसकवत थी

Q55) उततर (d)

सपषटीकरण

bull च ल पाणडय और करलपतर (चर) इन तीन ो का

उललख अश क क अवभलख ो म वकया गया ह

bull सोभितः य भौवतक सोसकवत क उततर

महापािावणक चरण म थ

Q56) उततर (d)

सपषटीकरण

bull भीमा-क रगाोि की लडाई ततीय आोगल-मराठा

यद का वहससा थी

Q57) उततर (b)

सपषटीकरण

bull राजकमार शकल न गाोिीजी क चोपारण आन तथा

वतनकवथया परणाली स जडी समसया की जाोच क

वलए रारी करन क वलए दश भर म उनका

अनसरण वकया था

bull बज वकश र राजदर परसाद महादि दसाई और

नरहरी पाररख चोपारण सतयागरह क दौरान गाोिी

जी क सहय गी थ

Q58) उततर (b)

सपषटीकरण

bull बराहमण ो और बौद मठिाररय ो क कर-मकत गााि

अनदान म दन की परथा सतिाहन ो न आरमभ की

थी

Q59) उततर (c)

सपषटीकरण

इस कायिकरम क उददशय वनमनानसार ह

(i) बवनयादी पयििन आिाररक सोरचना का विकास

करना

(ii) चयवनत (पहचान वकय गए) कषतर ो म आजीविका क

सजन क वलए दश क साोसकवतक और विरासत

मलय ो क बिािा दना

(iii) विरासत समारक थथल ो पर विशव सतरीय आिाररक

सोरचना विकवसत करक एक सतत तरीक स

पयििक आकििण म िसद करना

(iv) थथानीय समदाय ो की सवकरय भागीदारी क माधयम

स र रगार ो का सजन करना

(v) र रगार उतपादन और आवथिक विकास क वलए

पयििन कषमता का उन पर परभाि का उपय ग

करना तथा

(vi) िारणीय पयििन आिाररक सोरचना का विकास

करना और उसका उवचत सोचालन तथा

रखरखाि सवनवशचत करना

Q60) उततर (b)

सपषटीकरण

bull यह वनकाय ििि 1987 म अससततव म आया था

bull यह एक राषटर ीय सतर का शीिि सोगठन ह ज भारत

सरकार क जनजातीय मामल ो क मोतरालय क

परशासवनक वनयोतरण क अिीन काम कर रहा ह

bull इसका पोजीकत और परिान कायािलय नई वदलली

म सथथत ह

Q61) उततर (c)

सपषटीकरण

bull परमचोद क उपनयास ो म परमाशरम रोगभवम गबन

कमिभवम और ग दान शावमल ह

bull ग रा रिी ोदरनाथ िग र क दवारा रवचत उपनयास ह

bull अभी हाल ही म मोशी परमचोद की 138िी ो जयोती दश

भर म मनाई गई थी

Q62) उततर (b)

सपषटीकरण

bull ldquoवगदाrdquo पोजाब (भारत) एिो पावकसतान की

मवहलाओो क दवारा तयौहार क समय और फसल

की बिाई तथा किाई क अिसर पर वकया जान

िाला एक पारोपररक दहाती नतय ह

FC19H1003 38

bull इस नतय क माधयम स पोजाबी मवहलाऐो अपनी

परसननता परकि करती ह तथा वगदा क परदशिन क

माधयम स परि िचिसव िाल समाज म मवहलाओो

की दबी हई भािनाओो क परकि करती ह

bull चोवक इस नतय का परि ो क साथ क ई सोबोि नही ो

ह अतः किल मवहलाऐो ही इसम भाग ल सकती

bull हर साल तीज समार ह क दौरान पोजाब म वगदा

नतय वकया जाता ह

तीज भारत क कछ भाग ो म मवहलाओो क दवारा

मनाया जान िाल कई तयौहार ो क वलए एक

वयापक नाम ह

Q63) उततर (a)

सपषटीकरण

- मजम-उल-बहरीन या द समदर ो का सोगम

नामक उललखनीय रचना दारा वशक ह क दवारा

वलखी थी

- भारत क उपराषटर पवत शरी एम िकया नायड न कहा

ह वक राजकमार दारा वशक ह की रचनाएा शाोवत

और सदभाि क बिािा दन क वलए एक तारा सर त

क रप म सामन आ सकती ो ह

- उपराषटर पवत गत ििो क भला वदए गए राजकमार

दारा वशक ह क परदवशित परचवलत करन हत

आय वजत एक परदशिनी का दौरा करन क बाद एक

सभा क सोब वित कर रह थ

- इस परदशिनी का आय जन फर क इस गौवियर

(Francois Gautier) क दवारा lsquoइोवदरा गाोिी नशनल

सिर फॉर द आििसrsquo (The Indira Gandhi

National Centre for the Arts) नई वदलली म

वकया गया था

Q64) उततर (c)

सपषटीकरण

- ग मतशवर परवतमा जन भगिान बाहबली क

समवपित ह

- यह एक एक-चटटानी पतथर की मवति ह

- राषटर पवत राम नाथ क विोद न शरिणबलग ला

(कनाििक) म आय वजत वकय जान िाल भवय

अवभिक समार ह महामसतकावभिक का

उदघािन वकया था

- यह समार ह 12 ििो म एक बार ह ता ह

Q65) उततर (c)

सपषटीकरण

bull पराची घािी पराची नदी क चार ो ओर फली हई थी

bull पराची घािी िीर-िीर विलपत ह गई थी

bull पराची नदी भिनशवर स वनकलती ह

bull यह महानदी की एक सहायक नदी ह और यह

परी खदाि किक तथा जगतवसोहपर वजल ो स

ह कर बहती ह

bull इस नदी क पर कषतर क पराची घािी कहा जाता ह

bull यह नदी बोगाल की खाडी म वगरती ह

परातासतवक साकषय स पता चलता ह वक पराची घािी

सभयता हडपपा और म हनज दाड द न ो की

पिििती ह

Q66) उततर (d)

सपषटीकरण

य समारक छतरपर वजल (मधय परदश) म विोधयाचल

पिित शरोखला म सथथत ह

Q67) उततर (a)

सपषटीकरण

bull थॉिस ऑन पावकसतान नामक पसतक डॉ बी

आर अमबडकर न वलखी थी

bull डॉ बी आर अमबडकर की जयोती क अिसर पर

भारत क राषटर पवत न भारत की इस महान हसती

क शरदाोजवल अवपित की थी

bull डॉ बी आर अमबडकर न 1924 म वडपरथड

कलावसर इोसटीटयि (दवलत िगि सोथथान -

बवहषकत वहतकाररणी सभा) और 1927 म समाज

समता सोघ की थथापना की थी

bull अमबडकर का धयान वशकषा कषतर की ओर भी था

bull उनह ोन वशकषा क वनमन िगो म फलान क वलए

पीपलस एजकशन स साइिी (The Peoples

Education Society) क नाम स महाविदयालय ो क

नििकि और छातरािास ो की थथापना की थी

FC19H1003 39

Q68) उततर (b)

सपषटीकरण

bull महरगि भारतीय उपमहादवीप म एक परवसद

निपािाण बसती ह ज बलवचसतान पराोत

पावकसतान म सथथत ह

bull दचपलली (आोधर परदश) क पास नागलर नदी क

पिी ति ो पर चना पतथर क बलॉक क विशाल

विसतार म एक पिि-ऐवतहावसक रॉक आिि थथल की

ख ज की गई ह

bull इसन 1500-2000 ईसा पिि क दौरान गोिर (आोधर

परदश) म विकवसत निपािाण सभयता पर परकाश

डाला ह

Q69) उततर (c)

सपषटीकरण

bull 12िी ो सदी और 13िी ो सदी म काकाविय िोश का

उदय हआ था

bull ि पहल कलयाण क पवशचमी चालकय ो क सामोत थ

bull परारोभ म उनह ोन िारोगल (तलोगाना) क पास एक

छ ि स कषतर पर शासन वकया था

bull उनह ोन ldquoनायक वयिथथाrdquo की शरआत की थी

वजस बाद म विजयनगर क राय शासक ो न

अपनाया और विकवसत वकया था

Q70) उततर (a)

सपषटीकरण

bull गाोिीजी क अनशन स वमल मावलक ो पर दबाि

पडा था ज अोततः शरवमक ो क ितन म 35 परवतशत

की िसद करन क वलए सहमत हए थ

bull गगल (Google) न अनसया साराभाई वजनह ोन

भारत क शरवमक आोद लन म एक अगरणी भवमका

वनभाई थी की 132िी ो जयोती डडल (Doodle) का

वनमािण करक मनाई

Q71) उततर (d)

सपषटीकरण

भारत स यनसक की मानिता की अमति साोसकवतक

विरासत की परवतवनवि सची म वनमनवलसखत शावमल ह

bull कवडयटटम करल का सोसकत रोगमोच

bull मवडयिि करल का अनषठान रोगमोच और नतय

नाविका

bull िवदक मि जाप की परोपरा

bull राजथथान क कालबवलया ल क गीत और नतय

bull रामलीला रामायण का पारोपररक परदशिन

bull सोकीतिन मवणपर का अनषठान गायन ढ ल िादन

और नतय

bull रममन भारत क गििाल वहमालय का िावमिक

तयौहार और अनषठान रोगमोच

bull जाोदीयाला गर पोजाब क ठठर ो की पीतल और

ताोब क वशलप स वनवमित बतिन ो की पारोपररक कला

bull छाऊ नतय पिी भारतीय राजय ो म जनमी शासतरीय

भारतीय नतय कला

bull लददाख का बौद मि जाप िर ाोस-वहमालयी लददाख

कषतर तथा जमम-कशमीर म पवितर बौद गरोथ ो का पाठ

bull य ग

bull नौर र

bull को भ मला

Q72) उततर (b)

सपषटीकरण

bull भारत क राषटर पवत शरी राम नाथ क विोद न

वकसामा नागालड म हॉनिवबल मह रति और

राजय गठन वदिस समार ह का उदघािन वकया

था

bull हॉनिवबल मह रति का नाम भारतीय हॉनिवबल क

नाम पर पडा ह ज एक विशाल और रोगीन जोगली

पकषी ह

bull यह पकषी नागालड राजय की अविकतर जनजावतय ो

की ल ककथाओो म उसललसखत ह

bull नागालड की परमख मानयता परापत जनजावतयाा ह

अोगामी आओ चखसोग चाोग ककी रगमा और

रवलोग आवद

bull ओोग जारिा और ससिनलीस अोडमान-वनक बार

दवीप समह की जनजावतयाा ह

FC19H1003 40

Q73) उततर (c)

सपषटीकरण

bull दकन म राषटर कि शासन दसिी ो सदी क अोत तक

लगभग 200 ििो तक रहा था

bull राषटर कि शासक अपन िावमिक विचार ो म सवहषण

bull उनह ोन न किल शि िमि और िषणि िमि बसलक

जन िमि क भी सोरकषण वदया था

bull एल रा म वशि क परवसद रॉक कि मोवदर का

वनमािण नौिी ो सदी म राषटर कि राजा कषण परथम न

करिाया था

bull उसका उततराविकारी अम घििि जन था लवकन

उसन अनय िमो क भी सोरकषण परदान वकया था

bull राषटर कि ो न मसलमान वयापाररय ो क बसन की

अनमवत दी थी

bull उनह न अपन अविराजय ो म इसलाम क उपदश दन

की भी अनमवत दी थी

bull अभी हाल ही म पाोडिलागटटा (तलोगाना) क

परागवतहावसक चटटान वचतर ो क कषरण की बिती हई

घिनाएा एक गोभीर वचोता का वििय ह

bull यह परागवतहावसक चटटान क नकसान पहाचा

सकता ह

bull पाोडिलागटटा वनमनवलसखत क वलए जाना जाता ह

- 10000 ईसा पिि स 8000 ईसा पिि क वचवतरत

चटटानी आशरय ो क वलए

- राषटर कि काल क एक 8 िी ो सदी क

वशलालख क वलए और

- 12िी ो सदी क काकविय सामराजय क वभवतत

वचतर ो क वलए

Q74) उततर (b)

सपषटीकरण

bull 1828 म राजा राम म हन रॉय न एक नय िावमिक

समाज बरहम सभा की थथापना की थी वजस बाद

म बरहम समाज क नाम स जाना गया था

bull दिदरनाथ िग र न ततवब विनी सभा की अधयकषता

की थी ज आधयासिक सतय की ख ज म सोलि

थी

bull इसका उददशय वहोद िमि क शद करन का और

एकशवरिाद (एक ईशवर म आथथा) का परचार करना

था

bull नय समाज की थथापना क आिार थ कारण

(तकि ) क द सतमभ तथा िद और उपवनिद

bull अभी हाल ही म सािारण बरहम समाज का कछ

काननी मदद ो क लकर पवशचम बोगाल सरकार क

साथ काननी वििाद चल रहा ह

Q75) उततर (c)

सपषटीकरण

bull भारत म वचशती वसलवसल की थथापना खवाजा

म इनददीन वचशती क दवारा की गयी थी

bull ि 1192 ईसवी क आसपास भारत आय थ

bull वचशतीय ो क बारहिी ो शताबदी क उततरािि म भारत

म आन िाल सफीय ो क समह ो म सबस

परभािशाली माना जाता ह

bull उनह ोन थथानीय िातािरण क साथ सफलतापििक

अनकलन वकया और उनह ोन भारतीय भसकत

परोपराओो क कई पहलओो क अपनाया

bull अजमर म सफी अपरकि खवाजा म इनददीन वचशती

की ऐवतहावसक दरगाह क एक नया रप दन की

तयारी की जा रही ह

bull इस 13िी ो शताबदी की दरगाह क ldquoसवचछ

आइकॉवनक थथल ोrdquo (Swacch Iconic Places) म

शावमल वकया गया ह ज परवतवषठत विरासत

आधयासिक और साोसकवतक थथान ो पर क वदरत

य जना ह

FC19H1003 41

ANSWERS amp EXPLANATION OF

NCERT History Class VI-X + Current Affairs

(FC19E1003)

Q1) Answer c

Explanation

Rigveda consists of more than a

thousand hymns dedicated to gods and

goddesses These hymns were

composed by sages and learnt by men

however a few were composed by

women like Apala Ghosa Lopamudra

Maitreyi and Gargi

Rigveda consists of many hymns in the

form of dialogues We get an example of

a dialogue between a sage named

Vishwamitra and two rivers (Beas and

Sutlej) that were worshipped as

goddesses This suggests that he

belonged to the Vedic period

Q2) Answer b

Explanation

Traces of ash have been found from

Kurnool Caves suggesting that people

were familiar with the use of fire

It is situated in Andhra Pradesh

Q3) Answer c

Explanation

Burzahom is a prehistoric site in

present day Kashmir where people built

pit houses which were dug into the

ground with steps leading into them

These may have provided shelter in cold

weather

Q4) Answer c

Explanation

Epigraphy is defined as the study of

inscriptions

Manuscriptology is the study of history

and literature through the use of hand

written documents

Palaeography refers to the study of

ancient writing systems and the

deciphering and dating of historical

manuscripts

Numismatics refers to the study of

coins

Q5) Answer a

Explanation

Charaka Samhita was written by

Charaka and is an important book on

Ayurveda and medicine

He was a practitioner of the traditional

system of Indian medicine known as

Ayurveda

Charaka is thought to have flourished

sometime between the 2nd century BCE

and the 2nd century CE

Q6) Answer b

Explanation

Bhaga refers to the tax on crops which

was fixed at 16th of the production

Kammakaras is the term used for the

landless agricultural labour class

Ashvamedha also known as horse

sacrifice is a ritual where a horse is let

loose to wander freely and it was

guarded by the rajarsquos men

Q7) Answer (d)

Explanation

In the Rigvedic period horses were

yoked to chariots that were used in

battles fought to capture land cattle

etc This suggests that the use of horse

chariots began much before the period

of Mahajanapadas

The battles were fought in the Rigvedic

period for cattlersquos lands water an even

to capture people Most men took part

in these wars however there was no

regular army but there were assemblies

where people met and discussed

matters of war Regular armies became

a feature in the Mjahajanapada period

including vast armies of foot soldiers

chariots and elephants

RAUSIAS-FC19E1003 42

Q8) Answer (a)

Explanation

Buddha belonged to the Sakya clan and

passed away at Kusinara

Buddha taught in Prakrit which was the

common language of people

Q9) Answer c

Explanation

There were six schools of philosophy in

ancient India These are known as

Vaishesika Nyaya Samkhya Yoga

Purva Mimansa and Vedanata or Uttara

Mimansa They were founded by sages

Kanada Gautama Kapila Patanjali

Jamini and Vyasa respectively

Q10) Answer b

Explanation

The teachings of Mahavira were

compiled at Valabhi in 6th century AD

Q11) Answer (c)

Explanation

Chanakya is traditionally identified as

Kautilya or Vishnugupta who authored

the ancient Indian political treatise the

Arthashastra

Q12) Answer d

The national emblem of India is an

adaptation of the Lion Capital atop the

Ashoka Pillar of Sarnath Uttar Pradesh

and is combined with the National

Motto Satyameva Jayate

The Rampurva Bull gets the name from

the site of its discovery Rampurva in

Bihar

It is noted for its delicately sculpted

model demonstrating superior

representation of soft flesh sensitive

nostrils alert ears and strong legs It is

a mixture of Indian and Persian

elements

Sankissa is situated in Uttar Pradesh

India

Q13) Ans(a)

Kunwar Singh was a notable leader during the Revolt of 1857 He belonged

to a royal house of Jagdispur Bihar

Q14) Answer b

Explanation

The term Vellalar was used for large

landowners

Q15) Answer c

Explanation

Arikamedu was a coastal settlement

where ships unloaded goods from

distant lands Finds here include a

massive brick warehouse pottery

including amphorae and Arretine ware

Roman lamps glassware and gems have

also been found at the site

Q16) Answer a

Explanation

Muvendar is a Tamil word mentioned in

Sangam poems meaning three chiefs

used for the heads of three ruling

families the Cholas Cheras and

Pandyas

Q17) Ans (c)

Several tribal or kin-based assemblies

such as the Sabha Vidatha and gana

are mentioned in the Rig-veda The

Sabha and the samiti mattered a great

deal in early Vedic times so much so

that the chiefs or the kings showed an

eagerness to win their support

Q18) Ans (a)

Jainism recognised the existence of the

gods but placed them lower than the

jina and did not condemn the varna

system as Buddhism did

Q19) Answer (d)

Explanation

Cholas and Pandyas had developed

powerful coastal cities The most

important city of Cholas was Puhar or

Kaveripattinam and Madurai was the

capital of Pandyas

Q20) Answer b

Explanation

Buddhacharita is the biography of

Buddha and was written by

RAUSIAS-FC19E1003 43

Ashvaghosha

Q21) Answer (a)

Explanation

Tamil poet Appar was a Shiva devotee

So he was a Nayanar saint

Q22) Answer d

Explanation

Samudragupta was a prominent Gupta

ruler whose coins depict him playing a

veena indicating his love for music We

get important historic information from

his Allahabad Prashasti which was

composed by his court poet Harisena

Q23) Answer (b)

Explanation

Vikrama Samvat was founded by

Chandragupta II in the 58 BC as a

mark of victory over the Shakas and

assumed the title of Vikramaditya

Banabhatta wrote Harshavardhanarsquos

biography the Harshacharita in

Sanskrit

Q24) Answer c

Explanation

Sandhi-vigrahika was the minister of

war and peace

Sarthavaha was the leader of the

merchant caravans

Q25) Answer a

Explanation

Xuan Zang (Hsuan-tsang) was a

Chinese traveller who came during the

reign of Harshavardhana

In the decade that began in 630 AD

Xuan Zang came to India through

Kashmir after visiting Central Asia Iran

and Afghanistan

He travelled from north to east and lived

in Bihar for a couple of years

At Nalanda University Xuan Zang

interacted with students and scholars

mastered local languages and

discovered Buddhist stupas

Q26) Answer c

Explanation

Pradakshina patha is a circular path

laid around a stupa in Buddhist

architecture While the rest are a part of

temple architecture

Q27) Answer d

Explanation

All the above-mentioned temples have

an elaborate use of bricks (baked

bricks) along with stone

Q28) Ans (c)

Muhammad Quli Qutab was the Sultan

of Golconda He was a contemporary of

Akbar was very fond of literature and

architecture

The Sultan was a great poet and he

wrote in Dakhini Urdu Persian and

Telgu and has left an extensive diwan or

collection

Recently the Archaeological Survey of

India (ASI) will be using Ground

Penetrating Radar (GPR) to map the

contours of the area around the Bagh-e-

Naya Qila excavated garden inside the

Golconda Fort in Telangana

Q29) Answer a

Explanation

Silappadikaram is a famous Tamil epic

which was written by Ilango around

1800 years ago It is a story of a

merchant named Kovalan who fell in

love with a courtesan named Madhavi

Manimekalai tells the story of the

daughter of Kovalan and Madhavi

Q30) Answer (a)

Explanation

Charaka is the author of Charaka

Samhita which is an important work of

Ayurveda and medicines

Brahmaguptarsquos fame rests mostly on his

Brahma-sphuta-siddhanta which was

an astronomical work It was translated

into Arabic in Baghdad and had a major

impact on Islamic mathematics and

astronomy

Late in his life Brahmagupta wrote

Khandakhadyaka which was an

RAUSIAS-FC19E1003 44

astronomical handbook that employed

Aryabhatarsquos system of starting each day

at midnight

Q31) Answer (c)

Explanation

Amir Khusrau was a famous sufi

musician poet and scholar In 1318 he

noted that there was different language

in every region of this land (Hindustan)

Lahori Kashmiri Dvarsamudri (in

Southern Karnataka) Telangana (in

Andhra Pradesh) Gujari (in Gujarat)

Marsquobari (in Tamil Nadu) Awadhi (in

eastern Uttar Pradesh) and Hindawai (in

the area around in Delhi) etc He went

to explain that Sanskrit did not belong

to any region and that only brahmans

knew it

Q32) Answer c

Explanation

Hiranyagarbha refers to the golden

womb When this ritual was performed

with the help of Brahmanas it was

thought to lead to the rebirth of the

sacrificer as a Khastriya

Q33) Answer d

Explanation

Kadamai refers to a tax on land

revenue

Gwalior Prashasti describes the exploits

of Nagabhata who was a Pratihara king

Q34) Answer b

Explanation

Rajatarangini is a Sanskrit text written

by Kalhana in the 12th century

It was historical chronicle of early India

It is justifiably considered to be the best

and most authentic work of its kind

It covers the entire span of history in

the Kashmir region from the earliest

times to the date of its composition

Q35) Answer c

Explanation

ldquoUrrdquo was the general assembly of the

village ldquoUrrdquo consisted of all the

taxpaying residents of an ordinary

village

Q36) Answer (a)

Explanation

Tarikh was a form of history writing in

the Delhi Sultanate The authors of

tawarikhs were learned men which

included secretaries administrators etc

Q37 Answer (a)

Explanation

Alauddin chose to pay his soldiers salaries in cash rather than iqtas The soldiers would buy their supplies from merchants in Delhi and it was thus feared that merchants would raise their prices To stop this Alauddin controlled the prices of goods in Delhi Prices were carefully surveyed by officers and merchants who did not sell at the prescribed rates were punished

Q38) Answer (d)

Explanation

Delhi first became the capital of a

kingdom under the Tomara Rajputs

who were defeated in the middle of the

twelfth century by the Chauhans (also

referred to as Chahamanas) of Ajmer

It was under the Tomaras and

Chauhans that Delhi became an

important commercial centre Many rich

Jaina merchants lived in the city and

constructed several temples Coins

minted here called dehliwal had a wide

circulation

Q39) Answer (c)

Explanation

Moth ki Masjid was built in the reign of

Sikandar Lodi by his minister

Begumpuri mosque built in the reign of

Muhammad Tughluq was the main

mosque of Jahanpanah the ldquoSanctuary

of the Worldrdquo and his new capital in

Delhi

Quwwat al ndash Islam mosque was

enlarged by Iltutmish and Alauddin

Khalji The minar was built by three

Sultansndash Qutbuddin Aybak Iltutmish

and Firuz Shah Tughluq

RAUSIAS-FC19E1003 45

Q40) Answer (c)

Explanation

Under the Mughals mansabdar was

referred to an individual who held a

mansab ie rank and he received his

salary as revenue assignments called

jagirs

Q41) Ans (b)

The Quit India Movement was a

spontaneous revolt of people against

British rule

The All India Congress Committee met

at Bombay on 8 August 1942 It passed

the famous resolution Quit India and

proposed the starting of a non-violent

mass struggle under Gandhis

leadership to achieve this aim But on

the very next day Gandhi and other

eminent leaders of the Congress were

arrested The Congress was once again

declared illegal

Q42) Ans (c)

The Simon Commission refers to a

group of seven MPs from the United

Kingdom constituted to suggest

constitutional reforms for British India

The Commission consisted of only

British members headed by one of the

senior British politicians Sir John

Simon

So the people of India agitated against

the arrival of Simon Commission

Q43) Ans (a)

He was widely known for his

unfavourable opinion of the economic

consequences of the British rule in

India

In his many writings and speeches and

especially in Poverty and Un-British

Rule in India Naoroji argued that India

was too highly taxed and that its wealth

was being drained away to England

He did not interpret the ancient Indian

texts and restored the self-confidence of

Indians And also he did not stress the

need for eradication of all the social

evils before anything else

Q44) Ans (c)

In August 1932 Prime Minister

MacDonald announced his Communal

Award Great Britainrsquos unilateral

attempt to resolve the various conflicts

among Indiarsquos many communal

interests

The award which was later

incorporated into the act of 1935

expanded the separate-electorate

formula reserved for Muslims to other

minorities including Sikhs Indian

Christians Anglo-Indians Europeans

distinct regional groups Gandhi

undertook a ldquofast unto deathrdquo against

that offer which he viewed as a

nefarious British plot to divide the

Indian society

Q45) Ans (b)

In British India apart from existing

imports and exports there was also a

particular amount of money which

colonial India contributed towards

administration maintenance of the

army war expenses pensions to retired

officers and other expenses accrued by

Britain towards maintenance of her

colony These were known as Home

charges and were paid for almost

entirely by India

The Home charges was made of

following components-

- Interest payable on Indian debt

- Dividend to shareholders of East

India Company

- Funds used to support the India

Office in London

- Funds used to pay salaries and

pensions of British personnel

engaged in India

- Interest on the railways

- Civil and military charges

- Store purchases in England

Q46) Ans (b)

The Lahore session of the Indian

National Congress was held in 1929

under the Presidentship of Jawaharlal

Nehru

The Lahore session of the Indian

National Congress witnessed significant

RAUSIAS-FC19E1003 46

developments in the Indian national

movement

- First the election of Jawaharlal

Nehru to the post of Presidentship of

the Congress was a clear indication

of the growing strength of the

Leftists in the Congress

- Secondly it was in this session that

the Congress for the first time raised

the demand for complete

independence Such demand was

not raised from the Congress

platform earlier

Q47) Ans (b)

It did not provide for separate

electorates for any community or

weightage for minorities However it did

allow for the reservation of minority

seats in provinces having minorities of

at least ten per cent but this was to be

in strict proportion to the size of the

community

There was no provision for complete

Independence for India

Q48) Ans (c)

The religion of early Vedic Aryans was

primarily of worship of nature and

Yajnas

The early Aryan religion was kind of

nature worship Actually the forces

around them which they could not

control or understand were invested

with divinity and were personified as

male or female gods And they

performed some Yajnas also

Q49) Ans (b)

The roads and river-routes were not

immune from robbery It is notable that

Yuan Chwang (Hiuen Tsang) was

robbed of his belongings during

Harshvardanarsquos period

Q50) Ans (c)

Q51) Ans (b)

Purandara Dasa was a saint and great

devotee of Lord Krishna

There is much speculation about where

Purandara Dasa regarded as the

Pitamaha of Carnatic music was born

Recently an expert committee

constituted by the Kannada University

Hampi has come to the conclusion that

Kshemapura Shivamogga district

Karnataka is the birth place of

Purandara Dasa

Q52) Ans (c)

Sri Tyagaraja Sri Shyama Shastry and Sri Muthuswami Dikshitar are considered the trinity of Carnatic music and with them came the golden age in Carnatic music in the 18th-19th

century

Q53) Ans d)

Recently a rare sarcophagus (stone

coffin) which is 2000 years old from the

Iron AgendashMegalithic era was discovered

from a rock-cut cave at Viyur village of

Kollam near Koyilandy in Kozhikode

district Kerala

The coffin containing bone fragments

was found during an excavation ldquoSo

far such a rare finding has been

discovered only from two sites

in Kerala Both these sarcophagi were

recovered from Megalithic sites at

Chevayur and Atholi also in Kozhikode

district

Q54) Ans a)

The megalithic culture in South India was a full-fledged Iron Age culture

Q55) Ans d)

The Cholas Pandyas and Keralaputras

(Cheras) mentioned in Ashokan

inscriptions were probably in the late

megalithic phase of material culture

Q56) Ans d)

Q57) Ans (b)

Raj Kumar Shukla followed Gandhiji all

over the country to persuade him to

come to Champaran to investigate the

problem associated with tinkathia

system

RAUSIAS-FC19E1003 47

Brij Kishore Rajendra Prasad Mahadev

Desai and Narhari Parikh accompanied

Gandhi ji during the Champaran

Satyagraha

Q58) Ans (b)

The Satvahanas started the practice of granting tax-free villages to brahmanas and Buddhist monks

Q59) Ans c)

The objectives of the Programme are

listed as under

- Developing basic tourism

infrastructure

- Promoting cultural and heritage

value of the country to generate

livelihoods in the identified regions

- Enhancing the tourist attractiveness

in a sustainable manner by

developing world-class

infrastructure at the heritage

monument sites

- Creating employment through active

involvement of local communities

- Harnessing tourism potential for its

effects on employment generation

and economic development

- Developing sustainable tourism

infrastructure and ensuring proper

Operations and maintenance

therein

Q60) Ans (b)

The Tribal Cooperative Marketing

Development Federation of India

(TRIFED) came into existence in 1987

It is a national-level apex organization

functioning under the administrative

control of Ministry of Tribal Affairs

Govt of India

TRIFED has its registered and Head

Office located in New Delhi

Q61) Ans (c)

Premchandrsquos novels include

Premashram Rangabhumi Ghaban

Karmabhumi and Godan

Gora is a novel written by Rabindranath

Tagore

138th birth anniversary of Munshi

Premchand was celebrated across the

country

Q62) Ans (b)

Giddha is a traditional pastoral dance

performed by the women of the Punjab

India and Pakistan at festival times

and at the sowing and reaping of the

harvest

By this dance the Punjabi women

reveal their joy expel their suppressed

feelings in a male dominated society

through the performance of Giddha

Since this dance has nothing to do with

men only women can participate in it

During the Teej celebrations Giddha

dance is celebrated in Punjab every

year Teej is a generic name for a

number of festivals that are celebrated

by women in some parts of India

Q63) Ans (a)

Dara Shukoh wrote the remarkable

work called ldquoMajma-ul-Bahrainrdquo or the

ldquoThe confluence of two seasrdquo

The Vice President of India Shri M

Venkaiah Naidu has said that Prince

Dara Shukohrsquos writings can come as a

refreshing source for infusing peace and

harmony He was addressing the

gathering after visiting the exhibition

that showcases the forgotten Prince of

yesteryears Dara Shukoh organized by

Mr Francois Gautier at Indira Gandhi

National Centre for the Arts in New

Delhi

Q64) Ans (c)

The statue Gommateshwara is

dedicated to the Jain God Bahubali

It is a monolithic statue

President Ram Nath Kovind

inaugurated the grand anointing

ceremony mdash Mahamastakabhisheka mdash

held once in 12 years at

Shravanabelagola (Karnataka)

Q65) Ans (c)

Prachi Valley had come up around the

Prachi river Prachi Valley gradually

disappeared

RAUSIAS-FC19E1003 48

The Prachi river originates from

Bhubaneswar

It is a tributary of the Mahanadi and

flows through the districts of Puri

Khurda Cuttack and Jagatsinghpur

and the entire region of the river is

termed as the Prachi Valley

It falls into the Bay of Bengal

Archaeological evidence shows that the

Prachi Valley Civilisation predates both

Harappa and Mohenjo-Daro

The Prachi river originates from

Bhubaneswar

Q66) Ans (d)

These monuments are located in

Chhatarpur district Madhya Pradesh

within Vindhya mountain range

Q67) Ans (a)

The book lsquoThoughts on Pakistanrsquo was

written by Dr BR Ambedkar

On the occasion of the birth anniversary

of Dr BR Ambedkar the president of

India pays homage to this icon of India

In 1924 he founded the Depressed

Classes Institute (Bahishkrit Hitkarini

Sabha) and in 1927 the Samaj Samata

Sangh

Another area of attention for Ambedkar

was education For its spread among

the low classes he set up a network of

colleges by the name of Peoples

Education Society and founded hostels

Q68) Ans(b)

Mehrgarh is a famous Neolithic

settlement in the Indian subcontinent

which is situated in Baluchistan

province Pakistan

A pre-historic rock art site is discovered

in the vast expanse of limestone blocks

on the eastern banks of Naguleru river

near Dachepalli (Andhra Pradesh) It

has thrown light on the Neolithic

civilisation that flourished in Guntur

(Andhra Pradesh) during 1500-2000

BC

Q69) Ans (c)

The 12th and the 13th centuries saw

the emergence of the Kakatiyas They

were at first the feudatories of the

Western Chalukyas of Kalyana Initially

they ruled over a small territory near

Warangal (Telangana)

They introduced Nayakships which was

later adopted and developed by the

Rayas of Vijayanagara

Q70) Ans (a)

The fast had effect of putting pressure

on mill owners who finally agreed to

give the workers a 35 per cent increase

in wages

Google celebrated with a doodle the

132nd birth anniversary of Anasuya

Sarabhai who played a pioneering role

in Indiarsquos labour movement

Q71) Ans (d)

The UNESCOrsquos list of the representative

list of the intangible cultural heritage of

humanity from India are

- Koodiyattam Sanskrit Theatre of

Kerala

- Mudiyettu ritual theatre and dance

drama of Kerala

- Tradition of Vedic Chanting

- Kalbelia folk songs and dances of

Rajasthan

- Ramlila Traditional Performance of

the Ramayana

- Sankirtana ritual singing

drumming and dancing of Manipur

- Ramman religious festival and

ritual theatre of the Garhwal

Himalayas India

- Traditional brass and copper craft of

utensil making among the Thatheras

of Jandiala Guru Punjab India

- Chhau dance classical Indian dance

originated in the eastern Indian

states

- Buddhist chanting of Ladakh

recitation of sacred Buddhist texts

in the trans-Himalayan Ladakh

region Jammu and Kashmir India

- Yoga

- Nouroz

- Kumbh Mela

RAUSIAS-FC19E1003 49

Q72) Ans(b)

The President of India Shri Ram Nath Kovind inaugurated the Hornbill Festival and State Formation Day celebrations of Nagaland in Kisama

The festival is named after the Indian hornbill the large and colourful forest bird which is displayed in the folklore of most of the states tribes

The major recognized tribes of Nagaland are Angami Ao Chakhesang Chang

Kuki Rengma and Zeling etc

Onge Jarawa and Sentinelese are the

tribes of Andman amp Nicobar Islands

Q73) Ans (c)

The Rashtrakutas rule in the Deccan lasted for almost two hundred years till the end of the tenth century The Rashtrakutas rulers were tolerant in their religious views and patronized not only Shaivism and Vaishnavism but

Jainism as well

The famous rock-cut temple of Shiva at Ellora was built by one of the Rashtrakutas kings Krishna I in the ninth century His successor Amoghavarsha was a Jain but he also

patronized other faiths

The Rashtrakutas allowed Muslims traders to settle and permitted Islam to

be preached in their dominions

Recently increasing defacement at the prehistoric rock paintings of Pandavulagutta Telangana has created a cause for grave concern It can spoil

the prehistoric rock

Pandavulagutta is home to

- Painted rock shelters dating to

10000 BC-8000 BC

- An 8th century inscription of the

Rashtrakuta period and

- Painted frescoes from the 12th century Kakatiya empire

Q74) Ans (b)

In 1828 Raja Ram Mohan Roy founded a new religious society the Brahma Sabha later known as the Brahmo

Samaj

Debendranath Tagore headed the Tattvabodhini Sabha which was

engaged in search of spiritual truth

Its purpose was to purify Hinduism and to preach monotheism or belief in one God

The new society was to be based on the twin pillars of reason and the Vedas and

Upanishads

Recently Sadharan Brahmo Samaj (SBS) has entered into a legal battle with the West Bengal government due

to some legal issue

Q75) Ans (c)

The Chishti order was established in India by Khwaja Moinuddin Chishti who came to India around 1192 The Chishtirsquos are considered to be the most influential of the groups of Sufis who migrated to India in the late twelfth century They adapted successfully to the local environment and adopted several features of Indian devotional

traditions

The historical dargah of Sufi mystic Khwaja Moinuddin Chishti in Ajmer is all set to get a facelift This 13 th century dargah has been included among the Swachh Iconic Places a clean-up initiative focused on iconic

heritage spiritual and cultural places

Page 2: GENERAL STUDIES (PAPER I) · Test is part of Rau’s IAS Test series for Preliminary Exam 2019 FOUNDATION + CURRENT AFFAIRS GENERAL STUDIES (PAPER –I) FOUNDATION TEST –III TOPIC:

RAUSIAS-FC19E1003 2

Q1) निमननिखित कथि ो म स कौि-सास सही हह

1 ऋगवद म कछ छो द मनहिाओो क दवारा रच गए

2 निशवानमतर िनदक काि म एक महतवपरण ऋनि

िीच नदए गए कट का परय ग कर सही उततर चनिए

(a) किि 1

(b) किि 2

(c) 1 और 2 द ि ो

(d) ि त 1 ि ही 2

Q2) निमननिखित म स कौि-स सथि स अनि क उपय ग क

साकषय परापत हए ह

(a) भीमबटका गफाएो

(b) करिि गफाएो

(c) एनिफ टा गफाएो

(d) किहरी गफाएो

Q3) निमननिखित म स नकस सथि स भनमगत गडढ क घर ो

(Pit Houses) क साकषय पाए गए ह

(a) नचराोद

(b) हललर

(c) बरणह म

(d) महागारा

Q4) पराचीि ििि पररानिय ो का अधययि और ऐनतहानसक

पाोडनिनपय ो क समझि तथा नतनथ निराणरर क

निमननिखित म स कया कहा जाता ह

(a) परािि-निदया (Epigraphy)

(b) पाोडनिनप निजञाि (Manuscriptology)

(c) पिीओगराफी (Palaeography)

(d) नयनमजमनटकस (Numismatics)

Q5) निमननिखित यग ो पर निचार कीनजए

1 िौह उपकरर महापािानरक कबरगाह

2 चरक सोनहता गनरत की पसतक

उपयणकततत यग ो म स कौि-सास सही समनित हह

(a) किि 1

(b) किि 2

(c) 1 और 2 द ि ो

(d) ि त 1 ि ही 2

Q6) निमननिखित यग ो पर निचार कीनजए

1 भाग भनम पर कर

2 कममकार भनमहीि कनि मजदर

3 अशवमर अनय शासक ो क कषतर ो म घमि क

निए छ ड़ा गया एक घ ड़ा (अशव)

उपयणकत यग ो म स कौि-स सही समनित ह

(a) किि 1 और 2

(b) किि 2 और 3

(c) किि 1 और 3

(d) 1 2 और 3

Q7) निमननिखित कथि ो पर निचार कीनजए

1 घ ड़ ो क दवारा िी ोच जाि िाि रथ ो का उपय ग

महाजिपद काि क दौराि शर हआ था

2 नियनमत सथायी सिाए रिि की शरआत

ऋगवनदक शासक ो क दवारा की गई थी

उपयणकत कथि ो म स कौि-सास सही हह

(a) किि 1

(b) किि 2

(c) 1 और 2 द ि ो

(d) ि त 1 ि ही 2

RAUSIAS-FC19E1003 3

Q1) Which of the following statements

isare correct

1 Some verses in Rigveda were

composed by women

2 Vishwamitra was an important

sage in the Vedic period

Select the correct answer using the code

given below

(a) 1 only

(b) 2 only

(c) Both 1 and 2

(d) Neither 1 nor 2

Q2) Which of the following sites yields traces

of the use of fire

(a) Bhimbetka Caves

(b) Kurnool Caves

(c) Elephanta Caves

(d) Kanheri Caves

Q3) Which of the following sites has

evidences of underground pit houses

(a) Chirand

(b) Hallur

(c) Burzahom

(d) Mahagara

Q4) The study of ancient writing systems

and the deciphering and dating of

historical manuscripts is known as-

(a) Epigraphy

(b) Manuscriptology

(c) Palaeography

(d) Numismatics

Q5) Consider the following pairs

1 Iron implements Megalithic

burials

2 Charaka Samhita Book on

Mathematics

Which of the pairs given above isare

correct

(a) 1 only

(b) 2 only

(c) Both 1 and 2

(d) Neither 1 nor 2

Q6) Consider the following pairs

1 Bhaga Tax on land

2 Kammakaras Landless

Agricultural Labourers

3 Ashvamedha A horse is let loose

to wander into the territories of

other rulers

Which of the pairs given above isare

correct

(a) 1 and 2

(b) 2 and 3

(c) 1 and 3

(d) 1 2 and 3

Q7) Consider the following statements

1 The use of Horse drawn chariots

began during the Mahajanapada

period

2 Regular standing armies were

introduced by Rigvedic rulers

Which of the statements given above

isare correct

(a) 1 only

(b) 2 only

(c) Both 1 and 2

(d) Neither 1 nor 2

RAUSIAS-FC19E1003 4

Q8) बदध क जीिि क सोदभण म निमननिखित कथि ो पर

निचार कीनजए

1 बदध ि िारारसी क पास सारिाथ म अपिा

पहिा उपदश नदया था

2 ि निचछिी कि स सोबोनरत थ और कशीिारा म

उिका निरि हआ था

उपयणकत कथि ो म स कौि-सास सही हह

(a) किि 1

(b) किि 2

(c) 1 और 2 द ि ो

(d) ि त 1 ि ही 2

Q9) निमननिखित यग ो पर निचार कीनजए

1 समखया कनपि

2 िशनिक जानमिी

3 नयाय गौतम

उपयणकत यग ो म स कौि-स सही समनित ह

(a) किि 1 और 2

(b) किि 2 और 3

(c) किि 1 और 3

(d) 1 2 और 3

Q10) निमननिखित कथि ो पर निचार कीनजए

1 महािीर की नशकषाऐो िशािी म सोकनित की गई

थी ो

2 नििय नपटक बौदध सोघ क बार म ह

उपयणकत कथि ो म स कौि-सास सही हह

(a) किि 1

(b) किि 2

(c) 1 और 2 द ि ो

(d) ि त 1 ि ही 2

Q11) निमननिखित कथि ो पर निचार कीनजए

1 अथणशासतर का सोकिि निषणगपत ि नकया था

2 मगसथिीस ज चनदरगपत मौयण क दरबार म

आया था सलयकस का राजदत था

उपयणकत कथि ो म स कौि-सास सही हह

(a) किि 1

(b) किि 2

(c) 1 और 2 द ि ो

(d) ि त 1 ि ही 2

Q12) निमननिखित यग ो पर निचार कीनजए

सतभ कपिटल सथान

1 नसोह कनपटि सारिाथ

2 बि कनपटि रामपिण

3 हाथी कनपटि सोनकससा

उपयणकत यग ो म स कौि-सास सही समनित हह

(a) किि 1 और 2

(b) किि 3

(c) किि 1 और 3

(d) 1 2 और 3

Q13) 1857 क नसपाही निदर ह क सोदभण म निमननिखित

कथि ो म स कौि-सास सही हह

1 अहमदललाह शाह फजाबाद क मौििी ि

अोगरज ो क निरदध ि ग ो क एकनतरत नकया

2 किर नसोह ज एक महाि य दधा थ उततर परदश

स सोबोनरत थ

िीच नदए गए कट का परय ग कर सही उततर चनिए

(a) किि 1

(b) किि 2

(c) 1 और 2 द ि ो

(d) ि त 1 ि ही 2

RAUSIAS-FC19E1003 5

Q8) In the context of the life of Buddha

consider the following statements

1 Buddha gave his first sermon at

Sarnath near Varanasi

2 He belonged to the Licchavi clan

and passed away at Kusinara

Which of the above statements isare

correct

(a) 1 only

(b) 2 only

(c) Both 1 and 2

(d) Neither 1 nor 2

Q9) Consider the following pairs

1 Samkhya Kapila

2 Vaisheshika Jamini

3 Nyaya Gautama

Which of the pairs given above isare

correctly matched

(a) 1 and 2 only

(b) 2 and 3 only

(c) 1 and 3 only

(d) 1 2 and 3

Q10) Consider the following statements

1 The teachings of Mahavira were

compiled at Vaishali

2 Vinaya Pitaka talks about the

Buddhist Sangha

Which of the statements given above

isare correct

(a) 1 only

(b) 2 only

(c) Both 1 and 2

(d) Neither 1and 2

Q11) Consider the following statements

1 Arthashastra was compiled by

Vishnugupta

2 Magasthenes who came to the

court of Chandragupta Maurya

was the ambassador of Seleucus

Which of the statements given above

isare correct

(a) 1 only

(b) 2 only

(c) Both 1 and 2

(d) Neither 1 nor 2

Q12) Consider the following pairs

Pillar Capital Place

1 Lion Capital Sarnath

2 Bull Capital Rampurva

3 Elephant Capital Sankissa

Which of the pairs given above isare

matched correctly

(a) 1 and 2

(b) 3 only

(c) 1 and 3

(d) 1 2 and 3

Q13) Consider the following statements

regarding the Revolt of 1857

1 Ahmadullah Shah a maulvi from

Faizabad caught the imagination

of the people and raised a huge

force of supporters against the

Britishers

2 Kunwar Singh was a great fighter

from Uttar Pradesh

Which of the statements given above

isare correct

(a) 1 only

(b) 2 only

(c) Both 1 and 2

(d) Neither 1 nor 2

RAUSIAS-FC19E1003 6

Q14) निमननिखित यग ो पर निचार कीनजए

1 िललािर सनिक

2 आनदमाई दास (गिाम)

3 कदनसयर भनमहीि मजदर

उपयणकत यग ो म स कौि-स सही समनित ह

(a) किि 1 और 2

(b) किि 2 और 3

(c) किि 1 और 3

(d) 1 2 और 3

Q15) निमननिखित सथि ो म स उस सथि की पहचाि कीनजए

ज एक परनसदध तटीय बसती थी और जहा एरटाइि

(Arretine) मदभाोड पाया गया था

(a) महाबिीपरम

(b) तामरनिखपत

(c) अररकमड

(d) अतरोजीिरा

Q16) सोगम सानहतय क सोदभण म निमननिखित कथि ो पर

निचार कीनजए

1 मदरई म सोगमसभाए आय नजत की जाती ो

थी ो

2 मिनदर शबद का परय ग सोगम कनिताओो म

नकया जाता था नजसका अथण ह द राजाओो

का एक समह

उपयणकत कथि ो म स कौि-सास सही हह

(a) किि 1

(b) किि 2

(c) 1 और 2 द ि ो

(d) ि त 1 ि ही 2

Q17) निमननिखित कथि ो पर निचार कीनजए

1 आरखिक िनदक काि म मनहिाऐो सभाओो

और निदाथाओो म भाग िती थी ो

2 आरखिक िनदक काि म िनशषठ ज एक

पजारी ह ता था की एक महतवपरण भनमका

ह ती थी

उपयणकत कथि ो म स कौि-सास सही हह

(a) किि 1

(b) किि 2

(c) 1 और 2 द ि ो

(d) ि त 1 ि ही 2

Q18) निमननिखित कथि ो पर निचार कीनजए

1 किाणटक म जि रमण क निसतार का शरय

चनदरगपत मौयण क जाता ह

2 जि रमण ि ईशवर क अखसततव क मानयता दी ह

तथा िरण पररािी की भरतणिा की ह

उपयणकत कथि ो म स कौि-सास सही हह

(a) किि 1

(b) किि 2

(c) 1 और 2 द ि ो

(d) ि त 1 ि ही 2

Q19) निमननिखित यग ो पर निचार कीनजए

1 पहार पाणडय

2 मदरई च ि

उपयणकत यग ो म स कौि-सास सही समनित हह

(a) किि 1

(b) किि 2

(c) 1 और 2 द ि ो

(d) ि त 1 ि ही 2

RAUSIAS-FC19E1003 7

Q14) Consider the following pairs

1 Vellalar Soldier

2 Adimai Slaves

3 Kadaisiyar Landless labourers

Which of the pairs given above isare

correctly matched

(a) 1 and 2 only

(b) 2 and 3 only

(c) 1 and 3 only

(d) 1 2 and 3

Q15) Identify the site which was a famous

coastal settlement from which Arretine

ware has been found

(a) Mahabalipuram

(b) Tamralipti

(c) Arikamedu

(d) Atranjikhera

Q16) With respect to the Sangam literature

consider the following statements

1 The Sangamsassemblies were

held at Madurai

2 The term muvendar was used in

sangam poems which mean a

group of two kings

Which of the statements given above

isare correct

(a) 1 only

(b) 2 only

(c) Both 1 and 2

(d) Neither 1 nor 2

Q17) Consider the following statements

1 During Early Vedic Period women

attended the Sabha and Vidatha

2 The priest ndash Vasishtha played

important role in Early Vedic

Period

Which of the statements given above

isare correct

(a) 1 only

(b) 2 only

(c) Both 1 and 2

(d) Neither 1 nor 2

Q18) Consider the following statements

1 The spread of Jainism in

Karnataka is attributed to

Chandragupta Maurya

2 Jainism recognised the existence

of the gods and condemns the

Varna System

Which of the statements given above

isare correct

(a) 1 only

(b) 2 only

(c) Both 1 and 2

(d) Neither 1 nor 2

Q19) Consider the following pairs

1 Puhar Pandyas

2 Madurai Cholas

Which of the pairs given above isare

correct

(a) 1 only

(b) 2 only

(c) Both 1 and 2

(d) Neither 1 nor 2

RAUSIAS-FC19E1003 8

Q20) निमननिखित कथि ो म स कौि-सास सही हह

1 बदधचररत का ििक िागसि ह

2 ब नरसतव की पजा महायाि बौदधमत का एक

महतवपरण भाग थी

िीच नदए गए कट का परय ग कर सही उततर चनिए

(a) किि 1

(b) किि 2

(c) 1 और 2 द ि ो

(d) ि त 1 ि ही 2

Q21) निमननिखित कथि ो म स कौि-सास सही हह

1 भखकत क निचार क भागित गीता म सपषट

नकया गया ह

2 तनमि कनि अपपर एक अििर सोत थ

िीच नदए गए कट का परय ग कर सही उततर चनिएः

(a) किि 1

(b) किि 2

(c) 1 और 2 द ि ो

(d) ि त 1 ि ही 2

Q22) निमननिखित म स नकस शासक ि िीरा बजात हए

अपिी छनि नसक ो पर अोनकत करिाई थी

(a) नमिाोदर

(b) चनदरगपत मौयण

(c) गौतमीपतर सतकरी

(d) समदरगपत

Q23) निमननिखित कथि ो म स कौि-सास सही हह

1 सवतोतर भारत ि निकरम सोित क राषटर ीय

किनडर क रप म अपिाया और यह 68 ईसा

पिण म आरि हआ था

2 बािभटट हिणिरणि क दरबार का एक कनि था

िीच नदए गए कट का परय ग कर सही उततर चनिएः

(a) किि 1

(b) किि 2

(c) 1 और 2 द ि ो

(d) ि त 1 ि ही 2

Q24) निमननिखित यग ो पर निचार कीनजएः

1 सोनर-निगरानहका वयापार मोतरी

2 परथम-कनिका परमि नशलपकार

3 साथणिाह परमि बकर

उपयणकत यग ो म स कौि-सास सही समनित हह

(a) किि 1

(b) किि 1 और 3

(c) किि 2

(d) किि 2 और 3

Q25) निमननिखित कथि ो म स कौि-सास सही हह

1 lsquoएह ि नशिाििrsquo पिकनशि नदवतीय स

सोबोनरत ह और रनिकनत क दवारा इसकी रचिा

की गई थी

2 जआि झाोग समदरगपत नदवतीय क शासिकाि

म भारत आया था

िीच नदए गए कट का परय ग कर सही उततर चनिएः

(a) किि 1

(b) किि 2

(c) 1 और 2 द ि ो

(d) ि त 1 ि ही 2

RAUSIAS-FC19E1003 9

Q20) Which of the following statements

isare correct

1 Buddhacharita is authored by

Nagasena

2 The worship of Bodhisattvas was

an important part of Mahayana

Buddhsim

Select the correct answer using the code

given below

(a) 1 only

(b) 2 only

(c) Both 1 and 2

(d) Neither 1 nor 2

Q21) Which of the following statements

isare correct

1 The idea of Bhakti is elucidated in

Bhagavata Gita

2 Tamil poet Appar was an Alvar

saint

Select the correct answer using the code

given below

(a) 1 only

(b) 2 only

(c) Both 1 and 2

(d) Neither 1 nor 2

Q22) Which of the following rulers had his

image inscribed in the coins while

playing a veena

(a) Meander

(b) Chandragupta Maurya

(c) Gautamiputra Satkarni

(d) Samudragupta

Q23) Which of the following statements

isare correct

1 Vikrama Samvat is adopted as the

national calendar by independent

India and it began in 68 BC

2 Banabhatta was a court poet of

Harshavardhana

Select the correct answer using the code

given below

(a) 1 only

(b) 2 only

(c) Both 1 and 2

(d) Neither 1 nor 2

Q24) Consider the following pairs

1 Sandhi-vigrahika Minister of trade

2 Prathama-kulika Chief craftsman

3 Sarthavaha Chief banker

Which of the pairs given above isare

correct

(a) 1 only

(b) 1 and 3 only

(c) 2 only

(d) 2 and 3 only

Q25) Which of the following statements

isare correct

1 Aihole inscription belongs to

Pulakeshin II and was composed

by Ravikriti

2 Xuan Zang came to India during

the reign of Chandragupta II

Select the correct answer using the code

given below

(a) 1 only

(b) 2 only

(c) Both 1 and 2

(d) Neither 1 nor 2

RAUSIAS-FC19E1003 10

Q26) िासतकिा स सोबोनरत निमननिखित ततव ो म स कौि-स

ततव किि नहोद मखनदर ो की िासतकिा क भाग ह

1 नशिर

2 मणडप

3 परदनकषरा पथ

4 गभणगह

िीच नदए गए कट का परय ग कर सही उततर चनिएः

(a) किि 1 3 और 4

(b) किि 2 3 और 4

(c) किि 1 2 और 4

(d) 1 2 3 और 4

Q27) निमननिखित मोनदर ो म स कौि-स मोनदर ईोट ो स बि ह

1 दिगढ़ मोनदर

2 भीतरगाि मोनदर

3 िकषमर मोनदर नसरपर

4 बहदशवर मोनदर

िीच नदए गए कट का परय ग कर सही उततर चनिएः

(a) किि 1 2 और 3

(b) किि 2 3 और 4

(c) किि 1 3 और 4

(d) 1 2 3 और 4

Q28) निमननिखित कथि ो म स कौि-सास सही हह

1 सलताि महममद किी कतब शाह अकबर का

समकािीि था

2 िासतकिा क कषतर म महममद किी कतब शाह

ि कई ईमारत ो का निमाणर करिाया था नजिम

स चार मीिार सिाणनरक परनसदध ह

िीच नदए गए कट का परय ग कर सही उततर चनिएः

(a) किि 1

(b) किि 2

(c) 1 और 2 द ि ो

(d) ि त 1 ि ही 2

Q29) निमननिखित यग ो पर निचार कीनजएः

1 मनिमकिाई सततिार

2 अनभजञाि शाको तिम कानिदास

3 नसिपपानदकारम क ििि

उपयणकत यग ो म स कौि-सास सही समनित हह

(a) किि 1 और 2

(b) किि 2

(c) किि 1 और 3

(d) 1 2 और 3

Q30) निमननिखित कथि ो म स कौि-सास सही हह

1 ldquoसशरत सोनहताrdquo नचनकरता पर एक महतवपरण

रचिा ह

2 बरहमगपत और चरक महतवपरण गनरतजञ थ

िीच नदए गए कट का परय ग कर सही उततर चनिए

(a) किि 1

(b) किि 2

(c) 1 और 2 द ि ो

(d) ि त 1 ि ही 2

Q31) अमीर िसर क सनदभण म निमननिखित कथि ो म स

कौि-सास सही हह

1 अमीर िसर ि अपिी रचिाओो म नििा ह

नक सोसकत नकसी भी कषतर स सोबोनरत िही ो थी

और किि बराहमर ही इस भािा का जञाि रित

2 उन ोि नहोदिी और अिरी क अखसततव का

उललि नकया था

िीच नदए गए कट का परय ग कर सही उततर चनिए

(a) किि 1

(b) किि 2

(c) 1 और 2 द ि ो

(d) ि त 1 ि ही 2

RAUSIAS-FC19E1003 11

Q26) Which of the following architectural

elements were only part of Hindu

temple architecture

1 Shikhara

2 Mandapa

3 Pradakshina patha

4 Garbhagriha

Select the correct answer using the code

given below

(a) 1 3 and 4 only

(b) 2 3 and 4 only

(c) 1 2 and 4 only

(d) 1 2 3 and 4

Q27) Which of the following temples isare

made of bricks

1 Deogarh Temple

2 Bhitargaon Temple

3 Lakshmana temple Sirpur

4 Brihadeshvara Temple

Select the correct answer using the code

given below

(a) 1 2 and 3 only

(b) 2 3 and 4 only

(c) 1 3 and 4 only

(d) 1 2 3 and 4

Q28) Which of the following statements

isare correct

1 Sultan Muhammad Quli Qutab

Shah was a contemporary of

Akbar

2 In the field of architecture

Muhammad Quli Qutab Shah

constructed many buildings the

most famous of which is the Char

Minar

Select the correct answer using the code

given below

(a) 1 only

(b) 2 only

(c) Both 1 and 2

(d) Neither 1 nor 2

Q29) Consider the following pairs

1 Manimekalai Sattanar

2 Abhijnana Shakuntalam Kalidasa

3 Silappadikaram Kovalan

Which of the pairs given above isare

correct

(a) 1 and 2 only

(b) 2 only

(c) 1 and 3 only

(d) 1 2 and 3

Q30) Which of the following statements

isare correct

1 Sushruta Samhita is an important

work on medicine

2 Brahmagupta and Charaka were

important mathematicians

Select the correct answer using the code

given below

(a) 1 only

(b) 2 only

(c) Both 1 and 2

(d) Neither 1 nor 2

Q31) Which of the following statements

isare correct about Amir Khusrau

1 Amir Khusrau records in his works

that Sanskrit did not belong to any

region and only the Brahmans

knew it

2 He recorded the existence of

Hindawi and Awadhi

Select the correct answer using the code

given below

(a) 1 only

(b) 2 only

(c) Both 1 and 2

(d) Neither 1 nor 2

RAUSIAS-FC19E1003 12

Q32) निमननिखित कथि ो पर निचार कीनजए

1 नहरणय-गभण अिषठाि क बार म ऐसा स चा जाता

था नक बनि दि िाि का एक कषनतरय क रप म

पिजणनम ह गा

2 मयरशमणि कदोब िोश का सोसथापक था

उपयणकत कथि ो म स कौि-सास सही हह

(a) किि 1

(b) किि 2

(c) 1 और 2 द ि ो

(d) ि त 1 ि ही 2

Q33) निमननिखित कथि ो म स कौि-सास सही हह

1 कदमई बगार (बिपिणक शरम) क रप म

निया जाि िािा कर था

2 गवानियर परशखसत म िागभट (ज एक चोदि

राजा था) क दवारा नकय गए श िर का िरणि

नकया गया ह

िीच नदए गए कट का परय ग कर सही उततर चनिए

(a) किि 1

(b) किि 2

(c) 1 और 2 द ि ो

(d) ि त 1 ि ही 2

Q34) निमननिखित कथि ो म स कौि-सास सही हह

1 राजतरो नगिी 11िी ो शताबदी म कलहि क दवारा

रनचत एक सोसकत पसतक (टकसट) ह

2 कननौज क निए नतरपकषीय सोघिण म पाि राजिोश

शानमि था

िीच नदए गए कट का परय ग कर सही उततर चनिए

(a) किि 1

(b) किि 2

(c) 1 और 2 द ि ो

(d) ि त 1 ि ही 2

Q35) निमननिखित यग ो पर निचार कीनजए

1 बरहदशवर मोनदर राजराजा च ि

2 उर मापि की इकाई

3 दिदाि मोनदर ो क भनम अिदाि

उपयणकत यग ो म स कौि-स सही समनित ह

(a) किि 1 और 2

(b) किि 2 और 3

(c) किि 1 और 3

(d) 1 2 और 3

Q36) निमननिखित कथि ो म स कौि-सास सही हह

1 नदलली क सलताि ो क अरीि परशासि की भािा

फारसी थी

2 नदलली सलतित म ldquoतारीितािरीिrdquo कनिता

का एक रप था

िीच नदए गए कट का परय ग कर सही उततर चनिए

(a) किि 1

(b) किि 2

(c) 1 और 2 द ि ो

(d) ि त 1 ि ही 2

Q37) निमननिखित कथि ो म स कौि-सास सही हह

1 अिाउददीि खििजी ि अपि सनिक ो क निए

नसरी िाम का एक िया दगणरकषक शहर

बिािाया था

2 िह अपि सनिक ो क िति का भगताि इकता

क रप म करता था

िीच नदए गए कट का परय ग कर सही उततर चनिए

(a) किि 1

(b) किि 2

(c) 1 और 2 द ि ो

(d) ि त 1 ि ही 2

RAUSIAS-FC19E1003 13

Q32) Consider the following statements

1 Hiranya-garbha ritual was thought

to lead to the rebirth of the

sacrificer as a Kshatriya

2 Mayurasharman was the founder

of the Kadamba dynasty

Which of the statements given above

isare correct

(a) 1 only

(b) 2 only

(c) Both 1 and 2

(d) Neither 1 nor 2

Q33) Which of the following statements

isare correct

1 Kadamai was tax taken in form of

forced labour

2 Gwalior Prashasti describes the

exploits of Nagabhata who was a

Chandella king

Select the correct answer using the code

given below

(a) 1 only

(b) 2 only

(c) Both 1 and 2

(d) Neither 1 nor 2

Q34) Which of the following statements

isare correct

1 Rajatarangini is a Sanskrit text

written by Kalhana in the 11th

century

2 Pala dynasty was included in the

tripartite struggle for Kannauj

Select the correct answer using the code

given below

(a) 1 only

(b) 2 only

(c) Both 1 and 2

(d) Neither 1 nor 2

Q35) Consider the following pairs

1 Brihadeshvara temple Rajaraja

Chola

2 ldquoUrrdquo Unit of measurement

3 Devadana Land grants made to

temples

Which of the pairs given above isare

correct

(a) 1 and 2 only

(b) 2 and 3 only

(c) 1 and 3 only

(d) 1 2 and 3

Q36) Which of the following statements

isare correct

1 The language of administration

under the Delhi Sultans was

Persian

2 Tarikhtawarikh was a form of

poetry in the Delhi Sultanate

Select the correct answer using the code

given below

(a) 1 only

(b) 2 only

(c) Both 1 and 2

(d) Neither 1 nor 2

Q37) Which of the following statements

isare correct

1 Alauddin Khilji constructed a new

garrison town named Siri for his

soldiers

2 He paid his soldiers their salaries

in the form of Iqta

Select the correct answer using the code

given below

(a) 1 only

(b) 2 only

(c) Both 1 and 2

(d) Neither 1 nor 2

RAUSIAS-FC19E1003 14

Q38) निमननिखित कथि ो म स कौि-सास सही हह

1 नदलली कतबददीि एबक क अरीि पहिी बार

नकसी सामराजय की राजरािी बिी थी

2 दहिीिाि नसक ो का मदरर मग़ि ो क दवारा

नकया गया था

िीच नदए गए कट का परय ग कर सही उततर चनिए

(a) किि 1

(b) किि 2

(c) 1 और 2 द ि ो

(d) ि त 1 ि ही 2

Q39) निमननिखित यग ो पर निचार कीनजए

1 म ठ की मखिद नसको दर ि दी

2 बगमपरी मखिद नफर ज शाह तगिक

3 कववत- अि - इसलाम कतबददीि ऐबक

उपयणकत यग ो म स कौि-स सही समनित ह

(a) किि 1 और 2

(b) किि 2 और 3

(c) किि 1 और 3

(d) 1 2 और 3

Q40) निमननिखित कथि ो म स कौि-सास सही हह

1 मिसबदार ो क अपिा िति राजसव कायो

नजन जागीर कहत थ क रप म परापत ह ता

था

2 मिसबदार क ज सनय उततरदानयतव सौोप जात

थ उसक अनतगणत उस एक निराणररत सखया म

सिार अथिा घड़सिार ो का रि-रिाि करिा

पड़ता था

िीच नदए गए कट का परय ग कर सही उततर चनिए

(a) किि 1

(b) किि 2

(c) 1 और 2 द ि ो

(d) ि त 1 ि ही 2

Q41) ldquo1942 क भारत छ ड़ आोद ििrdquo क बार म

निमननिखित अिि कि ो म स कौि-सा सतय िही ो ह

(a) यह एक अनहोसक आोद िि था

(b) इसका िततव महातमा गाोरी क दवारा नकया गया

था

(c) यह एक सवाभानिक आोद िि था

(d) इसि सामानयतया शरनमक िगण क आकनिणत

िही ो नकया था

Q42) भारत क ि ग ो ि ldquoसाइमि कमीशिrdquo क आगमि क

निरदध आोद िि नकया था कय ोनक

(a) भारतीय कभी भी 1919 क अनरनियम (The

Act of 1919) क काम की समीकषा िही ो करिा

चाहत थ

(b) साइमि कमीशि ि पराोत ो म दवर (द हर) शासि

क समापत करि की नसफाररश की थी

(c) साइमि कमीशि म क ई भारतीय सदसय िही ो

था

(d) साइमि कमीशि ि दश क निभाजि का

सझाि नदया था

Q43) निमननिखित कथि ो पर निचार कीनजए

भारतीय राषटर ीय आोद िि म दादाभाई िौर जी क दवारा

नकया गया सबस परभािी य गदाि यह था नक उन ोि

1 अोगरज ो क दवारा भारत क आनथणक श िर का

ििासा नकया था

2 पराचीि भारतीय गरोथ ो की वयाखया की थी और

भारतीय ो क आतमनिशवास क पिःसथानपत नकया

था

3 अनय नकसी भी बात स पहि सभी सामानजक

बराइय ो क उनमिि की आिशयकता पर बि

नदया था

उपयणकत कथि ो म स कौि-सास सही हह

(a) किि 1

(b) किि 2 और 3

(c) किि 1 और 3

(d) 1 2 और 3

RAUSIAS-FC19E1003 15

Q38) Which of the following statements

isare correct

1 Delhi first became the capital of a

kingdom under Qutubuddin

Aibak

2 Dehliwal coins were minted by the

Mughals

Select the correct answer using the code

given below

(a) 1 only

(b) 2 only

(c) Both 1 and 2

(d) Neither 1 nor 2

Q39) Consider the following pairs

1 Moth ki Masjid- Sikander Lodi

2 Begumpuri mosque- Firuz Shah

Tughluq

3 Quwwat al ndash Islam- Qutubuddin

Aibak

Which of the above pairs isare correct

(a) 1 and 2 only

(b) 2 and 3 only

(c) 1 and 3 only

(d) 1 2 and 3

Q40) Which of the following statements

isare correct

1 Mansabdars received their salaries

as revenue assignments called

jagirs

2 The mansabdarrsquos military

responsibilities required him to

maintain a specified number of

sawar or cavalrymen

Select the correct answer using the code

given below

(a) 1 only

(b) 2 only

(c) Both 1 and 2

(d) Neither 1 nor 2

Q41) Which one of the following observations

is not true about the Quit India

Movement of 1942

(a) It was a non-violent movement

(b) It was led by Mahatma Gandhi

(c) It was a spontaneous movement

(d) It did not attract the labour class

in general

Q42) The people of India agitated against the

arrival of the Simon Commission

because

(a) Indians never wanted the review of

the working of the Act of 1919

(b) Simon Commission recommended

the abolition of dyarchy in the

Provinces

(c) there was no Indian member in the

Simon Commission

(d) the Simon Commission suggested

the partition of the country

Q43) Consider the following statements

The most effective contribution made by

Dadabhai Naoroji to the cause of Indian

National Movement was that he-

1 exposed the economic exploitation

of India by the British

2 interpreted the ancient Indian

texts and restored the self-

confidence of Indians

3 stressed the need for eradication of

all the social evils before anything

else

Which of the statements given above

isare correct

(a) 1 only

(b) 2 and 3 only

(c) 1 and 3 only

(d) 1 2 and 3

RAUSIAS-FC19E1003 16

Q44) महातमा गाोरी ि 1932 म आमरर अिशि नकया था

कय ोनक

(a) ldquoग िमज सममििrdquo (The Round Table

Conference) भारतीय राजिीनतक

आकाोकषाओो क परा करि म असफि रहा था

(b) काोगरस और मखसलम िीग म मतभद थ

(c) रामस मकड िालड (Ramsay Macdonald)

ि ldquoसाोपरदानयक परसकारrdquo (The Communal

Award) की घ िरा की थी

(d) ldquoसनििय अिजञा आोद ििrdquo (The Civil

Disobedience Movement) असफि रहा

था

Q45) भारत म औपनििनशक शासि की अिनर क सोदभण म

भारत स रि क बनहगणमि का एक महतवपरण भाग गह

शलक (Home Charges) था निमननिखित म स

कौि-सास क ि गह शलक म सखममनित नकया गया

थानकय गए थ

1 िोदि म भारत कायाणिय क निए उपय ग नकय

जाि िािा क ि

2 भारत म नियकत नबरनटश कनमणय ो क िति और

पशि का भगताि करि क निए उपय ग नकय

जाि िािा क ि

3 अोगरज ो क दवारा भारत क बाहर यदध ो क निए

उपय ग नकय जाि िािा क ि

िीच नदए गए कट का परय ग कर सही उततर चनिए

(a) किि 1

(b) किि 1 और 2

(c) किि 2 और 3

(d) 1 2 और 3

Q46) सवतोतरता आोद िि क इनतहास म भारतीय राषटर ीय

काोगरस का 1929 का सतर महतवपरण ह कय ोनक इसम

(a) काोगरस क उददशय क रप म सथािीय सरकार

की पराखपत की घ िरा की गई थी

(b) परण सवराज की पराखपत क काोगरस क िकषय क

रप म अपिाया गया था

(c) असहय ग आोद िि शर नकया गया था

(d) िोदि म ldquoग ि मर सममििrdquo (The Round

Table Conference) म भाग िि का निरणय

निया गया था

Q47) भारतीय सवतोतरता सोगराम क सोदभण म िहर ररप टण

क दवारा निमननिखित म स नकसकी नसफाररश की गई

थीनकिकी नसफाररश की गई थी ो

1 भारत क निए परण सवतोतरता

2 अलपसोखयक ो क निए सीट ो क आरकषर क

निए सोयकत नििाणचक मोडि

3 सोनिराि म भारत क ि ग ो क निए मौनिक

अनरकार ो का परािराि

िीच नदए गए कट का परय ग कर सही उततर चनिए

(a) किि 1

(b) किि 2 और 3

(c) किि 1 और 3

(d) 1 2 और 3

Q48) आरो नभक िनदक आयो का रमण मखय रप स था

(a) भखकत

(b) मनतण पजा और यजञ

(c) परकनत की पजा और यजञ

(d) परकनत की पजा और भखकत

RAUSIAS-FC19E1003 17

Q44) Mahatma Gandhi undertook fast unto

death in 1932 mainly because

(a) The Round Table Conference failed

to satisfy Indian political

aspirations

(b) The Congress and Muslim League

had differences of opinion

(c) Ramsay Macdonald announced the

Communal Award

(d) The Civil Disobedience Movement

failed

Q45) With reference to the period of colonial

rule in India ldquoHome Chargesrdquo formed

an important part of drain of wealth

from India Which of the following funds

constituted ldquoHome Chargesrdquo

1 Funds used to support the India

Office in London

2 Funds used to pay salaries and

pensions of British personnel

engaged in India

3 Funds used for waging wars

outside India by the British

Select the correct answer using the code

given below

(a) 1 only

(b) 1 and 2 only

(c) 2 and 3 only

(d) 1 2 and 3

Q46) The 1929- Session of Indian National

Congress is of significance in the history

of the Freedom Movement because the-

(a) attainment of Self-Government

was declared as the objective of

the Congress

(b) attainment of Poorna Swaraj was

adopted as the goal of the

Congress

(c) Non-Cooperation Movement was

launched

(d) decision to participate in the

Round Table Conference in

London was taken

Q47) With reference to the period of Indian

freedom struggle which of the following

waswere recommended by the Nehru

report

1 Complete Independence for India

2 Joint electorates for reservation of

seats for minorities

3 Provision of fundamental rights for

the people of India in the

Constitution

Select the correct answer using the code

given below

(a) 1 only

(b) 2 and 3 only

(c) 1 and 3 only

(d) 1 2 and 3

Q48) The religion of the early Vedic Aryans was primarily of

(a) Bhakti

(b) image worship and Yajnas

(c) worship of nature and Yajnas

(d) worship of nature and Bhakti

RAUSIAS-FC19E1003 18

Q49) भारत की यातरा करि िाि चीिी यातरी यआि चिाोग

(हयएि साोग) ि समकािीि भारत की सामानय

खसथनतय ो और सोसकनत क दजण नकया था इस सोदभण म

निमननिखित कथि ो म स कौि-सास सही हह

1 सड़क और िदी-मागण (जि-मागण) डकती स

परण रप स सरनकषत थ

2 जहा तक अपरार ो क निए दणड की बात ह

उसक निए नकसी भी वयखकत की निदोिता

अथिा उसक अपरार क निराणररत करि क

निए अनि जि और निि परि क माधयम क

सारि थ

3 वयापाररय ो क घाट ो और परनतबोर सटशि ो पर

शलक ो का भगताि करिा पड़ता था

िीच नदए गए कट का परय ग कर सही उततर चनिए

(a) किि 1

(b) किि 2 और 3

(c) किि 1 और 3

(d) 1 2 और 3

Q50) नसोर घाटी सभयता क सोदभण म निमननिखित कथि ो पर

निचार कीनजए

1 यह मखय रप स एक रमणनिरपकष सभयता थी

तथा हािाोनक इसम रानमणक ततव मौजद था

िनकि िह परनतिश पर हािी िही ो था

2 इस काि क दौराि भारत म कपास का परय ग

कपड़ा बिाि क निए नकया जाता था

उपयणकत कथि ो म स कौि-सास सही हह

(a) किि 1

(b) किि 2

(c) 1 और 2 द ि ो

(d) ि त 1 ि ही 2

Q51) परोदर दास क सोदभण म निमननिखित कथि ो पर निचार

कीनजए

1 परोदर दास एक सोत और भगिाि नशि क एक

महाि भकत थ

2 ि एक सोगीतकार गायक और किाणटक सोगीत

क मखय सोसथापक-परसतािक ो म स एक थ

उपयणकत कथि ो म स कौि-सास सही हह

(a) किि 1

(b) किि 2

(c) 1 और 2 द ि ो

(d) ि त 1 ि ही 2

Q52) निमननिखित म स कौि-सास वयखकत किाणटक सोगीत

की नतरमनतण म शानमि हह

1 बािामरिी कषणा

2 शरी शयाम शासतरी

3 शरी मथसवामी दीनकषतर

िीच नदए गए कट का परय ग कर सही उततर चनिए

(a) किि 1

(b) किि 2

(c) किि 2 और 3

(d) 1 2 और 3

Q53) चियर (Chevayur) और अथ िी (Atholi) म खसथत

महापािार सथि निमननिखित म स नकस राजय म खसथत

(a) तनमििाड

(b) किाणटक

(c) पनिम बोगाि

(d) करि

RAUSIAS-FC19E1003 19

Q49) The Chinese traveller Yuan Chwang

(Hiuen Tsang) who visited India

recorded the general conditions and

culture of India at that time In this

context which of the following

statements isare correct

1 The roads and river-routes were

completely immune from robbery

2 As regards punishment for

offences ordeals by fire water and

poison were the instruments for

determining the innocence or guilt

of a person

3 The tradesmen had to pay duties

at ferries and barrier stations

Select the correct answer using the code

given below

(a) 1 only

(b) 2 and 3 only

(c) 1 and 3 only

(d) 1 2 and 3

Q50) Regarding the Indus Valley Civilization

consider the following statements

1 It was predominantly a secular

civilization and the religious

element though present did not

dominate the scene

2 During this period cotton was

used for manufacturing textiles in

India

Which of the statements given above

isare correct

(a) 1 only

(b) 2 only

(c) Both 1 and 2

(d) Neither 1 nor 2

Q51) Consider the following statements

regarding Purandara Dasa

1 Purandara Dasa was a saint and

great devotee of Lord Shiva

2 He was a composer singer and

one of the chief founding-

proponents of the Carnatic music

Which of the statements given above

isare correct

(a) 1 only

(b) 2 only

(c) Both 1 and 2

(d) Neither 1 nor 2

Q52) Which of the following persons isare

included in the trinity of Carnatic

music

1 Balamurali Krishna

2 Sri Shyama Shastry

3 Sri Muthuswami Dikshitar

Select the correct answer using the code

given below

(a) 1 only

(b) 2 only

(c) 2 and 3 only

(d) 1 2 and 3

Q53) Megalithic sites at Chevayur and Atholi

are located in which of the following

states

(a) Tamil Nadu

(b) Karnataka

(c) West Bengal

(d) Kerala

RAUSIAS-FC19E1003 20

Q54) निमननिखित कथि ो पर निचार कीनजए

1 महापािानरक ि ग कबर ो म िसतएो दफिात थ

2 दनकषर भारत म महापािार सोसकनत एक परण

निकनसत तामर यगीि सोसकनत थी

उपयणकत कथि ो म स कौि-सास सही हह

(a) किि 1

(b) किि 2

(c) 1 और 2 द ि ो

(d) ि त 1 ि ही 2

Q55) निमननिखित म स कौि-स सामराजयसामराजय ो का

अश क क अनभिि ो म उललि नकया गया ह

1 च ि

2 पाणडय

3 करिपतर (चर)

िीच नदए गए कट का परय ग कर सही उततर चनिए

(a) किि 1

(b) किि 1 और 2

(c) किि 3

(d) 1 2 और 3

Q56) भीमा-क रगाोि का यदध को पिी क सनिक ो और

बाजीराि नदवतीय क िततव म एक शखकतशािी पशिा

सिा (मराठ ो) क मधय िड़ा गया था यह यदध

निमननिखित म स नकसका नहससा था

(a) परथम आोगल-मराठा यदध का

(b) नदवतीय आोगल-मराठा यदध का

(c) ततीय आोगल-मसर यदध का

(d) ततीय आोगल-मराठा यदध का

Q57) निमननिखित कथि ो पर निचार कीनजए

1 महादि दसाई ि गाोरीजी क चोपारर आि तथा

नतिकनथया पररािी स जड़ी समसया की जाोच

क निए रारी करि क निए दश भर म उिका

अिसरर नकया था

2 िरहरी पाररि चोपारर सतयागरह क दौराि

गाोरीजी क साथ थ

उपयणकत कथि ो म स कौि-सास सही हह

(a) किि 1

(b) किि 2

(c) 1 और 2 द ि ो

(d) ि त 1 ि ही 2

Q58) निमननिखित कथि ो पर निचार कीनजए

1 िनद राज-िोश ि बराहमर ो और बौदध मठराररय ो

क कर-मकत गाि अिदाि म दि की परथा

आरि की थी

2 सतिाहि ो की आनरकाररक भािा पराकत थी

उपयणकत कथि ो म स कौि-सास सही हह

(a) किि 1

(b) किि 2

(c) 1 और 2 द ि ो

(d) ि त 1 ि ही 2

Q59) एक निरासत क अपिाइए (अडॉपट ए हररटज ndash

Adopt a Heritage) पररय जिा क उददशय ो क

सनदभण म निमननिखित कथि ो पर निचार कीनजए

1 यह पररय जिा र रगार उतपादि और आनथणक

निकास क निए पयणटि कषमता का उि पर

परभाि का उपय ग करगी

2 यह पररय जिा निरासत सथि ो पर निशव सतरीय

आराररक सोरचिा निकनसत करक एक सतत

तरीक स पयणटक आकिणर म िखदध करगी

उपयणकत कथि ो म स कौि-सास सही हह

(a) किि 1

(b) किि 2

(c) 1 और 2 द ि ो

(d) ि त 1 ि ही 2

RAUSIAS-FC19E1003 21

Q54) Consider the following statements

1 Megalithic people buried goods in

graves

2 The megalithic culture in South

India was a full-fledged Copper

Age culture

Which of the statements given above

isare correct

(a) 1 only

(b) 2 only

(c) Both 1 and 2

(d) Neither 1 nor 2

Q55) Which of the following kingdoms isare

mentioned in the Ashokan inscriptions

1 Cholas

2 Pandyas

3 Keralaputras (Cheras)

Select the correct answer using the code

given below

(a) 1 only

(b) 1 and 2 only

(c) 3 only

(d) 1 2 and 3

Q56) The Battle of Bhima-Koregaon was

fought between the soldiers of the

Company and the strong Peshwa army

(Marathas) under Bajirao II This war

was a part of the

(a) First Anglo-Maratha war

(b) Second Anglo-Maratha war

(c) Third Anglo- Mysore war

(d) Third Anglo-Maratha war

Q57) Consider the following statements

1 Mahadev Desai followed Gandhiji all over the country to persuade him to come to Champaran to investigate the problem associated

with tinkathia system

2 Narhari Parikh accompanied Gandhi ji during the Champaran

Satyagraha

Which of the statements given above isare correct

(a) 1 only

(b) 2 only

(c) Both 1 and 2

(d) Neither 1 nor 2

Q58) Consider the following statements

1 The Nanda Dynasty started the practice of granting tax-free villages to brahmanas and

Buddhist monks

2 The official language of the Satavahanas was Prakrit

Which of the statements given above

isare correct

(a) 1 only

(b) 2 only

(c) Both 1 and 2

(d) Neither 1 nor 2

Q59) Consider the following statements about the objectives of the lsquoadopt a heritagersquo

project

1 It will harness tourism potential for its effects on employment generation and economic

development

2 It will enhance the tourist attractiveness in a sustainable manner by developing world class infrastructure at heritage sites

Which of the statements given above

isare correct

(a) 1 only

(b) 2 only

(c) Both 1 and 2

(d) Neither 1 nor 2

RAUSIAS-FC19E1003 22

Q60) ldquoभारतीय जिजातीय सहकारी निपरि निकास सोघrdquo

(The Tribal Co-operative Marketing

Development Federation of India - TRIFED)

क सोदभण म निमननिखित कथि ो पर निचार कीनजए

1 यह एक राषटर ीय सतर का शीिण सोगठि ह ज

भारत सरकार क गह मोतरािय क परशासनिक

नियोतरर क अरीि काम कर रहा ह

2 इसका मखय उददशय दश म जिजातीय ि ग ो

का सामानजक-आनथणक निकास करिा ह

उपयणकत कथि ो म स कौि-सास सही हह

(a) किि 1

(b) किि 2

(c) 1 और 2 द ि ो

(d) ि त 1 ि ही 2

Q61) निमननिखित म स कौि-सास उपनयास परमचोद क

दवारा नििा गया हनिि गए ह

1 रोगभनम

2 ग दाि

3 ग रा

िीच नदए गए कट का परय ग कर सही उततर चनिए

(a) किि 1

(b) किि 2

(c) किि 1 और 2

(d) 1 2 और 3

Q62) नगदधा ितय क सोदभण म निमननिखित कथि ो पर निचार

कीनजए

1 नगदधा नबहार की मनहिाओो क दवारा तयौहार क

समय और फसि की बिाई तथा कटाई क

अिसर पर नकया जाि िािा एक पारोपररक

दहाती ितय ह

2 इस ितय क दवारा मनहिाऐो अपिी परसननता

परकट करती ह तथा नगदधा क परदशणि क

माधयम स परि िचणसव िाि समाज म

मनहिाओो की दबी हई भाििाओो क परकट

करती ह

उपयणकत कथि ो म स कौि-सास सही हह

(a) किि 1

(b) किि 2

(c) 1 और 2 द ि ो

(d) ि त 1 ि ही 2

Q63) निमननिखित कथि ो पर निचार कीनजए

1 मलला शाह बदखशी दारा नशक ह क

आधयाखतमक गर थ

2 औरोगरब ि मजम-उि-बहरीि या द समदर ो

का सोगम िामक उललििीय रचिा नििी थी

3 दारा नशक ह क अपि पिणज अकबर क गर ो

क उततरानरकारी क रप म दिा गया था

नजसम उसि रानमणक बहििाद और समनवयता

क बढ़ािा नदया था

उपयणकत कथि ो म स कौि-सास सही हह

(a) किि 1 और 3

(b) किि 2

(c) किि 1 और 2

(d) 1 2 और 3

RAUSIAS-FC19E1003 23

Q60) Consider the following statements about

the Tribal Cooperative Marketing

Development Federation of India

(TRIFED)

1 It is a national-level apex

organization functioning under the

administrative control of Ministry

of Home Affairs Government of

India

2 The main objective of TRIFED is

socio-economic development of

tribal people in the country

Which of the statements given above

isare correct

(a) 1 only

(b) 2 only

(c) Both 1 and 2

(d) Neither 1 nor 2

Q61) Which of the following novels isare

written by Premchand

1 Rangabhumi

2 Godan

3 Gora

Select the correct answer using the code

given below

(a) 1 only

(b) 2 only

(c) 1 and 2 only

(d) 1 2 and 3

Q62) Consider the following statements about

Giddha dance

1 Giddha is a traditional pastoral

dance performed by the women of

Bihar at festival times and at the

sowing and reaping of the harvest

2 By this dance the women reveal

their joy expel their suppressed

feelings in a male dominated

society through the performance of

Giddha

Which of the statements given above

isare correct

(a) 1 only

(b) 2 only

(c) Both 1 and 2

(d) Neither 1 nor 2

Q63) Consider the following statements

1 Mullah Shah Badakhshi was the

spiritual mentor of Dara Shukoh

2 Aurangzeb wrote the remarkable

work called ldquoMajma-ul-Bahrainrdquo or

the ldquoThe confluence of two seasrdquo

3 Dara Shukoh was seen as

inheriting the qualities of his

ancestor Akbar in that he

promoted religious pluralism and

syncretism

Which of the statements given above

isare correct

(a) 1 and 3 only

(b) 2 only

(c) 1 and 2 only

(d) 1 2 and 3

RAUSIAS-FC19E1003 24

Q64) निमननिखित कथि ो पर निचार कीनजए

1 ग मतशवर परनतमा निोधयनगरी पहाड़ी पर खसथत ह

2 शरिरबिग िा िह सथाि ह जहाो मौयण िोश क

सोसथापक चोदरगपत मौयण अपि नसोहासि क

तयागि क बाद जि तपसवी बि गए थ

उपयणकत कथि ो म स कौि-सास सही हह

(a) किि 1

(b) किि 2

(c) 1 और 2 द ि ो

(d) ि त 1 ि ही 2

Q65) निमननिखित कथि ो पर निचार कीनजए

1 पराताखतवक साकषय स पता चिता ह नक पराची

घाटी सभयता हड़पपा और म हिज दाड़ द ि ो

की पिणिती ह

2 पराची िदी भििशवर स निकिती ह

उपयणकत कथि ो म स कौि-सास सही हह

(a) किि 1

(b) किि 2

(c) 1 और 2 द ि ो

(d) ि त 1 ि ही 2

Q66) निमननिखित कथि ो म स कौि-सास सही हह

1 िजराह क समारक ो क समह का निमाणर

चोदि राजिोश क शासिकाि क दौराि हआ

था

2 य समारक हररिोदर पिणत शरोििा म खसथत ह

3 म रक क यातरी इबन बतता ि अपि सोसमरर ो

म िजराह क मोनदर ो की यातरा का उललि

नकया था तथा इन काजराण िाम स समब नरत

नकया था

िीच नदए गए कट का परय ग कर सही उततर चनिए

(a) किि 1

(b) किि 1 और 2

(c) किि 2 और 3

(d) किि 1 और 3

Q67) निमननिखित कथि ो म स कौि-सास सही हह

1 डॉ बी आर अमबडकर ि दी एनिनहिशि

ऑफ़ कासट (The Annihilation of Caste)

नििी थी नजसम उन ोि नहोद रमण म िोशािगत

पजारी की परथा क उनमिि की आिशयकता

पर बि नदया था

2 डॉ राजदर परसाद ि थॉटस ऑि पानकसताि

(Thoughts on Pakistan) िामक पसतक

नििी थी

िीच नदए गए कट का परय ग कर सही उततर चनिए

(a) किि 1

(b) किि 2

(c) 1 और 2 द ि ो

(d) ि त 1 ि ही 2

Q68) निमननिखित कथि ो म स कौि-सास सही हह

1 महरगढ़ भारतीय उपमहादवीप म एक परनसदध

ििपािार बसती ह ज नसोर पराोत पानकसताि म

खसथत ह

2 बरणह म म कतत ो क उिक सवामी क साथ कबर ो

म दफिाया जाता था

िीच नदए गए कट का परय ग कर सही उततर चनिए

(a) किि 1

(b) किि 2

(c) 1 और 2 द ि ो

(d) ि त 1 ि ही 2

Q69) निमननिखित कथि ो म स कौि-सास सही हह

1 काकानटय मोनदर अनरकतर नशि क समनपणत

2 हिमक ोडा म हजार-सतोभ िाि मोनदर (The

Thousand-Pillared Temple) का निमाणर

काकानटय समराट रदर ि करिाया था

िीच नदए गए कट का परय ग कर सही उततर चनिए

(a) किि 1

(b) किि 2

(c) 1 और 2 द ि ो

(d) ि त 1 ि ही 2

RAUSIAS-FC19E1003 25

Q64) Consider the following statements

1 Gommateshwara Statue is located

on the Vindyagiri Hill

2 Shravanabelagola is the place

where Chandragupta Maurya the

founder of the Mauryan dynasty

became a Jain ascetic after

relinquishing his throne

Which of the statements given above

isare correct

(a) 1 only

(b) 2 only

(c) Both 1 and 2

(d) Neither 1 nor 2

Q65) Consider the following statements

1 Archaeological evidence shows

that the Prachi Valley Civilisation

predates both Harappa and

Mohenjo-Daro

2 The Prachi river originates from

Bhubaneswar

Which of the statements given above

isare correct

(a) 1 only

(b) 2 only

(c) Both 1 and 2

(d) Neither 1 nor 2

Q66) Which of the following statements

isare correct

1 The Khajuraho group of

monuments was built during the

rule of the Chandela dynasty

2 These monuments are located in

Harischandra mountain range

3 Ibn Battuta the Moroccan

traveller in his memoirs mentioned

visiting Khajuraho temples and

called them Kajarra

Select the correct answer using the code

given below

(a) 1 only

(b) 1 and 2

(c) 2 and 3

(d) 1 and 3

Q67) Which of the following statements

isare correct

1 Dr BR Ambedkar wrote the

Annihilation of Caste emphasising

the need to do away with the

practice of hereditary priesthood in

Hinduism

2 The book lsquoThoughts on Pakistanrsquo

was written by Dr Rajendra

Prasad

Select the correct answer using the code

given below

(a) 1 only

(b) 2 only

(c) Both 1 and 2

(d) Neither 1 nor 2

Q68) Which of the following statements

isare correct

1 Mehrgarh is a famous Neolithic

settlement in the Indian

subcontinent which is situated in

Sindh province Pakistan

2 At Burzahom dogs were buried

with their masters in their graves

Select the correct answer using the code

given below

(a) 1 only

(b) 2 only

(c) Both 1 and 2

(d) Neither 1 nor 2

Q69) Which of the following statements

isare correct

1 The Kakatiya temples are

dedicated mostly to Siva

2 The Thousand-Pillared Temple at

Hanamkonda was built by the

Kakatiya king Rudra

Select the correct answer using the code

given below

(a) 1 only

(b) 2 only

(c) Both 1 and 2

(d) Neither 1 nor 2

RAUSIAS-FC19E1003 26

Q70) निमननिखित कथि ो म स कौि-सास सही हह

1 अहमदाबाद नमि हड़ताि क दौराि महातमा

गाोरी ि शरनमक ो क पकष क मजबत करि क

निए आमरर अिशि नकया था

2 अिशि स नमि मानिक ो पर दबाि पड़ा था ज

अोततः शरनमक ो क िति म 15 परनतशत की िखदध

करि क निए सहमत हए थ

िीच नदए गए कट का परय ग कर सही उततर चनिए

(a) किि 1

(b) किि 2

(c) 1 और 2 द ि ो

(d) ि त 1 ि ही 2

Q71) निमननिखित म स नकसक नकिक भारत स यिसक

की माििता की अमतण साोसकनतक निरासत की

परनतनिनर सची (The UNESCOrsquos List of the

Representative List of the Intangible

Cultural Heritage of Humanity) म शानमि

नकया गया ह

1 मनडयटट

2 सोकीतणि

3 को भ मिा

िीच नदए गए कट का परय ग कर सही उततर चनिए

(a) किि 1 और 2

(b) किि 2 और 3

(c) किि 3

(d) 1 2 और 3

Q72) निमननिखित जिजानतय ो म स कौि-सीसी ो

जिजानतजिजानतया िागािड स सोबोनरत हह

1 अोगामी

2 ककी

3 जारिा

िीच नदए गए कट का परय ग कर सही उततर चनिए

(a) किि 1

(b) किि 1 औऔ 2

(c) किि 2

(d) 1 2 और 3

Q73) निमननिखित कथि ो म स कौि-सास सही हह

1 राषटर कट सामराजय की सथापिा दोनतदगण ि की थी

नजसि मानयाित म अपिी राजरािी की

सथापिा की थी

2 राषटर कट समराट अम घििण एक ििक था और

उस कनिताओो पर पहिी कननड़ पसतक नििि

का शरय नदया जाता ह

िीच नदए गए कट का परय ग कर सही उततर चनिए

(a) किि 1

(b) किि 2

(c) 1 और 2 द ि ो

(d) ि त 1 ि ही 2

Q74) निमननिखित कथि ो म स कौि-सास सही हह

1 कशब चोदर सि ि ततवब नरिी सभा की

अधयकषता की थी ज आधयाखतमक सतय की

ि ज म सोिि थी

2 बरहम समाज ि मािि गररमा पर बि नदया

मनतणपजा का निर र नकया और सती परथा जसी

सामानजक बराइय ो की आि चिा की

िीच नदए गए कट का परय ग कर सही उततर चनिए

(a) किि 1

(b) किि 2

(c) 1 और 2 द ि ो

(d) ि त 1 ि ही 2

Q75) निमननिखित कथि ो म स कौि-सास सही हह

1 भारत म नचशती नसिनसिा खवाजा म इिददीि

नचशती क दवारा सथानपत नकया गया था

2 नचशती परोपरा की एक परमि निशिता

आतमसोयम थी नजसम साोसाररक म ह स दरी

बिाए रििा शानमि था

िीच नदए गए कट का परय ग कर सही उततर चनिए

(a) किि 1

(b) किि 2

(c) 1 और 2 द ि ो

(d) ि त 1 ि ही 2

RAUSIAS-FC19E1003 27

Q70) Which of the following statements

isare correct

1 During the Ahmedabad Mill Strike

Mahatma Gandhi undertook a fast

unto death to strengthen the

workersrsquo resolve

2 The fast had effect of putting

pressure on mill owners who

finally agreed to give the workers a

15 per cent increase in wages

Select the correct answer using the code

given below

(a) 1 only

(b) 2 only

(c) Both 1 and 2

(d) Neither 1 nor 2

Q71) Which of the following are included in

the UNESCOrsquos list of the representative

list of the intangible cultural heritage of

humanity from India

1 Mudiyettu

2 Sankirtana

3 Kumbh Mela

Select the correct answer using the code

given below

(a) 1 and 2 only

(b) 2 and 3 only

(c) 3 only

(d) 1 2 and 3

Q72) Which of the following tribes isare

related to Nagaland

1 Angami

2 Kuki

3 Jarawa

Select the correct answer using the code

given below

(a) 1 only

(b) 1 and 2 only

(c) 2 only

(d) 1 2 and 3

Q73) Which of the following statements

isare correct

1 Rashtrakuta kingdom was founded by Dantidurga who established his capital at Manyakhet

2 Amoghavarsha a Rashtrakuta king was an author and is credited with writing the first

Kannada book on poetics

Select the correct answer using the code given below

(a) 1 only

(b) 2 only

(c) Both 1 and 2

(d) Neither 1 nor 2

Q74) Which of the following statements isare correct

1 Keshab Chandra Sen headed the Tattvabodhini Sabha which was engaged in search of spiritual truth

2 The Brahmo Samaj laid emphasis on human dignity opposed idolatry and criticized such social

evils as the practice of Sati

Select the correct answer using the code given below

(a) 1 only

(b) 2 only

(c) Both 1 and 2

(d) Neither 1 nor 2

Q75) Which of the following statements isare correct

1 The Chishti order was established in India by Khwaja Moinuddin

Chishti

2 A major feature of the Chishti tradition was austerity including maintaining a distance from the

worldly power

Select the correct answer using the code

given below

(a) 1 only

(b) 2 only

(c) Both 1 and 2

(d) Neither 1 nor 2

T e s t i s p a r t o f R a u rsquo s I A S T e s t s e r i e s f o r P r e l i m i n a r y E x a m 2 0 1 9

FOUNDATION + CURRENT AFFAIRS

GENERAL STUDIES (PAPER ndashI)

FOUNDATION TEST ndashIII

SUBJECT NCERT History Class VI-X + Current Affairs

Time Allowed 1frac12 Hours Maximum Marks 150

I NSTRUCT IONS

1 IMMEDIATELY AFTER THE COMMENCEMENT OF THE EXAMINATION YOU SHOULD CHECK

THAT THIS TEST BOOKLET DOES NOT HAVE ANY UNPRINTED OR TORN or MISSING PAGES OR

ITEMS ETC IF SO GET IT REPLACED BY A COMPLETE TEST BOOKLET

2 This Test Booklet contains 75 items (questions) Each item is printed both in Hindi and English

Each item comprises four responses (answers) You will select the response which you want to mark

on the Answer Sheet In case you feel that there is more than one correct response mark the

response which you consider the best In any case choose ONLY ONE response for each item

3 You have to mark all your responses ONLY on the separate Answer Sheet (OMR sheet) provided

Read the directions in the Answer Sheet

4 All items carry equal marks

5 Before you proceed to mark in the Answer Sheet the response to various items in the Test booklet

you have to fill in some particulars in the Answer Sheet as per instructions contained therein

6 After you have completed filling in all your responses on the Answer Sheet and the examination has

concluded you should hand over to the Invigilator only the Answer Sheet You are permitted to

take away with you the Test Booklet

7 Penalty for wrong answers

THERE WILL BE PENALTY FOR WRONG ANSWERS MARKED BY A CANDIDATE IN THE

OBJECTIVE TYPE QUESTION PAPERS

(i) There are four alternatives for the answer to every question For each question for which a

wrong answer has been given by the candidate one-third of the marks assigned to that

question will be deducted as penalty

(ii) If a candidate gives more than one answer it will be treated as a wrong answer even if one of

the given answers happens to be correct and there will be same penalty as above to that

question

(iii) If a question is left blank ie no answer is given by the candidate there will be no penalty for

that question

T h i s t e s t i s p a r t o f R a u rsquo s I A S T e s t s e r i e s f o r P r e l i m i n a r y E x a m 2 0 1 9

Test Code

FC19E1003

FC19H1003 29

Answers and Explanations of

NCERT History Class VI-X + Current Affairs (FC19E1003)

Q1) उततर (c)

सपषटीकरण

- ऋगवद म दविय ो और दिताओो क समवपित एक

हजार स अविक सत तर (शल क) ह

- य शल क ऋविय ो क दवारा रच गए थ और परि ो

दवारा सीख जात थ

- हालाोवक कछ शल क मवहलाओो (जस वक अपाला

घ सा ल पामदरा मतरयी और गागी) क दवारा भी रच

गए थ

- ऋगवद म सोिाद क रप म कई शल क मौजद ह

- हम विशवावमतर नामक एक ऋवि और दविय ो क

रप म पजी जान िाली द नवदय ो (वयास और

सतलज) क बीच िाताि का उदाहरण वमलता ह

- इसस पता चलता ह वक विशवावमतर िवदक काल स

सोबोवित थ

Q2) उततर (b)

सपषटीकरण

- करनल गफाओो स राख क अिशि परापत हए ह

ज इस ओर सोकत करत ह वक ततकालीन ल ग

अवि क उपय ग स पररवचत थ

- य गफाएो आोधर परदश म सथथत ह

Q3) उततर (c)

सपषटीकरण

bull बरािह म ितिमान कशमीर म सथथत एक

परागवतहावसक थथल ह जहाो ल ग गडढ क घर ो का

वनमािण करत थ

bull य घर जमीन क ख द कर बनाए जात थ तथा नीच

जान क वलए सीवियाा ह ती थी

bull ऐसा अनमान लगाया जाता ह वक य घर ठो ड क

मौसम म आशरय परदान करत थ

Q4) उततर (c)

सपषटीकरण

bull परालख-विदया (Epigraphy) क वशलालख ो क

अधययन क रप म पररभावित वकया जाता ह

bull हसतवलसखत दसतािज ो क माधयम स इवतहास

और सावहतय क अधययन क पाोडवलवप विजञान

(Manuscriptology) कहत ह

bull पराचीन लखन परणावलय ो क अधययन और

ऐवतहावसक पाोडवलवपय ो क समझन तथा वतवथ

वनिािरण क पलीओगराफी (Palaeography) कहा

जाता ह

bull नयवमजमविकस (Numismatics) वसक ो क

अधययन क सोदवभित करता ह

Q5) उततर (a)

सपषटीकरण

- चरक सोवहता चरक क दवारा वलखी गई आयिद

और िदयक-शासर पर एक महतवपणि पसतक ह

- ि भारतीय िदयक-शासर की पारमपररक परणाली

वजस आयिद क नाम स जाना जाता ह क

अभयासकताि थ

- ऐसा माना जाता ह वक चरक का विकास दसरी

शताबदी (ईसा पिि) और दसरी शताबदी (ईसवी) क

मधय हआ था

Q6) उततर (b)

सपषटीकरण

- भाग फसल ो पर वलए जान िाल कर क सोदवभित

करता ह ज कल फसल उतपादन का 16 िाो भाग

था

- ldquoकममकारrdquo शबद भवमहीन कवि शरवमक िगि क

वलए परय ग वकया जाता था

- ldquoअशवमिrdquo (वजस घ ड क बवलदान क रप म भी

जाना जाता ह) एक अनषठान ह ता था वजसम एक

घ ड क सवतोतर रप स घमन क वलए छ ड वदया

FC19H1003 30

जाता ह और राजा क सवनक उसकी रखिाली

करत थ

Q7) उततर (d)

सपषटीकरण

- ऋगववदक काल म घ ड ो क रथ ो म ज ता जाता था

ज (रथ) भवम मिवशय ो आवद पर कबजा करन क

वलए लड गए यद ो म उपय ग वकए जात थ

- इसस यह पता चलता ह वक घ ड ो यकत रथ ो का

उपय ग महाजनपद काल स काफी पहल आरमभ

हआ था

- ऋगववदक काल म मिवशय ो भवम जल आवद पर

कबजा करन क वलए तथा ल ग ो क पकडन क

वलए यद वकय जात थ

- अविकाोश परि इन यद ो म भाग वलया करत थ

- हालाोवक उस समय क ई वनयवमत सना नही ो ह ती

थी लवकन उस काल म सभाऐो ह ती थी ो वजनम

ल ग यद क मामल ो पर चचाि करत थ

- वनयवमत सनाएा महाजनपद काल का िवशषटय थी

वजनम पदल सवनक ो की विशाल सनाएा रथ तथा

हाथी शावमल ह त थ

Q8) उततर (a)

सपषटीकरण

- बद शाकय कल स सोबोवित थ और कशीनारा म

उनका वनिन हआ था

- बद न अपनी वशकषाएा पराकत भािा म दी थी ो ज

आम ल ग ो की भािा थी

Q9) उततर (c)

सपषटीकरण

- पराचीन भारत म दशिनशासर की छह शाखाएा थी ो

िशविक नयाय समखया य ग पिि वममाोसा और

िदाोत या उततर वममाोसा

- इनकी थथापना करमश कनाद गौतम कवपल

पतोजवल जावमनी और वयास ऋविय ो न की थी

Q10) उततर (b)

सपषटीकरण

महािीर की वशकषाऐो छठी शताबदी म िललभी म

सोकवलत की गई थी ो

Q11) उततर (c)

सपषटीकरण

- पारमपररक रप स चाणकय क कौविलय अथिा

विषणगपत क नाम स जाना जाता ह

- उसन अथिशासतर ज एक पराचीन भारतीय

राजनवतक आलख ह वलखा था

Q12) उततर (d)

सपषटीकरण

- भारत का राषटर ीय वचनह सारनाथ (उततर परदश) क

अश क सतमभ क ऊपर (शीिि पर) वसोह कवपिल

का एक अनरपण ह

- इस राषटर ीय वसदाोत सतयमि जयत क साथ

सोय वजत वकया गया ह

- रामपिि बल का नाम रामपिि (वबहार) क नाम पर

पडा जहाा इसकी ख ज हई थी

- यह अपन नाजक नकाशी मॉडल क वलए परवसदद

ह वजसम क मल तवचा सोिदनशील नथन ो सतकि

कान और मरबत िााग ो क शरषठतर परवतरप क

परदवशित वकया गया ह

- यह भारतीय और फारसी ततव ो का एक ससममशरण

- सोवकससा उततर परदश म सथथत ह

Q13) उततर (a)

सपषटीकरण

का िर वसोह ज एक महान य दा थ वबहार स

सोबोवित थ

Q14) उततर (b)

सपषटीकरण

िललालर शबद बड भ-सवावमय ो क वलए परय ग

वकया जाता था

FC19H1003 31

Q15) उततर (c)

सपषटीकरण

- अररकमड एक तिीय बसती थी जहाो दर दश ो स

आन िाल जहाज ो का माल उतारा जाता था

- यहाो पर ईोि ो का एक विशाल ग दाम वमटटी क

बतिन (वजनम एमफ रा - द हरी मवठय ो का लोबा

घडा - शावमल ह) और एरिाइन (Arretine)

मदभाोड पाए गए थ

- इस थथान पर र मन दीपक काोच क बन पातर और

रतन भी पाए गए थ

Q16) उततर (a)

सपषटीकरण

- मिनदर सोगम कविताओो म उसललसखत एक

तवमल शबद ह वजसका अथि ह ldquoतीन परमखrdquo

- यह तीन सततारि पररिार ो क मसखयाओो क वलए

परय ग वकया जाता ह च ल चर और पाणडय

Q17) उततर (c)

सपषटीकरण

- ऋग िद म सभा विदाथा तथा गण जसी

जनजावतय ो पर अथिा किोब पर आिाररत

सभाओो का उललख ह

- आरसमभक िवदक काल म सभाओो और सवमवतय ो

का विशि महतव ह ता था

- यहाा तक की मसखया अथिा राजा भी उनका

समथिन परापत करन क वलए आतर रहत थ

Q18) उततर (a)

सपषटीकरण

- जन िमि न ईशवर क अससततव क मानयता त दी ह

वकनत उसन ईशवर क वजना क पद स नीच रखा

- जन िमि न बौद िमि की तरह िणि परणाली की

भरतिना नही ो की थी

Q19) उततर (d)

सपषटीकरण

- च ल ो और पाणडय ो न शसकतशाली तिीय शहर ो का

विकास वकया था

- च ल ो का सबस महतवपणि शहर पहार (या

कािरीपटटीनम) था |

- मदरई पाणडय ो की राजिानी थी

Q20) उततर (b)

सपषटीकरण

- ldquoबदचररतrdquo बद का जीिन-ितताोत ह

- इस अशवघ ि क दवारा वलखा गया था

Q21) उततर (a)

सपषटीकरणः

- तवमल कवि अपपर भगिान वशि क भकत थ

- इस परकार ि एक नयनार सोत थ

Q22) उततर (d)

सपषटीकरणः

- समदरगपत एक परवसद गपत शासक था

- उसन वसक ो पर िीणा बजात हए अपनी छवि

अोवकत करिाई थी

- यह सोगीत क परवत उसक परम क दशािता ह

- हम उसकी इलाहाबाद परशससत स महतवपणि

ऐवतहावसक जानकारी वमलती ह वजसकी रचना

उसक दरबार क कवि हररसन न की थी

Q23) उततर (b)

सपषटीकरणः

- विकरम सोित की शरआत ििि 58 ईसा पिि म

चनदरगपत वदवतीय न की थी

- यह शक ो पर उसकी जीत और उस विकरमावदतय

की पदिी वमलन क उपलकषय म आरमभ वकया गया

था

FC19H1003 32

- बानभटट न हिििििन का जीिन-ितताोत हििचररत

(ज सोसकत म थी) वलखी थी

Q24) उततर (c)

सपषटीकरणः

- सोवि-विगरावहका यद एिो शाोवत का मोतरी

- साथििाह वयापाररय ो क कावफल ो का नता

Q25) उततर (a)

सपषटीकरणः

- जआन झाोग (हसआन रताोग ndash Hsuang Tsang)

एक चीनी यातरी था ज हिििििन क शासनकाल म

भारत आया था

- ििि 630 ईसवी स ज दशक आरमभ हआ था उसम

जआन झाोग मधय एवशया ईरान और

अफग़ावनसतान की यातरा करन क पशचात कशमीर

क रासत स भारत आया था

- उसन उततर स पिि तक की यातरा की और िह

लगभग 2 ििि वबहार म रहा

- जआन झाोग न नालनदा विशवविदयालय म विदयावथिय ो

और विदवान ो क साथ पारसपररक विचार-विमशि

वकया थथानीय भािाओ ा म वनपणता परापत की तथा

बौद सतप ो की ख ज की

Q26) उततर (c)

सपषटीकरणः

- परदवकषणा पथ बौद िासतकला म सतप क चार ो

ओर बनाया जान िाला एक घमािदार पथ ह ता

- परशन म वदए गए बाकी क तीन ो ततव वहोद मसनदर ो की

िासतकला क भाग ह

Q27) उततर (d)

सपषटीकरणः

परशन म वदए गए सभी मोवदर ो म वयापक रप स

ईोि ो (पकी ईोि ो) का परय ग पतथर ो क साथ हआ

Q28) उततर (c)

सपषटीकरण

- महममद कली कतब शाह ग लकणडा का सलतान

था

- िह अकबर का समकालीन था

- सावहतय और िासतकला म उसकी अतयाविक

रवच थी

- िह एक महान कवि था

- िह दसखनी उदि फारसी और तलग म वलखता था

- उसन अपन पीछ एक विसतत वदिान (सोगरह)

छ डा ह

- अभी हाल ही म तलोगाना म ग लकणडा क वकल

क अनदर खदाई वकय गए बाग-ए-नाया वकला

बाग क चार ो ओर रप-रखा क मानवचतरण क

वलए भारतीय परातासतवक सिकषण (The

Archaeological Survey of India ndash ASI)

गराउणड पनीिर विोग रडार (Ground Penetrating

Radar) का परय ग करगा

Q29) उततर (a)

सपषटीकरणः

- वसलपपावदकारम एक तवमल महाकावय ह वजसकी

रचना इलाोग क दवारा लगभग 1800 ििि पिि की

गई थी

- यह क िलन नामक एक वयापारी की कहानी ह

ज माििी नामक एक गवणका (िशया) स परम

करन लगा था

- मवनमकलाई क िलन और माििी की पतरी की

कहानी ह

Q30) उततर (a)

सपषटीकरण

- चरक आयिद और वचवकरता की एक महतवपणि

रचना चरक सोवहता क लखक ह

- बरहमगपत क अपनी रचना बरहम-सफि-वसदानत

(ज एक खग लीय रचना ह) क कारण परवससद

वमली

FC19H1003 33

- बगदाद म इसका अनिाद अरबी भािा म वकया

गया था

- इसका इसलावमक गवणत और खग ल-विजञान पर

महतवपणि परभाि पडा था

- बाद म अपन जीिनकाल म बरहमगपत न

ldquoखोडखयाकrdquo वलखी ज एक खग लीय पससतका

(एक छ िी पसतक) थी

- इसम आयिभटट की अिि-रावतर क परतयक वदन की

शरआत परणाली का परय ग वकया गया था

Q31) उततर (c)

सपषटीकरण

- अमीर खसर एक परवसद सफी सोगीतकार कवि

और विदवान थ

- 1318 म उनह ोन पाया वक इस भवम (वहोदसतान) क

हर कषतर म अलग-अलग भािा थी लाहौरी

कशमीरी दवारसमदरी (दवकषणी कनाििक म)

तलोगाना (आोधर परदश म) गजरी (गजरात म)

माबारी (तवमलनाड म ) अििी (पिी उततर परदश

म) और वहोदिी (वदलली क आस-पास क कषतर म)

आवद

- उनह न यह बताया वक सोसकत वकसी भी कषतर स

सोबोवित नही ो थी और किल बराहमण ही इस भािा

का जञान रखत थ

Q32) उततर (c)

सपषटीकरण

- वहरणय-गभि सववणिम गभि क सोदवभित करता ह

- जब बराहमण ो की सहायता स यह अनषठान वकया

जाता था त यह माना जाता था वक बवल दन िाल

का कषवतरय क रप म पनजिनम ह गा

Q33) उततर (d)

सपषटीकरण

- कदमई भवम राजसव पर कर क सोदवभित करता

- गवावलयर परशससत म नागभि क दवारा वकय गए

श िण का िणिन वकया गया ह |

- नागभि एक परवतहार राजा था

Q34) उततर (b)

सपषटीकरण

- राजतरो वगनी 12िी ो शताबदी म कलहन क दवारा

रवचत एक सोसकत पसतक (िकसट) ह

- यह परारसमभक भारत की ऐवतहावसक इवतितत थी

- तकि सोगत रप स इस अपन परकार की सिोततम

और सिािविक विशवसनीय कवत माना जाता ह

- यह कशमीर कषतर क पराचीनतम समय स लकर

उसकी रचना की तारीख तक क समपणि इवतहास

का आचछादन करती ह

Q35) उततर (c)

सपषटीकरण

- गााि की आम सभा क ldquoउरrdquo कहा जाता था

- ldquoउरrdquo म गााि क सभी कर दन िाल वनिासी

शावमल ह त थ

Q36) उततर (a)

सपषटीकरण

- वदलली सलतनत म ldquoतारीखrdquo इवतहास लखन का

एक रप था

- ldquoतािरीखrdquo क लखक विदवान परि ह त थ वजनम

सवचि परशासक इतयावद शावमल थ

Q37) उततर (a)

सपषटीकरण

- अलाउददीन सखलजी अपन सवनक ो क ितन का

भगतान नकद म करता था न वक इकता क रप

- सवनक अपना सामान वदलली म वयापाररय ो स

खरीदत थ अतः इस बात का भय था वक वयापारी

कही ो िसतओो का मलय न बिा द

- इसकी र कथाम क वलए अलाउददीन सखलजी न

वदलली म कीमत ो क वनयसित वकया

FC19H1003 34

- अविकारीगण धयानपििक मलय ो का सिकषण करत

थ तथा ज वयापारी वनिािररत मलय पर माल नही ो

बचत थ उनक दसणडत वकया जाता था

Q38) उततर (d)

सपषटीकरण

- वदलली सििपरथम त मर राजपत ो क अिीन उनक

सामराजय की राजिानी बनी थी

- 12िी ो शताबदी क मधय म अजमर क चौहान ो

(वजनह चाहमान ो क नाम स भी जाना जाता ह) न

त मर राजपत ो क परावजत वकया था

- त मर ो और चौहान ो क अिीन वदलली एक

महतवपणि िावणसजयक क दर बन गया था

- कई जन वयापारी यहाा रहन लग थ और उनह ोन

कई मोवदर भी बनिाए

- यहाा पर मवदरत वसक वजनह ldquoदहलीिालrdquo क नाम

स जाना जाता था वयापक रप स परचलन म थ

Q39) उततर (c)

सपषटीकरण

- म ठ की मसिद का वनमािण वसको दर ल दी क

राजयकाल म उसक मिी क दवारा करिाया गया

था

- बगमपरी मसिद का वनमािण महममद तगलक क

शासनकाल म हआ था

- यह मसिद विशव का पणयथथान (The

Sanctuary of the World) और वदलली म महममद

तगलक की नई राजिानी जहाोपनाह की मखय

मसिद थी

- कववत- अल - इसलाम मसिद का विसतार

इलतसिश और अलाउददीन सखलजी न वकया था

- मीनार का वनमािण तीन सलतान ो कतबददीन ऐबक

इलतसिश और वफर ज शाह तगलक क दवारा

करिाया गया था

Q40) उततर (c)

सपषटीकरण

- मगल ो क अिीन मनसबदार शबद उस वयसकत क

वलए सोदवभित वकया जाता था वजसक पास मनसब

(अथाित पद) ह ता था

- उस अपना ितन राजसव कायो वजनह जागीर कहत

थ क रप म परापत ह ता था

Q41) उततर (b)

सपषटीकरण

- ldquoभारत छ ड आोद लनrdquo वबरविश शासन क

सखलाफ ल ग ो का एक सवाभाविक विदर ह था

- असखल भारतीय काोगरस सवमवत न 8 अगसत 1942

क बमबई म एक बठक का आय जन वकया था

- इस बठक म परवसद सोकलप ldquoभारत छ ड rdquo क

पाररत वकया गया और इस उददशय क परापत करन

क वलए गाोिी क नततव म एक अवहोसक जन सोघिि

आोद लन की शरआत का परसताि वदया गया

- लवकन अगल ही वदन गाोिी और काोगरस क अनय

परमख नताओो क वगरफतार कर वलया गया

- काोगरस क एक बार वफर अिि घ वित वकया गया

था

Q42) उततर (c)

सपषटीकरण

- साइमन कमीशन यनाइविड वको गडम क सात

साोसद ो का एक समह था

- इस वबरविश भारत क वलए सोििावनक सिार ो का

सझाि दन क वलए गवठत वकया गया था

- इस आय ग म िररषठ वबरविश राजनता सर जॉन

साइमन क नततव म किल वबरविश सदसय ही

शावमल थ

- इसवलए भारत क ल ग ो न साइमन कमीशन क

आगमन क विरद आोद लन वकया था

Q43) उततर (a)

सपषटीकरण

bull दादा भाई नौर जी भारत म वबरविश शासन क

आवथिक पररणाम ो क बार म अपनी विर िी

(परवतकल) राय क वलए जान जात थ

FC19H1003 35

bull अपन कई लख ो और भािण ो म विशि रप स

ldquoपाििी एो ड अन-वबरविश रल इन इसणडया

(Poverty and Un-British Rule in India) म

नौर जी न यह तकि वदया वक भारत पर अतयविक

कर लगाया गया था और इसकी सोपवतत इोगलड की

ओर परिावहत की जा रही थी

bull उनह ोन पराचीन भारतीय गरोथ ो की वयाखया करन

का और भारतीय ो क आिविशवास क बहाल

करन पर कायि नही ो वकया था

उनह ोन वकसी और बात स पहल सभी सामावजक

बराइय ो क उनमलन की आिशयकता पर भी बल

नही ो वदया था

Q44) उततर (c)

सपषटीकरण

bull अगसत 1932 म वबरविश परिानमोतरी मकड नालड न

अपन साोपरदावयक परसकार (The Communal

Award) की घ िणा की थी

bull यह भारत क कई साोपरदावयक वहत ो क बीच विवभनन

सोघिो क हल करन क वलए वबरिन का एकतरफा

परयास था

bull यह परसकार (Award) बाद म 1935 क

अविवनयम (The Act of 1935) म शावमल वकया

गया था

bull इस साोपरदावयक परसकार न मससलम ो क वलए

आरवकषत एक अलग वनिािचक मणडल फॉमिल का

विसतार अनय अलपसोखयक ो क वलए वकया था

वजसम वसख ो भारतीय ईसाइय ो आोगल-भारतीय

समदाय यर पीय समदाय तथा विवशषट कषतरीय

समह ो क शावमल वकया गया था

bull गाोिी न इस परसताि क भारतीय समाज क

विभावजत करन क वलए एक घवणत वबरविश

सावजश क रप म दखा और उसक सखलाफ

आमरण अनशन वकया

Q45) उततर (b)

सपषटीकरण

मौजदा आयात और वनयाित क अवतररक़त

औपवनिवशक भारत क वनमनवलसखत खचो क

वलए एक विशिवनवशचत िन रावश भी दनी पडती

थी

(i) परशासन क वयय

(ii) सना क रख-रखाि क वयय

(iii) यद क वयय

(iv) सिावनितत अविकाररय ो की पशन तथा

(v) वबरिन दवारा अपनी उपवनिश बसती

(कॉल नी) क रख-रखाि क वयय

इनह गह शलक (Home Charges) क रप म

जाना जाता था और लगभग परी तरह स भारत क

दवारा इनका भगतान वकया जाता था

bull गह शलक म वनमनवलसखत घिक शावमल थ

(i) भारतीय ऋण पर दय बयाज

(ii) ईसट इोवडया को पनी क शयरिारक ो क

लाभाोश

(iii) लोदन म भारत कायािलय चलान क वलए िन

(iv) भारत म वनयकत वबरविश कवमिय ो क ितन

और पशन का भगतान करन क वलए िन

(v) रलि पर बयाज

(vi) नागररक और सनय शलक

(vii) इोगलड म सट र (सामगरी) की खरीद

Q46) उततर (b)

सपषटीकरण

bull भारतीय राषटर ीय काोगरस का लाहौर सतर 1929 म

जिाहरलाल नहर की अधयकषता म आय वजत

वकया गया था

bull इस सतर म भारतीय राषटर ीय आोद लन स समबसित

कई महतवपणि पररणाम सामन आय थ

(i) सििपरथम इस सतर म काोगरस क अधयकष पद

पर जिाहरलाल नहर क चना गया था ज

काोगरस म िामपोवथय ो की बिती हई ताकत

का सपषट सोकत था

(ii) दसरा इस सतर म पहली बार काोगरस न पणि

सवतोतरता की माोग क उठाया था

इस परकार की माोग काोगरस मोच स पहल कभी भी

नही ो उठाई गई थी

Q47) उततर (b)

सपषटीकरण

FC19H1003 36

bull इस ररप िि न वकसी भी समदाय क वलए पथक

वनिािचक मोडल अथिा अलपसोखयक ो क वलए

भाराोश की वसफाररश नही ो की थी

bull तथावप इस ररप िि न उन पराोत ो म अलपसोखयक

सीि ो क आरकषण की अनमवत दी थी जहाा पर कम

स कम दस परवतशत अलपसोखयक ह

bull लवकन यह समदाय क आकार क अनपात म ह ना

चावहए था

bull इस ररप िि म भारत क वलए पणि सवतोतरता क

वलए क ई पराििान नही ो था

Q48) उततर (c)

सपषटीकरण

bull आरो वभक िवदक आयो का िमि मखय रप स

परकवत की पजा और यजञ था

bull परारो वभक आयि िमि परकवत की पजा क समान था

bull िासति म उनक चार ो ओर की शसकतयाा वजनह न

त ि वनयोवतरत कर सकत थ और न ही समझ पाए

थ उनह वदवयता क साथ वनिवशत वकया गया तथा

उनह मादा या नर दिीदिताओो क रप म

परतीकतव वकया गया था

bull उनह ोन कछ यजञ ो का भी वनषपादन वकया था

Q49) उततर (b)

सपषटीकरण

bull सडक और नदी-मागि (जल-मागि) डकती स

सरवकषत नही ो थ

bull उललखनीय ह वक हिििििन क शासनकाल क

दौरान यआन चिाोग (हयएन साोग) का सारा

सामान लि वलया गया था

Q50) उततर (c)

सपषटीकरण

परशन म वदए गए द न ो कथन सही ह

Q51) उततर (b)

सपषटीकरण

bull परोदर दास एक सोत और भगिान कषण क एक

महान भकत थ

bull परोदर दास क कनाििक सोगीत क वपतामह क

रप म जाना जाता ह

bull यदयवप उनक जनम-थथान क बार म काफी

अिकल लगाई जाती रही ह

bull तथावप अब कननड विशवविदयालय हमपी क दवारा

गवठत एक विशिजञ सवमवत इस वनषकिि पर पहोची

ह वक उनका जनम थथान सोभितया कनाििक का

एक छ िा-सा गााि कषमपरा (वशिम गगा वजला)

था

Q52) उततर (c)

सपषटीकरण

bull शरी तयागराज शरी शयाम शासतरी और शरी मथसवामी

दीवकषतर क कनाििक सोगीत की वतरमवति माना

जाता ह

bull उनक कारण ही 18िी ो-19िी ो शताबदी म कनाििक

सोगीत का सववणिम यग आया था

Q53) उततर (d)

सपषटीकरण

bull अभी हाल ही म लौह यगीन-महापािावणक काल

का 2000 ििि पराना एक दलिभ सारक फगस

(Sarcophagus) (पतथर का ताबत) क ललम क

वियर गाोि (क वयलडी क पास वजला क वझक ड

करल राजय) की एक रॉक-कि गफा स ख जा गया

bull यह ताबत वजसम हविय ो क िकड थ खदाई क

दौरान वमला

bull अभी तक इस परकार की दलिभ ख ज करल क

मातर द ही थथान ो स हई ह

bull य द न ो सारक फगी (Sarcophagi) (पतथर क

ताबत) चियर और अथ ली (वजला क वझक ड) क

महापािाण थथल ो स वमल ह

Q54) उततर (a)

सपषटीकरण

FC19H1003 37

दवकषण भारत म महापािाण सोसकवत एक पणि

विकवसत लौह यगीन सोसकवत थी

Q55) उततर (d)

सपषटीकरण

bull च ल पाणडय और करलपतर (चर) इन तीन ो का

उललख अश क क अवभलख ो म वकया गया ह

bull सोभितः य भौवतक सोसकवत क उततर

महापािावणक चरण म थ

Q56) उततर (d)

सपषटीकरण

bull भीमा-क रगाोि की लडाई ततीय आोगल-मराठा

यद का वहससा थी

Q57) उततर (b)

सपषटीकरण

bull राजकमार शकल न गाोिीजी क चोपारण आन तथा

वतनकवथया परणाली स जडी समसया की जाोच क

वलए रारी करन क वलए दश भर म उनका

अनसरण वकया था

bull बज वकश र राजदर परसाद महादि दसाई और

नरहरी पाररख चोपारण सतयागरह क दौरान गाोिी

जी क सहय गी थ

Q58) उततर (b)

सपषटीकरण

bull बराहमण ो और बौद मठिाररय ो क कर-मकत गााि

अनदान म दन की परथा सतिाहन ो न आरमभ की

थी

Q59) उततर (c)

सपषटीकरण

इस कायिकरम क उददशय वनमनानसार ह

(i) बवनयादी पयििन आिाररक सोरचना का विकास

करना

(ii) चयवनत (पहचान वकय गए) कषतर ो म आजीविका क

सजन क वलए दश क साोसकवतक और विरासत

मलय ो क बिािा दना

(iii) विरासत समारक थथल ो पर विशव सतरीय आिाररक

सोरचना विकवसत करक एक सतत तरीक स

पयििक आकििण म िसद करना

(iv) थथानीय समदाय ो की सवकरय भागीदारी क माधयम

स र रगार ो का सजन करना

(v) र रगार उतपादन और आवथिक विकास क वलए

पयििन कषमता का उन पर परभाि का उपय ग

करना तथा

(vi) िारणीय पयििन आिाररक सोरचना का विकास

करना और उसका उवचत सोचालन तथा

रखरखाि सवनवशचत करना

Q60) उततर (b)

सपषटीकरण

bull यह वनकाय ििि 1987 म अससततव म आया था

bull यह एक राषटर ीय सतर का शीिि सोगठन ह ज भारत

सरकार क जनजातीय मामल ो क मोतरालय क

परशासवनक वनयोतरण क अिीन काम कर रहा ह

bull इसका पोजीकत और परिान कायािलय नई वदलली

म सथथत ह

Q61) उततर (c)

सपषटीकरण

bull परमचोद क उपनयास ो म परमाशरम रोगभवम गबन

कमिभवम और ग दान शावमल ह

bull ग रा रिी ोदरनाथ िग र क दवारा रवचत उपनयास ह

bull अभी हाल ही म मोशी परमचोद की 138िी ो जयोती दश

भर म मनाई गई थी

Q62) उततर (b)

सपषटीकरण

bull ldquoवगदाrdquo पोजाब (भारत) एिो पावकसतान की

मवहलाओो क दवारा तयौहार क समय और फसल

की बिाई तथा किाई क अिसर पर वकया जान

िाला एक पारोपररक दहाती नतय ह

FC19H1003 38

bull इस नतय क माधयम स पोजाबी मवहलाऐो अपनी

परसननता परकि करती ह तथा वगदा क परदशिन क

माधयम स परि िचिसव िाल समाज म मवहलाओो

की दबी हई भािनाओो क परकि करती ह

bull चोवक इस नतय का परि ो क साथ क ई सोबोि नही ो

ह अतः किल मवहलाऐो ही इसम भाग ल सकती

bull हर साल तीज समार ह क दौरान पोजाब म वगदा

नतय वकया जाता ह

तीज भारत क कछ भाग ो म मवहलाओो क दवारा

मनाया जान िाल कई तयौहार ो क वलए एक

वयापक नाम ह

Q63) उततर (a)

सपषटीकरण

- मजम-उल-बहरीन या द समदर ो का सोगम

नामक उललखनीय रचना दारा वशक ह क दवारा

वलखी थी

- भारत क उपराषटर पवत शरी एम िकया नायड न कहा

ह वक राजकमार दारा वशक ह की रचनाएा शाोवत

और सदभाि क बिािा दन क वलए एक तारा सर त

क रप म सामन आ सकती ो ह

- उपराषटर पवत गत ििो क भला वदए गए राजकमार

दारा वशक ह क परदवशित परचवलत करन हत

आय वजत एक परदशिनी का दौरा करन क बाद एक

सभा क सोब वित कर रह थ

- इस परदशिनी का आय जन फर क इस गौवियर

(Francois Gautier) क दवारा lsquoइोवदरा गाोिी नशनल

सिर फॉर द आििसrsquo (The Indira Gandhi

National Centre for the Arts) नई वदलली म

वकया गया था

Q64) उततर (c)

सपषटीकरण

- ग मतशवर परवतमा जन भगिान बाहबली क

समवपित ह

- यह एक एक-चटटानी पतथर की मवति ह

- राषटर पवत राम नाथ क विोद न शरिणबलग ला

(कनाििक) म आय वजत वकय जान िाल भवय

अवभिक समार ह महामसतकावभिक का

उदघािन वकया था

- यह समार ह 12 ििो म एक बार ह ता ह

Q65) उततर (c)

सपषटीकरण

bull पराची घािी पराची नदी क चार ो ओर फली हई थी

bull पराची घािी िीर-िीर विलपत ह गई थी

bull पराची नदी भिनशवर स वनकलती ह

bull यह महानदी की एक सहायक नदी ह और यह

परी खदाि किक तथा जगतवसोहपर वजल ो स

ह कर बहती ह

bull इस नदी क पर कषतर क पराची घािी कहा जाता ह

bull यह नदी बोगाल की खाडी म वगरती ह

परातासतवक साकषय स पता चलता ह वक पराची घािी

सभयता हडपपा और म हनज दाड द न ो की

पिििती ह

Q66) उततर (d)

सपषटीकरण

य समारक छतरपर वजल (मधय परदश) म विोधयाचल

पिित शरोखला म सथथत ह

Q67) उततर (a)

सपषटीकरण

bull थॉिस ऑन पावकसतान नामक पसतक डॉ बी

आर अमबडकर न वलखी थी

bull डॉ बी आर अमबडकर की जयोती क अिसर पर

भारत क राषटर पवत न भारत की इस महान हसती

क शरदाोजवल अवपित की थी

bull डॉ बी आर अमबडकर न 1924 म वडपरथड

कलावसर इोसटीटयि (दवलत िगि सोथथान -

बवहषकत वहतकाररणी सभा) और 1927 म समाज

समता सोघ की थथापना की थी

bull अमबडकर का धयान वशकषा कषतर की ओर भी था

bull उनह ोन वशकषा क वनमन िगो म फलान क वलए

पीपलस एजकशन स साइिी (The Peoples

Education Society) क नाम स महाविदयालय ो क

नििकि और छातरािास ो की थथापना की थी

FC19H1003 39

Q68) उततर (b)

सपषटीकरण

bull महरगि भारतीय उपमहादवीप म एक परवसद

निपािाण बसती ह ज बलवचसतान पराोत

पावकसतान म सथथत ह

bull दचपलली (आोधर परदश) क पास नागलर नदी क

पिी ति ो पर चना पतथर क बलॉक क विशाल

विसतार म एक पिि-ऐवतहावसक रॉक आिि थथल की

ख ज की गई ह

bull इसन 1500-2000 ईसा पिि क दौरान गोिर (आोधर

परदश) म विकवसत निपािाण सभयता पर परकाश

डाला ह

Q69) उततर (c)

सपषटीकरण

bull 12िी ो सदी और 13िी ो सदी म काकाविय िोश का

उदय हआ था

bull ि पहल कलयाण क पवशचमी चालकय ो क सामोत थ

bull परारोभ म उनह ोन िारोगल (तलोगाना) क पास एक

छ ि स कषतर पर शासन वकया था

bull उनह ोन ldquoनायक वयिथथाrdquo की शरआत की थी

वजस बाद म विजयनगर क राय शासक ो न

अपनाया और विकवसत वकया था

Q70) उततर (a)

सपषटीकरण

bull गाोिीजी क अनशन स वमल मावलक ो पर दबाि

पडा था ज अोततः शरवमक ो क ितन म 35 परवतशत

की िसद करन क वलए सहमत हए थ

bull गगल (Google) न अनसया साराभाई वजनह ोन

भारत क शरवमक आोद लन म एक अगरणी भवमका

वनभाई थी की 132िी ो जयोती डडल (Doodle) का

वनमािण करक मनाई

Q71) उततर (d)

सपषटीकरण

भारत स यनसक की मानिता की अमति साोसकवतक

विरासत की परवतवनवि सची म वनमनवलसखत शावमल ह

bull कवडयटटम करल का सोसकत रोगमोच

bull मवडयिि करल का अनषठान रोगमोच और नतय

नाविका

bull िवदक मि जाप की परोपरा

bull राजथथान क कालबवलया ल क गीत और नतय

bull रामलीला रामायण का पारोपररक परदशिन

bull सोकीतिन मवणपर का अनषठान गायन ढ ल िादन

और नतय

bull रममन भारत क गििाल वहमालय का िावमिक

तयौहार और अनषठान रोगमोच

bull जाोदीयाला गर पोजाब क ठठर ो की पीतल और

ताोब क वशलप स वनवमित बतिन ो की पारोपररक कला

bull छाऊ नतय पिी भारतीय राजय ो म जनमी शासतरीय

भारतीय नतय कला

bull लददाख का बौद मि जाप िर ाोस-वहमालयी लददाख

कषतर तथा जमम-कशमीर म पवितर बौद गरोथ ो का पाठ

bull य ग

bull नौर र

bull को भ मला

Q72) उततर (b)

सपषटीकरण

bull भारत क राषटर पवत शरी राम नाथ क विोद न

वकसामा नागालड म हॉनिवबल मह रति और

राजय गठन वदिस समार ह का उदघािन वकया

था

bull हॉनिवबल मह रति का नाम भारतीय हॉनिवबल क

नाम पर पडा ह ज एक विशाल और रोगीन जोगली

पकषी ह

bull यह पकषी नागालड राजय की अविकतर जनजावतय ो

की ल ककथाओो म उसललसखत ह

bull नागालड की परमख मानयता परापत जनजावतयाा ह

अोगामी आओ चखसोग चाोग ककी रगमा और

रवलोग आवद

bull ओोग जारिा और ससिनलीस अोडमान-वनक बार

दवीप समह की जनजावतयाा ह

FC19H1003 40

Q73) उततर (c)

सपषटीकरण

bull दकन म राषटर कि शासन दसिी ो सदी क अोत तक

लगभग 200 ििो तक रहा था

bull राषटर कि शासक अपन िावमिक विचार ो म सवहषण

bull उनह ोन न किल शि िमि और िषणि िमि बसलक

जन िमि क भी सोरकषण वदया था

bull एल रा म वशि क परवसद रॉक कि मोवदर का

वनमािण नौिी ो सदी म राषटर कि राजा कषण परथम न

करिाया था

bull उसका उततराविकारी अम घििि जन था लवकन

उसन अनय िमो क भी सोरकषण परदान वकया था

bull राषटर कि ो न मसलमान वयापाररय ो क बसन की

अनमवत दी थी

bull उनह न अपन अविराजय ो म इसलाम क उपदश दन

की भी अनमवत दी थी

bull अभी हाल ही म पाोडिलागटटा (तलोगाना) क

परागवतहावसक चटटान वचतर ो क कषरण की बिती हई

घिनाएा एक गोभीर वचोता का वििय ह

bull यह परागवतहावसक चटटान क नकसान पहाचा

सकता ह

bull पाोडिलागटटा वनमनवलसखत क वलए जाना जाता ह

- 10000 ईसा पिि स 8000 ईसा पिि क वचवतरत

चटटानी आशरय ो क वलए

- राषटर कि काल क एक 8 िी ो सदी क

वशलालख क वलए और

- 12िी ो सदी क काकविय सामराजय क वभवतत

वचतर ो क वलए

Q74) उततर (b)

सपषटीकरण

bull 1828 म राजा राम म हन रॉय न एक नय िावमिक

समाज बरहम सभा की थथापना की थी वजस बाद

म बरहम समाज क नाम स जाना गया था

bull दिदरनाथ िग र न ततवब विनी सभा की अधयकषता

की थी ज आधयासिक सतय की ख ज म सोलि

थी

bull इसका उददशय वहोद िमि क शद करन का और

एकशवरिाद (एक ईशवर म आथथा) का परचार करना

था

bull नय समाज की थथापना क आिार थ कारण

(तकि ) क द सतमभ तथा िद और उपवनिद

bull अभी हाल ही म सािारण बरहम समाज का कछ

काननी मदद ो क लकर पवशचम बोगाल सरकार क

साथ काननी वििाद चल रहा ह

Q75) उततर (c)

सपषटीकरण

bull भारत म वचशती वसलवसल की थथापना खवाजा

म इनददीन वचशती क दवारा की गयी थी

bull ि 1192 ईसवी क आसपास भारत आय थ

bull वचशतीय ो क बारहिी ो शताबदी क उततरािि म भारत

म आन िाल सफीय ो क समह ो म सबस

परभािशाली माना जाता ह

bull उनह ोन थथानीय िातािरण क साथ सफलतापििक

अनकलन वकया और उनह ोन भारतीय भसकत

परोपराओो क कई पहलओो क अपनाया

bull अजमर म सफी अपरकि खवाजा म इनददीन वचशती

की ऐवतहावसक दरगाह क एक नया रप दन की

तयारी की जा रही ह

bull इस 13िी ो शताबदी की दरगाह क ldquoसवचछ

आइकॉवनक थथल ोrdquo (Swacch Iconic Places) म

शावमल वकया गया ह ज परवतवषठत विरासत

आधयासिक और साोसकवतक थथान ो पर क वदरत

य जना ह

FC19H1003 41

ANSWERS amp EXPLANATION OF

NCERT History Class VI-X + Current Affairs

(FC19E1003)

Q1) Answer c

Explanation

Rigveda consists of more than a

thousand hymns dedicated to gods and

goddesses These hymns were

composed by sages and learnt by men

however a few were composed by

women like Apala Ghosa Lopamudra

Maitreyi and Gargi

Rigveda consists of many hymns in the

form of dialogues We get an example of

a dialogue between a sage named

Vishwamitra and two rivers (Beas and

Sutlej) that were worshipped as

goddesses This suggests that he

belonged to the Vedic period

Q2) Answer b

Explanation

Traces of ash have been found from

Kurnool Caves suggesting that people

were familiar with the use of fire

It is situated in Andhra Pradesh

Q3) Answer c

Explanation

Burzahom is a prehistoric site in

present day Kashmir where people built

pit houses which were dug into the

ground with steps leading into them

These may have provided shelter in cold

weather

Q4) Answer c

Explanation

Epigraphy is defined as the study of

inscriptions

Manuscriptology is the study of history

and literature through the use of hand

written documents

Palaeography refers to the study of

ancient writing systems and the

deciphering and dating of historical

manuscripts

Numismatics refers to the study of

coins

Q5) Answer a

Explanation

Charaka Samhita was written by

Charaka and is an important book on

Ayurveda and medicine

He was a practitioner of the traditional

system of Indian medicine known as

Ayurveda

Charaka is thought to have flourished

sometime between the 2nd century BCE

and the 2nd century CE

Q6) Answer b

Explanation

Bhaga refers to the tax on crops which

was fixed at 16th of the production

Kammakaras is the term used for the

landless agricultural labour class

Ashvamedha also known as horse

sacrifice is a ritual where a horse is let

loose to wander freely and it was

guarded by the rajarsquos men

Q7) Answer (d)

Explanation

In the Rigvedic period horses were

yoked to chariots that were used in

battles fought to capture land cattle

etc This suggests that the use of horse

chariots began much before the period

of Mahajanapadas

The battles were fought in the Rigvedic

period for cattlersquos lands water an even

to capture people Most men took part

in these wars however there was no

regular army but there were assemblies

where people met and discussed

matters of war Regular armies became

a feature in the Mjahajanapada period

including vast armies of foot soldiers

chariots and elephants

RAUSIAS-FC19E1003 42

Q8) Answer (a)

Explanation

Buddha belonged to the Sakya clan and

passed away at Kusinara

Buddha taught in Prakrit which was the

common language of people

Q9) Answer c

Explanation

There were six schools of philosophy in

ancient India These are known as

Vaishesika Nyaya Samkhya Yoga

Purva Mimansa and Vedanata or Uttara

Mimansa They were founded by sages

Kanada Gautama Kapila Patanjali

Jamini and Vyasa respectively

Q10) Answer b

Explanation

The teachings of Mahavira were

compiled at Valabhi in 6th century AD

Q11) Answer (c)

Explanation

Chanakya is traditionally identified as

Kautilya or Vishnugupta who authored

the ancient Indian political treatise the

Arthashastra

Q12) Answer d

The national emblem of India is an

adaptation of the Lion Capital atop the

Ashoka Pillar of Sarnath Uttar Pradesh

and is combined with the National

Motto Satyameva Jayate

The Rampurva Bull gets the name from

the site of its discovery Rampurva in

Bihar

It is noted for its delicately sculpted

model demonstrating superior

representation of soft flesh sensitive

nostrils alert ears and strong legs It is

a mixture of Indian and Persian

elements

Sankissa is situated in Uttar Pradesh

India

Q13) Ans(a)

Kunwar Singh was a notable leader during the Revolt of 1857 He belonged

to a royal house of Jagdispur Bihar

Q14) Answer b

Explanation

The term Vellalar was used for large

landowners

Q15) Answer c

Explanation

Arikamedu was a coastal settlement

where ships unloaded goods from

distant lands Finds here include a

massive brick warehouse pottery

including amphorae and Arretine ware

Roman lamps glassware and gems have

also been found at the site

Q16) Answer a

Explanation

Muvendar is a Tamil word mentioned in

Sangam poems meaning three chiefs

used for the heads of three ruling

families the Cholas Cheras and

Pandyas

Q17) Ans (c)

Several tribal or kin-based assemblies

such as the Sabha Vidatha and gana

are mentioned in the Rig-veda The

Sabha and the samiti mattered a great

deal in early Vedic times so much so

that the chiefs or the kings showed an

eagerness to win their support

Q18) Ans (a)

Jainism recognised the existence of the

gods but placed them lower than the

jina and did not condemn the varna

system as Buddhism did

Q19) Answer (d)

Explanation

Cholas and Pandyas had developed

powerful coastal cities The most

important city of Cholas was Puhar or

Kaveripattinam and Madurai was the

capital of Pandyas

Q20) Answer b

Explanation

Buddhacharita is the biography of

Buddha and was written by

RAUSIAS-FC19E1003 43

Ashvaghosha

Q21) Answer (a)

Explanation

Tamil poet Appar was a Shiva devotee

So he was a Nayanar saint

Q22) Answer d

Explanation

Samudragupta was a prominent Gupta

ruler whose coins depict him playing a

veena indicating his love for music We

get important historic information from

his Allahabad Prashasti which was

composed by his court poet Harisena

Q23) Answer (b)

Explanation

Vikrama Samvat was founded by

Chandragupta II in the 58 BC as a

mark of victory over the Shakas and

assumed the title of Vikramaditya

Banabhatta wrote Harshavardhanarsquos

biography the Harshacharita in

Sanskrit

Q24) Answer c

Explanation

Sandhi-vigrahika was the minister of

war and peace

Sarthavaha was the leader of the

merchant caravans

Q25) Answer a

Explanation

Xuan Zang (Hsuan-tsang) was a

Chinese traveller who came during the

reign of Harshavardhana

In the decade that began in 630 AD

Xuan Zang came to India through

Kashmir after visiting Central Asia Iran

and Afghanistan

He travelled from north to east and lived

in Bihar for a couple of years

At Nalanda University Xuan Zang

interacted with students and scholars

mastered local languages and

discovered Buddhist stupas

Q26) Answer c

Explanation

Pradakshina patha is a circular path

laid around a stupa in Buddhist

architecture While the rest are a part of

temple architecture

Q27) Answer d

Explanation

All the above-mentioned temples have

an elaborate use of bricks (baked

bricks) along with stone

Q28) Ans (c)

Muhammad Quli Qutab was the Sultan

of Golconda He was a contemporary of

Akbar was very fond of literature and

architecture

The Sultan was a great poet and he

wrote in Dakhini Urdu Persian and

Telgu and has left an extensive diwan or

collection

Recently the Archaeological Survey of

India (ASI) will be using Ground

Penetrating Radar (GPR) to map the

contours of the area around the Bagh-e-

Naya Qila excavated garden inside the

Golconda Fort in Telangana

Q29) Answer a

Explanation

Silappadikaram is a famous Tamil epic

which was written by Ilango around

1800 years ago It is a story of a

merchant named Kovalan who fell in

love with a courtesan named Madhavi

Manimekalai tells the story of the

daughter of Kovalan and Madhavi

Q30) Answer (a)

Explanation

Charaka is the author of Charaka

Samhita which is an important work of

Ayurveda and medicines

Brahmaguptarsquos fame rests mostly on his

Brahma-sphuta-siddhanta which was

an astronomical work It was translated

into Arabic in Baghdad and had a major

impact on Islamic mathematics and

astronomy

Late in his life Brahmagupta wrote

Khandakhadyaka which was an

RAUSIAS-FC19E1003 44

astronomical handbook that employed

Aryabhatarsquos system of starting each day

at midnight

Q31) Answer (c)

Explanation

Amir Khusrau was a famous sufi

musician poet and scholar In 1318 he

noted that there was different language

in every region of this land (Hindustan)

Lahori Kashmiri Dvarsamudri (in

Southern Karnataka) Telangana (in

Andhra Pradesh) Gujari (in Gujarat)

Marsquobari (in Tamil Nadu) Awadhi (in

eastern Uttar Pradesh) and Hindawai (in

the area around in Delhi) etc He went

to explain that Sanskrit did not belong

to any region and that only brahmans

knew it

Q32) Answer c

Explanation

Hiranyagarbha refers to the golden

womb When this ritual was performed

with the help of Brahmanas it was

thought to lead to the rebirth of the

sacrificer as a Khastriya

Q33) Answer d

Explanation

Kadamai refers to a tax on land

revenue

Gwalior Prashasti describes the exploits

of Nagabhata who was a Pratihara king

Q34) Answer b

Explanation

Rajatarangini is a Sanskrit text written

by Kalhana in the 12th century

It was historical chronicle of early India

It is justifiably considered to be the best

and most authentic work of its kind

It covers the entire span of history in

the Kashmir region from the earliest

times to the date of its composition

Q35) Answer c

Explanation

ldquoUrrdquo was the general assembly of the

village ldquoUrrdquo consisted of all the

taxpaying residents of an ordinary

village

Q36) Answer (a)

Explanation

Tarikh was a form of history writing in

the Delhi Sultanate The authors of

tawarikhs were learned men which

included secretaries administrators etc

Q37 Answer (a)

Explanation

Alauddin chose to pay his soldiers salaries in cash rather than iqtas The soldiers would buy their supplies from merchants in Delhi and it was thus feared that merchants would raise their prices To stop this Alauddin controlled the prices of goods in Delhi Prices were carefully surveyed by officers and merchants who did not sell at the prescribed rates were punished

Q38) Answer (d)

Explanation

Delhi first became the capital of a

kingdom under the Tomara Rajputs

who were defeated in the middle of the

twelfth century by the Chauhans (also

referred to as Chahamanas) of Ajmer

It was under the Tomaras and

Chauhans that Delhi became an

important commercial centre Many rich

Jaina merchants lived in the city and

constructed several temples Coins

minted here called dehliwal had a wide

circulation

Q39) Answer (c)

Explanation

Moth ki Masjid was built in the reign of

Sikandar Lodi by his minister

Begumpuri mosque built in the reign of

Muhammad Tughluq was the main

mosque of Jahanpanah the ldquoSanctuary

of the Worldrdquo and his new capital in

Delhi

Quwwat al ndash Islam mosque was

enlarged by Iltutmish and Alauddin

Khalji The minar was built by three

Sultansndash Qutbuddin Aybak Iltutmish

and Firuz Shah Tughluq

RAUSIAS-FC19E1003 45

Q40) Answer (c)

Explanation

Under the Mughals mansabdar was

referred to an individual who held a

mansab ie rank and he received his

salary as revenue assignments called

jagirs

Q41) Ans (b)

The Quit India Movement was a

spontaneous revolt of people against

British rule

The All India Congress Committee met

at Bombay on 8 August 1942 It passed

the famous resolution Quit India and

proposed the starting of a non-violent

mass struggle under Gandhis

leadership to achieve this aim But on

the very next day Gandhi and other

eminent leaders of the Congress were

arrested The Congress was once again

declared illegal

Q42) Ans (c)

The Simon Commission refers to a

group of seven MPs from the United

Kingdom constituted to suggest

constitutional reforms for British India

The Commission consisted of only

British members headed by one of the

senior British politicians Sir John

Simon

So the people of India agitated against

the arrival of Simon Commission

Q43) Ans (a)

He was widely known for his

unfavourable opinion of the economic

consequences of the British rule in

India

In his many writings and speeches and

especially in Poverty and Un-British

Rule in India Naoroji argued that India

was too highly taxed and that its wealth

was being drained away to England

He did not interpret the ancient Indian

texts and restored the self-confidence of

Indians And also he did not stress the

need for eradication of all the social

evils before anything else

Q44) Ans (c)

In August 1932 Prime Minister

MacDonald announced his Communal

Award Great Britainrsquos unilateral

attempt to resolve the various conflicts

among Indiarsquos many communal

interests

The award which was later

incorporated into the act of 1935

expanded the separate-electorate

formula reserved for Muslims to other

minorities including Sikhs Indian

Christians Anglo-Indians Europeans

distinct regional groups Gandhi

undertook a ldquofast unto deathrdquo against

that offer which he viewed as a

nefarious British plot to divide the

Indian society

Q45) Ans (b)

In British India apart from existing

imports and exports there was also a

particular amount of money which

colonial India contributed towards

administration maintenance of the

army war expenses pensions to retired

officers and other expenses accrued by

Britain towards maintenance of her

colony These were known as Home

charges and were paid for almost

entirely by India

The Home charges was made of

following components-

- Interest payable on Indian debt

- Dividend to shareholders of East

India Company

- Funds used to support the India

Office in London

- Funds used to pay salaries and

pensions of British personnel

engaged in India

- Interest on the railways

- Civil and military charges

- Store purchases in England

Q46) Ans (b)

The Lahore session of the Indian

National Congress was held in 1929

under the Presidentship of Jawaharlal

Nehru

The Lahore session of the Indian

National Congress witnessed significant

RAUSIAS-FC19E1003 46

developments in the Indian national

movement

- First the election of Jawaharlal

Nehru to the post of Presidentship of

the Congress was a clear indication

of the growing strength of the

Leftists in the Congress

- Secondly it was in this session that

the Congress for the first time raised

the demand for complete

independence Such demand was

not raised from the Congress

platform earlier

Q47) Ans (b)

It did not provide for separate

electorates for any community or

weightage for minorities However it did

allow for the reservation of minority

seats in provinces having minorities of

at least ten per cent but this was to be

in strict proportion to the size of the

community

There was no provision for complete

Independence for India

Q48) Ans (c)

The religion of early Vedic Aryans was

primarily of worship of nature and

Yajnas

The early Aryan religion was kind of

nature worship Actually the forces

around them which they could not

control or understand were invested

with divinity and were personified as

male or female gods And they

performed some Yajnas also

Q49) Ans (b)

The roads and river-routes were not

immune from robbery It is notable that

Yuan Chwang (Hiuen Tsang) was

robbed of his belongings during

Harshvardanarsquos period

Q50) Ans (c)

Q51) Ans (b)

Purandara Dasa was a saint and great

devotee of Lord Krishna

There is much speculation about where

Purandara Dasa regarded as the

Pitamaha of Carnatic music was born

Recently an expert committee

constituted by the Kannada University

Hampi has come to the conclusion that

Kshemapura Shivamogga district

Karnataka is the birth place of

Purandara Dasa

Q52) Ans (c)

Sri Tyagaraja Sri Shyama Shastry and Sri Muthuswami Dikshitar are considered the trinity of Carnatic music and with them came the golden age in Carnatic music in the 18th-19th

century

Q53) Ans d)

Recently a rare sarcophagus (stone

coffin) which is 2000 years old from the

Iron AgendashMegalithic era was discovered

from a rock-cut cave at Viyur village of

Kollam near Koyilandy in Kozhikode

district Kerala

The coffin containing bone fragments

was found during an excavation ldquoSo

far such a rare finding has been

discovered only from two sites

in Kerala Both these sarcophagi were

recovered from Megalithic sites at

Chevayur and Atholi also in Kozhikode

district

Q54) Ans a)

The megalithic culture in South India was a full-fledged Iron Age culture

Q55) Ans d)

The Cholas Pandyas and Keralaputras

(Cheras) mentioned in Ashokan

inscriptions were probably in the late

megalithic phase of material culture

Q56) Ans d)

Q57) Ans (b)

Raj Kumar Shukla followed Gandhiji all

over the country to persuade him to

come to Champaran to investigate the

problem associated with tinkathia

system

RAUSIAS-FC19E1003 47

Brij Kishore Rajendra Prasad Mahadev

Desai and Narhari Parikh accompanied

Gandhi ji during the Champaran

Satyagraha

Q58) Ans (b)

The Satvahanas started the practice of granting tax-free villages to brahmanas and Buddhist monks

Q59) Ans c)

The objectives of the Programme are

listed as under

- Developing basic tourism

infrastructure

- Promoting cultural and heritage

value of the country to generate

livelihoods in the identified regions

- Enhancing the tourist attractiveness

in a sustainable manner by

developing world-class

infrastructure at the heritage

monument sites

- Creating employment through active

involvement of local communities

- Harnessing tourism potential for its

effects on employment generation

and economic development

- Developing sustainable tourism

infrastructure and ensuring proper

Operations and maintenance

therein

Q60) Ans (b)

The Tribal Cooperative Marketing

Development Federation of India

(TRIFED) came into existence in 1987

It is a national-level apex organization

functioning under the administrative

control of Ministry of Tribal Affairs

Govt of India

TRIFED has its registered and Head

Office located in New Delhi

Q61) Ans (c)

Premchandrsquos novels include

Premashram Rangabhumi Ghaban

Karmabhumi and Godan

Gora is a novel written by Rabindranath

Tagore

138th birth anniversary of Munshi

Premchand was celebrated across the

country

Q62) Ans (b)

Giddha is a traditional pastoral dance

performed by the women of the Punjab

India and Pakistan at festival times

and at the sowing and reaping of the

harvest

By this dance the Punjabi women

reveal their joy expel their suppressed

feelings in a male dominated society

through the performance of Giddha

Since this dance has nothing to do with

men only women can participate in it

During the Teej celebrations Giddha

dance is celebrated in Punjab every

year Teej is a generic name for a

number of festivals that are celebrated

by women in some parts of India

Q63) Ans (a)

Dara Shukoh wrote the remarkable

work called ldquoMajma-ul-Bahrainrdquo or the

ldquoThe confluence of two seasrdquo

The Vice President of India Shri M

Venkaiah Naidu has said that Prince

Dara Shukohrsquos writings can come as a

refreshing source for infusing peace and

harmony He was addressing the

gathering after visiting the exhibition

that showcases the forgotten Prince of

yesteryears Dara Shukoh organized by

Mr Francois Gautier at Indira Gandhi

National Centre for the Arts in New

Delhi

Q64) Ans (c)

The statue Gommateshwara is

dedicated to the Jain God Bahubali

It is a monolithic statue

President Ram Nath Kovind

inaugurated the grand anointing

ceremony mdash Mahamastakabhisheka mdash

held once in 12 years at

Shravanabelagola (Karnataka)

Q65) Ans (c)

Prachi Valley had come up around the

Prachi river Prachi Valley gradually

disappeared

RAUSIAS-FC19E1003 48

The Prachi river originates from

Bhubaneswar

It is a tributary of the Mahanadi and

flows through the districts of Puri

Khurda Cuttack and Jagatsinghpur

and the entire region of the river is

termed as the Prachi Valley

It falls into the Bay of Bengal

Archaeological evidence shows that the

Prachi Valley Civilisation predates both

Harappa and Mohenjo-Daro

The Prachi river originates from

Bhubaneswar

Q66) Ans (d)

These monuments are located in

Chhatarpur district Madhya Pradesh

within Vindhya mountain range

Q67) Ans (a)

The book lsquoThoughts on Pakistanrsquo was

written by Dr BR Ambedkar

On the occasion of the birth anniversary

of Dr BR Ambedkar the president of

India pays homage to this icon of India

In 1924 he founded the Depressed

Classes Institute (Bahishkrit Hitkarini

Sabha) and in 1927 the Samaj Samata

Sangh

Another area of attention for Ambedkar

was education For its spread among

the low classes he set up a network of

colleges by the name of Peoples

Education Society and founded hostels

Q68) Ans(b)

Mehrgarh is a famous Neolithic

settlement in the Indian subcontinent

which is situated in Baluchistan

province Pakistan

A pre-historic rock art site is discovered

in the vast expanse of limestone blocks

on the eastern banks of Naguleru river

near Dachepalli (Andhra Pradesh) It

has thrown light on the Neolithic

civilisation that flourished in Guntur

(Andhra Pradesh) during 1500-2000

BC

Q69) Ans (c)

The 12th and the 13th centuries saw

the emergence of the Kakatiyas They

were at first the feudatories of the

Western Chalukyas of Kalyana Initially

they ruled over a small territory near

Warangal (Telangana)

They introduced Nayakships which was

later adopted and developed by the

Rayas of Vijayanagara

Q70) Ans (a)

The fast had effect of putting pressure

on mill owners who finally agreed to

give the workers a 35 per cent increase

in wages

Google celebrated with a doodle the

132nd birth anniversary of Anasuya

Sarabhai who played a pioneering role

in Indiarsquos labour movement

Q71) Ans (d)

The UNESCOrsquos list of the representative

list of the intangible cultural heritage of

humanity from India are

- Koodiyattam Sanskrit Theatre of

Kerala

- Mudiyettu ritual theatre and dance

drama of Kerala

- Tradition of Vedic Chanting

- Kalbelia folk songs and dances of

Rajasthan

- Ramlila Traditional Performance of

the Ramayana

- Sankirtana ritual singing

drumming and dancing of Manipur

- Ramman religious festival and

ritual theatre of the Garhwal

Himalayas India

- Traditional brass and copper craft of

utensil making among the Thatheras

of Jandiala Guru Punjab India

- Chhau dance classical Indian dance

originated in the eastern Indian

states

- Buddhist chanting of Ladakh

recitation of sacred Buddhist texts

in the trans-Himalayan Ladakh

region Jammu and Kashmir India

- Yoga

- Nouroz

- Kumbh Mela

RAUSIAS-FC19E1003 49

Q72) Ans(b)

The President of India Shri Ram Nath Kovind inaugurated the Hornbill Festival and State Formation Day celebrations of Nagaland in Kisama

The festival is named after the Indian hornbill the large and colourful forest bird which is displayed in the folklore of most of the states tribes

The major recognized tribes of Nagaland are Angami Ao Chakhesang Chang

Kuki Rengma and Zeling etc

Onge Jarawa and Sentinelese are the

tribes of Andman amp Nicobar Islands

Q73) Ans (c)

The Rashtrakutas rule in the Deccan lasted for almost two hundred years till the end of the tenth century The Rashtrakutas rulers were tolerant in their religious views and patronized not only Shaivism and Vaishnavism but

Jainism as well

The famous rock-cut temple of Shiva at Ellora was built by one of the Rashtrakutas kings Krishna I in the ninth century His successor Amoghavarsha was a Jain but he also

patronized other faiths

The Rashtrakutas allowed Muslims traders to settle and permitted Islam to

be preached in their dominions

Recently increasing defacement at the prehistoric rock paintings of Pandavulagutta Telangana has created a cause for grave concern It can spoil

the prehistoric rock

Pandavulagutta is home to

- Painted rock shelters dating to

10000 BC-8000 BC

- An 8th century inscription of the

Rashtrakuta period and

- Painted frescoes from the 12th century Kakatiya empire

Q74) Ans (b)

In 1828 Raja Ram Mohan Roy founded a new religious society the Brahma Sabha later known as the Brahmo

Samaj

Debendranath Tagore headed the Tattvabodhini Sabha which was

engaged in search of spiritual truth

Its purpose was to purify Hinduism and to preach monotheism or belief in one God

The new society was to be based on the twin pillars of reason and the Vedas and

Upanishads

Recently Sadharan Brahmo Samaj (SBS) has entered into a legal battle with the West Bengal government due

to some legal issue

Q75) Ans (c)

The Chishti order was established in India by Khwaja Moinuddin Chishti who came to India around 1192 The Chishtirsquos are considered to be the most influential of the groups of Sufis who migrated to India in the late twelfth century They adapted successfully to the local environment and adopted several features of Indian devotional

traditions

The historical dargah of Sufi mystic Khwaja Moinuddin Chishti in Ajmer is all set to get a facelift This 13 th century dargah has been included among the Swachh Iconic Places a clean-up initiative focused on iconic

heritage spiritual and cultural places

Page 3: GENERAL STUDIES (PAPER I) · Test is part of Rau’s IAS Test series for Preliminary Exam 2019 FOUNDATION + CURRENT AFFAIRS GENERAL STUDIES (PAPER –I) FOUNDATION TEST –III TOPIC:

RAUSIAS-FC19E1003 3

Q1) Which of the following statements

isare correct

1 Some verses in Rigveda were

composed by women

2 Vishwamitra was an important

sage in the Vedic period

Select the correct answer using the code

given below

(a) 1 only

(b) 2 only

(c) Both 1 and 2

(d) Neither 1 nor 2

Q2) Which of the following sites yields traces

of the use of fire

(a) Bhimbetka Caves

(b) Kurnool Caves

(c) Elephanta Caves

(d) Kanheri Caves

Q3) Which of the following sites has

evidences of underground pit houses

(a) Chirand

(b) Hallur

(c) Burzahom

(d) Mahagara

Q4) The study of ancient writing systems

and the deciphering and dating of

historical manuscripts is known as-

(a) Epigraphy

(b) Manuscriptology

(c) Palaeography

(d) Numismatics

Q5) Consider the following pairs

1 Iron implements Megalithic

burials

2 Charaka Samhita Book on

Mathematics

Which of the pairs given above isare

correct

(a) 1 only

(b) 2 only

(c) Both 1 and 2

(d) Neither 1 nor 2

Q6) Consider the following pairs

1 Bhaga Tax on land

2 Kammakaras Landless

Agricultural Labourers

3 Ashvamedha A horse is let loose

to wander into the territories of

other rulers

Which of the pairs given above isare

correct

(a) 1 and 2

(b) 2 and 3

(c) 1 and 3

(d) 1 2 and 3

Q7) Consider the following statements

1 The use of Horse drawn chariots

began during the Mahajanapada

period

2 Regular standing armies were

introduced by Rigvedic rulers

Which of the statements given above

isare correct

(a) 1 only

(b) 2 only

(c) Both 1 and 2

(d) Neither 1 nor 2

RAUSIAS-FC19E1003 4

Q8) बदध क जीिि क सोदभण म निमननिखित कथि ो पर

निचार कीनजए

1 बदध ि िारारसी क पास सारिाथ म अपिा

पहिा उपदश नदया था

2 ि निचछिी कि स सोबोनरत थ और कशीिारा म

उिका निरि हआ था

उपयणकत कथि ो म स कौि-सास सही हह

(a) किि 1

(b) किि 2

(c) 1 और 2 द ि ो

(d) ि त 1 ि ही 2

Q9) निमननिखित यग ो पर निचार कीनजए

1 समखया कनपि

2 िशनिक जानमिी

3 नयाय गौतम

उपयणकत यग ो म स कौि-स सही समनित ह

(a) किि 1 और 2

(b) किि 2 और 3

(c) किि 1 और 3

(d) 1 2 और 3

Q10) निमननिखित कथि ो पर निचार कीनजए

1 महािीर की नशकषाऐो िशािी म सोकनित की गई

थी ो

2 नििय नपटक बौदध सोघ क बार म ह

उपयणकत कथि ो म स कौि-सास सही हह

(a) किि 1

(b) किि 2

(c) 1 और 2 द ि ो

(d) ि त 1 ि ही 2

Q11) निमननिखित कथि ो पर निचार कीनजए

1 अथणशासतर का सोकिि निषणगपत ि नकया था

2 मगसथिीस ज चनदरगपत मौयण क दरबार म

आया था सलयकस का राजदत था

उपयणकत कथि ो म स कौि-सास सही हह

(a) किि 1

(b) किि 2

(c) 1 और 2 द ि ो

(d) ि त 1 ि ही 2

Q12) निमननिखित यग ो पर निचार कीनजए

सतभ कपिटल सथान

1 नसोह कनपटि सारिाथ

2 बि कनपटि रामपिण

3 हाथी कनपटि सोनकससा

उपयणकत यग ो म स कौि-सास सही समनित हह

(a) किि 1 और 2

(b) किि 3

(c) किि 1 और 3

(d) 1 2 और 3

Q13) 1857 क नसपाही निदर ह क सोदभण म निमननिखित

कथि ो म स कौि-सास सही हह

1 अहमदललाह शाह फजाबाद क मौििी ि

अोगरज ो क निरदध ि ग ो क एकनतरत नकया

2 किर नसोह ज एक महाि य दधा थ उततर परदश

स सोबोनरत थ

िीच नदए गए कट का परय ग कर सही उततर चनिए

(a) किि 1

(b) किि 2

(c) 1 और 2 द ि ो

(d) ि त 1 ि ही 2

RAUSIAS-FC19E1003 5

Q8) In the context of the life of Buddha

consider the following statements

1 Buddha gave his first sermon at

Sarnath near Varanasi

2 He belonged to the Licchavi clan

and passed away at Kusinara

Which of the above statements isare

correct

(a) 1 only

(b) 2 only

(c) Both 1 and 2

(d) Neither 1 nor 2

Q9) Consider the following pairs

1 Samkhya Kapila

2 Vaisheshika Jamini

3 Nyaya Gautama

Which of the pairs given above isare

correctly matched

(a) 1 and 2 only

(b) 2 and 3 only

(c) 1 and 3 only

(d) 1 2 and 3

Q10) Consider the following statements

1 The teachings of Mahavira were

compiled at Vaishali

2 Vinaya Pitaka talks about the

Buddhist Sangha

Which of the statements given above

isare correct

(a) 1 only

(b) 2 only

(c) Both 1 and 2

(d) Neither 1and 2

Q11) Consider the following statements

1 Arthashastra was compiled by

Vishnugupta

2 Magasthenes who came to the

court of Chandragupta Maurya

was the ambassador of Seleucus

Which of the statements given above

isare correct

(a) 1 only

(b) 2 only

(c) Both 1 and 2

(d) Neither 1 nor 2

Q12) Consider the following pairs

Pillar Capital Place

1 Lion Capital Sarnath

2 Bull Capital Rampurva

3 Elephant Capital Sankissa

Which of the pairs given above isare

matched correctly

(a) 1 and 2

(b) 3 only

(c) 1 and 3

(d) 1 2 and 3

Q13) Consider the following statements

regarding the Revolt of 1857

1 Ahmadullah Shah a maulvi from

Faizabad caught the imagination

of the people and raised a huge

force of supporters against the

Britishers

2 Kunwar Singh was a great fighter

from Uttar Pradesh

Which of the statements given above

isare correct

(a) 1 only

(b) 2 only

(c) Both 1 and 2

(d) Neither 1 nor 2

RAUSIAS-FC19E1003 6

Q14) निमननिखित यग ो पर निचार कीनजए

1 िललािर सनिक

2 आनदमाई दास (गिाम)

3 कदनसयर भनमहीि मजदर

उपयणकत यग ो म स कौि-स सही समनित ह

(a) किि 1 और 2

(b) किि 2 और 3

(c) किि 1 और 3

(d) 1 2 और 3

Q15) निमननिखित सथि ो म स उस सथि की पहचाि कीनजए

ज एक परनसदध तटीय बसती थी और जहा एरटाइि

(Arretine) मदभाोड पाया गया था

(a) महाबिीपरम

(b) तामरनिखपत

(c) अररकमड

(d) अतरोजीिरा

Q16) सोगम सानहतय क सोदभण म निमननिखित कथि ो पर

निचार कीनजए

1 मदरई म सोगमसभाए आय नजत की जाती ो

थी ो

2 मिनदर शबद का परय ग सोगम कनिताओो म

नकया जाता था नजसका अथण ह द राजाओो

का एक समह

उपयणकत कथि ो म स कौि-सास सही हह

(a) किि 1

(b) किि 2

(c) 1 और 2 द ि ो

(d) ि त 1 ि ही 2

Q17) निमननिखित कथि ो पर निचार कीनजए

1 आरखिक िनदक काि म मनहिाऐो सभाओो

और निदाथाओो म भाग िती थी ो

2 आरखिक िनदक काि म िनशषठ ज एक

पजारी ह ता था की एक महतवपरण भनमका

ह ती थी

उपयणकत कथि ो म स कौि-सास सही हह

(a) किि 1

(b) किि 2

(c) 1 और 2 द ि ो

(d) ि त 1 ि ही 2

Q18) निमननिखित कथि ो पर निचार कीनजए

1 किाणटक म जि रमण क निसतार का शरय

चनदरगपत मौयण क जाता ह

2 जि रमण ि ईशवर क अखसततव क मानयता दी ह

तथा िरण पररािी की भरतणिा की ह

उपयणकत कथि ो म स कौि-सास सही हह

(a) किि 1

(b) किि 2

(c) 1 और 2 द ि ो

(d) ि त 1 ि ही 2

Q19) निमननिखित यग ो पर निचार कीनजए

1 पहार पाणडय

2 मदरई च ि

उपयणकत यग ो म स कौि-सास सही समनित हह

(a) किि 1

(b) किि 2

(c) 1 और 2 द ि ो

(d) ि त 1 ि ही 2

RAUSIAS-FC19E1003 7

Q14) Consider the following pairs

1 Vellalar Soldier

2 Adimai Slaves

3 Kadaisiyar Landless labourers

Which of the pairs given above isare

correctly matched

(a) 1 and 2 only

(b) 2 and 3 only

(c) 1 and 3 only

(d) 1 2 and 3

Q15) Identify the site which was a famous

coastal settlement from which Arretine

ware has been found

(a) Mahabalipuram

(b) Tamralipti

(c) Arikamedu

(d) Atranjikhera

Q16) With respect to the Sangam literature

consider the following statements

1 The Sangamsassemblies were

held at Madurai

2 The term muvendar was used in

sangam poems which mean a

group of two kings

Which of the statements given above

isare correct

(a) 1 only

(b) 2 only

(c) Both 1 and 2

(d) Neither 1 nor 2

Q17) Consider the following statements

1 During Early Vedic Period women

attended the Sabha and Vidatha

2 The priest ndash Vasishtha played

important role in Early Vedic

Period

Which of the statements given above

isare correct

(a) 1 only

(b) 2 only

(c) Both 1 and 2

(d) Neither 1 nor 2

Q18) Consider the following statements

1 The spread of Jainism in

Karnataka is attributed to

Chandragupta Maurya

2 Jainism recognised the existence

of the gods and condemns the

Varna System

Which of the statements given above

isare correct

(a) 1 only

(b) 2 only

(c) Both 1 and 2

(d) Neither 1 nor 2

Q19) Consider the following pairs

1 Puhar Pandyas

2 Madurai Cholas

Which of the pairs given above isare

correct

(a) 1 only

(b) 2 only

(c) Both 1 and 2

(d) Neither 1 nor 2

RAUSIAS-FC19E1003 8

Q20) निमननिखित कथि ो म स कौि-सास सही हह

1 बदधचररत का ििक िागसि ह

2 ब नरसतव की पजा महायाि बौदधमत का एक

महतवपरण भाग थी

िीच नदए गए कट का परय ग कर सही उततर चनिए

(a) किि 1

(b) किि 2

(c) 1 और 2 द ि ो

(d) ि त 1 ि ही 2

Q21) निमननिखित कथि ो म स कौि-सास सही हह

1 भखकत क निचार क भागित गीता म सपषट

नकया गया ह

2 तनमि कनि अपपर एक अििर सोत थ

िीच नदए गए कट का परय ग कर सही उततर चनिएः

(a) किि 1

(b) किि 2

(c) 1 और 2 द ि ो

(d) ि त 1 ि ही 2

Q22) निमननिखित म स नकस शासक ि िीरा बजात हए

अपिी छनि नसक ो पर अोनकत करिाई थी

(a) नमिाोदर

(b) चनदरगपत मौयण

(c) गौतमीपतर सतकरी

(d) समदरगपत

Q23) निमननिखित कथि ो म स कौि-सास सही हह

1 सवतोतर भारत ि निकरम सोित क राषटर ीय

किनडर क रप म अपिाया और यह 68 ईसा

पिण म आरि हआ था

2 बािभटट हिणिरणि क दरबार का एक कनि था

िीच नदए गए कट का परय ग कर सही उततर चनिएः

(a) किि 1

(b) किि 2

(c) 1 और 2 द ि ो

(d) ि त 1 ि ही 2

Q24) निमननिखित यग ो पर निचार कीनजएः

1 सोनर-निगरानहका वयापार मोतरी

2 परथम-कनिका परमि नशलपकार

3 साथणिाह परमि बकर

उपयणकत यग ो म स कौि-सास सही समनित हह

(a) किि 1

(b) किि 1 और 3

(c) किि 2

(d) किि 2 और 3

Q25) निमननिखित कथि ो म स कौि-सास सही हह

1 lsquoएह ि नशिाििrsquo पिकनशि नदवतीय स

सोबोनरत ह और रनिकनत क दवारा इसकी रचिा

की गई थी

2 जआि झाोग समदरगपत नदवतीय क शासिकाि

म भारत आया था

िीच नदए गए कट का परय ग कर सही उततर चनिएः

(a) किि 1

(b) किि 2

(c) 1 और 2 द ि ो

(d) ि त 1 ि ही 2

RAUSIAS-FC19E1003 9

Q20) Which of the following statements

isare correct

1 Buddhacharita is authored by

Nagasena

2 The worship of Bodhisattvas was

an important part of Mahayana

Buddhsim

Select the correct answer using the code

given below

(a) 1 only

(b) 2 only

(c) Both 1 and 2

(d) Neither 1 nor 2

Q21) Which of the following statements

isare correct

1 The idea of Bhakti is elucidated in

Bhagavata Gita

2 Tamil poet Appar was an Alvar

saint

Select the correct answer using the code

given below

(a) 1 only

(b) 2 only

(c) Both 1 and 2

(d) Neither 1 nor 2

Q22) Which of the following rulers had his

image inscribed in the coins while

playing a veena

(a) Meander

(b) Chandragupta Maurya

(c) Gautamiputra Satkarni

(d) Samudragupta

Q23) Which of the following statements

isare correct

1 Vikrama Samvat is adopted as the

national calendar by independent

India and it began in 68 BC

2 Banabhatta was a court poet of

Harshavardhana

Select the correct answer using the code

given below

(a) 1 only

(b) 2 only

(c) Both 1 and 2

(d) Neither 1 nor 2

Q24) Consider the following pairs

1 Sandhi-vigrahika Minister of trade

2 Prathama-kulika Chief craftsman

3 Sarthavaha Chief banker

Which of the pairs given above isare

correct

(a) 1 only

(b) 1 and 3 only

(c) 2 only

(d) 2 and 3 only

Q25) Which of the following statements

isare correct

1 Aihole inscription belongs to

Pulakeshin II and was composed

by Ravikriti

2 Xuan Zang came to India during

the reign of Chandragupta II

Select the correct answer using the code

given below

(a) 1 only

(b) 2 only

(c) Both 1 and 2

(d) Neither 1 nor 2

RAUSIAS-FC19E1003 10

Q26) िासतकिा स सोबोनरत निमननिखित ततव ो म स कौि-स

ततव किि नहोद मखनदर ो की िासतकिा क भाग ह

1 नशिर

2 मणडप

3 परदनकषरा पथ

4 गभणगह

िीच नदए गए कट का परय ग कर सही उततर चनिएः

(a) किि 1 3 और 4

(b) किि 2 3 और 4

(c) किि 1 2 और 4

(d) 1 2 3 और 4

Q27) निमननिखित मोनदर ो म स कौि-स मोनदर ईोट ो स बि ह

1 दिगढ़ मोनदर

2 भीतरगाि मोनदर

3 िकषमर मोनदर नसरपर

4 बहदशवर मोनदर

िीच नदए गए कट का परय ग कर सही उततर चनिएः

(a) किि 1 2 और 3

(b) किि 2 3 और 4

(c) किि 1 3 और 4

(d) 1 2 3 और 4

Q28) निमननिखित कथि ो म स कौि-सास सही हह

1 सलताि महममद किी कतब शाह अकबर का

समकािीि था

2 िासतकिा क कषतर म महममद किी कतब शाह

ि कई ईमारत ो का निमाणर करिाया था नजिम

स चार मीिार सिाणनरक परनसदध ह

िीच नदए गए कट का परय ग कर सही उततर चनिएः

(a) किि 1

(b) किि 2

(c) 1 और 2 द ि ो

(d) ि त 1 ि ही 2

Q29) निमननिखित यग ो पर निचार कीनजएः

1 मनिमकिाई सततिार

2 अनभजञाि शाको तिम कानिदास

3 नसिपपानदकारम क ििि

उपयणकत यग ो म स कौि-सास सही समनित हह

(a) किि 1 और 2

(b) किि 2

(c) किि 1 और 3

(d) 1 2 और 3

Q30) निमननिखित कथि ो म स कौि-सास सही हह

1 ldquoसशरत सोनहताrdquo नचनकरता पर एक महतवपरण

रचिा ह

2 बरहमगपत और चरक महतवपरण गनरतजञ थ

िीच नदए गए कट का परय ग कर सही उततर चनिए

(a) किि 1

(b) किि 2

(c) 1 और 2 द ि ो

(d) ि त 1 ि ही 2

Q31) अमीर िसर क सनदभण म निमननिखित कथि ो म स

कौि-सास सही हह

1 अमीर िसर ि अपिी रचिाओो म नििा ह

नक सोसकत नकसी भी कषतर स सोबोनरत िही ो थी

और किि बराहमर ही इस भािा का जञाि रित

2 उन ोि नहोदिी और अिरी क अखसततव का

उललि नकया था

िीच नदए गए कट का परय ग कर सही उततर चनिए

(a) किि 1

(b) किि 2

(c) 1 और 2 द ि ो

(d) ि त 1 ि ही 2

RAUSIAS-FC19E1003 11

Q26) Which of the following architectural

elements were only part of Hindu

temple architecture

1 Shikhara

2 Mandapa

3 Pradakshina patha

4 Garbhagriha

Select the correct answer using the code

given below

(a) 1 3 and 4 only

(b) 2 3 and 4 only

(c) 1 2 and 4 only

(d) 1 2 3 and 4

Q27) Which of the following temples isare

made of bricks

1 Deogarh Temple

2 Bhitargaon Temple

3 Lakshmana temple Sirpur

4 Brihadeshvara Temple

Select the correct answer using the code

given below

(a) 1 2 and 3 only

(b) 2 3 and 4 only

(c) 1 3 and 4 only

(d) 1 2 3 and 4

Q28) Which of the following statements

isare correct

1 Sultan Muhammad Quli Qutab

Shah was a contemporary of

Akbar

2 In the field of architecture

Muhammad Quli Qutab Shah

constructed many buildings the

most famous of which is the Char

Minar

Select the correct answer using the code

given below

(a) 1 only

(b) 2 only

(c) Both 1 and 2

(d) Neither 1 nor 2

Q29) Consider the following pairs

1 Manimekalai Sattanar

2 Abhijnana Shakuntalam Kalidasa

3 Silappadikaram Kovalan

Which of the pairs given above isare

correct

(a) 1 and 2 only

(b) 2 only

(c) 1 and 3 only

(d) 1 2 and 3

Q30) Which of the following statements

isare correct

1 Sushruta Samhita is an important

work on medicine

2 Brahmagupta and Charaka were

important mathematicians

Select the correct answer using the code

given below

(a) 1 only

(b) 2 only

(c) Both 1 and 2

(d) Neither 1 nor 2

Q31) Which of the following statements

isare correct about Amir Khusrau

1 Amir Khusrau records in his works

that Sanskrit did not belong to any

region and only the Brahmans

knew it

2 He recorded the existence of

Hindawi and Awadhi

Select the correct answer using the code

given below

(a) 1 only

(b) 2 only

(c) Both 1 and 2

(d) Neither 1 nor 2

RAUSIAS-FC19E1003 12

Q32) निमननिखित कथि ो पर निचार कीनजए

1 नहरणय-गभण अिषठाि क बार म ऐसा स चा जाता

था नक बनि दि िाि का एक कषनतरय क रप म

पिजणनम ह गा

2 मयरशमणि कदोब िोश का सोसथापक था

उपयणकत कथि ो म स कौि-सास सही हह

(a) किि 1

(b) किि 2

(c) 1 और 2 द ि ो

(d) ि त 1 ि ही 2

Q33) निमननिखित कथि ो म स कौि-सास सही हह

1 कदमई बगार (बिपिणक शरम) क रप म

निया जाि िािा कर था

2 गवानियर परशखसत म िागभट (ज एक चोदि

राजा था) क दवारा नकय गए श िर का िरणि

नकया गया ह

िीच नदए गए कट का परय ग कर सही उततर चनिए

(a) किि 1

(b) किि 2

(c) 1 और 2 द ि ो

(d) ि त 1 ि ही 2

Q34) निमननिखित कथि ो म स कौि-सास सही हह

1 राजतरो नगिी 11िी ो शताबदी म कलहि क दवारा

रनचत एक सोसकत पसतक (टकसट) ह

2 कननौज क निए नतरपकषीय सोघिण म पाि राजिोश

शानमि था

िीच नदए गए कट का परय ग कर सही उततर चनिए

(a) किि 1

(b) किि 2

(c) 1 और 2 द ि ो

(d) ि त 1 ि ही 2

Q35) निमननिखित यग ो पर निचार कीनजए

1 बरहदशवर मोनदर राजराजा च ि

2 उर मापि की इकाई

3 दिदाि मोनदर ो क भनम अिदाि

उपयणकत यग ो म स कौि-स सही समनित ह

(a) किि 1 और 2

(b) किि 2 और 3

(c) किि 1 और 3

(d) 1 2 और 3

Q36) निमननिखित कथि ो म स कौि-सास सही हह

1 नदलली क सलताि ो क अरीि परशासि की भािा

फारसी थी

2 नदलली सलतित म ldquoतारीितािरीिrdquo कनिता

का एक रप था

िीच नदए गए कट का परय ग कर सही उततर चनिए

(a) किि 1

(b) किि 2

(c) 1 और 2 द ि ो

(d) ि त 1 ि ही 2

Q37) निमननिखित कथि ो म स कौि-सास सही हह

1 अिाउददीि खििजी ि अपि सनिक ो क निए

नसरी िाम का एक िया दगणरकषक शहर

बिािाया था

2 िह अपि सनिक ो क िति का भगताि इकता

क रप म करता था

िीच नदए गए कट का परय ग कर सही उततर चनिए

(a) किि 1

(b) किि 2

(c) 1 और 2 द ि ो

(d) ि त 1 ि ही 2

RAUSIAS-FC19E1003 13

Q32) Consider the following statements

1 Hiranya-garbha ritual was thought

to lead to the rebirth of the

sacrificer as a Kshatriya

2 Mayurasharman was the founder

of the Kadamba dynasty

Which of the statements given above

isare correct

(a) 1 only

(b) 2 only

(c) Both 1 and 2

(d) Neither 1 nor 2

Q33) Which of the following statements

isare correct

1 Kadamai was tax taken in form of

forced labour

2 Gwalior Prashasti describes the

exploits of Nagabhata who was a

Chandella king

Select the correct answer using the code

given below

(a) 1 only

(b) 2 only

(c) Both 1 and 2

(d) Neither 1 nor 2

Q34) Which of the following statements

isare correct

1 Rajatarangini is a Sanskrit text

written by Kalhana in the 11th

century

2 Pala dynasty was included in the

tripartite struggle for Kannauj

Select the correct answer using the code

given below

(a) 1 only

(b) 2 only

(c) Both 1 and 2

(d) Neither 1 nor 2

Q35) Consider the following pairs

1 Brihadeshvara temple Rajaraja

Chola

2 ldquoUrrdquo Unit of measurement

3 Devadana Land grants made to

temples

Which of the pairs given above isare

correct

(a) 1 and 2 only

(b) 2 and 3 only

(c) 1 and 3 only

(d) 1 2 and 3

Q36) Which of the following statements

isare correct

1 The language of administration

under the Delhi Sultans was

Persian

2 Tarikhtawarikh was a form of

poetry in the Delhi Sultanate

Select the correct answer using the code

given below

(a) 1 only

(b) 2 only

(c) Both 1 and 2

(d) Neither 1 nor 2

Q37) Which of the following statements

isare correct

1 Alauddin Khilji constructed a new

garrison town named Siri for his

soldiers

2 He paid his soldiers their salaries

in the form of Iqta

Select the correct answer using the code

given below

(a) 1 only

(b) 2 only

(c) Both 1 and 2

(d) Neither 1 nor 2

RAUSIAS-FC19E1003 14

Q38) निमननिखित कथि ो म स कौि-सास सही हह

1 नदलली कतबददीि एबक क अरीि पहिी बार

नकसी सामराजय की राजरािी बिी थी

2 दहिीिाि नसक ो का मदरर मग़ि ो क दवारा

नकया गया था

िीच नदए गए कट का परय ग कर सही उततर चनिए

(a) किि 1

(b) किि 2

(c) 1 और 2 द ि ो

(d) ि त 1 ि ही 2

Q39) निमननिखित यग ो पर निचार कीनजए

1 म ठ की मखिद नसको दर ि दी

2 बगमपरी मखिद नफर ज शाह तगिक

3 कववत- अि - इसलाम कतबददीि ऐबक

उपयणकत यग ो म स कौि-स सही समनित ह

(a) किि 1 और 2

(b) किि 2 और 3

(c) किि 1 और 3

(d) 1 2 और 3

Q40) निमननिखित कथि ो म स कौि-सास सही हह

1 मिसबदार ो क अपिा िति राजसव कायो

नजन जागीर कहत थ क रप म परापत ह ता

था

2 मिसबदार क ज सनय उततरदानयतव सौोप जात

थ उसक अनतगणत उस एक निराणररत सखया म

सिार अथिा घड़सिार ो का रि-रिाि करिा

पड़ता था

िीच नदए गए कट का परय ग कर सही उततर चनिए

(a) किि 1

(b) किि 2

(c) 1 और 2 द ि ो

(d) ि त 1 ि ही 2

Q41) ldquo1942 क भारत छ ड़ आोद ििrdquo क बार म

निमननिखित अिि कि ो म स कौि-सा सतय िही ो ह

(a) यह एक अनहोसक आोद िि था

(b) इसका िततव महातमा गाोरी क दवारा नकया गया

था

(c) यह एक सवाभानिक आोद िि था

(d) इसि सामानयतया शरनमक िगण क आकनिणत

िही ो नकया था

Q42) भारत क ि ग ो ि ldquoसाइमि कमीशिrdquo क आगमि क

निरदध आोद िि नकया था कय ोनक

(a) भारतीय कभी भी 1919 क अनरनियम (The

Act of 1919) क काम की समीकषा िही ो करिा

चाहत थ

(b) साइमि कमीशि ि पराोत ो म दवर (द हर) शासि

क समापत करि की नसफाररश की थी

(c) साइमि कमीशि म क ई भारतीय सदसय िही ो

था

(d) साइमि कमीशि ि दश क निभाजि का

सझाि नदया था

Q43) निमननिखित कथि ो पर निचार कीनजए

भारतीय राषटर ीय आोद िि म दादाभाई िौर जी क दवारा

नकया गया सबस परभािी य गदाि यह था नक उन ोि

1 अोगरज ो क दवारा भारत क आनथणक श िर का

ििासा नकया था

2 पराचीि भारतीय गरोथ ो की वयाखया की थी और

भारतीय ो क आतमनिशवास क पिःसथानपत नकया

था

3 अनय नकसी भी बात स पहि सभी सामानजक

बराइय ो क उनमिि की आिशयकता पर बि

नदया था

उपयणकत कथि ो म स कौि-सास सही हह

(a) किि 1

(b) किि 2 और 3

(c) किि 1 और 3

(d) 1 2 और 3

RAUSIAS-FC19E1003 15

Q38) Which of the following statements

isare correct

1 Delhi first became the capital of a

kingdom under Qutubuddin

Aibak

2 Dehliwal coins were minted by the

Mughals

Select the correct answer using the code

given below

(a) 1 only

(b) 2 only

(c) Both 1 and 2

(d) Neither 1 nor 2

Q39) Consider the following pairs

1 Moth ki Masjid- Sikander Lodi

2 Begumpuri mosque- Firuz Shah

Tughluq

3 Quwwat al ndash Islam- Qutubuddin

Aibak

Which of the above pairs isare correct

(a) 1 and 2 only

(b) 2 and 3 only

(c) 1 and 3 only

(d) 1 2 and 3

Q40) Which of the following statements

isare correct

1 Mansabdars received their salaries

as revenue assignments called

jagirs

2 The mansabdarrsquos military

responsibilities required him to

maintain a specified number of

sawar or cavalrymen

Select the correct answer using the code

given below

(a) 1 only

(b) 2 only

(c) Both 1 and 2

(d) Neither 1 nor 2

Q41) Which one of the following observations

is not true about the Quit India

Movement of 1942

(a) It was a non-violent movement

(b) It was led by Mahatma Gandhi

(c) It was a spontaneous movement

(d) It did not attract the labour class

in general

Q42) The people of India agitated against the

arrival of the Simon Commission

because

(a) Indians never wanted the review of

the working of the Act of 1919

(b) Simon Commission recommended

the abolition of dyarchy in the

Provinces

(c) there was no Indian member in the

Simon Commission

(d) the Simon Commission suggested

the partition of the country

Q43) Consider the following statements

The most effective contribution made by

Dadabhai Naoroji to the cause of Indian

National Movement was that he-

1 exposed the economic exploitation

of India by the British

2 interpreted the ancient Indian

texts and restored the self-

confidence of Indians

3 stressed the need for eradication of

all the social evils before anything

else

Which of the statements given above

isare correct

(a) 1 only

(b) 2 and 3 only

(c) 1 and 3 only

(d) 1 2 and 3

RAUSIAS-FC19E1003 16

Q44) महातमा गाोरी ि 1932 म आमरर अिशि नकया था

कय ोनक

(a) ldquoग िमज सममििrdquo (The Round Table

Conference) भारतीय राजिीनतक

आकाोकषाओो क परा करि म असफि रहा था

(b) काोगरस और मखसलम िीग म मतभद थ

(c) रामस मकड िालड (Ramsay Macdonald)

ि ldquoसाोपरदानयक परसकारrdquo (The Communal

Award) की घ िरा की थी

(d) ldquoसनििय अिजञा आोद ििrdquo (The Civil

Disobedience Movement) असफि रहा

था

Q45) भारत म औपनििनशक शासि की अिनर क सोदभण म

भारत स रि क बनहगणमि का एक महतवपरण भाग गह

शलक (Home Charges) था निमननिखित म स

कौि-सास क ि गह शलक म सखममनित नकया गया

थानकय गए थ

1 िोदि म भारत कायाणिय क निए उपय ग नकय

जाि िािा क ि

2 भारत म नियकत नबरनटश कनमणय ो क िति और

पशि का भगताि करि क निए उपय ग नकय

जाि िािा क ि

3 अोगरज ो क दवारा भारत क बाहर यदध ो क निए

उपय ग नकय जाि िािा क ि

िीच नदए गए कट का परय ग कर सही उततर चनिए

(a) किि 1

(b) किि 1 और 2

(c) किि 2 और 3

(d) 1 2 और 3

Q46) सवतोतरता आोद िि क इनतहास म भारतीय राषटर ीय

काोगरस का 1929 का सतर महतवपरण ह कय ोनक इसम

(a) काोगरस क उददशय क रप म सथािीय सरकार

की पराखपत की घ िरा की गई थी

(b) परण सवराज की पराखपत क काोगरस क िकषय क

रप म अपिाया गया था

(c) असहय ग आोद िि शर नकया गया था

(d) िोदि म ldquoग ि मर सममििrdquo (The Round

Table Conference) म भाग िि का निरणय

निया गया था

Q47) भारतीय सवतोतरता सोगराम क सोदभण म िहर ररप टण

क दवारा निमननिखित म स नकसकी नसफाररश की गई

थीनकिकी नसफाररश की गई थी ो

1 भारत क निए परण सवतोतरता

2 अलपसोखयक ो क निए सीट ो क आरकषर क

निए सोयकत नििाणचक मोडि

3 सोनिराि म भारत क ि ग ो क निए मौनिक

अनरकार ो का परािराि

िीच नदए गए कट का परय ग कर सही उततर चनिए

(a) किि 1

(b) किि 2 और 3

(c) किि 1 और 3

(d) 1 2 और 3

Q48) आरो नभक िनदक आयो का रमण मखय रप स था

(a) भखकत

(b) मनतण पजा और यजञ

(c) परकनत की पजा और यजञ

(d) परकनत की पजा और भखकत

RAUSIAS-FC19E1003 17

Q44) Mahatma Gandhi undertook fast unto

death in 1932 mainly because

(a) The Round Table Conference failed

to satisfy Indian political

aspirations

(b) The Congress and Muslim League

had differences of opinion

(c) Ramsay Macdonald announced the

Communal Award

(d) The Civil Disobedience Movement

failed

Q45) With reference to the period of colonial

rule in India ldquoHome Chargesrdquo formed

an important part of drain of wealth

from India Which of the following funds

constituted ldquoHome Chargesrdquo

1 Funds used to support the India

Office in London

2 Funds used to pay salaries and

pensions of British personnel

engaged in India

3 Funds used for waging wars

outside India by the British

Select the correct answer using the code

given below

(a) 1 only

(b) 1 and 2 only

(c) 2 and 3 only

(d) 1 2 and 3

Q46) The 1929- Session of Indian National

Congress is of significance in the history

of the Freedom Movement because the-

(a) attainment of Self-Government

was declared as the objective of

the Congress

(b) attainment of Poorna Swaraj was

adopted as the goal of the

Congress

(c) Non-Cooperation Movement was

launched

(d) decision to participate in the

Round Table Conference in

London was taken

Q47) With reference to the period of Indian

freedom struggle which of the following

waswere recommended by the Nehru

report

1 Complete Independence for India

2 Joint electorates for reservation of

seats for minorities

3 Provision of fundamental rights for

the people of India in the

Constitution

Select the correct answer using the code

given below

(a) 1 only

(b) 2 and 3 only

(c) 1 and 3 only

(d) 1 2 and 3

Q48) The religion of the early Vedic Aryans was primarily of

(a) Bhakti

(b) image worship and Yajnas

(c) worship of nature and Yajnas

(d) worship of nature and Bhakti

RAUSIAS-FC19E1003 18

Q49) भारत की यातरा करि िाि चीिी यातरी यआि चिाोग

(हयएि साोग) ि समकािीि भारत की सामानय

खसथनतय ो और सोसकनत क दजण नकया था इस सोदभण म

निमननिखित कथि ो म स कौि-सास सही हह

1 सड़क और िदी-मागण (जि-मागण) डकती स

परण रप स सरनकषत थ

2 जहा तक अपरार ो क निए दणड की बात ह

उसक निए नकसी भी वयखकत की निदोिता

अथिा उसक अपरार क निराणररत करि क

निए अनि जि और निि परि क माधयम क

सारि थ

3 वयापाररय ो क घाट ो और परनतबोर सटशि ो पर

शलक ो का भगताि करिा पड़ता था

िीच नदए गए कट का परय ग कर सही उततर चनिए

(a) किि 1

(b) किि 2 और 3

(c) किि 1 और 3

(d) 1 2 और 3

Q50) नसोर घाटी सभयता क सोदभण म निमननिखित कथि ो पर

निचार कीनजए

1 यह मखय रप स एक रमणनिरपकष सभयता थी

तथा हािाोनक इसम रानमणक ततव मौजद था

िनकि िह परनतिश पर हािी िही ो था

2 इस काि क दौराि भारत म कपास का परय ग

कपड़ा बिाि क निए नकया जाता था

उपयणकत कथि ो म स कौि-सास सही हह

(a) किि 1

(b) किि 2

(c) 1 और 2 द ि ो

(d) ि त 1 ि ही 2

Q51) परोदर दास क सोदभण म निमननिखित कथि ो पर निचार

कीनजए

1 परोदर दास एक सोत और भगिाि नशि क एक

महाि भकत थ

2 ि एक सोगीतकार गायक और किाणटक सोगीत

क मखय सोसथापक-परसतािक ो म स एक थ

उपयणकत कथि ो म स कौि-सास सही हह

(a) किि 1

(b) किि 2

(c) 1 और 2 द ि ो

(d) ि त 1 ि ही 2

Q52) निमननिखित म स कौि-सास वयखकत किाणटक सोगीत

की नतरमनतण म शानमि हह

1 बािामरिी कषणा

2 शरी शयाम शासतरी

3 शरी मथसवामी दीनकषतर

िीच नदए गए कट का परय ग कर सही उततर चनिए

(a) किि 1

(b) किि 2

(c) किि 2 और 3

(d) 1 2 और 3

Q53) चियर (Chevayur) और अथ िी (Atholi) म खसथत

महापािार सथि निमननिखित म स नकस राजय म खसथत

(a) तनमििाड

(b) किाणटक

(c) पनिम बोगाि

(d) करि

RAUSIAS-FC19E1003 19

Q49) The Chinese traveller Yuan Chwang

(Hiuen Tsang) who visited India

recorded the general conditions and

culture of India at that time In this

context which of the following

statements isare correct

1 The roads and river-routes were

completely immune from robbery

2 As regards punishment for

offences ordeals by fire water and

poison were the instruments for

determining the innocence or guilt

of a person

3 The tradesmen had to pay duties

at ferries and barrier stations

Select the correct answer using the code

given below

(a) 1 only

(b) 2 and 3 only

(c) 1 and 3 only

(d) 1 2 and 3

Q50) Regarding the Indus Valley Civilization

consider the following statements

1 It was predominantly a secular

civilization and the religious

element though present did not

dominate the scene

2 During this period cotton was

used for manufacturing textiles in

India

Which of the statements given above

isare correct

(a) 1 only

(b) 2 only

(c) Both 1 and 2

(d) Neither 1 nor 2

Q51) Consider the following statements

regarding Purandara Dasa

1 Purandara Dasa was a saint and

great devotee of Lord Shiva

2 He was a composer singer and

one of the chief founding-

proponents of the Carnatic music

Which of the statements given above

isare correct

(a) 1 only

(b) 2 only

(c) Both 1 and 2

(d) Neither 1 nor 2

Q52) Which of the following persons isare

included in the trinity of Carnatic

music

1 Balamurali Krishna

2 Sri Shyama Shastry

3 Sri Muthuswami Dikshitar

Select the correct answer using the code

given below

(a) 1 only

(b) 2 only

(c) 2 and 3 only

(d) 1 2 and 3

Q53) Megalithic sites at Chevayur and Atholi

are located in which of the following

states

(a) Tamil Nadu

(b) Karnataka

(c) West Bengal

(d) Kerala

RAUSIAS-FC19E1003 20

Q54) निमननिखित कथि ो पर निचार कीनजए

1 महापािानरक ि ग कबर ो म िसतएो दफिात थ

2 दनकषर भारत म महापािार सोसकनत एक परण

निकनसत तामर यगीि सोसकनत थी

उपयणकत कथि ो म स कौि-सास सही हह

(a) किि 1

(b) किि 2

(c) 1 और 2 द ि ो

(d) ि त 1 ि ही 2

Q55) निमननिखित म स कौि-स सामराजयसामराजय ो का

अश क क अनभिि ो म उललि नकया गया ह

1 च ि

2 पाणडय

3 करिपतर (चर)

िीच नदए गए कट का परय ग कर सही उततर चनिए

(a) किि 1

(b) किि 1 और 2

(c) किि 3

(d) 1 2 और 3

Q56) भीमा-क रगाोि का यदध को पिी क सनिक ो और

बाजीराि नदवतीय क िततव म एक शखकतशािी पशिा

सिा (मराठ ो) क मधय िड़ा गया था यह यदध

निमननिखित म स नकसका नहससा था

(a) परथम आोगल-मराठा यदध का

(b) नदवतीय आोगल-मराठा यदध का

(c) ततीय आोगल-मसर यदध का

(d) ततीय आोगल-मराठा यदध का

Q57) निमननिखित कथि ो पर निचार कीनजए

1 महादि दसाई ि गाोरीजी क चोपारर आि तथा

नतिकनथया पररािी स जड़ी समसया की जाोच

क निए रारी करि क निए दश भर म उिका

अिसरर नकया था

2 िरहरी पाररि चोपारर सतयागरह क दौराि

गाोरीजी क साथ थ

उपयणकत कथि ो म स कौि-सास सही हह

(a) किि 1

(b) किि 2

(c) 1 और 2 द ि ो

(d) ि त 1 ि ही 2

Q58) निमननिखित कथि ो पर निचार कीनजए

1 िनद राज-िोश ि बराहमर ो और बौदध मठराररय ो

क कर-मकत गाि अिदाि म दि की परथा

आरि की थी

2 सतिाहि ो की आनरकाररक भािा पराकत थी

उपयणकत कथि ो म स कौि-सास सही हह

(a) किि 1

(b) किि 2

(c) 1 और 2 द ि ो

(d) ि त 1 ि ही 2

Q59) एक निरासत क अपिाइए (अडॉपट ए हररटज ndash

Adopt a Heritage) पररय जिा क उददशय ो क

सनदभण म निमननिखित कथि ो पर निचार कीनजए

1 यह पररय जिा र रगार उतपादि और आनथणक

निकास क निए पयणटि कषमता का उि पर

परभाि का उपय ग करगी

2 यह पररय जिा निरासत सथि ो पर निशव सतरीय

आराररक सोरचिा निकनसत करक एक सतत

तरीक स पयणटक आकिणर म िखदध करगी

उपयणकत कथि ो म स कौि-सास सही हह

(a) किि 1

(b) किि 2

(c) 1 और 2 द ि ो

(d) ि त 1 ि ही 2

RAUSIAS-FC19E1003 21

Q54) Consider the following statements

1 Megalithic people buried goods in

graves

2 The megalithic culture in South

India was a full-fledged Copper

Age culture

Which of the statements given above

isare correct

(a) 1 only

(b) 2 only

(c) Both 1 and 2

(d) Neither 1 nor 2

Q55) Which of the following kingdoms isare

mentioned in the Ashokan inscriptions

1 Cholas

2 Pandyas

3 Keralaputras (Cheras)

Select the correct answer using the code

given below

(a) 1 only

(b) 1 and 2 only

(c) 3 only

(d) 1 2 and 3

Q56) The Battle of Bhima-Koregaon was

fought between the soldiers of the

Company and the strong Peshwa army

(Marathas) under Bajirao II This war

was a part of the

(a) First Anglo-Maratha war

(b) Second Anglo-Maratha war

(c) Third Anglo- Mysore war

(d) Third Anglo-Maratha war

Q57) Consider the following statements

1 Mahadev Desai followed Gandhiji all over the country to persuade him to come to Champaran to investigate the problem associated

with tinkathia system

2 Narhari Parikh accompanied Gandhi ji during the Champaran

Satyagraha

Which of the statements given above isare correct

(a) 1 only

(b) 2 only

(c) Both 1 and 2

(d) Neither 1 nor 2

Q58) Consider the following statements

1 The Nanda Dynasty started the practice of granting tax-free villages to brahmanas and

Buddhist monks

2 The official language of the Satavahanas was Prakrit

Which of the statements given above

isare correct

(a) 1 only

(b) 2 only

(c) Both 1 and 2

(d) Neither 1 nor 2

Q59) Consider the following statements about the objectives of the lsquoadopt a heritagersquo

project

1 It will harness tourism potential for its effects on employment generation and economic

development

2 It will enhance the tourist attractiveness in a sustainable manner by developing world class infrastructure at heritage sites

Which of the statements given above

isare correct

(a) 1 only

(b) 2 only

(c) Both 1 and 2

(d) Neither 1 nor 2

RAUSIAS-FC19E1003 22

Q60) ldquoभारतीय जिजातीय सहकारी निपरि निकास सोघrdquo

(The Tribal Co-operative Marketing

Development Federation of India - TRIFED)

क सोदभण म निमननिखित कथि ो पर निचार कीनजए

1 यह एक राषटर ीय सतर का शीिण सोगठि ह ज

भारत सरकार क गह मोतरािय क परशासनिक

नियोतरर क अरीि काम कर रहा ह

2 इसका मखय उददशय दश म जिजातीय ि ग ो

का सामानजक-आनथणक निकास करिा ह

उपयणकत कथि ो म स कौि-सास सही हह

(a) किि 1

(b) किि 2

(c) 1 और 2 द ि ो

(d) ि त 1 ि ही 2

Q61) निमननिखित म स कौि-सास उपनयास परमचोद क

दवारा नििा गया हनिि गए ह

1 रोगभनम

2 ग दाि

3 ग रा

िीच नदए गए कट का परय ग कर सही उततर चनिए

(a) किि 1

(b) किि 2

(c) किि 1 और 2

(d) 1 2 और 3

Q62) नगदधा ितय क सोदभण म निमननिखित कथि ो पर निचार

कीनजए

1 नगदधा नबहार की मनहिाओो क दवारा तयौहार क

समय और फसि की बिाई तथा कटाई क

अिसर पर नकया जाि िािा एक पारोपररक

दहाती ितय ह

2 इस ितय क दवारा मनहिाऐो अपिी परसननता

परकट करती ह तथा नगदधा क परदशणि क

माधयम स परि िचणसव िाि समाज म

मनहिाओो की दबी हई भाििाओो क परकट

करती ह

उपयणकत कथि ो म स कौि-सास सही हह

(a) किि 1

(b) किि 2

(c) 1 और 2 द ि ो

(d) ि त 1 ि ही 2

Q63) निमननिखित कथि ो पर निचार कीनजए

1 मलला शाह बदखशी दारा नशक ह क

आधयाखतमक गर थ

2 औरोगरब ि मजम-उि-बहरीि या द समदर ो

का सोगम िामक उललििीय रचिा नििी थी

3 दारा नशक ह क अपि पिणज अकबर क गर ो

क उततरानरकारी क रप म दिा गया था

नजसम उसि रानमणक बहििाद और समनवयता

क बढ़ािा नदया था

उपयणकत कथि ो म स कौि-सास सही हह

(a) किि 1 और 3

(b) किि 2

(c) किि 1 और 2

(d) 1 2 और 3

RAUSIAS-FC19E1003 23

Q60) Consider the following statements about

the Tribal Cooperative Marketing

Development Federation of India

(TRIFED)

1 It is a national-level apex

organization functioning under the

administrative control of Ministry

of Home Affairs Government of

India

2 The main objective of TRIFED is

socio-economic development of

tribal people in the country

Which of the statements given above

isare correct

(a) 1 only

(b) 2 only

(c) Both 1 and 2

(d) Neither 1 nor 2

Q61) Which of the following novels isare

written by Premchand

1 Rangabhumi

2 Godan

3 Gora

Select the correct answer using the code

given below

(a) 1 only

(b) 2 only

(c) 1 and 2 only

(d) 1 2 and 3

Q62) Consider the following statements about

Giddha dance

1 Giddha is a traditional pastoral

dance performed by the women of

Bihar at festival times and at the

sowing and reaping of the harvest

2 By this dance the women reveal

their joy expel their suppressed

feelings in a male dominated

society through the performance of

Giddha

Which of the statements given above

isare correct

(a) 1 only

(b) 2 only

(c) Both 1 and 2

(d) Neither 1 nor 2

Q63) Consider the following statements

1 Mullah Shah Badakhshi was the

spiritual mentor of Dara Shukoh

2 Aurangzeb wrote the remarkable

work called ldquoMajma-ul-Bahrainrdquo or

the ldquoThe confluence of two seasrdquo

3 Dara Shukoh was seen as

inheriting the qualities of his

ancestor Akbar in that he

promoted religious pluralism and

syncretism

Which of the statements given above

isare correct

(a) 1 and 3 only

(b) 2 only

(c) 1 and 2 only

(d) 1 2 and 3

RAUSIAS-FC19E1003 24

Q64) निमननिखित कथि ो पर निचार कीनजए

1 ग मतशवर परनतमा निोधयनगरी पहाड़ी पर खसथत ह

2 शरिरबिग िा िह सथाि ह जहाो मौयण िोश क

सोसथापक चोदरगपत मौयण अपि नसोहासि क

तयागि क बाद जि तपसवी बि गए थ

उपयणकत कथि ो म स कौि-सास सही हह

(a) किि 1

(b) किि 2

(c) 1 और 2 द ि ो

(d) ि त 1 ि ही 2

Q65) निमननिखित कथि ो पर निचार कीनजए

1 पराताखतवक साकषय स पता चिता ह नक पराची

घाटी सभयता हड़पपा और म हिज दाड़ द ि ो

की पिणिती ह

2 पराची िदी भििशवर स निकिती ह

उपयणकत कथि ो म स कौि-सास सही हह

(a) किि 1

(b) किि 2

(c) 1 और 2 द ि ो

(d) ि त 1 ि ही 2

Q66) निमननिखित कथि ो म स कौि-सास सही हह

1 िजराह क समारक ो क समह का निमाणर

चोदि राजिोश क शासिकाि क दौराि हआ

था

2 य समारक हररिोदर पिणत शरोििा म खसथत ह

3 म रक क यातरी इबन बतता ि अपि सोसमरर ो

म िजराह क मोनदर ो की यातरा का उललि

नकया था तथा इन काजराण िाम स समब नरत

नकया था

िीच नदए गए कट का परय ग कर सही उततर चनिए

(a) किि 1

(b) किि 1 और 2

(c) किि 2 और 3

(d) किि 1 और 3

Q67) निमननिखित कथि ो म स कौि-सास सही हह

1 डॉ बी आर अमबडकर ि दी एनिनहिशि

ऑफ़ कासट (The Annihilation of Caste)

नििी थी नजसम उन ोि नहोद रमण म िोशािगत

पजारी की परथा क उनमिि की आिशयकता

पर बि नदया था

2 डॉ राजदर परसाद ि थॉटस ऑि पानकसताि

(Thoughts on Pakistan) िामक पसतक

नििी थी

िीच नदए गए कट का परय ग कर सही उततर चनिए

(a) किि 1

(b) किि 2

(c) 1 और 2 द ि ो

(d) ि त 1 ि ही 2

Q68) निमननिखित कथि ो म स कौि-सास सही हह

1 महरगढ़ भारतीय उपमहादवीप म एक परनसदध

ििपािार बसती ह ज नसोर पराोत पानकसताि म

खसथत ह

2 बरणह म म कतत ो क उिक सवामी क साथ कबर ो

म दफिाया जाता था

िीच नदए गए कट का परय ग कर सही उततर चनिए

(a) किि 1

(b) किि 2

(c) 1 और 2 द ि ो

(d) ि त 1 ि ही 2

Q69) निमननिखित कथि ो म स कौि-सास सही हह

1 काकानटय मोनदर अनरकतर नशि क समनपणत

2 हिमक ोडा म हजार-सतोभ िाि मोनदर (The

Thousand-Pillared Temple) का निमाणर

काकानटय समराट रदर ि करिाया था

िीच नदए गए कट का परय ग कर सही उततर चनिए

(a) किि 1

(b) किि 2

(c) 1 और 2 द ि ो

(d) ि त 1 ि ही 2

RAUSIAS-FC19E1003 25

Q64) Consider the following statements

1 Gommateshwara Statue is located

on the Vindyagiri Hill

2 Shravanabelagola is the place

where Chandragupta Maurya the

founder of the Mauryan dynasty

became a Jain ascetic after

relinquishing his throne

Which of the statements given above

isare correct

(a) 1 only

(b) 2 only

(c) Both 1 and 2

(d) Neither 1 nor 2

Q65) Consider the following statements

1 Archaeological evidence shows

that the Prachi Valley Civilisation

predates both Harappa and

Mohenjo-Daro

2 The Prachi river originates from

Bhubaneswar

Which of the statements given above

isare correct

(a) 1 only

(b) 2 only

(c) Both 1 and 2

(d) Neither 1 nor 2

Q66) Which of the following statements

isare correct

1 The Khajuraho group of

monuments was built during the

rule of the Chandela dynasty

2 These monuments are located in

Harischandra mountain range

3 Ibn Battuta the Moroccan

traveller in his memoirs mentioned

visiting Khajuraho temples and

called them Kajarra

Select the correct answer using the code

given below

(a) 1 only

(b) 1 and 2

(c) 2 and 3

(d) 1 and 3

Q67) Which of the following statements

isare correct

1 Dr BR Ambedkar wrote the

Annihilation of Caste emphasising

the need to do away with the

practice of hereditary priesthood in

Hinduism

2 The book lsquoThoughts on Pakistanrsquo

was written by Dr Rajendra

Prasad

Select the correct answer using the code

given below

(a) 1 only

(b) 2 only

(c) Both 1 and 2

(d) Neither 1 nor 2

Q68) Which of the following statements

isare correct

1 Mehrgarh is a famous Neolithic

settlement in the Indian

subcontinent which is situated in

Sindh province Pakistan

2 At Burzahom dogs were buried

with their masters in their graves

Select the correct answer using the code

given below

(a) 1 only

(b) 2 only

(c) Both 1 and 2

(d) Neither 1 nor 2

Q69) Which of the following statements

isare correct

1 The Kakatiya temples are

dedicated mostly to Siva

2 The Thousand-Pillared Temple at

Hanamkonda was built by the

Kakatiya king Rudra

Select the correct answer using the code

given below

(a) 1 only

(b) 2 only

(c) Both 1 and 2

(d) Neither 1 nor 2

RAUSIAS-FC19E1003 26

Q70) निमननिखित कथि ो म स कौि-सास सही हह

1 अहमदाबाद नमि हड़ताि क दौराि महातमा

गाोरी ि शरनमक ो क पकष क मजबत करि क

निए आमरर अिशि नकया था

2 अिशि स नमि मानिक ो पर दबाि पड़ा था ज

अोततः शरनमक ो क िति म 15 परनतशत की िखदध

करि क निए सहमत हए थ

िीच नदए गए कट का परय ग कर सही उततर चनिए

(a) किि 1

(b) किि 2

(c) 1 और 2 द ि ो

(d) ि त 1 ि ही 2

Q71) निमननिखित म स नकसक नकिक भारत स यिसक

की माििता की अमतण साोसकनतक निरासत की

परनतनिनर सची (The UNESCOrsquos List of the

Representative List of the Intangible

Cultural Heritage of Humanity) म शानमि

नकया गया ह

1 मनडयटट

2 सोकीतणि

3 को भ मिा

िीच नदए गए कट का परय ग कर सही उततर चनिए

(a) किि 1 और 2

(b) किि 2 और 3

(c) किि 3

(d) 1 2 और 3

Q72) निमननिखित जिजानतय ो म स कौि-सीसी ो

जिजानतजिजानतया िागािड स सोबोनरत हह

1 अोगामी

2 ककी

3 जारिा

िीच नदए गए कट का परय ग कर सही उततर चनिए

(a) किि 1

(b) किि 1 औऔ 2

(c) किि 2

(d) 1 2 और 3

Q73) निमननिखित कथि ो म स कौि-सास सही हह

1 राषटर कट सामराजय की सथापिा दोनतदगण ि की थी

नजसि मानयाित म अपिी राजरािी की

सथापिा की थी

2 राषटर कट समराट अम घििण एक ििक था और

उस कनिताओो पर पहिी कननड़ पसतक नििि

का शरय नदया जाता ह

िीच नदए गए कट का परय ग कर सही उततर चनिए

(a) किि 1

(b) किि 2

(c) 1 और 2 द ि ो

(d) ि त 1 ि ही 2

Q74) निमननिखित कथि ो म स कौि-सास सही हह

1 कशब चोदर सि ि ततवब नरिी सभा की

अधयकषता की थी ज आधयाखतमक सतय की

ि ज म सोिि थी

2 बरहम समाज ि मािि गररमा पर बि नदया

मनतणपजा का निर र नकया और सती परथा जसी

सामानजक बराइय ो की आि चिा की

िीच नदए गए कट का परय ग कर सही उततर चनिए

(a) किि 1

(b) किि 2

(c) 1 और 2 द ि ो

(d) ि त 1 ि ही 2

Q75) निमननिखित कथि ो म स कौि-सास सही हह

1 भारत म नचशती नसिनसिा खवाजा म इिददीि

नचशती क दवारा सथानपत नकया गया था

2 नचशती परोपरा की एक परमि निशिता

आतमसोयम थी नजसम साोसाररक म ह स दरी

बिाए रििा शानमि था

िीच नदए गए कट का परय ग कर सही उततर चनिए

(a) किि 1

(b) किि 2

(c) 1 और 2 द ि ो

(d) ि त 1 ि ही 2

RAUSIAS-FC19E1003 27

Q70) Which of the following statements

isare correct

1 During the Ahmedabad Mill Strike

Mahatma Gandhi undertook a fast

unto death to strengthen the

workersrsquo resolve

2 The fast had effect of putting

pressure on mill owners who

finally agreed to give the workers a

15 per cent increase in wages

Select the correct answer using the code

given below

(a) 1 only

(b) 2 only

(c) Both 1 and 2

(d) Neither 1 nor 2

Q71) Which of the following are included in

the UNESCOrsquos list of the representative

list of the intangible cultural heritage of

humanity from India

1 Mudiyettu

2 Sankirtana

3 Kumbh Mela

Select the correct answer using the code

given below

(a) 1 and 2 only

(b) 2 and 3 only

(c) 3 only

(d) 1 2 and 3

Q72) Which of the following tribes isare

related to Nagaland

1 Angami

2 Kuki

3 Jarawa

Select the correct answer using the code

given below

(a) 1 only

(b) 1 and 2 only

(c) 2 only

(d) 1 2 and 3

Q73) Which of the following statements

isare correct

1 Rashtrakuta kingdom was founded by Dantidurga who established his capital at Manyakhet

2 Amoghavarsha a Rashtrakuta king was an author and is credited with writing the first

Kannada book on poetics

Select the correct answer using the code given below

(a) 1 only

(b) 2 only

(c) Both 1 and 2

(d) Neither 1 nor 2

Q74) Which of the following statements isare correct

1 Keshab Chandra Sen headed the Tattvabodhini Sabha which was engaged in search of spiritual truth

2 The Brahmo Samaj laid emphasis on human dignity opposed idolatry and criticized such social

evils as the practice of Sati

Select the correct answer using the code given below

(a) 1 only

(b) 2 only

(c) Both 1 and 2

(d) Neither 1 nor 2

Q75) Which of the following statements isare correct

1 The Chishti order was established in India by Khwaja Moinuddin

Chishti

2 A major feature of the Chishti tradition was austerity including maintaining a distance from the

worldly power

Select the correct answer using the code

given below

(a) 1 only

(b) 2 only

(c) Both 1 and 2

(d) Neither 1 nor 2

T e s t i s p a r t o f R a u rsquo s I A S T e s t s e r i e s f o r P r e l i m i n a r y E x a m 2 0 1 9

FOUNDATION + CURRENT AFFAIRS

GENERAL STUDIES (PAPER ndashI)

FOUNDATION TEST ndashIII

SUBJECT NCERT History Class VI-X + Current Affairs

Time Allowed 1frac12 Hours Maximum Marks 150

I NSTRUCT IONS

1 IMMEDIATELY AFTER THE COMMENCEMENT OF THE EXAMINATION YOU SHOULD CHECK

THAT THIS TEST BOOKLET DOES NOT HAVE ANY UNPRINTED OR TORN or MISSING PAGES OR

ITEMS ETC IF SO GET IT REPLACED BY A COMPLETE TEST BOOKLET

2 This Test Booklet contains 75 items (questions) Each item is printed both in Hindi and English

Each item comprises four responses (answers) You will select the response which you want to mark

on the Answer Sheet In case you feel that there is more than one correct response mark the

response which you consider the best In any case choose ONLY ONE response for each item

3 You have to mark all your responses ONLY on the separate Answer Sheet (OMR sheet) provided

Read the directions in the Answer Sheet

4 All items carry equal marks

5 Before you proceed to mark in the Answer Sheet the response to various items in the Test booklet

you have to fill in some particulars in the Answer Sheet as per instructions contained therein

6 After you have completed filling in all your responses on the Answer Sheet and the examination has

concluded you should hand over to the Invigilator only the Answer Sheet You are permitted to

take away with you the Test Booklet

7 Penalty for wrong answers

THERE WILL BE PENALTY FOR WRONG ANSWERS MARKED BY A CANDIDATE IN THE

OBJECTIVE TYPE QUESTION PAPERS

(i) There are four alternatives for the answer to every question For each question for which a

wrong answer has been given by the candidate one-third of the marks assigned to that

question will be deducted as penalty

(ii) If a candidate gives more than one answer it will be treated as a wrong answer even if one of

the given answers happens to be correct and there will be same penalty as above to that

question

(iii) If a question is left blank ie no answer is given by the candidate there will be no penalty for

that question

T h i s t e s t i s p a r t o f R a u rsquo s I A S T e s t s e r i e s f o r P r e l i m i n a r y E x a m 2 0 1 9

Test Code

FC19E1003

FC19H1003 29

Answers and Explanations of

NCERT History Class VI-X + Current Affairs (FC19E1003)

Q1) उततर (c)

सपषटीकरण

- ऋगवद म दविय ो और दिताओो क समवपित एक

हजार स अविक सत तर (शल क) ह

- य शल क ऋविय ो क दवारा रच गए थ और परि ो

दवारा सीख जात थ

- हालाोवक कछ शल क मवहलाओो (जस वक अपाला

घ सा ल पामदरा मतरयी और गागी) क दवारा भी रच

गए थ

- ऋगवद म सोिाद क रप म कई शल क मौजद ह

- हम विशवावमतर नामक एक ऋवि और दविय ो क

रप म पजी जान िाली द नवदय ो (वयास और

सतलज) क बीच िाताि का उदाहरण वमलता ह

- इसस पता चलता ह वक विशवावमतर िवदक काल स

सोबोवित थ

Q2) उततर (b)

सपषटीकरण

- करनल गफाओो स राख क अिशि परापत हए ह

ज इस ओर सोकत करत ह वक ततकालीन ल ग

अवि क उपय ग स पररवचत थ

- य गफाएो आोधर परदश म सथथत ह

Q3) उततर (c)

सपषटीकरण

bull बरािह म ितिमान कशमीर म सथथत एक

परागवतहावसक थथल ह जहाो ल ग गडढ क घर ो का

वनमािण करत थ

bull य घर जमीन क ख द कर बनाए जात थ तथा नीच

जान क वलए सीवियाा ह ती थी

bull ऐसा अनमान लगाया जाता ह वक य घर ठो ड क

मौसम म आशरय परदान करत थ

Q4) उततर (c)

सपषटीकरण

bull परालख-विदया (Epigraphy) क वशलालख ो क

अधययन क रप म पररभावित वकया जाता ह

bull हसतवलसखत दसतािज ो क माधयम स इवतहास

और सावहतय क अधययन क पाोडवलवप विजञान

(Manuscriptology) कहत ह

bull पराचीन लखन परणावलय ो क अधययन और

ऐवतहावसक पाोडवलवपय ो क समझन तथा वतवथ

वनिािरण क पलीओगराफी (Palaeography) कहा

जाता ह

bull नयवमजमविकस (Numismatics) वसक ो क

अधययन क सोदवभित करता ह

Q5) उततर (a)

सपषटीकरण

- चरक सोवहता चरक क दवारा वलखी गई आयिद

और िदयक-शासर पर एक महतवपणि पसतक ह

- ि भारतीय िदयक-शासर की पारमपररक परणाली

वजस आयिद क नाम स जाना जाता ह क

अभयासकताि थ

- ऐसा माना जाता ह वक चरक का विकास दसरी

शताबदी (ईसा पिि) और दसरी शताबदी (ईसवी) क

मधय हआ था

Q6) उततर (b)

सपषटीकरण

- भाग फसल ो पर वलए जान िाल कर क सोदवभित

करता ह ज कल फसल उतपादन का 16 िाो भाग

था

- ldquoकममकारrdquo शबद भवमहीन कवि शरवमक िगि क

वलए परय ग वकया जाता था

- ldquoअशवमिrdquo (वजस घ ड क बवलदान क रप म भी

जाना जाता ह) एक अनषठान ह ता था वजसम एक

घ ड क सवतोतर रप स घमन क वलए छ ड वदया

FC19H1003 30

जाता ह और राजा क सवनक उसकी रखिाली

करत थ

Q7) उततर (d)

सपषटीकरण

- ऋगववदक काल म घ ड ो क रथ ो म ज ता जाता था

ज (रथ) भवम मिवशय ो आवद पर कबजा करन क

वलए लड गए यद ो म उपय ग वकए जात थ

- इसस यह पता चलता ह वक घ ड ो यकत रथ ो का

उपय ग महाजनपद काल स काफी पहल आरमभ

हआ था

- ऋगववदक काल म मिवशय ो भवम जल आवद पर

कबजा करन क वलए तथा ल ग ो क पकडन क

वलए यद वकय जात थ

- अविकाोश परि इन यद ो म भाग वलया करत थ

- हालाोवक उस समय क ई वनयवमत सना नही ो ह ती

थी लवकन उस काल म सभाऐो ह ती थी ो वजनम

ल ग यद क मामल ो पर चचाि करत थ

- वनयवमत सनाएा महाजनपद काल का िवशषटय थी

वजनम पदल सवनक ो की विशाल सनाएा रथ तथा

हाथी शावमल ह त थ

Q8) उततर (a)

सपषटीकरण

- बद शाकय कल स सोबोवित थ और कशीनारा म

उनका वनिन हआ था

- बद न अपनी वशकषाएा पराकत भािा म दी थी ो ज

आम ल ग ो की भािा थी

Q9) उततर (c)

सपषटीकरण

- पराचीन भारत म दशिनशासर की छह शाखाएा थी ो

िशविक नयाय समखया य ग पिि वममाोसा और

िदाोत या उततर वममाोसा

- इनकी थथापना करमश कनाद गौतम कवपल

पतोजवल जावमनी और वयास ऋविय ो न की थी

Q10) उततर (b)

सपषटीकरण

महािीर की वशकषाऐो छठी शताबदी म िललभी म

सोकवलत की गई थी ो

Q11) उततर (c)

सपषटीकरण

- पारमपररक रप स चाणकय क कौविलय अथिा

विषणगपत क नाम स जाना जाता ह

- उसन अथिशासतर ज एक पराचीन भारतीय

राजनवतक आलख ह वलखा था

Q12) उततर (d)

सपषटीकरण

- भारत का राषटर ीय वचनह सारनाथ (उततर परदश) क

अश क सतमभ क ऊपर (शीिि पर) वसोह कवपिल

का एक अनरपण ह

- इस राषटर ीय वसदाोत सतयमि जयत क साथ

सोय वजत वकया गया ह

- रामपिि बल का नाम रामपिि (वबहार) क नाम पर

पडा जहाा इसकी ख ज हई थी

- यह अपन नाजक नकाशी मॉडल क वलए परवसदद

ह वजसम क मल तवचा सोिदनशील नथन ो सतकि

कान और मरबत िााग ो क शरषठतर परवतरप क

परदवशित वकया गया ह

- यह भारतीय और फारसी ततव ो का एक ससममशरण

- सोवकससा उततर परदश म सथथत ह

Q13) उततर (a)

सपषटीकरण

का िर वसोह ज एक महान य दा थ वबहार स

सोबोवित थ

Q14) उततर (b)

सपषटीकरण

िललालर शबद बड भ-सवावमय ो क वलए परय ग

वकया जाता था

FC19H1003 31

Q15) उततर (c)

सपषटीकरण

- अररकमड एक तिीय बसती थी जहाो दर दश ो स

आन िाल जहाज ो का माल उतारा जाता था

- यहाो पर ईोि ो का एक विशाल ग दाम वमटटी क

बतिन (वजनम एमफ रा - द हरी मवठय ो का लोबा

घडा - शावमल ह) और एरिाइन (Arretine)

मदभाोड पाए गए थ

- इस थथान पर र मन दीपक काोच क बन पातर और

रतन भी पाए गए थ

Q16) उततर (a)

सपषटीकरण

- मिनदर सोगम कविताओो म उसललसखत एक

तवमल शबद ह वजसका अथि ह ldquoतीन परमखrdquo

- यह तीन सततारि पररिार ो क मसखयाओो क वलए

परय ग वकया जाता ह च ल चर और पाणडय

Q17) उततर (c)

सपषटीकरण

- ऋग िद म सभा विदाथा तथा गण जसी

जनजावतय ो पर अथिा किोब पर आिाररत

सभाओो का उललख ह

- आरसमभक िवदक काल म सभाओो और सवमवतय ो

का विशि महतव ह ता था

- यहाा तक की मसखया अथिा राजा भी उनका

समथिन परापत करन क वलए आतर रहत थ

Q18) उततर (a)

सपषटीकरण

- जन िमि न ईशवर क अससततव क मानयता त दी ह

वकनत उसन ईशवर क वजना क पद स नीच रखा

- जन िमि न बौद िमि की तरह िणि परणाली की

भरतिना नही ो की थी

Q19) उततर (d)

सपषटीकरण

- च ल ो और पाणडय ो न शसकतशाली तिीय शहर ो का

विकास वकया था

- च ल ो का सबस महतवपणि शहर पहार (या

कािरीपटटीनम) था |

- मदरई पाणडय ो की राजिानी थी

Q20) उततर (b)

सपषटीकरण

- ldquoबदचररतrdquo बद का जीिन-ितताोत ह

- इस अशवघ ि क दवारा वलखा गया था

Q21) उततर (a)

सपषटीकरणः

- तवमल कवि अपपर भगिान वशि क भकत थ

- इस परकार ि एक नयनार सोत थ

Q22) उततर (d)

सपषटीकरणः

- समदरगपत एक परवसद गपत शासक था

- उसन वसक ो पर िीणा बजात हए अपनी छवि

अोवकत करिाई थी

- यह सोगीत क परवत उसक परम क दशािता ह

- हम उसकी इलाहाबाद परशससत स महतवपणि

ऐवतहावसक जानकारी वमलती ह वजसकी रचना

उसक दरबार क कवि हररसन न की थी

Q23) उततर (b)

सपषटीकरणः

- विकरम सोित की शरआत ििि 58 ईसा पिि म

चनदरगपत वदवतीय न की थी

- यह शक ो पर उसकी जीत और उस विकरमावदतय

की पदिी वमलन क उपलकषय म आरमभ वकया गया

था

FC19H1003 32

- बानभटट न हिििििन का जीिन-ितताोत हििचररत

(ज सोसकत म थी) वलखी थी

Q24) उततर (c)

सपषटीकरणः

- सोवि-विगरावहका यद एिो शाोवत का मोतरी

- साथििाह वयापाररय ो क कावफल ो का नता

Q25) उततर (a)

सपषटीकरणः

- जआन झाोग (हसआन रताोग ndash Hsuang Tsang)

एक चीनी यातरी था ज हिििििन क शासनकाल म

भारत आया था

- ििि 630 ईसवी स ज दशक आरमभ हआ था उसम

जआन झाोग मधय एवशया ईरान और

अफग़ावनसतान की यातरा करन क पशचात कशमीर

क रासत स भारत आया था

- उसन उततर स पिि तक की यातरा की और िह

लगभग 2 ििि वबहार म रहा

- जआन झाोग न नालनदा विशवविदयालय म विदयावथिय ो

और विदवान ो क साथ पारसपररक विचार-विमशि

वकया थथानीय भािाओ ा म वनपणता परापत की तथा

बौद सतप ो की ख ज की

Q26) उततर (c)

सपषटीकरणः

- परदवकषणा पथ बौद िासतकला म सतप क चार ो

ओर बनाया जान िाला एक घमािदार पथ ह ता

- परशन म वदए गए बाकी क तीन ो ततव वहोद मसनदर ो की

िासतकला क भाग ह

Q27) उततर (d)

सपषटीकरणः

परशन म वदए गए सभी मोवदर ो म वयापक रप स

ईोि ो (पकी ईोि ो) का परय ग पतथर ो क साथ हआ

Q28) उततर (c)

सपषटीकरण

- महममद कली कतब शाह ग लकणडा का सलतान

था

- िह अकबर का समकालीन था

- सावहतय और िासतकला म उसकी अतयाविक

रवच थी

- िह एक महान कवि था

- िह दसखनी उदि फारसी और तलग म वलखता था

- उसन अपन पीछ एक विसतत वदिान (सोगरह)

छ डा ह

- अभी हाल ही म तलोगाना म ग लकणडा क वकल

क अनदर खदाई वकय गए बाग-ए-नाया वकला

बाग क चार ो ओर रप-रखा क मानवचतरण क

वलए भारतीय परातासतवक सिकषण (The

Archaeological Survey of India ndash ASI)

गराउणड पनीिर विोग रडार (Ground Penetrating

Radar) का परय ग करगा

Q29) उततर (a)

सपषटीकरणः

- वसलपपावदकारम एक तवमल महाकावय ह वजसकी

रचना इलाोग क दवारा लगभग 1800 ििि पिि की

गई थी

- यह क िलन नामक एक वयापारी की कहानी ह

ज माििी नामक एक गवणका (िशया) स परम

करन लगा था

- मवनमकलाई क िलन और माििी की पतरी की

कहानी ह

Q30) उततर (a)

सपषटीकरण

- चरक आयिद और वचवकरता की एक महतवपणि

रचना चरक सोवहता क लखक ह

- बरहमगपत क अपनी रचना बरहम-सफि-वसदानत

(ज एक खग लीय रचना ह) क कारण परवससद

वमली

FC19H1003 33

- बगदाद म इसका अनिाद अरबी भािा म वकया

गया था

- इसका इसलावमक गवणत और खग ल-विजञान पर

महतवपणि परभाि पडा था

- बाद म अपन जीिनकाल म बरहमगपत न

ldquoखोडखयाकrdquo वलखी ज एक खग लीय पससतका

(एक छ िी पसतक) थी

- इसम आयिभटट की अिि-रावतर क परतयक वदन की

शरआत परणाली का परय ग वकया गया था

Q31) उततर (c)

सपषटीकरण

- अमीर खसर एक परवसद सफी सोगीतकार कवि

और विदवान थ

- 1318 म उनह ोन पाया वक इस भवम (वहोदसतान) क

हर कषतर म अलग-अलग भािा थी लाहौरी

कशमीरी दवारसमदरी (दवकषणी कनाििक म)

तलोगाना (आोधर परदश म) गजरी (गजरात म)

माबारी (तवमलनाड म ) अििी (पिी उततर परदश

म) और वहोदिी (वदलली क आस-पास क कषतर म)

आवद

- उनह न यह बताया वक सोसकत वकसी भी कषतर स

सोबोवित नही ो थी और किल बराहमण ही इस भािा

का जञान रखत थ

Q32) उततर (c)

सपषटीकरण

- वहरणय-गभि सववणिम गभि क सोदवभित करता ह

- जब बराहमण ो की सहायता स यह अनषठान वकया

जाता था त यह माना जाता था वक बवल दन िाल

का कषवतरय क रप म पनजिनम ह गा

Q33) उततर (d)

सपषटीकरण

- कदमई भवम राजसव पर कर क सोदवभित करता

- गवावलयर परशससत म नागभि क दवारा वकय गए

श िण का िणिन वकया गया ह |

- नागभि एक परवतहार राजा था

Q34) उततर (b)

सपषटीकरण

- राजतरो वगनी 12िी ो शताबदी म कलहन क दवारा

रवचत एक सोसकत पसतक (िकसट) ह

- यह परारसमभक भारत की ऐवतहावसक इवतितत थी

- तकि सोगत रप स इस अपन परकार की सिोततम

और सिािविक विशवसनीय कवत माना जाता ह

- यह कशमीर कषतर क पराचीनतम समय स लकर

उसकी रचना की तारीख तक क समपणि इवतहास

का आचछादन करती ह

Q35) उततर (c)

सपषटीकरण

- गााि की आम सभा क ldquoउरrdquo कहा जाता था

- ldquoउरrdquo म गााि क सभी कर दन िाल वनिासी

शावमल ह त थ

Q36) उततर (a)

सपषटीकरण

- वदलली सलतनत म ldquoतारीखrdquo इवतहास लखन का

एक रप था

- ldquoतािरीखrdquo क लखक विदवान परि ह त थ वजनम

सवचि परशासक इतयावद शावमल थ

Q37) उततर (a)

सपषटीकरण

- अलाउददीन सखलजी अपन सवनक ो क ितन का

भगतान नकद म करता था न वक इकता क रप

- सवनक अपना सामान वदलली म वयापाररय ो स

खरीदत थ अतः इस बात का भय था वक वयापारी

कही ो िसतओो का मलय न बिा द

- इसकी र कथाम क वलए अलाउददीन सखलजी न

वदलली म कीमत ो क वनयसित वकया

FC19H1003 34

- अविकारीगण धयानपििक मलय ो का सिकषण करत

थ तथा ज वयापारी वनिािररत मलय पर माल नही ो

बचत थ उनक दसणडत वकया जाता था

Q38) उततर (d)

सपषटीकरण

- वदलली सििपरथम त मर राजपत ो क अिीन उनक

सामराजय की राजिानी बनी थी

- 12िी ो शताबदी क मधय म अजमर क चौहान ो

(वजनह चाहमान ो क नाम स भी जाना जाता ह) न

त मर राजपत ो क परावजत वकया था

- त मर ो और चौहान ो क अिीन वदलली एक

महतवपणि िावणसजयक क दर बन गया था

- कई जन वयापारी यहाा रहन लग थ और उनह ोन

कई मोवदर भी बनिाए

- यहाा पर मवदरत वसक वजनह ldquoदहलीिालrdquo क नाम

स जाना जाता था वयापक रप स परचलन म थ

Q39) उततर (c)

सपषटीकरण

- म ठ की मसिद का वनमािण वसको दर ल दी क

राजयकाल म उसक मिी क दवारा करिाया गया

था

- बगमपरी मसिद का वनमािण महममद तगलक क

शासनकाल म हआ था

- यह मसिद विशव का पणयथथान (The

Sanctuary of the World) और वदलली म महममद

तगलक की नई राजिानी जहाोपनाह की मखय

मसिद थी

- कववत- अल - इसलाम मसिद का विसतार

इलतसिश और अलाउददीन सखलजी न वकया था

- मीनार का वनमािण तीन सलतान ो कतबददीन ऐबक

इलतसिश और वफर ज शाह तगलक क दवारा

करिाया गया था

Q40) उततर (c)

सपषटीकरण

- मगल ो क अिीन मनसबदार शबद उस वयसकत क

वलए सोदवभित वकया जाता था वजसक पास मनसब

(अथाित पद) ह ता था

- उस अपना ितन राजसव कायो वजनह जागीर कहत

थ क रप म परापत ह ता था

Q41) उततर (b)

सपषटीकरण

- ldquoभारत छ ड आोद लनrdquo वबरविश शासन क

सखलाफ ल ग ो का एक सवाभाविक विदर ह था

- असखल भारतीय काोगरस सवमवत न 8 अगसत 1942

क बमबई म एक बठक का आय जन वकया था

- इस बठक म परवसद सोकलप ldquoभारत छ ड rdquo क

पाररत वकया गया और इस उददशय क परापत करन

क वलए गाोिी क नततव म एक अवहोसक जन सोघिि

आोद लन की शरआत का परसताि वदया गया

- लवकन अगल ही वदन गाोिी और काोगरस क अनय

परमख नताओो क वगरफतार कर वलया गया

- काोगरस क एक बार वफर अिि घ वित वकया गया

था

Q42) उततर (c)

सपषटीकरण

- साइमन कमीशन यनाइविड वको गडम क सात

साोसद ो का एक समह था

- इस वबरविश भारत क वलए सोििावनक सिार ो का

सझाि दन क वलए गवठत वकया गया था

- इस आय ग म िररषठ वबरविश राजनता सर जॉन

साइमन क नततव म किल वबरविश सदसय ही

शावमल थ

- इसवलए भारत क ल ग ो न साइमन कमीशन क

आगमन क विरद आोद लन वकया था

Q43) उततर (a)

सपषटीकरण

bull दादा भाई नौर जी भारत म वबरविश शासन क

आवथिक पररणाम ो क बार म अपनी विर िी

(परवतकल) राय क वलए जान जात थ

FC19H1003 35

bull अपन कई लख ो और भािण ो म विशि रप स

ldquoपाििी एो ड अन-वबरविश रल इन इसणडया

(Poverty and Un-British Rule in India) म

नौर जी न यह तकि वदया वक भारत पर अतयविक

कर लगाया गया था और इसकी सोपवतत इोगलड की

ओर परिावहत की जा रही थी

bull उनह ोन पराचीन भारतीय गरोथ ो की वयाखया करन

का और भारतीय ो क आिविशवास क बहाल

करन पर कायि नही ो वकया था

उनह ोन वकसी और बात स पहल सभी सामावजक

बराइय ो क उनमलन की आिशयकता पर भी बल

नही ो वदया था

Q44) उततर (c)

सपषटीकरण

bull अगसत 1932 म वबरविश परिानमोतरी मकड नालड न

अपन साोपरदावयक परसकार (The Communal

Award) की घ िणा की थी

bull यह भारत क कई साोपरदावयक वहत ो क बीच विवभनन

सोघिो क हल करन क वलए वबरिन का एकतरफा

परयास था

bull यह परसकार (Award) बाद म 1935 क

अविवनयम (The Act of 1935) म शावमल वकया

गया था

bull इस साोपरदावयक परसकार न मससलम ो क वलए

आरवकषत एक अलग वनिािचक मणडल फॉमिल का

विसतार अनय अलपसोखयक ो क वलए वकया था

वजसम वसख ो भारतीय ईसाइय ो आोगल-भारतीय

समदाय यर पीय समदाय तथा विवशषट कषतरीय

समह ो क शावमल वकया गया था

bull गाोिी न इस परसताि क भारतीय समाज क

विभावजत करन क वलए एक घवणत वबरविश

सावजश क रप म दखा और उसक सखलाफ

आमरण अनशन वकया

Q45) उततर (b)

सपषटीकरण

मौजदा आयात और वनयाित क अवतररक़त

औपवनिवशक भारत क वनमनवलसखत खचो क

वलए एक विशिवनवशचत िन रावश भी दनी पडती

थी

(i) परशासन क वयय

(ii) सना क रख-रखाि क वयय

(iii) यद क वयय

(iv) सिावनितत अविकाररय ो की पशन तथा

(v) वबरिन दवारा अपनी उपवनिश बसती

(कॉल नी) क रख-रखाि क वयय

इनह गह शलक (Home Charges) क रप म

जाना जाता था और लगभग परी तरह स भारत क

दवारा इनका भगतान वकया जाता था

bull गह शलक म वनमनवलसखत घिक शावमल थ

(i) भारतीय ऋण पर दय बयाज

(ii) ईसट इोवडया को पनी क शयरिारक ो क

लाभाोश

(iii) लोदन म भारत कायािलय चलान क वलए िन

(iv) भारत म वनयकत वबरविश कवमिय ो क ितन

और पशन का भगतान करन क वलए िन

(v) रलि पर बयाज

(vi) नागररक और सनय शलक

(vii) इोगलड म सट र (सामगरी) की खरीद

Q46) उततर (b)

सपषटीकरण

bull भारतीय राषटर ीय काोगरस का लाहौर सतर 1929 म

जिाहरलाल नहर की अधयकषता म आय वजत

वकया गया था

bull इस सतर म भारतीय राषटर ीय आोद लन स समबसित

कई महतवपणि पररणाम सामन आय थ

(i) सििपरथम इस सतर म काोगरस क अधयकष पद

पर जिाहरलाल नहर क चना गया था ज

काोगरस म िामपोवथय ो की बिती हई ताकत

का सपषट सोकत था

(ii) दसरा इस सतर म पहली बार काोगरस न पणि

सवतोतरता की माोग क उठाया था

इस परकार की माोग काोगरस मोच स पहल कभी भी

नही ो उठाई गई थी

Q47) उततर (b)

सपषटीकरण

FC19H1003 36

bull इस ररप िि न वकसी भी समदाय क वलए पथक

वनिािचक मोडल अथिा अलपसोखयक ो क वलए

भाराोश की वसफाररश नही ो की थी

bull तथावप इस ररप िि न उन पराोत ो म अलपसोखयक

सीि ो क आरकषण की अनमवत दी थी जहाा पर कम

स कम दस परवतशत अलपसोखयक ह

bull लवकन यह समदाय क आकार क अनपात म ह ना

चावहए था

bull इस ररप िि म भारत क वलए पणि सवतोतरता क

वलए क ई पराििान नही ो था

Q48) उततर (c)

सपषटीकरण

bull आरो वभक िवदक आयो का िमि मखय रप स

परकवत की पजा और यजञ था

bull परारो वभक आयि िमि परकवत की पजा क समान था

bull िासति म उनक चार ो ओर की शसकतयाा वजनह न

त ि वनयोवतरत कर सकत थ और न ही समझ पाए

थ उनह वदवयता क साथ वनिवशत वकया गया तथा

उनह मादा या नर दिीदिताओो क रप म

परतीकतव वकया गया था

bull उनह ोन कछ यजञ ो का भी वनषपादन वकया था

Q49) उततर (b)

सपषटीकरण

bull सडक और नदी-मागि (जल-मागि) डकती स

सरवकषत नही ो थ

bull उललखनीय ह वक हिििििन क शासनकाल क

दौरान यआन चिाोग (हयएन साोग) का सारा

सामान लि वलया गया था

Q50) उततर (c)

सपषटीकरण

परशन म वदए गए द न ो कथन सही ह

Q51) उततर (b)

सपषटीकरण

bull परोदर दास एक सोत और भगिान कषण क एक

महान भकत थ

bull परोदर दास क कनाििक सोगीत क वपतामह क

रप म जाना जाता ह

bull यदयवप उनक जनम-थथान क बार म काफी

अिकल लगाई जाती रही ह

bull तथावप अब कननड विशवविदयालय हमपी क दवारा

गवठत एक विशिजञ सवमवत इस वनषकिि पर पहोची

ह वक उनका जनम थथान सोभितया कनाििक का

एक छ िा-सा गााि कषमपरा (वशिम गगा वजला)

था

Q52) उततर (c)

सपषटीकरण

bull शरी तयागराज शरी शयाम शासतरी और शरी मथसवामी

दीवकषतर क कनाििक सोगीत की वतरमवति माना

जाता ह

bull उनक कारण ही 18िी ो-19िी ो शताबदी म कनाििक

सोगीत का सववणिम यग आया था

Q53) उततर (d)

सपषटीकरण

bull अभी हाल ही म लौह यगीन-महापािावणक काल

का 2000 ििि पराना एक दलिभ सारक फगस

(Sarcophagus) (पतथर का ताबत) क ललम क

वियर गाोि (क वयलडी क पास वजला क वझक ड

करल राजय) की एक रॉक-कि गफा स ख जा गया

bull यह ताबत वजसम हविय ो क िकड थ खदाई क

दौरान वमला

bull अभी तक इस परकार की दलिभ ख ज करल क

मातर द ही थथान ो स हई ह

bull य द न ो सारक फगी (Sarcophagi) (पतथर क

ताबत) चियर और अथ ली (वजला क वझक ड) क

महापािाण थथल ो स वमल ह

Q54) उततर (a)

सपषटीकरण

FC19H1003 37

दवकषण भारत म महापािाण सोसकवत एक पणि

विकवसत लौह यगीन सोसकवत थी

Q55) उततर (d)

सपषटीकरण

bull च ल पाणडय और करलपतर (चर) इन तीन ो का

उललख अश क क अवभलख ो म वकया गया ह

bull सोभितः य भौवतक सोसकवत क उततर

महापािावणक चरण म थ

Q56) उततर (d)

सपषटीकरण

bull भीमा-क रगाोि की लडाई ततीय आोगल-मराठा

यद का वहससा थी

Q57) उततर (b)

सपषटीकरण

bull राजकमार शकल न गाोिीजी क चोपारण आन तथा

वतनकवथया परणाली स जडी समसया की जाोच क

वलए रारी करन क वलए दश भर म उनका

अनसरण वकया था

bull बज वकश र राजदर परसाद महादि दसाई और

नरहरी पाररख चोपारण सतयागरह क दौरान गाोिी

जी क सहय गी थ

Q58) उततर (b)

सपषटीकरण

bull बराहमण ो और बौद मठिाररय ो क कर-मकत गााि

अनदान म दन की परथा सतिाहन ो न आरमभ की

थी

Q59) उततर (c)

सपषटीकरण

इस कायिकरम क उददशय वनमनानसार ह

(i) बवनयादी पयििन आिाररक सोरचना का विकास

करना

(ii) चयवनत (पहचान वकय गए) कषतर ो म आजीविका क

सजन क वलए दश क साोसकवतक और विरासत

मलय ो क बिािा दना

(iii) विरासत समारक थथल ो पर विशव सतरीय आिाररक

सोरचना विकवसत करक एक सतत तरीक स

पयििक आकििण म िसद करना

(iv) थथानीय समदाय ो की सवकरय भागीदारी क माधयम

स र रगार ो का सजन करना

(v) र रगार उतपादन और आवथिक विकास क वलए

पयििन कषमता का उन पर परभाि का उपय ग

करना तथा

(vi) िारणीय पयििन आिाररक सोरचना का विकास

करना और उसका उवचत सोचालन तथा

रखरखाि सवनवशचत करना

Q60) उततर (b)

सपषटीकरण

bull यह वनकाय ििि 1987 म अससततव म आया था

bull यह एक राषटर ीय सतर का शीिि सोगठन ह ज भारत

सरकार क जनजातीय मामल ो क मोतरालय क

परशासवनक वनयोतरण क अिीन काम कर रहा ह

bull इसका पोजीकत और परिान कायािलय नई वदलली

म सथथत ह

Q61) उततर (c)

सपषटीकरण

bull परमचोद क उपनयास ो म परमाशरम रोगभवम गबन

कमिभवम और ग दान शावमल ह

bull ग रा रिी ोदरनाथ िग र क दवारा रवचत उपनयास ह

bull अभी हाल ही म मोशी परमचोद की 138िी ो जयोती दश

भर म मनाई गई थी

Q62) उततर (b)

सपषटीकरण

bull ldquoवगदाrdquo पोजाब (भारत) एिो पावकसतान की

मवहलाओो क दवारा तयौहार क समय और फसल

की बिाई तथा किाई क अिसर पर वकया जान

िाला एक पारोपररक दहाती नतय ह

FC19H1003 38

bull इस नतय क माधयम स पोजाबी मवहलाऐो अपनी

परसननता परकि करती ह तथा वगदा क परदशिन क

माधयम स परि िचिसव िाल समाज म मवहलाओो

की दबी हई भािनाओो क परकि करती ह

bull चोवक इस नतय का परि ो क साथ क ई सोबोि नही ो

ह अतः किल मवहलाऐो ही इसम भाग ल सकती

bull हर साल तीज समार ह क दौरान पोजाब म वगदा

नतय वकया जाता ह

तीज भारत क कछ भाग ो म मवहलाओो क दवारा

मनाया जान िाल कई तयौहार ो क वलए एक

वयापक नाम ह

Q63) उततर (a)

सपषटीकरण

- मजम-उल-बहरीन या द समदर ो का सोगम

नामक उललखनीय रचना दारा वशक ह क दवारा

वलखी थी

- भारत क उपराषटर पवत शरी एम िकया नायड न कहा

ह वक राजकमार दारा वशक ह की रचनाएा शाोवत

और सदभाि क बिािा दन क वलए एक तारा सर त

क रप म सामन आ सकती ो ह

- उपराषटर पवत गत ििो क भला वदए गए राजकमार

दारा वशक ह क परदवशित परचवलत करन हत

आय वजत एक परदशिनी का दौरा करन क बाद एक

सभा क सोब वित कर रह थ

- इस परदशिनी का आय जन फर क इस गौवियर

(Francois Gautier) क दवारा lsquoइोवदरा गाोिी नशनल

सिर फॉर द आििसrsquo (The Indira Gandhi

National Centre for the Arts) नई वदलली म

वकया गया था

Q64) उततर (c)

सपषटीकरण

- ग मतशवर परवतमा जन भगिान बाहबली क

समवपित ह

- यह एक एक-चटटानी पतथर की मवति ह

- राषटर पवत राम नाथ क विोद न शरिणबलग ला

(कनाििक) म आय वजत वकय जान िाल भवय

अवभिक समार ह महामसतकावभिक का

उदघािन वकया था

- यह समार ह 12 ििो म एक बार ह ता ह

Q65) उततर (c)

सपषटीकरण

bull पराची घािी पराची नदी क चार ो ओर फली हई थी

bull पराची घािी िीर-िीर विलपत ह गई थी

bull पराची नदी भिनशवर स वनकलती ह

bull यह महानदी की एक सहायक नदी ह और यह

परी खदाि किक तथा जगतवसोहपर वजल ो स

ह कर बहती ह

bull इस नदी क पर कषतर क पराची घािी कहा जाता ह

bull यह नदी बोगाल की खाडी म वगरती ह

परातासतवक साकषय स पता चलता ह वक पराची घािी

सभयता हडपपा और म हनज दाड द न ो की

पिििती ह

Q66) उततर (d)

सपषटीकरण

य समारक छतरपर वजल (मधय परदश) म विोधयाचल

पिित शरोखला म सथथत ह

Q67) उततर (a)

सपषटीकरण

bull थॉिस ऑन पावकसतान नामक पसतक डॉ बी

आर अमबडकर न वलखी थी

bull डॉ बी आर अमबडकर की जयोती क अिसर पर

भारत क राषटर पवत न भारत की इस महान हसती

क शरदाोजवल अवपित की थी

bull डॉ बी आर अमबडकर न 1924 म वडपरथड

कलावसर इोसटीटयि (दवलत िगि सोथथान -

बवहषकत वहतकाररणी सभा) और 1927 म समाज

समता सोघ की थथापना की थी

bull अमबडकर का धयान वशकषा कषतर की ओर भी था

bull उनह ोन वशकषा क वनमन िगो म फलान क वलए

पीपलस एजकशन स साइिी (The Peoples

Education Society) क नाम स महाविदयालय ो क

नििकि और छातरािास ो की थथापना की थी

FC19H1003 39

Q68) उततर (b)

सपषटीकरण

bull महरगि भारतीय उपमहादवीप म एक परवसद

निपािाण बसती ह ज बलवचसतान पराोत

पावकसतान म सथथत ह

bull दचपलली (आोधर परदश) क पास नागलर नदी क

पिी ति ो पर चना पतथर क बलॉक क विशाल

विसतार म एक पिि-ऐवतहावसक रॉक आिि थथल की

ख ज की गई ह

bull इसन 1500-2000 ईसा पिि क दौरान गोिर (आोधर

परदश) म विकवसत निपािाण सभयता पर परकाश

डाला ह

Q69) उततर (c)

सपषटीकरण

bull 12िी ो सदी और 13िी ो सदी म काकाविय िोश का

उदय हआ था

bull ि पहल कलयाण क पवशचमी चालकय ो क सामोत थ

bull परारोभ म उनह ोन िारोगल (तलोगाना) क पास एक

छ ि स कषतर पर शासन वकया था

bull उनह ोन ldquoनायक वयिथथाrdquo की शरआत की थी

वजस बाद म विजयनगर क राय शासक ो न

अपनाया और विकवसत वकया था

Q70) उततर (a)

सपषटीकरण

bull गाोिीजी क अनशन स वमल मावलक ो पर दबाि

पडा था ज अोततः शरवमक ो क ितन म 35 परवतशत

की िसद करन क वलए सहमत हए थ

bull गगल (Google) न अनसया साराभाई वजनह ोन

भारत क शरवमक आोद लन म एक अगरणी भवमका

वनभाई थी की 132िी ो जयोती डडल (Doodle) का

वनमािण करक मनाई

Q71) उततर (d)

सपषटीकरण

भारत स यनसक की मानिता की अमति साोसकवतक

विरासत की परवतवनवि सची म वनमनवलसखत शावमल ह

bull कवडयटटम करल का सोसकत रोगमोच

bull मवडयिि करल का अनषठान रोगमोच और नतय

नाविका

bull िवदक मि जाप की परोपरा

bull राजथथान क कालबवलया ल क गीत और नतय

bull रामलीला रामायण का पारोपररक परदशिन

bull सोकीतिन मवणपर का अनषठान गायन ढ ल िादन

और नतय

bull रममन भारत क गििाल वहमालय का िावमिक

तयौहार और अनषठान रोगमोच

bull जाोदीयाला गर पोजाब क ठठर ो की पीतल और

ताोब क वशलप स वनवमित बतिन ो की पारोपररक कला

bull छाऊ नतय पिी भारतीय राजय ो म जनमी शासतरीय

भारतीय नतय कला

bull लददाख का बौद मि जाप िर ाोस-वहमालयी लददाख

कषतर तथा जमम-कशमीर म पवितर बौद गरोथ ो का पाठ

bull य ग

bull नौर र

bull को भ मला

Q72) उततर (b)

सपषटीकरण

bull भारत क राषटर पवत शरी राम नाथ क विोद न

वकसामा नागालड म हॉनिवबल मह रति और

राजय गठन वदिस समार ह का उदघािन वकया

था

bull हॉनिवबल मह रति का नाम भारतीय हॉनिवबल क

नाम पर पडा ह ज एक विशाल और रोगीन जोगली

पकषी ह

bull यह पकषी नागालड राजय की अविकतर जनजावतय ो

की ल ककथाओो म उसललसखत ह

bull नागालड की परमख मानयता परापत जनजावतयाा ह

अोगामी आओ चखसोग चाोग ककी रगमा और

रवलोग आवद

bull ओोग जारिा और ससिनलीस अोडमान-वनक बार

दवीप समह की जनजावतयाा ह

FC19H1003 40

Q73) उततर (c)

सपषटीकरण

bull दकन म राषटर कि शासन दसिी ो सदी क अोत तक

लगभग 200 ििो तक रहा था

bull राषटर कि शासक अपन िावमिक विचार ो म सवहषण

bull उनह ोन न किल शि िमि और िषणि िमि बसलक

जन िमि क भी सोरकषण वदया था

bull एल रा म वशि क परवसद रॉक कि मोवदर का

वनमािण नौिी ो सदी म राषटर कि राजा कषण परथम न

करिाया था

bull उसका उततराविकारी अम घििि जन था लवकन

उसन अनय िमो क भी सोरकषण परदान वकया था

bull राषटर कि ो न मसलमान वयापाररय ो क बसन की

अनमवत दी थी

bull उनह न अपन अविराजय ो म इसलाम क उपदश दन

की भी अनमवत दी थी

bull अभी हाल ही म पाोडिलागटटा (तलोगाना) क

परागवतहावसक चटटान वचतर ो क कषरण की बिती हई

घिनाएा एक गोभीर वचोता का वििय ह

bull यह परागवतहावसक चटटान क नकसान पहाचा

सकता ह

bull पाोडिलागटटा वनमनवलसखत क वलए जाना जाता ह

- 10000 ईसा पिि स 8000 ईसा पिि क वचवतरत

चटटानी आशरय ो क वलए

- राषटर कि काल क एक 8 िी ो सदी क

वशलालख क वलए और

- 12िी ो सदी क काकविय सामराजय क वभवतत

वचतर ो क वलए

Q74) उततर (b)

सपषटीकरण

bull 1828 म राजा राम म हन रॉय न एक नय िावमिक

समाज बरहम सभा की थथापना की थी वजस बाद

म बरहम समाज क नाम स जाना गया था

bull दिदरनाथ िग र न ततवब विनी सभा की अधयकषता

की थी ज आधयासिक सतय की ख ज म सोलि

थी

bull इसका उददशय वहोद िमि क शद करन का और

एकशवरिाद (एक ईशवर म आथथा) का परचार करना

था

bull नय समाज की थथापना क आिार थ कारण

(तकि ) क द सतमभ तथा िद और उपवनिद

bull अभी हाल ही म सािारण बरहम समाज का कछ

काननी मदद ो क लकर पवशचम बोगाल सरकार क

साथ काननी वििाद चल रहा ह

Q75) उततर (c)

सपषटीकरण

bull भारत म वचशती वसलवसल की थथापना खवाजा

म इनददीन वचशती क दवारा की गयी थी

bull ि 1192 ईसवी क आसपास भारत आय थ

bull वचशतीय ो क बारहिी ो शताबदी क उततरािि म भारत

म आन िाल सफीय ो क समह ो म सबस

परभािशाली माना जाता ह

bull उनह ोन थथानीय िातािरण क साथ सफलतापििक

अनकलन वकया और उनह ोन भारतीय भसकत

परोपराओो क कई पहलओो क अपनाया

bull अजमर म सफी अपरकि खवाजा म इनददीन वचशती

की ऐवतहावसक दरगाह क एक नया रप दन की

तयारी की जा रही ह

bull इस 13िी ो शताबदी की दरगाह क ldquoसवचछ

आइकॉवनक थथल ोrdquo (Swacch Iconic Places) म

शावमल वकया गया ह ज परवतवषठत विरासत

आधयासिक और साोसकवतक थथान ो पर क वदरत

य जना ह

FC19H1003 41

ANSWERS amp EXPLANATION OF

NCERT History Class VI-X + Current Affairs

(FC19E1003)

Q1) Answer c

Explanation

Rigveda consists of more than a

thousand hymns dedicated to gods and

goddesses These hymns were

composed by sages and learnt by men

however a few were composed by

women like Apala Ghosa Lopamudra

Maitreyi and Gargi

Rigveda consists of many hymns in the

form of dialogues We get an example of

a dialogue between a sage named

Vishwamitra and two rivers (Beas and

Sutlej) that were worshipped as

goddesses This suggests that he

belonged to the Vedic period

Q2) Answer b

Explanation

Traces of ash have been found from

Kurnool Caves suggesting that people

were familiar with the use of fire

It is situated in Andhra Pradesh

Q3) Answer c

Explanation

Burzahom is a prehistoric site in

present day Kashmir where people built

pit houses which were dug into the

ground with steps leading into them

These may have provided shelter in cold

weather

Q4) Answer c

Explanation

Epigraphy is defined as the study of

inscriptions

Manuscriptology is the study of history

and literature through the use of hand

written documents

Palaeography refers to the study of

ancient writing systems and the

deciphering and dating of historical

manuscripts

Numismatics refers to the study of

coins

Q5) Answer a

Explanation

Charaka Samhita was written by

Charaka and is an important book on

Ayurveda and medicine

He was a practitioner of the traditional

system of Indian medicine known as

Ayurveda

Charaka is thought to have flourished

sometime between the 2nd century BCE

and the 2nd century CE

Q6) Answer b

Explanation

Bhaga refers to the tax on crops which

was fixed at 16th of the production

Kammakaras is the term used for the

landless agricultural labour class

Ashvamedha also known as horse

sacrifice is a ritual where a horse is let

loose to wander freely and it was

guarded by the rajarsquos men

Q7) Answer (d)

Explanation

In the Rigvedic period horses were

yoked to chariots that were used in

battles fought to capture land cattle

etc This suggests that the use of horse

chariots began much before the period

of Mahajanapadas

The battles were fought in the Rigvedic

period for cattlersquos lands water an even

to capture people Most men took part

in these wars however there was no

regular army but there were assemblies

where people met and discussed

matters of war Regular armies became

a feature in the Mjahajanapada period

including vast armies of foot soldiers

chariots and elephants

RAUSIAS-FC19E1003 42

Q8) Answer (a)

Explanation

Buddha belonged to the Sakya clan and

passed away at Kusinara

Buddha taught in Prakrit which was the

common language of people

Q9) Answer c

Explanation

There were six schools of philosophy in

ancient India These are known as

Vaishesika Nyaya Samkhya Yoga

Purva Mimansa and Vedanata or Uttara

Mimansa They were founded by sages

Kanada Gautama Kapila Patanjali

Jamini and Vyasa respectively

Q10) Answer b

Explanation

The teachings of Mahavira were

compiled at Valabhi in 6th century AD

Q11) Answer (c)

Explanation

Chanakya is traditionally identified as

Kautilya or Vishnugupta who authored

the ancient Indian political treatise the

Arthashastra

Q12) Answer d

The national emblem of India is an

adaptation of the Lion Capital atop the

Ashoka Pillar of Sarnath Uttar Pradesh

and is combined with the National

Motto Satyameva Jayate

The Rampurva Bull gets the name from

the site of its discovery Rampurva in

Bihar

It is noted for its delicately sculpted

model demonstrating superior

representation of soft flesh sensitive

nostrils alert ears and strong legs It is

a mixture of Indian and Persian

elements

Sankissa is situated in Uttar Pradesh

India

Q13) Ans(a)

Kunwar Singh was a notable leader during the Revolt of 1857 He belonged

to a royal house of Jagdispur Bihar

Q14) Answer b

Explanation

The term Vellalar was used for large

landowners

Q15) Answer c

Explanation

Arikamedu was a coastal settlement

where ships unloaded goods from

distant lands Finds here include a

massive brick warehouse pottery

including amphorae and Arretine ware

Roman lamps glassware and gems have

also been found at the site

Q16) Answer a

Explanation

Muvendar is a Tamil word mentioned in

Sangam poems meaning three chiefs

used for the heads of three ruling

families the Cholas Cheras and

Pandyas

Q17) Ans (c)

Several tribal or kin-based assemblies

such as the Sabha Vidatha and gana

are mentioned in the Rig-veda The

Sabha and the samiti mattered a great

deal in early Vedic times so much so

that the chiefs or the kings showed an

eagerness to win their support

Q18) Ans (a)

Jainism recognised the existence of the

gods but placed them lower than the

jina and did not condemn the varna

system as Buddhism did

Q19) Answer (d)

Explanation

Cholas and Pandyas had developed

powerful coastal cities The most

important city of Cholas was Puhar or

Kaveripattinam and Madurai was the

capital of Pandyas

Q20) Answer b

Explanation

Buddhacharita is the biography of

Buddha and was written by

RAUSIAS-FC19E1003 43

Ashvaghosha

Q21) Answer (a)

Explanation

Tamil poet Appar was a Shiva devotee

So he was a Nayanar saint

Q22) Answer d

Explanation

Samudragupta was a prominent Gupta

ruler whose coins depict him playing a

veena indicating his love for music We

get important historic information from

his Allahabad Prashasti which was

composed by his court poet Harisena

Q23) Answer (b)

Explanation

Vikrama Samvat was founded by

Chandragupta II in the 58 BC as a

mark of victory over the Shakas and

assumed the title of Vikramaditya

Banabhatta wrote Harshavardhanarsquos

biography the Harshacharita in

Sanskrit

Q24) Answer c

Explanation

Sandhi-vigrahika was the minister of

war and peace

Sarthavaha was the leader of the

merchant caravans

Q25) Answer a

Explanation

Xuan Zang (Hsuan-tsang) was a

Chinese traveller who came during the

reign of Harshavardhana

In the decade that began in 630 AD

Xuan Zang came to India through

Kashmir after visiting Central Asia Iran

and Afghanistan

He travelled from north to east and lived

in Bihar for a couple of years

At Nalanda University Xuan Zang

interacted with students and scholars

mastered local languages and

discovered Buddhist stupas

Q26) Answer c

Explanation

Pradakshina patha is a circular path

laid around a stupa in Buddhist

architecture While the rest are a part of

temple architecture

Q27) Answer d

Explanation

All the above-mentioned temples have

an elaborate use of bricks (baked

bricks) along with stone

Q28) Ans (c)

Muhammad Quli Qutab was the Sultan

of Golconda He was a contemporary of

Akbar was very fond of literature and

architecture

The Sultan was a great poet and he

wrote in Dakhini Urdu Persian and

Telgu and has left an extensive diwan or

collection

Recently the Archaeological Survey of

India (ASI) will be using Ground

Penetrating Radar (GPR) to map the

contours of the area around the Bagh-e-

Naya Qila excavated garden inside the

Golconda Fort in Telangana

Q29) Answer a

Explanation

Silappadikaram is a famous Tamil epic

which was written by Ilango around

1800 years ago It is a story of a

merchant named Kovalan who fell in

love with a courtesan named Madhavi

Manimekalai tells the story of the

daughter of Kovalan and Madhavi

Q30) Answer (a)

Explanation

Charaka is the author of Charaka

Samhita which is an important work of

Ayurveda and medicines

Brahmaguptarsquos fame rests mostly on his

Brahma-sphuta-siddhanta which was

an astronomical work It was translated

into Arabic in Baghdad and had a major

impact on Islamic mathematics and

astronomy

Late in his life Brahmagupta wrote

Khandakhadyaka which was an

RAUSIAS-FC19E1003 44

astronomical handbook that employed

Aryabhatarsquos system of starting each day

at midnight

Q31) Answer (c)

Explanation

Amir Khusrau was a famous sufi

musician poet and scholar In 1318 he

noted that there was different language

in every region of this land (Hindustan)

Lahori Kashmiri Dvarsamudri (in

Southern Karnataka) Telangana (in

Andhra Pradesh) Gujari (in Gujarat)

Marsquobari (in Tamil Nadu) Awadhi (in

eastern Uttar Pradesh) and Hindawai (in

the area around in Delhi) etc He went

to explain that Sanskrit did not belong

to any region and that only brahmans

knew it

Q32) Answer c

Explanation

Hiranyagarbha refers to the golden

womb When this ritual was performed

with the help of Brahmanas it was

thought to lead to the rebirth of the

sacrificer as a Khastriya

Q33) Answer d

Explanation

Kadamai refers to a tax on land

revenue

Gwalior Prashasti describes the exploits

of Nagabhata who was a Pratihara king

Q34) Answer b

Explanation

Rajatarangini is a Sanskrit text written

by Kalhana in the 12th century

It was historical chronicle of early India

It is justifiably considered to be the best

and most authentic work of its kind

It covers the entire span of history in

the Kashmir region from the earliest

times to the date of its composition

Q35) Answer c

Explanation

ldquoUrrdquo was the general assembly of the

village ldquoUrrdquo consisted of all the

taxpaying residents of an ordinary

village

Q36) Answer (a)

Explanation

Tarikh was a form of history writing in

the Delhi Sultanate The authors of

tawarikhs were learned men which

included secretaries administrators etc

Q37 Answer (a)

Explanation

Alauddin chose to pay his soldiers salaries in cash rather than iqtas The soldiers would buy their supplies from merchants in Delhi and it was thus feared that merchants would raise their prices To stop this Alauddin controlled the prices of goods in Delhi Prices were carefully surveyed by officers and merchants who did not sell at the prescribed rates were punished

Q38) Answer (d)

Explanation

Delhi first became the capital of a

kingdom under the Tomara Rajputs

who were defeated in the middle of the

twelfth century by the Chauhans (also

referred to as Chahamanas) of Ajmer

It was under the Tomaras and

Chauhans that Delhi became an

important commercial centre Many rich

Jaina merchants lived in the city and

constructed several temples Coins

minted here called dehliwal had a wide

circulation

Q39) Answer (c)

Explanation

Moth ki Masjid was built in the reign of

Sikandar Lodi by his minister

Begumpuri mosque built in the reign of

Muhammad Tughluq was the main

mosque of Jahanpanah the ldquoSanctuary

of the Worldrdquo and his new capital in

Delhi

Quwwat al ndash Islam mosque was

enlarged by Iltutmish and Alauddin

Khalji The minar was built by three

Sultansndash Qutbuddin Aybak Iltutmish

and Firuz Shah Tughluq

RAUSIAS-FC19E1003 45

Q40) Answer (c)

Explanation

Under the Mughals mansabdar was

referred to an individual who held a

mansab ie rank and he received his

salary as revenue assignments called

jagirs

Q41) Ans (b)

The Quit India Movement was a

spontaneous revolt of people against

British rule

The All India Congress Committee met

at Bombay on 8 August 1942 It passed

the famous resolution Quit India and

proposed the starting of a non-violent

mass struggle under Gandhis

leadership to achieve this aim But on

the very next day Gandhi and other

eminent leaders of the Congress were

arrested The Congress was once again

declared illegal

Q42) Ans (c)

The Simon Commission refers to a

group of seven MPs from the United

Kingdom constituted to suggest

constitutional reforms for British India

The Commission consisted of only

British members headed by one of the

senior British politicians Sir John

Simon

So the people of India agitated against

the arrival of Simon Commission

Q43) Ans (a)

He was widely known for his

unfavourable opinion of the economic

consequences of the British rule in

India

In his many writings and speeches and

especially in Poverty and Un-British

Rule in India Naoroji argued that India

was too highly taxed and that its wealth

was being drained away to England

He did not interpret the ancient Indian

texts and restored the self-confidence of

Indians And also he did not stress the

need for eradication of all the social

evils before anything else

Q44) Ans (c)

In August 1932 Prime Minister

MacDonald announced his Communal

Award Great Britainrsquos unilateral

attempt to resolve the various conflicts

among Indiarsquos many communal

interests

The award which was later

incorporated into the act of 1935

expanded the separate-electorate

formula reserved for Muslims to other

minorities including Sikhs Indian

Christians Anglo-Indians Europeans

distinct regional groups Gandhi

undertook a ldquofast unto deathrdquo against

that offer which he viewed as a

nefarious British plot to divide the

Indian society

Q45) Ans (b)

In British India apart from existing

imports and exports there was also a

particular amount of money which

colonial India contributed towards

administration maintenance of the

army war expenses pensions to retired

officers and other expenses accrued by

Britain towards maintenance of her

colony These were known as Home

charges and were paid for almost

entirely by India

The Home charges was made of

following components-

- Interest payable on Indian debt

- Dividend to shareholders of East

India Company

- Funds used to support the India

Office in London

- Funds used to pay salaries and

pensions of British personnel

engaged in India

- Interest on the railways

- Civil and military charges

- Store purchases in England

Q46) Ans (b)

The Lahore session of the Indian

National Congress was held in 1929

under the Presidentship of Jawaharlal

Nehru

The Lahore session of the Indian

National Congress witnessed significant

RAUSIAS-FC19E1003 46

developments in the Indian national

movement

- First the election of Jawaharlal

Nehru to the post of Presidentship of

the Congress was a clear indication

of the growing strength of the

Leftists in the Congress

- Secondly it was in this session that

the Congress for the first time raised

the demand for complete

independence Such demand was

not raised from the Congress

platform earlier

Q47) Ans (b)

It did not provide for separate

electorates for any community or

weightage for minorities However it did

allow for the reservation of minority

seats in provinces having minorities of

at least ten per cent but this was to be

in strict proportion to the size of the

community

There was no provision for complete

Independence for India

Q48) Ans (c)

The religion of early Vedic Aryans was

primarily of worship of nature and

Yajnas

The early Aryan religion was kind of

nature worship Actually the forces

around them which they could not

control or understand were invested

with divinity and were personified as

male or female gods And they

performed some Yajnas also

Q49) Ans (b)

The roads and river-routes were not

immune from robbery It is notable that

Yuan Chwang (Hiuen Tsang) was

robbed of his belongings during

Harshvardanarsquos period

Q50) Ans (c)

Q51) Ans (b)

Purandara Dasa was a saint and great

devotee of Lord Krishna

There is much speculation about where

Purandara Dasa regarded as the

Pitamaha of Carnatic music was born

Recently an expert committee

constituted by the Kannada University

Hampi has come to the conclusion that

Kshemapura Shivamogga district

Karnataka is the birth place of

Purandara Dasa

Q52) Ans (c)

Sri Tyagaraja Sri Shyama Shastry and Sri Muthuswami Dikshitar are considered the trinity of Carnatic music and with them came the golden age in Carnatic music in the 18th-19th

century

Q53) Ans d)

Recently a rare sarcophagus (stone

coffin) which is 2000 years old from the

Iron AgendashMegalithic era was discovered

from a rock-cut cave at Viyur village of

Kollam near Koyilandy in Kozhikode

district Kerala

The coffin containing bone fragments

was found during an excavation ldquoSo

far such a rare finding has been

discovered only from two sites

in Kerala Both these sarcophagi were

recovered from Megalithic sites at

Chevayur and Atholi also in Kozhikode

district

Q54) Ans a)

The megalithic culture in South India was a full-fledged Iron Age culture

Q55) Ans d)

The Cholas Pandyas and Keralaputras

(Cheras) mentioned in Ashokan

inscriptions were probably in the late

megalithic phase of material culture

Q56) Ans d)

Q57) Ans (b)

Raj Kumar Shukla followed Gandhiji all

over the country to persuade him to

come to Champaran to investigate the

problem associated with tinkathia

system

RAUSIAS-FC19E1003 47

Brij Kishore Rajendra Prasad Mahadev

Desai and Narhari Parikh accompanied

Gandhi ji during the Champaran

Satyagraha

Q58) Ans (b)

The Satvahanas started the practice of granting tax-free villages to brahmanas and Buddhist monks

Q59) Ans c)

The objectives of the Programme are

listed as under

- Developing basic tourism

infrastructure

- Promoting cultural and heritage

value of the country to generate

livelihoods in the identified regions

- Enhancing the tourist attractiveness

in a sustainable manner by

developing world-class

infrastructure at the heritage

monument sites

- Creating employment through active

involvement of local communities

- Harnessing tourism potential for its

effects on employment generation

and economic development

- Developing sustainable tourism

infrastructure and ensuring proper

Operations and maintenance

therein

Q60) Ans (b)

The Tribal Cooperative Marketing

Development Federation of India

(TRIFED) came into existence in 1987

It is a national-level apex organization

functioning under the administrative

control of Ministry of Tribal Affairs

Govt of India

TRIFED has its registered and Head

Office located in New Delhi

Q61) Ans (c)

Premchandrsquos novels include

Premashram Rangabhumi Ghaban

Karmabhumi and Godan

Gora is a novel written by Rabindranath

Tagore

138th birth anniversary of Munshi

Premchand was celebrated across the

country

Q62) Ans (b)

Giddha is a traditional pastoral dance

performed by the women of the Punjab

India and Pakistan at festival times

and at the sowing and reaping of the

harvest

By this dance the Punjabi women

reveal their joy expel their suppressed

feelings in a male dominated society

through the performance of Giddha

Since this dance has nothing to do with

men only women can participate in it

During the Teej celebrations Giddha

dance is celebrated in Punjab every

year Teej is a generic name for a

number of festivals that are celebrated

by women in some parts of India

Q63) Ans (a)

Dara Shukoh wrote the remarkable

work called ldquoMajma-ul-Bahrainrdquo or the

ldquoThe confluence of two seasrdquo

The Vice President of India Shri M

Venkaiah Naidu has said that Prince

Dara Shukohrsquos writings can come as a

refreshing source for infusing peace and

harmony He was addressing the

gathering after visiting the exhibition

that showcases the forgotten Prince of

yesteryears Dara Shukoh organized by

Mr Francois Gautier at Indira Gandhi

National Centre for the Arts in New

Delhi

Q64) Ans (c)

The statue Gommateshwara is

dedicated to the Jain God Bahubali

It is a monolithic statue

President Ram Nath Kovind

inaugurated the grand anointing

ceremony mdash Mahamastakabhisheka mdash

held once in 12 years at

Shravanabelagola (Karnataka)

Q65) Ans (c)

Prachi Valley had come up around the

Prachi river Prachi Valley gradually

disappeared

RAUSIAS-FC19E1003 48

The Prachi river originates from

Bhubaneswar

It is a tributary of the Mahanadi and

flows through the districts of Puri

Khurda Cuttack and Jagatsinghpur

and the entire region of the river is

termed as the Prachi Valley

It falls into the Bay of Bengal

Archaeological evidence shows that the

Prachi Valley Civilisation predates both

Harappa and Mohenjo-Daro

The Prachi river originates from

Bhubaneswar

Q66) Ans (d)

These monuments are located in

Chhatarpur district Madhya Pradesh

within Vindhya mountain range

Q67) Ans (a)

The book lsquoThoughts on Pakistanrsquo was

written by Dr BR Ambedkar

On the occasion of the birth anniversary

of Dr BR Ambedkar the president of

India pays homage to this icon of India

In 1924 he founded the Depressed

Classes Institute (Bahishkrit Hitkarini

Sabha) and in 1927 the Samaj Samata

Sangh

Another area of attention for Ambedkar

was education For its spread among

the low classes he set up a network of

colleges by the name of Peoples

Education Society and founded hostels

Q68) Ans(b)

Mehrgarh is a famous Neolithic

settlement in the Indian subcontinent

which is situated in Baluchistan

province Pakistan

A pre-historic rock art site is discovered

in the vast expanse of limestone blocks

on the eastern banks of Naguleru river

near Dachepalli (Andhra Pradesh) It

has thrown light on the Neolithic

civilisation that flourished in Guntur

(Andhra Pradesh) during 1500-2000

BC

Q69) Ans (c)

The 12th and the 13th centuries saw

the emergence of the Kakatiyas They

were at first the feudatories of the

Western Chalukyas of Kalyana Initially

they ruled over a small territory near

Warangal (Telangana)

They introduced Nayakships which was

later adopted and developed by the

Rayas of Vijayanagara

Q70) Ans (a)

The fast had effect of putting pressure

on mill owners who finally agreed to

give the workers a 35 per cent increase

in wages

Google celebrated with a doodle the

132nd birth anniversary of Anasuya

Sarabhai who played a pioneering role

in Indiarsquos labour movement

Q71) Ans (d)

The UNESCOrsquos list of the representative

list of the intangible cultural heritage of

humanity from India are

- Koodiyattam Sanskrit Theatre of

Kerala

- Mudiyettu ritual theatre and dance

drama of Kerala

- Tradition of Vedic Chanting

- Kalbelia folk songs and dances of

Rajasthan

- Ramlila Traditional Performance of

the Ramayana

- Sankirtana ritual singing

drumming and dancing of Manipur

- Ramman religious festival and

ritual theatre of the Garhwal

Himalayas India

- Traditional brass and copper craft of

utensil making among the Thatheras

of Jandiala Guru Punjab India

- Chhau dance classical Indian dance

originated in the eastern Indian

states

- Buddhist chanting of Ladakh

recitation of sacred Buddhist texts

in the trans-Himalayan Ladakh

region Jammu and Kashmir India

- Yoga

- Nouroz

- Kumbh Mela

RAUSIAS-FC19E1003 49

Q72) Ans(b)

The President of India Shri Ram Nath Kovind inaugurated the Hornbill Festival and State Formation Day celebrations of Nagaland in Kisama

The festival is named after the Indian hornbill the large and colourful forest bird which is displayed in the folklore of most of the states tribes

The major recognized tribes of Nagaland are Angami Ao Chakhesang Chang

Kuki Rengma and Zeling etc

Onge Jarawa and Sentinelese are the

tribes of Andman amp Nicobar Islands

Q73) Ans (c)

The Rashtrakutas rule in the Deccan lasted for almost two hundred years till the end of the tenth century The Rashtrakutas rulers were tolerant in their religious views and patronized not only Shaivism and Vaishnavism but

Jainism as well

The famous rock-cut temple of Shiva at Ellora was built by one of the Rashtrakutas kings Krishna I in the ninth century His successor Amoghavarsha was a Jain but he also

patronized other faiths

The Rashtrakutas allowed Muslims traders to settle and permitted Islam to

be preached in their dominions

Recently increasing defacement at the prehistoric rock paintings of Pandavulagutta Telangana has created a cause for grave concern It can spoil

the prehistoric rock

Pandavulagutta is home to

- Painted rock shelters dating to

10000 BC-8000 BC

- An 8th century inscription of the

Rashtrakuta period and

- Painted frescoes from the 12th century Kakatiya empire

Q74) Ans (b)

In 1828 Raja Ram Mohan Roy founded a new religious society the Brahma Sabha later known as the Brahmo

Samaj

Debendranath Tagore headed the Tattvabodhini Sabha which was

engaged in search of spiritual truth

Its purpose was to purify Hinduism and to preach monotheism or belief in one God

The new society was to be based on the twin pillars of reason and the Vedas and

Upanishads

Recently Sadharan Brahmo Samaj (SBS) has entered into a legal battle with the West Bengal government due

to some legal issue

Q75) Ans (c)

The Chishti order was established in India by Khwaja Moinuddin Chishti who came to India around 1192 The Chishtirsquos are considered to be the most influential of the groups of Sufis who migrated to India in the late twelfth century They adapted successfully to the local environment and adopted several features of Indian devotional

traditions

The historical dargah of Sufi mystic Khwaja Moinuddin Chishti in Ajmer is all set to get a facelift This 13 th century dargah has been included among the Swachh Iconic Places a clean-up initiative focused on iconic

heritage spiritual and cultural places

Page 4: GENERAL STUDIES (PAPER I) · Test is part of Rau’s IAS Test series for Preliminary Exam 2019 FOUNDATION + CURRENT AFFAIRS GENERAL STUDIES (PAPER –I) FOUNDATION TEST –III TOPIC:

RAUSIAS-FC19E1003 4

Q8) बदध क जीिि क सोदभण म निमननिखित कथि ो पर

निचार कीनजए

1 बदध ि िारारसी क पास सारिाथ म अपिा

पहिा उपदश नदया था

2 ि निचछिी कि स सोबोनरत थ और कशीिारा म

उिका निरि हआ था

उपयणकत कथि ो म स कौि-सास सही हह

(a) किि 1

(b) किि 2

(c) 1 और 2 द ि ो

(d) ि त 1 ि ही 2

Q9) निमननिखित यग ो पर निचार कीनजए

1 समखया कनपि

2 िशनिक जानमिी

3 नयाय गौतम

उपयणकत यग ो म स कौि-स सही समनित ह

(a) किि 1 और 2

(b) किि 2 और 3

(c) किि 1 और 3

(d) 1 2 और 3

Q10) निमननिखित कथि ो पर निचार कीनजए

1 महािीर की नशकषाऐो िशािी म सोकनित की गई

थी ो

2 नििय नपटक बौदध सोघ क बार म ह

उपयणकत कथि ो म स कौि-सास सही हह

(a) किि 1

(b) किि 2

(c) 1 और 2 द ि ो

(d) ि त 1 ि ही 2

Q11) निमननिखित कथि ो पर निचार कीनजए

1 अथणशासतर का सोकिि निषणगपत ि नकया था

2 मगसथिीस ज चनदरगपत मौयण क दरबार म

आया था सलयकस का राजदत था

उपयणकत कथि ो म स कौि-सास सही हह

(a) किि 1

(b) किि 2

(c) 1 और 2 द ि ो

(d) ि त 1 ि ही 2

Q12) निमननिखित यग ो पर निचार कीनजए

सतभ कपिटल सथान

1 नसोह कनपटि सारिाथ

2 बि कनपटि रामपिण

3 हाथी कनपटि सोनकससा

उपयणकत यग ो म स कौि-सास सही समनित हह

(a) किि 1 और 2

(b) किि 3

(c) किि 1 और 3

(d) 1 2 और 3

Q13) 1857 क नसपाही निदर ह क सोदभण म निमननिखित

कथि ो म स कौि-सास सही हह

1 अहमदललाह शाह फजाबाद क मौििी ि

अोगरज ो क निरदध ि ग ो क एकनतरत नकया

2 किर नसोह ज एक महाि य दधा थ उततर परदश

स सोबोनरत थ

िीच नदए गए कट का परय ग कर सही उततर चनिए

(a) किि 1

(b) किि 2

(c) 1 और 2 द ि ो

(d) ि त 1 ि ही 2

RAUSIAS-FC19E1003 5

Q8) In the context of the life of Buddha

consider the following statements

1 Buddha gave his first sermon at

Sarnath near Varanasi

2 He belonged to the Licchavi clan

and passed away at Kusinara

Which of the above statements isare

correct

(a) 1 only

(b) 2 only

(c) Both 1 and 2

(d) Neither 1 nor 2

Q9) Consider the following pairs

1 Samkhya Kapila

2 Vaisheshika Jamini

3 Nyaya Gautama

Which of the pairs given above isare

correctly matched

(a) 1 and 2 only

(b) 2 and 3 only

(c) 1 and 3 only

(d) 1 2 and 3

Q10) Consider the following statements

1 The teachings of Mahavira were

compiled at Vaishali

2 Vinaya Pitaka talks about the

Buddhist Sangha

Which of the statements given above

isare correct

(a) 1 only

(b) 2 only

(c) Both 1 and 2

(d) Neither 1and 2

Q11) Consider the following statements

1 Arthashastra was compiled by

Vishnugupta

2 Magasthenes who came to the

court of Chandragupta Maurya

was the ambassador of Seleucus

Which of the statements given above

isare correct

(a) 1 only

(b) 2 only

(c) Both 1 and 2

(d) Neither 1 nor 2

Q12) Consider the following pairs

Pillar Capital Place

1 Lion Capital Sarnath

2 Bull Capital Rampurva

3 Elephant Capital Sankissa

Which of the pairs given above isare

matched correctly

(a) 1 and 2

(b) 3 only

(c) 1 and 3

(d) 1 2 and 3

Q13) Consider the following statements

regarding the Revolt of 1857

1 Ahmadullah Shah a maulvi from

Faizabad caught the imagination

of the people and raised a huge

force of supporters against the

Britishers

2 Kunwar Singh was a great fighter

from Uttar Pradesh

Which of the statements given above

isare correct

(a) 1 only

(b) 2 only

(c) Both 1 and 2

(d) Neither 1 nor 2

RAUSIAS-FC19E1003 6

Q14) निमननिखित यग ो पर निचार कीनजए

1 िललािर सनिक

2 आनदमाई दास (गिाम)

3 कदनसयर भनमहीि मजदर

उपयणकत यग ो म स कौि-स सही समनित ह

(a) किि 1 और 2

(b) किि 2 और 3

(c) किि 1 और 3

(d) 1 2 और 3

Q15) निमननिखित सथि ो म स उस सथि की पहचाि कीनजए

ज एक परनसदध तटीय बसती थी और जहा एरटाइि

(Arretine) मदभाोड पाया गया था

(a) महाबिीपरम

(b) तामरनिखपत

(c) अररकमड

(d) अतरोजीिरा

Q16) सोगम सानहतय क सोदभण म निमननिखित कथि ो पर

निचार कीनजए

1 मदरई म सोगमसभाए आय नजत की जाती ो

थी ो

2 मिनदर शबद का परय ग सोगम कनिताओो म

नकया जाता था नजसका अथण ह द राजाओो

का एक समह

उपयणकत कथि ो म स कौि-सास सही हह

(a) किि 1

(b) किि 2

(c) 1 और 2 द ि ो

(d) ि त 1 ि ही 2

Q17) निमननिखित कथि ो पर निचार कीनजए

1 आरखिक िनदक काि म मनहिाऐो सभाओो

और निदाथाओो म भाग िती थी ो

2 आरखिक िनदक काि म िनशषठ ज एक

पजारी ह ता था की एक महतवपरण भनमका

ह ती थी

उपयणकत कथि ो म स कौि-सास सही हह

(a) किि 1

(b) किि 2

(c) 1 और 2 द ि ो

(d) ि त 1 ि ही 2

Q18) निमननिखित कथि ो पर निचार कीनजए

1 किाणटक म जि रमण क निसतार का शरय

चनदरगपत मौयण क जाता ह

2 जि रमण ि ईशवर क अखसततव क मानयता दी ह

तथा िरण पररािी की भरतणिा की ह

उपयणकत कथि ो म स कौि-सास सही हह

(a) किि 1

(b) किि 2

(c) 1 और 2 द ि ो

(d) ि त 1 ि ही 2

Q19) निमननिखित यग ो पर निचार कीनजए

1 पहार पाणडय

2 मदरई च ि

उपयणकत यग ो म स कौि-सास सही समनित हह

(a) किि 1

(b) किि 2

(c) 1 और 2 द ि ो

(d) ि त 1 ि ही 2

RAUSIAS-FC19E1003 7

Q14) Consider the following pairs

1 Vellalar Soldier

2 Adimai Slaves

3 Kadaisiyar Landless labourers

Which of the pairs given above isare

correctly matched

(a) 1 and 2 only

(b) 2 and 3 only

(c) 1 and 3 only

(d) 1 2 and 3

Q15) Identify the site which was a famous

coastal settlement from which Arretine

ware has been found

(a) Mahabalipuram

(b) Tamralipti

(c) Arikamedu

(d) Atranjikhera

Q16) With respect to the Sangam literature

consider the following statements

1 The Sangamsassemblies were

held at Madurai

2 The term muvendar was used in

sangam poems which mean a

group of two kings

Which of the statements given above

isare correct

(a) 1 only

(b) 2 only

(c) Both 1 and 2

(d) Neither 1 nor 2

Q17) Consider the following statements

1 During Early Vedic Period women

attended the Sabha and Vidatha

2 The priest ndash Vasishtha played

important role in Early Vedic

Period

Which of the statements given above

isare correct

(a) 1 only

(b) 2 only

(c) Both 1 and 2

(d) Neither 1 nor 2

Q18) Consider the following statements

1 The spread of Jainism in

Karnataka is attributed to

Chandragupta Maurya

2 Jainism recognised the existence

of the gods and condemns the

Varna System

Which of the statements given above

isare correct

(a) 1 only

(b) 2 only

(c) Both 1 and 2

(d) Neither 1 nor 2

Q19) Consider the following pairs

1 Puhar Pandyas

2 Madurai Cholas

Which of the pairs given above isare

correct

(a) 1 only

(b) 2 only

(c) Both 1 and 2

(d) Neither 1 nor 2

RAUSIAS-FC19E1003 8

Q20) निमननिखित कथि ो म स कौि-सास सही हह

1 बदधचररत का ििक िागसि ह

2 ब नरसतव की पजा महायाि बौदधमत का एक

महतवपरण भाग थी

िीच नदए गए कट का परय ग कर सही उततर चनिए

(a) किि 1

(b) किि 2

(c) 1 और 2 द ि ो

(d) ि त 1 ि ही 2

Q21) निमननिखित कथि ो म स कौि-सास सही हह

1 भखकत क निचार क भागित गीता म सपषट

नकया गया ह

2 तनमि कनि अपपर एक अििर सोत थ

िीच नदए गए कट का परय ग कर सही उततर चनिएः

(a) किि 1

(b) किि 2

(c) 1 और 2 द ि ो

(d) ि त 1 ि ही 2

Q22) निमननिखित म स नकस शासक ि िीरा बजात हए

अपिी छनि नसक ो पर अोनकत करिाई थी

(a) नमिाोदर

(b) चनदरगपत मौयण

(c) गौतमीपतर सतकरी

(d) समदरगपत

Q23) निमननिखित कथि ो म स कौि-सास सही हह

1 सवतोतर भारत ि निकरम सोित क राषटर ीय

किनडर क रप म अपिाया और यह 68 ईसा

पिण म आरि हआ था

2 बािभटट हिणिरणि क दरबार का एक कनि था

िीच नदए गए कट का परय ग कर सही उततर चनिएः

(a) किि 1

(b) किि 2

(c) 1 और 2 द ि ो

(d) ि त 1 ि ही 2

Q24) निमननिखित यग ो पर निचार कीनजएः

1 सोनर-निगरानहका वयापार मोतरी

2 परथम-कनिका परमि नशलपकार

3 साथणिाह परमि बकर

उपयणकत यग ो म स कौि-सास सही समनित हह

(a) किि 1

(b) किि 1 और 3

(c) किि 2

(d) किि 2 और 3

Q25) निमननिखित कथि ो म स कौि-सास सही हह

1 lsquoएह ि नशिाििrsquo पिकनशि नदवतीय स

सोबोनरत ह और रनिकनत क दवारा इसकी रचिा

की गई थी

2 जआि झाोग समदरगपत नदवतीय क शासिकाि

म भारत आया था

िीच नदए गए कट का परय ग कर सही उततर चनिएः

(a) किि 1

(b) किि 2

(c) 1 और 2 द ि ो

(d) ि त 1 ि ही 2

RAUSIAS-FC19E1003 9

Q20) Which of the following statements

isare correct

1 Buddhacharita is authored by

Nagasena

2 The worship of Bodhisattvas was

an important part of Mahayana

Buddhsim

Select the correct answer using the code

given below

(a) 1 only

(b) 2 only

(c) Both 1 and 2

(d) Neither 1 nor 2

Q21) Which of the following statements

isare correct

1 The idea of Bhakti is elucidated in

Bhagavata Gita

2 Tamil poet Appar was an Alvar

saint

Select the correct answer using the code

given below

(a) 1 only

(b) 2 only

(c) Both 1 and 2

(d) Neither 1 nor 2

Q22) Which of the following rulers had his

image inscribed in the coins while

playing a veena

(a) Meander

(b) Chandragupta Maurya

(c) Gautamiputra Satkarni

(d) Samudragupta

Q23) Which of the following statements

isare correct

1 Vikrama Samvat is adopted as the

national calendar by independent

India and it began in 68 BC

2 Banabhatta was a court poet of

Harshavardhana

Select the correct answer using the code

given below

(a) 1 only

(b) 2 only

(c) Both 1 and 2

(d) Neither 1 nor 2

Q24) Consider the following pairs

1 Sandhi-vigrahika Minister of trade

2 Prathama-kulika Chief craftsman

3 Sarthavaha Chief banker

Which of the pairs given above isare

correct

(a) 1 only

(b) 1 and 3 only

(c) 2 only

(d) 2 and 3 only

Q25) Which of the following statements

isare correct

1 Aihole inscription belongs to

Pulakeshin II and was composed

by Ravikriti

2 Xuan Zang came to India during

the reign of Chandragupta II

Select the correct answer using the code

given below

(a) 1 only

(b) 2 only

(c) Both 1 and 2

(d) Neither 1 nor 2

RAUSIAS-FC19E1003 10

Q26) िासतकिा स सोबोनरत निमननिखित ततव ो म स कौि-स

ततव किि नहोद मखनदर ो की िासतकिा क भाग ह

1 नशिर

2 मणडप

3 परदनकषरा पथ

4 गभणगह

िीच नदए गए कट का परय ग कर सही उततर चनिएः

(a) किि 1 3 और 4

(b) किि 2 3 और 4

(c) किि 1 2 और 4

(d) 1 2 3 और 4

Q27) निमननिखित मोनदर ो म स कौि-स मोनदर ईोट ो स बि ह

1 दिगढ़ मोनदर

2 भीतरगाि मोनदर

3 िकषमर मोनदर नसरपर

4 बहदशवर मोनदर

िीच नदए गए कट का परय ग कर सही उततर चनिएः

(a) किि 1 2 और 3

(b) किि 2 3 और 4

(c) किि 1 3 और 4

(d) 1 2 3 और 4

Q28) निमननिखित कथि ो म स कौि-सास सही हह

1 सलताि महममद किी कतब शाह अकबर का

समकािीि था

2 िासतकिा क कषतर म महममद किी कतब शाह

ि कई ईमारत ो का निमाणर करिाया था नजिम

स चार मीिार सिाणनरक परनसदध ह

िीच नदए गए कट का परय ग कर सही उततर चनिएः

(a) किि 1

(b) किि 2

(c) 1 और 2 द ि ो

(d) ि त 1 ि ही 2

Q29) निमननिखित यग ो पर निचार कीनजएः

1 मनिमकिाई सततिार

2 अनभजञाि शाको तिम कानिदास

3 नसिपपानदकारम क ििि

उपयणकत यग ो म स कौि-सास सही समनित हह

(a) किि 1 और 2

(b) किि 2

(c) किि 1 और 3

(d) 1 2 और 3

Q30) निमननिखित कथि ो म स कौि-सास सही हह

1 ldquoसशरत सोनहताrdquo नचनकरता पर एक महतवपरण

रचिा ह

2 बरहमगपत और चरक महतवपरण गनरतजञ थ

िीच नदए गए कट का परय ग कर सही उततर चनिए

(a) किि 1

(b) किि 2

(c) 1 और 2 द ि ो

(d) ि त 1 ि ही 2

Q31) अमीर िसर क सनदभण म निमननिखित कथि ो म स

कौि-सास सही हह

1 अमीर िसर ि अपिी रचिाओो म नििा ह

नक सोसकत नकसी भी कषतर स सोबोनरत िही ो थी

और किि बराहमर ही इस भािा का जञाि रित

2 उन ोि नहोदिी और अिरी क अखसततव का

उललि नकया था

िीच नदए गए कट का परय ग कर सही उततर चनिए

(a) किि 1

(b) किि 2

(c) 1 और 2 द ि ो

(d) ि त 1 ि ही 2

RAUSIAS-FC19E1003 11

Q26) Which of the following architectural

elements were only part of Hindu

temple architecture

1 Shikhara

2 Mandapa

3 Pradakshina patha

4 Garbhagriha

Select the correct answer using the code

given below

(a) 1 3 and 4 only

(b) 2 3 and 4 only

(c) 1 2 and 4 only

(d) 1 2 3 and 4

Q27) Which of the following temples isare

made of bricks

1 Deogarh Temple

2 Bhitargaon Temple

3 Lakshmana temple Sirpur

4 Brihadeshvara Temple

Select the correct answer using the code

given below

(a) 1 2 and 3 only

(b) 2 3 and 4 only

(c) 1 3 and 4 only

(d) 1 2 3 and 4

Q28) Which of the following statements

isare correct

1 Sultan Muhammad Quli Qutab

Shah was a contemporary of

Akbar

2 In the field of architecture

Muhammad Quli Qutab Shah

constructed many buildings the

most famous of which is the Char

Minar

Select the correct answer using the code

given below

(a) 1 only

(b) 2 only

(c) Both 1 and 2

(d) Neither 1 nor 2

Q29) Consider the following pairs

1 Manimekalai Sattanar

2 Abhijnana Shakuntalam Kalidasa

3 Silappadikaram Kovalan

Which of the pairs given above isare

correct

(a) 1 and 2 only

(b) 2 only

(c) 1 and 3 only

(d) 1 2 and 3

Q30) Which of the following statements

isare correct

1 Sushruta Samhita is an important

work on medicine

2 Brahmagupta and Charaka were

important mathematicians

Select the correct answer using the code

given below

(a) 1 only

(b) 2 only

(c) Both 1 and 2

(d) Neither 1 nor 2

Q31) Which of the following statements

isare correct about Amir Khusrau

1 Amir Khusrau records in his works

that Sanskrit did not belong to any

region and only the Brahmans

knew it

2 He recorded the existence of

Hindawi and Awadhi

Select the correct answer using the code

given below

(a) 1 only

(b) 2 only

(c) Both 1 and 2

(d) Neither 1 nor 2

RAUSIAS-FC19E1003 12

Q32) निमननिखित कथि ो पर निचार कीनजए

1 नहरणय-गभण अिषठाि क बार म ऐसा स चा जाता

था नक बनि दि िाि का एक कषनतरय क रप म

पिजणनम ह गा

2 मयरशमणि कदोब िोश का सोसथापक था

उपयणकत कथि ो म स कौि-सास सही हह

(a) किि 1

(b) किि 2

(c) 1 और 2 द ि ो

(d) ि त 1 ि ही 2

Q33) निमननिखित कथि ो म स कौि-सास सही हह

1 कदमई बगार (बिपिणक शरम) क रप म

निया जाि िािा कर था

2 गवानियर परशखसत म िागभट (ज एक चोदि

राजा था) क दवारा नकय गए श िर का िरणि

नकया गया ह

िीच नदए गए कट का परय ग कर सही उततर चनिए

(a) किि 1

(b) किि 2

(c) 1 और 2 द ि ो

(d) ि त 1 ि ही 2

Q34) निमननिखित कथि ो म स कौि-सास सही हह

1 राजतरो नगिी 11िी ो शताबदी म कलहि क दवारा

रनचत एक सोसकत पसतक (टकसट) ह

2 कननौज क निए नतरपकषीय सोघिण म पाि राजिोश

शानमि था

िीच नदए गए कट का परय ग कर सही उततर चनिए

(a) किि 1

(b) किि 2

(c) 1 और 2 द ि ो

(d) ि त 1 ि ही 2

Q35) निमननिखित यग ो पर निचार कीनजए

1 बरहदशवर मोनदर राजराजा च ि

2 उर मापि की इकाई

3 दिदाि मोनदर ो क भनम अिदाि

उपयणकत यग ो म स कौि-स सही समनित ह

(a) किि 1 और 2

(b) किि 2 और 3

(c) किि 1 और 3

(d) 1 2 और 3

Q36) निमननिखित कथि ो म स कौि-सास सही हह

1 नदलली क सलताि ो क अरीि परशासि की भािा

फारसी थी

2 नदलली सलतित म ldquoतारीितािरीिrdquo कनिता

का एक रप था

िीच नदए गए कट का परय ग कर सही उततर चनिए

(a) किि 1

(b) किि 2

(c) 1 और 2 द ि ो

(d) ि त 1 ि ही 2

Q37) निमननिखित कथि ो म स कौि-सास सही हह

1 अिाउददीि खििजी ि अपि सनिक ो क निए

नसरी िाम का एक िया दगणरकषक शहर

बिािाया था

2 िह अपि सनिक ो क िति का भगताि इकता

क रप म करता था

िीच नदए गए कट का परय ग कर सही उततर चनिए

(a) किि 1

(b) किि 2

(c) 1 और 2 द ि ो

(d) ि त 1 ि ही 2

RAUSIAS-FC19E1003 13

Q32) Consider the following statements

1 Hiranya-garbha ritual was thought

to lead to the rebirth of the

sacrificer as a Kshatriya

2 Mayurasharman was the founder

of the Kadamba dynasty

Which of the statements given above

isare correct

(a) 1 only

(b) 2 only

(c) Both 1 and 2

(d) Neither 1 nor 2

Q33) Which of the following statements

isare correct

1 Kadamai was tax taken in form of

forced labour

2 Gwalior Prashasti describes the

exploits of Nagabhata who was a

Chandella king

Select the correct answer using the code

given below

(a) 1 only

(b) 2 only

(c) Both 1 and 2

(d) Neither 1 nor 2

Q34) Which of the following statements

isare correct

1 Rajatarangini is a Sanskrit text

written by Kalhana in the 11th

century

2 Pala dynasty was included in the

tripartite struggle for Kannauj

Select the correct answer using the code

given below

(a) 1 only

(b) 2 only

(c) Both 1 and 2

(d) Neither 1 nor 2

Q35) Consider the following pairs

1 Brihadeshvara temple Rajaraja

Chola

2 ldquoUrrdquo Unit of measurement

3 Devadana Land grants made to

temples

Which of the pairs given above isare

correct

(a) 1 and 2 only

(b) 2 and 3 only

(c) 1 and 3 only

(d) 1 2 and 3

Q36) Which of the following statements

isare correct

1 The language of administration

under the Delhi Sultans was

Persian

2 Tarikhtawarikh was a form of

poetry in the Delhi Sultanate

Select the correct answer using the code

given below

(a) 1 only

(b) 2 only

(c) Both 1 and 2

(d) Neither 1 nor 2

Q37) Which of the following statements

isare correct

1 Alauddin Khilji constructed a new

garrison town named Siri for his

soldiers

2 He paid his soldiers their salaries

in the form of Iqta

Select the correct answer using the code

given below

(a) 1 only

(b) 2 only

(c) Both 1 and 2

(d) Neither 1 nor 2

RAUSIAS-FC19E1003 14

Q38) निमननिखित कथि ो म स कौि-सास सही हह

1 नदलली कतबददीि एबक क अरीि पहिी बार

नकसी सामराजय की राजरािी बिी थी

2 दहिीिाि नसक ो का मदरर मग़ि ो क दवारा

नकया गया था

िीच नदए गए कट का परय ग कर सही उततर चनिए

(a) किि 1

(b) किि 2

(c) 1 और 2 द ि ो

(d) ि त 1 ि ही 2

Q39) निमननिखित यग ो पर निचार कीनजए

1 म ठ की मखिद नसको दर ि दी

2 बगमपरी मखिद नफर ज शाह तगिक

3 कववत- अि - इसलाम कतबददीि ऐबक

उपयणकत यग ो म स कौि-स सही समनित ह

(a) किि 1 और 2

(b) किि 2 और 3

(c) किि 1 और 3

(d) 1 2 और 3

Q40) निमननिखित कथि ो म स कौि-सास सही हह

1 मिसबदार ो क अपिा िति राजसव कायो

नजन जागीर कहत थ क रप म परापत ह ता

था

2 मिसबदार क ज सनय उततरदानयतव सौोप जात

थ उसक अनतगणत उस एक निराणररत सखया म

सिार अथिा घड़सिार ो का रि-रिाि करिा

पड़ता था

िीच नदए गए कट का परय ग कर सही उततर चनिए

(a) किि 1

(b) किि 2

(c) 1 और 2 द ि ो

(d) ि त 1 ि ही 2

Q41) ldquo1942 क भारत छ ड़ आोद ििrdquo क बार म

निमननिखित अिि कि ो म स कौि-सा सतय िही ो ह

(a) यह एक अनहोसक आोद िि था

(b) इसका िततव महातमा गाोरी क दवारा नकया गया

था

(c) यह एक सवाभानिक आोद िि था

(d) इसि सामानयतया शरनमक िगण क आकनिणत

िही ो नकया था

Q42) भारत क ि ग ो ि ldquoसाइमि कमीशिrdquo क आगमि क

निरदध आोद िि नकया था कय ोनक

(a) भारतीय कभी भी 1919 क अनरनियम (The

Act of 1919) क काम की समीकषा िही ो करिा

चाहत थ

(b) साइमि कमीशि ि पराोत ो म दवर (द हर) शासि

क समापत करि की नसफाररश की थी

(c) साइमि कमीशि म क ई भारतीय सदसय िही ो

था

(d) साइमि कमीशि ि दश क निभाजि का

सझाि नदया था

Q43) निमननिखित कथि ो पर निचार कीनजए

भारतीय राषटर ीय आोद िि म दादाभाई िौर जी क दवारा

नकया गया सबस परभािी य गदाि यह था नक उन ोि

1 अोगरज ो क दवारा भारत क आनथणक श िर का

ििासा नकया था

2 पराचीि भारतीय गरोथ ो की वयाखया की थी और

भारतीय ो क आतमनिशवास क पिःसथानपत नकया

था

3 अनय नकसी भी बात स पहि सभी सामानजक

बराइय ो क उनमिि की आिशयकता पर बि

नदया था

उपयणकत कथि ो म स कौि-सास सही हह

(a) किि 1

(b) किि 2 और 3

(c) किि 1 और 3

(d) 1 2 और 3

RAUSIAS-FC19E1003 15

Q38) Which of the following statements

isare correct

1 Delhi first became the capital of a

kingdom under Qutubuddin

Aibak

2 Dehliwal coins were minted by the

Mughals

Select the correct answer using the code

given below

(a) 1 only

(b) 2 only

(c) Both 1 and 2

(d) Neither 1 nor 2

Q39) Consider the following pairs

1 Moth ki Masjid- Sikander Lodi

2 Begumpuri mosque- Firuz Shah

Tughluq

3 Quwwat al ndash Islam- Qutubuddin

Aibak

Which of the above pairs isare correct

(a) 1 and 2 only

(b) 2 and 3 only

(c) 1 and 3 only

(d) 1 2 and 3

Q40) Which of the following statements

isare correct

1 Mansabdars received their salaries

as revenue assignments called

jagirs

2 The mansabdarrsquos military

responsibilities required him to

maintain a specified number of

sawar or cavalrymen

Select the correct answer using the code

given below

(a) 1 only

(b) 2 only

(c) Both 1 and 2

(d) Neither 1 nor 2

Q41) Which one of the following observations

is not true about the Quit India

Movement of 1942

(a) It was a non-violent movement

(b) It was led by Mahatma Gandhi

(c) It was a spontaneous movement

(d) It did not attract the labour class

in general

Q42) The people of India agitated against the

arrival of the Simon Commission

because

(a) Indians never wanted the review of

the working of the Act of 1919

(b) Simon Commission recommended

the abolition of dyarchy in the

Provinces

(c) there was no Indian member in the

Simon Commission

(d) the Simon Commission suggested

the partition of the country

Q43) Consider the following statements

The most effective contribution made by

Dadabhai Naoroji to the cause of Indian

National Movement was that he-

1 exposed the economic exploitation

of India by the British

2 interpreted the ancient Indian

texts and restored the self-

confidence of Indians

3 stressed the need for eradication of

all the social evils before anything

else

Which of the statements given above

isare correct

(a) 1 only

(b) 2 and 3 only

(c) 1 and 3 only

(d) 1 2 and 3

RAUSIAS-FC19E1003 16

Q44) महातमा गाोरी ि 1932 म आमरर अिशि नकया था

कय ोनक

(a) ldquoग िमज सममििrdquo (The Round Table

Conference) भारतीय राजिीनतक

आकाोकषाओो क परा करि म असफि रहा था

(b) काोगरस और मखसलम िीग म मतभद थ

(c) रामस मकड िालड (Ramsay Macdonald)

ि ldquoसाोपरदानयक परसकारrdquo (The Communal

Award) की घ िरा की थी

(d) ldquoसनििय अिजञा आोद ििrdquo (The Civil

Disobedience Movement) असफि रहा

था

Q45) भारत म औपनििनशक शासि की अिनर क सोदभण म

भारत स रि क बनहगणमि का एक महतवपरण भाग गह

शलक (Home Charges) था निमननिखित म स

कौि-सास क ि गह शलक म सखममनित नकया गया

थानकय गए थ

1 िोदि म भारत कायाणिय क निए उपय ग नकय

जाि िािा क ि

2 भारत म नियकत नबरनटश कनमणय ो क िति और

पशि का भगताि करि क निए उपय ग नकय

जाि िािा क ि

3 अोगरज ो क दवारा भारत क बाहर यदध ो क निए

उपय ग नकय जाि िािा क ि

िीच नदए गए कट का परय ग कर सही उततर चनिए

(a) किि 1

(b) किि 1 और 2

(c) किि 2 और 3

(d) 1 2 और 3

Q46) सवतोतरता आोद िि क इनतहास म भारतीय राषटर ीय

काोगरस का 1929 का सतर महतवपरण ह कय ोनक इसम

(a) काोगरस क उददशय क रप म सथािीय सरकार

की पराखपत की घ िरा की गई थी

(b) परण सवराज की पराखपत क काोगरस क िकषय क

रप म अपिाया गया था

(c) असहय ग आोद िि शर नकया गया था

(d) िोदि म ldquoग ि मर सममििrdquo (The Round

Table Conference) म भाग िि का निरणय

निया गया था

Q47) भारतीय सवतोतरता सोगराम क सोदभण म िहर ररप टण

क दवारा निमननिखित म स नकसकी नसफाररश की गई

थीनकिकी नसफाररश की गई थी ो

1 भारत क निए परण सवतोतरता

2 अलपसोखयक ो क निए सीट ो क आरकषर क

निए सोयकत नििाणचक मोडि

3 सोनिराि म भारत क ि ग ो क निए मौनिक

अनरकार ो का परािराि

िीच नदए गए कट का परय ग कर सही उततर चनिए

(a) किि 1

(b) किि 2 और 3

(c) किि 1 और 3

(d) 1 2 और 3

Q48) आरो नभक िनदक आयो का रमण मखय रप स था

(a) भखकत

(b) मनतण पजा और यजञ

(c) परकनत की पजा और यजञ

(d) परकनत की पजा और भखकत

RAUSIAS-FC19E1003 17

Q44) Mahatma Gandhi undertook fast unto

death in 1932 mainly because

(a) The Round Table Conference failed

to satisfy Indian political

aspirations

(b) The Congress and Muslim League

had differences of opinion

(c) Ramsay Macdonald announced the

Communal Award

(d) The Civil Disobedience Movement

failed

Q45) With reference to the period of colonial

rule in India ldquoHome Chargesrdquo formed

an important part of drain of wealth

from India Which of the following funds

constituted ldquoHome Chargesrdquo

1 Funds used to support the India

Office in London

2 Funds used to pay salaries and

pensions of British personnel

engaged in India

3 Funds used for waging wars

outside India by the British

Select the correct answer using the code

given below

(a) 1 only

(b) 1 and 2 only

(c) 2 and 3 only

(d) 1 2 and 3

Q46) The 1929- Session of Indian National

Congress is of significance in the history

of the Freedom Movement because the-

(a) attainment of Self-Government

was declared as the objective of

the Congress

(b) attainment of Poorna Swaraj was

adopted as the goal of the

Congress

(c) Non-Cooperation Movement was

launched

(d) decision to participate in the

Round Table Conference in

London was taken

Q47) With reference to the period of Indian

freedom struggle which of the following

waswere recommended by the Nehru

report

1 Complete Independence for India

2 Joint electorates for reservation of

seats for minorities

3 Provision of fundamental rights for

the people of India in the

Constitution

Select the correct answer using the code

given below

(a) 1 only

(b) 2 and 3 only

(c) 1 and 3 only

(d) 1 2 and 3

Q48) The religion of the early Vedic Aryans was primarily of

(a) Bhakti

(b) image worship and Yajnas

(c) worship of nature and Yajnas

(d) worship of nature and Bhakti

RAUSIAS-FC19E1003 18

Q49) भारत की यातरा करि िाि चीिी यातरी यआि चिाोग

(हयएि साोग) ि समकािीि भारत की सामानय

खसथनतय ो और सोसकनत क दजण नकया था इस सोदभण म

निमननिखित कथि ो म स कौि-सास सही हह

1 सड़क और िदी-मागण (जि-मागण) डकती स

परण रप स सरनकषत थ

2 जहा तक अपरार ो क निए दणड की बात ह

उसक निए नकसी भी वयखकत की निदोिता

अथिा उसक अपरार क निराणररत करि क

निए अनि जि और निि परि क माधयम क

सारि थ

3 वयापाररय ो क घाट ो और परनतबोर सटशि ो पर

शलक ो का भगताि करिा पड़ता था

िीच नदए गए कट का परय ग कर सही उततर चनिए

(a) किि 1

(b) किि 2 और 3

(c) किि 1 और 3

(d) 1 2 और 3

Q50) नसोर घाटी सभयता क सोदभण म निमननिखित कथि ो पर

निचार कीनजए

1 यह मखय रप स एक रमणनिरपकष सभयता थी

तथा हािाोनक इसम रानमणक ततव मौजद था

िनकि िह परनतिश पर हािी िही ो था

2 इस काि क दौराि भारत म कपास का परय ग

कपड़ा बिाि क निए नकया जाता था

उपयणकत कथि ो म स कौि-सास सही हह

(a) किि 1

(b) किि 2

(c) 1 और 2 द ि ो

(d) ि त 1 ि ही 2

Q51) परोदर दास क सोदभण म निमननिखित कथि ो पर निचार

कीनजए

1 परोदर दास एक सोत और भगिाि नशि क एक

महाि भकत थ

2 ि एक सोगीतकार गायक और किाणटक सोगीत

क मखय सोसथापक-परसतािक ो म स एक थ

उपयणकत कथि ो म स कौि-सास सही हह

(a) किि 1

(b) किि 2

(c) 1 और 2 द ि ो

(d) ि त 1 ि ही 2

Q52) निमननिखित म स कौि-सास वयखकत किाणटक सोगीत

की नतरमनतण म शानमि हह

1 बािामरिी कषणा

2 शरी शयाम शासतरी

3 शरी मथसवामी दीनकषतर

िीच नदए गए कट का परय ग कर सही उततर चनिए

(a) किि 1

(b) किि 2

(c) किि 2 और 3

(d) 1 2 और 3

Q53) चियर (Chevayur) और अथ िी (Atholi) म खसथत

महापािार सथि निमननिखित म स नकस राजय म खसथत

(a) तनमििाड

(b) किाणटक

(c) पनिम बोगाि

(d) करि

RAUSIAS-FC19E1003 19

Q49) The Chinese traveller Yuan Chwang

(Hiuen Tsang) who visited India

recorded the general conditions and

culture of India at that time In this

context which of the following

statements isare correct

1 The roads and river-routes were

completely immune from robbery

2 As regards punishment for

offences ordeals by fire water and

poison were the instruments for

determining the innocence or guilt

of a person

3 The tradesmen had to pay duties

at ferries and barrier stations

Select the correct answer using the code

given below

(a) 1 only

(b) 2 and 3 only

(c) 1 and 3 only

(d) 1 2 and 3

Q50) Regarding the Indus Valley Civilization

consider the following statements

1 It was predominantly a secular

civilization and the religious

element though present did not

dominate the scene

2 During this period cotton was

used for manufacturing textiles in

India

Which of the statements given above

isare correct

(a) 1 only

(b) 2 only

(c) Both 1 and 2

(d) Neither 1 nor 2

Q51) Consider the following statements

regarding Purandara Dasa

1 Purandara Dasa was a saint and

great devotee of Lord Shiva

2 He was a composer singer and

one of the chief founding-

proponents of the Carnatic music

Which of the statements given above

isare correct

(a) 1 only

(b) 2 only

(c) Both 1 and 2

(d) Neither 1 nor 2

Q52) Which of the following persons isare

included in the trinity of Carnatic

music

1 Balamurali Krishna

2 Sri Shyama Shastry

3 Sri Muthuswami Dikshitar

Select the correct answer using the code

given below

(a) 1 only

(b) 2 only

(c) 2 and 3 only

(d) 1 2 and 3

Q53) Megalithic sites at Chevayur and Atholi

are located in which of the following

states

(a) Tamil Nadu

(b) Karnataka

(c) West Bengal

(d) Kerala

RAUSIAS-FC19E1003 20

Q54) निमननिखित कथि ो पर निचार कीनजए

1 महापािानरक ि ग कबर ो म िसतएो दफिात थ

2 दनकषर भारत म महापािार सोसकनत एक परण

निकनसत तामर यगीि सोसकनत थी

उपयणकत कथि ो म स कौि-सास सही हह

(a) किि 1

(b) किि 2

(c) 1 और 2 द ि ो

(d) ि त 1 ि ही 2

Q55) निमननिखित म स कौि-स सामराजयसामराजय ो का

अश क क अनभिि ो म उललि नकया गया ह

1 च ि

2 पाणडय

3 करिपतर (चर)

िीच नदए गए कट का परय ग कर सही उततर चनिए

(a) किि 1

(b) किि 1 और 2

(c) किि 3

(d) 1 2 और 3

Q56) भीमा-क रगाोि का यदध को पिी क सनिक ो और

बाजीराि नदवतीय क िततव म एक शखकतशािी पशिा

सिा (मराठ ो) क मधय िड़ा गया था यह यदध

निमननिखित म स नकसका नहससा था

(a) परथम आोगल-मराठा यदध का

(b) नदवतीय आोगल-मराठा यदध का

(c) ततीय आोगल-मसर यदध का

(d) ततीय आोगल-मराठा यदध का

Q57) निमननिखित कथि ो पर निचार कीनजए

1 महादि दसाई ि गाोरीजी क चोपारर आि तथा

नतिकनथया पररािी स जड़ी समसया की जाोच

क निए रारी करि क निए दश भर म उिका

अिसरर नकया था

2 िरहरी पाररि चोपारर सतयागरह क दौराि

गाोरीजी क साथ थ

उपयणकत कथि ो म स कौि-सास सही हह

(a) किि 1

(b) किि 2

(c) 1 और 2 द ि ो

(d) ि त 1 ि ही 2

Q58) निमननिखित कथि ो पर निचार कीनजए

1 िनद राज-िोश ि बराहमर ो और बौदध मठराररय ो

क कर-मकत गाि अिदाि म दि की परथा

आरि की थी

2 सतिाहि ो की आनरकाररक भािा पराकत थी

उपयणकत कथि ो म स कौि-सास सही हह

(a) किि 1

(b) किि 2

(c) 1 और 2 द ि ो

(d) ि त 1 ि ही 2

Q59) एक निरासत क अपिाइए (अडॉपट ए हररटज ndash

Adopt a Heritage) पररय जिा क उददशय ो क

सनदभण म निमननिखित कथि ो पर निचार कीनजए

1 यह पररय जिा र रगार उतपादि और आनथणक

निकास क निए पयणटि कषमता का उि पर

परभाि का उपय ग करगी

2 यह पररय जिा निरासत सथि ो पर निशव सतरीय

आराररक सोरचिा निकनसत करक एक सतत

तरीक स पयणटक आकिणर म िखदध करगी

उपयणकत कथि ो म स कौि-सास सही हह

(a) किि 1

(b) किि 2

(c) 1 और 2 द ि ो

(d) ि त 1 ि ही 2

RAUSIAS-FC19E1003 21

Q54) Consider the following statements

1 Megalithic people buried goods in

graves

2 The megalithic culture in South

India was a full-fledged Copper

Age culture

Which of the statements given above

isare correct

(a) 1 only

(b) 2 only

(c) Both 1 and 2

(d) Neither 1 nor 2

Q55) Which of the following kingdoms isare

mentioned in the Ashokan inscriptions

1 Cholas

2 Pandyas

3 Keralaputras (Cheras)

Select the correct answer using the code

given below

(a) 1 only

(b) 1 and 2 only

(c) 3 only

(d) 1 2 and 3

Q56) The Battle of Bhima-Koregaon was

fought between the soldiers of the

Company and the strong Peshwa army

(Marathas) under Bajirao II This war

was a part of the

(a) First Anglo-Maratha war

(b) Second Anglo-Maratha war

(c) Third Anglo- Mysore war

(d) Third Anglo-Maratha war

Q57) Consider the following statements

1 Mahadev Desai followed Gandhiji all over the country to persuade him to come to Champaran to investigate the problem associated

with tinkathia system

2 Narhari Parikh accompanied Gandhi ji during the Champaran

Satyagraha

Which of the statements given above isare correct

(a) 1 only

(b) 2 only

(c) Both 1 and 2

(d) Neither 1 nor 2

Q58) Consider the following statements

1 The Nanda Dynasty started the practice of granting tax-free villages to brahmanas and

Buddhist monks

2 The official language of the Satavahanas was Prakrit

Which of the statements given above

isare correct

(a) 1 only

(b) 2 only

(c) Both 1 and 2

(d) Neither 1 nor 2

Q59) Consider the following statements about the objectives of the lsquoadopt a heritagersquo

project

1 It will harness tourism potential for its effects on employment generation and economic

development

2 It will enhance the tourist attractiveness in a sustainable manner by developing world class infrastructure at heritage sites

Which of the statements given above

isare correct

(a) 1 only

(b) 2 only

(c) Both 1 and 2

(d) Neither 1 nor 2

RAUSIAS-FC19E1003 22

Q60) ldquoभारतीय जिजातीय सहकारी निपरि निकास सोघrdquo

(The Tribal Co-operative Marketing

Development Federation of India - TRIFED)

क सोदभण म निमननिखित कथि ो पर निचार कीनजए

1 यह एक राषटर ीय सतर का शीिण सोगठि ह ज

भारत सरकार क गह मोतरािय क परशासनिक

नियोतरर क अरीि काम कर रहा ह

2 इसका मखय उददशय दश म जिजातीय ि ग ो

का सामानजक-आनथणक निकास करिा ह

उपयणकत कथि ो म स कौि-सास सही हह

(a) किि 1

(b) किि 2

(c) 1 और 2 द ि ो

(d) ि त 1 ि ही 2

Q61) निमननिखित म स कौि-सास उपनयास परमचोद क

दवारा नििा गया हनिि गए ह

1 रोगभनम

2 ग दाि

3 ग रा

िीच नदए गए कट का परय ग कर सही उततर चनिए

(a) किि 1

(b) किि 2

(c) किि 1 और 2

(d) 1 2 और 3

Q62) नगदधा ितय क सोदभण म निमननिखित कथि ो पर निचार

कीनजए

1 नगदधा नबहार की मनहिाओो क दवारा तयौहार क

समय और फसि की बिाई तथा कटाई क

अिसर पर नकया जाि िािा एक पारोपररक

दहाती ितय ह

2 इस ितय क दवारा मनहिाऐो अपिी परसननता

परकट करती ह तथा नगदधा क परदशणि क

माधयम स परि िचणसव िाि समाज म

मनहिाओो की दबी हई भाििाओो क परकट

करती ह

उपयणकत कथि ो म स कौि-सास सही हह

(a) किि 1

(b) किि 2

(c) 1 और 2 द ि ो

(d) ि त 1 ि ही 2

Q63) निमननिखित कथि ो पर निचार कीनजए

1 मलला शाह बदखशी दारा नशक ह क

आधयाखतमक गर थ

2 औरोगरब ि मजम-उि-बहरीि या द समदर ो

का सोगम िामक उललििीय रचिा नििी थी

3 दारा नशक ह क अपि पिणज अकबर क गर ो

क उततरानरकारी क रप म दिा गया था

नजसम उसि रानमणक बहििाद और समनवयता

क बढ़ािा नदया था

उपयणकत कथि ो म स कौि-सास सही हह

(a) किि 1 और 3

(b) किि 2

(c) किि 1 और 2

(d) 1 2 और 3

RAUSIAS-FC19E1003 23

Q60) Consider the following statements about

the Tribal Cooperative Marketing

Development Federation of India

(TRIFED)

1 It is a national-level apex

organization functioning under the

administrative control of Ministry

of Home Affairs Government of

India

2 The main objective of TRIFED is

socio-economic development of

tribal people in the country

Which of the statements given above

isare correct

(a) 1 only

(b) 2 only

(c) Both 1 and 2

(d) Neither 1 nor 2

Q61) Which of the following novels isare

written by Premchand

1 Rangabhumi

2 Godan

3 Gora

Select the correct answer using the code

given below

(a) 1 only

(b) 2 only

(c) 1 and 2 only

(d) 1 2 and 3

Q62) Consider the following statements about

Giddha dance

1 Giddha is a traditional pastoral

dance performed by the women of

Bihar at festival times and at the

sowing and reaping of the harvest

2 By this dance the women reveal

their joy expel their suppressed

feelings in a male dominated

society through the performance of

Giddha

Which of the statements given above

isare correct

(a) 1 only

(b) 2 only

(c) Both 1 and 2

(d) Neither 1 nor 2

Q63) Consider the following statements

1 Mullah Shah Badakhshi was the

spiritual mentor of Dara Shukoh

2 Aurangzeb wrote the remarkable

work called ldquoMajma-ul-Bahrainrdquo or

the ldquoThe confluence of two seasrdquo

3 Dara Shukoh was seen as

inheriting the qualities of his

ancestor Akbar in that he

promoted religious pluralism and

syncretism

Which of the statements given above

isare correct

(a) 1 and 3 only

(b) 2 only

(c) 1 and 2 only

(d) 1 2 and 3

RAUSIAS-FC19E1003 24

Q64) निमननिखित कथि ो पर निचार कीनजए

1 ग मतशवर परनतमा निोधयनगरी पहाड़ी पर खसथत ह

2 शरिरबिग िा िह सथाि ह जहाो मौयण िोश क

सोसथापक चोदरगपत मौयण अपि नसोहासि क

तयागि क बाद जि तपसवी बि गए थ

उपयणकत कथि ो म स कौि-सास सही हह

(a) किि 1

(b) किि 2

(c) 1 और 2 द ि ो

(d) ि त 1 ि ही 2

Q65) निमननिखित कथि ो पर निचार कीनजए

1 पराताखतवक साकषय स पता चिता ह नक पराची

घाटी सभयता हड़पपा और म हिज दाड़ द ि ो

की पिणिती ह

2 पराची िदी भििशवर स निकिती ह

उपयणकत कथि ो म स कौि-सास सही हह

(a) किि 1

(b) किि 2

(c) 1 और 2 द ि ो

(d) ि त 1 ि ही 2

Q66) निमननिखित कथि ो म स कौि-सास सही हह

1 िजराह क समारक ो क समह का निमाणर

चोदि राजिोश क शासिकाि क दौराि हआ

था

2 य समारक हररिोदर पिणत शरोििा म खसथत ह

3 म रक क यातरी इबन बतता ि अपि सोसमरर ो

म िजराह क मोनदर ो की यातरा का उललि

नकया था तथा इन काजराण िाम स समब नरत

नकया था

िीच नदए गए कट का परय ग कर सही उततर चनिए

(a) किि 1

(b) किि 1 और 2

(c) किि 2 और 3

(d) किि 1 और 3

Q67) निमननिखित कथि ो म स कौि-सास सही हह

1 डॉ बी आर अमबडकर ि दी एनिनहिशि

ऑफ़ कासट (The Annihilation of Caste)

नििी थी नजसम उन ोि नहोद रमण म िोशािगत

पजारी की परथा क उनमिि की आिशयकता

पर बि नदया था

2 डॉ राजदर परसाद ि थॉटस ऑि पानकसताि

(Thoughts on Pakistan) िामक पसतक

नििी थी

िीच नदए गए कट का परय ग कर सही उततर चनिए

(a) किि 1

(b) किि 2

(c) 1 और 2 द ि ो

(d) ि त 1 ि ही 2

Q68) निमननिखित कथि ो म स कौि-सास सही हह

1 महरगढ़ भारतीय उपमहादवीप म एक परनसदध

ििपािार बसती ह ज नसोर पराोत पानकसताि म

खसथत ह

2 बरणह म म कतत ो क उिक सवामी क साथ कबर ो

म दफिाया जाता था

िीच नदए गए कट का परय ग कर सही उततर चनिए

(a) किि 1

(b) किि 2

(c) 1 और 2 द ि ो

(d) ि त 1 ि ही 2

Q69) निमननिखित कथि ो म स कौि-सास सही हह

1 काकानटय मोनदर अनरकतर नशि क समनपणत

2 हिमक ोडा म हजार-सतोभ िाि मोनदर (The

Thousand-Pillared Temple) का निमाणर

काकानटय समराट रदर ि करिाया था

िीच नदए गए कट का परय ग कर सही उततर चनिए

(a) किि 1

(b) किि 2

(c) 1 और 2 द ि ो

(d) ि त 1 ि ही 2

RAUSIAS-FC19E1003 25

Q64) Consider the following statements

1 Gommateshwara Statue is located

on the Vindyagiri Hill

2 Shravanabelagola is the place

where Chandragupta Maurya the

founder of the Mauryan dynasty

became a Jain ascetic after

relinquishing his throne

Which of the statements given above

isare correct

(a) 1 only

(b) 2 only

(c) Both 1 and 2

(d) Neither 1 nor 2

Q65) Consider the following statements

1 Archaeological evidence shows

that the Prachi Valley Civilisation

predates both Harappa and

Mohenjo-Daro

2 The Prachi river originates from

Bhubaneswar

Which of the statements given above

isare correct

(a) 1 only

(b) 2 only

(c) Both 1 and 2

(d) Neither 1 nor 2

Q66) Which of the following statements

isare correct

1 The Khajuraho group of

monuments was built during the

rule of the Chandela dynasty

2 These monuments are located in

Harischandra mountain range

3 Ibn Battuta the Moroccan

traveller in his memoirs mentioned

visiting Khajuraho temples and

called them Kajarra

Select the correct answer using the code

given below

(a) 1 only

(b) 1 and 2

(c) 2 and 3

(d) 1 and 3

Q67) Which of the following statements

isare correct

1 Dr BR Ambedkar wrote the

Annihilation of Caste emphasising

the need to do away with the

practice of hereditary priesthood in

Hinduism

2 The book lsquoThoughts on Pakistanrsquo

was written by Dr Rajendra

Prasad

Select the correct answer using the code

given below

(a) 1 only

(b) 2 only

(c) Both 1 and 2

(d) Neither 1 nor 2

Q68) Which of the following statements

isare correct

1 Mehrgarh is a famous Neolithic

settlement in the Indian

subcontinent which is situated in

Sindh province Pakistan

2 At Burzahom dogs were buried

with their masters in their graves

Select the correct answer using the code

given below

(a) 1 only

(b) 2 only

(c) Both 1 and 2

(d) Neither 1 nor 2

Q69) Which of the following statements

isare correct

1 The Kakatiya temples are

dedicated mostly to Siva

2 The Thousand-Pillared Temple at

Hanamkonda was built by the

Kakatiya king Rudra

Select the correct answer using the code

given below

(a) 1 only

(b) 2 only

(c) Both 1 and 2

(d) Neither 1 nor 2

RAUSIAS-FC19E1003 26

Q70) निमननिखित कथि ो म स कौि-सास सही हह

1 अहमदाबाद नमि हड़ताि क दौराि महातमा

गाोरी ि शरनमक ो क पकष क मजबत करि क

निए आमरर अिशि नकया था

2 अिशि स नमि मानिक ो पर दबाि पड़ा था ज

अोततः शरनमक ो क िति म 15 परनतशत की िखदध

करि क निए सहमत हए थ

िीच नदए गए कट का परय ग कर सही उततर चनिए

(a) किि 1

(b) किि 2

(c) 1 और 2 द ि ो

(d) ि त 1 ि ही 2

Q71) निमननिखित म स नकसक नकिक भारत स यिसक

की माििता की अमतण साोसकनतक निरासत की

परनतनिनर सची (The UNESCOrsquos List of the

Representative List of the Intangible

Cultural Heritage of Humanity) म शानमि

नकया गया ह

1 मनडयटट

2 सोकीतणि

3 को भ मिा

िीच नदए गए कट का परय ग कर सही उततर चनिए

(a) किि 1 और 2

(b) किि 2 और 3

(c) किि 3

(d) 1 2 और 3

Q72) निमननिखित जिजानतय ो म स कौि-सीसी ो

जिजानतजिजानतया िागािड स सोबोनरत हह

1 अोगामी

2 ककी

3 जारिा

िीच नदए गए कट का परय ग कर सही उततर चनिए

(a) किि 1

(b) किि 1 औऔ 2

(c) किि 2

(d) 1 2 और 3

Q73) निमननिखित कथि ो म स कौि-सास सही हह

1 राषटर कट सामराजय की सथापिा दोनतदगण ि की थी

नजसि मानयाित म अपिी राजरािी की

सथापिा की थी

2 राषटर कट समराट अम घििण एक ििक था और

उस कनिताओो पर पहिी कननड़ पसतक नििि

का शरय नदया जाता ह

िीच नदए गए कट का परय ग कर सही उततर चनिए

(a) किि 1

(b) किि 2

(c) 1 और 2 द ि ो

(d) ि त 1 ि ही 2

Q74) निमननिखित कथि ो म स कौि-सास सही हह

1 कशब चोदर सि ि ततवब नरिी सभा की

अधयकषता की थी ज आधयाखतमक सतय की

ि ज म सोिि थी

2 बरहम समाज ि मािि गररमा पर बि नदया

मनतणपजा का निर र नकया और सती परथा जसी

सामानजक बराइय ो की आि चिा की

िीच नदए गए कट का परय ग कर सही उततर चनिए

(a) किि 1

(b) किि 2

(c) 1 और 2 द ि ो

(d) ि त 1 ि ही 2

Q75) निमननिखित कथि ो म स कौि-सास सही हह

1 भारत म नचशती नसिनसिा खवाजा म इिददीि

नचशती क दवारा सथानपत नकया गया था

2 नचशती परोपरा की एक परमि निशिता

आतमसोयम थी नजसम साोसाररक म ह स दरी

बिाए रििा शानमि था

िीच नदए गए कट का परय ग कर सही उततर चनिए

(a) किि 1

(b) किि 2

(c) 1 और 2 द ि ो

(d) ि त 1 ि ही 2

RAUSIAS-FC19E1003 27

Q70) Which of the following statements

isare correct

1 During the Ahmedabad Mill Strike

Mahatma Gandhi undertook a fast

unto death to strengthen the

workersrsquo resolve

2 The fast had effect of putting

pressure on mill owners who

finally agreed to give the workers a

15 per cent increase in wages

Select the correct answer using the code

given below

(a) 1 only

(b) 2 only

(c) Both 1 and 2

(d) Neither 1 nor 2

Q71) Which of the following are included in

the UNESCOrsquos list of the representative

list of the intangible cultural heritage of

humanity from India

1 Mudiyettu

2 Sankirtana

3 Kumbh Mela

Select the correct answer using the code

given below

(a) 1 and 2 only

(b) 2 and 3 only

(c) 3 only

(d) 1 2 and 3

Q72) Which of the following tribes isare

related to Nagaland

1 Angami

2 Kuki

3 Jarawa

Select the correct answer using the code

given below

(a) 1 only

(b) 1 and 2 only

(c) 2 only

(d) 1 2 and 3

Q73) Which of the following statements

isare correct

1 Rashtrakuta kingdom was founded by Dantidurga who established his capital at Manyakhet

2 Amoghavarsha a Rashtrakuta king was an author and is credited with writing the first

Kannada book on poetics

Select the correct answer using the code given below

(a) 1 only

(b) 2 only

(c) Both 1 and 2

(d) Neither 1 nor 2

Q74) Which of the following statements isare correct

1 Keshab Chandra Sen headed the Tattvabodhini Sabha which was engaged in search of spiritual truth

2 The Brahmo Samaj laid emphasis on human dignity opposed idolatry and criticized such social

evils as the practice of Sati

Select the correct answer using the code given below

(a) 1 only

(b) 2 only

(c) Both 1 and 2

(d) Neither 1 nor 2

Q75) Which of the following statements isare correct

1 The Chishti order was established in India by Khwaja Moinuddin

Chishti

2 A major feature of the Chishti tradition was austerity including maintaining a distance from the

worldly power

Select the correct answer using the code

given below

(a) 1 only

(b) 2 only

(c) Both 1 and 2

(d) Neither 1 nor 2

T e s t i s p a r t o f R a u rsquo s I A S T e s t s e r i e s f o r P r e l i m i n a r y E x a m 2 0 1 9

FOUNDATION + CURRENT AFFAIRS

GENERAL STUDIES (PAPER ndashI)

FOUNDATION TEST ndashIII

SUBJECT NCERT History Class VI-X + Current Affairs

Time Allowed 1frac12 Hours Maximum Marks 150

I NSTRUCT IONS

1 IMMEDIATELY AFTER THE COMMENCEMENT OF THE EXAMINATION YOU SHOULD CHECK

THAT THIS TEST BOOKLET DOES NOT HAVE ANY UNPRINTED OR TORN or MISSING PAGES OR

ITEMS ETC IF SO GET IT REPLACED BY A COMPLETE TEST BOOKLET

2 This Test Booklet contains 75 items (questions) Each item is printed both in Hindi and English

Each item comprises four responses (answers) You will select the response which you want to mark

on the Answer Sheet In case you feel that there is more than one correct response mark the

response which you consider the best In any case choose ONLY ONE response for each item

3 You have to mark all your responses ONLY on the separate Answer Sheet (OMR sheet) provided

Read the directions in the Answer Sheet

4 All items carry equal marks

5 Before you proceed to mark in the Answer Sheet the response to various items in the Test booklet

you have to fill in some particulars in the Answer Sheet as per instructions contained therein

6 After you have completed filling in all your responses on the Answer Sheet and the examination has

concluded you should hand over to the Invigilator only the Answer Sheet You are permitted to

take away with you the Test Booklet

7 Penalty for wrong answers

THERE WILL BE PENALTY FOR WRONG ANSWERS MARKED BY A CANDIDATE IN THE

OBJECTIVE TYPE QUESTION PAPERS

(i) There are four alternatives for the answer to every question For each question for which a

wrong answer has been given by the candidate one-third of the marks assigned to that

question will be deducted as penalty

(ii) If a candidate gives more than one answer it will be treated as a wrong answer even if one of

the given answers happens to be correct and there will be same penalty as above to that

question

(iii) If a question is left blank ie no answer is given by the candidate there will be no penalty for

that question

T h i s t e s t i s p a r t o f R a u rsquo s I A S T e s t s e r i e s f o r P r e l i m i n a r y E x a m 2 0 1 9

Test Code

FC19E1003

FC19H1003 29

Answers and Explanations of

NCERT History Class VI-X + Current Affairs (FC19E1003)

Q1) उततर (c)

सपषटीकरण

- ऋगवद म दविय ो और दिताओो क समवपित एक

हजार स अविक सत तर (शल क) ह

- य शल क ऋविय ो क दवारा रच गए थ और परि ो

दवारा सीख जात थ

- हालाोवक कछ शल क मवहलाओो (जस वक अपाला

घ सा ल पामदरा मतरयी और गागी) क दवारा भी रच

गए थ

- ऋगवद म सोिाद क रप म कई शल क मौजद ह

- हम विशवावमतर नामक एक ऋवि और दविय ो क

रप म पजी जान िाली द नवदय ो (वयास और

सतलज) क बीच िाताि का उदाहरण वमलता ह

- इसस पता चलता ह वक विशवावमतर िवदक काल स

सोबोवित थ

Q2) उततर (b)

सपषटीकरण

- करनल गफाओो स राख क अिशि परापत हए ह

ज इस ओर सोकत करत ह वक ततकालीन ल ग

अवि क उपय ग स पररवचत थ

- य गफाएो आोधर परदश म सथथत ह

Q3) उततर (c)

सपषटीकरण

bull बरािह म ितिमान कशमीर म सथथत एक

परागवतहावसक थथल ह जहाो ल ग गडढ क घर ो का

वनमािण करत थ

bull य घर जमीन क ख द कर बनाए जात थ तथा नीच

जान क वलए सीवियाा ह ती थी

bull ऐसा अनमान लगाया जाता ह वक य घर ठो ड क

मौसम म आशरय परदान करत थ

Q4) उततर (c)

सपषटीकरण

bull परालख-विदया (Epigraphy) क वशलालख ो क

अधययन क रप म पररभावित वकया जाता ह

bull हसतवलसखत दसतािज ो क माधयम स इवतहास

और सावहतय क अधययन क पाोडवलवप विजञान

(Manuscriptology) कहत ह

bull पराचीन लखन परणावलय ो क अधययन और

ऐवतहावसक पाोडवलवपय ो क समझन तथा वतवथ

वनिािरण क पलीओगराफी (Palaeography) कहा

जाता ह

bull नयवमजमविकस (Numismatics) वसक ो क

अधययन क सोदवभित करता ह

Q5) उततर (a)

सपषटीकरण

- चरक सोवहता चरक क दवारा वलखी गई आयिद

और िदयक-शासर पर एक महतवपणि पसतक ह

- ि भारतीय िदयक-शासर की पारमपररक परणाली

वजस आयिद क नाम स जाना जाता ह क

अभयासकताि थ

- ऐसा माना जाता ह वक चरक का विकास दसरी

शताबदी (ईसा पिि) और दसरी शताबदी (ईसवी) क

मधय हआ था

Q6) उततर (b)

सपषटीकरण

- भाग फसल ो पर वलए जान िाल कर क सोदवभित

करता ह ज कल फसल उतपादन का 16 िाो भाग

था

- ldquoकममकारrdquo शबद भवमहीन कवि शरवमक िगि क

वलए परय ग वकया जाता था

- ldquoअशवमिrdquo (वजस घ ड क बवलदान क रप म भी

जाना जाता ह) एक अनषठान ह ता था वजसम एक

घ ड क सवतोतर रप स घमन क वलए छ ड वदया

FC19H1003 30

जाता ह और राजा क सवनक उसकी रखिाली

करत थ

Q7) उततर (d)

सपषटीकरण

- ऋगववदक काल म घ ड ो क रथ ो म ज ता जाता था

ज (रथ) भवम मिवशय ो आवद पर कबजा करन क

वलए लड गए यद ो म उपय ग वकए जात थ

- इसस यह पता चलता ह वक घ ड ो यकत रथ ो का

उपय ग महाजनपद काल स काफी पहल आरमभ

हआ था

- ऋगववदक काल म मिवशय ो भवम जल आवद पर

कबजा करन क वलए तथा ल ग ो क पकडन क

वलए यद वकय जात थ

- अविकाोश परि इन यद ो म भाग वलया करत थ

- हालाोवक उस समय क ई वनयवमत सना नही ो ह ती

थी लवकन उस काल म सभाऐो ह ती थी ो वजनम

ल ग यद क मामल ो पर चचाि करत थ

- वनयवमत सनाएा महाजनपद काल का िवशषटय थी

वजनम पदल सवनक ो की विशाल सनाएा रथ तथा

हाथी शावमल ह त थ

Q8) उततर (a)

सपषटीकरण

- बद शाकय कल स सोबोवित थ और कशीनारा म

उनका वनिन हआ था

- बद न अपनी वशकषाएा पराकत भािा म दी थी ो ज

आम ल ग ो की भािा थी

Q9) उततर (c)

सपषटीकरण

- पराचीन भारत म दशिनशासर की छह शाखाएा थी ो

िशविक नयाय समखया य ग पिि वममाोसा और

िदाोत या उततर वममाोसा

- इनकी थथापना करमश कनाद गौतम कवपल

पतोजवल जावमनी और वयास ऋविय ो न की थी

Q10) उततर (b)

सपषटीकरण

महािीर की वशकषाऐो छठी शताबदी म िललभी म

सोकवलत की गई थी ो

Q11) उततर (c)

सपषटीकरण

- पारमपररक रप स चाणकय क कौविलय अथिा

विषणगपत क नाम स जाना जाता ह

- उसन अथिशासतर ज एक पराचीन भारतीय

राजनवतक आलख ह वलखा था

Q12) उततर (d)

सपषटीकरण

- भारत का राषटर ीय वचनह सारनाथ (उततर परदश) क

अश क सतमभ क ऊपर (शीिि पर) वसोह कवपिल

का एक अनरपण ह

- इस राषटर ीय वसदाोत सतयमि जयत क साथ

सोय वजत वकया गया ह

- रामपिि बल का नाम रामपिि (वबहार) क नाम पर

पडा जहाा इसकी ख ज हई थी

- यह अपन नाजक नकाशी मॉडल क वलए परवसदद

ह वजसम क मल तवचा सोिदनशील नथन ो सतकि

कान और मरबत िााग ो क शरषठतर परवतरप क

परदवशित वकया गया ह

- यह भारतीय और फारसी ततव ो का एक ससममशरण

- सोवकससा उततर परदश म सथथत ह

Q13) उततर (a)

सपषटीकरण

का िर वसोह ज एक महान य दा थ वबहार स

सोबोवित थ

Q14) उततर (b)

सपषटीकरण

िललालर शबद बड भ-सवावमय ो क वलए परय ग

वकया जाता था

FC19H1003 31

Q15) उततर (c)

सपषटीकरण

- अररकमड एक तिीय बसती थी जहाो दर दश ो स

आन िाल जहाज ो का माल उतारा जाता था

- यहाो पर ईोि ो का एक विशाल ग दाम वमटटी क

बतिन (वजनम एमफ रा - द हरी मवठय ो का लोबा

घडा - शावमल ह) और एरिाइन (Arretine)

मदभाोड पाए गए थ

- इस थथान पर र मन दीपक काोच क बन पातर और

रतन भी पाए गए थ

Q16) उततर (a)

सपषटीकरण

- मिनदर सोगम कविताओो म उसललसखत एक

तवमल शबद ह वजसका अथि ह ldquoतीन परमखrdquo

- यह तीन सततारि पररिार ो क मसखयाओो क वलए

परय ग वकया जाता ह च ल चर और पाणडय

Q17) उततर (c)

सपषटीकरण

- ऋग िद म सभा विदाथा तथा गण जसी

जनजावतय ो पर अथिा किोब पर आिाररत

सभाओो का उललख ह

- आरसमभक िवदक काल म सभाओो और सवमवतय ो

का विशि महतव ह ता था

- यहाा तक की मसखया अथिा राजा भी उनका

समथिन परापत करन क वलए आतर रहत थ

Q18) उततर (a)

सपषटीकरण

- जन िमि न ईशवर क अससततव क मानयता त दी ह

वकनत उसन ईशवर क वजना क पद स नीच रखा

- जन िमि न बौद िमि की तरह िणि परणाली की

भरतिना नही ो की थी

Q19) उततर (d)

सपषटीकरण

- च ल ो और पाणडय ो न शसकतशाली तिीय शहर ो का

विकास वकया था

- च ल ो का सबस महतवपणि शहर पहार (या

कािरीपटटीनम) था |

- मदरई पाणडय ो की राजिानी थी

Q20) उततर (b)

सपषटीकरण

- ldquoबदचररतrdquo बद का जीिन-ितताोत ह

- इस अशवघ ि क दवारा वलखा गया था

Q21) उततर (a)

सपषटीकरणः

- तवमल कवि अपपर भगिान वशि क भकत थ

- इस परकार ि एक नयनार सोत थ

Q22) उततर (d)

सपषटीकरणः

- समदरगपत एक परवसद गपत शासक था

- उसन वसक ो पर िीणा बजात हए अपनी छवि

अोवकत करिाई थी

- यह सोगीत क परवत उसक परम क दशािता ह

- हम उसकी इलाहाबाद परशससत स महतवपणि

ऐवतहावसक जानकारी वमलती ह वजसकी रचना

उसक दरबार क कवि हररसन न की थी

Q23) उततर (b)

सपषटीकरणः

- विकरम सोित की शरआत ििि 58 ईसा पिि म

चनदरगपत वदवतीय न की थी

- यह शक ो पर उसकी जीत और उस विकरमावदतय

की पदिी वमलन क उपलकषय म आरमभ वकया गया

था

FC19H1003 32

- बानभटट न हिििििन का जीिन-ितताोत हििचररत

(ज सोसकत म थी) वलखी थी

Q24) उततर (c)

सपषटीकरणः

- सोवि-विगरावहका यद एिो शाोवत का मोतरी

- साथििाह वयापाररय ो क कावफल ो का नता

Q25) उततर (a)

सपषटीकरणः

- जआन झाोग (हसआन रताोग ndash Hsuang Tsang)

एक चीनी यातरी था ज हिििििन क शासनकाल म

भारत आया था

- ििि 630 ईसवी स ज दशक आरमभ हआ था उसम

जआन झाोग मधय एवशया ईरान और

अफग़ावनसतान की यातरा करन क पशचात कशमीर

क रासत स भारत आया था

- उसन उततर स पिि तक की यातरा की और िह

लगभग 2 ििि वबहार म रहा

- जआन झाोग न नालनदा विशवविदयालय म विदयावथिय ो

और विदवान ो क साथ पारसपररक विचार-विमशि

वकया थथानीय भािाओ ा म वनपणता परापत की तथा

बौद सतप ो की ख ज की

Q26) उततर (c)

सपषटीकरणः

- परदवकषणा पथ बौद िासतकला म सतप क चार ो

ओर बनाया जान िाला एक घमािदार पथ ह ता

- परशन म वदए गए बाकी क तीन ो ततव वहोद मसनदर ो की

िासतकला क भाग ह

Q27) उततर (d)

सपषटीकरणः

परशन म वदए गए सभी मोवदर ो म वयापक रप स

ईोि ो (पकी ईोि ो) का परय ग पतथर ो क साथ हआ

Q28) उततर (c)

सपषटीकरण

- महममद कली कतब शाह ग लकणडा का सलतान

था

- िह अकबर का समकालीन था

- सावहतय और िासतकला म उसकी अतयाविक

रवच थी

- िह एक महान कवि था

- िह दसखनी उदि फारसी और तलग म वलखता था

- उसन अपन पीछ एक विसतत वदिान (सोगरह)

छ डा ह

- अभी हाल ही म तलोगाना म ग लकणडा क वकल

क अनदर खदाई वकय गए बाग-ए-नाया वकला

बाग क चार ो ओर रप-रखा क मानवचतरण क

वलए भारतीय परातासतवक सिकषण (The

Archaeological Survey of India ndash ASI)

गराउणड पनीिर विोग रडार (Ground Penetrating

Radar) का परय ग करगा

Q29) उततर (a)

सपषटीकरणः

- वसलपपावदकारम एक तवमल महाकावय ह वजसकी

रचना इलाोग क दवारा लगभग 1800 ििि पिि की

गई थी

- यह क िलन नामक एक वयापारी की कहानी ह

ज माििी नामक एक गवणका (िशया) स परम

करन लगा था

- मवनमकलाई क िलन और माििी की पतरी की

कहानी ह

Q30) उततर (a)

सपषटीकरण

- चरक आयिद और वचवकरता की एक महतवपणि

रचना चरक सोवहता क लखक ह

- बरहमगपत क अपनी रचना बरहम-सफि-वसदानत

(ज एक खग लीय रचना ह) क कारण परवससद

वमली

FC19H1003 33

- बगदाद म इसका अनिाद अरबी भािा म वकया

गया था

- इसका इसलावमक गवणत और खग ल-विजञान पर

महतवपणि परभाि पडा था

- बाद म अपन जीिनकाल म बरहमगपत न

ldquoखोडखयाकrdquo वलखी ज एक खग लीय पससतका

(एक छ िी पसतक) थी

- इसम आयिभटट की अिि-रावतर क परतयक वदन की

शरआत परणाली का परय ग वकया गया था

Q31) उततर (c)

सपषटीकरण

- अमीर खसर एक परवसद सफी सोगीतकार कवि

और विदवान थ

- 1318 म उनह ोन पाया वक इस भवम (वहोदसतान) क

हर कषतर म अलग-अलग भािा थी लाहौरी

कशमीरी दवारसमदरी (दवकषणी कनाििक म)

तलोगाना (आोधर परदश म) गजरी (गजरात म)

माबारी (तवमलनाड म ) अििी (पिी उततर परदश

म) और वहोदिी (वदलली क आस-पास क कषतर म)

आवद

- उनह न यह बताया वक सोसकत वकसी भी कषतर स

सोबोवित नही ो थी और किल बराहमण ही इस भािा

का जञान रखत थ

Q32) उततर (c)

सपषटीकरण

- वहरणय-गभि सववणिम गभि क सोदवभित करता ह

- जब बराहमण ो की सहायता स यह अनषठान वकया

जाता था त यह माना जाता था वक बवल दन िाल

का कषवतरय क रप म पनजिनम ह गा

Q33) उततर (d)

सपषटीकरण

- कदमई भवम राजसव पर कर क सोदवभित करता

- गवावलयर परशससत म नागभि क दवारा वकय गए

श िण का िणिन वकया गया ह |

- नागभि एक परवतहार राजा था

Q34) उततर (b)

सपषटीकरण

- राजतरो वगनी 12िी ो शताबदी म कलहन क दवारा

रवचत एक सोसकत पसतक (िकसट) ह

- यह परारसमभक भारत की ऐवतहावसक इवतितत थी

- तकि सोगत रप स इस अपन परकार की सिोततम

और सिािविक विशवसनीय कवत माना जाता ह

- यह कशमीर कषतर क पराचीनतम समय स लकर

उसकी रचना की तारीख तक क समपणि इवतहास

का आचछादन करती ह

Q35) उततर (c)

सपषटीकरण

- गााि की आम सभा क ldquoउरrdquo कहा जाता था

- ldquoउरrdquo म गााि क सभी कर दन िाल वनिासी

शावमल ह त थ

Q36) उततर (a)

सपषटीकरण

- वदलली सलतनत म ldquoतारीखrdquo इवतहास लखन का

एक रप था

- ldquoतािरीखrdquo क लखक विदवान परि ह त थ वजनम

सवचि परशासक इतयावद शावमल थ

Q37) उततर (a)

सपषटीकरण

- अलाउददीन सखलजी अपन सवनक ो क ितन का

भगतान नकद म करता था न वक इकता क रप

- सवनक अपना सामान वदलली म वयापाररय ो स

खरीदत थ अतः इस बात का भय था वक वयापारी

कही ो िसतओो का मलय न बिा द

- इसकी र कथाम क वलए अलाउददीन सखलजी न

वदलली म कीमत ो क वनयसित वकया

FC19H1003 34

- अविकारीगण धयानपििक मलय ो का सिकषण करत

थ तथा ज वयापारी वनिािररत मलय पर माल नही ो

बचत थ उनक दसणडत वकया जाता था

Q38) उततर (d)

सपषटीकरण

- वदलली सििपरथम त मर राजपत ो क अिीन उनक

सामराजय की राजिानी बनी थी

- 12िी ो शताबदी क मधय म अजमर क चौहान ो

(वजनह चाहमान ो क नाम स भी जाना जाता ह) न

त मर राजपत ो क परावजत वकया था

- त मर ो और चौहान ो क अिीन वदलली एक

महतवपणि िावणसजयक क दर बन गया था

- कई जन वयापारी यहाा रहन लग थ और उनह ोन

कई मोवदर भी बनिाए

- यहाा पर मवदरत वसक वजनह ldquoदहलीिालrdquo क नाम

स जाना जाता था वयापक रप स परचलन म थ

Q39) उततर (c)

सपषटीकरण

- म ठ की मसिद का वनमािण वसको दर ल दी क

राजयकाल म उसक मिी क दवारा करिाया गया

था

- बगमपरी मसिद का वनमािण महममद तगलक क

शासनकाल म हआ था

- यह मसिद विशव का पणयथथान (The

Sanctuary of the World) और वदलली म महममद

तगलक की नई राजिानी जहाोपनाह की मखय

मसिद थी

- कववत- अल - इसलाम मसिद का विसतार

इलतसिश और अलाउददीन सखलजी न वकया था

- मीनार का वनमािण तीन सलतान ो कतबददीन ऐबक

इलतसिश और वफर ज शाह तगलक क दवारा

करिाया गया था

Q40) उततर (c)

सपषटीकरण

- मगल ो क अिीन मनसबदार शबद उस वयसकत क

वलए सोदवभित वकया जाता था वजसक पास मनसब

(अथाित पद) ह ता था

- उस अपना ितन राजसव कायो वजनह जागीर कहत

थ क रप म परापत ह ता था

Q41) उततर (b)

सपषटीकरण

- ldquoभारत छ ड आोद लनrdquo वबरविश शासन क

सखलाफ ल ग ो का एक सवाभाविक विदर ह था

- असखल भारतीय काोगरस सवमवत न 8 अगसत 1942

क बमबई म एक बठक का आय जन वकया था

- इस बठक म परवसद सोकलप ldquoभारत छ ड rdquo क

पाररत वकया गया और इस उददशय क परापत करन

क वलए गाोिी क नततव म एक अवहोसक जन सोघिि

आोद लन की शरआत का परसताि वदया गया

- लवकन अगल ही वदन गाोिी और काोगरस क अनय

परमख नताओो क वगरफतार कर वलया गया

- काोगरस क एक बार वफर अिि घ वित वकया गया

था

Q42) उततर (c)

सपषटीकरण

- साइमन कमीशन यनाइविड वको गडम क सात

साोसद ो का एक समह था

- इस वबरविश भारत क वलए सोििावनक सिार ो का

सझाि दन क वलए गवठत वकया गया था

- इस आय ग म िररषठ वबरविश राजनता सर जॉन

साइमन क नततव म किल वबरविश सदसय ही

शावमल थ

- इसवलए भारत क ल ग ो न साइमन कमीशन क

आगमन क विरद आोद लन वकया था

Q43) उततर (a)

सपषटीकरण

bull दादा भाई नौर जी भारत म वबरविश शासन क

आवथिक पररणाम ो क बार म अपनी विर िी

(परवतकल) राय क वलए जान जात थ

FC19H1003 35

bull अपन कई लख ो और भािण ो म विशि रप स

ldquoपाििी एो ड अन-वबरविश रल इन इसणडया

(Poverty and Un-British Rule in India) म

नौर जी न यह तकि वदया वक भारत पर अतयविक

कर लगाया गया था और इसकी सोपवतत इोगलड की

ओर परिावहत की जा रही थी

bull उनह ोन पराचीन भारतीय गरोथ ो की वयाखया करन

का और भारतीय ो क आिविशवास क बहाल

करन पर कायि नही ो वकया था

उनह ोन वकसी और बात स पहल सभी सामावजक

बराइय ो क उनमलन की आिशयकता पर भी बल

नही ो वदया था

Q44) उततर (c)

सपषटीकरण

bull अगसत 1932 म वबरविश परिानमोतरी मकड नालड न

अपन साोपरदावयक परसकार (The Communal

Award) की घ िणा की थी

bull यह भारत क कई साोपरदावयक वहत ो क बीच विवभनन

सोघिो क हल करन क वलए वबरिन का एकतरफा

परयास था

bull यह परसकार (Award) बाद म 1935 क

अविवनयम (The Act of 1935) म शावमल वकया

गया था

bull इस साोपरदावयक परसकार न मससलम ो क वलए

आरवकषत एक अलग वनिािचक मणडल फॉमिल का

विसतार अनय अलपसोखयक ो क वलए वकया था

वजसम वसख ो भारतीय ईसाइय ो आोगल-भारतीय

समदाय यर पीय समदाय तथा विवशषट कषतरीय

समह ो क शावमल वकया गया था

bull गाोिी न इस परसताि क भारतीय समाज क

विभावजत करन क वलए एक घवणत वबरविश

सावजश क रप म दखा और उसक सखलाफ

आमरण अनशन वकया

Q45) उततर (b)

सपषटीकरण

मौजदा आयात और वनयाित क अवतररक़त

औपवनिवशक भारत क वनमनवलसखत खचो क

वलए एक विशिवनवशचत िन रावश भी दनी पडती

थी

(i) परशासन क वयय

(ii) सना क रख-रखाि क वयय

(iii) यद क वयय

(iv) सिावनितत अविकाररय ो की पशन तथा

(v) वबरिन दवारा अपनी उपवनिश बसती

(कॉल नी) क रख-रखाि क वयय

इनह गह शलक (Home Charges) क रप म

जाना जाता था और लगभग परी तरह स भारत क

दवारा इनका भगतान वकया जाता था

bull गह शलक म वनमनवलसखत घिक शावमल थ

(i) भारतीय ऋण पर दय बयाज

(ii) ईसट इोवडया को पनी क शयरिारक ो क

लाभाोश

(iii) लोदन म भारत कायािलय चलान क वलए िन

(iv) भारत म वनयकत वबरविश कवमिय ो क ितन

और पशन का भगतान करन क वलए िन

(v) रलि पर बयाज

(vi) नागररक और सनय शलक

(vii) इोगलड म सट र (सामगरी) की खरीद

Q46) उततर (b)

सपषटीकरण

bull भारतीय राषटर ीय काोगरस का लाहौर सतर 1929 म

जिाहरलाल नहर की अधयकषता म आय वजत

वकया गया था

bull इस सतर म भारतीय राषटर ीय आोद लन स समबसित

कई महतवपणि पररणाम सामन आय थ

(i) सििपरथम इस सतर म काोगरस क अधयकष पद

पर जिाहरलाल नहर क चना गया था ज

काोगरस म िामपोवथय ो की बिती हई ताकत

का सपषट सोकत था

(ii) दसरा इस सतर म पहली बार काोगरस न पणि

सवतोतरता की माोग क उठाया था

इस परकार की माोग काोगरस मोच स पहल कभी भी

नही ो उठाई गई थी

Q47) उततर (b)

सपषटीकरण

FC19H1003 36

bull इस ररप िि न वकसी भी समदाय क वलए पथक

वनिािचक मोडल अथिा अलपसोखयक ो क वलए

भाराोश की वसफाररश नही ो की थी

bull तथावप इस ररप िि न उन पराोत ो म अलपसोखयक

सीि ो क आरकषण की अनमवत दी थी जहाा पर कम

स कम दस परवतशत अलपसोखयक ह

bull लवकन यह समदाय क आकार क अनपात म ह ना

चावहए था

bull इस ररप िि म भारत क वलए पणि सवतोतरता क

वलए क ई पराििान नही ो था

Q48) उततर (c)

सपषटीकरण

bull आरो वभक िवदक आयो का िमि मखय रप स

परकवत की पजा और यजञ था

bull परारो वभक आयि िमि परकवत की पजा क समान था

bull िासति म उनक चार ो ओर की शसकतयाा वजनह न

त ि वनयोवतरत कर सकत थ और न ही समझ पाए

थ उनह वदवयता क साथ वनिवशत वकया गया तथा

उनह मादा या नर दिीदिताओो क रप म

परतीकतव वकया गया था

bull उनह ोन कछ यजञ ो का भी वनषपादन वकया था

Q49) उततर (b)

सपषटीकरण

bull सडक और नदी-मागि (जल-मागि) डकती स

सरवकषत नही ो थ

bull उललखनीय ह वक हिििििन क शासनकाल क

दौरान यआन चिाोग (हयएन साोग) का सारा

सामान लि वलया गया था

Q50) उततर (c)

सपषटीकरण

परशन म वदए गए द न ो कथन सही ह

Q51) उततर (b)

सपषटीकरण

bull परोदर दास एक सोत और भगिान कषण क एक

महान भकत थ

bull परोदर दास क कनाििक सोगीत क वपतामह क

रप म जाना जाता ह

bull यदयवप उनक जनम-थथान क बार म काफी

अिकल लगाई जाती रही ह

bull तथावप अब कननड विशवविदयालय हमपी क दवारा

गवठत एक विशिजञ सवमवत इस वनषकिि पर पहोची

ह वक उनका जनम थथान सोभितया कनाििक का

एक छ िा-सा गााि कषमपरा (वशिम गगा वजला)

था

Q52) उततर (c)

सपषटीकरण

bull शरी तयागराज शरी शयाम शासतरी और शरी मथसवामी

दीवकषतर क कनाििक सोगीत की वतरमवति माना

जाता ह

bull उनक कारण ही 18िी ो-19िी ो शताबदी म कनाििक

सोगीत का सववणिम यग आया था

Q53) उततर (d)

सपषटीकरण

bull अभी हाल ही म लौह यगीन-महापािावणक काल

का 2000 ििि पराना एक दलिभ सारक फगस

(Sarcophagus) (पतथर का ताबत) क ललम क

वियर गाोि (क वयलडी क पास वजला क वझक ड

करल राजय) की एक रॉक-कि गफा स ख जा गया

bull यह ताबत वजसम हविय ो क िकड थ खदाई क

दौरान वमला

bull अभी तक इस परकार की दलिभ ख ज करल क

मातर द ही थथान ो स हई ह

bull य द न ो सारक फगी (Sarcophagi) (पतथर क

ताबत) चियर और अथ ली (वजला क वझक ड) क

महापािाण थथल ो स वमल ह

Q54) उततर (a)

सपषटीकरण

FC19H1003 37

दवकषण भारत म महापािाण सोसकवत एक पणि

विकवसत लौह यगीन सोसकवत थी

Q55) उततर (d)

सपषटीकरण

bull च ल पाणडय और करलपतर (चर) इन तीन ो का

उललख अश क क अवभलख ो म वकया गया ह

bull सोभितः य भौवतक सोसकवत क उततर

महापािावणक चरण म थ

Q56) उततर (d)

सपषटीकरण

bull भीमा-क रगाोि की लडाई ततीय आोगल-मराठा

यद का वहससा थी

Q57) उततर (b)

सपषटीकरण

bull राजकमार शकल न गाोिीजी क चोपारण आन तथा

वतनकवथया परणाली स जडी समसया की जाोच क

वलए रारी करन क वलए दश भर म उनका

अनसरण वकया था

bull बज वकश र राजदर परसाद महादि दसाई और

नरहरी पाररख चोपारण सतयागरह क दौरान गाोिी

जी क सहय गी थ

Q58) उततर (b)

सपषटीकरण

bull बराहमण ो और बौद मठिाररय ो क कर-मकत गााि

अनदान म दन की परथा सतिाहन ो न आरमभ की

थी

Q59) उततर (c)

सपषटीकरण

इस कायिकरम क उददशय वनमनानसार ह

(i) बवनयादी पयििन आिाररक सोरचना का विकास

करना

(ii) चयवनत (पहचान वकय गए) कषतर ो म आजीविका क

सजन क वलए दश क साोसकवतक और विरासत

मलय ो क बिािा दना

(iii) विरासत समारक थथल ो पर विशव सतरीय आिाररक

सोरचना विकवसत करक एक सतत तरीक स

पयििक आकििण म िसद करना

(iv) थथानीय समदाय ो की सवकरय भागीदारी क माधयम

स र रगार ो का सजन करना

(v) र रगार उतपादन और आवथिक विकास क वलए

पयििन कषमता का उन पर परभाि का उपय ग

करना तथा

(vi) िारणीय पयििन आिाररक सोरचना का विकास

करना और उसका उवचत सोचालन तथा

रखरखाि सवनवशचत करना

Q60) उततर (b)

सपषटीकरण

bull यह वनकाय ििि 1987 म अससततव म आया था

bull यह एक राषटर ीय सतर का शीिि सोगठन ह ज भारत

सरकार क जनजातीय मामल ो क मोतरालय क

परशासवनक वनयोतरण क अिीन काम कर रहा ह

bull इसका पोजीकत और परिान कायािलय नई वदलली

म सथथत ह

Q61) उततर (c)

सपषटीकरण

bull परमचोद क उपनयास ो म परमाशरम रोगभवम गबन

कमिभवम और ग दान शावमल ह

bull ग रा रिी ोदरनाथ िग र क दवारा रवचत उपनयास ह

bull अभी हाल ही म मोशी परमचोद की 138िी ो जयोती दश

भर म मनाई गई थी

Q62) उततर (b)

सपषटीकरण

bull ldquoवगदाrdquo पोजाब (भारत) एिो पावकसतान की

मवहलाओो क दवारा तयौहार क समय और फसल

की बिाई तथा किाई क अिसर पर वकया जान

िाला एक पारोपररक दहाती नतय ह

FC19H1003 38

bull इस नतय क माधयम स पोजाबी मवहलाऐो अपनी

परसननता परकि करती ह तथा वगदा क परदशिन क

माधयम स परि िचिसव िाल समाज म मवहलाओो

की दबी हई भािनाओो क परकि करती ह

bull चोवक इस नतय का परि ो क साथ क ई सोबोि नही ो

ह अतः किल मवहलाऐो ही इसम भाग ल सकती

bull हर साल तीज समार ह क दौरान पोजाब म वगदा

नतय वकया जाता ह

तीज भारत क कछ भाग ो म मवहलाओो क दवारा

मनाया जान िाल कई तयौहार ो क वलए एक

वयापक नाम ह

Q63) उततर (a)

सपषटीकरण

- मजम-उल-बहरीन या द समदर ो का सोगम

नामक उललखनीय रचना दारा वशक ह क दवारा

वलखी थी

- भारत क उपराषटर पवत शरी एम िकया नायड न कहा

ह वक राजकमार दारा वशक ह की रचनाएा शाोवत

और सदभाि क बिािा दन क वलए एक तारा सर त

क रप म सामन आ सकती ो ह

- उपराषटर पवत गत ििो क भला वदए गए राजकमार

दारा वशक ह क परदवशित परचवलत करन हत

आय वजत एक परदशिनी का दौरा करन क बाद एक

सभा क सोब वित कर रह थ

- इस परदशिनी का आय जन फर क इस गौवियर

(Francois Gautier) क दवारा lsquoइोवदरा गाोिी नशनल

सिर फॉर द आििसrsquo (The Indira Gandhi

National Centre for the Arts) नई वदलली म

वकया गया था

Q64) उततर (c)

सपषटीकरण

- ग मतशवर परवतमा जन भगिान बाहबली क

समवपित ह

- यह एक एक-चटटानी पतथर की मवति ह

- राषटर पवत राम नाथ क विोद न शरिणबलग ला

(कनाििक) म आय वजत वकय जान िाल भवय

अवभिक समार ह महामसतकावभिक का

उदघािन वकया था

- यह समार ह 12 ििो म एक बार ह ता ह

Q65) उततर (c)

सपषटीकरण

bull पराची घािी पराची नदी क चार ो ओर फली हई थी

bull पराची घािी िीर-िीर विलपत ह गई थी

bull पराची नदी भिनशवर स वनकलती ह

bull यह महानदी की एक सहायक नदी ह और यह

परी खदाि किक तथा जगतवसोहपर वजल ो स

ह कर बहती ह

bull इस नदी क पर कषतर क पराची घािी कहा जाता ह

bull यह नदी बोगाल की खाडी म वगरती ह

परातासतवक साकषय स पता चलता ह वक पराची घािी

सभयता हडपपा और म हनज दाड द न ो की

पिििती ह

Q66) उततर (d)

सपषटीकरण

य समारक छतरपर वजल (मधय परदश) म विोधयाचल

पिित शरोखला म सथथत ह

Q67) उततर (a)

सपषटीकरण

bull थॉिस ऑन पावकसतान नामक पसतक डॉ बी

आर अमबडकर न वलखी थी

bull डॉ बी आर अमबडकर की जयोती क अिसर पर

भारत क राषटर पवत न भारत की इस महान हसती

क शरदाोजवल अवपित की थी

bull डॉ बी आर अमबडकर न 1924 म वडपरथड

कलावसर इोसटीटयि (दवलत िगि सोथथान -

बवहषकत वहतकाररणी सभा) और 1927 म समाज

समता सोघ की थथापना की थी

bull अमबडकर का धयान वशकषा कषतर की ओर भी था

bull उनह ोन वशकषा क वनमन िगो म फलान क वलए

पीपलस एजकशन स साइिी (The Peoples

Education Society) क नाम स महाविदयालय ो क

नििकि और छातरािास ो की थथापना की थी

FC19H1003 39

Q68) उततर (b)

सपषटीकरण

bull महरगि भारतीय उपमहादवीप म एक परवसद

निपािाण बसती ह ज बलवचसतान पराोत

पावकसतान म सथथत ह

bull दचपलली (आोधर परदश) क पास नागलर नदी क

पिी ति ो पर चना पतथर क बलॉक क विशाल

विसतार म एक पिि-ऐवतहावसक रॉक आिि थथल की

ख ज की गई ह

bull इसन 1500-2000 ईसा पिि क दौरान गोिर (आोधर

परदश) म विकवसत निपािाण सभयता पर परकाश

डाला ह

Q69) उततर (c)

सपषटीकरण

bull 12िी ो सदी और 13िी ो सदी म काकाविय िोश का

उदय हआ था

bull ि पहल कलयाण क पवशचमी चालकय ो क सामोत थ

bull परारोभ म उनह ोन िारोगल (तलोगाना) क पास एक

छ ि स कषतर पर शासन वकया था

bull उनह ोन ldquoनायक वयिथथाrdquo की शरआत की थी

वजस बाद म विजयनगर क राय शासक ो न

अपनाया और विकवसत वकया था

Q70) उततर (a)

सपषटीकरण

bull गाोिीजी क अनशन स वमल मावलक ो पर दबाि

पडा था ज अोततः शरवमक ो क ितन म 35 परवतशत

की िसद करन क वलए सहमत हए थ

bull गगल (Google) न अनसया साराभाई वजनह ोन

भारत क शरवमक आोद लन म एक अगरणी भवमका

वनभाई थी की 132िी ो जयोती डडल (Doodle) का

वनमािण करक मनाई

Q71) उततर (d)

सपषटीकरण

भारत स यनसक की मानिता की अमति साोसकवतक

विरासत की परवतवनवि सची म वनमनवलसखत शावमल ह

bull कवडयटटम करल का सोसकत रोगमोच

bull मवडयिि करल का अनषठान रोगमोच और नतय

नाविका

bull िवदक मि जाप की परोपरा

bull राजथथान क कालबवलया ल क गीत और नतय

bull रामलीला रामायण का पारोपररक परदशिन

bull सोकीतिन मवणपर का अनषठान गायन ढ ल िादन

और नतय

bull रममन भारत क गििाल वहमालय का िावमिक

तयौहार और अनषठान रोगमोच

bull जाोदीयाला गर पोजाब क ठठर ो की पीतल और

ताोब क वशलप स वनवमित बतिन ो की पारोपररक कला

bull छाऊ नतय पिी भारतीय राजय ो म जनमी शासतरीय

भारतीय नतय कला

bull लददाख का बौद मि जाप िर ाोस-वहमालयी लददाख

कषतर तथा जमम-कशमीर म पवितर बौद गरोथ ो का पाठ

bull य ग

bull नौर र

bull को भ मला

Q72) उततर (b)

सपषटीकरण

bull भारत क राषटर पवत शरी राम नाथ क विोद न

वकसामा नागालड म हॉनिवबल मह रति और

राजय गठन वदिस समार ह का उदघािन वकया

था

bull हॉनिवबल मह रति का नाम भारतीय हॉनिवबल क

नाम पर पडा ह ज एक विशाल और रोगीन जोगली

पकषी ह

bull यह पकषी नागालड राजय की अविकतर जनजावतय ो

की ल ककथाओो म उसललसखत ह

bull नागालड की परमख मानयता परापत जनजावतयाा ह

अोगामी आओ चखसोग चाोग ककी रगमा और

रवलोग आवद

bull ओोग जारिा और ससिनलीस अोडमान-वनक बार

दवीप समह की जनजावतयाा ह

FC19H1003 40

Q73) उततर (c)

सपषटीकरण

bull दकन म राषटर कि शासन दसिी ो सदी क अोत तक

लगभग 200 ििो तक रहा था

bull राषटर कि शासक अपन िावमिक विचार ो म सवहषण

bull उनह ोन न किल शि िमि और िषणि िमि बसलक

जन िमि क भी सोरकषण वदया था

bull एल रा म वशि क परवसद रॉक कि मोवदर का

वनमािण नौिी ो सदी म राषटर कि राजा कषण परथम न

करिाया था

bull उसका उततराविकारी अम घििि जन था लवकन

उसन अनय िमो क भी सोरकषण परदान वकया था

bull राषटर कि ो न मसलमान वयापाररय ो क बसन की

अनमवत दी थी

bull उनह न अपन अविराजय ो म इसलाम क उपदश दन

की भी अनमवत दी थी

bull अभी हाल ही म पाोडिलागटटा (तलोगाना) क

परागवतहावसक चटटान वचतर ो क कषरण की बिती हई

घिनाएा एक गोभीर वचोता का वििय ह

bull यह परागवतहावसक चटटान क नकसान पहाचा

सकता ह

bull पाोडिलागटटा वनमनवलसखत क वलए जाना जाता ह

- 10000 ईसा पिि स 8000 ईसा पिि क वचवतरत

चटटानी आशरय ो क वलए

- राषटर कि काल क एक 8 िी ो सदी क

वशलालख क वलए और

- 12िी ो सदी क काकविय सामराजय क वभवतत

वचतर ो क वलए

Q74) उततर (b)

सपषटीकरण

bull 1828 म राजा राम म हन रॉय न एक नय िावमिक

समाज बरहम सभा की थथापना की थी वजस बाद

म बरहम समाज क नाम स जाना गया था

bull दिदरनाथ िग र न ततवब विनी सभा की अधयकषता

की थी ज आधयासिक सतय की ख ज म सोलि

थी

bull इसका उददशय वहोद िमि क शद करन का और

एकशवरिाद (एक ईशवर म आथथा) का परचार करना

था

bull नय समाज की थथापना क आिार थ कारण

(तकि ) क द सतमभ तथा िद और उपवनिद

bull अभी हाल ही म सािारण बरहम समाज का कछ

काननी मदद ो क लकर पवशचम बोगाल सरकार क

साथ काननी वििाद चल रहा ह

Q75) उततर (c)

सपषटीकरण

bull भारत म वचशती वसलवसल की थथापना खवाजा

म इनददीन वचशती क दवारा की गयी थी

bull ि 1192 ईसवी क आसपास भारत आय थ

bull वचशतीय ो क बारहिी ो शताबदी क उततरािि म भारत

म आन िाल सफीय ो क समह ो म सबस

परभािशाली माना जाता ह

bull उनह ोन थथानीय िातािरण क साथ सफलतापििक

अनकलन वकया और उनह ोन भारतीय भसकत

परोपराओो क कई पहलओो क अपनाया

bull अजमर म सफी अपरकि खवाजा म इनददीन वचशती

की ऐवतहावसक दरगाह क एक नया रप दन की

तयारी की जा रही ह

bull इस 13िी ो शताबदी की दरगाह क ldquoसवचछ

आइकॉवनक थथल ोrdquo (Swacch Iconic Places) म

शावमल वकया गया ह ज परवतवषठत विरासत

आधयासिक और साोसकवतक थथान ो पर क वदरत

य जना ह

FC19H1003 41

ANSWERS amp EXPLANATION OF

NCERT History Class VI-X + Current Affairs

(FC19E1003)

Q1) Answer c

Explanation

Rigveda consists of more than a

thousand hymns dedicated to gods and

goddesses These hymns were

composed by sages and learnt by men

however a few were composed by

women like Apala Ghosa Lopamudra

Maitreyi and Gargi

Rigveda consists of many hymns in the

form of dialogues We get an example of

a dialogue between a sage named

Vishwamitra and two rivers (Beas and

Sutlej) that were worshipped as

goddesses This suggests that he

belonged to the Vedic period

Q2) Answer b

Explanation

Traces of ash have been found from

Kurnool Caves suggesting that people

were familiar with the use of fire

It is situated in Andhra Pradesh

Q3) Answer c

Explanation

Burzahom is a prehistoric site in

present day Kashmir where people built

pit houses which were dug into the

ground with steps leading into them

These may have provided shelter in cold

weather

Q4) Answer c

Explanation

Epigraphy is defined as the study of

inscriptions

Manuscriptology is the study of history

and literature through the use of hand

written documents

Palaeography refers to the study of

ancient writing systems and the

deciphering and dating of historical

manuscripts

Numismatics refers to the study of

coins

Q5) Answer a

Explanation

Charaka Samhita was written by

Charaka and is an important book on

Ayurveda and medicine

He was a practitioner of the traditional

system of Indian medicine known as

Ayurveda

Charaka is thought to have flourished

sometime between the 2nd century BCE

and the 2nd century CE

Q6) Answer b

Explanation

Bhaga refers to the tax on crops which

was fixed at 16th of the production

Kammakaras is the term used for the

landless agricultural labour class

Ashvamedha also known as horse

sacrifice is a ritual where a horse is let

loose to wander freely and it was

guarded by the rajarsquos men

Q7) Answer (d)

Explanation

In the Rigvedic period horses were

yoked to chariots that were used in

battles fought to capture land cattle

etc This suggests that the use of horse

chariots began much before the period

of Mahajanapadas

The battles were fought in the Rigvedic

period for cattlersquos lands water an even

to capture people Most men took part

in these wars however there was no

regular army but there were assemblies

where people met and discussed

matters of war Regular armies became

a feature in the Mjahajanapada period

including vast armies of foot soldiers

chariots and elephants

RAUSIAS-FC19E1003 42

Q8) Answer (a)

Explanation

Buddha belonged to the Sakya clan and

passed away at Kusinara

Buddha taught in Prakrit which was the

common language of people

Q9) Answer c

Explanation

There were six schools of philosophy in

ancient India These are known as

Vaishesika Nyaya Samkhya Yoga

Purva Mimansa and Vedanata or Uttara

Mimansa They were founded by sages

Kanada Gautama Kapila Patanjali

Jamini and Vyasa respectively

Q10) Answer b

Explanation

The teachings of Mahavira were

compiled at Valabhi in 6th century AD

Q11) Answer (c)

Explanation

Chanakya is traditionally identified as

Kautilya or Vishnugupta who authored

the ancient Indian political treatise the

Arthashastra

Q12) Answer d

The national emblem of India is an

adaptation of the Lion Capital atop the

Ashoka Pillar of Sarnath Uttar Pradesh

and is combined with the National

Motto Satyameva Jayate

The Rampurva Bull gets the name from

the site of its discovery Rampurva in

Bihar

It is noted for its delicately sculpted

model demonstrating superior

representation of soft flesh sensitive

nostrils alert ears and strong legs It is

a mixture of Indian and Persian

elements

Sankissa is situated in Uttar Pradesh

India

Q13) Ans(a)

Kunwar Singh was a notable leader during the Revolt of 1857 He belonged

to a royal house of Jagdispur Bihar

Q14) Answer b

Explanation

The term Vellalar was used for large

landowners

Q15) Answer c

Explanation

Arikamedu was a coastal settlement

where ships unloaded goods from

distant lands Finds here include a

massive brick warehouse pottery

including amphorae and Arretine ware

Roman lamps glassware and gems have

also been found at the site

Q16) Answer a

Explanation

Muvendar is a Tamil word mentioned in

Sangam poems meaning three chiefs

used for the heads of three ruling

families the Cholas Cheras and

Pandyas

Q17) Ans (c)

Several tribal or kin-based assemblies

such as the Sabha Vidatha and gana

are mentioned in the Rig-veda The

Sabha and the samiti mattered a great

deal in early Vedic times so much so

that the chiefs or the kings showed an

eagerness to win their support

Q18) Ans (a)

Jainism recognised the existence of the

gods but placed them lower than the

jina and did not condemn the varna

system as Buddhism did

Q19) Answer (d)

Explanation

Cholas and Pandyas had developed

powerful coastal cities The most

important city of Cholas was Puhar or

Kaveripattinam and Madurai was the

capital of Pandyas

Q20) Answer b

Explanation

Buddhacharita is the biography of

Buddha and was written by

RAUSIAS-FC19E1003 43

Ashvaghosha

Q21) Answer (a)

Explanation

Tamil poet Appar was a Shiva devotee

So he was a Nayanar saint

Q22) Answer d

Explanation

Samudragupta was a prominent Gupta

ruler whose coins depict him playing a

veena indicating his love for music We

get important historic information from

his Allahabad Prashasti which was

composed by his court poet Harisena

Q23) Answer (b)

Explanation

Vikrama Samvat was founded by

Chandragupta II in the 58 BC as a

mark of victory over the Shakas and

assumed the title of Vikramaditya

Banabhatta wrote Harshavardhanarsquos

biography the Harshacharita in

Sanskrit

Q24) Answer c

Explanation

Sandhi-vigrahika was the minister of

war and peace

Sarthavaha was the leader of the

merchant caravans

Q25) Answer a

Explanation

Xuan Zang (Hsuan-tsang) was a

Chinese traveller who came during the

reign of Harshavardhana

In the decade that began in 630 AD

Xuan Zang came to India through

Kashmir after visiting Central Asia Iran

and Afghanistan

He travelled from north to east and lived

in Bihar for a couple of years

At Nalanda University Xuan Zang

interacted with students and scholars

mastered local languages and

discovered Buddhist stupas

Q26) Answer c

Explanation

Pradakshina patha is a circular path

laid around a stupa in Buddhist

architecture While the rest are a part of

temple architecture

Q27) Answer d

Explanation

All the above-mentioned temples have

an elaborate use of bricks (baked

bricks) along with stone

Q28) Ans (c)

Muhammad Quli Qutab was the Sultan

of Golconda He was a contemporary of

Akbar was very fond of literature and

architecture

The Sultan was a great poet and he

wrote in Dakhini Urdu Persian and

Telgu and has left an extensive diwan or

collection

Recently the Archaeological Survey of

India (ASI) will be using Ground

Penetrating Radar (GPR) to map the

contours of the area around the Bagh-e-

Naya Qila excavated garden inside the

Golconda Fort in Telangana

Q29) Answer a

Explanation

Silappadikaram is a famous Tamil epic

which was written by Ilango around

1800 years ago It is a story of a

merchant named Kovalan who fell in

love with a courtesan named Madhavi

Manimekalai tells the story of the

daughter of Kovalan and Madhavi

Q30) Answer (a)

Explanation

Charaka is the author of Charaka

Samhita which is an important work of

Ayurveda and medicines

Brahmaguptarsquos fame rests mostly on his

Brahma-sphuta-siddhanta which was

an astronomical work It was translated

into Arabic in Baghdad and had a major

impact on Islamic mathematics and

astronomy

Late in his life Brahmagupta wrote

Khandakhadyaka which was an

RAUSIAS-FC19E1003 44

astronomical handbook that employed

Aryabhatarsquos system of starting each day

at midnight

Q31) Answer (c)

Explanation

Amir Khusrau was a famous sufi

musician poet and scholar In 1318 he

noted that there was different language

in every region of this land (Hindustan)

Lahori Kashmiri Dvarsamudri (in

Southern Karnataka) Telangana (in

Andhra Pradesh) Gujari (in Gujarat)

Marsquobari (in Tamil Nadu) Awadhi (in

eastern Uttar Pradesh) and Hindawai (in

the area around in Delhi) etc He went

to explain that Sanskrit did not belong

to any region and that only brahmans

knew it

Q32) Answer c

Explanation

Hiranyagarbha refers to the golden

womb When this ritual was performed

with the help of Brahmanas it was

thought to lead to the rebirth of the

sacrificer as a Khastriya

Q33) Answer d

Explanation

Kadamai refers to a tax on land

revenue

Gwalior Prashasti describes the exploits

of Nagabhata who was a Pratihara king

Q34) Answer b

Explanation

Rajatarangini is a Sanskrit text written

by Kalhana in the 12th century

It was historical chronicle of early India

It is justifiably considered to be the best

and most authentic work of its kind

It covers the entire span of history in

the Kashmir region from the earliest

times to the date of its composition

Q35) Answer c

Explanation

ldquoUrrdquo was the general assembly of the

village ldquoUrrdquo consisted of all the

taxpaying residents of an ordinary

village

Q36) Answer (a)

Explanation

Tarikh was a form of history writing in

the Delhi Sultanate The authors of

tawarikhs were learned men which

included secretaries administrators etc

Q37 Answer (a)

Explanation

Alauddin chose to pay his soldiers salaries in cash rather than iqtas The soldiers would buy their supplies from merchants in Delhi and it was thus feared that merchants would raise their prices To stop this Alauddin controlled the prices of goods in Delhi Prices were carefully surveyed by officers and merchants who did not sell at the prescribed rates were punished

Q38) Answer (d)

Explanation

Delhi first became the capital of a

kingdom under the Tomara Rajputs

who were defeated in the middle of the

twelfth century by the Chauhans (also

referred to as Chahamanas) of Ajmer

It was under the Tomaras and

Chauhans that Delhi became an

important commercial centre Many rich

Jaina merchants lived in the city and

constructed several temples Coins

minted here called dehliwal had a wide

circulation

Q39) Answer (c)

Explanation

Moth ki Masjid was built in the reign of

Sikandar Lodi by his minister

Begumpuri mosque built in the reign of

Muhammad Tughluq was the main

mosque of Jahanpanah the ldquoSanctuary

of the Worldrdquo and his new capital in

Delhi

Quwwat al ndash Islam mosque was

enlarged by Iltutmish and Alauddin

Khalji The minar was built by three

Sultansndash Qutbuddin Aybak Iltutmish

and Firuz Shah Tughluq

RAUSIAS-FC19E1003 45

Q40) Answer (c)

Explanation

Under the Mughals mansabdar was

referred to an individual who held a

mansab ie rank and he received his

salary as revenue assignments called

jagirs

Q41) Ans (b)

The Quit India Movement was a

spontaneous revolt of people against

British rule

The All India Congress Committee met

at Bombay on 8 August 1942 It passed

the famous resolution Quit India and

proposed the starting of a non-violent

mass struggle under Gandhis

leadership to achieve this aim But on

the very next day Gandhi and other

eminent leaders of the Congress were

arrested The Congress was once again

declared illegal

Q42) Ans (c)

The Simon Commission refers to a

group of seven MPs from the United

Kingdom constituted to suggest

constitutional reforms for British India

The Commission consisted of only

British members headed by one of the

senior British politicians Sir John

Simon

So the people of India agitated against

the arrival of Simon Commission

Q43) Ans (a)

He was widely known for his

unfavourable opinion of the economic

consequences of the British rule in

India

In his many writings and speeches and

especially in Poverty and Un-British

Rule in India Naoroji argued that India

was too highly taxed and that its wealth

was being drained away to England

He did not interpret the ancient Indian

texts and restored the self-confidence of

Indians And also he did not stress the

need for eradication of all the social

evils before anything else

Q44) Ans (c)

In August 1932 Prime Minister

MacDonald announced his Communal

Award Great Britainrsquos unilateral

attempt to resolve the various conflicts

among Indiarsquos many communal

interests

The award which was later

incorporated into the act of 1935

expanded the separate-electorate

formula reserved for Muslims to other

minorities including Sikhs Indian

Christians Anglo-Indians Europeans

distinct regional groups Gandhi

undertook a ldquofast unto deathrdquo against

that offer which he viewed as a

nefarious British plot to divide the

Indian society

Q45) Ans (b)

In British India apart from existing

imports and exports there was also a

particular amount of money which

colonial India contributed towards

administration maintenance of the

army war expenses pensions to retired

officers and other expenses accrued by

Britain towards maintenance of her

colony These were known as Home

charges and were paid for almost

entirely by India

The Home charges was made of

following components-

- Interest payable on Indian debt

- Dividend to shareholders of East

India Company

- Funds used to support the India

Office in London

- Funds used to pay salaries and

pensions of British personnel

engaged in India

- Interest on the railways

- Civil and military charges

- Store purchases in England

Q46) Ans (b)

The Lahore session of the Indian

National Congress was held in 1929

under the Presidentship of Jawaharlal

Nehru

The Lahore session of the Indian

National Congress witnessed significant

RAUSIAS-FC19E1003 46

developments in the Indian national

movement

- First the election of Jawaharlal

Nehru to the post of Presidentship of

the Congress was a clear indication

of the growing strength of the

Leftists in the Congress

- Secondly it was in this session that

the Congress for the first time raised

the demand for complete

independence Such demand was

not raised from the Congress

platform earlier

Q47) Ans (b)

It did not provide for separate

electorates for any community or

weightage for minorities However it did

allow for the reservation of minority

seats in provinces having minorities of

at least ten per cent but this was to be

in strict proportion to the size of the

community

There was no provision for complete

Independence for India

Q48) Ans (c)

The religion of early Vedic Aryans was

primarily of worship of nature and

Yajnas

The early Aryan religion was kind of

nature worship Actually the forces

around them which they could not

control or understand were invested

with divinity and were personified as

male or female gods And they

performed some Yajnas also

Q49) Ans (b)

The roads and river-routes were not

immune from robbery It is notable that

Yuan Chwang (Hiuen Tsang) was

robbed of his belongings during

Harshvardanarsquos period

Q50) Ans (c)

Q51) Ans (b)

Purandara Dasa was a saint and great

devotee of Lord Krishna

There is much speculation about where

Purandara Dasa regarded as the

Pitamaha of Carnatic music was born

Recently an expert committee

constituted by the Kannada University

Hampi has come to the conclusion that

Kshemapura Shivamogga district

Karnataka is the birth place of

Purandara Dasa

Q52) Ans (c)

Sri Tyagaraja Sri Shyama Shastry and Sri Muthuswami Dikshitar are considered the trinity of Carnatic music and with them came the golden age in Carnatic music in the 18th-19th

century

Q53) Ans d)

Recently a rare sarcophagus (stone

coffin) which is 2000 years old from the

Iron AgendashMegalithic era was discovered

from a rock-cut cave at Viyur village of

Kollam near Koyilandy in Kozhikode

district Kerala

The coffin containing bone fragments

was found during an excavation ldquoSo

far such a rare finding has been

discovered only from two sites

in Kerala Both these sarcophagi were

recovered from Megalithic sites at

Chevayur and Atholi also in Kozhikode

district

Q54) Ans a)

The megalithic culture in South India was a full-fledged Iron Age culture

Q55) Ans d)

The Cholas Pandyas and Keralaputras

(Cheras) mentioned in Ashokan

inscriptions were probably in the late

megalithic phase of material culture

Q56) Ans d)

Q57) Ans (b)

Raj Kumar Shukla followed Gandhiji all

over the country to persuade him to

come to Champaran to investigate the

problem associated with tinkathia

system

RAUSIAS-FC19E1003 47

Brij Kishore Rajendra Prasad Mahadev

Desai and Narhari Parikh accompanied

Gandhi ji during the Champaran

Satyagraha

Q58) Ans (b)

The Satvahanas started the practice of granting tax-free villages to brahmanas and Buddhist monks

Q59) Ans c)

The objectives of the Programme are

listed as under

- Developing basic tourism

infrastructure

- Promoting cultural and heritage

value of the country to generate

livelihoods in the identified regions

- Enhancing the tourist attractiveness

in a sustainable manner by

developing world-class

infrastructure at the heritage

monument sites

- Creating employment through active

involvement of local communities

- Harnessing tourism potential for its

effects on employment generation

and economic development

- Developing sustainable tourism

infrastructure and ensuring proper

Operations and maintenance

therein

Q60) Ans (b)

The Tribal Cooperative Marketing

Development Federation of India

(TRIFED) came into existence in 1987

It is a national-level apex organization

functioning under the administrative

control of Ministry of Tribal Affairs

Govt of India

TRIFED has its registered and Head

Office located in New Delhi

Q61) Ans (c)

Premchandrsquos novels include

Premashram Rangabhumi Ghaban

Karmabhumi and Godan

Gora is a novel written by Rabindranath

Tagore

138th birth anniversary of Munshi

Premchand was celebrated across the

country

Q62) Ans (b)

Giddha is a traditional pastoral dance

performed by the women of the Punjab

India and Pakistan at festival times

and at the sowing and reaping of the

harvest

By this dance the Punjabi women

reveal their joy expel their suppressed

feelings in a male dominated society

through the performance of Giddha

Since this dance has nothing to do with

men only women can participate in it

During the Teej celebrations Giddha

dance is celebrated in Punjab every

year Teej is a generic name for a

number of festivals that are celebrated

by women in some parts of India

Q63) Ans (a)

Dara Shukoh wrote the remarkable

work called ldquoMajma-ul-Bahrainrdquo or the

ldquoThe confluence of two seasrdquo

The Vice President of India Shri M

Venkaiah Naidu has said that Prince

Dara Shukohrsquos writings can come as a

refreshing source for infusing peace and

harmony He was addressing the

gathering after visiting the exhibition

that showcases the forgotten Prince of

yesteryears Dara Shukoh organized by

Mr Francois Gautier at Indira Gandhi

National Centre for the Arts in New

Delhi

Q64) Ans (c)

The statue Gommateshwara is

dedicated to the Jain God Bahubali

It is a monolithic statue

President Ram Nath Kovind

inaugurated the grand anointing

ceremony mdash Mahamastakabhisheka mdash

held once in 12 years at

Shravanabelagola (Karnataka)

Q65) Ans (c)

Prachi Valley had come up around the

Prachi river Prachi Valley gradually

disappeared

RAUSIAS-FC19E1003 48

The Prachi river originates from

Bhubaneswar

It is a tributary of the Mahanadi and

flows through the districts of Puri

Khurda Cuttack and Jagatsinghpur

and the entire region of the river is

termed as the Prachi Valley

It falls into the Bay of Bengal

Archaeological evidence shows that the

Prachi Valley Civilisation predates both

Harappa and Mohenjo-Daro

The Prachi river originates from

Bhubaneswar

Q66) Ans (d)

These monuments are located in

Chhatarpur district Madhya Pradesh

within Vindhya mountain range

Q67) Ans (a)

The book lsquoThoughts on Pakistanrsquo was

written by Dr BR Ambedkar

On the occasion of the birth anniversary

of Dr BR Ambedkar the president of

India pays homage to this icon of India

In 1924 he founded the Depressed

Classes Institute (Bahishkrit Hitkarini

Sabha) and in 1927 the Samaj Samata

Sangh

Another area of attention for Ambedkar

was education For its spread among

the low classes he set up a network of

colleges by the name of Peoples

Education Society and founded hostels

Q68) Ans(b)

Mehrgarh is a famous Neolithic

settlement in the Indian subcontinent

which is situated in Baluchistan

province Pakistan

A pre-historic rock art site is discovered

in the vast expanse of limestone blocks

on the eastern banks of Naguleru river

near Dachepalli (Andhra Pradesh) It

has thrown light on the Neolithic

civilisation that flourished in Guntur

(Andhra Pradesh) during 1500-2000

BC

Q69) Ans (c)

The 12th and the 13th centuries saw

the emergence of the Kakatiyas They

were at first the feudatories of the

Western Chalukyas of Kalyana Initially

they ruled over a small territory near

Warangal (Telangana)

They introduced Nayakships which was

later adopted and developed by the

Rayas of Vijayanagara

Q70) Ans (a)

The fast had effect of putting pressure

on mill owners who finally agreed to

give the workers a 35 per cent increase

in wages

Google celebrated with a doodle the

132nd birth anniversary of Anasuya

Sarabhai who played a pioneering role

in Indiarsquos labour movement

Q71) Ans (d)

The UNESCOrsquos list of the representative

list of the intangible cultural heritage of

humanity from India are

- Koodiyattam Sanskrit Theatre of

Kerala

- Mudiyettu ritual theatre and dance

drama of Kerala

- Tradition of Vedic Chanting

- Kalbelia folk songs and dances of

Rajasthan

- Ramlila Traditional Performance of

the Ramayana

- Sankirtana ritual singing

drumming and dancing of Manipur

- Ramman religious festival and

ritual theatre of the Garhwal

Himalayas India

- Traditional brass and copper craft of

utensil making among the Thatheras

of Jandiala Guru Punjab India

- Chhau dance classical Indian dance

originated in the eastern Indian

states

- Buddhist chanting of Ladakh

recitation of sacred Buddhist texts

in the trans-Himalayan Ladakh

region Jammu and Kashmir India

- Yoga

- Nouroz

- Kumbh Mela

RAUSIAS-FC19E1003 49

Q72) Ans(b)

The President of India Shri Ram Nath Kovind inaugurated the Hornbill Festival and State Formation Day celebrations of Nagaland in Kisama

The festival is named after the Indian hornbill the large and colourful forest bird which is displayed in the folklore of most of the states tribes

The major recognized tribes of Nagaland are Angami Ao Chakhesang Chang

Kuki Rengma and Zeling etc

Onge Jarawa and Sentinelese are the

tribes of Andman amp Nicobar Islands

Q73) Ans (c)

The Rashtrakutas rule in the Deccan lasted for almost two hundred years till the end of the tenth century The Rashtrakutas rulers were tolerant in their religious views and patronized not only Shaivism and Vaishnavism but

Jainism as well

The famous rock-cut temple of Shiva at Ellora was built by one of the Rashtrakutas kings Krishna I in the ninth century His successor Amoghavarsha was a Jain but he also

patronized other faiths

The Rashtrakutas allowed Muslims traders to settle and permitted Islam to

be preached in their dominions

Recently increasing defacement at the prehistoric rock paintings of Pandavulagutta Telangana has created a cause for grave concern It can spoil

the prehistoric rock

Pandavulagutta is home to

- Painted rock shelters dating to

10000 BC-8000 BC

- An 8th century inscription of the

Rashtrakuta period and

- Painted frescoes from the 12th century Kakatiya empire

Q74) Ans (b)

In 1828 Raja Ram Mohan Roy founded a new religious society the Brahma Sabha later known as the Brahmo

Samaj

Debendranath Tagore headed the Tattvabodhini Sabha which was

engaged in search of spiritual truth

Its purpose was to purify Hinduism and to preach monotheism or belief in one God

The new society was to be based on the twin pillars of reason and the Vedas and

Upanishads

Recently Sadharan Brahmo Samaj (SBS) has entered into a legal battle with the West Bengal government due

to some legal issue

Q75) Ans (c)

The Chishti order was established in India by Khwaja Moinuddin Chishti who came to India around 1192 The Chishtirsquos are considered to be the most influential of the groups of Sufis who migrated to India in the late twelfth century They adapted successfully to the local environment and adopted several features of Indian devotional

traditions

The historical dargah of Sufi mystic Khwaja Moinuddin Chishti in Ajmer is all set to get a facelift This 13 th century dargah has been included among the Swachh Iconic Places a clean-up initiative focused on iconic

heritage spiritual and cultural places

Page 5: GENERAL STUDIES (PAPER I) · Test is part of Rau’s IAS Test series for Preliminary Exam 2019 FOUNDATION + CURRENT AFFAIRS GENERAL STUDIES (PAPER –I) FOUNDATION TEST –III TOPIC:

RAUSIAS-FC19E1003 5

Q8) In the context of the life of Buddha

consider the following statements

1 Buddha gave his first sermon at

Sarnath near Varanasi

2 He belonged to the Licchavi clan

and passed away at Kusinara

Which of the above statements isare

correct

(a) 1 only

(b) 2 only

(c) Both 1 and 2

(d) Neither 1 nor 2

Q9) Consider the following pairs

1 Samkhya Kapila

2 Vaisheshika Jamini

3 Nyaya Gautama

Which of the pairs given above isare

correctly matched

(a) 1 and 2 only

(b) 2 and 3 only

(c) 1 and 3 only

(d) 1 2 and 3

Q10) Consider the following statements

1 The teachings of Mahavira were

compiled at Vaishali

2 Vinaya Pitaka talks about the

Buddhist Sangha

Which of the statements given above

isare correct

(a) 1 only

(b) 2 only

(c) Both 1 and 2

(d) Neither 1and 2

Q11) Consider the following statements

1 Arthashastra was compiled by

Vishnugupta

2 Magasthenes who came to the

court of Chandragupta Maurya

was the ambassador of Seleucus

Which of the statements given above

isare correct

(a) 1 only

(b) 2 only

(c) Both 1 and 2

(d) Neither 1 nor 2

Q12) Consider the following pairs

Pillar Capital Place

1 Lion Capital Sarnath

2 Bull Capital Rampurva

3 Elephant Capital Sankissa

Which of the pairs given above isare

matched correctly

(a) 1 and 2

(b) 3 only

(c) 1 and 3

(d) 1 2 and 3

Q13) Consider the following statements

regarding the Revolt of 1857

1 Ahmadullah Shah a maulvi from

Faizabad caught the imagination

of the people and raised a huge

force of supporters against the

Britishers

2 Kunwar Singh was a great fighter

from Uttar Pradesh

Which of the statements given above

isare correct

(a) 1 only

(b) 2 only

(c) Both 1 and 2

(d) Neither 1 nor 2

RAUSIAS-FC19E1003 6

Q14) निमननिखित यग ो पर निचार कीनजए

1 िललािर सनिक

2 आनदमाई दास (गिाम)

3 कदनसयर भनमहीि मजदर

उपयणकत यग ो म स कौि-स सही समनित ह

(a) किि 1 और 2

(b) किि 2 और 3

(c) किि 1 और 3

(d) 1 2 और 3

Q15) निमननिखित सथि ो म स उस सथि की पहचाि कीनजए

ज एक परनसदध तटीय बसती थी और जहा एरटाइि

(Arretine) मदभाोड पाया गया था

(a) महाबिीपरम

(b) तामरनिखपत

(c) अररकमड

(d) अतरोजीिरा

Q16) सोगम सानहतय क सोदभण म निमननिखित कथि ो पर

निचार कीनजए

1 मदरई म सोगमसभाए आय नजत की जाती ो

थी ो

2 मिनदर शबद का परय ग सोगम कनिताओो म

नकया जाता था नजसका अथण ह द राजाओो

का एक समह

उपयणकत कथि ो म स कौि-सास सही हह

(a) किि 1

(b) किि 2

(c) 1 और 2 द ि ो

(d) ि त 1 ि ही 2

Q17) निमननिखित कथि ो पर निचार कीनजए

1 आरखिक िनदक काि म मनहिाऐो सभाओो

और निदाथाओो म भाग िती थी ो

2 आरखिक िनदक काि म िनशषठ ज एक

पजारी ह ता था की एक महतवपरण भनमका

ह ती थी

उपयणकत कथि ो म स कौि-सास सही हह

(a) किि 1

(b) किि 2

(c) 1 और 2 द ि ो

(d) ि त 1 ि ही 2

Q18) निमननिखित कथि ो पर निचार कीनजए

1 किाणटक म जि रमण क निसतार का शरय

चनदरगपत मौयण क जाता ह

2 जि रमण ि ईशवर क अखसततव क मानयता दी ह

तथा िरण पररािी की भरतणिा की ह

उपयणकत कथि ो म स कौि-सास सही हह

(a) किि 1

(b) किि 2

(c) 1 और 2 द ि ो

(d) ि त 1 ि ही 2

Q19) निमननिखित यग ो पर निचार कीनजए

1 पहार पाणडय

2 मदरई च ि

उपयणकत यग ो म स कौि-सास सही समनित हह

(a) किि 1

(b) किि 2

(c) 1 और 2 द ि ो

(d) ि त 1 ि ही 2

RAUSIAS-FC19E1003 7

Q14) Consider the following pairs

1 Vellalar Soldier

2 Adimai Slaves

3 Kadaisiyar Landless labourers

Which of the pairs given above isare

correctly matched

(a) 1 and 2 only

(b) 2 and 3 only

(c) 1 and 3 only

(d) 1 2 and 3

Q15) Identify the site which was a famous

coastal settlement from which Arretine

ware has been found

(a) Mahabalipuram

(b) Tamralipti

(c) Arikamedu

(d) Atranjikhera

Q16) With respect to the Sangam literature

consider the following statements

1 The Sangamsassemblies were

held at Madurai

2 The term muvendar was used in

sangam poems which mean a

group of two kings

Which of the statements given above

isare correct

(a) 1 only

(b) 2 only

(c) Both 1 and 2

(d) Neither 1 nor 2

Q17) Consider the following statements

1 During Early Vedic Period women

attended the Sabha and Vidatha

2 The priest ndash Vasishtha played

important role in Early Vedic

Period

Which of the statements given above

isare correct

(a) 1 only

(b) 2 only

(c) Both 1 and 2

(d) Neither 1 nor 2

Q18) Consider the following statements

1 The spread of Jainism in

Karnataka is attributed to

Chandragupta Maurya

2 Jainism recognised the existence

of the gods and condemns the

Varna System

Which of the statements given above

isare correct

(a) 1 only

(b) 2 only

(c) Both 1 and 2

(d) Neither 1 nor 2

Q19) Consider the following pairs

1 Puhar Pandyas

2 Madurai Cholas

Which of the pairs given above isare

correct

(a) 1 only

(b) 2 only

(c) Both 1 and 2

(d) Neither 1 nor 2

RAUSIAS-FC19E1003 8

Q20) निमननिखित कथि ो म स कौि-सास सही हह

1 बदधचररत का ििक िागसि ह

2 ब नरसतव की पजा महायाि बौदधमत का एक

महतवपरण भाग थी

िीच नदए गए कट का परय ग कर सही उततर चनिए

(a) किि 1

(b) किि 2

(c) 1 और 2 द ि ो

(d) ि त 1 ि ही 2

Q21) निमननिखित कथि ो म स कौि-सास सही हह

1 भखकत क निचार क भागित गीता म सपषट

नकया गया ह

2 तनमि कनि अपपर एक अििर सोत थ

िीच नदए गए कट का परय ग कर सही उततर चनिएः

(a) किि 1

(b) किि 2

(c) 1 और 2 द ि ो

(d) ि त 1 ि ही 2

Q22) निमननिखित म स नकस शासक ि िीरा बजात हए

अपिी छनि नसक ो पर अोनकत करिाई थी

(a) नमिाोदर

(b) चनदरगपत मौयण

(c) गौतमीपतर सतकरी

(d) समदरगपत

Q23) निमननिखित कथि ो म स कौि-सास सही हह

1 सवतोतर भारत ि निकरम सोित क राषटर ीय

किनडर क रप म अपिाया और यह 68 ईसा

पिण म आरि हआ था

2 बािभटट हिणिरणि क दरबार का एक कनि था

िीच नदए गए कट का परय ग कर सही उततर चनिएः

(a) किि 1

(b) किि 2

(c) 1 और 2 द ि ो

(d) ि त 1 ि ही 2

Q24) निमननिखित यग ो पर निचार कीनजएः

1 सोनर-निगरानहका वयापार मोतरी

2 परथम-कनिका परमि नशलपकार

3 साथणिाह परमि बकर

उपयणकत यग ो म स कौि-सास सही समनित हह

(a) किि 1

(b) किि 1 और 3

(c) किि 2

(d) किि 2 और 3

Q25) निमननिखित कथि ो म स कौि-सास सही हह

1 lsquoएह ि नशिाििrsquo पिकनशि नदवतीय स

सोबोनरत ह और रनिकनत क दवारा इसकी रचिा

की गई थी

2 जआि झाोग समदरगपत नदवतीय क शासिकाि

म भारत आया था

िीच नदए गए कट का परय ग कर सही उततर चनिएः

(a) किि 1

(b) किि 2

(c) 1 और 2 द ि ो

(d) ि त 1 ि ही 2

RAUSIAS-FC19E1003 9

Q20) Which of the following statements

isare correct

1 Buddhacharita is authored by

Nagasena

2 The worship of Bodhisattvas was

an important part of Mahayana

Buddhsim

Select the correct answer using the code

given below

(a) 1 only

(b) 2 only

(c) Both 1 and 2

(d) Neither 1 nor 2

Q21) Which of the following statements

isare correct

1 The idea of Bhakti is elucidated in

Bhagavata Gita

2 Tamil poet Appar was an Alvar

saint

Select the correct answer using the code

given below

(a) 1 only

(b) 2 only

(c) Both 1 and 2

(d) Neither 1 nor 2

Q22) Which of the following rulers had his

image inscribed in the coins while

playing a veena

(a) Meander

(b) Chandragupta Maurya

(c) Gautamiputra Satkarni

(d) Samudragupta

Q23) Which of the following statements

isare correct

1 Vikrama Samvat is adopted as the

national calendar by independent

India and it began in 68 BC

2 Banabhatta was a court poet of

Harshavardhana

Select the correct answer using the code

given below

(a) 1 only

(b) 2 only

(c) Both 1 and 2

(d) Neither 1 nor 2

Q24) Consider the following pairs

1 Sandhi-vigrahika Minister of trade

2 Prathama-kulika Chief craftsman

3 Sarthavaha Chief banker

Which of the pairs given above isare

correct

(a) 1 only

(b) 1 and 3 only

(c) 2 only

(d) 2 and 3 only

Q25) Which of the following statements

isare correct

1 Aihole inscription belongs to

Pulakeshin II and was composed

by Ravikriti

2 Xuan Zang came to India during

the reign of Chandragupta II

Select the correct answer using the code

given below

(a) 1 only

(b) 2 only

(c) Both 1 and 2

(d) Neither 1 nor 2

RAUSIAS-FC19E1003 10

Q26) िासतकिा स सोबोनरत निमननिखित ततव ो म स कौि-स

ततव किि नहोद मखनदर ो की िासतकिा क भाग ह

1 नशिर

2 मणडप

3 परदनकषरा पथ

4 गभणगह

िीच नदए गए कट का परय ग कर सही उततर चनिएः

(a) किि 1 3 और 4

(b) किि 2 3 और 4

(c) किि 1 2 और 4

(d) 1 2 3 और 4

Q27) निमननिखित मोनदर ो म स कौि-स मोनदर ईोट ो स बि ह

1 दिगढ़ मोनदर

2 भीतरगाि मोनदर

3 िकषमर मोनदर नसरपर

4 बहदशवर मोनदर

िीच नदए गए कट का परय ग कर सही उततर चनिएः

(a) किि 1 2 और 3

(b) किि 2 3 और 4

(c) किि 1 3 और 4

(d) 1 2 3 और 4

Q28) निमननिखित कथि ो म स कौि-सास सही हह

1 सलताि महममद किी कतब शाह अकबर का

समकािीि था

2 िासतकिा क कषतर म महममद किी कतब शाह

ि कई ईमारत ो का निमाणर करिाया था नजिम

स चार मीिार सिाणनरक परनसदध ह

िीच नदए गए कट का परय ग कर सही उततर चनिएः

(a) किि 1

(b) किि 2

(c) 1 और 2 द ि ो

(d) ि त 1 ि ही 2

Q29) निमननिखित यग ो पर निचार कीनजएः

1 मनिमकिाई सततिार

2 अनभजञाि शाको तिम कानिदास

3 नसिपपानदकारम क ििि

उपयणकत यग ो म स कौि-सास सही समनित हह

(a) किि 1 और 2

(b) किि 2

(c) किि 1 और 3

(d) 1 2 और 3

Q30) निमननिखित कथि ो म स कौि-सास सही हह

1 ldquoसशरत सोनहताrdquo नचनकरता पर एक महतवपरण

रचिा ह

2 बरहमगपत और चरक महतवपरण गनरतजञ थ

िीच नदए गए कट का परय ग कर सही उततर चनिए

(a) किि 1

(b) किि 2

(c) 1 और 2 द ि ो

(d) ि त 1 ि ही 2

Q31) अमीर िसर क सनदभण म निमननिखित कथि ो म स

कौि-सास सही हह

1 अमीर िसर ि अपिी रचिाओो म नििा ह

नक सोसकत नकसी भी कषतर स सोबोनरत िही ो थी

और किि बराहमर ही इस भािा का जञाि रित

2 उन ोि नहोदिी और अिरी क अखसततव का

उललि नकया था

िीच नदए गए कट का परय ग कर सही उततर चनिए

(a) किि 1

(b) किि 2

(c) 1 और 2 द ि ो

(d) ि त 1 ि ही 2

RAUSIAS-FC19E1003 11

Q26) Which of the following architectural

elements were only part of Hindu

temple architecture

1 Shikhara

2 Mandapa

3 Pradakshina patha

4 Garbhagriha

Select the correct answer using the code

given below

(a) 1 3 and 4 only

(b) 2 3 and 4 only

(c) 1 2 and 4 only

(d) 1 2 3 and 4

Q27) Which of the following temples isare

made of bricks

1 Deogarh Temple

2 Bhitargaon Temple

3 Lakshmana temple Sirpur

4 Brihadeshvara Temple

Select the correct answer using the code

given below

(a) 1 2 and 3 only

(b) 2 3 and 4 only

(c) 1 3 and 4 only

(d) 1 2 3 and 4

Q28) Which of the following statements

isare correct

1 Sultan Muhammad Quli Qutab

Shah was a contemporary of

Akbar

2 In the field of architecture

Muhammad Quli Qutab Shah

constructed many buildings the

most famous of which is the Char

Minar

Select the correct answer using the code

given below

(a) 1 only

(b) 2 only

(c) Both 1 and 2

(d) Neither 1 nor 2

Q29) Consider the following pairs

1 Manimekalai Sattanar

2 Abhijnana Shakuntalam Kalidasa

3 Silappadikaram Kovalan

Which of the pairs given above isare

correct

(a) 1 and 2 only

(b) 2 only

(c) 1 and 3 only

(d) 1 2 and 3

Q30) Which of the following statements

isare correct

1 Sushruta Samhita is an important

work on medicine

2 Brahmagupta and Charaka were

important mathematicians

Select the correct answer using the code

given below

(a) 1 only

(b) 2 only

(c) Both 1 and 2

(d) Neither 1 nor 2

Q31) Which of the following statements

isare correct about Amir Khusrau

1 Amir Khusrau records in his works

that Sanskrit did not belong to any

region and only the Brahmans

knew it

2 He recorded the existence of

Hindawi and Awadhi

Select the correct answer using the code

given below

(a) 1 only

(b) 2 only

(c) Both 1 and 2

(d) Neither 1 nor 2

RAUSIAS-FC19E1003 12

Q32) निमननिखित कथि ो पर निचार कीनजए

1 नहरणय-गभण अिषठाि क बार म ऐसा स चा जाता

था नक बनि दि िाि का एक कषनतरय क रप म

पिजणनम ह गा

2 मयरशमणि कदोब िोश का सोसथापक था

उपयणकत कथि ो म स कौि-सास सही हह

(a) किि 1

(b) किि 2

(c) 1 और 2 द ि ो

(d) ि त 1 ि ही 2

Q33) निमननिखित कथि ो म स कौि-सास सही हह

1 कदमई बगार (बिपिणक शरम) क रप म

निया जाि िािा कर था

2 गवानियर परशखसत म िागभट (ज एक चोदि

राजा था) क दवारा नकय गए श िर का िरणि

नकया गया ह

िीच नदए गए कट का परय ग कर सही उततर चनिए

(a) किि 1

(b) किि 2

(c) 1 और 2 द ि ो

(d) ि त 1 ि ही 2

Q34) निमननिखित कथि ो म स कौि-सास सही हह

1 राजतरो नगिी 11िी ो शताबदी म कलहि क दवारा

रनचत एक सोसकत पसतक (टकसट) ह

2 कननौज क निए नतरपकषीय सोघिण म पाि राजिोश

शानमि था

िीच नदए गए कट का परय ग कर सही उततर चनिए

(a) किि 1

(b) किि 2

(c) 1 और 2 द ि ो

(d) ि त 1 ि ही 2

Q35) निमननिखित यग ो पर निचार कीनजए

1 बरहदशवर मोनदर राजराजा च ि

2 उर मापि की इकाई

3 दिदाि मोनदर ो क भनम अिदाि

उपयणकत यग ो म स कौि-स सही समनित ह

(a) किि 1 और 2

(b) किि 2 और 3

(c) किि 1 और 3

(d) 1 2 और 3

Q36) निमननिखित कथि ो म स कौि-सास सही हह

1 नदलली क सलताि ो क अरीि परशासि की भािा

फारसी थी

2 नदलली सलतित म ldquoतारीितािरीिrdquo कनिता

का एक रप था

िीच नदए गए कट का परय ग कर सही उततर चनिए

(a) किि 1

(b) किि 2

(c) 1 और 2 द ि ो

(d) ि त 1 ि ही 2

Q37) निमननिखित कथि ो म स कौि-सास सही हह

1 अिाउददीि खििजी ि अपि सनिक ो क निए

नसरी िाम का एक िया दगणरकषक शहर

बिािाया था

2 िह अपि सनिक ो क िति का भगताि इकता

क रप म करता था

िीच नदए गए कट का परय ग कर सही उततर चनिए

(a) किि 1

(b) किि 2

(c) 1 और 2 द ि ो

(d) ि त 1 ि ही 2

RAUSIAS-FC19E1003 13

Q32) Consider the following statements

1 Hiranya-garbha ritual was thought

to lead to the rebirth of the

sacrificer as a Kshatriya

2 Mayurasharman was the founder

of the Kadamba dynasty

Which of the statements given above

isare correct

(a) 1 only

(b) 2 only

(c) Both 1 and 2

(d) Neither 1 nor 2

Q33) Which of the following statements

isare correct

1 Kadamai was tax taken in form of

forced labour

2 Gwalior Prashasti describes the

exploits of Nagabhata who was a

Chandella king

Select the correct answer using the code

given below

(a) 1 only

(b) 2 only

(c) Both 1 and 2

(d) Neither 1 nor 2

Q34) Which of the following statements

isare correct

1 Rajatarangini is a Sanskrit text

written by Kalhana in the 11th

century

2 Pala dynasty was included in the

tripartite struggle for Kannauj

Select the correct answer using the code

given below

(a) 1 only

(b) 2 only

(c) Both 1 and 2

(d) Neither 1 nor 2

Q35) Consider the following pairs

1 Brihadeshvara temple Rajaraja

Chola

2 ldquoUrrdquo Unit of measurement

3 Devadana Land grants made to

temples

Which of the pairs given above isare

correct

(a) 1 and 2 only

(b) 2 and 3 only

(c) 1 and 3 only

(d) 1 2 and 3

Q36) Which of the following statements

isare correct

1 The language of administration

under the Delhi Sultans was

Persian

2 Tarikhtawarikh was a form of

poetry in the Delhi Sultanate

Select the correct answer using the code

given below

(a) 1 only

(b) 2 only

(c) Both 1 and 2

(d) Neither 1 nor 2

Q37) Which of the following statements

isare correct

1 Alauddin Khilji constructed a new

garrison town named Siri for his

soldiers

2 He paid his soldiers their salaries

in the form of Iqta

Select the correct answer using the code

given below

(a) 1 only

(b) 2 only

(c) Both 1 and 2

(d) Neither 1 nor 2

RAUSIAS-FC19E1003 14

Q38) निमननिखित कथि ो म स कौि-सास सही हह

1 नदलली कतबददीि एबक क अरीि पहिी बार

नकसी सामराजय की राजरािी बिी थी

2 दहिीिाि नसक ो का मदरर मग़ि ो क दवारा

नकया गया था

िीच नदए गए कट का परय ग कर सही उततर चनिए

(a) किि 1

(b) किि 2

(c) 1 और 2 द ि ो

(d) ि त 1 ि ही 2

Q39) निमननिखित यग ो पर निचार कीनजए

1 म ठ की मखिद नसको दर ि दी

2 बगमपरी मखिद नफर ज शाह तगिक

3 कववत- अि - इसलाम कतबददीि ऐबक

उपयणकत यग ो म स कौि-स सही समनित ह

(a) किि 1 और 2

(b) किि 2 और 3

(c) किि 1 और 3

(d) 1 2 और 3

Q40) निमननिखित कथि ो म स कौि-सास सही हह

1 मिसबदार ो क अपिा िति राजसव कायो

नजन जागीर कहत थ क रप म परापत ह ता

था

2 मिसबदार क ज सनय उततरदानयतव सौोप जात

थ उसक अनतगणत उस एक निराणररत सखया म

सिार अथिा घड़सिार ो का रि-रिाि करिा

पड़ता था

िीच नदए गए कट का परय ग कर सही उततर चनिए

(a) किि 1

(b) किि 2

(c) 1 और 2 द ि ो

(d) ि त 1 ि ही 2

Q41) ldquo1942 क भारत छ ड़ आोद ििrdquo क बार म

निमननिखित अिि कि ो म स कौि-सा सतय िही ो ह

(a) यह एक अनहोसक आोद िि था

(b) इसका िततव महातमा गाोरी क दवारा नकया गया

था

(c) यह एक सवाभानिक आोद िि था

(d) इसि सामानयतया शरनमक िगण क आकनिणत

िही ो नकया था

Q42) भारत क ि ग ो ि ldquoसाइमि कमीशिrdquo क आगमि क

निरदध आोद िि नकया था कय ोनक

(a) भारतीय कभी भी 1919 क अनरनियम (The

Act of 1919) क काम की समीकषा िही ो करिा

चाहत थ

(b) साइमि कमीशि ि पराोत ो म दवर (द हर) शासि

क समापत करि की नसफाररश की थी

(c) साइमि कमीशि म क ई भारतीय सदसय िही ो

था

(d) साइमि कमीशि ि दश क निभाजि का

सझाि नदया था

Q43) निमननिखित कथि ो पर निचार कीनजए

भारतीय राषटर ीय आोद िि म दादाभाई िौर जी क दवारा

नकया गया सबस परभािी य गदाि यह था नक उन ोि

1 अोगरज ो क दवारा भारत क आनथणक श िर का

ििासा नकया था

2 पराचीि भारतीय गरोथ ो की वयाखया की थी और

भारतीय ो क आतमनिशवास क पिःसथानपत नकया

था

3 अनय नकसी भी बात स पहि सभी सामानजक

बराइय ो क उनमिि की आिशयकता पर बि

नदया था

उपयणकत कथि ो म स कौि-सास सही हह

(a) किि 1

(b) किि 2 और 3

(c) किि 1 और 3

(d) 1 2 और 3

RAUSIAS-FC19E1003 15

Q38) Which of the following statements

isare correct

1 Delhi first became the capital of a

kingdom under Qutubuddin

Aibak

2 Dehliwal coins were minted by the

Mughals

Select the correct answer using the code

given below

(a) 1 only

(b) 2 only

(c) Both 1 and 2

(d) Neither 1 nor 2

Q39) Consider the following pairs

1 Moth ki Masjid- Sikander Lodi

2 Begumpuri mosque- Firuz Shah

Tughluq

3 Quwwat al ndash Islam- Qutubuddin

Aibak

Which of the above pairs isare correct

(a) 1 and 2 only

(b) 2 and 3 only

(c) 1 and 3 only

(d) 1 2 and 3

Q40) Which of the following statements

isare correct

1 Mansabdars received their salaries

as revenue assignments called

jagirs

2 The mansabdarrsquos military

responsibilities required him to

maintain a specified number of

sawar or cavalrymen

Select the correct answer using the code

given below

(a) 1 only

(b) 2 only

(c) Both 1 and 2

(d) Neither 1 nor 2

Q41) Which one of the following observations

is not true about the Quit India

Movement of 1942

(a) It was a non-violent movement

(b) It was led by Mahatma Gandhi

(c) It was a spontaneous movement

(d) It did not attract the labour class

in general

Q42) The people of India agitated against the

arrival of the Simon Commission

because

(a) Indians never wanted the review of

the working of the Act of 1919

(b) Simon Commission recommended

the abolition of dyarchy in the

Provinces

(c) there was no Indian member in the

Simon Commission

(d) the Simon Commission suggested

the partition of the country

Q43) Consider the following statements

The most effective contribution made by

Dadabhai Naoroji to the cause of Indian

National Movement was that he-

1 exposed the economic exploitation

of India by the British

2 interpreted the ancient Indian

texts and restored the self-

confidence of Indians

3 stressed the need for eradication of

all the social evils before anything

else

Which of the statements given above

isare correct

(a) 1 only

(b) 2 and 3 only

(c) 1 and 3 only

(d) 1 2 and 3

RAUSIAS-FC19E1003 16

Q44) महातमा गाोरी ि 1932 म आमरर अिशि नकया था

कय ोनक

(a) ldquoग िमज सममििrdquo (The Round Table

Conference) भारतीय राजिीनतक

आकाोकषाओो क परा करि म असफि रहा था

(b) काोगरस और मखसलम िीग म मतभद थ

(c) रामस मकड िालड (Ramsay Macdonald)

ि ldquoसाोपरदानयक परसकारrdquo (The Communal

Award) की घ िरा की थी

(d) ldquoसनििय अिजञा आोद ििrdquo (The Civil

Disobedience Movement) असफि रहा

था

Q45) भारत म औपनििनशक शासि की अिनर क सोदभण म

भारत स रि क बनहगणमि का एक महतवपरण भाग गह

शलक (Home Charges) था निमननिखित म स

कौि-सास क ि गह शलक म सखममनित नकया गया

थानकय गए थ

1 िोदि म भारत कायाणिय क निए उपय ग नकय

जाि िािा क ि

2 भारत म नियकत नबरनटश कनमणय ो क िति और

पशि का भगताि करि क निए उपय ग नकय

जाि िािा क ि

3 अोगरज ो क दवारा भारत क बाहर यदध ो क निए

उपय ग नकय जाि िािा क ि

िीच नदए गए कट का परय ग कर सही उततर चनिए

(a) किि 1

(b) किि 1 और 2

(c) किि 2 और 3

(d) 1 2 और 3

Q46) सवतोतरता आोद िि क इनतहास म भारतीय राषटर ीय

काोगरस का 1929 का सतर महतवपरण ह कय ोनक इसम

(a) काोगरस क उददशय क रप म सथािीय सरकार

की पराखपत की घ िरा की गई थी

(b) परण सवराज की पराखपत क काोगरस क िकषय क

रप म अपिाया गया था

(c) असहय ग आोद िि शर नकया गया था

(d) िोदि म ldquoग ि मर सममििrdquo (The Round

Table Conference) म भाग िि का निरणय

निया गया था

Q47) भारतीय सवतोतरता सोगराम क सोदभण म िहर ररप टण

क दवारा निमननिखित म स नकसकी नसफाररश की गई

थीनकिकी नसफाररश की गई थी ो

1 भारत क निए परण सवतोतरता

2 अलपसोखयक ो क निए सीट ो क आरकषर क

निए सोयकत नििाणचक मोडि

3 सोनिराि म भारत क ि ग ो क निए मौनिक

अनरकार ो का परािराि

िीच नदए गए कट का परय ग कर सही उततर चनिए

(a) किि 1

(b) किि 2 और 3

(c) किि 1 और 3

(d) 1 2 और 3

Q48) आरो नभक िनदक आयो का रमण मखय रप स था

(a) भखकत

(b) मनतण पजा और यजञ

(c) परकनत की पजा और यजञ

(d) परकनत की पजा और भखकत

RAUSIAS-FC19E1003 17

Q44) Mahatma Gandhi undertook fast unto

death in 1932 mainly because

(a) The Round Table Conference failed

to satisfy Indian political

aspirations

(b) The Congress and Muslim League

had differences of opinion

(c) Ramsay Macdonald announced the

Communal Award

(d) The Civil Disobedience Movement

failed

Q45) With reference to the period of colonial

rule in India ldquoHome Chargesrdquo formed

an important part of drain of wealth

from India Which of the following funds

constituted ldquoHome Chargesrdquo

1 Funds used to support the India

Office in London

2 Funds used to pay salaries and

pensions of British personnel

engaged in India

3 Funds used for waging wars

outside India by the British

Select the correct answer using the code

given below

(a) 1 only

(b) 1 and 2 only

(c) 2 and 3 only

(d) 1 2 and 3

Q46) The 1929- Session of Indian National

Congress is of significance in the history

of the Freedom Movement because the-

(a) attainment of Self-Government

was declared as the objective of

the Congress

(b) attainment of Poorna Swaraj was

adopted as the goal of the

Congress

(c) Non-Cooperation Movement was

launched

(d) decision to participate in the

Round Table Conference in

London was taken

Q47) With reference to the period of Indian

freedom struggle which of the following

waswere recommended by the Nehru

report

1 Complete Independence for India

2 Joint electorates for reservation of

seats for minorities

3 Provision of fundamental rights for

the people of India in the

Constitution

Select the correct answer using the code

given below

(a) 1 only

(b) 2 and 3 only

(c) 1 and 3 only

(d) 1 2 and 3

Q48) The religion of the early Vedic Aryans was primarily of

(a) Bhakti

(b) image worship and Yajnas

(c) worship of nature and Yajnas

(d) worship of nature and Bhakti

RAUSIAS-FC19E1003 18

Q49) भारत की यातरा करि िाि चीिी यातरी यआि चिाोग

(हयएि साोग) ि समकािीि भारत की सामानय

खसथनतय ो और सोसकनत क दजण नकया था इस सोदभण म

निमननिखित कथि ो म स कौि-सास सही हह

1 सड़क और िदी-मागण (जि-मागण) डकती स

परण रप स सरनकषत थ

2 जहा तक अपरार ो क निए दणड की बात ह

उसक निए नकसी भी वयखकत की निदोिता

अथिा उसक अपरार क निराणररत करि क

निए अनि जि और निि परि क माधयम क

सारि थ

3 वयापाररय ो क घाट ो और परनतबोर सटशि ो पर

शलक ो का भगताि करिा पड़ता था

िीच नदए गए कट का परय ग कर सही उततर चनिए

(a) किि 1

(b) किि 2 और 3

(c) किि 1 और 3

(d) 1 2 और 3

Q50) नसोर घाटी सभयता क सोदभण म निमननिखित कथि ो पर

निचार कीनजए

1 यह मखय रप स एक रमणनिरपकष सभयता थी

तथा हािाोनक इसम रानमणक ततव मौजद था

िनकि िह परनतिश पर हािी िही ो था

2 इस काि क दौराि भारत म कपास का परय ग

कपड़ा बिाि क निए नकया जाता था

उपयणकत कथि ो म स कौि-सास सही हह

(a) किि 1

(b) किि 2

(c) 1 और 2 द ि ो

(d) ि त 1 ि ही 2

Q51) परोदर दास क सोदभण म निमननिखित कथि ो पर निचार

कीनजए

1 परोदर दास एक सोत और भगिाि नशि क एक

महाि भकत थ

2 ि एक सोगीतकार गायक और किाणटक सोगीत

क मखय सोसथापक-परसतािक ो म स एक थ

उपयणकत कथि ो म स कौि-सास सही हह

(a) किि 1

(b) किि 2

(c) 1 और 2 द ि ो

(d) ि त 1 ि ही 2

Q52) निमननिखित म स कौि-सास वयखकत किाणटक सोगीत

की नतरमनतण म शानमि हह

1 बािामरिी कषणा

2 शरी शयाम शासतरी

3 शरी मथसवामी दीनकषतर

िीच नदए गए कट का परय ग कर सही उततर चनिए

(a) किि 1

(b) किि 2

(c) किि 2 और 3

(d) 1 2 और 3

Q53) चियर (Chevayur) और अथ िी (Atholi) म खसथत

महापािार सथि निमननिखित म स नकस राजय म खसथत

(a) तनमििाड

(b) किाणटक

(c) पनिम बोगाि

(d) करि

RAUSIAS-FC19E1003 19

Q49) The Chinese traveller Yuan Chwang

(Hiuen Tsang) who visited India

recorded the general conditions and

culture of India at that time In this

context which of the following

statements isare correct

1 The roads and river-routes were

completely immune from robbery

2 As regards punishment for

offences ordeals by fire water and

poison were the instruments for

determining the innocence or guilt

of a person

3 The tradesmen had to pay duties

at ferries and barrier stations

Select the correct answer using the code

given below

(a) 1 only

(b) 2 and 3 only

(c) 1 and 3 only

(d) 1 2 and 3

Q50) Regarding the Indus Valley Civilization

consider the following statements

1 It was predominantly a secular

civilization and the religious

element though present did not

dominate the scene

2 During this period cotton was

used for manufacturing textiles in

India

Which of the statements given above

isare correct

(a) 1 only

(b) 2 only

(c) Both 1 and 2

(d) Neither 1 nor 2

Q51) Consider the following statements

regarding Purandara Dasa

1 Purandara Dasa was a saint and

great devotee of Lord Shiva

2 He was a composer singer and

one of the chief founding-

proponents of the Carnatic music

Which of the statements given above

isare correct

(a) 1 only

(b) 2 only

(c) Both 1 and 2

(d) Neither 1 nor 2

Q52) Which of the following persons isare

included in the trinity of Carnatic

music

1 Balamurali Krishna

2 Sri Shyama Shastry

3 Sri Muthuswami Dikshitar

Select the correct answer using the code

given below

(a) 1 only

(b) 2 only

(c) 2 and 3 only

(d) 1 2 and 3

Q53) Megalithic sites at Chevayur and Atholi

are located in which of the following

states

(a) Tamil Nadu

(b) Karnataka

(c) West Bengal

(d) Kerala

RAUSIAS-FC19E1003 20

Q54) निमननिखित कथि ो पर निचार कीनजए

1 महापािानरक ि ग कबर ो म िसतएो दफिात थ

2 दनकषर भारत म महापािार सोसकनत एक परण

निकनसत तामर यगीि सोसकनत थी

उपयणकत कथि ो म स कौि-सास सही हह

(a) किि 1

(b) किि 2

(c) 1 और 2 द ि ो

(d) ि त 1 ि ही 2

Q55) निमननिखित म स कौि-स सामराजयसामराजय ो का

अश क क अनभिि ो म उललि नकया गया ह

1 च ि

2 पाणडय

3 करिपतर (चर)

िीच नदए गए कट का परय ग कर सही उततर चनिए

(a) किि 1

(b) किि 1 और 2

(c) किि 3

(d) 1 2 और 3

Q56) भीमा-क रगाोि का यदध को पिी क सनिक ो और

बाजीराि नदवतीय क िततव म एक शखकतशािी पशिा

सिा (मराठ ो) क मधय िड़ा गया था यह यदध

निमननिखित म स नकसका नहससा था

(a) परथम आोगल-मराठा यदध का

(b) नदवतीय आोगल-मराठा यदध का

(c) ततीय आोगल-मसर यदध का

(d) ततीय आोगल-मराठा यदध का

Q57) निमननिखित कथि ो पर निचार कीनजए

1 महादि दसाई ि गाोरीजी क चोपारर आि तथा

नतिकनथया पररािी स जड़ी समसया की जाोच

क निए रारी करि क निए दश भर म उिका

अिसरर नकया था

2 िरहरी पाररि चोपारर सतयागरह क दौराि

गाोरीजी क साथ थ

उपयणकत कथि ो म स कौि-सास सही हह

(a) किि 1

(b) किि 2

(c) 1 और 2 द ि ो

(d) ि त 1 ि ही 2

Q58) निमननिखित कथि ो पर निचार कीनजए

1 िनद राज-िोश ि बराहमर ो और बौदध मठराररय ो

क कर-मकत गाि अिदाि म दि की परथा

आरि की थी

2 सतिाहि ो की आनरकाररक भािा पराकत थी

उपयणकत कथि ो म स कौि-सास सही हह

(a) किि 1

(b) किि 2

(c) 1 और 2 द ि ो

(d) ि त 1 ि ही 2

Q59) एक निरासत क अपिाइए (अडॉपट ए हररटज ndash

Adopt a Heritage) पररय जिा क उददशय ो क

सनदभण म निमननिखित कथि ो पर निचार कीनजए

1 यह पररय जिा र रगार उतपादि और आनथणक

निकास क निए पयणटि कषमता का उि पर

परभाि का उपय ग करगी

2 यह पररय जिा निरासत सथि ो पर निशव सतरीय

आराररक सोरचिा निकनसत करक एक सतत

तरीक स पयणटक आकिणर म िखदध करगी

उपयणकत कथि ो म स कौि-सास सही हह

(a) किि 1

(b) किि 2

(c) 1 और 2 द ि ो

(d) ि त 1 ि ही 2

RAUSIAS-FC19E1003 21

Q54) Consider the following statements

1 Megalithic people buried goods in

graves

2 The megalithic culture in South

India was a full-fledged Copper

Age culture

Which of the statements given above

isare correct

(a) 1 only

(b) 2 only

(c) Both 1 and 2

(d) Neither 1 nor 2

Q55) Which of the following kingdoms isare

mentioned in the Ashokan inscriptions

1 Cholas

2 Pandyas

3 Keralaputras (Cheras)

Select the correct answer using the code

given below

(a) 1 only

(b) 1 and 2 only

(c) 3 only

(d) 1 2 and 3

Q56) The Battle of Bhima-Koregaon was

fought between the soldiers of the

Company and the strong Peshwa army

(Marathas) under Bajirao II This war

was a part of the

(a) First Anglo-Maratha war

(b) Second Anglo-Maratha war

(c) Third Anglo- Mysore war

(d) Third Anglo-Maratha war

Q57) Consider the following statements

1 Mahadev Desai followed Gandhiji all over the country to persuade him to come to Champaran to investigate the problem associated

with tinkathia system

2 Narhari Parikh accompanied Gandhi ji during the Champaran

Satyagraha

Which of the statements given above isare correct

(a) 1 only

(b) 2 only

(c) Both 1 and 2

(d) Neither 1 nor 2

Q58) Consider the following statements

1 The Nanda Dynasty started the practice of granting tax-free villages to brahmanas and

Buddhist monks

2 The official language of the Satavahanas was Prakrit

Which of the statements given above

isare correct

(a) 1 only

(b) 2 only

(c) Both 1 and 2

(d) Neither 1 nor 2

Q59) Consider the following statements about the objectives of the lsquoadopt a heritagersquo

project

1 It will harness tourism potential for its effects on employment generation and economic

development

2 It will enhance the tourist attractiveness in a sustainable manner by developing world class infrastructure at heritage sites

Which of the statements given above

isare correct

(a) 1 only

(b) 2 only

(c) Both 1 and 2

(d) Neither 1 nor 2

RAUSIAS-FC19E1003 22

Q60) ldquoभारतीय जिजातीय सहकारी निपरि निकास सोघrdquo

(The Tribal Co-operative Marketing

Development Federation of India - TRIFED)

क सोदभण म निमननिखित कथि ो पर निचार कीनजए

1 यह एक राषटर ीय सतर का शीिण सोगठि ह ज

भारत सरकार क गह मोतरािय क परशासनिक

नियोतरर क अरीि काम कर रहा ह

2 इसका मखय उददशय दश म जिजातीय ि ग ो

का सामानजक-आनथणक निकास करिा ह

उपयणकत कथि ो म स कौि-सास सही हह

(a) किि 1

(b) किि 2

(c) 1 और 2 द ि ो

(d) ि त 1 ि ही 2

Q61) निमननिखित म स कौि-सास उपनयास परमचोद क

दवारा नििा गया हनिि गए ह

1 रोगभनम

2 ग दाि

3 ग रा

िीच नदए गए कट का परय ग कर सही उततर चनिए

(a) किि 1

(b) किि 2

(c) किि 1 और 2

(d) 1 2 और 3

Q62) नगदधा ितय क सोदभण म निमननिखित कथि ो पर निचार

कीनजए

1 नगदधा नबहार की मनहिाओो क दवारा तयौहार क

समय और फसि की बिाई तथा कटाई क

अिसर पर नकया जाि िािा एक पारोपररक

दहाती ितय ह

2 इस ितय क दवारा मनहिाऐो अपिी परसननता

परकट करती ह तथा नगदधा क परदशणि क

माधयम स परि िचणसव िाि समाज म

मनहिाओो की दबी हई भाििाओो क परकट

करती ह

उपयणकत कथि ो म स कौि-सास सही हह

(a) किि 1

(b) किि 2

(c) 1 और 2 द ि ो

(d) ि त 1 ि ही 2

Q63) निमननिखित कथि ो पर निचार कीनजए

1 मलला शाह बदखशी दारा नशक ह क

आधयाखतमक गर थ

2 औरोगरब ि मजम-उि-बहरीि या द समदर ो

का सोगम िामक उललििीय रचिा नििी थी

3 दारा नशक ह क अपि पिणज अकबर क गर ो

क उततरानरकारी क रप म दिा गया था

नजसम उसि रानमणक बहििाद और समनवयता

क बढ़ािा नदया था

उपयणकत कथि ो म स कौि-सास सही हह

(a) किि 1 और 3

(b) किि 2

(c) किि 1 और 2

(d) 1 2 और 3

RAUSIAS-FC19E1003 23

Q60) Consider the following statements about

the Tribal Cooperative Marketing

Development Federation of India

(TRIFED)

1 It is a national-level apex

organization functioning under the

administrative control of Ministry

of Home Affairs Government of

India

2 The main objective of TRIFED is

socio-economic development of

tribal people in the country

Which of the statements given above

isare correct

(a) 1 only

(b) 2 only

(c) Both 1 and 2

(d) Neither 1 nor 2

Q61) Which of the following novels isare

written by Premchand

1 Rangabhumi

2 Godan

3 Gora

Select the correct answer using the code

given below

(a) 1 only

(b) 2 only

(c) 1 and 2 only

(d) 1 2 and 3

Q62) Consider the following statements about

Giddha dance

1 Giddha is a traditional pastoral

dance performed by the women of

Bihar at festival times and at the

sowing and reaping of the harvest

2 By this dance the women reveal

their joy expel their suppressed

feelings in a male dominated

society through the performance of

Giddha

Which of the statements given above

isare correct

(a) 1 only

(b) 2 only

(c) Both 1 and 2

(d) Neither 1 nor 2

Q63) Consider the following statements

1 Mullah Shah Badakhshi was the

spiritual mentor of Dara Shukoh

2 Aurangzeb wrote the remarkable

work called ldquoMajma-ul-Bahrainrdquo or

the ldquoThe confluence of two seasrdquo

3 Dara Shukoh was seen as

inheriting the qualities of his

ancestor Akbar in that he

promoted religious pluralism and

syncretism

Which of the statements given above

isare correct

(a) 1 and 3 only

(b) 2 only

(c) 1 and 2 only

(d) 1 2 and 3

RAUSIAS-FC19E1003 24

Q64) निमननिखित कथि ो पर निचार कीनजए

1 ग मतशवर परनतमा निोधयनगरी पहाड़ी पर खसथत ह

2 शरिरबिग िा िह सथाि ह जहाो मौयण िोश क

सोसथापक चोदरगपत मौयण अपि नसोहासि क

तयागि क बाद जि तपसवी बि गए थ

उपयणकत कथि ो म स कौि-सास सही हह

(a) किि 1

(b) किि 2

(c) 1 और 2 द ि ो

(d) ि त 1 ि ही 2

Q65) निमननिखित कथि ो पर निचार कीनजए

1 पराताखतवक साकषय स पता चिता ह नक पराची

घाटी सभयता हड़पपा और म हिज दाड़ द ि ो

की पिणिती ह

2 पराची िदी भििशवर स निकिती ह

उपयणकत कथि ो म स कौि-सास सही हह

(a) किि 1

(b) किि 2

(c) 1 और 2 द ि ो

(d) ि त 1 ि ही 2

Q66) निमननिखित कथि ो म स कौि-सास सही हह

1 िजराह क समारक ो क समह का निमाणर

चोदि राजिोश क शासिकाि क दौराि हआ

था

2 य समारक हररिोदर पिणत शरोििा म खसथत ह

3 म रक क यातरी इबन बतता ि अपि सोसमरर ो

म िजराह क मोनदर ो की यातरा का उललि

नकया था तथा इन काजराण िाम स समब नरत

नकया था

िीच नदए गए कट का परय ग कर सही उततर चनिए

(a) किि 1

(b) किि 1 और 2

(c) किि 2 और 3

(d) किि 1 और 3

Q67) निमननिखित कथि ो म स कौि-सास सही हह

1 डॉ बी आर अमबडकर ि दी एनिनहिशि

ऑफ़ कासट (The Annihilation of Caste)

नििी थी नजसम उन ोि नहोद रमण म िोशािगत

पजारी की परथा क उनमिि की आिशयकता

पर बि नदया था

2 डॉ राजदर परसाद ि थॉटस ऑि पानकसताि

(Thoughts on Pakistan) िामक पसतक

नििी थी

िीच नदए गए कट का परय ग कर सही उततर चनिए

(a) किि 1

(b) किि 2

(c) 1 और 2 द ि ो

(d) ि त 1 ि ही 2

Q68) निमननिखित कथि ो म स कौि-सास सही हह

1 महरगढ़ भारतीय उपमहादवीप म एक परनसदध

ििपािार बसती ह ज नसोर पराोत पानकसताि म

खसथत ह

2 बरणह म म कतत ो क उिक सवामी क साथ कबर ो

म दफिाया जाता था

िीच नदए गए कट का परय ग कर सही उततर चनिए

(a) किि 1

(b) किि 2

(c) 1 और 2 द ि ो

(d) ि त 1 ि ही 2

Q69) निमननिखित कथि ो म स कौि-सास सही हह

1 काकानटय मोनदर अनरकतर नशि क समनपणत

2 हिमक ोडा म हजार-सतोभ िाि मोनदर (The

Thousand-Pillared Temple) का निमाणर

काकानटय समराट रदर ि करिाया था

िीच नदए गए कट का परय ग कर सही उततर चनिए

(a) किि 1

(b) किि 2

(c) 1 और 2 द ि ो

(d) ि त 1 ि ही 2

RAUSIAS-FC19E1003 25

Q64) Consider the following statements

1 Gommateshwara Statue is located

on the Vindyagiri Hill

2 Shravanabelagola is the place

where Chandragupta Maurya the

founder of the Mauryan dynasty

became a Jain ascetic after

relinquishing his throne

Which of the statements given above

isare correct

(a) 1 only

(b) 2 only

(c) Both 1 and 2

(d) Neither 1 nor 2

Q65) Consider the following statements

1 Archaeological evidence shows

that the Prachi Valley Civilisation

predates both Harappa and

Mohenjo-Daro

2 The Prachi river originates from

Bhubaneswar

Which of the statements given above

isare correct

(a) 1 only

(b) 2 only

(c) Both 1 and 2

(d) Neither 1 nor 2

Q66) Which of the following statements

isare correct

1 The Khajuraho group of

monuments was built during the

rule of the Chandela dynasty

2 These monuments are located in

Harischandra mountain range

3 Ibn Battuta the Moroccan

traveller in his memoirs mentioned

visiting Khajuraho temples and

called them Kajarra

Select the correct answer using the code

given below

(a) 1 only

(b) 1 and 2

(c) 2 and 3

(d) 1 and 3

Q67) Which of the following statements

isare correct

1 Dr BR Ambedkar wrote the

Annihilation of Caste emphasising

the need to do away with the

practice of hereditary priesthood in

Hinduism

2 The book lsquoThoughts on Pakistanrsquo

was written by Dr Rajendra

Prasad

Select the correct answer using the code

given below

(a) 1 only

(b) 2 only

(c) Both 1 and 2

(d) Neither 1 nor 2

Q68) Which of the following statements

isare correct

1 Mehrgarh is a famous Neolithic

settlement in the Indian

subcontinent which is situated in

Sindh province Pakistan

2 At Burzahom dogs were buried

with their masters in their graves

Select the correct answer using the code

given below

(a) 1 only

(b) 2 only

(c) Both 1 and 2

(d) Neither 1 nor 2

Q69) Which of the following statements

isare correct

1 The Kakatiya temples are

dedicated mostly to Siva

2 The Thousand-Pillared Temple at

Hanamkonda was built by the

Kakatiya king Rudra

Select the correct answer using the code

given below

(a) 1 only

(b) 2 only

(c) Both 1 and 2

(d) Neither 1 nor 2

RAUSIAS-FC19E1003 26

Q70) निमननिखित कथि ो म स कौि-सास सही हह

1 अहमदाबाद नमि हड़ताि क दौराि महातमा

गाोरी ि शरनमक ो क पकष क मजबत करि क

निए आमरर अिशि नकया था

2 अिशि स नमि मानिक ो पर दबाि पड़ा था ज

अोततः शरनमक ो क िति म 15 परनतशत की िखदध

करि क निए सहमत हए थ

िीच नदए गए कट का परय ग कर सही उततर चनिए

(a) किि 1

(b) किि 2

(c) 1 और 2 द ि ो

(d) ि त 1 ि ही 2

Q71) निमननिखित म स नकसक नकिक भारत स यिसक

की माििता की अमतण साोसकनतक निरासत की

परनतनिनर सची (The UNESCOrsquos List of the

Representative List of the Intangible

Cultural Heritage of Humanity) म शानमि

नकया गया ह

1 मनडयटट

2 सोकीतणि

3 को भ मिा

िीच नदए गए कट का परय ग कर सही उततर चनिए

(a) किि 1 और 2

(b) किि 2 और 3

(c) किि 3

(d) 1 2 और 3

Q72) निमननिखित जिजानतय ो म स कौि-सीसी ो

जिजानतजिजानतया िागािड स सोबोनरत हह

1 अोगामी

2 ककी

3 जारिा

िीच नदए गए कट का परय ग कर सही उततर चनिए

(a) किि 1

(b) किि 1 औऔ 2

(c) किि 2

(d) 1 2 और 3

Q73) निमननिखित कथि ो म स कौि-सास सही हह

1 राषटर कट सामराजय की सथापिा दोनतदगण ि की थी

नजसि मानयाित म अपिी राजरािी की

सथापिा की थी

2 राषटर कट समराट अम घििण एक ििक था और

उस कनिताओो पर पहिी कननड़ पसतक नििि

का शरय नदया जाता ह

िीच नदए गए कट का परय ग कर सही उततर चनिए

(a) किि 1

(b) किि 2

(c) 1 और 2 द ि ो

(d) ि त 1 ि ही 2

Q74) निमननिखित कथि ो म स कौि-सास सही हह

1 कशब चोदर सि ि ततवब नरिी सभा की

अधयकषता की थी ज आधयाखतमक सतय की

ि ज म सोिि थी

2 बरहम समाज ि मािि गररमा पर बि नदया

मनतणपजा का निर र नकया और सती परथा जसी

सामानजक बराइय ो की आि चिा की

िीच नदए गए कट का परय ग कर सही उततर चनिए

(a) किि 1

(b) किि 2

(c) 1 और 2 द ि ो

(d) ि त 1 ि ही 2

Q75) निमननिखित कथि ो म स कौि-सास सही हह

1 भारत म नचशती नसिनसिा खवाजा म इिददीि

नचशती क दवारा सथानपत नकया गया था

2 नचशती परोपरा की एक परमि निशिता

आतमसोयम थी नजसम साोसाररक म ह स दरी

बिाए रििा शानमि था

िीच नदए गए कट का परय ग कर सही उततर चनिए

(a) किि 1

(b) किि 2

(c) 1 और 2 द ि ो

(d) ि त 1 ि ही 2

RAUSIAS-FC19E1003 27

Q70) Which of the following statements

isare correct

1 During the Ahmedabad Mill Strike

Mahatma Gandhi undertook a fast

unto death to strengthen the

workersrsquo resolve

2 The fast had effect of putting

pressure on mill owners who

finally agreed to give the workers a

15 per cent increase in wages

Select the correct answer using the code

given below

(a) 1 only

(b) 2 only

(c) Both 1 and 2

(d) Neither 1 nor 2

Q71) Which of the following are included in

the UNESCOrsquos list of the representative

list of the intangible cultural heritage of

humanity from India

1 Mudiyettu

2 Sankirtana

3 Kumbh Mela

Select the correct answer using the code

given below

(a) 1 and 2 only

(b) 2 and 3 only

(c) 3 only

(d) 1 2 and 3

Q72) Which of the following tribes isare

related to Nagaland

1 Angami

2 Kuki

3 Jarawa

Select the correct answer using the code

given below

(a) 1 only

(b) 1 and 2 only

(c) 2 only

(d) 1 2 and 3

Q73) Which of the following statements

isare correct

1 Rashtrakuta kingdom was founded by Dantidurga who established his capital at Manyakhet

2 Amoghavarsha a Rashtrakuta king was an author and is credited with writing the first

Kannada book on poetics

Select the correct answer using the code given below

(a) 1 only

(b) 2 only

(c) Both 1 and 2

(d) Neither 1 nor 2

Q74) Which of the following statements isare correct

1 Keshab Chandra Sen headed the Tattvabodhini Sabha which was engaged in search of spiritual truth

2 The Brahmo Samaj laid emphasis on human dignity opposed idolatry and criticized such social

evils as the practice of Sati

Select the correct answer using the code given below

(a) 1 only

(b) 2 only

(c) Both 1 and 2

(d) Neither 1 nor 2

Q75) Which of the following statements isare correct

1 The Chishti order was established in India by Khwaja Moinuddin

Chishti

2 A major feature of the Chishti tradition was austerity including maintaining a distance from the

worldly power

Select the correct answer using the code

given below

(a) 1 only

(b) 2 only

(c) Both 1 and 2

(d) Neither 1 nor 2

T e s t i s p a r t o f R a u rsquo s I A S T e s t s e r i e s f o r P r e l i m i n a r y E x a m 2 0 1 9

FOUNDATION + CURRENT AFFAIRS

GENERAL STUDIES (PAPER ndashI)

FOUNDATION TEST ndashIII

SUBJECT NCERT History Class VI-X + Current Affairs

Time Allowed 1frac12 Hours Maximum Marks 150

I NSTRUCT IONS

1 IMMEDIATELY AFTER THE COMMENCEMENT OF THE EXAMINATION YOU SHOULD CHECK

THAT THIS TEST BOOKLET DOES NOT HAVE ANY UNPRINTED OR TORN or MISSING PAGES OR

ITEMS ETC IF SO GET IT REPLACED BY A COMPLETE TEST BOOKLET

2 This Test Booklet contains 75 items (questions) Each item is printed both in Hindi and English

Each item comprises four responses (answers) You will select the response which you want to mark

on the Answer Sheet In case you feel that there is more than one correct response mark the

response which you consider the best In any case choose ONLY ONE response for each item

3 You have to mark all your responses ONLY on the separate Answer Sheet (OMR sheet) provided

Read the directions in the Answer Sheet

4 All items carry equal marks

5 Before you proceed to mark in the Answer Sheet the response to various items in the Test booklet

you have to fill in some particulars in the Answer Sheet as per instructions contained therein

6 After you have completed filling in all your responses on the Answer Sheet and the examination has

concluded you should hand over to the Invigilator only the Answer Sheet You are permitted to

take away with you the Test Booklet

7 Penalty for wrong answers

THERE WILL BE PENALTY FOR WRONG ANSWERS MARKED BY A CANDIDATE IN THE

OBJECTIVE TYPE QUESTION PAPERS

(i) There are four alternatives for the answer to every question For each question for which a

wrong answer has been given by the candidate one-third of the marks assigned to that

question will be deducted as penalty

(ii) If a candidate gives more than one answer it will be treated as a wrong answer even if one of

the given answers happens to be correct and there will be same penalty as above to that

question

(iii) If a question is left blank ie no answer is given by the candidate there will be no penalty for

that question

T h i s t e s t i s p a r t o f R a u rsquo s I A S T e s t s e r i e s f o r P r e l i m i n a r y E x a m 2 0 1 9

Test Code

FC19E1003

FC19H1003 29

Answers and Explanations of

NCERT History Class VI-X + Current Affairs (FC19E1003)

Q1) उततर (c)

सपषटीकरण

- ऋगवद म दविय ो और दिताओो क समवपित एक

हजार स अविक सत तर (शल क) ह

- य शल क ऋविय ो क दवारा रच गए थ और परि ो

दवारा सीख जात थ

- हालाोवक कछ शल क मवहलाओो (जस वक अपाला

घ सा ल पामदरा मतरयी और गागी) क दवारा भी रच

गए थ

- ऋगवद म सोिाद क रप म कई शल क मौजद ह

- हम विशवावमतर नामक एक ऋवि और दविय ो क

रप म पजी जान िाली द नवदय ो (वयास और

सतलज) क बीच िाताि का उदाहरण वमलता ह

- इसस पता चलता ह वक विशवावमतर िवदक काल स

सोबोवित थ

Q2) उततर (b)

सपषटीकरण

- करनल गफाओो स राख क अिशि परापत हए ह

ज इस ओर सोकत करत ह वक ततकालीन ल ग

अवि क उपय ग स पररवचत थ

- य गफाएो आोधर परदश म सथथत ह

Q3) उततर (c)

सपषटीकरण

bull बरािह म ितिमान कशमीर म सथथत एक

परागवतहावसक थथल ह जहाो ल ग गडढ क घर ो का

वनमािण करत थ

bull य घर जमीन क ख द कर बनाए जात थ तथा नीच

जान क वलए सीवियाा ह ती थी

bull ऐसा अनमान लगाया जाता ह वक य घर ठो ड क

मौसम म आशरय परदान करत थ

Q4) उततर (c)

सपषटीकरण

bull परालख-विदया (Epigraphy) क वशलालख ो क

अधययन क रप म पररभावित वकया जाता ह

bull हसतवलसखत दसतािज ो क माधयम स इवतहास

और सावहतय क अधययन क पाोडवलवप विजञान

(Manuscriptology) कहत ह

bull पराचीन लखन परणावलय ो क अधययन और

ऐवतहावसक पाोडवलवपय ो क समझन तथा वतवथ

वनिािरण क पलीओगराफी (Palaeography) कहा

जाता ह

bull नयवमजमविकस (Numismatics) वसक ो क

अधययन क सोदवभित करता ह

Q5) उततर (a)

सपषटीकरण

- चरक सोवहता चरक क दवारा वलखी गई आयिद

और िदयक-शासर पर एक महतवपणि पसतक ह

- ि भारतीय िदयक-शासर की पारमपररक परणाली

वजस आयिद क नाम स जाना जाता ह क

अभयासकताि थ

- ऐसा माना जाता ह वक चरक का विकास दसरी

शताबदी (ईसा पिि) और दसरी शताबदी (ईसवी) क

मधय हआ था

Q6) उततर (b)

सपषटीकरण

- भाग फसल ो पर वलए जान िाल कर क सोदवभित

करता ह ज कल फसल उतपादन का 16 िाो भाग

था

- ldquoकममकारrdquo शबद भवमहीन कवि शरवमक िगि क

वलए परय ग वकया जाता था

- ldquoअशवमिrdquo (वजस घ ड क बवलदान क रप म भी

जाना जाता ह) एक अनषठान ह ता था वजसम एक

घ ड क सवतोतर रप स घमन क वलए छ ड वदया

FC19H1003 30

जाता ह और राजा क सवनक उसकी रखिाली

करत थ

Q7) उततर (d)

सपषटीकरण

- ऋगववदक काल म घ ड ो क रथ ो म ज ता जाता था

ज (रथ) भवम मिवशय ो आवद पर कबजा करन क

वलए लड गए यद ो म उपय ग वकए जात थ

- इसस यह पता चलता ह वक घ ड ो यकत रथ ो का

उपय ग महाजनपद काल स काफी पहल आरमभ

हआ था

- ऋगववदक काल म मिवशय ो भवम जल आवद पर

कबजा करन क वलए तथा ल ग ो क पकडन क

वलए यद वकय जात थ

- अविकाोश परि इन यद ो म भाग वलया करत थ

- हालाोवक उस समय क ई वनयवमत सना नही ो ह ती

थी लवकन उस काल म सभाऐो ह ती थी ो वजनम

ल ग यद क मामल ो पर चचाि करत थ

- वनयवमत सनाएा महाजनपद काल का िवशषटय थी

वजनम पदल सवनक ो की विशाल सनाएा रथ तथा

हाथी शावमल ह त थ

Q8) उततर (a)

सपषटीकरण

- बद शाकय कल स सोबोवित थ और कशीनारा म

उनका वनिन हआ था

- बद न अपनी वशकषाएा पराकत भािा म दी थी ो ज

आम ल ग ो की भािा थी

Q9) उततर (c)

सपषटीकरण

- पराचीन भारत म दशिनशासर की छह शाखाएा थी ो

िशविक नयाय समखया य ग पिि वममाोसा और

िदाोत या उततर वममाोसा

- इनकी थथापना करमश कनाद गौतम कवपल

पतोजवल जावमनी और वयास ऋविय ो न की थी

Q10) उततर (b)

सपषटीकरण

महािीर की वशकषाऐो छठी शताबदी म िललभी म

सोकवलत की गई थी ो

Q11) उततर (c)

सपषटीकरण

- पारमपररक रप स चाणकय क कौविलय अथिा

विषणगपत क नाम स जाना जाता ह

- उसन अथिशासतर ज एक पराचीन भारतीय

राजनवतक आलख ह वलखा था

Q12) उततर (d)

सपषटीकरण

- भारत का राषटर ीय वचनह सारनाथ (उततर परदश) क

अश क सतमभ क ऊपर (शीिि पर) वसोह कवपिल

का एक अनरपण ह

- इस राषटर ीय वसदाोत सतयमि जयत क साथ

सोय वजत वकया गया ह

- रामपिि बल का नाम रामपिि (वबहार) क नाम पर

पडा जहाा इसकी ख ज हई थी

- यह अपन नाजक नकाशी मॉडल क वलए परवसदद

ह वजसम क मल तवचा सोिदनशील नथन ो सतकि

कान और मरबत िााग ो क शरषठतर परवतरप क

परदवशित वकया गया ह

- यह भारतीय और फारसी ततव ो का एक ससममशरण

- सोवकससा उततर परदश म सथथत ह

Q13) उततर (a)

सपषटीकरण

का िर वसोह ज एक महान य दा थ वबहार स

सोबोवित थ

Q14) उततर (b)

सपषटीकरण

िललालर शबद बड भ-सवावमय ो क वलए परय ग

वकया जाता था

FC19H1003 31

Q15) उततर (c)

सपषटीकरण

- अररकमड एक तिीय बसती थी जहाो दर दश ो स

आन िाल जहाज ो का माल उतारा जाता था

- यहाो पर ईोि ो का एक विशाल ग दाम वमटटी क

बतिन (वजनम एमफ रा - द हरी मवठय ो का लोबा

घडा - शावमल ह) और एरिाइन (Arretine)

मदभाोड पाए गए थ

- इस थथान पर र मन दीपक काोच क बन पातर और

रतन भी पाए गए थ

Q16) उततर (a)

सपषटीकरण

- मिनदर सोगम कविताओो म उसललसखत एक

तवमल शबद ह वजसका अथि ह ldquoतीन परमखrdquo

- यह तीन सततारि पररिार ो क मसखयाओो क वलए

परय ग वकया जाता ह च ल चर और पाणडय

Q17) उततर (c)

सपषटीकरण

- ऋग िद म सभा विदाथा तथा गण जसी

जनजावतय ो पर अथिा किोब पर आिाररत

सभाओो का उललख ह

- आरसमभक िवदक काल म सभाओो और सवमवतय ो

का विशि महतव ह ता था

- यहाा तक की मसखया अथिा राजा भी उनका

समथिन परापत करन क वलए आतर रहत थ

Q18) उततर (a)

सपषटीकरण

- जन िमि न ईशवर क अससततव क मानयता त दी ह

वकनत उसन ईशवर क वजना क पद स नीच रखा

- जन िमि न बौद िमि की तरह िणि परणाली की

भरतिना नही ो की थी

Q19) उततर (d)

सपषटीकरण

- च ल ो और पाणडय ो न शसकतशाली तिीय शहर ो का

विकास वकया था

- च ल ो का सबस महतवपणि शहर पहार (या

कािरीपटटीनम) था |

- मदरई पाणडय ो की राजिानी थी

Q20) उततर (b)

सपषटीकरण

- ldquoबदचररतrdquo बद का जीिन-ितताोत ह

- इस अशवघ ि क दवारा वलखा गया था

Q21) उततर (a)

सपषटीकरणः

- तवमल कवि अपपर भगिान वशि क भकत थ

- इस परकार ि एक नयनार सोत थ

Q22) उततर (d)

सपषटीकरणः

- समदरगपत एक परवसद गपत शासक था

- उसन वसक ो पर िीणा बजात हए अपनी छवि

अोवकत करिाई थी

- यह सोगीत क परवत उसक परम क दशािता ह

- हम उसकी इलाहाबाद परशससत स महतवपणि

ऐवतहावसक जानकारी वमलती ह वजसकी रचना

उसक दरबार क कवि हररसन न की थी

Q23) उततर (b)

सपषटीकरणः

- विकरम सोित की शरआत ििि 58 ईसा पिि म

चनदरगपत वदवतीय न की थी

- यह शक ो पर उसकी जीत और उस विकरमावदतय

की पदिी वमलन क उपलकषय म आरमभ वकया गया

था

FC19H1003 32

- बानभटट न हिििििन का जीिन-ितताोत हििचररत

(ज सोसकत म थी) वलखी थी

Q24) उततर (c)

सपषटीकरणः

- सोवि-विगरावहका यद एिो शाोवत का मोतरी

- साथििाह वयापाररय ो क कावफल ो का नता

Q25) उततर (a)

सपषटीकरणः

- जआन झाोग (हसआन रताोग ndash Hsuang Tsang)

एक चीनी यातरी था ज हिििििन क शासनकाल म

भारत आया था

- ििि 630 ईसवी स ज दशक आरमभ हआ था उसम

जआन झाोग मधय एवशया ईरान और

अफग़ावनसतान की यातरा करन क पशचात कशमीर

क रासत स भारत आया था

- उसन उततर स पिि तक की यातरा की और िह

लगभग 2 ििि वबहार म रहा

- जआन झाोग न नालनदा विशवविदयालय म विदयावथिय ो

और विदवान ो क साथ पारसपररक विचार-विमशि

वकया थथानीय भािाओ ा म वनपणता परापत की तथा

बौद सतप ो की ख ज की

Q26) उततर (c)

सपषटीकरणः

- परदवकषणा पथ बौद िासतकला म सतप क चार ो

ओर बनाया जान िाला एक घमािदार पथ ह ता

- परशन म वदए गए बाकी क तीन ो ततव वहोद मसनदर ो की

िासतकला क भाग ह

Q27) उततर (d)

सपषटीकरणः

परशन म वदए गए सभी मोवदर ो म वयापक रप स

ईोि ो (पकी ईोि ो) का परय ग पतथर ो क साथ हआ

Q28) उततर (c)

सपषटीकरण

- महममद कली कतब शाह ग लकणडा का सलतान

था

- िह अकबर का समकालीन था

- सावहतय और िासतकला म उसकी अतयाविक

रवच थी

- िह एक महान कवि था

- िह दसखनी उदि फारसी और तलग म वलखता था

- उसन अपन पीछ एक विसतत वदिान (सोगरह)

छ डा ह

- अभी हाल ही म तलोगाना म ग लकणडा क वकल

क अनदर खदाई वकय गए बाग-ए-नाया वकला

बाग क चार ो ओर रप-रखा क मानवचतरण क

वलए भारतीय परातासतवक सिकषण (The

Archaeological Survey of India ndash ASI)

गराउणड पनीिर विोग रडार (Ground Penetrating

Radar) का परय ग करगा

Q29) उततर (a)

सपषटीकरणः

- वसलपपावदकारम एक तवमल महाकावय ह वजसकी

रचना इलाोग क दवारा लगभग 1800 ििि पिि की

गई थी

- यह क िलन नामक एक वयापारी की कहानी ह

ज माििी नामक एक गवणका (िशया) स परम

करन लगा था

- मवनमकलाई क िलन और माििी की पतरी की

कहानी ह

Q30) उततर (a)

सपषटीकरण

- चरक आयिद और वचवकरता की एक महतवपणि

रचना चरक सोवहता क लखक ह

- बरहमगपत क अपनी रचना बरहम-सफि-वसदानत

(ज एक खग लीय रचना ह) क कारण परवससद

वमली

FC19H1003 33

- बगदाद म इसका अनिाद अरबी भािा म वकया

गया था

- इसका इसलावमक गवणत और खग ल-विजञान पर

महतवपणि परभाि पडा था

- बाद म अपन जीिनकाल म बरहमगपत न

ldquoखोडखयाकrdquo वलखी ज एक खग लीय पससतका

(एक छ िी पसतक) थी

- इसम आयिभटट की अिि-रावतर क परतयक वदन की

शरआत परणाली का परय ग वकया गया था

Q31) उततर (c)

सपषटीकरण

- अमीर खसर एक परवसद सफी सोगीतकार कवि

और विदवान थ

- 1318 म उनह ोन पाया वक इस भवम (वहोदसतान) क

हर कषतर म अलग-अलग भािा थी लाहौरी

कशमीरी दवारसमदरी (दवकषणी कनाििक म)

तलोगाना (आोधर परदश म) गजरी (गजरात म)

माबारी (तवमलनाड म ) अििी (पिी उततर परदश

म) और वहोदिी (वदलली क आस-पास क कषतर म)

आवद

- उनह न यह बताया वक सोसकत वकसी भी कषतर स

सोबोवित नही ो थी और किल बराहमण ही इस भािा

का जञान रखत थ

Q32) उततर (c)

सपषटीकरण

- वहरणय-गभि सववणिम गभि क सोदवभित करता ह

- जब बराहमण ो की सहायता स यह अनषठान वकया

जाता था त यह माना जाता था वक बवल दन िाल

का कषवतरय क रप म पनजिनम ह गा

Q33) उततर (d)

सपषटीकरण

- कदमई भवम राजसव पर कर क सोदवभित करता

- गवावलयर परशससत म नागभि क दवारा वकय गए

श िण का िणिन वकया गया ह |

- नागभि एक परवतहार राजा था

Q34) उततर (b)

सपषटीकरण

- राजतरो वगनी 12िी ो शताबदी म कलहन क दवारा

रवचत एक सोसकत पसतक (िकसट) ह

- यह परारसमभक भारत की ऐवतहावसक इवतितत थी

- तकि सोगत रप स इस अपन परकार की सिोततम

और सिािविक विशवसनीय कवत माना जाता ह

- यह कशमीर कषतर क पराचीनतम समय स लकर

उसकी रचना की तारीख तक क समपणि इवतहास

का आचछादन करती ह

Q35) उततर (c)

सपषटीकरण

- गााि की आम सभा क ldquoउरrdquo कहा जाता था

- ldquoउरrdquo म गााि क सभी कर दन िाल वनिासी

शावमल ह त थ

Q36) उततर (a)

सपषटीकरण

- वदलली सलतनत म ldquoतारीखrdquo इवतहास लखन का

एक रप था

- ldquoतािरीखrdquo क लखक विदवान परि ह त थ वजनम

सवचि परशासक इतयावद शावमल थ

Q37) उततर (a)

सपषटीकरण

- अलाउददीन सखलजी अपन सवनक ो क ितन का

भगतान नकद म करता था न वक इकता क रप

- सवनक अपना सामान वदलली म वयापाररय ो स

खरीदत थ अतः इस बात का भय था वक वयापारी

कही ो िसतओो का मलय न बिा द

- इसकी र कथाम क वलए अलाउददीन सखलजी न

वदलली म कीमत ो क वनयसित वकया

FC19H1003 34

- अविकारीगण धयानपििक मलय ो का सिकषण करत

थ तथा ज वयापारी वनिािररत मलय पर माल नही ो

बचत थ उनक दसणडत वकया जाता था

Q38) उततर (d)

सपषटीकरण

- वदलली सििपरथम त मर राजपत ो क अिीन उनक

सामराजय की राजिानी बनी थी

- 12िी ो शताबदी क मधय म अजमर क चौहान ो

(वजनह चाहमान ो क नाम स भी जाना जाता ह) न

त मर राजपत ो क परावजत वकया था

- त मर ो और चौहान ो क अिीन वदलली एक

महतवपणि िावणसजयक क दर बन गया था

- कई जन वयापारी यहाा रहन लग थ और उनह ोन

कई मोवदर भी बनिाए

- यहाा पर मवदरत वसक वजनह ldquoदहलीिालrdquo क नाम

स जाना जाता था वयापक रप स परचलन म थ

Q39) उततर (c)

सपषटीकरण

- म ठ की मसिद का वनमािण वसको दर ल दी क

राजयकाल म उसक मिी क दवारा करिाया गया

था

- बगमपरी मसिद का वनमािण महममद तगलक क

शासनकाल म हआ था

- यह मसिद विशव का पणयथथान (The

Sanctuary of the World) और वदलली म महममद

तगलक की नई राजिानी जहाोपनाह की मखय

मसिद थी

- कववत- अल - इसलाम मसिद का विसतार

इलतसिश और अलाउददीन सखलजी न वकया था

- मीनार का वनमािण तीन सलतान ो कतबददीन ऐबक

इलतसिश और वफर ज शाह तगलक क दवारा

करिाया गया था

Q40) उततर (c)

सपषटीकरण

- मगल ो क अिीन मनसबदार शबद उस वयसकत क

वलए सोदवभित वकया जाता था वजसक पास मनसब

(अथाित पद) ह ता था

- उस अपना ितन राजसव कायो वजनह जागीर कहत

थ क रप म परापत ह ता था

Q41) उततर (b)

सपषटीकरण

- ldquoभारत छ ड आोद लनrdquo वबरविश शासन क

सखलाफ ल ग ो का एक सवाभाविक विदर ह था

- असखल भारतीय काोगरस सवमवत न 8 अगसत 1942

क बमबई म एक बठक का आय जन वकया था

- इस बठक म परवसद सोकलप ldquoभारत छ ड rdquo क

पाररत वकया गया और इस उददशय क परापत करन

क वलए गाोिी क नततव म एक अवहोसक जन सोघिि

आोद लन की शरआत का परसताि वदया गया

- लवकन अगल ही वदन गाोिी और काोगरस क अनय

परमख नताओो क वगरफतार कर वलया गया

- काोगरस क एक बार वफर अिि घ वित वकया गया

था

Q42) उततर (c)

सपषटीकरण

- साइमन कमीशन यनाइविड वको गडम क सात

साोसद ो का एक समह था

- इस वबरविश भारत क वलए सोििावनक सिार ो का

सझाि दन क वलए गवठत वकया गया था

- इस आय ग म िररषठ वबरविश राजनता सर जॉन

साइमन क नततव म किल वबरविश सदसय ही

शावमल थ

- इसवलए भारत क ल ग ो न साइमन कमीशन क

आगमन क विरद आोद लन वकया था

Q43) उततर (a)

सपषटीकरण

bull दादा भाई नौर जी भारत म वबरविश शासन क

आवथिक पररणाम ो क बार म अपनी विर िी

(परवतकल) राय क वलए जान जात थ

FC19H1003 35

bull अपन कई लख ो और भािण ो म विशि रप स

ldquoपाििी एो ड अन-वबरविश रल इन इसणडया

(Poverty and Un-British Rule in India) म

नौर जी न यह तकि वदया वक भारत पर अतयविक

कर लगाया गया था और इसकी सोपवतत इोगलड की

ओर परिावहत की जा रही थी

bull उनह ोन पराचीन भारतीय गरोथ ो की वयाखया करन

का और भारतीय ो क आिविशवास क बहाल

करन पर कायि नही ो वकया था

उनह ोन वकसी और बात स पहल सभी सामावजक

बराइय ो क उनमलन की आिशयकता पर भी बल

नही ो वदया था

Q44) उततर (c)

सपषटीकरण

bull अगसत 1932 म वबरविश परिानमोतरी मकड नालड न

अपन साोपरदावयक परसकार (The Communal

Award) की घ िणा की थी

bull यह भारत क कई साोपरदावयक वहत ो क बीच विवभनन

सोघिो क हल करन क वलए वबरिन का एकतरफा

परयास था

bull यह परसकार (Award) बाद म 1935 क

अविवनयम (The Act of 1935) म शावमल वकया

गया था

bull इस साोपरदावयक परसकार न मससलम ो क वलए

आरवकषत एक अलग वनिािचक मणडल फॉमिल का

विसतार अनय अलपसोखयक ो क वलए वकया था

वजसम वसख ो भारतीय ईसाइय ो आोगल-भारतीय

समदाय यर पीय समदाय तथा विवशषट कषतरीय

समह ो क शावमल वकया गया था

bull गाोिी न इस परसताि क भारतीय समाज क

विभावजत करन क वलए एक घवणत वबरविश

सावजश क रप म दखा और उसक सखलाफ

आमरण अनशन वकया

Q45) उततर (b)

सपषटीकरण

मौजदा आयात और वनयाित क अवतररक़त

औपवनिवशक भारत क वनमनवलसखत खचो क

वलए एक विशिवनवशचत िन रावश भी दनी पडती

थी

(i) परशासन क वयय

(ii) सना क रख-रखाि क वयय

(iii) यद क वयय

(iv) सिावनितत अविकाररय ो की पशन तथा

(v) वबरिन दवारा अपनी उपवनिश बसती

(कॉल नी) क रख-रखाि क वयय

इनह गह शलक (Home Charges) क रप म

जाना जाता था और लगभग परी तरह स भारत क

दवारा इनका भगतान वकया जाता था

bull गह शलक म वनमनवलसखत घिक शावमल थ

(i) भारतीय ऋण पर दय बयाज

(ii) ईसट इोवडया को पनी क शयरिारक ो क

लाभाोश

(iii) लोदन म भारत कायािलय चलान क वलए िन

(iv) भारत म वनयकत वबरविश कवमिय ो क ितन

और पशन का भगतान करन क वलए िन

(v) रलि पर बयाज

(vi) नागररक और सनय शलक

(vii) इोगलड म सट र (सामगरी) की खरीद

Q46) उततर (b)

सपषटीकरण

bull भारतीय राषटर ीय काोगरस का लाहौर सतर 1929 म

जिाहरलाल नहर की अधयकषता म आय वजत

वकया गया था

bull इस सतर म भारतीय राषटर ीय आोद लन स समबसित

कई महतवपणि पररणाम सामन आय थ

(i) सििपरथम इस सतर म काोगरस क अधयकष पद

पर जिाहरलाल नहर क चना गया था ज

काोगरस म िामपोवथय ो की बिती हई ताकत

का सपषट सोकत था

(ii) दसरा इस सतर म पहली बार काोगरस न पणि

सवतोतरता की माोग क उठाया था

इस परकार की माोग काोगरस मोच स पहल कभी भी

नही ो उठाई गई थी

Q47) उततर (b)

सपषटीकरण

FC19H1003 36

bull इस ररप िि न वकसी भी समदाय क वलए पथक

वनिािचक मोडल अथिा अलपसोखयक ो क वलए

भाराोश की वसफाररश नही ो की थी

bull तथावप इस ररप िि न उन पराोत ो म अलपसोखयक

सीि ो क आरकषण की अनमवत दी थी जहाा पर कम

स कम दस परवतशत अलपसोखयक ह

bull लवकन यह समदाय क आकार क अनपात म ह ना

चावहए था

bull इस ररप िि म भारत क वलए पणि सवतोतरता क

वलए क ई पराििान नही ो था

Q48) उततर (c)

सपषटीकरण

bull आरो वभक िवदक आयो का िमि मखय रप स

परकवत की पजा और यजञ था

bull परारो वभक आयि िमि परकवत की पजा क समान था

bull िासति म उनक चार ो ओर की शसकतयाा वजनह न

त ि वनयोवतरत कर सकत थ और न ही समझ पाए

थ उनह वदवयता क साथ वनिवशत वकया गया तथा

उनह मादा या नर दिीदिताओो क रप म

परतीकतव वकया गया था

bull उनह ोन कछ यजञ ो का भी वनषपादन वकया था

Q49) उततर (b)

सपषटीकरण

bull सडक और नदी-मागि (जल-मागि) डकती स

सरवकषत नही ो थ

bull उललखनीय ह वक हिििििन क शासनकाल क

दौरान यआन चिाोग (हयएन साोग) का सारा

सामान लि वलया गया था

Q50) उततर (c)

सपषटीकरण

परशन म वदए गए द न ो कथन सही ह

Q51) उततर (b)

सपषटीकरण

bull परोदर दास एक सोत और भगिान कषण क एक

महान भकत थ

bull परोदर दास क कनाििक सोगीत क वपतामह क

रप म जाना जाता ह

bull यदयवप उनक जनम-थथान क बार म काफी

अिकल लगाई जाती रही ह

bull तथावप अब कननड विशवविदयालय हमपी क दवारा

गवठत एक विशिजञ सवमवत इस वनषकिि पर पहोची

ह वक उनका जनम थथान सोभितया कनाििक का

एक छ िा-सा गााि कषमपरा (वशिम गगा वजला)

था

Q52) उततर (c)

सपषटीकरण

bull शरी तयागराज शरी शयाम शासतरी और शरी मथसवामी

दीवकषतर क कनाििक सोगीत की वतरमवति माना

जाता ह

bull उनक कारण ही 18िी ो-19िी ो शताबदी म कनाििक

सोगीत का सववणिम यग आया था

Q53) उततर (d)

सपषटीकरण

bull अभी हाल ही म लौह यगीन-महापािावणक काल

का 2000 ििि पराना एक दलिभ सारक फगस

(Sarcophagus) (पतथर का ताबत) क ललम क

वियर गाोि (क वयलडी क पास वजला क वझक ड

करल राजय) की एक रॉक-कि गफा स ख जा गया

bull यह ताबत वजसम हविय ो क िकड थ खदाई क

दौरान वमला

bull अभी तक इस परकार की दलिभ ख ज करल क

मातर द ही थथान ो स हई ह

bull य द न ो सारक फगी (Sarcophagi) (पतथर क

ताबत) चियर और अथ ली (वजला क वझक ड) क

महापािाण थथल ो स वमल ह

Q54) उततर (a)

सपषटीकरण

FC19H1003 37

दवकषण भारत म महापािाण सोसकवत एक पणि

विकवसत लौह यगीन सोसकवत थी

Q55) उततर (d)

सपषटीकरण

bull च ल पाणडय और करलपतर (चर) इन तीन ो का

उललख अश क क अवभलख ो म वकया गया ह

bull सोभितः य भौवतक सोसकवत क उततर

महापािावणक चरण म थ

Q56) उततर (d)

सपषटीकरण

bull भीमा-क रगाोि की लडाई ततीय आोगल-मराठा

यद का वहससा थी

Q57) उततर (b)

सपषटीकरण

bull राजकमार शकल न गाोिीजी क चोपारण आन तथा

वतनकवथया परणाली स जडी समसया की जाोच क

वलए रारी करन क वलए दश भर म उनका

अनसरण वकया था

bull बज वकश र राजदर परसाद महादि दसाई और

नरहरी पाररख चोपारण सतयागरह क दौरान गाोिी

जी क सहय गी थ

Q58) उततर (b)

सपषटीकरण

bull बराहमण ो और बौद मठिाररय ो क कर-मकत गााि

अनदान म दन की परथा सतिाहन ो न आरमभ की

थी

Q59) उततर (c)

सपषटीकरण

इस कायिकरम क उददशय वनमनानसार ह

(i) बवनयादी पयििन आिाररक सोरचना का विकास

करना

(ii) चयवनत (पहचान वकय गए) कषतर ो म आजीविका क

सजन क वलए दश क साोसकवतक और विरासत

मलय ो क बिािा दना

(iii) विरासत समारक थथल ो पर विशव सतरीय आिाररक

सोरचना विकवसत करक एक सतत तरीक स

पयििक आकििण म िसद करना

(iv) थथानीय समदाय ो की सवकरय भागीदारी क माधयम

स र रगार ो का सजन करना

(v) र रगार उतपादन और आवथिक विकास क वलए

पयििन कषमता का उन पर परभाि का उपय ग

करना तथा

(vi) िारणीय पयििन आिाररक सोरचना का विकास

करना और उसका उवचत सोचालन तथा

रखरखाि सवनवशचत करना

Q60) उततर (b)

सपषटीकरण

bull यह वनकाय ििि 1987 म अससततव म आया था

bull यह एक राषटर ीय सतर का शीिि सोगठन ह ज भारत

सरकार क जनजातीय मामल ो क मोतरालय क

परशासवनक वनयोतरण क अिीन काम कर रहा ह

bull इसका पोजीकत और परिान कायािलय नई वदलली

म सथथत ह

Q61) उततर (c)

सपषटीकरण

bull परमचोद क उपनयास ो म परमाशरम रोगभवम गबन

कमिभवम और ग दान शावमल ह

bull ग रा रिी ोदरनाथ िग र क दवारा रवचत उपनयास ह

bull अभी हाल ही म मोशी परमचोद की 138िी ो जयोती दश

भर म मनाई गई थी

Q62) उततर (b)

सपषटीकरण

bull ldquoवगदाrdquo पोजाब (भारत) एिो पावकसतान की

मवहलाओो क दवारा तयौहार क समय और फसल

की बिाई तथा किाई क अिसर पर वकया जान

िाला एक पारोपररक दहाती नतय ह

FC19H1003 38

bull इस नतय क माधयम स पोजाबी मवहलाऐो अपनी

परसननता परकि करती ह तथा वगदा क परदशिन क

माधयम स परि िचिसव िाल समाज म मवहलाओो

की दबी हई भािनाओो क परकि करती ह

bull चोवक इस नतय का परि ो क साथ क ई सोबोि नही ो

ह अतः किल मवहलाऐो ही इसम भाग ल सकती

bull हर साल तीज समार ह क दौरान पोजाब म वगदा

नतय वकया जाता ह

तीज भारत क कछ भाग ो म मवहलाओो क दवारा

मनाया जान िाल कई तयौहार ो क वलए एक

वयापक नाम ह

Q63) उततर (a)

सपषटीकरण

- मजम-उल-बहरीन या द समदर ो का सोगम

नामक उललखनीय रचना दारा वशक ह क दवारा

वलखी थी

- भारत क उपराषटर पवत शरी एम िकया नायड न कहा

ह वक राजकमार दारा वशक ह की रचनाएा शाोवत

और सदभाि क बिािा दन क वलए एक तारा सर त

क रप म सामन आ सकती ो ह

- उपराषटर पवत गत ििो क भला वदए गए राजकमार

दारा वशक ह क परदवशित परचवलत करन हत

आय वजत एक परदशिनी का दौरा करन क बाद एक

सभा क सोब वित कर रह थ

- इस परदशिनी का आय जन फर क इस गौवियर

(Francois Gautier) क दवारा lsquoइोवदरा गाोिी नशनल

सिर फॉर द आििसrsquo (The Indira Gandhi

National Centre for the Arts) नई वदलली म

वकया गया था

Q64) उततर (c)

सपषटीकरण

- ग मतशवर परवतमा जन भगिान बाहबली क

समवपित ह

- यह एक एक-चटटानी पतथर की मवति ह

- राषटर पवत राम नाथ क विोद न शरिणबलग ला

(कनाििक) म आय वजत वकय जान िाल भवय

अवभिक समार ह महामसतकावभिक का

उदघािन वकया था

- यह समार ह 12 ििो म एक बार ह ता ह

Q65) उततर (c)

सपषटीकरण

bull पराची घािी पराची नदी क चार ो ओर फली हई थी

bull पराची घािी िीर-िीर विलपत ह गई थी

bull पराची नदी भिनशवर स वनकलती ह

bull यह महानदी की एक सहायक नदी ह और यह

परी खदाि किक तथा जगतवसोहपर वजल ो स

ह कर बहती ह

bull इस नदी क पर कषतर क पराची घािी कहा जाता ह

bull यह नदी बोगाल की खाडी म वगरती ह

परातासतवक साकषय स पता चलता ह वक पराची घािी

सभयता हडपपा और म हनज दाड द न ो की

पिििती ह

Q66) उततर (d)

सपषटीकरण

य समारक छतरपर वजल (मधय परदश) म विोधयाचल

पिित शरोखला म सथथत ह

Q67) उततर (a)

सपषटीकरण

bull थॉिस ऑन पावकसतान नामक पसतक डॉ बी

आर अमबडकर न वलखी थी

bull डॉ बी आर अमबडकर की जयोती क अिसर पर

भारत क राषटर पवत न भारत की इस महान हसती

क शरदाोजवल अवपित की थी

bull डॉ बी आर अमबडकर न 1924 म वडपरथड

कलावसर इोसटीटयि (दवलत िगि सोथथान -

बवहषकत वहतकाररणी सभा) और 1927 म समाज

समता सोघ की थथापना की थी

bull अमबडकर का धयान वशकषा कषतर की ओर भी था

bull उनह ोन वशकषा क वनमन िगो म फलान क वलए

पीपलस एजकशन स साइिी (The Peoples

Education Society) क नाम स महाविदयालय ो क

नििकि और छातरािास ो की थथापना की थी

FC19H1003 39

Q68) उततर (b)

सपषटीकरण

bull महरगि भारतीय उपमहादवीप म एक परवसद

निपािाण बसती ह ज बलवचसतान पराोत

पावकसतान म सथथत ह

bull दचपलली (आोधर परदश) क पास नागलर नदी क

पिी ति ो पर चना पतथर क बलॉक क विशाल

विसतार म एक पिि-ऐवतहावसक रॉक आिि थथल की

ख ज की गई ह

bull इसन 1500-2000 ईसा पिि क दौरान गोिर (आोधर

परदश) म विकवसत निपािाण सभयता पर परकाश

डाला ह

Q69) उततर (c)

सपषटीकरण

bull 12िी ो सदी और 13िी ो सदी म काकाविय िोश का

उदय हआ था

bull ि पहल कलयाण क पवशचमी चालकय ो क सामोत थ

bull परारोभ म उनह ोन िारोगल (तलोगाना) क पास एक

छ ि स कषतर पर शासन वकया था

bull उनह ोन ldquoनायक वयिथथाrdquo की शरआत की थी

वजस बाद म विजयनगर क राय शासक ो न

अपनाया और विकवसत वकया था

Q70) उततर (a)

सपषटीकरण

bull गाोिीजी क अनशन स वमल मावलक ो पर दबाि

पडा था ज अोततः शरवमक ो क ितन म 35 परवतशत

की िसद करन क वलए सहमत हए थ

bull गगल (Google) न अनसया साराभाई वजनह ोन

भारत क शरवमक आोद लन म एक अगरणी भवमका

वनभाई थी की 132िी ो जयोती डडल (Doodle) का

वनमािण करक मनाई

Q71) उततर (d)

सपषटीकरण

भारत स यनसक की मानिता की अमति साोसकवतक

विरासत की परवतवनवि सची म वनमनवलसखत शावमल ह

bull कवडयटटम करल का सोसकत रोगमोच

bull मवडयिि करल का अनषठान रोगमोच और नतय

नाविका

bull िवदक मि जाप की परोपरा

bull राजथथान क कालबवलया ल क गीत और नतय

bull रामलीला रामायण का पारोपररक परदशिन

bull सोकीतिन मवणपर का अनषठान गायन ढ ल िादन

और नतय

bull रममन भारत क गििाल वहमालय का िावमिक

तयौहार और अनषठान रोगमोच

bull जाोदीयाला गर पोजाब क ठठर ो की पीतल और

ताोब क वशलप स वनवमित बतिन ो की पारोपररक कला

bull छाऊ नतय पिी भारतीय राजय ो म जनमी शासतरीय

भारतीय नतय कला

bull लददाख का बौद मि जाप िर ाोस-वहमालयी लददाख

कषतर तथा जमम-कशमीर म पवितर बौद गरोथ ो का पाठ

bull य ग

bull नौर र

bull को भ मला

Q72) उततर (b)

सपषटीकरण

bull भारत क राषटर पवत शरी राम नाथ क विोद न

वकसामा नागालड म हॉनिवबल मह रति और

राजय गठन वदिस समार ह का उदघािन वकया

था

bull हॉनिवबल मह रति का नाम भारतीय हॉनिवबल क

नाम पर पडा ह ज एक विशाल और रोगीन जोगली

पकषी ह

bull यह पकषी नागालड राजय की अविकतर जनजावतय ो

की ल ककथाओो म उसललसखत ह

bull नागालड की परमख मानयता परापत जनजावतयाा ह

अोगामी आओ चखसोग चाोग ककी रगमा और

रवलोग आवद

bull ओोग जारिा और ससिनलीस अोडमान-वनक बार

दवीप समह की जनजावतयाा ह

FC19H1003 40

Q73) उततर (c)

सपषटीकरण

bull दकन म राषटर कि शासन दसिी ो सदी क अोत तक

लगभग 200 ििो तक रहा था

bull राषटर कि शासक अपन िावमिक विचार ो म सवहषण

bull उनह ोन न किल शि िमि और िषणि िमि बसलक

जन िमि क भी सोरकषण वदया था

bull एल रा म वशि क परवसद रॉक कि मोवदर का

वनमािण नौिी ो सदी म राषटर कि राजा कषण परथम न

करिाया था

bull उसका उततराविकारी अम घििि जन था लवकन

उसन अनय िमो क भी सोरकषण परदान वकया था

bull राषटर कि ो न मसलमान वयापाररय ो क बसन की

अनमवत दी थी

bull उनह न अपन अविराजय ो म इसलाम क उपदश दन

की भी अनमवत दी थी

bull अभी हाल ही म पाोडिलागटटा (तलोगाना) क

परागवतहावसक चटटान वचतर ो क कषरण की बिती हई

घिनाएा एक गोभीर वचोता का वििय ह

bull यह परागवतहावसक चटटान क नकसान पहाचा

सकता ह

bull पाोडिलागटटा वनमनवलसखत क वलए जाना जाता ह

- 10000 ईसा पिि स 8000 ईसा पिि क वचवतरत

चटटानी आशरय ो क वलए

- राषटर कि काल क एक 8 िी ो सदी क

वशलालख क वलए और

- 12िी ो सदी क काकविय सामराजय क वभवतत

वचतर ो क वलए

Q74) उततर (b)

सपषटीकरण

bull 1828 म राजा राम म हन रॉय न एक नय िावमिक

समाज बरहम सभा की थथापना की थी वजस बाद

म बरहम समाज क नाम स जाना गया था

bull दिदरनाथ िग र न ततवब विनी सभा की अधयकषता

की थी ज आधयासिक सतय की ख ज म सोलि

थी

bull इसका उददशय वहोद िमि क शद करन का और

एकशवरिाद (एक ईशवर म आथथा) का परचार करना

था

bull नय समाज की थथापना क आिार थ कारण

(तकि ) क द सतमभ तथा िद और उपवनिद

bull अभी हाल ही म सािारण बरहम समाज का कछ

काननी मदद ो क लकर पवशचम बोगाल सरकार क

साथ काननी वििाद चल रहा ह

Q75) उततर (c)

सपषटीकरण

bull भारत म वचशती वसलवसल की थथापना खवाजा

म इनददीन वचशती क दवारा की गयी थी

bull ि 1192 ईसवी क आसपास भारत आय थ

bull वचशतीय ो क बारहिी ो शताबदी क उततरािि म भारत

म आन िाल सफीय ो क समह ो म सबस

परभािशाली माना जाता ह

bull उनह ोन थथानीय िातािरण क साथ सफलतापििक

अनकलन वकया और उनह ोन भारतीय भसकत

परोपराओो क कई पहलओो क अपनाया

bull अजमर म सफी अपरकि खवाजा म इनददीन वचशती

की ऐवतहावसक दरगाह क एक नया रप दन की

तयारी की जा रही ह

bull इस 13िी ो शताबदी की दरगाह क ldquoसवचछ

आइकॉवनक थथल ोrdquo (Swacch Iconic Places) म

शावमल वकया गया ह ज परवतवषठत विरासत

आधयासिक और साोसकवतक थथान ो पर क वदरत

य जना ह

FC19H1003 41

ANSWERS amp EXPLANATION OF

NCERT History Class VI-X + Current Affairs

(FC19E1003)

Q1) Answer c

Explanation

Rigveda consists of more than a

thousand hymns dedicated to gods and

goddesses These hymns were

composed by sages and learnt by men

however a few were composed by

women like Apala Ghosa Lopamudra

Maitreyi and Gargi

Rigveda consists of many hymns in the

form of dialogues We get an example of

a dialogue between a sage named

Vishwamitra and two rivers (Beas and

Sutlej) that were worshipped as

goddesses This suggests that he

belonged to the Vedic period

Q2) Answer b

Explanation

Traces of ash have been found from

Kurnool Caves suggesting that people

were familiar with the use of fire

It is situated in Andhra Pradesh

Q3) Answer c

Explanation

Burzahom is a prehistoric site in

present day Kashmir where people built

pit houses which were dug into the

ground with steps leading into them

These may have provided shelter in cold

weather

Q4) Answer c

Explanation

Epigraphy is defined as the study of

inscriptions

Manuscriptology is the study of history

and literature through the use of hand

written documents

Palaeography refers to the study of

ancient writing systems and the

deciphering and dating of historical

manuscripts

Numismatics refers to the study of

coins

Q5) Answer a

Explanation

Charaka Samhita was written by

Charaka and is an important book on

Ayurveda and medicine

He was a practitioner of the traditional

system of Indian medicine known as

Ayurveda

Charaka is thought to have flourished

sometime between the 2nd century BCE

and the 2nd century CE

Q6) Answer b

Explanation

Bhaga refers to the tax on crops which

was fixed at 16th of the production

Kammakaras is the term used for the

landless agricultural labour class

Ashvamedha also known as horse

sacrifice is a ritual where a horse is let

loose to wander freely and it was

guarded by the rajarsquos men

Q7) Answer (d)

Explanation

In the Rigvedic period horses were

yoked to chariots that were used in

battles fought to capture land cattle

etc This suggests that the use of horse

chariots began much before the period

of Mahajanapadas

The battles were fought in the Rigvedic

period for cattlersquos lands water an even

to capture people Most men took part

in these wars however there was no

regular army but there were assemblies

where people met and discussed

matters of war Regular armies became

a feature in the Mjahajanapada period

including vast armies of foot soldiers

chariots and elephants

RAUSIAS-FC19E1003 42

Q8) Answer (a)

Explanation

Buddha belonged to the Sakya clan and

passed away at Kusinara

Buddha taught in Prakrit which was the

common language of people

Q9) Answer c

Explanation

There were six schools of philosophy in

ancient India These are known as

Vaishesika Nyaya Samkhya Yoga

Purva Mimansa and Vedanata or Uttara

Mimansa They were founded by sages

Kanada Gautama Kapila Patanjali

Jamini and Vyasa respectively

Q10) Answer b

Explanation

The teachings of Mahavira were

compiled at Valabhi in 6th century AD

Q11) Answer (c)

Explanation

Chanakya is traditionally identified as

Kautilya or Vishnugupta who authored

the ancient Indian political treatise the

Arthashastra

Q12) Answer d

The national emblem of India is an

adaptation of the Lion Capital atop the

Ashoka Pillar of Sarnath Uttar Pradesh

and is combined with the National

Motto Satyameva Jayate

The Rampurva Bull gets the name from

the site of its discovery Rampurva in

Bihar

It is noted for its delicately sculpted

model demonstrating superior

representation of soft flesh sensitive

nostrils alert ears and strong legs It is

a mixture of Indian and Persian

elements

Sankissa is situated in Uttar Pradesh

India

Q13) Ans(a)

Kunwar Singh was a notable leader during the Revolt of 1857 He belonged

to a royal house of Jagdispur Bihar

Q14) Answer b

Explanation

The term Vellalar was used for large

landowners

Q15) Answer c

Explanation

Arikamedu was a coastal settlement

where ships unloaded goods from

distant lands Finds here include a

massive brick warehouse pottery

including amphorae and Arretine ware

Roman lamps glassware and gems have

also been found at the site

Q16) Answer a

Explanation

Muvendar is a Tamil word mentioned in

Sangam poems meaning three chiefs

used for the heads of three ruling

families the Cholas Cheras and

Pandyas

Q17) Ans (c)

Several tribal or kin-based assemblies

such as the Sabha Vidatha and gana

are mentioned in the Rig-veda The

Sabha and the samiti mattered a great

deal in early Vedic times so much so

that the chiefs or the kings showed an

eagerness to win their support

Q18) Ans (a)

Jainism recognised the existence of the

gods but placed them lower than the

jina and did not condemn the varna

system as Buddhism did

Q19) Answer (d)

Explanation

Cholas and Pandyas had developed

powerful coastal cities The most

important city of Cholas was Puhar or

Kaveripattinam and Madurai was the

capital of Pandyas

Q20) Answer b

Explanation

Buddhacharita is the biography of

Buddha and was written by

RAUSIAS-FC19E1003 43

Ashvaghosha

Q21) Answer (a)

Explanation

Tamil poet Appar was a Shiva devotee

So he was a Nayanar saint

Q22) Answer d

Explanation

Samudragupta was a prominent Gupta

ruler whose coins depict him playing a

veena indicating his love for music We

get important historic information from

his Allahabad Prashasti which was

composed by his court poet Harisena

Q23) Answer (b)

Explanation

Vikrama Samvat was founded by

Chandragupta II in the 58 BC as a

mark of victory over the Shakas and

assumed the title of Vikramaditya

Banabhatta wrote Harshavardhanarsquos

biography the Harshacharita in

Sanskrit

Q24) Answer c

Explanation

Sandhi-vigrahika was the minister of

war and peace

Sarthavaha was the leader of the

merchant caravans

Q25) Answer a

Explanation

Xuan Zang (Hsuan-tsang) was a

Chinese traveller who came during the

reign of Harshavardhana

In the decade that began in 630 AD

Xuan Zang came to India through

Kashmir after visiting Central Asia Iran

and Afghanistan

He travelled from north to east and lived

in Bihar for a couple of years

At Nalanda University Xuan Zang

interacted with students and scholars

mastered local languages and

discovered Buddhist stupas

Q26) Answer c

Explanation

Pradakshina patha is a circular path

laid around a stupa in Buddhist

architecture While the rest are a part of

temple architecture

Q27) Answer d

Explanation

All the above-mentioned temples have

an elaborate use of bricks (baked

bricks) along with stone

Q28) Ans (c)

Muhammad Quli Qutab was the Sultan

of Golconda He was a contemporary of

Akbar was very fond of literature and

architecture

The Sultan was a great poet and he

wrote in Dakhini Urdu Persian and

Telgu and has left an extensive diwan or

collection

Recently the Archaeological Survey of

India (ASI) will be using Ground

Penetrating Radar (GPR) to map the

contours of the area around the Bagh-e-

Naya Qila excavated garden inside the

Golconda Fort in Telangana

Q29) Answer a

Explanation

Silappadikaram is a famous Tamil epic

which was written by Ilango around

1800 years ago It is a story of a

merchant named Kovalan who fell in

love with a courtesan named Madhavi

Manimekalai tells the story of the

daughter of Kovalan and Madhavi

Q30) Answer (a)

Explanation

Charaka is the author of Charaka

Samhita which is an important work of

Ayurveda and medicines

Brahmaguptarsquos fame rests mostly on his

Brahma-sphuta-siddhanta which was

an astronomical work It was translated

into Arabic in Baghdad and had a major

impact on Islamic mathematics and

astronomy

Late in his life Brahmagupta wrote

Khandakhadyaka which was an

RAUSIAS-FC19E1003 44

astronomical handbook that employed

Aryabhatarsquos system of starting each day

at midnight

Q31) Answer (c)

Explanation

Amir Khusrau was a famous sufi

musician poet and scholar In 1318 he

noted that there was different language

in every region of this land (Hindustan)

Lahori Kashmiri Dvarsamudri (in

Southern Karnataka) Telangana (in

Andhra Pradesh) Gujari (in Gujarat)

Marsquobari (in Tamil Nadu) Awadhi (in

eastern Uttar Pradesh) and Hindawai (in

the area around in Delhi) etc He went

to explain that Sanskrit did not belong

to any region and that only brahmans

knew it

Q32) Answer c

Explanation

Hiranyagarbha refers to the golden

womb When this ritual was performed

with the help of Brahmanas it was

thought to lead to the rebirth of the

sacrificer as a Khastriya

Q33) Answer d

Explanation

Kadamai refers to a tax on land

revenue

Gwalior Prashasti describes the exploits

of Nagabhata who was a Pratihara king

Q34) Answer b

Explanation

Rajatarangini is a Sanskrit text written

by Kalhana in the 12th century

It was historical chronicle of early India

It is justifiably considered to be the best

and most authentic work of its kind

It covers the entire span of history in

the Kashmir region from the earliest

times to the date of its composition

Q35) Answer c

Explanation

ldquoUrrdquo was the general assembly of the

village ldquoUrrdquo consisted of all the

taxpaying residents of an ordinary

village

Q36) Answer (a)

Explanation

Tarikh was a form of history writing in

the Delhi Sultanate The authors of

tawarikhs were learned men which

included secretaries administrators etc

Q37 Answer (a)

Explanation

Alauddin chose to pay his soldiers salaries in cash rather than iqtas The soldiers would buy their supplies from merchants in Delhi and it was thus feared that merchants would raise their prices To stop this Alauddin controlled the prices of goods in Delhi Prices were carefully surveyed by officers and merchants who did not sell at the prescribed rates were punished

Q38) Answer (d)

Explanation

Delhi first became the capital of a

kingdom under the Tomara Rajputs

who were defeated in the middle of the

twelfth century by the Chauhans (also

referred to as Chahamanas) of Ajmer

It was under the Tomaras and

Chauhans that Delhi became an

important commercial centre Many rich

Jaina merchants lived in the city and

constructed several temples Coins

minted here called dehliwal had a wide

circulation

Q39) Answer (c)

Explanation

Moth ki Masjid was built in the reign of

Sikandar Lodi by his minister

Begumpuri mosque built in the reign of

Muhammad Tughluq was the main

mosque of Jahanpanah the ldquoSanctuary

of the Worldrdquo and his new capital in

Delhi

Quwwat al ndash Islam mosque was

enlarged by Iltutmish and Alauddin

Khalji The minar was built by three

Sultansndash Qutbuddin Aybak Iltutmish

and Firuz Shah Tughluq

RAUSIAS-FC19E1003 45

Q40) Answer (c)

Explanation

Under the Mughals mansabdar was

referred to an individual who held a

mansab ie rank and he received his

salary as revenue assignments called

jagirs

Q41) Ans (b)

The Quit India Movement was a

spontaneous revolt of people against

British rule

The All India Congress Committee met

at Bombay on 8 August 1942 It passed

the famous resolution Quit India and

proposed the starting of a non-violent

mass struggle under Gandhis

leadership to achieve this aim But on

the very next day Gandhi and other

eminent leaders of the Congress were

arrested The Congress was once again

declared illegal

Q42) Ans (c)

The Simon Commission refers to a

group of seven MPs from the United

Kingdom constituted to suggest

constitutional reforms for British India

The Commission consisted of only

British members headed by one of the

senior British politicians Sir John

Simon

So the people of India agitated against

the arrival of Simon Commission

Q43) Ans (a)

He was widely known for his

unfavourable opinion of the economic

consequences of the British rule in

India

In his many writings and speeches and

especially in Poverty and Un-British

Rule in India Naoroji argued that India

was too highly taxed and that its wealth

was being drained away to England

He did not interpret the ancient Indian

texts and restored the self-confidence of

Indians And also he did not stress the

need for eradication of all the social

evils before anything else

Q44) Ans (c)

In August 1932 Prime Minister

MacDonald announced his Communal

Award Great Britainrsquos unilateral

attempt to resolve the various conflicts

among Indiarsquos many communal

interests

The award which was later

incorporated into the act of 1935

expanded the separate-electorate

formula reserved for Muslims to other

minorities including Sikhs Indian

Christians Anglo-Indians Europeans

distinct regional groups Gandhi

undertook a ldquofast unto deathrdquo against

that offer which he viewed as a

nefarious British plot to divide the

Indian society

Q45) Ans (b)

In British India apart from existing

imports and exports there was also a

particular amount of money which

colonial India contributed towards

administration maintenance of the

army war expenses pensions to retired

officers and other expenses accrued by

Britain towards maintenance of her

colony These were known as Home

charges and were paid for almost

entirely by India

The Home charges was made of

following components-

- Interest payable on Indian debt

- Dividend to shareholders of East

India Company

- Funds used to support the India

Office in London

- Funds used to pay salaries and

pensions of British personnel

engaged in India

- Interest on the railways

- Civil and military charges

- Store purchases in England

Q46) Ans (b)

The Lahore session of the Indian

National Congress was held in 1929

under the Presidentship of Jawaharlal

Nehru

The Lahore session of the Indian

National Congress witnessed significant

RAUSIAS-FC19E1003 46

developments in the Indian national

movement

- First the election of Jawaharlal

Nehru to the post of Presidentship of

the Congress was a clear indication

of the growing strength of the

Leftists in the Congress

- Secondly it was in this session that

the Congress for the first time raised

the demand for complete

independence Such demand was

not raised from the Congress

platform earlier

Q47) Ans (b)

It did not provide for separate

electorates for any community or

weightage for minorities However it did

allow for the reservation of minority

seats in provinces having minorities of

at least ten per cent but this was to be

in strict proportion to the size of the

community

There was no provision for complete

Independence for India

Q48) Ans (c)

The religion of early Vedic Aryans was

primarily of worship of nature and

Yajnas

The early Aryan religion was kind of

nature worship Actually the forces

around them which they could not

control or understand were invested

with divinity and were personified as

male or female gods And they

performed some Yajnas also

Q49) Ans (b)

The roads and river-routes were not

immune from robbery It is notable that

Yuan Chwang (Hiuen Tsang) was

robbed of his belongings during

Harshvardanarsquos period

Q50) Ans (c)

Q51) Ans (b)

Purandara Dasa was a saint and great

devotee of Lord Krishna

There is much speculation about where

Purandara Dasa regarded as the

Pitamaha of Carnatic music was born

Recently an expert committee

constituted by the Kannada University

Hampi has come to the conclusion that

Kshemapura Shivamogga district

Karnataka is the birth place of

Purandara Dasa

Q52) Ans (c)

Sri Tyagaraja Sri Shyama Shastry and Sri Muthuswami Dikshitar are considered the trinity of Carnatic music and with them came the golden age in Carnatic music in the 18th-19th

century

Q53) Ans d)

Recently a rare sarcophagus (stone

coffin) which is 2000 years old from the

Iron AgendashMegalithic era was discovered

from a rock-cut cave at Viyur village of

Kollam near Koyilandy in Kozhikode

district Kerala

The coffin containing bone fragments

was found during an excavation ldquoSo

far such a rare finding has been

discovered only from two sites

in Kerala Both these sarcophagi were

recovered from Megalithic sites at

Chevayur and Atholi also in Kozhikode

district

Q54) Ans a)

The megalithic culture in South India was a full-fledged Iron Age culture

Q55) Ans d)

The Cholas Pandyas and Keralaputras

(Cheras) mentioned in Ashokan

inscriptions were probably in the late

megalithic phase of material culture

Q56) Ans d)

Q57) Ans (b)

Raj Kumar Shukla followed Gandhiji all

over the country to persuade him to

come to Champaran to investigate the

problem associated with tinkathia

system

RAUSIAS-FC19E1003 47

Brij Kishore Rajendra Prasad Mahadev

Desai and Narhari Parikh accompanied

Gandhi ji during the Champaran

Satyagraha

Q58) Ans (b)

The Satvahanas started the practice of granting tax-free villages to brahmanas and Buddhist monks

Q59) Ans c)

The objectives of the Programme are

listed as under

- Developing basic tourism

infrastructure

- Promoting cultural and heritage

value of the country to generate

livelihoods in the identified regions

- Enhancing the tourist attractiveness

in a sustainable manner by

developing world-class

infrastructure at the heritage

monument sites

- Creating employment through active

involvement of local communities

- Harnessing tourism potential for its

effects on employment generation

and economic development

- Developing sustainable tourism

infrastructure and ensuring proper

Operations and maintenance

therein

Q60) Ans (b)

The Tribal Cooperative Marketing

Development Federation of India

(TRIFED) came into existence in 1987

It is a national-level apex organization

functioning under the administrative

control of Ministry of Tribal Affairs

Govt of India

TRIFED has its registered and Head

Office located in New Delhi

Q61) Ans (c)

Premchandrsquos novels include

Premashram Rangabhumi Ghaban

Karmabhumi and Godan

Gora is a novel written by Rabindranath

Tagore

138th birth anniversary of Munshi

Premchand was celebrated across the

country

Q62) Ans (b)

Giddha is a traditional pastoral dance

performed by the women of the Punjab

India and Pakistan at festival times

and at the sowing and reaping of the

harvest

By this dance the Punjabi women

reveal their joy expel their suppressed

feelings in a male dominated society

through the performance of Giddha

Since this dance has nothing to do with

men only women can participate in it

During the Teej celebrations Giddha

dance is celebrated in Punjab every

year Teej is a generic name for a

number of festivals that are celebrated

by women in some parts of India

Q63) Ans (a)

Dara Shukoh wrote the remarkable

work called ldquoMajma-ul-Bahrainrdquo or the

ldquoThe confluence of two seasrdquo

The Vice President of India Shri M

Venkaiah Naidu has said that Prince

Dara Shukohrsquos writings can come as a

refreshing source for infusing peace and

harmony He was addressing the

gathering after visiting the exhibition

that showcases the forgotten Prince of

yesteryears Dara Shukoh organized by

Mr Francois Gautier at Indira Gandhi

National Centre for the Arts in New

Delhi

Q64) Ans (c)

The statue Gommateshwara is

dedicated to the Jain God Bahubali

It is a monolithic statue

President Ram Nath Kovind

inaugurated the grand anointing

ceremony mdash Mahamastakabhisheka mdash

held once in 12 years at

Shravanabelagola (Karnataka)

Q65) Ans (c)

Prachi Valley had come up around the

Prachi river Prachi Valley gradually

disappeared

RAUSIAS-FC19E1003 48

The Prachi river originates from

Bhubaneswar

It is a tributary of the Mahanadi and

flows through the districts of Puri

Khurda Cuttack and Jagatsinghpur

and the entire region of the river is

termed as the Prachi Valley

It falls into the Bay of Bengal

Archaeological evidence shows that the

Prachi Valley Civilisation predates both

Harappa and Mohenjo-Daro

The Prachi river originates from

Bhubaneswar

Q66) Ans (d)

These monuments are located in

Chhatarpur district Madhya Pradesh

within Vindhya mountain range

Q67) Ans (a)

The book lsquoThoughts on Pakistanrsquo was

written by Dr BR Ambedkar

On the occasion of the birth anniversary

of Dr BR Ambedkar the president of

India pays homage to this icon of India

In 1924 he founded the Depressed

Classes Institute (Bahishkrit Hitkarini

Sabha) and in 1927 the Samaj Samata

Sangh

Another area of attention for Ambedkar

was education For its spread among

the low classes he set up a network of

colleges by the name of Peoples

Education Society and founded hostels

Q68) Ans(b)

Mehrgarh is a famous Neolithic

settlement in the Indian subcontinent

which is situated in Baluchistan

province Pakistan

A pre-historic rock art site is discovered

in the vast expanse of limestone blocks

on the eastern banks of Naguleru river

near Dachepalli (Andhra Pradesh) It

has thrown light on the Neolithic

civilisation that flourished in Guntur

(Andhra Pradesh) during 1500-2000

BC

Q69) Ans (c)

The 12th and the 13th centuries saw

the emergence of the Kakatiyas They

were at first the feudatories of the

Western Chalukyas of Kalyana Initially

they ruled over a small territory near

Warangal (Telangana)

They introduced Nayakships which was

later adopted and developed by the

Rayas of Vijayanagara

Q70) Ans (a)

The fast had effect of putting pressure

on mill owners who finally agreed to

give the workers a 35 per cent increase

in wages

Google celebrated with a doodle the

132nd birth anniversary of Anasuya

Sarabhai who played a pioneering role

in Indiarsquos labour movement

Q71) Ans (d)

The UNESCOrsquos list of the representative

list of the intangible cultural heritage of

humanity from India are

- Koodiyattam Sanskrit Theatre of

Kerala

- Mudiyettu ritual theatre and dance

drama of Kerala

- Tradition of Vedic Chanting

- Kalbelia folk songs and dances of

Rajasthan

- Ramlila Traditional Performance of

the Ramayana

- Sankirtana ritual singing

drumming and dancing of Manipur

- Ramman religious festival and

ritual theatre of the Garhwal

Himalayas India

- Traditional brass and copper craft of

utensil making among the Thatheras

of Jandiala Guru Punjab India

- Chhau dance classical Indian dance

originated in the eastern Indian

states

- Buddhist chanting of Ladakh

recitation of sacred Buddhist texts

in the trans-Himalayan Ladakh

region Jammu and Kashmir India

- Yoga

- Nouroz

- Kumbh Mela

RAUSIAS-FC19E1003 49

Q72) Ans(b)

The President of India Shri Ram Nath Kovind inaugurated the Hornbill Festival and State Formation Day celebrations of Nagaland in Kisama

The festival is named after the Indian hornbill the large and colourful forest bird which is displayed in the folklore of most of the states tribes

The major recognized tribes of Nagaland are Angami Ao Chakhesang Chang

Kuki Rengma and Zeling etc

Onge Jarawa and Sentinelese are the

tribes of Andman amp Nicobar Islands

Q73) Ans (c)

The Rashtrakutas rule in the Deccan lasted for almost two hundred years till the end of the tenth century The Rashtrakutas rulers were tolerant in their religious views and patronized not only Shaivism and Vaishnavism but

Jainism as well

The famous rock-cut temple of Shiva at Ellora was built by one of the Rashtrakutas kings Krishna I in the ninth century His successor Amoghavarsha was a Jain but he also

patronized other faiths

The Rashtrakutas allowed Muslims traders to settle and permitted Islam to

be preached in their dominions

Recently increasing defacement at the prehistoric rock paintings of Pandavulagutta Telangana has created a cause for grave concern It can spoil

the prehistoric rock

Pandavulagutta is home to

- Painted rock shelters dating to

10000 BC-8000 BC

- An 8th century inscription of the

Rashtrakuta period and

- Painted frescoes from the 12th century Kakatiya empire

Q74) Ans (b)

In 1828 Raja Ram Mohan Roy founded a new religious society the Brahma Sabha later known as the Brahmo

Samaj

Debendranath Tagore headed the Tattvabodhini Sabha which was

engaged in search of spiritual truth

Its purpose was to purify Hinduism and to preach monotheism or belief in one God

The new society was to be based on the twin pillars of reason and the Vedas and

Upanishads

Recently Sadharan Brahmo Samaj (SBS) has entered into a legal battle with the West Bengal government due

to some legal issue

Q75) Ans (c)

The Chishti order was established in India by Khwaja Moinuddin Chishti who came to India around 1192 The Chishtirsquos are considered to be the most influential of the groups of Sufis who migrated to India in the late twelfth century They adapted successfully to the local environment and adopted several features of Indian devotional

traditions

The historical dargah of Sufi mystic Khwaja Moinuddin Chishti in Ajmer is all set to get a facelift This 13 th century dargah has been included among the Swachh Iconic Places a clean-up initiative focused on iconic

heritage spiritual and cultural places

Page 6: GENERAL STUDIES (PAPER I) · Test is part of Rau’s IAS Test series for Preliminary Exam 2019 FOUNDATION + CURRENT AFFAIRS GENERAL STUDIES (PAPER –I) FOUNDATION TEST –III TOPIC:

RAUSIAS-FC19E1003 6

Q14) निमननिखित यग ो पर निचार कीनजए

1 िललािर सनिक

2 आनदमाई दास (गिाम)

3 कदनसयर भनमहीि मजदर

उपयणकत यग ो म स कौि-स सही समनित ह

(a) किि 1 और 2

(b) किि 2 और 3

(c) किि 1 और 3

(d) 1 2 और 3

Q15) निमननिखित सथि ो म स उस सथि की पहचाि कीनजए

ज एक परनसदध तटीय बसती थी और जहा एरटाइि

(Arretine) मदभाोड पाया गया था

(a) महाबिीपरम

(b) तामरनिखपत

(c) अररकमड

(d) अतरोजीिरा

Q16) सोगम सानहतय क सोदभण म निमननिखित कथि ो पर

निचार कीनजए

1 मदरई म सोगमसभाए आय नजत की जाती ो

थी ो

2 मिनदर शबद का परय ग सोगम कनिताओो म

नकया जाता था नजसका अथण ह द राजाओो

का एक समह

उपयणकत कथि ो म स कौि-सास सही हह

(a) किि 1

(b) किि 2

(c) 1 और 2 द ि ो

(d) ि त 1 ि ही 2

Q17) निमननिखित कथि ो पर निचार कीनजए

1 आरखिक िनदक काि म मनहिाऐो सभाओो

और निदाथाओो म भाग िती थी ो

2 आरखिक िनदक काि म िनशषठ ज एक

पजारी ह ता था की एक महतवपरण भनमका

ह ती थी

उपयणकत कथि ो म स कौि-सास सही हह

(a) किि 1

(b) किि 2

(c) 1 और 2 द ि ो

(d) ि त 1 ि ही 2

Q18) निमननिखित कथि ो पर निचार कीनजए

1 किाणटक म जि रमण क निसतार का शरय

चनदरगपत मौयण क जाता ह

2 जि रमण ि ईशवर क अखसततव क मानयता दी ह

तथा िरण पररािी की भरतणिा की ह

उपयणकत कथि ो म स कौि-सास सही हह

(a) किि 1

(b) किि 2

(c) 1 और 2 द ि ो

(d) ि त 1 ि ही 2

Q19) निमननिखित यग ो पर निचार कीनजए

1 पहार पाणडय

2 मदरई च ि

उपयणकत यग ो म स कौि-सास सही समनित हह

(a) किि 1

(b) किि 2

(c) 1 और 2 द ि ो

(d) ि त 1 ि ही 2

RAUSIAS-FC19E1003 7

Q14) Consider the following pairs

1 Vellalar Soldier

2 Adimai Slaves

3 Kadaisiyar Landless labourers

Which of the pairs given above isare

correctly matched

(a) 1 and 2 only

(b) 2 and 3 only

(c) 1 and 3 only

(d) 1 2 and 3

Q15) Identify the site which was a famous

coastal settlement from which Arretine

ware has been found

(a) Mahabalipuram

(b) Tamralipti

(c) Arikamedu

(d) Atranjikhera

Q16) With respect to the Sangam literature

consider the following statements

1 The Sangamsassemblies were

held at Madurai

2 The term muvendar was used in

sangam poems which mean a

group of two kings

Which of the statements given above

isare correct

(a) 1 only

(b) 2 only

(c) Both 1 and 2

(d) Neither 1 nor 2

Q17) Consider the following statements

1 During Early Vedic Period women

attended the Sabha and Vidatha

2 The priest ndash Vasishtha played

important role in Early Vedic

Period

Which of the statements given above

isare correct

(a) 1 only

(b) 2 only

(c) Both 1 and 2

(d) Neither 1 nor 2

Q18) Consider the following statements

1 The spread of Jainism in

Karnataka is attributed to

Chandragupta Maurya

2 Jainism recognised the existence

of the gods and condemns the

Varna System

Which of the statements given above

isare correct

(a) 1 only

(b) 2 only

(c) Both 1 and 2

(d) Neither 1 nor 2

Q19) Consider the following pairs

1 Puhar Pandyas

2 Madurai Cholas

Which of the pairs given above isare

correct

(a) 1 only

(b) 2 only

(c) Both 1 and 2

(d) Neither 1 nor 2

RAUSIAS-FC19E1003 8

Q20) निमननिखित कथि ो म स कौि-सास सही हह

1 बदधचररत का ििक िागसि ह

2 ब नरसतव की पजा महायाि बौदधमत का एक

महतवपरण भाग थी

िीच नदए गए कट का परय ग कर सही उततर चनिए

(a) किि 1

(b) किि 2

(c) 1 और 2 द ि ो

(d) ि त 1 ि ही 2

Q21) निमननिखित कथि ो म स कौि-सास सही हह

1 भखकत क निचार क भागित गीता म सपषट

नकया गया ह

2 तनमि कनि अपपर एक अििर सोत थ

िीच नदए गए कट का परय ग कर सही उततर चनिएः

(a) किि 1

(b) किि 2

(c) 1 और 2 द ि ो

(d) ि त 1 ि ही 2

Q22) निमननिखित म स नकस शासक ि िीरा बजात हए

अपिी छनि नसक ो पर अोनकत करिाई थी

(a) नमिाोदर

(b) चनदरगपत मौयण

(c) गौतमीपतर सतकरी

(d) समदरगपत

Q23) निमननिखित कथि ो म स कौि-सास सही हह

1 सवतोतर भारत ि निकरम सोित क राषटर ीय

किनडर क रप म अपिाया और यह 68 ईसा

पिण म आरि हआ था

2 बािभटट हिणिरणि क दरबार का एक कनि था

िीच नदए गए कट का परय ग कर सही उततर चनिएः

(a) किि 1

(b) किि 2

(c) 1 और 2 द ि ो

(d) ि त 1 ि ही 2

Q24) निमननिखित यग ो पर निचार कीनजएः

1 सोनर-निगरानहका वयापार मोतरी

2 परथम-कनिका परमि नशलपकार

3 साथणिाह परमि बकर

उपयणकत यग ो म स कौि-सास सही समनित हह

(a) किि 1

(b) किि 1 और 3

(c) किि 2

(d) किि 2 और 3

Q25) निमननिखित कथि ो म स कौि-सास सही हह

1 lsquoएह ि नशिाििrsquo पिकनशि नदवतीय स

सोबोनरत ह और रनिकनत क दवारा इसकी रचिा

की गई थी

2 जआि झाोग समदरगपत नदवतीय क शासिकाि

म भारत आया था

िीच नदए गए कट का परय ग कर सही उततर चनिएः

(a) किि 1

(b) किि 2

(c) 1 और 2 द ि ो

(d) ि त 1 ि ही 2

RAUSIAS-FC19E1003 9

Q20) Which of the following statements

isare correct

1 Buddhacharita is authored by

Nagasena

2 The worship of Bodhisattvas was

an important part of Mahayana

Buddhsim

Select the correct answer using the code

given below

(a) 1 only

(b) 2 only

(c) Both 1 and 2

(d) Neither 1 nor 2

Q21) Which of the following statements

isare correct

1 The idea of Bhakti is elucidated in

Bhagavata Gita

2 Tamil poet Appar was an Alvar

saint

Select the correct answer using the code

given below

(a) 1 only

(b) 2 only

(c) Both 1 and 2

(d) Neither 1 nor 2

Q22) Which of the following rulers had his

image inscribed in the coins while

playing a veena

(a) Meander

(b) Chandragupta Maurya

(c) Gautamiputra Satkarni

(d) Samudragupta

Q23) Which of the following statements

isare correct

1 Vikrama Samvat is adopted as the

national calendar by independent

India and it began in 68 BC

2 Banabhatta was a court poet of

Harshavardhana

Select the correct answer using the code

given below

(a) 1 only

(b) 2 only

(c) Both 1 and 2

(d) Neither 1 nor 2

Q24) Consider the following pairs

1 Sandhi-vigrahika Minister of trade

2 Prathama-kulika Chief craftsman

3 Sarthavaha Chief banker

Which of the pairs given above isare

correct

(a) 1 only

(b) 1 and 3 only

(c) 2 only

(d) 2 and 3 only

Q25) Which of the following statements

isare correct

1 Aihole inscription belongs to

Pulakeshin II and was composed

by Ravikriti

2 Xuan Zang came to India during

the reign of Chandragupta II

Select the correct answer using the code

given below

(a) 1 only

(b) 2 only

(c) Both 1 and 2

(d) Neither 1 nor 2

RAUSIAS-FC19E1003 10

Q26) िासतकिा स सोबोनरत निमननिखित ततव ो म स कौि-स

ततव किि नहोद मखनदर ो की िासतकिा क भाग ह

1 नशिर

2 मणडप

3 परदनकषरा पथ

4 गभणगह

िीच नदए गए कट का परय ग कर सही उततर चनिएः

(a) किि 1 3 और 4

(b) किि 2 3 और 4

(c) किि 1 2 और 4

(d) 1 2 3 और 4

Q27) निमननिखित मोनदर ो म स कौि-स मोनदर ईोट ो स बि ह

1 दिगढ़ मोनदर

2 भीतरगाि मोनदर

3 िकषमर मोनदर नसरपर

4 बहदशवर मोनदर

िीच नदए गए कट का परय ग कर सही उततर चनिएः

(a) किि 1 2 और 3

(b) किि 2 3 और 4

(c) किि 1 3 और 4

(d) 1 2 3 और 4

Q28) निमननिखित कथि ो म स कौि-सास सही हह

1 सलताि महममद किी कतब शाह अकबर का

समकािीि था

2 िासतकिा क कषतर म महममद किी कतब शाह

ि कई ईमारत ो का निमाणर करिाया था नजिम

स चार मीिार सिाणनरक परनसदध ह

िीच नदए गए कट का परय ग कर सही उततर चनिएः

(a) किि 1

(b) किि 2

(c) 1 और 2 द ि ो

(d) ि त 1 ि ही 2

Q29) निमननिखित यग ो पर निचार कीनजएः

1 मनिमकिाई सततिार

2 अनभजञाि शाको तिम कानिदास

3 नसिपपानदकारम क ििि

उपयणकत यग ो म स कौि-सास सही समनित हह

(a) किि 1 और 2

(b) किि 2

(c) किि 1 और 3

(d) 1 2 और 3

Q30) निमननिखित कथि ो म स कौि-सास सही हह

1 ldquoसशरत सोनहताrdquo नचनकरता पर एक महतवपरण

रचिा ह

2 बरहमगपत और चरक महतवपरण गनरतजञ थ

िीच नदए गए कट का परय ग कर सही उततर चनिए

(a) किि 1

(b) किि 2

(c) 1 और 2 द ि ो

(d) ि त 1 ि ही 2

Q31) अमीर िसर क सनदभण म निमननिखित कथि ो म स

कौि-सास सही हह

1 अमीर िसर ि अपिी रचिाओो म नििा ह

नक सोसकत नकसी भी कषतर स सोबोनरत िही ो थी

और किि बराहमर ही इस भािा का जञाि रित

2 उन ोि नहोदिी और अिरी क अखसततव का

उललि नकया था

िीच नदए गए कट का परय ग कर सही उततर चनिए

(a) किि 1

(b) किि 2

(c) 1 और 2 द ि ो

(d) ि त 1 ि ही 2

RAUSIAS-FC19E1003 11

Q26) Which of the following architectural

elements were only part of Hindu

temple architecture

1 Shikhara

2 Mandapa

3 Pradakshina patha

4 Garbhagriha

Select the correct answer using the code

given below

(a) 1 3 and 4 only

(b) 2 3 and 4 only

(c) 1 2 and 4 only

(d) 1 2 3 and 4

Q27) Which of the following temples isare

made of bricks

1 Deogarh Temple

2 Bhitargaon Temple

3 Lakshmana temple Sirpur

4 Brihadeshvara Temple

Select the correct answer using the code

given below

(a) 1 2 and 3 only

(b) 2 3 and 4 only

(c) 1 3 and 4 only

(d) 1 2 3 and 4

Q28) Which of the following statements

isare correct

1 Sultan Muhammad Quli Qutab

Shah was a contemporary of

Akbar

2 In the field of architecture

Muhammad Quli Qutab Shah

constructed many buildings the

most famous of which is the Char

Minar

Select the correct answer using the code

given below

(a) 1 only

(b) 2 only

(c) Both 1 and 2

(d) Neither 1 nor 2

Q29) Consider the following pairs

1 Manimekalai Sattanar

2 Abhijnana Shakuntalam Kalidasa

3 Silappadikaram Kovalan

Which of the pairs given above isare

correct

(a) 1 and 2 only

(b) 2 only

(c) 1 and 3 only

(d) 1 2 and 3

Q30) Which of the following statements

isare correct

1 Sushruta Samhita is an important

work on medicine

2 Brahmagupta and Charaka were

important mathematicians

Select the correct answer using the code

given below

(a) 1 only

(b) 2 only

(c) Both 1 and 2

(d) Neither 1 nor 2

Q31) Which of the following statements

isare correct about Amir Khusrau

1 Amir Khusrau records in his works

that Sanskrit did not belong to any

region and only the Brahmans

knew it

2 He recorded the existence of

Hindawi and Awadhi

Select the correct answer using the code

given below

(a) 1 only

(b) 2 only

(c) Both 1 and 2

(d) Neither 1 nor 2

RAUSIAS-FC19E1003 12

Q32) निमननिखित कथि ो पर निचार कीनजए

1 नहरणय-गभण अिषठाि क बार म ऐसा स चा जाता

था नक बनि दि िाि का एक कषनतरय क रप म

पिजणनम ह गा

2 मयरशमणि कदोब िोश का सोसथापक था

उपयणकत कथि ो म स कौि-सास सही हह

(a) किि 1

(b) किि 2

(c) 1 और 2 द ि ो

(d) ि त 1 ि ही 2

Q33) निमननिखित कथि ो म स कौि-सास सही हह

1 कदमई बगार (बिपिणक शरम) क रप म

निया जाि िािा कर था

2 गवानियर परशखसत म िागभट (ज एक चोदि

राजा था) क दवारा नकय गए श िर का िरणि

नकया गया ह

िीच नदए गए कट का परय ग कर सही उततर चनिए

(a) किि 1

(b) किि 2

(c) 1 और 2 द ि ो

(d) ि त 1 ि ही 2

Q34) निमननिखित कथि ो म स कौि-सास सही हह

1 राजतरो नगिी 11िी ो शताबदी म कलहि क दवारा

रनचत एक सोसकत पसतक (टकसट) ह

2 कननौज क निए नतरपकषीय सोघिण म पाि राजिोश

शानमि था

िीच नदए गए कट का परय ग कर सही उततर चनिए

(a) किि 1

(b) किि 2

(c) 1 और 2 द ि ो

(d) ि त 1 ि ही 2

Q35) निमननिखित यग ो पर निचार कीनजए

1 बरहदशवर मोनदर राजराजा च ि

2 उर मापि की इकाई

3 दिदाि मोनदर ो क भनम अिदाि

उपयणकत यग ो म स कौि-स सही समनित ह

(a) किि 1 और 2

(b) किि 2 और 3

(c) किि 1 और 3

(d) 1 2 और 3

Q36) निमननिखित कथि ो म स कौि-सास सही हह

1 नदलली क सलताि ो क अरीि परशासि की भािा

फारसी थी

2 नदलली सलतित म ldquoतारीितािरीिrdquo कनिता

का एक रप था

िीच नदए गए कट का परय ग कर सही उततर चनिए

(a) किि 1

(b) किि 2

(c) 1 और 2 द ि ो

(d) ि त 1 ि ही 2

Q37) निमननिखित कथि ो म स कौि-सास सही हह

1 अिाउददीि खििजी ि अपि सनिक ो क निए

नसरी िाम का एक िया दगणरकषक शहर

बिािाया था

2 िह अपि सनिक ो क िति का भगताि इकता

क रप म करता था

िीच नदए गए कट का परय ग कर सही उततर चनिए

(a) किि 1

(b) किि 2

(c) 1 और 2 द ि ो

(d) ि त 1 ि ही 2

RAUSIAS-FC19E1003 13

Q32) Consider the following statements

1 Hiranya-garbha ritual was thought

to lead to the rebirth of the

sacrificer as a Kshatriya

2 Mayurasharman was the founder

of the Kadamba dynasty

Which of the statements given above

isare correct

(a) 1 only

(b) 2 only

(c) Both 1 and 2

(d) Neither 1 nor 2

Q33) Which of the following statements

isare correct

1 Kadamai was tax taken in form of

forced labour

2 Gwalior Prashasti describes the

exploits of Nagabhata who was a

Chandella king

Select the correct answer using the code

given below

(a) 1 only

(b) 2 only

(c) Both 1 and 2

(d) Neither 1 nor 2

Q34) Which of the following statements

isare correct

1 Rajatarangini is a Sanskrit text

written by Kalhana in the 11th

century

2 Pala dynasty was included in the

tripartite struggle for Kannauj

Select the correct answer using the code

given below

(a) 1 only

(b) 2 only

(c) Both 1 and 2

(d) Neither 1 nor 2

Q35) Consider the following pairs

1 Brihadeshvara temple Rajaraja

Chola

2 ldquoUrrdquo Unit of measurement

3 Devadana Land grants made to

temples

Which of the pairs given above isare

correct

(a) 1 and 2 only

(b) 2 and 3 only

(c) 1 and 3 only

(d) 1 2 and 3

Q36) Which of the following statements

isare correct

1 The language of administration

under the Delhi Sultans was

Persian

2 Tarikhtawarikh was a form of

poetry in the Delhi Sultanate

Select the correct answer using the code

given below

(a) 1 only

(b) 2 only

(c) Both 1 and 2

(d) Neither 1 nor 2

Q37) Which of the following statements

isare correct

1 Alauddin Khilji constructed a new

garrison town named Siri for his

soldiers

2 He paid his soldiers their salaries

in the form of Iqta

Select the correct answer using the code

given below

(a) 1 only

(b) 2 only

(c) Both 1 and 2

(d) Neither 1 nor 2

RAUSIAS-FC19E1003 14

Q38) निमननिखित कथि ो म स कौि-सास सही हह

1 नदलली कतबददीि एबक क अरीि पहिी बार

नकसी सामराजय की राजरािी बिी थी

2 दहिीिाि नसक ो का मदरर मग़ि ो क दवारा

नकया गया था

िीच नदए गए कट का परय ग कर सही उततर चनिए

(a) किि 1

(b) किि 2

(c) 1 और 2 द ि ो

(d) ि त 1 ि ही 2

Q39) निमननिखित यग ो पर निचार कीनजए

1 म ठ की मखिद नसको दर ि दी

2 बगमपरी मखिद नफर ज शाह तगिक

3 कववत- अि - इसलाम कतबददीि ऐबक

उपयणकत यग ो म स कौि-स सही समनित ह

(a) किि 1 और 2

(b) किि 2 और 3

(c) किि 1 और 3

(d) 1 2 और 3

Q40) निमननिखित कथि ो म स कौि-सास सही हह

1 मिसबदार ो क अपिा िति राजसव कायो

नजन जागीर कहत थ क रप म परापत ह ता

था

2 मिसबदार क ज सनय उततरदानयतव सौोप जात

थ उसक अनतगणत उस एक निराणररत सखया म

सिार अथिा घड़सिार ो का रि-रिाि करिा

पड़ता था

िीच नदए गए कट का परय ग कर सही उततर चनिए

(a) किि 1

(b) किि 2

(c) 1 और 2 द ि ो

(d) ि त 1 ि ही 2

Q41) ldquo1942 क भारत छ ड़ आोद ििrdquo क बार म

निमननिखित अिि कि ो म स कौि-सा सतय िही ो ह

(a) यह एक अनहोसक आोद िि था

(b) इसका िततव महातमा गाोरी क दवारा नकया गया

था

(c) यह एक सवाभानिक आोद िि था

(d) इसि सामानयतया शरनमक िगण क आकनिणत

िही ो नकया था

Q42) भारत क ि ग ो ि ldquoसाइमि कमीशिrdquo क आगमि क

निरदध आोद िि नकया था कय ोनक

(a) भारतीय कभी भी 1919 क अनरनियम (The

Act of 1919) क काम की समीकषा िही ो करिा

चाहत थ

(b) साइमि कमीशि ि पराोत ो म दवर (द हर) शासि

क समापत करि की नसफाररश की थी

(c) साइमि कमीशि म क ई भारतीय सदसय िही ो

था

(d) साइमि कमीशि ि दश क निभाजि का

सझाि नदया था

Q43) निमननिखित कथि ो पर निचार कीनजए

भारतीय राषटर ीय आोद िि म दादाभाई िौर जी क दवारा

नकया गया सबस परभािी य गदाि यह था नक उन ोि

1 अोगरज ो क दवारा भारत क आनथणक श िर का

ििासा नकया था

2 पराचीि भारतीय गरोथ ो की वयाखया की थी और

भारतीय ो क आतमनिशवास क पिःसथानपत नकया

था

3 अनय नकसी भी बात स पहि सभी सामानजक

बराइय ो क उनमिि की आिशयकता पर बि

नदया था

उपयणकत कथि ो म स कौि-सास सही हह

(a) किि 1

(b) किि 2 और 3

(c) किि 1 और 3

(d) 1 2 और 3

RAUSIAS-FC19E1003 15

Q38) Which of the following statements

isare correct

1 Delhi first became the capital of a

kingdom under Qutubuddin

Aibak

2 Dehliwal coins were minted by the

Mughals

Select the correct answer using the code

given below

(a) 1 only

(b) 2 only

(c) Both 1 and 2

(d) Neither 1 nor 2

Q39) Consider the following pairs

1 Moth ki Masjid- Sikander Lodi

2 Begumpuri mosque- Firuz Shah

Tughluq

3 Quwwat al ndash Islam- Qutubuddin

Aibak

Which of the above pairs isare correct

(a) 1 and 2 only

(b) 2 and 3 only

(c) 1 and 3 only

(d) 1 2 and 3

Q40) Which of the following statements

isare correct

1 Mansabdars received their salaries

as revenue assignments called

jagirs

2 The mansabdarrsquos military

responsibilities required him to

maintain a specified number of

sawar or cavalrymen

Select the correct answer using the code

given below

(a) 1 only

(b) 2 only

(c) Both 1 and 2

(d) Neither 1 nor 2

Q41) Which one of the following observations

is not true about the Quit India

Movement of 1942

(a) It was a non-violent movement

(b) It was led by Mahatma Gandhi

(c) It was a spontaneous movement

(d) It did not attract the labour class

in general

Q42) The people of India agitated against the

arrival of the Simon Commission

because

(a) Indians never wanted the review of

the working of the Act of 1919

(b) Simon Commission recommended

the abolition of dyarchy in the

Provinces

(c) there was no Indian member in the

Simon Commission

(d) the Simon Commission suggested

the partition of the country

Q43) Consider the following statements

The most effective contribution made by

Dadabhai Naoroji to the cause of Indian

National Movement was that he-

1 exposed the economic exploitation

of India by the British

2 interpreted the ancient Indian

texts and restored the self-

confidence of Indians

3 stressed the need for eradication of

all the social evils before anything

else

Which of the statements given above

isare correct

(a) 1 only

(b) 2 and 3 only

(c) 1 and 3 only

(d) 1 2 and 3

RAUSIAS-FC19E1003 16

Q44) महातमा गाोरी ि 1932 म आमरर अिशि नकया था

कय ोनक

(a) ldquoग िमज सममििrdquo (The Round Table

Conference) भारतीय राजिीनतक

आकाोकषाओो क परा करि म असफि रहा था

(b) काोगरस और मखसलम िीग म मतभद थ

(c) रामस मकड िालड (Ramsay Macdonald)

ि ldquoसाोपरदानयक परसकारrdquo (The Communal

Award) की घ िरा की थी

(d) ldquoसनििय अिजञा आोद ििrdquo (The Civil

Disobedience Movement) असफि रहा

था

Q45) भारत म औपनििनशक शासि की अिनर क सोदभण म

भारत स रि क बनहगणमि का एक महतवपरण भाग गह

शलक (Home Charges) था निमननिखित म स

कौि-सास क ि गह शलक म सखममनित नकया गया

थानकय गए थ

1 िोदि म भारत कायाणिय क निए उपय ग नकय

जाि िािा क ि

2 भारत म नियकत नबरनटश कनमणय ो क िति और

पशि का भगताि करि क निए उपय ग नकय

जाि िािा क ि

3 अोगरज ो क दवारा भारत क बाहर यदध ो क निए

उपय ग नकय जाि िािा क ि

िीच नदए गए कट का परय ग कर सही उततर चनिए

(a) किि 1

(b) किि 1 और 2

(c) किि 2 और 3

(d) 1 2 और 3

Q46) सवतोतरता आोद िि क इनतहास म भारतीय राषटर ीय

काोगरस का 1929 का सतर महतवपरण ह कय ोनक इसम

(a) काोगरस क उददशय क रप म सथािीय सरकार

की पराखपत की घ िरा की गई थी

(b) परण सवराज की पराखपत क काोगरस क िकषय क

रप म अपिाया गया था

(c) असहय ग आोद िि शर नकया गया था

(d) िोदि म ldquoग ि मर सममििrdquo (The Round

Table Conference) म भाग िि का निरणय

निया गया था

Q47) भारतीय सवतोतरता सोगराम क सोदभण म िहर ररप टण

क दवारा निमननिखित म स नकसकी नसफाररश की गई

थीनकिकी नसफाररश की गई थी ो

1 भारत क निए परण सवतोतरता

2 अलपसोखयक ो क निए सीट ो क आरकषर क

निए सोयकत नििाणचक मोडि

3 सोनिराि म भारत क ि ग ो क निए मौनिक

अनरकार ो का परािराि

िीच नदए गए कट का परय ग कर सही उततर चनिए

(a) किि 1

(b) किि 2 और 3

(c) किि 1 और 3

(d) 1 2 और 3

Q48) आरो नभक िनदक आयो का रमण मखय रप स था

(a) भखकत

(b) मनतण पजा और यजञ

(c) परकनत की पजा और यजञ

(d) परकनत की पजा और भखकत

RAUSIAS-FC19E1003 17

Q44) Mahatma Gandhi undertook fast unto

death in 1932 mainly because

(a) The Round Table Conference failed

to satisfy Indian political

aspirations

(b) The Congress and Muslim League

had differences of opinion

(c) Ramsay Macdonald announced the

Communal Award

(d) The Civil Disobedience Movement

failed

Q45) With reference to the period of colonial

rule in India ldquoHome Chargesrdquo formed

an important part of drain of wealth

from India Which of the following funds

constituted ldquoHome Chargesrdquo

1 Funds used to support the India

Office in London

2 Funds used to pay salaries and

pensions of British personnel

engaged in India

3 Funds used for waging wars

outside India by the British

Select the correct answer using the code

given below

(a) 1 only

(b) 1 and 2 only

(c) 2 and 3 only

(d) 1 2 and 3

Q46) The 1929- Session of Indian National

Congress is of significance in the history

of the Freedom Movement because the-

(a) attainment of Self-Government

was declared as the objective of

the Congress

(b) attainment of Poorna Swaraj was

adopted as the goal of the

Congress

(c) Non-Cooperation Movement was

launched

(d) decision to participate in the

Round Table Conference in

London was taken

Q47) With reference to the period of Indian

freedom struggle which of the following

waswere recommended by the Nehru

report

1 Complete Independence for India

2 Joint electorates for reservation of

seats for minorities

3 Provision of fundamental rights for

the people of India in the

Constitution

Select the correct answer using the code

given below

(a) 1 only

(b) 2 and 3 only

(c) 1 and 3 only

(d) 1 2 and 3

Q48) The religion of the early Vedic Aryans was primarily of

(a) Bhakti

(b) image worship and Yajnas

(c) worship of nature and Yajnas

(d) worship of nature and Bhakti

RAUSIAS-FC19E1003 18

Q49) भारत की यातरा करि िाि चीिी यातरी यआि चिाोग

(हयएि साोग) ि समकािीि भारत की सामानय

खसथनतय ो और सोसकनत क दजण नकया था इस सोदभण म

निमननिखित कथि ो म स कौि-सास सही हह

1 सड़क और िदी-मागण (जि-मागण) डकती स

परण रप स सरनकषत थ

2 जहा तक अपरार ो क निए दणड की बात ह

उसक निए नकसी भी वयखकत की निदोिता

अथिा उसक अपरार क निराणररत करि क

निए अनि जि और निि परि क माधयम क

सारि थ

3 वयापाररय ो क घाट ो और परनतबोर सटशि ो पर

शलक ो का भगताि करिा पड़ता था

िीच नदए गए कट का परय ग कर सही उततर चनिए

(a) किि 1

(b) किि 2 और 3

(c) किि 1 और 3

(d) 1 2 और 3

Q50) नसोर घाटी सभयता क सोदभण म निमननिखित कथि ो पर

निचार कीनजए

1 यह मखय रप स एक रमणनिरपकष सभयता थी

तथा हािाोनक इसम रानमणक ततव मौजद था

िनकि िह परनतिश पर हािी िही ो था

2 इस काि क दौराि भारत म कपास का परय ग

कपड़ा बिाि क निए नकया जाता था

उपयणकत कथि ो म स कौि-सास सही हह

(a) किि 1

(b) किि 2

(c) 1 और 2 द ि ो

(d) ि त 1 ि ही 2

Q51) परोदर दास क सोदभण म निमननिखित कथि ो पर निचार

कीनजए

1 परोदर दास एक सोत और भगिाि नशि क एक

महाि भकत थ

2 ि एक सोगीतकार गायक और किाणटक सोगीत

क मखय सोसथापक-परसतािक ो म स एक थ

उपयणकत कथि ो म स कौि-सास सही हह

(a) किि 1

(b) किि 2

(c) 1 और 2 द ि ो

(d) ि त 1 ि ही 2

Q52) निमननिखित म स कौि-सास वयखकत किाणटक सोगीत

की नतरमनतण म शानमि हह

1 बािामरिी कषणा

2 शरी शयाम शासतरी

3 शरी मथसवामी दीनकषतर

िीच नदए गए कट का परय ग कर सही उततर चनिए

(a) किि 1

(b) किि 2

(c) किि 2 और 3

(d) 1 2 और 3

Q53) चियर (Chevayur) और अथ िी (Atholi) म खसथत

महापािार सथि निमननिखित म स नकस राजय म खसथत

(a) तनमििाड

(b) किाणटक

(c) पनिम बोगाि

(d) करि

RAUSIAS-FC19E1003 19

Q49) The Chinese traveller Yuan Chwang

(Hiuen Tsang) who visited India

recorded the general conditions and

culture of India at that time In this

context which of the following

statements isare correct

1 The roads and river-routes were

completely immune from robbery

2 As regards punishment for

offences ordeals by fire water and

poison were the instruments for

determining the innocence or guilt

of a person

3 The tradesmen had to pay duties

at ferries and barrier stations

Select the correct answer using the code

given below

(a) 1 only

(b) 2 and 3 only

(c) 1 and 3 only

(d) 1 2 and 3

Q50) Regarding the Indus Valley Civilization

consider the following statements

1 It was predominantly a secular

civilization and the religious

element though present did not

dominate the scene

2 During this period cotton was

used for manufacturing textiles in

India

Which of the statements given above

isare correct

(a) 1 only

(b) 2 only

(c) Both 1 and 2

(d) Neither 1 nor 2

Q51) Consider the following statements

regarding Purandara Dasa

1 Purandara Dasa was a saint and

great devotee of Lord Shiva

2 He was a composer singer and

one of the chief founding-

proponents of the Carnatic music

Which of the statements given above

isare correct

(a) 1 only

(b) 2 only

(c) Both 1 and 2

(d) Neither 1 nor 2

Q52) Which of the following persons isare

included in the trinity of Carnatic

music

1 Balamurali Krishna

2 Sri Shyama Shastry

3 Sri Muthuswami Dikshitar

Select the correct answer using the code

given below

(a) 1 only

(b) 2 only

(c) 2 and 3 only

(d) 1 2 and 3

Q53) Megalithic sites at Chevayur and Atholi

are located in which of the following

states

(a) Tamil Nadu

(b) Karnataka

(c) West Bengal

(d) Kerala

RAUSIAS-FC19E1003 20

Q54) निमननिखित कथि ो पर निचार कीनजए

1 महापािानरक ि ग कबर ो म िसतएो दफिात थ

2 दनकषर भारत म महापािार सोसकनत एक परण

निकनसत तामर यगीि सोसकनत थी

उपयणकत कथि ो म स कौि-सास सही हह

(a) किि 1

(b) किि 2

(c) 1 और 2 द ि ो

(d) ि त 1 ि ही 2

Q55) निमननिखित म स कौि-स सामराजयसामराजय ो का

अश क क अनभिि ो म उललि नकया गया ह

1 च ि

2 पाणडय

3 करिपतर (चर)

िीच नदए गए कट का परय ग कर सही उततर चनिए

(a) किि 1

(b) किि 1 और 2

(c) किि 3

(d) 1 2 और 3

Q56) भीमा-क रगाोि का यदध को पिी क सनिक ो और

बाजीराि नदवतीय क िततव म एक शखकतशािी पशिा

सिा (मराठ ो) क मधय िड़ा गया था यह यदध

निमननिखित म स नकसका नहससा था

(a) परथम आोगल-मराठा यदध का

(b) नदवतीय आोगल-मराठा यदध का

(c) ततीय आोगल-मसर यदध का

(d) ततीय आोगल-मराठा यदध का

Q57) निमननिखित कथि ो पर निचार कीनजए

1 महादि दसाई ि गाोरीजी क चोपारर आि तथा

नतिकनथया पररािी स जड़ी समसया की जाोच

क निए रारी करि क निए दश भर म उिका

अिसरर नकया था

2 िरहरी पाररि चोपारर सतयागरह क दौराि

गाोरीजी क साथ थ

उपयणकत कथि ो म स कौि-सास सही हह

(a) किि 1

(b) किि 2

(c) 1 और 2 द ि ो

(d) ि त 1 ि ही 2

Q58) निमननिखित कथि ो पर निचार कीनजए

1 िनद राज-िोश ि बराहमर ो और बौदध मठराररय ो

क कर-मकत गाि अिदाि म दि की परथा

आरि की थी

2 सतिाहि ो की आनरकाररक भािा पराकत थी

उपयणकत कथि ो म स कौि-सास सही हह

(a) किि 1

(b) किि 2

(c) 1 और 2 द ि ो

(d) ि त 1 ि ही 2

Q59) एक निरासत क अपिाइए (अडॉपट ए हररटज ndash

Adopt a Heritage) पररय जिा क उददशय ो क

सनदभण म निमननिखित कथि ो पर निचार कीनजए

1 यह पररय जिा र रगार उतपादि और आनथणक

निकास क निए पयणटि कषमता का उि पर

परभाि का उपय ग करगी

2 यह पररय जिा निरासत सथि ो पर निशव सतरीय

आराररक सोरचिा निकनसत करक एक सतत

तरीक स पयणटक आकिणर म िखदध करगी

उपयणकत कथि ो म स कौि-सास सही हह

(a) किि 1

(b) किि 2

(c) 1 और 2 द ि ो

(d) ि त 1 ि ही 2

RAUSIAS-FC19E1003 21

Q54) Consider the following statements

1 Megalithic people buried goods in

graves

2 The megalithic culture in South

India was a full-fledged Copper

Age culture

Which of the statements given above

isare correct

(a) 1 only

(b) 2 only

(c) Both 1 and 2

(d) Neither 1 nor 2

Q55) Which of the following kingdoms isare

mentioned in the Ashokan inscriptions

1 Cholas

2 Pandyas

3 Keralaputras (Cheras)

Select the correct answer using the code

given below

(a) 1 only

(b) 1 and 2 only

(c) 3 only

(d) 1 2 and 3

Q56) The Battle of Bhima-Koregaon was

fought between the soldiers of the

Company and the strong Peshwa army

(Marathas) under Bajirao II This war

was a part of the

(a) First Anglo-Maratha war

(b) Second Anglo-Maratha war

(c) Third Anglo- Mysore war

(d) Third Anglo-Maratha war

Q57) Consider the following statements

1 Mahadev Desai followed Gandhiji all over the country to persuade him to come to Champaran to investigate the problem associated

with tinkathia system

2 Narhari Parikh accompanied Gandhi ji during the Champaran

Satyagraha

Which of the statements given above isare correct

(a) 1 only

(b) 2 only

(c) Both 1 and 2

(d) Neither 1 nor 2

Q58) Consider the following statements

1 The Nanda Dynasty started the practice of granting tax-free villages to brahmanas and

Buddhist monks

2 The official language of the Satavahanas was Prakrit

Which of the statements given above

isare correct

(a) 1 only

(b) 2 only

(c) Both 1 and 2

(d) Neither 1 nor 2

Q59) Consider the following statements about the objectives of the lsquoadopt a heritagersquo

project

1 It will harness tourism potential for its effects on employment generation and economic

development

2 It will enhance the tourist attractiveness in a sustainable manner by developing world class infrastructure at heritage sites

Which of the statements given above

isare correct

(a) 1 only

(b) 2 only

(c) Both 1 and 2

(d) Neither 1 nor 2

RAUSIAS-FC19E1003 22

Q60) ldquoभारतीय जिजातीय सहकारी निपरि निकास सोघrdquo

(The Tribal Co-operative Marketing

Development Federation of India - TRIFED)

क सोदभण म निमननिखित कथि ो पर निचार कीनजए

1 यह एक राषटर ीय सतर का शीिण सोगठि ह ज

भारत सरकार क गह मोतरािय क परशासनिक

नियोतरर क अरीि काम कर रहा ह

2 इसका मखय उददशय दश म जिजातीय ि ग ो

का सामानजक-आनथणक निकास करिा ह

उपयणकत कथि ो म स कौि-सास सही हह

(a) किि 1

(b) किि 2

(c) 1 और 2 द ि ो

(d) ि त 1 ि ही 2

Q61) निमननिखित म स कौि-सास उपनयास परमचोद क

दवारा नििा गया हनिि गए ह

1 रोगभनम

2 ग दाि

3 ग रा

िीच नदए गए कट का परय ग कर सही उततर चनिए

(a) किि 1

(b) किि 2

(c) किि 1 और 2

(d) 1 2 और 3

Q62) नगदधा ितय क सोदभण म निमननिखित कथि ो पर निचार

कीनजए

1 नगदधा नबहार की मनहिाओो क दवारा तयौहार क

समय और फसि की बिाई तथा कटाई क

अिसर पर नकया जाि िािा एक पारोपररक

दहाती ितय ह

2 इस ितय क दवारा मनहिाऐो अपिी परसननता

परकट करती ह तथा नगदधा क परदशणि क

माधयम स परि िचणसव िाि समाज म

मनहिाओो की दबी हई भाििाओो क परकट

करती ह

उपयणकत कथि ो म स कौि-सास सही हह

(a) किि 1

(b) किि 2

(c) 1 और 2 द ि ो

(d) ि त 1 ि ही 2

Q63) निमननिखित कथि ो पर निचार कीनजए

1 मलला शाह बदखशी दारा नशक ह क

आधयाखतमक गर थ

2 औरोगरब ि मजम-उि-बहरीि या द समदर ो

का सोगम िामक उललििीय रचिा नििी थी

3 दारा नशक ह क अपि पिणज अकबर क गर ो

क उततरानरकारी क रप म दिा गया था

नजसम उसि रानमणक बहििाद और समनवयता

क बढ़ािा नदया था

उपयणकत कथि ो म स कौि-सास सही हह

(a) किि 1 और 3

(b) किि 2

(c) किि 1 और 2

(d) 1 2 और 3

RAUSIAS-FC19E1003 23

Q60) Consider the following statements about

the Tribal Cooperative Marketing

Development Federation of India

(TRIFED)

1 It is a national-level apex

organization functioning under the

administrative control of Ministry

of Home Affairs Government of

India

2 The main objective of TRIFED is

socio-economic development of

tribal people in the country

Which of the statements given above

isare correct

(a) 1 only

(b) 2 only

(c) Both 1 and 2

(d) Neither 1 nor 2

Q61) Which of the following novels isare

written by Premchand

1 Rangabhumi

2 Godan

3 Gora

Select the correct answer using the code

given below

(a) 1 only

(b) 2 only

(c) 1 and 2 only

(d) 1 2 and 3

Q62) Consider the following statements about

Giddha dance

1 Giddha is a traditional pastoral

dance performed by the women of

Bihar at festival times and at the

sowing and reaping of the harvest

2 By this dance the women reveal

their joy expel their suppressed

feelings in a male dominated

society through the performance of

Giddha

Which of the statements given above

isare correct

(a) 1 only

(b) 2 only

(c) Both 1 and 2

(d) Neither 1 nor 2

Q63) Consider the following statements

1 Mullah Shah Badakhshi was the

spiritual mentor of Dara Shukoh

2 Aurangzeb wrote the remarkable

work called ldquoMajma-ul-Bahrainrdquo or

the ldquoThe confluence of two seasrdquo

3 Dara Shukoh was seen as

inheriting the qualities of his

ancestor Akbar in that he

promoted religious pluralism and

syncretism

Which of the statements given above

isare correct

(a) 1 and 3 only

(b) 2 only

(c) 1 and 2 only

(d) 1 2 and 3

RAUSIAS-FC19E1003 24

Q64) निमननिखित कथि ो पर निचार कीनजए

1 ग मतशवर परनतमा निोधयनगरी पहाड़ी पर खसथत ह

2 शरिरबिग िा िह सथाि ह जहाो मौयण िोश क

सोसथापक चोदरगपत मौयण अपि नसोहासि क

तयागि क बाद जि तपसवी बि गए थ

उपयणकत कथि ो म स कौि-सास सही हह

(a) किि 1

(b) किि 2

(c) 1 और 2 द ि ो

(d) ि त 1 ि ही 2

Q65) निमननिखित कथि ो पर निचार कीनजए

1 पराताखतवक साकषय स पता चिता ह नक पराची

घाटी सभयता हड़पपा और म हिज दाड़ द ि ो

की पिणिती ह

2 पराची िदी भििशवर स निकिती ह

उपयणकत कथि ो म स कौि-सास सही हह

(a) किि 1

(b) किि 2

(c) 1 और 2 द ि ो

(d) ि त 1 ि ही 2

Q66) निमननिखित कथि ो म स कौि-सास सही हह

1 िजराह क समारक ो क समह का निमाणर

चोदि राजिोश क शासिकाि क दौराि हआ

था

2 य समारक हररिोदर पिणत शरोििा म खसथत ह

3 म रक क यातरी इबन बतता ि अपि सोसमरर ो

म िजराह क मोनदर ो की यातरा का उललि

नकया था तथा इन काजराण िाम स समब नरत

नकया था

िीच नदए गए कट का परय ग कर सही उततर चनिए

(a) किि 1

(b) किि 1 और 2

(c) किि 2 और 3

(d) किि 1 और 3

Q67) निमननिखित कथि ो म स कौि-सास सही हह

1 डॉ बी आर अमबडकर ि दी एनिनहिशि

ऑफ़ कासट (The Annihilation of Caste)

नििी थी नजसम उन ोि नहोद रमण म िोशािगत

पजारी की परथा क उनमिि की आिशयकता

पर बि नदया था

2 डॉ राजदर परसाद ि थॉटस ऑि पानकसताि

(Thoughts on Pakistan) िामक पसतक

नििी थी

िीच नदए गए कट का परय ग कर सही उततर चनिए

(a) किि 1

(b) किि 2

(c) 1 और 2 द ि ो

(d) ि त 1 ि ही 2

Q68) निमननिखित कथि ो म स कौि-सास सही हह

1 महरगढ़ भारतीय उपमहादवीप म एक परनसदध

ििपािार बसती ह ज नसोर पराोत पानकसताि म

खसथत ह

2 बरणह म म कतत ो क उिक सवामी क साथ कबर ो

म दफिाया जाता था

िीच नदए गए कट का परय ग कर सही उततर चनिए

(a) किि 1

(b) किि 2

(c) 1 और 2 द ि ो

(d) ि त 1 ि ही 2

Q69) निमननिखित कथि ो म स कौि-सास सही हह

1 काकानटय मोनदर अनरकतर नशि क समनपणत

2 हिमक ोडा म हजार-सतोभ िाि मोनदर (The

Thousand-Pillared Temple) का निमाणर

काकानटय समराट रदर ि करिाया था

िीच नदए गए कट का परय ग कर सही उततर चनिए

(a) किि 1

(b) किि 2

(c) 1 और 2 द ि ो

(d) ि त 1 ि ही 2

RAUSIAS-FC19E1003 25

Q64) Consider the following statements

1 Gommateshwara Statue is located

on the Vindyagiri Hill

2 Shravanabelagola is the place

where Chandragupta Maurya the

founder of the Mauryan dynasty

became a Jain ascetic after

relinquishing his throne

Which of the statements given above

isare correct

(a) 1 only

(b) 2 only

(c) Both 1 and 2

(d) Neither 1 nor 2

Q65) Consider the following statements

1 Archaeological evidence shows

that the Prachi Valley Civilisation

predates both Harappa and

Mohenjo-Daro

2 The Prachi river originates from

Bhubaneswar

Which of the statements given above

isare correct

(a) 1 only

(b) 2 only

(c) Both 1 and 2

(d) Neither 1 nor 2

Q66) Which of the following statements

isare correct

1 The Khajuraho group of

monuments was built during the

rule of the Chandela dynasty

2 These monuments are located in

Harischandra mountain range

3 Ibn Battuta the Moroccan

traveller in his memoirs mentioned

visiting Khajuraho temples and

called them Kajarra

Select the correct answer using the code

given below

(a) 1 only

(b) 1 and 2

(c) 2 and 3

(d) 1 and 3

Q67) Which of the following statements

isare correct

1 Dr BR Ambedkar wrote the

Annihilation of Caste emphasising

the need to do away with the

practice of hereditary priesthood in

Hinduism

2 The book lsquoThoughts on Pakistanrsquo

was written by Dr Rajendra

Prasad

Select the correct answer using the code

given below

(a) 1 only

(b) 2 only

(c) Both 1 and 2

(d) Neither 1 nor 2

Q68) Which of the following statements

isare correct

1 Mehrgarh is a famous Neolithic

settlement in the Indian

subcontinent which is situated in

Sindh province Pakistan

2 At Burzahom dogs were buried

with their masters in their graves

Select the correct answer using the code

given below

(a) 1 only

(b) 2 only

(c) Both 1 and 2

(d) Neither 1 nor 2

Q69) Which of the following statements

isare correct

1 The Kakatiya temples are

dedicated mostly to Siva

2 The Thousand-Pillared Temple at

Hanamkonda was built by the

Kakatiya king Rudra

Select the correct answer using the code

given below

(a) 1 only

(b) 2 only

(c) Both 1 and 2

(d) Neither 1 nor 2

RAUSIAS-FC19E1003 26

Q70) निमननिखित कथि ो म स कौि-सास सही हह

1 अहमदाबाद नमि हड़ताि क दौराि महातमा

गाोरी ि शरनमक ो क पकष क मजबत करि क

निए आमरर अिशि नकया था

2 अिशि स नमि मानिक ो पर दबाि पड़ा था ज

अोततः शरनमक ो क िति म 15 परनतशत की िखदध

करि क निए सहमत हए थ

िीच नदए गए कट का परय ग कर सही उततर चनिए

(a) किि 1

(b) किि 2

(c) 1 और 2 द ि ो

(d) ि त 1 ि ही 2

Q71) निमननिखित म स नकसक नकिक भारत स यिसक

की माििता की अमतण साोसकनतक निरासत की

परनतनिनर सची (The UNESCOrsquos List of the

Representative List of the Intangible

Cultural Heritage of Humanity) म शानमि

नकया गया ह

1 मनडयटट

2 सोकीतणि

3 को भ मिा

िीच नदए गए कट का परय ग कर सही उततर चनिए

(a) किि 1 और 2

(b) किि 2 और 3

(c) किि 3

(d) 1 2 और 3

Q72) निमननिखित जिजानतय ो म स कौि-सीसी ो

जिजानतजिजानतया िागािड स सोबोनरत हह

1 अोगामी

2 ककी

3 जारिा

िीच नदए गए कट का परय ग कर सही उततर चनिए

(a) किि 1

(b) किि 1 औऔ 2

(c) किि 2

(d) 1 2 और 3

Q73) निमननिखित कथि ो म स कौि-सास सही हह

1 राषटर कट सामराजय की सथापिा दोनतदगण ि की थी

नजसि मानयाित म अपिी राजरािी की

सथापिा की थी

2 राषटर कट समराट अम घििण एक ििक था और

उस कनिताओो पर पहिी कननड़ पसतक नििि

का शरय नदया जाता ह

िीच नदए गए कट का परय ग कर सही उततर चनिए

(a) किि 1

(b) किि 2

(c) 1 और 2 द ि ो

(d) ि त 1 ि ही 2

Q74) निमननिखित कथि ो म स कौि-सास सही हह

1 कशब चोदर सि ि ततवब नरिी सभा की

अधयकषता की थी ज आधयाखतमक सतय की

ि ज म सोिि थी

2 बरहम समाज ि मािि गररमा पर बि नदया

मनतणपजा का निर र नकया और सती परथा जसी

सामानजक बराइय ो की आि चिा की

िीच नदए गए कट का परय ग कर सही उततर चनिए

(a) किि 1

(b) किि 2

(c) 1 और 2 द ि ो

(d) ि त 1 ि ही 2

Q75) निमननिखित कथि ो म स कौि-सास सही हह

1 भारत म नचशती नसिनसिा खवाजा म इिददीि

नचशती क दवारा सथानपत नकया गया था

2 नचशती परोपरा की एक परमि निशिता

आतमसोयम थी नजसम साोसाररक म ह स दरी

बिाए रििा शानमि था

िीच नदए गए कट का परय ग कर सही उततर चनिए

(a) किि 1

(b) किि 2

(c) 1 और 2 द ि ो

(d) ि त 1 ि ही 2

RAUSIAS-FC19E1003 27

Q70) Which of the following statements

isare correct

1 During the Ahmedabad Mill Strike

Mahatma Gandhi undertook a fast

unto death to strengthen the

workersrsquo resolve

2 The fast had effect of putting

pressure on mill owners who

finally agreed to give the workers a

15 per cent increase in wages

Select the correct answer using the code

given below

(a) 1 only

(b) 2 only

(c) Both 1 and 2

(d) Neither 1 nor 2

Q71) Which of the following are included in

the UNESCOrsquos list of the representative

list of the intangible cultural heritage of

humanity from India

1 Mudiyettu

2 Sankirtana

3 Kumbh Mela

Select the correct answer using the code

given below

(a) 1 and 2 only

(b) 2 and 3 only

(c) 3 only

(d) 1 2 and 3

Q72) Which of the following tribes isare

related to Nagaland

1 Angami

2 Kuki

3 Jarawa

Select the correct answer using the code

given below

(a) 1 only

(b) 1 and 2 only

(c) 2 only

(d) 1 2 and 3

Q73) Which of the following statements

isare correct

1 Rashtrakuta kingdom was founded by Dantidurga who established his capital at Manyakhet

2 Amoghavarsha a Rashtrakuta king was an author and is credited with writing the first

Kannada book on poetics

Select the correct answer using the code given below

(a) 1 only

(b) 2 only

(c) Both 1 and 2

(d) Neither 1 nor 2

Q74) Which of the following statements isare correct

1 Keshab Chandra Sen headed the Tattvabodhini Sabha which was engaged in search of spiritual truth

2 The Brahmo Samaj laid emphasis on human dignity opposed idolatry and criticized such social

evils as the practice of Sati

Select the correct answer using the code given below

(a) 1 only

(b) 2 only

(c) Both 1 and 2

(d) Neither 1 nor 2

Q75) Which of the following statements isare correct

1 The Chishti order was established in India by Khwaja Moinuddin

Chishti

2 A major feature of the Chishti tradition was austerity including maintaining a distance from the

worldly power

Select the correct answer using the code

given below

(a) 1 only

(b) 2 only

(c) Both 1 and 2

(d) Neither 1 nor 2

T e s t i s p a r t o f R a u rsquo s I A S T e s t s e r i e s f o r P r e l i m i n a r y E x a m 2 0 1 9

FOUNDATION + CURRENT AFFAIRS

GENERAL STUDIES (PAPER ndashI)

FOUNDATION TEST ndashIII

SUBJECT NCERT History Class VI-X + Current Affairs

Time Allowed 1frac12 Hours Maximum Marks 150

I NSTRUCT IONS

1 IMMEDIATELY AFTER THE COMMENCEMENT OF THE EXAMINATION YOU SHOULD CHECK

THAT THIS TEST BOOKLET DOES NOT HAVE ANY UNPRINTED OR TORN or MISSING PAGES OR

ITEMS ETC IF SO GET IT REPLACED BY A COMPLETE TEST BOOKLET

2 This Test Booklet contains 75 items (questions) Each item is printed both in Hindi and English

Each item comprises four responses (answers) You will select the response which you want to mark

on the Answer Sheet In case you feel that there is more than one correct response mark the

response which you consider the best In any case choose ONLY ONE response for each item

3 You have to mark all your responses ONLY on the separate Answer Sheet (OMR sheet) provided

Read the directions in the Answer Sheet

4 All items carry equal marks

5 Before you proceed to mark in the Answer Sheet the response to various items in the Test booklet

you have to fill in some particulars in the Answer Sheet as per instructions contained therein

6 After you have completed filling in all your responses on the Answer Sheet and the examination has

concluded you should hand over to the Invigilator only the Answer Sheet You are permitted to

take away with you the Test Booklet

7 Penalty for wrong answers

THERE WILL BE PENALTY FOR WRONG ANSWERS MARKED BY A CANDIDATE IN THE

OBJECTIVE TYPE QUESTION PAPERS

(i) There are four alternatives for the answer to every question For each question for which a

wrong answer has been given by the candidate one-third of the marks assigned to that

question will be deducted as penalty

(ii) If a candidate gives more than one answer it will be treated as a wrong answer even if one of

the given answers happens to be correct and there will be same penalty as above to that

question

(iii) If a question is left blank ie no answer is given by the candidate there will be no penalty for

that question

T h i s t e s t i s p a r t o f R a u rsquo s I A S T e s t s e r i e s f o r P r e l i m i n a r y E x a m 2 0 1 9

Test Code

FC19E1003

FC19H1003 29

Answers and Explanations of

NCERT History Class VI-X + Current Affairs (FC19E1003)

Q1) उततर (c)

सपषटीकरण

- ऋगवद म दविय ो और दिताओो क समवपित एक

हजार स अविक सत तर (शल क) ह

- य शल क ऋविय ो क दवारा रच गए थ और परि ो

दवारा सीख जात थ

- हालाोवक कछ शल क मवहलाओो (जस वक अपाला

घ सा ल पामदरा मतरयी और गागी) क दवारा भी रच

गए थ

- ऋगवद म सोिाद क रप म कई शल क मौजद ह

- हम विशवावमतर नामक एक ऋवि और दविय ो क

रप म पजी जान िाली द नवदय ो (वयास और

सतलज) क बीच िाताि का उदाहरण वमलता ह

- इसस पता चलता ह वक विशवावमतर िवदक काल स

सोबोवित थ

Q2) उततर (b)

सपषटीकरण

- करनल गफाओो स राख क अिशि परापत हए ह

ज इस ओर सोकत करत ह वक ततकालीन ल ग

अवि क उपय ग स पररवचत थ

- य गफाएो आोधर परदश म सथथत ह

Q3) उततर (c)

सपषटीकरण

bull बरािह म ितिमान कशमीर म सथथत एक

परागवतहावसक थथल ह जहाो ल ग गडढ क घर ो का

वनमािण करत थ

bull य घर जमीन क ख द कर बनाए जात थ तथा नीच

जान क वलए सीवियाा ह ती थी

bull ऐसा अनमान लगाया जाता ह वक य घर ठो ड क

मौसम म आशरय परदान करत थ

Q4) उततर (c)

सपषटीकरण

bull परालख-विदया (Epigraphy) क वशलालख ो क

अधययन क रप म पररभावित वकया जाता ह

bull हसतवलसखत दसतािज ो क माधयम स इवतहास

और सावहतय क अधययन क पाोडवलवप विजञान

(Manuscriptology) कहत ह

bull पराचीन लखन परणावलय ो क अधययन और

ऐवतहावसक पाोडवलवपय ो क समझन तथा वतवथ

वनिािरण क पलीओगराफी (Palaeography) कहा

जाता ह

bull नयवमजमविकस (Numismatics) वसक ो क

अधययन क सोदवभित करता ह

Q5) उततर (a)

सपषटीकरण

- चरक सोवहता चरक क दवारा वलखी गई आयिद

और िदयक-शासर पर एक महतवपणि पसतक ह

- ि भारतीय िदयक-शासर की पारमपररक परणाली

वजस आयिद क नाम स जाना जाता ह क

अभयासकताि थ

- ऐसा माना जाता ह वक चरक का विकास दसरी

शताबदी (ईसा पिि) और दसरी शताबदी (ईसवी) क

मधय हआ था

Q6) उततर (b)

सपषटीकरण

- भाग फसल ो पर वलए जान िाल कर क सोदवभित

करता ह ज कल फसल उतपादन का 16 िाो भाग

था

- ldquoकममकारrdquo शबद भवमहीन कवि शरवमक िगि क

वलए परय ग वकया जाता था

- ldquoअशवमिrdquo (वजस घ ड क बवलदान क रप म भी

जाना जाता ह) एक अनषठान ह ता था वजसम एक

घ ड क सवतोतर रप स घमन क वलए छ ड वदया

FC19H1003 30

जाता ह और राजा क सवनक उसकी रखिाली

करत थ

Q7) उततर (d)

सपषटीकरण

- ऋगववदक काल म घ ड ो क रथ ो म ज ता जाता था

ज (रथ) भवम मिवशय ो आवद पर कबजा करन क

वलए लड गए यद ो म उपय ग वकए जात थ

- इसस यह पता चलता ह वक घ ड ो यकत रथ ो का

उपय ग महाजनपद काल स काफी पहल आरमभ

हआ था

- ऋगववदक काल म मिवशय ो भवम जल आवद पर

कबजा करन क वलए तथा ल ग ो क पकडन क

वलए यद वकय जात थ

- अविकाोश परि इन यद ो म भाग वलया करत थ

- हालाोवक उस समय क ई वनयवमत सना नही ो ह ती

थी लवकन उस काल म सभाऐो ह ती थी ो वजनम

ल ग यद क मामल ो पर चचाि करत थ

- वनयवमत सनाएा महाजनपद काल का िवशषटय थी

वजनम पदल सवनक ो की विशाल सनाएा रथ तथा

हाथी शावमल ह त थ

Q8) उततर (a)

सपषटीकरण

- बद शाकय कल स सोबोवित थ और कशीनारा म

उनका वनिन हआ था

- बद न अपनी वशकषाएा पराकत भािा म दी थी ो ज

आम ल ग ो की भािा थी

Q9) उततर (c)

सपषटीकरण

- पराचीन भारत म दशिनशासर की छह शाखाएा थी ो

िशविक नयाय समखया य ग पिि वममाोसा और

िदाोत या उततर वममाोसा

- इनकी थथापना करमश कनाद गौतम कवपल

पतोजवल जावमनी और वयास ऋविय ो न की थी

Q10) उततर (b)

सपषटीकरण

महािीर की वशकषाऐो छठी शताबदी म िललभी म

सोकवलत की गई थी ो

Q11) उततर (c)

सपषटीकरण

- पारमपररक रप स चाणकय क कौविलय अथिा

विषणगपत क नाम स जाना जाता ह

- उसन अथिशासतर ज एक पराचीन भारतीय

राजनवतक आलख ह वलखा था

Q12) उततर (d)

सपषटीकरण

- भारत का राषटर ीय वचनह सारनाथ (उततर परदश) क

अश क सतमभ क ऊपर (शीिि पर) वसोह कवपिल

का एक अनरपण ह

- इस राषटर ीय वसदाोत सतयमि जयत क साथ

सोय वजत वकया गया ह

- रामपिि बल का नाम रामपिि (वबहार) क नाम पर

पडा जहाा इसकी ख ज हई थी

- यह अपन नाजक नकाशी मॉडल क वलए परवसदद

ह वजसम क मल तवचा सोिदनशील नथन ो सतकि

कान और मरबत िााग ो क शरषठतर परवतरप क

परदवशित वकया गया ह

- यह भारतीय और फारसी ततव ो का एक ससममशरण

- सोवकससा उततर परदश म सथथत ह

Q13) उततर (a)

सपषटीकरण

का िर वसोह ज एक महान य दा थ वबहार स

सोबोवित थ

Q14) उततर (b)

सपषटीकरण

िललालर शबद बड भ-सवावमय ो क वलए परय ग

वकया जाता था

FC19H1003 31

Q15) उततर (c)

सपषटीकरण

- अररकमड एक तिीय बसती थी जहाो दर दश ो स

आन िाल जहाज ो का माल उतारा जाता था

- यहाो पर ईोि ो का एक विशाल ग दाम वमटटी क

बतिन (वजनम एमफ रा - द हरी मवठय ो का लोबा

घडा - शावमल ह) और एरिाइन (Arretine)

मदभाोड पाए गए थ

- इस थथान पर र मन दीपक काोच क बन पातर और

रतन भी पाए गए थ

Q16) उततर (a)

सपषटीकरण

- मिनदर सोगम कविताओो म उसललसखत एक

तवमल शबद ह वजसका अथि ह ldquoतीन परमखrdquo

- यह तीन सततारि पररिार ो क मसखयाओो क वलए

परय ग वकया जाता ह च ल चर और पाणडय

Q17) उततर (c)

सपषटीकरण

- ऋग िद म सभा विदाथा तथा गण जसी

जनजावतय ो पर अथिा किोब पर आिाररत

सभाओो का उललख ह

- आरसमभक िवदक काल म सभाओो और सवमवतय ो

का विशि महतव ह ता था

- यहाा तक की मसखया अथिा राजा भी उनका

समथिन परापत करन क वलए आतर रहत थ

Q18) उततर (a)

सपषटीकरण

- जन िमि न ईशवर क अससततव क मानयता त दी ह

वकनत उसन ईशवर क वजना क पद स नीच रखा

- जन िमि न बौद िमि की तरह िणि परणाली की

भरतिना नही ो की थी

Q19) उततर (d)

सपषटीकरण

- च ल ो और पाणडय ो न शसकतशाली तिीय शहर ो का

विकास वकया था

- च ल ो का सबस महतवपणि शहर पहार (या

कािरीपटटीनम) था |

- मदरई पाणडय ो की राजिानी थी

Q20) उततर (b)

सपषटीकरण

- ldquoबदचररतrdquo बद का जीिन-ितताोत ह

- इस अशवघ ि क दवारा वलखा गया था

Q21) उततर (a)

सपषटीकरणः

- तवमल कवि अपपर भगिान वशि क भकत थ

- इस परकार ि एक नयनार सोत थ

Q22) उततर (d)

सपषटीकरणः

- समदरगपत एक परवसद गपत शासक था

- उसन वसक ो पर िीणा बजात हए अपनी छवि

अोवकत करिाई थी

- यह सोगीत क परवत उसक परम क दशािता ह

- हम उसकी इलाहाबाद परशससत स महतवपणि

ऐवतहावसक जानकारी वमलती ह वजसकी रचना

उसक दरबार क कवि हररसन न की थी

Q23) उततर (b)

सपषटीकरणः

- विकरम सोित की शरआत ििि 58 ईसा पिि म

चनदरगपत वदवतीय न की थी

- यह शक ो पर उसकी जीत और उस विकरमावदतय

की पदिी वमलन क उपलकषय म आरमभ वकया गया

था

FC19H1003 32

- बानभटट न हिििििन का जीिन-ितताोत हििचररत

(ज सोसकत म थी) वलखी थी

Q24) उततर (c)

सपषटीकरणः

- सोवि-विगरावहका यद एिो शाोवत का मोतरी

- साथििाह वयापाररय ो क कावफल ो का नता

Q25) उततर (a)

सपषटीकरणः

- जआन झाोग (हसआन रताोग ndash Hsuang Tsang)

एक चीनी यातरी था ज हिििििन क शासनकाल म

भारत आया था

- ििि 630 ईसवी स ज दशक आरमभ हआ था उसम

जआन झाोग मधय एवशया ईरान और

अफग़ावनसतान की यातरा करन क पशचात कशमीर

क रासत स भारत आया था

- उसन उततर स पिि तक की यातरा की और िह

लगभग 2 ििि वबहार म रहा

- जआन झाोग न नालनदा विशवविदयालय म विदयावथिय ो

और विदवान ो क साथ पारसपररक विचार-विमशि

वकया थथानीय भािाओ ा म वनपणता परापत की तथा

बौद सतप ो की ख ज की

Q26) उततर (c)

सपषटीकरणः

- परदवकषणा पथ बौद िासतकला म सतप क चार ो

ओर बनाया जान िाला एक घमािदार पथ ह ता

- परशन म वदए गए बाकी क तीन ो ततव वहोद मसनदर ो की

िासतकला क भाग ह

Q27) उततर (d)

सपषटीकरणः

परशन म वदए गए सभी मोवदर ो म वयापक रप स

ईोि ो (पकी ईोि ो) का परय ग पतथर ो क साथ हआ

Q28) उततर (c)

सपषटीकरण

- महममद कली कतब शाह ग लकणडा का सलतान

था

- िह अकबर का समकालीन था

- सावहतय और िासतकला म उसकी अतयाविक

रवच थी

- िह एक महान कवि था

- िह दसखनी उदि फारसी और तलग म वलखता था

- उसन अपन पीछ एक विसतत वदिान (सोगरह)

छ डा ह

- अभी हाल ही म तलोगाना म ग लकणडा क वकल

क अनदर खदाई वकय गए बाग-ए-नाया वकला

बाग क चार ो ओर रप-रखा क मानवचतरण क

वलए भारतीय परातासतवक सिकषण (The

Archaeological Survey of India ndash ASI)

गराउणड पनीिर विोग रडार (Ground Penetrating

Radar) का परय ग करगा

Q29) उततर (a)

सपषटीकरणः

- वसलपपावदकारम एक तवमल महाकावय ह वजसकी

रचना इलाोग क दवारा लगभग 1800 ििि पिि की

गई थी

- यह क िलन नामक एक वयापारी की कहानी ह

ज माििी नामक एक गवणका (िशया) स परम

करन लगा था

- मवनमकलाई क िलन और माििी की पतरी की

कहानी ह

Q30) उततर (a)

सपषटीकरण

- चरक आयिद और वचवकरता की एक महतवपणि

रचना चरक सोवहता क लखक ह

- बरहमगपत क अपनी रचना बरहम-सफि-वसदानत

(ज एक खग लीय रचना ह) क कारण परवससद

वमली

FC19H1003 33

- बगदाद म इसका अनिाद अरबी भािा म वकया

गया था

- इसका इसलावमक गवणत और खग ल-विजञान पर

महतवपणि परभाि पडा था

- बाद म अपन जीिनकाल म बरहमगपत न

ldquoखोडखयाकrdquo वलखी ज एक खग लीय पससतका

(एक छ िी पसतक) थी

- इसम आयिभटट की अिि-रावतर क परतयक वदन की

शरआत परणाली का परय ग वकया गया था

Q31) उततर (c)

सपषटीकरण

- अमीर खसर एक परवसद सफी सोगीतकार कवि

और विदवान थ

- 1318 म उनह ोन पाया वक इस भवम (वहोदसतान) क

हर कषतर म अलग-अलग भािा थी लाहौरी

कशमीरी दवारसमदरी (दवकषणी कनाििक म)

तलोगाना (आोधर परदश म) गजरी (गजरात म)

माबारी (तवमलनाड म ) अििी (पिी उततर परदश

म) और वहोदिी (वदलली क आस-पास क कषतर म)

आवद

- उनह न यह बताया वक सोसकत वकसी भी कषतर स

सोबोवित नही ो थी और किल बराहमण ही इस भािा

का जञान रखत थ

Q32) उततर (c)

सपषटीकरण

- वहरणय-गभि सववणिम गभि क सोदवभित करता ह

- जब बराहमण ो की सहायता स यह अनषठान वकया

जाता था त यह माना जाता था वक बवल दन िाल

का कषवतरय क रप म पनजिनम ह गा

Q33) उततर (d)

सपषटीकरण

- कदमई भवम राजसव पर कर क सोदवभित करता

- गवावलयर परशससत म नागभि क दवारा वकय गए

श िण का िणिन वकया गया ह |

- नागभि एक परवतहार राजा था

Q34) उततर (b)

सपषटीकरण

- राजतरो वगनी 12िी ो शताबदी म कलहन क दवारा

रवचत एक सोसकत पसतक (िकसट) ह

- यह परारसमभक भारत की ऐवतहावसक इवतितत थी

- तकि सोगत रप स इस अपन परकार की सिोततम

और सिािविक विशवसनीय कवत माना जाता ह

- यह कशमीर कषतर क पराचीनतम समय स लकर

उसकी रचना की तारीख तक क समपणि इवतहास

का आचछादन करती ह

Q35) उततर (c)

सपषटीकरण

- गााि की आम सभा क ldquoउरrdquo कहा जाता था

- ldquoउरrdquo म गााि क सभी कर दन िाल वनिासी

शावमल ह त थ

Q36) उततर (a)

सपषटीकरण

- वदलली सलतनत म ldquoतारीखrdquo इवतहास लखन का

एक रप था

- ldquoतािरीखrdquo क लखक विदवान परि ह त थ वजनम

सवचि परशासक इतयावद शावमल थ

Q37) उततर (a)

सपषटीकरण

- अलाउददीन सखलजी अपन सवनक ो क ितन का

भगतान नकद म करता था न वक इकता क रप

- सवनक अपना सामान वदलली म वयापाररय ो स

खरीदत थ अतः इस बात का भय था वक वयापारी

कही ो िसतओो का मलय न बिा द

- इसकी र कथाम क वलए अलाउददीन सखलजी न

वदलली म कीमत ो क वनयसित वकया

FC19H1003 34

- अविकारीगण धयानपििक मलय ो का सिकषण करत

थ तथा ज वयापारी वनिािररत मलय पर माल नही ो

बचत थ उनक दसणडत वकया जाता था

Q38) उततर (d)

सपषटीकरण

- वदलली सििपरथम त मर राजपत ो क अिीन उनक

सामराजय की राजिानी बनी थी

- 12िी ो शताबदी क मधय म अजमर क चौहान ो

(वजनह चाहमान ो क नाम स भी जाना जाता ह) न

त मर राजपत ो क परावजत वकया था

- त मर ो और चौहान ो क अिीन वदलली एक

महतवपणि िावणसजयक क दर बन गया था

- कई जन वयापारी यहाा रहन लग थ और उनह ोन

कई मोवदर भी बनिाए

- यहाा पर मवदरत वसक वजनह ldquoदहलीिालrdquo क नाम

स जाना जाता था वयापक रप स परचलन म थ

Q39) उततर (c)

सपषटीकरण

- म ठ की मसिद का वनमािण वसको दर ल दी क

राजयकाल म उसक मिी क दवारा करिाया गया

था

- बगमपरी मसिद का वनमािण महममद तगलक क

शासनकाल म हआ था

- यह मसिद विशव का पणयथथान (The

Sanctuary of the World) और वदलली म महममद

तगलक की नई राजिानी जहाोपनाह की मखय

मसिद थी

- कववत- अल - इसलाम मसिद का विसतार

इलतसिश और अलाउददीन सखलजी न वकया था

- मीनार का वनमािण तीन सलतान ो कतबददीन ऐबक

इलतसिश और वफर ज शाह तगलक क दवारा

करिाया गया था

Q40) उततर (c)

सपषटीकरण

- मगल ो क अिीन मनसबदार शबद उस वयसकत क

वलए सोदवभित वकया जाता था वजसक पास मनसब

(अथाित पद) ह ता था

- उस अपना ितन राजसव कायो वजनह जागीर कहत

थ क रप म परापत ह ता था

Q41) उततर (b)

सपषटीकरण

- ldquoभारत छ ड आोद लनrdquo वबरविश शासन क

सखलाफ ल ग ो का एक सवाभाविक विदर ह था

- असखल भारतीय काोगरस सवमवत न 8 अगसत 1942

क बमबई म एक बठक का आय जन वकया था

- इस बठक म परवसद सोकलप ldquoभारत छ ड rdquo क

पाररत वकया गया और इस उददशय क परापत करन

क वलए गाोिी क नततव म एक अवहोसक जन सोघिि

आोद लन की शरआत का परसताि वदया गया

- लवकन अगल ही वदन गाोिी और काोगरस क अनय

परमख नताओो क वगरफतार कर वलया गया

- काोगरस क एक बार वफर अिि घ वित वकया गया

था

Q42) उततर (c)

सपषटीकरण

- साइमन कमीशन यनाइविड वको गडम क सात

साोसद ो का एक समह था

- इस वबरविश भारत क वलए सोििावनक सिार ो का

सझाि दन क वलए गवठत वकया गया था

- इस आय ग म िररषठ वबरविश राजनता सर जॉन

साइमन क नततव म किल वबरविश सदसय ही

शावमल थ

- इसवलए भारत क ल ग ो न साइमन कमीशन क

आगमन क विरद आोद लन वकया था

Q43) उततर (a)

सपषटीकरण

bull दादा भाई नौर जी भारत म वबरविश शासन क

आवथिक पररणाम ो क बार म अपनी विर िी

(परवतकल) राय क वलए जान जात थ

FC19H1003 35

bull अपन कई लख ो और भािण ो म विशि रप स

ldquoपाििी एो ड अन-वबरविश रल इन इसणडया

(Poverty and Un-British Rule in India) म

नौर जी न यह तकि वदया वक भारत पर अतयविक

कर लगाया गया था और इसकी सोपवतत इोगलड की

ओर परिावहत की जा रही थी

bull उनह ोन पराचीन भारतीय गरोथ ो की वयाखया करन

का और भारतीय ो क आिविशवास क बहाल

करन पर कायि नही ो वकया था

उनह ोन वकसी और बात स पहल सभी सामावजक

बराइय ो क उनमलन की आिशयकता पर भी बल

नही ो वदया था

Q44) उततर (c)

सपषटीकरण

bull अगसत 1932 म वबरविश परिानमोतरी मकड नालड न

अपन साोपरदावयक परसकार (The Communal

Award) की घ िणा की थी

bull यह भारत क कई साोपरदावयक वहत ो क बीच विवभनन

सोघिो क हल करन क वलए वबरिन का एकतरफा

परयास था

bull यह परसकार (Award) बाद म 1935 क

अविवनयम (The Act of 1935) म शावमल वकया

गया था

bull इस साोपरदावयक परसकार न मससलम ो क वलए

आरवकषत एक अलग वनिािचक मणडल फॉमिल का

विसतार अनय अलपसोखयक ो क वलए वकया था

वजसम वसख ो भारतीय ईसाइय ो आोगल-भारतीय

समदाय यर पीय समदाय तथा विवशषट कषतरीय

समह ो क शावमल वकया गया था

bull गाोिी न इस परसताि क भारतीय समाज क

विभावजत करन क वलए एक घवणत वबरविश

सावजश क रप म दखा और उसक सखलाफ

आमरण अनशन वकया

Q45) उततर (b)

सपषटीकरण

मौजदा आयात और वनयाित क अवतररक़त

औपवनिवशक भारत क वनमनवलसखत खचो क

वलए एक विशिवनवशचत िन रावश भी दनी पडती

थी

(i) परशासन क वयय

(ii) सना क रख-रखाि क वयय

(iii) यद क वयय

(iv) सिावनितत अविकाररय ो की पशन तथा

(v) वबरिन दवारा अपनी उपवनिश बसती

(कॉल नी) क रख-रखाि क वयय

इनह गह शलक (Home Charges) क रप म

जाना जाता था और लगभग परी तरह स भारत क

दवारा इनका भगतान वकया जाता था

bull गह शलक म वनमनवलसखत घिक शावमल थ

(i) भारतीय ऋण पर दय बयाज

(ii) ईसट इोवडया को पनी क शयरिारक ो क

लाभाोश

(iii) लोदन म भारत कायािलय चलान क वलए िन

(iv) भारत म वनयकत वबरविश कवमिय ो क ितन

और पशन का भगतान करन क वलए िन

(v) रलि पर बयाज

(vi) नागररक और सनय शलक

(vii) इोगलड म सट र (सामगरी) की खरीद

Q46) उततर (b)

सपषटीकरण

bull भारतीय राषटर ीय काोगरस का लाहौर सतर 1929 म

जिाहरलाल नहर की अधयकषता म आय वजत

वकया गया था

bull इस सतर म भारतीय राषटर ीय आोद लन स समबसित

कई महतवपणि पररणाम सामन आय थ

(i) सििपरथम इस सतर म काोगरस क अधयकष पद

पर जिाहरलाल नहर क चना गया था ज

काोगरस म िामपोवथय ो की बिती हई ताकत

का सपषट सोकत था

(ii) दसरा इस सतर म पहली बार काोगरस न पणि

सवतोतरता की माोग क उठाया था

इस परकार की माोग काोगरस मोच स पहल कभी भी

नही ो उठाई गई थी

Q47) उततर (b)

सपषटीकरण

FC19H1003 36

bull इस ररप िि न वकसी भी समदाय क वलए पथक

वनिािचक मोडल अथिा अलपसोखयक ो क वलए

भाराोश की वसफाररश नही ो की थी

bull तथावप इस ररप िि न उन पराोत ो म अलपसोखयक

सीि ो क आरकषण की अनमवत दी थी जहाा पर कम

स कम दस परवतशत अलपसोखयक ह

bull लवकन यह समदाय क आकार क अनपात म ह ना

चावहए था

bull इस ररप िि म भारत क वलए पणि सवतोतरता क

वलए क ई पराििान नही ो था

Q48) उततर (c)

सपषटीकरण

bull आरो वभक िवदक आयो का िमि मखय रप स

परकवत की पजा और यजञ था

bull परारो वभक आयि िमि परकवत की पजा क समान था

bull िासति म उनक चार ो ओर की शसकतयाा वजनह न

त ि वनयोवतरत कर सकत थ और न ही समझ पाए

थ उनह वदवयता क साथ वनिवशत वकया गया तथा

उनह मादा या नर दिीदिताओो क रप म

परतीकतव वकया गया था

bull उनह ोन कछ यजञ ो का भी वनषपादन वकया था

Q49) उततर (b)

सपषटीकरण

bull सडक और नदी-मागि (जल-मागि) डकती स

सरवकषत नही ो थ

bull उललखनीय ह वक हिििििन क शासनकाल क

दौरान यआन चिाोग (हयएन साोग) का सारा

सामान लि वलया गया था

Q50) उततर (c)

सपषटीकरण

परशन म वदए गए द न ो कथन सही ह

Q51) उततर (b)

सपषटीकरण

bull परोदर दास एक सोत और भगिान कषण क एक

महान भकत थ

bull परोदर दास क कनाििक सोगीत क वपतामह क

रप म जाना जाता ह

bull यदयवप उनक जनम-थथान क बार म काफी

अिकल लगाई जाती रही ह

bull तथावप अब कननड विशवविदयालय हमपी क दवारा

गवठत एक विशिजञ सवमवत इस वनषकिि पर पहोची

ह वक उनका जनम थथान सोभितया कनाििक का

एक छ िा-सा गााि कषमपरा (वशिम गगा वजला)

था

Q52) उततर (c)

सपषटीकरण

bull शरी तयागराज शरी शयाम शासतरी और शरी मथसवामी

दीवकषतर क कनाििक सोगीत की वतरमवति माना

जाता ह

bull उनक कारण ही 18िी ो-19िी ो शताबदी म कनाििक

सोगीत का सववणिम यग आया था

Q53) उततर (d)

सपषटीकरण

bull अभी हाल ही म लौह यगीन-महापािावणक काल

का 2000 ििि पराना एक दलिभ सारक फगस

(Sarcophagus) (पतथर का ताबत) क ललम क

वियर गाोि (क वयलडी क पास वजला क वझक ड

करल राजय) की एक रॉक-कि गफा स ख जा गया

bull यह ताबत वजसम हविय ो क िकड थ खदाई क

दौरान वमला

bull अभी तक इस परकार की दलिभ ख ज करल क

मातर द ही थथान ो स हई ह

bull य द न ो सारक फगी (Sarcophagi) (पतथर क

ताबत) चियर और अथ ली (वजला क वझक ड) क

महापािाण थथल ो स वमल ह

Q54) उततर (a)

सपषटीकरण

FC19H1003 37

दवकषण भारत म महापािाण सोसकवत एक पणि

विकवसत लौह यगीन सोसकवत थी

Q55) उततर (d)

सपषटीकरण

bull च ल पाणडय और करलपतर (चर) इन तीन ो का

उललख अश क क अवभलख ो म वकया गया ह

bull सोभितः य भौवतक सोसकवत क उततर

महापािावणक चरण म थ

Q56) उततर (d)

सपषटीकरण

bull भीमा-क रगाोि की लडाई ततीय आोगल-मराठा

यद का वहससा थी

Q57) उततर (b)

सपषटीकरण

bull राजकमार शकल न गाोिीजी क चोपारण आन तथा

वतनकवथया परणाली स जडी समसया की जाोच क

वलए रारी करन क वलए दश भर म उनका

अनसरण वकया था

bull बज वकश र राजदर परसाद महादि दसाई और

नरहरी पाररख चोपारण सतयागरह क दौरान गाोिी

जी क सहय गी थ

Q58) उततर (b)

सपषटीकरण

bull बराहमण ो और बौद मठिाररय ो क कर-मकत गााि

अनदान म दन की परथा सतिाहन ो न आरमभ की

थी

Q59) उततर (c)

सपषटीकरण

इस कायिकरम क उददशय वनमनानसार ह

(i) बवनयादी पयििन आिाररक सोरचना का विकास

करना

(ii) चयवनत (पहचान वकय गए) कषतर ो म आजीविका क

सजन क वलए दश क साोसकवतक और विरासत

मलय ो क बिािा दना

(iii) विरासत समारक थथल ो पर विशव सतरीय आिाररक

सोरचना विकवसत करक एक सतत तरीक स

पयििक आकििण म िसद करना

(iv) थथानीय समदाय ो की सवकरय भागीदारी क माधयम

स र रगार ो का सजन करना

(v) र रगार उतपादन और आवथिक विकास क वलए

पयििन कषमता का उन पर परभाि का उपय ग

करना तथा

(vi) िारणीय पयििन आिाररक सोरचना का विकास

करना और उसका उवचत सोचालन तथा

रखरखाि सवनवशचत करना

Q60) उततर (b)

सपषटीकरण

bull यह वनकाय ििि 1987 म अससततव म आया था

bull यह एक राषटर ीय सतर का शीिि सोगठन ह ज भारत

सरकार क जनजातीय मामल ो क मोतरालय क

परशासवनक वनयोतरण क अिीन काम कर रहा ह

bull इसका पोजीकत और परिान कायािलय नई वदलली

म सथथत ह

Q61) उततर (c)

सपषटीकरण

bull परमचोद क उपनयास ो म परमाशरम रोगभवम गबन

कमिभवम और ग दान शावमल ह

bull ग रा रिी ोदरनाथ िग र क दवारा रवचत उपनयास ह

bull अभी हाल ही म मोशी परमचोद की 138िी ो जयोती दश

भर म मनाई गई थी

Q62) उततर (b)

सपषटीकरण

bull ldquoवगदाrdquo पोजाब (भारत) एिो पावकसतान की

मवहलाओो क दवारा तयौहार क समय और फसल

की बिाई तथा किाई क अिसर पर वकया जान

िाला एक पारोपररक दहाती नतय ह

FC19H1003 38

bull इस नतय क माधयम स पोजाबी मवहलाऐो अपनी

परसननता परकि करती ह तथा वगदा क परदशिन क

माधयम स परि िचिसव िाल समाज म मवहलाओो

की दबी हई भािनाओो क परकि करती ह

bull चोवक इस नतय का परि ो क साथ क ई सोबोि नही ो

ह अतः किल मवहलाऐो ही इसम भाग ल सकती

bull हर साल तीज समार ह क दौरान पोजाब म वगदा

नतय वकया जाता ह

तीज भारत क कछ भाग ो म मवहलाओो क दवारा

मनाया जान िाल कई तयौहार ो क वलए एक

वयापक नाम ह

Q63) उततर (a)

सपषटीकरण

- मजम-उल-बहरीन या द समदर ो का सोगम

नामक उललखनीय रचना दारा वशक ह क दवारा

वलखी थी

- भारत क उपराषटर पवत शरी एम िकया नायड न कहा

ह वक राजकमार दारा वशक ह की रचनाएा शाोवत

और सदभाि क बिािा दन क वलए एक तारा सर त

क रप म सामन आ सकती ो ह

- उपराषटर पवत गत ििो क भला वदए गए राजकमार

दारा वशक ह क परदवशित परचवलत करन हत

आय वजत एक परदशिनी का दौरा करन क बाद एक

सभा क सोब वित कर रह थ

- इस परदशिनी का आय जन फर क इस गौवियर

(Francois Gautier) क दवारा lsquoइोवदरा गाोिी नशनल

सिर फॉर द आििसrsquo (The Indira Gandhi

National Centre for the Arts) नई वदलली म

वकया गया था

Q64) उततर (c)

सपषटीकरण

- ग मतशवर परवतमा जन भगिान बाहबली क

समवपित ह

- यह एक एक-चटटानी पतथर की मवति ह

- राषटर पवत राम नाथ क विोद न शरिणबलग ला

(कनाििक) म आय वजत वकय जान िाल भवय

अवभिक समार ह महामसतकावभिक का

उदघािन वकया था

- यह समार ह 12 ििो म एक बार ह ता ह

Q65) उततर (c)

सपषटीकरण

bull पराची घािी पराची नदी क चार ो ओर फली हई थी

bull पराची घािी िीर-िीर विलपत ह गई थी

bull पराची नदी भिनशवर स वनकलती ह

bull यह महानदी की एक सहायक नदी ह और यह

परी खदाि किक तथा जगतवसोहपर वजल ो स

ह कर बहती ह

bull इस नदी क पर कषतर क पराची घािी कहा जाता ह

bull यह नदी बोगाल की खाडी म वगरती ह

परातासतवक साकषय स पता चलता ह वक पराची घािी

सभयता हडपपा और म हनज दाड द न ो की

पिििती ह

Q66) उततर (d)

सपषटीकरण

य समारक छतरपर वजल (मधय परदश) म विोधयाचल

पिित शरोखला म सथथत ह

Q67) उततर (a)

सपषटीकरण

bull थॉिस ऑन पावकसतान नामक पसतक डॉ बी

आर अमबडकर न वलखी थी

bull डॉ बी आर अमबडकर की जयोती क अिसर पर

भारत क राषटर पवत न भारत की इस महान हसती

क शरदाोजवल अवपित की थी

bull डॉ बी आर अमबडकर न 1924 म वडपरथड

कलावसर इोसटीटयि (दवलत िगि सोथथान -

बवहषकत वहतकाररणी सभा) और 1927 म समाज

समता सोघ की थथापना की थी

bull अमबडकर का धयान वशकषा कषतर की ओर भी था

bull उनह ोन वशकषा क वनमन िगो म फलान क वलए

पीपलस एजकशन स साइिी (The Peoples

Education Society) क नाम स महाविदयालय ो क

नििकि और छातरािास ो की थथापना की थी

FC19H1003 39

Q68) उततर (b)

सपषटीकरण

bull महरगि भारतीय उपमहादवीप म एक परवसद

निपािाण बसती ह ज बलवचसतान पराोत

पावकसतान म सथथत ह

bull दचपलली (आोधर परदश) क पास नागलर नदी क

पिी ति ो पर चना पतथर क बलॉक क विशाल

विसतार म एक पिि-ऐवतहावसक रॉक आिि थथल की

ख ज की गई ह

bull इसन 1500-2000 ईसा पिि क दौरान गोिर (आोधर

परदश) म विकवसत निपािाण सभयता पर परकाश

डाला ह

Q69) उततर (c)

सपषटीकरण

bull 12िी ो सदी और 13िी ो सदी म काकाविय िोश का

उदय हआ था

bull ि पहल कलयाण क पवशचमी चालकय ो क सामोत थ

bull परारोभ म उनह ोन िारोगल (तलोगाना) क पास एक

छ ि स कषतर पर शासन वकया था

bull उनह ोन ldquoनायक वयिथथाrdquo की शरआत की थी

वजस बाद म विजयनगर क राय शासक ो न

अपनाया और विकवसत वकया था

Q70) उततर (a)

सपषटीकरण

bull गाोिीजी क अनशन स वमल मावलक ो पर दबाि

पडा था ज अोततः शरवमक ो क ितन म 35 परवतशत

की िसद करन क वलए सहमत हए थ

bull गगल (Google) न अनसया साराभाई वजनह ोन

भारत क शरवमक आोद लन म एक अगरणी भवमका

वनभाई थी की 132िी ो जयोती डडल (Doodle) का

वनमािण करक मनाई

Q71) उततर (d)

सपषटीकरण

भारत स यनसक की मानिता की अमति साोसकवतक

विरासत की परवतवनवि सची म वनमनवलसखत शावमल ह

bull कवडयटटम करल का सोसकत रोगमोच

bull मवडयिि करल का अनषठान रोगमोच और नतय

नाविका

bull िवदक मि जाप की परोपरा

bull राजथथान क कालबवलया ल क गीत और नतय

bull रामलीला रामायण का पारोपररक परदशिन

bull सोकीतिन मवणपर का अनषठान गायन ढ ल िादन

और नतय

bull रममन भारत क गििाल वहमालय का िावमिक

तयौहार और अनषठान रोगमोच

bull जाोदीयाला गर पोजाब क ठठर ो की पीतल और

ताोब क वशलप स वनवमित बतिन ो की पारोपररक कला

bull छाऊ नतय पिी भारतीय राजय ो म जनमी शासतरीय

भारतीय नतय कला

bull लददाख का बौद मि जाप िर ाोस-वहमालयी लददाख

कषतर तथा जमम-कशमीर म पवितर बौद गरोथ ो का पाठ

bull य ग

bull नौर र

bull को भ मला

Q72) उततर (b)

सपषटीकरण

bull भारत क राषटर पवत शरी राम नाथ क विोद न

वकसामा नागालड म हॉनिवबल मह रति और

राजय गठन वदिस समार ह का उदघािन वकया

था

bull हॉनिवबल मह रति का नाम भारतीय हॉनिवबल क

नाम पर पडा ह ज एक विशाल और रोगीन जोगली

पकषी ह

bull यह पकषी नागालड राजय की अविकतर जनजावतय ो

की ल ककथाओो म उसललसखत ह

bull नागालड की परमख मानयता परापत जनजावतयाा ह

अोगामी आओ चखसोग चाोग ककी रगमा और

रवलोग आवद

bull ओोग जारिा और ससिनलीस अोडमान-वनक बार

दवीप समह की जनजावतयाा ह

FC19H1003 40

Q73) उततर (c)

सपषटीकरण

bull दकन म राषटर कि शासन दसिी ो सदी क अोत तक

लगभग 200 ििो तक रहा था

bull राषटर कि शासक अपन िावमिक विचार ो म सवहषण

bull उनह ोन न किल शि िमि और िषणि िमि बसलक

जन िमि क भी सोरकषण वदया था

bull एल रा म वशि क परवसद रॉक कि मोवदर का

वनमािण नौिी ो सदी म राषटर कि राजा कषण परथम न

करिाया था

bull उसका उततराविकारी अम घििि जन था लवकन

उसन अनय िमो क भी सोरकषण परदान वकया था

bull राषटर कि ो न मसलमान वयापाररय ो क बसन की

अनमवत दी थी

bull उनह न अपन अविराजय ो म इसलाम क उपदश दन

की भी अनमवत दी थी

bull अभी हाल ही म पाोडिलागटटा (तलोगाना) क

परागवतहावसक चटटान वचतर ो क कषरण की बिती हई

घिनाएा एक गोभीर वचोता का वििय ह

bull यह परागवतहावसक चटटान क नकसान पहाचा

सकता ह

bull पाोडिलागटटा वनमनवलसखत क वलए जाना जाता ह

- 10000 ईसा पिि स 8000 ईसा पिि क वचवतरत

चटटानी आशरय ो क वलए

- राषटर कि काल क एक 8 िी ो सदी क

वशलालख क वलए और

- 12िी ो सदी क काकविय सामराजय क वभवतत

वचतर ो क वलए

Q74) उततर (b)

सपषटीकरण

bull 1828 म राजा राम म हन रॉय न एक नय िावमिक

समाज बरहम सभा की थथापना की थी वजस बाद

म बरहम समाज क नाम स जाना गया था

bull दिदरनाथ िग र न ततवब विनी सभा की अधयकषता

की थी ज आधयासिक सतय की ख ज म सोलि

थी

bull इसका उददशय वहोद िमि क शद करन का और

एकशवरिाद (एक ईशवर म आथथा) का परचार करना

था

bull नय समाज की थथापना क आिार थ कारण

(तकि ) क द सतमभ तथा िद और उपवनिद

bull अभी हाल ही म सािारण बरहम समाज का कछ

काननी मदद ो क लकर पवशचम बोगाल सरकार क

साथ काननी वििाद चल रहा ह

Q75) उततर (c)

सपषटीकरण

bull भारत म वचशती वसलवसल की थथापना खवाजा

म इनददीन वचशती क दवारा की गयी थी

bull ि 1192 ईसवी क आसपास भारत आय थ

bull वचशतीय ो क बारहिी ो शताबदी क उततरािि म भारत

म आन िाल सफीय ो क समह ो म सबस

परभािशाली माना जाता ह

bull उनह ोन थथानीय िातािरण क साथ सफलतापििक

अनकलन वकया और उनह ोन भारतीय भसकत

परोपराओो क कई पहलओो क अपनाया

bull अजमर म सफी अपरकि खवाजा म इनददीन वचशती

की ऐवतहावसक दरगाह क एक नया रप दन की

तयारी की जा रही ह

bull इस 13िी ो शताबदी की दरगाह क ldquoसवचछ

आइकॉवनक थथल ोrdquo (Swacch Iconic Places) म

शावमल वकया गया ह ज परवतवषठत विरासत

आधयासिक और साोसकवतक थथान ो पर क वदरत

य जना ह

FC19H1003 41

ANSWERS amp EXPLANATION OF

NCERT History Class VI-X + Current Affairs

(FC19E1003)

Q1) Answer c

Explanation

Rigveda consists of more than a

thousand hymns dedicated to gods and

goddesses These hymns were

composed by sages and learnt by men

however a few were composed by

women like Apala Ghosa Lopamudra

Maitreyi and Gargi

Rigveda consists of many hymns in the

form of dialogues We get an example of

a dialogue between a sage named

Vishwamitra and two rivers (Beas and

Sutlej) that were worshipped as

goddesses This suggests that he

belonged to the Vedic period

Q2) Answer b

Explanation

Traces of ash have been found from

Kurnool Caves suggesting that people

were familiar with the use of fire

It is situated in Andhra Pradesh

Q3) Answer c

Explanation

Burzahom is a prehistoric site in

present day Kashmir where people built

pit houses which were dug into the

ground with steps leading into them

These may have provided shelter in cold

weather

Q4) Answer c

Explanation

Epigraphy is defined as the study of

inscriptions

Manuscriptology is the study of history

and literature through the use of hand

written documents

Palaeography refers to the study of

ancient writing systems and the

deciphering and dating of historical

manuscripts

Numismatics refers to the study of

coins

Q5) Answer a

Explanation

Charaka Samhita was written by

Charaka and is an important book on

Ayurveda and medicine

He was a practitioner of the traditional

system of Indian medicine known as

Ayurveda

Charaka is thought to have flourished

sometime between the 2nd century BCE

and the 2nd century CE

Q6) Answer b

Explanation

Bhaga refers to the tax on crops which

was fixed at 16th of the production

Kammakaras is the term used for the

landless agricultural labour class

Ashvamedha also known as horse

sacrifice is a ritual where a horse is let

loose to wander freely and it was

guarded by the rajarsquos men

Q7) Answer (d)

Explanation

In the Rigvedic period horses were

yoked to chariots that were used in

battles fought to capture land cattle

etc This suggests that the use of horse

chariots began much before the period

of Mahajanapadas

The battles were fought in the Rigvedic

period for cattlersquos lands water an even

to capture people Most men took part

in these wars however there was no

regular army but there were assemblies

where people met and discussed

matters of war Regular armies became

a feature in the Mjahajanapada period

including vast armies of foot soldiers

chariots and elephants

RAUSIAS-FC19E1003 42

Q8) Answer (a)

Explanation

Buddha belonged to the Sakya clan and

passed away at Kusinara

Buddha taught in Prakrit which was the

common language of people

Q9) Answer c

Explanation

There were six schools of philosophy in

ancient India These are known as

Vaishesika Nyaya Samkhya Yoga

Purva Mimansa and Vedanata or Uttara

Mimansa They were founded by sages

Kanada Gautama Kapila Patanjali

Jamini and Vyasa respectively

Q10) Answer b

Explanation

The teachings of Mahavira were

compiled at Valabhi in 6th century AD

Q11) Answer (c)

Explanation

Chanakya is traditionally identified as

Kautilya or Vishnugupta who authored

the ancient Indian political treatise the

Arthashastra

Q12) Answer d

The national emblem of India is an

adaptation of the Lion Capital atop the

Ashoka Pillar of Sarnath Uttar Pradesh

and is combined with the National

Motto Satyameva Jayate

The Rampurva Bull gets the name from

the site of its discovery Rampurva in

Bihar

It is noted for its delicately sculpted

model demonstrating superior

representation of soft flesh sensitive

nostrils alert ears and strong legs It is

a mixture of Indian and Persian

elements

Sankissa is situated in Uttar Pradesh

India

Q13) Ans(a)

Kunwar Singh was a notable leader during the Revolt of 1857 He belonged

to a royal house of Jagdispur Bihar

Q14) Answer b

Explanation

The term Vellalar was used for large

landowners

Q15) Answer c

Explanation

Arikamedu was a coastal settlement

where ships unloaded goods from

distant lands Finds here include a

massive brick warehouse pottery

including amphorae and Arretine ware

Roman lamps glassware and gems have

also been found at the site

Q16) Answer a

Explanation

Muvendar is a Tamil word mentioned in

Sangam poems meaning three chiefs

used for the heads of three ruling

families the Cholas Cheras and

Pandyas

Q17) Ans (c)

Several tribal or kin-based assemblies

such as the Sabha Vidatha and gana

are mentioned in the Rig-veda The

Sabha and the samiti mattered a great

deal in early Vedic times so much so

that the chiefs or the kings showed an

eagerness to win their support

Q18) Ans (a)

Jainism recognised the existence of the

gods but placed them lower than the

jina and did not condemn the varna

system as Buddhism did

Q19) Answer (d)

Explanation

Cholas and Pandyas had developed

powerful coastal cities The most

important city of Cholas was Puhar or

Kaveripattinam and Madurai was the

capital of Pandyas

Q20) Answer b

Explanation

Buddhacharita is the biography of

Buddha and was written by

RAUSIAS-FC19E1003 43

Ashvaghosha

Q21) Answer (a)

Explanation

Tamil poet Appar was a Shiva devotee

So he was a Nayanar saint

Q22) Answer d

Explanation

Samudragupta was a prominent Gupta

ruler whose coins depict him playing a

veena indicating his love for music We

get important historic information from

his Allahabad Prashasti which was

composed by his court poet Harisena

Q23) Answer (b)

Explanation

Vikrama Samvat was founded by

Chandragupta II in the 58 BC as a

mark of victory over the Shakas and

assumed the title of Vikramaditya

Banabhatta wrote Harshavardhanarsquos

biography the Harshacharita in

Sanskrit

Q24) Answer c

Explanation

Sandhi-vigrahika was the minister of

war and peace

Sarthavaha was the leader of the

merchant caravans

Q25) Answer a

Explanation

Xuan Zang (Hsuan-tsang) was a

Chinese traveller who came during the

reign of Harshavardhana

In the decade that began in 630 AD

Xuan Zang came to India through

Kashmir after visiting Central Asia Iran

and Afghanistan

He travelled from north to east and lived

in Bihar for a couple of years

At Nalanda University Xuan Zang

interacted with students and scholars

mastered local languages and

discovered Buddhist stupas

Q26) Answer c

Explanation

Pradakshina patha is a circular path

laid around a stupa in Buddhist

architecture While the rest are a part of

temple architecture

Q27) Answer d

Explanation

All the above-mentioned temples have

an elaborate use of bricks (baked

bricks) along with stone

Q28) Ans (c)

Muhammad Quli Qutab was the Sultan

of Golconda He was a contemporary of

Akbar was very fond of literature and

architecture

The Sultan was a great poet and he

wrote in Dakhini Urdu Persian and

Telgu and has left an extensive diwan or

collection

Recently the Archaeological Survey of

India (ASI) will be using Ground

Penetrating Radar (GPR) to map the

contours of the area around the Bagh-e-

Naya Qila excavated garden inside the

Golconda Fort in Telangana

Q29) Answer a

Explanation

Silappadikaram is a famous Tamil epic

which was written by Ilango around

1800 years ago It is a story of a

merchant named Kovalan who fell in

love with a courtesan named Madhavi

Manimekalai tells the story of the

daughter of Kovalan and Madhavi

Q30) Answer (a)

Explanation

Charaka is the author of Charaka

Samhita which is an important work of

Ayurveda and medicines

Brahmaguptarsquos fame rests mostly on his

Brahma-sphuta-siddhanta which was

an astronomical work It was translated

into Arabic in Baghdad and had a major

impact on Islamic mathematics and

astronomy

Late in his life Brahmagupta wrote

Khandakhadyaka which was an

RAUSIAS-FC19E1003 44

astronomical handbook that employed

Aryabhatarsquos system of starting each day

at midnight

Q31) Answer (c)

Explanation

Amir Khusrau was a famous sufi

musician poet and scholar In 1318 he

noted that there was different language

in every region of this land (Hindustan)

Lahori Kashmiri Dvarsamudri (in

Southern Karnataka) Telangana (in

Andhra Pradesh) Gujari (in Gujarat)

Marsquobari (in Tamil Nadu) Awadhi (in

eastern Uttar Pradesh) and Hindawai (in

the area around in Delhi) etc He went

to explain that Sanskrit did not belong

to any region and that only brahmans

knew it

Q32) Answer c

Explanation

Hiranyagarbha refers to the golden

womb When this ritual was performed

with the help of Brahmanas it was

thought to lead to the rebirth of the

sacrificer as a Khastriya

Q33) Answer d

Explanation

Kadamai refers to a tax on land

revenue

Gwalior Prashasti describes the exploits

of Nagabhata who was a Pratihara king

Q34) Answer b

Explanation

Rajatarangini is a Sanskrit text written

by Kalhana in the 12th century

It was historical chronicle of early India

It is justifiably considered to be the best

and most authentic work of its kind

It covers the entire span of history in

the Kashmir region from the earliest

times to the date of its composition

Q35) Answer c

Explanation

ldquoUrrdquo was the general assembly of the

village ldquoUrrdquo consisted of all the

taxpaying residents of an ordinary

village

Q36) Answer (a)

Explanation

Tarikh was a form of history writing in

the Delhi Sultanate The authors of

tawarikhs were learned men which

included secretaries administrators etc

Q37 Answer (a)

Explanation

Alauddin chose to pay his soldiers salaries in cash rather than iqtas The soldiers would buy their supplies from merchants in Delhi and it was thus feared that merchants would raise their prices To stop this Alauddin controlled the prices of goods in Delhi Prices were carefully surveyed by officers and merchants who did not sell at the prescribed rates were punished

Q38) Answer (d)

Explanation

Delhi first became the capital of a

kingdom under the Tomara Rajputs

who were defeated in the middle of the

twelfth century by the Chauhans (also

referred to as Chahamanas) of Ajmer

It was under the Tomaras and

Chauhans that Delhi became an

important commercial centre Many rich

Jaina merchants lived in the city and

constructed several temples Coins

minted here called dehliwal had a wide

circulation

Q39) Answer (c)

Explanation

Moth ki Masjid was built in the reign of

Sikandar Lodi by his minister

Begumpuri mosque built in the reign of

Muhammad Tughluq was the main

mosque of Jahanpanah the ldquoSanctuary

of the Worldrdquo and his new capital in

Delhi

Quwwat al ndash Islam mosque was

enlarged by Iltutmish and Alauddin

Khalji The minar was built by three

Sultansndash Qutbuddin Aybak Iltutmish

and Firuz Shah Tughluq

RAUSIAS-FC19E1003 45

Q40) Answer (c)

Explanation

Under the Mughals mansabdar was

referred to an individual who held a

mansab ie rank and he received his

salary as revenue assignments called

jagirs

Q41) Ans (b)

The Quit India Movement was a

spontaneous revolt of people against

British rule

The All India Congress Committee met

at Bombay on 8 August 1942 It passed

the famous resolution Quit India and

proposed the starting of a non-violent

mass struggle under Gandhis

leadership to achieve this aim But on

the very next day Gandhi and other

eminent leaders of the Congress were

arrested The Congress was once again

declared illegal

Q42) Ans (c)

The Simon Commission refers to a

group of seven MPs from the United

Kingdom constituted to suggest

constitutional reforms for British India

The Commission consisted of only

British members headed by one of the

senior British politicians Sir John

Simon

So the people of India agitated against

the arrival of Simon Commission

Q43) Ans (a)

He was widely known for his

unfavourable opinion of the economic

consequences of the British rule in

India

In his many writings and speeches and

especially in Poverty and Un-British

Rule in India Naoroji argued that India

was too highly taxed and that its wealth

was being drained away to England

He did not interpret the ancient Indian

texts and restored the self-confidence of

Indians And also he did not stress the

need for eradication of all the social

evils before anything else

Q44) Ans (c)

In August 1932 Prime Minister

MacDonald announced his Communal

Award Great Britainrsquos unilateral

attempt to resolve the various conflicts

among Indiarsquos many communal

interests

The award which was later

incorporated into the act of 1935

expanded the separate-electorate

formula reserved for Muslims to other

minorities including Sikhs Indian

Christians Anglo-Indians Europeans

distinct regional groups Gandhi

undertook a ldquofast unto deathrdquo against

that offer which he viewed as a

nefarious British plot to divide the

Indian society

Q45) Ans (b)

In British India apart from existing

imports and exports there was also a

particular amount of money which

colonial India contributed towards

administration maintenance of the

army war expenses pensions to retired

officers and other expenses accrued by

Britain towards maintenance of her

colony These were known as Home

charges and were paid for almost

entirely by India

The Home charges was made of

following components-

- Interest payable on Indian debt

- Dividend to shareholders of East

India Company

- Funds used to support the India

Office in London

- Funds used to pay salaries and

pensions of British personnel

engaged in India

- Interest on the railways

- Civil and military charges

- Store purchases in England

Q46) Ans (b)

The Lahore session of the Indian

National Congress was held in 1929

under the Presidentship of Jawaharlal

Nehru

The Lahore session of the Indian

National Congress witnessed significant

RAUSIAS-FC19E1003 46

developments in the Indian national

movement

- First the election of Jawaharlal

Nehru to the post of Presidentship of

the Congress was a clear indication

of the growing strength of the

Leftists in the Congress

- Secondly it was in this session that

the Congress for the first time raised

the demand for complete

independence Such demand was

not raised from the Congress

platform earlier

Q47) Ans (b)

It did not provide for separate

electorates for any community or

weightage for minorities However it did

allow for the reservation of minority

seats in provinces having minorities of

at least ten per cent but this was to be

in strict proportion to the size of the

community

There was no provision for complete

Independence for India

Q48) Ans (c)

The religion of early Vedic Aryans was

primarily of worship of nature and

Yajnas

The early Aryan religion was kind of

nature worship Actually the forces

around them which they could not

control or understand were invested

with divinity and were personified as

male or female gods And they

performed some Yajnas also

Q49) Ans (b)

The roads and river-routes were not

immune from robbery It is notable that

Yuan Chwang (Hiuen Tsang) was

robbed of his belongings during

Harshvardanarsquos period

Q50) Ans (c)

Q51) Ans (b)

Purandara Dasa was a saint and great

devotee of Lord Krishna

There is much speculation about where

Purandara Dasa regarded as the

Pitamaha of Carnatic music was born

Recently an expert committee

constituted by the Kannada University

Hampi has come to the conclusion that

Kshemapura Shivamogga district

Karnataka is the birth place of

Purandara Dasa

Q52) Ans (c)

Sri Tyagaraja Sri Shyama Shastry and Sri Muthuswami Dikshitar are considered the trinity of Carnatic music and with them came the golden age in Carnatic music in the 18th-19th

century

Q53) Ans d)

Recently a rare sarcophagus (stone

coffin) which is 2000 years old from the

Iron AgendashMegalithic era was discovered

from a rock-cut cave at Viyur village of

Kollam near Koyilandy in Kozhikode

district Kerala

The coffin containing bone fragments

was found during an excavation ldquoSo

far such a rare finding has been

discovered only from two sites

in Kerala Both these sarcophagi were

recovered from Megalithic sites at

Chevayur and Atholi also in Kozhikode

district

Q54) Ans a)

The megalithic culture in South India was a full-fledged Iron Age culture

Q55) Ans d)

The Cholas Pandyas and Keralaputras

(Cheras) mentioned in Ashokan

inscriptions were probably in the late

megalithic phase of material culture

Q56) Ans d)

Q57) Ans (b)

Raj Kumar Shukla followed Gandhiji all

over the country to persuade him to

come to Champaran to investigate the

problem associated with tinkathia

system

RAUSIAS-FC19E1003 47

Brij Kishore Rajendra Prasad Mahadev

Desai and Narhari Parikh accompanied

Gandhi ji during the Champaran

Satyagraha

Q58) Ans (b)

The Satvahanas started the practice of granting tax-free villages to brahmanas and Buddhist monks

Q59) Ans c)

The objectives of the Programme are

listed as under

- Developing basic tourism

infrastructure

- Promoting cultural and heritage

value of the country to generate

livelihoods in the identified regions

- Enhancing the tourist attractiveness

in a sustainable manner by

developing world-class

infrastructure at the heritage

monument sites

- Creating employment through active

involvement of local communities

- Harnessing tourism potential for its

effects on employment generation

and economic development

- Developing sustainable tourism

infrastructure and ensuring proper

Operations and maintenance

therein

Q60) Ans (b)

The Tribal Cooperative Marketing

Development Federation of India

(TRIFED) came into existence in 1987

It is a national-level apex organization

functioning under the administrative

control of Ministry of Tribal Affairs

Govt of India

TRIFED has its registered and Head

Office located in New Delhi

Q61) Ans (c)

Premchandrsquos novels include

Premashram Rangabhumi Ghaban

Karmabhumi and Godan

Gora is a novel written by Rabindranath

Tagore

138th birth anniversary of Munshi

Premchand was celebrated across the

country

Q62) Ans (b)

Giddha is a traditional pastoral dance

performed by the women of the Punjab

India and Pakistan at festival times

and at the sowing and reaping of the

harvest

By this dance the Punjabi women

reveal their joy expel their suppressed

feelings in a male dominated society

through the performance of Giddha

Since this dance has nothing to do with

men only women can participate in it

During the Teej celebrations Giddha

dance is celebrated in Punjab every

year Teej is a generic name for a

number of festivals that are celebrated

by women in some parts of India

Q63) Ans (a)

Dara Shukoh wrote the remarkable

work called ldquoMajma-ul-Bahrainrdquo or the

ldquoThe confluence of two seasrdquo

The Vice President of India Shri M

Venkaiah Naidu has said that Prince

Dara Shukohrsquos writings can come as a

refreshing source for infusing peace and

harmony He was addressing the

gathering after visiting the exhibition

that showcases the forgotten Prince of

yesteryears Dara Shukoh organized by

Mr Francois Gautier at Indira Gandhi

National Centre for the Arts in New

Delhi

Q64) Ans (c)

The statue Gommateshwara is

dedicated to the Jain God Bahubali

It is a monolithic statue

President Ram Nath Kovind

inaugurated the grand anointing

ceremony mdash Mahamastakabhisheka mdash

held once in 12 years at

Shravanabelagola (Karnataka)

Q65) Ans (c)

Prachi Valley had come up around the

Prachi river Prachi Valley gradually

disappeared

RAUSIAS-FC19E1003 48

The Prachi river originates from

Bhubaneswar

It is a tributary of the Mahanadi and

flows through the districts of Puri

Khurda Cuttack and Jagatsinghpur

and the entire region of the river is

termed as the Prachi Valley

It falls into the Bay of Bengal

Archaeological evidence shows that the

Prachi Valley Civilisation predates both

Harappa and Mohenjo-Daro

The Prachi river originates from

Bhubaneswar

Q66) Ans (d)

These monuments are located in

Chhatarpur district Madhya Pradesh

within Vindhya mountain range

Q67) Ans (a)

The book lsquoThoughts on Pakistanrsquo was

written by Dr BR Ambedkar

On the occasion of the birth anniversary

of Dr BR Ambedkar the president of

India pays homage to this icon of India

In 1924 he founded the Depressed

Classes Institute (Bahishkrit Hitkarini

Sabha) and in 1927 the Samaj Samata

Sangh

Another area of attention for Ambedkar

was education For its spread among

the low classes he set up a network of

colleges by the name of Peoples

Education Society and founded hostels

Q68) Ans(b)

Mehrgarh is a famous Neolithic

settlement in the Indian subcontinent

which is situated in Baluchistan

province Pakistan

A pre-historic rock art site is discovered

in the vast expanse of limestone blocks

on the eastern banks of Naguleru river

near Dachepalli (Andhra Pradesh) It

has thrown light on the Neolithic

civilisation that flourished in Guntur

(Andhra Pradesh) during 1500-2000

BC

Q69) Ans (c)

The 12th and the 13th centuries saw

the emergence of the Kakatiyas They

were at first the feudatories of the

Western Chalukyas of Kalyana Initially

they ruled over a small territory near

Warangal (Telangana)

They introduced Nayakships which was

later adopted and developed by the

Rayas of Vijayanagara

Q70) Ans (a)

The fast had effect of putting pressure

on mill owners who finally agreed to

give the workers a 35 per cent increase

in wages

Google celebrated with a doodle the

132nd birth anniversary of Anasuya

Sarabhai who played a pioneering role

in Indiarsquos labour movement

Q71) Ans (d)

The UNESCOrsquos list of the representative

list of the intangible cultural heritage of

humanity from India are

- Koodiyattam Sanskrit Theatre of

Kerala

- Mudiyettu ritual theatre and dance

drama of Kerala

- Tradition of Vedic Chanting

- Kalbelia folk songs and dances of

Rajasthan

- Ramlila Traditional Performance of

the Ramayana

- Sankirtana ritual singing

drumming and dancing of Manipur

- Ramman religious festival and

ritual theatre of the Garhwal

Himalayas India

- Traditional brass and copper craft of

utensil making among the Thatheras

of Jandiala Guru Punjab India

- Chhau dance classical Indian dance

originated in the eastern Indian

states

- Buddhist chanting of Ladakh

recitation of sacred Buddhist texts

in the trans-Himalayan Ladakh

region Jammu and Kashmir India

- Yoga

- Nouroz

- Kumbh Mela

RAUSIAS-FC19E1003 49

Q72) Ans(b)

The President of India Shri Ram Nath Kovind inaugurated the Hornbill Festival and State Formation Day celebrations of Nagaland in Kisama

The festival is named after the Indian hornbill the large and colourful forest bird which is displayed in the folklore of most of the states tribes

The major recognized tribes of Nagaland are Angami Ao Chakhesang Chang

Kuki Rengma and Zeling etc

Onge Jarawa and Sentinelese are the

tribes of Andman amp Nicobar Islands

Q73) Ans (c)

The Rashtrakutas rule in the Deccan lasted for almost two hundred years till the end of the tenth century The Rashtrakutas rulers were tolerant in their religious views and patronized not only Shaivism and Vaishnavism but

Jainism as well

The famous rock-cut temple of Shiva at Ellora was built by one of the Rashtrakutas kings Krishna I in the ninth century His successor Amoghavarsha was a Jain but he also

patronized other faiths

The Rashtrakutas allowed Muslims traders to settle and permitted Islam to

be preached in their dominions

Recently increasing defacement at the prehistoric rock paintings of Pandavulagutta Telangana has created a cause for grave concern It can spoil

the prehistoric rock

Pandavulagutta is home to

- Painted rock shelters dating to

10000 BC-8000 BC

- An 8th century inscription of the

Rashtrakuta period and

- Painted frescoes from the 12th century Kakatiya empire

Q74) Ans (b)

In 1828 Raja Ram Mohan Roy founded a new religious society the Brahma Sabha later known as the Brahmo

Samaj

Debendranath Tagore headed the Tattvabodhini Sabha which was

engaged in search of spiritual truth

Its purpose was to purify Hinduism and to preach monotheism or belief in one God

The new society was to be based on the twin pillars of reason and the Vedas and

Upanishads

Recently Sadharan Brahmo Samaj (SBS) has entered into a legal battle with the West Bengal government due

to some legal issue

Q75) Ans (c)

The Chishti order was established in India by Khwaja Moinuddin Chishti who came to India around 1192 The Chishtirsquos are considered to be the most influential of the groups of Sufis who migrated to India in the late twelfth century They adapted successfully to the local environment and adopted several features of Indian devotional

traditions

The historical dargah of Sufi mystic Khwaja Moinuddin Chishti in Ajmer is all set to get a facelift This 13 th century dargah has been included among the Swachh Iconic Places a clean-up initiative focused on iconic

heritage spiritual and cultural places

Page 7: GENERAL STUDIES (PAPER I) · Test is part of Rau’s IAS Test series for Preliminary Exam 2019 FOUNDATION + CURRENT AFFAIRS GENERAL STUDIES (PAPER –I) FOUNDATION TEST –III TOPIC:

RAUSIAS-FC19E1003 7

Q14) Consider the following pairs

1 Vellalar Soldier

2 Adimai Slaves

3 Kadaisiyar Landless labourers

Which of the pairs given above isare

correctly matched

(a) 1 and 2 only

(b) 2 and 3 only

(c) 1 and 3 only

(d) 1 2 and 3

Q15) Identify the site which was a famous

coastal settlement from which Arretine

ware has been found

(a) Mahabalipuram

(b) Tamralipti

(c) Arikamedu

(d) Atranjikhera

Q16) With respect to the Sangam literature

consider the following statements

1 The Sangamsassemblies were

held at Madurai

2 The term muvendar was used in

sangam poems which mean a

group of two kings

Which of the statements given above

isare correct

(a) 1 only

(b) 2 only

(c) Both 1 and 2

(d) Neither 1 nor 2

Q17) Consider the following statements

1 During Early Vedic Period women

attended the Sabha and Vidatha

2 The priest ndash Vasishtha played

important role in Early Vedic

Period

Which of the statements given above

isare correct

(a) 1 only

(b) 2 only

(c) Both 1 and 2

(d) Neither 1 nor 2

Q18) Consider the following statements

1 The spread of Jainism in

Karnataka is attributed to

Chandragupta Maurya

2 Jainism recognised the existence

of the gods and condemns the

Varna System

Which of the statements given above

isare correct

(a) 1 only

(b) 2 only

(c) Both 1 and 2

(d) Neither 1 nor 2

Q19) Consider the following pairs

1 Puhar Pandyas

2 Madurai Cholas

Which of the pairs given above isare

correct

(a) 1 only

(b) 2 only

(c) Both 1 and 2

(d) Neither 1 nor 2

RAUSIAS-FC19E1003 8

Q20) निमननिखित कथि ो म स कौि-सास सही हह

1 बदधचररत का ििक िागसि ह

2 ब नरसतव की पजा महायाि बौदधमत का एक

महतवपरण भाग थी

िीच नदए गए कट का परय ग कर सही उततर चनिए

(a) किि 1

(b) किि 2

(c) 1 और 2 द ि ो

(d) ि त 1 ि ही 2

Q21) निमननिखित कथि ो म स कौि-सास सही हह

1 भखकत क निचार क भागित गीता म सपषट

नकया गया ह

2 तनमि कनि अपपर एक अििर सोत थ

िीच नदए गए कट का परय ग कर सही उततर चनिएः

(a) किि 1

(b) किि 2

(c) 1 और 2 द ि ो

(d) ि त 1 ि ही 2

Q22) निमननिखित म स नकस शासक ि िीरा बजात हए

अपिी छनि नसक ो पर अोनकत करिाई थी

(a) नमिाोदर

(b) चनदरगपत मौयण

(c) गौतमीपतर सतकरी

(d) समदरगपत

Q23) निमननिखित कथि ो म स कौि-सास सही हह

1 सवतोतर भारत ि निकरम सोित क राषटर ीय

किनडर क रप म अपिाया और यह 68 ईसा

पिण म आरि हआ था

2 बािभटट हिणिरणि क दरबार का एक कनि था

िीच नदए गए कट का परय ग कर सही उततर चनिएः

(a) किि 1

(b) किि 2

(c) 1 और 2 द ि ो

(d) ि त 1 ि ही 2

Q24) निमननिखित यग ो पर निचार कीनजएः

1 सोनर-निगरानहका वयापार मोतरी

2 परथम-कनिका परमि नशलपकार

3 साथणिाह परमि बकर

उपयणकत यग ो म स कौि-सास सही समनित हह

(a) किि 1

(b) किि 1 और 3

(c) किि 2

(d) किि 2 और 3

Q25) निमननिखित कथि ो म स कौि-सास सही हह

1 lsquoएह ि नशिाििrsquo पिकनशि नदवतीय स

सोबोनरत ह और रनिकनत क दवारा इसकी रचिा

की गई थी

2 जआि झाोग समदरगपत नदवतीय क शासिकाि

म भारत आया था

िीच नदए गए कट का परय ग कर सही उततर चनिएः

(a) किि 1

(b) किि 2

(c) 1 और 2 द ि ो

(d) ि त 1 ि ही 2

RAUSIAS-FC19E1003 9

Q20) Which of the following statements

isare correct

1 Buddhacharita is authored by

Nagasena

2 The worship of Bodhisattvas was

an important part of Mahayana

Buddhsim

Select the correct answer using the code

given below

(a) 1 only

(b) 2 only

(c) Both 1 and 2

(d) Neither 1 nor 2

Q21) Which of the following statements

isare correct

1 The idea of Bhakti is elucidated in

Bhagavata Gita

2 Tamil poet Appar was an Alvar

saint

Select the correct answer using the code

given below

(a) 1 only

(b) 2 only

(c) Both 1 and 2

(d) Neither 1 nor 2

Q22) Which of the following rulers had his

image inscribed in the coins while

playing a veena

(a) Meander

(b) Chandragupta Maurya

(c) Gautamiputra Satkarni

(d) Samudragupta

Q23) Which of the following statements

isare correct

1 Vikrama Samvat is adopted as the

national calendar by independent

India and it began in 68 BC

2 Banabhatta was a court poet of

Harshavardhana

Select the correct answer using the code

given below

(a) 1 only

(b) 2 only

(c) Both 1 and 2

(d) Neither 1 nor 2

Q24) Consider the following pairs

1 Sandhi-vigrahika Minister of trade

2 Prathama-kulika Chief craftsman

3 Sarthavaha Chief banker

Which of the pairs given above isare

correct

(a) 1 only

(b) 1 and 3 only

(c) 2 only

(d) 2 and 3 only

Q25) Which of the following statements

isare correct

1 Aihole inscription belongs to

Pulakeshin II and was composed

by Ravikriti

2 Xuan Zang came to India during

the reign of Chandragupta II

Select the correct answer using the code

given below

(a) 1 only

(b) 2 only

(c) Both 1 and 2

(d) Neither 1 nor 2

RAUSIAS-FC19E1003 10

Q26) िासतकिा स सोबोनरत निमननिखित ततव ो म स कौि-स

ततव किि नहोद मखनदर ो की िासतकिा क भाग ह

1 नशिर

2 मणडप

3 परदनकषरा पथ

4 गभणगह

िीच नदए गए कट का परय ग कर सही उततर चनिएः

(a) किि 1 3 और 4

(b) किि 2 3 और 4

(c) किि 1 2 और 4

(d) 1 2 3 और 4

Q27) निमननिखित मोनदर ो म स कौि-स मोनदर ईोट ो स बि ह

1 दिगढ़ मोनदर

2 भीतरगाि मोनदर

3 िकषमर मोनदर नसरपर

4 बहदशवर मोनदर

िीच नदए गए कट का परय ग कर सही उततर चनिएः

(a) किि 1 2 और 3

(b) किि 2 3 और 4

(c) किि 1 3 और 4

(d) 1 2 3 और 4

Q28) निमननिखित कथि ो म स कौि-सास सही हह

1 सलताि महममद किी कतब शाह अकबर का

समकािीि था

2 िासतकिा क कषतर म महममद किी कतब शाह

ि कई ईमारत ो का निमाणर करिाया था नजिम

स चार मीिार सिाणनरक परनसदध ह

िीच नदए गए कट का परय ग कर सही उततर चनिएः

(a) किि 1

(b) किि 2

(c) 1 और 2 द ि ो

(d) ि त 1 ि ही 2

Q29) निमननिखित यग ो पर निचार कीनजएः

1 मनिमकिाई सततिार

2 अनभजञाि शाको तिम कानिदास

3 नसिपपानदकारम क ििि

उपयणकत यग ो म स कौि-सास सही समनित हह

(a) किि 1 और 2

(b) किि 2

(c) किि 1 और 3

(d) 1 2 और 3

Q30) निमननिखित कथि ो म स कौि-सास सही हह

1 ldquoसशरत सोनहताrdquo नचनकरता पर एक महतवपरण

रचिा ह

2 बरहमगपत और चरक महतवपरण गनरतजञ थ

िीच नदए गए कट का परय ग कर सही उततर चनिए

(a) किि 1

(b) किि 2

(c) 1 और 2 द ि ो

(d) ि त 1 ि ही 2

Q31) अमीर िसर क सनदभण म निमननिखित कथि ो म स

कौि-सास सही हह

1 अमीर िसर ि अपिी रचिाओो म नििा ह

नक सोसकत नकसी भी कषतर स सोबोनरत िही ो थी

और किि बराहमर ही इस भािा का जञाि रित

2 उन ोि नहोदिी और अिरी क अखसततव का

उललि नकया था

िीच नदए गए कट का परय ग कर सही उततर चनिए

(a) किि 1

(b) किि 2

(c) 1 और 2 द ि ो

(d) ि त 1 ि ही 2

RAUSIAS-FC19E1003 11

Q26) Which of the following architectural

elements were only part of Hindu

temple architecture

1 Shikhara

2 Mandapa

3 Pradakshina patha

4 Garbhagriha

Select the correct answer using the code

given below

(a) 1 3 and 4 only

(b) 2 3 and 4 only

(c) 1 2 and 4 only

(d) 1 2 3 and 4

Q27) Which of the following temples isare

made of bricks

1 Deogarh Temple

2 Bhitargaon Temple

3 Lakshmana temple Sirpur

4 Brihadeshvara Temple

Select the correct answer using the code

given below

(a) 1 2 and 3 only

(b) 2 3 and 4 only

(c) 1 3 and 4 only

(d) 1 2 3 and 4

Q28) Which of the following statements

isare correct

1 Sultan Muhammad Quli Qutab

Shah was a contemporary of

Akbar

2 In the field of architecture

Muhammad Quli Qutab Shah

constructed many buildings the

most famous of which is the Char

Minar

Select the correct answer using the code

given below

(a) 1 only

(b) 2 only

(c) Both 1 and 2

(d) Neither 1 nor 2

Q29) Consider the following pairs

1 Manimekalai Sattanar

2 Abhijnana Shakuntalam Kalidasa

3 Silappadikaram Kovalan

Which of the pairs given above isare

correct

(a) 1 and 2 only

(b) 2 only

(c) 1 and 3 only

(d) 1 2 and 3

Q30) Which of the following statements

isare correct

1 Sushruta Samhita is an important

work on medicine

2 Brahmagupta and Charaka were

important mathematicians

Select the correct answer using the code

given below

(a) 1 only

(b) 2 only

(c) Both 1 and 2

(d) Neither 1 nor 2

Q31) Which of the following statements

isare correct about Amir Khusrau

1 Amir Khusrau records in his works

that Sanskrit did not belong to any

region and only the Brahmans

knew it

2 He recorded the existence of

Hindawi and Awadhi

Select the correct answer using the code

given below

(a) 1 only

(b) 2 only

(c) Both 1 and 2

(d) Neither 1 nor 2

RAUSIAS-FC19E1003 12

Q32) निमननिखित कथि ो पर निचार कीनजए

1 नहरणय-गभण अिषठाि क बार म ऐसा स चा जाता

था नक बनि दि िाि का एक कषनतरय क रप म

पिजणनम ह गा

2 मयरशमणि कदोब िोश का सोसथापक था

उपयणकत कथि ो म स कौि-सास सही हह

(a) किि 1

(b) किि 2

(c) 1 और 2 द ि ो

(d) ि त 1 ि ही 2

Q33) निमननिखित कथि ो म स कौि-सास सही हह

1 कदमई बगार (बिपिणक शरम) क रप म

निया जाि िािा कर था

2 गवानियर परशखसत म िागभट (ज एक चोदि

राजा था) क दवारा नकय गए श िर का िरणि

नकया गया ह

िीच नदए गए कट का परय ग कर सही उततर चनिए

(a) किि 1

(b) किि 2

(c) 1 और 2 द ि ो

(d) ि त 1 ि ही 2

Q34) निमननिखित कथि ो म स कौि-सास सही हह

1 राजतरो नगिी 11िी ो शताबदी म कलहि क दवारा

रनचत एक सोसकत पसतक (टकसट) ह

2 कननौज क निए नतरपकषीय सोघिण म पाि राजिोश

शानमि था

िीच नदए गए कट का परय ग कर सही उततर चनिए

(a) किि 1

(b) किि 2

(c) 1 और 2 द ि ो

(d) ि त 1 ि ही 2

Q35) निमननिखित यग ो पर निचार कीनजए

1 बरहदशवर मोनदर राजराजा च ि

2 उर मापि की इकाई

3 दिदाि मोनदर ो क भनम अिदाि

उपयणकत यग ो म स कौि-स सही समनित ह

(a) किि 1 और 2

(b) किि 2 और 3

(c) किि 1 और 3

(d) 1 2 और 3

Q36) निमननिखित कथि ो म स कौि-सास सही हह

1 नदलली क सलताि ो क अरीि परशासि की भािा

फारसी थी

2 नदलली सलतित म ldquoतारीितािरीिrdquo कनिता

का एक रप था

िीच नदए गए कट का परय ग कर सही उततर चनिए

(a) किि 1

(b) किि 2

(c) 1 और 2 द ि ो

(d) ि त 1 ि ही 2

Q37) निमननिखित कथि ो म स कौि-सास सही हह

1 अिाउददीि खििजी ि अपि सनिक ो क निए

नसरी िाम का एक िया दगणरकषक शहर

बिािाया था

2 िह अपि सनिक ो क िति का भगताि इकता

क रप म करता था

िीच नदए गए कट का परय ग कर सही उततर चनिए

(a) किि 1

(b) किि 2

(c) 1 और 2 द ि ो

(d) ि त 1 ि ही 2

RAUSIAS-FC19E1003 13

Q32) Consider the following statements

1 Hiranya-garbha ritual was thought

to lead to the rebirth of the

sacrificer as a Kshatriya

2 Mayurasharman was the founder

of the Kadamba dynasty

Which of the statements given above

isare correct

(a) 1 only

(b) 2 only

(c) Both 1 and 2

(d) Neither 1 nor 2

Q33) Which of the following statements

isare correct

1 Kadamai was tax taken in form of

forced labour

2 Gwalior Prashasti describes the

exploits of Nagabhata who was a

Chandella king

Select the correct answer using the code

given below

(a) 1 only

(b) 2 only

(c) Both 1 and 2

(d) Neither 1 nor 2

Q34) Which of the following statements

isare correct

1 Rajatarangini is a Sanskrit text

written by Kalhana in the 11th

century

2 Pala dynasty was included in the

tripartite struggle for Kannauj

Select the correct answer using the code

given below

(a) 1 only

(b) 2 only

(c) Both 1 and 2

(d) Neither 1 nor 2

Q35) Consider the following pairs

1 Brihadeshvara temple Rajaraja

Chola

2 ldquoUrrdquo Unit of measurement

3 Devadana Land grants made to

temples

Which of the pairs given above isare

correct

(a) 1 and 2 only

(b) 2 and 3 only

(c) 1 and 3 only

(d) 1 2 and 3

Q36) Which of the following statements

isare correct

1 The language of administration

under the Delhi Sultans was

Persian

2 Tarikhtawarikh was a form of

poetry in the Delhi Sultanate

Select the correct answer using the code

given below

(a) 1 only

(b) 2 only

(c) Both 1 and 2

(d) Neither 1 nor 2

Q37) Which of the following statements

isare correct

1 Alauddin Khilji constructed a new

garrison town named Siri for his

soldiers

2 He paid his soldiers their salaries

in the form of Iqta

Select the correct answer using the code

given below

(a) 1 only

(b) 2 only

(c) Both 1 and 2

(d) Neither 1 nor 2

RAUSIAS-FC19E1003 14

Q38) निमननिखित कथि ो म स कौि-सास सही हह

1 नदलली कतबददीि एबक क अरीि पहिी बार

नकसी सामराजय की राजरािी बिी थी

2 दहिीिाि नसक ो का मदरर मग़ि ो क दवारा

नकया गया था

िीच नदए गए कट का परय ग कर सही उततर चनिए

(a) किि 1

(b) किि 2

(c) 1 और 2 द ि ो

(d) ि त 1 ि ही 2

Q39) निमननिखित यग ो पर निचार कीनजए

1 म ठ की मखिद नसको दर ि दी

2 बगमपरी मखिद नफर ज शाह तगिक

3 कववत- अि - इसलाम कतबददीि ऐबक

उपयणकत यग ो म स कौि-स सही समनित ह

(a) किि 1 और 2

(b) किि 2 और 3

(c) किि 1 और 3

(d) 1 2 और 3

Q40) निमननिखित कथि ो म स कौि-सास सही हह

1 मिसबदार ो क अपिा िति राजसव कायो

नजन जागीर कहत थ क रप म परापत ह ता

था

2 मिसबदार क ज सनय उततरदानयतव सौोप जात

थ उसक अनतगणत उस एक निराणररत सखया म

सिार अथिा घड़सिार ो का रि-रिाि करिा

पड़ता था

िीच नदए गए कट का परय ग कर सही उततर चनिए

(a) किि 1

(b) किि 2

(c) 1 और 2 द ि ो

(d) ि त 1 ि ही 2

Q41) ldquo1942 क भारत छ ड़ आोद ििrdquo क बार म

निमननिखित अिि कि ो म स कौि-सा सतय िही ो ह

(a) यह एक अनहोसक आोद िि था

(b) इसका िततव महातमा गाोरी क दवारा नकया गया

था

(c) यह एक सवाभानिक आोद िि था

(d) इसि सामानयतया शरनमक िगण क आकनिणत

िही ो नकया था

Q42) भारत क ि ग ो ि ldquoसाइमि कमीशिrdquo क आगमि क

निरदध आोद िि नकया था कय ोनक

(a) भारतीय कभी भी 1919 क अनरनियम (The

Act of 1919) क काम की समीकषा िही ो करिा

चाहत थ

(b) साइमि कमीशि ि पराोत ो म दवर (द हर) शासि

क समापत करि की नसफाररश की थी

(c) साइमि कमीशि म क ई भारतीय सदसय िही ो

था

(d) साइमि कमीशि ि दश क निभाजि का

सझाि नदया था

Q43) निमननिखित कथि ो पर निचार कीनजए

भारतीय राषटर ीय आोद िि म दादाभाई िौर जी क दवारा

नकया गया सबस परभािी य गदाि यह था नक उन ोि

1 अोगरज ो क दवारा भारत क आनथणक श िर का

ििासा नकया था

2 पराचीि भारतीय गरोथ ो की वयाखया की थी और

भारतीय ो क आतमनिशवास क पिःसथानपत नकया

था

3 अनय नकसी भी बात स पहि सभी सामानजक

बराइय ो क उनमिि की आिशयकता पर बि

नदया था

उपयणकत कथि ो म स कौि-सास सही हह

(a) किि 1

(b) किि 2 और 3

(c) किि 1 और 3

(d) 1 2 और 3

RAUSIAS-FC19E1003 15

Q38) Which of the following statements

isare correct

1 Delhi first became the capital of a

kingdom under Qutubuddin

Aibak

2 Dehliwal coins were minted by the

Mughals

Select the correct answer using the code

given below

(a) 1 only

(b) 2 only

(c) Both 1 and 2

(d) Neither 1 nor 2

Q39) Consider the following pairs

1 Moth ki Masjid- Sikander Lodi

2 Begumpuri mosque- Firuz Shah

Tughluq

3 Quwwat al ndash Islam- Qutubuddin

Aibak

Which of the above pairs isare correct

(a) 1 and 2 only

(b) 2 and 3 only

(c) 1 and 3 only

(d) 1 2 and 3

Q40) Which of the following statements

isare correct

1 Mansabdars received their salaries

as revenue assignments called

jagirs

2 The mansabdarrsquos military

responsibilities required him to

maintain a specified number of

sawar or cavalrymen

Select the correct answer using the code

given below

(a) 1 only

(b) 2 only

(c) Both 1 and 2

(d) Neither 1 nor 2

Q41) Which one of the following observations

is not true about the Quit India

Movement of 1942

(a) It was a non-violent movement

(b) It was led by Mahatma Gandhi

(c) It was a spontaneous movement

(d) It did not attract the labour class

in general

Q42) The people of India agitated against the

arrival of the Simon Commission

because

(a) Indians never wanted the review of

the working of the Act of 1919

(b) Simon Commission recommended

the abolition of dyarchy in the

Provinces

(c) there was no Indian member in the

Simon Commission

(d) the Simon Commission suggested

the partition of the country

Q43) Consider the following statements

The most effective contribution made by

Dadabhai Naoroji to the cause of Indian

National Movement was that he-

1 exposed the economic exploitation

of India by the British

2 interpreted the ancient Indian

texts and restored the self-

confidence of Indians

3 stressed the need for eradication of

all the social evils before anything

else

Which of the statements given above

isare correct

(a) 1 only

(b) 2 and 3 only

(c) 1 and 3 only

(d) 1 2 and 3

RAUSIAS-FC19E1003 16

Q44) महातमा गाोरी ि 1932 म आमरर अिशि नकया था

कय ोनक

(a) ldquoग िमज सममििrdquo (The Round Table

Conference) भारतीय राजिीनतक

आकाोकषाओो क परा करि म असफि रहा था

(b) काोगरस और मखसलम िीग म मतभद थ

(c) रामस मकड िालड (Ramsay Macdonald)

ि ldquoसाोपरदानयक परसकारrdquo (The Communal

Award) की घ िरा की थी

(d) ldquoसनििय अिजञा आोद ििrdquo (The Civil

Disobedience Movement) असफि रहा

था

Q45) भारत म औपनििनशक शासि की अिनर क सोदभण म

भारत स रि क बनहगणमि का एक महतवपरण भाग गह

शलक (Home Charges) था निमननिखित म स

कौि-सास क ि गह शलक म सखममनित नकया गया

थानकय गए थ

1 िोदि म भारत कायाणिय क निए उपय ग नकय

जाि िािा क ि

2 भारत म नियकत नबरनटश कनमणय ो क िति और

पशि का भगताि करि क निए उपय ग नकय

जाि िािा क ि

3 अोगरज ो क दवारा भारत क बाहर यदध ो क निए

उपय ग नकय जाि िािा क ि

िीच नदए गए कट का परय ग कर सही उततर चनिए

(a) किि 1

(b) किि 1 और 2

(c) किि 2 और 3

(d) 1 2 और 3

Q46) सवतोतरता आोद िि क इनतहास म भारतीय राषटर ीय

काोगरस का 1929 का सतर महतवपरण ह कय ोनक इसम

(a) काोगरस क उददशय क रप म सथािीय सरकार

की पराखपत की घ िरा की गई थी

(b) परण सवराज की पराखपत क काोगरस क िकषय क

रप म अपिाया गया था

(c) असहय ग आोद िि शर नकया गया था

(d) िोदि म ldquoग ि मर सममििrdquo (The Round

Table Conference) म भाग िि का निरणय

निया गया था

Q47) भारतीय सवतोतरता सोगराम क सोदभण म िहर ररप टण

क दवारा निमननिखित म स नकसकी नसफाररश की गई

थीनकिकी नसफाररश की गई थी ो

1 भारत क निए परण सवतोतरता

2 अलपसोखयक ो क निए सीट ो क आरकषर क

निए सोयकत नििाणचक मोडि

3 सोनिराि म भारत क ि ग ो क निए मौनिक

अनरकार ो का परािराि

िीच नदए गए कट का परय ग कर सही उततर चनिए

(a) किि 1

(b) किि 2 और 3

(c) किि 1 और 3

(d) 1 2 और 3

Q48) आरो नभक िनदक आयो का रमण मखय रप स था

(a) भखकत

(b) मनतण पजा और यजञ

(c) परकनत की पजा और यजञ

(d) परकनत की पजा और भखकत

RAUSIAS-FC19E1003 17

Q44) Mahatma Gandhi undertook fast unto

death in 1932 mainly because

(a) The Round Table Conference failed

to satisfy Indian political

aspirations

(b) The Congress and Muslim League

had differences of opinion

(c) Ramsay Macdonald announced the

Communal Award

(d) The Civil Disobedience Movement

failed

Q45) With reference to the period of colonial

rule in India ldquoHome Chargesrdquo formed

an important part of drain of wealth

from India Which of the following funds

constituted ldquoHome Chargesrdquo

1 Funds used to support the India

Office in London

2 Funds used to pay salaries and

pensions of British personnel

engaged in India

3 Funds used for waging wars

outside India by the British

Select the correct answer using the code

given below

(a) 1 only

(b) 1 and 2 only

(c) 2 and 3 only

(d) 1 2 and 3

Q46) The 1929- Session of Indian National

Congress is of significance in the history

of the Freedom Movement because the-

(a) attainment of Self-Government

was declared as the objective of

the Congress

(b) attainment of Poorna Swaraj was

adopted as the goal of the

Congress

(c) Non-Cooperation Movement was

launched

(d) decision to participate in the

Round Table Conference in

London was taken

Q47) With reference to the period of Indian

freedom struggle which of the following

waswere recommended by the Nehru

report

1 Complete Independence for India

2 Joint electorates for reservation of

seats for minorities

3 Provision of fundamental rights for

the people of India in the

Constitution

Select the correct answer using the code

given below

(a) 1 only

(b) 2 and 3 only

(c) 1 and 3 only

(d) 1 2 and 3

Q48) The religion of the early Vedic Aryans was primarily of

(a) Bhakti

(b) image worship and Yajnas

(c) worship of nature and Yajnas

(d) worship of nature and Bhakti

RAUSIAS-FC19E1003 18

Q49) भारत की यातरा करि िाि चीिी यातरी यआि चिाोग

(हयएि साोग) ि समकािीि भारत की सामानय

खसथनतय ो और सोसकनत क दजण नकया था इस सोदभण म

निमननिखित कथि ो म स कौि-सास सही हह

1 सड़क और िदी-मागण (जि-मागण) डकती स

परण रप स सरनकषत थ

2 जहा तक अपरार ो क निए दणड की बात ह

उसक निए नकसी भी वयखकत की निदोिता

अथिा उसक अपरार क निराणररत करि क

निए अनि जि और निि परि क माधयम क

सारि थ

3 वयापाररय ो क घाट ो और परनतबोर सटशि ो पर

शलक ो का भगताि करिा पड़ता था

िीच नदए गए कट का परय ग कर सही उततर चनिए

(a) किि 1

(b) किि 2 और 3

(c) किि 1 और 3

(d) 1 2 और 3

Q50) नसोर घाटी सभयता क सोदभण म निमननिखित कथि ो पर

निचार कीनजए

1 यह मखय रप स एक रमणनिरपकष सभयता थी

तथा हािाोनक इसम रानमणक ततव मौजद था

िनकि िह परनतिश पर हािी िही ो था

2 इस काि क दौराि भारत म कपास का परय ग

कपड़ा बिाि क निए नकया जाता था

उपयणकत कथि ो म स कौि-सास सही हह

(a) किि 1

(b) किि 2

(c) 1 और 2 द ि ो

(d) ि त 1 ि ही 2

Q51) परोदर दास क सोदभण म निमननिखित कथि ो पर निचार

कीनजए

1 परोदर दास एक सोत और भगिाि नशि क एक

महाि भकत थ

2 ि एक सोगीतकार गायक और किाणटक सोगीत

क मखय सोसथापक-परसतािक ो म स एक थ

उपयणकत कथि ो म स कौि-सास सही हह

(a) किि 1

(b) किि 2

(c) 1 और 2 द ि ो

(d) ि त 1 ि ही 2

Q52) निमननिखित म स कौि-सास वयखकत किाणटक सोगीत

की नतरमनतण म शानमि हह

1 बािामरिी कषणा

2 शरी शयाम शासतरी

3 शरी मथसवामी दीनकषतर

िीच नदए गए कट का परय ग कर सही उततर चनिए

(a) किि 1

(b) किि 2

(c) किि 2 और 3

(d) 1 2 और 3

Q53) चियर (Chevayur) और अथ िी (Atholi) म खसथत

महापािार सथि निमननिखित म स नकस राजय म खसथत

(a) तनमििाड

(b) किाणटक

(c) पनिम बोगाि

(d) करि

RAUSIAS-FC19E1003 19

Q49) The Chinese traveller Yuan Chwang

(Hiuen Tsang) who visited India

recorded the general conditions and

culture of India at that time In this

context which of the following

statements isare correct

1 The roads and river-routes were

completely immune from robbery

2 As regards punishment for

offences ordeals by fire water and

poison were the instruments for

determining the innocence or guilt

of a person

3 The tradesmen had to pay duties

at ferries and barrier stations

Select the correct answer using the code

given below

(a) 1 only

(b) 2 and 3 only

(c) 1 and 3 only

(d) 1 2 and 3

Q50) Regarding the Indus Valley Civilization

consider the following statements

1 It was predominantly a secular

civilization and the religious

element though present did not

dominate the scene

2 During this period cotton was

used for manufacturing textiles in

India

Which of the statements given above

isare correct

(a) 1 only

(b) 2 only

(c) Both 1 and 2

(d) Neither 1 nor 2

Q51) Consider the following statements

regarding Purandara Dasa

1 Purandara Dasa was a saint and

great devotee of Lord Shiva

2 He was a composer singer and

one of the chief founding-

proponents of the Carnatic music

Which of the statements given above

isare correct

(a) 1 only

(b) 2 only

(c) Both 1 and 2

(d) Neither 1 nor 2

Q52) Which of the following persons isare

included in the trinity of Carnatic

music

1 Balamurali Krishna

2 Sri Shyama Shastry

3 Sri Muthuswami Dikshitar

Select the correct answer using the code

given below

(a) 1 only

(b) 2 only

(c) 2 and 3 only

(d) 1 2 and 3

Q53) Megalithic sites at Chevayur and Atholi

are located in which of the following

states

(a) Tamil Nadu

(b) Karnataka

(c) West Bengal

(d) Kerala

RAUSIAS-FC19E1003 20

Q54) निमननिखित कथि ो पर निचार कीनजए

1 महापािानरक ि ग कबर ो म िसतएो दफिात थ

2 दनकषर भारत म महापािार सोसकनत एक परण

निकनसत तामर यगीि सोसकनत थी

उपयणकत कथि ो म स कौि-सास सही हह

(a) किि 1

(b) किि 2

(c) 1 और 2 द ि ो

(d) ि त 1 ि ही 2

Q55) निमननिखित म स कौि-स सामराजयसामराजय ो का

अश क क अनभिि ो म उललि नकया गया ह

1 च ि

2 पाणडय

3 करिपतर (चर)

िीच नदए गए कट का परय ग कर सही उततर चनिए

(a) किि 1

(b) किि 1 और 2

(c) किि 3

(d) 1 2 और 3

Q56) भीमा-क रगाोि का यदध को पिी क सनिक ो और

बाजीराि नदवतीय क िततव म एक शखकतशािी पशिा

सिा (मराठ ो) क मधय िड़ा गया था यह यदध

निमननिखित म स नकसका नहससा था

(a) परथम आोगल-मराठा यदध का

(b) नदवतीय आोगल-मराठा यदध का

(c) ततीय आोगल-मसर यदध का

(d) ततीय आोगल-मराठा यदध का

Q57) निमननिखित कथि ो पर निचार कीनजए

1 महादि दसाई ि गाोरीजी क चोपारर आि तथा

नतिकनथया पररािी स जड़ी समसया की जाोच

क निए रारी करि क निए दश भर म उिका

अिसरर नकया था

2 िरहरी पाररि चोपारर सतयागरह क दौराि

गाोरीजी क साथ थ

उपयणकत कथि ो म स कौि-सास सही हह

(a) किि 1

(b) किि 2

(c) 1 और 2 द ि ो

(d) ि त 1 ि ही 2

Q58) निमननिखित कथि ो पर निचार कीनजए

1 िनद राज-िोश ि बराहमर ो और बौदध मठराररय ो

क कर-मकत गाि अिदाि म दि की परथा

आरि की थी

2 सतिाहि ो की आनरकाररक भािा पराकत थी

उपयणकत कथि ो म स कौि-सास सही हह

(a) किि 1

(b) किि 2

(c) 1 और 2 द ि ो

(d) ि त 1 ि ही 2

Q59) एक निरासत क अपिाइए (अडॉपट ए हररटज ndash

Adopt a Heritage) पररय जिा क उददशय ो क

सनदभण म निमननिखित कथि ो पर निचार कीनजए

1 यह पररय जिा र रगार उतपादि और आनथणक

निकास क निए पयणटि कषमता का उि पर

परभाि का उपय ग करगी

2 यह पररय जिा निरासत सथि ो पर निशव सतरीय

आराररक सोरचिा निकनसत करक एक सतत

तरीक स पयणटक आकिणर म िखदध करगी

उपयणकत कथि ो म स कौि-सास सही हह

(a) किि 1

(b) किि 2

(c) 1 और 2 द ि ो

(d) ि त 1 ि ही 2

RAUSIAS-FC19E1003 21

Q54) Consider the following statements

1 Megalithic people buried goods in

graves

2 The megalithic culture in South

India was a full-fledged Copper

Age culture

Which of the statements given above

isare correct

(a) 1 only

(b) 2 only

(c) Both 1 and 2

(d) Neither 1 nor 2

Q55) Which of the following kingdoms isare

mentioned in the Ashokan inscriptions

1 Cholas

2 Pandyas

3 Keralaputras (Cheras)

Select the correct answer using the code

given below

(a) 1 only

(b) 1 and 2 only

(c) 3 only

(d) 1 2 and 3

Q56) The Battle of Bhima-Koregaon was

fought between the soldiers of the

Company and the strong Peshwa army

(Marathas) under Bajirao II This war

was a part of the

(a) First Anglo-Maratha war

(b) Second Anglo-Maratha war

(c) Third Anglo- Mysore war

(d) Third Anglo-Maratha war

Q57) Consider the following statements

1 Mahadev Desai followed Gandhiji all over the country to persuade him to come to Champaran to investigate the problem associated

with tinkathia system

2 Narhari Parikh accompanied Gandhi ji during the Champaran

Satyagraha

Which of the statements given above isare correct

(a) 1 only

(b) 2 only

(c) Both 1 and 2

(d) Neither 1 nor 2

Q58) Consider the following statements

1 The Nanda Dynasty started the practice of granting tax-free villages to brahmanas and

Buddhist monks

2 The official language of the Satavahanas was Prakrit

Which of the statements given above

isare correct

(a) 1 only

(b) 2 only

(c) Both 1 and 2

(d) Neither 1 nor 2

Q59) Consider the following statements about the objectives of the lsquoadopt a heritagersquo

project

1 It will harness tourism potential for its effects on employment generation and economic

development

2 It will enhance the tourist attractiveness in a sustainable manner by developing world class infrastructure at heritage sites

Which of the statements given above

isare correct

(a) 1 only

(b) 2 only

(c) Both 1 and 2

(d) Neither 1 nor 2

RAUSIAS-FC19E1003 22

Q60) ldquoभारतीय जिजातीय सहकारी निपरि निकास सोघrdquo

(The Tribal Co-operative Marketing

Development Federation of India - TRIFED)

क सोदभण म निमननिखित कथि ो पर निचार कीनजए

1 यह एक राषटर ीय सतर का शीिण सोगठि ह ज

भारत सरकार क गह मोतरािय क परशासनिक

नियोतरर क अरीि काम कर रहा ह

2 इसका मखय उददशय दश म जिजातीय ि ग ो

का सामानजक-आनथणक निकास करिा ह

उपयणकत कथि ो म स कौि-सास सही हह

(a) किि 1

(b) किि 2

(c) 1 और 2 द ि ो

(d) ि त 1 ि ही 2

Q61) निमननिखित म स कौि-सास उपनयास परमचोद क

दवारा नििा गया हनिि गए ह

1 रोगभनम

2 ग दाि

3 ग रा

िीच नदए गए कट का परय ग कर सही उततर चनिए

(a) किि 1

(b) किि 2

(c) किि 1 और 2

(d) 1 2 और 3

Q62) नगदधा ितय क सोदभण म निमननिखित कथि ो पर निचार

कीनजए

1 नगदधा नबहार की मनहिाओो क दवारा तयौहार क

समय और फसि की बिाई तथा कटाई क

अिसर पर नकया जाि िािा एक पारोपररक

दहाती ितय ह

2 इस ितय क दवारा मनहिाऐो अपिी परसननता

परकट करती ह तथा नगदधा क परदशणि क

माधयम स परि िचणसव िाि समाज म

मनहिाओो की दबी हई भाििाओो क परकट

करती ह

उपयणकत कथि ो म स कौि-सास सही हह

(a) किि 1

(b) किि 2

(c) 1 और 2 द ि ो

(d) ि त 1 ि ही 2

Q63) निमननिखित कथि ो पर निचार कीनजए

1 मलला शाह बदखशी दारा नशक ह क

आधयाखतमक गर थ

2 औरोगरब ि मजम-उि-बहरीि या द समदर ो

का सोगम िामक उललििीय रचिा नििी थी

3 दारा नशक ह क अपि पिणज अकबर क गर ो

क उततरानरकारी क रप म दिा गया था

नजसम उसि रानमणक बहििाद और समनवयता

क बढ़ािा नदया था

उपयणकत कथि ो म स कौि-सास सही हह

(a) किि 1 और 3

(b) किि 2

(c) किि 1 और 2

(d) 1 2 और 3

RAUSIAS-FC19E1003 23

Q60) Consider the following statements about

the Tribal Cooperative Marketing

Development Federation of India

(TRIFED)

1 It is a national-level apex

organization functioning under the

administrative control of Ministry

of Home Affairs Government of

India

2 The main objective of TRIFED is

socio-economic development of

tribal people in the country

Which of the statements given above

isare correct

(a) 1 only

(b) 2 only

(c) Both 1 and 2

(d) Neither 1 nor 2

Q61) Which of the following novels isare

written by Premchand

1 Rangabhumi

2 Godan

3 Gora

Select the correct answer using the code

given below

(a) 1 only

(b) 2 only

(c) 1 and 2 only

(d) 1 2 and 3

Q62) Consider the following statements about

Giddha dance

1 Giddha is a traditional pastoral

dance performed by the women of

Bihar at festival times and at the

sowing and reaping of the harvest

2 By this dance the women reveal

their joy expel their suppressed

feelings in a male dominated

society through the performance of

Giddha

Which of the statements given above

isare correct

(a) 1 only

(b) 2 only

(c) Both 1 and 2

(d) Neither 1 nor 2

Q63) Consider the following statements

1 Mullah Shah Badakhshi was the

spiritual mentor of Dara Shukoh

2 Aurangzeb wrote the remarkable

work called ldquoMajma-ul-Bahrainrdquo or

the ldquoThe confluence of two seasrdquo

3 Dara Shukoh was seen as

inheriting the qualities of his

ancestor Akbar in that he

promoted religious pluralism and

syncretism

Which of the statements given above

isare correct

(a) 1 and 3 only

(b) 2 only

(c) 1 and 2 only

(d) 1 2 and 3

RAUSIAS-FC19E1003 24

Q64) निमननिखित कथि ो पर निचार कीनजए

1 ग मतशवर परनतमा निोधयनगरी पहाड़ी पर खसथत ह

2 शरिरबिग िा िह सथाि ह जहाो मौयण िोश क

सोसथापक चोदरगपत मौयण अपि नसोहासि क

तयागि क बाद जि तपसवी बि गए थ

उपयणकत कथि ो म स कौि-सास सही हह

(a) किि 1

(b) किि 2

(c) 1 और 2 द ि ो

(d) ि त 1 ि ही 2

Q65) निमननिखित कथि ो पर निचार कीनजए

1 पराताखतवक साकषय स पता चिता ह नक पराची

घाटी सभयता हड़पपा और म हिज दाड़ द ि ो

की पिणिती ह

2 पराची िदी भििशवर स निकिती ह

उपयणकत कथि ो म स कौि-सास सही हह

(a) किि 1

(b) किि 2

(c) 1 और 2 द ि ो

(d) ि त 1 ि ही 2

Q66) निमननिखित कथि ो म स कौि-सास सही हह

1 िजराह क समारक ो क समह का निमाणर

चोदि राजिोश क शासिकाि क दौराि हआ

था

2 य समारक हररिोदर पिणत शरोििा म खसथत ह

3 म रक क यातरी इबन बतता ि अपि सोसमरर ो

म िजराह क मोनदर ो की यातरा का उललि

नकया था तथा इन काजराण िाम स समब नरत

नकया था

िीच नदए गए कट का परय ग कर सही उततर चनिए

(a) किि 1

(b) किि 1 और 2

(c) किि 2 और 3

(d) किि 1 और 3

Q67) निमननिखित कथि ो म स कौि-सास सही हह

1 डॉ बी आर अमबडकर ि दी एनिनहिशि

ऑफ़ कासट (The Annihilation of Caste)

नििी थी नजसम उन ोि नहोद रमण म िोशािगत

पजारी की परथा क उनमिि की आिशयकता

पर बि नदया था

2 डॉ राजदर परसाद ि थॉटस ऑि पानकसताि

(Thoughts on Pakistan) िामक पसतक

नििी थी

िीच नदए गए कट का परय ग कर सही उततर चनिए

(a) किि 1

(b) किि 2

(c) 1 और 2 द ि ो

(d) ि त 1 ि ही 2

Q68) निमननिखित कथि ो म स कौि-सास सही हह

1 महरगढ़ भारतीय उपमहादवीप म एक परनसदध

ििपािार बसती ह ज नसोर पराोत पानकसताि म

खसथत ह

2 बरणह म म कतत ो क उिक सवामी क साथ कबर ो

म दफिाया जाता था

िीच नदए गए कट का परय ग कर सही उततर चनिए

(a) किि 1

(b) किि 2

(c) 1 और 2 द ि ो

(d) ि त 1 ि ही 2

Q69) निमननिखित कथि ो म स कौि-सास सही हह

1 काकानटय मोनदर अनरकतर नशि क समनपणत

2 हिमक ोडा म हजार-सतोभ िाि मोनदर (The

Thousand-Pillared Temple) का निमाणर

काकानटय समराट रदर ि करिाया था

िीच नदए गए कट का परय ग कर सही उततर चनिए

(a) किि 1

(b) किि 2

(c) 1 और 2 द ि ो

(d) ि त 1 ि ही 2

RAUSIAS-FC19E1003 25

Q64) Consider the following statements

1 Gommateshwara Statue is located

on the Vindyagiri Hill

2 Shravanabelagola is the place

where Chandragupta Maurya the

founder of the Mauryan dynasty

became a Jain ascetic after

relinquishing his throne

Which of the statements given above

isare correct

(a) 1 only

(b) 2 only

(c) Both 1 and 2

(d) Neither 1 nor 2

Q65) Consider the following statements

1 Archaeological evidence shows

that the Prachi Valley Civilisation

predates both Harappa and

Mohenjo-Daro

2 The Prachi river originates from

Bhubaneswar

Which of the statements given above

isare correct

(a) 1 only

(b) 2 only

(c) Both 1 and 2

(d) Neither 1 nor 2

Q66) Which of the following statements

isare correct

1 The Khajuraho group of

monuments was built during the

rule of the Chandela dynasty

2 These monuments are located in

Harischandra mountain range

3 Ibn Battuta the Moroccan

traveller in his memoirs mentioned

visiting Khajuraho temples and

called them Kajarra

Select the correct answer using the code

given below

(a) 1 only

(b) 1 and 2

(c) 2 and 3

(d) 1 and 3

Q67) Which of the following statements

isare correct

1 Dr BR Ambedkar wrote the

Annihilation of Caste emphasising

the need to do away with the

practice of hereditary priesthood in

Hinduism

2 The book lsquoThoughts on Pakistanrsquo

was written by Dr Rajendra

Prasad

Select the correct answer using the code

given below

(a) 1 only

(b) 2 only

(c) Both 1 and 2

(d) Neither 1 nor 2

Q68) Which of the following statements

isare correct

1 Mehrgarh is a famous Neolithic

settlement in the Indian

subcontinent which is situated in

Sindh province Pakistan

2 At Burzahom dogs were buried

with their masters in their graves

Select the correct answer using the code

given below

(a) 1 only

(b) 2 only

(c) Both 1 and 2

(d) Neither 1 nor 2

Q69) Which of the following statements

isare correct

1 The Kakatiya temples are

dedicated mostly to Siva

2 The Thousand-Pillared Temple at

Hanamkonda was built by the

Kakatiya king Rudra

Select the correct answer using the code

given below

(a) 1 only

(b) 2 only

(c) Both 1 and 2

(d) Neither 1 nor 2

RAUSIAS-FC19E1003 26

Q70) निमननिखित कथि ो म स कौि-सास सही हह

1 अहमदाबाद नमि हड़ताि क दौराि महातमा

गाोरी ि शरनमक ो क पकष क मजबत करि क

निए आमरर अिशि नकया था

2 अिशि स नमि मानिक ो पर दबाि पड़ा था ज

अोततः शरनमक ो क िति म 15 परनतशत की िखदध

करि क निए सहमत हए थ

िीच नदए गए कट का परय ग कर सही उततर चनिए

(a) किि 1

(b) किि 2

(c) 1 और 2 द ि ो

(d) ि त 1 ि ही 2

Q71) निमननिखित म स नकसक नकिक भारत स यिसक

की माििता की अमतण साोसकनतक निरासत की

परनतनिनर सची (The UNESCOrsquos List of the

Representative List of the Intangible

Cultural Heritage of Humanity) म शानमि

नकया गया ह

1 मनडयटट

2 सोकीतणि

3 को भ मिा

िीच नदए गए कट का परय ग कर सही उततर चनिए

(a) किि 1 और 2

(b) किि 2 और 3

(c) किि 3

(d) 1 2 और 3

Q72) निमननिखित जिजानतय ो म स कौि-सीसी ो

जिजानतजिजानतया िागािड स सोबोनरत हह

1 अोगामी

2 ककी

3 जारिा

िीच नदए गए कट का परय ग कर सही उततर चनिए

(a) किि 1

(b) किि 1 औऔ 2

(c) किि 2

(d) 1 2 और 3

Q73) निमननिखित कथि ो म स कौि-सास सही हह

1 राषटर कट सामराजय की सथापिा दोनतदगण ि की थी

नजसि मानयाित म अपिी राजरािी की

सथापिा की थी

2 राषटर कट समराट अम घििण एक ििक था और

उस कनिताओो पर पहिी कननड़ पसतक नििि

का शरय नदया जाता ह

िीच नदए गए कट का परय ग कर सही उततर चनिए

(a) किि 1

(b) किि 2

(c) 1 और 2 द ि ो

(d) ि त 1 ि ही 2

Q74) निमननिखित कथि ो म स कौि-सास सही हह

1 कशब चोदर सि ि ततवब नरिी सभा की

अधयकषता की थी ज आधयाखतमक सतय की

ि ज म सोिि थी

2 बरहम समाज ि मािि गररमा पर बि नदया

मनतणपजा का निर र नकया और सती परथा जसी

सामानजक बराइय ो की आि चिा की

िीच नदए गए कट का परय ग कर सही उततर चनिए

(a) किि 1

(b) किि 2

(c) 1 और 2 द ि ो

(d) ि त 1 ि ही 2

Q75) निमननिखित कथि ो म स कौि-सास सही हह

1 भारत म नचशती नसिनसिा खवाजा म इिददीि

नचशती क दवारा सथानपत नकया गया था

2 नचशती परोपरा की एक परमि निशिता

आतमसोयम थी नजसम साोसाररक म ह स दरी

बिाए रििा शानमि था

िीच नदए गए कट का परय ग कर सही उततर चनिए

(a) किि 1

(b) किि 2

(c) 1 और 2 द ि ो

(d) ि त 1 ि ही 2

RAUSIAS-FC19E1003 27

Q70) Which of the following statements

isare correct

1 During the Ahmedabad Mill Strike

Mahatma Gandhi undertook a fast

unto death to strengthen the

workersrsquo resolve

2 The fast had effect of putting

pressure on mill owners who

finally agreed to give the workers a

15 per cent increase in wages

Select the correct answer using the code

given below

(a) 1 only

(b) 2 only

(c) Both 1 and 2

(d) Neither 1 nor 2

Q71) Which of the following are included in

the UNESCOrsquos list of the representative

list of the intangible cultural heritage of

humanity from India

1 Mudiyettu

2 Sankirtana

3 Kumbh Mela

Select the correct answer using the code

given below

(a) 1 and 2 only

(b) 2 and 3 only

(c) 3 only

(d) 1 2 and 3

Q72) Which of the following tribes isare

related to Nagaland

1 Angami

2 Kuki

3 Jarawa

Select the correct answer using the code

given below

(a) 1 only

(b) 1 and 2 only

(c) 2 only

(d) 1 2 and 3

Q73) Which of the following statements

isare correct

1 Rashtrakuta kingdom was founded by Dantidurga who established his capital at Manyakhet

2 Amoghavarsha a Rashtrakuta king was an author and is credited with writing the first

Kannada book on poetics

Select the correct answer using the code given below

(a) 1 only

(b) 2 only

(c) Both 1 and 2

(d) Neither 1 nor 2

Q74) Which of the following statements isare correct

1 Keshab Chandra Sen headed the Tattvabodhini Sabha which was engaged in search of spiritual truth

2 The Brahmo Samaj laid emphasis on human dignity opposed idolatry and criticized such social

evils as the practice of Sati

Select the correct answer using the code given below

(a) 1 only

(b) 2 only

(c) Both 1 and 2

(d) Neither 1 nor 2

Q75) Which of the following statements isare correct

1 The Chishti order was established in India by Khwaja Moinuddin

Chishti

2 A major feature of the Chishti tradition was austerity including maintaining a distance from the

worldly power

Select the correct answer using the code

given below

(a) 1 only

(b) 2 only

(c) Both 1 and 2

(d) Neither 1 nor 2

T e s t i s p a r t o f R a u rsquo s I A S T e s t s e r i e s f o r P r e l i m i n a r y E x a m 2 0 1 9

FOUNDATION + CURRENT AFFAIRS

GENERAL STUDIES (PAPER ndashI)

FOUNDATION TEST ndashIII

SUBJECT NCERT History Class VI-X + Current Affairs

Time Allowed 1frac12 Hours Maximum Marks 150

I NSTRUCT IONS

1 IMMEDIATELY AFTER THE COMMENCEMENT OF THE EXAMINATION YOU SHOULD CHECK

THAT THIS TEST BOOKLET DOES NOT HAVE ANY UNPRINTED OR TORN or MISSING PAGES OR

ITEMS ETC IF SO GET IT REPLACED BY A COMPLETE TEST BOOKLET

2 This Test Booklet contains 75 items (questions) Each item is printed both in Hindi and English

Each item comprises four responses (answers) You will select the response which you want to mark

on the Answer Sheet In case you feel that there is more than one correct response mark the

response which you consider the best In any case choose ONLY ONE response for each item

3 You have to mark all your responses ONLY on the separate Answer Sheet (OMR sheet) provided

Read the directions in the Answer Sheet

4 All items carry equal marks

5 Before you proceed to mark in the Answer Sheet the response to various items in the Test booklet

you have to fill in some particulars in the Answer Sheet as per instructions contained therein

6 After you have completed filling in all your responses on the Answer Sheet and the examination has

concluded you should hand over to the Invigilator only the Answer Sheet You are permitted to

take away with you the Test Booklet

7 Penalty for wrong answers

THERE WILL BE PENALTY FOR WRONG ANSWERS MARKED BY A CANDIDATE IN THE

OBJECTIVE TYPE QUESTION PAPERS

(i) There are four alternatives for the answer to every question For each question for which a

wrong answer has been given by the candidate one-third of the marks assigned to that

question will be deducted as penalty

(ii) If a candidate gives more than one answer it will be treated as a wrong answer even if one of

the given answers happens to be correct and there will be same penalty as above to that

question

(iii) If a question is left blank ie no answer is given by the candidate there will be no penalty for

that question

T h i s t e s t i s p a r t o f R a u rsquo s I A S T e s t s e r i e s f o r P r e l i m i n a r y E x a m 2 0 1 9

Test Code

FC19E1003

FC19H1003 29

Answers and Explanations of

NCERT History Class VI-X + Current Affairs (FC19E1003)

Q1) उततर (c)

सपषटीकरण

- ऋगवद म दविय ो और दिताओो क समवपित एक

हजार स अविक सत तर (शल क) ह

- य शल क ऋविय ो क दवारा रच गए थ और परि ो

दवारा सीख जात थ

- हालाोवक कछ शल क मवहलाओो (जस वक अपाला

घ सा ल पामदरा मतरयी और गागी) क दवारा भी रच

गए थ

- ऋगवद म सोिाद क रप म कई शल क मौजद ह

- हम विशवावमतर नामक एक ऋवि और दविय ो क

रप म पजी जान िाली द नवदय ो (वयास और

सतलज) क बीच िाताि का उदाहरण वमलता ह

- इसस पता चलता ह वक विशवावमतर िवदक काल स

सोबोवित थ

Q2) उततर (b)

सपषटीकरण

- करनल गफाओो स राख क अिशि परापत हए ह

ज इस ओर सोकत करत ह वक ततकालीन ल ग

अवि क उपय ग स पररवचत थ

- य गफाएो आोधर परदश म सथथत ह

Q3) उततर (c)

सपषटीकरण

bull बरािह म ितिमान कशमीर म सथथत एक

परागवतहावसक थथल ह जहाो ल ग गडढ क घर ो का

वनमािण करत थ

bull य घर जमीन क ख द कर बनाए जात थ तथा नीच

जान क वलए सीवियाा ह ती थी

bull ऐसा अनमान लगाया जाता ह वक य घर ठो ड क

मौसम म आशरय परदान करत थ

Q4) उततर (c)

सपषटीकरण

bull परालख-विदया (Epigraphy) क वशलालख ो क

अधययन क रप म पररभावित वकया जाता ह

bull हसतवलसखत दसतािज ो क माधयम स इवतहास

और सावहतय क अधययन क पाोडवलवप विजञान

(Manuscriptology) कहत ह

bull पराचीन लखन परणावलय ो क अधययन और

ऐवतहावसक पाोडवलवपय ो क समझन तथा वतवथ

वनिािरण क पलीओगराफी (Palaeography) कहा

जाता ह

bull नयवमजमविकस (Numismatics) वसक ो क

अधययन क सोदवभित करता ह

Q5) उततर (a)

सपषटीकरण

- चरक सोवहता चरक क दवारा वलखी गई आयिद

और िदयक-शासर पर एक महतवपणि पसतक ह

- ि भारतीय िदयक-शासर की पारमपररक परणाली

वजस आयिद क नाम स जाना जाता ह क

अभयासकताि थ

- ऐसा माना जाता ह वक चरक का विकास दसरी

शताबदी (ईसा पिि) और दसरी शताबदी (ईसवी) क

मधय हआ था

Q6) उततर (b)

सपषटीकरण

- भाग फसल ो पर वलए जान िाल कर क सोदवभित

करता ह ज कल फसल उतपादन का 16 िाो भाग

था

- ldquoकममकारrdquo शबद भवमहीन कवि शरवमक िगि क

वलए परय ग वकया जाता था

- ldquoअशवमिrdquo (वजस घ ड क बवलदान क रप म भी

जाना जाता ह) एक अनषठान ह ता था वजसम एक

घ ड क सवतोतर रप स घमन क वलए छ ड वदया

FC19H1003 30

जाता ह और राजा क सवनक उसकी रखिाली

करत थ

Q7) उततर (d)

सपषटीकरण

- ऋगववदक काल म घ ड ो क रथ ो म ज ता जाता था

ज (रथ) भवम मिवशय ो आवद पर कबजा करन क

वलए लड गए यद ो म उपय ग वकए जात थ

- इसस यह पता चलता ह वक घ ड ो यकत रथ ो का

उपय ग महाजनपद काल स काफी पहल आरमभ

हआ था

- ऋगववदक काल म मिवशय ो भवम जल आवद पर

कबजा करन क वलए तथा ल ग ो क पकडन क

वलए यद वकय जात थ

- अविकाोश परि इन यद ो म भाग वलया करत थ

- हालाोवक उस समय क ई वनयवमत सना नही ो ह ती

थी लवकन उस काल म सभाऐो ह ती थी ो वजनम

ल ग यद क मामल ो पर चचाि करत थ

- वनयवमत सनाएा महाजनपद काल का िवशषटय थी

वजनम पदल सवनक ो की विशाल सनाएा रथ तथा

हाथी शावमल ह त थ

Q8) उततर (a)

सपषटीकरण

- बद शाकय कल स सोबोवित थ और कशीनारा म

उनका वनिन हआ था

- बद न अपनी वशकषाएा पराकत भािा म दी थी ो ज

आम ल ग ो की भािा थी

Q9) उततर (c)

सपषटीकरण

- पराचीन भारत म दशिनशासर की छह शाखाएा थी ो

िशविक नयाय समखया य ग पिि वममाोसा और

िदाोत या उततर वममाोसा

- इनकी थथापना करमश कनाद गौतम कवपल

पतोजवल जावमनी और वयास ऋविय ो न की थी

Q10) उततर (b)

सपषटीकरण

महािीर की वशकषाऐो छठी शताबदी म िललभी म

सोकवलत की गई थी ो

Q11) उततर (c)

सपषटीकरण

- पारमपररक रप स चाणकय क कौविलय अथिा

विषणगपत क नाम स जाना जाता ह

- उसन अथिशासतर ज एक पराचीन भारतीय

राजनवतक आलख ह वलखा था

Q12) उततर (d)

सपषटीकरण

- भारत का राषटर ीय वचनह सारनाथ (उततर परदश) क

अश क सतमभ क ऊपर (शीिि पर) वसोह कवपिल

का एक अनरपण ह

- इस राषटर ीय वसदाोत सतयमि जयत क साथ

सोय वजत वकया गया ह

- रामपिि बल का नाम रामपिि (वबहार) क नाम पर

पडा जहाा इसकी ख ज हई थी

- यह अपन नाजक नकाशी मॉडल क वलए परवसदद

ह वजसम क मल तवचा सोिदनशील नथन ो सतकि

कान और मरबत िााग ो क शरषठतर परवतरप क

परदवशित वकया गया ह

- यह भारतीय और फारसी ततव ो का एक ससममशरण

- सोवकससा उततर परदश म सथथत ह

Q13) उततर (a)

सपषटीकरण

का िर वसोह ज एक महान य दा थ वबहार स

सोबोवित थ

Q14) उततर (b)

सपषटीकरण

िललालर शबद बड भ-सवावमय ो क वलए परय ग

वकया जाता था

FC19H1003 31

Q15) उततर (c)

सपषटीकरण

- अररकमड एक तिीय बसती थी जहाो दर दश ो स

आन िाल जहाज ो का माल उतारा जाता था

- यहाो पर ईोि ो का एक विशाल ग दाम वमटटी क

बतिन (वजनम एमफ रा - द हरी मवठय ो का लोबा

घडा - शावमल ह) और एरिाइन (Arretine)

मदभाोड पाए गए थ

- इस थथान पर र मन दीपक काोच क बन पातर और

रतन भी पाए गए थ

Q16) उततर (a)

सपषटीकरण

- मिनदर सोगम कविताओो म उसललसखत एक

तवमल शबद ह वजसका अथि ह ldquoतीन परमखrdquo

- यह तीन सततारि पररिार ो क मसखयाओो क वलए

परय ग वकया जाता ह च ल चर और पाणडय

Q17) उततर (c)

सपषटीकरण

- ऋग िद म सभा विदाथा तथा गण जसी

जनजावतय ो पर अथिा किोब पर आिाररत

सभाओो का उललख ह

- आरसमभक िवदक काल म सभाओो और सवमवतय ो

का विशि महतव ह ता था

- यहाा तक की मसखया अथिा राजा भी उनका

समथिन परापत करन क वलए आतर रहत थ

Q18) उततर (a)

सपषटीकरण

- जन िमि न ईशवर क अससततव क मानयता त दी ह

वकनत उसन ईशवर क वजना क पद स नीच रखा

- जन िमि न बौद िमि की तरह िणि परणाली की

भरतिना नही ो की थी

Q19) उततर (d)

सपषटीकरण

- च ल ो और पाणडय ो न शसकतशाली तिीय शहर ो का

विकास वकया था

- च ल ो का सबस महतवपणि शहर पहार (या

कािरीपटटीनम) था |

- मदरई पाणडय ो की राजिानी थी

Q20) उततर (b)

सपषटीकरण

- ldquoबदचररतrdquo बद का जीिन-ितताोत ह

- इस अशवघ ि क दवारा वलखा गया था

Q21) उततर (a)

सपषटीकरणः

- तवमल कवि अपपर भगिान वशि क भकत थ

- इस परकार ि एक नयनार सोत थ

Q22) उततर (d)

सपषटीकरणः

- समदरगपत एक परवसद गपत शासक था

- उसन वसक ो पर िीणा बजात हए अपनी छवि

अोवकत करिाई थी

- यह सोगीत क परवत उसक परम क दशािता ह

- हम उसकी इलाहाबाद परशससत स महतवपणि

ऐवतहावसक जानकारी वमलती ह वजसकी रचना

उसक दरबार क कवि हररसन न की थी

Q23) उततर (b)

सपषटीकरणः

- विकरम सोित की शरआत ििि 58 ईसा पिि म

चनदरगपत वदवतीय न की थी

- यह शक ो पर उसकी जीत और उस विकरमावदतय

की पदिी वमलन क उपलकषय म आरमभ वकया गया

था

FC19H1003 32

- बानभटट न हिििििन का जीिन-ितताोत हििचररत

(ज सोसकत म थी) वलखी थी

Q24) उततर (c)

सपषटीकरणः

- सोवि-विगरावहका यद एिो शाोवत का मोतरी

- साथििाह वयापाररय ो क कावफल ो का नता

Q25) उततर (a)

सपषटीकरणः

- जआन झाोग (हसआन रताोग ndash Hsuang Tsang)

एक चीनी यातरी था ज हिििििन क शासनकाल म

भारत आया था

- ििि 630 ईसवी स ज दशक आरमभ हआ था उसम

जआन झाोग मधय एवशया ईरान और

अफग़ावनसतान की यातरा करन क पशचात कशमीर

क रासत स भारत आया था

- उसन उततर स पिि तक की यातरा की और िह

लगभग 2 ििि वबहार म रहा

- जआन झाोग न नालनदा विशवविदयालय म विदयावथिय ो

और विदवान ो क साथ पारसपररक विचार-विमशि

वकया थथानीय भािाओ ा म वनपणता परापत की तथा

बौद सतप ो की ख ज की

Q26) उततर (c)

सपषटीकरणः

- परदवकषणा पथ बौद िासतकला म सतप क चार ो

ओर बनाया जान िाला एक घमािदार पथ ह ता

- परशन म वदए गए बाकी क तीन ो ततव वहोद मसनदर ो की

िासतकला क भाग ह

Q27) उततर (d)

सपषटीकरणः

परशन म वदए गए सभी मोवदर ो म वयापक रप स

ईोि ो (पकी ईोि ो) का परय ग पतथर ो क साथ हआ

Q28) उततर (c)

सपषटीकरण

- महममद कली कतब शाह ग लकणडा का सलतान

था

- िह अकबर का समकालीन था

- सावहतय और िासतकला म उसकी अतयाविक

रवच थी

- िह एक महान कवि था

- िह दसखनी उदि फारसी और तलग म वलखता था

- उसन अपन पीछ एक विसतत वदिान (सोगरह)

छ डा ह

- अभी हाल ही म तलोगाना म ग लकणडा क वकल

क अनदर खदाई वकय गए बाग-ए-नाया वकला

बाग क चार ो ओर रप-रखा क मानवचतरण क

वलए भारतीय परातासतवक सिकषण (The

Archaeological Survey of India ndash ASI)

गराउणड पनीिर विोग रडार (Ground Penetrating

Radar) का परय ग करगा

Q29) उततर (a)

सपषटीकरणः

- वसलपपावदकारम एक तवमल महाकावय ह वजसकी

रचना इलाोग क दवारा लगभग 1800 ििि पिि की

गई थी

- यह क िलन नामक एक वयापारी की कहानी ह

ज माििी नामक एक गवणका (िशया) स परम

करन लगा था

- मवनमकलाई क िलन और माििी की पतरी की

कहानी ह

Q30) उततर (a)

सपषटीकरण

- चरक आयिद और वचवकरता की एक महतवपणि

रचना चरक सोवहता क लखक ह

- बरहमगपत क अपनी रचना बरहम-सफि-वसदानत

(ज एक खग लीय रचना ह) क कारण परवससद

वमली

FC19H1003 33

- बगदाद म इसका अनिाद अरबी भािा म वकया

गया था

- इसका इसलावमक गवणत और खग ल-विजञान पर

महतवपणि परभाि पडा था

- बाद म अपन जीिनकाल म बरहमगपत न

ldquoखोडखयाकrdquo वलखी ज एक खग लीय पससतका

(एक छ िी पसतक) थी

- इसम आयिभटट की अिि-रावतर क परतयक वदन की

शरआत परणाली का परय ग वकया गया था

Q31) उततर (c)

सपषटीकरण

- अमीर खसर एक परवसद सफी सोगीतकार कवि

और विदवान थ

- 1318 म उनह ोन पाया वक इस भवम (वहोदसतान) क

हर कषतर म अलग-अलग भािा थी लाहौरी

कशमीरी दवारसमदरी (दवकषणी कनाििक म)

तलोगाना (आोधर परदश म) गजरी (गजरात म)

माबारी (तवमलनाड म ) अििी (पिी उततर परदश

म) और वहोदिी (वदलली क आस-पास क कषतर म)

आवद

- उनह न यह बताया वक सोसकत वकसी भी कषतर स

सोबोवित नही ो थी और किल बराहमण ही इस भािा

का जञान रखत थ

Q32) उततर (c)

सपषटीकरण

- वहरणय-गभि सववणिम गभि क सोदवभित करता ह

- जब बराहमण ो की सहायता स यह अनषठान वकया

जाता था त यह माना जाता था वक बवल दन िाल

का कषवतरय क रप म पनजिनम ह गा

Q33) उततर (d)

सपषटीकरण

- कदमई भवम राजसव पर कर क सोदवभित करता

- गवावलयर परशससत म नागभि क दवारा वकय गए

श िण का िणिन वकया गया ह |

- नागभि एक परवतहार राजा था

Q34) उततर (b)

सपषटीकरण

- राजतरो वगनी 12िी ो शताबदी म कलहन क दवारा

रवचत एक सोसकत पसतक (िकसट) ह

- यह परारसमभक भारत की ऐवतहावसक इवतितत थी

- तकि सोगत रप स इस अपन परकार की सिोततम

और सिािविक विशवसनीय कवत माना जाता ह

- यह कशमीर कषतर क पराचीनतम समय स लकर

उसकी रचना की तारीख तक क समपणि इवतहास

का आचछादन करती ह

Q35) उततर (c)

सपषटीकरण

- गााि की आम सभा क ldquoउरrdquo कहा जाता था

- ldquoउरrdquo म गााि क सभी कर दन िाल वनिासी

शावमल ह त थ

Q36) उततर (a)

सपषटीकरण

- वदलली सलतनत म ldquoतारीखrdquo इवतहास लखन का

एक रप था

- ldquoतािरीखrdquo क लखक विदवान परि ह त थ वजनम

सवचि परशासक इतयावद शावमल थ

Q37) उततर (a)

सपषटीकरण

- अलाउददीन सखलजी अपन सवनक ो क ितन का

भगतान नकद म करता था न वक इकता क रप

- सवनक अपना सामान वदलली म वयापाररय ो स

खरीदत थ अतः इस बात का भय था वक वयापारी

कही ो िसतओो का मलय न बिा द

- इसकी र कथाम क वलए अलाउददीन सखलजी न

वदलली म कीमत ो क वनयसित वकया

FC19H1003 34

- अविकारीगण धयानपििक मलय ो का सिकषण करत

थ तथा ज वयापारी वनिािररत मलय पर माल नही ो

बचत थ उनक दसणडत वकया जाता था

Q38) उततर (d)

सपषटीकरण

- वदलली सििपरथम त मर राजपत ो क अिीन उनक

सामराजय की राजिानी बनी थी

- 12िी ो शताबदी क मधय म अजमर क चौहान ो

(वजनह चाहमान ो क नाम स भी जाना जाता ह) न

त मर राजपत ो क परावजत वकया था

- त मर ो और चौहान ो क अिीन वदलली एक

महतवपणि िावणसजयक क दर बन गया था

- कई जन वयापारी यहाा रहन लग थ और उनह ोन

कई मोवदर भी बनिाए

- यहाा पर मवदरत वसक वजनह ldquoदहलीिालrdquo क नाम

स जाना जाता था वयापक रप स परचलन म थ

Q39) उततर (c)

सपषटीकरण

- म ठ की मसिद का वनमािण वसको दर ल दी क

राजयकाल म उसक मिी क दवारा करिाया गया

था

- बगमपरी मसिद का वनमािण महममद तगलक क

शासनकाल म हआ था

- यह मसिद विशव का पणयथथान (The

Sanctuary of the World) और वदलली म महममद

तगलक की नई राजिानी जहाोपनाह की मखय

मसिद थी

- कववत- अल - इसलाम मसिद का विसतार

इलतसिश और अलाउददीन सखलजी न वकया था

- मीनार का वनमािण तीन सलतान ो कतबददीन ऐबक

इलतसिश और वफर ज शाह तगलक क दवारा

करिाया गया था

Q40) उततर (c)

सपषटीकरण

- मगल ो क अिीन मनसबदार शबद उस वयसकत क

वलए सोदवभित वकया जाता था वजसक पास मनसब

(अथाित पद) ह ता था

- उस अपना ितन राजसव कायो वजनह जागीर कहत

थ क रप म परापत ह ता था

Q41) उततर (b)

सपषटीकरण

- ldquoभारत छ ड आोद लनrdquo वबरविश शासन क

सखलाफ ल ग ो का एक सवाभाविक विदर ह था

- असखल भारतीय काोगरस सवमवत न 8 अगसत 1942

क बमबई म एक बठक का आय जन वकया था

- इस बठक म परवसद सोकलप ldquoभारत छ ड rdquo क

पाररत वकया गया और इस उददशय क परापत करन

क वलए गाोिी क नततव म एक अवहोसक जन सोघिि

आोद लन की शरआत का परसताि वदया गया

- लवकन अगल ही वदन गाोिी और काोगरस क अनय

परमख नताओो क वगरफतार कर वलया गया

- काोगरस क एक बार वफर अिि घ वित वकया गया

था

Q42) उततर (c)

सपषटीकरण

- साइमन कमीशन यनाइविड वको गडम क सात

साोसद ो का एक समह था

- इस वबरविश भारत क वलए सोििावनक सिार ो का

सझाि दन क वलए गवठत वकया गया था

- इस आय ग म िररषठ वबरविश राजनता सर जॉन

साइमन क नततव म किल वबरविश सदसय ही

शावमल थ

- इसवलए भारत क ल ग ो न साइमन कमीशन क

आगमन क विरद आोद लन वकया था

Q43) उततर (a)

सपषटीकरण

bull दादा भाई नौर जी भारत म वबरविश शासन क

आवथिक पररणाम ो क बार म अपनी विर िी

(परवतकल) राय क वलए जान जात थ

FC19H1003 35

bull अपन कई लख ो और भािण ो म विशि रप स

ldquoपाििी एो ड अन-वबरविश रल इन इसणडया

(Poverty and Un-British Rule in India) म

नौर जी न यह तकि वदया वक भारत पर अतयविक

कर लगाया गया था और इसकी सोपवतत इोगलड की

ओर परिावहत की जा रही थी

bull उनह ोन पराचीन भारतीय गरोथ ो की वयाखया करन

का और भारतीय ो क आिविशवास क बहाल

करन पर कायि नही ो वकया था

उनह ोन वकसी और बात स पहल सभी सामावजक

बराइय ो क उनमलन की आिशयकता पर भी बल

नही ो वदया था

Q44) उततर (c)

सपषटीकरण

bull अगसत 1932 म वबरविश परिानमोतरी मकड नालड न

अपन साोपरदावयक परसकार (The Communal

Award) की घ िणा की थी

bull यह भारत क कई साोपरदावयक वहत ो क बीच विवभनन

सोघिो क हल करन क वलए वबरिन का एकतरफा

परयास था

bull यह परसकार (Award) बाद म 1935 क

अविवनयम (The Act of 1935) म शावमल वकया

गया था

bull इस साोपरदावयक परसकार न मससलम ो क वलए

आरवकषत एक अलग वनिािचक मणडल फॉमिल का

विसतार अनय अलपसोखयक ो क वलए वकया था

वजसम वसख ो भारतीय ईसाइय ो आोगल-भारतीय

समदाय यर पीय समदाय तथा विवशषट कषतरीय

समह ो क शावमल वकया गया था

bull गाोिी न इस परसताि क भारतीय समाज क

विभावजत करन क वलए एक घवणत वबरविश

सावजश क रप म दखा और उसक सखलाफ

आमरण अनशन वकया

Q45) उततर (b)

सपषटीकरण

मौजदा आयात और वनयाित क अवतररक़त

औपवनिवशक भारत क वनमनवलसखत खचो क

वलए एक विशिवनवशचत िन रावश भी दनी पडती

थी

(i) परशासन क वयय

(ii) सना क रख-रखाि क वयय

(iii) यद क वयय

(iv) सिावनितत अविकाररय ो की पशन तथा

(v) वबरिन दवारा अपनी उपवनिश बसती

(कॉल नी) क रख-रखाि क वयय

इनह गह शलक (Home Charges) क रप म

जाना जाता था और लगभग परी तरह स भारत क

दवारा इनका भगतान वकया जाता था

bull गह शलक म वनमनवलसखत घिक शावमल थ

(i) भारतीय ऋण पर दय बयाज

(ii) ईसट इोवडया को पनी क शयरिारक ो क

लाभाोश

(iii) लोदन म भारत कायािलय चलान क वलए िन

(iv) भारत म वनयकत वबरविश कवमिय ो क ितन

और पशन का भगतान करन क वलए िन

(v) रलि पर बयाज

(vi) नागररक और सनय शलक

(vii) इोगलड म सट र (सामगरी) की खरीद

Q46) उततर (b)

सपषटीकरण

bull भारतीय राषटर ीय काोगरस का लाहौर सतर 1929 म

जिाहरलाल नहर की अधयकषता म आय वजत

वकया गया था

bull इस सतर म भारतीय राषटर ीय आोद लन स समबसित

कई महतवपणि पररणाम सामन आय थ

(i) सििपरथम इस सतर म काोगरस क अधयकष पद

पर जिाहरलाल नहर क चना गया था ज

काोगरस म िामपोवथय ो की बिती हई ताकत

का सपषट सोकत था

(ii) दसरा इस सतर म पहली बार काोगरस न पणि

सवतोतरता की माोग क उठाया था

इस परकार की माोग काोगरस मोच स पहल कभी भी

नही ो उठाई गई थी

Q47) उततर (b)

सपषटीकरण

FC19H1003 36

bull इस ररप िि न वकसी भी समदाय क वलए पथक

वनिािचक मोडल अथिा अलपसोखयक ो क वलए

भाराोश की वसफाररश नही ो की थी

bull तथावप इस ररप िि न उन पराोत ो म अलपसोखयक

सीि ो क आरकषण की अनमवत दी थी जहाा पर कम

स कम दस परवतशत अलपसोखयक ह

bull लवकन यह समदाय क आकार क अनपात म ह ना

चावहए था

bull इस ररप िि म भारत क वलए पणि सवतोतरता क

वलए क ई पराििान नही ो था

Q48) उततर (c)

सपषटीकरण

bull आरो वभक िवदक आयो का िमि मखय रप स

परकवत की पजा और यजञ था

bull परारो वभक आयि िमि परकवत की पजा क समान था

bull िासति म उनक चार ो ओर की शसकतयाा वजनह न

त ि वनयोवतरत कर सकत थ और न ही समझ पाए

थ उनह वदवयता क साथ वनिवशत वकया गया तथा

उनह मादा या नर दिीदिताओो क रप म

परतीकतव वकया गया था

bull उनह ोन कछ यजञ ो का भी वनषपादन वकया था

Q49) उततर (b)

सपषटीकरण

bull सडक और नदी-मागि (जल-मागि) डकती स

सरवकषत नही ो थ

bull उललखनीय ह वक हिििििन क शासनकाल क

दौरान यआन चिाोग (हयएन साोग) का सारा

सामान लि वलया गया था

Q50) उततर (c)

सपषटीकरण

परशन म वदए गए द न ो कथन सही ह

Q51) उततर (b)

सपषटीकरण

bull परोदर दास एक सोत और भगिान कषण क एक

महान भकत थ

bull परोदर दास क कनाििक सोगीत क वपतामह क

रप म जाना जाता ह

bull यदयवप उनक जनम-थथान क बार म काफी

अिकल लगाई जाती रही ह

bull तथावप अब कननड विशवविदयालय हमपी क दवारा

गवठत एक विशिजञ सवमवत इस वनषकिि पर पहोची

ह वक उनका जनम थथान सोभितया कनाििक का

एक छ िा-सा गााि कषमपरा (वशिम गगा वजला)

था

Q52) उततर (c)

सपषटीकरण

bull शरी तयागराज शरी शयाम शासतरी और शरी मथसवामी

दीवकषतर क कनाििक सोगीत की वतरमवति माना

जाता ह

bull उनक कारण ही 18िी ो-19िी ो शताबदी म कनाििक

सोगीत का सववणिम यग आया था

Q53) उततर (d)

सपषटीकरण

bull अभी हाल ही म लौह यगीन-महापािावणक काल

का 2000 ििि पराना एक दलिभ सारक फगस

(Sarcophagus) (पतथर का ताबत) क ललम क

वियर गाोि (क वयलडी क पास वजला क वझक ड

करल राजय) की एक रॉक-कि गफा स ख जा गया

bull यह ताबत वजसम हविय ो क िकड थ खदाई क

दौरान वमला

bull अभी तक इस परकार की दलिभ ख ज करल क

मातर द ही थथान ो स हई ह

bull य द न ो सारक फगी (Sarcophagi) (पतथर क

ताबत) चियर और अथ ली (वजला क वझक ड) क

महापािाण थथल ो स वमल ह

Q54) उततर (a)

सपषटीकरण

FC19H1003 37

दवकषण भारत म महापािाण सोसकवत एक पणि

विकवसत लौह यगीन सोसकवत थी

Q55) उततर (d)

सपषटीकरण

bull च ल पाणडय और करलपतर (चर) इन तीन ो का

उललख अश क क अवभलख ो म वकया गया ह

bull सोभितः य भौवतक सोसकवत क उततर

महापािावणक चरण म थ

Q56) उततर (d)

सपषटीकरण

bull भीमा-क रगाोि की लडाई ततीय आोगल-मराठा

यद का वहससा थी

Q57) उततर (b)

सपषटीकरण

bull राजकमार शकल न गाोिीजी क चोपारण आन तथा

वतनकवथया परणाली स जडी समसया की जाोच क

वलए रारी करन क वलए दश भर म उनका

अनसरण वकया था

bull बज वकश र राजदर परसाद महादि दसाई और

नरहरी पाररख चोपारण सतयागरह क दौरान गाोिी

जी क सहय गी थ

Q58) उततर (b)

सपषटीकरण

bull बराहमण ो और बौद मठिाररय ो क कर-मकत गााि

अनदान म दन की परथा सतिाहन ो न आरमभ की

थी

Q59) उततर (c)

सपषटीकरण

इस कायिकरम क उददशय वनमनानसार ह

(i) बवनयादी पयििन आिाररक सोरचना का विकास

करना

(ii) चयवनत (पहचान वकय गए) कषतर ो म आजीविका क

सजन क वलए दश क साोसकवतक और विरासत

मलय ो क बिािा दना

(iii) विरासत समारक थथल ो पर विशव सतरीय आिाररक

सोरचना विकवसत करक एक सतत तरीक स

पयििक आकििण म िसद करना

(iv) थथानीय समदाय ो की सवकरय भागीदारी क माधयम

स र रगार ो का सजन करना

(v) र रगार उतपादन और आवथिक विकास क वलए

पयििन कषमता का उन पर परभाि का उपय ग

करना तथा

(vi) िारणीय पयििन आिाररक सोरचना का विकास

करना और उसका उवचत सोचालन तथा

रखरखाि सवनवशचत करना

Q60) उततर (b)

सपषटीकरण

bull यह वनकाय ििि 1987 म अससततव म आया था

bull यह एक राषटर ीय सतर का शीिि सोगठन ह ज भारत

सरकार क जनजातीय मामल ो क मोतरालय क

परशासवनक वनयोतरण क अिीन काम कर रहा ह

bull इसका पोजीकत और परिान कायािलय नई वदलली

म सथथत ह

Q61) उततर (c)

सपषटीकरण

bull परमचोद क उपनयास ो म परमाशरम रोगभवम गबन

कमिभवम और ग दान शावमल ह

bull ग रा रिी ोदरनाथ िग र क दवारा रवचत उपनयास ह

bull अभी हाल ही म मोशी परमचोद की 138िी ो जयोती दश

भर म मनाई गई थी

Q62) उततर (b)

सपषटीकरण

bull ldquoवगदाrdquo पोजाब (भारत) एिो पावकसतान की

मवहलाओो क दवारा तयौहार क समय और फसल

की बिाई तथा किाई क अिसर पर वकया जान

िाला एक पारोपररक दहाती नतय ह

FC19H1003 38

bull इस नतय क माधयम स पोजाबी मवहलाऐो अपनी

परसननता परकि करती ह तथा वगदा क परदशिन क

माधयम स परि िचिसव िाल समाज म मवहलाओो

की दबी हई भािनाओो क परकि करती ह

bull चोवक इस नतय का परि ो क साथ क ई सोबोि नही ो

ह अतः किल मवहलाऐो ही इसम भाग ल सकती

bull हर साल तीज समार ह क दौरान पोजाब म वगदा

नतय वकया जाता ह

तीज भारत क कछ भाग ो म मवहलाओो क दवारा

मनाया जान िाल कई तयौहार ो क वलए एक

वयापक नाम ह

Q63) उततर (a)

सपषटीकरण

- मजम-उल-बहरीन या द समदर ो का सोगम

नामक उललखनीय रचना दारा वशक ह क दवारा

वलखी थी

- भारत क उपराषटर पवत शरी एम िकया नायड न कहा

ह वक राजकमार दारा वशक ह की रचनाएा शाोवत

और सदभाि क बिािा दन क वलए एक तारा सर त

क रप म सामन आ सकती ो ह

- उपराषटर पवत गत ििो क भला वदए गए राजकमार

दारा वशक ह क परदवशित परचवलत करन हत

आय वजत एक परदशिनी का दौरा करन क बाद एक

सभा क सोब वित कर रह थ

- इस परदशिनी का आय जन फर क इस गौवियर

(Francois Gautier) क दवारा lsquoइोवदरा गाोिी नशनल

सिर फॉर द आििसrsquo (The Indira Gandhi

National Centre for the Arts) नई वदलली म

वकया गया था

Q64) उततर (c)

सपषटीकरण

- ग मतशवर परवतमा जन भगिान बाहबली क

समवपित ह

- यह एक एक-चटटानी पतथर की मवति ह

- राषटर पवत राम नाथ क विोद न शरिणबलग ला

(कनाििक) म आय वजत वकय जान िाल भवय

अवभिक समार ह महामसतकावभिक का

उदघािन वकया था

- यह समार ह 12 ििो म एक बार ह ता ह

Q65) उततर (c)

सपषटीकरण

bull पराची घािी पराची नदी क चार ो ओर फली हई थी

bull पराची घािी िीर-िीर विलपत ह गई थी

bull पराची नदी भिनशवर स वनकलती ह

bull यह महानदी की एक सहायक नदी ह और यह

परी खदाि किक तथा जगतवसोहपर वजल ो स

ह कर बहती ह

bull इस नदी क पर कषतर क पराची घािी कहा जाता ह

bull यह नदी बोगाल की खाडी म वगरती ह

परातासतवक साकषय स पता चलता ह वक पराची घािी

सभयता हडपपा और म हनज दाड द न ो की

पिििती ह

Q66) उततर (d)

सपषटीकरण

य समारक छतरपर वजल (मधय परदश) म विोधयाचल

पिित शरोखला म सथथत ह

Q67) उततर (a)

सपषटीकरण

bull थॉिस ऑन पावकसतान नामक पसतक डॉ बी

आर अमबडकर न वलखी थी

bull डॉ बी आर अमबडकर की जयोती क अिसर पर

भारत क राषटर पवत न भारत की इस महान हसती

क शरदाोजवल अवपित की थी

bull डॉ बी आर अमबडकर न 1924 म वडपरथड

कलावसर इोसटीटयि (दवलत िगि सोथथान -

बवहषकत वहतकाररणी सभा) और 1927 म समाज

समता सोघ की थथापना की थी

bull अमबडकर का धयान वशकषा कषतर की ओर भी था

bull उनह ोन वशकषा क वनमन िगो म फलान क वलए

पीपलस एजकशन स साइिी (The Peoples

Education Society) क नाम स महाविदयालय ो क

नििकि और छातरािास ो की थथापना की थी

FC19H1003 39

Q68) उततर (b)

सपषटीकरण

bull महरगि भारतीय उपमहादवीप म एक परवसद

निपािाण बसती ह ज बलवचसतान पराोत

पावकसतान म सथथत ह

bull दचपलली (आोधर परदश) क पास नागलर नदी क

पिी ति ो पर चना पतथर क बलॉक क विशाल

विसतार म एक पिि-ऐवतहावसक रॉक आिि थथल की

ख ज की गई ह

bull इसन 1500-2000 ईसा पिि क दौरान गोिर (आोधर

परदश) म विकवसत निपािाण सभयता पर परकाश

डाला ह

Q69) उततर (c)

सपषटीकरण

bull 12िी ो सदी और 13िी ो सदी म काकाविय िोश का

उदय हआ था

bull ि पहल कलयाण क पवशचमी चालकय ो क सामोत थ

bull परारोभ म उनह ोन िारोगल (तलोगाना) क पास एक

छ ि स कषतर पर शासन वकया था

bull उनह ोन ldquoनायक वयिथथाrdquo की शरआत की थी

वजस बाद म विजयनगर क राय शासक ो न

अपनाया और विकवसत वकया था

Q70) उततर (a)

सपषटीकरण

bull गाोिीजी क अनशन स वमल मावलक ो पर दबाि

पडा था ज अोततः शरवमक ो क ितन म 35 परवतशत

की िसद करन क वलए सहमत हए थ

bull गगल (Google) न अनसया साराभाई वजनह ोन

भारत क शरवमक आोद लन म एक अगरणी भवमका

वनभाई थी की 132िी ो जयोती डडल (Doodle) का

वनमािण करक मनाई

Q71) उततर (d)

सपषटीकरण

भारत स यनसक की मानिता की अमति साोसकवतक

विरासत की परवतवनवि सची म वनमनवलसखत शावमल ह

bull कवडयटटम करल का सोसकत रोगमोच

bull मवडयिि करल का अनषठान रोगमोच और नतय

नाविका

bull िवदक मि जाप की परोपरा

bull राजथथान क कालबवलया ल क गीत और नतय

bull रामलीला रामायण का पारोपररक परदशिन

bull सोकीतिन मवणपर का अनषठान गायन ढ ल िादन

और नतय

bull रममन भारत क गििाल वहमालय का िावमिक

तयौहार और अनषठान रोगमोच

bull जाोदीयाला गर पोजाब क ठठर ो की पीतल और

ताोब क वशलप स वनवमित बतिन ो की पारोपररक कला

bull छाऊ नतय पिी भारतीय राजय ो म जनमी शासतरीय

भारतीय नतय कला

bull लददाख का बौद मि जाप िर ाोस-वहमालयी लददाख

कषतर तथा जमम-कशमीर म पवितर बौद गरोथ ो का पाठ

bull य ग

bull नौर र

bull को भ मला

Q72) उततर (b)

सपषटीकरण

bull भारत क राषटर पवत शरी राम नाथ क विोद न

वकसामा नागालड म हॉनिवबल मह रति और

राजय गठन वदिस समार ह का उदघािन वकया

था

bull हॉनिवबल मह रति का नाम भारतीय हॉनिवबल क

नाम पर पडा ह ज एक विशाल और रोगीन जोगली

पकषी ह

bull यह पकषी नागालड राजय की अविकतर जनजावतय ो

की ल ककथाओो म उसललसखत ह

bull नागालड की परमख मानयता परापत जनजावतयाा ह

अोगामी आओ चखसोग चाोग ककी रगमा और

रवलोग आवद

bull ओोग जारिा और ससिनलीस अोडमान-वनक बार

दवीप समह की जनजावतयाा ह

FC19H1003 40

Q73) उततर (c)

सपषटीकरण

bull दकन म राषटर कि शासन दसिी ो सदी क अोत तक

लगभग 200 ििो तक रहा था

bull राषटर कि शासक अपन िावमिक विचार ो म सवहषण

bull उनह ोन न किल शि िमि और िषणि िमि बसलक

जन िमि क भी सोरकषण वदया था

bull एल रा म वशि क परवसद रॉक कि मोवदर का

वनमािण नौिी ो सदी म राषटर कि राजा कषण परथम न

करिाया था

bull उसका उततराविकारी अम घििि जन था लवकन

उसन अनय िमो क भी सोरकषण परदान वकया था

bull राषटर कि ो न मसलमान वयापाररय ो क बसन की

अनमवत दी थी

bull उनह न अपन अविराजय ो म इसलाम क उपदश दन

की भी अनमवत दी थी

bull अभी हाल ही म पाोडिलागटटा (तलोगाना) क

परागवतहावसक चटटान वचतर ो क कषरण की बिती हई

घिनाएा एक गोभीर वचोता का वििय ह

bull यह परागवतहावसक चटटान क नकसान पहाचा

सकता ह

bull पाोडिलागटटा वनमनवलसखत क वलए जाना जाता ह

- 10000 ईसा पिि स 8000 ईसा पिि क वचवतरत

चटटानी आशरय ो क वलए

- राषटर कि काल क एक 8 िी ो सदी क

वशलालख क वलए और

- 12िी ो सदी क काकविय सामराजय क वभवतत

वचतर ो क वलए

Q74) उततर (b)

सपषटीकरण

bull 1828 म राजा राम म हन रॉय न एक नय िावमिक

समाज बरहम सभा की थथापना की थी वजस बाद

म बरहम समाज क नाम स जाना गया था

bull दिदरनाथ िग र न ततवब विनी सभा की अधयकषता

की थी ज आधयासिक सतय की ख ज म सोलि

थी

bull इसका उददशय वहोद िमि क शद करन का और

एकशवरिाद (एक ईशवर म आथथा) का परचार करना

था

bull नय समाज की थथापना क आिार थ कारण

(तकि ) क द सतमभ तथा िद और उपवनिद

bull अभी हाल ही म सािारण बरहम समाज का कछ

काननी मदद ो क लकर पवशचम बोगाल सरकार क

साथ काननी वििाद चल रहा ह

Q75) उततर (c)

सपषटीकरण

bull भारत म वचशती वसलवसल की थथापना खवाजा

म इनददीन वचशती क दवारा की गयी थी

bull ि 1192 ईसवी क आसपास भारत आय थ

bull वचशतीय ो क बारहिी ो शताबदी क उततरािि म भारत

म आन िाल सफीय ो क समह ो म सबस

परभािशाली माना जाता ह

bull उनह ोन थथानीय िातािरण क साथ सफलतापििक

अनकलन वकया और उनह ोन भारतीय भसकत

परोपराओो क कई पहलओो क अपनाया

bull अजमर म सफी अपरकि खवाजा म इनददीन वचशती

की ऐवतहावसक दरगाह क एक नया रप दन की

तयारी की जा रही ह

bull इस 13िी ो शताबदी की दरगाह क ldquoसवचछ

आइकॉवनक थथल ोrdquo (Swacch Iconic Places) म

शावमल वकया गया ह ज परवतवषठत विरासत

आधयासिक और साोसकवतक थथान ो पर क वदरत

य जना ह

FC19H1003 41

ANSWERS amp EXPLANATION OF

NCERT History Class VI-X + Current Affairs

(FC19E1003)

Q1) Answer c

Explanation

Rigveda consists of more than a

thousand hymns dedicated to gods and

goddesses These hymns were

composed by sages and learnt by men

however a few were composed by

women like Apala Ghosa Lopamudra

Maitreyi and Gargi

Rigveda consists of many hymns in the

form of dialogues We get an example of

a dialogue between a sage named

Vishwamitra and two rivers (Beas and

Sutlej) that were worshipped as

goddesses This suggests that he

belonged to the Vedic period

Q2) Answer b

Explanation

Traces of ash have been found from

Kurnool Caves suggesting that people

were familiar with the use of fire

It is situated in Andhra Pradesh

Q3) Answer c

Explanation

Burzahom is a prehistoric site in

present day Kashmir where people built

pit houses which were dug into the

ground with steps leading into them

These may have provided shelter in cold

weather

Q4) Answer c

Explanation

Epigraphy is defined as the study of

inscriptions

Manuscriptology is the study of history

and literature through the use of hand

written documents

Palaeography refers to the study of

ancient writing systems and the

deciphering and dating of historical

manuscripts

Numismatics refers to the study of

coins

Q5) Answer a

Explanation

Charaka Samhita was written by

Charaka and is an important book on

Ayurveda and medicine

He was a practitioner of the traditional

system of Indian medicine known as

Ayurveda

Charaka is thought to have flourished

sometime between the 2nd century BCE

and the 2nd century CE

Q6) Answer b

Explanation

Bhaga refers to the tax on crops which

was fixed at 16th of the production

Kammakaras is the term used for the

landless agricultural labour class

Ashvamedha also known as horse

sacrifice is a ritual where a horse is let

loose to wander freely and it was

guarded by the rajarsquos men

Q7) Answer (d)

Explanation

In the Rigvedic period horses were

yoked to chariots that were used in

battles fought to capture land cattle

etc This suggests that the use of horse

chariots began much before the period

of Mahajanapadas

The battles were fought in the Rigvedic

period for cattlersquos lands water an even

to capture people Most men took part

in these wars however there was no

regular army but there were assemblies

where people met and discussed

matters of war Regular armies became

a feature in the Mjahajanapada period

including vast armies of foot soldiers

chariots and elephants

RAUSIAS-FC19E1003 42

Q8) Answer (a)

Explanation

Buddha belonged to the Sakya clan and

passed away at Kusinara

Buddha taught in Prakrit which was the

common language of people

Q9) Answer c

Explanation

There were six schools of philosophy in

ancient India These are known as

Vaishesika Nyaya Samkhya Yoga

Purva Mimansa and Vedanata or Uttara

Mimansa They were founded by sages

Kanada Gautama Kapila Patanjali

Jamini and Vyasa respectively

Q10) Answer b

Explanation

The teachings of Mahavira were

compiled at Valabhi in 6th century AD

Q11) Answer (c)

Explanation

Chanakya is traditionally identified as

Kautilya or Vishnugupta who authored

the ancient Indian political treatise the

Arthashastra

Q12) Answer d

The national emblem of India is an

adaptation of the Lion Capital atop the

Ashoka Pillar of Sarnath Uttar Pradesh

and is combined with the National

Motto Satyameva Jayate

The Rampurva Bull gets the name from

the site of its discovery Rampurva in

Bihar

It is noted for its delicately sculpted

model demonstrating superior

representation of soft flesh sensitive

nostrils alert ears and strong legs It is

a mixture of Indian and Persian

elements

Sankissa is situated in Uttar Pradesh

India

Q13) Ans(a)

Kunwar Singh was a notable leader during the Revolt of 1857 He belonged

to a royal house of Jagdispur Bihar

Q14) Answer b

Explanation

The term Vellalar was used for large

landowners

Q15) Answer c

Explanation

Arikamedu was a coastal settlement

where ships unloaded goods from

distant lands Finds here include a

massive brick warehouse pottery

including amphorae and Arretine ware

Roman lamps glassware and gems have

also been found at the site

Q16) Answer a

Explanation

Muvendar is a Tamil word mentioned in

Sangam poems meaning three chiefs

used for the heads of three ruling

families the Cholas Cheras and

Pandyas

Q17) Ans (c)

Several tribal or kin-based assemblies

such as the Sabha Vidatha and gana

are mentioned in the Rig-veda The

Sabha and the samiti mattered a great

deal in early Vedic times so much so

that the chiefs or the kings showed an

eagerness to win their support

Q18) Ans (a)

Jainism recognised the existence of the

gods but placed them lower than the

jina and did not condemn the varna

system as Buddhism did

Q19) Answer (d)

Explanation

Cholas and Pandyas had developed

powerful coastal cities The most

important city of Cholas was Puhar or

Kaveripattinam and Madurai was the

capital of Pandyas

Q20) Answer b

Explanation

Buddhacharita is the biography of

Buddha and was written by

RAUSIAS-FC19E1003 43

Ashvaghosha

Q21) Answer (a)

Explanation

Tamil poet Appar was a Shiva devotee

So he was a Nayanar saint

Q22) Answer d

Explanation

Samudragupta was a prominent Gupta

ruler whose coins depict him playing a

veena indicating his love for music We

get important historic information from

his Allahabad Prashasti which was

composed by his court poet Harisena

Q23) Answer (b)

Explanation

Vikrama Samvat was founded by

Chandragupta II in the 58 BC as a

mark of victory over the Shakas and

assumed the title of Vikramaditya

Banabhatta wrote Harshavardhanarsquos

biography the Harshacharita in

Sanskrit

Q24) Answer c

Explanation

Sandhi-vigrahika was the minister of

war and peace

Sarthavaha was the leader of the

merchant caravans

Q25) Answer a

Explanation

Xuan Zang (Hsuan-tsang) was a

Chinese traveller who came during the

reign of Harshavardhana

In the decade that began in 630 AD

Xuan Zang came to India through

Kashmir after visiting Central Asia Iran

and Afghanistan

He travelled from north to east and lived

in Bihar for a couple of years

At Nalanda University Xuan Zang

interacted with students and scholars

mastered local languages and

discovered Buddhist stupas

Q26) Answer c

Explanation

Pradakshina patha is a circular path

laid around a stupa in Buddhist

architecture While the rest are a part of

temple architecture

Q27) Answer d

Explanation

All the above-mentioned temples have

an elaborate use of bricks (baked

bricks) along with stone

Q28) Ans (c)

Muhammad Quli Qutab was the Sultan

of Golconda He was a contemporary of

Akbar was very fond of literature and

architecture

The Sultan was a great poet and he

wrote in Dakhini Urdu Persian and

Telgu and has left an extensive diwan or

collection

Recently the Archaeological Survey of

India (ASI) will be using Ground

Penetrating Radar (GPR) to map the

contours of the area around the Bagh-e-

Naya Qila excavated garden inside the

Golconda Fort in Telangana

Q29) Answer a

Explanation

Silappadikaram is a famous Tamil epic

which was written by Ilango around

1800 years ago It is a story of a

merchant named Kovalan who fell in

love with a courtesan named Madhavi

Manimekalai tells the story of the

daughter of Kovalan and Madhavi

Q30) Answer (a)

Explanation

Charaka is the author of Charaka

Samhita which is an important work of

Ayurveda and medicines

Brahmaguptarsquos fame rests mostly on his

Brahma-sphuta-siddhanta which was

an astronomical work It was translated

into Arabic in Baghdad and had a major

impact on Islamic mathematics and

astronomy

Late in his life Brahmagupta wrote

Khandakhadyaka which was an

RAUSIAS-FC19E1003 44

astronomical handbook that employed

Aryabhatarsquos system of starting each day

at midnight

Q31) Answer (c)

Explanation

Amir Khusrau was a famous sufi

musician poet and scholar In 1318 he

noted that there was different language

in every region of this land (Hindustan)

Lahori Kashmiri Dvarsamudri (in

Southern Karnataka) Telangana (in

Andhra Pradesh) Gujari (in Gujarat)

Marsquobari (in Tamil Nadu) Awadhi (in

eastern Uttar Pradesh) and Hindawai (in

the area around in Delhi) etc He went

to explain that Sanskrit did not belong

to any region and that only brahmans

knew it

Q32) Answer c

Explanation

Hiranyagarbha refers to the golden

womb When this ritual was performed

with the help of Brahmanas it was

thought to lead to the rebirth of the

sacrificer as a Khastriya

Q33) Answer d

Explanation

Kadamai refers to a tax on land

revenue

Gwalior Prashasti describes the exploits

of Nagabhata who was a Pratihara king

Q34) Answer b

Explanation

Rajatarangini is a Sanskrit text written

by Kalhana in the 12th century

It was historical chronicle of early India

It is justifiably considered to be the best

and most authentic work of its kind

It covers the entire span of history in

the Kashmir region from the earliest

times to the date of its composition

Q35) Answer c

Explanation

ldquoUrrdquo was the general assembly of the

village ldquoUrrdquo consisted of all the

taxpaying residents of an ordinary

village

Q36) Answer (a)

Explanation

Tarikh was a form of history writing in

the Delhi Sultanate The authors of

tawarikhs were learned men which

included secretaries administrators etc

Q37 Answer (a)

Explanation

Alauddin chose to pay his soldiers salaries in cash rather than iqtas The soldiers would buy their supplies from merchants in Delhi and it was thus feared that merchants would raise their prices To stop this Alauddin controlled the prices of goods in Delhi Prices were carefully surveyed by officers and merchants who did not sell at the prescribed rates were punished

Q38) Answer (d)

Explanation

Delhi first became the capital of a

kingdom under the Tomara Rajputs

who were defeated in the middle of the

twelfth century by the Chauhans (also

referred to as Chahamanas) of Ajmer

It was under the Tomaras and

Chauhans that Delhi became an

important commercial centre Many rich

Jaina merchants lived in the city and

constructed several temples Coins

minted here called dehliwal had a wide

circulation

Q39) Answer (c)

Explanation

Moth ki Masjid was built in the reign of

Sikandar Lodi by his minister

Begumpuri mosque built in the reign of

Muhammad Tughluq was the main

mosque of Jahanpanah the ldquoSanctuary

of the Worldrdquo and his new capital in

Delhi

Quwwat al ndash Islam mosque was

enlarged by Iltutmish and Alauddin

Khalji The minar was built by three

Sultansndash Qutbuddin Aybak Iltutmish

and Firuz Shah Tughluq

RAUSIAS-FC19E1003 45

Q40) Answer (c)

Explanation

Under the Mughals mansabdar was

referred to an individual who held a

mansab ie rank and he received his

salary as revenue assignments called

jagirs

Q41) Ans (b)

The Quit India Movement was a

spontaneous revolt of people against

British rule

The All India Congress Committee met

at Bombay on 8 August 1942 It passed

the famous resolution Quit India and

proposed the starting of a non-violent

mass struggle under Gandhis

leadership to achieve this aim But on

the very next day Gandhi and other

eminent leaders of the Congress were

arrested The Congress was once again

declared illegal

Q42) Ans (c)

The Simon Commission refers to a

group of seven MPs from the United

Kingdom constituted to suggest

constitutional reforms for British India

The Commission consisted of only

British members headed by one of the

senior British politicians Sir John

Simon

So the people of India agitated against

the arrival of Simon Commission

Q43) Ans (a)

He was widely known for his

unfavourable opinion of the economic

consequences of the British rule in

India

In his many writings and speeches and

especially in Poverty and Un-British

Rule in India Naoroji argued that India

was too highly taxed and that its wealth

was being drained away to England

He did not interpret the ancient Indian

texts and restored the self-confidence of

Indians And also he did not stress the

need for eradication of all the social

evils before anything else

Q44) Ans (c)

In August 1932 Prime Minister

MacDonald announced his Communal

Award Great Britainrsquos unilateral

attempt to resolve the various conflicts

among Indiarsquos many communal

interests

The award which was later

incorporated into the act of 1935

expanded the separate-electorate

formula reserved for Muslims to other

minorities including Sikhs Indian

Christians Anglo-Indians Europeans

distinct regional groups Gandhi

undertook a ldquofast unto deathrdquo against

that offer which he viewed as a

nefarious British plot to divide the

Indian society

Q45) Ans (b)

In British India apart from existing

imports and exports there was also a

particular amount of money which

colonial India contributed towards

administration maintenance of the

army war expenses pensions to retired

officers and other expenses accrued by

Britain towards maintenance of her

colony These were known as Home

charges and were paid for almost

entirely by India

The Home charges was made of

following components-

- Interest payable on Indian debt

- Dividend to shareholders of East

India Company

- Funds used to support the India

Office in London

- Funds used to pay salaries and

pensions of British personnel

engaged in India

- Interest on the railways

- Civil and military charges

- Store purchases in England

Q46) Ans (b)

The Lahore session of the Indian

National Congress was held in 1929

under the Presidentship of Jawaharlal

Nehru

The Lahore session of the Indian

National Congress witnessed significant

RAUSIAS-FC19E1003 46

developments in the Indian national

movement

- First the election of Jawaharlal

Nehru to the post of Presidentship of

the Congress was a clear indication

of the growing strength of the

Leftists in the Congress

- Secondly it was in this session that

the Congress for the first time raised

the demand for complete

independence Such demand was

not raised from the Congress

platform earlier

Q47) Ans (b)

It did not provide for separate

electorates for any community or

weightage for minorities However it did

allow for the reservation of minority

seats in provinces having minorities of

at least ten per cent but this was to be

in strict proportion to the size of the

community

There was no provision for complete

Independence for India

Q48) Ans (c)

The religion of early Vedic Aryans was

primarily of worship of nature and

Yajnas

The early Aryan religion was kind of

nature worship Actually the forces

around them which they could not

control or understand were invested

with divinity and were personified as

male or female gods And they

performed some Yajnas also

Q49) Ans (b)

The roads and river-routes were not

immune from robbery It is notable that

Yuan Chwang (Hiuen Tsang) was

robbed of his belongings during

Harshvardanarsquos period

Q50) Ans (c)

Q51) Ans (b)

Purandara Dasa was a saint and great

devotee of Lord Krishna

There is much speculation about where

Purandara Dasa regarded as the

Pitamaha of Carnatic music was born

Recently an expert committee

constituted by the Kannada University

Hampi has come to the conclusion that

Kshemapura Shivamogga district

Karnataka is the birth place of

Purandara Dasa

Q52) Ans (c)

Sri Tyagaraja Sri Shyama Shastry and Sri Muthuswami Dikshitar are considered the trinity of Carnatic music and with them came the golden age in Carnatic music in the 18th-19th

century

Q53) Ans d)

Recently a rare sarcophagus (stone

coffin) which is 2000 years old from the

Iron AgendashMegalithic era was discovered

from a rock-cut cave at Viyur village of

Kollam near Koyilandy in Kozhikode

district Kerala

The coffin containing bone fragments

was found during an excavation ldquoSo

far such a rare finding has been

discovered only from two sites

in Kerala Both these sarcophagi were

recovered from Megalithic sites at

Chevayur and Atholi also in Kozhikode

district

Q54) Ans a)

The megalithic culture in South India was a full-fledged Iron Age culture

Q55) Ans d)

The Cholas Pandyas and Keralaputras

(Cheras) mentioned in Ashokan

inscriptions were probably in the late

megalithic phase of material culture

Q56) Ans d)

Q57) Ans (b)

Raj Kumar Shukla followed Gandhiji all

over the country to persuade him to

come to Champaran to investigate the

problem associated with tinkathia

system

RAUSIAS-FC19E1003 47

Brij Kishore Rajendra Prasad Mahadev

Desai and Narhari Parikh accompanied

Gandhi ji during the Champaran

Satyagraha

Q58) Ans (b)

The Satvahanas started the practice of granting tax-free villages to brahmanas and Buddhist monks

Q59) Ans c)

The objectives of the Programme are

listed as under

- Developing basic tourism

infrastructure

- Promoting cultural and heritage

value of the country to generate

livelihoods in the identified regions

- Enhancing the tourist attractiveness

in a sustainable manner by

developing world-class

infrastructure at the heritage

monument sites

- Creating employment through active

involvement of local communities

- Harnessing tourism potential for its

effects on employment generation

and economic development

- Developing sustainable tourism

infrastructure and ensuring proper

Operations and maintenance

therein

Q60) Ans (b)

The Tribal Cooperative Marketing

Development Federation of India

(TRIFED) came into existence in 1987

It is a national-level apex organization

functioning under the administrative

control of Ministry of Tribal Affairs

Govt of India

TRIFED has its registered and Head

Office located in New Delhi

Q61) Ans (c)

Premchandrsquos novels include

Premashram Rangabhumi Ghaban

Karmabhumi and Godan

Gora is a novel written by Rabindranath

Tagore

138th birth anniversary of Munshi

Premchand was celebrated across the

country

Q62) Ans (b)

Giddha is a traditional pastoral dance

performed by the women of the Punjab

India and Pakistan at festival times

and at the sowing and reaping of the

harvest

By this dance the Punjabi women

reveal their joy expel their suppressed

feelings in a male dominated society

through the performance of Giddha

Since this dance has nothing to do with

men only women can participate in it

During the Teej celebrations Giddha

dance is celebrated in Punjab every

year Teej is a generic name for a

number of festivals that are celebrated

by women in some parts of India

Q63) Ans (a)

Dara Shukoh wrote the remarkable

work called ldquoMajma-ul-Bahrainrdquo or the

ldquoThe confluence of two seasrdquo

The Vice President of India Shri M

Venkaiah Naidu has said that Prince

Dara Shukohrsquos writings can come as a

refreshing source for infusing peace and

harmony He was addressing the

gathering after visiting the exhibition

that showcases the forgotten Prince of

yesteryears Dara Shukoh organized by

Mr Francois Gautier at Indira Gandhi

National Centre for the Arts in New

Delhi

Q64) Ans (c)

The statue Gommateshwara is

dedicated to the Jain God Bahubali

It is a monolithic statue

President Ram Nath Kovind

inaugurated the grand anointing

ceremony mdash Mahamastakabhisheka mdash

held once in 12 years at

Shravanabelagola (Karnataka)

Q65) Ans (c)

Prachi Valley had come up around the

Prachi river Prachi Valley gradually

disappeared

RAUSIAS-FC19E1003 48

The Prachi river originates from

Bhubaneswar

It is a tributary of the Mahanadi and

flows through the districts of Puri

Khurda Cuttack and Jagatsinghpur

and the entire region of the river is

termed as the Prachi Valley

It falls into the Bay of Bengal

Archaeological evidence shows that the

Prachi Valley Civilisation predates both

Harappa and Mohenjo-Daro

The Prachi river originates from

Bhubaneswar

Q66) Ans (d)

These monuments are located in

Chhatarpur district Madhya Pradesh

within Vindhya mountain range

Q67) Ans (a)

The book lsquoThoughts on Pakistanrsquo was

written by Dr BR Ambedkar

On the occasion of the birth anniversary

of Dr BR Ambedkar the president of

India pays homage to this icon of India

In 1924 he founded the Depressed

Classes Institute (Bahishkrit Hitkarini

Sabha) and in 1927 the Samaj Samata

Sangh

Another area of attention for Ambedkar

was education For its spread among

the low classes he set up a network of

colleges by the name of Peoples

Education Society and founded hostels

Q68) Ans(b)

Mehrgarh is a famous Neolithic

settlement in the Indian subcontinent

which is situated in Baluchistan

province Pakistan

A pre-historic rock art site is discovered

in the vast expanse of limestone blocks

on the eastern banks of Naguleru river

near Dachepalli (Andhra Pradesh) It

has thrown light on the Neolithic

civilisation that flourished in Guntur

(Andhra Pradesh) during 1500-2000

BC

Q69) Ans (c)

The 12th and the 13th centuries saw

the emergence of the Kakatiyas They

were at first the feudatories of the

Western Chalukyas of Kalyana Initially

they ruled over a small territory near

Warangal (Telangana)

They introduced Nayakships which was

later adopted and developed by the

Rayas of Vijayanagara

Q70) Ans (a)

The fast had effect of putting pressure

on mill owners who finally agreed to

give the workers a 35 per cent increase

in wages

Google celebrated with a doodle the

132nd birth anniversary of Anasuya

Sarabhai who played a pioneering role

in Indiarsquos labour movement

Q71) Ans (d)

The UNESCOrsquos list of the representative

list of the intangible cultural heritage of

humanity from India are

- Koodiyattam Sanskrit Theatre of

Kerala

- Mudiyettu ritual theatre and dance

drama of Kerala

- Tradition of Vedic Chanting

- Kalbelia folk songs and dances of

Rajasthan

- Ramlila Traditional Performance of

the Ramayana

- Sankirtana ritual singing

drumming and dancing of Manipur

- Ramman religious festival and

ritual theatre of the Garhwal

Himalayas India

- Traditional brass and copper craft of

utensil making among the Thatheras

of Jandiala Guru Punjab India

- Chhau dance classical Indian dance

originated in the eastern Indian

states

- Buddhist chanting of Ladakh

recitation of sacred Buddhist texts

in the trans-Himalayan Ladakh

region Jammu and Kashmir India

- Yoga

- Nouroz

- Kumbh Mela

RAUSIAS-FC19E1003 49

Q72) Ans(b)

The President of India Shri Ram Nath Kovind inaugurated the Hornbill Festival and State Formation Day celebrations of Nagaland in Kisama

The festival is named after the Indian hornbill the large and colourful forest bird which is displayed in the folklore of most of the states tribes

The major recognized tribes of Nagaland are Angami Ao Chakhesang Chang

Kuki Rengma and Zeling etc

Onge Jarawa and Sentinelese are the

tribes of Andman amp Nicobar Islands

Q73) Ans (c)

The Rashtrakutas rule in the Deccan lasted for almost two hundred years till the end of the tenth century The Rashtrakutas rulers were tolerant in their religious views and patronized not only Shaivism and Vaishnavism but

Jainism as well

The famous rock-cut temple of Shiva at Ellora was built by one of the Rashtrakutas kings Krishna I in the ninth century His successor Amoghavarsha was a Jain but he also

patronized other faiths

The Rashtrakutas allowed Muslims traders to settle and permitted Islam to

be preached in their dominions

Recently increasing defacement at the prehistoric rock paintings of Pandavulagutta Telangana has created a cause for grave concern It can spoil

the prehistoric rock

Pandavulagutta is home to

- Painted rock shelters dating to

10000 BC-8000 BC

- An 8th century inscription of the

Rashtrakuta period and

- Painted frescoes from the 12th century Kakatiya empire

Q74) Ans (b)

In 1828 Raja Ram Mohan Roy founded a new religious society the Brahma Sabha later known as the Brahmo

Samaj

Debendranath Tagore headed the Tattvabodhini Sabha which was

engaged in search of spiritual truth

Its purpose was to purify Hinduism and to preach monotheism or belief in one God

The new society was to be based on the twin pillars of reason and the Vedas and

Upanishads

Recently Sadharan Brahmo Samaj (SBS) has entered into a legal battle with the West Bengal government due

to some legal issue

Q75) Ans (c)

The Chishti order was established in India by Khwaja Moinuddin Chishti who came to India around 1192 The Chishtirsquos are considered to be the most influential of the groups of Sufis who migrated to India in the late twelfth century They adapted successfully to the local environment and adopted several features of Indian devotional

traditions

The historical dargah of Sufi mystic Khwaja Moinuddin Chishti in Ajmer is all set to get a facelift This 13 th century dargah has been included among the Swachh Iconic Places a clean-up initiative focused on iconic

heritage spiritual and cultural places

Page 8: GENERAL STUDIES (PAPER I) · Test is part of Rau’s IAS Test series for Preliminary Exam 2019 FOUNDATION + CURRENT AFFAIRS GENERAL STUDIES (PAPER –I) FOUNDATION TEST –III TOPIC:

RAUSIAS-FC19E1003 8

Q20) निमननिखित कथि ो म स कौि-सास सही हह

1 बदधचररत का ििक िागसि ह

2 ब नरसतव की पजा महायाि बौदधमत का एक

महतवपरण भाग थी

िीच नदए गए कट का परय ग कर सही उततर चनिए

(a) किि 1

(b) किि 2

(c) 1 और 2 द ि ो

(d) ि त 1 ि ही 2

Q21) निमननिखित कथि ो म स कौि-सास सही हह

1 भखकत क निचार क भागित गीता म सपषट

नकया गया ह

2 तनमि कनि अपपर एक अििर सोत थ

िीच नदए गए कट का परय ग कर सही उततर चनिएः

(a) किि 1

(b) किि 2

(c) 1 और 2 द ि ो

(d) ि त 1 ि ही 2

Q22) निमननिखित म स नकस शासक ि िीरा बजात हए

अपिी छनि नसक ो पर अोनकत करिाई थी

(a) नमिाोदर

(b) चनदरगपत मौयण

(c) गौतमीपतर सतकरी

(d) समदरगपत

Q23) निमननिखित कथि ो म स कौि-सास सही हह

1 सवतोतर भारत ि निकरम सोित क राषटर ीय

किनडर क रप म अपिाया और यह 68 ईसा

पिण म आरि हआ था

2 बािभटट हिणिरणि क दरबार का एक कनि था

िीच नदए गए कट का परय ग कर सही उततर चनिएः

(a) किि 1

(b) किि 2

(c) 1 और 2 द ि ो

(d) ि त 1 ि ही 2

Q24) निमननिखित यग ो पर निचार कीनजएः

1 सोनर-निगरानहका वयापार मोतरी

2 परथम-कनिका परमि नशलपकार

3 साथणिाह परमि बकर

उपयणकत यग ो म स कौि-सास सही समनित हह

(a) किि 1

(b) किि 1 और 3

(c) किि 2

(d) किि 2 और 3

Q25) निमननिखित कथि ो म स कौि-सास सही हह

1 lsquoएह ि नशिाििrsquo पिकनशि नदवतीय स

सोबोनरत ह और रनिकनत क दवारा इसकी रचिा

की गई थी

2 जआि झाोग समदरगपत नदवतीय क शासिकाि

म भारत आया था

िीच नदए गए कट का परय ग कर सही उततर चनिएः

(a) किि 1

(b) किि 2

(c) 1 और 2 द ि ो

(d) ि त 1 ि ही 2

RAUSIAS-FC19E1003 9

Q20) Which of the following statements

isare correct

1 Buddhacharita is authored by

Nagasena

2 The worship of Bodhisattvas was

an important part of Mahayana

Buddhsim

Select the correct answer using the code

given below

(a) 1 only

(b) 2 only

(c) Both 1 and 2

(d) Neither 1 nor 2

Q21) Which of the following statements

isare correct

1 The idea of Bhakti is elucidated in

Bhagavata Gita

2 Tamil poet Appar was an Alvar

saint

Select the correct answer using the code

given below

(a) 1 only

(b) 2 only

(c) Both 1 and 2

(d) Neither 1 nor 2

Q22) Which of the following rulers had his

image inscribed in the coins while

playing a veena

(a) Meander

(b) Chandragupta Maurya

(c) Gautamiputra Satkarni

(d) Samudragupta

Q23) Which of the following statements

isare correct

1 Vikrama Samvat is adopted as the

national calendar by independent

India and it began in 68 BC

2 Banabhatta was a court poet of

Harshavardhana

Select the correct answer using the code

given below

(a) 1 only

(b) 2 only

(c) Both 1 and 2

(d) Neither 1 nor 2

Q24) Consider the following pairs

1 Sandhi-vigrahika Minister of trade

2 Prathama-kulika Chief craftsman

3 Sarthavaha Chief banker

Which of the pairs given above isare

correct

(a) 1 only

(b) 1 and 3 only

(c) 2 only

(d) 2 and 3 only

Q25) Which of the following statements

isare correct

1 Aihole inscription belongs to

Pulakeshin II and was composed

by Ravikriti

2 Xuan Zang came to India during

the reign of Chandragupta II

Select the correct answer using the code

given below

(a) 1 only

(b) 2 only

(c) Both 1 and 2

(d) Neither 1 nor 2

RAUSIAS-FC19E1003 10

Q26) िासतकिा स सोबोनरत निमननिखित ततव ो म स कौि-स

ततव किि नहोद मखनदर ो की िासतकिा क भाग ह

1 नशिर

2 मणडप

3 परदनकषरा पथ

4 गभणगह

िीच नदए गए कट का परय ग कर सही उततर चनिएः

(a) किि 1 3 और 4

(b) किि 2 3 और 4

(c) किि 1 2 और 4

(d) 1 2 3 और 4

Q27) निमननिखित मोनदर ो म स कौि-स मोनदर ईोट ो स बि ह

1 दिगढ़ मोनदर

2 भीतरगाि मोनदर

3 िकषमर मोनदर नसरपर

4 बहदशवर मोनदर

िीच नदए गए कट का परय ग कर सही उततर चनिएः

(a) किि 1 2 और 3

(b) किि 2 3 और 4

(c) किि 1 3 और 4

(d) 1 2 3 और 4

Q28) निमननिखित कथि ो म स कौि-सास सही हह

1 सलताि महममद किी कतब शाह अकबर का

समकािीि था

2 िासतकिा क कषतर म महममद किी कतब शाह

ि कई ईमारत ो का निमाणर करिाया था नजिम

स चार मीिार सिाणनरक परनसदध ह

िीच नदए गए कट का परय ग कर सही उततर चनिएः

(a) किि 1

(b) किि 2

(c) 1 और 2 द ि ो

(d) ि त 1 ि ही 2

Q29) निमननिखित यग ो पर निचार कीनजएः

1 मनिमकिाई सततिार

2 अनभजञाि शाको तिम कानिदास

3 नसिपपानदकारम क ििि

उपयणकत यग ो म स कौि-सास सही समनित हह

(a) किि 1 और 2

(b) किि 2

(c) किि 1 और 3

(d) 1 2 और 3

Q30) निमननिखित कथि ो म स कौि-सास सही हह

1 ldquoसशरत सोनहताrdquo नचनकरता पर एक महतवपरण

रचिा ह

2 बरहमगपत और चरक महतवपरण गनरतजञ थ

िीच नदए गए कट का परय ग कर सही उततर चनिए

(a) किि 1

(b) किि 2

(c) 1 और 2 द ि ो

(d) ि त 1 ि ही 2

Q31) अमीर िसर क सनदभण म निमननिखित कथि ो म स

कौि-सास सही हह

1 अमीर िसर ि अपिी रचिाओो म नििा ह

नक सोसकत नकसी भी कषतर स सोबोनरत िही ो थी

और किि बराहमर ही इस भािा का जञाि रित

2 उन ोि नहोदिी और अिरी क अखसततव का

उललि नकया था

िीच नदए गए कट का परय ग कर सही उततर चनिए

(a) किि 1

(b) किि 2

(c) 1 और 2 द ि ो

(d) ि त 1 ि ही 2

RAUSIAS-FC19E1003 11

Q26) Which of the following architectural

elements were only part of Hindu

temple architecture

1 Shikhara

2 Mandapa

3 Pradakshina patha

4 Garbhagriha

Select the correct answer using the code

given below

(a) 1 3 and 4 only

(b) 2 3 and 4 only

(c) 1 2 and 4 only

(d) 1 2 3 and 4

Q27) Which of the following temples isare

made of bricks

1 Deogarh Temple

2 Bhitargaon Temple

3 Lakshmana temple Sirpur

4 Brihadeshvara Temple

Select the correct answer using the code

given below

(a) 1 2 and 3 only

(b) 2 3 and 4 only

(c) 1 3 and 4 only

(d) 1 2 3 and 4

Q28) Which of the following statements

isare correct

1 Sultan Muhammad Quli Qutab

Shah was a contemporary of

Akbar

2 In the field of architecture

Muhammad Quli Qutab Shah

constructed many buildings the

most famous of which is the Char

Minar

Select the correct answer using the code

given below

(a) 1 only

(b) 2 only

(c) Both 1 and 2

(d) Neither 1 nor 2

Q29) Consider the following pairs

1 Manimekalai Sattanar

2 Abhijnana Shakuntalam Kalidasa

3 Silappadikaram Kovalan

Which of the pairs given above isare

correct

(a) 1 and 2 only

(b) 2 only

(c) 1 and 3 only

(d) 1 2 and 3

Q30) Which of the following statements

isare correct

1 Sushruta Samhita is an important

work on medicine

2 Brahmagupta and Charaka were

important mathematicians

Select the correct answer using the code

given below

(a) 1 only

(b) 2 only

(c) Both 1 and 2

(d) Neither 1 nor 2

Q31) Which of the following statements

isare correct about Amir Khusrau

1 Amir Khusrau records in his works

that Sanskrit did not belong to any

region and only the Brahmans

knew it

2 He recorded the existence of

Hindawi and Awadhi

Select the correct answer using the code

given below

(a) 1 only

(b) 2 only

(c) Both 1 and 2

(d) Neither 1 nor 2

RAUSIAS-FC19E1003 12

Q32) निमननिखित कथि ो पर निचार कीनजए

1 नहरणय-गभण अिषठाि क बार म ऐसा स चा जाता

था नक बनि दि िाि का एक कषनतरय क रप म

पिजणनम ह गा

2 मयरशमणि कदोब िोश का सोसथापक था

उपयणकत कथि ो म स कौि-सास सही हह

(a) किि 1

(b) किि 2

(c) 1 और 2 द ि ो

(d) ि त 1 ि ही 2

Q33) निमननिखित कथि ो म स कौि-सास सही हह

1 कदमई बगार (बिपिणक शरम) क रप म

निया जाि िािा कर था

2 गवानियर परशखसत म िागभट (ज एक चोदि

राजा था) क दवारा नकय गए श िर का िरणि

नकया गया ह

िीच नदए गए कट का परय ग कर सही उततर चनिए

(a) किि 1

(b) किि 2

(c) 1 और 2 द ि ो

(d) ि त 1 ि ही 2

Q34) निमननिखित कथि ो म स कौि-सास सही हह

1 राजतरो नगिी 11िी ो शताबदी म कलहि क दवारा

रनचत एक सोसकत पसतक (टकसट) ह

2 कननौज क निए नतरपकषीय सोघिण म पाि राजिोश

शानमि था

िीच नदए गए कट का परय ग कर सही उततर चनिए

(a) किि 1

(b) किि 2

(c) 1 और 2 द ि ो

(d) ि त 1 ि ही 2

Q35) निमननिखित यग ो पर निचार कीनजए

1 बरहदशवर मोनदर राजराजा च ि

2 उर मापि की इकाई

3 दिदाि मोनदर ो क भनम अिदाि

उपयणकत यग ो म स कौि-स सही समनित ह

(a) किि 1 और 2

(b) किि 2 और 3

(c) किि 1 और 3

(d) 1 2 और 3

Q36) निमननिखित कथि ो म स कौि-सास सही हह

1 नदलली क सलताि ो क अरीि परशासि की भािा

फारसी थी

2 नदलली सलतित म ldquoतारीितािरीिrdquo कनिता

का एक रप था

िीच नदए गए कट का परय ग कर सही उततर चनिए

(a) किि 1

(b) किि 2

(c) 1 और 2 द ि ो

(d) ि त 1 ि ही 2

Q37) निमननिखित कथि ो म स कौि-सास सही हह

1 अिाउददीि खििजी ि अपि सनिक ो क निए

नसरी िाम का एक िया दगणरकषक शहर

बिािाया था

2 िह अपि सनिक ो क िति का भगताि इकता

क रप म करता था

िीच नदए गए कट का परय ग कर सही उततर चनिए

(a) किि 1

(b) किि 2

(c) 1 और 2 द ि ो

(d) ि त 1 ि ही 2

RAUSIAS-FC19E1003 13

Q32) Consider the following statements

1 Hiranya-garbha ritual was thought

to lead to the rebirth of the

sacrificer as a Kshatriya

2 Mayurasharman was the founder

of the Kadamba dynasty

Which of the statements given above

isare correct

(a) 1 only

(b) 2 only

(c) Both 1 and 2

(d) Neither 1 nor 2

Q33) Which of the following statements

isare correct

1 Kadamai was tax taken in form of

forced labour

2 Gwalior Prashasti describes the

exploits of Nagabhata who was a

Chandella king

Select the correct answer using the code

given below

(a) 1 only

(b) 2 only

(c) Both 1 and 2

(d) Neither 1 nor 2

Q34) Which of the following statements

isare correct

1 Rajatarangini is a Sanskrit text

written by Kalhana in the 11th

century

2 Pala dynasty was included in the

tripartite struggle for Kannauj

Select the correct answer using the code

given below

(a) 1 only

(b) 2 only

(c) Both 1 and 2

(d) Neither 1 nor 2

Q35) Consider the following pairs

1 Brihadeshvara temple Rajaraja

Chola

2 ldquoUrrdquo Unit of measurement

3 Devadana Land grants made to

temples

Which of the pairs given above isare

correct

(a) 1 and 2 only

(b) 2 and 3 only

(c) 1 and 3 only

(d) 1 2 and 3

Q36) Which of the following statements

isare correct

1 The language of administration

under the Delhi Sultans was

Persian

2 Tarikhtawarikh was a form of

poetry in the Delhi Sultanate

Select the correct answer using the code

given below

(a) 1 only

(b) 2 only

(c) Both 1 and 2

(d) Neither 1 nor 2

Q37) Which of the following statements

isare correct

1 Alauddin Khilji constructed a new

garrison town named Siri for his

soldiers

2 He paid his soldiers their salaries

in the form of Iqta

Select the correct answer using the code

given below

(a) 1 only

(b) 2 only

(c) Both 1 and 2

(d) Neither 1 nor 2

RAUSIAS-FC19E1003 14

Q38) निमननिखित कथि ो म स कौि-सास सही हह

1 नदलली कतबददीि एबक क अरीि पहिी बार

नकसी सामराजय की राजरािी बिी थी

2 दहिीिाि नसक ो का मदरर मग़ि ो क दवारा

नकया गया था

िीच नदए गए कट का परय ग कर सही उततर चनिए

(a) किि 1

(b) किि 2

(c) 1 और 2 द ि ो

(d) ि त 1 ि ही 2

Q39) निमननिखित यग ो पर निचार कीनजए

1 म ठ की मखिद नसको दर ि दी

2 बगमपरी मखिद नफर ज शाह तगिक

3 कववत- अि - इसलाम कतबददीि ऐबक

उपयणकत यग ो म स कौि-स सही समनित ह

(a) किि 1 और 2

(b) किि 2 और 3

(c) किि 1 और 3

(d) 1 2 और 3

Q40) निमननिखित कथि ो म स कौि-सास सही हह

1 मिसबदार ो क अपिा िति राजसव कायो

नजन जागीर कहत थ क रप म परापत ह ता

था

2 मिसबदार क ज सनय उततरदानयतव सौोप जात

थ उसक अनतगणत उस एक निराणररत सखया म

सिार अथिा घड़सिार ो का रि-रिाि करिा

पड़ता था

िीच नदए गए कट का परय ग कर सही उततर चनिए

(a) किि 1

(b) किि 2

(c) 1 और 2 द ि ो

(d) ि त 1 ि ही 2

Q41) ldquo1942 क भारत छ ड़ आोद ििrdquo क बार म

निमननिखित अिि कि ो म स कौि-सा सतय िही ो ह

(a) यह एक अनहोसक आोद िि था

(b) इसका िततव महातमा गाोरी क दवारा नकया गया

था

(c) यह एक सवाभानिक आोद िि था

(d) इसि सामानयतया शरनमक िगण क आकनिणत

िही ो नकया था

Q42) भारत क ि ग ो ि ldquoसाइमि कमीशिrdquo क आगमि क

निरदध आोद िि नकया था कय ोनक

(a) भारतीय कभी भी 1919 क अनरनियम (The

Act of 1919) क काम की समीकषा िही ो करिा

चाहत थ

(b) साइमि कमीशि ि पराोत ो म दवर (द हर) शासि

क समापत करि की नसफाररश की थी

(c) साइमि कमीशि म क ई भारतीय सदसय िही ो

था

(d) साइमि कमीशि ि दश क निभाजि का

सझाि नदया था

Q43) निमननिखित कथि ो पर निचार कीनजए

भारतीय राषटर ीय आोद िि म दादाभाई िौर जी क दवारा

नकया गया सबस परभािी य गदाि यह था नक उन ोि

1 अोगरज ो क दवारा भारत क आनथणक श िर का

ििासा नकया था

2 पराचीि भारतीय गरोथ ो की वयाखया की थी और

भारतीय ो क आतमनिशवास क पिःसथानपत नकया

था

3 अनय नकसी भी बात स पहि सभी सामानजक

बराइय ो क उनमिि की आिशयकता पर बि

नदया था

उपयणकत कथि ो म स कौि-सास सही हह

(a) किि 1

(b) किि 2 और 3

(c) किि 1 और 3

(d) 1 2 और 3

RAUSIAS-FC19E1003 15

Q38) Which of the following statements

isare correct

1 Delhi first became the capital of a

kingdom under Qutubuddin

Aibak

2 Dehliwal coins were minted by the

Mughals

Select the correct answer using the code

given below

(a) 1 only

(b) 2 only

(c) Both 1 and 2

(d) Neither 1 nor 2

Q39) Consider the following pairs

1 Moth ki Masjid- Sikander Lodi

2 Begumpuri mosque- Firuz Shah

Tughluq

3 Quwwat al ndash Islam- Qutubuddin

Aibak

Which of the above pairs isare correct

(a) 1 and 2 only

(b) 2 and 3 only

(c) 1 and 3 only

(d) 1 2 and 3

Q40) Which of the following statements

isare correct

1 Mansabdars received their salaries

as revenue assignments called

jagirs

2 The mansabdarrsquos military

responsibilities required him to

maintain a specified number of

sawar or cavalrymen

Select the correct answer using the code

given below

(a) 1 only

(b) 2 only

(c) Both 1 and 2

(d) Neither 1 nor 2

Q41) Which one of the following observations

is not true about the Quit India

Movement of 1942

(a) It was a non-violent movement

(b) It was led by Mahatma Gandhi

(c) It was a spontaneous movement

(d) It did not attract the labour class

in general

Q42) The people of India agitated against the

arrival of the Simon Commission

because

(a) Indians never wanted the review of

the working of the Act of 1919

(b) Simon Commission recommended

the abolition of dyarchy in the

Provinces

(c) there was no Indian member in the

Simon Commission

(d) the Simon Commission suggested

the partition of the country

Q43) Consider the following statements

The most effective contribution made by

Dadabhai Naoroji to the cause of Indian

National Movement was that he-

1 exposed the economic exploitation

of India by the British

2 interpreted the ancient Indian

texts and restored the self-

confidence of Indians

3 stressed the need for eradication of

all the social evils before anything

else

Which of the statements given above

isare correct

(a) 1 only

(b) 2 and 3 only

(c) 1 and 3 only

(d) 1 2 and 3

RAUSIAS-FC19E1003 16

Q44) महातमा गाोरी ि 1932 म आमरर अिशि नकया था

कय ोनक

(a) ldquoग िमज सममििrdquo (The Round Table

Conference) भारतीय राजिीनतक

आकाोकषाओो क परा करि म असफि रहा था

(b) काोगरस और मखसलम िीग म मतभद थ

(c) रामस मकड िालड (Ramsay Macdonald)

ि ldquoसाोपरदानयक परसकारrdquo (The Communal

Award) की घ िरा की थी

(d) ldquoसनििय अिजञा आोद ििrdquo (The Civil

Disobedience Movement) असफि रहा

था

Q45) भारत म औपनििनशक शासि की अिनर क सोदभण म

भारत स रि क बनहगणमि का एक महतवपरण भाग गह

शलक (Home Charges) था निमननिखित म स

कौि-सास क ि गह शलक म सखममनित नकया गया

थानकय गए थ

1 िोदि म भारत कायाणिय क निए उपय ग नकय

जाि िािा क ि

2 भारत म नियकत नबरनटश कनमणय ो क िति और

पशि का भगताि करि क निए उपय ग नकय

जाि िािा क ि

3 अोगरज ो क दवारा भारत क बाहर यदध ो क निए

उपय ग नकय जाि िािा क ि

िीच नदए गए कट का परय ग कर सही उततर चनिए

(a) किि 1

(b) किि 1 और 2

(c) किि 2 और 3

(d) 1 2 और 3

Q46) सवतोतरता आोद िि क इनतहास म भारतीय राषटर ीय

काोगरस का 1929 का सतर महतवपरण ह कय ोनक इसम

(a) काोगरस क उददशय क रप म सथािीय सरकार

की पराखपत की घ िरा की गई थी

(b) परण सवराज की पराखपत क काोगरस क िकषय क

रप म अपिाया गया था

(c) असहय ग आोद िि शर नकया गया था

(d) िोदि म ldquoग ि मर सममििrdquo (The Round

Table Conference) म भाग िि का निरणय

निया गया था

Q47) भारतीय सवतोतरता सोगराम क सोदभण म िहर ररप टण

क दवारा निमननिखित म स नकसकी नसफाररश की गई

थीनकिकी नसफाररश की गई थी ो

1 भारत क निए परण सवतोतरता

2 अलपसोखयक ो क निए सीट ो क आरकषर क

निए सोयकत नििाणचक मोडि

3 सोनिराि म भारत क ि ग ो क निए मौनिक

अनरकार ो का परािराि

िीच नदए गए कट का परय ग कर सही उततर चनिए

(a) किि 1

(b) किि 2 और 3

(c) किि 1 और 3

(d) 1 2 और 3

Q48) आरो नभक िनदक आयो का रमण मखय रप स था

(a) भखकत

(b) मनतण पजा और यजञ

(c) परकनत की पजा और यजञ

(d) परकनत की पजा और भखकत

RAUSIAS-FC19E1003 17

Q44) Mahatma Gandhi undertook fast unto

death in 1932 mainly because

(a) The Round Table Conference failed

to satisfy Indian political

aspirations

(b) The Congress and Muslim League

had differences of opinion

(c) Ramsay Macdonald announced the

Communal Award

(d) The Civil Disobedience Movement

failed

Q45) With reference to the period of colonial

rule in India ldquoHome Chargesrdquo formed

an important part of drain of wealth

from India Which of the following funds

constituted ldquoHome Chargesrdquo

1 Funds used to support the India

Office in London

2 Funds used to pay salaries and

pensions of British personnel

engaged in India

3 Funds used for waging wars

outside India by the British

Select the correct answer using the code

given below

(a) 1 only

(b) 1 and 2 only

(c) 2 and 3 only

(d) 1 2 and 3

Q46) The 1929- Session of Indian National

Congress is of significance in the history

of the Freedom Movement because the-

(a) attainment of Self-Government

was declared as the objective of

the Congress

(b) attainment of Poorna Swaraj was

adopted as the goal of the

Congress

(c) Non-Cooperation Movement was

launched

(d) decision to participate in the

Round Table Conference in

London was taken

Q47) With reference to the period of Indian

freedom struggle which of the following

waswere recommended by the Nehru

report

1 Complete Independence for India

2 Joint electorates for reservation of

seats for minorities

3 Provision of fundamental rights for

the people of India in the

Constitution

Select the correct answer using the code

given below

(a) 1 only

(b) 2 and 3 only

(c) 1 and 3 only

(d) 1 2 and 3

Q48) The religion of the early Vedic Aryans was primarily of

(a) Bhakti

(b) image worship and Yajnas

(c) worship of nature and Yajnas

(d) worship of nature and Bhakti

RAUSIAS-FC19E1003 18

Q49) भारत की यातरा करि िाि चीिी यातरी यआि चिाोग

(हयएि साोग) ि समकािीि भारत की सामानय

खसथनतय ो और सोसकनत क दजण नकया था इस सोदभण म

निमननिखित कथि ो म स कौि-सास सही हह

1 सड़क और िदी-मागण (जि-मागण) डकती स

परण रप स सरनकषत थ

2 जहा तक अपरार ो क निए दणड की बात ह

उसक निए नकसी भी वयखकत की निदोिता

अथिा उसक अपरार क निराणररत करि क

निए अनि जि और निि परि क माधयम क

सारि थ

3 वयापाररय ो क घाट ो और परनतबोर सटशि ो पर

शलक ो का भगताि करिा पड़ता था

िीच नदए गए कट का परय ग कर सही उततर चनिए

(a) किि 1

(b) किि 2 और 3

(c) किि 1 और 3

(d) 1 2 और 3

Q50) नसोर घाटी सभयता क सोदभण म निमननिखित कथि ो पर

निचार कीनजए

1 यह मखय रप स एक रमणनिरपकष सभयता थी

तथा हािाोनक इसम रानमणक ततव मौजद था

िनकि िह परनतिश पर हािी िही ो था

2 इस काि क दौराि भारत म कपास का परय ग

कपड़ा बिाि क निए नकया जाता था

उपयणकत कथि ो म स कौि-सास सही हह

(a) किि 1

(b) किि 2

(c) 1 और 2 द ि ो

(d) ि त 1 ि ही 2

Q51) परोदर दास क सोदभण म निमननिखित कथि ो पर निचार

कीनजए

1 परोदर दास एक सोत और भगिाि नशि क एक

महाि भकत थ

2 ि एक सोगीतकार गायक और किाणटक सोगीत

क मखय सोसथापक-परसतािक ो म स एक थ

उपयणकत कथि ो म स कौि-सास सही हह

(a) किि 1

(b) किि 2

(c) 1 और 2 द ि ो

(d) ि त 1 ि ही 2

Q52) निमननिखित म स कौि-सास वयखकत किाणटक सोगीत

की नतरमनतण म शानमि हह

1 बािामरिी कषणा

2 शरी शयाम शासतरी

3 शरी मथसवामी दीनकषतर

िीच नदए गए कट का परय ग कर सही उततर चनिए

(a) किि 1

(b) किि 2

(c) किि 2 और 3

(d) 1 2 और 3

Q53) चियर (Chevayur) और अथ िी (Atholi) म खसथत

महापािार सथि निमननिखित म स नकस राजय म खसथत

(a) तनमििाड

(b) किाणटक

(c) पनिम बोगाि

(d) करि

RAUSIAS-FC19E1003 19

Q49) The Chinese traveller Yuan Chwang

(Hiuen Tsang) who visited India

recorded the general conditions and

culture of India at that time In this

context which of the following

statements isare correct

1 The roads and river-routes were

completely immune from robbery

2 As regards punishment for

offences ordeals by fire water and

poison were the instruments for

determining the innocence or guilt

of a person

3 The tradesmen had to pay duties

at ferries and barrier stations

Select the correct answer using the code

given below

(a) 1 only

(b) 2 and 3 only

(c) 1 and 3 only

(d) 1 2 and 3

Q50) Regarding the Indus Valley Civilization

consider the following statements

1 It was predominantly a secular

civilization and the religious

element though present did not

dominate the scene

2 During this period cotton was

used for manufacturing textiles in

India

Which of the statements given above

isare correct

(a) 1 only

(b) 2 only

(c) Both 1 and 2

(d) Neither 1 nor 2

Q51) Consider the following statements

regarding Purandara Dasa

1 Purandara Dasa was a saint and

great devotee of Lord Shiva

2 He was a composer singer and

one of the chief founding-

proponents of the Carnatic music

Which of the statements given above

isare correct

(a) 1 only

(b) 2 only

(c) Both 1 and 2

(d) Neither 1 nor 2

Q52) Which of the following persons isare

included in the trinity of Carnatic

music

1 Balamurali Krishna

2 Sri Shyama Shastry

3 Sri Muthuswami Dikshitar

Select the correct answer using the code

given below

(a) 1 only

(b) 2 only

(c) 2 and 3 only

(d) 1 2 and 3

Q53) Megalithic sites at Chevayur and Atholi

are located in which of the following

states

(a) Tamil Nadu

(b) Karnataka

(c) West Bengal

(d) Kerala

RAUSIAS-FC19E1003 20

Q54) निमननिखित कथि ो पर निचार कीनजए

1 महापािानरक ि ग कबर ो म िसतएो दफिात थ

2 दनकषर भारत म महापािार सोसकनत एक परण

निकनसत तामर यगीि सोसकनत थी

उपयणकत कथि ो म स कौि-सास सही हह

(a) किि 1

(b) किि 2

(c) 1 और 2 द ि ो

(d) ि त 1 ि ही 2

Q55) निमननिखित म स कौि-स सामराजयसामराजय ो का

अश क क अनभिि ो म उललि नकया गया ह

1 च ि

2 पाणडय

3 करिपतर (चर)

िीच नदए गए कट का परय ग कर सही उततर चनिए

(a) किि 1

(b) किि 1 और 2

(c) किि 3

(d) 1 2 और 3

Q56) भीमा-क रगाोि का यदध को पिी क सनिक ो और

बाजीराि नदवतीय क िततव म एक शखकतशािी पशिा

सिा (मराठ ो) क मधय िड़ा गया था यह यदध

निमननिखित म स नकसका नहससा था

(a) परथम आोगल-मराठा यदध का

(b) नदवतीय आोगल-मराठा यदध का

(c) ततीय आोगल-मसर यदध का

(d) ततीय आोगल-मराठा यदध का

Q57) निमननिखित कथि ो पर निचार कीनजए

1 महादि दसाई ि गाोरीजी क चोपारर आि तथा

नतिकनथया पररािी स जड़ी समसया की जाोच

क निए रारी करि क निए दश भर म उिका

अिसरर नकया था

2 िरहरी पाररि चोपारर सतयागरह क दौराि

गाोरीजी क साथ थ

उपयणकत कथि ो म स कौि-सास सही हह

(a) किि 1

(b) किि 2

(c) 1 और 2 द ि ो

(d) ि त 1 ि ही 2

Q58) निमननिखित कथि ो पर निचार कीनजए

1 िनद राज-िोश ि बराहमर ो और बौदध मठराररय ो

क कर-मकत गाि अिदाि म दि की परथा

आरि की थी

2 सतिाहि ो की आनरकाररक भािा पराकत थी

उपयणकत कथि ो म स कौि-सास सही हह

(a) किि 1

(b) किि 2

(c) 1 और 2 द ि ो

(d) ि त 1 ि ही 2

Q59) एक निरासत क अपिाइए (अडॉपट ए हररटज ndash

Adopt a Heritage) पररय जिा क उददशय ो क

सनदभण म निमननिखित कथि ो पर निचार कीनजए

1 यह पररय जिा र रगार उतपादि और आनथणक

निकास क निए पयणटि कषमता का उि पर

परभाि का उपय ग करगी

2 यह पररय जिा निरासत सथि ो पर निशव सतरीय

आराररक सोरचिा निकनसत करक एक सतत

तरीक स पयणटक आकिणर म िखदध करगी

उपयणकत कथि ो म स कौि-सास सही हह

(a) किि 1

(b) किि 2

(c) 1 और 2 द ि ो

(d) ि त 1 ि ही 2

RAUSIAS-FC19E1003 21

Q54) Consider the following statements

1 Megalithic people buried goods in

graves

2 The megalithic culture in South

India was a full-fledged Copper

Age culture

Which of the statements given above

isare correct

(a) 1 only

(b) 2 only

(c) Both 1 and 2

(d) Neither 1 nor 2

Q55) Which of the following kingdoms isare

mentioned in the Ashokan inscriptions

1 Cholas

2 Pandyas

3 Keralaputras (Cheras)

Select the correct answer using the code

given below

(a) 1 only

(b) 1 and 2 only

(c) 3 only

(d) 1 2 and 3

Q56) The Battle of Bhima-Koregaon was

fought between the soldiers of the

Company and the strong Peshwa army

(Marathas) under Bajirao II This war

was a part of the

(a) First Anglo-Maratha war

(b) Second Anglo-Maratha war

(c) Third Anglo- Mysore war

(d) Third Anglo-Maratha war

Q57) Consider the following statements

1 Mahadev Desai followed Gandhiji all over the country to persuade him to come to Champaran to investigate the problem associated

with tinkathia system

2 Narhari Parikh accompanied Gandhi ji during the Champaran

Satyagraha

Which of the statements given above isare correct

(a) 1 only

(b) 2 only

(c) Both 1 and 2

(d) Neither 1 nor 2

Q58) Consider the following statements

1 The Nanda Dynasty started the practice of granting tax-free villages to brahmanas and

Buddhist monks

2 The official language of the Satavahanas was Prakrit

Which of the statements given above

isare correct

(a) 1 only

(b) 2 only

(c) Both 1 and 2

(d) Neither 1 nor 2

Q59) Consider the following statements about the objectives of the lsquoadopt a heritagersquo

project

1 It will harness tourism potential for its effects on employment generation and economic

development

2 It will enhance the tourist attractiveness in a sustainable manner by developing world class infrastructure at heritage sites

Which of the statements given above

isare correct

(a) 1 only

(b) 2 only

(c) Both 1 and 2

(d) Neither 1 nor 2

RAUSIAS-FC19E1003 22

Q60) ldquoभारतीय जिजातीय सहकारी निपरि निकास सोघrdquo

(The Tribal Co-operative Marketing

Development Federation of India - TRIFED)

क सोदभण म निमननिखित कथि ो पर निचार कीनजए

1 यह एक राषटर ीय सतर का शीिण सोगठि ह ज

भारत सरकार क गह मोतरािय क परशासनिक

नियोतरर क अरीि काम कर रहा ह

2 इसका मखय उददशय दश म जिजातीय ि ग ो

का सामानजक-आनथणक निकास करिा ह

उपयणकत कथि ो म स कौि-सास सही हह

(a) किि 1

(b) किि 2

(c) 1 और 2 द ि ो

(d) ि त 1 ि ही 2

Q61) निमननिखित म स कौि-सास उपनयास परमचोद क

दवारा नििा गया हनिि गए ह

1 रोगभनम

2 ग दाि

3 ग रा

िीच नदए गए कट का परय ग कर सही उततर चनिए

(a) किि 1

(b) किि 2

(c) किि 1 और 2

(d) 1 2 और 3

Q62) नगदधा ितय क सोदभण म निमननिखित कथि ो पर निचार

कीनजए

1 नगदधा नबहार की मनहिाओो क दवारा तयौहार क

समय और फसि की बिाई तथा कटाई क

अिसर पर नकया जाि िािा एक पारोपररक

दहाती ितय ह

2 इस ितय क दवारा मनहिाऐो अपिी परसननता

परकट करती ह तथा नगदधा क परदशणि क

माधयम स परि िचणसव िाि समाज म

मनहिाओो की दबी हई भाििाओो क परकट

करती ह

उपयणकत कथि ो म स कौि-सास सही हह

(a) किि 1

(b) किि 2

(c) 1 और 2 द ि ो

(d) ि त 1 ि ही 2

Q63) निमननिखित कथि ो पर निचार कीनजए

1 मलला शाह बदखशी दारा नशक ह क

आधयाखतमक गर थ

2 औरोगरब ि मजम-उि-बहरीि या द समदर ो

का सोगम िामक उललििीय रचिा नििी थी

3 दारा नशक ह क अपि पिणज अकबर क गर ो

क उततरानरकारी क रप म दिा गया था

नजसम उसि रानमणक बहििाद और समनवयता

क बढ़ािा नदया था

उपयणकत कथि ो म स कौि-सास सही हह

(a) किि 1 और 3

(b) किि 2

(c) किि 1 और 2

(d) 1 2 और 3

RAUSIAS-FC19E1003 23

Q60) Consider the following statements about

the Tribal Cooperative Marketing

Development Federation of India

(TRIFED)

1 It is a national-level apex

organization functioning under the

administrative control of Ministry

of Home Affairs Government of

India

2 The main objective of TRIFED is

socio-economic development of

tribal people in the country

Which of the statements given above

isare correct

(a) 1 only

(b) 2 only

(c) Both 1 and 2

(d) Neither 1 nor 2

Q61) Which of the following novels isare

written by Premchand

1 Rangabhumi

2 Godan

3 Gora

Select the correct answer using the code

given below

(a) 1 only

(b) 2 only

(c) 1 and 2 only

(d) 1 2 and 3

Q62) Consider the following statements about

Giddha dance

1 Giddha is a traditional pastoral

dance performed by the women of

Bihar at festival times and at the

sowing and reaping of the harvest

2 By this dance the women reveal

their joy expel their suppressed

feelings in a male dominated

society through the performance of

Giddha

Which of the statements given above

isare correct

(a) 1 only

(b) 2 only

(c) Both 1 and 2

(d) Neither 1 nor 2

Q63) Consider the following statements

1 Mullah Shah Badakhshi was the

spiritual mentor of Dara Shukoh

2 Aurangzeb wrote the remarkable

work called ldquoMajma-ul-Bahrainrdquo or

the ldquoThe confluence of two seasrdquo

3 Dara Shukoh was seen as

inheriting the qualities of his

ancestor Akbar in that he

promoted religious pluralism and

syncretism

Which of the statements given above

isare correct

(a) 1 and 3 only

(b) 2 only

(c) 1 and 2 only

(d) 1 2 and 3

RAUSIAS-FC19E1003 24

Q64) निमननिखित कथि ो पर निचार कीनजए

1 ग मतशवर परनतमा निोधयनगरी पहाड़ी पर खसथत ह

2 शरिरबिग िा िह सथाि ह जहाो मौयण िोश क

सोसथापक चोदरगपत मौयण अपि नसोहासि क

तयागि क बाद जि तपसवी बि गए थ

उपयणकत कथि ो म स कौि-सास सही हह

(a) किि 1

(b) किि 2

(c) 1 और 2 द ि ो

(d) ि त 1 ि ही 2

Q65) निमननिखित कथि ो पर निचार कीनजए

1 पराताखतवक साकषय स पता चिता ह नक पराची

घाटी सभयता हड़पपा और म हिज दाड़ द ि ो

की पिणिती ह

2 पराची िदी भििशवर स निकिती ह

उपयणकत कथि ो म स कौि-सास सही हह

(a) किि 1

(b) किि 2

(c) 1 और 2 द ि ो

(d) ि त 1 ि ही 2

Q66) निमननिखित कथि ो म स कौि-सास सही हह

1 िजराह क समारक ो क समह का निमाणर

चोदि राजिोश क शासिकाि क दौराि हआ

था

2 य समारक हररिोदर पिणत शरोििा म खसथत ह

3 म रक क यातरी इबन बतता ि अपि सोसमरर ो

म िजराह क मोनदर ो की यातरा का उललि

नकया था तथा इन काजराण िाम स समब नरत

नकया था

िीच नदए गए कट का परय ग कर सही उततर चनिए

(a) किि 1

(b) किि 1 और 2

(c) किि 2 और 3

(d) किि 1 और 3

Q67) निमननिखित कथि ो म स कौि-सास सही हह

1 डॉ बी आर अमबडकर ि दी एनिनहिशि

ऑफ़ कासट (The Annihilation of Caste)

नििी थी नजसम उन ोि नहोद रमण म िोशािगत

पजारी की परथा क उनमिि की आिशयकता

पर बि नदया था

2 डॉ राजदर परसाद ि थॉटस ऑि पानकसताि

(Thoughts on Pakistan) िामक पसतक

नििी थी

िीच नदए गए कट का परय ग कर सही उततर चनिए

(a) किि 1

(b) किि 2

(c) 1 और 2 द ि ो

(d) ि त 1 ि ही 2

Q68) निमननिखित कथि ो म स कौि-सास सही हह

1 महरगढ़ भारतीय उपमहादवीप म एक परनसदध

ििपािार बसती ह ज नसोर पराोत पानकसताि म

खसथत ह

2 बरणह म म कतत ो क उिक सवामी क साथ कबर ो

म दफिाया जाता था

िीच नदए गए कट का परय ग कर सही उततर चनिए

(a) किि 1

(b) किि 2

(c) 1 और 2 द ि ो

(d) ि त 1 ि ही 2

Q69) निमननिखित कथि ो म स कौि-सास सही हह

1 काकानटय मोनदर अनरकतर नशि क समनपणत

2 हिमक ोडा म हजार-सतोभ िाि मोनदर (The

Thousand-Pillared Temple) का निमाणर

काकानटय समराट रदर ि करिाया था

िीच नदए गए कट का परय ग कर सही उततर चनिए

(a) किि 1

(b) किि 2

(c) 1 और 2 द ि ो

(d) ि त 1 ि ही 2

RAUSIAS-FC19E1003 25

Q64) Consider the following statements

1 Gommateshwara Statue is located

on the Vindyagiri Hill

2 Shravanabelagola is the place

where Chandragupta Maurya the

founder of the Mauryan dynasty

became a Jain ascetic after

relinquishing his throne

Which of the statements given above

isare correct

(a) 1 only

(b) 2 only

(c) Both 1 and 2

(d) Neither 1 nor 2

Q65) Consider the following statements

1 Archaeological evidence shows

that the Prachi Valley Civilisation

predates both Harappa and

Mohenjo-Daro

2 The Prachi river originates from

Bhubaneswar

Which of the statements given above

isare correct

(a) 1 only

(b) 2 only

(c) Both 1 and 2

(d) Neither 1 nor 2

Q66) Which of the following statements

isare correct

1 The Khajuraho group of

monuments was built during the

rule of the Chandela dynasty

2 These monuments are located in

Harischandra mountain range

3 Ibn Battuta the Moroccan

traveller in his memoirs mentioned

visiting Khajuraho temples and

called them Kajarra

Select the correct answer using the code

given below

(a) 1 only

(b) 1 and 2

(c) 2 and 3

(d) 1 and 3

Q67) Which of the following statements

isare correct

1 Dr BR Ambedkar wrote the

Annihilation of Caste emphasising

the need to do away with the

practice of hereditary priesthood in

Hinduism

2 The book lsquoThoughts on Pakistanrsquo

was written by Dr Rajendra

Prasad

Select the correct answer using the code

given below

(a) 1 only

(b) 2 only

(c) Both 1 and 2

(d) Neither 1 nor 2

Q68) Which of the following statements

isare correct

1 Mehrgarh is a famous Neolithic

settlement in the Indian

subcontinent which is situated in

Sindh province Pakistan

2 At Burzahom dogs were buried

with their masters in their graves

Select the correct answer using the code

given below

(a) 1 only

(b) 2 only

(c) Both 1 and 2

(d) Neither 1 nor 2

Q69) Which of the following statements

isare correct

1 The Kakatiya temples are

dedicated mostly to Siva

2 The Thousand-Pillared Temple at

Hanamkonda was built by the

Kakatiya king Rudra

Select the correct answer using the code

given below

(a) 1 only

(b) 2 only

(c) Both 1 and 2

(d) Neither 1 nor 2

RAUSIAS-FC19E1003 26

Q70) निमननिखित कथि ो म स कौि-सास सही हह

1 अहमदाबाद नमि हड़ताि क दौराि महातमा

गाोरी ि शरनमक ो क पकष क मजबत करि क

निए आमरर अिशि नकया था

2 अिशि स नमि मानिक ो पर दबाि पड़ा था ज

अोततः शरनमक ो क िति म 15 परनतशत की िखदध

करि क निए सहमत हए थ

िीच नदए गए कट का परय ग कर सही उततर चनिए

(a) किि 1

(b) किि 2

(c) 1 और 2 द ि ो

(d) ि त 1 ि ही 2

Q71) निमननिखित म स नकसक नकिक भारत स यिसक

की माििता की अमतण साोसकनतक निरासत की

परनतनिनर सची (The UNESCOrsquos List of the

Representative List of the Intangible

Cultural Heritage of Humanity) म शानमि

नकया गया ह

1 मनडयटट

2 सोकीतणि

3 को भ मिा

िीच नदए गए कट का परय ग कर सही उततर चनिए

(a) किि 1 और 2

(b) किि 2 और 3

(c) किि 3

(d) 1 2 और 3

Q72) निमननिखित जिजानतय ो म स कौि-सीसी ो

जिजानतजिजानतया िागािड स सोबोनरत हह

1 अोगामी

2 ककी

3 जारिा

िीच नदए गए कट का परय ग कर सही उततर चनिए

(a) किि 1

(b) किि 1 औऔ 2

(c) किि 2

(d) 1 2 और 3

Q73) निमननिखित कथि ो म स कौि-सास सही हह

1 राषटर कट सामराजय की सथापिा दोनतदगण ि की थी

नजसि मानयाित म अपिी राजरािी की

सथापिा की थी

2 राषटर कट समराट अम घििण एक ििक था और

उस कनिताओो पर पहिी कननड़ पसतक नििि

का शरय नदया जाता ह

िीच नदए गए कट का परय ग कर सही उततर चनिए

(a) किि 1

(b) किि 2

(c) 1 और 2 द ि ो

(d) ि त 1 ि ही 2

Q74) निमननिखित कथि ो म स कौि-सास सही हह

1 कशब चोदर सि ि ततवब नरिी सभा की

अधयकषता की थी ज आधयाखतमक सतय की

ि ज म सोिि थी

2 बरहम समाज ि मािि गररमा पर बि नदया

मनतणपजा का निर र नकया और सती परथा जसी

सामानजक बराइय ो की आि चिा की

िीच नदए गए कट का परय ग कर सही उततर चनिए

(a) किि 1

(b) किि 2

(c) 1 और 2 द ि ो

(d) ि त 1 ि ही 2

Q75) निमननिखित कथि ो म स कौि-सास सही हह

1 भारत म नचशती नसिनसिा खवाजा म इिददीि

नचशती क दवारा सथानपत नकया गया था

2 नचशती परोपरा की एक परमि निशिता

आतमसोयम थी नजसम साोसाररक म ह स दरी

बिाए रििा शानमि था

िीच नदए गए कट का परय ग कर सही उततर चनिए

(a) किि 1

(b) किि 2

(c) 1 और 2 द ि ो

(d) ि त 1 ि ही 2

RAUSIAS-FC19E1003 27

Q70) Which of the following statements

isare correct

1 During the Ahmedabad Mill Strike

Mahatma Gandhi undertook a fast

unto death to strengthen the

workersrsquo resolve

2 The fast had effect of putting

pressure on mill owners who

finally agreed to give the workers a

15 per cent increase in wages

Select the correct answer using the code

given below

(a) 1 only

(b) 2 only

(c) Both 1 and 2

(d) Neither 1 nor 2

Q71) Which of the following are included in

the UNESCOrsquos list of the representative

list of the intangible cultural heritage of

humanity from India

1 Mudiyettu

2 Sankirtana

3 Kumbh Mela

Select the correct answer using the code

given below

(a) 1 and 2 only

(b) 2 and 3 only

(c) 3 only

(d) 1 2 and 3

Q72) Which of the following tribes isare

related to Nagaland

1 Angami

2 Kuki

3 Jarawa

Select the correct answer using the code

given below

(a) 1 only

(b) 1 and 2 only

(c) 2 only

(d) 1 2 and 3

Q73) Which of the following statements

isare correct

1 Rashtrakuta kingdom was founded by Dantidurga who established his capital at Manyakhet

2 Amoghavarsha a Rashtrakuta king was an author and is credited with writing the first

Kannada book on poetics

Select the correct answer using the code given below

(a) 1 only

(b) 2 only

(c) Both 1 and 2

(d) Neither 1 nor 2

Q74) Which of the following statements isare correct

1 Keshab Chandra Sen headed the Tattvabodhini Sabha which was engaged in search of spiritual truth

2 The Brahmo Samaj laid emphasis on human dignity opposed idolatry and criticized such social

evils as the practice of Sati

Select the correct answer using the code given below

(a) 1 only

(b) 2 only

(c) Both 1 and 2

(d) Neither 1 nor 2

Q75) Which of the following statements isare correct

1 The Chishti order was established in India by Khwaja Moinuddin

Chishti

2 A major feature of the Chishti tradition was austerity including maintaining a distance from the

worldly power

Select the correct answer using the code

given below

(a) 1 only

(b) 2 only

(c) Both 1 and 2

(d) Neither 1 nor 2

T e s t i s p a r t o f R a u rsquo s I A S T e s t s e r i e s f o r P r e l i m i n a r y E x a m 2 0 1 9

FOUNDATION + CURRENT AFFAIRS

GENERAL STUDIES (PAPER ndashI)

FOUNDATION TEST ndashIII

SUBJECT NCERT History Class VI-X + Current Affairs

Time Allowed 1frac12 Hours Maximum Marks 150

I NSTRUCT IONS

1 IMMEDIATELY AFTER THE COMMENCEMENT OF THE EXAMINATION YOU SHOULD CHECK

THAT THIS TEST BOOKLET DOES NOT HAVE ANY UNPRINTED OR TORN or MISSING PAGES OR

ITEMS ETC IF SO GET IT REPLACED BY A COMPLETE TEST BOOKLET

2 This Test Booklet contains 75 items (questions) Each item is printed both in Hindi and English

Each item comprises four responses (answers) You will select the response which you want to mark

on the Answer Sheet In case you feel that there is more than one correct response mark the

response which you consider the best In any case choose ONLY ONE response for each item

3 You have to mark all your responses ONLY on the separate Answer Sheet (OMR sheet) provided

Read the directions in the Answer Sheet

4 All items carry equal marks

5 Before you proceed to mark in the Answer Sheet the response to various items in the Test booklet

you have to fill in some particulars in the Answer Sheet as per instructions contained therein

6 After you have completed filling in all your responses on the Answer Sheet and the examination has

concluded you should hand over to the Invigilator only the Answer Sheet You are permitted to

take away with you the Test Booklet

7 Penalty for wrong answers

THERE WILL BE PENALTY FOR WRONG ANSWERS MARKED BY A CANDIDATE IN THE

OBJECTIVE TYPE QUESTION PAPERS

(i) There are four alternatives for the answer to every question For each question for which a

wrong answer has been given by the candidate one-third of the marks assigned to that

question will be deducted as penalty

(ii) If a candidate gives more than one answer it will be treated as a wrong answer even if one of

the given answers happens to be correct and there will be same penalty as above to that

question

(iii) If a question is left blank ie no answer is given by the candidate there will be no penalty for

that question

T h i s t e s t i s p a r t o f R a u rsquo s I A S T e s t s e r i e s f o r P r e l i m i n a r y E x a m 2 0 1 9

Test Code

FC19E1003

FC19H1003 29

Answers and Explanations of

NCERT History Class VI-X + Current Affairs (FC19E1003)

Q1) उततर (c)

सपषटीकरण

- ऋगवद म दविय ो और दिताओो क समवपित एक

हजार स अविक सत तर (शल क) ह

- य शल क ऋविय ो क दवारा रच गए थ और परि ो

दवारा सीख जात थ

- हालाोवक कछ शल क मवहलाओो (जस वक अपाला

घ सा ल पामदरा मतरयी और गागी) क दवारा भी रच

गए थ

- ऋगवद म सोिाद क रप म कई शल क मौजद ह

- हम विशवावमतर नामक एक ऋवि और दविय ो क

रप म पजी जान िाली द नवदय ो (वयास और

सतलज) क बीच िाताि का उदाहरण वमलता ह

- इसस पता चलता ह वक विशवावमतर िवदक काल स

सोबोवित थ

Q2) उततर (b)

सपषटीकरण

- करनल गफाओो स राख क अिशि परापत हए ह

ज इस ओर सोकत करत ह वक ततकालीन ल ग

अवि क उपय ग स पररवचत थ

- य गफाएो आोधर परदश म सथथत ह

Q3) उततर (c)

सपषटीकरण

bull बरािह म ितिमान कशमीर म सथथत एक

परागवतहावसक थथल ह जहाो ल ग गडढ क घर ो का

वनमािण करत थ

bull य घर जमीन क ख द कर बनाए जात थ तथा नीच

जान क वलए सीवियाा ह ती थी

bull ऐसा अनमान लगाया जाता ह वक य घर ठो ड क

मौसम म आशरय परदान करत थ

Q4) उततर (c)

सपषटीकरण

bull परालख-विदया (Epigraphy) क वशलालख ो क

अधययन क रप म पररभावित वकया जाता ह

bull हसतवलसखत दसतािज ो क माधयम स इवतहास

और सावहतय क अधययन क पाोडवलवप विजञान

(Manuscriptology) कहत ह

bull पराचीन लखन परणावलय ो क अधययन और

ऐवतहावसक पाोडवलवपय ो क समझन तथा वतवथ

वनिािरण क पलीओगराफी (Palaeography) कहा

जाता ह

bull नयवमजमविकस (Numismatics) वसक ो क

अधययन क सोदवभित करता ह

Q5) उततर (a)

सपषटीकरण

- चरक सोवहता चरक क दवारा वलखी गई आयिद

और िदयक-शासर पर एक महतवपणि पसतक ह

- ि भारतीय िदयक-शासर की पारमपररक परणाली

वजस आयिद क नाम स जाना जाता ह क

अभयासकताि थ

- ऐसा माना जाता ह वक चरक का विकास दसरी

शताबदी (ईसा पिि) और दसरी शताबदी (ईसवी) क

मधय हआ था

Q6) उततर (b)

सपषटीकरण

- भाग फसल ो पर वलए जान िाल कर क सोदवभित

करता ह ज कल फसल उतपादन का 16 िाो भाग

था

- ldquoकममकारrdquo शबद भवमहीन कवि शरवमक िगि क

वलए परय ग वकया जाता था

- ldquoअशवमिrdquo (वजस घ ड क बवलदान क रप म भी

जाना जाता ह) एक अनषठान ह ता था वजसम एक

घ ड क सवतोतर रप स घमन क वलए छ ड वदया

FC19H1003 30

जाता ह और राजा क सवनक उसकी रखिाली

करत थ

Q7) उततर (d)

सपषटीकरण

- ऋगववदक काल म घ ड ो क रथ ो म ज ता जाता था

ज (रथ) भवम मिवशय ो आवद पर कबजा करन क

वलए लड गए यद ो म उपय ग वकए जात थ

- इसस यह पता चलता ह वक घ ड ो यकत रथ ो का

उपय ग महाजनपद काल स काफी पहल आरमभ

हआ था

- ऋगववदक काल म मिवशय ो भवम जल आवद पर

कबजा करन क वलए तथा ल ग ो क पकडन क

वलए यद वकय जात थ

- अविकाोश परि इन यद ो म भाग वलया करत थ

- हालाोवक उस समय क ई वनयवमत सना नही ो ह ती

थी लवकन उस काल म सभाऐो ह ती थी ो वजनम

ल ग यद क मामल ो पर चचाि करत थ

- वनयवमत सनाएा महाजनपद काल का िवशषटय थी

वजनम पदल सवनक ो की विशाल सनाएा रथ तथा

हाथी शावमल ह त थ

Q8) उततर (a)

सपषटीकरण

- बद शाकय कल स सोबोवित थ और कशीनारा म

उनका वनिन हआ था

- बद न अपनी वशकषाएा पराकत भािा म दी थी ो ज

आम ल ग ो की भािा थी

Q9) उततर (c)

सपषटीकरण

- पराचीन भारत म दशिनशासर की छह शाखाएा थी ो

िशविक नयाय समखया य ग पिि वममाोसा और

िदाोत या उततर वममाोसा

- इनकी थथापना करमश कनाद गौतम कवपल

पतोजवल जावमनी और वयास ऋविय ो न की थी

Q10) उततर (b)

सपषटीकरण

महािीर की वशकषाऐो छठी शताबदी म िललभी म

सोकवलत की गई थी ो

Q11) उततर (c)

सपषटीकरण

- पारमपररक रप स चाणकय क कौविलय अथिा

विषणगपत क नाम स जाना जाता ह

- उसन अथिशासतर ज एक पराचीन भारतीय

राजनवतक आलख ह वलखा था

Q12) उततर (d)

सपषटीकरण

- भारत का राषटर ीय वचनह सारनाथ (उततर परदश) क

अश क सतमभ क ऊपर (शीिि पर) वसोह कवपिल

का एक अनरपण ह

- इस राषटर ीय वसदाोत सतयमि जयत क साथ

सोय वजत वकया गया ह

- रामपिि बल का नाम रामपिि (वबहार) क नाम पर

पडा जहाा इसकी ख ज हई थी

- यह अपन नाजक नकाशी मॉडल क वलए परवसदद

ह वजसम क मल तवचा सोिदनशील नथन ो सतकि

कान और मरबत िााग ो क शरषठतर परवतरप क

परदवशित वकया गया ह

- यह भारतीय और फारसी ततव ो का एक ससममशरण

- सोवकससा उततर परदश म सथथत ह

Q13) उततर (a)

सपषटीकरण

का िर वसोह ज एक महान य दा थ वबहार स

सोबोवित थ

Q14) उततर (b)

सपषटीकरण

िललालर शबद बड भ-सवावमय ो क वलए परय ग

वकया जाता था

FC19H1003 31

Q15) उततर (c)

सपषटीकरण

- अररकमड एक तिीय बसती थी जहाो दर दश ो स

आन िाल जहाज ो का माल उतारा जाता था

- यहाो पर ईोि ो का एक विशाल ग दाम वमटटी क

बतिन (वजनम एमफ रा - द हरी मवठय ो का लोबा

घडा - शावमल ह) और एरिाइन (Arretine)

मदभाोड पाए गए थ

- इस थथान पर र मन दीपक काोच क बन पातर और

रतन भी पाए गए थ

Q16) उततर (a)

सपषटीकरण

- मिनदर सोगम कविताओो म उसललसखत एक

तवमल शबद ह वजसका अथि ह ldquoतीन परमखrdquo

- यह तीन सततारि पररिार ो क मसखयाओो क वलए

परय ग वकया जाता ह च ल चर और पाणडय

Q17) उततर (c)

सपषटीकरण

- ऋग िद म सभा विदाथा तथा गण जसी

जनजावतय ो पर अथिा किोब पर आिाररत

सभाओो का उललख ह

- आरसमभक िवदक काल म सभाओो और सवमवतय ो

का विशि महतव ह ता था

- यहाा तक की मसखया अथिा राजा भी उनका

समथिन परापत करन क वलए आतर रहत थ

Q18) उततर (a)

सपषटीकरण

- जन िमि न ईशवर क अससततव क मानयता त दी ह

वकनत उसन ईशवर क वजना क पद स नीच रखा

- जन िमि न बौद िमि की तरह िणि परणाली की

भरतिना नही ो की थी

Q19) उततर (d)

सपषटीकरण

- च ल ो और पाणडय ो न शसकतशाली तिीय शहर ो का

विकास वकया था

- च ल ो का सबस महतवपणि शहर पहार (या

कािरीपटटीनम) था |

- मदरई पाणडय ो की राजिानी थी

Q20) उततर (b)

सपषटीकरण

- ldquoबदचररतrdquo बद का जीिन-ितताोत ह

- इस अशवघ ि क दवारा वलखा गया था

Q21) उततर (a)

सपषटीकरणः

- तवमल कवि अपपर भगिान वशि क भकत थ

- इस परकार ि एक नयनार सोत थ

Q22) उततर (d)

सपषटीकरणः

- समदरगपत एक परवसद गपत शासक था

- उसन वसक ो पर िीणा बजात हए अपनी छवि

अोवकत करिाई थी

- यह सोगीत क परवत उसक परम क दशािता ह

- हम उसकी इलाहाबाद परशससत स महतवपणि

ऐवतहावसक जानकारी वमलती ह वजसकी रचना

उसक दरबार क कवि हररसन न की थी

Q23) उततर (b)

सपषटीकरणः

- विकरम सोित की शरआत ििि 58 ईसा पिि म

चनदरगपत वदवतीय न की थी

- यह शक ो पर उसकी जीत और उस विकरमावदतय

की पदिी वमलन क उपलकषय म आरमभ वकया गया

था

FC19H1003 32

- बानभटट न हिििििन का जीिन-ितताोत हििचररत

(ज सोसकत म थी) वलखी थी

Q24) उततर (c)

सपषटीकरणः

- सोवि-विगरावहका यद एिो शाोवत का मोतरी

- साथििाह वयापाररय ो क कावफल ो का नता

Q25) उततर (a)

सपषटीकरणः

- जआन झाोग (हसआन रताोग ndash Hsuang Tsang)

एक चीनी यातरी था ज हिििििन क शासनकाल म

भारत आया था

- ििि 630 ईसवी स ज दशक आरमभ हआ था उसम

जआन झाोग मधय एवशया ईरान और

अफग़ावनसतान की यातरा करन क पशचात कशमीर

क रासत स भारत आया था

- उसन उततर स पिि तक की यातरा की और िह

लगभग 2 ििि वबहार म रहा

- जआन झाोग न नालनदा विशवविदयालय म विदयावथिय ो

और विदवान ो क साथ पारसपररक विचार-विमशि

वकया थथानीय भािाओ ा म वनपणता परापत की तथा

बौद सतप ो की ख ज की

Q26) उततर (c)

सपषटीकरणः

- परदवकषणा पथ बौद िासतकला म सतप क चार ो

ओर बनाया जान िाला एक घमािदार पथ ह ता

- परशन म वदए गए बाकी क तीन ो ततव वहोद मसनदर ो की

िासतकला क भाग ह

Q27) उततर (d)

सपषटीकरणः

परशन म वदए गए सभी मोवदर ो म वयापक रप स

ईोि ो (पकी ईोि ो) का परय ग पतथर ो क साथ हआ

Q28) उततर (c)

सपषटीकरण

- महममद कली कतब शाह ग लकणडा का सलतान

था

- िह अकबर का समकालीन था

- सावहतय और िासतकला म उसकी अतयाविक

रवच थी

- िह एक महान कवि था

- िह दसखनी उदि फारसी और तलग म वलखता था

- उसन अपन पीछ एक विसतत वदिान (सोगरह)

छ डा ह

- अभी हाल ही म तलोगाना म ग लकणडा क वकल

क अनदर खदाई वकय गए बाग-ए-नाया वकला

बाग क चार ो ओर रप-रखा क मानवचतरण क

वलए भारतीय परातासतवक सिकषण (The

Archaeological Survey of India ndash ASI)

गराउणड पनीिर विोग रडार (Ground Penetrating

Radar) का परय ग करगा

Q29) उततर (a)

सपषटीकरणः

- वसलपपावदकारम एक तवमल महाकावय ह वजसकी

रचना इलाोग क दवारा लगभग 1800 ििि पिि की

गई थी

- यह क िलन नामक एक वयापारी की कहानी ह

ज माििी नामक एक गवणका (िशया) स परम

करन लगा था

- मवनमकलाई क िलन और माििी की पतरी की

कहानी ह

Q30) उततर (a)

सपषटीकरण

- चरक आयिद और वचवकरता की एक महतवपणि

रचना चरक सोवहता क लखक ह

- बरहमगपत क अपनी रचना बरहम-सफि-वसदानत

(ज एक खग लीय रचना ह) क कारण परवससद

वमली

FC19H1003 33

- बगदाद म इसका अनिाद अरबी भािा म वकया

गया था

- इसका इसलावमक गवणत और खग ल-विजञान पर

महतवपणि परभाि पडा था

- बाद म अपन जीिनकाल म बरहमगपत न

ldquoखोडखयाकrdquo वलखी ज एक खग लीय पससतका

(एक छ िी पसतक) थी

- इसम आयिभटट की अिि-रावतर क परतयक वदन की

शरआत परणाली का परय ग वकया गया था

Q31) उततर (c)

सपषटीकरण

- अमीर खसर एक परवसद सफी सोगीतकार कवि

और विदवान थ

- 1318 म उनह ोन पाया वक इस भवम (वहोदसतान) क

हर कषतर म अलग-अलग भािा थी लाहौरी

कशमीरी दवारसमदरी (दवकषणी कनाििक म)

तलोगाना (आोधर परदश म) गजरी (गजरात म)

माबारी (तवमलनाड म ) अििी (पिी उततर परदश

म) और वहोदिी (वदलली क आस-पास क कषतर म)

आवद

- उनह न यह बताया वक सोसकत वकसी भी कषतर स

सोबोवित नही ो थी और किल बराहमण ही इस भािा

का जञान रखत थ

Q32) उततर (c)

सपषटीकरण

- वहरणय-गभि सववणिम गभि क सोदवभित करता ह

- जब बराहमण ो की सहायता स यह अनषठान वकया

जाता था त यह माना जाता था वक बवल दन िाल

का कषवतरय क रप म पनजिनम ह गा

Q33) उततर (d)

सपषटीकरण

- कदमई भवम राजसव पर कर क सोदवभित करता

- गवावलयर परशससत म नागभि क दवारा वकय गए

श िण का िणिन वकया गया ह |

- नागभि एक परवतहार राजा था

Q34) उततर (b)

सपषटीकरण

- राजतरो वगनी 12िी ो शताबदी म कलहन क दवारा

रवचत एक सोसकत पसतक (िकसट) ह

- यह परारसमभक भारत की ऐवतहावसक इवतितत थी

- तकि सोगत रप स इस अपन परकार की सिोततम

और सिािविक विशवसनीय कवत माना जाता ह

- यह कशमीर कषतर क पराचीनतम समय स लकर

उसकी रचना की तारीख तक क समपणि इवतहास

का आचछादन करती ह

Q35) उततर (c)

सपषटीकरण

- गााि की आम सभा क ldquoउरrdquo कहा जाता था

- ldquoउरrdquo म गााि क सभी कर दन िाल वनिासी

शावमल ह त थ

Q36) उततर (a)

सपषटीकरण

- वदलली सलतनत म ldquoतारीखrdquo इवतहास लखन का

एक रप था

- ldquoतािरीखrdquo क लखक विदवान परि ह त थ वजनम

सवचि परशासक इतयावद शावमल थ

Q37) उततर (a)

सपषटीकरण

- अलाउददीन सखलजी अपन सवनक ो क ितन का

भगतान नकद म करता था न वक इकता क रप

- सवनक अपना सामान वदलली म वयापाररय ो स

खरीदत थ अतः इस बात का भय था वक वयापारी

कही ो िसतओो का मलय न बिा द

- इसकी र कथाम क वलए अलाउददीन सखलजी न

वदलली म कीमत ो क वनयसित वकया

FC19H1003 34

- अविकारीगण धयानपििक मलय ो का सिकषण करत

थ तथा ज वयापारी वनिािररत मलय पर माल नही ो

बचत थ उनक दसणडत वकया जाता था

Q38) उततर (d)

सपषटीकरण

- वदलली सििपरथम त मर राजपत ो क अिीन उनक

सामराजय की राजिानी बनी थी

- 12िी ो शताबदी क मधय म अजमर क चौहान ो

(वजनह चाहमान ो क नाम स भी जाना जाता ह) न

त मर राजपत ो क परावजत वकया था

- त मर ो और चौहान ो क अिीन वदलली एक

महतवपणि िावणसजयक क दर बन गया था

- कई जन वयापारी यहाा रहन लग थ और उनह ोन

कई मोवदर भी बनिाए

- यहाा पर मवदरत वसक वजनह ldquoदहलीिालrdquo क नाम

स जाना जाता था वयापक रप स परचलन म थ

Q39) उततर (c)

सपषटीकरण

- म ठ की मसिद का वनमािण वसको दर ल दी क

राजयकाल म उसक मिी क दवारा करिाया गया

था

- बगमपरी मसिद का वनमािण महममद तगलक क

शासनकाल म हआ था

- यह मसिद विशव का पणयथथान (The

Sanctuary of the World) और वदलली म महममद

तगलक की नई राजिानी जहाोपनाह की मखय

मसिद थी

- कववत- अल - इसलाम मसिद का विसतार

इलतसिश और अलाउददीन सखलजी न वकया था

- मीनार का वनमािण तीन सलतान ो कतबददीन ऐबक

इलतसिश और वफर ज शाह तगलक क दवारा

करिाया गया था

Q40) उततर (c)

सपषटीकरण

- मगल ो क अिीन मनसबदार शबद उस वयसकत क

वलए सोदवभित वकया जाता था वजसक पास मनसब

(अथाित पद) ह ता था

- उस अपना ितन राजसव कायो वजनह जागीर कहत

थ क रप म परापत ह ता था

Q41) उततर (b)

सपषटीकरण

- ldquoभारत छ ड आोद लनrdquo वबरविश शासन क

सखलाफ ल ग ो का एक सवाभाविक विदर ह था

- असखल भारतीय काोगरस सवमवत न 8 अगसत 1942

क बमबई म एक बठक का आय जन वकया था

- इस बठक म परवसद सोकलप ldquoभारत छ ड rdquo क

पाररत वकया गया और इस उददशय क परापत करन

क वलए गाोिी क नततव म एक अवहोसक जन सोघिि

आोद लन की शरआत का परसताि वदया गया

- लवकन अगल ही वदन गाोिी और काोगरस क अनय

परमख नताओो क वगरफतार कर वलया गया

- काोगरस क एक बार वफर अिि घ वित वकया गया

था

Q42) उततर (c)

सपषटीकरण

- साइमन कमीशन यनाइविड वको गडम क सात

साोसद ो का एक समह था

- इस वबरविश भारत क वलए सोििावनक सिार ो का

सझाि दन क वलए गवठत वकया गया था

- इस आय ग म िररषठ वबरविश राजनता सर जॉन

साइमन क नततव म किल वबरविश सदसय ही

शावमल थ

- इसवलए भारत क ल ग ो न साइमन कमीशन क

आगमन क विरद आोद लन वकया था

Q43) उततर (a)

सपषटीकरण

bull दादा भाई नौर जी भारत म वबरविश शासन क

आवथिक पररणाम ो क बार म अपनी विर िी

(परवतकल) राय क वलए जान जात थ

FC19H1003 35

bull अपन कई लख ो और भािण ो म विशि रप स

ldquoपाििी एो ड अन-वबरविश रल इन इसणडया

(Poverty and Un-British Rule in India) म

नौर जी न यह तकि वदया वक भारत पर अतयविक

कर लगाया गया था और इसकी सोपवतत इोगलड की

ओर परिावहत की जा रही थी

bull उनह ोन पराचीन भारतीय गरोथ ो की वयाखया करन

का और भारतीय ो क आिविशवास क बहाल

करन पर कायि नही ो वकया था

उनह ोन वकसी और बात स पहल सभी सामावजक

बराइय ो क उनमलन की आिशयकता पर भी बल

नही ो वदया था

Q44) उततर (c)

सपषटीकरण

bull अगसत 1932 म वबरविश परिानमोतरी मकड नालड न

अपन साोपरदावयक परसकार (The Communal

Award) की घ िणा की थी

bull यह भारत क कई साोपरदावयक वहत ो क बीच विवभनन

सोघिो क हल करन क वलए वबरिन का एकतरफा

परयास था

bull यह परसकार (Award) बाद म 1935 क

अविवनयम (The Act of 1935) म शावमल वकया

गया था

bull इस साोपरदावयक परसकार न मससलम ो क वलए

आरवकषत एक अलग वनिािचक मणडल फॉमिल का

विसतार अनय अलपसोखयक ो क वलए वकया था

वजसम वसख ो भारतीय ईसाइय ो आोगल-भारतीय

समदाय यर पीय समदाय तथा विवशषट कषतरीय

समह ो क शावमल वकया गया था

bull गाोिी न इस परसताि क भारतीय समाज क

विभावजत करन क वलए एक घवणत वबरविश

सावजश क रप म दखा और उसक सखलाफ

आमरण अनशन वकया

Q45) उततर (b)

सपषटीकरण

मौजदा आयात और वनयाित क अवतररक़त

औपवनिवशक भारत क वनमनवलसखत खचो क

वलए एक विशिवनवशचत िन रावश भी दनी पडती

थी

(i) परशासन क वयय

(ii) सना क रख-रखाि क वयय

(iii) यद क वयय

(iv) सिावनितत अविकाररय ो की पशन तथा

(v) वबरिन दवारा अपनी उपवनिश बसती

(कॉल नी) क रख-रखाि क वयय

इनह गह शलक (Home Charges) क रप म

जाना जाता था और लगभग परी तरह स भारत क

दवारा इनका भगतान वकया जाता था

bull गह शलक म वनमनवलसखत घिक शावमल थ

(i) भारतीय ऋण पर दय बयाज

(ii) ईसट इोवडया को पनी क शयरिारक ो क

लाभाोश

(iii) लोदन म भारत कायािलय चलान क वलए िन

(iv) भारत म वनयकत वबरविश कवमिय ो क ितन

और पशन का भगतान करन क वलए िन

(v) रलि पर बयाज

(vi) नागररक और सनय शलक

(vii) इोगलड म सट र (सामगरी) की खरीद

Q46) उततर (b)

सपषटीकरण

bull भारतीय राषटर ीय काोगरस का लाहौर सतर 1929 म

जिाहरलाल नहर की अधयकषता म आय वजत

वकया गया था

bull इस सतर म भारतीय राषटर ीय आोद लन स समबसित

कई महतवपणि पररणाम सामन आय थ

(i) सििपरथम इस सतर म काोगरस क अधयकष पद

पर जिाहरलाल नहर क चना गया था ज

काोगरस म िामपोवथय ो की बिती हई ताकत

का सपषट सोकत था

(ii) दसरा इस सतर म पहली बार काोगरस न पणि

सवतोतरता की माोग क उठाया था

इस परकार की माोग काोगरस मोच स पहल कभी भी

नही ो उठाई गई थी

Q47) उततर (b)

सपषटीकरण

FC19H1003 36

bull इस ररप िि न वकसी भी समदाय क वलए पथक

वनिािचक मोडल अथिा अलपसोखयक ो क वलए

भाराोश की वसफाररश नही ो की थी

bull तथावप इस ररप िि न उन पराोत ो म अलपसोखयक

सीि ो क आरकषण की अनमवत दी थी जहाा पर कम

स कम दस परवतशत अलपसोखयक ह

bull लवकन यह समदाय क आकार क अनपात म ह ना

चावहए था

bull इस ररप िि म भारत क वलए पणि सवतोतरता क

वलए क ई पराििान नही ो था

Q48) उततर (c)

सपषटीकरण

bull आरो वभक िवदक आयो का िमि मखय रप स

परकवत की पजा और यजञ था

bull परारो वभक आयि िमि परकवत की पजा क समान था

bull िासति म उनक चार ो ओर की शसकतयाा वजनह न

त ि वनयोवतरत कर सकत थ और न ही समझ पाए

थ उनह वदवयता क साथ वनिवशत वकया गया तथा

उनह मादा या नर दिीदिताओो क रप म

परतीकतव वकया गया था

bull उनह ोन कछ यजञ ो का भी वनषपादन वकया था

Q49) उततर (b)

सपषटीकरण

bull सडक और नदी-मागि (जल-मागि) डकती स

सरवकषत नही ो थ

bull उललखनीय ह वक हिििििन क शासनकाल क

दौरान यआन चिाोग (हयएन साोग) का सारा

सामान लि वलया गया था

Q50) उततर (c)

सपषटीकरण

परशन म वदए गए द न ो कथन सही ह

Q51) उततर (b)

सपषटीकरण

bull परोदर दास एक सोत और भगिान कषण क एक

महान भकत थ

bull परोदर दास क कनाििक सोगीत क वपतामह क

रप म जाना जाता ह

bull यदयवप उनक जनम-थथान क बार म काफी

अिकल लगाई जाती रही ह

bull तथावप अब कननड विशवविदयालय हमपी क दवारा

गवठत एक विशिजञ सवमवत इस वनषकिि पर पहोची

ह वक उनका जनम थथान सोभितया कनाििक का

एक छ िा-सा गााि कषमपरा (वशिम गगा वजला)

था

Q52) उततर (c)

सपषटीकरण

bull शरी तयागराज शरी शयाम शासतरी और शरी मथसवामी

दीवकषतर क कनाििक सोगीत की वतरमवति माना

जाता ह

bull उनक कारण ही 18िी ो-19िी ो शताबदी म कनाििक

सोगीत का सववणिम यग आया था

Q53) उततर (d)

सपषटीकरण

bull अभी हाल ही म लौह यगीन-महापािावणक काल

का 2000 ििि पराना एक दलिभ सारक फगस

(Sarcophagus) (पतथर का ताबत) क ललम क

वियर गाोि (क वयलडी क पास वजला क वझक ड

करल राजय) की एक रॉक-कि गफा स ख जा गया

bull यह ताबत वजसम हविय ो क िकड थ खदाई क

दौरान वमला

bull अभी तक इस परकार की दलिभ ख ज करल क

मातर द ही थथान ो स हई ह

bull य द न ो सारक फगी (Sarcophagi) (पतथर क

ताबत) चियर और अथ ली (वजला क वझक ड) क

महापािाण थथल ो स वमल ह

Q54) उततर (a)

सपषटीकरण

FC19H1003 37

दवकषण भारत म महापािाण सोसकवत एक पणि

विकवसत लौह यगीन सोसकवत थी

Q55) उततर (d)

सपषटीकरण

bull च ल पाणडय और करलपतर (चर) इन तीन ो का

उललख अश क क अवभलख ो म वकया गया ह

bull सोभितः य भौवतक सोसकवत क उततर

महापािावणक चरण म थ

Q56) उततर (d)

सपषटीकरण

bull भीमा-क रगाोि की लडाई ततीय आोगल-मराठा

यद का वहससा थी

Q57) उततर (b)

सपषटीकरण

bull राजकमार शकल न गाोिीजी क चोपारण आन तथा

वतनकवथया परणाली स जडी समसया की जाोच क

वलए रारी करन क वलए दश भर म उनका

अनसरण वकया था

bull बज वकश र राजदर परसाद महादि दसाई और

नरहरी पाररख चोपारण सतयागरह क दौरान गाोिी

जी क सहय गी थ

Q58) उततर (b)

सपषटीकरण

bull बराहमण ो और बौद मठिाररय ो क कर-मकत गााि

अनदान म दन की परथा सतिाहन ो न आरमभ की

थी

Q59) उततर (c)

सपषटीकरण

इस कायिकरम क उददशय वनमनानसार ह

(i) बवनयादी पयििन आिाररक सोरचना का विकास

करना

(ii) चयवनत (पहचान वकय गए) कषतर ो म आजीविका क

सजन क वलए दश क साोसकवतक और विरासत

मलय ो क बिािा दना

(iii) विरासत समारक थथल ो पर विशव सतरीय आिाररक

सोरचना विकवसत करक एक सतत तरीक स

पयििक आकििण म िसद करना

(iv) थथानीय समदाय ो की सवकरय भागीदारी क माधयम

स र रगार ो का सजन करना

(v) र रगार उतपादन और आवथिक विकास क वलए

पयििन कषमता का उन पर परभाि का उपय ग

करना तथा

(vi) िारणीय पयििन आिाररक सोरचना का विकास

करना और उसका उवचत सोचालन तथा

रखरखाि सवनवशचत करना

Q60) उततर (b)

सपषटीकरण

bull यह वनकाय ििि 1987 म अससततव म आया था

bull यह एक राषटर ीय सतर का शीिि सोगठन ह ज भारत

सरकार क जनजातीय मामल ो क मोतरालय क

परशासवनक वनयोतरण क अिीन काम कर रहा ह

bull इसका पोजीकत और परिान कायािलय नई वदलली

म सथथत ह

Q61) उततर (c)

सपषटीकरण

bull परमचोद क उपनयास ो म परमाशरम रोगभवम गबन

कमिभवम और ग दान शावमल ह

bull ग रा रिी ोदरनाथ िग र क दवारा रवचत उपनयास ह

bull अभी हाल ही म मोशी परमचोद की 138िी ो जयोती दश

भर म मनाई गई थी

Q62) उततर (b)

सपषटीकरण

bull ldquoवगदाrdquo पोजाब (भारत) एिो पावकसतान की

मवहलाओो क दवारा तयौहार क समय और फसल

की बिाई तथा किाई क अिसर पर वकया जान

िाला एक पारोपररक दहाती नतय ह

FC19H1003 38

bull इस नतय क माधयम स पोजाबी मवहलाऐो अपनी

परसननता परकि करती ह तथा वगदा क परदशिन क

माधयम स परि िचिसव िाल समाज म मवहलाओो

की दबी हई भािनाओो क परकि करती ह

bull चोवक इस नतय का परि ो क साथ क ई सोबोि नही ो

ह अतः किल मवहलाऐो ही इसम भाग ल सकती

bull हर साल तीज समार ह क दौरान पोजाब म वगदा

नतय वकया जाता ह

तीज भारत क कछ भाग ो म मवहलाओो क दवारा

मनाया जान िाल कई तयौहार ो क वलए एक

वयापक नाम ह

Q63) उततर (a)

सपषटीकरण

- मजम-उल-बहरीन या द समदर ो का सोगम

नामक उललखनीय रचना दारा वशक ह क दवारा

वलखी थी

- भारत क उपराषटर पवत शरी एम िकया नायड न कहा

ह वक राजकमार दारा वशक ह की रचनाएा शाोवत

और सदभाि क बिािा दन क वलए एक तारा सर त

क रप म सामन आ सकती ो ह

- उपराषटर पवत गत ििो क भला वदए गए राजकमार

दारा वशक ह क परदवशित परचवलत करन हत

आय वजत एक परदशिनी का दौरा करन क बाद एक

सभा क सोब वित कर रह थ

- इस परदशिनी का आय जन फर क इस गौवियर

(Francois Gautier) क दवारा lsquoइोवदरा गाोिी नशनल

सिर फॉर द आििसrsquo (The Indira Gandhi

National Centre for the Arts) नई वदलली म

वकया गया था

Q64) उततर (c)

सपषटीकरण

- ग मतशवर परवतमा जन भगिान बाहबली क

समवपित ह

- यह एक एक-चटटानी पतथर की मवति ह

- राषटर पवत राम नाथ क विोद न शरिणबलग ला

(कनाििक) म आय वजत वकय जान िाल भवय

अवभिक समार ह महामसतकावभिक का

उदघािन वकया था

- यह समार ह 12 ििो म एक बार ह ता ह

Q65) उततर (c)

सपषटीकरण

bull पराची घािी पराची नदी क चार ो ओर फली हई थी

bull पराची घािी िीर-िीर विलपत ह गई थी

bull पराची नदी भिनशवर स वनकलती ह

bull यह महानदी की एक सहायक नदी ह और यह

परी खदाि किक तथा जगतवसोहपर वजल ो स

ह कर बहती ह

bull इस नदी क पर कषतर क पराची घािी कहा जाता ह

bull यह नदी बोगाल की खाडी म वगरती ह

परातासतवक साकषय स पता चलता ह वक पराची घािी

सभयता हडपपा और म हनज दाड द न ो की

पिििती ह

Q66) उततर (d)

सपषटीकरण

य समारक छतरपर वजल (मधय परदश) म विोधयाचल

पिित शरोखला म सथथत ह

Q67) उततर (a)

सपषटीकरण

bull थॉिस ऑन पावकसतान नामक पसतक डॉ बी

आर अमबडकर न वलखी थी

bull डॉ बी आर अमबडकर की जयोती क अिसर पर

भारत क राषटर पवत न भारत की इस महान हसती

क शरदाोजवल अवपित की थी

bull डॉ बी आर अमबडकर न 1924 म वडपरथड

कलावसर इोसटीटयि (दवलत िगि सोथथान -

बवहषकत वहतकाररणी सभा) और 1927 म समाज

समता सोघ की थथापना की थी

bull अमबडकर का धयान वशकषा कषतर की ओर भी था

bull उनह ोन वशकषा क वनमन िगो म फलान क वलए

पीपलस एजकशन स साइिी (The Peoples

Education Society) क नाम स महाविदयालय ो क

नििकि और छातरािास ो की थथापना की थी

FC19H1003 39

Q68) उततर (b)

सपषटीकरण

bull महरगि भारतीय उपमहादवीप म एक परवसद

निपािाण बसती ह ज बलवचसतान पराोत

पावकसतान म सथथत ह

bull दचपलली (आोधर परदश) क पास नागलर नदी क

पिी ति ो पर चना पतथर क बलॉक क विशाल

विसतार म एक पिि-ऐवतहावसक रॉक आिि थथल की

ख ज की गई ह

bull इसन 1500-2000 ईसा पिि क दौरान गोिर (आोधर

परदश) म विकवसत निपािाण सभयता पर परकाश

डाला ह

Q69) उततर (c)

सपषटीकरण

bull 12िी ो सदी और 13िी ो सदी म काकाविय िोश का

उदय हआ था

bull ि पहल कलयाण क पवशचमी चालकय ो क सामोत थ

bull परारोभ म उनह ोन िारोगल (तलोगाना) क पास एक

छ ि स कषतर पर शासन वकया था

bull उनह ोन ldquoनायक वयिथथाrdquo की शरआत की थी

वजस बाद म विजयनगर क राय शासक ो न

अपनाया और विकवसत वकया था

Q70) उततर (a)

सपषटीकरण

bull गाोिीजी क अनशन स वमल मावलक ो पर दबाि

पडा था ज अोततः शरवमक ो क ितन म 35 परवतशत

की िसद करन क वलए सहमत हए थ

bull गगल (Google) न अनसया साराभाई वजनह ोन

भारत क शरवमक आोद लन म एक अगरणी भवमका

वनभाई थी की 132िी ो जयोती डडल (Doodle) का

वनमािण करक मनाई

Q71) उततर (d)

सपषटीकरण

भारत स यनसक की मानिता की अमति साोसकवतक

विरासत की परवतवनवि सची म वनमनवलसखत शावमल ह

bull कवडयटटम करल का सोसकत रोगमोच

bull मवडयिि करल का अनषठान रोगमोच और नतय

नाविका

bull िवदक मि जाप की परोपरा

bull राजथथान क कालबवलया ल क गीत और नतय

bull रामलीला रामायण का पारोपररक परदशिन

bull सोकीतिन मवणपर का अनषठान गायन ढ ल िादन

और नतय

bull रममन भारत क गििाल वहमालय का िावमिक

तयौहार और अनषठान रोगमोच

bull जाोदीयाला गर पोजाब क ठठर ो की पीतल और

ताोब क वशलप स वनवमित बतिन ो की पारोपररक कला

bull छाऊ नतय पिी भारतीय राजय ो म जनमी शासतरीय

भारतीय नतय कला

bull लददाख का बौद मि जाप िर ाोस-वहमालयी लददाख

कषतर तथा जमम-कशमीर म पवितर बौद गरोथ ो का पाठ

bull य ग

bull नौर र

bull को भ मला

Q72) उततर (b)

सपषटीकरण

bull भारत क राषटर पवत शरी राम नाथ क विोद न

वकसामा नागालड म हॉनिवबल मह रति और

राजय गठन वदिस समार ह का उदघािन वकया

था

bull हॉनिवबल मह रति का नाम भारतीय हॉनिवबल क

नाम पर पडा ह ज एक विशाल और रोगीन जोगली

पकषी ह

bull यह पकषी नागालड राजय की अविकतर जनजावतय ो

की ल ककथाओो म उसललसखत ह

bull नागालड की परमख मानयता परापत जनजावतयाा ह

अोगामी आओ चखसोग चाोग ककी रगमा और

रवलोग आवद

bull ओोग जारिा और ससिनलीस अोडमान-वनक बार

दवीप समह की जनजावतयाा ह

FC19H1003 40

Q73) उततर (c)

सपषटीकरण

bull दकन म राषटर कि शासन दसिी ो सदी क अोत तक

लगभग 200 ििो तक रहा था

bull राषटर कि शासक अपन िावमिक विचार ो म सवहषण

bull उनह ोन न किल शि िमि और िषणि िमि बसलक

जन िमि क भी सोरकषण वदया था

bull एल रा म वशि क परवसद रॉक कि मोवदर का

वनमािण नौिी ो सदी म राषटर कि राजा कषण परथम न

करिाया था

bull उसका उततराविकारी अम घििि जन था लवकन

उसन अनय िमो क भी सोरकषण परदान वकया था

bull राषटर कि ो न मसलमान वयापाररय ो क बसन की

अनमवत दी थी

bull उनह न अपन अविराजय ो म इसलाम क उपदश दन

की भी अनमवत दी थी

bull अभी हाल ही म पाोडिलागटटा (तलोगाना) क

परागवतहावसक चटटान वचतर ो क कषरण की बिती हई

घिनाएा एक गोभीर वचोता का वििय ह

bull यह परागवतहावसक चटटान क नकसान पहाचा

सकता ह

bull पाोडिलागटटा वनमनवलसखत क वलए जाना जाता ह

- 10000 ईसा पिि स 8000 ईसा पिि क वचवतरत

चटटानी आशरय ो क वलए

- राषटर कि काल क एक 8 िी ो सदी क

वशलालख क वलए और

- 12िी ो सदी क काकविय सामराजय क वभवतत

वचतर ो क वलए

Q74) उततर (b)

सपषटीकरण

bull 1828 म राजा राम म हन रॉय न एक नय िावमिक

समाज बरहम सभा की थथापना की थी वजस बाद

म बरहम समाज क नाम स जाना गया था

bull दिदरनाथ िग र न ततवब विनी सभा की अधयकषता

की थी ज आधयासिक सतय की ख ज म सोलि

थी

bull इसका उददशय वहोद िमि क शद करन का और

एकशवरिाद (एक ईशवर म आथथा) का परचार करना

था

bull नय समाज की थथापना क आिार थ कारण

(तकि ) क द सतमभ तथा िद और उपवनिद

bull अभी हाल ही म सािारण बरहम समाज का कछ

काननी मदद ो क लकर पवशचम बोगाल सरकार क

साथ काननी वििाद चल रहा ह

Q75) उततर (c)

सपषटीकरण

bull भारत म वचशती वसलवसल की थथापना खवाजा

म इनददीन वचशती क दवारा की गयी थी

bull ि 1192 ईसवी क आसपास भारत आय थ

bull वचशतीय ो क बारहिी ो शताबदी क उततरािि म भारत

म आन िाल सफीय ो क समह ो म सबस

परभािशाली माना जाता ह

bull उनह ोन थथानीय िातािरण क साथ सफलतापििक

अनकलन वकया और उनह ोन भारतीय भसकत

परोपराओो क कई पहलओो क अपनाया

bull अजमर म सफी अपरकि खवाजा म इनददीन वचशती

की ऐवतहावसक दरगाह क एक नया रप दन की

तयारी की जा रही ह

bull इस 13िी ो शताबदी की दरगाह क ldquoसवचछ

आइकॉवनक थथल ोrdquo (Swacch Iconic Places) म

शावमल वकया गया ह ज परवतवषठत विरासत

आधयासिक और साोसकवतक थथान ो पर क वदरत

य जना ह

FC19H1003 41

ANSWERS amp EXPLANATION OF

NCERT History Class VI-X + Current Affairs

(FC19E1003)

Q1) Answer c

Explanation

Rigveda consists of more than a

thousand hymns dedicated to gods and

goddesses These hymns were

composed by sages and learnt by men

however a few were composed by

women like Apala Ghosa Lopamudra

Maitreyi and Gargi

Rigveda consists of many hymns in the

form of dialogues We get an example of

a dialogue between a sage named

Vishwamitra and two rivers (Beas and

Sutlej) that were worshipped as

goddesses This suggests that he

belonged to the Vedic period

Q2) Answer b

Explanation

Traces of ash have been found from

Kurnool Caves suggesting that people

were familiar with the use of fire

It is situated in Andhra Pradesh

Q3) Answer c

Explanation

Burzahom is a prehistoric site in

present day Kashmir where people built

pit houses which were dug into the

ground with steps leading into them

These may have provided shelter in cold

weather

Q4) Answer c

Explanation

Epigraphy is defined as the study of

inscriptions

Manuscriptology is the study of history

and literature through the use of hand

written documents

Palaeography refers to the study of

ancient writing systems and the

deciphering and dating of historical

manuscripts

Numismatics refers to the study of

coins

Q5) Answer a

Explanation

Charaka Samhita was written by

Charaka and is an important book on

Ayurveda and medicine

He was a practitioner of the traditional

system of Indian medicine known as

Ayurveda

Charaka is thought to have flourished

sometime between the 2nd century BCE

and the 2nd century CE

Q6) Answer b

Explanation

Bhaga refers to the tax on crops which

was fixed at 16th of the production

Kammakaras is the term used for the

landless agricultural labour class

Ashvamedha also known as horse

sacrifice is a ritual where a horse is let

loose to wander freely and it was

guarded by the rajarsquos men

Q7) Answer (d)

Explanation

In the Rigvedic period horses were

yoked to chariots that were used in

battles fought to capture land cattle

etc This suggests that the use of horse

chariots began much before the period

of Mahajanapadas

The battles were fought in the Rigvedic

period for cattlersquos lands water an even

to capture people Most men took part

in these wars however there was no

regular army but there were assemblies

where people met and discussed

matters of war Regular armies became

a feature in the Mjahajanapada period

including vast armies of foot soldiers

chariots and elephants

RAUSIAS-FC19E1003 42

Q8) Answer (a)

Explanation

Buddha belonged to the Sakya clan and

passed away at Kusinara

Buddha taught in Prakrit which was the

common language of people

Q9) Answer c

Explanation

There were six schools of philosophy in

ancient India These are known as

Vaishesika Nyaya Samkhya Yoga

Purva Mimansa and Vedanata or Uttara

Mimansa They were founded by sages

Kanada Gautama Kapila Patanjali

Jamini and Vyasa respectively

Q10) Answer b

Explanation

The teachings of Mahavira were

compiled at Valabhi in 6th century AD

Q11) Answer (c)

Explanation

Chanakya is traditionally identified as

Kautilya or Vishnugupta who authored

the ancient Indian political treatise the

Arthashastra

Q12) Answer d

The national emblem of India is an

adaptation of the Lion Capital atop the

Ashoka Pillar of Sarnath Uttar Pradesh

and is combined with the National

Motto Satyameva Jayate

The Rampurva Bull gets the name from

the site of its discovery Rampurva in

Bihar

It is noted for its delicately sculpted

model demonstrating superior

representation of soft flesh sensitive

nostrils alert ears and strong legs It is

a mixture of Indian and Persian

elements

Sankissa is situated in Uttar Pradesh

India

Q13) Ans(a)

Kunwar Singh was a notable leader during the Revolt of 1857 He belonged

to a royal house of Jagdispur Bihar

Q14) Answer b

Explanation

The term Vellalar was used for large

landowners

Q15) Answer c

Explanation

Arikamedu was a coastal settlement

where ships unloaded goods from

distant lands Finds here include a

massive brick warehouse pottery

including amphorae and Arretine ware

Roman lamps glassware and gems have

also been found at the site

Q16) Answer a

Explanation

Muvendar is a Tamil word mentioned in

Sangam poems meaning three chiefs

used for the heads of three ruling

families the Cholas Cheras and

Pandyas

Q17) Ans (c)

Several tribal or kin-based assemblies

such as the Sabha Vidatha and gana

are mentioned in the Rig-veda The

Sabha and the samiti mattered a great

deal in early Vedic times so much so

that the chiefs or the kings showed an

eagerness to win their support

Q18) Ans (a)

Jainism recognised the existence of the

gods but placed them lower than the

jina and did not condemn the varna

system as Buddhism did

Q19) Answer (d)

Explanation

Cholas and Pandyas had developed

powerful coastal cities The most

important city of Cholas was Puhar or

Kaveripattinam and Madurai was the

capital of Pandyas

Q20) Answer b

Explanation

Buddhacharita is the biography of

Buddha and was written by

RAUSIAS-FC19E1003 43

Ashvaghosha

Q21) Answer (a)

Explanation

Tamil poet Appar was a Shiva devotee

So he was a Nayanar saint

Q22) Answer d

Explanation

Samudragupta was a prominent Gupta

ruler whose coins depict him playing a

veena indicating his love for music We

get important historic information from

his Allahabad Prashasti which was

composed by his court poet Harisena

Q23) Answer (b)

Explanation

Vikrama Samvat was founded by

Chandragupta II in the 58 BC as a

mark of victory over the Shakas and

assumed the title of Vikramaditya

Banabhatta wrote Harshavardhanarsquos

biography the Harshacharita in

Sanskrit

Q24) Answer c

Explanation

Sandhi-vigrahika was the minister of

war and peace

Sarthavaha was the leader of the

merchant caravans

Q25) Answer a

Explanation

Xuan Zang (Hsuan-tsang) was a

Chinese traveller who came during the

reign of Harshavardhana

In the decade that began in 630 AD

Xuan Zang came to India through

Kashmir after visiting Central Asia Iran

and Afghanistan

He travelled from north to east and lived

in Bihar for a couple of years

At Nalanda University Xuan Zang

interacted with students and scholars

mastered local languages and

discovered Buddhist stupas

Q26) Answer c

Explanation

Pradakshina patha is a circular path

laid around a stupa in Buddhist

architecture While the rest are a part of

temple architecture

Q27) Answer d

Explanation

All the above-mentioned temples have

an elaborate use of bricks (baked

bricks) along with stone

Q28) Ans (c)

Muhammad Quli Qutab was the Sultan

of Golconda He was a contemporary of

Akbar was very fond of literature and

architecture

The Sultan was a great poet and he

wrote in Dakhini Urdu Persian and

Telgu and has left an extensive diwan or

collection

Recently the Archaeological Survey of

India (ASI) will be using Ground

Penetrating Radar (GPR) to map the

contours of the area around the Bagh-e-

Naya Qila excavated garden inside the

Golconda Fort in Telangana

Q29) Answer a

Explanation

Silappadikaram is a famous Tamil epic

which was written by Ilango around

1800 years ago It is a story of a

merchant named Kovalan who fell in

love with a courtesan named Madhavi

Manimekalai tells the story of the

daughter of Kovalan and Madhavi

Q30) Answer (a)

Explanation

Charaka is the author of Charaka

Samhita which is an important work of

Ayurveda and medicines

Brahmaguptarsquos fame rests mostly on his

Brahma-sphuta-siddhanta which was

an astronomical work It was translated

into Arabic in Baghdad and had a major

impact on Islamic mathematics and

astronomy

Late in his life Brahmagupta wrote

Khandakhadyaka which was an

RAUSIAS-FC19E1003 44

astronomical handbook that employed

Aryabhatarsquos system of starting each day

at midnight

Q31) Answer (c)

Explanation

Amir Khusrau was a famous sufi

musician poet and scholar In 1318 he

noted that there was different language

in every region of this land (Hindustan)

Lahori Kashmiri Dvarsamudri (in

Southern Karnataka) Telangana (in

Andhra Pradesh) Gujari (in Gujarat)

Marsquobari (in Tamil Nadu) Awadhi (in

eastern Uttar Pradesh) and Hindawai (in

the area around in Delhi) etc He went

to explain that Sanskrit did not belong

to any region and that only brahmans

knew it

Q32) Answer c

Explanation

Hiranyagarbha refers to the golden

womb When this ritual was performed

with the help of Brahmanas it was

thought to lead to the rebirth of the

sacrificer as a Khastriya

Q33) Answer d

Explanation

Kadamai refers to a tax on land

revenue

Gwalior Prashasti describes the exploits

of Nagabhata who was a Pratihara king

Q34) Answer b

Explanation

Rajatarangini is a Sanskrit text written

by Kalhana in the 12th century

It was historical chronicle of early India

It is justifiably considered to be the best

and most authentic work of its kind

It covers the entire span of history in

the Kashmir region from the earliest

times to the date of its composition

Q35) Answer c

Explanation

ldquoUrrdquo was the general assembly of the

village ldquoUrrdquo consisted of all the

taxpaying residents of an ordinary

village

Q36) Answer (a)

Explanation

Tarikh was a form of history writing in

the Delhi Sultanate The authors of

tawarikhs were learned men which

included secretaries administrators etc

Q37 Answer (a)

Explanation

Alauddin chose to pay his soldiers salaries in cash rather than iqtas The soldiers would buy their supplies from merchants in Delhi and it was thus feared that merchants would raise their prices To stop this Alauddin controlled the prices of goods in Delhi Prices were carefully surveyed by officers and merchants who did not sell at the prescribed rates were punished

Q38) Answer (d)

Explanation

Delhi first became the capital of a

kingdom under the Tomara Rajputs

who were defeated in the middle of the

twelfth century by the Chauhans (also

referred to as Chahamanas) of Ajmer

It was under the Tomaras and

Chauhans that Delhi became an

important commercial centre Many rich

Jaina merchants lived in the city and

constructed several temples Coins

minted here called dehliwal had a wide

circulation

Q39) Answer (c)

Explanation

Moth ki Masjid was built in the reign of

Sikandar Lodi by his minister

Begumpuri mosque built in the reign of

Muhammad Tughluq was the main

mosque of Jahanpanah the ldquoSanctuary

of the Worldrdquo and his new capital in

Delhi

Quwwat al ndash Islam mosque was

enlarged by Iltutmish and Alauddin

Khalji The minar was built by three

Sultansndash Qutbuddin Aybak Iltutmish

and Firuz Shah Tughluq

RAUSIAS-FC19E1003 45

Q40) Answer (c)

Explanation

Under the Mughals mansabdar was

referred to an individual who held a

mansab ie rank and he received his

salary as revenue assignments called

jagirs

Q41) Ans (b)

The Quit India Movement was a

spontaneous revolt of people against

British rule

The All India Congress Committee met

at Bombay on 8 August 1942 It passed

the famous resolution Quit India and

proposed the starting of a non-violent

mass struggle under Gandhis

leadership to achieve this aim But on

the very next day Gandhi and other

eminent leaders of the Congress were

arrested The Congress was once again

declared illegal

Q42) Ans (c)

The Simon Commission refers to a

group of seven MPs from the United

Kingdom constituted to suggest

constitutional reforms for British India

The Commission consisted of only

British members headed by one of the

senior British politicians Sir John

Simon

So the people of India agitated against

the arrival of Simon Commission

Q43) Ans (a)

He was widely known for his

unfavourable opinion of the economic

consequences of the British rule in

India

In his many writings and speeches and

especially in Poverty and Un-British

Rule in India Naoroji argued that India

was too highly taxed and that its wealth

was being drained away to England

He did not interpret the ancient Indian

texts and restored the self-confidence of

Indians And also he did not stress the

need for eradication of all the social

evils before anything else

Q44) Ans (c)

In August 1932 Prime Minister

MacDonald announced his Communal

Award Great Britainrsquos unilateral

attempt to resolve the various conflicts

among Indiarsquos many communal

interests

The award which was later

incorporated into the act of 1935

expanded the separate-electorate

formula reserved for Muslims to other

minorities including Sikhs Indian

Christians Anglo-Indians Europeans

distinct regional groups Gandhi

undertook a ldquofast unto deathrdquo against

that offer which he viewed as a

nefarious British plot to divide the

Indian society

Q45) Ans (b)

In British India apart from existing

imports and exports there was also a

particular amount of money which

colonial India contributed towards

administration maintenance of the

army war expenses pensions to retired

officers and other expenses accrued by

Britain towards maintenance of her

colony These were known as Home

charges and were paid for almost

entirely by India

The Home charges was made of

following components-

- Interest payable on Indian debt

- Dividend to shareholders of East

India Company

- Funds used to support the India

Office in London

- Funds used to pay salaries and

pensions of British personnel

engaged in India

- Interest on the railways

- Civil and military charges

- Store purchases in England

Q46) Ans (b)

The Lahore session of the Indian

National Congress was held in 1929

under the Presidentship of Jawaharlal

Nehru

The Lahore session of the Indian

National Congress witnessed significant

RAUSIAS-FC19E1003 46

developments in the Indian national

movement

- First the election of Jawaharlal

Nehru to the post of Presidentship of

the Congress was a clear indication

of the growing strength of the

Leftists in the Congress

- Secondly it was in this session that

the Congress for the first time raised

the demand for complete

independence Such demand was

not raised from the Congress

platform earlier

Q47) Ans (b)

It did not provide for separate

electorates for any community or

weightage for minorities However it did

allow for the reservation of minority

seats in provinces having minorities of

at least ten per cent but this was to be

in strict proportion to the size of the

community

There was no provision for complete

Independence for India

Q48) Ans (c)

The religion of early Vedic Aryans was

primarily of worship of nature and

Yajnas

The early Aryan religion was kind of

nature worship Actually the forces

around them which they could not

control or understand were invested

with divinity and were personified as

male or female gods And they

performed some Yajnas also

Q49) Ans (b)

The roads and river-routes were not

immune from robbery It is notable that

Yuan Chwang (Hiuen Tsang) was

robbed of his belongings during

Harshvardanarsquos period

Q50) Ans (c)

Q51) Ans (b)

Purandara Dasa was a saint and great

devotee of Lord Krishna

There is much speculation about where

Purandara Dasa regarded as the

Pitamaha of Carnatic music was born

Recently an expert committee

constituted by the Kannada University

Hampi has come to the conclusion that

Kshemapura Shivamogga district

Karnataka is the birth place of

Purandara Dasa

Q52) Ans (c)

Sri Tyagaraja Sri Shyama Shastry and Sri Muthuswami Dikshitar are considered the trinity of Carnatic music and with them came the golden age in Carnatic music in the 18th-19th

century

Q53) Ans d)

Recently a rare sarcophagus (stone

coffin) which is 2000 years old from the

Iron AgendashMegalithic era was discovered

from a rock-cut cave at Viyur village of

Kollam near Koyilandy in Kozhikode

district Kerala

The coffin containing bone fragments

was found during an excavation ldquoSo

far such a rare finding has been

discovered only from two sites

in Kerala Both these sarcophagi were

recovered from Megalithic sites at

Chevayur and Atholi also in Kozhikode

district

Q54) Ans a)

The megalithic culture in South India was a full-fledged Iron Age culture

Q55) Ans d)

The Cholas Pandyas and Keralaputras

(Cheras) mentioned in Ashokan

inscriptions were probably in the late

megalithic phase of material culture

Q56) Ans d)

Q57) Ans (b)

Raj Kumar Shukla followed Gandhiji all

over the country to persuade him to

come to Champaran to investigate the

problem associated with tinkathia

system

RAUSIAS-FC19E1003 47

Brij Kishore Rajendra Prasad Mahadev

Desai and Narhari Parikh accompanied

Gandhi ji during the Champaran

Satyagraha

Q58) Ans (b)

The Satvahanas started the practice of granting tax-free villages to brahmanas and Buddhist monks

Q59) Ans c)

The objectives of the Programme are

listed as under

- Developing basic tourism

infrastructure

- Promoting cultural and heritage

value of the country to generate

livelihoods in the identified regions

- Enhancing the tourist attractiveness

in a sustainable manner by

developing world-class

infrastructure at the heritage

monument sites

- Creating employment through active

involvement of local communities

- Harnessing tourism potential for its

effects on employment generation

and economic development

- Developing sustainable tourism

infrastructure and ensuring proper

Operations and maintenance

therein

Q60) Ans (b)

The Tribal Cooperative Marketing

Development Federation of India

(TRIFED) came into existence in 1987

It is a national-level apex organization

functioning under the administrative

control of Ministry of Tribal Affairs

Govt of India

TRIFED has its registered and Head

Office located in New Delhi

Q61) Ans (c)

Premchandrsquos novels include

Premashram Rangabhumi Ghaban

Karmabhumi and Godan

Gora is a novel written by Rabindranath

Tagore

138th birth anniversary of Munshi

Premchand was celebrated across the

country

Q62) Ans (b)

Giddha is a traditional pastoral dance

performed by the women of the Punjab

India and Pakistan at festival times

and at the sowing and reaping of the

harvest

By this dance the Punjabi women

reveal their joy expel their suppressed

feelings in a male dominated society

through the performance of Giddha

Since this dance has nothing to do with

men only women can participate in it

During the Teej celebrations Giddha

dance is celebrated in Punjab every

year Teej is a generic name for a

number of festivals that are celebrated

by women in some parts of India

Q63) Ans (a)

Dara Shukoh wrote the remarkable

work called ldquoMajma-ul-Bahrainrdquo or the

ldquoThe confluence of two seasrdquo

The Vice President of India Shri M

Venkaiah Naidu has said that Prince

Dara Shukohrsquos writings can come as a

refreshing source for infusing peace and

harmony He was addressing the

gathering after visiting the exhibition

that showcases the forgotten Prince of

yesteryears Dara Shukoh organized by

Mr Francois Gautier at Indira Gandhi

National Centre for the Arts in New

Delhi

Q64) Ans (c)

The statue Gommateshwara is

dedicated to the Jain God Bahubali

It is a monolithic statue

President Ram Nath Kovind

inaugurated the grand anointing

ceremony mdash Mahamastakabhisheka mdash

held once in 12 years at

Shravanabelagola (Karnataka)

Q65) Ans (c)

Prachi Valley had come up around the

Prachi river Prachi Valley gradually

disappeared

RAUSIAS-FC19E1003 48

The Prachi river originates from

Bhubaneswar

It is a tributary of the Mahanadi and

flows through the districts of Puri

Khurda Cuttack and Jagatsinghpur

and the entire region of the river is

termed as the Prachi Valley

It falls into the Bay of Bengal

Archaeological evidence shows that the

Prachi Valley Civilisation predates both

Harappa and Mohenjo-Daro

The Prachi river originates from

Bhubaneswar

Q66) Ans (d)

These monuments are located in

Chhatarpur district Madhya Pradesh

within Vindhya mountain range

Q67) Ans (a)

The book lsquoThoughts on Pakistanrsquo was

written by Dr BR Ambedkar

On the occasion of the birth anniversary

of Dr BR Ambedkar the president of

India pays homage to this icon of India

In 1924 he founded the Depressed

Classes Institute (Bahishkrit Hitkarini

Sabha) and in 1927 the Samaj Samata

Sangh

Another area of attention for Ambedkar

was education For its spread among

the low classes he set up a network of

colleges by the name of Peoples

Education Society and founded hostels

Q68) Ans(b)

Mehrgarh is a famous Neolithic

settlement in the Indian subcontinent

which is situated in Baluchistan

province Pakistan

A pre-historic rock art site is discovered

in the vast expanse of limestone blocks

on the eastern banks of Naguleru river

near Dachepalli (Andhra Pradesh) It

has thrown light on the Neolithic

civilisation that flourished in Guntur

(Andhra Pradesh) during 1500-2000

BC

Q69) Ans (c)

The 12th and the 13th centuries saw

the emergence of the Kakatiyas They

were at first the feudatories of the

Western Chalukyas of Kalyana Initially

they ruled over a small territory near

Warangal (Telangana)

They introduced Nayakships which was

later adopted and developed by the

Rayas of Vijayanagara

Q70) Ans (a)

The fast had effect of putting pressure

on mill owners who finally agreed to

give the workers a 35 per cent increase

in wages

Google celebrated with a doodle the

132nd birth anniversary of Anasuya

Sarabhai who played a pioneering role

in Indiarsquos labour movement

Q71) Ans (d)

The UNESCOrsquos list of the representative

list of the intangible cultural heritage of

humanity from India are

- Koodiyattam Sanskrit Theatre of

Kerala

- Mudiyettu ritual theatre and dance

drama of Kerala

- Tradition of Vedic Chanting

- Kalbelia folk songs and dances of

Rajasthan

- Ramlila Traditional Performance of

the Ramayana

- Sankirtana ritual singing

drumming and dancing of Manipur

- Ramman religious festival and

ritual theatre of the Garhwal

Himalayas India

- Traditional brass and copper craft of

utensil making among the Thatheras

of Jandiala Guru Punjab India

- Chhau dance classical Indian dance

originated in the eastern Indian

states

- Buddhist chanting of Ladakh

recitation of sacred Buddhist texts

in the trans-Himalayan Ladakh

region Jammu and Kashmir India

- Yoga

- Nouroz

- Kumbh Mela

RAUSIAS-FC19E1003 49

Q72) Ans(b)

The President of India Shri Ram Nath Kovind inaugurated the Hornbill Festival and State Formation Day celebrations of Nagaland in Kisama

The festival is named after the Indian hornbill the large and colourful forest bird which is displayed in the folklore of most of the states tribes

The major recognized tribes of Nagaland are Angami Ao Chakhesang Chang

Kuki Rengma and Zeling etc

Onge Jarawa and Sentinelese are the

tribes of Andman amp Nicobar Islands

Q73) Ans (c)

The Rashtrakutas rule in the Deccan lasted for almost two hundred years till the end of the tenth century The Rashtrakutas rulers were tolerant in their religious views and patronized not only Shaivism and Vaishnavism but

Jainism as well

The famous rock-cut temple of Shiva at Ellora was built by one of the Rashtrakutas kings Krishna I in the ninth century His successor Amoghavarsha was a Jain but he also

patronized other faiths

The Rashtrakutas allowed Muslims traders to settle and permitted Islam to

be preached in their dominions

Recently increasing defacement at the prehistoric rock paintings of Pandavulagutta Telangana has created a cause for grave concern It can spoil

the prehistoric rock

Pandavulagutta is home to

- Painted rock shelters dating to

10000 BC-8000 BC

- An 8th century inscription of the

Rashtrakuta period and

- Painted frescoes from the 12th century Kakatiya empire

Q74) Ans (b)

In 1828 Raja Ram Mohan Roy founded a new religious society the Brahma Sabha later known as the Brahmo

Samaj

Debendranath Tagore headed the Tattvabodhini Sabha which was

engaged in search of spiritual truth

Its purpose was to purify Hinduism and to preach monotheism or belief in one God

The new society was to be based on the twin pillars of reason and the Vedas and

Upanishads

Recently Sadharan Brahmo Samaj (SBS) has entered into a legal battle with the West Bengal government due

to some legal issue

Q75) Ans (c)

The Chishti order was established in India by Khwaja Moinuddin Chishti who came to India around 1192 The Chishtirsquos are considered to be the most influential of the groups of Sufis who migrated to India in the late twelfth century They adapted successfully to the local environment and adopted several features of Indian devotional

traditions

The historical dargah of Sufi mystic Khwaja Moinuddin Chishti in Ajmer is all set to get a facelift This 13 th century dargah has been included among the Swachh Iconic Places a clean-up initiative focused on iconic

heritage spiritual and cultural places

Page 9: GENERAL STUDIES (PAPER I) · Test is part of Rau’s IAS Test series for Preliminary Exam 2019 FOUNDATION + CURRENT AFFAIRS GENERAL STUDIES (PAPER –I) FOUNDATION TEST –III TOPIC:

RAUSIAS-FC19E1003 9

Q20) Which of the following statements

isare correct

1 Buddhacharita is authored by

Nagasena

2 The worship of Bodhisattvas was

an important part of Mahayana

Buddhsim

Select the correct answer using the code

given below

(a) 1 only

(b) 2 only

(c) Both 1 and 2

(d) Neither 1 nor 2

Q21) Which of the following statements

isare correct

1 The idea of Bhakti is elucidated in

Bhagavata Gita

2 Tamil poet Appar was an Alvar

saint

Select the correct answer using the code

given below

(a) 1 only

(b) 2 only

(c) Both 1 and 2

(d) Neither 1 nor 2

Q22) Which of the following rulers had his

image inscribed in the coins while

playing a veena

(a) Meander

(b) Chandragupta Maurya

(c) Gautamiputra Satkarni

(d) Samudragupta

Q23) Which of the following statements

isare correct

1 Vikrama Samvat is adopted as the

national calendar by independent

India and it began in 68 BC

2 Banabhatta was a court poet of

Harshavardhana

Select the correct answer using the code

given below

(a) 1 only

(b) 2 only

(c) Both 1 and 2

(d) Neither 1 nor 2

Q24) Consider the following pairs

1 Sandhi-vigrahika Minister of trade

2 Prathama-kulika Chief craftsman

3 Sarthavaha Chief banker

Which of the pairs given above isare

correct

(a) 1 only

(b) 1 and 3 only

(c) 2 only

(d) 2 and 3 only

Q25) Which of the following statements

isare correct

1 Aihole inscription belongs to

Pulakeshin II and was composed

by Ravikriti

2 Xuan Zang came to India during

the reign of Chandragupta II

Select the correct answer using the code

given below

(a) 1 only

(b) 2 only

(c) Both 1 and 2

(d) Neither 1 nor 2

RAUSIAS-FC19E1003 10

Q26) िासतकिा स सोबोनरत निमननिखित ततव ो म स कौि-स

ततव किि नहोद मखनदर ो की िासतकिा क भाग ह

1 नशिर

2 मणडप

3 परदनकषरा पथ

4 गभणगह

िीच नदए गए कट का परय ग कर सही उततर चनिएः

(a) किि 1 3 और 4

(b) किि 2 3 और 4

(c) किि 1 2 और 4

(d) 1 2 3 और 4

Q27) निमननिखित मोनदर ो म स कौि-स मोनदर ईोट ो स बि ह

1 दिगढ़ मोनदर

2 भीतरगाि मोनदर

3 िकषमर मोनदर नसरपर

4 बहदशवर मोनदर

िीच नदए गए कट का परय ग कर सही उततर चनिएः

(a) किि 1 2 और 3

(b) किि 2 3 और 4

(c) किि 1 3 और 4

(d) 1 2 3 और 4

Q28) निमननिखित कथि ो म स कौि-सास सही हह

1 सलताि महममद किी कतब शाह अकबर का

समकािीि था

2 िासतकिा क कषतर म महममद किी कतब शाह

ि कई ईमारत ो का निमाणर करिाया था नजिम

स चार मीिार सिाणनरक परनसदध ह

िीच नदए गए कट का परय ग कर सही उततर चनिएः

(a) किि 1

(b) किि 2

(c) 1 और 2 द ि ो

(d) ि त 1 ि ही 2

Q29) निमननिखित यग ो पर निचार कीनजएः

1 मनिमकिाई सततिार

2 अनभजञाि शाको तिम कानिदास

3 नसिपपानदकारम क ििि

उपयणकत यग ो म स कौि-सास सही समनित हह

(a) किि 1 और 2

(b) किि 2

(c) किि 1 और 3

(d) 1 2 और 3

Q30) निमननिखित कथि ो म स कौि-सास सही हह

1 ldquoसशरत सोनहताrdquo नचनकरता पर एक महतवपरण

रचिा ह

2 बरहमगपत और चरक महतवपरण गनरतजञ थ

िीच नदए गए कट का परय ग कर सही उततर चनिए

(a) किि 1

(b) किि 2

(c) 1 और 2 द ि ो

(d) ि त 1 ि ही 2

Q31) अमीर िसर क सनदभण म निमननिखित कथि ो म स

कौि-सास सही हह

1 अमीर िसर ि अपिी रचिाओो म नििा ह

नक सोसकत नकसी भी कषतर स सोबोनरत िही ो थी

और किि बराहमर ही इस भािा का जञाि रित

2 उन ोि नहोदिी और अिरी क अखसततव का

उललि नकया था

िीच नदए गए कट का परय ग कर सही उततर चनिए

(a) किि 1

(b) किि 2

(c) 1 और 2 द ि ो

(d) ि त 1 ि ही 2

RAUSIAS-FC19E1003 11

Q26) Which of the following architectural

elements were only part of Hindu

temple architecture

1 Shikhara

2 Mandapa

3 Pradakshina patha

4 Garbhagriha

Select the correct answer using the code

given below

(a) 1 3 and 4 only

(b) 2 3 and 4 only

(c) 1 2 and 4 only

(d) 1 2 3 and 4

Q27) Which of the following temples isare

made of bricks

1 Deogarh Temple

2 Bhitargaon Temple

3 Lakshmana temple Sirpur

4 Brihadeshvara Temple

Select the correct answer using the code

given below

(a) 1 2 and 3 only

(b) 2 3 and 4 only

(c) 1 3 and 4 only

(d) 1 2 3 and 4

Q28) Which of the following statements

isare correct

1 Sultan Muhammad Quli Qutab

Shah was a contemporary of

Akbar

2 In the field of architecture

Muhammad Quli Qutab Shah

constructed many buildings the

most famous of which is the Char

Minar

Select the correct answer using the code

given below

(a) 1 only

(b) 2 only

(c) Both 1 and 2

(d) Neither 1 nor 2

Q29) Consider the following pairs

1 Manimekalai Sattanar

2 Abhijnana Shakuntalam Kalidasa

3 Silappadikaram Kovalan

Which of the pairs given above isare

correct

(a) 1 and 2 only

(b) 2 only

(c) 1 and 3 only

(d) 1 2 and 3

Q30) Which of the following statements

isare correct

1 Sushruta Samhita is an important

work on medicine

2 Brahmagupta and Charaka were

important mathematicians

Select the correct answer using the code

given below

(a) 1 only

(b) 2 only

(c) Both 1 and 2

(d) Neither 1 nor 2

Q31) Which of the following statements

isare correct about Amir Khusrau

1 Amir Khusrau records in his works

that Sanskrit did not belong to any

region and only the Brahmans

knew it

2 He recorded the existence of

Hindawi and Awadhi

Select the correct answer using the code

given below

(a) 1 only

(b) 2 only

(c) Both 1 and 2

(d) Neither 1 nor 2

RAUSIAS-FC19E1003 12

Q32) निमननिखित कथि ो पर निचार कीनजए

1 नहरणय-गभण अिषठाि क बार म ऐसा स चा जाता

था नक बनि दि िाि का एक कषनतरय क रप म

पिजणनम ह गा

2 मयरशमणि कदोब िोश का सोसथापक था

उपयणकत कथि ो म स कौि-सास सही हह

(a) किि 1

(b) किि 2

(c) 1 और 2 द ि ो

(d) ि त 1 ि ही 2

Q33) निमननिखित कथि ो म स कौि-सास सही हह

1 कदमई बगार (बिपिणक शरम) क रप म

निया जाि िािा कर था

2 गवानियर परशखसत म िागभट (ज एक चोदि

राजा था) क दवारा नकय गए श िर का िरणि

नकया गया ह

िीच नदए गए कट का परय ग कर सही उततर चनिए

(a) किि 1

(b) किि 2

(c) 1 और 2 द ि ो

(d) ि त 1 ि ही 2

Q34) निमननिखित कथि ो म स कौि-सास सही हह

1 राजतरो नगिी 11िी ो शताबदी म कलहि क दवारा

रनचत एक सोसकत पसतक (टकसट) ह

2 कननौज क निए नतरपकषीय सोघिण म पाि राजिोश

शानमि था

िीच नदए गए कट का परय ग कर सही उततर चनिए

(a) किि 1

(b) किि 2

(c) 1 और 2 द ि ो

(d) ि त 1 ि ही 2

Q35) निमननिखित यग ो पर निचार कीनजए

1 बरहदशवर मोनदर राजराजा च ि

2 उर मापि की इकाई

3 दिदाि मोनदर ो क भनम अिदाि

उपयणकत यग ो म स कौि-स सही समनित ह

(a) किि 1 और 2

(b) किि 2 और 3

(c) किि 1 और 3

(d) 1 2 और 3

Q36) निमननिखित कथि ो म स कौि-सास सही हह

1 नदलली क सलताि ो क अरीि परशासि की भािा

फारसी थी

2 नदलली सलतित म ldquoतारीितािरीिrdquo कनिता

का एक रप था

िीच नदए गए कट का परय ग कर सही उततर चनिए

(a) किि 1

(b) किि 2

(c) 1 और 2 द ि ो

(d) ि त 1 ि ही 2

Q37) निमननिखित कथि ो म स कौि-सास सही हह

1 अिाउददीि खििजी ि अपि सनिक ो क निए

नसरी िाम का एक िया दगणरकषक शहर

बिािाया था

2 िह अपि सनिक ो क िति का भगताि इकता

क रप म करता था

िीच नदए गए कट का परय ग कर सही उततर चनिए

(a) किि 1

(b) किि 2

(c) 1 और 2 द ि ो

(d) ि त 1 ि ही 2

RAUSIAS-FC19E1003 13

Q32) Consider the following statements

1 Hiranya-garbha ritual was thought

to lead to the rebirth of the

sacrificer as a Kshatriya

2 Mayurasharman was the founder

of the Kadamba dynasty

Which of the statements given above

isare correct

(a) 1 only

(b) 2 only

(c) Both 1 and 2

(d) Neither 1 nor 2

Q33) Which of the following statements

isare correct

1 Kadamai was tax taken in form of

forced labour

2 Gwalior Prashasti describes the

exploits of Nagabhata who was a

Chandella king

Select the correct answer using the code

given below

(a) 1 only

(b) 2 only

(c) Both 1 and 2

(d) Neither 1 nor 2

Q34) Which of the following statements

isare correct

1 Rajatarangini is a Sanskrit text

written by Kalhana in the 11th

century

2 Pala dynasty was included in the

tripartite struggle for Kannauj

Select the correct answer using the code

given below

(a) 1 only

(b) 2 only

(c) Both 1 and 2

(d) Neither 1 nor 2

Q35) Consider the following pairs

1 Brihadeshvara temple Rajaraja

Chola

2 ldquoUrrdquo Unit of measurement

3 Devadana Land grants made to

temples

Which of the pairs given above isare

correct

(a) 1 and 2 only

(b) 2 and 3 only

(c) 1 and 3 only

(d) 1 2 and 3

Q36) Which of the following statements

isare correct

1 The language of administration

under the Delhi Sultans was

Persian

2 Tarikhtawarikh was a form of

poetry in the Delhi Sultanate

Select the correct answer using the code

given below

(a) 1 only

(b) 2 only

(c) Both 1 and 2

(d) Neither 1 nor 2

Q37) Which of the following statements

isare correct

1 Alauddin Khilji constructed a new

garrison town named Siri for his

soldiers

2 He paid his soldiers their salaries

in the form of Iqta

Select the correct answer using the code

given below

(a) 1 only

(b) 2 only

(c) Both 1 and 2

(d) Neither 1 nor 2

RAUSIAS-FC19E1003 14

Q38) निमननिखित कथि ो म स कौि-सास सही हह

1 नदलली कतबददीि एबक क अरीि पहिी बार

नकसी सामराजय की राजरािी बिी थी

2 दहिीिाि नसक ो का मदरर मग़ि ो क दवारा

नकया गया था

िीच नदए गए कट का परय ग कर सही उततर चनिए

(a) किि 1

(b) किि 2

(c) 1 और 2 द ि ो

(d) ि त 1 ि ही 2

Q39) निमननिखित यग ो पर निचार कीनजए

1 म ठ की मखिद नसको दर ि दी

2 बगमपरी मखिद नफर ज शाह तगिक

3 कववत- अि - इसलाम कतबददीि ऐबक

उपयणकत यग ो म स कौि-स सही समनित ह

(a) किि 1 और 2

(b) किि 2 और 3

(c) किि 1 और 3

(d) 1 2 और 3

Q40) निमननिखित कथि ो म स कौि-सास सही हह

1 मिसबदार ो क अपिा िति राजसव कायो

नजन जागीर कहत थ क रप म परापत ह ता

था

2 मिसबदार क ज सनय उततरदानयतव सौोप जात

थ उसक अनतगणत उस एक निराणररत सखया म

सिार अथिा घड़सिार ो का रि-रिाि करिा

पड़ता था

िीच नदए गए कट का परय ग कर सही उततर चनिए

(a) किि 1

(b) किि 2

(c) 1 और 2 द ि ो

(d) ि त 1 ि ही 2

Q41) ldquo1942 क भारत छ ड़ आोद ििrdquo क बार म

निमननिखित अिि कि ो म स कौि-सा सतय िही ो ह

(a) यह एक अनहोसक आोद िि था

(b) इसका िततव महातमा गाोरी क दवारा नकया गया

था

(c) यह एक सवाभानिक आोद िि था

(d) इसि सामानयतया शरनमक िगण क आकनिणत

िही ो नकया था

Q42) भारत क ि ग ो ि ldquoसाइमि कमीशिrdquo क आगमि क

निरदध आोद िि नकया था कय ोनक

(a) भारतीय कभी भी 1919 क अनरनियम (The

Act of 1919) क काम की समीकषा िही ो करिा

चाहत थ

(b) साइमि कमीशि ि पराोत ो म दवर (द हर) शासि

क समापत करि की नसफाररश की थी

(c) साइमि कमीशि म क ई भारतीय सदसय िही ो

था

(d) साइमि कमीशि ि दश क निभाजि का

सझाि नदया था

Q43) निमननिखित कथि ो पर निचार कीनजए

भारतीय राषटर ीय आोद िि म दादाभाई िौर जी क दवारा

नकया गया सबस परभािी य गदाि यह था नक उन ोि

1 अोगरज ो क दवारा भारत क आनथणक श िर का

ििासा नकया था

2 पराचीि भारतीय गरोथ ो की वयाखया की थी और

भारतीय ो क आतमनिशवास क पिःसथानपत नकया

था

3 अनय नकसी भी बात स पहि सभी सामानजक

बराइय ो क उनमिि की आिशयकता पर बि

नदया था

उपयणकत कथि ो म स कौि-सास सही हह

(a) किि 1

(b) किि 2 और 3

(c) किि 1 और 3

(d) 1 2 और 3

RAUSIAS-FC19E1003 15

Q38) Which of the following statements

isare correct

1 Delhi first became the capital of a

kingdom under Qutubuddin

Aibak

2 Dehliwal coins were minted by the

Mughals

Select the correct answer using the code

given below

(a) 1 only

(b) 2 only

(c) Both 1 and 2

(d) Neither 1 nor 2

Q39) Consider the following pairs

1 Moth ki Masjid- Sikander Lodi

2 Begumpuri mosque- Firuz Shah

Tughluq

3 Quwwat al ndash Islam- Qutubuddin

Aibak

Which of the above pairs isare correct

(a) 1 and 2 only

(b) 2 and 3 only

(c) 1 and 3 only

(d) 1 2 and 3

Q40) Which of the following statements

isare correct

1 Mansabdars received their salaries

as revenue assignments called

jagirs

2 The mansabdarrsquos military

responsibilities required him to

maintain a specified number of

sawar or cavalrymen

Select the correct answer using the code

given below

(a) 1 only

(b) 2 only

(c) Both 1 and 2

(d) Neither 1 nor 2

Q41) Which one of the following observations

is not true about the Quit India

Movement of 1942

(a) It was a non-violent movement

(b) It was led by Mahatma Gandhi

(c) It was a spontaneous movement

(d) It did not attract the labour class

in general

Q42) The people of India agitated against the

arrival of the Simon Commission

because

(a) Indians never wanted the review of

the working of the Act of 1919

(b) Simon Commission recommended

the abolition of dyarchy in the

Provinces

(c) there was no Indian member in the

Simon Commission

(d) the Simon Commission suggested

the partition of the country

Q43) Consider the following statements

The most effective contribution made by

Dadabhai Naoroji to the cause of Indian

National Movement was that he-

1 exposed the economic exploitation

of India by the British

2 interpreted the ancient Indian

texts and restored the self-

confidence of Indians

3 stressed the need for eradication of

all the social evils before anything

else

Which of the statements given above

isare correct

(a) 1 only

(b) 2 and 3 only

(c) 1 and 3 only

(d) 1 2 and 3

RAUSIAS-FC19E1003 16

Q44) महातमा गाोरी ि 1932 म आमरर अिशि नकया था

कय ोनक

(a) ldquoग िमज सममििrdquo (The Round Table

Conference) भारतीय राजिीनतक

आकाोकषाओो क परा करि म असफि रहा था

(b) काोगरस और मखसलम िीग म मतभद थ

(c) रामस मकड िालड (Ramsay Macdonald)

ि ldquoसाोपरदानयक परसकारrdquo (The Communal

Award) की घ िरा की थी

(d) ldquoसनििय अिजञा आोद ििrdquo (The Civil

Disobedience Movement) असफि रहा

था

Q45) भारत म औपनििनशक शासि की अिनर क सोदभण म

भारत स रि क बनहगणमि का एक महतवपरण भाग गह

शलक (Home Charges) था निमननिखित म स

कौि-सास क ि गह शलक म सखममनित नकया गया

थानकय गए थ

1 िोदि म भारत कायाणिय क निए उपय ग नकय

जाि िािा क ि

2 भारत म नियकत नबरनटश कनमणय ो क िति और

पशि का भगताि करि क निए उपय ग नकय

जाि िािा क ि

3 अोगरज ो क दवारा भारत क बाहर यदध ो क निए

उपय ग नकय जाि िािा क ि

िीच नदए गए कट का परय ग कर सही उततर चनिए

(a) किि 1

(b) किि 1 और 2

(c) किि 2 और 3

(d) 1 2 और 3

Q46) सवतोतरता आोद िि क इनतहास म भारतीय राषटर ीय

काोगरस का 1929 का सतर महतवपरण ह कय ोनक इसम

(a) काोगरस क उददशय क रप म सथािीय सरकार

की पराखपत की घ िरा की गई थी

(b) परण सवराज की पराखपत क काोगरस क िकषय क

रप म अपिाया गया था

(c) असहय ग आोद िि शर नकया गया था

(d) िोदि म ldquoग ि मर सममििrdquo (The Round

Table Conference) म भाग िि का निरणय

निया गया था

Q47) भारतीय सवतोतरता सोगराम क सोदभण म िहर ररप टण

क दवारा निमननिखित म स नकसकी नसफाररश की गई

थीनकिकी नसफाररश की गई थी ो

1 भारत क निए परण सवतोतरता

2 अलपसोखयक ो क निए सीट ो क आरकषर क

निए सोयकत नििाणचक मोडि

3 सोनिराि म भारत क ि ग ो क निए मौनिक

अनरकार ो का परािराि

िीच नदए गए कट का परय ग कर सही उततर चनिए

(a) किि 1

(b) किि 2 और 3

(c) किि 1 और 3

(d) 1 2 और 3

Q48) आरो नभक िनदक आयो का रमण मखय रप स था

(a) भखकत

(b) मनतण पजा और यजञ

(c) परकनत की पजा और यजञ

(d) परकनत की पजा और भखकत

RAUSIAS-FC19E1003 17

Q44) Mahatma Gandhi undertook fast unto

death in 1932 mainly because

(a) The Round Table Conference failed

to satisfy Indian political

aspirations

(b) The Congress and Muslim League

had differences of opinion

(c) Ramsay Macdonald announced the

Communal Award

(d) The Civil Disobedience Movement

failed

Q45) With reference to the period of colonial

rule in India ldquoHome Chargesrdquo formed

an important part of drain of wealth

from India Which of the following funds

constituted ldquoHome Chargesrdquo

1 Funds used to support the India

Office in London

2 Funds used to pay salaries and

pensions of British personnel

engaged in India

3 Funds used for waging wars

outside India by the British

Select the correct answer using the code

given below

(a) 1 only

(b) 1 and 2 only

(c) 2 and 3 only

(d) 1 2 and 3

Q46) The 1929- Session of Indian National

Congress is of significance in the history

of the Freedom Movement because the-

(a) attainment of Self-Government

was declared as the objective of

the Congress

(b) attainment of Poorna Swaraj was

adopted as the goal of the

Congress

(c) Non-Cooperation Movement was

launched

(d) decision to participate in the

Round Table Conference in

London was taken

Q47) With reference to the period of Indian

freedom struggle which of the following

waswere recommended by the Nehru

report

1 Complete Independence for India

2 Joint electorates for reservation of

seats for minorities

3 Provision of fundamental rights for

the people of India in the

Constitution

Select the correct answer using the code

given below

(a) 1 only

(b) 2 and 3 only

(c) 1 and 3 only

(d) 1 2 and 3

Q48) The religion of the early Vedic Aryans was primarily of

(a) Bhakti

(b) image worship and Yajnas

(c) worship of nature and Yajnas

(d) worship of nature and Bhakti

RAUSIAS-FC19E1003 18

Q49) भारत की यातरा करि िाि चीिी यातरी यआि चिाोग

(हयएि साोग) ि समकािीि भारत की सामानय

खसथनतय ो और सोसकनत क दजण नकया था इस सोदभण म

निमननिखित कथि ो म स कौि-सास सही हह

1 सड़क और िदी-मागण (जि-मागण) डकती स

परण रप स सरनकषत थ

2 जहा तक अपरार ो क निए दणड की बात ह

उसक निए नकसी भी वयखकत की निदोिता

अथिा उसक अपरार क निराणररत करि क

निए अनि जि और निि परि क माधयम क

सारि थ

3 वयापाररय ो क घाट ो और परनतबोर सटशि ो पर

शलक ो का भगताि करिा पड़ता था

िीच नदए गए कट का परय ग कर सही उततर चनिए

(a) किि 1

(b) किि 2 और 3

(c) किि 1 और 3

(d) 1 2 और 3

Q50) नसोर घाटी सभयता क सोदभण म निमननिखित कथि ो पर

निचार कीनजए

1 यह मखय रप स एक रमणनिरपकष सभयता थी

तथा हािाोनक इसम रानमणक ततव मौजद था

िनकि िह परनतिश पर हािी िही ो था

2 इस काि क दौराि भारत म कपास का परय ग

कपड़ा बिाि क निए नकया जाता था

उपयणकत कथि ो म स कौि-सास सही हह

(a) किि 1

(b) किि 2

(c) 1 और 2 द ि ो

(d) ि त 1 ि ही 2

Q51) परोदर दास क सोदभण म निमननिखित कथि ो पर निचार

कीनजए

1 परोदर दास एक सोत और भगिाि नशि क एक

महाि भकत थ

2 ि एक सोगीतकार गायक और किाणटक सोगीत

क मखय सोसथापक-परसतािक ो म स एक थ

उपयणकत कथि ो म स कौि-सास सही हह

(a) किि 1

(b) किि 2

(c) 1 और 2 द ि ो

(d) ि त 1 ि ही 2

Q52) निमननिखित म स कौि-सास वयखकत किाणटक सोगीत

की नतरमनतण म शानमि हह

1 बािामरिी कषणा

2 शरी शयाम शासतरी

3 शरी मथसवामी दीनकषतर

िीच नदए गए कट का परय ग कर सही उततर चनिए

(a) किि 1

(b) किि 2

(c) किि 2 और 3

(d) 1 2 और 3

Q53) चियर (Chevayur) और अथ िी (Atholi) म खसथत

महापािार सथि निमननिखित म स नकस राजय म खसथत

(a) तनमििाड

(b) किाणटक

(c) पनिम बोगाि

(d) करि

RAUSIAS-FC19E1003 19

Q49) The Chinese traveller Yuan Chwang

(Hiuen Tsang) who visited India

recorded the general conditions and

culture of India at that time In this

context which of the following

statements isare correct

1 The roads and river-routes were

completely immune from robbery

2 As regards punishment for

offences ordeals by fire water and

poison were the instruments for

determining the innocence or guilt

of a person

3 The tradesmen had to pay duties

at ferries and barrier stations

Select the correct answer using the code

given below

(a) 1 only

(b) 2 and 3 only

(c) 1 and 3 only

(d) 1 2 and 3

Q50) Regarding the Indus Valley Civilization

consider the following statements

1 It was predominantly a secular

civilization and the religious

element though present did not

dominate the scene

2 During this period cotton was

used for manufacturing textiles in

India

Which of the statements given above

isare correct

(a) 1 only

(b) 2 only

(c) Both 1 and 2

(d) Neither 1 nor 2

Q51) Consider the following statements

regarding Purandara Dasa

1 Purandara Dasa was a saint and

great devotee of Lord Shiva

2 He was a composer singer and

one of the chief founding-

proponents of the Carnatic music

Which of the statements given above

isare correct

(a) 1 only

(b) 2 only

(c) Both 1 and 2

(d) Neither 1 nor 2

Q52) Which of the following persons isare

included in the trinity of Carnatic

music

1 Balamurali Krishna

2 Sri Shyama Shastry

3 Sri Muthuswami Dikshitar

Select the correct answer using the code

given below

(a) 1 only

(b) 2 only

(c) 2 and 3 only

(d) 1 2 and 3

Q53) Megalithic sites at Chevayur and Atholi

are located in which of the following

states

(a) Tamil Nadu

(b) Karnataka

(c) West Bengal

(d) Kerala

RAUSIAS-FC19E1003 20

Q54) निमननिखित कथि ो पर निचार कीनजए

1 महापािानरक ि ग कबर ो म िसतएो दफिात थ

2 दनकषर भारत म महापािार सोसकनत एक परण

निकनसत तामर यगीि सोसकनत थी

उपयणकत कथि ो म स कौि-सास सही हह

(a) किि 1

(b) किि 2

(c) 1 और 2 द ि ो

(d) ि त 1 ि ही 2

Q55) निमननिखित म स कौि-स सामराजयसामराजय ो का

अश क क अनभिि ो म उललि नकया गया ह

1 च ि

2 पाणडय

3 करिपतर (चर)

िीच नदए गए कट का परय ग कर सही उततर चनिए

(a) किि 1

(b) किि 1 और 2

(c) किि 3

(d) 1 2 और 3

Q56) भीमा-क रगाोि का यदध को पिी क सनिक ो और

बाजीराि नदवतीय क िततव म एक शखकतशािी पशिा

सिा (मराठ ो) क मधय िड़ा गया था यह यदध

निमननिखित म स नकसका नहससा था

(a) परथम आोगल-मराठा यदध का

(b) नदवतीय आोगल-मराठा यदध का

(c) ततीय आोगल-मसर यदध का

(d) ततीय आोगल-मराठा यदध का

Q57) निमननिखित कथि ो पर निचार कीनजए

1 महादि दसाई ि गाोरीजी क चोपारर आि तथा

नतिकनथया पररािी स जड़ी समसया की जाोच

क निए रारी करि क निए दश भर म उिका

अिसरर नकया था

2 िरहरी पाररि चोपारर सतयागरह क दौराि

गाोरीजी क साथ थ

उपयणकत कथि ो म स कौि-सास सही हह

(a) किि 1

(b) किि 2

(c) 1 और 2 द ि ो

(d) ि त 1 ि ही 2

Q58) निमननिखित कथि ो पर निचार कीनजए

1 िनद राज-िोश ि बराहमर ो और बौदध मठराररय ो

क कर-मकत गाि अिदाि म दि की परथा

आरि की थी

2 सतिाहि ो की आनरकाररक भािा पराकत थी

उपयणकत कथि ो म स कौि-सास सही हह

(a) किि 1

(b) किि 2

(c) 1 और 2 द ि ो

(d) ि त 1 ि ही 2

Q59) एक निरासत क अपिाइए (अडॉपट ए हररटज ndash

Adopt a Heritage) पररय जिा क उददशय ो क

सनदभण म निमननिखित कथि ो पर निचार कीनजए

1 यह पररय जिा र रगार उतपादि और आनथणक

निकास क निए पयणटि कषमता का उि पर

परभाि का उपय ग करगी

2 यह पररय जिा निरासत सथि ो पर निशव सतरीय

आराररक सोरचिा निकनसत करक एक सतत

तरीक स पयणटक आकिणर म िखदध करगी

उपयणकत कथि ो म स कौि-सास सही हह

(a) किि 1

(b) किि 2

(c) 1 और 2 द ि ो

(d) ि त 1 ि ही 2

RAUSIAS-FC19E1003 21

Q54) Consider the following statements

1 Megalithic people buried goods in

graves

2 The megalithic culture in South

India was a full-fledged Copper

Age culture

Which of the statements given above

isare correct

(a) 1 only

(b) 2 only

(c) Both 1 and 2

(d) Neither 1 nor 2

Q55) Which of the following kingdoms isare

mentioned in the Ashokan inscriptions

1 Cholas

2 Pandyas

3 Keralaputras (Cheras)

Select the correct answer using the code

given below

(a) 1 only

(b) 1 and 2 only

(c) 3 only

(d) 1 2 and 3

Q56) The Battle of Bhima-Koregaon was

fought between the soldiers of the

Company and the strong Peshwa army

(Marathas) under Bajirao II This war

was a part of the

(a) First Anglo-Maratha war

(b) Second Anglo-Maratha war

(c) Third Anglo- Mysore war

(d) Third Anglo-Maratha war

Q57) Consider the following statements

1 Mahadev Desai followed Gandhiji all over the country to persuade him to come to Champaran to investigate the problem associated

with tinkathia system

2 Narhari Parikh accompanied Gandhi ji during the Champaran

Satyagraha

Which of the statements given above isare correct

(a) 1 only

(b) 2 only

(c) Both 1 and 2

(d) Neither 1 nor 2

Q58) Consider the following statements

1 The Nanda Dynasty started the practice of granting tax-free villages to brahmanas and

Buddhist monks

2 The official language of the Satavahanas was Prakrit

Which of the statements given above

isare correct

(a) 1 only

(b) 2 only

(c) Both 1 and 2

(d) Neither 1 nor 2

Q59) Consider the following statements about the objectives of the lsquoadopt a heritagersquo

project

1 It will harness tourism potential for its effects on employment generation and economic

development

2 It will enhance the tourist attractiveness in a sustainable manner by developing world class infrastructure at heritage sites

Which of the statements given above

isare correct

(a) 1 only

(b) 2 only

(c) Both 1 and 2

(d) Neither 1 nor 2

RAUSIAS-FC19E1003 22

Q60) ldquoभारतीय जिजातीय सहकारी निपरि निकास सोघrdquo

(The Tribal Co-operative Marketing

Development Federation of India - TRIFED)

क सोदभण म निमननिखित कथि ो पर निचार कीनजए

1 यह एक राषटर ीय सतर का शीिण सोगठि ह ज

भारत सरकार क गह मोतरािय क परशासनिक

नियोतरर क अरीि काम कर रहा ह

2 इसका मखय उददशय दश म जिजातीय ि ग ो

का सामानजक-आनथणक निकास करिा ह

उपयणकत कथि ो म स कौि-सास सही हह

(a) किि 1

(b) किि 2

(c) 1 और 2 द ि ो

(d) ि त 1 ि ही 2

Q61) निमननिखित म स कौि-सास उपनयास परमचोद क

दवारा नििा गया हनिि गए ह

1 रोगभनम

2 ग दाि

3 ग रा

िीच नदए गए कट का परय ग कर सही उततर चनिए

(a) किि 1

(b) किि 2

(c) किि 1 और 2

(d) 1 2 और 3

Q62) नगदधा ितय क सोदभण म निमननिखित कथि ो पर निचार

कीनजए

1 नगदधा नबहार की मनहिाओो क दवारा तयौहार क

समय और फसि की बिाई तथा कटाई क

अिसर पर नकया जाि िािा एक पारोपररक

दहाती ितय ह

2 इस ितय क दवारा मनहिाऐो अपिी परसननता

परकट करती ह तथा नगदधा क परदशणि क

माधयम स परि िचणसव िाि समाज म

मनहिाओो की दबी हई भाििाओो क परकट

करती ह

उपयणकत कथि ो म स कौि-सास सही हह

(a) किि 1

(b) किि 2

(c) 1 और 2 द ि ो

(d) ि त 1 ि ही 2

Q63) निमननिखित कथि ो पर निचार कीनजए

1 मलला शाह बदखशी दारा नशक ह क

आधयाखतमक गर थ

2 औरोगरब ि मजम-उि-बहरीि या द समदर ो

का सोगम िामक उललििीय रचिा नििी थी

3 दारा नशक ह क अपि पिणज अकबर क गर ो

क उततरानरकारी क रप म दिा गया था

नजसम उसि रानमणक बहििाद और समनवयता

क बढ़ािा नदया था

उपयणकत कथि ो म स कौि-सास सही हह

(a) किि 1 और 3

(b) किि 2

(c) किि 1 और 2

(d) 1 2 और 3

RAUSIAS-FC19E1003 23

Q60) Consider the following statements about

the Tribal Cooperative Marketing

Development Federation of India

(TRIFED)

1 It is a national-level apex

organization functioning under the

administrative control of Ministry

of Home Affairs Government of

India

2 The main objective of TRIFED is

socio-economic development of

tribal people in the country

Which of the statements given above

isare correct

(a) 1 only

(b) 2 only

(c) Both 1 and 2

(d) Neither 1 nor 2

Q61) Which of the following novels isare

written by Premchand

1 Rangabhumi

2 Godan

3 Gora

Select the correct answer using the code

given below

(a) 1 only

(b) 2 only

(c) 1 and 2 only

(d) 1 2 and 3

Q62) Consider the following statements about

Giddha dance

1 Giddha is a traditional pastoral

dance performed by the women of

Bihar at festival times and at the

sowing and reaping of the harvest

2 By this dance the women reveal

their joy expel their suppressed

feelings in a male dominated

society through the performance of

Giddha

Which of the statements given above

isare correct

(a) 1 only

(b) 2 only

(c) Both 1 and 2

(d) Neither 1 nor 2

Q63) Consider the following statements

1 Mullah Shah Badakhshi was the

spiritual mentor of Dara Shukoh

2 Aurangzeb wrote the remarkable

work called ldquoMajma-ul-Bahrainrdquo or

the ldquoThe confluence of two seasrdquo

3 Dara Shukoh was seen as

inheriting the qualities of his

ancestor Akbar in that he

promoted religious pluralism and

syncretism

Which of the statements given above

isare correct

(a) 1 and 3 only

(b) 2 only

(c) 1 and 2 only

(d) 1 2 and 3

RAUSIAS-FC19E1003 24

Q64) निमननिखित कथि ो पर निचार कीनजए

1 ग मतशवर परनतमा निोधयनगरी पहाड़ी पर खसथत ह

2 शरिरबिग िा िह सथाि ह जहाो मौयण िोश क

सोसथापक चोदरगपत मौयण अपि नसोहासि क

तयागि क बाद जि तपसवी बि गए थ

उपयणकत कथि ो म स कौि-सास सही हह

(a) किि 1

(b) किि 2

(c) 1 और 2 द ि ो

(d) ि त 1 ि ही 2

Q65) निमननिखित कथि ो पर निचार कीनजए

1 पराताखतवक साकषय स पता चिता ह नक पराची

घाटी सभयता हड़पपा और म हिज दाड़ द ि ो

की पिणिती ह

2 पराची िदी भििशवर स निकिती ह

उपयणकत कथि ो म स कौि-सास सही हह

(a) किि 1

(b) किि 2

(c) 1 और 2 द ि ो

(d) ि त 1 ि ही 2

Q66) निमननिखित कथि ो म स कौि-सास सही हह

1 िजराह क समारक ो क समह का निमाणर

चोदि राजिोश क शासिकाि क दौराि हआ

था

2 य समारक हररिोदर पिणत शरोििा म खसथत ह

3 म रक क यातरी इबन बतता ि अपि सोसमरर ो

म िजराह क मोनदर ो की यातरा का उललि

नकया था तथा इन काजराण िाम स समब नरत

नकया था

िीच नदए गए कट का परय ग कर सही उततर चनिए

(a) किि 1

(b) किि 1 और 2

(c) किि 2 और 3

(d) किि 1 और 3

Q67) निमननिखित कथि ो म स कौि-सास सही हह

1 डॉ बी आर अमबडकर ि दी एनिनहिशि

ऑफ़ कासट (The Annihilation of Caste)

नििी थी नजसम उन ोि नहोद रमण म िोशािगत

पजारी की परथा क उनमिि की आिशयकता

पर बि नदया था

2 डॉ राजदर परसाद ि थॉटस ऑि पानकसताि

(Thoughts on Pakistan) िामक पसतक

नििी थी

िीच नदए गए कट का परय ग कर सही उततर चनिए

(a) किि 1

(b) किि 2

(c) 1 और 2 द ि ो

(d) ि त 1 ि ही 2

Q68) निमननिखित कथि ो म स कौि-सास सही हह

1 महरगढ़ भारतीय उपमहादवीप म एक परनसदध

ििपािार बसती ह ज नसोर पराोत पानकसताि म

खसथत ह

2 बरणह म म कतत ो क उिक सवामी क साथ कबर ो

म दफिाया जाता था

िीच नदए गए कट का परय ग कर सही उततर चनिए

(a) किि 1

(b) किि 2

(c) 1 और 2 द ि ो

(d) ि त 1 ि ही 2

Q69) निमननिखित कथि ो म स कौि-सास सही हह

1 काकानटय मोनदर अनरकतर नशि क समनपणत

2 हिमक ोडा म हजार-सतोभ िाि मोनदर (The

Thousand-Pillared Temple) का निमाणर

काकानटय समराट रदर ि करिाया था

िीच नदए गए कट का परय ग कर सही उततर चनिए

(a) किि 1

(b) किि 2

(c) 1 और 2 द ि ो

(d) ि त 1 ि ही 2

RAUSIAS-FC19E1003 25

Q64) Consider the following statements

1 Gommateshwara Statue is located

on the Vindyagiri Hill

2 Shravanabelagola is the place

where Chandragupta Maurya the

founder of the Mauryan dynasty

became a Jain ascetic after

relinquishing his throne

Which of the statements given above

isare correct

(a) 1 only

(b) 2 only

(c) Both 1 and 2

(d) Neither 1 nor 2

Q65) Consider the following statements

1 Archaeological evidence shows

that the Prachi Valley Civilisation

predates both Harappa and

Mohenjo-Daro

2 The Prachi river originates from

Bhubaneswar

Which of the statements given above

isare correct

(a) 1 only

(b) 2 only

(c) Both 1 and 2

(d) Neither 1 nor 2

Q66) Which of the following statements

isare correct

1 The Khajuraho group of

monuments was built during the

rule of the Chandela dynasty

2 These monuments are located in

Harischandra mountain range

3 Ibn Battuta the Moroccan

traveller in his memoirs mentioned

visiting Khajuraho temples and

called them Kajarra

Select the correct answer using the code

given below

(a) 1 only

(b) 1 and 2

(c) 2 and 3

(d) 1 and 3

Q67) Which of the following statements

isare correct

1 Dr BR Ambedkar wrote the

Annihilation of Caste emphasising

the need to do away with the

practice of hereditary priesthood in

Hinduism

2 The book lsquoThoughts on Pakistanrsquo

was written by Dr Rajendra

Prasad

Select the correct answer using the code

given below

(a) 1 only

(b) 2 only

(c) Both 1 and 2

(d) Neither 1 nor 2

Q68) Which of the following statements

isare correct

1 Mehrgarh is a famous Neolithic

settlement in the Indian

subcontinent which is situated in

Sindh province Pakistan

2 At Burzahom dogs were buried

with their masters in their graves

Select the correct answer using the code

given below

(a) 1 only

(b) 2 only

(c) Both 1 and 2

(d) Neither 1 nor 2

Q69) Which of the following statements

isare correct

1 The Kakatiya temples are

dedicated mostly to Siva

2 The Thousand-Pillared Temple at

Hanamkonda was built by the

Kakatiya king Rudra

Select the correct answer using the code

given below

(a) 1 only

(b) 2 only

(c) Both 1 and 2

(d) Neither 1 nor 2

RAUSIAS-FC19E1003 26

Q70) निमननिखित कथि ो म स कौि-सास सही हह

1 अहमदाबाद नमि हड़ताि क दौराि महातमा

गाोरी ि शरनमक ो क पकष क मजबत करि क

निए आमरर अिशि नकया था

2 अिशि स नमि मानिक ो पर दबाि पड़ा था ज

अोततः शरनमक ो क िति म 15 परनतशत की िखदध

करि क निए सहमत हए थ

िीच नदए गए कट का परय ग कर सही उततर चनिए

(a) किि 1

(b) किि 2

(c) 1 और 2 द ि ो

(d) ि त 1 ि ही 2

Q71) निमननिखित म स नकसक नकिक भारत स यिसक

की माििता की अमतण साोसकनतक निरासत की

परनतनिनर सची (The UNESCOrsquos List of the

Representative List of the Intangible

Cultural Heritage of Humanity) म शानमि

नकया गया ह

1 मनडयटट

2 सोकीतणि

3 को भ मिा

िीच नदए गए कट का परय ग कर सही उततर चनिए

(a) किि 1 और 2

(b) किि 2 और 3

(c) किि 3

(d) 1 2 और 3

Q72) निमननिखित जिजानतय ो म स कौि-सीसी ो

जिजानतजिजानतया िागािड स सोबोनरत हह

1 अोगामी

2 ककी

3 जारिा

िीच नदए गए कट का परय ग कर सही उततर चनिए

(a) किि 1

(b) किि 1 औऔ 2

(c) किि 2

(d) 1 2 और 3

Q73) निमननिखित कथि ो म स कौि-सास सही हह

1 राषटर कट सामराजय की सथापिा दोनतदगण ि की थी

नजसि मानयाित म अपिी राजरािी की

सथापिा की थी

2 राषटर कट समराट अम घििण एक ििक था और

उस कनिताओो पर पहिी कननड़ पसतक नििि

का शरय नदया जाता ह

िीच नदए गए कट का परय ग कर सही उततर चनिए

(a) किि 1

(b) किि 2

(c) 1 और 2 द ि ो

(d) ि त 1 ि ही 2

Q74) निमननिखित कथि ो म स कौि-सास सही हह

1 कशब चोदर सि ि ततवब नरिी सभा की

अधयकषता की थी ज आधयाखतमक सतय की

ि ज म सोिि थी

2 बरहम समाज ि मािि गररमा पर बि नदया

मनतणपजा का निर र नकया और सती परथा जसी

सामानजक बराइय ो की आि चिा की

िीच नदए गए कट का परय ग कर सही उततर चनिए

(a) किि 1

(b) किि 2

(c) 1 और 2 द ि ो

(d) ि त 1 ि ही 2

Q75) निमननिखित कथि ो म स कौि-सास सही हह

1 भारत म नचशती नसिनसिा खवाजा म इिददीि

नचशती क दवारा सथानपत नकया गया था

2 नचशती परोपरा की एक परमि निशिता

आतमसोयम थी नजसम साोसाररक म ह स दरी

बिाए रििा शानमि था

िीच नदए गए कट का परय ग कर सही उततर चनिए

(a) किि 1

(b) किि 2

(c) 1 और 2 द ि ो

(d) ि त 1 ि ही 2

RAUSIAS-FC19E1003 27

Q70) Which of the following statements

isare correct

1 During the Ahmedabad Mill Strike

Mahatma Gandhi undertook a fast

unto death to strengthen the

workersrsquo resolve

2 The fast had effect of putting

pressure on mill owners who

finally agreed to give the workers a

15 per cent increase in wages

Select the correct answer using the code

given below

(a) 1 only

(b) 2 only

(c) Both 1 and 2

(d) Neither 1 nor 2

Q71) Which of the following are included in

the UNESCOrsquos list of the representative

list of the intangible cultural heritage of

humanity from India

1 Mudiyettu

2 Sankirtana

3 Kumbh Mela

Select the correct answer using the code

given below

(a) 1 and 2 only

(b) 2 and 3 only

(c) 3 only

(d) 1 2 and 3

Q72) Which of the following tribes isare

related to Nagaland

1 Angami

2 Kuki

3 Jarawa

Select the correct answer using the code

given below

(a) 1 only

(b) 1 and 2 only

(c) 2 only

(d) 1 2 and 3

Q73) Which of the following statements

isare correct

1 Rashtrakuta kingdom was founded by Dantidurga who established his capital at Manyakhet

2 Amoghavarsha a Rashtrakuta king was an author and is credited with writing the first

Kannada book on poetics

Select the correct answer using the code given below

(a) 1 only

(b) 2 only

(c) Both 1 and 2

(d) Neither 1 nor 2

Q74) Which of the following statements isare correct

1 Keshab Chandra Sen headed the Tattvabodhini Sabha which was engaged in search of spiritual truth

2 The Brahmo Samaj laid emphasis on human dignity opposed idolatry and criticized such social

evils as the practice of Sati

Select the correct answer using the code given below

(a) 1 only

(b) 2 only

(c) Both 1 and 2

(d) Neither 1 nor 2

Q75) Which of the following statements isare correct

1 The Chishti order was established in India by Khwaja Moinuddin

Chishti

2 A major feature of the Chishti tradition was austerity including maintaining a distance from the

worldly power

Select the correct answer using the code

given below

(a) 1 only

(b) 2 only

(c) Both 1 and 2

(d) Neither 1 nor 2

T e s t i s p a r t o f R a u rsquo s I A S T e s t s e r i e s f o r P r e l i m i n a r y E x a m 2 0 1 9

FOUNDATION + CURRENT AFFAIRS

GENERAL STUDIES (PAPER ndashI)

FOUNDATION TEST ndashIII

SUBJECT NCERT History Class VI-X + Current Affairs

Time Allowed 1frac12 Hours Maximum Marks 150

I NSTRUCT IONS

1 IMMEDIATELY AFTER THE COMMENCEMENT OF THE EXAMINATION YOU SHOULD CHECK

THAT THIS TEST BOOKLET DOES NOT HAVE ANY UNPRINTED OR TORN or MISSING PAGES OR

ITEMS ETC IF SO GET IT REPLACED BY A COMPLETE TEST BOOKLET

2 This Test Booklet contains 75 items (questions) Each item is printed both in Hindi and English

Each item comprises four responses (answers) You will select the response which you want to mark

on the Answer Sheet In case you feel that there is more than one correct response mark the

response which you consider the best In any case choose ONLY ONE response for each item

3 You have to mark all your responses ONLY on the separate Answer Sheet (OMR sheet) provided

Read the directions in the Answer Sheet

4 All items carry equal marks

5 Before you proceed to mark in the Answer Sheet the response to various items in the Test booklet

you have to fill in some particulars in the Answer Sheet as per instructions contained therein

6 After you have completed filling in all your responses on the Answer Sheet and the examination has

concluded you should hand over to the Invigilator only the Answer Sheet You are permitted to

take away with you the Test Booklet

7 Penalty for wrong answers

THERE WILL BE PENALTY FOR WRONG ANSWERS MARKED BY A CANDIDATE IN THE

OBJECTIVE TYPE QUESTION PAPERS

(i) There are four alternatives for the answer to every question For each question for which a

wrong answer has been given by the candidate one-third of the marks assigned to that

question will be deducted as penalty

(ii) If a candidate gives more than one answer it will be treated as a wrong answer even if one of

the given answers happens to be correct and there will be same penalty as above to that

question

(iii) If a question is left blank ie no answer is given by the candidate there will be no penalty for

that question

T h i s t e s t i s p a r t o f R a u rsquo s I A S T e s t s e r i e s f o r P r e l i m i n a r y E x a m 2 0 1 9

Test Code

FC19E1003

FC19H1003 29

Answers and Explanations of

NCERT History Class VI-X + Current Affairs (FC19E1003)

Q1) उततर (c)

सपषटीकरण

- ऋगवद म दविय ो और दिताओो क समवपित एक

हजार स अविक सत तर (शल क) ह

- य शल क ऋविय ो क दवारा रच गए थ और परि ो

दवारा सीख जात थ

- हालाोवक कछ शल क मवहलाओो (जस वक अपाला

घ सा ल पामदरा मतरयी और गागी) क दवारा भी रच

गए थ

- ऋगवद म सोिाद क रप म कई शल क मौजद ह

- हम विशवावमतर नामक एक ऋवि और दविय ो क

रप म पजी जान िाली द नवदय ो (वयास और

सतलज) क बीच िाताि का उदाहरण वमलता ह

- इसस पता चलता ह वक विशवावमतर िवदक काल स

सोबोवित थ

Q2) उततर (b)

सपषटीकरण

- करनल गफाओो स राख क अिशि परापत हए ह

ज इस ओर सोकत करत ह वक ततकालीन ल ग

अवि क उपय ग स पररवचत थ

- य गफाएो आोधर परदश म सथथत ह

Q3) उततर (c)

सपषटीकरण

bull बरािह म ितिमान कशमीर म सथथत एक

परागवतहावसक थथल ह जहाो ल ग गडढ क घर ो का

वनमािण करत थ

bull य घर जमीन क ख द कर बनाए जात थ तथा नीच

जान क वलए सीवियाा ह ती थी

bull ऐसा अनमान लगाया जाता ह वक य घर ठो ड क

मौसम म आशरय परदान करत थ

Q4) उततर (c)

सपषटीकरण

bull परालख-विदया (Epigraphy) क वशलालख ो क

अधययन क रप म पररभावित वकया जाता ह

bull हसतवलसखत दसतािज ो क माधयम स इवतहास

और सावहतय क अधययन क पाोडवलवप विजञान

(Manuscriptology) कहत ह

bull पराचीन लखन परणावलय ो क अधययन और

ऐवतहावसक पाोडवलवपय ो क समझन तथा वतवथ

वनिािरण क पलीओगराफी (Palaeography) कहा

जाता ह

bull नयवमजमविकस (Numismatics) वसक ो क

अधययन क सोदवभित करता ह

Q5) उततर (a)

सपषटीकरण

- चरक सोवहता चरक क दवारा वलखी गई आयिद

और िदयक-शासर पर एक महतवपणि पसतक ह

- ि भारतीय िदयक-शासर की पारमपररक परणाली

वजस आयिद क नाम स जाना जाता ह क

अभयासकताि थ

- ऐसा माना जाता ह वक चरक का विकास दसरी

शताबदी (ईसा पिि) और दसरी शताबदी (ईसवी) क

मधय हआ था

Q6) उततर (b)

सपषटीकरण

- भाग फसल ो पर वलए जान िाल कर क सोदवभित

करता ह ज कल फसल उतपादन का 16 िाो भाग

था

- ldquoकममकारrdquo शबद भवमहीन कवि शरवमक िगि क

वलए परय ग वकया जाता था

- ldquoअशवमिrdquo (वजस घ ड क बवलदान क रप म भी

जाना जाता ह) एक अनषठान ह ता था वजसम एक

घ ड क सवतोतर रप स घमन क वलए छ ड वदया

FC19H1003 30

जाता ह और राजा क सवनक उसकी रखिाली

करत थ

Q7) उततर (d)

सपषटीकरण

- ऋगववदक काल म घ ड ो क रथ ो म ज ता जाता था

ज (रथ) भवम मिवशय ो आवद पर कबजा करन क

वलए लड गए यद ो म उपय ग वकए जात थ

- इसस यह पता चलता ह वक घ ड ो यकत रथ ो का

उपय ग महाजनपद काल स काफी पहल आरमभ

हआ था

- ऋगववदक काल म मिवशय ो भवम जल आवद पर

कबजा करन क वलए तथा ल ग ो क पकडन क

वलए यद वकय जात थ

- अविकाोश परि इन यद ो म भाग वलया करत थ

- हालाोवक उस समय क ई वनयवमत सना नही ो ह ती

थी लवकन उस काल म सभाऐो ह ती थी ो वजनम

ल ग यद क मामल ो पर चचाि करत थ

- वनयवमत सनाएा महाजनपद काल का िवशषटय थी

वजनम पदल सवनक ो की विशाल सनाएा रथ तथा

हाथी शावमल ह त थ

Q8) उततर (a)

सपषटीकरण

- बद शाकय कल स सोबोवित थ और कशीनारा म

उनका वनिन हआ था

- बद न अपनी वशकषाएा पराकत भािा म दी थी ो ज

आम ल ग ो की भािा थी

Q9) उततर (c)

सपषटीकरण

- पराचीन भारत म दशिनशासर की छह शाखाएा थी ो

िशविक नयाय समखया य ग पिि वममाोसा और

िदाोत या उततर वममाोसा

- इनकी थथापना करमश कनाद गौतम कवपल

पतोजवल जावमनी और वयास ऋविय ो न की थी

Q10) उततर (b)

सपषटीकरण

महािीर की वशकषाऐो छठी शताबदी म िललभी म

सोकवलत की गई थी ो

Q11) उततर (c)

सपषटीकरण

- पारमपररक रप स चाणकय क कौविलय अथिा

विषणगपत क नाम स जाना जाता ह

- उसन अथिशासतर ज एक पराचीन भारतीय

राजनवतक आलख ह वलखा था

Q12) उततर (d)

सपषटीकरण

- भारत का राषटर ीय वचनह सारनाथ (उततर परदश) क

अश क सतमभ क ऊपर (शीिि पर) वसोह कवपिल

का एक अनरपण ह

- इस राषटर ीय वसदाोत सतयमि जयत क साथ

सोय वजत वकया गया ह

- रामपिि बल का नाम रामपिि (वबहार) क नाम पर

पडा जहाा इसकी ख ज हई थी

- यह अपन नाजक नकाशी मॉडल क वलए परवसदद

ह वजसम क मल तवचा सोिदनशील नथन ो सतकि

कान और मरबत िााग ो क शरषठतर परवतरप क

परदवशित वकया गया ह

- यह भारतीय और फारसी ततव ो का एक ससममशरण

- सोवकससा उततर परदश म सथथत ह

Q13) उततर (a)

सपषटीकरण

का िर वसोह ज एक महान य दा थ वबहार स

सोबोवित थ

Q14) उततर (b)

सपषटीकरण

िललालर शबद बड भ-सवावमय ो क वलए परय ग

वकया जाता था

FC19H1003 31

Q15) उततर (c)

सपषटीकरण

- अररकमड एक तिीय बसती थी जहाो दर दश ो स

आन िाल जहाज ो का माल उतारा जाता था

- यहाो पर ईोि ो का एक विशाल ग दाम वमटटी क

बतिन (वजनम एमफ रा - द हरी मवठय ो का लोबा

घडा - शावमल ह) और एरिाइन (Arretine)

मदभाोड पाए गए थ

- इस थथान पर र मन दीपक काोच क बन पातर और

रतन भी पाए गए थ

Q16) उततर (a)

सपषटीकरण

- मिनदर सोगम कविताओो म उसललसखत एक

तवमल शबद ह वजसका अथि ह ldquoतीन परमखrdquo

- यह तीन सततारि पररिार ो क मसखयाओो क वलए

परय ग वकया जाता ह च ल चर और पाणडय

Q17) उततर (c)

सपषटीकरण

- ऋग िद म सभा विदाथा तथा गण जसी

जनजावतय ो पर अथिा किोब पर आिाररत

सभाओो का उललख ह

- आरसमभक िवदक काल म सभाओो और सवमवतय ो

का विशि महतव ह ता था

- यहाा तक की मसखया अथिा राजा भी उनका

समथिन परापत करन क वलए आतर रहत थ

Q18) उततर (a)

सपषटीकरण

- जन िमि न ईशवर क अससततव क मानयता त दी ह

वकनत उसन ईशवर क वजना क पद स नीच रखा

- जन िमि न बौद िमि की तरह िणि परणाली की

भरतिना नही ो की थी

Q19) उततर (d)

सपषटीकरण

- च ल ो और पाणडय ो न शसकतशाली तिीय शहर ो का

विकास वकया था

- च ल ो का सबस महतवपणि शहर पहार (या

कािरीपटटीनम) था |

- मदरई पाणडय ो की राजिानी थी

Q20) उततर (b)

सपषटीकरण

- ldquoबदचररतrdquo बद का जीिन-ितताोत ह

- इस अशवघ ि क दवारा वलखा गया था

Q21) उततर (a)

सपषटीकरणः

- तवमल कवि अपपर भगिान वशि क भकत थ

- इस परकार ि एक नयनार सोत थ

Q22) उततर (d)

सपषटीकरणः

- समदरगपत एक परवसद गपत शासक था

- उसन वसक ो पर िीणा बजात हए अपनी छवि

अोवकत करिाई थी

- यह सोगीत क परवत उसक परम क दशािता ह

- हम उसकी इलाहाबाद परशससत स महतवपणि

ऐवतहावसक जानकारी वमलती ह वजसकी रचना

उसक दरबार क कवि हररसन न की थी

Q23) उततर (b)

सपषटीकरणः

- विकरम सोित की शरआत ििि 58 ईसा पिि म

चनदरगपत वदवतीय न की थी

- यह शक ो पर उसकी जीत और उस विकरमावदतय

की पदिी वमलन क उपलकषय म आरमभ वकया गया

था

FC19H1003 32

- बानभटट न हिििििन का जीिन-ितताोत हििचररत

(ज सोसकत म थी) वलखी थी

Q24) उततर (c)

सपषटीकरणः

- सोवि-विगरावहका यद एिो शाोवत का मोतरी

- साथििाह वयापाररय ो क कावफल ो का नता

Q25) उततर (a)

सपषटीकरणः

- जआन झाोग (हसआन रताोग ndash Hsuang Tsang)

एक चीनी यातरी था ज हिििििन क शासनकाल म

भारत आया था

- ििि 630 ईसवी स ज दशक आरमभ हआ था उसम

जआन झाोग मधय एवशया ईरान और

अफग़ावनसतान की यातरा करन क पशचात कशमीर

क रासत स भारत आया था

- उसन उततर स पिि तक की यातरा की और िह

लगभग 2 ििि वबहार म रहा

- जआन झाोग न नालनदा विशवविदयालय म विदयावथिय ो

और विदवान ो क साथ पारसपररक विचार-विमशि

वकया थथानीय भािाओ ा म वनपणता परापत की तथा

बौद सतप ो की ख ज की

Q26) उततर (c)

सपषटीकरणः

- परदवकषणा पथ बौद िासतकला म सतप क चार ो

ओर बनाया जान िाला एक घमािदार पथ ह ता

- परशन म वदए गए बाकी क तीन ो ततव वहोद मसनदर ो की

िासतकला क भाग ह

Q27) उततर (d)

सपषटीकरणः

परशन म वदए गए सभी मोवदर ो म वयापक रप स

ईोि ो (पकी ईोि ो) का परय ग पतथर ो क साथ हआ

Q28) उततर (c)

सपषटीकरण

- महममद कली कतब शाह ग लकणडा का सलतान

था

- िह अकबर का समकालीन था

- सावहतय और िासतकला म उसकी अतयाविक

रवच थी

- िह एक महान कवि था

- िह दसखनी उदि फारसी और तलग म वलखता था

- उसन अपन पीछ एक विसतत वदिान (सोगरह)

छ डा ह

- अभी हाल ही म तलोगाना म ग लकणडा क वकल

क अनदर खदाई वकय गए बाग-ए-नाया वकला

बाग क चार ो ओर रप-रखा क मानवचतरण क

वलए भारतीय परातासतवक सिकषण (The

Archaeological Survey of India ndash ASI)

गराउणड पनीिर विोग रडार (Ground Penetrating

Radar) का परय ग करगा

Q29) उततर (a)

सपषटीकरणः

- वसलपपावदकारम एक तवमल महाकावय ह वजसकी

रचना इलाोग क दवारा लगभग 1800 ििि पिि की

गई थी

- यह क िलन नामक एक वयापारी की कहानी ह

ज माििी नामक एक गवणका (िशया) स परम

करन लगा था

- मवनमकलाई क िलन और माििी की पतरी की

कहानी ह

Q30) उततर (a)

सपषटीकरण

- चरक आयिद और वचवकरता की एक महतवपणि

रचना चरक सोवहता क लखक ह

- बरहमगपत क अपनी रचना बरहम-सफि-वसदानत

(ज एक खग लीय रचना ह) क कारण परवससद

वमली

FC19H1003 33

- बगदाद म इसका अनिाद अरबी भािा म वकया

गया था

- इसका इसलावमक गवणत और खग ल-विजञान पर

महतवपणि परभाि पडा था

- बाद म अपन जीिनकाल म बरहमगपत न

ldquoखोडखयाकrdquo वलखी ज एक खग लीय पससतका

(एक छ िी पसतक) थी

- इसम आयिभटट की अिि-रावतर क परतयक वदन की

शरआत परणाली का परय ग वकया गया था

Q31) उततर (c)

सपषटीकरण

- अमीर खसर एक परवसद सफी सोगीतकार कवि

और विदवान थ

- 1318 म उनह ोन पाया वक इस भवम (वहोदसतान) क

हर कषतर म अलग-अलग भािा थी लाहौरी

कशमीरी दवारसमदरी (दवकषणी कनाििक म)

तलोगाना (आोधर परदश म) गजरी (गजरात म)

माबारी (तवमलनाड म ) अििी (पिी उततर परदश

म) और वहोदिी (वदलली क आस-पास क कषतर म)

आवद

- उनह न यह बताया वक सोसकत वकसी भी कषतर स

सोबोवित नही ो थी और किल बराहमण ही इस भािा

का जञान रखत थ

Q32) उततर (c)

सपषटीकरण

- वहरणय-गभि सववणिम गभि क सोदवभित करता ह

- जब बराहमण ो की सहायता स यह अनषठान वकया

जाता था त यह माना जाता था वक बवल दन िाल

का कषवतरय क रप म पनजिनम ह गा

Q33) उततर (d)

सपषटीकरण

- कदमई भवम राजसव पर कर क सोदवभित करता

- गवावलयर परशससत म नागभि क दवारा वकय गए

श िण का िणिन वकया गया ह |

- नागभि एक परवतहार राजा था

Q34) उततर (b)

सपषटीकरण

- राजतरो वगनी 12िी ो शताबदी म कलहन क दवारा

रवचत एक सोसकत पसतक (िकसट) ह

- यह परारसमभक भारत की ऐवतहावसक इवतितत थी

- तकि सोगत रप स इस अपन परकार की सिोततम

और सिािविक विशवसनीय कवत माना जाता ह

- यह कशमीर कषतर क पराचीनतम समय स लकर

उसकी रचना की तारीख तक क समपणि इवतहास

का आचछादन करती ह

Q35) उततर (c)

सपषटीकरण

- गााि की आम सभा क ldquoउरrdquo कहा जाता था

- ldquoउरrdquo म गााि क सभी कर दन िाल वनिासी

शावमल ह त थ

Q36) उततर (a)

सपषटीकरण

- वदलली सलतनत म ldquoतारीखrdquo इवतहास लखन का

एक रप था

- ldquoतािरीखrdquo क लखक विदवान परि ह त थ वजनम

सवचि परशासक इतयावद शावमल थ

Q37) उततर (a)

सपषटीकरण

- अलाउददीन सखलजी अपन सवनक ो क ितन का

भगतान नकद म करता था न वक इकता क रप

- सवनक अपना सामान वदलली म वयापाररय ो स

खरीदत थ अतः इस बात का भय था वक वयापारी

कही ो िसतओो का मलय न बिा द

- इसकी र कथाम क वलए अलाउददीन सखलजी न

वदलली म कीमत ो क वनयसित वकया

FC19H1003 34

- अविकारीगण धयानपििक मलय ो का सिकषण करत

थ तथा ज वयापारी वनिािररत मलय पर माल नही ो

बचत थ उनक दसणडत वकया जाता था

Q38) उततर (d)

सपषटीकरण

- वदलली सििपरथम त मर राजपत ो क अिीन उनक

सामराजय की राजिानी बनी थी

- 12िी ो शताबदी क मधय म अजमर क चौहान ो

(वजनह चाहमान ो क नाम स भी जाना जाता ह) न

त मर राजपत ो क परावजत वकया था

- त मर ो और चौहान ो क अिीन वदलली एक

महतवपणि िावणसजयक क दर बन गया था

- कई जन वयापारी यहाा रहन लग थ और उनह ोन

कई मोवदर भी बनिाए

- यहाा पर मवदरत वसक वजनह ldquoदहलीिालrdquo क नाम

स जाना जाता था वयापक रप स परचलन म थ

Q39) उततर (c)

सपषटीकरण

- म ठ की मसिद का वनमािण वसको दर ल दी क

राजयकाल म उसक मिी क दवारा करिाया गया

था

- बगमपरी मसिद का वनमािण महममद तगलक क

शासनकाल म हआ था

- यह मसिद विशव का पणयथथान (The

Sanctuary of the World) और वदलली म महममद

तगलक की नई राजिानी जहाोपनाह की मखय

मसिद थी

- कववत- अल - इसलाम मसिद का विसतार

इलतसिश और अलाउददीन सखलजी न वकया था

- मीनार का वनमािण तीन सलतान ो कतबददीन ऐबक

इलतसिश और वफर ज शाह तगलक क दवारा

करिाया गया था

Q40) उततर (c)

सपषटीकरण

- मगल ो क अिीन मनसबदार शबद उस वयसकत क

वलए सोदवभित वकया जाता था वजसक पास मनसब

(अथाित पद) ह ता था

- उस अपना ितन राजसव कायो वजनह जागीर कहत

थ क रप म परापत ह ता था

Q41) उततर (b)

सपषटीकरण

- ldquoभारत छ ड आोद लनrdquo वबरविश शासन क

सखलाफ ल ग ो का एक सवाभाविक विदर ह था

- असखल भारतीय काोगरस सवमवत न 8 अगसत 1942

क बमबई म एक बठक का आय जन वकया था

- इस बठक म परवसद सोकलप ldquoभारत छ ड rdquo क

पाररत वकया गया और इस उददशय क परापत करन

क वलए गाोिी क नततव म एक अवहोसक जन सोघिि

आोद लन की शरआत का परसताि वदया गया

- लवकन अगल ही वदन गाोिी और काोगरस क अनय

परमख नताओो क वगरफतार कर वलया गया

- काोगरस क एक बार वफर अिि घ वित वकया गया

था

Q42) उततर (c)

सपषटीकरण

- साइमन कमीशन यनाइविड वको गडम क सात

साोसद ो का एक समह था

- इस वबरविश भारत क वलए सोििावनक सिार ो का

सझाि दन क वलए गवठत वकया गया था

- इस आय ग म िररषठ वबरविश राजनता सर जॉन

साइमन क नततव म किल वबरविश सदसय ही

शावमल थ

- इसवलए भारत क ल ग ो न साइमन कमीशन क

आगमन क विरद आोद लन वकया था

Q43) उततर (a)

सपषटीकरण

bull दादा भाई नौर जी भारत म वबरविश शासन क

आवथिक पररणाम ो क बार म अपनी विर िी

(परवतकल) राय क वलए जान जात थ

FC19H1003 35

bull अपन कई लख ो और भािण ो म विशि रप स

ldquoपाििी एो ड अन-वबरविश रल इन इसणडया

(Poverty and Un-British Rule in India) म

नौर जी न यह तकि वदया वक भारत पर अतयविक

कर लगाया गया था और इसकी सोपवतत इोगलड की

ओर परिावहत की जा रही थी

bull उनह ोन पराचीन भारतीय गरोथ ो की वयाखया करन

का और भारतीय ो क आिविशवास क बहाल

करन पर कायि नही ो वकया था

उनह ोन वकसी और बात स पहल सभी सामावजक

बराइय ो क उनमलन की आिशयकता पर भी बल

नही ो वदया था

Q44) उततर (c)

सपषटीकरण

bull अगसत 1932 म वबरविश परिानमोतरी मकड नालड न

अपन साोपरदावयक परसकार (The Communal

Award) की घ िणा की थी

bull यह भारत क कई साोपरदावयक वहत ो क बीच विवभनन

सोघिो क हल करन क वलए वबरिन का एकतरफा

परयास था

bull यह परसकार (Award) बाद म 1935 क

अविवनयम (The Act of 1935) म शावमल वकया

गया था

bull इस साोपरदावयक परसकार न मससलम ो क वलए

आरवकषत एक अलग वनिािचक मणडल फॉमिल का

विसतार अनय अलपसोखयक ो क वलए वकया था

वजसम वसख ो भारतीय ईसाइय ो आोगल-भारतीय

समदाय यर पीय समदाय तथा विवशषट कषतरीय

समह ो क शावमल वकया गया था

bull गाोिी न इस परसताि क भारतीय समाज क

विभावजत करन क वलए एक घवणत वबरविश

सावजश क रप म दखा और उसक सखलाफ

आमरण अनशन वकया

Q45) उततर (b)

सपषटीकरण

मौजदा आयात और वनयाित क अवतररक़त

औपवनिवशक भारत क वनमनवलसखत खचो क

वलए एक विशिवनवशचत िन रावश भी दनी पडती

थी

(i) परशासन क वयय

(ii) सना क रख-रखाि क वयय

(iii) यद क वयय

(iv) सिावनितत अविकाररय ो की पशन तथा

(v) वबरिन दवारा अपनी उपवनिश बसती

(कॉल नी) क रख-रखाि क वयय

इनह गह शलक (Home Charges) क रप म

जाना जाता था और लगभग परी तरह स भारत क

दवारा इनका भगतान वकया जाता था

bull गह शलक म वनमनवलसखत घिक शावमल थ

(i) भारतीय ऋण पर दय बयाज

(ii) ईसट इोवडया को पनी क शयरिारक ो क

लाभाोश

(iii) लोदन म भारत कायािलय चलान क वलए िन

(iv) भारत म वनयकत वबरविश कवमिय ो क ितन

और पशन का भगतान करन क वलए िन

(v) रलि पर बयाज

(vi) नागररक और सनय शलक

(vii) इोगलड म सट र (सामगरी) की खरीद

Q46) उततर (b)

सपषटीकरण

bull भारतीय राषटर ीय काोगरस का लाहौर सतर 1929 म

जिाहरलाल नहर की अधयकषता म आय वजत

वकया गया था

bull इस सतर म भारतीय राषटर ीय आोद लन स समबसित

कई महतवपणि पररणाम सामन आय थ

(i) सििपरथम इस सतर म काोगरस क अधयकष पद

पर जिाहरलाल नहर क चना गया था ज

काोगरस म िामपोवथय ो की बिती हई ताकत

का सपषट सोकत था

(ii) दसरा इस सतर म पहली बार काोगरस न पणि

सवतोतरता की माोग क उठाया था

इस परकार की माोग काोगरस मोच स पहल कभी भी

नही ो उठाई गई थी

Q47) उततर (b)

सपषटीकरण

FC19H1003 36

bull इस ररप िि न वकसी भी समदाय क वलए पथक

वनिािचक मोडल अथिा अलपसोखयक ो क वलए

भाराोश की वसफाररश नही ो की थी

bull तथावप इस ररप िि न उन पराोत ो म अलपसोखयक

सीि ो क आरकषण की अनमवत दी थी जहाा पर कम

स कम दस परवतशत अलपसोखयक ह

bull लवकन यह समदाय क आकार क अनपात म ह ना

चावहए था

bull इस ररप िि म भारत क वलए पणि सवतोतरता क

वलए क ई पराििान नही ो था

Q48) उततर (c)

सपषटीकरण

bull आरो वभक िवदक आयो का िमि मखय रप स

परकवत की पजा और यजञ था

bull परारो वभक आयि िमि परकवत की पजा क समान था

bull िासति म उनक चार ो ओर की शसकतयाा वजनह न

त ि वनयोवतरत कर सकत थ और न ही समझ पाए

थ उनह वदवयता क साथ वनिवशत वकया गया तथा

उनह मादा या नर दिीदिताओो क रप म

परतीकतव वकया गया था

bull उनह ोन कछ यजञ ो का भी वनषपादन वकया था

Q49) उततर (b)

सपषटीकरण

bull सडक और नदी-मागि (जल-मागि) डकती स

सरवकषत नही ो थ

bull उललखनीय ह वक हिििििन क शासनकाल क

दौरान यआन चिाोग (हयएन साोग) का सारा

सामान लि वलया गया था

Q50) उततर (c)

सपषटीकरण

परशन म वदए गए द न ो कथन सही ह

Q51) उततर (b)

सपषटीकरण

bull परोदर दास एक सोत और भगिान कषण क एक

महान भकत थ

bull परोदर दास क कनाििक सोगीत क वपतामह क

रप म जाना जाता ह

bull यदयवप उनक जनम-थथान क बार म काफी

अिकल लगाई जाती रही ह

bull तथावप अब कननड विशवविदयालय हमपी क दवारा

गवठत एक विशिजञ सवमवत इस वनषकिि पर पहोची

ह वक उनका जनम थथान सोभितया कनाििक का

एक छ िा-सा गााि कषमपरा (वशिम गगा वजला)

था

Q52) उततर (c)

सपषटीकरण

bull शरी तयागराज शरी शयाम शासतरी और शरी मथसवामी

दीवकषतर क कनाििक सोगीत की वतरमवति माना

जाता ह

bull उनक कारण ही 18िी ो-19िी ो शताबदी म कनाििक

सोगीत का सववणिम यग आया था

Q53) उततर (d)

सपषटीकरण

bull अभी हाल ही म लौह यगीन-महापािावणक काल

का 2000 ििि पराना एक दलिभ सारक फगस

(Sarcophagus) (पतथर का ताबत) क ललम क

वियर गाोि (क वयलडी क पास वजला क वझक ड

करल राजय) की एक रॉक-कि गफा स ख जा गया

bull यह ताबत वजसम हविय ो क िकड थ खदाई क

दौरान वमला

bull अभी तक इस परकार की दलिभ ख ज करल क

मातर द ही थथान ो स हई ह

bull य द न ो सारक फगी (Sarcophagi) (पतथर क

ताबत) चियर और अथ ली (वजला क वझक ड) क

महापािाण थथल ो स वमल ह

Q54) उततर (a)

सपषटीकरण

FC19H1003 37

दवकषण भारत म महापािाण सोसकवत एक पणि

विकवसत लौह यगीन सोसकवत थी

Q55) उततर (d)

सपषटीकरण

bull च ल पाणडय और करलपतर (चर) इन तीन ो का

उललख अश क क अवभलख ो म वकया गया ह

bull सोभितः य भौवतक सोसकवत क उततर

महापािावणक चरण म थ

Q56) उततर (d)

सपषटीकरण

bull भीमा-क रगाोि की लडाई ततीय आोगल-मराठा

यद का वहससा थी

Q57) उततर (b)

सपषटीकरण

bull राजकमार शकल न गाोिीजी क चोपारण आन तथा

वतनकवथया परणाली स जडी समसया की जाोच क

वलए रारी करन क वलए दश भर म उनका

अनसरण वकया था

bull बज वकश र राजदर परसाद महादि दसाई और

नरहरी पाररख चोपारण सतयागरह क दौरान गाोिी

जी क सहय गी थ

Q58) उततर (b)

सपषटीकरण

bull बराहमण ो और बौद मठिाररय ो क कर-मकत गााि

अनदान म दन की परथा सतिाहन ो न आरमभ की

थी

Q59) उततर (c)

सपषटीकरण

इस कायिकरम क उददशय वनमनानसार ह

(i) बवनयादी पयििन आिाररक सोरचना का विकास

करना

(ii) चयवनत (पहचान वकय गए) कषतर ो म आजीविका क

सजन क वलए दश क साोसकवतक और विरासत

मलय ो क बिािा दना

(iii) विरासत समारक थथल ो पर विशव सतरीय आिाररक

सोरचना विकवसत करक एक सतत तरीक स

पयििक आकििण म िसद करना

(iv) थथानीय समदाय ो की सवकरय भागीदारी क माधयम

स र रगार ो का सजन करना

(v) र रगार उतपादन और आवथिक विकास क वलए

पयििन कषमता का उन पर परभाि का उपय ग

करना तथा

(vi) िारणीय पयििन आिाररक सोरचना का विकास

करना और उसका उवचत सोचालन तथा

रखरखाि सवनवशचत करना

Q60) उततर (b)

सपषटीकरण

bull यह वनकाय ििि 1987 म अससततव म आया था

bull यह एक राषटर ीय सतर का शीिि सोगठन ह ज भारत

सरकार क जनजातीय मामल ो क मोतरालय क

परशासवनक वनयोतरण क अिीन काम कर रहा ह

bull इसका पोजीकत और परिान कायािलय नई वदलली

म सथथत ह

Q61) उततर (c)

सपषटीकरण

bull परमचोद क उपनयास ो म परमाशरम रोगभवम गबन

कमिभवम और ग दान शावमल ह

bull ग रा रिी ोदरनाथ िग र क दवारा रवचत उपनयास ह

bull अभी हाल ही म मोशी परमचोद की 138िी ो जयोती दश

भर म मनाई गई थी

Q62) उततर (b)

सपषटीकरण

bull ldquoवगदाrdquo पोजाब (भारत) एिो पावकसतान की

मवहलाओो क दवारा तयौहार क समय और फसल

की बिाई तथा किाई क अिसर पर वकया जान

िाला एक पारोपररक दहाती नतय ह

FC19H1003 38

bull इस नतय क माधयम स पोजाबी मवहलाऐो अपनी

परसननता परकि करती ह तथा वगदा क परदशिन क

माधयम स परि िचिसव िाल समाज म मवहलाओो

की दबी हई भािनाओो क परकि करती ह

bull चोवक इस नतय का परि ो क साथ क ई सोबोि नही ो

ह अतः किल मवहलाऐो ही इसम भाग ल सकती

bull हर साल तीज समार ह क दौरान पोजाब म वगदा

नतय वकया जाता ह

तीज भारत क कछ भाग ो म मवहलाओो क दवारा

मनाया जान िाल कई तयौहार ो क वलए एक

वयापक नाम ह

Q63) उततर (a)

सपषटीकरण

- मजम-उल-बहरीन या द समदर ो का सोगम

नामक उललखनीय रचना दारा वशक ह क दवारा

वलखी थी

- भारत क उपराषटर पवत शरी एम िकया नायड न कहा

ह वक राजकमार दारा वशक ह की रचनाएा शाोवत

और सदभाि क बिािा दन क वलए एक तारा सर त

क रप म सामन आ सकती ो ह

- उपराषटर पवत गत ििो क भला वदए गए राजकमार

दारा वशक ह क परदवशित परचवलत करन हत

आय वजत एक परदशिनी का दौरा करन क बाद एक

सभा क सोब वित कर रह थ

- इस परदशिनी का आय जन फर क इस गौवियर

(Francois Gautier) क दवारा lsquoइोवदरा गाोिी नशनल

सिर फॉर द आििसrsquo (The Indira Gandhi

National Centre for the Arts) नई वदलली म

वकया गया था

Q64) उततर (c)

सपषटीकरण

- ग मतशवर परवतमा जन भगिान बाहबली क

समवपित ह

- यह एक एक-चटटानी पतथर की मवति ह

- राषटर पवत राम नाथ क विोद न शरिणबलग ला

(कनाििक) म आय वजत वकय जान िाल भवय

अवभिक समार ह महामसतकावभिक का

उदघािन वकया था

- यह समार ह 12 ििो म एक बार ह ता ह

Q65) उततर (c)

सपषटीकरण

bull पराची घािी पराची नदी क चार ो ओर फली हई थी

bull पराची घािी िीर-िीर विलपत ह गई थी

bull पराची नदी भिनशवर स वनकलती ह

bull यह महानदी की एक सहायक नदी ह और यह

परी खदाि किक तथा जगतवसोहपर वजल ो स

ह कर बहती ह

bull इस नदी क पर कषतर क पराची घािी कहा जाता ह

bull यह नदी बोगाल की खाडी म वगरती ह

परातासतवक साकषय स पता चलता ह वक पराची घािी

सभयता हडपपा और म हनज दाड द न ो की

पिििती ह

Q66) उततर (d)

सपषटीकरण

य समारक छतरपर वजल (मधय परदश) म विोधयाचल

पिित शरोखला म सथथत ह

Q67) उततर (a)

सपषटीकरण

bull थॉिस ऑन पावकसतान नामक पसतक डॉ बी

आर अमबडकर न वलखी थी

bull डॉ बी आर अमबडकर की जयोती क अिसर पर

भारत क राषटर पवत न भारत की इस महान हसती

क शरदाोजवल अवपित की थी

bull डॉ बी आर अमबडकर न 1924 म वडपरथड

कलावसर इोसटीटयि (दवलत िगि सोथथान -

बवहषकत वहतकाररणी सभा) और 1927 म समाज

समता सोघ की थथापना की थी

bull अमबडकर का धयान वशकषा कषतर की ओर भी था

bull उनह ोन वशकषा क वनमन िगो म फलान क वलए

पीपलस एजकशन स साइिी (The Peoples

Education Society) क नाम स महाविदयालय ो क

नििकि और छातरािास ो की थथापना की थी

FC19H1003 39

Q68) उततर (b)

सपषटीकरण

bull महरगि भारतीय उपमहादवीप म एक परवसद

निपािाण बसती ह ज बलवचसतान पराोत

पावकसतान म सथथत ह

bull दचपलली (आोधर परदश) क पास नागलर नदी क

पिी ति ो पर चना पतथर क बलॉक क विशाल

विसतार म एक पिि-ऐवतहावसक रॉक आिि थथल की

ख ज की गई ह

bull इसन 1500-2000 ईसा पिि क दौरान गोिर (आोधर

परदश) म विकवसत निपािाण सभयता पर परकाश

डाला ह

Q69) उततर (c)

सपषटीकरण

bull 12िी ो सदी और 13िी ो सदी म काकाविय िोश का

उदय हआ था

bull ि पहल कलयाण क पवशचमी चालकय ो क सामोत थ

bull परारोभ म उनह ोन िारोगल (तलोगाना) क पास एक

छ ि स कषतर पर शासन वकया था

bull उनह ोन ldquoनायक वयिथथाrdquo की शरआत की थी

वजस बाद म विजयनगर क राय शासक ो न

अपनाया और विकवसत वकया था

Q70) उततर (a)

सपषटीकरण

bull गाोिीजी क अनशन स वमल मावलक ो पर दबाि

पडा था ज अोततः शरवमक ो क ितन म 35 परवतशत

की िसद करन क वलए सहमत हए थ

bull गगल (Google) न अनसया साराभाई वजनह ोन

भारत क शरवमक आोद लन म एक अगरणी भवमका

वनभाई थी की 132िी ो जयोती डडल (Doodle) का

वनमािण करक मनाई

Q71) उततर (d)

सपषटीकरण

भारत स यनसक की मानिता की अमति साोसकवतक

विरासत की परवतवनवि सची म वनमनवलसखत शावमल ह

bull कवडयटटम करल का सोसकत रोगमोच

bull मवडयिि करल का अनषठान रोगमोच और नतय

नाविका

bull िवदक मि जाप की परोपरा

bull राजथथान क कालबवलया ल क गीत और नतय

bull रामलीला रामायण का पारोपररक परदशिन

bull सोकीतिन मवणपर का अनषठान गायन ढ ल िादन

और नतय

bull रममन भारत क गििाल वहमालय का िावमिक

तयौहार और अनषठान रोगमोच

bull जाोदीयाला गर पोजाब क ठठर ो की पीतल और

ताोब क वशलप स वनवमित बतिन ो की पारोपररक कला

bull छाऊ नतय पिी भारतीय राजय ो म जनमी शासतरीय

भारतीय नतय कला

bull लददाख का बौद मि जाप िर ाोस-वहमालयी लददाख

कषतर तथा जमम-कशमीर म पवितर बौद गरोथ ो का पाठ

bull य ग

bull नौर र

bull को भ मला

Q72) उततर (b)

सपषटीकरण

bull भारत क राषटर पवत शरी राम नाथ क विोद न

वकसामा नागालड म हॉनिवबल मह रति और

राजय गठन वदिस समार ह का उदघािन वकया

था

bull हॉनिवबल मह रति का नाम भारतीय हॉनिवबल क

नाम पर पडा ह ज एक विशाल और रोगीन जोगली

पकषी ह

bull यह पकषी नागालड राजय की अविकतर जनजावतय ो

की ल ककथाओो म उसललसखत ह

bull नागालड की परमख मानयता परापत जनजावतयाा ह

अोगामी आओ चखसोग चाोग ककी रगमा और

रवलोग आवद

bull ओोग जारिा और ससिनलीस अोडमान-वनक बार

दवीप समह की जनजावतयाा ह

FC19H1003 40

Q73) उततर (c)

सपषटीकरण

bull दकन म राषटर कि शासन दसिी ो सदी क अोत तक

लगभग 200 ििो तक रहा था

bull राषटर कि शासक अपन िावमिक विचार ो म सवहषण

bull उनह ोन न किल शि िमि और िषणि िमि बसलक

जन िमि क भी सोरकषण वदया था

bull एल रा म वशि क परवसद रॉक कि मोवदर का

वनमािण नौिी ो सदी म राषटर कि राजा कषण परथम न

करिाया था

bull उसका उततराविकारी अम घििि जन था लवकन

उसन अनय िमो क भी सोरकषण परदान वकया था

bull राषटर कि ो न मसलमान वयापाररय ो क बसन की

अनमवत दी थी

bull उनह न अपन अविराजय ो म इसलाम क उपदश दन

की भी अनमवत दी थी

bull अभी हाल ही म पाोडिलागटटा (तलोगाना) क

परागवतहावसक चटटान वचतर ो क कषरण की बिती हई

घिनाएा एक गोभीर वचोता का वििय ह

bull यह परागवतहावसक चटटान क नकसान पहाचा

सकता ह

bull पाोडिलागटटा वनमनवलसखत क वलए जाना जाता ह

- 10000 ईसा पिि स 8000 ईसा पिि क वचवतरत

चटटानी आशरय ो क वलए

- राषटर कि काल क एक 8 िी ो सदी क

वशलालख क वलए और

- 12िी ो सदी क काकविय सामराजय क वभवतत

वचतर ो क वलए

Q74) उततर (b)

सपषटीकरण

bull 1828 म राजा राम म हन रॉय न एक नय िावमिक

समाज बरहम सभा की थथापना की थी वजस बाद

म बरहम समाज क नाम स जाना गया था

bull दिदरनाथ िग र न ततवब विनी सभा की अधयकषता

की थी ज आधयासिक सतय की ख ज म सोलि

थी

bull इसका उददशय वहोद िमि क शद करन का और

एकशवरिाद (एक ईशवर म आथथा) का परचार करना

था

bull नय समाज की थथापना क आिार थ कारण

(तकि ) क द सतमभ तथा िद और उपवनिद

bull अभी हाल ही म सािारण बरहम समाज का कछ

काननी मदद ो क लकर पवशचम बोगाल सरकार क

साथ काननी वििाद चल रहा ह

Q75) उततर (c)

सपषटीकरण

bull भारत म वचशती वसलवसल की थथापना खवाजा

म इनददीन वचशती क दवारा की गयी थी

bull ि 1192 ईसवी क आसपास भारत आय थ

bull वचशतीय ो क बारहिी ो शताबदी क उततरािि म भारत

म आन िाल सफीय ो क समह ो म सबस

परभािशाली माना जाता ह

bull उनह ोन थथानीय िातािरण क साथ सफलतापििक

अनकलन वकया और उनह ोन भारतीय भसकत

परोपराओो क कई पहलओो क अपनाया

bull अजमर म सफी अपरकि खवाजा म इनददीन वचशती

की ऐवतहावसक दरगाह क एक नया रप दन की

तयारी की जा रही ह

bull इस 13िी ो शताबदी की दरगाह क ldquoसवचछ

आइकॉवनक थथल ोrdquo (Swacch Iconic Places) म

शावमल वकया गया ह ज परवतवषठत विरासत

आधयासिक और साोसकवतक थथान ो पर क वदरत

य जना ह

FC19H1003 41

ANSWERS amp EXPLANATION OF

NCERT History Class VI-X + Current Affairs

(FC19E1003)

Q1) Answer c

Explanation

Rigveda consists of more than a

thousand hymns dedicated to gods and

goddesses These hymns were

composed by sages and learnt by men

however a few were composed by

women like Apala Ghosa Lopamudra

Maitreyi and Gargi

Rigveda consists of many hymns in the

form of dialogues We get an example of

a dialogue between a sage named

Vishwamitra and two rivers (Beas and

Sutlej) that were worshipped as

goddesses This suggests that he

belonged to the Vedic period

Q2) Answer b

Explanation

Traces of ash have been found from

Kurnool Caves suggesting that people

were familiar with the use of fire

It is situated in Andhra Pradesh

Q3) Answer c

Explanation

Burzahom is a prehistoric site in

present day Kashmir where people built

pit houses which were dug into the

ground with steps leading into them

These may have provided shelter in cold

weather

Q4) Answer c

Explanation

Epigraphy is defined as the study of

inscriptions

Manuscriptology is the study of history

and literature through the use of hand

written documents

Palaeography refers to the study of

ancient writing systems and the

deciphering and dating of historical

manuscripts

Numismatics refers to the study of

coins

Q5) Answer a

Explanation

Charaka Samhita was written by

Charaka and is an important book on

Ayurveda and medicine

He was a practitioner of the traditional

system of Indian medicine known as

Ayurveda

Charaka is thought to have flourished

sometime between the 2nd century BCE

and the 2nd century CE

Q6) Answer b

Explanation

Bhaga refers to the tax on crops which

was fixed at 16th of the production

Kammakaras is the term used for the

landless agricultural labour class

Ashvamedha also known as horse

sacrifice is a ritual where a horse is let

loose to wander freely and it was

guarded by the rajarsquos men

Q7) Answer (d)

Explanation

In the Rigvedic period horses were

yoked to chariots that were used in

battles fought to capture land cattle

etc This suggests that the use of horse

chariots began much before the period

of Mahajanapadas

The battles were fought in the Rigvedic

period for cattlersquos lands water an even

to capture people Most men took part

in these wars however there was no

regular army but there were assemblies

where people met and discussed

matters of war Regular armies became

a feature in the Mjahajanapada period

including vast armies of foot soldiers

chariots and elephants

RAUSIAS-FC19E1003 42

Q8) Answer (a)

Explanation

Buddha belonged to the Sakya clan and

passed away at Kusinara

Buddha taught in Prakrit which was the

common language of people

Q9) Answer c

Explanation

There were six schools of philosophy in

ancient India These are known as

Vaishesika Nyaya Samkhya Yoga

Purva Mimansa and Vedanata or Uttara

Mimansa They were founded by sages

Kanada Gautama Kapila Patanjali

Jamini and Vyasa respectively

Q10) Answer b

Explanation

The teachings of Mahavira were

compiled at Valabhi in 6th century AD

Q11) Answer (c)

Explanation

Chanakya is traditionally identified as

Kautilya or Vishnugupta who authored

the ancient Indian political treatise the

Arthashastra

Q12) Answer d

The national emblem of India is an

adaptation of the Lion Capital atop the

Ashoka Pillar of Sarnath Uttar Pradesh

and is combined with the National

Motto Satyameva Jayate

The Rampurva Bull gets the name from

the site of its discovery Rampurva in

Bihar

It is noted for its delicately sculpted

model demonstrating superior

representation of soft flesh sensitive

nostrils alert ears and strong legs It is

a mixture of Indian and Persian

elements

Sankissa is situated in Uttar Pradesh

India

Q13) Ans(a)

Kunwar Singh was a notable leader during the Revolt of 1857 He belonged

to a royal house of Jagdispur Bihar

Q14) Answer b

Explanation

The term Vellalar was used for large

landowners

Q15) Answer c

Explanation

Arikamedu was a coastal settlement

where ships unloaded goods from

distant lands Finds here include a

massive brick warehouse pottery

including amphorae and Arretine ware

Roman lamps glassware and gems have

also been found at the site

Q16) Answer a

Explanation

Muvendar is a Tamil word mentioned in

Sangam poems meaning three chiefs

used for the heads of three ruling

families the Cholas Cheras and

Pandyas

Q17) Ans (c)

Several tribal or kin-based assemblies

such as the Sabha Vidatha and gana

are mentioned in the Rig-veda The

Sabha and the samiti mattered a great

deal in early Vedic times so much so

that the chiefs or the kings showed an

eagerness to win their support

Q18) Ans (a)

Jainism recognised the existence of the

gods but placed them lower than the

jina and did not condemn the varna

system as Buddhism did

Q19) Answer (d)

Explanation

Cholas and Pandyas had developed

powerful coastal cities The most

important city of Cholas was Puhar or

Kaveripattinam and Madurai was the

capital of Pandyas

Q20) Answer b

Explanation

Buddhacharita is the biography of

Buddha and was written by

RAUSIAS-FC19E1003 43

Ashvaghosha

Q21) Answer (a)

Explanation

Tamil poet Appar was a Shiva devotee

So he was a Nayanar saint

Q22) Answer d

Explanation

Samudragupta was a prominent Gupta

ruler whose coins depict him playing a

veena indicating his love for music We

get important historic information from

his Allahabad Prashasti which was

composed by his court poet Harisena

Q23) Answer (b)

Explanation

Vikrama Samvat was founded by

Chandragupta II in the 58 BC as a

mark of victory over the Shakas and

assumed the title of Vikramaditya

Banabhatta wrote Harshavardhanarsquos

biography the Harshacharita in

Sanskrit

Q24) Answer c

Explanation

Sandhi-vigrahika was the minister of

war and peace

Sarthavaha was the leader of the

merchant caravans

Q25) Answer a

Explanation

Xuan Zang (Hsuan-tsang) was a

Chinese traveller who came during the

reign of Harshavardhana

In the decade that began in 630 AD

Xuan Zang came to India through

Kashmir after visiting Central Asia Iran

and Afghanistan

He travelled from north to east and lived

in Bihar for a couple of years

At Nalanda University Xuan Zang

interacted with students and scholars

mastered local languages and

discovered Buddhist stupas

Q26) Answer c

Explanation

Pradakshina patha is a circular path

laid around a stupa in Buddhist

architecture While the rest are a part of

temple architecture

Q27) Answer d

Explanation

All the above-mentioned temples have

an elaborate use of bricks (baked

bricks) along with stone

Q28) Ans (c)

Muhammad Quli Qutab was the Sultan

of Golconda He was a contemporary of

Akbar was very fond of literature and

architecture

The Sultan was a great poet and he

wrote in Dakhini Urdu Persian and

Telgu and has left an extensive diwan or

collection

Recently the Archaeological Survey of

India (ASI) will be using Ground

Penetrating Radar (GPR) to map the

contours of the area around the Bagh-e-

Naya Qila excavated garden inside the

Golconda Fort in Telangana

Q29) Answer a

Explanation

Silappadikaram is a famous Tamil epic

which was written by Ilango around

1800 years ago It is a story of a

merchant named Kovalan who fell in

love with a courtesan named Madhavi

Manimekalai tells the story of the

daughter of Kovalan and Madhavi

Q30) Answer (a)

Explanation

Charaka is the author of Charaka

Samhita which is an important work of

Ayurveda and medicines

Brahmaguptarsquos fame rests mostly on his

Brahma-sphuta-siddhanta which was

an astronomical work It was translated

into Arabic in Baghdad and had a major

impact on Islamic mathematics and

astronomy

Late in his life Brahmagupta wrote

Khandakhadyaka which was an

RAUSIAS-FC19E1003 44

astronomical handbook that employed

Aryabhatarsquos system of starting each day

at midnight

Q31) Answer (c)

Explanation

Amir Khusrau was a famous sufi

musician poet and scholar In 1318 he

noted that there was different language

in every region of this land (Hindustan)

Lahori Kashmiri Dvarsamudri (in

Southern Karnataka) Telangana (in

Andhra Pradesh) Gujari (in Gujarat)

Marsquobari (in Tamil Nadu) Awadhi (in

eastern Uttar Pradesh) and Hindawai (in

the area around in Delhi) etc He went

to explain that Sanskrit did not belong

to any region and that only brahmans

knew it

Q32) Answer c

Explanation

Hiranyagarbha refers to the golden

womb When this ritual was performed

with the help of Brahmanas it was

thought to lead to the rebirth of the

sacrificer as a Khastriya

Q33) Answer d

Explanation

Kadamai refers to a tax on land

revenue

Gwalior Prashasti describes the exploits

of Nagabhata who was a Pratihara king

Q34) Answer b

Explanation

Rajatarangini is a Sanskrit text written

by Kalhana in the 12th century

It was historical chronicle of early India

It is justifiably considered to be the best

and most authentic work of its kind

It covers the entire span of history in

the Kashmir region from the earliest

times to the date of its composition

Q35) Answer c

Explanation

ldquoUrrdquo was the general assembly of the

village ldquoUrrdquo consisted of all the

taxpaying residents of an ordinary

village

Q36) Answer (a)

Explanation

Tarikh was a form of history writing in

the Delhi Sultanate The authors of

tawarikhs were learned men which

included secretaries administrators etc

Q37 Answer (a)

Explanation

Alauddin chose to pay his soldiers salaries in cash rather than iqtas The soldiers would buy their supplies from merchants in Delhi and it was thus feared that merchants would raise their prices To stop this Alauddin controlled the prices of goods in Delhi Prices were carefully surveyed by officers and merchants who did not sell at the prescribed rates were punished

Q38) Answer (d)

Explanation

Delhi first became the capital of a

kingdom under the Tomara Rajputs

who were defeated in the middle of the

twelfth century by the Chauhans (also

referred to as Chahamanas) of Ajmer

It was under the Tomaras and

Chauhans that Delhi became an

important commercial centre Many rich

Jaina merchants lived in the city and

constructed several temples Coins

minted here called dehliwal had a wide

circulation

Q39) Answer (c)

Explanation

Moth ki Masjid was built in the reign of

Sikandar Lodi by his minister

Begumpuri mosque built in the reign of

Muhammad Tughluq was the main

mosque of Jahanpanah the ldquoSanctuary

of the Worldrdquo and his new capital in

Delhi

Quwwat al ndash Islam mosque was

enlarged by Iltutmish and Alauddin

Khalji The minar was built by three

Sultansndash Qutbuddin Aybak Iltutmish

and Firuz Shah Tughluq

RAUSIAS-FC19E1003 45

Q40) Answer (c)

Explanation

Under the Mughals mansabdar was

referred to an individual who held a

mansab ie rank and he received his

salary as revenue assignments called

jagirs

Q41) Ans (b)

The Quit India Movement was a

spontaneous revolt of people against

British rule

The All India Congress Committee met

at Bombay on 8 August 1942 It passed

the famous resolution Quit India and

proposed the starting of a non-violent

mass struggle under Gandhis

leadership to achieve this aim But on

the very next day Gandhi and other

eminent leaders of the Congress were

arrested The Congress was once again

declared illegal

Q42) Ans (c)

The Simon Commission refers to a

group of seven MPs from the United

Kingdom constituted to suggest

constitutional reforms for British India

The Commission consisted of only

British members headed by one of the

senior British politicians Sir John

Simon

So the people of India agitated against

the arrival of Simon Commission

Q43) Ans (a)

He was widely known for his

unfavourable opinion of the economic

consequences of the British rule in

India

In his many writings and speeches and

especially in Poverty and Un-British

Rule in India Naoroji argued that India

was too highly taxed and that its wealth

was being drained away to England

He did not interpret the ancient Indian

texts and restored the self-confidence of

Indians And also he did not stress the

need for eradication of all the social

evils before anything else

Q44) Ans (c)

In August 1932 Prime Minister

MacDonald announced his Communal

Award Great Britainrsquos unilateral

attempt to resolve the various conflicts

among Indiarsquos many communal

interests

The award which was later

incorporated into the act of 1935

expanded the separate-electorate

formula reserved for Muslims to other

minorities including Sikhs Indian

Christians Anglo-Indians Europeans

distinct regional groups Gandhi

undertook a ldquofast unto deathrdquo against

that offer which he viewed as a

nefarious British plot to divide the

Indian society

Q45) Ans (b)

In British India apart from existing

imports and exports there was also a

particular amount of money which

colonial India contributed towards

administration maintenance of the

army war expenses pensions to retired

officers and other expenses accrued by

Britain towards maintenance of her

colony These were known as Home

charges and were paid for almost

entirely by India

The Home charges was made of

following components-

- Interest payable on Indian debt

- Dividend to shareholders of East

India Company

- Funds used to support the India

Office in London

- Funds used to pay salaries and

pensions of British personnel

engaged in India

- Interest on the railways

- Civil and military charges

- Store purchases in England

Q46) Ans (b)

The Lahore session of the Indian

National Congress was held in 1929

under the Presidentship of Jawaharlal

Nehru

The Lahore session of the Indian

National Congress witnessed significant

RAUSIAS-FC19E1003 46

developments in the Indian national

movement

- First the election of Jawaharlal

Nehru to the post of Presidentship of

the Congress was a clear indication

of the growing strength of the

Leftists in the Congress

- Secondly it was in this session that

the Congress for the first time raised

the demand for complete

independence Such demand was

not raised from the Congress

platform earlier

Q47) Ans (b)

It did not provide for separate

electorates for any community or

weightage for minorities However it did

allow for the reservation of minority

seats in provinces having minorities of

at least ten per cent but this was to be

in strict proportion to the size of the

community

There was no provision for complete

Independence for India

Q48) Ans (c)

The religion of early Vedic Aryans was

primarily of worship of nature and

Yajnas

The early Aryan religion was kind of

nature worship Actually the forces

around them which they could not

control or understand were invested

with divinity and were personified as

male or female gods And they

performed some Yajnas also

Q49) Ans (b)

The roads and river-routes were not

immune from robbery It is notable that

Yuan Chwang (Hiuen Tsang) was

robbed of his belongings during

Harshvardanarsquos period

Q50) Ans (c)

Q51) Ans (b)

Purandara Dasa was a saint and great

devotee of Lord Krishna

There is much speculation about where

Purandara Dasa regarded as the

Pitamaha of Carnatic music was born

Recently an expert committee

constituted by the Kannada University

Hampi has come to the conclusion that

Kshemapura Shivamogga district

Karnataka is the birth place of

Purandara Dasa

Q52) Ans (c)

Sri Tyagaraja Sri Shyama Shastry and Sri Muthuswami Dikshitar are considered the trinity of Carnatic music and with them came the golden age in Carnatic music in the 18th-19th

century

Q53) Ans d)

Recently a rare sarcophagus (stone

coffin) which is 2000 years old from the

Iron AgendashMegalithic era was discovered

from a rock-cut cave at Viyur village of

Kollam near Koyilandy in Kozhikode

district Kerala

The coffin containing bone fragments

was found during an excavation ldquoSo

far such a rare finding has been

discovered only from two sites

in Kerala Both these sarcophagi were

recovered from Megalithic sites at

Chevayur and Atholi also in Kozhikode

district

Q54) Ans a)

The megalithic culture in South India was a full-fledged Iron Age culture

Q55) Ans d)

The Cholas Pandyas and Keralaputras

(Cheras) mentioned in Ashokan

inscriptions were probably in the late

megalithic phase of material culture

Q56) Ans d)

Q57) Ans (b)

Raj Kumar Shukla followed Gandhiji all

over the country to persuade him to

come to Champaran to investigate the

problem associated with tinkathia

system

RAUSIAS-FC19E1003 47

Brij Kishore Rajendra Prasad Mahadev

Desai and Narhari Parikh accompanied

Gandhi ji during the Champaran

Satyagraha

Q58) Ans (b)

The Satvahanas started the practice of granting tax-free villages to brahmanas and Buddhist monks

Q59) Ans c)

The objectives of the Programme are

listed as under

- Developing basic tourism

infrastructure

- Promoting cultural and heritage

value of the country to generate

livelihoods in the identified regions

- Enhancing the tourist attractiveness

in a sustainable manner by

developing world-class

infrastructure at the heritage

monument sites

- Creating employment through active

involvement of local communities

- Harnessing tourism potential for its

effects on employment generation

and economic development

- Developing sustainable tourism

infrastructure and ensuring proper

Operations and maintenance

therein

Q60) Ans (b)

The Tribal Cooperative Marketing

Development Federation of India

(TRIFED) came into existence in 1987

It is a national-level apex organization

functioning under the administrative

control of Ministry of Tribal Affairs

Govt of India

TRIFED has its registered and Head

Office located in New Delhi

Q61) Ans (c)

Premchandrsquos novels include

Premashram Rangabhumi Ghaban

Karmabhumi and Godan

Gora is a novel written by Rabindranath

Tagore

138th birth anniversary of Munshi

Premchand was celebrated across the

country

Q62) Ans (b)

Giddha is a traditional pastoral dance

performed by the women of the Punjab

India and Pakistan at festival times

and at the sowing and reaping of the

harvest

By this dance the Punjabi women

reveal their joy expel their suppressed

feelings in a male dominated society

through the performance of Giddha

Since this dance has nothing to do with

men only women can participate in it

During the Teej celebrations Giddha

dance is celebrated in Punjab every

year Teej is a generic name for a

number of festivals that are celebrated

by women in some parts of India

Q63) Ans (a)

Dara Shukoh wrote the remarkable

work called ldquoMajma-ul-Bahrainrdquo or the

ldquoThe confluence of two seasrdquo

The Vice President of India Shri M

Venkaiah Naidu has said that Prince

Dara Shukohrsquos writings can come as a

refreshing source for infusing peace and

harmony He was addressing the

gathering after visiting the exhibition

that showcases the forgotten Prince of

yesteryears Dara Shukoh organized by

Mr Francois Gautier at Indira Gandhi

National Centre for the Arts in New

Delhi

Q64) Ans (c)

The statue Gommateshwara is

dedicated to the Jain God Bahubali

It is a monolithic statue

President Ram Nath Kovind

inaugurated the grand anointing

ceremony mdash Mahamastakabhisheka mdash

held once in 12 years at

Shravanabelagola (Karnataka)

Q65) Ans (c)

Prachi Valley had come up around the

Prachi river Prachi Valley gradually

disappeared

RAUSIAS-FC19E1003 48

The Prachi river originates from

Bhubaneswar

It is a tributary of the Mahanadi and

flows through the districts of Puri

Khurda Cuttack and Jagatsinghpur

and the entire region of the river is

termed as the Prachi Valley

It falls into the Bay of Bengal

Archaeological evidence shows that the

Prachi Valley Civilisation predates both

Harappa and Mohenjo-Daro

The Prachi river originates from

Bhubaneswar

Q66) Ans (d)

These monuments are located in

Chhatarpur district Madhya Pradesh

within Vindhya mountain range

Q67) Ans (a)

The book lsquoThoughts on Pakistanrsquo was

written by Dr BR Ambedkar

On the occasion of the birth anniversary

of Dr BR Ambedkar the president of

India pays homage to this icon of India

In 1924 he founded the Depressed

Classes Institute (Bahishkrit Hitkarini

Sabha) and in 1927 the Samaj Samata

Sangh

Another area of attention for Ambedkar

was education For its spread among

the low classes he set up a network of

colleges by the name of Peoples

Education Society and founded hostels

Q68) Ans(b)

Mehrgarh is a famous Neolithic

settlement in the Indian subcontinent

which is situated in Baluchistan

province Pakistan

A pre-historic rock art site is discovered

in the vast expanse of limestone blocks

on the eastern banks of Naguleru river

near Dachepalli (Andhra Pradesh) It

has thrown light on the Neolithic

civilisation that flourished in Guntur

(Andhra Pradesh) during 1500-2000

BC

Q69) Ans (c)

The 12th and the 13th centuries saw

the emergence of the Kakatiyas They

were at first the feudatories of the

Western Chalukyas of Kalyana Initially

they ruled over a small territory near

Warangal (Telangana)

They introduced Nayakships which was

later adopted and developed by the

Rayas of Vijayanagara

Q70) Ans (a)

The fast had effect of putting pressure

on mill owners who finally agreed to

give the workers a 35 per cent increase

in wages

Google celebrated with a doodle the

132nd birth anniversary of Anasuya

Sarabhai who played a pioneering role

in Indiarsquos labour movement

Q71) Ans (d)

The UNESCOrsquos list of the representative

list of the intangible cultural heritage of

humanity from India are

- Koodiyattam Sanskrit Theatre of

Kerala

- Mudiyettu ritual theatre and dance

drama of Kerala

- Tradition of Vedic Chanting

- Kalbelia folk songs and dances of

Rajasthan

- Ramlila Traditional Performance of

the Ramayana

- Sankirtana ritual singing

drumming and dancing of Manipur

- Ramman religious festival and

ritual theatre of the Garhwal

Himalayas India

- Traditional brass and copper craft of

utensil making among the Thatheras

of Jandiala Guru Punjab India

- Chhau dance classical Indian dance

originated in the eastern Indian

states

- Buddhist chanting of Ladakh

recitation of sacred Buddhist texts

in the trans-Himalayan Ladakh

region Jammu and Kashmir India

- Yoga

- Nouroz

- Kumbh Mela

RAUSIAS-FC19E1003 49

Q72) Ans(b)

The President of India Shri Ram Nath Kovind inaugurated the Hornbill Festival and State Formation Day celebrations of Nagaland in Kisama

The festival is named after the Indian hornbill the large and colourful forest bird which is displayed in the folklore of most of the states tribes

The major recognized tribes of Nagaland are Angami Ao Chakhesang Chang

Kuki Rengma and Zeling etc

Onge Jarawa and Sentinelese are the

tribes of Andman amp Nicobar Islands

Q73) Ans (c)

The Rashtrakutas rule in the Deccan lasted for almost two hundred years till the end of the tenth century The Rashtrakutas rulers were tolerant in their religious views and patronized not only Shaivism and Vaishnavism but

Jainism as well

The famous rock-cut temple of Shiva at Ellora was built by one of the Rashtrakutas kings Krishna I in the ninth century His successor Amoghavarsha was a Jain but he also

patronized other faiths

The Rashtrakutas allowed Muslims traders to settle and permitted Islam to

be preached in their dominions

Recently increasing defacement at the prehistoric rock paintings of Pandavulagutta Telangana has created a cause for grave concern It can spoil

the prehistoric rock

Pandavulagutta is home to

- Painted rock shelters dating to

10000 BC-8000 BC

- An 8th century inscription of the

Rashtrakuta period and

- Painted frescoes from the 12th century Kakatiya empire

Q74) Ans (b)

In 1828 Raja Ram Mohan Roy founded a new religious society the Brahma Sabha later known as the Brahmo

Samaj

Debendranath Tagore headed the Tattvabodhini Sabha which was

engaged in search of spiritual truth

Its purpose was to purify Hinduism and to preach monotheism or belief in one God

The new society was to be based on the twin pillars of reason and the Vedas and

Upanishads

Recently Sadharan Brahmo Samaj (SBS) has entered into a legal battle with the West Bengal government due

to some legal issue

Q75) Ans (c)

The Chishti order was established in India by Khwaja Moinuddin Chishti who came to India around 1192 The Chishtirsquos are considered to be the most influential of the groups of Sufis who migrated to India in the late twelfth century They adapted successfully to the local environment and adopted several features of Indian devotional

traditions

The historical dargah of Sufi mystic Khwaja Moinuddin Chishti in Ajmer is all set to get a facelift This 13 th century dargah has been included among the Swachh Iconic Places a clean-up initiative focused on iconic

heritage spiritual and cultural places

Page 10: GENERAL STUDIES (PAPER I) · Test is part of Rau’s IAS Test series for Preliminary Exam 2019 FOUNDATION + CURRENT AFFAIRS GENERAL STUDIES (PAPER –I) FOUNDATION TEST –III TOPIC:

RAUSIAS-FC19E1003 10

Q26) िासतकिा स सोबोनरत निमननिखित ततव ो म स कौि-स

ततव किि नहोद मखनदर ो की िासतकिा क भाग ह

1 नशिर

2 मणडप

3 परदनकषरा पथ

4 गभणगह

िीच नदए गए कट का परय ग कर सही उततर चनिएः

(a) किि 1 3 और 4

(b) किि 2 3 और 4

(c) किि 1 2 और 4

(d) 1 2 3 और 4

Q27) निमननिखित मोनदर ो म स कौि-स मोनदर ईोट ो स बि ह

1 दिगढ़ मोनदर

2 भीतरगाि मोनदर

3 िकषमर मोनदर नसरपर

4 बहदशवर मोनदर

िीच नदए गए कट का परय ग कर सही उततर चनिएः

(a) किि 1 2 और 3

(b) किि 2 3 और 4

(c) किि 1 3 और 4

(d) 1 2 3 और 4

Q28) निमननिखित कथि ो म स कौि-सास सही हह

1 सलताि महममद किी कतब शाह अकबर का

समकािीि था

2 िासतकिा क कषतर म महममद किी कतब शाह

ि कई ईमारत ो का निमाणर करिाया था नजिम

स चार मीिार सिाणनरक परनसदध ह

िीच नदए गए कट का परय ग कर सही उततर चनिएः

(a) किि 1

(b) किि 2

(c) 1 और 2 द ि ो

(d) ि त 1 ि ही 2

Q29) निमननिखित यग ो पर निचार कीनजएः

1 मनिमकिाई सततिार

2 अनभजञाि शाको तिम कानिदास

3 नसिपपानदकारम क ििि

उपयणकत यग ो म स कौि-सास सही समनित हह

(a) किि 1 और 2

(b) किि 2

(c) किि 1 और 3

(d) 1 2 और 3

Q30) निमननिखित कथि ो म स कौि-सास सही हह

1 ldquoसशरत सोनहताrdquo नचनकरता पर एक महतवपरण

रचिा ह

2 बरहमगपत और चरक महतवपरण गनरतजञ थ

िीच नदए गए कट का परय ग कर सही उततर चनिए

(a) किि 1

(b) किि 2

(c) 1 और 2 द ि ो

(d) ि त 1 ि ही 2

Q31) अमीर िसर क सनदभण म निमननिखित कथि ो म स

कौि-सास सही हह

1 अमीर िसर ि अपिी रचिाओो म नििा ह

नक सोसकत नकसी भी कषतर स सोबोनरत िही ो थी

और किि बराहमर ही इस भािा का जञाि रित

2 उन ोि नहोदिी और अिरी क अखसततव का

उललि नकया था

िीच नदए गए कट का परय ग कर सही उततर चनिए

(a) किि 1

(b) किि 2

(c) 1 और 2 द ि ो

(d) ि त 1 ि ही 2

RAUSIAS-FC19E1003 11

Q26) Which of the following architectural

elements were only part of Hindu

temple architecture

1 Shikhara

2 Mandapa

3 Pradakshina patha

4 Garbhagriha

Select the correct answer using the code

given below

(a) 1 3 and 4 only

(b) 2 3 and 4 only

(c) 1 2 and 4 only

(d) 1 2 3 and 4

Q27) Which of the following temples isare

made of bricks

1 Deogarh Temple

2 Bhitargaon Temple

3 Lakshmana temple Sirpur

4 Brihadeshvara Temple

Select the correct answer using the code

given below

(a) 1 2 and 3 only

(b) 2 3 and 4 only

(c) 1 3 and 4 only

(d) 1 2 3 and 4

Q28) Which of the following statements

isare correct

1 Sultan Muhammad Quli Qutab

Shah was a contemporary of

Akbar

2 In the field of architecture

Muhammad Quli Qutab Shah

constructed many buildings the

most famous of which is the Char

Minar

Select the correct answer using the code

given below

(a) 1 only

(b) 2 only

(c) Both 1 and 2

(d) Neither 1 nor 2

Q29) Consider the following pairs

1 Manimekalai Sattanar

2 Abhijnana Shakuntalam Kalidasa

3 Silappadikaram Kovalan

Which of the pairs given above isare

correct

(a) 1 and 2 only

(b) 2 only

(c) 1 and 3 only

(d) 1 2 and 3

Q30) Which of the following statements

isare correct

1 Sushruta Samhita is an important

work on medicine

2 Brahmagupta and Charaka were

important mathematicians

Select the correct answer using the code

given below

(a) 1 only

(b) 2 only

(c) Both 1 and 2

(d) Neither 1 nor 2

Q31) Which of the following statements

isare correct about Amir Khusrau

1 Amir Khusrau records in his works

that Sanskrit did not belong to any

region and only the Brahmans

knew it

2 He recorded the existence of

Hindawi and Awadhi

Select the correct answer using the code

given below

(a) 1 only

(b) 2 only

(c) Both 1 and 2

(d) Neither 1 nor 2

RAUSIAS-FC19E1003 12

Q32) निमननिखित कथि ो पर निचार कीनजए

1 नहरणय-गभण अिषठाि क बार म ऐसा स चा जाता

था नक बनि दि िाि का एक कषनतरय क रप म

पिजणनम ह गा

2 मयरशमणि कदोब िोश का सोसथापक था

उपयणकत कथि ो म स कौि-सास सही हह

(a) किि 1

(b) किि 2

(c) 1 और 2 द ि ो

(d) ि त 1 ि ही 2

Q33) निमननिखित कथि ो म स कौि-सास सही हह

1 कदमई बगार (बिपिणक शरम) क रप म

निया जाि िािा कर था

2 गवानियर परशखसत म िागभट (ज एक चोदि

राजा था) क दवारा नकय गए श िर का िरणि

नकया गया ह

िीच नदए गए कट का परय ग कर सही उततर चनिए

(a) किि 1

(b) किि 2

(c) 1 और 2 द ि ो

(d) ि त 1 ि ही 2

Q34) निमननिखित कथि ो म स कौि-सास सही हह

1 राजतरो नगिी 11िी ो शताबदी म कलहि क दवारा

रनचत एक सोसकत पसतक (टकसट) ह

2 कननौज क निए नतरपकषीय सोघिण म पाि राजिोश

शानमि था

िीच नदए गए कट का परय ग कर सही उततर चनिए

(a) किि 1

(b) किि 2

(c) 1 और 2 द ि ो

(d) ि त 1 ि ही 2

Q35) निमननिखित यग ो पर निचार कीनजए

1 बरहदशवर मोनदर राजराजा च ि

2 उर मापि की इकाई

3 दिदाि मोनदर ो क भनम अिदाि

उपयणकत यग ो म स कौि-स सही समनित ह

(a) किि 1 और 2

(b) किि 2 और 3

(c) किि 1 और 3

(d) 1 2 और 3

Q36) निमननिखित कथि ो म स कौि-सास सही हह

1 नदलली क सलताि ो क अरीि परशासि की भािा

फारसी थी

2 नदलली सलतित म ldquoतारीितािरीिrdquo कनिता

का एक रप था

िीच नदए गए कट का परय ग कर सही उततर चनिए

(a) किि 1

(b) किि 2

(c) 1 और 2 द ि ो

(d) ि त 1 ि ही 2

Q37) निमननिखित कथि ो म स कौि-सास सही हह

1 अिाउददीि खििजी ि अपि सनिक ो क निए

नसरी िाम का एक िया दगणरकषक शहर

बिािाया था

2 िह अपि सनिक ो क िति का भगताि इकता

क रप म करता था

िीच नदए गए कट का परय ग कर सही उततर चनिए

(a) किि 1

(b) किि 2

(c) 1 और 2 द ि ो

(d) ि त 1 ि ही 2

RAUSIAS-FC19E1003 13

Q32) Consider the following statements

1 Hiranya-garbha ritual was thought

to lead to the rebirth of the

sacrificer as a Kshatriya

2 Mayurasharman was the founder

of the Kadamba dynasty

Which of the statements given above

isare correct

(a) 1 only

(b) 2 only

(c) Both 1 and 2

(d) Neither 1 nor 2

Q33) Which of the following statements

isare correct

1 Kadamai was tax taken in form of

forced labour

2 Gwalior Prashasti describes the

exploits of Nagabhata who was a

Chandella king

Select the correct answer using the code

given below

(a) 1 only

(b) 2 only

(c) Both 1 and 2

(d) Neither 1 nor 2

Q34) Which of the following statements

isare correct

1 Rajatarangini is a Sanskrit text

written by Kalhana in the 11th

century

2 Pala dynasty was included in the

tripartite struggle for Kannauj

Select the correct answer using the code

given below

(a) 1 only

(b) 2 only

(c) Both 1 and 2

(d) Neither 1 nor 2

Q35) Consider the following pairs

1 Brihadeshvara temple Rajaraja

Chola

2 ldquoUrrdquo Unit of measurement

3 Devadana Land grants made to

temples

Which of the pairs given above isare

correct

(a) 1 and 2 only

(b) 2 and 3 only

(c) 1 and 3 only

(d) 1 2 and 3

Q36) Which of the following statements

isare correct

1 The language of administration

under the Delhi Sultans was

Persian

2 Tarikhtawarikh was a form of

poetry in the Delhi Sultanate

Select the correct answer using the code

given below

(a) 1 only

(b) 2 only

(c) Both 1 and 2

(d) Neither 1 nor 2

Q37) Which of the following statements

isare correct

1 Alauddin Khilji constructed a new

garrison town named Siri for his

soldiers

2 He paid his soldiers their salaries

in the form of Iqta

Select the correct answer using the code

given below

(a) 1 only

(b) 2 only

(c) Both 1 and 2

(d) Neither 1 nor 2

RAUSIAS-FC19E1003 14

Q38) निमननिखित कथि ो म स कौि-सास सही हह

1 नदलली कतबददीि एबक क अरीि पहिी बार

नकसी सामराजय की राजरािी बिी थी

2 दहिीिाि नसक ो का मदरर मग़ि ो क दवारा

नकया गया था

िीच नदए गए कट का परय ग कर सही उततर चनिए

(a) किि 1

(b) किि 2

(c) 1 और 2 द ि ो

(d) ि त 1 ि ही 2

Q39) निमननिखित यग ो पर निचार कीनजए

1 म ठ की मखिद नसको दर ि दी

2 बगमपरी मखिद नफर ज शाह तगिक

3 कववत- अि - इसलाम कतबददीि ऐबक

उपयणकत यग ो म स कौि-स सही समनित ह

(a) किि 1 और 2

(b) किि 2 और 3

(c) किि 1 और 3

(d) 1 2 और 3

Q40) निमननिखित कथि ो म स कौि-सास सही हह

1 मिसबदार ो क अपिा िति राजसव कायो

नजन जागीर कहत थ क रप म परापत ह ता

था

2 मिसबदार क ज सनय उततरदानयतव सौोप जात

थ उसक अनतगणत उस एक निराणररत सखया म

सिार अथिा घड़सिार ो का रि-रिाि करिा

पड़ता था

िीच नदए गए कट का परय ग कर सही उततर चनिए

(a) किि 1

(b) किि 2

(c) 1 और 2 द ि ो

(d) ि त 1 ि ही 2

Q41) ldquo1942 क भारत छ ड़ आोद ििrdquo क बार म

निमननिखित अिि कि ो म स कौि-सा सतय िही ो ह

(a) यह एक अनहोसक आोद िि था

(b) इसका िततव महातमा गाोरी क दवारा नकया गया

था

(c) यह एक सवाभानिक आोद िि था

(d) इसि सामानयतया शरनमक िगण क आकनिणत

िही ो नकया था

Q42) भारत क ि ग ो ि ldquoसाइमि कमीशिrdquo क आगमि क

निरदध आोद िि नकया था कय ोनक

(a) भारतीय कभी भी 1919 क अनरनियम (The

Act of 1919) क काम की समीकषा िही ो करिा

चाहत थ

(b) साइमि कमीशि ि पराोत ो म दवर (द हर) शासि

क समापत करि की नसफाररश की थी

(c) साइमि कमीशि म क ई भारतीय सदसय िही ो

था

(d) साइमि कमीशि ि दश क निभाजि का

सझाि नदया था

Q43) निमननिखित कथि ो पर निचार कीनजए

भारतीय राषटर ीय आोद िि म दादाभाई िौर जी क दवारा

नकया गया सबस परभािी य गदाि यह था नक उन ोि

1 अोगरज ो क दवारा भारत क आनथणक श िर का

ििासा नकया था

2 पराचीि भारतीय गरोथ ो की वयाखया की थी और

भारतीय ो क आतमनिशवास क पिःसथानपत नकया

था

3 अनय नकसी भी बात स पहि सभी सामानजक

बराइय ो क उनमिि की आिशयकता पर बि

नदया था

उपयणकत कथि ो म स कौि-सास सही हह

(a) किि 1

(b) किि 2 और 3

(c) किि 1 और 3

(d) 1 2 और 3

RAUSIAS-FC19E1003 15

Q38) Which of the following statements

isare correct

1 Delhi first became the capital of a

kingdom under Qutubuddin

Aibak

2 Dehliwal coins were minted by the

Mughals

Select the correct answer using the code

given below

(a) 1 only

(b) 2 only

(c) Both 1 and 2

(d) Neither 1 nor 2

Q39) Consider the following pairs

1 Moth ki Masjid- Sikander Lodi

2 Begumpuri mosque- Firuz Shah

Tughluq

3 Quwwat al ndash Islam- Qutubuddin

Aibak

Which of the above pairs isare correct

(a) 1 and 2 only

(b) 2 and 3 only

(c) 1 and 3 only

(d) 1 2 and 3

Q40) Which of the following statements

isare correct

1 Mansabdars received their salaries

as revenue assignments called

jagirs

2 The mansabdarrsquos military

responsibilities required him to

maintain a specified number of

sawar or cavalrymen

Select the correct answer using the code

given below

(a) 1 only

(b) 2 only

(c) Both 1 and 2

(d) Neither 1 nor 2

Q41) Which one of the following observations

is not true about the Quit India

Movement of 1942

(a) It was a non-violent movement

(b) It was led by Mahatma Gandhi

(c) It was a spontaneous movement

(d) It did not attract the labour class

in general

Q42) The people of India agitated against the

arrival of the Simon Commission

because

(a) Indians never wanted the review of

the working of the Act of 1919

(b) Simon Commission recommended

the abolition of dyarchy in the

Provinces

(c) there was no Indian member in the

Simon Commission

(d) the Simon Commission suggested

the partition of the country

Q43) Consider the following statements

The most effective contribution made by

Dadabhai Naoroji to the cause of Indian

National Movement was that he-

1 exposed the economic exploitation

of India by the British

2 interpreted the ancient Indian

texts and restored the self-

confidence of Indians

3 stressed the need for eradication of

all the social evils before anything

else

Which of the statements given above

isare correct

(a) 1 only

(b) 2 and 3 only

(c) 1 and 3 only

(d) 1 2 and 3

RAUSIAS-FC19E1003 16

Q44) महातमा गाोरी ि 1932 म आमरर अिशि नकया था

कय ोनक

(a) ldquoग िमज सममििrdquo (The Round Table

Conference) भारतीय राजिीनतक

आकाोकषाओो क परा करि म असफि रहा था

(b) काोगरस और मखसलम िीग म मतभद थ

(c) रामस मकड िालड (Ramsay Macdonald)

ि ldquoसाोपरदानयक परसकारrdquo (The Communal

Award) की घ िरा की थी

(d) ldquoसनििय अिजञा आोद ििrdquo (The Civil

Disobedience Movement) असफि रहा

था

Q45) भारत म औपनििनशक शासि की अिनर क सोदभण म

भारत स रि क बनहगणमि का एक महतवपरण भाग गह

शलक (Home Charges) था निमननिखित म स

कौि-सास क ि गह शलक म सखममनित नकया गया

थानकय गए थ

1 िोदि म भारत कायाणिय क निए उपय ग नकय

जाि िािा क ि

2 भारत म नियकत नबरनटश कनमणय ो क िति और

पशि का भगताि करि क निए उपय ग नकय

जाि िािा क ि

3 अोगरज ो क दवारा भारत क बाहर यदध ो क निए

उपय ग नकय जाि िािा क ि

िीच नदए गए कट का परय ग कर सही उततर चनिए

(a) किि 1

(b) किि 1 और 2

(c) किि 2 और 3

(d) 1 2 और 3

Q46) सवतोतरता आोद िि क इनतहास म भारतीय राषटर ीय

काोगरस का 1929 का सतर महतवपरण ह कय ोनक इसम

(a) काोगरस क उददशय क रप म सथािीय सरकार

की पराखपत की घ िरा की गई थी

(b) परण सवराज की पराखपत क काोगरस क िकषय क

रप म अपिाया गया था

(c) असहय ग आोद िि शर नकया गया था

(d) िोदि म ldquoग ि मर सममििrdquo (The Round

Table Conference) म भाग िि का निरणय

निया गया था

Q47) भारतीय सवतोतरता सोगराम क सोदभण म िहर ररप टण

क दवारा निमननिखित म स नकसकी नसफाररश की गई

थीनकिकी नसफाररश की गई थी ो

1 भारत क निए परण सवतोतरता

2 अलपसोखयक ो क निए सीट ो क आरकषर क

निए सोयकत नििाणचक मोडि

3 सोनिराि म भारत क ि ग ो क निए मौनिक

अनरकार ो का परािराि

िीच नदए गए कट का परय ग कर सही उततर चनिए

(a) किि 1

(b) किि 2 और 3

(c) किि 1 और 3

(d) 1 2 और 3

Q48) आरो नभक िनदक आयो का रमण मखय रप स था

(a) भखकत

(b) मनतण पजा और यजञ

(c) परकनत की पजा और यजञ

(d) परकनत की पजा और भखकत

RAUSIAS-FC19E1003 17

Q44) Mahatma Gandhi undertook fast unto

death in 1932 mainly because

(a) The Round Table Conference failed

to satisfy Indian political

aspirations

(b) The Congress and Muslim League

had differences of opinion

(c) Ramsay Macdonald announced the

Communal Award

(d) The Civil Disobedience Movement

failed

Q45) With reference to the period of colonial

rule in India ldquoHome Chargesrdquo formed

an important part of drain of wealth

from India Which of the following funds

constituted ldquoHome Chargesrdquo

1 Funds used to support the India

Office in London

2 Funds used to pay salaries and

pensions of British personnel

engaged in India

3 Funds used for waging wars

outside India by the British

Select the correct answer using the code

given below

(a) 1 only

(b) 1 and 2 only

(c) 2 and 3 only

(d) 1 2 and 3

Q46) The 1929- Session of Indian National

Congress is of significance in the history

of the Freedom Movement because the-

(a) attainment of Self-Government

was declared as the objective of

the Congress

(b) attainment of Poorna Swaraj was

adopted as the goal of the

Congress

(c) Non-Cooperation Movement was

launched

(d) decision to participate in the

Round Table Conference in

London was taken

Q47) With reference to the period of Indian

freedom struggle which of the following

waswere recommended by the Nehru

report

1 Complete Independence for India

2 Joint electorates for reservation of

seats for minorities

3 Provision of fundamental rights for

the people of India in the

Constitution

Select the correct answer using the code

given below

(a) 1 only

(b) 2 and 3 only

(c) 1 and 3 only

(d) 1 2 and 3

Q48) The religion of the early Vedic Aryans was primarily of

(a) Bhakti

(b) image worship and Yajnas

(c) worship of nature and Yajnas

(d) worship of nature and Bhakti

RAUSIAS-FC19E1003 18

Q49) भारत की यातरा करि िाि चीिी यातरी यआि चिाोग

(हयएि साोग) ि समकािीि भारत की सामानय

खसथनतय ो और सोसकनत क दजण नकया था इस सोदभण म

निमननिखित कथि ो म स कौि-सास सही हह

1 सड़क और िदी-मागण (जि-मागण) डकती स

परण रप स सरनकषत थ

2 जहा तक अपरार ो क निए दणड की बात ह

उसक निए नकसी भी वयखकत की निदोिता

अथिा उसक अपरार क निराणररत करि क

निए अनि जि और निि परि क माधयम क

सारि थ

3 वयापाररय ो क घाट ो और परनतबोर सटशि ो पर

शलक ो का भगताि करिा पड़ता था

िीच नदए गए कट का परय ग कर सही उततर चनिए

(a) किि 1

(b) किि 2 और 3

(c) किि 1 और 3

(d) 1 2 और 3

Q50) नसोर घाटी सभयता क सोदभण म निमननिखित कथि ो पर

निचार कीनजए

1 यह मखय रप स एक रमणनिरपकष सभयता थी

तथा हािाोनक इसम रानमणक ततव मौजद था

िनकि िह परनतिश पर हािी िही ो था

2 इस काि क दौराि भारत म कपास का परय ग

कपड़ा बिाि क निए नकया जाता था

उपयणकत कथि ो म स कौि-सास सही हह

(a) किि 1

(b) किि 2

(c) 1 और 2 द ि ो

(d) ि त 1 ि ही 2

Q51) परोदर दास क सोदभण म निमननिखित कथि ो पर निचार

कीनजए

1 परोदर दास एक सोत और भगिाि नशि क एक

महाि भकत थ

2 ि एक सोगीतकार गायक और किाणटक सोगीत

क मखय सोसथापक-परसतािक ो म स एक थ

उपयणकत कथि ो म स कौि-सास सही हह

(a) किि 1

(b) किि 2

(c) 1 और 2 द ि ो

(d) ि त 1 ि ही 2

Q52) निमननिखित म स कौि-सास वयखकत किाणटक सोगीत

की नतरमनतण म शानमि हह

1 बािामरिी कषणा

2 शरी शयाम शासतरी

3 शरी मथसवामी दीनकषतर

िीच नदए गए कट का परय ग कर सही उततर चनिए

(a) किि 1

(b) किि 2

(c) किि 2 और 3

(d) 1 2 और 3

Q53) चियर (Chevayur) और अथ िी (Atholi) म खसथत

महापािार सथि निमननिखित म स नकस राजय म खसथत

(a) तनमििाड

(b) किाणटक

(c) पनिम बोगाि

(d) करि

RAUSIAS-FC19E1003 19

Q49) The Chinese traveller Yuan Chwang

(Hiuen Tsang) who visited India

recorded the general conditions and

culture of India at that time In this

context which of the following

statements isare correct

1 The roads and river-routes were

completely immune from robbery

2 As regards punishment for

offences ordeals by fire water and

poison were the instruments for

determining the innocence or guilt

of a person

3 The tradesmen had to pay duties

at ferries and barrier stations

Select the correct answer using the code

given below

(a) 1 only

(b) 2 and 3 only

(c) 1 and 3 only

(d) 1 2 and 3

Q50) Regarding the Indus Valley Civilization

consider the following statements

1 It was predominantly a secular

civilization and the religious

element though present did not

dominate the scene

2 During this period cotton was

used for manufacturing textiles in

India

Which of the statements given above

isare correct

(a) 1 only

(b) 2 only

(c) Both 1 and 2

(d) Neither 1 nor 2

Q51) Consider the following statements

regarding Purandara Dasa

1 Purandara Dasa was a saint and

great devotee of Lord Shiva

2 He was a composer singer and

one of the chief founding-

proponents of the Carnatic music

Which of the statements given above

isare correct

(a) 1 only

(b) 2 only

(c) Both 1 and 2

(d) Neither 1 nor 2

Q52) Which of the following persons isare

included in the trinity of Carnatic

music

1 Balamurali Krishna

2 Sri Shyama Shastry

3 Sri Muthuswami Dikshitar

Select the correct answer using the code

given below

(a) 1 only

(b) 2 only

(c) 2 and 3 only

(d) 1 2 and 3

Q53) Megalithic sites at Chevayur and Atholi

are located in which of the following

states

(a) Tamil Nadu

(b) Karnataka

(c) West Bengal

(d) Kerala

RAUSIAS-FC19E1003 20

Q54) निमननिखित कथि ो पर निचार कीनजए

1 महापािानरक ि ग कबर ो म िसतएो दफिात थ

2 दनकषर भारत म महापािार सोसकनत एक परण

निकनसत तामर यगीि सोसकनत थी

उपयणकत कथि ो म स कौि-सास सही हह

(a) किि 1

(b) किि 2

(c) 1 और 2 द ि ो

(d) ि त 1 ि ही 2

Q55) निमननिखित म स कौि-स सामराजयसामराजय ो का

अश क क अनभिि ो म उललि नकया गया ह

1 च ि

2 पाणडय

3 करिपतर (चर)

िीच नदए गए कट का परय ग कर सही उततर चनिए

(a) किि 1

(b) किि 1 और 2

(c) किि 3

(d) 1 2 और 3

Q56) भीमा-क रगाोि का यदध को पिी क सनिक ो और

बाजीराि नदवतीय क िततव म एक शखकतशािी पशिा

सिा (मराठ ो) क मधय िड़ा गया था यह यदध

निमननिखित म स नकसका नहससा था

(a) परथम आोगल-मराठा यदध का

(b) नदवतीय आोगल-मराठा यदध का

(c) ततीय आोगल-मसर यदध का

(d) ततीय आोगल-मराठा यदध का

Q57) निमननिखित कथि ो पर निचार कीनजए

1 महादि दसाई ि गाोरीजी क चोपारर आि तथा

नतिकनथया पररािी स जड़ी समसया की जाोच

क निए रारी करि क निए दश भर म उिका

अिसरर नकया था

2 िरहरी पाररि चोपारर सतयागरह क दौराि

गाोरीजी क साथ थ

उपयणकत कथि ो म स कौि-सास सही हह

(a) किि 1

(b) किि 2

(c) 1 और 2 द ि ो

(d) ि त 1 ि ही 2

Q58) निमननिखित कथि ो पर निचार कीनजए

1 िनद राज-िोश ि बराहमर ो और बौदध मठराररय ो

क कर-मकत गाि अिदाि म दि की परथा

आरि की थी

2 सतिाहि ो की आनरकाररक भािा पराकत थी

उपयणकत कथि ो म स कौि-सास सही हह

(a) किि 1

(b) किि 2

(c) 1 और 2 द ि ो

(d) ि त 1 ि ही 2

Q59) एक निरासत क अपिाइए (अडॉपट ए हररटज ndash

Adopt a Heritage) पररय जिा क उददशय ो क

सनदभण म निमननिखित कथि ो पर निचार कीनजए

1 यह पररय जिा र रगार उतपादि और आनथणक

निकास क निए पयणटि कषमता का उि पर

परभाि का उपय ग करगी

2 यह पररय जिा निरासत सथि ो पर निशव सतरीय

आराररक सोरचिा निकनसत करक एक सतत

तरीक स पयणटक आकिणर म िखदध करगी

उपयणकत कथि ो म स कौि-सास सही हह

(a) किि 1

(b) किि 2

(c) 1 और 2 द ि ो

(d) ि त 1 ि ही 2

RAUSIAS-FC19E1003 21

Q54) Consider the following statements

1 Megalithic people buried goods in

graves

2 The megalithic culture in South

India was a full-fledged Copper

Age culture

Which of the statements given above

isare correct

(a) 1 only

(b) 2 only

(c) Both 1 and 2

(d) Neither 1 nor 2

Q55) Which of the following kingdoms isare

mentioned in the Ashokan inscriptions

1 Cholas

2 Pandyas

3 Keralaputras (Cheras)

Select the correct answer using the code

given below

(a) 1 only

(b) 1 and 2 only

(c) 3 only

(d) 1 2 and 3

Q56) The Battle of Bhima-Koregaon was

fought between the soldiers of the

Company and the strong Peshwa army

(Marathas) under Bajirao II This war

was a part of the

(a) First Anglo-Maratha war

(b) Second Anglo-Maratha war

(c) Third Anglo- Mysore war

(d) Third Anglo-Maratha war

Q57) Consider the following statements

1 Mahadev Desai followed Gandhiji all over the country to persuade him to come to Champaran to investigate the problem associated

with tinkathia system

2 Narhari Parikh accompanied Gandhi ji during the Champaran

Satyagraha

Which of the statements given above isare correct

(a) 1 only

(b) 2 only

(c) Both 1 and 2

(d) Neither 1 nor 2

Q58) Consider the following statements

1 The Nanda Dynasty started the practice of granting tax-free villages to brahmanas and

Buddhist monks

2 The official language of the Satavahanas was Prakrit

Which of the statements given above

isare correct

(a) 1 only

(b) 2 only

(c) Both 1 and 2

(d) Neither 1 nor 2

Q59) Consider the following statements about the objectives of the lsquoadopt a heritagersquo

project

1 It will harness tourism potential for its effects on employment generation and economic

development

2 It will enhance the tourist attractiveness in a sustainable manner by developing world class infrastructure at heritage sites

Which of the statements given above

isare correct

(a) 1 only

(b) 2 only

(c) Both 1 and 2

(d) Neither 1 nor 2

RAUSIAS-FC19E1003 22

Q60) ldquoभारतीय जिजातीय सहकारी निपरि निकास सोघrdquo

(The Tribal Co-operative Marketing

Development Federation of India - TRIFED)

क सोदभण म निमननिखित कथि ो पर निचार कीनजए

1 यह एक राषटर ीय सतर का शीिण सोगठि ह ज

भारत सरकार क गह मोतरािय क परशासनिक

नियोतरर क अरीि काम कर रहा ह

2 इसका मखय उददशय दश म जिजातीय ि ग ो

का सामानजक-आनथणक निकास करिा ह

उपयणकत कथि ो म स कौि-सास सही हह

(a) किि 1

(b) किि 2

(c) 1 और 2 द ि ो

(d) ि त 1 ि ही 2

Q61) निमननिखित म स कौि-सास उपनयास परमचोद क

दवारा नििा गया हनिि गए ह

1 रोगभनम

2 ग दाि

3 ग रा

िीच नदए गए कट का परय ग कर सही उततर चनिए

(a) किि 1

(b) किि 2

(c) किि 1 और 2

(d) 1 2 और 3

Q62) नगदधा ितय क सोदभण म निमननिखित कथि ो पर निचार

कीनजए

1 नगदधा नबहार की मनहिाओो क दवारा तयौहार क

समय और फसि की बिाई तथा कटाई क

अिसर पर नकया जाि िािा एक पारोपररक

दहाती ितय ह

2 इस ितय क दवारा मनहिाऐो अपिी परसननता

परकट करती ह तथा नगदधा क परदशणि क

माधयम स परि िचणसव िाि समाज म

मनहिाओो की दबी हई भाििाओो क परकट

करती ह

उपयणकत कथि ो म स कौि-सास सही हह

(a) किि 1

(b) किि 2

(c) 1 और 2 द ि ो

(d) ि त 1 ि ही 2

Q63) निमननिखित कथि ो पर निचार कीनजए

1 मलला शाह बदखशी दारा नशक ह क

आधयाखतमक गर थ

2 औरोगरब ि मजम-उि-बहरीि या द समदर ो

का सोगम िामक उललििीय रचिा नििी थी

3 दारा नशक ह क अपि पिणज अकबर क गर ो

क उततरानरकारी क रप म दिा गया था

नजसम उसि रानमणक बहििाद और समनवयता

क बढ़ािा नदया था

उपयणकत कथि ो म स कौि-सास सही हह

(a) किि 1 और 3

(b) किि 2

(c) किि 1 और 2

(d) 1 2 और 3

RAUSIAS-FC19E1003 23

Q60) Consider the following statements about

the Tribal Cooperative Marketing

Development Federation of India

(TRIFED)

1 It is a national-level apex

organization functioning under the

administrative control of Ministry

of Home Affairs Government of

India

2 The main objective of TRIFED is

socio-economic development of

tribal people in the country

Which of the statements given above

isare correct

(a) 1 only

(b) 2 only

(c) Both 1 and 2

(d) Neither 1 nor 2

Q61) Which of the following novels isare

written by Premchand

1 Rangabhumi

2 Godan

3 Gora

Select the correct answer using the code

given below

(a) 1 only

(b) 2 only

(c) 1 and 2 only

(d) 1 2 and 3

Q62) Consider the following statements about

Giddha dance

1 Giddha is a traditional pastoral

dance performed by the women of

Bihar at festival times and at the

sowing and reaping of the harvest

2 By this dance the women reveal

their joy expel their suppressed

feelings in a male dominated

society through the performance of

Giddha

Which of the statements given above

isare correct

(a) 1 only

(b) 2 only

(c) Both 1 and 2

(d) Neither 1 nor 2

Q63) Consider the following statements

1 Mullah Shah Badakhshi was the

spiritual mentor of Dara Shukoh

2 Aurangzeb wrote the remarkable

work called ldquoMajma-ul-Bahrainrdquo or

the ldquoThe confluence of two seasrdquo

3 Dara Shukoh was seen as

inheriting the qualities of his

ancestor Akbar in that he

promoted religious pluralism and

syncretism

Which of the statements given above

isare correct

(a) 1 and 3 only

(b) 2 only

(c) 1 and 2 only

(d) 1 2 and 3

RAUSIAS-FC19E1003 24

Q64) निमननिखित कथि ो पर निचार कीनजए

1 ग मतशवर परनतमा निोधयनगरी पहाड़ी पर खसथत ह

2 शरिरबिग िा िह सथाि ह जहाो मौयण िोश क

सोसथापक चोदरगपत मौयण अपि नसोहासि क

तयागि क बाद जि तपसवी बि गए थ

उपयणकत कथि ो म स कौि-सास सही हह

(a) किि 1

(b) किि 2

(c) 1 और 2 द ि ो

(d) ि त 1 ि ही 2

Q65) निमननिखित कथि ो पर निचार कीनजए

1 पराताखतवक साकषय स पता चिता ह नक पराची

घाटी सभयता हड़पपा और म हिज दाड़ द ि ो

की पिणिती ह

2 पराची िदी भििशवर स निकिती ह

उपयणकत कथि ो म स कौि-सास सही हह

(a) किि 1

(b) किि 2

(c) 1 और 2 द ि ो

(d) ि त 1 ि ही 2

Q66) निमननिखित कथि ो म स कौि-सास सही हह

1 िजराह क समारक ो क समह का निमाणर

चोदि राजिोश क शासिकाि क दौराि हआ

था

2 य समारक हररिोदर पिणत शरोििा म खसथत ह

3 म रक क यातरी इबन बतता ि अपि सोसमरर ो

म िजराह क मोनदर ो की यातरा का उललि

नकया था तथा इन काजराण िाम स समब नरत

नकया था

िीच नदए गए कट का परय ग कर सही उततर चनिए

(a) किि 1

(b) किि 1 और 2

(c) किि 2 और 3

(d) किि 1 और 3

Q67) निमननिखित कथि ो म स कौि-सास सही हह

1 डॉ बी आर अमबडकर ि दी एनिनहिशि

ऑफ़ कासट (The Annihilation of Caste)

नििी थी नजसम उन ोि नहोद रमण म िोशािगत

पजारी की परथा क उनमिि की आिशयकता

पर बि नदया था

2 डॉ राजदर परसाद ि थॉटस ऑि पानकसताि

(Thoughts on Pakistan) िामक पसतक

नििी थी

िीच नदए गए कट का परय ग कर सही उततर चनिए

(a) किि 1

(b) किि 2

(c) 1 और 2 द ि ो

(d) ि त 1 ि ही 2

Q68) निमननिखित कथि ो म स कौि-सास सही हह

1 महरगढ़ भारतीय उपमहादवीप म एक परनसदध

ििपािार बसती ह ज नसोर पराोत पानकसताि म

खसथत ह

2 बरणह म म कतत ो क उिक सवामी क साथ कबर ो

म दफिाया जाता था

िीच नदए गए कट का परय ग कर सही उततर चनिए

(a) किि 1

(b) किि 2

(c) 1 और 2 द ि ो

(d) ि त 1 ि ही 2

Q69) निमननिखित कथि ो म स कौि-सास सही हह

1 काकानटय मोनदर अनरकतर नशि क समनपणत

2 हिमक ोडा म हजार-सतोभ िाि मोनदर (The

Thousand-Pillared Temple) का निमाणर

काकानटय समराट रदर ि करिाया था

िीच नदए गए कट का परय ग कर सही उततर चनिए

(a) किि 1

(b) किि 2

(c) 1 और 2 द ि ो

(d) ि त 1 ि ही 2

RAUSIAS-FC19E1003 25

Q64) Consider the following statements

1 Gommateshwara Statue is located

on the Vindyagiri Hill

2 Shravanabelagola is the place

where Chandragupta Maurya the

founder of the Mauryan dynasty

became a Jain ascetic after

relinquishing his throne

Which of the statements given above

isare correct

(a) 1 only

(b) 2 only

(c) Both 1 and 2

(d) Neither 1 nor 2

Q65) Consider the following statements

1 Archaeological evidence shows

that the Prachi Valley Civilisation

predates both Harappa and

Mohenjo-Daro

2 The Prachi river originates from

Bhubaneswar

Which of the statements given above

isare correct

(a) 1 only

(b) 2 only

(c) Both 1 and 2

(d) Neither 1 nor 2

Q66) Which of the following statements

isare correct

1 The Khajuraho group of

monuments was built during the

rule of the Chandela dynasty

2 These monuments are located in

Harischandra mountain range

3 Ibn Battuta the Moroccan

traveller in his memoirs mentioned

visiting Khajuraho temples and

called them Kajarra

Select the correct answer using the code

given below

(a) 1 only

(b) 1 and 2

(c) 2 and 3

(d) 1 and 3

Q67) Which of the following statements

isare correct

1 Dr BR Ambedkar wrote the

Annihilation of Caste emphasising

the need to do away with the

practice of hereditary priesthood in

Hinduism

2 The book lsquoThoughts on Pakistanrsquo

was written by Dr Rajendra

Prasad

Select the correct answer using the code

given below

(a) 1 only

(b) 2 only

(c) Both 1 and 2

(d) Neither 1 nor 2

Q68) Which of the following statements

isare correct

1 Mehrgarh is a famous Neolithic

settlement in the Indian

subcontinent which is situated in

Sindh province Pakistan

2 At Burzahom dogs were buried

with their masters in their graves

Select the correct answer using the code

given below

(a) 1 only

(b) 2 only

(c) Both 1 and 2

(d) Neither 1 nor 2

Q69) Which of the following statements

isare correct

1 The Kakatiya temples are

dedicated mostly to Siva

2 The Thousand-Pillared Temple at

Hanamkonda was built by the

Kakatiya king Rudra

Select the correct answer using the code

given below

(a) 1 only

(b) 2 only

(c) Both 1 and 2

(d) Neither 1 nor 2

RAUSIAS-FC19E1003 26

Q70) निमननिखित कथि ो म स कौि-सास सही हह

1 अहमदाबाद नमि हड़ताि क दौराि महातमा

गाोरी ि शरनमक ो क पकष क मजबत करि क

निए आमरर अिशि नकया था

2 अिशि स नमि मानिक ो पर दबाि पड़ा था ज

अोततः शरनमक ो क िति म 15 परनतशत की िखदध

करि क निए सहमत हए थ

िीच नदए गए कट का परय ग कर सही उततर चनिए

(a) किि 1

(b) किि 2

(c) 1 और 2 द ि ो

(d) ि त 1 ि ही 2

Q71) निमननिखित म स नकसक नकिक भारत स यिसक

की माििता की अमतण साोसकनतक निरासत की

परनतनिनर सची (The UNESCOrsquos List of the

Representative List of the Intangible

Cultural Heritage of Humanity) म शानमि

नकया गया ह

1 मनडयटट

2 सोकीतणि

3 को भ मिा

िीच नदए गए कट का परय ग कर सही उततर चनिए

(a) किि 1 और 2

(b) किि 2 और 3

(c) किि 3

(d) 1 2 और 3

Q72) निमननिखित जिजानतय ो म स कौि-सीसी ो

जिजानतजिजानतया िागािड स सोबोनरत हह

1 अोगामी

2 ककी

3 जारिा

िीच नदए गए कट का परय ग कर सही उततर चनिए

(a) किि 1

(b) किि 1 औऔ 2

(c) किि 2

(d) 1 2 और 3

Q73) निमननिखित कथि ो म स कौि-सास सही हह

1 राषटर कट सामराजय की सथापिा दोनतदगण ि की थी

नजसि मानयाित म अपिी राजरािी की

सथापिा की थी

2 राषटर कट समराट अम घििण एक ििक था और

उस कनिताओो पर पहिी कननड़ पसतक नििि

का शरय नदया जाता ह

िीच नदए गए कट का परय ग कर सही उततर चनिए

(a) किि 1

(b) किि 2

(c) 1 और 2 द ि ो

(d) ि त 1 ि ही 2

Q74) निमननिखित कथि ो म स कौि-सास सही हह

1 कशब चोदर सि ि ततवब नरिी सभा की

अधयकषता की थी ज आधयाखतमक सतय की

ि ज म सोिि थी

2 बरहम समाज ि मािि गररमा पर बि नदया

मनतणपजा का निर र नकया और सती परथा जसी

सामानजक बराइय ो की आि चिा की

िीच नदए गए कट का परय ग कर सही उततर चनिए

(a) किि 1

(b) किि 2

(c) 1 और 2 द ि ो

(d) ि त 1 ि ही 2

Q75) निमननिखित कथि ो म स कौि-सास सही हह

1 भारत म नचशती नसिनसिा खवाजा म इिददीि

नचशती क दवारा सथानपत नकया गया था

2 नचशती परोपरा की एक परमि निशिता

आतमसोयम थी नजसम साोसाररक म ह स दरी

बिाए रििा शानमि था

िीच नदए गए कट का परय ग कर सही उततर चनिए

(a) किि 1

(b) किि 2

(c) 1 और 2 द ि ो

(d) ि त 1 ि ही 2

RAUSIAS-FC19E1003 27

Q70) Which of the following statements

isare correct

1 During the Ahmedabad Mill Strike

Mahatma Gandhi undertook a fast

unto death to strengthen the

workersrsquo resolve

2 The fast had effect of putting

pressure on mill owners who

finally agreed to give the workers a

15 per cent increase in wages

Select the correct answer using the code

given below

(a) 1 only

(b) 2 only

(c) Both 1 and 2

(d) Neither 1 nor 2

Q71) Which of the following are included in

the UNESCOrsquos list of the representative

list of the intangible cultural heritage of

humanity from India

1 Mudiyettu

2 Sankirtana

3 Kumbh Mela

Select the correct answer using the code

given below

(a) 1 and 2 only

(b) 2 and 3 only

(c) 3 only

(d) 1 2 and 3

Q72) Which of the following tribes isare

related to Nagaland

1 Angami

2 Kuki

3 Jarawa

Select the correct answer using the code

given below

(a) 1 only

(b) 1 and 2 only

(c) 2 only

(d) 1 2 and 3

Q73) Which of the following statements

isare correct

1 Rashtrakuta kingdom was founded by Dantidurga who established his capital at Manyakhet

2 Amoghavarsha a Rashtrakuta king was an author and is credited with writing the first

Kannada book on poetics

Select the correct answer using the code given below

(a) 1 only

(b) 2 only

(c) Both 1 and 2

(d) Neither 1 nor 2

Q74) Which of the following statements isare correct

1 Keshab Chandra Sen headed the Tattvabodhini Sabha which was engaged in search of spiritual truth

2 The Brahmo Samaj laid emphasis on human dignity opposed idolatry and criticized such social

evils as the practice of Sati

Select the correct answer using the code given below

(a) 1 only

(b) 2 only

(c) Both 1 and 2

(d) Neither 1 nor 2

Q75) Which of the following statements isare correct

1 The Chishti order was established in India by Khwaja Moinuddin

Chishti

2 A major feature of the Chishti tradition was austerity including maintaining a distance from the

worldly power

Select the correct answer using the code

given below

(a) 1 only

(b) 2 only

(c) Both 1 and 2

(d) Neither 1 nor 2

T e s t i s p a r t o f R a u rsquo s I A S T e s t s e r i e s f o r P r e l i m i n a r y E x a m 2 0 1 9

FOUNDATION + CURRENT AFFAIRS

GENERAL STUDIES (PAPER ndashI)

FOUNDATION TEST ndashIII

SUBJECT NCERT History Class VI-X + Current Affairs

Time Allowed 1frac12 Hours Maximum Marks 150

I NSTRUCT IONS

1 IMMEDIATELY AFTER THE COMMENCEMENT OF THE EXAMINATION YOU SHOULD CHECK

THAT THIS TEST BOOKLET DOES NOT HAVE ANY UNPRINTED OR TORN or MISSING PAGES OR

ITEMS ETC IF SO GET IT REPLACED BY A COMPLETE TEST BOOKLET

2 This Test Booklet contains 75 items (questions) Each item is printed both in Hindi and English

Each item comprises four responses (answers) You will select the response which you want to mark

on the Answer Sheet In case you feel that there is more than one correct response mark the

response which you consider the best In any case choose ONLY ONE response for each item

3 You have to mark all your responses ONLY on the separate Answer Sheet (OMR sheet) provided

Read the directions in the Answer Sheet

4 All items carry equal marks

5 Before you proceed to mark in the Answer Sheet the response to various items in the Test booklet

you have to fill in some particulars in the Answer Sheet as per instructions contained therein

6 After you have completed filling in all your responses on the Answer Sheet and the examination has

concluded you should hand over to the Invigilator only the Answer Sheet You are permitted to

take away with you the Test Booklet

7 Penalty for wrong answers

THERE WILL BE PENALTY FOR WRONG ANSWERS MARKED BY A CANDIDATE IN THE

OBJECTIVE TYPE QUESTION PAPERS

(i) There are four alternatives for the answer to every question For each question for which a

wrong answer has been given by the candidate one-third of the marks assigned to that

question will be deducted as penalty

(ii) If a candidate gives more than one answer it will be treated as a wrong answer even if one of

the given answers happens to be correct and there will be same penalty as above to that

question

(iii) If a question is left blank ie no answer is given by the candidate there will be no penalty for

that question

T h i s t e s t i s p a r t o f R a u rsquo s I A S T e s t s e r i e s f o r P r e l i m i n a r y E x a m 2 0 1 9

Test Code

FC19E1003

FC19H1003 29

Answers and Explanations of

NCERT History Class VI-X + Current Affairs (FC19E1003)

Q1) उततर (c)

सपषटीकरण

- ऋगवद म दविय ो और दिताओो क समवपित एक

हजार स अविक सत तर (शल क) ह

- य शल क ऋविय ो क दवारा रच गए थ और परि ो

दवारा सीख जात थ

- हालाोवक कछ शल क मवहलाओो (जस वक अपाला

घ सा ल पामदरा मतरयी और गागी) क दवारा भी रच

गए थ

- ऋगवद म सोिाद क रप म कई शल क मौजद ह

- हम विशवावमतर नामक एक ऋवि और दविय ो क

रप म पजी जान िाली द नवदय ो (वयास और

सतलज) क बीच िाताि का उदाहरण वमलता ह

- इसस पता चलता ह वक विशवावमतर िवदक काल स

सोबोवित थ

Q2) उततर (b)

सपषटीकरण

- करनल गफाओो स राख क अिशि परापत हए ह

ज इस ओर सोकत करत ह वक ततकालीन ल ग

अवि क उपय ग स पररवचत थ

- य गफाएो आोधर परदश म सथथत ह

Q3) उततर (c)

सपषटीकरण

bull बरािह म ितिमान कशमीर म सथथत एक

परागवतहावसक थथल ह जहाो ल ग गडढ क घर ो का

वनमािण करत थ

bull य घर जमीन क ख द कर बनाए जात थ तथा नीच

जान क वलए सीवियाा ह ती थी

bull ऐसा अनमान लगाया जाता ह वक य घर ठो ड क

मौसम म आशरय परदान करत थ

Q4) उततर (c)

सपषटीकरण

bull परालख-विदया (Epigraphy) क वशलालख ो क

अधययन क रप म पररभावित वकया जाता ह

bull हसतवलसखत दसतािज ो क माधयम स इवतहास

और सावहतय क अधययन क पाोडवलवप विजञान

(Manuscriptology) कहत ह

bull पराचीन लखन परणावलय ो क अधययन और

ऐवतहावसक पाोडवलवपय ो क समझन तथा वतवथ

वनिािरण क पलीओगराफी (Palaeography) कहा

जाता ह

bull नयवमजमविकस (Numismatics) वसक ो क

अधययन क सोदवभित करता ह

Q5) उततर (a)

सपषटीकरण

- चरक सोवहता चरक क दवारा वलखी गई आयिद

और िदयक-शासर पर एक महतवपणि पसतक ह

- ि भारतीय िदयक-शासर की पारमपररक परणाली

वजस आयिद क नाम स जाना जाता ह क

अभयासकताि थ

- ऐसा माना जाता ह वक चरक का विकास दसरी

शताबदी (ईसा पिि) और दसरी शताबदी (ईसवी) क

मधय हआ था

Q6) उततर (b)

सपषटीकरण

- भाग फसल ो पर वलए जान िाल कर क सोदवभित

करता ह ज कल फसल उतपादन का 16 िाो भाग

था

- ldquoकममकारrdquo शबद भवमहीन कवि शरवमक िगि क

वलए परय ग वकया जाता था

- ldquoअशवमिrdquo (वजस घ ड क बवलदान क रप म भी

जाना जाता ह) एक अनषठान ह ता था वजसम एक

घ ड क सवतोतर रप स घमन क वलए छ ड वदया

FC19H1003 30

जाता ह और राजा क सवनक उसकी रखिाली

करत थ

Q7) उततर (d)

सपषटीकरण

- ऋगववदक काल म घ ड ो क रथ ो म ज ता जाता था

ज (रथ) भवम मिवशय ो आवद पर कबजा करन क

वलए लड गए यद ो म उपय ग वकए जात थ

- इसस यह पता चलता ह वक घ ड ो यकत रथ ो का

उपय ग महाजनपद काल स काफी पहल आरमभ

हआ था

- ऋगववदक काल म मिवशय ो भवम जल आवद पर

कबजा करन क वलए तथा ल ग ो क पकडन क

वलए यद वकय जात थ

- अविकाोश परि इन यद ो म भाग वलया करत थ

- हालाोवक उस समय क ई वनयवमत सना नही ो ह ती

थी लवकन उस काल म सभाऐो ह ती थी ो वजनम

ल ग यद क मामल ो पर चचाि करत थ

- वनयवमत सनाएा महाजनपद काल का िवशषटय थी

वजनम पदल सवनक ो की विशाल सनाएा रथ तथा

हाथी शावमल ह त थ

Q8) उततर (a)

सपषटीकरण

- बद शाकय कल स सोबोवित थ और कशीनारा म

उनका वनिन हआ था

- बद न अपनी वशकषाएा पराकत भािा म दी थी ो ज

आम ल ग ो की भािा थी

Q9) उततर (c)

सपषटीकरण

- पराचीन भारत म दशिनशासर की छह शाखाएा थी ो

िशविक नयाय समखया य ग पिि वममाोसा और

िदाोत या उततर वममाोसा

- इनकी थथापना करमश कनाद गौतम कवपल

पतोजवल जावमनी और वयास ऋविय ो न की थी

Q10) उततर (b)

सपषटीकरण

महािीर की वशकषाऐो छठी शताबदी म िललभी म

सोकवलत की गई थी ो

Q11) उततर (c)

सपषटीकरण

- पारमपररक रप स चाणकय क कौविलय अथिा

विषणगपत क नाम स जाना जाता ह

- उसन अथिशासतर ज एक पराचीन भारतीय

राजनवतक आलख ह वलखा था

Q12) उततर (d)

सपषटीकरण

- भारत का राषटर ीय वचनह सारनाथ (उततर परदश) क

अश क सतमभ क ऊपर (शीिि पर) वसोह कवपिल

का एक अनरपण ह

- इस राषटर ीय वसदाोत सतयमि जयत क साथ

सोय वजत वकया गया ह

- रामपिि बल का नाम रामपिि (वबहार) क नाम पर

पडा जहाा इसकी ख ज हई थी

- यह अपन नाजक नकाशी मॉडल क वलए परवसदद

ह वजसम क मल तवचा सोिदनशील नथन ो सतकि

कान और मरबत िााग ो क शरषठतर परवतरप क

परदवशित वकया गया ह

- यह भारतीय और फारसी ततव ो का एक ससममशरण

- सोवकससा उततर परदश म सथथत ह

Q13) उततर (a)

सपषटीकरण

का िर वसोह ज एक महान य दा थ वबहार स

सोबोवित थ

Q14) उततर (b)

सपषटीकरण

िललालर शबद बड भ-सवावमय ो क वलए परय ग

वकया जाता था

FC19H1003 31

Q15) उततर (c)

सपषटीकरण

- अररकमड एक तिीय बसती थी जहाो दर दश ो स

आन िाल जहाज ो का माल उतारा जाता था

- यहाो पर ईोि ो का एक विशाल ग दाम वमटटी क

बतिन (वजनम एमफ रा - द हरी मवठय ो का लोबा

घडा - शावमल ह) और एरिाइन (Arretine)

मदभाोड पाए गए थ

- इस थथान पर र मन दीपक काोच क बन पातर और

रतन भी पाए गए थ

Q16) उततर (a)

सपषटीकरण

- मिनदर सोगम कविताओो म उसललसखत एक

तवमल शबद ह वजसका अथि ह ldquoतीन परमखrdquo

- यह तीन सततारि पररिार ो क मसखयाओो क वलए

परय ग वकया जाता ह च ल चर और पाणडय

Q17) उततर (c)

सपषटीकरण

- ऋग िद म सभा विदाथा तथा गण जसी

जनजावतय ो पर अथिा किोब पर आिाररत

सभाओो का उललख ह

- आरसमभक िवदक काल म सभाओो और सवमवतय ो

का विशि महतव ह ता था

- यहाा तक की मसखया अथिा राजा भी उनका

समथिन परापत करन क वलए आतर रहत थ

Q18) उततर (a)

सपषटीकरण

- जन िमि न ईशवर क अससततव क मानयता त दी ह

वकनत उसन ईशवर क वजना क पद स नीच रखा

- जन िमि न बौद िमि की तरह िणि परणाली की

भरतिना नही ो की थी

Q19) उततर (d)

सपषटीकरण

- च ल ो और पाणडय ो न शसकतशाली तिीय शहर ो का

विकास वकया था

- च ल ो का सबस महतवपणि शहर पहार (या

कािरीपटटीनम) था |

- मदरई पाणडय ो की राजिानी थी

Q20) उततर (b)

सपषटीकरण

- ldquoबदचररतrdquo बद का जीिन-ितताोत ह

- इस अशवघ ि क दवारा वलखा गया था

Q21) उततर (a)

सपषटीकरणः

- तवमल कवि अपपर भगिान वशि क भकत थ

- इस परकार ि एक नयनार सोत थ

Q22) उततर (d)

सपषटीकरणः

- समदरगपत एक परवसद गपत शासक था

- उसन वसक ो पर िीणा बजात हए अपनी छवि

अोवकत करिाई थी

- यह सोगीत क परवत उसक परम क दशािता ह

- हम उसकी इलाहाबाद परशससत स महतवपणि

ऐवतहावसक जानकारी वमलती ह वजसकी रचना

उसक दरबार क कवि हररसन न की थी

Q23) उततर (b)

सपषटीकरणः

- विकरम सोित की शरआत ििि 58 ईसा पिि म

चनदरगपत वदवतीय न की थी

- यह शक ो पर उसकी जीत और उस विकरमावदतय

की पदिी वमलन क उपलकषय म आरमभ वकया गया

था

FC19H1003 32

- बानभटट न हिििििन का जीिन-ितताोत हििचररत

(ज सोसकत म थी) वलखी थी

Q24) उततर (c)

सपषटीकरणः

- सोवि-विगरावहका यद एिो शाोवत का मोतरी

- साथििाह वयापाररय ो क कावफल ो का नता

Q25) उततर (a)

सपषटीकरणः

- जआन झाोग (हसआन रताोग ndash Hsuang Tsang)

एक चीनी यातरी था ज हिििििन क शासनकाल म

भारत आया था

- ििि 630 ईसवी स ज दशक आरमभ हआ था उसम

जआन झाोग मधय एवशया ईरान और

अफग़ावनसतान की यातरा करन क पशचात कशमीर

क रासत स भारत आया था

- उसन उततर स पिि तक की यातरा की और िह

लगभग 2 ििि वबहार म रहा

- जआन झाोग न नालनदा विशवविदयालय म विदयावथिय ो

और विदवान ो क साथ पारसपररक विचार-विमशि

वकया थथानीय भािाओ ा म वनपणता परापत की तथा

बौद सतप ो की ख ज की

Q26) उततर (c)

सपषटीकरणः

- परदवकषणा पथ बौद िासतकला म सतप क चार ो

ओर बनाया जान िाला एक घमािदार पथ ह ता

- परशन म वदए गए बाकी क तीन ो ततव वहोद मसनदर ो की

िासतकला क भाग ह

Q27) उततर (d)

सपषटीकरणः

परशन म वदए गए सभी मोवदर ो म वयापक रप स

ईोि ो (पकी ईोि ो) का परय ग पतथर ो क साथ हआ

Q28) उततर (c)

सपषटीकरण

- महममद कली कतब शाह ग लकणडा का सलतान

था

- िह अकबर का समकालीन था

- सावहतय और िासतकला म उसकी अतयाविक

रवच थी

- िह एक महान कवि था

- िह दसखनी उदि फारसी और तलग म वलखता था

- उसन अपन पीछ एक विसतत वदिान (सोगरह)

छ डा ह

- अभी हाल ही म तलोगाना म ग लकणडा क वकल

क अनदर खदाई वकय गए बाग-ए-नाया वकला

बाग क चार ो ओर रप-रखा क मानवचतरण क

वलए भारतीय परातासतवक सिकषण (The

Archaeological Survey of India ndash ASI)

गराउणड पनीिर विोग रडार (Ground Penetrating

Radar) का परय ग करगा

Q29) उततर (a)

सपषटीकरणः

- वसलपपावदकारम एक तवमल महाकावय ह वजसकी

रचना इलाोग क दवारा लगभग 1800 ििि पिि की

गई थी

- यह क िलन नामक एक वयापारी की कहानी ह

ज माििी नामक एक गवणका (िशया) स परम

करन लगा था

- मवनमकलाई क िलन और माििी की पतरी की

कहानी ह

Q30) उततर (a)

सपषटीकरण

- चरक आयिद और वचवकरता की एक महतवपणि

रचना चरक सोवहता क लखक ह

- बरहमगपत क अपनी रचना बरहम-सफि-वसदानत

(ज एक खग लीय रचना ह) क कारण परवससद

वमली

FC19H1003 33

- बगदाद म इसका अनिाद अरबी भािा म वकया

गया था

- इसका इसलावमक गवणत और खग ल-विजञान पर

महतवपणि परभाि पडा था

- बाद म अपन जीिनकाल म बरहमगपत न

ldquoखोडखयाकrdquo वलखी ज एक खग लीय पससतका

(एक छ िी पसतक) थी

- इसम आयिभटट की अिि-रावतर क परतयक वदन की

शरआत परणाली का परय ग वकया गया था

Q31) उततर (c)

सपषटीकरण

- अमीर खसर एक परवसद सफी सोगीतकार कवि

और विदवान थ

- 1318 म उनह ोन पाया वक इस भवम (वहोदसतान) क

हर कषतर म अलग-अलग भािा थी लाहौरी

कशमीरी दवारसमदरी (दवकषणी कनाििक म)

तलोगाना (आोधर परदश म) गजरी (गजरात म)

माबारी (तवमलनाड म ) अििी (पिी उततर परदश

म) और वहोदिी (वदलली क आस-पास क कषतर म)

आवद

- उनह न यह बताया वक सोसकत वकसी भी कषतर स

सोबोवित नही ो थी और किल बराहमण ही इस भािा

का जञान रखत थ

Q32) उततर (c)

सपषटीकरण

- वहरणय-गभि सववणिम गभि क सोदवभित करता ह

- जब बराहमण ो की सहायता स यह अनषठान वकया

जाता था त यह माना जाता था वक बवल दन िाल

का कषवतरय क रप म पनजिनम ह गा

Q33) उततर (d)

सपषटीकरण

- कदमई भवम राजसव पर कर क सोदवभित करता

- गवावलयर परशससत म नागभि क दवारा वकय गए

श िण का िणिन वकया गया ह |

- नागभि एक परवतहार राजा था

Q34) उततर (b)

सपषटीकरण

- राजतरो वगनी 12िी ो शताबदी म कलहन क दवारा

रवचत एक सोसकत पसतक (िकसट) ह

- यह परारसमभक भारत की ऐवतहावसक इवतितत थी

- तकि सोगत रप स इस अपन परकार की सिोततम

और सिािविक विशवसनीय कवत माना जाता ह

- यह कशमीर कषतर क पराचीनतम समय स लकर

उसकी रचना की तारीख तक क समपणि इवतहास

का आचछादन करती ह

Q35) उततर (c)

सपषटीकरण

- गााि की आम सभा क ldquoउरrdquo कहा जाता था

- ldquoउरrdquo म गााि क सभी कर दन िाल वनिासी

शावमल ह त थ

Q36) उततर (a)

सपषटीकरण

- वदलली सलतनत म ldquoतारीखrdquo इवतहास लखन का

एक रप था

- ldquoतािरीखrdquo क लखक विदवान परि ह त थ वजनम

सवचि परशासक इतयावद शावमल थ

Q37) उततर (a)

सपषटीकरण

- अलाउददीन सखलजी अपन सवनक ो क ितन का

भगतान नकद म करता था न वक इकता क रप

- सवनक अपना सामान वदलली म वयापाररय ो स

खरीदत थ अतः इस बात का भय था वक वयापारी

कही ो िसतओो का मलय न बिा द

- इसकी र कथाम क वलए अलाउददीन सखलजी न

वदलली म कीमत ो क वनयसित वकया

FC19H1003 34

- अविकारीगण धयानपििक मलय ो का सिकषण करत

थ तथा ज वयापारी वनिािररत मलय पर माल नही ो

बचत थ उनक दसणडत वकया जाता था

Q38) उततर (d)

सपषटीकरण

- वदलली सििपरथम त मर राजपत ो क अिीन उनक

सामराजय की राजिानी बनी थी

- 12िी ो शताबदी क मधय म अजमर क चौहान ो

(वजनह चाहमान ो क नाम स भी जाना जाता ह) न

त मर राजपत ो क परावजत वकया था

- त मर ो और चौहान ो क अिीन वदलली एक

महतवपणि िावणसजयक क दर बन गया था

- कई जन वयापारी यहाा रहन लग थ और उनह ोन

कई मोवदर भी बनिाए

- यहाा पर मवदरत वसक वजनह ldquoदहलीिालrdquo क नाम

स जाना जाता था वयापक रप स परचलन म थ

Q39) उततर (c)

सपषटीकरण

- म ठ की मसिद का वनमािण वसको दर ल दी क

राजयकाल म उसक मिी क दवारा करिाया गया

था

- बगमपरी मसिद का वनमािण महममद तगलक क

शासनकाल म हआ था

- यह मसिद विशव का पणयथथान (The

Sanctuary of the World) और वदलली म महममद

तगलक की नई राजिानी जहाोपनाह की मखय

मसिद थी

- कववत- अल - इसलाम मसिद का विसतार

इलतसिश और अलाउददीन सखलजी न वकया था

- मीनार का वनमािण तीन सलतान ो कतबददीन ऐबक

इलतसिश और वफर ज शाह तगलक क दवारा

करिाया गया था

Q40) उततर (c)

सपषटीकरण

- मगल ो क अिीन मनसबदार शबद उस वयसकत क

वलए सोदवभित वकया जाता था वजसक पास मनसब

(अथाित पद) ह ता था

- उस अपना ितन राजसव कायो वजनह जागीर कहत

थ क रप म परापत ह ता था

Q41) उततर (b)

सपषटीकरण

- ldquoभारत छ ड आोद लनrdquo वबरविश शासन क

सखलाफ ल ग ो का एक सवाभाविक विदर ह था

- असखल भारतीय काोगरस सवमवत न 8 अगसत 1942

क बमबई म एक बठक का आय जन वकया था

- इस बठक म परवसद सोकलप ldquoभारत छ ड rdquo क

पाररत वकया गया और इस उददशय क परापत करन

क वलए गाोिी क नततव म एक अवहोसक जन सोघिि

आोद लन की शरआत का परसताि वदया गया

- लवकन अगल ही वदन गाोिी और काोगरस क अनय

परमख नताओो क वगरफतार कर वलया गया

- काोगरस क एक बार वफर अिि घ वित वकया गया

था

Q42) उततर (c)

सपषटीकरण

- साइमन कमीशन यनाइविड वको गडम क सात

साोसद ो का एक समह था

- इस वबरविश भारत क वलए सोििावनक सिार ो का

सझाि दन क वलए गवठत वकया गया था

- इस आय ग म िररषठ वबरविश राजनता सर जॉन

साइमन क नततव म किल वबरविश सदसय ही

शावमल थ

- इसवलए भारत क ल ग ो न साइमन कमीशन क

आगमन क विरद आोद लन वकया था

Q43) उततर (a)

सपषटीकरण

bull दादा भाई नौर जी भारत म वबरविश शासन क

आवथिक पररणाम ो क बार म अपनी विर िी

(परवतकल) राय क वलए जान जात थ

FC19H1003 35

bull अपन कई लख ो और भािण ो म विशि रप स

ldquoपाििी एो ड अन-वबरविश रल इन इसणडया

(Poverty and Un-British Rule in India) म

नौर जी न यह तकि वदया वक भारत पर अतयविक

कर लगाया गया था और इसकी सोपवतत इोगलड की

ओर परिावहत की जा रही थी

bull उनह ोन पराचीन भारतीय गरोथ ो की वयाखया करन

का और भारतीय ो क आिविशवास क बहाल

करन पर कायि नही ो वकया था

उनह ोन वकसी और बात स पहल सभी सामावजक

बराइय ो क उनमलन की आिशयकता पर भी बल

नही ो वदया था

Q44) उततर (c)

सपषटीकरण

bull अगसत 1932 म वबरविश परिानमोतरी मकड नालड न

अपन साोपरदावयक परसकार (The Communal

Award) की घ िणा की थी

bull यह भारत क कई साोपरदावयक वहत ो क बीच विवभनन

सोघिो क हल करन क वलए वबरिन का एकतरफा

परयास था

bull यह परसकार (Award) बाद म 1935 क

अविवनयम (The Act of 1935) म शावमल वकया

गया था

bull इस साोपरदावयक परसकार न मससलम ो क वलए

आरवकषत एक अलग वनिािचक मणडल फॉमिल का

विसतार अनय अलपसोखयक ो क वलए वकया था

वजसम वसख ो भारतीय ईसाइय ो आोगल-भारतीय

समदाय यर पीय समदाय तथा विवशषट कषतरीय

समह ो क शावमल वकया गया था

bull गाोिी न इस परसताि क भारतीय समाज क

विभावजत करन क वलए एक घवणत वबरविश

सावजश क रप म दखा और उसक सखलाफ

आमरण अनशन वकया

Q45) उततर (b)

सपषटीकरण

मौजदा आयात और वनयाित क अवतररक़त

औपवनिवशक भारत क वनमनवलसखत खचो क

वलए एक विशिवनवशचत िन रावश भी दनी पडती

थी

(i) परशासन क वयय

(ii) सना क रख-रखाि क वयय

(iii) यद क वयय

(iv) सिावनितत अविकाररय ो की पशन तथा

(v) वबरिन दवारा अपनी उपवनिश बसती

(कॉल नी) क रख-रखाि क वयय

इनह गह शलक (Home Charges) क रप म

जाना जाता था और लगभग परी तरह स भारत क

दवारा इनका भगतान वकया जाता था

bull गह शलक म वनमनवलसखत घिक शावमल थ

(i) भारतीय ऋण पर दय बयाज

(ii) ईसट इोवडया को पनी क शयरिारक ो क

लाभाोश

(iii) लोदन म भारत कायािलय चलान क वलए िन

(iv) भारत म वनयकत वबरविश कवमिय ो क ितन

और पशन का भगतान करन क वलए िन

(v) रलि पर बयाज

(vi) नागररक और सनय शलक

(vii) इोगलड म सट र (सामगरी) की खरीद

Q46) उततर (b)

सपषटीकरण

bull भारतीय राषटर ीय काोगरस का लाहौर सतर 1929 म

जिाहरलाल नहर की अधयकषता म आय वजत

वकया गया था

bull इस सतर म भारतीय राषटर ीय आोद लन स समबसित

कई महतवपणि पररणाम सामन आय थ

(i) सििपरथम इस सतर म काोगरस क अधयकष पद

पर जिाहरलाल नहर क चना गया था ज

काोगरस म िामपोवथय ो की बिती हई ताकत

का सपषट सोकत था

(ii) दसरा इस सतर म पहली बार काोगरस न पणि

सवतोतरता की माोग क उठाया था

इस परकार की माोग काोगरस मोच स पहल कभी भी

नही ो उठाई गई थी

Q47) उततर (b)

सपषटीकरण

FC19H1003 36

bull इस ररप िि न वकसी भी समदाय क वलए पथक

वनिािचक मोडल अथिा अलपसोखयक ो क वलए

भाराोश की वसफाररश नही ो की थी

bull तथावप इस ररप िि न उन पराोत ो म अलपसोखयक

सीि ो क आरकषण की अनमवत दी थी जहाा पर कम

स कम दस परवतशत अलपसोखयक ह

bull लवकन यह समदाय क आकार क अनपात म ह ना

चावहए था

bull इस ररप िि म भारत क वलए पणि सवतोतरता क

वलए क ई पराििान नही ो था

Q48) उततर (c)

सपषटीकरण

bull आरो वभक िवदक आयो का िमि मखय रप स

परकवत की पजा और यजञ था

bull परारो वभक आयि िमि परकवत की पजा क समान था

bull िासति म उनक चार ो ओर की शसकतयाा वजनह न

त ि वनयोवतरत कर सकत थ और न ही समझ पाए

थ उनह वदवयता क साथ वनिवशत वकया गया तथा

उनह मादा या नर दिीदिताओो क रप म

परतीकतव वकया गया था

bull उनह ोन कछ यजञ ो का भी वनषपादन वकया था

Q49) उततर (b)

सपषटीकरण

bull सडक और नदी-मागि (जल-मागि) डकती स

सरवकषत नही ो थ

bull उललखनीय ह वक हिििििन क शासनकाल क

दौरान यआन चिाोग (हयएन साोग) का सारा

सामान लि वलया गया था

Q50) उततर (c)

सपषटीकरण

परशन म वदए गए द न ो कथन सही ह

Q51) उततर (b)

सपषटीकरण

bull परोदर दास एक सोत और भगिान कषण क एक

महान भकत थ

bull परोदर दास क कनाििक सोगीत क वपतामह क

रप म जाना जाता ह

bull यदयवप उनक जनम-थथान क बार म काफी

अिकल लगाई जाती रही ह

bull तथावप अब कननड विशवविदयालय हमपी क दवारा

गवठत एक विशिजञ सवमवत इस वनषकिि पर पहोची

ह वक उनका जनम थथान सोभितया कनाििक का

एक छ िा-सा गााि कषमपरा (वशिम गगा वजला)

था

Q52) उततर (c)

सपषटीकरण

bull शरी तयागराज शरी शयाम शासतरी और शरी मथसवामी

दीवकषतर क कनाििक सोगीत की वतरमवति माना

जाता ह

bull उनक कारण ही 18िी ो-19िी ो शताबदी म कनाििक

सोगीत का सववणिम यग आया था

Q53) उततर (d)

सपषटीकरण

bull अभी हाल ही म लौह यगीन-महापािावणक काल

का 2000 ििि पराना एक दलिभ सारक फगस

(Sarcophagus) (पतथर का ताबत) क ललम क

वियर गाोि (क वयलडी क पास वजला क वझक ड

करल राजय) की एक रॉक-कि गफा स ख जा गया

bull यह ताबत वजसम हविय ो क िकड थ खदाई क

दौरान वमला

bull अभी तक इस परकार की दलिभ ख ज करल क

मातर द ही थथान ो स हई ह

bull य द न ो सारक फगी (Sarcophagi) (पतथर क

ताबत) चियर और अथ ली (वजला क वझक ड) क

महापािाण थथल ो स वमल ह

Q54) उततर (a)

सपषटीकरण

FC19H1003 37

दवकषण भारत म महापािाण सोसकवत एक पणि

विकवसत लौह यगीन सोसकवत थी

Q55) उततर (d)

सपषटीकरण

bull च ल पाणडय और करलपतर (चर) इन तीन ो का

उललख अश क क अवभलख ो म वकया गया ह

bull सोभितः य भौवतक सोसकवत क उततर

महापािावणक चरण म थ

Q56) उततर (d)

सपषटीकरण

bull भीमा-क रगाोि की लडाई ततीय आोगल-मराठा

यद का वहससा थी

Q57) उततर (b)

सपषटीकरण

bull राजकमार शकल न गाोिीजी क चोपारण आन तथा

वतनकवथया परणाली स जडी समसया की जाोच क

वलए रारी करन क वलए दश भर म उनका

अनसरण वकया था

bull बज वकश र राजदर परसाद महादि दसाई और

नरहरी पाररख चोपारण सतयागरह क दौरान गाोिी

जी क सहय गी थ

Q58) उततर (b)

सपषटीकरण

bull बराहमण ो और बौद मठिाररय ो क कर-मकत गााि

अनदान म दन की परथा सतिाहन ो न आरमभ की

थी

Q59) उततर (c)

सपषटीकरण

इस कायिकरम क उददशय वनमनानसार ह

(i) बवनयादी पयििन आिाररक सोरचना का विकास

करना

(ii) चयवनत (पहचान वकय गए) कषतर ो म आजीविका क

सजन क वलए दश क साोसकवतक और विरासत

मलय ो क बिािा दना

(iii) विरासत समारक थथल ो पर विशव सतरीय आिाररक

सोरचना विकवसत करक एक सतत तरीक स

पयििक आकििण म िसद करना

(iv) थथानीय समदाय ो की सवकरय भागीदारी क माधयम

स र रगार ो का सजन करना

(v) र रगार उतपादन और आवथिक विकास क वलए

पयििन कषमता का उन पर परभाि का उपय ग

करना तथा

(vi) िारणीय पयििन आिाररक सोरचना का विकास

करना और उसका उवचत सोचालन तथा

रखरखाि सवनवशचत करना

Q60) उततर (b)

सपषटीकरण

bull यह वनकाय ििि 1987 म अससततव म आया था

bull यह एक राषटर ीय सतर का शीिि सोगठन ह ज भारत

सरकार क जनजातीय मामल ो क मोतरालय क

परशासवनक वनयोतरण क अिीन काम कर रहा ह

bull इसका पोजीकत और परिान कायािलय नई वदलली

म सथथत ह

Q61) उततर (c)

सपषटीकरण

bull परमचोद क उपनयास ो म परमाशरम रोगभवम गबन

कमिभवम और ग दान शावमल ह

bull ग रा रिी ोदरनाथ िग र क दवारा रवचत उपनयास ह

bull अभी हाल ही म मोशी परमचोद की 138िी ो जयोती दश

भर म मनाई गई थी

Q62) उततर (b)

सपषटीकरण

bull ldquoवगदाrdquo पोजाब (भारत) एिो पावकसतान की

मवहलाओो क दवारा तयौहार क समय और फसल

की बिाई तथा किाई क अिसर पर वकया जान

िाला एक पारोपररक दहाती नतय ह

FC19H1003 38

bull इस नतय क माधयम स पोजाबी मवहलाऐो अपनी

परसननता परकि करती ह तथा वगदा क परदशिन क

माधयम स परि िचिसव िाल समाज म मवहलाओो

की दबी हई भािनाओो क परकि करती ह

bull चोवक इस नतय का परि ो क साथ क ई सोबोि नही ो

ह अतः किल मवहलाऐो ही इसम भाग ल सकती

bull हर साल तीज समार ह क दौरान पोजाब म वगदा

नतय वकया जाता ह

तीज भारत क कछ भाग ो म मवहलाओो क दवारा

मनाया जान िाल कई तयौहार ो क वलए एक

वयापक नाम ह

Q63) उततर (a)

सपषटीकरण

- मजम-उल-बहरीन या द समदर ो का सोगम

नामक उललखनीय रचना दारा वशक ह क दवारा

वलखी थी

- भारत क उपराषटर पवत शरी एम िकया नायड न कहा

ह वक राजकमार दारा वशक ह की रचनाएा शाोवत

और सदभाि क बिािा दन क वलए एक तारा सर त

क रप म सामन आ सकती ो ह

- उपराषटर पवत गत ििो क भला वदए गए राजकमार

दारा वशक ह क परदवशित परचवलत करन हत

आय वजत एक परदशिनी का दौरा करन क बाद एक

सभा क सोब वित कर रह थ

- इस परदशिनी का आय जन फर क इस गौवियर

(Francois Gautier) क दवारा lsquoइोवदरा गाोिी नशनल

सिर फॉर द आििसrsquo (The Indira Gandhi

National Centre for the Arts) नई वदलली म

वकया गया था

Q64) उततर (c)

सपषटीकरण

- ग मतशवर परवतमा जन भगिान बाहबली क

समवपित ह

- यह एक एक-चटटानी पतथर की मवति ह

- राषटर पवत राम नाथ क विोद न शरिणबलग ला

(कनाििक) म आय वजत वकय जान िाल भवय

अवभिक समार ह महामसतकावभिक का

उदघािन वकया था

- यह समार ह 12 ििो म एक बार ह ता ह

Q65) उततर (c)

सपषटीकरण

bull पराची घािी पराची नदी क चार ो ओर फली हई थी

bull पराची घािी िीर-िीर विलपत ह गई थी

bull पराची नदी भिनशवर स वनकलती ह

bull यह महानदी की एक सहायक नदी ह और यह

परी खदाि किक तथा जगतवसोहपर वजल ो स

ह कर बहती ह

bull इस नदी क पर कषतर क पराची घािी कहा जाता ह

bull यह नदी बोगाल की खाडी म वगरती ह

परातासतवक साकषय स पता चलता ह वक पराची घािी

सभयता हडपपा और म हनज दाड द न ो की

पिििती ह

Q66) उततर (d)

सपषटीकरण

य समारक छतरपर वजल (मधय परदश) म विोधयाचल

पिित शरोखला म सथथत ह

Q67) उततर (a)

सपषटीकरण

bull थॉिस ऑन पावकसतान नामक पसतक डॉ बी

आर अमबडकर न वलखी थी

bull डॉ बी आर अमबडकर की जयोती क अिसर पर

भारत क राषटर पवत न भारत की इस महान हसती

क शरदाोजवल अवपित की थी

bull डॉ बी आर अमबडकर न 1924 म वडपरथड

कलावसर इोसटीटयि (दवलत िगि सोथथान -

बवहषकत वहतकाररणी सभा) और 1927 म समाज

समता सोघ की थथापना की थी

bull अमबडकर का धयान वशकषा कषतर की ओर भी था

bull उनह ोन वशकषा क वनमन िगो म फलान क वलए

पीपलस एजकशन स साइिी (The Peoples

Education Society) क नाम स महाविदयालय ो क

नििकि और छातरािास ो की थथापना की थी

FC19H1003 39

Q68) उततर (b)

सपषटीकरण

bull महरगि भारतीय उपमहादवीप म एक परवसद

निपािाण बसती ह ज बलवचसतान पराोत

पावकसतान म सथथत ह

bull दचपलली (आोधर परदश) क पास नागलर नदी क

पिी ति ो पर चना पतथर क बलॉक क विशाल

विसतार म एक पिि-ऐवतहावसक रॉक आिि थथल की

ख ज की गई ह

bull इसन 1500-2000 ईसा पिि क दौरान गोिर (आोधर

परदश) म विकवसत निपािाण सभयता पर परकाश

डाला ह

Q69) उततर (c)

सपषटीकरण

bull 12िी ो सदी और 13िी ो सदी म काकाविय िोश का

उदय हआ था

bull ि पहल कलयाण क पवशचमी चालकय ो क सामोत थ

bull परारोभ म उनह ोन िारोगल (तलोगाना) क पास एक

छ ि स कषतर पर शासन वकया था

bull उनह ोन ldquoनायक वयिथथाrdquo की शरआत की थी

वजस बाद म विजयनगर क राय शासक ो न

अपनाया और विकवसत वकया था

Q70) उततर (a)

सपषटीकरण

bull गाोिीजी क अनशन स वमल मावलक ो पर दबाि

पडा था ज अोततः शरवमक ो क ितन म 35 परवतशत

की िसद करन क वलए सहमत हए थ

bull गगल (Google) न अनसया साराभाई वजनह ोन

भारत क शरवमक आोद लन म एक अगरणी भवमका

वनभाई थी की 132िी ो जयोती डडल (Doodle) का

वनमािण करक मनाई

Q71) उततर (d)

सपषटीकरण

भारत स यनसक की मानिता की अमति साोसकवतक

विरासत की परवतवनवि सची म वनमनवलसखत शावमल ह

bull कवडयटटम करल का सोसकत रोगमोच

bull मवडयिि करल का अनषठान रोगमोच और नतय

नाविका

bull िवदक मि जाप की परोपरा

bull राजथथान क कालबवलया ल क गीत और नतय

bull रामलीला रामायण का पारोपररक परदशिन

bull सोकीतिन मवणपर का अनषठान गायन ढ ल िादन

और नतय

bull रममन भारत क गििाल वहमालय का िावमिक

तयौहार और अनषठान रोगमोच

bull जाोदीयाला गर पोजाब क ठठर ो की पीतल और

ताोब क वशलप स वनवमित बतिन ो की पारोपररक कला

bull छाऊ नतय पिी भारतीय राजय ो म जनमी शासतरीय

भारतीय नतय कला

bull लददाख का बौद मि जाप िर ाोस-वहमालयी लददाख

कषतर तथा जमम-कशमीर म पवितर बौद गरोथ ो का पाठ

bull य ग

bull नौर र

bull को भ मला

Q72) उततर (b)

सपषटीकरण

bull भारत क राषटर पवत शरी राम नाथ क विोद न

वकसामा नागालड म हॉनिवबल मह रति और

राजय गठन वदिस समार ह का उदघािन वकया

था

bull हॉनिवबल मह रति का नाम भारतीय हॉनिवबल क

नाम पर पडा ह ज एक विशाल और रोगीन जोगली

पकषी ह

bull यह पकषी नागालड राजय की अविकतर जनजावतय ो

की ल ककथाओो म उसललसखत ह

bull नागालड की परमख मानयता परापत जनजावतयाा ह

अोगामी आओ चखसोग चाोग ककी रगमा और

रवलोग आवद

bull ओोग जारिा और ससिनलीस अोडमान-वनक बार

दवीप समह की जनजावतयाा ह

FC19H1003 40

Q73) उततर (c)

सपषटीकरण

bull दकन म राषटर कि शासन दसिी ो सदी क अोत तक

लगभग 200 ििो तक रहा था

bull राषटर कि शासक अपन िावमिक विचार ो म सवहषण

bull उनह ोन न किल शि िमि और िषणि िमि बसलक

जन िमि क भी सोरकषण वदया था

bull एल रा म वशि क परवसद रॉक कि मोवदर का

वनमािण नौिी ो सदी म राषटर कि राजा कषण परथम न

करिाया था

bull उसका उततराविकारी अम घििि जन था लवकन

उसन अनय िमो क भी सोरकषण परदान वकया था

bull राषटर कि ो न मसलमान वयापाररय ो क बसन की

अनमवत दी थी

bull उनह न अपन अविराजय ो म इसलाम क उपदश दन

की भी अनमवत दी थी

bull अभी हाल ही म पाोडिलागटटा (तलोगाना) क

परागवतहावसक चटटान वचतर ो क कषरण की बिती हई

घिनाएा एक गोभीर वचोता का वििय ह

bull यह परागवतहावसक चटटान क नकसान पहाचा

सकता ह

bull पाोडिलागटटा वनमनवलसखत क वलए जाना जाता ह

- 10000 ईसा पिि स 8000 ईसा पिि क वचवतरत

चटटानी आशरय ो क वलए

- राषटर कि काल क एक 8 िी ो सदी क

वशलालख क वलए और

- 12िी ो सदी क काकविय सामराजय क वभवतत

वचतर ो क वलए

Q74) उततर (b)

सपषटीकरण

bull 1828 म राजा राम म हन रॉय न एक नय िावमिक

समाज बरहम सभा की थथापना की थी वजस बाद

म बरहम समाज क नाम स जाना गया था

bull दिदरनाथ िग र न ततवब विनी सभा की अधयकषता

की थी ज आधयासिक सतय की ख ज म सोलि

थी

bull इसका उददशय वहोद िमि क शद करन का और

एकशवरिाद (एक ईशवर म आथथा) का परचार करना

था

bull नय समाज की थथापना क आिार थ कारण

(तकि ) क द सतमभ तथा िद और उपवनिद

bull अभी हाल ही म सािारण बरहम समाज का कछ

काननी मदद ो क लकर पवशचम बोगाल सरकार क

साथ काननी वििाद चल रहा ह

Q75) उततर (c)

सपषटीकरण

bull भारत म वचशती वसलवसल की थथापना खवाजा

म इनददीन वचशती क दवारा की गयी थी

bull ि 1192 ईसवी क आसपास भारत आय थ

bull वचशतीय ो क बारहिी ो शताबदी क उततरािि म भारत

म आन िाल सफीय ो क समह ो म सबस

परभािशाली माना जाता ह

bull उनह ोन थथानीय िातािरण क साथ सफलतापििक

अनकलन वकया और उनह ोन भारतीय भसकत

परोपराओो क कई पहलओो क अपनाया

bull अजमर म सफी अपरकि खवाजा म इनददीन वचशती

की ऐवतहावसक दरगाह क एक नया रप दन की

तयारी की जा रही ह

bull इस 13िी ो शताबदी की दरगाह क ldquoसवचछ

आइकॉवनक थथल ोrdquo (Swacch Iconic Places) म

शावमल वकया गया ह ज परवतवषठत विरासत

आधयासिक और साोसकवतक थथान ो पर क वदरत

य जना ह

FC19H1003 41

ANSWERS amp EXPLANATION OF

NCERT History Class VI-X + Current Affairs

(FC19E1003)

Q1) Answer c

Explanation

Rigveda consists of more than a

thousand hymns dedicated to gods and

goddesses These hymns were

composed by sages and learnt by men

however a few were composed by

women like Apala Ghosa Lopamudra

Maitreyi and Gargi

Rigveda consists of many hymns in the

form of dialogues We get an example of

a dialogue between a sage named

Vishwamitra and two rivers (Beas and

Sutlej) that were worshipped as

goddesses This suggests that he

belonged to the Vedic period

Q2) Answer b

Explanation

Traces of ash have been found from

Kurnool Caves suggesting that people

were familiar with the use of fire

It is situated in Andhra Pradesh

Q3) Answer c

Explanation

Burzahom is a prehistoric site in

present day Kashmir where people built

pit houses which were dug into the

ground with steps leading into them

These may have provided shelter in cold

weather

Q4) Answer c

Explanation

Epigraphy is defined as the study of

inscriptions

Manuscriptology is the study of history

and literature through the use of hand

written documents

Palaeography refers to the study of

ancient writing systems and the

deciphering and dating of historical

manuscripts

Numismatics refers to the study of

coins

Q5) Answer a

Explanation

Charaka Samhita was written by

Charaka and is an important book on

Ayurveda and medicine

He was a practitioner of the traditional

system of Indian medicine known as

Ayurveda

Charaka is thought to have flourished

sometime between the 2nd century BCE

and the 2nd century CE

Q6) Answer b

Explanation

Bhaga refers to the tax on crops which

was fixed at 16th of the production

Kammakaras is the term used for the

landless agricultural labour class

Ashvamedha also known as horse

sacrifice is a ritual where a horse is let

loose to wander freely and it was

guarded by the rajarsquos men

Q7) Answer (d)

Explanation

In the Rigvedic period horses were

yoked to chariots that were used in

battles fought to capture land cattle

etc This suggests that the use of horse

chariots began much before the period

of Mahajanapadas

The battles were fought in the Rigvedic

period for cattlersquos lands water an even

to capture people Most men took part

in these wars however there was no

regular army but there were assemblies

where people met and discussed

matters of war Regular armies became

a feature in the Mjahajanapada period

including vast armies of foot soldiers

chariots and elephants

RAUSIAS-FC19E1003 42

Q8) Answer (a)

Explanation

Buddha belonged to the Sakya clan and

passed away at Kusinara

Buddha taught in Prakrit which was the

common language of people

Q9) Answer c

Explanation

There were six schools of philosophy in

ancient India These are known as

Vaishesika Nyaya Samkhya Yoga

Purva Mimansa and Vedanata or Uttara

Mimansa They were founded by sages

Kanada Gautama Kapila Patanjali

Jamini and Vyasa respectively

Q10) Answer b

Explanation

The teachings of Mahavira were

compiled at Valabhi in 6th century AD

Q11) Answer (c)

Explanation

Chanakya is traditionally identified as

Kautilya or Vishnugupta who authored

the ancient Indian political treatise the

Arthashastra

Q12) Answer d

The national emblem of India is an

adaptation of the Lion Capital atop the

Ashoka Pillar of Sarnath Uttar Pradesh

and is combined with the National

Motto Satyameva Jayate

The Rampurva Bull gets the name from

the site of its discovery Rampurva in

Bihar

It is noted for its delicately sculpted

model demonstrating superior

representation of soft flesh sensitive

nostrils alert ears and strong legs It is

a mixture of Indian and Persian

elements

Sankissa is situated in Uttar Pradesh

India

Q13) Ans(a)

Kunwar Singh was a notable leader during the Revolt of 1857 He belonged

to a royal house of Jagdispur Bihar

Q14) Answer b

Explanation

The term Vellalar was used for large

landowners

Q15) Answer c

Explanation

Arikamedu was a coastal settlement

where ships unloaded goods from

distant lands Finds here include a

massive brick warehouse pottery

including amphorae and Arretine ware

Roman lamps glassware and gems have

also been found at the site

Q16) Answer a

Explanation

Muvendar is a Tamil word mentioned in

Sangam poems meaning three chiefs

used for the heads of three ruling

families the Cholas Cheras and

Pandyas

Q17) Ans (c)

Several tribal or kin-based assemblies

such as the Sabha Vidatha and gana

are mentioned in the Rig-veda The

Sabha and the samiti mattered a great

deal in early Vedic times so much so

that the chiefs or the kings showed an

eagerness to win their support

Q18) Ans (a)

Jainism recognised the existence of the

gods but placed them lower than the

jina and did not condemn the varna

system as Buddhism did

Q19) Answer (d)

Explanation

Cholas and Pandyas had developed

powerful coastal cities The most

important city of Cholas was Puhar or

Kaveripattinam and Madurai was the

capital of Pandyas

Q20) Answer b

Explanation

Buddhacharita is the biography of

Buddha and was written by

RAUSIAS-FC19E1003 43

Ashvaghosha

Q21) Answer (a)

Explanation

Tamil poet Appar was a Shiva devotee

So he was a Nayanar saint

Q22) Answer d

Explanation

Samudragupta was a prominent Gupta

ruler whose coins depict him playing a

veena indicating his love for music We

get important historic information from

his Allahabad Prashasti which was

composed by his court poet Harisena

Q23) Answer (b)

Explanation

Vikrama Samvat was founded by

Chandragupta II in the 58 BC as a

mark of victory over the Shakas and

assumed the title of Vikramaditya

Banabhatta wrote Harshavardhanarsquos

biography the Harshacharita in

Sanskrit

Q24) Answer c

Explanation

Sandhi-vigrahika was the minister of

war and peace

Sarthavaha was the leader of the

merchant caravans

Q25) Answer a

Explanation

Xuan Zang (Hsuan-tsang) was a

Chinese traveller who came during the

reign of Harshavardhana

In the decade that began in 630 AD

Xuan Zang came to India through

Kashmir after visiting Central Asia Iran

and Afghanistan

He travelled from north to east and lived

in Bihar for a couple of years

At Nalanda University Xuan Zang

interacted with students and scholars

mastered local languages and

discovered Buddhist stupas

Q26) Answer c

Explanation

Pradakshina patha is a circular path

laid around a stupa in Buddhist

architecture While the rest are a part of

temple architecture

Q27) Answer d

Explanation

All the above-mentioned temples have

an elaborate use of bricks (baked

bricks) along with stone

Q28) Ans (c)

Muhammad Quli Qutab was the Sultan

of Golconda He was a contemporary of

Akbar was very fond of literature and

architecture

The Sultan was a great poet and he

wrote in Dakhini Urdu Persian and

Telgu and has left an extensive diwan or

collection

Recently the Archaeological Survey of

India (ASI) will be using Ground

Penetrating Radar (GPR) to map the

contours of the area around the Bagh-e-

Naya Qila excavated garden inside the

Golconda Fort in Telangana

Q29) Answer a

Explanation

Silappadikaram is a famous Tamil epic

which was written by Ilango around

1800 years ago It is a story of a

merchant named Kovalan who fell in

love with a courtesan named Madhavi

Manimekalai tells the story of the

daughter of Kovalan and Madhavi

Q30) Answer (a)

Explanation

Charaka is the author of Charaka

Samhita which is an important work of

Ayurveda and medicines

Brahmaguptarsquos fame rests mostly on his

Brahma-sphuta-siddhanta which was

an astronomical work It was translated

into Arabic in Baghdad and had a major

impact on Islamic mathematics and

astronomy

Late in his life Brahmagupta wrote

Khandakhadyaka which was an

RAUSIAS-FC19E1003 44

astronomical handbook that employed

Aryabhatarsquos system of starting each day

at midnight

Q31) Answer (c)

Explanation

Amir Khusrau was a famous sufi

musician poet and scholar In 1318 he

noted that there was different language

in every region of this land (Hindustan)

Lahori Kashmiri Dvarsamudri (in

Southern Karnataka) Telangana (in

Andhra Pradesh) Gujari (in Gujarat)

Marsquobari (in Tamil Nadu) Awadhi (in

eastern Uttar Pradesh) and Hindawai (in

the area around in Delhi) etc He went

to explain that Sanskrit did not belong

to any region and that only brahmans

knew it

Q32) Answer c

Explanation

Hiranyagarbha refers to the golden

womb When this ritual was performed

with the help of Brahmanas it was

thought to lead to the rebirth of the

sacrificer as a Khastriya

Q33) Answer d

Explanation

Kadamai refers to a tax on land

revenue

Gwalior Prashasti describes the exploits

of Nagabhata who was a Pratihara king

Q34) Answer b

Explanation

Rajatarangini is a Sanskrit text written

by Kalhana in the 12th century

It was historical chronicle of early India

It is justifiably considered to be the best

and most authentic work of its kind

It covers the entire span of history in

the Kashmir region from the earliest

times to the date of its composition

Q35) Answer c

Explanation

ldquoUrrdquo was the general assembly of the

village ldquoUrrdquo consisted of all the

taxpaying residents of an ordinary

village

Q36) Answer (a)

Explanation

Tarikh was a form of history writing in

the Delhi Sultanate The authors of

tawarikhs were learned men which

included secretaries administrators etc

Q37 Answer (a)

Explanation

Alauddin chose to pay his soldiers salaries in cash rather than iqtas The soldiers would buy their supplies from merchants in Delhi and it was thus feared that merchants would raise their prices To stop this Alauddin controlled the prices of goods in Delhi Prices were carefully surveyed by officers and merchants who did not sell at the prescribed rates were punished

Q38) Answer (d)

Explanation

Delhi first became the capital of a

kingdom under the Tomara Rajputs

who were defeated in the middle of the

twelfth century by the Chauhans (also

referred to as Chahamanas) of Ajmer

It was under the Tomaras and

Chauhans that Delhi became an

important commercial centre Many rich

Jaina merchants lived in the city and

constructed several temples Coins

minted here called dehliwal had a wide

circulation

Q39) Answer (c)

Explanation

Moth ki Masjid was built in the reign of

Sikandar Lodi by his minister

Begumpuri mosque built in the reign of

Muhammad Tughluq was the main

mosque of Jahanpanah the ldquoSanctuary

of the Worldrdquo and his new capital in

Delhi

Quwwat al ndash Islam mosque was

enlarged by Iltutmish and Alauddin

Khalji The minar was built by three

Sultansndash Qutbuddin Aybak Iltutmish

and Firuz Shah Tughluq

RAUSIAS-FC19E1003 45

Q40) Answer (c)

Explanation

Under the Mughals mansabdar was

referred to an individual who held a

mansab ie rank and he received his

salary as revenue assignments called

jagirs

Q41) Ans (b)

The Quit India Movement was a

spontaneous revolt of people against

British rule

The All India Congress Committee met

at Bombay on 8 August 1942 It passed

the famous resolution Quit India and

proposed the starting of a non-violent

mass struggle under Gandhis

leadership to achieve this aim But on

the very next day Gandhi and other

eminent leaders of the Congress were

arrested The Congress was once again

declared illegal

Q42) Ans (c)

The Simon Commission refers to a

group of seven MPs from the United

Kingdom constituted to suggest

constitutional reforms for British India

The Commission consisted of only

British members headed by one of the

senior British politicians Sir John

Simon

So the people of India agitated against

the arrival of Simon Commission

Q43) Ans (a)

He was widely known for his

unfavourable opinion of the economic

consequences of the British rule in

India

In his many writings and speeches and

especially in Poverty and Un-British

Rule in India Naoroji argued that India

was too highly taxed and that its wealth

was being drained away to England

He did not interpret the ancient Indian

texts and restored the self-confidence of

Indians And also he did not stress the

need for eradication of all the social

evils before anything else

Q44) Ans (c)

In August 1932 Prime Minister

MacDonald announced his Communal

Award Great Britainrsquos unilateral

attempt to resolve the various conflicts

among Indiarsquos many communal

interests

The award which was later

incorporated into the act of 1935

expanded the separate-electorate

formula reserved for Muslims to other

minorities including Sikhs Indian

Christians Anglo-Indians Europeans

distinct regional groups Gandhi

undertook a ldquofast unto deathrdquo against

that offer which he viewed as a

nefarious British plot to divide the

Indian society

Q45) Ans (b)

In British India apart from existing

imports and exports there was also a

particular amount of money which

colonial India contributed towards

administration maintenance of the

army war expenses pensions to retired

officers and other expenses accrued by

Britain towards maintenance of her

colony These were known as Home

charges and were paid for almost

entirely by India

The Home charges was made of

following components-

- Interest payable on Indian debt

- Dividend to shareholders of East

India Company

- Funds used to support the India

Office in London

- Funds used to pay salaries and

pensions of British personnel

engaged in India

- Interest on the railways

- Civil and military charges

- Store purchases in England

Q46) Ans (b)

The Lahore session of the Indian

National Congress was held in 1929

under the Presidentship of Jawaharlal

Nehru

The Lahore session of the Indian

National Congress witnessed significant

RAUSIAS-FC19E1003 46

developments in the Indian national

movement

- First the election of Jawaharlal

Nehru to the post of Presidentship of

the Congress was a clear indication

of the growing strength of the

Leftists in the Congress

- Secondly it was in this session that

the Congress for the first time raised

the demand for complete

independence Such demand was

not raised from the Congress

platform earlier

Q47) Ans (b)

It did not provide for separate

electorates for any community or

weightage for minorities However it did

allow for the reservation of minority

seats in provinces having minorities of

at least ten per cent but this was to be

in strict proportion to the size of the

community

There was no provision for complete

Independence for India

Q48) Ans (c)

The religion of early Vedic Aryans was

primarily of worship of nature and

Yajnas

The early Aryan religion was kind of

nature worship Actually the forces

around them which they could not

control or understand were invested

with divinity and were personified as

male or female gods And they

performed some Yajnas also

Q49) Ans (b)

The roads and river-routes were not

immune from robbery It is notable that

Yuan Chwang (Hiuen Tsang) was

robbed of his belongings during

Harshvardanarsquos period

Q50) Ans (c)

Q51) Ans (b)

Purandara Dasa was a saint and great

devotee of Lord Krishna

There is much speculation about where

Purandara Dasa regarded as the

Pitamaha of Carnatic music was born

Recently an expert committee

constituted by the Kannada University

Hampi has come to the conclusion that

Kshemapura Shivamogga district

Karnataka is the birth place of

Purandara Dasa

Q52) Ans (c)

Sri Tyagaraja Sri Shyama Shastry and Sri Muthuswami Dikshitar are considered the trinity of Carnatic music and with them came the golden age in Carnatic music in the 18th-19th

century

Q53) Ans d)

Recently a rare sarcophagus (stone

coffin) which is 2000 years old from the

Iron AgendashMegalithic era was discovered

from a rock-cut cave at Viyur village of

Kollam near Koyilandy in Kozhikode

district Kerala

The coffin containing bone fragments

was found during an excavation ldquoSo

far such a rare finding has been

discovered only from two sites

in Kerala Both these sarcophagi were

recovered from Megalithic sites at

Chevayur and Atholi also in Kozhikode

district

Q54) Ans a)

The megalithic culture in South India was a full-fledged Iron Age culture

Q55) Ans d)

The Cholas Pandyas and Keralaputras

(Cheras) mentioned in Ashokan

inscriptions were probably in the late

megalithic phase of material culture

Q56) Ans d)

Q57) Ans (b)

Raj Kumar Shukla followed Gandhiji all

over the country to persuade him to

come to Champaran to investigate the

problem associated with tinkathia

system

RAUSIAS-FC19E1003 47

Brij Kishore Rajendra Prasad Mahadev

Desai and Narhari Parikh accompanied

Gandhi ji during the Champaran

Satyagraha

Q58) Ans (b)

The Satvahanas started the practice of granting tax-free villages to brahmanas and Buddhist monks

Q59) Ans c)

The objectives of the Programme are

listed as under

- Developing basic tourism

infrastructure

- Promoting cultural and heritage

value of the country to generate

livelihoods in the identified regions

- Enhancing the tourist attractiveness

in a sustainable manner by

developing world-class

infrastructure at the heritage

monument sites

- Creating employment through active

involvement of local communities

- Harnessing tourism potential for its

effects on employment generation

and economic development

- Developing sustainable tourism

infrastructure and ensuring proper

Operations and maintenance

therein

Q60) Ans (b)

The Tribal Cooperative Marketing

Development Federation of India

(TRIFED) came into existence in 1987

It is a national-level apex organization

functioning under the administrative

control of Ministry of Tribal Affairs

Govt of India

TRIFED has its registered and Head

Office located in New Delhi

Q61) Ans (c)

Premchandrsquos novels include

Premashram Rangabhumi Ghaban

Karmabhumi and Godan

Gora is a novel written by Rabindranath

Tagore

138th birth anniversary of Munshi

Premchand was celebrated across the

country

Q62) Ans (b)

Giddha is a traditional pastoral dance

performed by the women of the Punjab

India and Pakistan at festival times

and at the sowing and reaping of the

harvest

By this dance the Punjabi women

reveal their joy expel their suppressed

feelings in a male dominated society

through the performance of Giddha

Since this dance has nothing to do with

men only women can participate in it

During the Teej celebrations Giddha

dance is celebrated in Punjab every

year Teej is a generic name for a

number of festivals that are celebrated

by women in some parts of India

Q63) Ans (a)

Dara Shukoh wrote the remarkable

work called ldquoMajma-ul-Bahrainrdquo or the

ldquoThe confluence of two seasrdquo

The Vice President of India Shri M

Venkaiah Naidu has said that Prince

Dara Shukohrsquos writings can come as a

refreshing source for infusing peace and

harmony He was addressing the

gathering after visiting the exhibition

that showcases the forgotten Prince of

yesteryears Dara Shukoh organized by

Mr Francois Gautier at Indira Gandhi

National Centre for the Arts in New

Delhi

Q64) Ans (c)

The statue Gommateshwara is

dedicated to the Jain God Bahubali

It is a monolithic statue

President Ram Nath Kovind

inaugurated the grand anointing

ceremony mdash Mahamastakabhisheka mdash

held once in 12 years at

Shravanabelagola (Karnataka)

Q65) Ans (c)

Prachi Valley had come up around the

Prachi river Prachi Valley gradually

disappeared

RAUSIAS-FC19E1003 48

The Prachi river originates from

Bhubaneswar

It is a tributary of the Mahanadi and

flows through the districts of Puri

Khurda Cuttack and Jagatsinghpur

and the entire region of the river is

termed as the Prachi Valley

It falls into the Bay of Bengal

Archaeological evidence shows that the

Prachi Valley Civilisation predates both

Harappa and Mohenjo-Daro

The Prachi river originates from

Bhubaneswar

Q66) Ans (d)

These monuments are located in

Chhatarpur district Madhya Pradesh

within Vindhya mountain range

Q67) Ans (a)

The book lsquoThoughts on Pakistanrsquo was

written by Dr BR Ambedkar

On the occasion of the birth anniversary

of Dr BR Ambedkar the president of

India pays homage to this icon of India

In 1924 he founded the Depressed

Classes Institute (Bahishkrit Hitkarini

Sabha) and in 1927 the Samaj Samata

Sangh

Another area of attention for Ambedkar

was education For its spread among

the low classes he set up a network of

colleges by the name of Peoples

Education Society and founded hostels

Q68) Ans(b)

Mehrgarh is a famous Neolithic

settlement in the Indian subcontinent

which is situated in Baluchistan

province Pakistan

A pre-historic rock art site is discovered

in the vast expanse of limestone blocks

on the eastern banks of Naguleru river

near Dachepalli (Andhra Pradesh) It

has thrown light on the Neolithic

civilisation that flourished in Guntur

(Andhra Pradesh) during 1500-2000

BC

Q69) Ans (c)

The 12th and the 13th centuries saw

the emergence of the Kakatiyas They

were at first the feudatories of the

Western Chalukyas of Kalyana Initially

they ruled over a small territory near

Warangal (Telangana)

They introduced Nayakships which was

later adopted and developed by the

Rayas of Vijayanagara

Q70) Ans (a)

The fast had effect of putting pressure

on mill owners who finally agreed to

give the workers a 35 per cent increase

in wages

Google celebrated with a doodle the

132nd birth anniversary of Anasuya

Sarabhai who played a pioneering role

in Indiarsquos labour movement

Q71) Ans (d)

The UNESCOrsquos list of the representative

list of the intangible cultural heritage of

humanity from India are

- Koodiyattam Sanskrit Theatre of

Kerala

- Mudiyettu ritual theatre and dance

drama of Kerala

- Tradition of Vedic Chanting

- Kalbelia folk songs and dances of

Rajasthan

- Ramlila Traditional Performance of

the Ramayana

- Sankirtana ritual singing

drumming and dancing of Manipur

- Ramman religious festival and

ritual theatre of the Garhwal

Himalayas India

- Traditional brass and copper craft of

utensil making among the Thatheras

of Jandiala Guru Punjab India

- Chhau dance classical Indian dance

originated in the eastern Indian

states

- Buddhist chanting of Ladakh

recitation of sacred Buddhist texts

in the trans-Himalayan Ladakh

region Jammu and Kashmir India

- Yoga

- Nouroz

- Kumbh Mela

RAUSIAS-FC19E1003 49

Q72) Ans(b)

The President of India Shri Ram Nath Kovind inaugurated the Hornbill Festival and State Formation Day celebrations of Nagaland in Kisama

The festival is named after the Indian hornbill the large and colourful forest bird which is displayed in the folklore of most of the states tribes

The major recognized tribes of Nagaland are Angami Ao Chakhesang Chang

Kuki Rengma and Zeling etc

Onge Jarawa and Sentinelese are the

tribes of Andman amp Nicobar Islands

Q73) Ans (c)

The Rashtrakutas rule in the Deccan lasted for almost two hundred years till the end of the tenth century The Rashtrakutas rulers were tolerant in their religious views and patronized not only Shaivism and Vaishnavism but

Jainism as well

The famous rock-cut temple of Shiva at Ellora was built by one of the Rashtrakutas kings Krishna I in the ninth century His successor Amoghavarsha was a Jain but he also

patronized other faiths

The Rashtrakutas allowed Muslims traders to settle and permitted Islam to

be preached in their dominions

Recently increasing defacement at the prehistoric rock paintings of Pandavulagutta Telangana has created a cause for grave concern It can spoil

the prehistoric rock

Pandavulagutta is home to

- Painted rock shelters dating to

10000 BC-8000 BC

- An 8th century inscription of the

Rashtrakuta period and

- Painted frescoes from the 12th century Kakatiya empire

Q74) Ans (b)

In 1828 Raja Ram Mohan Roy founded a new religious society the Brahma Sabha later known as the Brahmo

Samaj

Debendranath Tagore headed the Tattvabodhini Sabha which was

engaged in search of spiritual truth

Its purpose was to purify Hinduism and to preach monotheism or belief in one God

The new society was to be based on the twin pillars of reason and the Vedas and

Upanishads

Recently Sadharan Brahmo Samaj (SBS) has entered into a legal battle with the West Bengal government due

to some legal issue

Q75) Ans (c)

The Chishti order was established in India by Khwaja Moinuddin Chishti who came to India around 1192 The Chishtirsquos are considered to be the most influential of the groups of Sufis who migrated to India in the late twelfth century They adapted successfully to the local environment and adopted several features of Indian devotional

traditions

The historical dargah of Sufi mystic Khwaja Moinuddin Chishti in Ajmer is all set to get a facelift This 13 th century dargah has been included among the Swachh Iconic Places a clean-up initiative focused on iconic

heritage spiritual and cultural places

Page 11: GENERAL STUDIES (PAPER I) · Test is part of Rau’s IAS Test series for Preliminary Exam 2019 FOUNDATION + CURRENT AFFAIRS GENERAL STUDIES (PAPER –I) FOUNDATION TEST –III TOPIC:

RAUSIAS-FC19E1003 11

Q26) Which of the following architectural

elements were only part of Hindu

temple architecture

1 Shikhara

2 Mandapa

3 Pradakshina patha

4 Garbhagriha

Select the correct answer using the code

given below

(a) 1 3 and 4 only

(b) 2 3 and 4 only

(c) 1 2 and 4 only

(d) 1 2 3 and 4

Q27) Which of the following temples isare

made of bricks

1 Deogarh Temple

2 Bhitargaon Temple

3 Lakshmana temple Sirpur

4 Brihadeshvara Temple

Select the correct answer using the code

given below

(a) 1 2 and 3 only

(b) 2 3 and 4 only

(c) 1 3 and 4 only

(d) 1 2 3 and 4

Q28) Which of the following statements

isare correct

1 Sultan Muhammad Quli Qutab

Shah was a contemporary of

Akbar

2 In the field of architecture

Muhammad Quli Qutab Shah

constructed many buildings the

most famous of which is the Char

Minar

Select the correct answer using the code

given below

(a) 1 only

(b) 2 only

(c) Both 1 and 2

(d) Neither 1 nor 2

Q29) Consider the following pairs

1 Manimekalai Sattanar

2 Abhijnana Shakuntalam Kalidasa

3 Silappadikaram Kovalan

Which of the pairs given above isare

correct

(a) 1 and 2 only

(b) 2 only

(c) 1 and 3 only

(d) 1 2 and 3

Q30) Which of the following statements

isare correct

1 Sushruta Samhita is an important

work on medicine

2 Brahmagupta and Charaka were

important mathematicians

Select the correct answer using the code

given below

(a) 1 only

(b) 2 only

(c) Both 1 and 2

(d) Neither 1 nor 2

Q31) Which of the following statements

isare correct about Amir Khusrau

1 Amir Khusrau records in his works

that Sanskrit did not belong to any

region and only the Brahmans

knew it

2 He recorded the existence of

Hindawi and Awadhi

Select the correct answer using the code

given below

(a) 1 only

(b) 2 only

(c) Both 1 and 2

(d) Neither 1 nor 2

RAUSIAS-FC19E1003 12

Q32) निमननिखित कथि ो पर निचार कीनजए

1 नहरणय-गभण अिषठाि क बार म ऐसा स चा जाता

था नक बनि दि िाि का एक कषनतरय क रप म

पिजणनम ह गा

2 मयरशमणि कदोब िोश का सोसथापक था

उपयणकत कथि ो म स कौि-सास सही हह

(a) किि 1

(b) किि 2

(c) 1 और 2 द ि ो

(d) ि त 1 ि ही 2

Q33) निमननिखित कथि ो म स कौि-सास सही हह

1 कदमई बगार (बिपिणक शरम) क रप म

निया जाि िािा कर था

2 गवानियर परशखसत म िागभट (ज एक चोदि

राजा था) क दवारा नकय गए श िर का िरणि

नकया गया ह

िीच नदए गए कट का परय ग कर सही उततर चनिए

(a) किि 1

(b) किि 2

(c) 1 और 2 द ि ो

(d) ि त 1 ि ही 2

Q34) निमननिखित कथि ो म स कौि-सास सही हह

1 राजतरो नगिी 11िी ो शताबदी म कलहि क दवारा

रनचत एक सोसकत पसतक (टकसट) ह

2 कननौज क निए नतरपकषीय सोघिण म पाि राजिोश

शानमि था

िीच नदए गए कट का परय ग कर सही उततर चनिए

(a) किि 1

(b) किि 2

(c) 1 और 2 द ि ो

(d) ि त 1 ि ही 2

Q35) निमननिखित यग ो पर निचार कीनजए

1 बरहदशवर मोनदर राजराजा च ि

2 उर मापि की इकाई

3 दिदाि मोनदर ो क भनम अिदाि

उपयणकत यग ो म स कौि-स सही समनित ह

(a) किि 1 और 2

(b) किि 2 और 3

(c) किि 1 और 3

(d) 1 2 और 3

Q36) निमननिखित कथि ो म स कौि-सास सही हह

1 नदलली क सलताि ो क अरीि परशासि की भािा

फारसी थी

2 नदलली सलतित म ldquoतारीितािरीिrdquo कनिता

का एक रप था

िीच नदए गए कट का परय ग कर सही उततर चनिए

(a) किि 1

(b) किि 2

(c) 1 और 2 द ि ो

(d) ि त 1 ि ही 2

Q37) निमननिखित कथि ो म स कौि-सास सही हह

1 अिाउददीि खििजी ि अपि सनिक ो क निए

नसरी िाम का एक िया दगणरकषक शहर

बिािाया था

2 िह अपि सनिक ो क िति का भगताि इकता

क रप म करता था

िीच नदए गए कट का परय ग कर सही उततर चनिए

(a) किि 1

(b) किि 2

(c) 1 और 2 द ि ो

(d) ि त 1 ि ही 2

RAUSIAS-FC19E1003 13

Q32) Consider the following statements

1 Hiranya-garbha ritual was thought

to lead to the rebirth of the

sacrificer as a Kshatriya

2 Mayurasharman was the founder

of the Kadamba dynasty

Which of the statements given above

isare correct

(a) 1 only

(b) 2 only

(c) Both 1 and 2

(d) Neither 1 nor 2

Q33) Which of the following statements

isare correct

1 Kadamai was tax taken in form of

forced labour

2 Gwalior Prashasti describes the

exploits of Nagabhata who was a

Chandella king

Select the correct answer using the code

given below

(a) 1 only

(b) 2 only

(c) Both 1 and 2

(d) Neither 1 nor 2

Q34) Which of the following statements

isare correct

1 Rajatarangini is a Sanskrit text

written by Kalhana in the 11th

century

2 Pala dynasty was included in the

tripartite struggle for Kannauj

Select the correct answer using the code

given below

(a) 1 only

(b) 2 only

(c) Both 1 and 2

(d) Neither 1 nor 2

Q35) Consider the following pairs

1 Brihadeshvara temple Rajaraja

Chola

2 ldquoUrrdquo Unit of measurement

3 Devadana Land grants made to

temples

Which of the pairs given above isare

correct

(a) 1 and 2 only

(b) 2 and 3 only

(c) 1 and 3 only

(d) 1 2 and 3

Q36) Which of the following statements

isare correct

1 The language of administration

under the Delhi Sultans was

Persian

2 Tarikhtawarikh was a form of

poetry in the Delhi Sultanate

Select the correct answer using the code

given below

(a) 1 only

(b) 2 only

(c) Both 1 and 2

(d) Neither 1 nor 2

Q37) Which of the following statements

isare correct

1 Alauddin Khilji constructed a new

garrison town named Siri for his

soldiers

2 He paid his soldiers their salaries

in the form of Iqta

Select the correct answer using the code

given below

(a) 1 only

(b) 2 only

(c) Both 1 and 2

(d) Neither 1 nor 2

RAUSIAS-FC19E1003 14

Q38) निमननिखित कथि ो म स कौि-सास सही हह

1 नदलली कतबददीि एबक क अरीि पहिी बार

नकसी सामराजय की राजरािी बिी थी

2 दहिीिाि नसक ो का मदरर मग़ि ो क दवारा

नकया गया था

िीच नदए गए कट का परय ग कर सही उततर चनिए

(a) किि 1

(b) किि 2

(c) 1 और 2 द ि ो

(d) ि त 1 ि ही 2

Q39) निमननिखित यग ो पर निचार कीनजए

1 म ठ की मखिद नसको दर ि दी

2 बगमपरी मखिद नफर ज शाह तगिक

3 कववत- अि - इसलाम कतबददीि ऐबक

उपयणकत यग ो म स कौि-स सही समनित ह

(a) किि 1 और 2

(b) किि 2 और 3

(c) किि 1 और 3

(d) 1 2 और 3

Q40) निमननिखित कथि ो म स कौि-सास सही हह

1 मिसबदार ो क अपिा िति राजसव कायो

नजन जागीर कहत थ क रप म परापत ह ता

था

2 मिसबदार क ज सनय उततरदानयतव सौोप जात

थ उसक अनतगणत उस एक निराणररत सखया म

सिार अथिा घड़सिार ो का रि-रिाि करिा

पड़ता था

िीच नदए गए कट का परय ग कर सही उततर चनिए

(a) किि 1

(b) किि 2

(c) 1 और 2 द ि ो

(d) ि त 1 ि ही 2

Q41) ldquo1942 क भारत छ ड़ आोद ििrdquo क बार म

निमननिखित अिि कि ो म स कौि-सा सतय िही ो ह

(a) यह एक अनहोसक आोद िि था

(b) इसका िततव महातमा गाोरी क दवारा नकया गया

था

(c) यह एक सवाभानिक आोद िि था

(d) इसि सामानयतया शरनमक िगण क आकनिणत

िही ो नकया था

Q42) भारत क ि ग ो ि ldquoसाइमि कमीशिrdquo क आगमि क

निरदध आोद िि नकया था कय ोनक

(a) भारतीय कभी भी 1919 क अनरनियम (The

Act of 1919) क काम की समीकषा िही ो करिा

चाहत थ

(b) साइमि कमीशि ि पराोत ो म दवर (द हर) शासि

क समापत करि की नसफाररश की थी

(c) साइमि कमीशि म क ई भारतीय सदसय िही ो

था

(d) साइमि कमीशि ि दश क निभाजि का

सझाि नदया था

Q43) निमननिखित कथि ो पर निचार कीनजए

भारतीय राषटर ीय आोद िि म दादाभाई िौर जी क दवारा

नकया गया सबस परभािी य गदाि यह था नक उन ोि

1 अोगरज ो क दवारा भारत क आनथणक श िर का

ििासा नकया था

2 पराचीि भारतीय गरोथ ो की वयाखया की थी और

भारतीय ो क आतमनिशवास क पिःसथानपत नकया

था

3 अनय नकसी भी बात स पहि सभी सामानजक

बराइय ो क उनमिि की आिशयकता पर बि

नदया था

उपयणकत कथि ो म स कौि-सास सही हह

(a) किि 1

(b) किि 2 और 3

(c) किि 1 और 3

(d) 1 2 और 3

RAUSIAS-FC19E1003 15

Q38) Which of the following statements

isare correct

1 Delhi first became the capital of a

kingdom under Qutubuddin

Aibak

2 Dehliwal coins were minted by the

Mughals

Select the correct answer using the code

given below

(a) 1 only

(b) 2 only

(c) Both 1 and 2

(d) Neither 1 nor 2

Q39) Consider the following pairs

1 Moth ki Masjid- Sikander Lodi

2 Begumpuri mosque- Firuz Shah

Tughluq

3 Quwwat al ndash Islam- Qutubuddin

Aibak

Which of the above pairs isare correct

(a) 1 and 2 only

(b) 2 and 3 only

(c) 1 and 3 only

(d) 1 2 and 3

Q40) Which of the following statements

isare correct

1 Mansabdars received their salaries

as revenue assignments called

jagirs

2 The mansabdarrsquos military

responsibilities required him to

maintain a specified number of

sawar or cavalrymen

Select the correct answer using the code

given below

(a) 1 only

(b) 2 only

(c) Both 1 and 2

(d) Neither 1 nor 2

Q41) Which one of the following observations

is not true about the Quit India

Movement of 1942

(a) It was a non-violent movement

(b) It was led by Mahatma Gandhi

(c) It was a spontaneous movement

(d) It did not attract the labour class

in general

Q42) The people of India agitated against the

arrival of the Simon Commission

because

(a) Indians never wanted the review of

the working of the Act of 1919

(b) Simon Commission recommended

the abolition of dyarchy in the

Provinces

(c) there was no Indian member in the

Simon Commission

(d) the Simon Commission suggested

the partition of the country

Q43) Consider the following statements

The most effective contribution made by

Dadabhai Naoroji to the cause of Indian

National Movement was that he-

1 exposed the economic exploitation

of India by the British

2 interpreted the ancient Indian

texts and restored the self-

confidence of Indians

3 stressed the need for eradication of

all the social evils before anything

else

Which of the statements given above

isare correct

(a) 1 only

(b) 2 and 3 only

(c) 1 and 3 only

(d) 1 2 and 3

RAUSIAS-FC19E1003 16

Q44) महातमा गाोरी ि 1932 म आमरर अिशि नकया था

कय ोनक

(a) ldquoग िमज सममििrdquo (The Round Table

Conference) भारतीय राजिीनतक

आकाोकषाओो क परा करि म असफि रहा था

(b) काोगरस और मखसलम िीग म मतभद थ

(c) रामस मकड िालड (Ramsay Macdonald)

ि ldquoसाोपरदानयक परसकारrdquo (The Communal

Award) की घ िरा की थी

(d) ldquoसनििय अिजञा आोद ििrdquo (The Civil

Disobedience Movement) असफि रहा

था

Q45) भारत म औपनििनशक शासि की अिनर क सोदभण म

भारत स रि क बनहगणमि का एक महतवपरण भाग गह

शलक (Home Charges) था निमननिखित म स

कौि-सास क ि गह शलक म सखममनित नकया गया

थानकय गए थ

1 िोदि म भारत कायाणिय क निए उपय ग नकय

जाि िािा क ि

2 भारत म नियकत नबरनटश कनमणय ो क िति और

पशि का भगताि करि क निए उपय ग नकय

जाि िािा क ि

3 अोगरज ो क दवारा भारत क बाहर यदध ो क निए

उपय ग नकय जाि िािा क ि

िीच नदए गए कट का परय ग कर सही उततर चनिए

(a) किि 1

(b) किि 1 और 2

(c) किि 2 और 3

(d) 1 2 और 3

Q46) सवतोतरता आोद िि क इनतहास म भारतीय राषटर ीय

काोगरस का 1929 का सतर महतवपरण ह कय ोनक इसम

(a) काोगरस क उददशय क रप म सथािीय सरकार

की पराखपत की घ िरा की गई थी

(b) परण सवराज की पराखपत क काोगरस क िकषय क

रप म अपिाया गया था

(c) असहय ग आोद िि शर नकया गया था

(d) िोदि म ldquoग ि मर सममििrdquo (The Round

Table Conference) म भाग िि का निरणय

निया गया था

Q47) भारतीय सवतोतरता सोगराम क सोदभण म िहर ररप टण

क दवारा निमननिखित म स नकसकी नसफाररश की गई

थीनकिकी नसफाररश की गई थी ो

1 भारत क निए परण सवतोतरता

2 अलपसोखयक ो क निए सीट ो क आरकषर क

निए सोयकत नििाणचक मोडि

3 सोनिराि म भारत क ि ग ो क निए मौनिक

अनरकार ो का परािराि

िीच नदए गए कट का परय ग कर सही उततर चनिए

(a) किि 1

(b) किि 2 और 3

(c) किि 1 और 3

(d) 1 2 और 3

Q48) आरो नभक िनदक आयो का रमण मखय रप स था

(a) भखकत

(b) मनतण पजा और यजञ

(c) परकनत की पजा और यजञ

(d) परकनत की पजा और भखकत

RAUSIAS-FC19E1003 17

Q44) Mahatma Gandhi undertook fast unto

death in 1932 mainly because

(a) The Round Table Conference failed

to satisfy Indian political

aspirations

(b) The Congress and Muslim League

had differences of opinion

(c) Ramsay Macdonald announced the

Communal Award

(d) The Civil Disobedience Movement

failed

Q45) With reference to the period of colonial

rule in India ldquoHome Chargesrdquo formed

an important part of drain of wealth

from India Which of the following funds

constituted ldquoHome Chargesrdquo

1 Funds used to support the India

Office in London

2 Funds used to pay salaries and

pensions of British personnel

engaged in India

3 Funds used for waging wars

outside India by the British

Select the correct answer using the code

given below

(a) 1 only

(b) 1 and 2 only

(c) 2 and 3 only

(d) 1 2 and 3

Q46) The 1929- Session of Indian National

Congress is of significance in the history

of the Freedom Movement because the-

(a) attainment of Self-Government

was declared as the objective of

the Congress

(b) attainment of Poorna Swaraj was

adopted as the goal of the

Congress

(c) Non-Cooperation Movement was

launched

(d) decision to participate in the

Round Table Conference in

London was taken

Q47) With reference to the period of Indian

freedom struggle which of the following

waswere recommended by the Nehru

report

1 Complete Independence for India

2 Joint electorates for reservation of

seats for minorities

3 Provision of fundamental rights for

the people of India in the

Constitution

Select the correct answer using the code

given below

(a) 1 only

(b) 2 and 3 only

(c) 1 and 3 only

(d) 1 2 and 3

Q48) The religion of the early Vedic Aryans was primarily of

(a) Bhakti

(b) image worship and Yajnas

(c) worship of nature and Yajnas

(d) worship of nature and Bhakti

RAUSIAS-FC19E1003 18

Q49) भारत की यातरा करि िाि चीिी यातरी यआि चिाोग

(हयएि साोग) ि समकािीि भारत की सामानय

खसथनतय ो और सोसकनत क दजण नकया था इस सोदभण म

निमननिखित कथि ो म स कौि-सास सही हह

1 सड़क और िदी-मागण (जि-मागण) डकती स

परण रप स सरनकषत थ

2 जहा तक अपरार ो क निए दणड की बात ह

उसक निए नकसी भी वयखकत की निदोिता

अथिा उसक अपरार क निराणररत करि क

निए अनि जि और निि परि क माधयम क

सारि थ

3 वयापाररय ो क घाट ो और परनतबोर सटशि ो पर

शलक ो का भगताि करिा पड़ता था

िीच नदए गए कट का परय ग कर सही उततर चनिए

(a) किि 1

(b) किि 2 और 3

(c) किि 1 और 3

(d) 1 2 और 3

Q50) नसोर घाटी सभयता क सोदभण म निमननिखित कथि ो पर

निचार कीनजए

1 यह मखय रप स एक रमणनिरपकष सभयता थी

तथा हािाोनक इसम रानमणक ततव मौजद था

िनकि िह परनतिश पर हािी िही ो था

2 इस काि क दौराि भारत म कपास का परय ग

कपड़ा बिाि क निए नकया जाता था

उपयणकत कथि ो म स कौि-सास सही हह

(a) किि 1

(b) किि 2

(c) 1 और 2 द ि ो

(d) ि त 1 ि ही 2

Q51) परोदर दास क सोदभण म निमननिखित कथि ो पर निचार

कीनजए

1 परोदर दास एक सोत और भगिाि नशि क एक

महाि भकत थ

2 ि एक सोगीतकार गायक और किाणटक सोगीत

क मखय सोसथापक-परसतािक ो म स एक थ

उपयणकत कथि ो म स कौि-सास सही हह

(a) किि 1

(b) किि 2

(c) 1 और 2 द ि ो

(d) ि त 1 ि ही 2

Q52) निमननिखित म स कौि-सास वयखकत किाणटक सोगीत

की नतरमनतण म शानमि हह

1 बािामरिी कषणा

2 शरी शयाम शासतरी

3 शरी मथसवामी दीनकषतर

िीच नदए गए कट का परय ग कर सही उततर चनिए

(a) किि 1

(b) किि 2

(c) किि 2 और 3

(d) 1 2 और 3

Q53) चियर (Chevayur) और अथ िी (Atholi) म खसथत

महापािार सथि निमननिखित म स नकस राजय म खसथत

(a) तनमििाड

(b) किाणटक

(c) पनिम बोगाि

(d) करि

RAUSIAS-FC19E1003 19

Q49) The Chinese traveller Yuan Chwang

(Hiuen Tsang) who visited India

recorded the general conditions and

culture of India at that time In this

context which of the following

statements isare correct

1 The roads and river-routes were

completely immune from robbery

2 As regards punishment for

offences ordeals by fire water and

poison were the instruments for

determining the innocence or guilt

of a person

3 The tradesmen had to pay duties

at ferries and barrier stations

Select the correct answer using the code

given below

(a) 1 only

(b) 2 and 3 only

(c) 1 and 3 only

(d) 1 2 and 3

Q50) Regarding the Indus Valley Civilization

consider the following statements

1 It was predominantly a secular

civilization and the religious

element though present did not

dominate the scene

2 During this period cotton was

used for manufacturing textiles in

India

Which of the statements given above

isare correct

(a) 1 only

(b) 2 only

(c) Both 1 and 2

(d) Neither 1 nor 2

Q51) Consider the following statements

regarding Purandara Dasa

1 Purandara Dasa was a saint and

great devotee of Lord Shiva

2 He was a composer singer and

one of the chief founding-

proponents of the Carnatic music

Which of the statements given above

isare correct

(a) 1 only

(b) 2 only

(c) Both 1 and 2

(d) Neither 1 nor 2

Q52) Which of the following persons isare

included in the trinity of Carnatic

music

1 Balamurali Krishna

2 Sri Shyama Shastry

3 Sri Muthuswami Dikshitar

Select the correct answer using the code

given below

(a) 1 only

(b) 2 only

(c) 2 and 3 only

(d) 1 2 and 3

Q53) Megalithic sites at Chevayur and Atholi

are located in which of the following

states

(a) Tamil Nadu

(b) Karnataka

(c) West Bengal

(d) Kerala

RAUSIAS-FC19E1003 20

Q54) निमननिखित कथि ो पर निचार कीनजए

1 महापािानरक ि ग कबर ो म िसतएो दफिात थ

2 दनकषर भारत म महापािार सोसकनत एक परण

निकनसत तामर यगीि सोसकनत थी

उपयणकत कथि ो म स कौि-सास सही हह

(a) किि 1

(b) किि 2

(c) 1 और 2 द ि ो

(d) ि त 1 ि ही 2

Q55) निमननिखित म स कौि-स सामराजयसामराजय ो का

अश क क अनभिि ो म उललि नकया गया ह

1 च ि

2 पाणडय

3 करिपतर (चर)

िीच नदए गए कट का परय ग कर सही उततर चनिए

(a) किि 1

(b) किि 1 और 2

(c) किि 3

(d) 1 2 और 3

Q56) भीमा-क रगाोि का यदध को पिी क सनिक ो और

बाजीराि नदवतीय क िततव म एक शखकतशािी पशिा

सिा (मराठ ो) क मधय िड़ा गया था यह यदध

निमननिखित म स नकसका नहससा था

(a) परथम आोगल-मराठा यदध का

(b) नदवतीय आोगल-मराठा यदध का

(c) ततीय आोगल-मसर यदध का

(d) ततीय आोगल-मराठा यदध का

Q57) निमननिखित कथि ो पर निचार कीनजए

1 महादि दसाई ि गाोरीजी क चोपारर आि तथा

नतिकनथया पररािी स जड़ी समसया की जाोच

क निए रारी करि क निए दश भर म उिका

अिसरर नकया था

2 िरहरी पाररि चोपारर सतयागरह क दौराि

गाोरीजी क साथ थ

उपयणकत कथि ो म स कौि-सास सही हह

(a) किि 1

(b) किि 2

(c) 1 और 2 द ि ो

(d) ि त 1 ि ही 2

Q58) निमननिखित कथि ो पर निचार कीनजए

1 िनद राज-िोश ि बराहमर ो और बौदध मठराररय ो

क कर-मकत गाि अिदाि म दि की परथा

आरि की थी

2 सतिाहि ो की आनरकाररक भािा पराकत थी

उपयणकत कथि ो म स कौि-सास सही हह

(a) किि 1

(b) किि 2

(c) 1 और 2 द ि ो

(d) ि त 1 ि ही 2

Q59) एक निरासत क अपिाइए (अडॉपट ए हररटज ndash

Adopt a Heritage) पररय जिा क उददशय ो क

सनदभण म निमननिखित कथि ो पर निचार कीनजए

1 यह पररय जिा र रगार उतपादि और आनथणक

निकास क निए पयणटि कषमता का उि पर

परभाि का उपय ग करगी

2 यह पररय जिा निरासत सथि ो पर निशव सतरीय

आराररक सोरचिा निकनसत करक एक सतत

तरीक स पयणटक आकिणर म िखदध करगी

उपयणकत कथि ो म स कौि-सास सही हह

(a) किि 1

(b) किि 2

(c) 1 और 2 द ि ो

(d) ि त 1 ि ही 2

RAUSIAS-FC19E1003 21

Q54) Consider the following statements

1 Megalithic people buried goods in

graves

2 The megalithic culture in South

India was a full-fledged Copper

Age culture

Which of the statements given above

isare correct

(a) 1 only

(b) 2 only

(c) Both 1 and 2

(d) Neither 1 nor 2

Q55) Which of the following kingdoms isare

mentioned in the Ashokan inscriptions

1 Cholas

2 Pandyas

3 Keralaputras (Cheras)

Select the correct answer using the code

given below

(a) 1 only

(b) 1 and 2 only

(c) 3 only

(d) 1 2 and 3

Q56) The Battle of Bhima-Koregaon was

fought between the soldiers of the

Company and the strong Peshwa army

(Marathas) under Bajirao II This war

was a part of the

(a) First Anglo-Maratha war

(b) Second Anglo-Maratha war

(c) Third Anglo- Mysore war

(d) Third Anglo-Maratha war

Q57) Consider the following statements

1 Mahadev Desai followed Gandhiji all over the country to persuade him to come to Champaran to investigate the problem associated

with tinkathia system

2 Narhari Parikh accompanied Gandhi ji during the Champaran

Satyagraha

Which of the statements given above isare correct

(a) 1 only

(b) 2 only

(c) Both 1 and 2

(d) Neither 1 nor 2

Q58) Consider the following statements

1 The Nanda Dynasty started the practice of granting tax-free villages to brahmanas and

Buddhist monks

2 The official language of the Satavahanas was Prakrit

Which of the statements given above

isare correct

(a) 1 only

(b) 2 only

(c) Both 1 and 2

(d) Neither 1 nor 2

Q59) Consider the following statements about the objectives of the lsquoadopt a heritagersquo

project

1 It will harness tourism potential for its effects on employment generation and economic

development

2 It will enhance the tourist attractiveness in a sustainable manner by developing world class infrastructure at heritage sites

Which of the statements given above

isare correct

(a) 1 only

(b) 2 only

(c) Both 1 and 2

(d) Neither 1 nor 2

RAUSIAS-FC19E1003 22

Q60) ldquoभारतीय जिजातीय सहकारी निपरि निकास सोघrdquo

(The Tribal Co-operative Marketing

Development Federation of India - TRIFED)

क सोदभण म निमननिखित कथि ो पर निचार कीनजए

1 यह एक राषटर ीय सतर का शीिण सोगठि ह ज

भारत सरकार क गह मोतरािय क परशासनिक

नियोतरर क अरीि काम कर रहा ह

2 इसका मखय उददशय दश म जिजातीय ि ग ो

का सामानजक-आनथणक निकास करिा ह

उपयणकत कथि ो म स कौि-सास सही हह

(a) किि 1

(b) किि 2

(c) 1 और 2 द ि ो

(d) ि त 1 ि ही 2

Q61) निमननिखित म स कौि-सास उपनयास परमचोद क

दवारा नििा गया हनिि गए ह

1 रोगभनम

2 ग दाि

3 ग रा

िीच नदए गए कट का परय ग कर सही उततर चनिए

(a) किि 1

(b) किि 2

(c) किि 1 और 2

(d) 1 2 और 3

Q62) नगदधा ितय क सोदभण म निमननिखित कथि ो पर निचार

कीनजए

1 नगदधा नबहार की मनहिाओो क दवारा तयौहार क

समय और फसि की बिाई तथा कटाई क

अिसर पर नकया जाि िािा एक पारोपररक

दहाती ितय ह

2 इस ितय क दवारा मनहिाऐो अपिी परसननता

परकट करती ह तथा नगदधा क परदशणि क

माधयम स परि िचणसव िाि समाज म

मनहिाओो की दबी हई भाििाओो क परकट

करती ह

उपयणकत कथि ो म स कौि-सास सही हह

(a) किि 1

(b) किि 2

(c) 1 और 2 द ि ो

(d) ि त 1 ि ही 2

Q63) निमननिखित कथि ो पर निचार कीनजए

1 मलला शाह बदखशी दारा नशक ह क

आधयाखतमक गर थ

2 औरोगरब ि मजम-उि-बहरीि या द समदर ो

का सोगम िामक उललििीय रचिा नििी थी

3 दारा नशक ह क अपि पिणज अकबर क गर ो

क उततरानरकारी क रप म दिा गया था

नजसम उसि रानमणक बहििाद और समनवयता

क बढ़ािा नदया था

उपयणकत कथि ो म स कौि-सास सही हह

(a) किि 1 और 3

(b) किि 2

(c) किि 1 और 2

(d) 1 2 और 3

RAUSIAS-FC19E1003 23

Q60) Consider the following statements about

the Tribal Cooperative Marketing

Development Federation of India

(TRIFED)

1 It is a national-level apex

organization functioning under the

administrative control of Ministry

of Home Affairs Government of

India

2 The main objective of TRIFED is

socio-economic development of

tribal people in the country

Which of the statements given above

isare correct

(a) 1 only

(b) 2 only

(c) Both 1 and 2

(d) Neither 1 nor 2

Q61) Which of the following novels isare

written by Premchand

1 Rangabhumi

2 Godan

3 Gora

Select the correct answer using the code

given below

(a) 1 only

(b) 2 only

(c) 1 and 2 only

(d) 1 2 and 3

Q62) Consider the following statements about

Giddha dance

1 Giddha is a traditional pastoral

dance performed by the women of

Bihar at festival times and at the

sowing and reaping of the harvest

2 By this dance the women reveal

their joy expel their suppressed

feelings in a male dominated

society through the performance of

Giddha

Which of the statements given above

isare correct

(a) 1 only

(b) 2 only

(c) Both 1 and 2

(d) Neither 1 nor 2

Q63) Consider the following statements

1 Mullah Shah Badakhshi was the

spiritual mentor of Dara Shukoh

2 Aurangzeb wrote the remarkable

work called ldquoMajma-ul-Bahrainrdquo or

the ldquoThe confluence of two seasrdquo

3 Dara Shukoh was seen as

inheriting the qualities of his

ancestor Akbar in that he

promoted religious pluralism and

syncretism

Which of the statements given above

isare correct

(a) 1 and 3 only

(b) 2 only

(c) 1 and 2 only

(d) 1 2 and 3

RAUSIAS-FC19E1003 24

Q64) निमननिखित कथि ो पर निचार कीनजए

1 ग मतशवर परनतमा निोधयनगरी पहाड़ी पर खसथत ह

2 शरिरबिग िा िह सथाि ह जहाो मौयण िोश क

सोसथापक चोदरगपत मौयण अपि नसोहासि क

तयागि क बाद जि तपसवी बि गए थ

उपयणकत कथि ो म स कौि-सास सही हह

(a) किि 1

(b) किि 2

(c) 1 और 2 द ि ो

(d) ि त 1 ि ही 2

Q65) निमननिखित कथि ो पर निचार कीनजए

1 पराताखतवक साकषय स पता चिता ह नक पराची

घाटी सभयता हड़पपा और म हिज दाड़ द ि ो

की पिणिती ह

2 पराची िदी भििशवर स निकिती ह

उपयणकत कथि ो म स कौि-सास सही हह

(a) किि 1

(b) किि 2

(c) 1 और 2 द ि ो

(d) ि त 1 ि ही 2

Q66) निमननिखित कथि ो म स कौि-सास सही हह

1 िजराह क समारक ो क समह का निमाणर

चोदि राजिोश क शासिकाि क दौराि हआ

था

2 य समारक हररिोदर पिणत शरोििा म खसथत ह

3 म रक क यातरी इबन बतता ि अपि सोसमरर ो

म िजराह क मोनदर ो की यातरा का उललि

नकया था तथा इन काजराण िाम स समब नरत

नकया था

िीच नदए गए कट का परय ग कर सही उततर चनिए

(a) किि 1

(b) किि 1 और 2

(c) किि 2 और 3

(d) किि 1 और 3

Q67) निमननिखित कथि ो म स कौि-सास सही हह

1 डॉ बी आर अमबडकर ि दी एनिनहिशि

ऑफ़ कासट (The Annihilation of Caste)

नििी थी नजसम उन ोि नहोद रमण म िोशािगत

पजारी की परथा क उनमिि की आिशयकता

पर बि नदया था

2 डॉ राजदर परसाद ि थॉटस ऑि पानकसताि

(Thoughts on Pakistan) िामक पसतक

नििी थी

िीच नदए गए कट का परय ग कर सही उततर चनिए

(a) किि 1

(b) किि 2

(c) 1 और 2 द ि ो

(d) ि त 1 ि ही 2

Q68) निमननिखित कथि ो म स कौि-सास सही हह

1 महरगढ़ भारतीय उपमहादवीप म एक परनसदध

ििपािार बसती ह ज नसोर पराोत पानकसताि म

खसथत ह

2 बरणह म म कतत ो क उिक सवामी क साथ कबर ो

म दफिाया जाता था

िीच नदए गए कट का परय ग कर सही उततर चनिए

(a) किि 1

(b) किि 2

(c) 1 और 2 द ि ो

(d) ि त 1 ि ही 2

Q69) निमननिखित कथि ो म स कौि-सास सही हह

1 काकानटय मोनदर अनरकतर नशि क समनपणत

2 हिमक ोडा म हजार-सतोभ िाि मोनदर (The

Thousand-Pillared Temple) का निमाणर

काकानटय समराट रदर ि करिाया था

िीच नदए गए कट का परय ग कर सही उततर चनिए

(a) किि 1

(b) किि 2

(c) 1 और 2 द ि ो

(d) ि त 1 ि ही 2

RAUSIAS-FC19E1003 25

Q64) Consider the following statements

1 Gommateshwara Statue is located

on the Vindyagiri Hill

2 Shravanabelagola is the place

where Chandragupta Maurya the

founder of the Mauryan dynasty

became a Jain ascetic after

relinquishing his throne

Which of the statements given above

isare correct

(a) 1 only

(b) 2 only

(c) Both 1 and 2

(d) Neither 1 nor 2

Q65) Consider the following statements

1 Archaeological evidence shows

that the Prachi Valley Civilisation

predates both Harappa and

Mohenjo-Daro

2 The Prachi river originates from

Bhubaneswar

Which of the statements given above

isare correct

(a) 1 only

(b) 2 only

(c) Both 1 and 2

(d) Neither 1 nor 2

Q66) Which of the following statements

isare correct

1 The Khajuraho group of

monuments was built during the

rule of the Chandela dynasty

2 These monuments are located in

Harischandra mountain range

3 Ibn Battuta the Moroccan

traveller in his memoirs mentioned

visiting Khajuraho temples and

called them Kajarra

Select the correct answer using the code

given below

(a) 1 only

(b) 1 and 2

(c) 2 and 3

(d) 1 and 3

Q67) Which of the following statements

isare correct

1 Dr BR Ambedkar wrote the

Annihilation of Caste emphasising

the need to do away with the

practice of hereditary priesthood in

Hinduism

2 The book lsquoThoughts on Pakistanrsquo

was written by Dr Rajendra

Prasad

Select the correct answer using the code

given below

(a) 1 only

(b) 2 only

(c) Both 1 and 2

(d) Neither 1 nor 2

Q68) Which of the following statements

isare correct

1 Mehrgarh is a famous Neolithic

settlement in the Indian

subcontinent which is situated in

Sindh province Pakistan

2 At Burzahom dogs were buried

with their masters in their graves

Select the correct answer using the code

given below

(a) 1 only

(b) 2 only

(c) Both 1 and 2

(d) Neither 1 nor 2

Q69) Which of the following statements

isare correct

1 The Kakatiya temples are

dedicated mostly to Siva

2 The Thousand-Pillared Temple at

Hanamkonda was built by the

Kakatiya king Rudra

Select the correct answer using the code

given below

(a) 1 only

(b) 2 only

(c) Both 1 and 2

(d) Neither 1 nor 2

RAUSIAS-FC19E1003 26

Q70) निमननिखित कथि ो म स कौि-सास सही हह

1 अहमदाबाद नमि हड़ताि क दौराि महातमा

गाोरी ि शरनमक ो क पकष क मजबत करि क

निए आमरर अिशि नकया था

2 अिशि स नमि मानिक ो पर दबाि पड़ा था ज

अोततः शरनमक ो क िति म 15 परनतशत की िखदध

करि क निए सहमत हए थ

िीच नदए गए कट का परय ग कर सही उततर चनिए

(a) किि 1

(b) किि 2

(c) 1 और 2 द ि ो

(d) ि त 1 ि ही 2

Q71) निमननिखित म स नकसक नकिक भारत स यिसक

की माििता की अमतण साोसकनतक निरासत की

परनतनिनर सची (The UNESCOrsquos List of the

Representative List of the Intangible

Cultural Heritage of Humanity) म शानमि

नकया गया ह

1 मनडयटट

2 सोकीतणि

3 को भ मिा

िीच नदए गए कट का परय ग कर सही उततर चनिए

(a) किि 1 और 2

(b) किि 2 और 3

(c) किि 3

(d) 1 2 और 3

Q72) निमननिखित जिजानतय ो म स कौि-सीसी ो

जिजानतजिजानतया िागािड स सोबोनरत हह

1 अोगामी

2 ककी

3 जारिा

िीच नदए गए कट का परय ग कर सही उततर चनिए

(a) किि 1

(b) किि 1 औऔ 2

(c) किि 2

(d) 1 2 और 3

Q73) निमननिखित कथि ो म स कौि-सास सही हह

1 राषटर कट सामराजय की सथापिा दोनतदगण ि की थी

नजसि मानयाित म अपिी राजरािी की

सथापिा की थी

2 राषटर कट समराट अम घििण एक ििक था और

उस कनिताओो पर पहिी कननड़ पसतक नििि

का शरय नदया जाता ह

िीच नदए गए कट का परय ग कर सही उततर चनिए

(a) किि 1

(b) किि 2

(c) 1 और 2 द ि ो

(d) ि त 1 ि ही 2

Q74) निमननिखित कथि ो म स कौि-सास सही हह

1 कशब चोदर सि ि ततवब नरिी सभा की

अधयकषता की थी ज आधयाखतमक सतय की

ि ज म सोिि थी

2 बरहम समाज ि मािि गररमा पर बि नदया

मनतणपजा का निर र नकया और सती परथा जसी

सामानजक बराइय ो की आि चिा की

िीच नदए गए कट का परय ग कर सही उततर चनिए

(a) किि 1

(b) किि 2

(c) 1 और 2 द ि ो

(d) ि त 1 ि ही 2

Q75) निमननिखित कथि ो म स कौि-सास सही हह

1 भारत म नचशती नसिनसिा खवाजा म इिददीि

नचशती क दवारा सथानपत नकया गया था

2 नचशती परोपरा की एक परमि निशिता

आतमसोयम थी नजसम साोसाररक म ह स दरी

बिाए रििा शानमि था

िीच नदए गए कट का परय ग कर सही उततर चनिए

(a) किि 1

(b) किि 2

(c) 1 और 2 द ि ो

(d) ि त 1 ि ही 2

RAUSIAS-FC19E1003 27

Q70) Which of the following statements

isare correct

1 During the Ahmedabad Mill Strike

Mahatma Gandhi undertook a fast

unto death to strengthen the

workersrsquo resolve

2 The fast had effect of putting

pressure on mill owners who

finally agreed to give the workers a

15 per cent increase in wages

Select the correct answer using the code

given below

(a) 1 only

(b) 2 only

(c) Both 1 and 2

(d) Neither 1 nor 2

Q71) Which of the following are included in

the UNESCOrsquos list of the representative

list of the intangible cultural heritage of

humanity from India

1 Mudiyettu

2 Sankirtana

3 Kumbh Mela

Select the correct answer using the code

given below

(a) 1 and 2 only

(b) 2 and 3 only

(c) 3 only

(d) 1 2 and 3

Q72) Which of the following tribes isare

related to Nagaland

1 Angami

2 Kuki

3 Jarawa

Select the correct answer using the code

given below

(a) 1 only

(b) 1 and 2 only

(c) 2 only

(d) 1 2 and 3

Q73) Which of the following statements

isare correct

1 Rashtrakuta kingdom was founded by Dantidurga who established his capital at Manyakhet

2 Amoghavarsha a Rashtrakuta king was an author and is credited with writing the first

Kannada book on poetics

Select the correct answer using the code given below

(a) 1 only

(b) 2 only

(c) Both 1 and 2

(d) Neither 1 nor 2

Q74) Which of the following statements isare correct

1 Keshab Chandra Sen headed the Tattvabodhini Sabha which was engaged in search of spiritual truth

2 The Brahmo Samaj laid emphasis on human dignity opposed idolatry and criticized such social

evils as the practice of Sati

Select the correct answer using the code given below

(a) 1 only

(b) 2 only

(c) Both 1 and 2

(d) Neither 1 nor 2

Q75) Which of the following statements isare correct

1 The Chishti order was established in India by Khwaja Moinuddin

Chishti

2 A major feature of the Chishti tradition was austerity including maintaining a distance from the

worldly power

Select the correct answer using the code

given below

(a) 1 only

(b) 2 only

(c) Both 1 and 2

(d) Neither 1 nor 2

T e s t i s p a r t o f R a u rsquo s I A S T e s t s e r i e s f o r P r e l i m i n a r y E x a m 2 0 1 9

FOUNDATION + CURRENT AFFAIRS

GENERAL STUDIES (PAPER ndashI)

FOUNDATION TEST ndashIII

SUBJECT NCERT History Class VI-X + Current Affairs

Time Allowed 1frac12 Hours Maximum Marks 150

I NSTRUCT IONS

1 IMMEDIATELY AFTER THE COMMENCEMENT OF THE EXAMINATION YOU SHOULD CHECK

THAT THIS TEST BOOKLET DOES NOT HAVE ANY UNPRINTED OR TORN or MISSING PAGES OR

ITEMS ETC IF SO GET IT REPLACED BY A COMPLETE TEST BOOKLET

2 This Test Booklet contains 75 items (questions) Each item is printed both in Hindi and English

Each item comprises four responses (answers) You will select the response which you want to mark

on the Answer Sheet In case you feel that there is more than one correct response mark the

response which you consider the best In any case choose ONLY ONE response for each item

3 You have to mark all your responses ONLY on the separate Answer Sheet (OMR sheet) provided

Read the directions in the Answer Sheet

4 All items carry equal marks

5 Before you proceed to mark in the Answer Sheet the response to various items in the Test booklet

you have to fill in some particulars in the Answer Sheet as per instructions contained therein

6 After you have completed filling in all your responses on the Answer Sheet and the examination has

concluded you should hand over to the Invigilator only the Answer Sheet You are permitted to

take away with you the Test Booklet

7 Penalty for wrong answers

THERE WILL BE PENALTY FOR WRONG ANSWERS MARKED BY A CANDIDATE IN THE

OBJECTIVE TYPE QUESTION PAPERS

(i) There are four alternatives for the answer to every question For each question for which a

wrong answer has been given by the candidate one-third of the marks assigned to that

question will be deducted as penalty

(ii) If a candidate gives more than one answer it will be treated as a wrong answer even if one of

the given answers happens to be correct and there will be same penalty as above to that

question

(iii) If a question is left blank ie no answer is given by the candidate there will be no penalty for

that question

T h i s t e s t i s p a r t o f R a u rsquo s I A S T e s t s e r i e s f o r P r e l i m i n a r y E x a m 2 0 1 9

Test Code

FC19E1003

FC19H1003 29

Answers and Explanations of

NCERT History Class VI-X + Current Affairs (FC19E1003)

Q1) उततर (c)

सपषटीकरण

- ऋगवद म दविय ो और दिताओो क समवपित एक

हजार स अविक सत तर (शल क) ह

- य शल क ऋविय ो क दवारा रच गए थ और परि ो

दवारा सीख जात थ

- हालाोवक कछ शल क मवहलाओो (जस वक अपाला

घ सा ल पामदरा मतरयी और गागी) क दवारा भी रच

गए थ

- ऋगवद म सोिाद क रप म कई शल क मौजद ह

- हम विशवावमतर नामक एक ऋवि और दविय ो क

रप म पजी जान िाली द नवदय ो (वयास और

सतलज) क बीच िाताि का उदाहरण वमलता ह

- इसस पता चलता ह वक विशवावमतर िवदक काल स

सोबोवित थ

Q2) उततर (b)

सपषटीकरण

- करनल गफाओो स राख क अिशि परापत हए ह

ज इस ओर सोकत करत ह वक ततकालीन ल ग

अवि क उपय ग स पररवचत थ

- य गफाएो आोधर परदश म सथथत ह

Q3) उततर (c)

सपषटीकरण

bull बरािह म ितिमान कशमीर म सथथत एक

परागवतहावसक थथल ह जहाो ल ग गडढ क घर ो का

वनमािण करत थ

bull य घर जमीन क ख द कर बनाए जात थ तथा नीच

जान क वलए सीवियाा ह ती थी

bull ऐसा अनमान लगाया जाता ह वक य घर ठो ड क

मौसम म आशरय परदान करत थ

Q4) उततर (c)

सपषटीकरण

bull परालख-विदया (Epigraphy) क वशलालख ो क

अधययन क रप म पररभावित वकया जाता ह

bull हसतवलसखत दसतािज ो क माधयम स इवतहास

और सावहतय क अधययन क पाोडवलवप विजञान

(Manuscriptology) कहत ह

bull पराचीन लखन परणावलय ो क अधययन और

ऐवतहावसक पाोडवलवपय ो क समझन तथा वतवथ

वनिािरण क पलीओगराफी (Palaeography) कहा

जाता ह

bull नयवमजमविकस (Numismatics) वसक ो क

अधययन क सोदवभित करता ह

Q5) उततर (a)

सपषटीकरण

- चरक सोवहता चरक क दवारा वलखी गई आयिद

और िदयक-शासर पर एक महतवपणि पसतक ह

- ि भारतीय िदयक-शासर की पारमपररक परणाली

वजस आयिद क नाम स जाना जाता ह क

अभयासकताि थ

- ऐसा माना जाता ह वक चरक का विकास दसरी

शताबदी (ईसा पिि) और दसरी शताबदी (ईसवी) क

मधय हआ था

Q6) उततर (b)

सपषटीकरण

- भाग फसल ो पर वलए जान िाल कर क सोदवभित

करता ह ज कल फसल उतपादन का 16 िाो भाग

था

- ldquoकममकारrdquo शबद भवमहीन कवि शरवमक िगि क

वलए परय ग वकया जाता था

- ldquoअशवमिrdquo (वजस घ ड क बवलदान क रप म भी

जाना जाता ह) एक अनषठान ह ता था वजसम एक

घ ड क सवतोतर रप स घमन क वलए छ ड वदया

FC19H1003 30

जाता ह और राजा क सवनक उसकी रखिाली

करत थ

Q7) उततर (d)

सपषटीकरण

- ऋगववदक काल म घ ड ो क रथ ो म ज ता जाता था

ज (रथ) भवम मिवशय ो आवद पर कबजा करन क

वलए लड गए यद ो म उपय ग वकए जात थ

- इसस यह पता चलता ह वक घ ड ो यकत रथ ो का

उपय ग महाजनपद काल स काफी पहल आरमभ

हआ था

- ऋगववदक काल म मिवशय ो भवम जल आवद पर

कबजा करन क वलए तथा ल ग ो क पकडन क

वलए यद वकय जात थ

- अविकाोश परि इन यद ो म भाग वलया करत थ

- हालाोवक उस समय क ई वनयवमत सना नही ो ह ती

थी लवकन उस काल म सभाऐो ह ती थी ो वजनम

ल ग यद क मामल ो पर चचाि करत थ

- वनयवमत सनाएा महाजनपद काल का िवशषटय थी

वजनम पदल सवनक ो की विशाल सनाएा रथ तथा

हाथी शावमल ह त थ

Q8) उततर (a)

सपषटीकरण

- बद शाकय कल स सोबोवित थ और कशीनारा म

उनका वनिन हआ था

- बद न अपनी वशकषाएा पराकत भािा म दी थी ो ज

आम ल ग ो की भािा थी

Q9) उततर (c)

सपषटीकरण

- पराचीन भारत म दशिनशासर की छह शाखाएा थी ो

िशविक नयाय समखया य ग पिि वममाोसा और

िदाोत या उततर वममाोसा

- इनकी थथापना करमश कनाद गौतम कवपल

पतोजवल जावमनी और वयास ऋविय ो न की थी

Q10) उततर (b)

सपषटीकरण

महािीर की वशकषाऐो छठी शताबदी म िललभी म

सोकवलत की गई थी ो

Q11) उततर (c)

सपषटीकरण

- पारमपररक रप स चाणकय क कौविलय अथिा

विषणगपत क नाम स जाना जाता ह

- उसन अथिशासतर ज एक पराचीन भारतीय

राजनवतक आलख ह वलखा था

Q12) उततर (d)

सपषटीकरण

- भारत का राषटर ीय वचनह सारनाथ (उततर परदश) क

अश क सतमभ क ऊपर (शीिि पर) वसोह कवपिल

का एक अनरपण ह

- इस राषटर ीय वसदाोत सतयमि जयत क साथ

सोय वजत वकया गया ह

- रामपिि बल का नाम रामपिि (वबहार) क नाम पर

पडा जहाा इसकी ख ज हई थी

- यह अपन नाजक नकाशी मॉडल क वलए परवसदद

ह वजसम क मल तवचा सोिदनशील नथन ो सतकि

कान और मरबत िााग ो क शरषठतर परवतरप क

परदवशित वकया गया ह

- यह भारतीय और फारसी ततव ो का एक ससममशरण

- सोवकससा उततर परदश म सथथत ह

Q13) उततर (a)

सपषटीकरण

का िर वसोह ज एक महान य दा थ वबहार स

सोबोवित थ

Q14) उततर (b)

सपषटीकरण

िललालर शबद बड भ-सवावमय ो क वलए परय ग

वकया जाता था

FC19H1003 31

Q15) उततर (c)

सपषटीकरण

- अररकमड एक तिीय बसती थी जहाो दर दश ो स

आन िाल जहाज ो का माल उतारा जाता था

- यहाो पर ईोि ो का एक विशाल ग दाम वमटटी क

बतिन (वजनम एमफ रा - द हरी मवठय ो का लोबा

घडा - शावमल ह) और एरिाइन (Arretine)

मदभाोड पाए गए थ

- इस थथान पर र मन दीपक काोच क बन पातर और

रतन भी पाए गए थ

Q16) उततर (a)

सपषटीकरण

- मिनदर सोगम कविताओो म उसललसखत एक

तवमल शबद ह वजसका अथि ह ldquoतीन परमखrdquo

- यह तीन सततारि पररिार ो क मसखयाओो क वलए

परय ग वकया जाता ह च ल चर और पाणडय

Q17) उततर (c)

सपषटीकरण

- ऋग िद म सभा विदाथा तथा गण जसी

जनजावतय ो पर अथिा किोब पर आिाररत

सभाओो का उललख ह

- आरसमभक िवदक काल म सभाओो और सवमवतय ो

का विशि महतव ह ता था

- यहाा तक की मसखया अथिा राजा भी उनका

समथिन परापत करन क वलए आतर रहत थ

Q18) उततर (a)

सपषटीकरण

- जन िमि न ईशवर क अससततव क मानयता त दी ह

वकनत उसन ईशवर क वजना क पद स नीच रखा

- जन िमि न बौद िमि की तरह िणि परणाली की

भरतिना नही ो की थी

Q19) उततर (d)

सपषटीकरण

- च ल ो और पाणडय ो न शसकतशाली तिीय शहर ो का

विकास वकया था

- च ल ो का सबस महतवपणि शहर पहार (या

कािरीपटटीनम) था |

- मदरई पाणडय ो की राजिानी थी

Q20) उततर (b)

सपषटीकरण

- ldquoबदचररतrdquo बद का जीिन-ितताोत ह

- इस अशवघ ि क दवारा वलखा गया था

Q21) उततर (a)

सपषटीकरणः

- तवमल कवि अपपर भगिान वशि क भकत थ

- इस परकार ि एक नयनार सोत थ

Q22) उततर (d)

सपषटीकरणः

- समदरगपत एक परवसद गपत शासक था

- उसन वसक ो पर िीणा बजात हए अपनी छवि

अोवकत करिाई थी

- यह सोगीत क परवत उसक परम क दशािता ह

- हम उसकी इलाहाबाद परशससत स महतवपणि

ऐवतहावसक जानकारी वमलती ह वजसकी रचना

उसक दरबार क कवि हररसन न की थी

Q23) उततर (b)

सपषटीकरणः

- विकरम सोित की शरआत ििि 58 ईसा पिि म

चनदरगपत वदवतीय न की थी

- यह शक ो पर उसकी जीत और उस विकरमावदतय

की पदिी वमलन क उपलकषय म आरमभ वकया गया

था

FC19H1003 32

- बानभटट न हिििििन का जीिन-ितताोत हििचररत

(ज सोसकत म थी) वलखी थी

Q24) उततर (c)

सपषटीकरणः

- सोवि-विगरावहका यद एिो शाोवत का मोतरी

- साथििाह वयापाररय ो क कावफल ो का नता

Q25) उततर (a)

सपषटीकरणः

- जआन झाोग (हसआन रताोग ndash Hsuang Tsang)

एक चीनी यातरी था ज हिििििन क शासनकाल म

भारत आया था

- ििि 630 ईसवी स ज दशक आरमभ हआ था उसम

जआन झाोग मधय एवशया ईरान और

अफग़ावनसतान की यातरा करन क पशचात कशमीर

क रासत स भारत आया था

- उसन उततर स पिि तक की यातरा की और िह

लगभग 2 ििि वबहार म रहा

- जआन झाोग न नालनदा विशवविदयालय म विदयावथिय ो

और विदवान ो क साथ पारसपररक विचार-विमशि

वकया थथानीय भािाओ ा म वनपणता परापत की तथा

बौद सतप ो की ख ज की

Q26) उततर (c)

सपषटीकरणः

- परदवकषणा पथ बौद िासतकला म सतप क चार ो

ओर बनाया जान िाला एक घमािदार पथ ह ता

- परशन म वदए गए बाकी क तीन ो ततव वहोद मसनदर ो की

िासतकला क भाग ह

Q27) उततर (d)

सपषटीकरणः

परशन म वदए गए सभी मोवदर ो म वयापक रप स

ईोि ो (पकी ईोि ो) का परय ग पतथर ो क साथ हआ

Q28) उततर (c)

सपषटीकरण

- महममद कली कतब शाह ग लकणडा का सलतान

था

- िह अकबर का समकालीन था

- सावहतय और िासतकला म उसकी अतयाविक

रवच थी

- िह एक महान कवि था

- िह दसखनी उदि फारसी और तलग म वलखता था

- उसन अपन पीछ एक विसतत वदिान (सोगरह)

छ डा ह

- अभी हाल ही म तलोगाना म ग लकणडा क वकल

क अनदर खदाई वकय गए बाग-ए-नाया वकला

बाग क चार ो ओर रप-रखा क मानवचतरण क

वलए भारतीय परातासतवक सिकषण (The

Archaeological Survey of India ndash ASI)

गराउणड पनीिर विोग रडार (Ground Penetrating

Radar) का परय ग करगा

Q29) उततर (a)

सपषटीकरणः

- वसलपपावदकारम एक तवमल महाकावय ह वजसकी

रचना इलाोग क दवारा लगभग 1800 ििि पिि की

गई थी

- यह क िलन नामक एक वयापारी की कहानी ह

ज माििी नामक एक गवणका (िशया) स परम

करन लगा था

- मवनमकलाई क िलन और माििी की पतरी की

कहानी ह

Q30) उततर (a)

सपषटीकरण

- चरक आयिद और वचवकरता की एक महतवपणि

रचना चरक सोवहता क लखक ह

- बरहमगपत क अपनी रचना बरहम-सफि-वसदानत

(ज एक खग लीय रचना ह) क कारण परवससद

वमली

FC19H1003 33

- बगदाद म इसका अनिाद अरबी भािा म वकया

गया था

- इसका इसलावमक गवणत और खग ल-विजञान पर

महतवपणि परभाि पडा था

- बाद म अपन जीिनकाल म बरहमगपत न

ldquoखोडखयाकrdquo वलखी ज एक खग लीय पससतका

(एक छ िी पसतक) थी

- इसम आयिभटट की अिि-रावतर क परतयक वदन की

शरआत परणाली का परय ग वकया गया था

Q31) उततर (c)

सपषटीकरण

- अमीर खसर एक परवसद सफी सोगीतकार कवि

और विदवान थ

- 1318 म उनह ोन पाया वक इस भवम (वहोदसतान) क

हर कषतर म अलग-अलग भािा थी लाहौरी

कशमीरी दवारसमदरी (दवकषणी कनाििक म)

तलोगाना (आोधर परदश म) गजरी (गजरात म)

माबारी (तवमलनाड म ) अििी (पिी उततर परदश

म) और वहोदिी (वदलली क आस-पास क कषतर म)

आवद

- उनह न यह बताया वक सोसकत वकसी भी कषतर स

सोबोवित नही ो थी और किल बराहमण ही इस भािा

का जञान रखत थ

Q32) उततर (c)

सपषटीकरण

- वहरणय-गभि सववणिम गभि क सोदवभित करता ह

- जब बराहमण ो की सहायता स यह अनषठान वकया

जाता था त यह माना जाता था वक बवल दन िाल

का कषवतरय क रप म पनजिनम ह गा

Q33) उततर (d)

सपषटीकरण

- कदमई भवम राजसव पर कर क सोदवभित करता

- गवावलयर परशससत म नागभि क दवारा वकय गए

श िण का िणिन वकया गया ह |

- नागभि एक परवतहार राजा था

Q34) उततर (b)

सपषटीकरण

- राजतरो वगनी 12िी ो शताबदी म कलहन क दवारा

रवचत एक सोसकत पसतक (िकसट) ह

- यह परारसमभक भारत की ऐवतहावसक इवतितत थी

- तकि सोगत रप स इस अपन परकार की सिोततम

और सिािविक विशवसनीय कवत माना जाता ह

- यह कशमीर कषतर क पराचीनतम समय स लकर

उसकी रचना की तारीख तक क समपणि इवतहास

का आचछादन करती ह

Q35) उततर (c)

सपषटीकरण

- गााि की आम सभा क ldquoउरrdquo कहा जाता था

- ldquoउरrdquo म गााि क सभी कर दन िाल वनिासी

शावमल ह त थ

Q36) उततर (a)

सपषटीकरण

- वदलली सलतनत म ldquoतारीखrdquo इवतहास लखन का

एक रप था

- ldquoतािरीखrdquo क लखक विदवान परि ह त थ वजनम

सवचि परशासक इतयावद शावमल थ

Q37) उततर (a)

सपषटीकरण

- अलाउददीन सखलजी अपन सवनक ो क ितन का

भगतान नकद म करता था न वक इकता क रप

- सवनक अपना सामान वदलली म वयापाररय ो स

खरीदत थ अतः इस बात का भय था वक वयापारी

कही ो िसतओो का मलय न बिा द

- इसकी र कथाम क वलए अलाउददीन सखलजी न

वदलली म कीमत ो क वनयसित वकया

FC19H1003 34

- अविकारीगण धयानपििक मलय ो का सिकषण करत

थ तथा ज वयापारी वनिािररत मलय पर माल नही ो

बचत थ उनक दसणडत वकया जाता था

Q38) उततर (d)

सपषटीकरण

- वदलली सििपरथम त मर राजपत ो क अिीन उनक

सामराजय की राजिानी बनी थी

- 12िी ो शताबदी क मधय म अजमर क चौहान ो

(वजनह चाहमान ो क नाम स भी जाना जाता ह) न

त मर राजपत ो क परावजत वकया था

- त मर ो और चौहान ो क अिीन वदलली एक

महतवपणि िावणसजयक क दर बन गया था

- कई जन वयापारी यहाा रहन लग थ और उनह ोन

कई मोवदर भी बनिाए

- यहाा पर मवदरत वसक वजनह ldquoदहलीिालrdquo क नाम

स जाना जाता था वयापक रप स परचलन म थ

Q39) उततर (c)

सपषटीकरण

- म ठ की मसिद का वनमािण वसको दर ल दी क

राजयकाल म उसक मिी क दवारा करिाया गया

था

- बगमपरी मसिद का वनमािण महममद तगलक क

शासनकाल म हआ था

- यह मसिद विशव का पणयथथान (The

Sanctuary of the World) और वदलली म महममद

तगलक की नई राजिानी जहाोपनाह की मखय

मसिद थी

- कववत- अल - इसलाम मसिद का विसतार

इलतसिश और अलाउददीन सखलजी न वकया था

- मीनार का वनमािण तीन सलतान ो कतबददीन ऐबक

इलतसिश और वफर ज शाह तगलक क दवारा

करिाया गया था

Q40) उततर (c)

सपषटीकरण

- मगल ो क अिीन मनसबदार शबद उस वयसकत क

वलए सोदवभित वकया जाता था वजसक पास मनसब

(अथाित पद) ह ता था

- उस अपना ितन राजसव कायो वजनह जागीर कहत

थ क रप म परापत ह ता था

Q41) उततर (b)

सपषटीकरण

- ldquoभारत छ ड आोद लनrdquo वबरविश शासन क

सखलाफ ल ग ो का एक सवाभाविक विदर ह था

- असखल भारतीय काोगरस सवमवत न 8 अगसत 1942

क बमबई म एक बठक का आय जन वकया था

- इस बठक म परवसद सोकलप ldquoभारत छ ड rdquo क

पाररत वकया गया और इस उददशय क परापत करन

क वलए गाोिी क नततव म एक अवहोसक जन सोघिि

आोद लन की शरआत का परसताि वदया गया

- लवकन अगल ही वदन गाोिी और काोगरस क अनय

परमख नताओो क वगरफतार कर वलया गया

- काोगरस क एक बार वफर अिि घ वित वकया गया

था

Q42) उततर (c)

सपषटीकरण

- साइमन कमीशन यनाइविड वको गडम क सात

साोसद ो का एक समह था

- इस वबरविश भारत क वलए सोििावनक सिार ो का

सझाि दन क वलए गवठत वकया गया था

- इस आय ग म िररषठ वबरविश राजनता सर जॉन

साइमन क नततव म किल वबरविश सदसय ही

शावमल थ

- इसवलए भारत क ल ग ो न साइमन कमीशन क

आगमन क विरद आोद लन वकया था

Q43) उततर (a)

सपषटीकरण

bull दादा भाई नौर जी भारत म वबरविश शासन क

आवथिक पररणाम ो क बार म अपनी विर िी

(परवतकल) राय क वलए जान जात थ

FC19H1003 35

bull अपन कई लख ो और भािण ो म विशि रप स

ldquoपाििी एो ड अन-वबरविश रल इन इसणडया

(Poverty and Un-British Rule in India) म

नौर जी न यह तकि वदया वक भारत पर अतयविक

कर लगाया गया था और इसकी सोपवतत इोगलड की

ओर परिावहत की जा रही थी

bull उनह ोन पराचीन भारतीय गरोथ ो की वयाखया करन

का और भारतीय ो क आिविशवास क बहाल

करन पर कायि नही ो वकया था

उनह ोन वकसी और बात स पहल सभी सामावजक

बराइय ो क उनमलन की आिशयकता पर भी बल

नही ो वदया था

Q44) उततर (c)

सपषटीकरण

bull अगसत 1932 म वबरविश परिानमोतरी मकड नालड न

अपन साोपरदावयक परसकार (The Communal

Award) की घ िणा की थी

bull यह भारत क कई साोपरदावयक वहत ो क बीच विवभनन

सोघिो क हल करन क वलए वबरिन का एकतरफा

परयास था

bull यह परसकार (Award) बाद म 1935 क

अविवनयम (The Act of 1935) म शावमल वकया

गया था

bull इस साोपरदावयक परसकार न मससलम ो क वलए

आरवकषत एक अलग वनिािचक मणडल फॉमिल का

विसतार अनय अलपसोखयक ो क वलए वकया था

वजसम वसख ो भारतीय ईसाइय ो आोगल-भारतीय

समदाय यर पीय समदाय तथा विवशषट कषतरीय

समह ो क शावमल वकया गया था

bull गाोिी न इस परसताि क भारतीय समाज क

विभावजत करन क वलए एक घवणत वबरविश

सावजश क रप म दखा और उसक सखलाफ

आमरण अनशन वकया

Q45) उततर (b)

सपषटीकरण

मौजदा आयात और वनयाित क अवतररक़त

औपवनिवशक भारत क वनमनवलसखत खचो क

वलए एक विशिवनवशचत िन रावश भी दनी पडती

थी

(i) परशासन क वयय

(ii) सना क रख-रखाि क वयय

(iii) यद क वयय

(iv) सिावनितत अविकाररय ो की पशन तथा

(v) वबरिन दवारा अपनी उपवनिश बसती

(कॉल नी) क रख-रखाि क वयय

इनह गह शलक (Home Charges) क रप म

जाना जाता था और लगभग परी तरह स भारत क

दवारा इनका भगतान वकया जाता था

bull गह शलक म वनमनवलसखत घिक शावमल थ

(i) भारतीय ऋण पर दय बयाज

(ii) ईसट इोवडया को पनी क शयरिारक ो क

लाभाोश

(iii) लोदन म भारत कायािलय चलान क वलए िन

(iv) भारत म वनयकत वबरविश कवमिय ो क ितन

और पशन का भगतान करन क वलए िन

(v) रलि पर बयाज

(vi) नागररक और सनय शलक

(vii) इोगलड म सट र (सामगरी) की खरीद

Q46) उततर (b)

सपषटीकरण

bull भारतीय राषटर ीय काोगरस का लाहौर सतर 1929 म

जिाहरलाल नहर की अधयकषता म आय वजत

वकया गया था

bull इस सतर म भारतीय राषटर ीय आोद लन स समबसित

कई महतवपणि पररणाम सामन आय थ

(i) सििपरथम इस सतर म काोगरस क अधयकष पद

पर जिाहरलाल नहर क चना गया था ज

काोगरस म िामपोवथय ो की बिती हई ताकत

का सपषट सोकत था

(ii) दसरा इस सतर म पहली बार काोगरस न पणि

सवतोतरता की माोग क उठाया था

इस परकार की माोग काोगरस मोच स पहल कभी भी

नही ो उठाई गई थी

Q47) उततर (b)

सपषटीकरण

FC19H1003 36

bull इस ररप िि न वकसी भी समदाय क वलए पथक

वनिािचक मोडल अथिा अलपसोखयक ो क वलए

भाराोश की वसफाररश नही ो की थी

bull तथावप इस ररप िि न उन पराोत ो म अलपसोखयक

सीि ो क आरकषण की अनमवत दी थी जहाा पर कम

स कम दस परवतशत अलपसोखयक ह

bull लवकन यह समदाय क आकार क अनपात म ह ना

चावहए था

bull इस ररप िि म भारत क वलए पणि सवतोतरता क

वलए क ई पराििान नही ो था

Q48) उततर (c)

सपषटीकरण

bull आरो वभक िवदक आयो का िमि मखय रप स

परकवत की पजा और यजञ था

bull परारो वभक आयि िमि परकवत की पजा क समान था

bull िासति म उनक चार ो ओर की शसकतयाा वजनह न

त ि वनयोवतरत कर सकत थ और न ही समझ पाए

थ उनह वदवयता क साथ वनिवशत वकया गया तथा

उनह मादा या नर दिीदिताओो क रप म

परतीकतव वकया गया था

bull उनह ोन कछ यजञ ो का भी वनषपादन वकया था

Q49) उततर (b)

सपषटीकरण

bull सडक और नदी-मागि (जल-मागि) डकती स

सरवकषत नही ो थ

bull उललखनीय ह वक हिििििन क शासनकाल क

दौरान यआन चिाोग (हयएन साोग) का सारा

सामान लि वलया गया था

Q50) उततर (c)

सपषटीकरण

परशन म वदए गए द न ो कथन सही ह

Q51) उततर (b)

सपषटीकरण

bull परोदर दास एक सोत और भगिान कषण क एक

महान भकत थ

bull परोदर दास क कनाििक सोगीत क वपतामह क

रप म जाना जाता ह

bull यदयवप उनक जनम-थथान क बार म काफी

अिकल लगाई जाती रही ह

bull तथावप अब कननड विशवविदयालय हमपी क दवारा

गवठत एक विशिजञ सवमवत इस वनषकिि पर पहोची

ह वक उनका जनम थथान सोभितया कनाििक का

एक छ िा-सा गााि कषमपरा (वशिम गगा वजला)

था

Q52) उततर (c)

सपषटीकरण

bull शरी तयागराज शरी शयाम शासतरी और शरी मथसवामी

दीवकषतर क कनाििक सोगीत की वतरमवति माना

जाता ह

bull उनक कारण ही 18िी ो-19िी ो शताबदी म कनाििक

सोगीत का सववणिम यग आया था

Q53) उततर (d)

सपषटीकरण

bull अभी हाल ही म लौह यगीन-महापािावणक काल

का 2000 ििि पराना एक दलिभ सारक फगस

(Sarcophagus) (पतथर का ताबत) क ललम क

वियर गाोि (क वयलडी क पास वजला क वझक ड

करल राजय) की एक रॉक-कि गफा स ख जा गया

bull यह ताबत वजसम हविय ो क िकड थ खदाई क

दौरान वमला

bull अभी तक इस परकार की दलिभ ख ज करल क

मातर द ही थथान ो स हई ह

bull य द न ो सारक फगी (Sarcophagi) (पतथर क

ताबत) चियर और अथ ली (वजला क वझक ड) क

महापािाण थथल ो स वमल ह

Q54) उततर (a)

सपषटीकरण

FC19H1003 37

दवकषण भारत म महापािाण सोसकवत एक पणि

विकवसत लौह यगीन सोसकवत थी

Q55) उततर (d)

सपषटीकरण

bull च ल पाणडय और करलपतर (चर) इन तीन ो का

उललख अश क क अवभलख ो म वकया गया ह

bull सोभितः य भौवतक सोसकवत क उततर

महापािावणक चरण म थ

Q56) उततर (d)

सपषटीकरण

bull भीमा-क रगाोि की लडाई ततीय आोगल-मराठा

यद का वहससा थी

Q57) उततर (b)

सपषटीकरण

bull राजकमार शकल न गाोिीजी क चोपारण आन तथा

वतनकवथया परणाली स जडी समसया की जाोच क

वलए रारी करन क वलए दश भर म उनका

अनसरण वकया था

bull बज वकश र राजदर परसाद महादि दसाई और

नरहरी पाररख चोपारण सतयागरह क दौरान गाोिी

जी क सहय गी थ

Q58) उततर (b)

सपषटीकरण

bull बराहमण ो और बौद मठिाररय ो क कर-मकत गााि

अनदान म दन की परथा सतिाहन ो न आरमभ की

थी

Q59) उततर (c)

सपषटीकरण

इस कायिकरम क उददशय वनमनानसार ह

(i) बवनयादी पयििन आिाररक सोरचना का विकास

करना

(ii) चयवनत (पहचान वकय गए) कषतर ो म आजीविका क

सजन क वलए दश क साोसकवतक और विरासत

मलय ो क बिािा दना

(iii) विरासत समारक थथल ो पर विशव सतरीय आिाररक

सोरचना विकवसत करक एक सतत तरीक स

पयििक आकििण म िसद करना

(iv) थथानीय समदाय ो की सवकरय भागीदारी क माधयम

स र रगार ो का सजन करना

(v) र रगार उतपादन और आवथिक विकास क वलए

पयििन कषमता का उन पर परभाि का उपय ग

करना तथा

(vi) िारणीय पयििन आिाररक सोरचना का विकास

करना और उसका उवचत सोचालन तथा

रखरखाि सवनवशचत करना

Q60) उततर (b)

सपषटीकरण

bull यह वनकाय ििि 1987 म अससततव म आया था

bull यह एक राषटर ीय सतर का शीिि सोगठन ह ज भारत

सरकार क जनजातीय मामल ो क मोतरालय क

परशासवनक वनयोतरण क अिीन काम कर रहा ह

bull इसका पोजीकत और परिान कायािलय नई वदलली

म सथथत ह

Q61) उततर (c)

सपषटीकरण

bull परमचोद क उपनयास ो म परमाशरम रोगभवम गबन

कमिभवम और ग दान शावमल ह

bull ग रा रिी ोदरनाथ िग र क दवारा रवचत उपनयास ह

bull अभी हाल ही म मोशी परमचोद की 138िी ो जयोती दश

भर म मनाई गई थी

Q62) उततर (b)

सपषटीकरण

bull ldquoवगदाrdquo पोजाब (भारत) एिो पावकसतान की

मवहलाओो क दवारा तयौहार क समय और फसल

की बिाई तथा किाई क अिसर पर वकया जान

िाला एक पारोपररक दहाती नतय ह

FC19H1003 38

bull इस नतय क माधयम स पोजाबी मवहलाऐो अपनी

परसननता परकि करती ह तथा वगदा क परदशिन क

माधयम स परि िचिसव िाल समाज म मवहलाओो

की दबी हई भािनाओो क परकि करती ह

bull चोवक इस नतय का परि ो क साथ क ई सोबोि नही ो

ह अतः किल मवहलाऐो ही इसम भाग ल सकती

bull हर साल तीज समार ह क दौरान पोजाब म वगदा

नतय वकया जाता ह

तीज भारत क कछ भाग ो म मवहलाओो क दवारा

मनाया जान िाल कई तयौहार ो क वलए एक

वयापक नाम ह

Q63) उततर (a)

सपषटीकरण

- मजम-उल-बहरीन या द समदर ो का सोगम

नामक उललखनीय रचना दारा वशक ह क दवारा

वलखी थी

- भारत क उपराषटर पवत शरी एम िकया नायड न कहा

ह वक राजकमार दारा वशक ह की रचनाएा शाोवत

और सदभाि क बिािा दन क वलए एक तारा सर त

क रप म सामन आ सकती ो ह

- उपराषटर पवत गत ििो क भला वदए गए राजकमार

दारा वशक ह क परदवशित परचवलत करन हत

आय वजत एक परदशिनी का दौरा करन क बाद एक

सभा क सोब वित कर रह थ

- इस परदशिनी का आय जन फर क इस गौवियर

(Francois Gautier) क दवारा lsquoइोवदरा गाोिी नशनल

सिर फॉर द आििसrsquo (The Indira Gandhi

National Centre for the Arts) नई वदलली म

वकया गया था

Q64) उततर (c)

सपषटीकरण

- ग मतशवर परवतमा जन भगिान बाहबली क

समवपित ह

- यह एक एक-चटटानी पतथर की मवति ह

- राषटर पवत राम नाथ क विोद न शरिणबलग ला

(कनाििक) म आय वजत वकय जान िाल भवय

अवभिक समार ह महामसतकावभिक का

उदघािन वकया था

- यह समार ह 12 ििो म एक बार ह ता ह

Q65) उततर (c)

सपषटीकरण

bull पराची घािी पराची नदी क चार ो ओर फली हई थी

bull पराची घािी िीर-िीर विलपत ह गई थी

bull पराची नदी भिनशवर स वनकलती ह

bull यह महानदी की एक सहायक नदी ह और यह

परी खदाि किक तथा जगतवसोहपर वजल ो स

ह कर बहती ह

bull इस नदी क पर कषतर क पराची घािी कहा जाता ह

bull यह नदी बोगाल की खाडी म वगरती ह

परातासतवक साकषय स पता चलता ह वक पराची घािी

सभयता हडपपा और म हनज दाड द न ो की

पिििती ह

Q66) उततर (d)

सपषटीकरण

य समारक छतरपर वजल (मधय परदश) म विोधयाचल

पिित शरोखला म सथथत ह

Q67) उततर (a)

सपषटीकरण

bull थॉिस ऑन पावकसतान नामक पसतक डॉ बी

आर अमबडकर न वलखी थी

bull डॉ बी आर अमबडकर की जयोती क अिसर पर

भारत क राषटर पवत न भारत की इस महान हसती

क शरदाोजवल अवपित की थी

bull डॉ बी आर अमबडकर न 1924 म वडपरथड

कलावसर इोसटीटयि (दवलत िगि सोथथान -

बवहषकत वहतकाररणी सभा) और 1927 म समाज

समता सोघ की थथापना की थी

bull अमबडकर का धयान वशकषा कषतर की ओर भी था

bull उनह ोन वशकषा क वनमन िगो म फलान क वलए

पीपलस एजकशन स साइिी (The Peoples

Education Society) क नाम स महाविदयालय ो क

नििकि और छातरािास ो की थथापना की थी

FC19H1003 39

Q68) उततर (b)

सपषटीकरण

bull महरगि भारतीय उपमहादवीप म एक परवसद

निपािाण बसती ह ज बलवचसतान पराोत

पावकसतान म सथथत ह

bull दचपलली (आोधर परदश) क पास नागलर नदी क

पिी ति ो पर चना पतथर क बलॉक क विशाल

विसतार म एक पिि-ऐवतहावसक रॉक आिि थथल की

ख ज की गई ह

bull इसन 1500-2000 ईसा पिि क दौरान गोिर (आोधर

परदश) म विकवसत निपािाण सभयता पर परकाश

डाला ह

Q69) उततर (c)

सपषटीकरण

bull 12िी ो सदी और 13िी ो सदी म काकाविय िोश का

उदय हआ था

bull ि पहल कलयाण क पवशचमी चालकय ो क सामोत थ

bull परारोभ म उनह ोन िारोगल (तलोगाना) क पास एक

छ ि स कषतर पर शासन वकया था

bull उनह ोन ldquoनायक वयिथथाrdquo की शरआत की थी

वजस बाद म विजयनगर क राय शासक ो न

अपनाया और विकवसत वकया था

Q70) उततर (a)

सपषटीकरण

bull गाोिीजी क अनशन स वमल मावलक ो पर दबाि

पडा था ज अोततः शरवमक ो क ितन म 35 परवतशत

की िसद करन क वलए सहमत हए थ

bull गगल (Google) न अनसया साराभाई वजनह ोन

भारत क शरवमक आोद लन म एक अगरणी भवमका

वनभाई थी की 132िी ो जयोती डडल (Doodle) का

वनमािण करक मनाई

Q71) उततर (d)

सपषटीकरण

भारत स यनसक की मानिता की अमति साोसकवतक

विरासत की परवतवनवि सची म वनमनवलसखत शावमल ह

bull कवडयटटम करल का सोसकत रोगमोच

bull मवडयिि करल का अनषठान रोगमोच और नतय

नाविका

bull िवदक मि जाप की परोपरा

bull राजथथान क कालबवलया ल क गीत और नतय

bull रामलीला रामायण का पारोपररक परदशिन

bull सोकीतिन मवणपर का अनषठान गायन ढ ल िादन

और नतय

bull रममन भारत क गििाल वहमालय का िावमिक

तयौहार और अनषठान रोगमोच

bull जाोदीयाला गर पोजाब क ठठर ो की पीतल और

ताोब क वशलप स वनवमित बतिन ो की पारोपररक कला

bull छाऊ नतय पिी भारतीय राजय ो म जनमी शासतरीय

भारतीय नतय कला

bull लददाख का बौद मि जाप िर ाोस-वहमालयी लददाख

कषतर तथा जमम-कशमीर म पवितर बौद गरोथ ो का पाठ

bull य ग

bull नौर र

bull को भ मला

Q72) उततर (b)

सपषटीकरण

bull भारत क राषटर पवत शरी राम नाथ क विोद न

वकसामा नागालड म हॉनिवबल मह रति और

राजय गठन वदिस समार ह का उदघािन वकया

था

bull हॉनिवबल मह रति का नाम भारतीय हॉनिवबल क

नाम पर पडा ह ज एक विशाल और रोगीन जोगली

पकषी ह

bull यह पकषी नागालड राजय की अविकतर जनजावतय ो

की ल ककथाओो म उसललसखत ह

bull नागालड की परमख मानयता परापत जनजावतयाा ह

अोगामी आओ चखसोग चाोग ककी रगमा और

रवलोग आवद

bull ओोग जारिा और ससिनलीस अोडमान-वनक बार

दवीप समह की जनजावतयाा ह

FC19H1003 40

Q73) उततर (c)

सपषटीकरण

bull दकन म राषटर कि शासन दसिी ो सदी क अोत तक

लगभग 200 ििो तक रहा था

bull राषटर कि शासक अपन िावमिक विचार ो म सवहषण

bull उनह ोन न किल शि िमि और िषणि िमि बसलक

जन िमि क भी सोरकषण वदया था

bull एल रा म वशि क परवसद रॉक कि मोवदर का

वनमािण नौिी ो सदी म राषटर कि राजा कषण परथम न

करिाया था

bull उसका उततराविकारी अम घििि जन था लवकन

उसन अनय िमो क भी सोरकषण परदान वकया था

bull राषटर कि ो न मसलमान वयापाररय ो क बसन की

अनमवत दी थी

bull उनह न अपन अविराजय ो म इसलाम क उपदश दन

की भी अनमवत दी थी

bull अभी हाल ही म पाोडिलागटटा (तलोगाना) क

परागवतहावसक चटटान वचतर ो क कषरण की बिती हई

घिनाएा एक गोभीर वचोता का वििय ह

bull यह परागवतहावसक चटटान क नकसान पहाचा

सकता ह

bull पाोडिलागटटा वनमनवलसखत क वलए जाना जाता ह

- 10000 ईसा पिि स 8000 ईसा पिि क वचवतरत

चटटानी आशरय ो क वलए

- राषटर कि काल क एक 8 िी ो सदी क

वशलालख क वलए और

- 12िी ो सदी क काकविय सामराजय क वभवतत

वचतर ो क वलए

Q74) उततर (b)

सपषटीकरण

bull 1828 म राजा राम म हन रॉय न एक नय िावमिक

समाज बरहम सभा की थथापना की थी वजस बाद

म बरहम समाज क नाम स जाना गया था

bull दिदरनाथ िग र न ततवब विनी सभा की अधयकषता

की थी ज आधयासिक सतय की ख ज म सोलि

थी

bull इसका उददशय वहोद िमि क शद करन का और

एकशवरिाद (एक ईशवर म आथथा) का परचार करना

था

bull नय समाज की थथापना क आिार थ कारण

(तकि ) क द सतमभ तथा िद और उपवनिद

bull अभी हाल ही म सािारण बरहम समाज का कछ

काननी मदद ो क लकर पवशचम बोगाल सरकार क

साथ काननी वििाद चल रहा ह

Q75) उततर (c)

सपषटीकरण

bull भारत म वचशती वसलवसल की थथापना खवाजा

म इनददीन वचशती क दवारा की गयी थी

bull ि 1192 ईसवी क आसपास भारत आय थ

bull वचशतीय ो क बारहिी ो शताबदी क उततरािि म भारत

म आन िाल सफीय ो क समह ो म सबस

परभािशाली माना जाता ह

bull उनह ोन थथानीय िातािरण क साथ सफलतापििक

अनकलन वकया और उनह ोन भारतीय भसकत

परोपराओो क कई पहलओो क अपनाया

bull अजमर म सफी अपरकि खवाजा म इनददीन वचशती

की ऐवतहावसक दरगाह क एक नया रप दन की

तयारी की जा रही ह

bull इस 13िी ो शताबदी की दरगाह क ldquoसवचछ

आइकॉवनक थथल ोrdquo (Swacch Iconic Places) म

शावमल वकया गया ह ज परवतवषठत विरासत

आधयासिक और साोसकवतक थथान ो पर क वदरत

य जना ह

FC19H1003 41

ANSWERS amp EXPLANATION OF

NCERT History Class VI-X + Current Affairs

(FC19E1003)

Q1) Answer c

Explanation

Rigveda consists of more than a

thousand hymns dedicated to gods and

goddesses These hymns were

composed by sages and learnt by men

however a few were composed by

women like Apala Ghosa Lopamudra

Maitreyi and Gargi

Rigveda consists of many hymns in the

form of dialogues We get an example of

a dialogue between a sage named

Vishwamitra and two rivers (Beas and

Sutlej) that were worshipped as

goddesses This suggests that he

belonged to the Vedic period

Q2) Answer b

Explanation

Traces of ash have been found from

Kurnool Caves suggesting that people

were familiar with the use of fire

It is situated in Andhra Pradesh

Q3) Answer c

Explanation

Burzahom is a prehistoric site in

present day Kashmir where people built

pit houses which were dug into the

ground with steps leading into them

These may have provided shelter in cold

weather

Q4) Answer c

Explanation

Epigraphy is defined as the study of

inscriptions

Manuscriptology is the study of history

and literature through the use of hand

written documents

Palaeography refers to the study of

ancient writing systems and the

deciphering and dating of historical

manuscripts

Numismatics refers to the study of

coins

Q5) Answer a

Explanation

Charaka Samhita was written by

Charaka and is an important book on

Ayurveda and medicine

He was a practitioner of the traditional

system of Indian medicine known as

Ayurveda

Charaka is thought to have flourished

sometime between the 2nd century BCE

and the 2nd century CE

Q6) Answer b

Explanation

Bhaga refers to the tax on crops which

was fixed at 16th of the production

Kammakaras is the term used for the

landless agricultural labour class

Ashvamedha also known as horse

sacrifice is a ritual where a horse is let

loose to wander freely and it was

guarded by the rajarsquos men

Q7) Answer (d)

Explanation

In the Rigvedic period horses were

yoked to chariots that were used in

battles fought to capture land cattle

etc This suggests that the use of horse

chariots began much before the period

of Mahajanapadas

The battles were fought in the Rigvedic

period for cattlersquos lands water an even

to capture people Most men took part

in these wars however there was no

regular army but there were assemblies

where people met and discussed

matters of war Regular armies became

a feature in the Mjahajanapada period

including vast armies of foot soldiers

chariots and elephants

RAUSIAS-FC19E1003 42

Q8) Answer (a)

Explanation

Buddha belonged to the Sakya clan and

passed away at Kusinara

Buddha taught in Prakrit which was the

common language of people

Q9) Answer c

Explanation

There were six schools of philosophy in

ancient India These are known as

Vaishesika Nyaya Samkhya Yoga

Purva Mimansa and Vedanata or Uttara

Mimansa They were founded by sages

Kanada Gautama Kapila Patanjali

Jamini and Vyasa respectively

Q10) Answer b

Explanation

The teachings of Mahavira were

compiled at Valabhi in 6th century AD

Q11) Answer (c)

Explanation

Chanakya is traditionally identified as

Kautilya or Vishnugupta who authored

the ancient Indian political treatise the

Arthashastra

Q12) Answer d

The national emblem of India is an

adaptation of the Lion Capital atop the

Ashoka Pillar of Sarnath Uttar Pradesh

and is combined with the National

Motto Satyameva Jayate

The Rampurva Bull gets the name from

the site of its discovery Rampurva in

Bihar

It is noted for its delicately sculpted

model demonstrating superior

representation of soft flesh sensitive

nostrils alert ears and strong legs It is

a mixture of Indian and Persian

elements

Sankissa is situated in Uttar Pradesh

India

Q13) Ans(a)

Kunwar Singh was a notable leader during the Revolt of 1857 He belonged

to a royal house of Jagdispur Bihar

Q14) Answer b

Explanation

The term Vellalar was used for large

landowners

Q15) Answer c

Explanation

Arikamedu was a coastal settlement

where ships unloaded goods from

distant lands Finds here include a

massive brick warehouse pottery

including amphorae and Arretine ware

Roman lamps glassware and gems have

also been found at the site

Q16) Answer a

Explanation

Muvendar is a Tamil word mentioned in

Sangam poems meaning three chiefs

used for the heads of three ruling

families the Cholas Cheras and

Pandyas

Q17) Ans (c)

Several tribal or kin-based assemblies

such as the Sabha Vidatha and gana

are mentioned in the Rig-veda The

Sabha and the samiti mattered a great

deal in early Vedic times so much so

that the chiefs or the kings showed an

eagerness to win their support

Q18) Ans (a)

Jainism recognised the existence of the

gods but placed them lower than the

jina and did not condemn the varna

system as Buddhism did

Q19) Answer (d)

Explanation

Cholas and Pandyas had developed

powerful coastal cities The most

important city of Cholas was Puhar or

Kaveripattinam and Madurai was the

capital of Pandyas

Q20) Answer b

Explanation

Buddhacharita is the biography of

Buddha and was written by

RAUSIAS-FC19E1003 43

Ashvaghosha

Q21) Answer (a)

Explanation

Tamil poet Appar was a Shiva devotee

So he was a Nayanar saint

Q22) Answer d

Explanation

Samudragupta was a prominent Gupta

ruler whose coins depict him playing a

veena indicating his love for music We

get important historic information from

his Allahabad Prashasti which was

composed by his court poet Harisena

Q23) Answer (b)

Explanation

Vikrama Samvat was founded by

Chandragupta II in the 58 BC as a

mark of victory over the Shakas and

assumed the title of Vikramaditya

Banabhatta wrote Harshavardhanarsquos

biography the Harshacharita in

Sanskrit

Q24) Answer c

Explanation

Sandhi-vigrahika was the minister of

war and peace

Sarthavaha was the leader of the

merchant caravans

Q25) Answer a

Explanation

Xuan Zang (Hsuan-tsang) was a

Chinese traveller who came during the

reign of Harshavardhana

In the decade that began in 630 AD

Xuan Zang came to India through

Kashmir after visiting Central Asia Iran

and Afghanistan

He travelled from north to east and lived

in Bihar for a couple of years

At Nalanda University Xuan Zang

interacted with students and scholars

mastered local languages and

discovered Buddhist stupas

Q26) Answer c

Explanation

Pradakshina patha is a circular path

laid around a stupa in Buddhist

architecture While the rest are a part of

temple architecture

Q27) Answer d

Explanation

All the above-mentioned temples have

an elaborate use of bricks (baked

bricks) along with stone

Q28) Ans (c)

Muhammad Quli Qutab was the Sultan

of Golconda He was a contemporary of

Akbar was very fond of literature and

architecture

The Sultan was a great poet and he

wrote in Dakhini Urdu Persian and

Telgu and has left an extensive diwan or

collection

Recently the Archaeological Survey of

India (ASI) will be using Ground

Penetrating Radar (GPR) to map the

contours of the area around the Bagh-e-

Naya Qila excavated garden inside the

Golconda Fort in Telangana

Q29) Answer a

Explanation

Silappadikaram is a famous Tamil epic

which was written by Ilango around

1800 years ago It is a story of a

merchant named Kovalan who fell in

love with a courtesan named Madhavi

Manimekalai tells the story of the

daughter of Kovalan and Madhavi

Q30) Answer (a)

Explanation

Charaka is the author of Charaka

Samhita which is an important work of

Ayurveda and medicines

Brahmaguptarsquos fame rests mostly on his

Brahma-sphuta-siddhanta which was

an astronomical work It was translated

into Arabic in Baghdad and had a major

impact on Islamic mathematics and

astronomy

Late in his life Brahmagupta wrote

Khandakhadyaka which was an

RAUSIAS-FC19E1003 44

astronomical handbook that employed

Aryabhatarsquos system of starting each day

at midnight

Q31) Answer (c)

Explanation

Amir Khusrau was a famous sufi

musician poet and scholar In 1318 he

noted that there was different language

in every region of this land (Hindustan)

Lahori Kashmiri Dvarsamudri (in

Southern Karnataka) Telangana (in

Andhra Pradesh) Gujari (in Gujarat)

Marsquobari (in Tamil Nadu) Awadhi (in

eastern Uttar Pradesh) and Hindawai (in

the area around in Delhi) etc He went

to explain that Sanskrit did not belong

to any region and that only brahmans

knew it

Q32) Answer c

Explanation

Hiranyagarbha refers to the golden

womb When this ritual was performed

with the help of Brahmanas it was

thought to lead to the rebirth of the

sacrificer as a Khastriya

Q33) Answer d

Explanation

Kadamai refers to a tax on land

revenue

Gwalior Prashasti describes the exploits

of Nagabhata who was a Pratihara king

Q34) Answer b

Explanation

Rajatarangini is a Sanskrit text written

by Kalhana in the 12th century

It was historical chronicle of early India

It is justifiably considered to be the best

and most authentic work of its kind

It covers the entire span of history in

the Kashmir region from the earliest

times to the date of its composition

Q35) Answer c

Explanation

ldquoUrrdquo was the general assembly of the

village ldquoUrrdquo consisted of all the

taxpaying residents of an ordinary

village

Q36) Answer (a)

Explanation

Tarikh was a form of history writing in

the Delhi Sultanate The authors of

tawarikhs were learned men which

included secretaries administrators etc

Q37 Answer (a)

Explanation

Alauddin chose to pay his soldiers salaries in cash rather than iqtas The soldiers would buy their supplies from merchants in Delhi and it was thus feared that merchants would raise their prices To stop this Alauddin controlled the prices of goods in Delhi Prices were carefully surveyed by officers and merchants who did not sell at the prescribed rates were punished

Q38) Answer (d)

Explanation

Delhi first became the capital of a

kingdom under the Tomara Rajputs

who were defeated in the middle of the

twelfth century by the Chauhans (also

referred to as Chahamanas) of Ajmer

It was under the Tomaras and

Chauhans that Delhi became an

important commercial centre Many rich

Jaina merchants lived in the city and

constructed several temples Coins

minted here called dehliwal had a wide

circulation

Q39) Answer (c)

Explanation

Moth ki Masjid was built in the reign of

Sikandar Lodi by his minister

Begumpuri mosque built in the reign of

Muhammad Tughluq was the main

mosque of Jahanpanah the ldquoSanctuary

of the Worldrdquo and his new capital in

Delhi

Quwwat al ndash Islam mosque was

enlarged by Iltutmish and Alauddin

Khalji The minar was built by three

Sultansndash Qutbuddin Aybak Iltutmish

and Firuz Shah Tughluq

RAUSIAS-FC19E1003 45

Q40) Answer (c)

Explanation

Under the Mughals mansabdar was

referred to an individual who held a

mansab ie rank and he received his

salary as revenue assignments called

jagirs

Q41) Ans (b)

The Quit India Movement was a

spontaneous revolt of people against

British rule

The All India Congress Committee met

at Bombay on 8 August 1942 It passed

the famous resolution Quit India and

proposed the starting of a non-violent

mass struggle under Gandhis

leadership to achieve this aim But on

the very next day Gandhi and other

eminent leaders of the Congress were

arrested The Congress was once again

declared illegal

Q42) Ans (c)

The Simon Commission refers to a

group of seven MPs from the United

Kingdom constituted to suggest

constitutional reforms for British India

The Commission consisted of only

British members headed by one of the

senior British politicians Sir John

Simon

So the people of India agitated against

the arrival of Simon Commission

Q43) Ans (a)

He was widely known for his

unfavourable opinion of the economic

consequences of the British rule in

India

In his many writings and speeches and

especially in Poverty and Un-British

Rule in India Naoroji argued that India

was too highly taxed and that its wealth

was being drained away to England

He did not interpret the ancient Indian

texts and restored the self-confidence of

Indians And also he did not stress the

need for eradication of all the social

evils before anything else

Q44) Ans (c)

In August 1932 Prime Minister

MacDonald announced his Communal

Award Great Britainrsquos unilateral

attempt to resolve the various conflicts

among Indiarsquos many communal

interests

The award which was later

incorporated into the act of 1935

expanded the separate-electorate

formula reserved for Muslims to other

minorities including Sikhs Indian

Christians Anglo-Indians Europeans

distinct regional groups Gandhi

undertook a ldquofast unto deathrdquo against

that offer which he viewed as a

nefarious British plot to divide the

Indian society

Q45) Ans (b)

In British India apart from existing

imports and exports there was also a

particular amount of money which

colonial India contributed towards

administration maintenance of the

army war expenses pensions to retired

officers and other expenses accrued by

Britain towards maintenance of her

colony These were known as Home

charges and were paid for almost

entirely by India

The Home charges was made of

following components-

- Interest payable on Indian debt

- Dividend to shareholders of East

India Company

- Funds used to support the India

Office in London

- Funds used to pay salaries and

pensions of British personnel

engaged in India

- Interest on the railways

- Civil and military charges

- Store purchases in England

Q46) Ans (b)

The Lahore session of the Indian

National Congress was held in 1929

under the Presidentship of Jawaharlal

Nehru

The Lahore session of the Indian

National Congress witnessed significant

RAUSIAS-FC19E1003 46

developments in the Indian national

movement

- First the election of Jawaharlal

Nehru to the post of Presidentship of

the Congress was a clear indication

of the growing strength of the

Leftists in the Congress

- Secondly it was in this session that

the Congress for the first time raised

the demand for complete

independence Such demand was

not raised from the Congress

platform earlier

Q47) Ans (b)

It did not provide for separate

electorates for any community or

weightage for minorities However it did

allow for the reservation of minority

seats in provinces having minorities of

at least ten per cent but this was to be

in strict proportion to the size of the

community

There was no provision for complete

Independence for India

Q48) Ans (c)

The religion of early Vedic Aryans was

primarily of worship of nature and

Yajnas

The early Aryan religion was kind of

nature worship Actually the forces

around them which they could not

control or understand were invested

with divinity and were personified as

male or female gods And they

performed some Yajnas also

Q49) Ans (b)

The roads and river-routes were not

immune from robbery It is notable that

Yuan Chwang (Hiuen Tsang) was

robbed of his belongings during

Harshvardanarsquos period

Q50) Ans (c)

Q51) Ans (b)

Purandara Dasa was a saint and great

devotee of Lord Krishna

There is much speculation about where

Purandara Dasa regarded as the

Pitamaha of Carnatic music was born

Recently an expert committee

constituted by the Kannada University

Hampi has come to the conclusion that

Kshemapura Shivamogga district

Karnataka is the birth place of

Purandara Dasa

Q52) Ans (c)

Sri Tyagaraja Sri Shyama Shastry and Sri Muthuswami Dikshitar are considered the trinity of Carnatic music and with them came the golden age in Carnatic music in the 18th-19th

century

Q53) Ans d)

Recently a rare sarcophagus (stone

coffin) which is 2000 years old from the

Iron AgendashMegalithic era was discovered

from a rock-cut cave at Viyur village of

Kollam near Koyilandy in Kozhikode

district Kerala

The coffin containing bone fragments

was found during an excavation ldquoSo

far such a rare finding has been

discovered only from two sites

in Kerala Both these sarcophagi were

recovered from Megalithic sites at

Chevayur and Atholi also in Kozhikode

district

Q54) Ans a)

The megalithic culture in South India was a full-fledged Iron Age culture

Q55) Ans d)

The Cholas Pandyas and Keralaputras

(Cheras) mentioned in Ashokan

inscriptions were probably in the late

megalithic phase of material culture

Q56) Ans d)

Q57) Ans (b)

Raj Kumar Shukla followed Gandhiji all

over the country to persuade him to

come to Champaran to investigate the

problem associated with tinkathia

system

RAUSIAS-FC19E1003 47

Brij Kishore Rajendra Prasad Mahadev

Desai and Narhari Parikh accompanied

Gandhi ji during the Champaran

Satyagraha

Q58) Ans (b)

The Satvahanas started the practice of granting tax-free villages to brahmanas and Buddhist monks

Q59) Ans c)

The objectives of the Programme are

listed as under

- Developing basic tourism

infrastructure

- Promoting cultural and heritage

value of the country to generate

livelihoods in the identified regions

- Enhancing the tourist attractiveness

in a sustainable manner by

developing world-class

infrastructure at the heritage

monument sites

- Creating employment through active

involvement of local communities

- Harnessing tourism potential for its

effects on employment generation

and economic development

- Developing sustainable tourism

infrastructure and ensuring proper

Operations and maintenance

therein

Q60) Ans (b)

The Tribal Cooperative Marketing

Development Federation of India

(TRIFED) came into existence in 1987

It is a national-level apex organization

functioning under the administrative

control of Ministry of Tribal Affairs

Govt of India

TRIFED has its registered and Head

Office located in New Delhi

Q61) Ans (c)

Premchandrsquos novels include

Premashram Rangabhumi Ghaban

Karmabhumi and Godan

Gora is a novel written by Rabindranath

Tagore

138th birth anniversary of Munshi

Premchand was celebrated across the

country

Q62) Ans (b)

Giddha is a traditional pastoral dance

performed by the women of the Punjab

India and Pakistan at festival times

and at the sowing and reaping of the

harvest

By this dance the Punjabi women

reveal their joy expel their suppressed

feelings in a male dominated society

through the performance of Giddha

Since this dance has nothing to do with

men only women can participate in it

During the Teej celebrations Giddha

dance is celebrated in Punjab every

year Teej is a generic name for a

number of festivals that are celebrated

by women in some parts of India

Q63) Ans (a)

Dara Shukoh wrote the remarkable

work called ldquoMajma-ul-Bahrainrdquo or the

ldquoThe confluence of two seasrdquo

The Vice President of India Shri M

Venkaiah Naidu has said that Prince

Dara Shukohrsquos writings can come as a

refreshing source for infusing peace and

harmony He was addressing the

gathering after visiting the exhibition

that showcases the forgotten Prince of

yesteryears Dara Shukoh organized by

Mr Francois Gautier at Indira Gandhi

National Centre for the Arts in New

Delhi

Q64) Ans (c)

The statue Gommateshwara is

dedicated to the Jain God Bahubali

It is a monolithic statue

President Ram Nath Kovind

inaugurated the grand anointing

ceremony mdash Mahamastakabhisheka mdash

held once in 12 years at

Shravanabelagola (Karnataka)

Q65) Ans (c)

Prachi Valley had come up around the

Prachi river Prachi Valley gradually

disappeared

RAUSIAS-FC19E1003 48

The Prachi river originates from

Bhubaneswar

It is a tributary of the Mahanadi and

flows through the districts of Puri

Khurda Cuttack and Jagatsinghpur

and the entire region of the river is

termed as the Prachi Valley

It falls into the Bay of Bengal

Archaeological evidence shows that the

Prachi Valley Civilisation predates both

Harappa and Mohenjo-Daro

The Prachi river originates from

Bhubaneswar

Q66) Ans (d)

These monuments are located in

Chhatarpur district Madhya Pradesh

within Vindhya mountain range

Q67) Ans (a)

The book lsquoThoughts on Pakistanrsquo was

written by Dr BR Ambedkar

On the occasion of the birth anniversary

of Dr BR Ambedkar the president of

India pays homage to this icon of India

In 1924 he founded the Depressed

Classes Institute (Bahishkrit Hitkarini

Sabha) and in 1927 the Samaj Samata

Sangh

Another area of attention for Ambedkar

was education For its spread among

the low classes he set up a network of

colleges by the name of Peoples

Education Society and founded hostels

Q68) Ans(b)

Mehrgarh is a famous Neolithic

settlement in the Indian subcontinent

which is situated in Baluchistan

province Pakistan

A pre-historic rock art site is discovered

in the vast expanse of limestone blocks

on the eastern banks of Naguleru river

near Dachepalli (Andhra Pradesh) It

has thrown light on the Neolithic

civilisation that flourished in Guntur

(Andhra Pradesh) during 1500-2000

BC

Q69) Ans (c)

The 12th and the 13th centuries saw

the emergence of the Kakatiyas They

were at first the feudatories of the

Western Chalukyas of Kalyana Initially

they ruled over a small territory near

Warangal (Telangana)

They introduced Nayakships which was

later adopted and developed by the

Rayas of Vijayanagara

Q70) Ans (a)

The fast had effect of putting pressure

on mill owners who finally agreed to

give the workers a 35 per cent increase

in wages

Google celebrated with a doodle the

132nd birth anniversary of Anasuya

Sarabhai who played a pioneering role

in Indiarsquos labour movement

Q71) Ans (d)

The UNESCOrsquos list of the representative

list of the intangible cultural heritage of

humanity from India are

- Koodiyattam Sanskrit Theatre of

Kerala

- Mudiyettu ritual theatre and dance

drama of Kerala

- Tradition of Vedic Chanting

- Kalbelia folk songs and dances of

Rajasthan

- Ramlila Traditional Performance of

the Ramayana

- Sankirtana ritual singing

drumming and dancing of Manipur

- Ramman religious festival and

ritual theatre of the Garhwal

Himalayas India

- Traditional brass and copper craft of

utensil making among the Thatheras

of Jandiala Guru Punjab India

- Chhau dance classical Indian dance

originated in the eastern Indian

states

- Buddhist chanting of Ladakh

recitation of sacred Buddhist texts

in the trans-Himalayan Ladakh

region Jammu and Kashmir India

- Yoga

- Nouroz

- Kumbh Mela

RAUSIAS-FC19E1003 49

Q72) Ans(b)

The President of India Shri Ram Nath Kovind inaugurated the Hornbill Festival and State Formation Day celebrations of Nagaland in Kisama

The festival is named after the Indian hornbill the large and colourful forest bird which is displayed in the folklore of most of the states tribes

The major recognized tribes of Nagaland are Angami Ao Chakhesang Chang

Kuki Rengma and Zeling etc

Onge Jarawa and Sentinelese are the

tribes of Andman amp Nicobar Islands

Q73) Ans (c)

The Rashtrakutas rule in the Deccan lasted for almost two hundred years till the end of the tenth century The Rashtrakutas rulers were tolerant in their religious views and patronized not only Shaivism and Vaishnavism but

Jainism as well

The famous rock-cut temple of Shiva at Ellora was built by one of the Rashtrakutas kings Krishna I in the ninth century His successor Amoghavarsha was a Jain but he also

patronized other faiths

The Rashtrakutas allowed Muslims traders to settle and permitted Islam to

be preached in their dominions

Recently increasing defacement at the prehistoric rock paintings of Pandavulagutta Telangana has created a cause for grave concern It can spoil

the prehistoric rock

Pandavulagutta is home to

- Painted rock shelters dating to

10000 BC-8000 BC

- An 8th century inscription of the

Rashtrakuta period and

- Painted frescoes from the 12th century Kakatiya empire

Q74) Ans (b)

In 1828 Raja Ram Mohan Roy founded a new religious society the Brahma Sabha later known as the Brahmo

Samaj

Debendranath Tagore headed the Tattvabodhini Sabha which was

engaged in search of spiritual truth

Its purpose was to purify Hinduism and to preach monotheism or belief in one God

The new society was to be based on the twin pillars of reason and the Vedas and

Upanishads

Recently Sadharan Brahmo Samaj (SBS) has entered into a legal battle with the West Bengal government due

to some legal issue

Q75) Ans (c)

The Chishti order was established in India by Khwaja Moinuddin Chishti who came to India around 1192 The Chishtirsquos are considered to be the most influential of the groups of Sufis who migrated to India in the late twelfth century They adapted successfully to the local environment and adopted several features of Indian devotional

traditions

The historical dargah of Sufi mystic Khwaja Moinuddin Chishti in Ajmer is all set to get a facelift This 13 th century dargah has been included among the Swachh Iconic Places a clean-up initiative focused on iconic

heritage spiritual and cultural places

Page 12: GENERAL STUDIES (PAPER I) · Test is part of Rau’s IAS Test series for Preliminary Exam 2019 FOUNDATION + CURRENT AFFAIRS GENERAL STUDIES (PAPER –I) FOUNDATION TEST –III TOPIC:

RAUSIAS-FC19E1003 12

Q32) निमननिखित कथि ो पर निचार कीनजए

1 नहरणय-गभण अिषठाि क बार म ऐसा स चा जाता

था नक बनि दि िाि का एक कषनतरय क रप म

पिजणनम ह गा

2 मयरशमणि कदोब िोश का सोसथापक था

उपयणकत कथि ो म स कौि-सास सही हह

(a) किि 1

(b) किि 2

(c) 1 और 2 द ि ो

(d) ि त 1 ि ही 2

Q33) निमननिखित कथि ो म स कौि-सास सही हह

1 कदमई बगार (बिपिणक शरम) क रप म

निया जाि िािा कर था

2 गवानियर परशखसत म िागभट (ज एक चोदि

राजा था) क दवारा नकय गए श िर का िरणि

नकया गया ह

िीच नदए गए कट का परय ग कर सही उततर चनिए

(a) किि 1

(b) किि 2

(c) 1 और 2 द ि ो

(d) ि त 1 ि ही 2

Q34) निमननिखित कथि ो म स कौि-सास सही हह

1 राजतरो नगिी 11िी ो शताबदी म कलहि क दवारा

रनचत एक सोसकत पसतक (टकसट) ह

2 कननौज क निए नतरपकषीय सोघिण म पाि राजिोश

शानमि था

िीच नदए गए कट का परय ग कर सही उततर चनिए

(a) किि 1

(b) किि 2

(c) 1 और 2 द ि ो

(d) ि त 1 ि ही 2

Q35) निमननिखित यग ो पर निचार कीनजए

1 बरहदशवर मोनदर राजराजा च ि

2 उर मापि की इकाई

3 दिदाि मोनदर ो क भनम अिदाि

उपयणकत यग ो म स कौि-स सही समनित ह

(a) किि 1 और 2

(b) किि 2 और 3

(c) किि 1 और 3

(d) 1 2 और 3

Q36) निमननिखित कथि ो म स कौि-सास सही हह

1 नदलली क सलताि ो क अरीि परशासि की भािा

फारसी थी

2 नदलली सलतित म ldquoतारीितािरीिrdquo कनिता

का एक रप था

िीच नदए गए कट का परय ग कर सही उततर चनिए

(a) किि 1

(b) किि 2

(c) 1 और 2 द ि ो

(d) ि त 1 ि ही 2

Q37) निमननिखित कथि ो म स कौि-सास सही हह

1 अिाउददीि खििजी ि अपि सनिक ो क निए

नसरी िाम का एक िया दगणरकषक शहर

बिािाया था

2 िह अपि सनिक ो क िति का भगताि इकता

क रप म करता था

िीच नदए गए कट का परय ग कर सही उततर चनिए

(a) किि 1

(b) किि 2

(c) 1 और 2 द ि ो

(d) ि त 1 ि ही 2

RAUSIAS-FC19E1003 13

Q32) Consider the following statements

1 Hiranya-garbha ritual was thought

to lead to the rebirth of the

sacrificer as a Kshatriya

2 Mayurasharman was the founder

of the Kadamba dynasty

Which of the statements given above

isare correct

(a) 1 only

(b) 2 only

(c) Both 1 and 2

(d) Neither 1 nor 2

Q33) Which of the following statements

isare correct

1 Kadamai was tax taken in form of

forced labour

2 Gwalior Prashasti describes the

exploits of Nagabhata who was a

Chandella king

Select the correct answer using the code

given below

(a) 1 only

(b) 2 only

(c) Both 1 and 2

(d) Neither 1 nor 2

Q34) Which of the following statements

isare correct

1 Rajatarangini is a Sanskrit text

written by Kalhana in the 11th

century

2 Pala dynasty was included in the

tripartite struggle for Kannauj

Select the correct answer using the code

given below

(a) 1 only

(b) 2 only

(c) Both 1 and 2

(d) Neither 1 nor 2

Q35) Consider the following pairs

1 Brihadeshvara temple Rajaraja

Chola

2 ldquoUrrdquo Unit of measurement

3 Devadana Land grants made to

temples

Which of the pairs given above isare

correct

(a) 1 and 2 only

(b) 2 and 3 only

(c) 1 and 3 only

(d) 1 2 and 3

Q36) Which of the following statements

isare correct

1 The language of administration

under the Delhi Sultans was

Persian

2 Tarikhtawarikh was a form of

poetry in the Delhi Sultanate

Select the correct answer using the code

given below

(a) 1 only

(b) 2 only

(c) Both 1 and 2

(d) Neither 1 nor 2

Q37) Which of the following statements

isare correct

1 Alauddin Khilji constructed a new

garrison town named Siri for his

soldiers

2 He paid his soldiers their salaries

in the form of Iqta

Select the correct answer using the code

given below

(a) 1 only

(b) 2 only

(c) Both 1 and 2

(d) Neither 1 nor 2

RAUSIAS-FC19E1003 14

Q38) निमननिखित कथि ो म स कौि-सास सही हह

1 नदलली कतबददीि एबक क अरीि पहिी बार

नकसी सामराजय की राजरािी बिी थी

2 दहिीिाि नसक ो का मदरर मग़ि ो क दवारा

नकया गया था

िीच नदए गए कट का परय ग कर सही उततर चनिए

(a) किि 1

(b) किि 2

(c) 1 और 2 द ि ो

(d) ि त 1 ि ही 2

Q39) निमननिखित यग ो पर निचार कीनजए

1 म ठ की मखिद नसको दर ि दी

2 बगमपरी मखिद नफर ज शाह तगिक

3 कववत- अि - इसलाम कतबददीि ऐबक

उपयणकत यग ो म स कौि-स सही समनित ह

(a) किि 1 और 2

(b) किि 2 और 3

(c) किि 1 और 3

(d) 1 2 और 3

Q40) निमननिखित कथि ो म स कौि-सास सही हह

1 मिसबदार ो क अपिा िति राजसव कायो

नजन जागीर कहत थ क रप म परापत ह ता

था

2 मिसबदार क ज सनय उततरदानयतव सौोप जात

थ उसक अनतगणत उस एक निराणररत सखया म

सिार अथिा घड़सिार ो का रि-रिाि करिा

पड़ता था

िीच नदए गए कट का परय ग कर सही उततर चनिए

(a) किि 1

(b) किि 2

(c) 1 और 2 द ि ो

(d) ि त 1 ि ही 2

Q41) ldquo1942 क भारत छ ड़ आोद ििrdquo क बार म

निमननिखित अिि कि ो म स कौि-सा सतय िही ो ह

(a) यह एक अनहोसक आोद िि था

(b) इसका िततव महातमा गाोरी क दवारा नकया गया

था

(c) यह एक सवाभानिक आोद िि था

(d) इसि सामानयतया शरनमक िगण क आकनिणत

िही ो नकया था

Q42) भारत क ि ग ो ि ldquoसाइमि कमीशिrdquo क आगमि क

निरदध आोद िि नकया था कय ोनक

(a) भारतीय कभी भी 1919 क अनरनियम (The

Act of 1919) क काम की समीकषा िही ो करिा

चाहत थ

(b) साइमि कमीशि ि पराोत ो म दवर (द हर) शासि

क समापत करि की नसफाररश की थी

(c) साइमि कमीशि म क ई भारतीय सदसय िही ो

था

(d) साइमि कमीशि ि दश क निभाजि का

सझाि नदया था

Q43) निमननिखित कथि ो पर निचार कीनजए

भारतीय राषटर ीय आोद िि म दादाभाई िौर जी क दवारा

नकया गया सबस परभािी य गदाि यह था नक उन ोि

1 अोगरज ो क दवारा भारत क आनथणक श िर का

ििासा नकया था

2 पराचीि भारतीय गरोथ ो की वयाखया की थी और

भारतीय ो क आतमनिशवास क पिःसथानपत नकया

था

3 अनय नकसी भी बात स पहि सभी सामानजक

बराइय ो क उनमिि की आिशयकता पर बि

नदया था

उपयणकत कथि ो म स कौि-सास सही हह

(a) किि 1

(b) किि 2 और 3

(c) किि 1 और 3

(d) 1 2 और 3

RAUSIAS-FC19E1003 15

Q38) Which of the following statements

isare correct

1 Delhi first became the capital of a

kingdom under Qutubuddin

Aibak

2 Dehliwal coins were minted by the

Mughals

Select the correct answer using the code

given below

(a) 1 only

(b) 2 only

(c) Both 1 and 2

(d) Neither 1 nor 2

Q39) Consider the following pairs

1 Moth ki Masjid- Sikander Lodi

2 Begumpuri mosque- Firuz Shah

Tughluq

3 Quwwat al ndash Islam- Qutubuddin

Aibak

Which of the above pairs isare correct

(a) 1 and 2 only

(b) 2 and 3 only

(c) 1 and 3 only

(d) 1 2 and 3

Q40) Which of the following statements

isare correct

1 Mansabdars received their salaries

as revenue assignments called

jagirs

2 The mansabdarrsquos military

responsibilities required him to

maintain a specified number of

sawar or cavalrymen

Select the correct answer using the code

given below

(a) 1 only

(b) 2 only

(c) Both 1 and 2

(d) Neither 1 nor 2

Q41) Which one of the following observations

is not true about the Quit India

Movement of 1942

(a) It was a non-violent movement

(b) It was led by Mahatma Gandhi

(c) It was a spontaneous movement

(d) It did not attract the labour class

in general

Q42) The people of India agitated against the

arrival of the Simon Commission

because

(a) Indians never wanted the review of

the working of the Act of 1919

(b) Simon Commission recommended

the abolition of dyarchy in the

Provinces

(c) there was no Indian member in the

Simon Commission

(d) the Simon Commission suggested

the partition of the country

Q43) Consider the following statements

The most effective contribution made by

Dadabhai Naoroji to the cause of Indian

National Movement was that he-

1 exposed the economic exploitation

of India by the British

2 interpreted the ancient Indian

texts and restored the self-

confidence of Indians

3 stressed the need for eradication of

all the social evils before anything

else

Which of the statements given above

isare correct

(a) 1 only

(b) 2 and 3 only

(c) 1 and 3 only

(d) 1 2 and 3

RAUSIAS-FC19E1003 16

Q44) महातमा गाोरी ि 1932 म आमरर अिशि नकया था

कय ोनक

(a) ldquoग िमज सममििrdquo (The Round Table

Conference) भारतीय राजिीनतक

आकाोकषाओो क परा करि म असफि रहा था

(b) काोगरस और मखसलम िीग म मतभद थ

(c) रामस मकड िालड (Ramsay Macdonald)

ि ldquoसाोपरदानयक परसकारrdquo (The Communal

Award) की घ िरा की थी

(d) ldquoसनििय अिजञा आोद ििrdquo (The Civil

Disobedience Movement) असफि रहा

था

Q45) भारत म औपनििनशक शासि की अिनर क सोदभण म

भारत स रि क बनहगणमि का एक महतवपरण भाग गह

शलक (Home Charges) था निमननिखित म स

कौि-सास क ि गह शलक म सखममनित नकया गया

थानकय गए थ

1 िोदि म भारत कायाणिय क निए उपय ग नकय

जाि िािा क ि

2 भारत म नियकत नबरनटश कनमणय ो क िति और

पशि का भगताि करि क निए उपय ग नकय

जाि िािा क ि

3 अोगरज ो क दवारा भारत क बाहर यदध ो क निए

उपय ग नकय जाि िािा क ि

िीच नदए गए कट का परय ग कर सही उततर चनिए

(a) किि 1

(b) किि 1 और 2

(c) किि 2 और 3

(d) 1 2 और 3

Q46) सवतोतरता आोद िि क इनतहास म भारतीय राषटर ीय

काोगरस का 1929 का सतर महतवपरण ह कय ोनक इसम

(a) काोगरस क उददशय क रप म सथािीय सरकार

की पराखपत की घ िरा की गई थी

(b) परण सवराज की पराखपत क काोगरस क िकषय क

रप म अपिाया गया था

(c) असहय ग आोद िि शर नकया गया था

(d) िोदि म ldquoग ि मर सममििrdquo (The Round

Table Conference) म भाग िि का निरणय

निया गया था

Q47) भारतीय सवतोतरता सोगराम क सोदभण म िहर ररप टण

क दवारा निमननिखित म स नकसकी नसफाररश की गई

थीनकिकी नसफाररश की गई थी ो

1 भारत क निए परण सवतोतरता

2 अलपसोखयक ो क निए सीट ो क आरकषर क

निए सोयकत नििाणचक मोडि

3 सोनिराि म भारत क ि ग ो क निए मौनिक

अनरकार ो का परािराि

िीच नदए गए कट का परय ग कर सही उततर चनिए

(a) किि 1

(b) किि 2 और 3

(c) किि 1 और 3

(d) 1 2 और 3

Q48) आरो नभक िनदक आयो का रमण मखय रप स था

(a) भखकत

(b) मनतण पजा और यजञ

(c) परकनत की पजा और यजञ

(d) परकनत की पजा और भखकत

RAUSIAS-FC19E1003 17

Q44) Mahatma Gandhi undertook fast unto

death in 1932 mainly because

(a) The Round Table Conference failed

to satisfy Indian political

aspirations

(b) The Congress and Muslim League

had differences of opinion

(c) Ramsay Macdonald announced the

Communal Award

(d) The Civil Disobedience Movement

failed

Q45) With reference to the period of colonial

rule in India ldquoHome Chargesrdquo formed

an important part of drain of wealth

from India Which of the following funds

constituted ldquoHome Chargesrdquo

1 Funds used to support the India

Office in London

2 Funds used to pay salaries and

pensions of British personnel

engaged in India

3 Funds used for waging wars

outside India by the British

Select the correct answer using the code

given below

(a) 1 only

(b) 1 and 2 only

(c) 2 and 3 only

(d) 1 2 and 3

Q46) The 1929- Session of Indian National

Congress is of significance in the history

of the Freedom Movement because the-

(a) attainment of Self-Government

was declared as the objective of

the Congress

(b) attainment of Poorna Swaraj was

adopted as the goal of the

Congress

(c) Non-Cooperation Movement was

launched

(d) decision to participate in the

Round Table Conference in

London was taken

Q47) With reference to the period of Indian

freedom struggle which of the following

waswere recommended by the Nehru

report

1 Complete Independence for India

2 Joint electorates for reservation of

seats for minorities

3 Provision of fundamental rights for

the people of India in the

Constitution

Select the correct answer using the code

given below

(a) 1 only

(b) 2 and 3 only

(c) 1 and 3 only

(d) 1 2 and 3

Q48) The religion of the early Vedic Aryans was primarily of

(a) Bhakti

(b) image worship and Yajnas

(c) worship of nature and Yajnas

(d) worship of nature and Bhakti

RAUSIAS-FC19E1003 18

Q49) भारत की यातरा करि िाि चीिी यातरी यआि चिाोग

(हयएि साोग) ि समकािीि भारत की सामानय

खसथनतय ो और सोसकनत क दजण नकया था इस सोदभण म

निमननिखित कथि ो म स कौि-सास सही हह

1 सड़क और िदी-मागण (जि-मागण) डकती स

परण रप स सरनकषत थ

2 जहा तक अपरार ो क निए दणड की बात ह

उसक निए नकसी भी वयखकत की निदोिता

अथिा उसक अपरार क निराणररत करि क

निए अनि जि और निि परि क माधयम क

सारि थ

3 वयापाररय ो क घाट ो और परनतबोर सटशि ो पर

शलक ो का भगताि करिा पड़ता था

िीच नदए गए कट का परय ग कर सही उततर चनिए

(a) किि 1

(b) किि 2 और 3

(c) किि 1 और 3

(d) 1 2 और 3

Q50) नसोर घाटी सभयता क सोदभण म निमननिखित कथि ो पर

निचार कीनजए

1 यह मखय रप स एक रमणनिरपकष सभयता थी

तथा हािाोनक इसम रानमणक ततव मौजद था

िनकि िह परनतिश पर हािी िही ो था

2 इस काि क दौराि भारत म कपास का परय ग

कपड़ा बिाि क निए नकया जाता था

उपयणकत कथि ो म स कौि-सास सही हह

(a) किि 1

(b) किि 2

(c) 1 और 2 द ि ो

(d) ि त 1 ि ही 2

Q51) परोदर दास क सोदभण म निमननिखित कथि ो पर निचार

कीनजए

1 परोदर दास एक सोत और भगिाि नशि क एक

महाि भकत थ

2 ि एक सोगीतकार गायक और किाणटक सोगीत

क मखय सोसथापक-परसतािक ो म स एक थ

उपयणकत कथि ो म स कौि-सास सही हह

(a) किि 1

(b) किि 2

(c) 1 और 2 द ि ो

(d) ि त 1 ि ही 2

Q52) निमननिखित म स कौि-सास वयखकत किाणटक सोगीत

की नतरमनतण म शानमि हह

1 बािामरिी कषणा

2 शरी शयाम शासतरी

3 शरी मथसवामी दीनकषतर

िीच नदए गए कट का परय ग कर सही उततर चनिए

(a) किि 1

(b) किि 2

(c) किि 2 और 3

(d) 1 2 और 3

Q53) चियर (Chevayur) और अथ िी (Atholi) म खसथत

महापािार सथि निमननिखित म स नकस राजय म खसथत

(a) तनमििाड

(b) किाणटक

(c) पनिम बोगाि

(d) करि

RAUSIAS-FC19E1003 19

Q49) The Chinese traveller Yuan Chwang

(Hiuen Tsang) who visited India

recorded the general conditions and

culture of India at that time In this

context which of the following

statements isare correct

1 The roads and river-routes were

completely immune from robbery

2 As regards punishment for

offences ordeals by fire water and

poison were the instruments for

determining the innocence or guilt

of a person

3 The tradesmen had to pay duties

at ferries and barrier stations

Select the correct answer using the code

given below

(a) 1 only

(b) 2 and 3 only

(c) 1 and 3 only

(d) 1 2 and 3

Q50) Regarding the Indus Valley Civilization

consider the following statements

1 It was predominantly a secular

civilization and the religious

element though present did not

dominate the scene

2 During this period cotton was

used for manufacturing textiles in

India

Which of the statements given above

isare correct

(a) 1 only

(b) 2 only

(c) Both 1 and 2

(d) Neither 1 nor 2

Q51) Consider the following statements

regarding Purandara Dasa

1 Purandara Dasa was a saint and

great devotee of Lord Shiva

2 He was a composer singer and

one of the chief founding-

proponents of the Carnatic music

Which of the statements given above

isare correct

(a) 1 only

(b) 2 only

(c) Both 1 and 2

(d) Neither 1 nor 2

Q52) Which of the following persons isare

included in the trinity of Carnatic

music

1 Balamurali Krishna

2 Sri Shyama Shastry

3 Sri Muthuswami Dikshitar

Select the correct answer using the code

given below

(a) 1 only

(b) 2 only

(c) 2 and 3 only

(d) 1 2 and 3

Q53) Megalithic sites at Chevayur and Atholi

are located in which of the following

states

(a) Tamil Nadu

(b) Karnataka

(c) West Bengal

(d) Kerala

RAUSIAS-FC19E1003 20

Q54) निमननिखित कथि ो पर निचार कीनजए

1 महापािानरक ि ग कबर ो म िसतएो दफिात थ

2 दनकषर भारत म महापािार सोसकनत एक परण

निकनसत तामर यगीि सोसकनत थी

उपयणकत कथि ो म स कौि-सास सही हह

(a) किि 1

(b) किि 2

(c) 1 और 2 द ि ो

(d) ि त 1 ि ही 2

Q55) निमननिखित म स कौि-स सामराजयसामराजय ो का

अश क क अनभिि ो म उललि नकया गया ह

1 च ि

2 पाणडय

3 करिपतर (चर)

िीच नदए गए कट का परय ग कर सही उततर चनिए

(a) किि 1

(b) किि 1 और 2

(c) किि 3

(d) 1 2 और 3

Q56) भीमा-क रगाोि का यदध को पिी क सनिक ो और

बाजीराि नदवतीय क िततव म एक शखकतशािी पशिा

सिा (मराठ ो) क मधय िड़ा गया था यह यदध

निमननिखित म स नकसका नहससा था

(a) परथम आोगल-मराठा यदध का

(b) नदवतीय आोगल-मराठा यदध का

(c) ततीय आोगल-मसर यदध का

(d) ततीय आोगल-मराठा यदध का

Q57) निमननिखित कथि ो पर निचार कीनजए

1 महादि दसाई ि गाोरीजी क चोपारर आि तथा

नतिकनथया पररािी स जड़ी समसया की जाोच

क निए रारी करि क निए दश भर म उिका

अिसरर नकया था

2 िरहरी पाररि चोपारर सतयागरह क दौराि

गाोरीजी क साथ थ

उपयणकत कथि ो म स कौि-सास सही हह

(a) किि 1

(b) किि 2

(c) 1 और 2 द ि ो

(d) ि त 1 ि ही 2

Q58) निमननिखित कथि ो पर निचार कीनजए

1 िनद राज-िोश ि बराहमर ो और बौदध मठराररय ो

क कर-मकत गाि अिदाि म दि की परथा

आरि की थी

2 सतिाहि ो की आनरकाररक भािा पराकत थी

उपयणकत कथि ो म स कौि-सास सही हह

(a) किि 1

(b) किि 2

(c) 1 और 2 द ि ो

(d) ि त 1 ि ही 2

Q59) एक निरासत क अपिाइए (अडॉपट ए हररटज ndash

Adopt a Heritage) पररय जिा क उददशय ो क

सनदभण म निमननिखित कथि ो पर निचार कीनजए

1 यह पररय जिा र रगार उतपादि और आनथणक

निकास क निए पयणटि कषमता का उि पर

परभाि का उपय ग करगी

2 यह पररय जिा निरासत सथि ो पर निशव सतरीय

आराररक सोरचिा निकनसत करक एक सतत

तरीक स पयणटक आकिणर म िखदध करगी

उपयणकत कथि ो म स कौि-सास सही हह

(a) किि 1

(b) किि 2

(c) 1 और 2 द ि ो

(d) ि त 1 ि ही 2

RAUSIAS-FC19E1003 21

Q54) Consider the following statements

1 Megalithic people buried goods in

graves

2 The megalithic culture in South

India was a full-fledged Copper

Age culture

Which of the statements given above

isare correct

(a) 1 only

(b) 2 only

(c) Both 1 and 2

(d) Neither 1 nor 2

Q55) Which of the following kingdoms isare

mentioned in the Ashokan inscriptions

1 Cholas

2 Pandyas

3 Keralaputras (Cheras)

Select the correct answer using the code

given below

(a) 1 only

(b) 1 and 2 only

(c) 3 only

(d) 1 2 and 3

Q56) The Battle of Bhima-Koregaon was

fought between the soldiers of the

Company and the strong Peshwa army

(Marathas) under Bajirao II This war

was a part of the

(a) First Anglo-Maratha war

(b) Second Anglo-Maratha war

(c) Third Anglo- Mysore war

(d) Third Anglo-Maratha war

Q57) Consider the following statements

1 Mahadev Desai followed Gandhiji all over the country to persuade him to come to Champaran to investigate the problem associated

with tinkathia system

2 Narhari Parikh accompanied Gandhi ji during the Champaran

Satyagraha

Which of the statements given above isare correct

(a) 1 only

(b) 2 only

(c) Both 1 and 2

(d) Neither 1 nor 2

Q58) Consider the following statements

1 The Nanda Dynasty started the practice of granting tax-free villages to brahmanas and

Buddhist monks

2 The official language of the Satavahanas was Prakrit

Which of the statements given above

isare correct

(a) 1 only

(b) 2 only

(c) Both 1 and 2

(d) Neither 1 nor 2

Q59) Consider the following statements about the objectives of the lsquoadopt a heritagersquo

project

1 It will harness tourism potential for its effects on employment generation and economic

development

2 It will enhance the tourist attractiveness in a sustainable manner by developing world class infrastructure at heritage sites

Which of the statements given above

isare correct

(a) 1 only

(b) 2 only

(c) Both 1 and 2

(d) Neither 1 nor 2

RAUSIAS-FC19E1003 22

Q60) ldquoभारतीय जिजातीय सहकारी निपरि निकास सोघrdquo

(The Tribal Co-operative Marketing

Development Federation of India - TRIFED)

क सोदभण म निमननिखित कथि ो पर निचार कीनजए

1 यह एक राषटर ीय सतर का शीिण सोगठि ह ज

भारत सरकार क गह मोतरािय क परशासनिक

नियोतरर क अरीि काम कर रहा ह

2 इसका मखय उददशय दश म जिजातीय ि ग ो

का सामानजक-आनथणक निकास करिा ह

उपयणकत कथि ो म स कौि-सास सही हह

(a) किि 1

(b) किि 2

(c) 1 और 2 द ि ो

(d) ि त 1 ि ही 2

Q61) निमननिखित म स कौि-सास उपनयास परमचोद क

दवारा नििा गया हनिि गए ह

1 रोगभनम

2 ग दाि

3 ग रा

िीच नदए गए कट का परय ग कर सही उततर चनिए

(a) किि 1

(b) किि 2

(c) किि 1 और 2

(d) 1 2 और 3

Q62) नगदधा ितय क सोदभण म निमननिखित कथि ो पर निचार

कीनजए

1 नगदधा नबहार की मनहिाओो क दवारा तयौहार क

समय और फसि की बिाई तथा कटाई क

अिसर पर नकया जाि िािा एक पारोपररक

दहाती ितय ह

2 इस ितय क दवारा मनहिाऐो अपिी परसननता

परकट करती ह तथा नगदधा क परदशणि क

माधयम स परि िचणसव िाि समाज म

मनहिाओो की दबी हई भाििाओो क परकट

करती ह

उपयणकत कथि ो म स कौि-सास सही हह

(a) किि 1

(b) किि 2

(c) 1 और 2 द ि ो

(d) ि त 1 ि ही 2

Q63) निमननिखित कथि ो पर निचार कीनजए

1 मलला शाह बदखशी दारा नशक ह क

आधयाखतमक गर थ

2 औरोगरब ि मजम-उि-बहरीि या द समदर ो

का सोगम िामक उललििीय रचिा नििी थी

3 दारा नशक ह क अपि पिणज अकबर क गर ो

क उततरानरकारी क रप म दिा गया था

नजसम उसि रानमणक बहििाद और समनवयता

क बढ़ािा नदया था

उपयणकत कथि ो म स कौि-सास सही हह

(a) किि 1 और 3

(b) किि 2

(c) किि 1 और 2

(d) 1 2 और 3

RAUSIAS-FC19E1003 23

Q60) Consider the following statements about

the Tribal Cooperative Marketing

Development Federation of India

(TRIFED)

1 It is a national-level apex

organization functioning under the

administrative control of Ministry

of Home Affairs Government of

India

2 The main objective of TRIFED is

socio-economic development of

tribal people in the country

Which of the statements given above

isare correct

(a) 1 only

(b) 2 only

(c) Both 1 and 2

(d) Neither 1 nor 2

Q61) Which of the following novels isare

written by Premchand

1 Rangabhumi

2 Godan

3 Gora

Select the correct answer using the code

given below

(a) 1 only

(b) 2 only

(c) 1 and 2 only

(d) 1 2 and 3

Q62) Consider the following statements about

Giddha dance

1 Giddha is a traditional pastoral

dance performed by the women of

Bihar at festival times and at the

sowing and reaping of the harvest

2 By this dance the women reveal

their joy expel their suppressed

feelings in a male dominated

society through the performance of

Giddha

Which of the statements given above

isare correct

(a) 1 only

(b) 2 only

(c) Both 1 and 2

(d) Neither 1 nor 2

Q63) Consider the following statements

1 Mullah Shah Badakhshi was the

spiritual mentor of Dara Shukoh

2 Aurangzeb wrote the remarkable

work called ldquoMajma-ul-Bahrainrdquo or

the ldquoThe confluence of two seasrdquo

3 Dara Shukoh was seen as

inheriting the qualities of his

ancestor Akbar in that he

promoted religious pluralism and

syncretism

Which of the statements given above

isare correct

(a) 1 and 3 only

(b) 2 only

(c) 1 and 2 only

(d) 1 2 and 3

RAUSIAS-FC19E1003 24

Q64) निमननिखित कथि ो पर निचार कीनजए

1 ग मतशवर परनतमा निोधयनगरी पहाड़ी पर खसथत ह

2 शरिरबिग िा िह सथाि ह जहाो मौयण िोश क

सोसथापक चोदरगपत मौयण अपि नसोहासि क

तयागि क बाद जि तपसवी बि गए थ

उपयणकत कथि ो म स कौि-सास सही हह

(a) किि 1

(b) किि 2

(c) 1 और 2 द ि ो

(d) ि त 1 ि ही 2

Q65) निमननिखित कथि ो पर निचार कीनजए

1 पराताखतवक साकषय स पता चिता ह नक पराची

घाटी सभयता हड़पपा और म हिज दाड़ द ि ो

की पिणिती ह

2 पराची िदी भििशवर स निकिती ह

उपयणकत कथि ो म स कौि-सास सही हह

(a) किि 1

(b) किि 2

(c) 1 और 2 द ि ो

(d) ि त 1 ि ही 2

Q66) निमननिखित कथि ो म स कौि-सास सही हह

1 िजराह क समारक ो क समह का निमाणर

चोदि राजिोश क शासिकाि क दौराि हआ

था

2 य समारक हररिोदर पिणत शरोििा म खसथत ह

3 म रक क यातरी इबन बतता ि अपि सोसमरर ो

म िजराह क मोनदर ो की यातरा का उललि

नकया था तथा इन काजराण िाम स समब नरत

नकया था

िीच नदए गए कट का परय ग कर सही उततर चनिए

(a) किि 1

(b) किि 1 और 2

(c) किि 2 और 3

(d) किि 1 और 3

Q67) निमननिखित कथि ो म स कौि-सास सही हह

1 डॉ बी आर अमबडकर ि दी एनिनहिशि

ऑफ़ कासट (The Annihilation of Caste)

नििी थी नजसम उन ोि नहोद रमण म िोशािगत

पजारी की परथा क उनमिि की आिशयकता

पर बि नदया था

2 डॉ राजदर परसाद ि थॉटस ऑि पानकसताि

(Thoughts on Pakistan) िामक पसतक

नििी थी

िीच नदए गए कट का परय ग कर सही उततर चनिए

(a) किि 1

(b) किि 2

(c) 1 और 2 द ि ो

(d) ि त 1 ि ही 2

Q68) निमननिखित कथि ो म स कौि-सास सही हह

1 महरगढ़ भारतीय उपमहादवीप म एक परनसदध

ििपािार बसती ह ज नसोर पराोत पानकसताि म

खसथत ह

2 बरणह म म कतत ो क उिक सवामी क साथ कबर ो

म दफिाया जाता था

िीच नदए गए कट का परय ग कर सही उततर चनिए

(a) किि 1

(b) किि 2

(c) 1 और 2 द ि ो

(d) ि त 1 ि ही 2

Q69) निमननिखित कथि ो म स कौि-सास सही हह

1 काकानटय मोनदर अनरकतर नशि क समनपणत

2 हिमक ोडा म हजार-सतोभ िाि मोनदर (The

Thousand-Pillared Temple) का निमाणर

काकानटय समराट रदर ि करिाया था

िीच नदए गए कट का परय ग कर सही उततर चनिए

(a) किि 1

(b) किि 2

(c) 1 और 2 द ि ो

(d) ि त 1 ि ही 2

RAUSIAS-FC19E1003 25

Q64) Consider the following statements

1 Gommateshwara Statue is located

on the Vindyagiri Hill

2 Shravanabelagola is the place

where Chandragupta Maurya the

founder of the Mauryan dynasty

became a Jain ascetic after

relinquishing his throne

Which of the statements given above

isare correct

(a) 1 only

(b) 2 only

(c) Both 1 and 2

(d) Neither 1 nor 2

Q65) Consider the following statements

1 Archaeological evidence shows

that the Prachi Valley Civilisation

predates both Harappa and

Mohenjo-Daro

2 The Prachi river originates from

Bhubaneswar

Which of the statements given above

isare correct

(a) 1 only

(b) 2 only

(c) Both 1 and 2

(d) Neither 1 nor 2

Q66) Which of the following statements

isare correct

1 The Khajuraho group of

monuments was built during the

rule of the Chandela dynasty

2 These monuments are located in

Harischandra mountain range

3 Ibn Battuta the Moroccan

traveller in his memoirs mentioned

visiting Khajuraho temples and

called them Kajarra

Select the correct answer using the code

given below

(a) 1 only

(b) 1 and 2

(c) 2 and 3

(d) 1 and 3

Q67) Which of the following statements

isare correct

1 Dr BR Ambedkar wrote the

Annihilation of Caste emphasising

the need to do away with the

practice of hereditary priesthood in

Hinduism

2 The book lsquoThoughts on Pakistanrsquo

was written by Dr Rajendra

Prasad

Select the correct answer using the code

given below

(a) 1 only

(b) 2 only

(c) Both 1 and 2

(d) Neither 1 nor 2

Q68) Which of the following statements

isare correct

1 Mehrgarh is a famous Neolithic

settlement in the Indian

subcontinent which is situated in

Sindh province Pakistan

2 At Burzahom dogs were buried

with their masters in their graves

Select the correct answer using the code

given below

(a) 1 only

(b) 2 only

(c) Both 1 and 2

(d) Neither 1 nor 2

Q69) Which of the following statements

isare correct

1 The Kakatiya temples are

dedicated mostly to Siva

2 The Thousand-Pillared Temple at

Hanamkonda was built by the

Kakatiya king Rudra

Select the correct answer using the code

given below

(a) 1 only

(b) 2 only

(c) Both 1 and 2

(d) Neither 1 nor 2

RAUSIAS-FC19E1003 26

Q70) निमननिखित कथि ो म स कौि-सास सही हह

1 अहमदाबाद नमि हड़ताि क दौराि महातमा

गाोरी ि शरनमक ो क पकष क मजबत करि क

निए आमरर अिशि नकया था

2 अिशि स नमि मानिक ो पर दबाि पड़ा था ज

अोततः शरनमक ो क िति म 15 परनतशत की िखदध

करि क निए सहमत हए थ

िीच नदए गए कट का परय ग कर सही उततर चनिए

(a) किि 1

(b) किि 2

(c) 1 और 2 द ि ो

(d) ि त 1 ि ही 2

Q71) निमननिखित म स नकसक नकिक भारत स यिसक

की माििता की अमतण साोसकनतक निरासत की

परनतनिनर सची (The UNESCOrsquos List of the

Representative List of the Intangible

Cultural Heritage of Humanity) म शानमि

नकया गया ह

1 मनडयटट

2 सोकीतणि

3 को भ मिा

िीच नदए गए कट का परय ग कर सही उततर चनिए

(a) किि 1 और 2

(b) किि 2 और 3

(c) किि 3

(d) 1 2 और 3

Q72) निमननिखित जिजानतय ो म स कौि-सीसी ो

जिजानतजिजानतया िागािड स सोबोनरत हह

1 अोगामी

2 ककी

3 जारिा

िीच नदए गए कट का परय ग कर सही उततर चनिए

(a) किि 1

(b) किि 1 औऔ 2

(c) किि 2

(d) 1 2 और 3

Q73) निमननिखित कथि ो म स कौि-सास सही हह

1 राषटर कट सामराजय की सथापिा दोनतदगण ि की थी

नजसि मानयाित म अपिी राजरािी की

सथापिा की थी

2 राषटर कट समराट अम घििण एक ििक था और

उस कनिताओो पर पहिी कननड़ पसतक नििि

का शरय नदया जाता ह

िीच नदए गए कट का परय ग कर सही उततर चनिए

(a) किि 1

(b) किि 2

(c) 1 और 2 द ि ो

(d) ि त 1 ि ही 2

Q74) निमननिखित कथि ो म स कौि-सास सही हह

1 कशब चोदर सि ि ततवब नरिी सभा की

अधयकषता की थी ज आधयाखतमक सतय की

ि ज म सोिि थी

2 बरहम समाज ि मािि गररमा पर बि नदया

मनतणपजा का निर र नकया और सती परथा जसी

सामानजक बराइय ो की आि चिा की

िीच नदए गए कट का परय ग कर सही उततर चनिए

(a) किि 1

(b) किि 2

(c) 1 और 2 द ि ो

(d) ि त 1 ि ही 2

Q75) निमननिखित कथि ो म स कौि-सास सही हह

1 भारत म नचशती नसिनसिा खवाजा म इिददीि

नचशती क दवारा सथानपत नकया गया था

2 नचशती परोपरा की एक परमि निशिता

आतमसोयम थी नजसम साोसाररक म ह स दरी

बिाए रििा शानमि था

िीच नदए गए कट का परय ग कर सही उततर चनिए

(a) किि 1

(b) किि 2

(c) 1 और 2 द ि ो

(d) ि त 1 ि ही 2

RAUSIAS-FC19E1003 27

Q70) Which of the following statements

isare correct

1 During the Ahmedabad Mill Strike

Mahatma Gandhi undertook a fast

unto death to strengthen the

workersrsquo resolve

2 The fast had effect of putting

pressure on mill owners who

finally agreed to give the workers a

15 per cent increase in wages

Select the correct answer using the code

given below

(a) 1 only

(b) 2 only

(c) Both 1 and 2

(d) Neither 1 nor 2

Q71) Which of the following are included in

the UNESCOrsquos list of the representative

list of the intangible cultural heritage of

humanity from India

1 Mudiyettu

2 Sankirtana

3 Kumbh Mela

Select the correct answer using the code

given below

(a) 1 and 2 only

(b) 2 and 3 only

(c) 3 only

(d) 1 2 and 3

Q72) Which of the following tribes isare

related to Nagaland

1 Angami

2 Kuki

3 Jarawa

Select the correct answer using the code

given below

(a) 1 only

(b) 1 and 2 only

(c) 2 only

(d) 1 2 and 3

Q73) Which of the following statements

isare correct

1 Rashtrakuta kingdom was founded by Dantidurga who established his capital at Manyakhet

2 Amoghavarsha a Rashtrakuta king was an author and is credited with writing the first

Kannada book on poetics

Select the correct answer using the code given below

(a) 1 only

(b) 2 only

(c) Both 1 and 2

(d) Neither 1 nor 2

Q74) Which of the following statements isare correct

1 Keshab Chandra Sen headed the Tattvabodhini Sabha which was engaged in search of spiritual truth

2 The Brahmo Samaj laid emphasis on human dignity opposed idolatry and criticized such social

evils as the practice of Sati

Select the correct answer using the code given below

(a) 1 only

(b) 2 only

(c) Both 1 and 2

(d) Neither 1 nor 2

Q75) Which of the following statements isare correct

1 The Chishti order was established in India by Khwaja Moinuddin

Chishti

2 A major feature of the Chishti tradition was austerity including maintaining a distance from the

worldly power

Select the correct answer using the code

given below

(a) 1 only

(b) 2 only

(c) Both 1 and 2

(d) Neither 1 nor 2

T e s t i s p a r t o f R a u rsquo s I A S T e s t s e r i e s f o r P r e l i m i n a r y E x a m 2 0 1 9

FOUNDATION + CURRENT AFFAIRS

GENERAL STUDIES (PAPER ndashI)

FOUNDATION TEST ndashIII

SUBJECT NCERT History Class VI-X + Current Affairs

Time Allowed 1frac12 Hours Maximum Marks 150

I NSTRUCT IONS

1 IMMEDIATELY AFTER THE COMMENCEMENT OF THE EXAMINATION YOU SHOULD CHECK

THAT THIS TEST BOOKLET DOES NOT HAVE ANY UNPRINTED OR TORN or MISSING PAGES OR

ITEMS ETC IF SO GET IT REPLACED BY A COMPLETE TEST BOOKLET

2 This Test Booklet contains 75 items (questions) Each item is printed both in Hindi and English

Each item comprises four responses (answers) You will select the response which you want to mark

on the Answer Sheet In case you feel that there is more than one correct response mark the

response which you consider the best In any case choose ONLY ONE response for each item

3 You have to mark all your responses ONLY on the separate Answer Sheet (OMR sheet) provided

Read the directions in the Answer Sheet

4 All items carry equal marks

5 Before you proceed to mark in the Answer Sheet the response to various items in the Test booklet

you have to fill in some particulars in the Answer Sheet as per instructions contained therein

6 After you have completed filling in all your responses on the Answer Sheet and the examination has

concluded you should hand over to the Invigilator only the Answer Sheet You are permitted to

take away with you the Test Booklet

7 Penalty for wrong answers

THERE WILL BE PENALTY FOR WRONG ANSWERS MARKED BY A CANDIDATE IN THE

OBJECTIVE TYPE QUESTION PAPERS

(i) There are four alternatives for the answer to every question For each question for which a

wrong answer has been given by the candidate one-third of the marks assigned to that

question will be deducted as penalty

(ii) If a candidate gives more than one answer it will be treated as a wrong answer even if one of

the given answers happens to be correct and there will be same penalty as above to that

question

(iii) If a question is left blank ie no answer is given by the candidate there will be no penalty for

that question

T h i s t e s t i s p a r t o f R a u rsquo s I A S T e s t s e r i e s f o r P r e l i m i n a r y E x a m 2 0 1 9

Test Code

FC19E1003

FC19H1003 29

Answers and Explanations of

NCERT History Class VI-X + Current Affairs (FC19E1003)

Q1) उततर (c)

सपषटीकरण

- ऋगवद म दविय ो और दिताओो क समवपित एक

हजार स अविक सत तर (शल क) ह

- य शल क ऋविय ो क दवारा रच गए थ और परि ो

दवारा सीख जात थ

- हालाोवक कछ शल क मवहलाओो (जस वक अपाला

घ सा ल पामदरा मतरयी और गागी) क दवारा भी रच

गए थ

- ऋगवद म सोिाद क रप म कई शल क मौजद ह

- हम विशवावमतर नामक एक ऋवि और दविय ो क

रप म पजी जान िाली द नवदय ो (वयास और

सतलज) क बीच िाताि का उदाहरण वमलता ह

- इसस पता चलता ह वक विशवावमतर िवदक काल स

सोबोवित थ

Q2) उततर (b)

सपषटीकरण

- करनल गफाओो स राख क अिशि परापत हए ह

ज इस ओर सोकत करत ह वक ततकालीन ल ग

अवि क उपय ग स पररवचत थ

- य गफाएो आोधर परदश म सथथत ह

Q3) उततर (c)

सपषटीकरण

bull बरािह म ितिमान कशमीर म सथथत एक

परागवतहावसक थथल ह जहाो ल ग गडढ क घर ो का

वनमािण करत थ

bull य घर जमीन क ख द कर बनाए जात थ तथा नीच

जान क वलए सीवियाा ह ती थी

bull ऐसा अनमान लगाया जाता ह वक य घर ठो ड क

मौसम म आशरय परदान करत थ

Q4) उततर (c)

सपषटीकरण

bull परालख-विदया (Epigraphy) क वशलालख ो क

अधययन क रप म पररभावित वकया जाता ह

bull हसतवलसखत दसतािज ो क माधयम स इवतहास

और सावहतय क अधययन क पाोडवलवप विजञान

(Manuscriptology) कहत ह

bull पराचीन लखन परणावलय ो क अधययन और

ऐवतहावसक पाोडवलवपय ो क समझन तथा वतवथ

वनिािरण क पलीओगराफी (Palaeography) कहा

जाता ह

bull नयवमजमविकस (Numismatics) वसक ो क

अधययन क सोदवभित करता ह

Q5) उततर (a)

सपषटीकरण

- चरक सोवहता चरक क दवारा वलखी गई आयिद

और िदयक-शासर पर एक महतवपणि पसतक ह

- ि भारतीय िदयक-शासर की पारमपररक परणाली

वजस आयिद क नाम स जाना जाता ह क

अभयासकताि थ

- ऐसा माना जाता ह वक चरक का विकास दसरी

शताबदी (ईसा पिि) और दसरी शताबदी (ईसवी) क

मधय हआ था

Q6) उततर (b)

सपषटीकरण

- भाग फसल ो पर वलए जान िाल कर क सोदवभित

करता ह ज कल फसल उतपादन का 16 िाो भाग

था

- ldquoकममकारrdquo शबद भवमहीन कवि शरवमक िगि क

वलए परय ग वकया जाता था

- ldquoअशवमिrdquo (वजस घ ड क बवलदान क रप म भी

जाना जाता ह) एक अनषठान ह ता था वजसम एक

घ ड क सवतोतर रप स घमन क वलए छ ड वदया

FC19H1003 30

जाता ह और राजा क सवनक उसकी रखिाली

करत थ

Q7) उततर (d)

सपषटीकरण

- ऋगववदक काल म घ ड ो क रथ ो म ज ता जाता था

ज (रथ) भवम मिवशय ो आवद पर कबजा करन क

वलए लड गए यद ो म उपय ग वकए जात थ

- इसस यह पता चलता ह वक घ ड ो यकत रथ ो का

उपय ग महाजनपद काल स काफी पहल आरमभ

हआ था

- ऋगववदक काल म मिवशय ो भवम जल आवद पर

कबजा करन क वलए तथा ल ग ो क पकडन क

वलए यद वकय जात थ

- अविकाोश परि इन यद ो म भाग वलया करत थ

- हालाोवक उस समय क ई वनयवमत सना नही ो ह ती

थी लवकन उस काल म सभाऐो ह ती थी ो वजनम

ल ग यद क मामल ो पर चचाि करत थ

- वनयवमत सनाएा महाजनपद काल का िवशषटय थी

वजनम पदल सवनक ो की विशाल सनाएा रथ तथा

हाथी शावमल ह त थ

Q8) उततर (a)

सपषटीकरण

- बद शाकय कल स सोबोवित थ और कशीनारा म

उनका वनिन हआ था

- बद न अपनी वशकषाएा पराकत भािा म दी थी ो ज

आम ल ग ो की भािा थी

Q9) उततर (c)

सपषटीकरण

- पराचीन भारत म दशिनशासर की छह शाखाएा थी ो

िशविक नयाय समखया य ग पिि वममाोसा और

िदाोत या उततर वममाोसा

- इनकी थथापना करमश कनाद गौतम कवपल

पतोजवल जावमनी और वयास ऋविय ो न की थी

Q10) उततर (b)

सपषटीकरण

महािीर की वशकषाऐो छठी शताबदी म िललभी म

सोकवलत की गई थी ो

Q11) उततर (c)

सपषटीकरण

- पारमपररक रप स चाणकय क कौविलय अथिा

विषणगपत क नाम स जाना जाता ह

- उसन अथिशासतर ज एक पराचीन भारतीय

राजनवतक आलख ह वलखा था

Q12) उततर (d)

सपषटीकरण

- भारत का राषटर ीय वचनह सारनाथ (उततर परदश) क

अश क सतमभ क ऊपर (शीिि पर) वसोह कवपिल

का एक अनरपण ह

- इस राषटर ीय वसदाोत सतयमि जयत क साथ

सोय वजत वकया गया ह

- रामपिि बल का नाम रामपिि (वबहार) क नाम पर

पडा जहाा इसकी ख ज हई थी

- यह अपन नाजक नकाशी मॉडल क वलए परवसदद

ह वजसम क मल तवचा सोिदनशील नथन ो सतकि

कान और मरबत िााग ो क शरषठतर परवतरप क

परदवशित वकया गया ह

- यह भारतीय और फारसी ततव ो का एक ससममशरण

- सोवकससा उततर परदश म सथथत ह

Q13) उततर (a)

सपषटीकरण

का िर वसोह ज एक महान य दा थ वबहार स

सोबोवित थ

Q14) उततर (b)

सपषटीकरण

िललालर शबद बड भ-सवावमय ो क वलए परय ग

वकया जाता था

FC19H1003 31

Q15) उततर (c)

सपषटीकरण

- अररकमड एक तिीय बसती थी जहाो दर दश ो स

आन िाल जहाज ो का माल उतारा जाता था

- यहाो पर ईोि ो का एक विशाल ग दाम वमटटी क

बतिन (वजनम एमफ रा - द हरी मवठय ो का लोबा

घडा - शावमल ह) और एरिाइन (Arretine)

मदभाोड पाए गए थ

- इस थथान पर र मन दीपक काोच क बन पातर और

रतन भी पाए गए थ

Q16) उततर (a)

सपषटीकरण

- मिनदर सोगम कविताओो म उसललसखत एक

तवमल शबद ह वजसका अथि ह ldquoतीन परमखrdquo

- यह तीन सततारि पररिार ो क मसखयाओो क वलए

परय ग वकया जाता ह च ल चर और पाणडय

Q17) उततर (c)

सपषटीकरण

- ऋग िद म सभा विदाथा तथा गण जसी

जनजावतय ो पर अथिा किोब पर आिाररत

सभाओो का उललख ह

- आरसमभक िवदक काल म सभाओो और सवमवतय ो

का विशि महतव ह ता था

- यहाा तक की मसखया अथिा राजा भी उनका

समथिन परापत करन क वलए आतर रहत थ

Q18) उततर (a)

सपषटीकरण

- जन िमि न ईशवर क अससततव क मानयता त दी ह

वकनत उसन ईशवर क वजना क पद स नीच रखा

- जन िमि न बौद िमि की तरह िणि परणाली की

भरतिना नही ो की थी

Q19) उततर (d)

सपषटीकरण

- च ल ो और पाणडय ो न शसकतशाली तिीय शहर ो का

विकास वकया था

- च ल ो का सबस महतवपणि शहर पहार (या

कािरीपटटीनम) था |

- मदरई पाणडय ो की राजिानी थी

Q20) उततर (b)

सपषटीकरण

- ldquoबदचररतrdquo बद का जीिन-ितताोत ह

- इस अशवघ ि क दवारा वलखा गया था

Q21) उततर (a)

सपषटीकरणः

- तवमल कवि अपपर भगिान वशि क भकत थ

- इस परकार ि एक नयनार सोत थ

Q22) उततर (d)

सपषटीकरणः

- समदरगपत एक परवसद गपत शासक था

- उसन वसक ो पर िीणा बजात हए अपनी छवि

अोवकत करिाई थी

- यह सोगीत क परवत उसक परम क दशािता ह

- हम उसकी इलाहाबाद परशससत स महतवपणि

ऐवतहावसक जानकारी वमलती ह वजसकी रचना

उसक दरबार क कवि हररसन न की थी

Q23) उततर (b)

सपषटीकरणः

- विकरम सोित की शरआत ििि 58 ईसा पिि म

चनदरगपत वदवतीय न की थी

- यह शक ो पर उसकी जीत और उस विकरमावदतय

की पदिी वमलन क उपलकषय म आरमभ वकया गया

था

FC19H1003 32

- बानभटट न हिििििन का जीिन-ितताोत हििचररत

(ज सोसकत म थी) वलखी थी

Q24) उततर (c)

सपषटीकरणः

- सोवि-विगरावहका यद एिो शाोवत का मोतरी

- साथििाह वयापाररय ो क कावफल ो का नता

Q25) उततर (a)

सपषटीकरणः

- जआन झाोग (हसआन रताोग ndash Hsuang Tsang)

एक चीनी यातरी था ज हिििििन क शासनकाल म

भारत आया था

- ििि 630 ईसवी स ज दशक आरमभ हआ था उसम

जआन झाोग मधय एवशया ईरान और

अफग़ावनसतान की यातरा करन क पशचात कशमीर

क रासत स भारत आया था

- उसन उततर स पिि तक की यातरा की और िह

लगभग 2 ििि वबहार म रहा

- जआन झाोग न नालनदा विशवविदयालय म विदयावथिय ो

और विदवान ो क साथ पारसपररक विचार-विमशि

वकया थथानीय भािाओ ा म वनपणता परापत की तथा

बौद सतप ो की ख ज की

Q26) उततर (c)

सपषटीकरणः

- परदवकषणा पथ बौद िासतकला म सतप क चार ो

ओर बनाया जान िाला एक घमािदार पथ ह ता

- परशन म वदए गए बाकी क तीन ो ततव वहोद मसनदर ो की

िासतकला क भाग ह

Q27) उततर (d)

सपषटीकरणः

परशन म वदए गए सभी मोवदर ो म वयापक रप स

ईोि ो (पकी ईोि ो) का परय ग पतथर ो क साथ हआ

Q28) उततर (c)

सपषटीकरण

- महममद कली कतब शाह ग लकणडा का सलतान

था

- िह अकबर का समकालीन था

- सावहतय और िासतकला म उसकी अतयाविक

रवच थी

- िह एक महान कवि था

- िह दसखनी उदि फारसी और तलग म वलखता था

- उसन अपन पीछ एक विसतत वदिान (सोगरह)

छ डा ह

- अभी हाल ही म तलोगाना म ग लकणडा क वकल

क अनदर खदाई वकय गए बाग-ए-नाया वकला

बाग क चार ो ओर रप-रखा क मानवचतरण क

वलए भारतीय परातासतवक सिकषण (The

Archaeological Survey of India ndash ASI)

गराउणड पनीिर विोग रडार (Ground Penetrating

Radar) का परय ग करगा

Q29) उततर (a)

सपषटीकरणः

- वसलपपावदकारम एक तवमल महाकावय ह वजसकी

रचना इलाोग क दवारा लगभग 1800 ििि पिि की

गई थी

- यह क िलन नामक एक वयापारी की कहानी ह

ज माििी नामक एक गवणका (िशया) स परम

करन लगा था

- मवनमकलाई क िलन और माििी की पतरी की

कहानी ह

Q30) उततर (a)

सपषटीकरण

- चरक आयिद और वचवकरता की एक महतवपणि

रचना चरक सोवहता क लखक ह

- बरहमगपत क अपनी रचना बरहम-सफि-वसदानत

(ज एक खग लीय रचना ह) क कारण परवससद

वमली

FC19H1003 33

- बगदाद म इसका अनिाद अरबी भािा म वकया

गया था

- इसका इसलावमक गवणत और खग ल-विजञान पर

महतवपणि परभाि पडा था

- बाद म अपन जीिनकाल म बरहमगपत न

ldquoखोडखयाकrdquo वलखी ज एक खग लीय पससतका

(एक छ िी पसतक) थी

- इसम आयिभटट की अिि-रावतर क परतयक वदन की

शरआत परणाली का परय ग वकया गया था

Q31) उततर (c)

सपषटीकरण

- अमीर खसर एक परवसद सफी सोगीतकार कवि

और विदवान थ

- 1318 म उनह ोन पाया वक इस भवम (वहोदसतान) क

हर कषतर म अलग-अलग भािा थी लाहौरी

कशमीरी दवारसमदरी (दवकषणी कनाििक म)

तलोगाना (आोधर परदश म) गजरी (गजरात म)

माबारी (तवमलनाड म ) अििी (पिी उततर परदश

म) और वहोदिी (वदलली क आस-पास क कषतर म)

आवद

- उनह न यह बताया वक सोसकत वकसी भी कषतर स

सोबोवित नही ो थी और किल बराहमण ही इस भािा

का जञान रखत थ

Q32) उततर (c)

सपषटीकरण

- वहरणय-गभि सववणिम गभि क सोदवभित करता ह

- जब बराहमण ो की सहायता स यह अनषठान वकया

जाता था त यह माना जाता था वक बवल दन िाल

का कषवतरय क रप म पनजिनम ह गा

Q33) उततर (d)

सपषटीकरण

- कदमई भवम राजसव पर कर क सोदवभित करता

- गवावलयर परशससत म नागभि क दवारा वकय गए

श िण का िणिन वकया गया ह |

- नागभि एक परवतहार राजा था

Q34) उततर (b)

सपषटीकरण

- राजतरो वगनी 12िी ो शताबदी म कलहन क दवारा

रवचत एक सोसकत पसतक (िकसट) ह

- यह परारसमभक भारत की ऐवतहावसक इवतितत थी

- तकि सोगत रप स इस अपन परकार की सिोततम

और सिािविक विशवसनीय कवत माना जाता ह

- यह कशमीर कषतर क पराचीनतम समय स लकर

उसकी रचना की तारीख तक क समपणि इवतहास

का आचछादन करती ह

Q35) उततर (c)

सपषटीकरण

- गााि की आम सभा क ldquoउरrdquo कहा जाता था

- ldquoउरrdquo म गााि क सभी कर दन िाल वनिासी

शावमल ह त थ

Q36) उततर (a)

सपषटीकरण

- वदलली सलतनत म ldquoतारीखrdquo इवतहास लखन का

एक रप था

- ldquoतािरीखrdquo क लखक विदवान परि ह त थ वजनम

सवचि परशासक इतयावद शावमल थ

Q37) उततर (a)

सपषटीकरण

- अलाउददीन सखलजी अपन सवनक ो क ितन का

भगतान नकद म करता था न वक इकता क रप

- सवनक अपना सामान वदलली म वयापाररय ो स

खरीदत थ अतः इस बात का भय था वक वयापारी

कही ो िसतओो का मलय न बिा द

- इसकी र कथाम क वलए अलाउददीन सखलजी न

वदलली म कीमत ो क वनयसित वकया

FC19H1003 34

- अविकारीगण धयानपििक मलय ो का सिकषण करत

थ तथा ज वयापारी वनिािररत मलय पर माल नही ो

बचत थ उनक दसणडत वकया जाता था

Q38) उततर (d)

सपषटीकरण

- वदलली सििपरथम त मर राजपत ो क अिीन उनक

सामराजय की राजिानी बनी थी

- 12िी ो शताबदी क मधय म अजमर क चौहान ो

(वजनह चाहमान ो क नाम स भी जाना जाता ह) न

त मर राजपत ो क परावजत वकया था

- त मर ो और चौहान ो क अिीन वदलली एक

महतवपणि िावणसजयक क दर बन गया था

- कई जन वयापारी यहाा रहन लग थ और उनह ोन

कई मोवदर भी बनिाए

- यहाा पर मवदरत वसक वजनह ldquoदहलीिालrdquo क नाम

स जाना जाता था वयापक रप स परचलन म थ

Q39) उततर (c)

सपषटीकरण

- म ठ की मसिद का वनमािण वसको दर ल दी क

राजयकाल म उसक मिी क दवारा करिाया गया

था

- बगमपरी मसिद का वनमािण महममद तगलक क

शासनकाल म हआ था

- यह मसिद विशव का पणयथथान (The

Sanctuary of the World) और वदलली म महममद

तगलक की नई राजिानी जहाोपनाह की मखय

मसिद थी

- कववत- अल - इसलाम मसिद का विसतार

इलतसिश और अलाउददीन सखलजी न वकया था

- मीनार का वनमािण तीन सलतान ो कतबददीन ऐबक

इलतसिश और वफर ज शाह तगलक क दवारा

करिाया गया था

Q40) उततर (c)

सपषटीकरण

- मगल ो क अिीन मनसबदार शबद उस वयसकत क

वलए सोदवभित वकया जाता था वजसक पास मनसब

(अथाित पद) ह ता था

- उस अपना ितन राजसव कायो वजनह जागीर कहत

थ क रप म परापत ह ता था

Q41) उततर (b)

सपषटीकरण

- ldquoभारत छ ड आोद लनrdquo वबरविश शासन क

सखलाफ ल ग ो का एक सवाभाविक विदर ह था

- असखल भारतीय काोगरस सवमवत न 8 अगसत 1942

क बमबई म एक बठक का आय जन वकया था

- इस बठक म परवसद सोकलप ldquoभारत छ ड rdquo क

पाररत वकया गया और इस उददशय क परापत करन

क वलए गाोिी क नततव म एक अवहोसक जन सोघिि

आोद लन की शरआत का परसताि वदया गया

- लवकन अगल ही वदन गाोिी और काोगरस क अनय

परमख नताओो क वगरफतार कर वलया गया

- काोगरस क एक बार वफर अिि घ वित वकया गया

था

Q42) उततर (c)

सपषटीकरण

- साइमन कमीशन यनाइविड वको गडम क सात

साोसद ो का एक समह था

- इस वबरविश भारत क वलए सोििावनक सिार ो का

सझाि दन क वलए गवठत वकया गया था

- इस आय ग म िररषठ वबरविश राजनता सर जॉन

साइमन क नततव म किल वबरविश सदसय ही

शावमल थ

- इसवलए भारत क ल ग ो न साइमन कमीशन क

आगमन क विरद आोद लन वकया था

Q43) उततर (a)

सपषटीकरण

bull दादा भाई नौर जी भारत म वबरविश शासन क

आवथिक पररणाम ो क बार म अपनी विर िी

(परवतकल) राय क वलए जान जात थ

FC19H1003 35

bull अपन कई लख ो और भािण ो म विशि रप स

ldquoपाििी एो ड अन-वबरविश रल इन इसणडया

(Poverty and Un-British Rule in India) म

नौर जी न यह तकि वदया वक भारत पर अतयविक

कर लगाया गया था और इसकी सोपवतत इोगलड की

ओर परिावहत की जा रही थी

bull उनह ोन पराचीन भारतीय गरोथ ो की वयाखया करन

का और भारतीय ो क आिविशवास क बहाल

करन पर कायि नही ो वकया था

उनह ोन वकसी और बात स पहल सभी सामावजक

बराइय ो क उनमलन की आिशयकता पर भी बल

नही ो वदया था

Q44) उततर (c)

सपषटीकरण

bull अगसत 1932 म वबरविश परिानमोतरी मकड नालड न

अपन साोपरदावयक परसकार (The Communal

Award) की घ िणा की थी

bull यह भारत क कई साोपरदावयक वहत ो क बीच विवभनन

सोघिो क हल करन क वलए वबरिन का एकतरफा

परयास था

bull यह परसकार (Award) बाद म 1935 क

अविवनयम (The Act of 1935) म शावमल वकया

गया था

bull इस साोपरदावयक परसकार न मससलम ो क वलए

आरवकषत एक अलग वनिािचक मणडल फॉमिल का

विसतार अनय अलपसोखयक ो क वलए वकया था

वजसम वसख ो भारतीय ईसाइय ो आोगल-भारतीय

समदाय यर पीय समदाय तथा विवशषट कषतरीय

समह ो क शावमल वकया गया था

bull गाोिी न इस परसताि क भारतीय समाज क

विभावजत करन क वलए एक घवणत वबरविश

सावजश क रप म दखा और उसक सखलाफ

आमरण अनशन वकया

Q45) उततर (b)

सपषटीकरण

मौजदा आयात और वनयाित क अवतररक़त

औपवनिवशक भारत क वनमनवलसखत खचो क

वलए एक विशिवनवशचत िन रावश भी दनी पडती

थी

(i) परशासन क वयय

(ii) सना क रख-रखाि क वयय

(iii) यद क वयय

(iv) सिावनितत अविकाररय ो की पशन तथा

(v) वबरिन दवारा अपनी उपवनिश बसती

(कॉल नी) क रख-रखाि क वयय

इनह गह शलक (Home Charges) क रप म

जाना जाता था और लगभग परी तरह स भारत क

दवारा इनका भगतान वकया जाता था

bull गह शलक म वनमनवलसखत घिक शावमल थ

(i) भारतीय ऋण पर दय बयाज

(ii) ईसट इोवडया को पनी क शयरिारक ो क

लाभाोश

(iii) लोदन म भारत कायािलय चलान क वलए िन

(iv) भारत म वनयकत वबरविश कवमिय ो क ितन

और पशन का भगतान करन क वलए िन

(v) रलि पर बयाज

(vi) नागररक और सनय शलक

(vii) इोगलड म सट र (सामगरी) की खरीद

Q46) उततर (b)

सपषटीकरण

bull भारतीय राषटर ीय काोगरस का लाहौर सतर 1929 म

जिाहरलाल नहर की अधयकषता म आय वजत

वकया गया था

bull इस सतर म भारतीय राषटर ीय आोद लन स समबसित

कई महतवपणि पररणाम सामन आय थ

(i) सििपरथम इस सतर म काोगरस क अधयकष पद

पर जिाहरलाल नहर क चना गया था ज

काोगरस म िामपोवथय ो की बिती हई ताकत

का सपषट सोकत था

(ii) दसरा इस सतर म पहली बार काोगरस न पणि

सवतोतरता की माोग क उठाया था

इस परकार की माोग काोगरस मोच स पहल कभी भी

नही ो उठाई गई थी

Q47) उततर (b)

सपषटीकरण

FC19H1003 36

bull इस ररप िि न वकसी भी समदाय क वलए पथक

वनिािचक मोडल अथिा अलपसोखयक ो क वलए

भाराोश की वसफाररश नही ो की थी

bull तथावप इस ररप िि न उन पराोत ो म अलपसोखयक

सीि ो क आरकषण की अनमवत दी थी जहाा पर कम

स कम दस परवतशत अलपसोखयक ह

bull लवकन यह समदाय क आकार क अनपात म ह ना

चावहए था

bull इस ररप िि म भारत क वलए पणि सवतोतरता क

वलए क ई पराििान नही ो था

Q48) उततर (c)

सपषटीकरण

bull आरो वभक िवदक आयो का िमि मखय रप स

परकवत की पजा और यजञ था

bull परारो वभक आयि िमि परकवत की पजा क समान था

bull िासति म उनक चार ो ओर की शसकतयाा वजनह न

त ि वनयोवतरत कर सकत थ और न ही समझ पाए

थ उनह वदवयता क साथ वनिवशत वकया गया तथा

उनह मादा या नर दिीदिताओो क रप म

परतीकतव वकया गया था

bull उनह ोन कछ यजञ ो का भी वनषपादन वकया था

Q49) उततर (b)

सपषटीकरण

bull सडक और नदी-मागि (जल-मागि) डकती स

सरवकषत नही ो थ

bull उललखनीय ह वक हिििििन क शासनकाल क

दौरान यआन चिाोग (हयएन साोग) का सारा

सामान लि वलया गया था

Q50) उततर (c)

सपषटीकरण

परशन म वदए गए द न ो कथन सही ह

Q51) उततर (b)

सपषटीकरण

bull परोदर दास एक सोत और भगिान कषण क एक

महान भकत थ

bull परोदर दास क कनाििक सोगीत क वपतामह क

रप म जाना जाता ह

bull यदयवप उनक जनम-थथान क बार म काफी

अिकल लगाई जाती रही ह

bull तथावप अब कननड विशवविदयालय हमपी क दवारा

गवठत एक विशिजञ सवमवत इस वनषकिि पर पहोची

ह वक उनका जनम थथान सोभितया कनाििक का

एक छ िा-सा गााि कषमपरा (वशिम गगा वजला)

था

Q52) उततर (c)

सपषटीकरण

bull शरी तयागराज शरी शयाम शासतरी और शरी मथसवामी

दीवकषतर क कनाििक सोगीत की वतरमवति माना

जाता ह

bull उनक कारण ही 18िी ो-19िी ो शताबदी म कनाििक

सोगीत का सववणिम यग आया था

Q53) उततर (d)

सपषटीकरण

bull अभी हाल ही म लौह यगीन-महापािावणक काल

का 2000 ििि पराना एक दलिभ सारक फगस

(Sarcophagus) (पतथर का ताबत) क ललम क

वियर गाोि (क वयलडी क पास वजला क वझक ड

करल राजय) की एक रॉक-कि गफा स ख जा गया

bull यह ताबत वजसम हविय ो क िकड थ खदाई क

दौरान वमला

bull अभी तक इस परकार की दलिभ ख ज करल क

मातर द ही थथान ो स हई ह

bull य द न ो सारक फगी (Sarcophagi) (पतथर क

ताबत) चियर और अथ ली (वजला क वझक ड) क

महापािाण थथल ो स वमल ह

Q54) उततर (a)

सपषटीकरण

FC19H1003 37

दवकषण भारत म महापािाण सोसकवत एक पणि

विकवसत लौह यगीन सोसकवत थी

Q55) उततर (d)

सपषटीकरण

bull च ल पाणडय और करलपतर (चर) इन तीन ो का

उललख अश क क अवभलख ो म वकया गया ह

bull सोभितः य भौवतक सोसकवत क उततर

महापािावणक चरण म थ

Q56) उततर (d)

सपषटीकरण

bull भीमा-क रगाोि की लडाई ततीय आोगल-मराठा

यद का वहससा थी

Q57) उततर (b)

सपषटीकरण

bull राजकमार शकल न गाोिीजी क चोपारण आन तथा

वतनकवथया परणाली स जडी समसया की जाोच क

वलए रारी करन क वलए दश भर म उनका

अनसरण वकया था

bull बज वकश र राजदर परसाद महादि दसाई और

नरहरी पाररख चोपारण सतयागरह क दौरान गाोिी

जी क सहय गी थ

Q58) उततर (b)

सपषटीकरण

bull बराहमण ो और बौद मठिाररय ो क कर-मकत गााि

अनदान म दन की परथा सतिाहन ो न आरमभ की

थी

Q59) उततर (c)

सपषटीकरण

इस कायिकरम क उददशय वनमनानसार ह

(i) बवनयादी पयििन आिाररक सोरचना का विकास

करना

(ii) चयवनत (पहचान वकय गए) कषतर ो म आजीविका क

सजन क वलए दश क साोसकवतक और विरासत

मलय ो क बिािा दना

(iii) विरासत समारक थथल ो पर विशव सतरीय आिाररक

सोरचना विकवसत करक एक सतत तरीक स

पयििक आकििण म िसद करना

(iv) थथानीय समदाय ो की सवकरय भागीदारी क माधयम

स र रगार ो का सजन करना

(v) र रगार उतपादन और आवथिक विकास क वलए

पयििन कषमता का उन पर परभाि का उपय ग

करना तथा

(vi) िारणीय पयििन आिाररक सोरचना का विकास

करना और उसका उवचत सोचालन तथा

रखरखाि सवनवशचत करना

Q60) उततर (b)

सपषटीकरण

bull यह वनकाय ििि 1987 म अससततव म आया था

bull यह एक राषटर ीय सतर का शीिि सोगठन ह ज भारत

सरकार क जनजातीय मामल ो क मोतरालय क

परशासवनक वनयोतरण क अिीन काम कर रहा ह

bull इसका पोजीकत और परिान कायािलय नई वदलली

म सथथत ह

Q61) उततर (c)

सपषटीकरण

bull परमचोद क उपनयास ो म परमाशरम रोगभवम गबन

कमिभवम और ग दान शावमल ह

bull ग रा रिी ोदरनाथ िग र क दवारा रवचत उपनयास ह

bull अभी हाल ही म मोशी परमचोद की 138िी ो जयोती दश

भर म मनाई गई थी

Q62) उततर (b)

सपषटीकरण

bull ldquoवगदाrdquo पोजाब (भारत) एिो पावकसतान की

मवहलाओो क दवारा तयौहार क समय और फसल

की बिाई तथा किाई क अिसर पर वकया जान

िाला एक पारोपररक दहाती नतय ह

FC19H1003 38

bull इस नतय क माधयम स पोजाबी मवहलाऐो अपनी

परसननता परकि करती ह तथा वगदा क परदशिन क

माधयम स परि िचिसव िाल समाज म मवहलाओो

की दबी हई भािनाओो क परकि करती ह

bull चोवक इस नतय का परि ो क साथ क ई सोबोि नही ो

ह अतः किल मवहलाऐो ही इसम भाग ल सकती

bull हर साल तीज समार ह क दौरान पोजाब म वगदा

नतय वकया जाता ह

तीज भारत क कछ भाग ो म मवहलाओो क दवारा

मनाया जान िाल कई तयौहार ो क वलए एक

वयापक नाम ह

Q63) उततर (a)

सपषटीकरण

- मजम-उल-बहरीन या द समदर ो का सोगम

नामक उललखनीय रचना दारा वशक ह क दवारा

वलखी थी

- भारत क उपराषटर पवत शरी एम िकया नायड न कहा

ह वक राजकमार दारा वशक ह की रचनाएा शाोवत

और सदभाि क बिािा दन क वलए एक तारा सर त

क रप म सामन आ सकती ो ह

- उपराषटर पवत गत ििो क भला वदए गए राजकमार

दारा वशक ह क परदवशित परचवलत करन हत

आय वजत एक परदशिनी का दौरा करन क बाद एक

सभा क सोब वित कर रह थ

- इस परदशिनी का आय जन फर क इस गौवियर

(Francois Gautier) क दवारा lsquoइोवदरा गाोिी नशनल

सिर फॉर द आििसrsquo (The Indira Gandhi

National Centre for the Arts) नई वदलली म

वकया गया था

Q64) उततर (c)

सपषटीकरण

- ग मतशवर परवतमा जन भगिान बाहबली क

समवपित ह

- यह एक एक-चटटानी पतथर की मवति ह

- राषटर पवत राम नाथ क विोद न शरिणबलग ला

(कनाििक) म आय वजत वकय जान िाल भवय

अवभिक समार ह महामसतकावभिक का

उदघािन वकया था

- यह समार ह 12 ििो म एक बार ह ता ह

Q65) उततर (c)

सपषटीकरण

bull पराची घािी पराची नदी क चार ो ओर फली हई थी

bull पराची घािी िीर-िीर विलपत ह गई थी

bull पराची नदी भिनशवर स वनकलती ह

bull यह महानदी की एक सहायक नदी ह और यह

परी खदाि किक तथा जगतवसोहपर वजल ो स

ह कर बहती ह

bull इस नदी क पर कषतर क पराची घािी कहा जाता ह

bull यह नदी बोगाल की खाडी म वगरती ह

परातासतवक साकषय स पता चलता ह वक पराची घािी

सभयता हडपपा और म हनज दाड द न ो की

पिििती ह

Q66) उततर (d)

सपषटीकरण

य समारक छतरपर वजल (मधय परदश) म विोधयाचल

पिित शरोखला म सथथत ह

Q67) उततर (a)

सपषटीकरण

bull थॉिस ऑन पावकसतान नामक पसतक डॉ बी

आर अमबडकर न वलखी थी

bull डॉ बी आर अमबडकर की जयोती क अिसर पर

भारत क राषटर पवत न भारत की इस महान हसती

क शरदाोजवल अवपित की थी

bull डॉ बी आर अमबडकर न 1924 म वडपरथड

कलावसर इोसटीटयि (दवलत िगि सोथथान -

बवहषकत वहतकाररणी सभा) और 1927 म समाज

समता सोघ की थथापना की थी

bull अमबडकर का धयान वशकषा कषतर की ओर भी था

bull उनह ोन वशकषा क वनमन िगो म फलान क वलए

पीपलस एजकशन स साइिी (The Peoples

Education Society) क नाम स महाविदयालय ो क

नििकि और छातरािास ो की थथापना की थी

FC19H1003 39

Q68) उततर (b)

सपषटीकरण

bull महरगि भारतीय उपमहादवीप म एक परवसद

निपािाण बसती ह ज बलवचसतान पराोत

पावकसतान म सथथत ह

bull दचपलली (आोधर परदश) क पास नागलर नदी क

पिी ति ो पर चना पतथर क बलॉक क विशाल

विसतार म एक पिि-ऐवतहावसक रॉक आिि थथल की

ख ज की गई ह

bull इसन 1500-2000 ईसा पिि क दौरान गोिर (आोधर

परदश) म विकवसत निपािाण सभयता पर परकाश

डाला ह

Q69) उततर (c)

सपषटीकरण

bull 12िी ो सदी और 13िी ो सदी म काकाविय िोश का

उदय हआ था

bull ि पहल कलयाण क पवशचमी चालकय ो क सामोत थ

bull परारोभ म उनह ोन िारोगल (तलोगाना) क पास एक

छ ि स कषतर पर शासन वकया था

bull उनह ोन ldquoनायक वयिथथाrdquo की शरआत की थी

वजस बाद म विजयनगर क राय शासक ो न

अपनाया और विकवसत वकया था

Q70) उततर (a)

सपषटीकरण

bull गाोिीजी क अनशन स वमल मावलक ो पर दबाि

पडा था ज अोततः शरवमक ो क ितन म 35 परवतशत

की िसद करन क वलए सहमत हए थ

bull गगल (Google) न अनसया साराभाई वजनह ोन

भारत क शरवमक आोद लन म एक अगरणी भवमका

वनभाई थी की 132िी ो जयोती डडल (Doodle) का

वनमािण करक मनाई

Q71) उततर (d)

सपषटीकरण

भारत स यनसक की मानिता की अमति साोसकवतक

विरासत की परवतवनवि सची म वनमनवलसखत शावमल ह

bull कवडयटटम करल का सोसकत रोगमोच

bull मवडयिि करल का अनषठान रोगमोच और नतय

नाविका

bull िवदक मि जाप की परोपरा

bull राजथथान क कालबवलया ल क गीत और नतय

bull रामलीला रामायण का पारोपररक परदशिन

bull सोकीतिन मवणपर का अनषठान गायन ढ ल िादन

और नतय

bull रममन भारत क गििाल वहमालय का िावमिक

तयौहार और अनषठान रोगमोच

bull जाोदीयाला गर पोजाब क ठठर ो की पीतल और

ताोब क वशलप स वनवमित बतिन ो की पारोपररक कला

bull छाऊ नतय पिी भारतीय राजय ो म जनमी शासतरीय

भारतीय नतय कला

bull लददाख का बौद मि जाप िर ाोस-वहमालयी लददाख

कषतर तथा जमम-कशमीर म पवितर बौद गरोथ ो का पाठ

bull य ग

bull नौर र

bull को भ मला

Q72) उततर (b)

सपषटीकरण

bull भारत क राषटर पवत शरी राम नाथ क विोद न

वकसामा नागालड म हॉनिवबल मह रति और

राजय गठन वदिस समार ह का उदघािन वकया

था

bull हॉनिवबल मह रति का नाम भारतीय हॉनिवबल क

नाम पर पडा ह ज एक विशाल और रोगीन जोगली

पकषी ह

bull यह पकषी नागालड राजय की अविकतर जनजावतय ो

की ल ककथाओो म उसललसखत ह

bull नागालड की परमख मानयता परापत जनजावतयाा ह

अोगामी आओ चखसोग चाोग ककी रगमा और

रवलोग आवद

bull ओोग जारिा और ससिनलीस अोडमान-वनक बार

दवीप समह की जनजावतयाा ह

FC19H1003 40

Q73) उततर (c)

सपषटीकरण

bull दकन म राषटर कि शासन दसिी ो सदी क अोत तक

लगभग 200 ििो तक रहा था

bull राषटर कि शासक अपन िावमिक विचार ो म सवहषण

bull उनह ोन न किल शि िमि और िषणि िमि बसलक

जन िमि क भी सोरकषण वदया था

bull एल रा म वशि क परवसद रॉक कि मोवदर का

वनमािण नौिी ो सदी म राषटर कि राजा कषण परथम न

करिाया था

bull उसका उततराविकारी अम घििि जन था लवकन

उसन अनय िमो क भी सोरकषण परदान वकया था

bull राषटर कि ो न मसलमान वयापाररय ो क बसन की

अनमवत दी थी

bull उनह न अपन अविराजय ो म इसलाम क उपदश दन

की भी अनमवत दी थी

bull अभी हाल ही म पाोडिलागटटा (तलोगाना) क

परागवतहावसक चटटान वचतर ो क कषरण की बिती हई

घिनाएा एक गोभीर वचोता का वििय ह

bull यह परागवतहावसक चटटान क नकसान पहाचा

सकता ह

bull पाोडिलागटटा वनमनवलसखत क वलए जाना जाता ह

- 10000 ईसा पिि स 8000 ईसा पिि क वचवतरत

चटटानी आशरय ो क वलए

- राषटर कि काल क एक 8 िी ो सदी क

वशलालख क वलए और

- 12िी ो सदी क काकविय सामराजय क वभवतत

वचतर ो क वलए

Q74) उततर (b)

सपषटीकरण

bull 1828 म राजा राम म हन रॉय न एक नय िावमिक

समाज बरहम सभा की थथापना की थी वजस बाद

म बरहम समाज क नाम स जाना गया था

bull दिदरनाथ िग र न ततवब विनी सभा की अधयकषता

की थी ज आधयासिक सतय की ख ज म सोलि

थी

bull इसका उददशय वहोद िमि क शद करन का और

एकशवरिाद (एक ईशवर म आथथा) का परचार करना

था

bull नय समाज की थथापना क आिार थ कारण

(तकि ) क द सतमभ तथा िद और उपवनिद

bull अभी हाल ही म सािारण बरहम समाज का कछ

काननी मदद ो क लकर पवशचम बोगाल सरकार क

साथ काननी वििाद चल रहा ह

Q75) उततर (c)

सपषटीकरण

bull भारत म वचशती वसलवसल की थथापना खवाजा

म इनददीन वचशती क दवारा की गयी थी

bull ि 1192 ईसवी क आसपास भारत आय थ

bull वचशतीय ो क बारहिी ो शताबदी क उततरािि म भारत

म आन िाल सफीय ो क समह ो म सबस

परभािशाली माना जाता ह

bull उनह ोन थथानीय िातािरण क साथ सफलतापििक

अनकलन वकया और उनह ोन भारतीय भसकत

परोपराओो क कई पहलओो क अपनाया

bull अजमर म सफी अपरकि खवाजा म इनददीन वचशती

की ऐवतहावसक दरगाह क एक नया रप दन की

तयारी की जा रही ह

bull इस 13िी ो शताबदी की दरगाह क ldquoसवचछ

आइकॉवनक थथल ोrdquo (Swacch Iconic Places) म

शावमल वकया गया ह ज परवतवषठत विरासत

आधयासिक और साोसकवतक थथान ो पर क वदरत

य जना ह

FC19H1003 41

ANSWERS amp EXPLANATION OF

NCERT History Class VI-X + Current Affairs

(FC19E1003)

Q1) Answer c

Explanation

Rigveda consists of more than a

thousand hymns dedicated to gods and

goddesses These hymns were

composed by sages and learnt by men

however a few were composed by

women like Apala Ghosa Lopamudra

Maitreyi and Gargi

Rigveda consists of many hymns in the

form of dialogues We get an example of

a dialogue between a sage named

Vishwamitra and two rivers (Beas and

Sutlej) that were worshipped as

goddesses This suggests that he

belonged to the Vedic period

Q2) Answer b

Explanation

Traces of ash have been found from

Kurnool Caves suggesting that people

were familiar with the use of fire

It is situated in Andhra Pradesh

Q3) Answer c

Explanation

Burzahom is a prehistoric site in

present day Kashmir where people built

pit houses which were dug into the

ground with steps leading into them

These may have provided shelter in cold

weather

Q4) Answer c

Explanation

Epigraphy is defined as the study of

inscriptions

Manuscriptology is the study of history

and literature through the use of hand

written documents

Palaeography refers to the study of

ancient writing systems and the

deciphering and dating of historical

manuscripts

Numismatics refers to the study of

coins

Q5) Answer a

Explanation

Charaka Samhita was written by

Charaka and is an important book on

Ayurveda and medicine

He was a practitioner of the traditional

system of Indian medicine known as

Ayurveda

Charaka is thought to have flourished

sometime between the 2nd century BCE

and the 2nd century CE

Q6) Answer b

Explanation

Bhaga refers to the tax on crops which

was fixed at 16th of the production

Kammakaras is the term used for the

landless agricultural labour class

Ashvamedha also known as horse

sacrifice is a ritual where a horse is let

loose to wander freely and it was

guarded by the rajarsquos men

Q7) Answer (d)

Explanation

In the Rigvedic period horses were

yoked to chariots that were used in

battles fought to capture land cattle

etc This suggests that the use of horse

chariots began much before the period

of Mahajanapadas

The battles were fought in the Rigvedic

period for cattlersquos lands water an even

to capture people Most men took part

in these wars however there was no

regular army but there were assemblies

where people met and discussed

matters of war Regular armies became

a feature in the Mjahajanapada period

including vast armies of foot soldiers

chariots and elephants

RAUSIAS-FC19E1003 42

Q8) Answer (a)

Explanation

Buddha belonged to the Sakya clan and

passed away at Kusinara

Buddha taught in Prakrit which was the

common language of people

Q9) Answer c

Explanation

There were six schools of philosophy in

ancient India These are known as

Vaishesika Nyaya Samkhya Yoga

Purva Mimansa and Vedanata or Uttara

Mimansa They were founded by sages

Kanada Gautama Kapila Patanjali

Jamini and Vyasa respectively

Q10) Answer b

Explanation

The teachings of Mahavira were

compiled at Valabhi in 6th century AD

Q11) Answer (c)

Explanation

Chanakya is traditionally identified as

Kautilya or Vishnugupta who authored

the ancient Indian political treatise the

Arthashastra

Q12) Answer d

The national emblem of India is an

adaptation of the Lion Capital atop the

Ashoka Pillar of Sarnath Uttar Pradesh

and is combined with the National

Motto Satyameva Jayate

The Rampurva Bull gets the name from

the site of its discovery Rampurva in

Bihar

It is noted for its delicately sculpted

model demonstrating superior

representation of soft flesh sensitive

nostrils alert ears and strong legs It is

a mixture of Indian and Persian

elements

Sankissa is situated in Uttar Pradesh

India

Q13) Ans(a)

Kunwar Singh was a notable leader during the Revolt of 1857 He belonged

to a royal house of Jagdispur Bihar

Q14) Answer b

Explanation

The term Vellalar was used for large

landowners

Q15) Answer c

Explanation

Arikamedu was a coastal settlement

where ships unloaded goods from

distant lands Finds here include a

massive brick warehouse pottery

including amphorae and Arretine ware

Roman lamps glassware and gems have

also been found at the site

Q16) Answer a

Explanation

Muvendar is a Tamil word mentioned in

Sangam poems meaning three chiefs

used for the heads of three ruling

families the Cholas Cheras and

Pandyas

Q17) Ans (c)

Several tribal or kin-based assemblies

such as the Sabha Vidatha and gana

are mentioned in the Rig-veda The

Sabha and the samiti mattered a great

deal in early Vedic times so much so

that the chiefs or the kings showed an

eagerness to win their support

Q18) Ans (a)

Jainism recognised the existence of the

gods but placed them lower than the

jina and did not condemn the varna

system as Buddhism did

Q19) Answer (d)

Explanation

Cholas and Pandyas had developed

powerful coastal cities The most

important city of Cholas was Puhar or

Kaveripattinam and Madurai was the

capital of Pandyas

Q20) Answer b

Explanation

Buddhacharita is the biography of

Buddha and was written by

RAUSIAS-FC19E1003 43

Ashvaghosha

Q21) Answer (a)

Explanation

Tamil poet Appar was a Shiva devotee

So he was a Nayanar saint

Q22) Answer d

Explanation

Samudragupta was a prominent Gupta

ruler whose coins depict him playing a

veena indicating his love for music We

get important historic information from

his Allahabad Prashasti which was

composed by his court poet Harisena

Q23) Answer (b)

Explanation

Vikrama Samvat was founded by

Chandragupta II in the 58 BC as a

mark of victory over the Shakas and

assumed the title of Vikramaditya

Banabhatta wrote Harshavardhanarsquos

biography the Harshacharita in

Sanskrit

Q24) Answer c

Explanation

Sandhi-vigrahika was the minister of

war and peace

Sarthavaha was the leader of the

merchant caravans

Q25) Answer a

Explanation

Xuan Zang (Hsuan-tsang) was a

Chinese traveller who came during the

reign of Harshavardhana

In the decade that began in 630 AD

Xuan Zang came to India through

Kashmir after visiting Central Asia Iran

and Afghanistan

He travelled from north to east and lived

in Bihar for a couple of years

At Nalanda University Xuan Zang

interacted with students and scholars

mastered local languages and

discovered Buddhist stupas

Q26) Answer c

Explanation

Pradakshina patha is a circular path

laid around a stupa in Buddhist

architecture While the rest are a part of

temple architecture

Q27) Answer d

Explanation

All the above-mentioned temples have

an elaborate use of bricks (baked

bricks) along with stone

Q28) Ans (c)

Muhammad Quli Qutab was the Sultan

of Golconda He was a contemporary of

Akbar was very fond of literature and

architecture

The Sultan was a great poet and he

wrote in Dakhini Urdu Persian and

Telgu and has left an extensive diwan or

collection

Recently the Archaeological Survey of

India (ASI) will be using Ground

Penetrating Radar (GPR) to map the

contours of the area around the Bagh-e-

Naya Qila excavated garden inside the

Golconda Fort in Telangana

Q29) Answer a

Explanation

Silappadikaram is a famous Tamil epic

which was written by Ilango around

1800 years ago It is a story of a

merchant named Kovalan who fell in

love with a courtesan named Madhavi

Manimekalai tells the story of the

daughter of Kovalan and Madhavi

Q30) Answer (a)

Explanation

Charaka is the author of Charaka

Samhita which is an important work of

Ayurveda and medicines

Brahmaguptarsquos fame rests mostly on his

Brahma-sphuta-siddhanta which was

an astronomical work It was translated

into Arabic in Baghdad and had a major

impact on Islamic mathematics and

astronomy

Late in his life Brahmagupta wrote

Khandakhadyaka which was an

RAUSIAS-FC19E1003 44

astronomical handbook that employed

Aryabhatarsquos system of starting each day

at midnight

Q31) Answer (c)

Explanation

Amir Khusrau was a famous sufi

musician poet and scholar In 1318 he

noted that there was different language

in every region of this land (Hindustan)

Lahori Kashmiri Dvarsamudri (in

Southern Karnataka) Telangana (in

Andhra Pradesh) Gujari (in Gujarat)

Marsquobari (in Tamil Nadu) Awadhi (in

eastern Uttar Pradesh) and Hindawai (in

the area around in Delhi) etc He went

to explain that Sanskrit did not belong

to any region and that only brahmans

knew it

Q32) Answer c

Explanation

Hiranyagarbha refers to the golden

womb When this ritual was performed

with the help of Brahmanas it was

thought to lead to the rebirth of the

sacrificer as a Khastriya

Q33) Answer d

Explanation

Kadamai refers to a tax on land

revenue

Gwalior Prashasti describes the exploits

of Nagabhata who was a Pratihara king

Q34) Answer b

Explanation

Rajatarangini is a Sanskrit text written

by Kalhana in the 12th century

It was historical chronicle of early India

It is justifiably considered to be the best

and most authentic work of its kind

It covers the entire span of history in

the Kashmir region from the earliest

times to the date of its composition

Q35) Answer c

Explanation

ldquoUrrdquo was the general assembly of the

village ldquoUrrdquo consisted of all the

taxpaying residents of an ordinary

village

Q36) Answer (a)

Explanation

Tarikh was a form of history writing in

the Delhi Sultanate The authors of

tawarikhs were learned men which

included secretaries administrators etc

Q37 Answer (a)

Explanation

Alauddin chose to pay his soldiers salaries in cash rather than iqtas The soldiers would buy their supplies from merchants in Delhi and it was thus feared that merchants would raise their prices To stop this Alauddin controlled the prices of goods in Delhi Prices were carefully surveyed by officers and merchants who did not sell at the prescribed rates were punished

Q38) Answer (d)

Explanation

Delhi first became the capital of a

kingdom under the Tomara Rajputs

who were defeated in the middle of the

twelfth century by the Chauhans (also

referred to as Chahamanas) of Ajmer

It was under the Tomaras and

Chauhans that Delhi became an

important commercial centre Many rich

Jaina merchants lived in the city and

constructed several temples Coins

minted here called dehliwal had a wide

circulation

Q39) Answer (c)

Explanation

Moth ki Masjid was built in the reign of

Sikandar Lodi by his minister

Begumpuri mosque built in the reign of

Muhammad Tughluq was the main

mosque of Jahanpanah the ldquoSanctuary

of the Worldrdquo and his new capital in

Delhi

Quwwat al ndash Islam mosque was

enlarged by Iltutmish and Alauddin

Khalji The minar was built by three

Sultansndash Qutbuddin Aybak Iltutmish

and Firuz Shah Tughluq

RAUSIAS-FC19E1003 45

Q40) Answer (c)

Explanation

Under the Mughals mansabdar was

referred to an individual who held a

mansab ie rank and he received his

salary as revenue assignments called

jagirs

Q41) Ans (b)

The Quit India Movement was a

spontaneous revolt of people against

British rule

The All India Congress Committee met

at Bombay on 8 August 1942 It passed

the famous resolution Quit India and

proposed the starting of a non-violent

mass struggle under Gandhis

leadership to achieve this aim But on

the very next day Gandhi and other

eminent leaders of the Congress were

arrested The Congress was once again

declared illegal

Q42) Ans (c)

The Simon Commission refers to a

group of seven MPs from the United

Kingdom constituted to suggest

constitutional reforms for British India

The Commission consisted of only

British members headed by one of the

senior British politicians Sir John

Simon

So the people of India agitated against

the arrival of Simon Commission

Q43) Ans (a)

He was widely known for his

unfavourable opinion of the economic

consequences of the British rule in

India

In his many writings and speeches and

especially in Poverty and Un-British

Rule in India Naoroji argued that India

was too highly taxed and that its wealth

was being drained away to England

He did not interpret the ancient Indian

texts and restored the self-confidence of

Indians And also he did not stress the

need for eradication of all the social

evils before anything else

Q44) Ans (c)

In August 1932 Prime Minister

MacDonald announced his Communal

Award Great Britainrsquos unilateral

attempt to resolve the various conflicts

among Indiarsquos many communal

interests

The award which was later

incorporated into the act of 1935

expanded the separate-electorate

formula reserved for Muslims to other

minorities including Sikhs Indian

Christians Anglo-Indians Europeans

distinct regional groups Gandhi

undertook a ldquofast unto deathrdquo against

that offer which he viewed as a

nefarious British plot to divide the

Indian society

Q45) Ans (b)

In British India apart from existing

imports and exports there was also a

particular amount of money which

colonial India contributed towards

administration maintenance of the

army war expenses pensions to retired

officers and other expenses accrued by

Britain towards maintenance of her

colony These were known as Home

charges and were paid for almost

entirely by India

The Home charges was made of

following components-

- Interest payable on Indian debt

- Dividend to shareholders of East

India Company

- Funds used to support the India

Office in London

- Funds used to pay salaries and

pensions of British personnel

engaged in India

- Interest on the railways

- Civil and military charges

- Store purchases in England

Q46) Ans (b)

The Lahore session of the Indian

National Congress was held in 1929

under the Presidentship of Jawaharlal

Nehru

The Lahore session of the Indian

National Congress witnessed significant

RAUSIAS-FC19E1003 46

developments in the Indian national

movement

- First the election of Jawaharlal

Nehru to the post of Presidentship of

the Congress was a clear indication

of the growing strength of the

Leftists in the Congress

- Secondly it was in this session that

the Congress for the first time raised

the demand for complete

independence Such demand was

not raised from the Congress

platform earlier

Q47) Ans (b)

It did not provide for separate

electorates for any community or

weightage for minorities However it did

allow for the reservation of minority

seats in provinces having minorities of

at least ten per cent but this was to be

in strict proportion to the size of the

community

There was no provision for complete

Independence for India

Q48) Ans (c)

The religion of early Vedic Aryans was

primarily of worship of nature and

Yajnas

The early Aryan religion was kind of

nature worship Actually the forces

around them which they could not

control or understand were invested

with divinity and were personified as

male or female gods And they

performed some Yajnas also

Q49) Ans (b)

The roads and river-routes were not

immune from robbery It is notable that

Yuan Chwang (Hiuen Tsang) was

robbed of his belongings during

Harshvardanarsquos period

Q50) Ans (c)

Q51) Ans (b)

Purandara Dasa was a saint and great

devotee of Lord Krishna

There is much speculation about where

Purandara Dasa regarded as the

Pitamaha of Carnatic music was born

Recently an expert committee

constituted by the Kannada University

Hampi has come to the conclusion that

Kshemapura Shivamogga district

Karnataka is the birth place of

Purandara Dasa

Q52) Ans (c)

Sri Tyagaraja Sri Shyama Shastry and Sri Muthuswami Dikshitar are considered the trinity of Carnatic music and with them came the golden age in Carnatic music in the 18th-19th

century

Q53) Ans d)

Recently a rare sarcophagus (stone

coffin) which is 2000 years old from the

Iron AgendashMegalithic era was discovered

from a rock-cut cave at Viyur village of

Kollam near Koyilandy in Kozhikode

district Kerala

The coffin containing bone fragments

was found during an excavation ldquoSo

far such a rare finding has been

discovered only from two sites

in Kerala Both these sarcophagi were

recovered from Megalithic sites at

Chevayur and Atholi also in Kozhikode

district

Q54) Ans a)

The megalithic culture in South India was a full-fledged Iron Age culture

Q55) Ans d)

The Cholas Pandyas and Keralaputras

(Cheras) mentioned in Ashokan

inscriptions were probably in the late

megalithic phase of material culture

Q56) Ans d)

Q57) Ans (b)

Raj Kumar Shukla followed Gandhiji all

over the country to persuade him to

come to Champaran to investigate the

problem associated with tinkathia

system

RAUSIAS-FC19E1003 47

Brij Kishore Rajendra Prasad Mahadev

Desai and Narhari Parikh accompanied

Gandhi ji during the Champaran

Satyagraha

Q58) Ans (b)

The Satvahanas started the practice of granting tax-free villages to brahmanas and Buddhist monks

Q59) Ans c)

The objectives of the Programme are

listed as under

- Developing basic tourism

infrastructure

- Promoting cultural and heritage

value of the country to generate

livelihoods in the identified regions

- Enhancing the tourist attractiveness

in a sustainable manner by

developing world-class

infrastructure at the heritage

monument sites

- Creating employment through active

involvement of local communities

- Harnessing tourism potential for its

effects on employment generation

and economic development

- Developing sustainable tourism

infrastructure and ensuring proper

Operations and maintenance

therein

Q60) Ans (b)

The Tribal Cooperative Marketing

Development Federation of India

(TRIFED) came into existence in 1987

It is a national-level apex organization

functioning under the administrative

control of Ministry of Tribal Affairs

Govt of India

TRIFED has its registered and Head

Office located in New Delhi

Q61) Ans (c)

Premchandrsquos novels include

Premashram Rangabhumi Ghaban

Karmabhumi and Godan

Gora is a novel written by Rabindranath

Tagore

138th birth anniversary of Munshi

Premchand was celebrated across the

country

Q62) Ans (b)

Giddha is a traditional pastoral dance

performed by the women of the Punjab

India and Pakistan at festival times

and at the sowing and reaping of the

harvest

By this dance the Punjabi women

reveal their joy expel their suppressed

feelings in a male dominated society

through the performance of Giddha

Since this dance has nothing to do with

men only women can participate in it

During the Teej celebrations Giddha

dance is celebrated in Punjab every

year Teej is a generic name for a

number of festivals that are celebrated

by women in some parts of India

Q63) Ans (a)

Dara Shukoh wrote the remarkable

work called ldquoMajma-ul-Bahrainrdquo or the

ldquoThe confluence of two seasrdquo

The Vice President of India Shri M

Venkaiah Naidu has said that Prince

Dara Shukohrsquos writings can come as a

refreshing source for infusing peace and

harmony He was addressing the

gathering after visiting the exhibition

that showcases the forgotten Prince of

yesteryears Dara Shukoh organized by

Mr Francois Gautier at Indira Gandhi

National Centre for the Arts in New

Delhi

Q64) Ans (c)

The statue Gommateshwara is

dedicated to the Jain God Bahubali

It is a monolithic statue

President Ram Nath Kovind

inaugurated the grand anointing

ceremony mdash Mahamastakabhisheka mdash

held once in 12 years at

Shravanabelagola (Karnataka)

Q65) Ans (c)

Prachi Valley had come up around the

Prachi river Prachi Valley gradually

disappeared

RAUSIAS-FC19E1003 48

The Prachi river originates from

Bhubaneswar

It is a tributary of the Mahanadi and

flows through the districts of Puri

Khurda Cuttack and Jagatsinghpur

and the entire region of the river is

termed as the Prachi Valley

It falls into the Bay of Bengal

Archaeological evidence shows that the

Prachi Valley Civilisation predates both

Harappa and Mohenjo-Daro

The Prachi river originates from

Bhubaneswar

Q66) Ans (d)

These monuments are located in

Chhatarpur district Madhya Pradesh

within Vindhya mountain range

Q67) Ans (a)

The book lsquoThoughts on Pakistanrsquo was

written by Dr BR Ambedkar

On the occasion of the birth anniversary

of Dr BR Ambedkar the president of

India pays homage to this icon of India

In 1924 he founded the Depressed

Classes Institute (Bahishkrit Hitkarini

Sabha) and in 1927 the Samaj Samata

Sangh

Another area of attention for Ambedkar

was education For its spread among

the low classes he set up a network of

colleges by the name of Peoples

Education Society and founded hostels

Q68) Ans(b)

Mehrgarh is a famous Neolithic

settlement in the Indian subcontinent

which is situated in Baluchistan

province Pakistan

A pre-historic rock art site is discovered

in the vast expanse of limestone blocks

on the eastern banks of Naguleru river

near Dachepalli (Andhra Pradesh) It

has thrown light on the Neolithic

civilisation that flourished in Guntur

(Andhra Pradesh) during 1500-2000

BC

Q69) Ans (c)

The 12th and the 13th centuries saw

the emergence of the Kakatiyas They

were at first the feudatories of the

Western Chalukyas of Kalyana Initially

they ruled over a small territory near

Warangal (Telangana)

They introduced Nayakships which was

later adopted and developed by the

Rayas of Vijayanagara

Q70) Ans (a)

The fast had effect of putting pressure

on mill owners who finally agreed to

give the workers a 35 per cent increase

in wages

Google celebrated with a doodle the

132nd birth anniversary of Anasuya

Sarabhai who played a pioneering role

in Indiarsquos labour movement

Q71) Ans (d)

The UNESCOrsquos list of the representative

list of the intangible cultural heritage of

humanity from India are

- Koodiyattam Sanskrit Theatre of

Kerala

- Mudiyettu ritual theatre and dance

drama of Kerala

- Tradition of Vedic Chanting

- Kalbelia folk songs and dances of

Rajasthan

- Ramlila Traditional Performance of

the Ramayana

- Sankirtana ritual singing

drumming and dancing of Manipur

- Ramman religious festival and

ritual theatre of the Garhwal

Himalayas India

- Traditional brass and copper craft of

utensil making among the Thatheras

of Jandiala Guru Punjab India

- Chhau dance classical Indian dance

originated in the eastern Indian

states

- Buddhist chanting of Ladakh

recitation of sacred Buddhist texts

in the trans-Himalayan Ladakh

region Jammu and Kashmir India

- Yoga

- Nouroz

- Kumbh Mela

RAUSIAS-FC19E1003 49

Q72) Ans(b)

The President of India Shri Ram Nath Kovind inaugurated the Hornbill Festival and State Formation Day celebrations of Nagaland in Kisama

The festival is named after the Indian hornbill the large and colourful forest bird which is displayed in the folklore of most of the states tribes

The major recognized tribes of Nagaland are Angami Ao Chakhesang Chang

Kuki Rengma and Zeling etc

Onge Jarawa and Sentinelese are the

tribes of Andman amp Nicobar Islands

Q73) Ans (c)

The Rashtrakutas rule in the Deccan lasted for almost two hundred years till the end of the tenth century The Rashtrakutas rulers were tolerant in their religious views and patronized not only Shaivism and Vaishnavism but

Jainism as well

The famous rock-cut temple of Shiva at Ellora was built by one of the Rashtrakutas kings Krishna I in the ninth century His successor Amoghavarsha was a Jain but he also

patronized other faiths

The Rashtrakutas allowed Muslims traders to settle and permitted Islam to

be preached in their dominions

Recently increasing defacement at the prehistoric rock paintings of Pandavulagutta Telangana has created a cause for grave concern It can spoil

the prehistoric rock

Pandavulagutta is home to

- Painted rock shelters dating to

10000 BC-8000 BC

- An 8th century inscription of the

Rashtrakuta period and

- Painted frescoes from the 12th century Kakatiya empire

Q74) Ans (b)

In 1828 Raja Ram Mohan Roy founded a new religious society the Brahma Sabha later known as the Brahmo

Samaj

Debendranath Tagore headed the Tattvabodhini Sabha which was

engaged in search of spiritual truth

Its purpose was to purify Hinduism and to preach monotheism or belief in one God

The new society was to be based on the twin pillars of reason and the Vedas and

Upanishads

Recently Sadharan Brahmo Samaj (SBS) has entered into a legal battle with the West Bengal government due

to some legal issue

Q75) Ans (c)

The Chishti order was established in India by Khwaja Moinuddin Chishti who came to India around 1192 The Chishtirsquos are considered to be the most influential of the groups of Sufis who migrated to India in the late twelfth century They adapted successfully to the local environment and adopted several features of Indian devotional

traditions

The historical dargah of Sufi mystic Khwaja Moinuddin Chishti in Ajmer is all set to get a facelift This 13 th century dargah has been included among the Swachh Iconic Places a clean-up initiative focused on iconic

heritage spiritual and cultural places

Page 13: GENERAL STUDIES (PAPER I) · Test is part of Rau’s IAS Test series for Preliminary Exam 2019 FOUNDATION + CURRENT AFFAIRS GENERAL STUDIES (PAPER –I) FOUNDATION TEST –III TOPIC:

RAUSIAS-FC19E1003 13

Q32) Consider the following statements

1 Hiranya-garbha ritual was thought

to lead to the rebirth of the

sacrificer as a Kshatriya

2 Mayurasharman was the founder

of the Kadamba dynasty

Which of the statements given above

isare correct

(a) 1 only

(b) 2 only

(c) Both 1 and 2

(d) Neither 1 nor 2

Q33) Which of the following statements

isare correct

1 Kadamai was tax taken in form of

forced labour

2 Gwalior Prashasti describes the

exploits of Nagabhata who was a

Chandella king

Select the correct answer using the code

given below

(a) 1 only

(b) 2 only

(c) Both 1 and 2

(d) Neither 1 nor 2

Q34) Which of the following statements

isare correct

1 Rajatarangini is a Sanskrit text

written by Kalhana in the 11th

century

2 Pala dynasty was included in the

tripartite struggle for Kannauj

Select the correct answer using the code

given below

(a) 1 only

(b) 2 only

(c) Both 1 and 2

(d) Neither 1 nor 2

Q35) Consider the following pairs

1 Brihadeshvara temple Rajaraja

Chola

2 ldquoUrrdquo Unit of measurement

3 Devadana Land grants made to

temples

Which of the pairs given above isare

correct

(a) 1 and 2 only

(b) 2 and 3 only

(c) 1 and 3 only

(d) 1 2 and 3

Q36) Which of the following statements

isare correct

1 The language of administration

under the Delhi Sultans was

Persian

2 Tarikhtawarikh was a form of

poetry in the Delhi Sultanate

Select the correct answer using the code

given below

(a) 1 only

(b) 2 only

(c) Both 1 and 2

(d) Neither 1 nor 2

Q37) Which of the following statements

isare correct

1 Alauddin Khilji constructed a new

garrison town named Siri for his

soldiers

2 He paid his soldiers their salaries

in the form of Iqta

Select the correct answer using the code

given below

(a) 1 only

(b) 2 only

(c) Both 1 and 2

(d) Neither 1 nor 2

RAUSIAS-FC19E1003 14

Q38) निमननिखित कथि ो म स कौि-सास सही हह

1 नदलली कतबददीि एबक क अरीि पहिी बार

नकसी सामराजय की राजरािी बिी थी

2 दहिीिाि नसक ो का मदरर मग़ि ो क दवारा

नकया गया था

िीच नदए गए कट का परय ग कर सही उततर चनिए

(a) किि 1

(b) किि 2

(c) 1 और 2 द ि ो

(d) ि त 1 ि ही 2

Q39) निमननिखित यग ो पर निचार कीनजए

1 म ठ की मखिद नसको दर ि दी

2 बगमपरी मखिद नफर ज शाह तगिक

3 कववत- अि - इसलाम कतबददीि ऐबक

उपयणकत यग ो म स कौि-स सही समनित ह

(a) किि 1 और 2

(b) किि 2 और 3

(c) किि 1 और 3

(d) 1 2 और 3

Q40) निमननिखित कथि ो म स कौि-सास सही हह

1 मिसबदार ो क अपिा िति राजसव कायो

नजन जागीर कहत थ क रप म परापत ह ता

था

2 मिसबदार क ज सनय उततरदानयतव सौोप जात

थ उसक अनतगणत उस एक निराणररत सखया म

सिार अथिा घड़सिार ो का रि-रिाि करिा

पड़ता था

िीच नदए गए कट का परय ग कर सही उततर चनिए

(a) किि 1

(b) किि 2

(c) 1 और 2 द ि ो

(d) ि त 1 ि ही 2

Q41) ldquo1942 क भारत छ ड़ आोद ििrdquo क बार म

निमननिखित अिि कि ो म स कौि-सा सतय िही ो ह

(a) यह एक अनहोसक आोद िि था

(b) इसका िततव महातमा गाोरी क दवारा नकया गया

था

(c) यह एक सवाभानिक आोद िि था

(d) इसि सामानयतया शरनमक िगण क आकनिणत

िही ो नकया था

Q42) भारत क ि ग ो ि ldquoसाइमि कमीशिrdquo क आगमि क

निरदध आोद िि नकया था कय ोनक

(a) भारतीय कभी भी 1919 क अनरनियम (The

Act of 1919) क काम की समीकषा िही ो करिा

चाहत थ

(b) साइमि कमीशि ि पराोत ो म दवर (द हर) शासि

क समापत करि की नसफाररश की थी

(c) साइमि कमीशि म क ई भारतीय सदसय िही ो

था

(d) साइमि कमीशि ि दश क निभाजि का

सझाि नदया था

Q43) निमननिखित कथि ो पर निचार कीनजए

भारतीय राषटर ीय आोद िि म दादाभाई िौर जी क दवारा

नकया गया सबस परभािी य गदाि यह था नक उन ोि

1 अोगरज ो क दवारा भारत क आनथणक श िर का

ििासा नकया था

2 पराचीि भारतीय गरोथ ो की वयाखया की थी और

भारतीय ो क आतमनिशवास क पिःसथानपत नकया

था

3 अनय नकसी भी बात स पहि सभी सामानजक

बराइय ो क उनमिि की आिशयकता पर बि

नदया था

उपयणकत कथि ो म स कौि-सास सही हह

(a) किि 1

(b) किि 2 और 3

(c) किि 1 और 3

(d) 1 2 और 3

RAUSIAS-FC19E1003 15

Q38) Which of the following statements

isare correct

1 Delhi first became the capital of a

kingdom under Qutubuddin

Aibak

2 Dehliwal coins were minted by the

Mughals

Select the correct answer using the code

given below

(a) 1 only

(b) 2 only

(c) Both 1 and 2

(d) Neither 1 nor 2

Q39) Consider the following pairs

1 Moth ki Masjid- Sikander Lodi

2 Begumpuri mosque- Firuz Shah

Tughluq

3 Quwwat al ndash Islam- Qutubuddin

Aibak

Which of the above pairs isare correct

(a) 1 and 2 only

(b) 2 and 3 only

(c) 1 and 3 only

(d) 1 2 and 3

Q40) Which of the following statements

isare correct

1 Mansabdars received their salaries

as revenue assignments called

jagirs

2 The mansabdarrsquos military

responsibilities required him to

maintain a specified number of

sawar or cavalrymen

Select the correct answer using the code

given below

(a) 1 only

(b) 2 only

(c) Both 1 and 2

(d) Neither 1 nor 2

Q41) Which one of the following observations

is not true about the Quit India

Movement of 1942

(a) It was a non-violent movement

(b) It was led by Mahatma Gandhi

(c) It was a spontaneous movement

(d) It did not attract the labour class

in general

Q42) The people of India agitated against the

arrival of the Simon Commission

because

(a) Indians never wanted the review of

the working of the Act of 1919

(b) Simon Commission recommended

the abolition of dyarchy in the

Provinces

(c) there was no Indian member in the

Simon Commission

(d) the Simon Commission suggested

the partition of the country

Q43) Consider the following statements

The most effective contribution made by

Dadabhai Naoroji to the cause of Indian

National Movement was that he-

1 exposed the economic exploitation

of India by the British

2 interpreted the ancient Indian

texts and restored the self-

confidence of Indians

3 stressed the need for eradication of

all the social evils before anything

else

Which of the statements given above

isare correct

(a) 1 only

(b) 2 and 3 only

(c) 1 and 3 only

(d) 1 2 and 3

RAUSIAS-FC19E1003 16

Q44) महातमा गाोरी ि 1932 म आमरर अिशि नकया था

कय ोनक

(a) ldquoग िमज सममििrdquo (The Round Table

Conference) भारतीय राजिीनतक

आकाोकषाओो क परा करि म असफि रहा था

(b) काोगरस और मखसलम िीग म मतभद थ

(c) रामस मकड िालड (Ramsay Macdonald)

ि ldquoसाोपरदानयक परसकारrdquo (The Communal

Award) की घ िरा की थी

(d) ldquoसनििय अिजञा आोद ििrdquo (The Civil

Disobedience Movement) असफि रहा

था

Q45) भारत म औपनििनशक शासि की अिनर क सोदभण म

भारत स रि क बनहगणमि का एक महतवपरण भाग गह

शलक (Home Charges) था निमननिखित म स

कौि-सास क ि गह शलक म सखममनित नकया गया

थानकय गए थ

1 िोदि म भारत कायाणिय क निए उपय ग नकय

जाि िािा क ि

2 भारत म नियकत नबरनटश कनमणय ो क िति और

पशि का भगताि करि क निए उपय ग नकय

जाि िािा क ि

3 अोगरज ो क दवारा भारत क बाहर यदध ो क निए

उपय ग नकय जाि िािा क ि

िीच नदए गए कट का परय ग कर सही उततर चनिए

(a) किि 1

(b) किि 1 और 2

(c) किि 2 और 3

(d) 1 2 और 3

Q46) सवतोतरता आोद िि क इनतहास म भारतीय राषटर ीय

काोगरस का 1929 का सतर महतवपरण ह कय ोनक इसम

(a) काोगरस क उददशय क रप म सथािीय सरकार

की पराखपत की घ िरा की गई थी

(b) परण सवराज की पराखपत क काोगरस क िकषय क

रप म अपिाया गया था

(c) असहय ग आोद िि शर नकया गया था

(d) िोदि म ldquoग ि मर सममििrdquo (The Round

Table Conference) म भाग िि का निरणय

निया गया था

Q47) भारतीय सवतोतरता सोगराम क सोदभण म िहर ररप टण

क दवारा निमननिखित म स नकसकी नसफाररश की गई

थीनकिकी नसफाररश की गई थी ो

1 भारत क निए परण सवतोतरता

2 अलपसोखयक ो क निए सीट ो क आरकषर क

निए सोयकत नििाणचक मोडि

3 सोनिराि म भारत क ि ग ो क निए मौनिक

अनरकार ो का परािराि

िीच नदए गए कट का परय ग कर सही उततर चनिए

(a) किि 1

(b) किि 2 और 3

(c) किि 1 और 3

(d) 1 2 और 3

Q48) आरो नभक िनदक आयो का रमण मखय रप स था

(a) भखकत

(b) मनतण पजा और यजञ

(c) परकनत की पजा और यजञ

(d) परकनत की पजा और भखकत

RAUSIAS-FC19E1003 17

Q44) Mahatma Gandhi undertook fast unto

death in 1932 mainly because

(a) The Round Table Conference failed

to satisfy Indian political

aspirations

(b) The Congress and Muslim League

had differences of opinion

(c) Ramsay Macdonald announced the

Communal Award

(d) The Civil Disobedience Movement

failed

Q45) With reference to the period of colonial

rule in India ldquoHome Chargesrdquo formed

an important part of drain of wealth

from India Which of the following funds

constituted ldquoHome Chargesrdquo

1 Funds used to support the India

Office in London

2 Funds used to pay salaries and

pensions of British personnel

engaged in India

3 Funds used for waging wars

outside India by the British

Select the correct answer using the code

given below

(a) 1 only

(b) 1 and 2 only

(c) 2 and 3 only

(d) 1 2 and 3

Q46) The 1929- Session of Indian National

Congress is of significance in the history

of the Freedom Movement because the-

(a) attainment of Self-Government

was declared as the objective of

the Congress

(b) attainment of Poorna Swaraj was

adopted as the goal of the

Congress

(c) Non-Cooperation Movement was

launched

(d) decision to participate in the

Round Table Conference in

London was taken

Q47) With reference to the period of Indian

freedom struggle which of the following

waswere recommended by the Nehru

report

1 Complete Independence for India

2 Joint electorates for reservation of

seats for minorities

3 Provision of fundamental rights for

the people of India in the

Constitution

Select the correct answer using the code

given below

(a) 1 only

(b) 2 and 3 only

(c) 1 and 3 only

(d) 1 2 and 3

Q48) The religion of the early Vedic Aryans was primarily of

(a) Bhakti

(b) image worship and Yajnas

(c) worship of nature and Yajnas

(d) worship of nature and Bhakti

RAUSIAS-FC19E1003 18

Q49) भारत की यातरा करि िाि चीिी यातरी यआि चिाोग

(हयएि साोग) ि समकािीि भारत की सामानय

खसथनतय ो और सोसकनत क दजण नकया था इस सोदभण म

निमननिखित कथि ो म स कौि-सास सही हह

1 सड़क और िदी-मागण (जि-मागण) डकती स

परण रप स सरनकषत थ

2 जहा तक अपरार ो क निए दणड की बात ह

उसक निए नकसी भी वयखकत की निदोिता

अथिा उसक अपरार क निराणररत करि क

निए अनि जि और निि परि क माधयम क

सारि थ

3 वयापाररय ो क घाट ो और परनतबोर सटशि ो पर

शलक ो का भगताि करिा पड़ता था

िीच नदए गए कट का परय ग कर सही उततर चनिए

(a) किि 1

(b) किि 2 और 3

(c) किि 1 और 3

(d) 1 2 और 3

Q50) नसोर घाटी सभयता क सोदभण म निमननिखित कथि ो पर

निचार कीनजए

1 यह मखय रप स एक रमणनिरपकष सभयता थी

तथा हािाोनक इसम रानमणक ततव मौजद था

िनकि िह परनतिश पर हािी िही ो था

2 इस काि क दौराि भारत म कपास का परय ग

कपड़ा बिाि क निए नकया जाता था

उपयणकत कथि ो म स कौि-सास सही हह

(a) किि 1

(b) किि 2

(c) 1 और 2 द ि ो

(d) ि त 1 ि ही 2

Q51) परोदर दास क सोदभण म निमननिखित कथि ो पर निचार

कीनजए

1 परोदर दास एक सोत और भगिाि नशि क एक

महाि भकत थ

2 ि एक सोगीतकार गायक और किाणटक सोगीत

क मखय सोसथापक-परसतािक ो म स एक थ

उपयणकत कथि ो म स कौि-सास सही हह

(a) किि 1

(b) किि 2

(c) 1 और 2 द ि ो

(d) ि त 1 ि ही 2

Q52) निमननिखित म स कौि-सास वयखकत किाणटक सोगीत

की नतरमनतण म शानमि हह

1 बािामरिी कषणा

2 शरी शयाम शासतरी

3 शरी मथसवामी दीनकषतर

िीच नदए गए कट का परय ग कर सही उततर चनिए

(a) किि 1

(b) किि 2

(c) किि 2 और 3

(d) 1 2 और 3

Q53) चियर (Chevayur) और अथ िी (Atholi) म खसथत

महापािार सथि निमननिखित म स नकस राजय म खसथत

(a) तनमििाड

(b) किाणटक

(c) पनिम बोगाि

(d) करि

RAUSIAS-FC19E1003 19

Q49) The Chinese traveller Yuan Chwang

(Hiuen Tsang) who visited India

recorded the general conditions and

culture of India at that time In this

context which of the following

statements isare correct

1 The roads and river-routes were

completely immune from robbery

2 As regards punishment for

offences ordeals by fire water and

poison were the instruments for

determining the innocence or guilt

of a person

3 The tradesmen had to pay duties

at ferries and barrier stations

Select the correct answer using the code

given below

(a) 1 only

(b) 2 and 3 only

(c) 1 and 3 only

(d) 1 2 and 3

Q50) Regarding the Indus Valley Civilization

consider the following statements

1 It was predominantly a secular

civilization and the religious

element though present did not

dominate the scene

2 During this period cotton was

used for manufacturing textiles in

India

Which of the statements given above

isare correct

(a) 1 only

(b) 2 only

(c) Both 1 and 2

(d) Neither 1 nor 2

Q51) Consider the following statements

regarding Purandara Dasa

1 Purandara Dasa was a saint and

great devotee of Lord Shiva

2 He was a composer singer and

one of the chief founding-

proponents of the Carnatic music

Which of the statements given above

isare correct

(a) 1 only

(b) 2 only

(c) Both 1 and 2

(d) Neither 1 nor 2

Q52) Which of the following persons isare

included in the trinity of Carnatic

music

1 Balamurali Krishna

2 Sri Shyama Shastry

3 Sri Muthuswami Dikshitar

Select the correct answer using the code

given below

(a) 1 only

(b) 2 only

(c) 2 and 3 only

(d) 1 2 and 3

Q53) Megalithic sites at Chevayur and Atholi

are located in which of the following

states

(a) Tamil Nadu

(b) Karnataka

(c) West Bengal

(d) Kerala

RAUSIAS-FC19E1003 20

Q54) निमननिखित कथि ो पर निचार कीनजए

1 महापािानरक ि ग कबर ो म िसतएो दफिात थ

2 दनकषर भारत म महापािार सोसकनत एक परण

निकनसत तामर यगीि सोसकनत थी

उपयणकत कथि ो म स कौि-सास सही हह

(a) किि 1

(b) किि 2

(c) 1 और 2 द ि ो

(d) ि त 1 ि ही 2

Q55) निमननिखित म स कौि-स सामराजयसामराजय ो का

अश क क अनभिि ो म उललि नकया गया ह

1 च ि

2 पाणडय

3 करिपतर (चर)

िीच नदए गए कट का परय ग कर सही उततर चनिए

(a) किि 1

(b) किि 1 और 2

(c) किि 3

(d) 1 2 और 3

Q56) भीमा-क रगाोि का यदध को पिी क सनिक ो और

बाजीराि नदवतीय क िततव म एक शखकतशािी पशिा

सिा (मराठ ो) क मधय िड़ा गया था यह यदध

निमननिखित म स नकसका नहससा था

(a) परथम आोगल-मराठा यदध का

(b) नदवतीय आोगल-मराठा यदध का

(c) ततीय आोगल-मसर यदध का

(d) ततीय आोगल-मराठा यदध का

Q57) निमननिखित कथि ो पर निचार कीनजए

1 महादि दसाई ि गाोरीजी क चोपारर आि तथा

नतिकनथया पररािी स जड़ी समसया की जाोच

क निए रारी करि क निए दश भर म उिका

अिसरर नकया था

2 िरहरी पाररि चोपारर सतयागरह क दौराि

गाोरीजी क साथ थ

उपयणकत कथि ो म स कौि-सास सही हह

(a) किि 1

(b) किि 2

(c) 1 और 2 द ि ो

(d) ि त 1 ि ही 2

Q58) निमननिखित कथि ो पर निचार कीनजए

1 िनद राज-िोश ि बराहमर ो और बौदध मठराररय ो

क कर-मकत गाि अिदाि म दि की परथा

आरि की थी

2 सतिाहि ो की आनरकाररक भािा पराकत थी

उपयणकत कथि ो म स कौि-सास सही हह

(a) किि 1

(b) किि 2

(c) 1 और 2 द ि ो

(d) ि त 1 ि ही 2

Q59) एक निरासत क अपिाइए (अडॉपट ए हररटज ndash

Adopt a Heritage) पररय जिा क उददशय ो क

सनदभण म निमननिखित कथि ो पर निचार कीनजए

1 यह पररय जिा र रगार उतपादि और आनथणक

निकास क निए पयणटि कषमता का उि पर

परभाि का उपय ग करगी

2 यह पररय जिा निरासत सथि ो पर निशव सतरीय

आराररक सोरचिा निकनसत करक एक सतत

तरीक स पयणटक आकिणर म िखदध करगी

उपयणकत कथि ो म स कौि-सास सही हह

(a) किि 1

(b) किि 2

(c) 1 और 2 द ि ो

(d) ि त 1 ि ही 2

RAUSIAS-FC19E1003 21

Q54) Consider the following statements

1 Megalithic people buried goods in

graves

2 The megalithic culture in South

India was a full-fledged Copper

Age culture

Which of the statements given above

isare correct

(a) 1 only

(b) 2 only

(c) Both 1 and 2

(d) Neither 1 nor 2

Q55) Which of the following kingdoms isare

mentioned in the Ashokan inscriptions

1 Cholas

2 Pandyas

3 Keralaputras (Cheras)

Select the correct answer using the code

given below

(a) 1 only

(b) 1 and 2 only

(c) 3 only

(d) 1 2 and 3

Q56) The Battle of Bhima-Koregaon was

fought between the soldiers of the

Company and the strong Peshwa army

(Marathas) under Bajirao II This war

was a part of the

(a) First Anglo-Maratha war

(b) Second Anglo-Maratha war

(c) Third Anglo- Mysore war

(d) Third Anglo-Maratha war

Q57) Consider the following statements

1 Mahadev Desai followed Gandhiji all over the country to persuade him to come to Champaran to investigate the problem associated

with tinkathia system

2 Narhari Parikh accompanied Gandhi ji during the Champaran

Satyagraha

Which of the statements given above isare correct

(a) 1 only

(b) 2 only

(c) Both 1 and 2

(d) Neither 1 nor 2

Q58) Consider the following statements

1 The Nanda Dynasty started the practice of granting tax-free villages to brahmanas and

Buddhist monks

2 The official language of the Satavahanas was Prakrit

Which of the statements given above

isare correct

(a) 1 only

(b) 2 only

(c) Both 1 and 2

(d) Neither 1 nor 2

Q59) Consider the following statements about the objectives of the lsquoadopt a heritagersquo

project

1 It will harness tourism potential for its effects on employment generation and economic

development

2 It will enhance the tourist attractiveness in a sustainable manner by developing world class infrastructure at heritage sites

Which of the statements given above

isare correct

(a) 1 only

(b) 2 only

(c) Both 1 and 2

(d) Neither 1 nor 2

RAUSIAS-FC19E1003 22

Q60) ldquoभारतीय जिजातीय सहकारी निपरि निकास सोघrdquo

(The Tribal Co-operative Marketing

Development Federation of India - TRIFED)

क सोदभण म निमननिखित कथि ो पर निचार कीनजए

1 यह एक राषटर ीय सतर का शीिण सोगठि ह ज

भारत सरकार क गह मोतरािय क परशासनिक

नियोतरर क अरीि काम कर रहा ह

2 इसका मखय उददशय दश म जिजातीय ि ग ो

का सामानजक-आनथणक निकास करिा ह

उपयणकत कथि ो म स कौि-सास सही हह

(a) किि 1

(b) किि 2

(c) 1 और 2 द ि ो

(d) ि त 1 ि ही 2

Q61) निमननिखित म स कौि-सास उपनयास परमचोद क

दवारा नििा गया हनिि गए ह

1 रोगभनम

2 ग दाि

3 ग रा

िीच नदए गए कट का परय ग कर सही उततर चनिए

(a) किि 1

(b) किि 2

(c) किि 1 और 2

(d) 1 2 और 3

Q62) नगदधा ितय क सोदभण म निमननिखित कथि ो पर निचार

कीनजए

1 नगदधा नबहार की मनहिाओो क दवारा तयौहार क

समय और फसि की बिाई तथा कटाई क

अिसर पर नकया जाि िािा एक पारोपररक

दहाती ितय ह

2 इस ितय क दवारा मनहिाऐो अपिी परसननता

परकट करती ह तथा नगदधा क परदशणि क

माधयम स परि िचणसव िाि समाज म

मनहिाओो की दबी हई भाििाओो क परकट

करती ह

उपयणकत कथि ो म स कौि-सास सही हह

(a) किि 1

(b) किि 2

(c) 1 और 2 द ि ो

(d) ि त 1 ि ही 2

Q63) निमननिखित कथि ो पर निचार कीनजए

1 मलला शाह बदखशी दारा नशक ह क

आधयाखतमक गर थ

2 औरोगरब ि मजम-उि-बहरीि या द समदर ो

का सोगम िामक उललििीय रचिा नििी थी

3 दारा नशक ह क अपि पिणज अकबर क गर ो

क उततरानरकारी क रप म दिा गया था

नजसम उसि रानमणक बहििाद और समनवयता

क बढ़ािा नदया था

उपयणकत कथि ो म स कौि-सास सही हह

(a) किि 1 और 3

(b) किि 2

(c) किि 1 और 2

(d) 1 2 और 3

RAUSIAS-FC19E1003 23

Q60) Consider the following statements about

the Tribal Cooperative Marketing

Development Federation of India

(TRIFED)

1 It is a national-level apex

organization functioning under the

administrative control of Ministry

of Home Affairs Government of

India

2 The main objective of TRIFED is

socio-economic development of

tribal people in the country

Which of the statements given above

isare correct

(a) 1 only

(b) 2 only

(c) Both 1 and 2

(d) Neither 1 nor 2

Q61) Which of the following novels isare

written by Premchand

1 Rangabhumi

2 Godan

3 Gora

Select the correct answer using the code

given below

(a) 1 only

(b) 2 only

(c) 1 and 2 only

(d) 1 2 and 3

Q62) Consider the following statements about

Giddha dance

1 Giddha is a traditional pastoral

dance performed by the women of

Bihar at festival times and at the

sowing and reaping of the harvest

2 By this dance the women reveal

their joy expel their suppressed

feelings in a male dominated

society through the performance of

Giddha

Which of the statements given above

isare correct

(a) 1 only

(b) 2 only

(c) Both 1 and 2

(d) Neither 1 nor 2

Q63) Consider the following statements

1 Mullah Shah Badakhshi was the

spiritual mentor of Dara Shukoh

2 Aurangzeb wrote the remarkable

work called ldquoMajma-ul-Bahrainrdquo or

the ldquoThe confluence of two seasrdquo

3 Dara Shukoh was seen as

inheriting the qualities of his

ancestor Akbar in that he

promoted religious pluralism and

syncretism

Which of the statements given above

isare correct

(a) 1 and 3 only

(b) 2 only

(c) 1 and 2 only

(d) 1 2 and 3

RAUSIAS-FC19E1003 24

Q64) निमननिखित कथि ो पर निचार कीनजए

1 ग मतशवर परनतमा निोधयनगरी पहाड़ी पर खसथत ह

2 शरिरबिग िा िह सथाि ह जहाो मौयण िोश क

सोसथापक चोदरगपत मौयण अपि नसोहासि क

तयागि क बाद जि तपसवी बि गए थ

उपयणकत कथि ो म स कौि-सास सही हह

(a) किि 1

(b) किि 2

(c) 1 और 2 द ि ो

(d) ि त 1 ि ही 2

Q65) निमननिखित कथि ो पर निचार कीनजए

1 पराताखतवक साकषय स पता चिता ह नक पराची

घाटी सभयता हड़पपा और म हिज दाड़ द ि ो

की पिणिती ह

2 पराची िदी भििशवर स निकिती ह

उपयणकत कथि ो म स कौि-सास सही हह

(a) किि 1

(b) किि 2

(c) 1 और 2 द ि ो

(d) ि त 1 ि ही 2

Q66) निमननिखित कथि ो म स कौि-सास सही हह

1 िजराह क समारक ो क समह का निमाणर

चोदि राजिोश क शासिकाि क दौराि हआ

था

2 य समारक हररिोदर पिणत शरोििा म खसथत ह

3 म रक क यातरी इबन बतता ि अपि सोसमरर ो

म िजराह क मोनदर ो की यातरा का उललि

नकया था तथा इन काजराण िाम स समब नरत

नकया था

िीच नदए गए कट का परय ग कर सही उततर चनिए

(a) किि 1

(b) किि 1 और 2

(c) किि 2 और 3

(d) किि 1 और 3

Q67) निमननिखित कथि ो म स कौि-सास सही हह

1 डॉ बी आर अमबडकर ि दी एनिनहिशि

ऑफ़ कासट (The Annihilation of Caste)

नििी थी नजसम उन ोि नहोद रमण म िोशािगत

पजारी की परथा क उनमिि की आिशयकता

पर बि नदया था

2 डॉ राजदर परसाद ि थॉटस ऑि पानकसताि

(Thoughts on Pakistan) िामक पसतक

नििी थी

िीच नदए गए कट का परय ग कर सही उततर चनिए

(a) किि 1

(b) किि 2

(c) 1 और 2 द ि ो

(d) ि त 1 ि ही 2

Q68) निमननिखित कथि ो म स कौि-सास सही हह

1 महरगढ़ भारतीय उपमहादवीप म एक परनसदध

ििपािार बसती ह ज नसोर पराोत पानकसताि म

खसथत ह

2 बरणह म म कतत ो क उिक सवामी क साथ कबर ो

म दफिाया जाता था

िीच नदए गए कट का परय ग कर सही उततर चनिए

(a) किि 1

(b) किि 2

(c) 1 और 2 द ि ो

(d) ि त 1 ि ही 2

Q69) निमननिखित कथि ो म स कौि-सास सही हह

1 काकानटय मोनदर अनरकतर नशि क समनपणत

2 हिमक ोडा म हजार-सतोभ िाि मोनदर (The

Thousand-Pillared Temple) का निमाणर

काकानटय समराट रदर ि करिाया था

िीच नदए गए कट का परय ग कर सही उततर चनिए

(a) किि 1

(b) किि 2

(c) 1 और 2 द ि ो

(d) ि त 1 ि ही 2

RAUSIAS-FC19E1003 25

Q64) Consider the following statements

1 Gommateshwara Statue is located

on the Vindyagiri Hill

2 Shravanabelagola is the place

where Chandragupta Maurya the

founder of the Mauryan dynasty

became a Jain ascetic after

relinquishing his throne

Which of the statements given above

isare correct

(a) 1 only

(b) 2 only

(c) Both 1 and 2

(d) Neither 1 nor 2

Q65) Consider the following statements

1 Archaeological evidence shows

that the Prachi Valley Civilisation

predates both Harappa and

Mohenjo-Daro

2 The Prachi river originates from

Bhubaneswar

Which of the statements given above

isare correct

(a) 1 only

(b) 2 only

(c) Both 1 and 2

(d) Neither 1 nor 2

Q66) Which of the following statements

isare correct

1 The Khajuraho group of

monuments was built during the

rule of the Chandela dynasty

2 These monuments are located in

Harischandra mountain range

3 Ibn Battuta the Moroccan

traveller in his memoirs mentioned

visiting Khajuraho temples and

called them Kajarra

Select the correct answer using the code

given below

(a) 1 only

(b) 1 and 2

(c) 2 and 3

(d) 1 and 3

Q67) Which of the following statements

isare correct

1 Dr BR Ambedkar wrote the

Annihilation of Caste emphasising

the need to do away with the

practice of hereditary priesthood in

Hinduism

2 The book lsquoThoughts on Pakistanrsquo

was written by Dr Rajendra

Prasad

Select the correct answer using the code

given below

(a) 1 only

(b) 2 only

(c) Both 1 and 2

(d) Neither 1 nor 2

Q68) Which of the following statements

isare correct

1 Mehrgarh is a famous Neolithic

settlement in the Indian

subcontinent which is situated in

Sindh province Pakistan

2 At Burzahom dogs were buried

with their masters in their graves

Select the correct answer using the code

given below

(a) 1 only

(b) 2 only

(c) Both 1 and 2

(d) Neither 1 nor 2

Q69) Which of the following statements

isare correct

1 The Kakatiya temples are

dedicated mostly to Siva

2 The Thousand-Pillared Temple at

Hanamkonda was built by the

Kakatiya king Rudra

Select the correct answer using the code

given below

(a) 1 only

(b) 2 only

(c) Both 1 and 2

(d) Neither 1 nor 2

RAUSIAS-FC19E1003 26

Q70) निमननिखित कथि ो म स कौि-सास सही हह

1 अहमदाबाद नमि हड़ताि क दौराि महातमा

गाोरी ि शरनमक ो क पकष क मजबत करि क

निए आमरर अिशि नकया था

2 अिशि स नमि मानिक ो पर दबाि पड़ा था ज

अोततः शरनमक ो क िति म 15 परनतशत की िखदध

करि क निए सहमत हए थ

िीच नदए गए कट का परय ग कर सही उततर चनिए

(a) किि 1

(b) किि 2

(c) 1 और 2 द ि ो

(d) ि त 1 ि ही 2

Q71) निमननिखित म स नकसक नकिक भारत स यिसक

की माििता की अमतण साोसकनतक निरासत की

परनतनिनर सची (The UNESCOrsquos List of the

Representative List of the Intangible

Cultural Heritage of Humanity) म शानमि

नकया गया ह

1 मनडयटट

2 सोकीतणि

3 को भ मिा

िीच नदए गए कट का परय ग कर सही उततर चनिए

(a) किि 1 और 2

(b) किि 2 और 3

(c) किि 3

(d) 1 2 और 3

Q72) निमननिखित जिजानतय ो म स कौि-सीसी ो

जिजानतजिजानतया िागािड स सोबोनरत हह

1 अोगामी

2 ककी

3 जारिा

िीच नदए गए कट का परय ग कर सही उततर चनिए

(a) किि 1

(b) किि 1 औऔ 2

(c) किि 2

(d) 1 2 और 3

Q73) निमननिखित कथि ो म स कौि-सास सही हह

1 राषटर कट सामराजय की सथापिा दोनतदगण ि की थी

नजसि मानयाित म अपिी राजरािी की

सथापिा की थी

2 राषटर कट समराट अम घििण एक ििक था और

उस कनिताओो पर पहिी कननड़ पसतक नििि

का शरय नदया जाता ह

िीच नदए गए कट का परय ग कर सही उततर चनिए

(a) किि 1

(b) किि 2

(c) 1 और 2 द ि ो

(d) ि त 1 ि ही 2

Q74) निमननिखित कथि ो म स कौि-सास सही हह

1 कशब चोदर सि ि ततवब नरिी सभा की

अधयकषता की थी ज आधयाखतमक सतय की

ि ज म सोिि थी

2 बरहम समाज ि मािि गररमा पर बि नदया

मनतणपजा का निर र नकया और सती परथा जसी

सामानजक बराइय ो की आि चिा की

िीच नदए गए कट का परय ग कर सही उततर चनिए

(a) किि 1

(b) किि 2

(c) 1 और 2 द ि ो

(d) ि त 1 ि ही 2

Q75) निमननिखित कथि ो म स कौि-सास सही हह

1 भारत म नचशती नसिनसिा खवाजा म इिददीि

नचशती क दवारा सथानपत नकया गया था

2 नचशती परोपरा की एक परमि निशिता

आतमसोयम थी नजसम साोसाररक म ह स दरी

बिाए रििा शानमि था

िीच नदए गए कट का परय ग कर सही उततर चनिए

(a) किि 1

(b) किि 2

(c) 1 और 2 द ि ो

(d) ि त 1 ि ही 2

RAUSIAS-FC19E1003 27

Q70) Which of the following statements

isare correct

1 During the Ahmedabad Mill Strike

Mahatma Gandhi undertook a fast

unto death to strengthen the

workersrsquo resolve

2 The fast had effect of putting

pressure on mill owners who

finally agreed to give the workers a

15 per cent increase in wages

Select the correct answer using the code

given below

(a) 1 only

(b) 2 only

(c) Both 1 and 2

(d) Neither 1 nor 2

Q71) Which of the following are included in

the UNESCOrsquos list of the representative

list of the intangible cultural heritage of

humanity from India

1 Mudiyettu

2 Sankirtana

3 Kumbh Mela

Select the correct answer using the code

given below

(a) 1 and 2 only

(b) 2 and 3 only

(c) 3 only

(d) 1 2 and 3

Q72) Which of the following tribes isare

related to Nagaland

1 Angami

2 Kuki

3 Jarawa

Select the correct answer using the code

given below

(a) 1 only

(b) 1 and 2 only

(c) 2 only

(d) 1 2 and 3

Q73) Which of the following statements

isare correct

1 Rashtrakuta kingdom was founded by Dantidurga who established his capital at Manyakhet

2 Amoghavarsha a Rashtrakuta king was an author and is credited with writing the first

Kannada book on poetics

Select the correct answer using the code given below

(a) 1 only

(b) 2 only

(c) Both 1 and 2

(d) Neither 1 nor 2

Q74) Which of the following statements isare correct

1 Keshab Chandra Sen headed the Tattvabodhini Sabha which was engaged in search of spiritual truth

2 The Brahmo Samaj laid emphasis on human dignity opposed idolatry and criticized such social

evils as the practice of Sati

Select the correct answer using the code given below

(a) 1 only

(b) 2 only

(c) Both 1 and 2

(d) Neither 1 nor 2

Q75) Which of the following statements isare correct

1 The Chishti order was established in India by Khwaja Moinuddin

Chishti

2 A major feature of the Chishti tradition was austerity including maintaining a distance from the

worldly power

Select the correct answer using the code

given below

(a) 1 only

(b) 2 only

(c) Both 1 and 2

(d) Neither 1 nor 2

T e s t i s p a r t o f R a u rsquo s I A S T e s t s e r i e s f o r P r e l i m i n a r y E x a m 2 0 1 9

FOUNDATION + CURRENT AFFAIRS

GENERAL STUDIES (PAPER ndashI)

FOUNDATION TEST ndashIII

SUBJECT NCERT History Class VI-X + Current Affairs

Time Allowed 1frac12 Hours Maximum Marks 150

I NSTRUCT IONS

1 IMMEDIATELY AFTER THE COMMENCEMENT OF THE EXAMINATION YOU SHOULD CHECK

THAT THIS TEST BOOKLET DOES NOT HAVE ANY UNPRINTED OR TORN or MISSING PAGES OR

ITEMS ETC IF SO GET IT REPLACED BY A COMPLETE TEST BOOKLET

2 This Test Booklet contains 75 items (questions) Each item is printed both in Hindi and English

Each item comprises four responses (answers) You will select the response which you want to mark

on the Answer Sheet In case you feel that there is more than one correct response mark the

response which you consider the best In any case choose ONLY ONE response for each item

3 You have to mark all your responses ONLY on the separate Answer Sheet (OMR sheet) provided

Read the directions in the Answer Sheet

4 All items carry equal marks

5 Before you proceed to mark in the Answer Sheet the response to various items in the Test booklet

you have to fill in some particulars in the Answer Sheet as per instructions contained therein

6 After you have completed filling in all your responses on the Answer Sheet and the examination has

concluded you should hand over to the Invigilator only the Answer Sheet You are permitted to

take away with you the Test Booklet

7 Penalty for wrong answers

THERE WILL BE PENALTY FOR WRONG ANSWERS MARKED BY A CANDIDATE IN THE

OBJECTIVE TYPE QUESTION PAPERS

(i) There are four alternatives for the answer to every question For each question for which a

wrong answer has been given by the candidate one-third of the marks assigned to that

question will be deducted as penalty

(ii) If a candidate gives more than one answer it will be treated as a wrong answer even if one of

the given answers happens to be correct and there will be same penalty as above to that

question

(iii) If a question is left blank ie no answer is given by the candidate there will be no penalty for

that question

T h i s t e s t i s p a r t o f R a u rsquo s I A S T e s t s e r i e s f o r P r e l i m i n a r y E x a m 2 0 1 9

Test Code

FC19E1003

FC19H1003 29

Answers and Explanations of

NCERT History Class VI-X + Current Affairs (FC19E1003)

Q1) उततर (c)

सपषटीकरण

- ऋगवद म दविय ो और दिताओो क समवपित एक

हजार स अविक सत तर (शल क) ह

- य शल क ऋविय ो क दवारा रच गए थ और परि ो

दवारा सीख जात थ

- हालाोवक कछ शल क मवहलाओो (जस वक अपाला

घ सा ल पामदरा मतरयी और गागी) क दवारा भी रच

गए थ

- ऋगवद म सोिाद क रप म कई शल क मौजद ह

- हम विशवावमतर नामक एक ऋवि और दविय ो क

रप म पजी जान िाली द नवदय ो (वयास और

सतलज) क बीच िाताि का उदाहरण वमलता ह

- इसस पता चलता ह वक विशवावमतर िवदक काल स

सोबोवित थ

Q2) उततर (b)

सपषटीकरण

- करनल गफाओो स राख क अिशि परापत हए ह

ज इस ओर सोकत करत ह वक ततकालीन ल ग

अवि क उपय ग स पररवचत थ

- य गफाएो आोधर परदश म सथथत ह

Q3) उततर (c)

सपषटीकरण

bull बरािह म ितिमान कशमीर म सथथत एक

परागवतहावसक थथल ह जहाो ल ग गडढ क घर ो का

वनमािण करत थ

bull य घर जमीन क ख द कर बनाए जात थ तथा नीच

जान क वलए सीवियाा ह ती थी

bull ऐसा अनमान लगाया जाता ह वक य घर ठो ड क

मौसम म आशरय परदान करत थ

Q4) उततर (c)

सपषटीकरण

bull परालख-विदया (Epigraphy) क वशलालख ो क

अधययन क रप म पररभावित वकया जाता ह

bull हसतवलसखत दसतािज ो क माधयम स इवतहास

और सावहतय क अधययन क पाोडवलवप विजञान

(Manuscriptology) कहत ह

bull पराचीन लखन परणावलय ो क अधययन और

ऐवतहावसक पाोडवलवपय ो क समझन तथा वतवथ

वनिािरण क पलीओगराफी (Palaeography) कहा

जाता ह

bull नयवमजमविकस (Numismatics) वसक ो क

अधययन क सोदवभित करता ह

Q5) उततर (a)

सपषटीकरण

- चरक सोवहता चरक क दवारा वलखी गई आयिद

और िदयक-शासर पर एक महतवपणि पसतक ह

- ि भारतीय िदयक-शासर की पारमपररक परणाली

वजस आयिद क नाम स जाना जाता ह क

अभयासकताि थ

- ऐसा माना जाता ह वक चरक का विकास दसरी

शताबदी (ईसा पिि) और दसरी शताबदी (ईसवी) क

मधय हआ था

Q6) उततर (b)

सपषटीकरण

- भाग फसल ो पर वलए जान िाल कर क सोदवभित

करता ह ज कल फसल उतपादन का 16 िाो भाग

था

- ldquoकममकारrdquo शबद भवमहीन कवि शरवमक िगि क

वलए परय ग वकया जाता था

- ldquoअशवमिrdquo (वजस घ ड क बवलदान क रप म भी

जाना जाता ह) एक अनषठान ह ता था वजसम एक

घ ड क सवतोतर रप स घमन क वलए छ ड वदया

FC19H1003 30

जाता ह और राजा क सवनक उसकी रखिाली

करत थ

Q7) उततर (d)

सपषटीकरण

- ऋगववदक काल म घ ड ो क रथ ो म ज ता जाता था

ज (रथ) भवम मिवशय ो आवद पर कबजा करन क

वलए लड गए यद ो म उपय ग वकए जात थ

- इसस यह पता चलता ह वक घ ड ो यकत रथ ो का

उपय ग महाजनपद काल स काफी पहल आरमभ

हआ था

- ऋगववदक काल म मिवशय ो भवम जल आवद पर

कबजा करन क वलए तथा ल ग ो क पकडन क

वलए यद वकय जात थ

- अविकाोश परि इन यद ो म भाग वलया करत थ

- हालाोवक उस समय क ई वनयवमत सना नही ो ह ती

थी लवकन उस काल म सभाऐो ह ती थी ो वजनम

ल ग यद क मामल ो पर चचाि करत थ

- वनयवमत सनाएा महाजनपद काल का िवशषटय थी

वजनम पदल सवनक ो की विशाल सनाएा रथ तथा

हाथी शावमल ह त थ

Q8) उततर (a)

सपषटीकरण

- बद शाकय कल स सोबोवित थ और कशीनारा म

उनका वनिन हआ था

- बद न अपनी वशकषाएा पराकत भािा म दी थी ो ज

आम ल ग ो की भािा थी

Q9) उततर (c)

सपषटीकरण

- पराचीन भारत म दशिनशासर की छह शाखाएा थी ो

िशविक नयाय समखया य ग पिि वममाोसा और

िदाोत या उततर वममाोसा

- इनकी थथापना करमश कनाद गौतम कवपल

पतोजवल जावमनी और वयास ऋविय ो न की थी

Q10) उततर (b)

सपषटीकरण

महािीर की वशकषाऐो छठी शताबदी म िललभी म

सोकवलत की गई थी ो

Q11) उततर (c)

सपषटीकरण

- पारमपररक रप स चाणकय क कौविलय अथिा

विषणगपत क नाम स जाना जाता ह

- उसन अथिशासतर ज एक पराचीन भारतीय

राजनवतक आलख ह वलखा था

Q12) उततर (d)

सपषटीकरण

- भारत का राषटर ीय वचनह सारनाथ (उततर परदश) क

अश क सतमभ क ऊपर (शीिि पर) वसोह कवपिल

का एक अनरपण ह

- इस राषटर ीय वसदाोत सतयमि जयत क साथ

सोय वजत वकया गया ह

- रामपिि बल का नाम रामपिि (वबहार) क नाम पर

पडा जहाा इसकी ख ज हई थी

- यह अपन नाजक नकाशी मॉडल क वलए परवसदद

ह वजसम क मल तवचा सोिदनशील नथन ो सतकि

कान और मरबत िााग ो क शरषठतर परवतरप क

परदवशित वकया गया ह

- यह भारतीय और फारसी ततव ो का एक ससममशरण

- सोवकससा उततर परदश म सथथत ह

Q13) उततर (a)

सपषटीकरण

का िर वसोह ज एक महान य दा थ वबहार स

सोबोवित थ

Q14) उततर (b)

सपषटीकरण

िललालर शबद बड भ-सवावमय ो क वलए परय ग

वकया जाता था

FC19H1003 31

Q15) उततर (c)

सपषटीकरण

- अररकमड एक तिीय बसती थी जहाो दर दश ो स

आन िाल जहाज ो का माल उतारा जाता था

- यहाो पर ईोि ो का एक विशाल ग दाम वमटटी क

बतिन (वजनम एमफ रा - द हरी मवठय ो का लोबा

घडा - शावमल ह) और एरिाइन (Arretine)

मदभाोड पाए गए थ

- इस थथान पर र मन दीपक काोच क बन पातर और

रतन भी पाए गए थ

Q16) उततर (a)

सपषटीकरण

- मिनदर सोगम कविताओो म उसललसखत एक

तवमल शबद ह वजसका अथि ह ldquoतीन परमखrdquo

- यह तीन सततारि पररिार ो क मसखयाओो क वलए

परय ग वकया जाता ह च ल चर और पाणडय

Q17) उततर (c)

सपषटीकरण

- ऋग िद म सभा विदाथा तथा गण जसी

जनजावतय ो पर अथिा किोब पर आिाररत

सभाओो का उललख ह

- आरसमभक िवदक काल म सभाओो और सवमवतय ो

का विशि महतव ह ता था

- यहाा तक की मसखया अथिा राजा भी उनका

समथिन परापत करन क वलए आतर रहत थ

Q18) उततर (a)

सपषटीकरण

- जन िमि न ईशवर क अससततव क मानयता त दी ह

वकनत उसन ईशवर क वजना क पद स नीच रखा

- जन िमि न बौद िमि की तरह िणि परणाली की

भरतिना नही ो की थी

Q19) उततर (d)

सपषटीकरण

- च ल ो और पाणडय ो न शसकतशाली तिीय शहर ो का

विकास वकया था

- च ल ो का सबस महतवपणि शहर पहार (या

कािरीपटटीनम) था |

- मदरई पाणडय ो की राजिानी थी

Q20) उततर (b)

सपषटीकरण

- ldquoबदचररतrdquo बद का जीिन-ितताोत ह

- इस अशवघ ि क दवारा वलखा गया था

Q21) उततर (a)

सपषटीकरणः

- तवमल कवि अपपर भगिान वशि क भकत थ

- इस परकार ि एक नयनार सोत थ

Q22) उततर (d)

सपषटीकरणः

- समदरगपत एक परवसद गपत शासक था

- उसन वसक ो पर िीणा बजात हए अपनी छवि

अोवकत करिाई थी

- यह सोगीत क परवत उसक परम क दशािता ह

- हम उसकी इलाहाबाद परशससत स महतवपणि

ऐवतहावसक जानकारी वमलती ह वजसकी रचना

उसक दरबार क कवि हररसन न की थी

Q23) उततर (b)

सपषटीकरणः

- विकरम सोित की शरआत ििि 58 ईसा पिि म

चनदरगपत वदवतीय न की थी

- यह शक ो पर उसकी जीत और उस विकरमावदतय

की पदिी वमलन क उपलकषय म आरमभ वकया गया

था

FC19H1003 32

- बानभटट न हिििििन का जीिन-ितताोत हििचररत

(ज सोसकत म थी) वलखी थी

Q24) उततर (c)

सपषटीकरणः

- सोवि-विगरावहका यद एिो शाोवत का मोतरी

- साथििाह वयापाररय ो क कावफल ो का नता

Q25) उततर (a)

सपषटीकरणः

- जआन झाोग (हसआन रताोग ndash Hsuang Tsang)

एक चीनी यातरी था ज हिििििन क शासनकाल म

भारत आया था

- ििि 630 ईसवी स ज दशक आरमभ हआ था उसम

जआन झाोग मधय एवशया ईरान और

अफग़ावनसतान की यातरा करन क पशचात कशमीर

क रासत स भारत आया था

- उसन उततर स पिि तक की यातरा की और िह

लगभग 2 ििि वबहार म रहा

- जआन झाोग न नालनदा विशवविदयालय म विदयावथिय ो

और विदवान ो क साथ पारसपररक विचार-विमशि

वकया थथानीय भािाओ ा म वनपणता परापत की तथा

बौद सतप ो की ख ज की

Q26) उततर (c)

सपषटीकरणः

- परदवकषणा पथ बौद िासतकला म सतप क चार ो

ओर बनाया जान िाला एक घमािदार पथ ह ता

- परशन म वदए गए बाकी क तीन ो ततव वहोद मसनदर ो की

िासतकला क भाग ह

Q27) उततर (d)

सपषटीकरणः

परशन म वदए गए सभी मोवदर ो म वयापक रप स

ईोि ो (पकी ईोि ो) का परय ग पतथर ो क साथ हआ

Q28) उततर (c)

सपषटीकरण

- महममद कली कतब शाह ग लकणडा का सलतान

था

- िह अकबर का समकालीन था

- सावहतय और िासतकला म उसकी अतयाविक

रवच थी

- िह एक महान कवि था

- िह दसखनी उदि फारसी और तलग म वलखता था

- उसन अपन पीछ एक विसतत वदिान (सोगरह)

छ डा ह

- अभी हाल ही म तलोगाना म ग लकणडा क वकल

क अनदर खदाई वकय गए बाग-ए-नाया वकला

बाग क चार ो ओर रप-रखा क मानवचतरण क

वलए भारतीय परातासतवक सिकषण (The

Archaeological Survey of India ndash ASI)

गराउणड पनीिर विोग रडार (Ground Penetrating

Radar) का परय ग करगा

Q29) उततर (a)

सपषटीकरणः

- वसलपपावदकारम एक तवमल महाकावय ह वजसकी

रचना इलाोग क दवारा लगभग 1800 ििि पिि की

गई थी

- यह क िलन नामक एक वयापारी की कहानी ह

ज माििी नामक एक गवणका (िशया) स परम

करन लगा था

- मवनमकलाई क िलन और माििी की पतरी की

कहानी ह

Q30) उततर (a)

सपषटीकरण

- चरक आयिद और वचवकरता की एक महतवपणि

रचना चरक सोवहता क लखक ह

- बरहमगपत क अपनी रचना बरहम-सफि-वसदानत

(ज एक खग लीय रचना ह) क कारण परवससद

वमली

FC19H1003 33

- बगदाद म इसका अनिाद अरबी भािा म वकया

गया था

- इसका इसलावमक गवणत और खग ल-विजञान पर

महतवपणि परभाि पडा था

- बाद म अपन जीिनकाल म बरहमगपत न

ldquoखोडखयाकrdquo वलखी ज एक खग लीय पससतका

(एक छ िी पसतक) थी

- इसम आयिभटट की अिि-रावतर क परतयक वदन की

शरआत परणाली का परय ग वकया गया था

Q31) उततर (c)

सपषटीकरण

- अमीर खसर एक परवसद सफी सोगीतकार कवि

और विदवान थ

- 1318 म उनह ोन पाया वक इस भवम (वहोदसतान) क

हर कषतर म अलग-अलग भािा थी लाहौरी

कशमीरी दवारसमदरी (दवकषणी कनाििक म)

तलोगाना (आोधर परदश म) गजरी (गजरात म)

माबारी (तवमलनाड म ) अििी (पिी उततर परदश

म) और वहोदिी (वदलली क आस-पास क कषतर म)

आवद

- उनह न यह बताया वक सोसकत वकसी भी कषतर स

सोबोवित नही ो थी और किल बराहमण ही इस भािा

का जञान रखत थ

Q32) उततर (c)

सपषटीकरण

- वहरणय-गभि सववणिम गभि क सोदवभित करता ह

- जब बराहमण ो की सहायता स यह अनषठान वकया

जाता था त यह माना जाता था वक बवल दन िाल

का कषवतरय क रप म पनजिनम ह गा

Q33) उततर (d)

सपषटीकरण

- कदमई भवम राजसव पर कर क सोदवभित करता

- गवावलयर परशससत म नागभि क दवारा वकय गए

श िण का िणिन वकया गया ह |

- नागभि एक परवतहार राजा था

Q34) उततर (b)

सपषटीकरण

- राजतरो वगनी 12िी ो शताबदी म कलहन क दवारा

रवचत एक सोसकत पसतक (िकसट) ह

- यह परारसमभक भारत की ऐवतहावसक इवतितत थी

- तकि सोगत रप स इस अपन परकार की सिोततम

और सिािविक विशवसनीय कवत माना जाता ह

- यह कशमीर कषतर क पराचीनतम समय स लकर

उसकी रचना की तारीख तक क समपणि इवतहास

का आचछादन करती ह

Q35) उततर (c)

सपषटीकरण

- गााि की आम सभा क ldquoउरrdquo कहा जाता था

- ldquoउरrdquo म गााि क सभी कर दन िाल वनिासी

शावमल ह त थ

Q36) उततर (a)

सपषटीकरण

- वदलली सलतनत म ldquoतारीखrdquo इवतहास लखन का

एक रप था

- ldquoतािरीखrdquo क लखक विदवान परि ह त थ वजनम

सवचि परशासक इतयावद शावमल थ

Q37) उततर (a)

सपषटीकरण

- अलाउददीन सखलजी अपन सवनक ो क ितन का

भगतान नकद म करता था न वक इकता क रप

- सवनक अपना सामान वदलली म वयापाररय ो स

खरीदत थ अतः इस बात का भय था वक वयापारी

कही ो िसतओो का मलय न बिा द

- इसकी र कथाम क वलए अलाउददीन सखलजी न

वदलली म कीमत ो क वनयसित वकया

FC19H1003 34

- अविकारीगण धयानपििक मलय ो का सिकषण करत

थ तथा ज वयापारी वनिािररत मलय पर माल नही ो

बचत थ उनक दसणडत वकया जाता था

Q38) उततर (d)

सपषटीकरण

- वदलली सििपरथम त मर राजपत ो क अिीन उनक

सामराजय की राजिानी बनी थी

- 12िी ो शताबदी क मधय म अजमर क चौहान ो

(वजनह चाहमान ो क नाम स भी जाना जाता ह) न

त मर राजपत ो क परावजत वकया था

- त मर ो और चौहान ो क अिीन वदलली एक

महतवपणि िावणसजयक क दर बन गया था

- कई जन वयापारी यहाा रहन लग थ और उनह ोन

कई मोवदर भी बनिाए

- यहाा पर मवदरत वसक वजनह ldquoदहलीिालrdquo क नाम

स जाना जाता था वयापक रप स परचलन म थ

Q39) उततर (c)

सपषटीकरण

- म ठ की मसिद का वनमािण वसको दर ल दी क

राजयकाल म उसक मिी क दवारा करिाया गया

था

- बगमपरी मसिद का वनमािण महममद तगलक क

शासनकाल म हआ था

- यह मसिद विशव का पणयथथान (The

Sanctuary of the World) और वदलली म महममद

तगलक की नई राजिानी जहाोपनाह की मखय

मसिद थी

- कववत- अल - इसलाम मसिद का विसतार

इलतसिश और अलाउददीन सखलजी न वकया था

- मीनार का वनमािण तीन सलतान ो कतबददीन ऐबक

इलतसिश और वफर ज शाह तगलक क दवारा

करिाया गया था

Q40) उततर (c)

सपषटीकरण

- मगल ो क अिीन मनसबदार शबद उस वयसकत क

वलए सोदवभित वकया जाता था वजसक पास मनसब

(अथाित पद) ह ता था

- उस अपना ितन राजसव कायो वजनह जागीर कहत

थ क रप म परापत ह ता था

Q41) उततर (b)

सपषटीकरण

- ldquoभारत छ ड आोद लनrdquo वबरविश शासन क

सखलाफ ल ग ो का एक सवाभाविक विदर ह था

- असखल भारतीय काोगरस सवमवत न 8 अगसत 1942

क बमबई म एक बठक का आय जन वकया था

- इस बठक म परवसद सोकलप ldquoभारत छ ड rdquo क

पाररत वकया गया और इस उददशय क परापत करन

क वलए गाोिी क नततव म एक अवहोसक जन सोघिि

आोद लन की शरआत का परसताि वदया गया

- लवकन अगल ही वदन गाोिी और काोगरस क अनय

परमख नताओो क वगरफतार कर वलया गया

- काोगरस क एक बार वफर अिि घ वित वकया गया

था

Q42) उततर (c)

सपषटीकरण

- साइमन कमीशन यनाइविड वको गडम क सात

साोसद ो का एक समह था

- इस वबरविश भारत क वलए सोििावनक सिार ो का

सझाि दन क वलए गवठत वकया गया था

- इस आय ग म िररषठ वबरविश राजनता सर जॉन

साइमन क नततव म किल वबरविश सदसय ही

शावमल थ

- इसवलए भारत क ल ग ो न साइमन कमीशन क

आगमन क विरद आोद लन वकया था

Q43) उततर (a)

सपषटीकरण

bull दादा भाई नौर जी भारत म वबरविश शासन क

आवथिक पररणाम ो क बार म अपनी विर िी

(परवतकल) राय क वलए जान जात थ

FC19H1003 35

bull अपन कई लख ो और भािण ो म विशि रप स

ldquoपाििी एो ड अन-वबरविश रल इन इसणडया

(Poverty and Un-British Rule in India) म

नौर जी न यह तकि वदया वक भारत पर अतयविक

कर लगाया गया था और इसकी सोपवतत इोगलड की

ओर परिावहत की जा रही थी

bull उनह ोन पराचीन भारतीय गरोथ ो की वयाखया करन

का और भारतीय ो क आिविशवास क बहाल

करन पर कायि नही ो वकया था

उनह ोन वकसी और बात स पहल सभी सामावजक

बराइय ो क उनमलन की आिशयकता पर भी बल

नही ो वदया था

Q44) उततर (c)

सपषटीकरण

bull अगसत 1932 म वबरविश परिानमोतरी मकड नालड न

अपन साोपरदावयक परसकार (The Communal

Award) की घ िणा की थी

bull यह भारत क कई साोपरदावयक वहत ो क बीच विवभनन

सोघिो क हल करन क वलए वबरिन का एकतरफा

परयास था

bull यह परसकार (Award) बाद म 1935 क

अविवनयम (The Act of 1935) म शावमल वकया

गया था

bull इस साोपरदावयक परसकार न मससलम ो क वलए

आरवकषत एक अलग वनिािचक मणडल फॉमिल का

विसतार अनय अलपसोखयक ो क वलए वकया था

वजसम वसख ो भारतीय ईसाइय ो आोगल-भारतीय

समदाय यर पीय समदाय तथा विवशषट कषतरीय

समह ो क शावमल वकया गया था

bull गाोिी न इस परसताि क भारतीय समाज क

विभावजत करन क वलए एक घवणत वबरविश

सावजश क रप म दखा और उसक सखलाफ

आमरण अनशन वकया

Q45) उततर (b)

सपषटीकरण

मौजदा आयात और वनयाित क अवतररक़त

औपवनिवशक भारत क वनमनवलसखत खचो क

वलए एक विशिवनवशचत िन रावश भी दनी पडती

थी

(i) परशासन क वयय

(ii) सना क रख-रखाि क वयय

(iii) यद क वयय

(iv) सिावनितत अविकाररय ो की पशन तथा

(v) वबरिन दवारा अपनी उपवनिश बसती

(कॉल नी) क रख-रखाि क वयय

इनह गह शलक (Home Charges) क रप म

जाना जाता था और लगभग परी तरह स भारत क

दवारा इनका भगतान वकया जाता था

bull गह शलक म वनमनवलसखत घिक शावमल थ

(i) भारतीय ऋण पर दय बयाज

(ii) ईसट इोवडया को पनी क शयरिारक ो क

लाभाोश

(iii) लोदन म भारत कायािलय चलान क वलए िन

(iv) भारत म वनयकत वबरविश कवमिय ो क ितन

और पशन का भगतान करन क वलए िन

(v) रलि पर बयाज

(vi) नागररक और सनय शलक

(vii) इोगलड म सट र (सामगरी) की खरीद

Q46) उततर (b)

सपषटीकरण

bull भारतीय राषटर ीय काोगरस का लाहौर सतर 1929 म

जिाहरलाल नहर की अधयकषता म आय वजत

वकया गया था

bull इस सतर म भारतीय राषटर ीय आोद लन स समबसित

कई महतवपणि पररणाम सामन आय थ

(i) सििपरथम इस सतर म काोगरस क अधयकष पद

पर जिाहरलाल नहर क चना गया था ज

काोगरस म िामपोवथय ो की बिती हई ताकत

का सपषट सोकत था

(ii) दसरा इस सतर म पहली बार काोगरस न पणि

सवतोतरता की माोग क उठाया था

इस परकार की माोग काोगरस मोच स पहल कभी भी

नही ो उठाई गई थी

Q47) उततर (b)

सपषटीकरण

FC19H1003 36

bull इस ररप िि न वकसी भी समदाय क वलए पथक

वनिािचक मोडल अथिा अलपसोखयक ो क वलए

भाराोश की वसफाररश नही ो की थी

bull तथावप इस ररप िि न उन पराोत ो म अलपसोखयक

सीि ो क आरकषण की अनमवत दी थी जहाा पर कम

स कम दस परवतशत अलपसोखयक ह

bull लवकन यह समदाय क आकार क अनपात म ह ना

चावहए था

bull इस ररप िि म भारत क वलए पणि सवतोतरता क

वलए क ई पराििान नही ो था

Q48) उततर (c)

सपषटीकरण

bull आरो वभक िवदक आयो का िमि मखय रप स

परकवत की पजा और यजञ था

bull परारो वभक आयि िमि परकवत की पजा क समान था

bull िासति म उनक चार ो ओर की शसकतयाा वजनह न

त ि वनयोवतरत कर सकत थ और न ही समझ पाए

थ उनह वदवयता क साथ वनिवशत वकया गया तथा

उनह मादा या नर दिीदिताओो क रप म

परतीकतव वकया गया था

bull उनह ोन कछ यजञ ो का भी वनषपादन वकया था

Q49) उततर (b)

सपषटीकरण

bull सडक और नदी-मागि (जल-मागि) डकती स

सरवकषत नही ो थ

bull उललखनीय ह वक हिििििन क शासनकाल क

दौरान यआन चिाोग (हयएन साोग) का सारा

सामान लि वलया गया था

Q50) उततर (c)

सपषटीकरण

परशन म वदए गए द न ो कथन सही ह

Q51) उततर (b)

सपषटीकरण

bull परोदर दास एक सोत और भगिान कषण क एक

महान भकत थ

bull परोदर दास क कनाििक सोगीत क वपतामह क

रप म जाना जाता ह

bull यदयवप उनक जनम-थथान क बार म काफी

अिकल लगाई जाती रही ह

bull तथावप अब कननड विशवविदयालय हमपी क दवारा

गवठत एक विशिजञ सवमवत इस वनषकिि पर पहोची

ह वक उनका जनम थथान सोभितया कनाििक का

एक छ िा-सा गााि कषमपरा (वशिम गगा वजला)

था

Q52) उततर (c)

सपषटीकरण

bull शरी तयागराज शरी शयाम शासतरी और शरी मथसवामी

दीवकषतर क कनाििक सोगीत की वतरमवति माना

जाता ह

bull उनक कारण ही 18िी ो-19िी ो शताबदी म कनाििक

सोगीत का सववणिम यग आया था

Q53) उततर (d)

सपषटीकरण

bull अभी हाल ही म लौह यगीन-महापािावणक काल

का 2000 ििि पराना एक दलिभ सारक फगस

(Sarcophagus) (पतथर का ताबत) क ललम क

वियर गाोि (क वयलडी क पास वजला क वझक ड

करल राजय) की एक रॉक-कि गफा स ख जा गया

bull यह ताबत वजसम हविय ो क िकड थ खदाई क

दौरान वमला

bull अभी तक इस परकार की दलिभ ख ज करल क

मातर द ही थथान ो स हई ह

bull य द न ो सारक फगी (Sarcophagi) (पतथर क

ताबत) चियर और अथ ली (वजला क वझक ड) क

महापािाण थथल ो स वमल ह

Q54) उततर (a)

सपषटीकरण

FC19H1003 37

दवकषण भारत म महापािाण सोसकवत एक पणि

विकवसत लौह यगीन सोसकवत थी

Q55) उततर (d)

सपषटीकरण

bull च ल पाणडय और करलपतर (चर) इन तीन ो का

उललख अश क क अवभलख ो म वकया गया ह

bull सोभितः य भौवतक सोसकवत क उततर

महापािावणक चरण म थ

Q56) उततर (d)

सपषटीकरण

bull भीमा-क रगाोि की लडाई ततीय आोगल-मराठा

यद का वहससा थी

Q57) उततर (b)

सपषटीकरण

bull राजकमार शकल न गाोिीजी क चोपारण आन तथा

वतनकवथया परणाली स जडी समसया की जाोच क

वलए रारी करन क वलए दश भर म उनका

अनसरण वकया था

bull बज वकश र राजदर परसाद महादि दसाई और

नरहरी पाररख चोपारण सतयागरह क दौरान गाोिी

जी क सहय गी थ

Q58) उततर (b)

सपषटीकरण

bull बराहमण ो और बौद मठिाररय ो क कर-मकत गााि

अनदान म दन की परथा सतिाहन ो न आरमभ की

थी

Q59) उततर (c)

सपषटीकरण

इस कायिकरम क उददशय वनमनानसार ह

(i) बवनयादी पयििन आिाररक सोरचना का विकास

करना

(ii) चयवनत (पहचान वकय गए) कषतर ो म आजीविका क

सजन क वलए दश क साोसकवतक और विरासत

मलय ो क बिािा दना

(iii) विरासत समारक थथल ो पर विशव सतरीय आिाररक

सोरचना विकवसत करक एक सतत तरीक स

पयििक आकििण म िसद करना

(iv) थथानीय समदाय ो की सवकरय भागीदारी क माधयम

स र रगार ो का सजन करना

(v) र रगार उतपादन और आवथिक विकास क वलए

पयििन कषमता का उन पर परभाि का उपय ग

करना तथा

(vi) िारणीय पयििन आिाररक सोरचना का विकास

करना और उसका उवचत सोचालन तथा

रखरखाि सवनवशचत करना

Q60) उततर (b)

सपषटीकरण

bull यह वनकाय ििि 1987 म अससततव म आया था

bull यह एक राषटर ीय सतर का शीिि सोगठन ह ज भारत

सरकार क जनजातीय मामल ो क मोतरालय क

परशासवनक वनयोतरण क अिीन काम कर रहा ह

bull इसका पोजीकत और परिान कायािलय नई वदलली

म सथथत ह

Q61) उततर (c)

सपषटीकरण

bull परमचोद क उपनयास ो म परमाशरम रोगभवम गबन

कमिभवम और ग दान शावमल ह

bull ग रा रिी ोदरनाथ िग र क दवारा रवचत उपनयास ह

bull अभी हाल ही म मोशी परमचोद की 138िी ो जयोती दश

भर म मनाई गई थी

Q62) उततर (b)

सपषटीकरण

bull ldquoवगदाrdquo पोजाब (भारत) एिो पावकसतान की

मवहलाओो क दवारा तयौहार क समय और फसल

की बिाई तथा किाई क अिसर पर वकया जान

िाला एक पारोपररक दहाती नतय ह

FC19H1003 38

bull इस नतय क माधयम स पोजाबी मवहलाऐो अपनी

परसननता परकि करती ह तथा वगदा क परदशिन क

माधयम स परि िचिसव िाल समाज म मवहलाओो

की दबी हई भािनाओो क परकि करती ह

bull चोवक इस नतय का परि ो क साथ क ई सोबोि नही ो

ह अतः किल मवहलाऐो ही इसम भाग ल सकती

bull हर साल तीज समार ह क दौरान पोजाब म वगदा

नतय वकया जाता ह

तीज भारत क कछ भाग ो म मवहलाओो क दवारा

मनाया जान िाल कई तयौहार ो क वलए एक

वयापक नाम ह

Q63) उततर (a)

सपषटीकरण

- मजम-उल-बहरीन या द समदर ो का सोगम

नामक उललखनीय रचना दारा वशक ह क दवारा

वलखी थी

- भारत क उपराषटर पवत शरी एम िकया नायड न कहा

ह वक राजकमार दारा वशक ह की रचनाएा शाोवत

और सदभाि क बिािा दन क वलए एक तारा सर त

क रप म सामन आ सकती ो ह

- उपराषटर पवत गत ििो क भला वदए गए राजकमार

दारा वशक ह क परदवशित परचवलत करन हत

आय वजत एक परदशिनी का दौरा करन क बाद एक

सभा क सोब वित कर रह थ

- इस परदशिनी का आय जन फर क इस गौवियर

(Francois Gautier) क दवारा lsquoइोवदरा गाोिी नशनल

सिर फॉर द आििसrsquo (The Indira Gandhi

National Centre for the Arts) नई वदलली म

वकया गया था

Q64) उततर (c)

सपषटीकरण

- ग मतशवर परवतमा जन भगिान बाहबली क

समवपित ह

- यह एक एक-चटटानी पतथर की मवति ह

- राषटर पवत राम नाथ क विोद न शरिणबलग ला

(कनाििक) म आय वजत वकय जान िाल भवय

अवभिक समार ह महामसतकावभिक का

उदघािन वकया था

- यह समार ह 12 ििो म एक बार ह ता ह

Q65) उततर (c)

सपषटीकरण

bull पराची घािी पराची नदी क चार ो ओर फली हई थी

bull पराची घािी िीर-िीर विलपत ह गई थी

bull पराची नदी भिनशवर स वनकलती ह

bull यह महानदी की एक सहायक नदी ह और यह

परी खदाि किक तथा जगतवसोहपर वजल ो स

ह कर बहती ह

bull इस नदी क पर कषतर क पराची घािी कहा जाता ह

bull यह नदी बोगाल की खाडी म वगरती ह

परातासतवक साकषय स पता चलता ह वक पराची घािी

सभयता हडपपा और म हनज दाड द न ो की

पिििती ह

Q66) उततर (d)

सपषटीकरण

य समारक छतरपर वजल (मधय परदश) म विोधयाचल

पिित शरोखला म सथथत ह

Q67) उततर (a)

सपषटीकरण

bull थॉिस ऑन पावकसतान नामक पसतक डॉ बी

आर अमबडकर न वलखी थी

bull डॉ बी आर अमबडकर की जयोती क अिसर पर

भारत क राषटर पवत न भारत की इस महान हसती

क शरदाोजवल अवपित की थी

bull डॉ बी आर अमबडकर न 1924 म वडपरथड

कलावसर इोसटीटयि (दवलत िगि सोथथान -

बवहषकत वहतकाररणी सभा) और 1927 म समाज

समता सोघ की थथापना की थी

bull अमबडकर का धयान वशकषा कषतर की ओर भी था

bull उनह ोन वशकषा क वनमन िगो म फलान क वलए

पीपलस एजकशन स साइिी (The Peoples

Education Society) क नाम स महाविदयालय ो क

नििकि और छातरािास ो की थथापना की थी

FC19H1003 39

Q68) उततर (b)

सपषटीकरण

bull महरगि भारतीय उपमहादवीप म एक परवसद

निपािाण बसती ह ज बलवचसतान पराोत

पावकसतान म सथथत ह

bull दचपलली (आोधर परदश) क पास नागलर नदी क

पिी ति ो पर चना पतथर क बलॉक क विशाल

विसतार म एक पिि-ऐवतहावसक रॉक आिि थथल की

ख ज की गई ह

bull इसन 1500-2000 ईसा पिि क दौरान गोिर (आोधर

परदश) म विकवसत निपािाण सभयता पर परकाश

डाला ह

Q69) उततर (c)

सपषटीकरण

bull 12िी ो सदी और 13िी ो सदी म काकाविय िोश का

उदय हआ था

bull ि पहल कलयाण क पवशचमी चालकय ो क सामोत थ

bull परारोभ म उनह ोन िारोगल (तलोगाना) क पास एक

छ ि स कषतर पर शासन वकया था

bull उनह ोन ldquoनायक वयिथथाrdquo की शरआत की थी

वजस बाद म विजयनगर क राय शासक ो न

अपनाया और विकवसत वकया था

Q70) उततर (a)

सपषटीकरण

bull गाोिीजी क अनशन स वमल मावलक ो पर दबाि

पडा था ज अोततः शरवमक ो क ितन म 35 परवतशत

की िसद करन क वलए सहमत हए थ

bull गगल (Google) न अनसया साराभाई वजनह ोन

भारत क शरवमक आोद लन म एक अगरणी भवमका

वनभाई थी की 132िी ो जयोती डडल (Doodle) का

वनमािण करक मनाई

Q71) उततर (d)

सपषटीकरण

भारत स यनसक की मानिता की अमति साोसकवतक

विरासत की परवतवनवि सची म वनमनवलसखत शावमल ह

bull कवडयटटम करल का सोसकत रोगमोच

bull मवडयिि करल का अनषठान रोगमोच और नतय

नाविका

bull िवदक मि जाप की परोपरा

bull राजथथान क कालबवलया ल क गीत और नतय

bull रामलीला रामायण का पारोपररक परदशिन

bull सोकीतिन मवणपर का अनषठान गायन ढ ल िादन

और नतय

bull रममन भारत क गििाल वहमालय का िावमिक

तयौहार और अनषठान रोगमोच

bull जाोदीयाला गर पोजाब क ठठर ो की पीतल और

ताोब क वशलप स वनवमित बतिन ो की पारोपररक कला

bull छाऊ नतय पिी भारतीय राजय ो म जनमी शासतरीय

भारतीय नतय कला

bull लददाख का बौद मि जाप िर ाोस-वहमालयी लददाख

कषतर तथा जमम-कशमीर म पवितर बौद गरोथ ो का पाठ

bull य ग

bull नौर र

bull को भ मला

Q72) उततर (b)

सपषटीकरण

bull भारत क राषटर पवत शरी राम नाथ क विोद न

वकसामा नागालड म हॉनिवबल मह रति और

राजय गठन वदिस समार ह का उदघािन वकया

था

bull हॉनिवबल मह रति का नाम भारतीय हॉनिवबल क

नाम पर पडा ह ज एक विशाल और रोगीन जोगली

पकषी ह

bull यह पकषी नागालड राजय की अविकतर जनजावतय ो

की ल ककथाओो म उसललसखत ह

bull नागालड की परमख मानयता परापत जनजावतयाा ह

अोगामी आओ चखसोग चाोग ककी रगमा और

रवलोग आवद

bull ओोग जारिा और ससिनलीस अोडमान-वनक बार

दवीप समह की जनजावतयाा ह

FC19H1003 40

Q73) उततर (c)

सपषटीकरण

bull दकन म राषटर कि शासन दसिी ो सदी क अोत तक

लगभग 200 ििो तक रहा था

bull राषटर कि शासक अपन िावमिक विचार ो म सवहषण

bull उनह ोन न किल शि िमि और िषणि िमि बसलक

जन िमि क भी सोरकषण वदया था

bull एल रा म वशि क परवसद रॉक कि मोवदर का

वनमािण नौिी ो सदी म राषटर कि राजा कषण परथम न

करिाया था

bull उसका उततराविकारी अम घििि जन था लवकन

उसन अनय िमो क भी सोरकषण परदान वकया था

bull राषटर कि ो न मसलमान वयापाररय ो क बसन की

अनमवत दी थी

bull उनह न अपन अविराजय ो म इसलाम क उपदश दन

की भी अनमवत दी थी

bull अभी हाल ही म पाोडिलागटटा (तलोगाना) क

परागवतहावसक चटटान वचतर ो क कषरण की बिती हई

घिनाएा एक गोभीर वचोता का वििय ह

bull यह परागवतहावसक चटटान क नकसान पहाचा

सकता ह

bull पाोडिलागटटा वनमनवलसखत क वलए जाना जाता ह

- 10000 ईसा पिि स 8000 ईसा पिि क वचवतरत

चटटानी आशरय ो क वलए

- राषटर कि काल क एक 8 िी ो सदी क

वशलालख क वलए और

- 12िी ो सदी क काकविय सामराजय क वभवतत

वचतर ो क वलए

Q74) उततर (b)

सपषटीकरण

bull 1828 म राजा राम म हन रॉय न एक नय िावमिक

समाज बरहम सभा की थथापना की थी वजस बाद

म बरहम समाज क नाम स जाना गया था

bull दिदरनाथ िग र न ततवब विनी सभा की अधयकषता

की थी ज आधयासिक सतय की ख ज म सोलि

थी

bull इसका उददशय वहोद िमि क शद करन का और

एकशवरिाद (एक ईशवर म आथथा) का परचार करना

था

bull नय समाज की थथापना क आिार थ कारण

(तकि ) क द सतमभ तथा िद और उपवनिद

bull अभी हाल ही म सािारण बरहम समाज का कछ

काननी मदद ो क लकर पवशचम बोगाल सरकार क

साथ काननी वििाद चल रहा ह

Q75) उततर (c)

सपषटीकरण

bull भारत म वचशती वसलवसल की थथापना खवाजा

म इनददीन वचशती क दवारा की गयी थी

bull ि 1192 ईसवी क आसपास भारत आय थ

bull वचशतीय ो क बारहिी ो शताबदी क उततरािि म भारत

म आन िाल सफीय ो क समह ो म सबस

परभािशाली माना जाता ह

bull उनह ोन थथानीय िातािरण क साथ सफलतापििक

अनकलन वकया और उनह ोन भारतीय भसकत

परोपराओो क कई पहलओो क अपनाया

bull अजमर म सफी अपरकि खवाजा म इनददीन वचशती

की ऐवतहावसक दरगाह क एक नया रप दन की

तयारी की जा रही ह

bull इस 13िी ो शताबदी की दरगाह क ldquoसवचछ

आइकॉवनक थथल ोrdquo (Swacch Iconic Places) म

शावमल वकया गया ह ज परवतवषठत विरासत

आधयासिक और साोसकवतक थथान ो पर क वदरत

य जना ह

FC19H1003 41

ANSWERS amp EXPLANATION OF

NCERT History Class VI-X + Current Affairs

(FC19E1003)

Q1) Answer c

Explanation

Rigveda consists of more than a

thousand hymns dedicated to gods and

goddesses These hymns were

composed by sages and learnt by men

however a few were composed by

women like Apala Ghosa Lopamudra

Maitreyi and Gargi

Rigveda consists of many hymns in the

form of dialogues We get an example of

a dialogue between a sage named

Vishwamitra and two rivers (Beas and

Sutlej) that were worshipped as

goddesses This suggests that he

belonged to the Vedic period

Q2) Answer b

Explanation

Traces of ash have been found from

Kurnool Caves suggesting that people

were familiar with the use of fire

It is situated in Andhra Pradesh

Q3) Answer c

Explanation

Burzahom is a prehistoric site in

present day Kashmir where people built

pit houses which were dug into the

ground with steps leading into them

These may have provided shelter in cold

weather

Q4) Answer c

Explanation

Epigraphy is defined as the study of

inscriptions

Manuscriptology is the study of history

and literature through the use of hand

written documents

Palaeography refers to the study of

ancient writing systems and the

deciphering and dating of historical

manuscripts

Numismatics refers to the study of

coins

Q5) Answer a

Explanation

Charaka Samhita was written by

Charaka and is an important book on

Ayurveda and medicine

He was a practitioner of the traditional

system of Indian medicine known as

Ayurveda

Charaka is thought to have flourished

sometime between the 2nd century BCE

and the 2nd century CE

Q6) Answer b

Explanation

Bhaga refers to the tax on crops which

was fixed at 16th of the production

Kammakaras is the term used for the

landless agricultural labour class

Ashvamedha also known as horse

sacrifice is a ritual where a horse is let

loose to wander freely and it was

guarded by the rajarsquos men

Q7) Answer (d)

Explanation

In the Rigvedic period horses were

yoked to chariots that were used in

battles fought to capture land cattle

etc This suggests that the use of horse

chariots began much before the period

of Mahajanapadas

The battles were fought in the Rigvedic

period for cattlersquos lands water an even

to capture people Most men took part

in these wars however there was no

regular army but there were assemblies

where people met and discussed

matters of war Regular armies became

a feature in the Mjahajanapada period

including vast armies of foot soldiers

chariots and elephants

RAUSIAS-FC19E1003 42

Q8) Answer (a)

Explanation

Buddha belonged to the Sakya clan and

passed away at Kusinara

Buddha taught in Prakrit which was the

common language of people

Q9) Answer c

Explanation

There were six schools of philosophy in

ancient India These are known as

Vaishesika Nyaya Samkhya Yoga

Purva Mimansa and Vedanata or Uttara

Mimansa They were founded by sages

Kanada Gautama Kapila Patanjali

Jamini and Vyasa respectively

Q10) Answer b

Explanation

The teachings of Mahavira were

compiled at Valabhi in 6th century AD

Q11) Answer (c)

Explanation

Chanakya is traditionally identified as

Kautilya or Vishnugupta who authored

the ancient Indian political treatise the

Arthashastra

Q12) Answer d

The national emblem of India is an

adaptation of the Lion Capital atop the

Ashoka Pillar of Sarnath Uttar Pradesh

and is combined with the National

Motto Satyameva Jayate

The Rampurva Bull gets the name from

the site of its discovery Rampurva in

Bihar

It is noted for its delicately sculpted

model demonstrating superior

representation of soft flesh sensitive

nostrils alert ears and strong legs It is

a mixture of Indian and Persian

elements

Sankissa is situated in Uttar Pradesh

India

Q13) Ans(a)

Kunwar Singh was a notable leader during the Revolt of 1857 He belonged

to a royal house of Jagdispur Bihar

Q14) Answer b

Explanation

The term Vellalar was used for large

landowners

Q15) Answer c

Explanation

Arikamedu was a coastal settlement

where ships unloaded goods from

distant lands Finds here include a

massive brick warehouse pottery

including amphorae and Arretine ware

Roman lamps glassware and gems have

also been found at the site

Q16) Answer a

Explanation

Muvendar is a Tamil word mentioned in

Sangam poems meaning three chiefs

used for the heads of three ruling

families the Cholas Cheras and

Pandyas

Q17) Ans (c)

Several tribal or kin-based assemblies

such as the Sabha Vidatha and gana

are mentioned in the Rig-veda The

Sabha and the samiti mattered a great

deal in early Vedic times so much so

that the chiefs or the kings showed an

eagerness to win their support

Q18) Ans (a)

Jainism recognised the existence of the

gods but placed them lower than the

jina and did not condemn the varna

system as Buddhism did

Q19) Answer (d)

Explanation

Cholas and Pandyas had developed

powerful coastal cities The most

important city of Cholas was Puhar or

Kaveripattinam and Madurai was the

capital of Pandyas

Q20) Answer b

Explanation

Buddhacharita is the biography of

Buddha and was written by

RAUSIAS-FC19E1003 43

Ashvaghosha

Q21) Answer (a)

Explanation

Tamil poet Appar was a Shiva devotee

So he was a Nayanar saint

Q22) Answer d

Explanation

Samudragupta was a prominent Gupta

ruler whose coins depict him playing a

veena indicating his love for music We

get important historic information from

his Allahabad Prashasti which was

composed by his court poet Harisena

Q23) Answer (b)

Explanation

Vikrama Samvat was founded by

Chandragupta II in the 58 BC as a

mark of victory over the Shakas and

assumed the title of Vikramaditya

Banabhatta wrote Harshavardhanarsquos

biography the Harshacharita in

Sanskrit

Q24) Answer c

Explanation

Sandhi-vigrahika was the minister of

war and peace

Sarthavaha was the leader of the

merchant caravans

Q25) Answer a

Explanation

Xuan Zang (Hsuan-tsang) was a

Chinese traveller who came during the

reign of Harshavardhana

In the decade that began in 630 AD

Xuan Zang came to India through

Kashmir after visiting Central Asia Iran

and Afghanistan

He travelled from north to east and lived

in Bihar for a couple of years

At Nalanda University Xuan Zang

interacted with students and scholars

mastered local languages and

discovered Buddhist stupas

Q26) Answer c

Explanation

Pradakshina patha is a circular path

laid around a stupa in Buddhist

architecture While the rest are a part of

temple architecture

Q27) Answer d

Explanation

All the above-mentioned temples have

an elaborate use of bricks (baked

bricks) along with stone

Q28) Ans (c)

Muhammad Quli Qutab was the Sultan

of Golconda He was a contemporary of

Akbar was very fond of literature and

architecture

The Sultan was a great poet and he

wrote in Dakhini Urdu Persian and

Telgu and has left an extensive diwan or

collection

Recently the Archaeological Survey of

India (ASI) will be using Ground

Penetrating Radar (GPR) to map the

contours of the area around the Bagh-e-

Naya Qila excavated garden inside the

Golconda Fort in Telangana

Q29) Answer a

Explanation

Silappadikaram is a famous Tamil epic

which was written by Ilango around

1800 years ago It is a story of a

merchant named Kovalan who fell in

love with a courtesan named Madhavi

Manimekalai tells the story of the

daughter of Kovalan and Madhavi

Q30) Answer (a)

Explanation

Charaka is the author of Charaka

Samhita which is an important work of

Ayurveda and medicines

Brahmaguptarsquos fame rests mostly on his

Brahma-sphuta-siddhanta which was

an astronomical work It was translated

into Arabic in Baghdad and had a major

impact on Islamic mathematics and

astronomy

Late in his life Brahmagupta wrote

Khandakhadyaka which was an

RAUSIAS-FC19E1003 44

astronomical handbook that employed

Aryabhatarsquos system of starting each day

at midnight

Q31) Answer (c)

Explanation

Amir Khusrau was a famous sufi

musician poet and scholar In 1318 he

noted that there was different language

in every region of this land (Hindustan)

Lahori Kashmiri Dvarsamudri (in

Southern Karnataka) Telangana (in

Andhra Pradesh) Gujari (in Gujarat)

Marsquobari (in Tamil Nadu) Awadhi (in

eastern Uttar Pradesh) and Hindawai (in

the area around in Delhi) etc He went

to explain that Sanskrit did not belong

to any region and that only brahmans

knew it

Q32) Answer c

Explanation

Hiranyagarbha refers to the golden

womb When this ritual was performed

with the help of Brahmanas it was

thought to lead to the rebirth of the

sacrificer as a Khastriya

Q33) Answer d

Explanation

Kadamai refers to a tax on land

revenue

Gwalior Prashasti describes the exploits

of Nagabhata who was a Pratihara king

Q34) Answer b

Explanation

Rajatarangini is a Sanskrit text written

by Kalhana in the 12th century

It was historical chronicle of early India

It is justifiably considered to be the best

and most authentic work of its kind

It covers the entire span of history in

the Kashmir region from the earliest

times to the date of its composition

Q35) Answer c

Explanation

ldquoUrrdquo was the general assembly of the

village ldquoUrrdquo consisted of all the

taxpaying residents of an ordinary

village

Q36) Answer (a)

Explanation

Tarikh was a form of history writing in

the Delhi Sultanate The authors of

tawarikhs were learned men which

included secretaries administrators etc

Q37 Answer (a)

Explanation

Alauddin chose to pay his soldiers salaries in cash rather than iqtas The soldiers would buy their supplies from merchants in Delhi and it was thus feared that merchants would raise their prices To stop this Alauddin controlled the prices of goods in Delhi Prices were carefully surveyed by officers and merchants who did not sell at the prescribed rates were punished

Q38) Answer (d)

Explanation

Delhi first became the capital of a

kingdom under the Tomara Rajputs

who were defeated in the middle of the

twelfth century by the Chauhans (also

referred to as Chahamanas) of Ajmer

It was under the Tomaras and

Chauhans that Delhi became an

important commercial centre Many rich

Jaina merchants lived in the city and

constructed several temples Coins

minted here called dehliwal had a wide

circulation

Q39) Answer (c)

Explanation

Moth ki Masjid was built in the reign of

Sikandar Lodi by his minister

Begumpuri mosque built in the reign of

Muhammad Tughluq was the main

mosque of Jahanpanah the ldquoSanctuary

of the Worldrdquo and his new capital in

Delhi

Quwwat al ndash Islam mosque was

enlarged by Iltutmish and Alauddin

Khalji The minar was built by three

Sultansndash Qutbuddin Aybak Iltutmish

and Firuz Shah Tughluq

RAUSIAS-FC19E1003 45

Q40) Answer (c)

Explanation

Under the Mughals mansabdar was

referred to an individual who held a

mansab ie rank and he received his

salary as revenue assignments called

jagirs

Q41) Ans (b)

The Quit India Movement was a

spontaneous revolt of people against

British rule

The All India Congress Committee met

at Bombay on 8 August 1942 It passed

the famous resolution Quit India and

proposed the starting of a non-violent

mass struggle under Gandhis

leadership to achieve this aim But on

the very next day Gandhi and other

eminent leaders of the Congress were

arrested The Congress was once again

declared illegal

Q42) Ans (c)

The Simon Commission refers to a

group of seven MPs from the United

Kingdom constituted to suggest

constitutional reforms for British India

The Commission consisted of only

British members headed by one of the

senior British politicians Sir John

Simon

So the people of India agitated against

the arrival of Simon Commission

Q43) Ans (a)

He was widely known for his

unfavourable opinion of the economic

consequences of the British rule in

India

In his many writings and speeches and

especially in Poverty and Un-British

Rule in India Naoroji argued that India

was too highly taxed and that its wealth

was being drained away to England

He did not interpret the ancient Indian

texts and restored the self-confidence of

Indians And also he did not stress the

need for eradication of all the social

evils before anything else

Q44) Ans (c)

In August 1932 Prime Minister

MacDonald announced his Communal

Award Great Britainrsquos unilateral

attempt to resolve the various conflicts

among Indiarsquos many communal

interests

The award which was later

incorporated into the act of 1935

expanded the separate-electorate

formula reserved for Muslims to other

minorities including Sikhs Indian

Christians Anglo-Indians Europeans

distinct regional groups Gandhi

undertook a ldquofast unto deathrdquo against

that offer which he viewed as a

nefarious British plot to divide the

Indian society

Q45) Ans (b)

In British India apart from existing

imports and exports there was also a

particular amount of money which

colonial India contributed towards

administration maintenance of the

army war expenses pensions to retired

officers and other expenses accrued by

Britain towards maintenance of her

colony These were known as Home

charges and were paid for almost

entirely by India

The Home charges was made of

following components-

- Interest payable on Indian debt

- Dividend to shareholders of East

India Company

- Funds used to support the India

Office in London

- Funds used to pay salaries and

pensions of British personnel

engaged in India

- Interest on the railways

- Civil and military charges

- Store purchases in England

Q46) Ans (b)

The Lahore session of the Indian

National Congress was held in 1929

under the Presidentship of Jawaharlal

Nehru

The Lahore session of the Indian

National Congress witnessed significant

RAUSIAS-FC19E1003 46

developments in the Indian national

movement

- First the election of Jawaharlal

Nehru to the post of Presidentship of

the Congress was a clear indication

of the growing strength of the

Leftists in the Congress

- Secondly it was in this session that

the Congress for the first time raised

the demand for complete

independence Such demand was

not raised from the Congress

platform earlier

Q47) Ans (b)

It did not provide for separate

electorates for any community or

weightage for minorities However it did

allow for the reservation of minority

seats in provinces having minorities of

at least ten per cent but this was to be

in strict proportion to the size of the

community

There was no provision for complete

Independence for India

Q48) Ans (c)

The religion of early Vedic Aryans was

primarily of worship of nature and

Yajnas

The early Aryan religion was kind of

nature worship Actually the forces

around them which they could not

control or understand were invested

with divinity and were personified as

male or female gods And they

performed some Yajnas also

Q49) Ans (b)

The roads and river-routes were not

immune from robbery It is notable that

Yuan Chwang (Hiuen Tsang) was

robbed of his belongings during

Harshvardanarsquos period

Q50) Ans (c)

Q51) Ans (b)

Purandara Dasa was a saint and great

devotee of Lord Krishna

There is much speculation about where

Purandara Dasa regarded as the

Pitamaha of Carnatic music was born

Recently an expert committee

constituted by the Kannada University

Hampi has come to the conclusion that

Kshemapura Shivamogga district

Karnataka is the birth place of

Purandara Dasa

Q52) Ans (c)

Sri Tyagaraja Sri Shyama Shastry and Sri Muthuswami Dikshitar are considered the trinity of Carnatic music and with them came the golden age in Carnatic music in the 18th-19th

century

Q53) Ans d)

Recently a rare sarcophagus (stone

coffin) which is 2000 years old from the

Iron AgendashMegalithic era was discovered

from a rock-cut cave at Viyur village of

Kollam near Koyilandy in Kozhikode

district Kerala

The coffin containing bone fragments

was found during an excavation ldquoSo

far such a rare finding has been

discovered only from two sites

in Kerala Both these sarcophagi were

recovered from Megalithic sites at

Chevayur and Atholi also in Kozhikode

district

Q54) Ans a)

The megalithic culture in South India was a full-fledged Iron Age culture

Q55) Ans d)

The Cholas Pandyas and Keralaputras

(Cheras) mentioned in Ashokan

inscriptions were probably in the late

megalithic phase of material culture

Q56) Ans d)

Q57) Ans (b)

Raj Kumar Shukla followed Gandhiji all

over the country to persuade him to

come to Champaran to investigate the

problem associated with tinkathia

system

RAUSIAS-FC19E1003 47

Brij Kishore Rajendra Prasad Mahadev

Desai and Narhari Parikh accompanied

Gandhi ji during the Champaran

Satyagraha

Q58) Ans (b)

The Satvahanas started the practice of granting tax-free villages to brahmanas and Buddhist monks

Q59) Ans c)

The objectives of the Programme are

listed as under

- Developing basic tourism

infrastructure

- Promoting cultural and heritage

value of the country to generate

livelihoods in the identified regions

- Enhancing the tourist attractiveness

in a sustainable manner by

developing world-class

infrastructure at the heritage

monument sites

- Creating employment through active

involvement of local communities

- Harnessing tourism potential for its

effects on employment generation

and economic development

- Developing sustainable tourism

infrastructure and ensuring proper

Operations and maintenance

therein

Q60) Ans (b)

The Tribal Cooperative Marketing

Development Federation of India

(TRIFED) came into existence in 1987

It is a national-level apex organization

functioning under the administrative

control of Ministry of Tribal Affairs

Govt of India

TRIFED has its registered and Head

Office located in New Delhi

Q61) Ans (c)

Premchandrsquos novels include

Premashram Rangabhumi Ghaban

Karmabhumi and Godan

Gora is a novel written by Rabindranath

Tagore

138th birth anniversary of Munshi

Premchand was celebrated across the

country

Q62) Ans (b)

Giddha is a traditional pastoral dance

performed by the women of the Punjab

India and Pakistan at festival times

and at the sowing and reaping of the

harvest

By this dance the Punjabi women

reveal their joy expel their suppressed

feelings in a male dominated society

through the performance of Giddha

Since this dance has nothing to do with

men only women can participate in it

During the Teej celebrations Giddha

dance is celebrated in Punjab every

year Teej is a generic name for a

number of festivals that are celebrated

by women in some parts of India

Q63) Ans (a)

Dara Shukoh wrote the remarkable

work called ldquoMajma-ul-Bahrainrdquo or the

ldquoThe confluence of two seasrdquo

The Vice President of India Shri M

Venkaiah Naidu has said that Prince

Dara Shukohrsquos writings can come as a

refreshing source for infusing peace and

harmony He was addressing the

gathering after visiting the exhibition

that showcases the forgotten Prince of

yesteryears Dara Shukoh organized by

Mr Francois Gautier at Indira Gandhi

National Centre for the Arts in New

Delhi

Q64) Ans (c)

The statue Gommateshwara is

dedicated to the Jain God Bahubali

It is a monolithic statue

President Ram Nath Kovind

inaugurated the grand anointing

ceremony mdash Mahamastakabhisheka mdash

held once in 12 years at

Shravanabelagola (Karnataka)

Q65) Ans (c)

Prachi Valley had come up around the

Prachi river Prachi Valley gradually

disappeared

RAUSIAS-FC19E1003 48

The Prachi river originates from

Bhubaneswar

It is a tributary of the Mahanadi and

flows through the districts of Puri

Khurda Cuttack and Jagatsinghpur

and the entire region of the river is

termed as the Prachi Valley

It falls into the Bay of Bengal

Archaeological evidence shows that the

Prachi Valley Civilisation predates both

Harappa and Mohenjo-Daro

The Prachi river originates from

Bhubaneswar

Q66) Ans (d)

These monuments are located in

Chhatarpur district Madhya Pradesh

within Vindhya mountain range

Q67) Ans (a)

The book lsquoThoughts on Pakistanrsquo was

written by Dr BR Ambedkar

On the occasion of the birth anniversary

of Dr BR Ambedkar the president of

India pays homage to this icon of India

In 1924 he founded the Depressed

Classes Institute (Bahishkrit Hitkarini

Sabha) and in 1927 the Samaj Samata

Sangh

Another area of attention for Ambedkar

was education For its spread among

the low classes he set up a network of

colleges by the name of Peoples

Education Society and founded hostels

Q68) Ans(b)

Mehrgarh is a famous Neolithic

settlement in the Indian subcontinent

which is situated in Baluchistan

province Pakistan

A pre-historic rock art site is discovered

in the vast expanse of limestone blocks

on the eastern banks of Naguleru river

near Dachepalli (Andhra Pradesh) It

has thrown light on the Neolithic

civilisation that flourished in Guntur

(Andhra Pradesh) during 1500-2000

BC

Q69) Ans (c)

The 12th and the 13th centuries saw

the emergence of the Kakatiyas They

were at first the feudatories of the

Western Chalukyas of Kalyana Initially

they ruled over a small territory near

Warangal (Telangana)

They introduced Nayakships which was

later adopted and developed by the

Rayas of Vijayanagara

Q70) Ans (a)

The fast had effect of putting pressure

on mill owners who finally agreed to

give the workers a 35 per cent increase

in wages

Google celebrated with a doodle the

132nd birth anniversary of Anasuya

Sarabhai who played a pioneering role

in Indiarsquos labour movement

Q71) Ans (d)

The UNESCOrsquos list of the representative

list of the intangible cultural heritage of

humanity from India are

- Koodiyattam Sanskrit Theatre of

Kerala

- Mudiyettu ritual theatre and dance

drama of Kerala

- Tradition of Vedic Chanting

- Kalbelia folk songs and dances of

Rajasthan

- Ramlila Traditional Performance of

the Ramayana

- Sankirtana ritual singing

drumming and dancing of Manipur

- Ramman religious festival and

ritual theatre of the Garhwal

Himalayas India

- Traditional brass and copper craft of

utensil making among the Thatheras

of Jandiala Guru Punjab India

- Chhau dance classical Indian dance

originated in the eastern Indian

states

- Buddhist chanting of Ladakh

recitation of sacred Buddhist texts

in the trans-Himalayan Ladakh

region Jammu and Kashmir India

- Yoga

- Nouroz

- Kumbh Mela

RAUSIAS-FC19E1003 49

Q72) Ans(b)

The President of India Shri Ram Nath Kovind inaugurated the Hornbill Festival and State Formation Day celebrations of Nagaland in Kisama

The festival is named after the Indian hornbill the large and colourful forest bird which is displayed in the folklore of most of the states tribes

The major recognized tribes of Nagaland are Angami Ao Chakhesang Chang

Kuki Rengma and Zeling etc

Onge Jarawa and Sentinelese are the

tribes of Andman amp Nicobar Islands

Q73) Ans (c)

The Rashtrakutas rule in the Deccan lasted for almost two hundred years till the end of the tenth century The Rashtrakutas rulers were tolerant in their religious views and patronized not only Shaivism and Vaishnavism but

Jainism as well

The famous rock-cut temple of Shiva at Ellora was built by one of the Rashtrakutas kings Krishna I in the ninth century His successor Amoghavarsha was a Jain but he also

patronized other faiths

The Rashtrakutas allowed Muslims traders to settle and permitted Islam to

be preached in their dominions

Recently increasing defacement at the prehistoric rock paintings of Pandavulagutta Telangana has created a cause for grave concern It can spoil

the prehistoric rock

Pandavulagutta is home to

- Painted rock shelters dating to

10000 BC-8000 BC

- An 8th century inscription of the

Rashtrakuta period and

- Painted frescoes from the 12th century Kakatiya empire

Q74) Ans (b)

In 1828 Raja Ram Mohan Roy founded a new religious society the Brahma Sabha later known as the Brahmo

Samaj

Debendranath Tagore headed the Tattvabodhini Sabha which was

engaged in search of spiritual truth

Its purpose was to purify Hinduism and to preach monotheism or belief in one God

The new society was to be based on the twin pillars of reason and the Vedas and

Upanishads

Recently Sadharan Brahmo Samaj (SBS) has entered into a legal battle with the West Bengal government due

to some legal issue

Q75) Ans (c)

The Chishti order was established in India by Khwaja Moinuddin Chishti who came to India around 1192 The Chishtirsquos are considered to be the most influential of the groups of Sufis who migrated to India in the late twelfth century They adapted successfully to the local environment and adopted several features of Indian devotional

traditions

The historical dargah of Sufi mystic Khwaja Moinuddin Chishti in Ajmer is all set to get a facelift This 13 th century dargah has been included among the Swachh Iconic Places a clean-up initiative focused on iconic

heritage spiritual and cultural places

Page 14: GENERAL STUDIES (PAPER I) · Test is part of Rau’s IAS Test series for Preliminary Exam 2019 FOUNDATION + CURRENT AFFAIRS GENERAL STUDIES (PAPER –I) FOUNDATION TEST –III TOPIC:

RAUSIAS-FC19E1003 14

Q38) निमननिखित कथि ो म स कौि-सास सही हह

1 नदलली कतबददीि एबक क अरीि पहिी बार

नकसी सामराजय की राजरािी बिी थी

2 दहिीिाि नसक ो का मदरर मग़ि ो क दवारा

नकया गया था

िीच नदए गए कट का परय ग कर सही उततर चनिए

(a) किि 1

(b) किि 2

(c) 1 और 2 द ि ो

(d) ि त 1 ि ही 2

Q39) निमननिखित यग ो पर निचार कीनजए

1 म ठ की मखिद नसको दर ि दी

2 बगमपरी मखिद नफर ज शाह तगिक

3 कववत- अि - इसलाम कतबददीि ऐबक

उपयणकत यग ो म स कौि-स सही समनित ह

(a) किि 1 और 2

(b) किि 2 और 3

(c) किि 1 और 3

(d) 1 2 और 3

Q40) निमननिखित कथि ो म स कौि-सास सही हह

1 मिसबदार ो क अपिा िति राजसव कायो

नजन जागीर कहत थ क रप म परापत ह ता

था

2 मिसबदार क ज सनय उततरदानयतव सौोप जात

थ उसक अनतगणत उस एक निराणररत सखया म

सिार अथिा घड़सिार ो का रि-रिाि करिा

पड़ता था

िीच नदए गए कट का परय ग कर सही उततर चनिए

(a) किि 1

(b) किि 2

(c) 1 और 2 द ि ो

(d) ि त 1 ि ही 2

Q41) ldquo1942 क भारत छ ड़ आोद ििrdquo क बार म

निमननिखित अिि कि ो म स कौि-सा सतय िही ो ह

(a) यह एक अनहोसक आोद िि था

(b) इसका िततव महातमा गाोरी क दवारा नकया गया

था

(c) यह एक सवाभानिक आोद िि था

(d) इसि सामानयतया शरनमक िगण क आकनिणत

िही ो नकया था

Q42) भारत क ि ग ो ि ldquoसाइमि कमीशिrdquo क आगमि क

निरदध आोद िि नकया था कय ोनक

(a) भारतीय कभी भी 1919 क अनरनियम (The

Act of 1919) क काम की समीकषा िही ो करिा

चाहत थ

(b) साइमि कमीशि ि पराोत ो म दवर (द हर) शासि

क समापत करि की नसफाररश की थी

(c) साइमि कमीशि म क ई भारतीय सदसय िही ो

था

(d) साइमि कमीशि ि दश क निभाजि का

सझाि नदया था

Q43) निमननिखित कथि ो पर निचार कीनजए

भारतीय राषटर ीय आोद िि म दादाभाई िौर जी क दवारा

नकया गया सबस परभािी य गदाि यह था नक उन ोि

1 अोगरज ो क दवारा भारत क आनथणक श िर का

ििासा नकया था

2 पराचीि भारतीय गरोथ ो की वयाखया की थी और

भारतीय ो क आतमनिशवास क पिःसथानपत नकया

था

3 अनय नकसी भी बात स पहि सभी सामानजक

बराइय ो क उनमिि की आिशयकता पर बि

नदया था

उपयणकत कथि ो म स कौि-सास सही हह

(a) किि 1

(b) किि 2 और 3

(c) किि 1 और 3

(d) 1 2 और 3

RAUSIAS-FC19E1003 15

Q38) Which of the following statements

isare correct

1 Delhi first became the capital of a

kingdom under Qutubuddin

Aibak

2 Dehliwal coins were minted by the

Mughals

Select the correct answer using the code

given below

(a) 1 only

(b) 2 only

(c) Both 1 and 2

(d) Neither 1 nor 2

Q39) Consider the following pairs

1 Moth ki Masjid- Sikander Lodi

2 Begumpuri mosque- Firuz Shah

Tughluq

3 Quwwat al ndash Islam- Qutubuddin

Aibak

Which of the above pairs isare correct

(a) 1 and 2 only

(b) 2 and 3 only

(c) 1 and 3 only

(d) 1 2 and 3

Q40) Which of the following statements

isare correct

1 Mansabdars received their salaries

as revenue assignments called

jagirs

2 The mansabdarrsquos military

responsibilities required him to

maintain a specified number of

sawar or cavalrymen

Select the correct answer using the code

given below

(a) 1 only

(b) 2 only

(c) Both 1 and 2

(d) Neither 1 nor 2

Q41) Which one of the following observations

is not true about the Quit India

Movement of 1942

(a) It was a non-violent movement

(b) It was led by Mahatma Gandhi

(c) It was a spontaneous movement

(d) It did not attract the labour class

in general

Q42) The people of India agitated against the

arrival of the Simon Commission

because

(a) Indians never wanted the review of

the working of the Act of 1919

(b) Simon Commission recommended

the abolition of dyarchy in the

Provinces

(c) there was no Indian member in the

Simon Commission

(d) the Simon Commission suggested

the partition of the country

Q43) Consider the following statements

The most effective contribution made by

Dadabhai Naoroji to the cause of Indian

National Movement was that he-

1 exposed the economic exploitation

of India by the British

2 interpreted the ancient Indian

texts and restored the self-

confidence of Indians

3 stressed the need for eradication of

all the social evils before anything

else

Which of the statements given above

isare correct

(a) 1 only

(b) 2 and 3 only

(c) 1 and 3 only

(d) 1 2 and 3

RAUSIAS-FC19E1003 16

Q44) महातमा गाोरी ि 1932 म आमरर अिशि नकया था

कय ोनक

(a) ldquoग िमज सममििrdquo (The Round Table

Conference) भारतीय राजिीनतक

आकाोकषाओो क परा करि म असफि रहा था

(b) काोगरस और मखसलम िीग म मतभद थ

(c) रामस मकड िालड (Ramsay Macdonald)

ि ldquoसाोपरदानयक परसकारrdquo (The Communal

Award) की घ िरा की थी

(d) ldquoसनििय अिजञा आोद ििrdquo (The Civil

Disobedience Movement) असफि रहा

था

Q45) भारत म औपनििनशक शासि की अिनर क सोदभण म

भारत स रि क बनहगणमि का एक महतवपरण भाग गह

शलक (Home Charges) था निमननिखित म स

कौि-सास क ि गह शलक म सखममनित नकया गया

थानकय गए थ

1 िोदि म भारत कायाणिय क निए उपय ग नकय

जाि िािा क ि

2 भारत म नियकत नबरनटश कनमणय ो क िति और

पशि का भगताि करि क निए उपय ग नकय

जाि िािा क ि

3 अोगरज ो क दवारा भारत क बाहर यदध ो क निए

उपय ग नकय जाि िािा क ि

िीच नदए गए कट का परय ग कर सही उततर चनिए

(a) किि 1

(b) किि 1 और 2

(c) किि 2 और 3

(d) 1 2 और 3

Q46) सवतोतरता आोद िि क इनतहास म भारतीय राषटर ीय

काोगरस का 1929 का सतर महतवपरण ह कय ोनक इसम

(a) काोगरस क उददशय क रप म सथािीय सरकार

की पराखपत की घ िरा की गई थी

(b) परण सवराज की पराखपत क काोगरस क िकषय क

रप म अपिाया गया था

(c) असहय ग आोद िि शर नकया गया था

(d) िोदि म ldquoग ि मर सममििrdquo (The Round

Table Conference) म भाग िि का निरणय

निया गया था

Q47) भारतीय सवतोतरता सोगराम क सोदभण म िहर ररप टण

क दवारा निमननिखित म स नकसकी नसफाररश की गई

थीनकिकी नसफाररश की गई थी ो

1 भारत क निए परण सवतोतरता

2 अलपसोखयक ो क निए सीट ो क आरकषर क

निए सोयकत नििाणचक मोडि

3 सोनिराि म भारत क ि ग ो क निए मौनिक

अनरकार ो का परािराि

िीच नदए गए कट का परय ग कर सही उततर चनिए

(a) किि 1

(b) किि 2 और 3

(c) किि 1 और 3

(d) 1 2 और 3

Q48) आरो नभक िनदक आयो का रमण मखय रप स था

(a) भखकत

(b) मनतण पजा और यजञ

(c) परकनत की पजा और यजञ

(d) परकनत की पजा और भखकत

RAUSIAS-FC19E1003 17

Q44) Mahatma Gandhi undertook fast unto

death in 1932 mainly because

(a) The Round Table Conference failed

to satisfy Indian political

aspirations

(b) The Congress and Muslim League

had differences of opinion

(c) Ramsay Macdonald announced the

Communal Award

(d) The Civil Disobedience Movement

failed

Q45) With reference to the period of colonial

rule in India ldquoHome Chargesrdquo formed

an important part of drain of wealth

from India Which of the following funds

constituted ldquoHome Chargesrdquo

1 Funds used to support the India

Office in London

2 Funds used to pay salaries and

pensions of British personnel

engaged in India

3 Funds used for waging wars

outside India by the British

Select the correct answer using the code

given below

(a) 1 only

(b) 1 and 2 only

(c) 2 and 3 only

(d) 1 2 and 3

Q46) The 1929- Session of Indian National

Congress is of significance in the history

of the Freedom Movement because the-

(a) attainment of Self-Government

was declared as the objective of

the Congress

(b) attainment of Poorna Swaraj was

adopted as the goal of the

Congress

(c) Non-Cooperation Movement was

launched

(d) decision to participate in the

Round Table Conference in

London was taken

Q47) With reference to the period of Indian

freedom struggle which of the following

waswere recommended by the Nehru

report

1 Complete Independence for India

2 Joint electorates for reservation of

seats for minorities

3 Provision of fundamental rights for

the people of India in the

Constitution

Select the correct answer using the code

given below

(a) 1 only

(b) 2 and 3 only

(c) 1 and 3 only

(d) 1 2 and 3

Q48) The religion of the early Vedic Aryans was primarily of

(a) Bhakti

(b) image worship and Yajnas

(c) worship of nature and Yajnas

(d) worship of nature and Bhakti

RAUSIAS-FC19E1003 18

Q49) भारत की यातरा करि िाि चीिी यातरी यआि चिाोग

(हयएि साोग) ि समकािीि भारत की सामानय

खसथनतय ो और सोसकनत क दजण नकया था इस सोदभण म

निमननिखित कथि ो म स कौि-सास सही हह

1 सड़क और िदी-मागण (जि-मागण) डकती स

परण रप स सरनकषत थ

2 जहा तक अपरार ो क निए दणड की बात ह

उसक निए नकसी भी वयखकत की निदोिता

अथिा उसक अपरार क निराणररत करि क

निए अनि जि और निि परि क माधयम क

सारि थ

3 वयापाररय ो क घाट ो और परनतबोर सटशि ो पर

शलक ो का भगताि करिा पड़ता था

िीच नदए गए कट का परय ग कर सही उततर चनिए

(a) किि 1

(b) किि 2 और 3

(c) किि 1 और 3

(d) 1 2 और 3

Q50) नसोर घाटी सभयता क सोदभण म निमननिखित कथि ो पर

निचार कीनजए

1 यह मखय रप स एक रमणनिरपकष सभयता थी

तथा हािाोनक इसम रानमणक ततव मौजद था

िनकि िह परनतिश पर हािी िही ो था

2 इस काि क दौराि भारत म कपास का परय ग

कपड़ा बिाि क निए नकया जाता था

उपयणकत कथि ो म स कौि-सास सही हह

(a) किि 1

(b) किि 2

(c) 1 और 2 द ि ो

(d) ि त 1 ि ही 2

Q51) परोदर दास क सोदभण म निमननिखित कथि ो पर निचार

कीनजए

1 परोदर दास एक सोत और भगिाि नशि क एक

महाि भकत थ

2 ि एक सोगीतकार गायक और किाणटक सोगीत

क मखय सोसथापक-परसतािक ो म स एक थ

उपयणकत कथि ो म स कौि-सास सही हह

(a) किि 1

(b) किि 2

(c) 1 और 2 द ि ो

(d) ि त 1 ि ही 2

Q52) निमननिखित म स कौि-सास वयखकत किाणटक सोगीत

की नतरमनतण म शानमि हह

1 बािामरिी कषणा

2 शरी शयाम शासतरी

3 शरी मथसवामी दीनकषतर

िीच नदए गए कट का परय ग कर सही उततर चनिए

(a) किि 1

(b) किि 2

(c) किि 2 और 3

(d) 1 2 और 3

Q53) चियर (Chevayur) और अथ िी (Atholi) म खसथत

महापािार सथि निमननिखित म स नकस राजय म खसथत

(a) तनमििाड

(b) किाणटक

(c) पनिम बोगाि

(d) करि

RAUSIAS-FC19E1003 19

Q49) The Chinese traveller Yuan Chwang

(Hiuen Tsang) who visited India

recorded the general conditions and

culture of India at that time In this

context which of the following

statements isare correct

1 The roads and river-routes were

completely immune from robbery

2 As regards punishment for

offences ordeals by fire water and

poison were the instruments for

determining the innocence or guilt

of a person

3 The tradesmen had to pay duties

at ferries and barrier stations

Select the correct answer using the code

given below

(a) 1 only

(b) 2 and 3 only

(c) 1 and 3 only

(d) 1 2 and 3

Q50) Regarding the Indus Valley Civilization

consider the following statements

1 It was predominantly a secular

civilization and the religious

element though present did not

dominate the scene

2 During this period cotton was

used for manufacturing textiles in

India

Which of the statements given above

isare correct

(a) 1 only

(b) 2 only

(c) Both 1 and 2

(d) Neither 1 nor 2

Q51) Consider the following statements

regarding Purandara Dasa

1 Purandara Dasa was a saint and

great devotee of Lord Shiva

2 He was a composer singer and

one of the chief founding-

proponents of the Carnatic music

Which of the statements given above

isare correct

(a) 1 only

(b) 2 only

(c) Both 1 and 2

(d) Neither 1 nor 2

Q52) Which of the following persons isare

included in the trinity of Carnatic

music

1 Balamurali Krishna

2 Sri Shyama Shastry

3 Sri Muthuswami Dikshitar

Select the correct answer using the code

given below

(a) 1 only

(b) 2 only

(c) 2 and 3 only

(d) 1 2 and 3

Q53) Megalithic sites at Chevayur and Atholi

are located in which of the following

states

(a) Tamil Nadu

(b) Karnataka

(c) West Bengal

(d) Kerala

RAUSIAS-FC19E1003 20

Q54) निमननिखित कथि ो पर निचार कीनजए

1 महापािानरक ि ग कबर ो म िसतएो दफिात थ

2 दनकषर भारत म महापािार सोसकनत एक परण

निकनसत तामर यगीि सोसकनत थी

उपयणकत कथि ो म स कौि-सास सही हह

(a) किि 1

(b) किि 2

(c) 1 और 2 द ि ो

(d) ि त 1 ि ही 2

Q55) निमननिखित म स कौि-स सामराजयसामराजय ो का

अश क क अनभिि ो म उललि नकया गया ह

1 च ि

2 पाणडय

3 करिपतर (चर)

िीच नदए गए कट का परय ग कर सही उततर चनिए

(a) किि 1

(b) किि 1 और 2

(c) किि 3

(d) 1 2 और 3

Q56) भीमा-क रगाोि का यदध को पिी क सनिक ो और

बाजीराि नदवतीय क िततव म एक शखकतशािी पशिा

सिा (मराठ ो) क मधय िड़ा गया था यह यदध

निमननिखित म स नकसका नहससा था

(a) परथम आोगल-मराठा यदध का

(b) नदवतीय आोगल-मराठा यदध का

(c) ततीय आोगल-मसर यदध का

(d) ततीय आोगल-मराठा यदध का

Q57) निमननिखित कथि ो पर निचार कीनजए

1 महादि दसाई ि गाोरीजी क चोपारर आि तथा

नतिकनथया पररािी स जड़ी समसया की जाोच

क निए रारी करि क निए दश भर म उिका

अिसरर नकया था

2 िरहरी पाररि चोपारर सतयागरह क दौराि

गाोरीजी क साथ थ

उपयणकत कथि ो म स कौि-सास सही हह

(a) किि 1

(b) किि 2

(c) 1 और 2 द ि ो

(d) ि त 1 ि ही 2

Q58) निमननिखित कथि ो पर निचार कीनजए

1 िनद राज-िोश ि बराहमर ो और बौदध मठराररय ो

क कर-मकत गाि अिदाि म दि की परथा

आरि की थी

2 सतिाहि ो की आनरकाररक भािा पराकत थी

उपयणकत कथि ो म स कौि-सास सही हह

(a) किि 1

(b) किि 2

(c) 1 और 2 द ि ो

(d) ि त 1 ि ही 2

Q59) एक निरासत क अपिाइए (अडॉपट ए हररटज ndash

Adopt a Heritage) पररय जिा क उददशय ो क

सनदभण म निमननिखित कथि ो पर निचार कीनजए

1 यह पररय जिा र रगार उतपादि और आनथणक

निकास क निए पयणटि कषमता का उि पर

परभाि का उपय ग करगी

2 यह पररय जिा निरासत सथि ो पर निशव सतरीय

आराररक सोरचिा निकनसत करक एक सतत

तरीक स पयणटक आकिणर म िखदध करगी

उपयणकत कथि ो म स कौि-सास सही हह

(a) किि 1

(b) किि 2

(c) 1 और 2 द ि ो

(d) ि त 1 ि ही 2

RAUSIAS-FC19E1003 21

Q54) Consider the following statements

1 Megalithic people buried goods in

graves

2 The megalithic culture in South

India was a full-fledged Copper

Age culture

Which of the statements given above

isare correct

(a) 1 only

(b) 2 only

(c) Both 1 and 2

(d) Neither 1 nor 2

Q55) Which of the following kingdoms isare

mentioned in the Ashokan inscriptions

1 Cholas

2 Pandyas

3 Keralaputras (Cheras)

Select the correct answer using the code

given below

(a) 1 only

(b) 1 and 2 only

(c) 3 only

(d) 1 2 and 3

Q56) The Battle of Bhima-Koregaon was

fought between the soldiers of the

Company and the strong Peshwa army

(Marathas) under Bajirao II This war

was a part of the

(a) First Anglo-Maratha war

(b) Second Anglo-Maratha war

(c) Third Anglo- Mysore war

(d) Third Anglo-Maratha war

Q57) Consider the following statements

1 Mahadev Desai followed Gandhiji all over the country to persuade him to come to Champaran to investigate the problem associated

with tinkathia system

2 Narhari Parikh accompanied Gandhi ji during the Champaran

Satyagraha

Which of the statements given above isare correct

(a) 1 only

(b) 2 only

(c) Both 1 and 2

(d) Neither 1 nor 2

Q58) Consider the following statements

1 The Nanda Dynasty started the practice of granting tax-free villages to brahmanas and

Buddhist monks

2 The official language of the Satavahanas was Prakrit

Which of the statements given above

isare correct

(a) 1 only

(b) 2 only

(c) Both 1 and 2

(d) Neither 1 nor 2

Q59) Consider the following statements about the objectives of the lsquoadopt a heritagersquo

project

1 It will harness tourism potential for its effects on employment generation and economic

development

2 It will enhance the tourist attractiveness in a sustainable manner by developing world class infrastructure at heritage sites

Which of the statements given above

isare correct

(a) 1 only

(b) 2 only

(c) Both 1 and 2

(d) Neither 1 nor 2

RAUSIAS-FC19E1003 22

Q60) ldquoभारतीय जिजातीय सहकारी निपरि निकास सोघrdquo

(The Tribal Co-operative Marketing

Development Federation of India - TRIFED)

क सोदभण म निमननिखित कथि ो पर निचार कीनजए

1 यह एक राषटर ीय सतर का शीिण सोगठि ह ज

भारत सरकार क गह मोतरािय क परशासनिक

नियोतरर क अरीि काम कर रहा ह

2 इसका मखय उददशय दश म जिजातीय ि ग ो

का सामानजक-आनथणक निकास करिा ह

उपयणकत कथि ो म स कौि-सास सही हह

(a) किि 1

(b) किि 2

(c) 1 और 2 द ि ो

(d) ि त 1 ि ही 2

Q61) निमननिखित म स कौि-सास उपनयास परमचोद क

दवारा नििा गया हनिि गए ह

1 रोगभनम

2 ग दाि

3 ग रा

िीच नदए गए कट का परय ग कर सही उततर चनिए

(a) किि 1

(b) किि 2

(c) किि 1 और 2

(d) 1 2 और 3

Q62) नगदधा ितय क सोदभण म निमननिखित कथि ो पर निचार

कीनजए

1 नगदधा नबहार की मनहिाओो क दवारा तयौहार क

समय और फसि की बिाई तथा कटाई क

अिसर पर नकया जाि िािा एक पारोपररक

दहाती ितय ह

2 इस ितय क दवारा मनहिाऐो अपिी परसननता

परकट करती ह तथा नगदधा क परदशणि क

माधयम स परि िचणसव िाि समाज म

मनहिाओो की दबी हई भाििाओो क परकट

करती ह

उपयणकत कथि ो म स कौि-सास सही हह

(a) किि 1

(b) किि 2

(c) 1 और 2 द ि ो

(d) ि त 1 ि ही 2

Q63) निमननिखित कथि ो पर निचार कीनजए

1 मलला शाह बदखशी दारा नशक ह क

आधयाखतमक गर थ

2 औरोगरब ि मजम-उि-बहरीि या द समदर ो

का सोगम िामक उललििीय रचिा नििी थी

3 दारा नशक ह क अपि पिणज अकबर क गर ो

क उततरानरकारी क रप म दिा गया था

नजसम उसि रानमणक बहििाद और समनवयता

क बढ़ािा नदया था

उपयणकत कथि ो म स कौि-सास सही हह

(a) किि 1 और 3

(b) किि 2

(c) किि 1 और 2

(d) 1 2 और 3

RAUSIAS-FC19E1003 23

Q60) Consider the following statements about

the Tribal Cooperative Marketing

Development Federation of India

(TRIFED)

1 It is a national-level apex

organization functioning under the

administrative control of Ministry

of Home Affairs Government of

India

2 The main objective of TRIFED is

socio-economic development of

tribal people in the country

Which of the statements given above

isare correct

(a) 1 only

(b) 2 only

(c) Both 1 and 2

(d) Neither 1 nor 2

Q61) Which of the following novels isare

written by Premchand

1 Rangabhumi

2 Godan

3 Gora

Select the correct answer using the code

given below

(a) 1 only

(b) 2 only

(c) 1 and 2 only

(d) 1 2 and 3

Q62) Consider the following statements about

Giddha dance

1 Giddha is a traditional pastoral

dance performed by the women of

Bihar at festival times and at the

sowing and reaping of the harvest

2 By this dance the women reveal

their joy expel their suppressed

feelings in a male dominated

society through the performance of

Giddha

Which of the statements given above

isare correct

(a) 1 only

(b) 2 only

(c) Both 1 and 2

(d) Neither 1 nor 2

Q63) Consider the following statements

1 Mullah Shah Badakhshi was the

spiritual mentor of Dara Shukoh

2 Aurangzeb wrote the remarkable

work called ldquoMajma-ul-Bahrainrdquo or

the ldquoThe confluence of two seasrdquo

3 Dara Shukoh was seen as

inheriting the qualities of his

ancestor Akbar in that he

promoted religious pluralism and

syncretism

Which of the statements given above

isare correct

(a) 1 and 3 only

(b) 2 only

(c) 1 and 2 only

(d) 1 2 and 3

RAUSIAS-FC19E1003 24

Q64) निमननिखित कथि ो पर निचार कीनजए

1 ग मतशवर परनतमा निोधयनगरी पहाड़ी पर खसथत ह

2 शरिरबिग िा िह सथाि ह जहाो मौयण िोश क

सोसथापक चोदरगपत मौयण अपि नसोहासि क

तयागि क बाद जि तपसवी बि गए थ

उपयणकत कथि ो म स कौि-सास सही हह

(a) किि 1

(b) किि 2

(c) 1 और 2 द ि ो

(d) ि त 1 ि ही 2

Q65) निमननिखित कथि ो पर निचार कीनजए

1 पराताखतवक साकषय स पता चिता ह नक पराची

घाटी सभयता हड़पपा और म हिज दाड़ द ि ो

की पिणिती ह

2 पराची िदी भििशवर स निकिती ह

उपयणकत कथि ो म स कौि-सास सही हह

(a) किि 1

(b) किि 2

(c) 1 और 2 द ि ो

(d) ि त 1 ि ही 2

Q66) निमननिखित कथि ो म स कौि-सास सही हह

1 िजराह क समारक ो क समह का निमाणर

चोदि राजिोश क शासिकाि क दौराि हआ

था

2 य समारक हररिोदर पिणत शरोििा म खसथत ह

3 म रक क यातरी इबन बतता ि अपि सोसमरर ो

म िजराह क मोनदर ो की यातरा का उललि

नकया था तथा इन काजराण िाम स समब नरत

नकया था

िीच नदए गए कट का परय ग कर सही उततर चनिए

(a) किि 1

(b) किि 1 और 2

(c) किि 2 और 3

(d) किि 1 और 3

Q67) निमननिखित कथि ो म स कौि-सास सही हह

1 डॉ बी आर अमबडकर ि दी एनिनहिशि

ऑफ़ कासट (The Annihilation of Caste)

नििी थी नजसम उन ोि नहोद रमण म िोशािगत

पजारी की परथा क उनमिि की आिशयकता

पर बि नदया था

2 डॉ राजदर परसाद ि थॉटस ऑि पानकसताि

(Thoughts on Pakistan) िामक पसतक

नििी थी

िीच नदए गए कट का परय ग कर सही उततर चनिए

(a) किि 1

(b) किि 2

(c) 1 और 2 द ि ो

(d) ि त 1 ि ही 2

Q68) निमननिखित कथि ो म स कौि-सास सही हह

1 महरगढ़ भारतीय उपमहादवीप म एक परनसदध

ििपािार बसती ह ज नसोर पराोत पानकसताि म

खसथत ह

2 बरणह म म कतत ो क उिक सवामी क साथ कबर ो

म दफिाया जाता था

िीच नदए गए कट का परय ग कर सही उततर चनिए

(a) किि 1

(b) किि 2

(c) 1 और 2 द ि ो

(d) ि त 1 ि ही 2

Q69) निमननिखित कथि ो म स कौि-सास सही हह

1 काकानटय मोनदर अनरकतर नशि क समनपणत

2 हिमक ोडा म हजार-सतोभ िाि मोनदर (The

Thousand-Pillared Temple) का निमाणर

काकानटय समराट रदर ि करिाया था

िीच नदए गए कट का परय ग कर सही उततर चनिए

(a) किि 1

(b) किि 2

(c) 1 और 2 द ि ो

(d) ि त 1 ि ही 2

RAUSIAS-FC19E1003 25

Q64) Consider the following statements

1 Gommateshwara Statue is located

on the Vindyagiri Hill

2 Shravanabelagola is the place

where Chandragupta Maurya the

founder of the Mauryan dynasty

became a Jain ascetic after

relinquishing his throne

Which of the statements given above

isare correct

(a) 1 only

(b) 2 only

(c) Both 1 and 2

(d) Neither 1 nor 2

Q65) Consider the following statements

1 Archaeological evidence shows

that the Prachi Valley Civilisation

predates both Harappa and

Mohenjo-Daro

2 The Prachi river originates from

Bhubaneswar

Which of the statements given above

isare correct

(a) 1 only

(b) 2 only

(c) Both 1 and 2

(d) Neither 1 nor 2

Q66) Which of the following statements

isare correct

1 The Khajuraho group of

monuments was built during the

rule of the Chandela dynasty

2 These monuments are located in

Harischandra mountain range

3 Ibn Battuta the Moroccan

traveller in his memoirs mentioned

visiting Khajuraho temples and

called them Kajarra

Select the correct answer using the code

given below

(a) 1 only

(b) 1 and 2

(c) 2 and 3

(d) 1 and 3

Q67) Which of the following statements

isare correct

1 Dr BR Ambedkar wrote the

Annihilation of Caste emphasising

the need to do away with the

practice of hereditary priesthood in

Hinduism

2 The book lsquoThoughts on Pakistanrsquo

was written by Dr Rajendra

Prasad

Select the correct answer using the code

given below

(a) 1 only

(b) 2 only

(c) Both 1 and 2

(d) Neither 1 nor 2

Q68) Which of the following statements

isare correct

1 Mehrgarh is a famous Neolithic

settlement in the Indian

subcontinent which is situated in

Sindh province Pakistan

2 At Burzahom dogs were buried

with their masters in their graves

Select the correct answer using the code

given below

(a) 1 only

(b) 2 only

(c) Both 1 and 2

(d) Neither 1 nor 2

Q69) Which of the following statements

isare correct

1 The Kakatiya temples are

dedicated mostly to Siva

2 The Thousand-Pillared Temple at

Hanamkonda was built by the

Kakatiya king Rudra

Select the correct answer using the code

given below

(a) 1 only

(b) 2 only

(c) Both 1 and 2

(d) Neither 1 nor 2

RAUSIAS-FC19E1003 26

Q70) निमननिखित कथि ो म स कौि-सास सही हह

1 अहमदाबाद नमि हड़ताि क दौराि महातमा

गाोरी ि शरनमक ो क पकष क मजबत करि क

निए आमरर अिशि नकया था

2 अिशि स नमि मानिक ो पर दबाि पड़ा था ज

अोततः शरनमक ो क िति म 15 परनतशत की िखदध

करि क निए सहमत हए थ

िीच नदए गए कट का परय ग कर सही उततर चनिए

(a) किि 1

(b) किि 2

(c) 1 और 2 द ि ो

(d) ि त 1 ि ही 2

Q71) निमननिखित म स नकसक नकिक भारत स यिसक

की माििता की अमतण साोसकनतक निरासत की

परनतनिनर सची (The UNESCOrsquos List of the

Representative List of the Intangible

Cultural Heritage of Humanity) म शानमि

नकया गया ह

1 मनडयटट

2 सोकीतणि

3 को भ मिा

िीच नदए गए कट का परय ग कर सही उततर चनिए

(a) किि 1 और 2

(b) किि 2 और 3

(c) किि 3

(d) 1 2 और 3

Q72) निमननिखित जिजानतय ो म स कौि-सीसी ो

जिजानतजिजानतया िागािड स सोबोनरत हह

1 अोगामी

2 ककी

3 जारिा

िीच नदए गए कट का परय ग कर सही उततर चनिए

(a) किि 1

(b) किि 1 औऔ 2

(c) किि 2

(d) 1 2 और 3

Q73) निमननिखित कथि ो म स कौि-सास सही हह

1 राषटर कट सामराजय की सथापिा दोनतदगण ि की थी

नजसि मानयाित म अपिी राजरािी की

सथापिा की थी

2 राषटर कट समराट अम घििण एक ििक था और

उस कनिताओो पर पहिी कननड़ पसतक नििि

का शरय नदया जाता ह

िीच नदए गए कट का परय ग कर सही उततर चनिए

(a) किि 1

(b) किि 2

(c) 1 और 2 द ि ो

(d) ि त 1 ि ही 2

Q74) निमननिखित कथि ो म स कौि-सास सही हह

1 कशब चोदर सि ि ततवब नरिी सभा की

अधयकषता की थी ज आधयाखतमक सतय की

ि ज म सोिि थी

2 बरहम समाज ि मािि गररमा पर बि नदया

मनतणपजा का निर र नकया और सती परथा जसी

सामानजक बराइय ो की आि चिा की

िीच नदए गए कट का परय ग कर सही उततर चनिए

(a) किि 1

(b) किि 2

(c) 1 और 2 द ि ो

(d) ि त 1 ि ही 2

Q75) निमननिखित कथि ो म स कौि-सास सही हह

1 भारत म नचशती नसिनसिा खवाजा म इिददीि

नचशती क दवारा सथानपत नकया गया था

2 नचशती परोपरा की एक परमि निशिता

आतमसोयम थी नजसम साोसाररक म ह स दरी

बिाए रििा शानमि था

िीच नदए गए कट का परय ग कर सही उततर चनिए

(a) किि 1

(b) किि 2

(c) 1 और 2 द ि ो

(d) ि त 1 ि ही 2

RAUSIAS-FC19E1003 27

Q70) Which of the following statements

isare correct

1 During the Ahmedabad Mill Strike

Mahatma Gandhi undertook a fast

unto death to strengthen the

workersrsquo resolve

2 The fast had effect of putting

pressure on mill owners who

finally agreed to give the workers a

15 per cent increase in wages

Select the correct answer using the code

given below

(a) 1 only

(b) 2 only

(c) Both 1 and 2

(d) Neither 1 nor 2

Q71) Which of the following are included in

the UNESCOrsquos list of the representative

list of the intangible cultural heritage of

humanity from India

1 Mudiyettu

2 Sankirtana

3 Kumbh Mela

Select the correct answer using the code

given below

(a) 1 and 2 only

(b) 2 and 3 only

(c) 3 only

(d) 1 2 and 3

Q72) Which of the following tribes isare

related to Nagaland

1 Angami

2 Kuki

3 Jarawa

Select the correct answer using the code

given below

(a) 1 only

(b) 1 and 2 only

(c) 2 only

(d) 1 2 and 3

Q73) Which of the following statements

isare correct

1 Rashtrakuta kingdom was founded by Dantidurga who established his capital at Manyakhet

2 Amoghavarsha a Rashtrakuta king was an author and is credited with writing the first

Kannada book on poetics

Select the correct answer using the code given below

(a) 1 only

(b) 2 only

(c) Both 1 and 2

(d) Neither 1 nor 2

Q74) Which of the following statements isare correct

1 Keshab Chandra Sen headed the Tattvabodhini Sabha which was engaged in search of spiritual truth

2 The Brahmo Samaj laid emphasis on human dignity opposed idolatry and criticized such social

evils as the practice of Sati

Select the correct answer using the code given below

(a) 1 only

(b) 2 only

(c) Both 1 and 2

(d) Neither 1 nor 2

Q75) Which of the following statements isare correct

1 The Chishti order was established in India by Khwaja Moinuddin

Chishti

2 A major feature of the Chishti tradition was austerity including maintaining a distance from the

worldly power

Select the correct answer using the code

given below

(a) 1 only

(b) 2 only

(c) Both 1 and 2

(d) Neither 1 nor 2

T e s t i s p a r t o f R a u rsquo s I A S T e s t s e r i e s f o r P r e l i m i n a r y E x a m 2 0 1 9

FOUNDATION + CURRENT AFFAIRS

GENERAL STUDIES (PAPER ndashI)

FOUNDATION TEST ndashIII

SUBJECT NCERT History Class VI-X + Current Affairs

Time Allowed 1frac12 Hours Maximum Marks 150

I NSTRUCT IONS

1 IMMEDIATELY AFTER THE COMMENCEMENT OF THE EXAMINATION YOU SHOULD CHECK

THAT THIS TEST BOOKLET DOES NOT HAVE ANY UNPRINTED OR TORN or MISSING PAGES OR

ITEMS ETC IF SO GET IT REPLACED BY A COMPLETE TEST BOOKLET

2 This Test Booklet contains 75 items (questions) Each item is printed both in Hindi and English

Each item comprises four responses (answers) You will select the response which you want to mark

on the Answer Sheet In case you feel that there is more than one correct response mark the

response which you consider the best In any case choose ONLY ONE response for each item

3 You have to mark all your responses ONLY on the separate Answer Sheet (OMR sheet) provided

Read the directions in the Answer Sheet

4 All items carry equal marks

5 Before you proceed to mark in the Answer Sheet the response to various items in the Test booklet

you have to fill in some particulars in the Answer Sheet as per instructions contained therein

6 After you have completed filling in all your responses on the Answer Sheet and the examination has

concluded you should hand over to the Invigilator only the Answer Sheet You are permitted to

take away with you the Test Booklet

7 Penalty for wrong answers

THERE WILL BE PENALTY FOR WRONG ANSWERS MARKED BY A CANDIDATE IN THE

OBJECTIVE TYPE QUESTION PAPERS

(i) There are four alternatives for the answer to every question For each question for which a

wrong answer has been given by the candidate one-third of the marks assigned to that

question will be deducted as penalty

(ii) If a candidate gives more than one answer it will be treated as a wrong answer even if one of

the given answers happens to be correct and there will be same penalty as above to that

question

(iii) If a question is left blank ie no answer is given by the candidate there will be no penalty for

that question

T h i s t e s t i s p a r t o f R a u rsquo s I A S T e s t s e r i e s f o r P r e l i m i n a r y E x a m 2 0 1 9

Test Code

FC19E1003

FC19H1003 29

Answers and Explanations of

NCERT History Class VI-X + Current Affairs (FC19E1003)

Q1) उततर (c)

सपषटीकरण

- ऋगवद म दविय ो और दिताओो क समवपित एक

हजार स अविक सत तर (शल क) ह

- य शल क ऋविय ो क दवारा रच गए थ और परि ो

दवारा सीख जात थ

- हालाोवक कछ शल क मवहलाओो (जस वक अपाला

घ सा ल पामदरा मतरयी और गागी) क दवारा भी रच

गए थ

- ऋगवद म सोिाद क रप म कई शल क मौजद ह

- हम विशवावमतर नामक एक ऋवि और दविय ो क

रप म पजी जान िाली द नवदय ो (वयास और

सतलज) क बीच िाताि का उदाहरण वमलता ह

- इसस पता चलता ह वक विशवावमतर िवदक काल स

सोबोवित थ

Q2) उततर (b)

सपषटीकरण

- करनल गफाओो स राख क अिशि परापत हए ह

ज इस ओर सोकत करत ह वक ततकालीन ल ग

अवि क उपय ग स पररवचत थ

- य गफाएो आोधर परदश म सथथत ह

Q3) उततर (c)

सपषटीकरण

bull बरािह म ितिमान कशमीर म सथथत एक

परागवतहावसक थथल ह जहाो ल ग गडढ क घर ो का

वनमािण करत थ

bull य घर जमीन क ख द कर बनाए जात थ तथा नीच

जान क वलए सीवियाा ह ती थी

bull ऐसा अनमान लगाया जाता ह वक य घर ठो ड क

मौसम म आशरय परदान करत थ

Q4) उततर (c)

सपषटीकरण

bull परालख-विदया (Epigraphy) क वशलालख ो क

अधययन क रप म पररभावित वकया जाता ह

bull हसतवलसखत दसतािज ो क माधयम स इवतहास

और सावहतय क अधययन क पाोडवलवप विजञान

(Manuscriptology) कहत ह

bull पराचीन लखन परणावलय ो क अधययन और

ऐवतहावसक पाोडवलवपय ो क समझन तथा वतवथ

वनिािरण क पलीओगराफी (Palaeography) कहा

जाता ह

bull नयवमजमविकस (Numismatics) वसक ो क

अधययन क सोदवभित करता ह

Q5) उततर (a)

सपषटीकरण

- चरक सोवहता चरक क दवारा वलखी गई आयिद

और िदयक-शासर पर एक महतवपणि पसतक ह

- ि भारतीय िदयक-शासर की पारमपररक परणाली

वजस आयिद क नाम स जाना जाता ह क

अभयासकताि थ

- ऐसा माना जाता ह वक चरक का विकास दसरी

शताबदी (ईसा पिि) और दसरी शताबदी (ईसवी) क

मधय हआ था

Q6) उततर (b)

सपषटीकरण

- भाग फसल ो पर वलए जान िाल कर क सोदवभित

करता ह ज कल फसल उतपादन का 16 िाो भाग

था

- ldquoकममकारrdquo शबद भवमहीन कवि शरवमक िगि क

वलए परय ग वकया जाता था

- ldquoअशवमिrdquo (वजस घ ड क बवलदान क रप म भी

जाना जाता ह) एक अनषठान ह ता था वजसम एक

घ ड क सवतोतर रप स घमन क वलए छ ड वदया

FC19H1003 30

जाता ह और राजा क सवनक उसकी रखिाली

करत थ

Q7) उततर (d)

सपषटीकरण

- ऋगववदक काल म घ ड ो क रथ ो म ज ता जाता था

ज (रथ) भवम मिवशय ो आवद पर कबजा करन क

वलए लड गए यद ो म उपय ग वकए जात थ

- इसस यह पता चलता ह वक घ ड ो यकत रथ ो का

उपय ग महाजनपद काल स काफी पहल आरमभ

हआ था

- ऋगववदक काल म मिवशय ो भवम जल आवद पर

कबजा करन क वलए तथा ल ग ो क पकडन क

वलए यद वकय जात थ

- अविकाोश परि इन यद ो म भाग वलया करत थ

- हालाोवक उस समय क ई वनयवमत सना नही ो ह ती

थी लवकन उस काल म सभाऐो ह ती थी ो वजनम

ल ग यद क मामल ो पर चचाि करत थ

- वनयवमत सनाएा महाजनपद काल का िवशषटय थी

वजनम पदल सवनक ो की विशाल सनाएा रथ तथा

हाथी शावमल ह त थ

Q8) उततर (a)

सपषटीकरण

- बद शाकय कल स सोबोवित थ और कशीनारा म

उनका वनिन हआ था

- बद न अपनी वशकषाएा पराकत भािा म दी थी ो ज

आम ल ग ो की भािा थी

Q9) उततर (c)

सपषटीकरण

- पराचीन भारत म दशिनशासर की छह शाखाएा थी ो

िशविक नयाय समखया य ग पिि वममाोसा और

िदाोत या उततर वममाोसा

- इनकी थथापना करमश कनाद गौतम कवपल

पतोजवल जावमनी और वयास ऋविय ो न की थी

Q10) उततर (b)

सपषटीकरण

महािीर की वशकषाऐो छठी शताबदी म िललभी म

सोकवलत की गई थी ो

Q11) उततर (c)

सपषटीकरण

- पारमपररक रप स चाणकय क कौविलय अथिा

विषणगपत क नाम स जाना जाता ह

- उसन अथिशासतर ज एक पराचीन भारतीय

राजनवतक आलख ह वलखा था

Q12) उततर (d)

सपषटीकरण

- भारत का राषटर ीय वचनह सारनाथ (उततर परदश) क

अश क सतमभ क ऊपर (शीिि पर) वसोह कवपिल

का एक अनरपण ह

- इस राषटर ीय वसदाोत सतयमि जयत क साथ

सोय वजत वकया गया ह

- रामपिि बल का नाम रामपिि (वबहार) क नाम पर

पडा जहाा इसकी ख ज हई थी

- यह अपन नाजक नकाशी मॉडल क वलए परवसदद

ह वजसम क मल तवचा सोिदनशील नथन ो सतकि

कान और मरबत िााग ो क शरषठतर परवतरप क

परदवशित वकया गया ह

- यह भारतीय और फारसी ततव ो का एक ससममशरण

- सोवकससा उततर परदश म सथथत ह

Q13) उततर (a)

सपषटीकरण

का िर वसोह ज एक महान य दा थ वबहार स

सोबोवित थ

Q14) उततर (b)

सपषटीकरण

िललालर शबद बड भ-सवावमय ो क वलए परय ग

वकया जाता था

FC19H1003 31

Q15) उततर (c)

सपषटीकरण

- अररकमड एक तिीय बसती थी जहाो दर दश ो स

आन िाल जहाज ो का माल उतारा जाता था

- यहाो पर ईोि ो का एक विशाल ग दाम वमटटी क

बतिन (वजनम एमफ रा - द हरी मवठय ो का लोबा

घडा - शावमल ह) और एरिाइन (Arretine)

मदभाोड पाए गए थ

- इस थथान पर र मन दीपक काोच क बन पातर और

रतन भी पाए गए थ

Q16) उततर (a)

सपषटीकरण

- मिनदर सोगम कविताओो म उसललसखत एक

तवमल शबद ह वजसका अथि ह ldquoतीन परमखrdquo

- यह तीन सततारि पररिार ो क मसखयाओो क वलए

परय ग वकया जाता ह च ल चर और पाणडय

Q17) उततर (c)

सपषटीकरण

- ऋग िद म सभा विदाथा तथा गण जसी

जनजावतय ो पर अथिा किोब पर आिाररत

सभाओो का उललख ह

- आरसमभक िवदक काल म सभाओो और सवमवतय ो

का विशि महतव ह ता था

- यहाा तक की मसखया अथिा राजा भी उनका

समथिन परापत करन क वलए आतर रहत थ

Q18) उततर (a)

सपषटीकरण

- जन िमि न ईशवर क अससततव क मानयता त दी ह

वकनत उसन ईशवर क वजना क पद स नीच रखा

- जन िमि न बौद िमि की तरह िणि परणाली की

भरतिना नही ो की थी

Q19) उततर (d)

सपषटीकरण

- च ल ो और पाणडय ो न शसकतशाली तिीय शहर ो का

विकास वकया था

- च ल ो का सबस महतवपणि शहर पहार (या

कािरीपटटीनम) था |

- मदरई पाणडय ो की राजिानी थी

Q20) उततर (b)

सपषटीकरण

- ldquoबदचररतrdquo बद का जीिन-ितताोत ह

- इस अशवघ ि क दवारा वलखा गया था

Q21) उततर (a)

सपषटीकरणः

- तवमल कवि अपपर भगिान वशि क भकत थ

- इस परकार ि एक नयनार सोत थ

Q22) उततर (d)

सपषटीकरणः

- समदरगपत एक परवसद गपत शासक था

- उसन वसक ो पर िीणा बजात हए अपनी छवि

अोवकत करिाई थी

- यह सोगीत क परवत उसक परम क दशािता ह

- हम उसकी इलाहाबाद परशससत स महतवपणि

ऐवतहावसक जानकारी वमलती ह वजसकी रचना

उसक दरबार क कवि हररसन न की थी

Q23) उततर (b)

सपषटीकरणः

- विकरम सोित की शरआत ििि 58 ईसा पिि म

चनदरगपत वदवतीय न की थी

- यह शक ो पर उसकी जीत और उस विकरमावदतय

की पदिी वमलन क उपलकषय म आरमभ वकया गया

था

FC19H1003 32

- बानभटट न हिििििन का जीिन-ितताोत हििचररत

(ज सोसकत म थी) वलखी थी

Q24) उततर (c)

सपषटीकरणः

- सोवि-विगरावहका यद एिो शाोवत का मोतरी

- साथििाह वयापाररय ो क कावफल ो का नता

Q25) उततर (a)

सपषटीकरणः

- जआन झाोग (हसआन रताोग ndash Hsuang Tsang)

एक चीनी यातरी था ज हिििििन क शासनकाल म

भारत आया था

- ििि 630 ईसवी स ज दशक आरमभ हआ था उसम

जआन झाोग मधय एवशया ईरान और

अफग़ावनसतान की यातरा करन क पशचात कशमीर

क रासत स भारत आया था

- उसन उततर स पिि तक की यातरा की और िह

लगभग 2 ििि वबहार म रहा

- जआन झाोग न नालनदा विशवविदयालय म विदयावथिय ो

और विदवान ो क साथ पारसपररक विचार-विमशि

वकया थथानीय भािाओ ा म वनपणता परापत की तथा

बौद सतप ो की ख ज की

Q26) उततर (c)

सपषटीकरणः

- परदवकषणा पथ बौद िासतकला म सतप क चार ो

ओर बनाया जान िाला एक घमािदार पथ ह ता

- परशन म वदए गए बाकी क तीन ो ततव वहोद मसनदर ो की

िासतकला क भाग ह

Q27) उततर (d)

सपषटीकरणः

परशन म वदए गए सभी मोवदर ो म वयापक रप स

ईोि ो (पकी ईोि ो) का परय ग पतथर ो क साथ हआ

Q28) उततर (c)

सपषटीकरण

- महममद कली कतब शाह ग लकणडा का सलतान

था

- िह अकबर का समकालीन था

- सावहतय और िासतकला म उसकी अतयाविक

रवच थी

- िह एक महान कवि था

- िह दसखनी उदि फारसी और तलग म वलखता था

- उसन अपन पीछ एक विसतत वदिान (सोगरह)

छ डा ह

- अभी हाल ही म तलोगाना म ग लकणडा क वकल

क अनदर खदाई वकय गए बाग-ए-नाया वकला

बाग क चार ो ओर रप-रखा क मानवचतरण क

वलए भारतीय परातासतवक सिकषण (The

Archaeological Survey of India ndash ASI)

गराउणड पनीिर विोग रडार (Ground Penetrating

Radar) का परय ग करगा

Q29) उततर (a)

सपषटीकरणः

- वसलपपावदकारम एक तवमल महाकावय ह वजसकी

रचना इलाोग क दवारा लगभग 1800 ििि पिि की

गई थी

- यह क िलन नामक एक वयापारी की कहानी ह

ज माििी नामक एक गवणका (िशया) स परम

करन लगा था

- मवनमकलाई क िलन और माििी की पतरी की

कहानी ह

Q30) उततर (a)

सपषटीकरण

- चरक आयिद और वचवकरता की एक महतवपणि

रचना चरक सोवहता क लखक ह

- बरहमगपत क अपनी रचना बरहम-सफि-वसदानत

(ज एक खग लीय रचना ह) क कारण परवससद

वमली

FC19H1003 33

- बगदाद म इसका अनिाद अरबी भािा म वकया

गया था

- इसका इसलावमक गवणत और खग ल-विजञान पर

महतवपणि परभाि पडा था

- बाद म अपन जीिनकाल म बरहमगपत न

ldquoखोडखयाकrdquo वलखी ज एक खग लीय पससतका

(एक छ िी पसतक) थी

- इसम आयिभटट की अिि-रावतर क परतयक वदन की

शरआत परणाली का परय ग वकया गया था

Q31) उततर (c)

सपषटीकरण

- अमीर खसर एक परवसद सफी सोगीतकार कवि

और विदवान थ

- 1318 म उनह ोन पाया वक इस भवम (वहोदसतान) क

हर कषतर म अलग-अलग भािा थी लाहौरी

कशमीरी दवारसमदरी (दवकषणी कनाििक म)

तलोगाना (आोधर परदश म) गजरी (गजरात म)

माबारी (तवमलनाड म ) अििी (पिी उततर परदश

म) और वहोदिी (वदलली क आस-पास क कषतर म)

आवद

- उनह न यह बताया वक सोसकत वकसी भी कषतर स

सोबोवित नही ो थी और किल बराहमण ही इस भािा

का जञान रखत थ

Q32) उततर (c)

सपषटीकरण

- वहरणय-गभि सववणिम गभि क सोदवभित करता ह

- जब बराहमण ो की सहायता स यह अनषठान वकया

जाता था त यह माना जाता था वक बवल दन िाल

का कषवतरय क रप म पनजिनम ह गा

Q33) उततर (d)

सपषटीकरण

- कदमई भवम राजसव पर कर क सोदवभित करता

- गवावलयर परशससत म नागभि क दवारा वकय गए

श िण का िणिन वकया गया ह |

- नागभि एक परवतहार राजा था

Q34) उततर (b)

सपषटीकरण

- राजतरो वगनी 12िी ो शताबदी म कलहन क दवारा

रवचत एक सोसकत पसतक (िकसट) ह

- यह परारसमभक भारत की ऐवतहावसक इवतितत थी

- तकि सोगत रप स इस अपन परकार की सिोततम

और सिािविक विशवसनीय कवत माना जाता ह

- यह कशमीर कषतर क पराचीनतम समय स लकर

उसकी रचना की तारीख तक क समपणि इवतहास

का आचछादन करती ह

Q35) उततर (c)

सपषटीकरण

- गााि की आम सभा क ldquoउरrdquo कहा जाता था

- ldquoउरrdquo म गााि क सभी कर दन िाल वनिासी

शावमल ह त थ

Q36) उततर (a)

सपषटीकरण

- वदलली सलतनत म ldquoतारीखrdquo इवतहास लखन का

एक रप था

- ldquoतािरीखrdquo क लखक विदवान परि ह त थ वजनम

सवचि परशासक इतयावद शावमल थ

Q37) उततर (a)

सपषटीकरण

- अलाउददीन सखलजी अपन सवनक ो क ितन का

भगतान नकद म करता था न वक इकता क रप

- सवनक अपना सामान वदलली म वयापाररय ो स

खरीदत थ अतः इस बात का भय था वक वयापारी

कही ो िसतओो का मलय न बिा द

- इसकी र कथाम क वलए अलाउददीन सखलजी न

वदलली म कीमत ो क वनयसित वकया

FC19H1003 34

- अविकारीगण धयानपििक मलय ो का सिकषण करत

थ तथा ज वयापारी वनिािररत मलय पर माल नही ो

बचत थ उनक दसणडत वकया जाता था

Q38) उततर (d)

सपषटीकरण

- वदलली सििपरथम त मर राजपत ो क अिीन उनक

सामराजय की राजिानी बनी थी

- 12िी ो शताबदी क मधय म अजमर क चौहान ो

(वजनह चाहमान ो क नाम स भी जाना जाता ह) न

त मर राजपत ो क परावजत वकया था

- त मर ो और चौहान ो क अिीन वदलली एक

महतवपणि िावणसजयक क दर बन गया था

- कई जन वयापारी यहाा रहन लग थ और उनह ोन

कई मोवदर भी बनिाए

- यहाा पर मवदरत वसक वजनह ldquoदहलीिालrdquo क नाम

स जाना जाता था वयापक रप स परचलन म थ

Q39) उततर (c)

सपषटीकरण

- म ठ की मसिद का वनमािण वसको दर ल दी क

राजयकाल म उसक मिी क दवारा करिाया गया

था

- बगमपरी मसिद का वनमािण महममद तगलक क

शासनकाल म हआ था

- यह मसिद विशव का पणयथथान (The

Sanctuary of the World) और वदलली म महममद

तगलक की नई राजिानी जहाोपनाह की मखय

मसिद थी

- कववत- अल - इसलाम मसिद का विसतार

इलतसिश और अलाउददीन सखलजी न वकया था

- मीनार का वनमािण तीन सलतान ो कतबददीन ऐबक

इलतसिश और वफर ज शाह तगलक क दवारा

करिाया गया था

Q40) उततर (c)

सपषटीकरण

- मगल ो क अिीन मनसबदार शबद उस वयसकत क

वलए सोदवभित वकया जाता था वजसक पास मनसब

(अथाित पद) ह ता था

- उस अपना ितन राजसव कायो वजनह जागीर कहत

थ क रप म परापत ह ता था

Q41) उततर (b)

सपषटीकरण

- ldquoभारत छ ड आोद लनrdquo वबरविश शासन क

सखलाफ ल ग ो का एक सवाभाविक विदर ह था

- असखल भारतीय काोगरस सवमवत न 8 अगसत 1942

क बमबई म एक बठक का आय जन वकया था

- इस बठक म परवसद सोकलप ldquoभारत छ ड rdquo क

पाररत वकया गया और इस उददशय क परापत करन

क वलए गाोिी क नततव म एक अवहोसक जन सोघिि

आोद लन की शरआत का परसताि वदया गया

- लवकन अगल ही वदन गाोिी और काोगरस क अनय

परमख नताओो क वगरफतार कर वलया गया

- काोगरस क एक बार वफर अिि घ वित वकया गया

था

Q42) उततर (c)

सपषटीकरण

- साइमन कमीशन यनाइविड वको गडम क सात

साोसद ो का एक समह था

- इस वबरविश भारत क वलए सोििावनक सिार ो का

सझाि दन क वलए गवठत वकया गया था

- इस आय ग म िररषठ वबरविश राजनता सर जॉन

साइमन क नततव म किल वबरविश सदसय ही

शावमल थ

- इसवलए भारत क ल ग ो न साइमन कमीशन क

आगमन क विरद आोद लन वकया था

Q43) उततर (a)

सपषटीकरण

bull दादा भाई नौर जी भारत म वबरविश शासन क

आवथिक पररणाम ो क बार म अपनी विर िी

(परवतकल) राय क वलए जान जात थ

FC19H1003 35

bull अपन कई लख ो और भािण ो म विशि रप स

ldquoपाििी एो ड अन-वबरविश रल इन इसणडया

(Poverty and Un-British Rule in India) म

नौर जी न यह तकि वदया वक भारत पर अतयविक

कर लगाया गया था और इसकी सोपवतत इोगलड की

ओर परिावहत की जा रही थी

bull उनह ोन पराचीन भारतीय गरोथ ो की वयाखया करन

का और भारतीय ो क आिविशवास क बहाल

करन पर कायि नही ो वकया था

उनह ोन वकसी और बात स पहल सभी सामावजक

बराइय ो क उनमलन की आिशयकता पर भी बल

नही ो वदया था

Q44) उततर (c)

सपषटीकरण

bull अगसत 1932 म वबरविश परिानमोतरी मकड नालड न

अपन साोपरदावयक परसकार (The Communal

Award) की घ िणा की थी

bull यह भारत क कई साोपरदावयक वहत ो क बीच विवभनन

सोघिो क हल करन क वलए वबरिन का एकतरफा

परयास था

bull यह परसकार (Award) बाद म 1935 क

अविवनयम (The Act of 1935) म शावमल वकया

गया था

bull इस साोपरदावयक परसकार न मससलम ो क वलए

आरवकषत एक अलग वनिािचक मणडल फॉमिल का

विसतार अनय अलपसोखयक ो क वलए वकया था

वजसम वसख ो भारतीय ईसाइय ो आोगल-भारतीय

समदाय यर पीय समदाय तथा विवशषट कषतरीय

समह ो क शावमल वकया गया था

bull गाोिी न इस परसताि क भारतीय समाज क

विभावजत करन क वलए एक घवणत वबरविश

सावजश क रप म दखा और उसक सखलाफ

आमरण अनशन वकया

Q45) उततर (b)

सपषटीकरण

मौजदा आयात और वनयाित क अवतररक़त

औपवनिवशक भारत क वनमनवलसखत खचो क

वलए एक विशिवनवशचत िन रावश भी दनी पडती

थी

(i) परशासन क वयय

(ii) सना क रख-रखाि क वयय

(iii) यद क वयय

(iv) सिावनितत अविकाररय ो की पशन तथा

(v) वबरिन दवारा अपनी उपवनिश बसती

(कॉल नी) क रख-रखाि क वयय

इनह गह शलक (Home Charges) क रप म

जाना जाता था और लगभग परी तरह स भारत क

दवारा इनका भगतान वकया जाता था

bull गह शलक म वनमनवलसखत घिक शावमल थ

(i) भारतीय ऋण पर दय बयाज

(ii) ईसट इोवडया को पनी क शयरिारक ो क

लाभाोश

(iii) लोदन म भारत कायािलय चलान क वलए िन

(iv) भारत म वनयकत वबरविश कवमिय ो क ितन

और पशन का भगतान करन क वलए िन

(v) रलि पर बयाज

(vi) नागररक और सनय शलक

(vii) इोगलड म सट र (सामगरी) की खरीद

Q46) उततर (b)

सपषटीकरण

bull भारतीय राषटर ीय काोगरस का लाहौर सतर 1929 म

जिाहरलाल नहर की अधयकषता म आय वजत

वकया गया था

bull इस सतर म भारतीय राषटर ीय आोद लन स समबसित

कई महतवपणि पररणाम सामन आय थ

(i) सििपरथम इस सतर म काोगरस क अधयकष पद

पर जिाहरलाल नहर क चना गया था ज

काोगरस म िामपोवथय ो की बिती हई ताकत

का सपषट सोकत था

(ii) दसरा इस सतर म पहली बार काोगरस न पणि

सवतोतरता की माोग क उठाया था

इस परकार की माोग काोगरस मोच स पहल कभी भी

नही ो उठाई गई थी

Q47) उततर (b)

सपषटीकरण

FC19H1003 36

bull इस ररप िि न वकसी भी समदाय क वलए पथक

वनिािचक मोडल अथिा अलपसोखयक ो क वलए

भाराोश की वसफाररश नही ो की थी

bull तथावप इस ररप िि न उन पराोत ो म अलपसोखयक

सीि ो क आरकषण की अनमवत दी थी जहाा पर कम

स कम दस परवतशत अलपसोखयक ह

bull लवकन यह समदाय क आकार क अनपात म ह ना

चावहए था

bull इस ररप िि म भारत क वलए पणि सवतोतरता क

वलए क ई पराििान नही ो था

Q48) उततर (c)

सपषटीकरण

bull आरो वभक िवदक आयो का िमि मखय रप स

परकवत की पजा और यजञ था

bull परारो वभक आयि िमि परकवत की पजा क समान था

bull िासति म उनक चार ो ओर की शसकतयाा वजनह न

त ि वनयोवतरत कर सकत थ और न ही समझ पाए

थ उनह वदवयता क साथ वनिवशत वकया गया तथा

उनह मादा या नर दिीदिताओो क रप म

परतीकतव वकया गया था

bull उनह ोन कछ यजञ ो का भी वनषपादन वकया था

Q49) उततर (b)

सपषटीकरण

bull सडक और नदी-मागि (जल-मागि) डकती स

सरवकषत नही ो थ

bull उललखनीय ह वक हिििििन क शासनकाल क

दौरान यआन चिाोग (हयएन साोग) का सारा

सामान लि वलया गया था

Q50) उततर (c)

सपषटीकरण

परशन म वदए गए द न ो कथन सही ह

Q51) उततर (b)

सपषटीकरण

bull परोदर दास एक सोत और भगिान कषण क एक

महान भकत थ

bull परोदर दास क कनाििक सोगीत क वपतामह क

रप म जाना जाता ह

bull यदयवप उनक जनम-थथान क बार म काफी

अिकल लगाई जाती रही ह

bull तथावप अब कननड विशवविदयालय हमपी क दवारा

गवठत एक विशिजञ सवमवत इस वनषकिि पर पहोची

ह वक उनका जनम थथान सोभितया कनाििक का

एक छ िा-सा गााि कषमपरा (वशिम गगा वजला)

था

Q52) उततर (c)

सपषटीकरण

bull शरी तयागराज शरी शयाम शासतरी और शरी मथसवामी

दीवकषतर क कनाििक सोगीत की वतरमवति माना

जाता ह

bull उनक कारण ही 18िी ो-19िी ो शताबदी म कनाििक

सोगीत का सववणिम यग आया था

Q53) उततर (d)

सपषटीकरण

bull अभी हाल ही म लौह यगीन-महापािावणक काल

का 2000 ििि पराना एक दलिभ सारक फगस

(Sarcophagus) (पतथर का ताबत) क ललम क

वियर गाोि (क वयलडी क पास वजला क वझक ड

करल राजय) की एक रॉक-कि गफा स ख जा गया

bull यह ताबत वजसम हविय ो क िकड थ खदाई क

दौरान वमला

bull अभी तक इस परकार की दलिभ ख ज करल क

मातर द ही थथान ो स हई ह

bull य द न ो सारक फगी (Sarcophagi) (पतथर क

ताबत) चियर और अथ ली (वजला क वझक ड) क

महापािाण थथल ो स वमल ह

Q54) उततर (a)

सपषटीकरण

FC19H1003 37

दवकषण भारत म महापािाण सोसकवत एक पणि

विकवसत लौह यगीन सोसकवत थी

Q55) उततर (d)

सपषटीकरण

bull च ल पाणडय और करलपतर (चर) इन तीन ो का

उललख अश क क अवभलख ो म वकया गया ह

bull सोभितः य भौवतक सोसकवत क उततर

महापािावणक चरण म थ

Q56) उततर (d)

सपषटीकरण

bull भीमा-क रगाोि की लडाई ततीय आोगल-मराठा

यद का वहससा थी

Q57) उततर (b)

सपषटीकरण

bull राजकमार शकल न गाोिीजी क चोपारण आन तथा

वतनकवथया परणाली स जडी समसया की जाोच क

वलए रारी करन क वलए दश भर म उनका

अनसरण वकया था

bull बज वकश र राजदर परसाद महादि दसाई और

नरहरी पाररख चोपारण सतयागरह क दौरान गाोिी

जी क सहय गी थ

Q58) उततर (b)

सपषटीकरण

bull बराहमण ो और बौद मठिाररय ो क कर-मकत गााि

अनदान म दन की परथा सतिाहन ो न आरमभ की

थी

Q59) उततर (c)

सपषटीकरण

इस कायिकरम क उददशय वनमनानसार ह

(i) बवनयादी पयििन आिाररक सोरचना का विकास

करना

(ii) चयवनत (पहचान वकय गए) कषतर ो म आजीविका क

सजन क वलए दश क साोसकवतक और विरासत

मलय ो क बिािा दना

(iii) विरासत समारक थथल ो पर विशव सतरीय आिाररक

सोरचना विकवसत करक एक सतत तरीक स

पयििक आकििण म िसद करना

(iv) थथानीय समदाय ो की सवकरय भागीदारी क माधयम

स र रगार ो का सजन करना

(v) र रगार उतपादन और आवथिक विकास क वलए

पयििन कषमता का उन पर परभाि का उपय ग

करना तथा

(vi) िारणीय पयििन आिाररक सोरचना का विकास

करना और उसका उवचत सोचालन तथा

रखरखाि सवनवशचत करना

Q60) उततर (b)

सपषटीकरण

bull यह वनकाय ििि 1987 म अससततव म आया था

bull यह एक राषटर ीय सतर का शीिि सोगठन ह ज भारत

सरकार क जनजातीय मामल ो क मोतरालय क

परशासवनक वनयोतरण क अिीन काम कर रहा ह

bull इसका पोजीकत और परिान कायािलय नई वदलली

म सथथत ह

Q61) उततर (c)

सपषटीकरण

bull परमचोद क उपनयास ो म परमाशरम रोगभवम गबन

कमिभवम और ग दान शावमल ह

bull ग रा रिी ोदरनाथ िग र क दवारा रवचत उपनयास ह

bull अभी हाल ही म मोशी परमचोद की 138िी ो जयोती दश

भर म मनाई गई थी

Q62) उततर (b)

सपषटीकरण

bull ldquoवगदाrdquo पोजाब (भारत) एिो पावकसतान की

मवहलाओो क दवारा तयौहार क समय और फसल

की बिाई तथा किाई क अिसर पर वकया जान

िाला एक पारोपररक दहाती नतय ह

FC19H1003 38

bull इस नतय क माधयम स पोजाबी मवहलाऐो अपनी

परसननता परकि करती ह तथा वगदा क परदशिन क

माधयम स परि िचिसव िाल समाज म मवहलाओो

की दबी हई भािनाओो क परकि करती ह

bull चोवक इस नतय का परि ो क साथ क ई सोबोि नही ो

ह अतः किल मवहलाऐो ही इसम भाग ल सकती

bull हर साल तीज समार ह क दौरान पोजाब म वगदा

नतय वकया जाता ह

तीज भारत क कछ भाग ो म मवहलाओो क दवारा

मनाया जान िाल कई तयौहार ो क वलए एक

वयापक नाम ह

Q63) उततर (a)

सपषटीकरण

- मजम-उल-बहरीन या द समदर ो का सोगम

नामक उललखनीय रचना दारा वशक ह क दवारा

वलखी थी

- भारत क उपराषटर पवत शरी एम िकया नायड न कहा

ह वक राजकमार दारा वशक ह की रचनाएा शाोवत

और सदभाि क बिािा दन क वलए एक तारा सर त

क रप म सामन आ सकती ो ह

- उपराषटर पवत गत ििो क भला वदए गए राजकमार

दारा वशक ह क परदवशित परचवलत करन हत

आय वजत एक परदशिनी का दौरा करन क बाद एक

सभा क सोब वित कर रह थ

- इस परदशिनी का आय जन फर क इस गौवियर

(Francois Gautier) क दवारा lsquoइोवदरा गाोिी नशनल

सिर फॉर द आििसrsquo (The Indira Gandhi

National Centre for the Arts) नई वदलली म

वकया गया था

Q64) उततर (c)

सपषटीकरण

- ग मतशवर परवतमा जन भगिान बाहबली क

समवपित ह

- यह एक एक-चटटानी पतथर की मवति ह

- राषटर पवत राम नाथ क विोद न शरिणबलग ला

(कनाििक) म आय वजत वकय जान िाल भवय

अवभिक समार ह महामसतकावभिक का

उदघािन वकया था

- यह समार ह 12 ििो म एक बार ह ता ह

Q65) उततर (c)

सपषटीकरण

bull पराची घािी पराची नदी क चार ो ओर फली हई थी

bull पराची घािी िीर-िीर विलपत ह गई थी

bull पराची नदी भिनशवर स वनकलती ह

bull यह महानदी की एक सहायक नदी ह और यह

परी खदाि किक तथा जगतवसोहपर वजल ो स

ह कर बहती ह

bull इस नदी क पर कषतर क पराची घािी कहा जाता ह

bull यह नदी बोगाल की खाडी म वगरती ह

परातासतवक साकषय स पता चलता ह वक पराची घािी

सभयता हडपपा और म हनज दाड द न ो की

पिििती ह

Q66) उततर (d)

सपषटीकरण

य समारक छतरपर वजल (मधय परदश) म विोधयाचल

पिित शरोखला म सथथत ह

Q67) उततर (a)

सपषटीकरण

bull थॉिस ऑन पावकसतान नामक पसतक डॉ बी

आर अमबडकर न वलखी थी

bull डॉ बी आर अमबडकर की जयोती क अिसर पर

भारत क राषटर पवत न भारत की इस महान हसती

क शरदाोजवल अवपित की थी

bull डॉ बी आर अमबडकर न 1924 म वडपरथड

कलावसर इोसटीटयि (दवलत िगि सोथथान -

बवहषकत वहतकाररणी सभा) और 1927 म समाज

समता सोघ की थथापना की थी

bull अमबडकर का धयान वशकषा कषतर की ओर भी था

bull उनह ोन वशकषा क वनमन िगो म फलान क वलए

पीपलस एजकशन स साइिी (The Peoples

Education Society) क नाम स महाविदयालय ो क

नििकि और छातरािास ो की थथापना की थी

FC19H1003 39

Q68) उततर (b)

सपषटीकरण

bull महरगि भारतीय उपमहादवीप म एक परवसद

निपािाण बसती ह ज बलवचसतान पराोत

पावकसतान म सथथत ह

bull दचपलली (आोधर परदश) क पास नागलर नदी क

पिी ति ो पर चना पतथर क बलॉक क विशाल

विसतार म एक पिि-ऐवतहावसक रॉक आिि थथल की

ख ज की गई ह

bull इसन 1500-2000 ईसा पिि क दौरान गोिर (आोधर

परदश) म विकवसत निपािाण सभयता पर परकाश

डाला ह

Q69) उततर (c)

सपषटीकरण

bull 12िी ो सदी और 13िी ो सदी म काकाविय िोश का

उदय हआ था

bull ि पहल कलयाण क पवशचमी चालकय ो क सामोत थ

bull परारोभ म उनह ोन िारोगल (तलोगाना) क पास एक

छ ि स कषतर पर शासन वकया था

bull उनह ोन ldquoनायक वयिथथाrdquo की शरआत की थी

वजस बाद म विजयनगर क राय शासक ो न

अपनाया और विकवसत वकया था

Q70) उततर (a)

सपषटीकरण

bull गाोिीजी क अनशन स वमल मावलक ो पर दबाि

पडा था ज अोततः शरवमक ो क ितन म 35 परवतशत

की िसद करन क वलए सहमत हए थ

bull गगल (Google) न अनसया साराभाई वजनह ोन

भारत क शरवमक आोद लन म एक अगरणी भवमका

वनभाई थी की 132िी ो जयोती डडल (Doodle) का

वनमािण करक मनाई

Q71) उततर (d)

सपषटीकरण

भारत स यनसक की मानिता की अमति साोसकवतक

विरासत की परवतवनवि सची म वनमनवलसखत शावमल ह

bull कवडयटटम करल का सोसकत रोगमोच

bull मवडयिि करल का अनषठान रोगमोच और नतय

नाविका

bull िवदक मि जाप की परोपरा

bull राजथथान क कालबवलया ल क गीत और नतय

bull रामलीला रामायण का पारोपररक परदशिन

bull सोकीतिन मवणपर का अनषठान गायन ढ ल िादन

और नतय

bull रममन भारत क गििाल वहमालय का िावमिक

तयौहार और अनषठान रोगमोच

bull जाोदीयाला गर पोजाब क ठठर ो की पीतल और

ताोब क वशलप स वनवमित बतिन ो की पारोपररक कला

bull छाऊ नतय पिी भारतीय राजय ो म जनमी शासतरीय

भारतीय नतय कला

bull लददाख का बौद मि जाप िर ाोस-वहमालयी लददाख

कषतर तथा जमम-कशमीर म पवितर बौद गरोथ ो का पाठ

bull य ग

bull नौर र

bull को भ मला

Q72) उततर (b)

सपषटीकरण

bull भारत क राषटर पवत शरी राम नाथ क विोद न

वकसामा नागालड म हॉनिवबल मह रति और

राजय गठन वदिस समार ह का उदघािन वकया

था

bull हॉनिवबल मह रति का नाम भारतीय हॉनिवबल क

नाम पर पडा ह ज एक विशाल और रोगीन जोगली

पकषी ह

bull यह पकषी नागालड राजय की अविकतर जनजावतय ो

की ल ककथाओो म उसललसखत ह

bull नागालड की परमख मानयता परापत जनजावतयाा ह

अोगामी आओ चखसोग चाोग ककी रगमा और

रवलोग आवद

bull ओोग जारिा और ससिनलीस अोडमान-वनक बार

दवीप समह की जनजावतयाा ह

FC19H1003 40

Q73) उततर (c)

सपषटीकरण

bull दकन म राषटर कि शासन दसिी ो सदी क अोत तक

लगभग 200 ििो तक रहा था

bull राषटर कि शासक अपन िावमिक विचार ो म सवहषण

bull उनह ोन न किल शि िमि और िषणि िमि बसलक

जन िमि क भी सोरकषण वदया था

bull एल रा म वशि क परवसद रॉक कि मोवदर का

वनमािण नौिी ो सदी म राषटर कि राजा कषण परथम न

करिाया था

bull उसका उततराविकारी अम घििि जन था लवकन

उसन अनय िमो क भी सोरकषण परदान वकया था

bull राषटर कि ो न मसलमान वयापाररय ो क बसन की

अनमवत दी थी

bull उनह न अपन अविराजय ो म इसलाम क उपदश दन

की भी अनमवत दी थी

bull अभी हाल ही म पाोडिलागटटा (तलोगाना) क

परागवतहावसक चटटान वचतर ो क कषरण की बिती हई

घिनाएा एक गोभीर वचोता का वििय ह

bull यह परागवतहावसक चटटान क नकसान पहाचा

सकता ह

bull पाोडिलागटटा वनमनवलसखत क वलए जाना जाता ह

- 10000 ईसा पिि स 8000 ईसा पिि क वचवतरत

चटटानी आशरय ो क वलए

- राषटर कि काल क एक 8 िी ो सदी क

वशलालख क वलए और

- 12िी ो सदी क काकविय सामराजय क वभवतत

वचतर ो क वलए

Q74) उततर (b)

सपषटीकरण

bull 1828 म राजा राम म हन रॉय न एक नय िावमिक

समाज बरहम सभा की थथापना की थी वजस बाद

म बरहम समाज क नाम स जाना गया था

bull दिदरनाथ िग र न ततवब विनी सभा की अधयकषता

की थी ज आधयासिक सतय की ख ज म सोलि

थी

bull इसका उददशय वहोद िमि क शद करन का और

एकशवरिाद (एक ईशवर म आथथा) का परचार करना

था

bull नय समाज की थथापना क आिार थ कारण

(तकि ) क द सतमभ तथा िद और उपवनिद

bull अभी हाल ही म सािारण बरहम समाज का कछ

काननी मदद ो क लकर पवशचम बोगाल सरकार क

साथ काननी वििाद चल रहा ह

Q75) उततर (c)

सपषटीकरण

bull भारत म वचशती वसलवसल की थथापना खवाजा

म इनददीन वचशती क दवारा की गयी थी

bull ि 1192 ईसवी क आसपास भारत आय थ

bull वचशतीय ो क बारहिी ो शताबदी क उततरािि म भारत

म आन िाल सफीय ो क समह ो म सबस

परभािशाली माना जाता ह

bull उनह ोन थथानीय िातािरण क साथ सफलतापििक

अनकलन वकया और उनह ोन भारतीय भसकत

परोपराओो क कई पहलओो क अपनाया

bull अजमर म सफी अपरकि खवाजा म इनददीन वचशती

की ऐवतहावसक दरगाह क एक नया रप दन की

तयारी की जा रही ह

bull इस 13िी ो शताबदी की दरगाह क ldquoसवचछ

आइकॉवनक थथल ोrdquo (Swacch Iconic Places) म

शावमल वकया गया ह ज परवतवषठत विरासत

आधयासिक और साोसकवतक थथान ो पर क वदरत

य जना ह

FC19H1003 41

ANSWERS amp EXPLANATION OF

NCERT History Class VI-X + Current Affairs

(FC19E1003)

Q1) Answer c

Explanation

Rigveda consists of more than a

thousand hymns dedicated to gods and

goddesses These hymns were

composed by sages and learnt by men

however a few were composed by

women like Apala Ghosa Lopamudra

Maitreyi and Gargi

Rigveda consists of many hymns in the

form of dialogues We get an example of

a dialogue between a sage named

Vishwamitra and two rivers (Beas and

Sutlej) that were worshipped as

goddesses This suggests that he

belonged to the Vedic period

Q2) Answer b

Explanation

Traces of ash have been found from

Kurnool Caves suggesting that people

were familiar with the use of fire

It is situated in Andhra Pradesh

Q3) Answer c

Explanation

Burzahom is a prehistoric site in

present day Kashmir where people built

pit houses which were dug into the

ground with steps leading into them

These may have provided shelter in cold

weather

Q4) Answer c

Explanation

Epigraphy is defined as the study of

inscriptions

Manuscriptology is the study of history

and literature through the use of hand

written documents

Palaeography refers to the study of

ancient writing systems and the

deciphering and dating of historical

manuscripts

Numismatics refers to the study of

coins

Q5) Answer a

Explanation

Charaka Samhita was written by

Charaka and is an important book on

Ayurveda and medicine

He was a practitioner of the traditional

system of Indian medicine known as

Ayurveda

Charaka is thought to have flourished

sometime between the 2nd century BCE

and the 2nd century CE

Q6) Answer b

Explanation

Bhaga refers to the tax on crops which

was fixed at 16th of the production

Kammakaras is the term used for the

landless agricultural labour class

Ashvamedha also known as horse

sacrifice is a ritual where a horse is let

loose to wander freely and it was

guarded by the rajarsquos men

Q7) Answer (d)

Explanation

In the Rigvedic period horses were

yoked to chariots that were used in

battles fought to capture land cattle

etc This suggests that the use of horse

chariots began much before the period

of Mahajanapadas

The battles were fought in the Rigvedic

period for cattlersquos lands water an even

to capture people Most men took part

in these wars however there was no

regular army but there were assemblies

where people met and discussed

matters of war Regular armies became

a feature in the Mjahajanapada period

including vast armies of foot soldiers

chariots and elephants

RAUSIAS-FC19E1003 42

Q8) Answer (a)

Explanation

Buddha belonged to the Sakya clan and

passed away at Kusinara

Buddha taught in Prakrit which was the

common language of people

Q9) Answer c

Explanation

There were six schools of philosophy in

ancient India These are known as

Vaishesika Nyaya Samkhya Yoga

Purva Mimansa and Vedanata or Uttara

Mimansa They were founded by sages

Kanada Gautama Kapila Patanjali

Jamini and Vyasa respectively

Q10) Answer b

Explanation

The teachings of Mahavira were

compiled at Valabhi in 6th century AD

Q11) Answer (c)

Explanation

Chanakya is traditionally identified as

Kautilya or Vishnugupta who authored

the ancient Indian political treatise the

Arthashastra

Q12) Answer d

The national emblem of India is an

adaptation of the Lion Capital atop the

Ashoka Pillar of Sarnath Uttar Pradesh

and is combined with the National

Motto Satyameva Jayate

The Rampurva Bull gets the name from

the site of its discovery Rampurva in

Bihar

It is noted for its delicately sculpted

model demonstrating superior

representation of soft flesh sensitive

nostrils alert ears and strong legs It is

a mixture of Indian and Persian

elements

Sankissa is situated in Uttar Pradesh

India

Q13) Ans(a)

Kunwar Singh was a notable leader during the Revolt of 1857 He belonged

to a royal house of Jagdispur Bihar

Q14) Answer b

Explanation

The term Vellalar was used for large

landowners

Q15) Answer c

Explanation

Arikamedu was a coastal settlement

where ships unloaded goods from

distant lands Finds here include a

massive brick warehouse pottery

including amphorae and Arretine ware

Roman lamps glassware and gems have

also been found at the site

Q16) Answer a

Explanation

Muvendar is a Tamil word mentioned in

Sangam poems meaning three chiefs

used for the heads of three ruling

families the Cholas Cheras and

Pandyas

Q17) Ans (c)

Several tribal or kin-based assemblies

such as the Sabha Vidatha and gana

are mentioned in the Rig-veda The

Sabha and the samiti mattered a great

deal in early Vedic times so much so

that the chiefs or the kings showed an

eagerness to win their support

Q18) Ans (a)

Jainism recognised the existence of the

gods but placed them lower than the

jina and did not condemn the varna

system as Buddhism did

Q19) Answer (d)

Explanation

Cholas and Pandyas had developed

powerful coastal cities The most

important city of Cholas was Puhar or

Kaveripattinam and Madurai was the

capital of Pandyas

Q20) Answer b

Explanation

Buddhacharita is the biography of

Buddha and was written by

RAUSIAS-FC19E1003 43

Ashvaghosha

Q21) Answer (a)

Explanation

Tamil poet Appar was a Shiva devotee

So he was a Nayanar saint

Q22) Answer d

Explanation

Samudragupta was a prominent Gupta

ruler whose coins depict him playing a

veena indicating his love for music We

get important historic information from

his Allahabad Prashasti which was

composed by his court poet Harisena

Q23) Answer (b)

Explanation

Vikrama Samvat was founded by

Chandragupta II in the 58 BC as a

mark of victory over the Shakas and

assumed the title of Vikramaditya

Banabhatta wrote Harshavardhanarsquos

biography the Harshacharita in

Sanskrit

Q24) Answer c

Explanation

Sandhi-vigrahika was the minister of

war and peace

Sarthavaha was the leader of the

merchant caravans

Q25) Answer a

Explanation

Xuan Zang (Hsuan-tsang) was a

Chinese traveller who came during the

reign of Harshavardhana

In the decade that began in 630 AD

Xuan Zang came to India through

Kashmir after visiting Central Asia Iran

and Afghanistan

He travelled from north to east and lived

in Bihar for a couple of years

At Nalanda University Xuan Zang

interacted with students and scholars

mastered local languages and

discovered Buddhist stupas

Q26) Answer c

Explanation

Pradakshina patha is a circular path

laid around a stupa in Buddhist

architecture While the rest are a part of

temple architecture

Q27) Answer d

Explanation

All the above-mentioned temples have

an elaborate use of bricks (baked

bricks) along with stone

Q28) Ans (c)

Muhammad Quli Qutab was the Sultan

of Golconda He was a contemporary of

Akbar was very fond of literature and

architecture

The Sultan was a great poet and he

wrote in Dakhini Urdu Persian and

Telgu and has left an extensive diwan or

collection

Recently the Archaeological Survey of

India (ASI) will be using Ground

Penetrating Radar (GPR) to map the

contours of the area around the Bagh-e-

Naya Qila excavated garden inside the

Golconda Fort in Telangana

Q29) Answer a

Explanation

Silappadikaram is a famous Tamil epic

which was written by Ilango around

1800 years ago It is a story of a

merchant named Kovalan who fell in

love with a courtesan named Madhavi

Manimekalai tells the story of the

daughter of Kovalan and Madhavi

Q30) Answer (a)

Explanation

Charaka is the author of Charaka

Samhita which is an important work of

Ayurveda and medicines

Brahmaguptarsquos fame rests mostly on his

Brahma-sphuta-siddhanta which was

an astronomical work It was translated

into Arabic in Baghdad and had a major

impact on Islamic mathematics and

astronomy

Late in his life Brahmagupta wrote

Khandakhadyaka which was an

RAUSIAS-FC19E1003 44

astronomical handbook that employed

Aryabhatarsquos system of starting each day

at midnight

Q31) Answer (c)

Explanation

Amir Khusrau was a famous sufi

musician poet and scholar In 1318 he

noted that there was different language

in every region of this land (Hindustan)

Lahori Kashmiri Dvarsamudri (in

Southern Karnataka) Telangana (in

Andhra Pradesh) Gujari (in Gujarat)

Marsquobari (in Tamil Nadu) Awadhi (in

eastern Uttar Pradesh) and Hindawai (in

the area around in Delhi) etc He went

to explain that Sanskrit did not belong

to any region and that only brahmans

knew it

Q32) Answer c

Explanation

Hiranyagarbha refers to the golden

womb When this ritual was performed

with the help of Brahmanas it was

thought to lead to the rebirth of the

sacrificer as a Khastriya

Q33) Answer d

Explanation

Kadamai refers to a tax on land

revenue

Gwalior Prashasti describes the exploits

of Nagabhata who was a Pratihara king

Q34) Answer b

Explanation

Rajatarangini is a Sanskrit text written

by Kalhana in the 12th century

It was historical chronicle of early India

It is justifiably considered to be the best

and most authentic work of its kind

It covers the entire span of history in

the Kashmir region from the earliest

times to the date of its composition

Q35) Answer c

Explanation

ldquoUrrdquo was the general assembly of the

village ldquoUrrdquo consisted of all the

taxpaying residents of an ordinary

village

Q36) Answer (a)

Explanation

Tarikh was a form of history writing in

the Delhi Sultanate The authors of

tawarikhs were learned men which

included secretaries administrators etc

Q37 Answer (a)

Explanation

Alauddin chose to pay his soldiers salaries in cash rather than iqtas The soldiers would buy their supplies from merchants in Delhi and it was thus feared that merchants would raise their prices To stop this Alauddin controlled the prices of goods in Delhi Prices were carefully surveyed by officers and merchants who did not sell at the prescribed rates were punished

Q38) Answer (d)

Explanation

Delhi first became the capital of a

kingdom under the Tomara Rajputs

who were defeated in the middle of the

twelfth century by the Chauhans (also

referred to as Chahamanas) of Ajmer

It was under the Tomaras and

Chauhans that Delhi became an

important commercial centre Many rich

Jaina merchants lived in the city and

constructed several temples Coins

minted here called dehliwal had a wide

circulation

Q39) Answer (c)

Explanation

Moth ki Masjid was built in the reign of

Sikandar Lodi by his minister

Begumpuri mosque built in the reign of

Muhammad Tughluq was the main

mosque of Jahanpanah the ldquoSanctuary

of the Worldrdquo and his new capital in

Delhi

Quwwat al ndash Islam mosque was

enlarged by Iltutmish and Alauddin

Khalji The minar was built by three

Sultansndash Qutbuddin Aybak Iltutmish

and Firuz Shah Tughluq

RAUSIAS-FC19E1003 45

Q40) Answer (c)

Explanation

Under the Mughals mansabdar was

referred to an individual who held a

mansab ie rank and he received his

salary as revenue assignments called

jagirs

Q41) Ans (b)

The Quit India Movement was a

spontaneous revolt of people against

British rule

The All India Congress Committee met

at Bombay on 8 August 1942 It passed

the famous resolution Quit India and

proposed the starting of a non-violent

mass struggle under Gandhis

leadership to achieve this aim But on

the very next day Gandhi and other

eminent leaders of the Congress were

arrested The Congress was once again

declared illegal

Q42) Ans (c)

The Simon Commission refers to a

group of seven MPs from the United

Kingdom constituted to suggest

constitutional reforms for British India

The Commission consisted of only

British members headed by one of the

senior British politicians Sir John

Simon

So the people of India agitated against

the arrival of Simon Commission

Q43) Ans (a)

He was widely known for his

unfavourable opinion of the economic

consequences of the British rule in

India

In his many writings and speeches and

especially in Poverty and Un-British

Rule in India Naoroji argued that India

was too highly taxed and that its wealth

was being drained away to England

He did not interpret the ancient Indian

texts and restored the self-confidence of

Indians And also he did not stress the

need for eradication of all the social

evils before anything else

Q44) Ans (c)

In August 1932 Prime Minister

MacDonald announced his Communal

Award Great Britainrsquos unilateral

attempt to resolve the various conflicts

among Indiarsquos many communal

interests

The award which was later

incorporated into the act of 1935

expanded the separate-electorate

formula reserved for Muslims to other

minorities including Sikhs Indian

Christians Anglo-Indians Europeans

distinct regional groups Gandhi

undertook a ldquofast unto deathrdquo against

that offer which he viewed as a

nefarious British plot to divide the

Indian society

Q45) Ans (b)

In British India apart from existing

imports and exports there was also a

particular amount of money which

colonial India contributed towards

administration maintenance of the

army war expenses pensions to retired

officers and other expenses accrued by

Britain towards maintenance of her

colony These were known as Home

charges and were paid for almost

entirely by India

The Home charges was made of

following components-

- Interest payable on Indian debt

- Dividend to shareholders of East

India Company

- Funds used to support the India

Office in London

- Funds used to pay salaries and

pensions of British personnel

engaged in India

- Interest on the railways

- Civil and military charges

- Store purchases in England

Q46) Ans (b)

The Lahore session of the Indian

National Congress was held in 1929

under the Presidentship of Jawaharlal

Nehru

The Lahore session of the Indian

National Congress witnessed significant

RAUSIAS-FC19E1003 46

developments in the Indian national

movement

- First the election of Jawaharlal

Nehru to the post of Presidentship of

the Congress was a clear indication

of the growing strength of the

Leftists in the Congress

- Secondly it was in this session that

the Congress for the first time raised

the demand for complete

independence Such demand was

not raised from the Congress

platform earlier

Q47) Ans (b)

It did not provide for separate

electorates for any community or

weightage for minorities However it did

allow for the reservation of minority

seats in provinces having minorities of

at least ten per cent but this was to be

in strict proportion to the size of the

community

There was no provision for complete

Independence for India

Q48) Ans (c)

The religion of early Vedic Aryans was

primarily of worship of nature and

Yajnas

The early Aryan religion was kind of

nature worship Actually the forces

around them which they could not

control or understand were invested

with divinity and were personified as

male or female gods And they

performed some Yajnas also

Q49) Ans (b)

The roads and river-routes were not

immune from robbery It is notable that

Yuan Chwang (Hiuen Tsang) was

robbed of his belongings during

Harshvardanarsquos period

Q50) Ans (c)

Q51) Ans (b)

Purandara Dasa was a saint and great

devotee of Lord Krishna

There is much speculation about where

Purandara Dasa regarded as the

Pitamaha of Carnatic music was born

Recently an expert committee

constituted by the Kannada University

Hampi has come to the conclusion that

Kshemapura Shivamogga district

Karnataka is the birth place of

Purandara Dasa

Q52) Ans (c)

Sri Tyagaraja Sri Shyama Shastry and Sri Muthuswami Dikshitar are considered the trinity of Carnatic music and with them came the golden age in Carnatic music in the 18th-19th

century

Q53) Ans d)

Recently a rare sarcophagus (stone

coffin) which is 2000 years old from the

Iron AgendashMegalithic era was discovered

from a rock-cut cave at Viyur village of

Kollam near Koyilandy in Kozhikode

district Kerala

The coffin containing bone fragments

was found during an excavation ldquoSo

far such a rare finding has been

discovered only from two sites

in Kerala Both these sarcophagi were

recovered from Megalithic sites at

Chevayur and Atholi also in Kozhikode

district

Q54) Ans a)

The megalithic culture in South India was a full-fledged Iron Age culture

Q55) Ans d)

The Cholas Pandyas and Keralaputras

(Cheras) mentioned in Ashokan

inscriptions were probably in the late

megalithic phase of material culture

Q56) Ans d)

Q57) Ans (b)

Raj Kumar Shukla followed Gandhiji all

over the country to persuade him to

come to Champaran to investigate the

problem associated with tinkathia

system

RAUSIAS-FC19E1003 47

Brij Kishore Rajendra Prasad Mahadev

Desai and Narhari Parikh accompanied

Gandhi ji during the Champaran

Satyagraha

Q58) Ans (b)

The Satvahanas started the practice of granting tax-free villages to brahmanas and Buddhist monks

Q59) Ans c)

The objectives of the Programme are

listed as under

- Developing basic tourism

infrastructure

- Promoting cultural and heritage

value of the country to generate

livelihoods in the identified regions

- Enhancing the tourist attractiveness

in a sustainable manner by

developing world-class

infrastructure at the heritage

monument sites

- Creating employment through active

involvement of local communities

- Harnessing tourism potential for its

effects on employment generation

and economic development

- Developing sustainable tourism

infrastructure and ensuring proper

Operations and maintenance

therein

Q60) Ans (b)

The Tribal Cooperative Marketing

Development Federation of India

(TRIFED) came into existence in 1987

It is a national-level apex organization

functioning under the administrative

control of Ministry of Tribal Affairs

Govt of India

TRIFED has its registered and Head

Office located in New Delhi

Q61) Ans (c)

Premchandrsquos novels include

Premashram Rangabhumi Ghaban

Karmabhumi and Godan

Gora is a novel written by Rabindranath

Tagore

138th birth anniversary of Munshi

Premchand was celebrated across the

country

Q62) Ans (b)

Giddha is a traditional pastoral dance

performed by the women of the Punjab

India and Pakistan at festival times

and at the sowing and reaping of the

harvest

By this dance the Punjabi women

reveal their joy expel their suppressed

feelings in a male dominated society

through the performance of Giddha

Since this dance has nothing to do with

men only women can participate in it

During the Teej celebrations Giddha

dance is celebrated in Punjab every

year Teej is a generic name for a

number of festivals that are celebrated

by women in some parts of India

Q63) Ans (a)

Dara Shukoh wrote the remarkable

work called ldquoMajma-ul-Bahrainrdquo or the

ldquoThe confluence of two seasrdquo

The Vice President of India Shri M

Venkaiah Naidu has said that Prince

Dara Shukohrsquos writings can come as a

refreshing source for infusing peace and

harmony He was addressing the

gathering after visiting the exhibition

that showcases the forgotten Prince of

yesteryears Dara Shukoh organized by

Mr Francois Gautier at Indira Gandhi

National Centre for the Arts in New

Delhi

Q64) Ans (c)

The statue Gommateshwara is

dedicated to the Jain God Bahubali

It is a monolithic statue

President Ram Nath Kovind

inaugurated the grand anointing

ceremony mdash Mahamastakabhisheka mdash

held once in 12 years at

Shravanabelagola (Karnataka)

Q65) Ans (c)

Prachi Valley had come up around the

Prachi river Prachi Valley gradually

disappeared

RAUSIAS-FC19E1003 48

The Prachi river originates from

Bhubaneswar

It is a tributary of the Mahanadi and

flows through the districts of Puri

Khurda Cuttack and Jagatsinghpur

and the entire region of the river is

termed as the Prachi Valley

It falls into the Bay of Bengal

Archaeological evidence shows that the

Prachi Valley Civilisation predates both

Harappa and Mohenjo-Daro

The Prachi river originates from

Bhubaneswar

Q66) Ans (d)

These monuments are located in

Chhatarpur district Madhya Pradesh

within Vindhya mountain range

Q67) Ans (a)

The book lsquoThoughts on Pakistanrsquo was

written by Dr BR Ambedkar

On the occasion of the birth anniversary

of Dr BR Ambedkar the president of

India pays homage to this icon of India

In 1924 he founded the Depressed

Classes Institute (Bahishkrit Hitkarini

Sabha) and in 1927 the Samaj Samata

Sangh

Another area of attention for Ambedkar

was education For its spread among

the low classes he set up a network of

colleges by the name of Peoples

Education Society and founded hostels

Q68) Ans(b)

Mehrgarh is a famous Neolithic

settlement in the Indian subcontinent

which is situated in Baluchistan

province Pakistan

A pre-historic rock art site is discovered

in the vast expanse of limestone blocks

on the eastern banks of Naguleru river

near Dachepalli (Andhra Pradesh) It

has thrown light on the Neolithic

civilisation that flourished in Guntur

(Andhra Pradesh) during 1500-2000

BC

Q69) Ans (c)

The 12th and the 13th centuries saw

the emergence of the Kakatiyas They

were at first the feudatories of the

Western Chalukyas of Kalyana Initially

they ruled over a small territory near

Warangal (Telangana)

They introduced Nayakships which was

later adopted and developed by the

Rayas of Vijayanagara

Q70) Ans (a)

The fast had effect of putting pressure

on mill owners who finally agreed to

give the workers a 35 per cent increase

in wages

Google celebrated with a doodle the

132nd birth anniversary of Anasuya

Sarabhai who played a pioneering role

in Indiarsquos labour movement

Q71) Ans (d)

The UNESCOrsquos list of the representative

list of the intangible cultural heritage of

humanity from India are

- Koodiyattam Sanskrit Theatre of

Kerala

- Mudiyettu ritual theatre and dance

drama of Kerala

- Tradition of Vedic Chanting

- Kalbelia folk songs and dances of

Rajasthan

- Ramlila Traditional Performance of

the Ramayana

- Sankirtana ritual singing

drumming and dancing of Manipur

- Ramman religious festival and

ritual theatre of the Garhwal

Himalayas India

- Traditional brass and copper craft of

utensil making among the Thatheras

of Jandiala Guru Punjab India

- Chhau dance classical Indian dance

originated in the eastern Indian

states

- Buddhist chanting of Ladakh

recitation of sacred Buddhist texts

in the trans-Himalayan Ladakh

region Jammu and Kashmir India

- Yoga

- Nouroz

- Kumbh Mela

RAUSIAS-FC19E1003 49

Q72) Ans(b)

The President of India Shri Ram Nath Kovind inaugurated the Hornbill Festival and State Formation Day celebrations of Nagaland in Kisama

The festival is named after the Indian hornbill the large and colourful forest bird which is displayed in the folklore of most of the states tribes

The major recognized tribes of Nagaland are Angami Ao Chakhesang Chang

Kuki Rengma and Zeling etc

Onge Jarawa and Sentinelese are the

tribes of Andman amp Nicobar Islands

Q73) Ans (c)

The Rashtrakutas rule in the Deccan lasted for almost two hundred years till the end of the tenth century The Rashtrakutas rulers were tolerant in their religious views and patronized not only Shaivism and Vaishnavism but

Jainism as well

The famous rock-cut temple of Shiva at Ellora was built by one of the Rashtrakutas kings Krishna I in the ninth century His successor Amoghavarsha was a Jain but he also

patronized other faiths

The Rashtrakutas allowed Muslims traders to settle and permitted Islam to

be preached in their dominions

Recently increasing defacement at the prehistoric rock paintings of Pandavulagutta Telangana has created a cause for grave concern It can spoil

the prehistoric rock

Pandavulagutta is home to

- Painted rock shelters dating to

10000 BC-8000 BC

- An 8th century inscription of the

Rashtrakuta period and

- Painted frescoes from the 12th century Kakatiya empire

Q74) Ans (b)

In 1828 Raja Ram Mohan Roy founded a new religious society the Brahma Sabha later known as the Brahmo

Samaj

Debendranath Tagore headed the Tattvabodhini Sabha which was

engaged in search of spiritual truth

Its purpose was to purify Hinduism and to preach monotheism or belief in one God

The new society was to be based on the twin pillars of reason and the Vedas and

Upanishads

Recently Sadharan Brahmo Samaj (SBS) has entered into a legal battle with the West Bengal government due

to some legal issue

Q75) Ans (c)

The Chishti order was established in India by Khwaja Moinuddin Chishti who came to India around 1192 The Chishtirsquos are considered to be the most influential of the groups of Sufis who migrated to India in the late twelfth century They adapted successfully to the local environment and adopted several features of Indian devotional

traditions

The historical dargah of Sufi mystic Khwaja Moinuddin Chishti in Ajmer is all set to get a facelift This 13 th century dargah has been included among the Swachh Iconic Places a clean-up initiative focused on iconic

heritage spiritual and cultural places

Page 15: GENERAL STUDIES (PAPER I) · Test is part of Rau’s IAS Test series for Preliminary Exam 2019 FOUNDATION + CURRENT AFFAIRS GENERAL STUDIES (PAPER –I) FOUNDATION TEST –III TOPIC:

RAUSIAS-FC19E1003 15

Q38) Which of the following statements

isare correct

1 Delhi first became the capital of a

kingdom under Qutubuddin

Aibak

2 Dehliwal coins were minted by the

Mughals

Select the correct answer using the code

given below

(a) 1 only

(b) 2 only

(c) Both 1 and 2

(d) Neither 1 nor 2

Q39) Consider the following pairs

1 Moth ki Masjid- Sikander Lodi

2 Begumpuri mosque- Firuz Shah

Tughluq

3 Quwwat al ndash Islam- Qutubuddin

Aibak

Which of the above pairs isare correct

(a) 1 and 2 only

(b) 2 and 3 only

(c) 1 and 3 only

(d) 1 2 and 3

Q40) Which of the following statements

isare correct

1 Mansabdars received their salaries

as revenue assignments called

jagirs

2 The mansabdarrsquos military

responsibilities required him to

maintain a specified number of

sawar or cavalrymen

Select the correct answer using the code

given below

(a) 1 only

(b) 2 only

(c) Both 1 and 2

(d) Neither 1 nor 2

Q41) Which one of the following observations

is not true about the Quit India

Movement of 1942

(a) It was a non-violent movement

(b) It was led by Mahatma Gandhi

(c) It was a spontaneous movement

(d) It did not attract the labour class

in general

Q42) The people of India agitated against the

arrival of the Simon Commission

because

(a) Indians never wanted the review of

the working of the Act of 1919

(b) Simon Commission recommended

the abolition of dyarchy in the

Provinces

(c) there was no Indian member in the

Simon Commission

(d) the Simon Commission suggested

the partition of the country

Q43) Consider the following statements

The most effective contribution made by

Dadabhai Naoroji to the cause of Indian

National Movement was that he-

1 exposed the economic exploitation

of India by the British

2 interpreted the ancient Indian

texts and restored the self-

confidence of Indians

3 stressed the need for eradication of

all the social evils before anything

else

Which of the statements given above

isare correct

(a) 1 only

(b) 2 and 3 only

(c) 1 and 3 only

(d) 1 2 and 3

RAUSIAS-FC19E1003 16

Q44) महातमा गाोरी ि 1932 म आमरर अिशि नकया था

कय ोनक

(a) ldquoग िमज सममििrdquo (The Round Table

Conference) भारतीय राजिीनतक

आकाोकषाओो क परा करि म असफि रहा था

(b) काोगरस और मखसलम िीग म मतभद थ

(c) रामस मकड िालड (Ramsay Macdonald)

ि ldquoसाोपरदानयक परसकारrdquo (The Communal

Award) की घ िरा की थी

(d) ldquoसनििय अिजञा आोद ििrdquo (The Civil

Disobedience Movement) असफि रहा

था

Q45) भारत म औपनििनशक शासि की अिनर क सोदभण म

भारत स रि क बनहगणमि का एक महतवपरण भाग गह

शलक (Home Charges) था निमननिखित म स

कौि-सास क ि गह शलक म सखममनित नकया गया

थानकय गए थ

1 िोदि म भारत कायाणिय क निए उपय ग नकय

जाि िािा क ि

2 भारत म नियकत नबरनटश कनमणय ो क िति और

पशि का भगताि करि क निए उपय ग नकय

जाि िािा क ि

3 अोगरज ो क दवारा भारत क बाहर यदध ो क निए

उपय ग नकय जाि िािा क ि

िीच नदए गए कट का परय ग कर सही उततर चनिए

(a) किि 1

(b) किि 1 और 2

(c) किि 2 और 3

(d) 1 2 और 3

Q46) सवतोतरता आोद िि क इनतहास म भारतीय राषटर ीय

काोगरस का 1929 का सतर महतवपरण ह कय ोनक इसम

(a) काोगरस क उददशय क रप म सथािीय सरकार

की पराखपत की घ िरा की गई थी

(b) परण सवराज की पराखपत क काोगरस क िकषय क

रप म अपिाया गया था

(c) असहय ग आोद िि शर नकया गया था

(d) िोदि म ldquoग ि मर सममििrdquo (The Round

Table Conference) म भाग िि का निरणय

निया गया था

Q47) भारतीय सवतोतरता सोगराम क सोदभण म िहर ररप टण

क दवारा निमननिखित म स नकसकी नसफाररश की गई

थीनकिकी नसफाररश की गई थी ो

1 भारत क निए परण सवतोतरता

2 अलपसोखयक ो क निए सीट ो क आरकषर क

निए सोयकत नििाणचक मोडि

3 सोनिराि म भारत क ि ग ो क निए मौनिक

अनरकार ो का परािराि

िीच नदए गए कट का परय ग कर सही उततर चनिए

(a) किि 1

(b) किि 2 और 3

(c) किि 1 और 3

(d) 1 2 और 3

Q48) आरो नभक िनदक आयो का रमण मखय रप स था

(a) भखकत

(b) मनतण पजा और यजञ

(c) परकनत की पजा और यजञ

(d) परकनत की पजा और भखकत

RAUSIAS-FC19E1003 17

Q44) Mahatma Gandhi undertook fast unto

death in 1932 mainly because

(a) The Round Table Conference failed

to satisfy Indian political

aspirations

(b) The Congress and Muslim League

had differences of opinion

(c) Ramsay Macdonald announced the

Communal Award

(d) The Civil Disobedience Movement

failed

Q45) With reference to the period of colonial

rule in India ldquoHome Chargesrdquo formed

an important part of drain of wealth

from India Which of the following funds

constituted ldquoHome Chargesrdquo

1 Funds used to support the India

Office in London

2 Funds used to pay salaries and

pensions of British personnel

engaged in India

3 Funds used for waging wars

outside India by the British

Select the correct answer using the code

given below

(a) 1 only

(b) 1 and 2 only

(c) 2 and 3 only

(d) 1 2 and 3

Q46) The 1929- Session of Indian National

Congress is of significance in the history

of the Freedom Movement because the-

(a) attainment of Self-Government

was declared as the objective of

the Congress

(b) attainment of Poorna Swaraj was

adopted as the goal of the

Congress

(c) Non-Cooperation Movement was

launched

(d) decision to participate in the

Round Table Conference in

London was taken

Q47) With reference to the period of Indian

freedom struggle which of the following

waswere recommended by the Nehru

report

1 Complete Independence for India

2 Joint electorates for reservation of

seats for minorities

3 Provision of fundamental rights for

the people of India in the

Constitution

Select the correct answer using the code

given below

(a) 1 only

(b) 2 and 3 only

(c) 1 and 3 only

(d) 1 2 and 3

Q48) The religion of the early Vedic Aryans was primarily of

(a) Bhakti

(b) image worship and Yajnas

(c) worship of nature and Yajnas

(d) worship of nature and Bhakti

RAUSIAS-FC19E1003 18

Q49) भारत की यातरा करि िाि चीिी यातरी यआि चिाोग

(हयएि साोग) ि समकािीि भारत की सामानय

खसथनतय ो और सोसकनत क दजण नकया था इस सोदभण म

निमननिखित कथि ो म स कौि-सास सही हह

1 सड़क और िदी-मागण (जि-मागण) डकती स

परण रप स सरनकषत थ

2 जहा तक अपरार ो क निए दणड की बात ह

उसक निए नकसी भी वयखकत की निदोिता

अथिा उसक अपरार क निराणररत करि क

निए अनि जि और निि परि क माधयम क

सारि थ

3 वयापाररय ो क घाट ो और परनतबोर सटशि ो पर

शलक ो का भगताि करिा पड़ता था

िीच नदए गए कट का परय ग कर सही उततर चनिए

(a) किि 1

(b) किि 2 और 3

(c) किि 1 और 3

(d) 1 2 और 3

Q50) नसोर घाटी सभयता क सोदभण म निमननिखित कथि ो पर

निचार कीनजए

1 यह मखय रप स एक रमणनिरपकष सभयता थी

तथा हािाोनक इसम रानमणक ततव मौजद था

िनकि िह परनतिश पर हािी िही ो था

2 इस काि क दौराि भारत म कपास का परय ग

कपड़ा बिाि क निए नकया जाता था

उपयणकत कथि ो म स कौि-सास सही हह

(a) किि 1

(b) किि 2

(c) 1 और 2 द ि ो

(d) ि त 1 ि ही 2

Q51) परोदर दास क सोदभण म निमननिखित कथि ो पर निचार

कीनजए

1 परोदर दास एक सोत और भगिाि नशि क एक

महाि भकत थ

2 ि एक सोगीतकार गायक और किाणटक सोगीत

क मखय सोसथापक-परसतािक ो म स एक थ

उपयणकत कथि ो म स कौि-सास सही हह

(a) किि 1

(b) किि 2

(c) 1 और 2 द ि ो

(d) ि त 1 ि ही 2

Q52) निमननिखित म स कौि-सास वयखकत किाणटक सोगीत

की नतरमनतण म शानमि हह

1 बािामरिी कषणा

2 शरी शयाम शासतरी

3 शरी मथसवामी दीनकषतर

िीच नदए गए कट का परय ग कर सही उततर चनिए

(a) किि 1

(b) किि 2

(c) किि 2 और 3

(d) 1 2 और 3

Q53) चियर (Chevayur) और अथ िी (Atholi) म खसथत

महापािार सथि निमननिखित म स नकस राजय म खसथत

(a) तनमििाड

(b) किाणटक

(c) पनिम बोगाि

(d) करि

RAUSIAS-FC19E1003 19

Q49) The Chinese traveller Yuan Chwang

(Hiuen Tsang) who visited India

recorded the general conditions and

culture of India at that time In this

context which of the following

statements isare correct

1 The roads and river-routes were

completely immune from robbery

2 As regards punishment for

offences ordeals by fire water and

poison were the instruments for

determining the innocence or guilt

of a person

3 The tradesmen had to pay duties

at ferries and barrier stations

Select the correct answer using the code

given below

(a) 1 only

(b) 2 and 3 only

(c) 1 and 3 only

(d) 1 2 and 3

Q50) Regarding the Indus Valley Civilization

consider the following statements

1 It was predominantly a secular

civilization and the religious

element though present did not

dominate the scene

2 During this period cotton was

used for manufacturing textiles in

India

Which of the statements given above

isare correct

(a) 1 only

(b) 2 only

(c) Both 1 and 2

(d) Neither 1 nor 2

Q51) Consider the following statements

regarding Purandara Dasa

1 Purandara Dasa was a saint and

great devotee of Lord Shiva

2 He was a composer singer and

one of the chief founding-

proponents of the Carnatic music

Which of the statements given above

isare correct

(a) 1 only

(b) 2 only

(c) Both 1 and 2

(d) Neither 1 nor 2

Q52) Which of the following persons isare

included in the trinity of Carnatic

music

1 Balamurali Krishna

2 Sri Shyama Shastry

3 Sri Muthuswami Dikshitar

Select the correct answer using the code

given below

(a) 1 only

(b) 2 only

(c) 2 and 3 only

(d) 1 2 and 3

Q53) Megalithic sites at Chevayur and Atholi

are located in which of the following

states

(a) Tamil Nadu

(b) Karnataka

(c) West Bengal

(d) Kerala

RAUSIAS-FC19E1003 20

Q54) निमननिखित कथि ो पर निचार कीनजए

1 महापािानरक ि ग कबर ो म िसतएो दफिात थ

2 दनकषर भारत म महापािार सोसकनत एक परण

निकनसत तामर यगीि सोसकनत थी

उपयणकत कथि ो म स कौि-सास सही हह

(a) किि 1

(b) किि 2

(c) 1 और 2 द ि ो

(d) ि त 1 ि ही 2

Q55) निमननिखित म स कौि-स सामराजयसामराजय ो का

अश क क अनभिि ो म उललि नकया गया ह

1 च ि

2 पाणडय

3 करिपतर (चर)

िीच नदए गए कट का परय ग कर सही उततर चनिए

(a) किि 1

(b) किि 1 और 2

(c) किि 3

(d) 1 2 और 3

Q56) भीमा-क रगाोि का यदध को पिी क सनिक ो और

बाजीराि नदवतीय क िततव म एक शखकतशािी पशिा

सिा (मराठ ो) क मधय िड़ा गया था यह यदध

निमननिखित म स नकसका नहससा था

(a) परथम आोगल-मराठा यदध का

(b) नदवतीय आोगल-मराठा यदध का

(c) ततीय आोगल-मसर यदध का

(d) ततीय आोगल-मराठा यदध का

Q57) निमननिखित कथि ो पर निचार कीनजए

1 महादि दसाई ि गाोरीजी क चोपारर आि तथा

नतिकनथया पररािी स जड़ी समसया की जाोच

क निए रारी करि क निए दश भर म उिका

अिसरर नकया था

2 िरहरी पाररि चोपारर सतयागरह क दौराि

गाोरीजी क साथ थ

उपयणकत कथि ो म स कौि-सास सही हह

(a) किि 1

(b) किि 2

(c) 1 और 2 द ि ो

(d) ि त 1 ि ही 2

Q58) निमननिखित कथि ो पर निचार कीनजए

1 िनद राज-िोश ि बराहमर ो और बौदध मठराररय ो

क कर-मकत गाि अिदाि म दि की परथा

आरि की थी

2 सतिाहि ो की आनरकाररक भािा पराकत थी

उपयणकत कथि ो म स कौि-सास सही हह

(a) किि 1

(b) किि 2

(c) 1 और 2 द ि ो

(d) ि त 1 ि ही 2

Q59) एक निरासत क अपिाइए (अडॉपट ए हररटज ndash

Adopt a Heritage) पररय जिा क उददशय ो क

सनदभण म निमननिखित कथि ो पर निचार कीनजए

1 यह पररय जिा र रगार उतपादि और आनथणक

निकास क निए पयणटि कषमता का उि पर

परभाि का उपय ग करगी

2 यह पररय जिा निरासत सथि ो पर निशव सतरीय

आराररक सोरचिा निकनसत करक एक सतत

तरीक स पयणटक आकिणर म िखदध करगी

उपयणकत कथि ो म स कौि-सास सही हह

(a) किि 1

(b) किि 2

(c) 1 और 2 द ि ो

(d) ि त 1 ि ही 2

RAUSIAS-FC19E1003 21

Q54) Consider the following statements

1 Megalithic people buried goods in

graves

2 The megalithic culture in South

India was a full-fledged Copper

Age culture

Which of the statements given above

isare correct

(a) 1 only

(b) 2 only

(c) Both 1 and 2

(d) Neither 1 nor 2

Q55) Which of the following kingdoms isare

mentioned in the Ashokan inscriptions

1 Cholas

2 Pandyas

3 Keralaputras (Cheras)

Select the correct answer using the code

given below

(a) 1 only

(b) 1 and 2 only

(c) 3 only

(d) 1 2 and 3

Q56) The Battle of Bhima-Koregaon was

fought between the soldiers of the

Company and the strong Peshwa army

(Marathas) under Bajirao II This war

was a part of the

(a) First Anglo-Maratha war

(b) Second Anglo-Maratha war

(c) Third Anglo- Mysore war

(d) Third Anglo-Maratha war

Q57) Consider the following statements

1 Mahadev Desai followed Gandhiji all over the country to persuade him to come to Champaran to investigate the problem associated

with tinkathia system

2 Narhari Parikh accompanied Gandhi ji during the Champaran

Satyagraha

Which of the statements given above isare correct

(a) 1 only

(b) 2 only

(c) Both 1 and 2

(d) Neither 1 nor 2

Q58) Consider the following statements

1 The Nanda Dynasty started the practice of granting tax-free villages to brahmanas and

Buddhist monks

2 The official language of the Satavahanas was Prakrit

Which of the statements given above

isare correct

(a) 1 only

(b) 2 only

(c) Both 1 and 2

(d) Neither 1 nor 2

Q59) Consider the following statements about the objectives of the lsquoadopt a heritagersquo

project

1 It will harness tourism potential for its effects on employment generation and economic

development

2 It will enhance the tourist attractiveness in a sustainable manner by developing world class infrastructure at heritage sites

Which of the statements given above

isare correct

(a) 1 only

(b) 2 only

(c) Both 1 and 2

(d) Neither 1 nor 2

RAUSIAS-FC19E1003 22

Q60) ldquoभारतीय जिजातीय सहकारी निपरि निकास सोघrdquo

(The Tribal Co-operative Marketing

Development Federation of India - TRIFED)

क सोदभण म निमननिखित कथि ो पर निचार कीनजए

1 यह एक राषटर ीय सतर का शीिण सोगठि ह ज

भारत सरकार क गह मोतरािय क परशासनिक

नियोतरर क अरीि काम कर रहा ह

2 इसका मखय उददशय दश म जिजातीय ि ग ो

का सामानजक-आनथणक निकास करिा ह

उपयणकत कथि ो म स कौि-सास सही हह

(a) किि 1

(b) किि 2

(c) 1 और 2 द ि ो

(d) ि त 1 ि ही 2

Q61) निमननिखित म स कौि-सास उपनयास परमचोद क

दवारा नििा गया हनिि गए ह

1 रोगभनम

2 ग दाि

3 ग रा

िीच नदए गए कट का परय ग कर सही उततर चनिए

(a) किि 1

(b) किि 2

(c) किि 1 और 2

(d) 1 2 और 3

Q62) नगदधा ितय क सोदभण म निमननिखित कथि ो पर निचार

कीनजए

1 नगदधा नबहार की मनहिाओो क दवारा तयौहार क

समय और फसि की बिाई तथा कटाई क

अिसर पर नकया जाि िािा एक पारोपररक

दहाती ितय ह

2 इस ितय क दवारा मनहिाऐो अपिी परसननता

परकट करती ह तथा नगदधा क परदशणि क

माधयम स परि िचणसव िाि समाज म

मनहिाओो की दबी हई भाििाओो क परकट

करती ह

उपयणकत कथि ो म स कौि-सास सही हह

(a) किि 1

(b) किि 2

(c) 1 और 2 द ि ो

(d) ि त 1 ि ही 2

Q63) निमननिखित कथि ो पर निचार कीनजए

1 मलला शाह बदखशी दारा नशक ह क

आधयाखतमक गर थ

2 औरोगरब ि मजम-उि-बहरीि या द समदर ो

का सोगम िामक उललििीय रचिा नििी थी

3 दारा नशक ह क अपि पिणज अकबर क गर ो

क उततरानरकारी क रप म दिा गया था

नजसम उसि रानमणक बहििाद और समनवयता

क बढ़ािा नदया था

उपयणकत कथि ो म स कौि-सास सही हह

(a) किि 1 और 3

(b) किि 2

(c) किि 1 और 2

(d) 1 2 और 3

RAUSIAS-FC19E1003 23

Q60) Consider the following statements about

the Tribal Cooperative Marketing

Development Federation of India

(TRIFED)

1 It is a national-level apex

organization functioning under the

administrative control of Ministry

of Home Affairs Government of

India

2 The main objective of TRIFED is

socio-economic development of

tribal people in the country

Which of the statements given above

isare correct

(a) 1 only

(b) 2 only

(c) Both 1 and 2

(d) Neither 1 nor 2

Q61) Which of the following novels isare

written by Premchand

1 Rangabhumi

2 Godan

3 Gora

Select the correct answer using the code

given below

(a) 1 only

(b) 2 only

(c) 1 and 2 only

(d) 1 2 and 3

Q62) Consider the following statements about

Giddha dance

1 Giddha is a traditional pastoral

dance performed by the women of

Bihar at festival times and at the

sowing and reaping of the harvest

2 By this dance the women reveal

their joy expel their suppressed

feelings in a male dominated

society through the performance of

Giddha

Which of the statements given above

isare correct

(a) 1 only

(b) 2 only

(c) Both 1 and 2

(d) Neither 1 nor 2

Q63) Consider the following statements

1 Mullah Shah Badakhshi was the

spiritual mentor of Dara Shukoh

2 Aurangzeb wrote the remarkable

work called ldquoMajma-ul-Bahrainrdquo or

the ldquoThe confluence of two seasrdquo

3 Dara Shukoh was seen as

inheriting the qualities of his

ancestor Akbar in that he

promoted religious pluralism and

syncretism

Which of the statements given above

isare correct

(a) 1 and 3 only

(b) 2 only

(c) 1 and 2 only

(d) 1 2 and 3

RAUSIAS-FC19E1003 24

Q64) निमननिखित कथि ो पर निचार कीनजए

1 ग मतशवर परनतमा निोधयनगरी पहाड़ी पर खसथत ह

2 शरिरबिग िा िह सथाि ह जहाो मौयण िोश क

सोसथापक चोदरगपत मौयण अपि नसोहासि क

तयागि क बाद जि तपसवी बि गए थ

उपयणकत कथि ो म स कौि-सास सही हह

(a) किि 1

(b) किि 2

(c) 1 और 2 द ि ो

(d) ि त 1 ि ही 2

Q65) निमननिखित कथि ो पर निचार कीनजए

1 पराताखतवक साकषय स पता चिता ह नक पराची

घाटी सभयता हड़पपा और म हिज दाड़ द ि ो

की पिणिती ह

2 पराची िदी भििशवर स निकिती ह

उपयणकत कथि ो म स कौि-सास सही हह

(a) किि 1

(b) किि 2

(c) 1 और 2 द ि ो

(d) ि त 1 ि ही 2

Q66) निमननिखित कथि ो म स कौि-सास सही हह

1 िजराह क समारक ो क समह का निमाणर

चोदि राजिोश क शासिकाि क दौराि हआ

था

2 य समारक हररिोदर पिणत शरोििा म खसथत ह

3 म रक क यातरी इबन बतता ि अपि सोसमरर ो

म िजराह क मोनदर ो की यातरा का उललि

नकया था तथा इन काजराण िाम स समब नरत

नकया था

िीच नदए गए कट का परय ग कर सही उततर चनिए

(a) किि 1

(b) किि 1 और 2

(c) किि 2 और 3

(d) किि 1 और 3

Q67) निमननिखित कथि ो म स कौि-सास सही हह

1 डॉ बी आर अमबडकर ि दी एनिनहिशि

ऑफ़ कासट (The Annihilation of Caste)

नििी थी नजसम उन ोि नहोद रमण म िोशािगत

पजारी की परथा क उनमिि की आिशयकता

पर बि नदया था

2 डॉ राजदर परसाद ि थॉटस ऑि पानकसताि

(Thoughts on Pakistan) िामक पसतक

नििी थी

िीच नदए गए कट का परय ग कर सही उततर चनिए

(a) किि 1

(b) किि 2

(c) 1 और 2 द ि ो

(d) ि त 1 ि ही 2

Q68) निमननिखित कथि ो म स कौि-सास सही हह

1 महरगढ़ भारतीय उपमहादवीप म एक परनसदध

ििपािार बसती ह ज नसोर पराोत पानकसताि म

खसथत ह

2 बरणह म म कतत ो क उिक सवामी क साथ कबर ो

म दफिाया जाता था

िीच नदए गए कट का परय ग कर सही उततर चनिए

(a) किि 1

(b) किि 2

(c) 1 और 2 द ि ो

(d) ि त 1 ि ही 2

Q69) निमननिखित कथि ो म स कौि-सास सही हह

1 काकानटय मोनदर अनरकतर नशि क समनपणत

2 हिमक ोडा म हजार-सतोभ िाि मोनदर (The

Thousand-Pillared Temple) का निमाणर

काकानटय समराट रदर ि करिाया था

िीच नदए गए कट का परय ग कर सही उततर चनिए

(a) किि 1

(b) किि 2

(c) 1 और 2 द ि ो

(d) ि त 1 ि ही 2

RAUSIAS-FC19E1003 25

Q64) Consider the following statements

1 Gommateshwara Statue is located

on the Vindyagiri Hill

2 Shravanabelagola is the place

where Chandragupta Maurya the

founder of the Mauryan dynasty

became a Jain ascetic after

relinquishing his throne

Which of the statements given above

isare correct

(a) 1 only

(b) 2 only

(c) Both 1 and 2

(d) Neither 1 nor 2

Q65) Consider the following statements

1 Archaeological evidence shows

that the Prachi Valley Civilisation

predates both Harappa and

Mohenjo-Daro

2 The Prachi river originates from

Bhubaneswar

Which of the statements given above

isare correct

(a) 1 only

(b) 2 only

(c) Both 1 and 2

(d) Neither 1 nor 2

Q66) Which of the following statements

isare correct

1 The Khajuraho group of

monuments was built during the

rule of the Chandela dynasty

2 These monuments are located in

Harischandra mountain range

3 Ibn Battuta the Moroccan

traveller in his memoirs mentioned

visiting Khajuraho temples and

called them Kajarra

Select the correct answer using the code

given below

(a) 1 only

(b) 1 and 2

(c) 2 and 3

(d) 1 and 3

Q67) Which of the following statements

isare correct

1 Dr BR Ambedkar wrote the

Annihilation of Caste emphasising

the need to do away with the

practice of hereditary priesthood in

Hinduism

2 The book lsquoThoughts on Pakistanrsquo

was written by Dr Rajendra

Prasad

Select the correct answer using the code

given below

(a) 1 only

(b) 2 only

(c) Both 1 and 2

(d) Neither 1 nor 2

Q68) Which of the following statements

isare correct

1 Mehrgarh is a famous Neolithic

settlement in the Indian

subcontinent which is situated in

Sindh province Pakistan

2 At Burzahom dogs were buried

with their masters in their graves

Select the correct answer using the code

given below

(a) 1 only

(b) 2 only

(c) Both 1 and 2

(d) Neither 1 nor 2

Q69) Which of the following statements

isare correct

1 The Kakatiya temples are

dedicated mostly to Siva

2 The Thousand-Pillared Temple at

Hanamkonda was built by the

Kakatiya king Rudra

Select the correct answer using the code

given below

(a) 1 only

(b) 2 only

(c) Both 1 and 2

(d) Neither 1 nor 2

RAUSIAS-FC19E1003 26

Q70) निमननिखित कथि ो म स कौि-सास सही हह

1 अहमदाबाद नमि हड़ताि क दौराि महातमा

गाोरी ि शरनमक ो क पकष क मजबत करि क

निए आमरर अिशि नकया था

2 अिशि स नमि मानिक ो पर दबाि पड़ा था ज

अोततः शरनमक ो क िति म 15 परनतशत की िखदध

करि क निए सहमत हए थ

िीच नदए गए कट का परय ग कर सही उततर चनिए

(a) किि 1

(b) किि 2

(c) 1 और 2 द ि ो

(d) ि त 1 ि ही 2

Q71) निमननिखित म स नकसक नकिक भारत स यिसक

की माििता की अमतण साोसकनतक निरासत की

परनतनिनर सची (The UNESCOrsquos List of the

Representative List of the Intangible

Cultural Heritage of Humanity) म शानमि

नकया गया ह

1 मनडयटट

2 सोकीतणि

3 को भ मिा

िीच नदए गए कट का परय ग कर सही उततर चनिए

(a) किि 1 और 2

(b) किि 2 और 3

(c) किि 3

(d) 1 2 और 3

Q72) निमननिखित जिजानतय ो म स कौि-सीसी ो

जिजानतजिजानतया िागािड स सोबोनरत हह

1 अोगामी

2 ककी

3 जारिा

िीच नदए गए कट का परय ग कर सही उततर चनिए

(a) किि 1

(b) किि 1 औऔ 2

(c) किि 2

(d) 1 2 और 3

Q73) निमननिखित कथि ो म स कौि-सास सही हह

1 राषटर कट सामराजय की सथापिा दोनतदगण ि की थी

नजसि मानयाित म अपिी राजरािी की

सथापिा की थी

2 राषटर कट समराट अम घििण एक ििक था और

उस कनिताओो पर पहिी कननड़ पसतक नििि

का शरय नदया जाता ह

िीच नदए गए कट का परय ग कर सही उततर चनिए

(a) किि 1

(b) किि 2

(c) 1 और 2 द ि ो

(d) ि त 1 ि ही 2

Q74) निमननिखित कथि ो म स कौि-सास सही हह

1 कशब चोदर सि ि ततवब नरिी सभा की

अधयकषता की थी ज आधयाखतमक सतय की

ि ज म सोिि थी

2 बरहम समाज ि मािि गररमा पर बि नदया

मनतणपजा का निर र नकया और सती परथा जसी

सामानजक बराइय ो की आि चिा की

िीच नदए गए कट का परय ग कर सही उततर चनिए

(a) किि 1

(b) किि 2

(c) 1 और 2 द ि ो

(d) ि त 1 ि ही 2

Q75) निमननिखित कथि ो म स कौि-सास सही हह

1 भारत म नचशती नसिनसिा खवाजा म इिददीि

नचशती क दवारा सथानपत नकया गया था

2 नचशती परोपरा की एक परमि निशिता

आतमसोयम थी नजसम साोसाररक म ह स दरी

बिाए रििा शानमि था

िीच नदए गए कट का परय ग कर सही उततर चनिए

(a) किि 1

(b) किि 2

(c) 1 और 2 द ि ो

(d) ि त 1 ि ही 2

RAUSIAS-FC19E1003 27

Q70) Which of the following statements

isare correct

1 During the Ahmedabad Mill Strike

Mahatma Gandhi undertook a fast

unto death to strengthen the

workersrsquo resolve

2 The fast had effect of putting

pressure on mill owners who

finally agreed to give the workers a

15 per cent increase in wages

Select the correct answer using the code

given below

(a) 1 only

(b) 2 only

(c) Both 1 and 2

(d) Neither 1 nor 2

Q71) Which of the following are included in

the UNESCOrsquos list of the representative

list of the intangible cultural heritage of

humanity from India

1 Mudiyettu

2 Sankirtana

3 Kumbh Mela

Select the correct answer using the code

given below

(a) 1 and 2 only

(b) 2 and 3 only

(c) 3 only

(d) 1 2 and 3

Q72) Which of the following tribes isare

related to Nagaland

1 Angami

2 Kuki

3 Jarawa

Select the correct answer using the code

given below

(a) 1 only

(b) 1 and 2 only

(c) 2 only

(d) 1 2 and 3

Q73) Which of the following statements

isare correct

1 Rashtrakuta kingdom was founded by Dantidurga who established his capital at Manyakhet

2 Amoghavarsha a Rashtrakuta king was an author and is credited with writing the first

Kannada book on poetics

Select the correct answer using the code given below

(a) 1 only

(b) 2 only

(c) Both 1 and 2

(d) Neither 1 nor 2

Q74) Which of the following statements isare correct

1 Keshab Chandra Sen headed the Tattvabodhini Sabha which was engaged in search of spiritual truth

2 The Brahmo Samaj laid emphasis on human dignity opposed idolatry and criticized such social

evils as the practice of Sati

Select the correct answer using the code given below

(a) 1 only

(b) 2 only

(c) Both 1 and 2

(d) Neither 1 nor 2

Q75) Which of the following statements isare correct

1 The Chishti order was established in India by Khwaja Moinuddin

Chishti

2 A major feature of the Chishti tradition was austerity including maintaining a distance from the

worldly power

Select the correct answer using the code

given below

(a) 1 only

(b) 2 only

(c) Both 1 and 2

(d) Neither 1 nor 2

T e s t i s p a r t o f R a u rsquo s I A S T e s t s e r i e s f o r P r e l i m i n a r y E x a m 2 0 1 9

FOUNDATION + CURRENT AFFAIRS

GENERAL STUDIES (PAPER ndashI)

FOUNDATION TEST ndashIII

SUBJECT NCERT History Class VI-X + Current Affairs

Time Allowed 1frac12 Hours Maximum Marks 150

I NSTRUCT IONS

1 IMMEDIATELY AFTER THE COMMENCEMENT OF THE EXAMINATION YOU SHOULD CHECK

THAT THIS TEST BOOKLET DOES NOT HAVE ANY UNPRINTED OR TORN or MISSING PAGES OR

ITEMS ETC IF SO GET IT REPLACED BY A COMPLETE TEST BOOKLET

2 This Test Booklet contains 75 items (questions) Each item is printed both in Hindi and English

Each item comprises four responses (answers) You will select the response which you want to mark

on the Answer Sheet In case you feel that there is more than one correct response mark the

response which you consider the best In any case choose ONLY ONE response for each item

3 You have to mark all your responses ONLY on the separate Answer Sheet (OMR sheet) provided

Read the directions in the Answer Sheet

4 All items carry equal marks

5 Before you proceed to mark in the Answer Sheet the response to various items in the Test booklet

you have to fill in some particulars in the Answer Sheet as per instructions contained therein

6 After you have completed filling in all your responses on the Answer Sheet and the examination has

concluded you should hand over to the Invigilator only the Answer Sheet You are permitted to

take away with you the Test Booklet

7 Penalty for wrong answers

THERE WILL BE PENALTY FOR WRONG ANSWERS MARKED BY A CANDIDATE IN THE

OBJECTIVE TYPE QUESTION PAPERS

(i) There are four alternatives for the answer to every question For each question for which a

wrong answer has been given by the candidate one-third of the marks assigned to that

question will be deducted as penalty

(ii) If a candidate gives more than one answer it will be treated as a wrong answer even if one of

the given answers happens to be correct and there will be same penalty as above to that

question

(iii) If a question is left blank ie no answer is given by the candidate there will be no penalty for

that question

T h i s t e s t i s p a r t o f R a u rsquo s I A S T e s t s e r i e s f o r P r e l i m i n a r y E x a m 2 0 1 9

Test Code

FC19E1003

FC19H1003 29

Answers and Explanations of

NCERT History Class VI-X + Current Affairs (FC19E1003)

Q1) उततर (c)

सपषटीकरण

- ऋगवद म दविय ो और दिताओो क समवपित एक

हजार स अविक सत तर (शल क) ह

- य शल क ऋविय ो क दवारा रच गए थ और परि ो

दवारा सीख जात थ

- हालाोवक कछ शल क मवहलाओो (जस वक अपाला

घ सा ल पामदरा मतरयी और गागी) क दवारा भी रच

गए थ

- ऋगवद म सोिाद क रप म कई शल क मौजद ह

- हम विशवावमतर नामक एक ऋवि और दविय ो क

रप म पजी जान िाली द नवदय ो (वयास और

सतलज) क बीच िाताि का उदाहरण वमलता ह

- इसस पता चलता ह वक विशवावमतर िवदक काल स

सोबोवित थ

Q2) उततर (b)

सपषटीकरण

- करनल गफाओो स राख क अिशि परापत हए ह

ज इस ओर सोकत करत ह वक ततकालीन ल ग

अवि क उपय ग स पररवचत थ

- य गफाएो आोधर परदश म सथथत ह

Q3) उततर (c)

सपषटीकरण

bull बरािह म ितिमान कशमीर म सथथत एक

परागवतहावसक थथल ह जहाो ल ग गडढ क घर ो का

वनमािण करत थ

bull य घर जमीन क ख द कर बनाए जात थ तथा नीच

जान क वलए सीवियाा ह ती थी

bull ऐसा अनमान लगाया जाता ह वक य घर ठो ड क

मौसम म आशरय परदान करत थ

Q4) उततर (c)

सपषटीकरण

bull परालख-विदया (Epigraphy) क वशलालख ो क

अधययन क रप म पररभावित वकया जाता ह

bull हसतवलसखत दसतािज ो क माधयम स इवतहास

और सावहतय क अधययन क पाोडवलवप विजञान

(Manuscriptology) कहत ह

bull पराचीन लखन परणावलय ो क अधययन और

ऐवतहावसक पाोडवलवपय ो क समझन तथा वतवथ

वनिािरण क पलीओगराफी (Palaeography) कहा

जाता ह

bull नयवमजमविकस (Numismatics) वसक ो क

अधययन क सोदवभित करता ह

Q5) उततर (a)

सपषटीकरण

- चरक सोवहता चरक क दवारा वलखी गई आयिद

और िदयक-शासर पर एक महतवपणि पसतक ह

- ि भारतीय िदयक-शासर की पारमपररक परणाली

वजस आयिद क नाम स जाना जाता ह क

अभयासकताि थ

- ऐसा माना जाता ह वक चरक का विकास दसरी

शताबदी (ईसा पिि) और दसरी शताबदी (ईसवी) क

मधय हआ था

Q6) उततर (b)

सपषटीकरण

- भाग फसल ो पर वलए जान िाल कर क सोदवभित

करता ह ज कल फसल उतपादन का 16 िाो भाग

था

- ldquoकममकारrdquo शबद भवमहीन कवि शरवमक िगि क

वलए परय ग वकया जाता था

- ldquoअशवमिrdquo (वजस घ ड क बवलदान क रप म भी

जाना जाता ह) एक अनषठान ह ता था वजसम एक

घ ड क सवतोतर रप स घमन क वलए छ ड वदया

FC19H1003 30

जाता ह और राजा क सवनक उसकी रखिाली

करत थ

Q7) उततर (d)

सपषटीकरण

- ऋगववदक काल म घ ड ो क रथ ो म ज ता जाता था

ज (रथ) भवम मिवशय ो आवद पर कबजा करन क

वलए लड गए यद ो म उपय ग वकए जात थ

- इसस यह पता चलता ह वक घ ड ो यकत रथ ो का

उपय ग महाजनपद काल स काफी पहल आरमभ

हआ था

- ऋगववदक काल म मिवशय ो भवम जल आवद पर

कबजा करन क वलए तथा ल ग ो क पकडन क

वलए यद वकय जात थ

- अविकाोश परि इन यद ो म भाग वलया करत थ

- हालाोवक उस समय क ई वनयवमत सना नही ो ह ती

थी लवकन उस काल म सभाऐो ह ती थी ो वजनम

ल ग यद क मामल ो पर चचाि करत थ

- वनयवमत सनाएा महाजनपद काल का िवशषटय थी

वजनम पदल सवनक ो की विशाल सनाएा रथ तथा

हाथी शावमल ह त थ

Q8) उततर (a)

सपषटीकरण

- बद शाकय कल स सोबोवित थ और कशीनारा म

उनका वनिन हआ था

- बद न अपनी वशकषाएा पराकत भािा म दी थी ो ज

आम ल ग ो की भािा थी

Q9) उततर (c)

सपषटीकरण

- पराचीन भारत म दशिनशासर की छह शाखाएा थी ो

िशविक नयाय समखया य ग पिि वममाोसा और

िदाोत या उततर वममाोसा

- इनकी थथापना करमश कनाद गौतम कवपल

पतोजवल जावमनी और वयास ऋविय ो न की थी

Q10) उततर (b)

सपषटीकरण

महािीर की वशकषाऐो छठी शताबदी म िललभी म

सोकवलत की गई थी ो

Q11) उततर (c)

सपषटीकरण

- पारमपररक रप स चाणकय क कौविलय अथिा

विषणगपत क नाम स जाना जाता ह

- उसन अथिशासतर ज एक पराचीन भारतीय

राजनवतक आलख ह वलखा था

Q12) उततर (d)

सपषटीकरण

- भारत का राषटर ीय वचनह सारनाथ (उततर परदश) क

अश क सतमभ क ऊपर (शीिि पर) वसोह कवपिल

का एक अनरपण ह

- इस राषटर ीय वसदाोत सतयमि जयत क साथ

सोय वजत वकया गया ह

- रामपिि बल का नाम रामपिि (वबहार) क नाम पर

पडा जहाा इसकी ख ज हई थी

- यह अपन नाजक नकाशी मॉडल क वलए परवसदद

ह वजसम क मल तवचा सोिदनशील नथन ो सतकि

कान और मरबत िााग ो क शरषठतर परवतरप क

परदवशित वकया गया ह

- यह भारतीय और फारसी ततव ो का एक ससममशरण

- सोवकससा उततर परदश म सथथत ह

Q13) उततर (a)

सपषटीकरण

का िर वसोह ज एक महान य दा थ वबहार स

सोबोवित थ

Q14) उततर (b)

सपषटीकरण

िललालर शबद बड भ-सवावमय ो क वलए परय ग

वकया जाता था

FC19H1003 31

Q15) उततर (c)

सपषटीकरण

- अररकमड एक तिीय बसती थी जहाो दर दश ो स

आन िाल जहाज ो का माल उतारा जाता था

- यहाो पर ईोि ो का एक विशाल ग दाम वमटटी क

बतिन (वजनम एमफ रा - द हरी मवठय ो का लोबा

घडा - शावमल ह) और एरिाइन (Arretine)

मदभाोड पाए गए थ

- इस थथान पर र मन दीपक काोच क बन पातर और

रतन भी पाए गए थ

Q16) उततर (a)

सपषटीकरण

- मिनदर सोगम कविताओो म उसललसखत एक

तवमल शबद ह वजसका अथि ह ldquoतीन परमखrdquo

- यह तीन सततारि पररिार ो क मसखयाओो क वलए

परय ग वकया जाता ह च ल चर और पाणडय

Q17) उततर (c)

सपषटीकरण

- ऋग िद म सभा विदाथा तथा गण जसी

जनजावतय ो पर अथिा किोब पर आिाररत

सभाओो का उललख ह

- आरसमभक िवदक काल म सभाओो और सवमवतय ो

का विशि महतव ह ता था

- यहाा तक की मसखया अथिा राजा भी उनका

समथिन परापत करन क वलए आतर रहत थ

Q18) उततर (a)

सपषटीकरण

- जन िमि न ईशवर क अससततव क मानयता त दी ह

वकनत उसन ईशवर क वजना क पद स नीच रखा

- जन िमि न बौद िमि की तरह िणि परणाली की

भरतिना नही ो की थी

Q19) उततर (d)

सपषटीकरण

- च ल ो और पाणडय ो न शसकतशाली तिीय शहर ो का

विकास वकया था

- च ल ो का सबस महतवपणि शहर पहार (या

कािरीपटटीनम) था |

- मदरई पाणडय ो की राजिानी थी

Q20) उततर (b)

सपषटीकरण

- ldquoबदचररतrdquo बद का जीिन-ितताोत ह

- इस अशवघ ि क दवारा वलखा गया था

Q21) उततर (a)

सपषटीकरणः

- तवमल कवि अपपर भगिान वशि क भकत थ

- इस परकार ि एक नयनार सोत थ

Q22) उततर (d)

सपषटीकरणः

- समदरगपत एक परवसद गपत शासक था

- उसन वसक ो पर िीणा बजात हए अपनी छवि

अोवकत करिाई थी

- यह सोगीत क परवत उसक परम क दशािता ह

- हम उसकी इलाहाबाद परशससत स महतवपणि

ऐवतहावसक जानकारी वमलती ह वजसकी रचना

उसक दरबार क कवि हररसन न की थी

Q23) उततर (b)

सपषटीकरणः

- विकरम सोित की शरआत ििि 58 ईसा पिि म

चनदरगपत वदवतीय न की थी

- यह शक ो पर उसकी जीत और उस विकरमावदतय

की पदिी वमलन क उपलकषय म आरमभ वकया गया

था

FC19H1003 32

- बानभटट न हिििििन का जीिन-ितताोत हििचररत

(ज सोसकत म थी) वलखी थी

Q24) उततर (c)

सपषटीकरणः

- सोवि-विगरावहका यद एिो शाोवत का मोतरी

- साथििाह वयापाररय ो क कावफल ो का नता

Q25) उततर (a)

सपषटीकरणः

- जआन झाोग (हसआन रताोग ndash Hsuang Tsang)

एक चीनी यातरी था ज हिििििन क शासनकाल म

भारत आया था

- ििि 630 ईसवी स ज दशक आरमभ हआ था उसम

जआन झाोग मधय एवशया ईरान और

अफग़ावनसतान की यातरा करन क पशचात कशमीर

क रासत स भारत आया था

- उसन उततर स पिि तक की यातरा की और िह

लगभग 2 ििि वबहार म रहा

- जआन झाोग न नालनदा विशवविदयालय म विदयावथिय ो

और विदवान ो क साथ पारसपररक विचार-विमशि

वकया थथानीय भािाओ ा म वनपणता परापत की तथा

बौद सतप ो की ख ज की

Q26) उततर (c)

सपषटीकरणः

- परदवकषणा पथ बौद िासतकला म सतप क चार ो

ओर बनाया जान िाला एक घमािदार पथ ह ता

- परशन म वदए गए बाकी क तीन ो ततव वहोद मसनदर ो की

िासतकला क भाग ह

Q27) उततर (d)

सपषटीकरणः

परशन म वदए गए सभी मोवदर ो म वयापक रप स

ईोि ो (पकी ईोि ो) का परय ग पतथर ो क साथ हआ

Q28) उततर (c)

सपषटीकरण

- महममद कली कतब शाह ग लकणडा का सलतान

था

- िह अकबर का समकालीन था

- सावहतय और िासतकला म उसकी अतयाविक

रवच थी

- िह एक महान कवि था

- िह दसखनी उदि फारसी और तलग म वलखता था

- उसन अपन पीछ एक विसतत वदिान (सोगरह)

छ डा ह

- अभी हाल ही म तलोगाना म ग लकणडा क वकल

क अनदर खदाई वकय गए बाग-ए-नाया वकला

बाग क चार ो ओर रप-रखा क मानवचतरण क

वलए भारतीय परातासतवक सिकषण (The

Archaeological Survey of India ndash ASI)

गराउणड पनीिर विोग रडार (Ground Penetrating

Radar) का परय ग करगा

Q29) उततर (a)

सपषटीकरणः

- वसलपपावदकारम एक तवमल महाकावय ह वजसकी

रचना इलाोग क दवारा लगभग 1800 ििि पिि की

गई थी

- यह क िलन नामक एक वयापारी की कहानी ह

ज माििी नामक एक गवणका (िशया) स परम

करन लगा था

- मवनमकलाई क िलन और माििी की पतरी की

कहानी ह

Q30) उततर (a)

सपषटीकरण

- चरक आयिद और वचवकरता की एक महतवपणि

रचना चरक सोवहता क लखक ह

- बरहमगपत क अपनी रचना बरहम-सफि-वसदानत

(ज एक खग लीय रचना ह) क कारण परवससद

वमली

FC19H1003 33

- बगदाद म इसका अनिाद अरबी भािा म वकया

गया था

- इसका इसलावमक गवणत और खग ल-विजञान पर

महतवपणि परभाि पडा था

- बाद म अपन जीिनकाल म बरहमगपत न

ldquoखोडखयाकrdquo वलखी ज एक खग लीय पससतका

(एक छ िी पसतक) थी

- इसम आयिभटट की अिि-रावतर क परतयक वदन की

शरआत परणाली का परय ग वकया गया था

Q31) उततर (c)

सपषटीकरण

- अमीर खसर एक परवसद सफी सोगीतकार कवि

और विदवान थ

- 1318 म उनह ोन पाया वक इस भवम (वहोदसतान) क

हर कषतर म अलग-अलग भािा थी लाहौरी

कशमीरी दवारसमदरी (दवकषणी कनाििक म)

तलोगाना (आोधर परदश म) गजरी (गजरात म)

माबारी (तवमलनाड म ) अििी (पिी उततर परदश

म) और वहोदिी (वदलली क आस-पास क कषतर म)

आवद

- उनह न यह बताया वक सोसकत वकसी भी कषतर स

सोबोवित नही ो थी और किल बराहमण ही इस भािा

का जञान रखत थ

Q32) उततर (c)

सपषटीकरण

- वहरणय-गभि सववणिम गभि क सोदवभित करता ह

- जब बराहमण ो की सहायता स यह अनषठान वकया

जाता था त यह माना जाता था वक बवल दन िाल

का कषवतरय क रप म पनजिनम ह गा

Q33) उततर (d)

सपषटीकरण

- कदमई भवम राजसव पर कर क सोदवभित करता

- गवावलयर परशससत म नागभि क दवारा वकय गए

श िण का िणिन वकया गया ह |

- नागभि एक परवतहार राजा था

Q34) उततर (b)

सपषटीकरण

- राजतरो वगनी 12िी ो शताबदी म कलहन क दवारा

रवचत एक सोसकत पसतक (िकसट) ह

- यह परारसमभक भारत की ऐवतहावसक इवतितत थी

- तकि सोगत रप स इस अपन परकार की सिोततम

और सिािविक विशवसनीय कवत माना जाता ह

- यह कशमीर कषतर क पराचीनतम समय स लकर

उसकी रचना की तारीख तक क समपणि इवतहास

का आचछादन करती ह

Q35) उततर (c)

सपषटीकरण

- गााि की आम सभा क ldquoउरrdquo कहा जाता था

- ldquoउरrdquo म गााि क सभी कर दन िाल वनिासी

शावमल ह त थ

Q36) उततर (a)

सपषटीकरण

- वदलली सलतनत म ldquoतारीखrdquo इवतहास लखन का

एक रप था

- ldquoतािरीखrdquo क लखक विदवान परि ह त थ वजनम

सवचि परशासक इतयावद शावमल थ

Q37) उततर (a)

सपषटीकरण

- अलाउददीन सखलजी अपन सवनक ो क ितन का

भगतान नकद म करता था न वक इकता क रप

- सवनक अपना सामान वदलली म वयापाररय ो स

खरीदत थ अतः इस बात का भय था वक वयापारी

कही ो िसतओो का मलय न बिा द

- इसकी र कथाम क वलए अलाउददीन सखलजी न

वदलली म कीमत ो क वनयसित वकया

FC19H1003 34

- अविकारीगण धयानपििक मलय ो का सिकषण करत

थ तथा ज वयापारी वनिािररत मलय पर माल नही ो

बचत थ उनक दसणडत वकया जाता था

Q38) उततर (d)

सपषटीकरण

- वदलली सििपरथम त मर राजपत ो क अिीन उनक

सामराजय की राजिानी बनी थी

- 12िी ो शताबदी क मधय म अजमर क चौहान ो

(वजनह चाहमान ो क नाम स भी जाना जाता ह) न

त मर राजपत ो क परावजत वकया था

- त मर ो और चौहान ो क अिीन वदलली एक

महतवपणि िावणसजयक क दर बन गया था

- कई जन वयापारी यहाा रहन लग थ और उनह ोन

कई मोवदर भी बनिाए

- यहाा पर मवदरत वसक वजनह ldquoदहलीिालrdquo क नाम

स जाना जाता था वयापक रप स परचलन म थ

Q39) उततर (c)

सपषटीकरण

- म ठ की मसिद का वनमािण वसको दर ल दी क

राजयकाल म उसक मिी क दवारा करिाया गया

था

- बगमपरी मसिद का वनमािण महममद तगलक क

शासनकाल म हआ था

- यह मसिद विशव का पणयथथान (The

Sanctuary of the World) और वदलली म महममद

तगलक की नई राजिानी जहाोपनाह की मखय

मसिद थी

- कववत- अल - इसलाम मसिद का विसतार

इलतसिश और अलाउददीन सखलजी न वकया था

- मीनार का वनमािण तीन सलतान ो कतबददीन ऐबक

इलतसिश और वफर ज शाह तगलक क दवारा

करिाया गया था

Q40) उततर (c)

सपषटीकरण

- मगल ो क अिीन मनसबदार शबद उस वयसकत क

वलए सोदवभित वकया जाता था वजसक पास मनसब

(अथाित पद) ह ता था

- उस अपना ितन राजसव कायो वजनह जागीर कहत

थ क रप म परापत ह ता था

Q41) उततर (b)

सपषटीकरण

- ldquoभारत छ ड आोद लनrdquo वबरविश शासन क

सखलाफ ल ग ो का एक सवाभाविक विदर ह था

- असखल भारतीय काोगरस सवमवत न 8 अगसत 1942

क बमबई म एक बठक का आय जन वकया था

- इस बठक म परवसद सोकलप ldquoभारत छ ड rdquo क

पाररत वकया गया और इस उददशय क परापत करन

क वलए गाोिी क नततव म एक अवहोसक जन सोघिि

आोद लन की शरआत का परसताि वदया गया

- लवकन अगल ही वदन गाोिी और काोगरस क अनय

परमख नताओो क वगरफतार कर वलया गया

- काोगरस क एक बार वफर अिि घ वित वकया गया

था

Q42) उततर (c)

सपषटीकरण

- साइमन कमीशन यनाइविड वको गडम क सात

साोसद ो का एक समह था

- इस वबरविश भारत क वलए सोििावनक सिार ो का

सझाि दन क वलए गवठत वकया गया था

- इस आय ग म िररषठ वबरविश राजनता सर जॉन

साइमन क नततव म किल वबरविश सदसय ही

शावमल थ

- इसवलए भारत क ल ग ो न साइमन कमीशन क

आगमन क विरद आोद लन वकया था

Q43) उततर (a)

सपषटीकरण

bull दादा भाई नौर जी भारत म वबरविश शासन क

आवथिक पररणाम ो क बार म अपनी विर िी

(परवतकल) राय क वलए जान जात थ

FC19H1003 35

bull अपन कई लख ो और भािण ो म विशि रप स

ldquoपाििी एो ड अन-वबरविश रल इन इसणडया

(Poverty and Un-British Rule in India) म

नौर जी न यह तकि वदया वक भारत पर अतयविक

कर लगाया गया था और इसकी सोपवतत इोगलड की

ओर परिावहत की जा रही थी

bull उनह ोन पराचीन भारतीय गरोथ ो की वयाखया करन

का और भारतीय ो क आिविशवास क बहाल

करन पर कायि नही ो वकया था

उनह ोन वकसी और बात स पहल सभी सामावजक

बराइय ो क उनमलन की आिशयकता पर भी बल

नही ो वदया था

Q44) उततर (c)

सपषटीकरण

bull अगसत 1932 म वबरविश परिानमोतरी मकड नालड न

अपन साोपरदावयक परसकार (The Communal

Award) की घ िणा की थी

bull यह भारत क कई साोपरदावयक वहत ो क बीच विवभनन

सोघिो क हल करन क वलए वबरिन का एकतरफा

परयास था

bull यह परसकार (Award) बाद म 1935 क

अविवनयम (The Act of 1935) म शावमल वकया

गया था

bull इस साोपरदावयक परसकार न मससलम ो क वलए

आरवकषत एक अलग वनिािचक मणडल फॉमिल का

विसतार अनय अलपसोखयक ो क वलए वकया था

वजसम वसख ो भारतीय ईसाइय ो आोगल-भारतीय

समदाय यर पीय समदाय तथा विवशषट कषतरीय

समह ो क शावमल वकया गया था

bull गाोिी न इस परसताि क भारतीय समाज क

विभावजत करन क वलए एक घवणत वबरविश

सावजश क रप म दखा और उसक सखलाफ

आमरण अनशन वकया

Q45) उततर (b)

सपषटीकरण

मौजदा आयात और वनयाित क अवतररक़त

औपवनिवशक भारत क वनमनवलसखत खचो क

वलए एक विशिवनवशचत िन रावश भी दनी पडती

थी

(i) परशासन क वयय

(ii) सना क रख-रखाि क वयय

(iii) यद क वयय

(iv) सिावनितत अविकाररय ो की पशन तथा

(v) वबरिन दवारा अपनी उपवनिश बसती

(कॉल नी) क रख-रखाि क वयय

इनह गह शलक (Home Charges) क रप म

जाना जाता था और लगभग परी तरह स भारत क

दवारा इनका भगतान वकया जाता था

bull गह शलक म वनमनवलसखत घिक शावमल थ

(i) भारतीय ऋण पर दय बयाज

(ii) ईसट इोवडया को पनी क शयरिारक ो क

लाभाोश

(iii) लोदन म भारत कायािलय चलान क वलए िन

(iv) भारत म वनयकत वबरविश कवमिय ो क ितन

और पशन का भगतान करन क वलए िन

(v) रलि पर बयाज

(vi) नागररक और सनय शलक

(vii) इोगलड म सट र (सामगरी) की खरीद

Q46) उततर (b)

सपषटीकरण

bull भारतीय राषटर ीय काोगरस का लाहौर सतर 1929 म

जिाहरलाल नहर की अधयकषता म आय वजत

वकया गया था

bull इस सतर म भारतीय राषटर ीय आोद लन स समबसित

कई महतवपणि पररणाम सामन आय थ

(i) सििपरथम इस सतर म काोगरस क अधयकष पद

पर जिाहरलाल नहर क चना गया था ज

काोगरस म िामपोवथय ो की बिती हई ताकत

का सपषट सोकत था

(ii) दसरा इस सतर म पहली बार काोगरस न पणि

सवतोतरता की माोग क उठाया था

इस परकार की माोग काोगरस मोच स पहल कभी भी

नही ो उठाई गई थी

Q47) उततर (b)

सपषटीकरण

FC19H1003 36

bull इस ररप िि न वकसी भी समदाय क वलए पथक

वनिािचक मोडल अथिा अलपसोखयक ो क वलए

भाराोश की वसफाररश नही ो की थी

bull तथावप इस ररप िि न उन पराोत ो म अलपसोखयक

सीि ो क आरकषण की अनमवत दी थी जहाा पर कम

स कम दस परवतशत अलपसोखयक ह

bull लवकन यह समदाय क आकार क अनपात म ह ना

चावहए था

bull इस ररप िि म भारत क वलए पणि सवतोतरता क

वलए क ई पराििान नही ो था

Q48) उततर (c)

सपषटीकरण

bull आरो वभक िवदक आयो का िमि मखय रप स

परकवत की पजा और यजञ था

bull परारो वभक आयि िमि परकवत की पजा क समान था

bull िासति म उनक चार ो ओर की शसकतयाा वजनह न

त ि वनयोवतरत कर सकत थ और न ही समझ पाए

थ उनह वदवयता क साथ वनिवशत वकया गया तथा

उनह मादा या नर दिीदिताओो क रप म

परतीकतव वकया गया था

bull उनह ोन कछ यजञ ो का भी वनषपादन वकया था

Q49) उततर (b)

सपषटीकरण

bull सडक और नदी-मागि (जल-मागि) डकती स

सरवकषत नही ो थ

bull उललखनीय ह वक हिििििन क शासनकाल क

दौरान यआन चिाोग (हयएन साोग) का सारा

सामान लि वलया गया था

Q50) उततर (c)

सपषटीकरण

परशन म वदए गए द न ो कथन सही ह

Q51) उततर (b)

सपषटीकरण

bull परोदर दास एक सोत और भगिान कषण क एक

महान भकत थ

bull परोदर दास क कनाििक सोगीत क वपतामह क

रप म जाना जाता ह

bull यदयवप उनक जनम-थथान क बार म काफी

अिकल लगाई जाती रही ह

bull तथावप अब कननड विशवविदयालय हमपी क दवारा

गवठत एक विशिजञ सवमवत इस वनषकिि पर पहोची

ह वक उनका जनम थथान सोभितया कनाििक का

एक छ िा-सा गााि कषमपरा (वशिम गगा वजला)

था

Q52) उततर (c)

सपषटीकरण

bull शरी तयागराज शरी शयाम शासतरी और शरी मथसवामी

दीवकषतर क कनाििक सोगीत की वतरमवति माना

जाता ह

bull उनक कारण ही 18िी ो-19िी ो शताबदी म कनाििक

सोगीत का सववणिम यग आया था

Q53) उततर (d)

सपषटीकरण

bull अभी हाल ही म लौह यगीन-महापािावणक काल

का 2000 ििि पराना एक दलिभ सारक फगस

(Sarcophagus) (पतथर का ताबत) क ललम क

वियर गाोि (क वयलडी क पास वजला क वझक ड

करल राजय) की एक रॉक-कि गफा स ख जा गया

bull यह ताबत वजसम हविय ो क िकड थ खदाई क

दौरान वमला

bull अभी तक इस परकार की दलिभ ख ज करल क

मातर द ही थथान ो स हई ह

bull य द न ो सारक फगी (Sarcophagi) (पतथर क

ताबत) चियर और अथ ली (वजला क वझक ड) क

महापािाण थथल ो स वमल ह

Q54) उततर (a)

सपषटीकरण

FC19H1003 37

दवकषण भारत म महापािाण सोसकवत एक पणि

विकवसत लौह यगीन सोसकवत थी

Q55) उततर (d)

सपषटीकरण

bull च ल पाणडय और करलपतर (चर) इन तीन ो का

उललख अश क क अवभलख ो म वकया गया ह

bull सोभितः य भौवतक सोसकवत क उततर

महापािावणक चरण म थ

Q56) उततर (d)

सपषटीकरण

bull भीमा-क रगाोि की लडाई ततीय आोगल-मराठा

यद का वहससा थी

Q57) उततर (b)

सपषटीकरण

bull राजकमार शकल न गाोिीजी क चोपारण आन तथा

वतनकवथया परणाली स जडी समसया की जाोच क

वलए रारी करन क वलए दश भर म उनका

अनसरण वकया था

bull बज वकश र राजदर परसाद महादि दसाई और

नरहरी पाररख चोपारण सतयागरह क दौरान गाोिी

जी क सहय गी थ

Q58) उततर (b)

सपषटीकरण

bull बराहमण ो और बौद मठिाररय ो क कर-मकत गााि

अनदान म दन की परथा सतिाहन ो न आरमभ की

थी

Q59) उततर (c)

सपषटीकरण

इस कायिकरम क उददशय वनमनानसार ह

(i) बवनयादी पयििन आिाररक सोरचना का विकास

करना

(ii) चयवनत (पहचान वकय गए) कषतर ो म आजीविका क

सजन क वलए दश क साोसकवतक और विरासत

मलय ो क बिािा दना

(iii) विरासत समारक थथल ो पर विशव सतरीय आिाररक

सोरचना विकवसत करक एक सतत तरीक स

पयििक आकििण म िसद करना

(iv) थथानीय समदाय ो की सवकरय भागीदारी क माधयम

स र रगार ो का सजन करना

(v) र रगार उतपादन और आवथिक विकास क वलए

पयििन कषमता का उन पर परभाि का उपय ग

करना तथा

(vi) िारणीय पयििन आिाररक सोरचना का विकास

करना और उसका उवचत सोचालन तथा

रखरखाि सवनवशचत करना

Q60) उततर (b)

सपषटीकरण

bull यह वनकाय ििि 1987 म अससततव म आया था

bull यह एक राषटर ीय सतर का शीिि सोगठन ह ज भारत

सरकार क जनजातीय मामल ो क मोतरालय क

परशासवनक वनयोतरण क अिीन काम कर रहा ह

bull इसका पोजीकत और परिान कायािलय नई वदलली

म सथथत ह

Q61) उततर (c)

सपषटीकरण

bull परमचोद क उपनयास ो म परमाशरम रोगभवम गबन

कमिभवम और ग दान शावमल ह

bull ग रा रिी ोदरनाथ िग र क दवारा रवचत उपनयास ह

bull अभी हाल ही म मोशी परमचोद की 138िी ो जयोती दश

भर म मनाई गई थी

Q62) उततर (b)

सपषटीकरण

bull ldquoवगदाrdquo पोजाब (भारत) एिो पावकसतान की

मवहलाओो क दवारा तयौहार क समय और फसल

की बिाई तथा किाई क अिसर पर वकया जान

िाला एक पारोपररक दहाती नतय ह

FC19H1003 38

bull इस नतय क माधयम स पोजाबी मवहलाऐो अपनी

परसननता परकि करती ह तथा वगदा क परदशिन क

माधयम स परि िचिसव िाल समाज म मवहलाओो

की दबी हई भािनाओो क परकि करती ह

bull चोवक इस नतय का परि ो क साथ क ई सोबोि नही ो

ह अतः किल मवहलाऐो ही इसम भाग ल सकती

bull हर साल तीज समार ह क दौरान पोजाब म वगदा

नतय वकया जाता ह

तीज भारत क कछ भाग ो म मवहलाओो क दवारा

मनाया जान िाल कई तयौहार ो क वलए एक

वयापक नाम ह

Q63) उततर (a)

सपषटीकरण

- मजम-उल-बहरीन या द समदर ो का सोगम

नामक उललखनीय रचना दारा वशक ह क दवारा

वलखी थी

- भारत क उपराषटर पवत शरी एम िकया नायड न कहा

ह वक राजकमार दारा वशक ह की रचनाएा शाोवत

और सदभाि क बिािा दन क वलए एक तारा सर त

क रप म सामन आ सकती ो ह

- उपराषटर पवत गत ििो क भला वदए गए राजकमार

दारा वशक ह क परदवशित परचवलत करन हत

आय वजत एक परदशिनी का दौरा करन क बाद एक

सभा क सोब वित कर रह थ

- इस परदशिनी का आय जन फर क इस गौवियर

(Francois Gautier) क दवारा lsquoइोवदरा गाोिी नशनल

सिर फॉर द आििसrsquo (The Indira Gandhi

National Centre for the Arts) नई वदलली म

वकया गया था

Q64) उततर (c)

सपषटीकरण

- ग मतशवर परवतमा जन भगिान बाहबली क

समवपित ह

- यह एक एक-चटटानी पतथर की मवति ह

- राषटर पवत राम नाथ क विोद न शरिणबलग ला

(कनाििक) म आय वजत वकय जान िाल भवय

अवभिक समार ह महामसतकावभिक का

उदघािन वकया था

- यह समार ह 12 ििो म एक बार ह ता ह

Q65) उततर (c)

सपषटीकरण

bull पराची घािी पराची नदी क चार ो ओर फली हई थी

bull पराची घािी िीर-िीर विलपत ह गई थी

bull पराची नदी भिनशवर स वनकलती ह

bull यह महानदी की एक सहायक नदी ह और यह

परी खदाि किक तथा जगतवसोहपर वजल ो स

ह कर बहती ह

bull इस नदी क पर कषतर क पराची घािी कहा जाता ह

bull यह नदी बोगाल की खाडी म वगरती ह

परातासतवक साकषय स पता चलता ह वक पराची घािी

सभयता हडपपा और म हनज दाड द न ो की

पिििती ह

Q66) उततर (d)

सपषटीकरण

य समारक छतरपर वजल (मधय परदश) म विोधयाचल

पिित शरोखला म सथथत ह

Q67) उततर (a)

सपषटीकरण

bull थॉिस ऑन पावकसतान नामक पसतक डॉ बी

आर अमबडकर न वलखी थी

bull डॉ बी आर अमबडकर की जयोती क अिसर पर

भारत क राषटर पवत न भारत की इस महान हसती

क शरदाोजवल अवपित की थी

bull डॉ बी आर अमबडकर न 1924 म वडपरथड

कलावसर इोसटीटयि (दवलत िगि सोथथान -

बवहषकत वहतकाररणी सभा) और 1927 म समाज

समता सोघ की थथापना की थी

bull अमबडकर का धयान वशकषा कषतर की ओर भी था

bull उनह ोन वशकषा क वनमन िगो म फलान क वलए

पीपलस एजकशन स साइिी (The Peoples

Education Society) क नाम स महाविदयालय ो क

नििकि और छातरािास ो की थथापना की थी

FC19H1003 39

Q68) उततर (b)

सपषटीकरण

bull महरगि भारतीय उपमहादवीप म एक परवसद

निपािाण बसती ह ज बलवचसतान पराोत

पावकसतान म सथथत ह

bull दचपलली (आोधर परदश) क पास नागलर नदी क

पिी ति ो पर चना पतथर क बलॉक क विशाल

विसतार म एक पिि-ऐवतहावसक रॉक आिि थथल की

ख ज की गई ह

bull इसन 1500-2000 ईसा पिि क दौरान गोिर (आोधर

परदश) म विकवसत निपािाण सभयता पर परकाश

डाला ह

Q69) उततर (c)

सपषटीकरण

bull 12िी ो सदी और 13िी ो सदी म काकाविय िोश का

उदय हआ था

bull ि पहल कलयाण क पवशचमी चालकय ो क सामोत थ

bull परारोभ म उनह ोन िारोगल (तलोगाना) क पास एक

छ ि स कषतर पर शासन वकया था

bull उनह ोन ldquoनायक वयिथथाrdquo की शरआत की थी

वजस बाद म विजयनगर क राय शासक ो न

अपनाया और विकवसत वकया था

Q70) उततर (a)

सपषटीकरण

bull गाोिीजी क अनशन स वमल मावलक ो पर दबाि

पडा था ज अोततः शरवमक ो क ितन म 35 परवतशत

की िसद करन क वलए सहमत हए थ

bull गगल (Google) न अनसया साराभाई वजनह ोन

भारत क शरवमक आोद लन म एक अगरणी भवमका

वनभाई थी की 132िी ो जयोती डडल (Doodle) का

वनमािण करक मनाई

Q71) उततर (d)

सपषटीकरण

भारत स यनसक की मानिता की अमति साोसकवतक

विरासत की परवतवनवि सची म वनमनवलसखत शावमल ह

bull कवडयटटम करल का सोसकत रोगमोच

bull मवडयिि करल का अनषठान रोगमोच और नतय

नाविका

bull िवदक मि जाप की परोपरा

bull राजथथान क कालबवलया ल क गीत और नतय

bull रामलीला रामायण का पारोपररक परदशिन

bull सोकीतिन मवणपर का अनषठान गायन ढ ल िादन

और नतय

bull रममन भारत क गििाल वहमालय का िावमिक

तयौहार और अनषठान रोगमोच

bull जाोदीयाला गर पोजाब क ठठर ो की पीतल और

ताोब क वशलप स वनवमित बतिन ो की पारोपररक कला

bull छाऊ नतय पिी भारतीय राजय ो म जनमी शासतरीय

भारतीय नतय कला

bull लददाख का बौद मि जाप िर ाोस-वहमालयी लददाख

कषतर तथा जमम-कशमीर म पवितर बौद गरोथ ो का पाठ

bull य ग

bull नौर र

bull को भ मला

Q72) उततर (b)

सपषटीकरण

bull भारत क राषटर पवत शरी राम नाथ क विोद न

वकसामा नागालड म हॉनिवबल मह रति और

राजय गठन वदिस समार ह का उदघािन वकया

था

bull हॉनिवबल मह रति का नाम भारतीय हॉनिवबल क

नाम पर पडा ह ज एक विशाल और रोगीन जोगली

पकषी ह

bull यह पकषी नागालड राजय की अविकतर जनजावतय ो

की ल ककथाओो म उसललसखत ह

bull नागालड की परमख मानयता परापत जनजावतयाा ह

अोगामी आओ चखसोग चाोग ककी रगमा और

रवलोग आवद

bull ओोग जारिा और ससिनलीस अोडमान-वनक बार

दवीप समह की जनजावतयाा ह

FC19H1003 40

Q73) उततर (c)

सपषटीकरण

bull दकन म राषटर कि शासन दसिी ो सदी क अोत तक

लगभग 200 ििो तक रहा था

bull राषटर कि शासक अपन िावमिक विचार ो म सवहषण

bull उनह ोन न किल शि िमि और िषणि िमि बसलक

जन िमि क भी सोरकषण वदया था

bull एल रा म वशि क परवसद रॉक कि मोवदर का

वनमािण नौिी ो सदी म राषटर कि राजा कषण परथम न

करिाया था

bull उसका उततराविकारी अम घििि जन था लवकन

उसन अनय िमो क भी सोरकषण परदान वकया था

bull राषटर कि ो न मसलमान वयापाररय ो क बसन की

अनमवत दी थी

bull उनह न अपन अविराजय ो म इसलाम क उपदश दन

की भी अनमवत दी थी

bull अभी हाल ही म पाोडिलागटटा (तलोगाना) क

परागवतहावसक चटटान वचतर ो क कषरण की बिती हई

घिनाएा एक गोभीर वचोता का वििय ह

bull यह परागवतहावसक चटटान क नकसान पहाचा

सकता ह

bull पाोडिलागटटा वनमनवलसखत क वलए जाना जाता ह

- 10000 ईसा पिि स 8000 ईसा पिि क वचवतरत

चटटानी आशरय ो क वलए

- राषटर कि काल क एक 8 िी ो सदी क

वशलालख क वलए और

- 12िी ो सदी क काकविय सामराजय क वभवतत

वचतर ो क वलए

Q74) उततर (b)

सपषटीकरण

bull 1828 म राजा राम म हन रॉय न एक नय िावमिक

समाज बरहम सभा की थथापना की थी वजस बाद

म बरहम समाज क नाम स जाना गया था

bull दिदरनाथ िग र न ततवब विनी सभा की अधयकषता

की थी ज आधयासिक सतय की ख ज म सोलि

थी

bull इसका उददशय वहोद िमि क शद करन का और

एकशवरिाद (एक ईशवर म आथथा) का परचार करना

था

bull नय समाज की थथापना क आिार थ कारण

(तकि ) क द सतमभ तथा िद और उपवनिद

bull अभी हाल ही म सािारण बरहम समाज का कछ

काननी मदद ो क लकर पवशचम बोगाल सरकार क

साथ काननी वििाद चल रहा ह

Q75) उततर (c)

सपषटीकरण

bull भारत म वचशती वसलवसल की थथापना खवाजा

म इनददीन वचशती क दवारा की गयी थी

bull ि 1192 ईसवी क आसपास भारत आय थ

bull वचशतीय ो क बारहिी ो शताबदी क उततरािि म भारत

म आन िाल सफीय ो क समह ो म सबस

परभािशाली माना जाता ह

bull उनह ोन थथानीय िातािरण क साथ सफलतापििक

अनकलन वकया और उनह ोन भारतीय भसकत

परोपराओो क कई पहलओो क अपनाया

bull अजमर म सफी अपरकि खवाजा म इनददीन वचशती

की ऐवतहावसक दरगाह क एक नया रप दन की

तयारी की जा रही ह

bull इस 13िी ो शताबदी की दरगाह क ldquoसवचछ

आइकॉवनक थथल ोrdquo (Swacch Iconic Places) म

शावमल वकया गया ह ज परवतवषठत विरासत

आधयासिक और साोसकवतक थथान ो पर क वदरत

य जना ह

FC19H1003 41

ANSWERS amp EXPLANATION OF

NCERT History Class VI-X + Current Affairs

(FC19E1003)

Q1) Answer c

Explanation

Rigveda consists of more than a

thousand hymns dedicated to gods and

goddesses These hymns were

composed by sages and learnt by men

however a few were composed by

women like Apala Ghosa Lopamudra

Maitreyi and Gargi

Rigveda consists of many hymns in the

form of dialogues We get an example of

a dialogue between a sage named

Vishwamitra and two rivers (Beas and

Sutlej) that were worshipped as

goddesses This suggests that he

belonged to the Vedic period

Q2) Answer b

Explanation

Traces of ash have been found from

Kurnool Caves suggesting that people

were familiar with the use of fire

It is situated in Andhra Pradesh

Q3) Answer c

Explanation

Burzahom is a prehistoric site in

present day Kashmir where people built

pit houses which were dug into the

ground with steps leading into them

These may have provided shelter in cold

weather

Q4) Answer c

Explanation

Epigraphy is defined as the study of

inscriptions

Manuscriptology is the study of history

and literature through the use of hand

written documents

Palaeography refers to the study of

ancient writing systems and the

deciphering and dating of historical

manuscripts

Numismatics refers to the study of

coins

Q5) Answer a

Explanation

Charaka Samhita was written by

Charaka and is an important book on

Ayurveda and medicine

He was a practitioner of the traditional

system of Indian medicine known as

Ayurveda

Charaka is thought to have flourished

sometime between the 2nd century BCE

and the 2nd century CE

Q6) Answer b

Explanation

Bhaga refers to the tax on crops which

was fixed at 16th of the production

Kammakaras is the term used for the

landless agricultural labour class

Ashvamedha also known as horse

sacrifice is a ritual where a horse is let

loose to wander freely and it was

guarded by the rajarsquos men

Q7) Answer (d)

Explanation

In the Rigvedic period horses were

yoked to chariots that were used in

battles fought to capture land cattle

etc This suggests that the use of horse

chariots began much before the period

of Mahajanapadas

The battles were fought in the Rigvedic

period for cattlersquos lands water an even

to capture people Most men took part

in these wars however there was no

regular army but there were assemblies

where people met and discussed

matters of war Regular armies became

a feature in the Mjahajanapada period

including vast armies of foot soldiers

chariots and elephants

RAUSIAS-FC19E1003 42

Q8) Answer (a)

Explanation

Buddha belonged to the Sakya clan and

passed away at Kusinara

Buddha taught in Prakrit which was the

common language of people

Q9) Answer c

Explanation

There were six schools of philosophy in

ancient India These are known as

Vaishesika Nyaya Samkhya Yoga

Purva Mimansa and Vedanata or Uttara

Mimansa They were founded by sages

Kanada Gautama Kapila Patanjali

Jamini and Vyasa respectively

Q10) Answer b

Explanation

The teachings of Mahavira were

compiled at Valabhi in 6th century AD

Q11) Answer (c)

Explanation

Chanakya is traditionally identified as

Kautilya or Vishnugupta who authored

the ancient Indian political treatise the

Arthashastra

Q12) Answer d

The national emblem of India is an

adaptation of the Lion Capital atop the

Ashoka Pillar of Sarnath Uttar Pradesh

and is combined with the National

Motto Satyameva Jayate

The Rampurva Bull gets the name from

the site of its discovery Rampurva in

Bihar

It is noted for its delicately sculpted

model demonstrating superior

representation of soft flesh sensitive

nostrils alert ears and strong legs It is

a mixture of Indian and Persian

elements

Sankissa is situated in Uttar Pradesh

India

Q13) Ans(a)

Kunwar Singh was a notable leader during the Revolt of 1857 He belonged

to a royal house of Jagdispur Bihar

Q14) Answer b

Explanation

The term Vellalar was used for large

landowners

Q15) Answer c

Explanation

Arikamedu was a coastal settlement

where ships unloaded goods from

distant lands Finds here include a

massive brick warehouse pottery

including amphorae and Arretine ware

Roman lamps glassware and gems have

also been found at the site

Q16) Answer a

Explanation

Muvendar is a Tamil word mentioned in

Sangam poems meaning three chiefs

used for the heads of three ruling

families the Cholas Cheras and

Pandyas

Q17) Ans (c)

Several tribal or kin-based assemblies

such as the Sabha Vidatha and gana

are mentioned in the Rig-veda The

Sabha and the samiti mattered a great

deal in early Vedic times so much so

that the chiefs or the kings showed an

eagerness to win their support

Q18) Ans (a)

Jainism recognised the existence of the

gods but placed them lower than the

jina and did not condemn the varna

system as Buddhism did

Q19) Answer (d)

Explanation

Cholas and Pandyas had developed

powerful coastal cities The most

important city of Cholas was Puhar or

Kaveripattinam and Madurai was the

capital of Pandyas

Q20) Answer b

Explanation

Buddhacharita is the biography of

Buddha and was written by

RAUSIAS-FC19E1003 43

Ashvaghosha

Q21) Answer (a)

Explanation

Tamil poet Appar was a Shiva devotee

So he was a Nayanar saint

Q22) Answer d

Explanation

Samudragupta was a prominent Gupta

ruler whose coins depict him playing a

veena indicating his love for music We

get important historic information from

his Allahabad Prashasti which was

composed by his court poet Harisena

Q23) Answer (b)

Explanation

Vikrama Samvat was founded by

Chandragupta II in the 58 BC as a

mark of victory over the Shakas and

assumed the title of Vikramaditya

Banabhatta wrote Harshavardhanarsquos

biography the Harshacharita in

Sanskrit

Q24) Answer c

Explanation

Sandhi-vigrahika was the minister of

war and peace

Sarthavaha was the leader of the

merchant caravans

Q25) Answer a

Explanation

Xuan Zang (Hsuan-tsang) was a

Chinese traveller who came during the

reign of Harshavardhana

In the decade that began in 630 AD

Xuan Zang came to India through

Kashmir after visiting Central Asia Iran

and Afghanistan

He travelled from north to east and lived

in Bihar for a couple of years

At Nalanda University Xuan Zang

interacted with students and scholars

mastered local languages and

discovered Buddhist stupas

Q26) Answer c

Explanation

Pradakshina patha is a circular path

laid around a stupa in Buddhist

architecture While the rest are a part of

temple architecture

Q27) Answer d

Explanation

All the above-mentioned temples have

an elaborate use of bricks (baked

bricks) along with stone

Q28) Ans (c)

Muhammad Quli Qutab was the Sultan

of Golconda He was a contemporary of

Akbar was very fond of literature and

architecture

The Sultan was a great poet and he

wrote in Dakhini Urdu Persian and

Telgu and has left an extensive diwan or

collection

Recently the Archaeological Survey of

India (ASI) will be using Ground

Penetrating Radar (GPR) to map the

contours of the area around the Bagh-e-

Naya Qila excavated garden inside the

Golconda Fort in Telangana

Q29) Answer a

Explanation

Silappadikaram is a famous Tamil epic

which was written by Ilango around

1800 years ago It is a story of a

merchant named Kovalan who fell in

love with a courtesan named Madhavi

Manimekalai tells the story of the

daughter of Kovalan and Madhavi

Q30) Answer (a)

Explanation

Charaka is the author of Charaka

Samhita which is an important work of

Ayurveda and medicines

Brahmaguptarsquos fame rests mostly on his

Brahma-sphuta-siddhanta which was

an astronomical work It was translated

into Arabic in Baghdad and had a major

impact on Islamic mathematics and

astronomy

Late in his life Brahmagupta wrote

Khandakhadyaka which was an

RAUSIAS-FC19E1003 44

astronomical handbook that employed

Aryabhatarsquos system of starting each day

at midnight

Q31) Answer (c)

Explanation

Amir Khusrau was a famous sufi

musician poet and scholar In 1318 he

noted that there was different language

in every region of this land (Hindustan)

Lahori Kashmiri Dvarsamudri (in

Southern Karnataka) Telangana (in

Andhra Pradesh) Gujari (in Gujarat)

Marsquobari (in Tamil Nadu) Awadhi (in

eastern Uttar Pradesh) and Hindawai (in

the area around in Delhi) etc He went

to explain that Sanskrit did not belong

to any region and that only brahmans

knew it

Q32) Answer c

Explanation

Hiranyagarbha refers to the golden

womb When this ritual was performed

with the help of Brahmanas it was

thought to lead to the rebirth of the

sacrificer as a Khastriya

Q33) Answer d

Explanation

Kadamai refers to a tax on land

revenue

Gwalior Prashasti describes the exploits

of Nagabhata who was a Pratihara king

Q34) Answer b

Explanation

Rajatarangini is a Sanskrit text written

by Kalhana in the 12th century

It was historical chronicle of early India

It is justifiably considered to be the best

and most authentic work of its kind

It covers the entire span of history in

the Kashmir region from the earliest

times to the date of its composition

Q35) Answer c

Explanation

ldquoUrrdquo was the general assembly of the

village ldquoUrrdquo consisted of all the

taxpaying residents of an ordinary

village

Q36) Answer (a)

Explanation

Tarikh was a form of history writing in

the Delhi Sultanate The authors of

tawarikhs were learned men which

included secretaries administrators etc

Q37 Answer (a)

Explanation

Alauddin chose to pay his soldiers salaries in cash rather than iqtas The soldiers would buy their supplies from merchants in Delhi and it was thus feared that merchants would raise their prices To stop this Alauddin controlled the prices of goods in Delhi Prices were carefully surveyed by officers and merchants who did not sell at the prescribed rates were punished

Q38) Answer (d)

Explanation

Delhi first became the capital of a

kingdom under the Tomara Rajputs

who were defeated in the middle of the

twelfth century by the Chauhans (also

referred to as Chahamanas) of Ajmer

It was under the Tomaras and

Chauhans that Delhi became an

important commercial centre Many rich

Jaina merchants lived in the city and

constructed several temples Coins

minted here called dehliwal had a wide

circulation

Q39) Answer (c)

Explanation

Moth ki Masjid was built in the reign of

Sikandar Lodi by his minister

Begumpuri mosque built in the reign of

Muhammad Tughluq was the main

mosque of Jahanpanah the ldquoSanctuary

of the Worldrdquo and his new capital in

Delhi

Quwwat al ndash Islam mosque was

enlarged by Iltutmish and Alauddin

Khalji The minar was built by three

Sultansndash Qutbuddin Aybak Iltutmish

and Firuz Shah Tughluq

RAUSIAS-FC19E1003 45

Q40) Answer (c)

Explanation

Under the Mughals mansabdar was

referred to an individual who held a

mansab ie rank and he received his

salary as revenue assignments called

jagirs

Q41) Ans (b)

The Quit India Movement was a

spontaneous revolt of people against

British rule

The All India Congress Committee met

at Bombay on 8 August 1942 It passed

the famous resolution Quit India and

proposed the starting of a non-violent

mass struggle under Gandhis

leadership to achieve this aim But on

the very next day Gandhi and other

eminent leaders of the Congress were

arrested The Congress was once again

declared illegal

Q42) Ans (c)

The Simon Commission refers to a

group of seven MPs from the United

Kingdom constituted to suggest

constitutional reforms for British India

The Commission consisted of only

British members headed by one of the

senior British politicians Sir John

Simon

So the people of India agitated against

the arrival of Simon Commission

Q43) Ans (a)

He was widely known for his

unfavourable opinion of the economic

consequences of the British rule in

India

In his many writings and speeches and

especially in Poverty and Un-British

Rule in India Naoroji argued that India

was too highly taxed and that its wealth

was being drained away to England

He did not interpret the ancient Indian

texts and restored the self-confidence of

Indians And also he did not stress the

need for eradication of all the social

evils before anything else

Q44) Ans (c)

In August 1932 Prime Minister

MacDonald announced his Communal

Award Great Britainrsquos unilateral

attempt to resolve the various conflicts

among Indiarsquos many communal

interests

The award which was later

incorporated into the act of 1935

expanded the separate-electorate

formula reserved for Muslims to other

minorities including Sikhs Indian

Christians Anglo-Indians Europeans

distinct regional groups Gandhi

undertook a ldquofast unto deathrdquo against

that offer which he viewed as a

nefarious British plot to divide the

Indian society

Q45) Ans (b)

In British India apart from existing

imports and exports there was also a

particular amount of money which

colonial India contributed towards

administration maintenance of the

army war expenses pensions to retired

officers and other expenses accrued by

Britain towards maintenance of her

colony These were known as Home

charges and were paid for almost

entirely by India

The Home charges was made of

following components-

- Interest payable on Indian debt

- Dividend to shareholders of East

India Company

- Funds used to support the India

Office in London

- Funds used to pay salaries and

pensions of British personnel

engaged in India

- Interest on the railways

- Civil and military charges

- Store purchases in England

Q46) Ans (b)

The Lahore session of the Indian

National Congress was held in 1929

under the Presidentship of Jawaharlal

Nehru

The Lahore session of the Indian

National Congress witnessed significant

RAUSIAS-FC19E1003 46

developments in the Indian national

movement

- First the election of Jawaharlal

Nehru to the post of Presidentship of

the Congress was a clear indication

of the growing strength of the

Leftists in the Congress

- Secondly it was in this session that

the Congress for the first time raised

the demand for complete

independence Such demand was

not raised from the Congress

platform earlier

Q47) Ans (b)

It did not provide for separate

electorates for any community or

weightage for minorities However it did

allow for the reservation of minority

seats in provinces having minorities of

at least ten per cent but this was to be

in strict proportion to the size of the

community

There was no provision for complete

Independence for India

Q48) Ans (c)

The religion of early Vedic Aryans was

primarily of worship of nature and

Yajnas

The early Aryan religion was kind of

nature worship Actually the forces

around them which they could not

control or understand were invested

with divinity and were personified as

male or female gods And they

performed some Yajnas also

Q49) Ans (b)

The roads and river-routes were not

immune from robbery It is notable that

Yuan Chwang (Hiuen Tsang) was

robbed of his belongings during

Harshvardanarsquos period

Q50) Ans (c)

Q51) Ans (b)

Purandara Dasa was a saint and great

devotee of Lord Krishna

There is much speculation about where

Purandara Dasa regarded as the

Pitamaha of Carnatic music was born

Recently an expert committee

constituted by the Kannada University

Hampi has come to the conclusion that

Kshemapura Shivamogga district

Karnataka is the birth place of

Purandara Dasa

Q52) Ans (c)

Sri Tyagaraja Sri Shyama Shastry and Sri Muthuswami Dikshitar are considered the trinity of Carnatic music and with them came the golden age in Carnatic music in the 18th-19th

century

Q53) Ans d)

Recently a rare sarcophagus (stone

coffin) which is 2000 years old from the

Iron AgendashMegalithic era was discovered

from a rock-cut cave at Viyur village of

Kollam near Koyilandy in Kozhikode

district Kerala

The coffin containing bone fragments

was found during an excavation ldquoSo

far such a rare finding has been

discovered only from two sites

in Kerala Both these sarcophagi were

recovered from Megalithic sites at

Chevayur and Atholi also in Kozhikode

district

Q54) Ans a)

The megalithic culture in South India was a full-fledged Iron Age culture

Q55) Ans d)

The Cholas Pandyas and Keralaputras

(Cheras) mentioned in Ashokan

inscriptions were probably in the late

megalithic phase of material culture

Q56) Ans d)

Q57) Ans (b)

Raj Kumar Shukla followed Gandhiji all

over the country to persuade him to

come to Champaran to investigate the

problem associated with tinkathia

system

RAUSIAS-FC19E1003 47

Brij Kishore Rajendra Prasad Mahadev

Desai and Narhari Parikh accompanied

Gandhi ji during the Champaran

Satyagraha

Q58) Ans (b)

The Satvahanas started the practice of granting tax-free villages to brahmanas and Buddhist monks

Q59) Ans c)

The objectives of the Programme are

listed as under

- Developing basic tourism

infrastructure

- Promoting cultural and heritage

value of the country to generate

livelihoods in the identified regions

- Enhancing the tourist attractiveness

in a sustainable manner by

developing world-class

infrastructure at the heritage

monument sites

- Creating employment through active

involvement of local communities

- Harnessing tourism potential for its

effects on employment generation

and economic development

- Developing sustainable tourism

infrastructure and ensuring proper

Operations and maintenance

therein

Q60) Ans (b)

The Tribal Cooperative Marketing

Development Federation of India

(TRIFED) came into existence in 1987

It is a national-level apex organization

functioning under the administrative

control of Ministry of Tribal Affairs

Govt of India

TRIFED has its registered and Head

Office located in New Delhi

Q61) Ans (c)

Premchandrsquos novels include

Premashram Rangabhumi Ghaban

Karmabhumi and Godan

Gora is a novel written by Rabindranath

Tagore

138th birth anniversary of Munshi

Premchand was celebrated across the

country

Q62) Ans (b)

Giddha is a traditional pastoral dance

performed by the women of the Punjab

India and Pakistan at festival times

and at the sowing and reaping of the

harvest

By this dance the Punjabi women

reveal their joy expel their suppressed

feelings in a male dominated society

through the performance of Giddha

Since this dance has nothing to do with

men only women can participate in it

During the Teej celebrations Giddha

dance is celebrated in Punjab every

year Teej is a generic name for a

number of festivals that are celebrated

by women in some parts of India

Q63) Ans (a)

Dara Shukoh wrote the remarkable

work called ldquoMajma-ul-Bahrainrdquo or the

ldquoThe confluence of two seasrdquo

The Vice President of India Shri M

Venkaiah Naidu has said that Prince

Dara Shukohrsquos writings can come as a

refreshing source for infusing peace and

harmony He was addressing the

gathering after visiting the exhibition

that showcases the forgotten Prince of

yesteryears Dara Shukoh organized by

Mr Francois Gautier at Indira Gandhi

National Centre for the Arts in New

Delhi

Q64) Ans (c)

The statue Gommateshwara is

dedicated to the Jain God Bahubali

It is a monolithic statue

President Ram Nath Kovind

inaugurated the grand anointing

ceremony mdash Mahamastakabhisheka mdash

held once in 12 years at

Shravanabelagola (Karnataka)

Q65) Ans (c)

Prachi Valley had come up around the

Prachi river Prachi Valley gradually

disappeared

RAUSIAS-FC19E1003 48

The Prachi river originates from

Bhubaneswar

It is a tributary of the Mahanadi and

flows through the districts of Puri

Khurda Cuttack and Jagatsinghpur

and the entire region of the river is

termed as the Prachi Valley

It falls into the Bay of Bengal

Archaeological evidence shows that the

Prachi Valley Civilisation predates both

Harappa and Mohenjo-Daro

The Prachi river originates from

Bhubaneswar

Q66) Ans (d)

These monuments are located in

Chhatarpur district Madhya Pradesh

within Vindhya mountain range

Q67) Ans (a)

The book lsquoThoughts on Pakistanrsquo was

written by Dr BR Ambedkar

On the occasion of the birth anniversary

of Dr BR Ambedkar the president of

India pays homage to this icon of India

In 1924 he founded the Depressed

Classes Institute (Bahishkrit Hitkarini

Sabha) and in 1927 the Samaj Samata

Sangh

Another area of attention for Ambedkar

was education For its spread among

the low classes he set up a network of

colleges by the name of Peoples

Education Society and founded hostels

Q68) Ans(b)

Mehrgarh is a famous Neolithic

settlement in the Indian subcontinent

which is situated in Baluchistan

province Pakistan

A pre-historic rock art site is discovered

in the vast expanse of limestone blocks

on the eastern banks of Naguleru river

near Dachepalli (Andhra Pradesh) It

has thrown light on the Neolithic

civilisation that flourished in Guntur

(Andhra Pradesh) during 1500-2000

BC

Q69) Ans (c)

The 12th and the 13th centuries saw

the emergence of the Kakatiyas They

were at first the feudatories of the

Western Chalukyas of Kalyana Initially

they ruled over a small territory near

Warangal (Telangana)

They introduced Nayakships which was

later adopted and developed by the

Rayas of Vijayanagara

Q70) Ans (a)

The fast had effect of putting pressure

on mill owners who finally agreed to

give the workers a 35 per cent increase

in wages

Google celebrated with a doodle the

132nd birth anniversary of Anasuya

Sarabhai who played a pioneering role

in Indiarsquos labour movement

Q71) Ans (d)

The UNESCOrsquos list of the representative

list of the intangible cultural heritage of

humanity from India are

- Koodiyattam Sanskrit Theatre of

Kerala

- Mudiyettu ritual theatre and dance

drama of Kerala

- Tradition of Vedic Chanting

- Kalbelia folk songs and dances of

Rajasthan

- Ramlila Traditional Performance of

the Ramayana

- Sankirtana ritual singing

drumming and dancing of Manipur

- Ramman religious festival and

ritual theatre of the Garhwal

Himalayas India

- Traditional brass and copper craft of

utensil making among the Thatheras

of Jandiala Guru Punjab India

- Chhau dance classical Indian dance

originated in the eastern Indian

states

- Buddhist chanting of Ladakh

recitation of sacred Buddhist texts

in the trans-Himalayan Ladakh

region Jammu and Kashmir India

- Yoga

- Nouroz

- Kumbh Mela

RAUSIAS-FC19E1003 49

Q72) Ans(b)

The President of India Shri Ram Nath Kovind inaugurated the Hornbill Festival and State Formation Day celebrations of Nagaland in Kisama

The festival is named after the Indian hornbill the large and colourful forest bird which is displayed in the folklore of most of the states tribes

The major recognized tribes of Nagaland are Angami Ao Chakhesang Chang

Kuki Rengma and Zeling etc

Onge Jarawa and Sentinelese are the

tribes of Andman amp Nicobar Islands

Q73) Ans (c)

The Rashtrakutas rule in the Deccan lasted for almost two hundred years till the end of the tenth century The Rashtrakutas rulers were tolerant in their religious views and patronized not only Shaivism and Vaishnavism but

Jainism as well

The famous rock-cut temple of Shiva at Ellora was built by one of the Rashtrakutas kings Krishna I in the ninth century His successor Amoghavarsha was a Jain but he also

patronized other faiths

The Rashtrakutas allowed Muslims traders to settle and permitted Islam to

be preached in their dominions

Recently increasing defacement at the prehistoric rock paintings of Pandavulagutta Telangana has created a cause for grave concern It can spoil

the prehistoric rock

Pandavulagutta is home to

- Painted rock shelters dating to

10000 BC-8000 BC

- An 8th century inscription of the

Rashtrakuta period and

- Painted frescoes from the 12th century Kakatiya empire

Q74) Ans (b)

In 1828 Raja Ram Mohan Roy founded a new religious society the Brahma Sabha later known as the Brahmo

Samaj

Debendranath Tagore headed the Tattvabodhini Sabha which was

engaged in search of spiritual truth

Its purpose was to purify Hinduism and to preach monotheism or belief in one God

The new society was to be based on the twin pillars of reason and the Vedas and

Upanishads

Recently Sadharan Brahmo Samaj (SBS) has entered into a legal battle with the West Bengal government due

to some legal issue

Q75) Ans (c)

The Chishti order was established in India by Khwaja Moinuddin Chishti who came to India around 1192 The Chishtirsquos are considered to be the most influential of the groups of Sufis who migrated to India in the late twelfth century They adapted successfully to the local environment and adopted several features of Indian devotional

traditions

The historical dargah of Sufi mystic Khwaja Moinuddin Chishti in Ajmer is all set to get a facelift This 13 th century dargah has been included among the Swachh Iconic Places a clean-up initiative focused on iconic

heritage spiritual and cultural places

Page 16: GENERAL STUDIES (PAPER I) · Test is part of Rau’s IAS Test series for Preliminary Exam 2019 FOUNDATION + CURRENT AFFAIRS GENERAL STUDIES (PAPER –I) FOUNDATION TEST –III TOPIC:

RAUSIAS-FC19E1003 16

Q44) महातमा गाोरी ि 1932 म आमरर अिशि नकया था

कय ोनक

(a) ldquoग िमज सममििrdquo (The Round Table

Conference) भारतीय राजिीनतक

आकाोकषाओो क परा करि म असफि रहा था

(b) काोगरस और मखसलम िीग म मतभद थ

(c) रामस मकड िालड (Ramsay Macdonald)

ि ldquoसाोपरदानयक परसकारrdquo (The Communal

Award) की घ िरा की थी

(d) ldquoसनििय अिजञा आोद ििrdquo (The Civil

Disobedience Movement) असफि रहा

था

Q45) भारत म औपनििनशक शासि की अिनर क सोदभण म

भारत स रि क बनहगणमि का एक महतवपरण भाग गह

शलक (Home Charges) था निमननिखित म स

कौि-सास क ि गह शलक म सखममनित नकया गया

थानकय गए थ

1 िोदि म भारत कायाणिय क निए उपय ग नकय

जाि िािा क ि

2 भारत म नियकत नबरनटश कनमणय ो क िति और

पशि का भगताि करि क निए उपय ग नकय

जाि िािा क ि

3 अोगरज ो क दवारा भारत क बाहर यदध ो क निए

उपय ग नकय जाि िािा क ि

िीच नदए गए कट का परय ग कर सही उततर चनिए

(a) किि 1

(b) किि 1 और 2

(c) किि 2 और 3

(d) 1 2 और 3

Q46) सवतोतरता आोद िि क इनतहास म भारतीय राषटर ीय

काोगरस का 1929 का सतर महतवपरण ह कय ोनक इसम

(a) काोगरस क उददशय क रप म सथािीय सरकार

की पराखपत की घ िरा की गई थी

(b) परण सवराज की पराखपत क काोगरस क िकषय क

रप म अपिाया गया था

(c) असहय ग आोद िि शर नकया गया था

(d) िोदि म ldquoग ि मर सममििrdquo (The Round

Table Conference) म भाग िि का निरणय

निया गया था

Q47) भारतीय सवतोतरता सोगराम क सोदभण म िहर ररप टण

क दवारा निमननिखित म स नकसकी नसफाररश की गई

थीनकिकी नसफाररश की गई थी ो

1 भारत क निए परण सवतोतरता

2 अलपसोखयक ो क निए सीट ो क आरकषर क

निए सोयकत नििाणचक मोडि

3 सोनिराि म भारत क ि ग ो क निए मौनिक

अनरकार ो का परािराि

िीच नदए गए कट का परय ग कर सही उततर चनिए

(a) किि 1

(b) किि 2 और 3

(c) किि 1 और 3

(d) 1 2 और 3

Q48) आरो नभक िनदक आयो का रमण मखय रप स था

(a) भखकत

(b) मनतण पजा और यजञ

(c) परकनत की पजा और यजञ

(d) परकनत की पजा और भखकत

RAUSIAS-FC19E1003 17

Q44) Mahatma Gandhi undertook fast unto

death in 1932 mainly because

(a) The Round Table Conference failed

to satisfy Indian political

aspirations

(b) The Congress and Muslim League

had differences of opinion

(c) Ramsay Macdonald announced the

Communal Award

(d) The Civil Disobedience Movement

failed

Q45) With reference to the period of colonial

rule in India ldquoHome Chargesrdquo formed

an important part of drain of wealth

from India Which of the following funds

constituted ldquoHome Chargesrdquo

1 Funds used to support the India

Office in London

2 Funds used to pay salaries and

pensions of British personnel

engaged in India

3 Funds used for waging wars

outside India by the British

Select the correct answer using the code

given below

(a) 1 only

(b) 1 and 2 only

(c) 2 and 3 only

(d) 1 2 and 3

Q46) The 1929- Session of Indian National

Congress is of significance in the history

of the Freedom Movement because the-

(a) attainment of Self-Government

was declared as the objective of

the Congress

(b) attainment of Poorna Swaraj was

adopted as the goal of the

Congress

(c) Non-Cooperation Movement was

launched

(d) decision to participate in the

Round Table Conference in

London was taken

Q47) With reference to the period of Indian

freedom struggle which of the following

waswere recommended by the Nehru

report

1 Complete Independence for India

2 Joint electorates for reservation of

seats for minorities

3 Provision of fundamental rights for

the people of India in the

Constitution

Select the correct answer using the code

given below

(a) 1 only

(b) 2 and 3 only

(c) 1 and 3 only

(d) 1 2 and 3

Q48) The religion of the early Vedic Aryans was primarily of

(a) Bhakti

(b) image worship and Yajnas

(c) worship of nature and Yajnas

(d) worship of nature and Bhakti

RAUSIAS-FC19E1003 18

Q49) भारत की यातरा करि िाि चीिी यातरी यआि चिाोग

(हयएि साोग) ि समकािीि भारत की सामानय

खसथनतय ो और सोसकनत क दजण नकया था इस सोदभण म

निमननिखित कथि ो म स कौि-सास सही हह

1 सड़क और िदी-मागण (जि-मागण) डकती स

परण रप स सरनकषत थ

2 जहा तक अपरार ो क निए दणड की बात ह

उसक निए नकसी भी वयखकत की निदोिता

अथिा उसक अपरार क निराणररत करि क

निए अनि जि और निि परि क माधयम क

सारि थ

3 वयापाररय ो क घाट ो और परनतबोर सटशि ो पर

शलक ो का भगताि करिा पड़ता था

िीच नदए गए कट का परय ग कर सही उततर चनिए

(a) किि 1

(b) किि 2 और 3

(c) किि 1 और 3

(d) 1 2 और 3

Q50) नसोर घाटी सभयता क सोदभण म निमननिखित कथि ो पर

निचार कीनजए

1 यह मखय रप स एक रमणनिरपकष सभयता थी

तथा हािाोनक इसम रानमणक ततव मौजद था

िनकि िह परनतिश पर हािी िही ो था

2 इस काि क दौराि भारत म कपास का परय ग

कपड़ा बिाि क निए नकया जाता था

उपयणकत कथि ो म स कौि-सास सही हह

(a) किि 1

(b) किि 2

(c) 1 और 2 द ि ो

(d) ि त 1 ि ही 2

Q51) परोदर दास क सोदभण म निमननिखित कथि ो पर निचार

कीनजए

1 परोदर दास एक सोत और भगिाि नशि क एक

महाि भकत थ

2 ि एक सोगीतकार गायक और किाणटक सोगीत

क मखय सोसथापक-परसतािक ो म स एक थ

उपयणकत कथि ो म स कौि-सास सही हह

(a) किि 1

(b) किि 2

(c) 1 और 2 द ि ो

(d) ि त 1 ि ही 2

Q52) निमननिखित म स कौि-सास वयखकत किाणटक सोगीत

की नतरमनतण म शानमि हह

1 बािामरिी कषणा

2 शरी शयाम शासतरी

3 शरी मथसवामी दीनकषतर

िीच नदए गए कट का परय ग कर सही उततर चनिए

(a) किि 1

(b) किि 2

(c) किि 2 और 3

(d) 1 2 और 3

Q53) चियर (Chevayur) और अथ िी (Atholi) म खसथत

महापािार सथि निमननिखित म स नकस राजय म खसथत

(a) तनमििाड

(b) किाणटक

(c) पनिम बोगाि

(d) करि

RAUSIAS-FC19E1003 19

Q49) The Chinese traveller Yuan Chwang

(Hiuen Tsang) who visited India

recorded the general conditions and

culture of India at that time In this

context which of the following

statements isare correct

1 The roads and river-routes were

completely immune from robbery

2 As regards punishment for

offences ordeals by fire water and

poison were the instruments for

determining the innocence or guilt

of a person

3 The tradesmen had to pay duties

at ferries and barrier stations

Select the correct answer using the code

given below

(a) 1 only

(b) 2 and 3 only

(c) 1 and 3 only

(d) 1 2 and 3

Q50) Regarding the Indus Valley Civilization

consider the following statements

1 It was predominantly a secular

civilization and the religious

element though present did not

dominate the scene

2 During this period cotton was

used for manufacturing textiles in

India

Which of the statements given above

isare correct

(a) 1 only

(b) 2 only

(c) Both 1 and 2

(d) Neither 1 nor 2

Q51) Consider the following statements

regarding Purandara Dasa

1 Purandara Dasa was a saint and

great devotee of Lord Shiva

2 He was a composer singer and

one of the chief founding-

proponents of the Carnatic music

Which of the statements given above

isare correct

(a) 1 only

(b) 2 only

(c) Both 1 and 2

(d) Neither 1 nor 2

Q52) Which of the following persons isare

included in the trinity of Carnatic

music

1 Balamurali Krishna

2 Sri Shyama Shastry

3 Sri Muthuswami Dikshitar

Select the correct answer using the code

given below

(a) 1 only

(b) 2 only

(c) 2 and 3 only

(d) 1 2 and 3

Q53) Megalithic sites at Chevayur and Atholi

are located in which of the following

states

(a) Tamil Nadu

(b) Karnataka

(c) West Bengal

(d) Kerala

RAUSIAS-FC19E1003 20

Q54) निमननिखित कथि ो पर निचार कीनजए

1 महापािानरक ि ग कबर ो म िसतएो दफिात थ

2 दनकषर भारत म महापािार सोसकनत एक परण

निकनसत तामर यगीि सोसकनत थी

उपयणकत कथि ो म स कौि-सास सही हह

(a) किि 1

(b) किि 2

(c) 1 और 2 द ि ो

(d) ि त 1 ि ही 2

Q55) निमननिखित म स कौि-स सामराजयसामराजय ो का

अश क क अनभिि ो म उललि नकया गया ह

1 च ि

2 पाणडय

3 करिपतर (चर)

िीच नदए गए कट का परय ग कर सही उततर चनिए

(a) किि 1

(b) किि 1 और 2

(c) किि 3

(d) 1 2 और 3

Q56) भीमा-क रगाोि का यदध को पिी क सनिक ो और

बाजीराि नदवतीय क िततव म एक शखकतशािी पशिा

सिा (मराठ ो) क मधय िड़ा गया था यह यदध

निमननिखित म स नकसका नहससा था

(a) परथम आोगल-मराठा यदध का

(b) नदवतीय आोगल-मराठा यदध का

(c) ततीय आोगल-मसर यदध का

(d) ततीय आोगल-मराठा यदध का

Q57) निमननिखित कथि ो पर निचार कीनजए

1 महादि दसाई ि गाोरीजी क चोपारर आि तथा

नतिकनथया पररािी स जड़ी समसया की जाोच

क निए रारी करि क निए दश भर म उिका

अिसरर नकया था

2 िरहरी पाररि चोपारर सतयागरह क दौराि

गाोरीजी क साथ थ

उपयणकत कथि ो म स कौि-सास सही हह

(a) किि 1

(b) किि 2

(c) 1 और 2 द ि ो

(d) ि त 1 ि ही 2

Q58) निमननिखित कथि ो पर निचार कीनजए

1 िनद राज-िोश ि बराहमर ो और बौदध मठराररय ो

क कर-मकत गाि अिदाि म दि की परथा

आरि की थी

2 सतिाहि ो की आनरकाररक भािा पराकत थी

उपयणकत कथि ो म स कौि-सास सही हह

(a) किि 1

(b) किि 2

(c) 1 और 2 द ि ो

(d) ि त 1 ि ही 2

Q59) एक निरासत क अपिाइए (अडॉपट ए हररटज ndash

Adopt a Heritage) पररय जिा क उददशय ो क

सनदभण म निमननिखित कथि ो पर निचार कीनजए

1 यह पररय जिा र रगार उतपादि और आनथणक

निकास क निए पयणटि कषमता का उि पर

परभाि का उपय ग करगी

2 यह पररय जिा निरासत सथि ो पर निशव सतरीय

आराररक सोरचिा निकनसत करक एक सतत

तरीक स पयणटक आकिणर म िखदध करगी

उपयणकत कथि ो म स कौि-सास सही हह

(a) किि 1

(b) किि 2

(c) 1 और 2 द ि ो

(d) ि त 1 ि ही 2

RAUSIAS-FC19E1003 21

Q54) Consider the following statements

1 Megalithic people buried goods in

graves

2 The megalithic culture in South

India was a full-fledged Copper

Age culture

Which of the statements given above

isare correct

(a) 1 only

(b) 2 only

(c) Both 1 and 2

(d) Neither 1 nor 2

Q55) Which of the following kingdoms isare

mentioned in the Ashokan inscriptions

1 Cholas

2 Pandyas

3 Keralaputras (Cheras)

Select the correct answer using the code

given below

(a) 1 only

(b) 1 and 2 only

(c) 3 only

(d) 1 2 and 3

Q56) The Battle of Bhima-Koregaon was

fought between the soldiers of the

Company and the strong Peshwa army

(Marathas) under Bajirao II This war

was a part of the

(a) First Anglo-Maratha war

(b) Second Anglo-Maratha war

(c) Third Anglo- Mysore war

(d) Third Anglo-Maratha war

Q57) Consider the following statements

1 Mahadev Desai followed Gandhiji all over the country to persuade him to come to Champaran to investigate the problem associated

with tinkathia system

2 Narhari Parikh accompanied Gandhi ji during the Champaran

Satyagraha

Which of the statements given above isare correct

(a) 1 only

(b) 2 only

(c) Both 1 and 2

(d) Neither 1 nor 2

Q58) Consider the following statements

1 The Nanda Dynasty started the practice of granting tax-free villages to brahmanas and

Buddhist monks

2 The official language of the Satavahanas was Prakrit

Which of the statements given above

isare correct

(a) 1 only

(b) 2 only

(c) Both 1 and 2

(d) Neither 1 nor 2

Q59) Consider the following statements about the objectives of the lsquoadopt a heritagersquo

project

1 It will harness tourism potential for its effects on employment generation and economic

development

2 It will enhance the tourist attractiveness in a sustainable manner by developing world class infrastructure at heritage sites

Which of the statements given above

isare correct

(a) 1 only

(b) 2 only

(c) Both 1 and 2

(d) Neither 1 nor 2

RAUSIAS-FC19E1003 22

Q60) ldquoभारतीय जिजातीय सहकारी निपरि निकास सोघrdquo

(The Tribal Co-operative Marketing

Development Federation of India - TRIFED)

क सोदभण म निमननिखित कथि ो पर निचार कीनजए

1 यह एक राषटर ीय सतर का शीिण सोगठि ह ज

भारत सरकार क गह मोतरािय क परशासनिक

नियोतरर क अरीि काम कर रहा ह

2 इसका मखय उददशय दश म जिजातीय ि ग ो

का सामानजक-आनथणक निकास करिा ह

उपयणकत कथि ो म स कौि-सास सही हह

(a) किि 1

(b) किि 2

(c) 1 और 2 द ि ो

(d) ि त 1 ि ही 2

Q61) निमननिखित म स कौि-सास उपनयास परमचोद क

दवारा नििा गया हनिि गए ह

1 रोगभनम

2 ग दाि

3 ग रा

िीच नदए गए कट का परय ग कर सही उततर चनिए

(a) किि 1

(b) किि 2

(c) किि 1 और 2

(d) 1 2 और 3

Q62) नगदधा ितय क सोदभण म निमननिखित कथि ो पर निचार

कीनजए

1 नगदधा नबहार की मनहिाओो क दवारा तयौहार क

समय और फसि की बिाई तथा कटाई क

अिसर पर नकया जाि िािा एक पारोपररक

दहाती ितय ह

2 इस ितय क दवारा मनहिाऐो अपिी परसननता

परकट करती ह तथा नगदधा क परदशणि क

माधयम स परि िचणसव िाि समाज म

मनहिाओो की दबी हई भाििाओो क परकट

करती ह

उपयणकत कथि ो म स कौि-सास सही हह

(a) किि 1

(b) किि 2

(c) 1 और 2 द ि ो

(d) ि त 1 ि ही 2

Q63) निमननिखित कथि ो पर निचार कीनजए

1 मलला शाह बदखशी दारा नशक ह क

आधयाखतमक गर थ

2 औरोगरब ि मजम-उि-बहरीि या द समदर ो

का सोगम िामक उललििीय रचिा नििी थी

3 दारा नशक ह क अपि पिणज अकबर क गर ो

क उततरानरकारी क रप म दिा गया था

नजसम उसि रानमणक बहििाद और समनवयता

क बढ़ािा नदया था

उपयणकत कथि ो म स कौि-सास सही हह

(a) किि 1 और 3

(b) किि 2

(c) किि 1 और 2

(d) 1 2 और 3

RAUSIAS-FC19E1003 23

Q60) Consider the following statements about

the Tribal Cooperative Marketing

Development Federation of India

(TRIFED)

1 It is a national-level apex

organization functioning under the

administrative control of Ministry

of Home Affairs Government of

India

2 The main objective of TRIFED is

socio-economic development of

tribal people in the country

Which of the statements given above

isare correct

(a) 1 only

(b) 2 only

(c) Both 1 and 2

(d) Neither 1 nor 2

Q61) Which of the following novels isare

written by Premchand

1 Rangabhumi

2 Godan

3 Gora

Select the correct answer using the code

given below

(a) 1 only

(b) 2 only

(c) 1 and 2 only

(d) 1 2 and 3

Q62) Consider the following statements about

Giddha dance

1 Giddha is a traditional pastoral

dance performed by the women of

Bihar at festival times and at the

sowing and reaping of the harvest

2 By this dance the women reveal

their joy expel their suppressed

feelings in a male dominated

society through the performance of

Giddha

Which of the statements given above

isare correct

(a) 1 only

(b) 2 only

(c) Both 1 and 2

(d) Neither 1 nor 2

Q63) Consider the following statements

1 Mullah Shah Badakhshi was the

spiritual mentor of Dara Shukoh

2 Aurangzeb wrote the remarkable

work called ldquoMajma-ul-Bahrainrdquo or

the ldquoThe confluence of two seasrdquo

3 Dara Shukoh was seen as

inheriting the qualities of his

ancestor Akbar in that he

promoted religious pluralism and

syncretism

Which of the statements given above

isare correct

(a) 1 and 3 only

(b) 2 only

(c) 1 and 2 only

(d) 1 2 and 3

RAUSIAS-FC19E1003 24

Q64) निमननिखित कथि ो पर निचार कीनजए

1 ग मतशवर परनतमा निोधयनगरी पहाड़ी पर खसथत ह

2 शरिरबिग िा िह सथाि ह जहाो मौयण िोश क

सोसथापक चोदरगपत मौयण अपि नसोहासि क

तयागि क बाद जि तपसवी बि गए थ

उपयणकत कथि ो म स कौि-सास सही हह

(a) किि 1

(b) किि 2

(c) 1 और 2 द ि ो

(d) ि त 1 ि ही 2

Q65) निमननिखित कथि ो पर निचार कीनजए

1 पराताखतवक साकषय स पता चिता ह नक पराची

घाटी सभयता हड़पपा और म हिज दाड़ द ि ो

की पिणिती ह

2 पराची िदी भििशवर स निकिती ह

उपयणकत कथि ो म स कौि-सास सही हह

(a) किि 1

(b) किि 2

(c) 1 और 2 द ि ो

(d) ि त 1 ि ही 2

Q66) निमननिखित कथि ो म स कौि-सास सही हह

1 िजराह क समारक ो क समह का निमाणर

चोदि राजिोश क शासिकाि क दौराि हआ

था

2 य समारक हररिोदर पिणत शरोििा म खसथत ह

3 म रक क यातरी इबन बतता ि अपि सोसमरर ो

म िजराह क मोनदर ो की यातरा का उललि

नकया था तथा इन काजराण िाम स समब नरत

नकया था

िीच नदए गए कट का परय ग कर सही उततर चनिए

(a) किि 1

(b) किि 1 और 2

(c) किि 2 और 3

(d) किि 1 और 3

Q67) निमननिखित कथि ो म स कौि-सास सही हह

1 डॉ बी आर अमबडकर ि दी एनिनहिशि

ऑफ़ कासट (The Annihilation of Caste)

नििी थी नजसम उन ोि नहोद रमण म िोशािगत

पजारी की परथा क उनमिि की आिशयकता

पर बि नदया था

2 डॉ राजदर परसाद ि थॉटस ऑि पानकसताि

(Thoughts on Pakistan) िामक पसतक

नििी थी

िीच नदए गए कट का परय ग कर सही उततर चनिए

(a) किि 1

(b) किि 2

(c) 1 और 2 द ि ो

(d) ि त 1 ि ही 2

Q68) निमननिखित कथि ो म स कौि-सास सही हह

1 महरगढ़ भारतीय उपमहादवीप म एक परनसदध

ििपािार बसती ह ज नसोर पराोत पानकसताि म

खसथत ह

2 बरणह म म कतत ो क उिक सवामी क साथ कबर ो

म दफिाया जाता था

िीच नदए गए कट का परय ग कर सही उततर चनिए

(a) किि 1

(b) किि 2

(c) 1 और 2 द ि ो

(d) ि त 1 ि ही 2

Q69) निमननिखित कथि ो म स कौि-सास सही हह

1 काकानटय मोनदर अनरकतर नशि क समनपणत

2 हिमक ोडा म हजार-सतोभ िाि मोनदर (The

Thousand-Pillared Temple) का निमाणर

काकानटय समराट रदर ि करिाया था

िीच नदए गए कट का परय ग कर सही उततर चनिए

(a) किि 1

(b) किि 2

(c) 1 और 2 द ि ो

(d) ि त 1 ि ही 2

RAUSIAS-FC19E1003 25

Q64) Consider the following statements

1 Gommateshwara Statue is located

on the Vindyagiri Hill

2 Shravanabelagola is the place

where Chandragupta Maurya the

founder of the Mauryan dynasty

became a Jain ascetic after

relinquishing his throne

Which of the statements given above

isare correct

(a) 1 only

(b) 2 only

(c) Both 1 and 2

(d) Neither 1 nor 2

Q65) Consider the following statements

1 Archaeological evidence shows

that the Prachi Valley Civilisation

predates both Harappa and

Mohenjo-Daro

2 The Prachi river originates from

Bhubaneswar

Which of the statements given above

isare correct

(a) 1 only

(b) 2 only

(c) Both 1 and 2

(d) Neither 1 nor 2

Q66) Which of the following statements

isare correct

1 The Khajuraho group of

monuments was built during the

rule of the Chandela dynasty

2 These monuments are located in

Harischandra mountain range

3 Ibn Battuta the Moroccan

traveller in his memoirs mentioned

visiting Khajuraho temples and

called them Kajarra

Select the correct answer using the code

given below

(a) 1 only

(b) 1 and 2

(c) 2 and 3

(d) 1 and 3

Q67) Which of the following statements

isare correct

1 Dr BR Ambedkar wrote the

Annihilation of Caste emphasising

the need to do away with the

practice of hereditary priesthood in

Hinduism

2 The book lsquoThoughts on Pakistanrsquo

was written by Dr Rajendra

Prasad

Select the correct answer using the code

given below

(a) 1 only

(b) 2 only

(c) Both 1 and 2

(d) Neither 1 nor 2

Q68) Which of the following statements

isare correct

1 Mehrgarh is a famous Neolithic

settlement in the Indian

subcontinent which is situated in

Sindh province Pakistan

2 At Burzahom dogs were buried

with their masters in their graves

Select the correct answer using the code

given below

(a) 1 only

(b) 2 only

(c) Both 1 and 2

(d) Neither 1 nor 2

Q69) Which of the following statements

isare correct

1 The Kakatiya temples are

dedicated mostly to Siva

2 The Thousand-Pillared Temple at

Hanamkonda was built by the

Kakatiya king Rudra

Select the correct answer using the code

given below

(a) 1 only

(b) 2 only

(c) Both 1 and 2

(d) Neither 1 nor 2

RAUSIAS-FC19E1003 26

Q70) निमननिखित कथि ो म स कौि-सास सही हह

1 अहमदाबाद नमि हड़ताि क दौराि महातमा

गाोरी ि शरनमक ो क पकष क मजबत करि क

निए आमरर अिशि नकया था

2 अिशि स नमि मानिक ो पर दबाि पड़ा था ज

अोततः शरनमक ो क िति म 15 परनतशत की िखदध

करि क निए सहमत हए थ

िीच नदए गए कट का परय ग कर सही उततर चनिए

(a) किि 1

(b) किि 2

(c) 1 और 2 द ि ो

(d) ि त 1 ि ही 2

Q71) निमननिखित म स नकसक नकिक भारत स यिसक

की माििता की अमतण साोसकनतक निरासत की

परनतनिनर सची (The UNESCOrsquos List of the

Representative List of the Intangible

Cultural Heritage of Humanity) म शानमि

नकया गया ह

1 मनडयटट

2 सोकीतणि

3 को भ मिा

िीच नदए गए कट का परय ग कर सही उततर चनिए

(a) किि 1 और 2

(b) किि 2 और 3

(c) किि 3

(d) 1 2 और 3

Q72) निमननिखित जिजानतय ो म स कौि-सीसी ो

जिजानतजिजानतया िागािड स सोबोनरत हह

1 अोगामी

2 ककी

3 जारिा

िीच नदए गए कट का परय ग कर सही उततर चनिए

(a) किि 1

(b) किि 1 औऔ 2

(c) किि 2

(d) 1 2 और 3

Q73) निमननिखित कथि ो म स कौि-सास सही हह

1 राषटर कट सामराजय की सथापिा दोनतदगण ि की थी

नजसि मानयाित म अपिी राजरािी की

सथापिा की थी

2 राषटर कट समराट अम घििण एक ििक था और

उस कनिताओो पर पहिी कननड़ पसतक नििि

का शरय नदया जाता ह

िीच नदए गए कट का परय ग कर सही उततर चनिए

(a) किि 1

(b) किि 2

(c) 1 और 2 द ि ो

(d) ि त 1 ि ही 2

Q74) निमननिखित कथि ो म स कौि-सास सही हह

1 कशब चोदर सि ि ततवब नरिी सभा की

अधयकषता की थी ज आधयाखतमक सतय की

ि ज म सोिि थी

2 बरहम समाज ि मािि गररमा पर बि नदया

मनतणपजा का निर र नकया और सती परथा जसी

सामानजक बराइय ो की आि चिा की

िीच नदए गए कट का परय ग कर सही उततर चनिए

(a) किि 1

(b) किि 2

(c) 1 और 2 द ि ो

(d) ि त 1 ि ही 2

Q75) निमननिखित कथि ो म स कौि-सास सही हह

1 भारत म नचशती नसिनसिा खवाजा म इिददीि

नचशती क दवारा सथानपत नकया गया था

2 नचशती परोपरा की एक परमि निशिता

आतमसोयम थी नजसम साोसाररक म ह स दरी

बिाए रििा शानमि था

िीच नदए गए कट का परय ग कर सही उततर चनिए

(a) किि 1

(b) किि 2

(c) 1 और 2 द ि ो

(d) ि त 1 ि ही 2

RAUSIAS-FC19E1003 27

Q70) Which of the following statements

isare correct

1 During the Ahmedabad Mill Strike

Mahatma Gandhi undertook a fast

unto death to strengthen the

workersrsquo resolve

2 The fast had effect of putting

pressure on mill owners who

finally agreed to give the workers a

15 per cent increase in wages

Select the correct answer using the code

given below

(a) 1 only

(b) 2 only

(c) Both 1 and 2

(d) Neither 1 nor 2

Q71) Which of the following are included in

the UNESCOrsquos list of the representative

list of the intangible cultural heritage of

humanity from India

1 Mudiyettu

2 Sankirtana

3 Kumbh Mela

Select the correct answer using the code

given below

(a) 1 and 2 only

(b) 2 and 3 only

(c) 3 only

(d) 1 2 and 3

Q72) Which of the following tribes isare

related to Nagaland

1 Angami

2 Kuki

3 Jarawa

Select the correct answer using the code

given below

(a) 1 only

(b) 1 and 2 only

(c) 2 only

(d) 1 2 and 3

Q73) Which of the following statements

isare correct

1 Rashtrakuta kingdom was founded by Dantidurga who established his capital at Manyakhet

2 Amoghavarsha a Rashtrakuta king was an author and is credited with writing the first

Kannada book on poetics

Select the correct answer using the code given below

(a) 1 only

(b) 2 only

(c) Both 1 and 2

(d) Neither 1 nor 2

Q74) Which of the following statements isare correct

1 Keshab Chandra Sen headed the Tattvabodhini Sabha which was engaged in search of spiritual truth

2 The Brahmo Samaj laid emphasis on human dignity opposed idolatry and criticized such social

evils as the practice of Sati

Select the correct answer using the code given below

(a) 1 only

(b) 2 only

(c) Both 1 and 2

(d) Neither 1 nor 2

Q75) Which of the following statements isare correct

1 The Chishti order was established in India by Khwaja Moinuddin

Chishti

2 A major feature of the Chishti tradition was austerity including maintaining a distance from the

worldly power

Select the correct answer using the code

given below

(a) 1 only

(b) 2 only

(c) Both 1 and 2

(d) Neither 1 nor 2

T e s t i s p a r t o f R a u rsquo s I A S T e s t s e r i e s f o r P r e l i m i n a r y E x a m 2 0 1 9

FOUNDATION + CURRENT AFFAIRS

GENERAL STUDIES (PAPER ndashI)

FOUNDATION TEST ndashIII

SUBJECT NCERT History Class VI-X + Current Affairs

Time Allowed 1frac12 Hours Maximum Marks 150

I NSTRUCT IONS

1 IMMEDIATELY AFTER THE COMMENCEMENT OF THE EXAMINATION YOU SHOULD CHECK

THAT THIS TEST BOOKLET DOES NOT HAVE ANY UNPRINTED OR TORN or MISSING PAGES OR

ITEMS ETC IF SO GET IT REPLACED BY A COMPLETE TEST BOOKLET

2 This Test Booklet contains 75 items (questions) Each item is printed both in Hindi and English

Each item comprises four responses (answers) You will select the response which you want to mark

on the Answer Sheet In case you feel that there is more than one correct response mark the

response which you consider the best In any case choose ONLY ONE response for each item

3 You have to mark all your responses ONLY on the separate Answer Sheet (OMR sheet) provided

Read the directions in the Answer Sheet

4 All items carry equal marks

5 Before you proceed to mark in the Answer Sheet the response to various items in the Test booklet

you have to fill in some particulars in the Answer Sheet as per instructions contained therein

6 After you have completed filling in all your responses on the Answer Sheet and the examination has

concluded you should hand over to the Invigilator only the Answer Sheet You are permitted to

take away with you the Test Booklet

7 Penalty for wrong answers

THERE WILL BE PENALTY FOR WRONG ANSWERS MARKED BY A CANDIDATE IN THE

OBJECTIVE TYPE QUESTION PAPERS

(i) There are four alternatives for the answer to every question For each question for which a

wrong answer has been given by the candidate one-third of the marks assigned to that

question will be deducted as penalty

(ii) If a candidate gives more than one answer it will be treated as a wrong answer even if one of

the given answers happens to be correct and there will be same penalty as above to that

question

(iii) If a question is left blank ie no answer is given by the candidate there will be no penalty for

that question

T h i s t e s t i s p a r t o f R a u rsquo s I A S T e s t s e r i e s f o r P r e l i m i n a r y E x a m 2 0 1 9

Test Code

FC19E1003

FC19H1003 29

Answers and Explanations of

NCERT History Class VI-X + Current Affairs (FC19E1003)

Q1) उततर (c)

सपषटीकरण

- ऋगवद म दविय ो और दिताओो क समवपित एक

हजार स अविक सत तर (शल क) ह

- य शल क ऋविय ो क दवारा रच गए थ और परि ो

दवारा सीख जात थ

- हालाोवक कछ शल क मवहलाओो (जस वक अपाला

घ सा ल पामदरा मतरयी और गागी) क दवारा भी रच

गए थ

- ऋगवद म सोिाद क रप म कई शल क मौजद ह

- हम विशवावमतर नामक एक ऋवि और दविय ो क

रप म पजी जान िाली द नवदय ो (वयास और

सतलज) क बीच िाताि का उदाहरण वमलता ह

- इसस पता चलता ह वक विशवावमतर िवदक काल स

सोबोवित थ

Q2) उततर (b)

सपषटीकरण

- करनल गफाओो स राख क अिशि परापत हए ह

ज इस ओर सोकत करत ह वक ततकालीन ल ग

अवि क उपय ग स पररवचत थ

- य गफाएो आोधर परदश म सथथत ह

Q3) उततर (c)

सपषटीकरण

bull बरािह म ितिमान कशमीर म सथथत एक

परागवतहावसक थथल ह जहाो ल ग गडढ क घर ो का

वनमािण करत थ

bull य घर जमीन क ख द कर बनाए जात थ तथा नीच

जान क वलए सीवियाा ह ती थी

bull ऐसा अनमान लगाया जाता ह वक य घर ठो ड क

मौसम म आशरय परदान करत थ

Q4) उततर (c)

सपषटीकरण

bull परालख-विदया (Epigraphy) क वशलालख ो क

अधययन क रप म पररभावित वकया जाता ह

bull हसतवलसखत दसतािज ो क माधयम स इवतहास

और सावहतय क अधययन क पाोडवलवप विजञान

(Manuscriptology) कहत ह

bull पराचीन लखन परणावलय ो क अधययन और

ऐवतहावसक पाोडवलवपय ो क समझन तथा वतवथ

वनिािरण क पलीओगराफी (Palaeography) कहा

जाता ह

bull नयवमजमविकस (Numismatics) वसक ो क

अधययन क सोदवभित करता ह

Q5) उततर (a)

सपषटीकरण

- चरक सोवहता चरक क दवारा वलखी गई आयिद

और िदयक-शासर पर एक महतवपणि पसतक ह

- ि भारतीय िदयक-शासर की पारमपररक परणाली

वजस आयिद क नाम स जाना जाता ह क

अभयासकताि थ

- ऐसा माना जाता ह वक चरक का विकास दसरी

शताबदी (ईसा पिि) और दसरी शताबदी (ईसवी) क

मधय हआ था

Q6) उततर (b)

सपषटीकरण

- भाग फसल ो पर वलए जान िाल कर क सोदवभित

करता ह ज कल फसल उतपादन का 16 िाो भाग

था

- ldquoकममकारrdquo शबद भवमहीन कवि शरवमक िगि क

वलए परय ग वकया जाता था

- ldquoअशवमिrdquo (वजस घ ड क बवलदान क रप म भी

जाना जाता ह) एक अनषठान ह ता था वजसम एक

घ ड क सवतोतर रप स घमन क वलए छ ड वदया

FC19H1003 30

जाता ह और राजा क सवनक उसकी रखिाली

करत थ

Q7) उततर (d)

सपषटीकरण

- ऋगववदक काल म घ ड ो क रथ ो म ज ता जाता था

ज (रथ) भवम मिवशय ो आवद पर कबजा करन क

वलए लड गए यद ो म उपय ग वकए जात थ

- इसस यह पता चलता ह वक घ ड ो यकत रथ ो का

उपय ग महाजनपद काल स काफी पहल आरमभ

हआ था

- ऋगववदक काल म मिवशय ो भवम जल आवद पर

कबजा करन क वलए तथा ल ग ो क पकडन क

वलए यद वकय जात थ

- अविकाोश परि इन यद ो म भाग वलया करत थ

- हालाोवक उस समय क ई वनयवमत सना नही ो ह ती

थी लवकन उस काल म सभाऐो ह ती थी ो वजनम

ल ग यद क मामल ो पर चचाि करत थ

- वनयवमत सनाएा महाजनपद काल का िवशषटय थी

वजनम पदल सवनक ो की विशाल सनाएा रथ तथा

हाथी शावमल ह त थ

Q8) उततर (a)

सपषटीकरण

- बद शाकय कल स सोबोवित थ और कशीनारा म

उनका वनिन हआ था

- बद न अपनी वशकषाएा पराकत भािा म दी थी ो ज

आम ल ग ो की भािा थी

Q9) उततर (c)

सपषटीकरण

- पराचीन भारत म दशिनशासर की छह शाखाएा थी ो

िशविक नयाय समखया य ग पिि वममाोसा और

िदाोत या उततर वममाोसा

- इनकी थथापना करमश कनाद गौतम कवपल

पतोजवल जावमनी और वयास ऋविय ो न की थी

Q10) उततर (b)

सपषटीकरण

महािीर की वशकषाऐो छठी शताबदी म िललभी म

सोकवलत की गई थी ो

Q11) उततर (c)

सपषटीकरण

- पारमपररक रप स चाणकय क कौविलय अथिा

विषणगपत क नाम स जाना जाता ह

- उसन अथिशासतर ज एक पराचीन भारतीय

राजनवतक आलख ह वलखा था

Q12) उततर (d)

सपषटीकरण

- भारत का राषटर ीय वचनह सारनाथ (उततर परदश) क

अश क सतमभ क ऊपर (शीिि पर) वसोह कवपिल

का एक अनरपण ह

- इस राषटर ीय वसदाोत सतयमि जयत क साथ

सोय वजत वकया गया ह

- रामपिि बल का नाम रामपिि (वबहार) क नाम पर

पडा जहाा इसकी ख ज हई थी

- यह अपन नाजक नकाशी मॉडल क वलए परवसदद

ह वजसम क मल तवचा सोिदनशील नथन ो सतकि

कान और मरबत िााग ो क शरषठतर परवतरप क

परदवशित वकया गया ह

- यह भारतीय और फारसी ततव ो का एक ससममशरण

- सोवकससा उततर परदश म सथथत ह

Q13) उततर (a)

सपषटीकरण

का िर वसोह ज एक महान य दा थ वबहार स

सोबोवित थ

Q14) उततर (b)

सपषटीकरण

िललालर शबद बड भ-सवावमय ो क वलए परय ग

वकया जाता था

FC19H1003 31

Q15) उततर (c)

सपषटीकरण

- अररकमड एक तिीय बसती थी जहाो दर दश ो स

आन िाल जहाज ो का माल उतारा जाता था

- यहाो पर ईोि ो का एक विशाल ग दाम वमटटी क

बतिन (वजनम एमफ रा - द हरी मवठय ो का लोबा

घडा - शावमल ह) और एरिाइन (Arretine)

मदभाोड पाए गए थ

- इस थथान पर र मन दीपक काोच क बन पातर और

रतन भी पाए गए थ

Q16) उततर (a)

सपषटीकरण

- मिनदर सोगम कविताओो म उसललसखत एक

तवमल शबद ह वजसका अथि ह ldquoतीन परमखrdquo

- यह तीन सततारि पररिार ो क मसखयाओो क वलए

परय ग वकया जाता ह च ल चर और पाणडय

Q17) उततर (c)

सपषटीकरण

- ऋग िद म सभा विदाथा तथा गण जसी

जनजावतय ो पर अथिा किोब पर आिाररत

सभाओो का उललख ह

- आरसमभक िवदक काल म सभाओो और सवमवतय ो

का विशि महतव ह ता था

- यहाा तक की मसखया अथिा राजा भी उनका

समथिन परापत करन क वलए आतर रहत थ

Q18) उततर (a)

सपषटीकरण

- जन िमि न ईशवर क अससततव क मानयता त दी ह

वकनत उसन ईशवर क वजना क पद स नीच रखा

- जन िमि न बौद िमि की तरह िणि परणाली की

भरतिना नही ो की थी

Q19) उततर (d)

सपषटीकरण

- च ल ो और पाणडय ो न शसकतशाली तिीय शहर ो का

विकास वकया था

- च ल ो का सबस महतवपणि शहर पहार (या

कािरीपटटीनम) था |

- मदरई पाणडय ो की राजिानी थी

Q20) उततर (b)

सपषटीकरण

- ldquoबदचररतrdquo बद का जीिन-ितताोत ह

- इस अशवघ ि क दवारा वलखा गया था

Q21) उततर (a)

सपषटीकरणः

- तवमल कवि अपपर भगिान वशि क भकत थ

- इस परकार ि एक नयनार सोत थ

Q22) उततर (d)

सपषटीकरणः

- समदरगपत एक परवसद गपत शासक था

- उसन वसक ो पर िीणा बजात हए अपनी छवि

अोवकत करिाई थी

- यह सोगीत क परवत उसक परम क दशािता ह

- हम उसकी इलाहाबाद परशससत स महतवपणि

ऐवतहावसक जानकारी वमलती ह वजसकी रचना

उसक दरबार क कवि हररसन न की थी

Q23) उततर (b)

सपषटीकरणः

- विकरम सोित की शरआत ििि 58 ईसा पिि म

चनदरगपत वदवतीय न की थी

- यह शक ो पर उसकी जीत और उस विकरमावदतय

की पदिी वमलन क उपलकषय म आरमभ वकया गया

था

FC19H1003 32

- बानभटट न हिििििन का जीिन-ितताोत हििचररत

(ज सोसकत म थी) वलखी थी

Q24) उततर (c)

सपषटीकरणः

- सोवि-विगरावहका यद एिो शाोवत का मोतरी

- साथििाह वयापाररय ो क कावफल ो का नता

Q25) उततर (a)

सपषटीकरणः

- जआन झाोग (हसआन रताोग ndash Hsuang Tsang)

एक चीनी यातरी था ज हिििििन क शासनकाल म

भारत आया था

- ििि 630 ईसवी स ज दशक आरमभ हआ था उसम

जआन झाोग मधय एवशया ईरान और

अफग़ावनसतान की यातरा करन क पशचात कशमीर

क रासत स भारत आया था

- उसन उततर स पिि तक की यातरा की और िह

लगभग 2 ििि वबहार म रहा

- जआन झाोग न नालनदा विशवविदयालय म विदयावथिय ो

और विदवान ो क साथ पारसपररक विचार-विमशि

वकया थथानीय भािाओ ा म वनपणता परापत की तथा

बौद सतप ो की ख ज की

Q26) उततर (c)

सपषटीकरणः

- परदवकषणा पथ बौद िासतकला म सतप क चार ो

ओर बनाया जान िाला एक घमािदार पथ ह ता

- परशन म वदए गए बाकी क तीन ो ततव वहोद मसनदर ो की

िासतकला क भाग ह

Q27) उततर (d)

सपषटीकरणः

परशन म वदए गए सभी मोवदर ो म वयापक रप स

ईोि ो (पकी ईोि ो) का परय ग पतथर ो क साथ हआ

Q28) उततर (c)

सपषटीकरण

- महममद कली कतब शाह ग लकणडा का सलतान

था

- िह अकबर का समकालीन था

- सावहतय और िासतकला म उसकी अतयाविक

रवच थी

- िह एक महान कवि था

- िह दसखनी उदि फारसी और तलग म वलखता था

- उसन अपन पीछ एक विसतत वदिान (सोगरह)

छ डा ह

- अभी हाल ही म तलोगाना म ग लकणडा क वकल

क अनदर खदाई वकय गए बाग-ए-नाया वकला

बाग क चार ो ओर रप-रखा क मानवचतरण क

वलए भारतीय परातासतवक सिकषण (The

Archaeological Survey of India ndash ASI)

गराउणड पनीिर विोग रडार (Ground Penetrating

Radar) का परय ग करगा

Q29) उततर (a)

सपषटीकरणः

- वसलपपावदकारम एक तवमल महाकावय ह वजसकी

रचना इलाोग क दवारा लगभग 1800 ििि पिि की

गई थी

- यह क िलन नामक एक वयापारी की कहानी ह

ज माििी नामक एक गवणका (िशया) स परम

करन लगा था

- मवनमकलाई क िलन और माििी की पतरी की

कहानी ह

Q30) उततर (a)

सपषटीकरण

- चरक आयिद और वचवकरता की एक महतवपणि

रचना चरक सोवहता क लखक ह

- बरहमगपत क अपनी रचना बरहम-सफि-वसदानत

(ज एक खग लीय रचना ह) क कारण परवससद

वमली

FC19H1003 33

- बगदाद म इसका अनिाद अरबी भािा म वकया

गया था

- इसका इसलावमक गवणत और खग ल-विजञान पर

महतवपणि परभाि पडा था

- बाद म अपन जीिनकाल म बरहमगपत न

ldquoखोडखयाकrdquo वलखी ज एक खग लीय पससतका

(एक छ िी पसतक) थी

- इसम आयिभटट की अिि-रावतर क परतयक वदन की

शरआत परणाली का परय ग वकया गया था

Q31) उततर (c)

सपषटीकरण

- अमीर खसर एक परवसद सफी सोगीतकार कवि

और विदवान थ

- 1318 म उनह ोन पाया वक इस भवम (वहोदसतान) क

हर कषतर म अलग-अलग भािा थी लाहौरी

कशमीरी दवारसमदरी (दवकषणी कनाििक म)

तलोगाना (आोधर परदश म) गजरी (गजरात म)

माबारी (तवमलनाड म ) अििी (पिी उततर परदश

म) और वहोदिी (वदलली क आस-पास क कषतर म)

आवद

- उनह न यह बताया वक सोसकत वकसी भी कषतर स

सोबोवित नही ो थी और किल बराहमण ही इस भािा

का जञान रखत थ

Q32) उततर (c)

सपषटीकरण

- वहरणय-गभि सववणिम गभि क सोदवभित करता ह

- जब बराहमण ो की सहायता स यह अनषठान वकया

जाता था त यह माना जाता था वक बवल दन िाल

का कषवतरय क रप म पनजिनम ह गा

Q33) उततर (d)

सपषटीकरण

- कदमई भवम राजसव पर कर क सोदवभित करता

- गवावलयर परशससत म नागभि क दवारा वकय गए

श िण का िणिन वकया गया ह |

- नागभि एक परवतहार राजा था

Q34) उततर (b)

सपषटीकरण

- राजतरो वगनी 12िी ो शताबदी म कलहन क दवारा

रवचत एक सोसकत पसतक (िकसट) ह

- यह परारसमभक भारत की ऐवतहावसक इवतितत थी

- तकि सोगत रप स इस अपन परकार की सिोततम

और सिािविक विशवसनीय कवत माना जाता ह

- यह कशमीर कषतर क पराचीनतम समय स लकर

उसकी रचना की तारीख तक क समपणि इवतहास

का आचछादन करती ह

Q35) उततर (c)

सपषटीकरण

- गााि की आम सभा क ldquoउरrdquo कहा जाता था

- ldquoउरrdquo म गााि क सभी कर दन िाल वनिासी

शावमल ह त थ

Q36) उततर (a)

सपषटीकरण

- वदलली सलतनत म ldquoतारीखrdquo इवतहास लखन का

एक रप था

- ldquoतािरीखrdquo क लखक विदवान परि ह त थ वजनम

सवचि परशासक इतयावद शावमल थ

Q37) उततर (a)

सपषटीकरण

- अलाउददीन सखलजी अपन सवनक ो क ितन का

भगतान नकद म करता था न वक इकता क रप

- सवनक अपना सामान वदलली म वयापाररय ो स

खरीदत थ अतः इस बात का भय था वक वयापारी

कही ो िसतओो का मलय न बिा द

- इसकी र कथाम क वलए अलाउददीन सखलजी न

वदलली म कीमत ो क वनयसित वकया

FC19H1003 34

- अविकारीगण धयानपििक मलय ो का सिकषण करत

थ तथा ज वयापारी वनिािररत मलय पर माल नही ो

बचत थ उनक दसणडत वकया जाता था

Q38) उततर (d)

सपषटीकरण

- वदलली सििपरथम त मर राजपत ो क अिीन उनक

सामराजय की राजिानी बनी थी

- 12िी ो शताबदी क मधय म अजमर क चौहान ो

(वजनह चाहमान ो क नाम स भी जाना जाता ह) न

त मर राजपत ो क परावजत वकया था

- त मर ो और चौहान ो क अिीन वदलली एक

महतवपणि िावणसजयक क दर बन गया था

- कई जन वयापारी यहाा रहन लग थ और उनह ोन

कई मोवदर भी बनिाए

- यहाा पर मवदरत वसक वजनह ldquoदहलीिालrdquo क नाम

स जाना जाता था वयापक रप स परचलन म थ

Q39) उततर (c)

सपषटीकरण

- म ठ की मसिद का वनमािण वसको दर ल दी क

राजयकाल म उसक मिी क दवारा करिाया गया

था

- बगमपरी मसिद का वनमािण महममद तगलक क

शासनकाल म हआ था

- यह मसिद विशव का पणयथथान (The

Sanctuary of the World) और वदलली म महममद

तगलक की नई राजिानी जहाोपनाह की मखय

मसिद थी

- कववत- अल - इसलाम मसिद का विसतार

इलतसिश और अलाउददीन सखलजी न वकया था

- मीनार का वनमािण तीन सलतान ो कतबददीन ऐबक

इलतसिश और वफर ज शाह तगलक क दवारा

करिाया गया था

Q40) उततर (c)

सपषटीकरण

- मगल ो क अिीन मनसबदार शबद उस वयसकत क

वलए सोदवभित वकया जाता था वजसक पास मनसब

(अथाित पद) ह ता था

- उस अपना ितन राजसव कायो वजनह जागीर कहत

थ क रप म परापत ह ता था

Q41) उततर (b)

सपषटीकरण

- ldquoभारत छ ड आोद लनrdquo वबरविश शासन क

सखलाफ ल ग ो का एक सवाभाविक विदर ह था

- असखल भारतीय काोगरस सवमवत न 8 अगसत 1942

क बमबई म एक बठक का आय जन वकया था

- इस बठक म परवसद सोकलप ldquoभारत छ ड rdquo क

पाररत वकया गया और इस उददशय क परापत करन

क वलए गाोिी क नततव म एक अवहोसक जन सोघिि

आोद लन की शरआत का परसताि वदया गया

- लवकन अगल ही वदन गाोिी और काोगरस क अनय

परमख नताओो क वगरफतार कर वलया गया

- काोगरस क एक बार वफर अिि घ वित वकया गया

था

Q42) उततर (c)

सपषटीकरण

- साइमन कमीशन यनाइविड वको गडम क सात

साोसद ो का एक समह था

- इस वबरविश भारत क वलए सोििावनक सिार ो का

सझाि दन क वलए गवठत वकया गया था

- इस आय ग म िररषठ वबरविश राजनता सर जॉन

साइमन क नततव म किल वबरविश सदसय ही

शावमल थ

- इसवलए भारत क ल ग ो न साइमन कमीशन क

आगमन क विरद आोद लन वकया था

Q43) उततर (a)

सपषटीकरण

bull दादा भाई नौर जी भारत म वबरविश शासन क

आवथिक पररणाम ो क बार म अपनी विर िी

(परवतकल) राय क वलए जान जात थ

FC19H1003 35

bull अपन कई लख ो और भािण ो म विशि रप स

ldquoपाििी एो ड अन-वबरविश रल इन इसणडया

(Poverty and Un-British Rule in India) म

नौर जी न यह तकि वदया वक भारत पर अतयविक

कर लगाया गया था और इसकी सोपवतत इोगलड की

ओर परिावहत की जा रही थी

bull उनह ोन पराचीन भारतीय गरोथ ो की वयाखया करन

का और भारतीय ो क आिविशवास क बहाल

करन पर कायि नही ो वकया था

उनह ोन वकसी और बात स पहल सभी सामावजक

बराइय ो क उनमलन की आिशयकता पर भी बल

नही ो वदया था

Q44) उततर (c)

सपषटीकरण

bull अगसत 1932 म वबरविश परिानमोतरी मकड नालड न

अपन साोपरदावयक परसकार (The Communal

Award) की घ िणा की थी

bull यह भारत क कई साोपरदावयक वहत ो क बीच विवभनन

सोघिो क हल करन क वलए वबरिन का एकतरफा

परयास था

bull यह परसकार (Award) बाद म 1935 क

अविवनयम (The Act of 1935) म शावमल वकया

गया था

bull इस साोपरदावयक परसकार न मससलम ो क वलए

आरवकषत एक अलग वनिािचक मणडल फॉमिल का

विसतार अनय अलपसोखयक ो क वलए वकया था

वजसम वसख ो भारतीय ईसाइय ो आोगल-भारतीय

समदाय यर पीय समदाय तथा विवशषट कषतरीय

समह ो क शावमल वकया गया था

bull गाोिी न इस परसताि क भारतीय समाज क

विभावजत करन क वलए एक घवणत वबरविश

सावजश क रप म दखा और उसक सखलाफ

आमरण अनशन वकया

Q45) उततर (b)

सपषटीकरण

मौजदा आयात और वनयाित क अवतररक़त

औपवनिवशक भारत क वनमनवलसखत खचो क

वलए एक विशिवनवशचत िन रावश भी दनी पडती

थी

(i) परशासन क वयय

(ii) सना क रख-रखाि क वयय

(iii) यद क वयय

(iv) सिावनितत अविकाररय ो की पशन तथा

(v) वबरिन दवारा अपनी उपवनिश बसती

(कॉल नी) क रख-रखाि क वयय

इनह गह शलक (Home Charges) क रप म

जाना जाता था और लगभग परी तरह स भारत क

दवारा इनका भगतान वकया जाता था

bull गह शलक म वनमनवलसखत घिक शावमल थ

(i) भारतीय ऋण पर दय बयाज

(ii) ईसट इोवडया को पनी क शयरिारक ो क

लाभाोश

(iii) लोदन म भारत कायािलय चलान क वलए िन

(iv) भारत म वनयकत वबरविश कवमिय ो क ितन

और पशन का भगतान करन क वलए िन

(v) रलि पर बयाज

(vi) नागररक और सनय शलक

(vii) इोगलड म सट र (सामगरी) की खरीद

Q46) उततर (b)

सपषटीकरण

bull भारतीय राषटर ीय काोगरस का लाहौर सतर 1929 म

जिाहरलाल नहर की अधयकषता म आय वजत

वकया गया था

bull इस सतर म भारतीय राषटर ीय आोद लन स समबसित

कई महतवपणि पररणाम सामन आय थ

(i) सििपरथम इस सतर म काोगरस क अधयकष पद

पर जिाहरलाल नहर क चना गया था ज

काोगरस म िामपोवथय ो की बिती हई ताकत

का सपषट सोकत था

(ii) दसरा इस सतर म पहली बार काोगरस न पणि

सवतोतरता की माोग क उठाया था

इस परकार की माोग काोगरस मोच स पहल कभी भी

नही ो उठाई गई थी

Q47) उततर (b)

सपषटीकरण

FC19H1003 36

bull इस ररप िि न वकसी भी समदाय क वलए पथक

वनिािचक मोडल अथिा अलपसोखयक ो क वलए

भाराोश की वसफाररश नही ो की थी

bull तथावप इस ररप िि न उन पराोत ो म अलपसोखयक

सीि ो क आरकषण की अनमवत दी थी जहाा पर कम

स कम दस परवतशत अलपसोखयक ह

bull लवकन यह समदाय क आकार क अनपात म ह ना

चावहए था

bull इस ररप िि म भारत क वलए पणि सवतोतरता क

वलए क ई पराििान नही ो था

Q48) उततर (c)

सपषटीकरण

bull आरो वभक िवदक आयो का िमि मखय रप स

परकवत की पजा और यजञ था

bull परारो वभक आयि िमि परकवत की पजा क समान था

bull िासति म उनक चार ो ओर की शसकतयाा वजनह न

त ि वनयोवतरत कर सकत थ और न ही समझ पाए

थ उनह वदवयता क साथ वनिवशत वकया गया तथा

उनह मादा या नर दिीदिताओो क रप म

परतीकतव वकया गया था

bull उनह ोन कछ यजञ ो का भी वनषपादन वकया था

Q49) उततर (b)

सपषटीकरण

bull सडक और नदी-मागि (जल-मागि) डकती स

सरवकषत नही ो थ

bull उललखनीय ह वक हिििििन क शासनकाल क

दौरान यआन चिाोग (हयएन साोग) का सारा

सामान लि वलया गया था

Q50) उततर (c)

सपषटीकरण

परशन म वदए गए द न ो कथन सही ह

Q51) उततर (b)

सपषटीकरण

bull परोदर दास एक सोत और भगिान कषण क एक

महान भकत थ

bull परोदर दास क कनाििक सोगीत क वपतामह क

रप म जाना जाता ह

bull यदयवप उनक जनम-थथान क बार म काफी

अिकल लगाई जाती रही ह

bull तथावप अब कननड विशवविदयालय हमपी क दवारा

गवठत एक विशिजञ सवमवत इस वनषकिि पर पहोची

ह वक उनका जनम थथान सोभितया कनाििक का

एक छ िा-सा गााि कषमपरा (वशिम गगा वजला)

था

Q52) उततर (c)

सपषटीकरण

bull शरी तयागराज शरी शयाम शासतरी और शरी मथसवामी

दीवकषतर क कनाििक सोगीत की वतरमवति माना

जाता ह

bull उनक कारण ही 18िी ो-19िी ो शताबदी म कनाििक

सोगीत का सववणिम यग आया था

Q53) उततर (d)

सपषटीकरण

bull अभी हाल ही म लौह यगीन-महापािावणक काल

का 2000 ििि पराना एक दलिभ सारक फगस

(Sarcophagus) (पतथर का ताबत) क ललम क

वियर गाोि (क वयलडी क पास वजला क वझक ड

करल राजय) की एक रॉक-कि गफा स ख जा गया

bull यह ताबत वजसम हविय ो क िकड थ खदाई क

दौरान वमला

bull अभी तक इस परकार की दलिभ ख ज करल क

मातर द ही थथान ो स हई ह

bull य द न ो सारक फगी (Sarcophagi) (पतथर क

ताबत) चियर और अथ ली (वजला क वझक ड) क

महापािाण थथल ो स वमल ह

Q54) उततर (a)

सपषटीकरण

FC19H1003 37

दवकषण भारत म महापािाण सोसकवत एक पणि

विकवसत लौह यगीन सोसकवत थी

Q55) उततर (d)

सपषटीकरण

bull च ल पाणडय और करलपतर (चर) इन तीन ो का

उललख अश क क अवभलख ो म वकया गया ह

bull सोभितः य भौवतक सोसकवत क उततर

महापािावणक चरण म थ

Q56) उततर (d)

सपषटीकरण

bull भीमा-क रगाोि की लडाई ततीय आोगल-मराठा

यद का वहससा थी

Q57) उततर (b)

सपषटीकरण

bull राजकमार शकल न गाोिीजी क चोपारण आन तथा

वतनकवथया परणाली स जडी समसया की जाोच क

वलए रारी करन क वलए दश भर म उनका

अनसरण वकया था

bull बज वकश र राजदर परसाद महादि दसाई और

नरहरी पाररख चोपारण सतयागरह क दौरान गाोिी

जी क सहय गी थ

Q58) उततर (b)

सपषटीकरण

bull बराहमण ो और बौद मठिाररय ो क कर-मकत गााि

अनदान म दन की परथा सतिाहन ो न आरमभ की

थी

Q59) उततर (c)

सपषटीकरण

इस कायिकरम क उददशय वनमनानसार ह

(i) बवनयादी पयििन आिाररक सोरचना का विकास

करना

(ii) चयवनत (पहचान वकय गए) कषतर ो म आजीविका क

सजन क वलए दश क साोसकवतक और विरासत

मलय ो क बिािा दना

(iii) विरासत समारक थथल ो पर विशव सतरीय आिाररक

सोरचना विकवसत करक एक सतत तरीक स

पयििक आकििण म िसद करना

(iv) थथानीय समदाय ो की सवकरय भागीदारी क माधयम

स र रगार ो का सजन करना

(v) र रगार उतपादन और आवथिक विकास क वलए

पयििन कषमता का उन पर परभाि का उपय ग

करना तथा

(vi) िारणीय पयििन आिाररक सोरचना का विकास

करना और उसका उवचत सोचालन तथा

रखरखाि सवनवशचत करना

Q60) उततर (b)

सपषटीकरण

bull यह वनकाय ििि 1987 म अससततव म आया था

bull यह एक राषटर ीय सतर का शीिि सोगठन ह ज भारत

सरकार क जनजातीय मामल ो क मोतरालय क

परशासवनक वनयोतरण क अिीन काम कर रहा ह

bull इसका पोजीकत और परिान कायािलय नई वदलली

म सथथत ह

Q61) उततर (c)

सपषटीकरण

bull परमचोद क उपनयास ो म परमाशरम रोगभवम गबन

कमिभवम और ग दान शावमल ह

bull ग रा रिी ोदरनाथ िग र क दवारा रवचत उपनयास ह

bull अभी हाल ही म मोशी परमचोद की 138िी ो जयोती दश

भर म मनाई गई थी

Q62) उततर (b)

सपषटीकरण

bull ldquoवगदाrdquo पोजाब (भारत) एिो पावकसतान की

मवहलाओो क दवारा तयौहार क समय और फसल

की बिाई तथा किाई क अिसर पर वकया जान

िाला एक पारोपररक दहाती नतय ह

FC19H1003 38

bull इस नतय क माधयम स पोजाबी मवहलाऐो अपनी

परसननता परकि करती ह तथा वगदा क परदशिन क

माधयम स परि िचिसव िाल समाज म मवहलाओो

की दबी हई भािनाओो क परकि करती ह

bull चोवक इस नतय का परि ो क साथ क ई सोबोि नही ो

ह अतः किल मवहलाऐो ही इसम भाग ल सकती

bull हर साल तीज समार ह क दौरान पोजाब म वगदा

नतय वकया जाता ह

तीज भारत क कछ भाग ो म मवहलाओो क दवारा

मनाया जान िाल कई तयौहार ो क वलए एक

वयापक नाम ह

Q63) उततर (a)

सपषटीकरण

- मजम-उल-बहरीन या द समदर ो का सोगम

नामक उललखनीय रचना दारा वशक ह क दवारा

वलखी थी

- भारत क उपराषटर पवत शरी एम िकया नायड न कहा

ह वक राजकमार दारा वशक ह की रचनाएा शाोवत

और सदभाि क बिािा दन क वलए एक तारा सर त

क रप म सामन आ सकती ो ह

- उपराषटर पवत गत ििो क भला वदए गए राजकमार

दारा वशक ह क परदवशित परचवलत करन हत

आय वजत एक परदशिनी का दौरा करन क बाद एक

सभा क सोब वित कर रह थ

- इस परदशिनी का आय जन फर क इस गौवियर

(Francois Gautier) क दवारा lsquoइोवदरा गाोिी नशनल

सिर फॉर द आििसrsquo (The Indira Gandhi

National Centre for the Arts) नई वदलली म

वकया गया था

Q64) उततर (c)

सपषटीकरण

- ग मतशवर परवतमा जन भगिान बाहबली क

समवपित ह

- यह एक एक-चटटानी पतथर की मवति ह

- राषटर पवत राम नाथ क विोद न शरिणबलग ला

(कनाििक) म आय वजत वकय जान िाल भवय

अवभिक समार ह महामसतकावभिक का

उदघािन वकया था

- यह समार ह 12 ििो म एक बार ह ता ह

Q65) उततर (c)

सपषटीकरण

bull पराची घािी पराची नदी क चार ो ओर फली हई थी

bull पराची घािी िीर-िीर विलपत ह गई थी

bull पराची नदी भिनशवर स वनकलती ह

bull यह महानदी की एक सहायक नदी ह और यह

परी खदाि किक तथा जगतवसोहपर वजल ो स

ह कर बहती ह

bull इस नदी क पर कषतर क पराची घािी कहा जाता ह

bull यह नदी बोगाल की खाडी म वगरती ह

परातासतवक साकषय स पता चलता ह वक पराची घािी

सभयता हडपपा और म हनज दाड द न ो की

पिििती ह

Q66) उततर (d)

सपषटीकरण

य समारक छतरपर वजल (मधय परदश) म विोधयाचल

पिित शरोखला म सथथत ह

Q67) उततर (a)

सपषटीकरण

bull थॉिस ऑन पावकसतान नामक पसतक डॉ बी

आर अमबडकर न वलखी थी

bull डॉ बी आर अमबडकर की जयोती क अिसर पर

भारत क राषटर पवत न भारत की इस महान हसती

क शरदाोजवल अवपित की थी

bull डॉ बी आर अमबडकर न 1924 म वडपरथड

कलावसर इोसटीटयि (दवलत िगि सोथथान -

बवहषकत वहतकाररणी सभा) और 1927 म समाज

समता सोघ की थथापना की थी

bull अमबडकर का धयान वशकषा कषतर की ओर भी था

bull उनह ोन वशकषा क वनमन िगो म फलान क वलए

पीपलस एजकशन स साइिी (The Peoples

Education Society) क नाम स महाविदयालय ो क

नििकि और छातरािास ो की थथापना की थी

FC19H1003 39

Q68) उततर (b)

सपषटीकरण

bull महरगि भारतीय उपमहादवीप म एक परवसद

निपािाण बसती ह ज बलवचसतान पराोत

पावकसतान म सथथत ह

bull दचपलली (आोधर परदश) क पास नागलर नदी क

पिी ति ो पर चना पतथर क बलॉक क विशाल

विसतार म एक पिि-ऐवतहावसक रॉक आिि थथल की

ख ज की गई ह

bull इसन 1500-2000 ईसा पिि क दौरान गोिर (आोधर

परदश) म विकवसत निपािाण सभयता पर परकाश

डाला ह

Q69) उततर (c)

सपषटीकरण

bull 12िी ो सदी और 13िी ो सदी म काकाविय िोश का

उदय हआ था

bull ि पहल कलयाण क पवशचमी चालकय ो क सामोत थ

bull परारोभ म उनह ोन िारोगल (तलोगाना) क पास एक

छ ि स कषतर पर शासन वकया था

bull उनह ोन ldquoनायक वयिथथाrdquo की शरआत की थी

वजस बाद म विजयनगर क राय शासक ो न

अपनाया और विकवसत वकया था

Q70) उततर (a)

सपषटीकरण

bull गाोिीजी क अनशन स वमल मावलक ो पर दबाि

पडा था ज अोततः शरवमक ो क ितन म 35 परवतशत

की िसद करन क वलए सहमत हए थ

bull गगल (Google) न अनसया साराभाई वजनह ोन

भारत क शरवमक आोद लन म एक अगरणी भवमका

वनभाई थी की 132िी ो जयोती डडल (Doodle) का

वनमािण करक मनाई

Q71) उततर (d)

सपषटीकरण

भारत स यनसक की मानिता की अमति साोसकवतक

विरासत की परवतवनवि सची म वनमनवलसखत शावमल ह

bull कवडयटटम करल का सोसकत रोगमोच

bull मवडयिि करल का अनषठान रोगमोच और नतय

नाविका

bull िवदक मि जाप की परोपरा

bull राजथथान क कालबवलया ल क गीत और नतय

bull रामलीला रामायण का पारोपररक परदशिन

bull सोकीतिन मवणपर का अनषठान गायन ढ ल िादन

और नतय

bull रममन भारत क गििाल वहमालय का िावमिक

तयौहार और अनषठान रोगमोच

bull जाोदीयाला गर पोजाब क ठठर ो की पीतल और

ताोब क वशलप स वनवमित बतिन ो की पारोपररक कला

bull छाऊ नतय पिी भारतीय राजय ो म जनमी शासतरीय

भारतीय नतय कला

bull लददाख का बौद मि जाप िर ाोस-वहमालयी लददाख

कषतर तथा जमम-कशमीर म पवितर बौद गरोथ ो का पाठ

bull य ग

bull नौर र

bull को भ मला

Q72) उततर (b)

सपषटीकरण

bull भारत क राषटर पवत शरी राम नाथ क विोद न

वकसामा नागालड म हॉनिवबल मह रति और

राजय गठन वदिस समार ह का उदघािन वकया

था

bull हॉनिवबल मह रति का नाम भारतीय हॉनिवबल क

नाम पर पडा ह ज एक विशाल और रोगीन जोगली

पकषी ह

bull यह पकषी नागालड राजय की अविकतर जनजावतय ो

की ल ककथाओो म उसललसखत ह

bull नागालड की परमख मानयता परापत जनजावतयाा ह

अोगामी आओ चखसोग चाोग ककी रगमा और

रवलोग आवद

bull ओोग जारिा और ससिनलीस अोडमान-वनक बार

दवीप समह की जनजावतयाा ह

FC19H1003 40

Q73) उततर (c)

सपषटीकरण

bull दकन म राषटर कि शासन दसिी ो सदी क अोत तक

लगभग 200 ििो तक रहा था

bull राषटर कि शासक अपन िावमिक विचार ो म सवहषण

bull उनह ोन न किल शि िमि और िषणि िमि बसलक

जन िमि क भी सोरकषण वदया था

bull एल रा म वशि क परवसद रॉक कि मोवदर का

वनमािण नौिी ो सदी म राषटर कि राजा कषण परथम न

करिाया था

bull उसका उततराविकारी अम घििि जन था लवकन

उसन अनय िमो क भी सोरकषण परदान वकया था

bull राषटर कि ो न मसलमान वयापाररय ो क बसन की

अनमवत दी थी

bull उनह न अपन अविराजय ो म इसलाम क उपदश दन

की भी अनमवत दी थी

bull अभी हाल ही म पाोडिलागटटा (तलोगाना) क

परागवतहावसक चटटान वचतर ो क कषरण की बिती हई

घिनाएा एक गोभीर वचोता का वििय ह

bull यह परागवतहावसक चटटान क नकसान पहाचा

सकता ह

bull पाोडिलागटटा वनमनवलसखत क वलए जाना जाता ह

- 10000 ईसा पिि स 8000 ईसा पिि क वचवतरत

चटटानी आशरय ो क वलए

- राषटर कि काल क एक 8 िी ो सदी क

वशलालख क वलए और

- 12िी ो सदी क काकविय सामराजय क वभवतत

वचतर ो क वलए

Q74) उततर (b)

सपषटीकरण

bull 1828 म राजा राम म हन रॉय न एक नय िावमिक

समाज बरहम सभा की थथापना की थी वजस बाद

म बरहम समाज क नाम स जाना गया था

bull दिदरनाथ िग र न ततवब विनी सभा की अधयकषता

की थी ज आधयासिक सतय की ख ज म सोलि

थी

bull इसका उददशय वहोद िमि क शद करन का और

एकशवरिाद (एक ईशवर म आथथा) का परचार करना

था

bull नय समाज की थथापना क आिार थ कारण

(तकि ) क द सतमभ तथा िद और उपवनिद

bull अभी हाल ही म सािारण बरहम समाज का कछ

काननी मदद ो क लकर पवशचम बोगाल सरकार क

साथ काननी वििाद चल रहा ह

Q75) उततर (c)

सपषटीकरण

bull भारत म वचशती वसलवसल की थथापना खवाजा

म इनददीन वचशती क दवारा की गयी थी

bull ि 1192 ईसवी क आसपास भारत आय थ

bull वचशतीय ो क बारहिी ो शताबदी क उततरािि म भारत

म आन िाल सफीय ो क समह ो म सबस

परभािशाली माना जाता ह

bull उनह ोन थथानीय िातािरण क साथ सफलतापििक

अनकलन वकया और उनह ोन भारतीय भसकत

परोपराओो क कई पहलओो क अपनाया

bull अजमर म सफी अपरकि खवाजा म इनददीन वचशती

की ऐवतहावसक दरगाह क एक नया रप दन की

तयारी की जा रही ह

bull इस 13िी ो शताबदी की दरगाह क ldquoसवचछ

आइकॉवनक थथल ोrdquo (Swacch Iconic Places) म

शावमल वकया गया ह ज परवतवषठत विरासत

आधयासिक और साोसकवतक थथान ो पर क वदरत

य जना ह

FC19H1003 41

ANSWERS amp EXPLANATION OF

NCERT History Class VI-X + Current Affairs

(FC19E1003)

Q1) Answer c

Explanation

Rigveda consists of more than a

thousand hymns dedicated to gods and

goddesses These hymns were

composed by sages and learnt by men

however a few were composed by

women like Apala Ghosa Lopamudra

Maitreyi and Gargi

Rigveda consists of many hymns in the

form of dialogues We get an example of

a dialogue between a sage named

Vishwamitra and two rivers (Beas and

Sutlej) that were worshipped as

goddesses This suggests that he

belonged to the Vedic period

Q2) Answer b

Explanation

Traces of ash have been found from

Kurnool Caves suggesting that people

were familiar with the use of fire

It is situated in Andhra Pradesh

Q3) Answer c

Explanation

Burzahom is a prehistoric site in

present day Kashmir where people built

pit houses which were dug into the

ground with steps leading into them

These may have provided shelter in cold

weather

Q4) Answer c

Explanation

Epigraphy is defined as the study of

inscriptions

Manuscriptology is the study of history

and literature through the use of hand

written documents

Palaeography refers to the study of

ancient writing systems and the

deciphering and dating of historical

manuscripts

Numismatics refers to the study of

coins

Q5) Answer a

Explanation

Charaka Samhita was written by

Charaka and is an important book on

Ayurveda and medicine

He was a practitioner of the traditional

system of Indian medicine known as

Ayurveda

Charaka is thought to have flourished

sometime between the 2nd century BCE

and the 2nd century CE

Q6) Answer b

Explanation

Bhaga refers to the tax on crops which

was fixed at 16th of the production

Kammakaras is the term used for the

landless agricultural labour class

Ashvamedha also known as horse

sacrifice is a ritual where a horse is let

loose to wander freely and it was

guarded by the rajarsquos men

Q7) Answer (d)

Explanation

In the Rigvedic period horses were

yoked to chariots that were used in

battles fought to capture land cattle

etc This suggests that the use of horse

chariots began much before the period

of Mahajanapadas

The battles were fought in the Rigvedic

period for cattlersquos lands water an even

to capture people Most men took part

in these wars however there was no

regular army but there were assemblies

where people met and discussed

matters of war Regular armies became

a feature in the Mjahajanapada period

including vast armies of foot soldiers

chariots and elephants

RAUSIAS-FC19E1003 42

Q8) Answer (a)

Explanation

Buddha belonged to the Sakya clan and

passed away at Kusinara

Buddha taught in Prakrit which was the

common language of people

Q9) Answer c

Explanation

There were six schools of philosophy in

ancient India These are known as

Vaishesika Nyaya Samkhya Yoga

Purva Mimansa and Vedanata or Uttara

Mimansa They were founded by sages

Kanada Gautama Kapila Patanjali

Jamini and Vyasa respectively

Q10) Answer b

Explanation

The teachings of Mahavira were

compiled at Valabhi in 6th century AD

Q11) Answer (c)

Explanation

Chanakya is traditionally identified as

Kautilya or Vishnugupta who authored

the ancient Indian political treatise the

Arthashastra

Q12) Answer d

The national emblem of India is an

adaptation of the Lion Capital atop the

Ashoka Pillar of Sarnath Uttar Pradesh

and is combined with the National

Motto Satyameva Jayate

The Rampurva Bull gets the name from

the site of its discovery Rampurva in

Bihar

It is noted for its delicately sculpted

model demonstrating superior

representation of soft flesh sensitive

nostrils alert ears and strong legs It is

a mixture of Indian and Persian

elements

Sankissa is situated in Uttar Pradesh

India

Q13) Ans(a)

Kunwar Singh was a notable leader during the Revolt of 1857 He belonged

to a royal house of Jagdispur Bihar

Q14) Answer b

Explanation

The term Vellalar was used for large

landowners

Q15) Answer c

Explanation

Arikamedu was a coastal settlement

where ships unloaded goods from

distant lands Finds here include a

massive brick warehouse pottery

including amphorae and Arretine ware

Roman lamps glassware and gems have

also been found at the site

Q16) Answer a

Explanation

Muvendar is a Tamil word mentioned in

Sangam poems meaning three chiefs

used for the heads of three ruling

families the Cholas Cheras and

Pandyas

Q17) Ans (c)

Several tribal or kin-based assemblies

such as the Sabha Vidatha and gana

are mentioned in the Rig-veda The

Sabha and the samiti mattered a great

deal in early Vedic times so much so

that the chiefs or the kings showed an

eagerness to win their support

Q18) Ans (a)

Jainism recognised the existence of the

gods but placed them lower than the

jina and did not condemn the varna

system as Buddhism did

Q19) Answer (d)

Explanation

Cholas and Pandyas had developed

powerful coastal cities The most

important city of Cholas was Puhar or

Kaveripattinam and Madurai was the

capital of Pandyas

Q20) Answer b

Explanation

Buddhacharita is the biography of

Buddha and was written by

RAUSIAS-FC19E1003 43

Ashvaghosha

Q21) Answer (a)

Explanation

Tamil poet Appar was a Shiva devotee

So he was a Nayanar saint

Q22) Answer d

Explanation

Samudragupta was a prominent Gupta

ruler whose coins depict him playing a

veena indicating his love for music We

get important historic information from

his Allahabad Prashasti which was

composed by his court poet Harisena

Q23) Answer (b)

Explanation

Vikrama Samvat was founded by

Chandragupta II in the 58 BC as a

mark of victory over the Shakas and

assumed the title of Vikramaditya

Banabhatta wrote Harshavardhanarsquos

biography the Harshacharita in

Sanskrit

Q24) Answer c

Explanation

Sandhi-vigrahika was the minister of

war and peace

Sarthavaha was the leader of the

merchant caravans

Q25) Answer a

Explanation

Xuan Zang (Hsuan-tsang) was a

Chinese traveller who came during the

reign of Harshavardhana

In the decade that began in 630 AD

Xuan Zang came to India through

Kashmir after visiting Central Asia Iran

and Afghanistan

He travelled from north to east and lived

in Bihar for a couple of years

At Nalanda University Xuan Zang

interacted with students and scholars

mastered local languages and

discovered Buddhist stupas

Q26) Answer c

Explanation

Pradakshina patha is a circular path

laid around a stupa in Buddhist

architecture While the rest are a part of

temple architecture

Q27) Answer d

Explanation

All the above-mentioned temples have

an elaborate use of bricks (baked

bricks) along with stone

Q28) Ans (c)

Muhammad Quli Qutab was the Sultan

of Golconda He was a contemporary of

Akbar was very fond of literature and

architecture

The Sultan was a great poet and he

wrote in Dakhini Urdu Persian and

Telgu and has left an extensive diwan or

collection

Recently the Archaeological Survey of

India (ASI) will be using Ground

Penetrating Radar (GPR) to map the

contours of the area around the Bagh-e-

Naya Qila excavated garden inside the

Golconda Fort in Telangana

Q29) Answer a

Explanation

Silappadikaram is a famous Tamil epic

which was written by Ilango around

1800 years ago It is a story of a

merchant named Kovalan who fell in

love with a courtesan named Madhavi

Manimekalai tells the story of the

daughter of Kovalan and Madhavi

Q30) Answer (a)

Explanation

Charaka is the author of Charaka

Samhita which is an important work of

Ayurveda and medicines

Brahmaguptarsquos fame rests mostly on his

Brahma-sphuta-siddhanta which was

an astronomical work It was translated

into Arabic in Baghdad and had a major

impact on Islamic mathematics and

astronomy

Late in his life Brahmagupta wrote

Khandakhadyaka which was an

RAUSIAS-FC19E1003 44

astronomical handbook that employed

Aryabhatarsquos system of starting each day

at midnight

Q31) Answer (c)

Explanation

Amir Khusrau was a famous sufi

musician poet and scholar In 1318 he

noted that there was different language

in every region of this land (Hindustan)

Lahori Kashmiri Dvarsamudri (in

Southern Karnataka) Telangana (in

Andhra Pradesh) Gujari (in Gujarat)

Marsquobari (in Tamil Nadu) Awadhi (in

eastern Uttar Pradesh) and Hindawai (in

the area around in Delhi) etc He went

to explain that Sanskrit did not belong

to any region and that only brahmans

knew it

Q32) Answer c

Explanation

Hiranyagarbha refers to the golden

womb When this ritual was performed

with the help of Brahmanas it was

thought to lead to the rebirth of the

sacrificer as a Khastriya

Q33) Answer d

Explanation

Kadamai refers to a tax on land

revenue

Gwalior Prashasti describes the exploits

of Nagabhata who was a Pratihara king

Q34) Answer b

Explanation

Rajatarangini is a Sanskrit text written

by Kalhana in the 12th century

It was historical chronicle of early India

It is justifiably considered to be the best

and most authentic work of its kind

It covers the entire span of history in

the Kashmir region from the earliest

times to the date of its composition

Q35) Answer c

Explanation

ldquoUrrdquo was the general assembly of the

village ldquoUrrdquo consisted of all the

taxpaying residents of an ordinary

village

Q36) Answer (a)

Explanation

Tarikh was a form of history writing in

the Delhi Sultanate The authors of

tawarikhs were learned men which

included secretaries administrators etc

Q37 Answer (a)

Explanation

Alauddin chose to pay his soldiers salaries in cash rather than iqtas The soldiers would buy their supplies from merchants in Delhi and it was thus feared that merchants would raise their prices To stop this Alauddin controlled the prices of goods in Delhi Prices were carefully surveyed by officers and merchants who did not sell at the prescribed rates were punished

Q38) Answer (d)

Explanation

Delhi first became the capital of a

kingdom under the Tomara Rajputs

who were defeated in the middle of the

twelfth century by the Chauhans (also

referred to as Chahamanas) of Ajmer

It was under the Tomaras and

Chauhans that Delhi became an

important commercial centre Many rich

Jaina merchants lived in the city and

constructed several temples Coins

minted here called dehliwal had a wide

circulation

Q39) Answer (c)

Explanation

Moth ki Masjid was built in the reign of

Sikandar Lodi by his minister

Begumpuri mosque built in the reign of

Muhammad Tughluq was the main

mosque of Jahanpanah the ldquoSanctuary

of the Worldrdquo and his new capital in

Delhi

Quwwat al ndash Islam mosque was

enlarged by Iltutmish and Alauddin

Khalji The minar was built by three

Sultansndash Qutbuddin Aybak Iltutmish

and Firuz Shah Tughluq

RAUSIAS-FC19E1003 45

Q40) Answer (c)

Explanation

Under the Mughals mansabdar was

referred to an individual who held a

mansab ie rank and he received his

salary as revenue assignments called

jagirs

Q41) Ans (b)

The Quit India Movement was a

spontaneous revolt of people against

British rule

The All India Congress Committee met

at Bombay on 8 August 1942 It passed

the famous resolution Quit India and

proposed the starting of a non-violent

mass struggle under Gandhis

leadership to achieve this aim But on

the very next day Gandhi and other

eminent leaders of the Congress were

arrested The Congress was once again

declared illegal

Q42) Ans (c)

The Simon Commission refers to a

group of seven MPs from the United

Kingdom constituted to suggest

constitutional reforms for British India

The Commission consisted of only

British members headed by one of the

senior British politicians Sir John

Simon

So the people of India agitated against

the arrival of Simon Commission

Q43) Ans (a)

He was widely known for his

unfavourable opinion of the economic

consequences of the British rule in

India

In his many writings and speeches and

especially in Poverty and Un-British

Rule in India Naoroji argued that India

was too highly taxed and that its wealth

was being drained away to England

He did not interpret the ancient Indian

texts and restored the self-confidence of

Indians And also he did not stress the

need for eradication of all the social

evils before anything else

Q44) Ans (c)

In August 1932 Prime Minister

MacDonald announced his Communal

Award Great Britainrsquos unilateral

attempt to resolve the various conflicts

among Indiarsquos many communal

interests

The award which was later

incorporated into the act of 1935

expanded the separate-electorate

formula reserved for Muslims to other

minorities including Sikhs Indian

Christians Anglo-Indians Europeans

distinct regional groups Gandhi

undertook a ldquofast unto deathrdquo against

that offer which he viewed as a

nefarious British plot to divide the

Indian society

Q45) Ans (b)

In British India apart from existing

imports and exports there was also a

particular amount of money which

colonial India contributed towards

administration maintenance of the

army war expenses pensions to retired

officers and other expenses accrued by

Britain towards maintenance of her

colony These were known as Home

charges and were paid for almost

entirely by India

The Home charges was made of

following components-

- Interest payable on Indian debt

- Dividend to shareholders of East

India Company

- Funds used to support the India

Office in London

- Funds used to pay salaries and

pensions of British personnel

engaged in India

- Interest on the railways

- Civil and military charges

- Store purchases in England

Q46) Ans (b)

The Lahore session of the Indian

National Congress was held in 1929

under the Presidentship of Jawaharlal

Nehru

The Lahore session of the Indian

National Congress witnessed significant

RAUSIAS-FC19E1003 46

developments in the Indian national

movement

- First the election of Jawaharlal

Nehru to the post of Presidentship of

the Congress was a clear indication

of the growing strength of the

Leftists in the Congress

- Secondly it was in this session that

the Congress for the first time raised

the demand for complete

independence Such demand was

not raised from the Congress

platform earlier

Q47) Ans (b)

It did not provide for separate

electorates for any community or

weightage for minorities However it did

allow for the reservation of minority

seats in provinces having minorities of

at least ten per cent but this was to be

in strict proportion to the size of the

community

There was no provision for complete

Independence for India

Q48) Ans (c)

The religion of early Vedic Aryans was

primarily of worship of nature and

Yajnas

The early Aryan religion was kind of

nature worship Actually the forces

around them which they could not

control or understand were invested

with divinity and were personified as

male or female gods And they

performed some Yajnas also

Q49) Ans (b)

The roads and river-routes were not

immune from robbery It is notable that

Yuan Chwang (Hiuen Tsang) was

robbed of his belongings during

Harshvardanarsquos period

Q50) Ans (c)

Q51) Ans (b)

Purandara Dasa was a saint and great

devotee of Lord Krishna

There is much speculation about where

Purandara Dasa regarded as the

Pitamaha of Carnatic music was born

Recently an expert committee

constituted by the Kannada University

Hampi has come to the conclusion that

Kshemapura Shivamogga district

Karnataka is the birth place of

Purandara Dasa

Q52) Ans (c)

Sri Tyagaraja Sri Shyama Shastry and Sri Muthuswami Dikshitar are considered the trinity of Carnatic music and with them came the golden age in Carnatic music in the 18th-19th

century

Q53) Ans d)

Recently a rare sarcophagus (stone

coffin) which is 2000 years old from the

Iron AgendashMegalithic era was discovered

from a rock-cut cave at Viyur village of

Kollam near Koyilandy in Kozhikode

district Kerala

The coffin containing bone fragments

was found during an excavation ldquoSo

far such a rare finding has been

discovered only from two sites

in Kerala Both these sarcophagi were

recovered from Megalithic sites at

Chevayur and Atholi also in Kozhikode

district

Q54) Ans a)

The megalithic culture in South India was a full-fledged Iron Age culture

Q55) Ans d)

The Cholas Pandyas and Keralaputras

(Cheras) mentioned in Ashokan

inscriptions were probably in the late

megalithic phase of material culture

Q56) Ans d)

Q57) Ans (b)

Raj Kumar Shukla followed Gandhiji all

over the country to persuade him to

come to Champaran to investigate the

problem associated with tinkathia

system

RAUSIAS-FC19E1003 47

Brij Kishore Rajendra Prasad Mahadev

Desai and Narhari Parikh accompanied

Gandhi ji during the Champaran

Satyagraha

Q58) Ans (b)

The Satvahanas started the practice of granting tax-free villages to brahmanas and Buddhist monks

Q59) Ans c)

The objectives of the Programme are

listed as under

- Developing basic tourism

infrastructure

- Promoting cultural and heritage

value of the country to generate

livelihoods in the identified regions

- Enhancing the tourist attractiveness

in a sustainable manner by

developing world-class

infrastructure at the heritage

monument sites

- Creating employment through active

involvement of local communities

- Harnessing tourism potential for its

effects on employment generation

and economic development

- Developing sustainable tourism

infrastructure and ensuring proper

Operations and maintenance

therein

Q60) Ans (b)

The Tribal Cooperative Marketing

Development Federation of India

(TRIFED) came into existence in 1987

It is a national-level apex organization

functioning under the administrative

control of Ministry of Tribal Affairs

Govt of India

TRIFED has its registered and Head

Office located in New Delhi

Q61) Ans (c)

Premchandrsquos novels include

Premashram Rangabhumi Ghaban

Karmabhumi and Godan

Gora is a novel written by Rabindranath

Tagore

138th birth anniversary of Munshi

Premchand was celebrated across the

country

Q62) Ans (b)

Giddha is a traditional pastoral dance

performed by the women of the Punjab

India and Pakistan at festival times

and at the sowing and reaping of the

harvest

By this dance the Punjabi women

reveal their joy expel their suppressed

feelings in a male dominated society

through the performance of Giddha

Since this dance has nothing to do with

men only women can participate in it

During the Teej celebrations Giddha

dance is celebrated in Punjab every

year Teej is a generic name for a

number of festivals that are celebrated

by women in some parts of India

Q63) Ans (a)

Dara Shukoh wrote the remarkable

work called ldquoMajma-ul-Bahrainrdquo or the

ldquoThe confluence of two seasrdquo

The Vice President of India Shri M

Venkaiah Naidu has said that Prince

Dara Shukohrsquos writings can come as a

refreshing source for infusing peace and

harmony He was addressing the

gathering after visiting the exhibition

that showcases the forgotten Prince of

yesteryears Dara Shukoh organized by

Mr Francois Gautier at Indira Gandhi

National Centre for the Arts in New

Delhi

Q64) Ans (c)

The statue Gommateshwara is

dedicated to the Jain God Bahubali

It is a monolithic statue

President Ram Nath Kovind

inaugurated the grand anointing

ceremony mdash Mahamastakabhisheka mdash

held once in 12 years at

Shravanabelagola (Karnataka)

Q65) Ans (c)

Prachi Valley had come up around the

Prachi river Prachi Valley gradually

disappeared

RAUSIAS-FC19E1003 48

The Prachi river originates from

Bhubaneswar

It is a tributary of the Mahanadi and

flows through the districts of Puri

Khurda Cuttack and Jagatsinghpur

and the entire region of the river is

termed as the Prachi Valley

It falls into the Bay of Bengal

Archaeological evidence shows that the

Prachi Valley Civilisation predates both

Harappa and Mohenjo-Daro

The Prachi river originates from

Bhubaneswar

Q66) Ans (d)

These monuments are located in

Chhatarpur district Madhya Pradesh

within Vindhya mountain range

Q67) Ans (a)

The book lsquoThoughts on Pakistanrsquo was

written by Dr BR Ambedkar

On the occasion of the birth anniversary

of Dr BR Ambedkar the president of

India pays homage to this icon of India

In 1924 he founded the Depressed

Classes Institute (Bahishkrit Hitkarini

Sabha) and in 1927 the Samaj Samata

Sangh

Another area of attention for Ambedkar

was education For its spread among

the low classes he set up a network of

colleges by the name of Peoples

Education Society and founded hostels

Q68) Ans(b)

Mehrgarh is a famous Neolithic

settlement in the Indian subcontinent

which is situated in Baluchistan

province Pakistan

A pre-historic rock art site is discovered

in the vast expanse of limestone blocks

on the eastern banks of Naguleru river

near Dachepalli (Andhra Pradesh) It

has thrown light on the Neolithic

civilisation that flourished in Guntur

(Andhra Pradesh) during 1500-2000

BC

Q69) Ans (c)

The 12th and the 13th centuries saw

the emergence of the Kakatiyas They

were at first the feudatories of the

Western Chalukyas of Kalyana Initially

they ruled over a small territory near

Warangal (Telangana)

They introduced Nayakships which was

later adopted and developed by the

Rayas of Vijayanagara

Q70) Ans (a)

The fast had effect of putting pressure

on mill owners who finally agreed to

give the workers a 35 per cent increase

in wages

Google celebrated with a doodle the

132nd birth anniversary of Anasuya

Sarabhai who played a pioneering role

in Indiarsquos labour movement

Q71) Ans (d)

The UNESCOrsquos list of the representative

list of the intangible cultural heritage of

humanity from India are

- Koodiyattam Sanskrit Theatre of

Kerala

- Mudiyettu ritual theatre and dance

drama of Kerala

- Tradition of Vedic Chanting

- Kalbelia folk songs and dances of

Rajasthan

- Ramlila Traditional Performance of

the Ramayana

- Sankirtana ritual singing

drumming and dancing of Manipur

- Ramman religious festival and

ritual theatre of the Garhwal

Himalayas India

- Traditional brass and copper craft of

utensil making among the Thatheras

of Jandiala Guru Punjab India

- Chhau dance classical Indian dance

originated in the eastern Indian

states

- Buddhist chanting of Ladakh

recitation of sacred Buddhist texts

in the trans-Himalayan Ladakh

region Jammu and Kashmir India

- Yoga

- Nouroz

- Kumbh Mela

RAUSIAS-FC19E1003 49

Q72) Ans(b)

The President of India Shri Ram Nath Kovind inaugurated the Hornbill Festival and State Formation Day celebrations of Nagaland in Kisama

The festival is named after the Indian hornbill the large and colourful forest bird which is displayed in the folklore of most of the states tribes

The major recognized tribes of Nagaland are Angami Ao Chakhesang Chang

Kuki Rengma and Zeling etc

Onge Jarawa and Sentinelese are the

tribes of Andman amp Nicobar Islands

Q73) Ans (c)

The Rashtrakutas rule in the Deccan lasted for almost two hundred years till the end of the tenth century The Rashtrakutas rulers were tolerant in their religious views and patronized not only Shaivism and Vaishnavism but

Jainism as well

The famous rock-cut temple of Shiva at Ellora was built by one of the Rashtrakutas kings Krishna I in the ninth century His successor Amoghavarsha was a Jain but he also

patronized other faiths

The Rashtrakutas allowed Muslims traders to settle and permitted Islam to

be preached in their dominions

Recently increasing defacement at the prehistoric rock paintings of Pandavulagutta Telangana has created a cause for grave concern It can spoil

the prehistoric rock

Pandavulagutta is home to

- Painted rock shelters dating to

10000 BC-8000 BC

- An 8th century inscription of the

Rashtrakuta period and

- Painted frescoes from the 12th century Kakatiya empire

Q74) Ans (b)

In 1828 Raja Ram Mohan Roy founded a new religious society the Brahma Sabha later known as the Brahmo

Samaj

Debendranath Tagore headed the Tattvabodhini Sabha which was

engaged in search of spiritual truth

Its purpose was to purify Hinduism and to preach monotheism or belief in one God

The new society was to be based on the twin pillars of reason and the Vedas and

Upanishads

Recently Sadharan Brahmo Samaj (SBS) has entered into a legal battle with the West Bengal government due

to some legal issue

Q75) Ans (c)

The Chishti order was established in India by Khwaja Moinuddin Chishti who came to India around 1192 The Chishtirsquos are considered to be the most influential of the groups of Sufis who migrated to India in the late twelfth century They adapted successfully to the local environment and adopted several features of Indian devotional

traditions

The historical dargah of Sufi mystic Khwaja Moinuddin Chishti in Ajmer is all set to get a facelift This 13 th century dargah has been included among the Swachh Iconic Places a clean-up initiative focused on iconic

heritage spiritual and cultural places

Page 17: GENERAL STUDIES (PAPER I) · Test is part of Rau’s IAS Test series for Preliminary Exam 2019 FOUNDATION + CURRENT AFFAIRS GENERAL STUDIES (PAPER –I) FOUNDATION TEST –III TOPIC:

RAUSIAS-FC19E1003 17

Q44) Mahatma Gandhi undertook fast unto

death in 1932 mainly because

(a) The Round Table Conference failed

to satisfy Indian political

aspirations

(b) The Congress and Muslim League

had differences of opinion

(c) Ramsay Macdonald announced the

Communal Award

(d) The Civil Disobedience Movement

failed

Q45) With reference to the period of colonial

rule in India ldquoHome Chargesrdquo formed

an important part of drain of wealth

from India Which of the following funds

constituted ldquoHome Chargesrdquo

1 Funds used to support the India

Office in London

2 Funds used to pay salaries and

pensions of British personnel

engaged in India

3 Funds used for waging wars

outside India by the British

Select the correct answer using the code

given below

(a) 1 only

(b) 1 and 2 only

(c) 2 and 3 only

(d) 1 2 and 3

Q46) The 1929- Session of Indian National

Congress is of significance in the history

of the Freedom Movement because the-

(a) attainment of Self-Government

was declared as the objective of

the Congress

(b) attainment of Poorna Swaraj was

adopted as the goal of the

Congress

(c) Non-Cooperation Movement was

launched

(d) decision to participate in the

Round Table Conference in

London was taken

Q47) With reference to the period of Indian

freedom struggle which of the following

waswere recommended by the Nehru

report

1 Complete Independence for India

2 Joint electorates for reservation of

seats for minorities

3 Provision of fundamental rights for

the people of India in the

Constitution

Select the correct answer using the code

given below

(a) 1 only

(b) 2 and 3 only

(c) 1 and 3 only

(d) 1 2 and 3

Q48) The religion of the early Vedic Aryans was primarily of

(a) Bhakti

(b) image worship and Yajnas

(c) worship of nature and Yajnas

(d) worship of nature and Bhakti

RAUSIAS-FC19E1003 18

Q49) भारत की यातरा करि िाि चीिी यातरी यआि चिाोग

(हयएि साोग) ि समकािीि भारत की सामानय

खसथनतय ो और सोसकनत क दजण नकया था इस सोदभण म

निमननिखित कथि ो म स कौि-सास सही हह

1 सड़क और िदी-मागण (जि-मागण) डकती स

परण रप स सरनकषत थ

2 जहा तक अपरार ो क निए दणड की बात ह

उसक निए नकसी भी वयखकत की निदोिता

अथिा उसक अपरार क निराणररत करि क

निए अनि जि और निि परि क माधयम क

सारि थ

3 वयापाररय ो क घाट ो और परनतबोर सटशि ो पर

शलक ो का भगताि करिा पड़ता था

िीच नदए गए कट का परय ग कर सही उततर चनिए

(a) किि 1

(b) किि 2 और 3

(c) किि 1 और 3

(d) 1 2 और 3

Q50) नसोर घाटी सभयता क सोदभण म निमननिखित कथि ो पर

निचार कीनजए

1 यह मखय रप स एक रमणनिरपकष सभयता थी

तथा हािाोनक इसम रानमणक ततव मौजद था

िनकि िह परनतिश पर हािी िही ो था

2 इस काि क दौराि भारत म कपास का परय ग

कपड़ा बिाि क निए नकया जाता था

उपयणकत कथि ो म स कौि-सास सही हह

(a) किि 1

(b) किि 2

(c) 1 और 2 द ि ो

(d) ि त 1 ि ही 2

Q51) परोदर दास क सोदभण म निमननिखित कथि ो पर निचार

कीनजए

1 परोदर दास एक सोत और भगिाि नशि क एक

महाि भकत थ

2 ि एक सोगीतकार गायक और किाणटक सोगीत

क मखय सोसथापक-परसतािक ो म स एक थ

उपयणकत कथि ो म स कौि-सास सही हह

(a) किि 1

(b) किि 2

(c) 1 और 2 द ि ो

(d) ि त 1 ि ही 2

Q52) निमननिखित म स कौि-सास वयखकत किाणटक सोगीत

की नतरमनतण म शानमि हह

1 बािामरिी कषणा

2 शरी शयाम शासतरी

3 शरी मथसवामी दीनकषतर

िीच नदए गए कट का परय ग कर सही उततर चनिए

(a) किि 1

(b) किि 2

(c) किि 2 और 3

(d) 1 2 और 3

Q53) चियर (Chevayur) और अथ िी (Atholi) म खसथत

महापािार सथि निमननिखित म स नकस राजय म खसथत

(a) तनमििाड

(b) किाणटक

(c) पनिम बोगाि

(d) करि

RAUSIAS-FC19E1003 19

Q49) The Chinese traveller Yuan Chwang

(Hiuen Tsang) who visited India

recorded the general conditions and

culture of India at that time In this

context which of the following

statements isare correct

1 The roads and river-routes were

completely immune from robbery

2 As regards punishment for

offences ordeals by fire water and

poison were the instruments for

determining the innocence or guilt

of a person

3 The tradesmen had to pay duties

at ferries and barrier stations

Select the correct answer using the code

given below

(a) 1 only

(b) 2 and 3 only

(c) 1 and 3 only

(d) 1 2 and 3

Q50) Regarding the Indus Valley Civilization

consider the following statements

1 It was predominantly a secular

civilization and the religious

element though present did not

dominate the scene

2 During this period cotton was

used for manufacturing textiles in

India

Which of the statements given above

isare correct

(a) 1 only

(b) 2 only

(c) Both 1 and 2

(d) Neither 1 nor 2

Q51) Consider the following statements

regarding Purandara Dasa

1 Purandara Dasa was a saint and

great devotee of Lord Shiva

2 He was a composer singer and

one of the chief founding-

proponents of the Carnatic music

Which of the statements given above

isare correct

(a) 1 only

(b) 2 only

(c) Both 1 and 2

(d) Neither 1 nor 2

Q52) Which of the following persons isare

included in the trinity of Carnatic

music

1 Balamurali Krishna

2 Sri Shyama Shastry

3 Sri Muthuswami Dikshitar

Select the correct answer using the code

given below

(a) 1 only

(b) 2 only

(c) 2 and 3 only

(d) 1 2 and 3

Q53) Megalithic sites at Chevayur and Atholi

are located in which of the following

states

(a) Tamil Nadu

(b) Karnataka

(c) West Bengal

(d) Kerala

RAUSIAS-FC19E1003 20

Q54) निमननिखित कथि ो पर निचार कीनजए

1 महापािानरक ि ग कबर ो म िसतएो दफिात थ

2 दनकषर भारत म महापािार सोसकनत एक परण

निकनसत तामर यगीि सोसकनत थी

उपयणकत कथि ो म स कौि-सास सही हह

(a) किि 1

(b) किि 2

(c) 1 और 2 द ि ो

(d) ि त 1 ि ही 2

Q55) निमननिखित म स कौि-स सामराजयसामराजय ो का

अश क क अनभिि ो म उललि नकया गया ह

1 च ि

2 पाणडय

3 करिपतर (चर)

िीच नदए गए कट का परय ग कर सही उततर चनिए

(a) किि 1

(b) किि 1 और 2

(c) किि 3

(d) 1 2 और 3

Q56) भीमा-क रगाोि का यदध को पिी क सनिक ो और

बाजीराि नदवतीय क िततव म एक शखकतशािी पशिा

सिा (मराठ ो) क मधय िड़ा गया था यह यदध

निमननिखित म स नकसका नहससा था

(a) परथम आोगल-मराठा यदध का

(b) नदवतीय आोगल-मराठा यदध का

(c) ततीय आोगल-मसर यदध का

(d) ततीय आोगल-मराठा यदध का

Q57) निमननिखित कथि ो पर निचार कीनजए

1 महादि दसाई ि गाोरीजी क चोपारर आि तथा

नतिकनथया पररािी स जड़ी समसया की जाोच

क निए रारी करि क निए दश भर म उिका

अिसरर नकया था

2 िरहरी पाररि चोपारर सतयागरह क दौराि

गाोरीजी क साथ थ

उपयणकत कथि ो म स कौि-सास सही हह

(a) किि 1

(b) किि 2

(c) 1 और 2 द ि ो

(d) ि त 1 ि ही 2

Q58) निमननिखित कथि ो पर निचार कीनजए

1 िनद राज-िोश ि बराहमर ो और बौदध मठराररय ो

क कर-मकत गाि अिदाि म दि की परथा

आरि की थी

2 सतिाहि ो की आनरकाररक भािा पराकत थी

उपयणकत कथि ो म स कौि-सास सही हह

(a) किि 1

(b) किि 2

(c) 1 और 2 द ि ो

(d) ि त 1 ि ही 2

Q59) एक निरासत क अपिाइए (अडॉपट ए हररटज ndash

Adopt a Heritage) पररय जिा क उददशय ो क

सनदभण म निमननिखित कथि ो पर निचार कीनजए

1 यह पररय जिा र रगार उतपादि और आनथणक

निकास क निए पयणटि कषमता का उि पर

परभाि का उपय ग करगी

2 यह पररय जिा निरासत सथि ो पर निशव सतरीय

आराररक सोरचिा निकनसत करक एक सतत

तरीक स पयणटक आकिणर म िखदध करगी

उपयणकत कथि ो म स कौि-सास सही हह

(a) किि 1

(b) किि 2

(c) 1 और 2 द ि ो

(d) ि त 1 ि ही 2

RAUSIAS-FC19E1003 21

Q54) Consider the following statements

1 Megalithic people buried goods in

graves

2 The megalithic culture in South

India was a full-fledged Copper

Age culture

Which of the statements given above

isare correct

(a) 1 only

(b) 2 only

(c) Both 1 and 2

(d) Neither 1 nor 2

Q55) Which of the following kingdoms isare

mentioned in the Ashokan inscriptions

1 Cholas

2 Pandyas

3 Keralaputras (Cheras)

Select the correct answer using the code

given below

(a) 1 only

(b) 1 and 2 only

(c) 3 only

(d) 1 2 and 3

Q56) The Battle of Bhima-Koregaon was

fought between the soldiers of the

Company and the strong Peshwa army

(Marathas) under Bajirao II This war

was a part of the

(a) First Anglo-Maratha war

(b) Second Anglo-Maratha war

(c) Third Anglo- Mysore war

(d) Third Anglo-Maratha war

Q57) Consider the following statements

1 Mahadev Desai followed Gandhiji all over the country to persuade him to come to Champaran to investigate the problem associated

with tinkathia system

2 Narhari Parikh accompanied Gandhi ji during the Champaran

Satyagraha

Which of the statements given above isare correct

(a) 1 only

(b) 2 only

(c) Both 1 and 2

(d) Neither 1 nor 2

Q58) Consider the following statements

1 The Nanda Dynasty started the practice of granting tax-free villages to brahmanas and

Buddhist monks

2 The official language of the Satavahanas was Prakrit

Which of the statements given above

isare correct

(a) 1 only

(b) 2 only

(c) Both 1 and 2

(d) Neither 1 nor 2

Q59) Consider the following statements about the objectives of the lsquoadopt a heritagersquo

project

1 It will harness tourism potential for its effects on employment generation and economic

development

2 It will enhance the tourist attractiveness in a sustainable manner by developing world class infrastructure at heritage sites

Which of the statements given above

isare correct

(a) 1 only

(b) 2 only

(c) Both 1 and 2

(d) Neither 1 nor 2

RAUSIAS-FC19E1003 22

Q60) ldquoभारतीय जिजातीय सहकारी निपरि निकास सोघrdquo

(The Tribal Co-operative Marketing

Development Federation of India - TRIFED)

क सोदभण म निमननिखित कथि ो पर निचार कीनजए

1 यह एक राषटर ीय सतर का शीिण सोगठि ह ज

भारत सरकार क गह मोतरािय क परशासनिक

नियोतरर क अरीि काम कर रहा ह

2 इसका मखय उददशय दश म जिजातीय ि ग ो

का सामानजक-आनथणक निकास करिा ह

उपयणकत कथि ो म स कौि-सास सही हह

(a) किि 1

(b) किि 2

(c) 1 और 2 द ि ो

(d) ि त 1 ि ही 2

Q61) निमननिखित म स कौि-सास उपनयास परमचोद क

दवारा नििा गया हनिि गए ह

1 रोगभनम

2 ग दाि

3 ग रा

िीच नदए गए कट का परय ग कर सही उततर चनिए

(a) किि 1

(b) किि 2

(c) किि 1 और 2

(d) 1 2 और 3

Q62) नगदधा ितय क सोदभण म निमननिखित कथि ो पर निचार

कीनजए

1 नगदधा नबहार की मनहिाओो क दवारा तयौहार क

समय और फसि की बिाई तथा कटाई क

अिसर पर नकया जाि िािा एक पारोपररक

दहाती ितय ह

2 इस ितय क दवारा मनहिाऐो अपिी परसननता

परकट करती ह तथा नगदधा क परदशणि क

माधयम स परि िचणसव िाि समाज म

मनहिाओो की दबी हई भाििाओो क परकट

करती ह

उपयणकत कथि ो म स कौि-सास सही हह

(a) किि 1

(b) किि 2

(c) 1 और 2 द ि ो

(d) ि त 1 ि ही 2

Q63) निमननिखित कथि ो पर निचार कीनजए

1 मलला शाह बदखशी दारा नशक ह क

आधयाखतमक गर थ

2 औरोगरब ि मजम-उि-बहरीि या द समदर ो

का सोगम िामक उललििीय रचिा नििी थी

3 दारा नशक ह क अपि पिणज अकबर क गर ो

क उततरानरकारी क रप म दिा गया था

नजसम उसि रानमणक बहििाद और समनवयता

क बढ़ािा नदया था

उपयणकत कथि ो म स कौि-सास सही हह

(a) किि 1 और 3

(b) किि 2

(c) किि 1 और 2

(d) 1 2 और 3

RAUSIAS-FC19E1003 23

Q60) Consider the following statements about

the Tribal Cooperative Marketing

Development Federation of India

(TRIFED)

1 It is a national-level apex

organization functioning under the

administrative control of Ministry

of Home Affairs Government of

India

2 The main objective of TRIFED is

socio-economic development of

tribal people in the country

Which of the statements given above

isare correct

(a) 1 only

(b) 2 only

(c) Both 1 and 2

(d) Neither 1 nor 2

Q61) Which of the following novels isare

written by Premchand

1 Rangabhumi

2 Godan

3 Gora

Select the correct answer using the code

given below

(a) 1 only

(b) 2 only

(c) 1 and 2 only

(d) 1 2 and 3

Q62) Consider the following statements about

Giddha dance

1 Giddha is a traditional pastoral

dance performed by the women of

Bihar at festival times and at the

sowing and reaping of the harvest

2 By this dance the women reveal

their joy expel their suppressed

feelings in a male dominated

society through the performance of

Giddha

Which of the statements given above

isare correct

(a) 1 only

(b) 2 only

(c) Both 1 and 2

(d) Neither 1 nor 2

Q63) Consider the following statements

1 Mullah Shah Badakhshi was the

spiritual mentor of Dara Shukoh

2 Aurangzeb wrote the remarkable

work called ldquoMajma-ul-Bahrainrdquo or

the ldquoThe confluence of two seasrdquo

3 Dara Shukoh was seen as

inheriting the qualities of his

ancestor Akbar in that he

promoted religious pluralism and

syncretism

Which of the statements given above

isare correct

(a) 1 and 3 only

(b) 2 only

(c) 1 and 2 only

(d) 1 2 and 3

RAUSIAS-FC19E1003 24

Q64) निमननिखित कथि ो पर निचार कीनजए

1 ग मतशवर परनतमा निोधयनगरी पहाड़ी पर खसथत ह

2 शरिरबिग िा िह सथाि ह जहाो मौयण िोश क

सोसथापक चोदरगपत मौयण अपि नसोहासि क

तयागि क बाद जि तपसवी बि गए थ

उपयणकत कथि ो म स कौि-सास सही हह

(a) किि 1

(b) किि 2

(c) 1 और 2 द ि ो

(d) ि त 1 ि ही 2

Q65) निमननिखित कथि ो पर निचार कीनजए

1 पराताखतवक साकषय स पता चिता ह नक पराची

घाटी सभयता हड़पपा और म हिज दाड़ द ि ो

की पिणिती ह

2 पराची िदी भििशवर स निकिती ह

उपयणकत कथि ो म स कौि-सास सही हह

(a) किि 1

(b) किि 2

(c) 1 और 2 द ि ो

(d) ि त 1 ि ही 2

Q66) निमननिखित कथि ो म स कौि-सास सही हह

1 िजराह क समारक ो क समह का निमाणर

चोदि राजिोश क शासिकाि क दौराि हआ

था

2 य समारक हररिोदर पिणत शरोििा म खसथत ह

3 म रक क यातरी इबन बतता ि अपि सोसमरर ो

म िजराह क मोनदर ो की यातरा का उललि

नकया था तथा इन काजराण िाम स समब नरत

नकया था

िीच नदए गए कट का परय ग कर सही उततर चनिए

(a) किि 1

(b) किि 1 और 2

(c) किि 2 और 3

(d) किि 1 और 3

Q67) निमननिखित कथि ो म स कौि-सास सही हह

1 डॉ बी आर अमबडकर ि दी एनिनहिशि

ऑफ़ कासट (The Annihilation of Caste)

नििी थी नजसम उन ोि नहोद रमण म िोशािगत

पजारी की परथा क उनमिि की आिशयकता

पर बि नदया था

2 डॉ राजदर परसाद ि थॉटस ऑि पानकसताि

(Thoughts on Pakistan) िामक पसतक

नििी थी

िीच नदए गए कट का परय ग कर सही उततर चनिए

(a) किि 1

(b) किि 2

(c) 1 और 2 द ि ो

(d) ि त 1 ि ही 2

Q68) निमननिखित कथि ो म स कौि-सास सही हह

1 महरगढ़ भारतीय उपमहादवीप म एक परनसदध

ििपािार बसती ह ज नसोर पराोत पानकसताि म

खसथत ह

2 बरणह म म कतत ो क उिक सवामी क साथ कबर ो

म दफिाया जाता था

िीच नदए गए कट का परय ग कर सही उततर चनिए

(a) किि 1

(b) किि 2

(c) 1 और 2 द ि ो

(d) ि त 1 ि ही 2

Q69) निमननिखित कथि ो म स कौि-सास सही हह

1 काकानटय मोनदर अनरकतर नशि क समनपणत

2 हिमक ोडा म हजार-सतोभ िाि मोनदर (The

Thousand-Pillared Temple) का निमाणर

काकानटय समराट रदर ि करिाया था

िीच नदए गए कट का परय ग कर सही उततर चनिए

(a) किि 1

(b) किि 2

(c) 1 और 2 द ि ो

(d) ि त 1 ि ही 2

RAUSIAS-FC19E1003 25

Q64) Consider the following statements

1 Gommateshwara Statue is located

on the Vindyagiri Hill

2 Shravanabelagola is the place

where Chandragupta Maurya the

founder of the Mauryan dynasty

became a Jain ascetic after

relinquishing his throne

Which of the statements given above

isare correct

(a) 1 only

(b) 2 only

(c) Both 1 and 2

(d) Neither 1 nor 2

Q65) Consider the following statements

1 Archaeological evidence shows

that the Prachi Valley Civilisation

predates both Harappa and

Mohenjo-Daro

2 The Prachi river originates from

Bhubaneswar

Which of the statements given above

isare correct

(a) 1 only

(b) 2 only

(c) Both 1 and 2

(d) Neither 1 nor 2

Q66) Which of the following statements

isare correct

1 The Khajuraho group of

monuments was built during the

rule of the Chandela dynasty

2 These monuments are located in

Harischandra mountain range

3 Ibn Battuta the Moroccan

traveller in his memoirs mentioned

visiting Khajuraho temples and

called them Kajarra

Select the correct answer using the code

given below

(a) 1 only

(b) 1 and 2

(c) 2 and 3

(d) 1 and 3

Q67) Which of the following statements

isare correct

1 Dr BR Ambedkar wrote the

Annihilation of Caste emphasising

the need to do away with the

practice of hereditary priesthood in

Hinduism

2 The book lsquoThoughts on Pakistanrsquo

was written by Dr Rajendra

Prasad

Select the correct answer using the code

given below

(a) 1 only

(b) 2 only

(c) Both 1 and 2

(d) Neither 1 nor 2

Q68) Which of the following statements

isare correct

1 Mehrgarh is a famous Neolithic

settlement in the Indian

subcontinent which is situated in

Sindh province Pakistan

2 At Burzahom dogs were buried

with their masters in their graves

Select the correct answer using the code

given below

(a) 1 only

(b) 2 only

(c) Both 1 and 2

(d) Neither 1 nor 2

Q69) Which of the following statements

isare correct

1 The Kakatiya temples are

dedicated mostly to Siva

2 The Thousand-Pillared Temple at

Hanamkonda was built by the

Kakatiya king Rudra

Select the correct answer using the code

given below

(a) 1 only

(b) 2 only

(c) Both 1 and 2

(d) Neither 1 nor 2

RAUSIAS-FC19E1003 26

Q70) निमननिखित कथि ो म स कौि-सास सही हह

1 अहमदाबाद नमि हड़ताि क दौराि महातमा

गाोरी ि शरनमक ो क पकष क मजबत करि क

निए आमरर अिशि नकया था

2 अिशि स नमि मानिक ो पर दबाि पड़ा था ज

अोततः शरनमक ो क िति म 15 परनतशत की िखदध

करि क निए सहमत हए थ

िीच नदए गए कट का परय ग कर सही उततर चनिए

(a) किि 1

(b) किि 2

(c) 1 और 2 द ि ो

(d) ि त 1 ि ही 2

Q71) निमननिखित म स नकसक नकिक भारत स यिसक

की माििता की अमतण साोसकनतक निरासत की

परनतनिनर सची (The UNESCOrsquos List of the

Representative List of the Intangible

Cultural Heritage of Humanity) म शानमि

नकया गया ह

1 मनडयटट

2 सोकीतणि

3 को भ मिा

िीच नदए गए कट का परय ग कर सही उततर चनिए

(a) किि 1 और 2

(b) किि 2 और 3

(c) किि 3

(d) 1 2 और 3

Q72) निमननिखित जिजानतय ो म स कौि-सीसी ो

जिजानतजिजानतया िागािड स सोबोनरत हह

1 अोगामी

2 ककी

3 जारिा

िीच नदए गए कट का परय ग कर सही उततर चनिए

(a) किि 1

(b) किि 1 औऔ 2

(c) किि 2

(d) 1 2 और 3

Q73) निमननिखित कथि ो म स कौि-सास सही हह

1 राषटर कट सामराजय की सथापिा दोनतदगण ि की थी

नजसि मानयाित म अपिी राजरािी की

सथापिा की थी

2 राषटर कट समराट अम घििण एक ििक था और

उस कनिताओो पर पहिी कननड़ पसतक नििि

का शरय नदया जाता ह

िीच नदए गए कट का परय ग कर सही उततर चनिए

(a) किि 1

(b) किि 2

(c) 1 और 2 द ि ो

(d) ि त 1 ि ही 2

Q74) निमननिखित कथि ो म स कौि-सास सही हह

1 कशब चोदर सि ि ततवब नरिी सभा की

अधयकषता की थी ज आधयाखतमक सतय की

ि ज म सोिि थी

2 बरहम समाज ि मािि गररमा पर बि नदया

मनतणपजा का निर र नकया और सती परथा जसी

सामानजक बराइय ो की आि चिा की

िीच नदए गए कट का परय ग कर सही उततर चनिए

(a) किि 1

(b) किि 2

(c) 1 और 2 द ि ो

(d) ि त 1 ि ही 2

Q75) निमननिखित कथि ो म स कौि-सास सही हह

1 भारत म नचशती नसिनसिा खवाजा म इिददीि

नचशती क दवारा सथानपत नकया गया था

2 नचशती परोपरा की एक परमि निशिता

आतमसोयम थी नजसम साोसाररक म ह स दरी

बिाए रििा शानमि था

िीच नदए गए कट का परय ग कर सही उततर चनिए

(a) किि 1

(b) किि 2

(c) 1 और 2 द ि ो

(d) ि त 1 ि ही 2

RAUSIAS-FC19E1003 27

Q70) Which of the following statements

isare correct

1 During the Ahmedabad Mill Strike

Mahatma Gandhi undertook a fast

unto death to strengthen the

workersrsquo resolve

2 The fast had effect of putting

pressure on mill owners who

finally agreed to give the workers a

15 per cent increase in wages

Select the correct answer using the code

given below

(a) 1 only

(b) 2 only

(c) Both 1 and 2

(d) Neither 1 nor 2

Q71) Which of the following are included in

the UNESCOrsquos list of the representative

list of the intangible cultural heritage of

humanity from India

1 Mudiyettu

2 Sankirtana

3 Kumbh Mela

Select the correct answer using the code

given below

(a) 1 and 2 only

(b) 2 and 3 only

(c) 3 only

(d) 1 2 and 3

Q72) Which of the following tribes isare

related to Nagaland

1 Angami

2 Kuki

3 Jarawa

Select the correct answer using the code

given below

(a) 1 only

(b) 1 and 2 only

(c) 2 only

(d) 1 2 and 3

Q73) Which of the following statements

isare correct

1 Rashtrakuta kingdom was founded by Dantidurga who established his capital at Manyakhet

2 Amoghavarsha a Rashtrakuta king was an author and is credited with writing the first

Kannada book on poetics

Select the correct answer using the code given below

(a) 1 only

(b) 2 only

(c) Both 1 and 2

(d) Neither 1 nor 2

Q74) Which of the following statements isare correct

1 Keshab Chandra Sen headed the Tattvabodhini Sabha which was engaged in search of spiritual truth

2 The Brahmo Samaj laid emphasis on human dignity opposed idolatry and criticized such social

evils as the practice of Sati

Select the correct answer using the code given below

(a) 1 only

(b) 2 only

(c) Both 1 and 2

(d) Neither 1 nor 2

Q75) Which of the following statements isare correct

1 The Chishti order was established in India by Khwaja Moinuddin

Chishti

2 A major feature of the Chishti tradition was austerity including maintaining a distance from the

worldly power

Select the correct answer using the code

given below

(a) 1 only

(b) 2 only

(c) Both 1 and 2

(d) Neither 1 nor 2

T e s t i s p a r t o f R a u rsquo s I A S T e s t s e r i e s f o r P r e l i m i n a r y E x a m 2 0 1 9

FOUNDATION + CURRENT AFFAIRS

GENERAL STUDIES (PAPER ndashI)

FOUNDATION TEST ndashIII

SUBJECT NCERT History Class VI-X + Current Affairs

Time Allowed 1frac12 Hours Maximum Marks 150

I NSTRUCT IONS

1 IMMEDIATELY AFTER THE COMMENCEMENT OF THE EXAMINATION YOU SHOULD CHECK

THAT THIS TEST BOOKLET DOES NOT HAVE ANY UNPRINTED OR TORN or MISSING PAGES OR

ITEMS ETC IF SO GET IT REPLACED BY A COMPLETE TEST BOOKLET

2 This Test Booklet contains 75 items (questions) Each item is printed both in Hindi and English

Each item comprises four responses (answers) You will select the response which you want to mark

on the Answer Sheet In case you feel that there is more than one correct response mark the

response which you consider the best In any case choose ONLY ONE response for each item

3 You have to mark all your responses ONLY on the separate Answer Sheet (OMR sheet) provided

Read the directions in the Answer Sheet

4 All items carry equal marks

5 Before you proceed to mark in the Answer Sheet the response to various items in the Test booklet

you have to fill in some particulars in the Answer Sheet as per instructions contained therein

6 After you have completed filling in all your responses on the Answer Sheet and the examination has

concluded you should hand over to the Invigilator only the Answer Sheet You are permitted to

take away with you the Test Booklet

7 Penalty for wrong answers

THERE WILL BE PENALTY FOR WRONG ANSWERS MARKED BY A CANDIDATE IN THE

OBJECTIVE TYPE QUESTION PAPERS

(i) There are four alternatives for the answer to every question For each question for which a

wrong answer has been given by the candidate one-third of the marks assigned to that

question will be deducted as penalty

(ii) If a candidate gives more than one answer it will be treated as a wrong answer even if one of

the given answers happens to be correct and there will be same penalty as above to that

question

(iii) If a question is left blank ie no answer is given by the candidate there will be no penalty for

that question

T h i s t e s t i s p a r t o f R a u rsquo s I A S T e s t s e r i e s f o r P r e l i m i n a r y E x a m 2 0 1 9

Test Code

FC19E1003

FC19H1003 29

Answers and Explanations of

NCERT History Class VI-X + Current Affairs (FC19E1003)

Q1) उततर (c)

सपषटीकरण

- ऋगवद म दविय ो और दिताओो क समवपित एक

हजार स अविक सत तर (शल क) ह

- य शल क ऋविय ो क दवारा रच गए थ और परि ो

दवारा सीख जात थ

- हालाोवक कछ शल क मवहलाओो (जस वक अपाला

घ सा ल पामदरा मतरयी और गागी) क दवारा भी रच

गए थ

- ऋगवद म सोिाद क रप म कई शल क मौजद ह

- हम विशवावमतर नामक एक ऋवि और दविय ो क

रप म पजी जान िाली द नवदय ो (वयास और

सतलज) क बीच िाताि का उदाहरण वमलता ह

- इसस पता चलता ह वक विशवावमतर िवदक काल स

सोबोवित थ

Q2) उततर (b)

सपषटीकरण

- करनल गफाओो स राख क अिशि परापत हए ह

ज इस ओर सोकत करत ह वक ततकालीन ल ग

अवि क उपय ग स पररवचत थ

- य गफाएो आोधर परदश म सथथत ह

Q3) उततर (c)

सपषटीकरण

bull बरािह म ितिमान कशमीर म सथथत एक

परागवतहावसक थथल ह जहाो ल ग गडढ क घर ो का

वनमािण करत थ

bull य घर जमीन क ख द कर बनाए जात थ तथा नीच

जान क वलए सीवियाा ह ती थी

bull ऐसा अनमान लगाया जाता ह वक य घर ठो ड क

मौसम म आशरय परदान करत थ

Q4) उततर (c)

सपषटीकरण

bull परालख-विदया (Epigraphy) क वशलालख ो क

अधययन क रप म पररभावित वकया जाता ह

bull हसतवलसखत दसतािज ो क माधयम स इवतहास

और सावहतय क अधययन क पाोडवलवप विजञान

(Manuscriptology) कहत ह

bull पराचीन लखन परणावलय ो क अधययन और

ऐवतहावसक पाोडवलवपय ो क समझन तथा वतवथ

वनिािरण क पलीओगराफी (Palaeography) कहा

जाता ह

bull नयवमजमविकस (Numismatics) वसक ो क

अधययन क सोदवभित करता ह

Q5) उततर (a)

सपषटीकरण

- चरक सोवहता चरक क दवारा वलखी गई आयिद

और िदयक-शासर पर एक महतवपणि पसतक ह

- ि भारतीय िदयक-शासर की पारमपररक परणाली

वजस आयिद क नाम स जाना जाता ह क

अभयासकताि थ

- ऐसा माना जाता ह वक चरक का विकास दसरी

शताबदी (ईसा पिि) और दसरी शताबदी (ईसवी) क

मधय हआ था

Q6) उततर (b)

सपषटीकरण

- भाग फसल ो पर वलए जान िाल कर क सोदवभित

करता ह ज कल फसल उतपादन का 16 िाो भाग

था

- ldquoकममकारrdquo शबद भवमहीन कवि शरवमक िगि क

वलए परय ग वकया जाता था

- ldquoअशवमिrdquo (वजस घ ड क बवलदान क रप म भी

जाना जाता ह) एक अनषठान ह ता था वजसम एक

घ ड क सवतोतर रप स घमन क वलए छ ड वदया

FC19H1003 30

जाता ह और राजा क सवनक उसकी रखिाली

करत थ

Q7) उततर (d)

सपषटीकरण

- ऋगववदक काल म घ ड ो क रथ ो म ज ता जाता था

ज (रथ) भवम मिवशय ो आवद पर कबजा करन क

वलए लड गए यद ो म उपय ग वकए जात थ

- इसस यह पता चलता ह वक घ ड ो यकत रथ ो का

उपय ग महाजनपद काल स काफी पहल आरमभ

हआ था

- ऋगववदक काल म मिवशय ो भवम जल आवद पर

कबजा करन क वलए तथा ल ग ो क पकडन क

वलए यद वकय जात थ

- अविकाोश परि इन यद ो म भाग वलया करत थ

- हालाोवक उस समय क ई वनयवमत सना नही ो ह ती

थी लवकन उस काल म सभाऐो ह ती थी ो वजनम

ल ग यद क मामल ो पर चचाि करत थ

- वनयवमत सनाएा महाजनपद काल का िवशषटय थी

वजनम पदल सवनक ो की विशाल सनाएा रथ तथा

हाथी शावमल ह त थ

Q8) उततर (a)

सपषटीकरण

- बद शाकय कल स सोबोवित थ और कशीनारा म

उनका वनिन हआ था

- बद न अपनी वशकषाएा पराकत भािा म दी थी ो ज

आम ल ग ो की भािा थी

Q9) उततर (c)

सपषटीकरण

- पराचीन भारत म दशिनशासर की छह शाखाएा थी ो

िशविक नयाय समखया य ग पिि वममाोसा और

िदाोत या उततर वममाोसा

- इनकी थथापना करमश कनाद गौतम कवपल

पतोजवल जावमनी और वयास ऋविय ो न की थी

Q10) उततर (b)

सपषटीकरण

महािीर की वशकषाऐो छठी शताबदी म िललभी म

सोकवलत की गई थी ो

Q11) उततर (c)

सपषटीकरण

- पारमपररक रप स चाणकय क कौविलय अथिा

विषणगपत क नाम स जाना जाता ह

- उसन अथिशासतर ज एक पराचीन भारतीय

राजनवतक आलख ह वलखा था

Q12) उततर (d)

सपषटीकरण

- भारत का राषटर ीय वचनह सारनाथ (उततर परदश) क

अश क सतमभ क ऊपर (शीिि पर) वसोह कवपिल

का एक अनरपण ह

- इस राषटर ीय वसदाोत सतयमि जयत क साथ

सोय वजत वकया गया ह

- रामपिि बल का नाम रामपिि (वबहार) क नाम पर

पडा जहाा इसकी ख ज हई थी

- यह अपन नाजक नकाशी मॉडल क वलए परवसदद

ह वजसम क मल तवचा सोिदनशील नथन ो सतकि

कान और मरबत िााग ो क शरषठतर परवतरप क

परदवशित वकया गया ह

- यह भारतीय और फारसी ततव ो का एक ससममशरण

- सोवकससा उततर परदश म सथथत ह

Q13) उततर (a)

सपषटीकरण

का िर वसोह ज एक महान य दा थ वबहार स

सोबोवित थ

Q14) उततर (b)

सपषटीकरण

िललालर शबद बड भ-सवावमय ो क वलए परय ग

वकया जाता था

FC19H1003 31

Q15) उततर (c)

सपषटीकरण

- अररकमड एक तिीय बसती थी जहाो दर दश ो स

आन िाल जहाज ो का माल उतारा जाता था

- यहाो पर ईोि ो का एक विशाल ग दाम वमटटी क

बतिन (वजनम एमफ रा - द हरी मवठय ो का लोबा

घडा - शावमल ह) और एरिाइन (Arretine)

मदभाोड पाए गए थ

- इस थथान पर र मन दीपक काोच क बन पातर और

रतन भी पाए गए थ

Q16) उततर (a)

सपषटीकरण

- मिनदर सोगम कविताओो म उसललसखत एक

तवमल शबद ह वजसका अथि ह ldquoतीन परमखrdquo

- यह तीन सततारि पररिार ो क मसखयाओो क वलए

परय ग वकया जाता ह च ल चर और पाणडय

Q17) उततर (c)

सपषटीकरण

- ऋग िद म सभा विदाथा तथा गण जसी

जनजावतय ो पर अथिा किोब पर आिाररत

सभाओो का उललख ह

- आरसमभक िवदक काल म सभाओो और सवमवतय ो

का विशि महतव ह ता था

- यहाा तक की मसखया अथिा राजा भी उनका

समथिन परापत करन क वलए आतर रहत थ

Q18) उततर (a)

सपषटीकरण

- जन िमि न ईशवर क अससततव क मानयता त दी ह

वकनत उसन ईशवर क वजना क पद स नीच रखा

- जन िमि न बौद िमि की तरह िणि परणाली की

भरतिना नही ो की थी

Q19) उततर (d)

सपषटीकरण

- च ल ो और पाणडय ो न शसकतशाली तिीय शहर ो का

विकास वकया था

- च ल ो का सबस महतवपणि शहर पहार (या

कािरीपटटीनम) था |

- मदरई पाणडय ो की राजिानी थी

Q20) उततर (b)

सपषटीकरण

- ldquoबदचररतrdquo बद का जीिन-ितताोत ह

- इस अशवघ ि क दवारा वलखा गया था

Q21) उततर (a)

सपषटीकरणः

- तवमल कवि अपपर भगिान वशि क भकत थ

- इस परकार ि एक नयनार सोत थ

Q22) उततर (d)

सपषटीकरणः

- समदरगपत एक परवसद गपत शासक था

- उसन वसक ो पर िीणा बजात हए अपनी छवि

अोवकत करिाई थी

- यह सोगीत क परवत उसक परम क दशािता ह

- हम उसकी इलाहाबाद परशससत स महतवपणि

ऐवतहावसक जानकारी वमलती ह वजसकी रचना

उसक दरबार क कवि हररसन न की थी

Q23) उततर (b)

सपषटीकरणः

- विकरम सोित की शरआत ििि 58 ईसा पिि म

चनदरगपत वदवतीय न की थी

- यह शक ो पर उसकी जीत और उस विकरमावदतय

की पदिी वमलन क उपलकषय म आरमभ वकया गया

था

FC19H1003 32

- बानभटट न हिििििन का जीिन-ितताोत हििचररत

(ज सोसकत म थी) वलखी थी

Q24) उततर (c)

सपषटीकरणः

- सोवि-विगरावहका यद एिो शाोवत का मोतरी

- साथििाह वयापाररय ो क कावफल ो का नता

Q25) उततर (a)

सपषटीकरणः

- जआन झाोग (हसआन रताोग ndash Hsuang Tsang)

एक चीनी यातरी था ज हिििििन क शासनकाल म

भारत आया था

- ििि 630 ईसवी स ज दशक आरमभ हआ था उसम

जआन झाोग मधय एवशया ईरान और

अफग़ावनसतान की यातरा करन क पशचात कशमीर

क रासत स भारत आया था

- उसन उततर स पिि तक की यातरा की और िह

लगभग 2 ििि वबहार म रहा

- जआन झाोग न नालनदा विशवविदयालय म विदयावथिय ो

और विदवान ो क साथ पारसपररक विचार-विमशि

वकया थथानीय भािाओ ा म वनपणता परापत की तथा

बौद सतप ो की ख ज की

Q26) उततर (c)

सपषटीकरणः

- परदवकषणा पथ बौद िासतकला म सतप क चार ो

ओर बनाया जान िाला एक घमािदार पथ ह ता

- परशन म वदए गए बाकी क तीन ो ततव वहोद मसनदर ो की

िासतकला क भाग ह

Q27) उततर (d)

सपषटीकरणः

परशन म वदए गए सभी मोवदर ो म वयापक रप स

ईोि ो (पकी ईोि ो) का परय ग पतथर ो क साथ हआ

Q28) उततर (c)

सपषटीकरण

- महममद कली कतब शाह ग लकणडा का सलतान

था

- िह अकबर का समकालीन था

- सावहतय और िासतकला म उसकी अतयाविक

रवच थी

- िह एक महान कवि था

- िह दसखनी उदि फारसी और तलग म वलखता था

- उसन अपन पीछ एक विसतत वदिान (सोगरह)

छ डा ह

- अभी हाल ही म तलोगाना म ग लकणडा क वकल

क अनदर खदाई वकय गए बाग-ए-नाया वकला

बाग क चार ो ओर रप-रखा क मानवचतरण क

वलए भारतीय परातासतवक सिकषण (The

Archaeological Survey of India ndash ASI)

गराउणड पनीिर विोग रडार (Ground Penetrating

Radar) का परय ग करगा

Q29) उततर (a)

सपषटीकरणः

- वसलपपावदकारम एक तवमल महाकावय ह वजसकी

रचना इलाोग क दवारा लगभग 1800 ििि पिि की

गई थी

- यह क िलन नामक एक वयापारी की कहानी ह

ज माििी नामक एक गवणका (िशया) स परम

करन लगा था

- मवनमकलाई क िलन और माििी की पतरी की

कहानी ह

Q30) उततर (a)

सपषटीकरण

- चरक आयिद और वचवकरता की एक महतवपणि

रचना चरक सोवहता क लखक ह

- बरहमगपत क अपनी रचना बरहम-सफि-वसदानत

(ज एक खग लीय रचना ह) क कारण परवससद

वमली

FC19H1003 33

- बगदाद म इसका अनिाद अरबी भािा म वकया

गया था

- इसका इसलावमक गवणत और खग ल-विजञान पर

महतवपणि परभाि पडा था

- बाद म अपन जीिनकाल म बरहमगपत न

ldquoखोडखयाकrdquo वलखी ज एक खग लीय पससतका

(एक छ िी पसतक) थी

- इसम आयिभटट की अिि-रावतर क परतयक वदन की

शरआत परणाली का परय ग वकया गया था

Q31) उततर (c)

सपषटीकरण

- अमीर खसर एक परवसद सफी सोगीतकार कवि

और विदवान थ

- 1318 म उनह ोन पाया वक इस भवम (वहोदसतान) क

हर कषतर म अलग-अलग भािा थी लाहौरी

कशमीरी दवारसमदरी (दवकषणी कनाििक म)

तलोगाना (आोधर परदश म) गजरी (गजरात म)

माबारी (तवमलनाड म ) अििी (पिी उततर परदश

म) और वहोदिी (वदलली क आस-पास क कषतर म)

आवद

- उनह न यह बताया वक सोसकत वकसी भी कषतर स

सोबोवित नही ो थी और किल बराहमण ही इस भािा

का जञान रखत थ

Q32) उततर (c)

सपषटीकरण

- वहरणय-गभि सववणिम गभि क सोदवभित करता ह

- जब बराहमण ो की सहायता स यह अनषठान वकया

जाता था त यह माना जाता था वक बवल दन िाल

का कषवतरय क रप म पनजिनम ह गा

Q33) उततर (d)

सपषटीकरण

- कदमई भवम राजसव पर कर क सोदवभित करता

- गवावलयर परशससत म नागभि क दवारा वकय गए

श िण का िणिन वकया गया ह |

- नागभि एक परवतहार राजा था

Q34) उततर (b)

सपषटीकरण

- राजतरो वगनी 12िी ो शताबदी म कलहन क दवारा

रवचत एक सोसकत पसतक (िकसट) ह

- यह परारसमभक भारत की ऐवतहावसक इवतितत थी

- तकि सोगत रप स इस अपन परकार की सिोततम

और सिािविक विशवसनीय कवत माना जाता ह

- यह कशमीर कषतर क पराचीनतम समय स लकर

उसकी रचना की तारीख तक क समपणि इवतहास

का आचछादन करती ह

Q35) उततर (c)

सपषटीकरण

- गााि की आम सभा क ldquoउरrdquo कहा जाता था

- ldquoउरrdquo म गााि क सभी कर दन िाल वनिासी

शावमल ह त थ

Q36) उततर (a)

सपषटीकरण

- वदलली सलतनत म ldquoतारीखrdquo इवतहास लखन का

एक रप था

- ldquoतािरीखrdquo क लखक विदवान परि ह त थ वजनम

सवचि परशासक इतयावद शावमल थ

Q37) उततर (a)

सपषटीकरण

- अलाउददीन सखलजी अपन सवनक ो क ितन का

भगतान नकद म करता था न वक इकता क रप

- सवनक अपना सामान वदलली म वयापाररय ो स

खरीदत थ अतः इस बात का भय था वक वयापारी

कही ो िसतओो का मलय न बिा द

- इसकी र कथाम क वलए अलाउददीन सखलजी न

वदलली म कीमत ो क वनयसित वकया

FC19H1003 34

- अविकारीगण धयानपििक मलय ो का सिकषण करत

थ तथा ज वयापारी वनिािररत मलय पर माल नही ो

बचत थ उनक दसणडत वकया जाता था

Q38) उततर (d)

सपषटीकरण

- वदलली सििपरथम त मर राजपत ो क अिीन उनक

सामराजय की राजिानी बनी थी

- 12िी ो शताबदी क मधय म अजमर क चौहान ो

(वजनह चाहमान ो क नाम स भी जाना जाता ह) न

त मर राजपत ो क परावजत वकया था

- त मर ो और चौहान ो क अिीन वदलली एक

महतवपणि िावणसजयक क दर बन गया था

- कई जन वयापारी यहाा रहन लग थ और उनह ोन

कई मोवदर भी बनिाए

- यहाा पर मवदरत वसक वजनह ldquoदहलीिालrdquo क नाम

स जाना जाता था वयापक रप स परचलन म थ

Q39) उततर (c)

सपषटीकरण

- म ठ की मसिद का वनमािण वसको दर ल दी क

राजयकाल म उसक मिी क दवारा करिाया गया

था

- बगमपरी मसिद का वनमािण महममद तगलक क

शासनकाल म हआ था

- यह मसिद विशव का पणयथथान (The

Sanctuary of the World) और वदलली म महममद

तगलक की नई राजिानी जहाोपनाह की मखय

मसिद थी

- कववत- अल - इसलाम मसिद का विसतार

इलतसिश और अलाउददीन सखलजी न वकया था

- मीनार का वनमािण तीन सलतान ो कतबददीन ऐबक

इलतसिश और वफर ज शाह तगलक क दवारा

करिाया गया था

Q40) उततर (c)

सपषटीकरण

- मगल ो क अिीन मनसबदार शबद उस वयसकत क

वलए सोदवभित वकया जाता था वजसक पास मनसब

(अथाित पद) ह ता था

- उस अपना ितन राजसव कायो वजनह जागीर कहत

थ क रप म परापत ह ता था

Q41) उततर (b)

सपषटीकरण

- ldquoभारत छ ड आोद लनrdquo वबरविश शासन क

सखलाफ ल ग ो का एक सवाभाविक विदर ह था

- असखल भारतीय काोगरस सवमवत न 8 अगसत 1942

क बमबई म एक बठक का आय जन वकया था

- इस बठक म परवसद सोकलप ldquoभारत छ ड rdquo क

पाररत वकया गया और इस उददशय क परापत करन

क वलए गाोिी क नततव म एक अवहोसक जन सोघिि

आोद लन की शरआत का परसताि वदया गया

- लवकन अगल ही वदन गाोिी और काोगरस क अनय

परमख नताओो क वगरफतार कर वलया गया

- काोगरस क एक बार वफर अिि घ वित वकया गया

था

Q42) उततर (c)

सपषटीकरण

- साइमन कमीशन यनाइविड वको गडम क सात

साोसद ो का एक समह था

- इस वबरविश भारत क वलए सोििावनक सिार ो का

सझाि दन क वलए गवठत वकया गया था

- इस आय ग म िररषठ वबरविश राजनता सर जॉन

साइमन क नततव म किल वबरविश सदसय ही

शावमल थ

- इसवलए भारत क ल ग ो न साइमन कमीशन क

आगमन क विरद आोद लन वकया था

Q43) उततर (a)

सपषटीकरण

bull दादा भाई नौर जी भारत म वबरविश शासन क

आवथिक पररणाम ो क बार म अपनी विर िी

(परवतकल) राय क वलए जान जात थ

FC19H1003 35

bull अपन कई लख ो और भािण ो म विशि रप स

ldquoपाििी एो ड अन-वबरविश रल इन इसणडया

(Poverty and Un-British Rule in India) म

नौर जी न यह तकि वदया वक भारत पर अतयविक

कर लगाया गया था और इसकी सोपवतत इोगलड की

ओर परिावहत की जा रही थी

bull उनह ोन पराचीन भारतीय गरोथ ो की वयाखया करन

का और भारतीय ो क आिविशवास क बहाल

करन पर कायि नही ो वकया था

उनह ोन वकसी और बात स पहल सभी सामावजक

बराइय ो क उनमलन की आिशयकता पर भी बल

नही ो वदया था

Q44) उततर (c)

सपषटीकरण

bull अगसत 1932 म वबरविश परिानमोतरी मकड नालड न

अपन साोपरदावयक परसकार (The Communal

Award) की घ िणा की थी

bull यह भारत क कई साोपरदावयक वहत ो क बीच विवभनन

सोघिो क हल करन क वलए वबरिन का एकतरफा

परयास था

bull यह परसकार (Award) बाद म 1935 क

अविवनयम (The Act of 1935) म शावमल वकया

गया था

bull इस साोपरदावयक परसकार न मससलम ो क वलए

आरवकषत एक अलग वनिािचक मणडल फॉमिल का

विसतार अनय अलपसोखयक ो क वलए वकया था

वजसम वसख ो भारतीय ईसाइय ो आोगल-भारतीय

समदाय यर पीय समदाय तथा विवशषट कषतरीय

समह ो क शावमल वकया गया था

bull गाोिी न इस परसताि क भारतीय समाज क

विभावजत करन क वलए एक घवणत वबरविश

सावजश क रप म दखा और उसक सखलाफ

आमरण अनशन वकया

Q45) उततर (b)

सपषटीकरण

मौजदा आयात और वनयाित क अवतररक़त

औपवनिवशक भारत क वनमनवलसखत खचो क

वलए एक विशिवनवशचत िन रावश भी दनी पडती

थी

(i) परशासन क वयय

(ii) सना क रख-रखाि क वयय

(iii) यद क वयय

(iv) सिावनितत अविकाररय ो की पशन तथा

(v) वबरिन दवारा अपनी उपवनिश बसती

(कॉल नी) क रख-रखाि क वयय

इनह गह शलक (Home Charges) क रप म

जाना जाता था और लगभग परी तरह स भारत क

दवारा इनका भगतान वकया जाता था

bull गह शलक म वनमनवलसखत घिक शावमल थ

(i) भारतीय ऋण पर दय बयाज

(ii) ईसट इोवडया को पनी क शयरिारक ो क

लाभाोश

(iii) लोदन म भारत कायािलय चलान क वलए िन

(iv) भारत म वनयकत वबरविश कवमिय ो क ितन

और पशन का भगतान करन क वलए िन

(v) रलि पर बयाज

(vi) नागररक और सनय शलक

(vii) इोगलड म सट र (सामगरी) की खरीद

Q46) उततर (b)

सपषटीकरण

bull भारतीय राषटर ीय काोगरस का लाहौर सतर 1929 म

जिाहरलाल नहर की अधयकषता म आय वजत

वकया गया था

bull इस सतर म भारतीय राषटर ीय आोद लन स समबसित

कई महतवपणि पररणाम सामन आय थ

(i) सििपरथम इस सतर म काोगरस क अधयकष पद

पर जिाहरलाल नहर क चना गया था ज

काोगरस म िामपोवथय ो की बिती हई ताकत

का सपषट सोकत था

(ii) दसरा इस सतर म पहली बार काोगरस न पणि

सवतोतरता की माोग क उठाया था

इस परकार की माोग काोगरस मोच स पहल कभी भी

नही ो उठाई गई थी

Q47) उततर (b)

सपषटीकरण

FC19H1003 36

bull इस ररप िि न वकसी भी समदाय क वलए पथक

वनिािचक मोडल अथिा अलपसोखयक ो क वलए

भाराोश की वसफाररश नही ो की थी

bull तथावप इस ररप िि न उन पराोत ो म अलपसोखयक

सीि ो क आरकषण की अनमवत दी थी जहाा पर कम

स कम दस परवतशत अलपसोखयक ह

bull लवकन यह समदाय क आकार क अनपात म ह ना

चावहए था

bull इस ररप िि म भारत क वलए पणि सवतोतरता क

वलए क ई पराििान नही ो था

Q48) उततर (c)

सपषटीकरण

bull आरो वभक िवदक आयो का िमि मखय रप स

परकवत की पजा और यजञ था

bull परारो वभक आयि िमि परकवत की पजा क समान था

bull िासति म उनक चार ो ओर की शसकतयाा वजनह न

त ि वनयोवतरत कर सकत थ और न ही समझ पाए

थ उनह वदवयता क साथ वनिवशत वकया गया तथा

उनह मादा या नर दिीदिताओो क रप म

परतीकतव वकया गया था

bull उनह ोन कछ यजञ ो का भी वनषपादन वकया था

Q49) उततर (b)

सपषटीकरण

bull सडक और नदी-मागि (जल-मागि) डकती स

सरवकषत नही ो थ

bull उललखनीय ह वक हिििििन क शासनकाल क

दौरान यआन चिाोग (हयएन साोग) का सारा

सामान लि वलया गया था

Q50) उततर (c)

सपषटीकरण

परशन म वदए गए द न ो कथन सही ह

Q51) उततर (b)

सपषटीकरण

bull परोदर दास एक सोत और भगिान कषण क एक

महान भकत थ

bull परोदर दास क कनाििक सोगीत क वपतामह क

रप म जाना जाता ह

bull यदयवप उनक जनम-थथान क बार म काफी

अिकल लगाई जाती रही ह

bull तथावप अब कननड विशवविदयालय हमपी क दवारा

गवठत एक विशिजञ सवमवत इस वनषकिि पर पहोची

ह वक उनका जनम थथान सोभितया कनाििक का

एक छ िा-सा गााि कषमपरा (वशिम गगा वजला)

था

Q52) उततर (c)

सपषटीकरण

bull शरी तयागराज शरी शयाम शासतरी और शरी मथसवामी

दीवकषतर क कनाििक सोगीत की वतरमवति माना

जाता ह

bull उनक कारण ही 18िी ो-19िी ो शताबदी म कनाििक

सोगीत का सववणिम यग आया था

Q53) उततर (d)

सपषटीकरण

bull अभी हाल ही म लौह यगीन-महापािावणक काल

का 2000 ििि पराना एक दलिभ सारक फगस

(Sarcophagus) (पतथर का ताबत) क ललम क

वियर गाोि (क वयलडी क पास वजला क वझक ड

करल राजय) की एक रॉक-कि गफा स ख जा गया

bull यह ताबत वजसम हविय ो क िकड थ खदाई क

दौरान वमला

bull अभी तक इस परकार की दलिभ ख ज करल क

मातर द ही थथान ो स हई ह

bull य द न ो सारक फगी (Sarcophagi) (पतथर क

ताबत) चियर और अथ ली (वजला क वझक ड) क

महापािाण थथल ो स वमल ह

Q54) उततर (a)

सपषटीकरण

FC19H1003 37

दवकषण भारत म महापािाण सोसकवत एक पणि

विकवसत लौह यगीन सोसकवत थी

Q55) उततर (d)

सपषटीकरण

bull च ल पाणडय और करलपतर (चर) इन तीन ो का

उललख अश क क अवभलख ो म वकया गया ह

bull सोभितः य भौवतक सोसकवत क उततर

महापािावणक चरण म थ

Q56) उततर (d)

सपषटीकरण

bull भीमा-क रगाोि की लडाई ततीय आोगल-मराठा

यद का वहससा थी

Q57) उततर (b)

सपषटीकरण

bull राजकमार शकल न गाोिीजी क चोपारण आन तथा

वतनकवथया परणाली स जडी समसया की जाोच क

वलए रारी करन क वलए दश भर म उनका

अनसरण वकया था

bull बज वकश र राजदर परसाद महादि दसाई और

नरहरी पाररख चोपारण सतयागरह क दौरान गाोिी

जी क सहय गी थ

Q58) उततर (b)

सपषटीकरण

bull बराहमण ो और बौद मठिाररय ो क कर-मकत गााि

अनदान म दन की परथा सतिाहन ो न आरमभ की

थी

Q59) उततर (c)

सपषटीकरण

इस कायिकरम क उददशय वनमनानसार ह

(i) बवनयादी पयििन आिाररक सोरचना का विकास

करना

(ii) चयवनत (पहचान वकय गए) कषतर ो म आजीविका क

सजन क वलए दश क साोसकवतक और विरासत

मलय ो क बिािा दना

(iii) विरासत समारक थथल ो पर विशव सतरीय आिाररक

सोरचना विकवसत करक एक सतत तरीक स

पयििक आकििण म िसद करना

(iv) थथानीय समदाय ो की सवकरय भागीदारी क माधयम

स र रगार ो का सजन करना

(v) र रगार उतपादन और आवथिक विकास क वलए

पयििन कषमता का उन पर परभाि का उपय ग

करना तथा

(vi) िारणीय पयििन आिाररक सोरचना का विकास

करना और उसका उवचत सोचालन तथा

रखरखाि सवनवशचत करना

Q60) उततर (b)

सपषटीकरण

bull यह वनकाय ििि 1987 म अससततव म आया था

bull यह एक राषटर ीय सतर का शीिि सोगठन ह ज भारत

सरकार क जनजातीय मामल ो क मोतरालय क

परशासवनक वनयोतरण क अिीन काम कर रहा ह

bull इसका पोजीकत और परिान कायािलय नई वदलली

म सथथत ह

Q61) उततर (c)

सपषटीकरण

bull परमचोद क उपनयास ो म परमाशरम रोगभवम गबन

कमिभवम और ग दान शावमल ह

bull ग रा रिी ोदरनाथ िग र क दवारा रवचत उपनयास ह

bull अभी हाल ही म मोशी परमचोद की 138िी ो जयोती दश

भर म मनाई गई थी

Q62) उततर (b)

सपषटीकरण

bull ldquoवगदाrdquo पोजाब (भारत) एिो पावकसतान की

मवहलाओो क दवारा तयौहार क समय और फसल

की बिाई तथा किाई क अिसर पर वकया जान

िाला एक पारोपररक दहाती नतय ह

FC19H1003 38

bull इस नतय क माधयम स पोजाबी मवहलाऐो अपनी

परसननता परकि करती ह तथा वगदा क परदशिन क

माधयम स परि िचिसव िाल समाज म मवहलाओो

की दबी हई भािनाओो क परकि करती ह

bull चोवक इस नतय का परि ो क साथ क ई सोबोि नही ो

ह अतः किल मवहलाऐो ही इसम भाग ल सकती

bull हर साल तीज समार ह क दौरान पोजाब म वगदा

नतय वकया जाता ह

तीज भारत क कछ भाग ो म मवहलाओो क दवारा

मनाया जान िाल कई तयौहार ो क वलए एक

वयापक नाम ह

Q63) उततर (a)

सपषटीकरण

- मजम-उल-बहरीन या द समदर ो का सोगम

नामक उललखनीय रचना दारा वशक ह क दवारा

वलखी थी

- भारत क उपराषटर पवत शरी एम िकया नायड न कहा

ह वक राजकमार दारा वशक ह की रचनाएा शाोवत

और सदभाि क बिािा दन क वलए एक तारा सर त

क रप म सामन आ सकती ो ह

- उपराषटर पवत गत ििो क भला वदए गए राजकमार

दारा वशक ह क परदवशित परचवलत करन हत

आय वजत एक परदशिनी का दौरा करन क बाद एक

सभा क सोब वित कर रह थ

- इस परदशिनी का आय जन फर क इस गौवियर

(Francois Gautier) क दवारा lsquoइोवदरा गाोिी नशनल

सिर फॉर द आििसrsquo (The Indira Gandhi

National Centre for the Arts) नई वदलली म

वकया गया था

Q64) उततर (c)

सपषटीकरण

- ग मतशवर परवतमा जन भगिान बाहबली क

समवपित ह

- यह एक एक-चटटानी पतथर की मवति ह

- राषटर पवत राम नाथ क विोद न शरिणबलग ला

(कनाििक) म आय वजत वकय जान िाल भवय

अवभिक समार ह महामसतकावभिक का

उदघािन वकया था

- यह समार ह 12 ििो म एक बार ह ता ह

Q65) उततर (c)

सपषटीकरण

bull पराची घािी पराची नदी क चार ो ओर फली हई थी

bull पराची घािी िीर-िीर विलपत ह गई थी

bull पराची नदी भिनशवर स वनकलती ह

bull यह महानदी की एक सहायक नदी ह और यह

परी खदाि किक तथा जगतवसोहपर वजल ो स

ह कर बहती ह

bull इस नदी क पर कषतर क पराची घािी कहा जाता ह

bull यह नदी बोगाल की खाडी म वगरती ह

परातासतवक साकषय स पता चलता ह वक पराची घािी

सभयता हडपपा और म हनज दाड द न ो की

पिििती ह

Q66) उततर (d)

सपषटीकरण

य समारक छतरपर वजल (मधय परदश) म विोधयाचल

पिित शरोखला म सथथत ह

Q67) उततर (a)

सपषटीकरण

bull थॉिस ऑन पावकसतान नामक पसतक डॉ बी

आर अमबडकर न वलखी थी

bull डॉ बी आर अमबडकर की जयोती क अिसर पर

भारत क राषटर पवत न भारत की इस महान हसती

क शरदाोजवल अवपित की थी

bull डॉ बी आर अमबडकर न 1924 म वडपरथड

कलावसर इोसटीटयि (दवलत िगि सोथथान -

बवहषकत वहतकाररणी सभा) और 1927 म समाज

समता सोघ की थथापना की थी

bull अमबडकर का धयान वशकषा कषतर की ओर भी था

bull उनह ोन वशकषा क वनमन िगो म फलान क वलए

पीपलस एजकशन स साइिी (The Peoples

Education Society) क नाम स महाविदयालय ो क

नििकि और छातरािास ो की थथापना की थी

FC19H1003 39

Q68) उततर (b)

सपषटीकरण

bull महरगि भारतीय उपमहादवीप म एक परवसद

निपािाण बसती ह ज बलवचसतान पराोत

पावकसतान म सथथत ह

bull दचपलली (आोधर परदश) क पास नागलर नदी क

पिी ति ो पर चना पतथर क बलॉक क विशाल

विसतार म एक पिि-ऐवतहावसक रॉक आिि थथल की

ख ज की गई ह

bull इसन 1500-2000 ईसा पिि क दौरान गोिर (आोधर

परदश) म विकवसत निपािाण सभयता पर परकाश

डाला ह

Q69) उततर (c)

सपषटीकरण

bull 12िी ो सदी और 13िी ो सदी म काकाविय िोश का

उदय हआ था

bull ि पहल कलयाण क पवशचमी चालकय ो क सामोत थ

bull परारोभ म उनह ोन िारोगल (तलोगाना) क पास एक

छ ि स कषतर पर शासन वकया था

bull उनह ोन ldquoनायक वयिथथाrdquo की शरआत की थी

वजस बाद म विजयनगर क राय शासक ो न

अपनाया और विकवसत वकया था

Q70) उततर (a)

सपषटीकरण

bull गाोिीजी क अनशन स वमल मावलक ो पर दबाि

पडा था ज अोततः शरवमक ो क ितन म 35 परवतशत

की िसद करन क वलए सहमत हए थ

bull गगल (Google) न अनसया साराभाई वजनह ोन

भारत क शरवमक आोद लन म एक अगरणी भवमका

वनभाई थी की 132िी ो जयोती डडल (Doodle) का

वनमािण करक मनाई

Q71) उततर (d)

सपषटीकरण

भारत स यनसक की मानिता की अमति साोसकवतक

विरासत की परवतवनवि सची म वनमनवलसखत शावमल ह

bull कवडयटटम करल का सोसकत रोगमोच

bull मवडयिि करल का अनषठान रोगमोच और नतय

नाविका

bull िवदक मि जाप की परोपरा

bull राजथथान क कालबवलया ल क गीत और नतय

bull रामलीला रामायण का पारोपररक परदशिन

bull सोकीतिन मवणपर का अनषठान गायन ढ ल िादन

और नतय

bull रममन भारत क गििाल वहमालय का िावमिक

तयौहार और अनषठान रोगमोच

bull जाोदीयाला गर पोजाब क ठठर ो की पीतल और

ताोब क वशलप स वनवमित बतिन ो की पारोपररक कला

bull छाऊ नतय पिी भारतीय राजय ो म जनमी शासतरीय

भारतीय नतय कला

bull लददाख का बौद मि जाप िर ाोस-वहमालयी लददाख

कषतर तथा जमम-कशमीर म पवितर बौद गरोथ ो का पाठ

bull य ग

bull नौर र

bull को भ मला

Q72) उततर (b)

सपषटीकरण

bull भारत क राषटर पवत शरी राम नाथ क विोद न

वकसामा नागालड म हॉनिवबल मह रति और

राजय गठन वदिस समार ह का उदघािन वकया

था

bull हॉनिवबल मह रति का नाम भारतीय हॉनिवबल क

नाम पर पडा ह ज एक विशाल और रोगीन जोगली

पकषी ह

bull यह पकषी नागालड राजय की अविकतर जनजावतय ो

की ल ककथाओो म उसललसखत ह

bull नागालड की परमख मानयता परापत जनजावतयाा ह

अोगामी आओ चखसोग चाोग ककी रगमा और

रवलोग आवद

bull ओोग जारिा और ससिनलीस अोडमान-वनक बार

दवीप समह की जनजावतयाा ह

FC19H1003 40

Q73) उततर (c)

सपषटीकरण

bull दकन म राषटर कि शासन दसिी ो सदी क अोत तक

लगभग 200 ििो तक रहा था

bull राषटर कि शासक अपन िावमिक विचार ो म सवहषण

bull उनह ोन न किल शि िमि और िषणि िमि बसलक

जन िमि क भी सोरकषण वदया था

bull एल रा म वशि क परवसद रॉक कि मोवदर का

वनमािण नौिी ो सदी म राषटर कि राजा कषण परथम न

करिाया था

bull उसका उततराविकारी अम घििि जन था लवकन

उसन अनय िमो क भी सोरकषण परदान वकया था

bull राषटर कि ो न मसलमान वयापाररय ो क बसन की

अनमवत दी थी

bull उनह न अपन अविराजय ो म इसलाम क उपदश दन

की भी अनमवत दी थी

bull अभी हाल ही म पाोडिलागटटा (तलोगाना) क

परागवतहावसक चटटान वचतर ो क कषरण की बिती हई

घिनाएा एक गोभीर वचोता का वििय ह

bull यह परागवतहावसक चटटान क नकसान पहाचा

सकता ह

bull पाोडिलागटटा वनमनवलसखत क वलए जाना जाता ह

- 10000 ईसा पिि स 8000 ईसा पिि क वचवतरत

चटटानी आशरय ो क वलए

- राषटर कि काल क एक 8 िी ो सदी क

वशलालख क वलए और

- 12िी ो सदी क काकविय सामराजय क वभवतत

वचतर ो क वलए

Q74) उततर (b)

सपषटीकरण

bull 1828 म राजा राम म हन रॉय न एक नय िावमिक

समाज बरहम सभा की थथापना की थी वजस बाद

म बरहम समाज क नाम स जाना गया था

bull दिदरनाथ िग र न ततवब विनी सभा की अधयकषता

की थी ज आधयासिक सतय की ख ज म सोलि

थी

bull इसका उददशय वहोद िमि क शद करन का और

एकशवरिाद (एक ईशवर म आथथा) का परचार करना

था

bull नय समाज की थथापना क आिार थ कारण

(तकि ) क द सतमभ तथा िद और उपवनिद

bull अभी हाल ही म सािारण बरहम समाज का कछ

काननी मदद ो क लकर पवशचम बोगाल सरकार क

साथ काननी वििाद चल रहा ह

Q75) उततर (c)

सपषटीकरण

bull भारत म वचशती वसलवसल की थथापना खवाजा

म इनददीन वचशती क दवारा की गयी थी

bull ि 1192 ईसवी क आसपास भारत आय थ

bull वचशतीय ो क बारहिी ो शताबदी क उततरािि म भारत

म आन िाल सफीय ो क समह ो म सबस

परभािशाली माना जाता ह

bull उनह ोन थथानीय िातािरण क साथ सफलतापििक

अनकलन वकया और उनह ोन भारतीय भसकत

परोपराओो क कई पहलओो क अपनाया

bull अजमर म सफी अपरकि खवाजा म इनददीन वचशती

की ऐवतहावसक दरगाह क एक नया रप दन की

तयारी की जा रही ह

bull इस 13िी ो शताबदी की दरगाह क ldquoसवचछ

आइकॉवनक थथल ोrdquo (Swacch Iconic Places) म

शावमल वकया गया ह ज परवतवषठत विरासत

आधयासिक और साोसकवतक थथान ो पर क वदरत

य जना ह

FC19H1003 41

ANSWERS amp EXPLANATION OF

NCERT History Class VI-X + Current Affairs

(FC19E1003)

Q1) Answer c

Explanation

Rigveda consists of more than a

thousand hymns dedicated to gods and

goddesses These hymns were

composed by sages and learnt by men

however a few were composed by

women like Apala Ghosa Lopamudra

Maitreyi and Gargi

Rigveda consists of many hymns in the

form of dialogues We get an example of

a dialogue between a sage named

Vishwamitra and two rivers (Beas and

Sutlej) that were worshipped as

goddesses This suggests that he

belonged to the Vedic period

Q2) Answer b

Explanation

Traces of ash have been found from

Kurnool Caves suggesting that people

were familiar with the use of fire

It is situated in Andhra Pradesh

Q3) Answer c

Explanation

Burzahom is a prehistoric site in

present day Kashmir where people built

pit houses which were dug into the

ground with steps leading into them

These may have provided shelter in cold

weather

Q4) Answer c

Explanation

Epigraphy is defined as the study of

inscriptions

Manuscriptology is the study of history

and literature through the use of hand

written documents

Palaeography refers to the study of

ancient writing systems and the

deciphering and dating of historical

manuscripts

Numismatics refers to the study of

coins

Q5) Answer a

Explanation

Charaka Samhita was written by

Charaka and is an important book on

Ayurveda and medicine

He was a practitioner of the traditional

system of Indian medicine known as

Ayurveda

Charaka is thought to have flourished

sometime between the 2nd century BCE

and the 2nd century CE

Q6) Answer b

Explanation

Bhaga refers to the tax on crops which

was fixed at 16th of the production

Kammakaras is the term used for the

landless agricultural labour class

Ashvamedha also known as horse

sacrifice is a ritual where a horse is let

loose to wander freely and it was

guarded by the rajarsquos men

Q7) Answer (d)

Explanation

In the Rigvedic period horses were

yoked to chariots that were used in

battles fought to capture land cattle

etc This suggests that the use of horse

chariots began much before the period

of Mahajanapadas

The battles were fought in the Rigvedic

period for cattlersquos lands water an even

to capture people Most men took part

in these wars however there was no

regular army but there were assemblies

where people met and discussed

matters of war Regular armies became

a feature in the Mjahajanapada period

including vast armies of foot soldiers

chariots and elephants

RAUSIAS-FC19E1003 42

Q8) Answer (a)

Explanation

Buddha belonged to the Sakya clan and

passed away at Kusinara

Buddha taught in Prakrit which was the

common language of people

Q9) Answer c

Explanation

There were six schools of philosophy in

ancient India These are known as

Vaishesika Nyaya Samkhya Yoga

Purva Mimansa and Vedanata or Uttara

Mimansa They were founded by sages

Kanada Gautama Kapila Patanjali

Jamini and Vyasa respectively

Q10) Answer b

Explanation

The teachings of Mahavira were

compiled at Valabhi in 6th century AD

Q11) Answer (c)

Explanation

Chanakya is traditionally identified as

Kautilya or Vishnugupta who authored

the ancient Indian political treatise the

Arthashastra

Q12) Answer d

The national emblem of India is an

adaptation of the Lion Capital atop the

Ashoka Pillar of Sarnath Uttar Pradesh

and is combined with the National

Motto Satyameva Jayate

The Rampurva Bull gets the name from

the site of its discovery Rampurva in

Bihar

It is noted for its delicately sculpted

model demonstrating superior

representation of soft flesh sensitive

nostrils alert ears and strong legs It is

a mixture of Indian and Persian

elements

Sankissa is situated in Uttar Pradesh

India

Q13) Ans(a)

Kunwar Singh was a notable leader during the Revolt of 1857 He belonged

to a royal house of Jagdispur Bihar

Q14) Answer b

Explanation

The term Vellalar was used for large

landowners

Q15) Answer c

Explanation

Arikamedu was a coastal settlement

where ships unloaded goods from

distant lands Finds here include a

massive brick warehouse pottery

including amphorae and Arretine ware

Roman lamps glassware and gems have

also been found at the site

Q16) Answer a

Explanation

Muvendar is a Tamil word mentioned in

Sangam poems meaning three chiefs

used for the heads of three ruling

families the Cholas Cheras and

Pandyas

Q17) Ans (c)

Several tribal or kin-based assemblies

such as the Sabha Vidatha and gana

are mentioned in the Rig-veda The

Sabha and the samiti mattered a great

deal in early Vedic times so much so

that the chiefs or the kings showed an

eagerness to win their support

Q18) Ans (a)

Jainism recognised the existence of the

gods but placed them lower than the

jina and did not condemn the varna

system as Buddhism did

Q19) Answer (d)

Explanation

Cholas and Pandyas had developed

powerful coastal cities The most

important city of Cholas was Puhar or

Kaveripattinam and Madurai was the

capital of Pandyas

Q20) Answer b

Explanation

Buddhacharita is the biography of

Buddha and was written by

RAUSIAS-FC19E1003 43

Ashvaghosha

Q21) Answer (a)

Explanation

Tamil poet Appar was a Shiva devotee

So he was a Nayanar saint

Q22) Answer d

Explanation

Samudragupta was a prominent Gupta

ruler whose coins depict him playing a

veena indicating his love for music We

get important historic information from

his Allahabad Prashasti which was

composed by his court poet Harisena

Q23) Answer (b)

Explanation

Vikrama Samvat was founded by

Chandragupta II in the 58 BC as a

mark of victory over the Shakas and

assumed the title of Vikramaditya

Banabhatta wrote Harshavardhanarsquos

biography the Harshacharita in

Sanskrit

Q24) Answer c

Explanation

Sandhi-vigrahika was the minister of

war and peace

Sarthavaha was the leader of the

merchant caravans

Q25) Answer a

Explanation

Xuan Zang (Hsuan-tsang) was a

Chinese traveller who came during the

reign of Harshavardhana

In the decade that began in 630 AD

Xuan Zang came to India through

Kashmir after visiting Central Asia Iran

and Afghanistan

He travelled from north to east and lived

in Bihar for a couple of years

At Nalanda University Xuan Zang

interacted with students and scholars

mastered local languages and

discovered Buddhist stupas

Q26) Answer c

Explanation

Pradakshina patha is a circular path

laid around a stupa in Buddhist

architecture While the rest are a part of

temple architecture

Q27) Answer d

Explanation

All the above-mentioned temples have

an elaborate use of bricks (baked

bricks) along with stone

Q28) Ans (c)

Muhammad Quli Qutab was the Sultan

of Golconda He was a contemporary of

Akbar was very fond of literature and

architecture

The Sultan was a great poet and he

wrote in Dakhini Urdu Persian and

Telgu and has left an extensive diwan or

collection

Recently the Archaeological Survey of

India (ASI) will be using Ground

Penetrating Radar (GPR) to map the

contours of the area around the Bagh-e-

Naya Qila excavated garden inside the

Golconda Fort in Telangana

Q29) Answer a

Explanation

Silappadikaram is a famous Tamil epic

which was written by Ilango around

1800 years ago It is a story of a

merchant named Kovalan who fell in

love with a courtesan named Madhavi

Manimekalai tells the story of the

daughter of Kovalan and Madhavi

Q30) Answer (a)

Explanation

Charaka is the author of Charaka

Samhita which is an important work of

Ayurveda and medicines

Brahmaguptarsquos fame rests mostly on his

Brahma-sphuta-siddhanta which was

an astronomical work It was translated

into Arabic in Baghdad and had a major

impact on Islamic mathematics and

astronomy

Late in his life Brahmagupta wrote

Khandakhadyaka which was an

RAUSIAS-FC19E1003 44

astronomical handbook that employed

Aryabhatarsquos system of starting each day

at midnight

Q31) Answer (c)

Explanation

Amir Khusrau was a famous sufi

musician poet and scholar In 1318 he

noted that there was different language

in every region of this land (Hindustan)

Lahori Kashmiri Dvarsamudri (in

Southern Karnataka) Telangana (in

Andhra Pradesh) Gujari (in Gujarat)

Marsquobari (in Tamil Nadu) Awadhi (in

eastern Uttar Pradesh) and Hindawai (in

the area around in Delhi) etc He went

to explain that Sanskrit did not belong

to any region and that only brahmans

knew it

Q32) Answer c

Explanation

Hiranyagarbha refers to the golden

womb When this ritual was performed

with the help of Brahmanas it was

thought to lead to the rebirth of the

sacrificer as a Khastriya

Q33) Answer d

Explanation

Kadamai refers to a tax on land

revenue

Gwalior Prashasti describes the exploits

of Nagabhata who was a Pratihara king

Q34) Answer b

Explanation

Rajatarangini is a Sanskrit text written

by Kalhana in the 12th century

It was historical chronicle of early India

It is justifiably considered to be the best

and most authentic work of its kind

It covers the entire span of history in

the Kashmir region from the earliest

times to the date of its composition

Q35) Answer c

Explanation

ldquoUrrdquo was the general assembly of the

village ldquoUrrdquo consisted of all the

taxpaying residents of an ordinary

village

Q36) Answer (a)

Explanation

Tarikh was a form of history writing in

the Delhi Sultanate The authors of

tawarikhs were learned men which

included secretaries administrators etc

Q37 Answer (a)

Explanation

Alauddin chose to pay his soldiers salaries in cash rather than iqtas The soldiers would buy their supplies from merchants in Delhi and it was thus feared that merchants would raise their prices To stop this Alauddin controlled the prices of goods in Delhi Prices were carefully surveyed by officers and merchants who did not sell at the prescribed rates were punished

Q38) Answer (d)

Explanation

Delhi first became the capital of a

kingdom under the Tomara Rajputs

who were defeated in the middle of the

twelfth century by the Chauhans (also

referred to as Chahamanas) of Ajmer

It was under the Tomaras and

Chauhans that Delhi became an

important commercial centre Many rich

Jaina merchants lived in the city and

constructed several temples Coins

minted here called dehliwal had a wide

circulation

Q39) Answer (c)

Explanation

Moth ki Masjid was built in the reign of

Sikandar Lodi by his minister

Begumpuri mosque built in the reign of

Muhammad Tughluq was the main

mosque of Jahanpanah the ldquoSanctuary

of the Worldrdquo and his new capital in

Delhi

Quwwat al ndash Islam mosque was

enlarged by Iltutmish and Alauddin

Khalji The minar was built by three

Sultansndash Qutbuddin Aybak Iltutmish

and Firuz Shah Tughluq

RAUSIAS-FC19E1003 45

Q40) Answer (c)

Explanation

Under the Mughals mansabdar was

referred to an individual who held a

mansab ie rank and he received his

salary as revenue assignments called

jagirs

Q41) Ans (b)

The Quit India Movement was a

spontaneous revolt of people against

British rule

The All India Congress Committee met

at Bombay on 8 August 1942 It passed

the famous resolution Quit India and

proposed the starting of a non-violent

mass struggle under Gandhis

leadership to achieve this aim But on

the very next day Gandhi and other

eminent leaders of the Congress were

arrested The Congress was once again

declared illegal

Q42) Ans (c)

The Simon Commission refers to a

group of seven MPs from the United

Kingdom constituted to suggest

constitutional reforms for British India

The Commission consisted of only

British members headed by one of the

senior British politicians Sir John

Simon

So the people of India agitated against

the arrival of Simon Commission

Q43) Ans (a)

He was widely known for his

unfavourable opinion of the economic

consequences of the British rule in

India

In his many writings and speeches and

especially in Poverty and Un-British

Rule in India Naoroji argued that India

was too highly taxed and that its wealth

was being drained away to England

He did not interpret the ancient Indian

texts and restored the self-confidence of

Indians And also he did not stress the

need for eradication of all the social

evils before anything else

Q44) Ans (c)

In August 1932 Prime Minister

MacDonald announced his Communal

Award Great Britainrsquos unilateral

attempt to resolve the various conflicts

among Indiarsquos many communal

interests

The award which was later

incorporated into the act of 1935

expanded the separate-electorate

formula reserved for Muslims to other

minorities including Sikhs Indian

Christians Anglo-Indians Europeans

distinct regional groups Gandhi

undertook a ldquofast unto deathrdquo against

that offer which he viewed as a

nefarious British plot to divide the

Indian society

Q45) Ans (b)

In British India apart from existing

imports and exports there was also a

particular amount of money which

colonial India contributed towards

administration maintenance of the

army war expenses pensions to retired

officers and other expenses accrued by

Britain towards maintenance of her

colony These were known as Home

charges and were paid for almost

entirely by India

The Home charges was made of

following components-

- Interest payable on Indian debt

- Dividend to shareholders of East

India Company

- Funds used to support the India

Office in London

- Funds used to pay salaries and

pensions of British personnel

engaged in India

- Interest on the railways

- Civil and military charges

- Store purchases in England

Q46) Ans (b)

The Lahore session of the Indian

National Congress was held in 1929

under the Presidentship of Jawaharlal

Nehru

The Lahore session of the Indian

National Congress witnessed significant

RAUSIAS-FC19E1003 46

developments in the Indian national

movement

- First the election of Jawaharlal

Nehru to the post of Presidentship of

the Congress was a clear indication

of the growing strength of the

Leftists in the Congress

- Secondly it was in this session that

the Congress for the first time raised

the demand for complete

independence Such demand was

not raised from the Congress

platform earlier

Q47) Ans (b)

It did not provide for separate

electorates for any community or

weightage for minorities However it did

allow for the reservation of minority

seats in provinces having minorities of

at least ten per cent but this was to be

in strict proportion to the size of the

community

There was no provision for complete

Independence for India

Q48) Ans (c)

The religion of early Vedic Aryans was

primarily of worship of nature and

Yajnas

The early Aryan religion was kind of

nature worship Actually the forces

around them which they could not

control or understand were invested

with divinity and were personified as

male or female gods And they

performed some Yajnas also

Q49) Ans (b)

The roads and river-routes were not

immune from robbery It is notable that

Yuan Chwang (Hiuen Tsang) was

robbed of his belongings during

Harshvardanarsquos period

Q50) Ans (c)

Q51) Ans (b)

Purandara Dasa was a saint and great

devotee of Lord Krishna

There is much speculation about where

Purandara Dasa regarded as the

Pitamaha of Carnatic music was born

Recently an expert committee

constituted by the Kannada University

Hampi has come to the conclusion that

Kshemapura Shivamogga district

Karnataka is the birth place of

Purandara Dasa

Q52) Ans (c)

Sri Tyagaraja Sri Shyama Shastry and Sri Muthuswami Dikshitar are considered the trinity of Carnatic music and with them came the golden age in Carnatic music in the 18th-19th

century

Q53) Ans d)

Recently a rare sarcophagus (stone

coffin) which is 2000 years old from the

Iron AgendashMegalithic era was discovered

from a rock-cut cave at Viyur village of

Kollam near Koyilandy in Kozhikode

district Kerala

The coffin containing bone fragments

was found during an excavation ldquoSo

far such a rare finding has been

discovered only from two sites

in Kerala Both these sarcophagi were

recovered from Megalithic sites at

Chevayur and Atholi also in Kozhikode

district

Q54) Ans a)

The megalithic culture in South India was a full-fledged Iron Age culture

Q55) Ans d)

The Cholas Pandyas and Keralaputras

(Cheras) mentioned in Ashokan

inscriptions were probably in the late

megalithic phase of material culture

Q56) Ans d)

Q57) Ans (b)

Raj Kumar Shukla followed Gandhiji all

over the country to persuade him to

come to Champaran to investigate the

problem associated with tinkathia

system

RAUSIAS-FC19E1003 47

Brij Kishore Rajendra Prasad Mahadev

Desai and Narhari Parikh accompanied

Gandhi ji during the Champaran

Satyagraha

Q58) Ans (b)

The Satvahanas started the practice of granting tax-free villages to brahmanas and Buddhist monks

Q59) Ans c)

The objectives of the Programme are

listed as under

- Developing basic tourism

infrastructure

- Promoting cultural and heritage

value of the country to generate

livelihoods in the identified regions

- Enhancing the tourist attractiveness

in a sustainable manner by

developing world-class

infrastructure at the heritage

monument sites

- Creating employment through active

involvement of local communities

- Harnessing tourism potential for its

effects on employment generation

and economic development

- Developing sustainable tourism

infrastructure and ensuring proper

Operations and maintenance

therein

Q60) Ans (b)

The Tribal Cooperative Marketing

Development Federation of India

(TRIFED) came into existence in 1987

It is a national-level apex organization

functioning under the administrative

control of Ministry of Tribal Affairs

Govt of India

TRIFED has its registered and Head

Office located in New Delhi

Q61) Ans (c)

Premchandrsquos novels include

Premashram Rangabhumi Ghaban

Karmabhumi and Godan

Gora is a novel written by Rabindranath

Tagore

138th birth anniversary of Munshi

Premchand was celebrated across the

country

Q62) Ans (b)

Giddha is a traditional pastoral dance

performed by the women of the Punjab

India and Pakistan at festival times

and at the sowing and reaping of the

harvest

By this dance the Punjabi women

reveal their joy expel their suppressed

feelings in a male dominated society

through the performance of Giddha

Since this dance has nothing to do with

men only women can participate in it

During the Teej celebrations Giddha

dance is celebrated in Punjab every

year Teej is a generic name for a

number of festivals that are celebrated

by women in some parts of India

Q63) Ans (a)

Dara Shukoh wrote the remarkable

work called ldquoMajma-ul-Bahrainrdquo or the

ldquoThe confluence of two seasrdquo

The Vice President of India Shri M

Venkaiah Naidu has said that Prince

Dara Shukohrsquos writings can come as a

refreshing source for infusing peace and

harmony He was addressing the

gathering after visiting the exhibition

that showcases the forgotten Prince of

yesteryears Dara Shukoh organized by

Mr Francois Gautier at Indira Gandhi

National Centre for the Arts in New

Delhi

Q64) Ans (c)

The statue Gommateshwara is

dedicated to the Jain God Bahubali

It is a monolithic statue

President Ram Nath Kovind

inaugurated the grand anointing

ceremony mdash Mahamastakabhisheka mdash

held once in 12 years at

Shravanabelagola (Karnataka)

Q65) Ans (c)

Prachi Valley had come up around the

Prachi river Prachi Valley gradually

disappeared

RAUSIAS-FC19E1003 48

The Prachi river originates from

Bhubaneswar

It is a tributary of the Mahanadi and

flows through the districts of Puri

Khurda Cuttack and Jagatsinghpur

and the entire region of the river is

termed as the Prachi Valley

It falls into the Bay of Bengal

Archaeological evidence shows that the

Prachi Valley Civilisation predates both

Harappa and Mohenjo-Daro

The Prachi river originates from

Bhubaneswar

Q66) Ans (d)

These monuments are located in

Chhatarpur district Madhya Pradesh

within Vindhya mountain range

Q67) Ans (a)

The book lsquoThoughts on Pakistanrsquo was

written by Dr BR Ambedkar

On the occasion of the birth anniversary

of Dr BR Ambedkar the president of

India pays homage to this icon of India

In 1924 he founded the Depressed

Classes Institute (Bahishkrit Hitkarini

Sabha) and in 1927 the Samaj Samata

Sangh

Another area of attention for Ambedkar

was education For its spread among

the low classes he set up a network of

colleges by the name of Peoples

Education Society and founded hostels

Q68) Ans(b)

Mehrgarh is a famous Neolithic

settlement in the Indian subcontinent

which is situated in Baluchistan

province Pakistan

A pre-historic rock art site is discovered

in the vast expanse of limestone blocks

on the eastern banks of Naguleru river

near Dachepalli (Andhra Pradesh) It

has thrown light on the Neolithic

civilisation that flourished in Guntur

(Andhra Pradesh) during 1500-2000

BC

Q69) Ans (c)

The 12th and the 13th centuries saw

the emergence of the Kakatiyas They

were at first the feudatories of the

Western Chalukyas of Kalyana Initially

they ruled over a small territory near

Warangal (Telangana)

They introduced Nayakships which was

later adopted and developed by the

Rayas of Vijayanagara

Q70) Ans (a)

The fast had effect of putting pressure

on mill owners who finally agreed to

give the workers a 35 per cent increase

in wages

Google celebrated with a doodle the

132nd birth anniversary of Anasuya

Sarabhai who played a pioneering role

in Indiarsquos labour movement

Q71) Ans (d)

The UNESCOrsquos list of the representative

list of the intangible cultural heritage of

humanity from India are

- Koodiyattam Sanskrit Theatre of

Kerala

- Mudiyettu ritual theatre and dance

drama of Kerala

- Tradition of Vedic Chanting

- Kalbelia folk songs and dances of

Rajasthan

- Ramlila Traditional Performance of

the Ramayana

- Sankirtana ritual singing

drumming and dancing of Manipur

- Ramman religious festival and

ritual theatre of the Garhwal

Himalayas India

- Traditional brass and copper craft of

utensil making among the Thatheras

of Jandiala Guru Punjab India

- Chhau dance classical Indian dance

originated in the eastern Indian

states

- Buddhist chanting of Ladakh

recitation of sacred Buddhist texts

in the trans-Himalayan Ladakh

region Jammu and Kashmir India

- Yoga

- Nouroz

- Kumbh Mela

RAUSIAS-FC19E1003 49

Q72) Ans(b)

The President of India Shri Ram Nath Kovind inaugurated the Hornbill Festival and State Formation Day celebrations of Nagaland in Kisama

The festival is named after the Indian hornbill the large and colourful forest bird which is displayed in the folklore of most of the states tribes

The major recognized tribes of Nagaland are Angami Ao Chakhesang Chang

Kuki Rengma and Zeling etc

Onge Jarawa and Sentinelese are the

tribes of Andman amp Nicobar Islands

Q73) Ans (c)

The Rashtrakutas rule in the Deccan lasted for almost two hundred years till the end of the tenth century The Rashtrakutas rulers were tolerant in their religious views and patronized not only Shaivism and Vaishnavism but

Jainism as well

The famous rock-cut temple of Shiva at Ellora was built by one of the Rashtrakutas kings Krishna I in the ninth century His successor Amoghavarsha was a Jain but he also

patronized other faiths

The Rashtrakutas allowed Muslims traders to settle and permitted Islam to

be preached in their dominions

Recently increasing defacement at the prehistoric rock paintings of Pandavulagutta Telangana has created a cause for grave concern It can spoil

the prehistoric rock

Pandavulagutta is home to

- Painted rock shelters dating to

10000 BC-8000 BC

- An 8th century inscription of the

Rashtrakuta period and

- Painted frescoes from the 12th century Kakatiya empire

Q74) Ans (b)

In 1828 Raja Ram Mohan Roy founded a new religious society the Brahma Sabha later known as the Brahmo

Samaj

Debendranath Tagore headed the Tattvabodhini Sabha which was

engaged in search of spiritual truth

Its purpose was to purify Hinduism and to preach monotheism or belief in one God

The new society was to be based on the twin pillars of reason and the Vedas and

Upanishads

Recently Sadharan Brahmo Samaj (SBS) has entered into a legal battle with the West Bengal government due

to some legal issue

Q75) Ans (c)

The Chishti order was established in India by Khwaja Moinuddin Chishti who came to India around 1192 The Chishtirsquos are considered to be the most influential of the groups of Sufis who migrated to India in the late twelfth century They adapted successfully to the local environment and adopted several features of Indian devotional

traditions

The historical dargah of Sufi mystic Khwaja Moinuddin Chishti in Ajmer is all set to get a facelift This 13 th century dargah has been included among the Swachh Iconic Places a clean-up initiative focused on iconic

heritage spiritual and cultural places

Page 18: GENERAL STUDIES (PAPER I) · Test is part of Rau’s IAS Test series for Preliminary Exam 2019 FOUNDATION + CURRENT AFFAIRS GENERAL STUDIES (PAPER –I) FOUNDATION TEST –III TOPIC:

RAUSIAS-FC19E1003 18

Q49) भारत की यातरा करि िाि चीिी यातरी यआि चिाोग

(हयएि साोग) ि समकािीि भारत की सामानय

खसथनतय ो और सोसकनत क दजण नकया था इस सोदभण म

निमननिखित कथि ो म स कौि-सास सही हह

1 सड़क और िदी-मागण (जि-मागण) डकती स

परण रप स सरनकषत थ

2 जहा तक अपरार ो क निए दणड की बात ह

उसक निए नकसी भी वयखकत की निदोिता

अथिा उसक अपरार क निराणररत करि क

निए अनि जि और निि परि क माधयम क

सारि थ

3 वयापाररय ो क घाट ो और परनतबोर सटशि ो पर

शलक ो का भगताि करिा पड़ता था

िीच नदए गए कट का परय ग कर सही उततर चनिए

(a) किि 1

(b) किि 2 और 3

(c) किि 1 और 3

(d) 1 2 और 3

Q50) नसोर घाटी सभयता क सोदभण म निमननिखित कथि ो पर

निचार कीनजए

1 यह मखय रप स एक रमणनिरपकष सभयता थी

तथा हािाोनक इसम रानमणक ततव मौजद था

िनकि िह परनतिश पर हािी िही ो था

2 इस काि क दौराि भारत म कपास का परय ग

कपड़ा बिाि क निए नकया जाता था

उपयणकत कथि ो म स कौि-सास सही हह

(a) किि 1

(b) किि 2

(c) 1 और 2 द ि ो

(d) ि त 1 ि ही 2

Q51) परोदर दास क सोदभण म निमननिखित कथि ो पर निचार

कीनजए

1 परोदर दास एक सोत और भगिाि नशि क एक

महाि भकत थ

2 ि एक सोगीतकार गायक और किाणटक सोगीत

क मखय सोसथापक-परसतािक ो म स एक थ

उपयणकत कथि ो म स कौि-सास सही हह

(a) किि 1

(b) किि 2

(c) 1 और 2 द ि ो

(d) ि त 1 ि ही 2

Q52) निमननिखित म स कौि-सास वयखकत किाणटक सोगीत

की नतरमनतण म शानमि हह

1 बािामरिी कषणा

2 शरी शयाम शासतरी

3 शरी मथसवामी दीनकषतर

िीच नदए गए कट का परय ग कर सही उततर चनिए

(a) किि 1

(b) किि 2

(c) किि 2 और 3

(d) 1 2 और 3

Q53) चियर (Chevayur) और अथ िी (Atholi) म खसथत

महापािार सथि निमननिखित म स नकस राजय म खसथत

(a) तनमििाड

(b) किाणटक

(c) पनिम बोगाि

(d) करि

RAUSIAS-FC19E1003 19

Q49) The Chinese traveller Yuan Chwang

(Hiuen Tsang) who visited India

recorded the general conditions and

culture of India at that time In this

context which of the following

statements isare correct

1 The roads and river-routes were

completely immune from robbery

2 As regards punishment for

offences ordeals by fire water and

poison were the instruments for

determining the innocence or guilt

of a person

3 The tradesmen had to pay duties

at ferries and barrier stations

Select the correct answer using the code

given below

(a) 1 only

(b) 2 and 3 only

(c) 1 and 3 only

(d) 1 2 and 3

Q50) Regarding the Indus Valley Civilization

consider the following statements

1 It was predominantly a secular

civilization and the religious

element though present did not

dominate the scene

2 During this period cotton was

used for manufacturing textiles in

India

Which of the statements given above

isare correct

(a) 1 only

(b) 2 only

(c) Both 1 and 2

(d) Neither 1 nor 2

Q51) Consider the following statements

regarding Purandara Dasa

1 Purandara Dasa was a saint and

great devotee of Lord Shiva

2 He was a composer singer and

one of the chief founding-

proponents of the Carnatic music

Which of the statements given above

isare correct

(a) 1 only

(b) 2 only

(c) Both 1 and 2

(d) Neither 1 nor 2

Q52) Which of the following persons isare

included in the trinity of Carnatic

music

1 Balamurali Krishna

2 Sri Shyama Shastry

3 Sri Muthuswami Dikshitar

Select the correct answer using the code

given below

(a) 1 only

(b) 2 only

(c) 2 and 3 only

(d) 1 2 and 3

Q53) Megalithic sites at Chevayur and Atholi

are located in which of the following

states

(a) Tamil Nadu

(b) Karnataka

(c) West Bengal

(d) Kerala

RAUSIAS-FC19E1003 20

Q54) निमननिखित कथि ो पर निचार कीनजए

1 महापािानरक ि ग कबर ो म िसतएो दफिात थ

2 दनकषर भारत म महापािार सोसकनत एक परण

निकनसत तामर यगीि सोसकनत थी

उपयणकत कथि ो म स कौि-सास सही हह

(a) किि 1

(b) किि 2

(c) 1 और 2 द ि ो

(d) ि त 1 ि ही 2

Q55) निमननिखित म स कौि-स सामराजयसामराजय ो का

अश क क अनभिि ो म उललि नकया गया ह

1 च ि

2 पाणडय

3 करिपतर (चर)

िीच नदए गए कट का परय ग कर सही उततर चनिए

(a) किि 1

(b) किि 1 और 2

(c) किि 3

(d) 1 2 और 3

Q56) भीमा-क रगाोि का यदध को पिी क सनिक ो और

बाजीराि नदवतीय क िततव म एक शखकतशािी पशिा

सिा (मराठ ो) क मधय िड़ा गया था यह यदध

निमननिखित म स नकसका नहससा था

(a) परथम आोगल-मराठा यदध का

(b) नदवतीय आोगल-मराठा यदध का

(c) ततीय आोगल-मसर यदध का

(d) ततीय आोगल-मराठा यदध का

Q57) निमननिखित कथि ो पर निचार कीनजए

1 महादि दसाई ि गाोरीजी क चोपारर आि तथा

नतिकनथया पररािी स जड़ी समसया की जाोच

क निए रारी करि क निए दश भर म उिका

अिसरर नकया था

2 िरहरी पाररि चोपारर सतयागरह क दौराि

गाोरीजी क साथ थ

उपयणकत कथि ो म स कौि-सास सही हह

(a) किि 1

(b) किि 2

(c) 1 और 2 द ि ो

(d) ि त 1 ि ही 2

Q58) निमननिखित कथि ो पर निचार कीनजए

1 िनद राज-िोश ि बराहमर ो और बौदध मठराररय ो

क कर-मकत गाि अिदाि म दि की परथा

आरि की थी

2 सतिाहि ो की आनरकाररक भािा पराकत थी

उपयणकत कथि ो म स कौि-सास सही हह

(a) किि 1

(b) किि 2

(c) 1 और 2 द ि ो

(d) ि त 1 ि ही 2

Q59) एक निरासत क अपिाइए (अडॉपट ए हररटज ndash

Adopt a Heritage) पररय जिा क उददशय ो क

सनदभण म निमननिखित कथि ो पर निचार कीनजए

1 यह पररय जिा र रगार उतपादि और आनथणक

निकास क निए पयणटि कषमता का उि पर

परभाि का उपय ग करगी

2 यह पररय जिा निरासत सथि ो पर निशव सतरीय

आराररक सोरचिा निकनसत करक एक सतत

तरीक स पयणटक आकिणर म िखदध करगी

उपयणकत कथि ो म स कौि-सास सही हह

(a) किि 1

(b) किि 2

(c) 1 और 2 द ि ो

(d) ि त 1 ि ही 2

RAUSIAS-FC19E1003 21

Q54) Consider the following statements

1 Megalithic people buried goods in

graves

2 The megalithic culture in South

India was a full-fledged Copper

Age culture

Which of the statements given above

isare correct

(a) 1 only

(b) 2 only

(c) Both 1 and 2

(d) Neither 1 nor 2

Q55) Which of the following kingdoms isare

mentioned in the Ashokan inscriptions

1 Cholas

2 Pandyas

3 Keralaputras (Cheras)

Select the correct answer using the code

given below

(a) 1 only

(b) 1 and 2 only

(c) 3 only

(d) 1 2 and 3

Q56) The Battle of Bhima-Koregaon was

fought between the soldiers of the

Company and the strong Peshwa army

(Marathas) under Bajirao II This war

was a part of the

(a) First Anglo-Maratha war

(b) Second Anglo-Maratha war

(c) Third Anglo- Mysore war

(d) Third Anglo-Maratha war

Q57) Consider the following statements

1 Mahadev Desai followed Gandhiji all over the country to persuade him to come to Champaran to investigate the problem associated

with tinkathia system

2 Narhari Parikh accompanied Gandhi ji during the Champaran

Satyagraha

Which of the statements given above isare correct

(a) 1 only

(b) 2 only

(c) Both 1 and 2

(d) Neither 1 nor 2

Q58) Consider the following statements

1 The Nanda Dynasty started the practice of granting tax-free villages to brahmanas and

Buddhist monks

2 The official language of the Satavahanas was Prakrit

Which of the statements given above

isare correct

(a) 1 only

(b) 2 only

(c) Both 1 and 2

(d) Neither 1 nor 2

Q59) Consider the following statements about the objectives of the lsquoadopt a heritagersquo

project

1 It will harness tourism potential for its effects on employment generation and economic

development

2 It will enhance the tourist attractiveness in a sustainable manner by developing world class infrastructure at heritage sites

Which of the statements given above

isare correct

(a) 1 only

(b) 2 only

(c) Both 1 and 2

(d) Neither 1 nor 2

RAUSIAS-FC19E1003 22

Q60) ldquoभारतीय जिजातीय सहकारी निपरि निकास सोघrdquo

(The Tribal Co-operative Marketing

Development Federation of India - TRIFED)

क सोदभण म निमननिखित कथि ो पर निचार कीनजए

1 यह एक राषटर ीय सतर का शीिण सोगठि ह ज

भारत सरकार क गह मोतरािय क परशासनिक

नियोतरर क अरीि काम कर रहा ह

2 इसका मखय उददशय दश म जिजातीय ि ग ो

का सामानजक-आनथणक निकास करिा ह

उपयणकत कथि ो म स कौि-सास सही हह

(a) किि 1

(b) किि 2

(c) 1 और 2 द ि ो

(d) ि त 1 ि ही 2

Q61) निमननिखित म स कौि-सास उपनयास परमचोद क

दवारा नििा गया हनिि गए ह

1 रोगभनम

2 ग दाि

3 ग रा

िीच नदए गए कट का परय ग कर सही उततर चनिए

(a) किि 1

(b) किि 2

(c) किि 1 और 2

(d) 1 2 और 3

Q62) नगदधा ितय क सोदभण म निमननिखित कथि ो पर निचार

कीनजए

1 नगदधा नबहार की मनहिाओो क दवारा तयौहार क

समय और फसि की बिाई तथा कटाई क

अिसर पर नकया जाि िािा एक पारोपररक

दहाती ितय ह

2 इस ितय क दवारा मनहिाऐो अपिी परसननता

परकट करती ह तथा नगदधा क परदशणि क

माधयम स परि िचणसव िाि समाज म

मनहिाओो की दबी हई भाििाओो क परकट

करती ह

उपयणकत कथि ो म स कौि-सास सही हह

(a) किि 1

(b) किि 2

(c) 1 और 2 द ि ो

(d) ि त 1 ि ही 2

Q63) निमननिखित कथि ो पर निचार कीनजए

1 मलला शाह बदखशी दारा नशक ह क

आधयाखतमक गर थ

2 औरोगरब ि मजम-उि-बहरीि या द समदर ो

का सोगम िामक उललििीय रचिा नििी थी

3 दारा नशक ह क अपि पिणज अकबर क गर ो

क उततरानरकारी क रप म दिा गया था

नजसम उसि रानमणक बहििाद और समनवयता

क बढ़ािा नदया था

उपयणकत कथि ो म स कौि-सास सही हह

(a) किि 1 और 3

(b) किि 2

(c) किि 1 और 2

(d) 1 2 और 3

RAUSIAS-FC19E1003 23

Q60) Consider the following statements about

the Tribal Cooperative Marketing

Development Federation of India

(TRIFED)

1 It is a national-level apex

organization functioning under the

administrative control of Ministry

of Home Affairs Government of

India

2 The main objective of TRIFED is

socio-economic development of

tribal people in the country

Which of the statements given above

isare correct

(a) 1 only

(b) 2 only

(c) Both 1 and 2

(d) Neither 1 nor 2

Q61) Which of the following novels isare

written by Premchand

1 Rangabhumi

2 Godan

3 Gora

Select the correct answer using the code

given below

(a) 1 only

(b) 2 only

(c) 1 and 2 only

(d) 1 2 and 3

Q62) Consider the following statements about

Giddha dance

1 Giddha is a traditional pastoral

dance performed by the women of

Bihar at festival times and at the

sowing and reaping of the harvest

2 By this dance the women reveal

their joy expel their suppressed

feelings in a male dominated

society through the performance of

Giddha

Which of the statements given above

isare correct

(a) 1 only

(b) 2 only

(c) Both 1 and 2

(d) Neither 1 nor 2

Q63) Consider the following statements

1 Mullah Shah Badakhshi was the

spiritual mentor of Dara Shukoh

2 Aurangzeb wrote the remarkable

work called ldquoMajma-ul-Bahrainrdquo or

the ldquoThe confluence of two seasrdquo

3 Dara Shukoh was seen as

inheriting the qualities of his

ancestor Akbar in that he

promoted religious pluralism and

syncretism

Which of the statements given above

isare correct

(a) 1 and 3 only

(b) 2 only

(c) 1 and 2 only

(d) 1 2 and 3

RAUSIAS-FC19E1003 24

Q64) निमननिखित कथि ो पर निचार कीनजए

1 ग मतशवर परनतमा निोधयनगरी पहाड़ी पर खसथत ह

2 शरिरबिग िा िह सथाि ह जहाो मौयण िोश क

सोसथापक चोदरगपत मौयण अपि नसोहासि क

तयागि क बाद जि तपसवी बि गए थ

उपयणकत कथि ो म स कौि-सास सही हह

(a) किि 1

(b) किि 2

(c) 1 और 2 द ि ो

(d) ि त 1 ि ही 2

Q65) निमननिखित कथि ो पर निचार कीनजए

1 पराताखतवक साकषय स पता चिता ह नक पराची

घाटी सभयता हड़पपा और म हिज दाड़ द ि ो

की पिणिती ह

2 पराची िदी भििशवर स निकिती ह

उपयणकत कथि ो म स कौि-सास सही हह

(a) किि 1

(b) किि 2

(c) 1 और 2 द ि ो

(d) ि त 1 ि ही 2

Q66) निमननिखित कथि ो म स कौि-सास सही हह

1 िजराह क समारक ो क समह का निमाणर

चोदि राजिोश क शासिकाि क दौराि हआ

था

2 य समारक हररिोदर पिणत शरोििा म खसथत ह

3 म रक क यातरी इबन बतता ि अपि सोसमरर ो

म िजराह क मोनदर ो की यातरा का उललि

नकया था तथा इन काजराण िाम स समब नरत

नकया था

िीच नदए गए कट का परय ग कर सही उततर चनिए

(a) किि 1

(b) किि 1 और 2

(c) किि 2 और 3

(d) किि 1 और 3

Q67) निमननिखित कथि ो म स कौि-सास सही हह

1 डॉ बी आर अमबडकर ि दी एनिनहिशि

ऑफ़ कासट (The Annihilation of Caste)

नििी थी नजसम उन ोि नहोद रमण म िोशािगत

पजारी की परथा क उनमिि की आिशयकता

पर बि नदया था

2 डॉ राजदर परसाद ि थॉटस ऑि पानकसताि

(Thoughts on Pakistan) िामक पसतक

नििी थी

िीच नदए गए कट का परय ग कर सही उततर चनिए

(a) किि 1

(b) किि 2

(c) 1 और 2 द ि ो

(d) ि त 1 ि ही 2

Q68) निमननिखित कथि ो म स कौि-सास सही हह

1 महरगढ़ भारतीय उपमहादवीप म एक परनसदध

ििपािार बसती ह ज नसोर पराोत पानकसताि म

खसथत ह

2 बरणह म म कतत ो क उिक सवामी क साथ कबर ो

म दफिाया जाता था

िीच नदए गए कट का परय ग कर सही उततर चनिए

(a) किि 1

(b) किि 2

(c) 1 और 2 द ि ो

(d) ि त 1 ि ही 2

Q69) निमननिखित कथि ो म स कौि-सास सही हह

1 काकानटय मोनदर अनरकतर नशि क समनपणत

2 हिमक ोडा म हजार-सतोभ िाि मोनदर (The

Thousand-Pillared Temple) का निमाणर

काकानटय समराट रदर ि करिाया था

िीच नदए गए कट का परय ग कर सही उततर चनिए

(a) किि 1

(b) किि 2

(c) 1 और 2 द ि ो

(d) ि त 1 ि ही 2

RAUSIAS-FC19E1003 25

Q64) Consider the following statements

1 Gommateshwara Statue is located

on the Vindyagiri Hill

2 Shravanabelagola is the place

where Chandragupta Maurya the

founder of the Mauryan dynasty

became a Jain ascetic after

relinquishing his throne

Which of the statements given above

isare correct

(a) 1 only

(b) 2 only

(c) Both 1 and 2

(d) Neither 1 nor 2

Q65) Consider the following statements

1 Archaeological evidence shows

that the Prachi Valley Civilisation

predates both Harappa and

Mohenjo-Daro

2 The Prachi river originates from

Bhubaneswar

Which of the statements given above

isare correct

(a) 1 only

(b) 2 only

(c) Both 1 and 2

(d) Neither 1 nor 2

Q66) Which of the following statements

isare correct

1 The Khajuraho group of

monuments was built during the

rule of the Chandela dynasty

2 These monuments are located in

Harischandra mountain range

3 Ibn Battuta the Moroccan

traveller in his memoirs mentioned

visiting Khajuraho temples and

called them Kajarra

Select the correct answer using the code

given below

(a) 1 only

(b) 1 and 2

(c) 2 and 3

(d) 1 and 3

Q67) Which of the following statements

isare correct

1 Dr BR Ambedkar wrote the

Annihilation of Caste emphasising

the need to do away with the

practice of hereditary priesthood in

Hinduism

2 The book lsquoThoughts on Pakistanrsquo

was written by Dr Rajendra

Prasad

Select the correct answer using the code

given below

(a) 1 only

(b) 2 only

(c) Both 1 and 2

(d) Neither 1 nor 2

Q68) Which of the following statements

isare correct

1 Mehrgarh is a famous Neolithic

settlement in the Indian

subcontinent which is situated in

Sindh province Pakistan

2 At Burzahom dogs were buried

with their masters in their graves

Select the correct answer using the code

given below

(a) 1 only

(b) 2 only

(c) Both 1 and 2

(d) Neither 1 nor 2

Q69) Which of the following statements

isare correct

1 The Kakatiya temples are

dedicated mostly to Siva

2 The Thousand-Pillared Temple at

Hanamkonda was built by the

Kakatiya king Rudra

Select the correct answer using the code

given below

(a) 1 only

(b) 2 only

(c) Both 1 and 2

(d) Neither 1 nor 2

RAUSIAS-FC19E1003 26

Q70) निमननिखित कथि ो म स कौि-सास सही हह

1 अहमदाबाद नमि हड़ताि क दौराि महातमा

गाोरी ि शरनमक ो क पकष क मजबत करि क

निए आमरर अिशि नकया था

2 अिशि स नमि मानिक ो पर दबाि पड़ा था ज

अोततः शरनमक ो क िति म 15 परनतशत की िखदध

करि क निए सहमत हए थ

िीच नदए गए कट का परय ग कर सही उततर चनिए

(a) किि 1

(b) किि 2

(c) 1 और 2 द ि ो

(d) ि त 1 ि ही 2

Q71) निमननिखित म स नकसक नकिक भारत स यिसक

की माििता की अमतण साोसकनतक निरासत की

परनतनिनर सची (The UNESCOrsquos List of the

Representative List of the Intangible

Cultural Heritage of Humanity) म शानमि

नकया गया ह

1 मनडयटट

2 सोकीतणि

3 को भ मिा

िीच नदए गए कट का परय ग कर सही उततर चनिए

(a) किि 1 और 2

(b) किि 2 और 3

(c) किि 3

(d) 1 2 और 3

Q72) निमननिखित जिजानतय ो म स कौि-सीसी ो

जिजानतजिजानतया िागािड स सोबोनरत हह

1 अोगामी

2 ककी

3 जारिा

िीच नदए गए कट का परय ग कर सही उततर चनिए

(a) किि 1

(b) किि 1 औऔ 2

(c) किि 2

(d) 1 2 और 3

Q73) निमननिखित कथि ो म स कौि-सास सही हह

1 राषटर कट सामराजय की सथापिा दोनतदगण ि की थी

नजसि मानयाित म अपिी राजरािी की

सथापिा की थी

2 राषटर कट समराट अम घििण एक ििक था और

उस कनिताओो पर पहिी कननड़ पसतक नििि

का शरय नदया जाता ह

िीच नदए गए कट का परय ग कर सही उततर चनिए

(a) किि 1

(b) किि 2

(c) 1 और 2 द ि ो

(d) ि त 1 ि ही 2

Q74) निमननिखित कथि ो म स कौि-सास सही हह

1 कशब चोदर सि ि ततवब नरिी सभा की

अधयकषता की थी ज आधयाखतमक सतय की

ि ज म सोिि थी

2 बरहम समाज ि मािि गररमा पर बि नदया

मनतणपजा का निर र नकया और सती परथा जसी

सामानजक बराइय ो की आि चिा की

िीच नदए गए कट का परय ग कर सही उततर चनिए

(a) किि 1

(b) किि 2

(c) 1 और 2 द ि ो

(d) ि त 1 ि ही 2

Q75) निमननिखित कथि ो म स कौि-सास सही हह

1 भारत म नचशती नसिनसिा खवाजा म इिददीि

नचशती क दवारा सथानपत नकया गया था

2 नचशती परोपरा की एक परमि निशिता

आतमसोयम थी नजसम साोसाररक म ह स दरी

बिाए रििा शानमि था

िीच नदए गए कट का परय ग कर सही उततर चनिए

(a) किि 1

(b) किि 2

(c) 1 और 2 द ि ो

(d) ि त 1 ि ही 2

RAUSIAS-FC19E1003 27

Q70) Which of the following statements

isare correct

1 During the Ahmedabad Mill Strike

Mahatma Gandhi undertook a fast

unto death to strengthen the

workersrsquo resolve

2 The fast had effect of putting

pressure on mill owners who

finally agreed to give the workers a

15 per cent increase in wages

Select the correct answer using the code

given below

(a) 1 only

(b) 2 only

(c) Both 1 and 2

(d) Neither 1 nor 2

Q71) Which of the following are included in

the UNESCOrsquos list of the representative

list of the intangible cultural heritage of

humanity from India

1 Mudiyettu

2 Sankirtana

3 Kumbh Mela

Select the correct answer using the code

given below

(a) 1 and 2 only

(b) 2 and 3 only

(c) 3 only

(d) 1 2 and 3

Q72) Which of the following tribes isare

related to Nagaland

1 Angami

2 Kuki

3 Jarawa

Select the correct answer using the code

given below

(a) 1 only

(b) 1 and 2 only

(c) 2 only

(d) 1 2 and 3

Q73) Which of the following statements

isare correct

1 Rashtrakuta kingdom was founded by Dantidurga who established his capital at Manyakhet

2 Amoghavarsha a Rashtrakuta king was an author and is credited with writing the first

Kannada book on poetics

Select the correct answer using the code given below

(a) 1 only

(b) 2 only

(c) Both 1 and 2

(d) Neither 1 nor 2

Q74) Which of the following statements isare correct

1 Keshab Chandra Sen headed the Tattvabodhini Sabha which was engaged in search of spiritual truth

2 The Brahmo Samaj laid emphasis on human dignity opposed idolatry and criticized such social

evils as the practice of Sati

Select the correct answer using the code given below

(a) 1 only

(b) 2 only

(c) Both 1 and 2

(d) Neither 1 nor 2

Q75) Which of the following statements isare correct

1 The Chishti order was established in India by Khwaja Moinuddin

Chishti

2 A major feature of the Chishti tradition was austerity including maintaining a distance from the

worldly power

Select the correct answer using the code

given below

(a) 1 only

(b) 2 only

(c) Both 1 and 2

(d) Neither 1 nor 2

T e s t i s p a r t o f R a u rsquo s I A S T e s t s e r i e s f o r P r e l i m i n a r y E x a m 2 0 1 9

FOUNDATION + CURRENT AFFAIRS

GENERAL STUDIES (PAPER ndashI)

FOUNDATION TEST ndashIII

SUBJECT NCERT History Class VI-X + Current Affairs

Time Allowed 1frac12 Hours Maximum Marks 150

I NSTRUCT IONS

1 IMMEDIATELY AFTER THE COMMENCEMENT OF THE EXAMINATION YOU SHOULD CHECK

THAT THIS TEST BOOKLET DOES NOT HAVE ANY UNPRINTED OR TORN or MISSING PAGES OR

ITEMS ETC IF SO GET IT REPLACED BY A COMPLETE TEST BOOKLET

2 This Test Booklet contains 75 items (questions) Each item is printed both in Hindi and English

Each item comprises four responses (answers) You will select the response which you want to mark

on the Answer Sheet In case you feel that there is more than one correct response mark the

response which you consider the best In any case choose ONLY ONE response for each item

3 You have to mark all your responses ONLY on the separate Answer Sheet (OMR sheet) provided

Read the directions in the Answer Sheet

4 All items carry equal marks

5 Before you proceed to mark in the Answer Sheet the response to various items in the Test booklet

you have to fill in some particulars in the Answer Sheet as per instructions contained therein

6 After you have completed filling in all your responses on the Answer Sheet and the examination has

concluded you should hand over to the Invigilator only the Answer Sheet You are permitted to

take away with you the Test Booklet

7 Penalty for wrong answers

THERE WILL BE PENALTY FOR WRONG ANSWERS MARKED BY A CANDIDATE IN THE

OBJECTIVE TYPE QUESTION PAPERS

(i) There are four alternatives for the answer to every question For each question for which a

wrong answer has been given by the candidate one-third of the marks assigned to that

question will be deducted as penalty

(ii) If a candidate gives more than one answer it will be treated as a wrong answer even if one of

the given answers happens to be correct and there will be same penalty as above to that

question

(iii) If a question is left blank ie no answer is given by the candidate there will be no penalty for

that question

T h i s t e s t i s p a r t o f R a u rsquo s I A S T e s t s e r i e s f o r P r e l i m i n a r y E x a m 2 0 1 9

Test Code

FC19E1003

FC19H1003 29

Answers and Explanations of

NCERT History Class VI-X + Current Affairs (FC19E1003)

Q1) उततर (c)

सपषटीकरण

- ऋगवद म दविय ो और दिताओो क समवपित एक

हजार स अविक सत तर (शल क) ह

- य शल क ऋविय ो क दवारा रच गए थ और परि ो

दवारा सीख जात थ

- हालाोवक कछ शल क मवहलाओो (जस वक अपाला

घ सा ल पामदरा मतरयी और गागी) क दवारा भी रच

गए थ

- ऋगवद म सोिाद क रप म कई शल क मौजद ह

- हम विशवावमतर नामक एक ऋवि और दविय ो क

रप म पजी जान िाली द नवदय ो (वयास और

सतलज) क बीच िाताि का उदाहरण वमलता ह

- इसस पता चलता ह वक विशवावमतर िवदक काल स

सोबोवित थ

Q2) उततर (b)

सपषटीकरण

- करनल गफाओो स राख क अिशि परापत हए ह

ज इस ओर सोकत करत ह वक ततकालीन ल ग

अवि क उपय ग स पररवचत थ

- य गफाएो आोधर परदश म सथथत ह

Q3) उततर (c)

सपषटीकरण

bull बरािह म ितिमान कशमीर म सथथत एक

परागवतहावसक थथल ह जहाो ल ग गडढ क घर ो का

वनमािण करत थ

bull य घर जमीन क ख द कर बनाए जात थ तथा नीच

जान क वलए सीवियाा ह ती थी

bull ऐसा अनमान लगाया जाता ह वक य घर ठो ड क

मौसम म आशरय परदान करत थ

Q4) उततर (c)

सपषटीकरण

bull परालख-विदया (Epigraphy) क वशलालख ो क

अधययन क रप म पररभावित वकया जाता ह

bull हसतवलसखत दसतािज ो क माधयम स इवतहास

और सावहतय क अधययन क पाोडवलवप विजञान

(Manuscriptology) कहत ह

bull पराचीन लखन परणावलय ो क अधययन और

ऐवतहावसक पाोडवलवपय ो क समझन तथा वतवथ

वनिािरण क पलीओगराफी (Palaeography) कहा

जाता ह

bull नयवमजमविकस (Numismatics) वसक ो क

अधययन क सोदवभित करता ह

Q5) उततर (a)

सपषटीकरण

- चरक सोवहता चरक क दवारा वलखी गई आयिद

और िदयक-शासर पर एक महतवपणि पसतक ह

- ि भारतीय िदयक-शासर की पारमपररक परणाली

वजस आयिद क नाम स जाना जाता ह क

अभयासकताि थ

- ऐसा माना जाता ह वक चरक का विकास दसरी

शताबदी (ईसा पिि) और दसरी शताबदी (ईसवी) क

मधय हआ था

Q6) उततर (b)

सपषटीकरण

- भाग फसल ो पर वलए जान िाल कर क सोदवभित

करता ह ज कल फसल उतपादन का 16 िाो भाग

था

- ldquoकममकारrdquo शबद भवमहीन कवि शरवमक िगि क

वलए परय ग वकया जाता था

- ldquoअशवमिrdquo (वजस घ ड क बवलदान क रप म भी

जाना जाता ह) एक अनषठान ह ता था वजसम एक

घ ड क सवतोतर रप स घमन क वलए छ ड वदया

FC19H1003 30

जाता ह और राजा क सवनक उसकी रखिाली

करत थ

Q7) उततर (d)

सपषटीकरण

- ऋगववदक काल म घ ड ो क रथ ो म ज ता जाता था

ज (रथ) भवम मिवशय ो आवद पर कबजा करन क

वलए लड गए यद ो म उपय ग वकए जात थ

- इसस यह पता चलता ह वक घ ड ो यकत रथ ो का

उपय ग महाजनपद काल स काफी पहल आरमभ

हआ था

- ऋगववदक काल म मिवशय ो भवम जल आवद पर

कबजा करन क वलए तथा ल ग ो क पकडन क

वलए यद वकय जात थ

- अविकाोश परि इन यद ो म भाग वलया करत थ

- हालाोवक उस समय क ई वनयवमत सना नही ो ह ती

थी लवकन उस काल म सभाऐो ह ती थी ो वजनम

ल ग यद क मामल ो पर चचाि करत थ

- वनयवमत सनाएा महाजनपद काल का िवशषटय थी

वजनम पदल सवनक ो की विशाल सनाएा रथ तथा

हाथी शावमल ह त थ

Q8) उततर (a)

सपषटीकरण

- बद शाकय कल स सोबोवित थ और कशीनारा म

उनका वनिन हआ था

- बद न अपनी वशकषाएा पराकत भािा म दी थी ो ज

आम ल ग ो की भािा थी

Q9) उततर (c)

सपषटीकरण

- पराचीन भारत म दशिनशासर की छह शाखाएा थी ो

िशविक नयाय समखया य ग पिि वममाोसा और

िदाोत या उततर वममाोसा

- इनकी थथापना करमश कनाद गौतम कवपल

पतोजवल जावमनी और वयास ऋविय ो न की थी

Q10) उततर (b)

सपषटीकरण

महािीर की वशकषाऐो छठी शताबदी म िललभी म

सोकवलत की गई थी ो

Q11) उततर (c)

सपषटीकरण

- पारमपररक रप स चाणकय क कौविलय अथिा

विषणगपत क नाम स जाना जाता ह

- उसन अथिशासतर ज एक पराचीन भारतीय

राजनवतक आलख ह वलखा था

Q12) उततर (d)

सपषटीकरण

- भारत का राषटर ीय वचनह सारनाथ (उततर परदश) क

अश क सतमभ क ऊपर (शीिि पर) वसोह कवपिल

का एक अनरपण ह

- इस राषटर ीय वसदाोत सतयमि जयत क साथ

सोय वजत वकया गया ह

- रामपिि बल का नाम रामपिि (वबहार) क नाम पर

पडा जहाा इसकी ख ज हई थी

- यह अपन नाजक नकाशी मॉडल क वलए परवसदद

ह वजसम क मल तवचा सोिदनशील नथन ो सतकि

कान और मरबत िााग ो क शरषठतर परवतरप क

परदवशित वकया गया ह

- यह भारतीय और फारसी ततव ो का एक ससममशरण

- सोवकससा उततर परदश म सथथत ह

Q13) उततर (a)

सपषटीकरण

का िर वसोह ज एक महान य दा थ वबहार स

सोबोवित थ

Q14) उततर (b)

सपषटीकरण

िललालर शबद बड भ-सवावमय ो क वलए परय ग

वकया जाता था

FC19H1003 31

Q15) उततर (c)

सपषटीकरण

- अररकमड एक तिीय बसती थी जहाो दर दश ो स

आन िाल जहाज ो का माल उतारा जाता था

- यहाो पर ईोि ो का एक विशाल ग दाम वमटटी क

बतिन (वजनम एमफ रा - द हरी मवठय ो का लोबा

घडा - शावमल ह) और एरिाइन (Arretine)

मदभाोड पाए गए थ

- इस थथान पर र मन दीपक काोच क बन पातर और

रतन भी पाए गए थ

Q16) उततर (a)

सपषटीकरण

- मिनदर सोगम कविताओो म उसललसखत एक

तवमल शबद ह वजसका अथि ह ldquoतीन परमखrdquo

- यह तीन सततारि पररिार ो क मसखयाओो क वलए

परय ग वकया जाता ह च ल चर और पाणडय

Q17) उततर (c)

सपषटीकरण

- ऋग िद म सभा विदाथा तथा गण जसी

जनजावतय ो पर अथिा किोब पर आिाररत

सभाओो का उललख ह

- आरसमभक िवदक काल म सभाओो और सवमवतय ो

का विशि महतव ह ता था

- यहाा तक की मसखया अथिा राजा भी उनका

समथिन परापत करन क वलए आतर रहत थ

Q18) उततर (a)

सपषटीकरण

- जन िमि न ईशवर क अससततव क मानयता त दी ह

वकनत उसन ईशवर क वजना क पद स नीच रखा

- जन िमि न बौद िमि की तरह िणि परणाली की

भरतिना नही ो की थी

Q19) उततर (d)

सपषटीकरण

- च ल ो और पाणडय ो न शसकतशाली तिीय शहर ो का

विकास वकया था

- च ल ो का सबस महतवपणि शहर पहार (या

कािरीपटटीनम) था |

- मदरई पाणडय ो की राजिानी थी

Q20) उततर (b)

सपषटीकरण

- ldquoबदचररतrdquo बद का जीिन-ितताोत ह

- इस अशवघ ि क दवारा वलखा गया था

Q21) उततर (a)

सपषटीकरणः

- तवमल कवि अपपर भगिान वशि क भकत थ

- इस परकार ि एक नयनार सोत थ

Q22) उततर (d)

सपषटीकरणः

- समदरगपत एक परवसद गपत शासक था

- उसन वसक ो पर िीणा बजात हए अपनी छवि

अोवकत करिाई थी

- यह सोगीत क परवत उसक परम क दशािता ह

- हम उसकी इलाहाबाद परशससत स महतवपणि

ऐवतहावसक जानकारी वमलती ह वजसकी रचना

उसक दरबार क कवि हररसन न की थी

Q23) उततर (b)

सपषटीकरणः

- विकरम सोित की शरआत ििि 58 ईसा पिि म

चनदरगपत वदवतीय न की थी

- यह शक ो पर उसकी जीत और उस विकरमावदतय

की पदिी वमलन क उपलकषय म आरमभ वकया गया

था

FC19H1003 32

- बानभटट न हिििििन का जीिन-ितताोत हििचररत

(ज सोसकत म थी) वलखी थी

Q24) उततर (c)

सपषटीकरणः

- सोवि-विगरावहका यद एिो शाोवत का मोतरी

- साथििाह वयापाररय ो क कावफल ो का नता

Q25) उततर (a)

सपषटीकरणः

- जआन झाोग (हसआन रताोग ndash Hsuang Tsang)

एक चीनी यातरी था ज हिििििन क शासनकाल म

भारत आया था

- ििि 630 ईसवी स ज दशक आरमभ हआ था उसम

जआन झाोग मधय एवशया ईरान और

अफग़ावनसतान की यातरा करन क पशचात कशमीर

क रासत स भारत आया था

- उसन उततर स पिि तक की यातरा की और िह

लगभग 2 ििि वबहार म रहा

- जआन झाोग न नालनदा विशवविदयालय म विदयावथिय ो

और विदवान ो क साथ पारसपररक विचार-विमशि

वकया थथानीय भािाओ ा म वनपणता परापत की तथा

बौद सतप ो की ख ज की

Q26) उततर (c)

सपषटीकरणः

- परदवकषणा पथ बौद िासतकला म सतप क चार ो

ओर बनाया जान िाला एक घमािदार पथ ह ता

- परशन म वदए गए बाकी क तीन ो ततव वहोद मसनदर ो की

िासतकला क भाग ह

Q27) उततर (d)

सपषटीकरणः

परशन म वदए गए सभी मोवदर ो म वयापक रप स

ईोि ो (पकी ईोि ो) का परय ग पतथर ो क साथ हआ

Q28) उततर (c)

सपषटीकरण

- महममद कली कतब शाह ग लकणडा का सलतान

था

- िह अकबर का समकालीन था

- सावहतय और िासतकला म उसकी अतयाविक

रवच थी

- िह एक महान कवि था

- िह दसखनी उदि फारसी और तलग म वलखता था

- उसन अपन पीछ एक विसतत वदिान (सोगरह)

छ डा ह

- अभी हाल ही म तलोगाना म ग लकणडा क वकल

क अनदर खदाई वकय गए बाग-ए-नाया वकला

बाग क चार ो ओर रप-रखा क मानवचतरण क

वलए भारतीय परातासतवक सिकषण (The

Archaeological Survey of India ndash ASI)

गराउणड पनीिर विोग रडार (Ground Penetrating

Radar) का परय ग करगा

Q29) उततर (a)

सपषटीकरणः

- वसलपपावदकारम एक तवमल महाकावय ह वजसकी

रचना इलाोग क दवारा लगभग 1800 ििि पिि की

गई थी

- यह क िलन नामक एक वयापारी की कहानी ह

ज माििी नामक एक गवणका (िशया) स परम

करन लगा था

- मवनमकलाई क िलन और माििी की पतरी की

कहानी ह

Q30) उततर (a)

सपषटीकरण

- चरक आयिद और वचवकरता की एक महतवपणि

रचना चरक सोवहता क लखक ह

- बरहमगपत क अपनी रचना बरहम-सफि-वसदानत

(ज एक खग लीय रचना ह) क कारण परवससद

वमली

FC19H1003 33

- बगदाद म इसका अनिाद अरबी भािा म वकया

गया था

- इसका इसलावमक गवणत और खग ल-विजञान पर

महतवपणि परभाि पडा था

- बाद म अपन जीिनकाल म बरहमगपत न

ldquoखोडखयाकrdquo वलखी ज एक खग लीय पससतका

(एक छ िी पसतक) थी

- इसम आयिभटट की अिि-रावतर क परतयक वदन की

शरआत परणाली का परय ग वकया गया था

Q31) उततर (c)

सपषटीकरण

- अमीर खसर एक परवसद सफी सोगीतकार कवि

और विदवान थ

- 1318 म उनह ोन पाया वक इस भवम (वहोदसतान) क

हर कषतर म अलग-अलग भािा थी लाहौरी

कशमीरी दवारसमदरी (दवकषणी कनाििक म)

तलोगाना (आोधर परदश म) गजरी (गजरात म)

माबारी (तवमलनाड म ) अििी (पिी उततर परदश

म) और वहोदिी (वदलली क आस-पास क कषतर म)

आवद

- उनह न यह बताया वक सोसकत वकसी भी कषतर स

सोबोवित नही ो थी और किल बराहमण ही इस भािा

का जञान रखत थ

Q32) उततर (c)

सपषटीकरण

- वहरणय-गभि सववणिम गभि क सोदवभित करता ह

- जब बराहमण ो की सहायता स यह अनषठान वकया

जाता था त यह माना जाता था वक बवल दन िाल

का कषवतरय क रप म पनजिनम ह गा

Q33) उततर (d)

सपषटीकरण

- कदमई भवम राजसव पर कर क सोदवभित करता

- गवावलयर परशससत म नागभि क दवारा वकय गए

श िण का िणिन वकया गया ह |

- नागभि एक परवतहार राजा था

Q34) उततर (b)

सपषटीकरण

- राजतरो वगनी 12िी ो शताबदी म कलहन क दवारा

रवचत एक सोसकत पसतक (िकसट) ह

- यह परारसमभक भारत की ऐवतहावसक इवतितत थी

- तकि सोगत रप स इस अपन परकार की सिोततम

और सिािविक विशवसनीय कवत माना जाता ह

- यह कशमीर कषतर क पराचीनतम समय स लकर

उसकी रचना की तारीख तक क समपणि इवतहास

का आचछादन करती ह

Q35) उततर (c)

सपषटीकरण

- गााि की आम सभा क ldquoउरrdquo कहा जाता था

- ldquoउरrdquo म गााि क सभी कर दन िाल वनिासी

शावमल ह त थ

Q36) उततर (a)

सपषटीकरण

- वदलली सलतनत म ldquoतारीखrdquo इवतहास लखन का

एक रप था

- ldquoतािरीखrdquo क लखक विदवान परि ह त थ वजनम

सवचि परशासक इतयावद शावमल थ

Q37) उततर (a)

सपषटीकरण

- अलाउददीन सखलजी अपन सवनक ो क ितन का

भगतान नकद म करता था न वक इकता क रप

- सवनक अपना सामान वदलली म वयापाररय ो स

खरीदत थ अतः इस बात का भय था वक वयापारी

कही ो िसतओो का मलय न बिा द

- इसकी र कथाम क वलए अलाउददीन सखलजी न

वदलली म कीमत ो क वनयसित वकया

FC19H1003 34

- अविकारीगण धयानपििक मलय ो का सिकषण करत

थ तथा ज वयापारी वनिािररत मलय पर माल नही ो

बचत थ उनक दसणडत वकया जाता था

Q38) उततर (d)

सपषटीकरण

- वदलली सििपरथम त मर राजपत ो क अिीन उनक

सामराजय की राजिानी बनी थी

- 12िी ो शताबदी क मधय म अजमर क चौहान ो

(वजनह चाहमान ो क नाम स भी जाना जाता ह) न

त मर राजपत ो क परावजत वकया था

- त मर ो और चौहान ो क अिीन वदलली एक

महतवपणि िावणसजयक क दर बन गया था

- कई जन वयापारी यहाा रहन लग थ और उनह ोन

कई मोवदर भी बनिाए

- यहाा पर मवदरत वसक वजनह ldquoदहलीिालrdquo क नाम

स जाना जाता था वयापक रप स परचलन म थ

Q39) उततर (c)

सपषटीकरण

- म ठ की मसिद का वनमािण वसको दर ल दी क

राजयकाल म उसक मिी क दवारा करिाया गया

था

- बगमपरी मसिद का वनमािण महममद तगलक क

शासनकाल म हआ था

- यह मसिद विशव का पणयथथान (The

Sanctuary of the World) और वदलली म महममद

तगलक की नई राजिानी जहाोपनाह की मखय

मसिद थी

- कववत- अल - इसलाम मसिद का विसतार

इलतसिश और अलाउददीन सखलजी न वकया था

- मीनार का वनमािण तीन सलतान ो कतबददीन ऐबक

इलतसिश और वफर ज शाह तगलक क दवारा

करिाया गया था

Q40) उततर (c)

सपषटीकरण

- मगल ो क अिीन मनसबदार शबद उस वयसकत क

वलए सोदवभित वकया जाता था वजसक पास मनसब

(अथाित पद) ह ता था

- उस अपना ितन राजसव कायो वजनह जागीर कहत

थ क रप म परापत ह ता था

Q41) उततर (b)

सपषटीकरण

- ldquoभारत छ ड आोद लनrdquo वबरविश शासन क

सखलाफ ल ग ो का एक सवाभाविक विदर ह था

- असखल भारतीय काोगरस सवमवत न 8 अगसत 1942

क बमबई म एक बठक का आय जन वकया था

- इस बठक म परवसद सोकलप ldquoभारत छ ड rdquo क

पाररत वकया गया और इस उददशय क परापत करन

क वलए गाोिी क नततव म एक अवहोसक जन सोघिि

आोद लन की शरआत का परसताि वदया गया

- लवकन अगल ही वदन गाोिी और काोगरस क अनय

परमख नताओो क वगरफतार कर वलया गया

- काोगरस क एक बार वफर अिि घ वित वकया गया

था

Q42) उततर (c)

सपषटीकरण

- साइमन कमीशन यनाइविड वको गडम क सात

साोसद ो का एक समह था

- इस वबरविश भारत क वलए सोििावनक सिार ो का

सझाि दन क वलए गवठत वकया गया था

- इस आय ग म िररषठ वबरविश राजनता सर जॉन

साइमन क नततव म किल वबरविश सदसय ही

शावमल थ

- इसवलए भारत क ल ग ो न साइमन कमीशन क

आगमन क विरद आोद लन वकया था

Q43) उततर (a)

सपषटीकरण

bull दादा भाई नौर जी भारत म वबरविश शासन क

आवथिक पररणाम ो क बार म अपनी विर िी

(परवतकल) राय क वलए जान जात थ

FC19H1003 35

bull अपन कई लख ो और भािण ो म विशि रप स

ldquoपाििी एो ड अन-वबरविश रल इन इसणडया

(Poverty and Un-British Rule in India) म

नौर जी न यह तकि वदया वक भारत पर अतयविक

कर लगाया गया था और इसकी सोपवतत इोगलड की

ओर परिावहत की जा रही थी

bull उनह ोन पराचीन भारतीय गरोथ ो की वयाखया करन

का और भारतीय ो क आिविशवास क बहाल

करन पर कायि नही ो वकया था

उनह ोन वकसी और बात स पहल सभी सामावजक

बराइय ो क उनमलन की आिशयकता पर भी बल

नही ो वदया था

Q44) उततर (c)

सपषटीकरण

bull अगसत 1932 म वबरविश परिानमोतरी मकड नालड न

अपन साोपरदावयक परसकार (The Communal

Award) की घ िणा की थी

bull यह भारत क कई साोपरदावयक वहत ो क बीच विवभनन

सोघिो क हल करन क वलए वबरिन का एकतरफा

परयास था

bull यह परसकार (Award) बाद म 1935 क

अविवनयम (The Act of 1935) म शावमल वकया

गया था

bull इस साोपरदावयक परसकार न मससलम ो क वलए

आरवकषत एक अलग वनिािचक मणडल फॉमिल का

विसतार अनय अलपसोखयक ो क वलए वकया था

वजसम वसख ो भारतीय ईसाइय ो आोगल-भारतीय

समदाय यर पीय समदाय तथा विवशषट कषतरीय

समह ो क शावमल वकया गया था

bull गाोिी न इस परसताि क भारतीय समाज क

विभावजत करन क वलए एक घवणत वबरविश

सावजश क रप म दखा और उसक सखलाफ

आमरण अनशन वकया

Q45) उततर (b)

सपषटीकरण

मौजदा आयात और वनयाित क अवतररक़त

औपवनिवशक भारत क वनमनवलसखत खचो क

वलए एक विशिवनवशचत िन रावश भी दनी पडती

थी

(i) परशासन क वयय

(ii) सना क रख-रखाि क वयय

(iii) यद क वयय

(iv) सिावनितत अविकाररय ो की पशन तथा

(v) वबरिन दवारा अपनी उपवनिश बसती

(कॉल नी) क रख-रखाि क वयय

इनह गह शलक (Home Charges) क रप म

जाना जाता था और लगभग परी तरह स भारत क

दवारा इनका भगतान वकया जाता था

bull गह शलक म वनमनवलसखत घिक शावमल थ

(i) भारतीय ऋण पर दय बयाज

(ii) ईसट इोवडया को पनी क शयरिारक ो क

लाभाोश

(iii) लोदन म भारत कायािलय चलान क वलए िन

(iv) भारत म वनयकत वबरविश कवमिय ो क ितन

और पशन का भगतान करन क वलए िन

(v) रलि पर बयाज

(vi) नागररक और सनय शलक

(vii) इोगलड म सट र (सामगरी) की खरीद

Q46) उततर (b)

सपषटीकरण

bull भारतीय राषटर ीय काोगरस का लाहौर सतर 1929 म

जिाहरलाल नहर की अधयकषता म आय वजत

वकया गया था

bull इस सतर म भारतीय राषटर ीय आोद लन स समबसित

कई महतवपणि पररणाम सामन आय थ

(i) सििपरथम इस सतर म काोगरस क अधयकष पद

पर जिाहरलाल नहर क चना गया था ज

काोगरस म िामपोवथय ो की बिती हई ताकत

का सपषट सोकत था

(ii) दसरा इस सतर म पहली बार काोगरस न पणि

सवतोतरता की माोग क उठाया था

इस परकार की माोग काोगरस मोच स पहल कभी भी

नही ो उठाई गई थी

Q47) उततर (b)

सपषटीकरण

FC19H1003 36

bull इस ररप िि न वकसी भी समदाय क वलए पथक

वनिािचक मोडल अथिा अलपसोखयक ो क वलए

भाराोश की वसफाररश नही ो की थी

bull तथावप इस ररप िि न उन पराोत ो म अलपसोखयक

सीि ो क आरकषण की अनमवत दी थी जहाा पर कम

स कम दस परवतशत अलपसोखयक ह

bull लवकन यह समदाय क आकार क अनपात म ह ना

चावहए था

bull इस ररप िि म भारत क वलए पणि सवतोतरता क

वलए क ई पराििान नही ो था

Q48) उततर (c)

सपषटीकरण

bull आरो वभक िवदक आयो का िमि मखय रप स

परकवत की पजा और यजञ था

bull परारो वभक आयि िमि परकवत की पजा क समान था

bull िासति म उनक चार ो ओर की शसकतयाा वजनह न

त ि वनयोवतरत कर सकत थ और न ही समझ पाए

थ उनह वदवयता क साथ वनिवशत वकया गया तथा

उनह मादा या नर दिीदिताओो क रप म

परतीकतव वकया गया था

bull उनह ोन कछ यजञ ो का भी वनषपादन वकया था

Q49) उततर (b)

सपषटीकरण

bull सडक और नदी-मागि (जल-मागि) डकती स

सरवकषत नही ो थ

bull उललखनीय ह वक हिििििन क शासनकाल क

दौरान यआन चिाोग (हयएन साोग) का सारा

सामान लि वलया गया था

Q50) उततर (c)

सपषटीकरण

परशन म वदए गए द न ो कथन सही ह

Q51) उततर (b)

सपषटीकरण

bull परोदर दास एक सोत और भगिान कषण क एक

महान भकत थ

bull परोदर दास क कनाििक सोगीत क वपतामह क

रप म जाना जाता ह

bull यदयवप उनक जनम-थथान क बार म काफी

अिकल लगाई जाती रही ह

bull तथावप अब कननड विशवविदयालय हमपी क दवारा

गवठत एक विशिजञ सवमवत इस वनषकिि पर पहोची

ह वक उनका जनम थथान सोभितया कनाििक का

एक छ िा-सा गााि कषमपरा (वशिम गगा वजला)

था

Q52) उततर (c)

सपषटीकरण

bull शरी तयागराज शरी शयाम शासतरी और शरी मथसवामी

दीवकषतर क कनाििक सोगीत की वतरमवति माना

जाता ह

bull उनक कारण ही 18िी ो-19िी ो शताबदी म कनाििक

सोगीत का सववणिम यग आया था

Q53) उततर (d)

सपषटीकरण

bull अभी हाल ही म लौह यगीन-महापािावणक काल

का 2000 ििि पराना एक दलिभ सारक फगस

(Sarcophagus) (पतथर का ताबत) क ललम क

वियर गाोि (क वयलडी क पास वजला क वझक ड

करल राजय) की एक रॉक-कि गफा स ख जा गया

bull यह ताबत वजसम हविय ो क िकड थ खदाई क

दौरान वमला

bull अभी तक इस परकार की दलिभ ख ज करल क

मातर द ही थथान ो स हई ह

bull य द न ो सारक फगी (Sarcophagi) (पतथर क

ताबत) चियर और अथ ली (वजला क वझक ड) क

महापािाण थथल ो स वमल ह

Q54) उततर (a)

सपषटीकरण

FC19H1003 37

दवकषण भारत म महापािाण सोसकवत एक पणि

विकवसत लौह यगीन सोसकवत थी

Q55) उततर (d)

सपषटीकरण

bull च ल पाणडय और करलपतर (चर) इन तीन ो का

उललख अश क क अवभलख ो म वकया गया ह

bull सोभितः य भौवतक सोसकवत क उततर

महापािावणक चरण म थ

Q56) उततर (d)

सपषटीकरण

bull भीमा-क रगाोि की लडाई ततीय आोगल-मराठा

यद का वहससा थी

Q57) उततर (b)

सपषटीकरण

bull राजकमार शकल न गाोिीजी क चोपारण आन तथा

वतनकवथया परणाली स जडी समसया की जाोच क

वलए रारी करन क वलए दश भर म उनका

अनसरण वकया था

bull बज वकश र राजदर परसाद महादि दसाई और

नरहरी पाररख चोपारण सतयागरह क दौरान गाोिी

जी क सहय गी थ

Q58) उततर (b)

सपषटीकरण

bull बराहमण ो और बौद मठिाररय ो क कर-मकत गााि

अनदान म दन की परथा सतिाहन ो न आरमभ की

थी

Q59) उततर (c)

सपषटीकरण

इस कायिकरम क उददशय वनमनानसार ह

(i) बवनयादी पयििन आिाररक सोरचना का विकास

करना

(ii) चयवनत (पहचान वकय गए) कषतर ो म आजीविका क

सजन क वलए दश क साोसकवतक और विरासत

मलय ो क बिािा दना

(iii) विरासत समारक थथल ो पर विशव सतरीय आिाररक

सोरचना विकवसत करक एक सतत तरीक स

पयििक आकििण म िसद करना

(iv) थथानीय समदाय ो की सवकरय भागीदारी क माधयम

स र रगार ो का सजन करना

(v) र रगार उतपादन और आवथिक विकास क वलए

पयििन कषमता का उन पर परभाि का उपय ग

करना तथा

(vi) िारणीय पयििन आिाररक सोरचना का विकास

करना और उसका उवचत सोचालन तथा

रखरखाि सवनवशचत करना

Q60) उततर (b)

सपषटीकरण

bull यह वनकाय ििि 1987 म अससततव म आया था

bull यह एक राषटर ीय सतर का शीिि सोगठन ह ज भारत

सरकार क जनजातीय मामल ो क मोतरालय क

परशासवनक वनयोतरण क अिीन काम कर रहा ह

bull इसका पोजीकत और परिान कायािलय नई वदलली

म सथथत ह

Q61) उततर (c)

सपषटीकरण

bull परमचोद क उपनयास ो म परमाशरम रोगभवम गबन

कमिभवम और ग दान शावमल ह

bull ग रा रिी ोदरनाथ िग र क दवारा रवचत उपनयास ह

bull अभी हाल ही म मोशी परमचोद की 138िी ो जयोती दश

भर म मनाई गई थी

Q62) उततर (b)

सपषटीकरण

bull ldquoवगदाrdquo पोजाब (भारत) एिो पावकसतान की

मवहलाओो क दवारा तयौहार क समय और फसल

की बिाई तथा किाई क अिसर पर वकया जान

िाला एक पारोपररक दहाती नतय ह

FC19H1003 38

bull इस नतय क माधयम स पोजाबी मवहलाऐो अपनी

परसननता परकि करती ह तथा वगदा क परदशिन क

माधयम स परि िचिसव िाल समाज म मवहलाओो

की दबी हई भािनाओो क परकि करती ह

bull चोवक इस नतय का परि ो क साथ क ई सोबोि नही ो

ह अतः किल मवहलाऐो ही इसम भाग ल सकती

bull हर साल तीज समार ह क दौरान पोजाब म वगदा

नतय वकया जाता ह

तीज भारत क कछ भाग ो म मवहलाओो क दवारा

मनाया जान िाल कई तयौहार ो क वलए एक

वयापक नाम ह

Q63) उततर (a)

सपषटीकरण

- मजम-उल-बहरीन या द समदर ो का सोगम

नामक उललखनीय रचना दारा वशक ह क दवारा

वलखी थी

- भारत क उपराषटर पवत शरी एम िकया नायड न कहा

ह वक राजकमार दारा वशक ह की रचनाएा शाोवत

और सदभाि क बिािा दन क वलए एक तारा सर त

क रप म सामन आ सकती ो ह

- उपराषटर पवत गत ििो क भला वदए गए राजकमार

दारा वशक ह क परदवशित परचवलत करन हत

आय वजत एक परदशिनी का दौरा करन क बाद एक

सभा क सोब वित कर रह थ

- इस परदशिनी का आय जन फर क इस गौवियर

(Francois Gautier) क दवारा lsquoइोवदरा गाोिी नशनल

सिर फॉर द आििसrsquo (The Indira Gandhi

National Centre for the Arts) नई वदलली म

वकया गया था

Q64) उततर (c)

सपषटीकरण

- ग मतशवर परवतमा जन भगिान बाहबली क

समवपित ह

- यह एक एक-चटटानी पतथर की मवति ह

- राषटर पवत राम नाथ क विोद न शरिणबलग ला

(कनाििक) म आय वजत वकय जान िाल भवय

अवभिक समार ह महामसतकावभिक का

उदघािन वकया था

- यह समार ह 12 ििो म एक बार ह ता ह

Q65) उततर (c)

सपषटीकरण

bull पराची घािी पराची नदी क चार ो ओर फली हई थी

bull पराची घािी िीर-िीर विलपत ह गई थी

bull पराची नदी भिनशवर स वनकलती ह

bull यह महानदी की एक सहायक नदी ह और यह

परी खदाि किक तथा जगतवसोहपर वजल ो स

ह कर बहती ह

bull इस नदी क पर कषतर क पराची घािी कहा जाता ह

bull यह नदी बोगाल की खाडी म वगरती ह

परातासतवक साकषय स पता चलता ह वक पराची घािी

सभयता हडपपा और म हनज दाड द न ो की

पिििती ह

Q66) उततर (d)

सपषटीकरण

य समारक छतरपर वजल (मधय परदश) म विोधयाचल

पिित शरोखला म सथथत ह

Q67) उततर (a)

सपषटीकरण

bull थॉिस ऑन पावकसतान नामक पसतक डॉ बी

आर अमबडकर न वलखी थी

bull डॉ बी आर अमबडकर की जयोती क अिसर पर

भारत क राषटर पवत न भारत की इस महान हसती

क शरदाोजवल अवपित की थी

bull डॉ बी आर अमबडकर न 1924 म वडपरथड

कलावसर इोसटीटयि (दवलत िगि सोथथान -

बवहषकत वहतकाररणी सभा) और 1927 म समाज

समता सोघ की थथापना की थी

bull अमबडकर का धयान वशकषा कषतर की ओर भी था

bull उनह ोन वशकषा क वनमन िगो म फलान क वलए

पीपलस एजकशन स साइिी (The Peoples

Education Society) क नाम स महाविदयालय ो क

नििकि और छातरािास ो की थथापना की थी

FC19H1003 39

Q68) उततर (b)

सपषटीकरण

bull महरगि भारतीय उपमहादवीप म एक परवसद

निपािाण बसती ह ज बलवचसतान पराोत

पावकसतान म सथथत ह

bull दचपलली (आोधर परदश) क पास नागलर नदी क

पिी ति ो पर चना पतथर क बलॉक क विशाल

विसतार म एक पिि-ऐवतहावसक रॉक आिि थथल की

ख ज की गई ह

bull इसन 1500-2000 ईसा पिि क दौरान गोिर (आोधर

परदश) म विकवसत निपािाण सभयता पर परकाश

डाला ह

Q69) उततर (c)

सपषटीकरण

bull 12िी ो सदी और 13िी ो सदी म काकाविय िोश का

उदय हआ था

bull ि पहल कलयाण क पवशचमी चालकय ो क सामोत थ

bull परारोभ म उनह ोन िारोगल (तलोगाना) क पास एक

छ ि स कषतर पर शासन वकया था

bull उनह ोन ldquoनायक वयिथथाrdquo की शरआत की थी

वजस बाद म विजयनगर क राय शासक ो न

अपनाया और विकवसत वकया था

Q70) उततर (a)

सपषटीकरण

bull गाोिीजी क अनशन स वमल मावलक ो पर दबाि

पडा था ज अोततः शरवमक ो क ितन म 35 परवतशत

की िसद करन क वलए सहमत हए थ

bull गगल (Google) न अनसया साराभाई वजनह ोन

भारत क शरवमक आोद लन म एक अगरणी भवमका

वनभाई थी की 132िी ो जयोती डडल (Doodle) का

वनमािण करक मनाई

Q71) उततर (d)

सपषटीकरण

भारत स यनसक की मानिता की अमति साोसकवतक

विरासत की परवतवनवि सची म वनमनवलसखत शावमल ह

bull कवडयटटम करल का सोसकत रोगमोच

bull मवडयिि करल का अनषठान रोगमोच और नतय

नाविका

bull िवदक मि जाप की परोपरा

bull राजथथान क कालबवलया ल क गीत और नतय

bull रामलीला रामायण का पारोपररक परदशिन

bull सोकीतिन मवणपर का अनषठान गायन ढ ल िादन

और नतय

bull रममन भारत क गििाल वहमालय का िावमिक

तयौहार और अनषठान रोगमोच

bull जाोदीयाला गर पोजाब क ठठर ो की पीतल और

ताोब क वशलप स वनवमित बतिन ो की पारोपररक कला

bull छाऊ नतय पिी भारतीय राजय ो म जनमी शासतरीय

भारतीय नतय कला

bull लददाख का बौद मि जाप िर ाोस-वहमालयी लददाख

कषतर तथा जमम-कशमीर म पवितर बौद गरोथ ो का पाठ

bull य ग

bull नौर र

bull को भ मला

Q72) उततर (b)

सपषटीकरण

bull भारत क राषटर पवत शरी राम नाथ क विोद न

वकसामा नागालड म हॉनिवबल मह रति और

राजय गठन वदिस समार ह का उदघािन वकया

था

bull हॉनिवबल मह रति का नाम भारतीय हॉनिवबल क

नाम पर पडा ह ज एक विशाल और रोगीन जोगली

पकषी ह

bull यह पकषी नागालड राजय की अविकतर जनजावतय ो

की ल ककथाओो म उसललसखत ह

bull नागालड की परमख मानयता परापत जनजावतयाा ह

अोगामी आओ चखसोग चाोग ककी रगमा और

रवलोग आवद

bull ओोग जारिा और ससिनलीस अोडमान-वनक बार

दवीप समह की जनजावतयाा ह

FC19H1003 40

Q73) उततर (c)

सपषटीकरण

bull दकन म राषटर कि शासन दसिी ो सदी क अोत तक

लगभग 200 ििो तक रहा था

bull राषटर कि शासक अपन िावमिक विचार ो म सवहषण

bull उनह ोन न किल शि िमि और िषणि िमि बसलक

जन िमि क भी सोरकषण वदया था

bull एल रा म वशि क परवसद रॉक कि मोवदर का

वनमािण नौिी ो सदी म राषटर कि राजा कषण परथम न

करिाया था

bull उसका उततराविकारी अम घििि जन था लवकन

उसन अनय िमो क भी सोरकषण परदान वकया था

bull राषटर कि ो न मसलमान वयापाररय ो क बसन की

अनमवत दी थी

bull उनह न अपन अविराजय ो म इसलाम क उपदश दन

की भी अनमवत दी थी

bull अभी हाल ही म पाोडिलागटटा (तलोगाना) क

परागवतहावसक चटटान वचतर ो क कषरण की बिती हई

घिनाएा एक गोभीर वचोता का वििय ह

bull यह परागवतहावसक चटटान क नकसान पहाचा

सकता ह

bull पाोडिलागटटा वनमनवलसखत क वलए जाना जाता ह

- 10000 ईसा पिि स 8000 ईसा पिि क वचवतरत

चटटानी आशरय ो क वलए

- राषटर कि काल क एक 8 िी ो सदी क

वशलालख क वलए और

- 12िी ो सदी क काकविय सामराजय क वभवतत

वचतर ो क वलए

Q74) उततर (b)

सपषटीकरण

bull 1828 म राजा राम म हन रॉय न एक नय िावमिक

समाज बरहम सभा की थथापना की थी वजस बाद

म बरहम समाज क नाम स जाना गया था

bull दिदरनाथ िग र न ततवब विनी सभा की अधयकषता

की थी ज आधयासिक सतय की ख ज म सोलि

थी

bull इसका उददशय वहोद िमि क शद करन का और

एकशवरिाद (एक ईशवर म आथथा) का परचार करना

था

bull नय समाज की थथापना क आिार थ कारण

(तकि ) क द सतमभ तथा िद और उपवनिद

bull अभी हाल ही म सािारण बरहम समाज का कछ

काननी मदद ो क लकर पवशचम बोगाल सरकार क

साथ काननी वििाद चल रहा ह

Q75) उततर (c)

सपषटीकरण

bull भारत म वचशती वसलवसल की थथापना खवाजा

म इनददीन वचशती क दवारा की गयी थी

bull ि 1192 ईसवी क आसपास भारत आय थ

bull वचशतीय ो क बारहिी ो शताबदी क उततरािि म भारत

म आन िाल सफीय ो क समह ो म सबस

परभािशाली माना जाता ह

bull उनह ोन थथानीय िातािरण क साथ सफलतापििक

अनकलन वकया और उनह ोन भारतीय भसकत

परोपराओो क कई पहलओो क अपनाया

bull अजमर म सफी अपरकि खवाजा म इनददीन वचशती

की ऐवतहावसक दरगाह क एक नया रप दन की

तयारी की जा रही ह

bull इस 13िी ो शताबदी की दरगाह क ldquoसवचछ

आइकॉवनक थथल ोrdquo (Swacch Iconic Places) म

शावमल वकया गया ह ज परवतवषठत विरासत

आधयासिक और साोसकवतक थथान ो पर क वदरत

य जना ह

FC19H1003 41

ANSWERS amp EXPLANATION OF

NCERT History Class VI-X + Current Affairs

(FC19E1003)

Q1) Answer c

Explanation

Rigveda consists of more than a

thousand hymns dedicated to gods and

goddesses These hymns were

composed by sages and learnt by men

however a few were composed by

women like Apala Ghosa Lopamudra

Maitreyi and Gargi

Rigveda consists of many hymns in the

form of dialogues We get an example of

a dialogue between a sage named

Vishwamitra and two rivers (Beas and

Sutlej) that were worshipped as

goddesses This suggests that he

belonged to the Vedic period

Q2) Answer b

Explanation

Traces of ash have been found from

Kurnool Caves suggesting that people

were familiar with the use of fire

It is situated in Andhra Pradesh

Q3) Answer c

Explanation

Burzahom is a prehistoric site in

present day Kashmir where people built

pit houses which were dug into the

ground with steps leading into them

These may have provided shelter in cold

weather

Q4) Answer c

Explanation

Epigraphy is defined as the study of

inscriptions

Manuscriptology is the study of history

and literature through the use of hand

written documents

Palaeography refers to the study of

ancient writing systems and the

deciphering and dating of historical

manuscripts

Numismatics refers to the study of

coins

Q5) Answer a

Explanation

Charaka Samhita was written by

Charaka and is an important book on

Ayurveda and medicine

He was a practitioner of the traditional

system of Indian medicine known as

Ayurveda

Charaka is thought to have flourished

sometime between the 2nd century BCE

and the 2nd century CE

Q6) Answer b

Explanation

Bhaga refers to the tax on crops which

was fixed at 16th of the production

Kammakaras is the term used for the

landless agricultural labour class

Ashvamedha also known as horse

sacrifice is a ritual where a horse is let

loose to wander freely and it was

guarded by the rajarsquos men

Q7) Answer (d)

Explanation

In the Rigvedic period horses were

yoked to chariots that were used in

battles fought to capture land cattle

etc This suggests that the use of horse

chariots began much before the period

of Mahajanapadas

The battles were fought in the Rigvedic

period for cattlersquos lands water an even

to capture people Most men took part

in these wars however there was no

regular army but there were assemblies

where people met and discussed

matters of war Regular armies became

a feature in the Mjahajanapada period

including vast armies of foot soldiers

chariots and elephants

RAUSIAS-FC19E1003 42

Q8) Answer (a)

Explanation

Buddha belonged to the Sakya clan and

passed away at Kusinara

Buddha taught in Prakrit which was the

common language of people

Q9) Answer c

Explanation

There were six schools of philosophy in

ancient India These are known as

Vaishesika Nyaya Samkhya Yoga

Purva Mimansa and Vedanata or Uttara

Mimansa They were founded by sages

Kanada Gautama Kapila Patanjali

Jamini and Vyasa respectively

Q10) Answer b

Explanation

The teachings of Mahavira were

compiled at Valabhi in 6th century AD

Q11) Answer (c)

Explanation

Chanakya is traditionally identified as

Kautilya or Vishnugupta who authored

the ancient Indian political treatise the

Arthashastra

Q12) Answer d

The national emblem of India is an

adaptation of the Lion Capital atop the

Ashoka Pillar of Sarnath Uttar Pradesh

and is combined with the National

Motto Satyameva Jayate

The Rampurva Bull gets the name from

the site of its discovery Rampurva in

Bihar

It is noted for its delicately sculpted

model demonstrating superior

representation of soft flesh sensitive

nostrils alert ears and strong legs It is

a mixture of Indian and Persian

elements

Sankissa is situated in Uttar Pradesh

India

Q13) Ans(a)

Kunwar Singh was a notable leader during the Revolt of 1857 He belonged

to a royal house of Jagdispur Bihar

Q14) Answer b

Explanation

The term Vellalar was used for large

landowners

Q15) Answer c

Explanation

Arikamedu was a coastal settlement

where ships unloaded goods from

distant lands Finds here include a

massive brick warehouse pottery

including amphorae and Arretine ware

Roman lamps glassware and gems have

also been found at the site

Q16) Answer a

Explanation

Muvendar is a Tamil word mentioned in

Sangam poems meaning three chiefs

used for the heads of three ruling

families the Cholas Cheras and

Pandyas

Q17) Ans (c)

Several tribal or kin-based assemblies

such as the Sabha Vidatha and gana

are mentioned in the Rig-veda The

Sabha and the samiti mattered a great

deal in early Vedic times so much so

that the chiefs or the kings showed an

eagerness to win their support

Q18) Ans (a)

Jainism recognised the existence of the

gods but placed them lower than the

jina and did not condemn the varna

system as Buddhism did

Q19) Answer (d)

Explanation

Cholas and Pandyas had developed

powerful coastal cities The most

important city of Cholas was Puhar or

Kaveripattinam and Madurai was the

capital of Pandyas

Q20) Answer b

Explanation

Buddhacharita is the biography of

Buddha and was written by

RAUSIAS-FC19E1003 43

Ashvaghosha

Q21) Answer (a)

Explanation

Tamil poet Appar was a Shiva devotee

So he was a Nayanar saint

Q22) Answer d

Explanation

Samudragupta was a prominent Gupta

ruler whose coins depict him playing a

veena indicating his love for music We

get important historic information from

his Allahabad Prashasti which was

composed by his court poet Harisena

Q23) Answer (b)

Explanation

Vikrama Samvat was founded by

Chandragupta II in the 58 BC as a

mark of victory over the Shakas and

assumed the title of Vikramaditya

Banabhatta wrote Harshavardhanarsquos

biography the Harshacharita in

Sanskrit

Q24) Answer c

Explanation

Sandhi-vigrahika was the minister of

war and peace

Sarthavaha was the leader of the

merchant caravans

Q25) Answer a

Explanation

Xuan Zang (Hsuan-tsang) was a

Chinese traveller who came during the

reign of Harshavardhana

In the decade that began in 630 AD

Xuan Zang came to India through

Kashmir after visiting Central Asia Iran

and Afghanistan

He travelled from north to east and lived

in Bihar for a couple of years

At Nalanda University Xuan Zang

interacted with students and scholars

mastered local languages and

discovered Buddhist stupas

Q26) Answer c

Explanation

Pradakshina patha is a circular path

laid around a stupa in Buddhist

architecture While the rest are a part of

temple architecture

Q27) Answer d

Explanation

All the above-mentioned temples have

an elaborate use of bricks (baked

bricks) along with stone

Q28) Ans (c)

Muhammad Quli Qutab was the Sultan

of Golconda He was a contemporary of

Akbar was very fond of literature and

architecture

The Sultan was a great poet and he

wrote in Dakhini Urdu Persian and

Telgu and has left an extensive diwan or

collection

Recently the Archaeological Survey of

India (ASI) will be using Ground

Penetrating Radar (GPR) to map the

contours of the area around the Bagh-e-

Naya Qila excavated garden inside the

Golconda Fort in Telangana

Q29) Answer a

Explanation

Silappadikaram is a famous Tamil epic

which was written by Ilango around

1800 years ago It is a story of a

merchant named Kovalan who fell in

love with a courtesan named Madhavi

Manimekalai tells the story of the

daughter of Kovalan and Madhavi

Q30) Answer (a)

Explanation

Charaka is the author of Charaka

Samhita which is an important work of

Ayurveda and medicines

Brahmaguptarsquos fame rests mostly on his

Brahma-sphuta-siddhanta which was

an astronomical work It was translated

into Arabic in Baghdad and had a major

impact on Islamic mathematics and

astronomy

Late in his life Brahmagupta wrote

Khandakhadyaka which was an

RAUSIAS-FC19E1003 44

astronomical handbook that employed

Aryabhatarsquos system of starting each day

at midnight

Q31) Answer (c)

Explanation

Amir Khusrau was a famous sufi

musician poet and scholar In 1318 he

noted that there was different language

in every region of this land (Hindustan)

Lahori Kashmiri Dvarsamudri (in

Southern Karnataka) Telangana (in

Andhra Pradesh) Gujari (in Gujarat)

Marsquobari (in Tamil Nadu) Awadhi (in

eastern Uttar Pradesh) and Hindawai (in

the area around in Delhi) etc He went

to explain that Sanskrit did not belong

to any region and that only brahmans

knew it

Q32) Answer c

Explanation

Hiranyagarbha refers to the golden

womb When this ritual was performed

with the help of Brahmanas it was

thought to lead to the rebirth of the

sacrificer as a Khastriya

Q33) Answer d

Explanation

Kadamai refers to a tax on land

revenue

Gwalior Prashasti describes the exploits

of Nagabhata who was a Pratihara king

Q34) Answer b

Explanation

Rajatarangini is a Sanskrit text written

by Kalhana in the 12th century

It was historical chronicle of early India

It is justifiably considered to be the best

and most authentic work of its kind

It covers the entire span of history in

the Kashmir region from the earliest

times to the date of its composition

Q35) Answer c

Explanation

ldquoUrrdquo was the general assembly of the

village ldquoUrrdquo consisted of all the

taxpaying residents of an ordinary

village

Q36) Answer (a)

Explanation

Tarikh was a form of history writing in

the Delhi Sultanate The authors of

tawarikhs were learned men which

included secretaries administrators etc

Q37 Answer (a)

Explanation

Alauddin chose to pay his soldiers salaries in cash rather than iqtas The soldiers would buy their supplies from merchants in Delhi and it was thus feared that merchants would raise their prices To stop this Alauddin controlled the prices of goods in Delhi Prices were carefully surveyed by officers and merchants who did not sell at the prescribed rates were punished

Q38) Answer (d)

Explanation

Delhi first became the capital of a

kingdom under the Tomara Rajputs

who were defeated in the middle of the

twelfth century by the Chauhans (also

referred to as Chahamanas) of Ajmer

It was under the Tomaras and

Chauhans that Delhi became an

important commercial centre Many rich

Jaina merchants lived in the city and

constructed several temples Coins

minted here called dehliwal had a wide

circulation

Q39) Answer (c)

Explanation

Moth ki Masjid was built in the reign of

Sikandar Lodi by his minister

Begumpuri mosque built in the reign of

Muhammad Tughluq was the main

mosque of Jahanpanah the ldquoSanctuary

of the Worldrdquo and his new capital in

Delhi

Quwwat al ndash Islam mosque was

enlarged by Iltutmish and Alauddin

Khalji The minar was built by three

Sultansndash Qutbuddin Aybak Iltutmish

and Firuz Shah Tughluq

RAUSIAS-FC19E1003 45

Q40) Answer (c)

Explanation

Under the Mughals mansabdar was

referred to an individual who held a

mansab ie rank and he received his

salary as revenue assignments called

jagirs

Q41) Ans (b)

The Quit India Movement was a

spontaneous revolt of people against

British rule

The All India Congress Committee met

at Bombay on 8 August 1942 It passed

the famous resolution Quit India and

proposed the starting of a non-violent

mass struggle under Gandhis

leadership to achieve this aim But on

the very next day Gandhi and other

eminent leaders of the Congress were

arrested The Congress was once again

declared illegal

Q42) Ans (c)

The Simon Commission refers to a

group of seven MPs from the United

Kingdom constituted to suggest

constitutional reforms for British India

The Commission consisted of only

British members headed by one of the

senior British politicians Sir John

Simon

So the people of India agitated against

the arrival of Simon Commission

Q43) Ans (a)

He was widely known for his

unfavourable opinion of the economic

consequences of the British rule in

India

In his many writings and speeches and

especially in Poverty and Un-British

Rule in India Naoroji argued that India

was too highly taxed and that its wealth

was being drained away to England

He did not interpret the ancient Indian

texts and restored the self-confidence of

Indians And also he did not stress the

need for eradication of all the social

evils before anything else

Q44) Ans (c)

In August 1932 Prime Minister

MacDonald announced his Communal

Award Great Britainrsquos unilateral

attempt to resolve the various conflicts

among Indiarsquos many communal

interests

The award which was later

incorporated into the act of 1935

expanded the separate-electorate

formula reserved for Muslims to other

minorities including Sikhs Indian

Christians Anglo-Indians Europeans

distinct regional groups Gandhi

undertook a ldquofast unto deathrdquo against

that offer which he viewed as a

nefarious British plot to divide the

Indian society

Q45) Ans (b)

In British India apart from existing

imports and exports there was also a

particular amount of money which

colonial India contributed towards

administration maintenance of the

army war expenses pensions to retired

officers and other expenses accrued by

Britain towards maintenance of her

colony These were known as Home

charges and were paid for almost

entirely by India

The Home charges was made of

following components-

- Interest payable on Indian debt

- Dividend to shareholders of East

India Company

- Funds used to support the India

Office in London

- Funds used to pay salaries and

pensions of British personnel

engaged in India

- Interest on the railways

- Civil and military charges

- Store purchases in England

Q46) Ans (b)

The Lahore session of the Indian

National Congress was held in 1929

under the Presidentship of Jawaharlal

Nehru

The Lahore session of the Indian

National Congress witnessed significant

RAUSIAS-FC19E1003 46

developments in the Indian national

movement

- First the election of Jawaharlal

Nehru to the post of Presidentship of

the Congress was a clear indication

of the growing strength of the

Leftists in the Congress

- Secondly it was in this session that

the Congress for the first time raised

the demand for complete

independence Such demand was

not raised from the Congress

platform earlier

Q47) Ans (b)

It did not provide for separate

electorates for any community or

weightage for minorities However it did

allow for the reservation of minority

seats in provinces having minorities of

at least ten per cent but this was to be

in strict proportion to the size of the

community

There was no provision for complete

Independence for India

Q48) Ans (c)

The religion of early Vedic Aryans was

primarily of worship of nature and

Yajnas

The early Aryan religion was kind of

nature worship Actually the forces

around them which they could not

control or understand were invested

with divinity and were personified as

male or female gods And they

performed some Yajnas also

Q49) Ans (b)

The roads and river-routes were not

immune from robbery It is notable that

Yuan Chwang (Hiuen Tsang) was

robbed of his belongings during

Harshvardanarsquos period

Q50) Ans (c)

Q51) Ans (b)

Purandara Dasa was a saint and great

devotee of Lord Krishna

There is much speculation about where

Purandara Dasa regarded as the

Pitamaha of Carnatic music was born

Recently an expert committee

constituted by the Kannada University

Hampi has come to the conclusion that

Kshemapura Shivamogga district

Karnataka is the birth place of

Purandara Dasa

Q52) Ans (c)

Sri Tyagaraja Sri Shyama Shastry and Sri Muthuswami Dikshitar are considered the trinity of Carnatic music and with them came the golden age in Carnatic music in the 18th-19th

century

Q53) Ans d)

Recently a rare sarcophagus (stone

coffin) which is 2000 years old from the

Iron AgendashMegalithic era was discovered

from a rock-cut cave at Viyur village of

Kollam near Koyilandy in Kozhikode

district Kerala

The coffin containing bone fragments

was found during an excavation ldquoSo

far such a rare finding has been

discovered only from two sites

in Kerala Both these sarcophagi were

recovered from Megalithic sites at

Chevayur and Atholi also in Kozhikode

district

Q54) Ans a)

The megalithic culture in South India was a full-fledged Iron Age culture

Q55) Ans d)

The Cholas Pandyas and Keralaputras

(Cheras) mentioned in Ashokan

inscriptions were probably in the late

megalithic phase of material culture

Q56) Ans d)

Q57) Ans (b)

Raj Kumar Shukla followed Gandhiji all

over the country to persuade him to

come to Champaran to investigate the

problem associated with tinkathia

system

RAUSIAS-FC19E1003 47

Brij Kishore Rajendra Prasad Mahadev

Desai and Narhari Parikh accompanied

Gandhi ji during the Champaran

Satyagraha

Q58) Ans (b)

The Satvahanas started the practice of granting tax-free villages to brahmanas and Buddhist monks

Q59) Ans c)

The objectives of the Programme are

listed as under

- Developing basic tourism

infrastructure

- Promoting cultural and heritage

value of the country to generate

livelihoods in the identified regions

- Enhancing the tourist attractiveness

in a sustainable manner by

developing world-class

infrastructure at the heritage

monument sites

- Creating employment through active

involvement of local communities

- Harnessing tourism potential for its

effects on employment generation

and economic development

- Developing sustainable tourism

infrastructure and ensuring proper

Operations and maintenance

therein

Q60) Ans (b)

The Tribal Cooperative Marketing

Development Federation of India

(TRIFED) came into existence in 1987

It is a national-level apex organization

functioning under the administrative

control of Ministry of Tribal Affairs

Govt of India

TRIFED has its registered and Head

Office located in New Delhi

Q61) Ans (c)

Premchandrsquos novels include

Premashram Rangabhumi Ghaban

Karmabhumi and Godan

Gora is a novel written by Rabindranath

Tagore

138th birth anniversary of Munshi

Premchand was celebrated across the

country

Q62) Ans (b)

Giddha is a traditional pastoral dance

performed by the women of the Punjab

India and Pakistan at festival times

and at the sowing and reaping of the

harvest

By this dance the Punjabi women

reveal their joy expel their suppressed

feelings in a male dominated society

through the performance of Giddha

Since this dance has nothing to do with

men only women can participate in it

During the Teej celebrations Giddha

dance is celebrated in Punjab every

year Teej is a generic name for a

number of festivals that are celebrated

by women in some parts of India

Q63) Ans (a)

Dara Shukoh wrote the remarkable

work called ldquoMajma-ul-Bahrainrdquo or the

ldquoThe confluence of two seasrdquo

The Vice President of India Shri M

Venkaiah Naidu has said that Prince

Dara Shukohrsquos writings can come as a

refreshing source for infusing peace and

harmony He was addressing the

gathering after visiting the exhibition

that showcases the forgotten Prince of

yesteryears Dara Shukoh organized by

Mr Francois Gautier at Indira Gandhi

National Centre for the Arts in New

Delhi

Q64) Ans (c)

The statue Gommateshwara is

dedicated to the Jain God Bahubali

It is a monolithic statue

President Ram Nath Kovind

inaugurated the grand anointing

ceremony mdash Mahamastakabhisheka mdash

held once in 12 years at

Shravanabelagola (Karnataka)

Q65) Ans (c)

Prachi Valley had come up around the

Prachi river Prachi Valley gradually

disappeared

RAUSIAS-FC19E1003 48

The Prachi river originates from

Bhubaneswar

It is a tributary of the Mahanadi and

flows through the districts of Puri

Khurda Cuttack and Jagatsinghpur

and the entire region of the river is

termed as the Prachi Valley

It falls into the Bay of Bengal

Archaeological evidence shows that the

Prachi Valley Civilisation predates both

Harappa and Mohenjo-Daro

The Prachi river originates from

Bhubaneswar

Q66) Ans (d)

These monuments are located in

Chhatarpur district Madhya Pradesh

within Vindhya mountain range

Q67) Ans (a)

The book lsquoThoughts on Pakistanrsquo was

written by Dr BR Ambedkar

On the occasion of the birth anniversary

of Dr BR Ambedkar the president of

India pays homage to this icon of India

In 1924 he founded the Depressed

Classes Institute (Bahishkrit Hitkarini

Sabha) and in 1927 the Samaj Samata

Sangh

Another area of attention for Ambedkar

was education For its spread among

the low classes he set up a network of

colleges by the name of Peoples

Education Society and founded hostels

Q68) Ans(b)

Mehrgarh is a famous Neolithic

settlement in the Indian subcontinent

which is situated in Baluchistan

province Pakistan

A pre-historic rock art site is discovered

in the vast expanse of limestone blocks

on the eastern banks of Naguleru river

near Dachepalli (Andhra Pradesh) It

has thrown light on the Neolithic

civilisation that flourished in Guntur

(Andhra Pradesh) during 1500-2000

BC

Q69) Ans (c)

The 12th and the 13th centuries saw

the emergence of the Kakatiyas They

were at first the feudatories of the

Western Chalukyas of Kalyana Initially

they ruled over a small territory near

Warangal (Telangana)

They introduced Nayakships which was

later adopted and developed by the

Rayas of Vijayanagara

Q70) Ans (a)

The fast had effect of putting pressure

on mill owners who finally agreed to

give the workers a 35 per cent increase

in wages

Google celebrated with a doodle the

132nd birth anniversary of Anasuya

Sarabhai who played a pioneering role

in Indiarsquos labour movement

Q71) Ans (d)

The UNESCOrsquos list of the representative

list of the intangible cultural heritage of

humanity from India are

- Koodiyattam Sanskrit Theatre of

Kerala

- Mudiyettu ritual theatre and dance

drama of Kerala

- Tradition of Vedic Chanting

- Kalbelia folk songs and dances of

Rajasthan

- Ramlila Traditional Performance of

the Ramayana

- Sankirtana ritual singing

drumming and dancing of Manipur

- Ramman religious festival and

ritual theatre of the Garhwal

Himalayas India

- Traditional brass and copper craft of

utensil making among the Thatheras

of Jandiala Guru Punjab India

- Chhau dance classical Indian dance

originated in the eastern Indian

states

- Buddhist chanting of Ladakh

recitation of sacred Buddhist texts

in the trans-Himalayan Ladakh

region Jammu and Kashmir India

- Yoga

- Nouroz

- Kumbh Mela

RAUSIAS-FC19E1003 49

Q72) Ans(b)

The President of India Shri Ram Nath Kovind inaugurated the Hornbill Festival and State Formation Day celebrations of Nagaland in Kisama

The festival is named after the Indian hornbill the large and colourful forest bird which is displayed in the folklore of most of the states tribes

The major recognized tribes of Nagaland are Angami Ao Chakhesang Chang

Kuki Rengma and Zeling etc

Onge Jarawa and Sentinelese are the

tribes of Andman amp Nicobar Islands

Q73) Ans (c)

The Rashtrakutas rule in the Deccan lasted for almost two hundred years till the end of the tenth century The Rashtrakutas rulers were tolerant in their religious views and patronized not only Shaivism and Vaishnavism but

Jainism as well

The famous rock-cut temple of Shiva at Ellora was built by one of the Rashtrakutas kings Krishna I in the ninth century His successor Amoghavarsha was a Jain but he also

patronized other faiths

The Rashtrakutas allowed Muslims traders to settle and permitted Islam to

be preached in their dominions

Recently increasing defacement at the prehistoric rock paintings of Pandavulagutta Telangana has created a cause for grave concern It can spoil

the prehistoric rock

Pandavulagutta is home to

- Painted rock shelters dating to

10000 BC-8000 BC

- An 8th century inscription of the

Rashtrakuta period and

- Painted frescoes from the 12th century Kakatiya empire

Q74) Ans (b)

In 1828 Raja Ram Mohan Roy founded a new religious society the Brahma Sabha later known as the Brahmo

Samaj

Debendranath Tagore headed the Tattvabodhini Sabha which was

engaged in search of spiritual truth

Its purpose was to purify Hinduism and to preach monotheism or belief in one God

The new society was to be based on the twin pillars of reason and the Vedas and

Upanishads

Recently Sadharan Brahmo Samaj (SBS) has entered into a legal battle with the West Bengal government due

to some legal issue

Q75) Ans (c)

The Chishti order was established in India by Khwaja Moinuddin Chishti who came to India around 1192 The Chishtirsquos are considered to be the most influential of the groups of Sufis who migrated to India in the late twelfth century They adapted successfully to the local environment and adopted several features of Indian devotional

traditions

The historical dargah of Sufi mystic Khwaja Moinuddin Chishti in Ajmer is all set to get a facelift This 13 th century dargah has been included among the Swachh Iconic Places a clean-up initiative focused on iconic

heritage spiritual and cultural places

Page 19: GENERAL STUDIES (PAPER I) · Test is part of Rau’s IAS Test series for Preliminary Exam 2019 FOUNDATION + CURRENT AFFAIRS GENERAL STUDIES (PAPER –I) FOUNDATION TEST –III TOPIC:

RAUSIAS-FC19E1003 19

Q49) The Chinese traveller Yuan Chwang

(Hiuen Tsang) who visited India

recorded the general conditions and

culture of India at that time In this

context which of the following

statements isare correct

1 The roads and river-routes were

completely immune from robbery

2 As regards punishment for

offences ordeals by fire water and

poison were the instruments for

determining the innocence or guilt

of a person

3 The tradesmen had to pay duties

at ferries and barrier stations

Select the correct answer using the code

given below

(a) 1 only

(b) 2 and 3 only

(c) 1 and 3 only

(d) 1 2 and 3

Q50) Regarding the Indus Valley Civilization

consider the following statements

1 It was predominantly a secular

civilization and the religious

element though present did not

dominate the scene

2 During this period cotton was

used for manufacturing textiles in

India

Which of the statements given above

isare correct

(a) 1 only

(b) 2 only

(c) Both 1 and 2

(d) Neither 1 nor 2

Q51) Consider the following statements

regarding Purandara Dasa

1 Purandara Dasa was a saint and

great devotee of Lord Shiva

2 He was a composer singer and

one of the chief founding-

proponents of the Carnatic music

Which of the statements given above

isare correct

(a) 1 only

(b) 2 only

(c) Both 1 and 2

(d) Neither 1 nor 2

Q52) Which of the following persons isare

included in the trinity of Carnatic

music

1 Balamurali Krishna

2 Sri Shyama Shastry

3 Sri Muthuswami Dikshitar

Select the correct answer using the code

given below

(a) 1 only

(b) 2 only

(c) 2 and 3 only

(d) 1 2 and 3

Q53) Megalithic sites at Chevayur and Atholi

are located in which of the following

states

(a) Tamil Nadu

(b) Karnataka

(c) West Bengal

(d) Kerala

RAUSIAS-FC19E1003 20

Q54) निमननिखित कथि ो पर निचार कीनजए

1 महापािानरक ि ग कबर ो म िसतएो दफिात थ

2 दनकषर भारत म महापािार सोसकनत एक परण

निकनसत तामर यगीि सोसकनत थी

उपयणकत कथि ो म स कौि-सास सही हह

(a) किि 1

(b) किि 2

(c) 1 और 2 द ि ो

(d) ि त 1 ि ही 2

Q55) निमननिखित म स कौि-स सामराजयसामराजय ो का

अश क क अनभिि ो म उललि नकया गया ह

1 च ि

2 पाणडय

3 करिपतर (चर)

िीच नदए गए कट का परय ग कर सही उततर चनिए

(a) किि 1

(b) किि 1 और 2

(c) किि 3

(d) 1 2 और 3

Q56) भीमा-क रगाोि का यदध को पिी क सनिक ो और

बाजीराि नदवतीय क िततव म एक शखकतशािी पशिा

सिा (मराठ ो) क मधय िड़ा गया था यह यदध

निमननिखित म स नकसका नहससा था

(a) परथम आोगल-मराठा यदध का

(b) नदवतीय आोगल-मराठा यदध का

(c) ततीय आोगल-मसर यदध का

(d) ततीय आोगल-मराठा यदध का

Q57) निमननिखित कथि ो पर निचार कीनजए

1 महादि दसाई ि गाोरीजी क चोपारर आि तथा

नतिकनथया पररािी स जड़ी समसया की जाोच

क निए रारी करि क निए दश भर म उिका

अिसरर नकया था

2 िरहरी पाररि चोपारर सतयागरह क दौराि

गाोरीजी क साथ थ

उपयणकत कथि ो म स कौि-सास सही हह

(a) किि 1

(b) किि 2

(c) 1 और 2 द ि ो

(d) ि त 1 ि ही 2

Q58) निमननिखित कथि ो पर निचार कीनजए

1 िनद राज-िोश ि बराहमर ो और बौदध मठराररय ो

क कर-मकत गाि अिदाि म दि की परथा

आरि की थी

2 सतिाहि ो की आनरकाररक भािा पराकत थी

उपयणकत कथि ो म स कौि-सास सही हह

(a) किि 1

(b) किि 2

(c) 1 और 2 द ि ो

(d) ि त 1 ि ही 2

Q59) एक निरासत क अपिाइए (अडॉपट ए हररटज ndash

Adopt a Heritage) पररय जिा क उददशय ो क

सनदभण म निमननिखित कथि ो पर निचार कीनजए

1 यह पररय जिा र रगार उतपादि और आनथणक

निकास क निए पयणटि कषमता का उि पर

परभाि का उपय ग करगी

2 यह पररय जिा निरासत सथि ो पर निशव सतरीय

आराररक सोरचिा निकनसत करक एक सतत

तरीक स पयणटक आकिणर म िखदध करगी

उपयणकत कथि ो म स कौि-सास सही हह

(a) किि 1

(b) किि 2

(c) 1 और 2 द ि ो

(d) ि त 1 ि ही 2

RAUSIAS-FC19E1003 21

Q54) Consider the following statements

1 Megalithic people buried goods in

graves

2 The megalithic culture in South

India was a full-fledged Copper

Age culture

Which of the statements given above

isare correct

(a) 1 only

(b) 2 only

(c) Both 1 and 2

(d) Neither 1 nor 2

Q55) Which of the following kingdoms isare

mentioned in the Ashokan inscriptions

1 Cholas

2 Pandyas

3 Keralaputras (Cheras)

Select the correct answer using the code

given below

(a) 1 only

(b) 1 and 2 only

(c) 3 only

(d) 1 2 and 3

Q56) The Battle of Bhima-Koregaon was

fought between the soldiers of the

Company and the strong Peshwa army

(Marathas) under Bajirao II This war

was a part of the

(a) First Anglo-Maratha war

(b) Second Anglo-Maratha war

(c) Third Anglo- Mysore war

(d) Third Anglo-Maratha war

Q57) Consider the following statements

1 Mahadev Desai followed Gandhiji all over the country to persuade him to come to Champaran to investigate the problem associated

with tinkathia system

2 Narhari Parikh accompanied Gandhi ji during the Champaran

Satyagraha

Which of the statements given above isare correct

(a) 1 only

(b) 2 only

(c) Both 1 and 2

(d) Neither 1 nor 2

Q58) Consider the following statements

1 The Nanda Dynasty started the practice of granting tax-free villages to brahmanas and

Buddhist monks

2 The official language of the Satavahanas was Prakrit

Which of the statements given above

isare correct

(a) 1 only

(b) 2 only

(c) Both 1 and 2

(d) Neither 1 nor 2

Q59) Consider the following statements about the objectives of the lsquoadopt a heritagersquo

project

1 It will harness tourism potential for its effects on employment generation and economic

development

2 It will enhance the tourist attractiveness in a sustainable manner by developing world class infrastructure at heritage sites

Which of the statements given above

isare correct

(a) 1 only

(b) 2 only

(c) Both 1 and 2

(d) Neither 1 nor 2

RAUSIAS-FC19E1003 22

Q60) ldquoभारतीय जिजातीय सहकारी निपरि निकास सोघrdquo

(The Tribal Co-operative Marketing

Development Federation of India - TRIFED)

क सोदभण म निमननिखित कथि ो पर निचार कीनजए

1 यह एक राषटर ीय सतर का शीिण सोगठि ह ज

भारत सरकार क गह मोतरािय क परशासनिक

नियोतरर क अरीि काम कर रहा ह

2 इसका मखय उददशय दश म जिजातीय ि ग ो

का सामानजक-आनथणक निकास करिा ह

उपयणकत कथि ो म स कौि-सास सही हह

(a) किि 1

(b) किि 2

(c) 1 और 2 द ि ो

(d) ि त 1 ि ही 2

Q61) निमननिखित म स कौि-सास उपनयास परमचोद क

दवारा नििा गया हनिि गए ह

1 रोगभनम

2 ग दाि

3 ग रा

िीच नदए गए कट का परय ग कर सही उततर चनिए

(a) किि 1

(b) किि 2

(c) किि 1 और 2

(d) 1 2 और 3

Q62) नगदधा ितय क सोदभण म निमननिखित कथि ो पर निचार

कीनजए

1 नगदधा नबहार की मनहिाओो क दवारा तयौहार क

समय और फसि की बिाई तथा कटाई क

अिसर पर नकया जाि िािा एक पारोपररक

दहाती ितय ह

2 इस ितय क दवारा मनहिाऐो अपिी परसननता

परकट करती ह तथा नगदधा क परदशणि क

माधयम स परि िचणसव िाि समाज म

मनहिाओो की दबी हई भाििाओो क परकट

करती ह

उपयणकत कथि ो म स कौि-सास सही हह

(a) किि 1

(b) किि 2

(c) 1 और 2 द ि ो

(d) ि त 1 ि ही 2

Q63) निमननिखित कथि ो पर निचार कीनजए

1 मलला शाह बदखशी दारा नशक ह क

आधयाखतमक गर थ

2 औरोगरब ि मजम-उि-बहरीि या द समदर ो

का सोगम िामक उललििीय रचिा नििी थी

3 दारा नशक ह क अपि पिणज अकबर क गर ो

क उततरानरकारी क रप म दिा गया था

नजसम उसि रानमणक बहििाद और समनवयता

क बढ़ािा नदया था

उपयणकत कथि ो म स कौि-सास सही हह

(a) किि 1 और 3

(b) किि 2

(c) किि 1 और 2

(d) 1 2 और 3

RAUSIAS-FC19E1003 23

Q60) Consider the following statements about

the Tribal Cooperative Marketing

Development Federation of India

(TRIFED)

1 It is a national-level apex

organization functioning under the

administrative control of Ministry

of Home Affairs Government of

India

2 The main objective of TRIFED is

socio-economic development of

tribal people in the country

Which of the statements given above

isare correct

(a) 1 only

(b) 2 only

(c) Both 1 and 2

(d) Neither 1 nor 2

Q61) Which of the following novels isare

written by Premchand

1 Rangabhumi

2 Godan

3 Gora

Select the correct answer using the code

given below

(a) 1 only

(b) 2 only

(c) 1 and 2 only

(d) 1 2 and 3

Q62) Consider the following statements about

Giddha dance

1 Giddha is a traditional pastoral

dance performed by the women of

Bihar at festival times and at the

sowing and reaping of the harvest

2 By this dance the women reveal

their joy expel their suppressed

feelings in a male dominated

society through the performance of

Giddha

Which of the statements given above

isare correct

(a) 1 only

(b) 2 only

(c) Both 1 and 2

(d) Neither 1 nor 2

Q63) Consider the following statements

1 Mullah Shah Badakhshi was the

spiritual mentor of Dara Shukoh

2 Aurangzeb wrote the remarkable

work called ldquoMajma-ul-Bahrainrdquo or

the ldquoThe confluence of two seasrdquo

3 Dara Shukoh was seen as

inheriting the qualities of his

ancestor Akbar in that he

promoted religious pluralism and

syncretism

Which of the statements given above

isare correct

(a) 1 and 3 only

(b) 2 only

(c) 1 and 2 only

(d) 1 2 and 3

RAUSIAS-FC19E1003 24

Q64) निमननिखित कथि ो पर निचार कीनजए

1 ग मतशवर परनतमा निोधयनगरी पहाड़ी पर खसथत ह

2 शरिरबिग िा िह सथाि ह जहाो मौयण िोश क

सोसथापक चोदरगपत मौयण अपि नसोहासि क

तयागि क बाद जि तपसवी बि गए थ

उपयणकत कथि ो म स कौि-सास सही हह

(a) किि 1

(b) किि 2

(c) 1 और 2 द ि ो

(d) ि त 1 ि ही 2

Q65) निमननिखित कथि ो पर निचार कीनजए

1 पराताखतवक साकषय स पता चिता ह नक पराची

घाटी सभयता हड़पपा और म हिज दाड़ द ि ो

की पिणिती ह

2 पराची िदी भििशवर स निकिती ह

उपयणकत कथि ो म स कौि-सास सही हह

(a) किि 1

(b) किि 2

(c) 1 और 2 द ि ो

(d) ि त 1 ि ही 2

Q66) निमननिखित कथि ो म स कौि-सास सही हह

1 िजराह क समारक ो क समह का निमाणर

चोदि राजिोश क शासिकाि क दौराि हआ

था

2 य समारक हररिोदर पिणत शरोििा म खसथत ह

3 म रक क यातरी इबन बतता ि अपि सोसमरर ो

म िजराह क मोनदर ो की यातरा का उललि

नकया था तथा इन काजराण िाम स समब नरत

नकया था

िीच नदए गए कट का परय ग कर सही उततर चनिए

(a) किि 1

(b) किि 1 और 2

(c) किि 2 और 3

(d) किि 1 और 3

Q67) निमननिखित कथि ो म स कौि-सास सही हह

1 डॉ बी आर अमबडकर ि दी एनिनहिशि

ऑफ़ कासट (The Annihilation of Caste)

नििी थी नजसम उन ोि नहोद रमण म िोशािगत

पजारी की परथा क उनमिि की आिशयकता

पर बि नदया था

2 डॉ राजदर परसाद ि थॉटस ऑि पानकसताि

(Thoughts on Pakistan) िामक पसतक

नििी थी

िीच नदए गए कट का परय ग कर सही उततर चनिए

(a) किि 1

(b) किि 2

(c) 1 और 2 द ि ो

(d) ि त 1 ि ही 2

Q68) निमननिखित कथि ो म स कौि-सास सही हह

1 महरगढ़ भारतीय उपमहादवीप म एक परनसदध

ििपािार बसती ह ज नसोर पराोत पानकसताि म

खसथत ह

2 बरणह म म कतत ो क उिक सवामी क साथ कबर ो

म दफिाया जाता था

िीच नदए गए कट का परय ग कर सही उततर चनिए

(a) किि 1

(b) किि 2

(c) 1 और 2 द ि ो

(d) ि त 1 ि ही 2

Q69) निमननिखित कथि ो म स कौि-सास सही हह

1 काकानटय मोनदर अनरकतर नशि क समनपणत

2 हिमक ोडा म हजार-सतोभ िाि मोनदर (The

Thousand-Pillared Temple) का निमाणर

काकानटय समराट रदर ि करिाया था

िीच नदए गए कट का परय ग कर सही उततर चनिए

(a) किि 1

(b) किि 2

(c) 1 और 2 द ि ो

(d) ि त 1 ि ही 2

RAUSIAS-FC19E1003 25

Q64) Consider the following statements

1 Gommateshwara Statue is located

on the Vindyagiri Hill

2 Shravanabelagola is the place

where Chandragupta Maurya the

founder of the Mauryan dynasty

became a Jain ascetic after

relinquishing his throne

Which of the statements given above

isare correct

(a) 1 only

(b) 2 only

(c) Both 1 and 2

(d) Neither 1 nor 2

Q65) Consider the following statements

1 Archaeological evidence shows

that the Prachi Valley Civilisation

predates both Harappa and

Mohenjo-Daro

2 The Prachi river originates from

Bhubaneswar

Which of the statements given above

isare correct

(a) 1 only

(b) 2 only

(c) Both 1 and 2

(d) Neither 1 nor 2

Q66) Which of the following statements

isare correct

1 The Khajuraho group of

monuments was built during the

rule of the Chandela dynasty

2 These monuments are located in

Harischandra mountain range

3 Ibn Battuta the Moroccan

traveller in his memoirs mentioned

visiting Khajuraho temples and

called them Kajarra

Select the correct answer using the code

given below

(a) 1 only

(b) 1 and 2

(c) 2 and 3

(d) 1 and 3

Q67) Which of the following statements

isare correct

1 Dr BR Ambedkar wrote the

Annihilation of Caste emphasising

the need to do away with the

practice of hereditary priesthood in

Hinduism

2 The book lsquoThoughts on Pakistanrsquo

was written by Dr Rajendra

Prasad

Select the correct answer using the code

given below

(a) 1 only

(b) 2 only

(c) Both 1 and 2

(d) Neither 1 nor 2

Q68) Which of the following statements

isare correct

1 Mehrgarh is a famous Neolithic

settlement in the Indian

subcontinent which is situated in

Sindh province Pakistan

2 At Burzahom dogs were buried

with their masters in their graves

Select the correct answer using the code

given below

(a) 1 only

(b) 2 only

(c) Both 1 and 2

(d) Neither 1 nor 2

Q69) Which of the following statements

isare correct

1 The Kakatiya temples are

dedicated mostly to Siva

2 The Thousand-Pillared Temple at

Hanamkonda was built by the

Kakatiya king Rudra

Select the correct answer using the code

given below

(a) 1 only

(b) 2 only

(c) Both 1 and 2

(d) Neither 1 nor 2

RAUSIAS-FC19E1003 26

Q70) निमननिखित कथि ो म स कौि-सास सही हह

1 अहमदाबाद नमि हड़ताि क दौराि महातमा

गाोरी ि शरनमक ो क पकष क मजबत करि क

निए आमरर अिशि नकया था

2 अिशि स नमि मानिक ो पर दबाि पड़ा था ज

अोततः शरनमक ो क िति म 15 परनतशत की िखदध

करि क निए सहमत हए थ

िीच नदए गए कट का परय ग कर सही उततर चनिए

(a) किि 1

(b) किि 2

(c) 1 और 2 द ि ो

(d) ि त 1 ि ही 2

Q71) निमननिखित म स नकसक नकिक भारत स यिसक

की माििता की अमतण साोसकनतक निरासत की

परनतनिनर सची (The UNESCOrsquos List of the

Representative List of the Intangible

Cultural Heritage of Humanity) म शानमि

नकया गया ह

1 मनडयटट

2 सोकीतणि

3 को भ मिा

िीच नदए गए कट का परय ग कर सही उततर चनिए

(a) किि 1 और 2

(b) किि 2 और 3

(c) किि 3

(d) 1 2 और 3

Q72) निमननिखित जिजानतय ो म स कौि-सीसी ो

जिजानतजिजानतया िागािड स सोबोनरत हह

1 अोगामी

2 ककी

3 जारिा

िीच नदए गए कट का परय ग कर सही उततर चनिए

(a) किि 1

(b) किि 1 औऔ 2

(c) किि 2

(d) 1 2 और 3

Q73) निमननिखित कथि ो म स कौि-सास सही हह

1 राषटर कट सामराजय की सथापिा दोनतदगण ि की थी

नजसि मानयाित म अपिी राजरािी की

सथापिा की थी

2 राषटर कट समराट अम घििण एक ििक था और

उस कनिताओो पर पहिी कननड़ पसतक नििि

का शरय नदया जाता ह

िीच नदए गए कट का परय ग कर सही उततर चनिए

(a) किि 1

(b) किि 2

(c) 1 और 2 द ि ो

(d) ि त 1 ि ही 2

Q74) निमननिखित कथि ो म स कौि-सास सही हह

1 कशब चोदर सि ि ततवब नरिी सभा की

अधयकषता की थी ज आधयाखतमक सतय की

ि ज म सोिि थी

2 बरहम समाज ि मािि गररमा पर बि नदया

मनतणपजा का निर र नकया और सती परथा जसी

सामानजक बराइय ो की आि चिा की

िीच नदए गए कट का परय ग कर सही उततर चनिए

(a) किि 1

(b) किि 2

(c) 1 और 2 द ि ो

(d) ि त 1 ि ही 2

Q75) निमननिखित कथि ो म स कौि-सास सही हह

1 भारत म नचशती नसिनसिा खवाजा म इिददीि

नचशती क दवारा सथानपत नकया गया था

2 नचशती परोपरा की एक परमि निशिता

आतमसोयम थी नजसम साोसाररक म ह स दरी

बिाए रििा शानमि था

िीच नदए गए कट का परय ग कर सही उततर चनिए

(a) किि 1

(b) किि 2

(c) 1 और 2 द ि ो

(d) ि त 1 ि ही 2

RAUSIAS-FC19E1003 27

Q70) Which of the following statements

isare correct

1 During the Ahmedabad Mill Strike

Mahatma Gandhi undertook a fast

unto death to strengthen the

workersrsquo resolve

2 The fast had effect of putting

pressure on mill owners who

finally agreed to give the workers a

15 per cent increase in wages

Select the correct answer using the code

given below

(a) 1 only

(b) 2 only

(c) Both 1 and 2

(d) Neither 1 nor 2

Q71) Which of the following are included in

the UNESCOrsquos list of the representative

list of the intangible cultural heritage of

humanity from India

1 Mudiyettu

2 Sankirtana

3 Kumbh Mela

Select the correct answer using the code

given below

(a) 1 and 2 only

(b) 2 and 3 only

(c) 3 only

(d) 1 2 and 3

Q72) Which of the following tribes isare

related to Nagaland

1 Angami

2 Kuki

3 Jarawa

Select the correct answer using the code

given below

(a) 1 only

(b) 1 and 2 only

(c) 2 only

(d) 1 2 and 3

Q73) Which of the following statements

isare correct

1 Rashtrakuta kingdom was founded by Dantidurga who established his capital at Manyakhet

2 Amoghavarsha a Rashtrakuta king was an author and is credited with writing the first

Kannada book on poetics

Select the correct answer using the code given below

(a) 1 only

(b) 2 only

(c) Both 1 and 2

(d) Neither 1 nor 2

Q74) Which of the following statements isare correct

1 Keshab Chandra Sen headed the Tattvabodhini Sabha which was engaged in search of spiritual truth

2 The Brahmo Samaj laid emphasis on human dignity opposed idolatry and criticized such social

evils as the practice of Sati

Select the correct answer using the code given below

(a) 1 only

(b) 2 only

(c) Both 1 and 2

(d) Neither 1 nor 2

Q75) Which of the following statements isare correct

1 The Chishti order was established in India by Khwaja Moinuddin

Chishti

2 A major feature of the Chishti tradition was austerity including maintaining a distance from the

worldly power

Select the correct answer using the code

given below

(a) 1 only

(b) 2 only

(c) Both 1 and 2

(d) Neither 1 nor 2

T e s t i s p a r t o f R a u rsquo s I A S T e s t s e r i e s f o r P r e l i m i n a r y E x a m 2 0 1 9

FOUNDATION + CURRENT AFFAIRS

GENERAL STUDIES (PAPER ndashI)

FOUNDATION TEST ndashIII

SUBJECT NCERT History Class VI-X + Current Affairs

Time Allowed 1frac12 Hours Maximum Marks 150

I NSTRUCT IONS

1 IMMEDIATELY AFTER THE COMMENCEMENT OF THE EXAMINATION YOU SHOULD CHECK

THAT THIS TEST BOOKLET DOES NOT HAVE ANY UNPRINTED OR TORN or MISSING PAGES OR

ITEMS ETC IF SO GET IT REPLACED BY A COMPLETE TEST BOOKLET

2 This Test Booklet contains 75 items (questions) Each item is printed both in Hindi and English

Each item comprises four responses (answers) You will select the response which you want to mark

on the Answer Sheet In case you feel that there is more than one correct response mark the

response which you consider the best In any case choose ONLY ONE response for each item

3 You have to mark all your responses ONLY on the separate Answer Sheet (OMR sheet) provided

Read the directions in the Answer Sheet

4 All items carry equal marks

5 Before you proceed to mark in the Answer Sheet the response to various items in the Test booklet

you have to fill in some particulars in the Answer Sheet as per instructions contained therein

6 After you have completed filling in all your responses on the Answer Sheet and the examination has

concluded you should hand over to the Invigilator only the Answer Sheet You are permitted to

take away with you the Test Booklet

7 Penalty for wrong answers

THERE WILL BE PENALTY FOR WRONG ANSWERS MARKED BY A CANDIDATE IN THE

OBJECTIVE TYPE QUESTION PAPERS

(i) There are four alternatives for the answer to every question For each question for which a

wrong answer has been given by the candidate one-third of the marks assigned to that

question will be deducted as penalty

(ii) If a candidate gives more than one answer it will be treated as a wrong answer even if one of

the given answers happens to be correct and there will be same penalty as above to that

question

(iii) If a question is left blank ie no answer is given by the candidate there will be no penalty for

that question

T h i s t e s t i s p a r t o f R a u rsquo s I A S T e s t s e r i e s f o r P r e l i m i n a r y E x a m 2 0 1 9

Test Code

FC19E1003

FC19H1003 29

Answers and Explanations of

NCERT History Class VI-X + Current Affairs (FC19E1003)

Q1) उततर (c)

सपषटीकरण

- ऋगवद म दविय ो और दिताओो क समवपित एक

हजार स अविक सत तर (शल क) ह

- य शल क ऋविय ो क दवारा रच गए थ और परि ो

दवारा सीख जात थ

- हालाोवक कछ शल क मवहलाओो (जस वक अपाला

घ सा ल पामदरा मतरयी और गागी) क दवारा भी रच

गए थ

- ऋगवद म सोिाद क रप म कई शल क मौजद ह

- हम विशवावमतर नामक एक ऋवि और दविय ो क

रप म पजी जान िाली द नवदय ो (वयास और

सतलज) क बीच िाताि का उदाहरण वमलता ह

- इसस पता चलता ह वक विशवावमतर िवदक काल स

सोबोवित थ

Q2) उततर (b)

सपषटीकरण

- करनल गफाओो स राख क अिशि परापत हए ह

ज इस ओर सोकत करत ह वक ततकालीन ल ग

अवि क उपय ग स पररवचत थ

- य गफाएो आोधर परदश म सथथत ह

Q3) उततर (c)

सपषटीकरण

bull बरािह म ितिमान कशमीर म सथथत एक

परागवतहावसक थथल ह जहाो ल ग गडढ क घर ो का

वनमािण करत थ

bull य घर जमीन क ख द कर बनाए जात थ तथा नीच

जान क वलए सीवियाा ह ती थी

bull ऐसा अनमान लगाया जाता ह वक य घर ठो ड क

मौसम म आशरय परदान करत थ

Q4) उततर (c)

सपषटीकरण

bull परालख-विदया (Epigraphy) क वशलालख ो क

अधययन क रप म पररभावित वकया जाता ह

bull हसतवलसखत दसतािज ो क माधयम स इवतहास

और सावहतय क अधययन क पाोडवलवप विजञान

(Manuscriptology) कहत ह

bull पराचीन लखन परणावलय ो क अधययन और

ऐवतहावसक पाोडवलवपय ो क समझन तथा वतवथ

वनिािरण क पलीओगराफी (Palaeography) कहा

जाता ह

bull नयवमजमविकस (Numismatics) वसक ो क

अधययन क सोदवभित करता ह

Q5) उततर (a)

सपषटीकरण

- चरक सोवहता चरक क दवारा वलखी गई आयिद

और िदयक-शासर पर एक महतवपणि पसतक ह

- ि भारतीय िदयक-शासर की पारमपररक परणाली

वजस आयिद क नाम स जाना जाता ह क

अभयासकताि थ

- ऐसा माना जाता ह वक चरक का विकास दसरी

शताबदी (ईसा पिि) और दसरी शताबदी (ईसवी) क

मधय हआ था

Q6) उततर (b)

सपषटीकरण

- भाग फसल ो पर वलए जान िाल कर क सोदवभित

करता ह ज कल फसल उतपादन का 16 िाो भाग

था

- ldquoकममकारrdquo शबद भवमहीन कवि शरवमक िगि क

वलए परय ग वकया जाता था

- ldquoअशवमिrdquo (वजस घ ड क बवलदान क रप म भी

जाना जाता ह) एक अनषठान ह ता था वजसम एक

घ ड क सवतोतर रप स घमन क वलए छ ड वदया

FC19H1003 30

जाता ह और राजा क सवनक उसकी रखिाली

करत थ

Q7) उततर (d)

सपषटीकरण

- ऋगववदक काल म घ ड ो क रथ ो म ज ता जाता था

ज (रथ) भवम मिवशय ो आवद पर कबजा करन क

वलए लड गए यद ो म उपय ग वकए जात थ

- इसस यह पता चलता ह वक घ ड ो यकत रथ ो का

उपय ग महाजनपद काल स काफी पहल आरमभ

हआ था

- ऋगववदक काल म मिवशय ो भवम जल आवद पर

कबजा करन क वलए तथा ल ग ो क पकडन क

वलए यद वकय जात थ

- अविकाोश परि इन यद ो म भाग वलया करत थ

- हालाोवक उस समय क ई वनयवमत सना नही ो ह ती

थी लवकन उस काल म सभाऐो ह ती थी ो वजनम

ल ग यद क मामल ो पर चचाि करत थ

- वनयवमत सनाएा महाजनपद काल का िवशषटय थी

वजनम पदल सवनक ो की विशाल सनाएा रथ तथा

हाथी शावमल ह त थ

Q8) उततर (a)

सपषटीकरण

- बद शाकय कल स सोबोवित थ और कशीनारा म

उनका वनिन हआ था

- बद न अपनी वशकषाएा पराकत भािा म दी थी ो ज

आम ल ग ो की भािा थी

Q9) उततर (c)

सपषटीकरण

- पराचीन भारत म दशिनशासर की छह शाखाएा थी ो

िशविक नयाय समखया य ग पिि वममाोसा और

िदाोत या उततर वममाोसा

- इनकी थथापना करमश कनाद गौतम कवपल

पतोजवल जावमनी और वयास ऋविय ो न की थी

Q10) उततर (b)

सपषटीकरण

महािीर की वशकषाऐो छठी शताबदी म िललभी म

सोकवलत की गई थी ो

Q11) उततर (c)

सपषटीकरण

- पारमपररक रप स चाणकय क कौविलय अथिा

विषणगपत क नाम स जाना जाता ह

- उसन अथिशासतर ज एक पराचीन भारतीय

राजनवतक आलख ह वलखा था

Q12) उततर (d)

सपषटीकरण

- भारत का राषटर ीय वचनह सारनाथ (उततर परदश) क

अश क सतमभ क ऊपर (शीिि पर) वसोह कवपिल

का एक अनरपण ह

- इस राषटर ीय वसदाोत सतयमि जयत क साथ

सोय वजत वकया गया ह

- रामपिि बल का नाम रामपिि (वबहार) क नाम पर

पडा जहाा इसकी ख ज हई थी

- यह अपन नाजक नकाशी मॉडल क वलए परवसदद

ह वजसम क मल तवचा सोिदनशील नथन ो सतकि

कान और मरबत िााग ो क शरषठतर परवतरप क

परदवशित वकया गया ह

- यह भारतीय और फारसी ततव ो का एक ससममशरण

- सोवकससा उततर परदश म सथथत ह

Q13) उततर (a)

सपषटीकरण

का िर वसोह ज एक महान य दा थ वबहार स

सोबोवित थ

Q14) उततर (b)

सपषटीकरण

िललालर शबद बड भ-सवावमय ो क वलए परय ग

वकया जाता था

FC19H1003 31

Q15) उततर (c)

सपषटीकरण

- अररकमड एक तिीय बसती थी जहाो दर दश ो स

आन िाल जहाज ो का माल उतारा जाता था

- यहाो पर ईोि ो का एक विशाल ग दाम वमटटी क

बतिन (वजनम एमफ रा - द हरी मवठय ो का लोबा

घडा - शावमल ह) और एरिाइन (Arretine)

मदभाोड पाए गए थ

- इस थथान पर र मन दीपक काोच क बन पातर और

रतन भी पाए गए थ

Q16) उततर (a)

सपषटीकरण

- मिनदर सोगम कविताओो म उसललसखत एक

तवमल शबद ह वजसका अथि ह ldquoतीन परमखrdquo

- यह तीन सततारि पररिार ो क मसखयाओो क वलए

परय ग वकया जाता ह च ल चर और पाणडय

Q17) उततर (c)

सपषटीकरण

- ऋग िद म सभा विदाथा तथा गण जसी

जनजावतय ो पर अथिा किोब पर आिाररत

सभाओो का उललख ह

- आरसमभक िवदक काल म सभाओो और सवमवतय ो

का विशि महतव ह ता था

- यहाा तक की मसखया अथिा राजा भी उनका

समथिन परापत करन क वलए आतर रहत थ

Q18) उततर (a)

सपषटीकरण

- जन िमि न ईशवर क अससततव क मानयता त दी ह

वकनत उसन ईशवर क वजना क पद स नीच रखा

- जन िमि न बौद िमि की तरह िणि परणाली की

भरतिना नही ो की थी

Q19) उततर (d)

सपषटीकरण

- च ल ो और पाणडय ो न शसकतशाली तिीय शहर ो का

विकास वकया था

- च ल ो का सबस महतवपणि शहर पहार (या

कािरीपटटीनम) था |

- मदरई पाणडय ो की राजिानी थी

Q20) उततर (b)

सपषटीकरण

- ldquoबदचररतrdquo बद का जीिन-ितताोत ह

- इस अशवघ ि क दवारा वलखा गया था

Q21) उततर (a)

सपषटीकरणः

- तवमल कवि अपपर भगिान वशि क भकत थ

- इस परकार ि एक नयनार सोत थ

Q22) उततर (d)

सपषटीकरणः

- समदरगपत एक परवसद गपत शासक था

- उसन वसक ो पर िीणा बजात हए अपनी छवि

अोवकत करिाई थी

- यह सोगीत क परवत उसक परम क दशािता ह

- हम उसकी इलाहाबाद परशससत स महतवपणि

ऐवतहावसक जानकारी वमलती ह वजसकी रचना

उसक दरबार क कवि हररसन न की थी

Q23) उततर (b)

सपषटीकरणः

- विकरम सोित की शरआत ििि 58 ईसा पिि म

चनदरगपत वदवतीय न की थी

- यह शक ो पर उसकी जीत और उस विकरमावदतय

की पदिी वमलन क उपलकषय म आरमभ वकया गया

था

FC19H1003 32

- बानभटट न हिििििन का जीिन-ितताोत हििचररत

(ज सोसकत म थी) वलखी थी

Q24) उततर (c)

सपषटीकरणः

- सोवि-विगरावहका यद एिो शाोवत का मोतरी

- साथििाह वयापाररय ो क कावफल ो का नता

Q25) उततर (a)

सपषटीकरणः

- जआन झाोग (हसआन रताोग ndash Hsuang Tsang)

एक चीनी यातरी था ज हिििििन क शासनकाल म

भारत आया था

- ििि 630 ईसवी स ज दशक आरमभ हआ था उसम

जआन झाोग मधय एवशया ईरान और

अफग़ावनसतान की यातरा करन क पशचात कशमीर

क रासत स भारत आया था

- उसन उततर स पिि तक की यातरा की और िह

लगभग 2 ििि वबहार म रहा

- जआन झाोग न नालनदा विशवविदयालय म विदयावथिय ो

और विदवान ो क साथ पारसपररक विचार-विमशि

वकया थथानीय भािाओ ा म वनपणता परापत की तथा

बौद सतप ो की ख ज की

Q26) उततर (c)

सपषटीकरणः

- परदवकषणा पथ बौद िासतकला म सतप क चार ो

ओर बनाया जान िाला एक घमािदार पथ ह ता

- परशन म वदए गए बाकी क तीन ो ततव वहोद मसनदर ो की

िासतकला क भाग ह

Q27) उततर (d)

सपषटीकरणः

परशन म वदए गए सभी मोवदर ो म वयापक रप स

ईोि ो (पकी ईोि ो) का परय ग पतथर ो क साथ हआ

Q28) उततर (c)

सपषटीकरण

- महममद कली कतब शाह ग लकणडा का सलतान

था

- िह अकबर का समकालीन था

- सावहतय और िासतकला म उसकी अतयाविक

रवच थी

- िह एक महान कवि था

- िह दसखनी उदि फारसी और तलग म वलखता था

- उसन अपन पीछ एक विसतत वदिान (सोगरह)

छ डा ह

- अभी हाल ही म तलोगाना म ग लकणडा क वकल

क अनदर खदाई वकय गए बाग-ए-नाया वकला

बाग क चार ो ओर रप-रखा क मानवचतरण क

वलए भारतीय परातासतवक सिकषण (The

Archaeological Survey of India ndash ASI)

गराउणड पनीिर विोग रडार (Ground Penetrating

Radar) का परय ग करगा

Q29) उततर (a)

सपषटीकरणः

- वसलपपावदकारम एक तवमल महाकावय ह वजसकी

रचना इलाोग क दवारा लगभग 1800 ििि पिि की

गई थी

- यह क िलन नामक एक वयापारी की कहानी ह

ज माििी नामक एक गवणका (िशया) स परम

करन लगा था

- मवनमकलाई क िलन और माििी की पतरी की

कहानी ह

Q30) उततर (a)

सपषटीकरण

- चरक आयिद और वचवकरता की एक महतवपणि

रचना चरक सोवहता क लखक ह

- बरहमगपत क अपनी रचना बरहम-सफि-वसदानत

(ज एक खग लीय रचना ह) क कारण परवससद

वमली

FC19H1003 33

- बगदाद म इसका अनिाद अरबी भािा म वकया

गया था

- इसका इसलावमक गवणत और खग ल-विजञान पर

महतवपणि परभाि पडा था

- बाद म अपन जीिनकाल म बरहमगपत न

ldquoखोडखयाकrdquo वलखी ज एक खग लीय पससतका

(एक छ िी पसतक) थी

- इसम आयिभटट की अिि-रावतर क परतयक वदन की

शरआत परणाली का परय ग वकया गया था

Q31) उततर (c)

सपषटीकरण

- अमीर खसर एक परवसद सफी सोगीतकार कवि

और विदवान थ

- 1318 म उनह ोन पाया वक इस भवम (वहोदसतान) क

हर कषतर म अलग-अलग भािा थी लाहौरी

कशमीरी दवारसमदरी (दवकषणी कनाििक म)

तलोगाना (आोधर परदश म) गजरी (गजरात म)

माबारी (तवमलनाड म ) अििी (पिी उततर परदश

म) और वहोदिी (वदलली क आस-पास क कषतर म)

आवद

- उनह न यह बताया वक सोसकत वकसी भी कषतर स

सोबोवित नही ो थी और किल बराहमण ही इस भािा

का जञान रखत थ

Q32) उततर (c)

सपषटीकरण

- वहरणय-गभि सववणिम गभि क सोदवभित करता ह

- जब बराहमण ो की सहायता स यह अनषठान वकया

जाता था त यह माना जाता था वक बवल दन िाल

का कषवतरय क रप म पनजिनम ह गा

Q33) उततर (d)

सपषटीकरण

- कदमई भवम राजसव पर कर क सोदवभित करता

- गवावलयर परशससत म नागभि क दवारा वकय गए

श िण का िणिन वकया गया ह |

- नागभि एक परवतहार राजा था

Q34) उततर (b)

सपषटीकरण

- राजतरो वगनी 12िी ो शताबदी म कलहन क दवारा

रवचत एक सोसकत पसतक (िकसट) ह

- यह परारसमभक भारत की ऐवतहावसक इवतितत थी

- तकि सोगत रप स इस अपन परकार की सिोततम

और सिािविक विशवसनीय कवत माना जाता ह

- यह कशमीर कषतर क पराचीनतम समय स लकर

उसकी रचना की तारीख तक क समपणि इवतहास

का आचछादन करती ह

Q35) उततर (c)

सपषटीकरण

- गााि की आम सभा क ldquoउरrdquo कहा जाता था

- ldquoउरrdquo म गााि क सभी कर दन िाल वनिासी

शावमल ह त थ

Q36) उततर (a)

सपषटीकरण

- वदलली सलतनत म ldquoतारीखrdquo इवतहास लखन का

एक रप था

- ldquoतािरीखrdquo क लखक विदवान परि ह त थ वजनम

सवचि परशासक इतयावद शावमल थ

Q37) उततर (a)

सपषटीकरण

- अलाउददीन सखलजी अपन सवनक ो क ितन का

भगतान नकद म करता था न वक इकता क रप

- सवनक अपना सामान वदलली म वयापाररय ो स

खरीदत थ अतः इस बात का भय था वक वयापारी

कही ो िसतओो का मलय न बिा द

- इसकी र कथाम क वलए अलाउददीन सखलजी न

वदलली म कीमत ो क वनयसित वकया

FC19H1003 34

- अविकारीगण धयानपििक मलय ो का सिकषण करत

थ तथा ज वयापारी वनिािररत मलय पर माल नही ो

बचत थ उनक दसणडत वकया जाता था

Q38) उततर (d)

सपषटीकरण

- वदलली सििपरथम त मर राजपत ो क अिीन उनक

सामराजय की राजिानी बनी थी

- 12िी ो शताबदी क मधय म अजमर क चौहान ो

(वजनह चाहमान ो क नाम स भी जाना जाता ह) न

त मर राजपत ो क परावजत वकया था

- त मर ो और चौहान ो क अिीन वदलली एक

महतवपणि िावणसजयक क दर बन गया था

- कई जन वयापारी यहाा रहन लग थ और उनह ोन

कई मोवदर भी बनिाए

- यहाा पर मवदरत वसक वजनह ldquoदहलीिालrdquo क नाम

स जाना जाता था वयापक रप स परचलन म थ

Q39) उततर (c)

सपषटीकरण

- म ठ की मसिद का वनमािण वसको दर ल दी क

राजयकाल म उसक मिी क दवारा करिाया गया

था

- बगमपरी मसिद का वनमािण महममद तगलक क

शासनकाल म हआ था

- यह मसिद विशव का पणयथथान (The

Sanctuary of the World) और वदलली म महममद

तगलक की नई राजिानी जहाोपनाह की मखय

मसिद थी

- कववत- अल - इसलाम मसिद का विसतार

इलतसिश और अलाउददीन सखलजी न वकया था

- मीनार का वनमािण तीन सलतान ो कतबददीन ऐबक

इलतसिश और वफर ज शाह तगलक क दवारा

करिाया गया था

Q40) उततर (c)

सपषटीकरण

- मगल ो क अिीन मनसबदार शबद उस वयसकत क

वलए सोदवभित वकया जाता था वजसक पास मनसब

(अथाित पद) ह ता था

- उस अपना ितन राजसव कायो वजनह जागीर कहत

थ क रप म परापत ह ता था

Q41) उततर (b)

सपषटीकरण

- ldquoभारत छ ड आोद लनrdquo वबरविश शासन क

सखलाफ ल ग ो का एक सवाभाविक विदर ह था

- असखल भारतीय काोगरस सवमवत न 8 अगसत 1942

क बमबई म एक बठक का आय जन वकया था

- इस बठक म परवसद सोकलप ldquoभारत छ ड rdquo क

पाररत वकया गया और इस उददशय क परापत करन

क वलए गाोिी क नततव म एक अवहोसक जन सोघिि

आोद लन की शरआत का परसताि वदया गया

- लवकन अगल ही वदन गाोिी और काोगरस क अनय

परमख नताओो क वगरफतार कर वलया गया

- काोगरस क एक बार वफर अिि घ वित वकया गया

था

Q42) उततर (c)

सपषटीकरण

- साइमन कमीशन यनाइविड वको गडम क सात

साोसद ो का एक समह था

- इस वबरविश भारत क वलए सोििावनक सिार ो का

सझाि दन क वलए गवठत वकया गया था

- इस आय ग म िररषठ वबरविश राजनता सर जॉन

साइमन क नततव म किल वबरविश सदसय ही

शावमल थ

- इसवलए भारत क ल ग ो न साइमन कमीशन क

आगमन क विरद आोद लन वकया था

Q43) उततर (a)

सपषटीकरण

bull दादा भाई नौर जी भारत म वबरविश शासन क

आवथिक पररणाम ो क बार म अपनी विर िी

(परवतकल) राय क वलए जान जात थ

FC19H1003 35

bull अपन कई लख ो और भािण ो म विशि रप स

ldquoपाििी एो ड अन-वबरविश रल इन इसणडया

(Poverty and Un-British Rule in India) म

नौर जी न यह तकि वदया वक भारत पर अतयविक

कर लगाया गया था और इसकी सोपवतत इोगलड की

ओर परिावहत की जा रही थी

bull उनह ोन पराचीन भारतीय गरोथ ो की वयाखया करन

का और भारतीय ो क आिविशवास क बहाल

करन पर कायि नही ो वकया था

उनह ोन वकसी और बात स पहल सभी सामावजक

बराइय ो क उनमलन की आिशयकता पर भी बल

नही ो वदया था

Q44) उततर (c)

सपषटीकरण

bull अगसत 1932 म वबरविश परिानमोतरी मकड नालड न

अपन साोपरदावयक परसकार (The Communal

Award) की घ िणा की थी

bull यह भारत क कई साोपरदावयक वहत ो क बीच विवभनन

सोघिो क हल करन क वलए वबरिन का एकतरफा

परयास था

bull यह परसकार (Award) बाद म 1935 क

अविवनयम (The Act of 1935) म शावमल वकया

गया था

bull इस साोपरदावयक परसकार न मससलम ो क वलए

आरवकषत एक अलग वनिािचक मणडल फॉमिल का

विसतार अनय अलपसोखयक ो क वलए वकया था

वजसम वसख ो भारतीय ईसाइय ो आोगल-भारतीय

समदाय यर पीय समदाय तथा विवशषट कषतरीय

समह ो क शावमल वकया गया था

bull गाोिी न इस परसताि क भारतीय समाज क

विभावजत करन क वलए एक घवणत वबरविश

सावजश क रप म दखा और उसक सखलाफ

आमरण अनशन वकया

Q45) उततर (b)

सपषटीकरण

मौजदा आयात और वनयाित क अवतररक़त

औपवनिवशक भारत क वनमनवलसखत खचो क

वलए एक विशिवनवशचत िन रावश भी दनी पडती

थी

(i) परशासन क वयय

(ii) सना क रख-रखाि क वयय

(iii) यद क वयय

(iv) सिावनितत अविकाररय ो की पशन तथा

(v) वबरिन दवारा अपनी उपवनिश बसती

(कॉल नी) क रख-रखाि क वयय

इनह गह शलक (Home Charges) क रप म

जाना जाता था और लगभग परी तरह स भारत क

दवारा इनका भगतान वकया जाता था

bull गह शलक म वनमनवलसखत घिक शावमल थ

(i) भारतीय ऋण पर दय बयाज

(ii) ईसट इोवडया को पनी क शयरिारक ो क

लाभाोश

(iii) लोदन म भारत कायािलय चलान क वलए िन

(iv) भारत म वनयकत वबरविश कवमिय ो क ितन

और पशन का भगतान करन क वलए िन

(v) रलि पर बयाज

(vi) नागररक और सनय शलक

(vii) इोगलड म सट र (सामगरी) की खरीद

Q46) उततर (b)

सपषटीकरण

bull भारतीय राषटर ीय काोगरस का लाहौर सतर 1929 म

जिाहरलाल नहर की अधयकषता म आय वजत

वकया गया था

bull इस सतर म भारतीय राषटर ीय आोद लन स समबसित

कई महतवपणि पररणाम सामन आय थ

(i) सििपरथम इस सतर म काोगरस क अधयकष पद

पर जिाहरलाल नहर क चना गया था ज

काोगरस म िामपोवथय ो की बिती हई ताकत

का सपषट सोकत था

(ii) दसरा इस सतर म पहली बार काोगरस न पणि

सवतोतरता की माोग क उठाया था

इस परकार की माोग काोगरस मोच स पहल कभी भी

नही ो उठाई गई थी

Q47) उततर (b)

सपषटीकरण

FC19H1003 36

bull इस ररप िि न वकसी भी समदाय क वलए पथक

वनिािचक मोडल अथिा अलपसोखयक ो क वलए

भाराोश की वसफाररश नही ो की थी

bull तथावप इस ररप िि न उन पराोत ो म अलपसोखयक

सीि ो क आरकषण की अनमवत दी थी जहाा पर कम

स कम दस परवतशत अलपसोखयक ह

bull लवकन यह समदाय क आकार क अनपात म ह ना

चावहए था

bull इस ररप िि म भारत क वलए पणि सवतोतरता क

वलए क ई पराििान नही ो था

Q48) उततर (c)

सपषटीकरण

bull आरो वभक िवदक आयो का िमि मखय रप स

परकवत की पजा और यजञ था

bull परारो वभक आयि िमि परकवत की पजा क समान था

bull िासति म उनक चार ो ओर की शसकतयाा वजनह न

त ि वनयोवतरत कर सकत थ और न ही समझ पाए

थ उनह वदवयता क साथ वनिवशत वकया गया तथा

उनह मादा या नर दिीदिताओो क रप म

परतीकतव वकया गया था

bull उनह ोन कछ यजञ ो का भी वनषपादन वकया था

Q49) उततर (b)

सपषटीकरण

bull सडक और नदी-मागि (जल-मागि) डकती स

सरवकषत नही ो थ

bull उललखनीय ह वक हिििििन क शासनकाल क

दौरान यआन चिाोग (हयएन साोग) का सारा

सामान लि वलया गया था

Q50) उततर (c)

सपषटीकरण

परशन म वदए गए द न ो कथन सही ह

Q51) उततर (b)

सपषटीकरण

bull परोदर दास एक सोत और भगिान कषण क एक

महान भकत थ

bull परोदर दास क कनाििक सोगीत क वपतामह क

रप म जाना जाता ह

bull यदयवप उनक जनम-थथान क बार म काफी

अिकल लगाई जाती रही ह

bull तथावप अब कननड विशवविदयालय हमपी क दवारा

गवठत एक विशिजञ सवमवत इस वनषकिि पर पहोची

ह वक उनका जनम थथान सोभितया कनाििक का

एक छ िा-सा गााि कषमपरा (वशिम गगा वजला)

था

Q52) उततर (c)

सपषटीकरण

bull शरी तयागराज शरी शयाम शासतरी और शरी मथसवामी

दीवकषतर क कनाििक सोगीत की वतरमवति माना

जाता ह

bull उनक कारण ही 18िी ो-19िी ो शताबदी म कनाििक

सोगीत का सववणिम यग आया था

Q53) उततर (d)

सपषटीकरण

bull अभी हाल ही म लौह यगीन-महापािावणक काल

का 2000 ििि पराना एक दलिभ सारक फगस

(Sarcophagus) (पतथर का ताबत) क ललम क

वियर गाोि (क वयलडी क पास वजला क वझक ड

करल राजय) की एक रॉक-कि गफा स ख जा गया

bull यह ताबत वजसम हविय ो क िकड थ खदाई क

दौरान वमला

bull अभी तक इस परकार की दलिभ ख ज करल क

मातर द ही थथान ो स हई ह

bull य द न ो सारक फगी (Sarcophagi) (पतथर क

ताबत) चियर और अथ ली (वजला क वझक ड) क

महापािाण थथल ो स वमल ह

Q54) उततर (a)

सपषटीकरण

FC19H1003 37

दवकषण भारत म महापािाण सोसकवत एक पणि

विकवसत लौह यगीन सोसकवत थी

Q55) उततर (d)

सपषटीकरण

bull च ल पाणडय और करलपतर (चर) इन तीन ो का

उललख अश क क अवभलख ो म वकया गया ह

bull सोभितः य भौवतक सोसकवत क उततर

महापािावणक चरण म थ

Q56) उततर (d)

सपषटीकरण

bull भीमा-क रगाोि की लडाई ततीय आोगल-मराठा

यद का वहससा थी

Q57) उततर (b)

सपषटीकरण

bull राजकमार शकल न गाोिीजी क चोपारण आन तथा

वतनकवथया परणाली स जडी समसया की जाोच क

वलए रारी करन क वलए दश भर म उनका

अनसरण वकया था

bull बज वकश र राजदर परसाद महादि दसाई और

नरहरी पाररख चोपारण सतयागरह क दौरान गाोिी

जी क सहय गी थ

Q58) उततर (b)

सपषटीकरण

bull बराहमण ो और बौद मठिाररय ो क कर-मकत गााि

अनदान म दन की परथा सतिाहन ो न आरमभ की

थी

Q59) उततर (c)

सपषटीकरण

इस कायिकरम क उददशय वनमनानसार ह

(i) बवनयादी पयििन आिाररक सोरचना का विकास

करना

(ii) चयवनत (पहचान वकय गए) कषतर ो म आजीविका क

सजन क वलए दश क साोसकवतक और विरासत

मलय ो क बिािा दना

(iii) विरासत समारक थथल ो पर विशव सतरीय आिाररक

सोरचना विकवसत करक एक सतत तरीक स

पयििक आकििण म िसद करना

(iv) थथानीय समदाय ो की सवकरय भागीदारी क माधयम

स र रगार ो का सजन करना

(v) र रगार उतपादन और आवथिक विकास क वलए

पयििन कषमता का उन पर परभाि का उपय ग

करना तथा

(vi) िारणीय पयििन आिाररक सोरचना का विकास

करना और उसका उवचत सोचालन तथा

रखरखाि सवनवशचत करना

Q60) उततर (b)

सपषटीकरण

bull यह वनकाय ििि 1987 म अससततव म आया था

bull यह एक राषटर ीय सतर का शीिि सोगठन ह ज भारत

सरकार क जनजातीय मामल ो क मोतरालय क

परशासवनक वनयोतरण क अिीन काम कर रहा ह

bull इसका पोजीकत और परिान कायािलय नई वदलली

म सथथत ह

Q61) उततर (c)

सपषटीकरण

bull परमचोद क उपनयास ो म परमाशरम रोगभवम गबन

कमिभवम और ग दान शावमल ह

bull ग रा रिी ोदरनाथ िग र क दवारा रवचत उपनयास ह

bull अभी हाल ही म मोशी परमचोद की 138िी ो जयोती दश

भर म मनाई गई थी

Q62) उततर (b)

सपषटीकरण

bull ldquoवगदाrdquo पोजाब (भारत) एिो पावकसतान की

मवहलाओो क दवारा तयौहार क समय और फसल

की बिाई तथा किाई क अिसर पर वकया जान

िाला एक पारोपररक दहाती नतय ह

FC19H1003 38

bull इस नतय क माधयम स पोजाबी मवहलाऐो अपनी

परसननता परकि करती ह तथा वगदा क परदशिन क

माधयम स परि िचिसव िाल समाज म मवहलाओो

की दबी हई भािनाओो क परकि करती ह

bull चोवक इस नतय का परि ो क साथ क ई सोबोि नही ो

ह अतः किल मवहलाऐो ही इसम भाग ल सकती

bull हर साल तीज समार ह क दौरान पोजाब म वगदा

नतय वकया जाता ह

तीज भारत क कछ भाग ो म मवहलाओो क दवारा

मनाया जान िाल कई तयौहार ो क वलए एक

वयापक नाम ह

Q63) उततर (a)

सपषटीकरण

- मजम-उल-बहरीन या द समदर ो का सोगम

नामक उललखनीय रचना दारा वशक ह क दवारा

वलखी थी

- भारत क उपराषटर पवत शरी एम िकया नायड न कहा

ह वक राजकमार दारा वशक ह की रचनाएा शाोवत

और सदभाि क बिािा दन क वलए एक तारा सर त

क रप म सामन आ सकती ो ह

- उपराषटर पवत गत ििो क भला वदए गए राजकमार

दारा वशक ह क परदवशित परचवलत करन हत

आय वजत एक परदशिनी का दौरा करन क बाद एक

सभा क सोब वित कर रह थ

- इस परदशिनी का आय जन फर क इस गौवियर

(Francois Gautier) क दवारा lsquoइोवदरा गाोिी नशनल

सिर फॉर द आििसrsquo (The Indira Gandhi

National Centre for the Arts) नई वदलली म

वकया गया था

Q64) उततर (c)

सपषटीकरण

- ग मतशवर परवतमा जन भगिान बाहबली क

समवपित ह

- यह एक एक-चटटानी पतथर की मवति ह

- राषटर पवत राम नाथ क विोद न शरिणबलग ला

(कनाििक) म आय वजत वकय जान िाल भवय

अवभिक समार ह महामसतकावभिक का

उदघािन वकया था

- यह समार ह 12 ििो म एक बार ह ता ह

Q65) उततर (c)

सपषटीकरण

bull पराची घािी पराची नदी क चार ो ओर फली हई थी

bull पराची घािी िीर-िीर विलपत ह गई थी

bull पराची नदी भिनशवर स वनकलती ह

bull यह महानदी की एक सहायक नदी ह और यह

परी खदाि किक तथा जगतवसोहपर वजल ो स

ह कर बहती ह

bull इस नदी क पर कषतर क पराची घािी कहा जाता ह

bull यह नदी बोगाल की खाडी म वगरती ह

परातासतवक साकषय स पता चलता ह वक पराची घािी

सभयता हडपपा और म हनज दाड द न ो की

पिििती ह

Q66) उततर (d)

सपषटीकरण

य समारक छतरपर वजल (मधय परदश) म विोधयाचल

पिित शरोखला म सथथत ह

Q67) उततर (a)

सपषटीकरण

bull थॉिस ऑन पावकसतान नामक पसतक डॉ बी

आर अमबडकर न वलखी थी

bull डॉ बी आर अमबडकर की जयोती क अिसर पर

भारत क राषटर पवत न भारत की इस महान हसती

क शरदाोजवल अवपित की थी

bull डॉ बी आर अमबडकर न 1924 म वडपरथड

कलावसर इोसटीटयि (दवलत िगि सोथथान -

बवहषकत वहतकाररणी सभा) और 1927 म समाज

समता सोघ की थथापना की थी

bull अमबडकर का धयान वशकषा कषतर की ओर भी था

bull उनह ोन वशकषा क वनमन िगो म फलान क वलए

पीपलस एजकशन स साइिी (The Peoples

Education Society) क नाम स महाविदयालय ो क

नििकि और छातरािास ो की थथापना की थी

FC19H1003 39

Q68) उततर (b)

सपषटीकरण

bull महरगि भारतीय उपमहादवीप म एक परवसद

निपािाण बसती ह ज बलवचसतान पराोत

पावकसतान म सथथत ह

bull दचपलली (आोधर परदश) क पास नागलर नदी क

पिी ति ो पर चना पतथर क बलॉक क विशाल

विसतार म एक पिि-ऐवतहावसक रॉक आिि थथल की

ख ज की गई ह

bull इसन 1500-2000 ईसा पिि क दौरान गोिर (आोधर

परदश) म विकवसत निपािाण सभयता पर परकाश

डाला ह

Q69) उततर (c)

सपषटीकरण

bull 12िी ो सदी और 13िी ो सदी म काकाविय िोश का

उदय हआ था

bull ि पहल कलयाण क पवशचमी चालकय ो क सामोत थ

bull परारोभ म उनह ोन िारोगल (तलोगाना) क पास एक

छ ि स कषतर पर शासन वकया था

bull उनह ोन ldquoनायक वयिथथाrdquo की शरआत की थी

वजस बाद म विजयनगर क राय शासक ो न

अपनाया और विकवसत वकया था

Q70) उततर (a)

सपषटीकरण

bull गाोिीजी क अनशन स वमल मावलक ो पर दबाि

पडा था ज अोततः शरवमक ो क ितन म 35 परवतशत

की िसद करन क वलए सहमत हए थ

bull गगल (Google) न अनसया साराभाई वजनह ोन

भारत क शरवमक आोद लन म एक अगरणी भवमका

वनभाई थी की 132िी ो जयोती डडल (Doodle) का

वनमािण करक मनाई

Q71) उततर (d)

सपषटीकरण

भारत स यनसक की मानिता की अमति साोसकवतक

विरासत की परवतवनवि सची म वनमनवलसखत शावमल ह

bull कवडयटटम करल का सोसकत रोगमोच

bull मवडयिि करल का अनषठान रोगमोच और नतय

नाविका

bull िवदक मि जाप की परोपरा

bull राजथथान क कालबवलया ल क गीत और नतय

bull रामलीला रामायण का पारोपररक परदशिन

bull सोकीतिन मवणपर का अनषठान गायन ढ ल िादन

और नतय

bull रममन भारत क गििाल वहमालय का िावमिक

तयौहार और अनषठान रोगमोच

bull जाोदीयाला गर पोजाब क ठठर ो की पीतल और

ताोब क वशलप स वनवमित बतिन ो की पारोपररक कला

bull छाऊ नतय पिी भारतीय राजय ो म जनमी शासतरीय

भारतीय नतय कला

bull लददाख का बौद मि जाप िर ाोस-वहमालयी लददाख

कषतर तथा जमम-कशमीर म पवितर बौद गरोथ ो का पाठ

bull य ग

bull नौर र

bull को भ मला

Q72) उततर (b)

सपषटीकरण

bull भारत क राषटर पवत शरी राम नाथ क विोद न

वकसामा नागालड म हॉनिवबल मह रति और

राजय गठन वदिस समार ह का उदघािन वकया

था

bull हॉनिवबल मह रति का नाम भारतीय हॉनिवबल क

नाम पर पडा ह ज एक विशाल और रोगीन जोगली

पकषी ह

bull यह पकषी नागालड राजय की अविकतर जनजावतय ो

की ल ककथाओो म उसललसखत ह

bull नागालड की परमख मानयता परापत जनजावतयाा ह

अोगामी आओ चखसोग चाोग ककी रगमा और

रवलोग आवद

bull ओोग जारिा और ससिनलीस अोडमान-वनक बार

दवीप समह की जनजावतयाा ह

FC19H1003 40

Q73) उततर (c)

सपषटीकरण

bull दकन म राषटर कि शासन दसिी ो सदी क अोत तक

लगभग 200 ििो तक रहा था

bull राषटर कि शासक अपन िावमिक विचार ो म सवहषण

bull उनह ोन न किल शि िमि और िषणि िमि बसलक

जन िमि क भी सोरकषण वदया था

bull एल रा म वशि क परवसद रॉक कि मोवदर का

वनमािण नौिी ो सदी म राषटर कि राजा कषण परथम न

करिाया था

bull उसका उततराविकारी अम घििि जन था लवकन

उसन अनय िमो क भी सोरकषण परदान वकया था

bull राषटर कि ो न मसलमान वयापाररय ो क बसन की

अनमवत दी थी

bull उनह न अपन अविराजय ो म इसलाम क उपदश दन

की भी अनमवत दी थी

bull अभी हाल ही म पाोडिलागटटा (तलोगाना) क

परागवतहावसक चटटान वचतर ो क कषरण की बिती हई

घिनाएा एक गोभीर वचोता का वििय ह

bull यह परागवतहावसक चटटान क नकसान पहाचा

सकता ह

bull पाोडिलागटटा वनमनवलसखत क वलए जाना जाता ह

- 10000 ईसा पिि स 8000 ईसा पिि क वचवतरत

चटटानी आशरय ो क वलए

- राषटर कि काल क एक 8 िी ो सदी क

वशलालख क वलए और

- 12िी ो सदी क काकविय सामराजय क वभवतत

वचतर ो क वलए

Q74) उततर (b)

सपषटीकरण

bull 1828 म राजा राम म हन रॉय न एक नय िावमिक

समाज बरहम सभा की थथापना की थी वजस बाद

म बरहम समाज क नाम स जाना गया था

bull दिदरनाथ िग र न ततवब विनी सभा की अधयकषता

की थी ज आधयासिक सतय की ख ज म सोलि

थी

bull इसका उददशय वहोद िमि क शद करन का और

एकशवरिाद (एक ईशवर म आथथा) का परचार करना

था

bull नय समाज की थथापना क आिार थ कारण

(तकि ) क द सतमभ तथा िद और उपवनिद

bull अभी हाल ही म सािारण बरहम समाज का कछ

काननी मदद ो क लकर पवशचम बोगाल सरकार क

साथ काननी वििाद चल रहा ह

Q75) उततर (c)

सपषटीकरण

bull भारत म वचशती वसलवसल की थथापना खवाजा

म इनददीन वचशती क दवारा की गयी थी

bull ि 1192 ईसवी क आसपास भारत आय थ

bull वचशतीय ो क बारहिी ो शताबदी क उततरािि म भारत

म आन िाल सफीय ो क समह ो म सबस

परभािशाली माना जाता ह

bull उनह ोन थथानीय िातािरण क साथ सफलतापििक

अनकलन वकया और उनह ोन भारतीय भसकत

परोपराओो क कई पहलओो क अपनाया

bull अजमर म सफी अपरकि खवाजा म इनददीन वचशती

की ऐवतहावसक दरगाह क एक नया रप दन की

तयारी की जा रही ह

bull इस 13िी ो शताबदी की दरगाह क ldquoसवचछ

आइकॉवनक थथल ोrdquo (Swacch Iconic Places) म

शावमल वकया गया ह ज परवतवषठत विरासत

आधयासिक और साोसकवतक थथान ो पर क वदरत

य जना ह

FC19H1003 41

ANSWERS amp EXPLANATION OF

NCERT History Class VI-X + Current Affairs

(FC19E1003)

Q1) Answer c

Explanation

Rigveda consists of more than a

thousand hymns dedicated to gods and

goddesses These hymns were

composed by sages and learnt by men

however a few were composed by

women like Apala Ghosa Lopamudra

Maitreyi and Gargi

Rigveda consists of many hymns in the

form of dialogues We get an example of

a dialogue between a sage named

Vishwamitra and two rivers (Beas and

Sutlej) that were worshipped as

goddesses This suggests that he

belonged to the Vedic period

Q2) Answer b

Explanation

Traces of ash have been found from

Kurnool Caves suggesting that people

were familiar with the use of fire

It is situated in Andhra Pradesh

Q3) Answer c

Explanation

Burzahom is a prehistoric site in

present day Kashmir where people built

pit houses which were dug into the

ground with steps leading into them

These may have provided shelter in cold

weather

Q4) Answer c

Explanation

Epigraphy is defined as the study of

inscriptions

Manuscriptology is the study of history

and literature through the use of hand

written documents

Palaeography refers to the study of

ancient writing systems and the

deciphering and dating of historical

manuscripts

Numismatics refers to the study of

coins

Q5) Answer a

Explanation

Charaka Samhita was written by

Charaka and is an important book on

Ayurveda and medicine

He was a practitioner of the traditional

system of Indian medicine known as

Ayurveda

Charaka is thought to have flourished

sometime between the 2nd century BCE

and the 2nd century CE

Q6) Answer b

Explanation

Bhaga refers to the tax on crops which

was fixed at 16th of the production

Kammakaras is the term used for the

landless agricultural labour class

Ashvamedha also known as horse

sacrifice is a ritual where a horse is let

loose to wander freely and it was

guarded by the rajarsquos men

Q7) Answer (d)

Explanation

In the Rigvedic period horses were

yoked to chariots that were used in

battles fought to capture land cattle

etc This suggests that the use of horse

chariots began much before the period

of Mahajanapadas

The battles were fought in the Rigvedic

period for cattlersquos lands water an even

to capture people Most men took part

in these wars however there was no

regular army but there were assemblies

where people met and discussed

matters of war Regular armies became

a feature in the Mjahajanapada period

including vast armies of foot soldiers

chariots and elephants

RAUSIAS-FC19E1003 42

Q8) Answer (a)

Explanation

Buddha belonged to the Sakya clan and

passed away at Kusinara

Buddha taught in Prakrit which was the

common language of people

Q9) Answer c

Explanation

There were six schools of philosophy in

ancient India These are known as

Vaishesika Nyaya Samkhya Yoga

Purva Mimansa and Vedanata or Uttara

Mimansa They were founded by sages

Kanada Gautama Kapila Patanjali

Jamini and Vyasa respectively

Q10) Answer b

Explanation

The teachings of Mahavira were

compiled at Valabhi in 6th century AD

Q11) Answer (c)

Explanation

Chanakya is traditionally identified as

Kautilya or Vishnugupta who authored

the ancient Indian political treatise the

Arthashastra

Q12) Answer d

The national emblem of India is an

adaptation of the Lion Capital atop the

Ashoka Pillar of Sarnath Uttar Pradesh

and is combined with the National

Motto Satyameva Jayate

The Rampurva Bull gets the name from

the site of its discovery Rampurva in

Bihar

It is noted for its delicately sculpted

model demonstrating superior

representation of soft flesh sensitive

nostrils alert ears and strong legs It is

a mixture of Indian and Persian

elements

Sankissa is situated in Uttar Pradesh

India

Q13) Ans(a)

Kunwar Singh was a notable leader during the Revolt of 1857 He belonged

to a royal house of Jagdispur Bihar

Q14) Answer b

Explanation

The term Vellalar was used for large

landowners

Q15) Answer c

Explanation

Arikamedu was a coastal settlement

where ships unloaded goods from

distant lands Finds here include a

massive brick warehouse pottery

including amphorae and Arretine ware

Roman lamps glassware and gems have

also been found at the site

Q16) Answer a

Explanation

Muvendar is a Tamil word mentioned in

Sangam poems meaning three chiefs

used for the heads of three ruling

families the Cholas Cheras and

Pandyas

Q17) Ans (c)

Several tribal or kin-based assemblies

such as the Sabha Vidatha and gana

are mentioned in the Rig-veda The

Sabha and the samiti mattered a great

deal in early Vedic times so much so

that the chiefs or the kings showed an

eagerness to win their support

Q18) Ans (a)

Jainism recognised the existence of the

gods but placed them lower than the

jina and did not condemn the varna

system as Buddhism did

Q19) Answer (d)

Explanation

Cholas and Pandyas had developed

powerful coastal cities The most

important city of Cholas was Puhar or

Kaveripattinam and Madurai was the

capital of Pandyas

Q20) Answer b

Explanation

Buddhacharita is the biography of

Buddha and was written by

RAUSIAS-FC19E1003 43

Ashvaghosha

Q21) Answer (a)

Explanation

Tamil poet Appar was a Shiva devotee

So he was a Nayanar saint

Q22) Answer d

Explanation

Samudragupta was a prominent Gupta

ruler whose coins depict him playing a

veena indicating his love for music We

get important historic information from

his Allahabad Prashasti which was

composed by his court poet Harisena

Q23) Answer (b)

Explanation

Vikrama Samvat was founded by

Chandragupta II in the 58 BC as a

mark of victory over the Shakas and

assumed the title of Vikramaditya

Banabhatta wrote Harshavardhanarsquos

biography the Harshacharita in

Sanskrit

Q24) Answer c

Explanation

Sandhi-vigrahika was the minister of

war and peace

Sarthavaha was the leader of the

merchant caravans

Q25) Answer a

Explanation

Xuan Zang (Hsuan-tsang) was a

Chinese traveller who came during the

reign of Harshavardhana

In the decade that began in 630 AD

Xuan Zang came to India through

Kashmir after visiting Central Asia Iran

and Afghanistan

He travelled from north to east and lived

in Bihar for a couple of years

At Nalanda University Xuan Zang

interacted with students and scholars

mastered local languages and

discovered Buddhist stupas

Q26) Answer c

Explanation

Pradakshina patha is a circular path

laid around a stupa in Buddhist

architecture While the rest are a part of

temple architecture

Q27) Answer d

Explanation

All the above-mentioned temples have

an elaborate use of bricks (baked

bricks) along with stone

Q28) Ans (c)

Muhammad Quli Qutab was the Sultan

of Golconda He was a contemporary of

Akbar was very fond of literature and

architecture

The Sultan was a great poet and he

wrote in Dakhini Urdu Persian and

Telgu and has left an extensive diwan or

collection

Recently the Archaeological Survey of

India (ASI) will be using Ground

Penetrating Radar (GPR) to map the

contours of the area around the Bagh-e-

Naya Qila excavated garden inside the

Golconda Fort in Telangana

Q29) Answer a

Explanation

Silappadikaram is a famous Tamil epic

which was written by Ilango around

1800 years ago It is a story of a

merchant named Kovalan who fell in

love with a courtesan named Madhavi

Manimekalai tells the story of the

daughter of Kovalan and Madhavi

Q30) Answer (a)

Explanation

Charaka is the author of Charaka

Samhita which is an important work of

Ayurveda and medicines

Brahmaguptarsquos fame rests mostly on his

Brahma-sphuta-siddhanta which was

an astronomical work It was translated

into Arabic in Baghdad and had a major

impact on Islamic mathematics and

astronomy

Late in his life Brahmagupta wrote

Khandakhadyaka which was an

RAUSIAS-FC19E1003 44

astronomical handbook that employed

Aryabhatarsquos system of starting each day

at midnight

Q31) Answer (c)

Explanation

Amir Khusrau was a famous sufi

musician poet and scholar In 1318 he

noted that there was different language

in every region of this land (Hindustan)

Lahori Kashmiri Dvarsamudri (in

Southern Karnataka) Telangana (in

Andhra Pradesh) Gujari (in Gujarat)

Marsquobari (in Tamil Nadu) Awadhi (in

eastern Uttar Pradesh) and Hindawai (in

the area around in Delhi) etc He went

to explain that Sanskrit did not belong

to any region and that only brahmans

knew it

Q32) Answer c

Explanation

Hiranyagarbha refers to the golden

womb When this ritual was performed

with the help of Brahmanas it was

thought to lead to the rebirth of the

sacrificer as a Khastriya

Q33) Answer d

Explanation

Kadamai refers to a tax on land

revenue

Gwalior Prashasti describes the exploits

of Nagabhata who was a Pratihara king

Q34) Answer b

Explanation

Rajatarangini is a Sanskrit text written

by Kalhana in the 12th century

It was historical chronicle of early India

It is justifiably considered to be the best

and most authentic work of its kind

It covers the entire span of history in

the Kashmir region from the earliest

times to the date of its composition

Q35) Answer c

Explanation

ldquoUrrdquo was the general assembly of the

village ldquoUrrdquo consisted of all the

taxpaying residents of an ordinary

village

Q36) Answer (a)

Explanation

Tarikh was a form of history writing in

the Delhi Sultanate The authors of

tawarikhs were learned men which

included secretaries administrators etc

Q37 Answer (a)

Explanation

Alauddin chose to pay his soldiers salaries in cash rather than iqtas The soldiers would buy their supplies from merchants in Delhi and it was thus feared that merchants would raise their prices To stop this Alauddin controlled the prices of goods in Delhi Prices were carefully surveyed by officers and merchants who did not sell at the prescribed rates were punished

Q38) Answer (d)

Explanation

Delhi first became the capital of a

kingdom under the Tomara Rajputs

who were defeated in the middle of the

twelfth century by the Chauhans (also

referred to as Chahamanas) of Ajmer

It was under the Tomaras and

Chauhans that Delhi became an

important commercial centre Many rich

Jaina merchants lived in the city and

constructed several temples Coins

minted here called dehliwal had a wide

circulation

Q39) Answer (c)

Explanation

Moth ki Masjid was built in the reign of

Sikandar Lodi by his minister

Begumpuri mosque built in the reign of

Muhammad Tughluq was the main

mosque of Jahanpanah the ldquoSanctuary

of the Worldrdquo and his new capital in

Delhi

Quwwat al ndash Islam mosque was

enlarged by Iltutmish and Alauddin

Khalji The minar was built by three

Sultansndash Qutbuddin Aybak Iltutmish

and Firuz Shah Tughluq

RAUSIAS-FC19E1003 45

Q40) Answer (c)

Explanation

Under the Mughals mansabdar was

referred to an individual who held a

mansab ie rank and he received his

salary as revenue assignments called

jagirs

Q41) Ans (b)

The Quit India Movement was a

spontaneous revolt of people against

British rule

The All India Congress Committee met

at Bombay on 8 August 1942 It passed

the famous resolution Quit India and

proposed the starting of a non-violent

mass struggle under Gandhis

leadership to achieve this aim But on

the very next day Gandhi and other

eminent leaders of the Congress were

arrested The Congress was once again

declared illegal

Q42) Ans (c)

The Simon Commission refers to a

group of seven MPs from the United

Kingdom constituted to suggest

constitutional reforms for British India

The Commission consisted of only

British members headed by one of the

senior British politicians Sir John

Simon

So the people of India agitated against

the arrival of Simon Commission

Q43) Ans (a)

He was widely known for his

unfavourable opinion of the economic

consequences of the British rule in

India

In his many writings and speeches and

especially in Poverty and Un-British

Rule in India Naoroji argued that India

was too highly taxed and that its wealth

was being drained away to England

He did not interpret the ancient Indian

texts and restored the self-confidence of

Indians And also he did not stress the

need for eradication of all the social

evils before anything else

Q44) Ans (c)

In August 1932 Prime Minister

MacDonald announced his Communal

Award Great Britainrsquos unilateral

attempt to resolve the various conflicts

among Indiarsquos many communal

interests

The award which was later

incorporated into the act of 1935

expanded the separate-electorate

formula reserved for Muslims to other

minorities including Sikhs Indian

Christians Anglo-Indians Europeans

distinct regional groups Gandhi

undertook a ldquofast unto deathrdquo against

that offer which he viewed as a

nefarious British plot to divide the

Indian society

Q45) Ans (b)

In British India apart from existing

imports and exports there was also a

particular amount of money which

colonial India contributed towards

administration maintenance of the

army war expenses pensions to retired

officers and other expenses accrued by

Britain towards maintenance of her

colony These were known as Home

charges and were paid for almost

entirely by India

The Home charges was made of

following components-

- Interest payable on Indian debt

- Dividend to shareholders of East

India Company

- Funds used to support the India

Office in London

- Funds used to pay salaries and

pensions of British personnel

engaged in India

- Interest on the railways

- Civil and military charges

- Store purchases in England

Q46) Ans (b)

The Lahore session of the Indian

National Congress was held in 1929

under the Presidentship of Jawaharlal

Nehru

The Lahore session of the Indian

National Congress witnessed significant

RAUSIAS-FC19E1003 46

developments in the Indian national

movement

- First the election of Jawaharlal

Nehru to the post of Presidentship of

the Congress was a clear indication

of the growing strength of the

Leftists in the Congress

- Secondly it was in this session that

the Congress for the first time raised

the demand for complete

independence Such demand was

not raised from the Congress

platform earlier

Q47) Ans (b)

It did not provide for separate

electorates for any community or

weightage for minorities However it did

allow for the reservation of minority

seats in provinces having minorities of

at least ten per cent but this was to be

in strict proportion to the size of the

community

There was no provision for complete

Independence for India

Q48) Ans (c)

The religion of early Vedic Aryans was

primarily of worship of nature and

Yajnas

The early Aryan religion was kind of

nature worship Actually the forces

around them which they could not

control or understand were invested

with divinity and were personified as

male or female gods And they

performed some Yajnas also

Q49) Ans (b)

The roads and river-routes were not

immune from robbery It is notable that

Yuan Chwang (Hiuen Tsang) was

robbed of his belongings during

Harshvardanarsquos period

Q50) Ans (c)

Q51) Ans (b)

Purandara Dasa was a saint and great

devotee of Lord Krishna

There is much speculation about where

Purandara Dasa regarded as the

Pitamaha of Carnatic music was born

Recently an expert committee

constituted by the Kannada University

Hampi has come to the conclusion that

Kshemapura Shivamogga district

Karnataka is the birth place of

Purandara Dasa

Q52) Ans (c)

Sri Tyagaraja Sri Shyama Shastry and Sri Muthuswami Dikshitar are considered the trinity of Carnatic music and with them came the golden age in Carnatic music in the 18th-19th

century

Q53) Ans d)

Recently a rare sarcophagus (stone

coffin) which is 2000 years old from the

Iron AgendashMegalithic era was discovered

from a rock-cut cave at Viyur village of

Kollam near Koyilandy in Kozhikode

district Kerala

The coffin containing bone fragments

was found during an excavation ldquoSo

far such a rare finding has been

discovered only from two sites

in Kerala Both these sarcophagi were

recovered from Megalithic sites at

Chevayur and Atholi also in Kozhikode

district

Q54) Ans a)

The megalithic culture in South India was a full-fledged Iron Age culture

Q55) Ans d)

The Cholas Pandyas and Keralaputras

(Cheras) mentioned in Ashokan

inscriptions were probably in the late

megalithic phase of material culture

Q56) Ans d)

Q57) Ans (b)

Raj Kumar Shukla followed Gandhiji all

over the country to persuade him to

come to Champaran to investigate the

problem associated with tinkathia

system

RAUSIAS-FC19E1003 47

Brij Kishore Rajendra Prasad Mahadev

Desai and Narhari Parikh accompanied

Gandhi ji during the Champaran

Satyagraha

Q58) Ans (b)

The Satvahanas started the practice of granting tax-free villages to brahmanas and Buddhist monks

Q59) Ans c)

The objectives of the Programme are

listed as under

- Developing basic tourism

infrastructure

- Promoting cultural and heritage

value of the country to generate

livelihoods in the identified regions

- Enhancing the tourist attractiveness

in a sustainable manner by

developing world-class

infrastructure at the heritage

monument sites

- Creating employment through active

involvement of local communities

- Harnessing tourism potential for its

effects on employment generation

and economic development

- Developing sustainable tourism

infrastructure and ensuring proper

Operations and maintenance

therein

Q60) Ans (b)

The Tribal Cooperative Marketing

Development Federation of India

(TRIFED) came into existence in 1987

It is a national-level apex organization

functioning under the administrative

control of Ministry of Tribal Affairs

Govt of India

TRIFED has its registered and Head

Office located in New Delhi

Q61) Ans (c)

Premchandrsquos novels include

Premashram Rangabhumi Ghaban

Karmabhumi and Godan

Gora is a novel written by Rabindranath

Tagore

138th birth anniversary of Munshi

Premchand was celebrated across the

country

Q62) Ans (b)

Giddha is a traditional pastoral dance

performed by the women of the Punjab

India and Pakistan at festival times

and at the sowing and reaping of the

harvest

By this dance the Punjabi women

reveal their joy expel their suppressed

feelings in a male dominated society

through the performance of Giddha

Since this dance has nothing to do with

men only women can participate in it

During the Teej celebrations Giddha

dance is celebrated in Punjab every

year Teej is a generic name for a

number of festivals that are celebrated

by women in some parts of India

Q63) Ans (a)

Dara Shukoh wrote the remarkable

work called ldquoMajma-ul-Bahrainrdquo or the

ldquoThe confluence of two seasrdquo

The Vice President of India Shri M

Venkaiah Naidu has said that Prince

Dara Shukohrsquos writings can come as a

refreshing source for infusing peace and

harmony He was addressing the

gathering after visiting the exhibition

that showcases the forgotten Prince of

yesteryears Dara Shukoh organized by

Mr Francois Gautier at Indira Gandhi

National Centre for the Arts in New

Delhi

Q64) Ans (c)

The statue Gommateshwara is

dedicated to the Jain God Bahubali

It is a monolithic statue

President Ram Nath Kovind

inaugurated the grand anointing

ceremony mdash Mahamastakabhisheka mdash

held once in 12 years at

Shravanabelagola (Karnataka)

Q65) Ans (c)

Prachi Valley had come up around the

Prachi river Prachi Valley gradually

disappeared

RAUSIAS-FC19E1003 48

The Prachi river originates from

Bhubaneswar

It is a tributary of the Mahanadi and

flows through the districts of Puri

Khurda Cuttack and Jagatsinghpur

and the entire region of the river is

termed as the Prachi Valley

It falls into the Bay of Bengal

Archaeological evidence shows that the

Prachi Valley Civilisation predates both

Harappa and Mohenjo-Daro

The Prachi river originates from

Bhubaneswar

Q66) Ans (d)

These monuments are located in

Chhatarpur district Madhya Pradesh

within Vindhya mountain range

Q67) Ans (a)

The book lsquoThoughts on Pakistanrsquo was

written by Dr BR Ambedkar

On the occasion of the birth anniversary

of Dr BR Ambedkar the president of

India pays homage to this icon of India

In 1924 he founded the Depressed

Classes Institute (Bahishkrit Hitkarini

Sabha) and in 1927 the Samaj Samata

Sangh

Another area of attention for Ambedkar

was education For its spread among

the low classes he set up a network of

colleges by the name of Peoples

Education Society and founded hostels

Q68) Ans(b)

Mehrgarh is a famous Neolithic

settlement in the Indian subcontinent

which is situated in Baluchistan

province Pakistan

A pre-historic rock art site is discovered

in the vast expanse of limestone blocks

on the eastern banks of Naguleru river

near Dachepalli (Andhra Pradesh) It

has thrown light on the Neolithic

civilisation that flourished in Guntur

(Andhra Pradesh) during 1500-2000

BC

Q69) Ans (c)

The 12th and the 13th centuries saw

the emergence of the Kakatiyas They

were at first the feudatories of the

Western Chalukyas of Kalyana Initially

they ruled over a small territory near

Warangal (Telangana)

They introduced Nayakships which was

later adopted and developed by the

Rayas of Vijayanagara

Q70) Ans (a)

The fast had effect of putting pressure

on mill owners who finally agreed to

give the workers a 35 per cent increase

in wages

Google celebrated with a doodle the

132nd birth anniversary of Anasuya

Sarabhai who played a pioneering role

in Indiarsquos labour movement

Q71) Ans (d)

The UNESCOrsquos list of the representative

list of the intangible cultural heritage of

humanity from India are

- Koodiyattam Sanskrit Theatre of

Kerala

- Mudiyettu ritual theatre and dance

drama of Kerala

- Tradition of Vedic Chanting

- Kalbelia folk songs and dances of

Rajasthan

- Ramlila Traditional Performance of

the Ramayana

- Sankirtana ritual singing

drumming and dancing of Manipur

- Ramman religious festival and

ritual theatre of the Garhwal

Himalayas India

- Traditional brass and copper craft of

utensil making among the Thatheras

of Jandiala Guru Punjab India

- Chhau dance classical Indian dance

originated in the eastern Indian

states

- Buddhist chanting of Ladakh

recitation of sacred Buddhist texts

in the trans-Himalayan Ladakh

region Jammu and Kashmir India

- Yoga

- Nouroz

- Kumbh Mela

RAUSIAS-FC19E1003 49

Q72) Ans(b)

The President of India Shri Ram Nath Kovind inaugurated the Hornbill Festival and State Formation Day celebrations of Nagaland in Kisama

The festival is named after the Indian hornbill the large and colourful forest bird which is displayed in the folklore of most of the states tribes

The major recognized tribes of Nagaland are Angami Ao Chakhesang Chang

Kuki Rengma and Zeling etc

Onge Jarawa and Sentinelese are the

tribes of Andman amp Nicobar Islands

Q73) Ans (c)

The Rashtrakutas rule in the Deccan lasted for almost two hundred years till the end of the tenth century The Rashtrakutas rulers were tolerant in their religious views and patronized not only Shaivism and Vaishnavism but

Jainism as well

The famous rock-cut temple of Shiva at Ellora was built by one of the Rashtrakutas kings Krishna I in the ninth century His successor Amoghavarsha was a Jain but he also

patronized other faiths

The Rashtrakutas allowed Muslims traders to settle and permitted Islam to

be preached in their dominions

Recently increasing defacement at the prehistoric rock paintings of Pandavulagutta Telangana has created a cause for grave concern It can spoil

the prehistoric rock

Pandavulagutta is home to

- Painted rock shelters dating to

10000 BC-8000 BC

- An 8th century inscription of the

Rashtrakuta period and

- Painted frescoes from the 12th century Kakatiya empire

Q74) Ans (b)

In 1828 Raja Ram Mohan Roy founded a new religious society the Brahma Sabha later known as the Brahmo

Samaj

Debendranath Tagore headed the Tattvabodhini Sabha which was

engaged in search of spiritual truth

Its purpose was to purify Hinduism and to preach monotheism or belief in one God

The new society was to be based on the twin pillars of reason and the Vedas and

Upanishads

Recently Sadharan Brahmo Samaj (SBS) has entered into a legal battle with the West Bengal government due

to some legal issue

Q75) Ans (c)

The Chishti order was established in India by Khwaja Moinuddin Chishti who came to India around 1192 The Chishtirsquos are considered to be the most influential of the groups of Sufis who migrated to India in the late twelfth century They adapted successfully to the local environment and adopted several features of Indian devotional

traditions

The historical dargah of Sufi mystic Khwaja Moinuddin Chishti in Ajmer is all set to get a facelift This 13 th century dargah has been included among the Swachh Iconic Places a clean-up initiative focused on iconic

heritage spiritual and cultural places

Page 20: GENERAL STUDIES (PAPER I) · Test is part of Rau’s IAS Test series for Preliminary Exam 2019 FOUNDATION + CURRENT AFFAIRS GENERAL STUDIES (PAPER –I) FOUNDATION TEST –III TOPIC:

RAUSIAS-FC19E1003 20

Q54) निमननिखित कथि ो पर निचार कीनजए

1 महापािानरक ि ग कबर ो म िसतएो दफिात थ

2 दनकषर भारत म महापािार सोसकनत एक परण

निकनसत तामर यगीि सोसकनत थी

उपयणकत कथि ो म स कौि-सास सही हह

(a) किि 1

(b) किि 2

(c) 1 और 2 द ि ो

(d) ि त 1 ि ही 2

Q55) निमननिखित म स कौि-स सामराजयसामराजय ो का

अश क क अनभिि ो म उललि नकया गया ह

1 च ि

2 पाणडय

3 करिपतर (चर)

िीच नदए गए कट का परय ग कर सही उततर चनिए

(a) किि 1

(b) किि 1 और 2

(c) किि 3

(d) 1 2 और 3

Q56) भीमा-क रगाोि का यदध को पिी क सनिक ो और

बाजीराि नदवतीय क िततव म एक शखकतशािी पशिा

सिा (मराठ ो) क मधय िड़ा गया था यह यदध

निमननिखित म स नकसका नहससा था

(a) परथम आोगल-मराठा यदध का

(b) नदवतीय आोगल-मराठा यदध का

(c) ततीय आोगल-मसर यदध का

(d) ततीय आोगल-मराठा यदध का

Q57) निमननिखित कथि ो पर निचार कीनजए

1 महादि दसाई ि गाोरीजी क चोपारर आि तथा

नतिकनथया पररािी स जड़ी समसया की जाोच

क निए रारी करि क निए दश भर म उिका

अिसरर नकया था

2 िरहरी पाररि चोपारर सतयागरह क दौराि

गाोरीजी क साथ थ

उपयणकत कथि ो म स कौि-सास सही हह

(a) किि 1

(b) किि 2

(c) 1 और 2 द ि ो

(d) ि त 1 ि ही 2

Q58) निमननिखित कथि ो पर निचार कीनजए

1 िनद राज-िोश ि बराहमर ो और बौदध मठराररय ो

क कर-मकत गाि अिदाि म दि की परथा

आरि की थी

2 सतिाहि ो की आनरकाररक भािा पराकत थी

उपयणकत कथि ो म स कौि-सास सही हह

(a) किि 1

(b) किि 2

(c) 1 और 2 द ि ो

(d) ि त 1 ि ही 2

Q59) एक निरासत क अपिाइए (अडॉपट ए हररटज ndash

Adopt a Heritage) पररय जिा क उददशय ो क

सनदभण म निमननिखित कथि ो पर निचार कीनजए

1 यह पररय जिा र रगार उतपादि और आनथणक

निकास क निए पयणटि कषमता का उि पर

परभाि का उपय ग करगी

2 यह पररय जिा निरासत सथि ो पर निशव सतरीय

आराररक सोरचिा निकनसत करक एक सतत

तरीक स पयणटक आकिणर म िखदध करगी

उपयणकत कथि ो म स कौि-सास सही हह

(a) किि 1

(b) किि 2

(c) 1 और 2 द ि ो

(d) ि त 1 ि ही 2

RAUSIAS-FC19E1003 21

Q54) Consider the following statements

1 Megalithic people buried goods in

graves

2 The megalithic culture in South

India was a full-fledged Copper

Age culture

Which of the statements given above

isare correct

(a) 1 only

(b) 2 only

(c) Both 1 and 2

(d) Neither 1 nor 2

Q55) Which of the following kingdoms isare

mentioned in the Ashokan inscriptions

1 Cholas

2 Pandyas

3 Keralaputras (Cheras)

Select the correct answer using the code

given below

(a) 1 only

(b) 1 and 2 only

(c) 3 only

(d) 1 2 and 3

Q56) The Battle of Bhima-Koregaon was

fought between the soldiers of the

Company and the strong Peshwa army

(Marathas) under Bajirao II This war

was a part of the

(a) First Anglo-Maratha war

(b) Second Anglo-Maratha war

(c) Third Anglo- Mysore war

(d) Third Anglo-Maratha war

Q57) Consider the following statements

1 Mahadev Desai followed Gandhiji all over the country to persuade him to come to Champaran to investigate the problem associated

with tinkathia system

2 Narhari Parikh accompanied Gandhi ji during the Champaran

Satyagraha

Which of the statements given above isare correct

(a) 1 only

(b) 2 only

(c) Both 1 and 2

(d) Neither 1 nor 2

Q58) Consider the following statements

1 The Nanda Dynasty started the practice of granting tax-free villages to brahmanas and

Buddhist monks

2 The official language of the Satavahanas was Prakrit

Which of the statements given above

isare correct

(a) 1 only

(b) 2 only

(c) Both 1 and 2

(d) Neither 1 nor 2

Q59) Consider the following statements about the objectives of the lsquoadopt a heritagersquo

project

1 It will harness tourism potential for its effects on employment generation and economic

development

2 It will enhance the tourist attractiveness in a sustainable manner by developing world class infrastructure at heritage sites

Which of the statements given above

isare correct

(a) 1 only

(b) 2 only

(c) Both 1 and 2

(d) Neither 1 nor 2

RAUSIAS-FC19E1003 22

Q60) ldquoभारतीय जिजातीय सहकारी निपरि निकास सोघrdquo

(The Tribal Co-operative Marketing

Development Federation of India - TRIFED)

क सोदभण म निमननिखित कथि ो पर निचार कीनजए

1 यह एक राषटर ीय सतर का शीिण सोगठि ह ज

भारत सरकार क गह मोतरािय क परशासनिक

नियोतरर क अरीि काम कर रहा ह

2 इसका मखय उददशय दश म जिजातीय ि ग ो

का सामानजक-आनथणक निकास करिा ह

उपयणकत कथि ो म स कौि-सास सही हह

(a) किि 1

(b) किि 2

(c) 1 और 2 द ि ो

(d) ि त 1 ि ही 2

Q61) निमननिखित म स कौि-सास उपनयास परमचोद क

दवारा नििा गया हनिि गए ह

1 रोगभनम

2 ग दाि

3 ग रा

िीच नदए गए कट का परय ग कर सही उततर चनिए

(a) किि 1

(b) किि 2

(c) किि 1 और 2

(d) 1 2 और 3

Q62) नगदधा ितय क सोदभण म निमननिखित कथि ो पर निचार

कीनजए

1 नगदधा नबहार की मनहिाओो क दवारा तयौहार क

समय और फसि की बिाई तथा कटाई क

अिसर पर नकया जाि िािा एक पारोपररक

दहाती ितय ह

2 इस ितय क दवारा मनहिाऐो अपिी परसननता

परकट करती ह तथा नगदधा क परदशणि क

माधयम स परि िचणसव िाि समाज म

मनहिाओो की दबी हई भाििाओो क परकट

करती ह

उपयणकत कथि ो म स कौि-सास सही हह

(a) किि 1

(b) किि 2

(c) 1 और 2 द ि ो

(d) ि त 1 ि ही 2

Q63) निमननिखित कथि ो पर निचार कीनजए

1 मलला शाह बदखशी दारा नशक ह क

आधयाखतमक गर थ

2 औरोगरब ि मजम-उि-बहरीि या द समदर ो

का सोगम िामक उललििीय रचिा नििी थी

3 दारा नशक ह क अपि पिणज अकबर क गर ो

क उततरानरकारी क रप म दिा गया था

नजसम उसि रानमणक बहििाद और समनवयता

क बढ़ािा नदया था

उपयणकत कथि ो म स कौि-सास सही हह

(a) किि 1 और 3

(b) किि 2

(c) किि 1 और 2

(d) 1 2 और 3

RAUSIAS-FC19E1003 23

Q60) Consider the following statements about

the Tribal Cooperative Marketing

Development Federation of India

(TRIFED)

1 It is a national-level apex

organization functioning under the

administrative control of Ministry

of Home Affairs Government of

India

2 The main objective of TRIFED is

socio-economic development of

tribal people in the country

Which of the statements given above

isare correct

(a) 1 only

(b) 2 only

(c) Both 1 and 2

(d) Neither 1 nor 2

Q61) Which of the following novels isare

written by Premchand

1 Rangabhumi

2 Godan

3 Gora

Select the correct answer using the code

given below

(a) 1 only

(b) 2 only

(c) 1 and 2 only

(d) 1 2 and 3

Q62) Consider the following statements about

Giddha dance

1 Giddha is a traditional pastoral

dance performed by the women of

Bihar at festival times and at the

sowing and reaping of the harvest

2 By this dance the women reveal

their joy expel their suppressed

feelings in a male dominated

society through the performance of

Giddha

Which of the statements given above

isare correct

(a) 1 only

(b) 2 only

(c) Both 1 and 2

(d) Neither 1 nor 2

Q63) Consider the following statements

1 Mullah Shah Badakhshi was the

spiritual mentor of Dara Shukoh

2 Aurangzeb wrote the remarkable

work called ldquoMajma-ul-Bahrainrdquo or

the ldquoThe confluence of two seasrdquo

3 Dara Shukoh was seen as

inheriting the qualities of his

ancestor Akbar in that he

promoted religious pluralism and

syncretism

Which of the statements given above

isare correct

(a) 1 and 3 only

(b) 2 only

(c) 1 and 2 only

(d) 1 2 and 3

RAUSIAS-FC19E1003 24

Q64) निमननिखित कथि ो पर निचार कीनजए

1 ग मतशवर परनतमा निोधयनगरी पहाड़ी पर खसथत ह

2 शरिरबिग िा िह सथाि ह जहाो मौयण िोश क

सोसथापक चोदरगपत मौयण अपि नसोहासि क

तयागि क बाद जि तपसवी बि गए थ

उपयणकत कथि ो म स कौि-सास सही हह

(a) किि 1

(b) किि 2

(c) 1 और 2 द ि ो

(d) ि त 1 ि ही 2

Q65) निमननिखित कथि ो पर निचार कीनजए

1 पराताखतवक साकषय स पता चिता ह नक पराची

घाटी सभयता हड़पपा और म हिज दाड़ द ि ो

की पिणिती ह

2 पराची िदी भििशवर स निकिती ह

उपयणकत कथि ो म स कौि-सास सही हह

(a) किि 1

(b) किि 2

(c) 1 और 2 द ि ो

(d) ि त 1 ि ही 2

Q66) निमननिखित कथि ो म स कौि-सास सही हह

1 िजराह क समारक ो क समह का निमाणर

चोदि राजिोश क शासिकाि क दौराि हआ

था

2 य समारक हररिोदर पिणत शरोििा म खसथत ह

3 म रक क यातरी इबन बतता ि अपि सोसमरर ो

म िजराह क मोनदर ो की यातरा का उललि

नकया था तथा इन काजराण िाम स समब नरत

नकया था

िीच नदए गए कट का परय ग कर सही उततर चनिए

(a) किि 1

(b) किि 1 और 2

(c) किि 2 और 3

(d) किि 1 और 3

Q67) निमननिखित कथि ो म स कौि-सास सही हह

1 डॉ बी आर अमबडकर ि दी एनिनहिशि

ऑफ़ कासट (The Annihilation of Caste)

नििी थी नजसम उन ोि नहोद रमण म िोशािगत

पजारी की परथा क उनमिि की आिशयकता

पर बि नदया था

2 डॉ राजदर परसाद ि थॉटस ऑि पानकसताि

(Thoughts on Pakistan) िामक पसतक

नििी थी

िीच नदए गए कट का परय ग कर सही उततर चनिए

(a) किि 1

(b) किि 2

(c) 1 और 2 द ि ो

(d) ि त 1 ि ही 2

Q68) निमननिखित कथि ो म स कौि-सास सही हह

1 महरगढ़ भारतीय उपमहादवीप म एक परनसदध

ििपािार बसती ह ज नसोर पराोत पानकसताि म

खसथत ह

2 बरणह म म कतत ो क उिक सवामी क साथ कबर ो

म दफिाया जाता था

िीच नदए गए कट का परय ग कर सही उततर चनिए

(a) किि 1

(b) किि 2

(c) 1 और 2 द ि ो

(d) ि त 1 ि ही 2

Q69) निमननिखित कथि ो म स कौि-सास सही हह

1 काकानटय मोनदर अनरकतर नशि क समनपणत

2 हिमक ोडा म हजार-सतोभ िाि मोनदर (The

Thousand-Pillared Temple) का निमाणर

काकानटय समराट रदर ि करिाया था

िीच नदए गए कट का परय ग कर सही उततर चनिए

(a) किि 1

(b) किि 2

(c) 1 और 2 द ि ो

(d) ि त 1 ि ही 2

RAUSIAS-FC19E1003 25

Q64) Consider the following statements

1 Gommateshwara Statue is located

on the Vindyagiri Hill

2 Shravanabelagola is the place

where Chandragupta Maurya the

founder of the Mauryan dynasty

became a Jain ascetic after

relinquishing his throne

Which of the statements given above

isare correct

(a) 1 only

(b) 2 only

(c) Both 1 and 2

(d) Neither 1 nor 2

Q65) Consider the following statements

1 Archaeological evidence shows

that the Prachi Valley Civilisation

predates both Harappa and

Mohenjo-Daro

2 The Prachi river originates from

Bhubaneswar

Which of the statements given above

isare correct

(a) 1 only

(b) 2 only

(c) Both 1 and 2

(d) Neither 1 nor 2

Q66) Which of the following statements

isare correct

1 The Khajuraho group of

monuments was built during the

rule of the Chandela dynasty

2 These monuments are located in

Harischandra mountain range

3 Ibn Battuta the Moroccan

traveller in his memoirs mentioned

visiting Khajuraho temples and

called them Kajarra

Select the correct answer using the code

given below

(a) 1 only

(b) 1 and 2

(c) 2 and 3

(d) 1 and 3

Q67) Which of the following statements

isare correct

1 Dr BR Ambedkar wrote the

Annihilation of Caste emphasising

the need to do away with the

practice of hereditary priesthood in

Hinduism

2 The book lsquoThoughts on Pakistanrsquo

was written by Dr Rajendra

Prasad

Select the correct answer using the code

given below

(a) 1 only

(b) 2 only

(c) Both 1 and 2

(d) Neither 1 nor 2

Q68) Which of the following statements

isare correct

1 Mehrgarh is a famous Neolithic

settlement in the Indian

subcontinent which is situated in

Sindh province Pakistan

2 At Burzahom dogs were buried

with their masters in their graves

Select the correct answer using the code

given below

(a) 1 only

(b) 2 only

(c) Both 1 and 2

(d) Neither 1 nor 2

Q69) Which of the following statements

isare correct

1 The Kakatiya temples are

dedicated mostly to Siva

2 The Thousand-Pillared Temple at

Hanamkonda was built by the

Kakatiya king Rudra

Select the correct answer using the code

given below

(a) 1 only

(b) 2 only

(c) Both 1 and 2

(d) Neither 1 nor 2

RAUSIAS-FC19E1003 26

Q70) निमननिखित कथि ो म स कौि-सास सही हह

1 अहमदाबाद नमि हड़ताि क दौराि महातमा

गाोरी ि शरनमक ो क पकष क मजबत करि क

निए आमरर अिशि नकया था

2 अिशि स नमि मानिक ो पर दबाि पड़ा था ज

अोततः शरनमक ो क िति म 15 परनतशत की िखदध

करि क निए सहमत हए थ

िीच नदए गए कट का परय ग कर सही उततर चनिए

(a) किि 1

(b) किि 2

(c) 1 और 2 द ि ो

(d) ि त 1 ि ही 2

Q71) निमननिखित म स नकसक नकिक भारत स यिसक

की माििता की अमतण साोसकनतक निरासत की

परनतनिनर सची (The UNESCOrsquos List of the

Representative List of the Intangible

Cultural Heritage of Humanity) म शानमि

नकया गया ह

1 मनडयटट

2 सोकीतणि

3 को भ मिा

िीच नदए गए कट का परय ग कर सही उततर चनिए

(a) किि 1 और 2

(b) किि 2 और 3

(c) किि 3

(d) 1 2 और 3

Q72) निमननिखित जिजानतय ो म स कौि-सीसी ो

जिजानतजिजानतया िागािड स सोबोनरत हह

1 अोगामी

2 ककी

3 जारिा

िीच नदए गए कट का परय ग कर सही उततर चनिए

(a) किि 1

(b) किि 1 औऔ 2

(c) किि 2

(d) 1 2 और 3

Q73) निमननिखित कथि ो म स कौि-सास सही हह

1 राषटर कट सामराजय की सथापिा दोनतदगण ि की थी

नजसि मानयाित म अपिी राजरािी की

सथापिा की थी

2 राषटर कट समराट अम घििण एक ििक था और

उस कनिताओो पर पहिी कननड़ पसतक नििि

का शरय नदया जाता ह

िीच नदए गए कट का परय ग कर सही उततर चनिए

(a) किि 1

(b) किि 2

(c) 1 और 2 द ि ो

(d) ि त 1 ि ही 2

Q74) निमननिखित कथि ो म स कौि-सास सही हह

1 कशब चोदर सि ि ततवब नरिी सभा की

अधयकषता की थी ज आधयाखतमक सतय की

ि ज म सोिि थी

2 बरहम समाज ि मािि गररमा पर बि नदया

मनतणपजा का निर र नकया और सती परथा जसी

सामानजक बराइय ो की आि चिा की

िीच नदए गए कट का परय ग कर सही उततर चनिए

(a) किि 1

(b) किि 2

(c) 1 और 2 द ि ो

(d) ि त 1 ि ही 2

Q75) निमननिखित कथि ो म स कौि-सास सही हह

1 भारत म नचशती नसिनसिा खवाजा म इिददीि

नचशती क दवारा सथानपत नकया गया था

2 नचशती परोपरा की एक परमि निशिता

आतमसोयम थी नजसम साोसाररक म ह स दरी

बिाए रििा शानमि था

िीच नदए गए कट का परय ग कर सही उततर चनिए

(a) किि 1

(b) किि 2

(c) 1 और 2 द ि ो

(d) ि त 1 ि ही 2

RAUSIAS-FC19E1003 27

Q70) Which of the following statements

isare correct

1 During the Ahmedabad Mill Strike

Mahatma Gandhi undertook a fast

unto death to strengthen the

workersrsquo resolve

2 The fast had effect of putting

pressure on mill owners who

finally agreed to give the workers a

15 per cent increase in wages

Select the correct answer using the code

given below

(a) 1 only

(b) 2 only

(c) Both 1 and 2

(d) Neither 1 nor 2

Q71) Which of the following are included in

the UNESCOrsquos list of the representative

list of the intangible cultural heritage of

humanity from India

1 Mudiyettu

2 Sankirtana

3 Kumbh Mela

Select the correct answer using the code

given below

(a) 1 and 2 only

(b) 2 and 3 only

(c) 3 only

(d) 1 2 and 3

Q72) Which of the following tribes isare

related to Nagaland

1 Angami

2 Kuki

3 Jarawa

Select the correct answer using the code

given below

(a) 1 only

(b) 1 and 2 only

(c) 2 only

(d) 1 2 and 3

Q73) Which of the following statements

isare correct

1 Rashtrakuta kingdom was founded by Dantidurga who established his capital at Manyakhet

2 Amoghavarsha a Rashtrakuta king was an author and is credited with writing the first

Kannada book on poetics

Select the correct answer using the code given below

(a) 1 only

(b) 2 only

(c) Both 1 and 2

(d) Neither 1 nor 2

Q74) Which of the following statements isare correct

1 Keshab Chandra Sen headed the Tattvabodhini Sabha which was engaged in search of spiritual truth

2 The Brahmo Samaj laid emphasis on human dignity opposed idolatry and criticized such social

evils as the practice of Sati

Select the correct answer using the code given below

(a) 1 only

(b) 2 only

(c) Both 1 and 2

(d) Neither 1 nor 2

Q75) Which of the following statements isare correct

1 The Chishti order was established in India by Khwaja Moinuddin

Chishti

2 A major feature of the Chishti tradition was austerity including maintaining a distance from the

worldly power

Select the correct answer using the code

given below

(a) 1 only

(b) 2 only

(c) Both 1 and 2

(d) Neither 1 nor 2

T e s t i s p a r t o f R a u rsquo s I A S T e s t s e r i e s f o r P r e l i m i n a r y E x a m 2 0 1 9

FOUNDATION + CURRENT AFFAIRS

GENERAL STUDIES (PAPER ndashI)

FOUNDATION TEST ndashIII

SUBJECT NCERT History Class VI-X + Current Affairs

Time Allowed 1frac12 Hours Maximum Marks 150

I NSTRUCT IONS

1 IMMEDIATELY AFTER THE COMMENCEMENT OF THE EXAMINATION YOU SHOULD CHECK

THAT THIS TEST BOOKLET DOES NOT HAVE ANY UNPRINTED OR TORN or MISSING PAGES OR

ITEMS ETC IF SO GET IT REPLACED BY A COMPLETE TEST BOOKLET

2 This Test Booklet contains 75 items (questions) Each item is printed both in Hindi and English

Each item comprises four responses (answers) You will select the response which you want to mark

on the Answer Sheet In case you feel that there is more than one correct response mark the

response which you consider the best In any case choose ONLY ONE response for each item

3 You have to mark all your responses ONLY on the separate Answer Sheet (OMR sheet) provided

Read the directions in the Answer Sheet

4 All items carry equal marks

5 Before you proceed to mark in the Answer Sheet the response to various items in the Test booklet

you have to fill in some particulars in the Answer Sheet as per instructions contained therein

6 After you have completed filling in all your responses on the Answer Sheet and the examination has

concluded you should hand over to the Invigilator only the Answer Sheet You are permitted to

take away with you the Test Booklet

7 Penalty for wrong answers

THERE WILL BE PENALTY FOR WRONG ANSWERS MARKED BY A CANDIDATE IN THE

OBJECTIVE TYPE QUESTION PAPERS

(i) There are four alternatives for the answer to every question For each question for which a

wrong answer has been given by the candidate one-third of the marks assigned to that

question will be deducted as penalty

(ii) If a candidate gives more than one answer it will be treated as a wrong answer even if one of

the given answers happens to be correct and there will be same penalty as above to that

question

(iii) If a question is left blank ie no answer is given by the candidate there will be no penalty for

that question

T h i s t e s t i s p a r t o f R a u rsquo s I A S T e s t s e r i e s f o r P r e l i m i n a r y E x a m 2 0 1 9

Test Code

FC19E1003

FC19H1003 29

Answers and Explanations of

NCERT History Class VI-X + Current Affairs (FC19E1003)

Q1) उततर (c)

सपषटीकरण

- ऋगवद म दविय ो और दिताओो क समवपित एक

हजार स अविक सत तर (शल क) ह

- य शल क ऋविय ो क दवारा रच गए थ और परि ो

दवारा सीख जात थ

- हालाोवक कछ शल क मवहलाओो (जस वक अपाला

घ सा ल पामदरा मतरयी और गागी) क दवारा भी रच

गए थ

- ऋगवद म सोिाद क रप म कई शल क मौजद ह

- हम विशवावमतर नामक एक ऋवि और दविय ो क

रप म पजी जान िाली द नवदय ो (वयास और

सतलज) क बीच िाताि का उदाहरण वमलता ह

- इसस पता चलता ह वक विशवावमतर िवदक काल स

सोबोवित थ

Q2) उततर (b)

सपषटीकरण

- करनल गफाओो स राख क अिशि परापत हए ह

ज इस ओर सोकत करत ह वक ततकालीन ल ग

अवि क उपय ग स पररवचत थ

- य गफाएो आोधर परदश म सथथत ह

Q3) उततर (c)

सपषटीकरण

bull बरािह म ितिमान कशमीर म सथथत एक

परागवतहावसक थथल ह जहाो ल ग गडढ क घर ो का

वनमािण करत थ

bull य घर जमीन क ख द कर बनाए जात थ तथा नीच

जान क वलए सीवियाा ह ती थी

bull ऐसा अनमान लगाया जाता ह वक य घर ठो ड क

मौसम म आशरय परदान करत थ

Q4) उततर (c)

सपषटीकरण

bull परालख-विदया (Epigraphy) क वशलालख ो क

अधययन क रप म पररभावित वकया जाता ह

bull हसतवलसखत दसतािज ो क माधयम स इवतहास

और सावहतय क अधययन क पाोडवलवप विजञान

(Manuscriptology) कहत ह

bull पराचीन लखन परणावलय ो क अधययन और

ऐवतहावसक पाोडवलवपय ो क समझन तथा वतवथ

वनिािरण क पलीओगराफी (Palaeography) कहा

जाता ह

bull नयवमजमविकस (Numismatics) वसक ो क

अधययन क सोदवभित करता ह

Q5) उततर (a)

सपषटीकरण

- चरक सोवहता चरक क दवारा वलखी गई आयिद

और िदयक-शासर पर एक महतवपणि पसतक ह

- ि भारतीय िदयक-शासर की पारमपररक परणाली

वजस आयिद क नाम स जाना जाता ह क

अभयासकताि थ

- ऐसा माना जाता ह वक चरक का विकास दसरी

शताबदी (ईसा पिि) और दसरी शताबदी (ईसवी) क

मधय हआ था

Q6) उततर (b)

सपषटीकरण

- भाग फसल ो पर वलए जान िाल कर क सोदवभित

करता ह ज कल फसल उतपादन का 16 िाो भाग

था

- ldquoकममकारrdquo शबद भवमहीन कवि शरवमक िगि क

वलए परय ग वकया जाता था

- ldquoअशवमिrdquo (वजस घ ड क बवलदान क रप म भी

जाना जाता ह) एक अनषठान ह ता था वजसम एक

घ ड क सवतोतर रप स घमन क वलए छ ड वदया

FC19H1003 30

जाता ह और राजा क सवनक उसकी रखिाली

करत थ

Q7) उततर (d)

सपषटीकरण

- ऋगववदक काल म घ ड ो क रथ ो म ज ता जाता था

ज (रथ) भवम मिवशय ो आवद पर कबजा करन क

वलए लड गए यद ो म उपय ग वकए जात थ

- इसस यह पता चलता ह वक घ ड ो यकत रथ ो का

उपय ग महाजनपद काल स काफी पहल आरमभ

हआ था

- ऋगववदक काल म मिवशय ो भवम जल आवद पर

कबजा करन क वलए तथा ल ग ो क पकडन क

वलए यद वकय जात थ

- अविकाोश परि इन यद ो म भाग वलया करत थ

- हालाोवक उस समय क ई वनयवमत सना नही ो ह ती

थी लवकन उस काल म सभाऐो ह ती थी ो वजनम

ल ग यद क मामल ो पर चचाि करत थ

- वनयवमत सनाएा महाजनपद काल का िवशषटय थी

वजनम पदल सवनक ो की विशाल सनाएा रथ तथा

हाथी शावमल ह त थ

Q8) उततर (a)

सपषटीकरण

- बद शाकय कल स सोबोवित थ और कशीनारा म

उनका वनिन हआ था

- बद न अपनी वशकषाएा पराकत भािा म दी थी ो ज

आम ल ग ो की भािा थी

Q9) उततर (c)

सपषटीकरण

- पराचीन भारत म दशिनशासर की छह शाखाएा थी ो

िशविक नयाय समखया य ग पिि वममाोसा और

िदाोत या उततर वममाोसा

- इनकी थथापना करमश कनाद गौतम कवपल

पतोजवल जावमनी और वयास ऋविय ो न की थी

Q10) उततर (b)

सपषटीकरण

महािीर की वशकषाऐो छठी शताबदी म िललभी म

सोकवलत की गई थी ो

Q11) उततर (c)

सपषटीकरण

- पारमपररक रप स चाणकय क कौविलय अथिा

विषणगपत क नाम स जाना जाता ह

- उसन अथिशासतर ज एक पराचीन भारतीय

राजनवतक आलख ह वलखा था

Q12) उततर (d)

सपषटीकरण

- भारत का राषटर ीय वचनह सारनाथ (उततर परदश) क

अश क सतमभ क ऊपर (शीिि पर) वसोह कवपिल

का एक अनरपण ह

- इस राषटर ीय वसदाोत सतयमि जयत क साथ

सोय वजत वकया गया ह

- रामपिि बल का नाम रामपिि (वबहार) क नाम पर

पडा जहाा इसकी ख ज हई थी

- यह अपन नाजक नकाशी मॉडल क वलए परवसदद

ह वजसम क मल तवचा सोिदनशील नथन ो सतकि

कान और मरबत िााग ो क शरषठतर परवतरप क

परदवशित वकया गया ह

- यह भारतीय और फारसी ततव ो का एक ससममशरण

- सोवकससा उततर परदश म सथथत ह

Q13) उततर (a)

सपषटीकरण

का िर वसोह ज एक महान य दा थ वबहार स

सोबोवित थ

Q14) उततर (b)

सपषटीकरण

िललालर शबद बड भ-सवावमय ो क वलए परय ग

वकया जाता था

FC19H1003 31

Q15) उततर (c)

सपषटीकरण

- अररकमड एक तिीय बसती थी जहाो दर दश ो स

आन िाल जहाज ो का माल उतारा जाता था

- यहाो पर ईोि ो का एक विशाल ग दाम वमटटी क

बतिन (वजनम एमफ रा - द हरी मवठय ो का लोबा

घडा - शावमल ह) और एरिाइन (Arretine)

मदभाोड पाए गए थ

- इस थथान पर र मन दीपक काोच क बन पातर और

रतन भी पाए गए थ

Q16) उततर (a)

सपषटीकरण

- मिनदर सोगम कविताओो म उसललसखत एक

तवमल शबद ह वजसका अथि ह ldquoतीन परमखrdquo

- यह तीन सततारि पररिार ो क मसखयाओो क वलए

परय ग वकया जाता ह च ल चर और पाणडय

Q17) उततर (c)

सपषटीकरण

- ऋग िद म सभा विदाथा तथा गण जसी

जनजावतय ो पर अथिा किोब पर आिाररत

सभाओो का उललख ह

- आरसमभक िवदक काल म सभाओो और सवमवतय ो

का विशि महतव ह ता था

- यहाा तक की मसखया अथिा राजा भी उनका

समथिन परापत करन क वलए आतर रहत थ

Q18) उततर (a)

सपषटीकरण

- जन िमि न ईशवर क अससततव क मानयता त दी ह

वकनत उसन ईशवर क वजना क पद स नीच रखा

- जन िमि न बौद िमि की तरह िणि परणाली की

भरतिना नही ो की थी

Q19) उततर (d)

सपषटीकरण

- च ल ो और पाणडय ो न शसकतशाली तिीय शहर ो का

विकास वकया था

- च ल ो का सबस महतवपणि शहर पहार (या

कािरीपटटीनम) था |

- मदरई पाणडय ो की राजिानी थी

Q20) उततर (b)

सपषटीकरण

- ldquoबदचररतrdquo बद का जीिन-ितताोत ह

- इस अशवघ ि क दवारा वलखा गया था

Q21) उततर (a)

सपषटीकरणः

- तवमल कवि अपपर भगिान वशि क भकत थ

- इस परकार ि एक नयनार सोत थ

Q22) उततर (d)

सपषटीकरणः

- समदरगपत एक परवसद गपत शासक था

- उसन वसक ो पर िीणा बजात हए अपनी छवि

अोवकत करिाई थी

- यह सोगीत क परवत उसक परम क दशािता ह

- हम उसकी इलाहाबाद परशससत स महतवपणि

ऐवतहावसक जानकारी वमलती ह वजसकी रचना

उसक दरबार क कवि हररसन न की थी

Q23) उततर (b)

सपषटीकरणः

- विकरम सोित की शरआत ििि 58 ईसा पिि म

चनदरगपत वदवतीय न की थी

- यह शक ो पर उसकी जीत और उस विकरमावदतय

की पदिी वमलन क उपलकषय म आरमभ वकया गया

था

FC19H1003 32

- बानभटट न हिििििन का जीिन-ितताोत हििचररत

(ज सोसकत म थी) वलखी थी

Q24) उततर (c)

सपषटीकरणः

- सोवि-विगरावहका यद एिो शाोवत का मोतरी

- साथििाह वयापाररय ो क कावफल ो का नता

Q25) उततर (a)

सपषटीकरणः

- जआन झाोग (हसआन रताोग ndash Hsuang Tsang)

एक चीनी यातरी था ज हिििििन क शासनकाल म

भारत आया था

- ििि 630 ईसवी स ज दशक आरमभ हआ था उसम

जआन झाोग मधय एवशया ईरान और

अफग़ावनसतान की यातरा करन क पशचात कशमीर

क रासत स भारत आया था

- उसन उततर स पिि तक की यातरा की और िह

लगभग 2 ििि वबहार म रहा

- जआन झाोग न नालनदा विशवविदयालय म विदयावथिय ो

और विदवान ो क साथ पारसपररक विचार-विमशि

वकया थथानीय भािाओ ा म वनपणता परापत की तथा

बौद सतप ो की ख ज की

Q26) उततर (c)

सपषटीकरणः

- परदवकषणा पथ बौद िासतकला म सतप क चार ो

ओर बनाया जान िाला एक घमािदार पथ ह ता

- परशन म वदए गए बाकी क तीन ो ततव वहोद मसनदर ो की

िासतकला क भाग ह

Q27) उततर (d)

सपषटीकरणः

परशन म वदए गए सभी मोवदर ो म वयापक रप स

ईोि ो (पकी ईोि ो) का परय ग पतथर ो क साथ हआ

Q28) उततर (c)

सपषटीकरण

- महममद कली कतब शाह ग लकणडा का सलतान

था

- िह अकबर का समकालीन था

- सावहतय और िासतकला म उसकी अतयाविक

रवच थी

- िह एक महान कवि था

- िह दसखनी उदि फारसी और तलग म वलखता था

- उसन अपन पीछ एक विसतत वदिान (सोगरह)

छ डा ह

- अभी हाल ही म तलोगाना म ग लकणडा क वकल

क अनदर खदाई वकय गए बाग-ए-नाया वकला

बाग क चार ो ओर रप-रखा क मानवचतरण क

वलए भारतीय परातासतवक सिकषण (The

Archaeological Survey of India ndash ASI)

गराउणड पनीिर विोग रडार (Ground Penetrating

Radar) का परय ग करगा

Q29) उततर (a)

सपषटीकरणः

- वसलपपावदकारम एक तवमल महाकावय ह वजसकी

रचना इलाोग क दवारा लगभग 1800 ििि पिि की

गई थी

- यह क िलन नामक एक वयापारी की कहानी ह

ज माििी नामक एक गवणका (िशया) स परम

करन लगा था

- मवनमकलाई क िलन और माििी की पतरी की

कहानी ह

Q30) उततर (a)

सपषटीकरण

- चरक आयिद और वचवकरता की एक महतवपणि

रचना चरक सोवहता क लखक ह

- बरहमगपत क अपनी रचना बरहम-सफि-वसदानत

(ज एक खग लीय रचना ह) क कारण परवससद

वमली

FC19H1003 33

- बगदाद म इसका अनिाद अरबी भािा म वकया

गया था

- इसका इसलावमक गवणत और खग ल-विजञान पर

महतवपणि परभाि पडा था

- बाद म अपन जीिनकाल म बरहमगपत न

ldquoखोडखयाकrdquo वलखी ज एक खग लीय पससतका

(एक छ िी पसतक) थी

- इसम आयिभटट की अिि-रावतर क परतयक वदन की

शरआत परणाली का परय ग वकया गया था

Q31) उततर (c)

सपषटीकरण

- अमीर खसर एक परवसद सफी सोगीतकार कवि

और विदवान थ

- 1318 म उनह ोन पाया वक इस भवम (वहोदसतान) क

हर कषतर म अलग-अलग भािा थी लाहौरी

कशमीरी दवारसमदरी (दवकषणी कनाििक म)

तलोगाना (आोधर परदश म) गजरी (गजरात म)

माबारी (तवमलनाड म ) अििी (पिी उततर परदश

म) और वहोदिी (वदलली क आस-पास क कषतर म)

आवद

- उनह न यह बताया वक सोसकत वकसी भी कषतर स

सोबोवित नही ो थी और किल बराहमण ही इस भािा

का जञान रखत थ

Q32) उततर (c)

सपषटीकरण

- वहरणय-गभि सववणिम गभि क सोदवभित करता ह

- जब बराहमण ो की सहायता स यह अनषठान वकया

जाता था त यह माना जाता था वक बवल दन िाल

का कषवतरय क रप म पनजिनम ह गा

Q33) उततर (d)

सपषटीकरण

- कदमई भवम राजसव पर कर क सोदवभित करता

- गवावलयर परशससत म नागभि क दवारा वकय गए

श िण का िणिन वकया गया ह |

- नागभि एक परवतहार राजा था

Q34) उततर (b)

सपषटीकरण

- राजतरो वगनी 12िी ो शताबदी म कलहन क दवारा

रवचत एक सोसकत पसतक (िकसट) ह

- यह परारसमभक भारत की ऐवतहावसक इवतितत थी

- तकि सोगत रप स इस अपन परकार की सिोततम

और सिािविक विशवसनीय कवत माना जाता ह

- यह कशमीर कषतर क पराचीनतम समय स लकर

उसकी रचना की तारीख तक क समपणि इवतहास

का आचछादन करती ह

Q35) उततर (c)

सपषटीकरण

- गााि की आम सभा क ldquoउरrdquo कहा जाता था

- ldquoउरrdquo म गााि क सभी कर दन िाल वनिासी

शावमल ह त थ

Q36) उततर (a)

सपषटीकरण

- वदलली सलतनत म ldquoतारीखrdquo इवतहास लखन का

एक रप था

- ldquoतािरीखrdquo क लखक विदवान परि ह त थ वजनम

सवचि परशासक इतयावद शावमल थ

Q37) उततर (a)

सपषटीकरण

- अलाउददीन सखलजी अपन सवनक ो क ितन का

भगतान नकद म करता था न वक इकता क रप

- सवनक अपना सामान वदलली म वयापाररय ो स

खरीदत थ अतः इस बात का भय था वक वयापारी

कही ो िसतओो का मलय न बिा द

- इसकी र कथाम क वलए अलाउददीन सखलजी न

वदलली म कीमत ो क वनयसित वकया

FC19H1003 34

- अविकारीगण धयानपििक मलय ो का सिकषण करत

थ तथा ज वयापारी वनिािररत मलय पर माल नही ो

बचत थ उनक दसणडत वकया जाता था

Q38) उततर (d)

सपषटीकरण

- वदलली सििपरथम त मर राजपत ो क अिीन उनक

सामराजय की राजिानी बनी थी

- 12िी ो शताबदी क मधय म अजमर क चौहान ो

(वजनह चाहमान ो क नाम स भी जाना जाता ह) न

त मर राजपत ो क परावजत वकया था

- त मर ो और चौहान ो क अिीन वदलली एक

महतवपणि िावणसजयक क दर बन गया था

- कई जन वयापारी यहाा रहन लग थ और उनह ोन

कई मोवदर भी बनिाए

- यहाा पर मवदरत वसक वजनह ldquoदहलीिालrdquo क नाम

स जाना जाता था वयापक रप स परचलन म थ

Q39) उततर (c)

सपषटीकरण

- म ठ की मसिद का वनमािण वसको दर ल दी क

राजयकाल म उसक मिी क दवारा करिाया गया

था

- बगमपरी मसिद का वनमािण महममद तगलक क

शासनकाल म हआ था

- यह मसिद विशव का पणयथथान (The

Sanctuary of the World) और वदलली म महममद

तगलक की नई राजिानी जहाोपनाह की मखय

मसिद थी

- कववत- अल - इसलाम मसिद का विसतार

इलतसिश और अलाउददीन सखलजी न वकया था

- मीनार का वनमािण तीन सलतान ो कतबददीन ऐबक

इलतसिश और वफर ज शाह तगलक क दवारा

करिाया गया था

Q40) उततर (c)

सपषटीकरण

- मगल ो क अिीन मनसबदार शबद उस वयसकत क

वलए सोदवभित वकया जाता था वजसक पास मनसब

(अथाित पद) ह ता था

- उस अपना ितन राजसव कायो वजनह जागीर कहत

थ क रप म परापत ह ता था

Q41) उततर (b)

सपषटीकरण

- ldquoभारत छ ड आोद लनrdquo वबरविश शासन क

सखलाफ ल ग ो का एक सवाभाविक विदर ह था

- असखल भारतीय काोगरस सवमवत न 8 अगसत 1942

क बमबई म एक बठक का आय जन वकया था

- इस बठक म परवसद सोकलप ldquoभारत छ ड rdquo क

पाररत वकया गया और इस उददशय क परापत करन

क वलए गाोिी क नततव म एक अवहोसक जन सोघिि

आोद लन की शरआत का परसताि वदया गया

- लवकन अगल ही वदन गाोिी और काोगरस क अनय

परमख नताओो क वगरफतार कर वलया गया

- काोगरस क एक बार वफर अिि घ वित वकया गया

था

Q42) उततर (c)

सपषटीकरण

- साइमन कमीशन यनाइविड वको गडम क सात

साोसद ो का एक समह था

- इस वबरविश भारत क वलए सोििावनक सिार ो का

सझाि दन क वलए गवठत वकया गया था

- इस आय ग म िररषठ वबरविश राजनता सर जॉन

साइमन क नततव म किल वबरविश सदसय ही

शावमल थ

- इसवलए भारत क ल ग ो न साइमन कमीशन क

आगमन क विरद आोद लन वकया था

Q43) उततर (a)

सपषटीकरण

bull दादा भाई नौर जी भारत म वबरविश शासन क

आवथिक पररणाम ो क बार म अपनी विर िी

(परवतकल) राय क वलए जान जात थ

FC19H1003 35

bull अपन कई लख ो और भािण ो म विशि रप स

ldquoपाििी एो ड अन-वबरविश रल इन इसणडया

(Poverty and Un-British Rule in India) म

नौर जी न यह तकि वदया वक भारत पर अतयविक

कर लगाया गया था और इसकी सोपवतत इोगलड की

ओर परिावहत की जा रही थी

bull उनह ोन पराचीन भारतीय गरोथ ो की वयाखया करन

का और भारतीय ो क आिविशवास क बहाल

करन पर कायि नही ो वकया था

उनह ोन वकसी और बात स पहल सभी सामावजक

बराइय ो क उनमलन की आिशयकता पर भी बल

नही ो वदया था

Q44) उततर (c)

सपषटीकरण

bull अगसत 1932 म वबरविश परिानमोतरी मकड नालड न

अपन साोपरदावयक परसकार (The Communal

Award) की घ िणा की थी

bull यह भारत क कई साोपरदावयक वहत ो क बीच विवभनन

सोघिो क हल करन क वलए वबरिन का एकतरफा

परयास था

bull यह परसकार (Award) बाद म 1935 क

अविवनयम (The Act of 1935) म शावमल वकया

गया था

bull इस साोपरदावयक परसकार न मससलम ो क वलए

आरवकषत एक अलग वनिािचक मणडल फॉमिल का

विसतार अनय अलपसोखयक ो क वलए वकया था

वजसम वसख ो भारतीय ईसाइय ो आोगल-भारतीय

समदाय यर पीय समदाय तथा विवशषट कषतरीय

समह ो क शावमल वकया गया था

bull गाोिी न इस परसताि क भारतीय समाज क

विभावजत करन क वलए एक घवणत वबरविश

सावजश क रप म दखा और उसक सखलाफ

आमरण अनशन वकया

Q45) उततर (b)

सपषटीकरण

मौजदा आयात और वनयाित क अवतररक़त

औपवनिवशक भारत क वनमनवलसखत खचो क

वलए एक विशिवनवशचत िन रावश भी दनी पडती

थी

(i) परशासन क वयय

(ii) सना क रख-रखाि क वयय

(iii) यद क वयय

(iv) सिावनितत अविकाररय ो की पशन तथा

(v) वबरिन दवारा अपनी उपवनिश बसती

(कॉल नी) क रख-रखाि क वयय

इनह गह शलक (Home Charges) क रप म

जाना जाता था और लगभग परी तरह स भारत क

दवारा इनका भगतान वकया जाता था

bull गह शलक म वनमनवलसखत घिक शावमल थ

(i) भारतीय ऋण पर दय बयाज

(ii) ईसट इोवडया को पनी क शयरिारक ो क

लाभाोश

(iii) लोदन म भारत कायािलय चलान क वलए िन

(iv) भारत म वनयकत वबरविश कवमिय ो क ितन

और पशन का भगतान करन क वलए िन

(v) रलि पर बयाज

(vi) नागररक और सनय शलक

(vii) इोगलड म सट र (सामगरी) की खरीद

Q46) उततर (b)

सपषटीकरण

bull भारतीय राषटर ीय काोगरस का लाहौर सतर 1929 म

जिाहरलाल नहर की अधयकषता म आय वजत

वकया गया था

bull इस सतर म भारतीय राषटर ीय आोद लन स समबसित

कई महतवपणि पररणाम सामन आय थ

(i) सििपरथम इस सतर म काोगरस क अधयकष पद

पर जिाहरलाल नहर क चना गया था ज

काोगरस म िामपोवथय ो की बिती हई ताकत

का सपषट सोकत था

(ii) दसरा इस सतर म पहली बार काोगरस न पणि

सवतोतरता की माोग क उठाया था

इस परकार की माोग काोगरस मोच स पहल कभी भी

नही ो उठाई गई थी

Q47) उततर (b)

सपषटीकरण

FC19H1003 36

bull इस ररप िि न वकसी भी समदाय क वलए पथक

वनिािचक मोडल अथिा अलपसोखयक ो क वलए

भाराोश की वसफाररश नही ो की थी

bull तथावप इस ररप िि न उन पराोत ो म अलपसोखयक

सीि ो क आरकषण की अनमवत दी थी जहाा पर कम

स कम दस परवतशत अलपसोखयक ह

bull लवकन यह समदाय क आकार क अनपात म ह ना

चावहए था

bull इस ररप िि म भारत क वलए पणि सवतोतरता क

वलए क ई पराििान नही ो था

Q48) उततर (c)

सपषटीकरण

bull आरो वभक िवदक आयो का िमि मखय रप स

परकवत की पजा और यजञ था

bull परारो वभक आयि िमि परकवत की पजा क समान था

bull िासति म उनक चार ो ओर की शसकतयाा वजनह न

त ि वनयोवतरत कर सकत थ और न ही समझ पाए

थ उनह वदवयता क साथ वनिवशत वकया गया तथा

उनह मादा या नर दिीदिताओो क रप म

परतीकतव वकया गया था

bull उनह ोन कछ यजञ ो का भी वनषपादन वकया था

Q49) उततर (b)

सपषटीकरण

bull सडक और नदी-मागि (जल-मागि) डकती स

सरवकषत नही ो थ

bull उललखनीय ह वक हिििििन क शासनकाल क

दौरान यआन चिाोग (हयएन साोग) का सारा

सामान लि वलया गया था

Q50) उततर (c)

सपषटीकरण

परशन म वदए गए द न ो कथन सही ह

Q51) उततर (b)

सपषटीकरण

bull परोदर दास एक सोत और भगिान कषण क एक

महान भकत थ

bull परोदर दास क कनाििक सोगीत क वपतामह क

रप म जाना जाता ह

bull यदयवप उनक जनम-थथान क बार म काफी

अिकल लगाई जाती रही ह

bull तथावप अब कननड विशवविदयालय हमपी क दवारा

गवठत एक विशिजञ सवमवत इस वनषकिि पर पहोची

ह वक उनका जनम थथान सोभितया कनाििक का

एक छ िा-सा गााि कषमपरा (वशिम गगा वजला)

था

Q52) उततर (c)

सपषटीकरण

bull शरी तयागराज शरी शयाम शासतरी और शरी मथसवामी

दीवकषतर क कनाििक सोगीत की वतरमवति माना

जाता ह

bull उनक कारण ही 18िी ो-19िी ो शताबदी म कनाििक

सोगीत का सववणिम यग आया था

Q53) उततर (d)

सपषटीकरण

bull अभी हाल ही म लौह यगीन-महापािावणक काल

का 2000 ििि पराना एक दलिभ सारक फगस

(Sarcophagus) (पतथर का ताबत) क ललम क

वियर गाोि (क वयलडी क पास वजला क वझक ड

करल राजय) की एक रॉक-कि गफा स ख जा गया

bull यह ताबत वजसम हविय ो क िकड थ खदाई क

दौरान वमला

bull अभी तक इस परकार की दलिभ ख ज करल क

मातर द ही थथान ो स हई ह

bull य द न ो सारक फगी (Sarcophagi) (पतथर क

ताबत) चियर और अथ ली (वजला क वझक ड) क

महापािाण थथल ो स वमल ह

Q54) उततर (a)

सपषटीकरण

FC19H1003 37

दवकषण भारत म महापािाण सोसकवत एक पणि

विकवसत लौह यगीन सोसकवत थी

Q55) उततर (d)

सपषटीकरण

bull च ल पाणडय और करलपतर (चर) इन तीन ो का

उललख अश क क अवभलख ो म वकया गया ह

bull सोभितः य भौवतक सोसकवत क उततर

महापािावणक चरण म थ

Q56) उततर (d)

सपषटीकरण

bull भीमा-क रगाोि की लडाई ततीय आोगल-मराठा

यद का वहससा थी

Q57) उततर (b)

सपषटीकरण

bull राजकमार शकल न गाोिीजी क चोपारण आन तथा

वतनकवथया परणाली स जडी समसया की जाोच क

वलए रारी करन क वलए दश भर म उनका

अनसरण वकया था

bull बज वकश र राजदर परसाद महादि दसाई और

नरहरी पाररख चोपारण सतयागरह क दौरान गाोिी

जी क सहय गी थ

Q58) उततर (b)

सपषटीकरण

bull बराहमण ो और बौद मठिाररय ो क कर-मकत गााि

अनदान म दन की परथा सतिाहन ो न आरमभ की

थी

Q59) उततर (c)

सपषटीकरण

इस कायिकरम क उददशय वनमनानसार ह

(i) बवनयादी पयििन आिाररक सोरचना का विकास

करना

(ii) चयवनत (पहचान वकय गए) कषतर ो म आजीविका क

सजन क वलए दश क साोसकवतक और विरासत

मलय ो क बिािा दना

(iii) विरासत समारक थथल ो पर विशव सतरीय आिाररक

सोरचना विकवसत करक एक सतत तरीक स

पयििक आकििण म िसद करना

(iv) थथानीय समदाय ो की सवकरय भागीदारी क माधयम

स र रगार ो का सजन करना

(v) र रगार उतपादन और आवथिक विकास क वलए

पयििन कषमता का उन पर परभाि का उपय ग

करना तथा

(vi) िारणीय पयििन आिाररक सोरचना का विकास

करना और उसका उवचत सोचालन तथा

रखरखाि सवनवशचत करना

Q60) उततर (b)

सपषटीकरण

bull यह वनकाय ििि 1987 म अससततव म आया था

bull यह एक राषटर ीय सतर का शीिि सोगठन ह ज भारत

सरकार क जनजातीय मामल ो क मोतरालय क

परशासवनक वनयोतरण क अिीन काम कर रहा ह

bull इसका पोजीकत और परिान कायािलय नई वदलली

म सथथत ह

Q61) उततर (c)

सपषटीकरण

bull परमचोद क उपनयास ो म परमाशरम रोगभवम गबन

कमिभवम और ग दान शावमल ह

bull ग रा रिी ोदरनाथ िग र क दवारा रवचत उपनयास ह

bull अभी हाल ही म मोशी परमचोद की 138िी ो जयोती दश

भर म मनाई गई थी

Q62) उततर (b)

सपषटीकरण

bull ldquoवगदाrdquo पोजाब (भारत) एिो पावकसतान की

मवहलाओो क दवारा तयौहार क समय और फसल

की बिाई तथा किाई क अिसर पर वकया जान

िाला एक पारोपररक दहाती नतय ह

FC19H1003 38

bull इस नतय क माधयम स पोजाबी मवहलाऐो अपनी

परसननता परकि करती ह तथा वगदा क परदशिन क

माधयम स परि िचिसव िाल समाज म मवहलाओो

की दबी हई भािनाओो क परकि करती ह

bull चोवक इस नतय का परि ो क साथ क ई सोबोि नही ो

ह अतः किल मवहलाऐो ही इसम भाग ल सकती

bull हर साल तीज समार ह क दौरान पोजाब म वगदा

नतय वकया जाता ह

तीज भारत क कछ भाग ो म मवहलाओो क दवारा

मनाया जान िाल कई तयौहार ो क वलए एक

वयापक नाम ह

Q63) उततर (a)

सपषटीकरण

- मजम-उल-बहरीन या द समदर ो का सोगम

नामक उललखनीय रचना दारा वशक ह क दवारा

वलखी थी

- भारत क उपराषटर पवत शरी एम िकया नायड न कहा

ह वक राजकमार दारा वशक ह की रचनाएा शाोवत

और सदभाि क बिािा दन क वलए एक तारा सर त

क रप म सामन आ सकती ो ह

- उपराषटर पवत गत ििो क भला वदए गए राजकमार

दारा वशक ह क परदवशित परचवलत करन हत

आय वजत एक परदशिनी का दौरा करन क बाद एक

सभा क सोब वित कर रह थ

- इस परदशिनी का आय जन फर क इस गौवियर

(Francois Gautier) क दवारा lsquoइोवदरा गाोिी नशनल

सिर फॉर द आििसrsquo (The Indira Gandhi

National Centre for the Arts) नई वदलली म

वकया गया था

Q64) उततर (c)

सपषटीकरण

- ग मतशवर परवतमा जन भगिान बाहबली क

समवपित ह

- यह एक एक-चटटानी पतथर की मवति ह

- राषटर पवत राम नाथ क विोद न शरिणबलग ला

(कनाििक) म आय वजत वकय जान िाल भवय

अवभिक समार ह महामसतकावभिक का

उदघािन वकया था

- यह समार ह 12 ििो म एक बार ह ता ह

Q65) उततर (c)

सपषटीकरण

bull पराची घािी पराची नदी क चार ो ओर फली हई थी

bull पराची घािी िीर-िीर विलपत ह गई थी

bull पराची नदी भिनशवर स वनकलती ह

bull यह महानदी की एक सहायक नदी ह और यह

परी खदाि किक तथा जगतवसोहपर वजल ो स

ह कर बहती ह

bull इस नदी क पर कषतर क पराची घािी कहा जाता ह

bull यह नदी बोगाल की खाडी म वगरती ह

परातासतवक साकषय स पता चलता ह वक पराची घािी

सभयता हडपपा और म हनज दाड द न ो की

पिििती ह

Q66) उततर (d)

सपषटीकरण

य समारक छतरपर वजल (मधय परदश) म विोधयाचल

पिित शरोखला म सथथत ह

Q67) उततर (a)

सपषटीकरण

bull थॉिस ऑन पावकसतान नामक पसतक डॉ बी

आर अमबडकर न वलखी थी

bull डॉ बी आर अमबडकर की जयोती क अिसर पर

भारत क राषटर पवत न भारत की इस महान हसती

क शरदाोजवल अवपित की थी

bull डॉ बी आर अमबडकर न 1924 म वडपरथड

कलावसर इोसटीटयि (दवलत िगि सोथथान -

बवहषकत वहतकाररणी सभा) और 1927 म समाज

समता सोघ की थथापना की थी

bull अमबडकर का धयान वशकषा कषतर की ओर भी था

bull उनह ोन वशकषा क वनमन िगो म फलान क वलए

पीपलस एजकशन स साइिी (The Peoples

Education Society) क नाम स महाविदयालय ो क

नििकि और छातरािास ो की थथापना की थी

FC19H1003 39

Q68) उततर (b)

सपषटीकरण

bull महरगि भारतीय उपमहादवीप म एक परवसद

निपािाण बसती ह ज बलवचसतान पराोत

पावकसतान म सथथत ह

bull दचपलली (आोधर परदश) क पास नागलर नदी क

पिी ति ो पर चना पतथर क बलॉक क विशाल

विसतार म एक पिि-ऐवतहावसक रॉक आिि थथल की

ख ज की गई ह

bull इसन 1500-2000 ईसा पिि क दौरान गोिर (आोधर

परदश) म विकवसत निपािाण सभयता पर परकाश

डाला ह

Q69) उततर (c)

सपषटीकरण

bull 12िी ो सदी और 13िी ो सदी म काकाविय िोश का

उदय हआ था

bull ि पहल कलयाण क पवशचमी चालकय ो क सामोत थ

bull परारोभ म उनह ोन िारोगल (तलोगाना) क पास एक

छ ि स कषतर पर शासन वकया था

bull उनह ोन ldquoनायक वयिथथाrdquo की शरआत की थी

वजस बाद म विजयनगर क राय शासक ो न

अपनाया और विकवसत वकया था

Q70) उततर (a)

सपषटीकरण

bull गाोिीजी क अनशन स वमल मावलक ो पर दबाि

पडा था ज अोततः शरवमक ो क ितन म 35 परवतशत

की िसद करन क वलए सहमत हए थ

bull गगल (Google) न अनसया साराभाई वजनह ोन

भारत क शरवमक आोद लन म एक अगरणी भवमका

वनभाई थी की 132िी ो जयोती डडल (Doodle) का

वनमािण करक मनाई

Q71) उततर (d)

सपषटीकरण

भारत स यनसक की मानिता की अमति साोसकवतक

विरासत की परवतवनवि सची म वनमनवलसखत शावमल ह

bull कवडयटटम करल का सोसकत रोगमोच

bull मवडयिि करल का अनषठान रोगमोच और नतय

नाविका

bull िवदक मि जाप की परोपरा

bull राजथथान क कालबवलया ल क गीत और नतय

bull रामलीला रामायण का पारोपररक परदशिन

bull सोकीतिन मवणपर का अनषठान गायन ढ ल िादन

और नतय

bull रममन भारत क गििाल वहमालय का िावमिक

तयौहार और अनषठान रोगमोच

bull जाोदीयाला गर पोजाब क ठठर ो की पीतल और

ताोब क वशलप स वनवमित बतिन ो की पारोपररक कला

bull छाऊ नतय पिी भारतीय राजय ो म जनमी शासतरीय

भारतीय नतय कला

bull लददाख का बौद मि जाप िर ाोस-वहमालयी लददाख

कषतर तथा जमम-कशमीर म पवितर बौद गरोथ ो का पाठ

bull य ग

bull नौर र

bull को भ मला

Q72) उततर (b)

सपषटीकरण

bull भारत क राषटर पवत शरी राम नाथ क विोद न

वकसामा नागालड म हॉनिवबल मह रति और

राजय गठन वदिस समार ह का उदघािन वकया

था

bull हॉनिवबल मह रति का नाम भारतीय हॉनिवबल क

नाम पर पडा ह ज एक विशाल और रोगीन जोगली

पकषी ह

bull यह पकषी नागालड राजय की अविकतर जनजावतय ो

की ल ककथाओो म उसललसखत ह

bull नागालड की परमख मानयता परापत जनजावतयाा ह

अोगामी आओ चखसोग चाोग ककी रगमा और

रवलोग आवद

bull ओोग जारिा और ससिनलीस अोडमान-वनक बार

दवीप समह की जनजावतयाा ह

FC19H1003 40

Q73) उततर (c)

सपषटीकरण

bull दकन म राषटर कि शासन दसिी ो सदी क अोत तक

लगभग 200 ििो तक रहा था

bull राषटर कि शासक अपन िावमिक विचार ो म सवहषण

bull उनह ोन न किल शि िमि और िषणि िमि बसलक

जन िमि क भी सोरकषण वदया था

bull एल रा म वशि क परवसद रॉक कि मोवदर का

वनमािण नौिी ो सदी म राषटर कि राजा कषण परथम न

करिाया था

bull उसका उततराविकारी अम घििि जन था लवकन

उसन अनय िमो क भी सोरकषण परदान वकया था

bull राषटर कि ो न मसलमान वयापाररय ो क बसन की

अनमवत दी थी

bull उनह न अपन अविराजय ो म इसलाम क उपदश दन

की भी अनमवत दी थी

bull अभी हाल ही म पाोडिलागटटा (तलोगाना) क

परागवतहावसक चटटान वचतर ो क कषरण की बिती हई

घिनाएा एक गोभीर वचोता का वििय ह

bull यह परागवतहावसक चटटान क नकसान पहाचा

सकता ह

bull पाोडिलागटटा वनमनवलसखत क वलए जाना जाता ह

- 10000 ईसा पिि स 8000 ईसा पिि क वचवतरत

चटटानी आशरय ो क वलए

- राषटर कि काल क एक 8 िी ो सदी क

वशलालख क वलए और

- 12िी ो सदी क काकविय सामराजय क वभवतत

वचतर ो क वलए

Q74) उततर (b)

सपषटीकरण

bull 1828 म राजा राम म हन रॉय न एक नय िावमिक

समाज बरहम सभा की थथापना की थी वजस बाद

म बरहम समाज क नाम स जाना गया था

bull दिदरनाथ िग र न ततवब विनी सभा की अधयकषता

की थी ज आधयासिक सतय की ख ज म सोलि

थी

bull इसका उददशय वहोद िमि क शद करन का और

एकशवरिाद (एक ईशवर म आथथा) का परचार करना

था

bull नय समाज की थथापना क आिार थ कारण

(तकि ) क द सतमभ तथा िद और उपवनिद

bull अभी हाल ही म सािारण बरहम समाज का कछ

काननी मदद ो क लकर पवशचम बोगाल सरकार क

साथ काननी वििाद चल रहा ह

Q75) उततर (c)

सपषटीकरण

bull भारत म वचशती वसलवसल की थथापना खवाजा

म इनददीन वचशती क दवारा की गयी थी

bull ि 1192 ईसवी क आसपास भारत आय थ

bull वचशतीय ो क बारहिी ो शताबदी क उततरािि म भारत

म आन िाल सफीय ो क समह ो म सबस

परभािशाली माना जाता ह

bull उनह ोन थथानीय िातािरण क साथ सफलतापििक

अनकलन वकया और उनह ोन भारतीय भसकत

परोपराओो क कई पहलओो क अपनाया

bull अजमर म सफी अपरकि खवाजा म इनददीन वचशती

की ऐवतहावसक दरगाह क एक नया रप दन की

तयारी की जा रही ह

bull इस 13िी ो शताबदी की दरगाह क ldquoसवचछ

आइकॉवनक थथल ोrdquo (Swacch Iconic Places) म

शावमल वकया गया ह ज परवतवषठत विरासत

आधयासिक और साोसकवतक थथान ो पर क वदरत

य जना ह

FC19H1003 41

ANSWERS amp EXPLANATION OF

NCERT History Class VI-X + Current Affairs

(FC19E1003)

Q1) Answer c

Explanation

Rigveda consists of more than a

thousand hymns dedicated to gods and

goddesses These hymns were

composed by sages and learnt by men

however a few were composed by

women like Apala Ghosa Lopamudra

Maitreyi and Gargi

Rigveda consists of many hymns in the

form of dialogues We get an example of

a dialogue between a sage named

Vishwamitra and two rivers (Beas and

Sutlej) that were worshipped as

goddesses This suggests that he

belonged to the Vedic period

Q2) Answer b

Explanation

Traces of ash have been found from

Kurnool Caves suggesting that people

were familiar with the use of fire

It is situated in Andhra Pradesh

Q3) Answer c

Explanation

Burzahom is a prehistoric site in

present day Kashmir where people built

pit houses which were dug into the

ground with steps leading into them

These may have provided shelter in cold

weather

Q4) Answer c

Explanation

Epigraphy is defined as the study of

inscriptions

Manuscriptology is the study of history

and literature through the use of hand

written documents

Palaeography refers to the study of

ancient writing systems and the

deciphering and dating of historical

manuscripts

Numismatics refers to the study of

coins

Q5) Answer a

Explanation

Charaka Samhita was written by

Charaka and is an important book on

Ayurveda and medicine

He was a practitioner of the traditional

system of Indian medicine known as

Ayurveda

Charaka is thought to have flourished

sometime between the 2nd century BCE

and the 2nd century CE

Q6) Answer b

Explanation

Bhaga refers to the tax on crops which

was fixed at 16th of the production

Kammakaras is the term used for the

landless agricultural labour class

Ashvamedha also known as horse

sacrifice is a ritual where a horse is let

loose to wander freely and it was

guarded by the rajarsquos men

Q7) Answer (d)

Explanation

In the Rigvedic period horses were

yoked to chariots that were used in

battles fought to capture land cattle

etc This suggests that the use of horse

chariots began much before the period

of Mahajanapadas

The battles were fought in the Rigvedic

period for cattlersquos lands water an even

to capture people Most men took part

in these wars however there was no

regular army but there were assemblies

where people met and discussed

matters of war Regular armies became

a feature in the Mjahajanapada period

including vast armies of foot soldiers

chariots and elephants

RAUSIAS-FC19E1003 42

Q8) Answer (a)

Explanation

Buddha belonged to the Sakya clan and

passed away at Kusinara

Buddha taught in Prakrit which was the

common language of people

Q9) Answer c

Explanation

There were six schools of philosophy in

ancient India These are known as

Vaishesika Nyaya Samkhya Yoga

Purva Mimansa and Vedanata or Uttara

Mimansa They were founded by sages

Kanada Gautama Kapila Patanjali

Jamini and Vyasa respectively

Q10) Answer b

Explanation

The teachings of Mahavira were

compiled at Valabhi in 6th century AD

Q11) Answer (c)

Explanation

Chanakya is traditionally identified as

Kautilya or Vishnugupta who authored

the ancient Indian political treatise the

Arthashastra

Q12) Answer d

The national emblem of India is an

adaptation of the Lion Capital atop the

Ashoka Pillar of Sarnath Uttar Pradesh

and is combined with the National

Motto Satyameva Jayate

The Rampurva Bull gets the name from

the site of its discovery Rampurva in

Bihar

It is noted for its delicately sculpted

model demonstrating superior

representation of soft flesh sensitive

nostrils alert ears and strong legs It is

a mixture of Indian and Persian

elements

Sankissa is situated in Uttar Pradesh

India

Q13) Ans(a)

Kunwar Singh was a notable leader during the Revolt of 1857 He belonged

to a royal house of Jagdispur Bihar

Q14) Answer b

Explanation

The term Vellalar was used for large

landowners

Q15) Answer c

Explanation

Arikamedu was a coastal settlement

where ships unloaded goods from

distant lands Finds here include a

massive brick warehouse pottery

including amphorae and Arretine ware

Roman lamps glassware and gems have

also been found at the site

Q16) Answer a

Explanation

Muvendar is a Tamil word mentioned in

Sangam poems meaning three chiefs

used for the heads of three ruling

families the Cholas Cheras and

Pandyas

Q17) Ans (c)

Several tribal or kin-based assemblies

such as the Sabha Vidatha and gana

are mentioned in the Rig-veda The

Sabha and the samiti mattered a great

deal in early Vedic times so much so

that the chiefs or the kings showed an

eagerness to win their support

Q18) Ans (a)

Jainism recognised the existence of the

gods but placed them lower than the

jina and did not condemn the varna

system as Buddhism did

Q19) Answer (d)

Explanation

Cholas and Pandyas had developed

powerful coastal cities The most

important city of Cholas was Puhar or

Kaveripattinam and Madurai was the

capital of Pandyas

Q20) Answer b

Explanation

Buddhacharita is the biography of

Buddha and was written by

RAUSIAS-FC19E1003 43

Ashvaghosha

Q21) Answer (a)

Explanation

Tamil poet Appar was a Shiva devotee

So he was a Nayanar saint

Q22) Answer d

Explanation

Samudragupta was a prominent Gupta

ruler whose coins depict him playing a

veena indicating his love for music We

get important historic information from

his Allahabad Prashasti which was

composed by his court poet Harisena

Q23) Answer (b)

Explanation

Vikrama Samvat was founded by

Chandragupta II in the 58 BC as a

mark of victory over the Shakas and

assumed the title of Vikramaditya

Banabhatta wrote Harshavardhanarsquos

biography the Harshacharita in

Sanskrit

Q24) Answer c

Explanation

Sandhi-vigrahika was the minister of

war and peace

Sarthavaha was the leader of the

merchant caravans

Q25) Answer a

Explanation

Xuan Zang (Hsuan-tsang) was a

Chinese traveller who came during the

reign of Harshavardhana

In the decade that began in 630 AD

Xuan Zang came to India through

Kashmir after visiting Central Asia Iran

and Afghanistan

He travelled from north to east and lived

in Bihar for a couple of years

At Nalanda University Xuan Zang

interacted with students and scholars

mastered local languages and

discovered Buddhist stupas

Q26) Answer c

Explanation

Pradakshina patha is a circular path

laid around a stupa in Buddhist

architecture While the rest are a part of

temple architecture

Q27) Answer d

Explanation

All the above-mentioned temples have

an elaborate use of bricks (baked

bricks) along with stone

Q28) Ans (c)

Muhammad Quli Qutab was the Sultan

of Golconda He was a contemporary of

Akbar was very fond of literature and

architecture

The Sultan was a great poet and he

wrote in Dakhini Urdu Persian and

Telgu and has left an extensive diwan or

collection

Recently the Archaeological Survey of

India (ASI) will be using Ground

Penetrating Radar (GPR) to map the

contours of the area around the Bagh-e-

Naya Qila excavated garden inside the

Golconda Fort in Telangana

Q29) Answer a

Explanation

Silappadikaram is a famous Tamil epic

which was written by Ilango around

1800 years ago It is a story of a

merchant named Kovalan who fell in

love with a courtesan named Madhavi

Manimekalai tells the story of the

daughter of Kovalan and Madhavi

Q30) Answer (a)

Explanation

Charaka is the author of Charaka

Samhita which is an important work of

Ayurveda and medicines

Brahmaguptarsquos fame rests mostly on his

Brahma-sphuta-siddhanta which was

an astronomical work It was translated

into Arabic in Baghdad and had a major

impact on Islamic mathematics and

astronomy

Late in his life Brahmagupta wrote

Khandakhadyaka which was an

RAUSIAS-FC19E1003 44

astronomical handbook that employed

Aryabhatarsquos system of starting each day

at midnight

Q31) Answer (c)

Explanation

Amir Khusrau was a famous sufi

musician poet and scholar In 1318 he

noted that there was different language

in every region of this land (Hindustan)

Lahori Kashmiri Dvarsamudri (in

Southern Karnataka) Telangana (in

Andhra Pradesh) Gujari (in Gujarat)

Marsquobari (in Tamil Nadu) Awadhi (in

eastern Uttar Pradesh) and Hindawai (in

the area around in Delhi) etc He went

to explain that Sanskrit did not belong

to any region and that only brahmans

knew it

Q32) Answer c

Explanation

Hiranyagarbha refers to the golden

womb When this ritual was performed

with the help of Brahmanas it was

thought to lead to the rebirth of the

sacrificer as a Khastriya

Q33) Answer d

Explanation

Kadamai refers to a tax on land

revenue

Gwalior Prashasti describes the exploits

of Nagabhata who was a Pratihara king

Q34) Answer b

Explanation

Rajatarangini is a Sanskrit text written

by Kalhana in the 12th century

It was historical chronicle of early India

It is justifiably considered to be the best

and most authentic work of its kind

It covers the entire span of history in

the Kashmir region from the earliest

times to the date of its composition

Q35) Answer c

Explanation

ldquoUrrdquo was the general assembly of the

village ldquoUrrdquo consisted of all the

taxpaying residents of an ordinary

village

Q36) Answer (a)

Explanation

Tarikh was a form of history writing in

the Delhi Sultanate The authors of

tawarikhs were learned men which

included secretaries administrators etc

Q37 Answer (a)

Explanation

Alauddin chose to pay his soldiers salaries in cash rather than iqtas The soldiers would buy their supplies from merchants in Delhi and it was thus feared that merchants would raise their prices To stop this Alauddin controlled the prices of goods in Delhi Prices were carefully surveyed by officers and merchants who did not sell at the prescribed rates were punished

Q38) Answer (d)

Explanation

Delhi first became the capital of a

kingdom under the Tomara Rajputs

who were defeated in the middle of the

twelfth century by the Chauhans (also

referred to as Chahamanas) of Ajmer

It was under the Tomaras and

Chauhans that Delhi became an

important commercial centre Many rich

Jaina merchants lived in the city and

constructed several temples Coins

minted here called dehliwal had a wide

circulation

Q39) Answer (c)

Explanation

Moth ki Masjid was built in the reign of

Sikandar Lodi by his minister

Begumpuri mosque built in the reign of

Muhammad Tughluq was the main

mosque of Jahanpanah the ldquoSanctuary

of the Worldrdquo and his new capital in

Delhi

Quwwat al ndash Islam mosque was

enlarged by Iltutmish and Alauddin

Khalji The minar was built by three

Sultansndash Qutbuddin Aybak Iltutmish

and Firuz Shah Tughluq

RAUSIAS-FC19E1003 45

Q40) Answer (c)

Explanation

Under the Mughals mansabdar was

referred to an individual who held a

mansab ie rank and he received his

salary as revenue assignments called

jagirs

Q41) Ans (b)

The Quit India Movement was a

spontaneous revolt of people against

British rule

The All India Congress Committee met

at Bombay on 8 August 1942 It passed

the famous resolution Quit India and

proposed the starting of a non-violent

mass struggle under Gandhis

leadership to achieve this aim But on

the very next day Gandhi and other

eminent leaders of the Congress were

arrested The Congress was once again

declared illegal

Q42) Ans (c)

The Simon Commission refers to a

group of seven MPs from the United

Kingdom constituted to suggest

constitutional reforms for British India

The Commission consisted of only

British members headed by one of the

senior British politicians Sir John

Simon

So the people of India agitated against

the arrival of Simon Commission

Q43) Ans (a)

He was widely known for his

unfavourable opinion of the economic

consequences of the British rule in

India

In his many writings and speeches and

especially in Poverty and Un-British

Rule in India Naoroji argued that India

was too highly taxed and that its wealth

was being drained away to England

He did not interpret the ancient Indian

texts and restored the self-confidence of

Indians And also he did not stress the

need for eradication of all the social

evils before anything else

Q44) Ans (c)

In August 1932 Prime Minister

MacDonald announced his Communal

Award Great Britainrsquos unilateral

attempt to resolve the various conflicts

among Indiarsquos many communal

interests

The award which was later

incorporated into the act of 1935

expanded the separate-electorate

formula reserved for Muslims to other

minorities including Sikhs Indian

Christians Anglo-Indians Europeans

distinct regional groups Gandhi

undertook a ldquofast unto deathrdquo against

that offer which he viewed as a

nefarious British plot to divide the

Indian society

Q45) Ans (b)

In British India apart from existing

imports and exports there was also a

particular amount of money which

colonial India contributed towards

administration maintenance of the

army war expenses pensions to retired

officers and other expenses accrued by

Britain towards maintenance of her

colony These were known as Home

charges and were paid for almost

entirely by India

The Home charges was made of

following components-

- Interest payable on Indian debt

- Dividend to shareholders of East

India Company

- Funds used to support the India

Office in London

- Funds used to pay salaries and

pensions of British personnel

engaged in India

- Interest on the railways

- Civil and military charges

- Store purchases in England

Q46) Ans (b)

The Lahore session of the Indian

National Congress was held in 1929

under the Presidentship of Jawaharlal

Nehru

The Lahore session of the Indian

National Congress witnessed significant

RAUSIAS-FC19E1003 46

developments in the Indian national

movement

- First the election of Jawaharlal

Nehru to the post of Presidentship of

the Congress was a clear indication

of the growing strength of the

Leftists in the Congress

- Secondly it was in this session that

the Congress for the first time raised

the demand for complete

independence Such demand was

not raised from the Congress

platform earlier

Q47) Ans (b)

It did not provide for separate

electorates for any community or

weightage for minorities However it did

allow for the reservation of minority

seats in provinces having minorities of

at least ten per cent but this was to be

in strict proportion to the size of the

community

There was no provision for complete

Independence for India

Q48) Ans (c)

The religion of early Vedic Aryans was

primarily of worship of nature and

Yajnas

The early Aryan religion was kind of

nature worship Actually the forces

around them which they could not

control or understand were invested

with divinity and were personified as

male or female gods And they

performed some Yajnas also

Q49) Ans (b)

The roads and river-routes were not

immune from robbery It is notable that

Yuan Chwang (Hiuen Tsang) was

robbed of his belongings during

Harshvardanarsquos period

Q50) Ans (c)

Q51) Ans (b)

Purandara Dasa was a saint and great

devotee of Lord Krishna

There is much speculation about where

Purandara Dasa regarded as the

Pitamaha of Carnatic music was born

Recently an expert committee

constituted by the Kannada University

Hampi has come to the conclusion that

Kshemapura Shivamogga district

Karnataka is the birth place of

Purandara Dasa

Q52) Ans (c)

Sri Tyagaraja Sri Shyama Shastry and Sri Muthuswami Dikshitar are considered the trinity of Carnatic music and with them came the golden age in Carnatic music in the 18th-19th

century

Q53) Ans d)

Recently a rare sarcophagus (stone

coffin) which is 2000 years old from the

Iron AgendashMegalithic era was discovered

from a rock-cut cave at Viyur village of

Kollam near Koyilandy in Kozhikode

district Kerala

The coffin containing bone fragments

was found during an excavation ldquoSo

far such a rare finding has been

discovered only from two sites

in Kerala Both these sarcophagi were

recovered from Megalithic sites at

Chevayur and Atholi also in Kozhikode

district

Q54) Ans a)

The megalithic culture in South India was a full-fledged Iron Age culture

Q55) Ans d)

The Cholas Pandyas and Keralaputras

(Cheras) mentioned in Ashokan

inscriptions were probably in the late

megalithic phase of material culture

Q56) Ans d)

Q57) Ans (b)

Raj Kumar Shukla followed Gandhiji all

over the country to persuade him to

come to Champaran to investigate the

problem associated with tinkathia

system

RAUSIAS-FC19E1003 47

Brij Kishore Rajendra Prasad Mahadev

Desai and Narhari Parikh accompanied

Gandhi ji during the Champaran

Satyagraha

Q58) Ans (b)

The Satvahanas started the practice of granting tax-free villages to brahmanas and Buddhist monks

Q59) Ans c)

The objectives of the Programme are

listed as under

- Developing basic tourism

infrastructure

- Promoting cultural and heritage

value of the country to generate

livelihoods in the identified regions

- Enhancing the tourist attractiveness

in a sustainable manner by

developing world-class

infrastructure at the heritage

monument sites

- Creating employment through active

involvement of local communities

- Harnessing tourism potential for its

effects on employment generation

and economic development

- Developing sustainable tourism

infrastructure and ensuring proper

Operations and maintenance

therein

Q60) Ans (b)

The Tribal Cooperative Marketing

Development Federation of India

(TRIFED) came into existence in 1987

It is a national-level apex organization

functioning under the administrative

control of Ministry of Tribal Affairs

Govt of India

TRIFED has its registered and Head

Office located in New Delhi

Q61) Ans (c)

Premchandrsquos novels include

Premashram Rangabhumi Ghaban

Karmabhumi and Godan

Gora is a novel written by Rabindranath

Tagore

138th birth anniversary of Munshi

Premchand was celebrated across the

country

Q62) Ans (b)

Giddha is a traditional pastoral dance

performed by the women of the Punjab

India and Pakistan at festival times

and at the sowing and reaping of the

harvest

By this dance the Punjabi women

reveal their joy expel their suppressed

feelings in a male dominated society

through the performance of Giddha

Since this dance has nothing to do with

men only women can participate in it

During the Teej celebrations Giddha

dance is celebrated in Punjab every

year Teej is a generic name for a

number of festivals that are celebrated

by women in some parts of India

Q63) Ans (a)

Dara Shukoh wrote the remarkable

work called ldquoMajma-ul-Bahrainrdquo or the

ldquoThe confluence of two seasrdquo

The Vice President of India Shri M

Venkaiah Naidu has said that Prince

Dara Shukohrsquos writings can come as a

refreshing source for infusing peace and

harmony He was addressing the

gathering after visiting the exhibition

that showcases the forgotten Prince of

yesteryears Dara Shukoh organized by

Mr Francois Gautier at Indira Gandhi

National Centre for the Arts in New

Delhi

Q64) Ans (c)

The statue Gommateshwara is

dedicated to the Jain God Bahubali

It is a monolithic statue

President Ram Nath Kovind

inaugurated the grand anointing

ceremony mdash Mahamastakabhisheka mdash

held once in 12 years at

Shravanabelagola (Karnataka)

Q65) Ans (c)

Prachi Valley had come up around the

Prachi river Prachi Valley gradually

disappeared

RAUSIAS-FC19E1003 48

The Prachi river originates from

Bhubaneswar

It is a tributary of the Mahanadi and

flows through the districts of Puri

Khurda Cuttack and Jagatsinghpur

and the entire region of the river is

termed as the Prachi Valley

It falls into the Bay of Bengal

Archaeological evidence shows that the

Prachi Valley Civilisation predates both

Harappa and Mohenjo-Daro

The Prachi river originates from

Bhubaneswar

Q66) Ans (d)

These monuments are located in

Chhatarpur district Madhya Pradesh

within Vindhya mountain range

Q67) Ans (a)

The book lsquoThoughts on Pakistanrsquo was

written by Dr BR Ambedkar

On the occasion of the birth anniversary

of Dr BR Ambedkar the president of

India pays homage to this icon of India

In 1924 he founded the Depressed

Classes Institute (Bahishkrit Hitkarini

Sabha) and in 1927 the Samaj Samata

Sangh

Another area of attention for Ambedkar

was education For its spread among

the low classes he set up a network of

colleges by the name of Peoples

Education Society and founded hostels

Q68) Ans(b)

Mehrgarh is a famous Neolithic

settlement in the Indian subcontinent

which is situated in Baluchistan

province Pakistan

A pre-historic rock art site is discovered

in the vast expanse of limestone blocks

on the eastern banks of Naguleru river

near Dachepalli (Andhra Pradesh) It

has thrown light on the Neolithic

civilisation that flourished in Guntur

(Andhra Pradesh) during 1500-2000

BC

Q69) Ans (c)

The 12th and the 13th centuries saw

the emergence of the Kakatiyas They

were at first the feudatories of the

Western Chalukyas of Kalyana Initially

they ruled over a small territory near

Warangal (Telangana)

They introduced Nayakships which was

later adopted and developed by the

Rayas of Vijayanagara

Q70) Ans (a)

The fast had effect of putting pressure

on mill owners who finally agreed to

give the workers a 35 per cent increase

in wages

Google celebrated with a doodle the

132nd birth anniversary of Anasuya

Sarabhai who played a pioneering role

in Indiarsquos labour movement

Q71) Ans (d)

The UNESCOrsquos list of the representative

list of the intangible cultural heritage of

humanity from India are

- Koodiyattam Sanskrit Theatre of

Kerala

- Mudiyettu ritual theatre and dance

drama of Kerala

- Tradition of Vedic Chanting

- Kalbelia folk songs and dances of

Rajasthan

- Ramlila Traditional Performance of

the Ramayana

- Sankirtana ritual singing

drumming and dancing of Manipur

- Ramman religious festival and

ritual theatre of the Garhwal

Himalayas India

- Traditional brass and copper craft of

utensil making among the Thatheras

of Jandiala Guru Punjab India

- Chhau dance classical Indian dance

originated in the eastern Indian

states

- Buddhist chanting of Ladakh

recitation of sacred Buddhist texts

in the trans-Himalayan Ladakh

region Jammu and Kashmir India

- Yoga

- Nouroz

- Kumbh Mela

RAUSIAS-FC19E1003 49

Q72) Ans(b)

The President of India Shri Ram Nath Kovind inaugurated the Hornbill Festival and State Formation Day celebrations of Nagaland in Kisama

The festival is named after the Indian hornbill the large and colourful forest bird which is displayed in the folklore of most of the states tribes

The major recognized tribes of Nagaland are Angami Ao Chakhesang Chang

Kuki Rengma and Zeling etc

Onge Jarawa and Sentinelese are the

tribes of Andman amp Nicobar Islands

Q73) Ans (c)

The Rashtrakutas rule in the Deccan lasted for almost two hundred years till the end of the tenth century The Rashtrakutas rulers were tolerant in their religious views and patronized not only Shaivism and Vaishnavism but

Jainism as well

The famous rock-cut temple of Shiva at Ellora was built by one of the Rashtrakutas kings Krishna I in the ninth century His successor Amoghavarsha was a Jain but he also

patronized other faiths

The Rashtrakutas allowed Muslims traders to settle and permitted Islam to

be preached in their dominions

Recently increasing defacement at the prehistoric rock paintings of Pandavulagutta Telangana has created a cause for grave concern It can spoil

the prehistoric rock

Pandavulagutta is home to

- Painted rock shelters dating to

10000 BC-8000 BC

- An 8th century inscription of the

Rashtrakuta period and

- Painted frescoes from the 12th century Kakatiya empire

Q74) Ans (b)

In 1828 Raja Ram Mohan Roy founded a new religious society the Brahma Sabha later known as the Brahmo

Samaj

Debendranath Tagore headed the Tattvabodhini Sabha which was

engaged in search of spiritual truth

Its purpose was to purify Hinduism and to preach monotheism or belief in one God

The new society was to be based on the twin pillars of reason and the Vedas and

Upanishads

Recently Sadharan Brahmo Samaj (SBS) has entered into a legal battle with the West Bengal government due

to some legal issue

Q75) Ans (c)

The Chishti order was established in India by Khwaja Moinuddin Chishti who came to India around 1192 The Chishtirsquos are considered to be the most influential of the groups of Sufis who migrated to India in the late twelfth century They adapted successfully to the local environment and adopted several features of Indian devotional

traditions

The historical dargah of Sufi mystic Khwaja Moinuddin Chishti in Ajmer is all set to get a facelift This 13 th century dargah has been included among the Swachh Iconic Places a clean-up initiative focused on iconic

heritage spiritual and cultural places

Page 21: GENERAL STUDIES (PAPER I) · Test is part of Rau’s IAS Test series for Preliminary Exam 2019 FOUNDATION + CURRENT AFFAIRS GENERAL STUDIES (PAPER –I) FOUNDATION TEST –III TOPIC:

RAUSIAS-FC19E1003 21

Q54) Consider the following statements

1 Megalithic people buried goods in

graves

2 The megalithic culture in South

India was a full-fledged Copper

Age culture

Which of the statements given above

isare correct

(a) 1 only

(b) 2 only

(c) Both 1 and 2

(d) Neither 1 nor 2

Q55) Which of the following kingdoms isare

mentioned in the Ashokan inscriptions

1 Cholas

2 Pandyas

3 Keralaputras (Cheras)

Select the correct answer using the code

given below

(a) 1 only

(b) 1 and 2 only

(c) 3 only

(d) 1 2 and 3

Q56) The Battle of Bhima-Koregaon was

fought between the soldiers of the

Company and the strong Peshwa army

(Marathas) under Bajirao II This war

was a part of the

(a) First Anglo-Maratha war

(b) Second Anglo-Maratha war

(c) Third Anglo- Mysore war

(d) Third Anglo-Maratha war

Q57) Consider the following statements

1 Mahadev Desai followed Gandhiji all over the country to persuade him to come to Champaran to investigate the problem associated

with tinkathia system

2 Narhari Parikh accompanied Gandhi ji during the Champaran

Satyagraha

Which of the statements given above isare correct

(a) 1 only

(b) 2 only

(c) Both 1 and 2

(d) Neither 1 nor 2

Q58) Consider the following statements

1 The Nanda Dynasty started the practice of granting tax-free villages to brahmanas and

Buddhist monks

2 The official language of the Satavahanas was Prakrit

Which of the statements given above

isare correct

(a) 1 only

(b) 2 only

(c) Both 1 and 2

(d) Neither 1 nor 2

Q59) Consider the following statements about the objectives of the lsquoadopt a heritagersquo

project

1 It will harness tourism potential for its effects on employment generation and economic

development

2 It will enhance the tourist attractiveness in a sustainable manner by developing world class infrastructure at heritage sites

Which of the statements given above

isare correct

(a) 1 only

(b) 2 only

(c) Both 1 and 2

(d) Neither 1 nor 2

RAUSIAS-FC19E1003 22

Q60) ldquoभारतीय जिजातीय सहकारी निपरि निकास सोघrdquo

(The Tribal Co-operative Marketing

Development Federation of India - TRIFED)

क सोदभण म निमननिखित कथि ो पर निचार कीनजए

1 यह एक राषटर ीय सतर का शीिण सोगठि ह ज

भारत सरकार क गह मोतरािय क परशासनिक

नियोतरर क अरीि काम कर रहा ह

2 इसका मखय उददशय दश म जिजातीय ि ग ो

का सामानजक-आनथणक निकास करिा ह

उपयणकत कथि ो म स कौि-सास सही हह

(a) किि 1

(b) किि 2

(c) 1 और 2 द ि ो

(d) ि त 1 ि ही 2

Q61) निमननिखित म स कौि-सास उपनयास परमचोद क

दवारा नििा गया हनिि गए ह

1 रोगभनम

2 ग दाि

3 ग रा

िीच नदए गए कट का परय ग कर सही उततर चनिए

(a) किि 1

(b) किि 2

(c) किि 1 और 2

(d) 1 2 और 3

Q62) नगदधा ितय क सोदभण म निमननिखित कथि ो पर निचार

कीनजए

1 नगदधा नबहार की मनहिाओो क दवारा तयौहार क

समय और फसि की बिाई तथा कटाई क

अिसर पर नकया जाि िािा एक पारोपररक

दहाती ितय ह

2 इस ितय क दवारा मनहिाऐो अपिी परसननता

परकट करती ह तथा नगदधा क परदशणि क

माधयम स परि िचणसव िाि समाज म

मनहिाओो की दबी हई भाििाओो क परकट

करती ह

उपयणकत कथि ो म स कौि-सास सही हह

(a) किि 1

(b) किि 2

(c) 1 और 2 द ि ो

(d) ि त 1 ि ही 2

Q63) निमननिखित कथि ो पर निचार कीनजए

1 मलला शाह बदखशी दारा नशक ह क

आधयाखतमक गर थ

2 औरोगरब ि मजम-उि-बहरीि या द समदर ो

का सोगम िामक उललििीय रचिा नििी थी

3 दारा नशक ह क अपि पिणज अकबर क गर ो

क उततरानरकारी क रप म दिा गया था

नजसम उसि रानमणक बहििाद और समनवयता

क बढ़ािा नदया था

उपयणकत कथि ो म स कौि-सास सही हह

(a) किि 1 और 3

(b) किि 2

(c) किि 1 और 2

(d) 1 2 और 3

RAUSIAS-FC19E1003 23

Q60) Consider the following statements about

the Tribal Cooperative Marketing

Development Federation of India

(TRIFED)

1 It is a national-level apex

organization functioning under the

administrative control of Ministry

of Home Affairs Government of

India

2 The main objective of TRIFED is

socio-economic development of

tribal people in the country

Which of the statements given above

isare correct

(a) 1 only

(b) 2 only

(c) Both 1 and 2

(d) Neither 1 nor 2

Q61) Which of the following novels isare

written by Premchand

1 Rangabhumi

2 Godan

3 Gora

Select the correct answer using the code

given below

(a) 1 only

(b) 2 only

(c) 1 and 2 only

(d) 1 2 and 3

Q62) Consider the following statements about

Giddha dance

1 Giddha is a traditional pastoral

dance performed by the women of

Bihar at festival times and at the

sowing and reaping of the harvest

2 By this dance the women reveal

their joy expel their suppressed

feelings in a male dominated

society through the performance of

Giddha

Which of the statements given above

isare correct

(a) 1 only

(b) 2 only

(c) Both 1 and 2

(d) Neither 1 nor 2

Q63) Consider the following statements

1 Mullah Shah Badakhshi was the

spiritual mentor of Dara Shukoh

2 Aurangzeb wrote the remarkable

work called ldquoMajma-ul-Bahrainrdquo or

the ldquoThe confluence of two seasrdquo

3 Dara Shukoh was seen as

inheriting the qualities of his

ancestor Akbar in that he

promoted religious pluralism and

syncretism

Which of the statements given above

isare correct

(a) 1 and 3 only

(b) 2 only

(c) 1 and 2 only

(d) 1 2 and 3

RAUSIAS-FC19E1003 24

Q64) निमननिखित कथि ो पर निचार कीनजए

1 ग मतशवर परनतमा निोधयनगरी पहाड़ी पर खसथत ह

2 शरिरबिग िा िह सथाि ह जहाो मौयण िोश क

सोसथापक चोदरगपत मौयण अपि नसोहासि क

तयागि क बाद जि तपसवी बि गए थ

उपयणकत कथि ो म स कौि-सास सही हह

(a) किि 1

(b) किि 2

(c) 1 और 2 द ि ो

(d) ि त 1 ि ही 2

Q65) निमननिखित कथि ो पर निचार कीनजए

1 पराताखतवक साकषय स पता चिता ह नक पराची

घाटी सभयता हड़पपा और म हिज दाड़ द ि ो

की पिणिती ह

2 पराची िदी भििशवर स निकिती ह

उपयणकत कथि ो म स कौि-सास सही हह

(a) किि 1

(b) किि 2

(c) 1 और 2 द ि ो

(d) ि त 1 ि ही 2

Q66) निमननिखित कथि ो म स कौि-सास सही हह

1 िजराह क समारक ो क समह का निमाणर

चोदि राजिोश क शासिकाि क दौराि हआ

था

2 य समारक हररिोदर पिणत शरोििा म खसथत ह

3 म रक क यातरी इबन बतता ि अपि सोसमरर ो

म िजराह क मोनदर ो की यातरा का उललि

नकया था तथा इन काजराण िाम स समब नरत

नकया था

िीच नदए गए कट का परय ग कर सही उततर चनिए

(a) किि 1

(b) किि 1 और 2

(c) किि 2 और 3

(d) किि 1 और 3

Q67) निमननिखित कथि ो म स कौि-सास सही हह

1 डॉ बी आर अमबडकर ि दी एनिनहिशि

ऑफ़ कासट (The Annihilation of Caste)

नििी थी नजसम उन ोि नहोद रमण म िोशािगत

पजारी की परथा क उनमिि की आिशयकता

पर बि नदया था

2 डॉ राजदर परसाद ि थॉटस ऑि पानकसताि

(Thoughts on Pakistan) िामक पसतक

नििी थी

िीच नदए गए कट का परय ग कर सही उततर चनिए

(a) किि 1

(b) किि 2

(c) 1 और 2 द ि ो

(d) ि त 1 ि ही 2

Q68) निमननिखित कथि ो म स कौि-सास सही हह

1 महरगढ़ भारतीय उपमहादवीप म एक परनसदध

ििपािार बसती ह ज नसोर पराोत पानकसताि म

खसथत ह

2 बरणह म म कतत ो क उिक सवामी क साथ कबर ो

म दफिाया जाता था

िीच नदए गए कट का परय ग कर सही उततर चनिए

(a) किि 1

(b) किि 2

(c) 1 और 2 द ि ो

(d) ि त 1 ि ही 2

Q69) निमननिखित कथि ो म स कौि-सास सही हह

1 काकानटय मोनदर अनरकतर नशि क समनपणत

2 हिमक ोडा म हजार-सतोभ िाि मोनदर (The

Thousand-Pillared Temple) का निमाणर

काकानटय समराट रदर ि करिाया था

िीच नदए गए कट का परय ग कर सही उततर चनिए

(a) किि 1

(b) किि 2

(c) 1 और 2 द ि ो

(d) ि त 1 ि ही 2

RAUSIAS-FC19E1003 25

Q64) Consider the following statements

1 Gommateshwara Statue is located

on the Vindyagiri Hill

2 Shravanabelagola is the place

where Chandragupta Maurya the

founder of the Mauryan dynasty

became a Jain ascetic after

relinquishing his throne

Which of the statements given above

isare correct

(a) 1 only

(b) 2 only

(c) Both 1 and 2

(d) Neither 1 nor 2

Q65) Consider the following statements

1 Archaeological evidence shows

that the Prachi Valley Civilisation

predates both Harappa and

Mohenjo-Daro

2 The Prachi river originates from

Bhubaneswar

Which of the statements given above

isare correct

(a) 1 only

(b) 2 only

(c) Both 1 and 2

(d) Neither 1 nor 2

Q66) Which of the following statements

isare correct

1 The Khajuraho group of

monuments was built during the

rule of the Chandela dynasty

2 These monuments are located in

Harischandra mountain range

3 Ibn Battuta the Moroccan

traveller in his memoirs mentioned

visiting Khajuraho temples and

called them Kajarra

Select the correct answer using the code

given below

(a) 1 only

(b) 1 and 2

(c) 2 and 3

(d) 1 and 3

Q67) Which of the following statements

isare correct

1 Dr BR Ambedkar wrote the

Annihilation of Caste emphasising

the need to do away with the

practice of hereditary priesthood in

Hinduism

2 The book lsquoThoughts on Pakistanrsquo

was written by Dr Rajendra

Prasad

Select the correct answer using the code

given below

(a) 1 only

(b) 2 only

(c) Both 1 and 2

(d) Neither 1 nor 2

Q68) Which of the following statements

isare correct

1 Mehrgarh is a famous Neolithic

settlement in the Indian

subcontinent which is situated in

Sindh province Pakistan

2 At Burzahom dogs were buried

with their masters in their graves

Select the correct answer using the code

given below

(a) 1 only

(b) 2 only

(c) Both 1 and 2

(d) Neither 1 nor 2

Q69) Which of the following statements

isare correct

1 The Kakatiya temples are

dedicated mostly to Siva

2 The Thousand-Pillared Temple at

Hanamkonda was built by the

Kakatiya king Rudra

Select the correct answer using the code

given below

(a) 1 only

(b) 2 only

(c) Both 1 and 2

(d) Neither 1 nor 2

RAUSIAS-FC19E1003 26

Q70) निमननिखित कथि ो म स कौि-सास सही हह

1 अहमदाबाद नमि हड़ताि क दौराि महातमा

गाोरी ि शरनमक ो क पकष क मजबत करि क

निए आमरर अिशि नकया था

2 अिशि स नमि मानिक ो पर दबाि पड़ा था ज

अोततः शरनमक ो क िति म 15 परनतशत की िखदध

करि क निए सहमत हए थ

िीच नदए गए कट का परय ग कर सही उततर चनिए

(a) किि 1

(b) किि 2

(c) 1 और 2 द ि ो

(d) ि त 1 ि ही 2

Q71) निमननिखित म स नकसक नकिक भारत स यिसक

की माििता की अमतण साोसकनतक निरासत की

परनतनिनर सची (The UNESCOrsquos List of the

Representative List of the Intangible

Cultural Heritage of Humanity) म शानमि

नकया गया ह

1 मनडयटट

2 सोकीतणि

3 को भ मिा

िीच नदए गए कट का परय ग कर सही उततर चनिए

(a) किि 1 और 2

(b) किि 2 और 3

(c) किि 3

(d) 1 2 और 3

Q72) निमननिखित जिजानतय ो म स कौि-सीसी ो

जिजानतजिजानतया िागािड स सोबोनरत हह

1 अोगामी

2 ककी

3 जारिा

िीच नदए गए कट का परय ग कर सही उततर चनिए

(a) किि 1

(b) किि 1 औऔ 2

(c) किि 2

(d) 1 2 और 3

Q73) निमननिखित कथि ो म स कौि-सास सही हह

1 राषटर कट सामराजय की सथापिा दोनतदगण ि की थी

नजसि मानयाित म अपिी राजरािी की

सथापिा की थी

2 राषटर कट समराट अम घििण एक ििक था और

उस कनिताओो पर पहिी कननड़ पसतक नििि

का शरय नदया जाता ह

िीच नदए गए कट का परय ग कर सही उततर चनिए

(a) किि 1

(b) किि 2

(c) 1 और 2 द ि ो

(d) ि त 1 ि ही 2

Q74) निमननिखित कथि ो म स कौि-सास सही हह

1 कशब चोदर सि ि ततवब नरिी सभा की

अधयकषता की थी ज आधयाखतमक सतय की

ि ज म सोिि थी

2 बरहम समाज ि मािि गररमा पर बि नदया

मनतणपजा का निर र नकया और सती परथा जसी

सामानजक बराइय ो की आि चिा की

िीच नदए गए कट का परय ग कर सही उततर चनिए

(a) किि 1

(b) किि 2

(c) 1 और 2 द ि ो

(d) ि त 1 ि ही 2

Q75) निमननिखित कथि ो म स कौि-सास सही हह

1 भारत म नचशती नसिनसिा खवाजा म इिददीि

नचशती क दवारा सथानपत नकया गया था

2 नचशती परोपरा की एक परमि निशिता

आतमसोयम थी नजसम साोसाररक म ह स दरी

बिाए रििा शानमि था

िीच नदए गए कट का परय ग कर सही उततर चनिए

(a) किि 1

(b) किि 2

(c) 1 और 2 द ि ो

(d) ि त 1 ि ही 2

RAUSIAS-FC19E1003 27

Q70) Which of the following statements

isare correct

1 During the Ahmedabad Mill Strike

Mahatma Gandhi undertook a fast

unto death to strengthen the

workersrsquo resolve

2 The fast had effect of putting

pressure on mill owners who

finally agreed to give the workers a

15 per cent increase in wages

Select the correct answer using the code

given below

(a) 1 only

(b) 2 only

(c) Both 1 and 2

(d) Neither 1 nor 2

Q71) Which of the following are included in

the UNESCOrsquos list of the representative

list of the intangible cultural heritage of

humanity from India

1 Mudiyettu

2 Sankirtana

3 Kumbh Mela

Select the correct answer using the code

given below

(a) 1 and 2 only

(b) 2 and 3 only

(c) 3 only

(d) 1 2 and 3

Q72) Which of the following tribes isare

related to Nagaland

1 Angami

2 Kuki

3 Jarawa

Select the correct answer using the code

given below

(a) 1 only

(b) 1 and 2 only

(c) 2 only

(d) 1 2 and 3

Q73) Which of the following statements

isare correct

1 Rashtrakuta kingdom was founded by Dantidurga who established his capital at Manyakhet

2 Amoghavarsha a Rashtrakuta king was an author and is credited with writing the first

Kannada book on poetics

Select the correct answer using the code given below

(a) 1 only

(b) 2 only

(c) Both 1 and 2

(d) Neither 1 nor 2

Q74) Which of the following statements isare correct

1 Keshab Chandra Sen headed the Tattvabodhini Sabha which was engaged in search of spiritual truth

2 The Brahmo Samaj laid emphasis on human dignity opposed idolatry and criticized such social

evils as the practice of Sati

Select the correct answer using the code given below

(a) 1 only

(b) 2 only

(c) Both 1 and 2

(d) Neither 1 nor 2

Q75) Which of the following statements isare correct

1 The Chishti order was established in India by Khwaja Moinuddin

Chishti

2 A major feature of the Chishti tradition was austerity including maintaining a distance from the

worldly power

Select the correct answer using the code

given below

(a) 1 only

(b) 2 only

(c) Both 1 and 2

(d) Neither 1 nor 2

T e s t i s p a r t o f R a u rsquo s I A S T e s t s e r i e s f o r P r e l i m i n a r y E x a m 2 0 1 9

FOUNDATION + CURRENT AFFAIRS

GENERAL STUDIES (PAPER ndashI)

FOUNDATION TEST ndashIII

SUBJECT NCERT History Class VI-X + Current Affairs

Time Allowed 1frac12 Hours Maximum Marks 150

I NSTRUCT IONS

1 IMMEDIATELY AFTER THE COMMENCEMENT OF THE EXAMINATION YOU SHOULD CHECK

THAT THIS TEST BOOKLET DOES NOT HAVE ANY UNPRINTED OR TORN or MISSING PAGES OR

ITEMS ETC IF SO GET IT REPLACED BY A COMPLETE TEST BOOKLET

2 This Test Booklet contains 75 items (questions) Each item is printed both in Hindi and English

Each item comprises four responses (answers) You will select the response which you want to mark

on the Answer Sheet In case you feel that there is more than one correct response mark the

response which you consider the best In any case choose ONLY ONE response for each item

3 You have to mark all your responses ONLY on the separate Answer Sheet (OMR sheet) provided

Read the directions in the Answer Sheet

4 All items carry equal marks

5 Before you proceed to mark in the Answer Sheet the response to various items in the Test booklet

you have to fill in some particulars in the Answer Sheet as per instructions contained therein

6 After you have completed filling in all your responses on the Answer Sheet and the examination has

concluded you should hand over to the Invigilator only the Answer Sheet You are permitted to

take away with you the Test Booklet

7 Penalty for wrong answers

THERE WILL BE PENALTY FOR WRONG ANSWERS MARKED BY A CANDIDATE IN THE

OBJECTIVE TYPE QUESTION PAPERS

(i) There are four alternatives for the answer to every question For each question for which a

wrong answer has been given by the candidate one-third of the marks assigned to that

question will be deducted as penalty

(ii) If a candidate gives more than one answer it will be treated as a wrong answer even if one of

the given answers happens to be correct and there will be same penalty as above to that

question

(iii) If a question is left blank ie no answer is given by the candidate there will be no penalty for

that question

T h i s t e s t i s p a r t o f R a u rsquo s I A S T e s t s e r i e s f o r P r e l i m i n a r y E x a m 2 0 1 9

Test Code

FC19E1003

FC19H1003 29

Answers and Explanations of

NCERT History Class VI-X + Current Affairs (FC19E1003)

Q1) उततर (c)

सपषटीकरण

- ऋगवद म दविय ो और दिताओो क समवपित एक

हजार स अविक सत तर (शल क) ह

- य शल क ऋविय ो क दवारा रच गए थ और परि ो

दवारा सीख जात थ

- हालाोवक कछ शल क मवहलाओो (जस वक अपाला

घ सा ल पामदरा मतरयी और गागी) क दवारा भी रच

गए थ

- ऋगवद म सोिाद क रप म कई शल क मौजद ह

- हम विशवावमतर नामक एक ऋवि और दविय ो क

रप म पजी जान िाली द नवदय ो (वयास और

सतलज) क बीच िाताि का उदाहरण वमलता ह

- इसस पता चलता ह वक विशवावमतर िवदक काल स

सोबोवित थ

Q2) उततर (b)

सपषटीकरण

- करनल गफाओो स राख क अिशि परापत हए ह

ज इस ओर सोकत करत ह वक ततकालीन ल ग

अवि क उपय ग स पररवचत थ

- य गफाएो आोधर परदश म सथथत ह

Q3) उततर (c)

सपषटीकरण

bull बरािह म ितिमान कशमीर म सथथत एक

परागवतहावसक थथल ह जहाो ल ग गडढ क घर ो का

वनमािण करत थ

bull य घर जमीन क ख द कर बनाए जात थ तथा नीच

जान क वलए सीवियाा ह ती थी

bull ऐसा अनमान लगाया जाता ह वक य घर ठो ड क

मौसम म आशरय परदान करत थ

Q4) उततर (c)

सपषटीकरण

bull परालख-विदया (Epigraphy) क वशलालख ो क

अधययन क रप म पररभावित वकया जाता ह

bull हसतवलसखत दसतािज ो क माधयम स इवतहास

और सावहतय क अधययन क पाोडवलवप विजञान

(Manuscriptology) कहत ह

bull पराचीन लखन परणावलय ो क अधययन और

ऐवतहावसक पाोडवलवपय ो क समझन तथा वतवथ

वनिािरण क पलीओगराफी (Palaeography) कहा

जाता ह

bull नयवमजमविकस (Numismatics) वसक ो क

अधययन क सोदवभित करता ह

Q5) उततर (a)

सपषटीकरण

- चरक सोवहता चरक क दवारा वलखी गई आयिद

और िदयक-शासर पर एक महतवपणि पसतक ह

- ि भारतीय िदयक-शासर की पारमपररक परणाली

वजस आयिद क नाम स जाना जाता ह क

अभयासकताि थ

- ऐसा माना जाता ह वक चरक का विकास दसरी

शताबदी (ईसा पिि) और दसरी शताबदी (ईसवी) क

मधय हआ था

Q6) उततर (b)

सपषटीकरण

- भाग फसल ो पर वलए जान िाल कर क सोदवभित

करता ह ज कल फसल उतपादन का 16 िाो भाग

था

- ldquoकममकारrdquo शबद भवमहीन कवि शरवमक िगि क

वलए परय ग वकया जाता था

- ldquoअशवमिrdquo (वजस घ ड क बवलदान क रप म भी

जाना जाता ह) एक अनषठान ह ता था वजसम एक

घ ड क सवतोतर रप स घमन क वलए छ ड वदया

FC19H1003 30

जाता ह और राजा क सवनक उसकी रखिाली

करत थ

Q7) उततर (d)

सपषटीकरण

- ऋगववदक काल म घ ड ो क रथ ो म ज ता जाता था

ज (रथ) भवम मिवशय ो आवद पर कबजा करन क

वलए लड गए यद ो म उपय ग वकए जात थ

- इसस यह पता चलता ह वक घ ड ो यकत रथ ो का

उपय ग महाजनपद काल स काफी पहल आरमभ

हआ था

- ऋगववदक काल म मिवशय ो भवम जल आवद पर

कबजा करन क वलए तथा ल ग ो क पकडन क

वलए यद वकय जात थ

- अविकाोश परि इन यद ो म भाग वलया करत थ

- हालाोवक उस समय क ई वनयवमत सना नही ो ह ती

थी लवकन उस काल म सभाऐो ह ती थी ो वजनम

ल ग यद क मामल ो पर चचाि करत थ

- वनयवमत सनाएा महाजनपद काल का िवशषटय थी

वजनम पदल सवनक ो की विशाल सनाएा रथ तथा

हाथी शावमल ह त थ

Q8) उततर (a)

सपषटीकरण

- बद शाकय कल स सोबोवित थ और कशीनारा म

उनका वनिन हआ था

- बद न अपनी वशकषाएा पराकत भािा म दी थी ो ज

आम ल ग ो की भािा थी

Q9) उततर (c)

सपषटीकरण

- पराचीन भारत म दशिनशासर की छह शाखाएा थी ो

िशविक नयाय समखया य ग पिि वममाोसा और

िदाोत या उततर वममाोसा

- इनकी थथापना करमश कनाद गौतम कवपल

पतोजवल जावमनी और वयास ऋविय ो न की थी

Q10) उततर (b)

सपषटीकरण

महािीर की वशकषाऐो छठी शताबदी म िललभी म

सोकवलत की गई थी ो

Q11) उततर (c)

सपषटीकरण

- पारमपररक रप स चाणकय क कौविलय अथिा

विषणगपत क नाम स जाना जाता ह

- उसन अथिशासतर ज एक पराचीन भारतीय

राजनवतक आलख ह वलखा था

Q12) उततर (d)

सपषटीकरण

- भारत का राषटर ीय वचनह सारनाथ (उततर परदश) क

अश क सतमभ क ऊपर (शीिि पर) वसोह कवपिल

का एक अनरपण ह

- इस राषटर ीय वसदाोत सतयमि जयत क साथ

सोय वजत वकया गया ह

- रामपिि बल का नाम रामपिि (वबहार) क नाम पर

पडा जहाा इसकी ख ज हई थी

- यह अपन नाजक नकाशी मॉडल क वलए परवसदद

ह वजसम क मल तवचा सोिदनशील नथन ो सतकि

कान और मरबत िााग ो क शरषठतर परवतरप क

परदवशित वकया गया ह

- यह भारतीय और फारसी ततव ो का एक ससममशरण

- सोवकससा उततर परदश म सथथत ह

Q13) उततर (a)

सपषटीकरण

का िर वसोह ज एक महान य दा थ वबहार स

सोबोवित थ

Q14) उततर (b)

सपषटीकरण

िललालर शबद बड भ-सवावमय ो क वलए परय ग

वकया जाता था

FC19H1003 31

Q15) उततर (c)

सपषटीकरण

- अररकमड एक तिीय बसती थी जहाो दर दश ो स

आन िाल जहाज ो का माल उतारा जाता था

- यहाो पर ईोि ो का एक विशाल ग दाम वमटटी क

बतिन (वजनम एमफ रा - द हरी मवठय ो का लोबा

घडा - शावमल ह) और एरिाइन (Arretine)

मदभाोड पाए गए थ

- इस थथान पर र मन दीपक काोच क बन पातर और

रतन भी पाए गए थ

Q16) उततर (a)

सपषटीकरण

- मिनदर सोगम कविताओो म उसललसखत एक

तवमल शबद ह वजसका अथि ह ldquoतीन परमखrdquo

- यह तीन सततारि पररिार ो क मसखयाओो क वलए

परय ग वकया जाता ह च ल चर और पाणडय

Q17) उततर (c)

सपषटीकरण

- ऋग िद म सभा विदाथा तथा गण जसी

जनजावतय ो पर अथिा किोब पर आिाररत

सभाओो का उललख ह

- आरसमभक िवदक काल म सभाओो और सवमवतय ो

का विशि महतव ह ता था

- यहाा तक की मसखया अथिा राजा भी उनका

समथिन परापत करन क वलए आतर रहत थ

Q18) उततर (a)

सपषटीकरण

- जन िमि न ईशवर क अससततव क मानयता त दी ह

वकनत उसन ईशवर क वजना क पद स नीच रखा

- जन िमि न बौद िमि की तरह िणि परणाली की

भरतिना नही ो की थी

Q19) उततर (d)

सपषटीकरण

- च ल ो और पाणडय ो न शसकतशाली तिीय शहर ो का

विकास वकया था

- च ल ो का सबस महतवपणि शहर पहार (या

कािरीपटटीनम) था |

- मदरई पाणडय ो की राजिानी थी

Q20) उततर (b)

सपषटीकरण

- ldquoबदचररतrdquo बद का जीिन-ितताोत ह

- इस अशवघ ि क दवारा वलखा गया था

Q21) उततर (a)

सपषटीकरणः

- तवमल कवि अपपर भगिान वशि क भकत थ

- इस परकार ि एक नयनार सोत थ

Q22) उततर (d)

सपषटीकरणः

- समदरगपत एक परवसद गपत शासक था

- उसन वसक ो पर िीणा बजात हए अपनी छवि

अोवकत करिाई थी

- यह सोगीत क परवत उसक परम क दशािता ह

- हम उसकी इलाहाबाद परशससत स महतवपणि

ऐवतहावसक जानकारी वमलती ह वजसकी रचना

उसक दरबार क कवि हररसन न की थी

Q23) उततर (b)

सपषटीकरणः

- विकरम सोित की शरआत ििि 58 ईसा पिि म

चनदरगपत वदवतीय न की थी

- यह शक ो पर उसकी जीत और उस विकरमावदतय

की पदिी वमलन क उपलकषय म आरमभ वकया गया

था

FC19H1003 32

- बानभटट न हिििििन का जीिन-ितताोत हििचररत

(ज सोसकत म थी) वलखी थी

Q24) उततर (c)

सपषटीकरणः

- सोवि-विगरावहका यद एिो शाोवत का मोतरी

- साथििाह वयापाररय ो क कावफल ो का नता

Q25) उततर (a)

सपषटीकरणः

- जआन झाोग (हसआन रताोग ndash Hsuang Tsang)

एक चीनी यातरी था ज हिििििन क शासनकाल म

भारत आया था

- ििि 630 ईसवी स ज दशक आरमभ हआ था उसम

जआन झाोग मधय एवशया ईरान और

अफग़ावनसतान की यातरा करन क पशचात कशमीर

क रासत स भारत आया था

- उसन उततर स पिि तक की यातरा की और िह

लगभग 2 ििि वबहार म रहा

- जआन झाोग न नालनदा विशवविदयालय म विदयावथिय ो

और विदवान ो क साथ पारसपररक विचार-विमशि

वकया थथानीय भािाओ ा म वनपणता परापत की तथा

बौद सतप ो की ख ज की

Q26) उततर (c)

सपषटीकरणः

- परदवकषणा पथ बौद िासतकला म सतप क चार ो

ओर बनाया जान िाला एक घमािदार पथ ह ता

- परशन म वदए गए बाकी क तीन ो ततव वहोद मसनदर ो की

िासतकला क भाग ह

Q27) उततर (d)

सपषटीकरणः

परशन म वदए गए सभी मोवदर ो म वयापक रप स

ईोि ो (पकी ईोि ो) का परय ग पतथर ो क साथ हआ

Q28) उततर (c)

सपषटीकरण

- महममद कली कतब शाह ग लकणडा का सलतान

था

- िह अकबर का समकालीन था

- सावहतय और िासतकला म उसकी अतयाविक

रवच थी

- िह एक महान कवि था

- िह दसखनी उदि फारसी और तलग म वलखता था

- उसन अपन पीछ एक विसतत वदिान (सोगरह)

छ डा ह

- अभी हाल ही म तलोगाना म ग लकणडा क वकल

क अनदर खदाई वकय गए बाग-ए-नाया वकला

बाग क चार ो ओर रप-रखा क मानवचतरण क

वलए भारतीय परातासतवक सिकषण (The

Archaeological Survey of India ndash ASI)

गराउणड पनीिर विोग रडार (Ground Penetrating

Radar) का परय ग करगा

Q29) उततर (a)

सपषटीकरणः

- वसलपपावदकारम एक तवमल महाकावय ह वजसकी

रचना इलाोग क दवारा लगभग 1800 ििि पिि की

गई थी

- यह क िलन नामक एक वयापारी की कहानी ह

ज माििी नामक एक गवणका (िशया) स परम

करन लगा था

- मवनमकलाई क िलन और माििी की पतरी की

कहानी ह

Q30) उततर (a)

सपषटीकरण

- चरक आयिद और वचवकरता की एक महतवपणि

रचना चरक सोवहता क लखक ह

- बरहमगपत क अपनी रचना बरहम-सफि-वसदानत

(ज एक खग लीय रचना ह) क कारण परवससद

वमली

FC19H1003 33

- बगदाद म इसका अनिाद अरबी भािा म वकया

गया था

- इसका इसलावमक गवणत और खग ल-विजञान पर

महतवपणि परभाि पडा था

- बाद म अपन जीिनकाल म बरहमगपत न

ldquoखोडखयाकrdquo वलखी ज एक खग लीय पससतका

(एक छ िी पसतक) थी

- इसम आयिभटट की अिि-रावतर क परतयक वदन की

शरआत परणाली का परय ग वकया गया था

Q31) उततर (c)

सपषटीकरण

- अमीर खसर एक परवसद सफी सोगीतकार कवि

और विदवान थ

- 1318 म उनह ोन पाया वक इस भवम (वहोदसतान) क

हर कषतर म अलग-अलग भािा थी लाहौरी

कशमीरी दवारसमदरी (दवकषणी कनाििक म)

तलोगाना (आोधर परदश म) गजरी (गजरात म)

माबारी (तवमलनाड म ) अििी (पिी उततर परदश

म) और वहोदिी (वदलली क आस-पास क कषतर म)

आवद

- उनह न यह बताया वक सोसकत वकसी भी कषतर स

सोबोवित नही ो थी और किल बराहमण ही इस भािा

का जञान रखत थ

Q32) उततर (c)

सपषटीकरण

- वहरणय-गभि सववणिम गभि क सोदवभित करता ह

- जब बराहमण ो की सहायता स यह अनषठान वकया

जाता था त यह माना जाता था वक बवल दन िाल

का कषवतरय क रप म पनजिनम ह गा

Q33) उततर (d)

सपषटीकरण

- कदमई भवम राजसव पर कर क सोदवभित करता

- गवावलयर परशससत म नागभि क दवारा वकय गए

श िण का िणिन वकया गया ह |

- नागभि एक परवतहार राजा था

Q34) उततर (b)

सपषटीकरण

- राजतरो वगनी 12िी ो शताबदी म कलहन क दवारा

रवचत एक सोसकत पसतक (िकसट) ह

- यह परारसमभक भारत की ऐवतहावसक इवतितत थी

- तकि सोगत रप स इस अपन परकार की सिोततम

और सिािविक विशवसनीय कवत माना जाता ह

- यह कशमीर कषतर क पराचीनतम समय स लकर

उसकी रचना की तारीख तक क समपणि इवतहास

का आचछादन करती ह

Q35) उततर (c)

सपषटीकरण

- गााि की आम सभा क ldquoउरrdquo कहा जाता था

- ldquoउरrdquo म गााि क सभी कर दन िाल वनिासी

शावमल ह त थ

Q36) उततर (a)

सपषटीकरण

- वदलली सलतनत म ldquoतारीखrdquo इवतहास लखन का

एक रप था

- ldquoतािरीखrdquo क लखक विदवान परि ह त थ वजनम

सवचि परशासक इतयावद शावमल थ

Q37) उततर (a)

सपषटीकरण

- अलाउददीन सखलजी अपन सवनक ो क ितन का

भगतान नकद म करता था न वक इकता क रप

- सवनक अपना सामान वदलली म वयापाररय ो स

खरीदत थ अतः इस बात का भय था वक वयापारी

कही ो िसतओो का मलय न बिा द

- इसकी र कथाम क वलए अलाउददीन सखलजी न

वदलली म कीमत ो क वनयसित वकया

FC19H1003 34

- अविकारीगण धयानपििक मलय ो का सिकषण करत

थ तथा ज वयापारी वनिािररत मलय पर माल नही ो

बचत थ उनक दसणडत वकया जाता था

Q38) उततर (d)

सपषटीकरण

- वदलली सििपरथम त मर राजपत ो क अिीन उनक

सामराजय की राजिानी बनी थी

- 12िी ो शताबदी क मधय म अजमर क चौहान ो

(वजनह चाहमान ो क नाम स भी जाना जाता ह) न

त मर राजपत ो क परावजत वकया था

- त मर ो और चौहान ो क अिीन वदलली एक

महतवपणि िावणसजयक क दर बन गया था

- कई जन वयापारी यहाा रहन लग थ और उनह ोन

कई मोवदर भी बनिाए

- यहाा पर मवदरत वसक वजनह ldquoदहलीिालrdquo क नाम

स जाना जाता था वयापक रप स परचलन म थ

Q39) उततर (c)

सपषटीकरण

- म ठ की मसिद का वनमािण वसको दर ल दी क

राजयकाल म उसक मिी क दवारा करिाया गया

था

- बगमपरी मसिद का वनमािण महममद तगलक क

शासनकाल म हआ था

- यह मसिद विशव का पणयथथान (The

Sanctuary of the World) और वदलली म महममद

तगलक की नई राजिानी जहाोपनाह की मखय

मसिद थी

- कववत- अल - इसलाम मसिद का विसतार

इलतसिश और अलाउददीन सखलजी न वकया था

- मीनार का वनमािण तीन सलतान ो कतबददीन ऐबक

इलतसिश और वफर ज शाह तगलक क दवारा

करिाया गया था

Q40) उततर (c)

सपषटीकरण

- मगल ो क अिीन मनसबदार शबद उस वयसकत क

वलए सोदवभित वकया जाता था वजसक पास मनसब

(अथाित पद) ह ता था

- उस अपना ितन राजसव कायो वजनह जागीर कहत

थ क रप म परापत ह ता था

Q41) उततर (b)

सपषटीकरण

- ldquoभारत छ ड आोद लनrdquo वबरविश शासन क

सखलाफ ल ग ो का एक सवाभाविक विदर ह था

- असखल भारतीय काोगरस सवमवत न 8 अगसत 1942

क बमबई म एक बठक का आय जन वकया था

- इस बठक म परवसद सोकलप ldquoभारत छ ड rdquo क

पाररत वकया गया और इस उददशय क परापत करन

क वलए गाोिी क नततव म एक अवहोसक जन सोघिि

आोद लन की शरआत का परसताि वदया गया

- लवकन अगल ही वदन गाोिी और काोगरस क अनय

परमख नताओो क वगरफतार कर वलया गया

- काोगरस क एक बार वफर अिि घ वित वकया गया

था

Q42) उततर (c)

सपषटीकरण

- साइमन कमीशन यनाइविड वको गडम क सात

साोसद ो का एक समह था

- इस वबरविश भारत क वलए सोििावनक सिार ो का

सझाि दन क वलए गवठत वकया गया था

- इस आय ग म िररषठ वबरविश राजनता सर जॉन

साइमन क नततव म किल वबरविश सदसय ही

शावमल थ

- इसवलए भारत क ल ग ो न साइमन कमीशन क

आगमन क विरद आोद लन वकया था

Q43) उततर (a)

सपषटीकरण

bull दादा भाई नौर जी भारत म वबरविश शासन क

आवथिक पररणाम ो क बार म अपनी विर िी

(परवतकल) राय क वलए जान जात थ

FC19H1003 35

bull अपन कई लख ो और भािण ो म विशि रप स

ldquoपाििी एो ड अन-वबरविश रल इन इसणडया

(Poverty and Un-British Rule in India) म

नौर जी न यह तकि वदया वक भारत पर अतयविक

कर लगाया गया था और इसकी सोपवतत इोगलड की

ओर परिावहत की जा रही थी

bull उनह ोन पराचीन भारतीय गरोथ ो की वयाखया करन

का और भारतीय ो क आिविशवास क बहाल

करन पर कायि नही ो वकया था

उनह ोन वकसी और बात स पहल सभी सामावजक

बराइय ो क उनमलन की आिशयकता पर भी बल

नही ो वदया था

Q44) उततर (c)

सपषटीकरण

bull अगसत 1932 म वबरविश परिानमोतरी मकड नालड न

अपन साोपरदावयक परसकार (The Communal

Award) की घ िणा की थी

bull यह भारत क कई साोपरदावयक वहत ो क बीच विवभनन

सोघिो क हल करन क वलए वबरिन का एकतरफा

परयास था

bull यह परसकार (Award) बाद म 1935 क

अविवनयम (The Act of 1935) म शावमल वकया

गया था

bull इस साोपरदावयक परसकार न मससलम ो क वलए

आरवकषत एक अलग वनिािचक मणडल फॉमिल का

विसतार अनय अलपसोखयक ो क वलए वकया था

वजसम वसख ो भारतीय ईसाइय ो आोगल-भारतीय

समदाय यर पीय समदाय तथा विवशषट कषतरीय

समह ो क शावमल वकया गया था

bull गाोिी न इस परसताि क भारतीय समाज क

विभावजत करन क वलए एक घवणत वबरविश

सावजश क रप म दखा और उसक सखलाफ

आमरण अनशन वकया

Q45) उततर (b)

सपषटीकरण

मौजदा आयात और वनयाित क अवतररक़त

औपवनिवशक भारत क वनमनवलसखत खचो क

वलए एक विशिवनवशचत िन रावश भी दनी पडती

थी

(i) परशासन क वयय

(ii) सना क रख-रखाि क वयय

(iii) यद क वयय

(iv) सिावनितत अविकाररय ो की पशन तथा

(v) वबरिन दवारा अपनी उपवनिश बसती

(कॉल नी) क रख-रखाि क वयय

इनह गह शलक (Home Charges) क रप म

जाना जाता था और लगभग परी तरह स भारत क

दवारा इनका भगतान वकया जाता था

bull गह शलक म वनमनवलसखत घिक शावमल थ

(i) भारतीय ऋण पर दय बयाज

(ii) ईसट इोवडया को पनी क शयरिारक ो क

लाभाोश

(iii) लोदन म भारत कायािलय चलान क वलए िन

(iv) भारत म वनयकत वबरविश कवमिय ो क ितन

और पशन का भगतान करन क वलए िन

(v) रलि पर बयाज

(vi) नागररक और सनय शलक

(vii) इोगलड म सट र (सामगरी) की खरीद

Q46) उततर (b)

सपषटीकरण

bull भारतीय राषटर ीय काोगरस का लाहौर सतर 1929 म

जिाहरलाल नहर की अधयकषता म आय वजत

वकया गया था

bull इस सतर म भारतीय राषटर ीय आोद लन स समबसित

कई महतवपणि पररणाम सामन आय थ

(i) सििपरथम इस सतर म काोगरस क अधयकष पद

पर जिाहरलाल नहर क चना गया था ज

काोगरस म िामपोवथय ो की बिती हई ताकत

का सपषट सोकत था

(ii) दसरा इस सतर म पहली बार काोगरस न पणि

सवतोतरता की माोग क उठाया था

इस परकार की माोग काोगरस मोच स पहल कभी भी

नही ो उठाई गई थी

Q47) उततर (b)

सपषटीकरण

FC19H1003 36

bull इस ररप िि न वकसी भी समदाय क वलए पथक

वनिािचक मोडल अथिा अलपसोखयक ो क वलए

भाराोश की वसफाररश नही ो की थी

bull तथावप इस ररप िि न उन पराोत ो म अलपसोखयक

सीि ो क आरकषण की अनमवत दी थी जहाा पर कम

स कम दस परवतशत अलपसोखयक ह

bull लवकन यह समदाय क आकार क अनपात म ह ना

चावहए था

bull इस ररप िि म भारत क वलए पणि सवतोतरता क

वलए क ई पराििान नही ो था

Q48) उततर (c)

सपषटीकरण

bull आरो वभक िवदक आयो का िमि मखय रप स

परकवत की पजा और यजञ था

bull परारो वभक आयि िमि परकवत की पजा क समान था

bull िासति म उनक चार ो ओर की शसकतयाा वजनह न

त ि वनयोवतरत कर सकत थ और न ही समझ पाए

थ उनह वदवयता क साथ वनिवशत वकया गया तथा

उनह मादा या नर दिीदिताओो क रप म

परतीकतव वकया गया था

bull उनह ोन कछ यजञ ो का भी वनषपादन वकया था

Q49) उततर (b)

सपषटीकरण

bull सडक और नदी-मागि (जल-मागि) डकती स

सरवकषत नही ो थ

bull उललखनीय ह वक हिििििन क शासनकाल क

दौरान यआन चिाोग (हयएन साोग) का सारा

सामान लि वलया गया था

Q50) उततर (c)

सपषटीकरण

परशन म वदए गए द न ो कथन सही ह

Q51) उततर (b)

सपषटीकरण

bull परोदर दास एक सोत और भगिान कषण क एक

महान भकत थ

bull परोदर दास क कनाििक सोगीत क वपतामह क

रप म जाना जाता ह

bull यदयवप उनक जनम-थथान क बार म काफी

अिकल लगाई जाती रही ह

bull तथावप अब कननड विशवविदयालय हमपी क दवारा

गवठत एक विशिजञ सवमवत इस वनषकिि पर पहोची

ह वक उनका जनम थथान सोभितया कनाििक का

एक छ िा-सा गााि कषमपरा (वशिम गगा वजला)

था

Q52) उततर (c)

सपषटीकरण

bull शरी तयागराज शरी शयाम शासतरी और शरी मथसवामी

दीवकषतर क कनाििक सोगीत की वतरमवति माना

जाता ह

bull उनक कारण ही 18िी ो-19िी ो शताबदी म कनाििक

सोगीत का सववणिम यग आया था

Q53) उततर (d)

सपषटीकरण

bull अभी हाल ही म लौह यगीन-महापािावणक काल

का 2000 ििि पराना एक दलिभ सारक फगस

(Sarcophagus) (पतथर का ताबत) क ललम क

वियर गाोि (क वयलडी क पास वजला क वझक ड

करल राजय) की एक रॉक-कि गफा स ख जा गया

bull यह ताबत वजसम हविय ो क िकड थ खदाई क

दौरान वमला

bull अभी तक इस परकार की दलिभ ख ज करल क

मातर द ही थथान ो स हई ह

bull य द न ो सारक फगी (Sarcophagi) (पतथर क

ताबत) चियर और अथ ली (वजला क वझक ड) क

महापािाण थथल ो स वमल ह

Q54) उततर (a)

सपषटीकरण

FC19H1003 37

दवकषण भारत म महापािाण सोसकवत एक पणि

विकवसत लौह यगीन सोसकवत थी

Q55) उततर (d)

सपषटीकरण

bull च ल पाणडय और करलपतर (चर) इन तीन ो का

उललख अश क क अवभलख ो म वकया गया ह

bull सोभितः य भौवतक सोसकवत क उततर

महापािावणक चरण म थ

Q56) उततर (d)

सपषटीकरण

bull भीमा-क रगाोि की लडाई ततीय आोगल-मराठा

यद का वहससा थी

Q57) उततर (b)

सपषटीकरण

bull राजकमार शकल न गाोिीजी क चोपारण आन तथा

वतनकवथया परणाली स जडी समसया की जाोच क

वलए रारी करन क वलए दश भर म उनका

अनसरण वकया था

bull बज वकश र राजदर परसाद महादि दसाई और

नरहरी पाररख चोपारण सतयागरह क दौरान गाोिी

जी क सहय गी थ

Q58) उततर (b)

सपषटीकरण

bull बराहमण ो और बौद मठिाररय ो क कर-मकत गााि

अनदान म दन की परथा सतिाहन ो न आरमभ की

थी

Q59) उततर (c)

सपषटीकरण

इस कायिकरम क उददशय वनमनानसार ह

(i) बवनयादी पयििन आिाररक सोरचना का विकास

करना

(ii) चयवनत (पहचान वकय गए) कषतर ो म आजीविका क

सजन क वलए दश क साोसकवतक और विरासत

मलय ो क बिािा दना

(iii) विरासत समारक थथल ो पर विशव सतरीय आिाररक

सोरचना विकवसत करक एक सतत तरीक स

पयििक आकििण म िसद करना

(iv) थथानीय समदाय ो की सवकरय भागीदारी क माधयम

स र रगार ो का सजन करना

(v) र रगार उतपादन और आवथिक विकास क वलए

पयििन कषमता का उन पर परभाि का उपय ग

करना तथा

(vi) िारणीय पयििन आिाररक सोरचना का विकास

करना और उसका उवचत सोचालन तथा

रखरखाि सवनवशचत करना

Q60) उततर (b)

सपषटीकरण

bull यह वनकाय ििि 1987 म अससततव म आया था

bull यह एक राषटर ीय सतर का शीिि सोगठन ह ज भारत

सरकार क जनजातीय मामल ो क मोतरालय क

परशासवनक वनयोतरण क अिीन काम कर रहा ह

bull इसका पोजीकत और परिान कायािलय नई वदलली

म सथथत ह

Q61) उततर (c)

सपषटीकरण

bull परमचोद क उपनयास ो म परमाशरम रोगभवम गबन

कमिभवम और ग दान शावमल ह

bull ग रा रिी ोदरनाथ िग र क दवारा रवचत उपनयास ह

bull अभी हाल ही म मोशी परमचोद की 138िी ो जयोती दश

भर म मनाई गई थी

Q62) उततर (b)

सपषटीकरण

bull ldquoवगदाrdquo पोजाब (भारत) एिो पावकसतान की

मवहलाओो क दवारा तयौहार क समय और फसल

की बिाई तथा किाई क अिसर पर वकया जान

िाला एक पारोपररक दहाती नतय ह

FC19H1003 38

bull इस नतय क माधयम स पोजाबी मवहलाऐो अपनी

परसननता परकि करती ह तथा वगदा क परदशिन क

माधयम स परि िचिसव िाल समाज म मवहलाओो

की दबी हई भािनाओो क परकि करती ह

bull चोवक इस नतय का परि ो क साथ क ई सोबोि नही ो

ह अतः किल मवहलाऐो ही इसम भाग ल सकती

bull हर साल तीज समार ह क दौरान पोजाब म वगदा

नतय वकया जाता ह

तीज भारत क कछ भाग ो म मवहलाओो क दवारा

मनाया जान िाल कई तयौहार ो क वलए एक

वयापक नाम ह

Q63) उततर (a)

सपषटीकरण

- मजम-उल-बहरीन या द समदर ो का सोगम

नामक उललखनीय रचना दारा वशक ह क दवारा

वलखी थी

- भारत क उपराषटर पवत शरी एम िकया नायड न कहा

ह वक राजकमार दारा वशक ह की रचनाएा शाोवत

और सदभाि क बिािा दन क वलए एक तारा सर त

क रप म सामन आ सकती ो ह

- उपराषटर पवत गत ििो क भला वदए गए राजकमार

दारा वशक ह क परदवशित परचवलत करन हत

आय वजत एक परदशिनी का दौरा करन क बाद एक

सभा क सोब वित कर रह थ

- इस परदशिनी का आय जन फर क इस गौवियर

(Francois Gautier) क दवारा lsquoइोवदरा गाोिी नशनल

सिर फॉर द आििसrsquo (The Indira Gandhi

National Centre for the Arts) नई वदलली म

वकया गया था

Q64) उततर (c)

सपषटीकरण

- ग मतशवर परवतमा जन भगिान बाहबली क

समवपित ह

- यह एक एक-चटटानी पतथर की मवति ह

- राषटर पवत राम नाथ क विोद न शरिणबलग ला

(कनाििक) म आय वजत वकय जान िाल भवय

अवभिक समार ह महामसतकावभिक का

उदघािन वकया था

- यह समार ह 12 ििो म एक बार ह ता ह

Q65) उततर (c)

सपषटीकरण

bull पराची घािी पराची नदी क चार ो ओर फली हई थी

bull पराची घािी िीर-िीर विलपत ह गई थी

bull पराची नदी भिनशवर स वनकलती ह

bull यह महानदी की एक सहायक नदी ह और यह

परी खदाि किक तथा जगतवसोहपर वजल ो स

ह कर बहती ह

bull इस नदी क पर कषतर क पराची घािी कहा जाता ह

bull यह नदी बोगाल की खाडी म वगरती ह

परातासतवक साकषय स पता चलता ह वक पराची घािी

सभयता हडपपा और म हनज दाड द न ो की

पिििती ह

Q66) उततर (d)

सपषटीकरण

य समारक छतरपर वजल (मधय परदश) म विोधयाचल

पिित शरोखला म सथथत ह

Q67) उततर (a)

सपषटीकरण

bull थॉिस ऑन पावकसतान नामक पसतक डॉ बी

आर अमबडकर न वलखी थी

bull डॉ बी आर अमबडकर की जयोती क अिसर पर

भारत क राषटर पवत न भारत की इस महान हसती

क शरदाोजवल अवपित की थी

bull डॉ बी आर अमबडकर न 1924 म वडपरथड

कलावसर इोसटीटयि (दवलत िगि सोथथान -

बवहषकत वहतकाररणी सभा) और 1927 म समाज

समता सोघ की थथापना की थी

bull अमबडकर का धयान वशकषा कषतर की ओर भी था

bull उनह ोन वशकषा क वनमन िगो म फलान क वलए

पीपलस एजकशन स साइिी (The Peoples

Education Society) क नाम स महाविदयालय ो क

नििकि और छातरािास ो की थथापना की थी

FC19H1003 39

Q68) उततर (b)

सपषटीकरण

bull महरगि भारतीय उपमहादवीप म एक परवसद

निपािाण बसती ह ज बलवचसतान पराोत

पावकसतान म सथथत ह

bull दचपलली (आोधर परदश) क पास नागलर नदी क

पिी ति ो पर चना पतथर क बलॉक क विशाल

विसतार म एक पिि-ऐवतहावसक रॉक आिि थथल की

ख ज की गई ह

bull इसन 1500-2000 ईसा पिि क दौरान गोिर (आोधर

परदश) म विकवसत निपािाण सभयता पर परकाश

डाला ह

Q69) उततर (c)

सपषटीकरण

bull 12िी ो सदी और 13िी ो सदी म काकाविय िोश का

उदय हआ था

bull ि पहल कलयाण क पवशचमी चालकय ो क सामोत थ

bull परारोभ म उनह ोन िारोगल (तलोगाना) क पास एक

छ ि स कषतर पर शासन वकया था

bull उनह ोन ldquoनायक वयिथथाrdquo की शरआत की थी

वजस बाद म विजयनगर क राय शासक ो न

अपनाया और विकवसत वकया था

Q70) उततर (a)

सपषटीकरण

bull गाोिीजी क अनशन स वमल मावलक ो पर दबाि

पडा था ज अोततः शरवमक ो क ितन म 35 परवतशत

की िसद करन क वलए सहमत हए थ

bull गगल (Google) न अनसया साराभाई वजनह ोन

भारत क शरवमक आोद लन म एक अगरणी भवमका

वनभाई थी की 132िी ो जयोती डडल (Doodle) का

वनमािण करक मनाई

Q71) उततर (d)

सपषटीकरण

भारत स यनसक की मानिता की अमति साोसकवतक

विरासत की परवतवनवि सची म वनमनवलसखत शावमल ह

bull कवडयटटम करल का सोसकत रोगमोच

bull मवडयिि करल का अनषठान रोगमोच और नतय

नाविका

bull िवदक मि जाप की परोपरा

bull राजथथान क कालबवलया ल क गीत और नतय

bull रामलीला रामायण का पारोपररक परदशिन

bull सोकीतिन मवणपर का अनषठान गायन ढ ल िादन

और नतय

bull रममन भारत क गििाल वहमालय का िावमिक

तयौहार और अनषठान रोगमोच

bull जाोदीयाला गर पोजाब क ठठर ो की पीतल और

ताोब क वशलप स वनवमित बतिन ो की पारोपररक कला

bull छाऊ नतय पिी भारतीय राजय ो म जनमी शासतरीय

भारतीय नतय कला

bull लददाख का बौद मि जाप िर ाोस-वहमालयी लददाख

कषतर तथा जमम-कशमीर म पवितर बौद गरोथ ो का पाठ

bull य ग

bull नौर र

bull को भ मला

Q72) उततर (b)

सपषटीकरण

bull भारत क राषटर पवत शरी राम नाथ क विोद न

वकसामा नागालड म हॉनिवबल मह रति और

राजय गठन वदिस समार ह का उदघािन वकया

था

bull हॉनिवबल मह रति का नाम भारतीय हॉनिवबल क

नाम पर पडा ह ज एक विशाल और रोगीन जोगली

पकषी ह

bull यह पकषी नागालड राजय की अविकतर जनजावतय ो

की ल ककथाओो म उसललसखत ह

bull नागालड की परमख मानयता परापत जनजावतयाा ह

अोगामी आओ चखसोग चाोग ककी रगमा और

रवलोग आवद

bull ओोग जारिा और ससिनलीस अोडमान-वनक बार

दवीप समह की जनजावतयाा ह

FC19H1003 40

Q73) उततर (c)

सपषटीकरण

bull दकन म राषटर कि शासन दसिी ो सदी क अोत तक

लगभग 200 ििो तक रहा था

bull राषटर कि शासक अपन िावमिक विचार ो म सवहषण

bull उनह ोन न किल शि िमि और िषणि िमि बसलक

जन िमि क भी सोरकषण वदया था

bull एल रा म वशि क परवसद रॉक कि मोवदर का

वनमािण नौिी ो सदी म राषटर कि राजा कषण परथम न

करिाया था

bull उसका उततराविकारी अम घििि जन था लवकन

उसन अनय िमो क भी सोरकषण परदान वकया था

bull राषटर कि ो न मसलमान वयापाररय ो क बसन की

अनमवत दी थी

bull उनह न अपन अविराजय ो म इसलाम क उपदश दन

की भी अनमवत दी थी

bull अभी हाल ही म पाोडिलागटटा (तलोगाना) क

परागवतहावसक चटटान वचतर ो क कषरण की बिती हई

घिनाएा एक गोभीर वचोता का वििय ह

bull यह परागवतहावसक चटटान क नकसान पहाचा

सकता ह

bull पाोडिलागटटा वनमनवलसखत क वलए जाना जाता ह

- 10000 ईसा पिि स 8000 ईसा पिि क वचवतरत

चटटानी आशरय ो क वलए

- राषटर कि काल क एक 8 िी ो सदी क

वशलालख क वलए और

- 12िी ो सदी क काकविय सामराजय क वभवतत

वचतर ो क वलए

Q74) उततर (b)

सपषटीकरण

bull 1828 म राजा राम म हन रॉय न एक नय िावमिक

समाज बरहम सभा की थथापना की थी वजस बाद

म बरहम समाज क नाम स जाना गया था

bull दिदरनाथ िग र न ततवब विनी सभा की अधयकषता

की थी ज आधयासिक सतय की ख ज म सोलि

थी

bull इसका उददशय वहोद िमि क शद करन का और

एकशवरिाद (एक ईशवर म आथथा) का परचार करना

था

bull नय समाज की थथापना क आिार थ कारण

(तकि ) क द सतमभ तथा िद और उपवनिद

bull अभी हाल ही म सािारण बरहम समाज का कछ

काननी मदद ो क लकर पवशचम बोगाल सरकार क

साथ काननी वििाद चल रहा ह

Q75) उततर (c)

सपषटीकरण

bull भारत म वचशती वसलवसल की थथापना खवाजा

म इनददीन वचशती क दवारा की गयी थी

bull ि 1192 ईसवी क आसपास भारत आय थ

bull वचशतीय ो क बारहिी ो शताबदी क उततरािि म भारत

म आन िाल सफीय ो क समह ो म सबस

परभािशाली माना जाता ह

bull उनह ोन थथानीय िातािरण क साथ सफलतापििक

अनकलन वकया और उनह ोन भारतीय भसकत

परोपराओो क कई पहलओो क अपनाया

bull अजमर म सफी अपरकि खवाजा म इनददीन वचशती

की ऐवतहावसक दरगाह क एक नया रप दन की

तयारी की जा रही ह

bull इस 13िी ो शताबदी की दरगाह क ldquoसवचछ

आइकॉवनक थथल ोrdquo (Swacch Iconic Places) म

शावमल वकया गया ह ज परवतवषठत विरासत

आधयासिक और साोसकवतक थथान ो पर क वदरत

य जना ह

FC19H1003 41

ANSWERS amp EXPLANATION OF

NCERT History Class VI-X + Current Affairs

(FC19E1003)

Q1) Answer c

Explanation

Rigveda consists of more than a

thousand hymns dedicated to gods and

goddesses These hymns were

composed by sages and learnt by men

however a few were composed by

women like Apala Ghosa Lopamudra

Maitreyi and Gargi

Rigveda consists of many hymns in the

form of dialogues We get an example of

a dialogue between a sage named

Vishwamitra and two rivers (Beas and

Sutlej) that were worshipped as

goddesses This suggests that he

belonged to the Vedic period

Q2) Answer b

Explanation

Traces of ash have been found from

Kurnool Caves suggesting that people

were familiar with the use of fire

It is situated in Andhra Pradesh

Q3) Answer c

Explanation

Burzahom is a prehistoric site in

present day Kashmir where people built

pit houses which were dug into the

ground with steps leading into them

These may have provided shelter in cold

weather

Q4) Answer c

Explanation

Epigraphy is defined as the study of

inscriptions

Manuscriptology is the study of history

and literature through the use of hand

written documents

Palaeography refers to the study of

ancient writing systems and the

deciphering and dating of historical

manuscripts

Numismatics refers to the study of

coins

Q5) Answer a

Explanation

Charaka Samhita was written by

Charaka and is an important book on

Ayurveda and medicine

He was a practitioner of the traditional

system of Indian medicine known as

Ayurveda

Charaka is thought to have flourished

sometime between the 2nd century BCE

and the 2nd century CE

Q6) Answer b

Explanation

Bhaga refers to the tax on crops which

was fixed at 16th of the production

Kammakaras is the term used for the

landless agricultural labour class

Ashvamedha also known as horse

sacrifice is a ritual where a horse is let

loose to wander freely and it was

guarded by the rajarsquos men

Q7) Answer (d)

Explanation

In the Rigvedic period horses were

yoked to chariots that were used in

battles fought to capture land cattle

etc This suggests that the use of horse

chariots began much before the period

of Mahajanapadas

The battles were fought in the Rigvedic

period for cattlersquos lands water an even

to capture people Most men took part

in these wars however there was no

regular army but there were assemblies

where people met and discussed

matters of war Regular armies became

a feature in the Mjahajanapada period

including vast armies of foot soldiers

chariots and elephants

RAUSIAS-FC19E1003 42

Q8) Answer (a)

Explanation

Buddha belonged to the Sakya clan and

passed away at Kusinara

Buddha taught in Prakrit which was the

common language of people

Q9) Answer c

Explanation

There were six schools of philosophy in

ancient India These are known as

Vaishesika Nyaya Samkhya Yoga

Purva Mimansa and Vedanata or Uttara

Mimansa They were founded by sages

Kanada Gautama Kapila Patanjali

Jamini and Vyasa respectively

Q10) Answer b

Explanation

The teachings of Mahavira were

compiled at Valabhi in 6th century AD

Q11) Answer (c)

Explanation

Chanakya is traditionally identified as

Kautilya or Vishnugupta who authored

the ancient Indian political treatise the

Arthashastra

Q12) Answer d

The national emblem of India is an

adaptation of the Lion Capital atop the

Ashoka Pillar of Sarnath Uttar Pradesh

and is combined with the National

Motto Satyameva Jayate

The Rampurva Bull gets the name from

the site of its discovery Rampurva in

Bihar

It is noted for its delicately sculpted

model demonstrating superior

representation of soft flesh sensitive

nostrils alert ears and strong legs It is

a mixture of Indian and Persian

elements

Sankissa is situated in Uttar Pradesh

India

Q13) Ans(a)

Kunwar Singh was a notable leader during the Revolt of 1857 He belonged

to a royal house of Jagdispur Bihar

Q14) Answer b

Explanation

The term Vellalar was used for large

landowners

Q15) Answer c

Explanation

Arikamedu was a coastal settlement

where ships unloaded goods from

distant lands Finds here include a

massive brick warehouse pottery

including amphorae and Arretine ware

Roman lamps glassware and gems have

also been found at the site

Q16) Answer a

Explanation

Muvendar is a Tamil word mentioned in

Sangam poems meaning three chiefs

used for the heads of three ruling

families the Cholas Cheras and

Pandyas

Q17) Ans (c)

Several tribal or kin-based assemblies

such as the Sabha Vidatha and gana

are mentioned in the Rig-veda The

Sabha and the samiti mattered a great

deal in early Vedic times so much so

that the chiefs or the kings showed an

eagerness to win their support

Q18) Ans (a)

Jainism recognised the existence of the

gods but placed them lower than the

jina and did not condemn the varna

system as Buddhism did

Q19) Answer (d)

Explanation

Cholas and Pandyas had developed

powerful coastal cities The most

important city of Cholas was Puhar or

Kaveripattinam and Madurai was the

capital of Pandyas

Q20) Answer b

Explanation

Buddhacharita is the biography of

Buddha and was written by

RAUSIAS-FC19E1003 43

Ashvaghosha

Q21) Answer (a)

Explanation

Tamil poet Appar was a Shiva devotee

So he was a Nayanar saint

Q22) Answer d

Explanation

Samudragupta was a prominent Gupta

ruler whose coins depict him playing a

veena indicating his love for music We

get important historic information from

his Allahabad Prashasti which was

composed by his court poet Harisena

Q23) Answer (b)

Explanation

Vikrama Samvat was founded by

Chandragupta II in the 58 BC as a

mark of victory over the Shakas and

assumed the title of Vikramaditya

Banabhatta wrote Harshavardhanarsquos

biography the Harshacharita in

Sanskrit

Q24) Answer c

Explanation

Sandhi-vigrahika was the minister of

war and peace

Sarthavaha was the leader of the

merchant caravans

Q25) Answer a

Explanation

Xuan Zang (Hsuan-tsang) was a

Chinese traveller who came during the

reign of Harshavardhana

In the decade that began in 630 AD

Xuan Zang came to India through

Kashmir after visiting Central Asia Iran

and Afghanistan

He travelled from north to east and lived

in Bihar for a couple of years

At Nalanda University Xuan Zang

interacted with students and scholars

mastered local languages and

discovered Buddhist stupas

Q26) Answer c

Explanation

Pradakshina patha is a circular path

laid around a stupa in Buddhist

architecture While the rest are a part of

temple architecture

Q27) Answer d

Explanation

All the above-mentioned temples have

an elaborate use of bricks (baked

bricks) along with stone

Q28) Ans (c)

Muhammad Quli Qutab was the Sultan

of Golconda He was a contemporary of

Akbar was very fond of literature and

architecture

The Sultan was a great poet and he

wrote in Dakhini Urdu Persian and

Telgu and has left an extensive diwan or

collection

Recently the Archaeological Survey of

India (ASI) will be using Ground

Penetrating Radar (GPR) to map the

contours of the area around the Bagh-e-

Naya Qila excavated garden inside the

Golconda Fort in Telangana

Q29) Answer a

Explanation

Silappadikaram is a famous Tamil epic

which was written by Ilango around

1800 years ago It is a story of a

merchant named Kovalan who fell in

love with a courtesan named Madhavi

Manimekalai tells the story of the

daughter of Kovalan and Madhavi

Q30) Answer (a)

Explanation

Charaka is the author of Charaka

Samhita which is an important work of

Ayurveda and medicines

Brahmaguptarsquos fame rests mostly on his

Brahma-sphuta-siddhanta which was

an astronomical work It was translated

into Arabic in Baghdad and had a major

impact on Islamic mathematics and

astronomy

Late in his life Brahmagupta wrote

Khandakhadyaka which was an

RAUSIAS-FC19E1003 44

astronomical handbook that employed

Aryabhatarsquos system of starting each day

at midnight

Q31) Answer (c)

Explanation

Amir Khusrau was a famous sufi

musician poet and scholar In 1318 he

noted that there was different language

in every region of this land (Hindustan)

Lahori Kashmiri Dvarsamudri (in

Southern Karnataka) Telangana (in

Andhra Pradesh) Gujari (in Gujarat)

Marsquobari (in Tamil Nadu) Awadhi (in

eastern Uttar Pradesh) and Hindawai (in

the area around in Delhi) etc He went

to explain that Sanskrit did not belong

to any region and that only brahmans

knew it

Q32) Answer c

Explanation

Hiranyagarbha refers to the golden

womb When this ritual was performed

with the help of Brahmanas it was

thought to lead to the rebirth of the

sacrificer as a Khastriya

Q33) Answer d

Explanation

Kadamai refers to a tax on land

revenue

Gwalior Prashasti describes the exploits

of Nagabhata who was a Pratihara king

Q34) Answer b

Explanation

Rajatarangini is a Sanskrit text written

by Kalhana in the 12th century

It was historical chronicle of early India

It is justifiably considered to be the best

and most authentic work of its kind

It covers the entire span of history in

the Kashmir region from the earliest

times to the date of its composition

Q35) Answer c

Explanation

ldquoUrrdquo was the general assembly of the

village ldquoUrrdquo consisted of all the

taxpaying residents of an ordinary

village

Q36) Answer (a)

Explanation

Tarikh was a form of history writing in

the Delhi Sultanate The authors of

tawarikhs were learned men which

included secretaries administrators etc

Q37 Answer (a)

Explanation

Alauddin chose to pay his soldiers salaries in cash rather than iqtas The soldiers would buy their supplies from merchants in Delhi and it was thus feared that merchants would raise their prices To stop this Alauddin controlled the prices of goods in Delhi Prices were carefully surveyed by officers and merchants who did not sell at the prescribed rates were punished

Q38) Answer (d)

Explanation

Delhi first became the capital of a

kingdom under the Tomara Rajputs

who were defeated in the middle of the

twelfth century by the Chauhans (also

referred to as Chahamanas) of Ajmer

It was under the Tomaras and

Chauhans that Delhi became an

important commercial centre Many rich

Jaina merchants lived in the city and

constructed several temples Coins

minted here called dehliwal had a wide

circulation

Q39) Answer (c)

Explanation

Moth ki Masjid was built in the reign of

Sikandar Lodi by his minister

Begumpuri mosque built in the reign of

Muhammad Tughluq was the main

mosque of Jahanpanah the ldquoSanctuary

of the Worldrdquo and his new capital in

Delhi

Quwwat al ndash Islam mosque was

enlarged by Iltutmish and Alauddin

Khalji The minar was built by three

Sultansndash Qutbuddin Aybak Iltutmish

and Firuz Shah Tughluq

RAUSIAS-FC19E1003 45

Q40) Answer (c)

Explanation

Under the Mughals mansabdar was

referred to an individual who held a

mansab ie rank and he received his

salary as revenue assignments called

jagirs

Q41) Ans (b)

The Quit India Movement was a

spontaneous revolt of people against

British rule

The All India Congress Committee met

at Bombay on 8 August 1942 It passed

the famous resolution Quit India and

proposed the starting of a non-violent

mass struggle under Gandhis

leadership to achieve this aim But on

the very next day Gandhi and other

eminent leaders of the Congress were

arrested The Congress was once again

declared illegal

Q42) Ans (c)

The Simon Commission refers to a

group of seven MPs from the United

Kingdom constituted to suggest

constitutional reforms for British India

The Commission consisted of only

British members headed by one of the

senior British politicians Sir John

Simon

So the people of India agitated against

the arrival of Simon Commission

Q43) Ans (a)

He was widely known for his

unfavourable opinion of the economic

consequences of the British rule in

India

In his many writings and speeches and

especially in Poverty and Un-British

Rule in India Naoroji argued that India

was too highly taxed and that its wealth

was being drained away to England

He did not interpret the ancient Indian

texts and restored the self-confidence of

Indians And also he did not stress the

need for eradication of all the social

evils before anything else

Q44) Ans (c)

In August 1932 Prime Minister

MacDonald announced his Communal

Award Great Britainrsquos unilateral

attempt to resolve the various conflicts

among Indiarsquos many communal

interests

The award which was later

incorporated into the act of 1935

expanded the separate-electorate

formula reserved for Muslims to other

minorities including Sikhs Indian

Christians Anglo-Indians Europeans

distinct regional groups Gandhi

undertook a ldquofast unto deathrdquo against

that offer which he viewed as a

nefarious British plot to divide the

Indian society

Q45) Ans (b)

In British India apart from existing

imports and exports there was also a

particular amount of money which

colonial India contributed towards

administration maintenance of the

army war expenses pensions to retired

officers and other expenses accrued by

Britain towards maintenance of her

colony These were known as Home

charges and were paid for almost

entirely by India

The Home charges was made of

following components-

- Interest payable on Indian debt

- Dividend to shareholders of East

India Company

- Funds used to support the India

Office in London

- Funds used to pay salaries and

pensions of British personnel

engaged in India

- Interest on the railways

- Civil and military charges

- Store purchases in England

Q46) Ans (b)

The Lahore session of the Indian

National Congress was held in 1929

under the Presidentship of Jawaharlal

Nehru

The Lahore session of the Indian

National Congress witnessed significant

RAUSIAS-FC19E1003 46

developments in the Indian national

movement

- First the election of Jawaharlal

Nehru to the post of Presidentship of

the Congress was a clear indication

of the growing strength of the

Leftists in the Congress

- Secondly it was in this session that

the Congress for the first time raised

the demand for complete

independence Such demand was

not raised from the Congress

platform earlier

Q47) Ans (b)

It did not provide for separate

electorates for any community or

weightage for minorities However it did

allow for the reservation of minority

seats in provinces having minorities of

at least ten per cent but this was to be

in strict proportion to the size of the

community

There was no provision for complete

Independence for India

Q48) Ans (c)

The religion of early Vedic Aryans was

primarily of worship of nature and

Yajnas

The early Aryan religion was kind of

nature worship Actually the forces

around them which they could not

control or understand were invested

with divinity and were personified as

male or female gods And they

performed some Yajnas also

Q49) Ans (b)

The roads and river-routes were not

immune from robbery It is notable that

Yuan Chwang (Hiuen Tsang) was

robbed of his belongings during

Harshvardanarsquos period

Q50) Ans (c)

Q51) Ans (b)

Purandara Dasa was a saint and great

devotee of Lord Krishna

There is much speculation about where

Purandara Dasa regarded as the

Pitamaha of Carnatic music was born

Recently an expert committee

constituted by the Kannada University

Hampi has come to the conclusion that

Kshemapura Shivamogga district

Karnataka is the birth place of

Purandara Dasa

Q52) Ans (c)

Sri Tyagaraja Sri Shyama Shastry and Sri Muthuswami Dikshitar are considered the trinity of Carnatic music and with them came the golden age in Carnatic music in the 18th-19th

century

Q53) Ans d)

Recently a rare sarcophagus (stone

coffin) which is 2000 years old from the

Iron AgendashMegalithic era was discovered

from a rock-cut cave at Viyur village of

Kollam near Koyilandy in Kozhikode

district Kerala

The coffin containing bone fragments

was found during an excavation ldquoSo

far such a rare finding has been

discovered only from two sites

in Kerala Both these sarcophagi were

recovered from Megalithic sites at

Chevayur and Atholi also in Kozhikode

district

Q54) Ans a)

The megalithic culture in South India was a full-fledged Iron Age culture

Q55) Ans d)

The Cholas Pandyas and Keralaputras

(Cheras) mentioned in Ashokan

inscriptions were probably in the late

megalithic phase of material culture

Q56) Ans d)

Q57) Ans (b)

Raj Kumar Shukla followed Gandhiji all

over the country to persuade him to

come to Champaran to investigate the

problem associated with tinkathia

system

RAUSIAS-FC19E1003 47

Brij Kishore Rajendra Prasad Mahadev

Desai and Narhari Parikh accompanied

Gandhi ji during the Champaran

Satyagraha

Q58) Ans (b)

The Satvahanas started the practice of granting tax-free villages to brahmanas and Buddhist monks

Q59) Ans c)

The objectives of the Programme are

listed as under

- Developing basic tourism

infrastructure

- Promoting cultural and heritage

value of the country to generate

livelihoods in the identified regions

- Enhancing the tourist attractiveness

in a sustainable manner by

developing world-class

infrastructure at the heritage

monument sites

- Creating employment through active

involvement of local communities

- Harnessing tourism potential for its

effects on employment generation

and economic development

- Developing sustainable tourism

infrastructure and ensuring proper

Operations and maintenance

therein

Q60) Ans (b)

The Tribal Cooperative Marketing

Development Federation of India

(TRIFED) came into existence in 1987

It is a national-level apex organization

functioning under the administrative

control of Ministry of Tribal Affairs

Govt of India

TRIFED has its registered and Head

Office located in New Delhi

Q61) Ans (c)

Premchandrsquos novels include

Premashram Rangabhumi Ghaban

Karmabhumi and Godan

Gora is a novel written by Rabindranath

Tagore

138th birth anniversary of Munshi

Premchand was celebrated across the

country

Q62) Ans (b)

Giddha is a traditional pastoral dance

performed by the women of the Punjab

India and Pakistan at festival times

and at the sowing and reaping of the

harvest

By this dance the Punjabi women

reveal their joy expel their suppressed

feelings in a male dominated society

through the performance of Giddha

Since this dance has nothing to do with

men only women can participate in it

During the Teej celebrations Giddha

dance is celebrated in Punjab every

year Teej is a generic name for a

number of festivals that are celebrated

by women in some parts of India

Q63) Ans (a)

Dara Shukoh wrote the remarkable

work called ldquoMajma-ul-Bahrainrdquo or the

ldquoThe confluence of two seasrdquo

The Vice President of India Shri M

Venkaiah Naidu has said that Prince

Dara Shukohrsquos writings can come as a

refreshing source for infusing peace and

harmony He was addressing the

gathering after visiting the exhibition

that showcases the forgotten Prince of

yesteryears Dara Shukoh organized by

Mr Francois Gautier at Indira Gandhi

National Centre for the Arts in New

Delhi

Q64) Ans (c)

The statue Gommateshwara is

dedicated to the Jain God Bahubali

It is a monolithic statue

President Ram Nath Kovind

inaugurated the grand anointing

ceremony mdash Mahamastakabhisheka mdash

held once in 12 years at

Shravanabelagola (Karnataka)

Q65) Ans (c)

Prachi Valley had come up around the

Prachi river Prachi Valley gradually

disappeared

RAUSIAS-FC19E1003 48

The Prachi river originates from

Bhubaneswar

It is a tributary of the Mahanadi and

flows through the districts of Puri

Khurda Cuttack and Jagatsinghpur

and the entire region of the river is

termed as the Prachi Valley

It falls into the Bay of Bengal

Archaeological evidence shows that the

Prachi Valley Civilisation predates both

Harappa and Mohenjo-Daro

The Prachi river originates from

Bhubaneswar

Q66) Ans (d)

These monuments are located in

Chhatarpur district Madhya Pradesh

within Vindhya mountain range

Q67) Ans (a)

The book lsquoThoughts on Pakistanrsquo was

written by Dr BR Ambedkar

On the occasion of the birth anniversary

of Dr BR Ambedkar the president of

India pays homage to this icon of India

In 1924 he founded the Depressed

Classes Institute (Bahishkrit Hitkarini

Sabha) and in 1927 the Samaj Samata

Sangh

Another area of attention for Ambedkar

was education For its spread among

the low classes he set up a network of

colleges by the name of Peoples

Education Society and founded hostels

Q68) Ans(b)

Mehrgarh is a famous Neolithic

settlement in the Indian subcontinent

which is situated in Baluchistan

province Pakistan

A pre-historic rock art site is discovered

in the vast expanse of limestone blocks

on the eastern banks of Naguleru river

near Dachepalli (Andhra Pradesh) It

has thrown light on the Neolithic

civilisation that flourished in Guntur

(Andhra Pradesh) during 1500-2000

BC

Q69) Ans (c)

The 12th and the 13th centuries saw

the emergence of the Kakatiyas They

were at first the feudatories of the

Western Chalukyas of Kalyana Initially

they ruled over a small territory near

Warangal (Telangana)

They introduced Nayakships which was

later adopted and developed by the

Rayas of Vijayanagara

Q70) Ans (a)

The fast had effect of putting pressure

on mill owners who finally agreed to

give the workers a 35 per cent increase

in wages

Google celebrated with a doodle the

132nd birth anniversary of Anasuya

Sarabhai who played a pioneering role

in Indiarsquos labour movement

Q71) Ans (d)

The UNESCOrsquos list of the representative

list of the intangible cultural heritage of

humanity from India are

- Koodiyattam Sanskrit Theatre of

Kerala

- Mudiyettu ritual theatre and dance

drama of Kerala

- Tradition of Vedic Chanting

- Kalbelia folk songs and dances of

Rajasthan

- Ramlila Traditional Performance of

the Ramayana

- Sankirtana ritual singing

drumming and dancing of Manipur

- Ramman religious festival and

ritual theatre of the Garhwal

Himalayas India

- Traditional brass and copper craft of

utensil making among the Thatheras

of Jandiala Guru Punjab India

- Chhau dance classical Indian dance

originated in the eastern Indian

states

- Buddhist chanting of Ladakh

recitation of sacred Buddhist texts

in the trans-Himalayan Ladakh

region Jammu and Kashmir India

- Yoga

- Nouroz

- Kumbh Mela

RAUSIAS-FC19E1003 49

Q72) Ans(b)

The President of India Shri Ram Nath Kovind inaugurated the Hornbill Festival and State Formation Day celebrations of Nagaland in Kisama

The festival is named after the Indian hornbill the large and colourful forest bird which is displayed in the folklore of most of the states tribes

The major recognized tribes of Nagaland are Angami Ao Chakhesang Chang

Kuki Rengma and Zeling etc

Onge Jarawa and Sentinelese are the

tribes of Andman amp Nicobar Islands

Q73) Ans (c)

The Rashtrakutas rule in the Deccan lasted for almost two hundred years till the end of the tenth century The Rashtrakutas rulers were tolerant in their religious views and patronized not only Shaivism and Vaishnavism but

Jainism as well

The famous rock-cut temple of Shiva at Ellora was built by one of the Rashtrakutas kings Krishna I in the ninth century His successor Amoghavarsha was a Jain but he also

patronized other faiths

The Rashtrakutas allowed Muslims traders to settle and permitted Islam to

be preached in their dominions

Recently increasing defacement at the prehistoric rock paintings of Pandavulagutta Telangana has created a cause for grave concern It can spoil

the prehistoric rock

Pandavulagutta is home to

- Painted rock shelters dating to

10000 BC-8000 BC

- An 8th century inscription of the

Rashtrakuta period and

- Painted frescoes from the 12th century Kakatiya empire

Q74) Ans (b)

In 1828 Raja Ram Mohan Roy founded a new religious society the Brahma Sabha later known as the Brahmo

Samaj

Debendranath Tagore headed the Tattvabodhini Sabha which was

engaged in search of spiritual truth

Its purpose was to purify Hinduism and to preach monotheism or belief in one God

The new society was to be based on the twin pillars of reason and the Vedas and

Upanishads

Recently Sadharan Brahmo Samaj (SBS) has entered into a legal battle with the West Bengal government due

to some legal issue

Q75) Ans (c)

The Chishti order was established in India by Khwaja Moinuddin Chishti who came to India around 1192 The Chishtirsquos are considered to be the most influential of the groups of Sufis who migrated to India in the late twelfth century They adapted successfully to the local environment and adopted several features of Indian devotional

traditions

The historical dargah of Sufi mystic Khwaja Moinuddin Chishti in Ajmer is all set to get a facelift This 13 th century dargah has been included among the Swachh Iconic Places a clean-up initiative focused on iconic

heritage spiritual and cultural places

Page 22: GENERAL STUDIES (PAPER I) · Test is part of Rau’s IAS Test series for Preliminary Exam 2019 FOUNDATION + CURRENT AFFAIRS GENERAL STUDIES (PAPER –I) FOUNDATION TEST –III TOPIC:

RAUSIAS-FC19E1003 22

Q60) ldquoभारतीय जिजातीय सहकारी निपरि निकास सोघrdquo

(The Tribal Co-operative Marketing

Development Federation of India - TRIFED)

क सोदभण म निमननिखित कथि ो पर निचार कीनजए

1 यह एक राषटर ीय सतर का शीिण सोगठि ह ज

भारत सरकार क गह मोतरािय क परशासनिक

नियोतरर क अरीि काम कर रहा ह

2 इसका मखय उददशय दश म जिजातीय ि ग ो

का सामानजक-आनथणक निकास करिा ह

उपयणकत कथि ो म स कौि-सास सही हह

(a) किि 1

(b) किि 2

(c) 1 और 2 द ि ो

(d) ि त 1 ि ही 2

Q61) निमननिखित म स कौि-सास उपनयास परमचोद क

दवारा नििा गया हनिि गए ह

1 रोगभनम

2 ग दाि

3 ग रा

िीच नदए गए कट का परय ग कर सही उततर चनिए

(a) किि 1

(b) किि 2

(c) किि 1 और 2

(d) 1 2 और 3

Q62) नगदधा ितय क सोदभण म निमननिखित कथि ो पर निचार

कीनजए

1 नगदधा नबहार की मनहिाओो क दवारा तयौहार क

समय और फसि की बिाई तथा कटाई क

अिसर पर नकया जाि िािा एक पारोपररक

दहाती ितय ह

2 इस ितय क दवारा मनहिाऐो अपिी परसननता

परकट करती ह तथा नगदधा क परदशणि क

माधयम स परि िचणसव िाि समाज म

मनहिाओो की दबी हई भाििाओो क परकट

करती ह

उपयणकत कथि ो म स कौि-सास सही हह

(a) किि 1

(b) किि 2

(c) 1 और 2 द ि ो

(d) ि त 1 ि ही 2

Q63) निमननिखित कथि ो पर निचार कीनजए

1 मलला शाह बदखशी दारा नशक ह क

आधयाखतमक गर थ

2 औरोगरब ि मजम-उि-बहरीि या द समदर ो

का सोगम िामक उललििीय रचिा नििी थी

3 दारा नशक ह क अपि पिणज अकबर क गर ो

क उततरानरकारी क रप म दिा गया था

नजसम उसि रानमणक बहििाद और समनवयता

क बढ़ािा नदया था

उपयणकत कथि ो म स कौि-सास सही हह

(a) किि 1 और 3

(b) किि 2

(c) किि 1 और 2

(d) 1 2 और 3

RAUSIAS-FC19E1003 23

Q60) Consider the following statements about

the Tribal Cooperative Marketing

Development Federation of India

(TRIFED)

1 It is a national-level apex

organization functioning under the

administrative control of Ministry

of Home Affairs Government of

India

2 The main objective of TRIFED is

socio-economic development of

tribal people in the country

Which of the statements given above

isare correct

(a) 1 only

(b) 2 only

(c) Both 1 and 2

(d) Neither 1 nor 2

Q61) Which of the following novels isare

written by Premchand

1 Rangabhumi

2 Godan

3 Gora

Select the correct answer using the code

given below

(a) 1 only

(b) 2 only

(c) 1 and 2 only

(d) 1 2 and 3

Q62) Consider the following statements about

Giddha dance

1 Giddha is a traditional pastoral

dance performed by the women of

Bihar at festival times and at the

sowing and reaping of the harvest

2 By this dance the women reveal

their joy expel their suppressed

feelings in a male dominated

society through the performance of

Giddha

Which of the statements given above

isare correct

(a) 1 only

(b) 2 only

(c) Both 1 and 2

(d) Neither 1 nor 2

Q63) Consider the following statements

1 Mullah Shah Badakhshi was the

spiritual mentor of Dara Shukoh

2 Aurangzeb wrote the remarkable

work called ldquoMajma-ul-Bahrainrdquo or

the ldquoThe confluence of two seasrdquo

3 Dara Shukoh was seen as

inheriting the qualities of his

ancestor Akbar in that he

promoted religious pluralism and

syncretism

Which of the statements given above

isare correct

(a) 1 and 3 only

(b) 2 only

(c) 1 and 2 only

(d) 1 2 and 3

RAUSIAS-FC19E1003 24

Q64) निमननिखित कथि ो पर निचार कीनजए

1 ग मतशवर परनतमा निोधयनगरी पहाड़ी पर खसथत ह

2 शरिरबिग िा िह सथाि ह जहाो मौयण िोश क

सोसथापक चोदरगपत मौयण अपि नसोहासि क

तयागि क बाद जि तपसवी बि गए थ

उपयणकत कथि ो म स कौि-सास सही हह

(a) किि 1

(b) किि 2

(c) 1 और 2 द ि ो

(d) ि त 1 ि ही 2

Q65) निमननिखित कथि ो पर निचार कीनजए

1 पराताखतवक साकषय स पता चिता ह नक पराची

घाटी सभयता हड़पपा और म हिज दाड़ द ि ो

की पिणिती ह

2 पराची िदी भििशवर स निकिती ह

उपयणकत कथि ो म स कौि-सास सही हह

(a) किि 1

(b) किि 2

(c) 1 और 2 द ि ो

(d) ि त 1 ि ही 2

Q66) निमननिखित कथि ो म स कौि-सास सही हह

1 िजराह क समारक ो क समह का निमाणर

चोदि राजिोश क शासिकाि क दौराि हआ

था

2 य समारक हररिोदर पिणत शरोििा म खसथत ह

3 म रक क यातरी इबन बतता ि अपि सोसमरर ो

म िजराह क मोनदर ो की यातरा का उललि

नकया था तथा इन काजराण िाम स समब नरत

नकया था

िीच नदए गए कट का परय ग कर सही उततर चनिए

(a) किि 1

(b) किि 1 और 2

(c) किि 2 और 3

(d) किि 1 और 3

Q67) निमननिखित कथि ो म स कौि-सास सही हह

1 डॉ बी आर अमबडकर ि दी एनिनहिशि

ऑफ़ कासट (The Annihilation of Caste)

नििी थी नजसम उन ोि नहोद रमण म िोशािगत

पजारी की परथा क उनमिि की आिशयकता

पर बि नदया था

2 डॉ राजदर परसाद ि थॉटस ऑि पानकसताि

(Thoughts on Pakistan) िामक पसतक

नििी थी

िीच नदए गए कट का परय ग कर सही उततर चनिए

(a) किि 1

(b) किि 2

(c) 1 और 2 द ि ो

(d) ि त 1 ि ही 2

Q68) निमननिखित कथि ो म स कौि-सास सही हह

1 महरगढ़ भारतीय उपमहादवीप म एक परनसदध

ििपािार बसती ह ज नसोर पराोत पानकसताि म

खसथत ह

2 बरणह म म कतत ो क उिक सवामी क साथ कबर ो

म दफिाया जाता था

िीच नदए गए कट का परय ग कर सही उततर चनिए

(a) किि 1

(b) किि 2

(c) 1 और 2 द ि ो

(d) ि त 1 ि ही 2

Q69) निमननिखित कथि ो म स कौि-सास सही हह

1 काकानटय मोनदर अनरकतर नशि क समनपणत

2 हिमक ोडा म हजार-सतोभ िाि मोनदर (The

Thousand-Pillared Temple) का निमाणर

काकानटय समराट रदर ि करिाया था

िीच नदए गए कट का परय ग कर सही उततर चनिए

(a) किि 1

(b) किि 2

(c) 1 और 2 द ि ो

(d) ि त 1 ि ही 2

RAUSIAS-FC19E1003 25

Q64) Consider the following statements

1 Gommateshwara Statue is located

on the Vindyagiri Hill

2 Shravanabelagola is the place

where Chandragupta Maurya the

founder of the Mauryan dynasty

became a Jain ascetic after

relinquishing his throne

Which of the statements given above

isare correct

(a) 1 only

(b) 2 only

(c) Both 1 and 2

(d) Neither 1 nor 2

Q65) Consider the following statements

1 Archaeological evidence shows

that the Prachi Valley Civilisation

predates both Harappa and

Mohenjo-Daro

2 The Prachi river originates from

Bhubaneswar

Which of the statements given above

isare correct

(a) 1 only

(b) 2 only

(c) Both 1 and 2

(d) Neither 1 nor 2

Q66) Which of the following statements

isare correct

1 The Khajuraho group of

monuments was built during the

rule of the Chandela dynasty

2 These monuments are located in

Harischandra mountain range

3 Ibn Battuta the Moroccan

traveller in his memoirs mentioned

visiting Khajuraho temples and

called them Kajarra

Select the correct answer using the code

given below

(a) 1 only

(b) 1 and 2

(c) 2 and 3

(d) 1 and 3

Q67) Which of the following statements

isare correct

1 Dr BR Ambedkar wrote the

Annihilation of Caste emphasising

the need to do away with the

practice of hereditary priesthood in

Hinduism

2 The book lsquoThoughts on Pakistanrsquo

was written by Dr Rajendra

Prasad

Select the correct answer using the code

given below

(a) 1 only

(b) 2 only

(c) Both 1 and 2

(d) Neither 1 nor 2

Q68) Which of the following statements

isare correct

1 Mehrgarh is a famous Neolithic

settlement in the Indian

subcontinent which is situated in

Sindh province Pakistan

2 At Burzahom dogs were buried

with their masters in their graves

Select the correct answer using the code

given below

(a) 1 only

(b) 2 only

(c) Both 1 and 2

(d) Neither 1 nor 2

Q69) Which of the following statements

isare correct

1 The Kakatiya temples are

dedicated mostly to Siva

2 The Thousand-Pillared Temple at

Hanamkonda was built by the

Kakatiya king Rudra

Select the correct answer using the code

given below

(a) 1 only

(b) 2 only

(c) Both 1 and 2

(d) Neither 1 nor 2

RAUSIAS-FC19E1003 26

Q70) निमननिखित कथि ो म स कौि-सास सही हह

1 अहमदाबाद नमि हड़ताि क दौराि महातमा

गाोरी ि शरनमक ो क पकष क मजबत करि क

निए आमरर अिशि नकया था

2 अिशि स नमि मानिक ो पर दबाि पड़ा था ज

अोततः शरनमक ो क िति म 15 परनतशत की िखदध

करि क निए सहमत हए थ

िीच नदए गए कट का परय ग कर सही उततर चनिए

(a) किि 1

(b) किि 2

(c) 1 और 2 द ि ो

(d) ि त 1 ि ही 2

Q71) निमननिखित म स नकसक नकिक भारत स यिसक

की माििता की अमतण साोसकनतक निरासत की

परनतनिनर सची (The UNESCOrsquos List of the

Representative List of the Intangible

Cultural Heritage of Humanity) म शानमि

नकया गया ह

1 मनडयटट

2 सोकीतणि

3 को भ मिा

िीच नदए गए कट का परय ग कर सही उततर चनिए

(a) किि 1 और 2

(b) किि 2 और 3

(c) किि 3

(d) 1 2 और 3

Q72) निमननिखित जिजानतय ो म स कौि-सीसी ो

जिजानतजिजानतया िागािड स सोबोनरत हह

1 अोगामी

2 ककी

3 जारिा

िीच नदए गए कट का परय ग कर सही उततर चनिए

(a) किि 1

(b) किि 1 औऔ 2

(c) किि 2

(d) 1 2 और 3

Q73) निमननिखित कथि ो म स कौि-सास सही हह

1 राषटर कट सामराजय की सथापिा दोनतदगण ि की थी

नजसि मानयाित म अपिी राजरािी की

सथापिा की थी

2 राषटर कट समराट अम घििण एक ििक था और

उस कनिताओो पर पहिी कननड़ पसतक नििि

का शरय नदया जाता ह

िीच नदए गए कट का परय ग कर सही उततर चनिए

(a) किि 1

(b) किि 2

(c) 1 और 2 द ि ो

(d) ि त 1 ि ही 2

Q74) निमननिखित कथि ो म स कौि-सास सही हह

1 कशब चोदर सि ि ततवब नरिी सभा की

अधयकषता की थी ज आधयाखतमक सतय की

ि ज म सोिि थी

2 बरहम समाज ि मािि गररमा पर बि नदया

मनतणपजा का निर र नकया और सती परथा जसी

सामानजक बराइय ो की आि चिा की

िीच नदए गए कट का परय ग कर सही उततर चनिए

(a) किि 1

(b) किि 2

(c) 1 और 2 द ि ो

(d) ि त 1 ि ही 2

Q75) निमननिखित कथि ो म स कौि-सास सही हह

1 भारत म नचशती नसिनसिा खवाजा म इिददीि

नचशती क दवारा सथानपत नकया गया था

2 नचशती परोपरा की एक परमि निशिता

आतमसोयम थी नजसम साोसाररक म ह स दरी

बिाए रििा शानमि था

िीच नदए गए कट का परय ग कर सही उततर चनिए

(a) किि 1

(b) किि 2

(c) 1 और 2 द ि ो

(d) ि त 1 ि ही 2

RAUSIAS-FC19E1003 27

Q70) Which of the following statements

isare correct

1 During the Ahmedabad Mill Strike

Mahatma Gandhi undertook a fast

unto death to strengthen the

workersrsquo resolve

2 The fast had effect of putting

pressure on mill owners who

finally agreed to give the workers a

15 per cent increase in wages

Select the correct answer using the code

given below

(a) 1 only

(b) 2 only

(c) Both 1 and 2

(d) Neither 1 nor 2

Q71) Which of the following are included in

the UNESCOrsquos list of the representative

list of the intangible cultural heritage of

humanity from India

1 Mudiyettu

2 Sankirtana

3 Kumbh Mela

Select the correct answer using the code

given below

(a) 1 and 2 only

(b) 2 and 3 only

(c) 3 only

(d) 1 2 and 3

Q72) Which of the following tribes isare

related to Nagaland

1 Angami

2 Kuki

3 Jarawa

Select the correct answer using the code

given below

(a) 1 only

(b) 1 and 2 only

(c) 2 only

(d) 1 2 and 3

Q73) Which of the following statements

isare correct

1 Rashtrakuta kingdom was founded by Dantidurga who established his capital at Manyakhet

2 Amoghavarsha a Rashtrakuta king was an author and is credited with writing the first

Kannada book on poetics

Select the correct answer using the code given below

(a) 1 only

(b) 2 only

(c) Both 1 and 2

(d) Neither 1 nor 2

Q74) Which of the following statements isare correct

1 Keshab Chandra Sen headed the Tattvabodhini Sabha which was engaged in search of spiritual truth

2 The Brahmo Samaj laid emphasis on human dignity opposed idolatry and criticized such social

evils as the practice of Sati

Select the correct answer using the code given below

(a) 1 only

(b) 2 only

(c) Both 1 and 2

(d) Neither 1 nor 2

Q75) Which of the following statements isare correct

1 The Chishti order was established in India by Khwaja Moinuddin

Chishti

2 A major feature of the Chishti tradition was austerity including maintaining a distance from the

worldly power

Select the correct answer using the code

given below

(a) 1 only

(b) 2 only

(c) Both 1 and 2

(d) Neither 1 nor 2

T e s t i s p a r t o f R a u rsquo s I A S T e s t s e r i e s f o r P r e l i m i n a r y E x a m 2 0 1 9

FOUNDATION + CURRENT AFFAIRS

GENERAL STUDIES (PAPER ndashI)

FOUNDATION TEST ndashIII

SUBJECT NCERT History Class VI-X + Current Affairs

Time Allowed 1frac12 Hours Maximum Marks 150

I NSTRUCT IONS

1 IMMEDIATELY AFTER THE COMMENCEMENT OF THE EXAMINATION YOU SHOULD CHECK

THAT THIS TEST BOOKLET DOES NOT HAVE ANY UNPRINTED OR TORN or MISSING PAGES OR

ITEMS ETC IF SO GET IT REPLACED BY A COMPLETE TEST BOOKLET

2 This Test Booklet contains 75 items (questions) Each item is printed both in Hindi and English

Each item comprises four responses (answers) You will select the response which you want to mark

on the Answer Sheet In case you feel that there is more than one correct response mark the

response which you consider the best In any case choose ONLY ONE response for each item

3 You have to mark all your responses ONLY on the separate Answer Sheet (OMR sheet) provided

Read the directions in the Answer Sheet

4 All items carry equal marks

5 Before you proceed to mark in the Answer Sheet the response to various items in the Test booklet

you have to fill in some particulars in the Answer Sheet as per instructions contained therein

6 After you have completed filling in all your responses on the Answer Sheet and the examination has

concluded you should hand over to the Invigilator only the Answer Sheet You are permitted to

take away with you the Test Booklet

7 Penalty for wrong answers

THERE WILL BE PENALTY FOR WRONG ANSWERS MARKED BY A CANDIDATE IN THE

OBJECTIVE TYPE QUESTION PAPERS

(i) There are four alternatives for the answer to every question For each question for which a

wrong answer has been given by the candidate one-third of the marks assigned to that

question will be deducted as penalty

(ii) If a candidate gives more than one answer it will be treated as a wrong answer even if one of

the given answers happens to be correct and there will be same penalty as above to that

question

(iii) If a question is left blank ie no answer is given by the candidate there will be no penalty for

that question

T h i s t e s t i s p a r t o f R a u rsquo s I A S T e s t s e r i e s f o r P r e l i m i n a r y E x a m 2 0 1 9

Test Code

FC19E1003

FC19H1003 29

Answers and Explanations of

NCERT History Class VI-X + Current Affairs (FC19E1003)

Q1) उततर (c)

सपषटीकरण

- ऋगवद म दविय ो और दिताओो क समवपित एक

हजार स अविक सत तर (शल क) ह

- य शल क ऋविय ो क दवारा रच गए थ और परि ो

दवारा सीख जात थ

- हालाोवक कछ शल क मवहलाओो (जस वक अपाला

घ सा ल पामदरा मतरयी और गागी) क दवारा भी रच

गए थ

- ऋगवद म सोिाद क रप म कई शल क मौजद ह

- हम विशवावमतर नामक एक ऋवि और दविय ो क

रप म पजी जान िाली द नवदय ो (वयास और

सतलज) क बीच िाताि का उदाहरण वमलता ह

- इसस पता चलता ह वक विशवावमतर िवदक काल स

सोबोवित थ

Q2) उततर (b)

सपषटीकरण

- करनल गफाओो स राख क अिशि परापत हए ह

ज इस ओर सोकत करत ह वक ततकालीन ल ग

अवि क उपय ग स पररवचत थ

- य गफाएो आोधर परदश म सथथत ह

Q3) उततर (c)

सपषटीकरण

bull बरािह म ितिमान कशमीर म सथथत एक

परागवतहावसक थथल ह जहाो ल ग गडढ क घर ो का

वनमािण करत थ

bull य घर जमीन क ख द कर बनाए जात थ तथा नीच

जान क वलए सीवियाा ह ती थी

bull ऐसा अनमान लगाया जाता ह वक य घर ठो ड क

मौसम म आशरय परदान करत थ

Q4) उततर (c)

सपषटीकरण

bull परालख-विदया (Epigraphy) क वशलालख ो क

अधययन क रप म पररभावित वकया जाता ह

bull हसतवलसखत दसतािज ो क माधयम स इवतहास

और सावहतय क अधययन क पाोडवलवप विजञान

(Manuscriptology) कहत ह

bull पराचीन लखन परणावलय ो क अधययन और

ऐवतहावसक पाोडवलवपय ो क समझन तथा वतवथ

वनिािरण क पलीओगराफी (Palaeography) कहा

जाता ह

bull नयवमजमविकस (Numismatics) वसक ो क

अधययन क सोदवभित करता ह

Q5) उततर (a)

सपषटीकरण

- चरक सोवहता चरक क दवारा वलखी गई आयिद

और िदयक-शासर पर एक महतवपणि पसतक ह

- ि भारतीय िदयक-शासर की पारमपररक परणाली

वजस आयिद क नाम स जाना जाता ह क

अभयासकताि थ

- ऐसा माना जाता ह वक चरक का विकास दसरी

शताबदी (ईसा पिि) और दसरी शताबदी (ईसवी) क

मधय हआ था

Q6) उततर (b)

सपषटीकरण

- भाग फसल ो पर वलए जान िाल कर क सोदवभित

करता ह ज कल फसल उतपादन का 16 िाो भाग

था

- ldquoकममकारrdquo शबद भवमहीन कवि शरवमक िगि क

वलए परय ग वकया जाता था

- ldquoअशवमिrdquo (वजस घ ड क बवलदान क रप म भी

जाना जाता ह) एक अनषठान ह ता था वजसम एक

घ ड क सवतोतर रप स घमन क वलए छ ड वदया

FC19H1003 30

जाता ह और राजा क सवनक उसकी रखिाली

करत थ

Q7) उततर (d)

सपषटीकरण

- ऋगववदक काल म घ ड ो क रथ ो म ज ता जाता था

ज (रथ) भवम मिवशय ो आवद पर कबजा करन क

वलए लड गए यद ो म उपय ग वकए जात थ

- इसस यह पता चलता ह वक घ ड ो यकत रथ ो का

उपय ग महाजनपद काल स काफी पहल आरमभ

हआ था

- ऋगववदक काल म मिवशय ो भवम जल आवद पर

कबजा करन क वलए तथा ल ग ो क पकडन क

वलए यद वकय जात थ

- अविकाोश परि इन यद ो म भाग वलया करत थ

- हालाोवक उस समय क ई वनयवमत सना नही ो ह ती

थी लवकन उस काल म सभाऐो ह ती थी ो वजनम

ल ग यद क मामल ो पर चचाि करत थ

- वनयवमत सनाएा महाजनपद काल का िवशषटय थी

वजनम पदल सवनक ो की विशाल सनाएा रथ तथा

हाथी शावमल ह त थ

Q8) उततर (a)

सपषटीकरण

- बद शाकय कल स सोबोवित थ और कशीनारा म

उनका वनिन हआ था

- बद न अपनी वशकषाएा पराकत भािा म दी थी ो ज

आम ल ग ो की भािा थी

Q9) उततर (c)

सपषटीकरण

- पराचीन भारत म दशिनशासर की छह शाखाएा थी ो

िशविक नयाय समखया य ग पिि वममाोसा और

िदाोत या उततर वममाोसा

- इनकी थथापना करमश कनाद गौतम कवपल

पतोजवल जावमनी और वयास ऋविय ो न की थी

Q10) उततर (b)

सपषटीकरण

महािीर की वशकषाऐो छठी शताबदी म िललभी म

सोकवलत की गई थी ो

Q11) उततर (c)

सपषटीकरण

- पारमपररक रप स चाणकय क कौविलय अथिा

विषणगपत क नाम स जाना जाता ह

- उसन अथिशासतर ज एक पराचीन भारतीय

राजनवतक आलख ह वलखा था

Q12) उततर (d)

सपषटीकरण

- भारत का राषटर ीय वचनह सारनाथ (उततर परदश) क

अश क सतमभ क ऊपर (शीिि पर) वसोह कवपिल

का एक अनरपण ह

- इस राषटर ीय वसदाोत सतयमि जयत क साथ

सोय वजत वकया गया ह

- रामपिि बल का नाम रामपिि (वबहार) क नाम पर

पडा जहाा इसकी ख ज हई थी

- यह अपन नाजक नकाशी मॉडल क वलए परवसदद

ह वजसम क मल तवचा सोिदनशील नथन ो सतकि

कान और मरबत िााग ो क शरषठतर परवतरप क

परदवशित वकया गया ह

- यह भारतीय और फारसी ततव ो का एक ससममशरण

- सोवकससा उततर परदश म सथथत ह

Q13) उततर (a)

सपषटीकरण

का िर वसोह ज एक महान य दा थ वबहार स

सोबोवित थ

Q14) उततर (b)

सपषटीकरण

िललालर शबद बड भ-सवावमय ो क वलए परय ग

वकया जाता था

FC19H1003 31

Q15) उततर (c)

सपषटीकरण

- अररकमड एक तिीय बसती थी जहाो दर दश ो स

आन िाल जहाज ो का माल उतारा जाता था

- यहाो पर ईोि ो का एक विशाल ग दाम वमटटी क

बतिन (वजनम एमफ रा - द हरी मवठय ो का लोबा

घडा - शावमल ह) और एरिाइन (Arretine)

मदभाोड पाए गए थ

- इस थथान पर र मन दीपक काोच क बन पातर और

रतन भी पाए गए थ

Q16) उततर (a)

सपषटीकरण

- मिनदर सोगम कविताओो म उसललसखत एक

तवमल शबद ह वजसका अथि ह ldquoतीन परमखrdquo

- यह तीन सततारि पररिार ो क मसखयाओो क वलए

परय ग वकया जाता ह च ल चर और पाणडय

Q17) उततर (c)

सपषटीकरण

- ऋग िद म सभा विदाथा तथा गण जसी

जनजावतय ो पर अथिा किोब पर आिाररत

सभाओो का उललख ह

- आरसमभक िवदक काल म सभाओो और सवमवतय ो

का विशि महतव ह ता था

- यहाा तक की मसखया अथिा राजा भी उनका

समथिन परापत करन क वलए आतर रहत थ

Q18) उततर (a)

सपषटीकरण

- जन िमि न ईशवर क अससततव क मानयता त दी ह

वकनत उसन ईशवर क वजना क पद स नीच रखा

- जन िमि न बौद िमि की तरह िणि परणाली की

भरतिना नही ो की थी

Q19) उततर (d)

सपषटीकरण

- च ल ो और पाणडय ो न शसकतशाली तिीय शहर ो का

विकास वकया था

- च ल ो का सबस महतवपणि शहर पहार (या

कािरीपटटीनम) था |

- मदरई पाणडय ो की राजिानी थी

Q20) उततर (b)

सपषटीकरण

- ldquoबदचररतrdquo बद का जीिन-ितताोत ह

- इस अशवघ ि क दवारा वलखा गया था

Q21) उततर (a)

सपषटीकरणः

- तवमल कवि अपपर भगिान वशि क भकत थ

- इस परकार ि एक नयनार सोत थ

Q22) उततर (d)

सपषटीकरणः

- समदरगपत एक परवसद गपत शासक था

- उसन वसक ो पर िीणा बजात हए अपनी छवि

अोवकत करिाई थी

- यह सोगीत क परवत उसक परम क दशािता ह

- हम उसकी इलाहाबाद परशससत स महतवपणि

ऐवतहावसक जानकारी वमलती ह वजसकी रचना

उसक दरबार क कवि हररसन न की थी

Q23) उततर (b)

सपषटीकरणः

- विकरम सोित की शरआत ििि 58 ईसा पिि म

चनदरगपत वदवतीय न की थी

- यह शक ो पर उसकी जीत और उस विकरमावदतय

की पदिी वमलन क उपलकषय म आरमभ वकया गया

था

FC19H1003 32

- बानभटट न हिििििन का जीिन-ितताोत हििचररत

(ज सोसकत म थी) वलखी थी

Q24) उततर (c)

सपषटीकरणः

- सोवि-विगरावहका यद एिो शाोवत का मोतरी

- साथििाह वयापाररय ो क कावफल ो का नता

Q25) उततर (a)

सपषटीकरणः

- जआन झाोग (हसआन रताोग ndash Hsuang Tsang)

एक चीनी यातरी था ज हिििििन क शासनकाल म

भारत आया था

- ििि 630 ईसवी स ज दशक आरमभ हआ था उसम

जआन झाोग मधय एवशया ईरान और

अफग़ावनसतान की यातरा करन क पशचात कशमीर

क रासत स भारत आया था

- उसन उततर स पिि तक की यातरा की और िह

लगभग 2 ििि वबहार म रहा

- जआन झाोग न नालनदा विशवविदयालय म विदयावथिय ो

और विदवान ो क साथ पारसपररक विचार-विमशि

वकया थथानीय भािाओ ा म वनपणता परापत की तथा

बौद सतप ो की ख ज की

Q26) उततर (c)

सपषटीकरणः

- परदवकषणा पथ बौद िासतकला म सतप क चार ो

ओर बनाया जान िाला एक घमािदार पथ ह ता

- परशन म वदए गए बाकी क तीन ो ततव वहोद मसनदर ो की

िासतकला क भाग ह

Q27) उततर (d)

सपषटीकरणः

परशन म वदए गए सभी मोवदर ो म वयापक रप स

ईोि ो (पकी ईोि ो) का परय ग पतथर ो क साथ हआ

Q28) उततर (c)

सपषटीकरण

- महममद कली कतब शाह ग लकणडा का सलतान

था

- िह अकबर का समकालीन था

- सावहतय और िासतकला म उसकी अतयाविक

रवच थी

- िह एक महान कवि था

- िह दसखनी उदि फारसी और तलग म वलखता था

- उसन अपन पीछ एक विसतत वदिान (सोगरह)

छ डा ह

- अभी हाल ही म तलोगाना म ग लकणडा क वकल

क अनदर खदाई वकय गए बाग-ए-नाया वकला

बाग क चार ो ओर रप-रखा क मानवचतरण क

वलए भारतीय परातासतवक सिकषण (The

Archaeological Survey of India ndash ASI)

गराउणड पनीिर विोग रडार (Ground Penetrating

Radar) का परय ग करगा

Q29) उततर (a)

सपषटीकरणः

- वसलपपावदकारम एक तवमल महाकावय ह वजसकी

रचना इलाोग क दवारा लगभग 1800 ििि पिि की

गई थी

- यह क िलन नामक एक वयापारी की कहानी ह

ज माििी नामक एक गवणका (िशया) स परम

करन लगा था

- मवनमकलाई क िलन और माििी की पतरी की

कहानी ह

Q30) उततर (a)

सपषटीकरण

- चरक आयिद और वचवकरता की एक महतवपणि

रचना चरक सोवहता क लखक ह

- बरहमगपत क अपनी रचना बरहम-सफि-वसदानत

(ज एक खग लीय रचना ह) क कारण परवससद

वमली

FC19H1003 33

- बगदाद म इसका अनिाद अरबी भािा म वकया

गया था

- इसका इसलावमक गवणत और खग ल-विजञान पर

महतवपणि परभाि पडा था

- बाद म अपन जीिनकाल म बरहमगपत न

ldquoखोडखयाकrdquo वलखी ज एक खग लीय पससतका

(एक छ िी पसतक) थी

- इसम आयिभटट की अिि-रावतर क परतयक वदन की

शरआत परणाली का परय ग वकया गया था

Q31) उततर (c)

सपषटीकरण

- अमीर खसर एक परवसद सफी सोगीतकार कवि

और विदवान थ

- 1318 म उनह ोन पाया वक इस भवम (वहोदसतान) क

हर कषतर म अलग-अलग भािा थी लाहौरी

कशमीरी दवारसमदरी (दवकषणी कनाििक म)

तलोगाना (आोधर परदश म) गजरी (गजरात म)

माबारी (तवमलनाड म ) अििी (पिी उततर परदश

म) और वहोदिी (वदलली क आस-पास क कषतर म)

आवद

- उनह न यह बताया वक सोसकत वकसी भी कषतर स

सोबोवित नही ो थी और किल बराहमण ही इस भािा

का जञान रखत थ

Q32) उततर (c)

सपषटीकरण

- वहरणय-गभि सववणिम गभि क सोदवभित करता ह

- जब बराहमण ो की सहायता स यह अनषठान वकया

जाता था त यह माना जाता था वक बवल दन िाल

का कषवतरय क रप म पनजिनम ह गा

Q33) उततर (d)

सपषटीकरण

- कदमई भवम राजसव पर कर क सोदवभित करता

- गवावलयर परशससत म नागभि क दवारा वकय गए

श िण का िणिन वकया गया ह |

- नागभि एक परवतहार राजा था

Q34) उततर (b)

सपषटीकरण

- राजतरो वगनी 12िी ो शताबदी म कलहन क दवारा

रवचत एक सोसकत पसतक (िकसट) ह

- यह परारसमभक भारत की ऐवतहावसक इवतितत थी

- तकि सोगत रप स इस अपन परकार की सिोततम

और सिािविक विशवसनीय कवत माना जाता ह

- यह कशमीर कषतर क पराचीनतम समय स लकर

उसकी रचना की तारीख तक क समपणि इवतहास

का आचछादन करती ह

Q35) उततर (c)

सपषटीकरण

- गााि की आम सभा क ldquoउरrdquo कहा जाता था

- ldquoउरrdquo म गााि क सभी कर दन िाल वनिासी

शावमल ह त थ

Q36) उततर (a)

सपषटीकरण

- वदलली सलतनत म ldquoतारीखrdquo इवतहास लखन का

एक रप था

- ldquoतािरीखrdquo क लखक विदवान परि ह त थ वजनम

सवचि परशासक इतयावद शावमल थ

Q37) उततर (a)

सपषटीकरण

- अलाउददीन सखलजी अपन सवनक ो क ितन का

भगतान नकद म करता था न वक इकता क रप

- सवनक अपना सामान वदलली म वयापाररय ो स

खरीदत थ अतः इस बात का भय था वक वयापारी

कही ो िसतओो का मलय न बिा द

- इसकी र कथाम क वलए अलाउददीन सखलजी न

वदलली म कीमत ो क वनयसित वकया

FC19H1003 34

- अविकारीगण धयानपििक मलय ो का सिकषण करत

थ तथा ज वयापारी वनिािररत मलय पर माल नही ो

बचत थ उनक दसणडत वकया जाता था

Q38) उततर (d)

सपषटीकरण

- वदलली सििपरथम त मर राजपत ो क अिीन उनक

सामराजय की राजिानी बनी थी

- 12िी ो शताबदी क मधय म अजमर क चौहान ो

(वजनह चाहमान ो क नाम स भी जाना जाता ह) न

त मर राजपत ो क परावजत वकया था

- त मर ो और चौहान ो क अिीन वदलली एक

महतवपणि िावणसजयक क दर बन गया था

- कई जन वयापारी यहाा रहन लग थ और उनह ोन

कई मोवदर भी बनिाए

- यहाा पर मवदरत वसक वजनह ldquoदहलीिालrdquo क नाम

स जाना जाता था वयापक रप स परचलन म थ

Q39) उततर (c)

सपषटीकरण

- म ठ की मसिद का वनमािण वसको दर ल दी क

राजयकाल म उसक मिी क दवारा करिाया गया

था

- बगमपरी मसिद का वनमािण महममद तगलक क

शासनकाल म हआ था

- यह मसिद विशव का पणयथथान (The

Sanctuary of the World) और वदलली म महममद

तगलक की नई राजिानी जहाोपनाह की मखय

मसिद थी

- कववत- अल - इसलाम मसिद का विसतार

इलतसिश और अलाउददीन सखलजी न वकया था

- मीनार का वनमािण तीन सलतान ो कतबददीन ऐबक

इलतसिश और वफर ज शाह तगलक क दवारा

करिाया गया था

Q40) उततर (c)

सपषटीकरण

- मगल ो क अिीन मनसबदार शबद उस वयसकत क

वलए सोदवभित वकया जाता था वजसक पास मनसब

(अथाित पद) ह ता था

- उस अपना ितन राजसव कायो वजनह जागीर कहत

थ क रप म परापत ह ता था

Q41) उततर (b)

सपषटीकरण

- ldquoभारत छ ड आोद लनrdquo वबरविश शासन क

सखलाफ ल ग ो का एक सवाभाविक विदर ह था

- असखल भारतीय काोगरस सवमवत न 8 अगसत 1942

क बमबई म एक बठक का आय जन वकया था

- इस बठक म परवसद सोकलप ldquoभारत छ ड rdquo क

पाररत वकया गया और इस उददशय क परापत करन

क वलए गाोिी क नततव म एक अवहोसक जन सोघिि

आोद लन की शरआत का परसताि वदया गया

- लवकन अगल ही वदन गाोिी और काोगरस क अनय

परमख नताओो क वगरफतार कर वलया गया

- काोगरस क एक बार वफर अिि घ वित वकया गया

था

Q42) उततर (c)

सपषटीकरण

- साइमन कमीशन यनाइविड वको गडम क सात

साोसद ो का एक समह था

- इस वबरविश भारत क वलए सोििावनक सिार ो का

सझाि दन क वलए गवठत वकया गया था

- इस आय ग म िररषठ वबरविश राजनता सर जॉन

साइमन क नततव म किल वबरविश सदसय ही

शावमल थ

- इसवलए भारत क ल ग ो न साइमन कमीशन क

आगमन क विरद आोद लन वकया था

Q43) उततर (a)

सपषटीकरण

bull दादा भाई नौर जी भारत म वबरविश शासन क

आवथिक पररणाम ो क बार म अपनी विर िी

(परवतकल) राय क वलए जान जात थ

FC19H1003 35

bull अपन कई लख ो और भािण ो म विशि रप स

ldquoपाििी एो ड अन-वबरविश रल इन इसणडया

(Poverty and Un-British Rule in India) म

नौर जी न यह तकि वदया वक भारत पर अतयविक

कर लगाया गया था और इसकी सोपवतत इोगलड की

ओर परिावहत की जा रही थी

bull उनह ोन पराचीन भारतीय गरोथ ो की वयाखया करन

का और भारतीय ो क आिविशवास क बहाल

करन पर कायि नही ो वकया था

उनह ोन वकसी और बात स पहल सभी सामावजक

बराइय ो क उनमलन की आिशयकता पर भी बल

नही ो वदया था

Q44) उततर (c)

सपषटीकरण

bull अगसत 1932 म वबरविश परिानमोतरी मकड नालड न

अपन साोपरदावयक परसकार (The Communal

Award) की घ िणा की थी

bull यह भारत क कई साोपरदावयक वहत ो क बीच विवभनन

सोघिो क हल करन क वलए वबरिन का एकतरफा

परयास था

bull यह परसकार (Award) बाद म 1935 क

अविवनयम (The Act of 1935) म शावमल वकया

गया था

bull इस साोपरदावयक परसकार न मससलम ो क वलए

आरवकषत एक अलग वनिािचक मणडल फॉमिल का

विसतार अनय अलपसोखयक ो क वलए वकया था

वजसम वसख ो भारतीय ईसाइय ो आोगल-भारतीय

समदाय यर पीय समदाय तथा विवशषट कषतरीय

समह ो क शावमल वकया गया था

bull गाोिी न इस परसताि क भारतीय समाज क

विभावजत करन क वलए एक घवणत वबरविश

सावजश क रप म दखा और उसक सखलाफ

आमरण अनशन वकया

Q45) उततर (b)

सपषटीकरण

मौजदा आयात और वनयाित क अवतररक़त

औपवनिवशक भारत क वनमनवलसखत खचो क

वलए एक विशिवनवशचत िन रावश भी दनी पडती

थी

(i) परशासन क वयय

(ii) सना क रख-रखाि क वयय

(iii) यद क वयय

(iv) सिावनितत अविकाररय ो की पशन तथा

(v) वबरिन दवारा अपनी उपवनिश बसती

(कॉल नी) क रख-रखाि क वयय

इनह गह शलक (Home Charges) क रप म

जाना जाता था और लगभग परी तरह स भारत क

दवारा इनका भगतान वकया जाता था

bull गह शलक म वनमनवलसखत घिक शावमल थ

(i) भारतीय ऋण पर दय बयाज

(ii) ईसट इोवडया को पनी क शयरिारक ो क

लाभाोश

(iii) लोदन म भारत कायािलय चलान क वलए िन

(iv) भारत म वनयकत वबरविश कवमिय ो क ितन

और पशन का भगतान करन क वलए िन

(v) रलि पर बयाज

(vi) नागररक और सनय शलक

(vii) इोगलड म सट र (सामगरी) की खरीद

Q46) उततर (b)

सपषटीकरण

bull भारतीय राषटर ीय काोगरस का लाहौर सतर 1929 म

जिाहरलाल नहर की अधयकषता म आय वजत

वकया गया था

bull इस सतर म भारतीय राषटर ीय आोद लन स समबसित

कई महतवपणि पररणाम सामन आय थ

(i) सििपरथम इस सतर म काोगरस क अधयकष पद

पर जिाहरलाल नहर क चना गया था ज

काोगरस म िामपोवथय ो की बिती हई ताकत

का सपषट सोकत था

(ii) दसरा इस सतर म पहली बार काोगरस न पणि

सवतोतरता की माोग क उठाया था

इस परकार की माोग काोगरस मोच स पहल कभी भी

नही ो उठाई गई थी

Q47) उततर (b)

सपषटीकरण

FC19H1003 36

bull इस ररप िि न वकसी भी समदाय क वलए पथक

वनिािचक मोडल अथिा अलपसोखयक ो क वलए

भाराोश की वसफाररश नही ो की थी

bull तथावप इस ररप िि न उन पराोत ो म अलपसोखयक

सीि ो क आरकषण की अनमवत दी थी जहाा पर कम

स कम दस परवतशत अलपसोखयक ह

bull लवकन यह समदाय क आकार क अनपात म ह ना

चावहए था

bull इस ररप िि म भारत क वलए पणि सवतोतरता क

वलए क ई पराििान नही ो था

Q48) उततर (c)

सपषटीकरण

bull आरो वभक िवदक आयो का िमि मखय रप स

परकवत की पजा और यजञ था

bull परारो वभक आयि िमि परकवत की पजा क समान था

bull िासति म उनक चार ो ओर की शसकतयाा वजनह न

त ि वनयोवतरत कर सकत थ और न ही समझ पाए

थ उनह वदवयता क साथ वनिवशत वकया गया तथा

उनह मादा या नर दिीदिताओो क रप म

परतीकतव वकया गया था

bull उनह ोन कछ यजञ ो का भी वनषपादन वकया था

Q49) उततर (b)

सपषटीकरण

bull सडक और नदी-मागि (जल-मागि) डकती स

सरवकषत नही ो थ

bull उललखनीय ह वक हिििििन क शासनकाल क

दौरान यआन चिाोग (हयएन साोग) का सारा

सामान लि वलया गया था

Q50) उततर (c)

सपषटीकरण

परशन म वदए गए द न ो कथन सही ह

Q51) उततर (b)

सपषटीकरण

bull परोदर दास एक सोत और भगिान कषण क एक

महान भकत थ

bull परोदर दास क कनाििक सोगीत क वपतामह क

रप म जाना जाता ह

bull यदयवप उनक जनम-थथान क बार म काफी

अिकल लगाई जाती रही ह

bull तथावप अब कननड विशवविदयालय हमपी क दवारा

गवठत एक विशिजञ सवमवत इस वनषकिि पर पहोची

ह वक उनका जनम थथान सोभितया कनाििक का

एक छ िा-सा गााि कषमपरा (वशिम गगा वजला)

था

Q52) उततर (c)

सपषटीकरण

bull शरी तयागराज शरी शयाम शासतरी और शरी मथसवामी

दीवकषतर क कनाििक सोगीत की वतरमवति माना

जाता ह

bull उनक कारण ही 18िी ो-19िी ो शताबदी म कनाििक

सोगीत का सववणिम यग आया था

Q53) उततर (d)

सपषटीकरण

bull अभी हाल ही म लौह यगीन-महापािावणक काल

का 2000 ििि पराना एक दलिभ सारक फगस

(Sarcophagus) (पतथर का ताबत) क ललम क

वियर गाोि (क वयलडी क पास वजला क वझक ड

करल राजय) की एक रॉक-कि गफा स ख जा गया

bull यह ताबत वजसम हविय ो क िकड थ खदाई क

दौरान वमला

bull अभी तक इस परकार की दलिभ ख ज करल क

मातर द ही थथान ो स हई ह

bull य द न ो सारक फगी (Sarcophagi) (पतथर क

ताबत) चियर और अथ ली (वजला क वझक ड) क

महापािाण थथल ो स वमल ह

Q54) उततर (a)

सपषटीकरण

FC19H1003 37

दवकषण भारत म महापािाण सोसकवत एक पणि

विकवसत लौह यगीन सोसकवत थी

Q55) उततर (d)

सपषटीकरण

bull च ल पाणडय और करलपतर (चर) इन तीन ो का

उललख अश क क अवभलख ो म वकया गया ह

bull सोभितः य भौवतक सोसकवत क उततर

महापािावणक चरण म थ

Q56) उततर (d)

सपषटीकरण

bull भीमा-क रगाोि की लडाई ततीय आोगल-मराठा

यद का वहससा थी

Q57) उततर (b)

सपषटीकरण

bull राजकमार शकल न गाोिीजी क चोपारण आन तथा

वतनकवथया परणाली स जडी समसया की जाोच क

वलए रारी करन क वलए दश भर म उनका

अनसरण वकया था

bull बज वकश र राजदर परसाद महादि दसाई और

नरहरी पाररख चोपारण सतयागरह क दौरान गाोिी

जी क सहय गी थ

Q58) उततर (b)

सपषटीकरण

bull बराहमण ो और बौद मठिाररय ो क कर-मकत गााि

अनदान म दन की परथा सतिाहन ो न आरमभ की

थी

Q59) उततर (c)

सपषटीकरण

इस कायिकरम क उददशय वनमनानसार ह

(i) बवनयादी पयििन आिाररक सोरचना का विकास

करना

(ii) चयवनत (पहचान वकय गए) कषतर ो म आजीविका क

सजन क वलए दश क साोसकवतक और विरासत

मलय ो क बिािा दना

(iii) विरासत समारक थथल ो पर विशव सतरीय आिाररक

सोरचना विकवसत करक एक सतत तरीक स

पयििक आकििण म िसद करना

(iv) थथानीय समदाय ो की सवकरय भागीदारी क माधयम

स र रगार ो का सजन करना

(v) र रगार उतपादन और आवथिक विकास क वलए

पयििन कषमता का उन पर परभाि का उपय ग

करना तथा

(vi) िारणीय पयििन आिाररक सोरचना का विकास

करना और उसका उवचत सोचालन तथा

रखरखाि सवनवशचत करना

Q60) उततर (b)

सपषटीकरण

bull यह वनकाय ििि 1987 म अससततव म आया था

bull यह एक राषटर ीय सतर का शीिि सोगठन ह ज भारत

सरकार क जनजातीय मामल ो क मोतरालय क

परशासवनक वनयोतरण क अिीन काम कर रहा ह

bull इसका पोजीकत और परिान कायािलय नई वदलली

म सथथत ह

Q61) उततर (c)

सपषटीकरण

bull परमचोद क उपनयास ो म परमाशरम रोगभवम गबन

कमिभवम और ग दान शावमल ह

bull ग रा रिी ोदरनाथ िग र क दवारा रवचत उपनयास ह

bull अभी हाल ही म मोशी परमचोद की 138िी ो जयोती दश

भर म मनाई गई थी

Q62) उततर (b)

सपषटीकरण

bull ldquoवगदाrdquo पोजाब (भारत) एिो पावकसतान की

मवहलाओो क दवारा तयौहार क समय और फसल

की बिाई तथा किाई क अिसर पर वकया जान

िाला एक पारोपररक दहाती नतय ह

FC19H1003 38

bull इस नतय क माधयम स पोजाबी मवहलाऐो अपनी

परसननता परकि करती ह तथा वगदा क परदशिन क

माधयम स परि िचिसव िाल समाज म मवहलाओो

की दबी हई भािनाओो क परकि करती ह

bull चोवक इस नतय का परि ो क साथ क ई सोबोि नही ो

ह अतः किल मवहलाऐो ही इसम भाग ल सकती

bull हर साल तीज समार ह क दौरान पोजाब म वगदा

नतय वकया जाता ह

तीज भारत क कछ भाग ो म मवहलाओो क दवारा

मनाया जान िाल कई तयौहार ो क वलए एक

वयापक नाम ह

Q63) उततर (a)

सपषटीकरण

- मजम-उल-बहरीन या द समदर ो का सोगम

नामक उललखनीय रचना दारा वशक ह क दवारा

वलखी थी

- भारत क उपराषटर पवत शरी एम िकया नायड न कहा

ह वक राजकमार दारा वशक ह की रचनाएा शाोवत

और सदभाि क बिािा दन क वलए एक तारा सर त

क रप म सामन आ सकती ो ह

- उपराषटर पवत गत ििो क भला वदए गए राजकमार

दारा वशक ह क परदवशित परचवलत करन हत

आय वजत एक परदशिनी का दौरा करन क बाद एक

सभा क सोब वित कर रह थ

- इस परदशिनी का आय जन फर क इस गौवियर

(Francois Gautier) क दवारा lsquoइोवदरा गाोिी नशनल

सिर फॉर द आििसrsquo (The Indira Gandhi

National Centre for the Arts) नई वदलली म

वकया गया था

Q64) उततर (c)

सपषटीकरण

- ग मतशवर परवतमा जन भगिान बाहबली क

समवपित ह

- यह एक एक-चटटानी पतथर की मवति ह

- राषटर पवत राम नाथ क विोद न शरिणबलग ला

(कनाििक) म आय वजत वकय जान िाल भवय

अवभिक समार ह महामसतकावभिक का

उदघािन वकया था

- यह समार ह 12 ििो म एक बार ह ता ह

Q65) उततर (c)

सपषटीकरण

bull पराची घािी पराची नदी क चार ो ओर फली हई थी

bull पराची घािी िीर-िीर विलपत ह गई थी

bull पराची नदी भिनशवर स वनकलती ह

bull यह महानदी की एक सहायक नदी ह और यह

परी खदाि किक तथा जगतवसोहपर वजल ो स

ह कर बहती ह

bull इस नदी क पर कषतर क पराची घािी कहा जाता ह

bull यह नदी बोगाल की खाडी म वगरती ह

परातासतवक साकषय स पता चलता ह वक पराची घािी

सभयता हडपपा और म हनज दाड द न ो की

पिििती ह

Q66) उततर (d)

सपषटीकरण

य समारक छतरपर वजल (मधय परदश) म विोधयाचल

पिित शरोखला म सथथत ह

Q67) उततर (a)

सपषटीकरण

bull थॉिस ऑन पावकसतान नामक पसतक डॉ बी

आर अमबडकर न वलखी थी

bull डॉ बी आर अमबडकर की जयोती क अिसर पर

भारत क राषटर पवत न भारत की इस महान हसती

क शरदाोजवल अवपित की थी

bull डॉ बी आर अमबडकर न 1924 म वडपरथड

कलावसर इोसटीटयि (दवलत िगि सोथथान -

बवहषकत वहतकाररणी सभा) और 1927 म समाज

समता सोघ की थथापना की थी

bull अमबडकर का धयान वशकषा कषतर की ओर भी था

bull उनह ोन वशकषा क वनमन िगो म फलान क वलए

पीपलस एजकशन स साइिी (The Peoples

Education Society) क नाम स महाविदयालय ो क

नििकि और छातरािास ो की थथापना की थी

FC19H1003 39

Q68) उततर (b)

सपषटीकरण

bull महरगि भारतीय उपमहादवीप म एक परवसद

निपािाण बसती ह ज बलवचसतान पराोत

पावकसतान म सथथत ह

bull दचपलली (आोधर परदश) क पास नागलर नदी क

पिी ति ो पर चना पतथर क बलॉक क विशाल

विसतार म एक पिि-ऐवतहावसक रॉक आिि थथल की

ख ज की गई ह

bull इसन 1500-2000 ईसा पिि क दौरान गोिर (आोधर

परदश) म विकवसत निपािाण सभयता पर परकाश

डाला ह

Q69) उततर (c)

सपषटीकरण

bull 12िी ो सदी और 13िी ो सदी म काकाविय िोश का

उदय हआ था

bull ि पहल कलयाण क पवशचमी चालकय ो क सामोत थ

bull परारोभ म उनह ोन िारोगल (तलोगाना) क पास एक

छ ि स कषतर पर शासन वकया था

bull उनह ोन ldquoनायक वयिथथाrdquo की शरआत की थी

वजस बाद म विजयनगर क राय शासक ो न

अपनाया और विकवसत वकया था

Q70) उततर (a)

सपषटीकरण

bull गाोिीजी क अनशन स वमल मावलक ो पर दबाि

पडा था ज अोततः शरवमक ो क ितन म 35 परवतशत

की िसद करन क वलए सहमत हए थ

bull गगल (Google) न अनसया साराभाई वजनह ोन

भारत क शरवमक आोद लन म एक अगरणी भवमका

वनभाई थी की 132िी ो जयोती डडल (Doodle) का

वनमािण करक मनाई

Q71) उततर (d)

सपषटीकरण

भारत स यनसक की मानिता की अमति साोसकवतक

विरासत की परवतवनवि सची म वनमनवलसखत शावमल ह

bull कवडयटटम करल का सोसकत रोगमोच

bull मवडयिि करल का अनषठान रोगमोच और नतय

नाविका

bull िवदक मि जाप की परोपरा

bull राजथथान क कालबवलया ल क गीत और नतय

bull रामलीला रामायण का पारोपररक परदशिन

bull सोकीतिन मवणपर का अनषठान गायन ढ ल िादन

और नतय

bull रममन भारत क गििाल वहमालय का िावमिक

तयौहार और अनषठान रोगमोच

bull जाोदीयाला गर पोजाब क ठठर ो की पीतल और

ताोब क वशलप स वनवमित बतिन ो की पारोपररक कला

bull छाऊ नतय पिी भारतीय राजय ो म जनमी शासतरीय

भारतीय नतय कला

bull लददाख का बौद मि जाप िर ाोस-वहमालयी लददाख

कषतर तथा जमम-कशमीर म पवितर बौद गरोथ ो का पाठ

bull य ग

bull नौर र

bull को भ मला

Q72) उततर (b)

सपषटीकरण

bull भारत क राषटर पवत शरी राम नाथ क विोद न

वकसामा नागालड म हॉनिवबल मह रति और

राजय गठन वदिस समार ह का उदघािन वकया

था

bull हॉनिवबल मह रति का नाम भारतीय हॉनिवबल क

नाम पर पडा ह ज एक विशाल और रोगीन जोगली

पकषी ह

bull यह पकषी नागालड राजय की अविकतर जनजावतय ो

की ल ककथाओो म उसललसखत ह

bull नागालड की परमख मानयता परापत जनजावतयाा ह

अोगामी आओ चखसोग चाोग ककी रगमा और

रवलोग आवद

bull ओोग जारिा और ससिनलीस अोडमान-वनक बार

दवीप समह की जनजावतयाा ह

FC19H1003 40

Q73) उततर (c)

सपषटीकरण

bull दकन म राषटर कि शासन दसिी ो सदी क अोत तक

लगभग 200 ििो तक रहा था

bull राषटर कि शासक अपन िावमिक विचार ो म सवहषण

bull उनह ोन न किल शि िमि और िषणि िमि बसलक

जन िमि क भी सोरकषण वदया था

bull एल रा म वशि क परवसद रॉक कि मोवदर का

वनमािण नौिी ो सदी म राषटर कि राजा कषण परथम न

करिाया था

bull उसका उततराविकारी अम घििि जन था लवकन

उसन अनय िमो क भी सोरकषण परदान वकया था

bull राषटर कि ो न मसलमान वयापाररय ो क बसन की

अनमवत दी थी

bull उनह न अपन अविराजय ो म इसलाम क उपदश दन

की भी अनमवत दी थी

bull अभी हाल ही म पाोडिलागटटा (तलोगाना) क

परागवतहावसक चटटान वचतर ो क कषरण की बिती हई

घिनाएा एक गोभीर वचोता का वििय ह

bull यह परागवतहावसक चटटान क नकसान पहाचा

सकता ह

bull पाोडिलागटटा वनमनवलसखत क वलए जाना जाता ह

- 10000 ईसा पिि स 8000 ईसा पिि क वचवतरत

चटटानी आशरय ो क वलए

- राषटर कि काल क एक 8 िी ो सदी क

वशलालख क वलए और

- 12िी ो सदी क काकविय सामराजय क वभवतत

वचतर ो क वलए

Q74) उततर (b)

सपषटीकरण

bull 1828 म राजा राम म हन रॉय न एक नय िावमिक

समाज बरहम सभा की थथापना की थी वजस बाद

म बरहम समाज क नाम स जाना गया था

bull दिदरनाथ िग र न ततवब विनी सभा की अधयकषता

की थी ज आधयासिक सतय की ख ज म सोलि

थी

bull इसका उददशय वहोद िमि क शद करन का और

एकशवरिाद (एक ईशवर म आथथा) का परचार करना

था

bull नय समाज की थथापना क आिार थ कारण

(तकि ) क द सतमभ तथा िद और उपवनिद

bull अभी हाल ही म सािारण बरहम समाज का कछ

काननी मदद ो क लकर पवशचम बोगाल सरकार क

साथ काननी वििाद चल रहा ह

Q75) उततर (c)

सपषटीकरण

bull भारत म वचशती वसलवसल की थथापना खवाजा

म इनददीन वचशती क दवारा की गयी थी

bull ि 1192 ईसवी क आसपास भारत आय थ

bull वचशतीय ो क बारहिी ो शताबदी क उततरािि म भारत

म आन िाल सफीय ो क समह ो म सबस

परभािशाली माना जाता ह

bull उनह ोन थथानीय िातािरण क साथ सफलतापििक

अनकलन वकया और उनह ोन भारतीय भसकत

परोपराओो क कई पहलओो क अपनाया

bull अजमर म सफी अपरकि खवाजा म इनददीन वचशती

की ऐवतहावसक दरगाह क एक नया रप दन की

तयारी की जा रही ह

bull इस 13िी ो शताबदी की दरगाह क ldquoसवचछ

आइकॉवनक थथल ोrdquo (Swacch Iconic Places) म

शावमल वकया गया ह ज परवतवषठत विरासत

आधयासिक और साोसकवतक थथान ो पर क वदरत

य जना ह

FC19H1003 41

ANSWERS amp EXPLANATION OF

NCERT History Class VI-X + Current Affairs

(FC19E1003)

Q1) Answer c

Explanation

Rigveda consists of more than a

thousand hymns dedicated to gods and

goddesses These hymns were

composed by sages and learnt by men

however a few were composed by

women like Apala Ghosa Lopamudra

Maitreyi and Gargi

Rigveda consists of many hymns in the

form of dialogues We get an example of

a dialogue between a sage named

Vishwamitra and two rivers (Beas and

Sutlej) that were worshipped as

goddesses This suggests that he

belonged to the Vedic period

Q2) Answer b

Explanation

Traces of ash have been found from

Kurnool Caves suggesting that people

were familiar with the use of fire

It is situated in Andhra Pradesh

Q3) Answer c

Explanation

Burzahom is a prehistoric site in

present day Kashmir where people built

pit houses which were dug into the

ground with steps leading into them

These may have provided shelter in cold

weather

Q4) Answer c

Explanation

Epigraphy is defined as the study of

inscriptions

Manuscriptology is the study of history

and literature through the use of hand

written documents

Palaeography refers to the study of

ancient writing systems and the

deciphering and dating of historical

manuscripts

Numismatics refers to the study of

coins

Q5) Answer a

Explanation

Charaka Samhita was written by

Charaka and is an important book on

Ayurveda and medicine

He was a practitioner of the traditional

system of Indian medicine known as

Ayurveda

Charaka is thought to have flourished

sometime between the 2nd century BCE

and the 2nd century CE

Q6) Answer b

Explanation

Bhaga refers to the tax on crops which

was fixed at 16th of the production

Kammakaras is the term used for the

landless agricultural labour class

Ashvamedha also known as horse

sacrifice is a ritual where a horse is let

loose to wander freely and it was

guarded by the rajarsquos men

Q7) Answer (d)

Explanation

In the Rigvedic period horses were

yoked to chariots that were used in

battles fought to capture land cattle

etc This suggests that the use of horse

chariots began much before the period

of Mahajanapadas

The battles were fought in the Rigvedic

period for cattlersquos lands water an even

to capture people Most men took part

in these wars however there was no

regular army but there were assemblies

where people met and discussed

matters of war Regular armies became

a feature in the Mjahajanapada period

including vast armies of foot soldiers

chariots and elephants

RAUSIAS-FC19E1003 42

Q8) Answer (a)

Explanation

Buddha belonged to the Sakya clan and

passed away at Kusinara

Buddha taught in Prakrit which was the

common language of people

Q9) Answer c

Explanation

There were six schools of philosophy in

ancient India These are known as

Vaishesika Nyaya Samkhya Yoga

Purva Mimansa and Vedanata or Uttara

Mimansa They were founded by sages

Kanada Gautama Kapila Patanjali

Jamini and Vyasa respectively

Q10) Answer b

Explanation

The teachings of Mahavira were

compiled at Valabhi in 6th century AD

Q11) Answer (c)

Explanation

Chanakya is traditionally identified as

Kautilya or Vishnugupta who authored

the ancient Indian political treatise the

Arthashastra

Q12) Answer d

The national emblem of India is an

adaptation of the Lion Capital atop the

Ashoka Pillar of Sarnath Uttar Pradesh

and is combined with the National

Motto Satyameva Jayate

The Rampurva Bull gets the name from

the site of its discovery Rampurva in

Bihar

It is noted for its delicately sculpted

model demonstrating superior

representation of soft flesh sensitive

nostrils alert ears and strong legs It is

a mixture of Indian and Persian

elements

Sankissa is situated in Uttar Pradesh

India

Q13) Ans(a)

Kunwar Singh was a notable leader during the Revolt of 1857 He belonged

to a royal house of Jagdispur Bihar

Q14) Answer b

Explanation

The term Vellalar was used for large

landowners

Q15) Answer c

Explanation

Arikamedu was a coastal settlement

where ships unloaded goods from

distant lands Finds here include a

massive brick warehouse pottery

including amphorae and Arretine ware

Roman lamps glassware and gems have

also been found at the site

Q16) Answer a

Explanation

Muvendar is a Tamil word mentioned in

Sangam poems meaning three chiefs

used for the heads of three ruling

families the Cholas Cheras and

Pandyas

Q17) Ans (c)

Several tribal or kin-based assemblies

such as the Sabha Vidatha and gana

are mentioned in the Rig-veda The

Sabha and the samiti mattered a great

deal in early Vedic times so much so

that the chiefs or the kings showed an

eagerness to win their support

Q18) Ans (a)

Jainism recognised the existence of the

gods but placed them lower than the

jina and did not condemn the varna

system as Buddhism did

Q19) Answer (d)

Explanation

Cholas and Pandyas had developed

powerful coastal cities The most

important city of Cholas was Puhar or

Kaveripattinam and Madurai was the

capital of Pandyas

Q20) Answer b

Explanation

Buddhacharita is the biography of

Buddha and was written by

RAUSIAS-FC19E1003 43

Ashvaghosha

Q21) Answer (a)

Explanation

Tamil poet Appar was a Shiva devotee

So he was a Nayanar saint

Q22) Answer d

Explanation

Samudragupta was a prominent Gupta

ruler whose coins depict him playing a

veena indicating his love for music We

get important historic information from

his Allahabad Prashasti which was

composed by his court poet Harisena

Q23) Answer (b)

Explanation

Vikrama Samvat was founded by

Chandragupta II in the 58 BC as a

mark of victory over the Shakas and

assumed the title of Vikramaditya

Banabhatta wrote Harshavardhanarsquos

biography the Harshacharita in

Sanskrit

Q24) Answer c

Explanation

Sandhi-vigrahika was the minister of

war and peace

Sarthavaha was the leader of the

merchant caravans

Q25) Answer a

Explanation

Xuan Zang (Hsuan-tsang) was a

Chinese traveller who came during the

reign of Harshavardhana

In the decade that began in 630 AD

Xuan Zang came to India through

Kashmir after visiting Central Asia Iran

and Afghanistan

He travelled from north to east and lived

in Bihar for a couple of years

At Nalanda University Xuan Zang

interacted with students and scholars

mastered local languages and

discovered Buddhist stupas

Q26) Answer c

Explanation

Pradakshina patha is a circular path

laid around a stupa in Buddhist

architecture While the rest are a part of

temple architecture

Q27) Answer d

Explanation

All the above-mentioned temples have

an elaborate use of bricks (baked

bricks) along with stone

Q28) Ans (c)

Muhammad Quli Qutab was the Sultan

of Golconda He was a contemporary of

Akbar was very fond of literature and

architecture

The Sultan was a great poet and he

wrote in Dakhini Urdu Persian and

Telgu and has left an extensive diwan or

collection

Recently the Archaeological Survey of

India (ASI) will be using Ground

Penetrating Radar (GPR) to map the

contours of the area around the Bagh-e-

Naya Qila excavated garden inside the

Golconda Fort in Telangana

Q29) Answer a

Explanation

Silappadikaram is a famous Tamil epic

which was written by Ilango around

1800 years ago It is a story of a

merchant named Kovalan who fell in

love with a courtesan named Madhavi

Manimekalai tells the story of the

daughter of Kovalan and Madhavi

Q30) Answer (a)

Explanation

Charaka is the author of Charaka

Samhita which is an important work of

Ayurveda and medicines

Brahmaguptarsquos fame rests mostly on his

Brahma-sphuta-siddhanta which was

an astronomical work It was translated

into Arabic in Baghdad and had a major

impact on Islamic mathematics and

astronomy

Late in his life Brahmagupta wrote

Khandakhadyaka which was an

RAUSIAS-FC19E1003 44

astronomical handbook that employed

Aryabhatarsquos system of starting each day

at midnight

Q31) Answer (c)

Explanation

Amir Khusrau was a famous sufi

musician poet and scholar In 1318 he

noted that there was different language

in every region of this land (Hindustan)

Lahori Kashmiri Dvarsamudri (in

Southern Karnataka) Telangana (in

Andhra Pradesh) Gujari (in Gujarat)

Marsquobari (in Tamil Nadu) Awadhi (in

eastern Uttar Pradesh) and Hindawai (in

the area around in Delhi) etc He went

to explain that Sanskrit did not belong

to any region and that only brahmans

knew it

Q32) Answer c

Explanation

Hiranyagarbha refers to the golden

womb When this ritual was performed

with the help of Brahmanas it was

thought to lead to the rebirth of the

sacrificer as a Khastriya

Q33) Answer d

Explanation

Kadamai refers to a tax on land

revenue

Gwalior Prashasti describes the exploits

of Nagabhata who was a Pratihara king

Q34) Answer b

Explanation

Rajatarangini is a Sanskrit text written

by Kalhana in the 12th century

It was historical chronicle of early India

It is justifiably considered to be the best

and most authentic work of its kind

It covers the entire span of history in

the Kashmir region from the earliest

times to the date of its composition

Q35) Answer c

Explanation

ldquoUrrdquo was the general assembly of the

village ldquoUrrdquo consisted of all the

taxpaying residents of an ordinary

village

Q36) Answer (a)

Explanation

Tarikh was a form of history writing in

the Delhi Sultanate The authors of

tawarikhs were learned men which

included secretaries administrators etc

Q37 Answer (a)

Explanation

Alauddin chose to pay his soldiers salaries in cash rather than iqtas The soldiers would buy their supplies from merchants in Delhi and it was thus feared that merchants would raise their prices To stop this Alauddin controlled the prices of goods in Delhi Prices were carefully surveyed by officers and merchants who did not sell at the prescribed rates were punished

Q38) Answer (d)

Explanation

Delhi first became the capital of a

kingdom under the Tomara Rajputs

who were defeated in the middle of the

twelfth century by the Chauhans (also

referred to as Chahamanas) of Ajmer

It was under the Tomaras and

Chauhans that Delhi became an

important commercial centre Many rich

Jaina merchants lived in the city and

constructed several temples Coins

minted here called dehliwal had a wide

circulation

Q39) Answer (c)

Explanation

Moth ki Masjid was built in the reign of

Sikandar Lodi by his minister

Begumpuri mosque built in the reign of

Muhammad Tughluq was the main

mosque of Jahanpanah the ldquoSanctuary

of the Worldrdquo and his new capital in

Delhi

Quwwat al ndash Islam mosque was

enlarged by Iltutmish and Alauddin

Khalji The minar was built by three

Sultansndash Qutbuddin Aybak Iltutmish

and Firuz Shah Tughluq

RAUSIAS-FC19E1003 45

Q40) Answer (c)

Explanation

Under the Mughals mansabdar was

referred to an individual who held a

mansab ie rank and he received his

salary as revenue assignments called

jagirs

Q41) Ans (b)

The Quit India Movement was a

spontaneous revolt of people against

British rule

The All India Congress Committee met

at Bombay on 8 August 1942 It passed

the famous resolution Quit India and

proposed the starting of a non-violent

mass struggle under Gandhis

leadership to achieve this aim But on

the very next day Gandhi and other

eminent leaders of the Congress were

arrested The Congress was once again

declared illegal

Q42) Ans (c)

The Simon Commission refers to a

group of seven MPs from the United

Kingdom constituted to suggest

constitutional reforms for British India

The Commission consisted of only

British members headed by one of the

senior British politicians Sir John

Simon

So the people of India agitated against

the arrival of Simon Commission

Q43) Ans (a)

He was widely known for his

unfavourable opinion of the economic

consequences of the British rule in

India

In his many writings and speeches and

especially in Poverty and Un-British

Rule in India Naoroji argued that India

was too highly taxed and that its wealth

was being drained away to England

He did not interpret the ancient Indian

texts and restored the self-confidence of

Indians And also he did not stress the

need for eradication of all the social

evils before anything else

Q44) Ans (c)

In August 1932 Prime Minister

MacDonald announced his Communal

Award Great Britainrsquos unilateral

attempt to resolve the various conflicts

among Indiarsquos many communal

interests

The award which was later

incorporated into the act of 1935

expanded the separate-electorate

formula reserved for Muslims to other

minorities including Sikhs Indian

Christians Anglo-Indians Europeans

distinct regional groups Gandhi

undertook a ldquofast unto deathrdquo against

that offer which he viewed as a

nefarious British plot to divide the

Indian society

Q45) Ans (b)

In British India apart from existing

imports and exports there was also a

particular amount of money which

colonial India contributed towards

administration maintenance of the

army war expenses pensions to retired

officers and other expenses accrued by

Britain towards maintenance of her

colony These were known as Home

charges and were paid for almost

entirely by India

The Home charges was made of

following components-

- Interest payable on Indian debt

- Dividend to shareholders of East

India Company

- Funds used to support the India

Office in London

- Funds used to pay salaries and

pensions of British personnel

engaged in India

- Interest on the railways

- Civil and military charges

- Store purchases in England

Q46) Ans (b)

The Lahore session of the Indian

National Congress was held in 1929

under the Presidentship of Jawaharlal

Nehru

The Lahore session of the Indian

National Congress witnessed significant

RAUSIAS-FC19E1003 46

developments in the Indian national

movement

- First the election of Jawaharlal

Nehru to the post of Presidentship of

the Congress was a clear indication

of the growing strength of the

Leftists in the Congress

- Secondly it was in this session that

the Congress for the first time raised

the demand for complete

independence Such demand was

not raised from the Congress

platform earlier

Q47) Ans (b)

It did not provide for separate

electorates for any community or

weightage for minorities However it did

allow for the reservation of minority

seats in provinces having minorities of

at least ten per cent but this was to be

in strict proportion to the size of the

community

There was no provision for complete

Independence for India

Q48) Ans (c)

The religion of early Vedic Aryans was

primarily of worship of nature and

Yajnas

The early Aryan religion was kind of

nature worship Actually the forces

around them which they could not

control or understand were invested

with divinity and were personified as

male or female gods And they

performed some Yajnas also

Q49) Ans (b)

The roads and river-routes were not

immune from robbery It is notable that

Yuan Chwang (Hiuen Tsang) was

robbed of his belongings during

Harshvardanarsquos period

Q50) Ans (c)

Q51) Ans (b)

Purandara Dasa was a saint and great

devotee of Lord Krishna

There is much speculation about where

Purandara Dasa regarded as the

Pitamaha of Carnatic music was born

Recently an expert committee

constituted by the Kannada University

Hampi has come to the conclusion that

Kshemapura Shivamogga district

Karnataka is the birth place of

Purandara Dasa

Q52) Ans (c)

Sri Tyagaraja Sri Shyama Shastry and Sri Muthuswami Dikshitar are considered the trinity of Carnatic music and with them came the golden age in Carnatic music in the 18th-19th

century

Q53) Ans d)

Recently a rare sarcophagus (stone

coffin) which is 2000 years old from the

Iron AgendashMegalithic era was discovered

from a rock-cut cave at Viyur village of

Kollam near Koyilandy in Kozhikode

district Kerala

The coffin containing bone fragments

was found during an excavation ldquoSo

far such a rare finding has been

discovered only from two sites

in Kerala Both these sarcophagi were

recovered from Megalithic sites at

Chevayur and Atholi also in Kozhikode

district

Q54) Ans a)

The megalithic culture in South India was a full-fledged Iron Age culture

Q55) Ans d)

The Cholas Pandyas and Keralaputras

(Cheras) mentioned in Ashokan

inscriptions were probably in the late

megalithic phase of material culture

Q56) Ans d)

Q57) Ans (b)

Raj Kumar Shukla followed Gandhiji all

over the country to persuade him to

come to Champaran to investigate the

problem associated with tinkathia

system

RAUSIAS-FC19E1003 47

Brij Kishore Rajendra Prasad Mahadev

Desai and Narhari Parikh accompanied

Gandhi ji during the Champaran

Satyagraha

Q58) Ans (b)

The Satvahanas started the practice of granting tax-free villages to brahmanas and Buddhist monks

Q59) Ans c)

The objectives of the Programme are

listed as under

- Developing basic tourism

infrastructure

- Promoting cultural and heritage

value of the country to generate

livelihoods in the identified regions

- Enhancing the tourist attractiveness

in a sustainable manner by

developing world-class

infrastructure at the heritage

monument sites

- Creating employment through active

involvement of local communities

- Harnessing tourism potential for its

effects on employment generation

and economic development

- Developing sustainable tourism

infrastructure and ensuring proper

Operations and maintenance

therein

Q60) Ans (b)

The Tribal Cooperative Marketing

Development Federation of India

(TRIFED) came into existence in 1987

It is a national-level apex organization

functioning under the administrative

control of Ministry of Tribal Affairs

Govt of India

TRIFED has its registered and Head

Office located in New Delhi

Q61) Ans (c)

Premchandrsquos novels include

Premashram Rangabhumi Ghaban

Karmabhumi and Godan

Gora is a novel written by Rabindranath

Tagore

138th birth anniversary of Munshi

Premchand was celebrated across the

country

Q62) Ans (b)

Giddha is a traditional pastoral dance

performed by the women of the Punjab

India and Pakistan at festival times

and at the sowing and reaping of the

harvest

By this dance the Punjabi women

reveal their joy expel their suppressed

feelings in a male dominated society

through the performance of Giddha

Since this dance has nothing to do with

men only women can participate in it

During the Teej celebrations Giddha

dance is celebrated in Punjab every

year Teej is a generic name for a

number of festivals that are celebrated

by women in some parts of India

Q63) Ans (a)

Dara Shukoh wrote the remarkable

work called ldquoMajma-ul-Bahrainrdquo or the

ldquoThe confluence of two seasrdquo

The Vice President of India Shri M

Venkaiah Naidu has said that Prince

Dara Shukohrsquos writings can come as a

refreshing source for infusing peace and

harmony He was addressing the

gathering after visiting the exhibition

that showcases the forgotten Prince of

yesteryears Dara Shukoh organized by

Mr Francois Gautier at Indira Gandhi

National Centre for the Arts in New

Delhi

Q64) Ans (c)

The statue Gommateshwara is

dedicated to the Jain God Bahubali

It is a monolithic statue

President Ram Nath Kovind

inaugurated the grand anointing

ceremony mdash Mahamastakabhisheka mdash

held once in 12 years at

Shravanabelagola (Karnataka)

Q65) Ans (c)

Prachi Valley had come up around the

Prachi river Prachi Valley gradually

disappeared

RAUSIAS-FC19E1003 48

The Prachi river originates from

Bhubaneswar

It is a tributary of the Mahanadi and

flows through the districts of Puri

Khurda Cuttack and Jagatsinghpur

and the entire region of the river is

termed as the Prachi Valley

It falls into the Bay of Bengal

Archaeological evidence shows that the

Prachi Valley Civilisation predates both

Harappa and Mohenjo-Daro

The Prachi river originates from

Bhubaneswar

Q66) Ans (d)

These monuments are located in

Chhatarpur district Madhya Pradesh

within Vindhya mountain range

Q67) Ans (a)

The book lsquoThoughts on Pakistanrsquo was

written by Dr BR Ambedkar

On the occasion of the birth anniversary

of Dr BR Ambedkar the president of

India pays homage to this icon of India

In 1924 he founded the Depressed

Classes Institute (Bahishkrit Hitkarini

Sabha) and in 1927 the Samaj Samata

Sangh

Another area of attention for Ambedkar

was education For its spread among

the low classes he set up a network of

colleges by the name of Peoples

Education Society and founded hostels

Q68) Ans(b)

Mehrgarh is a famous Neolithic

settlement in the Indian subcontinent

which is situated in Baluchistan

province Pakistan

A pre-historic rock art site is discovered

in the vast expanse of limestone blocks

on the eastern banks of Naguleru river

near Dachepalli (Andhra Pradesh) It

has thrown light on the Neolithic

civilisation that flourished in Guntur

(Andhra Pradesh) during 1500-2000

BC

Q69) Ans (c)

The 12th and the 13th centuries saw

the emergence of the Kakatiyas They

were at first the feudatories of the

Western Chalukyas of Kalyana Initially

they ruled over a small territory near

Warangal (Telangana)

They introduced Nayakships which was

later adopted and developed by the

Rayas of Vijayanagara

Q70) Ans (a)

The fast had effect of putting pressure

on mill owners who finally agreed to

give the workers a 35 per cent increase

in wages

Google celebrated with a doodle the

132nd birth anniversary of Anasuya

Sarabhai who played a pioneering role

in Indiarsquos labour movement

Q71) Ans (d)

The UNESCOrsquos list of the representative

list of the intangible cultural heritage of

humanity from India are

- Koodiyattam Sanskrit Theatre of

Kerala

- Mudiyettu ritual theatre and dance

drama of Kerala

- Tradition of Vedic Chanting

- Kalbelia folk songs and dances of

Rajasthan

- Ramlila Traditional Performance of

the Ramayana

- Sankirtana ritual singing

drumming and dancing of Manipur

- Ramman religious festival and

ritual theatre of the Garhwal

Himalayas India

- Traditional brass and copper craft of

utensil making among the Thatheras

of Jandiala Guru Punjab India

- Chhau dance classical Indian dance

originated in the eastern Indian

states

- Buddhist chanting of Ladakh

recitation of sacred Buddhist texts

in the trans-Himalayan Ladakh

region Jammu and Kashmir India

- Yoga

- Nouroz

- Kumbh Mela

RAUSIAS-FC19E1003 49

Q72) Ans(b)

The President of India Shri Ram Nath Kovind inaugurated the Hornbill Festival and State Formation Day celebrations of Nagaland in Kisama

The festival is named after the Indian hornbill the large and colourful forest bird which is displayed in the folklore of most of the states tribes

The major recognized tribes of Nagaland are Angami Ao Chakhesang Chang

Kuki Rengma and Zeling etc

Onge Jarawa and Sentinelese are the

tribes of Andman amp Nicobar Islands

Q73) Ans (c)

The Rashtrakutas rule in the Deccan lasted for almost two hundred years till the end of the tenth century The Rashtrakutas rulers were tolerant in their religious views and patronized not only Shaivism and Vaishnavism but

Jainism as well

The famous rock-cut temple of Shiva at Ellora was built by one of the Rashtrakutas kings Krishna I in the ninth century His successor Amoghavarsha was a Jain but he also

patronized other faiths

The Rashtrakutas allowed Muslims traders to settle and permitted Islam to

be preached in their dominions

Recently increasing defacement at the prehistoric rock paintings of Pandavulagutta Telangana has created a cause for grave concern It can spoil

the prehistoric rock

Pandavulagutta is home to

- Painted rock shelters dating to

10000 BC-8000 BC

- An 8th century inscription of the

Rashtrakuta period and

- Painted frescoes from the 12th century Kakatiya empire

Q74) Ans (b)

In 1828 Raja Ram Mohan Roy founded a new religious society the Brahma Sabha later known as the Brahmo

Samaj

Debendranath Tagore headed the Tattvabodhini Sabha which was

engaged in search of spiritual truth

Its purpose was to purify Hinduism and to preach monotheism or belief in one God

The new society was to be based on the twin pillars of reason and the Vedas and

Upanishads

Recently Sadharan Brahmo Samaj (SBS) has entered into a legal battle with the West Bengal government due

to some legal issue

Q75) Ans (c)

The Chishti order was established in India by Khwaja Moinuddin Chishti who came to India around 1192 The Chishtirsquos are considered to be the most influential of the groups of Sufis who migrated to India in the late twelfth century They adapted successfully to the local environment and adopted several features of Indian devotional

traditions

The historical dargah of Sufi mystic Khwaja Moinuddin Chishti in Ajmer is all set to get a facelift This 13 th century dargah has been included among the Swachh Iconic Places a clean-up initiative focused on iconic

heritage spiritual and cultural places

Page 23: GENERAL STUDIES (PAPER I) · Test is part of Rau’s IAS Test series for Preliminary Exam 2019 FOUNDATION + CURRENT AFFAIRS GENERAL STUDIES (PAPER –I) FOUNDATION TEST –III TOPIC:

RAUSIAS-FC19E1003 23

Q60) Consider the following statements about

the Tribal Cooperative Marketing

Development Federation of India

(TRIFED)

1 It is a national-level apex

organization functioning under the

administrative control of Ministry

of Home Affairs Government of

India

2 The main objective of TRIFED is

socio-economic development of

tribal people in the country

Which of the statements given above

isare correct

(a) 1 only

(b) 2 only

(c) Both 1 and 2

(d) Neither 1 nor 2

Q61) Which of the following novels isare

written by Premchand

1 Rangabhumi

2 Godan

3 Gora

Select the correct answer using the code

given below

(a) 1 only

(b) 2 only

(c) 1 and 2 only

(d) 1 2 and 3

Q62) Consider the following statements about

Giddha dance

1 Giddha is a traditional pastoral

dance performed by the women of

Bihar at festival times and at the

sowing and reaping of the harvest

2 By this dance the women reveal

their joy expel their suppressed

feelings in a male dominated

society through the performance of

Giddha

Which of the statements given above

isare correct

(a) 1 only

(b) 2 only

(c) Both 1 and 2

(d) Neither 1 nor 2

Q63) Consider the following statements

1 Mullah Shah Badakhshi was the

spiritual mentor of Dara Shukoh

2 Aurangzeb wrote the remarkable

work called ldquoMajma-ul-Bahrainrdquo or

the ldquoThe confluence of two seasrdquo

3 Dara Shukoh was seen as

inheriting the qualities of his

ancestor Akbar in that he

promoted religious pluralism and

syncretism

Which of the statements given above

isare correct

(a) 1 and 3 only

(b) 2 only

(c) 1 and 2 only

(d) 1 2 and 3

RAUSIAS-FC19E1003 24

Q64) निमननिखित कथि ो पर निचार कीनजए

1 ग मतशवर परनतमा निोधयनगरी पहाड़ी पर खसथत ह

2 शरिरबिग िा िह सथाि ह जहाो मौयण िोश क

सोसथापक चोदरगपत मौयण अपि नसोहासि क

तयागि क बाद जि तपसवी बि गए थ

उपयणकत कथि ो म स कौि-सास सही हह

(a) किि 1

(b) किि 2

(c) 1 और 2 द ि ो

(d) ि त 1 ि ही 2

Q65) निमननिखित कथि ो पर निचार कीनजए

1 पराताखतवक साकषय स पता चिता ह नक पराची

घाटी सभयता हड़पपा और म हिज दाड़ द ि ो

की पिणिती ह

2 पराची िदी भििशवर स निकिती ह

उपयणकत कथि ो म स कौि-सास सही हह

(a) किि 1

(b) किि 2

(c) 1 और 2 द ि ो

(d) ि त 1 ि ही 2

Q66) निमननिखित कथि ो म स कौि-सास सही हह

1 िजराह क समारक ो क समह का निमाणर

चोदि राजिोश क शासिकाि क दौराि हआ

था

2 य समारक हररिोदर पिणत शरोििा म खसथत ह

3 म रक क यातरी इबन बतता ि अपि सोसमरर ो

म िजराह क मोनदर ो की यातरा का उललि

नकया था तथा इन काजराण िाम स समब नरत

नकया था

िीच नदए गए कट का परय ग कर सही उततर चनिए

(a) किि 1

(b) किि 1 और 2

(c) किि 2 और 3

(d) किि 1 और 3

Q67) निमननिखित कथि ो म स कौि-सास सही हह

1 डॉ बी आर अमबडकर ि दी एनिनहिशि

ऑफ़ कासट (The Annihilation of Caste)

नििी थी नजसम उन ोि नहोद रमण म िोशािगत

पजारी की परथा क उनमिि की आिशयकता

पर बि नदया था

2 डॉ राजदर परसाद ि थॉटस ऑि पानकसताि

(Thoughts on Pakistan) िामक पसतक

नििी थी

िीच नदए गए कट का परय ग कर सही उततर चनिए

(a) किि 1

(b) किि 2

(c) 1 और 2 द ि ो

(d) ि त 1 ि ही 2

Q68) निमननिखित कथि ो म स कौि-सास सही हह

1 महरगढ़ भारतीय उपमहादवीप म एक परनसदध

ििपािार बसती ह ज नसोर पराोत पानकसताि म

खसथत ह

2 बरणह म म कतत ो क उिक सवामी क साथ कबर ो

म दफिाया जाता था

िीच नदए गए कट का परय ग कर सही उततर चनिए

(a) किि 1

(b) किि 2

(c) 1 और 2 द ि ो

(d) ि त 1 ि ही 2

Q69) निमननिखित कथि ो म स कौि-सास सही हह

1 काकानटय मोनदर अनरकतर नशि क समनपणत

2 हिमक ोडा म हजार-सतोभ िाि मोनदर (The

Thousand-Pillared Temple) का निमाणर

काकानटय समराट रदर ि करिाया था

िीच नदए गए कट का परय ग कर सही उततर चनिए

(a) किि 1

(b) किि 2

(c) 1 और 2 द ि ो

(d) ि त 1 ि ही 2

RAUSIAS-FC19E1003 25

Q64) Consider the following statements

1 Gommateshwara Statue is located

on the Vindyagiri Hill

2 Shravanabelagola is the place

where Chandragupta Maurya the

founder of the Mauryan dynasty

became a Jain ascetic after

relinquishing his throne

Which of the statements given above

isare correct

(a) 1 only

(b) 2 only

(c) Both 1 and 2

(d) Neither 1 nor 2

Q65) Consider the following statements

1 Archaeological evidence shows

that the Prachi Valley Civilisation

predates both Harappa and

Mohenjo-Daro

2 The Prachi river originates from

Bhubaneswar

Which of the statements given above

isare correct

(a) 1 only

(b) 2 only

(c) Both 1 and 2

(d) Neither 1 nor 2

Q66) Which of the following statements

isare correct

1 The Khajuraho group of

monuments was built during the

rule of the Chandela dynasty

2 These monuments are located in

Harischandra mountain range

3 Ibn Battuta the Moroccan

traveller in his memoirs mentioned

visiting Khajuraho temples and

called them Kajarra

Select the correct answer using the code

given below

(a) 1 only

(b) 1 and 2

(c) 2 and 3

(d) 1 and 3

Q67) Which of the following statements

isare correct

1 Dr BR Ambedkar wrote the

Annihilation of Caste emphasising

the need to do away with the

practice of hereditary priesthood in

Hinduism

2 The book lsquoThoughts on Pakistanrsquo

was written by Dr Rajendra

Prasad

Select the correct answer using the code

given below

(a) 1 only

(b) 2 only

(c) Both 1 and 2

(d) Neither 1 nor 2

Q68) Which of the following statements

isare correct

1 Mehrgarh is a famous Neolithic

settlement in the Indian

subcontinent which is situated in

Sindh province Pakistan

2 At Burzahom dogs were buried

with their masters in their graves

Select the correct answer using the code

given below

(a) 1 only

(b) 2 only

(c) Both 1 and 2

(d) Neither 1 nor 2

Q69) Which of the following statements

isare correct

1 The Kakatiya temples are

dedicated mostly to Siva

2 The Thousand-Pillared Temple at

Hanamkonda was built by the

Kakatiya king Rudra

Select the correct answer using the code

given below

(a) 1 only

(b) 2 only

(c) Both 1 and 2

(d) Neither 1 nor 2

RAUSIAS-FC19E1003 26

Q70) निमननिखित कथि ो म स कौि-सास सही हह

1 अहमदाबाद नमि हड़ताि क दौराि महातमा

गाोरी ि शरनमक ो क पकष क मजबत करि क

निए आमरर अिशि नकया था

2 अिशि स नमि मानिक ो पर दबाि पड़ा था ज

अोततः शरनमक ो क िति म 15 परनतशत की िखदध

करि क निए सहमत हए थ

िीच नदए गए कट का परय ग कर सही उततर चनिए

(a) किि 1

(b) किि 2

(c) 1 और 2 द ि ो

(d) ि त 1 ि ही 2

Q71) निमननिखित म स नकसक नकिक भारत स यिसक

की माििता की अमतण साोसकनतक निरासत की

परनतनिनर सची (The UNESCOrsquos List of the

Representative List of the Intangible

Cultural Heritage of Humanity) म शानमि

नकया गया ह

1 मनडयटट

2 सोकीतणि

3 को भ मिा

िीच नदए गए कट का परय ग कर सही उततर चनिए

(a) किि 1 और 2

(b) किि 2 और 3

(c) किि 3

(d) 1 2 और 3

Q72) निमननिखित जिजानतय ो म स कौि-सीसी ो

जिजानतजिजानतया िागािड स सोबोनरत हह

1 अोगामी

2 ककी

3 जारिा

िीच नदए गए कट का परय ग कर सही उततर चनिए

(a) किि 1

(b) किि 1 औऔ 2

(c) किि 2

(d) 1 2 और 3

Q73) निमननिखित कथि ो म स कौि-सास सही हह

1 राषटर कट सामराजय की सथापिा दोनतदगण ि की थी

नजसि मानयाित म अपिी राजरािी की

सथापिा की थी

2 राषटर कट समराट अम घििण एक ििक था और

उस कनिताओो पर पहिी कननड़ पसतक नििि

का शरय नदया जाता ह

िीच नदए गए कट का परय ग कर सही उततर चनिए

(a) किि 1

(b) किि 2

(c) 1 और 2 द ि ो

(d) ि त 1 ि ही 2

Q74) निमननिखित कथि ो म स कौि-सास सही हह

1 कशब चोदर सि ि ततवब नरिी सभा की

अधयकषता की थी ज आधयाखतमक सतय की

ि ज म सोिि थी

2 बरहम समाज ि मािि गररमा पर बि नदया

मनतणपजा का निर र नकया और सती परथा जसी

सामानजक बराइय ो की आि चिा की

िीच नदए गए कट का परय ग कर सही उततर चनिए

(a) किि 1

(b) किि 2

(c) 1 और 2 द ि ो

(d) ि त 1 ि ही 2

Q75) निमननिखित कथि ो म स कौि-सास सही हह

1 भारत म नचशती नसिनसिा खवाजा म इिददीि

नचशती क दवारा सथानपत नकया गया था

2 नचशती परोपरा की एक परमि निशिता

आतमसोयम थी नजसम साोसाररक म ह स दरी

बिाए रििा शानमि था

िीच नदए गए कट का परय ग कर सही उततर चनिए

(a) किि 1

(b) किि 2

(c) 1 और 2 द ि ो

(d) ि त 1 ि ही 2

RAUSIAS-FC19E1003 27

Q70) Which of the following statements

isare correct

1 During the Ahmedabad Mill Strike

Mahatma Gandhi undertook a fast

unto death to strengthen the

workersrsquo resolve

2 The fast had effect of putting

pressure on mill owners who

finally agreed to give the workers a

15 per cent increase in wages

Select the correct answer using the code

given below

(a) 1 only

(b) 2 only

(c) Both 1 and 2

(d) Neither 1 nor 2

Q71) Which of the following are included in

the UNESCOrsquos list of the representative

list of the intangible cultural heritage of

humanity from India

1 Mudiyettu

2 Sankirtana

3 Kumbh Mela

Select the correct answer using the code

given below

(a) 1 and 2 only

(b) 2 and 3 only

(c) 3 only

(d) 1 2 and 3

Q72) Which of the following tribes isare

related to Nagaland

1 Angami

2 Kuki

3 Jarawa

Select the correct answer using the code

given below

(a) 1 only

(b) 1 and 2 only

(c) 2 only

(d) 1 2 and 3

Q73) Which of the following statements

isare correct

1 Rashtrakuta kingdom was founded by Dantidurga who established his capital at Manyakhet

2 Amoghavarsha a Rashtrakuta king was an author and is credited with writing the first

Kannada book on poetics

Select the correct answer using the code given below

(a) 1 only

(b) 2 only

(c) Both 1 and 2

(d) Neither 1 nor 2

Q74) Which of the following statements isare correct

1 Keshab Chandra Sen headed the Tattvabodhini Sabha which was engaged in search of spiritual truth

2 The Brahmo Samaj laid emphasis on human dignity opposed idolatry and criticized such social

evils as the practice of Sati

Select the correct answer using the code given below

(a) 1 only

(b) 2 only

(c) Both 1 and 2

(d) Neither 1 nor 2

Q75) Which of the following statements isare correct

1 The Chishti order was established in India by Khwaja Moinuddin

Chishti

2 A major feature of the Chishti tradition was austerity including maintaining a distance from the

worldly power

Select the correct answer using the code

given below

(a) 1 only

(b) 2 only

(c) Both 1 and 2

(d) Neither 1 nor 2

T e s t i s p a r t o f R a u rsquo s I A S T e s t s e r i e s f o r P r e l i m i n a r y E x a m 2 0 1 9

FOUNDATION + CURRENT AFFAIRS

GENERAL STUDIES (PAPER ndashI)

FOUNDATION TEST ndashIII

SUBJECT NCERT History Class VI-X + Current Affairs

Time Allowed 1frac12 Hours Maximum Marks 150

I NSTRUCT IONS

1 IMMEDIATELY AFTER THE COMMENCEMENT OF THE EXAMINATION YOU SHOULD CHECK

THAT THIS TEST BOOKLET DOES NOT HAVE ANY UNPRINTED OR TORN or MISSING PAGES OR

ITEMS ETC IF SO GET IT REPLACED BY A COMPLETE TEST BOOKLET

2 This Test Booklet contains 75 items (questions) Each item is printed both in Hindi and English

Each item comprises four responses (answers) You will select the response which you want to mark

on the Answer Sheet In case you feel that there is more than one correct response mark the

response which you consider the best In any case choose ONLY ONE response for each item

3 You have to mark all your responses ONLY on the separate Answer Sheet (OMR sheet) provided

Read the directions in the Answer Sheet

4 All items carry equal marks

5 Before you proceed to mark in the Answer Sheet the response to various items in the Test booklet

you have to fill in some particulars in the Answer Sheet as per instructions contained therein

6 After you have completed filling in all your responses on the Answer Sheet and the examination has

concluded you should hand over to the Invigilator only the Answer Sheet You are permitted to

take away with you the Test Booklet

7 Penalty for wrong answers

THERE WILL BE PENALTY FOR WRONG ANSWERS MARKED BY A CANDIDATE IN THE

OBJECTIVE TYPE QUESTION PAPERS

(i) There are four alternatives for the answer to every question For each question for which a

wrong answer has been given by the candidate one-third of the marks assigned to that

question will be deducted as penalty

(ii) If a candidate gives more than one answer it will be treated as a wrong answer even if one of

the given answers happens to be correct and there will be same penalty as above to that

question

(iii) If a question is left blank ie no answer is given by the candidate there will be no penalty for

that question

T h i s t e s t i s p a r t o f R a u rsquo s I A S T e s t s e r i e s f o r P r e l i m i n a r y E x a m 2 0 1 9

Test Code

FC19E1003

FC19H1003 29

Answers and Explanations of

NCERT History Class VI-X + Current Affairs (FC19E1003)

Q1) उततर (c)

सपषटीकरण

- ऋगवद म दविय ो और दिताओो क समवपित एक

हजार स अविक सत तर (शल क) ह

- य शल क ऋविय ो क दवारा रच गए थ और परि ो

दवारा सीख जात थ

- हालाोवक कछ शल क मवहलाओो (जस वक अपाला

घ सा ल पामदरा मतरयी और गागी) क दवारा भी रच

गए थ

- ऋगवद म सोिाद क रप म कई शल क मौजद ह

- हम विशवावमतर नामक एक ऋवि और दविय ो क

रप म पजी जान िाली द नवदय ो (वयास और

सतलज) क बीच िाताि का उदाहरण वमलता ह

- इसस पता चलता ह वक विशवावमतर िवदक काल स

सोबोवित थ

Q2) उततर (b)

सपषटीकरण

- करनल गफाओो स राख क अिशि परापत हए ह

ज इस ओर सोकत करत ह वक ततकालीन ल ग

अवि क उपय ग स पररवचत थ

- य गफाएो आोधर परदश म सथथत ह

Q3) उततर (c)

सपषटीकरण

bull बरािह म ितिमान कशमीर म सथथत एक

परागवतहावसक थथल ह जहाो ल ग गडढ क घर ो का

वनमािण करत थ

bull य घर जमीन क ख द कर बनाए जात थ तथा नीच

जान क वलए सीवियाा ह ती थी

bull ऐसा अनमान लगाया जाता ह वक य घर ठो ड क

मौसम म आशरय परदान करत थ

Q4) उततर (c)

सपषटीकरण

bull परालख-विदया (Epigraphy) क वशलालख ो क

अधययन क रप म पररभावित वकया जाता ह

bull हसतवलसखत दसतािज ो क माधयम स इवतहास

और सावहतय क अधययन क पाोडवलवप विजञान

(Manuscriptology) कहत ह

bull पराचीन लखन परणावलय ो क अधययन और

ऐवतहावसक पाोडवलवपय ो क समझन तथा वतवथ

वनिािरण क पलीओगराफी (Palaeography) कहा

जाता ह

bull नयवमजमविकस (Numismatics) वसक ो क

अधययन क सोदवभित करता ह

Q5) उततर (a)

सपषटीकरण

- चरक सोवहता चरक क दवारा वलखी गई आयिद

और िदयक-शासर पर एक महतवपणि पसतक ह

- ि भारतीय िदयक-शासर की पारमपररक परणाली

वजस आयिद क नाम स जाना जाता ह क

अभयासकताि थ

- ऐसा माना जाता ह वक चरक का विकास दसरी

शताबदी (ईसा पिि) और दसरी शताबदी (ईसवी) क

मधय हआ था

Q6) उततर (b)

सपषटीकरण

- भाग फसल ो पर वलए जान िाल कर क सोदवभित

करता ह ज कल फसल उतपादन का 16 िाो भाग

था

- ldquoकममकारrdquo शबद भवमहीन कवि शरवमक िगि क

वलए परय ग वकया जाता था

- ldquoअशवमिrdquo (वजस घ ड क बवलदान क रप म भी

जाना जाता ह) एक अनषठान ह ता था वजसम एक

घ ड क सवतोतर रप स घमन क वलए छ ड वदया

FC19H1003 30

जाता ह और राजा क सवनक उसकी रखिाली

करत थ

Q7) उततर (d)

सपषटीकरण

- ऋगववदक काल म घ ड ो क रथ ो म ज ता जाता था

ज (रथ) भवम मिवशय ो आवद पर कबजा करन क

वलए लड गए यद ो म उपय ग वकए जात थ

- इसस यह पता चलता ह वक घ ड ो यकत रथ ो का

उपय ग महाजनपद काल स काफी पहल आरमभ

हआ था

- ऋगववदक काल म मिवशय ो भवम जल आवद पर

कबजा करन क वलए तथा ल ग ो क पकडन क

वलए यद वकय जात थ

- अविकाोश परि इन यद ो म भाग वलया करत थ

- हालाोवक उस समय क ई वनयवमत सना नही ो ह ती

थी लवकन उस काल म सभाऐो ह ती थी ो वजनम

ल ग यद क मामल ो पर चचाि करत थ

- वनयवमत सनाएा महाजनपद काल का िवशषटय थी

वजनम पदल सवनक ो की विशाल सनाएा रथ तथा

हाथी शावमल ह त थ

Q8) उततर (a)

सपषटीकरण

- बद शाकय कल स सोबोवित थ और कशीनारा म

उनका वनिन हआ था

- बद न अपनी वशकषाएा पराकत भािा म दी थी ो ज

आम ल ग ो की भािा थी

Q9) उततर (c)

सपषटीकरण

- पराचीन भारत म दशिनशासर की छह शाखाएा थी ो

िशविक नयाय समखया य ग पिि वममाोसा और

िदाोत या उततर वममाोसा

- इनकी थथापना करमश कनाद गौतम कवपल

पतोजवल जावमनी और वयास ऋविय ो न की थी

Q10) उततर (b)

सपषटीकरण

महािीर की वशकषाऐो छठी शताबदी म िललभी म

सोकवलत की गई थी ो

Q11) उततर (c)

सपषटीकरण

- पारमपररक रप स चाणकय क कौविलय अथिा

विषणगपत क नाम स जाना जाता ह

- उसन अथिशासतर ज एक पराचीन भारतीय

राजनवतक आलख ह वलखा था

Q12) उततर (d)

सपषटीकरण

- भारत का राषटर ीय वचनह सारनाथ (उततर परदश) क

अश क सतमभ क ऊपर (शीिि पर) वसोह कवपिल

का एक अनरपण ह

- इस राषटर ीय वसदाोत सतयमि जयत क साथ

सोय वजत वकया गया ह

- रामपिि बल का नाम रामपिि (वबहार) क नाम पर

पडा जहाा इसकी ख ज हई थी

- यह अपन नाजक नकाशी मॉडल क वलए परवसदद

ह वजसम क मल तवचा सोिदनशील नथन ो सतकि

कान और मरबत िााग ो क शरषठतर परवतरप क

परदवशित वकया गया ह

- यह भारतीय और फारसी ततव ो का एक ससममशरण

- सोवकससा उततर परदश म सथथत ह

Q13) उततर (a)

सपषटीकरण

का िर वसोह ज एक महान य दा थ वबहार स

सोबोवित थ

Q14) उततर (b)

सपषटीकरण

िललालर शबद बड भ-सवावमय ो क वलए परय ग

वकया जाता था

FC19H1003 31

Q15) उततर (c)

सपषटीकरण

- अररकमड एक तिीय बसती थी जहाो दर दश ो स

आन िाल जहाज ो का माल उतारा जाता था

- यहाो पर ईोि ो का एक विशाल ग दाम वमटटी क

बतिन (वजनम एमफ रा - द हरी मवठय ो का लोबा

घडा - शावमल ह) और एरिाइन (Arretine)

मदभाोड पाए गए थ

- इस थथान पर र मन दीपक काोच क बन पातर और

रतन भी पाए गए थ

Q16) उततर (a)

सपषटीकरण

- मिनदर सोगम कविताओो म उसललसखत एक

तवमल शबद ह वजसका अथि ह ldquoतीन परमखrdquo

- यह तीन सततारि पररिार ो क मसखयाओो क वलए

परय ग वकया जाता ह च ल चर और पाणडय

Q17) उततर (c)

सपषटीकरण

- ऋग िद म सभा विदाथा तथा गण जसी

जनजावतय ो पर अथिा किोब पर आिाररत

सभाओो का उललख ह

- आरसमभक िवदक काल म सभाओो और सवमवतय ो

का विशि महतव ह ता था

- यहाा तक की मसखया अथिा राजा भी उनका

समथिन परापत करन क वलए आतर रहत थ

Q18) उततर (a)

सपषटीकरण

- जन िमि न ईशवर क अससततव क मानयता त दी ह

वकनत उसन ईशवर क वजना क पद स नीच रखा

- जन िमि न बौद िमि की तरह िणि परणाली की

भरतिना नही ो की थी

Q19) उततर (d)

सपषटीकरण

- च ल ो और पाणडय ो न शसकतशाली तिीय शहर ो का

विकास वकया था

- च ल ो का सबस महतवपणि शहर पहार (या

कािरीपटटीनम) था |

- मदरई पाणडय ो की राजिानी थी

Q20) उततर (b)

सपषटीकरण

- ldquoबदचररतrdquo बद का जीिन-ितताोत ह

- इस अशवघ ि क दवारा वलखा गया था

Q21) उततर (a)

सपषटीकरणः

- तवमल कवि अपपर भगिान वशि क भकत थ

- इस परकार ि एक नयनार सोत थ

Q22) उततर (d)

सपषटीकरणः

- समदरगपत एक परवसद गपत शासक था

- उसन वसक ो पर िीणा बजात हए अपनी छवि

अोवकत करिाई थी

- यह सोगीत क परवत उसक परम क दशािता ह

- हम उसकी इलाहाबाद परशससत स महतवपणि

ऐवतहावसक जानकारी वमलती ह वजसकी रचना

उसक दरबार क कवि हररसन न की थी

Q23) उततर (b)

सपषटीकरणः

- विकरम सोित की शरआत ििि 58 ईसा पिि म

चनदरगपत वदवतीय न की थी

- यह शक ो पर उसकी जीत और उस विकरमावदतय

की पदिी वमलन क उपलकषय म आरमभ वकया गया

था

FC19H1003 32

- बानभटट न हिििििन का जीिन-ितताोत हििचररत

(ज सोसकत म थी) वलखी थी

Q24) उततर (c)

सपषटीकरणः

- सोवि-विगरावहका यद एिो शाोवत का मोतरी

- साथििाह वयापाररय ो क कावफल ो का नता

Q25) उततर (a)

सपषटीकरणः

- जआन झाोग (हसआन रताोग ndash Hsuang Tsang)

एक चीनी यातरी था ज हिििििन क शासनकाल म

भारत आया था

- ििि 630 ईसवी स ज दशक आरमभ हआ था उसम

जआन झाोग मधय एवशया ईरान और

अफग़ावनसतान की यातरा करन क पशचात कशमीर

क रासत स भारत आया था

- उसन उततर स पिि तक की यातरा की और िह

लगभग 2 ििि वबहार म रहा

- जआन झाोग न नालनदा विशवविदयालय म विदयावथिय ो

और विदवान ो क साथ पारसपररक विचार-विमशि

वकया थथानीय भािाओ ा म वनपणता परापत की तथा

बौद सतप ो की ख ज की

Q26) उततर (c)

सपषटीकरणः

- परदवकषणा पथ बौद िासतकला म सतप क चार ो

ओर बनाया जान िाला एक घमािदार पथ ह ता

- परशन म वदए गए बाकी क तीन ो ततव वहोद मसनदर ो की

िासतकला क भाग ह

Q27) उततर (d)

सपषटीकरणः

परशन म वदए गए सभी मोवदर ो म वयापक रप स

ईोि ो (पकी ईोि ो) का परय ग पतथर ो क साथ हआ

Q28) उततर (c)

सपषटीकरण

- महममद कली कतब शाह ग लकणडा का सलतान

था

- िह अकबर का समकालीन था

- सावहतय और िासतकला म उसकी अतयाविक

रवच थी

- िह एक महान कवि था

- िह दसखनी उदि फारसी और तलग म वलखता था

- उसन अपन पीछ एक विसतत वदिान (सोगरह)

छ डा ह

- अभी हाल ही म तलोगाना म ग लकणडा क वकल

क अनदर खदाई वकय गए बाग-ए-नाया वकला

बाग क चार ो ओर रप-रखा क मानवचतरण क

वलए भारतीय परातासतवक सिकषण (The

Archaeological Survey of India ndash ASI)

गराउणड पनीिर विोग रडार (Ground Penetrating

Radar) का परय ग करगा

Q29) उततर (a)

सपषटीकरणः

- वसलपपावदकारम एक तवमल महाकावय ह वजसकी

रचना इलाोग क दवारा लगभग 1800 ििि पिि की

गई थी

- यह क िलन नामक एक वयापारी की कहानी ह

ज माििी नामक एक गवणका (िशया) स परम

करन लगा था

- मवनमकलाई क िलन और माििी की पतरी की

कहानी ह

Q30) उततर (a)

सपषटीकरण

- चरक आयिद और वचवकरता की एक महतवपणि

रचना चरक सोवहता क लखक ह

- बरहमगपत क अपनी रचना बरहम-सफि-वसदानत

(ज एक खग लीय रचना ह) क कारण परवससद

वमली

FC19H1003 33

- बगदाद म इसका अनिाद अरबी भािा म वकया

गया था

- इसका इसलावमक गवणत और खग ल-विजञान पर

महतवपणि परभाि पडा था

- बाद म अपन जीिनकाल म बरहमगपत न

ldquoखोडखयाकrdquo वलखी ज एक खग लीय पससतका

(एक छ िी पसतक) थी

- इसम आयिभटट की अिि-रावतर क परतयक वदन की

शरआत परणाली का परय ग वकया गया था

Q31) उततर (c)

सपषटीकरण

- अमीर खसर एक परवसद सफी सोगीतकार कवि

और विदवान थ

- 1318 म उनह ोन पाया वक इस भवम (वहोदसतान) क

हर कषतर म अलग-अलग भािा थी लाहौरी

कशमीरी दवारसमदरी (दवकषणी कनाििक म)

तलोगाना (आोधर परदश म) गजरी (गजरात म)

माबारी (तवमलनाड म ) अििी (पिी उततर परदश

म) और वहोदिी (वदलली क आस-पास क कषतर म)

आवद

- उनह न यह बताया वक सोसकत वकसी भी कषतर स

सोबोवित नही ो थी और किल बराहमण ही इस भािा

का जञान रखत थ

Q32) उततर (c)

सपषटीकरण

- वहरणय-गभि सववणिम गभि क सोदवभित करता ह

- जब बराहमण ो की सहायता स यह अनषठान वकया

जाता था त यह माना जाता था वक बवल दन िाल

का कषवतरय क रप म पनजिनम ह गा

Q33) उततर (d)

सपषटीकरण

- कदमई भवम राजसव पर कर क सोदवभित करता

- गवावलयर परशससत म नागभि क दवारा वकय गए

श िण का िणिन वकया गया ह |

- नागभि एक परवतहार राजा था

Q34) उततर (b)

सपषटीकरण

- राजतरो वगनी 12िी ो शताबदी म कलहन क दवारा

रवचत एक सोसकत पसतक (िकसट) ह

- यह परारसमभक भारत की ऐवतहावसक इवतितत थी

- तकि सोगत रप स इस अपन परकार की सिोततम

और सिािविक विशवसनीय कवत माना जाता ह

- यह कशमीर कषतर क पराचीनतम समय स लकर

उसकी रचना की तारीख तक क समपणि इवतहास

का आचछादन करती ह

Q35) उततर (c)

सपषटीकरण

- गााि की आम सभा क ldquoउरrdquo कहा जाता था

- ldquoउरrdquo म गााि क सभी कर दन िाल वनिासी

शावमल ह त थ

Q36) उततर (a)

सपषटीकरण

- वदलली सलतनत म ldquoतारीखrdquo इवतहास लखन का

एक रप था

- ldquoतािरीखrdquo क लखक विदवान परि ह त थ वजनम

सवचि परशासक इतयावद शावमल थ

Q37) उततर (a)

सपषटीकरण

- अलाउददीन सखलजी अपन सवनक ो क ितन का

भगतान नकद म करता था न वक इकता क रप

- सवनक अपना सामान वदलली म वयापाररय ो स

खरीदत थ अतः इस बात का भय था वक वयापारी

कही ो िसतओो का मलय न बिा द

- इसकी र कथाम क वलए अलाउददीन सखलजी न

वदलली म कीमत ो क वनयसित वकया

FC19H1003 34

- अविकारीगण धयानपििक मलय ो का सिकषण करत

थ तथा ज वयापारी वनिािररत मलय पर माल नही ो

बचत थ उनक दसणडत वकया जाता था

Q38) उततर (d)

सपषटीकरण

- वदलली सििपरथम त मर राजपत ो क अिीन उनक

सामराजय की राजिानी बनी थी

- 12िी ो शताबदी क मधय म अजमर क चौहान ो

(वजनह चाहमान ो क नाम स भी जाना जाता ह) न

त मर राजपत ो क परावजत वकया था

- त मर ो और चौहान ो क अिीन वदलली एक

महतवपणि िावणसजयक क दर बन गया था

- कई जन वयापारी यहाा रहन लग थ और उनह ोन

कई मोवदर भी बनिाए

- यहाा पर मवदरत वसक वजनह ldquoदहलीिालrdquo क नाम

स जाना जाता था वयापक रप स परचलन म थ

Q39) उततर (c)

सपषटीकरण

- म ठ की मसिद का वनमािण वसको दर ल दी क

राजयकाल म उसक मिी क दवारा करिाया गया

था

- बगमपरी मसिद का वनमािण महममद तगलक क

शासनकाल म हआ था

- यह मसिद विशव का पणयथथान (The

Sanctuary of the World) और वदलली म महममद

तगलक की नई राजिानी जहाोपनाह की मखय

मसिद थी

- कववत- अल - इसलाम मसिद का विसतार

इलतसिश और अलाउददीन सखलजी न वकया था

- मीनार का वनमािण तीन सलतान ो कतबददीन ऐबक

इलतसिश और वफर ज शाह तगलक क दवारा

करिाया गया था

Q40) उततर (c)

सपषटीकरण

- मगल ो क अिीन मनसबदार शबद उस वयसकत क

वलए सोदवभित वकया जाता था वजसक पास मनसब

(अथाित पद) ह ता था

- उस अपना ितन राजसव कायो वजनह जागीर कहत

थ क रप म परापत ह ता था

Q41) उततर (b)

सपषटीकरण

- ldquoभारत छ ड आोद लनrdquo वबरविश शासन क

सखलाफ ल ग ो का एक सवाभाविक विदर ह था

- असखल भारतीय काोगरस सवमवत न 8 अगसत 1942

क बमबई म एक बठक का आय जन वकया था

- इस बठक म परवसद सोकलप ldquoभारत छ ड rdquo क

पाररत वकया गया और इस उददशय क परापत करन

क वलए गाोिी क नततव म एक अवहोसक जन सोघिि

आोद लन की शरआत का परसताि वदया गया

- लवकन अगल ही वदन गाोिी और काोगरस क अनय

परमख नताओो क वगरफतार कर वलया गया

- काोगरस क एक बार वफर अिि घ वित वकया गया

था

Q42) उततर (c)

सपषटीकरण

- साइमन कमीशन यनाइविड वको गडम क सात

साोसद ो का एक समह था

- इस वबरविश भारत क वलए सोििावनक सिार ो का

सझाि दन क वलए गवठत वकया गया था

- इस आय ग म िररषठ वबरविश राजनता सर जॉन

साइमन क नततव म किल वबरविश सदसय ही

शावमल थ

- इसवलए भारत क ल ग ो न साइमन कमीशन क

आगमन क विरद आोद लन वकया था

Q43) उततर (a)

सपषटीकरण

bull दादा भाई नौर जी भारत म वबरविश शासन क

आवथिक पररणाम ो क बार म अपनी विर िी

(परवतकल) राय क वलए जान जात थ

FC19H1003 35

bull अपन कई लख ो और भािण ो म विशि रप स

ldquoपाििी एो ड अन-वबरविश रल इन इसणडया

(Poverty and Un-British Rule in India) म

नौर जी न यह तकि वदया वक भारत पर अतयविक

कर लगाया गया था और इसकी सोपवतत इोगलड की

ओर परिावहत की जा रही थी

bull उनह ोन पराचीन भारतीय गरोथ ो की वयाखया करन

का और भारतीय ो क आिविशवास क बहाल

करन पर कायि नही ो वकया था

उनह ोन वकसी और बात स पहल सभी सामावजक

बराइय ो क उनमलन की आिशयकता पर भी बल

नही ो वदया था

Q44) उततर (c)

सपषटीकरण

bull अगसत 1932 म वबरविश परिानमोतरी मकड नालड न

अपन साोपरदावयक परसकार (The Communal

Award) की घ िणा की थी

bull यह भारत क कई साोपरदावयक वहत ो क बीच विवभनन

सोघिो क हल करन क वलए वबरिन का एकतरफा

परयास था

bull यह परसकार (Award) बाद म 1935 क

अविवनयम (The Act of 1935) म शावमल वकया

गया था

bull इस साोपरदावयक परसकार न मससलम ो क वलए

आरवकषत एक अलग वनिािचक मणडल फॉमिल का

विसतार अनय अलपसोखयक ो क वलए वकया था

वजसम वसख ो भारतीय ईसाइय ो आोगल-भारतीय

समदाय यर पीय समदाय तथा विवशषट कषतरीय

समह ो क शावमल वकया गया था

bull गाोिी न इस परसताि क भारतीय समाज क

विभावजत करन क वलए एक घवणत वबरविश

सावजश क रप म दखा और उसक सखलाफ

आमरण अनशन वकया

Q45) उततर (b)

सपषटीकरण

मौजदा आयात और वनयाित क अवतररक़त

औपवनिवशक भारत क वनमनवलसखत खचो क

वलए एक विशिवनवशचत िन रावश भी दनी पडती

थी

(i) परशासन क वयय

(ii) सना क रख-रखाि क वयय

(iii) यद क वयय

(iv) सिावनितत अविकाररय ो की पशन तथा

(v) वबरिन दवारा अपनी उपवनिश बसती

(कॉल नी) क रख-रखाि क वयय

इनह गह शलक (Home Charges) क रप म

जाना जाता था और लगभग परी तरह स भारत क

दवारा इनका भगतान वकया जाता था

bull गह शलक म वनमनवलसखत घिक शावमल थ

(i) भारतीय ऋण पर दय बयाज

(ii) ईसट इोवडया को पनी क शयरिारक ो क

लाभाोश

(iii) लोदन म भारत कायािलय चलान क वलए िन

(iv) भारत म वनयकत वबरविश कवमिय ो क ितन

और पशन का भगतान करन क वलए िन

(v) रलि पर बयाज

(vi) नागररक और सनय शलक

(vii) इोगलड म सट र (सामगरी) की खरीद

Q46) उततर (b)

सपषटीकरण

bull भारतीय राषटर ीय काोगरस का लाहौर सतर 1929 म

जिाहरलाल नहर की अधयकषता म आय वजत

वकया गया था

bull इस सतर म भारतीय राषटर ीय आोद लन स समबसित

कई महतवपणि पररणाम सामन आय थ

(i) सििपरथम इस सतर म काोगरस क अधयकष पद

पर जिाहरलाल नहर क चना गया था ज

काोगरस म िामपोवथय ो की बिती हई ताकत

का सपषट सोकत था

(ii) दसरा इस सतर म पहली बार काोगरस न पणि

सवतोतरता की माोग क उठाया था

इस परकार की माोग काोगरस मोच स पहल कभी भी

नही ो उठाई गई थी

Q47) उततर (b)

सपषटीकरण

FC19H1003 36

bull इस ररप िि न वकसी भी समदाय क वलए पथक

वनिािचक मोडल अथिा अलपसोखयक ो क वलए

भाराोश की वसफाररश नही ो की थी

bull तथावप इस ररप िि न उन पराोत ो म अलपसोखयक

सीि ो क आरकषण की अनमवत दी थी जहाा पर कम

स कम दस परवतशत अलपसोखयक ह

bull लवकन यह समदाय क आकार क अनपात म ह ना

चावहए था

bull इस ररप िि म भारत क वलए पणि सवतोतरता क

वलए क ई पराििान नही ो था

Q48) उततर (c)

सपषटीकरण

bull आरो वभक िवदक आयो का िमि मखय रप स

परकवत की पजा और यजञ था

bull परारो वभक आयि िमि परकवत की पजा क समान था

bull िासति म उनक चार ो ओर की शसकतयाा वजनह न

त ि वनयोवतरत कर सकत थ और न ही समझ पाए

थ उनह वदवयता क साथ वनिवशत वकया गया तथा

उनह मादा या नर दिीदिताओो क रप म

परतीकतव वकया गया था

bull उनह ोन कछ यजञ ो का भी वनषपादन वकया था

Q49) उततर (b)

सपषटीकरण

bull सडक और नदी-मागि (जल-मागि) डकती स

सरवकषत नही ो थ

bull उललखनीय ह वक हिििििन क शासनकाल क

दौरान यआन चिाोग (हयएन साोग) का सारा

सामान लि वलया गया था

Q50) उततर (c)

सपषटीकरण

परशन म वदए गए द न ो कथन सही ह

Q51) उततर (b)

सपषटीकरण

bull परोदर दास एक सोत और भगिान कषण क एक

महान भकत थ

bull परोदर दास क कनाििक सोगीत क वपतामह क

रप म जाना जाता ह

bull यदयवप उनक जनम-थथान क बार म काफी

अिकल लगाई जाती रही ह

bull तथावप अब कननड विशवविदयालय हमपी क दवारा

गवठत एक विशिजञ सवमवत इस वनषकिि पर पहोची

ह वक उनका जनम थथान सोभितया कनाििक का

एक छ िा-सा गााि कषमपरा (वशिम गगा वजला)

था

Q52) उततर (c)

सपषटीकरण

bull शरी तयागराज शरी शयाम शासतरी और शरी मथसवामी

दीवकषतर क कनाििक सोगीत की वतरमवति माना

जाता ह

bull उनक कारण ही 18िी ो-19िी ो शताबदी म कनाििक

सोगीत का सववणिम यग आया था

Q53) उततर (d)

सपषटीकरण

bull अभी हाल ही म लौह यगीन-महापािावणक काल

का 2000 ििि पराना एक दलिभ सारक फगस

(Sarcophagus) (पतथर का ताबत) क ललम क

वियर गाोि (क वयलडी क पास वजला क वझक ड

करल राजय) की एक रॉक-कि गफा स ख जा गया

bull यह ताबत वजसम हविय ो क िकड थ खदाई क

दौरान वमला

bull अभी तक इस परकार की दलिभ ख ज करल क

मातर द ही थथान ो स हई ह

bull य द न ो सारक फगी (Sarcophagi) (पतथर क

ताबत) चियर और अथ ली (वजला क वझक ड) क

महापािाण थथल ो स वमल ह

Q54) उततर (a)

सपषटीकरण

FC19H1003 37

दवकषण भारत म महापािाण सोसकवत एक पणि

विकवसत लौह यगीन सोसकवत थी

Q55) उततर (d)

सपषटीकरण

bull च ल पाणडय और करलपतर (चर) इन तीन ो का

उललख अश क क अवभलख ो म वकया गया ह

bull सोभितः य भौवतक सोसकवत क उततर

महापािावणक चरण म थ

Q56) उततर (d)

सपषटीकरण

bull भीमा-क रगाोि की लडाई ततीय आोगल-मराठा

यद का वहससा थी

Q57) उततर (b)

सपषटीकरण

bull राजकमार शकल न गाोिीजी क चोपारण आन तथा

वतनकवथया परणाली स जडी समसया की जाोच क

वलए रारी करन क वलए दश भर म उनका

अनसरण वकया था

bull बज वकश र राजदर परसाद महादि दसाई और

नरहरी पाररख चोपारण सतयागरह क दौरान गाोिी

जी क सहय गी थ

Q58) उततर (b)

सपषटीकरण

bull बराहमण ो और बौद मठिाररय ो क कर-मकत गााि

अनदान म दन की परथा सतिाहन ो न आरमभ की

थी

Q59) उततर (c)

सपषटीकरण

इस कायिकरम क उददशय वनमनानसार ह

(i) बवनयादी पयििन आिाररक सोरचना का विकास

करना

(ii) चयवनत (पहचान वकय गए) कषतर ो म आजीविका क

सजन क वलए दश क साोसकवतक और विरासत

मलय ो क बिािा दना

(iii) विरासत समारक थथल ो पर विशव सतरीय आिाररक

सोरचना विकवसत करक एक सतत तरीक स

पयििक आकििण म िसद करना

(iv) थथानीय समदाय ो की सवकरय भागीदारी क माधयम

स र रगार ो का सजन करना

(v) र रगार उतपादन और आवथिक विकास क वलए

पयििन कषमता का उन पर परभाि का उपय ग

करना तथा

(vi) िारणीय पयििन आिाररक सोरचना का विकास

करना और उसका उवचत सोचालन तथा

रखरखाि सवनवशचत करना

Q60) उततर (b)

सपषटीकरण

bull यह वनकाय ििि 1987 म अससततव म आया था

bull यह एक राषटर ीय सतर का शीिि सोगठन ह ज भारत

सरकार क जनजातीय मामल ो क मोतरालय क

परशासवनक वनयोतरण क अिीन काम कर रहा ह

bull इसका पोजीकत और परिान कायािलय नई वदलली

म सथथत ह

Q61) उततर (c)

सपषटीकरण

bull परमचोद क उपनयास ो म परमाशरम रोगभवम गबन

कमिभवम और ग दान शावमल ह

bull ग रा रिी ोदरनाथ िग र क दवारा रवचत उपनयास ह

bull अभी हाल ही म मोशी परमचोद की 138िी ो जयोती दश

भर म मनाई गई थी

Q62) उततर (b)

सपषटीकरण

bull ldquoवगदाrdquo पोजाब (भारत) एिो पावकसतान की

मवहलाओो क दवारा तयौहार क समय और फसल

की बिाई तथा किाई क अिसर पर वकया जान

िाला एक पारोपररक दहाती नतय ह

FC19H1003 38

bull इस नतय क माधयम स पोजाबी मवहलाऐो अपनी

परसननता परकि करती ह तथा वगदा क परदशिन क

माधयम स परि िचिसव िाल समाज म मवहलाओो

की दबी हई भािनाओो क परकि करती ह

bull चोवक इस नतय का परि ो क साथ क ई सोबोि नही ो

ह अतः किल मवहलाऐो ही इसम भाग ल सकती

bull हर साल तीज समार ह क दौरान पोजाब म वगदा

नतय वकया जाता ह

तीज भारत क कछ भाग ो म मवहलाओो क दवारा

मनाया जान िाल कई तयौहार ो क वलए एक

वयापक नाम ह

Q63) उततर (a)

सपषटीकरण

- मजम-उल-बहरीन या द समदर ो का सोगम

नामक उललखनीय रचना दारा वशक ह क दवारा

वलखी थी

- भारत क उपराषटर पवत शरी एम िकया नायड न कहा

ह वक राजकमार दारा वशक ह की रचनाएा शाोवत

और सदभाि क बिािा दन क वलए एक तारा सर त

क रप म सामन आ सकती ो ह

- उपराषटर पवत गत ििो क भला वदए गए राजकमार

दारा वशक ह क परदवशित परचवलत करन हत

आय वजत एक परदशिनी का दौरा करन क बाद एक

सभा क सोब वित कर रह थ

- इस परदशिनी का आय जन फर क इस गौवियर

(Francois Gautier) क दवारा lsquoइोवदरा गाोिी नशनल

सिर फॉर द आििसrsquo (The Indira Gandhi

National Centre for the Arts) नई वदलली म

वकया गया था

Q64) उततर (c)

सपषटीकरण

- ग मतशवर परवतमा जन भगिान बाहबली क

समवपित ह

- यह एक एक-चटटानी पतथर की मवति ह

- राषटर पवत राम नाथ क विोद न शरिणबलग ला

(कनाििक) म आय वजत वकय जान िाल भवय

अवभिक समार ह महामसतकावभिक का

उदघािन वकया था

- यह समार ह 12 ििो म एक बार ह ता ह

Q65) उततर (c)

सपषटीकरण

bull पराची घािी पराची नदी क चार ो ओर फली हई थी

bull पराची घािी िीर-िीर विलपत ह गई थी

bull पराची नदी भिनशवर स वनकलती ह

bull यह महानदी की एक सहायक नदी ह और यह

परी खदाि किक तथा जगतवसोहपर वजल ो स

ह कर बहती ह

bull इस नदी क पर कषतर क पराची घािी कहा जाता ह

bull यह नदी बोगाल की खाडी म वगरती ह

परातासतवक साकषय स पता चलता ह वक पराची घािी

सभयता हडपपा और म हनज दाड द न ो की

पिििती ह

Q66) उततर (d)

सपषटीकरण

य समारक छतरपर वजल (मधय परदश) म विोधयाचल

पिित शरोखला म सथथत ह

Q67) उततर (a)

सपषटीकरण

bull थॉिस ऑन पावकसतान नामक पसतक डॉ बी

आर अमबडकर न वलखी थी

bull डॉ बी आर अमबडकर की जयोती क अिसर पर

भारत क राषटर पवत न भारत की इस महान हसती

क शरदाोजवल अवपित की थी

bull डॉ बी आर अमबडकर न 1924 म वडपरथड

कलावसर इोसटीटयि (दवलत िगि सोथथान -

बवहषकत वहतकाररणी सभा) और 1927 म समाज

समता सोघ की थथापना की थी

bull अमबडकर का धयान वशकषा कषतर की ओर भी था

bull उनह ोन वशकषा क वनमन िगो म फलान क वलए

पीपलस एजकशन स साइिी (The Peoples

Education Society) क नाम स महाविदयालय ो क

नििकि और छातरािास ो की थथापना की थी

FC19H1003 39

Q68) उततर (b)

सपषटीकरण

bull महरगि भारतीय उपमहादवीप म एक परवसद

निपािाण बसती ह ज बलवचसतान पराोत

पावकसतान म सथथत ह

bull दचपलली (आोधर परदश) क पास नागलर नदी क

पिी ति ो पर चना पतथर क बलॉक क विशाल

विसतार म एक पिि-ऐवतहावसक रॉक आिि थथल की

ख ज की गई ह

bull इसन 1500-2000 ईसा पिि क दौरान गोिर (आोधर

परदश) म विकवसत निपािाण सभयता पर परकाश

डाला ह

Q69) उततर (c)

सपषटीकरण

bull 12िी ो सदी और 13िी ो सदी म काकाविय िोश का

उदय हआ था

bull ि पहल कलयाण क पवशचमी चालकय ो क सामोत थ

bull परारोभ म उनह ोन िारोगल (तलोगाना) क पास एक

छ ि स कषतर पर शासन वकया था

bull उनह ोन ldquoनायक वयिथथाrdquo की शरआत की थी

वजस बाद म विजयनगर क राय शासक ो न

अपनाया और विकवसत वकया था

Q70) उततर (a)

सपषटीकरण

bull गाोिीजी क अनशन स वमल मावलक ो पर दबाि

पडा था ज अोततः शरवमक ो क ितन म 35 परवतशत

की िसद करन क वलए सहमत हए थ

bull गगल (Google) न अनसया साराभाई वजनह ोन

भारत क शरवमक आोद लन म एक अगरणी भवमका

वनभाई थी की 132िी ो जयोती डडल (Doodle) का

वनमािण करक मनाई

Q71) उततर (d)

सपषटीकरण

भारत स यनसक की मानिता की अमति साोसकवतक

विरासत की परवतवनवि सची म वनमनवलसखत शावमल ह

bull कवडयटटम करल का सोसकत रोगमोच

bull मवडयिि करल का अनषठान रोगमोच और नतय

नाविका

bull िवदक मि जाप की परोपरा

bull राजथथान क कालबवलया ल क गीत और नतय

bull रामलीला रामायण का पारोपररक परदशिन

bull सोकीतिन मवणपर का अनषठान गायन ढ ल िादन

और नतय

bull रममन भारत क गििाल वहमालय का िावमिक

तयौहार और अनषठान रोगमोच

bull जाोदीयाला गर पोजाब क ठठर ो की पीतल और

ताोब क वशलप स वनवमित बतिन ो की पारोपररक कला

bull छाऊ नतय पिी भारतीय राजय ो म जनमी शासतरीय

भारतीय नतय कला

bull लददाख का बौद मि जाप िर ाोस-वहमालयी लददाख

कषतर तथा जमम-कशमीर म पवितर बौद गरोथ ो का पाठ

bull य ग

bull नौर र

bull को भ मला

Q72) उततर (b)

सपषटीकरण

bull भारत क राषटर पवत शरी राम नाथ क विोद न

वकसामा नागालड म हॉनिवबल मह रति और

राजय गठन वदिस समार ह का उदघािन वकया

था

bull हॉनिवबल मह रति का नाम भारतीय हॉनिवबल क

नाम पर पडा ह ज एक विशाल और रोगीन जोगली

पकषी ह

bull यह पकषी नागालड राजय की अविकतर जनजावतय ो

की ल ककथाओो म उसललसखत ह

bull नागालड की परमख मानयता परापत जनजावतयाा ह

अोगामी आओ चखसोग चाोग ककी रगमा और

रवलोग आवद

bull ओोग जारिा और ससिनलीस अोडमान-वनक बार

दवीप समह की जनजावतयाा ह

FC19H1003 40

Q73) उततर (c)

सपषटीकरण

bull दकन म राषटर कि शासन दसिी ो सदी क अोत तक

लगभग 200 ििो तक रहा था

bull राषटर कि शासक अपन िावमिक विचार ो म सवहषण

bull उनह ोन न किल शि िमि और िषणि िमि बसलक

जन िमि क भी सोरकषण वदया था

bull एल रा म वशि क परवसद रॉक कि मोवदर का

वनमािण नौिी ो सदी म राषटर कि राजा कषण परथम न

करिाया था

bull उसका उततराविकारी अम घििि जन था लवकन

उसन अनय िमो क भी सोरकषण परदान वकया था

bull राषटर कि ो न मसलमान वयापाररय ो क बसन की

अनमवत दी थी

bull उनह न अपन अविराजय ो म इसलाम क उपदश दन

की भी अनमवत दी थी

bull अभी हाल ही म पाोडिलागटटा (तलोगाना) क

परागवतहावसक चटटान वचतर ो क कषरण की बिती हई

घिनाएा एक गोभीर वचोता का वििय ह

bull यह परागवतहावसक चटटान क नकसान पहाचा

सकता ह

bull पाोडिलागटटा वनमनवलसखत क वलए जाना जाता ह

- 10000 ईसा पिि स 8000 ईसा पिि क वचवतरत

चटटानी आशरय ो क वलए

- राषटर कि काल क एक 8 िी ो सदी क

वशलालख क वलए और

- 12िी ो सदी क काकविय सामराजय क वभवतत

वचतर ो क वलए

Q74) उततर (b)

सपषटीकरण

bull 1828 म राजा राम म हन रॉय न एक नय िावमिक

समाज बरहम सभा की थथापना की थी वजस बाद

म बरहम समाज क नाम स जाना गया था

bull दिदरनाथ िग र न ततवब विनी सभा की अधयकषता

की थी ज आधयासिक सतय की ख ज म सोलि

थी

bull इसका उददशय वहोद िमि क शद करन का और

एकशवरिाद (एक ईशवर म आथथा) का परचार करना

था

bull नय समाज की थथापना क आिार थ कारण

(तकि ) क द सतमभ तथा िद और उपवनिद

bull अभी हाल ही म सािारण बरहम समाज का कछ

काननी मदद ो क लकर पवशचम बोगाल सरकार क

साथ काननी वििाद चल रहा ह

Q75) उततर (c)

सपषटीकरण

bull भारत म वचशती वसलवसल की थथापना खवाजा

म इनददीन वचशती क दवारा की गयी थी

bull ि 1192 ईसवी क आसपास भारत आय थ

bull वचशतीय ो क बारहिी ो शताबदी क उततरािि म भारत

म आन िाल सफीय ो क समह ो म सबस

परभािशाली माना जाता ह

bull उनह ोन थथानीय िातािरण क साथ सफलतापििक

अनकलन वकया और उनह ोन भारतीय भसकत

परोपराओो क कई पहलओो क अपनाया

bull अजमर म सफी अपरकि खवाजा म इनददीन वचशती

की ऐवतहावसक दरगाह क एक नया रप दन की

तयारी की जा रही ह

bull इस 13िी ो शताबदी की दरगाह क ldquoसवचछ

आइकॉवनक थथल ोrdquo (Swacch Iconic Places) म

शावमल वकया गया ह ज परवतवषठत विरासत

आधयासिक और साोसकवतक थथान ो पर क वदरत

य जना ह

FC19H1003 41

ANSWERS amp EXPLANATION OF

NCERT History Class VI-X + Current Affairs

(FC19E1003)

Q1) Answer c

Explanation

Rigveda consists of more than a

thousand hymns dedicated to gods and

goddesses These hymns were

composed by sages and learnt by men

however a few were composed by

women like Apala Ghosa Lopamudra

Maitreyi and Gargi

Rigveda consists of many hymns in the

form of dialogues We get an example of

a dialogue between a sage named

Vishwamitra and two rivers (Beas and

Sutlej) that were worshipped as

goddesses This suggests that he

belonged to the Vedic period

Q2) Answer b

Explanation

Traces of ash have been found from

Kurnool Caves suggesting that people

were familiar with the use of fire

It is situated in Andhra Pradesh

Q3) Answer c

Explanation

Burzahom is a prehistoric site in

present day Kashmir where people built

pit houses which were dug into the

ground with steps leading into them

These may have provided shelter in cold

weather

Q4) Answer c

Explanation

Epigraphy is defined as the study of

inscriptions

Manuscriptology is the study of history

and literature through the use of hand

written documents

Palaeography refers to the study of

ancient writing systems and the

deciphering and dating of historical

manuscripts

Numismatics refers to the study of

coins

Q5) Answer a

Explanation

Charaka Samhita was written by

Charaka and is an important book on

Ayurveda and medicine

He was a practitioner of the traditional

system of Indian medicine known as

Ayurveda

Charaka is thought to have flourished

sometime between the 2nd century BCE

and the 2nd century CE

Q6) Answer b

Explanation

Bhaga refers to the tax on crops which

was fixed at 16th of the production

Kammakaras is the term used for the

landless agricultural labour class

Ashvamedha also known as horse

sacrifice is a ritual where a horse is let

loose to wander freely and it was

guarded by the rajarsquos men

Q7) Answer (d)

Explanation

In the Rigvedic period horses were

yoked to chariots that were used in

battles fought to capture land cattle

etc This suggests that the use of horse

chariots began much before the period

of Mahajanapadas

The battles were fought in the Rigvedic

period for cattlersquos lands water an even

to capture people Most men took part

in these wars however there was no

regular army but there were assemblies

where people met and discussed

matters of war Regular armies became

a feature in the Mjahajanapada period

including vast armies of foot soldiers

chariots and elephants

RAUSIAS-FC19E1003 42

Q8) Answer (a)

Explanation

Buddha belonged to the Sakya clan and

passed away at Kusinara

Buddha taught in Prakrit which was the

common language of people

Q9) Answer c

Explanation

There were six schools of philosophy in

ancient India These are known as

Vaishesika Nyaya Samkhya Yoga

Purva Mimansa and Vedanata or Uttara

Mimansa They were founded by sages

Kanada Gautama Kapila Patanjali

Jamini and Vyasa respectively

Q10) Answer b

Explanation

The teachings of Mahavira were

compiled at Valabhi in 6th century AD

Q11) Answer (c)

Explanation

Chanakya is traditionally identified as

Kautilya or Vishnugupta who authored

the ancient Indian political treatise the

Arthashastra

Q12) Answer d

The national emblem of India is an

adaptation of the Lion Capital atop the

Ashoka Pillar of Sarnath Uttar Pradesh

and is combined with the National

Motto Satyameva Jayate

The Rampurva Bull gets the name from

the site of its discovery Rampurva in

Bihar

It is noted for its delicately sculpted

model demonstrating superior

representation of soft flesh sensitive

nostrils alert ears and strong legs It is

a mixture of Indian and Persian

elements

Sankissa is situated in Uttar Pradesh

India

Q13) Ans(a)

Kunwar Singh was a notable leader during the Revolt of 1857 He belonged

to a royal house of Jagdispur Bihar

Q14) Answer b

Explanation

The term Vellalar was used for large

landowners

Q15) Answer c

Explanation

Arikamedu was a coastal settlement

where ships unloaded goods from

distant lands Finds here include a

massive brick warehouse pottery

including amphorae and Arretine ware

Roman lamps glassware and gems have

also been found at the site

Q16) Answer a

Explanation

Muvendar is a Tamil word mentioned in

Sangam poems meaning three chiefs

used for the heads of three ruling

families the Cholas Cheras and

Pandyas

Q17) Ans (c)

Several tribal or kin-based assemblies

such as the Sabha Vidatha and gana

are mentioned in the Rig-veda The

Sabha and the samiti mattered a great

deal in early Vedic times so much so

that the chiefs or the kings showed an

eagerness to win their support

Q18) Ans (a)

Jainism recognised the existence of the

gods but placed them lower than the

jina and did not condemn the varna

system as Buddhism did

Q19) Answer (d)

Explanation

Cholas and Pandyas had developed

powerful coastal cities The most

important city of Cholas was Puhar or

Kaveripattinam and Madurai was the

capital of Pandyas

Q20) Answer b

Explanation

Buddhacharita is the biography of

Buddha and was written by

RAUSIAS-FC19E1003 43

Ashvaghosha

Q21) Answer (a)

Explanation

Tamil poet Appar was a Shiva devotee

So he was a Nayanar saint

Q22) Answer d

Explanation

Samudragupta was a prominent Gupta

ruler whose coins depict him playing a

veena indicating his love for music We

get important historic information from

his Allahabad Prashasti which was

composed by his court poet Harisena

Q23) Answer (b)

Explanation

Vikrama Samvat was founded by

Chandragupta II in the 58 BC as a

mark of victory over the Shakas and

assumed the title of Vikramaditya

Banabhatta wrote Harshavardhanarsquos

biography the Harshacharita in

Sanskrit

Q24) Answer c

Explanation

Sandhi-vigrahika was the minister of

war and peace

Sarthavaha was the leader of the

merchant caravans

Q25) Answer a

Explanation

Xuan Zang (Hsuan-tsang) was a

Chinese traveller who came during the

reign of Harshavardhana

In the decade that began in 630 AD

Xuan Zang came to India through

Kashmir after visiting Central Asia Iran

and Afghanistan

He travelled from north to east and lived

in Bihar for a couple of years

At Nalanda University Xuan Zang

interacted with students and scholars

mastered local languages and

discovered Buddhist stupas

Q26) Answer c

Explanation

Pradakshina patha is a circular path

laid around a stupa in Buddhist

architecture While the rest are a part of

temple architecture

Q27) Answer d

Explanation

All the above-mentioned temples have

an elaborate use of bricks (baked

bricks) along with stone

Q28) Ans (c)

Muhammad Quli Qutab was the Sultan

of Golconda He was a contemporary of

Akbar was very fond of literature and

architecture

The Sultan was a great poet and he

wrote in Dakhini Urdu Persian and

Telgu and has left an extensive diwan or

collection

Recently the Archaeological Survey of

India (ASI) will be using Ground

Penetrating Radar (GPR) to map the

contours of the area around the Bagh-e-

Naya Qila excavated garden inside the

Golconda Fort in Telangana

Q29) Answer a

Explanation

Silappadikaram is a famous Tamil epic

which was written by Ilango around

1800 years ago It is a story of a

merchant named Kovalan who fell in

love with a courtesan named Madhavi

Manimekalai tells the story of the

daughter of Kovalan and Madhavi

Q30) Answer (a)

Explanation

Charaka is the author of Charaka

Samhita which is an important work of

Ayurveda and medicines

Brahmaguptarsquos fame rests mostly on his

Brahma-sphuta-siddhanta which was

an astronomical work It was translated

into Arabic in Baghdad and had a major

impact on Islamic mathematics and

astronomy

Late in his life Brahmagupta wrote

Khandakhadyaka which was an

RAUSIAS-FC19E1003 44

astronomical handbook that employed

Aryabhatarsquos system of starting each day

at midnight

Q31) Answer (c)

Explanation

Amir Khusrau was a famous sufi

musician poet and scholar In 1318 he

noted that there was different language

in every region of this land (Hindustan)

Lahori Kashmiri Dvarsamudri (in

Southern Karnataka) Telangana (in

Andhra Pradesh) Gujari (in Gujarat)

Marsquobari (in Tamil Nadu) Awadhi (in

eastern Uttar Pradesh) and Hindawai (in

the area around in Delhi) etc He went

to explain that Sanskrit did not belong

to any region and that only brahmans

knew it

Q32) Answer c

Explanation

Hiranyagarbha refers to the golden

womb When this ritual was performed

with the help of Brahmanas it was

thought to lead to the rebirth of the

sacrificer as a Khastriya

Q33) Answer d

Explanation

Kadamai refers to a tax on land

revenue

Gwalior Prashasti describes the exploits

of Nagabhata who was a Pratihara king

Q34) Answer b

Explanation

Rajatarangini is a Sanskrit text written

by Kalhana in the 12th century

It was historical chronicle of early India

It is justifiably considered to be the best

and most authentic work of its kind

It covers the entire span of history in

the Kashmir region from the earliest

times to the date of its composition

Q35) Answer c

Explanation

ldquoUrrdquo was the general assembly of the

village ldquoUrrdquo consisted of all the

taxpaying residents of an ordinary

village

Q36) Answer (a)

Explanation

Tarikh was a form of history writing in

the Delhi Sultanate The authors of

tawarikhs were learned men which

included secretaries administrators etc

Q37 Answer (a)

Explanation

Alauddin chose to pay his soldiers salaries in cash rather than iqtas The soldiers would buy their supplies from merchants in Delhi and it was thus feared that merchants would raise their prices To stop this Alauddin controlled the prices of goods in Delhi Prices were carefully surveyed by officers and merchants who did not sell at the prescribed rates were punished

Q38) Answer (d)

Explanation

Delhi first became the capital of a

kingdom under the Tomara Rajputs

who were defeated in the middle of the

twelfth century by the Chauhans (also

referred to as Chahamanas) of Ajmer

It was under the Tomaras and

Chauhans that Delhi became an

important commercial centre Many rich

Jaina merchants lived in the city and

constructed several temples Coins

minted here called dehliwal had a wide

circulation

Q39) Answer (c)

Explanation

Moth ki Masjid was built in the reign of

Sikandar Lodi by his minister

Begumpuri mosque built in the reign of

Muhammad Tughluq was the main

mosque of Jahanpanah the ldquoSanctuary

of the Worldrdquo and his new capital in

Delhi

Quwwat al ndash Islam mosque was

enlarged by Iltutmish and Alauddin

Khalji The minar was built by three

Sultansndash Qutbuddin Aybak Iltutmish

and Firuz Shah Tughluq

RAUSIAS-FC19E1003 45

Q40) Answer (c)

Explanation

Under the Mughals mansabdar was

referred to an individual who held a

mansab ie rank and he received his

salary as revenue assignments called

jagirs

Q41) Ans (b)

The Quit India Movement was a

spontaneous revolt of people against

British rule

The All India Congress Committee met

at Bombay on 8 August 1942 It passed

the famous resolution Quit India and

proposed the starting of a non-violent

mass struggle under Gandhis

leadership to achieve this aim But on

the very next day Gandhi and other

eminent leaders of the Congress were

arrested The Congress was once again

declared illegal

Q42) Ans (c)

The Simon Commission refers to a

group of seven MPs from the United

Kingdom constituted to suggest

constitutional reforms for British India

The Commission consisted of only

British members headed by one of the

senior British politicians Sir John

Simon

So the people of India agitated against

the arrival of Simon Commission

Q43) Ans (a)

He was widely known for his

unfavourable opinion of the economic

consequences of the British rule in

India

In his many writings and speeches and

especially in Poverty and Un-British

Rule in India Naoroji argued that India

was too highly taxed and that its wealth

was being drained away to England

He did not interpret the ancient Indian

texts and restored the self-confidence of

Indians And also he did not stress the

need for eradication of all the social

evils before anything else

Q44) Ans (c)

In August 1932 Prime Minister

MacDonald announced his Communal

Award Great Britainrsquos unilateral

attempt to resolve the various conflicts

among Indiarsquos many communal

interests

The award which was later

incorporated into the act of 1935

expanded the separate-electorate

formula reserved for Muslims to other

minorities including Sikhs Indian

Christians Anglo-Indians Europeans

distinct regional groups Gandhi

undertook a ldquofast unto deathrdquo against

that offer which he viewed as a

nefarious British plot to divide the

Indian society

Q45) Ans (b)

In British India apart from existing

imports and exports there was also a

particular amount of money which

colonial India contributed towards

administration maintenance of the

army war expenses pensions to retired

officers and other expenses accrued by

Britain towards maintenance of her

colony These were known as Home

charges and were paid for almost

entirely by India

The Home charges was made of

following components-

- Interest payable on Indian debt

- Dividend to shareholders of East

India Company

- Funds used to support the India

Office in London

- Funds used to pay salaries and

pensions of British personnel

engaged in India

- Interest on the railways

- Civil and military charges

- Store purchases in England

Q46) Ans (b)

The Lahore session of the Indian

National Congress was held in 1929

under the Presidentship of Jawaharlal

Nehru

The Lahore session of the Indian

National Congress witnessed significant

RAUSIAS-FC19E1003 46

developments in the Indian national

movement

- First the election of Jawaharlal

Nehru to the post of Presidentship of

the Congress was a clear indication

of the growing strength of the

Leftists in the Congress

- Secondly it was in this session that

the Congress for the first time raised

the demand for complete

independence Such demand was

not raised from the Congress

platform earlier

Q47) Ans (b)

It did not provide for separate

electorates for any community or

weightage for minorities However it did

allow for the reservation of minority

seats in provinces having minorities of

at least ten per cent but this was to be

in strict proportion to the size of the

community

There was no provision for complete

Independence for India

Q48) Ans (c)

The religion of early Vedic Aryans was

primarily of worship of nature and

Yajnas

The early Aryan religion was kind of

nature worship Actually the forces

around them which they could not

control or understand were invested

with divinity and were personified as

male or female gods And they

performed some Yajnas also

Q49) Ans (b)

The roads and river-routes were not

immune from robbery It is notable that

Yuan Chwang (Hiuen Tsang) was

robbed of his belongings during

Harshvardanarsquos period

Q50) Ans (c)

Q51) Ans (b)

Purandara Dasa was a saint and great

devotee of Lord Krishna

There is much speculation about where

Purandara Dasa regarded as the

Pitamaha of Carnatic music was born

Recently an expert committee

constituted by the Kannada University

Hampi has come to the conclusion that

Kshemapura Shivamogga district

Karnataka is the birth place of

Purandara Dasa

Q52) Ans (c)

Sri Tyagaraja Sri Shyama Shastry and Sri Muthuswami Dikshitar are considered the trinity of Carnatic music and with them came the golden age in Carnatic music in the 18th-19th

century

Q53) Ans d)

Recently a rare sarcophagus (stone

coffin) which is 2000 years old from the

Iron AgendashMegalithic era was discovered

from a rock-cut cave at Viyur village of

Kollam near Koyilandy in Kozhikode

district Kerala

The coffin containing bone fragments

was found during an excavation ldquoSo

far such a rare finding has been

discovered only from two sites

in Kerala Both these sarcophagi were

recovered from Megalithic sites at

Chevayur and Atholi also in Kozhikode

district

Q54) Ans a)

The megalithic culture in South India was a full-fledged Iron Age culture

Q55) Ans d)

The Cholas Pandyas and Keralaputras

(Cheras) mentioned in Ashokan

inscriptions were probably in the late

megalithic phase of material culture

Q56) Ans d)

Q57) Ans (b)

Raj Kumar Shukla followed Gandhiji all

over the country to persuade him to

come to Champaran to investigate the

problem associated with tinkathia

system

RAUSIAS-FC19E1003 47

Brij Kishore Rajendra Prasad Mahadev

Desai and Narhari Parikh accompanied

Gandhi ji during the Champaran

Satyagraha

Q58) Ans (b)

The Satvahanas started the practice of granting tax-free villages to brahmanas and Buddhist monks

Q59) Ans c)

The objectives of the Programme are

listed as under

- Developing basic tourism

infrastructure

- Promoting cultural and heritage

value of the country to generate

livelihoods in the identified regions

- Enhancing the tourist attractiveness

in a sustainable manner by

developing world-class

infrastructure at the heritage

monument sites

- Creating employment through active

involvement of local communities

- Harnessing tourism potential for its

effects on employment generation

and economic development

- Developing sustainable tourism

infrastructure and ensuring proper

Operations and maintenance

therein

Q60) Ans (b)

The Tribal Cooperative Marketing

Development Federation of India

(TRIFED) came into existence in 1987

It is a national-level apex organization

functioning under the administrative

control of Ministry of Tribal Affairs

Govt of India

TRIFED has its registered and Head

Office located in New Delhi

Q61) Ans (c)

Premchandrsquos novels include

Premashram Rangabhumi Ghaban

Karmabhumi and Godan

Gora is a novel written by Rabindranath

Tagore

138th birth anniversary of Munshi

Premchand was celebrated across the

country

Q62) Ans (b)

Giddha is a traditional pastoral dance

performed by the women of the Punjab

India and Pakistan at festival times

and at the sowing and reaping of the

harvest

By this dance the Punjabi women

reveal their joy expel their suppressed

feelings in a male dominated society

through the performance of Giddha

Since this dance has nothing to do with

men only women can participate in it

During the Teej celebrations Giddha

dance is celebrated in Punjab every

year Teej is a generic name for a

number of festivals that are celebrated

by women in some parts of India

Q63) Ans (a)

Dara Shukoh wrote the remarkable

work called ldquoMajma-ul-Bahrainrdquo or the

ldquoThe confluence of two seasrdquo

The Vice President of India Shri M

Venkaiah Naidu has said that Prince

Dara Shukohrsquos writings can come as a

refreshing source for infusing peace and

harmony He was addressing the

gathering after visiting the exhibition

that showcases the forgotten Prince of

yesteryears Dara Shukoh organized by

Mr Francois Gautier at Indira Gandhi

National Centre for the Arts in New

Delhi

Q64) Ans (c)

The statue Gommateshwara is

dedicated to the Jain God Bahubali

It is a monolithic statue

President Ram Nath Kovind

inaugurated the grand anointing

ceremony mdash Mahamastakabhisheka mdash

held once in 12 years at

Shravanabelagola (Karnataka)

Q65) Ans (c)

Prachi Valley had come up around the

Prachi river Prachi Valley gradually

disappeared

RAUSIAS-FC19E1003 48

The Prachi river originates from

Bhubaneswar

It is a tributary of the Mahanadi and

flows through the districts of Puri

Khurda Cuttack and Jagatsinghpur

and the entire region of the river is

termed as the Prachi Valley

It falls into the Bay of Bengal

Archaeological evidence shows that the

Prachi Valley Civilisation predates both

Harappa and Mohenjo-Daro

The Prachi river originates from

Bhubaneswar

Q66) Ans (d)

These monuments are located in

Chhatarpur district Madhya Pradesh

within Vindhya mountain range

Q67) Ans (a)

The book lsquoThoughts on Pakistanrsquo was

written by Dr BR Ambedkar

On the occasion of the birth anniversary

of Dr BR Ambedkar the president of

India pays homage to this icon of India

In 1924 he founded the Depressed

Classes Institute (Bahishkrit Hitkarini

Sabha) and in 1927 the Samaj Samata

Sangh

Another area of attention for Ambedkar

was education For its spread among

the low classes he set up a network of

colleges by the name of Peoples

Education Society and founded hostels

Q68) Ans(b)

Mehrgarh is a famous Neolithic

settlement in the Indian subcontinent

which is situated in Baluchistan

province Pakistan

A pre-historic rock art site is discovered

in the vast expanse of limestone blocks

on the eastern banks of Naguleru river

near Dachepalli (Andhra Pradesh) It

has thrown light on the Neolithic

civilisation that flourished in Guntur

(Andhra Pradesh) during 1500-2000

BC

Q69) Ans (c)

The 12th and the 13th centuries saw

the emergence of the Kakatiyas They

were at first the feudatories of the

Western Chalukyas of Kalyana Initially

they ruled over a small territory near

Warangal (Telangana)

They introduced Nayakships which was

later adopted and developed by the

Rayas of Vijayanagara

Q70) Ans (a)

The fast had effect of putting pressure

on mill owners who finally agreed to

give the workers a 35 per cent increase

in wages

Google celebrated with a doodle the

132nd birth anniversary of Anasuya

Sarabhai who played a pioneering role

in Indiarsquos labour movement

Q71) Ans (d)

The UNESCOrsquos list of the representative

list of the intangible cultural heritage of

humanity from India are

- Koodiyattam Sanskrit Theatre of

Kerala

- Mudiyettu ritual theatre and dance

drama of Kerala

- Tradition of Vedic Chanting

- Kalbelia folk songs and dances of

Rajasthan

- Ramlila Traditional Performance of

the Ramayana

- Sankirtana ritual singing

drumming and dancing of Manipur

- Ramman religious festival and

ritual theatre of the Garhwal

Himalayas India

- Traditional brass and copper craft of

utensil making among the Thatheras

of Jandiala Guru Punjab India

- Chhau dance classical Indian dance

originated in the eastern Indian

states

- Buddhist chanting of Ladakh

recitation of sacred Buddhist texts

in the trans-Himalayan Ladakh

region Jammu and Kashmir India

- Yoga

- Nouroz

- Kumbh Mela

RAUSIAS-FC19E1003 49

Q72) Ans(b)

The President of India Shri Ram Nath Kovind inaugurated the Hornbill Festival and State Formation Day celebrations of Nagaland in Kisama

The festival is named after the Indian hornbill the large and colourful forest bird which is displayed in the folklore of most of the states tribes

The major recognized tribes of Nagaland are Angami Ao Chakhesang Chang

Kuki Rengma and Zeling etc

Onge Jarawa and Sentinelese are the

tribes of Andman amp Nicobar Islands

Q73) Ans (c)

The Rashtrakutas rule in the Deccan lasted for almost two hundred years till the end of the tenth century The Rashtrakutas rulers were tolerant in their religious views and patronized not only Shaivism and Vaishnavism but

Jainism as well

The famous rock-cut temple of Shiva at Ellora was built by one of the Rashtrakutas kings Krishna I in the ninth century His successor Amoghavarsha was a Jain but he also

patronized other faiths

The Rashtrakutas allowed Muslims traders to settle and permitted Islam to

be preached in their dominions

Recently increasing defacement at the prehistoric rock paintings of Pandavulagutta Telangana has created a cause for grave concern It can spoil

the prehistoric rock

Pandavulagutta is home to

- Painted rock shelters dating to

10000 BC-8000 BC

- An 8th century inscription of the

Rashtrakuta period and

- Painted frescoes from the 12th century Kakatiya empire

Q74) Ans (b)

In 1828 Raja Ram Mohan Roy founded a new religious society the Brahma Sabha later known as the Brahmo

Samaj

Debendranath Tagore headed the Tattvabodhini Sabha which was

engaged in search of spiritual truth

Its purpose was to purify Hinduism and to preach monotheism or belief in one God

The new society was to be based on the twin pillars of reason and the Vedas and

Upanishads

Recently Sadharan Brahmo Samaj (SBS) has entered into a legal battle with the West Bengal government due

to some legal issue

Q75) Ans (c)

The Chishti order was established in India by Khwaja Moinuddin Chishti who came to India around 1192 The Chishtirsquos are considered to be the most influential of the groups of Sufis who migrated to India in the late twelfth century They adapted successfully to the local environment and adopted several features of Indian devotional

traditions

The historical dargah of Sufi mystic Khwaja Moinuddin Chishti in Ajmer is all set to get a facelift This 13 th century dargah has been included among the Swachh Iconic Places a clean-up initiative focused on iconic

heritage spiritual and cultural places

Page 24: GENERAL STUDIES (PAPER I) · Test is part of Rau’s IAS Test series for Preliminary Exam 2019 FOUNDATION + CURRENT AFFAIRS GENERAL STUDIES (PAPER –I) FOUNDATION TEST –III TOPIC:

RAUSIAS-FC19E1003 24

Q64) निमननिखित कथि ो पर निचार कीनजए

1 ग मतशवर परनतमा निोधयनगरी पहाड़ी पर खसथत ह

2 शरिरबिग िा िह सथाि ह जहाो मौयण िोश क

सोसथापक चोदरगपत मौयण अपि नसोहासि क

तयागि क बाद जि तपसवी बि गए थ

उपयणकत कथि ो म स कौि-सास सही हह

(a) किि 1

(b) किि 2

(c) 1 और 2 द ि ो

(d) ि त 1 ि ही 2

Q65) निमननिखित कथि ो पर निचार कीनजए

1 पराताखतवक साकषय स पता चिता ह नक पराची

घाटी सभयता हड़पपा और म हिज दाड़ द ि ो

की पिणिती ह

2 पराची िदी भििशवर स निकिती ह

उपयणकत कथि ो म स कौि-सास सही हह

(a) किि 1

(b) किि 2

(c) 1 और 2 द ि ो

(d) ि त 1 ि ही 2

Q66) निमननिखित कथि ो म स कौि-सास सही हह

1 िजराह क समारक ो क समह का निमाणर

चोदि राजिोश क शासिकाि क दौराि हआ

था

2 य समारक हररिोदर पिणत शरोििा म खसथत ह

3 म रक क यातरी इबन बतता ि अपि सोसमरर ो

म िजराह क मोनदर ो की यातरा का उललि

नकया था तथा इन काजराण िाम स समब नरत

नकया था

िीच नदए गए कट का परय ग कर सही उततर चनिए

(a) किि 1

(b) किि 1 और 2

(c) किि 2 और 3

(d) किि 1 और 3

Q67) निमननिखित कथि ो म स कौि-सास सही हह

1 डॉ बी आर अमबडकर ि दी एनिनहिशि

ऑफ़ कासट (The Annihilation of Caste)

नििी थी नजसम उन ोि नहोद रमण म िोशािगत

पजारी की परथा क उनमिि की आिशयकता

पर बि नदया था

2 डॉ राजदर परसाद ि थॉटस ऑि पानकसताि

(Thoughts on Pakistan) िामक पसतक

नििी थी

िीच नदए गए कट का परय ग कर सही उततर चनिए

(a) किि 1

(b) किि 2

(c) 1 और 2 द ि ो

(d) ि त 1 ि ही 2

Q68) निमननिखित कथि ो म स कौि-सास सही हह

1 महरगढ़ भारतीय उपमहादवीप म एक परनसदध

ििपािार बसती ह ज नसोर पराोत पानकसताि म

खसथत ह

2 बरणह म म कतत ो क उिक सवामी क साथ कबर ो

म दफिाया जाता था

िीच नदए गए कट का परय ग कर सही उततर चनिए

(a) किि 1

(b) किि 2

(c) 1 और 2 द ि ो

(d) ि त 1 ि ही 2

Q69) निमननिखित कथि ो म स कौि-सास सही हह

1 काकानटय मोनदर अनरकतर नशि क समनपणत

2 हिमक ोडा म हजार-सतोभ िाि मोनदर (The

Thousand-Pillared Temple) का निमाणर

काकानटय समराट रदर ि करिाया था

िीच नदए गए कट का परय ग कर सही उततर चनिए

(a) किि 1

(b) किि 2

(c) 1 और 2 द ि ो

(d) ि त 1 ि ही 2

RAUSIAS-FC19E1003 25

Q64) Consider the following statements

1 Gommateshwara Statue is located

on the Vindyagiri Hill

2 Shravanabelagola is the place

where Chandragupta Maurya the

founder of the Mauryan dynasty

became a Jain ascetic after

relinquishing his throne

Which of the statements given above

isare correct

(a) 1 only

(b) 2 only

(c) Both 1 and 2

(d) Neither 1 nor 2

Q65) Consider the following statements

1 Archaeological evidence shows

that the Prachi Valley Civilisation

predates both Harappa and

Mohenjo-Daro

2 The Prachi river originates from

Bhubaneswar

Which of the statements given above

isare correct

(a) 1 only

(b) 2 only

(c) Both 1 and 2

(d) Neither 1 nor 2

Q66) Which of the following statements

isare correct

1 The Khajuraho group of

monuments was built during the

rule of the Chandela dynasty

2 These monuments are located in

Harischandra mountain range

3 Ibn Battuta the Moroccan

traveller in his memoirs mentioned

visiting Khajuraho temples and

called them Kajarra

Select the correct answer using the code

given below

(a) 1 only

(b) 1 and 2

(c) 2 and 3

(d) 1 and 3

Q67) Which of the following statements

isare correct

1 Dr BR Ambedkar wrote the

Annihilation of Caste emphasising

the need to do away with the

practice of hereditary priesthood in

Hinduism

2 The book lsquoThoughts on Pakistanrsquo

was written by Dr Rajendra

Prasad

Select the correct answer using the code

given below

(a) 1 only

(b) 2 only

(c) Both 1 and 2

(d) Neither 1 nor 2

Q68) Which of the following statements

isare correct

1 Mehrgarh is a famous Neolithic

settlement in the Indian

subcontinent which is situated in

Sindh province Pakistan

2 At Burzahom dogs were buried

with their masters in their graves

Select the correct answer using the code

given below

(a) 1 only

(b) 2 only

(c) Both 1 and 2

(d) Neither 1 nor 2

Q69) Which of the following statements

isare correct

1 The Kakatiya temples are

dedicated mostly to Siva

2 The Thousand-Pillared Temple at

Hanamkonda was built by the

Kakatiya king Rudra

Select the correct answer using the code

given below

(a) 1 only

(b) 2 only

(c) Both 1 and 2

(d) Neither 1 nor 2

RAUSIAS-FC19E1003 26

Q70) निमननिखित कथि ो म स कौि-सास सही हह

1 अहमदाबाद नमि हड़ताि क दौराि महातमा

गाोरी ि शरनमक ो क पकष क मजबत करि क

निए आमरर अिशि नकया था

2 अिशि स नमि मानिक ो पर दबाि पड़ा था ज

अोततः शरनमक ो क िति म 15 परनतशत की िखदध

करि क निए सहमत हए थ

िीच नदए गए कट का परय ग कर सही उततर चनिए

(a) किि 1

(b) किि 2

(c) 1 और 2 द ि ो

(d) ि त 1 ि ही 2

Q71) निमननिखित म स नकसक नकिक भारत स यिसक

की माििता की अमतण साोसकनतक निरासत की

परनतनिनर सची (The UNESCOrsquos List of the

Representative List of the Intangible

Cultural Heritage of Humanity) म शानमि

नकया गया ह

1 मनडयटट

2 सोकीतणि

3 को भ मिा

िीच नदए गए कट का परय ग कर सही उततर चनिए

(a) किि 1 और 2

(b) किि 2 और 3

(c) किि 3

(d) 1 2 और 3

Q72) निमननिखित जिजानतय ो म स कौि-सीसी ो

जिजानतजिजानतया िागािड स सोबोनरत हह

1 अोगामी

2 ककी

3 जारिा

िीच नदए गए कट का परय ग कर सही उततर चनिए

(a) किि 1

(b) किि 1 औऔ 2

(c) किि 2

(d) 1 2 और 3

Q73) निमननिखित कथि ो म स कौि-सास सही हह

1 राषटर कट सामराजय की सथापिा दोनतदगण ि की थी

नजसि मानयाित म अपिी राजरािी की

सथापिा की थी

2 राषटर कट समराट अम घििण एक ििक था और

उस कनिताओो पर पहिी कननड़ पसतक नििि

का शरय नदया जाता ह

िीच नदए गए कट का परय ग कर सही उततर चनिए

(a) किि 1

(b) किि 2

(c) 1 और 2 द ि ो

(d) ि त 1 ि ही 2

Q74) निमननिखित कथि ो म स कौि-सास सही हह

1 कशब चोदर सि ि ततवब नरिी सभा की

अधयकषता की थी ज आधयाखतमक सतय की

ि ज म सोिि थी

2 बरहम समाज ि मािि गररमा पर बि नदया

मनतणपजा का निर र नकया और सती परथा जसी

सामानजक बराइय ो की आि चिा की

िीच नदए गए कट का परय ग कर सही उततर चनिए

(a) किि 1

(b) किि 2

(c) 1 और 2 द ि ो

(d) ि त 1 ि ही 2

Q75) निमननिखित कथि ो म स कौि-सास सही हह

1 भारत म नचशती नसिनसिा खवाजा म इिददीि

नचशती क दवारा सथानपत नकया गया था

2 नचशती परोपरा की एक परमि निशिता

आतमसोयम थी नजसम साोसाररक म ह स दरी

बिाए रििा शानमि था

िीच नदए गए कट का परय ग कर सही उततर चनिए

(a) किि 1

(b) किि 2

(c) 1 और 2 द ि ो

(d) ि त 1 ि ही 2

RAUSIAS-FC19E1003 27

Q70) Which of the following statements

isare correct

1 During the Ahmedabad Mill Strike

Mahatma Gandhi undertook a fast

unto death to strengthen the

workersrsquo resolve

2 The fast had effect of putting

pressure on mill owners who

finally agreed to give the workers a

15 per cent increase in wages

Select the correct answer using the code

given below

(a) 1 only

(b) 2 only

(c) Both 1 and 2

(d) Neither 1 nor 2

Q71) Which of the following are included in

the UNESCOrsquos list of the representative

list of the intangible cultural heritage of

humanity from India

1 Mudiyettu

2 Sankirtana

3 Kumbh Mela

Select the correct answer using the code

given below

(a) 1 and 2 only

(b) 2 and 3 only

(c) 3 only

(d) 1 2 and 3

Q72) Which of the following tribes isare

related to Nagaland

1 Angami

2 Kuki

3 Jarawa

Select the correct answer using the code

given below

(a) 1 only

(b) 1 and 2 only

(c) 2 only

(d) 1 2 and 3

Q73) Which of the following statements

isare correct

1 Rashtrakuta kingdom was founded by Dantidurga who established his capital at Manyakhet

2 Amoghavarsha a Rashtrakuta king was an author and is credited with writing the first

Kannada book on poetics

Select the correct answer using the code given below

(a) 1 only

(b) 2 only

(c) Both 1 and 2

(d) Neither 1 nor 2

Q74) Which of the following statements isare correct

1 Keshab Chandra Sen headed the Tattvabodhini Sabha which was engaged in search of spiritual truth

2 The Brahmo Samaj laid emphasis on human dignity opposed idolatry and criticized such social

evils as the practice of Sati

Select the correct answer using the code given below

(a) 1 only

(b) 2 only

(c) Both 1 and 2

(d) Neither 1 nor 2

Q75) Which of the following statements isare correct

1 The Chishti order was established in India by Khwaja Moinuddin

Chishti

2 A major feature of the Chishti tradition was austerity including maintaining a distance from the

worldly power

Select the correct answer using the code

given below

(a) 1 only

(b) 2 only

(c) Both 1 and 2

(d) Neither 1 nor 2

T e s t i s p a r t o f R a u rsquo s I A S T e s t s e r i e s f o r P r e l i m i n a r y E x a m 2 0 1 9

FOUNDATION + CURRENT AFFAIRS

GENERAL STUDIES (PAPER ndashI)

FOUNDATION TEST ndashIII

SUBJECT NCERT History Class VI-X + Current Affairs

Time Allowed 1frac12 Hours Maximum Marks 150

I NSTRUCT IONS

1 IMMEDIATELY AFTER THE COMMENCEMENT OF THE EXAMINATION YOU SHOULD CHECK

THAT THIS TEST BOOKLET DOES NOT HAVE ANY UNPRINTED OR TORN or MISSING PAGES OR

ITEMS ETC IF SO GET IT REPLACED BY A COMPLETE TEST BOOKLET

2 This Test Booklet contains 75 items (questions) Each item is printed both in Hindi and English

Each item comprises four responses (answers) You will select the response which you want to mark

on the Answer Sheet In case you feel that there is more than one correct response mark the

response which you consider the best In any case choose ONLY ONE response for each item

3 You have to mark all your responses ONLY on the separate Answer Sheet (OMR sheet) provided

Read the directions in the Answer Sheet

4 All items carry equal marks

5 Before you proceed to mark in the Answer Sheet the response to various items in the Test booklet

you have to fill in some particulars in the Answer Sheet as per instructions contained therein

6 After you have completed filling in all your responses on the Answer Sheet and the examination has

concluded you should hand over to the Invigilator only the Answer Sheet You are permitted to

take away with you the Test Booklet

7 Penalty for wrong answers

THERE WILL BE PENALTY FOR WRONG ANSWERS MARKED BY A CANDIDATE IN THE

OBJECTIVE TYPE QUESTION PAPERS

(i) There are four alternatives for the answer to every question For each question for which a

wrong answer has been given by the candidate one-third of the marks assigned to that

question will be deducted as penalty

(ii) If a candidate gives more than one answer it will be treated as a wrong answer even if one of

the given answers happens to be correct and there will be same penalty as above to that

question

(iii) If a question is left blank ie no answer is given by the candidate there will be no penalty for

that question

T h i s t e s t i s p a r t o f R a u rsquo s I A S T e s t s e r i e s f o r P r e l i m i n a r y E x a m 2 0 1 9

Test Code

FC19E1003

FC19H1003 29

Answers and Explanations of

NCERT History Class VI-X + Current Affairs (FC19E1003)

Q1) उततर (c)

सपषटीकरण

- ऋगवद म दविय ो और दिताओो क समवपित एक

हजार स अविक सत तर (शल क) ह

- य शल क ऋविय ो क दवारा रच गए थ और परि ो

दवारा सीख जात थ

- हालाोवक कछ शल क मवहलाओो (जस वक अपाला

घ सा ल पामदरा मतरयी और गागी) क दवारा भी रच

गए थ

- ऋगवद म सोिाद क रप म कई शल क मौजद ह

- हम विशवावमतर नामक एक ऋवि और दविय ो क

रप म पजी जान िाली द नवदय ो (वयास और

सतलज) क बीच िाताि का उदाहरण वमलता ह

- इसस पता चलता ह वक विशवावमतर िवदक काल स

सोबोवित थ

Q2) उततर (b)

सपषटीकरण

- करनल गफाओो स राख क अिशि परापत हए ह

ज इस ओर सोकत करत ह वक ततकालीन ल ग

अवि क उपय ग स पररवचत थ

- य गफाएो आोधर परदश म सथथत ह

Q3) उततर (c)

सपषटीकरण

bull बरािह म ितिमान कशमीर म सथथत एक

परागवतहावसक थथल ह जहाो ल ग गडढ क घर ो का

वनमािण करत थ

bull य घर जमीन क ख द कर बनाए जात थ तथा नीच

जान क वलए सीवियाा ह ती थी

bull ऐसा अनमान लगाया जाता ह वक य घर ठो ड क

मौसम म आशरय परदान करत थ

Q4) उततर (c)

सपषटीकरण

bull परालख-विदया (Epigraphy) क वशलालख ो क

अधययन क रप म पररभावित वकया जाता ह

bull हसतवलसखत दसतािज ो क माधयम स इवतहास

और सावहतय क अधययन क पाोडवलवप विजञान

(Manuscriptology) कहत ह

bull पराचीन लखन परणावलय ो क अधययन और

ऐवतहावसक पाोडवलवपय ो क समझन तथा वतवथ

वनिािरण क पलीओगराफी (Palaeography) कहा

जाता ह

bull नयवमजमविकस (Numismatics) वसक ो क

अधययन क सोदवभित करता ह

Q5) उततर (a)

सपषटीकरण

- चरक सोवहता चरक क दवारा वलखी गई आयिद

और िदयक-शासर पर एक महतवपणि पसतक ह

- ि भारतीय िदयक-शासर की पारमपररक परणाली

वजस आयिद क नाम स जाना जाता ह क

अभयासकताि थ

- ऐसा माना जाता ह वक चरक का विकास दसरी

शताबदी (ईसा पिि) और दसरी शताबदी (ईसवी) क

मधय हआ था

Q6) उततर (b)

सपषटीकरण

- भाग फसल ो पर वलए जान िाल कर क सोदवभित

करता ह ज कल फसल उतपादन का 16 िाो भाग

था

- ldquoकममकारrdquo शबद भवमहीन कवि शरवमक िगि क

वलए परय ग वकया जाता था

- ldquoअशवमिrdquo (वजस घ ड क बवलदान क रप म भी

जाना जाता ह) एक अनषठान ह ता था वजसम एक

घ ड क सवतोतर रप स घमन क वलए छ ड वदया

FC19H1003 30

जाता ह और राजा क सवनक उसकी रखिाली

करत थ

Q7) उततर (d)

सपषटीकरण

- ऋगववदक काल म घ ड ो क रथ ो म ज ता जाता था

ज (रथ) भवम मिवशय ो आवद पर कबजा करन क

वलए लड गए यद ो म उपय ग वकए जात थ

- इसस यह पता चलता ह वक घ ड ो यकत रथ ो का

उपय ग महाजनपद काल स काफी पहल आरमभ

हआ था

- ऋगववदक काल म मिवशय ो भवम जल आवद पर

कबजा करन क वलए तथा ल ग ो क पकडन क

वलए यद वकय जात थ

- अविकाोश परि इन यद ो म भाग वलया करत थ

- हालाोवक उस समय क ई वनयवमत सना नही ो ह ती

थी लवकन उस काल म सभाऐो ह ती थी ो वजनम

ल ग यद क मामल ो पर चचाि करत थ

- वनयवमत सनाएा महाजनपद काल का िवशषटय थी

वजनम पदल सवनक ो की विशाल सनाएा रथ तथा

हाथी शावमल ह त थ

Q8) उततर (a)

सपषटीकरण

- बद शाकय कल स सोबोवित थ और कशीनारा म

उनका वनिन हआ था

- बद न अपनी वशकषाएा पराकत भािा म दी थी ो ज

आम ल ग ो की भािा थी

Q9) उततर (c)

सपषटीकरण

- पराचीन भारत म दशिनशासर की छह शाखाएा थी ो

िशविक नयाय समखया य ग पिि वममाोसा और

िदाोत या उततर वममाोसा

- इनकी थथापना करमश कनाद गौतम कवपल

पतोजवल जावमनी और वयास ऋविय ो न की थी

Q10) उततर (b)

सपषटीकरण

महािीर की वशकषाऐो छठी शताबदी म िललभी म

सोकवलत की गई थी ो

Q11) उततर (c)

सपषटीकरण

- पारमपररक रप स चाणकय क कौविलय अथिा

विषणगपत क नाम स जाना जाता ह

- उसन अथिशासतर ज एक पराचीन भारतीय

राजनवतक आलख ह वलखा था

Q12) उततर (d)

सपषटीकरण

- भारत का राषटर ीय वचनह सारनाथ (उततर परदश) क

अश क सतमभ क ऊपर (शीिि पर) वसोह कवपिल

का एक अनरपण ह

- इस राषटर ीय वसदाोत सतयमि जयत क साथ

सोय वजत वकया गया ह

- रामपिि बल का नाम रामपिि (वबहार) क नाम पर

पडा जहाा इसकी ख ज हई थी

- यह अपन नाजक नकाशी मॉडल क वलए परवसदद

ह वजसम क मल तवचा सोिदनशील नथन ो सतकि

कान और मरबत िााग ो क शरषठतर परवतरप क

परदवशित वकया गया ह

- यह भारतीय और फारसी ततव ो का एक ससममशरण

- सोवकससा उततर परदश म सथथत ह

Q13) उततर (a)

सपषटीकरण

का िर वसोह ज एक महान य दा थ वबहार स

सोबोवित थ

Q14) उततर (b)

सपषटीकरण

िललालर शबद बड भ-सवावमय ो क वलए परय ग

वकया जाता था

FC19H1003 31

Q15) उततर (c)

सपषटीकरण

- अररकमड एक तिीय बसती थी जहाो दर दश ो स

आन िाल जहाज ो का माल उतारा जाता था

- यहाो पर ईोि ो का एक विशाल ग दाम वमटटी क

बतिन (वजनम एमफ रा - द हरी मवठय ो का लोबा

घडा - शावमल ह) और एरिाइन (Arretine)

मदभाोड पाए गए थ

- इस थथान पर र मन दीपक काोच क बन पातर और

रतन भी पाए गए थ

Q16) उततर (a)

सपषटीकरण

- मिनदर सोगम कविताओो म उसललसखत एक

तवमल शबद ह वजसका अथि ह ldquoतीन परमखrdquo

- यह तीन सततारि पररिार ो क मसखयाओो क वलए

परय ग वकया जाता ह च ल चर और पाणडय

Q17) उततर (c)

सपषटीकरण

- ऋग िद म सभा विदाथा तथा गण जसी

जनजावतय ो पर अथिा किोब पर आिाररत

सभाओो का उललख ह

- आरसमभक िवदक काल म सभाओो और सवमवतय ो

का विशि महतव ह ता था

- यहाा तक की मसखया अथिा राजा भी उनका

समथिन परापत करन क वलए आतर रहत थ

Q18) उततर (a)

सपषटीकरण

- जन िमि न ईशवर क अससततव क मानयता त दी ह

वकनत उसन ईशवर क वजना क पद स नीच रखा

- जन िमि न बौद िमि की तरह िणि परणाली की

भरतिना नही ो की थी

Q19) उततर (d)

सपषटीकरण

- च ल ो और पाणडय ो न शसकतशाली तिीय शहर ो का

विकास वकया था

- च ल ो का सबस महतवपणि शहर पहार (या

कािरीपटटीनम) था |

- मदरई पाणडय ो की राजिानी थी

Q20) उततर (b)

सपषटीकरण

- ldquoबदचररतrdquo बद का जीिन-ितताोत ह

- इस अशवघ ि क दवारा वलखा गया था

Q21) उततर (a)

सपषटीकरणः

- तवमल कवि अपपर भगिान वशि क भकत थ

- इस परकार ि एक नयनार सोत थ

Q22) उततर (d)

सपषटीकरणः

- समदरगपत एक परवसद गपत शासक था

- उसन वसक ो पर िीणा बजात हए अपनी छवि

अोवकत करिाई थी

- यह सोगीत क परवत उसक परम क दशािता ह

- हम उसकी इलाहाबाद परशससत स महतवपणि

ऐवतहावसक जानकारी वमलती ह वजसकी रचना

उसक दरबार क कवि हररसन न की थी

Q23) उततर (b)

सपषटीकरणः

- विकरम सोित की शरआत ििि 58 ईसा पिि म

चनदरगपत वदवतीय न की थी

- यह शक ो पर उसकी जीत और उस विकरमावदतय

की पदिी वमलन क उपलकषय म आरमभ वकया गया

था

FC19H1003 32

- बानभटट न हिििििन का जीिन-ितताोत हििचररत

(ज सोसकत म थी) वलखी थी

Q24) उततर (c)

सपषटीकरणः

- सोवि-विगरावहका यद एिो शाोवत का मोतरी

- साथििाह वयापाररय ो क कावफल ो का नता

Q25) उततर (a)

सपषटीकरणः

- जआन झाोग (हसआन रताोग ndash Hsuang Tsang)

एक चीनी यातरी था ज हिििििन क शासनकाल म

भारत आया था

- ििि 630 ईसवी स ज दशक आरमभ हआ था उसम

जआन झाोग मधय एवशया ईरान और

अफग़ावनसतान की यातरा करन क पशचात कशमीर

क रासत स भारत आया था

- उसन उततर स पिि तक की यातरा की और िह

लगभग 2 ििि वबहार म रहा

- जआन झाोग न नालनदा विशवविदयालय म विदयावथिय ो

और विदवान ो क साथ पारसपररक विचार-विमशि

वकया थथानीय भािाओ ा म वनपणता परापत की तथा

बौद सतप ो की ख ज की

Q26) उततर (c)

सपषटीकरणः

- परदवकषणा पथ बौद िासतकला म सतप क चार ो

ओर बनाया जान िाला एक घमािदार पथ ह ता

- परशन म वदए गए बाकी क तीन ो ततव वहोद मसनदर ो की

िासतकला क भाग ह

Q27) उततर (d)

सपषटीकरणः

परशन म वदए गए सभी मोवदर ो म वयापक रप स

ईोि ो (पकी ईोि ो) का परय ग पतथर ो क साथ हआ

Q28) उततर (c)

सपषटीकरण

- महममद कली कतब शाह ग लकणडा का सलतान

था

- िह अकबर का समकालीन था

- सावहतय और िासतकला म उसकी अतयाविक

रवच थी

- िह एक महान कवि था

- िह दसखनी उदि फारसी और तलग म वलखता था

- उसन अपन पीछ एक विसतत वदिान (सोगरह)

छ डा ह

- अभी हाल ही म तलोगाना म ग लकणडा क वकल

क अनदर खदाई वकय गए बाग-ए-नाया वकला

बाग क चार ो ओर रप-रखा क मानवचतरण क

वलए भारतीय परातासतवक सिकषण (The

Archaeological Survey of India ndash ASI)

गराउणड पनीिर विोग रडार (Ground Penetrating

Radar) का परय ग करगा

Q29) उततर (a)

सपषटीकरणः

- वसलपपावदकारम एक तवमल महाकावय ह वजसकी

रचना इलाोग क दवारा लगभग 1800 ििि पिि की

गई थी

- यह क िलन नामक एक वयापारी की कहानी ह

ज माििी नामक एक गवणका (िशया) स परम

करन लगा था

- मवनमकलाई क िलन और माििी की पतरी की

कहानी ह

Q30) उततर (a)

सपषटीकरण

- चरक आयिद और वचवकरता की एक महतवपणि

रचना चरक सोवहता क लखक ह

- बरहमगपत क अपनी रचना बरहम-सफि-वसदानत

(ज एक खग लीय रचना ह) क कारण परवससद

वमली

FC19H1003 33

- बगदाद म इसका अनिाद अरबी भािा म वकया

गया था

- इसका इसलावमक गवणत और खग ल-विजञान पर

महतवपणि परभाि पडा था

- बाद म अपन जीिनकाल म बरहमगपत न

ldquoखोडखयाकrdquo वलखी ज एक खग लीय पससतका

(एक छ िी पसतक) थी

- इसम आयिभटट की अिि-रावतर क परतयक वदन की

शरआत परणाली का परय ग वकया गया था

Q31) उततर (c)

सपषटीकरण

- अमीर खसर एक परवसद सफी सोगीतकार कवि

और विदवान थ

- 1318 म उनह ोन पाया वक इस भवम (वहोदसतान) क

हर कषतर म अलग-अलग भािा थी लाहौरी

कशमीरी दवारसमदरी (दवकषणी कनाििक म)

तलोगाना (आोधर परदश म) गजरी (गजरात म)

माबारी (तवमलनाड म ) अििी (पिी उततर परदश

म) और वहोदिी (वदलली क आस-पास क कषतर म)

आवद

- उनह न यह बताया वक सोसकत वकसी भी कषतर स

सोबोवित नही ो थी और किल बराहमण ही इस भािा

का जञान रखत थ

Q32) उततर (c)

सपषटीकरण

- वहरणय-गभि सववणिम गभि क सोदवभित करता ह

- जब बराहमण ो की सहायता स यह अनषठान वकया

जाता था त यह माना जाता था वक बवल दन िाल

का कषवतरय क रप म पनजिनम ह गा

Q33) उततर (d)

सपषटीकरण

- कदमई भवम राजसव पर कर क सोदवभित करता

- गवावलयर परशससत म नागभि क दवारा वकय गए

श िण का िणिन वकया गया ह |

- नागभि एक परवतहार राजा था

Q34) उततर (b)

सपषटीकरण

- राजतरो वगनी 12िी ो शताबदी म कलहन क दवारा

रवचत एक सोसकत पसतक (िकसट) ह

- यह परारसमभक भारत की ऐवतहावसक इवतितत थी

- तकि सोगत रप स इस अपन परकार की सिोततम

और सिािविक विशवसनीय कवत माना जाता ह

- यह कशमीर कषतर क पराचीनतम समय स लकर

उसकी रचना की तारीख तक क समपणि इवतहास

का आचछादन करती ह

Q35) उततर (c)

सपषटीकरण

- गााि की आम सभा क ldquoउरrdquo कहा जाता था

- ldquoउरrdquo म गााि क सभी कर दन िाल वनिासी

शावमल ह त थ

Q36) उततर (a)

सपषटीकरण

- वदलली सलतनत म ldquoतारीखrdquo इवतहास लखन का

एक रप था

- ldquoतािरीखrdquo क लखक विदवान परि ह त थ वजनम

सवचि परशासक इतयावद शावमल थ

Q37) उततर (a)

सपषटीकरण

- अलाउददीन सखलजी अपन सवनक ो क ितन का

भगतान नकद म करता था न वक इकता क रप

- सवनक अपना सामान वदलली म वयापाररय ो स

खरीदत थ अतः इस बात का भय था वक वयापारी

कही ो िसतओो का मलय न बिा द

- इसकी र कथाम क वलए अलाउददीन सखलजी न

वदलली म कीमत ो क वनयसित वकया

FC19H1003 34

- अविकारीगण धयानपििक मलय ो का सिकषण करत

थ तथा ज वयापारी वनिािररत मलय पर माल नही ो

बचत थ उनक दसणडत वकया जाता था

Q38) उततर (d)

सपषटीकरण

- वदलली सििपरथम त मर राजपत ो क अिीन उनक

सामराजय की राजिानी बनी थी

- 12िी ो शताबदी क मधय म अजमर क चौहान ो

(वजनह चाहमान ो क नाम स भी जाना जाता ह) न

त मर राजपत ो क परावजत वकया था

- त मर ो और चौहान ो क अिीन वदलली एक

महतवपणि िावणसजयक क दर बन गया था

- कई जन वयापारी यहाा रहन लग थ और उनह ोन

कई मोवदर भी बनिाए

- यहाा पर मवदरत वसक वजनह ldquoदहलीिालrdquo क नाम

स जाना जाता था वयापक रप स परचलन म थ

Q39) उततर (c)

सपषटीकरण

- म ठ की मसिद का वनमािण वसको दर ल दी क

राजयकाल म उसक मिी क दवारा करिाया गया

था

- बगमपरी मसिद का वनमािण महममद तगलक क

शासनकाल म हआ था

- यह मसिद विशव का पणयथथान (The

Sanctuary of the World) और वदलली म महममद

तगलक की नई राजिानी जहाोपनाह की मखय

मसिद थी

- कववत- अल - इसलाम मसिद का विसतार

इलतसिश और अलाउददीन सखलजी न वकया था

- मीनार का वनमािण तीन सलतान ो कतबददीन ऐबक

इलतसिश और वफर ज शाह तगलक क दवारा

करिाया गया था

Q40) उततर (c)

सपषटीकरण

- मगल ो क अिीन मनसबदार शबद उस वयसकत क

वलए सोदवभित वकया जाता था वजसक पास मनसब

(अथाित पद) ह ता था

- उस अपना ितन राजसव कायो वजनह जागीर कहत

थ क रप म परापत ह ता था

Q41) उततर (b)

सपषटीकरण

- ldquoभारत छ ड आोद लनrdquo वबरविश शासन क

सखलाफ ल ग ो का एक सवाभाविक विदर ह था

- असखल भारतीय काोगरस सवमवत न 8 अगसत 1942

क बमबई म एक बठक का आय जन वकया था

- इस बठक म परवसद सोकलप ldquoभारत छ ड rdquo क

पाररत वकया गया और इस उददशय क परापत करन

क वलए गाोिी क नततव म एक अवहोसक जन सोघिि

आोद लन की शरआत का परसताि वदया गया

- लवकन अगल ही वदन गाोिी और काोगरस क अनय

परमख नताओो क वगरफतार कर वलया गया

- काोगरस क एक बार वफर अिि घ वित वकया गया

था

Q42) उततर (c)

सपषटीकरण

- साइमन कमीशन यनाइविड वको गडम क सात

साोसद ो का एक समह था

- इस वबरविश भारत क वलए सोििावनक सिार ो का

सझाि दन क वलए गवठत वकया गया था

- इस आय ग म िररषठ वबरविश राजनता सर जॉन

साइमन क नततव म किल वबरविश सदसय ही

शावमल थ

- इसवलए भारत क ल ग ो न साइमन कमीशन क

आगमन क विरद आोद लन वकया था

Q43) उततर (a)

सपषटीकरण

bull दादा भाई नौर जी भारत म वबरविश शासन क

आवथिक पररणाम ो क बार म अपनी विर िी

(परवतकल) राय क वलए जान जात थ

FC19H1003 35

bull अपन कई लख ो और भािण ो म विशि रप स

ldquoपाििी एो ड अन-वबरविश रल इन इसणडया

(Poverty and Un-British Rule in India) म

नौर जी न यह तकि वदया वक भारत पर अतयविक

कर लगाया गया था और इसकी सोपवतत इोगलड की

ओर परिावहत की जा रही थी

bull उनह ोन पराचीन भारतीय गरोथ ो की वयाखया करन

का और भारतीय ो क आिविशवास क बहाल

करन पर कायि नही ो वकया था

उनह ोन वकसी और बात स पहल सभी सामावजक

बराइय ो क उनमलन की आिशयकता पर भी बल

नही ो वदया था

Q44) उततर (c)

सपषटीकरण

bull अगसत 1932 म वबरविश परिानमोतरी मकड नालड न

अपन साोपरदावयक परसकार (The Communal

Award) की घ िणा की थी

bull यह भारत क कई साोपरदावयक वहत ो क बीच विवभनन

सोघिो क हल करन क वलए वबरिन का एकतरफा

परयास था

bull यह परसकार (Award) बाद म 1935 क

अविवनयम (The Act of 1935) म शावमल वकया

गया था

bull इस साोपरदावयक परसकार न मससलम ो क वलए

आरवकषत एक अलग वनिािचक मणडल फॉमिल का

विसतार अनय अलपसोखयक ो क वलए वकया था

वजसम वसख ो भारतीय ईसाइय ो आोगल-भारतीय

समदाय यर पीय समदाय तथा विवशषट कषतरीय

समह ो क शावमल वकया गया था

bull गाोिी न इस परसताि क भारतीय समाज क

विभावजत करन क वलए एक घवणत वबरविश

सावजश क रप म दखा और उसक सखलाफ

आमरण अनशन वकया

Q45) उततर (b)

सपषटीकरण

मौजदा आयात और वनयाित क अवतररक़त

औपवनिवशक भारत क वनमनवलसखत खचो क

वलए एक विशिवनवशचत िन रावश भी दनी पडती

थी

(i) परशासन क वयय

(ii) सना क रख-रखाि क वयय

(iii) यद क वयय

(iv) सिावनितत अविकाररय ो की पशन तथा

(v) वबरिन दवारा अपनी उपवनिश बसती

(कॉल नी) क रख-रखाि क वयय

इनह गह शलक (Home Charges) क रप म

जाना जाता था और लगभग परी तरह स भारत क

दवारा इनका भगतान वकया जाता था

bull गह शलक म वनमनवलसखत घिक शावमल थ

(i) भारतीय ऋण पर दय बयाज

(ii) ईसट इोवडया को पनी क शयरिारक ो क

लाभाोश

(iii) लोदन म भारत कायािलय चलान क वलए िन

(iv) भारत म वनयकत वबरविश कवमिय ो क ितन

और पशन का भगतान करन क वलए िन

(v) रलि पर बयाज

(vi) नागररक और सनय शलक

(vii) इोगलड म सट र (सामगरी) की खरीद

Q46) उततर (b)

सपषटीकरण

bull भारतीय राषटर ीय काोगरस का लाहौर सतर 1929 म

जिाहरलाल नहर की अधयकषता म आय वजत

वकया गया था

bull इस सतर म भारतीय राषटर ीय आोद लन स समबसित

कई महतवपणि पररणाम सामन आय थ

(i) सििपरथम इस सतर म काोगरस क अधयकष पद

पर जिाहरलाल नहर क चना गया था ज

काोगरस म िामपोवथय ो की बिती हई ताकत

का सपषट सोकत था

(ii) दसरा इस सतर म पहली बार काोगरस न पणि

सवतोतरता की माोग क उठाया था

इस परकार की माोग काोगरस मोच स पहल कभी भी

नही ो उठाई गई थी

Q47) उततर (b)

सपषटीकरण

FC19H1003 36

bull इस ररप िि न वकसी भी समदाय क वलए पथक

वनिािचक मोडल अथिा अलपसोखयक ो क वलए

भाराोश की वसफाररश नही ो की थी

bull तथावप इस ररप िि न उन पराोत ो म अलपसोखयक

सीि ो क आरकषण की अनमवत दी थी जहाा पर कम

स कम दस परवतशत अलपसोखयक ह

bull लवकन यह समदाय क आकार क अनपात म ह ना

चावहए था

bull इस ररप िि म भारत क वलए पणि सवतोतरता क

वलए क ई पराििान नही ो था

Q48) उततर (c)

सपषटीकरण

bull आरो वभक िवदक आयो का िमि मखय रप स

परकवत की पजा और यजञ था

bull परारो वभक आयि िमि परकवत की पजा क समान था

bull िासति म उनक चार ो ओर की शसकतयाा वजनह न

त ि वनयोवतरत कर सकत थ और न ही समझ पाए

थ उनह वदवयता क साथ वनिवशत वकया गया तथा

उनह मादा या नर दिीदिताओो क रप म

परतीकतव वकया गया था

bull उनह ोन कछ यजञ ो का भी वनषपादन वकया था

Q49) उततर (b)

सपषटीकरण

bull सडक और नदी-मागि (जल-मागि) डकती स

सरवकषत नही ो थ

bull उललखनीय ह वक हिििििन क शासनकाल क

दौरान यआन चिाोग (हयएन साोग) का सारा

सामान लि वलया गया था

Q50) उततर (c)

सपषटीकरण

परशन म वदए गए द न ो कथन सही ह

Q51) उततर (b)

सपषटीकरण

bull परोदर दास एक सोत और भगिान कषण क एक

महान भकत थ

bull परोदर दास क कनाििक सोगीत क वपतामह क

रप म जाना जाता ह

bull यदयवप उनक जनम-थथान क बार म काफी

अिकल लगाई जाती रही ह

bull तथावप अब कननड विशवविदयालय हमपी क दवारा

गवठत एक विशिजञ सवमवत इस वनषकिि पर पहोची

ह वक उनका जनम थथान सोभितया कनाििक का

एक छ िा-सा गााि कषमपरा (वशिम गगा वजला)

था

Q52) उततर (c)

सपषटीकरण

bull शरी तयागराज शरी शयाम शासतरी और शरी मथसवामी

दीवकषतर क कनाििक सोगीत की वतरमवति माना

जाता ह

bull उनक कारण ही 18िी ो-19िी ो शताबदी म कनाििक

सोगीत का सववणिम यग आया था

Q53) उततर (d)

सपषटीकरण

bull अभी हाल ही म लौह यगीन-महापािावणक काल

का 2000 ििि पराना एक दलिभ सारक फगस

(Sarcophagus) (पतथर का ताबत) क ललम क

वियर गाोि (क वयलडी क पास वजला क वझक ड

करल राजय) की एक रॉक-कि गफा स ख जा गया

bull यह ताबत वजसम हविय ो क िकड थ खदाई क

दौरान वमला

bull अभी तक इस परकार की दलिभ ख ज करल क

मातर द ही थथान ो स हई ह

bull य द न ो सारक फगी (Sarcophagi) (पतथर क

ताबत) चियर और अथ ली (वजला क वझक ड) क

महापािाण थथल ो स वमल ह

Q54) उततर (a)

सपषटीकरण

FC19H1003 37

दवकषण भारत म महापािाण सोसकवत एक पणि

विकवसत लौह यगीन सोसकवत थी

Q55) उततर (d)

सपषटीकरण

bull च ल पाणडय और करलपतर (चर) इन तीन ो का

उललख अश क क अवभलख ो म वकया गया ह

bull सोभितः य भौवतक सोसकवत क उततर

महापािावणक चरण म थ

Q56) उततर (d)

सपषटीकरण

bull भीमा-क रगाोि की लडाई ततीय आोगल-मराठा

यद का वहससा थी

Q57) उततर (b)

सपषटीकरण

bull राजकमार शकल न गाोिीजी क चोपारण आन तथा

वतनकवथया परणाली स जडी समसया की जाोच क

वलए रारी करन क वलए दश भर म उनका

अनसरण वकया था

bull बज वकश र राजदर परसाद महादि दसाई और

नरहरी पाररख चोपारण सतयागरह क दौरान गाोिी

जी क सहय गी थ

Q58) उततर (b)

सपषटीकरण

bull बराहमण ो और बौद मठिाररय ो क कर-मकत गााि

अनदान म दन की परथा सतिाहन ो न आरमभ की

थी

Q59) उततर (c)

सपषटीकरण

इस कायिकरम क उददशय वनमनानसार ह

(i) बवनयादी पयििन आिाररक सोरचना का विकास

करना

(ii) चयवनत (पहचान वकय गए) कषतर ो म आजीविका क

सजन क वलए दश क साोसकवतक और विरासत

मलय ो क बिािा दना

(iii) विरासत समारक थथल ो पर विशव सतरीय आिाररक

सोरचना विकवसत करक एक सतत तरीक स

पयििक आकििण म िसद करना

(iv) थथानीय समदाय ो की सवकरय भागीदारी क माधयम

स र रगार ो का सजन करना

(v) र रगार उतपादन और आवथिक विकास क वलए

पयििन कषमता का उन पर परभाि का उपय ग

करना तथा

(vi) िारणीय पयििन आिाररक सोरचना का विकास

करना और उसका उवचत सोचालन तथा

रखरखाि सवनवशचत करना

Q60) उततर (b)

सपषटीकरण

bull यह वनकाय ििि 1987 म अससततव म आया था

bull यह एक राषटर ीय सतर का शीिि सोगठन ह ज भारत

सरकार क जनजातीय मामल ो क मोतरालय क

परशासवनक वनयोतरण क अिीन काम कर रहा ह

bull इसका पोजीकत और परिान कायािलय नई वदलली

म सथथत ह

Q61) उततर (c)

सपषटीकरण

bull परमचोद क उपनयास ो म परमाशरम रोगभवम गबन

कमिभवम और ग दान शावमल ह

bull ग रा रिी ोदरनाथ िग र क दवारा रवचत उपनयास ह

bull अभी हाल ही म मोशी परमचोद की 138िी ो जयोती दश

भर म मनाई गई थी

Q62) उततर (b)

सपषटीकरण

bull ldquoवगदाrdquo पोजाब (भारत) एिो पावकसतान की

मवहलाओो क दवारा तयौहार क समय और फसल

की बिाई तथा किाई क अिसर पर वकया जान

िाला एक पारोपररक दहाती नतय ह

FC19H1003 38

bull इस नतय क माधयम स पोजाबी मवहलाऐो अपनी

परसननता परकि करती ह तथा वगदा क परदशिन क

माधयम स परि िचिसव िाल समाज म मवहलाओो

की दबी हई भािनाओो क परकि करती ह

bull चोवक इस नतय का परि ो क साथ क ई सोबोि नही ो

ह अतः किल मवहलाऐो ही इसम भाग ल सकती

bull हर साल तीज समार ह क दौरान पोजाब म वगदा

नतय वकया जाता ह

तीज भारत क कछ भाग ो म मवहलाओो क दवारा

मनाया जान िाल कई तयौहार ो क वलए एक

वयापक नाम ह

Q63) उततर (a)

सपषटीकरण

- मजम-उल-बहरीन या द समदर ो का सोगम

नामक उललखनीय रचना दारा वशक ह क दवारा

वलखी थी

- भारत क उपराषटर पवत शरी एम िकया नायड न कहा

ह वक राजकमार दारा वशक ह की रचनाएा शाोवत

और सदभाि क बिािा दन क वलए एक तारा सर त

क रप म सामन आ सकती ो ह

- उपराषटर पवत गत ििो क भला वदए गए राजकमार

दारा वशक ह क परदवशित परचवलत करन हत

आय वजत एक परदशिनी का दौरा करन क बाद एक

सभा क सोब वित कर रह थ

- इस परदशिनी का आय जन फर क इस गौवियर

(Francois Gautier) क दवारा lsquoइोवदरा गाोिी नशनल

सिर फॉर द आििसrsquo (The Indira Gandhi

National Centre for the Arts) नई वदलली म

वकया गया था

Q64) उततर (c)

सपषटीकरण

- ग मतशवर परवतमा जन भगिान बाहबली क

समवपित ह

- यह एक एक-चटटानी पतथर की मवति ह

- राषटर पवत राम नाथ क विोद न शरिणबलग ला

(कनाििक) म आय वजत वकय जान िाल भवय

अवभिक समार ह महामसतकावभिक का

उदघािन वकया था

- यह समार ह 12 ििो म एक बार ह ता ह

Q65) उततर (c)

सपषटीकरण

bull पराची घािी पराची नदी क चार ो ओर फली हई थी

bull पराची घािी िीर-िीर विलपत ह गई थी

bull पराची नदी भिनशवर स वनकलती ह

bull यह महानदी की एक सहायक नदी ह और यह

परी खदाि किक तथा जगतवसोहपर वजल ो स

ह कर बहती ह

bull इस नदी क पर कषतर क पराची घािी कहा जाता ह

bull यह नदी बोगाल की खाडी म वगरती ह

परातासतवक साकषय स पता चलता ह वक पराची घािी

सभयता हडपपा और म हनज दाड द न ो की

पिििती ह

Q66) उततर (d)

सपषटीकरण

य समारक छतरपर वजल (मधय परदश) म विोधयाचल

पिित शरोखला म सथथत ह

Q67) उततर (a)

सपषटीकरण

bull थॉिस ऑन पावकसतान नामक पसतक डॉ बी

आर अमबडकर न वलखी थी

bull डॉ बी आर अमबडकर की जयोती क अिसर पर

भारत क राषटर पवत न भारत की इस महान हसती

क शरदाोजवल अवपित की थी

bull डॉ बी आर अमबडकर न 1924 म वडपरथड

कलावसर इोसटीटयि (दवलत िगि सोथथान -

बवहषकत वहतकाररणी सभा) और 1927 म समाज

समता सोघ की थथापना की थी

bull अमबडकर का धयान वशकषा कषतर की ओर भी था

bull उनह ोन वशकषा क वनमन िगो म फलान क वलए

पीपलस एजकशन स साइिी (The Peoples

Education Society) क नाम स महाविदयालय ो क

नििकि और छातरािास ो की थथापना की थी

FC19H1003 39

Q68) उततर (b)

सपषटीकरण

bull महरगि भारतीय उपमहादवीप म एक परवसद

निपािाण बसती ह ज बलवचसतान पराोत

पावकसतान म सथथत ह

bull दचपलली (आोधर परदश) क पास नागलर नदी क

पिी ति ो पर चना पतथर क बलॉक क विशाल

विसतार म एक पिि-ऐवतहावसक रॉक आिि थथल की

ख ज की गई ह

bull इसन 1500-2000 ईसा पिि क दौरान गोिर (आोधर

परदश) म विकवसत निपािाण सभयता पर परकाश

डाला ह

Q69) उततर (c)

सपषटीकरण

bull 12िी ो सदी और 13िी ो सदी म काकाविय िोश का

उदय हआ था

bull ि पहल कलयाण क पवशचमी चालकय ो क सामोत थ

bull परारोभ म उनह ोन िारोगल (तलोगाना) क पास एक

छ ि स कषतर पर शासन वकया था

bull उनह ोन ldquoनायक वयिथथाrdquo की शरआत की थी

वजस बाद म विजयनगर क राय शासक ो न

अपनाया और विकवसत वकया था

Q70) उततर (a)

सपषटीकरण

bull गाोिीजी क अनशन स वमल मावलक ो पर दबाि

पडा था ज अोततः शरवमक ो क ितन म 35 परवतशत

की िसद करन क वलए सहमत हए थ

bull गगल (Google) न अनसया साराभाई वजनह ोन

भारत क शरवमक आोद लन म एक अगरणी भवमका

वनभाई थी की 132िी ो जयोती डडल (Doodle) का

वनमािण करक मनाई

Q71) उततर (d)

सपषटीकरण

भारत स यनसक की मानिता की अमति साोसकवतक

विरासत की परवतवनवि सची म वनमनवलसखत शावमल ह

bull कवडयटटम करल का सोसकत रोगमोच

bull मवडयिि करल का अनषठान रोगमोच और नतय

नाविका

bull िवदक मि जाप की परोपरा

bull राजथथान क कालबवलया ल क गीत और नतय

bull रामलीला रामायण का पारोपररक परदशिन

bull सोकीतिन मवणपर का अनषठान गायन ढ ल िादन

और नतय

bull रममन भारत क गििाल वहमालय का िावमिक

तयौहार और अनषठान रोगमोच

bull जाोदीयाला गर पोजाब क ठठर ो की पीतल और

ताोब क वशलप स वनवमित बतिन ो की पारोपररक कला

bull छाऊ नतय पिी भारतीय राजय ो म जनमी शासतरीय

भारतीय नतय कला

bull लददाख का बौद मि जाप िर ाोस-वहमालयी लददाख

कषतर तथा जमम-कशमीर म पवितर बौद गरोथ ो का पाठ

bull य ग

bull नौर र

bull को भ मला

Q72) उततर (b)

सपषटीकरण

bull भारत क राषटर पवत शरी राम नाथ क विोद न

वकसामा नागालड म हॉनिवबल मह रति और

राजय गठन वदिस समार ह का उदघािन वकया

था

bull हॉनिवबल मह रति का नाम भारतीय हॉनिवबल क

नाम पर पडा ह ज एक विशाल और रोगीन जोगली

पकषी ह

bull यह पकषी नागालड राजय की अविकतर जनजावतय ो

की ल ककथाओो म उसललसखत ह

bull नागालड की परमख मानयता परापत जनजावतयाा ह

अोगामी आओ चखसोग चाोग ककी रगमा और

रवलोग आवद

bull ओोग जारिा और ससिनलीस अोडमान-वनक बार

दवीप समह की जनजावतयाा ह

FC19H1003 40

Q73) उततर (c)

सपषटीकरण

bull दकन म राषटर कि शासन दसिी ो सदी क अोत तक

लगभग 200 ििो तक रहा था

bull राषटर कि शासक अपन िावमिक विचार ो म सवहषण

bull उनह ोन न किल शि िमि और िषणि िमि बसलक

जन िमि क भी सोरकषण वदया था

bull एल रा म वशि क परवसद रॉक कि मोवदर का

वनमािण नौिी ो सदी म राषटर कि राजा कषण परथम न

करिाया था

bull उसका उततराविकारी अम घििि जन था लवकन

उसन अनय िमो क भी सोरकषण परदान वकया था

bull राषटर कि ो न मसलमान वयापाररय ो क बसन की

अनमवत दी थी

bull उनह न अपन अविराजय ो म इसलाम क उपदश दन

की भी अनमवत दी थी

bull अभी हाल ही म पाोडिलागटटा (तलोगाना) क

परागवतहावसक चटटान वचतर ो क कषरण की बिती हई

घिनाएा एक गोभीर वचोता का वििय ह

bull यह परागवतहावसक चटटान क नकसान पहाचा

सकता ह

bull पाोडिलागटटा वनमनवलसखत क वलए जाना जाता ह

- 10000 ईसा पिि स 8000 ईसा पिि क वचवतरत

चटटानी आशरय ो क वलए

- राषटर कि काल क एक 8 िी ो सदी क

वशलालख क वलए और

- 12िी ो सदी क काकविय सामराजय क वभवतत

वचतर ो क वलए

Q74) उततर (b)

सपषटीकरण

bull 1828 म राजा राम म हन रॉय न एक नय िावमिक

समाज बरहम सभा की थथापना की थी वजस बाद

म बरहम समाज क नाम स जाना गया था

bull दिदरनाथ िग र न ततवब विनी सभा की अधयकषता

की थी ज आधयासिक सतय की ख ज म सोलि

थी

bull इसका उददशय वहोद िमि क शद करन का और

एकशवरिाद (एक ईशवर म आथथा) का परचार करना

था

bull नय समाज की थथापना क आिार थ कारण

(तकि ) क द सतमभ तथा िद और उपवनिद

bull अभी हाल ही म सािारण बरहम समाज का कछ

काननी मदद ो क लकर पवशचम बोगाल सरकार क

साथ काननी वििाद चल रहा ह

Q75) उततर (c)

सपषटीकरण

bull भारत म वचशती वसलवसल की थथापना खवाजा

म इनददीन वचशती क दवारा की गयी थी

bull ि 1192 ईसवी क आसपास भारत आय थ

bull वचशतीय ो क बारहिी ो शताबदी क उततरािि म भारत

म आन िाल सफीय ो क समह ो म सबस

परभािशाली माना जाता ह

bull उनह ोन थथानीय िातािरण क साथ सफलतापििक

अनकलन वकया और उनह ोन भारतीय भसकत

परोपराओो क कई पहलओो क अपनाया

bull अजमर म सफी अपरकि खवाजा म इनददीन वचशती

की ऐवतहावसक दरगाह क एक नया रप दन की

तयारी की जा रही ह

bull इस 13िी ो शताबदी की दरगाह क ldquoसवचछ

आइकॉवनक थथल ोrdquo (Swacch Iconic Places) म

शावमल वकया गया ह ज परवतवषठत विरासत

आधयासिक और साोसकवतक थथान ो पर क वदरत

य जना ह

FC19H1003 41

ANSWERS amp EXPLANATION OF

NCERT History Class VI-X + Current Affairs

(FC19E1003)

Q1) Answer c

Explanation

Rigveda consists of more than a

thousand hymns dedicated to gods and

goddesses These hymns were

composed by sages and learnt by men

however a few were composed by

women like Apala Ghosa Lopamudra

Maitreyi and Gargi

Rigveda consists of many hymns in the

form of dialogues We get an example of

a dialogue between a sage named

Vishwamitra and two rivers (Beas and

Sutlej) that were worshipped as

goddesses This suggests that he

belonged to the Vedic period

Q2) Answer b

Explanation

Traces of ash have been found from

Kurnool Caves suggesting that people

were familiar with the use of fire

It is situated in Andhra Pradesh

Q3) Answer c

Explanation

Burzahom is a prehistoric site in

present day Kashmir where people built

pit houses which were dug into the

ground with steps leading into them

These may have provided shelter in cold

weather

Q4) Answer c

Explanation

Epigraphy is defined as the study of

inscriptions

Manuscriptology is the study of history

and literature through the use of hand

written documents

Palaeography refers to the study of

ancient writing systems and the

deciphering and dating of historical

manuscripts

Numismatics refers to the study of

coins

Q5) Answer a

Explanation

Charaka Samhita was written by

Charaka and is an important book on

Ayurveda and medicine

He was a practitioner of the traditional

system of Indian medicine known as

Ayurveda

Charaka is thought to have flourished

sometime between the 2nd century BCE

and the 2nd century CE

Q6) Answer b

Explanation

Bhaga refers to the tax on crops which

was fixed at 16th of the production

Kammakaras is the term used for the

landless agricultural labour class

Ashvamedha also known as horse

sacrifice is a ritual where a horse is let

loose to wander freely and it was

guarded by the rajarsquos men

Q7) Answer (d)

Explanation

In the Rigvedic period horses were

yoked to chariots that were used in

battles fought to capture land cattle

etc This suggests that the use of horse

chariots began much before the period

of Mahajanapadas

The battles were fought in the Rigvedic

period for cattlersquos lands water an even

to capture people Most men took part

in these wars however there was no

regular army but there were assemblies

where people met and discussed

matters of war Regular armies became

a feature in the Mjahajanapada period

including vast armies of foot soldiers

chariots and elephants

RAUSIAS-FC19E1003 42

Q8) Answer (a)

Explanation

Buddha belonged to the Sakya clan and

passed away at Kusinara

Buddha taught in Prakrit which was the

common language of people

Q9) Answer c

Explanation

There were six schools of philosophy in

ancient India These are known as

Vaishesika Nyaya Samkhya Yoga

Purva Mimansa and Vedanata or Uttara

Mimansa They were founded by sages

Kanada Gautama Kapila Patanjali

Jamini and Vyasa respectively

Q10) Answer b

Explanation

The teachings of Mahavira were

compiled at Valabhi in 6th century AD

Q11) Answer (c)

Explanation

Chanakya is traditionally identified as

Kautilya or Vishnugupta who authored

the ancient Indian political treatise the

Arthashastra

Q12) Answer d

The national emblem of India is an

adaptation of the Lion Capital atop the

Ashoka Pillar of Sarnath Uttar Pradesh

and is combined with the National

Motto Satyameva Jayate

The Rampurva Bull gets the name from

the site of its discovery Rampurva in

Bihar

It is noted for its delicately sculpted

model demonstrating superior

representation of soft flesh sensitive

nostrils alert ears and strong legs It is

a mixture of Indian and Persian

elements

Sankissa is situated in Uttar Pradesh

India

Q13) Ans(a)

Kunwar Singh was a notable leader during the Revolt of 1857 He belonged

to a royal house of Jagdispur Bihar

Q14) Answer b

Explanation

The term Vellalar was used for large

landowners

Q15) Answer c

Explanation

Arikamedu was a coastal settlement

where ships unloaded goods from

distant lands Finds here include a

massive brick warehouse pottery

including amphorae and Arretine ware

Roman lamps glassware and gems have

also been found at the site

Q16) Answer a

Explanation

Muvendar is a Tamil word mentioned in

Sangam poems meaning three chiefs

used for the heads of three ruling

families the Cholas Cheras and

Pandyas

Q17) Ans (c)

Several tribal or kin-based assemblies

such as the Sabha Vidatha and gana

are mentioned in the Rig-veda The

Sabha and the samiti mattered a great

deal in early Vedic times so much so

that the chiefs or the kings showed an

eagerness to win their support

Q18) Ans (a)

Jainism recognised the existence of the

gods but placed them lower than the

jina and did not condemn the varna

system as Buddhism did

Q19) Answer (d)

Explanation

Cholas and Pandyas had developed

powerful coastal cities The most

important city of Cholas was Puhar or

Kaveripattinam and Madurai was the

capital of Pandyas

Q20) Answer b

Explanation

Buddhacharita is the biography of

Buddha and was written by

RAUSIAS-FC19E1003 43

Ashvaghosha

Q21) Answer (a)

Explanation

Tamil poet Appar was a Shiva devotee

So he was a Nayanar saint

Q22) Answer d

Explanation

Samudragupta was a prominent Gupta

ruler whose coins depict him playing a

veena indicating his love for music We

get important historic information from

his Allahabad Prashasti which was

composed by his court poet Harisena

Q23) Answer (b)

Explanation

Vikrama Samvat was founded by

Chandragupta II in the 58 BC as a

mark of victory over the Shakas and

assumed the title of Vikramaditya

Banabhatta wrote Harshavardhanarsquos

biography the Harshacharita in

Sanskrit

Q24) Answer c

Explanation

Sandhi-vigrahika was the minister of

war and peace

Sarthavaha was the leader of the

merchant caravans

Q25) Answer a

Explanation

Xuan Zang (Hsuan-tsang) was a

Chinese traveller who came during the

reign of Harshavardhana

In the decade that began in 630 AD

Xuan Zang came to India through

Kashmir after visiting Central Asia Iran

and Afghanistan

He travelled from north to east and lived

in Bihar for a couple of years

At Nalanda University Xuan Zang

interacted with students and scholars

mastered local languages and

discovered Buddhist stupas

Q26) Answer c

Explanation

Pradakshina patha is a circular path

laid around a stupa in Buddhist

architecture While the rest are a part of

temple architecture

Q27) Answer d

Explanation

All the above-mentioned temples have

an elaborate use of bricks (baked

bricks) along with stone

Q28) Ans (c)

Muhammad Quli Qutab was the Sultan

of Golconda He was a contemporary of

Akbar was very fond of literature and

architecture

The Sultan was a great poet and he

wrote in Dakhini Urdu Persian and

Telgu and has left an extensive diwan or

collection

Recently the Archaeological Survey of

India (ASI) will be using Ground

Penetrating Radar (GPR) to map the

contours of the area around the Bagh-e-

Naya Qila excavated garden inside the

Golconda Fort in Telangana

Q29) Answer a

Explanation

Silappadikaram is a famous Tamil epic

which was written by Ilango around

1800 years ago It is a story of a

merchant named Kovalan who fell in

love with a courtesan named Madhavi

Manimekalai tells the story of the

daughter of Kovalan and Madhavi

Q30) Answer (a)

Explanation

Charaka is the author of Charaka

Samhita which is an important work of

Ayurveda and medicines

Brahmaguptarsquos fame rests mostly on his

Brahma-sphuta-siddhanta which was

an astronomical work It was translated

into Arabic in Baghdad and had a major

impact on Islamic mathematics and

astronomy

Late in his life Brahmagupta wrote

Khandakhadyaka which was an

RAUSIAS-FC19E1003 44

astronomical handbook that employed

Aryabhatarsquos system of starting each day

at midnight

Q31) Answer (c)

Explanation

Amir Khusrau was a famous sufi

musician poet and scholar In 1318 he

noted that there was different language

in every region of this land (Hindustan)

Lahori Kashmiri Dvarsamudri (in

Southern Karnataka) Telangana (in

Andhra Pradesh) Gujari (in Gujarat)

Marsquobari (in Tamil Nadu) Awadhi (in

eastern Uttar Pradesh) and Hindawai (in

the area around in Delhi) etc He went

to explain that Sanskrit did not belong

to any region and that only brahmans

knew it

Q32) Answer c

Explanation

Hiranyagarbha refers to the golden

womb When this ritual was performed

with the help of Brahmanas it was

thought to lead to the rebirth of the

sacrificer as a Khastriya

Q33) Answer d

Explanation

Kadamai refers to a tax on land

revenue

Gwalior Prashasti describes the exploits

of Nagabhata who was a Pratihara king

Q34) Answer b

Explanation

Rajatarangini is a Sanskrit text written

by Kalhana in the 12th century

It was historical chronicle of early India

It is justifiably considered to be the best

and most authentic work of its kind

It covers the entire span of history in

the Kashmir region from the earliest

times to the date of its composition

Q35) Answer c

Explanation

ldquoUrrdquo was the general assembly of the

village ldquoUrrdquo consisted of all the

taxpaying residents of an ordinary

village

Q36) Answer (a)

Explanation

Tarikh was a form of history writing in

the Delhi Sultanate The authors of

tawarikhs were learned men which

included secretaries administrators etc

Q37 Answer (a)

Explanation

Alauddin chose to pay his soldiers salaries in cash rather than iqtas The soldiers would buy their supplies from merchants in Delhi and it was thus feared that merchants would raise their prices To stop this Alauddin controlled the prices of goods in Delhi Prices were carefully surveyed by officers and merchants who did not sell at the prescribed rates were punished

Q38) Answer (d)

Explanation

Delhi first became the capital of a

kingdom under the Tomara Rajputs

who were defeated in the middle of the

twelfth century by the Chauhans (also

referred to as Chahamanas) of Ajmer

It was under the Tomaras and

Chauhans that Delhi became an

important commercial centre Many rich

Jaina merchants lived in the city and

constructed several temples Coins

minted here called dehliwal had a wide

circulation

Q39) Answer (c)

Explanation

Moth ki Masjid was built in the reign of

Sikandar Lodi by his minister

Begumpuri mosque built in the reign of

Muhammad Tughluq was the main

mosque of Jahanpanah the ldquoSanctuary

of the Worldrdquo and his new capital in

Delhi

Quwwat al ndash Islam mosque was

enlarged by Iltutmish and Alauddin

Khalji The minar was built by three

Sultansndash Qutbuddin Aybak Iltutmish

and Firuz Shah Tughluq

RAUSIAS-FC19E1003 45

Q40) Answer (c)

Explanation

Under the Mughals mansabdar was

referred to an individual who held a

mansab ie rank and he received his

salary as revenue assignments called

jagirs

Q41) Ans (b)

The Quit India Movement was a

spontaneous revolt of people against

British rule

The All India Congress Committee met

at Bombay on 8 August 1942 It passed

the famous resolution Quit India and

proposed the starting of a non-violent

mass struggle under Gandhis

leadership to achieve this aim But on

the very next day Gandhi and other

eminent leaders of the Congress were

arrested The Congress was once again

declared illegal

Q42) Ans (c)

The Simon Commission refers to a

group of seven MPs from the United

Kingdom constituted to suggest

constitutional reforms for British India

The Commission consisted of only

British members headed by one of the

senior British politicians Sir John

Simon

So the people of India agitated against

the arrival of Simon Commission

Q43) Ans (a)

He was widely known for his

unfavourable opinion of the economic

consequences of the British rule in

India

In his many writings and speeches and

especially in Poverty and Un-British

Rule in India Naoroji argued that India

was too highly taxed and that its wealth

was being drained away to England

He did not interpret the ancient Indian

texts and restored the self-confidence of

Indians And also he did not stress the

need for eradication of all the social

evils before anything else

Q44) Ans (c)

In August 1932 Prime Minister

MacDonald announced his Communal

Award Great Britainrsquos unilateral

attempt to resolve the various conflicts

among Indiarsquos many communal

interests

The award which was later

incorporated into the act of 1935

expanded the separate-electorate

formula reserved for Muslims to other

minorities including Sikhs Indian

Christians Anglo-Indians Europeans

distinct regional groups Gandhi

undertook a ldquofast unto deathrdquo against

that offer which he viewed as a

nefarious British plot to divide the

Indian society

Q45) Ans (b)

In British India apart from existing

imports and exports there was also a

particular amount of money which

colonial India contributed towards

administration maintenance of the

army war expenses pensions to retired

officers and other expenses accrued by

Britain towards maintenance of her

colony These were known as Home

charges and were paid for almost

entirely by India

The Home charges was made of

following components-

- Interest payable on Indian debt

- Dividend to shareholders of East

India Company

- Funds used to support the India

Office in London

- Funds used to pay salaries and

pensions of British personnel

engaged in India

- Interest on the railways

- Civil and military charges

- Store purchases in England

Q46) Ans (b)

The Lahore session of the Indian

National Congress was held in 1929

under the Presidentship of Jawaharlal

Nehru

The Lahore session of the Indian

National Congress witnessed significant

RAUSIAS-FC19E1003 46

developments in the Indian national

movement

- First the election of Jawaharlal

Nehru to the post of Presidentship of

the Congress was a clear indication

of the growing strength of the

Leftists in the Congress

- Secondly it was in this session that

the Congress for the first time raised

the demand for complete

independence Such demand was

not raised from the Congress

platform earlier

Q47) Ans (b)

It did not provide for separate

electorates for any community or

weightage for minorities However it did

allow for the reservation of minority

seats in provinces having minorities of

at least ten per cent but this was to be

in strict proportion to the size of the

community

There was no provision for complete

Independence for India

Q48) Ans (c)

The religion of early Vedic Aryans was

primarily of worship of nature and

Yajnas

The early Aryan religion was kind of

nature worship Actually the forces

around them which they could not

control or understand were invested

with divinity and were personified as

male or female gods And they

performed some Yajnas also

Q49) Ans (b)

The roads and river-routes were not

immune from robbery It is notable that

Yuan Chwang (Hiuen Tsang) was

robbed of his belongings during

Harshvardanarsquos period

Q50) Ans (c)

Q51) Ans (b)

Purandara Dasa was a saint and great

devotee of Lord Krishna

There is much speculation about where

Purandara Dasa regarded as the

Pitamaha of Carnatic music was born

Recently an expert committee

constituted by the Kannada University

Hampi has come to the conclusion that

Kshemapura Shivamogga district

Karnataka is the birth place of

Purandara Dasa

Q52) Ans (c)

Sri Tyagaraja Sri Shyama Shastry and Sri Muthuswami Dikshitar are considered the trinity of Carnatic music and with them came the golden age in Carnatic music in the 18th-19th

century

Q53) Ans d)

Recently a rare sarcophagus (stone

coffin) which is 2000 years old from the

Iron AgendashMegalithic era was discovered

from a rock-cut cave at Viyur village of

Kollam near Koyilandy in Kozhikode

district Kerala

The coffin containing bone fragments

was found during an excavation ldquoSo

far such a rare finding has been

discovered only from two sites

in Kerala Both these sarcophagi were

recovered from Megalithic sites at

Chevayur and Atholi also in Kozhikode

district

Q54) Ans a)

The megalithic culture in South India was a full-fledged Iron Age culture

Q55) Ans d)

The Cholas Pandyas and Keralaputras

(Cheras) mentioned in Ashokan

inscriptions were probably in the late

megalithic phase of material culture

Q56) Ans d)

Q57) Ans (b)

Raj Kumar Shukla followed Gandhiji all

over the country to persuade him to

come to Champaran to investigate the

problem associated with tinkathia

system

RAUSIAS-FC19E1003 47

Brij Kishore Rajendra Prasad Mahadev

Desai and Narhari Parikh accompanied

Gandhi ji during the Champaran

Satyagraha

Q58) Ans (b)

The Satvahanas started the practice of granting tax-free villages to brahmanas and Buddhist monks

Q59) Ans c)

The objectives of the Programme are

listed as under

- Developing basic tourism

infrastructure

- Promoting cultural and heritage

value of the country to generate

livelihoods in the identified regions

- Enhancing the tourist attractiveness

in a sustainable manner by

developing world-class

infrastructure at the heritage

monument sites

- Creating employment through active

involvement of local communities

- Harnessing tourism potential for its

effects on employment generation

and economic development

- Developing sustainable tourism

infrastructure and ensuring proper

Operations and maintenance

therein

Q60) Ans (b)

The Tribal Cooperative Marketing

Development Federation of India

(TRIFED) came into existence in 1987

It is a national-level apex organization

functioning under the administrative

control of Ministry of Tribal Affairs

Govt of India

TRIFED has its registered and Head

Office located in New Delhi

Q61) Ans (c)

Premchandrsquos novels include

Premashram Rangabhumi Ghaban

Karmabhumi and Godan

Gora is a novel written by Rabindranath

Tagore

138th birth anniversary of Munshi

Premchand was celebrated across the

country

Q62) Ans (b)

Giddha is a traditional pastoral dance

performed by the women of the Punjab

India and Pakistan at festival times

and at the sowing and reaping of the

harvest

By this dance the Punjabi women

reveal their joy expel their suppressed

feelings in a male dominated society

through the performance of Giddha

Since this dance has nothing to do with

men only women can participate in it

During the Teej celebrations Giddha

dance is celebrated in Punjab every

year Teej is a generic name for a

number of festivals that are celebrated

by women in some parts of India

Q63) Ans (a)

Dara Shukoh wrote the remarkable

work called ldquoMajma-ul-Bahrainrdquo or the

ldquoThe confluence of two seasrdquo

The Vice President of India Shri M

Venkaiah Naidu has said that Prince

Dara Shukohrsquos writings can come as a

refreshing source for infusing peace and

harmony He was addressing the

gathering after visiting the exhibition

that showcases the forgotten Prince of

yesteryears Dara Shukoh organized by

Mr Francois Gautier at Indira Gandhi

National Centre for the Arts in New

Delhi

Q64) Ans (c)

The statue Gommateshwara is

dedicated to the Jain God Bahubali

It is a monolithic statue

President Ram Nath Kovind

inaugurated the grand anointing

ceremony mdash Mahamastakabhisheka mdash

held once in 12 years at

Shravanabelagola (Karnataka)

Q65) Ans (c)

Prachi Valley had come up around the

Prachi river Prachi Valley gradually

disappeared

RAUSIAS-FC19E1003 48

The Prachi river originates from

Bhubaneswar

It is a tributary of the Mahanadi and

flows through the districts of Puri

Khurda Cuttack and Jagatsinghpur

and the entire region of the river is

termed as the Prachi Valley

It falls into the Bay of Bengal

Archaeological evidence shows that the

Prachi Valley Civilisation predates both

Harappa and Mohenjo-Daro

The Prachi river originates from

Bhubaneswar

Q66) Ans (d)

These monuments are located in

Chhatarpur district Madhya Pradesh

within Vindhya mountain range

Q67) Ans (a)

The book lsquoThoughts on Pakistanrsquo was

written by Dr BR Ambedkar

On the occasion of the birth anniversary

of Dr BR Ambedkar the president of

India pays homage to this icon of India

In 1924 he founded the Depressed

Classes Institute (Bahishkrit Hitkarini

Sabha) and in 1927 the Samaj Samata

Sangh

Another area of attention for Ambedkar

was education For its spread among

the low classes he set up a network of

colleges by the name of Peoples

Education Society and founded hostels

Q68) Ans(b)

Mehrgarh is a famous Neolithic

settlement in the Indian subcontinent

which is situated in Baluchistan

province Pakistan

A pre-historic rock art site is discovered

in the vast expanse of limestone blocks

on the eastern banks of Naguleru river

near Dachepalli (Andhra Pradesh) It

has thrown light on the Neolithic

civilisation that flourished in Guntur

(Andhra Pradesh) during 1500-2000

BC

Q69) Ans (c)

The 12th and the 13th centuries saw

the emergence of the Kakatiyas They

were at first the feudatories of the

Western Chalukyas of Kalyana Initially

they ruled over a small territory near

Warangal (Telangana)

They introduced Nayakships which was

later adopted and developed by the

Rayas of Vijayanagara

Q70) Ans (a)

The fast had effect of putting pressure

on mill owners who finally agreed to

give the workers a 35 per cent increase

in wages

Google celebrated with a doodle the

132nd birth anniversary of Anasuya

Sarabhai who played a pioneering role

in Indiarsquos labour movement

Q71) Ans (d)

The UNESCOrsquos list of the representative

list of the intangible cultural heritage of

humanity from India are

- Koodiyattam Sanskrit Theatre of

Kerala

- Mudiyettu ritual theatre and dance

drama of Kerala

- Tradition of Vedic Chanting

- Kalbelia folk songs and dances of

Rajasthan

- Ramlila Traditional Performance of

the Ramayana

- Sankirtana ritual singing

drumming and dancing of Manipur

- Ramman religious festival and

ritual theatre of the Garhwal

Himalayas India

- Traditional brass and copper craft of

utensil making among the Thatheras

of Jandiala Guru Punjab India

- Chhau dance classical Indian dance

originated in the eastern Indian

states

- Buddhist chanting of Ladakh

recitation of sacred Buddhist texts

in the trans-Himalayan Ladakh

region Jammu and Kashmir India

- Yoga

- Nouroz

- Kumbh Mela

RAUSIAS-FC19E1003 49

Q72) Ans(b)

The President of India Shri Ram Nath Kovind inaugurated the Hornbill Festival and State Formation Day celebrations of Nagaland in Kisama

The festival is named after the Indian hornbill the large and colourful forest bird which is displayed in the folklore of most of the states tribes

The major recognized tribes of Nagaland are Angami Ao Chakhesang Chang

Kuki Rengma and Zeling etc

Onge Jarawa and Sentinelese are the

tribes of Andman amp Nicobar Islands

Q73) Ans (c)

The Rashtrakutas rule in the Deccan lasted for almost two hundred years till the end of the tenth century The Rashtrakutas rulers were tolerant in their religious views and patronized not only Shaivism and Vaishnavism but

Jainism as well

The famous rock-cut temple of Shiva at Ellora was built by one of the Rashtrakutas kings Krishna I in the ninth century His successor Amoghavarsha was a Jain but he also

patronized other faiths

The Rashtrakutas allowed Muslims traders to settle and permitted Islam to

be preached in their dominions

Recently increasing defacement at the prehistoric rock paintings of Pandavulagutta Telangana has created a cause for grave concern It can spoil

the prehistoric rock

Pandavulagutta is home to

- Painted rock shelters dating to

10000 BC-8000 BC

- An 8th century inscription of the

Rashtrakuta period and

- Painted frescoes from the 12th century Kakatiya empire

Q74) Ans (b)

In 1828 Raja Ram Mohan Roy founded a new religious society the Brahma Sabha later known as the Brahmo

Samaj

Debendranath Tagore headed the Tattvabodhini Sabha which was

engaged in search of spiritual truth

Its purpose was to purify Hinduism and to preach monotheism or belief in one God

The new society was to be based on the twin pillars of reason and the Vedas and

Upanishads

Recently Sadharan Brahmo Samaj (SBS) has entered into a legal battle with the West Bengal government due

to some legal issue

Q75) Ans (c)

The Chishti order was established in India by Khwaja Moinuddin Chishti who came to India around 1192 The Chishtirsquos are considered to be the most influential of the groups of Sufis who migrated to India in the late twelfth century They adapted successfully to the local environment and adopted several features of Indian devotional

traditions

The historical dargah of Sufi mystic Khwaja Moinuddin Chishti in Ajmer is all set to get a facelift This 13 th century dargah has been included among the Swachh Iconic Places a clean-up initiative focused on iconic

heritage spiritual and cultural places

Page 25: GENERAL STUDIES (PAPER I) · Test is part of Rau’s IAS Test series for Preliminary Exam 2019 FOUNDATION + CURRENT AFFAIRS GENERAL STUDIES (PAPER –I) FOUNDATION TEST –III TOPIC:

RAUSIAS-FC19E1003 25

Q64) Consider the following statements

1 Gommateshwara Statue is located

on the Vindyagiri Hill

2 Shravanabelagola is the place

where Chandragupta Maurya the

founder of the Mauryan dynasty

became a Jain ascetic after

relinquishing his throne

Which of the statements given above

isare correct

(a) 1 only

(b) 2 only

(c) Both 1 and 2

(d) Neither 1 nor 2

Q65) Consider the following statements

1 Archaeological evidence shows

that the Prachi Valley Civilisation

predates both Harappa and

Mohenjo-Daro

2 The Prachi river originates from

Bhubaneswar

Which of the statements given above

isare correct

(a) 1 only

(b) 2 only

(c) Both 1 and 2

(d) Neither 1 nor 2

Q66) Which of the following statements

isare correct

1 The Khajuraho group of

monuments was built during the

rule of the Chandela dynasty

2 These monuments are located in

Harischandra mountain range

3 Ibn Battuta the Moroccan

traveller in his memoirs mentioned

visiting Khajuraho temples and

called them Kajarra

Select the correct answer using the code

given below

(a) 1 only

(b) 1 and 2

(c) 2 and 3

(d) 1 and 3

Q67) Which of the following statements

isare correct

1 Dr BR Ambedkar wrote the

Annihilation of Caste emphasising

the need to do away with the

practice of hereditary priesthood in

Hinduism

2 The book lsquoThoughts on Pakistanrsquo

was written by Dr Rajendra

Prasad

Select the correct answer using the code

given below

(a) 1 only

(b) 2 only

(c) Both 1 and 2

(d) Neither 1 nor 2

Q68) Which of the following statements

isare correct

1 Mehrgarh is a famous Neolithic

settlement in the Indian

subcontinent which is situated in

Sindh province Pakistan

2 At Burzahom dogs were buried

with their masters in their graves

Select the correct answer using the code

given below

(a) 1 only

(b) 2 only

(c) Both 1 and 2

(d) Neither 1 nor 2

Q69) Which of the following statements

isare correct

1 The Kakatiya temples are

dedicated mostly to Siva

2 The Thousand-Pillared Temple at

Hanamkonda was built by the

Kakatiya king Rudra

Select the correct answer using the code

given below

(a) 1 only

(b) 2 only

(c) Both 1 and 2

(d) Neither 1 nor 2

RAUSIAS-FC19E1003 26

Q70) निमननिखित कथि ो म स कौि-सास सही हह

1 अहमदाबाद नमि हड़ताि क दौराि महातमा

गाोरी ि शरनमक ो क पकष क मजबत करि क

निए आमरर अिशि नकया था

2 अिशि स नमि मानिक ो पर दबाि पड़ा था ज

अोततः शरनमक ो क िति म 15 परनतशत की िखदध

करि क निए सहमत हए थ

िीच नदए गए कट का परय ग कर सही उततर चनिए

(a) किि 1

(b) किि 2

(c) 1 और 2 द ि ो

(d) ि त 1 ि ही 2

Q71) निमननिखित म स नकसक नकिक भारत स यिसक

की माििता की अमतण साोसकनतक निरासत की

परनतनिनर सची (The UNESCOrsquos List of the

Representative List of the Intangible

Cultural Heritage of Humanity) म शानमि

नकया गया ह

1 मनडयटट

2 सोकीतणि

3 को भ मिा

िीच नदए गए कट का परय ग कर सही उततर चनिए

(a) किि 1 और 2

(b) किि 2 और 3

(c) किि 3

(d) 1 2 और 3

Q72) निमननिखित जिजानतय ो म स कौि-सीसी ो

जिजानतजिजानतया िागािड स सोबोनरत हह

1 अोगामी

2 ककी

3 जारिा

िीच नदए गए कट का परय ग कर सही उततर चनिए

(a) किि 1

(b) किि 1 औऔ 2

(c) किि 2

(d) 1 2 और 3

Q73) निमननिखित कथि ो म स कौि-सास सही हह

1 राषटर कट सामराजय की सथापिा दोनतदगण ि की थी

नजसि मानयाित म अपिी राजरािी की

सथापिा की थी

2 राषटर कट समराट अम घििण एक ििक था और

उस कनिताओो पर पहिी कननड़ पसतक नििि

का शरय नदया जाता ह

िीच नदए गए कट का परय ग कर सही उततर चनिए

(a) किि 1

(b) किि 2

(c) 1 और 2 द ि ो

(d) ि त 1 ि ही 2

Q74) निमननिखित कथि ो म स कौि-सास सही हह

1 कशब चोदर सि ि ततवब नरिी सभा की

अधयकषता की थी ज आधयाखतमक सतय की

ि ज म सोिि थी

2 बरहम समाज ि मािि गररमा पर बि नदया

मनतणपजा का निर र नकया और सती परथा जसी

सामानजक बराइय ो की आि चिा की

िीच नदए गए कट का परय ग कर सही उततर चनिए

(a) किि 1

(b) किि 2

(c) 1 और 2 द ि ो

(d) ि त 1 ि ही 2

Q75) निमननिखित कथि ो म स कौि-सास सही हह

1 भारत म नचशती नसिनसिा खवाजा म इिददीि

नचशती क दवारा सथानपत नकया गया था

2 नचशती परोपरा की एक परमि निशिता

आतमसोयम थी नजसम साोसाररक म ह स दरी

बिाए रििा शानमि था

िीच नदए गए कट का परय ग कर सही उततर चनिए

(a) किि 1

(b) किि 2

(c) 1 और 2 द ि ो

(d) ि त 1 ि ही 2

RAUSIAS-FC19E1003 27

Q70) Which of the following statements

isare correct

1 During the Ahmedabad Mill Strike

Mahatma Gandhi undertook a fast

unto death to strengthen the

workersrsquo resolve

2 The fast had effect of putting

pressure on mill owners who

finally agreed to give the workers a

15 per cent increase in wages

Select the correct answer using the code

given below

(a) 1 only

(b) 2 only

(c) Both 1 and 2

(d) Neither 1 nor 2

Q71) Which of the following are included in

the UNESCOrsquos list of the representative

list of the intangible cultural heritage of

humanity from India

1 Mudiyettu

2 Sankirtana

3 Kumbh Mela

Select the correct answer using the code

given below

(a) 1 and 2 only

(b) 2 and 3 only

(c) 3 only

(d) 1 2 and 3

Q72) Which of the following tribes isare

related to Nagaland

1 Angami

2 Kuki

3 Jarawa

Select the correct answer using the code

given below

(a) 1 only

(b) 1 and 2 only

(c) 2 only

(d) 1 2 and 3

Q73) Which of the following statements

isare correct

1 Rashtrakuta kingdom was founded by Dantidurga who established his capital at Manyakhet

2 Amoghavarsha a Rashtrakuta king was an author and is credited with writing the first

Kannada book on poetics

Select the correct answer using the code given below

(a) 1 only

(b) 2 only

(c) Both 1 and 2

(d) Neither 1 nor 2

Q74) Which of the following statements isare correct

1 Keshab Chandra Sen headed the Tattvabodhini Sabha which was engaged in search of spiritual truth

2 The Brahmo Samaj laid emphasis on human dignity opposed idolatry and criticized such social

evils as the practice of Sati

Select the correct answer using the code given below

(a) 1 only

(b) 2 only

(c) Both 1 and 2

(d) Neither 1 nor 2

Q75) Which of the following statements isare correct

1 The Chishti order was established in India by Khwaja Moinuddin

Chishti

2 A major feature of the Chishti tradition was austerity including maintaining a distance from the

worldly power

Select the correct answer using the code

given below

(a) 1 only

(b) 2 only

(c) Both 1 and 2

(d) Neither 1 nor 2

T e s t i s p a r t o f R a u rsquo s I A S T e s t s e r i e s f o r P r e l i m i n a r y E x a m 2 0 1 9

FOUNDATION + CURRENT AFFAIRS

GENERAL STUDIES (PAPER ndashI)

FOUNDATION TEST ndashIII

SUBJECT NCERT History Class VI-X + Current Affairs

Time Allowed 1frac12 Hours Maximum Marks 150

I NSTRUCT IONS

1 IMMEDIATELY AFTER THE COMMENCEMENT OF THE EXAMINATION YOU SHOULD CHECK

THAT THIS TEST BOOKLET DOES NOT HAVE ANY UNPRINTED OR TORN or MISSING PAGES OR

ITEMS ETC IF SO GET IT REPLACED BY A COMPLETE TEST BOOKLET

2 This Test Booklet contains 75 items (questions) Each item is printed both in Hindi and English

Each item comprises four responses (answers) You will select the response which you want to mark

on the Answer Sheet In case you feel that there is more than one correct response mark the

response which you consider the best In any case choose ONLY ONE response for each item

3 You have to mark all your responses ONLY on the separate Answer Sheet (OMR sheet) provided

Read the directions in the Answer Sheet

4 All items carry equal marks

5 Before you proceed to mark in the Answer Sheet the response to various items in the Test booklet

you have to fill in some particulars in the Answer Sheet as per instructions contained therein

6 After you have completed filling in all your responses on the Answer Sheet and the examination has

concluded you should hand over to the Invigilator only the Answer Sheet You are permitted to

take away with you the Test Booklet

7 Penalty for wrong answers

THERE WILL BE PENALTY FOR WRONG ANSWERS MARKED BY A CANDIDATE IN THE

OBJECTIVE TYPE QUESTION PAPERS

(i) There are four alternatives for the answer to every question For each question for which a

wrong answer has been given by the candidate one-third of the marks assigned to that

question will be deducted as penalty

(ii) If a candidate gives more than one answer it will be treated as a wrong answer even if one of

the given answers happens to be correct and there will be same penalty as above to that

question

(iii) If a question is left blank ie no answer is given by the candidate there will be no penalty for

that question

T h i s t e s t i s p a r t o f R a u rsquo s I A S T e s t s e r i e s f o r P r e l i m i n a r y E x a m 2 0 1 9

Test Code

FC19E1003

FC19H1003 29

Answers and Explanations of

NCERT History Class VI-X + Current Affairs (FC19E1003)

Q1) उततर (c)

सपषटीकरण

- ऋगवद म दविय ो और दिताओो क समवपित एक

हजार स अविक सत तर (शल क) ह

- य शल क ऋविय ो क दवारा रच गए थ और परि ो

दवारा सीख जात थ

- हालाोवक कछ शल क मवहलाओो (जस वक अपाला

घ सा ल पामदरा मतरयी और गागी) क दवारा भी रच

गए थ

- ऋगवद म सोिाद क रप म कई शल क मौजद ह

- हम विशवावमतर नामक एक ऋवि और दविय ो क

रप म पजी जान िाली द नवदय ो (वयास और

सतलज) क बीच िाताि का उदाहरण वमलता ह

- इसस पता चलता ह वक विशवावमतर िवदक काल स

सोबोवित थ

Q2) उततर (b)

सपषटीकरण

- करनल गफाओो स राख क अिशि परापत हए ह

ज इस ओर सोकत करत ह वक ततकालीन ल ग

अवि क उपय ग स पररवचत थ

- य गफाएो आोधर परदश म सथथत ह

Q3) उततर (c)

सपषटीकरण

bull बरािह म ितिमान कशमीर म सथथत एक

परागवतहावसक थथल ह जहाो ल ग गडढ क घर ो का

वनमािण करत थ

bull य घर जमीन क ख द कर बनाए जात थ तथा नीच

जान क वलए सीवियाा ह ती थी

bull ऐसा अनमान लगाया जाता ह वक य घर ठो ड क

मौसम म आशरय परदान करत थ

Q4) उततर (c)

सपषटीकरण

bull परालख-विदया (Epigraphy) क वशलालख ो क

अधययन क रप म पररभावित वकया जाता ह

bull हसतवलसखत दसतािज ो क माधयम स इवतहास

और सावहतय क अधययन क पाोडवलवप विजञान

(Manuscriptology) कहत ह

bull पराचीन लखन परणावलय ो क अधययन और

ऐवतहावसक पाोडवलवपय ो क समझन तथा वतवथ

वनिािरण क पलीओगराफी (Palaeography) कहा

जाता ह

bull नयवमजमविकस (Numismatics) वसक ो क

अधययन क सोदवभित करता ह

Q5) उततर (a)

सपषटीकरण

- चरक सोवहता चरक क दवारा वलखी गई आयिद

और िदयक-शासर पर एक महतवपणि पसतक ह

- ि भारतीय िदयक-शासर की पारमपररक परणाली

वजस आयिद क नाम स जाना जाता ह क

अभयासकताि थ

- ऐसा माना जाता ह वक चरक का विकास दसरी

शताबदी (ईसा पिि) और दसरी शताबदी (ईसवी) क

मधय हआ था

Q6) उततर (b)

सपषटीकरण

- भाग फसल ो पर वलए जान िाल कर क सोदवभित

करता ह ज कल फसल उतपादन का 16 िाो भाग

था

- ldquoकममकारrdquo शबद भवमहीन कवि शरवमक िगि क

वलए परय ग वकया जाता था

- ldquoअशवमिrdquo (वजस घ ड क बवलदान क रप म भी

जाना जाता ह) एक अनषठान ह ता था वजसम एक

घ ड क सवतोतर रप स घमन क वलए छ ड वदया

FC19H1003 30

जाता ह और राजा क सवनक उसकी रखिाली

करत थ

Q7) उततर (d)

सपषटीकरण

- ऋगववदक काल म घ ड ो क रथ ो म ज ता जाता था

ज (रथ) भवम मिवशय ो आवद पर कबजा करन क

वलए लड गए यद ो म उपय ग वकए जात थ

- इसस यह पता चलता ह वक घ ड ो यकत रथ ो का

उपय ग महाजनपद काल स काफी पहल आरमभ

हआ था

- ऋगववदक काल म मिवशय ो भवम जल आवद पर

कबजा करन क वलए तथा ल ग ो क पकडन क

वलए यद वकय जात थ

- अविकाोश परि इन यद ो म भाग वलया करत थ

- हालाोवक उस समय क ई वनयवमत सना नही ो ह ती

थी लवकन उस काल म सभाऐो ह ती थी ो वजनम

ल ग यद क मामल ो पर चचाि करत थ

- वनयवमत सनाएा महाजनपद काल का िवशषटय थी

वजनम पदल सवनक ो की विशाल सनाएा रथ तथा

हाथी शावमल ह त थ

Q8) उततर (a)

सपषटीकरण

- बद शाकय कल स सोबोवित थ और कशीनारा म

उनका वनिन हआ था

- बद न अपनी वशकषाएा पराकत भािा म दी थी ो ज

आम ल ग ो की भािा थी

Q9) उततर (c)

सपषटीकरण

- पराचीन भारत म दशिनशासर की छह शाखाएा थी ो

िशविक नयाय समखया य ग पिि वममाोसा और

िदाोत या उततर वममाोसा

- इनकी थथापना करमश कनाद गौतम कवपल

पतोजवल जावमनी और वयास ऋविय ो न की थी

Q10) उततर (b)

सपषटीकरण

महािीर की वशकषाऐो छठी शताबदी म िललभी म

सोकवलत की गई थी ो

Q11) उततर (c)

सपषटीकरण

- पारमपररक रप स चाणकय क कौविलय अथिा

विषणगपत क नाम स जाना जाता ह

- उसन अथिशासतर ज एक पराचीन भारतीय

राजनवतक आलख ह वलखा था

Q12) उततर (d)

सपषटीकरण

- भारत का राषटर ीय वचनह सारनाथ (उततर परदश) क

अश क सतमभ क ऊपर (शीिि पर) वसोह कवपिल

का एक अनरपण ह

- इस राषटर ीय वसदाोत सतयमि जयत क साथ

सोय वजत वकया गया ह

- रामपिि बल का नाम रामपिि (वबहार) क नाम पर

पडा जहाा इसकी ख ज हई थी

- यह अपन नाजक नकाशी मॉडल क वलए परवसदद

ह वजसम क मल तवचा सोिदनशील नथन ो सतकि

कान और मरबत िााग ो क शरषठतर परवतरप क

परदवशित वकया गया ह

- यह भारतीय और फारसी ततव ो का एक ससममशरण

- सोवकससा उततर परदश म सथथत ह

Q13) उततर (a)

सपषटीकरण

का िर वसोह ज एक महान य दा थ वबहार स

सोबोवित थ

Q14) उततर (b)

सपषटीकरण

िललालर शबद बड भ-सवावमय ो क वलए परय ग

वकया जाता था

FC19H1003 31

Q15) उततर (c)

सपषटीकरण

- अररकमड एक तिीय बसती थी जहाो दर दश ो स

आन िाल जहाज ो का माल उतारा जाता था

- यहाो पर ईोि ो का एक विशाल ग दाम वमटटी क

बतिन (वजनम एमफ रा - द हरी मवठय ो का लोबा

घडा - शावमल ह) और एरिाइन (Arretine)

मदभाोड पाए गए थ

- इस थथान पर र मन दीपक काोच क बन पातर और

रतन भी पाए गए थ

Q16) उततर (a)

सपषटीकरण

- मिनदर सोगम कविताओो म उसललसखत एक

तवमल शबद ह वजसका अथि ह ldquoतीन परमखrdquo

- यह तीन सततारि पररिार ो क मसखयाओो क वलए

परय ग वकया जाता ह च ल चर और पाणडय

Q17) उततर (c)

सपषटीकरण

- ऋग िद म सभा विदाथा तथा गण जसी

जनजावतय ो पर अथिा किोब पर आिाररत

सभाओो का उललख ह

- आरसमभक िवदक काल म सभाओो और सवमवतय ो

का विशि महतव ह ता था

- यहाा तक की मसखया अथिा राजा भी उनका

समथिन परापत करन क वलए आतर रहत थ

Q18) उततर (a)

सपषटीकरण

- जन िमि न ईशवर क अससततव क मानयता त दी ह

वकनत उसन ईशवर क वजना क पद स नीच रखा

- जन िमि न बौद िमि की तरह िणि परणाली की

भरतिना नही ो की थी

Q19) उततर (d)

सपषटीकरण

- च ल ो और पाणडय ो न शसकतशाली तिीय शहर ो का

विकास वकया था

- च ल ो का सबस महतवपणि शहर पहार (या

कािरीपटटीनम) था |

- मदरई पाणडय ो की राजिानी थी

Q20) उततर (b)

सपषटीकरण

- ldquoबदचररतrdquo बद का जीिन-ितताोत ह

- इस अशवघ ि क दवारा वलखा गया था

Q21) उततर (a)

सपषटीकरणः

- तवमल कवि अपपर भगिान वशि क भकत थ

- इस परकार ि एक नयनार सोत थ

Q22) उततर (d)

सपषटीकरणः

- समदरगपत एक परवसद गपत शासक था

- उसन वसक ो पर िीणा बजात हए अपनी छवि

अोवकत करिाई थी

- यह सोगीत क परवत उसक परम क दशािता ह

- हम उसकी इलाहाबाद परशससत स महतवपणि

ऐवतहावसक जानकारी वमलती ह वजसकी रचना

उसक दरबार क कवि हररसन न की थी

Q23) उततर (b)

सपषटीकरणः

- विकरम सोित की शरआत ििि 58 ईसा पिि म

चनदरगपत वदवतीय न की थी

- यह शक ो पर उसकी जीत और उस विकरमावदतय

की पदिी वमलन क उपलकषय म आरमभ वकया गया

था

FC19H1003 32

- बानभटट न हिििििन का जीिन-ितताोत हििचररत

(ज सोसकत म थी) वलखी थी

Q24) उततर (c)

सपषटीकरणः

- सोवि-विगरावहका यद एिो शाोवत का मोतरी

- साथििाह वयापाररय ो क कावफल ो का नता

Q25) उततर (a)

सपषटीकरणः

- जआन झाोग (हसआन रताोग ndash Hsuang Tsang)

एक चीनी यातरी था ज हिििििन क शासनकाल म

भारत आया था

- ििि 630 ईसवी स ज दशक आरमभ हआ था उसम

जआन झाोग मधय एवशया ईरान और

अफग़ावनसतान की यातरा करन क पशचात कशमीर

क रासत स भारत आया था

- उसन उततर स पिि तक की यातरा की और िह

लगभग 2 ििि वबहार म रहा

- जआन झाोग न नालनदा विशवविदयालय म विदयावथिय ो

और विदवान ो क साथ पारसपररक विचार-विमशि

वकया थथानीय भािाओ ा म वनपणता परापत की तथा

बौद सतप ो की ख ज की

Q26) उततर (c)

सपषटीकरणः

- परदवकषणा पथ बौद िासतकला म सतप क चार ो

ओर बनाया जान िाला एक घमािदार पथ ह ता

- परशन म वदए गए बाकी क तीन ो ततव वहोद मसनदर ो की

िासतकला क भाग ह

Q27) उततर (d)

सपषटीकरणः

परशन म वदए गए सभी मोवदर ो म वयापक रप स

ईोि ो (पकी ईोि ो) का परय ग पतथर ो क साथ हआ

Q28) उततर (c)

सपषटीकरण

- महममद कली कतब शाह ग लकणडा का सलतान

था

- िह अकबर का समकालीन था

- सावहतय और िासतकला म उसकी अतयाविक

रवच थी

- िह एक महान कवि था

- िह दसखनी उदि फारसी और तलग म वलखता था

- उसन अपन पीछ एक विसतत वदिान (सोगरह)

छ डा ह

- अभी हाल ही म तलोगाना म ग लकणडा क वकल

क अनदर खदाई वकय गए बाग-ए-नाया वकला

बाग क चार ो ओर रप-रखा क मानवचतरण क

वलए भारतीय परातासतवक सिकषण (The

Archaeological Survey of India ndash ASI)

गराउणड पनीिर विोग रडार (Ground Penetrating

Radar) का परय ग करगा

Q29) उततर (a)

सपषटीकरणः

- वसलपपावदकारम एक तवमल महाकावय ह वजसकी

रचना इलाोग क दवारा लगभग 1800 ििि पिि की

गई थी

- यह क िलन नामक एक वयापारी की कहानी ह

ज माििी नामक एक गवणका (िशया) स परम

करन लगा था

- मवनमकलाई क िलन और माििी की पतरी की

कहानी ह

Q30) उततर (a)

सपषटीकरण

- चरक आयिद और वचवकरता की एक महतवपणि

रचना चरक सोवहता क लखक ह

- बरहमगपत क अपनी रचना बरहम-सफि-वसदानत

(ज एक खग लीय रचना ह) क कारण परवससद

वमली

FC19H1003 33

- बगदाद म इसका अनिाद अरबी भािा म वकया

गया था

- इसका इसलावमक गवणत और खग ल-विजञान पर

महतवपणि परभाि पडा था

- बाद म अपन जीिनकाल म बरहमगपत न

ldquoखोडखयाकrdquo वलखी ज एक खग लीय पससतका

(एक छ िी पसतक) थी

- इसम आयिभटट की अिि-रावतर क परतयक वदन की

शरआत परणाली का परय ग वकया गया था

Q31) उततर (c)

सपषटीकरण

- अमीर खसर एक परवसद सफी सोगीतकार कवि

और विदवान थ

- 1318 म उनह ोन पाया वक इस भवम (वहोदसतान) क

हर कषतर म अलग-अलग भािा थी लाहौरी

कशमीरी दवारसमदरी (दवकषणी कनाििक म)

तलोगाना (आोधर परदश म) गजरी (गजरात म)

माबारी (तवमलनाड म ) अििी (पिी उततर परदश

म) और वहोदिी (वदलली क आस-पास क कषतर म)

आवद

- उनह न यह बताया वक सोसकत वकसी भी कषतर स

सोबोवित नही ो थी और किल बराहमण ही इस भािा

का जञान रखत थ

Q32) उततर (c)

सपषटीकरण

- वहरणय-गभि सववणिम गभि क सोदवभित करता ह

- जब बराहमण ो की सहायता स यह अनषठान वकया

जाता था त यह माना जाता था वक बवल दन िाल

का कषवतरय क रप म पनजिनम ह गा

Q33) उततर (d)

सपषटीकरण

- कदमई भवम राजसव पर कर क सोदवभित करता

- गवावलयर परशससत म नागभि क दवारा वकय गए

श िण का िणिन वकया गया ह |

- नागभि एक परवतहार राजा था

Q34) उततर (b)

सपषटीकरण

- राजतरो वगनी 12िी ो शताबदी म कलहन क दवारा

रवचत एक सोसकत पसतक (िकसट) ह

- यह परारसमभक भारत की ऐवतहावसक इवतितत थी

- तकि सोगत रप स इस अपन परकार की सिोततम

और सिािविक विशवसनीय कवत माना जाता ह

- यह कशमीर कषतर क पराचीनतम समय स लकर

उसकी रचना की तारीख तक क समपणि इवतहास

का आचछादन करती ह

Q35) उततर (c)

सपषटीकरण

- गााि की आम सभा क ldquoउरrdquo कहा जाता था

- ldquoउरrdquo म गााि क सभी कर दन िाल वनिासी

शावमल ह त थ

Q36) उततर (a)

सपषटीकरण

- वदलली सलतनत म ldquoतारीखrdquo इवतहास लखन का

एक रप था

- ldquoतािरीखrdquo क लखक विदवान परि ह त थ वजनम

सवचि परशासक इतयावद शावमल थ

Q37) उततर (a)

सपषटीकरण

- अलाउददीन सखलजी अपन सवनक ो क ितन का

भगतान नकद म करता था न वक इकता क रप

- सवनक अपना सामान वदलली म वयापाररय ो स

खरीदत थ अतः इस बात का भय था वक वयापारी

कही ो िसतओो का मलय न बिा द

- इसकी र कथाम क वलए अलाउददीन सखलजी न

वदलली म कीमत ो क वनयसित वकया

FC19H1003 34

- अविकारीगण धयानपििक मलय ो का सिकषण करत

थ तथा ज वयापारी वनिािररत मलय पर माल नही ो

बचत थ उनक दसणडत वकया जाता था

Q38) उततर (d)

सपषटीकरण

- वदलली सििपरथम त मर राजपत ो क अिीन उनक

सामराजय की राजिानी बनी थी

- 12िी ो शताबदी क मधय म अजमर क चौहान ो

(वजनह चाहमान ो क नाम स भी जाना जाता ह) न

त मर राजपत ो क परावजत वकया था

- त मर ो और चौहान ो क अिीन वदलली एक

महतवपणि िावणसजयक क दर बन गया था

- कई जन वयापारी यहाा रहन लग थ और उनह ोन

कई मोवदर भी बनिाए

- यहाा पर मवदरत वसक वजनह ldquoदहलीिालrdquo क नाम

स जाना जाता था वयापक रप स परचलन म थ

Q39) उततर (c)

सपषटीकरण

- म ठ की मसिद का वनमािण वसको दर ल दी क

राजयकाल म उसक मिी क दवारा करिाया गया

था

- बगमपरी मसिद का वनमािण महममद तगलक क

शासनकाल म हआ था

- यह मसिद विशव का पणयथथान (The

Sanctuary of the World) और वदलली म महममद

तगलक की नई राजिानी जहाोपनाह की मखय

मसिद थी

- कववत- अल - इसलाम मसिद का विसतार

इलतसिश और अलाउददीन सखलजी न वकया था

- मीनार का वनमािण तीन सलतान ो कतबददीन ऐबक

इलतसिश और वफर ज शाह तगलक क दवारा

करिाया गया था

Q40) उततर (c)

सपषटीकरण

- मगल ो क अिीन मनसबदार शबद उस वयसकत क

वलए सोदवभित वकया जाता था वजसक पास मनसब

(अथाित पद) ह ता था

- उस अपना ितन राजसव कायो वजनह जागीर कहत

थ क रप म परापत ह ता था

Q41) उततर (b)

सपषटीकरण

- ldquoभारत छ ड आोद लनrdquo वबरविश शासन क

सखलाफ ल ग ो का एक सवाभाविक विदर ह था

- असखल भारतीय काोगरस सवमवत न 8 अगसत 1942

क बमबई म एक बठक का आय जन वकया था

- इस बठक म परवसद सोकलप ldquoभारत छ ड rdquo क

पाररत वकया गया और इस उददशय क परापत करन

क वलए गाोिी क नततव म एक अवहोसक जन सोघिि

आोद लन की शरआत का परसताि वदया गया

- लवकन अगल ही वदन गाोिी और काोगरस क अनय

परमख नताओो क वगरफतार कर वलया गया

- काोगरस क एक बार वफर अिि घ वित वकया गया

था

Q42) उततर (c)

सपषटीकरण

- साइमन कमीशन यनाइविड वको गडम क सात

साोसद ो का एक समह था

- इस वबरविश भारत क वलए सोििावनक सिार ो का

सझाि दन क वलए गवठत वकया गया था

- इस आय ग म िररषठ वबरविश राजनता सर जॉन

साइमन क नततव म किल वबरविश सदसय ही

शावमल थ

- इसवलए भारत क ल ग ो न साइमन कमीशन क

आगमन क विरद आोद लन वकया था

Q43) उततर (a)

सपषटीकरण

bull दादा भाई नौर जी भारत म वबरविश शासन क

आवथिक पररणाम ो क बार म अपनी विर िी

(परवतकल) राय क वलए जान जात थ

FC19H1003 35

bull अपन कई लख ो और भािण ो म विशि रप स

ldquoपाििी एो ड अन-वबरविश रल इन इसणडया

(Poverty and Un-British Rule in India) म

नौर जी न यह तकि वदया वक भारत पर अतयविक

कर लगाया गया था और इसकी सोपवतत इोगलड की

ओर परिावहत की जा रही थी

bull उनह ोन पराचीन भारतीय गरोथ ो की वयाखया करन

का और भारतीय ो क आिविशवास क बहाल

करन पर कायि नही ो वकया था

उनह ोन वकसी और बात स पहल सभी सामावजक

बराइय ो क उनमलन की आिशयकता पर भी बल

नही ो वदया था

Q44) उततर (c)

सपषटीकरण

bull अगसत 1932 म वबरविश परिानमोतरी मकड नालड न

अपन साोपरदावयक परसकार (The Communal

Award) की घ िणा की थी

bull यह भारत क कई साोपरदावयक वहत ो क बीच विवभनन

सोघिो क हल करन क वलए वबरिन का एकतरफा

परयास था

bull यह परसकार (Award) बाद म 1935 क

अविवनयम (The Act of 1935) म शावमल वकया

गया था

bull इस साोपरदावयक परसकार न मससलम ो क वलए

आरवकषत एक अलग वनिािचक मणडल फॉमिल का

विसतार अनय अलपसोखयक ो क वलए वकया था

वजसम वसख ो भारतीय ईसाइय ो आोगल-भारतीय

समदाय यर पीय समदाय तथा विवशषट कषतरीय

समह ो क शावमल वकया गया था

bull गाोिी न इस परसताि क भारतीय समाज क

विभावजत करन क वलए एक घवणत वबरविश

सावजश क रप म दखा और उसक सखलाफ

आमरण अनशन वकया

Q45) उततर (b)

सपषटीकरण

मौजदा आयात और वनयाित क अवतररक़त

औपवनिवशक भारत क वनमनवलसखत खचो क

वलए एक विशिवनवशचत िन रावश भी दनी पडती

थी

(i) परशासन क वयय

(ii) सना क रख-रखाि क वयय

(iii) यद क वयय

(iv) सिावनितत अविकाररय ो की पशन तथा

(v) वबरिन दवारा अपनी उपवनिश बसती

(कॉल नी) क रख-रखाि क वयय

इनह गह शलक (Home Charges) क रप म

जाना जाता था और लगभग परी तरह स भारत क

दवारा इनका भगतान वकया जाता था

bull गह शलक म वनमनवलसखत घिक शावमल थ

(i) भारतीय ऋण पर दय बयाज

(ii) ईसट इोवडया को पनी क शयरिारक ो क

लाभाोश

(iii) लोदन म भारत कायािलय चलान क वलए िन

(iv) भारत म वनयकत वबरविश कवमिय ो क ितन

और पशन का भगतान करन क वलए िन

(v) रलि पर बयाज

(vi) नागररक और सनय शलक

(vii) इोगलड म सट र (सामगरी) की खरीद

Q46) उततर (b)

सपषटीकरण

bull भारतीय राषटर ीय काोगरस का लाहौर सतर 1929 म

जिाहरलाल नहर की अधयकषता म आय वजत

वकया गया था

bull इस सतर म भारतीय राषटर ीय आोद लन स समबसित

कई महतवपणि पररणाम सामन आय थ

(i) सििपरथम इस सतर म काोगरस क अधयकष पद

पर जिाहरलाल नहर क चना गया था ज

काोगरस म िामपोवथय ो की बिती हई ताकत

का सपषट सोकत था

(ii) दसरा इस सतर म पहली बार काोगरस न पणि

सवतोतरता की माोग क उठाया था

इस परकार की माोग काोगरस मोच स पहल कभी भी

नही ो उठाई गई थी

Q47) उततर (b)

सपषटीकरण

FC19H1003 36

bull इस ररप िि न वकसी भी समदाय क वलए पथक

वनिािचक मोडल अथिा अलपसोखयक ो क वलए

भाराोश की वसफाररश नही ो की थी

bull तथावप इस ररप िि न उन पराोत ो म अलपसोखयक

सीि ो क आरकषण की अनमवत दी थी जहाा पर कम

स कम दस परवतशत अलपसोखयक ह

bull लवकन यह समदाय क आकार क अनपात म ह ना

चावहए था

bull इस ररप िि म भारत क वलए पणि सवतोतरता क

वलए क ई पराििान नही ो था

Q48) उततर (c)

सपषटीकरण

bull आरो वभक िवदक आयो का िमि मखय रप स

परकवत की पजा और यजञ था

bull परारो वभक आयि िमि परकवत की पजा क समान था

bull िासति म उनक चार ो ओर की शसकतयाा वजनह न

त ि वनयोवतरत कर सकत थ और न ही समझ पाए

थ उनह वदवयता क साथ वनिवशत वकया गया तथा

उनह मादा या नर दिीदिताओो क रप म

परतीकतव वकया गया था

bull उनह ोन कछ यजञ ो का भी वनषपादन वकया था

Q49) उततर (b)

सपषटीकरण

bull सडक और नदी-मागि (जल-मागि) डकती स

सरवकषत नही ो थ

bull उललखनीय ह वक हिििििन क शासनकाल क

दौरान यआन चिाोग (हयएन साोग) का सारा

सामान लि वलया गया था

Q50) उततर (c)

सपषटीकरण

परशन म वदए गए द न ो कथन सही ह

Q51) उततर (b)

सपषटीकरण

bull परोदर दास एक सोत और भगिान कषण क एक

महान भकत थ

bull परोदर दास क कनाििक सोगीत क वपतामह क

रप म जाना जाता ह

bull यदयवप उनक जनम-थथान क बार म काफी

अिकल लगाई जाती रही ह

bull तथावप अब कननड विशवविदयालय हमपी क दवारा

गवठत एक विशिजञ सवमवत इस वनषकिि पर पहोची

ह वक उनका जनम थथान सोभितया कनाििक का

एक छ िा-सा गााि कषमपरा (वशिम गगा वजला)

था

Q52) उततर (c)

सपषटीकरण

bull शरी तयागराज शरी शयाम शासतरी और शरी मथसवामी

दीवकषतर क कनाििक सोगीत की वतरमवति माना

जाता ह

bull उनक कारण ही 18िी ो-19िी ो शताबदी म कनाििक

सोगीत का सववणिम यग आया था

Q53) उततर (d)

सपषटीकरण

bull अभी हाल ही म लौह यगीन-महापािावणक काल

का 2000 ििि पराना एक दलिभ सारक फगस

(Sarcophagus) (पतथर का ताबत) क ललम क

वियर गाोि (क वयलडी क पास वजला क वझक ड

करल राजय) की एक रॉक-कि गफा स ख जा गया

bull यह ताबत वजसम हविय ो क िकड थ खदाई क

दौरान वमला

bull अभी तक इस परकार की दलिभ ख ज करल क

मातर द ही थथान ो स हई ह

bull य द न ो सारक फगी (Sarcophagi) (पतथर क

ताबत) चियर और अथ ली (वजला क वझक ड) क

महापािाण थथल ो स वमल ह

Q54) उततर (a)

सपषटीकरण

FC19H1003 37

दवकषण भारत म महापािाण सोसकवत एक पणि

विकवसत लौह यगीन सोसकवत थी

Q55) उततर (d)

सपषटीकरण

bull च ल पाणडय और करलपतर (चर) इन तीन ो का

उललख अश क क अवभलख ो म वकया गया ह

bull सोभितः य भौवतक सोसकवत क उततर

महापािावणक चरण म थ

Q56) उततर (d)

सपषटीकरण

bull भीमा-क रगाोि की लडाई ततीय आोगल-मराठा

यद का वहससा थी

Q57) उततर (b)

सपषटीकरण

bull राजकमार शकल न गाोिीजी क चोपारण आन तथा

वतनकवथया परणाली स जडी समसया की जाोच क

वलए रारी करन क वलए दश भर म उनका

अनसरण वकया था

bull बज वकश र राजदर परसाद महादि दसाई और

नरहरी पाररख चोपारण सतयागरह क दौरान गाोिी

जी क सहय गी थ

Q58) उततर (b)

सपषटीकरण

bull बराहमण ो और बौद मठिाररय ो क कर-मकत गााि

अनदान म दन की परथा सतिाहन ो न आरमभ की

थी

Q59) उततर (c)

सपषटीकरण

इस कायिकरम क उददशय वनमनानसार ह

(i) बवनयादी पयििन आिाररक सोरचना का विकास

करना

(ii) चयवनत (पहचान वकय गए) कषतर ो म आजीविका क

सजन क वलए दश क साोसकवतक और विरासत

मलय ो क बिािा दना

(iii) विरासत समारक थथल ो पर विशव सतरीय आिाररक

सोरचना विकवसत करक एक सतत तरीक स

पयििक आकििण म िसद करना

(iv) थथानीय समदाय ो की सवकरय भागीदारी क माधयम

स र रगार ो का सजन करना

(v) र रगार उतपादन और आवथिक विकास क वलए

पयििन कषमता का उन पर परभाि का उपय ग

करना तथा

(vi) िारणीय पयििन आिाररक सोरचना का विकास

करना और उसका उवचत सोचालन तथा

रखरखाि सवनवशचत करना

Q60) उततर (b)

सपषटीकरण

bull यह वनकाय ििि 1987 म अससततव म आया था

bull यह एक राषटर ीय सतर का शीिि सोगठन ह ज भारत

सरकार क जनजातीय मामल ो क मोतरालय क

परशासवनक वनयोतरण क अिीन काम कर रहा ह

bull इसका पोजीकत और परिान कायािलय नई वदलली

म सथथत ह

Q61) उततर (c)

सपषटीकरण

bull परमचोद क उपनयास ो म परमाशरम रोगभवम गबन

कमिभवम और ग दान शावमल ह

bull ग रा रिी ोदरनाथ िग र क दवारा रवचत उपनयास ह

bull अभी हाल ही म मोशी परमचोद की 138िी ो जयोती दश

भर म मनाई गई थी

Q62) उततर (b)

सपषटीकरण

bull ldquoवगदाrdquo पोजाब (भारत) एिो पावकसतान की

मवहलाओो क दवारा तयौहार क समय और फसल

की बिाई तथा किाई क अिसर पर वकया जान

िाला एक पारोपररक दहाती नतय ह

FC19H1003 38

bull इस नतय क माधयम स पोजाबी मवहलाऐो अपनी

परसननता परकि करती ह तथा वगदा क परदशिन क

माधयम स परि िचिसव िाल समाज म मवहलाओो

की दबी हई भािनाओो क परकि करती ह

bull चोवक इस नतय का परि ो क साथ क ई सोबोि नही ो

ह अतः किल मवहलाऐो ही इसम भाग ल सकती

bull हर साल तीज समार ह क दौरान पोजाब म वगदा

नतय वकया जाता ह

तीज भारत क कछ भाग ो म मवहलाओो क दवारा

मनाया जान िाल कई तयौहार ो क वलए एक

वयापक नाम ह

Q63) उततर (a)

सपषटीकरण

- मजम-उल-बहरीन या द समदर ो का सोगम

नामक उललखनीय रचना दारा वशक ह क दवारा

वलखी थी

- भारत क उपराषटर पवत शरी एम िकया नायड न कहा

ह वक राजकमार दारा वशक ह की रचनाएा शाोवत

और सदभाि क बिािा दन क वलए एक तारा सर त

क रप म सामन आ सकती ो ह

- उपराषटर पवत गत ििो क भला वदए गए राजकमार

दारा वशक ह क परदवशित परचवलत करन हत

आय वजत एक परदशिनी का दौरा करन क बाद एक

सभा क सोब वित कर रह थ

- इस परदशिनी का आय जन फर क इस गौवियर

(Francois Gautier) क दवारा lsquoइोवदरा गाोिी नशनल

सिर फॉर द आििसrsquo (The Indira Gandhi

National Centre for the Arts) नई वदलली म

वकया गया था

Q64) उततर (c)

सपषटीकरण

- ग मतशवर परवतमा जन भगिान बाहबली क

समवपित ह

- यह एक एक-चटटानी पतथर की मवति ह

- राषटर पवत राम नाथ क विोद न शरिणबलग ला

(कनाििक) म आय वजत वकय जान िाल भवय

अवभिक समार ह महामसतकावभिक का

उदघािन वकया था

- यह समार ह 12 ििो म एक बार ह ता ह

Q65) उततर (c)

सपषटीकरण

bull पराची घािी पराची नदी क चार ो ओर फली हई थी

bull पराची घािी िीर-िीर विलपत ह गई थी

bull पराची नदी भिनशवर स वनकलती ह

bull यह महानदी की एक सहायक नदी ह और यह

परी खदाि किक तथा जगतवसोहपर वजल ो स

ह कर बहती ह

bull इस नदी क पर कषतर क पराची घािी कहा जाता ह

bull यह नदी बोगाल की खाडी म वगरती ह

परातासतवक साकषय स पता चलता ह वक पराची घािी

सभयता हडपपा और म हनज दाड द न ो की

पिििती ह

Q66) उततर (d)

सपषटीकरण

य समारक छतरपर वजल (मधय परदश) म विोधयाचल

पिित शरोखला म सथथत ह

Q67) उततर (a)

सपषटीकरण

bull थॉिस ऑन पावकसतान नामक पसतक डॉ बी

आर अमबडकर न वलखी थी

bull डॉ बी आर अमबडकर की जयोती क अिसर पर

भारत क राषटर पवत न भारत की इस महान हसती

क शरदाोजवल अवपित की थी

bull डॉ बी आर अमबडकर न 1924 म वडपरथड

कलावसर इोसटीटयि (दवलत िगि सोथथान -

बवहषकत वहतकाररणी सभा) और 1927 म समाज

समता सोघ की थथापना की थी

bull अमबडकर का धयान वशकषा कषतर की ओर भी था

bull उनह ोन वशकषा क वनमन िगो म फलान क वलए

पीपलस एजकशन स साइिी (The Peoples

Education Society) क नाम स महाविदयालय ो क

नििकि और छातरािास ो की थथापना की थी

FC19H1003 39

Q68) उततर (b)

सपषटीकरण

bull महरगि भारतीय उपमहादवीप म एक परवसद

निपािाण बसती ह ज बलवचसतान पराोत

पावकसतान म सथथत ह

bull दचपलली (आोधर परदश) क पास नागलर नदी क

पिी ति ो पर चना पतथर क बलॉक क विशाल

विसतार म एक पिि-ऐवतहावसक रॉक आिि थथल की

ख ज की गई ह

bull इसन 1500-2000 ईसा पिि क दौरान गोिर (आोधर

परदश) म विकवसत निपािाण सभयता पर परकाश

डाला ह

Q69) उततर (c)

सपषटीकरण

bull 12िी ो सदी और 13िी ो सदी म काकाविय िोश का

उदय हआ था

bull ि पहल कलयाण क पवशचमी चालकय ो क सामोत थ

bull परारोभ म उनह ोन िारोगल (तलोगाना) क पास एक

छ ि स कषतर पर शासन वकया था

bull उनह ोन ldquoनायक वयिथथाrdquo की शरआत की थी

वजस बाद म विजयनगर क राय शासक ो न

अपनाया और विकवसत वकया था

Q70) उततर (a)

सपषटीकरण

bull गाोिीजी क अनशन स वमल मावलक ो पर दबाि

पडा था ज अोततः शरवमक ो क ितन म 35 परवतशत

की िसद करन क वलए सहमत हए थ

bull गगल (Google) न अनसया साराभाई वजनह ोन

भारत क शरवमक आोद लन म एक अगरणी भवमका

वनभाई थी की 132िी ो जयोती डडल (Doodle) का

वनमािण करक मनाई

Q71) उततर (d)

सपषटीकरण

भारत स यनसक की मानिता की अमति साोसकवतक

विरासत की परवतवनवि सची म वनमनवलसखत शावमल ह

bull कवडयटटम करल का सोसकत रोगमोच

bull मवडयिि करल का अनषठान रोगमोच और नतय

नाविका

bull िवदक मि जाप की परोपरा

bull राजथथान क कालबवलया ल क गीत और नतय

bull रामलीला रामायण का पारोपररक परदशिन

bull सोकीतिन मवणपर का अनषठान गायन ढ ल िादन

और नतय

bull रममन भारत क गििाल वहमालय का िावमिक

तयौहार और अनषठान रोगमोच

bull जाोदीयाला गर पोजाब क ठठर ो की पीतल और

ताोब क वशलप स वनवमित बतिन ो की पारोपररक कला

bull छाऊ नतय पिी भारतीय राजय ो म जनमी शासतरीय

भारतीय नतय कला

bull लददाख का बौद मि जाप िर ाोस-वहमालयी लददाख

कषतर तथा जमम-कशमीर म पवितर बौद गरोथ ो का पाठ

bull य ग

bull नौर र

bull को भ मला

Q72) उततर (b)

सपषटीकरण

bull भारत क राषटर पवत शरी राम नाथ क विोद न

वकसामा नागालड म हॉनिवबल मह रति और

राजय गठन वदिस समार ह का उदघािन वकया

था

bull हॉनिवबल मह रति का नाम भारतीय हॉनिवबल क

नाम पर पडा ह ज एक विशाल और रोगीन जोगली

पकषी ह

bull यह पकषी नागालड राजय की अविकतर जनजावतय ो

की ल ककथाओो म उसललसखत ह

bull नागालड की परमख मानयता परापत जनजावतयाा ह

अोगामी आओ चखसोग चाोग ककी रगमा और

रवलोग आवद

bull ओोग जारिा और ससिनलीस अोडमान-वनक बार

दवीप समह की जनजावतयाा ह

FC19H1003 40

Q73) उततर (c)

सपषटीकरण

bull दकन म राषटर कि शासन दसिी ो सदी क अोत तक

लगभग 200 ििो तक रहा था

bull राषटर कि शासक अपन िावमिक विचार ो म सवहषण

bull उनह ोन न किल शि िमि और िषणि िमि बसलक

जन िमि क भी सोरकषण वदया था

bull एल रा म वशि क परवसद रॉक कि मोवदर का

वनमािण नौिी ो सदी म राषटर कि राजा कषण परथम न

करिाया था

bull उसका उततराविकारी अम घििि जन था लवकन

उसन अनय िमो क भी सोरकषण परदान वकया था

bull राषटर कि ो न मसलमान वयापाररय ो क बसन की

अनमवत दी थी

bull उनह न अपन अविराजय ो म इसलाम क उपदश दन

की भी अनमवत दी थी

bull अभी हाल ही म पाोडिलागटटा (तलोगाना) क

परागवतहावसक चटटान वचतर ो क कषरण की बिती हई

घिनाएा एक गोभीर वचोता का वििय ह

bull यह परागवतहावसक चटटान क नकसान पहाचा

सकता ह

bull पाोडिलागटटा वनमनवलसखत क वलए जाना जाता ह

- 10000 ईसा पिि स 8000 ईसा पिि क वचवतरत

चटटानी आशरय ो क वलए

- राषटर कि काल क एक 8 िी ो सदी क

वशलालख क वलए और

- 12िी ो सदी क काकविय सामराजय क वभवतत

वचतर ो क वलए

Q74) उततर (b)

सपषटीकरण

bull 1828 म राजा राम म हन रॉय न एक नय िावमिक

समाज बरहम सभा की थथापना की थी वजस बाद

म बरहम समाज क नाम स जाना गया था

bull दिदरनाथ िग र न ततवब विनी सभा की अधयकषता

की थी ज आधयासिक सतय की ख ज म सोलि

थी

bull इसका उददशय वहोद िमि क शद करन का और

एकशवरिाद (एक ईशवर म आथथा) का परचार करना

था

bull नय समाज की थथापना क आिार थ कारण

(तकि ) क द सतमभ तथा िद और उपवनिद

bull अभी हाल ही म सािारण बरहम समाज का कछ

काननी मदद ो क लकर पवशचम बोगाल सरकार क

साथ काननी वििाद चल रहा ह

Q75) उततर (c)

सपषटीकरण

bull भारत म वचशती वसलवसल की थथापना खवाजा

म इनददीन वचशती क दवारा की गयी थी

bull ि 1192 ईसवी क आसपास भारत आय थ

bull वचशतीय ो क बारहिी ो शताबदी क उततरािि म भारत

म आन िाल सफीय ो क समह ो म सबस

परभािशाली माना जाता ह

bull उनह ोन थथानीय िातािरण क साथ सफलतापििक

अनकलन वकया और उनह ोन भारतीय भसकत

परोपराओो क कई पहलओो क अपनाया

bull अजमर म सफी अपरकि खवाजा म इनददीन वचशती

की ऐवतहावसक दरगाह क एक नया रप दन की

तयारी की जा रही ह

bull इस 13िी ो शताबदी की दरगाह क ldquoसवचछ

आइकॉवनक थथल ोrdquo (Swacch Iconic Places) म

शावमल वकया गया ह ज परवतवषठत विरासत

आधयासिक और साोसकवतक थथान ो पर क वदरत

य जना ह

FC19H1003 41

ANSWERS amp EXPLANATION OF

NCERT History Class VI-X + Current Affairs

(FC19E1003)

Q1) Answer c

Explanation

Rigveda consists of more than a

thousand hymns dedicated to gods and

goddesses These hymns were

composed by sages and learnt by men

however a few were composed by

women like Apala Ghosa Lopamudra

Maitreyi and Gargi

Rigveda consists of many hymns in the

form of dialogues We get an example of

a dialogue between a sage named

Vishwamitra and two rivers (Beas and

Sutlej) that were worshipped as

goddesses This suggests that he

belonged to the Vedic period

Q2) Answer b

Explanation

Traces of ash have been found from

Kurnool Caves suggesting that people

were familiar with the use of fire

It is situated in Andhra Pradesh

Q3) Answer c

Explanation

Burzahom is a prehistoric site in

present day Kashmir where people built

pit houses which were dug into the

ground with steps leading into them

These may have provided shelter in cold

weather

Q4) Answer c

Explanation

Epigraphy is defined as the study of

inscriptions

Manuscriptology is the study of history

and literature through the use of hand

written documents

Palaeography refers to the study of

ancient writing systems and the

deciphering and dating of historical

manuscripts

Numismatics refers to the study of

coins

Q5) Answer a

Explanation

Charaka Samhita was written by

Charaka and is an important book on

Ayurveda and medicine

He was a practitioner of the traditional

system of Indian medicine known as

Ayurveda

Charaka is thought to have flourished

sometime between the 2nd century BCE

and the 2nd century CE

Q6) Answer b

Explanation

Bhaga refers to the tax on crops which

was fixed at 16th of the production

Kammakaras is the term used for the

landless agricultural labour class

Ashvamedha also known as horse

sacrifice is a ritual where a horse is let

loose to wander freely and it was

guarded by the rajarsquos men

Q7) Answer (d)

Explanation

In the Rigvedic period horses were

yoked to chariots that were used in

battles fought to capture land cattle

etc This suggests that the use of horse

chariots began much before the period

of Mahajanapadas

The battles were fought in the Rigvedic

period for cattlersquos lands water an even

to capture people Most men took part

in these wars however there was no

regular army but there were assemblies

where people met and discussed

matters of war Regular armies became

a feature in the Mjahajanapada period

including vast armies of foot soldiers

chariots and elephants

RAUSIAS-FC19E1003 42

Q8) Answer (a)

Explanation

Buddha belonged to the Sakya clan and

passed away at Kusinara

Buddha taught in Prakrit which was the

common language of people

Q9) Answer c

Explanation

There were six schools of philosophy in

ancient India These are known as

Vaishesika Nyaya Samkhya Yoga

Purva Mimansa and Vedanata or Uttara

Mimansa They were founded by sages

Kanada Gautama Kapila Patanjali

Jamini and Vyasa respectively

Q10) Answer b

Explanation

The teachings of Mahavira were

compiled at Valabhi in 6th century AD

Q11) Answer (c)

Explanation

Chanakya is traditionally identified as

Kautilya or Vishnugupta who authored

the ancient Indian political treatise the

Arthashastra

Q12) Answer d

The national emblem of India is an

adaptation of the Lion Capital atop the

Ashoka Pillar of Sarnath Uttar Pradesh

and is combined with the National

Motto Satyameva Jayate

The Rampurva Bull gets the name from

the site of its discovery Rampurva in

Bihar

It is noted for its delicately sculpted

model demonstrating superior

representation of soft flesh sensitive

nostrils alert ears and strong legs It is

a mixture of Indian and Persian

elements

Sankissa is situated in Uttar Pradesh

India

Q13) Ans(a)

Kunwar Singh was a notable leader during the Revolt of 1857 He belonged

to a royal house of Jagdispur Bihar

Q14) Answer b

Explanation

The term Vellalar was used for large

landowners

Q15) Answer c

Explanation

Arikamedu was a coastal settlement

where ships unloaded goods from

distant lands Finds here include a

massive brick warehouse pottery

including amphorae and Arretine ware

Roman lamps glassware and gems have

also been found at the site

Q16) Answer a

Explanation

Muvendar is a Tamil word mentioned in

Sangam poems meaning three chiefs

used for the heads of three ruling

families the Cholas Cheras and

Pandyas

Q17) Ans (c)

Several tribal or kin-based assemblies

such as the Sabha Vidatha and gana

are mentioned in the Rig-veda The

Sabha and the samiti mattered a great

deal in early Vedic times so much so

that the chiefs or the kings showed an

eagerness to win their support

Q18) Ans (a)

Jainism recognised the existence of the

gods but placed them lower than the

jina and did not condemn the varna

system as Buddhism did

Q19) Answer (d)

Explanation

Cholas and Pandyas had developed

powerful coastal cities The most

important city of Cholas was Puhar or

Kaveripattinam and Madurai was the

capital of Pandyas

Q20) Answer b

Explanation

Buddhacharita is the biography of

Buddha and was written by

RAUSIAS-FC19E1003 43

Ashvaghosha

Q21) Answer (a)

Explanation

Tamil poet Appar was a Shiva devotee

So he was a Nayanar saint

Q22) Answer d

Explanation

Samudragupta was a prominent Gupta

ruler whose coins depict him playing a

veena indicating his love for music We

get important historic information from

his Allahabad Prashasti which was

composed by his court poet Harisena

Q23) Answer (b)

Explanation

Vikrama Samvat was founded by

Chandragupta II in the 58 BC as a

mark of victory over the Shakas and

assumed the title of Vikramaditya

Banabhatta wrote Harshavardhanarsquos

biography the Harshacharita in

Sanskrit

Q24) Answer c

Explanation

Sandhi-vigrahika was the minister of

war and peace

Sarthavaha was the leader of the

merchant caravans

Q25) Answer a

Explanation

Xuan Zang (Hsuan-tsang) was a

Chinese traveller who came during the

reign of Harshavardhana

In the decade that began in 630 AD

Xuan Zang came to India through

Kashmir after visiting Central Asia Iran

and Afghanistan

He travelled from north to east and lived

in Bihar for a couple of years

At Nalanda University Xuan Zang

interacted with students and scholars

mastered local languages and

discovered Buddhist stupas

Q26) Answer c

Explanation

Pradakshina patha is a circular path

laid around a stupa in Buddhist

architecture While the rest are a part of

temple architecture

Q27) Answer d

Explanation

All the above-mentioned temples have

an elaborate use of bricks (baked

bricks) along with stone

Q28) Ans (c)

Muhammad Quli Qutab was the Sultan

of Golconda He was a contemporary of

Akbar was very fond of literature and

architecture

The Sultan was a great poet and he

wrote in Dakhini Urdu Persian and

Telgu and has left an extensive diwan or

collection

Recently the Archaeological Survey of

India (ASI) will be using Ground

Penetrating Radar (GPR) to map the

contours of the area around the Bagh-e-

Naya Qila excavated garden inside the

Golconda Fort in Telangana

Q29) Answer a

Explanation

Silappadikaram is a famous Tamil epic

which was written by Ilango around

1800 years ago It is a story of a

merchant named Kovalan who fell in

love with a courtesan named Madhavi

Manimekalai tells the story of the

daughter of Kovalan and Madhavi

Q30) Answer (a)

Explanation

Charaka is the author of Charaka

Samhita which is an important work of

Ayurveda and medicines

Brahmaguptarsquos fame rests mostly on his

Brahma-sphuta-siddhanta which was

an astronomical work It was translated

into Arabic in Baghdad and had a major

impact on Islamic mathematics and

astronomy

Late in his life Brahmagupta wrote

Khandakhadyaka which was an

RAUSIAS-FC19E1003 44

astronomical handbook that employed

Aryabhatarsquos system of starting each day

at midnight

Q31) Answer (c)

Explanation

Amir Khusrau was a famous sufi

musician poet and scholar In 1318 he

noted that there was different language

in every region of this land (Hindustan)

Lahori Kashmiri Dvarsamudri (in

Southern Karnataka) Telangana (in

Andhra Pradesh) Gujari (in Gujarat)

Marsquobari (in Tamil Nadu) Awadhi (in

eastern Uttar Pradesh) and Hindawai (in

the area around in Delhi) etc He went

to explain that Sanskrit did not belong

to any region and that only brahmans

knew it

Q32) Answer c

Explanation

Hiranyagarbha refers to the golden

womb When this ritual was performed

with the help of Brahmanas it was

thought to lead to the rebirth of the

sacrificer as a Khastriya

Q33) Answer d

Explanation

Kadamai refers to a tax on land

revenue

Gwalior Prashasti describes the exploits

of Nagabhata who was a Pratihara king

Q34) Answer b

Explanation

Rajatarangini is a Sanskrit text written

by Kalhana in the 12th century

It was historical chronicle of early India

It is justifiably considered to be the best

and most authentic work of its kind

It covers the entire span of history in

the Kashmir region from the earliest

times to the date of its composition

Q35) Answer c

Explanation

ldquoUrrdquo was the general assembly of the

village ldquoUrrdquo consisted of all the

taxpaying residents of an ordinary

village

Q36) Answer (a)

Explanation

Tarikh was a form of history writing in

the Delhi Sultanate The authors of

tawarikhs were learned men which

included secretaries administrators etc

Q37 Answer (a)

Explanation

Alauddin chose to pay his soldiers salaries in cash rather than iqtas The soldiers would buy their supplies from merchants in Delhi and it was thus feared that merchants would raise their prices To stop this Alauddin controlled the prices of goods in Delhi Prices were carefully surveyed by officers and merchants who did not sell at the prescribed rates were punished

Q38) Answer (d)

Explanation

Delhi first became the capital of a

kingdom under the Tomara Rajputs

who were defeated in the middle of the

twelfth century by the Chauhans (also

referred to as Chahamanas) of Ajmer

It was under the Tomaras and

Chauhans that Delhi became an

important commercial centre Many rich

Jaina merchants lived in the city and

constructed several temples Coins

minted here called dehliwal had a wide

circulation

Q39) Answer (c)

Explanation

Moth ki Masjid was built in the reign of

Sikandar Lodi by his minister

Begumpuri mosque built in the reign of

Muhammad Tughluq was the main

mosque of Jahanpanah the ldquoSanctuary

of the Worldrdquo and his new capital in

Delhi

Quwwat al ndash Islam mosque was

enlarged by Iltutmish and Alauddin

Khalji The minar was built by three

Sultansndash Qutbuddin Aybak Iltutmish

and Firuz Shah Tughluq

RAUSIAS-FC19E1003 45

Q40) Answer (c)

Explanation

Under the Mughals mansabdar was

referred to an individual who held a

mansab ie rank and he received his

salary as revenue assignments called

jagirs

Q41) Ans (b)

The Quit India Movement was a

spontaneous revolt of people against

British rule

The All India Congress Committee met

at Bombay on 8 August 1942 It passed

the famous resolution Quit India and

proposed the starting of a non-violent

mass struggle under Gandhis

leadership to achieve this aim But on

the very next day Gandhi and other

eminent leaders of the Congress were

arrested The Congress was once again

declared illegal

Q42) Ans (c)

The Simon Commission refers to a

group of seven MPs from the United

Kingdom constituted to suggest

constitutional reforms for British India

The Commission consisted of only

British members headed by one of the

senior British politicians Sir John

Simon

So the people of India agitated against

the arrival of Simon Commission

Q43) Ans (a)

He was widely known for his

unfavourable opinion of the economic

consequences of the British rule in

India

In his many writings and speeches and

especially in Poverty and Un-British

Rule in India Naoroji argued that India

was too highly taxed and that its wealth

was being drained away to England

He did not interpret the ancient Indian

texts and restored the self-confidence of

Indians And also he did not stress the

need for eradication of all the social

evils before anything else

Q44) Ans (c)

In August 1932 Prime Minister

MacDonald announced his Communal

Award Great Britainrsquos unilateral

attempt to resolve the various conflicts

among Indiarsquos many communal

interests

The award which was later

incorporated into the act of 1935

expanded the separate-electorate

formula reserved for Muslims to other

minorities including Sikhs Indian

Christians Anglo-Indians Europeans

distinct regional groups Gandhi

undertook a ldquofast unto deathrdquo against

that offer which he viewed as a

nefarious British plot to divide the

Indian society

Q45) Ans (b)

In British India apart from existing

imports and exports there was also a

particular amount of money which

colonial India contributed towards

administration maintenance of the

army war expenses pensions to retired

officers and other expenses accrued by

Britain towards maintenance of her

colony These were known as Home

charges and were paid for almost

entirely by India

The Home charges was made of

following components-

- Interest payable on Indian debt

- Dividend to shareholders of East

India Company

- Funds used to support the India

Office in London

- Funds used to pay salaries and

pensions of British personnel

engaged in India

- Interest on the railways

- Civil and military charges

- Store purchases in England

Q46) Ans (b)

The Lahore session of the Indian

National Congress was held in 1929

under the Presidentship of Jawaharlal

Nehru

The Lahore session of the Indian

National Congress witnessed significant

RAUSIAS-FC19E1003 46

developments in the Indian national

movement

- First the election of Jawaharlal

Nehru to the post of Presidentship of

the Congress was a clear indication

of the growing strength of the

Leftists in the Congress

- Secondly it was in this session that

the Congress for the first time raised

the demand for complete

independence Such demand was

not raised from the Congress

platform earlier

Q47) Ans (b)

It did not provide for separate

electorates for any community or

weightage for minorities However it did

allow for the reservation of minority

seats in provinces having minorities of

at least ten per cent but this was to be

in strict proportion to the size of the

community

There was no provision for complete

Independence for India

Q48) Ans (c)

The religion of early Vedic Aryans was

primarily of worship of nature and

Yajnas

The early Aryan religion was kind of

nature worship Actually the forces

around them which they could not

control or understand were invested

with divinity and were personified as

male or female gods And they

performed some Yajnas also

Q49) Ans (b)

The roads and river-routes were not

immune from robbery It is notable that

Yuan Chwang (Hiuen Tsang) was

robbed of his belongings during

Harshvardanarsquos period

Q50) Ans (c)

Q51) Ans (b)

Purandara Dasa was a saint and great

devotee of Lord Krishna

There is much speculation about where

Purandara Dasa regarded as the

Pitamaha of Carnatic music was born

Recently an expert committee

constituted by the Kannada University

Hampi has come to the conclusion that

Kshemapura Shivamogga district

Karnataka is the birth place of

Purandara Dasa

Q52) Ans (c)

Sri Tyagaraja Sri Shyama Shastry and Sri Muthuswami Dikshitar are considered the trinity of Carnatic music and with them came the golden age in Carnatic music in the 18th-19th

century

Q53) Ans d)

Recently a rare sarcophagus (stone

coffin) which is 2000 years old from the

Iron AgendashMegalithic era was discovered

from a rock-cut cave at Viyur village of

Kollam near Koyilandy in Kozhikode

district Kerala

The coffin containing bone fragments

was found during an excavation ldquoSo

far such a rare finding has been

discovered only from two sites

in Kerala Both these sarcophagi were

recovered from Megalithic sites at

Chevayur and Atholi also in Kozhikode

district

Q54) Ans a)

The megalithic culture in South India was a full-fledged Iron Age culture

Q55) Ans d)

The Cholas Pandyas and Keralaputras

(Cheras) mentioned in Ashokan

inscriptions were probably in the late

megalithic phase of material culture

Q56) Ans d)

Q57) Ans (b)

Raj Kumar Shukla followed Gandhiji all

over the country to persuade him to

come to Champaran to investigate the

problem associated with tinkathia

system

RAUSIAS-FC19E1003 47

Brij Kishore Rajendra Prasad Mahadev

Desai and Narhari Parikh accompanied

Gandhi ji during the Champaran

Satyagraha

Q58) Ans (b)

The Satvahanas started the practice of granting tax-free villages to brahmanas and Buddhist monks

Q59) Ans c)

The objectives of the Programme are

listed as under

- Developing basic tourism

infrastructure

- Promoting cultural and heritage

value of the country to generate

livelihoods in the identified regions

- Enhancing the tourist attractiveness

in a sustainable manner by

developing world-class

infrastructure at the heritage

monument sites

- Creating employment through active

involvement of local communities

- Harnessing tourism potential for its

effects on employment generation

and economic development

- Developing sustainable tourism

infrastructure and ensuring proper

Operations and maintenance

therein

Q60) Ans (b)

The Tribal Cooperative Marketing

Development Federation of India

(TRIFED) came into existence in 1987

It is a national-level apex organization

functioning under the administrative

control of Ministry of Tribal Affairs

Govt of India

TRIFED has its registered and Head

Office located in New Delhi

Q61) Ans (c)

Premchandrsquos novels include

Premashram Rangabhumi Ghaban

Karmabhumi and Godan

Gora is a novel written by Rabindranath

Tagore

138th birth anniversary of Munshi

Premchand was celebrated across the

country

Q62) Ans (b)

Giddha is a traditional pastoral dance

performed by the women of the Punjab

India and Pakistan at festival times

and at the sowing and reaping of the

harvest

By this dance the Punjabi women

reveal their joy expel their suppressed

feelings in a male dominated society

through the performance of Giddha

Since this dance has nothing to do with

men only women can participate in it

During the Teej celebrations Giddha

dance is celebrated in Punjab every

year Teej is a generic name for a

number of festivals that are celebrated

by women in some parts of India

Q63) Ans (a)

Dara Shukoh wrote the remarkable

work called ldquoMajma-ul-Bahrainrdquo or the

ldquoThe confluence of two seasrdquo

The Vice President of India Shri M

Venkaiah Naidu has said that Prince

Dara Shukohrsquos writings can come as a

refreshing source for infusing peace and

harmony He was addressing the

gathering after visiting the exhibition

that showcases the forgotten Prince of

yesteryears Dara Shukoh organized by

Mr Francois Gautier at Indira Gandhi

National Centre for the Arts in New

Delhi

Q64) Ans (c)

The statue Gommateshwara is

dedicated to the Jain God Bahubali

It is a monolithic statue

President Ram Nath Kovind

inaugurated the grand anointing

ceremony mdash Mahamastakabhisheka mdash

held once in 12 years at

Shravanabelagola (Karnataka)

Q65) Ans (c)

Prachi Valley had come up around the

Prachi river Prachi Valley gradually

disappeared

RAUSIAS-FC19E1003 48

The Prachi river originates from

Bhubaneswar

It is a tributary of the Mahanadi and

flows through the districts of Puri

Khurda Cuttack and Jagatsinghpur

and the entire region of the river is

termed as the Prachi Valley

It falls into the Bay of Bengal

Archaeological evidence shows that the

Prachi Valley Civilisation predates both

Harappa and Mohenjo-Daro

The Prachi river originates from

Bhubaneswar

Q66) Ans (d)

These monuments are located in

Chhatarpur district Madhya Pradesh

within Vindhya mountain range

Q67) Ans (a)

The book lsquoThoughts on Pakistanrsquo was

written by Dr BR Ambedkar

On the occasion of the birth anniversary

of Dr BR Ambedkar the president of

India pays homage to this icon of India

In 1924 he founded the Depressed

Classes Institute (Bahishkrit Hitkarini

Sabha) and in 1927 the Samaj Samata

Sangh

Another area of attention for Ambedkar

was education For its spread among

the low classes he set up a network of

colleges by the name of Peoples

Education Society and founded hostels

Q68) Ans(b)

Mehrgarh is a famous Neolithic

settlement in the Indian subcontinent

which is situated in Baluchistan

province Pakistan

A pre-historic rock art site is discovered

in the vast expanse of limestone blocks

on the eastern banks of Naguleru river

near Dachepalli (Andhra Pradesh) It

has thrown light on the Neolithic

civilisation that flourished in Guntur

(Andhra Pradesh) during 1500-2000

BC

Q69) Ans (c)

The 12th and the 13th centuries saw

the emergence of the Kakatiyas They

were at first the feudatories of the

Western Chalukyas of Kalyana Initially

they ruled over a small territory near

Warangal (Telangana)

They introduced Nayakships which was

later adopted and developed by the

Rayas of Vijayanagara

Q70) Ans (a)

The fast had effect of putting pressure

on mill owners who finally agreed to

give the workers a 35 per cent increase

in wages

Google celebrated with a doodle the

132nd birth anniversary of Anasuya

Sarabhai who played a pioneering role

in Indiarsquos labour movement

Q71) Ans (d)

The UNESCOrsquos list of the representative

list of the intangible cultural heritage of

humanity from India are

- Koodiyattam Sanskrit Theatre of

Kerala

- Mudiyettu ritual theatre and dance

drama of Kerala

- Tradition of Vedic Chanting

- Kalbelia folk songs and dances of

Rajasthan

- Ramlila Traditional Performance of

the Ramayana

- Sankirtana ritual singing

drumming and dancing of Manipur

- Ramman religious festival and

ritual theatre of the Garhwal

Himalayas India

- Traditional brass and copper craft of

utensil making among the Thatheras

of Jandiala Guru Punjab India

- Chhau dance classical Indian dance

originated in the eastern Indian

states

- Buddhist chanting of Ladakh

recitation of sacred Buddhist texts

in the trans-Himalayan Ladakh

region Jammu and Kashmir India

- Yoga

- Nouroz

- Kumbh Mela

RAUSIAS-FC19E1003 49

Q72) Ans(b)

The President of India Shri Ram Nath Kovind inaugurated the Hornbill Festival and State Formation Day celebrations of Nagaland in Kisama

The festival is named after the Indian hornbill the large and colourful forest bird which is displayed in the folklore of most of the states tribes

The major recognized tribes of Nagaland are Angami Ao Chakhesang Chang

Kuki Rengma and Zeling etc

Onge Jarawa and Sentinelese are the

tribes of Andman amp Nicobar Islands

Q73) Ans (c)

The Rashtrakutas rule in the Deccan lasted for almost two hundred years till the end of the tenth century The Rashtrakutas rulers were tolerant in their religious views and patronized not only Shaivism and Vaishnavism but

Jainism as well

The famous rock-cut temple of Shiva at Ellora was built by one of the Rashtrakutas kings Krishna I in the ninth century His successor Amoghavarsha was a Jain but he also

patronized other faiths

The Rashtrakutas allowed Muslims traders to settle and permitted Islam to

be preached in their dominions

Recently increasing defacement at the prehistoric rock paintings of Pandavulagutta Telangana has created a cause for grave concern It can spoil

the prehistoric rock

Pandavulagutta is home to

- Painted rock shelters dating to

10000 BC-8000 BC

- An 8th century inscription of the

Rashtrakuta period and

- Painted frescoes from the 12th century Kakatiya empire

Q74) Ans (b)

In 1828 Raja Ram Mohan Roy founded a new religious society the Brahma Sabha later known as the Brahmo

Samaj

Debendranath Tagore headed the Tattvabodhini Sabha which was

engaged in search of spiritual truth

Its purpose was to purify Hinduism and to preach monotheism or belief in one God

The new society was to be based on the twin pillars of reason and the Vedas and

Upanishads

Recently Sadharan Brahmo Samaj (SBS) has entered into a legal battle with the West Bengal government due

to some legal issue

Q75) Ans (c)

The Chishti order was established in India by Khwaja Moinuddin Chishti who came to India around 1192 The Chishtirsquos are considered to be the most influential of the groups of Sufis who migrated to India in the late twelfth century They adapted successfully to the local environment and adopted several features of Indian devotional

traditions

The historical dargah of Sufi mystic Khwaja Moinuddin Chishti in Ajmer is all set to get a facelift This 13 th century dargah has been included among the Swachh Iconic Places a clean-up initiative focused on iconic

heritage spiritual and cultural places

Page 26: GENERAL STUDIES (PAPER I) · Test is part of Rau’s IAS Test series for Preliminary Exam 2019 FOUNDATION + CURRENT AFFAIRS GENERAL STUDIES (PAPER –I) FOUNDATION TEST –III TOPIC:

RAUSIAS-FC19E1003 26

Q70) निमननिखित कथि ो म स कौि-सास सही हह

1 अहमदाबाद नमि हड़ताि क दौराि महातमा

गाोरी ि शरनमक ो क पकष क मजबत करि क

निए आमरर अिशि नकया था

2 अिशि स नमि मानिक ो पर दबाि पड़ा था ज

अोततः शरनमक ो क िति म 15 परनतशत की िखदध

करि क निए सहमत हए थ

िीच नदए गए कट का परय ग कर सही उततर चनिए

(a) किि 1

(b) किि 2

(c) 1 और 2 द ि ो

(d) ि त 1 ि ही 2

Q71) निमननिखित म स नकसक नकिक भारत स यिसक

की माििता की अमतण साोसकनतक निरासत की

परनतनिनर सची (The UNESCOrsquos List of the

Representative List of the Intangible

Cultural Heritage of Humanity) म शानमि

नकया गया ह

1 मनडयटट

2 सोकीतणि

3 को भ मिा

िीच नदए गए कट का परय ग कर सही उततर चनिए

(a) किि 1 और 2

(b) किि 2 और 3

(c) किि 3

(d) 1 2 और 3

Q72) निमननिखित जिजानतय ो म स कौि-सीसी ो

जिजानतजिजानतया िागािड स सोबोनरत हह

1 अोगामी

2 ककी

3 जारिा

िीच नदए गए कट का परय ग कर सही उततर चनिए

(a) किि 1

(b) किि 1 औऔ 2

(c) किि 2

(d) 1 2 और 3

Q73) निमननिखित कथि ो म स कौि-सास सही हह

1 राषटर कट सामराजय की सथापिा दोनतदगण ि की थी

नजसि मानयाित म अपिी राजरािी की

सथापिा की थी

2 राषटर कट समराट अम घििण एक ििक था और

उस कनिताओो पर पहिी कननड़ पसतक नििि

का शरय नदया जाता ह

िीच नदए गए कट का परय ग कर सही उततर चनिए

(a) किि 1

(b) किि 2

(c) 1 और 2 द ि ो

(d) ि त 1 ि ही 2

Q74) निमननिखित कथि ो म स कौि-सास सही हह

1 कशब चोदर सि ि ततवब नरिी सभा की

अधयकषता की थी ज आधयाखतमक सतय की

ि ज म सोिि थी

2 बरहम समाज ि मािि गररमा पर बि नदया

मनतणपजा का निर र नकया और सती परथा जसी

सामानजक बराइय ो की आि चिा की

िीच नदए गए कट का परय ग कर सही उततर चनिए

(a) किि 1

(b) किि 2

(c) 1 और 2 द ि ो

(d) ि त 1 ि ही 2

Q75) निमननिखित कथि ो म स कौि-सास सही हह

1 भारत म नचशती नसिनसिा खवाजा म इिददीि

नचशती क दवारा सथानपत नकया गया था

2 नचशती परोपरा की एक परमि निशिता

आतमसोयम थी नजसम साोसाररक म ह स दरी

बिाए रििा शानमि था

िीच नदए गए कट का परय ग कर सही उततर चनिए

(a) किि 1

(b) किि 2

(c) 1 और 2 द ि ो

(d) ि त 1 ि ही 2

RAUSIAS-FC19E1003 27

Q70) Which of the following statements

isare correct

1 During the Ahmedabad Mill Strike

Mahatma Gandhi undertook a fast

unto death to strengthen the

workersrsquo resolve

2 The fast had effect of putting

pressure on mill owners who

finally agreed to give the workers a

15 per cent increase in wages

Select the correct answer using the code

given below

(a) 1 only

(b) 2 only

(c) Both 1 and 2

(d) Neither 1 nor 2

Q71) Which of the following are included in

the UNESCOrsquos list of the representative

list of the intangible cultural heritage of

humanity from India

1 Mudiyettu

2 Sankirtana

3 Kumbh Mela

Select the correct answer using the code

given below

(a) 1 and 2 only

(b) 2 and 3 only

(c) 3 only

(d) 1 2 and 3

Q72) Which of the following tribes isare

related to Nagaland

1 Angami

2 Kuki

3 Jarawa

Select the correct answer using the code

given below

(a) 1 only

(b) 1 and 2 only

(c) 2 only

(d) 1 2 and 3

Q73) Which of the following statements

isare correct

1 Rashtrakuta kingdom was founded by Dantidurga who established his capital at Manyakhet

2 Amoghavarsha a Rashtrakuta king was an author and is credited with writing the first

Kannada book on poetics

Select the correct answer using the code given below

(a) 1 only

(b) 2 only

(c) Both 1 and 2

(d) Neither 1 nor 2

Q74) Which of the following statements isare correct

1 Keshab Chandra Sen headed the Tattvabodhini Sabha which was engaged in search of spiritual truth

2 The Brahmo Samaj laid emphasis on human dignity opposed idolatry and criticized such social

evils as the practice of Sati

Select the correct answer using the code given below

(a) 1 only

(b) 2 only

(c) Both 1 and 2

(d) Neither 1 nor 2

Q75) Which of the following statements isare correct

1 The Chishti order was established in India by Khwaja Moinuddin

Chishti

2 A major feature of the Chishti tradition was austerity including maintaining a distance from the

worldly power

Select the correct answer using the code

given below

(a) 1 only

(b) 2 only

(c) Both 1 and 2

(d) Neither 1 nor 2

T e s t i s p a r t o f R a u rsquo s I A S T e s t s e r i e s f o r P r e l i m i n a r y E x a m 2 0 1 9

FOUNDATION + CURRENT AFFAIRS

GENERAL STUDIES (PAPER ndashI)

FOUNDATION TEST ndashIII

SUBJECT NCERT History Class VI-X + Current Affairs

Time Allowed 1frac12 Hours Maximum Marks 150

I NSTRUCT IONS

1 IMMEDIATELY AFTER THE COMMENCEMENT OF THE EXAMINATION YOU SHOULD CHECK

THAT THIS TEST BOOKLET DOES NOT HAVE ANY UNPRINTED OR TORN or MISSING PAGES OR

ITEMS ETC IF SO GET IT REPLACED BY A COMPLETE TEST BOOKLET

2 This Test Booklet contains 75 items (questions) Each item is printed both in Hindi and English

Each item comprises four responses (answers) You will select the response which you want to mark

on the Answer Sheet In case you feel that there is more than one correct response mark the

response which you consider the best In any case choose ONLY ONE response for each item

3 You have to mark all your responses ONLY on the separate Answer Sheet (OMR sheet) provided

Read the directions in the Answer Sheet

4 All items carry equal marks

5 Before you proceed to mark in the Answer Sheet the response to various items in the Test booklet

you have to fill in some particulars in the Answer Sheet as per instructions contained therein

6 After you have completed filling in all your responses on the Answer Sheet and the examination has

concluded you should hand over to the Invigilator only the Answer Sheet You are permitted to

take away with you the Test Booklet

7 Penalty for wrong answers

THERE WILL BE PENALTY FOR WRONG ANSWERS MARKED BY A CANDIDATE IN THE

OBJECTIVE TYPE QUESTION PAPERS

(i) There are four alternatives for the answer to every question For each question for which a

wrong answer has been given by the candidate one-third of the marks assigned to that

question will be deducted as penalty

(ii) If a candidate gives more than one answer it will be treated as a wrong answer even if one of

the given answers happens to be correct and there will be same penalty as above to that

question

(iii) If a question is left blank ie no answer is given by the candidate there will be no penalty for

that question

T h i s t e s t i s p a r t o f R a u rsquo s I A S T e s t s e r i e s f o r P r e l i m i n a r y E x a m 2 0 1 9

Test Code

FC19E1003

FC19H1003 29

Answers and Explanations of

NCERT History Class VI-X + Current Affairs (FC19E1003)

Q1) उततर (c)

सपषटीकरण

- ऋगवद म दविय ो और दिताओो क समवपित एक

हजार स अविक सत तर (शल क) ह

- य शल क ऋविय ो क दवारा रच गए थ और परि ो

दवारा सीख जात थ

- हालाोवक कछ शल क मवहलाओो (जस वक अपाला

घ सा ल पामदरा मतरयी और गागी) क दवारा भी रच

गए थ

- ऋगवद म सोिाद क रप म कई शल क मौजद ह

- हम विशवावमतर नामक एक ऋवि और दविय ो क

रप म पजी जान िाली द नवदय ो (वयास और

सतलज) क बीच िाताि का उदाहरण वमलता ह

- इसस पता चलता ह वक विशवावमतर िवदक काल स

सोबोवित थ

Q2) उततर (b)

सपषटीकरण

- करनल गफाओो स राख क अिशि परापत हए ह

ज इस ओर सोकत करत ह वक ततकालीन ल ग

अवि क उपय ग स पररवचत थ

- य गफाएो आोधर परदश म सथथत ह

Q3) उततर (c)

सपषटीकरण

bull बरािह म ितिमान कशमीर म सथथत एक

परागवतहावसक थथल ह जहाो ल ग गडढ क घर ो का

वनमािण करत थ

bull य घर जमीन क ख द कर बनाए जात थ तथा नीच

जान क वलए सीवियाा ह ती थी

bull ऐसा अनमान लगाया जाता ह वक य घर ठो ड क

मौसम म आशरय परदान करत थ

Q4) उततर (c)

सपषटीकरण

bull परालख-विदया (Epigraphy) क वशलालख ो क

अधययन क रप म पररभावित वकया जाता ह

bull हसतवलसखत दसतािज ो क माधयम स इवतहास

और सावहतय क अधययन क पाोडवलवप विजञान

(Manuscriptology) कहत ह

bull पराचीन लखन परणावलय ो क अधययन और

ऐवतहावसक पाोडवलवपय ो क समझन तथा वतवथ

वनिािरण क पलीओगराफी (Palaeography) कहा

जाता ह

bull नयवमजमविकस (Numismatics) वसक ो क

अधययन क सोदवभित करता ह

Q5) उततर (a)

सपषटीकरण

- चरक सोवहता चरक क दवारा वलखी गई आयिद

और िदयक-शासर पर एक महतवपणि पसतक ह

- ि भारतीय िदयक-शासर की पारमपररक परणाली

वजस आयिद क नाम स जाना जाता ह क

अभयासकताि थ

- ऐसा माना जाता ह वक चरक का विकास दसरी

शताबदी (ईसा पिि) और दसरी शताबदी (ईसवी) क

मधय हआ था

Q6) उततर (b)

सपषटीकरण

- भाग फसल ो पर वलए जान िाल कर क सोदवभित

करता ह ज कल फसल उतपादन का 16 िाो भाग

था

- ldquoकममकारrdquo शबद भवमहीन कवि शरवमक िगि क

वलए परय ग वकया जाता था

- ldquoअशवमिrdquo (वजस घ ड क बवलदान क रप म भी

जाना जाता ह) एक अनषठान ह ता था वजसम एक

घ ड क सवतोतर रप स घमन क वलए छ ड वदया

FC19H1003 30

जाता ह और राजा क सवनक उसकी रखिाली

करत थ

Q7) उततर (d)

सपषटीकरण

- ऋगववदक काल म घ ड ो क रथ ो म ज ता जाता था

ज (रथ) भवम मिवशय ो आवद पर कबजा करन क

वलए लड गए यद ो म उपय ग वकए जात थ

- इसस यह पता चलता ह वक घ ड ो यकत रथ ो का

उपय ग महाजनपद काल स काफी पहल आरमभ

हआ था

- ऋगववदक काल म मिवशय ो भवम जल आवद पर

कबजा करन क वलए तथा ल ग ो क पकडन क

वलए यद वकय जात थ

- अविकाोश परि इन यद ो म भाग वलया करत थ

- हालाोवक उस समय क ई वनयवमत सना नही ो ह ती

थी लवकन उस काल म सभाऐो ह ती थी ो वजनम

ल ग यद क मामल ो पर चचाि करत थ

- वनयवमत सनाएा महाजनपद काल का िवशषटय थी

वजनम पदल सवनक ो की विशाल सनाएा रथ तथा

हाथी शावमल ह त थ

Q8) उततर (a)

सपषटीकरण

- बद शाकय कल स सोबोवित थ और कशीनारा म

उनका वनिन हआ था

- बद न अपनी वशकषाएा पराकत भािा म दी थी ो ज

आम ल ग ो की भािा थी

Q9) उततर (c)

सपषटीकरण

- पराचीन भारत म दशिनशासर की छह शाखाएा थी ो

िशविक नयाय समखया य ग पिि वममाोसा और

िदाोत या उततर वममाोसा

- इनकी थथापना करमश कनाद गौतम कवपल

पतोजवल जावमनी और वयास ऋविय ो न की थी

Q10) उततर (b)

सपषटीकरण

महािीर की वशकषाऐो छठी शताबदी म िललभी म

सोकवलत की गई थी ो

Q11) उततर (c)

सपषटीकरण

- पारमपररक रप स चाणकय क कौविलय अथिा

विषणगपत क नाम स जाना जाता ह

- उसन अथिशासतर ज एक पराचीन भारतीय

राजनवतक आलख ह वलखा था

Q12) उततर (d)

सपषटीकरण

- भारत का राषटर ीय वचनह सारनाथ (उततर परदश) क

अश क सतमभ क ऊपर (शीिि पर) वसोह कवपिल

का एक अनरपण ह

- इस राषटर ीय वसदाोत सतयमि जयत क साथ

सोय वजत वकया गया ह

- रामपिि बल का नाम रामपिि (वबहार) क नाम पर

पडा जहाा इसकी ख ज हई थी

- यह अपन नाजक नकाशी मॉडल क वलए परवसदद

ह वजसम क मल तवचा सोिदनशील नथन ो सतकि

कान और मरबत िााग ो क शरषठतर परवतरप क

परदवशित वकया गया ह

- यह भारतीय और फारसी ततव ो का एक ससममशरण

- सोवकससा उततर परदश म सथथत ह

Q13) उततर (a)

सपषटीकरण

का िर वसोह ज एक महान य दा थ वबहार स

सोबोवित थ

Q14) उततर (b)

सपषटीकरण

िललालर शबद बड भ-सवावमय ो क वलए परय ग

वकया जाता था

FC19H1003 31

Q15) उततर (c)

सपषटीकरण

- अररकमड एक तिीय बसती थी जहाो दर दश ो स

आन िाल जहाज ो का माल उतारा जाता था

- यहाो पर ईोि ो का एक विशाल ग दाम वमटटी क

बतिन (वजनम एमफ रा - द हरी मवठय ो का लोबा

घडा - शावमल ह) और एरिाइन (Arretine)

मदभाोड पाए गए थ

- इस थथान पर र मन दीपक काोच क बन पातर और

रतन भी पाए गए थ

Q16) उततर (a)

सपषटीकरण

- मिनदर सोगम कविताओो म उसललसखत एक

तवमल शबद ह वजसका अथि ह ldquoतीन परमखrdquo

- यह तीन सततारि पररिार ो क मसखयाओो क वलए

परय ग वकया जाता ह च ल चर और पाणडय

Q17) उततर (c)

सपषटीकरण

- ऋग िद म सभा विदाथा तथा गण जसी

जनजावतय ो पर अथिा किोब पर आिाररत

सभाओो का उललख ह

- आरसमभक िवदक काल म सभाओो और सवमवतय ो

का विशि महतव ह ता था

- यहाा तक की मसखया अथिा राजा भी उनका

समथिन परापत करन क वलए आतर रहत थ

Q18) उततर (a)

सपषटीकरण

- जन िमि न ईशवर क अससततव क मानयता त दी ह

वकनत उसन ईशवर क वजना क पद स नीच रखा

- जन िमि न बौद िमि की तरह िणि परणाली की

भरतिना नही ो की थी

Q19) उततर (d)

सपषटीकरण

- च ल ो और पाणडय ो न शसकतशाली तिीय शहर ो का

विकास वकया था

- च ल ो का सबस महतवपणि शहर पहार (या

कािरीपटटीनम) था |

- मदरई पाणडय ो की राजिानी थी

Q20) उततर (b)

सपषटीकरण

- ldquoबदचररतrdquo बद का जीिन-ितताोत ह

- इस अशवघ ि क दवारा वलखा गया था

Q21) उततर (a)

सपषटीकरणः

- तवमल कवि अपपर भगिान वशि क भकत थ

- इस परकार ि एक नयनार सोत थ

Q22) उततर (d)

सपषटीकरणः

- समदरगपत एक परवसद गपत शासक था

- उसन वसक ो पर िीणा बजात हए अपनी छवि

अोवकत करिाई थी

- यह सोगीत क परवत उसक परम क दशािता ह

- हम उसकी इलाहाबाद परशससत स महतवपणि

ऐवतहावसक जानकारी वमलती ह वजसकी रचना

उसक दरबार क कवि हररसन न की थी

Q23) उततर (b)

सपषटीकरणः

- विकरम सोित की शरआत ििि 58 ईसा पिि म

चनदरगपत वदवतीय न की थी

- यह शक ो पर उसकी जीत और उस विकरमावदतय

की पदिी वमलन क उपलकषय म आरमभ वकया गया

था

FC19H1003 32

- बानभटट न हिििििन का जीिन-ितताोत हििचररत

(ज सोसकत म थी) वलखी थी

Q24) उततर (c)

सपषटीकरणः

- सोवि-विगरावहका यद एिो शाोवत का मोतरी

- साथििाह वयापाररय ो क कावफल ो का नता

Q25) उततर (a)

सपषटीकरणः

- जआन झाोग (हसआन रताोग ndash Hsuang Tsang)

एक चीनी यातरी था ज हिििििन क शासनकाल म

भारत आया था

- ििि 630 ईसवी स ज दशक आरमभ हआ था उसम

जआन झाोग मधय एवशया ईरान और

अफग़ावनसतान की यातरा करन क पशचात कशमीर

क रासत स भारत आया था

- उसन उततर स पिि तक की यातरा की और िह

लगभग 2 ििि वबहार म रहा

- जआन झाोग न नालनदा विशवविदयालय म विदयावथिय ो

और विदवान ो क साथ पारसपररक विचार-विमशि

वकया थथानीय भािाओ ा म वनपणता परापत की तथा

बौद सतप ो की ख ज की

Q26) उततर (c)

सपषटीकरणः

- परदवकषणा पथ बौद िासतकला म सतप क चार ो

ओर बनाया जान िाला एक घमािदार पथ ह ता

- परशन म वदए गए बाकी क तीन ो ततव वहोद मसनदर ो की

िासतकला क भाग ह

Q27) उततर (d)

सपषटीकरणः

परशन म वदए गए सभी मोवदर ो म वयापक रप स

ईोि ो (पकी ईोि ो) का परय ग पतथर ो क साथ हआ

Q28) उततर (c)

सपषटीकरण

- महममद कली कतब शाह ग लकणडा का सलतान

था

- िह अकबर का समकालीन था

- सावहतय और िासतकला म उसकी अतयाविक

रवच थी

- िह एक महान कवि था

- िह दसखनी उदि फारसी और तलग म वलखता था

- उसन अपन पीछ एक विसतत वदिान (सोगरह)

छ डा ह

- अभी हाल ही म तलोगाना म ग लकणडा क वकल

क अनदर खदाई वकय गए बाग-ए-नाया वकला

बाग क चार ो ओर रप-रखा क मानवचतरण क

वलए भारतीय परातासतवक सिकषण (The

Archaeological Survey of India ndash ASI)

गराउणड पनीिर विोग रडार (Ground Penetrating

Radar) का परय ग करगा

Q29) उततर (a)

सपषटीकरणः

- वसलपपावदकारम एक तवमल महाकावय ह वजसकी

रचना इलाोग क दवारा लगभग 1800 ििि पिि की

गई थी

- यह क िलन नामक एक वयापारी की कहानी ह

ज माििी नामक एक गवणका (िशया) स परम

करन लगा था

- मवनमकलाई क िलन और माििी की पतरी की

कहानी ह

Q30) उततर (a)

सपषटीकरण

- चरक आयिद और वचवकरता की एक महतवपणि

रचना चरक सोवहता क लखक ह

- बरहमगपत क अपनी रचना बरहम-सफि-वसदानत

(ज एक खग लीय रचना ह) क कारण परवससद

वमली

FC19H1003 33

- बगदाद म इसका अनिाद अरबी भािा म वकया

गया था

- इसका इसलावमक गवणत और खग ल-विजञान पर

महतवपणि परभाि पडा था

- बाद म अपन जीिनकाल म बरहमगपत न

ldquoखोडखयाकrdquo वलखी ज एक खग लीय पससतका

(एक छ िी पसतक) थी

- इसम आयिभटट की अिि-रावतर क परतयक वदन की

शरआत परणाली का परय ग वकया गया था

Q31) उततर (c)

सपषटीकरण

- अमीर खसर एक परवसद सफी सोगीतकार कवि

और विदवान थ

- 1318 म उनह ोन पाया वक इस भवम (वहोदसतान) क

हर कषतर म अलग-अलग भािा थी लाहौरी

कशमीरी दवारसमदरी (दवकषणी कनाििक म)

तलोगाना (आोधर परदश म) गजरी (गजरात म)

माबारी (तवमलनाड म ) अििी (पिी उततर परदश

म) और वहोदिी (वदलली क आस-पास क कषतर म)

आवद

- उनह न यह बताया वक सोसकत वकसी भी कषतर स

सोबोवित नही ो थी और किल बराहमण ही इस भािा

का जञान रखत थ

Q32) उततर (c)

सपषटीकरण

- वहरणय-गभि सववणिम गभि क सोदवभित करता ह

- जब बराहमण ो की सहायता स यह अनषठान वकया

जाता था त यह माना जाता था वक बवल दन िाल

का कषवतरय क रप म पनजिनम ह गा

Q33) उततर (d)

सपषटीकरण

- कदमई भवम राजसव पर कर क सोदवभित करता

- गवावलयर परशससत म नागभि क दवारा वकय गए

श िण का िणिन वकया गया ह |

- नागभि एक परवतहार राजा था

Q34) उततर (b)

सपषटीकरण

- राजतरो वगनी 12िी ो शताबदी म कलहन क दवारा

रवचत एक सोसकत पसतक (िकसट) ह

- यह परारसमभक भारत की ऐवतहावसक इवतितत थी

- तकि सोगत रप स इस अपन परकार की सिोततम

और सिािविक विशवसनीय कवत माना जाता ह

- यह कशमीर कषतर क पराचीनतम समय स लकर

उसकी रचना की तारीख तक क समपणि इवतहास

का आचछादन करती ह

Q35) उततर (c)

सपषटीकरण

- गााि की आम सभा क ldquoउरrdquo कहा जाता था

- ldquoउरrdquo म गााि क सभी कर दन िाल वनिासी

शावमल ह त थ

Q36) उततर (a)

सपषटीकरण

- वदलली सलतनत म ldquoतारीखrdquo इवतहास लखन का

एक रप था

- ldquoतािरीखrdquo क लखक विदवान परि ह त थ वजनम

सवचि परशासक इतयावद शावमल थ

Q37) उततर (a)

सपषटीकरण

- अलाउददीन सखलजी अपन सवनक ो क ितन का

भगतान नकद म करता था न वक इकता क रप

- सवनक अपना सामान वदलली म वयापाररय ो स

खरीदत थ अतः इस बात का भय था वक वयापारी

कही ो िसतओो का मलय न बिा द

- इसकी र कथाम क वलए अलाउददीन सखलजी न

वदलली म कीमत ो क वनयसित वकया

FC19H1003 34

- अविकारीगण धयानपििक मलय ो का सिकषण करत

थ तथा ज वयापारी वनिािररत मलय पर माल नही ो

बचत थ उनक दसणडत वकया जाता था

Q38) उततर (d)

सपषटीकरण

- वदलली सििपरथम त मर राजपत ो क अिीन उनक

सामराजय की राजिानी बनी थी

- 12िी ो शताबदी क मधय म अजमर क चौहान ो

(वजनह चाहमान ो क नाम स भी जाना जाता ह) न

त मर राजपत ो क परावजत वकया था

- त मर ो और चौहान ो क अिीन वदलली एक

महतवपणि िावणसजयक क दर बन गया था

- कई जन वयापारी यहाा रहन लग थ और उनह ोन

कई मोवदर भी बनिाए

- यहाा पर मवदरत वसक वजनह ldquoदहलीिालrdquo क नाम

स जाना जाता था वयापक रप स परचलन म थ

Q39) उततर (c)

सपषटीकरण

- म ठ की मसिद का वनमािण वसको दर ल दी क

राजयकाल म उसक मिी क दवारा करिाया गया

था

- बगमपरी मसिद का वनमािण महममद तगलक क

शासनकाल म हआ था

- यह मसिद विशव का पणयथथान (The

Sanctuary of the World) और वदलली म महममद

तगलक की नई राजिानी जहाोपनाह की मखय

मसिद थी

- कववत- अल - इसलाम मसिद का विसतार

इलतसिश और अलाउददीन सखलजी न वकया था

- मीनार का वनमािण तीन सलतान ो कतबददीन ऐबक

इलतसिश और वफर ज शाह तगलक क दवारा

करिाया गया था

Q40) उततर (c)

सपषटीकरण

- मगल ो क अिीन मनसबदार शबद उस वयसकत क

वलए सोदवभित वकया जाता था वजसक पास मनसब

(अथाित पद) ह ता था

- उस अपना ितन राजसव कायो वजनह जागीर कहत

थ क रप म परापत ह ता था

Q41) उततर (b)

सपषटीकरण

- ldquoभारत छ ड आोद लनrdquo वबरविश शासन क

सखलाफ ल ग ो का एक सवाभाविक विदर ह था

- असखल भारतीय काोगरस सवमवत न 8 अगसत 1942

क बमबई म एक बठक का आय जन वकया था

- इस बठक म परवसद सोकलप ldquoभारत छ ड rdquo क

पाररत वकया गया और इस उददशय क परापत करन

क वलए गाोिी क नततव म एक अवहोसक जन सोघिि

आोद लन की शरआत का परसताि वदया गया

- लवकन अगल ही वदन गाोिी और काोगरस क अनय

परमख नताओो क वगरफतार कर वलया गया

- काोगरस क एक बार वफर अिि घ वित वकया गया

था

Q42) उततर (c)

सपषटीकरण

- साइमन कमीशन यनाइविड वको गडम क सात

साोसद ो का एक समह था

- इस वबरविश भारत क वलए सोििावनक सिार ो का

सझाि दन क वलए गवठत वकया गया था

- इस आय ग म िररषठ वबरविश राजनता सर जॉन

साइमन क नततव म किल वबरविश सदसय ही

शावमल थ

- इसवलए भारत क ल ग ो न साइमन कमीशन क

आगमन क विरद आोद लन वकया था

Q43) उततर (a)

सपषटीकरण

bull दादा भाई नौर जी भारत म वबरविश शासन क

आवथिक पररणाम ो क बार म अपनी विर िी

(परवतकल) राय क वलए जान जात थ

FC19H1003 35

bull अपन कई लख ो और भािण ो म विशि रप स

ldquoपाििी एो ड अन-वबरविश रल इन इसणडया

(Poverty and Un-British Rule in India) म

नौर जी न यह तकि वदया वक भारत पर अतयविक

कर लगाया गया था और इसकी सोपवतत इोगलड की

ओर परिावहत की जा रही थी

bull उनह ोन पराचीन भारतीय गरोथ ो की वयाखया करन

का और भारतीय ो क आिविशवास क बहाल

करन पर कायि नही ो वकया था

उनह ोन वकसी और बात स पहल सभी सामावजक

बराइय ो क उनमलन की आिशयकता पर भी बल

नही ो वदया था

Q44) उततर (c)

सपषटीकरण

bull अगसत 1932 म वबरविश परिानमोतरी मकड नालड न

अपन साोपरदावयक परसकार (The Communal

Award) की घ िणा की थी

bull यह भारत क कई साोपरदावयक वहत ो क बीच विवभनन

सोघिो क हल करन क वलए वबरिन का एकतरफा

परयास था

bull यह परसकार (Award) बाद म 1935 क

अविवनयम (The Act of 1935) म शावमल वकया

गया था

bull इस साोपरदावयक परसकार न मससलम ो क वलए

आरवकषत एक अलग वनिािचक मणडल फॉमिल का

विसतार अनय अलपसोखयक ो क वलए वकया था

वजसम वसख ो भारतीय ईसाइय ो आोगल-भारतीय

समदाय यर पीय समदाय तथा विवशषट कषतरीय

समह ो क शावमल वकया गया था

bull गाोिी न इस परसताि क भारतीय समाज क

विभावजत करन क वलए एक घवणत वबरविश

सावजश क रप म दखा और उसक सखलाफ

आमरण अनशन वकया

Q45) उततर (b)

सपषटीकरण

मौजदा आयात और वनयाित क अवतररक़त

औपवनिवशक भारत क वनमनवलसखत खचो क

वलए एक विशिवनवशचत िन रावश भी दनी पडती

थी

(i) परशासन क वयय

(ii) सना क रख-रखाि क वयय

(iii) यद क वयय

(iv) सिावनितत अविकाररय ो की पशन तथा

(v) वबरिन दवारा अपनी उपवनिश बसती

(कॉल नी) क रख-रखाि क वयय

इनह गह शलक (Home Charges) क रप म

जाना जाता था और लगभग परी तरह स भारत क

दवारा इनका भगतान वकया जाता था

bull गह शलक म वनमनवलसखत घिक शावमल थ

(i) भारतीय ऋण पर दय बयाज

(ii) ईसट इोवडया को पनी क शयरिारक ो क

लाभाोश

(iii) लोदन म भारत कायािलय चलान क वलए िन

(iv) भारत म वनयकत वबरविश कवमिय ो क ितन

और पशन का भगतान करन क वलए िन

(v) रलि पर बयाज

(vi) नागररक और सनय शलक

(vii) इोगलड म सट र (सामगरी) की खरीद

Q46) उततर (b)

सपषटीकरण

bull भारतीय राषटर ीय काोगरस का लाहौर सतर 1929 म

जिाहरलाल नहर की अधयकषता म आय वजत

वकया गया था

bull इस सतर म भारतीय राषटर ीय आोद लन स समबसित

कई महतवपणि पररणाम सामन आय थ

(i) सििपरथम इस सतर म काोगरस क अधयकष पद

पर जिाहरलाल नहर क चना गया था ज

काोगरस म िामपोवथय ो की बिती हई ताकत

का सपषट सोकत था

(ii) दसरा इस सतर म पहली बार काोगरस न पणि

सवतोतरता की माोग क उठाया था

इस परकार की माोग काोगरस मोच स पहल कभी भी

नही ो उठाई गई थी

Q47) उततर (b)

सपषटीकरण

FC19H1003 36

bull इस ररप िि न वकसी भी समदाय क वलए पथक

वनिािचक मोडल अथिा अलपसोखयक ो क वलए

भाराोश की वसफाररश नही ो की थी

bull तथावप इस ररप िि न उन पराोत ो म अलपसोखयक

सीि ो क आरकषण की अनमवत दी थी जहाा पर कम

स कम दस परवतशत अलपसोखयक ह

bull लवकन यह समदाय क आकार क अनपात म ह ना

चावहए था

bull इस ररप िि म भारत क वलए पणि सवतोतरता क

वलए क ई पराििान नही ो था

Q48) उततर (c)

सपषटीकरण

bull आरो वभक िवदक आयो का िमि मखय रप स

परकवत की पजा और यजञ था

bull परारो वभक आयि िमि परकवत की पजा क समान था

bull िासति म उनक चार ो ओर की शसकतयाा वजनह न

त ि वनयोवतरत कर सकत थ और न ही समझ पाए

थ उनह वदवयता क साथ वनिवशत वकया गया तथा

उनह मादा या नर दिीदिताओो क रप म

परतीकतव वकया गया था

bull उनह ोन कछ यजञ ो का भी वनषपादन वकया था

Q49) उततर (b)

सपषटीकरण

bull सडक और नदी-मागि (जल-मागि) डकती स

सरवकषत नही ो थ

bull उललखनीय ह वक हिििििन क शासनकाल क

दौरान यआन चिाोग (हयएन साोग) का सारा

सामान लि वलया गया था

Q50) उततर (c)

सपषटीकरण

परशन म वदए गए द न ो कथन सही ह

Q51) उततर (b)

सपषटीकरण

bull परोदर दास एक सोत और भगिान कषण क एक

महान भकत थ

bull परोदर दास क कनाििक सोगीत क वपतामह क

रप म जाना जाता ह

bull यदयवप उनक जनम-थथान क बार म काफी

अिकल लगाई जाती रही ह

bull तथावप अब कननड विशवविदयालय हमपी क दवारा

गवठत एक विशिजञ सवमवत इस वनषकिि पर पहोची

ह वक उनका जनम थथान सोभितया कनाििक का

एक छ िा-सा गााि कषमपरा (वशिम गगा वजला)

था

Q52) उततर (c)

सपषटीकरण

bull शरी तयागराज शरी शयाम शासतरी और शरी मथसवामी

दीवकषतर क कनाििक सोगीत की वतरमवति माना

जाता ह

bull उनक कारण ही 18िी ो-19िी ो शताबदी म कनाििक

सोगीत का सववणिम यग आया था

Q53) उततर (d)

सपषटीकरण

bull अभी हाल ही म लौह यगीन-महापािावणक काल

का 2000 ििि पराना एक दलिभ सारक फगस

(Sarcophagus) (पतथर का ताबत) क ललम क

वियर गाोि (क वयलडी क पास वजला क वझक ड

करल राजय) की एक रॉक-कि गफा स ख जा गया

bull यह ताबत वजसम हविय ो क िकड थ खदाई क

दौरान वमला

bull अभी तक इस परकार की दलिभ ख ज करल क

मातर द ही थथान ो स हई ह

bull य द न ो सारक फगी (Sarcophagi) (पतथर क

ताबत) चियर और अथ ली (वजला क वझक ड) क

महापािाण थथल ो स वमल ह

Q54) उततर (a)

सपषटीकरण

FC19H1003 37

दवकषण भारत म महापािाण सोसकवत एक पणि

विकवसत लौह यगीन सोसकवत थी

Q55) उततर (d)

सपषटीकरण

bull च ल पाणडय और करलपतर (चर) इन तीन ो का

उललख अश क क अवभलख ो म वकया गया ह

bull सोभितः य भौवतक सोसकवत क उततर

महापािावणक चरण म थ

Q56) उततर (d)

सपषटीकरण

bull भीमा-क रगाोि की लडाई ततीय आोगल-मराठा

यद का वहससा थी

Q57) उततर (b)

सपषटीकरण

bull राजकमार शकल न गाोिीजी क चोपारण आन तथा

वतनकवथया परणाली स जडी समसया की जाोच क

वलए रारी करन क वलए दश भर म उनका

अनसरण वकया था

bull बज वकश र राजदर परसाद महादि दसाई और

नरहरी पाररख चोपारण सतयागरह क दौरान गाोिी

जी क सहय गी थ

Q58) उततर (b)

सपषटीकरण

bull बराहमण ो और बौद मठिाररय ो क कर-मकत गााि

अनदान म दन की परथा सतिाहन ो न आरमभ की

थी

Q59) उततर (c)

सपषटीकरण

इस कायिकरम क उददशय वनमनानसार ह

(i) बवनयादी पयििन आिाररक सोरचना का विकास

करना

(ii) चयवनत (पहचान वकय गए) कषतर ो म आजीविका क

सजन क वलए दश क साोसकवतक और विरासत

मलय ो क बिािा दना

(iii) विरासत समारक थथल ो पर विशव सतरीय आिाररक

सोरचना विकवसत करक एक सतत तरीक स

पयििक आकििण म िसद करना

(iv) थथानीय समदाय ो की सवकरय भागीदारी क माधयम

स र रगार ो का सजन करना

(v) र रगार उतपादन और आवथिक विकास क वलए

पयििन कषमता का उन पर परभाि का उपय ग

करना तथा

(vi) िारणीय पयििन आिाररक सोरचना का विकास

करना और उसका उवचत सोचालन तथा

रखरखाि सवनवशचत करना

Q60) उततर (b)

सपषटीकरण

bull यह वनकाय ििि 1987 म अससततव म आया था

bull यह एक राषटर ीय सतर का शीिि सोगठन ह ज भारत

सरकार क जनजातीय मामल ो क मोतरालय क

परशासवनक वनयोतरण क अिीन काम कर रहा ह

bull इसका पोजीकत और परिान कायािलय नई वदलली

म सथथत ह

Q61) उततर (c)

सपषटीकरण

bull परमचोद क उपनयास ो म परमाशरम रोगभवम गबन

कमिभवम और ग दान शावमल ह

bull ग रा रिी ोदरनाथ िग र क दवारा रवचत उपनयास ह

bull अभी हाल ही म मोशी परमचोद की 138िी ो जयोती दश

भर म मनाई गई थी

Q62) उततर (b)

सपषटीकरण

bull ldquoवगदाrdquo पोजाब (भारत) एिो पावकसतान की

मवहलाओो क दवारा तयौहार क समय और फसल

की बिाई तथा किाई क अिसर पर वकया जान

िाला एक पारोपररक दहाती नतय ह

FC19H1003 38

bull इस नतय क माधयम स पोजाबी मवहलाऐो अपनी

परसननता परकि करती ह तथा वगदा क परदशिन क

माधयम स परि िचिसव िाल समाज म मवहलाओो

की दबी हई भािनाओो क परकि करती ह

bull चोवक इस नतय का परि ो क साथ क ई सोबोि नही ो

ह अतः किल मवहलाऐो ही इसम भाग ल सकती

bull हर साल तीज समार ह क दौरान पोजाब म वगदा

नतय वकया जाता ह

तीज भारत क कछ भाग ो म मवहलाओो क दवारा

मनाया जान िाल कई तयौहार ो क वलए एक

वयापक नाम ह

Q63) उततर (a)

सपषटीकरण

- मजम-उल-बहरीन या द समदर ो का सोगम

नामक उललखनीय रचना दारा वशक ह क दवारा

वलखी थी

- भारत क उपराषटर पवत शरी एम िकया नायड न कहा

ह वक राजकमार दारा वशक ह की रचनाएा शाोवत

और सदभाि क बिािा दन क वलए एक तारा सर त

क रप म सामन आ सकती ो ह

- उपराषटर पवत गत ििो क भला वदए गए राजकमार

दारा वशक ह क परदवशित परचवलत करन हत

आय वजत एक परदशिनी का दौरा करन क बाद एक

सभा क सोब वित कर रह थ

- इस परदशिनी का आय जन फर क इस गौवियर

(Francois Gautier) क दवारा lsquoइोवदरा गाोिी नशनल

सिर फॉर द आििसrsquo (The Indira Gandhi

National Centre for the Arts) नई वदलली म

वकया गया था

Q64) उततर (c)

सपषटीकरण

- ग मतशवर परवतमा जन भगिान बाहबली क

समवपित ह

- यह एक एक-चटटानी पतथर की मवति ह

- राषटर पवत राम नाथ क विोद न शरिणबलग ला

(कनाििक) म आय वजत वकय जान िाल भवय

अवभिक समार ह महामसतकावभिक का

उदघािन वकया था

- यह समार ह 12 ििो म एक बार ह ता ह

Q65) उततर (c)

सपषटीकरण

bull पराची घािी पराची नदी क चार ो ओर फली हई थी

bull पराची घािी िीर-िीर विलपत ह गई थी

bull पराची नदी भिनशवर स वनकलती ह

bull यह महानदी की एक सहायक नदी ह और यह

परी खदाि किक तथा जगतवसोहपर वजल ो स

ह कर बहती ह

bull इस नदी क पर कषतर क पराची घािी कहा जाता ह

bull यह नदी बोगाल की खाडी म वगरती ह

परातासतवक साकषय स पता चलता ह वक पराची घािी

सभयता हडपपा और म हनज दाड द न ो की

पिििती ह

Q66) उततर (d)

सपषटीकरण

य समारक छतरपर वजल (मधय परदश) म विोधयाचल

पिित शरोखला म सथथत ह

Q67) उततर (a)

सपषटीकरण

bull थॉिस ऑन पावकसतान नामक पसतक डॉ बी

आर अमबडकर न वलखी थी

bull डॉ बी आर अमबडकर की जयोती क अिसर पर

भारत क राषटर पवत न भारत की इस महान हसती

क शरदाोजवल अवपित की थी

bull डॉ बी आर अमबडकर न 1924 म वडपरथड

कलावसर इोसटीटयि (दवलत िगि सोथथान -

बवहषकत वहतकाररणी सभा) और 1927 म समाज

समता सोघ की थथापना की थी

bull अमबडकर का धयान वशकषा कषतर की ओर भी था

bull उनह ोन वशकषा क वनमन िगो म फलान क वलए

पीपलस एजकशन स साइिी (The Peoples

Education Society) क नाम स महाविदयालय ो क

नििकि और छातरािास ो की थथापना की थी

FC19H1003 39

Q68) उततर (b)

सपषटीकरण

bull महरगि भारतीय उपमहादवीप म एक परवसद

निपािाण बसती ह ज बलवचसतान पराोत

पावकसतान म सथथत ह

bull दचपलली (आोधर परदश) क पास नागलर नदी क

पिी ति ो पर चना पतथर क बलॉक क विशाल

विसतार म एक पिि-ऐवतहावसक रॉक आिि थथल की

ख ज की गई ह

bull इसन 1500-2000 ईसा पिि क दौरान गोिर (आोधर

परदश) म विकवसत निपािाण सभयता पर परकाश

डाला ह

Q69) उततर (c)

सपषटीकरण

bull 12िी ो सदी और 13िी ो सदी म काकाविय िोश का

उदय हआ था

bull ि पहल कलयाण क पवशचमी चालकय ो क सामोत थ

bull परारोभ म उनह ोन िारोगल (तलोगाना) क पास एक

छ ि स कषतर पर शासन वकया था

bull उनह ोन ldquoनायक वयिथथाrdquo की शरआत की थी

वजस बाद म विजयनगर क राय शासक ो न

अपनाया और विकवसत वकया था

Q70) उततर (a)

सपषटीकरण

bull गाोिीजी क अनशन स वमल मावलक ो पर दबाि

पडा था ज अोततः शरवमक ो क ितन म 35 परवतशत

की िसद करन क वलए सहमत हए थ

bull गगल (Google) न अनसया साराभाई वजनह ोन

भारत क शरवमक आोद लन म एक अगरणी भवमका

वनभाई थी की 132िी ो जयोती डडल (Doodle) का

वनमािण करक मनाई

Q71) उततर (d)

सपषटीकरण

भारत स यनसक की मानिता की अमति साोसकवतक

विरासत की परवतवनवि सची म वनमनवलसखत शावमल ह

bull कवडयटटम करल का सोसकत रोगमोच

bull मवडयिि करल का अनषठान रोगमोच और नतय

नाविका

bull िवदक मि जाप की परोपरा

bull राजथथान क कालबवलया ल क गीत और नतय

bull रामलीला रामायण का पारोपररक परदशिन

bull सोकीतिन मवणपर का अनषठान गायन ढ ल िादन

और नतय

bull रममन भारत क गििाल वहमालय का िावमिक

तयौहार और अनषठान रोगमोच

bull जाोदीयाला गर पोजाब क ठठर ो की पीतल और

ताोब क वशलप स वनवमित बतिन ो की पारोपररक कला

bull छाऊ नतय पिी भारतीय राजय ो म जनमी शासतरीय

भारतीय नतय कला

bull लददाख का बौद मि जाप िर ाोस-वहमालयी लददाख

कषतर तथा जमम-कशमीर म पवितर बौद गरोथ ो का पाठ

bull य ग

bull नौर र

bull को भ मला

Q72) उततर (b)

सपषटीकरण

bull भारत क राषटर पवत शरी राम नाथ क विोद न

वकसामा नागालड म हॉनिवबल मह रति और

राजय गठन वदिस समार ह का उदघािन वकया

था

bull हॉनिवबल मह रति का नाम भारतीय हॉनिवबल क

नाम पर पडा ह ज एक विशाल और रोगीन जोगली

पकषी ह

bull यह पकषी नागालड राजय की अविकतर जनजावतय ो

की ल ककथाओो म उसललसखत ह

bull नागालड की परमख मानयता परापत जनजावतयाा ह

अोगामी आओ चखसोग चाोग ककी रगमा और

रवलोग आवद

bull ओोग जारिा और ससिनलीस अोडमान-वनक बार

दवीप समह की जनजावतयाा ह

FC19H1003 40

Q73) उततर (c)

सपषटीकरण

bull दकन म राषटर कि शासन दसिी ो सदी क अोत तक

लगभग 200 ििो तक रहा था

bull राषटर कि शासक अपन िावमिक विचार ो म सवहषण

bull उनह ोन न किल शि िमि और िषणि िमि बसलक

जन िमि क भी सोरकषण वदया था

bull एल रा म वशि क परवसद रॉक कि मोवदर का

वनमािण नौिी ो सदी म राषटर कि राजा कषण परथम न

करिाया था

bull उसका उततराविकारी अम घििि जन था लवकन

उसन अनय िमो क भी सोरकषण परदान वकया था

bull राषटर कि ो न मसलमान वयापाररय ो क बसन की

अनमवत दी थी

bull उनह न अपन अविराजय ो म इसलाम क उपदश दन

की भी अनमवत दी थी

bull अभी हाल ही म पाोडिलागटटा (तलोगाना) क

परागवतहावसक चटटान वचतर ो क कषरण की बिती हई

घिनाएा एक गोभीर वचोता का वििय ह

bull यह परागवतहावसक चटटान क नकसान पहाचा

सकता ह

bull पाोडिलागटटा वनमनवलसखत क वलए जाना जाता ह

- 10000 ईसा पिि स 8000 ईसा पिि क वचवतरत

चटटानी आशरय ो क वलए

- राषटर कि काल क एक 8 िी ो सदी क

वशलालख क वलए और

- 12िी ो सदी क काकविय सामराजय क वभवतत

वचतर ो क वलए

Q74) उततर (b)

सपषटीकरण

bull 1828 म राजा राम म हन रॉय न एक नय िावमिक

समाज बरहम सभा की थथापना की थी वजस बाद

म बरहम समाज क नाम स जाना गया था

bull दिदरनाथ िग र न ततवब विनी सभा की अधयकषता

की थी ज आधयासिक सतय की ख ज म सोलि

थी

bull इसका उददशय वहोद िमि क शद करन का और

एकशवरिाद (एक ईशवर म आथथा) का परचार करना

था

bull नय समाज की थथापना क आिार थ कारण

(तकि ) क द सतमभ तथा िद और उपवनिद

bull अभी हाल ही म सािारण बरहम समाज का कछ

काननी मदद ो क लकर पवशचम बोगाल सरकार क

साथ काननी वििाद चल रहा ह

Q75) उततर (c)

सपषटीकरण

bull भारत म वचशती वसलवसल की थथापना खवाजा

म इनददीन वचशती क दवारा की गयी थी

bull ि 1192 ईसवी क आसपास भारत आय थ

bull वचशतीय ो क बारहिी ो शताबदी क उततरािि म भारत

म आन िाल सफीय ो क समह ो म सबस

परभािशाली माना जाता ह

bull उनह ोन थथानीय िातािरण क साथ सफलतापििक

अनकलन वकया और उनह ोन भारतीय भसकत

परोपराओो क कई पहलओो क अपनाया

bull अजमर म सफी अपरकि खवाजा म इनददीन वचशती

की ऐवतहावसक दरगाह क एक नया रप दन की

तयारी की जा रही ह

bull इस 13िी ो शताबदी की दरगाह क ldquoसवचछ

आइकॉवनक थथल ोrdquo (Swacch Iconic Places) म

शावमल वकया गया ह ज परवतवषठत विरासत

आधयासिक और साोसकवतक थथान ो पर क वदरत

य जना ह

FC19H1003 41

ANSWERS amp EXPLANATION OF

NCERT History Class VI-X + Current Affairs

(FC19E1003)

Q1) Answer c

Explanation

Rigveda consists of more than a

thousand hymns dedicated to gods and

goddesses These hymns were

composed by sages and learnt by men

however a few were composed by

women like Apala Ghosa Lopamudra

Maitreyi and Gargi

Rigveda consists of many hymns in the

form of dialogues We get an example of

a dialogue between a sage named

Vishwamitra and two rivers (Beas and

Sutlej) that were worshipped as

goddesses This suggests that he

belonged to the Vedic period

Q2) Answer b

Explanation

Traces of ash have been found from

Kurnool Caves suggesting that people

were familiar with the use of fire

It is situated in Andhra Pradesh

Q3) Answer c

Explanation

Burzahom is a prehistoric site in

present day Kashmir where people built

pit houses which were dug into the

ground with steps leading into them

These may have provided shelter in cold

weather

Q4) Answer c

Explanation

Epigraphy is defined as the study of

inscriptions

Manuscriptology is the study of history

and literature through the use of hand

written documents

Palaeography refers to the study of

ancient writing systems and the

deciphering and dating of historical

manuscripts

Numismatics refers to the study of

coins

Q5) Answer a

Explanation

Charaka Samhita was written by

Charaka and is an important book on

Ayurveda and medicine

He was a practitioner of the traditional

system of Indian medicine known as

Ayurveda

Charaka is thought to have flourished

sometime between the 2nd century BCE

and the 2nd century CE

Q6) Answer b

Explanation

Bhaga refers to the tax on crops which

was fixed at 16th of the production

Kammakaras is the term used for the

landless agricultural labour class

Ashvamedha also known as horse

sacrifice is a ritual where a horse is let

loose to wander freely and it was

guarded by the rajarsquos men

Q7) Answer (d)

Explanation

In the Rigvedic period horses were

yoked to chariots that were used in

battles fought to capture land cattle

etc This suggests that the use of horse

chariots began much before the period

of Mahajanapadas

The battles were fought in the Rigvedic

period for cattlersquos lands water an even

to capture people Most men took part

in these wars however there was no

regular army but there were assemblies

where people met and discussed

matters of war Regular armies became

a feature in the Mjahajanapada period

including vast armies of foot soldiers

chariots and elephants

RAUSIAS-FC19E1003 42

Q8) Answer (a)

Explanation

Buddha belonged to the Sakya clan and

passed away at Kusinara

Buddha taught in Prakrit which was the

common language of people

Q9) Answer c

Explanation

There were six schools of philosophy in

ancient India These are known as

Vaishesika Nyaya Samkhya Yoga

Purva Mimansa and Vedanata or Uttara

Mimansa They were founded by sages

Kanada Gautama Kapila Patanjali

Jamini and Vyasa respectively

Q10) Answer b

Explanation

The teachings of Mahavira were

compiled at Valabhi in 6th century AD

Q11) Answer (c)

Explanation

Chanakya is traditionally identified as

Kautilya or Vishnugupta who authored

the ancient Indian political treatise the

Arthashastra

Q12) Answer d

The national emblem of India is an

adaptation of the Lion Capital atop the

Ashoka Pillar of Sarnath Uttar Pradesh

and is combined with the National

Motto Satyameva Jayate

The Rampurva Bull gets the name from

the site of its discovery Rampurva in

Bihar

It is noted for its delicately sculpted

model demonstrating superior

representation of soft flesh sensitive

nostrils alert ears and strong legs It is

a mixture of Indian and Persian

elements

Sankissa is situated in Uttar Pradesh

India

Q13) Ans(a)

Kunwar Singh was a notable leader during the Revolt of 1857 He belonged

to a royal house of Jagdispur Bihar

Q14) Answer b

Explanation

The term Vellalar was used for large

landowners

Q15) Answer c

Explanation

Arikamedu was a coastal settlement

where ships unloaded goods from

distant lands Finds here include a

massive brick warehouse pottery

including amphorae and Arretine ware

Roman lamps glassware and gems have

also been found at the site

Q16) Answer a

Explanation

Muvendar is a Tamil word mentioned in

Sangam poems meaning three chiefs

used for the heads of three ruling

families the Cholas Cheras and

Pandyas

Q17) Ans (c)

Several tribal or kin-based assemblies

such as the Sabha Vidatha and gana

are mentioned in the Rig-veda The

Sabha and the samiti mattered a great

deal in early Vedic times so much so

that the chiefs or the kings showed an

eagerness to win their support

Q18) Ans (a)

Jainism recognised the existence of the

gods but placed them lower than the

jina and did not condemn the varna

system as Buddhism did

Q19) Answer (d)

Explanation

Cholas and Pandyas had developed

powerful coastal cities The most

important city of Cholas was Puhar or

Kaveripattinam and Madurai was the

capital of Pandyas

Q20) Answer b

Explanation

Buddhacharita is the biography of

Buddha and was written by

RAUSIAS-FC19E1003 43

Ashvaghosha

Q21) Answer (a)

Explanation

Tamil poet Appar was a Shiva devotee

So he was a Nayanar saint

Q22) Answer d

Explanation

Samudragupta was a prominent Gupta

ruler whose coins depict him playing a

veena indicating his love for music We

get important historic information from

his Allahabad Prashasti which was

composed by his court poet Harisena

Q23) Answer (b)

Explanation

Vikrama Samvat was founded by

Chandragupta II in the 58 BC as a

mark of victory over the Shakas and

assumed the title of Vikramaditya

Banabhatta wrote Harshavardhanarsquos

biography the Harshacharita in

Sanskrit

Q24) Answer c

Explanation

Sandhi-vigrahika was the minister of

war and peace

Sarthavaha was the leader of the

merchant caravans

Q25) Answer a

Explanation

Xuan Zang (Hsuan-tsang) was a

Chinese traveller who came during the

reign of Harshavardhana

In the decade that began in 630 AD

Xuan Zang came to India through

Kashmir after visiting Central Asia Iran

and Afghanistan

He travelled from north to east and lived

in Bihar for a couple of years

At Nalanda University Xuan Zang

interacted with students and scholars

mastered local languages and

discovered Buddhist stupas

Q26) Answer c

Explanation

Pradakshina patha is a circular path

laid around a stupa in Buddhist

architecture While the rest are a part of

temple architecture

Q27) Answer d

Explanation

All the above-mentioned temples have

an elaborate use of bricks (baked

bricks) along with stone

Q28) Ans (c)

Muhammad Quli Qutab was the Sultan

of Golconda He was a contemporary of

Akbar was very fond of literature and

architecture

The Sultan was a great poet and he

wrote in Dakhini Urdu Persian and

Telgu and has left an extensive diwan or

collection

Recently the Archaeological Survey of

India (ASI) will be using Ground

Penetrating Radar (GPR) to map the

contours of the area around the Bagh-e-

Naya Qila excavated garden inside the

Golconda Fort in Telangana

Q29) Answer a

Explanation

Silappadikaram is a famous Tamil epic

which was written by Ilango around

1800 years ago It is a story of a

merchant named Kovalan who fell in

love with a courtesan named Madhavi

Manimekalai tells the story of the

daughter of Kovalan and Madhavi

Q30) Answer (a)

Explanation

Charaka is the author of Charaka

Samhita which is an important work of

Ayurveda and medicines

Brahmaguptarsquos fame rests mostly on his

Brahma-sphuta-siddhanta which was

an astronomical work It was translated

into Arabic in Baghdad and had a major

impact on Islamic mathematics and

astronomy

Late in his life Brahmagupta wrote

Khandakhadyaka which was an

RAUSIAS-FC19E1003 44

astronomical handbook that employed

Aryabhatarsquos system of starting each day

at midnight

Q31) Answer (c)

Explanation

Amir Khusrau was a famous sufi

musician poet and scholar In 1318 he

noted that there was different language

in every region of this land (Hindustan)

Lahori Kashmiri Dvarsamudri (in

Southern Karnataka) Telangana (in

Andhra Pradesh) Gujari (in Gujarat)

Marsquobari (in Tamil Nadu) Awadhi (in

eastern Uttar Pradesh) and Hindawai (in

the area around in Delhi) etc He went

to explain that Sanskrit did not belong

to any region and that only brahmans

knew it

Q32) Answer c

Explanation

Hiranyagarbha refers to the golden

womb When this ritual was performed

with the help of Brahmanas it was

thought to lead to the rebirth of the

sacrificer as a Khastriya

Q33) Answer d

Explanation

Kadamai refers to a tax on land

revenue

Gwalior Prashasti describes the exploits

of Nagabhata who was a Pratihara king

Q34) Answer b

Explanation

Rajatarangini is a Sanskrit text written

by Kalhana in the 12th century

It was historical chronicle of early India

It is justifiably considered to be the best

and most authentic work of its kind

It covers the entire span of history in

the Kashmir region from the earliest

times to the date of its composition

Q35) Answer c

Explanation

ldquoUrrdquo was the general assembly of the

village ldquoUrrdquo consisted of all the

taxpaying residents of an ordinary

village

Q36) Answer (a)

Explanation

Tarikh was a form of history writing in

the Delhi Sultanate The authors of

tawarikhs were learned men which

included secretaries administrators etc

Q37 Answer (a)

Explanation

Alauddin chose to pay his soldiers salaries in cash rather than iqtas The soldiers would buy their supplies from merchants in Delhi and it was thus feared that merchants would raise their prices To stop this Alauddin controlled the prices of goods in Delhi Prices were carefully surveyed by officers and merchants who did not sell at the prescribed rates were punished

Q38) Answer (d)

Explanation

Delhi first became the capital of a

kingdom under the Tomara Rajputs

who were defeated in the middle of the

twelfth century by the Chauhans (also

referred to as Chahamanas) of Ajmer

It was under the Tomaras and

Chauhans that Delhi became an

important commercial centre Many rich

Jaina merchants lived in the city and

constructed several temples Coins

minted here called dehliwal had a wide

circulation

Q39) Answer (c)

Explanation

Moth ki Masjid was built in the reign of

Sikandar Lodi by his minister

Begumpuri mosque built in the reign of

Muhammad Tughluq was the main

mosque of Jahanpanah the ldquoSanctuary

of the Worldrdquo and his new capital in

Delhi

Quwwat al ndash Islam mosque was

enlarged by Iltutmish and Alauddin

Khalji The minar was built by three

Sultansndash Qutbuddin Aybak Iltutmish

and Firuz Shah Tughluq

RAUSIAS-FC19E1003 45

Q40) Answer (c)

Explanation

Under the Mughals mansabdar was

referred to an individual who held a

mansab ie rank and he received his

salary as revenue assignments called

jagirs

Q41) Ans (b)

The Quit India Movement was a

spontaneous revolt of people against

British rule

The All India Congress Committee met

at Bombay on 8 August 1942 It passed

the famous resolution Quit India and

proposed the starting of a non-violent

mass struggle under Gandhis

leadership to achieve this aim But on

the very next day Gandhi and other

eminent leaders of the Congress were

arrested The Congress was once again

declared illegal

Q42) Ans (c)

The Simon Commission refers to a

group of seven MPs from the United

Kingdom constituted to suggest

constitutional reforms for British India

The Commission consisted of only

British members headed by one of the

senior British politicians Sir John

Simon

So the people of India agitated against

the arrival of Simon Commission

Q43) Ans (a)

He was widely known for his

unfavourable opinion of the economic

consequences of the British rule in

India

In his many writings and speeches and

especially in Poverty and Un-British

Rule in India Naoroji argued that India

was too highly taxed and that its wealth

was being drained away to England

He did not interpret the ancient Indian

texts and restored the self-confidence of

Indians And also he did not stress the

need for eradication of all the social

evils before anything else

Q44) Ans (c)

In August 1932 Prime Minister

MacDonald announced his Communal

Award Great Britainrsquos unilateral

attempt to resolve the various conflicts

among Indiarsquos many communal

interests

The award which was later

incorporated into the act of 1935

expanded the separate-electorate

formula reserved for Muslims to other

minorities including Sikhs Indian

Christians Anglo-Indians Europeans

distinct regional groups Gandhi

undertook a ldquofast unto deathrdquo against

that offer which he viewed as a

nefarious British plot to divide the

Indian society

Q45) Ans (b)

In British India apart from existing

imports and exports there was also a

particular amount of money which

colonial India contributed towards

administration maintenance of the

army war expenses pensions to retired

officers and other expenses accrued by

Britain towards maintenance of her

colony These were known as Home

charges and were paid for almost

entirely by India

The Home charges was made of

following components-

- Interest payable on Indian debt

- Dividend to shareholders of East

India Company

- Funds used to support the India

Office in London

- Funds used to pay salaries and

pensions of British personnel

engaged in India

- Interest on the railways

- Civil and military charges

- Store purchases in England

Q46) Ans (b)

The Lahore session of the Indian

National Congress was held in 1929

under the Presidentship of Jawaharlal

Nehru

The Lahore session of the Indian

National Congress witnessed significant

RAUSIAS-FC19E1003 46

developments in the Indian national

movement

- First the election of Jawaharlal

Nehru to the post of Presidentship of

the Congress was a clear indication

of the growing strength of the

Leftists in the Congress

- Secondly it was in this session that

the Congress for the first time raised

the demand for complete

independence Such demand was

not raised from the Congress

platform earlier

Q47) Ans (b)

It did not provide for separate

electorates for any community or

weightage for minorities However it did

allow for the reservation of minority

seats in provinces having minorities of

at least ten per cent but this was to be

in strict proportion to the size of the

community

There was no provision for complete

Independence for India

Q48) Ans (c)

The religion of early Vedic Aryans was

primarily of worship of nature and

Yajnas

The early Aryan religion was kind of

nature worship Actually the forces

around them which they could not

control or understand were invested

with divinity and were personified as

male or female gods And they

performed some Yajnas also

Q49) Ans (b)

The roads and river-routes were not

immune from robbery It is notable that

Yuan Chwang (Hiuen Tsang) was

robbed of his belongings during

Harshvardanarsquos period

Q50) Ans (c)

Q51) Ans (b)

Purandara Dasa was a saint and great

devotee of Lord Krishna

There is much speculation about where

Purandara Dasa regarded as the

Pitamaha of Carnatic music was born

Recently an expert committee

constituted by the Kannada University

Hampi has come to the conclusion that

Kshemapura Shivamogga district

Karnataka is the birth place of

Purandara Dasa

Q52) Ans (c)

Sri Tyagaraja Sri Shyama Shastry and Sri Muthuswami Dikshitar are considered the trinity of Carnatic music and with them came the golden age in Carnatic music in the 18th-19th

century

Q53) Ans d)

Recently a rare sarcophagus (stone

coffin) which is 2000 years old from the

Iron AgendashMegalithic era was discovered

from a rock-cut cave at Viyur village of

Kollam near Koyilandy in Kozhikode

district Kerala

The coffin containing bone fragments

was found during an excavation ldquoSo

far such a rare finding has been

discovered only from two sites

in Kerala Both these sarcophagi were

recovered from Megalithic sites at

Chevayur and Atholi also in Kozhikode

district

Q54) Ans a)

The megalithic culture in South India was a full-fledged Iron Age culture

Q55) Ans d)

The Cholas Pandyas and Keralaputras

(Cheras) mentioned in Ashokan

inscriptions were probably in the late

megalithic phase of material culture

Q56) Ans d)

Q57) Ans (b)

Raj Kumar Shukla followed Gandhiji all

over the country to persuade him to

come to Champaran to investigate the

problem associated with tinkathia

system

RAUSIAS-FC19E1003 47

Brij Kishore Rajendra Prasad Mahadev

Desai and Narhari Parikh accompanied

Gandhi ji during the Champaran

Satyagraha

Q58) Ans (b)

The Satvahanas started the practice of granting tax-free villages to brahmanas and Buddhist monks

Q59) Ans c)

The objectives of the Programme are

listed as under

- Developing basic tourism

infrastructure

- Promoting cultural and heritage

value of the country to generate

livelihoods in the identified regions

- Enhancing the tourist attractiveness

in a sustainable manner by

developing world-class

infrastructure at the heritage

monument sites

- Creating employment through active

involvement of local communities

- Harnessing tourism potential for its

effects on employment generation

and economic development

- Developing sustainable tourism

infrastructure and ensuring proper

Operations and maintenance

therein

Q60) Ans (b)

The Tribal Cooperative Marketing

Development Federation of India

(TRIFED) came into existence in 1987

It is a national-level apex organization

functioning under the administrative

control of Ministry of Tribal Affairs

Govt of India

TRIFED has its registered and Head

Office located in New Delhi

Q61) Ans (c)

Premchandrsquos novels include

Premashram Rangabhumi Ghaban

Karmabhumi and Godan

Gora is a novel written by Rabindranath

Tagore

138th birth anniversary of Munshi

Premchand was celebrated across the

country

Q62) Ans (b)

Giddha is a traditional pastoral dance

performed by the women of the Punjab

India and Pakistan at festival times

and at the sowing and reaping of the

harvest

By this dance the Punjabi women

reveal their joy expel their suppressed

feelings in a male dominated society

through the performance of Giddha

Since this dance has nothing to do with

men only women can participate in it

During the Teej celebrations Giddha

dance is celebrated in Punjab every

year Teej is a generic name for a

number of festivals that are celebrated

by women in some parts of India

Q63) Ans (a)

Dara Shukoh wrote the remarkable

work called ldquoMajma-ul-Bahrainrdquo or the

ldquoThe confluence of two seasrdquo

The Vice President of India Shri M

Venkaiah Naidu has said that Prince

Dara Shukohrsquos writings can come as a

refreshing source for infusing peace and

harmony He was addressing the

gathering after visiting the exhibition

that showcases the forgotten Prince of

yesteryears Dara Shukoh organized by

Mr Francois Gautier at Indira Gandhi

National Centre for the Arts in New

Delhi

Q64) Ans (c)

The statue Gommateshwara is

dedicated to the Jain God Bahubali

It is a monolithic statue

President Ram Nath Kovind

inaugurated the grand anointing

ceremony mdash Mahamastakabhisheka mdash

held once in 12 years at

Shravanabelagola (Karnataka)

Q65) Ans (c)

Prachi Valley had come up around the

Prachi river Prachi Valley gradually

disappeared

RAUSIAS-FC19E1003 48

The Prachi river originates from

Bhubaneswar

It is a tributary of the Mahanadi and

flows through the districts of Puri

Khurda Cuttack and Jagatsinghpur

and the entire region of the river is

termed as the Prachi Valley

It falls into the Bay of Bengal

Archaeological evidence shows that the

Prachi Valley Civilisation predates both

Harappa and Mohenjo-Daro

The Prachi river originates from

Bhubaneswar

Q66) Ans (d)

These monuments are located in

Chhatarpur district Madhya Pradesh

within Vindhya mountain range

Q67) Ans (a)

The book lsquoThoughts on Pakistanrsquo was

written by Dr BR Ambedkar

On the occasion of the birth anniversary

of Dr BR Ambedkar the president of

India pays homage to this icon of India

In 1924 he founded the Depressed

Classes Institute (Bahishkrit Hitkarini

Sabha) and in 1927 the Samaj Samata

Sangh

Another area of attention for Ambedkar

was education For its spread among

the low classes he set up a network of

colleges by the name of Peoples

Education Society and founded hostels

Q68) Ans(b)

Mehrgarh is a famous Neolithic

settlement in the Indian subcontinent

which is situated in Baluchistan

province Pakistan

A pre-historic rock art site is discovered

in the vast expanse of limestone blocks

on the eastern banks of Naguleru river

near Dachepalli (Andhra Pradesh) It

has thrown light on the Neolithic

civilisation that flourished in Guntur

(Andhra Pradesh) during 1500-2000

BC

Q69) Ans (c)

The 12th and the 13th centuries saw

the emergence of the Kakatiyas They

were at first the feudatories of the

Western Chalukyas of Kalyana Initially

they ruled over a small territory near

Warangal (Telangana)

They introduced Nayakships which was

later adopted and developed by the

Rayas of Vijayanagara

Q70) Ans (a)

The fast had effect of putting pressure

on mill owners who finally agreed to

give the workers a 35 per cent increase

in wages

Google celebrated with a doodle the

132nd birth anniversary of Anasuya

Sarabhai who played a pioneering role

in Indiarsquos labour movement

Q71) Ans (d)

The UNESCOrsquos list of the representative

list of the intangible cultural heritage of

humanity from India are

- Koodiyattam Sanskrit Theatre of

Kerala

- Mudiyettu ritual theatre and dance

drama of Kerala

- Tradition of Vedic Chanting

- Kalbelia folk songs and dances of

Rajasthan

- Ramlila Traditional Performance of

the Ramayana

- Sankirtana ritual singing

drumming and dancing of Manipur

- Ramman religious festival and

ritual theatre of the Garhwal

Himalayas India

- Traditional brass and copper craft of

utensil making among the Thatheras

of Jandiala Guru Punjab India

- Chhau dance classical Indian dance

originated in the eastern Indian

states

- Buddhist chanting of Ladakh

recitation of sacred Buddhist texts

in the trans-Himalayan Ladakh

region Jammu and Kashmir India

- Yoga

- Nouroz

- Kumbh Mela

RAUSIAS-FC19E1003 49

Q72) Ans(b)

The President of India Shri Ram Nath Kovind inaugurated the Hornbill Festival and State Formation Day celebrations of Nagaland in Kisama

The festival is named after the Indian hornbill the large and colourful forest bird which is displayed in the folklore of most of the states tribes

The major recognized tribes of Nagaland are Angami Ao Chakhesang Chang

Kuki Rengma and Zeling etc

Onge Jarawa and Sentinelese are the

tribes of Andman amp Nicobar Islands

Q73) Ans (c)

The Rashtrakutas rule in the Deccan lasted for almost two hundred years till the end of the tenth century The Rashtrakutas rulers were tolerant in their religious views and patronized not only Shaivism and Vaishnavism but

Jainism as well

The famous rock-cut temple of Shiva at Ellora was built by one of the Rashtrakutas kings Krishna I in the ninth century His successor Amoghavarsha was a Jain but he also

patronized other faiths

The Rashtrakutas allowed Muslims traders to settle and permitted Islam to

be preached in their dominions

Recently increasing defacement at the prehistoric rock paintings of Pandavulagutta Telangana has created a cause for grave concern It can spoil

the prehistoric rock

Pandavulagutta is home to

- Painted rock shelters dating to

10000 BC-8000 BC

- An 8th century inscription of the

Rashtrakuta period and

- Painted frescoes from the 12th century Kakatiya empire

Q74) Ans (b)

In 1828 Raja Ram Mohan Roy founded a new religious society the Brahma Sabha later known as the Brahmo

Samaj

Debendranath Tagore headed the Tattvabodhini Sabha which was

engaged in search of spiritual truth

Its purpose was to purify Hinduism and to preach monotheism or belief in one God

The new society was to be based on the twin pillars of reason and the Vedas and

Upanishads

Recently Sadharan Brahmo Samaj (SBS) has entered into a legal battle with the West Bengal government due

to some legal issue

Q75) Ans (c)

The Chishti order was established in India by Khwaja Moinuddin Chishti who came to India around 1192 The Chishtirsquos are considered to be the most influential of the groups of Sufis who migrated to India in the late twelfth century They adapted successfully to the local environment and adopted several features of Indian devotional

traditions

The historical dargah of Sufi mystic Khwaja Moinuddin Chishti in Ajmer is all set to get a facelift This 13 th century dargah has been included among the Swachh Iconic Places a clean-up initiative focused on iconic

heritage spiritual and cultural places

Page 27: GENERAL STUDIES (PAPER I) · Test is part of Rau’s IAS Test series for Preliminary Exam 2019 FOUNDATION + CURRENT AFFAIRS GENERAL STUDIES (PAPER –I) FOUNDATION TEST –III TOPIC:

RAUSIAS-FC19E1003 27

Q70) Which of the following statements

isare correct

1 During the Ahmedabad Mill Strike

Mahatma Gandhi undertook a fast

unto death to strengthen the

workersrsquo resolve

2 The fast had effect of putting

pressure on mill owners who

finally agreed to give the workers a

15 per cent increase in wages

Select the correct answer using the code

given below

(a) 1 only

(b) 2 only

(c) Both 1 and 2

(d) Neither 1 nor 2

Q71) Which of the following are included in

the UNESCOrsquos list of the representative

list of the intangible cultural heritage of

humanity from India

1 Mudiyettu

2 Sankirtana

3 Kumbh Mela

Select the correct answer using the code

given below

(a) 1 and 2 only

(b) 2 and 3 only

(c) 3 only

(d) 1 2 and 3

Q72) Which of the following tribes isare

related to Nagaland

1 Angami

2 Kuki

3 Jarawa

Select the correct answer using the code

given below

(a) 1 only

(b) 1 and 2 only

(c) 2 only

(d) 1 2 and 3

Q73) Which of the following statements

isare correct

1 Rashtrakuta kingdom was founded by Dantidurga who established his capital at Manyakhet

2 Amoghavarsha a Rashtrakuta king was an author and is credited with writing the first

Kannada book on poetics

Select the correct answer using the code given below

(a) 1 only

(b) 2 only

(c) Both 1 and 2

(d) Neither 1 nor 2

Q74) Which of the following statements isare correct

1 Keshab Chandra Sen headed the Tattvabodhini Sabha which was engaged in search of spiritual truth

2 The Brahmo Samaj laid emphasis on human dignity opposed idolatry and criticized such social

evils as the practice of Sati

Select the correct answer using the code given below

(a) 1 only

(b) 2 only

(c) Both 1 and 2

(d) Neither 1 nor 2

Q75) Which of the following statements isare correct

1 The Chishti order was established in India by Khwaja Moinuddin

Chishti

2 A major feature of the Chishti tradition was austerity including maintaining a distance from the

worldly power

Select the correct answer using the code

given below

(a) 1 only

(b) 2 only

(c) Both 1 and 2

(d) Neither 1 nor 2

T e s t i s p a r t o f R a u rsquo s I A S T e s t s e r i e s f o r P r e l i m i n a r y E x a m 2 0 1 9

FOUNDATION + CURRENT AFFAIRS

GENERAL STUDIES (PAPER ndashI)

FOUNDATION TEST ndashIII

SUBJECT NCERT History Class VI-X + Current Affairs

Time Allowed 1frac12 Hours Maximum Marks 150

I NSTRUCT IONS

1 IMMEDIATELY AFTER THE COMMENCEMENT OF THE EXAMINATION YOU SHOULD CHECK

THAT THIS TEST BOOKLET DOES NOT HAVE ANY UNPRINTED OR TORN or MISSING PAGES OR

ITEMS ETC IF SO GET IT REPLACED BY A COMPLETE TEST BOOKLET

2 This Test Booklet contains 75 items (questions) Each item is printed both in Hindi and English

Each item comprises four responses (answers) You will select the response which you want to mark

on the Answer Sheet In case you feel that there is more than one correct response mark the

response which you consider the best In any case choose ONLY ONE response for each item

3 You have to mark all your responses ONLY on the separate Answer Sheet (OMR sheet) provided

Read the directions in the Answer Sheet

4 All items carry equal marks

5 Before you proceed to mark in the Answer Sheet the response to various items in the Test booklet

you have to fill in some particulars in the Answer Sheet as per instructions contained therein

6 After you have completed filling in all your responses on the Answer Sheet and the examination has

concluded you should hand over to the Invigilator only the Answer Sheet You are permitted to

take away with you the Test Booklet

7 Penalty for wrong answers

THERE WILL BE PENALTY FOR WRONG ANSWERS MARKED BY A CANDIDATE IN THE

OBJECTIVE TYPE QUESTION PAPERS

(i) There are four alternatives for the answer to every question For each question for which a

wrong answer has been given by the candidate one-third of the marks assigned to that

question will be deducted as penalty

(ii) If a candidate gives more than one answer it will be treated as a wrong answer even if one of

the given answers happens to be correct and there will be same penalty as above to that

question

(iii) If a question is left blank ie no answer is given by the candidate there will be no penalty for

that question

T h i s t e s t i s p a r t o f R a u rsquo s I A S T e s t s e r i e s f o r P r e l i m i n a r y E x a m 2 0 1 9

Test Code

FC19E1003

FC19H1003 29

Answers and Explanations of

NCERT History Class VI-X + Current Affairs (FC19E1003)

Q1) उततर (c)

सपषटीकरण

- ऋगवद म दविय ो और दिताओो क समवपित एक

हजार स अविक सत तर (शल क) ह

- य शल क ऋविय ो क दवारा रच गए थ और परि ो

दवारा सीख जात थ

- हालाोवक कछ शल क मवहलाओो (जस वक अपाला

घ सा ल पामदरा मतरयी और गागी) क दवारा भी रच

गए थ

- ऋगवद म सोिाद क रप म कई शल क मौजद ह

- हम विशवावमतर नामक एक ऋवि और दविय ो क

रप म पजी जान िाली द नवदय ो (वयास और

सतलज) क बीच िाताि का उदाहरण वमलता ह

- इसस पता चलता ह वक विशवावमतर िवदक काल स

सोबोवित थ

Q2) उततर (b)

सपषटीकरण

- करनल गफाओो स राख क अिशि परापत हए ह

ज इस ओर सोकत करत ह वक ततकालीन ल ग

अवि क उपय ग स पररवचत थ

- य गफाएो आोधर परदश म सथथत ह

Q3) उततर (c)

सपषटीकरण

bull बरािह म ितिमान कशमीर म सथथत एक

परागवतहावसक थथल ह जहाो ल ग गडढ क घर ो का

वनमािण करत थ

bull य घर जमीन क ख द कर बनाए जात थ तथा नीच

जान क वलए सीवियाा ह ती थी

bull ऐसा अनमान लगाया जाता ह वक य घर ठो ड क

मौसम म आशरय परदान करत थ

Q4) उततर (c)

सपषटीकरण

bull परालख-विदया (Epigraphy) क वशलालख ो क

अधययन क रप म पररभावित वकया जाता ह

bull हसतवलसखत दसतािज ो क माधयम स इवतहास

और सावहतय क अधययन क पाोडवलवप विजञान

(Manuscriptology) कहत ह

bull पराचीन लखन परणावलय ो क अधययन और

ऐवतहावसक पाोडवलवपय ो क समझन तथा वतवथ

वनिािरण क पलीओगराफी (Palaeography) कहा

जाता ह

bull नयवमजमविकस (Numismatics) वसक ो क

अधययन क सोदवभित करता ह

Q5) उततर (a)

सपषटीकरण

- चरक सोवहता चरक क दवारा वलखी गई आयिद

और िदयक-शासर पर एक महतवपणि पसतक ह

- ि भारतीय िदयक-शासर की पारमपररक परणाली

वजस आयिद क नाम स जाना जाता ह क

अभयासकताि थ

- ऐसा माना जाता ह वक चरक का विकास दसरी

शताबदी (ईसा पिि) और दसरी शताबदी (ईसवी) क

मधय हआ था

Q6) उततर (b)

सपषटीकरण

- भाग फसल ो पर वलए जान िाल कर क सोदवभित

करता ह ज कल फसल उतपादन का 16 िाो भाग

था

- ldquoकममकारrdquo शबद भवमहीन कवि शरवमक िगि क

वलए परय ग वकया जाता था

- ldquoअशवमिrdquo (वजस घ ड क बवलदान क रप म भी

जाना जाता ह) एक अनषठान ह ता था वजसम एक

घ ड क सवतोतर रप स घमन क वलए छ ड वदया

FC19H1003 30

जाता ह और राजा क सवनक उसकी रखिाली

करत थ

Q7) उततर (d)

सपषटीकरण

- ऋगववदक काल म घ ड ो क रथ ो म ज ता जाता था

ज (रथ) भवम मिवशय ो आवद पर कबजा करन क

वलए लड गए यद ो म उपय ग वकए जात थ

- इसस यह पता चलता ह वक घ ड ो यकत रथ ो का

उपय ग महाजनपद काल स काफी पहल आरमभ

हआ था

- ऋगववदक काल म मिवशय ो भवम जल आवद पर

कबजा करन क वलए तथा ल ग ो क पकडन क

वलए यद वकय जात थ

- अविकाोश परि इन यद ो म भाग वलया करत थ

- हालाोवक उस समय क ई वनयवमत सना नही ो ह ती

थी लवकन उस काल म सभाऐो ह ती थी ो वजनम

ल ग यद क मामल ो पर चचाि करत थ

- वनयवमत सनाएा महाजनपद काल का िवशषटय थी

वजनम पदल सवनक ो की विशाल सनाएा रथ तथा

हाथी शावमल ह त थ

Q8) उततर (a)

सपषटीकरण

- बद शाकय कल स सोबोवित थ और कशीनारा म

उनका वनिन हआ था

- बद न अपनी वशकषाएा पराकत भािा म दी थी ो ज

आम ल ग ो की भािा थी

Q9) उततर (c)

सपषटीकरण

- पराचीन भारत म दशिनशासर की छह शाखाएा थी ो

िशविक नयाय समखया य ग पिि वममाोसा और

िदाोत या उततर वममाोसा

- इनकी थथापना करमश कनाद गौतम कवपल

पतोजवल जावमनी और वयास ऋविय ो न की थी

Q10) उततर (b)

सपषटीकरण

महािीर की वशकषाऐो छठी शताबदी म िललभी म

सोकवलत की गई थी ो

Q11) उततर (c)

सपषटीकरण

- पारमपररक रप स चाणकय क कौविलय अथिा

विषणगपत क नाम स जाना जाता ह

- उसन अथिशासतर ज एक पराचीन भारतीय

राजनवतक आलख ह वलखा था

Q12) उततर (d)

सपषटीकरण

- भारत का राषटर ीय वचनह सारनाथ (उततर परदश) क

अश क सतमभ क ऊपर (शीिि पर) वसोह कवपिल

का एक अनरपण ह

- इस राषटर ीय वसदाोत सतयमि जयत क साथ

सोय वजत वकया गया ह

- रामपिि बल का नाम रामपिि (वबहार) क नाम पर

पडा जहाा इसकी ख ज हई थी

- यह अपन नाजक नकाशी मॉडल क वलए परवसदद

ह वजसम क मल तवचा सोिदनशील नथन ो सतकि

कान और मरबत िााग ो क शरषठतर परवतरप क

परदवशित वकया गया ह

- यह भारतीय और फारसी ततव ो का एक ससममशरण

- सोवकससा उततर परदश म सथथत ह

Q13) उततर (a)

सपषटीकरण

का िर वसोह ज एक महान य दा थ वबहार स

सोबोवित थ

Q14) उततर (b)

सपषटीकरण

िललालर शबद बड भ-सवावमय ो क वलए परय ग

वकया जाता था

FC19H1003 31

Q15) उततर (c)

सपषटीकरण

- अररकमड एक तिीय बसती थी जहाो दर दश ो स

आन िाल जहाज ो का माल उतारा जाता था

- यहाो पर ईोि ो का एक विशाल ग दाम वमटटी क

बतिन (वजनम एमफ रा - द हरी मवठय ो का लोबा

घडा - शावमल ह) और एरिाइन (Arretine)

मदभाोड पाए गए थ

- इस थथान पर र मन दीपक काोच क बन पातर और

रतन भी पाए गए थ

Q16) उततर (a)

सपषटीकरण

- मिनदर सोगम कविताओो म उसललसखत एक

तवमल शबद ह वजसका अथि ह ldquoतीन परमखrdquo

- यह तीन सततारि पररिार ो क मसखयाओो क वलए

परय ग वकया जाता ह च ल चर और पाणडय

Q17) उततर (c)

सपषटीकरण

- ऋग िद म सभा विदाथा तथा गण जसी

जनजावतय ो पर अथिा किोब पर आिाररत

सभाओो का उललख ह

- आरसमभक िवदक काल म सभाओो और सवमवतय ो

का विशि महतव ह ता था

- यहाा तक की मसखया अथिा राजा भी उनका

समथिन परापत करन क वलए आतर रहत थ

Q18) उततर (a)

सपषटीकरण

- जन िमि न ईशवर क अससततव क मानयता त दी ह

वकनत उसन ईशवर क वजना क पद स नीच रखा

- जन िमि न बौद िमि की तरह िणि परणाली की

भरतिना नही ो की थी

Q19) उततर (d)

सपषटीकरण

- च ल ो और पाणडय ो न शसकतशाली तिीय शहर ो का

विकास वकया था

- च ल ो का सबस महतवपणि शहर पहार (या

कािरीपटटीनम) था |

- मदरई पाणडय ो की राजिानी थी

Q20) उततर (b)

सपषटीकरण

- ldquoबदचररतrdquo बद का जीिन-ितताोत ह

- इस अशवघ ि क दवारा वलखा गया था

Q21) उततर (a)

सपषटीकरणः

- तवमल कवि अपपर भगिान वशि क भकत थ

- इस परकार ि एक नयनार सोत थ

Q22) उततर (d)

सपषटीकरणः

- समदरगपत एक परवसद गपत शासक था

- उसन वसक ो पर िीणा बजात हए अपनी छवि

अोवकत करिाई थी

- यह सोगीत क परवत उसक परम क दशािता ह

- हम उसकी इलाहाबाद परशससत स महतवपणि

ऐवतहावसक जानकारी वमलती ह वजसकी रचना

उसक दरबार क कवि हररसन न की थी

Q23) उततर (b)

सपषटीकरणः

- विकरम सोित की शरआत ििि 58 ईसा पिि म

चनदरगपत वदवतीय न की थी

- यह शक ो पर उसकी जीत और उस विकरमावदतय

की पदिी वमलन क उपलकषय म आरमभ वकया गया

था

FC19H1003 32

- बानभटट न हिििििन का जीिन-ितताोत हििचररत

(ज सोसकत म थी) वलखी थी

Q24) उततर (c)

सपषटीकरणः

- सोवि-विगरावहका यद एिो शाोवत का मोतरी

- साथििाह वयापाररय ो क कावफल ो का नता

Q25) उततर (a)

सपषटीकरणः

- जआन झाोग (हसआन रताोग ndash Hsuang Tsang)

एक चीनी यातरी था ज हिििििन क शासनकाल म

भारत आया था

- ििि 630 ईसवी स ज दशक आरमभ हआ था उसम

जआन झाोग मधय एवशया ईरान और

अफग़ावनसतान की यातरा करन क पशचात कशमीर

क रासत स भारत आया था

- उसन उततर स पिि तक की यातरा की और िह

लगभग 2 ििि वबहार म रहा

- जआन झाोग न नालनदा विशवविदयालय म विदयावथिय ो

और विदवान ो क साथ पारसपररक विचार-विमशि

वकया थथानीय भािाओ ा म वनपणता परापत की तथा

बौद सतप ो की ख ज की

Q26) उततर (c)

सपषटीकरणः

- परदवकषणा पथ बौद िासतकला म सतप क चार ो

ओर बनाया जान िाला एक घमािदार पथ ह ता

- परशन म वदए गए बाकी क तीन ो ततव वहोद मसनदर ो की

िासतकला क भाग ह

Q27) उततर (d)

सपषटीकरणः

परशन म वदए गए सभी मोवदर ो म वयापक रप स

ईोि ो (पकी ईोि ो) का परय ग पतथर ो क साथ हआ

Q28) उततर (c)

सपषटीकरण

- महममद कली कतब शाह ग लकणडा का सलतान

था

- िह अकबर का समकालीन था

- सावहतय और िासतकला म उसकी अतयाविक

रवच थी

- िह एक महान कवि था

- िह दसखनी उदि फारसी और तलग म वलखता था

- उसन अपन पीछ एक विसतत वदिान (सोगरह)

छ डा ह

- अभी हाल ही म तलोगाना म ग लकणडा क वकल

क अनदर खदाई वकय गए बाग-ए-नाया वकला

बाग क चार ो ओर रप-रखा क मानवचतरण क

वलए भारतीय परातासतवक सिकषण (The

Archaeological Survey of India ndash ASI)

गराउणड पनीिर विोग रडार (Ground Penetrating

Radar) का परय ग करगा

Q29) उततर (a)

सपषटीकरणः

- वसलपपावदकारम एक तवमल महाकावय ह वजसकी

रचना इलाोग क दवारा लगभग 1800 ििि पिि की

गई थी

- यह क िलन नामक एक वयापारी की कहानी ह

ज माििी नामक एक गवणका (िशया) स परम

करन लगा था

- मवनमकलाई क िलन और माििी की पतरी की

कहानी ह

Q30) उततर (a)

सपषटीकरण

- चरक आयिद और वचवकरता की एक महतवपणि

रचना चरक सोवहता क लखक ह

- बरहमगपत क अपनी रचना बरहम-सफि-वसदानत

(ज एक खग लीय रचना ह) क कारण परवससद

वमली

FC19H1003 33

- बगदाद म इसका अनिाद अरबी भािा म वकया

गया था

- इसका इसलावमक गवणत और खग ल-विजञान पर

महतवपणि परभाि पडा था

- बाद म अपन जीिनकाल म बरहमगपत न

ldquoखोडखयाकrdquo वलखी ज एक खग लीय पससतका

(एक छ िी पसतक) थी

- इसम आयिभटट की अिि-रावतर क परतयक वदन की

शरआत परणाली का परय ग वकया गया था

Q31) उततर (c)

सपषटीकरण

- अमीर खसर एक परवसद सफी सोगीतकार कवि

और विदवान थ

- 1318 म उनह ोन पाया वक इस भवम (वहोदसतान) क

हर कषतर म अलग-अलग भािा थी लाहौरी

कशमीरी दवारसमदरी (दवकषणी कनाििक म)

तलोगाना (आोधर परदश म) गजरी (गजरात म)

माबारी (तवमलनाड म ) अििी (पिी उततर परदश

म) और वहोदिी (वदलली क आस-पास क कषतर म)

आवद

- उनह न यह बताया वक सोसकत वकसी भी कषतर स

सोबोवित नही ो थी और किल बराहमण ही इस भािा

का जञान रखत थ

Q32) उततर (c)

सपषटीकरण

- वहरणय-गभि सववणिम गभि क सोदवभित करता ह

- जब बराहमण ो की सहायता स यह अनषठान वकया

जाता था त यह माना जाता था वक बवल दन िाल

का कषवतरय क रप म पनजिनम ह गा

Q33) उततर (d)

सपषटीकरण

- कदमई भवम राजसव पर कर क सोदवभित करता

- गवावलयर परशससत म नागभि क दवारा वकय गए

श िण का िणिन वकया गया ह |

- नागभि एक परवतहार राजा था

Q34) उततर (b)

सपषटीकरण

- राजतरो वगनी 12िी ो शताबदी म कलहन क दवारा

रवचत एक सोसकत पसतक (िकसट) ह

- यह परारसमभक भारत की ऐवतहावसक इवतितत थी

- तकि सोगत रप स इस अपन परकार की सिोततम

और सिािविक विशवसनीय कवत माना जाता ह

- यह कशमीर कषतर क पराचीनतम समय स लकर

उसकी रचना की तारीख तक क समपणि इवतहास

का आचछादन करती ह

Q35) उततर (c)

सपषटीकरण

- गााि की आम सभा क ldquoउरrdquo कहा जाता था

- ldquoउरrdquo म गााि क सभी कर दन िाल वनिासी

शावमल ह त थ

Q36) उततर (a)

सपषटीकरण

- वदलली सलतनत म ldquoतारीखrdquo इवतहास लखन का

एक रप था

- ldquoतािरीखrdquo क लखक विदवान परि ह त थ वजनम

सवचि परशासक इतयावद शावमल थ

Q37) उततर (a)

सपषटीकरण

- अलाउददीन सखलजी अपन सवनक ो क ितन का

भगतान नकद म करता था न वक इकता क रप

- सवनक अपना सामान वदलली म वयापाररय ो स

खरीदत थ अतः इस बात का भय था वक वयापारी

कही ो िसतओो का मलय न बिा द

- इसकी र कथाम क वलए अलाउददीन सखलजी न

वदलली म कीमत ो क वनयसित वकया

FC19H1003 34

- अविकारीगण धयानपििक मलय ो का सिकषण करत

थ तथा ज वयापारी वनिािररत मलय पर माल नही ो

बचत थ उनक दसणडत वकया जाता था

Q38) उततर (d)

सपषटीकरण

- वदलली सििपरथम त मर राजपत ो क अिीन उनक

सामराजय की राजिानी बनी थी

- 12िी ो शताबदी क मधय म अजमर क चौहान ो

(वजनह चाहमान ो क नाम स भी जाना जाता ह) न

त मर राजपत ो क परावजत वकया था

- त मर ो और चौहान ो क अिीन वदलली एक

महतवपणि िावणसजयक क दर बन गया था

- कई जन वयापारी यहाा रहन लग थ और उनह ोन

कई मोवदर भी बनिाए

- यहाा पर मवदरत वसक वजनह ldquoदहलीिालrdquo क नाम

स जाना जाता था वयापक रप स परचलन म थ

Q39) उततर (c)

सपषटीकरण

- म ठ की मसिद का वनमािण वसको दर ल दी क

राजयकाल म उसक मिी क दवारा करिाया गया

था

- बगमपरी मसिद का वनमािण महममद तगलक क

शासनकाल म हआ था

- यह मसिद विशव का पणयथथान (The

Sanctuary of the World) और वदलली म महममद

तगलक की नई राजिानी जहाोपनाह की मखय

मसिद थी

- कववत- अल - इसलाम मसिद का विसतार

इलतसिश और अलाउददीन सखलजी न वकया था

- मीनार का वनमािण तीन सलतान ो कतबददीन ऐबक

इलतसिश और वफर ज शाह तगलक क दवारा

करिाया गया था

Q40) उततर (c)

सपषटीकरण

- मगल ो क अिीन मनसबदार शबद उस वयसकत क

वलए सोदवभित वकया जाता था वजसक पास मनसब

(अथाित पद) ह ता था

- उस अपना ितन राजसव कायो वजनह जागीर कहत

थ क रप म परापत ह ता था

Q41) उततर (b)

सपषटीकरण

- ldquoभारत छ ड आोद लनrdquo वबरविश शासन क

सखलाफ ल ग ो का एक सवाभाविक विदर ह था

- असखल भारतीय काोगरस सवमवत न 8 अगसत 1942

क बमबई म एक बठक का आय जन वकया था

- इस बठक म परवसद सोकलप ldquoभारत छ ड rdquo क

पाररत वकया गया और इस उददशय क परापत करन

क वलए गाोिी क नततव म एक अवहोसक जन सोघिि

आोद लन की शरआत का परसताि वदया गया

- लवकन अगल ही वदन गाोिी और काोगरस क अनय

परमख नताओो क वगरफतार कर वलया गया

- काोगरस क एक बार वफर अिि घ वित वकया गया

था

Q42) उततर (c)

सपषटीकरण

- साइमन कमीशन यनाइविड वको गडम क सात

साोसद ो का एक समह था

- इस वबरविश भारत क वलए सोििावनक सिार ो का

सझाि दन क वलए गवठत वकया गया था

- इस आय ग म िररषठ वबरविश राजनता सर जॉन

साइमन क नततव म किल वबरविश सदसय ही

शावमल थ

- इसवलए भारत क ल ग ो न साइमन कमीशन क

आगमन क विरद आोद लन वकया था

Q43) उततर (a)

सपषटीकरण

bull दादा भाई नौर जी भारत म वबरविश शासन क

आवथिक पररणाम ो क बार म अपनी विर िी

(परवतकल) राय क वलए जान जात थ

FC19H1003 35

bull अपन कई लख ो और भािण ो म विशि रप स

ldquoपाििी एो ड अन-वबरविश रल इन इसणडया

(Poverty and Un-British Rule in India) म

नौर जी न यह तकि वदया वक भारत पर अतयविक

कर लगाया गया था और इसकी सोपवतत इोगलड की

ओर परिावहत की जा रही थी

bull उनह ोन पराचीन भारतीय गरोथ ो की वयाखया करन

का और भारतीय ो क आिविशवास क बहाल

करन पर कायि नही ो वकया था

उनह ोन वकसी और बात स पहल सभी सामावजक

बराइय ो क उनमलन की आिशयकता पर भी बल

नही ो वदया था

Q44) उततर (c)

सपषटीकरण

bull अगसत 1932 म वबरविश परिानमोतरी मकड नालड न

अपन साोपरदावयक परसकार (The Communal

Award) की घ िणा की थी

bull यह भारत क कई साोपरदावयक वहत ो क बीच विवभनन

सोघिो क हल करन क वलए वबरिन का एकतरफा

परयास था

bull यह परसकार (Award) बाद म 1935 क

अविवनयम (The Act of 1935) म शावमल वकया

गया था

bull इस साोपरदावयक परसकार न मससलम ो क वलए

आरवकषत एक अलग वनिािचक मणडल फॉमिल का

विसतार अनय अलपसोखयक ो क वलए वकया था

वजसम वसख ो भारतीय ईसाइय ो आोगल-भारतीय

समदाय यर पीय समदाय तथा विवशषट कषतरीय

समह ो क शावमल वकया गया था

bull गाोिी न इस परसताि क भारतीय समाज क

विभावजत करन क वलए एक घवणत वबरविश

सावजश क रप म दखा और उसक सखलाफ

आमरण अनशन वकया

Q45) उततर (b)

सपषटीकरण

मौजदा आयात और वनयाित क अवतररक़त

औपवनिवशक भारत क वनमनवलसखत खचो क

वलए एक विशिवनवशचत िन रावश भी दनी पडती

थी

(i) परशासन क वयय

(ii) सना क रख-रखाि क वयय

(iii) यद क वयय

(iv) सिावनितत अविकाररय ो की पशन तथा

(v) वबरिन दवारा अपनी उपवनिश बसती

(कॉल नी) क रख-रखाि क वयय

इनह गह शलक (Home Charges) क रप म

जाना जाता था और लगभग परी तरह स भारत क

दवारा इनका भगतान वकया जाता था

bull गह शलक म वनमनवलसखत घिक शावमल थ

(i) भारतीय ऋण पर दय बयाज

(ii) ईसट इोवडया को पनी क शयरिारक ो क

लाभाोश

(iii) लोदन म भारत कायािलय चलान क वलए िन

(iv) भारत म वनयकत वबरविश कवमिय ो क ितन

और पशन का भगतान करन क वलए िन

(v) रलि पर बयाज

(vi) नागररक और सनय शलक

(vii) इोगलड म सट र (सामगरी) की खरीद

Q46) उततर (b)

सपषटीकरण

bull भारतीय राषटर ीय काोगरस का लाहौर सतर 1929 म

जिाहरलाल नहर की अधयकषता म आय वजत

वकया गया था

bull इस सतर म भारतीय राषटर ीय आोद लन स समबसित

कई महतवपणि पररणाम सामन आय थ

(i) सििपरथम इस सतर म काोगरस क अधयकष पद

पर जिाहरलाल नहर क चना गया था ज

काोगरस म िामपोवथय ो की बिती हई ताकत

का सपषट सोकत था

(ii) दसरा इस सतर म पहली बार काोगरस न पणि

सवतोतरता की माोग क उठाया था

इस परकार की माोग काोगरस मोच स पहल कभी भी

नही ो उठाई गई थी

Q47) उततर (b)

सपषटीकरण

FC19H1003 36

bull इस ररप िि न वकसी भी समदाय क वलए पथक

वनिािचक मोडल अथिा अलपसोखयक ो क वलए

भाराोश की वसफाररश नही ो की थी

bull तथावप इस ररप िि न उन पराोत ो म अलपसोखयक

सीि ो क आरकषण की अनमवत दी थी जहाा पर कम

स कम दस परवतशत अलपसोखयक ह

bull लवकन यह समदाय क आकार क अनपात म ह ना

चावहए था

bull इस ररप िि म भारत क वलए पणि सवतोतरता क

वलए क ई पराििान नही ो था

Q48) उततर (c)

सपषटीकरण

bull आरो वभक िवदक आयो का िमि मखय रप स

परकवत की पजा और यजञ था

bull परारो वभक आयि िमि परकवत की पजा क समान था

bull िासति म उनक चार ो ओर की शसकतयाा वजनह न

त ि वनयोवतरत कर सकत थ और न ही समझ पाए

थ उनह वदवयता क साथ वनिवशत वकया गया तथा

उनह मादा या नर दिीदिताओो क रप म

परतीकतव वकया गया था

bull उनह ोन कछ यजञ ो का भी वनषपादन वकया था

Q49) उततर (b)

सपषटीकरण

bull सडक और नदी-मागि (जल-मागि) डकती स

सरवकषत नही ो थ

bull उललखनीय ह वक हिििििन क शासनकाल क

दौरान यआन चिाोग (हयएन साोग) का सारा

सामान लि वलया गया था

Q50) उततर (c)

सपषटीकरण

परशन म वदए गए द न ो कथन सही ह

Q51) उततर (b)

सपषटीकरण

bull परोदर दास एक सोत और भगिान कषण क एक

महान भकत थ

bull परोदर दास क कनाििक सोगीत क वपतामह क

रप म जाना जाता ह

bull यदयवप उनक जनम-थथान क बार म काफी

अिकल लगाई जाती रही ह

bull तथावप अब कननड विशवविदयालय हमपी क दवारा

गवठत एक विशिजञ सवमवत इस वनषकिि पर पहोची

ह वक उनका जनम थथान सोभितया कनाििक का

एक छ िा-सा गााि कषमपरा (वशिम गगा वजला)

था

Q52) उततर (c)

सपषटीकरण

bull शरी तयागराज शरी शयाम शासतरी और शरी मथसवामी

दीवकषतर क कनाििक सोगीत की वतरमवति माना

जाता ह

bull उनक कारण ही 18िी ो-19िी ो शताबदी म कनाििक

सोगीत का सववणिम यग आया था

Q53) उततर (d)

सपषटीकरण

bull अभी हाल ही म लौह यगीन-महापािावणक काल

का 2000 ििि पराना एक दलिभ सारक फगस

(Sarcophagus) (पतथर का ताबत) क ललम क

वियर गाोि (क वयलडी क पास वजला क वझक ड

करल राजय) की एक रॉक-कि गफा स ख जा गया

bull यह ताबत वजसम हविय ो क िकड थ खदाई क

दौरान वमला

bull अभी तक इस परकार की दलिभ ख ज करल क

मातर द ही थथान ो स हई ह

bull य द न ो सारक फगी (Sarcophagi) (पतथर क

ताबत) चियर और अथ ली (वजला क वझक ड) क

महापािाण थथल ो स वमल ह

Q54) उततर (a)

सपषटीकरण

FC19H1003 37

दवकषण भारत म महापािाण सोसकवत एक पणि

विकवसत लौह यगीन सोसकवत थी

Q55) उततर (d)

सपषटीकरण

bull च ल पाणडय और करलपतर (चर) इन तीन ो का

उललख अश क क अवभलख ो म वकया गया ह

bull सोभितः य भौवतक सोसकवत क उततर

महापािावणक चरण म थ

Q56) उततर (d)

सपषटीकरण

bull भीमा-क रगाोि की लडाई ततीय आोगल-मराठा

यद का वहससा थी

Q57) उततर (b)

सपषटीकरण

bull राजकमार शकल न गाोिीजी क चोपारण आन तथा

वतनकवथया परणाली स जडी समसया की जाोच क

वलए रारी करन क वलए दश भर म उनका

अनसरण वकया था

bull बज वकश र राजदर परसाद महादि दसाई और

नरहरी पाररख चोपारण सतयागरह क दौरान गाोिी

जी क सहय गी थ

Q58) उततर (b)

सपषटीकरण

bull बराहमण ो और बौद मठिाररय ो क कर-मकत गााि

अनदान म दन की परथा सतिाहन ो न आरमभ की

थी

Q59) उततर (c)

सपषटीकरण

इस कायिकरम क उददशय वनमनानसार ह

(i) बवनयादी पयििन आिाररक सोरचना का विकास

करना

(ii) चयवनत (पहचान वकय गए) कषतर ो म आजीविका क

सजन क वलए दश क साोसकवतक और विरासत

मलय ो क बिािा दना

(iii) विरासत समारक थथल ो पर विशव सतरीय आिाररक

सोरचना विकवसत करक एक सतत तरीक स

पयििक आकििण म िसद करना

(iv) थथानीय समदाय ो की सवकरय भागीदारी क माधयम

स र रगार ो का सजन करना

(v) र रगार उतपादन और आवथिक विकास क वलए

पयििन कषमता का उन पर परभाि का उपय ग

करना तथा

(vi) िारणीय पयििन आिाररक सोरचना का विकास

करना और उसका उवचत सोचालन तथा

रखरखाि सवनवशचत करना

Q60) उततर (b)

सपषटीकरण

bull यह वनकाय ििि 1987 म अससततव म आया था

bull यह एक राषटर ीय सतर का शीिि सोगठन ह ज भारत

सरकार क जनजातीय मामल ो क मोतरालय क

परशासवनक वनयोतरण क अिीन काम कर रहा ह

bull इसका पोजीकत और परिान कायािलय नई वदलली

म सथथत ह

Q61) उततर (c)

सपषटीकरण

bull परमचोद क उपनयास ो म परमाशरम रोगभवम गबन

कमिभवम और ग दान शावमल ह

bull ग रा रिी ोदरनाथ िग र क दवारा रवचत उपनयास ह

bull अभी हाल ही म मोशी परमचोद की 138िी ो जयोती दश

भर म मनाई गई थी

Q62) उततर (b)

सपषटीकरण

bull ldquoवगदाrdquo पोजाब (भारत) एिो पावकसतान की

मवहलाओो क दवारा तयौहार क समय और फसल

की बिाई तथा किाई क अिसर पर वकया जान

िाला एक पारोपररक दहाती नतय ह

FC19H1003 38

bull इस नतय क माधयम स पोजाबी मवहलाऐो अपनी

परसननता परकि करती ह तथा वगदा क परदशिन क

माधयम स परि िचिसव िाल समाज म मवहलाओो

की दबी हई भािनाओो क परकि करती ह

bull चोवक इस नतय का परि ो क साथ क ई सोबोि नही ो

ह अतः किल मवहलाऐो ही इसम भाग ल सकती

bull हर साल तीज समार ह क दौरान पोजाब म वगदा

नतय वकया जाता ह

तीज भारत क कछ भाग ो म मवहलाओो क दवारा

मनाया जान िाल कई तयौहार ो क वलए एक

वयापक नाम ह

Q63) उततर (a)

सपषटीकरण

- मजम-उल-बहरीन या द समदर ो का सोगम

नामक उललखनीय रचना दारा वशक ह क दवारा

वलखी थी

- भारत क उपराषटर पवत शरी एम िकया नायड न कहा

ह वक राजकमार दारा वशक ह की रचनाएा शाोवत

और सदभाि क बिािा दन क वलए एक तारा सर त

क रप म सामन आ सकती ो ह

- उपराषटर पवत गत ििो क भला वदए गए राजकमार

दारा वशक ह क परदवशित परचवलत करन हत

आय वजत एक परदशिनी का दौरा करन क बाद एक

सभा क सोब वित कर रह थ

- इस परदशिनी का आय जन फर क इस गौवियर

(Francois Gautier) क दवारा lsquoइोवदरा गाोिी नशनल

सिर फॉर द आििसrsquo (The Indira Gandhi

National Centre for the Arts) नई वदलली म

वकया गया था

Q64) उततर (c)

सपषटीकरण

- ग मतशवर परवतमा जन भगिान बाहबली क

समवपित ह

- यह एक एक-चटटानी पतथर की मवति ह

- राषटर पवत राम नाथ क विोद न शरिणबलग ला

(कनाििक) म आय वजत वकय जान िाल भवय

अवभिक समार ह महामसतकावभिक का

उदघािन वकया था

- यह समार ह 12 ििो म एक बार ह ता ह

Q65) उततर (c)

सपषटीकरण

bull पराची घािी पराची नदी क चार ो ओर फली हई थी

bull पराची घािी िीर-िीर विलपत ह गई थी

bull पराची नदी भिनशवर स वनकलती ह

bull यह महानदी की एक सहायक नदी ह और यह

परी खदाि किक तथा जगतवसोहपर वजल ो स

ह कर बहती ह

bull इस नदी क पर कषतर क पराची घािी कहा जाता ह

bull यह नदी बोगाल की खाडी म वगरती ह

परातासतवक साकषय स पता चलता ह वक पराची घािी

सभयता हडपपा और म हनज दाड द न ो की

पिििती ह

Q66) उततर (d)

सपषटीकरण

य समारक छतरपर वजल (मधय परदश) म विोधयाचल

पिित शरोखला म सथथत ह

Q67) उततर (a)

सपषटीकरण

bull थॉिस ऑन पावकसतान नामक पसतक डॉ बी

आर अमबडकर न वलखी थी

bull डॉ बी आर अमबडकर की जयोती क अिसर पर

भारत क राषटर पवत न भारत की इस महान हसती

क शरदाोजवल अवपित की थी

bull डॉ बी आर अमबडकर न 1924 म वडपरथड

कलावसर इोसटीटयि (दवलत िगि सोथथान -

बवहषकत वहतकाररणी सभा) और 1927 म समाज

समता सोघ की थथापना की थी

bull अमबडकर का धयान वशकषा कषतर की ओर भी था

bull उनह ोन वशकषा क वनमन िगो म फलान क वलए

पीपलस एजकशन स साइिी (The Peoples

Education Society) क नाम स महाविदयालय ो क

नििकि और छातरािास ो की थथापना की थी

FC19H1003 39

Q68) उततर (b)

सपषटीकरण

bull महरगि भारतीय उपमहादवीप म एक परवसद

निपािाण बसती ह ज बलवचसतान पराोत

पावकसतान म सथथत ह

bull दचपलली (आोधर परदश) क पास नागलर नदी क

पिी ति ो पर चना पतथर क बलॉक क विशाल

विसतार म एक पिि-ऐवतहावसक रॉक आिि थथल की

ख ज की गई ह

bull इसन 1500-2000 ईसा पिि क दौरान गोिर (आोधर

परदश) म विकवसत निपािाण सभयता पर परकाश

डाला ह

Q69) उततर (c)

सपषटीकरण

bull 12िी ो सदी और 13िी ो सदी म काकाविय िोश का

उदय हआ था

bull ि पहल कलयाण क पवशचमी चालकय ो क सामोत थ

bull परारोभ म उनह ोन िारोगल (तलोगाना) क पास एक

छ ि स कषतर पर शासन वकया था

bull उनह ोन ldquoनायक वयिथथाrdquo की शरआत की थी

वजस बाद म विजयनगर क राय शासक ो न

अपनाया और विकवसत वकया था

Q70) उततर (a)

सपषटीकरण

bull गाोिीजी क अनशन स वमल मावलक ो पर दबाि

पडा था ज अोततः शरवमक ो क ितन म 35 परवतशत

की िसद करन क वलए सहमत हए थ

bull गगल (Google) न अनसया साराभाई वजनह ोन

भारत क शरवमक आोद लन म एक अगरणी भवमका

वनभाई थी की 132िी ो जयोती डडल (Doodle) का

वनमािण करक मनाई

Q71) उततर (d)

सपषटीकरण

भारत स यनसक की मानिता की अमति साोसकवतक

विरासत की परवतवनवि सची म वनमनवलसखत शावमल ह

bull कवडयटटम करल का सोसकत रोगमोच

bull मवडयिि करल का अनषठान रोगमोच और नतय

नाविका

bull िवदक मि जाप की परोपरा

bull राजथथान क कालबवलया ल क गीत और नतय

bull रामलीला रामायण का पारोपररक परदशिन

bull सोकीतिन मवणपर का अनषठान गायन ढ ल िादन

और नतय

bull रममन भारत क गििाल वहमालय का िावमिक

तयौहार और अनषठान रोगमोच

bull जाोदीयाला गर पोजाब क ठठर ो की पीतल और

ताोब क वशलप स वनवमित बतिन ो की पारोपररक कला

bull छाऊ नतय पिी भारतीय राजय ो म जनमी शासतरीय

भारतीय नतय कला

bull लददाख का बौद मि जाप िर ाोस-वहमालयी लददाख

कषतर तथा जमम-कशमीर म पवितर बौद गरोथ ो का पाठ

bull य ग

bull नौर र

bull को भ मला

Q72) उततर (b)

सपषटीकरण

bull भारत क राषटर पवत शरी राम नाथ क विोद न

वकसामा नागालड म हॉनिवबल मह रति और

राजय गठन वदिस समार ह का उदघािन वकया

था

bull हॉनिवबल मह रति का नाम भारतीय हॉनिवबल क

नाम पर पडा ह ज एक विशाल और रोगीन जोगली

पकषी ह

bull यह पकषी नागालड राजय की अविकतर जनजावतय ो

की ल ककथाओो म उसललसखत ह

bull नागालड की परमख मानयता परापत जनजावतयाा ह

अोगामी आओ चखसोग चाोग ककी रगमा और

रवलोग आवद

bull ओोग जारिा और ससिनलीस अोडमान-वनक बार

दवीप समह की जनजावतयाा ह

FC19H1003 40

Q73) उततर (c)

सपषटीकरण

bull दकन म राषटर कि शासन दसिी ो सदी क अोत तक

लगभग 200 ििो तक रहा था

bull राषटर कि शासक अपन िावमिक विचार ो म सवहषण

bull उनह ोन न किल शि िमि और िषणि िमि बसलक

जन िमि क भी सोरकषण वदया था

bull एल रा म वशि क परवसद रॉक कि मोवदर का

वनमािण नौिी ो सदी म राषटर कि राजा कषण परथम न

करिाया था

bull उसका उततराविकारी अम घििि जन था लवकन

उसन अनय िमो क भी सोरकषण परदान वकया था

bull राषटर कि ो न मसलमान वयापाररय ो क बसन की

अनमवत दी थी

bull उनह न अपन अविराजय ो म इसलाम क उपदश दन

की भी अनमवत दी थी

bull अभी हाल ही म पाोडिलागटटा (तलोगाना) क

परागवतहावसक चटटान वचतर ो क कषरण की बिती हई

घिनाएा एक गोभीर वचोता का वििय ह

bull यह परागवतहावसक चटटान क नकसान पहाचा

सकता ह

bull पाोडिलागटटा वनमनवलसखत क वलए जाना जाता ह

- 10000 ईसा पिि स 8000 ईसा पिि क वचवतरत

चटटानी आशरय ो क वलए

- राषटर कि काल क एक 8 िी ो सदी क

वशलालख क वलए और

- 12िी ो सदी क काकविय सामराजय क वभवतत

वचतर ो क वलए

Q74) उततर (b)

सपषटीकरण

bull 1828 म राजा राम म हन रॉय न एक नय िावमिक

समाज बरहम सभा की थथापना की थी वजस बाद

म बरहम समाज क नाम स जाना गया था

bull दिदरनाथ िग र न ततवब विनी सभा की अधयकषता

की थी ज आधयासिक सतय की ख ज म सोलि

थी

bull इसका उददशय वहोद िमि क शद करन का और

एकशवरिाद (एक ईशवर म आथथा) का परचार करना

था

bull नय समाज की थथापना क आिार थ कारण

(तकि ) क द सतमभ तथा िद और उपवनिद

bull अभी हाल ही म सािारण बरहम समाज का कछ

काननी मदद ो क लकर पवशचम बोगाल सरकार क

साथ काननी वििाद चल रहा ह

Q75) उततर (c)

सपषटीकरण

bull भारत म वचशती वसलवसल की थथापना खवाजा

म इनददीन वचशती क दवारा की गयी थी

bull ि 1192 ईसवी क आसपास भारत आय थ

bull वचशतीय ो क बारहिी ो शताबदी क उततरािि म भारत

म आन िाल सफीय ो क समह ो म सबस

परभािशाली माना जाता ह

bull उनह ोन थथानीय िातािरण क साथ सफलतापििक

अनकलन वकया और उनह ोन भारतीय भसकत

परोपराओो क कई पहलओो क अपनाया

bull अजमर म सफी अपरकि खवाजा म इनददीन वचशती

की ऐवतहावसक दरगाह क एक नया रप दन की

तयारी की जा रही ह

bull इस 13िी ो शताबदी की दरगाह क ldquoसवचछ

आइकॉवनक थथल ोrdquo (Swacch Iconic Places) म

शावमल वकया गया ह ज परवतवषठत विरासत

आधयासिक और साोसकवतक थथान ो पर क वदरत

य जना ह

FC19H1003 41

ANSWERS amp EXPLANATION OF

NCERT History Class VI-X + Current Affairs

(FC19E1003)

Q1) Answer c

Explanation

Rigveda consists of more than a

thousand hymns dedicated to gods and

goddesses These hymns were

composed by sages and learnt by men

however a few were composed by

women like Apala Ghosa Lopamudra

Maitreyi and Gargi

Rigveda consists of many hymns in the

form of dialogues We get an example of

a dialogue between a sage named

Vishwamitra and two rivers (Beas and

Sutlej) that were worshipped as

goddesses This suggests that he

belonged to the Vedic period

Q2) Answer b

Explanation

Traces of ash have been found from

Kurnool Caves suggesting that people

were familiar with the use of fire

It is situated in Andhra Pradesh

Q3) Answer c

Explanation

Burzahom is a prehistoric site in

present day Kashmir where people built

pit houses which were dug into the

ground with steps leading into them

These may have provided shelter in cold

weather

Q4) Answer c

Explanation

Epigraphy is defined as the study of

inscriptions

Manuscriptology is the study of history

and literature through the use of hand

written documents

Palaeography refers to the study of

ancient writing systems and the

deciphering and dating of historical

manuscripts

Numismatics refers to the study of

coins

Q5) Answer a

Explanation

Charaka Samhita was written by

Charaka and is an important book on

Ayurveda and medicine

He was a practitioner of the traditional

system of Indian medicine known as

Ayurveda

Charaka is thought to have flourished

sometime between the 2nd century BCE

and the 2nd century CE

Q6) Answer b

Explanation

Bhaga refers to the tax on crops which

was fixed at 16th of the production

Kammakaras is the term used for the

landless agricultural labour class

Ashvamedha also known as horse

sacrifice is a ritual where a horse is let

loose to wander freely and it was

guarded by the rajarsquos men

Q7) Answer (d)

Explanation

In the Rigvedic period horses were

yoked to chariots that were used in

battles fought to capture land cattle

etc This suggests that the use of horse

chariots began much before the period

of Mahajanapadas

The battles were fought in the Rigvedic

period for cattlersquos lands water an even

to capture people Most men took part

in these wars however there was no

regular army but there were assemblies

where people met and discussed

matters of war Regular armies became

a feature in the Mjahajanapada period

including vast armies of foot soldiers

chariots and elephants

RAUSIAS-FC19E1003 42

Q8) Answer (a)

Explanation

Buddha belonged to the Sakya clan and

passed away at Kusinara

Buddha taught in Prakrit which was the

common language of people

Q9) Answer c

Explanation

There were six schools of philosophy in

ancient India These are known as

Vaishesika Nyaya Samkhya Yoga

Purva Mimansa and Vedanata or Uttara

Mimansa They were founded by sages

Kanada Gautama Kapila Patanjali

Jamini and Vyasa respectively

Q10) Answer b

Explanation

The teachings of Mahavira were

compiled at Valabhi in 6th century AD

Q11) Answer (c)

Explanation

Chanakya is traditionally identified as

Kautilya or Vishnugupta who authored

the ancient Indian political treatise the

Arthashastra

Q12) Answer d

The national emblem of India is an

adaptation of the Lion Capital atop the

Ashoka Pillar of Sarnath Uttar Pradesh

and is combined with the National

Motto Satyameva Jayate

The Rampurva Bull gets the name from

the site of its discovery Rampurva in

Bihar

It is noted for its delicately sculpted

model demonstrating superior

representation of soft flesh sensitive

nostrils alert ears and strong legs It is

a mixture of Indian and Persian

elements

Sankissa is situated in Uttar Pradesh

India

Q13) Ans(a)

Kunwar Singh was a notable leader during the Revolt of 1857 He belonged

to a royal house of Jagdispur Bihar

Q14) Answer b

Explanation

The term Vellalar was used for large

landowners

Q15) Answer c

Explanation

Arikamedu was a coastal settlement

where ships unloaded goods from

distant lands Finds here include a

massive brick warehouse pottery

including amphorae and Arretine ware

Roman lamps glassware and gems have

also been found at the site

Q16) Answer a

Explanation

Muvendar is a Tamil word mentioned in

Sangam poems meaning three chiefs

used for the heads of three ruling

families the Cholas Cheras and

Pandyas

Q17) Ans (c)

Several tribal or kin-based assemblies

such as the Sabha Vidatha and gana

are mentioned in the Rig-veda The

Sabha and the samiti mattered a great

deal in early Vedic times so much so

that the chiefs or the kings showed an

eagerness to win their support

Q18) Ans (a)

Jainism recognised the existence of the

gods but placed them lower than the

jina and did not condemn the varna

system as Buddhism did

Q19) Answer (d)

Explanation

Cholas and Pandyas had developed

powerful coastal cities The most

important city of Cholas was Puhar or

Kaveripattinam and Madurai was the

capital of Pandyas

Q20) Answer b

Explanation

Buddhacharita is the biography of

Buddha and was written by

RAUSIAS-FC19E1003 43

Ashvaghosha

Q21) Answer (a)

Explanation

Tamil poet Appar was a Shiva devotee

So he was a Nayanar saint

Q22) Answer d

Explanation

Samudragupta was a prominent Gupta

ruler whose coins depict him playing a

veena indicating his love for music We

get important historic information from

his Allahabad Prashasti which was

composed by his court poet Harisena

Q23) Answer (b)

Explanation

Vikrama Samvat was founded by

Chandragupta II in the 58 BC as a

mark of victory over the Shakas and

assumed the title of Vikramaditya

Banabhatta wrote Harshavardhanarsquos

biography the Harshacharita in

Sanskrit

Q24) Answer c

Explanation

Sandhi-vigrahika was the minister of

war and peace

Sarthavaha was the leader of the

merchant caravans

Q25) Answer a

Explanation

Xuan Zang (Hsuan-tsang) was a

Chinese traveller who came during the

reign of Harshavardhana

In the decade that began in 630 AD

Xuan Zang came to India through

Kashmir after visiting Central Asia Iran

and Afghanistan

He travelled from north to east and lived

in Bihar for a couple of years

At Nalanda University Xuan Zang

interacted with students and scholars

mastered local languages and

discovered Buddhist stupas

Q26) Answer c

Explanation

Pradakshina patha is a circular path

laid around a stupa in Buddhist

architecture While the rest are a part of

temple architecture

Q27) Answer d

Explanation

All the above-mentioned temples have

an elaborate use of bricks (baked

bricks) along with stone

Q28) Ans (c)

Muhammad Quli Qutab was the Sultan

of Golconda He was a contemporary of

Akbar was very fond of literature and

architecture

The Sultan was a great poet and he

wrote in Dakhini Urdu Persian and

Telgu and has left an extensive diwan or

collection

Recently the Archaeological Survey of

India (ASI) will be using Ground

Penetrating Radar (GPR) to map the

contours of the area around the Bagh-e-

Naya Qila excavated garden inside the

Golconda Fort in Telangana

Q29) Answer a

Explanation

Silappadikaram is a famous Tamil epic

which was written by Ilango around

1800 years ago It is a story of a

merchant named Kovalan who fell in

love with a courtesan named Madhavi

Manimekalai tells the story of the

daughter of Kovalan and Madhavi

Q30) Answer (a)

Explanation

Charaka is the author of Charaka

Samhita which is an important work of

Ayurveda and medicines

Brahmaguptarsquos fame rests mostly on his

Brahma-sphuta-siddhanta which was

an astronomical work It was translated

into Arabic in Baghdad and had a major

impact on Islamic mathematics and

astronomy

Late in his life Brahmagupta wrote

Khandakhadyaka which was an

RAUSIAS-FC19E1003 44

astronomical handbook that employed

Aryabhatarsquos system of starting each day

at midnight

Q31) Answer (c)

Explanation

Amir Khusrau was a famous sufi

musician poet and scholar In 1318 he

noted that there was different language

in every region of this land (Hindustan)

Lahori Kashmiri Dvarsamudri (in

Southern Karnataka) Telangana (in

Andhra Pradesh) Gujari (in Gujarat)

Marsquobari (in Tamil Nadu) Awadhi (in

eastern Uttar Pradesh) and Hindawai (in

the area around in Delhi) etc He went

to explain that Sanskrit did not belong

to any region and that only brahmans

knew it

Q32) Answer c

Explanation

Hiranyagarbha refers to the golden

womb When this ritual was performed

with the help of Brahmanas it was

thought to lead to the rebirth of the

sacrificer as a Khastriya

Q33) Answer d

Explanation

Kadamai refers to a tax on land

revenue

Gwalior Prashasti describes the exploits

of Nagabhata who was a Pratihara king

Q34) Answer b

Explanation

Rajatarangini is a Sanskrit text written

by Kalhana in the 12th century

It was historical chronicle of early India

It is justifiably considered to be the best

and most authentic work of its kind

It covers the entire span of history in

the Kashmir region from the earliest

times to the date of its composition

Q35) Answer c

Explanation

ldquoUrrdquo was the general assembly of the

village ldquoUrrdquo consisted of all the

taxpaying residents of an ordinary

village

Q36) Answer (a)

Explanation

Tarikh was a form of history writing in

the Delhi Sultanate The authors of

tawarikhs were learned men which

included secretaries administrators etc

Q37 Answer (a)

Explanation

Alauddin chose to pay his soldiers salaries in cash rather than iqtas The soldiers would buy their supplies from merchants in Delhi and it was thus feared that merchants would raise their prices To stop this Alauddin controlled the prices of goods in Delhi Prices were carefully surveyed by officers and merchants who did not sell at the prescribed rates were punished

Q38) Answer (d)

Explanation

Delhi first became the capital of a

kingdom under the Tomara Rajputs

who were defeated in the middle of the

twelfth century by the Chauhans (also

referred to as Chahamanas) of Ajmer

It was under the Tomaras and

Chauhans that Delhi became an

important commercial centre Many rich

Jaina merchants lived in the city and

constructed several temples Coins

minted here called dehliwal had a wide

circulation

Q39) Answer (c)

Explanation

Moth ki Masjid was built in the reign of

Sikandar Lodi by his minister

Begumpuri mosque built in the reign of

Muhammad Tughluq was the main

mosque of Jahanpanah the ldquoSanctuary

of the Worldrdquo and his new capital in

Delhi

Quwwat al ndash Islam mosque was

enlarged by Iltutmish and Alauddin

Khalji The minar was built by three

Sultansndash Qutbuddin Aybak Iltutmish

and Firuz Shah Tughluq

RAUSIAS-FC19E1003 45

Q40) Answer (c)

Explanation

Under the Mughals mansabdar was

referred to an individual who held a

mansab ie rank and he received his

salary as revenue assignments called

jagirs

Q41) Ans (b)

The Quit India Movement was a

spontaneous revolt of people against

British rule

The All India Congress Committee met

at Bombay on 8 August 1942 It passed

the famous resolution Quit India and

proposed the starting of a non-violent

mass struggle under Gandhis

leadership to achieve this aim But on

the very next day Gandhi and other

eminent leaders of the Congress were

arrested The Congress was once again

declared illegal

Q42) Ans (c)

The Simon Commission refers to a

group of seven MPs from the United

Kingdom constituted to suggest

constitutional reforms for British India

The Commission consisted of only

British members headed by one of the

senior British politicians Sir John

Simon

So the people of India agitated against

the arrival of Simon Commission

Q43) Ans (a)

He was widely known for his

unfavourable opinion of the economic

consequences of the British rule in

India

In his many writings and speeches and

especially in Poverty and Un-British

Rule in India Naoroji argued that India

was too highly taxed and that its wealth

was being drained away to England

He did not interpret the ancient Indian

texts and restored the self-confidence of

Indians And also he did not stress the

need for eradication of all the social

evils before anything else

Q44) Ans (c)

In August 1932 Prime Minister

MacDonald announced his Communal

Award Great Britainrsquos unilateral

attempt to resolve the various conflicts

among Indiarsquos many communal

interests

The award which was later

incorporated into the act of 1935

expanded the separate-electorate

formula reserved for Muslims to other

minorities including Sikhs Indian

Christians Anglo-Indians Europeans

distinct regional groups Gandhi

undertook a ldquofast unto deathrdquo against

that offer which he viewed as a

nefarious British plot to divide the

Indian society

Q45) Ans (b)

In British India apart from existing

imports and exports there was also a

particular amount of money which

colonial India contributed towards

administration maintenance of the

army war expenses pensions to retired

officers and other expenses accrued by

Britain towards maintenance of her

colony These were known as Home

charges and were paid for almost

entirely by India

The Home charges was made of

following components-

- Interest payable on Indian debt

- Dividend to shareholders of East

India Company

- Funds used to support the India

Office in London

- Funds used to pay salaries and

pensions of British personnel

engaged in India

- Interest on the railways

- Civil and military charges

- Store purchases in England

Q46) Ans (b)

The Lahore session of the Indian

National Congress was held in 1929

under the Presidentship of Jawaharlal

Nehru

The Lahore session of the Indian

National Congress witnessed significant

RAUSIAS-FC19E1003 46

developments in the Indian national

movement

- First the election of Jawaharlal

Nehru to the post of Presidentship of

the Congress was a clear indication

of the growing strength of the

Leftists in the Congress

- Secondly it was in this session that

the Congress for the first time raised

the demand for complete

independence Such demand was

not raised from the Congress

platform earlier

Q47) Ans (b)

It did not provide for separate

electorates for any community or

weightage for minorities However it did

allow for the reservation of minority

seats in provinces having minorities of

at least ten per cent but this was to be

in strict proportion to the size of the

community

There was no provision for complete

Independence for India

Q48) Ans (c)

The religion of early Vedic Aryans was

primarily of worship of nature and

Yajnas

The early Aryan religion was kind of

nature worship Actually the forces

around them which they could not

control or understand were invested

with divinity and were personified as

male or female gods And they

performed some Yajnas also

Q49) Ans (b)

The roads and river-routes were not

immune from robbery It is notable that

Yuan Chwang (Hiuen Tsang) was

robbed of his belongings during

Harshvardanarsquos period

Q50) Ans (c)

Q51) Ans (b)

Purandara Dasa was a saint and great

devotee of Lord Krishna

There is much speculation about where

Purandara Dasa regarded as the

Pitamaha of Carnatic music was born

Recently an expert committee

constituted by the Kannada University

Hampi has come to the conclusion that

Kshemapura Shivamogga district

Karnataka is the birth place of

Purandara Dasa

Q52) Ans (c)

Sri Tyagaraja Sri Shyama Shastry and Sri Muthuswami Dikshitar are considered the trinity of Carnatic music and with them came the golden age in Carnatic music in the 18th-19th

century

Q53) Ans d)

Recently a rare sarcophagus (stone

coffin) which is 2000 years old from the

Iron AgendashMegalithic era was discovered

from a rock-cut cave at Viyur village of

Kollam near Koyilandy in Kozhikode

district Kerala

The coffin containing bone fragments

was found during an excavation ldquoSo

far such a rare finding has been

discovered only from two sites

in Kerala Both these sarcophagi were

recovered from Megalithic sites at

Chevayur and Atholi also in Kozhikode

district

Q54) Ans a)

The megalithic culture in South India was a full-fledged Iron Age culture

Q55) Ans d)

The Cholas Pandyas and Keralaputras

(Cheras) mentioned in Ashokan

inscriptions were probably in the late

megalithic phase of material culture

Q56) Ans d)

Q57) Ans (b)

Raj Kumar Shukla followed Gandhiji all

over the country to persuade him to

come to Champaran to investigate the

problem associated with tinkathia

system

RAUSIAS-FC19E1003 47

Brij Kishore Rajendra Prasad Mahadev

Desai and Narhari Parikh accompanied

Gandhi ji during the Champaran

Satyagraha

Q58) Ans (b)

The Satvahanas started the practice of granting tax-free villages to brahmanas and Buddhist monks

Q59) Ans c)

The objectives of the Programme are

listed as under

- Developing basic tourism

infrastructure

- Promoting cultural and heritage

value of the country to generate

livelihoods in the identified regions

- Enhancing the tourist attractiveness

in a sustainable manner by

developing world-class

infrastructure at the heritage

monument sites

- Creating employment through active

involvement of local communities

- Harnessing tourism potential for its

effects on employment generation

and economic development

- Developing sustainable tourism

infrastructure and ensuring proper

Operations and maintenance

therein

Q60) Ans (b)

The Tribal Cooperative Marketing

Development Federation of India

(TRIFED) came into existence in 1987

It is a national-level apex organization

functioning under the administrative

control of Ministry of Tribal Affairs

Govt of India

TRIFED has its registered and Head

Office located in New Delhi

Q61) Ans (c)

Premchandrsquos novels include

Premashram Rangabhumi Ghaban

Karmabhumi and Godan

Gora is a novel written by Rabindranath

Tagore

138th birth anniversary of Munshi

Premchand was celebrated across the

country

Q62) Ans (b)

Giddha is a traditional pastoral dance

performed by the women of the Punjab

India and Pakistan at festival times

and at the sowing and reaping of the

harvest

By this dance the Punjabi women

reveal their joy expel their suppressed

feelings in a male dominated society

through the performance of Giddha

Since this dance has nothing to do with

men only women can participate in it

During the Teej celebrations Giddha

dance is celebrated in Punjab every

year Teej is a generic name for a

number of festivals that are celebrated

by women in some parts of India

Q63) Ans (a)

Dara Shukoh wrote the remarkable

work called ldquoMajma-ul-Bahrainrdquo or the

ldquoThe confluence of two seasrdquo

The Vice President of India Shri M

Venkaiah Naidu has said that Prince

Dara Shukohrsquos writings can come as a

refreshing source for infusing peace and

harmony He was addressing the

gathering after visiting the exhibition

that showcases the forgotten Prince of

yesteryears Dara Shukoh organized by

Mr Francois Gautier at Indira Gandhi

National Centre for the Arts in New

Delhi

Q64) Ans (c)

The statue Gommateshwara is

dedicated to the Jain God Bahubali

It is a monolithic statue

President Ram Nath Kovind

inaugurated the grand anointing

ceremony mdash Mahamastakabhisheka mdash

held once in 12 years at

Shravanabelagola (Karnataka)

Q65) Ans (c)

Prachi Valley had come up around the

Prachi river Prachi Valley gradually

disappeared

RAUSIAS-FC19E1003 48

The Prachi river originates from

Bhubaneswar

It is a tributary of the Mahanadi and

flows through the districts of Puri

Khurda Cuttack and Jagatsinghpur

and the entire region of the river is

termed as the Prachi Valley

It falls into the Bay of Bengal

Archaeological evidence shows that the

Prachi Valley Civilisation predates both

Harappa and Mohenjo-Daro

The Prachi river originates from

Bhubaneswar

Q66) Ans (d)

These monuments are located in

Chhatarpur district Madhya Pradesh

within Vindhya mountain range

Q67) Ans (a)

The book lsquoThoughts on Pakistanrsquo was

written by Dr BR Ambedkar

On the occasion of the birth anniversary

of Dr BR Ambedkar the president of

India pays homage to this icon of India

In 1924 he founded the Depressed

Classes Institute (Bahishkrit Hitkarini

Sabha) and in 1927 the Samaj Samata

Sangh

Another area of attention for Ambedkar

was education For its spread among

the low classes he set up a network of

colleges by the name of Peoples

Education Society and founded hostels

Q68) Ans(b)

Mehrgarh is a famous Neolithic

settlement in the Indian subcontinent

which is situated in Baluchistan

province Pakistan

A pre-historic rock art site is discovered

in the vast expanse of limestone blocks

on the eastern banks of Naguleru river

near Dachepalli (Andhra Pradesh) It

has thrown light on the Neolithic

civilisation that flourished in Guntur

(Andhra Pradesh) during 1500-2000

BC

Q69) Ans (c)

The 12th and the 13th centuries saw

the emergence of the Kakatiyas They

were at first the feudatories of the

Western Chalukyas of Kalyana Initially

they ruled over a small territory near

Warangal (Telangana)

They introduced Nayakships which was

later adopted and developed by the

Rayas of Vijayanagara

Q70) Ans (a)

The fast had effect of putting pressure

on mill owners who finally agreed to

give the workers a 35 per cent increase

in wages

Google celebrated with a doodle the

132nd birth anniversary of Anasuya

Sarabhai who played a pioneering role

in Indiarsquos labour movement

Q71) Ans (d)

The UNESCOrsquos list of the representative

list of the intangible cultural heritage of

humanity from India are

- Koodiyattam Sanskrit Theatre of

Kerala

- Mudiyettu ritual theatre and dance

drama of Kerala

- Tradition of Vedic Chanting

- Kalbelia folk songs and dances of

Rajasthan

- Ramlila Traditional Performance of

the Ramayana

- Sankirtana ritual singing

drumming and dancing of Manipur

- Ramman religious festival and

ritual theatre of the Garhwal

Himalayas India

- Traditional brass and copper craft of

utensil making among the Thatheras

of Jandiala Guru Punjab India

- Chhau dance classical Indian dance

originated in the eastern Indian

states

- Buddhist chanting of Ladakh

recitation of sacred Buddhist texts

in the trans-Himalayan Ladakh

region Jammu and Kashmir India

- Yoga

- Nouroz

- Kumbh Mela

RAUSIAS-FC19E1003 49

Q72) Ans(b)

The President of India Shri Ram Nath Kovind inaugurated the Hornbill Festival and State Formation Day celebrations of Nagaland in Kisama

The festival is named after the Indian hornbill the large and colourful forest bird which is displayed in the folklore of most of the states tribes

The major recognized tribes of Nagaland are Angami Ao Chakhesang Chang

Kuki Rengma and Zeling etc

Onge Jarawa and Sentinelese are the

tribes of Andman amp Nicobar Islands

Q73) Ans (c)

The Rashtrakutas rule in the Deccan lasted for almost two hundred years till the end of the tenth century The Rashtrakutas rulers were tolerant in their religious views and patronized not only Shaivism and Vaishnavism but

Jainism as well

The famous rock-cut temple of Shiva at Ellora was built by one of the Rashtrakutas kings Krishna I in the ninth century His successor Amoghavarsha was a Jain but he also

patronized other faiths

The Rashtrakutas allowed Muslims traders to settle and permitted Islam to

be preached in their dominions

Recently increasing defacement at the prehistoric rock paintings of Pandavulagutta Telangana has created a cause for grave concern It can spoil

the prehistoric rock

Pandavulagutta is home to

- Painted rock shelters dating to

10000 BC-8000 BC

- An 8th century inscription of the

Rashtrakuta period and

- Painted frescoes from the 12th century Kakatiya empire

Q74) Ans (b)

In 1828 Raja Ram Mohan Roy founded a new religious society the Brahma Sabha later known as the Brahmo

Samaj

Debendranath Tagore headed the Tattvabodhini Sabha which was

engaged in search of spiritual truth

Its purpose was to purify Hinduism and to preach monotheism or belief in one God

The new society was to be based on the twin pillars of reason and the Vedas and

Upanishads

Recently Sadharan Brahmo Samaj (SBS) has entered into a legal battle with the West Bengal government due

to some legal issue

Q75) Ans (c)

The Chishti order was established in India by Khwaja Moinuddin Chishti who came to India around 1192 The Chishtirsquos are considered to be the most influential of the groups of Sufis who migrated to India in the late twelfth century They adapted successfully to the local environment and adopted several features of Indian devotional

traditions

The historical dargah of Sufi mystic Khwaja Moinuddin Chishti in Ajmer is all set to get a facelift This 13 th century dargah has been included among the Swachh Iconic Places a clean-up initiative focused on iconic

heritage spiritual and cultural places

Page 28: GENERAL STUDIES (PAPER I) · Test is part of Rau’s IAS Test series for Preliminary Exam 2019 FOUNDATION + CURRENT AFFAIRS GENERAL STUDIES (PAPER –I) FOUNDATION TEST –III TOPIC:

T e s t i s p a r t o f R a u rsquo s I A S T e s t s e r i e s f o r P r e l i m i n a r y E x a m 2 0 1 9

FOUNDATION + CURRENT AFFAIRS

GENERAL STUDIES (PAPER ndashI)

FOUNDATION TEST ndashIII

SUBJECT NCERT History Class VI-X + Current Affairs

Time Allowed 1frac12 Hours Maximum Marks 150

I NSTRUCT IONS

1 IMMEDIATELY AFTER THE COMMENCEMENT OF THE EXAMINATION YOU SHOULD CHECK

THAT THIS TEST BOOKLET DOES NOT HAVE ANY UNPRINTED OR TORN or MISSING PAGES OR

ITEMS ETC IF SO GET IT REPLACED BY A COMPLETE TEST BOOKLET

2 This Test Booklet contains 75 items (questions) Each item is printed both in Hindi and English

Each item comprises four responses (answers) You will select the response which you want to mark

on the Answer Sheet In case you feel that there is more than one correct response mark the

response which you consider the best In any case choose ONLY ONE response for each item

3 You have to mark all your responses ONLY on the separate Answer Sheet (OMR sheet) provided

Read the directions in the Answer Sheet

4 All items carry equal marks

5 Before you proceed to mark in the Answer Sheet the response to various items in the Test booklet

you have to fill in some particulars in the Answer Sheet as per instructions contained therein

6 After you have completed filling in all your responses on the Answer Sheet and the examination has

concluded you should hand over to the Invigilator only the Answer Sheet You are permitted to

take away with you the Test Booklet

7 Penalty for wrong answers

THERE WILL BE PENALTY FOR WRONG ANSWERS MARKED BY A CANDIDATE IN THE

OBJECTIVE TYPE QUESTION PAPERS

(i) There are four alternatives for the answer to every question For each question for which a

wrong answer has been given by the candidate one-third of the marks assigned to that

question will be deducted as penalty

(ii) If a candidate gives more than one answer it will be treated as a wrong answer even if one of

the given answers happens to be correct and there will be same penalty as above to that

question

(iii) If a question is left blank ie no answer is given by the candidate there will be no penalty for

that question

T h i s t e s t i s p a r t o f R a u rsquo s I A S T e s t s e r i e s f o r P r e l i m i n a r y E x a m 2 0 1 9

Test Code

FC19E1003

FC19H1003 29

Answers and Explanations of

NCERT History Class VI-X + Current Affairs (FC19E1003)

Q1) उततर (c)

सपषटीकरण

- ऋगवद म दविय ो और दिताओो क समवपित एक

हजार स अविक सत तर (शल क) ह

- य शल क ऋविय ो क दवारा रच गए थ और परि ो

दवारा सीख जात थ

- हालाोवक कछ शल क मवहलाओो (जस वक अपाला

घ सा ल पामदरा मतरयी और गागी) क दवारा भी रच

गए थ

- ऋगवद म सोिाद क रप म कई शल क मौजद ह

- हम विशवावमतर नामक एक ऋवि और दविय ो क

रप म पजी जान िाली द नवदय ो (वयास और

सतलज) क बीच िाताि का उदाहरण वमलता ह

- इसस पता चलता ह वक विशवावमतर िवदक काल स

सोबोवित थ

Q2) उततर (b)

सपषटीकरण

- करनल गफाओो स राख क अिशि परापत हए ह

ज इस ओर सोकत करत ह वक ततकालीन ल ग

अवि क उपय ग स पररवचत थ

- य गफाएो आोधर परदश म सथथत ह

Q3) उततर (c)

सपषटीकरण

bull बरािह म ितिमान कशमीर म सथथत एक

परागवतहावसक थथल ह जहाो ल ग गडढ क घर ो का

वनमािण करत थ

bull य घर जमीन क ख द कर बनाए जात थ तथा नीच

जान क वलए सीवियाा ह ती थी

bull ऐसा अनमान लगाया जाता ह वक य घर ठो ड क

मौसम म आशरय परदान करत थ

Q4) उततर (c)

सपषटीकरण

bull परालख-विदया (Epigraphy) क वशलालख ो क

अधययन क रप म पररभावित वकया जाता ह

bull हसतवलसखत दसतािज ो क माधयम स इवतहास

और सावहतय क अधययन क पाोडवलवप विजञान

(Manuscriptology) कहत ह

bull पराचीन लखन परणावलय ो क अधययन और

ऐवतहावसक पाोडवलवपय ो क समझन तथा वतवथ

वनिािरण क पलीओगराफी (Palaeography) कहा

जाता ह

bull नयवमजमविकस (Numismatics) वसक ो क

अधययन क सोदवभित करता ह

Q5) उततर (a)

सपषटीकरण

- चरक सोवहता चरक क दवारा वलखी गई आयिद

और िदयक-शासर पर एक महतवपणि पसतक ह

- ि भारतीय िदयक-शासर की पारमपररक परणाली

वजस आयिद क नाम स जाना जाता ह क

अभयासकताि थ

- ऐसा माना जाता ह वक चरक का विकास दसरी

शताबदी (ईसा पिि) और दसरी शताबदी (ईसवी) क

मधय हआ था

Q6) उततर (b)

सपषटीकरण

- भाग फसल ो पर वलए जान िाल कर क सोदवभित

करता ह ज कल फसल उतपादन का 16 िाो भाग

था

- ldquoकममकारrdquo शबद भवमहीन कवि शरवमक िगि क

वलए परय ग वकया जाता था

- ldquoअशवमिrdquo (वजस घ ड क बवलदान क रप म भी

जाना जाता ह) एक अनषठान ह ता था वजसम एक

घ ड क सवतोतर रप स घमन क वलए छ ड वदया

FC19H1003 30

जाता ह और राजा क सवनक उसकी रखिाली

करत थ

Q7) उततर (d)

सपषटीकरण

- ऋगववदक काल म घ ड ो क रथ ो म ज ता जाता था

ज (रथ) भवम मिवशय ो आवद पर कबजा करन क

वलए लड गए यद ो म उपय ग वकए जात थ

- इसस यह पता चलता ह वक घ ड ो यकत रथ ो का

उपय ग महाजनपद काल स काफी पहल आरमभ

हआ था

- ऋगववदक काल म मिवशय ो भवम जल आवद पर

कबजा करन क वलए तथा ल ग ो क पकडन क

वलए यद वकय जात थ

- अविकाोश परि इन यद ो म भाग वलया करत थ

- हालाोवक उस समय क ई वनयवमत सना नही ो ह ती

थी लवकन उस काल म सभाऐो ह ती थी ो वजनम

ल ग यद क मामल ो पर चचाि करत थ

- वनयवमत सनाएा महाजनपद काल का िवशषटय थी

वजनम पदल सवनक ो की विशाल सनाएा रथ तथा

हाथी शावमल ह त थ

Q8) उततर (a)

सपषटीकरण

- बद शाकय कल स सोबोवित थ और कशीनारा म

उनका वनिन हआ था

- बद न अपनी वशकषाएा पराकत भािा म दी थी ो ज

आम ल ग ो की भािा थी

Q9) उततर (c)

सपषटीकरण

- पराचीन भारत म दशिनशासर की छह शाखाएा थी ो

िशविक नयाय समखया य ग पिि वममाोसा और

िदाोत या उततर वममाोसा

- इनकी थथापना करमश कनाद गौतम कवपल

पतोजवल जावमनी और वयास ऋविय ो न की थी

Q10) उततर (b)

सपषटीकरण

महािीर की वशकषाऐो छठी शताबदी म िललभी म

सोकवलत की गई थी ो

Q11) उततर (c)

सपषटीकरण

- पारमपररक रप स चाणकय क कौविलय अथिा

विषणगपत क नाम स जाना जाता ह

- उसन अथिशासतर ज एक पराचीन भारतीय

राजनवतक आलख ह वलखा था

Q12) उततर (d)

सपषटीकरण

- भारत का राषटर ीय वचनह सारनाथ (उततर परदश) क

अश क सतमभ क ऊपर (शीिि पर) वसोह कवपिल

का एक अनरपण ह

- इस राषटर ीय वसदाोत सतयमि जयत क साथ

सोय वजत वकया गया ह

- रामपिि बल का नाम रामपिि (वबहार) क नाम पर

पडा जहाा इसकी ख ज हई थी

- यह अपन नाजक नकाशी मॉडल क वलए परवसदद

ह वजसम क मल तवचा सोिदनशील नथन ो सतकि

कान और मरबत िााग ो क शरषठतर परवतरप क

परदवशित वकया गया ह

- यह भारतीय और फारसी ततव ो का एक ससममशरण

- सोवकससा उततर परदश म सथथत ह

Q13) उततर (a)

सपषटीकरण

का िर वसोह ज एक महान य दा थ वबहार स

सोबोवित थ

Q14) उततर (b)

सपषटीकरण

िललालर शबद बड भ-सवावमय ो क वलए परय ग

वकया जाता था

FC19H1003 31

Q15) उततर (c)

सपषटीकरण

- अररकमड एक तिीय बसती थी जहाो दर दश ो स

आन िाल जहाज ो का माल उतारा जाता था

- यहाो पर ईोि ो का एक विशाल ग दाम वमटटी क

बतिन (वजनम एमफ रा - द हरी मवठय ो का लोबा

घडा - शावमल ह) और एरिाइन (Arretine)

मदभाोड पाए गए थ

- इस थथान पर र मन दीपक काोच क बन पातर और

रतन भी पाए गए थ

Q16) उततर (a)

सपषटीकरण

- मिनदर सोगम कविताओो म उसललसखत एक

तवमल शबद ह वजसका अथि ह ldquoतीन परमखrdquo

- यह तीन सततारि पररिार ो क मसखयाओो क वलए

परय ग वकया जाता ह च ल चर और पाणडय

Q17) उततर (c)

सपषटीकरण

- ऋग िद म सभा विदाथा तथा गण जसी

जनजावतय ो पर अथिा किोब पर आिाररत

सभाओो का उललख ह

- आरसमभक िवदक काल म सभाओो और सवमवतय ो

का विशि महतव ह ता था

- यहाा तक की मसखया अथिा राजा भी उनका

समथिन परापत करन क वलए आतर रहत थ

Q18) उततर (a)

सपषटीकरण

- जन िमि न ईशवर क अससततव क मानयता त दी ह

वकनत उसन ईशवर क वजना क पद स नीच रखा

- जन िमि न बौद िमि की तरह िणि परणाली की

भरतिना नही ो की थी

Q19) उततर (d)

सपषटीकरण

- च ल ो और पाणडय ो न शसकतशाली तिीय शहर ो का

विकास वकया था

- च ल ो का सबस महतवपणि शहर पहार (या

कािरीपटटीनम) था |

- मदरई पाणडय ो की राजिानी थी

Q20) उततर (b)

सपषटीकरण

- ldquoबदचररतrdquo बद का जीिन-ितताोत ह

- इस अशवघ ि क दवारा वलखा गया था

Q21) उततर (a)

सपषटीकरणः

- तवमल कवि अपपर भगिान वशि क भकत थ

- इस परकार ि एक नयनार सोत थ

Q22) उततर (d)

सपषटीकरणः

- समदरगपत एक परवसद गपत शासक था

- उसन वसक ो पर िीणा बजात हए अपनी छवि

अोवकत करिाई थी

- यह सोगीत क परवत उसक परम क दशािता ह

- हम उसकी इलाहाबाद परशससत स महतवपणि

ऐवतहावसक जानकारी वमलती ह वजसकी रचना

उसक दरबार क कवि हररसन न की थी

Q23) उततर (b)

सपषटीकरणः

- विकरम सोित की शरआत ििि 58 ईसा पिि म

चनदरगपत वदवतीय न की थी

- यह शक ो पर उसकी जीत और उस विकरमावदतय

की पदिी वमलन क उपलकषय म आरमभ वकया गया

था

FC19H1003 32

- बानभटट न हिििििन का जीिन-ितताोत हििचररत

(ज सोसकत म थी) वलखी थी

Q24) उततर (c)

सपषटीकरणः

- सोवि-विगरावहका यद एिो शाोवत का मोतरी

- साथििाह वयापाररय ो क कावफल ो का नता

Q25) उततर (a)

सपषटीकरणः

- जआन झाोग (हसआन रताोग ndash Hsuang Tsang)

एक चीनी यातरी था ज हिििििन क शासनकाल म

भारत आया था

- ििि 630 ईसवी स ज दशक आरमभ हआ था उसम

जआन झाोग मधय एवशया ईरान और

अफग़ावनसतान की यातरा करन क पशचात कशमीर

क रासत स भारत आया था

- उसन उततर स पिि तक की यातरा की और िह

लगभग 2 ििि वबहार म रहा

- जआन झाोग न नालनदा विशवविदयालय म विदयावथिय ो

और विदवान ो क साथ पारसपररक विचार-विमशि

वकया थथानीय भािाओ ा म वनपणता परापत की तथा

बौद सतप ो की ख ज की

Q26) उततर (c)

सपषटीकरणः

- परदवकषणा पथ बौद िासतकला म सतप क चार ो

ओर बनाया जान िाला एक घमािदार पथ ह ता

- परशन म वदए गए बाकी क तीन ो ततव वहोद मसनदर ो की

िासतकला क भाग ह

Q27) उततर (d)

सपषटीकरणः

परशन म वदए गए सभी मोवदर ो म वयापक रप स

ईोि ो (पकी ईोि ो) का परय ग पतथर ो क साथ हआ

Q28) उततर (c)

सपषटीकरण

- महममद कली कतब शाह ग लकणडा का सलतान

था

- िह अकबर का समकालीन था

- सावहतय और िासतकला म उसकी अतयाविक

रवच थी

- िह एक महान कवि था

- िह दसखनी उदि फारसी और तलग म वलखता था

- उसन अपन पीछ एक विसतत वदिान (सोगरह)

छ डा ह

- अभी हाल ही म तलोगाना म ग लकणडा क वकल

क अनदर खदाई वकय गए बाग-ए-नाया वकला

बाग क चार ो ओर रप-रखा क मानवचतरण क

वलए भारतीय परातासतवक सिकषण (The

Archaeological Survey of India ndash ASI)

गराउणड पनीिर विोग रडार (Ground Penetrating

Radar) का परय ग करगा

Q29) उततर (a)

सपषटीकरणः

- वसलपपावदकारम एक तवमल महाकावय ह वजसकी

रचना इलाोग क दवारा लगभग 1800 ििि पिि की

गई थी

- यह क िलन नामक एक वयापारी की कहानी ह

ज माििी नामक एक गवणका (िशया) स परम

करन लगा था

- मवनमकलाई क िलन और माििी की पतरी की

कहानी ह

Q30) उततर (a)

सपषटीकरण

- चरक आयिद और वचवकरता की एक महतवपणि

रचना चरक सोवहता क लखक ह

- बरहमगपत क अपनी रचना बरहम-सफि-वसदानत

(ज एक खग लीय रचना ह) क कारण परवससद

वमली

FC19H1003 33

- बगदाद म इसका अनिाद अरबी भािा म वकया

गया था

- इसका इसलावमक गवणत और खग ल-विजञान पर

महतवपणि परभाि पडा था

- बाद म अपन जीिनकाल म बरहमगपत न

ldquoखोडखयाकrdquo वलखी ज एक खग लीय पससतका

(एक छ िी पसतक) थी

- इसम आयिभटट की अिि-रावतर क परतयक वदन की

शरआत परणाली का परय ग वकया गया था

Q31) उततर (c)

सपषटीकरण

- अमीर खसर एक परवसद सफी सोगीतकार कवि

और विदवान थ

- 1318 म उनह ोन पाया वक इस भवम (वहोदसतान) क

हर कषतर म अलग-अलग भािा थी लाहौरी

कशमीरी दवारसमदरी (दवकषणी कनाििक म)

तलोगाना (आोधर परदश म) गजरी (गजरात म)

माबारी (तवमलनाड म ) अििी (पिी उततर परदश

म) और वहोदिी (वदलली क आस-पास क कषतर म)

आवद

- उनह न यह बताया वक सोसकत वकसी भी कषतर स

सोबोवित नही ो थी और किल बराहमण ही इस भािा

का जञान रखत थ

Q32) उततर (c)

सपषटीकरण

- वहरणय-गभि सववणिम गभि क सोदवभित करता ह

- जब बराहमण ो की सहायता स यह अनषठान वकया

जाता था त यह माना जाता था वक बवल दन िाल

का कषवतरय क रप म पनजिनम ह गा

Q33) उततर (d)

सपषटीकरण

- कदमई भवम राजसव पर कर क सोदवभित करता

- गवावलयर परशससत म नागभि क दवारा वकय गए

श िण का िणिन वकया गया ह |

- नागभि एक परवतहार राजा था

Q34) उततर (b)

सपषटीकरण

- राजतरो वगनी 12िी ो शताबदी म कलहन क दवारा

रवचत एक सोसकत पसतक (िकसट) ह

- यह परारसमभक भारत की ऐवतहावसक इवतितत थी

- तकि सोगत रप स इस अपन परकार की सिोततम

और सिािविक विशवसनीय कवत माना जाता ह

- यह कशमीर कषतर क पराचीनतम समय स लकर

उसकी रचना की तारीख तक क समपणि इवतहास

का आचछादन करती ह

Q35) उततर (c)

सपषटीकरण

- गााि की आम सभा क ldquoउरrdquo कहा जाता था

- ldquoउरrdquo म गााि क सभी कर दन िाल वनिासी

शावमल ह त थ

Q36) उततर (a)

सपषटीकरण

- वदलली सलतनत म ldquoतारीखrdquo इवतहास लखन का

एक रप था

- ldquoतािरीखrdquo क लखक विदवान परि ह त थ वजनम

सवचि परशासक इतयावद शावमल थ

Q37) उततर (a)

सपषटीकरण

- अलाउददीन सखलजी अपन सवनक ो क ितन का

भगतान नकद म करता था न वक इकता क रप

- सवनक अपना सामान वदलली म वयापाररय ो स

खरीदत थ अतः इस बात का भय था वक वयापारी

कही ो िसतओो का मलय न बिा द

- इसकी र कथाम क वलए अलाउददीन सखलजी न

वदलली म कीमत ो क वनयसित वकया

FC19H1003 34

- अविकारीगण धयानपििक मलय ो का सिकषण करत

थ तथा ज वयापारी वनिािररत मलय पर माल नही ो

बचत थ उनक दसणडत वकया जाता था

Q38) उततर (d)

सपषटीकरण

- वदलली सििपरथम त मर राजपत ो क अिीन उनक

सामराजय की राजिानी बनी थी

- 12िी ो शताबदी क मधय म अजमर क चौहान ो

(वजनह चाहमान ो क नाम स भी जाना जाता ह) न

त मर राजपत ो क परावजत वकया था

- त मर ो और चौहान ो क अिीन वदलली एक

महतवपणि िावणसजयक क दर बन गया था

- कई जन वयापारी यहाा रहन लग थ और उनह ोन

कई मोवदर भी बनिाए

- यहाा पर मवदरत वसक वजनह ldquoदहलीिालrdquo क नाम

स जाना जाता था वयापक रप स परचलन म थ

Q39) उततर (c)

सपषटीकरण

- म ठ की मसिद का वनमािण वसको दर ल दी क

राजयकाल म उसक मिी क दवारा करिाया गया

था

- बगमपरी मसिद का वनमािण महममद तगलक क

शासनकाल म हआ था

- यह मसिद विशव का पणयथथान (The

Sanctuary of the World) और वदलली म महममद

तगलक की नई राजिानी जहाोपनाह की मखय

मसिद थी

- कववत- अल - इसलाम मसिद का विसतार

इलतसिश और अलाउददीन सखलजी न वकया था

- मीनार का वनमािण तीन सलतान ो कतबददीन ऐबक

इलतसिश और वफर ज शाह तगलक क दवारा

करिाया गया था

Q40) उततर (c)

सपषटीकरण

- मगल ो क अिीन मनसबदार शबद उस वयसकत क

वलए सोदवभित वकया जाता था वजसक पास मनसब

(अथाित पद) ह ता था

- उस अपना ितन राजसव कायो वजनह जागीर कहत

थ क रप म परापत ह ता था

Q41) उततर (b)

सपषटीकरण

- ldquoभारत छ ड आोद लनrdquo वबरविश शासन क

सखलाफ ल ग ो का एक सवाभाविक विदर ह था

- असखल भारतीय काोगरस सवमवत न 8 अगसत 1942

क बमबई म एक बठक का आय जन वकया था

- इस बठक म परवसद सोकलप ldquoभारत छ ड rdquo क

पाररत वकया गया और इस उददशय क परापत करन

क वलए गाोिी क नततव म एक अवहोसक जन सोघिि

आोद लन की शरआत का परसताि वदया गया

- लवकन अगल ही वदन गाोिी और काोगरस क अनय

परमख नताओो क वगरफतार कर वलया गया

- काोगरस क एक बार वफर अिि घ वित वकया गया

था

Q42) उततर (c)

सपषटीकरण

- साइमन कमीशन यनाइविड वको गडम क सात

साोसद ो का एक समह था

- इस वबरविश भारत क वलए सोििावनक सिार ो का

सझाि दन क वलए गवठत वकया गया था

- इस आय ग म िररषठ वबरविश राजनता सर जॉन

साइमन क नततव म किल वबरविश सदसय ही

शावमल थ

- इसवलए भारत क ल ग ो न साइमन कमीशन क

आगमन क विरद आोद लन वकया था

Q43) उततर (a)

सपषटीकरण

bull दादा भाई नौर जी भारत म वबरविश शासन क

आवथिक पररणाम ो क बार म अपनी विर िी

(परवतकल) राय क वलए जान जात थ

FC19H1003 35

bull अपन कई लख ो और भािण ो म विशि रप स

ldquoपाििी एो ड अन-वबरविश रल इन इसणडया

(Poverty and Un-British Rule in India) म

नौर जी न यह तकि वदया वक भारत पर अतयविक

कर लगाया गया था और इसकी सोपवतत इोगलड की

ओर परिावहत की जा रही थी

bull उनह ोन पराचीन भारतीय गरोथ ो की वयाखया करन

का और भारतीय ो क आिविशवास क बहाल

करन पर कायि नही ो वकया था

उनह ोन वकसी और बात स पहल सभी सामावजक

बराइय ो क उनमलन की आिशयकता पर भी बल

नही ो वदया था

Q44) उततर (c)

सपषटीकरण

bull अगसत 1932 म वबरविश परिानमोतरी मकड नालड न

अपन साोपरदावयक परसकार (The Communal

Award) की घ िणा की थी

bull यह भारत क कई साोपरदावयक वहत ो क बीच विवभनन

सोघिो क हल करन क वलए वबरिन का एकतरफा

परयास था

bull यह परसकार (Award) बाद म 1935 क

अविवनयम (The Act of 1935) म शावमल वकया

गया था

bull इस साोपरदावयक परसकार न मससलम ो क वलए

आरवकषत एक अलग वनिािचक मणडल फॉमिल का

विसतार अनय अलपसोखयक ो क वलए वकया था

वजसम वसख ो भारतीय ईसाइय ो आोगल-भारतीय

समदाय यर पीय समदाय तथा विवशषट कषतरीय

समह ो क शावमल वकया गया था

bull गाोिी न इस परसताि क भारतीय समाज क

विभावजत करन क वलए एक घवणत वबरविश

सावजश क रप म दखा और उसक सखलाफ

आमरण अनशन वकया

Q45) उततर (b)

सपषटीकरण

मौजदा आयात और वनयाित क अवतररक़त

औपवनिवशक भारत क वनमनवलसखत खचो क

वलए एक विशिवनवशचत िन रावश भी दनी पडती

थी

(i) परशासन क वयय

(ii) सना क रख-रखाि क वयय

(iii) यद क वयय

(iv) सिावनितत अविकाररय ो की पशन तथा

(v) वबरिन दवारा अपनी उपवनिश बसती

(कॉल नी) क रख-रखाि क वयय

इनह गह शलक (Home Charges) क रप म

जाना जाता था और लगभग परी तरह स भारत क

दवारा इनका भगतान वकया जाता था

bull गह शलक म वनमनवलसखत घिक शावमल थ

(i) भारतीय ऋण पर दय बयाज

(ii) ईसट इोवडया को पनी क शयरिारक ो क

लाभाोश

(iii) लोदन म भारत कायािलय चलान क वलए िन

(iv) भारत म वनयकत वबरविश कवमिय ो क ितन

और पशन का भगतान करन क वलए िन

(v) रलि पर बयाज

(vi) नागररक और सनय शलक

(vii) इोगलड म सट र (सामगरी) की खरीद

Q46) उततर (b)

सपषटीकरण

bull भारतीय राषटर ीय काोगरस का लाहौर सतर 1929 म

जिाहरलाल नहर की अधयकषता म आय वजत

वकया गया था

bull इस सतर म भारतीय राषटर ीय आोद लन स समबसित

कई महतवपणि पररणाम सामन आय थ

(i) सििपरथम इस सतर म काोगरस क अधयकष पद

पर जिाहरलाल नहर क चना गया था ज

काोगरस म िामपोवथय ो की बिती हई ताकत

का सपषट सोकत था

(ii) दसरा इस सतर म पहली बार काोगरस न पणि

सवतोतरता की माोग क उठाया था

इस परकार की माोग काोगरस मोच स पहल कभी भी

नही ो उठाई गई थी

Q47) उततर (b)

सपषटीकरण

FC19H1003 36

bull इस ररप िि न वकसी भी समदाय क वलए पथक

वनिािचक मोडल अथिा अलपसोखयक ो क वलए

भाराोश की वसफाररश नही ो की थी

bull तथावप इस ररप िि न उन पराोत ो म अलपसोखयक

सीि ो क आरकषण की अनमवत दी थी जहाा पर कम

स कम दस परवतशत अलपसोखयक ह

bull लवकन यह समदाय क आकार क अनपात म ह ना

चावहए था

bull इस ररप िि म भारत क वलए पणि सवतोतरता क

वलए क ई पराििान नही ो था

Q48) उततर (c)

सपषटीकरण

bull आरो वभक िवदक आयो का िमि मखय रप स

परकवत की पजा और यजञ था

bull परारो वभक आयि िमि परकवत की पजा क समान था

bull िासति म उनक चार ो ओर की शसकतयाा वजनह न

त ि वनयोवतरत कर सकत थ और न ही समझ पाए

थ उनह वदवयता क साथ वनिवशत वकया गया तथा

उनह मादा या नर दिीदिताओो क रप म

परतीकतव वकया गया था

bull उनह ोन कछ यजञ ो का भी वनषपादन वकया था

Q49) उततर (b)

सपषटीकरण

bull सडक और नदी-मागि (जल-मागि) डकती स

सरवकषत नही ो थ

bull उललखनीय ह वक हिििििन क शासनकाल क

दौरान यआन चिाोग (हयएन साोग) का सारा

सामान लि वलया गया था

Q50) उततर (c)

सपषटीकरण

परशन म वदए गए द न ो कथन सही ह

Q51) उततर (b)

सपषटीकरण

bull परोदर दास एक सोत और भगिान कषण क एक

महान भकत थ

bull परोदर दास क कनाििक सोगीत क वपतामह क

रप म जाना जाता ह

bull यदयवप उनक जनम-थथान क बार म काफी

अिकल लगाई जाती रही ह

bull तथावप अब कननड विशवविदयालय हमपी क दवारा

गवठत एक विशिजञ सवमवत इस वनषकिि पर पहोची

ह वक उनका जनम थथान सोभितया कनाििक का

एक छ िा-सा गााि कषमपरा (वशिम गगा वजला)

था

Q52) उततर (c)

सपषटीकरण

bull शरी तयागराज शरी शयाम शासतरी और शरी मथसवामी

दीवकषतर क कनाििक सोगीत की वतरमवति माना

जाता ह

bull उनक कारण ही 18िी ो-19िी ो शताबदी म कनाििक

सोगीत का सववणिम यग आया था

Q53) उततर (d)

सपषटीकरण

bull अभी हाल ही म लौह यगीन-महापािावणक काल

का 2000 ििि पराना एक दलिभ सारक फगस

(Sarcophagus) (पतथर का ताबत) क ललम क

वियर गाोि (क वयलडी क पास वजला क वझक ड

करल राजय) की एक रॉक-कि गफा स ख जा गया

bull यह ताबत वजसम हविय ो क िकड थ खदाई क

दौरान वमला

bull अभी तक इस परकार की दलिभ ख ज करल क

मातर द ही थथान ो स हई ह

bull य द न ो सारक फगी (Sarcophagi) (पतथर क

ताबत) चियर और अथ ली (वजला क वझक ड) क

महापािाण थथल ो स वमल ह

Q54) उततर (a)

सपषटीकरण

FC19H1003 37

दवकषण भारत म महापािाण सोसकवत एक पणि

विकवसत लौह यगीन सोसकवत थी

Q55) उततर (d)

सपषटीकरण

bull च ल पाणडय और करलपतर (चर) इन तीन ो का

उललख अश क क अवभलख ो म वकया गया ह

bull सोभितः य भौवतक सोसकवत क उततर

महापािावणक चरण म थ

Q56) उततर (d)

सपषटीकरण

bull भीमा-क रगाोि की लडाई ततीय आोगल-मराठा

यद का वहससा थी

Q57) उततर (b)

सपषटीकरण

bull राजकमार शकल न गाोिीजी क चोपारण आन तथा

वतनकवथया परणाली स जडी समसया की जाोच क

वलए रारी करन क वलए दश भर म उनका

अनसरण वकया था

bull बज वकश र राजदर परसाद महादि दसाई और

नरहरी पाररख चोपारण सतयागरह क दौरान गाोिी

जी क सहय गी थ

Q58) उततर (b)

सपषटीकरण

bull बराहमण ो और बौद मठिाररय ो क कर-मकत गााि

अनदान म दन की परथा सतिाहन ो न आरमभ की

थी

Q59) उततर (c)

सपषटीकरण

इस कायिकरम क उददशय वनमनानसार ह

(i) बवनयादी पयििन आिाररक सोरचना का विकास

करना

(ii) चयवनत (पहचान वकय गए) कषतर ो म आजीविका क

सजन क वलए दश क साोसकवतक और विरासत

मलय ो क बिािा दना

(iii) विरासत समारक थथल ो पर विशव सतरीय आिाररक

सोरचना विकवसत करक एक सतत तरीक स

पयििक आकििण म िसद करना

(iv) थथानीय समदाय ो की सवकरय भागीदारी क माधयम

स र रगार ो का सजन करना

(v) र रगार उतपादन और आवथिक विकास क वलए

पयििन कषमता का उन पर परभाि का उपय ग

करना तथा

(vi) िारणीय पयििन आिाररक सोरचना का विकास

करना और उसका उवचत सोचालन तथा

रखरखाि सवनवशचत करना

Q60) उततर (b)

सपषटीकरण

bull यह वनकाय ििि 1987 म अससततव म आया था

bull यह एक राषटर ीय सतर का शीिि सोगठन ह ज भारत

सरकार क जनजातीय मामल ो क मोतरालय क

परशासवनक वनयोतरण क अिीन काम कर रहा ह

bull इसका पोजीकत और परिान कायािलय नई वदलली

म सथथत ह

Q61) उततर (c)

सपषटीकरण

bull परमचोद क उपनयास ो म परमाशरम रोगभवम गबन

कमिभवम और ग दान शावमल ह

bull ग रा रिी ोदरनाथ िग र क दवारा रवचत उपनयास ह

bull अभी हाल ही म मोशी परमचोद की 138िी ो जयोती दश

भर म मनाई गई थी

Q62) उततर (b)

सपषटीकरण

bull ldquoवगदाrdquo पोजाब (भारत) एिो पावकसतान की

मवहलाओो क दवारा तयौहार क समय और फसल

की बिाई तथा किाई क अिसर पर वकया जान

िाला एक पारोपररक दहाती नतय ह

FC19H1003 38

bull इस नतय क माधयम स पोजाबी मवहलाऐो अपनी

परसननता परकि करती ह तथा वगदा क परदशिन क

माधयम स परि िचिसव िाल समाज म मवहलाओो

की दबी हई भािनाओो क परकि करती ह

bull चोवक इस नतय का परि ो क साथ क ई सोबोि नही ो

ह अतः किल मवहलाऐो ही इसम भाग ल सकती

bull हर साल तीज समार ह क दौरान पोजाब म वगदा

नतय वकया जाता ह

तीज भारत क कछ भाग ो म मवहलाओो क दवारा

मनाया जान िाल कई तयौहार ो क वलए एक

वयापक नाम ह

Q63) उततर (a)

सपषटीकरण

- मजम-उल-बहरीन या द समदर ो का सोगम

नामक उललखनीय रचना दारा वशक ह क दवारा

वलखी थी

- भारत क उपराषटर पवत शरी एम िकया नायड न कहा

ह वक राजकमार दारा वशक ह की रचनाएा शाोवत

और सदभाि क बिािा दन क वलए एक तारा सर त

क रप म सामन आ सकती ो ह

- उपराषटर पवत गत ििो क भला वदए गए राजकमार

दारा वशक ह क परदवशित परचवलत करन हत

आय वजत एक परदशिनी का दौरा करन क बाद एक

सभा क सोब वित कर रह थ

- इस परदशिनी का आय जन फर क इस गौवियर

(Francois Gautier) क दवारा lsquoइोवदरा गाोिी नशनल

सिर फॉर द आििसrsquo (The Indira Gandhi

National Centre for the Arts) नई वदलली म

वकया गया था

Q64) उततर (c)

सपषटीकरण

- ग मतशवर परवतमा जन भगिान बाहबली क

समवपित ह

- यह एक एक-चटटानी पतथर की मवति ह

- राषटर पवत राम नाथ क विोद न शरिणबलग ला

(कनाििक) म आय वजत वकय जान िाल भवय

अवभिक समार ह महामसतकावभिक का

उदघािन वकया था

- यह समार ह 12 ििो म एक बार ह ता ह

Q65) उततर (c)

सपषटीकरण

bull पराची घािी पराची नदी क चार ो ओर फली हई थी

bull पराची घािी िीर-िीर विलपत ह गई थी

bull पराची नदी भिनशवर स वनकलती ह

bull यह महानदी की एक सहायक नदी ह और यह

परी खदाि किक तथा जगतवसोहपर वजल ो स

ह कर बहती ह

bull इस नदी क पर कषतर क पराची घािी कहा जाता ह

bull यह नदी बोगाल की खाडी म वगरती ह

परातासतवक साकषय स पता चलता ह वक पराची घािी

सभयता हडपपा और म हनज दाड द न ो की

पिििती ह

Q66) उततर (d)

सपषटीकरण

य समारक छतरपर वजल (मधय परदश) म विोधयाचल

पिित शरोखला म सथथत ह

Q67) उततर (a)

सपषटीकरण

bull थॉिस ऑन पावकसतान नामक पसतक डॉ बी

आर अमबडकर न वलखी थी

bull डॉ बी आर अमबडकर की जयोती क अिसर पर

भारत क राषटर पवत न भारत की इस महान हसती

क शरदाोजवल अवपित की थी

bull डॉ बी आर अमबडकर न 1924 म वडपरथड

कलावसर इोसटीटयि (दवलत िगि सोथथान -

बवहषकत वहतकाररणी सभा) और 1927 म समाज

समता सोघ की थथापना की थी

bull अमबडकर का धयान वशकषा कषतर की ओर भी था

bull उनह ोन वशकषा क वनमन िगो म फलान क वलए

पीपलस एजकशन स साइिी (The Peoples

Education Society) क नाम स महाविदयालय ो क

नििकि और छातरािास ो की थथापना की थी

FC19H1003 39

Q68) उततर (b)

सपषटीकरण

bull महरगि भारतीय उपमहादवीप म एक परवसद

निपािाण बसती ह ज बलवचसतान पराोत

पावकसतान म सथथत ह

bull दचपलली (आोधर परदश) क पास नागलर नदी क

पिी ति ो पर चना पतथर क बलॉक क विशाल

विसतार म एक पिि-ऐवतहावसक रॉक आिि थथल की

ख ज की गई ह

bull इसन 1500-2000 ईसा पिि क दौरान गोिर (आोधर

परदश) म विकवसत निपािाण सभयता पर परकाश

डाला ह

Q69) उततर (c)

सपषटीकरण

bull 12िी ो सदी और 13िी ो सदी म काकाविय िोश का

उदय हआ था

bull ि पहल कलयाण क पवशचमी चालकय ो क सामोत थ

bull परारोभ म उनह ोन िारोगल (तलोगाना) क पास एक

छ ि स कषतर पर शासन वकया था

bull उनह ोन ldquoनायक वयिथथाrdquo की शरआत की थी

वजस बाद म विजयनगर क राय शासक ो न

अपनाया और विकवसत वकया था

Q70) उततर (a)

सपषटीकरण

bull गाोिीजी क अनशन स वमल मावलक ो पर दबाि

पडा था ज अोततः शरवमक ो क ितन म 35 परवतशत

की िसद करन क वलए सहमत हए थ

bull गगल (Google) न अनसया साराभाई वजनह ोन

भारत क शरवमक आोद लन म एक अगरणी भवमका

वनभाई थी की 132िी ो जयोती डडल (Doodle) का

वनमािण करक मनाई

Q71) उततर (d)

सपषटीकरण

भारत स यनसक की मानिता की अमति साोसकवतक

विरासत की परवतवनवि सची म वनमनवलसखत शावमल ह

bull कवडयटटम करल का सोसकत रोगमोच

bull मवडयिि करल का अनषठान रोगमोच और नतय

नाविका

bull िवदक मि जाप की परोपरा

bull राजथथान क कालबवलया ल क गीत और नतय

bull रामलीला रामायण का पारोपररक परदशिन

bull सोकीतिन मवणपर का अनषठान गायन ढ ल िादन

और नतय

bull रममन भारत क गििाल वहमालय का िावमिक

तयौहार और अनषठान रोगमोच

bull जाोदीयाला गर पोजाब क ठठर ो की पीतल और

ताोब क वशलप स वनवमित बतिन ो की पारोपररक कला

bull छाऊ नतय पिी भारतीय राजय ो म जनमी शासतरीय

भारतीय नतय कला

bull लददाख का बौद मि जाप िर ाोस-वहमालयी लददाख

कषतर तथा जमम-कशमीर म पवितर बौद गरोथ ो का पाठ

bull य ग

bull नौर र

bull को भ मला

Q72) उततर (b)

सपषटीकरण

bull भारत क राषटर पवत शरी राम नाथ क विोद न

वकसामा नागालड म हॉनिवबल मह रति और

राजय गठन वदिस समार ह का उदघािन वकया

था

bull हॉनिवबल मह रति का नाम भारतीय हॉनिवबल क

नाम पर पडा ह ज एक विशाल और रोगीन जोगली

पकषी ह

bull यह पकषी नागालड राजय की अविकतर जनजावतय ो

की ल ककथाओो म उसललसखत ह

bull नागालड की परमख मानयता परापत जनजावतयाा ह

अोगामी आओ चखसोग चाोग ककी रगमा और

रवलोग आवद

bull ओोग जारिा और ससिनलीस अोडमान-वनक बार

दवीप समह की जनजावतयाा ह

FC19H1003 40

Q73) उततर (c)

सपषटीकरण

bull दकन म राषटर कि शासन दसिी ो सदी क अोत तक

लगभग 200 ििो तक रहा था

bull राषटर कि शासक अपन िावमिक विचार ो म सवहषण

bull उनह ोन न किल शि िमि और िषणि िमि बसलक

जन िमि क भी सोरकषण वदया था

bull एल रा म वशि क परवसद रॉक कि मोवदर का

वनमािण नौिी ो सदी म राषटर कि राजा कषण परथम न

करिाया था

bull उसका उततराविकारी अम घििि जन था लवकन

उसन अनय िमो क भी सोरकषण परदान वकया था

bull राषटर कि ो न मसलमान वयापाररय ो क बसन की

अनमवत दी थी

bull उनह न अपन अविराजय ो म इसलाम क उपदश दन

की भी अनमवत दी थी

bull अभी हाल ही म पाोडिलागटटा (तलोगाना) क

परागवतहावसक चटटान वचतर ो क कषरण की बिती हई

घिनाएा एक गोभीर वचोता का वििय ह

bull यह परागवतहावसक चटटान क नकसान पहाचा

सकता ह

bull पाोडिलागटटा वनमनवलसखत क वलए जाना जाता ह

- 10000 ईसा पिि स 8000 ईसा पिि क वचवतरत

चटटानी आशरय ो क वलए

- राषटर कि काल क एक 8 िी ो सदी क

वशलालख क वलए और

- 12िी ो सदी क काकविय सामराजय क वभवतत

वचतर ो क वलए

Q74) उततर (b)

सपषटीकरण

bull 1828 म राजा राम म हन रॉय न एक नय िावमिक

समाज बरहम सभा की थथापना की थी वजस बाद

म बरहम समाज क नाम स जाना गया था

bull दिदरनाथ िग र न ततवब विनी सभा की अधयकषता

की थी ज आधयासिक सतय की ख ज म सोलि

थी

bull इसका उददशय वहोद िमि क शद करन का और

एकशवरिाद (एक ईशवर म आथथा) का परचार करना

था

bull नय समाज की थथापना क आिार थ कारण

(तकि ) क द सतमभ तथा िद और उपवनिद

bull अभी हाल ही म सािारण बरहम समाज का कछ

काननी मदद ो क लकर पवशचम बोगाल सरकार क

साथ काननी वििाद चल रहा ह

Q75) उततर (c)

सपषटीकरण

bull भारत म वचशती वसलवसल की थथापना खवाजा

म इनददीन वचशती क दवारा की गयी थी

bull ि 1192 ईसवी क आसपास भारत आय थ

bull वचशतीय ो क बारहिी ो शताबदी क उततरािि म भारत

म आन िाल सफीय ो क समह ो म सबस

परभािशाली माना जाता ह

bull उनह ोन थथानीय िातािरण क साथ सफलतापििक

अनकलन वकया और उनह ोन भारतीय भसकत

परोपराओो क कई पहलओो क अपनाया

bull अजमर म सफी अपरकि खवाजा म इनददीन वचशती

की ऐवतहावसक दरगाह क एक नया रप दन की

तयारी की जा रही ह

bull इस 13िी ो शताबदी की दरगाह क ldquoसवचछ

आइकॉवनक थथल ोrdquo (Swacch Iconic Places) म

शावमल वकया गया ह ज परवतवषठत विरासत

आधयासिक और साोसकवतक थथान ो पर क वदरत

य जना ह

FC19H1003 41

ANSWERS amp EXPLANATION OF

NCERT History Class VI-X + Current Affairs

(FC19E1003)

Q1) Answer c

Explanation

Rigveda consists of more than a

thousand hymns dedicated to gods and

goddesses These hymns were

composed by sages and learnt by men

however a few were composed by

women like Apala Ghosa Lopamudra

Maitreyi and Gargi

Rigveda consists of many hymns in the

form of dialogues We get an example of

a dialogue between a sage named

Vishwamitra and two rivers (Beas and

Sutlej) that were worshipped as

goddesses This suggests that he

belonged to the Vedic period

Q2) Answer b

Explanation

Traces of ash have been found from

Kurnool Caves suggesting that people

were familiar with the use of fire

It is situated in Andhra Pradesh

Q3) Answer c

Explanation

Burzahom is a prehistoric site in

present day Kashmir where people built

pit houses which were dug into the

ground with steps leading into them

These may have provided shelter in cold

weather

Q4) Answer c

Explanation

Epigraphy is defined as the study of

inscriptions

Manuscriptology is the study of history

and literature through the use of hand

written documents

Palaeography refers to the study of

ancient writing systems and the

deciphering and dating of historical

manuscripts

Numismatics refers to the study of

coins

Q5) Answer a

Explanation

Charaka Samhita was written by

Charaka and is an important book on

Ayurveda and medicine

He was a practitioner of the traditional

system of Indian medicine known as

Ayurveda

Charaka is thought to have flourished

sometime between the 2nd century BCE

and the 2nd century CE

Q6) Answer b

Explanation

Bhaga refers to the tax on crops which

was fixed at 16th of the production

Kammakaras is the term used for the

landless agricultural labour class

Ashvamedha also known as horse

sacrifice is a ritual where a horse is let

loose to wander freely and it was

guarded by the rajarsquos men

Q7) Answer (d)

Explanation

In the Rigvedic period horses were

yoked to chariots that were used in

battles fought to capture land cattle

etc This suggests that the use of horse

chariots began much before the period

of Mahajanapadas

The battles were fought in the Rigvedic

period for cattlersquos lands water an even

to capture people Most men took part

in these wars however there was no

regular army but there were assemblies

where people met and discussed

matters of war Regular armies became

a feature in the Mjahajanapada period

including vast armies of foot soldiers

chariots and elephants

RAUSIAS-FC19E1003 42

Q8) Answer (a)

Explanation

Buddha belonged to the Sakya clan and

passed away at Kusinara

Buddha taught in Prakrit which was the

common language of people

Q9) Answer c

Explanation

There were six schools of philosophy in

ancient India These are known as

Vaishesika Nyaya Samkhya Yoga

Purva Mimansa and Vedanata or Uttara

Mimansa They were founded by sages

Kanada Gautama Kapila Patanjali

Jamini and Vyasa respectively

Q10) Answer b

Explanation

The teachings of Mahavira were

compiled at Valabhi in 6th century AD

Q11) Answer (c)

Explanation

Chanakya is traditionally identified as

Kautilya or Vishnugupta who authored

the ancient Indian political treatise the

Arthashastra

Q12) Answer d

The national emblem of India is an

adaptation of the Lion Capital atop the

Ashoka Pillar of Sarnath Uttar Pradesh

and is combined with the National

Motto Satyameva Jayate

The Rampurva Bull gets the name from

the site of its discovery Rampurva in

Bihar

It is noted for its delicately sculpted

model demonstrating superior

representation of soft flesh sensitive

nostrils alert ears and strong legs It is

a mixture of Indian and Persian

elements

Sankissa is situated in Uttar Pradesh

India

Q13) Ans(a)

Kunwar Singh was a notable leader during the Revolt of 1857 He belonged

to a royal house of Jagdispur Bihar

Q14) Answer b

Explanation

The term Vellalar was used for large

landowners

Q15) Answer c

Explanation

Arikamedu was a coastal settlement

where ships unloaded goods from

distant lands Finds here include a

massive brick warehouse pottery

including amphorae and Arretine ware

Roman lamps glassware and gems have

also been found at the site

Q16) Answer a

Explanation

Muvendar is a Tamil word mentioned in

Sangam poems meaning three chiefs

used for the heads of three ruling

families the Cholas Cheras and

Pandyas

Q17) Ans (c)

Several tribal or kin-based assemblies

such as the Sabha Vidatha and gana

are mentioned in the Rig-veda The

Sabha and the samiti mattered a great

deal in early Vedic times so much so

that the chiefs or the kings showed an

eagerness to win their support

Q18) Ans (a)

Jainism recognised the existence of the

gods but placed them lower than the

jina and did not condemn the varna

system as Buddhism did

Q19) Answer (d)

Explanation

Cholas and Pandyas had developed

powerful coastal cities The most

important city of Cholas was Puhar or

Kaveripattinam and Madurai was the

capital of Pandyas

Q20) Answer b

Explanation

Buddhacharita is the biography of

Buddha and was written by

RAUSIAS-FC19E1003 43

Ashvaghosha

Q21) Answer (a)

Explanation

Tamil poet Appar was a Shiva devotee

So he was a Nayanar saint

Q22) Answer d

Explanation

Samudragupta was a prominent Gupta

ruler whose coins depict him playing a

veena indicating his love for music We

get important historic information from

his Allahabad Prashasti which was

composed by his court poet Harisena

Q23) Answer (b)

Explanation

Vikrama Samvat was founded by

Chandragupta II in the 58 BC as a

mark of victory over the Shakas and

assumed the title of Vikramaditya

Banabhatta wrote Harshavardhanarsquos

biography the Harshacharita in

Sanskrit

Q24) Answer c

Explanation

Sandhi-vigrahika was the minister of

war and peace

Sarthavaha was the leader of the

merchant caravans

Q25) Answer a

Explanation

Xuan Zang (Hsuan-tsang) was a

Chinese traveller who came during the

reign of Harshavardhana

In the decade that began in 630 AD

Xuan Zang came to India through

Kashmir after visiting Central Asia Iran

and Afghanistan

He travelled from north to east and lived

in Bihar for a couple of years

At Nalanda University Xuan Zang

interacted with students and scholars

mastered local languages and

discovered Buddhist stupas

Q26) Answer c

Explanation

Pradakshina patha is a circular path

laid around a stupa in Buddhist

architecture While the rest are a part of

temple architecture

Q27) Answer d

Explanation

All the above-mentioned temples have

an elaborate use of bricks (baked

bricks) along with stone

Q28) Ans (c)

Muhammad Quli Qutab was the Sultan

of Golconda He was a contemporary of

Akbar was very fond of literature and

architecture

The Sultan was a great poet and he

wrote in Dakhini Urdu Persian and

Telgu and has left an extensive diwan or

collection

Recently the Archaeological Survey of

India (ASI) will be using Ground

Penetrating Radar (GPR) to map the

contours of the area around the Bagh-e-

Naya Qila excavated garden inside the

Golconda Fort in Telangana

Q29) Answer a

Explanation

Silappadikaram is a famous Tamil epic

which was written by Ilango around

1800 years ago It is a story of a

merchant named Kovalan who fell in

love with a courtesan named Madhavi

Manimekalai tells the story of the

daughter of Kovalan and Madhavi

Q30) Answer (a)

Explanation

Charaka is the author of Charaka

Samhita which is an important work of

Ayurveda and medicines

Brahmaguptarsquos fame rests mostly on his

Brahma-sphuta-siddhanta which was

an astronomical work It was translated

into Arabic in Baghdad and had a major

impact on Islamic mathematics and

astronomy

Late in his life Brahmagupta wrote

Khandakhadyaka which was an

RAUSIAS-FC19E1003 44

astronomical handbook that employed

Aryabhatarsquos system of starting each day

at midnight

Q31) Answer (c)

Explanation

Amir Khusrau was a famous sufi

musician poet and scholar In 1318 he

noted that there was different language

in every region of this land (Hindustan)

Lahori Kashmiri Dvarsamudri (in

Southern Karnataka) Telangana (in

Andhra Pradesh) Gujari (in Gujarat)

Marsquobari (in Tamil Nadu) Awadhi (in

eastern Uttar Pradesh) and Hindawai (in

the area around in Delhi) etc He went

to explain that Sanskrit did not belong

to any region and that only brahmans

knew it

Q32) Answer c

Explanation

Hiranyagarbha refers to the golden

womb When this ritual was performed

with the help of Brahmanas it was

thought to lead to the rebirth of the

sacrificer as a Khastriya

Q33) Answer d

Explanation

Kadamai refers to a tax on land

revenue

Gwalior Prashasti describes the exploits

of Nagabhata who was a Pratihara king

Q34) Answer b

Explanation

Rajatarangini is a Sanskrit text written

by Kalhana in the 12th century

It was historical chronicle of early India

It is justifiably considered to be the best

and most authentic work of its kind

It covers the entire span of history in

the Kashmir region from the earliest

times to the date of its composition

Q35) Answer c

Explanation

ldquoUrrdquo was the general assembly of the

village ldquoUrrdquo consisted of all the

taxpaying residents of an ordinary

village

Q36) Answer (a)

Explanation

Tarikh was a form of history writing in

the Delhi Sultanate The authors of

tawarikhs were learned men which

included secretaries administrators etc

Q37 Answer (a)

Explanation

Alauddin chose to pay his soldiers salaries in cash rather than iqtas The soldiers would buy their supplies from merchants in Delhi and it was thus feared that merchants would raise their prices To stop this Alauddin controlled the prices of goods in Delhi Prices were carefully surveyed by officers and merchants who did not sell at the prescribed rates were punished

Q38) Answer (d)

Explanation

Delhi first became the capital of a

kingdom under the Tomara Rajputs

who were defeated in the middle of the

twelfth century by the Chauhans (also

referred to as Chahamanas) of Ajmer

It was under the Tomaras and

Chauhans that Delhi became an

important commercial centre Many rich

Jaina merchants lived in the city and

constructed several temples Coins

minted here called dehliwal had a wide

circulation

Q39) Answer (c)

Explanation

Moth ki Masjid was built in the reign of

Sikandar Lodi by his minister

Begumpuri mosque built in the reign of

Muhammad Tughluq was the main

mosque of Jahanpanah the ldquoSanctuary

of the Worldrdquo and his new capital in

Delhi

Quwwat al ndash Islam mosque was

enlarged by Iltutmish and Alauddin

Khalji The minar was built by three

Sultansndash Qutbuddin Aybak Iltutmish

and Firuz Shah Tughluq

RAUSIAS-FC19E1003 45

Q40) Answer (c)

Explanation

Under the Mughals mansabdar was

referred to an individual who held a

mansab ie rank and he received his

salary as revenue assignments called

jagirs

Q41) Ans (b)

The Quit India Movement was a

spontaneous revolt of people against

British rule

The All India Congress Committee met

at Bombay on 8 August 1942 It passed

the famous resolution Quit India and

proposed the starting of a non-violent

mass struggle under Gandhis

leadership to achieve this aim But on

the very next day Gandhi and other

eminent leaders of the Congress were

arrested The Congress was once again

declared illegal

Q42) Ans (c)

The Simon Commission refers to a

group of seven MPs from the United

Kingdom constituted to suggest

constitutional reforms for British India

The Commission consisted of only

British members headed by one of the

senior British politicians Sir John

Simon

So the people of India agitated against

the arrival of Simon Commission

Q43) Ans (a)

He was widely known for his

unfavourable opinion of the economic

consequences of the British rule in

India

In his many writings and speeches and

especially in Poverty and Un-British

Rule in India Naoroji argued that India

was too highly taxed and that its wealth

was being drained away to England

He did not interpret the ancient Indian

texts and restored the self-confidence of

Indians And also he did not stress the

need for eradication of all the social

evils before anything else

Q44) Ans (c)

In August 1932 Prime Minister

MacDonald announced his Communal

Award Great Britainrsquos unilateral

attempt to resolve the various conflicts

among Indiarsquos many communal

interests

The award which was later

incorporated into the act of 1935

expanded the separate-electorate

formula reserved for Muslims to other

minorities including Sikhs Indian

Christians Anglo-Indians Europeans

distinct regional groups Gandhi

undertook a ldquofast unto deathrdquo against

that offer which he viewed as a

nefarious British plot to divide the

Indian society

Q45) Ans (b)

In British India apart from existing

imports and exports there was also a

particular amount of money which

colonial India contributed towards

administration maintenance of the

army war expenses pensions to retired

officers and other expenses accrued by

Britain towards maintenance of her

colony These were known as Home

charges and were paid for almost

entirely by India

The Home charges was made of

following components-

- Interest payable on Indian debt

- Dividend to shareholders of East

India Company

- Funds used to support the India

Office in London

- Funds used to pay salaries and

pensions of British personnel

engaged in India

- Interest on the railways

- Civil and military charges

- Store purchases in England

Q46) Ans (b)

The Lahore session of the Indian

National Congress was held in 1929

under the Presidentship of Jawaharlal

Nehru

The Lahore session of the Indian

National Congress witnessed significant

RAUSIAS-FC19E1003 46

developments in the Indian national

movement

- First the election of Jawaharlal

Nehru to the post of Presidentship of

the Congress was a clear indication

of the growing strength of the

Leftists in the Congress

- Secondly it was in this session that

the Congress for the first time raised

the demand for complete

independence Such demand was

not raised from the Congress

platform earlier

Q47) Ans (b)

It did not provide for separate

electorates for any community or

weightage for minorities However it did

allow for the reservation of minority

seats in provinces having minorities of

at least ten per cent but this was to be

in strict proportion to the size of the

community

There was no provision for complete

Independence for India

Q48) Ans (c)

The religion of early Vedic Aryans was

primarily of worship of nature and

Yajnas

The early Aryan religion was kind of

nature worship Actually the forces

around them which they could not

control or understand were invested

with divinity and were personified as

male or female gods And they

performed some Yajnas also

Q49) Ans (b)

The roads and river-routes were not

immune from robbery It is notable that

Yuan Chwang (Hiuen Tsang) was

robbed of his belongings during

Harshvardanarsquos period

Q50) Ans (c)

Q51) Ans (b)

Purandara Dasa was a saint and great

devotee of Lord Krishna

There is much speculation about where

Purandara Dasa regarded as the

Pitamaha of Carnatic music was born

Recently an expert committee

constituted by the Kannada University

Hampi has come to the conclusion that

Kshemapura Shivamogga district

Karnataka is the birth place of

Purandara Dasa

Q52) Ans (c)

Sri Tyagaraja Sri Shyama Shastry and Sri Muthuswami Dikshitar are considered the trinity of Carnatic music and with them came the golden age in Carnatic music in the 18th-19th

century

Q53) Ans d)

Recently a rare sarcophagus (stone

coffin) which is 2000 years old from the

Iron AgendashMegalithic era was discovered

from a rock-cut cave at Viyur village of

Kollam near Koyilandy in Kozhikode

district Kerala

The coffin containing bone fragments

was found during an excavation ldquoSo

far such a rare finding has been

discovered only from two sites

in Kerala Both these sarcophagi were

recovered from Megalithic sites at

Chevayur and Atholi also in Kozhikode

district

Q54) Ans a)

The megalithic culture in South India was a full-fledged Iron Age culture

Q55) Ans d)

The Cholas Pandyas and Keralaputras

(Cheras) mentioned in Ashokan

inscriptions were probably in the late

megalithic phase of material culture

Q56) Ans d)

Q57) Ans (b)

Raj Kumar Shukla followed Gandhiji all

over the country to persuade him to

come to Champaran to investigate the

problem associated with tinkathia

system

RAUSIAS-FC19E1003 47

Brij Kishore Rajendra Prasad Mahadev

Desai and Narhari Parikh accompanied

Gandhi ji during the Champaran

Satyagraha

Q58) Ans (b)

The Satvahanas started the practice of granting tax-free villages to brahmanas and Buddhist monks

Q59) Ans c)

The objectives of the Programme are

listed as under

- Developing basic tourism

infrastructure

- Promoting cultural and heritage

value of the country to generate

livelihoods in the identified regions

- Enhancing the tourist attractiveness

in a sustainable manner by

developing world-class

infrastructure at the heritage

monument sites

- Creating employment through active

involvement of local communities

- Harnessing tourism potential for its

effects on employment generation

and economic development

- Developing sustainable tourism

infrastructure and ensuring proper

Operations and maintenance

therein

Q60) Ans (b)

The Tribal Cooperative Marketing

Development Federation of India

(TRIFED) came into existence in 1987

It is a national-level apex organization

functioning under the administrative

control of Ministry of Tribal Affairs

Govt of India

TRIFED has its registered and Head

Office located in New Delhi

Q61) Ans (c)

Premchandrsquos novels include

Premashram Rangabhumi Ghaban

Karmabhumi and Godan

Gora is a novel written by Rabindranath

Tagore

138th birth anniversary of Munshi

Premchand was celebrated across the

country

Q62) Ans (b)

Giddha is a traditional pastoral dance

performed by the women of the Punjab

India and Pakistan at festival times

and at the sowing and reaping of the

harvest

By this dance the Punjabi women

reveal their joy expel their suppressed

feelings in a male dominated society

through the performance of Giddha

Since this dance has nothing to do with

men only women can participate in it

During the Teej celebrations Giddha

dance is celebrated in Punjab every

year Teej is a generic name for a

number of festivals that are celebrated

by women in some parts of India

Q63) Ans (a)

Dara Shukoh wrote the remarkable

work called ldquoMajma-ul-Bahrainrdquo or the

ldquoThe confluence of two seasrdquo

The Vice President of India Shri M

Venkaiah Naidu has said that Prince

Dara Shukohrsquos writings can come as a

refreshing source for infusing peace and

harmony He was addressing the

gathering after visiting the exhibition

that showcases the forgotten Prince of

yesteryears Dara Shukoh organized by

Mr Francois Gautier at Indira Gandhi

National Centre for the Arts in New

Delhi

Q64) Ans (c)

The statue Gommateshwara is

dedicated to the Jain God Bahubali

It is a monolithic statue

President Ram Nath Kovind

inaugurated the grand anointing

ceremony mdash Mahamastakabhisheka mdash

held once in 12 years at

Shravanabelagola (Karnataka)

Q65) Ans (c)

Prachi Valley had come up around the

Prachi river Prachi Valley gradually

disappeared

RAUSIAS-FC19E1003 48

The Prachi river originates from

Bhubaneswar

It is a tributary of the Mahanadi and

flows through the districts of Puri

Khurda Cuttack and Jagatsinghpur

and the entire region of the river is

termed as the Prachi Valley

It falls into the Bay of Bengal

Archaeological evidence shows that the

Prachi Valley Civilisation predates both

Harappa and Mohenjo-Daro

The Prachi river originates from

Bhubaneswar

Q66) Ans (d)

These monuments are located in

Chhatarpur district Madhya Pradesh

within Vindhya mountain range

Q67) Ans (a)

The book lsquoThoughts on Pakistanrsquo was

written by Dr BR Ambedkar

On the occasion of the birth anniversary

of Dr BR Ambedkar the president of

India pays homage to this icon of India

In 1924 he founded the Depressed

Classes Institute (Bahishkrit Hitkarini

Sabha) and in 1927 the Samaj Samata

Sangh

Another area of attention for Ambedkar

was education For its spread among

the low classes he set up a network of

colleges by the name of Peoples

Education Society and founded hostels

Q68) Ans(b)

Mehrgarh is a famous Neolithic

settlement in the Indian subcontinent

which is situated in Baluchistan

province Pakistan

A pre-historic rock art site is discovered

in the vast expanse of limestone blocks

on the eastern banks of Naguleru river

near Dachepalli (Andhra Pradesh) It

has thrown light on the Neolithic

civilisation that flourished in Guntur

(Andhra Pradesh) during 1500-2000

BC

Q69) Ans (c)

The 12th and the 13th centuries saw

the emergence of the Kakatiyas They

were at first the feudatories of the

Western Chalukyas of Kalyana Initially

they ruled over a small territory near

Warangal (Telangana)

They introduced Nayakships which was

later adopted and developed by the

Rayas of Vijayanagara

Q70) Ans (a)

The fast had effect of putting pressure

on mill owners who finally agreed to

give the workers a 35 per cent increase

in wages

Google celebrated with a doodle the

132nd birth anniversary of Anasuya

Sarabhai who played a pioneering role

in Indiarsquos labour movement

Q71) Ans (d)

The UNESCOrsquos list of the representative

list of the intangible cultural heritage of

humanity from India are

- Koodiyattam Sanskrit Theatre of

Kerala

- Mudiyettu ritual theatre and dance

drama of Kerala

- Tradition of Vedic Chanting

- Kalbelia folk songs and dances of

Rajasthan

- Ramlila Traditional Performance of

the Ramayana

- Sankirtana ritual singing

drumming and dancing of Manipur

- Ramman religious festival and

ritual theatre of the Garhwal

Himalayas India

- Traditional brass and copper craft of

utensil making among the Thatheras

of Jandiala Guru Punjab India

- Chhau dance classical Indian dance

originated in the eastern Indian

states

- Buddhist chanting of Ladakh

recitation of sacred Buddhist texts

in the trans-Himalayan Ladakh

region Jammu and Kashmir India

- Yoga

- Nouroz

- Kumbh Mela

RAUSIAS-FC19E1003 49

Q72) Ans(b)

The President of India Shri Ram Nath Kovind inaugurated the Hornbill Festival and State Formation Day celebrations of Nagaland in Kisama

The festival is named after the Indian hornbill the large and colourful forest bird which is displayed in the folklore of most of the states tribes

The major recognized tribes of Nagaland are Angami Ao Chakhesang Chang

Kuki Rengma and Zeling etc

Onge Jarawa and Sentinelese are the

tribes of Andman amp Nicobar Islands

Q73) Ans (c)

The Rashtrakutas rule in the Deccan lasted for almost two hundred years till the end of the tenth century The Rashtrakutas rulers were tolerant in their religious views and patronized not only Shaivism and Vaishnavism but

Jainism as well

The famous rock-cut temple of Shiva at Ellora was built by one of the Rashtrakutas kings Krishna I in the ninth century His successor Amoghavarsha was a Jain but he also

patronized other faiths

The Rashtrakutas allowed Muslims traders to settle and permitted Islam to

be preached in their dominions

Recently increasing defacement at the prehistoric rock paintings of Pandavulagutta Telangana has created a cause for grave concern It can spoil

the prehistoric rock

Pandavulagutta is home to

- Painted rock shelters dating to

10000 BC-8000 BC

- An 8th century inscription of the

Rashtrakuta period and

- Painted frescoes from the 12th century Kakatiya empire

Q74) Ans (b)

In 1828 Raja Ram Mohan Roy founded a new religious society the Brahma Sabha later known as the Brahmo

Samaj

Debendranath Tagore headed the Tattvabodhini Sabha which was

engaged in search of spiritual truth

Its purpose was to purify Hinduism and to preach monotheism or belief in one God

The new society was to be based on the twin pillars of reason and the Vedas and

Upanishads

Recently Sadharan Brahmo Samaj (SBS) has entered into a legal battle with the West Bengal government due

to some legal issue

Q75) Ans (c)

The Chishti order was established in India by Khwaja Moinuddin Chishti who came to India around 1192 The Chishtirsquos are considered to be the most influential of the groups of Sufis who migrated to India in the late twelfth century They adapted successfully to the local environment and adopted several features of Indian devotional

traditions

The historical dargah of Sufi mystic Khwaja Moinuddin Chishti in Ajmer is all set to get a facelift This 13 th century dargah has been included among the Swachh Iconic Places a clean-up initiative focused on iconic

heritage spiritual and cultural places

Page 29: GENERAL STUDIES (PAPER I) · Test is part of Rau’s IAS Test series for Preliminary Exam 2019 FOUNDATION + CURRENT AFFAIRS GENERAL STUDIES (PAPER –I) FOUNDATION TEST –III TOPIC:

FC19H1003 29

Answers and Explanations of

NCERT History Class VI-X + Current Affairs (FC19E1003)

Q1) उततर (c)

सपषटीकरण

- ऋगवद म दविय ो और दिताओो क समवपित एक

हजार स अविक सत तर (शल क) ह

- य शल क ऋविय ो क दवारा रच गए थ और परि ो

दवारा सीख जात थ

- हालाोवक कछ शल क मवहलाओो (जस वक अपाला

घ सा ल पामदरा मतरयी और गागी) क दवारा भी रच

गए थ

- ऋगवद म सोिाद क रप म कई शल क मौजद ह

- हम विशवावमतर नामक एक ऋवि और दविय ो क

रप म पजी जान िाली द नवदय ो (वयास और

सतलज) क बीच िाताि का उदाहरण वमलता ह

- इसस पता चलता ह वक विशवावमतर िवदक काल स

सोबोवित थ

Q2) उततर (b)

सपषटीकरण

- करनल गफाओो स राख क अिशि परापत हए ह

ज इस ओर सोकत करत ह वक ततकालीन ल ग

अवि क उपय ग स पररवचत थ

- य गफाएो आोधर परदश म सथथत ह

Q3) उततर (c)

सपषटीकरण

bull बरािह म ितिमान कशमीर म सथथत एक

परागवतहावसक थथल ह जहाो ल ग गडढ क घर ो का

वनमािण करत थ

bull य घर जमीन क ख द कर बनाए जात थ तथा नीच

जान क वलए सीवियाा ह ती थी

bull ऐसा अनमान लगाया जाता ह वक य घर ठो ड क

मौसम म आशरय परदान करत थ

Q4) उततर (c)

सपषटीकरण

bull परालख-विदया (Epigraphy) क वशलालख ो क

अधययन क रप म पररभावित वकया जाता ह

bull हसतवलसखत दसतािज ो क माधयम स इवतहास

और सावहतय क अधययन क पाोडवलवप विजञान

(Manuscriptology) कहत ह

bull पराचीन लखन परणावलय ो क अधययन और

ऐवतहावसक पाोडवलवपय ो क समझन तथा वतवथ

वनिािरण क पलीओगराफी (Palaeography) कहा

जाता ह

bull नयवमजमविकस (Numismatics) वसक ो क

अधययन क सोदवभित करता ह

Q5) उततर (a)

सपषटीकरण

- चरक सोवहता चरक क दवारा वलखी गई आयिद

और िदयक-शासर पर एक महतवपणि पसतक ह

- ि भारतीय िदयक-शासर की पारमपररक परणाली

वजस आयिद क नाम स जाना जाता ह क

अभयासकताि थ

- ऐसा माना जाता ह वक चरक का विकास दसरी

शताबदी (ईसा पिि) और दसरी शताबदी (ईसवी) क

मधय हआ था

Q6) उततर (b)

सपषटीकरण

- भाग फसल ो पर वलए जान िाल कर क सोदवभित

करता ह ज कल फसल उतपादन का 16 िाो भाग

था

- ldquoकममकारrdquo शबद भवमहीन कवि शरवमक िगि क

वलए परय ग वकया जाता था

- ldquoअशवमिrdquo (वजस घ ड क बवलदान क रप म भी

जाना जाता ह) एक अनषठान ह ता था वजसम एक

घ ड क सवतोतर रप स घमन क वलए छ ड वदया

FC19H1003 30

जाता ह और राजा क सवनक उसकी रखिाली

करत थ

Q7) उततर (d)

सपषटीकरण

- ऋगववदक काल म घ ड ो क रथ ो म ज ता जाता था

ज (रथ) भवम मिवशय ो आवद पर कबजा करन क

वलए लड गए यद ो म उपय ग वकए जात थ

- इसस यह पता चलता ह वक घ ड ो यकत रथ ो का

उपय ग महाजनपद काल स काफी पहल आरमभ

हआ था

- ऋगववदक काल म मिवशय ो भवम जल आवद पर

कबजा करन क वलए तथा ल ग ो क पकडन क

वलए यद वकय जात थ

- अविकाोश परि इन यद ो म भाग वलया करत थ

- हालाोवक उस समय क ई वनयवमत सना नही ो ह ती

थी लवकन उस काल म सभाऐो ह ती थी ो वजनम

ल ग यद क मामल ो पर चचाि करत थ

- वनयवमत सनाएा महाजनपद काल का िवशषटय थी

वजनम पदल सवनक ो की विशाल सनाएा रथ तथा

हाथी शावमल ह त थ

Q8) उततर (a)

सपषटीकरण

- बद शाकय कल स सोबोवित थ और कशीनारा म

उनका वनिन हआ था

- बद न अपनी वशकषाएा पराकत भािा म दी थी ो ज

आम ल ग ो की भािा थी

Q9) उततर (c)

सपषटीकरण

- पराचीन भारत म दशिनशासर की छह शाखाएा थी ो

िशविक नयाय समखया य ग पिि वममाोसा और

िदाोत या उततर वममाोसा

- इनकी थथापना करमश कनाद गौतम कवपल

पतोजवल जावमनी और वयास ऋविय ो न की थी

Q10) उततर (b)

सपषटीकरण

महािीर की वशकषाऐो छठी शताबदी म िललभी म

सोकवलत की गई थी ो

Q11) उततर (c)

सपषटीकरण

- पारमपररक रप स चाणकय क कौविलय अथिा

विषणगपत क नाम स जाना जाता ह

- उसन अथिशासतर ज एक पराचीन भारतीय

राजनवतक आलख ह वलखा था

Q12) उततर (d)

सपषटीकरण

- भारत का राषटर ीय वचनह सारनाथ (उततर परदश) क

अश क सतमभ क ऊपर (शीिि पर) वसोह कवपिल

का एक अनरपण ह

- इस राषटर ीय वसदाोत सतयमि जयत क साथ

सोय वजत वकया गया ह

- रामपिि बल का नाम रामपिि (वबहार) क नाम पर

पडा जहाा इसकी ख ज हई थी

- यह अपन नाजक नकाशी मॉडल क वलए परवसदद

ह वजसम क मल तवचा सोिदनशील नथन ो सतकि

कान और मरबत िााग ो क शरषठतर परवतरप क

परदवशित वकया गया ह

- यह भारतीय और फारसी ततव ो का एक ससममशरण

- सोवकससा उततर परदश म सथथत ह

Q13) उततर (a)

सपषटीकरण

का िर वसोह ज एक महान य दा थ वबहार स

सोबोवित थ

Q14) उततर (b)

सपषटीकरण

िललालर शबद बड भ-सवावमय ो क वलए परय ग

वकया जाता था

FC19H1003 31

Q15) उततर (c)

सपषटीकरण

- अररकमड एक तिीय बसती थी जहाो दर दश ो स

आन िाल जहाज ो का माल उतारा जाता था

- यहाो पर ईोि ो का एक विशाल ग दाम वमटटी क

बतिन (वजनम एमफ रा - द हरी मवठय ो का लोबा

घडा - शावमल ह) और एरिाइन (Arretine)

मदभाोड पाए गए थ

- इस थथान पर र मन दीपक काोच क बन पातर और

रतन भी पाए गए थ

Q16) उततर (a)

सपषटीकरण

- मिनदर सोगम कविताओो म उसललसखत एक

तवमल शबद ह वजसका अथि ह ldquoतीन परमखrdquo

- यह तीन सततारि पररिार ो क मसखयाओो क वलए

परय ग वकया जाता ह च ल चर और पाणडय

Q17) उततर (c)

सपषटीकरण

- ऋग िद म सभा विदाथा तथा गण जसी

जनजावतय ो पर अथिा किोब पर आिाररत

सभाओो का उललख ह

- आरसमभक िवदक काल म सभाओो और सवमवतय ो

का विशि महतव ह ता था

- यहाा तक की मसखया अथिा राजा भी उनका

समथिन परापत करन क वलए आतर रहत थ

Q18) उततर (a)

सपषटीकरण

- जन िमि न ईशवर क अससततव क मानयता त दी ह

वकनत उसन ईशवर क वजना क पद स नीच रखा

- जन िमि न बौद िमि की तरह िणि परणाली की

भरतिना नही ो की थी

Q19) उततर (d)

सपषटीकरण

- च ल ो और पाणडय ो न शसकतशाली तिीय शहर ो का

विकास वकया था

- च ल ो का सबस महतवपणि शहर पहार (या

कािरीपटटीनम) था |

- मदरई पाणडय ो की राजिानी थी

Q20) उततर (b)

सपषटीकरण

- ldquoबदचररतrdquo बद का जीिन-ितताोत ह

- इस अशवघ ि क दवारा वलखा गया था

Q21) उततर (a)

सपषटीकरणः

- तवमल कवि अपपर भगिान वशि क भकत थ

- इस परकार ि एक नयनार सोत थ

Q22) उततर (d)

सपषटीकरणः

- समदरगपत एक परवसद गपत शासक था

- उसन वसक ो पर िीणा बजात हए अपनी छवि

अोवकत करिाई थी

- यह सोगीत क परवत उसक परम क दशािता ह

- हम उसकी इलाहाबाद परशससत स महतवपणि

ऐवतहावसक जानकारी वमलती ह वजसकी रचना

उसक दरबार क कवि हररसन न की थी

Q23) उततर (b)

सपषटीकरणः

- विकरम सोित की शरआत ििि 58 ईसा पिि म

चनदरगपत वदवतीय न की थी

- यह शक ो पर उसकी जीत और उस विकरमावदतय

की पदिी वमलन क उपलकषय म आरमभ वकया गया

था

FC19H1003 32

- बानभटट न हिििििन का जीिन-ितताोत हििचररत

(ज सोसकत म थी) वलखी थी

Q24) उततर (c)

सपषटीकरणः

- सोवि-विगरावहका यद एिो शाोवत का मोतरी

- साथििाह वयापाररय ो क कावफल ो का नता

Q25) उततर (a)

सपषटीकरणः

- जआन झाोग (हसआन रताोग ndash Hsuang Tsang)

एक चीनी यातरी था ज हिििििन क शासनकाल म

भारत आया था

- ििि 630 ईसवी स ज दशक आरमभ हआ था उसम

जआन झाोग मधय एवशया ईरान और

अफग़ावनसतान की यातरा करन क पशचात कशमीर

क रासत स भारत आया था

- उसन उततर स पिि तक की यातरा की और िह

लगभग 2 ििि वबहार म रहा

- जआन झाोग न नालनदा विशवविदयालय म विदयावथिय ो

और विदवान ो क साथ पारसपररक विचार-विमशि

वकया थथानीय भािाओ ा म वनपणता परापत की तथा

बौद सतप ो की ख ज की

Q26) उततर (c)

सपषटीकरणः

- परदवकषणा पथ बौद िासतकला म सतप क चार ो

ओर बनाया जान िाला एक घमािदार पथ ह ता

- परशन म वदए गए बाकी क तीन ो ततव वहोद मसनदर ो की

िासतकला क भाग ह

Q27) उततर (d)

सपषटीकरणः

परशन म वदए गए सभी मोवदर ो म वयापक रप स

ईोि ो (पकी ईोि ो) का परय ग पतथर ो क साथ हआ

Q28) उततर (c)

सपषटीकरण

- महममद कली कतब शाह ग लकणडा का सलतान

था

- िह अकबर का समकालीन था

- सावहतय और िासतकला म उसकी अतयाविक

रवच थी

- िह एक महान कवि था

- िह दसखनी उदि फारसी और तलग म वलखता था

- उसन अपन पीछ एक विसतत वदिान (सोगरह)

छ डा ह

- अभी हाल ही म तलोगाना म ग लकणडा क वकल

क अनदर खदाई वकय गए बाग-ए-नाया वकला

बाग क चार ो ओर रप-रखा क मानवचतरण क

वलए भारतीय परातासतवक सिकषण (The

Archaeological Survey of India ndash ASI)

गराउणड पनीिर विोग रडार (Ground Penetrating

Radar) का परय ग करगा

Q29) उततर (a)

सपषटीकरणः

- वसलपपावदकारम एक तवमल महाकावय ह वजसकी

रचना इलाोग क दवारा लगभग 1800 ििि पिि की

गई थी

- यह क िलन नामक एक वयापारी की कहानी ह

ज माििी नामक एक गवणका (िशया) स परम

करन लगा था

- मवनमकलाई क िलन और माििी की पतरी की

कहानी ह

Q30) उततर (a)

सपषटीकरण

- चरक आयिद और वचवकरता की एक महतवपणि

रचना चरक सोवहता क लखक ह

- बरहमगपत क अपनी रचना बरहम-सफि-वसदानत

(ज एक खग लीय रचना ह) क कारण परवससद

वमली

FC19H1003 33

- बगदाद म इसका अनिाद अरबी भािा म वकया

गया था

- इसका इसलावमक गवणत और खग ल-विजञान पर

महतवपणि परभाि पडा था

- बाद म अपन जीिनकाल म बरहमगपत न

ldquoखोडखयाकrdquo वलखी ज एक खग लीय पससतका

(एक छ िी पसतक) थी

- इसम आयिभटट की अिि-रावतर क परतयक वदन की

शरआत परणाली का परय ग वकया गया था

Q31) उततर (c)

सपषटीकरण

- अमीर खसर एक परवसद सफी सोगीतकार कवि

और विदवान थ

- 1318 म उनह ोन पाया वक इस भवम (वहोदसतान) क

हर कषतर म अलग-अलग भािा थी लाहौरी

कशमीरी दवारसमदरी (दवकषणी कनाििक म)

तलोगाना (आोधर परदश म) गजरी (गजरात म)

माबारी (तवमलनाड म ) अििी (पिी उततर परदश

म) और वहोदिी (वदलली क आस-पास क कषतर म)

आवद

- उनह न यह बताया वक सोसकत वकसी भी कषतर स

सोबोवित नही ो थी और किल बराहमण ही इस भािा

का जञान रखत थ

Q32) उततर (c)

सपषटीकरण

- वहरणय-गभि सववणिम गभि क सोदवभित करता ह

- जब बराहमण ो की सहायता स यह अनषठान वकया

जाता था त यह माना जाता था वक बवल दन िाल

का कषवतरय क रप म पनजिनम ह गा

Q33) उततर (d)

सपषटीकरण

- कदमई भवम राजसव पर कर क सोदवभित करता

- गवावलयर परशससत म नागभि क दवारा वकय गए

श िण का िणिन वकया गया ह |

- नागभि एक परवतहार राजा था

Q34) उततर (b)

सपषटीकरण

- राजतरो वगनी 12िी ो शताबदी म कलहन क दवारा

रवचत एक सोसकत पसतक (िकसट) ह

- यह परारसमभक भारत की ऐवतहावसक इवतितत थी

- तकि सोगत रप स इस अपन परकार की सिोततम

और सिािविक विशवसनीय कवत माना जाता ह

- यह कशमीर कषतर क पराचीनतम समय स लकर

उसकी रचना की तारीख तक क समपणि इवतहास

का आचछादन करती ह

Q35) उततर (c)

सपषटीकरण

- गााि की आम सभा क ldquoउरrdquo कहा जाता था

- ldquoउरrdquo म गााि क सभी कर दन िाल वनिासी

शावमल ह त थ

Q36) उततर (a)

सपषटीकरण

- वदलली सलतनत म ldquoतारीखrdquo इवतहास लखन का

एक रप था

- ldquoतािरीखrdquo क लखक विदवान परि ह त थ वजनम

सवचि परशासक इतयावद शावमल थ

Q37) उततर (a)

सपषटीकरण

- अलाउददीन सखलजी अपन सवनक ो क ितन का

भगतान नकद म करता था न वक इकता क रप

- सवनक अपना सामान वदलली म वयापाररय ो स

खरीदत थ अतः इस बात का भय था वक वयापारी

कही ो िसतओो का मलय न बिा द

- इसकी र कथाम क वलए अलाउददीन सखलजी न

वदलली म कीमत ो क वनयसित वकया

FC19H1003 34

- अविकारीगण धयानपििक मलय ो का सिकषण करत

थ तथा ज वयापारी वनिािररत मलय पर माल नही ो

बचत थ उनक दसणडत वकया जाता था

Q38) उततर (d)

सपषटीकरण

- वदलली सििपरथम त मर राजपत ो क अिीन उनक

सामराजय की राजिानी बनी थी

- 12िी ो शताबदी क मधय म अजमर क चौहान ो

(वजनह चाहमान ो क नाम स भी जाना जाता ह) न

त मर राजपत ो क परावजत वकया था

- त मर ो और चौहान ो क अिीन वदलली एक

महतवपणि िावणसजयक क दर बन गया था

- कई जन वयापारी यहाा रहन लग थ और उनह ोन

कई मोवदर भी बनिाए

- यहाा पर मवदरत वसक वजनह ldquoदहलीिालrdquo क नाम

स जाना जाता था वयापक रप स परचलन म थ

Q39) उततर (c)

सपषटीकरण

- म ठ की मसिद का वनमािण वसको दर ल दी क

राजयकाल म उसक मिी क दवारा करिाया गया

था

- बगमपरी मसिद का वनमािण महममद तगलक क

शासनकाल म हआ था

- यह मसिद विशव का पणयथथान (The

Sanctuary of the World) और वदलली म महममद

तगलक की नई राजिानी जहाोपनाह की मखय

मसिद थी

- कववत- अल - इसलाम मसिद का विसतार

इलतसिश और अलाउददीन सखलजी न वकया था

- मीनार का वनमािण तीन सलतान ो कतबददीन ऐबक

इलतसिश और वफर ज शाह तगलक क दवारा

करिाया गया था

Q40) उततर (c)

सपषटीकरण

- मगल ो क अिीन मनसबदार शबद उस वयसकत क

वलए सोदवभित वकया जाता था वजसक पास मनसब

(अथाित पद) ह ता था

- उस अपना ितन राजसव कायो वजनह जागीर कहत

थ क रप म परापत ह ता था

Q41) उततर (b)

सपषटीकरण

- ldquoभारत छ ड आोद लनrdquo वबरविश शासन क

सखलाफ ल ग ो का एक सवाभाविक विदर ह था

- असखल भारतीय काोगरस सवमवत न 8 अगसत 1942

क बमबई म एक बठक का आय जन वकया था

- इस बठक म परवसद सोकलप ldquoभारत छ ड rdquo क

पाररत वकया गया और इस उददशय क परापत करन

क वलए गाोिी क नततव म एक अवहोसक जन सोघिि

आोद लन की शरआत का परसताि वदया गया

- लवकन अगल ही वदन गाोिी और काोगरस क अनय

परमख नताओो क वगरफतार कर वलया गया

- काोगरस क एक बार वफर अिि घ वित वकया गया

था

Q42) उततर (c)

सपषटीकरण

- साइमन कमीशन यनाइविड वको गडम क सात

साोसद ो का एक समह था

- इस वबरविश भारत क वलए सोििावनक सिार ो का

सझाि दन क वलए गवठत वकया गया था

- इस आय ग म िररषठ वबरविश राजनता सर जॉन

साइमन क नततव म किल वबरविश सदसय ही

शावमल थ

- इसवलए भारत क ल ग ो न साइमन कमीशन क

आगमन क विरद आोद लन वकया था

Q43) उततर (a)

सपषटीकरण

bull दादा भाई नौर जी भारत म वबरविश शासन क

आवथिक पररणाम ो क बार म अपनी विर िी

(परवतकल) राय क वलए जान जात थ

FC19H1003 35

bull अपन कई लख ो और भािण ो म विशि रप स

ldquoपाििी एो ड अन-वबरविश रल इन इसणडया

(Poverty and Un-British Rule in India) म

नौर जी न यह तकि वदया वक भारत पर अतयविक

कर लगाया गया था और इसकी सोपवतत इोगलड की

ओर परिावहत की जा रही थी

bull उनह ोन पराचीन भारतीय गरोथ ो की वयाखया करन

का और भारतीय ो क आिविशवास क बहाल

करन पर कायि नही ो वकया था

उनह ोन वकसी और बात स पहल सभी सामावजक

बराइय ो क उनमलन की आिशयकता पर भी बल

नही ो वदया था

Q44) उततर (c)

सपषटीकरण

bull अगसत 1932 म वबरविश परिानमोतरी मकड नालड न

अपन साोपरदावयक परसकार (The Communal

Award) की घ िणा की थी

bull यह भारत क कई साोपरदावयक वहत ो क बीच विवभनन

सोघिो क हल करन क वलए वबरिन का एकतरफा

परयास था

bull यह परसकार (Award) बाद म 1935 क

अविवनयम (The Act of 1935) म शावमल वकया

गया था

bull इस साोपरदावयक परसकार न मससलम ो क वलए

आरवकषत एक अलग वनिािचक मणडल फॉमिल का

विसतार अनय अलपसोखयक ो क वलए वकया था

वजसम वसख ो भारतीय ईसाइय ो आोगल-भारतीय

समदाय यर पीय समदाय तथा विवशषट कषतरीय

समह ो क शावमल वकया गया था

bull गाोिी न इस परसताि क भारतीय समाज क

विभावजत करन क वलए एक घवणत वबरविश

सावजश क रप म दखा और उसक सखलाफ

आमरण अनशन वकया

Q45) उततर (b)

सपषटीकरण

मौजदा आयात और वनयाित क अवतररक़त

औपवनिवशक भारत क वनमनवलसखत खचो क

वलए एक विशिवनवशचत िन रावश भी दनी पडती

थी

(i) परशासन क वयय

(ii) सना क रख-रखाि क वयय

(iii) यद क वयय

(iv) सिावनितत अविकाररय ो की पशन तथा

(v) वबरिन दवारा अपनी उपवनिश बसती

(कॉल नी) क रख-रखाि क वयय

इनह गह शलक (Home Charges) क रप म

जाना जाता था और लगभग परी तरह स भारत क

दवारा इनका भगतान वकया जाता था

bull गह शलक म वनमनवलसखत घिक शावमल थ

(i) भारतीय ऋण पर दय बयाज

(ii) ईसट इोवडया को पनी क शयरिारक ो क

लाभाोश

(iii) लोदन म भारत कायािलय चलान क वलए िन

(iv) भारत म वनयकत वबरविश कवमिय ो क ितन

और पशन का भगतान करन क वलए िन

(v) रलि पर बयाज

(vi) नागररक और सनय शलक

(vii) इोगलड म सट र (सामगरी) की खरीद

Q46) उततर (b)

सपषटीकरण

bull भारतीय राषटर ीय काोगरस का लाहौर सतर 1929 म

जिाहरलाल नहर की अधयकषता म आय वजत

वकया गया था

bull इस सतर म भारतीय राषटर ीय आोद लन स समबसित

कई महतवपणि पररणाम सामन आय थ

(i) सििपरथम इस सतर म काोगरस क अधयकष पद

पर जिाहरलाल नहर क चना गया था ज

काोगरस म िामपोवथय ो की बिती हई ताकत

का सपषट सोकत था

(ii) दसरा इस सतर म पहली बार काोगरस न पणि

सवतोतरता की माोग क उठाया था

इस परकार की माोग काोगरस मोच स पहल कभी भी

नही ो उठाई गई थी

Q47) उततर (b)

सपषटीकरण

FC19H1003 36

bull इस ररप िि न वकसी भी समदाय क वलए पथक

वनिािचक मोडल अथिा अलपसोखयक ो क वलए

भाराोश की वसफाररश नही ो की थी

bull तथावप इस ररप िि न उन पराोत ो म अलपसोखयक

सीि ो क आरकषण की अनमवत दी थी जहाा पर कम

स कम दस परवतशत अलपसोखयक ह

bull लवकन यह समदाय क आकार क अनपात म ह ना

चावहए था

bull इस ररप िि म भारत क वलए पणि सवतोतरता क

वलए क ई पराििान नही ो था

Q48) उततर (c)

सपषटीकरण

bull आरो वभक िवदक आयो का िमि मखय रप स

परकवत की पजा और यजञ था

bull परारो वभक आयि िमि परकवत की पजा क समान था

bull िासति म उनक चार ो ओर की शसकतयाा वजनह न

त ि वनयोवतरत कर सकत थ और न ही समझ पाए

थ उनह वदवयता क साथ वनिवशत वकया गया तथा

उनह मादा या नर दिीदिताओो क रप म

परतीकतव वकया गया था

bull उनह ोन कछ यजञ ो का भी वनषपादन वकया था

Q49) उततर (b)

सपषटीकरण

bull सडक और नदी-मागि (जल-मागि) डकती स

सरवकषत नही ो थ

bull उललखनीय ह वक हिििििन क शासनकाल क

दौरान यआन चिाोग (हयएन साोग) का सारा

सामान लि वलया गया था

Q50) उततर (c)

सपषटीकरण

परशन म वदए गए द न ो कथन सही ह

Q51) उततर (b)

सपषटीकरण

bull परोदर दास एक सोत और भगिान कषण क एक

महान भकत थ

bull परोदर दास क कनाििक सोगीत क वपतामह क

रप म जाना जाता ह

bull यदयवप उनक जनम-थथान क बार म काफी

अिकल लगाई जाती रही ह

bull तथावप अब कननड विशवविदयालय हमपी क दवारा

गवठत एक विशिजञ सवमवत इस वनषकिि पर पहोची

ह वक उनका जनम थथान सोभितया कनाििक का

एक छ िा-सा गााि कषमपरा (वशिम गगा वजला)

था

Q52) उततर (c)

सपषटीकरण

bull शरी तयागराज शरी शयाम शासतरी और शरी मथसवामी

दीवकषतर क कनाििक सोगीत की वतरमवति माना

जाता ह

bull उनक कारण ही 18िी ो-19िी ो शताबदी म कनाििक

सोगीत का सववणिम यग आया था

Q53) उततर (d)

सपषटीकरण

bull अभी हाल ही म लौह यगीन-महापािावणक काल

का 2000 ििि पराना एक दलिभ सारक फगस

(Sarcophagus) (पतथर का ताबत) क ललम क

वियर गाोि (क वयलडी क पास वजला क वझक ड

करल राजय) की एक रॉक-कि गफा स ख जा गया

bull यह ताबत वजसम हविय ो क िकड थ खदाई क

दौरान वमला

bull अभी तक इस परकार की दलिभ ख ज करल क

मातर द ही थथान ो स हई ह

bull य द न ो सारक फगी (Sarcophagi) (पतथर क

ताबत) चियर और अथ ली (वजला क वझक ड) क

महापािाण थथल ो स वमल ह

Q54) उततर (a)

सपषटीकरण

FC19H1003 37

दवकषण भारत म महापािाण सोसकवत एक पणि

विकवसत लौह यगीन सोसकवत थी

Q55) उततर (d)

सपषटीकरण

bull च ल पाणडय और करलपतर (चर) इन तीन ो का

उललख अश क क अवभलख ो म वकया गया ह

bull सोभितः य भौवतक सोसकवत क उततर

महापािावणक चरण म थ

Q56) उततर (d)

सपषटीकरण

bull भीमा-क रगाोि की लडाई ततीय आोगल-मराठा

यद का वहससा थी

Q57) उततर (b)

सपषटीकरण

bull राजकमार शकल न गाोिीजी क चोपारण आन तथा

वतनकवथया परणाली स जडी समसया की जाोच क

वलए रारी करन क वलए दश भर म उनका

अनसरण वकया था

bull बज वकश र राजदर परसाद महादि दसाई और

नरहरी पाररख चोपारण सतयागरह क दौरान गाोिी

जी क सहय गी थ

Q58) उततर (b)

सपषटीकरण

bull बराहमण ो और बौद मठिाररय ो क कर-मकत गााि

अनदान म दन की परथा सतिाहन ो न आरमभ की

थी

Q59) उततर (c)

सपषटीकरण

इस कायिकरम क उददशय वनमनानसार ह

(i) बवनयादी पयििन आिाररक सोरचना का विकास

करना

(ii) चयवनत (पहचान वकय गए) कषतर ो म आजीविका क

सजन क वलए दश क साोसकवतक और विरासत

मलय ो क बिािा दना

(iii) विरासत समारक थथल ो पर विशव सतरीय आिाररक

सोरचना विकवसत करक एक सतत तरीक स

पयििक आकििण म िसद करना

(iv) थथानीय समदाय ो की सवकरय भागीदारी क माधयम

स र रगार ो का सजन करना

(v) र रगार उतपादन और आवथिक विकास क वलए

पयििन कषमता का उन पर परभाि का उपय ग

करना तथा

(vi) िारणीय पयििन आिाररक सोरचना का विकास

करना और उसका उवचत सोचालन तथा

रखरखाि सवनवशचत करना

Q60) उततर (b)

सपषटीकरण

bull यह वनकाय ििि 1987 म अससततव म आया था

bull यह एक राषटर ीय सतर का शीिि सोगठन ह ज भारत

सरकार क जनजातीय मामल ो क मोतरालय क

परशासवनक वनयोतरण क अिीन काम कर रहा ह

bull इसका पोजीकत और परिान कायािलय नई वदलली

म सथथत ह

Q61) उततर (c)

सपषटीकरण

bull परमचोद क उपनयास ो म परमाशरम रोगभवम गबन

कमिभवम और ग दान शावमल ह

bull ग रा रिी ोदरनाथ िग र क दवारा रवचत उपनयास ह

bull अभी हाल ही म मोशी परमचोद की 138िी ो जयोती दश

भर म मनाई गई थी

Q62) उततर (b)

सपषटीकरण

bull ldquoवगदाrdquo पोजाब (भारत) एिो पावकसतान की

मवहलाओो क दवारा तयौहार क समय और फसल

की बिाई तथा किाई क अिसर पर वकया जान

िाला एक पारोपररक दहाती नतय ह

FC19H1003 38

bull इस नतय क माधयम स पोजाबी मवहलाऐो अपनी

परसननता परकि करती ह तथा वगदा क परदशिन क

माधयम स परि िचिसव िाल समाज म मवहलाओो

की दबी हई भािनाओो क परकि करती ह

bull चोवक इस नतय का परि ो क साथ क ई सोबोि नही ो

ह अतः किल मवहलाऐो ही इसम भाग ल सकती

bull हर साल तीज समार ह क दौरान पोजाब म वगदा

नतय वकया जाता ह

तीज भारत क कछ भाग ो म मवहलाओो क दवारा

मनाया जान िाल कई तयौहार ो क वलए एक

वयापक नाम ह

Q63) उततर (a)

सपषटीकरण

- मजम-उल-बहरीन या द समदर ो का सोगम

नामक उललखनीय रचना दारा वशक ह क दवारा

वलखी थी

- भारत क उपराषटर पवत शरी एम िकया नायड न कहा

ह वक राजकमार दारा वशक ह की रचनाएा शाोवत

और सदभाि क बिािा दन क वलए एक तारा सर त

क रप म सामन आ सकती ो ह

- उपराषटर पवत गत ििो क भला वदए गए राजकमार

दारा वशक ह क परदवशित परचवलत करन हत

आय वजत एक परदशिनी का दौरा करन क बाद एक

सभा क सोब वित कर रह थ

- इस परदशिनी का आय जन फर क इस गौवियर

(Francois Gautier) क दवारा lsquoइोवदरा गाोिी नशनल

सिर फॉर द आििसrsquo (The Indira Gandhi

National Centre for the Arts) नई वदलली म

वकया गया था

Q64) उततर (c)

सपषटीकरण

- ग मतशवर परवतमा जन भगिान बाहबली क

समवपित ह

- यह एक एक-चटटानी पतथर की मवति ह

- राषटर पवत राम नाथ क विोद न शरिणबलग ला

(कनाििक) म आय वजत वकय जान िाल भवय

अवभिक समार ह महामसतकावभिक का

उदघािन वकया था

- यह समार ह 12 ििो म एक बार ह ता ह

Q65) उततर (c)

सपषटीकरण

bull पराची घािी पराची नदी क चार ो ओर फली हई थी

bull पराची घािी िीर-िीर विलपत ह गई थी

bull पराची नदी भिनशवर स वनकलती ह

bull यह महानदी की एक सहायक नदी ह और यह

परी खदाि किक तथा जगतवसोहपर वजल ो स

ह कर बहती ह

bull इस नदी क पर कषतर क पराची घािी कहा जाता ह

bull यह नदी बोगाल की खाडी म वगरती ह

परातासतवक साकषय स पता चलता ह वक पराची घािी

सभयता हडपपा और म हनज दाड द न ो की

पिििती ह

Q66) उततर (d)

सपषटीकरण

य समारक छतरपर वजल (मधय परदश) म विोधयाचल

पिित शरोखला म सथथत ह

Q67) उततर (a)

सपषटीकरण

bull थॉिस ऑन पावकसतान नामक पसतक डॉ बी

आर अमबडकर न वलखी थी

bull डॉ बी आर अमबडकर की जयोती क अिसर पर

भारत क राषटर पवत न भारत की इस महान हसती

क शरदाोजवल अवपित की थी

bull डॉ बी आर अमबडकर न 1924 म वडपरथड

कलावसर इोसटीटयि (दवलत िगि सोथथान -

बवहषकत वहतकाररणी सभा) और 1927 म समाज

समता सोघ की थथापना की थी

bull अमबडकर का धयान वशकषा कषतर की ओर भी था

bull उनह ोन वशकषा क वनमन िगो म फलान क वलए

पीपलस एजकशन स साइिी (The Peoples

Education Society) क नाम स महाविदयालय ो क

नििकि और छातरािास ो की थथापना की थी

FC19H1003 39

Q68) उततर (b)

सपषटीकरण

bull महरगि भारतीय उपमहादवीप म एक परवसद

निपािाण बसती ह ज बलवचसतान पराोत

पावकसतान म सथथत ह

bull दचपलली (आोधर परदश) क पास नागलर नदी क

पिी ति ो पर चना पतथर क बलॉक क विशाल

विसतार म एक पिि-ऐवतहावसक रॉक आिि थथल की

ख ज की गई ह

bull इसन 1500-2000 ईसा पिि क दौरान गोिर (आोधर

परदश) म विकवसत निपािाण सभयता पर परकाश

डाला ह

Q69) उततर (c)

सपषटीकरण

bull 12िी ो सदी और 13िी ो सदी म काकाविय िोश का

उदय हआ था

bull ि पहल कलयाण क पवशचमी चालकय ो क सामोत थ

bull परारोभ म उनह ोन िारोगल (तलोगाना) क पास एक

छ ि स कषतर पर शासन वकया था

bull उनह ोन ldquoनायक वयिथथाrdquo की शरआत की थी

वजस बाद म विजयनगर क राय शासक ो न

अपनाया और विकवसत वकया था

Q70) उततर (a)

सपषटीकरण

bull गाोिीजी क अनशन स वमल मावलक ो पर दबाि

पडा था ज अोततः शरवमक ो क ितन म 35 परवतशत

की िसद करन क वलए सहमत हए थ

bull गगल (Google) न अनसया साराभाई वजनह ोन

भारत क शरवमक आोद लन म एक अगरणी भवमका

वनभाई थी की 132िी ो जयोती डडल (Doodle) का

वनमािण करक मनाई

Q71) उततर (d)

सपषटीकरण

भारत स यनसक की मानिता की अमति साोसकवतक

विरासत की परवतवनवि सची म वनमनवलसखत शावमल ह

bull कवडयटटम करल का सोसकत रोगमोच

bull मवडयिि करल का अनषठान रोगमोच और नतय

नाविका

bull िवदक मि जाप की परोपरा

bull राजथथान क कालबवलया ल क गीत और नतय

bull रामलीला रामायण का पारोपररक परदशिन

bull सोकीतिन मवणपर का अनषठान गायन ढ ल िादन

और नतय

bull रममन भारत क गििाल वहमालय का िावमिक

तयौहार और अनषठान रोगमोच

bull जाोदीयाला गर पोजाब क ठठर ो की पीतल और

ताोब क वशलप स वनवमित बतिन ो की पारोपररक कला

bull छाऊ नतय पिी भारतीय राजय ो म जनमी शासतरीय

भारतीय नतय कला

bull लददाख का बौद मि जाप िर ाोस-वहमालयी लददाख

कषतर तथा जमम-कशमीर म पवितर बौद गरोथ ो का पाठ

bull य ग

bull नौर र

bull को भ मला

Q72) उततर (b)

सपषटीकरण

bull भारत क राषटर पवत शरी राम नाथ क विोद न

वकसामा नागालड म हॉनिवबल मह रति और

राजय गठन वदिस समार ह का उदघािन वकया

था

bull हॉनिवबल मह रति का नाम भारतीय हॉनिवबल क

नाम पर पडा ह ज एक विशाल और रोगीन जोगली

पकषी ह

bull यह पकषी नागालड राजय की अविकतर जनजावतय ो

की ल ककथाओो म उसललसखत ह

bull नागालड की परमख मानयता परापत जनजावतयाा ह

अोगामी आओ चखसोग चाोग ककी रगमा और

रवलोग आवद

bull ओोग जारिा और ससिनलीस अोडमान-वनक बार

दवीप समह की जनजावतयाा ह

FC19H1003 40

Q73) उततर (c)

सपषटीकरण

bull दकन म राषटर कि शासन दसिी ो सदी क अोत तक

लगभग 200 ििो तक रहा था

bull राषटर कि शासक अपन िावमिक विचार ो म सवहषण

bull उनह ोन न किल शि िमि और िषणि िमि बसलक

जन िमि क भी सोरकषण वदया था

bull एल रा म वशि क परवसद रॉक कि मोवदर का

वनमािण नौिी ो सदी म राषटर कि राजा कषण परथम न

करिाया था

bull उसका उततराविकारी अम घििि जन था लवकन

उसन अनय िमो क भी सोरकषण परदान वकया था

bull राषटर कि ो न मसलमान वयापाररय ो क बसन की

अनमवत दी थी

bull उनह न अपन अविराजय ो म इसलाम क उपदश दन

की भी अनमवत दी थी

bull अभी हाल ही म पाोडिलागटटा (तलोगाना) क

परागवतहावसक चटटान वचतर ो क कषरण की बिती हई

घिनाएा एक गोभीर वचोता का वििय ह

bull यह परागवतहावसक चटटान क नकसान पहाचा

सकता ह

bull पाोडिलागटटा वनमनवलसखत क वलए जाना जाता ह

- 10000 ईसा पिि स 8000 ईसा पिि क वचवतरत

चटटानी आशरय ो क वलए

- राषटर कि काल क एक 8 िी ो सदी क

वशलालख क वलए और

- 12िी ो सदी क काकविय सामराजय क वभवतत

वचतर ो क वलए

Q74) उततर (b)

सपषटीकरण

bull 1828 म राजा राम म हन रॉय न एक नय िावमिक

समाज बरहम सभा की थथापना की थी वजस बाद

म बरहम समाज क नाम स जाना गया था

bull दिदरनाथ िग र न ततवब विनी सभा की अधयकषता

की थी ज आधयासिक सतय की ख ज म सोलि

थी

bull इसका उददशय वहोद िमि क शद करन का और

एकशवरिाद (एक ईशवर म आथथा) का परचार करना

था

bull नय समाज की थथापना क आिार थ कारण

(तकि ) क द सतमभ तथा िद और उपवनिद

bull अभी हाल ही म सािारण बरहम समाज का कछ

काननी मदद ो क लकर पवशचम बोगाल सरकार क

साथ काननी वििाद चल रहा ह

Q75) उततर (c)

सपषटीकरण

bull भारत म वचशती वसलवसल की थथापना खवाजा

म इनददीन वचशती क दवारा की गयी थी

bull ि 1192 ईसवी क आसपास भारत आय थ

bull वचशतीय ो क बारहिी ो शताबदी क उततरािि म भारत

म आन िाल सफीय ो क समह ो म सबस

परभािशाली माना जाता ह

bull उनह ोन थथानीय िातािरण क साथ सफलतापििक

अनकलन वकया और उनह ोन भारतीय भसकत

परोपराओो क कई पहलओो क अपनाया

bull अजमर म सफी अपरकि खवाजा म इनददीन वचशती

की ऐवतहावसक दरगाह क एक नया रप दन की

तयारी की जा रही ह

bull इस 13िी ो शताबदी की दरगाह क ldquoसवचछ

आइकॉवनक थथल ोrdquo (Swacch Iconic Places) म

शावमल वकया गया ह ज परवतवषठत विरासत

आधयासिक और साोसकवतक थथान ो पर क वदरत

य जना ह

FC19H1003 41

ANSWERS amp EXPLANATION OF

NCERT History Class VI-X + Current Affairs

(FC19E1003)

Q1) Answer c

Explanation

Rigveda consists of more than a

thousand hymns dedicated to gods and

goddesses These hymns were

composed by sages and learnt by men

however a few were composed by

women like Apala Ghosa Lopamudra

Maitreyi and Gargi

Rigveda consists of many hymns in the

form of dialogues We get an example of

a dialogue between a sage named

Vishwamitra and two rivers (Beas and

Sutlej) that were worshipped as

goddesses This suggests that he

belonged to the Vedic period

Q2) Answer b

Explanation

Traces of ash have been found from

Kurnool Caves suggesting that people

were familiar with the use of fire

It is situated in Andhra Pradesh

Q3) Answer c

Explanation

Burzahom is a prehistoric site in

present day Kashmir where people built

pit houses which were dug into the

ground with steps leading into them

These may have provided shelter in cold

weather

Q4) Answer c

Explanation

Epigraphy is defined as the study of

inscriptions

Manuscriptology is the study of history

and literature through the use of hand

written documents

Palaeography refers to the study of

ancient writing systems and the

deciphering and dating of historical

manuscripts

Numismatics refers to the study of

coins

Q5) Answer a

Explanation

Charaka Samhita was written by

Charaka and is an important book on

Ayurveda and medicine

He was a practitioner of the traditional

system of Indian medicine known as

Ayurveda

Charaka is thought to have flourished

sometime between the 2nd century BCE

and the 2nd century CE

Q6) Answer b

Explanation

Bhaga refers to the tax on crops which

was fixed at 16th of the production

Kammakaras is the term used for the

landless agricultural labour class

Ashvamedha also known as horse

sacrifice is a ritual where a horse is let

loose to wander freely and it was

guarded by the rajarsquos men

Q7) Answer (d)

Explanation

In the Rigvedic period horses were

yoked to chariots that were used in

battles fought to capture land cattle

etc This suggests that the use of horse

chariots began much before the period

of Mahajanapadas

The battles were fought in the Rigvedic

period for cattlersquos lands water an even

to capture people Most men took part

in these wars however there was no

regular army but there were assemblies

where people met and discussed

matters of war Regular armies became

a feature in the Mjahajanapada period

including vast armies of foot soldiers

chariots and elephants

RAUSIAS-FC19E1003 42

Q8) Answer (a)

Explanation

Buddha belonged to the Sakya clan and

passed away at Kusinara

Buddha taught in Prakrit which was the

common language of people

Q9) Answer c

Explanation

There were six schools of philosophy in

ancient India These are known as

Vaishesika Nyaya Samkhya Yoga

Purva Mimansa and Vedanata or Uttara

Mimansa They were founded by sages

Kanada Gautama Kapila Patanjali

Jamini and Vyasa respectively

Q10) Answer b

Explanation

The teachings of Mahavira were

compiled at Valabhi in 6th century AD

Q11) Answer (c)

Explanation

Chanakya is traditionally identified as

Kautilya or Vishnugupta who authored

the ancient Indian political treatise the

Arthashastra

Q12) Answer d

The national emblem of India is an

adaptation of the Lion Capital atop the

Ashoka Pillar of Sarnath Uttar Pradesh

and is combined with the National

Motto Satyameva Jayate

The Rampurva Bull gets the name from

the site of its discovery Rampurva in

Bihar

It is noted for its delicately sculpted

model demonstrating superior

representation of soft flesh sensitive

nostrils alert ears and strong legs It is

a mixture of Indian and Persian

elements

Sankissa is situated in Uttar Pradesh

India

Q13) Ans(a)

Kunwar Singh was a notable leader during the Revolt of 1857 He belonged

to a royal house of Jagdispur Bihar

Q14) Answer b

Explanation

The term Vellalar was used for large

landowners

Q15) Answer c

Explanation

Arikamedu was a coastal settlement

where ships unloaded goods from

distant lands Finds here include a

massive brick warehouse pottery

including amphorae and Arretine ware

Roman lamps glassware and gems have

also been found at the site

Q16) Answer a

Explanation

Muvendar is a Tamil word mentioned in

Sangam poems meaning three chiefs

used for the heads of three ruling

families the Cholas Cheras and

Pandyas

Q17) Ans (c)

Several tribal or kin-based assemblies

such as the Sabha Vidatha and gana

are mentioned in the Rig-veda The

Sabha and the samiti mattered a great

deal in early Vedic times so much so

that the chiefs or the kings showed an

eagerness to win their support

Q18) Ans (a)

Jainism recognised the existence of the

gods but placed them lower than the

jina and did not condemn the varna

system as Buddhism did

Q19) Answer (d)

Explanation

Cholas and Pandyas had developed

powerful coastal cities The most

important city of Cholas was Puhar or

Kaveripattinam and Madurai was the

capital of Pandyas

Q20) Answer b

Explanation

Buddhacharita is the biography of

Buddha and was written by

RAUSIAS-FC19E1003 43

Ashvaghosha

Q21) Answer (a)

Explanation

Tamil poet Appar was a Shiva devotee

So he was a Nayanar saint

Q22) Answer d

Explanation

Samudragupta was a prominent Gupta

ruler whose coins depict him playing a

veena indicating his love for music We

get important historic information from

his Allahabad Prashasti which was

composed by his court poet Harisena

Q23) Answer (b)

Explanation

Vikrama Samvat was founded by

Chandragupta II in the 58 BC as a

mark of victory over the Shakas and

assumed the title of Vikramaditya

Banabhatta wrote Harshavardhanarsquos

biography the Harshacharita in

Sanskrit

Q24) Answer c

Explanation

Sandhi-vigrahika was the minister of

war and peace

Sarthavaha was the leader of the

merchant caravans

Q25) Answer a

Explanation

Xuan Zang (Hsuan-tsang) was a

Chinese traveller who came during the

reign of Harshavardhana

In the decade that began in 630 AD

Xuan Zang came to India through

Kashmir after visiting Central Asia Iran

and Afghanistan

He travelled from north to east and lived

in Bihar for a couple of years

At Nalanda University Xuan Zang

interacted with students and scholars

mastered local languages and

discovered Buddhist stupas

Q26) Answer c

Explanation

Pradakshina patha is a circular path

laid around a stupa in Buddhist

architecture While the rest are a part of

temple architecture

Q27) Answer d

Explanation

All the above-mentioned temples have

an elaborate use of bricks (baked

bricks) along with stone

Q28) Ans (c)

Muhammad Quli Qutab was the Sultan

of Golconda He was a contemporary of

Akbar was very fond of literature and

architecture

The Sultan was a great poet and he

wrote in Dakhini Urdu Persian and

Telgu and has left an extensive diwan or

collection

Recently the Archaeological Survey of

India (ASI) will be using Ground

Penetrating Radar (GPR) to map the

contours of the area around the Bagh-e-

Naya Qila excavated garden inside the

Golconda Fort in Telangana

Q29) Answer a

Explanation

Silappadikaram is a famous Tamil epic

which was written by Ilango around

1800 years ago It is a story of a

merchant named Kovalan who fell in

love with a courtesan named Madhavi

Manimekalai tells the story of the

daughter of Kovalan and Madhavi

Q30) Answer (a)

Explanation

Charaka is the author of Charaka

Samhita which is an important work of

Ayurveda and medicines

Brahmaguptarsquos fame rests mostly on his

Brahma-sphuta-siddhanta which was

an astronomical work It was translated

into Arabic in Baghdad and had a major

impact on Islamic mathematics and

astronomy

Late in his life Brahmagupta wrote

Khandakhadyaka which was an

RAUSIAS-FC19E1003 44

astronomical handbook that employed

Aryabhatarsquos system of starting each day

at midnight

Q31) Answer (c)

Explanation

Amir Khusrau was a famous sufi

musician poet and scholar In 1318 he

noted that there was different language

in every region of this land (Hindustan)

Lahori Kashmiri Dvarsamudri (in

Southern Karnataka) Telangana (in

Andhra Pradesh) Gujari (in Gujarat)

Marsquobari (in Tamil Nadu) Awadhi (in

eastern Uttar Pradesh) and Hindawai (in

the area around in Delhi) etc He went

to explain that Sanskrit did not belong

to any region and that only brahmans

knew it

Q32) Answer c

Explanation

Hiranyagarbha refers to the golden

womb When this ritual was performed

with the help of Brahmanas it was

thought to lead to the rebirth of the

sacrificer as a Khastriya

Q33) Answer d

Explanation

Kadamai refers to a tax on land

revenue

Gwalior Prashasti describes the exploits

of Nagabhata who was a Pratihara king

Q34) Answer b

Explanation

Rajatarangini is a Sanskrit text written

by Kalhana in the 12th century

It was historical chronicle of early India

It is justifiably considered to be the best

and most authentic work of its kind

It covers the entire span of history in

the Kashmir region from the earliest

times to the date of its composition

Q35) Answer c

Explanation

ldquoUrrdquo was the general assembly of the

village ldquoUrrdquo consisted of all the

taxpaying residents of an ordinary

village

Q36) Answer (a)

Explanation

Tarikh was a form of history writing in

the Delhi Sultanate The authors of

tawarikhs were learned men which

included secretaries administrators etc

Q37 Answer (a)

Explanation

Alauddin chose to pay his soldiers salaries in cash rather than iqtas The soldiers would buy their supplies from merchants in Delhi and it was thus feared that merchants would raise their prices To stop this Alauddin controlled the prices of goods in Delhi Prices were carefully surveyed by officers and merchants who did not sell at the prescribed rates were punished

Q38) Answer (d)

Explanation

Delhi first became the capital of a

kingdom under the Tomara Rajputs

who were defeated in the middle of the

twelfth century by the Chauhans (also

referred to as Chahamanas) of Ajmer

It was under the Tomaras and

Chauhans that Delhi became an

important commercial centre Many rich

Jaina merchants lived in the city and

constructed several temples Coins

minted here called dehliwal had a wide

circulation

Q39) Answer (c)

Explanation

Moth ki Masjid was built in the reign of

Sikandar Lodi by his minister

Begumpuri mosque built in the reign of

Muhammad Tughluq was the main

mosque of Jahanpanah the ldquoSanctuary

of the Worldrdquo and his new capital in

Delhi

Quwwat al ndash Islam mosque was

enlarged by Iltutmish and Alauddin

Khalji The minar was built by three

Sultansndash Qutbuddin Aybak Iltutmish

and Firuz Shah Tughluq

RAUSIAS-FC19E1003 45

Q40) Answer (c)

Explanation

Under the Mughals mansabdar was

referred to an individual who held a

mansab ie rank and he received his

salary as revenue assignments called

jagirs

Q41) Ans (b)

The Quit India Movement was a

spontaneous revolt of people against

British rule

The All India Congress Committee met

at Bombay on 8 August 1942 It passed

the famous resolution Quit India and

proposed the starting of a non-violent

mass struggle under Gandhis

leadership to achieve this aim But on

the very next day Gandhi and other

eminent leaders of the Congress were

arrested The Congress was once again

declared illegal

Q42) Ans (c)

The Simon Commission refers to a

group of seven MPs from the United

Kingdom constituted to suggest

constitutional reforms for British India

The Commission consisted of only

British members headed by one of the

senior British politicians Sir John

Simon

So the people of India agitated against

the arrival of Simon Commission

Q43) Ans (a)

He was widely known for his

unfavourable opinion of the economic

consequences of the British rule in

India

In his many writings and speeches and

especially in Poverty and Un-British

Rule in India Naoroji argued that India

was too highly taxed and that its wealth

was being drained away to England

He did not interpret the ancient Indian

texts and restored the self-confidence of

Indians And also he did not stress the

need for eradication of all the social

evils before anything else

Q44) Ans (c)

In August 1932 Prime Minister

MacDonald announced his Communal

Award Great Britainrsquos unilateral

attempt to resolve the various conflicts

among Indiarsquos many communal

interests

The award which was later

incorporated into the act of 1935

expanded the separate-electorate

formula reserved for Muslims to other

minorities including Sikhs Indian

Christians Anglo-Indians Europeans

distinct regional groups Gandhi

undertook a ldquofast unto deathrdquo against

that offer which he viewed as a

nefarious British plot to divide the

Indian society

Q45) Ans (b)

In British India apart from existing

imports and exports there was also a

particular amount of money which

colonial India contributed towards

administration maintenance of the

army war expenses pensions to retired

officers and other expenses accrued by

Britain towards maintenance of her

colony These were known as Home

charges and were paid for almost

entirely by India

The Home charges was made of

following components-

- Interest payable on Indian debt

- Dividend to shareholders of East

India Company

- Funds used to support the India

Office in London

- Funds used to pay salaries and

pensions of British personnel

engaged in India

- Interest on the railways

- Civil and military charges

- Store purchases in England

Q46) Ans (b)

The Lahore session of the Indian

National Congress was held in 1929

under the Presidentship of Jawaharlal

Nehru

The Lahore session of the Indian

National Congress witnessed significant

RAUSIAS-FC19E1003 46

developments in the Indian national

movement

- First the election of Jawaharlal

Nehru to the post of Presidentship of

the Congress was a clear indication

of the growing strength of the

Leftists in the Congress

- Secondly it was in this session that

the Congress for the first time raised

the demand for complete

independence Such demand was

not raised from the Congress

platform earlier

Q47) Ans (b)

It did not provide for separate

electorates for any community or

weightage for minorities However it did

allow for the reservation of minority

seats in provinces having minorities of

at least ten per cent but this was to be

in strict proportion to the size of the

community

There was no provision for complete

Independence for India

Q48) Ans (c)

The religion of early Vedic Aryans was

primarily of worship of nature and

Yajnas

The early Aryan religion was kind of

nature worship Actually the forces

around them which they could not

control or understand were invested

with divinity and were personified as

male or female gods And they

performed some Yajnas also

Q49) Ans (b)

The roads and river-routes were not

immune from robbery It is notable that

Yuan Chwang (Hiuen Tsang) was

robbed of his belongings during

Harshvardanarsquos period

Q50) Ans (c)

Q51) Ans (b)

Purandara Dasa was a saint and great

devotee of Lord Krishna

There is much speculation about where

Purandara Dasa regarded as the

Pitamaha of Carnatic music was born

Recently an expert committee

constituted by the Kannada University

Hampi has come to the conclusion that

Kshemapura Shivamogga district

Karnataka is the birth place of

Purandara Dasa

Q52) Ans (c)

Sri Tyagaraja Sri Shyama Shastry and Sri Muthuswami Dikshitar are considered the trinity of Carnatic music and with them came the golden age in Carnatic music in the 18th-19th

century

Q53) Ans d)

Recently a rare sarcophagus (stone

coffin) which is 2000 years old from the

Iron AgendashMegalithic era was discovered

from a rock-cut cave at Viyur village of

Kollam near Koyilandy in Kozhikode

district Kerala

The coffin containing bone fragments

was found during an excavation ldquoSo

far such a rare finding has been

discovered only from two sites

in Kerala Both these sarcophagi were

recovered from Megalithic sites at

Chevayur and Atholi also in Kozhikode

district

Q54) Ans a)

The megalithic culture in South India was a full-fledged Iron Age culture

Q55) Ans d)

The Cholas Pandyas and Keralaputras

(Cheras) mentioned in Ashokan

inscriptions were probably in the late

megalithic phase of material culture

Q56) Ans d)

Q57) Ans (b)

Raj Kumar Shukla followed Gandhiji all

over the country to persuade him to

come to Champaran to investigate the

problem associated with tinkathia

system

RAUSIAS-FC19E1003 47

Brij Kishore Rajendra Prasad Mahadev

Desai and Narhari Parikh accompanied

Gandhi ji during the Champaran

Satyagraha

Q58) Ans (b)

The Satvahanas started the practice of granting tax-free villages to brahmanas and Buddhist monks

Q59) Ans c)

The objectives of the Programme are

listed as under

- Developing basic tourism

infrastructure

- Promoting cultural and heritage

value of the country to generate

livelihoods in the identified regions

- Enhancing the tourist attractiveness

in a sustainable manner by

developing world-class

infrastructure at the heritage

monument sites

- Creating employment through active

involvement of local communities

- Harnessing tourism potential for its

effects on employment generation

and economic development

- Developing sustainable tourism

infrastructure and ensuring proper

Operations and maintenance

therein

Q60) Ans (b)

The Tribal Cooperative Marketing

Development Federation of India

(TRIFED) came into existence in 1987

It is a national-level apex organization

functioning under the administrative

control of Ministry of Tribal Affairs

Govt of India

TRIFED has its registered and Head

Office located in New Delhi

Q61) Ans (c)

Premchandrsquos novels include

Premashram Rangabhumi Ghaban

Karmabhumi and Godan

Gora is a novel written by Rabindranath

Tagore

138th birth anniversary of Munshi

Premchand was celebrated across the

country

Q62) Ans (b)

Giddha is a traditional pastoral dance

performed by the women of the Punjab

India and Pakistan at festival times

and at the sowing and reaping of the

harvest

By this dance the Punjabi women

reveal their joy expel their suppressed

feelings in a male dominated society

through the performance of Giddha

Since this dance has nothing to do with

men only women can participate in it

During the Teej celebrations Giddha

dance is celebrated in Punjab every

year Teej is a generic name for a

number of festivals that are celebrated

by women in some parts of India

Q63) Ans (a)

Dara Shukoh wrote the remarkable

work called ldquoMajma-ul-Bahrainrdquo or the

ldquoThe confluence of two seasrdquo

The Vice President of India Shri M

Venkaiah Naidu has said that Prince

Dara Shukohrsquos writings can come as a

refreshing source for infusing peace and

harmony He was addressing the

gathering after visiting the exhibition

that showcases the forgotten Prince of

yesteryears Dara Shukoh organized by

Mr Francois Gautier at Indira Gandhi

National Centre for the Arts in New

Delhi

Q64) Ans (c)

The statue Gommateshwara is

dedicated to the Jain God Bahubali

It is a monolithic statue

President Ram Nath Kovind

inaugurated the grand anointing

ceremony mdash Mahamastakabhisheka mdash

held once in 12 years at

Shravanabelagola (Karnataka)

Q65) Ans (c)

Prachi Valley had come up around the

Prachi river Prachi Valley gradually

disappeared

RAUSIAS-FC19E1003 48

The Prachi river originates from

Bhubaneswar

It is a tributary of the Mahanadi and

flows through the districts of Puri

Khurda Cuttack and Jagatsinghpur

and the entire region of the river is

termed as the Prachi Valley

It falls into the Bay of Bengal

Archaeological evidence shows that the

Prachi Valley Civilisation predates both

Harappa and Mohenjo-Daro

The Prachi river originates from

Bhubaneswar

Q66) Ans (d)

These monuments are located in

Chhatarpur district Madhya Pradesh

within Vindhya mountain range

Q67) Ans (a)

The book lsquoThoughts on Pakistanrsquo was

written by Dr BR Ambedkar

On the occasion of the birth anniversary

of Dr BR Ambedkar the president of

India pays homage to this icon of India

In 1924 he founded the Depressed

Classes Institute (Bahishkrit Hitkarini

Sabha) and in 1927 the Samaj Samata

Sangh

Another area of attention for Ambedkar

was education For its spread among

the low classes he set up a network of

colleges by the name of Peoples

Education Society and founded hostels

Q68) Ans(b)

Mehrgarh is a famous Neolithic

settlement in the Indian subcontinent

which is situated in Baluchistan

province Pakistan

A pre-historic rock art site is discovered

in the vast expanse of limestone blocks

on the eastern banks of Naguleru river

near Dachepalli (Andhra Pradesh) It

has thrown light on the Neolithic

civilisation that flourished in Guntur

(Andhra Pradesh) during 1500-2000

BC

Q69) Ans (c)

The 12th and the 13th centuries saw

the emergence of the Kakatiyas They

were at first the feudatories of the

Western Chalukyas of Kalyana Initially

they ruled over a small territory near

Warangal (Telangana)

They introduced Nayakships which was

later adopted and developed by the

Rayas of Vijayanagara

Q70) Ans (a)

The fast had effect of putting pressure

on mill owners who finally agreed to

give the workers a 35 per cent increase

in wages

Google celebrated with a doodle the

132nd birth anniversary of Anasuya

Sarabhai who played a pioneering role

in Indiarsquos labour movement

Q71) Ans (d)

The UNESCOrsquos list of the representative

list of the intangible cultural heritage of

humanity from India are

- Koodiyattam Sanskrit Theatre of

Kerala

- Mudiyettu ritual theatre and dance

drama of Kerala

- Tradition of Vedic Chanting

- Kalbelia folk songs and dances of

Rajasthan

- Ramlila Traditional Performance of

the Ramayana

- Sankirtana ritual singing

drumming and dancing of Manipur

- Ramman religious festival and

ritual theatre of the Garhwal

Himalayas India

- Traditional brass and copper craft of

utensil making among the Thatheras

of Jandiala Guru Punjab India

- Chhau dance classical Indian dance

originated in the eastern Indian

states

- Buddhist chanting of Ladakh

recitation of sacred Buddhist texts

in the trans-Himalayan Ladakh

region Jammu and Kashmir India

- Yoga

- Nouroz

- Kumbh Mela

RAUSIAS-FC19E1003 49

Q72) Ans(b)

The President of India Shri Ram Nath Kovind inaugurated the Hornbill Festival and State Formation Day celebrations of Nagaland in Kisama

The festival is named after the Indian hornbill the large and colourful forest bird which is displayed in the folklore of most of the states tribes

The major recognized tribes of Nagaland are Angami Ao Chakhesang Chang

Kuki Rengma and Zeling etc

Onge Jarawa and Sentinelese are the

tribes of Andman amp Nicobar Islands

Q73) Ans (c)

The Rashtrakutas rule in the Deccan lasted for almost two hundred years till the end of the tenth century The Rashtrakutas rulers were tolerant in their religious views and patronized not only Shaivism and Vaishnavism but

Jainism as well

The famous rock-cut temple of Shiva at Ellora was built by one of the Rashtrakutas kings Krishna I in the ninth century His successor Amoghavarsha was a Jain but he also

patronized other faiths

The Rashtrakutas allowed Muslims traders to settle and permitted Islam to

be preached in their dominions

Recently increasing defacement at the prehistoric rock paintings of Pandavulagutta Telangana has created a cause for grave concern It can spoil

the prehistoric rock

Pandavulagutta is home to

- Painted rock shelters dating to

10000 BC-8000 BC

- An 8th century inscription of the

Rashtrakuta period and

- Painted frescoes from the 12th century Kakatiya empire

Q74) Ans (b)

In 1828 Raja Ram Mohan Roy founded a new religious society the Brahma Sabha later known as the Brahmo

Samaj

Debendranath Tagore headed the Tattvabodhini Sabha which was

engaged in search of spiritual truth

Its purpose was to purify Hinduism and to preach monotheism or belief in one God

The new society was to be based on the twin pillars of reason and the Vedas and

Upanishads

Recently Sadharan Brahmo Samaj (SBS) has entered into a legal battle with the West Bengal government due

to some legal issue

Q75) Ans (c)

The Chishti order was established in India by Khwaja Moinuddin Chishti who came to India around 1192 The Chishtirsquos are considered to be the most influential of the groups of Sufis who migrated to India in the late twelfth century They adapted successfully to the local environment and adopted several features of Indian devotional

traditions

The historical dargah of Sufi mystic Khwaja Moinuddin Chishti in Ajmer is all set to get a facelift This 13 th century dargah has been included among the Swachh Iconic Places a clean-up initiative focused on iconic

heritage spiritual and cultural places

Page 30: GENERAL STUDIES (PAPER I) · Test is part of Rau’s IAS Test series for Preliminary Exam 2019 FOUNDATION + CURRENT AFFAIRS GENERAL STUDIES (PAPER –I) FOUNDATION TEST –III TOPIC:

FC19H1003 30

जाता ह और राजा क सवनक उसकी रखिाली

करत थ

Q7) उततर (d)

सपषटीकरण

- ऋगववदक काल म घ ड ो क रथ ो म ज ता जाता था

ज (रथ) भवम मिवशय ो आवद पर कबजा करन क

वलए लड गए यद ो म उपय ग वकए जात थ

- इसस यह पता चलता ह वक घ ड ो यकत रथ ो का

उपय ग महाजनपद काल स काफी पहल आरमभ

हआ था

- ऋगववदक काल म मिवशय ो भवम जल आवद पर

कबजा करन क वलए तथा ल ग ो क पकडन क

वलए यद वकय जात थ

- अविकाोश परि इन यद ो म भाग वलया करत थ

- हालाोवक उस समय क ई वनयवमत सना नही ो ह ती

थी लवकन उस काल म सभाऐो ह ती थी ो वजनम

ल ग यद क मामल ो पर चचाि करत थ

- वनयवमत सनाएा महाजनपद काल का िवशषटय थी

वजनम पदल सवनक ो की विशाल सनाएा रथ तथा

हाथी शावमल ह त थ

Q8) उततर (a)

सपषटीकरण

- बद शाकय कल स सोबोवित थ और कशीनारा म

उनका वनिन हआ था

- बद न अपनी वशकषाएा पराकत भािा म दी थी ो ज

आम ल ग ो की भािा थी

Q9) उततर (c)

सपषटीकरण

- पराचीन भारत म दशिनशासर की छह शाखाएा थी ो

िशविक नयाय समखया य ग पिि वममाोसा और

िदाोत या उततर वममाोसा

- इनकी थथापना करमश कनाद गौतम कवपल

पतोजवल जावमनी और वयास ऋविय ो न की थी

Q10) उततर (b)

सपषटीकरण

महािीर की वशकषाऐो छठी शताबदी म िललभी म

सोकवलत की गई थी ो

Q11) उततर (c)

सपषटीकरण

- पारमपररक रप स चाणकय क कौविलय अथिा

विषणगपत क नाम स जाना जाता ह

- उसन अथिशासतर ज एक पराचीन भारतीय

राजनवतक आलख ह वलखा था

Q12) उततर (d)

सपषटीकरण

- भारत का राषटर ीय वचनह सारनाथ (उततर परदश) क

अश क सतमभ क ऊपर (शीिि पर) वसोह कवपिल

का एक अनरपण ह

- इस राषटर ीय वसदाोत सतयमि जयत क साथ

सोय वजत वकया गया ह

- रामपिि बल का नाम रामपिि (वबहार) क नाम पर

पडा जहाा इसकी ख ज हई थी

- यह अपन नाजक नकाशी मॉडल क वलए परवसदद

ह वजसम क मल तवचा सोिदनशील नथन ो सतकि

कान और मरबत िााग ो क शरषठतर परवतरप क

परदवशित वकया गया ह

- यह भारतीय और फारसी ततव ो का एक ससममशरण

- सोवकससा उततर परदश म सथथत ह

Q13) उततर (a)

सपषटीकरण

का िर वसोह ज एक महान य दा थ वबहार स

सोबोवित थ

Q14) उततर (b)

सपषटीकरण

िललालर शबद बड भ-सवावमय ो क वलए परय ग

वकया जाता था

FC19H1003 31

Q15) उततर (c)

सपषटीकरण

- अररकमड एक तिीय बसती थी जहाो दर दश ो स

आन िाल जहाज ो का माल उतारा जाता था

- यहाो पर ईोि ो का एक विशाल ग दाम वमटटी क

बतिन (वजनम एमफ रा - द हरी मवठय ो का लोबा

घडा - शावमल ह) और एरिाइन (Arretine)

मदभाोड पाए गए थ

- इस थथान पर र मन दीपक काोच क बन पातर और

रतन भी पाए गए थ

Q16) उततर (a)

सपषटीकरण

- मिनदर सोगम कविताओो म उसललसखत एक

तवमल शबद ह वजसका अथि ह ldquoतीन परमखrdquo

- यह तीन सततारि पररिार ो क मसखयाओो क वलए

परय ग वकया जाता ह च ल चर और पाणडय

Q17) उततर (c)

सपषटीकरण

- ऋग िद म सभा विदाथा तथा गण जसी

जनजावतय ो पर अथिा किोब पर आिाररत

सभाओो का उललख ह

- आरसमभक िवदक काल म सभाओो और सवमवतय ो

का विशि महतव ह ता था

- यहाा तक की मसखया अथिा राजा भी उनका

समथिन परापत करन क वलए आतर रहत थ

Q18) उततर (a)

सपषटीकरण

- जन िमि न ईशवर क अससततव क मानयता त दी ह

वकनत उसन ईशवर क वजना क पद स नीच रखा

- जन िमि न बौद िमि की तरह िणि परणाली की

भरतिना नही ो की थी

Q19) उततर (d)

सपषटीकरण

- च ल ो और पाणडय ो न शसकतशाली तिीय शहर ो का

विकास वकया था

- च ल ो का सबस महतवपणि शहर पहार (या

कािरीपटटीनम) था |

- मदरई पाणडय ो की राजिानी थी

Q20) उततर (b)

सपषटीकरण

- ldquoबदचररतrdquo बद का जीिन-ितताोत ह

- इस अशवघ ि क दवारा वलखा गया था

Q21) उततर (a)

सपषटीकरणः

- तवमल कवि अपपर भगिान वशि क भकत थ

- इस परकार ि एक नयनार सोत थ

Q22) उततर (d)

सपषटीकरणः

- समदरगपत एक परवसद गपत शासक था

- उसन वसक ो पर िीणा बजात हए अपनी छवि

अोवकत करिाई थी

- यह सोगीत क परवत उसक परम क दशािता ह

- हम उसकी इलाहाबाद परशससत स महतवपणि

ऐवतहावसक जानकारी वमलती ह वजसकी रचना

उसक दरबार क कवि हररसन न की थी

Q23) उततर (b)

सपषटीकरणः

- विकरम सोित की शरआत ििि 58 ईसा पिि म

चनदरगपत वदवतीय न की थी

- यह शक ो पर उसकी जीत और उस विकरमावदतय

की पदिी वमलन क उपलकषय म आरमभ वकया गया

था

FC19H1003 32

- बानभटट न हिििििन का जीिन-ितताोत हििचररत

(ज सोसकत म थी) वलखी थी

Q24) उततर (c)

सपषटीकरणः

- सोवि-विगरावहका यद एिो शाोवत का मोतरी

- साथििाह वयापाररय ो क कावफल ो का नता

Q25) उततर (a)

सपषटीकरणः

- जआन झाोग (हसआन रताोग ndash Hsuang Tsang)

एक चीनी यातरी था ज हिििििन क शासनकाल म

भारत आया था

- ििि 630 ईसवी स ज दशक आरमभ हआ था उसम

जआन झाोग मधय एवशया ईरान और

अफग़ावनसतान की यातरा करन क पशचात कशमीर

क रासत स भारत आया था

- उसन उततर स पिि तक की यातरा की और िह

लगभग 2 ििि वबहार म रहा

- जआन झाोग न नालनदा विशवविदयालय म विदयावथिय ो

और विदवान ो क साथ पारसपररक विचार-विमशि

वकया थथानीय भािाओ ा म वनपणता परापत की तथा

बौद सतप ो की ख ज की

Q26) उततर (c)

सपषटीकरणः

- परदवकषणा पथ बौद िासतकला म सतप क चार ो

ओर बनाया जान िाला एक घमािदार पथ ह ता

- परशन म वदए गए बाकी क तीन ो ततव वहोद मसनदर ो की

िासतकला क भाग ह

Q27) उततर (d)

सपषटीकरणः

परशन म वदए गए सभी मोवदर ो म वयापक रप स

ईोि ो (पकी ईोि ो) का परय ग पतथर ो क साथ हआ

Q28) उततर (c)

सपषटीकरण

- महममद कली कतब शाह ग लकणडा का सलतान

था

- िह अकबर का समकालीन था

- सावहतय और िासतकला म उसकी अतयाविक

रवच थी

- िह एक महान कवि था

- िह दसखनी उदि फारसी और तलग म वलखता था

- उसन अपन पीछ एक विसतत वदिान (सोगरह)

छ डा ह

- अभी हाल ही म तलोगाना म ग लकणडा क वकल

क अनदर खदाई वकय गए बाग-ए-नाया वकला

बाग क चार ो ओर रप-रखा क मानवचतरण क

वलए भारतीय परातासतवक सिकषण (The

Archaeological Survey of India ndash ASI)

गराउणड पनीिर विोग रडार (Ground Penetrating

Radar) का परय ग करगा

Q29) उततर (a)

सपषटीकरणः

- वसलपपावदकारम एक तवमल महाकावय ह वजसकी

रचना इलाोग क दवारा लगभग 1800 ििि पिि की

गई थी

- यह क िलन नामक एक वयापारी की कहानी ह

ज माििी नामक एक गवणका (िशया) स परम

करन लगा था

- मवनमकलाई क िलन और माििी की पतरी की

कहानी ह

Q30) उततर (a)

सपषटीकरण

- चरक आयिद और वचवकरता की एक महतवपणि

रचना चरक सोवहता क लखक ह

- बरहमगपत क अपनी रचना बरहम-सफि-वसदानत

(ज एक खग लीय रचना ह) क कारण परवससद

वमली

FC19H1003 33

- बगदाद म इसका अनिाद अरबी भािा म वकया

गया था

- इसका इसलावमक गवणत और खग ल-विजञान पर

महतवपणि परभाि पडा था

- बाद म अपन जीिनकाल म बरहमगपत न

ldquoखोडखयाकrdquo वलखी ज एक खग लीय पससतका

(एक छ िी पसतक) थी

- इसम आयिभटट की अिि-रावतर क परतयक वदन की

शरआत परणाली का परय ग वकया गया था

Q31) उततर (c)

सपषटीकरण

- अमीर खसर एक परवसद सफी सोगीतकार कवि

और विदवान थ

- 1318 म उनह ोन पाया वक इस भवम (वहोदसतान) क

हर कषतर म अलग-अलग भािा थी लाहौरी

कशमीरी दवारसमदरी (दवकषणी कनाििक म)

तलोगाना (आोधर परदश म) गजरी (गजरात म)

माबारी (तवमलनाड म ) अििी (पिी उततर परदश

म) और वहोदिी (वदलली क आस-पास क कषतर म)

आवद

- उनह न यह बताया वक सोसकत वकसी भी कषतर स

सोबोवित नही ो थी और किल बराहमण ही इस भािा

का जञान रखत थ

Q32) उततर (c)

सपषटीकरण

- वहरणय-गभि सववणिम गभि क सोदवभित करता ह

- जब बराहमण ो की सहायता स यह अनषठान वकया

जाता था त यह माना जाता था वक बवल दन िाल

का कषवतरय क रप म पनजिनम ह गा

Q33) उततर (d)

सपषटीकरण

- कदमई भवम राजसव पर कर क सोदवभित करता

- गवावलयर परशससत म नागभि क दवारा वकय गए

श िण का िणिन वकया गया ह |

- नागभि एक परवतहार राजा था

Q34) उततर (b)

सपषटीकरण

- राजतरो वगनी 12िी ो शताबदी म कलहन क दवारा

रवचत एक सोसकत पसतक (िकसट) ह

- यह परारसमभक भारत की ऐवतहावसक इवतितत थी

- तकि सोगत रप स इस अपन परकार की सिोततम

और सिािविक विशवसनीय कवत माना जाता ह

- यह कशमीर कषतर क पराचीनतम समय स लकर

उसकी रचना की तारीख तक क समपणि इवतहास

का आचछादन करती ह

Q35) उततर (c)

सपषटीकरण

- गााि की आम सभा क ldquoउरrdquo कहा जाता था

- ldquoउरrdquo म गााि क सभी कर दन िाल वनिासी

शावमल ह त थ

Q36) उततर (a)

सपषटीकरण

- वदलली सलतनत म ldquoतारीखrdquo इवतहास लखन का

एक रप था

- ldquoतािरीखrdquo क लखक विदवान परि ह त थ वजनम

सवचि परशासक इतयावद शावमल थ

Q37) उततर (a)

सपषटीकरण

- अलाउददीन सखलजी अपन सवनक ो क ितन का

भगतान नकद म करता था न वक इकता क रप

- सवनक अपना सामान वदलली म वयापाररय ो स

खरीदत थ अतः इस बात का भय था वक वयापारी

कही ो िसतओो का मलय न बिा द

- इसकी र कथाम क वलए अलाउददीन सखलजी न

वदलली म कीमत ो क वनयसित वकया

FC19H1003 34

- अविकारीगण धयानपििक मलय ो का सिकषण करत

थ तथा ज वयापारी वनिािररत मलय पर माल नही ो

बचत थ उनक दसणडत वकया जाता था

Q38) उततर (d)

सपषटीकरण

- वदलली सििपरथम त मर राजपत ो क अिीन उनक

सामराजय की राजिानी बनी थी

- 12िी ो शताबदी क मधय म अजमर क चौहान ो

(वजनह चाहमान ो क नाम स भी जाना जाता ह) न

त मर राजपत ो क परावजत वकया था

- त मर ो और चौहान ो क अिीन वदलली एक

महतवपणि िावणसजयक क दर बन गया था

- कई जन वयापारी यहाा रहन लग थ और उनह ोन

कई मोवदर भी बनिाए

- यहाा पर मवदरत वसक वजनह ldquoदहलीिालrdquo क नाम

स जाना जाता था वयापक रप स परचलन म थ

Q39) उततर (c)

सपषटीकरण

- म ठ की मसिद का वनमािण वसको दर ल दी क

राजयकाल म उसक मिी क दवारा करिाया गया

था

- बगमपरी मसिद का वनमािण महममद तगलक क

शासनकाल म हआ था

- यह मसिद विशव का पणयथथान (The

Sanctuary of the World) और वदलली म महममद

तगलक की नई राजिानी जहाोपनाह की मखय

मसिद थी

- कववत- अल - इसलाम मसिद का विसतार

इलतसिश और अलाउददीन सखलजी न वकया था

- मीनार का वनमािण तीन सलतान ो कतबददीन ऐबक

इलतसिश और वफर ज शाह तगलक क दवारा

करिाया गया था

Q40) उततर (c)

सपषटीकरण

- मगल ो क अिीन मनसबदार शबद उस वयसकत क

वलए सोदवभित वकया जाता था वजसक पास मनसब

(अथाित पद) ह ता था

- उस अपना ितन राजसव कायो वजनह जागीर कहत

थ क रप म परापत ह ता था

Q41) उततर (b)

सपषटीकरण

- ldquoभारत छ ड आोद लनrdquo वबरविश शासन क

सखलाफ ल ग ो का एक सवाभाविक विदर ह था

- असखल भारतीय काोगरस सवमवत न 8 अगसत 1942

क बमबई म एक बठक का आय जन वकया था

- इस बठक म परवसद सोकलप ldquoभारत छ ड rdquo क

पाररत वकया गया और इस उददशय क परापत करन

क वलए गाोिी क नततव म एक अवहोसक जन सोघिि

आोद लन की शरआत का परसताि वदया गया

- लवकन अगल ही वदन गाोिी और काोगरस क अनय

परमख नताओो क वगरफतार कर वलया गया

- काोगरस क एक बार वफर अिि घ वित वकया गया

था

Q42) उततर (c)

सपषटीकरण

- साइमन कमीशन यनाइविड वको गडम क सात

साोसद ो का एक समह था

- इस वबरविश भारत क वलए सोििावनक सिार ो का

सझाि दन क वलए गवठत वकया गया था

- इस आय ग म िररषठ वबरविश राजनता सर जॉन

साइमन क नततव म किल वबरविश सदसय ही

शावमल थ

- इसवलए भारत क ल ग ो न साइमन कमीशन क

आगमन क विरद आोद लन वकया था

Q43) उततर (a)

सपषटीकरण

bull दादा भाई नौर जी भारत म वबरविश शासन क

आवथिक पररणाम ो क बार म अपनी विर िी

(परवतकल) राय क वलए जान जात थ

FC19H1003 35

bull अपन कई लख ो और भािण ो म विशि रप स

ldquoपाििी एो ड अन-वबरविश रल इन इसणडया

(Poverty and Un-British Rule in India) म

नौर जी न यह तकि वदया वक भारत पर अतयविक

कर लगाया गया था और इसकी सोपवतत इोगलड की

ओर परिावहत की जा रही थी

bull उनह ोन पराचीन भारतीय गरोथ ो की वयाखया करन

का और भारतीय ो क आिविशवास क बहाल

करन पर कायि नही ो वकया था

उनह ोन वकसी और बात स पहल सभी सामावजक

बराइय ो क उनमलन की आिशयकता पर भी बल

नही ो वदया था

Q44) उततर (c)

सपषटीकरण

bull अगसत 1932 म वबरविश परिानमोतरी मकड नालड न

अपन साोपरदावयक परसकार (The Communal

Award) की घ िणा की थी

bull यह भारत क कई साोपरदावयक वहत ो क बीच विवभनन

सोघिो क हल करन क वलए वबरिन का एकतरफा

परयास था

bull यह परसकार (Award) बाद म 1935 क

अविवनयम (The Act of 1935) म शावमल वकया

गया था

bull इस साोपरदावयक परसकार न मससलम ो क वलए

आरवकषत एक अलग वनिािचक मणडल फॉमिल का

विसतार अनय अलपसोखयक ो क वलए वकया था

वजसम वसख ो भारतीय ईसाइय ो आोगल-भारतीय

समदाय यर पीय समदाय तथा विवशषट कषतरीय

समह ो क शावमल वकया गया था

bull गाोिी न इस परसताि क भारतीय समाज क

विभावजत करन क वलए एक घवणत वबरविश

सावजश क रप म दखा और उसक सखलाफ

आमरण अनशन वकया

Q45) उततर (b)

सपषटीकरण

मौजदा आयात और वनयाित क अवतररक़त

औपवनिवशक भारत क वनमनवलसखत खचो क

वलए एक विशिवनवशचत िन रावश भी दनी पडती

थी

(i) परशासन क वयय

(ii) सना क रख-रखाि क वयय

(iii) यद क वयय

(iv) सिावनितत अविकाररय ो की पशन तथा

(v) वबरिन दवारा अपनी उपवनिश बसती

(कॉल नी) क रख-रखाि क वयय

इनह गह शलक (Home Charges) क रप म

जाना जाता था और लगभग परी तरह स भारत क

दवारा इनका भगतान वकया जाता था

bull गह शलक म वनमनवलसखत घिक शावमल थ

(i) भारतीय ऋण पर दय बयाज

(ii) ईसट इोवडया को पनी क शयरिारक ो क

लाभाोश

(iii) लोदन म भारत कायािलय चलान क वलए िन

(iv) भारत म वनयकत वबरविश कवमिय ो क ितन

और पशन का भगतान करन क वलए िन

(v) रलि पर बयाज

(vi) नागररक और सनय शलक

(vii) इोगलड म सट र (सामगरी) की खरीद

Q46) उततर (b)

सपषटीकरण

bull भारतीय राषटर ीय काोगरस का लाहौर सतर 1929 म

जिाहरलाल नहर की अधयकषता म आय वजत

वकया गया था

bull इस सतर म भारतीय राषटर ीय आोद लन स समबसित

कई महतवपणि पररणाम सामन आय थ

(i) सििपरथम इस सतर म काोगरस क अधयकष पद

पर जिाहरलाल नहर क चना गया था ज

काोगरस म िामपोवथय ो की बिती हई ताकत

का सपषट सोकत था

(ii) दसरा इस सतर म पहली बार काोगरस न पणि

सवतोतरता की माोग क उठाया था

इस परकार की माोग काोगरस मोच स पहल कभी भी

नही ो उठाई गई थी

Q47) उततर (b)

सपषटीकरण

FC19H1003 36

bull इस ररप िि न वकसी भी समदाय क वलए पथक

वनिािचक मोडल अथिा अलपसोखयक ो क वलए

भाराोश की वसफाररश नही ो की थी

bull तथावप इस ररप िि न उन पराोत ो म अलपसोखयक

सीि ो क आरकषण की अनमवत दी थी जहाा पर कम

स कम दस परवतशत अलपसोखयक ह

bull लवकन यह समदाय क आकार क अनपात म ह ना

चावहए था

bull इस ररप िि म भारत क वलए पणि सवतोतरता क

वलए क ई पराििान नही ो था

Q48) उततर (c)

सपषटीकरण

bull आरो वभक िवदक आयो का िमि मखय रप स

परकवत की पजा और यजञ था

bull परारो वभक आयि िमि परकवत की पजा क समान था

bull िासति म उनक चार ो ओर की शसकतयाा वजनह न

त ि वनयोवतरत कर सकत थ और न ही समझ पाए

थ उनह वदवयता क साथ वनिवशत वकया गया तथा

उनह मादा या नर दिीदिताओो क रप म

परतीकतव वकया गया था

bull उनह ोन कछ यजञ ो का भी वनषपादन वकया था

Q49) उततर (b)

सपषटीकरण

bull सडक और नदी-मागि (जल-मागि) डकती स

सरवकषत नही ो थ

bull उललखनीय ह वक हिििििन क शासनकाल क

दौरान यआन चिाोग (हयएन साोग) का सारा

सामान लि वलया गया था

Q50) उततर (c)

सपषटीकरण

परशन म वदए गए द न ो कथन सही ह

Q51) उततर (b)

सपषटीकरण

bull परोदर दास एक सोत और भगिान कषण क एक

महान भकत थ

bull परोदर दास क कनाििक सोगीत क वपतामह क

रप म जाना जाता ह

bull यदयवप उनक जनम-थथान क बार म काफी

अिकल लगाई जाती रही ह

bull तथावप अब कननड विशवविदयालय हमपी क दवारा

गवठत एक विशिजञ सवमवत इस वनषकिि पर पहोची

ह वक उनका जनम थथान सोभितया कनाििक का

एक छ िा-सा गााि कषमपरा (वशिम गगा वजला)

था

Q52) उततर (c)

सपषटीकरण

bull शरी तयागराज शरी शयाम शासतरी और शरी मथसवामी

दीवकषतर क कनाििक सोगीत की वतरमवति माना

जाता ह

bull उनक कारण ही 18िी ो-19िी ो शताबदी म कनाििक

सोगीत का सववणिम यग आया था

Q53) उततर (d)

सपषटीकरण

bull अभी हाल ही म लौह यगीन-महापािावणक काल

का 2000 ििि पराना एक दलिभ सारक फगस

(Sarcophagus) (पतथर का ताबत) क ललम क

वियर गाोि (क वयलडी क पास वजला क वझक ड

करल राजय) की एक रॉक-कि गफा स ख जा गया

bull यह ताबत वजसम हविय ो क िकड थ खदाई क

दौरान वमला

bull अभी तक इस परकार की दलिभ ख ज करल क

मातर द ही थथान ो स हई ह

bull य द न ो सारक फगी (Sarcophagi) (पतथर क

ताबत) चियर और अथ ली (वजला क वझक ड) क

महापािाण थथल ो स वमल ह

Q54) उततर (a)

सपषटीकरण

FC19H1003 37

दवकषण भारत म महापािाण सोसकवत एक पणि

विकवसत लौह यगीन सोसकवत थी

Q55) उततर (d)

सपषटीकरण

bull च ल पाणडय और करलपतर (चर) इन तीन ो का

उललख अश क क अवभलख ो म वकया गया ह

bull सोभितः य भौवतक सोसकवत क उततर

महापािावणक चरण म थ

Q56) उततर (d)

सपषटीकरण

bull भीमा-क रगाोि की लडाई ततीय आोगल-मराठा

यद का वहससा थी

Q57) उततर (b)

सपषटीकरण

bull राजकमार शकल न गाोिीजी क चोपारण आन तथा

वतनकवथया परणाली स जडी समसया की जाोच क

वलए रारी करन क वलए दश भर म उनका

अनसरण वकया था

bull बज वकश र राजदर परसाद महादि दसाई और

नरहरी पाररख चोपारण सतयागरह क दौरान गाोिी

जी क सहय गी थ

Q58) उततर (b)

सपषटीकरण

bull बराहमण ो और बौद मठिाररय ो क कर-मकत गााि

अनदान म दन की परथा सतिाहन ो न आरमभ की

थी

Q59) उततर (c)

सपषटीकरण

इस कायिकरम क उददशय वनमनानसार ह

(i) बवनयादी पयििन आिाररक सोरचना का विकास

करना

(ii) चयवनत (पहचान वकय गए) कषतर ो म आजीविका क

सजन क वलए दश क साोसकवतक और विरासत

मलय ो क बिािा दना

(iii) विरासत समारक थथल ो पर विशव सतरीय आिाररक

सोरचना विकवसत करक एक सतत तरीक स

पयििक आकििण म िसद करना

(iv) थथानीय समदाय ो की सवकरय भागीदारी क माधयम

स र रगार ो का सजन करना

(v) र रगार उतपादन और आवथिक विकास क वलए

पयििन कषमता का उन पर परभाि का उपय ग

करना तथा

(vi) िारणीय पयििन आिाररक सोरचना का विकास

करना और उसका उवचत सोचालन तथा

रखरखाि सवनवशचत करना

Q60) उततर (b)

सपषटीकरण

bull यह वनकाय ििि 1987 म अससततव म आया था

bull यह एक राषटर ीय सतर का शीिि सोगठन ह ज भारत

सरकार क जनजातीय मामल ो क मोतरालय क

परशासवनक वनयोतरण क अिीन काम कर रहा ह

bull इसका पोजीकत और परिान कायािलय नई वदलली

म सथथत ह

Q61) उततर (c)

सपषटीकरण

bull परमचोद क उपनयास ो म परमाशरम रोगभवम गबन

कमिभवम और ग दान शावमल ह

bull ग रा रिी ोदरनाथ िग र क दवारा रवचत उपनयास ह

bull अभी हाल ही म मोशी परमचोद की 138िी ो जयोती दश

भर म मनाई गई थी

Q62) उततर (b)

सपषटीकरण

bull ldquoवगदाrdquo पोजाब (भारत) एिो पावकसतान की

मवहलाओो क दवारा तयौहार क समय और फसल

की बिाई तथा किाई क अिसर पर वकया जान

िाला एक पारोपररक दहाती नतय ह

FC19H1003 38

bull इस नतय क माधयम स पोजाबी मवहलाऐो अपनी

परसननता परकि करती ह तथा वगदा क परदशिन क

माधयम स परि िचिसव िाल समाज म मवहलाओो

की दबी हई भािनाओो क परकि करती ह

bull चोवक इस नतय का परि ो क साथ क ई सोबोि नही ो

ह अतः किल मवहलाऐो ही इसम भाग ल सकती

bull हर साल तीज समार ह क दौरान पोजाब म वगदा

नतय वकया जाता ह

तीज भारत क कछ भाग ो म मवहलाओो क दवारा

मनाया जान िाल कई तयौहार ो क वलए एक

वयापक नाम ह

Q63) उततर (a)

सपषटीकरण

- मजम-उल-बहरीन या द समदर ो का सोगम

नामक उललखनीय रचना दारा वशक ह क दवारा

वलखी थी

- भारत क उपराषटर पवत शरी एम िकया नायड न कहा

ह वक राजकमार दारा वशक ह की रचनाएा शाोवत

और सदभाि क बिािा दन क वलए एक तारा सर त

क रप म सामन आ सकती ो ह

- उपराषटर पवत गत ििो क भला वदए गए राजकमार

दारा वशक ह क परदवशित परचवलत करन हत

आय वजत एक परदशिनी का दौरा करन क बाद एक

सभा क सोब वित कर रह थ

- इस परदशिनी का आय जन फर क इस गौवियर

(Francois Gautier) क दवारा lsquoइोवदरा गाोिी नशनल

सिर फॉर द आििसrsquo (The Indira Gandhi

National Centre for the Arts) नई वदलली म

वकया गया था

Q64) उततर (c)

सपषटीकरण

- ग मतशवर परवतमा जन भगिान बाहबली क

समवपित ह

- यह एक एक-चटटानी पतथर की मवति ह

- राषटर पवत राम नाथ क विोद न शरिणबलग ला

(कनाििक) म आय वजत वकय जान िाल भवय

अवभिक समार ह महामसतकावभिक का

उदघािन वकया था

- यह समार ह 12 ििो म एक बार ह ता ह

Q65) उततर (c)

सपषटीकरण

bull पराची घािी पराची नदी क चार ो ओर फली हई थी

bull पराची घािी िीर-िीर विलपत ह गई थी

bull पराची नदी भिनशवर स वनकलती ह

bull यह महानदी की एक सहायक नदी ह और यह

परी खदाि किक तथा जगतवसोहपर वजल ो स

ह कर बहती ह

bull इस नदी क पर कषतर क पराची घािी कहा जाता ह

bull यह नदी बोगाल की खाडी म वगरती ह

परातासतवक साकषय स पता चलता ह वक पराची घािी

सभयता हडपपा और म हनज दाड द न ो की

पिििती ह

Q66) उततर (d)

सपषटीकरण

य समारक छतरपर वजल (मधय परदश) म विोधयाचल

पिित शरोखला म सथथत ह

Q67) उततर (a)

सपषटीकरण

bull थॉिस ऑन पावकसतान नामक पसतक डॉ बी

आर अमबडकर न वलखी थी

bull डॉ बी आर अमबडकर की जयोती क अिसर पर

भारत क राषटर पवत न भारत की इस महान हसती

क शरदाोजवल अवपित की थी

bull डॉ बी आर अमबडकर न 1924 म वडपरथड

कलावसर इोसटीटयि (दवलत िगि सोथथान -

बवहषकत वहतकाररणी सभा) और 1927 म समाज

समता सोघ की थथापना की थी

bull अमबडकर का धयान वशकषा कषतर की ओर भी था

bull उनह ोन वशकषा क वनमन िगो म फलान क वलए

पीपलस एजकशन स साइिी (The Peoples

Education Society) क नाम स महाविदयालय ो क

नििकि और छातरािास ो की थथापना की थी

FC19H1003 39

Q68) उततर (b)

सपषटीकरण

bull महरगि भारतीय उपमहादवीप म एक परवसद

निपािाण बसती ह ज बलवचसतान पराोत

पावकसतान म सथथत ह

bull दचपलली (आोधर परदश) क पास नागलर नदी क

पिी ति ो पर चना पतथर क बलॉक क विशाल

विसतार म एक पिि-ऐवतहावसक रॉक आिि थथल की

ख ज की गई ह

bull इसन 1500-2000 ईसा पिि क दौरान गोिर (आोधर

परदश) म विकवसत निपािाण सभयता पर परकाश

डाला ह

Q69) उततर (c)

सपषटीकरण

bull 12िी ो सदी और 13िी ो सदी म काकाविय िोश का

उदय हआ था

bull ि पहल कलयाण क पवशचमी चालकय ो क सामोत थ

bull परारोभ म उनह ोन िारोगल (तलोगाना) क पास एक

छ ि स कषतर पर शासन वकया था

bull उनह ोन ldquoनायक वयिथथाrdquo की शरआत की थी

वजस बाद म विजयनगर क राय शासक ो न

अपनाया और विकवसत वकया था

Q70) उततर (a)

सपषटीकरण

bull गाोिीजी क अनशन स वमल मावलक ो पर दबाि

पडा था ज अोततः शरवमक ो क ितन म 35 परवतशत

की िसद करन क वलए सहमत हए थ

bull गगल (Google) न अनसया साराभाई वजनह ोन

भारत क शरवमक आोद लन म एक अगरणी भवमका

वनभाई थी की 132िी ो जयोती डडल (Doodle) का

वनमािण करक मनाई

Q71) उततर (d)

सपषटीकरण

भारत स यनसक की मानिता की अमति साोसकवतक

विरासत की परवतवनवि सची म वनमनवलसखत शावमल ह

bull कवडयटटम करल का सोसकत रोगमोच

bull मवडयिि करल का अनषठान रोगमोच और नतय

नाविका

bull िवदक मि जाप की परोपरा

bull राजथथान क कालबवलया ल क गीत और नतय

bull रामलीला रामायण का पारोपररक परदशिन

bull सोकीतिन मवणपर का अनषठान गायन ढ ल िादन

और नतय

bull रममन भारत क गििाल वहमालय का िावमिक

तयौहार और अनषठान रोगमोच

bull जाोदीयाला गर पोजाब क ठठर ो की पीतल और

ताोब क वशलप स वनवमित बतिन ो की पारोपररक कला

bull छाऊ नतय पिी भारतीय राजय ो म जनमी शासतरीय

भारतीय नतय कला

bull लददाख का बौद मि जाप िर ाोस-वहमालयी लददाख

कषतर तथा जमम-कशमीर म पवितर बौद गरोथ ो का पाठ

bull य ग

bull नौर र

bull को भ मला

Q72) उततर (b)

सपषटीकरण

bull भारत क राषटर पवत शरी राम नाथ क विोद न

वकसामा नागालड म हॉनिवबल मह रति और

राजय गठन वदिस समार ह का उदघािन वकया

था

bull हॉनिवबल मह रति का नाम भारतीय हॉनिवबल क

नाम पर पडा ह ज एक विशाल और रोगीन जोगली

पकषी ह

bull यह पकषी नागालड राजय की अविकतर जनजावतय ो

की ल ककथाओो म उसललसखत ह

bull नागालड की परमख मानयता परापत जनजावतयाा ह

अोगामी आओ चखसोग चाोग ककी रगमा और

रवलोग आवद

bull ओोग जारिा और ससिनलीस अोडमान-वनक बार

दवीप समह की जनजावतयाा ह

FC19H1003 40

Q73) उततर (c)

सपषटीकरण

bull दकन म राषटर कि शासन दसिी ो सदी क अोत तक

लगभग 200 ििो तक रहा था

bull राषटर कि शासक अपन िावमिक विचार ो म सवहषण

bull उनह ोन न किल शि िमि और िषणि िमि बसलक

जन िमि क भी सोरकषण वदया था

bull एल रा म वशि क परवसद रॉक कि मोवदर का

वनमािण नौिी ो सदी म राषटर कि राजा कषण परथम न

करिाया था

bull उसका उततराविकारी अम घििि जन था लवकन

उसन अनय िमो क भी सोरकषण परदान वकया था

bull राषटर कि ो न मसलमान वयापाररय ो क बसन की

अनमवत दी थी

bull उनह न अपन अविराजय ो म इसलाम क उपदश दन

की भी अनमवत दी थी

bull अभी हाल ही म पाोडिलागटटा (तलोगाना) क

परागवतहावसक चटटान वचतर ो क कषरण की बिती हई

घिनाएा एक गोभीर वचोता का वििय ह

bull यह परागवतहावसक चटटान क नकसान पहाचा

सकता ह

bull पाोडिलागटटा वनमनवलसखत क वलए जाना जाता ह

- 10000 ईसा पिि स 8000 ईसा पिि क वचवतरत

चटटानी आशरय ो क वलए

- राषटर कि काल क एक 8 िी ो सदी क

वशलालख क वलए और

- 12िी ो सदी क काकविय सामराजय क वभवतत

वचतर ो क वलए

Q74) उततर (b)

सपषटीकरण

bull 1828 म राजा राम म हन रॉय न एक नय िावमिक

समाज बरहम सभा की थथापना की थी वजस बाद

म बरहम समाज क नाम स जाना गया था

bull दिदरनाथ िग र न ततवब विनी सभा की अधयकषता

की थी ज आधयासिक सतय की ख ज म सोलि

थी

bull इसका उददशय वहोद िमि क शद करन का और

एकशवरिाद (एक ईशवर म आथथा) का परचार करना

था

bull नय समाज की थथापना क आिार थ कारण

(तकि ) क द सतमभ तथा िद और उपवनिद

bull अभी हाल ही म सािारण बरहम समाज का कछ

काननी मदद ो क लकर पवशचम बोगाल सरकार क

साथ काननी वििाद चल रहा ह

Q75) उततर (c)

सपषटीकरण

bull भारत म वचशती वसलवसल की थथापना खवाजा

म इनददीन वचशती क दवारा की गयी थी

bull ि 1192 ईसवी क आसपास भारत आय थ

bull वचशतीय ो क बारहिी ो शताबदी क उततरािि म भारत

म आन िाल सफीय ो क समह ो म सबस

परभािशाली माना जाता ह

bull उनह ोन थथानीय िातािरण क साथ सफलतापििक

अनकलन वकया और उनह ोन भारतीय भसकत

परोपराओो क कई पहलओो क अपनाया

bull अजमर म सफी अपरकि खवाजा म इनददीन वचशती

की ऐवतहावसक दरगाह क एक नया रप दन की

तयारी की जा रही ह

bull इस 13िी ो शताबदी की दरगाह क ldquoसवचछ

आइकॉवनक थथल ोrdquo (Swacch Iconic Places) म

शावमल वकया गया ह ज परवतवषठत विरासत

आधयासिक और साोसकवतक थथान ो पर क वदरत

य जना ह

FC19H1003 41

ANSWERS amp EXPLANATION OF

NCERT History Class VI-X + Current Affairs

(FC19E1003)

Q1) Answer c

Explanation

Rigveda consists of more than a

thousand hymns dedicated to gods and

goddesses These hymns were

composed by sages and learnt by men

however a few were composed by

women like Apala Ghosa Lopamudra

Maitreyi and Gargi

Rigveda consists of many hymns in the

form of dialogues We get an example of

a dialogue between a sage named

Vishwamitra and two rivers (Beas and

Sutlej) that were worshipped as

goddesses This suggests that he

belonged to the Vedic period

Q2) Answer b

Explanation

Traces of ash have been found from

Kurnool Caves suggesting that people

were familiar with the use of fire

It is situated in Andhra Pradesh

Q3) Answer c

Explanation

Burzahom is a prehistoric site in

present day Kashmir where people built

pit houses which were dug into the

ground with steps leading into them

These may have provided shelter in cold

weather

Q4) Answer c

Explanation

Epigraphy is defined as the study of

inscriptions

Manuscriptology is the study of history

and literature through the use of hand

written documents

Palaeography refers to the study of

ancient writing systems and the

deciphering and dating of historical

manuscripts

Numismatics refers to the study of

coins

Q5) Answer a

Explanation

Charaka Samhita was written by

Charaka and is an important book on

Ayurveda and medicine

He was a practitioner of the traditional

system of Indian medicine known as

Ayurveda

Charaka is thought to have flourished

sometime between the 2nd century BCE

and the 2nd century CE

Q6) Answer b

Explanation

Bhaga refers to the tax on crops which

was fixed at 16th of the production

Kammakaras is the term used for the

landless agricultural labour class

Ashvamedha also known as horse

sacrifice is a ritual where a horse is let

loose to wander freely and it was

guarded by the rajarsquos men

Q7) Answer (d)

Explanation

In the Rigvedic period horses were

yoked to chariots that were used in

battles fought to capture land cattle

etc This suggests that the use of horse

chariots began much before the period

of Mahajanapadas

The battles were fought in the Rigvedic

period for cattlersquos lands water an even

to capture people Most men took part

in these wars however there was no

regular army but there were assemblies

where people met and discussed

matters of war Regular armies became

a feature in the Mjahajanapada period

including vast armies of foot soldiers

chariots and elephants

RAUSIAS-FC19E1003 42

Q8) Answer (a)

Explanation

Buddha belonged to the Sakya clan and

passed away at Kusinara

Buddha taught in Prakrit which was the

common language of people

Q9) Answer c

Explanation

There were six schools of philosophy in

ancient India These are known as

Vaishesika Nyaya Samkhya Yoga

Purva Mimansa and Vedanata or Uttara

Mimansa They were founded by sages

Kanada Gautama Kapila Patanjali

Jamini and Vyasa respectively

Q10) Answer b

Explanation

The teachings of Mahavira were

compiled at Valabhi in 6th century AD

Q11) Answer (c)

Explanation

Chanakya is traditionally identified as

Kautilya or Vishnugupta who authored

the ancient Indian political treatise the

Arthashastra

Q12) Answer d

The national emblem of India is an

adaptation of the Lion Capital atop the

Ashoka Pillar of Sarnath Uttar Pradesh

and is combined with the National

Motto Satyameva Jayate

The Rampurva Bull gets the name from

the site of its discovery Rampurva in

Bihar

It is noted for its delicately sculpted

model demonstrating superior

representation of soft flesh sensitive

nostrils alert ears and strong legs It is

a mixture of Indian and Persian

elements

Sankissa is situated in Uttar Pradesh

India

Q13) Ans(a)

Kunwar Singh was a notable leader during the Revolt of 1857 He belonged

to a royal house of Jagdispur Bihar

Q14) Answer b

Explanation

The term Vellalar was used for large

landowners

Q15) Answer c

Explanation

Arikamedu was a coastal settlement

where ships unloaded goods from

distant lands Finds here include a

massive brick warehouse pottery

including amphorae and Arretine ware

Roman lamps glassware and gems have

also been found at the site

Q16) Answer a

Explanation

Muvendar is a Tamil word mentioned in

Sangam poems meaning three chiefs

used for the heads of three ruling

families the Cholas Cheras and

Pandyas

Q17) Ans (c)

Several tribal or kin-based assemblies

such as the Sabha Vidatha and gana

are mentioned in the Rig-veda The

Sabha and the samiti mattered a great

deal in early Vedic times so much so

that the chiefs or the kings showed an

eagerness to win their support

Q18) Ans (a)

Jainism recognised the existence of the

gods but placed them lower than the

jina and did not condemn the varna

system as Buddhism did

Q19) Answer (d)

Explanation

Cholas and Pandyas had developed

powerful coastal cities The most

important city of Cholas was Puhar or

Kaveripattinam and Madurai was the

capital of Pandyas

Q20) Answer b

Explanation

Buddhacharita is the biography of

Buddha and was written by

RAUSIAS-FC19E1003 43

Ashvaghosha

Q21) Answer (a)

Explanation

Tamil poet Appar was a Shiva devotee

So he was a Nayanar saint

Q22) Answer d

Explanation

Samudragupta was a prominent Gupta

ruler whose coins depict him playing a

veena indicating his love for music We

get important historic information from

his Allahabad Prashasti which was

composed by his court poet Harisena

Q23) Answer (b)

Explanation

Vikrama Samvat was founded by

Chandragupta II in the 58 BC as a

mark of victory over the Shakas and

assumed the title of Vikramaditya

Banabhatta wrote Harshavardhanarsquos

biography the Harshacharita in

Sanskrit

Q24) Answer c

Explanation

Sandhi-vigrahika was the minister of

war and peace

Sarthavaha was the leader of the

merchant caravans

Q25) Answer a

Explanation

Xuan Zang (Hsuan-tsang) was a

Chinese traveller who came during the

reign of Harshavardhana

In the decade that began in 630 AD

Xuan Zang came to India through

Kashmir after visiting Central Asia Iran

and Afghanistan

He travelled from north to east and lived

in Bihar for a couple of years

At Nalanda University Xuan Zang

interacted with students and scholars

mastered local languages and

discovered Buddhist stupas

Q26) Answer c

Explanation

Pradakshina patha is a circular path

laid around a stupa in Buddhist

architecture While the rest are a part of

temple architecture

Q27) Answer d

Explanation

All the above-mentioned temples have

an elaborate use of bricks (baked

bricks) along with stone

Q28) Ans (c)

Muhammad Quli Qutab was the Sultan

of Golconda He was a contemporary of

Akbar was very fond of literature and

architecture

The Sultan was a great poet and he

wrote in Dakhini Urdu Persian and

Telgu and has left an extensive diwan or

collection

Recently the Archaeological Survey of

India (ASI) will be using Ground

Penetrating Radar (GPR) to map the

contours of the area around the Bagh-e-

Naya Qila excavated garden inside the

Golconda Fort in Telangana

Q29) Answer a

Explanation

Silappadikaram is a famous Tamil epic

which was written by Ilango around

1800 years ago It is a story of a

merchant named Kovalan who fell in

love with a courtesan named Madhavi

Manimekalai tells the story of the

daughter of Kovalan and Madhavi

Q30) Answer (a)

Explanation

Charaka is the author of Charaka

Samhita which is an important work of

Ayurveda and medicines

Brahmaguptarsquos fame rests mostly on his

Brahma-sphuta-siddhanta which was

an astronomical work It was translated

into Arabic in Baghdad and had a major

impact on Islamic mathematics and

astronomy

Late in his life Brahmagupta wrote

Khandakhadyaka which was an

RAUSIAS-FC19E1003 44

astronomical handbook that employed

Aryabhatarsquos system of starting each day

at midnight

Q31) Answer (c)

Explanation

Amir Khusrau was a famous sufi

musician poet and scholar In 1318 he

noted that there was different language

in every region of this land (Hindustan)

Lahori Kashmiri Dvarsamudri (in

Southern Karnataka) Telangana (in

Andhra Pradesh) Gujari (in Gujarat)

Marsquobari (in Tamil Nadu) Awadhi (in

eastern Uttar Pradesh) and Hindawai (in

the area around in Delhi) etc He went

to explain that Sanskrit did not belong

to any region and that only brahmans

knew it

Q32) Answer c

Explanation

Hiranyagarbha refers to the golden

womb When this ritual was performed

with the help of Brahmanas it was

thought to lead to the rebirth of the

sacrificer as a Khastriya

Q33) Answer d

Explanation

Kadamai refers to a tax on land

revenue

Gwalior Prashasti describes the exploits

of Nagabhata who was a Pratihara king

Q34) Answer b

Explanation

Rajatarangini is a Sanskrit text written

by Kalhana in the 12th century

It was historical chronicle of early India

It is justifiably considered to be the best

and most authentic work of its kind

It covers the entire span of history in

the Kashmir region from the earliest

times to the date of its composition

Q35) Answer c

Explanation

ldquoUrrdquo was the general assembly of the

village ldquoUrrdquo consisted of all the

taxpaying residents of an ordinary

village

Q36) Answer (a)

Explanation

Tarikh was a form of history writing in

the Delhi Sultanate The authors of

tawarikhs were learned men which

included secretaries administrators etc

Q37 Answer (a)

Explanation

Alauddin chose to pay his soldiers salaries in cash rather than iqtas The soldiers would buy their supplies from merchants in Delhi and it was thus feared that merchants would raise their prices To stop this Alauddin controlled the prices of goods in Delhi Prices were carefully surveyed by officers and merchants who did not sell at the prescribed rates were punished

Q38) Answer (d)

Explanation

Delhi first became the capital of a

kingdom under the Tomara Rajputs

who were defeated in the middle of the

twelfth century by the Chauhans (also

referred to as Chahamanas) of Ajmer

It was under the Tomaras and

Chauhans that Delhi became an

important commercial centre Many rich

Jaina merchants lived in the city and

constructed several temples Coins

minted here called dehliwal had a wide

circulation

Q39) Answer (c)

Explanation

Moth ki Masjid was built in the reign of

Sikandar Lodi by his minister

Begumpuri mosque built in the reign of

Muhammad Tughluq was the main

mosque of Jahanpanah the ldquoSanctuary

of the Worldrdquo and his new capital in

Delhi

Quwwat al ndash Islam mosque was

enlarged by Iltutmish and Alauddin

Khalji The minar was built by three

Sultansndash Qutbuddin Aybak Iltutmish

and Firuz Shah Tughluq

RAUSIAS-FC19E1003 45

Q40) Answer (c)

Explanation

Under the Mughals mansabdar was

referred to an individual who held a

mansab ie rank and he received his

salary as revenue assignments called

jagirs

Q41) Ans (b)

The Quit India Movement was a

spontaneous revolt of people against

British rule

The All India Congress Committee met

at Bombay on 8 August 1942 It passed

the famous resolution Quit India and

proposed the starting of a non-violent

mass struggle under Gandhis

leadership to achieve this aim But on

the very next day Gandhi and other

eminent leaders of the Congress were

arrested The Congress was once again

declared illegal

Q42) Ans (c)

The Simon Commission refers to a

group of seven MPs from the United

Kingdom constituted to suggest

constitutional reforms for British India

The Commission consisted of only

British members headed by one of the

senior British politicians Sir John

Simon

So the people of India agitated against

the arrival of Simon Commission

Q43) Ans (a)

He was widely known for his

unfavourable opinion of the economic

consequences of the British rule in

India

In his many writings and speeches and

especially in Poverty and Un-British

Rule in India Naoroji argued that India

was too highly taxed and that its wealth

was being drained away to England

He did not interpret the ancient Indian

texts and restored the self-confidence of

Indians And also he did not stress the

need for eradication of all the social

evils before anything else

Q44) Ans (c)

In August 1932 Prime Minister

MacDonald announced his Communal

Award Great Britainrsquos unilateral

attempt to resolve the various conflicts

among Indiarsquos many communal

interests

The award which was later

incorporated into the act of 1935

expanded the separate-electorate

formula reserved for Muslims to other

minorities including Sikhs Indian

Christians Anglo-Indians Europeans

distinct regional groups Gandhi

undertook a ldquofast unto deathrdquo against

that offer which he viewed as a

nefarious British plot to divide the

Indian society

Q45) Ans (b)

In British India apart from existing

imports and exports there was also a

particular amount of money which

colonial India contributed towards

administration maintenance of the

army war expenses pensions to retired

officers and other expenses accrued by

Britain towards maintenance of her

colony These were known as Home

charges and were paid for almost

entirely by India

The Home charges was made of

following components-

- Interest payable on Indian debt

- Dividend to shareholders of East

India Company

- Funds used to support the India

Office in London

- Funds used to pay salaries and

pensions of British personnel

engaged in India

- Interest on the railways

- Civil and military charges

- Store purchases in England

Q46) Ans (b)

The Lahore session of the Indian

National Congress was held in 1929

under the Presidentship of Jawaharlal

Nehru

The Lahore session of the Indian

National Congress witnessed significant

RAUSIAS-FC19E1003 46

developments in the Indian national

movement

- First the election of Jawaharlal

Nehru to the post of Presidentship of

the Congress was a clear indication

of the growing strength of the

Leftists in the Congress

- Secondly it was in this session that

the Congress for the first time raised

the demand for complete

independence Such demand was

not raised from the Congress

platform earlier

Q47) Ans (b)

It did not provide for separate

electorates for any community or

weightage for minorities However it did

allow for the reservation of minority

seats in provinces having minorities of

at least ten per cent but this was to be

in strict proportion to the size of the

community

There was no provision for complete

Independence for India

Q48) Ans (c)

The religion of early Vedic Aryans was

primarily of worship of nature and

Yajnas

The early Aryan religion was kind of

nature worship Actually the forces

around them which they could not

control or understand were invested

with divinity and were personified as

male or female gods And they

performed some Yajnas also

Q49) Ans (b)

The roads and river-routes were not

immune from robbery It is notable that

Yuan Chwang (Hiuen Tsang) was

robbed of his belongings during

Harshvardanarsquos period

Q50) Ans (c)

Q51) Ans (b)

Purandara Dasa was a saint and great

devotee of Lord Krishna

There is much speculation about where

Purandara Dasa regarded as the

Pitamaha of Carnatic music was born

Recently an expert committee

constituted by the Kannada University

Hampi has come to the conclusion that

Kshemapura Shivamogga district

Karnataka is the birth place of

Purandara Dasa

Q52) Ans (c)

Sri Tyagaraja Sri Shyama Shastry and Sri Muthuswami Dikshitar are considered the trinity of Carnatic music and with them came the golden age in Carnatic music in the 18th-19th

century

Q53) Ans d)

Recently a rare sarcophagus (stone

coffin) which is 2000 years old from the

Iron AgendashMegalithic era was discovered

from a rock-cut cave at Viyur village of

Kollam near Koyilandy in Kozhikode

district Kerala

The coffin containing bone fragments

was found during an excavation ldquoSo

far such a rare finding has been

discovered only from two sites

in Kerala Both these sarcophagi were

recovered from Megalithic sites at

Chevayur and Atholi also in Kozhikode

district

Q54) Ans a)

The megalithic culture in South India was a full-fledged Iron Age culture

Q55) Ans d)

The Cholas Pandyas and Keralaputras

(Cheras) mentioned in Ashokan

inscriptions were probably in the late

megalithic phase of material culture

Q56) Ans d)

Q57) Ans (b)

Raj Kumar Shukla followed Gandhiji all

over the country to persuade him to

come to Champaran to investigate the

problem associated with tinkathia

system

RAUSIAS-FC19E1003 47

Brij Kishore Rajendra Prasad Mahadev

Desai and Narhari Parikh accompanied

Gandhi ji during the Champaran

Satyagraha

Q58) Ans (b)

The Satvahanas started the practice of granting tax-free villages to brahmanas and Buddhist monks

Q59) Ans c)

The objectives of the Programme are

listed as under

- Developing basic tourism

infrastructure

- Promoting cultural and heritage

value of the country to generate

livelihoods in the identified regions

- Enhancing the tourist attractiveness

in a sustainable manner by

developing world-class

infrastructure at the heritage

monument sites

- Creating employment through active

involvement of local communities

- Harnessing tourism potential for its

effects on employment generation

and economic development

- Developing sustainable tourism

infrastructure and ensuring proper

Operations and maintenance

therein

Q60) Ans (b)

The Tribal Cooperative Marketing

Development Federation of India

(TRIFED) came into existence in 1987

It is a national-level apex organization

functioning under the administrative

control of Ministry of Tribal Affairs

Govt of India

TRIFED has its registered and Head

Office located in New Delhi

Q61) Ans (c)

Premchandrsquos novels include

Premashram Rangabhumi Ghaban

Karmabhumi and Godan

Gora is a novel written by Rabindranath

Tagore

138th birth anniversary of Munshi

Premchand was celebrated across the

country

Q62) Ans (b)

Giddha is a traditional pastoral dance

performed by the women of the Punjab

India and Pakistan at festival times

and at the sowing and reaping of the

harvest

By this dance the Punjabi women

reveal their joy expel their suppressed

feelings in a male dominated society

through the performance of Giddha

Since this dance has nothing to do with

men only women can participate in it

During the Teej celebrations Giddha

dance is celebrated in Punjab every

year Teej is a generic name for a

number of festivals that are celebrated

by women in some parts of India

Q63) Ans (a)

Dara Shukoh wrote the remarkable

work called ldquoMajma-ul-Bahrainrdquo or the

ldquoThe confluence of two seasrdquo

The Vice President of India Shri M

Venkaiah Naidu has said that Prince

Dara Shukohrsquos writings can come as a

refreshing source for infusing peace and

harmony He was addressing the

gathering after visiting the exhibition

that showcases the forgotten Prince of

yesteryears Dara Shukoh organized by

Mr Francois Gautier at Indira Gandhi

National Centre for the Arts in New

Delhi

Q64) Ans (c)

The statue Gommateshwara is

dedicated to the Jain God Bahubali

It is a monolithic statue

President Ram Nath Kovind

inaugurated the grand anointing

ceremony mdash Mahamastakabhisheka mdash

held once in 12 years at

Shravanabelagola (Karnataka)

Q65) Ans (c)

Prachi Valley had come up around the

Prachi river Prachi Valley gradually

disappeared

RAUSIAS-FC19E1003 48

The Prachi river originates from

Bhubaneswar

It is a tributary of the Mahanadi and

flows through the districts of Puri

Khurda Cuttack and Jagatsinghpur

and the entire region of the river is

termed as the Prachi Valley

It falls into the Bay of Bengal

Archaeological evidence shows that the

Prachi Valley Civilisation predates both

Harappa and Mohenjo-Daro

The Prachi river originates from

Bhubaneswar

Q66) Ans (d)

These monuments are located in

Chhatarpur district Madhya Pradesh

within Vindhya mountain range

Q67) Ans (a)

The book lsquoThoughts on Pakistanrsquo was

written by Dr BR Ambedkar

On the occasion of the birth anniversary

of Dr BR Ambedkar the president of

India pays homage to this icon of India

In 1924 he founded the Depressed

Classes Institute (Bahishkrit Hitkarini

Sabha) and in 1927 the Samaj Samata

Sangh

Another area of attention for Ambedkar

was education For its spread among

the low classes he set up a network of

colleges by the name of Peoples

Education Society and founded hostels

Q68) Ans(b)

Mehrgarh is a famous Neolithic

settlement in the Indian subcontinent

which is situated in Baluchistan

province Pakistan

A pre-historic rock art site is discovered

in the vast expanse of limestone blocks

on the eastern banks of Naguleru river

near Dachepalli (Andhra Pradesh) It

has thrown light on the Neolithic

civilisation that flourished in Guntur

(Andhra Pradesh) during 1500-2000

BC

Q69) Ans (c)

The 12th and the 13th centuries saw

the emergence of the Kakatiyas They

were at first the feudatories of the

Western Chalukyas of Kalyana Initially

they ruled over a small territory near

Warangal (Telangana)

They introduced Nayakships which was

later adopted and developed by the

Rayas of Vijayanagara

Q70) Ans (a)

The fast had effect of putting pressure

on mill owners who finally agreed to

give the workers a 35 per cent increase

in wages

Google celebrated with a doodle the

132nd birth anniversary of Anasuya

Sarabhai who played a pioneering role

in Indiarsquos labour movement

Q71) Ans (d)

The UNESCOrsquos list of the representative

list of the intangible cultural heritage of

humanity from India are

- Koodiyattam Sanskrit Theatre of

Kerala

- Mudiyettu ritual theatre and dance

drama of Kerala

- Tradition of Vedic Chanting

- Kalbelia folk songs and dances of

Rajasthan

- Ramlila Traditional Performance of

the Ramayana

- Sankirtana ritual singing

drumming and dancing of Manipur

- Ramman religious festival and

ritual theatre of the Garhwal

Himalayas India

- Traditional brass and copper craft of

utensil making among the Thatheras

of Jandiala Guru Punjab India

- Chhau dance classical Indian dance

originated in the eastern Indian

states

- Buddhist chanting of Ladakh

recitation of sacred Buddhist texts

in the trans-Himalayan Ladakh

region Jammu and Kashmir India

- Yoga

- Nouroz

- Kumbh Mela

RAUSIAS-FC19E1003 49

Q72) Ans(b)

The President of India Shri Ram Nath Kovind inaugurated the Hornbill Festival and State Formation Day celebrations of Nagaland in Kisama

The festival is named after the Indian hornbill the large and colourful forest bird which is displayed in the folklore of most of the states tribes

The major recognized tribes of Nagaland are Angami Ao Chakhesang Chang

Kuki Rengma and Zeling etc

Onge Jarawa and Sentinelese are the

tribes of Andman amp Nicobar Islands

Q73) Ans (c)

The Rashtrakutas rule in the Deccan lasted for almost two hundred years till the end of the tenth century The Rashtrakutas rulers were tolerant in their religious views and patronized not only Shaivism and Vaishnavism but

Jainism as well

The famous rock-cut temple of Shiva at Ellora was built by one of the Rashtrakutas kings Krishna I in the ninth century His successor Amoghavarsha was a Jain but he also

patronized other faiths

The Rashtrakutas allowed Muslims traders to settle and permitted Islam to

be preached in their dominions

Recently increasing defacement at the prehistoric rock paintings of Pandavulagutta Telangana has created a cause for grave concern It can spoil

the prehistoric rock

Pandavulagutta is home to

- Painted rock shelters dating to

10000 BC-8000 BC

- An 8th century inscription of the

Rashtrakuta period and

- Painted frescoes from the 12th century Kakatiya empire

Q74) Ans (b)

In 1828 Raja Ram Mohan Roy founded a new religious society the Brahma Sabha later known as the Brahmo

Samaj

Debendranath Tagore headed the Tattvabodhini Sabha which was

engaged in search of spiritual truth

Its purpose was to purify Hinduism and to preach monotheism or belief in one God

The new society was to be based on the twin pillars of reason and the Vedas and

Upanishads

Recently Sadharan Brahmo Samaj (SBS) has entered into a legal battle with the West Bengal government due

to some legal issue

Q75) Ans (c)

The Chishti order was established in India by Khwaja Moinuddin Chishti who came to India around 1192 The Chishtirsquos are considered to be the most influential of the groups of Sufis who migrated to India in the late twelfth century They adapted successfully to the local environment and adopted several features of Indian devotional

traditions

The historical dargah of Sufi mystic Khwaja Moinuddin Chishti in Ajmer is all set to get a facelift This 13 th century dargah has been included among the Swachh Iconic Places a clean-up initiative focused on iconic

heritage spiritual and cultural places

Page 31: GENERAL STUDIES (PAPER I) · Test is part of Rau’s IAS Test series for Preliminary Exam 2019 FOUNDATION + CURRENT AFFAIRS GENERAL STUDIES (PAPER –I) FOUNDATION TEST –III TOPIC:

FC19H1003 31

Q15) उततर (c)

सपषटीकरण

- अररकमड एक तिीय बसती थी जहाो दर दश ो स

आन िाल जहाज ो का माल उतारा जाता था

- यहाो पर ईोि ो का एक विशाल ग दाम वमटटी क

बतिन (वजनम एमफ रा - द हरी मवठय ो का लोबा

घडा - शावमल ह) और एरिाइन (Arretine)

मदभाोड पाए गए थ

- इस थथान पर र मन दीपक काोच क बन पातर और

रतन भी पाए गए थ

Q16) उततर (a)

सपषटीकरण

- मिनदर सोगम कविताओो म उसललसखत एक

तवमल शबद ह वजसका अथि ह ldquoतीन परमखrdquo

- यह तीन सततारि पररिार ो क मसखयाओो क वलए

परय ग वकया जाता ह च ल चर और पाणडय

Q17) उततर (c)

सपषटीकरण

- ऋग िद म सभा विदाथा तथा गण जसी

जनजावतय ो पर अथिा किोब पर आिाररत

सभाओो का उललख ह

- आरसमभक िवदक काल म सभाओो और सवमवतय ो

का विशि महतव ह ता था

- यहाा तक की मसखया अथिा राजा भी उनका

समथिन परापत करन क वलए आतर रहत थ

Q18) उततर (a)

सपषटीकरण

- जन िमि न ईशवर क अससततव क मानयता त दी ह

वकनत उसन ईशवर क वजना क पद स नीच रखा

- जन िमि न बौद िमि की तरह िणि परणाली की

भरतिना नही ो की थी

Q19) उततर (d)

सपषटीकरण

- च ल ो और पाणडय ो न शसकतशाली तिीय शहर ो का

विकास वकया था

- च ल ो का सबस महतवपणि शहर पहार (या

कािरीपटटीनम) था |

- मदरई पाणडय ो की राजिानी थी

Q20) उततर (b)

सपषटीकरण

- ldquoबदचररतrdquo बद का जीिन-ितताोत ह

- इस अशवघ ि क दवारा वलखा गया था

Q21) उततर (a)

सपषटीकरणः

- तवमल कवि अपपर भगिान वशि क भकत थ

- इस परकार ि एक नयनार सोत थ

Q22) उततर (d)

सपषटीकरणः

- समदरगपत एक परवसद गपत शासक था

- उसन वसक ो पर िीणा बजात हए अपनी छवि

अोवकत करिाई थी

- यह सोगीत क परवत उसक परम क दशािता ह

- हम उसकी इलाहाबाद परशससत स महतवपणि

ऐवतहावसक जानकारी वमलती ह वजसकी रचना

उसक दरबार क कवि हररसन न की थी

Q23) उततर (b)

सपषटीकरणः

- विकरम सोित की शरआत ििि 58 ईसा पिि म

चनदरगपत वदवतीय न की थी

- यह शक ो पर उसकी जीत और उस विकरमावदतय

की पदिी वमलन क उपलकषय म आरमभ वकया गया

था

FC19H1003 32

- बानभटट न हिििििन का जीिन-ितताोत हििचररत

(ज सोसकत म थी) वलखी थी

Q24) उततर (c)

सपषटीकरणः

- सोवि-विगरावहका यद एिो शाोवत का मोतरी

- साथििाह वयापाररय ो क कावफल ो का नता

Q25) उततर (a)

सपषटीकरणः

- जआन झाोग (हसआन रताोग ndash Hsuang Tsang)

एक चीनी यातरी था ज हिििििन क शासनकाल म

भारत आया था

- ििि 630 ईसवी स ज दशक आरमभ हआ था उसम

जआन झाोग मधय एवशया ईरान और

अफग़ावनसतान की यातरा करन क पशचात कशमीर

क रासत स भारत आया था

- उसन उततर स पिि तक की यातरा की और िह

लगभग 2 ििि वबहार म रहा

- जआन झाोग न नालनदा विशवविदयालय म विदयावथिय ो

और विदवान ो क साथ पारसपररक विचार-विमशि

वकया थथानीय भािाओ ा म वनपणता परापत की तथा

बौद सतप ो की ख ज की

Q26) उततर (c)

सपषटीकरणः

- परदवकषणा पथ बौद िासतकला म सतप क चार ो

ओर बनाया जान िाला एक घमािदार पथ ह ता

- परशन म वदए गए बाकी क तीन ो ततव वहोद मसनदर ो की

िासतकला क भाग ह

Q27) उततर (d)

सपषटीकरणः

परशन म वदए गए सभी मोवदर ो म वयापक रप स

ईोि ो (पकी ईोि ो) का परय ग पतथर ो क साथ हआ

Q28) उततर (c)

सपषटीकरण

- महममद कली कतब शाह ग लकणडा का सलतान

था

- िह अकबर का समकालीन था

- सावहतय और िासतकला म उसकी अतयाविक

रवच थी

- िह एक महान कवि था

- िह दसखनी उदि फारसी और तलग म वलखता था

- उसन अपन पीछ एक विसतत वदिान (सोगरह)

छ डा ह

- अभी हाल ही म तलोगाना म ग लकणडा क वकल

क अनदर खदाई वकय गए बाग-ए-नाया वकला

बाग क चार ो ओर रप-रखा क मानवचतरण क

वलए भारतीय परातासतवक सिकषण (The

Archaeological Survey of India ndash ASI)

गराउणड पनीिर विोग रडार (Ground Penetrating

Radar) का परय ग करगा

Q29) उततर (a)

सपषटीकरणः

- वसलपपावदकारम एक तवमल महाकावय ह वजसकी

रचना इलाोग क दवारा लगभग 1800 ििि पिि की

गई थी

- यह क िलन नामक एक वयापारी की कहानी ह

ज माििी नामक एक गवणका (िशया) स परम

करन लगा था

- मवनमकलाई क िलन और माििी की पतरी की

कहानी ह

Q30) उततर (a)

सपषटीकरण

- चरक आयिद और वचवकरता की एक महतवपणि

रचना चरक सोवहता क लखक ह

- बरहमगपत क अपनी रचना बरहम-सफि-वसदानत

(ज एक खग लीय रचना ह) क कारण परवससद

वमली

FC19H1003 33

- बगदाद म इसका अनिाद अरबी भािा म वकया

गया था

- इसका इसलावमक गवणत और खग ल-विजञान पर

महतवपणि परभाि पडा था

- बाद म अपन जीिनकाल म बरहमगपत न

ldquoखोडखयाकrdquo वलखी ज एक खग लीय पससतका

(एक छ िी पसतक) थी

- इसम आयिभटट की अिि-रावतर क परतयक वदन की

शरआत परणाली का परय ग वकया गया था

Q31) उततर (c)

सपषटीकरण

- अमीर खसर एक परवसद सफी सोगीतकार कवि

और विदवान थ

- 1318 म उनह ोन पाया वक इस भवम (वहोदसतान) क

हर कषतर म अलग-अलग भािा थी लाहौरी

कशमीरी दवारसमदरी (दवकषणी कनाििक म)

तलोगाना (आोधर परदश म) गजरी (गजरात म)

माबारी (तवमलनाड म ) अििी (पिी उततर परदश

म) और वहोदिी (वदलली क आस-पास क कषतर म)

आवद

- उनह न यह बताया वक सोसकत वकसी भी कषतर स

सोबोवित नही ो थी और किल बराहमण ही इस भािा

का जञान रखत थ

Q32) उततर (c)

सपषटीकरण

- वहरणय-गभि सववणिम गभि क सोदवभित करता ह

- जब बराहमण ो की सहायता स यह अनषठान वकया

जाता था त यह माना जाता था वक बवल दन िाल

का कषवतरय क रप म पनजिनम ह गा

Q33) उततर (d)

सपषटीकरण

- कदमई भवम राजसव पर कर क सोदवभित करता

- गवावलयर परशससत म नागभि क दवारा वकय गए

श िण का िणिन वकया गया ह |

- नागभि एक परवतहार राजा था

Q34) उततर (b)

सपषटीकरण

- राजतरो वगनी 12िी ो शताबदी म कलहन क दवारा

रवचत एक सोसकत पसतक (िकसट) ह

- यह परारसमभक भारत की ऐवतहावसक इवतितत थी

- तकि सोगत रप स इस अपन परकार की सिोततम

और सिािविक विशवसनीय कवत माना जाता ह

- यह कशमीर कषतर क पराचीनतम समय स लकर

उसकी रचना की तारीख तक क समपणि इवतहास

का आचछादन करती ह

Q35) उततर (c)

सपषटीकरण

- गााि की आम सभा क ldquoउरrdquo कहा जाता था

- ldquoउरrdquo म गााि क सभी कर दन िाल वनिासी

शावमल ह त थ

Q36) उततर (a)

सपषटीकरण

- वदलली सलतनत म ldquoतारीखrdquo इवतहास लखन का

एक रप था

- ldquoतािरीखrdquo क लखक विदवान परि ह त थ वजनम

सवचि परशासक इतयावद शावमल थ

Q37) उततर (a)

सपषटीकरण

- अलाउददीन सखलजी अपन सवनक ो क ितन का

भगतान नकद म करता था न वक इकता क रप

- सवनक अपना सामान वदलली म वयापाररय ो स

खरीदत थ अतः इस बात का भय था वक वयापारी

कही ो िसतओो का मलय न बिा द

- इसकी र कथाम क वलए अलाउददीन सखलजी न

वदलली म कीमत ो क वनयसित वकया

FC19H1003 34

- अविकारीगण धयानपििक मलय ो का सिकषण करत

थ तथा ज वयापारी वनिािररत मलय पर माल नही ो

बचत थ उनक दसणडत वकया जाता था

Q38) उततर (d)

सपषटीकरण

- वदलली सििपरथम त मर राजपत ो क अिीन उनक

सामराजय की राजिानी बनी थी

- 12िी ो शताबदी क मधय म अजमर क चौहान ो

(वजनह चाहमान ो क नाम स भी जाना जाता ह) न

त मर राजपत ो क परावजत वकया था

- त मर ो और चौहान ो क अिीन वदलली एक

महतवपणि िावणसजयक क दर बन गया था

- कई जन वयापारी यहाा रहन लग थ और उनह ोन

कई मोवदर भी बनिाए

- यहाा पर मवदरत वसक वजनह ldquoदहलीिालrdquo क नाम

स जाना जाता था वयापक रप स परचलन म थ

Q39) उततर (c)

सपषटीकरण

- म ठ की मसिद का वनमािण वसको दर ल दी क

राजयकाल म उसक मिी क दवारा करिाया गया

था

- बगमपरी मसिद का वनमािण महममद तगलक क

शासनकाल म हआ था

- यह मसिद विशव का पणयथथान (The

Sanctuary of the World) और वदलली म महममद

तगलक की नई राजिानी जहाोपनाह की मखय

मसिद थी

- कववत- अल - इसलाम मसिद का विसतार

इलतसिश और अलाउददीन सखलजी न वकया था

- मीनार का वनमािण तीन सलतान ो कतबददीन ऐबक

इलतसिश और वफर ज शाह तगलक क दवारा

करिाया गया था

Q40) उततर (c)

सपषटीकरण

- मगल ो क अिीन मनसबदार शबद उस वयसकत क

वलए सोदवभित वकया जाता था वजसक पास मनसब

(अथाित पद) ह ता था

- उस अपना ितन राजसव कायो वजनह जागीर कहत

थ क रप म परापत ह ता था

Q41) उततर (b)

सपषटीकरण

- ldquoभारत छ ड आोद लनrdquo वबरविश शासन क

सखलाफ ल ग ो का एक सवाभाविक विदर ह था

- असखल भारतीय काोगरस सवमवत न 8 अगसत 1942

क बमबई म एक बठक का आय जन वकया था

- इस बठक म परवसद सोकलप ldquoभारत छ ड rdquo क

पाररत वकया गया और इस उददशय क परापत करन

क वलए गाोिी क नततव म एक अवहोसक जन सोघिि

आोद लन की शरआत का परसताि वदया गया

- लवकन अगल ही वदन गाोिी और काोगरस क अनय

परमख नताओो क वगरफतार कर वलया गया

- काोगरस क एक बार वफर अिि घ वित वकया गया

था

Q42) उततर (c)

सपषटीकरण

- साइमन कमीशन यनाइविड वको गडम क सात

साोसद ो का एक समह था

- इस वबरविश भारत क वलए सोििावनक सिार ो का

सझाि दन क वलए गवठत वकया गया था

- इस आय ग म िररषठ वबरविश राजनता सर जॉन

साइमन क नततव म किल वबरविश सदसय ही

शावमल थ

- इसवलए भारत क ल ग ो न साइमन कमीशन क

आगमन क विरद आोद लन वकया था

Q43) उततर (a)

सपषटीकरण

bull दादा भाई नौर जी भारत म वबरविश शासन क

आवथिक पररणाम ो क बार म अपनी विर िी

(परवतकल) राय क वलए जान जात थ

FC19H1003 35

bull अपन कई लख ो और भािण ो म विशि रप स

ldquoपाििी एो ड अन-वबरविश रल इन इसणडया

(Poverty and Un-British Rule in India) म

नौर जी न यह तकि वदया वक भारत पर अतयविक

कर लगाया गया था और इसकी सोपवतत इोगलड की

ओर परिावहत की जा रही थी

bull उनह ोन पराचीन भारतीय गरोथ ो की वयाखया करन

का और भारतीय ो क आिविशवास क बहाल

करन पर कायि नही ो वकया था

उनह ोन वकसी और बात स पहल सभी सामावजक

बराइय ो क उनमलन की आिशयकता पर भी बल

नही ो वदया था

Q44) उततर (c)

सपषटीकरण

bull अगसत 1932 म वबरविश परिानमोतरी मकड नालड न

अपन साोपरदावयक परसकार (The Communal

Award) की घ िणा की थी

bull यह भारत क कई साोपरदावयक वहत ो क बीच विवभनन

सोघिो क हल करन क वलए वबरिन का एकतरफा

परयास था

bull यह परसकार (Award) बाद म 1935 क

अविवनयम (The Act of 1935) म शावमल वकया

गया था

bull इस साोपरदावयक परसकार न मससलम ो क वलए

आरवकषत एक अलग वनिािचक मणडल फॉमिल का

विसतार अनय अलपसोखयक ो क वलए वकया था

वजसम वसख ो भारतीय ईसाइय ो आोगल-भारतीय

समदाय यर पीय समदाय तथा विवशषट कषतरीय

समह ो क शावमल वकया गया था

bull गाोिी न इस परसताि क भारतीय समाज क

विभावजत करन क वलए एक घवणत वबरविश

सावजश क रप म दखा और उसक सखलाफ

आमरण अनशन वकया

Q45) उततर (b)

सपषटीकरण

मौजदा आयात और वनयाित क अवतररक़त

औपवनिवशक भारत क वनमनवलसखत खचो क

वलए एक विशिवनवशचत िन रावश भी दनी पडती

थी

(i) परशासन क वयय

(ii) सना क रख-रखाि क वयय

(iii) यद क वयय

(iv) सिावनितत अविकाररय ो की पशन तथा

(v) वबरिन दवारा अपनी उपवनिश बसती

(कॉल नी) क रख-रखाि क वयय

इनह गह शलक (Home Charges) क रप म

जाना जाता था और लगभग परी तरह स भारत क

दवारा इनका भगतान वकया जाता था

bull गह शलक म वनमनवलसखत घिक शावमल थ

(i) भारतीय ऋण पर दय बयाज

(ii) ईसट इोवडया को पनी क शयरिारक ो क

लाभाोश

(iii) लोदन म भारत कायािलय चलान क वलए िन

(iv) भारत म वनयकत वबरविश कवमिय ो क ितन

और पशन का भगतान करन क वलए िन

(v) रलि पर बयाज

(vi) नागररक और सनय शलक

(vii) इोगलड म सट र (सामगरी) की खरीद

Q46) उततर (b)

सपषटीकरण

bull भारतीय राषटर ीय काोगरस का लाहौर सतर 1929 म

जिाहरलाल नहर की अधयकषता म आय वजत

वकया गया था

bull इस सतर म भारतीय राषटर ीय आोद लन स समबसित

कई महतवपणि पररणाम सामन आय थ

(i) सििपरथम इस सतर म काोगरस क अधयकष पद

पर जिाहरलाल नहर क चना गया था ज

काोगरस म िामपोवथय ो की बिती हई ताकत

का सपषट सोकत था

(ii) दसरा इस सतर म पहली बार काोगरस न पणि

सवतोतरता की माोग क उठाया था

इस परकार की माोग काोगरस मोच स पहल कभी भी

नही ो उठाई गई थी

Q47) उततर (b)

सपषटीकरण

FC19H1003 36

bull इस ररप िि न वकसी भी समदाय क वलए पथक

वनिािचक मोडल अथिा अलपसोखयक ो क वलए

भाराोश की वसफाररश नही ो की थी

bull तथावप इस ररप िि न उन पराोत ो म अलपसोखयक

सीि ो क आरकषण की अनमवत दी थी जहाा पर कम

स कम दस परवतशत अलपसोखयक ह

bull लवकन यह समदाय क आकार क अनपात म ह ना

चावहए था

bull इस ररप िि म भारत क वलए पणि सवतोतरता क

वलए क ई पराििान नही ो था

Q48) उततर (c)

सपषटीकरण

bull आरो वभक िवदक आयो का िमि मखय रप स

परकवत की पजा और यजञ था

bull परारो वभक आयि िमि परकवत की पजा क समान था

bull िासति म उनक चार ो ओर की शसकतयाा वजनह न

त ि वनयोवतरत कर सकत थ और न ही समझ पाए

थ उनह वदवयता क साथ वनिवशत वकया गया तथा

उनह मादा या नर दिीदिताओो क रप म

परतीकतव वकया गया था

bull उनह ोन कछ यजञ ो का भी वनषपादन वकया था

Q49) उततर (b)

सपषटीकरण

bull सडक और नदी-मागि (जल-मागि) डकती स

सरवकषत नही ो थ

bull उललखनीय ह वक हिििििन क शासनकाल क

दौरान यआन चिाोग (हयएन साोग) का सारा

सामान लि वलया गया था

Q50) उततर (c)

सपषटीकरण

परशन म वदए गए द न ो कथन सही ह

Q51) उततर (b)

सपषटीकरण

bull परोदर दास एक सोत और भगिान कषण क एक

महान भकत थ

bull परोदर दास क कनाििक सोगीत क वपतामह क

रप म जाना जाता ह

bull यदयवप उनक जनम-थथान क बार म काफी

अिकल लगाई जाती रही ह

bull तथावप अब कननड विशवविदयालय हमपी क दवारा

गवठत एक विशिजञ सवमवत इस वनषकिि पर पहोची

ह वक उनका जनम थथान सोभितया कनाििक का

एक छ िा-सा गााि कषमपरा (वशिम गगा वजला)

था

Q52) उततर (c)

सपषटीकरण

bull शरी तयागराज शरी शयाम शासतरी और शरी मथसवामी

दीवकषतर क कनाििक सोगीत की वतरमवति माना

जाता ह

bull उनक कारण ही 18िी ो-19िी ो शताबदी म कनाििक

सोगीत का सववणिम यग आया था

Q53) उततर (d)

सपषटीकरण

bull अभी हाल ही म लौह यगीन-महापािावणक काल

का 2000 ििि पराना एक दलिभ सारक फगस

(Sarcophagus) (पतथर का ताबत) क ललम क

वियर गाोि (क वयलडी क पास वजला क वझक ड

करल राजय) की एक रॉक-कि गफा स ख जा गया

bull यह ताबत वजसम हविय ो क िकड थ खदाई क

दौरान वमला

bull अभी तक इस परकार की दलिभ ख ज करल क

मातर द ही थथान ो स हई ह

bull य द न ो सारक फगी (Sarcophagi) (पतथर क

ताबत) चियर और अथ ली (वजला क वझक ड) क

महापािाण थथल ो स वमल ह

Q54) उततर (a)

सपषटीकरण

FC19H1003 37

दवकषण भारत म महापािाण सोसकवत एक पणि

विकवसत लौह यगीन सोसकवत थी

Q55) उततर (d)

सपषटीकरण

bull च ल पाणडय और करलपतर (चर) इन तीन ो का

उललख अश क क अवभलख ो म वकया गया ह

bull सोभितः य भौवतक सोसकवत क उततर

महापािावणक चरण म थ

Q56) उततर (d)

सपषटीकरण

bull भीमा-क रगाोि की लडाई ततीय आोगल-मराठा

यद का वहससा थी

Q57) उततर (b)

सपषटीकरण

bull राजकमार शकल न गाोिीजी क चोपारण आन तथा

वतनकवथया परणाली स जडी समसया की जाोच क

वलए रारी करन क वलए दश भर म उनका

अनसरण वकया था

bull बज वकश र राजदर परसाद महादि दसाई और

नरहरी पाररख चोपारण सतयागरह क दौरान गाोिी

जी क सहय गी थ

Q58) उततर (b)

सपषटीकरण

bull बराहमण ो और बौद मठिाररय ो क कर-मकत गााि

अनदान म दन की परथा सतिाहन ो न आरमभ की

थी

Q59) उततर (c)

सपषटीकरण

इस कायिकरम क उददशय वनमनानसार ह

(i) बवनयादी पयििन आिाररक सोरचना का विकास

करना

(ii) चयवनत (पहचान वकय गए) कषतर ो म आजीविका क

सजन क वलए दश क साोसकवतक और विरासत

मलय ो क बिािा दना

(iii) विरासत समारक थथल ो पर विशव सतरीय आिाररक

सोरचना विकवसत करक एक सतत तरीक स

पयििक आकििण म िसद करना

(iv) थथानीय समदाय ो की सवकरय भागीदारी क माधयम

स र रगार ो का सजन करना

(v) र रगार उतपादन और आवथिक विकास क वलए

पयििन कषमता का उन पर परभाि का उपय ग

करना तथा

(vi) िारणीय पयििन आिाररक सोरचना का विकास

करना और उसका उवचत सोचालन तथा

रखरखाि सवनवशचत करना

Q60) उततर (b)

सपषटीकरण

bull यह वनकाय ििि 1987 म अससततव म आया था

bull यह एक राषटर ीय सतर का शीिि सोगठन ह ज भारत

सरकार क जनजातीय मामल ो क मोतरालय क

परशासवनक वनयोतरण क अिीन काम कर रहा ह

bull इसका पोजीकत और परिान कायािलय नई वदलली

म सथथत ह

Q61) उततर (c)

सपषटीकरण

bull परमचोद क उपनयास ो म परमाशरम रोगभवम गबन

कमिभवम और ग दान शावमल ह

bull ग रा रिी ोदरनाथ िग र क दवारा रवचत उपनयास ह

bull अभी हाल ही म मोशी परमचोद की 138िी ो जयोती दश

भर म मनाई गई थी

Q62) उततर (b)

सपषटीकरण

bull ldquoवगदाrdquo पोजाब (भारत) एिो पावकसतान की

मवहलाओो क दवारा तयौहार क समय और फसल

की बिाई तथा किाई क अिसर पर वकया जान

िाला एक पारोपररक दहाती नतय ह

FC19H1003 38

bull इस नतय क माधयम स पोजाबी मवहलाऐो अपनी

परसननता परकि करती ह तथा वगदा क परदशिन क

माधयम स परि िचिसव िाल समाज म मवहलाओो

की दबी हई भािनाओो क परकि करती ह

bull चोवक इस नतय का परि ो क साथ क ई सोबोि नही ो

ह अतः किल मवहलाऐो ही इसम भाग ल सकती

bull हर साल तीज समार ह क दौरान पोजाब म वगदा

नतय वकया जाता ह

तीज भारत क कछ भाग ो म मवहलाओो क दवारा

मनाया जान िाल कई तयौहार ो क वलए एक

वयापक नाम ह

Q63) उततर (a)

सपषटीकरण

- मजम-उल-बहरीन या द समदर ो का सोगम

नामक उललखनीय रचना दारा वशक ह क दवारा

वलखी थी

- भारत क उपराषटर पवत शरी एम िकया नायड न कहा

ह वक राजकमार दारा वशक ह की रचनाएा शाोवत

और सदभाि क बिािा दन क वलए एक तारा सर त

क रप म सामन आ सकती ो ह

- उपराषटर पवत गत ििो क भला वदए गए राजकमार

दारा वशक ह क परदवशित परचवलत करन हत

आय वजत एक परदशिनी का दौरा करन क बाद एक

सभा क सोब वित कर रह थ

- इस परदशिनी का आय जन फर क इस गौवियर

(Francois Gautier) क दवारा lsquoइोवदरा गाोिी नशनल

सिर फॉर द आििसrsquo (The Indira Gandhi

National Centre for the Arts) नई वदलली म

वकया गया था

Q64) उततर (c)

सपषटीकरण

- ग मतशवर परवतमा जन भगिान बाहबली क

समवपित ह

- यह एक एक-चटटानी पतथर की मवति ह

- राषटर पवत राम नाथ क विोद न शरिणबलग ला

(कनाििक) म आय वजत वकय जान िाल भवय

अवभिक समार ह महामसतकावभिक का

उदघािन वकया था

- यह समार ह 12 ििो म एक बार ह ता ह

Q65) उततर (c)

सपषटीकरण

bull पराची घािी पराची नदी क चार ो ओर फली हई थी

bull पराची घािी िीर-िीर विलपत ह गई थी

bull पराची नदी भिनशवर स वनकलती ह

bull यह महानदी की एक सहायक नदी ह और यह

परी खदाि किक तथा जगतवसोहपर वजल ो स

ह कर बहती ह

bull इस नदी क पर कषतर क पराची घािी कहा जाता ह

bull यह नदी बोगाल की खाडी म वगरती ह

परातासतवक साकषय स पता चलता ह वक पराची घािी

सभयता हडपपा और म हनज दाड द न ो की

पिििती ह

Q66) उततर (d)

सपषटीकरण

य समारक छतरपर वजल (मधय परदश) म विोधयाचल

पिित शरोखला म सथथत ह

Q67) उततर (a)

सपषटीकरण

bull थॉिस ऑन पावकसतान नामक पसतक डॉ बी

आर अमबडकर न वलखी थी

bull डॉ बी आर अमबडकर की जयोती क अिसर पर

भारत क राषटर पवत न भारत की इस महान हसती

क शरदाोजवल अवपित की थी

bull डॉ बी आर अमबडकर न 1924 म वडपरथड

कलावसर इोसटीटयि (दवलत िगि सोथथान -

बवहषकत वहतकाररणी सभा) और 1927 म समाज

समता सोघ की थथापना की थी

bull अमबडकर का धयान वशकषा कषतर की ओर भी था

bull उनह ोन वशकषा क वनमन िगो म फलान क वलए

पीपलस एजकशन स साइिी (The Peoples

Education Society) क नाम स महाविदयालय ो क

नििकि और छातरािास ो की थथापना की थी

FC19H1003 39

Q68) उततर (b)

सपषटीकरण

bull महरगि भारतीय उपमहादवीप म एक परवसद

निपािाण बसती ह ज बलवचसतान पराोत

पावकसतान म सथथत ह

bull दचपलली (आोधर परदश) क पास नागलर नदी क

पिी ति ो पर चना पतथर क बलॉक क विशाल

विसतार म एक पिि-ऐवतहावसक रॉक आिि थथल की

ख ज की गई ह

bull इसन 1500-2000 ईसा पिि क दौरान गोिर (आोधर

परदश) म विकवसत निपािाण सभयता पर परकाश

डाला ह

Q69) उततर (c)

सपषटीकरण

bull 12िी ो सदी और 13िी ो सदी म काकाविय िोश का

उदय हआ था

bull ि पहल कलयाण क पवशचमी चालकय ो क सामोत थ

bull परारोभ म उनह ोन िारोगल (तलोगाना) क पास एक

छ ि स कषतर पर शासन वकया था

bull उनह ोन ldquoनायक वयिथथाrdquo की शरआत की थी

वजस बाद म विजयनगर क राय शासक ो न

अपनाया और विकवसत वकया था

Q70) उततर (a)

सपषटीकरण

bull गाोिीजी क अनशन स वमल मावलक ो पर दबाि

पडा था ज अोततः शरवमक ो क ितन म 35 परवतशत

की िसद करन क वलए सहमत हए थ

bull गगल (Google) न अनसया साराभाई वजनह ोन

भारत क शरवमक आोद लन म एक अगरणी भवमका

वनभाई थी की 132िी ो जयोती डडल (Doodle) का

वनमािण करक मनाई

Q71) उततर (d)

सपषटीकरण

भारत स यनसक की मानिता की अमति साोसकवतक

विरासत की परवतवनवि सची म वनमनवलसखत शावमल ह

bull कवडयटटम करल का सोसकत रोगमोच

bull मवडयिि करल का अनषठान रोगमोच और नतय

नाविका

bull िवदक मि जाप की परोपरा

bull राजथथान क कालबवलया ल क गीत और नतय

bull रामलीला रामायण का पारोपररक परदशिन

bull सोकीतिन मवणपर का अनषठान गायन ढ ल िादन

और नतय

bull रममन भारत क गििाल वहमालय का िावमिक

तयौहार और अनषठान रोगमोच

bull जाोदीयाला गर पोजाब क ठठर ो की पीतल और

ताोब क वशलप स वनवमित बतिन ो की पारोपररक कला

bull छाऊ नतय पिी भारतीय राजय ो म जनमी शासतरीय

भारतीय नतय कला

bull लददाख का बौद मि जाप िर ाोस-वहमालयी लददाख

कषतर तथा जमम-कशमीर म पवितर बौद गरोथ ो का पाठ

bull य ग

bull नौर र

bull को भ मला

Q72) उततर (b)

सपषटीकरण

bull भारत क राषटर पवत शरी राम नाथ क विोद न

वकसामा नागालड म हॉनिवबल मह रति और

राजय गठन वदिस समार ह का उदघािन वकया

था

bull हॉनिवबल मह रति का नाम भारतीय हॉनिवबल क

नाम पर पडा ह ज एक विशाल और रोगीन जोगली

पकषी ह

bull यह पकषी नागालड राजय की अविकतर जनजावतय ो

की ल ककथाओो म उसललसखत ह

bull नागालड की परमख मानयता परापत जनजावतयाा ह

अोगामी आओ चखसोग चाोग ककी रगमा और

रवलोग आवद

bull ओोग जारिा और ससिनलीस अोडमान-वनक बार

दवीप समह की जनजावतयाा ह

FC19H1003 40

Q73) उततर (c)

सपषटीकरण

bull दकन म राषटर कि शासन दसिी ो सदी क अोत तक

लगभग 200 ििो तक रहा था

bull राषटर कि शासक अपन िावमिक विचार ो म सवहषण

bull उनह ोन न किल शि िमि और िषणि िमि बसलक

जन िमि क भी सोरकषण वदया था

bull एल रा म वशि क परवसद रॉक कि मोवदर का

वनमािण नौिी ो सदी म राषटर कि राजा कषण परथम न

करिाया था

bull उसका उततराविकारी अम घििि जन था लवकन

उसन अनय िमो क भी सोरकषण परदान वकया था

bull राषटर कि ो न मसलमान वयापाररय ो क बसन की

अनमवत दी थी

bull उनह न अपन अविराजय ो म इसलाम क उपदश दन

की भी अनमवत दी थी

bull अभी हाल ही म पाोडिलागटटा (तलोगाना) क

परागवतहावसक चटटान वचतर ो क कषरण की बिती हई

घिनाएा एक गोभीर वचोता का वििय ह

bull यह परागवतहावसक चटटान क नकसान पहाचा

सकता ह

bull पाोडिलागटटा वनमनवलसखत क वलए जाना जाता ह

- 10000 ईसा पिि स 8000 ईसा पिि क वचवतरत

चटटानी आशरय ो क वलए

- राषटर कि काल क एक 8 िी ो सदी क

वशलालख क वलए और

- 12िी ो सदी क काकविय सामराजय क वभवतत

वचतर ो क वलए

Q74) उततर (b)

सपषटीकरण

bull 1828 म राजा राम म हन रॉय न एक नय िावमिक

समाज बरहम सभा की थथापना की थी वजस बाद

म बरहम समाज क नाम स जाना गया था

bull दिदरनाथ िग र न ततवब विनी सभा की अधयकषता

की थी ज आधयासिक सतय की ख ज म सोलि

थी

bull इसका उददशय वहोद िमि क शद करन का और

एकशवरिाद (एक ईशवर म आथथा) का परचार करना

था

bull नय समाज की थथापना क आिार थ कारण

(तकि ) क द सतमभ तथा िद और उपवनिद

bull अभी हाल ही म सािारण बरहम समाज का कछ

काननी मदद ो क लकर पवशचम बोगाल सरकार क

साथ काननी वििाद चल रहा ह

Q75) उततर (c)

सपषटीकरण

bull भारत म वचशती वसलवसल की थथापना खवाजा

म इनददीन वचशती क दवारा की गयी थी

bull ि 1192 ईसवी क आसपास भारत आय थ

bull वचशतीय ो क बारहिी ो शताबदी क उततरािि म भारत

म आन िाल सफीय ो क समह ो म सबस

परभािशाली माना जाता ह

bull उनह ोन थथानीय िातािरण क साथ सफलतापििक

अनकलन वकया और उनह ोन भारतीय भसकत

परोपराओो क कई पहलओो क अपनाया

bull अजमर म सफी अपरकि खवाजा म इनददीन वचशती

की ऐवतहावसक दरगाह क एक नया रप दन की

तयारी की जा रही ह

bull इस 13िी ो शताबदी की दरगाह क ldquoसवचछ

आइकॉवनक थथल ोrdquo (Swacch Iconic Places) म

शावमल वकया गया ह ज परवतवषठत विरासत

आधयासिक और साोसकवतक थथान ो पर क वदरत

य जना ह

FC19H1003 41

ANSWERS amp EXPLANATION OF

NCERT History Class VI-X + Current Affairs

(FC19E1003)

Q1) Answer c

Explanation

Rigveda consists of more than a

thousand hymns dedicated to gods and

goddesses These hymns were

composed by sages and learnt by men

however a few were composed by

women like Apala Ghosa Lopamudra

Maitreyi and Gargi

Rigveda consists of many hymns in the

form of dialogues We get an example of

a dialogue between a sage named

Vishwamitra and two rivers (Beas and

Sutlej) that were worshipped as

goddesses This suggests that he

belonged to the Vedic period

Q2) Answer b

Explanation

Traces of ash have been found from

Kurnool Caves suggesting that people

were familiar with the use of fire

It is situated in Andhra Pradesh

Q3) Answer c

Explanation

Burzahom is a prehistoric site in

present day Kashmir where people built

pit houses which were dug into the

ground with steps leading into them

These may have provided shelter in cold

weather

Q4) Answer c

Explanation

Epigraphy is defined as the study of

inscriptions

Manuscriptology is the study of history

and literature through the use of hand

written documents

Palaeography refers to the study of

ancient writing systems and the

deciphering and dating of historical

manuscripts

Numismatics refers to the study of

coins

Q5) Answer a

Explanation

Charaka Samhita was written by

Charaka and is an important book on

Ayurveda and medicine

He was a practitioner of the traditional

system of Indian medicine known as

Ayurveda

Charaka is thought to have flourished

sometime between the 2nd century BCE

and the 2nd century CE

Q6) Answer b

Explanation

Bhaga refers to the tax on crops which

was fixed at 16th of the production

Kammakaras is the term used for the

landless agricultural labour class

Ashvamedha also known as horse

sacrifice is a ritual where a horse is let

loose to wander freely and it was

guarded by the rajarsquos men

Q7) Answer (d)

Explanation

In the Rigvedic period horses were

yoked to chariots that were used in

battles fought to capture land cattle

etc This suggests that the use of horse

chariots began much before the period

of Mahajanapadas

The battles were fought in the Rigvedic

period for cattlersquos lands water an even

to capture people Most men took part

in these wars however there was no

regular army but there were assemblies

where people met and discussed

matters of war Regular armies became

a feature in the Mjahajanapada period

including vast armies of foot soldiers

chariots and elephants

RAUSIAS-FC19E1003 42

Q8) Answer (a)

Explanation

Buddha belonged to the Sakya clan and

passed away at Kusinara

Buddha taught in Prakrit which was the

common language of people

Q9) Answer c

Explanation

There were six schools of philosophy in

ancient India These are known as

Vaishesika Nyaya Samkhya Yoga

Purva Mimansa and Vedanata or Uttara

Mimansa They were founded by sages

Kanada Gautama Kapila Patanjali

Jamini and Vyasa respectively

Q10) Answer b

Explanation

The teachings of Mahavira were

compiled at Valabhi in 6th century AD

Q11) Answer (c)

Explanation

Chanakya is traditionally identified as

Kautilya or Vishnugupta who authored

the ancient Indian political treatise the

Arthashastra

Q12) Answer d

The national emblem of India is an

adaptation of the Lion Capital atop the

Ashoka Pillar of Sarnath Uttar Pradesh

and is combined with the National

Motto Satyameva Jayate

The Rampurva Bull gets the name from

the site of its discovery Rampurva in

Bihar

It is noted for its delicately sculpted

model demonstrating superior

representation of soft flesh sensitive

nostrils alert ears and strong legs It is

a mixture of Indian and Persian

elements

Sankissa is situated in Uttar Pradesh

India

Q13) Ans(a)

Kunwar Singh was a notable leader during the Revolt of 1857 He belonged

to a royal house of Jagdispur Bihar

Q14) Answer b

Explanation

The term Vellalar was used for large

landowners

Q15) Answer c

Explanation

Arikamedu was a coastal settlement

where ships unloaded goods from

distant lands Finds here include a

massive brick warehouse pottery

including amphorae and Arretine ware

Roman lamps glassware and gems have

also been found at the site

Q16) Answer a

Explanation

Muvendar is a Tamil word mentioned in

Sangam poems meaning three chiefs

used for the heads of three ruling

families the Cholas Cheras and

Pandyas

Q17) Ans (c)

Several tribal or kin-based assemblies

such as the Sabha Vidatha and gana

are mentioned in the Rig-veda The

Sabha and the samiti mattered a great

deal in early Vedic times so much so

that the chiefs or the kings showed an

eagerness to win their support

Q18) Ans (a)

Jainism recognised the existence of the

gods but placed them lower than the

jina and did not condemn the varna

system as Buddhism did

Q19) Answer (d)

Explanation

Cholas and Pandyas had developed

powerful coastal cities The most

important city of Cholas was Puhar or

Kaveripattinam and Madurai was the

capital of Pandyas

Q20) Answer b

Explanation

Buddhacharita is the biography of

Buddha and was written by

RAUSIAS-FC19E1003 43

Ashvaghosha

Q21) Answer (a)

Explanation

Tamil poet Appar was a Shiva devotee

So he was a Nayanar saint

Q22) Answer d

Explanation

Samudragupta was a prominent Gupta

ruler whose coins depict him playing a

veena indicating his love for music We

get important historic information from

his Allahabad Prashasti which was

composed by his court poet Harisena

Q23) Answer (b)

Explanation

Vikrama Samvat was founded by

Chandragupta II in the 58 BC as a

mark of victory over the Shakas and

assumed the title of Vikramaditya

Banabhatta wrote Harshavardhanarsquos

biography the Harshacharita in

Sanskrit

Q24) Answer c

Explanation

Sandhi-vigrahika was the minister of

war and peace

Sarthavaha was the leader of the

merchant caravans

Q25) Answer a

Explanation

Xuan Zang (Hsuan-tsang) was a

Chinese traveller who came during the

reign of Harshavardhana

In the decade that began in 630 AD

Xuan Zang came to India through

Kashmir after visiting Central Asia Iran

and Afghanistan

He travelled from north to east and lived

in Bihar for a couple of years

At Nalanda University Xuan Zang

interacted with students and scholars

mastered local languages and

discovered Buddhist stupas

Q26) Answer c

Explanation

Pradakshina patha is a circular path

laid around a stupa in Buddhist

architecture While the rest are a part of

temple architecture

Q27) Answer d

Explanation

All the above-mentioned temples have

an elaborate use of bricks (baked

bricks) along with stone

Q28) Ans (c)

Muhammad Quli Qutab was the Sultan

of Golconda He was a contemporary of

Akbar was very fond of literature and

architecture

The Sultan was a great poet and he

wrote in Dakhini Urdu Persian and

Telgu and has left an extensive diwan or

collection

Recently the Archaeological Survey of

India (ASI) will be using Ground

Penetrating Radar (GPR) to map the

contours of the area around the Bagh-e-

Naya Qila excavated garden inside the

Golconda Fort in Telangana

Q29) Answer a

Explanation

Silappadikaram is a famous Tamil epic

which was written by Ilango around

1800 years ago It is a story of a

merchant named Kovalan who fell in

love with a courtesan named Madhavi

Manimekalai tells the story of the

daughter of Kovalan and Madhavi

Q30) Answer (a)

Explanation

Charaka is the author of Charaka

Samhita which is an important work of

Ayurveda and medicines

Brahmaguptarsquos fame rests mostly on his

Brahma-sphuta-siddhanta which was

an astronomical work It was translated

into Arabic in Baghdad and had a major

impact on Islamic mathematics and

astronomy

Late in his life Brahmagupta wrote

Khandakhadyaka which was an

RAUSIAS-FC19E1003 44

astronomical handbook that employed

Aryabhatarsquos system of starting each day

at midnight

Q31) Answer (c)

Explanation

Amir Khusrau was a famous sufi

musician poet and scholar In 1318 he

noted that there was different language

in every region of this land (Hindustan)

Lahori Kashmiri Dvarsamudri (in

Southern Karnataka) Telangana (in

Andhra Pradesh) Gujari (in Gujarat)

Marsquobari (in Tamil Nadu) Awadhi (in

eastern Uttar Pradesh) and Hindawai (in

the area around in Delhi) etc He went

to explain that Sanskrit did not belong

to any region and that only brahmans

knew it

Q32) Answer c

Explanation

Hiranyagarbha refers to the golden

womb When this ritual was performed

with the help of Brahmanas it was

thought to lead to the rebirth of the

sacrificer as a Khastriya

Q33) Answer d

Explanation

Kadamai refers to a tax on land

revenue

Gwalior Prashasti describes the exploits

of Nagabhata who was a Pratihara king

Q34) Answer b

Explanation

Rajatarangini is a Sanskrit text written

by Kalhana in the 12th century

It was historical chronicle of early India

It is justifiably considered to be the best

and most authentic work of its kind

It covers the entire span of history in

the Kashmir region from the earliest

times to the date of its composition

Q35) Answer c

Explanation

ldquoUrrdquo was the general assembly of the

village ldquoUrrdquo consisted of all the

taxpaying residents of an ordinary

village

Q36) Answer (a)

Explanation

Tarikh was a form of history writing in

the Delhi Sultanate The authors of

tawarikhs were learned men which

included secretaries administrators etc

Q37 Answer (a)

Explanation

Alauddin chose to pay his soldiers salaries in cash rather than iqtas The soldiers would buy their supplies from merchants in Delhi and it was thus feared that merchants would raise their prices To stop this Alauddin controlled the prices of goods in Delhi Prices were carefully surveyed by officers and merchants who did not sell at the prescribed rates were punished

Q38) Answer (d)

Explanation

Delhi first became the capital of a

kingdom under the Tomara Rajputs

who were defeated in the middle of the

twelfth century by the Chauhans (also

referred to as Chahamanas) of Ajmer

It was under the Tomaras and

Chauhans that Delhi became an

important commercial centre Many rich

Jaina merchants lived in the city and

constructed several temples Coins

minted here called dehliwal had a wide

circulation

Q39) Answer (c)

Explanation

Moth ki Masjid was built in the reign of

Sikandar Lodi by his minister

Begumpuri mosque built in the reign of

Muhammad Tughluq was the main

mosque of Jahanpanah the ldquoSanctuary

of the Worldrdquo and his new capital in

Delhi

Quwwat al ndash Islam mosque was

enlarged by Iltutmish and Alauddin

Khalji The minar was built by three

Sultansndash Qutbuddin Aybak Iltutmish

and Firuz Shah Tughluq

RAUSIAS-FC19E1003 45

Q40) Answer (c)

Explanation

Under the Mughals mansabdar was

referred to an individual who held a

mansab ie rank and he received his

salary as revenue assignments called

jagirs

Q41) Ans (b)

The Quit India Movement was a

spontaneous revolt of people against

British rule

The All India Congress Committee met

at Bombay on 8 August 1942 It passed

the famous resolution Quit India and

proposed the starting of a non-violent

mass struggle under Gandhis

leadership to achieve this aim But on

the very next day Gandhi and other

eminent leaders of the Congress were

arrested The Congress was once again

declared illegal

Q42) Ans (c)

The Simon Commission refers to a

group of seven MPs from the United

Kingdom constituted to suggest

constitutional reforms for British India

The Commission consisted of only

British members headed by one of the

senior British politicians Sir John

Simon

So the people of India agitated against

the arrival of Simon Commission

Q43) Ans (a)

He was widely known for his

unfavourable opinion of the economic

consequences of the British rule in

India

In his many writings and speeches and

especially in Poverty and Un-British

Rule in India Naoroji argued that India

was too highly taxed and that its wealth

was being drained away to England

He did not interpret the ancient Indian

texts and restored the self-confidence of

Indians And also he did not stress the

need for eradication of all the social

evils before anything else

Q44) Ans (c)

In August 1932 Prime Minister

MacDonald announced his Communal

Award Great Britainrsquos unilateral

attempt to resolve the various conflicts

among Indiarsquos many communal

interests

The award which was later

incorporated into the act of 1935

expanded the separate-electorate

formula reserved for Muslims to other

minorities including Sikhs Indian

Christians Anglo-Indians Europeans

distinct regional groups Gandhi

undertook a ldquofast unto deathrdquo against

that offer which he viewed as a

nefarious British plot to divide the

Indian society

Q45) Ans (b)

In British India apart from existing

imports and exports there was also a

particular amount of money which

colonial India contributed towards

administration maintenance of the

army war expenses pensions to retired

officers and other expenses accrued by

Britain towards maintenance of her

colony These were known as Home

charges and were paid for almost

entirely by India

The Home charges was made of

following components-

- Interest payable on Indian debt

- Dividend to shareholders of East

India Company

- Funds used to support the India

Office in London

- Funds used to pay salaries and

pensions of British personnel

engaged in India

- Interest on the railways

- Civil and military charges

- Store purchases in England

Q46) Ans (b)

The Lahore session of the Indian

National Congress was held in 1929

under the Presidentship of Jawaharlal

Nehru

The Lahore session of the Indian

National Congress witnessed significant

RAUSIAS-FC19E1003 46

developments in the Indian national

movement

- First the election of Jawaharlal

Nehru to the post of Presidentship of

the Congress was a clear indication

of the growing strength of the

Leftists in the Congress

- Secondly it was in this session that

the Congress for the first time raised

the demand for complete

independence Such demand was

not raised from the Congress

platform earlier

Q47) Ans (b)

It did not provide for separate

electorates for any community or

weightage for minorities However it did

allow for the reservation of minority

seats in provinces having minorities of

at least ten per cent but this was to be

in strict proportion to the size of the

community

There was no provision for complete

Independence for India

Q48) Ans (c)

The religion of early Vedic Aryans was

primarily of worship of nature and

Yajnas

The early Aryan religion was kind of

nature worship Actually the forces

around them which they could not

control or understand were invested

with divinity and were personified as

male or female gods And they

performed some Yajnas also

Q49) Ans (b)

The roads and river-routes were not

immune from robbery It is notable that

Yuan Chwang (Hiuen Tsang) was

robbed of his belongings during

Harshvardanarsquos period

Q50) Ans (c)

Q51) Ans (b)

Purandara Dasa was a saint and great

devotee of Lord Krishna

There is much speculation about where

Purandara Dasa regarded as the

Pitamaha of Carnatic music was born

Recently an expert committee

constituted by the Kannada University

Hampi has come to the conclusion that

Kshemapura Shivamogga district

Karnataka is the birth place of

Purandara Dasa

Q52) Ans (c)

Sri Tyagaraja Sri Shyama Shastry and Sri Muthuswami Dikshitar are considered the trinity of Carnatic music and with them came the golden age in Carnatic music in the 18th-19th

century

Q53) Ans d)

Recently a rare sarcophagus (stone

coffin) which is 2000 years old from the

Iron AgendashMegalithic era was discovered

from a rock-cut cave at Viyur village of

Kollam near Koyilandy in Kozhikode

district Kerala

The coffin containing bone fragments

was found during an excavation ldquoSo

far such a rare finding has been

discovered only from two sites

in Kerala Both these sarcophagi were

recovered from Megalithic sites at

Chevayur and Atholi also in Kozhikode

district

Q54) Ans a)

The megalithic culture in South India was a full-fledged Iron Age culture

Q55) Ans d)

The Cholas Pandyas and Keralaputras

(Cheras) mentioned in Ashokan

inscriptions were probably in the late

megalithic phase of material culture

Q56) Ans d)

Q57) Ans (b)

Raj Kumar Shukla followed Gandhiji all

over the country to persuade him to

come to Champaran to investigate the

problem associated with tinkathia

system

RAUSIAS-FC19E1003 47

Brij Kishore Rajendra Prasad Mahadev

Desai and Narhari Parikh accompanied

Gandhi ji during the Champaran

Satyagraha

Q58) Ans (b)

The Satvahanas started the practice of granting tax-free villages to brahmanas and Buddhist monks

Q59) Ans c)

The objectives of the Programme are

listed as under

- Developing basic tourism

infrastructure

- Promoting cultural and heritage

value of the country to generate

livelihoods in the identified regions

- Enhancing the tourist attractiveness

in a sustainable manner by

developing world-class

infrastructure at the heritage

monument sites

- Creating employment through active

involvement of local communities

- Harnessing tourism potential for its

effects on employment generation

and economic development

- Developing sustainable tourism

infrastructure and ensuring proper

Operations and maintenance

therein

Q60) Ans (b)

The Tribal Cooperative Marketing

Development Federation of India

(TRIFED) came into existence in 1987

It is a national-level apex organization

functioning under the administrative

control of Ministry of Tribal Affairs

Govt of India

TRIFED has its registered and Head

Office located in New Delhi

Q61) Ans (c)

Premchandrsquos novels include

Premashram Rangabhumi Ghaban

Karmabhumi and Godan

Gora is a novel written by Rabindranath

Tagore

138th birth anniversary of Munshi

Premchand was celebrated across the

country

Q62) Ans (b)

Giddha is a traditional pastoral dance

performed by the women of the Punjab

India and Pakistan at festival times

and at the sowing and reaping of the

harvest

By this dance the Punjabi women

reveal their joy expel their suppressed

feelings in a male dominated society

through the performance of Giddha

Since this dance has nothing to do with

men only women can participate in it

During the Teej celebrations Giddha

dance is celebrated in Punjab every

year Teej is a generic name for a

number of festivals that are celebrated

by women in some parts of India

Q63) Ans (a)

Dara Shukoh wrote the remarkable

work called ldquoMajma-ul-Bahrainrdquo or the

ldquoThe confluence of two seasrdquo

The Vice President of India Shri M

Venkaiah Naidu has said that Prince

Dara Shukohrsquos writings can come as a

refreshing source for infusing peace and

harmony He was addressing the

gathering after visiting the exhibition

that showcases the forgotten Prince of

yesteryears Dara Shukoh organized by

Mr Francois Gautier at Indira Gandhi

National Centre for the Arts in New

Delhi

Q64) Ans (c)

The statue Gommateshwara is

dedicated to the Jain God Bahubali

It is a monolithic statue

President Ram Nath Kovind

inaugurated the grand anointing

ceremony mdash Mahamastakabhisheka mdash

held once in 12 years at

Shravanabelagola (Karnataka)

Q65) Ans (c)

Prachi Valley had come up around the

Prachi river Prachi Valley gradually

disappeared

RAUSIAS-FC19E1003 48

The Prachi river originates from

Bhubaneswar

It is a tributary of the Mahanadi and

flows through the districts of Puri

Khurda Cuttack and Jagatsinghpur

and the entire region of the river is

termed as the Prachi Valley

It falls into the Bay of Bengal

Archaeological evidence shows that the

Prachi Valley Civilisation predates both

Harappa and Mohenjo-Daro

The Prachi river originates from

Bhubaneswar

Q66) Ans (d)

These monuments are located in

Chhatarpur district Madhya Pradesh

within Vindhya mountain range

Q67) Ans (a)

The book lsquoThoughts on Pakistanrsquo was

written by Dr BR Ambedkar

On the occasion of the birth anniversary

of Dr BR Ambedkar the president of

India pays homage to this icon of India

In 1924 he founded the Depressed

Classes Institute (Bahishkrit Hitkarini

Sabha) and in 1927 the Samaj Samata

Sangh

Another area of attention for Ambedkar

was education For its spread among

the low classes he set up a network of

colleges by the name of Peoples

Education Society and founded hostels

Q68) Ans(b)

Mehrgarh is a famous Neolithic

settlement in the Indian subcontinent

which is situated in Baluchistan

province Pakistan

A pre-historic rock art site is discovered

in the vast expanse of limestone blocks

on the eastern banks of Naguleru river

near Dachepalli (Andhra Pradesh) It

has thrown light on the Neolithic

civilisation that flourished in Guntur

(Andhra Pradesh) during 1500-2000

BC

Q69) Ans (c)

The 12th and the 13th centuries saw

the emergence of the Kakatiyas They

were at first the feudatories of the

Western Chalukyas of Kalyana Initially

they ruled over a small territory near

Warangal (Telangana)

They introduced Nayakships which was

later adopted and developed by the

Rayas of Vijayanagara

Q70) Ans (a)

The fast had effect of putting pressure

on mill owners who finally agreed to

give the workers a 35 per cent increase

in wages

Google celebrated with a doodle the

132nd birth anniversary of Anasuya

Sarabhai who played a pioneering role

in Indiarsquos labour movement

Q71) Ans (d)

The UNESCOrsquos list of the representative

list of the intangible cultural heritage of

humanity from India are

- Koodiyattam Sanskrit Theatre of

Kerala

- Mudiyettu ritual theatre and dance

drama of Kerala

- Tradition of Vedic Chanting

- Kalbelia folk songs and dances of

Rajasthan

- Ramlila Traditional Performance of

the Ramayana

- Sankirtana ritual singing

drumming and dancing of Manipur

- Ramman religious festival and

ritual theatre of the Garhwal

Himalayas India

- Traditional brass and copper craft of

utensil making among the Thatheras

of Jandiala Guru Punjab India

- Chhau dance classical Indian dance

originated in the eastern Indian

states

- Buddhist chanting of Ladakh

recitation of sacred Buddhist texts

in the trans-Himalayan Ladakh

region Jammu and Kashmir India

- Yoga

- Nouroz

- Kumbh Mela

RAUSIAS-FC19E1003 49

Q72) Ans(b)

The President of India Shri Ram Nath Kovind inaugurated the Hornbill Festival and State Formation Day celebrations of Nagaland in Kisama

The festival is named after the Indian hornbill the large and colourful forest bird which is displayed in the folklore of most of the states tribes

The major recognized tribes of Nagaland are Angami Ao Chakhesang Chang

Kuki Rengma and Zeling etc

Onge Jarawa and Sentinelese are the

tribes of Andman amp Nicobar Islands

Q73) Ans (c)

The Rashtrakutas rule in the Deccan lasted for almost two hundred years till the end of the tenth century The Rashtrakutas rulers were tolerant in their religious views and patronized not only Shaivism and Vaishnavism but

Jainism as well

The famous rock-cut temple of Shiva at Ellora was built by one of the Rashtrakutas kings Krishna I in the ninth century His successor Amoghavarsha was a Jain but he also

patronized other faiths

The Rashtrakutas allowed Muslims traders to settle and permitted Islam to

be preached in their dominions

Recently increasing defacement at the prehistoric rock paintings of Pandavulagutta Telangana has created a cause for grave concern It can spoil

the prehistoric rock

Pandavulagutta is home to

- Painted rock shelters dating to

10000 BC-8000 BC

- An 8th century inscription of the

Rashtrakuta period and

- Painted frescoes from the 12th century Kakatiya empire

Q74) Ans (b)

In 1828 Raja Ram Mohan Roy founded a new religious society the Brahma Sabha later known as the Brahmo

Samaj

Debendranath Tagore headed the Tattvabodhini Sabha which was

engaged in search of spiritual truth

Its purpose was to purify Hinduism and to preach monotheism or belief in one God

The new society was to be based on the twin pillars of reason and the Vedas and

Upanishads

Recently Sadharan Brahmo Samaj (SBS) has entered into a legal battle with the West Bengal government due

to some legal issue

Q75) Ans (c)

The Chishti order was established in India by Khwaja Moinuddin Chishti who came to India around 1192 The Chishtirsquos are considered to be the most influential of the groups of Sufis who migrated to India in the late twelfth century They adapted successfully to the local environment and adopted several features of Indian devotional

traditions

The historical dargah of Sufi mystic Khwaja Moinuddin Chishti in Ajmer is all set to get a facelift This 13 th century dargah has been included among the Swachh Iconic Places a clean-up initiative focused on iconic

heritage spiritual and cultural places

Page 32: GENERAL STUDIES (PAPER I) · Test is part of Rau’s IAS Test series for Preliminary Exam 2019 FOUNDATION + CURRENT AFFAIRS GENERAL STUDIES (PAPER –I) FOUNDATION TEST –III TOPIC:

FC19H1003 32

- बानभटट न हिििििन का जीिन-ितताोत हििचररत

(ज सोसकत म थी) वलखी थी

Q24) उततर (c)

सपषटीकरणः

- सोवि-विगरावहका यद एिो शाोवत का मोतरी

- साथििाह वयापाररय ो क कावफल ो का नता

Q25) उततर (a)

सपषटीकरणः

- जआन झाोग (हसआन रताोग ndash Hsuang Tsang)

एक चीनी यातरी था ज हिििििन क शासनकाल म

भारत आया था

- ििि 630 ईसवी स ज दशक आरमभ हआ था उसम

जआन झाोग मधय एवशया ईरान और

अफग़ावनसतान की यातरा करन क पशचात कशमीर

क रासत स भारत आया था

- उसन उततर स पिि तक की यातरा की और िह

लगभग 2 ििि वबहार म रहा

- जआन झाोग न नालनदा विशवविदयालय म विदयावथिय ो

और विदवान ो क साथ पारसपररक विचार-विमशि

वकया थथानीय भािाओ ा म वनपणता परापत की तथा

बौद सतप ो की ख ज की

Q26) उततर (c)

सपषटीकरणः

- परदवकषणा पथ बौद िासतकला म सतप क चार ो

ओर बनाया जान िाला एक घमािदार पथ ह ता

- परशन म वदए गए बाकी क तीन ो ततव वहोद मसनदर ो की

िासतकला क भाग ह

Q27) उततर (d)

सपषटीकरणः

परशन म वदए गए सभी मोवदर ो म वयापक रप स

ईोि ो (पकी ईोि ो) का परय ग पतथर ो क साथ हआ

Q28) उततर (c)

सपषटीकरण

- महममद कली कतब शाह ग लकणडा का सलतान

था

- िह अकबर का समकालीन था

- सावहतय और िासतकला म उसकी अतयाविक

रवच थी

- िह एक महान कवि था

- िह दसखनी उदि फारसी और तलग म वलखता था

- उसन अपन पीछ एक विसतत वदिान (सोगरह)

छ डा ह

- अभी हाल ही म तलोगाना म ग लकणडा क वकल

क अनदर खदाई वकय गए बाग-ए-नाया वकला

बाग क चार ो ओर रप-रखा क मानवचतरण क

वलए भारतीय परातासतवक सिकषण (The

Archaeological Survey of India ndash ASI)

गराउणड पनीिर विोग रडार (Ground Penetrating

Radar) का परय ग करगा

Q29) उततर (a)

सपषटीकरणः

- वसलपपावदकारम एक तवमल महाकावय ह वजसकी

रचना इलाोग क दवारा लगभग 1800 ििि पिि की

गई थी

- यह क िलन नामक एक वयापारी की कहानी ह

ज माििी नामक एक गवणका (िशया) स परम

करन लगा था

- मवनमकलाई क िलन और माििी की पतरी की

कहानी ह

Q30) उततर (a)

सपषटीकरण

- चरक आयिद और वचवकरता की एक महतवपणि

रचना चरक सोवहता क लखक ह

- बरहमगपत क अपनी रचना बरहम-सफि-वसदानत

(ज एक खग लीय रचना ह) क कारण परवससद

वमली

FC19H1003 33

- बगदाद म इसका अनिाद अरबी भािा म वकया

गया था

- इसका इसलावमक गवणत और खग ल-विजञान पर

महतवपणि परभाि पडा था

- बाद म अपन जीिनकाल म बरहमगपत न

ldquoखोडखयाकrdquo वलखी ज एक खग लीय पससतका

(एक छ िी पसतक) थी

- इसम आयिभटट की अिि-रावतर क परतयक वदन की

शरआत परणाली का परय ग वकया गया था

Q31) उततर (c)

सपषटीकरण

- अमीर खसर एक परवसद सफी सोगीतकार कवि

और विदवान थ

- 1318 म उनह ोन पाया वक इस भवम (वहोदसतान) क

हर कषतर म अलग-अलग भािा थी लाहौरी

कशमीरी दवारसमदरी (दवकषणी कनाििक म)

तलोगाना (आोधर परदश म) गजरी (गजरात म)

माबारी (तवमलनाड म ) अििी (पिी उततर परदश

म) और वहोदिी (वदलली क आस-पास क कषतर म)

आवद

- उनह न यह बताया वक सोसकत वकसी भी कषतर स

सोबोवित नही ो थी और किल बराहमण ही इस भािा

का जञान रखत थ

Q32) उततर (c)

सपषटीकरण

- वहरणय-गभि सववणिम गभि क सोदवभित करता ह

- जब बराहमण ो की सहायता स यह अनषठान वकया

जाता था त यह माना जाता था वक बवल दन िाल

का कषवतरय क रप म पनजिनम ह गा

Q33) उततर (d)

सपषटीकरण

- कदमई भवम राजसव पर कर क सोदवभित करता

- गवावलयर परशससत म नागभि क दवारा वकय गए

श िण का िणिन वकया गया ह |

- नागभि एक परवतहार राजा था

Q34) उततर (b)

सपषटीकरण

- राजतरो वगनी 12िी ो शताबदी म कलहन क दवारा

रवचत एक सोसकत पसतक (िकसट) ह

- यह परारसमभक भारत की ऐवतहावसक इवतितत थी

- तकि सोगत रप स इस अपन परकार की सिोततम

और सिािविक विशवसनीय कवत माना जाता ह

- यह कशमीर कषतर क पराचीनतम समय स लकर

उसकी रचना की तारीख तक क समपणि इवतहास

का आचछादन करती ह

Q35) उततर (c)

सपषटीकरण

- गााि की आम सभा क ldquoउरrdquo कहा जाता था

- ldquoउरrdquo म गााि क सभी कर दन िाल वनिासी

शावमल ह त थ

Q36) उततर (a)

सपषटीकरण

- वदलली सलतनत म ldquoतारीखrdquo इवतहास लखन का

एक रप था

- ldquoतािरीखrdquo क लखक विदवान परि ह त थ वजनम

सवचि परशासक इतयावद शावमल थ

Q37) उततर (a)

सपषटीकरण

- अलाउददीन सखलजी अपन सवनक ो क ितन का

भगतान नकद म करता था न वक इकता क रप

- सवनक अपना सामान वदलली म वयापाररय ो स

खरीदत थ अतः इस बात का भय था वक वयापारी

कही ो िसतओो का मलय न बिा द

- इसकी र कथाम क वलए अलाउददीन सखलजी न

वदलली म कीमत ो क वनयसित वकया

FC19H1003 34

- अविकारीगण धयानपििक मलय ो का सिकषण करत

थ तथा ज वयापारी वनिािररत मलय पर माल नही ो

बचत थ उनक दसणडत वकया जाता था

Q38) उततर (d)

सपषटीकरण

- वदलली सििपरथम त मर राजपत ो क अिीन उनक

सामराजय की राजिानी बनी थी

- 12िी ो शताबदी क मधय म अजमर क चौहान ो

(वजनह चाहमान ो क नाम स भी जाना जाता ह) न

त मर राजपत ो क परावजत वकया था

- त मर ो और चौहान ो क अिीन वदलली एक

महतवपणि िावणसजयक क दर बन गया था

- कई जन वयापारी यहाा रहन लग थ और उनह ोन

कई मोवदर भी बनिाए

- यहाा पर मवदरत वसक वजनह ldquoदहलीिालrdquo क नाम

स जाना जाता था वयापक रप स परचलन म थ

Q39) उततर (c)

सपषटीकरण

- म ठ की मसिद का वनमािण वसको दर ल दी क

राजयकाल म उसक मिी क दवारा करिाया गया

था

- बगमपरी मसिद का वनमािण महममद तगलक क

शासनकाल म हआ था

- यह मसिद विशव का पणयथथान (The

Sanctuary of the World) और वदलली म महममद

तगलक की नई राजिानी जहाोपनाह की मखय

मसिद थी

- कववत- अल - इसलाम मसिद का विसतार

इलतसिश और अलाउददीन सखलजी न वकया था

- मीनार का वनमािण तीन सलतान ो कतबददीन ऐबक

इलतसिश और वफर ज शाह तगलक क दवारा

करिाया गया था

Q40) उततर (c)

सपषटीकरण

- मगल ो क अिीन मनसबदार शबद उस वयसकत क

वलए सोदवभित वकया जाता था वजसक पास मनसब

(अथाित पद) ह ता था

- उस अपना ितन राजसव कायो वजनह जागीर कहत

थ क रप म परापत ह ता था

Q41) उततर (b)

सपषटीकरण

- ldquoभारत छ ड आोद लनrdquo वबरविश शासन क

सखलाफ ल ग ो का एक सवाभाविक विदर ह था

- असखल भारतीय काोगरस सवमवत न 8 अगसत 1942

क बमबई म एक बठक का आय जन वकया था

- इस बठक म परवसद सोकलप ldquoभारत छ ड rdquo क

पाररत वकया गया और इस उददशय क परापत करन

क वलए गाोिी क नततव म एक अवहोसक जन सोघिि

आोद लन की शरआत का परसताि वदया गया

- लवकन अगल ही वदन गाोिी और काोगरस क अनय

परमख नताओो क वगरफतार कर वलया गया

- काोगरस क एक बार वफर अिि घ वित वकया गया

था

Q42) उततर (c)

सपषटीकरण

- साइमन कमीशन यनाइविड वको गडम क सात

साोसद ो का एक समह था

- इस वबरविश भारत क वलए सोििावनक सिार ो का

सझाि दन क वलए गवठत वकया गया था

- इस आय ग म िररषठ वबरविश राजनता सर जॉन

साइमन क नततव म किल वबरविश सदसय ही

शावमल थ

- इसवलए भारत क ल ग ो न साइमन कमीशन क

आगमन क विरद आोद लन वकया था

Q43) उततर (a)

सपषटीकरण

bull दादा भाई नौर जी भारत म वबरविश शासन क

आवथिक पररणाम ो क बार म अपनी विर िी

(परवतकल) राय क वलए जान जात थ

FC19H1003 35

bull अपन कई लख ो और भािण ो म विशि रप स

ldquoपाििी एो ड अन-वबरविश रल इन इसणडया

(Poverty and Un-British Rule in India) म

नौर जी न यह तकि वदया वक भारत पर अतयविक

कर लगाया गया था और इसकी सोपवतत इोगलड की

ओर परिावहत की जा रही थी

bull उनह ोन पराचीन भारतीय गरोथ ो की वयाखया करन

का और भारतीय ो क आिविशवास क बहाल

करन पर कायि नही ो वकया था

उनह ोन वकसी और बात स पहल सभी सामावजक

बराइय ो क उनमलन की आिशयकता पर भी बल

नही ो वदया था

Q44) उततर (c)

सपषटीकरण

bull अगसत 1932 म वबरविश परिानमोतरी मकड नालड न

अपन साोपरदावयक परसकार (The Communal

Award) की घ िणा की थी

bull यह भारत क कई साोपरदावयक वहत ो क बीच विवभनन

सोघिो क हल करन क वलए वबरिन का एकतरफा

परयास था

bull यह परसकार (Award) बाद म 1935 क

अविवनयम (The Act of 1935) म शावमल वकया

गया था

bull इस साोपरदावयक परसकार न मससलम ो क वलए

आरवकषत एक अलग वनिािचक मणडल फॉमिल का

विसतार अनय अलपसोखयक ो क वलए वकया था

वजसम वसख ो भारतीय ईसाइय ो आोगल-भारतीय

समदाय यर पीय समदाय तथा विवशषट कषतरीय

समह ो क शावमल वकया गया था

bull गाोिी न इस परसताि क भारतीय समाज क

विभावजत करन क वलए एक घवणत वबरविश

सावजश क रप म दखा और उसक सखलाफ

आमरण अनशन वकया

Q45) उततर (b)

सपषटीकरण

मौजदा आयात और वनयाित क अवतररक़त

औपवनिवशक भारत क वनमनवलसखत खचो क

वलए एक विशिवनवशचत िन रावश भी दनी पडती

थी

(i) परशासन क वयय

(ii) सना क रख-रखाि क वयय

(iii) यद क वयय

(iv) सिावनितत अविकाररय ो की पशन तथा

(v) वबरिन दवारा अपनी उपवनिश बसती

(कॉल नी) क रख-रखाि क वयय

इनह गह शलक (Home Charges) क रप म

जाना जाता था और लगभग परी तरह स भारत क

दवारा इनका भगतान वकया जाता था

bull गह शलक म वनमनवलसखत घिक शावमल थ

(i) भारतीय ऋण पर दय बयाज

(ii) ईसट इोवडया को पनी क शयरिारक ो क

लाभाोश

(iii) लोदन म भारत कायािलय चलान क वलए िन

(iv) भारत म वनयकत वबरविश कवमिय ो क ितन

और पशन का भगतान करन क वलए िन

(v) रलि पर बयाज

(vi) नागररक और सनय शलक

(vii) इोगलड म सट र (सामगरी) की खरीद

Q46) उततर (b)

सपषटीकरण

bull भारतीय राषटर ीय काोगरस का लाहौर सतर 1929 म

जिाहरलाल नहर की अधयकषता म आय वजत

वकया गया था

bull इस सतर म भारतीय राषटर ीय आोद लन स समबसित

कई महतवपणि पररणाम सामन आय थ

(i) सििपरथम इस सतर म काोगरस क अधयकष पद

पर जिाहरलाल नहर क चना गया था ज

काोगरस म िामपोवथय ो की बिती हई ताकत

का सपषट सोकत था

(ii) दसरा इस सतर म पहली बार काोगरस न पणि

सवतोतरता की माोग क उठाया था

इस परकार की माोग काोगरस मोच स पहल कभी भी

नही ो उठाई गई थी

Q47) उततर (b)

सपषटीकरण

FC19H1003 36

bull इस ररप िि न वकसी भी समदाय क वलए पथक

वनिािचक मोडल अथिा अलपसोखयक ो क वलए

भाराोश की वसफाररश नही ो की थी

bull तथावप इस ररप िि न उन पराोत ो म अलपसोखयक

सीि ो क आरकषण की अनमवत दी थी जहाा पर कम

स कम दस परवतशत अलपसोखयक ह

bull लवकन यह समदाय क आकार क अनपात म ह ना

चावहए था

bull इस ररप िि म भारत क वलए पणि सवतोतरता क

वलए क ई पराििान नही ो था

Q48) उततर (c)

सपषटीकरण

bull आरो वभक िवदक आयो का िमि मखय रप स

परकवत की पजा और यजञ था

bull परारो वभक आयि िमि परकवत की पजा क समान था

bull िासति म उनक चार ो ओर की शसकतयाा वजनह न

त ि वनयोवतरत कर सकत थ और न ही समझ पाए

थ उनह वदवयता क साथ वनिवशत वकया गया तथा

उनह मादा या नर दिीदिताओो क रप म

परतीकतव वकया गया था

bull उनह ोन कछ यजञ ो का भी वनषपादन वकया था

Q49) उततर (b)

सपषटीकरण

bull सडक और नदी-मागि (जल-मागि) डकती स

सरवकषत नही ो थ

bull उललखनीय ह वक हिििििन क शासनकाल क

दौरान यआन चिाोग (हयएन साोग) का सारा

सामान लि वलया गया था

Q50) उततर (c)

सपषटीकरण

परशन म वदए गए द न ो कथन सही ह

Q51) उततर (b)

सपषटीकरण

bull परोदर दास एक सोत और भगिान कषण क एक

महान भकत थ

bull परोदर दास क कनाििक सोगीत क वपतामह क

रप म जाना जाता ह

bull यदयवप उनक जनम-थथान क बार म काफी

अिकल लगाई जाती रही ह

bull तथावप अब कननड विशवविदयालय हमपी क दवारा

गवठत एक विशिजञ सवमवत इस वनषकिि पर पहोची

ह वक उनका जनम थथान सोभितया कनाििक का

एक छ िा-सा गााि कषमपरा (वशिम गगा वजला)

था

Q52) उततर (c)

सपषटीकरण

bull शरी तयागराज शरी शयाम शासतरी और शरी मथसवामी

दीवकषतर क कनाििक सोगीत की वतरमवति माना

जाता ह

bull उनक कारण ही 18िी ो-19िी ो शताबदी म कनाििक

सोगीत का सववणिम यग आया था

Q53) उततर (d)

सपषटीकरण

bull अभी हाल ही म लौह यगीन-महापािावणक काल

का 2000 ििि पराना एक दलिभ सारक फगस

(Sarcophagus) (पतथर का ताबत) क ललम क

वियर गाोि (क वयलडी क पास वजला क वझक ड

करल राजय) की एक रॉक-कि गफा स ख जा गया

bull यह ताबत वजसम हविय ो क िकड थ खदाई क

दौरान वमला

bull अभी तक इस परकार की दलिभ ख ज करल क

मातर द ही थथान ो स हई ह

bull य द न ो सारक फगी (Sarcophagi) (पतथर क

ताबत) चियर और अथ ली (वजला क वझक ड) क

महापािाण थथल ो स वमल ह

Q54) उततर (a)

सपषटीकरण

FC19H1003 37

दवकषण भारत म महापािाण सोसकवत एक पणि

विकवसत लौह यगीन सोसकवत थी

Q55) उततर (d)

सपषटीकरण

bull च ल पाणडय और करलपतर (चर) इन तीन ो का

उललख अश क क अवभलख ो म वकया गया ह

bull सोभितः य भौवतक सोसकवत क उततर

महापािावणक चरण म थ

Q56) उततर (d)

सपषटीकरण

bull भीमा-क रगाोि की लडाई ततीय आोगल-मराठा

यद का वहससा थी

Q57) उततर (b)

सपषटीकरण

bull राजकमार शकल न गाोिीजी क चोपारण आन तथा

वतनकवथया परणाली स जडी समसया की जाोच क

वलए रारी करन क वलए दश भर म उनका

अनसरण वकया था

bull बज वकश र राजदर परसाद महादि दसाई और

नरहरी पाररख चोपारण सतयागरह क दौरान गाोिी

जी क सहय गी थ

Q58) उततर (b)

सपषटीकरण

bull बराहमण ो और बौद मठिाररय ो क कर-मकत गााि

अनदान म दन की परथा सतिाहन ो न आरमभ की

थी

Q59) उततर (c)

सपषटीकरण

इस कायिकरम क उददशय वनमनानसार ह

(i) बवनयादी पयििन आिाररक सोरचना का विकास

करना

(ii) चयवनत (पहचान वकय गए) कषतर ो म आजीविका क

सजन क वलए दश क साोसकवतक और विरासत

मलय ो क बिािा दना

(iii) विरासत समारक थथल ो पर विशव सतरीय आिाररक

सोरचना विकवसत करक एक सतत तरीक स

पयििक आकििण म िसद करना

(iv) थथानीय समदाय ो की सवकरय भागीदारी क माधयम

स र रगार ो का सजन करना

(v) र रगार उतपादन और आवथिक विकास क वलए

पयििन कषमता का उन पर परभाि का उपय ग

करना तथा

(vi) िारणीय पयििन आिाररक सोरचना का विकास

करना और उसका उवचत सोचालन तथा

रखरखाि सवनवशचत करना

Q60) उततर (b)

सपषटीकरण

bull यह वनकाय ििि 1987 म अससततव म आया था

bull यह एक राषटर ीय सतर का शीिि सोगठन ह ज भारत

सरकार क जनजातीय मामल ो क मोतरालय क

परशासवनक वनयोतरण क अिीन काम कर रहा ह

bull इसका पोजीकत और परिान कायािलय नई वदलली

म सथथत ह

Q61) उततर (c)

सपषटीकरण

bull परमचोद क उपनयास ो म परमाशरम रोगभवम गबन

कमिभवम और ग दान शावमल ह

bull ग रा रिी ोदरनाथ िग र क दवारा रवचत उपनयास ह

bull अभी हाल ही म मोशी परमचोद की 138िी ो जयोती दश

भर म मनाई गई थी

Q62) उततर (b)

सपषटीकरण

bull ldquoवगदाrdquo पोजाब (भारत) एिो पावकसतान की

मवहलाओो क दवारा तयौहार क समय और फसल

की बिाई तथा किाई क अिसर पर वकया जान

िाला एक पारोपररक दहाती नतय ह

FC19H1003 38

bull इस नतय क माधयम स पोजाबी मवहलाऐो अपनी

परसननता परकि करती ह तथा वगदा क परदशिन क

माधयम स परि िचिसव िाल समाज म मवहलाओो

की दबी हई भािनाओो क परकि करती ह

bull चोवक इस नतय का परि ो क साथ क ई सोबोि नही ो

ह अतः किल मवहलाऐो ही इसम भाग ल सकती

bull हर साल तीज समार ह क दौरान पोजाब म वगदा

नतय वकया जाता ह

तीज भारत क कछ भाग ो म मवहलाओो क दवारा

मनाया जान िाल कई तयौहार ो क वलए एक

वयापक नाम ह

Q63) उततर (a)

सपषटीकरण

- मजम-उल-बहरीन या द समदर ो का सोगम

नामक उललखनीय रचना दारा वशक ह क दवारा

वलखी थी

- भारत क उपराषटर पवत शरी एम िकया नायड न कहा

ह वक राजकमार दारा वशक ह की रचनाएा शाोवत

और सदभाि क बिािा दन क वलए एक तारा सर त

क रप म सामन आ सकती ो ह

- उपराषटर पवत गत ििो क भला वदए गए राजकमार

दारा वशक ह क परदवशित परचवलत करन हत

आय वजत एक परदशिनी का दौरा करन क बाद एक

सभा क सोब वित कर रह थ

- इस परदशिनी का आय जन फर क इस गौवियर

(Francois Gautier) क दवारा lsquoइोवदरा गाोिी नशनल

सिर फॉर द आििसrsquo (The Indira Gandhi

National Centre for the Arts) नई वदलली म

वकया गया था

Q64) उततर (c)

सपषटीकरण

- ग मतशवर परवतमा जन भगिान बाहबली क

समवपित ह

- यह एक एक-चटटानी पतथर की मवति ह

- राषटर पवत राम नाथ क विोद न शरिणबलग ला

(कनाििक) म आय वजत वकय जान िाल भवय

अवभिक समार ह महामसतकावभिक का

उदघािन वकया था

- यह समार ह 12 ििो म एक बार ह ता ह

Q65) उततर (c)

सपषटीकरण

bull पराची घािी पराची नदी क चार ो ओर फली हई थी

bull पराची घािी िीर-िीर विलपत ह गई थी

bull पराची नदी भिनशवर स वनकलती ह

bull यह महानदी की एक सहायक नदी ह और यह

परी खदाि किक तथा जगतवसोहपर वजल ो स

ह कर बहती ह

bull इस नदी क पर कषतर क पराची घािी कहा जाता ह

bull यह नदी बोगाल की खाडी म वगरती ह

परातासतवक साकषय स पता चलता ह वक पराची घािी

सभयता हडपपा और म हनज दाड द न ो की

पिििती ह

Q66) उततर (d)

सपषटीकरण

य समारक छतरपर वजल (मधय परदश) म विोधयाचल

पिित शरोखला म सथथत ह

Q67) उततर (a)

सपषटीकरण

bull थॉिस ऑन पावकसतान नामक पसतक डॉ बी

आर अमबडकर न वलखी थी

bull डॉ बी आर अमबडकर की जयोती क अिसर पर

भारत क राषटर पवत न भारत की इस महान हसती

क शरदाोजवल अवपित की थी

bull डॉ बी आर अमबडकर न 1924 म वडपरथड

कलावसर इोसटीटयि (दवलत िगि सोथथान -

बवहषकत वहतकाररणी सभा) और 1927 म समाज

समता सोघ की थथापना की थी

bull अमबडकर का धयान वशकषा कषतर की ओर भी था

bull उनह ोन वशकषा क वनमन िगो म फलान क वलए

पीपलस एजकशन स साइिी (The Peoples

Education Society) क नाम स महाविदयालय ो क

नििकि और छातरािास ो की थथापना की थी

FC19H1003 39

Q68) उततर (b)

सपषटीकरण

bull महरगि भारतीय उपमहादवीप म एक परवसद

निपािाण बसती ह ज बलवचसतान पराोत

पावकसतान म सथथत ह

bull दचपलली (आोधर परदश) क पास नागलर नदी क

पिी ति ो पर चना पतथर क बलॉक क विशाल

विसतार म एक पिि-ऐवतहावसक रॉक आिि थथल की

ख ज की गई ह

bull इसन 1500-2000 ईसा पिि क दौरान गोिर (आोधर

परदश) म विकवसत निपािाण सभयता पर परकाश

डाला ह

Q69) उततर (c)

सपषटीकरण

bull 12िी ो सदी और 13िी ो सदी म काकाविय िोश का

उदय हआ था

bull ि पहल कलयाण क पवशचमी चालकय ो क सामोत थ

bull परारोभ म उनह ोन िारोगल (तलोगाना) क पास एक

छ ि स कषतर पर शासन वकया था

bull उनह ोन ldquoनायक वयिथथाrdquo की शरआत की थी

वजस बाद म विजयनगर क राय शासक ो न

अपनाया और विकवसत वकया था

Q70) उततर (a)

सपषटीकरण

bull गाोिीजी क अनशन स वमल मावलक ो पर दबाि

पडा था ज अोततः शरवमक ो क ितन म 35 परवतशत

की िसद करन क वलए सहमत हए थ

bull गगल (Google) न अनसया साराभाई वजनह ोन

भारत क शरवमक आोद लन म एक अगरणी भवमका

वनभाई थी की 132िी ो जयोती डडल (Doodle) का

वनमािण करक मनाई

Q71) उततर (d)

सपषटीकरण

भारत स यनसक की मानिता की अमति साोसकवतक

विरासत की परवतवनवि सची म वनमनवलसखत शावमल ह

bull कवडयटटम करल का सोसकत रोगमोच

bull मवडयिि करल का अनषठान रोगमोच और नतय

नाविका

bull िवदक मि जाप की परोपरा

bull राजथथान क कालबवलया ल क गीत और नतय

bull रामलीला रामायण का पारोपररक परदशिन

bull सोकीतिन मवणपर का अनषठान गायन ढ ल िादन

और नतय

bull रममन भारत क गििाल वहमालय का िावमिक

तयौहार और अनषठान रोगमोच

bull जाोदीयाला गर पोजाब क ठठर ो की पीतल और

ताोब क वशलप स वनवमित बतिन ो की पारोपररक कला

bull छाऊ नतय पिी भारतीय राजय ो म जनमी शासतरीय

भारतीय नतय कला

bull लददाख का बौद मि जाप िर ाोस-वहमालयी लददाख

कषतर तथा जमम-कशमीर म पवितर बौद गरोथ ो का पाठ

bull य ग

bull नौर र

bull को भ मला

Q72) उततर (b)

सपषटीकरण

bull भारत क राषटर पवत शरी राम नाथ क विोद न

वकसामा नागालड म हॉनिवबल मह रति और

राजय गठन वदिस समार ह का उदघािन वकया

था

bull हॉनिवबल मह रति का नाम भारतीय हॉनिवबल क

नाम पर पडा ह ज एक विशाल और रोगीन जोगली

पकषी ह

bull यह पकषी नागालड राजय की अविकतर जनजावतय ो

की ल ककथाओो म उसललसखत ह

bull नागालड की परमख मानयता परापत जनजावतयाा ह

अोगामी आओ चखसोग चाोग ककी रगमा और

रवलोग आवद

bull ओोग जारिा और ससिनलीस अोडमान-वनक बार

दवीप समह की जनजावतयाा ह

FC19H1003 40

Q73) उततर (c)

सपषटीकरण

bull दकन म राषटर कि शासन दसिी ो सदी क अोत तक

लगभग 200 ििो तक रहा था

bull राषटर कि शासक अपन िावमिक विचार ो म सवहषण

bull उनह ोन न किल शि िमि और िषणि िमि बसलक

जन िमि क भी सोरकषण वदया था

bull एल रा म वशि क परवसद रॉक कि मोवदर का

वनमािण नौिी ो सदी म राषटर कि राजा कषण परथम न

करिाया था

bull उसका उततराविकारी अम घििि जन था लवकन

उसन अनय िमो क भी सोरकषण परदान वकया था

bull राषटर कि ो न मसलमान वयापाररय ो क बसन की

अनमवत दी थी

bull उनह न अपन अविराजय ो म इसलाम क उपदश दन

की भी अनमवत दी थी

bull अभी हाल ही म पाोडिलागटटा (तलोगाना) क

परागवतहावसक चटटान वचतर ो क कषरण की बिती हई

घिनाएा एक गोभीर वचोता का वििय ह

bull यह परागवतहावसक चटटान क नकसान पहाचा

सकता ह

bull पाोडिलागटटा वनमनवलसखत क वलए जाना जाता ह

- 10000 ईसा पिि स 8000 ईसा पिि क वचवतरत

चटटानी आशरय ो क वलए

- राषटर कि काल क एक 8 िी ो सदी क

वशलालख क वलए और

- 12िी ो सदी क काकविय सामराजय क वभवतत

वचतर ो क वलए

Q74) उततर (b)

सपषटीकरण

bull 1828 म राजा राम म हन रॉय न एक नय िावमिक

समाज बरहम सभा की थथापना की थी वजस बाद

म बरहम समाज क नाम स जाना गया था

bull दिदरनाथ िग र न ततवब विनी सभा की अधयकषता

की थी ज आधयासिक सतय की ख ज म सोलि

थी

bull इसका उददशय वहोद िमि क शद करन का और

एकशवरिाद (एक ईशवर म आथथा) का परचार करना

था

bull नय समाज की थथापना क आिार थ कारण

(तकि ) क द सतमभ तथा िद और उपवनिद

bull अभी हाल ही म सािारण बरहम समाज का कछ

काननी मदद ो क लकर पवशचम बोगाल सरकार क

साथ काननी वििाद चल रहा ह

Q75) उततर (c)

सपषटीकरण

bull भारत म वचशती वसलवसल की थथापना खवाजा

म इनददीन वचशती क दवारा की गयी थी

bull ि 1192 ईसवी क आसपास भारत आय थ

bull वचशतीय ो क बारहिी ो शताबदी क उततरािि म भारत

म आन िाल सफीय ो क समह ो म सबस

परभािशाली माना जाता ह

bull उनह ोन थथानीय िातािरण क साथ सफलतापििक

अनकलन वकया और उनह ोन भारतीय भसकत

परोपराओो क कई पहलओो क अपनाया

bull अजमर म सफी अपरकि खवाजा म इनददीन वचशती

की ऐवतहावसक दरगाह क एक नया रप दन की

तयारी की जा रही ह

bull इस 13िी ो शताबदी की दरगाह क ldquoसवचछ

आइकॉवनक थथल ोrdquo (Swacch Iconic Places) म

शावमल वकया गया ह ज परवतवषठत विरासत

आधयासिक और साोसकवतक थथान ो पर क वदरत

य जना ह

FC19H1003 41

ANSWERS amp EXPLANATION OF

NCERT History Class VI-X + Current Affairs

(FC19E1003)

Q1) Answer c

Explanation

Rigveda consists of more than a

thousand hymns dedicated to gods and

goddesses These hymns were

composed by sages and learnt by men

however a few were composed by

women like Apala Ghosa Lopamudra

Maitreyi and Gargi

Rigveda consists of many hymns in the

form of dialogues We get an example of

a dialogue between a sage named

Vishwamitra and two rivers (Beas and

Sutlej) that were worshipped as

goddesses This suggests that he

belonged to the Vedic period

Q2) Answer b

Explanation

Traces of ash have been found from

Kurnool Caves suggesting that people

were familiar with the use of fire

It is situated in Andhra Pradesh

Q3) Answer c

Explanation

Burzahom is a prehistoric site in

present day Kashmir where people built

pit houses which were dug into the

ground with steps leading into them

These may have provided shelter in cold

weather

Q4) Answer c

Explanation

Epigraphy is defined as the study of

inscriptions

Manuscriptology is the study of history

and literature through the use of hand

written documents

Palaeography refers to the study of

ancient writing systems and the

deciphering and dating of historical

manuscripts

Numismatics refers to the study of

coins

Q5) Answer a

Explanation

Charaka Samhita was written by

Charaka and is an important book on

Ayurveda and medicine

He was a practitioner of the traditional

system of Indian medicine known as

Ayurveda

Charaka is thought to have flourished

sometime between the 2nd century BCE

and the 2nd century CE

Q6) Answer b

Explanation

Bhaga refers to the tax on crops which

was fixed at 16th of the production

Kammakaras is the term used for the

landless agricultural labour class

Ashvamedha also known as horse

sacrifice is a ritual where a horse is let

loose to wander freely and it was

guarded by the rajarsquos men

Q7) Answer (d)

Explanation

In the Rigvedic period horses were

yoked to chariots that were used in

battles fought to capture land cattle

etc This suggests that the use of horse

chariots began much before the period

of Mahajanapadas

The battles were fought in the Rigvedic

period for cattlersquos lands water an even

to capture people Most men took part

in these wars however there was no

regular army but there were assemblies

where people met and discussed

matters of war Regular armies became

a feature in the Mjahajanapada period

including vast armies of foot soldiers

chariots and elephants

RAUSIAS-FC19E1003 42

Q8) Answer (a)

Explanation

Buddha belonged to the Sakya clan and

passed away at Kusinara

Buddha taught in Prakrit which was the

common language of people

Q9) Answer c

Explanation

There were six schools of philosophy in

ancient India These are known as

Vaishesika Nyaya Samkhya Yoga

Purva Mimansa and Vedanata or Uttara

Mimansa They were founded by sages

Kanada Gautama Kapila Patanjali

Jamini and Vyasa respectively

Q10) Answer b

Explanation

The teachings of Mahavira were

compiled at Valabhi in 6th century AD

Q11) Answer (c)

Explanation

Chanakya is traditionally identified as

Kautilya or Vishnugupta who authored

the ancient Indian political treatise the

Arthashastra

Q12) Answer d

The national emblem of India is an

adaptation of the Lion Capital atop the

Ashoka Pillar of Sarnath Uttar Pradesh

and is combined with the National

Motto Satyameva Jayate

The Rampurva Bull gets the name from

the site of its discovery Rampurva in

Bihar

It is noted for its delicately sculpted

model demonstrating superior

representation of soft flesh sensitive

nostrils alert ears and strong legs It is

a mixture of Indian and Persian

elements

Sankissa is situated in Uttar Pradesh

India

Q13) Ans(a)

Kunwar Singh was a notable leader during the Revolt of 1857 He belonged

to a royal house of Jagdispur Bihar

Q14) Answer b

Explanation

The term Vellalar was used for large

landowners

Q15) Answer c

Explanation

Arikamedu was a coastal settlement

where ships unloaded goods from

distant lands Finds here include a

massive brick warehouse pottery

including amphorae and Arretine ware

Roman lamps glassware and gems have

also been found at the site

Q16) Answer a

Explanation

Muvendar is a Tamil word mentioned in

Sangam poems meaning three chiefs

used for the heads of three ruling

families the Cholas Cheras and

Pandyas

Q17) Ans (c)

Several tribal or kin-based assemblies

such as the Sabha Vidatha and gana

are mentioned in the Rig-veda The

Sabha and the samiti mattered a great

deal in early Vedic times so much so

that the chiefs or the kings showed an

eagerness to win their support

Q18) Ans (a)

Jainism recognised the existence of the

gods but placed them lower than the

jina and did not condemn the varna

system as Buddhism did

Q19) Answer (d)

Explanation

Cholas and Pandyas had developed

powerful coastal cities The most

important city of Cholas was Puhar or

Kaveripattinam and Madurai was the

capital of Pandyas

Q20) Answer b

Explanation

Buddhacharita is the biography of

Buddha and was written by

RAUSIAS-FC19E1003 43

Ashvaghosha

Q21) Answer (a)

Explanation

Tamil poet Appar was a Shiva devotee

So he was a Nayanar saint

Q22) Answer d

Explanation

Samudragupta was a prominent Gupta

ruler whose coins depict him playing a

veena indicating his love for music We

get important historic information from

his Allahabad Prashasti which was

composed by his court poet Harisena

Q23) Answer (b)

Explanation

Vikrama Samvat was founded by

Chandragupta II in the 58 BC as a

mark of victory over the Shakas and

assumed the title of Vikramaditya

Banabhatta wrote Harshavardhanarsquos

biography the Harshacharita in

Sanskrit

Q24) Answer c

Explanation

Sandhi-vigrahika was the minister of

war and peace

Sarthavaha was the leader of the

merchant caravans

Q25) Answer a

Explanation

Xuan Zang (Hsuan-tsang) was a

Chinese traveller who came during the

reign of Harshavardhana

In the decade that began in 630 AD

Xuan Zang came to India through

Kashmir after visiting Central Asia Iran

and Afghanistan

He travelled from north to east and lived

in Bihar for a couple of years

At Nalanda University Xuan Zang

interacted with students and scholars

mastered local languages and

discovered Buddhist stupas

Q26) Answer c

Explanation

Pradakshina patha is a circular path

laid around a stupa in Buddhist

architecture While the rest are a part of

temple architecture

Q27) Answer d

Explanation

All the above-mentioned temples have

an elaborate use of bricks (baked

bricks) along with stone

Q28) Ans (c)

Muhammad Quli Qutab was the Sultan

of Golconda He was a contemporary of

Akbar was very fond of literature and

architecture

The Sultan was a great poet and he

wrote in Dakhini Urdu Persian and

Telgu and has left an extensive diwan or

collection

Recently the Archaeological Survey of

India (ASI) will be using Ground

Penetrating Radar (GPR) to map the

contours of the area around the Bagh-e-

Naya Qila excavated garden inside the

Golconda Fort in Telangana

Q29) Answer a

Explanation

Silappadikaram is a famous Tamil epic

which was written by Ilango around

1800 years ago It is a story of a

merchant named Kovalan who fell in

love with a courtesan named Madhavi

Manimekalai tells the story of the

daughter of Kovalan and Madhavi

Q30) Answer (a)

Explanation

Charaka is the author of Charaka

Samhita which is an important work of

Ayurveda and medicines

Brahmaguptarsquos fame rests mostly on his

Brahma-sphuta-siddhanta which was

an astronomical work It was translated

into Arabic in Baghdad and had a major

impact on Islamic mathematics and

astronomy

Late in his life Brahmagupta wrote

Khandakhadyaka which was an

RAUSIAS-FC19E1003 44

astronomical handbook that employed

Aryabhatarsquos system of starting each day

at midnight

Q31) Answer (c)

Explanation

Amir Khusrau was a famous sufi

musician poet and scholar In 1318 he

noted that there was different language

in every region of this land (Hindustan)

Lahori Kashmiri Dvarsamudri (in

Southern Karnataka) Telangana (in

Andhra Pradesh) Gujari (in Gujarat)

Marsquobari (in Tamil Nadu) Awadhi (in

eastern Uttar Pradesh) and Hindawai (in

the area around in Delhi) etc He went

to explain that Sanskrit did not belong

to any region and that only brahmans

knew it

Q32) Answer c

Explanation

Hiranyagarbha refers to the golden

womb When this ritual was performed

with the help of Brahmanas it was

thought to lead to the rebirth of the

sacrificer as a Khastriya

Q33) Answer d

Explanation

Kadamai refers to a tax on land

revenue

Gwalior Prashasti describes the exploits

of Nagabhata who was a Pratihara king

Q34) Answer b

Explanation

Rajatarangini is a Sanskrit text written

by Kalhana in the 12th century

It was historical chronicle of early India

It is justifiably considered to be the best

and most authentic work of its kind

It covers the entire span of history in

the Kashmir region from the earliest

times to the date of its composition

Q35) Answer c

Explanation

ldquoUrrdquo was the general assembly of the

village ldquoUrrdquo consisted of all the

taxpaying residents of an ordinary

village

Q36) Answer (a)

Explanation

Tarikh was a form of history writing in

the Delhi Sultanate The authors of

tawarikhs were learned men which

included secretaries administrators etc

Q37 Answer (a)

Explanation

Alauddin chose to pay his soldiers salaries in cash rather than iqtas The soldiers would buy their supplies from merchants in Delhi and it was thus feared that merchants would raise their prices To stop this Alauddin controlled the prices of goods in Delhi Prices were carefully surveyed by officers and merchants who did not sell at the prescribed rates were punished

Q38) Answer (d)

Explanation

Delhi first became the capital of a

kingdom under the Tomara Rajputs

who were defeated in the middle of the

twelfth century by the Chauhans (also

referred to as Chahamanas) of Ajmer

It was under the Tomaras and

Chauhans that Delhi became an

important commercial centre Many rich

Jaina merchants lived in the city and

constructed several temples Coins

minted here called dehliwal had a wide

circulation

Q39) Answer (c)

Explanation

Moth ki Masjid was built in the reign of

Sikandar Lodi by his minister

Begumpuri mosque built in the reign of

Muhammad Tughluq was the main

mosque of Jahanpanah the ldquoSanctuary

of the Worldrdquo and his new capital in

Delhi

Quwwat al ndash Islam mosque was

enlarged by Iltutmish and Alauddin

Khalji The minar was built by three

Sultansndash Qutbuddin Aybak Iltutmish

and Firuz Shah Tughluq

RAUSIAS-FC19E1003 45

Q40) Answer (c)

Explanation

Under the Mughals mansabdar was

referred to an individual who held a

mansab ie rank and he received his

salary as revenue assignments called

jagirs

Q41) Ans (b)

The Quit India Movement was a

spontaneous revolt of people against

British rule

The All India Congress Committee met

at Bombay on 8 August 1942 It passed

the famous resolution Quit India and

proposed the starting of a non-violent

mass struggle under Gandhis

leadership to achieve this aim But on

the very next day Gandhi and other

eminent leaders of the Congress were

arrested The Congress was once again

declared illegal

Q42) Ans (c)

The Simon Commission refers to a

group of seven MPs from the United

Kingdom constituted to suggest

constitutional reforms for British India

The Commission consisted of only

British members headed by one of the

senior British politicians Sir John

Simon

So the people of India agitated against

the arrival of Simon Commission

Q43) Ans (a)

He was widely known for his

unfavourable opinion of the economic

consequences of the British rule in

India

In his many writings and speeches and

especially in Poverty and Un-British

Rule in India Naoroji argued that India

was too highly taxed and that its wealth

was being drained away to England

He did not interpret the ancient Indian

texts and restored the self-confidence of

Indians And also he did not stress the

need for eradication of all the social

evils before anything else

Q44) Ans (c)

In August 1932 Prime Minister

MacDonald announced his Communal

Award Great Britainrsquos unilateral

attempt to resolve the various conflicts

among Indiarsquos many communal

interests

The award which was later

incorporated into the act of 1935

expanded the separate-electorate

formula reserved for Muslims to other

minorities including Sikhs Indian

Christians Anglo-Indians Europeans

distinct regional groups Gandhi

undertook a ldquofast unto deathrdquo against

that offer which he viewed as a

nefarious British plot to divide the

Indian society

Q45) Ans (b)

In British India apart from existing

imports and exports there was also a

particular amount of money which

colonial India contributed towards

administration maintenance of the

army war expenses pensions to retired

officers and other expenses accrued by

Britain towards maintenance of her

colony These were known as Home

charges and were paid for almost

entirely by India

The Home charges was made of

following components-

- Interest payable on Indian debt

- Dividend to shareholders of East

India Company

- Funds used to support the India

Office in London

- Funds used to pay salaries and

pensions of British personnel

engaged in India

- Interest on the railways

- Civil and military charges

- Store purchases in England

Q46) Ans (b)

The Lahore session of the Indian

National Congress was held in 1929

under the Presidentship of Jawaharlal

Nehru

The Lahore session of the Indian

National Congress witnessed significant

RAUSIAS-FC19E1003 46

developments in the Indian national

movement

- First the election of Jawaharlal

Nehru to the post of Presidentship of

the Congress was a clear indication

of the growing strength of the

Leftists in the Congress

- Secondly it was in this session that

the Congress for the first time raised

the demand for complete

independence Such demand was

not raised from the Congress

platform earlier

Q47) Ans (b)

It did not provide for separate

electorates for any community or

weightage for minorities However it did

allow for the reservation of minority

seats in provinces having minorities of

at least ten per cent but this was to be

in strict proportion to the size of the

community

There was no provision for complete

Independence for India

Q48) Ans (c)

The religion of early Vedic Aryans was

primarily of worship of nature and

Yajnas

The early Aryan religion was kind of

nature worship Actually the forces

around them which they could not

control or understand were invested

with divinity and were personified as

male or female gods And they

performed some Yajnas also

Q49) Ans (b)

The roads and river-routes were not

immune from robbery It is notable that

Yuan Chwang (Hiuen Tsang) was

robbed of his belongings during

Harshvardanarsquos period

Q50) Ans (c)

Q51) Ans (b)

Purandara Dasa was a saint and great

devotee of Lord Krishna

There is much speculation about where

Purandara Dasa regarded as the

Pitamaha of Carnatic music was born

Recently an expert committee

constituted by the Kannada University

Hampi has come to the conclusion that

Kshemapura Shivamogga district

Karnataka is the birth place of

Purandara Dasa

Q52) Ans (c)

Sri Tyagaraja Sri Shyama Shastry and Sri Muthuswami Dikshitar are considered the trinity of Carnatic music and with them came the golden age in Carnatic music in the 18th-19th

century

Q53) Ans d)

Recently a rare sarcophagus (stone

coffin) which is 2000 years old from the

Iron AgendashMegalithic era was discovered

from a rock-cut cave at Viyur village of

Kollam near Koyilandy in Kozhikode

district Kerala

The coffin containing bone fragments

was found during an excavation ldquoSo

far such a rare finding has been

discovered only from two sites

in Kerala Both these sarcophagi were

recovered from Megalithic sites at

Chevayur and Atholi also in Kozhikode

district

Q54) Ans a)

The megalithic culture in South India was a full-fledged Iron Age culture

Q55) Ans d)

The Cholas Pandyas and Keralaputras

(Cheras) mentioned in Ashokan

inscriptions were probably in the late

megalithic phase of material culture

Q56) Ans d)

Q57) Ans (b)

Raj Kumar Shukla followed Gandhiji all

over the country to persuade him to

come to Champaran to investigate the

problem associated with tinkathia

system

RAUSIAS-FC19E1003 47

Brij Kishore Rajendra Prasad Mahadev

Desai and Narhari Parikh accompanied

Gandhi ji during the Champaran

Satyagraha

Q58) Ans (b)

The Satvahanas started the practice of granting tax-free villages to brahmanas and Buddhist monks

Q59) Ans c)

The objectives of the Programme are

listed as under

- Developing basic tourism

infrastructure

- Promoting cultural and heritage

value of the country to generate

livelihoods in the identified regions

- Enhancing the tourist attractiveness

in a sustainable manner by

developing world-class

infrastructure at the heritage

monument sites

- Creating employment through active

involvement of local communities

- Harnessing tourism potential for its

effects on employment generation

and economic development

- Developing sustainable tourism

infrastructure and ensuring proper

Operations and maintenance

therein

Q60) Ans (b)

The Tribal Cooperative Marketing

Development Federation of India

(TRIFED) came into existence in 1987

It is a national-level apex organization

functioning under the administrative

control of Ministry of Tribal Affairs

Govt of India

TRIFED has its registered and Head

Office located in New Delhi

Q61) Ans (c)

Premchandrsquos novels include

Premashram Rangabhumi Ghaban

Karmabhumi and Godan

Gora is a novel written by Rabindranath

Tagore

138th birth anniversary of Munshi

Premchand was celebrated across the

country

Q62) Ans (b)

Giddha is a traditional pastoral dance

performed by the women of the Punjab

India and Pakistan at festival times

and at the sowing and reaping of the

harvest

By this dance the Punjabi women

reveal their joy expel their suppressed

feelings in a male dominated society

through the performance of Giddha

Since this dance has nothing to do with

men only women can participate in it

During the Teej celebrations Giddha

dance is celebrated in Punjab every

year Teej is a generic name for a

number of festivals that are celebrated

by women in some parts of India

Q63) Ans (a)

Dara Shukoh wrote the remarkable

work called ldquoMajma-ul-Bahrainrdquo or the

ldquoThe confluence of two seasrdquo

The Vice President of India Shri M

Venkaiah Naidu has said that Prince

Dara Shukohrsquos writings can come as a

refreshing source for infusing peace and

harmony He was addressing the

gathering after visiting the exhibition

that showcases the forgotten Prince of

yesteryears Dara Shukoh organized by

Mr Francois Gautier at Indira Gandhi

National Centre for the Arts in New

Delhi

Q64) Ans (c)

The statue Gommateshwara is

dedicated to the Jain God Bahubali

It is a monolithic statue

President Ram Nath Kovind

inaugurated the grand anointing

ceremony mdash Mahamastakabhisheka mdash

held once in 12 years at

Shravanabelagola (Karnataka)

Q65) Ans (c)

Prachi Valley had come up around the

Prachi river Prachi Valley gradually

disappeared

RAUSIAS-FC19E1003 48

The Prachi river originates from

Bhubaneswar

It is a tributary of the Mahanadi and

flows through the districts of Puri

Khurda Cuttack and Jagatsinghpur

and the entire region of the river is

termed as the Prachi Valley

It falls into the Bay of Bengal

Archaeological evidence shows that the

Prachi Valley Civilisation predates both

Harappa and Mohenjo-Daro

The Prachi river originates from

Bhubaneswar

Q66) Ans (d)

These monuments are located in

Chhatarpur district Madhya Pradesh

within Vindhya mountain range

Q67) Ans (a)

The book lsquoThoughts on Pakistanrsquo was

written by Dr BR Ambedkar

On the occasion of the birth anniversary

of Dr BR Ambedkar the president of

India pays homage to this icon of India

In 1924 he founded the Depressed

Classes Institute (Bahishkrit Hitkarini

Sabha) and in 1927 the Samaj Samata

Sangh

Another area of attention for Ambedkar

was education For its spread among

the low classes he set up a network of

colleges by the name of Peoples

Education Society and founded hostels

Q68) Ans(b)

Mehrgarh is a famous Neolithic

settlement in the Indian subcontinent

which is situated in Baluchistan

province Pakistan

A pre-historic rock art site is discovered

in the vast expanse of limestone blocks

on the eastern banks of Naguleru river

near Dachepalli (Andhra Pradesh) It

has thrown light on the Neolithic

civilisation that flourished in Guntur

(Andhra Pradesh) during 1500-2000

BC

Q69) Ans (c)

The 12th and the 13th centuries saw

the emergence of the Kakatiyas They

were at first the feudatories of the

Western Chalukyas of Kalyana Initially

they ruled over a small territory near

Warangal (Telangana)

They introduced Nayakships which was

later adopted and developed by the

Rayas of Vijayanagara

Q70) Ans (a)

The fast had effect of putting pressure

on mill owners who finally agreed to

give the workers a 35 per cent increase

in wages

Google celebrated with a doodle the

132nd birth anniversary of Anasuya

Sarabhai who played a pioneering role

in Indiarsquos labour movement

Q71) Ans (d)

The UNESCOrsquos list of the representative

list of the intangible cultural heritage of

humanity from India are

- Koodiyattam Sanskrit Theatre of

Kerala

- Mudiyettu ritual theatre and dance

drama of Kerala

- Tradition of Vedic Chanting

- Kalbelia folk songs and dances of

Rajasthan

- Ramlila Traditional Performance of

the Ramayana

- Sankirtana ritual singing

drumming and dancing of Manipur

- Ramman religious festival and

ritual theatre of the Garhwal

Himalayas India

- Traditional brass and copper craft of

utensil making among the Thatheras

of Jandiala Guru Punjab India

- Chhau dance classical Indian dance

originated in the eastern Indian

states

- Buddhist chanting of Ladakh

recitation of sacred Buddhist texts

in the trans-Himalayan Ladakh

region Jammu and Kashmir India

- Yoga

- Nouroz

- Kumbh Mela

RAUSIAS-FC19E1003 49

Q72) Ans(b)

The President of India Shri Ram Nath Kovind inaugurated the Hornbill Festival and State Formation Day celebrations of Nagaland in Kisama

The festival is named after the Indian hornbill the large and colourful forest bird which is displayed in the folklore of most of the states tribes

The major recognized tribes of Nagaland are Angami Ao Chakhesang Chang

Kuki Rengma and Zeling etc

Onge Jarawa and Sentinelese are the

tribes of Andman amp Nicobar Islands

Q73) Ans (c)

The Rashtrakutas rule in the Deccan lasted for almost two hundred years till the end of the tenth century The Rashtrakutas rulers were tolerant in their religious views and patronized not only Shaivism and Vaishnavism but

Jainism as well

The famous rock-cut temple of Shiva at Ellora was built by one of the Rashtrakutas kings Krishna I in the ninth century His successor Amoghavarsha was a Jain but he also

patronized other faiths

The Rashtrakutas allowed Muslims traders to settle and permitted Islam to

be preached in their dominions

Recently increasing defacement at the prehistoric rock paintings of Pandavulagutta Telangana has created a cause for grave concern It can spoil

the prehistoric rock

Pandavulagutta is home to

- Painted rock shelters dating to

10000 BC-8000 BC

- An 8th century inscription of the

Rashtrakuta period and

- Painted frescoes from the 12th century Kakatiya empire

Q74) Ans (b)

In 1828 Raja Ram Mohan Roy founded a new religious society the Brahma Sabha later known as the Brahmo

Samaj

Debendranath Tagore headed the Tattvabodhini Sabha which was

engaged in search of spiritual truth

Its purpose was to purify Hinduism and to preach monotheism or belief in one God

The new society was to be based on the twin pillars of reason and the Vedas and

Upanishads

Recently Sadharan Brahmo Samaj (SBS) has entered into a legal battle with the West Bengal government due

to some legal issue

Q75) Ans (c)

The Chishti order was established in India by Khwaja Moinuddin Chishti who came to India around 1192 The Chishtirsquos are considered to be the most influential of the groups of Sufis who migrated to India in the late twelfth century They adapted successfully to the local environment and adopted several features of Indian devotional

traditions

The historical dargah of Sufi mystic Khwaja Moinuddin Chishti in Ajmer is all set to get a facelift This 13 th century dargah has been included among the Swachh Iconic Places a clean-up initiative focused on iconic

heritage spiritual and cultural places

Page 33: GENERAL STUDIES (PAPER I) · Test is part of Rau’s IAS Test series for Preliminary Exam 2019 FOUNDATION + CURRENT AFFAIRS GENERAL STUDIES (PAPER –I) FOUNDATION TEST –III TOPIC:

FC19H1003 33

- बगदाद म इसका अनिाद अरबी भािा म वकया

गया था

- इसका इसलावमक गवणत और खग ल-विजञान पर

महतवपणि परभाि पडा था

- बाद म अपन जीिनकाल म बरहमगपत न

ldquoखोडखयाकrdquo वलखी ज एक खग लीय पससतका

(एक छ िी पसतक) थी

- इसम आयिभटट की अिि-रावतर क परतयक वदन की

शरआत परणाली का परय ग वकया गया था

Q31) उततर (c)

सपषटीकरण

- अमीर खसर एक परवसद सफी सोगीतकार कवि

और विदवान थ

- 1318 म उनह ोन पाया वक इस भवम (वहोदसतान) क

हर कषतर म अलग-अलग भािा थी लाहौरी

कशमीरी दवारसमदरी (दवकषणी कनाििक म)

तलोगाना (आोधर परदश म) गजरी (गजरात म)

माबारी (तवमलनाड म ) अििी (पिी उततर परदश

म) और वहोदिी (वदलली क आस-पास क कषतर म)

आवद

- उनह न यह बताया वक सोसकत वकसी भी कषतर स

सोबोवित नही ो थी और किल बराहमण ही इस भािा

का जञान रखत थ

Q32) उततर (c)

सपषटीकरण

- वहरणय-गभि सववणिम गभि क सोदवभित करता ह

- जब बराहमण ो की सहायता स यह अनषठान वकया

जाता था त यह माना जाता था वक बवल दन िाल

का कषवतरय क रप म पनजिनम ह गा

Q33) उततर (d)

सपषटीकरण

- कदमई भवम राजसव पर कर क सोदवभित करता

- गवावलयर परशससत म नागभि क दवारा वकय गए

श िण का िणिन वकया गया ह |

- नागभि एक परवतहार राजा था

Q34) उततर (b)

सपषटीकरण

- राजतरो वगनी 12िी ो शताबदी म कलहन क दवारा

रवचत एक सोसकत पसतक (िकसट) ह

- यह परारसमभक भारत की ऐवतहावसक इवतितत थी

- तकि सोगत रप स इस अपन परकार की सिोततम

और सिािविक विशवसनीय कवत माना जाता ह

- यह कशमीर कषतर क पराचीनतम समय स लकर

उसकी रचना की तारीख तक क समपणि इवतहास

का आचछादन करती ह

Q35) उततर (c)

सपषटीकरण

- गााि की आम सभा क ldquoउरrdquo कहा जाता था

- ldquoउरrdquo म गााि क सभी कर दन िाल वनिासी

शावमल ह त थ

Q36) उततर (a)

सपषटीकरण

- वदलली सलतनत म ldquoतारीखrdquo इवतहास लखन का

एक रप था

- ldquoतािरीखrdquo क लखक विदवान परि ह त थ वजनम

सवचि परशासक इतयावद शावमल थ

Q37) उततर (a)

सपषटीकरण

- अलाउददीन सखलजी अपन सवनक ो क ितन का

भगतान नकद म करता था न वक इकता क रप

- सवनक अपना सामान वदलली म वयापाररय ो स

खरीदत थ अतः इस बात का भय था वक वयापारी

कही ो िसतओो का मलय न बिा द

- इसकी र कथाम क वलए अलाउददीन सखलजी न

वदलली म कीमत ो क वनयसित वकया

FC19H1003 34

- अविकारीगण धयानपििक मलय ो का सिकषण करत

थ तथा ज वयापारी वनिािररत मलय पर माल नही ो

बचत थ उनक दसणडत वकया जाता था

Q38) उततर (d)

सपषटीकरण

- वदलली सििपरथम त मर राजपत ो क अिीन उनक

सामराजय की राजिानी बनी थी

- 12िी ो शताबदी क मधय म अजमर क चौहान ो

(वजनह चाहमान ो क नाम स भी जाना जाता ह) न

त मर राजपत ो क परावजत वकया था

- त मर ो और चौहान ो क अिीन वदलली एक

महतवपणि िावणसजयक क दर बन गया था

- कई जन वयापारी यहाा रहन लग थ और उनह ोन

कई मोवदर भी बनिाए

- यहाा पर मवदरत वसक वजनह ldquoदहलीिालrdquo क नाम

स जाना जाता था वयापक रप स परचलन म थ

Q39) उततर (c)

सपषटीकरण

- म ठ की मसिद का वनमािण वसको दर ल दी क

राजयकाल म उसक मिी क दवारा करिाया गया

था

- बगमपरी मसिद का वनमािण महममद तगलक क

शासनकाल म हआ था

- यह मसिद विशव का पणयथथान (The

Sanctuary of the World) और वदलली म महममद

तगलक की नई राजिानी जहाोपनाह की मखय

मसिद थी

- कववत- अल - इसलाम मसिद का विसतार

इलतसिश और अलाउददीन सखलजी न वकया था

- मीनार का वनमािण तीन सलतान ो कतबददीन ऐबक

इलतसिश और वफर ज शाह तगलक क दवारा

करिाया गया था

Q40) उततर (c)

सपषटीकरण

- मगल ो क अिीन मनसबदार शबद उस वयसकत क

वलए सोदवभित वकया जाता था वजसक पास मनसब

(अथाित पद) ह ता था

- उस अपना ितन राजसव कायो वजनह जागीर कहत

थ क रप म परापत ह ता था

Q41) उततर (b)

सपषटीकरण

- ldquoभारत छ ड आोद लनrdquo वबरविश शासन क

सखलाफ ल ग ो का एक सवाभाविक विदर ह था

- असखल भारतीय काोगरस सवमवत न 8 अगसत 1942

क बमबई म एक बठक का आय जन वकया था

- इस बठक म परवसद सोकलप ldquoभारत छ ड rdquo क

पाररत वकया गया और इस उददशय क परापत करन

क वलए गाोिी क नततव म एक अवहोसक जन सोघिि

आोद लन की शरआत का परसताि वदया गया

- लवकन अगल ही वदन गाोिी और काोगरस क अनय

परमख नताओो क वगरफतार कर वलया गया

- काोगरस क एक बार वफर अिि घ वित वकया गया

था

Q42) उततर (c)

सपषटीकरण

- साइमन कमीशन यनाइविड वको गडम क सात

साोसद ो का एक समह था

- इस वबरविश भारत क वलए सोििावनक सिार ो का

सझाि दन क वलए गवठत वकया गया था

- इस आय ग म िररषठ वबरविश राजनता सर जॉन

साइमन क नततव म किल वबरविश सदसय ही

शावमल थ

- इसवलए भारत क ल ग ो न साइमन कमीशन क

आगमन क विरद आोद लन वकया था

Q43) उततर (a)

सपषटीकरण

bull दादा भाई नौर जी भारत म वबरविश शासन क

आवथिक पररणाम ो क बार म अपनी विर िी

(परवतकल) राय क वलए जान जात थ

FC19H1003 35

bull अपन कई लख ो और भािण ो म विशि रप स

ldquoपाििी एो ड अन-वबरविश रल इन इसणडया

(Poverty and Un-British Rule in India) म

नौर जी न यह तकि वदया वक भारत पर अतयविक

कर लगाया गया था और इसकी सोपवतत इोगलड की

ओर परिावहत की जा रही थी

bull उनह ोन पराचीन भारतीय गरोथ ो की वयाखया करन

का और भारतीय ो क आिविशवास क बहाल

करन पर कायि नही ो वकया था

उनह ोन वकसी और बात स पहल सभी सामावजक

बराइय ो क उनमलन की आिशयकता पर भी बल

नही ो वदया था

Q44) उततर (c)

सपषटीकरण

bull अगसत 1932 म वबरविश परिानमोतरी मकड नालड न

अपन साोपरदावयक परसकार (The Communal

Award) की घ िणा की थी

bull यह भारत क कई साोपरदावयक वहत ो क बीच विवभनन

सोघिो क हल करन क वलए वबरिन का एकतरफा

परयास था

bull यह परसकार (Award) बाद म 1935 क

अविवनयम (The Act of 1935) म शावमल वकया

गया था

bull इस साोपरदावयक परसकार न मससलम ो क वलए

आरवकषत एक अलग वनिािचक मणडल फॉमिल का

विसतार अनय अलपसोखयक ो क वलए वकया था

वजसम वसख ो भारतीय ईसाइय ो आोगल-भारतीय

समदाय यर पीय समदाय तथा विवशषट कषतरीय

समह ो क शावमल वकया गया था

bull गाोिी न इस परसताि क भारतीय समाज क

विभावजत करन क वलए एक घवणत वबरविश

सावजश क रप म दखा और उसक सखलाफ

आमरण अनशन वकया

Q45) उततर (b)

सपषटीकरण

मौजदा आयात और वनयाित क अवतररक़त

औपवनिवशक भारत क वनमनवलसखत खचो क

वलए एक विशिवनवशचत िन रावश भी दनी पडती

थी

(i) परशासन क वयय

(ii) सना क रख-रखाि क वयय

(iii) यद क वयय

(iv) सिावनितत अविकाररय ो की पशन तथा

(v) वबरिन दवारा अपनी उपवनिश बसती

(कॉल नी) क रख-रखाि क वयय

इनह गह शलक (Home Charges) क रप म

जाना जाता था और लगभग परी तरह स भारत क

दवारा इनका भगतान वकया जाता था

bull गह शलक म वनमनवलसखत घिक शावमल थ

(i) भारतीय ऋण पर दय बयाज

(ii) ईसट इोवडया को पनी क शयरिारक ो क

लाभाोश

(iii) लोदन म भारत कायािलय चलान क वलए िन

(iv) भारत म वनयकत वबरविश कवमिय ो क ितन

और पशन का भगतान करन क वलए िन

(v) रलि पर बयाज

(vi) नागररक और सनय शलक

(vii) इोगलड म सट र (सामगरी) की खरीद

Q46) उततर (b)

सपषटीकरण

bull भारतीय राषटर ीय काोगरस का लाहौर सतर 1929 म

जिाहरलाल नहर की अधयकषता म आय वजत

वकया गया था

bull इस सतर म भारतीय राषटर ीय आोद लन स समबसित

कई महतवपणि पररणाम सामन आय थ

(i) सििपरथम इस सतर म काोगरस क अधयकष पद

पर जिाहरलाल नहर क चना गया था ज

काोगरस म िामपोवथय ो की बिती हई ताकत

का सपषट सोकत था

(ii) दसरा इस सतर म पहली बार काोगरस न पणि

सवतोतरता की माोग क उठाया था

इस परकार की माोग काोगरस मोच स पहल कभी भी

नही ो उठाई गई थी

Q47) उततर (b)

सपषटीकरण

FC19H1003 36

bull इस ररप िि न वकसी भी समदाय क वलए पथक

वनिािचक मोडल अथिा अलपसोखयक ो क वलए

भाराोश की वसफाररश नही ो की थी

bull तथावप इस ररप िि न उन पराोत ो म अलपसोखयक

सीि ो क आरकषण की अनमवत दी थी जहाा पर कम

स कम दस परवतशत अलपसोखयक ह

bull लवकन यह समदाय क आकार क अनपात म ह ना

चावहए था

bull इस ररप िि म भारत क वलए पणि सवतोतरता क

वलए क ई पराििान नही ो था

Q48) उततर (c)

सपषटीकरण

bull आरो वभक िवदक आयो का िमि मखय रप स

परकवत की पजा और यजञ था

bull परारो वभक आयि िमि परकवत की पजा क समान था

bull िासति म उनक चार ो ओर की शसकतयाा वजनह न

त ि वनयोवतरत कर सकत थ और न ही समझ पाए

थ उनह वदवयता क साथ वनिवशत वकया गया तथा

उनह मादा या नर दिीदिताओो क रप म

परतीकतव वकया गया था

bull उनह ोन कछ यजञ ो का भी वनषपादन वकया था

Q49) उततर (b)

सपषटीकरण

bull सडक और नदी-मागि (जल-मागि) डकती स

सरवकषत नही ो थ

bull उललखनीय ह वक हिििििन क शासनकाल क

दौरान यआन चिाोग (हयएन साोग) का सारा

सामान लि वलया गया था

Q50) उततर (c)

सपषटीकरण

परशन म वदए गए द न ो कथन सही ह

Q51) उततर (b)

सपषटीकरण

bull परोदर दास एक सोत और भगिान कषण क एक

महान भकत थ

bull परोदर दास क कनाििक सोगीत क वपतामह क

रप म जाना जाता ह

bull यदयवप उनक जनम-थथान क बार म काफी

अिकल लगाई जाती रही ह

bull तथावप अब कननड विशवविदयालय हमपी क दवारा

गवठत एक विशिजञ सवमवत इस वनषकिि पर पहोची

ह वक उनका जनम थथान सोभितया कनाििक का

एक छ िा-सा गााि कषमपरा (वशिम गगा वजला)

था

Q52) उततर (c)

सपषटीकरण

bull शरी तयागराज शरी शयाम शासतरी और शरी मथसवामी

दीवकषतर क कनाििक सोगीत की वतरमवति माना

जाता ह

bull उनक कारण ही 18िी ो-19िी ो शताबदी म कनाििक

सोगीत का सववणिम यग आया था

Q53) उततर (d)

सपषटीकरण

bull अभी हाल ही म लौह यगीन-महापािावणक काल

का 2000 ििि पराना एक दलिभ सारक फगस

(Sarcophagus) (पतथर का ताबत) क ललम क

वियर गाोि (क वयलडी क पास वजला क वझक ड

करल राजय) की एक रॉक-कि गफा स ख जा गया

bull यह ताबत वजसम हविय ो क िकड थ खदाई क

दौरान वमला

bull अभी तक इस परकार की दलिभ ख ज करल क

मातर द ही थथान ो स हई ह

bull य द न ो सारक फगी (Sarcophagi) (पतथर क

ताबत) चियर और अथ ली (वजला क वझक ड) क

महापािाण थथल ो स वमल ह

Q54) उततर (a)

सपषटीकरण

FC19H1003 37

दवकषण भारत म महापािाण सोसकवत एक पणि

विकवसत लौह यगीन सोसकवत थी

Q55) उततर (d)

सपषटीकरण

bull च ल पाणडय और करलपतर (चर) इन तीन ो का

उललख अश क क अवभलख ो म वकया गया ह

bull सोभितः य भौवतक सोसकवत क उततर

महापािावणक चरण म थ

Q56) उततर (d)

सपषटीकरण

bull भीमा-क रगाोि की लडाई ततीय आोगल-मराठा

यद का वहससा थी

Q57) उततर (b)

सपषटीकरण

bull राजकमार शकल न गाोिीजी क चोपारण आन तथा

वतनकवथया परणाली स जडी समसया की जाोच क

वलए रारी करन क वलए दश भर म उनका

अनसरण वकया था

bull बज वकश र राजदर परसाद महादि दसाई और

नरहरी पाररख चोपारण सतयागरह क दौरान गाोिी

जी क सहय गी थ

Q58) उततर (b)

सपषटीकरण

bull बराहमण ो और बौद मठिाररय ो क कर-मकत गााि

अनदान म दन की परथा सतिाहन ो न आरमभ की

थी

Q59) उततर (c)

सपषटीकरण

इस कायिकरम क उददशय वनमनानसार ह

(i) बवनयादी पयििन आिाररक सोरचना का विकास

करना

(ii) चयवनत (पहचान वकय गए) कषतर ो म आजीविका क

सजन क वलए दश क साोसकवतक और विरासत

मलय ो क बिािा दना

(iii) विरासत समारक थथल ो पर विशव सतरीय आिाररक

सोरचना विकवसत करक एक सतत तरीक स

पयििक आकििण म िसद करना

(iv) थथानीय समदाय ो की सवकरय भागीदारी क माधयम

स र रगार ो का सजन करना

(v) र रगार उतपादन और आवथिक विकास क वलए

पयििन कषमता का उन पर परभाि का उपय ग

करना तथा

(vi) िारणीय पयििन आिाररक सोरचना का विकास

करना और उसका उवचत सोचालन तथा

रखरखाि सवनवशचत करना

Q60) उततर (b)

सपषटीकरण

bull यह वनकाय ििि 1987 म अससततव म आया था

bull यह एक राषटर ीय सतर का शीिि सोगठन ह ज भारत

सरकार क जनजातीय मामल ो क मोतरालय क

परशासवनक वनयोतरण क अिीन काम कर रहा ह

bull इसका पोजीकत और परिान कायािलय नई वदलली

म सथथत ह

Q61) उततर (c)

सपषटीकरण

bull परमचोद क उपनयास ो म परमाशरम रोगभवम गबन

कमिभवम और ग दान शावमल ह

bull ग रा रिी ोदरनाथ िग र क दवारा रवचत उपनयास ह

bull अभी हाल ही म मोशी परमचोद की 138िी ो जयोती दश

भर म मनाई गई थी

Q62) उततर (b)

सपषटीकरण

bull ldquoवगदाrdquo पोजाब (भारत) एिो पावकसतान की

मवहलाओो क दवारा तयौहार क समय और फसल

की बिाई तथा किाई क अिसर पर वकया जान

िाला एक पारोपररक दहाती नतय ह

FC19H1003 38

bull इस नतय क माधयम स पोजाबी मवहलाऐो अपनी

परसननता परकि करती ह तथा वगदा क परदशिन क

माधयम स परि िचिसव िाल समाज म मवहलाओो

की दबी हई भािनाओो क परकि करती ह

bull चोवक इस नतय का परि ो क साथ क ई सोबोि नही ो

ह अतः किल मवहलाऐो ही इसम भाग ल सकती

bull हर साल तीज समार ह क दौरान पोजाब म वगदा

नतय वकया जाता ह

तीज भारत क कछ भाग ो म मवहलाओो क दवारा

मनाया जान िाल कई तयौहार ो क वलए एक

वयापक नाम ह

Q63) उततर (a)

सपषटीकरण

- मजम-उल-बहरीन या द समदर ो का सोगम

नामक उललखनीय रचना दारा वशक ह क दवारा

वलखी थी

- भारत क उपराषटर पवत शरी एम िकया नायड न कहा

ह वक राजकमार दारा वशक ह की रचनाएा शाोवत

और सदभाि क बिािा दन क वलए एक तारा सर त

क रप म सामन आ सकती ो ह

- उपराषटर पवत गत ििो क भला वदए गए राजकमार

दारा वशक ह क परदवशित परचवलत करन हत

आय वजत एक परदशिनी का दौरा करन क बाद एक

सभा क सोब वित कर रह थ

- इस परदशिनी का आय जन फर क इस गौवियर

(Francois Gautier) क दवारा lsquoइोवदरा गाोिी नशनल

सिर फॉर द आििसrsquo (The Indira Gandhi

National Centre for the Arts) नई वदलली म

वकया गया था

Q64) उततर (c)

सपषटीकरण

- ग मतशवर परवतमा जन भगिान बाहबली क

समवपित ह

- यह एक एक-चटटानी पतथर की मवति ह

- राषटर पवत राम नाथ क विोद न शरिणबलग ला

(कनाििक) म आय वजत वकय जान िाल भवय

अवभिक समार ह महामसतकावभिक का

उदघािन वकया था

- यह समार ह 12 ििो म एक बार ह ता ह

Q65) उततर (c)

सपषटीकरण

bull पराची घािी पराची नदी क चार ो ओर फली हई थी

bull पराची घािी िीर-िीर विलपत ह गई थी

bull पराची नदी भिनशवर स वनकलती ह

bull यह महानदी की एक सहायक नदी ह और यह

परी खदाि किक तथा जगतवसोहपर वजल ो स

ह कर बहती ह

bull इस नदी क पर कषतर क पराची घािी कहा जाता ह

bull यह नदी बोगाल की खाडी म वगरती ह

परातासतवक साकषय स पता चलता ह वक पराची घािी

सभयता हडपपा और म हनज दाड द न ो की

पिििती ह

Q66) उततर (d)

सपषटीकरण

य समारक छतरपर वजल (मधय परदश) म विोधयाचल

पिित शरोखला म सथथत ह

Q67) उततर (a)

सपषटीकरण

bull थॉिस ऑन पावकसतान नामक पसतक डॉ बी

आर अमबडकर न वलखी थी

bull डॉ बी आर अमबडकर की जयोती क अिसर पर

भारत क राषटर पवत न भारत की इस महान हसती

क शरदाोजवल अवपित की थी

bull डॉ बी आर अमबडकर न 1924 म वडपरथड

कलावसर इोसटीटयि (दवलत िगि सोथथान -

बवहषकत वहतकाररणी सभा) और 1927 म समाज

समता सोघ की थथापना की थी

bull अमबडकर का धयान वशकषा कषतर की ओर भी था

bull उनह ोन वशकषा क वनमन िगो म फलान क वलए

पीपलस एजकशन स साइिी (The Peoples

Education Society) क नाम स महाविदयालय ो क

नििकि और छातरािास ो की थथापना की थी

FC19H1003 39

Q68) उततर (b)

सपषटीकरण

bull महरगि भारतीय उपमहादवीप म एक परवसद

निपािाण बसती ह ज बलवचसतान पराोत

पावकसतान म सथथत ह

bull दचपलली (आोधर परदश) क पास नागलर नदी क

पिी ति ो पर चना पतथर क बलॉक क विशाल

विसतार म एक पिि-ऐवतहावसक रॉक आिि थथल की

ख ज की गई ह

bull इसन 1500-2000 ईसा पिि क दौरान गोिर (आोधर

परदश) म विकवसत निपािाण सभयता पर परकाश

डाला ह

Q69) उततर (c)

सपषटीकरण

bull 12िी ो सदी और 13िी ो सदी म काकाविय िोश का

उदय हआ था

bull ि पहल कलयाण क पवशचमी चालकय ो क सामोत थ

bull परारोभ म उनह ोन िारोगल (तलोगाना) क पास एक

छ ि स कषतर पर शासन वकया था

bull उनह ोन ldquoनायक वयिथथाrdquo की शरआत की थी

वजस बाद म विजयनगर क राय शासक ो न

अपनाया और विकवसत वकया था

Q70) उततर (a)

सपषटीकरण

bull गाोिीजी क अनशन स वमल मावलक ो पर दबाि

पडा था ज अोततः शरवमक ो क ितन म 35 परवतशत

की िसद करन क वलए सहमत हए थ

bull गगल (Google) न अनसया साराभाई वजनह ोन

भारत क शरवमक आोद लन म एक अगरणी भवमका

वनभाई थी की 132िी ो जयोती डडल (Doodle) का

वनमािण करक मनाई

Q71) उततर (d)

सपषटीकरण

भारत स यनसक की मानिता की अमति साोसकवतक

विरासत की परवतवनवि सची म वनमनवलसखत शावमल ह

bull कवडयटटम करल का सोसकत रोगमोच

bull मवडयिि करल का अनषठान रोगमोच और नतय

नाविका

bull िवदक मि जाप की परोपरा

bull राजथथान क कालबवलया ल क गीत और नतय

bull रामलीला रामायण का पारोपररक परदशिन

bull सोकीतिन मवणपर का अनषठान गायन ढ ल िादन

और नतय

bull रममन भारत क गििाल वहमालय का िावमिक

तयौहार और अनषठान रोगमोच

bull जाोदीयाला गर पोजाब क ठठर ो की पीतल और

ताोब क वशलप स वनवमित बतिन ो की पारोपररक कला

bull छाऊ नतय पिी भारतीय राजय ो म जनमी शासतरीय

भारतीय नतय कला

bull लददाख का बौद मि जाप िर ाोस-वहमालयी लददाख

कषतर तथा जमम-कशमीर म पवितर बौद गरोथ ो का पाठ

bull य ग

bull नौर र

bull को भ मला

Q72) उततर (b)

सपषटीकरण

bull भारत क राषटर पवत शरी राम नाथ क विोद न

वकसामा नागालड म हॉनिवबल मह रति और

राजय गठन वदिस समार ह का उदघािन वकया

था

bull हॉनिवबल मह रति का नाम भारतीय हॉनिवबल क

नाम पर पडा ह ज एक विशाल और रोगीन जोगली

पकषी ह

bull यह पकषी नागालड राजय की अविकतर जनजावतय ो

की ल ककथाओो म उसललसखत ह

bull नागालड की परमख मानयता परापत जनजावतयाा ह

अोगामी आओ चखसोग चाोग ककी रगमा और

रवलोग आवद

bull ओोग जारिा और ससिनलीस अोडमान-वनक बार

दवीप समह की जनजावतयाा ह

FC19H1003 40

Q73) उततर (c)

सपषटीकरण

bull दकन म राषटर कि शासन दसिी ो सदी क अोत तक

लगभग 200 ििो तक रहा था

bull राषटर कि शासक अपन िावमिक विचार ो म सवहषण

bull उनह ोन न किल शि िमि और िषणि िमि बसलक

जन िमि क भी सोरकषण वदया था

bull एल रा म वशि क परवसद रॉक कि मोवदर का

वनमािण नौिी ो सदी म राषटर कि राजा कषण परथम न

करिाया था

bull उसका उततराविकारी अम घििि जन था लवकन

उसन अनय िमो क भी सोरकषण परदान वकया था

bull राषटर कि ो न मसलमान वयापाररय ो क बसन की

अनमवत दी थी

bull उनह न अपन अविराजय ो म इसलाम क उपदश दन

की भी अनमवत दी थी

bull अभी हाल ही म पाोडिलागटटा (तलोगाना) क

परागवतहावसक चटटान वचतर ो क कषरण की बिती हई

घिनाएा एक गोभीर वचोता का वििय ह

bull यह परागवतहावसक चटटान क नकसान पहाचा

सकता ह

bull पाोडिलागटटा वनमनवलसखत क वलए जाना जाता ह

- 10000 ईसा पिि स 8000 ईसा पिि क वचवतरत

चटटानी आशरय ो क वलए

- राषटर कि काल क एक 8 िी ो सदी क

वशलालख क वलए और

- 12िी ो सदी क काकविय सामराजय क वभवतत

वचतर ो क वलए

Q74) उततर (b)

सपषटीकरण

bull 1828 म राजा राम म हन रॉय न एक नय िावमिक

समाज बरहम सभा की थथापना की थी वजस बाद

म बरहम समाज क नाम स जाना गया था

bull दिदरनाथ िग र न ततवब विनी सभा की अधयकषता

की थी ज आधयासिक सतय की ख ज म सोलि

थी

bull इसका उददशय वहोद िमि क शद करन का और

एकशवरिाद (एक ईशवर म आथथा) का परचार करना

था

bull नय समाज की थथापना क आिार थ कारण

(तकि ) क द सतमभ तथा िद और उपवनिद

bull अभी हाल ही म सािारण बरहम समाज का कछ

काननी मदद ो क लकर पवशचम बोगाल सरकार क

साथ काननी वििाद चल रहा ह

Q75) उततर (c)

सपषटीकरण

bull भारत म वचशती वसलवसल की थथापना खवाजा

म इनददीन वचशती क दवारा की गयी थी

bull ि 1192 ईसवी क आसपास भारत आय थ

bull वचशतीय ो क बारहिी ो शताबदी क उततरािि म भारत

म आन िाल सफीय ो क समह ो म सबस

परभािशाली माना जाता ह

bull उनह ोन थथानीय िातािरण क साथ सफलतापििक

अनकलन वकया और उनह ोन भारतीय भसकत

परोपराओो क कई पहलओो क अपनाया

bull अजमर म सफी अपरकि खवाजा म इनददीन वचशती

की ऐवतहावसक दरगाह क एक नया रप दन की

तयारी की जा रही ह

bull इस 13िी ो शताबदी की दरगाह क ldquoसवचछ

आइकॉवनक थथल ोrdquo (Swacch Iconic Places) म

शावमल वकया गया ह ज परवतवषठत विरासत

आधयासिक और साोसकवतक थथान ो पर क वदरत

य जना ह

FC19H1003 41

ANSWERS amp EXPLANATION OF

NCERT History Class VI-X + Current Affairs

(FC19E1003)

Q1) Answer c

Explanation

Rigveda consists of more than a

thousand hymns dedicated to gods and

goddesses These hymns were

composed by sages and learnt by men

however a few were composed by

women like Apala Ghosa Lopamudra

Maitreyi and Gargi

Rigveda consists of many hymns in the

form of dialogues We get an example of

a dialogue between a sage named

Vishwamitra and two rivers (Beas and

Sutlej) that were worshipped as

goddesses This suggests that he

belonged to the Vedic period

Q2) Answer b

Explanation

Traces of ash have been found from

Kurnool Caves suggesting that people

were familiar with the use of fire

It is situated in Andhra Pradesh

Q3) Answer c

Explanation

Burzahom is a prehistoric site in

present day Kashmir where people built

pit houses which were dug into the

ground with steps leading into them

These may have provided shelter in cold

weather

Q4) Answer c

Explanation

Epigraphy is defined as the study of

inscriptions

Manuscriptology is the study of history

and literature through the use of hand

written documents

Palaeography refers to the study of

ancient writing systems and the

deciphering and dating of historical

manuscripts

Numismatics refers to the study of

coins

Q5) Answer a

Explanation

Charaka Samhita was written by

Charaka and is an important book on

Ayurveda and medicine

He was a practitioner of the traditional

system of Indian medicine known as

Ayurveda

Charaka is thought to have flourished

sometime between the 2nd century BCE

and the 2nd century CE

Q6) Answer b

Explanation

Bhaga refers to the tax on crops which

was fixed at 16th of the production

Kammakaras is the term used for the

landless agricultural labour class

Ashvamedha also known as horse

sacrifice is a ritual where a horse is let

loose to wander freely and it was

guarded by the rajarsquos men

Q7) Answer (d)

Explanation

In the Rigvedic period horses were

yoked to chariots that were used in

battles fought to capture land cattle

etc This suggests that the use of horse

chariots began much before the period

of Mahajanapadas

The battles were fought in the Rigvedic

period for cattlersquos lands water an even

to capture people Most men took part

in these wars however there was no

regular army but there were assemblies

where people met and discussed

matters of war Regular armies became

a feature in the Mjahajanapada period

including vast armies of foot soldiers

chariots and elephants

RAUSIAS-FC19E1003 42

Q8) Answer (a)

Explanation

Buddha belonged to the Sakya clan and

passed away at Kusinara

Buddha taught in Prakrit which was the

common language of people

Q9) Answer c

Explanation

There were six schools of philosophy in

ancient India These are known as

Vaishesika Nyaya Samkhya Yoga

Purva Mimansa and Vedanata or Uttara

Mimansa They were founded by sages

Kanada Gautama Kapila Patanjali

Jamini and Vyasa respectively

Q10) Answer b

Explanation

The teachings of Mahavira were

compiled at Valabhi in 6th century AD

Q11) Answer (c)

Explanation

Chanakya is traditionally identified as

Kautilya or Vishnugupta who authored

the ancient Indian political treatise the

Arthashastra

Q12) Answer d

The national emblem of India is an

adaptation of the Lion Capital atop the

Ashoka Pillar of Sarnath Uttar Pradesh

and is combined with the National

Motto Satyameva Jayate

The Rampurva Bull gets the name from

the site of its discovery Rampurva in

Bihar

It is noted for its delicately sculpted

model demonstrating superior

representation of soft flesh sensitive

nostrils alert ears and strong legs It is

a mixture of Indian and Persian

elements

Sankissa is situated in Uttar Pradesh

India

Q13) Ans(a)

Kunwar Singh was a notable leader during the Revolt of 1857 He belonged

to a royal house of Jagdispur Bihar

Q14) Answer b

Explanation

The term Vellalar was used for large

landowners

Q15) Answer c

Explanation

Arikamedu was a coastal settlement

where ships unloaded goods from

distant lands Finds here include a

massive brick warehouse pottery

including amphorae and Arretine ware

Roman lamps glassware and gems have

also been found at the site

Q16) Answer a

Explanation

Muvendar is a Tamil word mentioned in

Sangam poems meaning three chiefs

used for the heads of three ruling

families the Cholas Cheras and

Pandyas

Q17) Ans (c)

Several tribal or kin-based assemblies

such as the Sabha Vidatha and gana

are mentioned in the Rig-veda The

Sabha and the samiti mattered a great

deal in early Vedic times so much so

that the chiefs or the kings showed an

eagerness to win their support

Q18) Ans (a)

Jainism recognised the existence of the

gods but placed them lower than the

jina and did not condemn the varna

system as Buddhism did

Q19) Answer (d)

Explanation

Cholas and Pandyas had developed

powerful coastal cities The most

important city of Cholas was Puhar or

Kaveripattinam and Madurai was the

capital of Pandyas

Q20) Answer b

Explanation

Buddhacharita is the biography of

Buddha and was written by

RAUSIAS-FC19E1003 43

Ashvaghosha

Q21) Answer (a)

Explanation

Tamil poet Appar was a Shiva devotee

So he was a Nayanar saint

Q22) Answer d

Explanation

Samudragupta was a prominent Gupta

ruler whose coins depict him playing a

veena indicating his love for music We

get important historic information from

his Allahabad Prashasti which was

composed by his court poet Harisena

Q23) Answer (b)

Explanation

Vikrama Samvat was founded by

Chandragupta II in the 58 BC as a

mark of victory over the Shakas and

assumed the title of Vikramaditya

Banabhatta wrote Harshavardhanarsquos

biography the Harshacharita in

Sanskrit

Q24) Answer c

Explanation

Sandhi-vigrahika was the minister of

war and peace

Sarthavaha was the leader of the

merchant caravans

Q25) Answer a

Explanation

Xuan Zang (Hsuan-tsang) was a

Chinese traveller who came during the

reign of Harshavardhana

In the decade that began in 630 AD

Xuan Zang came to India through

Kashmir after visiting Central Asia Iran

and Afghanistan

He travelled from north to east and lived

in Bihar for a couple of years

At Nalanda University Xuan Zang

interacted with students and scholars

mastered local languages and

discovered Buddhist stupas

Q26) Answer c

Explanation

Pradakshina patha is a circular path

laid around a stupa in Buddhist

architecture While the rest are a part of

temple architecture

Q27) Answer d

Explanation

All the above-mentioned temples have

an elaborate use of bricks (baked

bricks) along with stone

Q28) Ans (c)

Muhammad Quli Qutab was the Sultan

of Golconda He was a contemporary of

Akbar was very fond of literature and

architecture

The Sultan was a great poet and he

wrote in Dakhini Urdu Persian and

Telgu and has left an extensive diwan or

collection

Recently the Archaeological Survey of

India (ASI) will be using Ground

Penetrating Radar (GPR) to map the

contours of the area around the Bagh-e-

Naya Qila excavated garden inside the

Golconda Fort in Telangana

Q29) Answer a

Explanation

Silappadikaram is a famous Tamil epic

which was written by Ilango around

1800 years ago It is a story of a

merchant named Kovalan who fell in

love with a courtesan named Madhavi

Manimekalai tells the story of the

daughter of Kovalan and Madhavi

Q30) Answer (a)

Explanation

Charaka is the author of Charaka

Samhita which is an important work of

Ayurveda and medicines

Brahmaguptarsquos fame rests mostly on his

Brahma-sphuta-siddhanta which was

an astronomical work It was translated

into Arabic in Baghdad and had a major

impact on Islamic mathematics and

astronomy

Late in his life Brahmagupta wrote

Khandakhadyaka which was an

RAUSIAS-FC19E1003 44

astronomical handbook that employed

Aryabhatarsquos system of starting each day

at midnight

Q31) Answer (c)

Explanation

Amir Khusrau was a famous sufi

musician poet and scholar In 1318 he

noted that there was different language

in every region of this land (Hindustan)

Lahori Kashmiri Dvarsamudri (in

Southern Karnataka) Telangana (in

Andhra Pradesh) Gujari (in Gujarat)

Marsquobari (in Tamil Nadu) Awadhi (in

eastern Uttar Pradesh) and Hindawai (in

the area around in Delhi) etc He went

to explain that Sanskrit did not belong

to any region and that only brahmans

knew it

Q32) Answer c

Explanation

Hiranyagarbha refers to the golden

womb When this ritual was performed

with the help of Brahmanas it was

thought to lead to the rebirth of the

sacrificer as a Khastriya

Q33) Answer d

Explanation

Kadamai refers to a tax on land

revenue

Gwalior Prashasti describes the exploits

of Nagabhata who was a Pratihara king

Q34) Answer b

Explanation

Rajatarangini is a Sanskrit text written

by Kalhana in the 12th century

It was historical chronicle of early India

It is justifiably considered to be the best

and most authentic work of its kind

It covers the entire span of history in

the Kashmir region from the earliest

times to the date of its composition

Q35) Answer c

Explanation

ldquoUrrdquo was the general assembly of the

village ldquoUrrdquo consisted of all the

taxpaying residents of an ordinary

village

Q36) Answer (a)

Explanation

Tarikh was a form of history writing in

the Delhi Sultanate The authors of

tawarikhs were learned men which

included secretaries administrators etc

Q37 Answer (a)

Explanation

Alauddin chose to pay his soldiers salaries in cash rather than iqtas The soldiers would buy their supplies from merchants in Delhi and it was thus feared that merchants would raise their prices To stop this Alauddin controlled the prices of goods in Delhi Prices were carefully surveyed by officers and merchants who did not sell at the prescribed rates were punished

Q38) Answer (d)

Explanation

Delhi first became the capital of a

kingdom under the Tomara Rajputs

who were defeated in the middle of the

twelfth century by the Chauhans (also

referred to as Chahamanas) of Ajmer

It was under the Tomaras and

Chauhans that Delhi became an

important commercial centre Many rich

Jaina merchants lived in the city and

constructed several temples Coins

minted here called dehliwal had a wide

circulation

Q39) Answer (c)

Explanation

Moth ki Masjid was built in the reign of

Sikandar Lodi by his minister

Begumpuri mosque built in the reign of

Muhammad Tughluq was the main

mosque of Jahanpanah the ldquoSanctuary

of the Worldrdquo and his new capital in

Delhi

Quwwat al ndash Islam mosque was

enlarged by Iltutmish and Alauddin

Khalji The minar was built by three

Sultansndash Qutbuddin Aybak Iltutmish

and Firuz Shah Tughluq

RAUSIAS-FC19E1003 45

Q40) Answer (c)

Explanation

Under the Mughals mansabdar was

referred to an individual who held a

mansab ie rank and he received his

salary as revenue assignments called

jagirs

Q41) Ans (b)

The Quit India Movement was a

spontaneous revolt of people against

British rule

The All India Congress Committee met

at Bombay on 8 August 1942 It passed

the famous resolution Quit India and

proposed the starting of a non-violent

mass struggle under Gandhis

leadership to achieve this aim But on

the very next day Gandhi and other

eminent leaders of the Congress were

arrested The Congress was once again

declared illegal

Q42) Ans (c)

The Simon Commission refers to a

group of seven MPs from the United

Kingdom constituted to suggest

constitutional reforms for British India

The Commission consisted of only

British members headed by one of the

senior British politicians Sir John

Simon

So the people of India agitated against

the arrival of Simon Commission

Q43) Ans (a)

He was widely known for his

unfavourable opinion of the economic

consequences of the British rule in

India

In his many writings and speeches and

especially in Poverty and Un-British

Rule in India Naoroji argued that India

was too highly taxed and that its wealth

was being drained away to England

He did not interpret the ancient Indian

texts and restored the self-confidence of

Indians And also he did not stress the

need for eradication of all the social

evils before anything else

Q44) Ans (c)

In August 1932 Prime Minister

MacDonald announced his Communal

Award Great Britainrsquos unilateral

attempt to resolve the various conflicts

among Indiarsquos many communal

interests

The award which was later

incorporated into the act of 1935

expanded the separate-electorate

formula reserved for Muslims to other

minorities including Sikhs Indian

Christians Anglo-Indians Europeans

distinct regional groups Gandhi

undertook a ldquofast unto deathrdquo against

that offer which he viewed as a

nefarious British plot to divide the

Indian society

Q45) Ans (b)

In British India apart from existing

imports and exports there was also a

particular amount of money which

colonial India contributed towards

administration maintenance of the

army war expenses pensions to retired

officers and other expenses accrued by

Britain towards maintenance of her

colony These were known as Home

charges and were paid for almost

entirely by India

The Home charges was made of

following components-

- Interest payable on Indian debt

- Dividend to shareholders of East

India Company

- Funds used to support the India

Office in London

- Funds used to pay salaries and

pensions of British personnel

engaged in India

- Interest on the railways

- Civil and military charges

- Store purchases in England

Q46) Ans (b)

The Lahore session of the Indian

National Congress was held in 1929

under the Presidentship of Jawaharlal

Nehru

The Lahore session of the Indian

National Congress witnessed significant

RAUSIAS-FC19E1003 46

developments in the Indian national

movement

- First the election of Jawaharlal

Nehru to the post of Presidentship of

the Congress was a clear indication

of the growing strength of the

Leftists in the Congress

- Secondly it was in this session that

the Congress for the first time raised

the demand for complete

independence Such demand was

not raised from the Congress

platform earlier

Q47) Ans (b)

It did not provide for separate

electorates for any community or

weightage for minorities However it did

allow for the reservation of minority

seats in provinces having minorities of

at least ten per cent but this was to be

in strict proportion to the size of the

community

There was no provision for complete

Independence for India

Q48) Ans (c)

The religion of early Vedic Aryans was

primarily of worship of nature and

Yajnas

The early Aryan religion was kind of

nature worship Actually the forces

around them which they could not

control or understand were invested

with divinity and were personified as

male or female gods And they

performed some Yajnas also

Q49) Ans (b)

The roads and river-routes were not

immune from robbery It is notable that

Yuan Chwang (Hiuen Tsang) was

robbed of his belongings during

Harshvardanarsquos period

Q50) Ans (c)

Q51) Ans (b)

Purandara Dasa was a saint and great

devotee of Lord Krishna

There is much speculation about where

Purandara Dasa regarded as the

Pitamaha of Carnatic music was born

Recently an expert committee

constituted by the Kannada University

Hampi has come to the conclusion that

Kshemapura Shivamogga district

Karnataka is the birth place of

Purandara Dasa

Q52) Ans (c)

Sri Tyagaraja Sri Shyama Shastry and Sri Muthuswami Dikshitar are considered the trinity of Carnatic music and with them came the golden age in Carnatic music in the 18th-19th

century

Q53) Ans d)

Recently a rare sarcophagus (stone

coffin) which is 2000 years old from the

Iron AgendashMegalithic era was discovered

from a rock-cut cave at Viyur village of

Kollam near Koyilandy in Kozhikode

district Kerala

The coffin containing bone fragments

was found during an excavation ldquoSo

far such a rare finding has been

discovered only from two sites

in Kerala Both these sarcophagi were

recovered from Megalithic sites at

Chevayur and Atholi also in Kozhikode

district

Q54) Ans a)

The megalithic culture in South India was a full-fledged Iron Age culture

Q55) Ans d)

The Cholas Pandyas and Keralaputras

(Cheras) mentioned in Ashokan

inscriptions were probably in the late

megalithic phase of material culture

Q56) Ans d)

Q57) Ans (b)

Raj Kumar Shukla followed Gandhiji all

over the country to persuade him to

come to Champaran to investigate the

problem associated with tinkathia

system

RAUSIAS-FC19E1003 47

Brij Kishore Rajendra Prasad Mahadev

Desai and Narhari Parikh accompanied

Gandhi ji during the Champaran

Satyagraha

Q58) Ans (b)

The Satvahanas started the practice of granting tax-free villages to brahmanas and Buddhist monks

Q59) Ans c)

The objectives of the Programme are

listed as under

- Developing basic tourism

infrastructure

- Promoting cultural and heritage

value of the country to generate

livelihoods in the identified regions

- Enhancing the tourist attractiveness

in a sustainable manner by

developing world-class

infrastructure at the heritage

monument sites

- Creating employment through active

involvement of local communities

- Harnessing tourism potential for its

effects on employment generation

and economic development

- Developing sustainable tourism

infrastructure and ensuring proper

Operations and maintenance

therein

Q60) Ans (b)

The Tribal Cooperative Marketing

Development Federation of India

(TRIFED) came into existence in 1987

It is a national-level apex organization

functioning under the administrative

control of Ministry of Tribal Affairs

Govt of India

TRIFED has its registered and Head

Office located in New Delhi

Q61) Ans (c)

Premchandrsquos novels include

Premashram Rangabhumi Ghaban

Karmabhumi and Godan

Gora is a novel written by Rabindranath

Tagore

138th birth anniversary of Munshi

Premchand was celebrated across the

country

Q62) Ans (b)

Giddha is a traditional pastoral dance

performed by the women of the Punjab

India and Pakistan at festival times

and at the sowing and reaping of the

harvest

By this dance the Punjabi women

reveal their joy expel their suppressed

feelings in a male dominated society

through the performance of Giddha

Since this dance has nothing to do with

men only women can participate in it

During the Teej celebrations Giddha

dance is celebrated in Punjab every

year Teej is a generic name for a

number of festivals that are celebrated

by women in some parts of India

Q63) Ans (a)

Dara Shukoh wrote the remarkable

work called ldquoMajma-ul-Bahrainrdquo or the

ldquoThe confluence of two seasrdquo

The Vice President of India Shri M

Venkaiah Naidu has said that Prince

Dara Shukohrsquos writings can come as a

refreshing source for infusing peace and

harmony He was addressing the

gathering after visiting the exhibition

that showcases the forgotten Prince of

yesteryears Dara Shukoh organized by

Mr Francois Gautier at Indira Gandhi

National Centre for the Arts in New

Delhi

Q64) Ans (c)

The statue Gommateshwara is

dedicated to the Jain God Bahubali

It is a monolithic statue

President Ram Nath Kovind

inaugurated the grand anointing

ceremony mdash Mahamastakabhisheka mdash

held once in 12 years at

Shravanabelagola (Karnataka)

Q65) Ans (c)

Prachi Valley had come up around the

Prachi river Prachi Valley gradually

disappeared

RAUSIAS-FC19E1003 48

The Prachi river originates from

Bhubaneswar

It is a tributary of the Mahanadi and

flows through the districts of Puri

Khurda Cuttack and Jagatsinghpur

and the entire region of the river is

termed as the Prachi Valley

It falls into the Bay of Bengal

Archaeological evidence shows that the

Prachi Valley Civilisation predates both

Harappa and Mohenjo-Daro

The Prachi river originates from

Bhubaneswar

Q66) Ans (d)

These monuments are located in

Chhatarpur district Madhya Pradesh

within Vindhya mountain range

Q67) Ans (a)

The book lsquoThoughts on Pakistanrsquo was

written by Dr BR Ambedkar

On the occasion of the birth anniversary

of Dr BR Ambedkar the president of

India pays homage to this icon of India

In 1924 he founded the Depressed

Classes Institute (Bahishkrit Hitkarini

Sabha) and in 1927 the Samaj Samata

Sangh

Another area of attention for Ambedkar

was education For its spread among

the low classes he set up a network of

colleges by the name of Peoples

Education Society and founded hostels

Q68) Ans(b)

Mehrgarh is a famous Neolithic

settlement in the Indian subcontinent

which is situated in Baluchistan

province Pakistan

A pre-historic rock art site is discovered

in the vast expanse of limestone blocks

on the eastern banks of Naguleru river

near Dachepalli (Andhra Pradesh) It

has thrown light on the Neolithic

civilisation that flourished in Guntur

(Andhra Pradesh) during 1500-2000

BC

Q69) Ans (c)

The 12th and the 13th centuries saw

the emergence of the Kakatiyas They

were at first the feudatories of the

Western Chalukyas of Kalyana Initially

they ruled over a small territory near

Warangal (Telangana)

They introduced Nayakships which was

later adopted and developed by the

Rayas of Vijayanagara

Q70) Ans (a)

The fast had effect of putting pressure

on mill owners who finally agreed to

give the workers a 35 per cent increase

in wages

Google celebrated with a doodle the

132nd birth anniversary of Anasuya

Sarabhai who played a pioneering role

in Indiarsquos labour movement

Q71) Ans (d)

The UNESCOrsquos list of the representative

list of the intangible cultural heritage of

humanity from India are

- Koodiyattam Sanskrit Theatre of

Kerala

- Mudiyettu ritual theatre and dance

drama of Kerala

- Tradition of Vedic Chanting

- Kalbelia folk songs and dances of

Rajasthan

- Ramlila Traditional Performance of

the Ramayana

- Sankirtana ritual singing

drumming and dancing of Manipur

- Ramman religious festival and

ritual theatre of the Garhwal

Himalayas India

- Traditional brass and copper craft of

utensil making among the Thatheras

of Jandiala Guru Punjab India

- Chhau dance classical Indian dance

originated in the eastern Indian

states

- Buddhist chanting of Ladakh

recitation of sacred Buddhist texts

in the trans-Himalayan Ladakh

region Jammu and Kashmir India

- Yoga

- Nouroz

- Kumbh Mela

RAUSIAS-FC19E1003 49

Q72) Ans(b)

The President of India Shri Ram Nath Kovind inaugurated the Hornbill Festival and State Formation Day celebrations of Nagaland in Kisama

The festival is named after the Indian hornbill the large and colourful forest bird which is displayed in the folklore of most of the states tribes

The major recognized tribes of Nagaland are Angami Ao Chakhesang Chang

Kuki Rengma and Zeling etc

Onge Jarawa and Sentinelese are the

tribes of Andman amp Nicobar Islands

Q73) Ans (c)

The Rashtrakutas rule in the Deccan lasted for almost two hundred years till the end of the tenth century The Rashtrakutas rulers were tolerant in their religious views and patronized not only Shaivism and Vaishnavism but

Jainism as well

The famous rock-cut temple of Shiva at Ellora was built by one of the Rashtrakutas kings Krishna I in the ninth century His successor Amoghavarsha was a Jain but he also

patronized other faiths

The Rashtrakutas allowed Muslims traders to settle and permitted Islam to

be preached in their dominions

Recently increasing defacement at the prehistoric rock paintings of Pandavulagutta Telangana has created a cause for grave concern It can spoil

the prehistoric rock

Pandavulagutta is home to

- Painted rock shelters dating to

10000 BC-8000 BC

- An 8th century inscription of the

Rashtrakuta period and

- Painted frescoes from the 12th century Kakatiya empire

Q74) Ans (b)

In 1828 Raja Ram Mohan Roy founded a new religious society the Brahma Sabha later known as the Brahmo

Samaj

Debendranath Tagore headed the Tattvabodhini Sabha which was

engaged in search of spiritual truth

Its purpose was to purify Hinduism and to preach monotheism or belief in one God

The new society was to be based on the twin pillars of reason and the Vedas and

Upanishads

Recently Sadharan Brahmo Samaj (SBS) has entered into a legal battle with the West Bengal government due

to some legal issue

Q75) Ans (c)

The Chishti order was established in India by Khwaja Moinuddin Chishti who came to India around 1192 The Chishtirsquos are considered to be the most influential of the groups of Sufis who migrated to India in the late twelfth century They adapted successfully to the local environment and adopted several features of Indian devotional

traditions

The historical dargah of Sufi mystic Khwaja Moinuddin Chishti in Ajmer is all set to get a facelift This 13 th century dargah has been included among the Swachh Iconic Places a clean-up initiative focused on iconic

heritage spiritual and cultural places

Page 34: GENERAL STUDIES (PAPER I) · Test is part of Rau’s IAS Test series for Preliminary Exam 2019 FOUNDATION + CURRENT AFFAIRS GENERAL STUDIES (PAPER –I) FOUNDATION TEST –III TOPIC:

FC19H1003 34

- अविकारीगण धयानपििक मलय ो का सिकषण करत

थ तथा ज वयापारी वनिािररत मलय पर माल नही ो

बचत थ उनक दसणडत वकया जाता था

Q38) उततर (d)

सपषटीकरण

- वदलली सििपरथम त मर राजपत ो क अिीन उनक

सामराजय की राजिानी बनी थी

- 12िी ो शताबदी क मधय म अजमर क चौहान ो

(वजनह चाहमान ो क नाम स भी जाना जाता ह) न

त मर राजपत ो क परावजत वकया था

- त मर ो और चौहान ो क अिीन वदलली एक

महतवपणि िावणसजयक क दर बन गया था

- कई जन वयापारी यहाा रहन लग थ और उनह ोन

कई मोवदर भी बनिाए

- यहाा पर मवदरत वसक वजनह ldquoदहलीिालrdquo क नाम

स जाना जाता था वयापक रप स परचलन म थ

Q39) उततर (c)

सपषटीकरण

- म ठ की मसिद का वनमािण वसको दर ल दी क

राजयकाल म उसक मिी क दवारा करिाया गया

था

- बगमपरी मसिद का वनमािण महममद तगलक क

शासनकाल म हआ था

- यह मसिद विशव का पणयथथान (The

Sanctuary of the World) और वदलली म महममद

तगलक की नई राजिानी जहाोपनाह की मखय

मसिद थी

- कववत- अल - इसलाम मसिद का विसतार

इलतसिश और अलाउददीन सखलजी न वकया था

- मीनार का वनमािण तीन सलतान ो कतबददीन ऐबक

इलतसिश और वफर ज शाह तगलक क दवारा

करिाया गया था

Q40) उततर (c)

सपषटीकरण

- मगल ो क अिीन मनसबदार शबद उस वयसकत क

वलए सोदवभित वकया जाता था वजसक पास मनसब

(अथाित पद) ह ता था

- उस अपना ितन राजसव कायो वजनह जागीर कहत

थ क रप म परापत ह ता था

Q41) उततर (b)

सपषटीकरण

- ldquoभारत छ ड आोद लनrdquo वबरविश शासन क

सखलाफ ल ग ो का एक सवाभाविक विदर ह था

- असखल भारतीय काोगरस सवमवत न 8 अगसत 1942

क बमबई म एक बठक का आय जन वकया था

- इस बठक म परवसद सोकलप ldquoभारत छ ड rdquo क

पाररत वकया गया और इस उददशय क परापत करन

क वलए गाोिी क नततव म एक अवहोसक जन सोघिि

आोद लन की शरआत का परसताि वदया गया

- लवकन अगल ही वदन गाोिी और काोगरस क अनय

परमख नताओो क वगरफतार कर वलया गया

- काोगरस क एक बार वफर अिि घ वित वकया गया

था

Q42) उततर (c)

सपषटीकरण

- साइमन कमीशन यनाइविड वको गडम क सात

साोसद ो का एक समह था

- इस वबरविश भारत क वलए सोििावनक सिार ो का

सझाि दन क वलए गवठत वकया गया था

- इस आय ग म िररषठ वबरविश राजनता सर जॉन

साइमन क नततव म किल वबरविश सदसय ही

शावमल थ

- इसवलए भारत क ल ग ो न साइमन कमीशन क

आगमन क विरद आोद लन वकया था

Q43) उततर (a)

सपषटीकरण

bull दादा भाई नौर जी भारत म वबरविश शासन क

आवथिक पररणाम ो क बार म अपनी विर िी

(परवतकल) राय क वलए जान जात थ

FC19H1003 35

bull अपन कई लख ो और भािण ो म विशि रप स

ldquoपाििी एो ड अन-वबरविश रल इन इसणडया

(Poverty and Un-British Rule in India) म

नौर जी न यह तकि वदया वक भारत पर अतयविक

कर लगाया गया था और इसकी सोपवतत इोगलड की

ओर परिावहत की जा रही थी

bull उनह ोन पराचीन भारतीय गरोथ ो की वयाखया करन

का और भारतीय ो क आिविशवास क बहाल

करन पर कायि नही ो वकया था

उनह ोन वकसी और बात स पहल सभी सामावजक

बराइय ो क उनमलन की आिशयकता पर भी बल

नही ो वदया था

Q44) उततर (c)

सपषटीकरण

bull अगसत 1932 म वबरविश परिानमोतरी मकड नालड न

अपन साोपरदावयक परसकार (The Communal

Award) की घ िणा की थी

bull यह भारत क कई साोपरदावयक वहत ो क बीच विवभनन

सोघिो क हल करन क वलए वबरिन का एकतरफा

परयास था

bull यह परसकार (Award) बाद म 1935 क

अविवनयम (The Act of 1935) म शावमल वकया

गया था

bull इस साोपरदावयक परसकार न मससलम ो क वलए

आरवकषत एक अलग वनिािचक मणडल फॉमिल का

विसतार अनय अलपसोखयक ो क वलए वकया था

वजसम वसख ो भारतीय ईसाइय ो आोगल-भारतीय

समदाय यर पीय समदाय तथा विवशषट कषतरीय

समह ो क शावमल वकया गया था

bull गाोिी न इस परसताि क भारतीय समाज क

विभावजत करन क वलए एक घवणत वबरविश

सावजश क रप म दखा और उसक सखलाफ

आमरण अनशन वकया

Q45) उततर (b)

सपषटीकरण

मौजदा आयात और वनयाित क अवतररक़त

औपवनिवशक भारत क वनमनवलसखत खचो क

वलए एक विशिवनवशचत िन रावश भी दनी पडती

थी

(i) परशासन क वयय

(ii) सना क रख-रखाि क वयय

(iii) यद क वयय

(iv) सिावनितत अविकाररय ो की पशन तथा

(v) वबरिन दवारा अपनी उपवनिश बसती

(कॉल नी) क रख-रखाि क वयय

इनह गह शलक (Home Charges) क रप म

जाना जाता था और लगभग परी तरह स भारत क

दवारा इनका भगतान वकया जाता था

bull गह शलक म वनमनवलसखत घिक शावमल थ

(i) भारतीय ऋण पर दय बयाज

(ii) ईसट इोवडया को पनी क शयरिारक ो क

लाभाोश

(iii) लोदन म भारत कायािलय चलान क वलए िन

(iv) भारत म वनयकत वबरविश कवमिय ो क ितन

और पशन का भगतान करन क वलए िन

(v) रलि पर बयाज

(vi) नागररक और सनय शलक

(vii) इोगलड म सट र (सामगरी) की खरीद

Q46) उततर (b)

सपषटीकरण

bull भारतीय राषटर ीय काोगरस का लाहौर सतर 1929 म

जिाहरलाल नहर की अधयकषता म आय वजत

वकया गया था

bull इस सतर म भारतीय राषटर ीय आोद लन स समबसित

कई महतवपणि पररणाम सामन आय थ

(i) सििपरथम इस सतर म काोगरस क अधयकष पद

पर जिाहरलाल नहर क चना गया था ज

काोगरस म िामपोवथय ो की बिती हई ताकत

का सपषट सोकत था

(ii) दसरा इस सतर म पहली बार काोगरस न पणि

सवतोतरता की माोग क उठाया था

इस परकार की माोग काोगरस मोच स पहल कभी भी

नही ो उठाई गई थी

Q47) उततर (b)

सपषटीकरण

FC19H1003 36

bull इस ररप िि न वकसी भी समदाय क वलए पथक

वनिािचक मोडल अथिा अलपसोखयक ो क वलए

भाराोश की वसफाररश नही ो की थी

bull तथावप इस ररप िि न उन पराोत ो म अलपसोखयक

सीि ो क आरकषण की अनमवत दी थी जहाा पर कम

स कम दस परवतशत अलपसोखयक ह

bull लवकन यह समदाय क आकार क अनपात म ह ना

चावहए था

bull इस ररप िि म भारत क वलए पणि सवतोतरता क

वलए क ई पराििान नही ो था

Q48) उततर (c)

सपषटीकरण

bull आरो वभक िवदक आयो का िमि मखय रप स

परकवत की पजा और यजञ था

bull परारो वभक आयि िमि परकवत की पजा क समान था

bull िासति म उनक चार ो ओर की शसकतयाा वजनह न

त ि वनयोवतरत कर सकत थ और न ही समझ पाए

थ उनह वदवयता क साथ वनिवशत वकया गया तथा

उनह मादा या नर दिीदिताओो क रप म

परतीकतव वकया गया था

bull उनह ोन कछ यजञ ो का भी वनषपादन वकया था

Q49) उततर (b)

सपषटीकरण

bull सडक और नदी-मागि (जल-मागि) डकती स

सरवकषत नही ो थ

bull उललखनीय ह वक हिििििन क शासनकाल क

दौरान यआन चिाोग (हयएन साोग) का सारा

सामान लि वलया गया था

Q50) उततर (c)

सपषटीकरण

परशन म वदए गए द न ो कथन सही ह

Q51) उततर (b)

सपषटीकरण

bull परोदर दास एक सोत और भगिान कषण क एक

महान भकत थ

bull परोदर दास क कनाििक सोगीत क वपतामह क

रप म जाना जाता ह

bull यदयवप उनक जनम-थथान क बार म काफी

अिकल लगाई जाती रही ह

bull तथावप अब कननड विशवविदयालय हमपी क दवारा

गवठत एक विशिजञ सवमवत इस वनषकिि पर पहोची

ह वक उनका जनम थथान सोभितया कनाििक का

एक छ िा-सा गााि कषमपरा (वशिम गगा वजला)

था

Q52) उततर (c)

सपषटीकरण

bull शरी तयागराज शरी शयाम शासतरी और शरी मथसवामी

दीवकषतर क कनाििक सोगीत की वतरमवति माना

जाता ह

bull उनक कारण ही 18िी ो-19िी ो शताबदी म कनाििक

सोगीत का सववणिम यग आया था

Q53) उततर (d)

सपषटीकरण

bull अभी हाल ही म लौह यगीन-महापािावणक काल

का 2000 ििि पराना एक दलिभ सारक फगस

(Sarcophagus) (पतथर का ताबत) क ललम क

वियर गाोि (क वयलडी क पास वजला क वझक ड

करल राजय) की एक रॉक-कि गफा स ख जा गया

bull यह ताबत वजसम हविय ो क िकड थ खदाई क

दौरान वमला

bull अभी तक इस परकार की दलिभ ख ज करल क

मातर द ही थथान ो स हई ह

bull य द न ो सारक फगी (Sarcophagi) (पतथर क

ताबत) चियर और अथ ली (वजला क वझक ड) क

महापािाण थथल ो स वमल ह

Q54) उततर (a)

सपषटीकरण

FC19H1003 37

दवकषण भारत म महापािाण सोसकवत एक पणि

विकवसत लौह यगीन सोसकवत थी

Q55) उततर (d)

सपषटीकरण

bull च ल पाणडय और करलपतर (चर) इन तीन ो का

उललख अश क क अवभलख ो म वकया गया ह

bull सोभितः य भौवतक सोसकवत क उततर

महापािावणक चरण म थ

Q56) उततर (d)

सपषटीकरण

bull भीमा-क रगाोि की लडाई ततीय आोगल-मराठा

यद का वहससा थी

Q57) उततर (b)

सपषटीकरण

bull राजकमार शकल न गाोिीजी क चोपारण आन तथा

वतनकवथया परणाली स जडी समसया की जाोच क

वलए रारी करन क वलए दश भर म उनका

अनसरण वकया था

bull बज वकश र राजदर परसाद महादि दसाई और

नरहरी पाररख चोपारण सतयागरह क दौरान गाोिी

जी क सहय गी थ

Q58) उततर (b)

सपषटीकरण

bull बराहमण ो और बौद मठिाररय ो क कर-मकत गााि

अनदान म दन की परथा सतिाहन ो न आरमभ की

थी

Q59) उततर (c)

सपषटीकरण

इस कायिकरम क उददशय वनमनानसार ह

(i) बवनयादी पयििन आिाररक सोरचना का विकास

करना

(ii) चयवनत (पहचान वकय गए) कषतर ो म आजीविका क

सजन क वलए दश क साोसकवतक और विरासत

मलय ो क बिािा दना

(iii) विरासत समारक थथल ो पर विशव सतरीय आिाररक

सोरचना विकवसत करक एक सतत तरीक स

पयििक आकििण म िसद करना

(iv) थथानीय समदाय ो की सवकरय भागीदारी क माधयम

स र रगार ो का सजन करना

(v) र रगार उतपादन और आवथिक विकास क वलए

पयििन कषमता का उन पर परभाि का उपय ग

करना तथा

(vi) िारणीय पयििन आिाररक सोरचना का विकास

करना और उसका उवचत सोचालन तथा

रखरखाि सवनवशचत करना

Q60) उततर (b)

सपषटीकरण

bull यह वनकाय ििि 1987 म अससततव म आया था

bull यह एक राषटर ीय सतर का शीिि सोगठन ह ज भारत

सरकार क जनजातीय मामल ो क मोतरालय क

परशासवनक वनयोतरण क अिीन काम कर रहा ह

bull इसका पोजीकत और परिान कायािलय नई वदलली

म सथथत ह

Q61) उततर (c)

सपषटीकरण

bull परमचोद क उपनयास ो म परमाशरम रोगभवम गबन

कमिभवम और ग दान शावमल ह

bull ग रा रिी ोदरनाथ िग र क दवारा रवचत उपनयास ह

bull अभी हाल ही म मोशी परमचोद की 138िी ो जयोती दश

भर म मनाई गई थी

Q62) उततर (b)

सपषटीकरण

bull ldquoवगदाrdquo पोजाब (भारत) एिो पावकसतान की

मवहलाओो क दवारा तयौहार क समय और फसल

की बिाई तथा किाई क अिसर पर वकया जान

िाला एक पारोपररक दहाती नतय ह

FC19H1003 38

bull इस नतय क माधयम स पोजाबी मवहलाऐो अपनी

परसननता परकि करती ह तथा वगदा क परदशिन क

माधयम स परि िचिसव िाल समाज म मवहलाओो

की दबी हई भािनाओो क परकि करती ह

bull चोवक इस नतय का परि ो क साथ क ई सोबोि नही ो

ह अतः किल मवहलाऐो ही इसम भाग ल सकती

bull हर साल तीज समार ह क दौरान पोजाब म वगदा

नतय वकया जाता ह

तीज भारत क कछ भाग ो म मवहलाओो क दवारा

मनाया जान िाल कई तयौहार ो क वलए एक

वयापक नाम ह

Q63) उततर (a)

सपषटीकरण

- मजम-उल-बहरीन या द समदर ो का सोगम

नामक उललखनीय रचना दारा वशक ह क दवारा

वलखी थी

- भारत क उपराषटर पवत शरी एम िकया नायड न कहा

ह वक राजकमार दारा वशक ह की रचनाएा शाोवत

और सदभाि क बिािा दन क वलए एक तारा सर त

क रप म सामन आ सकती ो ह

- उपराषटर पवत गत ििो क भला वदए गए राजकमार

दारा वशक ह क परदवशित परचवलत करन हत

आय वजत एक परदशिनी का दौरा करन क बाद एक

सभा क सोब वित कर रह थ

- इस परदशिनी का आय जन फर क इस गौवियर

(Francois Gautier) क दवारा lsquoइोवदरा गाोिी नशनल

सिर फॉर द आििसrsquo (The Indira Gandhi

National Centre for the Arts) नई वदलली म

वकया गया था

Q64) उततर (c)

सपषटीकरण

- ग मतशवर परवतमा जन भगिान बाहबली क

समवपित ह

- यह एक एक-चटटानी पतथर की मवति ह

- राषटर पवत राम नाथ क विोद न शरिणबलग ला

(कनाििक) म आय वजत वकय जान िाल भवय

अवभिक समार ह महामसतकावभिक का

उदघािन वकया था

- यह समार ह 12 ििो म एक बार ह ता ह

Q65) उततर (c)

सपषटीकरण

bull पराची घािी पराची नदी क चार ो ओर फली हई थी

bull पराची घािी िीर-िीर विलपत ह गई थी

bull पराची नदी भिनशवर स वनकलती ह

bull यह महानदी की एक सहायक नदी ह और यह

परी खदाि किक तथा जगतवसोहपर वजल ो स

ह कर बहती ह

bull इस नदी क पर कषतर क पराची घािी कहा जाता ह

bull यह नदी बोगाल की खाडी म वगरती ह

परातासतवक साकषय स पता चलता ह वक पराची घािी

सभयता हडपपा और म हनज दाड द न ो की

पिििती ह

Q66) उततर (d)

सपषटीकरण

य समारक छतरपर वजल (मधय परदश) म विोधयाचल

पिित शरोखला म सथथत ह

Q67) उततर (a)

सपषटीकरण

bull थॉिस ऑन पावकसतान नामक पसतक डॉ बी

आर अमबडकर न वलखी थी

bull डॉ बी आर अमबडकर की जयोती क अिसर पर

भारत क राषटर पवत न भारत की इस महान हसती

क शरदाोजवल अवपित की थी

bull डॉ बी आर अमबडकर न 1924 म वडपरथड

कलावसर इोसटीटयि (दवलत िगि सोथथान -

बवहषकत वहतकाररणी सभा) और 1927 म समाज

समता सोघ की थथापना की थी

bull अमबडकर का धयान वशकषा कषतर की ओर भी था

bull उनह ोन वशकषा क वनमन िगो म फलान क वलए

पीपलस एजकशन स साइिी (The Peoples

Education Society) क नाम स महाविदयालय ो क

नििकि और छातरािास ो की थथापना की थी

FC19H1003 39

Q68) उततर (b)

सपषटीकरण

bull महरगि भारतीय उपमहादवीप म एक परवसद

निपािाण बसती ह ज बलवचसतान पराोत

पावकसतान म सथथत ह

bull दचपलली (आोधर परदश) क पास नागलर नदी क

पिी ति ो पर चना पतथर क बलॉक क विशाल

विसतार म एक पिि-ऐवतहावसक रॉक आिि थथल की

ख ज की गई ह

bull इसन 1500-2000 ईसा पिि क दौरान गोिर (आोधर

परदश) म विकवसत निपािाण सभयता पर परकाश

डाला ह

Q69) उततर (c)

सपषटीकरण

bull 12िी ो सदी और 13िी ो सदी म काकाविय िोश का

उदय हआ था

bull ि पहल कलयाण क पवशचमी चालकय ो क सामोत थ

bull परारोभ म उनह ोन िारोगल (तलोगाना) क पास एक

छ ि स कषतर पर शासन वकया था

bull उनह ोन ldquoनायक वयिथथाrdquo की शरआत की थी

वजस बाद म विजयनगर क राय शासक ो न

अपनाया और विकवसत वकया था

Q70) उततर (a)

सपषटीकरण

bull गाोिीजी क अनशन स वमल मावलक ो पर दबाि

पडा था ज अोततः शरवमक ो क ितन म 35 परवतशत

की िसद करन क वलए सहमत हए थ

bull गगल (Google) न अनसया साराभाई वजनह ोन

भारत क शरवमक आोद लन म एक अगरणी भवमका

वनभाई थी की 132िी ो जयोती डडल (Doodle) का

वनमािण करक मनाई

Q71) उततर (d)

सपषटीकरण

भारत स यनसक की मानिता की अमति साोसकवतक

विरासत की परवतवनवि सची म वनमनवलसखत शावमल ह

bull कवडयटटम करल का सोसकत रोगमोच

bull मवडयिि करल का अनषठान रोगमोच और नतय

नाविका

bull िवदक मि जाप की परोपरा

bull राजथथान क कालबवलया ल क गीत और नतय

bull रामलीला रामायण का पारोपररक परदशिन

bull सोकीतिन मवणपर का अनषठान गायन ढ ल िादन

और नतय

bull रममन भारत क गििाल वहमालय का िावमिक

तयौहार और अनषठान रोगमोच

bull जाोदीयाला गर पोजाब क ठठर ो की पीतल और

ताोब क वशलप स वनवमित बतिन ो की पारोपररक कला

bull छाऊ नतय पिी भारतीय राजय ो म जनमी शासतरीय

भारतीय नतय कला

bull लददाख का बौद मि जाप िर ाोस-वहमालयी लददाख

कषतर तथा जमम-कशमीर म पवितर बौद गरोथ ो का पाठ

bull य ग

bull नौर र

bull को भ मला

Q72) उततर (b)

सपषटीकरण

bull भारत क राषटर पवत शरी राम नाथ क विोद न

वकसामा नागालड म हॉनिवबल मह रति और

राजय गठन वदिस समार ह का उदघािन वकया

था

bull हॉनिवबल मह रति का नाम भारतीय हॉनिवबल क

नाम पर पडा ह ज एक विशाल और रोगीन जोगली

पकषी ह

bull यह पकषी नागालड राजय की अविकतर जनजावतय ो

की ल ककथाओो म उसललसखत ह

bull नागालड की परमख मानयता परापत जनजावतयाा ह

अोगामी आओ चखसोग चाोग ककी रगमा और

रवलोग आवद

bull ओोग जारिा और ससिनलीस अोडमान-वनक बार

दवीप समह की जनजावतयाा ह

FC19H1003 40

Q73) उततर (c)

सपषटीकरण

bull दकन म राषटर कि शासन दसिी ो सदी क अोत तक

लगभग 200 ििो तक रहा था

bull राषटर कि शासक अपन िावमिक विचार ो म सवहषण

bull उनह ोन न किल शि िमि और िषणि िमि बसलक

जन िमि क भी सोरकषण वदया था

bull एल रा म वशि क परवसद रॉक कि मोवदर का

वनमािण नौिी ो सदी म राषटर कि राजा कषण परथम न

करिाया था

bull उसका उततराविकारी अम घििि जन था लवकन

उसन अनय िमो क भी सोरकषण परदान वकया था

bull राषटर कि ो न मसलमान वयापाररय ो क बसन की

अनमवत दी थी

bull उनह न अपन अविराजय ो म इसलाम क उपदश दन

की भी अनमवत दी थी

bull अभी हाल ही म पाोडिलागटटा (तलोगाना) क

परागवतहावसक चटटान वचतर ो क कषरण की बिती हई

घिनाएा एक गोभीर वचोता का वििय ह

bull यह परागवतहावसक चटटान क नकसान पहाचा

सकता ह

bull पाोडिलागटटा वनमनवलसखत क वलए जाना जाता ह

- 10000 ईसा पिि स 8000 ईसा पिि क वचवतरत

चटटानी आशरय ो क वलए

- राषटर कि काल क एक 8 िी ो सदी क

वशलालख क वलए और

- 12िी ो सदी क काकविय सामराजय क वभवतत

वचतर ो क वलए

Q74) उततर (b)

सपषटीकरण

bull 1828 म राजा राम म हन रॉय न एक नय िावमिक

समाज बरहम सभा की थथापना की थी वजस बाद

म बरहम समाज क नाम स जाना गया था

bull दिदरनाथ िग र न ततवब विनी सभा की अधयकषता

की थी ज आधयासिक सतय की ख ज म सोलि

थी

bull इसका उददशय वहोद िमि क शद करन का और

एकशवरिाद (एक ईशवर म आथथा) का परचार करना

था

bull नय समाज की थथापना क आिार थ कारण

(तकि ) क द सतमभ तथा िद और उपवनिद

bull अभी हाल ही म सािारण बरहम समाज का कछ

काननी मदद ो क लकर पवशचम बोगाल सरकार क

साथ काननी वििाद चल रहा ह

Q75) उततर (c)

सपषटीकरण

bull भारत म वचशती वसलवसल की थथापना खवाजा

म इनददीन वचशती क दवारा की गयी थी

bull ि 1192 ईसवी क आसपास भारत आय थ

bull वचशतीय ो क बारहिी ो शताबदी क उततरािि म भारत

म आन िाल सफीय ो क समह ो म सबस

परभािशाली माना जाता ह

bull उनह ोन थथानीय िातािरण क साथ सफलतापििक

अनकलन वकया और उनह ोन भारतीय भसकत

परोपराओो क कई पहलओो क अपनाया

bull अजमर म सफी अपरकि खवाजा म इनददीन वचशती

की ऐवतहावसक दरगाह क एक नया रप दन की

तयारी की जा रही ह

bull इस 13िी ो शताबदी की दरगाह क ldquoसवचछ

आइकॉवनक थथल ोrdquo (Swacch Iconic Places) म

शावमल वकया गया ह ज परवतवषठत विरासत

आधयासिक और साोसकवतक थथान ो पर क वदरत

य जना ह

FC19H1003 41

ANSWERS amp EXPLANATION OF

NCERT History Class VI-X + Current Affairs

(FC19E1003)

Q1) Answer c

Explanation

Rigveda consists of more than a

thousand hymns dedicated to gods and

goddesses These hymns were

composed by sages and learnt by men

however a few were composed by

women like Apala Ghosa Lopamudra

Maitreyi and Gargi

Rigveda consists of many hymns in the

form of dialogues We get an example of

a dialogue between a sage named

Vishwamitra and two rivers (Beas and

Sutlej) that were worshipped as

goddesses This suggests that he

belonged to the Vedic period

Q2) Answer b

Explanation

Traces of ash have been found from

Kurnool Caves suggesting that people

were familiar with the use of fire

It is situated in Andhra Pradesh

Q3) Answer c

Explanation

Burzahom is a prehistoric site in

present day Kashmir where people built

pit houses which were dug into the

ground with steps leading into them

These may have provided shelter in cold

weather

Q4) Answer c

Explanation

Epigraphy is defined as the study of

inscriptions

Manuscriptology is the study of history

and literature through the use of hand

written documents

Palaeography refers to the study of

ancient writing systems and the

deciphering and dating of historical

manuscripts

Numismatics refers to the study of

coins

Q5) Answer a

Explanation

Charaka Samhita was written by

Charaka and is an important book on

Ayurveda and medicine

He was a practitioner of the traditional

system of Indian medicine known as

Ayurveda

Charaka is thought to have flourished

sometime between the 2nd century BCE

and the 2nd century CE

Q6) Answer b

Explanation

Bhaga refers to the tax on crops which

was fixed at 16th of the production

Kammakaras is the term used for the

landless agricultural labour class

Ashvamedha also known as horse

sacrifice is a ritual where a horse is let

loose to wander freely and it was

guarded by the rajarsquos men

Q7) Answer (d)

Explanation

In the Rigvedic period horses were

yoked to chariots that were used in

battles fought to capture land cattle

etc This suggests that the use of horse

chariots began much before the period

of Mahajanapadas

The battles were fought in the Rigvedic

period for cattlersquos lands water an even

to capture people Most men took part

in these wars however there was no

regular army but there were assemblies

where people met and discussed

matters of war Regular armies became

a feature in the Mjahajanapada period

including vast armies of foot soldiers

chariots and elephants

RAUSIAS-FC19E1003 42

Q8) Answer (a)

Explanation

Buddha belonged to the Sakya clan and

passed away at Kusinara

Buddha taught in Prakrit which was the

common language of people

Q9) Answer c

Explanation

There were six schools of philosophy in

ancient India These are known as

Vaishesika Nyaya Samkhya Yoga

Purva Mimansa and Vedanata or Uttara

Mimansa They were founded by sages

Kanada Gautama Kapila Patanjali

Jamini and Vyasa respectively

Q10) Answer b

Explanation

The teachings of Mahavira were

compiled at Valabhi in 6th century AD

Q11) Answer (c)

Explanation

Chanakya is traditionally identified as

Kautilya or Vishnugupta who authored

the ancient Indian political treatise the

Arthashastra

Q12) Answer d

The national emblem of India is an

adaptation of the Lion Capital atop the

Ashoka Pillar of Sarnath Uttar Pradesh

and is combined with the National

Motto Satyameva Jayate

The Rampurva Bull gets the name from

the site of its discovery Rampurva in

Bihar

It is noted for its delicately sculpted

model demonstrating superior

representation of soft flesh sensitive

nostrils alert ears and strong legs It is

a mixture of Indian and Persian

elements

Sankissa is situated in Uttar Pradesh

India

Q13) Ans(a)

Kunwar Singh was a notable leader during the Revolt of 1857 He belonged

to a royal house of Jagdispur Bihar

Q14) Answer b

Explanation

The term Vellalar was used for large

landowners

Q15) Answer c

Explanation

Arikamedu was a coastal settlement

where ships unloaded goods from

distant lands Finds here include a

massive brick warehouse pottery

including amphorae and Arretine ware

Roman lamps glassware and gems have

also been found at the site

Q16) Answer a

Explanation

Muvendar is a Tamil word mentioned in

Sangam poems meaning three chiefs

used for the heads of three ruling

families the Cholas Cheras and

Pandyas

Q17) Ans (c)

Several tribal or kin-based assemblies

such as the Sabha Vidatha and gana

are mentioned in the Rig-veda The

Sabha and the samiti mattered a great

deal in early Vedic times so much so

that the chiefs or the kings showed an

eagerness to win their support

Q18) Ans (a)

Jainism recognised the existence of the

gods but placed them lower than the

jina and did not condemn the varna

system as Buddhism did

Q19) Answer (d)

Explanation

Cholas and Pandyas had developed

powerful coastal cities The most

important city of Cholas was Puhar or

Kaveripattinam and Madurai was the

capital of Pandyas

Q20) Answer b

Explanation

Buddhacharita is the biography of

Buddha and was written by

RAUSIAS-FC19E1003 43

Ashvaghosha

Q21) Answer (a)

Explanation

Tamil poet Appar was a Shiva devotee

So he was a Nayanar saint

Q22) Answer d

Explanation

Samudragupta was a prominent Gupta

ruler whose coins depict him playing a

veena indicating his love for music We

get important historic information from

his Allahabad Prashasti which was

composed by his court poet Harisena

Q23) Answer (b)

Explanation

Vikrama Samvat was founded by

Chandragupta II in the 58 BC as a

mark of victory over the Shakas and

assumed the title of Vikramaditya

Banabhatta wrote Harshavardhanarsquos

biography the Harshacharita in

Sanskrit

Q24) Answer c

Explanation

Sandhi-vigrahika was the minister of

war and peace

Sarthavaha was the leader of the

merchant caravans

Q25) Answer a

Explanation

Xuan Zang (Hsuan-tsang) was a

Chinese traveller who came during the

reign of Harshavardhana

In the decade that began in 630 AD

Xuan Zang came to India through

Kashmir after visiting Central Asia Iran

and Afghanistan

He travelled from north to east and lived

in Bihar for a couple of years

At Nalanda University Xuan Zang

interacted with students and scholars

mastered local languages and

discovered Buddhist stupas

Q26) Answer c

Explanation

Pradakshina patha is a circular path

laid around a stupa in Buddhist

architecture While the rest are a part of

temple architecture

Q27) Answer d

Explanation

All the above-mentioned temples have

an elaborate use of bricks (baked

bricks) along with stone

Q28) Ans (c)

Muhammad Quli Qutab was the Sultan

of Golconda He was a contemporary of

Akbar was very fond of literature and

architecture

The Sultan was a great poet and he

wrote in Dakhini Urdu Persian and

Telgu and has left an extensive diwan or

collection

Recently the Archaeological Survey of

India (ASI) will be using Ground

Penetrating Radar (GPR) to map the

contours of the area around the Bagh-e-

Naya Qila excavated garden inside the

Golconda Fort in Telangana

Q29) Answer a

Explanation

Silappadikaram is a famous Tamil epic

which was written by Ilango around

1800 years ago It is a story of a

merchant named Kovalan who fell in

love with a courtesan named Madhavi

Manimekalai tells the story of the

daughter of Kovalan and Madhavi

Q30) Answer (a)

Explanation

Charaka is the author of Charaka

Samhita which is an important work of

Ayurveda and medicines

Brahmaguptarsquos fame rests mostly on his

Brahma-sphuta-siddhanta which was

an astronomical work It was translated

into Arabic in Baghdad and had a major

impact on Islamic mathematics and

astronomy

Late in his life Brahmagupta wrote

Khandakhadyaka which was an

RAUSIAS-FC19E1003 44

astronomical handbook that employed

Aryabhatarsquos system of starting each day

at midnight

Q31) Answer (c)

Explanation

Amir Khusrau was a famous sufi

musician poet and scholar In 1318 he

noted that there was different language

in every region of this land (Hindustan)

Lahori Kashmiri Dvarsamudri (in

Southern Karnataka) Telangana (in

Andhra Pradesh) Gujari (in Gujarat)

Marsquobari (in Tamil Nadu) Awadhi (in

eastern Uttar Pradesh) and Hindawai (in

the area around in Delhi) etc He went

to explain that Sanskrit did not belong

to any region and that only brahmans

knew it

Q32) Answer c

Explanation

Hiranyagarbha refers to the golden

womb When this ritual was performed

with the help of Brahmanas it was

thought to lead to the rebirth of the

sacrificer as a Khastriya

Q33) Answer d

Explanation

Kadamai refers to a tax on land

revenue

Gwalior Prashasti describes the exploits

of Nagabhata who was a Pratihara king

Q34) Answer b

Explanation

Rajatarangini is a Sanskrit text written

by Kalhana in the 12th century

It was historical chronicle of early India

It is justifiably considered to be the best

and most authentic work of its kind

It covers the entire span of history in

the Kashmir region from the earliest

times to the date of its composition

Q35) Answer c

Explanation

ldquoUrrdquo was the general assembly of the

village ldquoUrrdquo consisted of all the

taxpaying residents of an ordinary

village

Q36) Answer (a)

Explanation

Tarikh was a form of history writing in

the Delhi Sultanate The authors of

tawarikhs were learned men which

included secretaries administrators etc

Q37 Answer (a)

Explanation

Alauddin chose to pay his soldiers salaries in cash rather than iqtas The soldiers would buy their supplies from merchants in Delhi and it was thus feared that merchants would raise their prices To stop this Alauddin controlled the prices of goods in Delhi Prices were carefully surveyed by officers and merchants who did not sell at the prescribed rates were punished

Q38) Answer (d)

Explanation

Delhi first became the capital of a

kingdom under the Tomara Rajputs

who were defeated in the middle of the

twelfth century by the Chauhans (also

referred to as Chahamanas) of Ajmer

It was under the Tomaras and

Chauhans that Delhi became an

important commercial centre Many rich

Jaina merchants lived in the city and

constructed several temples Coins

minted here called dehliwal had a wide

circulation

Q39) Answer (c)

Explanation

Moth ki Masjid was built in the reign of

Sikandar Lodi by his minister

Begumpuri mosque built in the reign of

Muhammad Tughluq was the main

mosque of Jahanpanah the ldquoSanctuary

of the Worldrdquo and his new capital in

Delhi

Quwwat al ndash Islam mosque was

enlarged by Iltutmish and Alauddin

Khalji The minar was built by three

Sultansndash Qutbuddin Aybak Iltutmish

and Firuz Shah Tughluq

RAUSIAS-FC19E1003 45

Q40) Answer (c)

Explanation

Under the Mughals mansabdar was

referred to an individual who held a

mansab ie rank and he received his

salary as revenue assignments called

jagirs

Q41) Ans (b)

The Quit India Movement was a

spontaneous revolt of people against

British rule

The All India Congress Committee met

at Bombay on 8 August 1942 It passed

the famous resolution Quit India and

proposed the starting of a non-violent

mass struggle under Gandhis

leadership to achieve this aim But on

the very next day Gandhi and other

eminent leaders of the Congress were

arrested The Congress was once again

declared illegal

Q42) Ans (c)

The Simon Commission refers to a

group of seven MPs from the United

Kingdom constituted to suggest

constitutional reforms for British India

The Commission consisted of only

British members headed by one of the

senior British politicians Sir John

Simon

So the people of India agitated against

the arrival of Simon Commission

Q43) Ans (a)

He was widely known for his

unfavourable opinion of the economic

consequences of the British rule in

India

In his many writings and speeches and

especially in Poverty and Un-British

Rule in India Naoroji argued that India

was too highly taxed and that its wealth

was being drained away to England

He did not interpret the ancient Indian

texts and restored the self-confidence of

Indians And also he did not stress the

need for eradication of all the social

evils before anything else

Q44) Ans (c)

In August 1932 Prime Minister

MacDonald announced his Communal

Award Great Britainrsquos unilateral

attempt to resolve the various conflicts

among Indiarsquos many communal

interests

The award which was later

incorporated into the act of 1935

expanded the separate-electorate

formula reserved for Muslims to other

minorities including Sikhs Indian

Christians Anglo-Indians Europeans

distinct regional groups Gandhi

undertook a ldquofast unto deathrdquo against

that offer which he viewed as a

nefarious British plot to divide the

Indian society

Q45) Ans (b)

In British India apart from existing

imports and exports there was also a

particular amount of money which

colonial India contributed towards

administration maintenance of the

army war expenses pensions to retired

officers and other expenses accrued by

Britain towards maintenance of her

colony These were known as Home

charges and were paid for almost

entirely by India

The Home charges was made of

following components-

- Interest payable on Indian debt

- Dividend to shareholders of East

India Company

- Funds used to support the India

Office in London

- Funds used to pay salaries and

pensions of British personnel

engaged in India

- Interest on the railways

- Civil and military charges

- Store purchases in England

Q46) Ans (b)

The Lahore session of the Indian

National Congress was held in 1929

under the Presidentship of Jawaharlal

Nehru

The Lahore session of the Indian

National Congress witnessed significant

RAUSIAS-FC19E1003 46

developments in the Indian national

movement

- First the election of Jawaharlal

Nehru to the post of Presidentship of

the Congress was a clear indication

of the growing strength of the

Leftists in the Congress

- Secondly it was in this session that

the Congress for the first time raised

the demand for complete

independence Such demand was

not raised from the Congress

platform earlier

Q47) Ans (b)

It did not provide for separate

electorates for any community or

weightage for minorities However it did

allow for the reservation of minority

seats in provinces having minorities of

at least ten per cent but this was to be

in strict proportion to the size of the

community

There was no provision for complete

Independence for India

Q48) Ans (c)

The religion of early Vedic Aryans was

primarily of worship of nature and

Yajnas

The early Aryan religion was kind of

nature worship Actually the forces

around them which they could not

control or understand were invested

with divinity and were personified as

male or female gods And they

performed some Yajnas also

Q49) Ans (b)

The roads and river-routes were not

immune from robbery It is notable that

Yuan Chwang (Hiuen Tsang) was

robbed of his belongings during

Harshvardanarsquos period

Q50) Ans (c)

Q51) Ans (b)

Purandara Dasa was a saint and great

devotee of Lord Krishna

There is much speculation about where

Purandara Dasa regarded as the

Pitamaha of Carnatic music was born

Recently an expert committee

constituted by the Kannada University

Hampi has come to the conclusion that

Kshemapura Shivamogga district

Karnataka is the birth place of

Purandara Dasa

Q52) Ans (c)

Sri Tyagaraja Sri Shyama Shastry and Sri Muthuswami Dikshitar are considered the trinity of Carnatic music and with them came the golden age in Carnatic music in the 18th-19th

century

Q53) Ans d)

Recently a rare sarcophagus (stone

coffin) which is 2000 years old from the

Iron AgendashMegalithic era was discovered

from a rock-cut cave at Viyur village of

Kollam near Koyilandy in Kozhikode

district Kerala

The coffin containing bone fragments

was found during an excavation ldquoSo

far such a rare finding has been

discovered only from two sites

in Kerala Both these sarcophagi were

recovered from Megalithic sites at

Chevayur and Atholi also in Kozhikode

district

Q54) Ans a)

The megalithic culture in South India was a full-fledged Iron Age culture

Q55) Ans d)

The Cholas Pandyas and Keralaputras

(Cheras) mentioned in Ashokan

inscriptions were probably in the late

megalithic phase of material culture

Q56) Ans d)

Q57) Ans (b)

Raj Kumar Shukla followed Gandhiji all

over the country to persuade him to

come to Champaran to investigate the

problem associated with tinkathia

system

RAUSIAS-FC19E1003 47

Brij Kishore Rajendra Prasad Mahadev

Desai and Narhari Parikh accompanied

Gandhi ji during the Champaran

Satyagraha

Q58) Ans (b)

The Satvahanas started the practice of granting tax-free villages to brahmanas and Buddhist monks

Q59) Ans c)

The objectives of the Programme are

listed as under

- Developing basic tourism

infrastructure

- Promoting cultural and heritage

value of the country to generate

livelihoods in the identified regions

- Enhancing the tourist attractiveness

in a sustainable manner by

developing world-class

infrastructure at the heritage

monument sites

- Creating employment through active

involvement of local communities

- Harnessing tourism potential for its

effects on employment generation

and economic development

- Developing sustainable tourism

infrastructure and ensuring proper

Operations and maintenance

therein

Q60) Ans (b)

The Tribal Cooperative Marketing

Development Federation of India

(TRIFED) came into existence in 1987

It is a national-level apex organization

functioning under the administrative

control of Ministry of Tribal Affairs

Govt of India

TRIFED has its registered and Head

Office located in New Delhi

Q61) Ans (c)

Premchandrsquos novels include

Premashram Rangabhumi Ghaban

Karmabhumi and Godan

Gora is a novel written by Rabindranath

Tagore

138th birth anniversary of Munshi

Premchand was celebrated across the

country

Q62) Ans (b)

Giddha is a traditional pastoral dance

performed by the women of the Punjab

India and Pakistan at festival times

and at the sowing and reaping of the

harvest

By this dance the Punjabi women

reveal their joy expel their suppressed

feelings in a male dominated society

through the performance of Giddha

Since this dance has nothing to do with

men only women can participate in it

During the Teej celebrations Giddha

dance is celebrated in Punjab every

year Teej is a generic name for a

number of festivals that are celebrated

by women in some parts of India

Q63) Ans (a)

Dara Shukoh wrote the remarkable

work called ldquoMajma-ul-Bahrainrdquo or the

ldquoThe confluence of two seasrdquo

The Vice President of India Shri M

Venkaiah Naidu has said that Prince

Dara Shukohrsquos writings can come as a

refreshing source for infusing peace and

harmony He was addressing the

gathering after visiting the exhibition

that showcases the forgotten Prince of

yesteryears Dara Shukoh organized by

Mr Francois Gautier at Indira Gandhi

National Centre for the Arts in New

Delhi

Q64) Ans (c)

The statue Gommateshwara is

dedicated to the Jain God Bahubali

It is a monolithic statue

President Ram Nath Kovind

inaugurated the grand anointing

ceremony mdash Mahamastakabhisheka mdash

held once in 12 years at

Shravanabelagola (Karnataka)

Q65) Ans (c)

Prachi Valley had come up around the

Prachi river Prachi Valley gradually

disappeared

RAUSIAS-FC19E1003 48

The Prachi river originates from

Bhubaneswar

It is a tributary of the Mahanadi and

flows through the districts of Puri

Khurda Cuttack and Jagatsinghpur

and the entire region of the river is

termed as the Prachi Valley

It falls into the Bay of Bengal

Archaeological evidence shows that the

Prachi Valley Civilisation predates both

Harappa and Mohenjo-Daro

The Prachi river originates from

Bhubaneswar

Q66) Ans (d)

These monuments are located in

Chhatarpur district Madhya Pradesh

within Vindhya mountain range

Q67) Ans (a)

The book lsquoThoughts on Pakistanrsquo was

written by Dr BR Ambedkar

On the occasion of the birth anniversary

of Dr BR Ambedkar the president of

India pays homage to this icon of India

In 1924 he founded the Depressed

Classes Institute (Bahishkrit Hitkarini

Sabha) and in 1927 the Samaj Samata

Sangh

Another area of attention for Ambedkar

was education For its spread among

the low classes he set up a network of

colleges by the name of Peoples

Education Society and founded hostels

Q68) Ans(b)

Mehrgarh is a famous Neolithic

settlement in the Indian subcontinent

which is situated in Baluchistan

province Pakistan

A pre-historic rock art site is discovered

in the vast expanse of limestone blocks

on the eastern banks of Naguleru river

near Dachepalli (Andhra Pradesh) It

has thrown light on the Neolithic

civilisation that flourished in Guntur

(Andhra Pradesh) during 1500-2000

BC

Q69) Ans (c)

The 12th and the 13th centuries saw

the emergence of the Kakatiyas They

were at first the feudatories of the

Western Chalukyas of Kalyana Initially

they ruled over a small territory near

Warangal (Telangana)

They introduced Nayakships which was

later adopted and developed by the

Rayas of Vijayanagara

Q70) Ans (a)

The fast had effect of putting pressure

on mill owners who finally agreed to

give the workers a 35 per cent increase

in wages

Google celebrated with a doodle the

132nd birth anniversary of Anasuya

Sarabhai who played a pioneering role

in Indiarsquos labour movement

Q71) Ans (d)

The UNESCOrsquos list of the representative

list of the intangible cultural heritage of

humanity from India are

- Koodiyattam Sanskrit Theatre of

Kerala

- Mudiyettu ritual theatre and dance

drama of Kerala

- Tradition of Vedic Chanting

- Kalbelia folk songs and dances of

Rajasthan

- Ramlila Traditional Performance of

the Ramayana

- Sankirtana ritual singing

drumming and dancing of Manipur

- Ramman religious festival and

ritual theatre of the Garhwal

Himalayas India

- Traditional brass and copper craft of

utensil making among the Thatheras

of Jandiala Guru Punjab India

- Chhau dance classical Indian dance

originated in the eastern Indian

states

- Buddhist chanting of Ladakh

recitation of sacred Buddhist texts

in the trans-Himalayan Ladakh

region Jammu and Kashmir India

- Yoga

- Nouroz

- Kumbh Mela

RAUSIAS-FC19E1003 49

Q72) Ans(b)

The President of India Shri Ram Nath Kovind inaugurated the Hornbill Festival and State Formation Day celebrations of Nagaland in Kisama

The festival is named after the Indian hornbill the large and colourful forest bird which is displayed in the folklore of most of the states tribes

The major recognized tribes of Nagaland are Angami Ao Chakhesang Chang

Kuki Rengma and Zeling etc

Onge Jarawa and Sentinelese are the

tribes of Andman amp Nicobar Islands

Q73) Ans (c)

The Rashtrakutas rule in the Deccan lasted for almost two hundred years till the end of the tenth century The Rashtrakutas rulers were tolerant in their religious views and patronized not only Shaivism and Vaishnavism but

Jainism as well

The famous rock-cut temple of Shiva at Ellora was built by one of the Rashtrakutas kings Krishna I in the ninth century His successor Amoghavarsha was a Jain but he also

patronized other faiths

The Rashtrakutas allowed Muslims traders to settle and permitted Islam to

be preached in their dominions

Recently increasing defacement at the prehistoric rock paintings of Pandavulagutta Telangana has created a cause for grave concern It can spoil

the prehistoric rock

Pandavulagutta is home to

- Painted rock shelters dating to

10000 BC-8000 BC

- An 8th century inscription of the

Rashtrakuta period and

- Painted frescoes from the 12th century Kakatiya empire

Q74) Ans (b)

In 1828 Raja Ram Mohan Roy founded a new religious society the Brahma Sabha later known as the Brahmo

Samaj

Debendranath Tagore headed the Tattvabodhini Sabha which was

engaged in search of spiritual truth

Its purpose was to purify Hinduism and to preach monotheism or belief in one God

The new society was to be based on the twin pillars of reason and the Vedas and

Upanishads

Recently Sadharan Brahmo Samaj (SBS) has entered into a legal battle with the West Bengal government due

to some legal issue

Q75) Ans (c)

The Chishti order was established in India by Khwaja Moinuddin Chishti who came to India around 1192 The Chishtirsquos are considered to be the most influential of the groups of Sufis who migrated to India in the late twelfth century They adapted successfully to the local environment and adopted several features of Indian devotional

traditions

The historical dargah of Sufi mystic Khwaja Moinuddin Chishti in Ajmer is all set to get a facelift This 13 th century dargah has been included among the Swachh Iconic Places a clean-up initiative focused on iconic

heritage spiritual and cultural places

Page 35: GENERAL STUDIES (PAPER I) · Test is part of Rau’s IAS Test series for Preliminary Exam 2019 FOUNDATION + CURRENT AFFAIRS GENERAL STUDIES (PAPER –I) FOUNDATION TEST –III TOPIC:

FC19H1003 35

bull अपन कई लख ो और भािण ो म विशि रप स

ldquoपाििी एो ड अन-वबरविश रल इन इसणडया

(Poverty and Un-British Rule in India) म

नौर जी न यह तकि वदया वक भारत पर अतयविक

कर लगाया गया था और इसकी सोपवतत इोगलड की

ओर परिावहत की जा रही थी

bull उनह ोन पराचीन भारतीय गरोथ ो की वयाखया करन

का और भारतीय ो क आिविशवास क बहाल

करन पर कायि नही ो वकया था

उनह ोन वकसी और बात स पहल सभी सामावजक

बराइय ो क उनमलन की आिशयकता पर भी बल

नही ो वदया था

Q44) उततर (c)

सपषटीकरण

bull अगसत 1932 म वबरविश परिानमोतरी मकड नालड न

अपन साोपरदावयक परसकार (The Communal

Award) की घ िणा की थी

bull यह भारत क कई साोपरदावयक वहत ो क बीच विवभनन

सोघिो क हल करन क वलए वबरिन का एकतरफा

परयास था

bull यह परसकार (Award) बाद म 1935 क

अविवनयम (The Act of 1935) म शावमल वकया

गया था

bull इस साोपरदावयक परसकार न मससलम ो क वलए

आरवकषत एक अलग वनिािचक मणडल फॉमिल का

विसतार अनय अलपसोखयक ो क वलए वकया था

वजसम वसख ो भारतीय ईसाइय ो आोगल-भारतीय

समदाय यर पीय समदाय तथा विवशषट कषतरीय

समह ो क शावमल वकया गया था

bull गाोिी न इस परसताि क भारतीय समाज क

विभावजत करन क वलए एक घवणत वबरविश

सावजश क रप म दखा और उसक सखलाफ

आमरण अनशन वकया

Q45) उततर (b)

सपषटीकरण

मौजदा आयात और वनयाित क अवतररक़त

औपवनिवशक भारत क वनमनवलसखत खचो क

वलए एक विशिवनवशचत िन रावश भी दनी पडती

थी

(i) परशासन क वयय

(ii) सना क रख-रखाि क वयय

(iii) यद क वयय

(iv) सिावनितत अविकाररय ो की पशन तथा

(v) वबरिन दवारा अपनी उपवनिश बसती

(कॉल नी) क रख-रखाि क वयय

इनह गह शलक (Home Charges) क रप म

जाना जाता था और लगभग परी तरह स भारत क

दवारा इनका भगतान वकया जाता था

bull गह शलक म वनमनवलसखत घिक शावमल थ

(i) भारतीय ऋण पर दय बयाज

(ii) ईसट इोवडया को पनी क शयरिारक ो क

लाभाोश

(iii) लोदन म भारत कायािलय चलान क वलए िन

(iv) भारत म वनयकत वबरविश कवमिय ो क ितन

और पशन का भगतान करन क वलए िन

(v) रलि पर बयाज

(vi) नागररक और सनय शलक

(vii) इोगलड म सट र (सामगरी) की खरीद

Q46) उततर (b)

सपषटीकरण

bull भारतीय राषटर ीय काोगरस का लाहौर सतर 1929 म

जिाहरलाल नहर की अधयकषता म आय वजत

वकया गया था

bull इस सतर म भारतीय राषटर ीय आोद लन स समबसित

कई महतवपणि पररणाम सामन आय थ

(i) सििपरथम इस सतर म काोगरस क अधयकष पद

पर जिाहरलाल नहर क चना गया था ज

काोगरस म िामपोवथय ो की बिती हई ताकत

का सपषट सोकत था

(ii) दसरा इस सतर म पहली बार काोगरस न पणि

सवतोतरता की माोग क उठाया था

इस परकार की माोग काोगरस मोच स पहल कभी भी

नही ो उठाई गई थी

Q47) उततर (b)

सपषटीकरण

FC19H1003 36

bull इस ररप िि न वकसी भी समदाय क वलए पथक

वनिािचक मोडल अथिा अलपसोखयक ो क वलए

भाराोश की वसफाररश नही ो की थी

bull तथावप इस ररप िि न उन पराोत ो म अलपसोखयक

सीि ो क आरकषण की अनमवत दी थी जहाा पर कम

स कम दस परवतशत अलपसोखयक ह

bull लवकन यह समदाय क आकार क अनपात म ह ना

चावहए था

bull इस ररप िि म भारत क वलए पणि सवतोतरता क

वलए क ई पराििान नही ो था

Q48) उततर (c)

सपषटीकरण

bull आरो वभक िवदक आयो का िमि मखय रप स

परकवत की पजा और यजञ था

bull परारो वभक आयि िमि परकवत की पजा क समान था

bull िासति म उनक चार ो ओर की शसकतयाा वजनह न

त ि वनयोवतरत कर सकत थ और न ही समझ पाए

थ उनह वदवयता क साथ वनिवशत वकया गया तथा

उनह मादा या नर दिीदिताओो क रप म

परतीकतव वकया गया था

bull उनह ोन कछ यजञ ो का भी वनषपादन वकया था

Q49) उततर (b)

सपषटीकरण

bull सडक और नदी-मागि (जल-मागि) डकती स

सरवकषत नही ो थ

bull उललखनीय ह वक हिििििन क शासनकाल क

दौरान यआन चिाोग (हयएन साोग) का सारा

सामान लि वलया गया था

Q50) उततर (c)

सपषटीकरण

परशन म वदए गए द न ो कथन सही ह

Q51) उततर (b)

सपषटीकरण

bull परोदर दास एक सोत और भगिान कषण क एक

महान भकत थ

bull परोदर दास क कनाििक सोगीत क वपतामह क

रप म जाना जाता ह

bull यदयवप उनक जनम-थथान क बार म काफी

अिकल लगाई जाती रही ह

bull तथावप अब कननड विशवविदयालय हमपी क दवारा

गवठत एक विशिजञ सवमवत इस वनषकिि पर पहोची

ह वक उनका जनम थथान सोभितया कनाििक का

एक छ िा-सा गााि कषमपरा (वशिम गगा वजला)

था

Q52) उततर (c)

सपषटीकरण

bull शरी तयागराज शरी शयाम शासतरी और शरी मथसवामी

दीवकषतर क कनाििक सोगीत की वतरमवति माना

जाता ह

bull उनक कारण ही 18िी ो-19िी ो शताबदी म कनाििक

सोगीत का सववणिम यग आया था

Q53) उततर (d)

सपषटीकरण

bull अभी हाल ही म लौह यगीन-महापािावणक काल

का 2000 ििि पराना एक दलिभ सारक फगस

(Sarcophagus) (पतथर का ताबत) क ललम क

वियर गाोि (क वयलडी क पास वजला क वझक ड

करल राजय) की एक रॉक-कि गफा स ख जा गया

bull यह ताबत वजसम हविय ो क िकड थ खदाई क

दौरान वमला

bull अभी तक इस परकार की दलिभ ख ज करल क

मातर द ही थथान ो स हई ह

bull य द न ो सारक फगी (Sarcophagi) (पतथर क

ताबत) चियर और अथ ली (वजला क वझक ड) क

महापािाण थथल ो स वमल ह

Q54) उततर (a)

सपषटीकरण

FC19H1003 37

दवकषण भारत म महापािाण सोसकवत एक पणि

विकवसत लौह यगीन सोसकवत थी

Q55) उततर (d)

सपषटीकरण

bull च ल पाणडय और करलपतर (चर) इन तीन ो का

उललख अश क क अवभलख ो म वकया गया ह

bull सोभितः य भौवतक सोसकवत क उततर

महापािावणक चरण म थ

Q56) उततर (d)

सपषटीकरण

bull भीमा-क रगाोि की लडाई ततीय आोगल-मराठा

यद का वहससा थी

Q57) उततर (b)

सपषटीकरण

bull राजकमार शकल न गाोिीजी क चोपारण आन तथा

वतनकवथया परणाली स जडी समसया की जाोच क

वलए रारी करन क वलए दश भर म उनका

अनसरण वकया था

bull बज वकश र राजदर परसाद महादि दसाई और

नरहरी पाररख चोपारण सतयागरह क दौरान गाोिी

जी क सहय गी थ

Q58) उततर (b)

सपषटीकरण

bull बराहमण ो और बौद मठिाररय ो क कर-मकत गााि

अनदान म दन की परथा सतिाहन ो न आरमभ की

थी

Q59) उततर (c)

सपषटीकरण

इस कायिकरम क उददशय वनमनानसार ह

(i) बवनयादी पयििन आिाररक सोरचना का विकास

करना

(ii) चयवनत (पहचान वकय गए) कषतर ो म आजीविका क

सजन क वलए दश क साोसकवतक और विरासत

मलय ो क बिािा दना

(iii) विरासत समारक थथल ो पर विशव सतरीय आिाररक

सोरचना विकवसत करक एक सतत तरीक स

पयििक आकििण म िसद करना

(iv) थथानीय समदाय ो की सवकरय भागीदारी क माधयम

स र रगार ो का सजन करना

(v) र रगार उतपादन और आवथिक विकास क वलए

पयििन कषमता का उन पर परभाि का उपय ग

करना तथा

(vi) िारणीय पयििन आिाररक सोरचना का विकास

करना और उसका उवचत सोचालन तथा

रखरखाि सवनवशचत करना

Q60) उततर (b)

सपषटीकरण

bull यह वनकाय ििि 1987 म अससततव म आया था

bull यह एक राषटर ीय सतर का शीिि सोगठन ह ज भारत

सरकार क जनजातीय मामल ो क मोतरालय क

परशासवनक वनयोतरण क अिीन काम कर रहा ह

bull इसका पोजीकत और परिान कायािलय नई वदलली

म सथथत ह

Q61) उततर (c)

सपषटीकरण

bull परमचोद क उपनयास ो म परमाशरम रोगभवम गबन

कमिभवम और ग दान शावमल ह

bull ग रा रिी ोदरनाथ िग र क दवारा रवचत उपनयास ह

bull अभी हाल ही म मोशी परमचोद की 138िी ो जयोती दश

भर म मनाई गई थी

Q62) उततर (b)

सपषटीकरण

bull ldquoवगदाrdquo पोजाब (भारत) एिो पावकसतान की

मवहलाओो क दवारा तयौहार क समय और फसल

की बिाई तथा किाई क अिसर पर वकया जान

िाला एक पारोपररक दहाती नतय ह

FC19H1003 38

bull इस नतय क माधयम स पोजाबी मवहलाऐो अपनी

परसननता परकि करती ह तथा वगदा क परदशिन क

माधयम स परि िचिसव िाल समाज म मवहलाओो

की दबी हई भािनाओो क परकि करती ह

bull चोवक इस नतय का परि ो क साथ क ई सोबोि नही ो

ह अतः किल मवहलाऐो ही इसम भाग ल सकती

bull हर साल तीज समार ह क दौरान पोजाब म वगदा

नतय वकया जाता ह

तीज भारत क कछ भाग ो म मवहलाओो क दवारा

मनाया जान िाल कई तयौहार ो क वलए एक

वयापक नाम ह

Q63) उततर (a)

सपषटीकरण

- मजम-उल-बहरीन या द समदर ो का सोगम

नामक उललखनीय रचना दारा वशक ह क दवारा

वलखी थी

- भारत क उपराषटर पवत शरी एम िकया नायड न कहा

ह वक राजकमार दारा वशक ह की रचनाएा शाोवत

और सदभाि क बिािा दन क वलए एक तारा सर त

क रप म सामन आ सकती ो ह

- उपराषटर पवत गत ििो क भला वदए गए राजकमार

दारा वशक ह क परदवशित परचवलत करन हत

आय वजत एक परदशिनी का दौरा करन क बाद एक

सभा क सोब वित कर रह थ

- इस परदशिनी का आय जन फर क इस गौवियर

(Francois Gautier) क दवारा lsquoइोवदरा गाोिी नशनल

सिर फॉर द आििसrsquo (The Indira Gandhi

National Centre for the Arts) नई वदलली म

वकया गया था

Q64) उततर (c)

सपषटीकरण

- ग मतशवर परवतमा जन भगिान बाहबली क

समवपित ह

- यह एक एक-चटटानी पतथर की मवति ह

- राषटर पवत राम नाथ क विोद न शरिणबलग ला

(कनाििक) म आय वजत वकय जान िाल भवय

अवभिक समार ह महामसतकावभिक का

उदघािन वकया था

- यह समार ह 12 ििो म एक बार ह ता ह

Q65) उततर (c)

सपषटीकरण

bull पराची घािी पराची नदी क चार ो ओर फली हई थी

bull पराची घािी िीर-िीर विलपत ह गई थी

bull पराची नदी भिनशवर स वनकलती ह

bull यह महानदी की एक सहायक नदी ह और यह

परी खदाि किक तथा जगतवसोहपर वजल ो स

ह कर बहती ह

bull इस नदी क पर कषतर क पराची घािी कहा जाता ह

bull यह नदी बोगाल की खाडी म वगरती ह

परातासतवक साकषय स पता चलता ह वक पराची घािी

सभयता हडपपा और म हनज दाड द न ो की

पिििती ह

Q66) उततर (d)

सपषटीकरण

य समारक छतरपर वजल (मधय परदश) म विोधयाचल

पिित शरोखला म सथथत ह

Q67) उततर (a)

सपषटीकरण

bull थॉिस ऑन पावकसतान नामक पसतक डॉ बी

आर अमबडकर न वलखी थी

bull डॉ बी आर अमबडकर की जयोती क अिसर पर

भारत क राषटर पवत न भारत की इस महान हसती

क शरदाोजवल अवपित की थी

bull डॉ बी आर अमबडकर न 1924 म वडपरथड

कलावसर इोसटीटयि (दवलत िगि सोथथान -

बवहषकत वहतकाररणी सभा) और 1927 म समाज

समता सोघ की थथापना की थी

bull अमबडकर का धयान वशकषा कषतर की ओर भी था

bull उनह ोन वशकषा क वनमन िगो म फलान क वलए

पीपलस एजकशन स साइिी (The Peoples

Education Society) क नाम स महाविदयालय ो क

नििकि और छातरािास ो की थथापना की थी

FC19H1003 39

Q68) उततर (b)

सपषटीकरण

bull महरगि भारतीय उपमहादवीप म एक परवसद

निपािाण बसती ह ज बलवचसतान पराोत

पावकसतान म सथथत ह

bull दचपलली (आोधर परदश) क पास नागलर नदी क

पिी ति ो पर चना पतथर क बलॉक क विशाल

विसतार म एक पिि-ऐवतहावसक रॉक आिि थथल की

ख ज की गई ह

bull इसन 1500-2000 ईसा पिि क दौरान गोिर (आोधर

परदश) म विकवसत निपािाण सभयता पर परकाश

डाला ह

Q69) उततर (c)

सपषटीकरण

bull 12िी ो सदी और 13िी ो सदी म काकाविय िोश का

उदय हआ था

bull ि पहल कलयाण क पवशचमी चालकय ो क सामोत थ

bull परारोभ म उनह ोन िारोगल (तलोगाना) क पास एक

छ ि स कषतर पर शासन वकया था

bull उनह ोन ldquoनायक वयिथथाrdquo की शरआत की थी

वजस बाद म विजयनगर क राय शासक ो न

अपनाया और विकवसत वकया था

Q70) उततर (a)

सपषटीकरण

bull गाोिीजी क अनशन स वमल मावलक ो पर दबाि

पडा था ज अोततः शरवमक ो क ितन म 35 परवतशत

की िसद करन क वलए सहमत हए थ

bull गगल (Google) न अनसया साराभाई वजनह ोन

भारत क शरवमक आोद लन म एक अगरणी भवमका

वनभाई थी की 132िी ो जयोती डडल (Doodle) का

वनमािण करक मनाई

Q71) उततर (d)

सपषटीकरण

भारत स यनसक की मानिता की अमति साोसकवतक

विरासत की परवतवनवि सची म वनमनवलसखत शावमल ह

bull कवडयटटम करल का सोसकत रोगमोच

bull मवडयिि करल का अनषठान रोगमोच और नतय

नाविका

bull िवदक मि जाप की परोपरा

bull राजथथान क कालबवलया ल क गीत और नतय

bull रामलीला रामायण का पारोपररक परदशिन

bull सोकीतिन मवणपर का अनषठान गायन ढ ल िादन

और नतय

bull रममन भारत क गििाल वहमालय का िावमिक

तयौहार और अनषठान रोगमोच

bull जाोदीयाला गर पोजाब क ठठर ो की पीतल और

ताोब क वशलप स वनवमित बतिन ो की पारोपररक कला

bull छाऊ नतय पिी भारतीय राजय ो म जनमी शासतरीय

भारतीय नतय कला

bull लददाख का बौद मि जाप िर ाोस-वहमालयी लददाख

कषतर तथा जमम-कशमीर म पवितर बौद गरोथ ो का पाठ

bull य ग

bull नौर र

bull को भ मला

Q72) उततर (b)

सपषटीकरण

bull भारत क राषटर पवत शरी राम नाथ क विोद न

वकसामा नागालड म हॉनिवबल मह रति और

राजय गठन वदिस समार ह का उदघािन वकया

था

bull हॉनिवबल मह रति का नाम भारतीय हॉनिवबल क

नाम पर पडा ह ज एक विशाल और रोगीन जोगली

पकषी ह

bull यह पकषी नागालड राजय की अविकतर जनजावतय ो

की ल ककथाओो म उसललसखत ह

bull नागालड की परमख मानयता परापत जनजावतयाा ह

अोगामी आओ चखसोग चाोग ककी रगमा और

रवलोग आवद

bull ओोग जारिा और ससिनलीस अोडमान-वनक बार

दवीप समह की जनजावतयाा ह

FC19H1003 40

Q73) उततर (c)

सपषटीकरण

bull दकन म राषटर कि शासन दसिी ो सदी क अोत तक

लगभग 200 ििो तक रहा था

bull राषटर कि शासक अपन िावमिक विचार ो म सवहषण

bull उनह ोन न किल शि िमि और िषणि िमि बसलक

जन िमि क भी सोरकषण वदया था

bull एल रा म वशि क परवसद रॉक कि मोवदर का

वनमािण नौिी ो सदी म राषटर कि राजा कषण परथम न

करिाया था

bull उसका उततराविकारी अम घििि जन था लवकन

उसन अनय िमो क भी सोरकषण परदान वकया था

bull राषटर कि ो न मसलमान वयापाररय ो क बसन की

अनमवत दी थी

bull उनह न अपन अविराजय ो म इसलाम क उपदश दन

की भी अनमवत दी थी

bull अभी हाल ही म पाोडिलागटटा (तलोगाना) क

परागवतहावसक चटटान वचतर ो क कषरण की बिती हई

घिनाएा एक गोभीर वचोता का वििय ह

bull यह परागवतहावसक चटटान क नकसान पहाचा

सकता ह

bull पाोडिलागटटा वनमनवलसखत क वलए जाना जाता ह

- 10000 ईसा पिि स 8000 ईसा पिि क वचवतरत

चटटानी आशरय ो क वलए

- राषटर कि काल क एक 8 िी ो सदी क

वशलालख क वलए और

- 12िी ो सदी क काकविय सामराजय क वभवतत

वचतर ो क वलए

Q74) उततर (b)

सपषटीकरण

bull 1828 म राजा राम म हन रॉय न एक नय िावमिक

समाज बरहम सभा की थथापना की थी वजस बाद

म बरहम समाज क नाम स जाना गया था

bull दिदरनाथ िग र न ततवब विनी सभा की अधयकषता

की थी ज आधयासिक सतय की ख ज म सोलि

थी

bull इसका उददशय वहोद िमि क शद करन का और

एकशवरिाद (एक ईशवर म आथथा) का परचार करना

था

bull नय समाज की थथापना क आिार थ कारण

(तकि ) क द सतमभ तथा िद और उपवनिद

bull अभी हाल ही म सािारण बरहम समाज का कछ

काननी मदद ो क लकर पवशचम बोगाल सरकार क

साथ काननी वििाद चल रहा ह

Q75) उततर (c)

सपषटीकरण

bull भारत म वचशती वसलवसल की थथापना खवाजा

म इनददीन वचशती क दवारा की गयी थी

bull ि 1192 ईसवी क आसपास भारत आय थ

bull वचशतीय ो क बारहिी ो शताबदी क उततरािि म भारत

म आन िाल सफीय ो क समह ो म सबस

परभािशाली माना जाता ह

bull उनह ोन थथानीय िातािरण क साथ सफलतापििक

अनकलन वकया और उनह ोन भारतीय भसकत

परोपराओो क कई पहलओो क अपनाया

bull अजमर म सफी अपरकि खवाजा म इनददीन वचशती

की ऐवतहावसक दरगाह क एक नया रप दन की

तयारी की जा रही ह

bull इस 13िी ो शताबदी की दरगाह क ldquoसवचछ

आइकॉवनक थथल ोrdquo (Swacch Iconic Places) म

शावमल वकया गया ह ज परवतवषठत विरासत

आधयासिक और साोसकवतक थथान ो पर क वदरत

य जना ह

FC19H1003 41

ANSWERS amp EXPLANATION OF

NCERT History Class VI-X + Current Affairs

(FC19E1003)

Q1) Answer c

Explanation

Rigveda consists of more than a

thousand hymns dedicated to gods and

goddesses These hymns were

composed by sages and learnt by men

however a few were composed by

women like Apala Ghosa Lopamudra

Maitreyi and Gargi

Rigveda consists of many hymns in the

form of dialogues We get an example of

a dialogue between a sage named

Vishwamitra and two rivers (Beas and

Sutlej) that were worshipped as

goddesses This suggests that he

belonged to the Vedic period

Q2) Answer b

Explanation

Traces of ash have been found from

Kurnool Caves suggesting that people

were familiar with the use of fire

It is situated in Andhra Pradesh

Q3) Answer c

Explanation

Burzahom is a prehistoric site in

present day Kashmir where people built

pit houses which were dug into the

ground with steps leading into them

These may have provided shelter in cold

weather

Q4) Answer c

Explanation

Epigraphy is defined as the study of

inscriptions

Manuscriptology is the study of history

and literature through the use of hand

written documents

Palaeography refers to the study of

ancient writing systems and the

deciphering and dating of historical

manuscripts

Numismatics refers to the study of

coins

Q5) Answer a

Explanation

Charaka Samhita was written by

Charaka and is an important book on

Ayurveda and medicine

He was a practitioner of the traditional

system of Indian medicine known as

Ayurveda

Charaka is thought to have flourished

sometime between the 2nd century BCE

and the 2nd century CE

Q6) Answer b

Explanation

Bhaga refers to the tax on crops which

was fixed at 16th of the production

Kammakaras is the term used for the

landless agricultural labour class

Ashvamedha also known as horse

sacrifice is a ritual where a horse is let

loose to wander freely and it was

guarded by the rajarsquos men

Q7) Answer (d)

Explanation

In the Rigvedic period horses were

yoked to chariots that were used in

battles fought to capture land cattle

etc This suggests that the use of horse

chariots began much before the period

of Mahajanapadas

The battles were fought in the Rigvedic

period for cattlersquos lands water an even

to capture people Most men took part

in these wars however there was no

regular army but there were assemblies

where people met and discussed

matters of war Regular armies became

a feature in the Mjahajanapada period

including vast armies of foot soldiers

chariots and elephants

RAUSIAS-FC19E1003 42

Q8) Answer (a)

Explanation

Buddha belonged to the Sakya clan and

passed away at Kusinara

Buddha taught in Prakrit which was the

common language of people

Q9) Answer c

Explanation

There were six schools of philosophy in

ancient India These are known as

Vaishesika Nyaya Samkhya Yoga

Purva Mimansa and Vedanata or Uttara

Mimansa They were founded by sages

Kanada Gautama Kapila Patanjali

Jamini and Vyasa respectively

Q10) Answer b

Explanation

The teachings of Mahavira were

compiled at Valabhi in 6th century AD

Q11) Answer (c)

Explanation

Chanakya is traditionally identified as

Kautilya or Vishnugupta who authored

the ancient Indian political treatise the

Arthashastra

Q12) Answer d

The national emblem of India is an

adaptation of the Lion Capital atop the

Ashoka Pillar of Sarnath Uttar Pradesh

and is combined with the National

Motto Satyameva Jayate

The Rampurva Bull gets the name from

the site of its discovery Rampurva in

Bihar

It is noted for its delicately sculpted

model demonstrating superior

representation of soft flesh sensitive

nostrils alert ears and strong legs It is

a mixture of Indian and Persian

elements

Sankissa is situated in Uttar Pradesh

India

Q13) Ans(a)

Kunwar Singh was a notable leader during the Revolt of 1857 He belonged

to a royal house of Jagdispur Bihar

Q14) Answer b

Explanation

The term Vellalar was used for large

landowners

Q15) Answer c

Explanation

Arikamedu was a coastal settlement

where ships unloaded goods from

distant lands Finds here include a

massive brick warehouse pottery

including amphorae and Arretine ware

Roman lamps glassware and gems have

also been found at the site

Q16) Answer a

Explanation

Muvendar is a Tamil word mentioned in

Sangam poems meaning three chiefs

used for the heads of three ruling

families the Cholas Cheras and

Pandyas

Q17) Ans (c)

Several tribal or kin-based assemblies

such as the Sabha Vidatha and gana

are mentioned in the Rig-veda The

Sabha and the samiti mattered a great

deal in early Vedic times so much so

that the chiefs or the kings showed an

eagerness to win their support

Q18) Ans (a)

Jainism recognised the existence of the

gods but placed them lower than the

jina and did not condemn the varna

system as Buddhism did

Q19) Answer (d)

Explanation

Cholas and Pandyas had developed

powerful coastal cities The most

important city of Cholas was Puhar or

Kaveripattinam and Madurai was the

capital of Pandyas

Q20) Answer b

Explanation

Buddhacharita is the biography of

Buddha and was written by

RAUSIAS-FC19E1003 43

Ashvaghosha

Q21) Answer (a)

Explanation

Tamil poet Appar was a Shiva devotee

So he was a Nayanar saint

Q22) Answer d

Explanation

Samudragupta was a prominent Gupta

ruler whose coins depict him playing a

veena indicating his love for music We

get important historic information from

his Allahabad Prashasti which was

composed by his court poet Harisena

Q23) Answer (b)

Explanation

Vikrama Samvat was founded by

Chandragupta II in the 58 BC as a

mark of victory over the Shakas and

assumed the title of Vikramaditya

Banabhatta wrote Harshavardhanarsquos

biography the Harshacharita in

Sanskrit

Q24) Answer c

Explanation

Sandhi-vigrahika was the minister of

war and peace

Sarthavaha was the leader of the

merchant caravans

Q25) Answer a

Explanation

Xuan Zang (Hsuan-tsang) was a

Chinese traveller who came during the

reign of Harshavardhana

In the decade that began in 630 AD

Xuan Zang came to India through

Kashmir after visiting Central Asia Iran

and Afghanistan

He travelled from north to east and lived

in Bihar for a couple of years

At Nalanda University Xuan Zang

interacted with students and scholars

mastered local languages and

discovered Buddhist stupas

Q26) Answer c

Explanation

Pradakshina patha is a circular path

laid around a stupa in Buddhist

architecture While the rest are a part of

temple architecture

Q27) Answer d

Explanation

All the above-mentioned temples have

an elaborate use of bricks (baked

bricks) along with stone

Q28) Ans (c)

Muhammad Quli Qutab was the Sultan

of Golconda He was a contemporary of

Akbar was very fond of literature and

architecture

The Sultan was a great poet and he

wrote in Dakhini Urdu Persian and

Telgu and has left an extensive diwan or

collection

Recently the Archaeological Survey of

India (ASI) will be using Ground

Penetrating Radar (GPR) to map the

contours of the area around the Bagh-e-

Naya Qila excavated garden inside the

Golconda Fort in Telangana

Q29) Answer a

Explanation

Silappadikaram is a famous Tamil epic

which was written by Ilango around

1800 years ago It is a story of a

merchant named Kovalan who fell in

love with a courtesan named Madhavi

Manimekalai tells the story of the

daughter of Kovalan and Madhavi

Q30) Answer (a)

Explanation

Charaka is the author of Charaka

Samhita which is an important work of

Ayurveda and medicines

Brahmaguptarsquos fame rests mostly on his

Brahma-sphuta-siddhanta which was

an astronomical work It was translated

into Arabic in Baghdad and had a major

impact on Islamic mathematics and

astronomy

Late in his life Brahmagupta wrote

Khandakhadyaka which was an

RAUSIAS-FC19E1003 44

astronomical handbook that employed

Aryabhatarsquos system of starting each day

at midnight

Q31) Answer (c)

Explanation

Amir Khusrau was a famous sufi

musician poet and scholar In 1318 he

noted that there was different language

in every region of this land (Hindustan)

Lahori Kashmiri Dvarsamudri (in

Southern Karnataka) Telangana (in

Andhra Pradesh) Gujari (in Gujarat)

Marsquobari (in Tamil Nadu) Awadhi (in

eastern Uttar Pradesh) and Hindawai (in

the area around in Delhi) etc He went

to explain that Sanskrit did not belong

to any region and that only brahmans

knew it

Q32) Answer c

Explanation

Hiranyagarbha refers to the golden

womb When this ritual was performed

with the help of Brahmanas it was

thought to lead to the rebirth of the

sacrificer as a Khastriya

Q33) Answer d

Explanation

Kadamai refers to a tax on land

revenue

Gwalior Prashasti describes the exploits

of Nagabhata who was a Pratihara king

Q34) Answer b

Explanation

Rajatarangini is a Sanskrit text written

by Kalhana in the 12th century

It was historical chronicle of early India

It is justifiably considered to be the best

and most authentic work of its kind

It covers the entire span of history in

the Kashmir region from the earliest

times to the date of its composition

Q35) Answer c

Explanation

ldquoUrrdquo was the general assembly of the

village ldquoUrrdquo consisted of all the

taxpaying residents of an ordinary

village

Q36) Answer (a)

Explanation

Tarikh was a form of history writing in

the Delhi Sultanate The authors of

tawarikhs were learned men which

included secretaries administrators etc

Q37 Answer (a)

Explanation

Alauddin chose to pay his soldiers salaries in cash rather than iqtas The soldiers would buy their supplies from merchants in Delhi and it was thus feared that merchants would raise their prices To stop this Alauddin controlled the prices of goods in Delhi Prices were carefully surveyed by officers and merchants who did not sell at the prescribed rates were punished

Q38) Answer (d)

Explanation

Delhi first became the capital of a

kingdom under the Tomara Rajputs

who were defeated in the middle of the

twelfth century by the Chauhans (also

referred to as Chahamanas) of Ajmer

It was under the Tomaras and

Chauhans that Delhi became an

important commercial centre Many rich

Jaina merchants lived in the city and

constructed several temples Coins

minted here called dehliwal had a wide

circulation

Q39) Answer (c)

Explanation

Moth ki Masjid was built in the reign of

Sikandar Lodi by his minister

Begumpuri mosque built in the reign of

Muhammad Tughluq was the main

mosque of Jahanpanah the ldquoSanctuary

of the Worldrdquo and his new capital in

Delhi

Quwwat al ndash Islam mosque was

enlarged by Iltutmish and Alauddin

Khalji The minar was built by three

Sultansndash Qutbuddin Aybak Iltutmish

and Firuz Shah Tughluq

RAUSIAS-FC19E1003 45

Q40) Answer (c)

Explanation

Under the Mughals mansabdar was

referred to an individual who held a

mansab ie rank and he received his

salary as revenue assignments called

jagirs

Q41) Ans (b)

The Quit India Movement was a

spontaneous revolt of people against

British rule

The All India Congress Committee met

at Bombay on 8 August 1942 It passed

the famous resolution Quit India and

proposed the starting of a non-violent

mass struggle under Gandhis

leadership to achieve this aim But on

the very next day Gandhi and other

eminent leaders of the Congress were

arrested The Congress was once again

declared illegal

Q42) Ans (c)

The Simon Commission refers to a

group of seven MPs from the United

Kingdom constituted to suggest

constitutional reforms for British India

The Commission consisted of only

British members headed by one of the

senior British politicians Sir John

Simon

So the people of India agitated against

the arrival of Simon Commission

Q43) Ans (a)

He was widely known for his

unfavourable opinion of the economic

consequences of the British rule in

India

In his many writings and speeches and

especially in Poverty and Un-British

Rule in India Naoroji argued that India

was too highly taxed and that its wealth

was being drained away to England

He did not interpret the ancient Indian

texts and restored the self-confidence of

Indians And also he did not stress the

need for eradication of all the social

evils before anything else

Q44) Ans (c)

In August 1932 Prime Minister

MacDonald announced his Communal

Award Great Britainrsquos unilateral

attempt to resolve the various conflicts

among Indiarsquos many communal

interests

The award which was later

incorporated into the act of 1935

expanded the separate-electorate

formula reserved for Muslims to other

minorities including Sikhs Indian

Christians Anglo-Indians Europeans

distinct regional groups Gandhi

undertook a ldquofast unto deathrdquo against

that offer which he viewed as a

nefarious British plot to divide the

Indian society

Q45) Ans (b)

In British India apart from existing

imports and exports there was also a

particular amount of money which

colonial India contributed towards

administration maintenance of the

army war expenses pensions to retired

officers and other expenses accrued by

Britain towards maintenance of her

colony These were known as Home

charges and were paid for almost

entirely by India

The Home charges was made of

following components-

- Interest payable on Indian debt

- Dividend to shareholders of East

India Company

- Funds used to support the India

Office in London

- Funds used to pay salaries and

pensions of British personnel

engaged in India

- Interest on the railways

- Civil and military charges

- Store purchases in England

Q46) Ans (b)

The Lahore session of the Indian

National Congress was held in 1929

under the Presidentship of Jawaharlal

Nehru

The Lahore session of the Indian

National Congress witnessed significant

RAUSIAS-FC19E1003 46

developments in the Indian national

movement

- First the election of Jawaharlal

Nehru to the post of Presidentship of

the Congress was a clear indication

of the growing strength of the

Leftists in the Congress

- Secondly it was in this session that

the Congress for the first time raised

the demand for complete

independence Such demand was

not raised from the Congress

platform earlier

Q47) Ans (b)

It did not provide for separate

electorates for any community or

weightage for minorities However it did

allow for the reservation of minority

seats in provinces having minorities of

at least ten per cent but this was to be

in strict proportion to the size of the

community

There was no provision for complete

Independence for India

Q48) Ans (c)

The religion of early Vedic Aryans was

primarily of worship of nature and

Yajnas

The early Aryan religion was kind of

nature worship Actually the forces

around them which they could not

control or understand were invested

with divinity and were personified as

male or female gods And they

performed some Yajnas also

Q49) Ans (b)

The roads and river-routes were not

immune from robbery It is notable that

Yuan Chwang (Hiuen Tsang) was

robbed of his belongings during

Harshvardanarsquos period

Q50) Ans (c)

Q51) Ans (b)

Purandara Dasa was a saint and great

devotee of Lord Krishna

There is much speculation about where

Purandara Dasa regarded as the

Pitamaha of Carnatic music was born

Recently an expert committee

constituted by the Kannada University

Hampi has come to the conclusion that

Kshemapura Shivamogga district

Karnataka is the birth place of

Purandara Dasa

Q52) Ans (c)

Sri Tyagaraja Sri Shyama Shastry and Sri Muthuswami Dikshitar are considered the trinity of Carnatic music and with them came the golden age in Carnatic music in the 18th-19th

century

Q53) Ans d)

Recently a rare sarcophagus (stone

coffin) which is 2000 years old from the

Iron AgendashMegalithic era was discovered

from a rock-cut cave at Viyur village of

Kollam near Koyilandy in Kozhikode

district Kerala

The coffin containing bone fragments

was found during an excavation ldquoSo

far such a rare finding has been

discovered only from two sites

in Kerala Both these sarcophagi were

recovered from Megalithic sites at

Chevayur and Atholi also in Kozhikode

district

Q54) Ans a)

The megalithic culture in South India was a full-fledged Iron Age culture

Q55) Ans d)

The Cholas Pandyas and Keralaputras

(Cheras) mentioned in Ashokan

inscriptions were probably in the late

megalithic phase of material culture

Q56) Ans d)

Q57) Ans (b)

Raj Kumar Shukla followed Gandhiji all

over the country to persuade him to

come to Champaran to investigate the

problem associated with tinkathia

system

RAUSIAS-FC19E1003 47

Brij Kishore Rajendra Prasad Mahadev

Desai and Narhari Parikh accompanied

Gandhi ji during the Champaran

Satyagraha

Q58) Ans (b)

The Satvahanas started the practice of granting tax-free villages to brahmanas and Buddhist monks

Q59) Ans c)

The objectives of the Programme are

listed as under

- Developing basic tourism

infrastructure

- Promoting cultural and heritage

value of the country to generate

livelihoods in the identified regions

- Enhancing the tourist attractiveness

in a sustainable manner by

developing world-class

infrastructure at the heritage

monument sites

- Creating employment through active

involvement of local communities

- Harnessing tourism potential for its

effects on employment generation

and economic development

- Developing sustainable tourism

infrastructure and ensuring proper

Operations and maintenance

therein

Q60) Ans (b)

The Tribal Cooperative Marketing

Development Federation of India

(TRIFED) came into existence in 1987

It is a national-level apex organization

functioning under the administrative

control of Ministry of Tribal Affairs

Govt of India

TRIFED has its registered and Head

Office located in New Delhi

Q61) Ans (c)

Premchandrsquos novels include

Premashram Rangabhumi Ghaban

Karmabhumi and Godan

Gora is a novel written by Rabindranath

Tagore

138th birth anniversary of Munshi

Premchand was celebrated across the

country

Q62) Ans (b)

Giddha is a traditional pastoral dance

performed by the women of the Punjab

India and Pakistan at festival times

and at the sowing and reaping of the

harvest

By this dance the Punjabi women

reveal their joy expel their suppressed

feelings in a male dominated society

through the performance of Giddha

Since this dance has nothing to do with

men only women can participate in it

During the Teej celebrations Giddha

dance is celebrated in Punjab every

year Teej is a generic name for a

number of festivals that are celebrated

by women in some parts of India

Q63) Ans (a)

Dara Shukoh wrote the remarkable

work called ldquoMajma-ul-Bahrainrdquo or the

ldquoThe confluence of two seasrdquo

The Vice President of India Shri M

Venkaiah Naidu has said that Prince

Dara Shukohrsquos writings can come as a

refreshing source for infusing peace and

harmony He was addressing the

gathering after visiting the exhibition

that showcases the forgotten Prince of

yesteryears Dara Shukoh organized by

Mr Francois Gautier at Indira Gandhi

National Centre for the Arts in New

Delhi

Q64) Ans (c)

The statue Gommateshwara is

dedicated to the Jain God Bahubali

It is a monolithic statue

President Ram Nath Kovind

inaugurated the grand anointing

ceremony mdash Mahamastakabhisheka mdash

held once in 12 years at

Shravanabelagola (Karnataka)

Q65) Ans (c)

Prachi Valley had come up around the

Prachi river Prachi Valley gradually

disappeared

RAUSIAS-FC19E1003 48

The Prachi river originates from

Bhubaneswar

It is a tributary of the Mahanadi and

flows through the districts of Puri

Khurda Cuttack and Jagatsinghpur

and the entire region of the river is

termed as the Prachi Valley

It falls into the Bay of Bengal

Archaeological evidence shows that the

Prachi Valley Civilisation predates both

Harappa and Mohenjo-Daro

The Prachi river originates from

Bhubaneswar

Q66) Ans (d)

These monuments are located in

Chhatarpur district Madhya Pradesh

within Vindhya mountain range

Q67) Ans (a)

The book lsquoThoughts on Pakistanrsquo was

written by Dr BR Ambedkar

On the occasion of the birth anniversary

of Dr BR Ambedkar the president of

India pays homage to this icon of India

In 1924 he founded the Depressed

Classes Institute (Bahishkrit Hitkarini

Sabha) and in 1927 the Samaj Samata

Sangh

Another area of attention for Ambedkar

was education For its spread among

the low classes he set up a network of

colleges by the name of Peoples

Education Society and founded hostels

Q68) Ans(b)

Mehrgarh is a famous Neolithic

settlement in the Indian subcontinent

which is situated in Baluchistan

province Pakistan

A pre-historic rock art site is discovered

in the vast expanse of limestone blocks

on the eastern banks of Naguleru river

near Dachepalli (Andhra Pradesh) It

has thrown light on the Neolithic

civilisation that flourished in Guntur

(Andhra Pradesh) during 1500-2000

BC

Q69) Ans (c)

The 12th and the 13th centuries saw

the emergence of the Kakatiyas They

were at first the feudatories of the

Western Chalukyas of Kalyana Initially

they ruled over a small territory near

Warangal (Telangana)

They introduced Nayakships which was

later adopted and developed by the

Rayas of Vijayanagara

Q70) Ans (a)

The fast had effect of putting pressure

on mill owners who finally agreed to

give the workers a 35 per cent increase

in wages

Google celebrated with a doodle the

132nd birth anniversary of Anasuya

Sarabhai who played a pioneering role

in Indiarsquos labour movement

Q71) Ans (d)

The UNESCOrsquos list of the representative

list of the intangible cultural heritage of

humanity from India are

- Koodiyattam Sanskrit Theatre of

Kerala

- Mudiyettu ritual theatre and dance

drama of Kerala

- Tradition of Vedic Chanting

- Kalbelia folk songs and dances of

Rajasthan

- Ramlila Traditional Performance of

the Ramayana

- Sankirtana ritual singing

drumming and dancing of Manipur

- Ramman religious festival and

ritual theatre of the Garhwal

Himalayas India

- Traditional brass and copper craft of

utensil making among the Thatheras

of Jandiala Guru Punjab India

- Chhau dance classical Indian dance

originated in the eastern Indian

states

- Buddhist chanting of Ladakh

recitation of sacred Buddhist texts

in the trans-Himalayan Ladakh

region Jammu and Kashmir India

- Yoga

- Nouroz

- Kumbh Mela

RAUSIAS-FC19E1003 49

Q72) Ans(b)

The President of India Shri Ram Nath Kovind inaugurated the Hornbill Festival and State Formation Day celebrations of Nagaland in Kisama

The festival is named after the Indian hornbill the large and colourful forest bird which is displayed in the folklore of most of the states tribes

The major recognized tribes of Nagaland are Angami Ao Chakhesang Chang

Kuki Rengma and Zeling etc

Onge Jarawa and Sentinelese are the

tribes of Andman amp Nicobar Islands

Q73) Ans (c)

The Rashtrakutas rule in the Deccan lasted for almost two hundred years till the end of the tenth century The Rashtrakutas rulers were tolerant in their religious views and patronized not only Shaivism and Vaishnavism but

Jainism as well

The famous rock-cut temple of Shiva at Ellora was built by one of the Rashtrakutas kings Krishna I in the ninth century His successor Amoghavarsha was a Jain but he also

patronized other faiths

The Rashtrakutas allowed Muslims traders to settle and permitted Islam to

be preached in their dominions

Recently increasing defacement at the prehistoric rock paintings of Pandavulagutta Telangana has created a cause for grave concern It can spoil

the prehistoric rock

Pandavulagutta is home to

- Painted rock shelters dating to

10000 BC-8000 BC

- An 8th century inscription of the

Rashtrakuta period and

- Painted frescoes from the 12th century Kakatiya empire

Q74) Ans (b)

In 1828 Raja Ram Mohan Roy founded a new religious society the Brahma Sabha later known as the Brahmo

Samaj

Debendranath Tagore headed the Tattvabodhini Sabha which was

engaged in search of spiritual truth

Its purpose was to purify Hinduism and to preach monotheism or belief in one God

The new society was to be based on the twin pillars of reason and the Vedas and

Upanishads

Recently Sadharan Brahmo Samaj (SBS) has entered into a legal battle with the West Bengal government due

to some legal issue

Q75) Ans (c)

The Chishti order was established in India by Khwaja Moinuddin Chishti who came to India around 1192 The Chishtirsquos are considered to be the most influential of the groups of Sufis who migrated to India in the late twelfth century They adapted successfully to the local environment and adopted several features of Indian devotional

traditions

The historical dargah of Sufi mystic Khwaja Moinuddin Chishti in Ajmer is all set to get a facelift This 13 th century dargah has been included among the Swachh Iconic Places a clean-up initiative focused on iconic

heritage spiritual and cultural places

Page 36: GENERAL STUDIES (PAPER I) · Test is part of Rau’s IAS Test series for Preliminary Exam 2019 FOUNDATION + CURRENT AFFAIRS GENERAL STUDIES (PAPER –I) FOUNDATION TEST –III TOPIC:

FC19H1003 36

bull इस ररप िि न वकसी भी समदाय क वलए पथक

वनिािचक मोडल अथिा अलपसोखयक ो क वलए

भाराोश की वसफाररश नही ो की थी

bull तथावप इस ररप िि न उन पराोत ो म अलपसोखयक

सीि ो क आरकषण की अनमवत दी थी जहाा पर कम

स कम दस परवतशत अलपसोखयक ह

bull लवकन यह समदाय क आकार क अनपात म ह ना

चावहए था

bull इस ररप िि म भारत क वलए पणि सवतोतरता क

वलए क ई पराििान नही ो था

Q48) उततर (c)

सपषटीकरण

bull आरो वभक िवदक आयो का िमि मखय रप स

परकवत की पजा और यजञ था

bull परारो वभक आयि िमि परकवत की पजा क समान था

bull िासति म उनक चार ो ओर की शसकतयाा वजनह न

त ि वनयोवतरत कर सकत थ और न ही समझ पाए

थ उनह वदवयता क साथ वनिवशत वकया गया तथा

उनह मादा या नर दिीदिताओो क रप म

परतीकतव वकया गया था

bull उनह ोन कछ यजञ ो का भी वनषपादन वकया था

Q49) उततर (b)

सपषटीकरण

bull सडक और नदी-मागि (जल-मागि) डकती स

सरवकषत नही ो थ

bull उललखनीय ह वक हिििििन क शासनकाल क

दौरान यआन चिाोग (हयएन साोग) का सारा

सामान लि वलया गया था

Q50) उततर (c)

सपषटीकरण

परशन म वदए गए द न ो कथन सही ह

Q51) उततर (b)

सपषटीकरण

bull परोदर दास एक सोत और भगिान कषण क एक

महान भकत थ

bull परोदर दास क कनाििक सोगीत क वपतामह क

रप म जाना जाता ह

bull यदयवप उनक जनम-थथान क बार म काफी

अिकल लगाई जाती रही ह

bull तथावप अब कननड विशवविदयालय हमपी क दवारा

गवठत एक विशिजञ सवमवत इस वनषकिि पर पहोची

ह वक उनका जनम थथान सोभितया कनाििक का

एक छ िा-सा गााि कषमपरा (वशिम गगा वजला)

था

Q52) उततर (c)

सपषटीकरण

bull शरी तयागराज शरी शयाम शासतरी और शरी मथसवामी

दीवकषतर क कनाििक सोगीत की वतरमवति माना

जाता ह

bull उनक कारण ही 18िी ो-19िी ो शताबदी म कनाििक

सोगीत का सववणिम यग आया था

Q53) उततर (d)

सपषटीकरण

bull अभी हाल ही म लौह यगीन-महापािावणक काल

का 2000 ििि पराना एक दलिभ सारक फगस

(Sarcophagus) (पतथर का ताबत) क ललम क

वियर गाोि (क वयलडी क पास वजला क वझक ड

करल राजय) की एक रॉक-कि गफा स ख जा गया

bull यह ताबत वजसम हविय ो क िकड थ खदाई क

दौरान वमला

bull अभी तक इस परकार की दलिभ ख ज करल क

मातर द ही थथान ो स हई ह

bull य द न ो सारक फगी (Sarcophagi) (पतथर क

ताबत) चियर और अथ ली (वजला क वझक ड) क

महापािाण थथल ो स वमल ह

Q54) उततर (a)

सपषटीकरण

FC19H1003 37

दवकषण भारत म महापािाण सोसकवत एक पणि

विकवसत लौह यगीन सोसकवत थी

Q55) उततर (d)

सपषटीकरण

bull च ल पाणडय और करलपतर (चर) इन तीन ो का

उललख अश क क अवभलख ो म वकया गया ह

bull सोभितः य भौवतक सोसकवत क उततर

महापािावणक चरण म थ

Q56) उततर (d)

सपषटीकरण

bull भीमा-क रगाोि की लडाई ततीय आोगल-मराठा

यद का वहससा थी

Q57) उततर (b)

सपषटीकरण

bull राजकमार शकल न गाोिीजी क चोपारण आन तथा

वतनकवथया परणाली स जडी समसया की जाोच क

वलए रारी करन क वलए दश भर म उनका

अनसरण वकया था

bull बज वकश र राजदर परसाद महादि दसाई और

नरहरी पाररख चोपारण सतयागरह क दौरान गाोिी

जी क सहय गी थ

Q58) उततर (b)

सपषटीकरण

bull बराहमण ो और बौद मठिाररय ो क कर-मकत गााि

अनदान म दन की परथा सतिाहन ो न आरमभ की

थी

Q59) उततर (c)

सपषटीकरण

इस कायिकरम क उददशय वनमनानसार ह

(i) बवनयादी पयििन आिाररक सोरचना का विकास

करना

(ii) चयवनत (पहचान वकय गए) कषतर ो म आजीविका क

सजन क वलए दश क साोसकवतक और विरासत

मलय ो क बिािा दना

(iii) विरासत समारक थथल ो पर विशव सतरीय आिाररक

सोरचना विकवसत करक एक सतत तरीक स

पयििक आकििण म िसद करना

(iv) थथानीय समदाय ो की सवकरय भागीदारी क माधयम

स र रगार ो का सजन करना

(v) र रगार उतपादन और आवथिक विकास क वलए

पयििन कषमता का उन पर परभाि का उपय ग

करना तथा

(vi) िारणीय पयििन आिाररक सोरचना का विकास

करना और उसका उवचत सोचालन तथा

रखरखाि सवनवशचत करना

Q60) उततर (b)

सपषटीकरण

bull यह वनकाय ििि 1987 म अससततव म आया था

bull यह एक राषटर ीय सतर का शीिि सोगठन ह ज भारत

सरकार क जनजातीय मामल ो क मोतरालय क

परशासवनक वनयोतरण क अिीन काम कर रहा ह

bull इसका पोजीकत और परिान कायािलय नई वदलली

म सथथत ह

Q61) उततर (c)

सपषटीकरण

bull परमचोद क उपनयास ो म परमाशरम रोगभवम गबन

कमिभवम और ग दान शावमल ह

bull ग रा रिी ोदरनाथ िग र क दवारा रवचत उपनयास ह

bull अभी हाल ही म मोशी परमचोद की 138िी ो जयोती दश

भर म मनाई गई थी

Q62) उततर (b)

सपषटीकरण

bull ldquoवगदाrdquo पोजाब (भारत) एिो पावकसतान की

मवहलाओो क दवारा तयौहार क समय और फसल

की बिाई तथा किाई क अिसर पर वकया जान

िाला एक पारोपररक दहाती नतय ह

FC19H1003 38

bull इस नतय क माधयम स पोजाबी मवहलाऐो अपनी

परसननता परकि करती ह तथा वगदा क परदशिन क

माधयम स परि िचिसव िाल समाज म मवहलाओो

की दबी हई भािनाओो क परकि करती ह

bull चोवक इस नतय का परि ो क साथ क ई सोबोि नही ो

ह अतः किल मवहलाऐो ही इसम भाग ल सकती

bull हर साल तीज समार ह क दौरान पोजाब म वगदा

नतय वकया जाता ह

तीज भारत क कछ भाग ो म मवहलाओो क दवारा

मनाया जान िाल कई तयौहार ो क वलए एक

वयापक नाम ह

Q63) उततर (a)

सपषटीकरण

- मजम-उल-बहरीन या द समदर ो का सोगम

नामक उललखनीय रचना दारा वशक ह क दवारा

वलखी थी

- भारत क उपराषटर पवत शरी एम िकया नायड न कहा

ह वक राजकमार दारा वशक ह की रचनाएा शाोवत

और सदभाि क बिािा दन क वलए एक तारा सर त

क रप म सामन आ सकती ो ह

- उपराषटर पवत गत ििो क भला वदए गए राजकमार

दारा वशक ह क परदवशित परचवलत करन हत

आय वजत एक परदशिनी का दौरा करन क बाद एक

सभा क सोब वित कर रह थ

- इस परदशिनी का आय जन फर क इस गौवियर

(Francois Gautier) क दवारा lsquoइोवदरा गाोिी नशनल

सिर फॉर द आििसrsquo (The Indira Gandhi

National Centre for the Arts) नई वदलली म

वकया गया था

Q64) उततर (c)

सपषटीकरण

- ग मतशवर परवतमा जन भगिान बाहबली क

समवपित ह

- यह एक एक-चटटानी पतथर की मवति ह

- राषटर पवत राम नाथ क विोद न शरिणबलग ला

(कनाििक) म आय वजत वकय जान िाल भवय

अवभिक समार ह महामसतकावभिक का

उदघािन वकया था

- यह समार ह 12 ििो म एक बार ह ता ह

Q65) उततर (c)

सपषटीकरण

bull पराची घािी पराची नदी क चार ो ओर फली हई थी

bull पराची घािी िीर-िीर विलपत ह गई थी

bull पराची नदी भिनशवर स वनकलती ह

bull यह महानदी की एक सहायक नदी ह और यह

परी खदाि किक तथा जगतवसोहपर वजल ो स

ह कर बहती ह

bull इस नदी क पर कषतर क पराची घािी कहा जाता ह

bull यह नदी बोगाल की खाडी म वगरती ह

परातासतवक साकषय स पता चलता ह वक पराची घािी

सभयता हडपपा और म हनज दाड द न ो की

पिििती ह

Q66) उततर (d)

सपषटीकरण

य समारक छतरपर वजल (मधय परदश) म विोधयाचल

पिित शरोखला म सथथत ह

Q67) उततर (a)

सपषटीकरण

bull थॉिस ऑन पावकसतान नामक पसतक डॉ बी

आर अमबडकर न वलखी थी

bull डॉ बी आर अमबडकर की जयोती क अिसर पर

भारत क राषटर पवत न भारत की इस महान हसती

क शरदाोजवल अवपित की थी

bull डॉ बी आर अमबडकर न 1924 म वडपरथड

कलावसर इोसटीटयि (दवलत िगि सोथथान -

बवहषकत वहतकाररणी सभा) और 1927 म समाज

समता सोघ की थथापना की थी

bull अमबडकर का धयान वशकषा कषतर की ओर भी था

bull उनह ोन वशकषा क वनमन िगो म फलान क वलए

पीपलस एजकशन स साइिी (The Peoples

Education Society) क नाम स महाविदयालय ो क

नििकि और छातरािास ो की थथापना की थी

FC19H1003 39

Q68) उततर (b)

सपषटीकरण

bull महरगि भारतीय उपमहादवीप म एक परवसद

निपािाण बसती ह ज बलवचसतान पराोत

पावकसतान म सथथत ह

bull दचपलली (आोधर परदश) क पास नागलर नदी क

पिी ति ो पर चना पतथर क बलॉक क विशाल

विसतार म एक पिि-ऐवतहावसक रॉक आिि थथल की

ख ज की गई ह

bull इसन 1500-2000 ईसा पिि क दौरान गोिर (आोधर

परदश) म विकवसत निपािाण सभयता पर परकाश

डाला ह

Q69) उततर (c)

सपषटीकरण

bull 12िी ो सदी और 13िी ो सदी म काकाविय िोश का

उदय हआ था

bull ि पहल कलयाण क पवशचमी चालकय ो क सामोत थ

bull परारोभ म उनह ोन िारोगल (तलोगाना) क पास एक

छ ि स कषतर पर शासन वकया था

bull उनह ोन ldquoनायक वयिथथाrdquo की शरआत की थी

वजस बाद म विजयनगर क राय शासक ो न

अपनाया और विकवसत वकया था

Q70) उततर (a)

सपषटीकरण

bull गाोिीजी क अनशन स वमल मावलक ो पर दबाि

पडा था ज अोततः शरवमक ो क ितन म 35 परवतशत

की िसद करन क वलए सहमत हए थ

bull गगल (Google) न अनसया साराभाई वजनह ोन

भारत क शरवमक आोद लन म एक अगरणी भवमका

वनभाई थी की 132िी ो जयोती डडल (Doodle) का

वनमािण करक मनाई

Q71) उततर (d)

सपषटीकरण

भारत स यनसक की मानिता की अमति साोसकवतक

विरासत की परवतवनवि सची म वनमनवलसखत शावमल ह

bull कवडयटटम करल का सोसकत रोगमोच

bull मवडयिि करल का अनषठान रोगमोच और नतय

नाविका

bull िवदक मि जाप की परोपरा

bull राजथथान क कालबवलया ल क गीत और नतय

bull रामलीला रामायण का पारोपररक परदशिन

bull सोकीतिन मवणपर का अनषठान गायन ढ ल िादन

और नतय

bull रममन भारत क गििाल वहमालय का िावमिक

तयौहार और अनषठान रोगमोच

bull जाोदीयाला गर पोजाब क ठठर ो की पीतल और

ताोब क वशलप स वनवमित बतिन ो की पारोपररक कला

bull छाऊ नतय पिी भारतीय राजय ो म जनमी शासतरीय

भारतीय नतय कला

bull लददाख का बौद मि जाप िर ाोस-वहमालयी लददाख

कषतर तथा जमम-कशमीर म पवितर बौद गरोथ ो का पाठ

bull य ग

bull नौर र

bull को भ मला

Q72) उततर (b)

सपषटीकरण

bull भारत क राषटर पवत शरी राम नाथ क विोद न

वकसामा नागालड म हॉनिवबल मह रति और

राजय गठन वदिस समार ह का उदघािन वकया

था

bull हॉनिवबल मह रति का नाम भारतीय हॉनिवबल क

नाम पर पडा ह ज एक विशाल और रोगीन जोगली

पकषी ह

bull यह पकषी नागालड राजय की अविकतर जनजावतय ो

की ल ककथाओो म उसललसखत ह

bull नागालड की परमख मानयता परापत जनजावतयाा ह

अोगामी आओ चखसोग चाोग ककी रगमा और

रवलोग आवद

bull ओोग जारिा और ससिनलीस अोडमान-वनक बार

दवीप समह की जनजावतयाा ह

FC19H1003 40

Q73) उततर (c)

सपषटीकरण

bull दकन म राषटर कि शासन दसिी ो सदी क अोत तक

लगभग 200 ििो तक रहा था

bull राषटर कि शासक अपन िावमिक विचार ो म सवहषण

bull उनह ोन न किल शि िमि और िषणि िमि बसलक

जन िमि क भी सोरकषण वदया था

bull एल रा म वशि क परवसद रॉक कि मोवदर का

वनमािण नौिी ो सदी म राषटर कि राजा कषण परथम न

करिाया था

bull उसका उततराविकारी अम घििि जन था लवकन

उसन अनय िमो क भी सोरकषण परदान वकया था

bull राषटर कि ो न मसलमान वयापाररय ो क बसन की

अनमवत दी थी

bull उनह न अपन अविराजय ो म इसलाम क उपदश दन

की भी अनमवत दी थी

bull अभी हाल ही म पाोडिलागटटा (तलोगाना) क

परागवतहावसक चटटान वचतर ो क कषरण की बिती हई

घिनाएा एक गोभीर वचोता का वििय ह

bull यह परागवतहावसक चटटान क नकसान पहाचा

सकता ह

bull पाोडिलागटटा वनमनवलसखत क वलए जाना जाता ह

- 10000 ईसा पिि स 8000 ईसा पिि क वचवतरत

चटटानी आशरय ो क वलए

- राषटर कि काल क एक 8 िी ो सदी क

वशलालख क वलए और

- 12िी ो सदी क काकविय सामराजय क वभवतत

वचतर ो क वलए

Q74) उततर (b)

सपषटीकरण

bull 1828 म राजा राम म हन रॉय न एक नय िावमिक

समाज बरहम सभा की थथापना की थी वजस बाद

म बरहम समाज क नाम स जाना गया था

bull दिदरनाथ िग र न ततवब विनी सभा की अधयकषता

की थी ज आधयासिक सतय की ख ज म सोलि

थी

bull इसका उददशय वहोद िमि क शद करन का और

एकशवरिाद (एक ईशवर म आथथा) का परचार करना

था

bull नय समाज की थथापना क आिार थ कारण

(तकि ) क द सतमभ तथा िद और उपवनिद

bull अभी हाल ही म सािारण बरहम समाज का कछ

काननी मदद ो क लकर पवशचम बोगाल सरकार क

साथ काननी वििाद चल रहा ह

Q75) उततर (c)

सपषटीकरण

bull भारत म वचशती वसलवसल की थथापना खवाजा

म इनददीन वचशती क दवारा की गयी थी

bull ि 1192 ईसवी क आसपास भारत आय थ

bull वचशतीय ो क बारहिी ो शताबदी क उततरािि म भारत

म आन िाल सफीय ो क समह ो म सबस

परभािशाली माना जाता ह

bull उनह ोन थथानीय िातािरण क साथ सफलतापििक

अनकलन वकया और उनह ोन भारतीय भसकत

परोपराओो क कई पहलओो क अपनाया

bull अजमर म सफी अपरकि खवाजा म इनददीन वचशती

की ऐवतहावसक दरगाह क एक नया रप दन की

तयारी की जा रही ह

bull इस 13िी ो शताबदी की दरगाह क ldquoसवचछ

आइकॉवनक थथल ोrdquo (Swacch Iconic Places) म

शावमल वकया गया ह ज परवतवषठत विरासत

आधयासिक और साोसकवतक थथान ो पर क वदरत

य जना ह

FC19H1003 41

ANSWERS amp EXPLANATION OF

NCERT History Class VI-X + Current Affairs

(FC19E1003)

Q1) Answer c

Explanation

Rigveda consists of more than a

thousand hymns dedicated to gods and

goddesses These hymns were

composed by sages and learnt by men

however a few were composed by

women like Apala Ghosa Lopamudra

Maitreyi and Gargi

Rigveda consists of many hymns in the

form of dialogues We get an example of

a dialogue between a sage named

Vishwamitra and two rivers (Beas and

Sutlej) that were worshipped as

goddesses This suggests that he

belonged to the Vedic period

Q2) Answer b

Explanation

Traces of ash have been found from

Kurnool Caves suggesting that people

were familiar with the use of fire

It is situated in Andhra Pradesh

Q3) Answer c

Explanation

Burzahom is a prehistoric site in

present day Kashmir where people built

pit houses which were dug into the

ground with steps leading into them

These may have provided shelter in cold

weather

Q4) Answer c

Explanation

Epigraphy is defined as the study of

inscriptions

Manuscriptology is the study of history

and literature through the use of hand

written documents

Palaeography refers to the study of

ancient writing systems and the

deciphering and dating of historical

manuscripts

Numismatics refers to the study of

coins

Q5) Answer a

Explanation

Charaka Samhita was written by

Charaka and is an important book on

Ayurveda and medicine

He was a practitioner of the traditional

system of Indian medicine known as

Ayurveda

Charaka is thought to have flourished

sometime between the 2nd century BCE

and the 2nd century CE

Q6) Answer b

Explanation

Bhaga refers to the tax on crops which

was fixed at 16th of the production

Kammakaras is the term used for the

landless agricultural labour class

Ashvamedha also known as horse

sacrifice is a ritual where a horse is let

loose to wander freely and it was

guarded by the rajarsquos men

Q7) Answer (d)

Explanation

In the Rigvedic period horses were

yoked to chariots that were used in

battles fought to capture land cattle

etc This suggests that the use of horse

chariots began much before the period

of Mahajanapadas

The battles were fought in the Rigvedic

period for cattlersquos lands water an even

to capture people Most men took part

in these wars however there was no

regular army but there were assemblies

where people met and discussed

matters of war Regular armies became

a feature in the Mjahajanapada period

including vast armies of foot soldiers

chariots and elephants

RAUSIAS-FC19E1003 42

Q8) Answer (a)

Explanation

Buddha belonged to the Sakya clan and

passed away at Kusinara

Buddha taught in Prakrit which was the

common language of people

Q9) Answer c

Explanation

There were six schools of philosophy in

ancient India These are known as

Vaishesika Nyaya Samkhya Yoga

Purva Mimansa and Vedanata or Uttara

Mimansa They were founded by sages

Kanada Gautama Kapila Patanjali

Jamini and Vyasa respectively

Q10) Answer b

Explanation

The teachings of Mahavira were

compiled at Valabhi in 6th century AD

Q11) Answer (c)

Explanation

Chanakya is traditionally identified as

Kautilya or Vishnugupta who authored

the ancient Indian political treatise the

Arthashastra

Q12) Answer d

The national emblem of India is an

adaptation of the Lion Capital atop the

Ashoka Pillar of Sarnath Uttar Pradesh

and is combined with the National

Motto Satyameva Jayate

The Rampurva Bull gets the name from

the site of its discovery Rampurva in

Bihar

It is noted for its delicately sculpted

model demonstrating superior

representation of soft flesh sensitive

nostrils alert ears and strong legs It is

a mixture of Indian and Persian

elements

Sankissa is situated in Uttar Pradesh

India

Q13) Ans(a)

Kunwar Singh was a notable leader during the Revolt of 1857 He belonged

to a royal house of Jagdispur Bihar

Q14) Answer b

Explanation

The term Vellalar was used for large

landowners

Q15) Answer c

Explanation

Arikamedu was a coastal settlement

where ships unloaded goods from

distant lands Finds here include a

massive brick warehouse pottery

including amphorae and Arretine ware

Roman lamps glassware and gems have

also been found at the site

Q16) Answer a

Explanation

Muvendar is a Tamil word mentioned in

Sangam poems meaning three chiefs

used for the heads of three ruling

families the Cholas Cheras and

Pandyas

Q17) Ans (c)

Several tribal or kin-based assemblies

such as the Sabha Vidatha and gana

are mentioned in the Rig-veda The

Sabha and the samiti mattered a great

deal in early Vedic times so much so

that the chiefs or the kings showed an

eagerness to win their support

Q18) Ans (a)

Jainism recognised the existence of the

gods but placed them lower than the

jina and did not condemn the varna

system as Buddhism did

Q19) Answer (d)

Explanation

Cholas and Pandyas had developed

powerful coastal cities The most

important city of Cholas was Puhar or

Kaveripattinam and Madurai was the

capital of Pandyas

Q20) Answer b

Explanation

Buddhacharita is the biography of

Buddha and was written by

RAUSIAS-FC19E1003 43

Ashvaghosha

Q21) Answer (a)

Explanation

Tamil poet Appar was a Shiva devotee

So he was a Nayanar saint

Q22) Answer d

Explanation

Samudragupta was a prominent Gupta

ruler whose coins depict him playing a

veena indicating his love for music We

get important historic information from

his Allahabad Prashasti which was

composed by his court poet Harisena

Q23) Answer (b)

Explanation

Vikrama Samvat was founded by

Chandragupta II in the 58 BC as a

mark of victory over the Shakas and

assumed the title of Vikramaditya

Banabhatta wrote Harshavardhanarsquos

biography the Harshacharita in

Sanskrit

Q24) Answer c

Explanation

Sandhi-vigrahika was the minister of

war and peace

Sarthavaha was the leader of the

merchant caravans

Q25) Answer a

Explanation

Xuan Zang (Hsuan-tsang) was a

Chinese traveller who came during the

reign of Harshavardhana

In the decade that began in 630 AD

Xuan Zang came to India through

Kashmir after visiting Central Asia Iran

and Afghanistan

He travelled from north to east and lived

in Bihar for a couple of years

At Nalanda University Xuan Zang

interacted with students and scholars

mastered local languages and

discovered Buddhist stupas

Q26) Answer c

Explanation

Pradakshina patha is a circular path

laid around a stupa in Buddhist

architecture While the rest are a part of

temple architecture

Q27) Answer d

Explanation

All the above-mentioned temples have

an elaborate use of bricks (baked

bricks) along with stone

Q28) Ans (c)

Muhammad Quli Qutab was the Sultan

of Golconda He was a contemporary of

Akbar was very fond of literature and

architecture

The Sultan was a great poet and he

wrote in Dakhini Urdu Persian and

Telgu and has left an extensive diwan or

collection

Recently the Archaeological Survey of

India (ASI) will be using Ground

Penetrating Radar (GPR) to map the

contours of the area around the Bagh-e-

Naya Qila excavated garden inside the

Golconda Fort in Telangana

Q29) Answer a

Explanation

Silappadikaram is a famous Tamil epic

which was written by Ilango around

1800 years ago It is a story of a

merchant named Kovalan who fell in

love with a courtesan named Madhavi

Manimekalai tells the story of the

daughter of Kovalan and Madhavi

Q30) Answer (a)

Explanation

Charaka is the author of Charaka

Samhita which is an important work of

Ayurveda and medicines

Brahmaguptarsquos fame rests mostly on his

Brahma-sphuta-siddhanta which was

an astronomical work It was translated

into Arabic in Baghdad and had a major

impact on Islamic mathematics and

astronomy

Late in his life Brahmagupta wrote

Khandakhadyaka which was an

RAUSIAS-FC19E1003 44

astronomical handbook that employed

Aryabhatarsquos system of starting each day

at midnight

Q31) Answer (c)

Explanation

Amir Khusrau was a famous sufi

musician poet and scholar In 1318 he

noted that there was different language

in every region of this land (Hindustan)

Lahori Kashmiri Dvarsamudri (in

Southern Karnataka) Telangana (in

Andhra Pradesh) Gujari (in Gujarat)

Marsquobari (in Tamil Nadu) Awadhi (in

eastern Uttar Pradesh) and Hindawai (in

the area around in Delhi) etc He went

to explain that Sanskrit did not belong

to any region and that only brahmans

knew it

Q32) Answer c

Explanation

Hiranyagarbha refers to the golden

womb When this ritual was performed

with the help of Brahmanas it was

thought to lead to the rebirth of the

sacrificer as a Khastriya

Q33) Answer d

Explanation

Kadamai refers to a tax on land

revenue

Gwalior Prashasti describes the exploits

of Nagabhata who was a Pratihara king

Q34) Answer b

Explanation

Rajatarangini is a Sanskrit text written

by Kalhana in the 12th century

It was historical chronicle of early India

It is justifiably considered to be the best

and most authentic work of its kind

It covers the entire span of history in

the Kashmir region from the earliest

times to the date of its composition

Q35) Answer c

Explanation

ldquoUrrdquo was the general assembly of the

village ldquoUrrdquo consisted of all the

taxpaying residents of an ordinary

village

Q36) Answer (a)

Explanation

Tarikh was a form of history writing in

the Delhi Sultanate The authors of

tawarikhs were learned men which

included secretaries administrators etc

Q37 Answer (a)

Explanation

Alauddin chose to pay his soldiers salaries in cash rather than iqtas The soldiers would buy their supplies from merchants in Delhi and it was thus feared that merchants would raise their prices To stop this Alauddin controlled the prices of goods in Delhi Prices were carefully surveyed by officers and merchants who did not sell at the prescribed rates were punished

Q38) Answer (d)

Explanation

Delhi first became the capital of a

kingdom under the Tomara Rajputs

who were defeated in the middle of the

twelfth century by the Chauhans (also

referred to as Chahamanas) of Ajmer

It was under the Tomaras and

Chauhans that Delhi became an

important commercial centre Many rich

Jaina merchants lived in the city and

constructed several temples Coins

minted here called dehliwal had a wide

circulation

Q39) Answer (c)

Explanation

Moth ki Masjid was built in the reign of

Sikandar Lodi by his minister

Begumpuri mosque built in the reign of

Muhammad Tughluq was the main

mosque of Jahanpanah the ldquoSanctuary

of the Worldrdquo and his new capital in

Delhi

Quwwat al ndash Islam mosque was

enlarged by Iltutmish and Alauddin

Khalji The minar was built by three

Sultansndash Qutbuddin Aybak Iltutmish

and Firuz Shah Tughluq

RAUSIAS-FC19E1003 45

Q40) Answer (c)

Explanation

Under the Mughals mansabdar was

referred to an individual who held a

mansab ie rank and he received his

salary as revenue assignments called

jagirs

Q41) Ans (b)

The Quit India Movement was a

spontaneous revolt of people against

British rule

The All India Congress Committee met

at Bombay on 8 August 1942 It passed

the famous resolution Quit India and

proposed the starting of a non-violent

mass struggle under Gandhis

leadership to achieve this aim But on

the very next day Gandhi and other

eminent leaders of the Congress were

arrested The Congress was once again

declared illegal

Q42) Ans (c)

The Simon Commission refers to a

group of seven MPs from the United

Kingdom constituted to suggest

constitutional reforms for British India

The Commission consisted of only

British members headed by one of the

senior British politicians Sir John

Simon

So the people of India agitated against

the arrival of Simon Commission

Q43) Ans (a)

He was widely known for his

unfavourable opinion of the economic

consequences of the British rule in

India

In his many writings and speeches and

especially in Poverty and Un-British

Rule in India Naoroji argued that India

was too highly taxed and that its wealth

was being drained away to England

He did not interpret the ancient Indian

texts and restored the self-confidence of

Indians And also he did not stress the

need for eradication of all the social

evils before anything else

Q44) Ans (c)

In August 1932 Prime Minister

MacDonald announced his Communal

Award Great Britainrsquos unilateral

attempt to resolve the various conflicts

among Indiarsquos many communal

interests

The award which was later

incorporated into the act of 1935

expanded the separate-electorate

formula reserved for Muslims to other

minorities including Sikhs Indian

Christians Anglo-Indians Europeans

distinct regional groups Gandhi

undertook a ldquofast unto deathrdquo against

that offer which he viewed as a

nefarious British plot to divide the

Indian society

Q45) Ans (b)

In British India apart from existing

imports and exports there was also a

particular amount of money which

colonial India contributed towards

administration maintenance of the

army war expenses pensions to retired

officers and other expenses accrued by

Britain towards maintenance of her

colony These were known as Home

charges and were paid for almost

entirely by India

The Home charges was made of

following components-

- Interest payable on Indian debt

- Dividend to shareholders of East

India Company

- Funds used to support the India

Office in London

- Funds used to pay salaries and

pensions of British personnel

engaged in India

- Interest on the railways

- Civil and military charges

- Store purchases in England

Q46) Ans (b)

The Lahore session of the Indian

National Congress was held in 1929

under the Presidentship of Jawaharlal

Nehru

The Lahore session of the Indian

National Congress witnessed significant

RAUSIAS-FC19E1003 46

developments in the Indian national

movement

- First the election of Jawaharlal

Nehru to the post of Presidentship of

the Congress was a clear indication

of the growing strength of the

Leftists in the Congress

- Secondly it was in this session that

the Congress for the first time raised

the demand for complete

independence Such demand was

not raised from the Congress

platform earlier

Q47) Ans (b)

It did not provide for separate

electorates for any community or

weightage for minorities However it did

allow for the reservation of minority

seats in provinces having minorities of

at least ten per cent but this was to be

in strict proportion to the size of the

community

There was no provision for complete

Independence for India

Q48) Ans (c)

The religion of early Vedic Aryans was

primarily of worship of nature and

Yajnas

The early Aryan religion was kind of

nature worship Actually the forces

around them which they could not

control or understand were invested

with divinity and were personified as

male or female gods And they

performed some Yajnas also

Q49) Ans (b)

The roads and river-routes were not

immune from robbery It is notable that

Yuan Chwang (Hiuen Tsang) was

robbed of his belongings during

Harshvardanarsquos period

Q50) Ans (c)

Q51) Ans (b)

Purandara Dasa was a saint and great

devotee of Lord Krishna

There is much speculation about where

Purandara Dasa regarded as the

Pitamaha of Carnatic music was born

Recently an expert committee

constituted by the Kannada University

Hampi has come to the conclusion that

Kshemapura Shivamogga district

Karnataka is the birth place of

Purandara Dasa

Q52) Ans (c)

Sri Tyagaraja Sri Shyama Shastry and Sri Muthuswami Dikshitar are considered the trinity of Carnatic music and with them came the golden age in Carnatic music in the 18th-19th

century

Q53) Ans d)

Recently a rare sarcophagus (stone

coffin) which is 2000 years old from the

Iron AgendashMegalithic era was discovered

from a rock-cut cave at Viyur village of

Kollam near Koyilandy in Kozhikode

district Kerala

The coffin containing bone fragments

was found during an excavation ldquoSo

far such a rare finding has been

discovered only from two sites

in Kerala Both these sarcophagi were

recovered from Megalithic sites at

Chevayur and Atholi also in Kozhikode

district

Q54) Ans a)

The megalithic culture in South India was a full-fledged Iron Age culture

Q55) Ans d)

The Cholas Pandyas and Keralaputras

(Cheras) mentioned in Ashokan

inscriptions were probably in the late

megalithic phase of material culture

Q56) Ans d)

Q57) Ans (b)

Raj Kumar Shukla followed Gandhiji all

over the country to persuade him to

come to Champaran to investigate the

problem associated with tinkathia

system

RAUSIAS-FC19E1003 47

Brij Kishore Rajendra Prasad Mahadev

Desai and Narhari Parikh accompanied

Gandhi ji during the Champaran

Satyagraha

Q58) Ans (b)

The Satvahanas started the practice of granting tax-free villages to brahmanas and Buddhist monks

Q59) Ans c)

The objectives of the Programme are

listed as under

- Developing basic tourism

infrastructure

- Promoting cultural and heritage

value of the country to generate

livelihoods in the identified regions

- Enhancing the tourist attractiveness

in a sustainable manner by

developing world-class

infrastructure at the heritage

monument sites

- Creating employment through active

involvement of local communities

- Harnessing tourism potential for its

effects on employment generation

and economic development

- Developing sustainable tourism

infrastructure and ensuring proper

Operations and maintenance

therein

Q60) Ans (b)

The Tribal Cooperative Marketing

Development Federation of India

(TRIFED) came into existence in 1987

It is a national-level apex organization

functioning under the administrative

control of Ministry of Tribal Affairs

Govt of India

TRIFED has its registered and Head

Office located in New Delhi

Q61) Ans (c)

Premchandrsquos novels include

Premashram Rangabhumi Ghaban

Karmabhumi and Godan

Gora is a novel written by Rabindranath

Tagore

138th birth anniversary of Munshi

Premchand was celebrated across the

country

Q62) Ans (b)

Giddha is a traditional pastoral dance

performed by the women of the Punjab

India and Pakistan at festival times

and at the sowing and reaping of the

harvest

By this dance the Punjabi women

reveal their joy expel their suppressed

feelings in a male dominated society

through the performance of Giddha

Since this dance has nothing to do with

men only women can participate in it

During the Teej celebrations Giddha

dance is celebrated in Punjab every

year Teej is a generic name for a

number of festivals that are celebrated

by women in some parts of India

Q63) Ans (a)

Dara Shukoh wrote the remarkable

work called ldquoMajma-ul-Bahrainrdquo or the

ldquoThe confluence of two seasrdquo

The Vice President of India Shri M

Venkaiah Naidu has said that Prince

Dara Shukohrsquos writings can come as a

refreshing source for infusing peace and

harmony He was addressing the

gathering after visiting the exhibition

that showcases the forgotten Prince of

yesteryears Dara Shukoh organized by

Mr Francois Gautier at Indira Gandhi

National Centre for the Arts in New

Delhi

Q64) Ans (c)

The statue Gommateshwara is

dedicated to the Jain God Bahubali

It is a monolithic statue

President Ram Nath Kovind

inaugurated the grand anointing

ceremony mdash Mahamastakabhisheka mdash

held once in 12 years at

Shravanabelagola (Karnataka)

Q65) Ans (c)

Prachi Valley had come up around the

Prachi river Prachi Valley gradually

disappeared

RAUSIAS-FC19E1003 48

The Prachi river originates from

Bhubaneswar

It is a tributary of the Mahanadi and

flows through the districts of Puri

Khurda Cuttack and Jagatsinghpur

and the entire region of the river is

termed as the Prachi Valley

It falls into the Bay of Bengal

Archaeological evidence shows that the

Prachi Valley Civilisation predates both

Harappa and Mohenjo-Daro

The Prachi river originates from

Bhubaneswar

Q66) Ans (d)

These monuments are located in

Chhatarpur district Madhya Pradesh

within Vindhya mountain range

Q67) Ans (a)

The book lsquoThoughts on Pakistanrsquo was

written by Dr BR Ambedkar

On the occasion of the birth anniversary

of Dr BR Ambedkar the president of

India pays homage to this icon of India

In 1924 he founded the Depressed

Classes Institute (Bahishkrit Hitkarini

Sabha) and in 1927 the Samaj Samata

Sangh

Another area of attention for Ambedkar

was education For its spread among

the low classes he set up a network of

colleges by the name of Peoples

Education Society and founded hostels

Q68) Ans(b)

Mehrgarh is a famous Neolithic

settlement in the Indian subcontinent

which is situated in Baluchistan

province Pakistan

A pre-historic rock art site is discovered

in the vast expanse of limestone blocks

on the eastern banks of Naguleru river

near Dachepalli (Andhra Pradesh) It

has thrown light on the Neolithic

civilisation that flourished in Guntur

(Andhra Pradesh) during 1500-2000

BC

Q69) Ans (c)

The 12th and the 13th centuries saw

the emergence of the Kakatiyas They

were at first the feudatories of the

Western Chalukyas of Kalyana Initially

they ruled over a small territory near

Warangal (Telangana)

They introduced Nayakships which was

later adopted and developed by the

Rayas of Vijayanagara

Q70) Ans (a)

The fast had effect of putting pressure

on mill owners who finally agreed to

give the workers a 35 per cent increase

in wages

Google celebrated with a doodle the

132nd birth anniversary of Anasuya

Sarabhai who played a pioneering role

in Indiarsquos labour movement

Q71) Ans (d)

The UNESCOrsquos list of the representative

list of the intangible cultural heritage of

humanity from India are

- Koodiyattam Sanskrit Theatre of

Kerala

- Mudiyettu ritual theatre and dance

drama of Kerala

- Tradition of Vedic Chanting

- Kalbelia folk songs and dances of

Rajasthan

- Ramlila Traditional Performance of

the Ramayana

- Sankirtana ritual singing

drumming and dancing of Manipur

- Ramman religious festival and

ritual theatre of the Garhwal

Himalayas India

- Traditional brass and copper craft of

utensil making among the Thatheras

of Jandiala Guru Punjab India

- Chhau dance classical Indian dance

originated in the eastern Indian

states

- Buddhist chanting of Ladakh

recitation of sacred Buddhist texts

in the trans-Himalayan Ladakh

region Jammu and Kashmir India

- Yoga

- Nouroz

- Kumbh Mela

RAUSIAS-FC19E1003 49

Q72) Ans(b)

The President of India Shri Ram Nath Kovind inaugurated the Hornbill Festival and State Formation Day celebrations of Nagaland in Kisama

The festival is named after the Indian hornbill the large and colourful forest bird which is displayed in the folklore of most of the states tribes

The major recognized tribes of Nagaland are Angami Ao Chakhesang Chang

Kuki Rengma and Zeling etc

Onge Jarawa and Sentinelese are the

tribes of Andman amp Nicobar Islands

Q73) Ans (c)

The Rashtrakutas rule in the Deccan lasted for almost two hundred years till the end of the tenth century The Rashtrakutas rulers were tolerant in their religious views and patronized not only Shaivism and Vaishnavism but

Jainism as well

The famous rock-cut temple of Shiva at Ellora was built by one of the Rashtrakutas kings Krishna I in the ninth century His successor Amoghavarsha was a Jain but he also

patronized other faiths

The Rashtrakutas allowed Muslims traders to settle and permitted Islam to

be preached in their dominions

Recently increasing defacement at the prehistoric rock paintings of Pandavulagutta Telangana has created a cause for grave concern It can spoil

the prehistoric rock

Pandavulagutta is home to

- Painted rock shelters dating to

10000 BC-8000 BC

- An 8th century inscription of the

Rashtrakuta period and

- Painted frescoes from the 12th century Kakatiya empire

Q74) Ans (b)

In 1828 Raja Ram Mohan Roy founded a new religious society the Brahma Sabha later known as the Brahmo

Samaj

Debendranath Tagore headed the Tattvabodhini Sabha which was

engaged in search of spiritual truth

Its purpose was to purify Hinduism and to preach monotheism or belief in one God

The new society was to be based on the twin pillars of reason and the Vedas and

Upanishads

Recently Sadharan Brahmo Samaj (SBS) has entered into a legal battle with the West Bengal government due

to some legal issue

Q75) Ans (c)

The Chishti order was established in India by Khwaja Moinuddin Chishti who came to India around 1192 The Chishtirsquos are considered to be the most influential of the groups of Sufis who migrated to India in the late twelfth century They adapted successfully to the local environment and adopted several features of Indian devotional

traditions

The historical dargah of Sufi mystic Khwaja Moinuddin Chishti in Ajmer is all set to get a facelift This 13 th century dargah has been included among the Swachh Iconic Places a clean-up initiative focused on iconic

heritage spiritual and cultural places

Page 37: GENERAL STUDIES (PAPER I) · Test is part of Rau’s IAS Test series for Preliminary Exam 2019 FOUNDATION + CURRENT AFFAIRS GENERAL STUDIES (PAPER –I) FOUNDATION TEST –III TOPIC:

FC19H1003 37

दवकषण भारत म महापािाण सोसकवत एक पणि

विकवसत लौह यगीन सोसकवत थी

Q55) उततर (d)

सपषटीकरण

bull च ल पाणडय और करलपतर (चर) इन तीन ो का

उललख अश क क अवभलख ो म वकया गया ह

bull सोभितः य भौवतक सोसकवत क उततर

महापािावणक चरण म थ

Q56) उततर (d)

सपषटीकरण

bull भीमा-क रगाोि की लडाई ततीय आोगल-मराठा

यद का वहससा थी

Q57) उततर (b)

सपषटीकरण

bull राजकमार शकल न गाोिीजी क चोपारण आन तथा

वतनकवथया परणाली स जडी समसया की जाोच क

वलए रारी करन क वलए दश भर म उनका

अनसरण वकया था

bull बज वकश र राजदर परसाद महादि दसाई और

नरहरी पाररख चोपारण सतयागरह क दौरान गाोिी

जी क सहय गी थ

Q58) उततर (b)

सपषटीकरण

bull बराहमण ो और बौद मठिाररय ो क कर-मकत गााि

अनदान म दन की परथा सतिाहन ो न आरमभ की

थी

Q59) उततर (c)

सपषटीकरण

इस कायिकरम क उददशय वनमनानसार ह

(i) बवनयादी पयििन आिाररक सोरचना का विकास

करना

(ii) चयवनत (पहचान वकय गए) कषतर ो म आजीविका क

सजन क वलए दश क साोसकवतक और विरासत

मलय ो क बिािा दना

(iii) विरासत समारक थथल ो पर विशव सतरीय आिाररक

सोरचना विकवसत करक एक सतत तरीक स

पयििक आकििण म िसद करना

(iv) थथानीय समदाय ो की सवकरय भागीदारी क माधयम

स र रगार ो का सजन करना

(v) र रगार उतपादन और आवथिक विकास क वलए

पयििन कषमता का उन पर परभाि का उपय ग

करना तथा

(vi) िारणीय पयििन आिाररक सोरचना का विकास

करना और उसका उवचत सोचालन तथा

रखरखाि सवनवशचत करना

Q60) उततर (b)

सपषटीकरण

bull यह वनकाय ििि 1987 म अससततव म आया था

bull यह एक राषटर ीय सतर का शीिि सोगठन ह ज भारत

सरकार क जनजातीय मामल ो क मोतरालय क

परशासवनक वनयोतरण क अिीन काम कर रहा ह

bull इसका पोजीकत और परिान कायािलय नई वदलली

म सथथत ह

Q61) उततर (c)

सपषटीकरण

bull परमचोद क उपनयास ो म परमाशरम रोगभवम गबन

कमिभवम और ग दान शावमल ह

bull ग रा रिी ोदरनाथ िग र क दवारा रवचत उपनयास ह

bull अभी हाल ही म मोशी परमचोद की 138िी ो जयोती दश

भर म मनाई गई थी

Q62) उततर (b)

सपषटीकरण

bull ldquoवगदाrdquo पोजाब (भारत) एिो पावकसतान की

मवहलाओो क दवारा तयौहार क समय और फसल

की बिाई तथा किाई क अिसर पर वकया जान

िाला एक पारोपररक दहाती नतय ह

FC19H1003 38

bull इस नतय क माधयम स पोजाबी मवहलाऐो अपनी

परसननता परकि करती ह तथा वगदा क परदशिन क

माधयम स परि िचिसव िाल समाज म मवहलाओो

की दबी हई भािनाओो क परकि करती ह

bull चोवक इस नतय का परि ो क साथ क ई सोबोि नही ो

ह अतः किल मवहलाऐो ही इसम भाग ल सकती

bull हर साल तीज समार ह क दौरान पोजाब म वगदा

नतय वकया जाता ह

तीज भारत क कछ भाग ो म मवहलाओो क दवारा

मनाया जान िाल कई तयौहार ो क वलए एक

वयापक नाम ह

Q63) उततर (a)

सपषटीकरण

- मजम-उल-बहरीन या द समदर ो का सोगम

नामक उललखनीय रचना दारा वशक ह क दवारा

वलखी थी

- भारत क उपराषटर पवत शरी एम िकया नायड न कहा

ह वक राजकमार दारा वशक ह की रचनाएा शाोवत

और सदभाि क बिािा दन क वलए एक तारा सर त

क रप म सामन आ सकती ो ह

- उपराषटर पवत गत ििो क भला वदए गए राजकमार

दारा वशक ह क परदवशित परचवलत करन हत

आय वजत एक परदशिनी का दौरा करन क बाद एक

सभा क सोब वित कर रह थ

- इस परदशिनी का आय जन फर क इस गौवियर

(Francois Gautier) क दवारा lsquoइोवदरा गाोिी नशनल

सिर फॉर द आििसrsquo (The Indira Gandhi

National Centre for the Arts) नई वदलली म

वकया गया था

Q64) उततर (c)

सपषटीकरण

- ग मतशवर परवतमा जन भगिान बाहबली क

समवपित ह

- यह एक एक-चटटानी पतथर की मवति ह

- राषटर पवत राम नाथ क विोद न शरिणबलग ला

(कनाििक) म आय वजत वकय जान िाल भवय

अवभिक समार ह महामसतकावभिक का

उदघािन वकया था

- यह समार ह 12 ििो म एक बार ह ता ह

Q65) उततर (c)

सपषटीकरण

bull पराची घािी पराची नदी क चार ो ओर फली हई थी

bull पराची घािी िीर-िीर विलपत ह गई थी

bull पराची नदी भिनशवर स वनकलती ह

bull यह महानदी की एक सहायक नदी ह और यह

परी खदाि किक तथा जगतवसोहपर वजल ो स

ह कर बहती ह

bull इस नदी क पर कषतर क पराची घािी कहा जाता ह

bull यह नदी बोगाल की खाडी म वगरती ह

परातासतवक साकषय स पता चलता ह वक पराची घािी

सभयता हडपपा और म हनज दाड द न ो की

पिििती ह

Q66) उततर (d)

सपषटीकरण

य समारक छतरपर वजल (मधय परदश) म विोधयाचल

पिित शरोखला म सथथत ह

Q67) उततर (a)

सपषटीकरण

bull थॉिस ऑन पावकसतान नामक पसतक डॉ बी

आर अमबडकर न वलखी थी

bull डॉ बी आर अमबडकर की जयोती क अिसर पर

भारत क राषटर पवत न भारत की इस महान हसती

क शरदाोजवल अवपित की थी

bull डॉ बी आर अमबडकर न 1924 म वडपरथड

कलावसर इोसटीटयि (दवलत िगि सोथथान -

बवहषकत वहतकाररणी सभा) और 1927 म समाज

समता सोघ की थथापना की थी

bull अमबडकर का धयान वशकषा कषतर की ओर भी था

bull उनह ोन वशकषा क वनमन िगो म फलान क वलए

पीपलस एजकशन स साइिी (The Peoples

Education Society) क नाम स महाविदयालय ो क

नििकि और छातरािास ो की थथापना की थी

FC19H1003 39

Q68) उततर (b)

सपषटीकरण

bull महरगि भारतीय उपमहादवीप म एक परवसद

निपािाण बसती ह ज बलवचसतान पराोत

पावकसतान म सथथत ह

bull दचपलली (आोधर परदश) क पास नागलर नदी क

पिी ति ो पर चना पतथर क बलॉक क विशाल

विसतार म एक पिि-ऐवतहावसक रॉक आिि थथल की

ख ज की गई ह

bull इसन 1500-2000 ईसा पिि क दौरान गोिर (आोधर

परदश) म विकवसत निपािाण सभयता पर परकाश

डाला ह

Q69) उततर (c)

सपषटीकरण

bull 12िी ो सदी और 13िी ो सदी म काकाविय िोश का

उदय हआ था

bull ि पहल कलयाण क पवशचमी चालकय ो क सामोत थ

bull परारोभ म उनह ोन िारोगल (तलोगाना) क पास एक

छ ि स कषतर पर शासन वकया था

bull उनह ोन ldquoनायक वयिथथाrdquo की शरआत की थी

वजस बाद म विजयनगर क राय शासक ो न

अपनाया और विकवसत वकया था

Q70) उततर (a)

सपषटीकरण

bull गाोिीजी क अनशन स वमल मावलक ो पर दबाि

पडा था ज अोततः शरवमक ो क ितन म 35 परवतशत

की िसद करन क वलए सहमत हए थ

bull गगल (Google) न अनसया साराभाई वजनह ोन

भारत क शरवमक आोद लन म एक अगरणी भवमका

वनभाई थी की 132िी ो जयोती डडल (Doodle) का

वनमािण करक मनाई

Q71) उततर (d)

सपषटीकरण

भारत स यनसक की मानिता की अमति साोसकवतक

विरासत की परवतवनवि सची म वनमनवलसखत शावमल ह

bull कवडयटटम करल का सोसकत रोगमोच

bull मवडयिि करल का अनषठान रोगमोच और नतय

नाविका

bull िवदक मि जाप की परोपरा

bull राजथथान क कालबवलया ल क गीत और नतय

bull रामलीला रामायण का पारोपररक परदशिन

bull सोकीतिन मवणपर का अनषठान गायन ढ ल िादन

और नतय

bull रममन भारत क गििाल वहमालय का िावमिक

तयौहार और अनषठान रोगमोच

bull जाोदीयाला गर पोजाब क ठठर ो की पीतल और

ताोब क वशलप स वनवमित बतिन ो की पारोपररक कला

bull छाऊ नतय पिी भारतीय राजय ो म जनमी शासतरीय

भारतीय नतय कला

bull लददाख का बौद मि जाप िर ाोस-वहमालयी लददाख

कषतर तथा जमम-कशमीर म पवितर बौद गरोथ ो का पाठ

bull य ग

bull नौर र

bull को भ मला

Q72) उततर (b)

सपषटीकरण

bull भारत क राषटर पवत शरी राम नाथ क विोद न

वकसामा नागालड म हॉनिवबल मह रति और

राजय गठन वदिस समार ह का उदघािन वकया

था

bull हॉनिवबल मह रति का नाम भारतीय हॉनिवबल क

नाम पर पडा ह ज एक विशाल और रोगीन जोगली

पकषी ह

bull यह पकषी नागालड राजय की अविकतर जनजावतय ो

की ल ककथाओो म उसललसखत ह

bull नागालड की परमख मानयता परापत जनजावतयाा ह

अोगामी आओ चखसोग चाोग ककी रगमा और

रवलोग आवद

bull ओोग जारिा और ससिनलीस अोडमान-वनक बार

दवीप समह की जनजावतयाा ह

FC19H1003 40

Q73) उततर (c)

सपषटीकरण

bull दकन म राषटर कि शासन दसिी ो सदी क अोत तक

लगभग 200 ििो तक रहा था

bull राषटर कि शासक अपन िावमिक विचार ो म सवहषण

bull उनह ोन न किल शि िमि और िषणि िमि बसलक

जन िमि क भी सोरकषण वदया था

bull एल रा म वशि क परवसद रॉक कि मोवदर का

वनमािण नौिी ो सदी म राषटर कि राजा कषण परथम न

करिाया था

bull उसका उततराविकारी अम घििि जन था लवकन

उसन अनय िमो क भी सोरकषण परदान वकया था

bull राषटर कि ो न मसलमान वयापाररय ो क बसन की

अनमवत दी थी

bull उनह न अपन अविराजय ो म इसलाम क उपदश दन

की भी अनमवत दी थी

bull अभी हाल ही म पाोडिलागटटा (तलोगाना) क

परागवतहावसक चटटान वचतर ो क कषरण की बिती हई

घिनाएा एक गोभीर वचोता का वििय ह

bull यह परागवतहावसक चटटान क नकसान पहाचा

सकता ह

bull पाोडिलागटटा वनमनवलसखत क वलए जाना जाता ह

- 10000 ईसा पिि स 8000 ईसा पिि क वचवतरत

चटटानी आशरय ो क वलए

- राषटर कि काल क एक 8 िी ो सदी क

वशलालख क वलए और

- 12िी ो सदी क काकविय सामराजय क वभवतत

वचतर ो क वलए

Q74) उततर (b)

सपषटीकरण

bull 1828 म राजा राम म हन रॉय न एक नय िावमिक

समाज बरहम सभा की थथापना की थी वजस बाद

म बरहम समाज क नाम स जाना गया था

bull दिदरनाथ िग र न ततवब विनी सभा की अधयकषता

की थी ज आधयासिक सतय की ख ज म सोलि

थी

bull इसका उददशय वहोद िमि क शद करन का और

एकशवरिाद (एक ईशवर म आथथा) का परचार करना

था

bull नय समाज की थथापना क आिार थ कारण

(तकि ) क द सतमभ तथा िद और उपवनिद

bull अभी हाल ही म सािारण बरहम समाज का कछ

काननी मदद ो क लकर पवशचम बोगाल सरकार क

साथ काननी वििाद चल रहा ह

Q75) उततर (c)

सपषटीकरण

bull भारत म वचशती वसलवसल की थथापना खवाजा

म इनददीन वचशती क दवारा की गयी थी

bull ि 1192 ईसवी क आसपास भारत आय थ

bull वचशतीय ो क बारहिी ो शताबदी क उततरािि म भारत

म आन िाल सफीय ो क समह ो म सबस

परभािशाली माना जाता ह

bull उनह ोन थथानीय िातािरण क साथ सफलतापििक

अनकलन वकया और उनह ोन भारतीय भसकत

परोपराओो क कई पहलओो क अपनाया

bull अजमर म सफी अपरकि खवाजा म इनददीन वचशती

की ऐवतहावसक दरगाह क एक नया रप दन की

तयारी की जा रही ह

bull इस 13िी ो शताबदी की दरगाह क ldquoसवचछ

आइकॉवनक थथल ोrdquo (Swacch Iconic Places) म

शावमल वकया गया ह ज परवतवषठत विरासत

आधयासिक और साोसकवतक थथान ो पर क वदरत

य जना ह

FC19H1003 41

ANSWERS amp EXPLANATION OF

NCERT History Class VI-X + Current Affairs

(FC19E1003)

Q1) Answer c

Explanation

Rigveda consists of more than a

thousand hymns dedicated to gods and

goddesses These hymns were

composed by sages and learnt by men

however a few were composed by

women like Apala Ghosa Lopamudra

Maitreyi and Gargi

Rigveda consists of many hymns in the

form of dialogues We get an example of

a dialogue between a sage named

Vishwamitra and two rivers (Beas and

Sutlej) that were worshipped as

goddesses This suggests that he

belonged to the Vedic period

Q2) Answer b

Explanation

Traces of ash have been found from

Kurnool Caves suggesting that people

were familiar with the use of fire

It is situated in Andhra Pradesh

Q3) Answer c

Explanation

Burzahom is a prehistoric site in

present day Kashmir where people built

pit houses which were dug into the

ground with steps leading into them

These may have provided shelter in cold

weather

Q4) Answer c

Explanation

Epigraphy is defined as the study of

inscriptions

Manuscriptology is the study of history

and literature through the use of hand

written documents

Palaeography refers to the study of

ancient writing systems and the

deciphering and dating of historical

manuscripts

Numismatics refers to the study of

coins

Q5) Answer a

Explanation

Charaka Samhita was written by

Charaka and is an important book on

Ayurveda and medicine

He was a practitioner of the traditional

system of Indian medicine known as

Ayurveda

Charaka is thought to have flourished

sometime between the 2nd century BCE

and the 2nd century CE

Q6) Answer b

Explanation

Bhaga refers to the tax on crops which

was fixed at 16th of the production

Kammakaras is the term used for the

landless agricultural labour class

Ashvamedha also known as horse

sacrifice is a ritual where a horse is let

loose to wander freely and it was

guarded by the rajarsquos men

Q7) Answer (d)

Explanation

In the Rigvedic period horses were

yoked to chariots that were used in

battles fought to capture land cattle

etc This suggests that the use of horse

chariots began much before the period

of Mahajanapadas

The battles were fought in the Rigvedic

period for cattlersquos lands water an even

to capture people Most men took part

in these wars however there was no

regular army but there were assemblies

where people met and discussed

matters of war Regular armies became

a feature in the Mjahajanapada period

including vast armies of foot soldiers

chariots and elephants

RAUSIAS-FC19E1003 42

Q8) Answer (a)

Explanation

Buddha belonged to the Sakya clan and

passed away at Kusinara

Buddha taught in Prakrit which was the

common language of people

Q9) Answer c

Explanation

There were six schools of philosophy in

ancient India These are known as

Vaishesika Nyaya Samkhya Yoga

Purva Mimansa and Vedanata or Uttara

Mimansa They were founded by sages

Kanada Gautama Kapila Patanjali

Jamini and Vyasa respectively

Q10) Answer b

Explanation

The teachings of Mahavira were

compiled at Valabhi in 6th century AD

Q11) Answer (c)

Explanation

Chanakya is traditionally identified as

Kautilya or Vishnugupta who authored

the ancient Indian political treatise the

Arthashastra

Q12) Answer d

The national emblem of India is an

adaptation of the Lion Capital atop the

Ashoka Pillar of Sarnath Uttar Pradesh

and is combined with the National

Motto Satyameva Jayate

The Rampurva Bull gets the name from

the site of its discovery Rampurva in

Bihar

It is noted for its delicately sculpted

model demonstrating superior

representation of soft flesh sensitive

nostrils alert ears and strong legs It is

a mixture of Indian and Persian

elements

Sankissa is situated in Uttar Pradesh

India

Q13) Ans(a)

Kunwar Singh was a notable leader during the Revolt of 1857 He belonged

to a royal house of Jagdispur Bihar

Q14) Answer b

Explanation

The term Vellalar was used for large

landowners

Q15) Answer c

Explanation

Arikamedu was a coastal settlement

where ships unloaded goods from

distant lands Finds here include a

massive brick warehouse pottery

including amphorae and Arretine ware

Roman lamps glassware and gems have

also been found at the site

Q16) Answer a

Explanation

Muvendar is a Tamil word mentioned in

Sangam poems meaning three chiefs

used for the heads of three ruling

families the Cholas Cheras and

Pandyas

Q17) Ans (c)

Several tribal or kin-based assemblies

such as the Sabha Vidatha and gana

are mentioned in the Rig-veda The

Sabha and the samiti mattered a great

deal in early Vedic times so much so

that the chiefs or the kings showed an

eagerness to win their support

Q18) Ans (a)

Jainism recognised the existence of the

gods but placed them lower than the

jina and did not condemn the varna

system as Buddhism did

Q19) Answer (d)

Explanation

Cholas and Pandyas had developed

powerful coastal cities The most

important city of Cholas was Puhar or

Kaveripattinam and Madurai was the

capital of Pandyas

Q20) Answer b

Explanation

Buddhacharita is the biography of

Buddha and was written by

RAUSIAS-FC19E1003 43

Ashvaghosha

Q21) Answer (a)

Explanation

Tamil poet Appar was a Shiva devotee

So he was a Nayanar saint

Q22) Answer d

Explanation

Samudragupta was a prominent Gupta

ruler whose coins depict him playing a

veena indicating his love for music We

get important historic information from

his Allahabad Prashasti which was

composed by his court poet Harisena

Q23) Answer (b)

Explanation

Vikrama Samvat was founded by

Chandragupta II in the 58 BC as a

mark of victory over the Shakas and

assumed the title of Vikramaditya

Banabhatta wrote Harshavardhanarsquos

biography the Harshacharita in

Sanskrit

Q24) Answer c

Explanation

Sandhi-vigrahika was the minister of

war and peace

Sarthavaha was the leader of the

merchant caravans

Q25) Answer a

Explanation

Xuan Zang (Hsuan-tsang) was a

Chinese traveller who came during the

reign of Harshavardhana

In the decade that began in 630 AD

Xuan Zang came to India through

Kashmir after visiting Central Asia Iran

and Afghanistan

He travelled from north to east and lived

in Bihar for a couple of years

At Nalanda University Xuan Zang

interacted with students and scholars

mastered local languages and

discovered Buddhist stupas

Q26) Answer c

Explanation

Pradakshina patha is a circular path

laid around a stupa in Buddhist

architecture While the rest are a part of

temple architecture

Q27) Answer d

Explanation

All the above-mentioned temples have

an elaborate use of bricks (baked

bricks) along with stone

Q28) Ans (c)

Muhammad Quli Qutab was the Sultan

of Golconda He was a contemporary of

Akbar was very fond of literature and

architecture

The Sultan was a great poet and he

wrote in Dakhini Urdu Persian and

Telgu and has left an extensive diwan or

collection

Recently the Archaeological Survey of

India (ASI) will be using Ground

Penetrating Radar (GPR) to map the

contours of the area around the Bagh-e-

Naya Qila excavated garden inside the

Golconda Fort in Telangana

Q29) Answer a

Explanation

Silappadikaram is a famous Tamil epic

which was written by Ilango around

1800 years ago It is a story of a

merchant named Kovalan who fell in

love with a courtesan named Madhavi

Manimekalai tells the story of the

daughter of Kovalan and Madhavi

Q30) Answer (a)

Explanation

Charaka is the author of Charaka

Samhita which is an important work of

Ayurveda and medicines

Brahmaguptarsquos fame rests mostly on his

Brahma-sphuta-siddhanta which was

an astronomical work It was translated

into Arabic in Baghdad and had a major

impact on Islamic mathematics and

astronomy

Late in his life Brahmagupta wrote

Khandakhadyaka which was an

RAUSIAS-FC19E1003 44

astronomical handbook that employed

Aryabhatarsquos system of starting each day

at midnight

Q31) Answer (c)

Explanation

Amir Khusrau was a famous sufi

musician poet and scholar In 1318 he

noted that there was different language

in every region of this land (Hindustan)

Lahori Kashmiri Dvarsamudri (in

Southern Karnataka) Telangana (in

Andhra Pradesh) Gujari (in Gujarat)

Marsquobari (in Tamil Nadu) Awadhi (in

eastern Uttar Pradesh) and Hindawai (in

the area around in Delhi) etc He went

to explain that Sanskrit did not belong

to any region and that only brahmans

knew it

Q32) Answer c

Explanation

Hiranyagarbha refers to the golden

womb When this ritual was performed

with the help of Brahmanas it was

thought to lead to the rebirth of the

sacrificer as a Khastriya

Q33) Answer d

Explanation

Kadamai refers to a tax on land

revenue

Gwalior Prashasti describes the exploits

of Nagabhata who was a Pratihara king

Q34) Answer b

Explanation

Rajatarangini is a Sanskrit text written

by Kalhana in the 12th century

It was historical chronicle of early India

It is justifiably considered to be the best

and most authentic work of its kind

It covers the entire span of history in

the Kashmir region from the earliest

times to the date of its composition

Q35) Answer c

Explanation

ldquoUrrdquo was the general assembly of the

village ldquoUrrdquo consisted of all the

taxpaying residents of an ordinary

village

Q36) Answer (a)

Explanation

Tarikh was a form of history writing in

the Delhi Sultanate The authors of

tawarikhs were learned men which

included secretaries administrators etc

Q37 Answer (a)

Explanation

Alauddin chose to pay his soldiers salaries in cash rather than iqtas The soldiers would buy their supplies from merchants in Delhi and it was thus feared that merchants would raise their prices To stop this Alauddin controlled the prices of goods in Delhi Prices were carefully surveyed by officers and merchants who did not sell at the prescribed rates were punished

Q38) Answer (d)

Explanation

Delhi first became the capital of a

kingdom under the Tomara Rajputs

who were defeated in the middle of the

twelfth century by the Chauhans (also

referred to as Chahamanas) of Ajmer

It was under the Tomaras and

Chauhans that Delhi became an

important commercial centre Many rich

Jaina merchants lived in the city and

constructed several temples Coins

minted here called dehliwal had a wide

circulation

Q39) Answer (c)

Explanation

Moth ki Masjid was built in the reign of

Sikandar Lodi by his minister

Begumpuri mosque built in the reign of

Muhammad Tughluq was the main

mosque of Jahanpanah the ldquoSanctuary

of the Worldrdquo and his new capital in

Delhi

Quwwat al ndash Islam mosque was

enlarged by Iltutmish and Alauddin

Khalji The minar was built by three

Sultansndash Qutbuddin Aybak Iltutmish

and Firuz Shah Tughluq

RAUSIAS-FC19E1003 45

Q40) Answer (c)

Explanation

Under the Mughals mansabdar was

referred to an individual who held a

mansab ie rank and he received his

salary as revenue assignments called

jagirs

Q41) Ans (b)

The Quit India Movement was a

spontaneous revolt of people against

British rule

The All India Congress Committee met

at Bombay on 8 August 1942 It passed

the famous resolution Quit India and

proposed the starting of a non-violent

mass struggle under Gandhis

leadership to achieve this aim But on

the very next day Gandhi and other

eminent leaders of the Congress were

arrested The Congress was once again

declared illegal

Q42) Ans (c)

The Simon Commission refers to a

group of seven MPs from the United

Kingdom constituted to suggest

constitutional reforms for British India

The Commission consisted of only

British members headed by one of the

senior British politicians Sir John

Simon

So the people of India agitated against

the arrival of Simon Commission

Q43) Ans (a)

He was widely known for his

unfavourable opinion of the economic

consequences of the British rule in

India

In his many writings and speeches and

especially in Poverty and Un-British

Rule in India Naoroji argued that India

was too highly taxed and that its wealth

was being drained away to England

He did not interpret the ancient Indian

texts and restored the self-confidence of

Indians And also he did not stress the

need for eradication of all the social

evils before anything else

Q44) Ans (c)

In August 1932 Prime Minister

MacDonald announced his Communal

Award Great Britainrsquos unilateral

attempt to resolve the various conflicts

among Indiarsquos many communal

interests

The award which was later

incorporated into the act of 1935

expanded the separate-electorate

formula reserved for Muslims to other

minorities including Sikhs Indian

Christians Anglo-Indians Europeans

distinct regional groups Gandhi

undertook a ldquofast unto deathrdquo against

that offer which he viewed as a

nefarious British plot to divide the

Indian society

Q45) Ans (b)

In British India apart from existing

imports and exports there was also a

particular amount of money which

colonial India contributed towards

administration maintenance of the

army war expenses pensions to retired

officers and other expenses accrued by

Britain towards maintenance of her

colony These were known as Home

charges and were paid for almost

entirely by India

The Home charges was made of

following components-

- Interest payable on Indian debt

- Dividend to shareholders of East

India Company

- Funds used to support the India

Office in London

- Funds used to pay salaries and

pensions of British personnel

engaged in India

- Interest on the railways

- Civil and military charges

- Store purchases in England

Q46) Ans (b)

The Lahore session of the Indian

National Congress was held in 1929

under the Presidentship of Jawaharlal

Nehru

The Lahore session of the Indian

National Congress witnessed significant

RAUSIAS-FC19E1003 46

developments in the Indian national

movement

- First the election of Jawaharlal

Nehru to the post of Presidentship of

the Congress was a clear indication

of the growing strength of the

Leftists in the Congress

- Secondly it was in this session that

the Congress for the first time raised

the demand for complete

independence Such demand was

not raised from the Congress

platform earlier

Q47) Ans (b)

It did not provide for separate

electorates for any community or

weightage for minorities However it did

allow for the reservation of minority

seats in provinces having minorities of

at least ten per cent but this was to be

in strict proportion to the size of the

community

There was no provision for complete

Independence for India

Q48) Ans (c)

The religion of early Vedic Aryans was

primarily of worship of nature and

Yajnas

The early Aryan religion was kind of

nature worship Actually the forces

around them which they could not

control or understand were invested

with divinity and were personified as

male or female gods And they

performed some Yajnas also

Q49) Ans (b)

The roads and river-routes were not

immune from robbery It is notable that

Yuan Chwang (Hiuen Tsang) was

robbed of his belongings during

Harshvardanarsquos period

Q50) Ans (c)

Q51) Ans (b)

Purandara Dasa was a saint and great

devotee of Lord Krishna

There is much speculation about where

Purandara Dasa regarded as the

Pitamaha of Carnatic music was born

Recently an expert committee

constituted by the Kannada University

Hampi has come to the conclusion that

Kshemapura Shivamogga district

Karnataka is the birth place of

Purandara Dasa

Q52) Ans (c)

Sri Tyagaraja Sri Shyama Shastry and Sri Muthuswami Dikshitar are considered the trinity of Carnatic music and with them came the golden age in Carnatic music in the 18th-19th

century

Q53) Ans d)

Recently a rare sarcophagus (stone

coffin) which is 2000 years old from the

Iron AgendashMegalithic era was discovered

from a rock-cut cave at Viyur village of

Kollam near Koyilandy in Kozhikode

district Kerala

The coffin containing bone fragments

was found during an excavation ldquoSo

far such a rare finding has been

discovered only from two sites

in Kerala Both these sarcophagi were

recovered from Megalithic sites at

Chevayur and Atholi also in Kozhikode

district

Q54) Ans a)

The megalithic culture in South India was a full-fledged Iron Age culture

Q55) Ans d)

The Cholas Pandyas and Keralaputras

(Cheras) mentioned in Ashokan

inscriptions were probably in the late

megalithic phase of material culture

Q56) Ans d)

Q57) Ans (b)

Raj Kumar Shukla followed Gandhiji all

over the country to persuade him to

come to Champaran to investigate the

problem associated with tinkathia

system

RAUSIAS-FC19E1003 47

Brij Kishore Rajendra Prasad Mahadev

Desai and Narhari Parikh accompanied

Gandhi ji during the Champaran

Satyagraha

Q58) Ans (b)

The Satvahanas started the practice of granting tax-free villages to brahmanas and Buddhist monks

Q59) Ans c)

The objectives of the Programme are

listed as under

- Developing basic tourism

infrastructure

- Promoting cultural and heritage

value of the country to generate

livelihoods in the identified regions

- Enhancing the tourist attractiveness

in a sustainable manner by

developing world-class

infrastructure at the heritage

monument sites

- Creating employment through active

involvement of local communities

- Harnessing tourism potential for its

effects on employment generation

and economic development

- Developing sustainable tourism

infrastructure and ensuring proper

Operations and maintenance

therein

Q60) Ans (b)

The Tribal Cooperative Marketing

Development Federation of India

(TRIFED) came into existence in 1987

It is a national-level apex organization

functioning under the administrative

control of Ministry of Tribal Affairs

Govt of India

TRIFED has its registered and Head

Office located in New Delhi

Q61) Ans (c)

Premchandrsquos novels include

Premashram Rangabhumi Ghaban

Karmabhumi and Godan

Gora is a novel written by Rabindranath

Tagore

138th birth anniversary of Munshi

Premchand was celebrated across the

country

Q62) Ans (b)

Giddha is a traditional pastoral dance

performed by the women of the Punjab

India and Pakistan at festival times

and at the sowing and reaping of the

harvest

By this dance the Punjabi women

reveal their joy expel their suppressed

feelings in a male dominated society

through the performance of Giddha

Since this dance has nothing to do with

men only women can participate in it

During the Teej celebrations Giddha

dance is celebrated in Punjab every

year Teej is a generic name for a

number of festivals that are celebrated

by women in some parts of India

Q63) Ans (a)

Dara Shukoh wrote the remarkable

work called ldquoMajma-ul-Bahrainrdquo or the

ldquoThe confluence of two seasrdquo

The Vice President of India Shri M

Venkaiah Naidu has said that Prince

Dara Shukohrsquos writings can come as a

refreshing source for infusing peace and

harmony He was addressing the

gathering after visiting the exhibition

that showcases the forgotten Prince of

yesteryears Dara Shukoh organized by

Mr Francois Gautier at Indira Gandhi

National Centre for the Arts in New

Delhi

Q64) Ans (c)

The statue Gommateshwara is

dedicated to the Jain God Bahubali

It is a monolithic statue

President Ram Nath Kovind

inaugurated the grand anointing

ceremony mdash Mahamastakabhisheka mdash

held once in 12 years at

Shravanabelagola (Karnataka)

Q65) Ans (c)

Prachi Valley had come up around the

Prachi river Prachi Valley gradually

disappeared

RAUSIAS-FC19E1003 48

The Prachi river originates from

Bhubaneswar

It is a tributary of the Mahanadi and

flows through the districts of Puri

Khurda Cuttack and Jagatsinghpur

and the entire region of the river is

termed as the Prachi Valley

It falls into the Bay of Bengal

Archaeological evidence shows that the

Prachi Valley Civilisation predates both

Harappa and Mohenjo-Daro

The Prachi river originates from

Bhubaneswar

Q66) Ans (d)

These monuments are located in

Chhatarpur district Madhya Pradesh

within Vindhya mountain range

Q67) Ans (a)

The book lsquoThoughts on Pakistanrsquo was

written by Dr BR Ambedkar

On the occasion of the birth anniversary

of Dr BR Ambedkar the president of

India pays homage to this icon of India

In 1924 he founded the Depressed

Classes Institute (Bahishkrit Hitkarini

Sabha) and in 1927 the Samaj Samata

Sangh

Another area of attention for Ambedkar

was education For its spread among

the low classes he set up a network of

colleges by the name of Peoples

Education Society and founded hostels

Q68) Ans(b)

Mehrgarh is a famous Neolithic

settlement in the Indian subcontinent

which is situated in Baluchistan

province Pakistan

A pre-historic rock art site is discovered

in the vast expanse of limestone blocks

on the eastern banks of Naguleru river

near Dachepalli (Andhra Pradesh) It

has thrown light on the Neolithic

civilisation that flourished in Guntur

(Andhra Pradesh) during 1500-2000

BC

Q69) Ans (c)

The 12th and the 13th centuries saw

the emergence of the Kakatiyas They

were at first the feudatories of the

Western Chalukyas of Kalyana Initially

they ruled over a small territory near

Warangal (Telangana)

They introduced Nayakships which was

later adopted and developed by the

Rayas of Vijayanagara

Q70) Ans (a)

The fast had effect of putting pressure

on mill owners who finally agreed to

give the workers a 35 per cent increase

in wages

Google celebrated with a doodle the

132nd birth anniversary of Anasuya

Sarabhai who played a pioneering role

in Indiarsquos labour movement

Q71) Ans (d)

The UNESCOrsquos list of the representative

list of the intangible cultural heritage of

humanity from India are

- Koodiyattam Sanskrit Theatre of

Kerala

- Mudiyettu ritual theatre and dance

drama of Kerala

- Tradition of Vedic Chanting

- Kalbelia folk songs and dances of

Rajasthan

- Ramlila Traditional Performance of

the Ramayana

- Sankirtana ritual singing

drumming and dancing of Manipur

- Ramman religious festival and

ritual theatre of the Garhwal

Himalayas India

- Traditional brass and copper craft of

utensil making among the Thatheras

of Jandiala Guru Punjab India

- Chhau dance classical Indian dance

originated in the eastern Indian

states

- Buddhist chanting of Ladakh

recitation of sacred Buddhist texts

in the trans-Himalayan Ladakh

region Jammu and Kashmir India

- Yoga

- Nouroz

- Kumbh Mela

RAUSIAS-FC19E1003 49

Q72) Ans(b)

The President of India Shri Ram Nath Kovind inaugurated the Hornbill Festival and State Formation Day celebrations of Nagaland in Kisama

The festival is named after the Indian hornbill the large and colourful forest bird which is displayed in the folklore of most of the states tribes

The major recognized tribes of Nagaland are Angami Ao Chakhesang Chang

Kuki Rengma and Zeling etc

Onge Jarawa and Sentinelese are the

tribes of Andman amp Nicobar Islands

Q73) Ans (c)

The Rashtrakutas rule in the Deccan lasted for almost two hundred years till the end of the tenth century The Rashtrakutas rulers were tolerant in their religious views and patronized not only Shaivism and Vaishnavism but

Jainism as well

The famous rock-cut temple of Shiva at Ellora was built by one of the Rashtrakutas kings Krishna I in the ninth century His successor Amoghavarsha was a Jain but he also

patronized other faiths

The Rashtrakutas allowed Muslims traders to settle and permitted Islam to

be preached in their dominions

Recently increasing defacement at the prehistoric rock paintings of Pandavulagutta Telangana has created a cause for grave concern It can spoil

the prehistoric rock

Pandavulagutta is home to

- Painted rock shelters dating to

10000 BC-8000 BC

- An 8th century inscription of the

Rashtrakuta period and

- Painted frescoes from the 12th century Kakatiya empire

Q74) Ans (b)

In 1828 Raja Ram Mohan Roy founded a new religious society the Brahma Sabha later known as the Brahmo

Samaj

Debendranath Tagore headed the Tattvabodhini Sabha which was

engaged in search of spiritual truth

Its purpose was to purify Hinduism and to preach monotheism or belief in one God

The new society was to be based on the twin pillars of reason and the Vedas and

Upanishads

Recently Sadharan Brahmo Samaj (SBS) has entered into a legal battle with the West Bengal government due

to some legal issue

Q75) Ans (c)

The Chishti order was established in India by Khwaja Moinuddin Chishti who came to India around 1192 The Chishtirsquos are considered to be the most influential of the groups of Sufis who migrated to India in the late twelfth century They adapted successfully to the local environment and adopted several features of Indian devotional

traditions

The historical dargah of Sufi mystic Khwaja Moinuddin Chishti in Ajmer is all set to get a facelift This 13 th century dargah has been included among the Swachh Iconic Places a clean-up initiative focused on iconic

heritage spiritual and cultural places

Page 38: GENERAL STUDIES (PAPER I) · Test is part of Rau’s IAS Test series for Preliminary Exam 2019 FOUNDATION + CURRENT AFFAIRS GENERAL STUDIES (PAPER –I) FOUNDATION TEST –III TOPIC:

FC19H1003 38

bull इस नतय क माधयम स पोजाबी मवहलाऐो अपनी

परसननता परकि करती ह तथा वगदा क परदशिन क

माधयम स परि िचिसव िाल समाज म मवहलाओो

की दबी हई भािनाओो क परकि करती ह

bull चोवक इस नतय का परि ो क साथ क ई सोबोि नही ो

ह अतः किल मवहलाऐो ही इसम भाग ल सकती

bull हर साल तीज समार ह क दौरान पोजाब म वगदा

नतय वकया जाता ह

तीज भारत क कछ भाग ो म मवहलाओो क दवारा

मनाया जान िाल कई तयौहार ो क वलए एक

वयापक नाम ह

Q63) उततर (a)

सपषटीकरण

- मजम-उल-बहरीन या द समदर ो का सोगम

नामक उललखनीय रचना दारा वशक ह क दवारा

वलखी थी

- भारत क उपराषटर पवत शरी एम िकया नायड न कहा

ह वक राजकमार दारा वशक ह की रचनाएा शाोवत

और सदभाि क बिािा दन क वलए एक तारा सर त

क रप म सामन आ सकती ो ह

- उपराषटर पवत गत ििो क भला वदए गए राजकमार

दारा वशक ह क परदवशित परचवलत करन हत

आय वजत एक परदशिनी का दौरा करन क बाद एक

सभा क सोब वित कर रह थ

- इस परदशिनी का आय जन फर क इस गौवियर

(Francois Gautier) क दवारा lsquoइोवदरा गाोिी नशनल

सिर फॉर द आििसrsquo (The Indira Gandhi

National Centre for the Arts) नई वदलली म

वकया गया था

Q64) उततर (c)

सपषटीकरण

- ग मतशवर परवतमा जन भगिान बाहबली क

समवपित ह

- यह एक एक-चटटानी पतथर की मवति ह

- राषटर पवत राम नाथ क विोद न शरिणबलग ला

(कनाििक) म आय वजत वकय जान िाल भवय

अवभिक समार ह महामसतकावभिक का

उदघािन वकया था

- यह समार ह 12 ििो म एक बार ह ता ह

Q65) उततर (c)

सपषटीकरण

bull पराची घािी पराची नदी क चार ो ओर फली हई थी

bull पराची घािी िीर-िीर विलपत ह गई थी

bull पराची नदी भिनशवर स वनकलती ह

bull यह महानदी की एक सहायक नदी ह और यह

परी खदाि किक तथा जगतवसोहपर वजल ो स

ह कर बहती ह

bull इस नदी क पर कषतर क पराची घािी कहा जाता ह

bull यह नदी बोगाल की खाडी म वगरती ह

परातासतवक साकषय स पता चलता ह वक पराची घािी

सभयता हडपपा और म हनज दाड द न ो की

पिििती ह

Q66) उततर (d)

सपषटीकरण

य समारक छतरपर वजल (मधय परदश) म विोधयाचल

पिित शरोखला म सथथत ह

Q67) उततर (a)

सपषटीकरण

bull थॉिस ऑन पावकसतान नामक पसतक डॉ बी

आर अमबडकर न वलखी थी

bull डॉ बी आर अमबडकर की जयोती क अिसर पर

भारत क राषटर पवत न भारत की इस महान हसती

क शरदाोजवल अवपित की थी

bull डॉ बी आर अमबडकर न 1924 म वडपरथड

कलावसर इोसटीटयि (दवलत िगि सोथथान -

बवहषकत वहतकाररणी सभा) और 1927 म समाज

समता सोघ की थथापना की थी

bull अमबडकर का धयान वशकषा कषतर की ओर भी था

bull उनह ोन वशकषा क वनमन िगो म फलान क वलए

पीपलस एजकशन स साइिी (The Peoples

Education Society) क नाम स महाविदयालय ो क

नििकि और छातरािास ो की थथापना की थी

FC19H1003 39

Q68) उततर (b)

सपषटीकरण

bull महरगि भारतीय उपमहादवीप म एक परवसद

निपािाण बसती ह ज बलवचसतान पराोत

पावकसतान म सथथत ह

bull दचपलली (आोधर परदश) क पास नागलर नदी क

पिी ति ो पर चना पतथर क बलॉक क विशाल

विसतार म एक पिि-ऐवतहावसक रॉक आिि थथल की

ख ज की गई ह

bull इसन 1500-2000 ईसा पिि क दौरान गोिर (आोधर

परदश) म विकवसत निपािाण सभयता पर परकाश

डाला ह

Q69) उततर (c)

सपषटीकरण

bull 12िी ो सदी और 13िी ो सदी म काकाविय िोश का

उदय हआ था

bull ि पहल कलयाण क पवशचमी चालकय ो क सामोत थ

bull परारोभ म उनह ोन िारोगल (तलोगाना) क पास एक

छ ि स कषतर पर शासन वकया था

bull उनह ोन ldquoनायक वयिथथाrdquo की शरआत की थी

वजस बाद म विजयनगर क राय शासक ो न

अपनाया और विकवसत वकया था

Q70) उततर (a)

सपषटीकरण

bull गाोिीजी क अनशन स वमल मावलक ो पर दबाि

पडा था ज अोततः शरवमक ो क ितन म 35 परवतशत

की िसद करन क वलए सहमत हए थ

bull गगल (Google) न अनसया साराभाई वजनह ोन

भारत क शरवमक आोद लन म एक अगरणी भवमका

वनभाई थी की 132िी ो जयोती डडल (Doodle) का

वनमािण करक मनाई

Q71) उततर (d)

सपषटीकरण

भारत स यनसक की मानिता की अमति साोसकवतक

विरासत की परवतवनवि सची म वनमनवलसखत शावमल ह

bull कवडयटटम करल का सोसकत रोगमोच

bull मवडयिि करल का अनषठान रोगमोच और नतय

नाविका

bull िवदक मि जाप की परोपरा

bull राजथथान क कालबवलया ल क गीत और नतय

bull रामलीला रामायण का पारोपररक परदशिन

bull सोकीतिन मवणपर का अनषठान गायन ढ ल िादन

और नतय

bull रममन भारत क गििाल वहमालय का िावमिक

तयौहार और अनषठान रोगमोच

bull जाोदीयाला गर पोजाब क ठठर ो की पीतल और

ताोब क वशलप स वनवमित बतिन ो की पारोपररक कला

bull छाऊ नतय पिी भारतीय राजय ो म जनमी शासतरीय

भारतीय नतय कला

bull लददाख का बौद मि जाप िर ाोस-वहमालयी लददाख

कषतर तथा जमम-कशमीर म पवितर बौद गरोथ ो का पाठ

bull य ग

bull नौर र

bull को भ मला

Q72) उततर (b)

सपषटीकरण

bull भारत क राषटर पवत शरी राम नाथ क विोद न

वकसामा नागालड म हॉनिवबल मह रति और

राजय गठन वदिस समार ह का उदघािन वकया

था

bull हॉनिवबल मह रति का नाम भारतीय हॉनिवबल क

नाम पर पडा ह ज एक विशाल और रोगीन जोगली

पकषी ह

bull यह पकषी नागालड राजय की अविकतर जनजावतय ो

की ल ककथाओो म उसललसखत ह

bull नागालड की परमख मानयता परापत जनजावतयाा ह

अोगामी आओ चखसोग चाोग ककी रगमा और

रवलोग आवद

bull ओोग जारिा और ससिनलीस अोडमान-वनक बार

दवीप समह की जनजावतयाा ह

FC19H1003 40

Q73) उततर (c)

सपषटीकरण

bull दकन म राषटर कि शासन दसिी ो सदी क अोत तक

लगभग 200 ििो तक रहा था

bull राषटर कि शासक अपन िावमिक विचार ो म सवहषण

bull उनह ोन न किल शि िमि और िषणि िमि बसलक

जन िमि क भी सोरकषण वदया था

bull एल रा म वशि क परवसद रॉक कि मोवदर का

वनमािण नौिी ो सदी म राषटर कि राजा कषण परथम न

करिाया था

bull उसका उततराविकारी अम घििि जन था लवकन

उसन अनय िमो क भी सोरकषण परदान वकया था

bull राषटर कि ो न मसलमान वयापाररय ो क बसन की

अनमवत दी थी

bull उनह न अपन अविराजय ो म इसलाम क उपदश दन

की भी अनमवत दी थी

bull अभी हाल ही म पाोडिलागटटा (तलोगाना) क

परागवतहावसक चटटान वचतर ो क कषरण की बिती हई

घिनाएा एक गोभीर वचोता का वििय ह

bull यह परागवतहावसक चटटान क नकसान पहाचा

सकता ह

bull पाोडिलागटटा वनमनवलसखत क वलए जाना जाता ह

- 10000 ईसा पिि स 8000 ईसा पिि क वचवतरत

चटटानी आशरय ो क वलए

- राषटर कि काल क एक 8 िी ो सदी क

वशलालख क वलए और

- 12िी ो सदी क काकविय सामराजय क वभवतत

वचतर ो क वलए

Q74) उततर (b)

सपषटीकरण

bull 1828 म राजा राम म हन रॉय न एक नय िावमिक

समाज बरहम सभा की थथापना की थी वजस बाद

म बरहम समाज क नाम स जाना गया था

bull दिदरनाथ िग र न ततवब विनी सभा की अधयकषता

की थी ज आधयासिक सतय की ख ज म सोलि

थी

bull इसका उददशय वहोद िमि क शद करन का और

एकशवरिाद (एक ईशवर म आथथा) का परचार करना

था

bull नय समाज की थथापना क आिार थ कारण

(तकि ) क द सतमभ तथा िद और उपवनिद

bull अभी हाल ही म सािारण बरहम समाज का कछ

काननी मदद ो क लकर पवशचम बोगाल सरकार क

साथ काननी वििाद चल रहा ह

Q75) उततर (c)

सपषटीकरण

bull भारत म वचशती वसलवसल की थथापना खवाजा

म इनददीन वचशती क दवारा की गयी थी

bull ि 1192 ईसवी क आसपास भारत आय थ

bull वचशतीय ो क बारहिी ो शताबदी क उततरािि म भारत

म आन िाल सफीय ो क समह ो म सबस

परभािशाली माना जाता ह

bull उनह ोन थथानीय िातािरण क साथ सफलतापििक

अनकलन वकया और उनह ोन भारतीय भसकत

परोपराओो क कई पहलओो क अपनाया

bull अजमर म सफी अपरकि खवाजा म इनददीन वचशती

की ऐवतहावसक दरगाह क एक नया रप दन की

तयारी की जा रही ह

bull इस 13िी ो शताबदी की दरगाह क ldquoसवचछ

आइकॉवनक थथल ोrdquo (Swacch Iconic Places) म

शावमल वकया गया ह ज परवतवषठत विरासत

आधयासिक और साोसकवतक थथान ो पर क वदरत

य जना ह

FC19H1003 41

ANSWERS amp EXPLANATION OF

NCERT History Class VI-X + Current Affairs

(FC19E1003)

Q1) Answer c

Explanation

Rigveda consists of more than a

thousand hymns dedicated to gods and

goddesses These hymns were

composed by sages and learnt by men

however a few were composed by

women like Apala Ghosa Lopamudra

Maitreyi and Gargi

Rigveda consists of many hymns in the

form of dialogues We get an example of

a dialogue between a sage named

Vishwamitra and two rivers (Beas and

Sutlej) that were worshipped as

goddesses This suggests that he

belonged to the Vedic period

Q2) Answer b

Explanation

Traces of ash have been found from

Kurnool Caves suggesting that people

were familiar with the use of fire

It is situated in Andhra Pradesh

Q3) Answer c

Explanation

Burzahom is a prehistoric site in

present day Kashmir where people built

pit houses which were dug into the

ground with steps leading into them

These may have provided shelter in cold

weather

Q4) Answer c

Explanation

Epigraphy is defined as the study of

inscriptions

Manuscriptology is the study of history

and literature through the use of hand

written documents

Palaeography refers to the study of

ancient writing systems and the

deciphering and dating of historical

manuscripts

Numismatics refers to the study of

coins

Q5) Answer a

Explanation

Charaka Samhita was written by

Charaka and is an important book on

Ayurveda and medicine

He was a practitioner of the traditional

system of Indian medicine known as

Ayurveda

Charaka is thought to have flourished

sometime between the 2nd century BCE

and the 2nd century CE

Q6) Answer b

Explanation

Bhaga refers to the tax on crops which

was fixed at 16th of the production

Kammakaras is the term used for the

landless agricultural labour class

Ashvamedha also known as horse

sacrifice is a ritual where a horse is let

loose to wander freely and it was

guarded by the rajarsquos men

Q7) Answer (d)

Explanation

In the Rigvedic period horses were

yoked to chariots that were used in

battles fought to capture land cattle

etc This suggests that the use of horse

chariots began much before the period

of Mahajanapadas

The battles were fought in the Rigvedic

period for cattlersquos lands water an even

to capture people Most men took part

in these wars however there was no

regular army but there were assemblies

where people met and discussed

matters of war Regular armies became

a feature in the Mjahajanapada period

including vast armies of foot soldiers

chariots and elephants

RAUSIAS-FC19E1003 42

Q8) Answer (a)

Explanation

Buddha belonged to the Sakya clan and

passed away at Kusinara

Buddha taught in Prakrit which was the

common language of people

Q9) Answer c

Explanation

There were six schools of philosophy in

ancient India These are known as

Vaishesika Nyaya Samkhya Yoga

Purva Mimansa and Vedanata or Uttara

Mimansa They were founded by sages

Kanada Gautama Kapila Patanjali

Jamini and Vyasa respectively

Q10) Answer b

Explanation

The teachings of Mahavira were

compiled at Valabhi in 6th century AD

Q11) Answer (c)

Explanation

Chanakya is traditionally identified as

Kautilya or Vishnugupta who authored

the ancient Indian political treatise the

Arthashastra

Q12) Answer d

The national emblem of India is an

adaptation of the Lion Capital atop the

Ashoka Pillar of Sarnath Uttar Pradesh

and is combined with the National

Motto Satyameva Jayate

The Rampurva Bull gets the name from

the site of its discovery Rampurva in

Bihar

It is noted for its delicately sculpted

model demonstrating superior

representation of soft flesh sensitive

nostrils alert ears and strong legs It is

a mixture of Indian and Persian

elements

Sankissa is situated in Uttar Pradesh

India

Q13) Ans(a)

Kunwar Singh was a notable leader during the Revolt of 1857 He belonged

to a royal house of Jagdispur Bihar

Q14) Answer b

Explanation

The term Vellalar was used for large

landowners

Q15) Answer c

Explanation

Arikamedu was a coastal settlement

where ships unloaded goods from

distant lands Finds here include a

massive brick warehouse pottery

including amphorae and Arretine ware

Roman lamps glassware and gems have

also been found at the site

Q16) Answer a

Explanation

Muvendar is a Tamil word mentioned in

Sangam poems meaning three chiefs

used for the heads of three ruling

families the Cholas Cheras and

Pandyas

Q17) Ans (c)

Several tribal or kin-based assemblies

such as the Sabha Vidatha and gana

are mentioned in the Rig-veda The

Sabha and the samiti mattered a great

deal in early Vedic times so much so

that the chiefs or the kings showed an

eagerness to win their support

Q18) Ans (a)

Jainism recognised the existence of the

gods but placed them lower than the

jina and did not condemn the varna

system as Buddhism did

Q19) Answer (d)

Explanation

Cholas and Pandyas had developed

powerful coastal cities The most

important city of Cholas was Puhar or

Kaveripattinam and Madurai was the

capital of Pandyas

Q20) Answer b

Explanation

Buddhacharita is the biography of

Buddha and was written by

RAUSIAS-FC19E1003 43

Ashvaghosha

Q21) Answer (a)

Explanation

Tamil poet Appar was a Shiva devotee

So he was a Nayanar saint

Q22) Answer d

Explanation

Samudragupta was a prominent Gupta

ruler whose coins depict him playing a

veena indicating his love for music We

get important historic information from

his Allahabad Prashasti which was

composed by his court poet Harisena

Q23) Answer (b)

Explanation

Vikrama Samvat was founded by

Chandragupta II in the 58 BC as a

mark of victory over the Shakas and

assumed the title of Vikramaditya

Banabhatta wrote Harshavardhanarsquos

biography the Harshacharita in

Sanskrit

Q24) Answer c

Explanation

Sandhi-vigrahika was the minister of

war and peace

Sarthavaha was the leader of the

merchant caravans

Q25) Answer a

Explanation

Xuan Zang (Hsuan-tsang) was a

Chinese traveller who came during the

reign of Harshavardhana

In the decade that began in 630 AD

Xuan Zang came to India through

Kashmir after visiting Central Asia Iran

and Afghanistan

He travelled from north to east and lived

in Bihar for a couple of years

At Nalanda University Xuan Zang

interacted with students and scholars

mastered local languages and

discovered Buddhist stupas

Q26) Answer c

Explanation

Pradakshina patha is a circular path

laid around a stupa in Buddhist

architecture While the rest are a part of

temple architecture

Q27) Answer d

Explanation

All the above-mentioned temples have

an elaborate use of bricks (baked

bricks) along with stone

Q28) Ans (c)

Muhammad Quli Qutab was the Sultan

of Golconda He was a contemporary of

Akbar was very fond of literature and

architecture

The Sultan was a great poet and he

wrote in Dakhini Urdu Persian and

Telgu and has left an extensive diwan or

collection

Recently the Archaeological Survey of

India (ASI) will be using Ground

Penetrating Radar (GPR) to map the

contours of the area around the Bagh-e-

Naya Qila excavated garden inside the

Golconda Fort in Telangana

Q29) Answer a

Explanation

Silappadikaram is a famous Tamil epic

which was written by Ilango around

1800 years ago It is a story of a

merchant named Kovalan who fell in

love with a courtesan named Madhavi

Manimekalai tells the story of the

daughter of Kovalan and Madhavi

Q30) Answer (a)

Explanation

Charaka is the author of Charaka

Samhita which is an important work of

Ayurveda and medicines

Brahmaguptarsquos fame rests mostly on his

Brahma-sphuta-siddhanta which was

an astronomical work It was translated

into Arabic in Baghdad and had a major

impact on Islamic mathematics and

astronomy

Late in his life Brahmagupta wrote

Khandakhadyaka which was an

RAUSIAS-FC19E1003 44

astronomical handbook that employed

Aryabhatarsquos system of starting each day

at midnight

Q31) Answer (c)

Explanation

Amir Khusrau was a famous sufi

musician poet and scholar In 1318 he

noted that there was different language

in every region of this land (Hindustan)

Lahori Kashmiri Dvarsamudri (in

Southern Karnataka) Telangana (in

Andhra Pradesh) Gujari (in Gujarat)

Marsquobari (in Tamil Nadu) Awadhi (in

eastern Uttar Pradesh) and Hindawai (in

the area around in Delhi) etc He went

to explain that Sanskrit did not belong

to any region and that only brahmans

knew it

Q32) Answer c

Explanation

Hiranyagarbha refers to the golden

womb When this ritual was performed

with the help of Brahmanas it was

thought to lead to the rebirth of the

sacrificer as a Khastriya

Q33) Answer d

Explanation

Kadamai refers to a tax on land

revenue

Gwalior Prashasti describes the exploits

of Nagabhata who was a Pratihara king

Q34) Answer b

Explanation

Rajatarangini is a Sanskrit text written

by Kalhana in the 12th century

It was historical chronicle of early India

It is justifiably considered to be the best

and most authentic work of its kind

It covers the entire span of history in

the Kashmir region from the earliest

times to the date of its composition

Q35) Answer c

Explanation

ldquoUrrdquo was the general assembly of the

village ldquoUrrdquo consisted of all the

taxpaying residents of an ordinary

village

Q36) Answer (a)

Explanation

Tarikh was a form of history writing in

the Delhi Sultanate The authors of

tawarikhs were learned men which

included secretaries administrators etc

Q37 Answer (a)

Explanation

Alauddin chose to pay his soldiers salaries in cash rather than iqtas The soldiers would buy their supplies from merchants in Delhi and it was thus feared that merchants would raise their prices To stop this Alauddin controlled the prices of goods in Delhi Prices were carefully surveyed by officers and merchants who did not sell at the prescribed rates were punished

Q38) Answer (d)

Explanation

Delhi first became the capital of a

kingdom under the Tomara Rajputs

who were defeated in the middle of the

twelfth century by the Chauhans (also

referred to as Chahamanas) of Ajmer

It was under the Tomaras and

Chauhans that Delhi became an

important commercial centre Many rich

Jaina merchants lived in the city and

constructed several temples Coins

minted here called dehliwal had a wide

circulation

Q39) Answer (c)

Explanation

Moth ki Masjid was built in the reign of

Sikandar Lodi by his minister

Begumpuri mosque built in the reign of

Muhammad Tughluq was the main

mosque of Jahanpanah the ldquoSanctuary

of the Worldrdquo and his new capital in

Delhi

Quwwat al ndash Islam mosque was

enlarged by Iltutmish and Alauddin

Khalji The minar was built by three

Sultansndash Qutbuddin Aybak Iltutmish

and Firuz Shah Tughluq

RAUSIAS-FC19E1003 45

Q40) Answer (c)

Explanation

Under the Mughals mansabdar was

referred to an individual who held a

mansab ie rank and he received his

salary as revenue assignments called

jagirs

Q41) Ans (b)

The Quit India Movement was a

spontaneous revolt of people against

British rule

The All India Congress Committee met

at Bombay on 8 August 1942 It passed

the famous resolution Quit India and

proposed the starting of a non-violent

mass struggle under Gandhis

leadership to achieve this aim But on

the very next day Gandhi and other

eminent leaders of the Congress were

arrested The Congress was once again

declared illegal

Q42) Ans (c)

The Simon Commission refers to a

group of seven MPs from the United

Kingdom constituted to suggest

constitutional reforms for British India

The Commission consisted of only

British members headed by one of the

senior British politicians Sir John

Simon

So the people of India agitated against

the arrival of Simon Commission

Q43) Ans (a)

He was widely known for his

unfavourable opinion of the economic

consequences of the British rule in

India

In his many writings and speeches and

especially in Poverty and Un-British

Rule in India Naoroji argued that India

was too highly taxed and that its wealth

was being drained away to England

He did not interpret the ancient Indian

texts and restored the self-confidence of

Indians And also he did not stress the

need for eradication of all the social

evils before anything else

Q44) Ans (c)

In August 1932 Prime Minister

MacDonald announced his Communal

Award Great Britainrsquos unilateral

attempt to resolve the various conflicts

among Indiarsquos many communal

interests

The award which was later

incorporated into the act of 1935

expanded the separate-electorate

formula reserved for Muslims to other

minorities including Sikhs Indian

Christians Anglo-Indians Europeans

distinct regional groups Gandhi

undertook a ldquofast unto deathrdquo against

that offer which he viewed as a

nefarious British plot to divide the

Indian society

Q45) Ans (b)

In British India apart from existing

imports and exports there was also a

particular amount of money which

colonial India contributed towards

administration maintenance of the

army war expenses pensions to retired

officers and other expenses accrued by

Britain towards maintenance of her

colony These were known as Home

charges and were paid for almost

entirely by India

The Home charges was made of

following components-

- Interest payable on Indian debt

- Dividend to shareholders of East

India Company

- Funds used to support the India

Office in London

- Funds used to pay salaries and

pensions of British personnel

engaged in India

- Interest on the railways

- Civil and military charges

- Store purchases in England

Q46) Ans (b)

The Lahore session of the Indian

National Congress was held in 1929

under the Presidentship of Jawaharlal

Nehru

The Lahore session of the Indian

National Congress witnessed significant

RAUSIAS-FC19E1003 46

developments in the Indian national

movement

- First the election of Jawaharlal

Nehru to the post of Presidentship of

the Congress was a clear indication

of the growing strength of the

Leftists in the Congress

- Secondly it was in this session that

the Congress for the first time raised

the demand for complete

independence Such demand was

not raised from the Congress

platform earlier

Q47) Ans (b)

It did not provide for separate

electorates for any community or

weightage for minorities However it did

allow for the reservation of minority

seats in provinces having minorities of

at least ten per cent but this was to be

in strict proportion to the size of the

community

There was no provision for complete

Independence for India

Q48) Ans (c)

The religion of early Vedic Aryans was

primarily of worship of nature and

Yajnas

The early Aryan religion was kind of

nature worship Actually the forces

around them which they could not

control or understand were invested

with divinity and were personified as

male or female gods And they

performed some Yajnas also

Q49) Ans (b)

The roads and river-routes were not

immune from robbery It is notable that

Yuan Chwang (Hiuen Tsang) was

robbed of his belongings during

Harshvardanarsquos period

Q50) Ans (c)

Q51) Ans (b)

Purandara Dasa was a saint and great

devotee of Lord Krishna

There is much speculation about where

Purandara Dasa regarded as the

Pitamaha of Carnatic music was born

Recently an expert committee

constituted by the Kannada University

Hampi has come to the conclusion that

Kshemapura Shivamogga district

Karnataka is the birth place of

Purandara Dasa

Q52) Ans (c)

Sri Tyagaraja Sri Shyama Shastry and Sri Muthuswami Dikshitar are considered the trinity of Carnatic music and with them came the golden age in Carnatic music in the 18th-19th

century

Q53) Ans d)

Recently a rare sarcophagus (stone

coffin) which is 2000 years old from the

Iron AgendashMegalithic era was discovered

from a rock-cut cave at Viyur village of

Kollam near Koyilandy in Kozhikode

district Kerala

The coffin containing bone fragments

was found during an excavation ldquoSo

far such a rare finding has been

discovered only from two sites

in Kerala Both these sarcophagi were

recovered from Megalithic sites at

Chevayur and Atholi also in Kozhikode

district

Q54) Ans a)

The megalithic culture in South India was a full-fledged Iron Age culture

Q55) Ans d)

The Cholas Pandyas and Keralaputras

(Cheras) mentioned in Ashokan

inscriptions were probably in the late

megalithic phase of material culture

Q56) Ans d)

Q57) Ans (b)

Raj Kumar Shukla followed Gandhiji all

over the country to persuade him to

come to Champaran to investigate the

problem associated with tinkathia

system

RAUSIAS-FC19E1003 47

Brij Kishore Rajendra Prasad Mahadev

Desai and Narhari Parikh accompanied

Gandhi ji during the Champaran

Satyagraha

Q58) Ans (b)

The Satvahanas started the practice of granting tax-free villages to brahmanas and Buddhist monks

Q59) Ans c)

The objectives of the Programme are

listed as under

- Developing basic tourism

infrastructure

- Promoting cultural and heritage

value of the country to generate

livelihoods in the identified regions

- Enhancing the tourist attractiveness

in a sustainable manner by

developing world-class

infrastructure at the heritage

monument sites

- Creating employment through active

involvement of local communities

- Harnessing tourism potential for its

effects on employment generation

and economic development

- Developing sustainable tourism

infrastructure and ensuring proper

Operations and maintenance

therein

Q60) Ans (b)

The Tribal Cooperative Marketing

Development Federation of India

(TRIFED) came into existence in 1987

It is a national-level apex organization

functioning under the administrative

control of Ministry of Tribal Affairs

Govt of India

TRIFED has its registered and Head

Office located in New Delhi

Q61) Ans (c)

Premchandrsquos novels include

Premashram Rangabhumi Ghaban

Karmabhumi and Godan

Gora is a novel written by Rabindranath

Tagore

138th birth anniversary of Munshi

Premchand was celebrated across the

country

Q62) Ans (b)

Giddha is a traditional pastoral dance

performed by the women of the Punjab

India and Pakistan at festival times

and at the sowing and reaping of the

harvest

By this dance the Punjabi women

reveal their joy expel their suppressed

feelings in a male dominated society

through the performance of Giddha

Since this dance has nothing to do with

men only women can participate in it

During the Teej celebrations Giddha

dance is celebrated in Punjab every

year Teej is a generic name for a

number of festivals that are celebrated

by women in some parts of India

Q63) Ans (a)

Dara Shukoh wrote the remarkable

work called ldquoMajma-ul-Bahrainrdquo or the

ldquoThe confluence of two seasrdquo

The Vice President of India Shri M

Venkaiah Naidu has said that Prince

Dara Shukohrsquos writings can come as a

refreshing source for infusing peace and

harmony He was addressing the

gathering after visiting the exhibition

that showcases the forgotten Prince of

yesteryears Dara Shukoh organized by

Mr Francois Gautier at Indira Gandhi

National Centre for the Arts in New

Delhi

Q64) Ans (c)

The statue Gommateshwara is

dedicated to the Jain God Bahubali

It is a monolithic statue

President Ram Nath Kovind

inaugurated the grand anointing

ceremony mdash Mahamastakabhisheka mdash

held once in 12 years at

Shravanabelagola (Karnataka)

Q65) Ans (c)

Prachi Valley had come up around the

Prachi river Prachi Valley gradually

disappeared

RAUSIAS-FC19E1003 48

The Prachi river originates from

Bhubaneswar

It is a tributary of the Mahanadi and

flows through the districts of Puri

Khurda Cuttack and Jagatsinghpur

and the entire region of the river is

termed as the Prachi Valley

It falls into the Bay of Bengal

Archaeological evidence shows that the

Prachi Valley Civilisation predates both

Harappa and Mohenjo-Daro

The Prachi river originates from

Bhubaneswar

Q66) Ans (d)

These monuments are located in

Chhatarpur district Madhya Pradesh

within Vindhya mountain range

Q67) Ans (a)

The book lsquoThoughts on Pakistanrsquo was

written by Dr BR Ambedkar

On the occasion of the birth anniversary

of Dr BR Ambedkar the president of

India pays homage to this icon of India

In 1924 he founded the Depressed

Classes Institute (Bahishkrit Hitkarini

Sabha) and in 1927 the Samaj Samata

Sangh

Another area of attention for Ambedkar

was education For its spread among

the low classes he set up a network of

colleges by the name of Peoples

Education Society and founded hostels

Q68) Ans(b)

Mehrgarh is a famous Neolithic

settlement in the Indian subcontinent

which is situated in Baluchistan

province Pakistan

A pre-historic rock art site is discovered

in the vast expanse of limestone blocks

on the eastern banks of Naguleru river

near Dachepalli (Andhra Pradesh) It

has thrown light on the Neolithic

civilisation that flourished in Guntur

(Andhra Pradesh) during 1500-2000

BC

Q69) Ans (c)

The 12th and the 13th centuries saw

the emergence of the Kakatiyas They

were at first the feudatories of the

Western Chalukyas of Kalyana Initially

they ruled over a small territory near

Warangal (Telangana)

They introduced Nayakships which was

later adopted and developed by the

Rayas of Vijayanagara

Q70) Ans (a)

The fast had effect of putting pressure

on mill owners who finally agreed to

give the workers a 35 per cent increase

in wages

Google celebrated with a doodle the

132nd birth anniversary of Anasuya

Sarabhai who played a pioneering role

in Indiarsquos labour movement

Q71) Ans (d)

The UNESCOrsquos list of the representative

list of the intangible cultural heritage of

humanity from India are

- Koodiyattam Sanskrit Theatre of

Kerala

- Mudiyettu ritual theatre and dance

drama of Kerala

- Tradition of Vedic Chanting

- Kalbelia folk songs and dances of

Rajasthan

- Ramlila Traditional Performance of

the Ramayana

- Sankirtana ritual singing

drumming and dancing of Manipur

- Ramman religious festival and

ritual theatre of the Garhwal

Himalayas India

- Traditional brass and copper craft of

utensil making among the Thatheras

of Jandiala Guru Punjab India

- Chhau dance classical Indian dance

originated in the eastern Indian

states

- Buddhist chanting of Ladakh

recitation of sacred Buddhist texts

in the trans-Himalayan Ladakh

region Jammu and Kashmir India

- Yoga

- Nouroz

- Kumbh Mela

RAUSIAS-FC19E1003 49

Q72) Ans(b)

The President of India Shri Ram Nath Kovind inaugurated the Hornbill Festival and State Formation Day celebrations of Nagaland in Kisama

The festival is named after the Indian hornbill the large and colourful forest bird which is displayed in the folklore of most of the states tribes

The major recognized tribes of Nagaland are Angami Ao Chakhesang Chang

Kuki Rengma and Zeling etc

Onge Jarawa and Sentinelese are the

tribes of Andman amp Nicobar Islands

Q73) Ans (c)

The Rashtrakutas rule in the Deccan lasted for almost two hundred years till the end of the tenth century The Rashtrakutas rulers were tolerant in their religious views and patronized not only Shaivism and Vaishnavism but

Jainism as well

The famous rock-cut temple of Shiva at Ellora was built by one of the Rashtrakutas kings Krishna I in the ninth century His successor Amoghavarsha was a Jain but he also

patronized other faiths

The Rashtrakutas allowed Muslims traders to settle and permitted Islam to

be preached in their dominions

Recently increasing defacement at the prehistoric rock paintings of Pandavulagutta Telangana has created a cause for grave concern It can spoil

the prehistoric rock

Pandavulagutta is home to

- Painted rock shelters dating to

10000 BC-8000 BC

- An 8th century inscription of the

Rashtrakuta period and

- Painted frescoes from the 12th century Kakatiya empire

Q74) Ans (b)

In 1828 Raja Ram Mohan Roy founded a new religious society the Brahma Sabha later known as the Brahmo

Samaj

Debendranath Tagore headed the Tattvabodhini Sabha which was

engaged in search of spiritual truth

Its purpose was to purify Hinduism and to preach monotheism or belief in one God

The new society was to be based on the twin pillars of reason and the Vedas and

Upanishads

Recently Sadharan Brahmo Samaj (SBS) has entered into a legal battle with the West Bengal government due

to some legal issue

Q75) Ans (c)

The Chishti order was established in India by Khwaja Moinuddin Chishti who came to India around 1192 The Chishtirsquos are considered to be the most influential of the groups of Sufis who migrated to India in the late twelfth century They adapted successfully to the local environment and adopted several features of Indian devotional

traditions

The historical dargah of Sufi mystic Khwaja Moinuddin Chishti in Ajmer is all set to get a facelift This 13 th century dargah has been included among the Swachh Iconic Places a clean-up initiative focused on iconic

heritage spiritual and cultural places

Page 39: GENERAL STUDIES (PAPER I) · Test is part of Rau’s IAS Test series for Preliminary Exam 2019 FOUNDATION + CURRENT AFFAIRS GENERAL STUDIES (PAPER –I) FOUNDATION TEST –III TOPIC:

FC19H1003 39

Q68) उततर (b)

सपषटीकरण

bull महरगि भारतीय उपमहादवीप म एक परवसद

निपािाण बसती ह ज बलवचसतान पराोत

पावकसतान म सथथत ह

bull दचपलली (आोधर परदश) क पास नागलर नदी क

पिी ति ो पर चना पतथर क बलॉक क विशाल

विसतार म एक पिि-ऐवतहावसक रॉक आिि थथल की

ख ज की गई ह

bull इसन 1500-2000 ईसा पिि क दौरान गोिर (आोधर

परदश) म विकवसत निपािाण सभयता पर परकाश

डाला ह

Q69) उततर (c)

सपषटीकरण

bull 12िी ो सदी और 13िी ो सदी म काकाविय िोश का

उदय हआ था

bull ि पहल कलयाण क पवशचमी चालकय ो क सामोत थ

bull परारोभ म उनह ोन िारोगल (तलोगाना) क पास एक

छ ि स कषतर पर शासन वकया था

bull उनह ोन ldquoनायक वयिथथाrdquo की शरआत की थी

वजस बाद म विजयनगर क राय शासक ो न

अपनाया और विकवसत वकया था

Q70) उततर (a)

सपषटीकरण

bull गाोिीजी क अनशन स वमल मावलक ो पर दबाि

पडा था ज अोततः शरवमक ो क ितन म 35 परवतशत

की िसद करन क वलए सहमत हए थ

bull गगल (Google) न अनसया साराभाई वजनह ोन

भारत क शरवमक आोद लन म एक अगरणी भवमका

वनभाई थी की 132िी ो जयोती डडल (Doodle) का

वनमािण करक मनाई

Q71) उततर (d)

सपषटीकरण

भारत स यनसक की मानिता की अमति साोसकवतक

विरासत की परवतवनवि सची म वनमनवलसखत शावमल ह

bull कवडयटटम करल का सोसकत रोगमोच

bull मवडयिि करल का अनषठान रोगमोच और नतय

नाविका

bull िवदक मि जाप की परोपरा

bull राजथथान क कालबवलया ल क गीत और नतय

bull रामलीला रामायण का पारोपररक परदशिन

bull सोकीतिन मवणपर का अनषठान गायन ढ ल िादन

और नतय

bull रममन भारत क गििाल वहमालय का िावमिक

तयौहार और अनषठान रोगमोच

bull जाोदीयाला गर पोजाब क ठठर ो की पीतल और

ताोब क वशलप स वनवमित बतिन ो की पारोपररक कला

bull छाऊ नतय पिी भारतीय राजय ो म जनमी शासतरीय

भारतीय नतय कला

bull लददाख का बौद मि जाप िर ाोस-वहमालयी लददाख

कषतर तथा जमम-कशमीर म पवितर बौद गरोथ ो का पाठ

bull य ग

bull नौर र

bull को भ मला

Q72) उततर (b)

सपषटीकरण

bull भारत क राषटर पवत शरी राम नाथ क विोद न

वकसामा नागालड म हॉनिवबल मह रति और

राजय गठन वदिस समार ह का उदघािन वकया

था

bull हॉनिवबल मह रति का नाम भारतीय हॉनिवबल क

नाम पर पडा ह ज एक विशाल और रोगीन जोगली

पकषी ह

bull यह पकषी नागालड राजय की अविकतर जनजावतय ो

की ल ककथाओो म उसललसखत ह

bull नागालड की परमख मानयता परापत जनजावतयाा ह

अोगामी आओ चखसोग चाोग ककी रगमा और

रवलोग आवद

bull ओोग जारिा और ससिनलीस अोडमान-वनक बार

दवीप समह की जनजावतयाा ह

FC19H1003 40

Q73) उततर (c)

सपषटीकरण

bull दकन म राषटर कि शासन दसिी ो सदी क अोत तक

लगभग 200 ििो तक रहा था

bull राषटर कि शासक अपन िावमिक विचार ो म सवहषण

bull उनह ोन न किल शि िमि और िषणि िमि बसलक

जन िमि क भी सोरकषण वदया था

bull एल रा म वशि क परवसद रॉक कि मोवदर का

वनमािण नौिी ो सदी म राषटर कि राजा कषण परथम न

करिाया था

bull उसका उततराविकारी अम घििि जन था लवकन

उसन अनय िमो क भी सोरकषण परदान वकया था

bull राषटर कि ो न मसलमान वयापाररय ो क बसन की

अनमवत दी थी

bull उनह न अपन अविराजय ो म इसलाम क उपदश दन

की भी अनमवत दी थी

bull अभी हाल ही म पाोडिलागटटा (तलोगाना) क

परागवतहावसक चटटान वचतर ो क कषरण की बिती हई

घिनाएा एक गोभीर वचोता का वििय ह

bull यह परागवतहावसक चटटान क नकसान पहाचा

सकता ह

bull पाोडिलागटटा वनमनवलसखत क वलए जाना जाता ह

- 10000 ईसा पिि स 8000 ईसा पिि क वचवतरत

चटटानी आशरय ो क वलए

- राषटर कि काल क एक 8 िी ो सदी क

वशलालख क वलए और

- 12िी ो सदी क काकविय सामराजय क वभवतत

वचतर ो क वलए

Q74) उततर (b)

सपषटीकरण

bull 1828 म राजा राम म हन रॉय न एक नय िावमिक

समाज बरहम सभा की थथापना की थी वजस बाद

म बरहम समाज क नाम स जाना गया था

bull दिदरनाथ िग र न ततवब विनी सभा की अधयकषता

की थी ज आधयासिक सतय की ख ज म सोलि

थी

bull इसका उददशय वहोद िमि क शद करन का और

एकशवरिाद (एक ईशवर म आथथा) का परचार करना

था

bull नय समाज की थथापना क आिार थ कारण

(तकि ) क द सतमभ तथा िद और उपवनिद

bull अभी हाल ही म सािारण बरहम समाज का कछ

काननी मदद ो क लकर पवशचम बोगाल सरकार क

साथ काननी वििाद चल रहा ह

Q75) उततर (c)

सपषटीकरण

bull भारत म वचशती वसलवसल की थथापना खवाजा

म इनददीन वचशती क दवारा की गयी थी

bull ि 1192 ईसवी क आसपास भारत आय थ

bull वचशतीय ो क बारहिी ो शताबदी क उततरािि म भारत

म आन िाल सफीय ो क समह ो म सबस

परभािशाली माना जाता ह

bull उनह ोन थथानीय िातािरण क साथ सफलतापििक

अनकलन वकया और उनह ोन भारतीय भसकत

परोपराओो क कई पहलओो क अपनाया

bull अजमर म सफी अपरकि खवाजा म इनददीन वचशती

की ऐवतहावसक दरगाह क एक नया रप दन की

तयारी की जा रही ह

bull इस 13िी ो शताबदी की दरगाह क ldquoसवचछ

आइकॉवनक थथल ोrdquo (Swacch Iconic Places) म

शावमल वकया गया ह ज परवतवषठत विरासत

आधयासिक और साोसकवतक थथान ो पर क वदरत

य जना ह

FC19H1003 41

ANSWERS amp EXPLANATION OF

NCERT History Class VI-X + Current Affairs

(FC19E1003)

Q1) Answer c

Explanation

Rigveda consists of more than a

thousand hymns dedicated to gods and

goddesses These hymns were

composed by sages and learnt by men

however a few were composed by

women like Apala Ghosa Lopamudra

Maitreyi and Gargi

Rigveda consists of many hymns in the

form of dialogues We get an example of

a dialogue between a sage named

Vishwamitra and two rivers (Beas and

Sutlej) that were worshipped as

goddesses This suggests that he

belonged to the Vedic period

Q2) Answer b

Explanation

Traces of ash have been found from

Kurnool Caves suggesting that people

were familiar with the use of fire

It is situated in Andhra Pradesh

Q3) Answer c

Explanation

Burzahom is a prehistoric site in

present day Kashmir where people built

pit houses which were dug into the

ground with steps leading into them

These may have provided shelter in cold

weather

Q4) Answer c

Explanation

Epigraphy is defined as the study of

inscriptions

Manuscriptology is the study of history

and literature through the use of hand

written documents

Palaeography refers to the study of

ancient writing systems and the

deciphering and dating of historical

manuscripts

Numismatics refers to the study of

coins

Q5) Answer a

Explanation

Charaka Samhita was written by

Charaka and is an important book on

Ayurveda and medicine

He was a practitioner of the traditional

system of Indian medicine known as

Ayurveda

Charaka is thought to have flourished

sometime between the 2nd century BCE

and the 2nd century CE

Q6) Answer b

Explanation

Bhaga refers to the tax on crops which

was fixed at 16th of the production

Kammakaras is the term used for the

landless agricultural labour class

Ashvamedha also known as horse

sacrifice is a ritual where a horse is let

loose to wander freely and it was

guarded by the rajarsquos men

Q7) Answer (d)

Explanation

In the Rigvedic period horses were

yoked to chariots that were used in

battles fought to capture land cattle

etc This suggests that the use of horse

chariots began much before the period

of Mahajanapadas

The battles were fought in the Rigvedic

period for cattlersquos lands water an even

to capture people Most men took part

in these wars however there was no

regular army but there were assemblies

where people met and discussed

matters of war Regular armies became

a feature in the Mjahajanapada period

including vast armies of foot soldiers

chariots and elephants

RAUSIAS-FC19E1003 42

Q8) Answer (a)

Explanation

Buddha belonged to the Sakya clan and

passed away at Kusinara

Buddha taught in Prakrit which was the

common language of people

Q9) Answer c

Explanation

There were six schools of philosophy in

ancient India These are known as

Vaishesika Nyaya Samkhya Yoga

Purva Mimansa and Vedanata or Uttara

Mimansa They were founded by sages

Kanada Gautama Kapila Patanjali

Jamini and Vyasa respectively

Q10) Answer b

Explanation

The teachings of Mahavira were

compiled at Valabhi in 6th century AD

Q11) Answer (c)

Explanation

Chanakya is traditionally identified as

Kautilya or Vishnugupta who authored

the ancient Indian political treatise the

Arthashastra

Q12) Answer d

The national emblem of India is an

adaptation of the Lion Capital atop the

Ashoka Pillar of Sarnath Uttar Pradesh

and is combined with the National

Motto Satyameva Jayate

The Rampurva Bull gets the name from

the site of its discovery Rampurva in

Bihar

It is noted for its delicately sculpted

model demonstrating superior

representation of soft flesh sensitive

nostrils alert ears and strong legs It is

a mixture of Indian and Persian

elements

Sankissa is situated in Uttar Pradesh

India

Q13) Ans(a)

Kunwar Singh was a notable leader during the Revolt of 1857 He belonged

to a royal house of Jagdispur Bihar

Q14) Answer b

Explanation

The term Vellalar was used for large

landowners

Q15) Answer c

Explanation

Arikamedu was a coastal settlement

where ships unloaded goods from

distant lands Finds here include a

massive brick warehouse pottery

including amphorae and Arretine ware

Roman lamps glassware and gems have

also been found at the site

Q16) Answer a

Explanation

Muvendar is a Tamil word mentioned in

Sangam poems meaning three chiefs

used for the heads of three ruling

families the Cholas Cheras and

Pandyas

Q17) Ans (c)

Several tribal or kin-based assemblies

such as the Sabha Vidatha and gana

are mentioned in the Rig-veda The

Sabha and the samiti mattered a great

deal in early Vedic times so much so

that the chiefs or the kings showed an

eagerness to win their support

Q18) Ans (a)

Jainism recognised the existence of the

gods but placed them lower than the

jina and did not condemn the varna

system as Buddhism did

Q19) Answer (d)

Explanation

Cholas and Pandyas had developed

powerful coastal cities The most

important city of Cholas was Puhar or

Kaveripattinam and Madurai was the

capital of Pandyas

Q20) Answer b

Explanation

Buddhacharita is the biography of

Buddha and was written by

RAUSIAS-FC19E1003 43

Ashvaghosha

Q21) Answer (a)

Explanation

Tamil poet Appar was a Shiva devotee

So he was a Nayanar saint

Q22) Answer d

Explanation

Samudragupta was a prominent Gupta

ruler whose coins depict him playing a

veena indicating his love for music We

get important historic information from

his Allahabad Prashasti which was

composed by his court poet Harisena

Q23) Answer (b)

Explanation

Vikrama Samvat was founded by

Chandragupta II in the 58 BC as a

mark of victory over the Shakas and

assumed the title of Vikramaditya

Banabhatta wrote Harshavardhanarsquos

biography the Harshacharita in

Sanskrit

Q24) Answer c

Explanation

Sandhi-vigrahika was the minister of

war and peace

Sarthavaha was the leader of the

merchant caravans

Q25) Answer a

Explanation

Xuan Zang (Hsuan-tsang) was a

Chinese traveller who came during the

reign of Harshavardhana

In the decade that began in 630 AD

Xuan Zang came to India through

Kashmir after visiting Central Asia Iran

and Afghanistan

He travelled from north to east and lived

in Bihar for a couple of years

At Nalanda University Xuan Zang

interacted with students and scholars

mastered local languages and

discovered Buddhist stupas

Q26) Answer c

Explanation

Pradakshina patha is a circular path

laid around a stupa in Buddhist

architecture While the rest are a part of

temple architecture

Q27) Answer d

Explanation

All the above-mentioned temples have

an elaborate use of bricks (baked

bricks) along with stone

Q28) Ans (c)

Muhammad Quli Qutab was the Sultan

of Golconda He was a contemporary of

Akbar was very fond of literature and

architecture

The Sultan was a great poet and he

wrote in Dakhini Urdu Persian and

Telgu and has left an extensive diwan or

collection

Recently the Archaeological Survey of

India (ASI) will be using Ground

Penetrating Radar (GPR) to map the

contours of the area around the Bagh-e-

Naya Qila excavated garden inside the

Golconda Fort in Telangana

Q29) Answer a

Explanation

Silappadikaram is a famous Tamil epic

which was written by Ilango around

1800 years ago It is a story of a

merchant named Kovalan who fell in

love with a courtesan named Madhavi

Manimekalai tells the story of the

daughter of Kovalan and Madhavi

Q30) Answer (a)

Explanation

Charaka is the author of Charaka

Samhita which is an important work of

Ayurveda and medicines

Brahmaguptarsquos fame rests mostly on his

Brahma-sphuta-siddhanta which was

an astronomical work It was translated

into Arabic in Baghdad and had a major

impact on Islamic mathematics and

astronomy

Late in his life Brahmagupta wrote

Khandakhadyaka which was an

RAUSIAS-FC19E1003 44

astronomical handbook that employed

Aryabhatarsquos system of starting each day

at midnight

Q31) Answer (c)

Explanation

Amir Khusrau was a famous sufi

musician poet and scholar In 1318 he

noted that there was different language

in every region of this land (Hindustan)

Lahori Kashmiri Dvarsamudri (in

Southern Karnataka) Telangana (in

Andhra Pradesh) Gujari (in Gujarat)

Marsquobari (in Tamil Nadu) Awadhi (in

eastern Uttar Pradesh) and Hindawai (in

the area around in Delhi) etc He went

to explain that Sanskrit did not belong

to any region and that only brahmans

knew it

Q32) Answer c

Explanation

Hiranyagarbha refers to the golden

womb When this ritual was performed

with the help of Brahmanas it was

thought to lead to the rebirth of the

sacrificer as a Khastriya

Q33) Answer d

Explanation

Kadamai refers to a tax on land

revenue

Gwalior Prashasti describes the exploits

of Nagabhata who was a Pratihara king

Q34) Answer b

Explanation

Rajatarangini is a Sanskrit text written

by Kalhana in the 12th century

It was historical chronicle of early India

It is justifiably considered to be the best

and most authentic work of its kind

It covers the entire span of history in

the Kashmir region from the earliest

times to the date of its composition

Q35) Answer c

Explanation

ldquoUrrdquo was the general assembly of the

village ldquoUrrdquo consisted of all the

taxpaying residents of an ordinary

village

Q36) Answer (a)

Explanation

Tarikh was a form of history writing in

the Delhi Sultanate The authors of

tawarikhs were learned men which

included secretaries administrators etc

Q37 Answer (a)

Explanation

Alauddin chose to pay his soldiers salaries in cash rather than iqtas The soldiers would buy their supplies from merchants in Delhi and it was thus feared that merchants would raise their prices To stop this Alauddin controlled the prices of goods in Delhi Prices were carefully surveyed by officers and merchants who did not sell at the prescribed rates were punished

Q38) Answer (d)

Explanation

Delhi first became the capital of a

kingdom under the Tomara Rajputs

who were defeated in the middle of the

twelfth century by the Chauhans (also

referred to as Chahamanas) of Ajmer

It was under the Tomaras and

Chauhans that Delhi became an

important commercial centre Many rich

Jaina merchants lived in the city and

constructed several temples Coins

minted here called dehliwal had a wide

circulation

Q39) Answer (c)

Explanation

Moth ki Masjid was built in the reign of

Sikandar Lodi by his minister

Begumpuri mosque built in the reign of

Muhammad Tughluq was the main

mosque of Jahanpanah the ldquoSanctuary

of the Worldrdquo and his new capital in

Delhi

Quwwat al ndash Islam mosque was

enlarged by Iltutmish and Alauddin

Khalji The minar was built by three

Sultansndash Qutbuddin Aybak Iltutmish

and Firuz Shah Tughluq

RAUSIAS-FC19E1003 45

Q40) Answer (c)

Explanation

Under the Mughals mansabdar was

referred to an individual who held a

mansab ie rank and he received his

salary as revenue assignments called

jagirs

Q41) Ans (b)

The Quit India Movement was a

spontaneous revolt of people against

British rule

The All India Congress Committee met

at Bombay on 8 August 1942 It passed

the famous resolution Quit India and

proposed the starting of a non-violent

mass struggle under Gandhis

leadership to achieve this aim But on

the very next day Gandhi and other

eminent leaders of the Congress were

arrested The Congress was once again

declared illegal

Q42) Ans (c)

The Simon Commission refers to a

group of seven MPs from the United

Kingdom constituted to suggest

constitutional reforms for British India

The Commission consisted of only

British members headed by one of the

senior British politicians Sir John

Simon

So the people of India agitated against

the arrival of Simon Commission

Q43) Ans (a)

He was widely known for his

unfavourable opinion of the economic

consequences of the British rule in

India

In his many writings and speeches and

especially in Poverty and Un-British

Rule in India Naoroji argued that India

was too highly taxed and that its wealth

was being drained away to England

He did not interpret the ancient Indian

texts and restored the self-confidence of

Indians And also he did not stress the

need for eradication of all the social

evils before anything else

Q44) Ans (c)

In August 1932 Prime Minister

MacDonald announced his Communal

Award Great Britainrsquos unilateral

attempt to resolve the various conflicts

among Indiarsquos many communal

interests

The award which was later

incorporated into the act of 1935

expanded the separate-electorate

formula reserved for Muslims to other

minorities including Sikhs Indian

Christians Anglo-Indians Europeans

distinct regional groups Gandhi

undertook a ldquofast unto deathrdquo against

that offer which he viewed as a

nefarious British plot to divide the

Indian society

Q45) Ans (b)

In British India apart from existing

imports and exports there was also a

particular amount of money which

colonial India contributed towards

administration maintenance of the

army war expenses pensions to retired

officers and other expenses accrued by

Britain towards maintenance of her

colony These were known as Home

charges and were paid for almost

entirely by India

The Home charges was made of

following components-

- Interest payable on Indian debt

- Dividend to shareholders of East

India Company

- Funds used to support the India

Office in London

- Funds used to pay salaries and

pensions of British personnel

engaged in India

- Interest on the railways

- Civil and military charges

- Store purchases in England

Q46) Ans (b)

The Lahore session of the Indian

National Congress was held in 1929

under the Presidentship of Jawaharlal

Nehru

The Lahore session of the Indian

National Congress witnessed significant

RAUSIAS-FC19E1003 46

developments in the Indian national

movement

- First the election of Jawaharlal

Nehru to the post of Presidentship of

the Congress was a clear indication

of the growing strength of the

Leftists in the Congress

- Secondly it was in this session that

the Congress for the first time raised

the demand for complete

independence Such demand was

not raised from the Congress

platform earlier

Q47) Ans (b)

It did not provide for separate

electorates for any community or

weightage for minorities However it did

allow for the reservation of minority

seats in provinces having minorities of

at least ten per cent but this was to be

in strict proportion to the size of the

community

There was no provision for complete

Independence for India

Q48) Ans (c)

The religion of early Vedic Aryans was

primarily of worship of nature and

Yajnas

The early Aryan religion was kind of

nature worship Actually the forces

around them which they could not

control or understand were invested

with divinity and were personified as

male or female gods And they

performed some Yajnas also

Q49) Ans (b)

The roads and river-routes were not

immune from robbery It is notable that

Yuan Chwang (Hiuen Tsang) was

robbed of his belongings during

Harshvardanarsquos period

Q50) Ans (c)

Q51) Ans (b)

Purandara Dasa was a saint and great

devotee of Lord Krishna

There is much speculation about where

Purandara Dasa regarded as the

Pitamaha of Carnatic music was born

Recently an expert committee

constituted by the Kannada University

Hampi has come to the conclusion that

Kshemapura Shivamogga district

Karnataka is the birth place of

Purandara Dasa

Q52) Ans (c)

Sri Tyagaraja Sri Shyama Shastry and Sri Muthuswami Dikshitar are considered the trinity of Carnatic music and with them came the golden age in Carnatic music in the 18th-19th

century

Q53) Ans d)

Recently a rare sarcophagus (stone

coffin) which is 2000 years old from the

Iron AgendashMegalithic era was discovered

from a rock-cut cave at Viyur village of

Kollam near Koyilandy in Kozhikode

district Kerala

The coffin containing bone fragments

was found during an excavation ldquoSo

far such a rare finding has been

discovered only from two sites

in Kerala Both these sarcophagi were

recovered from Megalithic sites at

Chevayur and Atholi also in Kozhikode

district

Q54) Ans a)

The megalithic culture in South India was a full-fledged Iron Age culture

Q55) Ans d)

The Cholas Pandyas and Keralaputras

(Cheras) mentioned in Ashokan

inscriptions were probably in the late

megalithic phase of material culture

Q56) Ans d)

Q57) Ans (b)

Raj Kumar Shukla followed Gandhiji all

over the country to persuade him to

come to Champaran to investigate the

problem associated with tinkathia

system

RAUSIAS-FC19E1003 47

Brij Kishore Rajendra Prasad Mahadev

Desai and Narhari Parikh accompanied

Gandhi ji during the Champaran

Satyagraha

Q58) Ans (b)

The Satvahanas started the practice of granting tax-free villages to brahmanas and Buddhist monks

Q59) Ans c)

The objectives of the Programme are

listed as under

- Developing basic tourism

infrastructure

- Promoting cultural and heritage

value of the country to generate

livelihoods in the identified regions

- Enhancing the tourist attractiveness

in a sustainable manner by

developing world-class

infrastructure at the heritage

monument sites

- Creating employment through active

involvement of local communities

- Harnessing tourism potential for its

effects on employment generation

and economic development

- Developing sustainable tourism

infrastructure and ensuring proper

Operations and maintenance

therein

Q60) Ans (b)

The Tribal Cooperative Marketing

Development Federation of India

(TRIFED) came into existence in 1987

It is a national-level apex organization

functioning under the administrative

control of Ministry of Tribal Affairs

Govt of India

TRIFED has its registered and Head

Office located in New Delhi

Q61) Ans (c)

Premchandrsquos novels include

Premashram Rangabhumi Ghaban

Karmabhumi and Godan

Gora is a novel written by Rabindranath

Tagore

138th birth anniversary of Munshi

Premchand was celebrated across the

country

Q62) Ans (b)

Giddha is a traditional pastoral dance

performed by the women of the Punjab

India and Pakistan at festival times

and at the sowing and reaping of the

harvest

By this dance the Punjabi women

reveal their joy expel their suppressed

feelings in a male dominated society

through the performance of Giddha

Since this dance has nothing to do with

men only women can participate in it

During the Teej celebrations Giddha

dance is celebrated in Punjab every

year Teej is a generic name for a

number of festivals that are celebrated

by women in some parts of India

Q63) Ans (a)

Dara Shukoh wrote the remarkable

work called ldquoMajma-ul-Bahrainrdquo or the

ldquoThe confluence of two seasrdquo

The Vice President of India Shri M

Venkaiah Naidu has said that Prince

Dara Shukohrsquos writings can come as a

refreshing source for infusing peace and

harmony He was addressing the

gathering after visiting the exhibition

that showcases the forgotten Prince of

yesteryears Dara Shukoh organized by

Mr Francois Gautier at Indira Gandhi

National Centre for the Arts in New

Delhi

Q64) Ans (c)

The statue Gommateshwara is

dedicated to the Jain God Bahubali

It is a monolithic statue

President Ram Nath Kovind

inaugurated the grand anointing

ceremony mdash Mahamastakabhisheka mdash

held once in 12 years at

Shravanabelagola (Karnataka)

Q65) Ans (c)

Prachi Valley had come up around the

Prachi river Prachi Valley gradually

disappeared

RAUSIAS-FC19E1003 48

The Prachi river originates from

Bhubaneswar

It is a tributary of the Mahanadi and

flows through the districts of Puri

Khurda Cuttack and Jagatsinghpur

and the entire region of the river is

termed as the Prachi Valley

It falls into the Bay of Bengal

Archaeological evidence shows that the

Prachi Valley Civilisation predates both

Harappa and Mohenjo-Daro

The Prachi river originates from

Bhubaneswar

Q66) Ans (d)

These monuments are located in

Chhatarpur district Madhya Pradesh

within Vindhya mountain range

Q67) Ans (a)

The book lsquoThoughts on Pakistanrsquo was

written by Dr BR Ambedkar

On the occasion of the birth anniversary

of Dr BR Ambedkar the president of

India pays homage to this icon of India

In 1924 he founded the Depressed

Classes Institute (Bahishkrit Hitkarini

Sabha) and in 1927 the Samaj Samata

Sangh

Another area of attention for Ambedkar

was education For its spread among

the low classes he set up a network of

colleges by the name of Peoples

Education Society and founded hostels

Q68) Ans(b)

Mehrgarh is a famous Neolithic

settlement in the Indian subcontinent

which is situated in Baluchistan

province Pakistan

A pre-historic rock art site is discovered

in the vast expanse of limestone blocks

on the eastern banks of Naguleru river

near Dachepalli (Andhra Pradesh) It

has thrown light on the Neolithic

civilisation that flourished in Guntur

(Andhra Pradesh) during 1500-2000

BC

Q69) Ans (c)

The 12th and the 13th centuries saw

the emergence of the Kakatiyas They

were at first the feudatories of the

Western Chalukyas of Kalyana Initially

they ruled over a small territory near

Warangal (Telangana)

They introduced Nayakships which was

later adopted and developed by the

Rayas of Vijayanagara

Q70) Ans (a)

The fast had effect of putting pressure

on mill owners who finally agreed to

give the workers a 35 per cent increase

in wages

Google celebrated with a doodle the

132nd birth anniversary of Anasuya

Sarabhai who played a pioneering role

in Indiarsquos labour movement

Q71) Ans (d)

The UNESCOrsquos list of the representative

list of the intangible cultural heritage of

humanity from India are

- Koodiyattam Sanskrit Theatre of

Kerala

- Mudiyettu ritual theatre and dance

drama of Kerala

- Tradition of Vedic Chanting

- Kalbelia folk songs and dances of

Rajasthan

- Ramlila Traditional Performance of

the Ramayana

- Sankirtana ritual singing

drumming and dancing of Manipur

- Ramman religious festival and

ritual theatre of the Garhwal

Himalayas India

- Traditional brass and copper craft of

utensil making among the Thatheras

of Jandiala Guru Punjab India

- Chhau dance classical Indian dance

originated in the eastern Indian

states

- Buddhist chanting of Ladakh

recitation of sacred Buddhist texts

in the trans-Himalayan Ladakh

region Jammu and Kashmir India

- Yoga

- Nouroz

- Kumbh Mela

RAUSIAS-FC19E1003 49

Q72) Ans(b)

The President of India Shri Ram Nath Kovind inaugurated the Hornbill Festival and State Formation Day celebrations of Nagaland in Kisama

The festival is named after the Indian hornbill the large and colourful forest bird which is displayed in the folklore of most of the states tribes

The major recognized tribes of Nagaland are Angami Ao Chakhesang Chang

Kuki Rengma and Zeling etc

Onge Jarawa and Sentinelese are the

tribes of Andman amp Nicobar Islands

Q73) Ans (c)

The Rashtrakutas rule in the Deccan lasted for almost two hundred years till the end of the tenth century The Rashtrakutas rulers were tolerant in their religious views and patronized not only Shaivism and Vaishnavism but

Jainism as well

The famous rock-cut temple of Shiva at Ellora was built by one of the Rashtrakutas kings Krishna I in the ninth century His successor Amoghavarsha was a Jain but he also

patronized other faiths

The Rashtrakutas allowed Muslims traders to settle and permitted Islam to

be preached in their dominions

Recently increasing defacement at the prehistoric rock paintings of Pandavulagutta Telangana has created a cause for grave concern It can spoil

the prehistoric rock

Pandavulagutta is home to

- Painted rock shelters dating to

10000 BC-8000 BC

- An 8th century inscription of the

Rashtrakuta period and

- Painted frescoes from the 12th century Kakatiya empire

Q74) Ans (b)

In 1828 Raja Ram Mohan Roy founded a new religious society the Brahma Sabha later known as the Brahmo

Samaj

Debendranath Tagore headed the Tattvabodhini Sabha which was

engaged in search of spiritual truth

Its purpose was to purify Hinduism and to preach monotheism or belief in one God

The new society was to be based on the twin pillars of reason and the Vedas and

Upanishads

Recently Sadharan Brahmo Samaj (SBS) has entered into a legal battle with the West Bengal government due

to some legal issue

Q75) Ans (c)

The Chishti order was established in India by Khwaja Moinuddin Chishti who came to India around 1192 The Chishtirsquos are considered to be the most influential of the groups of Sufis who migrated to India in the late twelfth century They adapted successfully to the local environment and adopted several features of Indian devotional

traditions

The historical dargah of Sufi mystic Khwaja Moinuddin Chishti in Ajmer is all set to get a facelift This 13 th century dargah has been included among the Swachh Iconic Places a clean-up initiative focused on iconic

heritage spiritual and cultural places

Page 40: GENERAL STUDIES (PAPER I) · Test is part of Rau’s IAS Test series for Preliminary Exam 2019 FOUNDATION + CURRENT AFFAIRS GENERAL STUDIES (PAPER –I) FOUNDATION TEST –III TOPIC:

FC19H1003 40

Q73) उततर (c)

सपषटीकरण

bull दकन म राषटर कि शासन दसिी ो सदी क अोत तक

लगभग 200 ििो तक रहा था

bull राषटर कि शासक अपन िावमिक विचार ो म सवहषण

bull उनह ोन न किल शि िमि और िषणि िमि बसलक

जन िमि क भी सोरकषण वदया था

bull एल रा म वशि क परवसद रॉक कि मोवदर का

वनमािण नौिी ो सदी म राषटर कि राजा कषण परथम न

करिाया था

bull उसका उततराविकारी अम घििि जन था लवकन

उसन अनय िमो क भी सोरकषण परदान वकया था

bull राषटर कि ो न मसलमान वयापाररय ो क बसन की

अनमवत दी थी

bull उनह न अपन अविराजय ो म इसलाम क उपदश दन

की भी अनमवत दी थी

bull अभी हाल ही म पाोडिलागटटा (तलोगाना) क

परागवतहावसक चटटान वचतर ो क कषरण की बिती हई

घिनाएा एक गोभीर वचोता का वििय ह

bull यह परागवतहावसक चटटान क नकसान पहाचा

सकता ह

bull पाोडिलागटटा वनमनवलसखत क वलए जाना जाता ह

- 10000 ईसा पिि स 8000 ईसा पिि क वचवतरत

चटटानी आशरय ो क वलए

- राषटर कि काल क एक 8 िी ो सदी क

वशलालख क वलए और

- 12िी ो सदी क काकविय सामराजय क वभवतत

वचतर ो क वलए

Q74) उततर (b)

सपषटीकरण

bull 1828 म राजा राम म हन रॉय न एक नय िावमिक

समाज बरहम सभा की थथापना की थी वजस बाद

म बरहम समाज क नाम स जाना गया था

bull दिदरनाथ िग र न ततवब विनी सभा की अधयकषता

की थी ज आधयासिक सतय की ख ज म सोलि

थी

bull इसका उददशय वहोद िमि क शद करन का और

एकशवरिाद (एक ईशवर म आथथा) का परचार करना

था

bull नय समाज की थथापना क आिार थ कारण

(तकि ) क द सतमभ तथा िद और उपवनिद

bull अभी हाल ही म सािारण बरहम समाज का कछ

काननी मदद ो क लकर पवशचम बोगाल सरकार क

साथ काननी वििाद चल रहा ह

Q75) उततर (c)

सपषटीकरण

bull भारत म वचशती वसलवसल की थथापना खवाजा

म इनददीन वचशती क दवारा की गयी थी

bull ि 1192 ईसवी क आसपास भारत आय थ

bull वचशतीय ो क बारहिी ो शताबदी क उततरािि म भारत

म आन िाल सफीय ो क समह ो म सबस

परभािशाली माना जाता ह

bull उनह ोन थथानीय िातािरण क साथ सफलतापििक

अनकलन वकया और उनह ोन भारतीय भसकत

परोपराओो क कई पहलओो क अपनाया

bull अजमर म सफी अपरकि खवाजा म इनददीन वचशती

की ऐवतहावसक दरगाह क एक नया रप दन की

तयारी की जा रही ह

bull इस 13िी ो शताबदी की दरगाह क ldquoसवचछ

आइकॉवनक थथल ोrdquo (Swacch Iconic Places) म

शावमल वकया गया ह ज परवतवषठत विरासत

आधयासिक और साोसकवतक थथान ो पर क वदरत

य जना ह

FC19H1003 41

ANSWERS amp EXPLANATION OF

NCERT History Class VI-X + Current Affairs

(FC19E1003)

Q1) Answer c

Explanation

Rigveda consists of more than a

thousand hymns dedicated to gods and

goddesses These hymns were

composed by sages and learnt by men

however a few were composed by

women like Apala Ghosa Lopamudra

Maitreyi and Gargi

Rigveda consists of many hymns in the

form of dialogues We get an example of

a dialogue between a sage named

Vishwamitra and two rivers (Beas and

Sutlej) that were worshipped as

goddesses This suggests that he

belonged to the Vedic period

Q2) Answer b

Explanation

Traces of ash have been found from

Kurnool Caves suggesting that people

were familiar with the use of fire

It is situated in Andhra Pradesh

Q3) Answer c

Explanation

Burzahom is a prehistoric site in

present day Kashmir where people built

pit houses which were dug into the

ground with steps leading into them

These may have provided shelter in cold

weather

Q4) Answer c

Explanation

Epigraphy is defined as the study of

inscriptions

Manuscriptology is the study of history

and literature through the use of hand

written documents

Palaeography refers to the study of

ancient writing systems and the

deciphering and dating of historical

manuscripts

Numismatics refers to the study of

coins

Q5) Answer a

Explanation

Charaka Samhita was written by

Charaka and is an important book on

Ayurveda and medicine

He was a practitioner of the traditional

system of Indian medicine known as

Ayurveda

Charaka is thought to have flourished

sometime between the 2nd century BCE

and the 2nd century CE

Q6) Answer b

Explanation

Bhaga refers to the tax on crops which

was fixed at 16th of the production

Kammakaras is the term used for the

landless agricultural labour class

Ashvamedha also known as horse

sacrifice is a ritual where a horse is let

loose to wander freely and it was

guarded by the rajarsquos men

Q7) Answer (d)

Explanation

In the Rigvedic period horses were

yoked to chariots that were used in

battles fought to capture land cattle

etc This suggests that the use of horse

chariots began much before the period

of Mahajanapadas

The battles were fought in the Rigvedic

period for cattlersquos lands water an even

to capture people Most men took part

in these wars however there was no

regular army but there were assemblies

where people met and discussed

matters of war Regular armies became

a feature in the Mjahajanapada period

including vast armies of foot soldiers

chariots and elephants

RAUSIAS-FC19E1003 42

Q8) Answer (a)

Explanation

Buddha belonged to the Sakya clan and

passed away at Kusinara

Buddha taught in Prakrit which was the

common language of people

Q9) Answer c

Explanation

There were six schools of philosophy in

ancient India These are known as

Vaishesika Nyaya Samkhya Yoga

Purva Mimansa and Vedanata or Uttara

Mimansa They were founded by sages

Kanada Gautama Kapila Patanjali

Jamini and Vyasa respectively

Q10) Answer b

Explanation

The teachings of Mahavira were

compiled at Valabhi in 6th century AD

Q11) Answer (c)

Explanation

Chanakya is traditionally identified as

Kautilya or Vishnugupta who authored

the ancient Indian political treatise the

Arthashastra

Q12) Answer d

The national emblem of India is an

adaptation of the Lion Capital atop the

Ashoka Pillar of Sarnath Uttar Pradesh

and is combined with the National

Motto Satyameva Jayate

The Rampurva Bull gets the name from

the site of its discovery Rampurva in

Bihar

It is noted for its delicately sculpted

model demonstrating superior

representation of soft flesh sensitive

nostrils alert ears and strong legs It is

a mixture of Indian and Persian

elements

Sankissa is situated in Uttar Pradesh

India

Q13) Ans(a)

Kunwar Singh was a notable leader during the Revolt of 1857 He belonged

to a royal house of Jagdispur Bihar

Q14) Answer b

Explanation

The term Vellalar was used for large

landowners

Q15) Answer c

Explanation

Arikamedu was a coastal settlement

where ships unloaded goods from

distant lands Finds here include a

massive brick warehouse pottery

including amphorae and Arretine ware

Roman lamps glassware and gems have

also been found at the site

Q16) Answer a

Explanation

Muvendar is a Tamil word mentioned in

Sangam poems meaning three chiefs

used for the heads of three ruling

families the Cholas Cheras and

Pandyas

Q17) Ans (c)

Several tribal or kin-based assemblies

such as the Sabha Vidatha and gana

are mentioned in the Rig-veda The

Sabha and the samiti mattered a great

deal in early Vedic times so much so

that the chiefs or the kings showed an

eagerness to win their support

Q18) Ans (a)

Jainism recognised the existence of the

gods but placed them lower than the

jina and did not condemn the varna

system as Buddhism did

Q19) Answer (d)

Explanation

Cholas and Pandyas had developed

powerful coastal cities The most

important city of Cholas was Puhar or

Kaveripattinam and Madurai was the

capital of Pandyas

Q20) Answer b

Explanation

Buddhacharita is the biography of

Buddha and was written by

RAUSIAS-FC19E1003 43

Ashvaghosha

Q21) Answer (a)

Explanation

Tamil poet Appar was a Shiva devotee

So he was a Nayanar saint

Q22) Answer d

Explanation

Samudragupta was a prominent Gupta

ruler whose coins depict him playing a

veena indicating his love for music We

get important historic information from

his Allahabad Prashasti which was

composed by his court poet Harisena

Q23) Answer (b)

Explanation

Vikrama Samvat was founded by

Chandragupta II in the 58 BC as a

mark of victory over the Shakas and

assumed the title of Vikramaditya

Banabhatta wrote Harshavardhanarsquos

biography the Harshacharita in

Sanskrit

Q24) Answer c

Explanation

Sandhi-vigrahika was the minister of

war and peace

Sarthavaha was the leader of the

merchant caravans

Q25) Answer a

Explanation

Xuan Zang (Hsuan-tsang) was a

Chinese traveller who came during the

reign of Harshavardhana

In the decade that began in 630 AD

Xuan Zang came to India through

Kashmir after visiting Central Asia Iran

and Afghanistan

He travelled from north to east and lived

in Bihar for a couple of years

At Nalanda University Xuan Zang

interacted with students and scholars

mastered local languages and

discovered Buddhist stupas

Q26) Answer c

Explanation

Pradakshina patha is a circular path

laid around a stupa in Buddhist

architecture While the rest are a part of

temple architecture

Q27) Answer d

Explanation

All the above-mentioned temples have

an elaborate use of bricks (baked

bricks) along with stone

Q28) Ans (c)

Muhammad Quli Qutab was the Sultan

of Golconda He was a contemporary of

Akbar was very fond of literature and

architecture

The Sultan was a great poet and he

wrote in Dakhini Urdu Persian and

Telgu and has left an extensive diwan or

collection

Recently the Archaeological Survey of

India (ASI) will be using Ground

Penetrating Radar (GPR) to map the

contours of the area around the Bagh-e-

Naya Qila excavated garden inside the

Golconda Fort in Telangana

Q29) Answer a

Explanation

Silappadikaram is a famous Tamil epic

which was written by Ilango around

1800 years ago It is a story of a

merchant named Kovalan who fell in

love with a courtesan named Madhavi

Manimekalai tells the story of the

daughter of Kovalan and Madhavi

Q30) Answer (a)

Explanation

Charaka is the author of Charaka

Samhita which is an important work of

Ayurveda and medicines

Brahmaguptarsquos fame rests mostly on his

Brahma-sphuta-siddhanta which was

an astronomical work It was translated

into Arabic in Baghdad and had a major

impact on Islamic mathematics and

astronomy

Late in his life Brahmagupta wrote

Khandakhadyaka which was an

RAUSIAS-FC19E1003 44

astronomical handbook that employed

Aryabhatarsquos system of starting each day

at midnight

Q31) Answer (c)

Explanation

Amir Khusrau was a famous sufi

musician poet and scholar In 1318 he

noted that there was different language

in every region of this land (Hindustan)

Lahori Kashmiri Dvarsamudri (in

Southern Karnataka) Telangana (in

Andhra Pradesh) Gujari (in Gujarat)

Marsquobari (in Tamil Nadu) Awadhi (in

eastern Uttar Pradesh) and Hindawai (in

the area around in Delhi) etc He went

to explain that Sanskrit did not belong

to any region and that only brahmans

knew it

Q32) Answer c

Explanation

Hiranyagarbha refers to the golden

womb When this ritual was performed

with the help of Brahmanas it was

thought to lead to the rebirth of the

sacrificer as a Khastriya

Q33) Answer d

Explanation

Kadamai refers to a tax on land

revenue

Gwalior Prashasti describes the exploits

of Nagabhata who was a Pratihara king

Q34) Answer b

Explanation

Rajatarangini is a Sanskrit text written

by Kalhana in the 12th century

It was historical chronicle of early India

It is justifiably considered to be the best

and most authentic work of its kind

It covers the entire span of history in

the Kashmir region from the earliest

times to the date of its composition

Q35) Answer c

Explanation

ldquoUrrdquo was the general assembly of the

village ldquoUrrdquo consisted of all the

taxpaying residents of an ordinary

village

Q36) Answer (a)

Explanation

Tarikh was a form of history writing in

the Delhi Sultanate The authors of

tawarikhs were learned men which

included secretaries administrators etc

Q37 Answer (a)

Explanation

Alauddin chose to pay his soldiers salaries in cash rather than iqtas The soldiers would buy their supplies from merchants in Delhi and it was thus feared that merchants would raise their prices To stop this Alauddin controlled the prices of goods in Delhi Prices were carefully surveyed by officers and merchants who did not sell at the prescribed rates were punished

Q38) Answer (d)

Explanation

Delhi first became the capital of a

kingdom under the Tomara Rajputs

who were defeated in the middle of the

twelfth century by the Chauhans (also

referred to as Chahamanas) of Ajmer

It was under the Tomaras and

Chauhans that Delhi became an

important commercial centre Many rich

Jaina merchants lived in the city and

constructed several temples Coins

minted here called dehliwal had a wide

circulation

Q39) Answer (c)

Explanation

Moth ki Masjid was built in the reign of

Sikandar Lodi by his minister

Begumpuri mosque built in the reign of

Muhammad Tughluq was the main

mosque of Jahanpanah the ldquoSanctuary

of the Worldrdquo and his new capital in

Delhi

Quwwat al ndash Islam mosque was

enlarged by Iltutmish and Alauddin

Khalji The minar was built by three

Sultansndash Qutbuddin Aybak Iltutmish

and Firuz Shah Tughluq

RAUSIAS-FC19E1003 45

Q40) Answer (c)

Explanation

Under the Mughals mansabdar was

referred to an individual who held a

mansab ie rank and he received his

salary as revenue assignments called

jagirs

Q41) Ans (b)

The Quit India Movement was a

spontaneous revolt of people against

British rule

The All India Congress Committee met

at Bombay on 8 August 1942 It passed

the famous resolution Quit India and

proposed the starting of a non-violent

mass struggle under Gandhis

leadership to achieve this aim But on

the very next day Gandhi and other

eminent leaders of the Congress were

arrested The Congress was once again

declared illegal

Q42) Ans (c)

The Simon Commission refers to a

group of seven MPs from the United

Kingdom constituted to suggest

constitutional reforms for British India

The Commission consisted of only

British members headed by one of the

senior British politicians Sir John

Simon

So the people of India agitated against

the arrival of Simon Commission

Q43) Ans (a)

He was widely known for his

unfavourable opinion of the economic

consequences of the British rule in

India

In his many writings and speeches and

especially in Poverty and Un-British

Rule in India Naoroji argued that India

was too highly taxed and that its wealth

was being drained away to England

He did not interpret the ancient Indian

texts and restored the self-confidence of

Indians And also he did not stress the

need for eradication of all the social

evils before anything else

Q44) Ans (c)

In August 1932 Prime Minister

MacDonald announced his Communal

Award Great Britainrsquos unilateral

attempt to resolve the various conflicts

among Indiarsquos many communal

interests

The award which was later

incorporated into the act of 1935

expanded the separate-electorate

formula reserved for Muslims to other

minorities including Sikhs Indian

Christians Anglo-Indians Europeans

distinct regional groups Gandhi

undertook a ldquofast unto deathrdquo against

that offer which he viewed as a

nefarious British plot to divide the

Indian society

Q45) Ans (b)

In British India apart from existing

imports and exports there was also a

particular amount of money which

colonial India contributed towards

administration maintenance of the

army war expenses pensions to retired

officers and other expenses accrued by

Britain towards maintenance of her

colony These were known as Home

charges and were paid for almost

entirely by India

The Home charges was made of

following components-

- Interest payable on Indian debt

- Dividend to shareholders of East

India Company

- Funds used to support the India

Office in London

- Funds used to pay salaries and

pensions of British personnel

engaged in India

- Interest on the railways

- Civil and military charges

- Store purchases in England

Q46) Ans (b)

The Lahore session of the Indian

National Congress was held in 1929

under the Presidentship of Jawaharlal

Nehru

The Lahore session of the Indian

National Congress witnessed significant

RAUSIAS-FC19E1003 46

developments in the Indian national

movement

- First the election of Jawaharlal

Nehru to the post of Presidentship of

the Congress was a clear indication

of the growing strength of the

Leftists in the Congress

- Secondly it was in this session that

the Congress for the first time raised

the demand for complete

independence Such demand was

not raised from the Congress

platform earlier

Q47) Ans (b)

It did not provide for separate

electorates for any community or

weightage for minorities However it did

allow for the reservation of minority

seats in provinces having minorities of

at least ten per cent but this was to be

in strict proportion to the size of the

community

There was no provision for complete

Independence for India

Q48) Ans (c)

The religion of early Vedic Aryans was

primarily of worship of nature and

Yajnas

The early Aryan religion was kind of

nature worship Actually the forces

around them which they could not

control or understand were invested

with divinity and were personified as

male or female gods And they

performed some Yajnas also

Q49) Ans (b)

The roads and river-routes were not

immune from robbery It is notable that

Yuan Chwang (Hiuen Tsang) was

robbed of his belongings during

Harshvardanarsquos period

Q50) Ans (c)

Q51) Ans (b)

Purandara Dasa was a saint and great

devotee of Lord Krishna

There is much speculation about where

Purandara Dasa regarded as the

Pitamaha of Carnatic music was born

Recently an expert committee

constituted by the Kannada University

Hampi has come to the conclusion that

Kshemapura Shivamogga district

Karnataka is the birth place of

Purandara Dasa

Q52) Ans (c)

Sri Tyagaraja Sri Shyama Shastry and Sri Muthuswami Dikshitar are considered the trinity of Carnatic music and with them came the golden age in Carnatic music in the 18th-19th

century

Q53) Ans d)

Recently a rare sarcophagus (stone

coffin) which is 2000 years old from the

Iron AgendashMegalithic era was discovered

from a rock-cut cave at Viyur village of

Kollam near Koyilandy in Kozhikode

district Kerala

The coffin containing bone fragments

was found during an excavation ldquoSo

far such a rare finding has been

discovered only from two sites

in Kerala Both these sarcophagi were

recovered from Megalithic sites at

Chevayur and Atholi also in Kozhikode

district

Q54) Ans a)

The megalithic culture in South India was a full-fledged Iron Age culture

Q55) Ans d)

The Cholas Pandyas and Keralaputras

(Cheras) mentioned in Ashokan

inscriptions were probably in the late

megalithic phase of material culture

Q56) Ans d)

Q57) Ans (b)

Raj Kumar Shukla followed Gandhiji all

over the country to persuade him to

come to Champaran to investigate the

problem associated with tinkathia

system

RAUSIAS-FC19E1003 47

Brij Kishore Rajendra Prasad Mahadev

Desai and Narhari Parikh accompanied

Gandhi ji during the Champaran

Satyagraha

Q58) Ans (b)

The Satvahanas started the practice of granting tax-free villages to brahmanas and Buddhist monks

Q59) Ans c)

The objectives of the Programme are

listed as under

- Developing basic tourism

infrastructure

- Promoting cultural and heritage

value of the country to generate

livelihoods in the identified regions

- Enhancing the tourist attractiveness

in a sustainable manner by

developing world-class

infrastructure at the heritage

monument sites

- Creating employment through active

involvement of local communities

- Harnessing tourism potential for its

effects on employment generation

and economic development

- Developing sustainable tourism

infrastructure and ensuring proper

Operations and maintenance

therein

Q60) Ans (b)

The Tribal Cooperative Marketing

Development Federation of India

(TRIFED) came into existence in 1987

It is a national-level apex organization

functioning under the administrative

control of Ministry of Tribal Affairs

Govt of India

TRIFED has its registered and Head

Office located in New Delhi

Q61) Ans (c)

Premchandrsquos novels include

Premashram Rangabhumi Ghaban

Karmabhumi and Godan

Gora is a novel written by Rabindranath

Tagore

138th birth anniversary of Munshi

Premchand was celebrated across the

country

Q62) Ans (b)

Giddha is a traditional pastoral dance

performed by the women of the Punjab

India and Pakistan at festival times

and at the sowing and reaping of the

harvest

By this dance the Punjabi women

reveal their joy expel their suppressed

feelings in a male dominated society

through the performance of Giddha

Since this dance has nothing to do with

men only women can participate in it

During the Teej celebrations Giddha

dance is celebrated in Punjab every

year Teej is a generic name for a

number of festivals that are celebrated

by women in some parts of India

Q63) Ans (a)

Dara Shukoh wrote the remarkable

work called ldquoMajma-ul-Bahrainrdquo or the

ldquoThe confluence of two seasrdquo

The Vice President of India Shri M

Venkaiah Naidu has said that Prince

Dara Shukohrsquos writings can come as a

refreshing source for infusing peace and

harmony He was addressing the

gathering after visiting the exhibition

that showcases the forgotten Prince of

yesteryears Dara Shukoh organized by

Mr Francois Gautier at Indira Gandhi

National Centre for the Arts in New

Delhi

Q64) Ans (c)

The statue Gommateshwara is

dedicated to the Jain God Bahubali

It is a monolithic statue

President Ram Nath Kovind

inaugurated the grand anointing

ceremony mdash Mahamastakabhisheka mdash

held once in 12 years at

Shravanabelagola (Karnataka)

Q65) Ans (c)

Prachi Valley had come up around the

Prachi river Prachi Valley gradually

disappeared

RAUSIAS-FC19E1003 48

The Prachi river originates from

Bhubaneswar

It is a tributary of the Mahanadi and

flows through the districts of Puri

Khurda Cuttack and Jagatsinghpur

and the entire region of the river is

termed as the Prachi Valley

It falls into the Bay of Bengal

Archaeological evidence shows that the

Prachi Valley Civilisation predates both

Harappa and Mohenjo-Daro

The Prachi river originates from

Bhubaneswar

Q66) Ans (d)

These monuments are located in

Chhatarpur district Madhya Pradesh

within Vindhya mountain range

Q67) Ans (a)

The book lsquoThoughts on Pakistanrsquo was

written by Dr BR Ambedkar

On the occasion of the birth anniversary

of Dr BR Ambedkar the president of

India pays homage to this icon of India

In 1924 he founded the Depressed

Classes Institute (Bahishkrit Hitkarini

Sabha) and in 1927 the Samaj Samata

Sangh

Another area of attention for Ambedkar

was education For its spread among

the low classes he set up a network of

colleges by the name of Peoples

Education Society and founded hostels

Q68) Ans(b)

Mehrgarh is a famous Neolithic

settlement in the Indian subcontinent

which is situated in Baluchistan

province Pakistan

A pre-historic rock art site is discovered

in the vast expanse of limestone blocks

on the eastern banks of Naguleru river

near Dachepalli (Andhra Pradesh) It

has thrown light on the Neolithic

civilisation that flourished in Guntur

(Andhra Pradesh) during 1500-2000

BC

Q69) Ans (c)

The 12th and the 13th centuries saw

the emergence of the Kakatiyas They

were at first the feudatories of the

Western Chalukyas of Kalyana Initially

they ruled over a small territory near

Warangal (Telangana)

They introduced Nayakships which was

later adopted and developed by the

Rayas of Vijayanagara

Q70) Ans (a)

The fast had effect of putting pressure

on mill owners who finally agreed to

give the workers a 35 per cent increase

in wages

Google celebrated with a doodle the

132nd birth anniversary of Anasuya

Sarabhai who played a pioneering role

in Indiarsquos labour movement

Q71) Ans (d)

The UNESCOrsquos list of the representative

list of the intangible cultural heritage of

humanity from India are

- Koodiyattam Sanskrit Theatre of

Kerala

- Mudiyettu ritual theatre and dance

drama of Kerala

- Tradition of Vedic Chanting

- Kalbelia folk songs and dances of

Rajasthan

- Ramlila Traditional Performance of

the Ramayana

- Sankirtana ritual singing

drumming and dancing of Manipur

- Ramman religious festival and

ritual theatre of the Garhwal

Himalayas India

- Traditional brass and copper craft of

utensil making among the Thatheras

of Jandiala Guru Punjab India

- Chhau dance classical Indian dance

originated in the eastern Indian

states

- Buddhist chanting of Ladakh

recitation of sacred Buddhist texts

in the trans-Himalayan Ladakh

region Jammu and Kashmir India

- Yoga

- Nouroz

- Kumbh Mela

RAUSIAS-FC19E1003 49

Q72) Ans(b)

The President of India Shri Ram Nath Kovind inaugurated the Hornbill Festival and State Formation Day celebrations of Nagaland in Kisama

The festival is named after the Indian hornbill the large and colourful forest bird which is displayed in the folklore of most of the states tribes

The major recognized tribes of Nagaland are Angami Ao Chakhesang Chang

Kuki Rengma and Zeling etc

Onge Jarawa and Sentinelese are the

tribes of Andman amp Nicobar Islands

Q73) Ans (c)

The Rashtrakutas rule in the Deccan lasted for almost two hundred years till the end of the tenth century The Rashtrakutas rulers were tolerant in their religious views and patronized not only Shaivism and Vaishnavism but

Jainism as well

The famous rock-cut temple of Shiva at Ellora was built by one of the Rashtrakutas kings Krishna I in the ninth century His successor Amoghavarsha was a Jain but he also

patronized other faiths

The Rashtrakutas allowed Muslims traders to settle and permitted Islam to

be preached in their dominions

Recently increasing defacement at the prehistoric rock paintings of Pandavulagutta Telangana has created a cause for grave concern It can spoil

the prehistoric rock

Pandavulagutta is home to

- Painted rock shelters dating to

10000 BC-8000 BC

- An 8th century inscription of the

Rashtrakuta period and

- Painted frescoes from the 12th century Kakatiya empire

Q74) Ans (b)

In 1828 Raja Ram Mohan Roy founded a new religious society the Brahma Sabha later known as the Brahmo

Samaj

Debendranath Tagore headed the Tattvabodhini Sabha which was

engaged in search of spiritual truth

Its purpose was to purify Hinduism and to preach monotheism or belief in one God

The new society was to be based on the twin pillars of reason and the Vedas and

Upanishads

Recently Sadharan Brahmo Samaj (SBS) has entered into a legal battle with the West Bengal government due

to some legal issue

Q75) Ans (c)

The Chishti order was established in India by Khwaja Moinuddin Chishti who came to India around 1192 The Chishtirsquos are considered to be the most influential of the groups of Sufis who migrated to India in the late twelfth century They adapted successfully to the local environment and adopted several features of Indian devotional

traditions

The historical dargah of Sufi mystic Khwaja Moinuddin Chishti in Ajmer is all set to get a facelift This 13 th century dargah has been included among the Swachh Iconic Places a clean-up initiative focused on iconic

heritage spiritual and cultural places

Page 41: GENERAL STUDIES (PAPER I) · Test is part of Rau’s IAS Test series for Preliminary Exam 2019 FOUNDATION + CURRENT AFFAIRS GENERAL STUDIES (PAPER –I) FOUNDATION TEST –III TOPIC:

FC19H1003 41

ANSWERS amp EXPLANATION OF

NCERT History Class VI-X + Current Affairs

(FC19E1003)

Q1) Answer c

Explanation

Rigveda consists of more than a

thousand hymns dedicated to gods and

goddesses These hymns were

composed by sages and learnt by men

however a few were composed by

women like Apala Ghosa Lopamudra

Maitreyi and Gargi

Rigveda consists of many hymns in the

form of dialogues We get an example of

a dialogue between a sage named

Vishwamitra and two rivers (Beas and

Sutlej) that were worshipped as

goddesses This suggests that he

belonged to the Vedic period

Q2) Answer b

Explanation

Traces of ash have been found from

Kurnool Caves suggesting that people

were familiar with the use of fire

It is situated in Andhra Pradesh

Q3) Answer c

Explanation

Burzahom is a prehistoric site in

present day Kashmir where people built

pit houses which were dug into the

ground with steps leading into them

These may have provided shelter in cold

weather

Q4) Answer c

Explanation

Epigraphy is defined as the study of

inscriptions

Manuscriptology is the study of history

and literature through the use of hand

written documents

Palaeography refers to the study of

ancient writing systems and the

deciphering and dating of historical

manuscripts

Numismatics refers to the study of

coins

Q5) Answer a

Explanation

Charaka Samhita was written by

Charaka and is an important book on

Ayurveda and medicine

He was a practitioner of the traditional

system of Indian medicine known as

Ayurveda

Charaka is thought to have flourished

sometime between the 2nd century BCE

and the 2nd century CE

Q6) Answer b

Explanation

Bhaga refers to the tax on crops which

was fixed at 16th of the production

Kammakaras is the term used for the

landless agricultural labour class

Ashvamedha also known as horse

sacrifice is a ritual where a horse is let

loose to wander freely and it was

guarded by the rajarsquos men

Q7) Answer (d)

Explanation

In the Rigvedic period horses were

yoked to chariots that were used in

battles fought to capture land cattle

etc This suggests that the use of horse

chariots began much before the period

of Mahajanapadas

The battles were fought in the Rigvedic

period for cattlersquos lands water an even

to capture people Most men took part

in these wars however there was no

regular army but there were assemblies

where people met and discussed

matters of war Regular armies became

a feature in the Mjahajanapada period

including vast armies of foot soldiers

chariots and elephants

RAUSIAS-FC19E1003 42

Q8) Answer (a)

Explanation

Buddha belonged to the Sakya clan and

passed away at Kusinara

Buddha taught in Prakrit which was the

common language of people

Q9) Answer c

Explanation

There were six schools of philosophy in

ancient India These are known as

Vaishesika Nyaya Samkhya Yoga

Purva Mimansa and Vedanata or Uttara

Mimansa They were founded by sages

Kanada Gautama Kapila Patanjali

Jamini and Vyasa respectively

Q10) Answer b

Explanation

The teachings of Mahavira were

compiled at Valabhi in 6th century AD

Q11) Answer (c)

Explanation

Chanakya is traditionally identified as

Kautilya or Vishnugupta who authored

the ancient Indian political treatise the

Arthashastra

Q12) Answer d

The national emblem of India is an

adaptation of the Lion Capital atop the

Ashoka Pillar of Sarnath Uttar Pradesh

and is combined with the National

Motto Satyameva Jayate

The Rampurva Bull gets the name from

the site of its discovery Rampurva in

Bihar

It is noted for its delicately sculpted

model demonstrating superior

representation of soft flesh sensitive

nostrils alert ears and strong legs It is

a mixture of Indian and Persian

elements

Sankissa is situated in Uttar Pradesh

India

Q13) Ans(a)

Kunwar Singh was a notable leader during the Revolt of 1857 He belonged

to a royal house of Jagdispur Bihar

Q14) Answer b

Explanation

The term Vellalar was used for large

landowners

Q15) Answer c

Explanation

Arikamedu was a coastal settlement

where ships unloaded goods from

distant lands Finds here include a

massive brick warehouse pottery

including amphorae and Arretine ware

Roman lamps glassware and gems have

also been found at the site

Q16) Answer a

Explanation

Muvendar is a Tamil word mentioned in

Sangam poems meaning three chiefs

used for the heads of three ruling

families the Cholas Cheras and

Pandyas

Q17) Ans (c)

Several tribal or kin-based assemblies

such as the Sabha Vidatha and gana

are mentioned in the Rig-veda The

Sabha and the samiti mattered a great

deal in early Vedic times so much so

that the chiefs or the kings showed an

eagerness to win their support

Q18) Ans (a)

Jainism recognised the existence of the

gods but placed them lower than the

jina and did not condemn the varna

system as Buddhism did

Q19) Answer (d)

Explanation

Cholas and Pandyas had developed

powerful coastal cities The most

important city of Cholas was Puhar or

Kaveripattinam and Madurai was the

capital of Pandyas

Q20) Answer b

Explanation

Buddhacharita is the biography of

Buddha and was written by

RAUSIAS-FC19E1003 43

Ashvaghosha

Q21) Answer (a)

Explanation

Tamil poet Appar was a Shiva devotee

So he was a Nayanar saint

Q22) Answer d

Explanation

Samudragupta was a prominent Gupta

ruler whose coins depict him playing a

veena indicating his love for music We

get important historic information from

his Allahabad Prashasti which was

composed by his court poet Harisena

Q23) Answer (b)

Explanation

Vikrama Samvat was founded by

Chandragupta II in the 58 BC as a

mark of victory over the Shakas and

assumed the title of Vikramaditya

Banabhatta wrote Harshavardhanarsquos

biography the Harshacharita in

Sanskrit

Q24) Answer c

Explanation

Sandhi-vigrahika was the minister of

war and peace

Sarthavaha was the leader of the

merchant caravans

Q25) Answer a

Explanation

Xuan Zang (Hsuan-tsang) was a

Chinese traveller who came during the

reign of Harshavardhana

In the decade that began in 630 AD

Xuan Zang came to India through

Kashmir after visiting Central Asia Iran

and Afghanistan

He travelled from north to east and lived

in Bihar for a couple of years

At Nalanda University Xuan Zang

interacted with students and scholars

mastered local languages and

discovered Buddhist stupas

Q26) Answer c

Explanation

Pradakshina patha is a circular path

laid around a stupa in Buddhist

architecture While the rest are a part of

temple architecture

Q27) Answer d

Explanation

All the above-mentioned temples have

an elaborate use of bricks (baked

bricks) along with stone

Q28) Ans (c)

Muhammad Quli Qutab was the Sultan

of Golconda He was a contemporary of

Akbar was very fond of literature and

architecture

The Sultan was a great poet and he

wrote in Dakhini Urdu Persian and

Telgu and has left an extensive diwan or

collection

Recently the Archaeological Survey of

India (ASI) will be using Ground

Penetrating Radar (GPR) to map the

contours of the area around the Bagh-e-

Naya Qila excavated garden inside the

Golconda Fort in Telangana

Q29) Answer a

Explanation

Silappadikaram is a famous Tamil epic

which was written by Ilango around

1800 years ago It is a story of a

merchant named Kovalan who fell in

love with a courtesan named Madhavi

Manimekalai tells the story of the

daughter of Kovalan and Madhavi

Q30) Answer (a)

Explanation

Charaka is the author of Charaka

Samhita which is an important work of

Ayurveda and medicines

Brahmaguptarsquos fame rests mostly on his

Brahma-sphuta-siddhanta which was

an astronomical work It was translated

into Arabic in Baghdad and had a major

impact on Islamic mathematics and

astronomy

Late in his life Brahmagupta wrote

Khandakhadyaka which was an

RAUSIAS-FC19E1003 44

astronomical handbook that employed

Aryabhatarsquos system of starting each day

at midnight

Q31) Answer (c)

Explanation

Amir Khusrau was a famous sufi

musician poet and scholar In 1318 he

noted that there was different language

in every region of this land (Hindustan)

Lahori Kashmiri Dvarsamudri (in

Southern Karnataka) Telangana (in

Andhra Pradesh) Gujari (in Gujarat)

Marsquobari (in Tamil Nadu) Awadhi (in

eastern Uttar Pradesh) and Hindawai (in

the area around in Delhi) etc He went

to explain that Sanskrit did not belong

to any region and that only brahmans

knew it

Q32) Answer c

Explanation

Hiranyagarbha refers to the golden

womb When this ritual was performed

with the help of Brahmanas it was

thought to lead to the rebirth of the

sacrificer as a Khastriya

Q33) Answer d

Explanation

Kadamai refers to a tax on land

revenue

Gwalior Prashasti describes the exploits

of Nagabhata who was a Pratihara king

Q34) Answer b

Explanation

Rajatarangini is a Sanskrit text written

by Kalhana in the 12th century

It was historical chronicle of early India

It is justifiably considered to be the best

and most authentic work of its kind

It covers the entire span of history in

the Kashmir region from the earliest

times to the date of its composition

Q35) Answer c

Explanation

ldquoUrrdquo was the general assembly of the

village ldquoUrrdquo consisted of all the

taxpaying residents of an ordinary

village

Q36) Answer (a)

Explanation

Tarikh was a form of history writing in

the Delhi Sultanate The authors of

tawarikhs were learned men which

included secretaries administrators etc

Q37 Answer (a)

Explanation

Alauddin chose to pay his soldiers salaries in cash rather than iqtas The soldiers would buy their supplies from merchants in Delhi and it was thus feared that merchants would raise their prices To stop this Alauddin controlled the prices of goods in Delhi Prices were carefully surveyed by officers and merchants who did not sell at the prescribed rates were punished

Q38) Answer (d)

Explanation

Delhi first became the capital of a

kingdom under the Tomara Rajputs

who were defeated in the middle of the

twelfth century by the Chauhans (also

referred to as Chahamanas) of Ajmer

It was under the Tomaras and

Chauhans that Delhi became an

important commercial centre Many rich

Jaina merchants lived in the city and

constructed several temples Coins

minted here called dehliwal had a wide

circulation

Q39) Answer (c)

Explanation

Moth ki Masjid was built in the reign of

Sikandar Lodi by his minister

Begumpuri mosque built in the reign of

Muhammad Tughluq was the main

mosque of Jahanpanah the ldquoSanctuary

of the Worldrdquo and his new capital in

Delhi

Quwwat al ndash Islam mosque was

enlarged by Iltutmish and Alauddin

Khalji The minar was built by three

Sultansndash Qutbuddin Aybak Iltutmish

and Firuz Shah Tughluq

RAUSIAS-FC19E1003 45

Q40) Answer (c)

Explanation

Under the Mughals mansabdar was

referred to an individual who held a

mansab ie rank and he received his

salary as revenue assignments called

jagirs

Q41) Ans (b)

The Quit India Movement was a

spontaneous revolt of people against

British rule

The All India Congress Committee met

at Bombay on 8 August 1942 It passed

the famous resolution Quit India and

proposed the starting of a non-violent

mass struggle under Gandhis

leadership to achieve this aim But on

the very next day Gandhi and other

eminent leaders of the Congress were

arrested The Congress was once again

declared illegal

Q42) Ans (c)

The Simon Commission refers to a

group of seven MPs from the United

Kingdom constituted to suggest

constitutional reforms for British India

The Commission consisted of only

British members headed by one of the

senior British politicians Sir John

Simon

So the people of India agitated against

the arrival of Simon Commission

Q43) Ans (a)

He was widely known for his

unfavourable opinion of the economic

consequences of the British rule in

India

In his many writings and speeches and

especially in Poverty and Un-British

Rule in India Naoroji argued that India

was too highly taxed and that its wealth

was being drained away to England

He did not interpret the ancient Indian

texts and restored the self-confidence of

Indians And also he did not stress the

need for eradication of all the social

evils before anything else

Q44) Ans (c)

In August 1932 Prime Minister

MacDonald announced his Communal

Award Great Britainrsquos unilateral

attempt to resolve the various conflicts

among Indiarsquos many communal

interests

The award which was later

incorporated into the act of 1935

expanded the separate-electorate

formula reserved for Muslims to other

minorities including Sikhs Indian

Christians Anglo-Indians Europeans

distinct regional groups Gandhi

undertook a ldquofast unto deathrdquo against

that offer which he viewed as a

nefarious British plot to divide the

Indian society

Q45) Ans (b)

In British India apart from existing

imports and exports there was also a

particular amount of money which

colonial India contributed towards

administration maintenance of the

army war expenses pensions to retired

officers and other expenses accrued by

Britain towards maintenance of her

colony These were known as Home

charges and were paid for almost

entirely by India

The Home charges was made of

following components-

- Interest payable on Indian debt

- Dividend to shareholders of East

India Company

- Funds used to support the India

Office in London

- Funds used to pay salaries and

pensions of British personnel

engaged in India

- Interest on the railways

- Civil and military charges

- Store purchases in England

Q46) Ans (b)

The Lahore session of the Indian

National Congress was held in 1929

under the Presidentship of Jawaharlal

Nehru

The Lahore session of the Indian

National Congress witnessed significant

RAUSIAS-FC19E1003 46

developments in the Indian national

movement

- First the election of Jawaharlal

Nehru to the post of Presidentship of

the Congress was a clear indication

of the growing strength of the

Leftists in the Congress

- Secondly it was in this session that

the Congress for the first time raised

the demand for complete

independence Such demand was

not raised from the Congress

platform earlier

Q47) Ans (b)

It did not provide for separate

electorates for any community or

weightage for minorities However it did

allow for the reservation of minority

seats in provinces having minorities of

at least ten per cent but this was to be

in strict proportion to the size of the

community

There was no provision for complete

Independence for India

Q48) Ans (c)

The religion of early Vedic Aryans was

primarily of worship of nature and

Yajnas

The early Aryan religion was kind of

nature worship Actually the forces

around them which they could not

control or understand were invested

with divinity and were personified as

male or female gods And they

performed some Yajnas also

Q49) Ans (b)

The roads and river-routes were not

immune from robbery It is notable that

Yuan Chwang (Hiuen Tsang) was

robbed of his belongings during

Harshvardanarsquos period

Q50) Ans (c)

Q51) Ans (b)

Purandara Dasa was a saint and great

devotee of Lord Krishna

There is much speculation about where

Purandara Dasa regarded as the

Pitamaha of Carnatic music was born

Recently an expert committee

constituted by the Kannada University

Hampi has come to the conclusion that

Kshemapura Shivamogga district

Karnataka is the birth place of

Purandara Dasa

Q52) Ans (c)

Sri Tyagaraja Sri Shyama Shastry and Sri Muthuswami Dikshitar are considered the trinity of Carnatic music and with them came the golden age in Carnatic music in the 18th-19th

century

Q53) Ans d)

Recently a rare sarcophagus (stone

coffin) which is 2000 years old from the

Iron AgendashMegalithic era was discovered

from a rock-cut cave at Viyur village of

Kollam near Koyilandy in Kozhikode

district Kerala

The coffin containing bone fragments

was found during an excavation ldquoSo

far such a rare finding has been

discovered only from two sites

in Kerala Both these sarcophagi were

recovered from Megalithic sites at

Chevayur and Atholi also in Kozhikode

district

Q54) Ans a)

The megalithic culture in South India was a full-fledged Iron Age culture

Q55) Ans d)

The Cholas Pandyas and Keralaputras

(Cheras) mentioned in Ashokan

inscriptions were probably in the late

megalithic phase of material culture

Q56) Ans d)

Q57) Ans (b)

Raj Kumar Shukla followed Gandhiji all

over the country to persuade him to

come to Champaran to investigate the

problem associated with tinkathia

system

RAUSIAS-FC19E1003 47

Brij Kishore Rajendra Prasad Mahadev

Desai and Narhari Parikh accompanied

Gandhi ji during the Champaran

Satyagraha

Q58) Ans (b)

The Satvahanas started the practice of granting tax-free villages to brahmanas and Buddhist monks

Q59) Ans c)

The objectives of the Programme are

listed as under

- Developing basic tourism

infrastructure

- Promoting cultural and heritage

value of the country to generate

livelihoods in the identified regions

- Enhancing the tourist attractiveness

in a sustainable manner by

developing world-class

infrastructure at the heritage

monument sites

- Creating employment through active

involvement of local communities

- Harnessing tourism potential for its

effects on employment generation

and economic development

- Developing sustainable tourism

infrastructure and ensuring proper

Operations and maintenance

therein

Q60) Ans (b)

The Tribal Cooperative Marketing

Development Federation of India

(TRIFED) came into existence in 1987

It is a national-level apex organization

functioning under the administrative

control of Ministry of Tribal Affairs

Govt of India

TRIFED has its registered and Head

Office located in New Delhi

Q61) Ans (c)

Premchandrsquos novels include

Premashram Rangabhumi Ghaban

Karmabhumi and Godan

Gora is a novel written by Rabindranath

Tagore

138th birth anniversary of Munshi

Premchand was celebrated across the

country

Q62) Ans (b)

Giddha is a traditional pastoral dance

performed by the women of the Punjab

India and Pakistan at festival times

and at the sowing and reaping of the

harvest

By this dance the Punjabi women

reveal their joy expel their suppressed

feelings in a male dominated society

through the performance of Giddha

Since this dance has nothing to do with

men only women can participate in it

During the Teej celebrations Giddha

dance is celebrated in Punjab every

year Teej is a generic name for a

number of festivals that are celebrated

by women in some parts of India

Q63) Ans (a)

Dara Shukoh wrote the remarkable

work called ldquoMajma-ul-Bahrainrdquo or the

ldquoThe confluence of two seasrdquo

The Vice President of India Shri M

Venkaiah Naidu has said that Prince

Dara Shukohrsquos writings can come as a

refreshing source for infusing peace and

harmony He was addressing the

gathering after visiting the exhibition

that showcases the forgotten Prince of

yesteryears Dara Shukoh organized by

Mr Francois Gautier at Indira Gandhi

National Centre for the Arts in New

Delhi

Q64) Ans (c)

The statue Gommateshwara is

dedicated to the Jain God Bahubali

It is a monolithic statue

President Ram Nath Kovind

inaugurated the grand anointing

ceremony mdash Mahamastakabhisheka mdash

held once in 12 years at

Shravanabelagola (Karnataka)

Q65) Ans (c)

Prachi Valley had come up around the

Prachi river Prachi Valley gradually

disappeared

RAUSIAS-FC19E1003 48

The Prachi river originates from

Bhubaneswar

It is a tributary of the Mahanadi and

flows through the districts of Puri

Khurda Cuttack and Jagatsinghpur

and the entire region of the river is

termed as the Prachi Valley

It falls into the Bay of Bengal

Archaeological evidence shows that the

Prachi Valley Civilisation predates both

Harappa and Mohenjo-Daro

The Prachi river originates from

Bhubaneswar

Q66) Ans (d)

These monuments are located in

Chhatarpur district Madhya Pradesh

within Vindhya mountain range

Q67) Ans (a)

The book lsquoThoughts on Pakistanrsquo was

written by Dr BR Ambedkar

On the occasion of the birth anniversary

of Dr BR Ambedkar the president of

India pays homage to this icon of India

In 1924 he founded the Depressed

Classes Institute (Bahishkrit Hitkarini

Sabha) and in 1927 the Samaj Samata

Sangh

Another area of attention for Ambedkar

was education For its spread among

the low classes he set up a network of

colleges by the name of Peoples

Education Society and founded hostels

Q68) Ans(b)

Mehrgarh is a famous Neolithic

settlement in the Indian subcontinent

which is situated in Baluchistan

province Pakistan

A pre-historic rock art site is discovered

in the vast expanse of limestone blocks

on the eastern banks of Naguleru river

near Dachepalli (Andhra Pradesh) It

has thrown light on the Neolithic

civilisation that flourished in Guntur

(Andhra Pradesh) during 1500-2000

BC

Q69) Ans (c)

The 12th and the 13th centuries saw

the emergence of the Kakatiyas They

were at first the feudatories of the

Western Chalukyas of Kalyana Initially

they ruled over a small territory near

Warangal (Telangana)

They introduced Nayakships which was

later adopted and developed by the

Rayas of Vijayanagara

Q70) Ans (a)

The fast had effect of putting pressure

on mill owners who finally agreed to

give the workers a 35 per cent increase

in wages

Google celebrated with a doodle the

132nd birth anniversary of Anasuya

Sarabhai who played a pioneering role

in Indiarsquos labour movement

Q71) Ans (d)

The UNESCOrsquos list of the representative

list of the intangible cultural heritage of

humanity from India are

- Koodiyattam Sanskrit Theatre of

Kerala

- Mudiyettu ritual theatre and dance

drama of Kerala

- Tradition of Vedic Chanting

- Kalbelia folk songs and dances of

Rajasthan

- Ramlila Traditional Performance of

the Ramayana

- Sankirtana ritual singing

drumming and dancing of Manipur

- Ramman religious festival and

ritual theatre of the Garhwal

Himalayas India

- Traditional brass and copper craft of

utensil making among the Thatheras

of Jandiala Guru Punjab India

- Chhau dance classical Indian dance

originated in the eastern Indian

states

- Buddhist chanting of Ladakh

recitation of sacred Buddhist texts

in the trans-Himalayan Ladakh

region Jammu and Kashmir India

- Yoga

- Nouroz

- Kumbh Mela

RAUSIAS-FC19E1003 49

Q72) Ans(b)

The President of India Shri Ram Nath Kovind inaugurated the Hornbill Festival and State Formation Day celebrations of Nagaland in Kisama

The festival is named after the Indian hornbill the large and colourful forest bird which is displayed in the folklore of most of the states tribes

The major recognized tribes of Nagaland are Angami Ao Chakhesang Chang

Kuki Rengma and Zeling etc

Onge Jarawa and Sentinelese are the

tribes of Andman amp Nicobar Islands

Q73) Ans (c)

The Rashtrakutas rule in the Deccan lasted for almost two hundred years till the end of the tenth century The Rashtrakutas rulers were tolerant in their religious views and patronized not only Shaivism and Vaishnavism but

Jainism as well

The famous rock-cut temple of Shiva at Ellora was built by one of the Rashtrakutas kings Krishna I in the ninth century His successor Amoghavarsha was a Jain but he also

patronized other faiths

The Rashtrakutas allowed Muslims traders to settle and permitted Islam to

be preached in their dominions

Recently increasing defacement at the prehistoric rock paintings of Pandavulagutta Telangana has created a cause for grave concern It can spoil

the prehistoric rock

Pandavulagutta is home to

- Painted rock shelters dating to

10000 BC-8000 BC

- An 8th century inscription of the

Rashtrakuta period and

- Painted frescoes from the 12th century Kakatiya empire

Q74) Ans (b)

In 1828 Raja Ram Mohan Roy founded a new religious society the Brahma Sabha later known as the Brahmo

Samaj

Debendranath Tagore headed the Tattvabodhini Sabha which was

engaged in search of spiritual truth

Its purpose was to purify Hinduism and to preach monotheism or belief in one God

The new society was to be based on the twin pillars of reason and the Vedas and

Upanishads

Recently Sadharan Brahmo Samaj (SBS) has entered into a legal battle with the West Bengal government due

to some legal issue

Q75) Ans (c)

The Chishti order was established in India by Khwaja Moinuddin Chishti who came to India around 1192 The Chishtirsquos are considered to be the most influential of the groups of Sufis who migrated to India in the late twelfth century They adapted successfully to the local environment and adopted several features of Indian devotional

traditions

The historical dargah of Sufi mystic Khwaja Moinuddin Chishti in Ajmer is all set to get a facelift This 13 th century dargah has been included among the Swachh Iconic Places a clean-up initiative focused on iconic

heritage spiritual and cultural places

Page 42: GENERAL STUDIES (PAPER I) · Test is part of Rau’s IAS Test series for Preliminary Exam 2019 FOUNDATION + CURRENT AFFAIRS GENERAL STUDIES (PAPER –I) FOUNDATION TEST –III TOPIC:

RAUSIAS-FC19E1003 42

Q8) Answer (a)

Explanation

Buddha belonged to the Sakya clan and

passed away at Kusinara

Buddha taught in Prakrit which was the

common language of people

Q9) Answer c

Explanation

There were six schools of philosophy in

ancient India These are known as

Vaishesika Nyaya Samkhya Yoga

Purva Mimansa and Vedanata or Uttara

Mimansa They were founded by sages

Kanada Gautama Kapila Patanjali

Jamini and Vyasa respectively

Q10) Answer b

Explanation

The teachings of Mahavira were

compiled at Valabhi in 6th century AD

Q11) Answer (c)

Explanation

Chanakya is traditionally identified as

Kautilya or Vishnugupta who authored

the ancient Indian political treatise the

Arthashastra

Q12) Answer d

The national emblem of India is an

adaptation of the Lion Capital atop the

Ashoka Pillar of Sarnath Uttar Pradesh

and is combined with the National

Motto Satyameva Jayate

The Rampurva Bull gets the name from

the site of its discovery Rampurva in

Bihar

It is noted for its delicately sculpted

model demonstrating superior

representation of soft flesh sensitive

nostrils alert ears and strong legs It is

a mixture of Indian and Persian

elements

Sankissa is situated in Uttar Pradesh

India

Q13) Ans(a)

Kunwar Singh was a notable leader during the Revolt of 1857 He belonged

to a royal house of Jagdispur Bihar

Q14) Answer b

Explanation

The term Vellalar was used for large

landowners

Q15) Answer c

Explanation

Arikamedu was a coastal settlement

where ships unloaded goods from

distant lands Finds here include a

massive brick warehouse pottery

including amphorae and Arretine ware

Roman lamps glassware and gems have

also been found at the site

Q16) Answer a

Explanation

Muvendar is a Tamil word mentioned in

Sangam poems meaning three chiefs

used for the heads of three ruling

families the Cholas Cheras and

Pandyas

Q17) Ans (c)

Several tribal or kin-based assemblies

such as the Sabha Vidatha and gana

are mentioned in the Rig-veda The

Sabha and the samiti mattered a great

deal in early Vedic times so much so

that the chiefs or the kings showed an

eagerness to win their support

Q18) Ans (a)

Jainism recognised the existence of the

gods but placed them lower than the

jina and did not condemn the varna

system as Buddhism did

Q19) Answer (d)

Explanation

Cholas and Pandyas had developed

powerful coastal cities The most

important city of Cholas was Puhar or

Kaveripattinam and Madurai was the

capital of Pandyas

Q20) Answer b

Explanation

Buddhacharita is the biography of

Buddha and was written by

RAUSIAS-FC19E1003 43

Ashvaghosha

Q21) Answer (a)

Explanation

Tamil poet Appar was a Shiva devotee

So he was a Nayanar saint

Q22) Answer d

Explanation

Samudragupta was a prominent Gupta

ruler whose coins depict him playing a

veena indicating his love for music We

get important historic information from

his Allahabad Prashasti which was

composed by his court poet Harisena

Q23) Answer (b)

Explanation

Vikrama Samvat was founded by

Chandragupta II in the 58 BC as a

mark of victory over the Shakas and

assumed the title of Vikramaditya

Banabhatta wrote Harshavardhanarsquos

biography the Harshacharita in

Sanskrit

Q24) Answer c

Explanation

Sandhi-vigrahika was the minister of

war and peace

Sarthavaha was the leader of the

merchant caravans

Q25) Answer a

Explanation

Xuan Zang (Hsuan-tsang) was a

Chinese traveller who came during the

reign of Harshavardhana

In the decade that began in 630 AD

Xuan Zang came to India through

Kashmir after visiting Central Asia Iran

and Afghanistan

He travelled from north to east and lived

in Bihar for a couple of years

At Nalanda University Xuan Zang

interacted with students and scholars

mastered local languages and

discovered Buddhist stupas

Q26) Answer c

Explanation

Pradakshina patha is a circular path

laid around a stupa in Buddhist

architecture While the rest are a part of

temple architecture

Q27) Answer d

Explanation

All the above-mentioned temples have

an elaborate use of bricks (baked

bricks) along with stone

Q28) Ans (c)

Muhammad Quli Qutab was the Sultan

of Golconda He was a contemporary of

Akbar was very fond of literature and

architecture

The Sultan was a great poet and he

wrote in Dakhini Urdu Persian and

Telgu and has left an extensive diwan or

collection

Recently the Archaeological Survey of

India (ASI) will be using Ground

Penetrating Radar (GPR) to map the

contours of the area around the Bagh-e-

Naya Qila excavated garden inside the

Golconda Fort in Telangana

Q29) Answer a

Explanation

Silappadikaram is a famous Tamil epic

which was written by Ilango around

1800 years ago It is a story of a

merchant named Kovalan who fell in

love with a courtesan named Madhavi

Manimekalai tells the story of the

daughter of Kovalan and Madhavi

Q30) Answer (a)

Explanation

Charaka is the author of Charaka

Samhita which is an important work of

Ayurveda and medicines

Brahmaguptarsquos fame rests mostly on his

Brahma-sphuta-siddhanta which was

an astronomical work It was translated

into Arabic in Baghdad and had a major

impact on Islamic mathematics and

astronomy

Late in his life Brahmagupta wrote

Khandakhadyaka which was an

RAUSIAS-FC19E1003 44

astronomical handbook that employed

Aryabhatarsquos system of starting each day

at midnight

Q31) Answer (c)

Explanation

Amir Khusrau was a famous sufi

musician poet and scholar In 1318 he

noted that there was different language

in every region of this land (Hindustan)

Lahori Kashmiri Dvarsamudri (in

Southern Karnataka) Telangana (in

Andhra Pradesh) Gujari (in Gujarat)

Marsquobari (in Tamil Nadu) Awadhi (in

eastern Uttar Pradesh) and Hindawai (in

the area around in Delhi) etc He went

to explain that Sanskrit did not belong

to any region and that only brahmans

knew it

Q32) Answer c

Explanation

Hiranyagarbha refers to the golden

womb When this ritual was performed

with the help of Brahmanas it was

thought to lead to the rebirth of the

sacrificer as a Khastriya

Q33) Answer d

Explanation

Kadamai refers to a tax on land

revenue

Gwalior Prashasti describes the exploits

of Nagabhata who was a Pratihara king

Q34) Answer b

Explanation

Rajatarangini is a Sanskrit text written

by Kalhana in the 12th century

It was historical chronicle of early India

It is justifiably considered to be the best

and most authentic work of its kind

It covers the entire span of history in

the Kashmir region from the earliest

times to the date of its composition

Q35) Answer c

Explanation

ldquoUrrdquo was the general assembly of the

village ldquoUrrdquo consisted of all the

taxpaying residents of an ordinary

village

Q36) Answer (a)

Explanation

Tarikh was a form of history writing in

the Delhi Sultanate The authors of

tawarikhs were learned men which

included secretaries administrators etc

Q37 Answer (a)

Explanation

Alauddin chose to pay his soldiers salaries in cash rather than iqtas The soldiers would buy their supplies from merchants in Delhi and it was thus feared that merchants would raise their prices To stop this Alauddin controlled the prices of goods in Delhi Prices were carefully surveyed by officers and merchants who did not sell at the prescribed rates were punished

Q38) Answer (d)

Explanation

Delhi first became the capital of a

kingdom under the Tomara Rajputs

who were defeated in the middle of the

twelfth century by the Chauhans (also

referred to as Chahamanas) of Ajmer

It was under the Tomaras and

Chauhans that Delhi became an

important commercial centre Many rich

Jaina merchants lived in the city and

constructed several temples Coins

minted here called dehliwal had a wide

circulation

Q39) Answer (c)

Explanation

Moth ki Masjid was built in the reign of

Sikandar Lodi by his minister

Begumpuri mosque built in the reign of

Muhammad Tughluq was the main

mosque of Jahanpanah the ldquoSanctuary

of the Worldrdquo and his new capital in

Delhi

Quwwat al ndash Islam mosque was

enlarged by Iltutmish and Alauddin

Khalji The minar was built by three

Sultansndash Qutbuddin Aybak Iltutmish

and Firuz Shah Tughluq

RAUSIAS-FC19E1003 45

Q40) Answer (c)

Explanation

Under the Mughals mansabdar was

referred to an individual who held a

mansab ie rank and he received his

salary as revenue assignments called

jagirs

Q41) Ans (b)

The Quit India Movement was a

spontaneous revolt of people against

British rule

The All India Congress Committee met

at Bombay on 8 August 1942 It passed

the famous resolution Quit India and

proposed the starting of a non-violent

mass struggle under Gandhis

leadership to achieve this aim But on

the very next day Gandhi and other

eminent leaders of the Congress were

arrested The Congress was once again

declared illegal

Q42) Ans (c)

The Simon Commission refers to a

group of seven MPs from the United

Kingdom constituted to suggest

constitutional reforms for British India

The Commission consisted of only

British members headed by one of the

senior British politicians Sir John

Simon

So the people of India agitated against

the arrival of Simon Commission

Q43) Ans (a)

He was widely known for his

unfavourable opinion of the economic

consequences of the British rule in

India

In his many writings and speeches and

especially in Poverty and Un-British

Rule in India Naoroji argued that India

was too highly taxed and that its wealth

was being drained away to England

He did not interpret the ancient Indian

texts and restored the self-confidence of

Indians And also he did not stress the

need for eradication of all the social

evils before anything else

Q44) Ans (c)

In August 1932 Prime Minister

MacDonald announced his Communal

Award Great Britainrsquos unilateral

attempt to resolve the various conflicts

among Indiarsquos many communal

interests

The award which was later

incorporated into the act of 1935

expanded the separate-electorate

formula reserved for Muslims to other

minorities including Sikhs Indian

Christians Anglo-Indians Europeans

distinct regional groups Gandhi

undertook a ldquofast unto deathrdquo against

that offer which he viewed as a

nefarious British plot to divide the

Indian society

Q45) Ans (b)

In British India apart from existing

imports and exports there was also a

particular amount of money which

colonial India contributed towards

administration maintenance of the

army war expenses pensions to retired

officers and other expenses accrued by

Britain towards maintenance of her

colony These were known as Home

charges and were paid for almost

entirely by India

The Home charges was made of

following components-

- Interest payable on Indian debt

- Dividend to shareholders of East

India Company

- Funds used to support the India

Office in London

- Funds used to pay salaries and

pensions of British personnel

engaged in India

- Interest on the railways

- Civil and military charges

- Store purchases in England

Q46) Ans (b)

The Lahore session of the Indian

National Congress was held in 1929

under the Presidentship of Jawaharlal

Nehru

The Lahore session of the Indian

National Congress witnessed significant

RAUSIAS-FC19E1003 46

developments in the Indian national

movement

- First the election of Jawaharlal

Nehru to the post of Presidentship of

the Congress was a clear indication

of the growing strength of the

Leftists in the Congress

- Secondly it was in this session that

the Congress for the first time raised

the demand for complete

independence Such demand was

not raised from the Congress

platform earlier

Q47) Ans (b)

It did not provide for separate

electorates for any community or

weightage for minorities However it did

allow for the reservation of minority

seats in provinces having minorities of

at least ten per cent but this was to be

in strict proportion to the size of the

community

There was no provision for complete

Independence for India

Q48) Ans (c)

The religion of early Vedic Aryans was

primarily of worship of nature and

Yajnas

The early Aryan religion was kind of

nature worship Actually the forces

around them which they could not

control or understand were invested

with divinity and were personified as

male or female gods And they

performed some Yajnas also

Q49) Ans (b)

The roads and river-routes were not

immune from robbery It is notable that

Yuan Chwang (Hiuen Tsang) was

robbed of his belongings during

Harshvardanarsquos period

Q50) Ans (c)

Q51) Ans (b)

Purandara Dasa was a saint and great

devotee of Lord Krishna

There is much speculation about where

Purandara Dasa regarded as the

Pitamaha of Carnatic music was born

Recently an expert committee

constituted by the Kannada University

Hampi has come to the conclusion that

Kshemapura Shivamogga district

Karnataka is the birth place of

Purandara Dasa

Q52) Ans (c)

Sri Tyagaraja Sri Shyama Shastry and Sri Muthuswami Dikshitar are considered the trinity of Carnatic music and with them came the golden age in Carnatic music in the 18th-19th

century

Q53) Ans d)

Recently a rare sarcophagus (stone

coffin) which is 2000 years old from the

Iron AgendashMegalithic era was discovered

from a rock-cut cave at Viyur village of

Kollam near Koyilandy in Kozhikode

district Kerala

The coffin containing bone fragments

was found during an excavation ldquoSo

far such a rare finding has been

discovered only from two sites

in Kerala Both these sarcophagi were

recovered from Megalithic sites at

Chevayur and Atholi also in Kozhikode

district

Q54) Ans a)

The megalithic culture in South India was a full-fledged Iron Age culture

Q55) Ans d)

The Cholas Pandyas and Keralaputras

(Cheras) mentioned in Ashokan

inscriptions were probably in the late

megalithic phase of material culture

Q56) Ans d)

Q57) Ans (b)

Raj Kumar Shukla followed Gandhiji all

over the country to persuade him to

come to Champaran to investigate the

problem associated with tinkathia

system

RAUSIAS-FC19E1003 47

Brij Kishore Rajendra Prasad Mahadev

Desai and Narhari Parikh accompanied

Gandhi ji during the Champaran

Satyagraha

Q58) Ans (b)

The Satvahanas started the practice of granting tax-free villages to brahmanas and Buddhist monks

Q59) Ans c)

The objectives of the Programme are

listed as under

- Developing basic tourism

infrastructure

- Promoting cultural and heritage

value of the country to generate

livelihoods in the identified regions

- Enhancing the tourist attractiveness

in a sustainable manner by

developing world-class

infrastructure at the heritage

monument sites

- Creating employment through active

involvement of local communities

- Harnessing tourism potential for its

effects on employment generation

and economic development

- Developing sustainable tourism

infrastructure and ensuring proper

Operations and maintenance

therein

Q60) Ans (b)

The Tribal Cooperative Marketing

Development Federation of India

(TRIFED) came into existence in 1987

It is a national-level apex organization

functioning under the administrative

control of Ministry of Tribal Affairs

Govt of India

TRIFED has its registered and Head

Office located in New Delhi

Q61) Ans (c)

Premchandrsquos novels include

Premashram Rangabhumi Ghaban

Karmabhumi and Godan

Gora is a novel written by Rabindranath

Tagore

138th birth anniversary of Munshi

Premchand was celebrated across the

country

Q62) Ans (b)

Giddha is a traditional pastoral dance

performed by the women of the Punjab

India and Pakistan at festival times

and at the sowing and reaping of the

harvest

By this dance the Punjabi women

reveal their joy expel their suppressed

feelings in a male dominated society

through the performance of Giddha

Since this dance has nothing to do with

men only women can participate in it

During the Teej celebrations Giddha

dance is celebrated in Punjab every

year Teej is a generic name for a

number of festivals that are celebrated

by women in some parts of India

Q63) Ans (a)

Dara Shukoh wrote the remarkable

work called ldquoMajma-ul-Bahrainrdquo or the

ldquoThe confluence of two seasrdquo

The Vice President of India Shri M

Venkaiah Naidu has said that Prince

Dara Shukohrsquos writings can come as a

refreshing source for infusing peace and

harmony He was addressing the

gathering after visiting the exhibition

that showcases the forgotten Prince of

yesteryears Dara Shukoh organized by

Mr Francois Gautier at Indira Gandhi

National Centre for the Arts in New

Delhi

Q64) Ans (c)

The statue Gommateshwara is

dedicated to the Jain God Bahubali

It is a monolithic statue

President Ram Nath Kovind

inaugurated the grand anointing

ceremony mdash Mahamastakabhisheka mdash

held once in 12 years at

Shravanabelagola (Karnataka)

Q65) Ans (c)

Prachi Valley had come up around the

Prachi river Prachi Valley gradually

disappeared

RAUSIAS-FC19E1003 48

The Prachi river originates from

Bhubaneswar

It is a tributary of the Mahanadi and

flows through the districts of Puri

Khurda Cuttack and Jagatsinghpur

and the entire region of the river is

termed as the Prachi Valley

It falls into the Bay of Bengal

Archaeological evidence shows that the

Prachi Valley Civilisation predates both

Harappa and Mohenjo-Daro

The Prachi river originates from

Bhubaneswar

Q66) Ans (d)

These monuments are located in

Chhatarpur district Madhya Pradesh

within Vindhya mountain range

Q67) Ans (a)

The book lsquoThoughts on Pakistanrsquo was

written by Dr BR Ambedkar

On the occasion of the birth anniversary

of Dr BR Ambedkar the president of

India pays homage to this icon of India

In 1924 he founded the Depressed

Classes Institute (Bahishkrit Hitkarini

Sabha) and in 1927 the Samaj Samata

Sangh

Another area of attention for Ambedkar

was education For its spread among

the low classes he set up a network of

colleges by the name of Peoples

Education Society and founded hostels

Q68) Ans(b)

Mehrgarh is a famous Neolithic

settlement in the Indian subcontinent

which is situated in Baluchistan

province Pakistan

A pre-historic rock art site is discovered

in the vast expanse of limestone blocks

on the eastern banks of Naguleru river

near Dachepalli (Andhra Pradesh) It

has thrown light on the Neolithic

civilisation that flourished in Guntur

(Andhra Pradesh) during 1500-2000

BC

Q69) Ans (c)

The 12th and the 13th centuries saw

the emergence of the Kakatiyas They

were at first the feudatories of the

Western Chalukyas of Kalyana Initially

they ruled over a small territory near

Warangal (Telangana)

They introduced Nayakships which was

later adopted and developed by the

Rayas of Vijayanagara

Q70) Ans (a)

The fast had effect of putting pressure

on mill owners who finally agreed to

give the workers a 35 per cent increase

in wages

Google celebrated with a doodle the

132nd birth anniversary of Anasuya

Sarabhai who played a pioneering role

in Indiarsquos labour movement

Q71) Ans (d)

The UNESCOrsquos list of the representative

list of the intangible cultural heritage of

humanity from India are

- Koodiyattam Sanskrit Theatre of

Kerala

- Mudiyettu ritual theatre and dance

drama of Kerala

- Tradition of Vedic Chanting

- Kalbelia folk songs and dances of

Rajasthan

- Ramlila Traditional Performance of

the Ramayana

- Sankirtana ritual singing

drumming and dancing of Manipur

- Ramman religious festival and

ritual theatre of the Garhwal

Himalayas India

- Traditional brass and copper craft of

utensil making among the Thatheras

of Jandiala Guru Punjab India

- Chhau dance classical Indian dance

originated in the eastern Indian

states

- Buddhist chanting of Ladakh

recitation of sacred Buddhist texts

in the trans-Himalayan Ladakh

region Jammu and Kashmir India

- Yoga

- Nouroz

- Kumbh Mela

RAUSIAS-FC19E1003 49

Q72) Ans(b)

The President of India Shri Ram Nath Kovind inaugurated the Hornbill Festival and State Formation Day celebrations of Nagaland in Kisama

The festival is named after the Indian hornbill the large and colourful forest bird which is displayed in the folklore of most of the states tribes

The major recognized tribes of Nagaland are Angami Ao Chakhesang Chang

Kuki Rengma and Zeling etc

Onge Jarawa and Sentinelese are the

tribes of Andman amp Nicobar Islands

Q73) Ans (c)

The Rashtrakutas rule in the Deccan lasted for almost two hundred years till the end of the tenth century The Rashtrakutas rulers were tolerant in their religious views and patronized not only Shaivism and Vaishnavism but

Jainism as well

The famous rock-cut temple of Shiva at Ellora was built by one of the Rashtrakutas kings Krishna I in the ninth century His successor Amoghavarsha was a Jain but he also

patronized other faiths

The Rashtrakutas allowed Muslims traders to settle and permitted Islam to

be preached in their dominions

Recently increasing defacement at the prehistoric rock paintings of Pandavulagutta Telangana has created a cause for grave concern It can spoil

the prehistoric rock

Pandavulagutta is home to

- Painted rock shelters dating to

10000 BC-8000 BC

- An 8th century inscription of the

Rashtrakuta period and

- Painted frescoes from the 12th century Kakatiya empire

Q74) Ans (b)

In 1828 Raja Ram Mohan Roy founded a new religious society the Brahma Sabha later known as the Brahmo

Samaj

Debendranath Tagore headed the Tattvabodhini Sabha which was

engaged in search of spiritual truth

Its purpose was to purify Hinduism and to preach monotheism or belief in one God

The new society was to be based on the twin pillars of reason and the Vedas and

Upanishads

Recently Sadharan Brahmo Samaj (SBS) has entered into a legal battle with the West Bengal government due

to some legal issue

Q75) Ans (c)

The Chishti order was established in India by Khwaja Moinuddin Chishti who came to India around 1192 The Chishtirsquos are considered to be the most influential of the groups of Sufis who migrated to India in the late twelfth century They adapted successfully to the local environment and adopted several features of Indian devotional

traditions

The historical dargah of Sufi mystic Khwaja Moinuddin Chishti in Ajmer is all set to get a facelift This 13 th century dargah has been included among the Swachh Iconic Places a clean-up initiative focused on iconic

heritage spiritual and cultural places

Page 43: GENERAL STUDIES (PAPER I) · Test is part of Rau’s IAS Test series for Preliminary Exam 2019 FOUNDATION + CURRENT AFFAIRS GENERAL STUDIES (PAPER –I) FOUNDATION TEST –III TOPIC:

RAUSIAS-FC19E1003 43

Ashvaghosha

Q21) Answer (a)

Explanation

Tamil poet Appar was a Shiva devotee

So he was a Nayanar saint

Q22) Answer d

Explanation

Samudragupta was a prominent Gupta

ruler whose coins depict him playing a

veena indicating his love for music We

get important historic information from

his Allahabad Prashasti which was

composed by his court poet Harisena

Q23) Answer (b)

Explanation

Vikrama Samvat was founded by

Chandragupta II in the 58 BC as a

mark of victory over the Shakas and

assumed the title of Vikramaditya

Banabhatta wrote Harshavardhanarsquos

biography the Harshacharita in

Sanskrit

Q24) Answer c

Explanation

Sandhi-vigrahika was the minister of

war and peace

Sarthavaha was the leader of the

merchant caravans

Q25) Answer a

Explanation

Xuan Zang (Hsuan-tsang) was a

Chinese traveller who came during the

reign of Harshavardhana

In the decade that began in 630 AD

Xuan Zang came to India through

Kashmir after visiting Central Asia Iran

and Afghanistan

He travelled from north to east and lived

in Bihar for a couple of years

At Nalanda University Xuan Zang

interacted with students and scholars

mastered local languages and

discovered Buddhist stupas

Q26) Answer c

Explanation

Pradakshina patha is a circular path

laid around a stupa in Buddhist

architecture While the rest are a part of

temple architecture

Q27) Answer d

Explanation

All the above-mentioned temples have

an elaborate use of bricks (baked

bricks) along with stone

Q28) Ans (c)

Muhammad Quli Qutab was the Sultan

of Golconda He was a contemporary of

Akbar was very fond of literature and

architecture

The Sultan was a great poet and he

wrote in Dakhini Urdu Persian and

Telgu and has left an extensive diwan or

collection

Recently the Archaeological Survey of

India (ASI) will be using Ground

Penetrating Radar (GPR) to map the

contours of the area around the Bagh-e-

Naya Qila excavated garden inside the

Golconda Fort in Telangana

Q29) Answer a

Explanation

Silappadikaram is a famous Tamil epic

which was written by Ilango around

1800 years ago It is a story of a

merchant named Kovalan who fell in

love with a courtesan named Madhavi

Manimekalai tells the story of the

daughter of Kovalan and Madhavi

Q30) Answer (a)

Explanation

Charaka is the author of Charaka

Samhita which is an important work of

Ayurveda and medicines

Brahmaguptarsquos fame rests mostly on his

Brahma-sphuta-siddhanta which was

an astronomical work It was translated

into Arabic in Baghdad and had a major

impact on Islamic mathematics and

astronomy

Late in his life Brahmagupta wrote

Khandakhadyaka which was an

RAUSIAS-FC19E1003 44

astronomical handbook that employed

Aryabhatarsquos system of starting each day

at midnight

Q31) Answer (c)

Explanation

Amir Khusrau was a famous sufi

musician poet and scholar In 1318 he

noted that there was different language

in every region of this land (Hindustan)

Lahori Kashmiri Dvarsamudri (in

Southern Karnataka) Telangana (in

Andhra Pradesh) Gujari (in Gujarat)

Marsquobari (in Tamil Nadu) Awadhi (in

eastern Uttar Pradesh) and Hindawai (in

the area around in Delhi) etc He went

to explain that Sanskrit did not belong

to any region and that only brahmans

knew it

Q32) Answer c

Explanation

Hiranyagarbha refers to the golden

womb When this ritual was performed

with the help of Brahmanas it was

thought to lead to the rebirth of the

sacrificer as a Khastriya

Q33) Answer d

Explanation

Kadamai refers to a tax on land

revenue

Gwalior Prashasti describes the exploits

of Nagabhata who was a Pratihara king

Q34) Answer b

Explanation

Rajatarangini is a Sanskrit text written

by Kalhana in the 12th century

It was historical chronicle of early India

It is justifiably considered to be the best

and most authentic work of its kind

It covers the entire span of history in

the Kashmir region from the earliest

times to the date of its composition

Q35) Answer c

Explanation

ldquoUrrdquo was the general assembly of the

village ldquoUrrdquo consisted of all the

taxpaying residents of an ordinary

village

Q36) Answer (a)

Explanation

Tarikh was a form of history writing in

the Delhi Sultanate The authors of

tawarikhs were learned men which

included secretaries administrators etc

Q37 Answer (a)

Explanation

Alauddin chose to pay his soldiers salaries in cash rather than iqtas The soldiers would buy their supplies from merchants in Delhi and it was thus feared that merchants would raise their prices To stop this Alauddin controlled the prices of goods in Delhi Prices were carefully surveyed by officers and merchants who did not sell at the prescribed rates were punished

Q38) Answer (d)

Explanation

Delhi first became the capital of a

kingdom under the Tomara Rajputs

who were defeated in the middle of the

twelfth century by the Chauhans (also

referred to as Chahamanas) of Ajmer

It was under the Tomaras and

Chauhans that Delhi became an

important commercial centre Many rich

Jaina merchants lived in the city and

constructed several temples Coins

minted here called dehliwal had a wide

circulation

Q39) Answer (c)

Explanation

Moth ki Masjid was built in the reign of

Sikandar Lodi by his minister

Begumpuri mosque built in the reign of

Muhammad Tughluq was the main

mosque of Jahanpanah the ldquoSanctuary

of the Worldrdquo and his new capital in

Delhi

Quwwat al ndash Islam mosque was

enlarged by Iltutmish and Alauddin

Khalji The minar was built by three

Sultansndash Qutbuddin Aybak Iltutmish

and Firuz Shah Tughluq

RAUSIAS-FC19E1003 45

Q40) Answer (c)

Explanation

Under the Mughals mansabdar was

referred to an individual who held a

mansab ie rank and he received his

salary as revenue assignments called

jagirs

Q41) Ans (b)

The Quit India Movement was a

spontaneous revolt of people against

British rule

The All India Congress Committee met

at Bombay on 8 August 1942 It passed

the famous resolution Quit India and

proposed the starting of a non-violent

mass struggle under Gandhis

leadership to achieve this aim But on

the very next day Gandhi and other

eminent leaders of the Congress were

arrested The Congress was once again

declared illegal

Q42) Ans (c)

The Simon Commission refers to a

group of seven MPs from the United

Kingdom constituted to suggest

constitutional reforms for British India

The Commission consisted of only

British members headed by one of the

senior British politicians Sir John

Simon

So the people of India agitated against

the arrival of Simon Commission

Q43) Ans (a)

He was widely known for his

unfavourable opinion of the economic

consequences of the British rule in

India

In his many writings and speeches and

especially in Poverty and Un-British

Rule in India Naoroji argued that India

was too highly taxed and that its wealth

was being drained away to England

He did not interpret the ancient Indian

texts and restored the self-confidence of

Indians And also he did not stress the

need for eradication of all the social

evils before anything else

Q44) Ans (c)

In August 1932 Prime Minister

MacDonald announced his Communal

Award Great Britainrsquos unilateral

attempt to resolve the various conflicts

among Indiarsquos many communal

interests

The award which was later

incorporated into the act of 1935

expanded the separate-electorate

formula reserved for Muslims to other

minorities including Sikhs Indian

Christians Anglo-Indians Europeans

distinct regional groups Gandhi

undertook a ldquofast unto deathrdquo against

that offer which he viewed as a

nefarious British plot to divide the

Indian society

Q45) Ans (b)

In British India apart from existing

imports and exports there was also a

particular amount of money which

colonial India contributed towards

administration maintenance of the

army war expenses pensions to retired

officers and other expenses accrued by

Britain towards maintenance of her

colony These were known as Home

charges and were paid for almost

entirely by India

The Home charges was made of

following components-

- Interest payable on Indian debt

- Dividend to shareholders of East

India Company

- Funds used to support the India

Office in London

- Funds used to pay salaries and

pensions of British personnel

engaged in India

- Interest on the railways

- Civil and military charges

- Store purchases in England

Q46) Ans (b)

The Lahore session of the Indian

National Congress was held in 1929

under the Presidentship of Jawaharlal

Nehru

The Lahore session of the Indian

National Congress witnessed significant

RAUSIAS-FC19E1003 46

developments in the Indian national

movement

- First the election of Jawaharlal

Nehru to the post of Presidentship of

the Congress was a clear indication

of the growing strength of the

Leftists in the Congress

- Secondly it was in this session that

the Congress for the first time raised

the demand for complete

independence Such demand was

not raised from the Congress

platform earlier

Q47) Ans (b)

It did not provide for separate

electorates for any community or

weightage for minorities However it did

allow for the reservation of minority

seats in provinces having minorities of

at least ten per cent but this was to be

in strict proportion to the size of the

community

There was no provision for complete

Independence for India

Q48) Ans (c)

The religion of early Vedic Aryans was

primarily of worship of nature and

Yajnas

The early Aryan religion was kind of

nature worship Actually the forces

around them which they could not

control or understand were invested

with divinity and were personified as

male or female gods And they

performed some Yajnas also

Q49) Ans (b)

The roads and river-routes were not

immune from robbery It is notable that

Yuan Chwang (Hiuen Tsang) was

robbed of his belongings during

Harshvardanarsquos period

Q50) Ans (c)

Q51) Ans (b)

Purandara Dasa was a saint and great

devotee of Lord Krishna

There is much speculation about where

Purandara Dasa regarded as the

Pitamaha of Carnatic music was born

Recently an expert committee

constituted by the Kannada University

Hampi has come to the conclusion that

Kshemapura Shivamogga district

Karnataka is the birth place of

Purandara Dasa

Q52) Ans (c)

Sri Tyagaraja Sri Shyama Shastry and Sri Muthuswami Dikshitar are considered the trinity of Carnatic music and with them came the golden age in Carnatic music in the 18th-19th

century

Q53) Ans d)

Recently a rare sarcophagus (stone

coffin) which is 2000 years old from the

Iron AgendashMegalithic era was discovered

from a rock-cut cave at Viyur village of

Kollam near Koyilandy in Kozhikode

district Kerala

The coffin containing bone fragments

was found during an excavation ldquoSo

far such a rare finding has been

discovered only from two sites

in Kerala Both these sarcophagi were

recovered from Megalithic sites at

Chevayur and Atholi also in Kozhikode

district

Q54) Ans a)

The megalithic culture in South India was a full-fledged Iron Age culture

Q55) Ans d)

The Cholas Pandyas and Keralaputras

(Cheras) mentioned in Ashokan

inscriptions were probably in the late

megalithic phase of material culture

Q56) Ans d)

Q57) Ans (b)

Raj Kumar Shukla followed Gandhiji all

over the country to persuade him to

come to Champaran to investigate the

problem associated with tinkathia

system

RAUSIAS-FC19E1003 47

Brij Kishore Rajendra Prasad Mahadev

Desai and Narhari Parikh accompanied

Gandhi ji during the Champaran

Satyagraha

Q58) Ans (b)

The Satvahanas started the practice of granting tax-free villages to brahmanas and Buddhist monks

Q59) Ans c)

The objectives of the Programme are

listed as under

- Developing basic tourism

infrastructure

- Promoting cultural and heritage

value of the country to generate

livelihoods in the identified regions

- Enhancing the tourist attractiveness

in a sustainable manner by

developing world-class

infrastructure at the heritage

monument sites

- Creating employment through active

involvement of local communities

- Harnessing tourism potential for its

effects on employment generation

and economic development

- Developing sustainable tourism

infrastructure and ensuring proper

Operations and maintenance

therein

Q60) Ans (b)

The Tribal Cooperative Marketing

Development Federation of India

(TRIFED) came into existence in 1987

It is a national-level apex organization

functioning under the administrative

control of Ministry of Tribal Affairs

Govt of India

TRIFED has its registered and Head

Office located in New Delhi

Q61) Ans (c)

Premchandrsquos novels include

Premashram Rangabhumi Ghaban

Karmabhumi and Godan

Gora is a novel written by Rabindranath

Tagore

138th birth anniversary of Munshi

Premchand was celebrated across the

country

Q62) Ans (b)

Giddha is a traditional pastoral dance

performed by the women of the Punjab

India and Pakistan at festival times

and at the sowing and reaping of the

harvest

By this dance the Punjabi women

reveal their joy expel their suppressed

feelings in a male dominated society

through the performance of Giddha

Since this dance has nothing to do with

men only women can participate in it

During the Teej celebrations Giddha

dance is celebrated in Punjab every

year Teej is a generic name for a

number of festivals that are celebrated

by women in some parts of India

Q63) Ans (a)

Dara Shukoh wrote the remarkable

work called ldquoMajma-ul-Bahrainrdquo or the

ldquoThe confluence of two seasrdquo

The Vice President of India Shri M

Venkaiah Naidu has said that Prince

Dara Shukohrsquos writings can come as a

refreshing source for infusing peace and

harmony He was addressing the

gathering after visiting the exhibition

that showcases the forgotten Prince of

yesteryears Dara Shukoh organized by

Mr Francois Gautier at Indira Gandhi

National Centre for the Arts in New

Delhi

Q64) Ans (c)

The statue Gommateshwara is

dedicated to the Jain God Bahubali

It is a monolithic statue

President Ram Nath Kovind

inaugurated the grand anointing

ceremony mdash Mahamastakabhisheka mdash

held once in 12 years at

Shravanabelagola (Karnataka)

Q65) Ans (c)

Prachi Valley had come up around the

Prachi river Prachi Valley gradually

disappeared

RAUSIAS-FC19E1003 48

The Prachi river originates from

Bhubaneswar

It is a tributary of the Mahanadi and

flows through the districts of Puri

Khurda Cuttack and Jagatsinghpur

and the entire region of the river is

termed as the Prachi Valley

It falls into the Bay of Bengal

Archaeological evidence shows that the

Prachi Valley Civilisation predates both

Harappa and Mohenjo-Daro

The Prachi river originates from

Bhubaneswar

Q66) Ans (d)

These monuments are located in

Chhatarpur district Madhya Pradesh

within Vindhya mountain range

Q67) Ans (a)

The book lsquoThoughts on Pakistanrsquo was

written by Dr BR Ambedkar

On the occasion of the birth anniversary

of Dr BR Ambedkar the president of

India pays homage to this icon of India

In 1924 he founded the Depressed

Classes Institute (Bahishkrit Hitkarini

Sabha) and in 1927 the Samaj Samata

Sangh

Another area of attention for Ambedkar

was education For its spread among

the low classes he set up a network of

colleges by the name of Peoples

Education Society and founded hostels

Q68) Ans(b)

Mehrgarh is a famous Neolithic

settlement in the Indian subcontinent

which is situated in Baluchistan

province Pakistan

A pre-historic rock art site is discovered

in the vast expanse of limestone blocks

on the eastern banks of Naguleru river

near Dachepalli (Andhra Pradesh) It

has thrown light on the Neolithic

civilisation that flourished in Guntur

(Andhra Pradesh) during 1500-2000

BC

Q69) Ans (c)

The 12th and the 13th centuries saw

the emergence of the Kakatiyas They

were at first the feudatories of the

Western Chalukyas of Kalyana Initially

they ruled over a small territory near

Warangal (Telangana)

They introduced Nayakships which was

later adopted and developed by the

Rayas of Vijayanagara

Q70) Ans (a)

The fast had effect of putting pressure

on mill owners who finally agreed to

give the workers a 35 per cent increase

in wages

Google celebrated with a doodle the

132nd birth anniversary of Anasuya

Sarabhai who played a pioneering role

in Indiarsquos labour movement

Q71) Ans (d)

The UNESCOrsquos list of the representative

list of the intangible cultural heritage of

humanity from India are

- Koodiyattam Sanskrit Theatre of

Kerala

- Mudiyettu ritual theatre and dance

drama of Kerala

- Tradition of Vedic Chanting

- Kalbelia folk songs and dances of

Rajasthan

- Ramlila Traditional Performance of

the Ramayana

- Sankirtana ritual singing

drumming and dancing of Manipur

- Ramman religious festival and

ritual theatre of the Garhwal

Himalayas India

- Traditional brass and copper craft of

utensil making among the Thatheras

of Jandiala Guru Punjab India

- Chhau dance classical Indian dance

originated in the eastern Indian

states

- Buddhist chanting of Ladakh

recitation of sacred Buddhist texts

in the trans-Himalayan Ladakh

region Jammu and Kashmir India

- Yoga

- Nouroz

- Kumbh Mela

RAUSIAS-FC19E1003 49

Q72) Ans(b)

The President of India Shri Ram Nath Kovind inaugurated the Hornbill Festival and State Formation Day celebrations of Nagaland in Kisama

The festival is named after the Indian hornbill the large and colourful forest bird which is displayed in the folklore of most of the states tribes

The major recognized tribes of Nagaland are Angami Ao Chakhesang Chang

Kuki Rengma and Zeling etc

Onge Jarawa and Sentinelese are the

tribes of Andman amp Nicobar Islands

Q73) Ans (c)

The Rashtrakutas rule in the Deccan lasted for almost two hundred years till the end of the tenth century The Rashtrakutas rulers were tolerant in their religious views and patronized not only Shaivism and Vaishnavism but

Jainism as well

The famous rock-cut temple of Shiva at Ellora was built by one of the Rashtrakutas kings Krishna I in the ninth century His successor Amoghavarsha was a Jain but he also

patronized other faiths

The Rashtrakutas allowed Muslims traders to settle and permitted Islam to

be preached in their dominions

Recently increasing defacement at the prehistoric rock paintings of Pandavulagutta Telangana has created a cause for grave concern It can spoil

the prehistoric rock

Pandavulagutta is home to

- Painted rock shelters dating to

10000 BC-8000 BC

- An 8th century inscription of the

Rashtrakuta period and

- Painted frescoes from the 12th century Kakatiya empire

Q74) Ans (b)

In 1828 Raja Ram Mohan Roy founded a new religious society the Brahma Sabha later known as the Brahmo

Samaj

Debendranath Tagore headed the Tattvabodhini Sabha which was

engaged in search of spiritual truth

Its purpose was to purify Hinduism and to preach monotheism or belief in one God

The new society was to be based on the twin pillars of reason and the Vedas and

Upanishads

Recently Sadharan Brahmo Samaj (SBS) has entered into a legal battle with the West Bengal government due

to some legal issue

Q75) Ans (c)

The Chishti order was established in India by Khwaja Moinuddin Chishti who came to India around 1192 The Chishtirsquos are considered to be the most influential of the groups of Sufis who migrated to India in the late twelfth century They adapted successfully to the local environment and adopted several features of Indian devotional

traditions

The historical dargah of Sufi mystic Khwaja Moinuddin Chishti in Ajmer is all set to get a facelift This 13 th century dargah has been included among the Swachh Iconic Places a clean-up initiative focused on iconic

heritage spiritual and cultural places

Page 44: GENERAL STUDIES (PAPER I) · Test is part of Rau’s IAS Test series for Preliminary Exam 2019 FOUNDATION + CURRENT AFFAIRS GENERAL STUDIES (PAPER –I) FOUNDATION TEST –III TOPIC:

RAUSIAS-FC19E1003 44

astronomical handbook that employed

Aryabhatarsquos system of starting each day

at midnight

Q31) Answer (c)

Explanation

Amir Khusrau was a famous sufi

musician poet and scholar In 1318 he

noted that there was different language

in every region of this land (Hindustan)

Lahori Kashmiri Dvarsamudri (in

Southern Karnataka) Telangana (in

Andhra Pradesh) Gujari (in Gujarat)

Marsquobari (in Tamil Nadu) Awadhi (in

eastern Uttar Pradesh) and Hindawai (in

the area around in Delhi) etc He went

to explain that Sanskrit did not belong

to any region and that only brahmans

knew it

Q32) Answer c

Explanation

Hiranyagarbha refers to the golden

womb When this ritual was performed

with the help of Brahmanas it was

thought to lead to the rebirth of the

sacrificer as a Khastriya

Q33) Answer d

Explanation

Kadamai refers to a tax on land

revenue

Gwalior Prashasti describes the exploits

of Nagabhata who was a Pratihara king

Q34) Answer b

Explanation

Rajatarangini is a Sanskrit text written

by Kalhana in the 12th century

It was historical chronicle of early India

It is justifiably considered to be the best

and most authentic work of its kind

It covers the entire span of history in

the Kashmir region from the earliest

times to the date of its composition

Q35) Answer c

Explanation

ldquoUrrdquo was the general assembly of the

village ldquoUrrdquo consisted of all the

taxpaying residents of an ordinary

village

Q36) Answer (a)

Explanation

Tarikh was a form of history writing in

the Delhi Sultanate The authors of

tawarikhs were learned men which

included secretaries administrators etc

Q37 Answer (a)

Explanation

Alauddin chose to pay his soldiers salaries in cash rather than iqtas The soldiers would buy their supplies from merchants in Delhi and it was thus feared that merchants would raise their prices To stop this Alauddin controlled the prices of goods in Delhi Prices were carefully surveyed by officers and merchants who did not sell at the prescribed rates were punished

Q38) Answer (d)

Explanation

Delhi first became the capital of a

kingdom under the Tomara Rajputs

who were defeated in the middle of the

twelfth century by the Chauhans (also

referred to as Chahamanas) of Ajmer

It was under the Tomaras and

Chauhans that Delhi became an

important commercial centre Many rich

Jaina merchants lived in the city and

constructed several temples Coins

minted here called dehliwal had a wide

circulation

Q39) Answer (c)

Explanation

Moth ki Masjid was built in the reign of

Sikandar Lodi by his minister

Begumpuri mosque built in the reign of

Muhammad Tughluq was the main

mosque of Jahanpanah the ldquoSanctuary

of the Worldrdquo and his new capital in

Delhi

Quwwat al ndash Islam mosque was

enlarged by Iltutmish and Alauddin

Khalji The minar was built by three

Sultansndash Qutbuddin Aybak Iltutmish

and Firuz Shah Tughluq

RAUSIAS-FC19E1003 45

Q40) Answer (c)

Explanation

Under the Mughals mansabdar was

referred to an individual who held a

mansab ie rank and he received his

salary as revenue assignments called

jagirs

Q41) Ans (b)

The Quit India Movement was a

spontaneous revolt of people against

British rule

The All India Congress Committee met

at Bombay on 8 August 1942 It passed

the famous resolution Quit India and

proposed the starting of a non-violent

mass struggle under Gandhis

leadership to achieve this aim But on

the very next day Gandhi and other

eminent leaders of the Congress were

arrested The Congress was once again

declared illegal

Q42) Ans (c)

The Simon Commission refers to a

group of seven MPs from the United

Kingdom constituted to suggest

constitutional reforms for British India

The Commission consisted of only

British members headed by one of the

senior British politicians Sir John

Simon

So the people of India agitated against

the arrival of Simon Commission

Q43) Ans (a)

He was widely known for his

unfavourable opinion of the economic

consequences of the British rule in

India

In his many writings and speeches and

especially in Poverty and Un-British

Rule in India Naoroji argued that India

was too highly taxed and that its wealth

was being drained away to England

He did not interpret the ancient Indian

texts and restored the self-confidence of

Indians And also he did not stress the

need for eradication of all the social

evils before anything else

Q44) Ans (c)

In August 1932 Prime Minister

MacDonald announced his Communal

Award Great Britainrsquos unilateral

attempt to resolve the various conflicts

among Indiarsquos many communal

interests

The award which was later

incorporated into the act of 1935

expanded the separate-electorate

formula reserved for Muslims to other

minorities including Sikhs Indian

Christians Anglo-Indians Europeans

distinct regional groups Gandhi

undertook a ldquofast unto deathrdquo against

that offer which he viewed as a

nefarious British plot to divide the

Indian society

Q45) Ans (b)

In British India apart from existing

imports and exports there was also a

particular amount of money which

colonial India contributed towards

administration maintenance of the

army war expenses pensions to retired

officers and other expenses accrued by

Britain towards maintenance of her

colony These were known as Home

charges and were paid for almost

entirely by India

The Home charges was made of

following components-

- Interest payable on Indian debt

- Dividend to shareholders of East

India Company

- Funds used to support the India

Office in London

- Funds used to pay salaries and

pensions of British personnel

engaged in India

- Interest on the railways

- Civil and military charges

- Store purchases in England

Q46) Ans (b)

The Lahore session of the Indian

National Congress was held in 1929

under the Presidentship of Jawaharlal

Nehru

The Lahore session of the Indian

National Congress witnessed significant

RAUSIAS-FC19E1003 46

developments in the Indian national

movement

- First the election of Jawaharlal

Nehru to the post of Presidentship of

the Congress was a clear indication

of the growing strength of the

Leftists in the Congress

- Secondly it was in this session that

the Congress for the first time raised

the demand for complete

independence Such demand was

not raised from the Congress

platform earlier

Q47) Ans (b)

It did not provide for separate

electorates for any community or

weightage for minorities However it did

allow for the reservation of minority

seats in provinces having minorities of

at least ten per cent but this was to be

in strict proportion to the size of the

community

There was no provision for complete

Independence for India

Q48) Ans (c)

The religion of early Vedic Aryans was

primarily of worship of nature and

Yajnas

The early Aryan religion was kind of

nature worship Actually the forces

around them which they could not

control or understand were invested

with divinity and were personified as

male or female gods And they

performed some Yajnas also

Q49) Ans (b)

The roads and river-routes were not

immune from robbery It is notable that

Yuan Chwang (Hiuen Tsang) was

robbed of his belongings during

Harshvardanarsquos period

Q50) Ans (c)

Q51) Ans (b)

Purandara Dasa was a saint and great

devotee of Lord Krishna

There is much speculation about where

Purandara Dasa regarded as the

Pitamaha of Carnatic music was born

Recently an expert committee

constituted by the Kannada University

Hampi has come to the conclusion that

Kshemapura Shivamogga district

Karnataka is the birth place of

Purandara Dasa

Q52) Ans (c)

Sri Tyagaraja Sri Shyama Shastry and Sri Muthuswami Dikshitar are considered the trinity of Carnatic music and with them came the golden age in Carnatic music in the 18th-19th

century

Q53) Ans d)

Recently a rare sarcophagus (stone

coffin) which is 2000 years old from the

Iron AgendashMegalithic era was discovered

from a rock-cut cave at Viyur village of

Kollam near Koyilandy in Kozhikode

district Kerala

The coffin containing bone fragments

was found during an excavation ldquoSo

far such a rare finding has been

discovered only from two sites

in Kerala Both these sarcophagi were

recovered from Megalithic sites at

Chevayur and Atholi also in Kozhikode

district

Q54) Ans a)

The megalithic culture in South India was a full-fledged Iron Age culture

Q55) Ans d)

The Cholas Pandyas and Keralaputras

(Cheras) mentioned in Ashokan

inscriptions were probably in the late

megalithic phase of material culture

Q56) Ans d)

Q57) Ans (b)

Raj Kumar Shukla followed Gandhiji all

over the country to persuade him to

come to Champaran to investigate the

problem associated with tinkathia

system

RAUSIAS-FC19E1003 47

Brij Kishore Rajendra Prasad Mahadev

Desai and Narhari Parikh accompanied

Gandhi ji during the Champaran

Satyagraha

Q58) Ans (b)

The Satvahanas started the practice of granting tax-free villages to brahmanas and Buddhist monks

Q59) Ans c)

The objectives of the Programme are

listed as under

- Developing basic tourism

infrastructure

- Promoting cultural and heritage

value of the country to generate

livelihoods in the identified regions

- Enhancing the tourist attractiveness

in a sustainable manner by

developing world-class

infrastructure at the heritage

monument sites

- Creating employment through active

involvement of local communities

- Harnessing tourism potential for its

effects on employment generation

and economic development

- Developing sustainable tourism

infrastructure and ensuring proper

Operations and maintenance

therein

Q60) Ans (b)

The Tribal Cooperative Marketing

Development Federation of India

(TRIFED) came into existence in 1987

It is a national-level apex organization

functioning under the administrative

control of Ministry of Tribal Affairs

Govt of India

TRIFED has its registered and Head

Office located in New Delhi

Q61) Ans (c)

Premchandrsquos novels include

Premashram Rangabhumi Ghaban

Karmabhumi and Godan

Gora is a novel written by Rabindranath

Tagore

138th birth anniversary of Munshi

Premchand was celebrated across the

country

Q62) Ans (b)

Giddha is a traditional pastoral dance

performed by the women of the Punjab

India and Pakistan at festival times

and at the sowing and reaping of the

harvest

By this dance the Punjabi women

reveal their joy expel their suppressed

feelings in a male dominated society

through the performance of Giddha

Since this dance has nothing to do with

men only women can participate in it

During the Teej celebrations Giddha

dance is celebrated in Punjab every

year Teej is a generic name for a

number of festivals that are celebrated

by women in some parts of India

Q63) Ans (a)

Dara Shukoh wrote the remarkable

work called ldquoMajma-ul-Bahrainrdquo or the

ldquoThe confluence of two seasrdquo

The Vice President of India Shri M

Venkaiah Naidu has said that Prince

Dara Shukohrsquos writings can come as a

refreshing source for infusing peace and

harmony He was addressing the

gathering after visiting the exhibition

that showcases the forgotten Prince of

yesteryears Dara Shukoh organized by

Mr Francois Gautier at Indira Gandhi

National Centre for the Arts in New

Delhi

Q64) Ans (c)

The statue Gommateshwara is

dedicated to the Jain God Bahubali

It is a monolithic statue

President Ram Nath Kovind

inaugurated the grand anointing

ceremony mdash Mahamastakabhisheka mdash

held once in 12 years at

Shravanabelagola (Karnataka)

Q65) Ans (c)

Prachi Valley had come up around the

Prachi river Prachi Valley gradually

disappeared

RAUSIAS-FC19E1003 48

The Prachi river originates from

Bhubaneswar

It is a tributary of the Mahanadi and

flows through the districts of Puri

Khurda Cuttack and Jagatsinghpur

and the entire region of the river is

termed as the Prachi Valley

It falls into the Bay of Bengal

Archaeological evidence shows that the

Prachi Valley Civilisation predates both

Harappa and Mohenjo-Daro

The Prachi river originates from

Bhubaneswar

Q66) Ans (d)

These monuments are located in

Chhatarpur district Madhya Pradesh

within Vindhya mountain range

Q67) Ans (a)

The book lsquoThoughts on Pakistanrsquo was

written by Dr BR Ambedkar

On the occasion of the birth anniversary

of Dr BR Ambedkar the president of

India pays homage to this icon of India

In 1924 he founded the Depressed

Classes Institute (Bahishkrit Hitkarini

Sabha) and in 1927 the Samaj Samata

Sangh

Another area of attention for Ambedkar

was education For its spread among

the low classes he set up a network of

colleges by the name of Peoples

Education Society and founded hostels

Q68) Ans(b)

Mehrgarh is a famous Neolithic

settlement in the Indian subcontinent

which is situated in Baluchistan

province Pakistan

A pre-historic rock art site is discovered

in the vast expanse of limestone blocks

on the eastern banks of Naguleru river

near Dachepalli (Andhra Pradesh) It

has thrown light on the Neolithic

civilisation that flourished in Guntur

(Andhra Pradesh) during 1500-2000

BC

Q69) Ans (c)

The 12th and the 13th centuries saw

the emergence of the Kakatiyas They

were at first the feudatories of the

Western Chalukyas of Kalyana Initially

they ruled over a small territory near

Warangal (Telangana)

They introduced Nayakships which was

later adopted and developed by the

Rayas of Vijayanagara

Q70) Ans (a)

The fast had effect of putting pressure

on mill owners who finally agreed to

give the workers a 35 per cent increase

in wages

Google celebrated with a doodle the

132nd birth anniversary of Anasuya

Sarabhai who played a pioneering role

in Indiarsquos labour movement

Q71) Ans (d)

The UNESCOrsquos list of the representative

list of the intangible cultural heritage of

humanity from India are

- Koodiyattam Sanskrit Theatre of

Kerala

- Mudiyettu ritual theatre and dance

drama of Kerala

- Tradition of Vedic Chanting

- Kalbelia folk songs and dances of

Rajasthan

- Ramlila Traditional Performance of

the Ramayana

- Sankirtana ritual singing

drumming and dancing of Manipur

- Ramman religious festival and

ritual theatre of the Garhwal

Himalayas India

- Traditional brass and copper craft of

utensil making among the Thatheras

of Jandiala Guru Punjab India

- Chhau dance classical Indian dance

originated in the eastern Indian

states

- Buddhist chanting of Ladakh

recitation of sacred Buddhist texts

in the trans-Himalayan Ladakh

region Jammu and Kashmir India

- Yoga

- Nouroz

- Kumbh Mela

RAUSIAS-FC19E1003 49

Q72) Ans(b)

The President of India Shri Ram Nath Kovind inaugurated the Hornbill Festival and State Formation Day celebrations of Nagaland in Kisama

The festival is named after the Indian hornbill the large and colourful forest bird which is displayed in the folklore of most of the states tribes

The major recognized tribes of Nagaland are Angami Ao Chakhesang Chang

Kuki Rengma and Zeling etc

Onge Jarawa and Sentinelese are the

tribes of Andman amp Nicobar Islands

Q73) Ans (c)

The Rashtrakutas rule in the Deccan lasted for almost two hundred years till the end of the tenth century The Rashtrakutas rulers were tolerant in their religious views and patronized not only Shaivism and Vaishnavism but

Jainism as well

The famous rock-cut temple of Shiva at Ellora was built by one of the Rashtrakutas kings Krishna I in the ninth century His successor Amoghavarsha was a Jain but he also

patronized other faiths

The Rashtrakutas allowed Muslims traders to settle and permitted Islam to

be preached in their dominions

Recently increasing defacement at the prehistoric rock paintings of Pandavulagutta Telangana has created a cause for grave concern It can spoil

the prehistoric rock

Pandavulagutta is home to

- Painted rock shelters dating to

10000 BC-8000 BC

- An 8th century inscription of the

Rashtrakuta period and

- Painted frescoes from the 12th century Kakatiya empire

Q74) Ans (b)

In 1828 Raja Ram Mohan Roy founded a new religious society the Brahma Sabha later known as the Brahmo

Samaj

Debendranath Tagore headed the Tattvabodhini Sabha which was

engaged in search of spiritual truth

Its purpose was to purify Hinduism and to preach monotheism or belief in one God

The new society was to be based on the twin pillars of reason and the Vedas and

Upanishads

Recently Sadharan Brahmo Samaj (SBS) has entered into a legal battle with the West Bengal government due

to some legal issue

Q75) Ans (c)

The Chishti order was established in India by Khwaja Moinuddin Chishti who came to India around 1192 The Chishtirsquos are considered to be the most influential of the groups of Sufis who migrated to India in the late twelfth century They adapted successfully to the local environment and adopted several features of Indian devotional

traditions

The historical dargah of Sufi mystic Khwaja Moinuddin Chishti in Ajmer is all set to get a facelift This 13 th century dargah has been included among the Swachh Iconic Places a clean-up initiative focused on iconic

heritage spiritual and cultural places

Page 45: GENERAL STUDIES (PAPER I) · Test is part of Rau’s IAS Test series for Preliminary Exam 2019 FOUNDATION + CURRENT AFFAIRS GENERAL STUDIES (PAPER –I) FOUNDATION TEST –III TOPIC:

RAUSIAS-FC19E1003 45

Q40) Answer (c)

Explanation

Under the Mughals mansabdar was

referred to an individual who held a

mansab ie rank and he received his

salary as revenue assignments called

jagirs

Q41) Ans (b)

The Quit India Movement was a

spontaneous revolt of people against

British rule

The All India Congress Committee met

at Bombay on 8 August 1942 It passed

the famous resolution Quit India and

proposed the starting of a non-violent

mass struggle under Gandhis

leadership to achieve this aim But on

the very next day Gandhi and other

eminent leaders of the Congress were

arrested The Congress was once again

declared illegal

Q42) Ans (c)

The Simon Commission refers to a

group of seven MPs from the United

Kingdom constituted to suggest

constitutional reforms for British India

The Commission consisted of only

British members headed by one of the

senior British politicians Sir John

Simon

So the people of India agitated against

the arrival of Simon Commission

Q43) Ans (a)

He was widely known for his

unfavourable opinion of the economic

consequences of the British rule in

India

In his many writings and speeches and

especially in Poverty and Un-British

Rule in India Naoroji argued that India

was too highly taxed and that its wealth

was being drained away to England

He did not interpret the ancient Indian

texts and restored the self-confidence of

Indians And also he did not stress the

need for eradication of all the social

evils before anything else

Q44) Ans (c)

In August 1932 Prime Minister

MacDonald announced his Communal

Award Great Britainrsquos unilateral

attempt to resolve the various conflicts

among Indiarsquos many communal

interests

The award which was later

incorporated into the act of 1935

expanded the separate-electorate

formula reserved for Muslims to other

minorities including Sikhs Indian

Christians Anglo-Indians Europeans

distinct regional groups Gandhi

undertook a ldquofast unto deathrdquo against

that offer which he viewed as a

nefarious British plot to divide the

Indian society

Q45) Ans (b)

In British India apart from existing

imports and exports there was also a

particular amount of money which

colonial India contributed towards

administration maintenance of the

army war expenses pensions to retired

officers and other expenses accrued by

Britain towards maintenance of her

colony These were known as Home

charges and were paid for almost

entirely by India

The Home charges was made of

following components-

- Interest payable on Indian debt

- Dividend to shareholders of East

India Company

- Funds used to support the India

Office in London

- Funds used to pay salaries and

pensions of British personnel

engaged in India

- Interest on the railways

- Civil and military charges

- Store purchases in England

Q46) Ans (b)

The Lahore session of the Indian

National Congress was held in 1929

under the Presidentship of Jawaharlal

Nehru

The Lahore session of the Indian

National Congress witnessed significant

RAUSIAS-FC19E1003 46

developments in the Indian national

movement

- First the election of Jawaharlal

Nehru to the post of Presidentship of

the Congress was a clear indication

of the growing strength of the

Leftists in the Congress

- Secondly it was in this session that

the Congress for the first time raised

the demand for complete

independence Such demand was

not raised from the Congress

platform earlier

Q47) Ans (b)

It did not provide for separate

electorates for any community or

weightage for minorities However it did

allow for the reservation of minority

seats in provinces having minorities of

at least ten per cent but this was to be

in strict proportion to the size of the

community

There was no provision for complete

Independence for India

Q48) Ans (c)

The religion of early Vedic Aryans was

primarily of worship of nature and

Yajnas

The early Aryan religion was kind of

nature worship Actually the forces

around them which they could not

control or understand were invested

with divinity and were personified as

male or female gods And they

performed some Yajnas also

Q49) Ans (b)

The roads and river-routes were not

immune from robbery It is notable that

Yuan Chwang (Hiuen Tsang) was

robbed of his belongings during

Harshvardanarsquos period

Q50) Ans (c)

Q51) Ans (b)

Purandara Dasa was a saint and great

devotee of Lord Krishna

There is much speculation about where

Purandara Dasa regarded as the

Pitamaha of Carnatic music was born

Recently an expert committee

constituted by the Kannada University

Hampi has come to the conclusion that

Kshemapura Shivamogga district

Karnataka is the birth place of

Purandara Dasa

Q52) Ans (c)

Sri Tyagaraja Sri Shyama Shastry and Sri Muthuswami Dikshitar are considered the trinity of Carnatic music and with them came the golden age in Carnatic music in the 18th-19th

century

Q53) Ans d)

Recently a rare sarcophagus (stone

coffin) which is 2000 years old from the

Iron AgendashMegalithic era was discovered

from a rock-cut cave at Viyur village of

Kollam near Koyilandy in Kozhikode

district Kerala

The coffin containing bone fragments

was found during an excavation ldquoSo

far such a rare finding has been

discovered only from two sites

in Kerala Both these sarcophagi were

recovered from Megalithic sites at

Chevayur and Atholi also in Kozhikode

district

Q54) Ans a)

The megalithic culture in South India was a full-fledged Iron Age culture

Q55) Ans d)

The Cholas Pandyas and Keralaputras

(Cheras) mentioned in Ashokan

inscriptions were probably in the late

megalithic phase of material culture

Q56) Ans d)

Q57) Ans (b)

Raj Kumar Shukla followed Gandhiji all

over the country to persuade him to

come to Champaran to investigate the

problem associated with tinkathia

system

RAUSIAS-FC19E1003 47

Brij Kishore Rajendra Prasad Mahadev

Desai and Narhari Parikh accompanied

Gandhi ji during the Champaran

Satyagraha

Q58) Ans (b)

The Satvahanas started the practice of granting tax-free villages to brahmanas and Buddhist monks

Q59) Ans c)

The objectives of the Programme are

listed as under

- Developing basic tourism

infrastructure

- Promoting cultural and heritage

value of the country to generate

livelihoods in the identified regions

- Enhancing the tourist attractiveness

in a sustainable manner by

developing world-class

infrastructure at the heritage

monument sites

- Creating employment through active

involvement of local communities

- Harnessing tourism potential for its

effects on employment generation

and economic development

- Developing sustainable tourism

infrastructure and ensuring proper

Operations and maintenance

therein

Q60) Ans (b)

The Tribal Cooperative Marketing

Development Federation of India

(TRIFED) came into existence in 1987

It is a national-level apex organization

functioning under the administrative

control of Ministry of Tribal Affairs

Govt of India

TRIFED has its registered and Head

Office located in New Delhi

Q61) Ans (c)

Premchandrsquos novels include

Premashram Rangabhumi Ghaban

Karmabhumi and Godan

Gora is a novel written by Rabindranath

Tagore

138th birth anniversary of Munshi

Premchand was celebrated across the

country

Q62) Ans (b)

Giddha is a traditional pastoral dance

performed by the women of the Punjab

India and Pakistan at festival times

and at the sowing and reaping of the

harvest

By this dance the Punjabi women

reveal their joy expel their suppressed

feelings in a male dominated society

through the performance of Giddha

Since this dance has nothing to do with

men only women can participate in it

During the Teej celebrations Giddha

dance is celebrated in Punjab every

year Teej is a generic name for a

number of festivals that are celebrated

by women in some parts of India

Q63) Ans (a)

Dara Shukoh wrote the remarkable

work called ldquoMajma-ul-Bahrainrdquo or the

ldquoThe confluence of two seasrdquo

The Vice President of India Shri M

Venkaiah Naidu has said that Prince

Dara Shukohrsquos writings can come as a

refreshing source for infusing peace and

harmony He was addressing the

gathering after visiting the exhibition

that showcases the forgotten Prince of

yesteryears Dara Shukoh organized by

Mr Francois Gautier at Indira Gandhi

National Centre for the Arts in New

Delhi

Q64) Ans (c)

The statue Gommateshwara is

dedicated to the Jain God Bahubali

It is a monolithic statue

President Ram Nath Kovind

inaugurated the grand anointing

ceremony mdash Mahamastakabhisheka mdash

held once in 12 years at

Shravanabelagola (Karnataka)

Q65) Ans (c)

Prachi Valley had come up around the

Prachi river Prachi Valley gradually

disappeared

RAUSIAS-FC19E1003 48

The Prachi river originates from

Bhubaneswar

It is a tributary of the Mahanadi and

flows through the districts of Puri

Khurda Cuttack and Jagatsinghpur

and the entire region of the river is

termed as the Prachi Valley

It falls into the Bay of Bengal

Archaeological evidence shows that the

Prachi Valley Civilisation predates both

Harappa and Mohenjo-Daro

The Prachi river originates from

Bhubaneswar

Q66) Ans (d)

These monuments are located in

Chhatarpur district Madhya Pradesh

within Vindhya mountain range

Q67) Ans (a)

The book lsquoThoughts on Pakistanrsquo was

written by Dr BR Ambedkar

On the occasion of the birth anniversary

of Dr BR Ambedkar the president of

India pays homage to this icon of India

In 1924 he founded the Depressed

Classes Institute (Bahishkrit Hitkarini

Sabha) and in 1927 the Samaj Samata

Sangh

Another area of attention for Ambedkar

was education For its spread among

the low classes he set up a network of

colleges by the name of Peoples

Education Society and founded hostels

Q68) Ans(b)

Mehrgarh is a famous Neolithic

settlement in the Indian subcontinent

which is situated in Baluchistan

province Pakistan

A pre-historic rock art site is discovered

in the vast expanse of limestone blocks

on the eastern banks of Naguleru river

near Dachepalli (Andhra Pradesh) It

has thrown light on the Neolithic

civilisation that flourished in Guntur

(Andhra Pradesh) during 1500-2000

BC

Q69) Ans (c)

The 12th and the 13th centuries saw

the emergence of the Kakatiyas They

were at first the feudatories of the

Western Chalukyas of Kalyana Initially

they ruled over a small territory near

Warangal (Telangana)

They introduced Nayakships which was

later adopted and developed by the

Rayas of Vijayanagara

Q70) Ans (a)

The fast had effect of putting pressure

on mill owners who finally agreed to

give the workers a 35 per cent increase

in wages

Google celebrated with a doodle the

132nd birth anniversary of Anasuya

Sarabhai who played a pioneering role

in Indiarsquos labour movement

Q71) Ans (d)

The UNESCOrsquos list of the representative

list of the intangible cultural heritage of

humanity from India are

- Koodiyattam Sanskrit Theatre of

Kerala

- Mudiyettu ritual theatre and dance

drama of Kerala

- Tradition of Vedic Chanting

- Kalbelia folk songs and dances of

Rajasthan

- Ramlila Traditional Performance of

the Ramayana

- Sankirtana ritual singing

drumming and dancing of Manipur

- Ramman religious festival and

ritual theatre of the Garhwal

Himalayas India

- Traditional brass and copper craft of

utensil making among the Thatheras

of Jandiala Guru Punjab India

- Chhau dance classical Indian dance

originated in the eastern Indian

states

- Buddhist chanting of Ladakh

recitation of sacred Buddhist texts

in the trans-Himalayan Ladakh

region Jammu and Kashmir India

- Yoga

- Nouroz

- Kumbh Mela

RAUSIAS-FC19E1003 49

Q72) Ans(b)

The President of India Shri Ram Nath Kovind inaugurated the Hornbill Festival and State Formation Day celebrations of Nagaland in Kisama

The festival is named after the Indian hornbill the large and colourful forest bird which is displayed in the folklore of most of the states tribes

The major recognized tribes of Nagaland are Angami Ao Chakhesang Chang

Kuki Rengma and Zeling etc

Onge Jarawa and Sentinelese are the

tribes of Andman amp Nicobar Islands

Q73) Ans (c)

The Rashtrakutas rule in the Deccan lasted for almost two hundred years till the end of the tenth century The Rashtrakutas rulers were tolerant in their religious views and patronized not only Shaivism and Vaishnavism but

Jainism as well

The famous rock-cut temple of Shiva at Ellora was built by one of the Rashtrakutas kings Krishna I in the ninth century His successor Amoghavarsha was a Jain but he also

patronized other faiths

The Rashtrakutas allowed Muslims traders to settle and permitted Islam to

be preached in their dominions

Recently increasing defacement at the prehistoric rock paintings of Pandavulagutta Telangana has created a cause for grave concern It can spoil

the prehistoric rock

Pandavulagutta is home to

- Painted rock shelters dating to

10000 BC-8000 BC

- An 8th century inscription of the

Rashtrakuta period and

- Painted frescoes from the 12th century Kakatiya empire

Q74) Ans (b)

In 1828 Raja Ram Mohan Roy founded a new religious society the Brahma Sabha later known as the Brahmo

Samaj

Debendranath Tagore headed the Tattvabodhini Sabha which was

engaged in search of spiritual truth

Its purpose was to purify Hinduism and to preach monotheism or belief in one God

The new society was to be based on the twin pillars of reason and the Vedas and

Upanishads

Recently Sadharan Brahmo Samaj (SBS) has entered into a legal battle with the West Bengal government due

to some legal issue

Q75) Ans (c)

The Chishti order was established in India by Khwaja Moinuddin Chishti who came to India around 1192 The Chishtirsquos are considered to be the most influential of the groups of Sufis who migrated to India in the late twelfth century They adapted successfully to the local environment and adopted several features of Indian devotional

traditions

The historical dargah of Sufi mystic Khwaja Moinuddin Chishti in Ajmer is all set to get a facelift This 13 th century dargah has been included among the Swachh Iconic Places a clean-up initiative focused on iconic

heritage spiritual and cultural places

Page 46: GENERAL STUDIES (PAPER I) · Test is part of Rau’s IAS Test series for Preliminary Exam 2019 FOUNDATION + CURRENT AFFAIRS GENERAL STUDIES (PAPER –I) FOUNDATION TEST –III TOPIC:

RAUSIAS-FC19E1003 46

developments in the Indian national

movement

- First the election of Jawaharlal

Nehru to the post of Presidentship of

the Congress was a clear indication

of the growing strength of the

Leftists in the Congress

- Secondly it was in this session that

the Congress for the first time raised

the demand for complete

independence Such demand was

not raised from the Congress

platform earlier

Q47) Ans (b)

It did not provide for separate

electorates for any community or

weightage for minorities However it did

allow for the reservation of minority

seats in provinces having minorities of

at least ten per cent but this was to be

in strict proportion to the size of the

community

There was no provision for complete

Independence for India

Q48) Ans (c)

The religion of early Vedic Aryans was

primarily of worship of nature and

Yajnas

The early Aryan religion was kind of

nature worship Actually the forces

around them which they could not

control or understand were invested

with divinity and were personified as

male or female gods And they

performed some Yajnas also

Q49) Ans (b)

The roads and river-routes were not

immune from robbery It is notable that

Yuan Chwang (Hiuen Tsang) was

robbed of his belongings during

Harshvardanarsquos period

Q50) Ans (c)

Q51) Ans (b)

Purandara Dasa was a saint and great

devotee of Lord Krishna

There is much speculation about where

Purandara Dasa regarded as the

Pitamaha of Carnatic music was born

Recently an expert committee

constituted by the Kannada University

Hampi has come to the conclusion that

Kshemapura Shivamogga district

Karnataka is the birth place of

Purandara Dasa

Q52) Ans (c)

Sri Tyagaraja Sri Shyama Shastry and Sri Muthuswami Dikshitar are considered the trinity of Carnatic music and with them came the golden age in Carnatic music in the 18th-19th

century

Q53) Ans d)

Recently a rare sarcophagus (stone

coffin) which is 2000 years old from the

Iron AgendashMegalithic era was discovered

from a rock-cut cave at Viyur village of

Kollam near Koyilandy in Kozhikode

district Kerala

The coffin containing bone fragments

was found during an excavation ldquoSo

far such a rare finding has been

discovered only from two sites

in Kerala Both these sarcophagi were

recovered from Megalithic sites at

Chevayur and Atholi also in Kozhikode

district

Q54) Ans a)

The megalithic culture in South India was a full-fledged Iron Age culture

Q55) Ans d)

The Cholas Pandyas and Keralaputras

(Cheras) mentioned in Ashokan

inscriptions were probably in the late

megalithic phase of material culture

Q56) Ans d)

Q57) Ans (b)

Raj Kumar Shukla followed Gandhiji all

over the country to persuade him to

come to Champaran to investigate the

problem associated with tinkathia

system

RAUSIAS-FC19E1003 47

Brij Kishore Rajendra Prasad Mahadev

Desai and Narhari Parikh accompanied

Gandhi ji during the Champaran

Satyagraha

Q58) Ans (b)

The Satvahanas started the practice of granting tax-free villages to brahmanas and Buddhist monks

Q59) Ans c)

The objectives of the Programme are

listed as under

- Developing basic tourism

infrastructure

- Promoting cultural and heritage

value of the country to generate

livelihoods in the identified regions

- Enhancing the tourist attractiveness

in a sustainable manner by

developing world-class

infrastructure at the heritage

monument sites

- Creating employment through active

involvement of local communities

- Harnessing tourism potential for its

effects on employment generation

and economic development

- Developing sustainable tourism

infrastructure and ensuring proper

Operations and maintenance

therein

Q60) Ans (b)

The Tribal Cooperative Marketing

Development Federation of India

(TRIFED) came into existence in 1987

It is a national-level apex organization

functioning under the administrative

control of Ministry of Tribal Affairs

Govt of India

TRIFED has its registered and Head

Office located in New Delhi

Q61) Ans (c)

Premchandrsquos novels include

Premashram Rangabhumi Ghaban

Karmabhumi and Godan

Gora is a novel written by Rabindranath

Tagore

138th birth anniversary of Munshi

Premchand was celebrated across the

country

Q62) Ans (b)

Giddha is a traditional pastoral dance

performed by the women of the Punjab

India and Pakistan at festival times

and at the sowing and reaping of the

harvest

By this dance the Punjabi women

reveal their joy expel their suppressed

feelings in a male dominated society

through the performance of Giddha

Since this dance has nothing to do with

men only women can participate in it

During the Teej celebrations Giddha

dance is celebrated in Punjab every

year Teej is a generic name for a

number of festivals that are celebrated

by women in some parts of India

Q63) Ans (a)

Dara Shukoh wrote the remarkable

work called ldquoMajma-ul-Bahrainrdquo or the

ldquoThe confluence of two seasrdquo

The Vice President of India Shri M

Venkaiah Naidu has said that Prince

Dara Shukohrsquos writings can come as a

refreshing source for infusing peace and

harmony He was addressing the

gathering after visiting the exhibition

that showcases the forgotten Prince of

yesteryears Dara Shukoh organized by

Mr Francois Gautier at Indira Gandhi

National Centre for the Arts in New

Delhi

Q64) Ans (c)

The statue Gommateshwara is

dedicated to the Jain God Bahubali

It is a monolithic statue

President Ram Nath Kovind

inaugurated the grand anointing

ceremony mdash Mahamastakabhisheka mdash

held once in 12 years at

Shravanabelagola (Karnataka)

Q65) Ans (c)

Prachi Valley had come up around the

Prachi river Prachi Valley gradually

disappeared

RAUSIAS-FC19E1003 48

The Prachi river originates from

Bhubaneswar

It is a tributary of the Mahanadi and

flows through the districts of Puri

Khurda Cuttack and Jagatsinghpur

and the entire region of the river is

termed as the Prachi Valley

It falls into the Bay of Bengal

Archaeological evidence shows that the

Prachi Valley Civilisation predates both

Harappa and Mohenjo-Daro

The Prachi river originates from

Bhubaneswar

Q66) Ans (d)

These monuments are located in

Chhatarpur district Madhya Pradesh

within Vindhya mountain range

Q67) Ans (a)

The book lsquoThoughts on Pakistanrsquo was

written by Dr BR Ambedkar

On the occasion of the birth anniversary

of Dr BR Ambedkar the president of

India pays homage to this icon of India

In 1924 he founded the Depressed

Classes Institute (Bahishkrit Hitkarini

Sabha) and in 1927 the Samaj Samata

Sangh

Another area of attention for Ambedkar

was education For its spread among

the low classes he set up a network of

colleges by the name of Peoples

Education Society and founded hostels

Q68) Ans(b)

Mehrgarh is a famous Neolithic

settlement in the Indian subcontinent

which is situated in Baluchistan

province Pakistan

A pre-historic rock art site is discovered

in the vast expanse of limestone blocks

on the eastern banks of Naguleru river

near Dachepalli (Andhra Pradesh) It

has thrown light on the Neolithic

civilisation that flourished in Guntur

(Andhra Pradesh) during 1500-2000

BC

Q69) Ans (c)

The 12th and the 13th centuries saw

the emergence of the Kakatiyas They

were at first the feudatories of the

Western Chalukyas of Kalyana Initially

they ruled over a small territory near

Warangal (Telangana)

They introduced Nayakships which was

later adopted and developed by the

Rayas of Vijayanagara

Q70) Ans (a)

The fast had effect of putting pressure

on mill owners who finally agreed to

give the workers a 35 per cent increase

in wages

Google celebrated with a doodle the

132nd birth anniversary of Anasuya

Sarabhai who played a pioneering role

in Indiarsquos labour movement

Q71) Ans (d)

The UNESCOrsquos list of the representative

list of the intangible cultural heritage of

humanity from India are

- Koodiyattam Sanskrit Theatre of

Kerala

- Mudiyettu ritual theatre and dance

drama of Kerala

- Tradition of Vedic Chanting

- Kalbelia folk songs and dances of

Rajasthan

- Ramlila Traditional Performance of

the Ramayana

- Sankirtana ritual singing

drumming and dancing of Manipur

- Ramman religious festival and

ritual theatre of the Garhwal

Himalayas India

- Traditional brass and copper craft of

utensil making among the Thatheras

of Jandiala Guru Punjab India

- Chhau dance classical Indian dance

originated in the eastern Indian

states

- Buddhist chanting of Ladakh

recitation of sacred Buddhist texts

in the trans-Himalayan Ladakh

region Jammu and Kashmir India

- Yoga

- Nouroz

- Kumbh Mela

RAUSIAS-FC19E1003 49

Q72) Ans(b)

The President of India Shri Ram Nath Kovind inaugurated the Hornbill Festival and State Formation Day celebrations of Nagaland in Kisama

The festival is named after the Indian hornbill the large and colourful forest bird which is displayed in the folklore of most of the states tribes

The major recognized tribes of Nagaland are Angami Ao Chakhesang Chang

Kuki Rengma and Zeling etc

Onge Jarawa and Sentinelese are the

tribes of Andman amp Nicobar Islands

Q73) Ans (c)

The Rashtrakutas rule in the Deccan lasted for almost two hundred years till the end of the tenth century The Rashtrakutas rulers were tolerant in their religious views and patronized not only Shaivism and Vaishnavism but

Jainism as well

The famous rock-cut temple of Shiva at Ellora was built by one of the Rashtrakutas kings Krishna I in the ninth century His successor Amoghavarsha was a Jain but he also

patronized other faiths

The Rashtrakutas allowed Muslims traders to settle and permitted Islam to

be preached in their dominions

Recently increasing defacement at the prehistoric rock paintings of Pandavulagutta Telangana has created a cause for grave concern It can spoil

the prehistoric rock

Pandavulagutta is home to

- Painted rock shelters dating to

10000 BC-8000 BC

- An 8th century inscription of the

Rashtrakuta period and

- Painted frescoes from the 12th century Kakatiya empire

Q74) Ans (b)

In 1828 Raja Ram Mohan Roy founded a new religious society the Brahma Sabha later known as the Brahmo

Samaj

Debendranath Tagore headed the Tattvabodhini Sabha which was

engaged in search of spiritual truth

Its purpose was to purify Hinduism and to preach monotheism or belief in one God

The new society was to be based on the twin pillars of reason and the Vedas and

Upanishads

Recently Sadharan Brahmo Samaj (SBS) has entered into a legal battle with the West Bengal government due

to some legal issue

Q75) Ans (c)

The Chishti order was established in India by Khwaja Moinuddin Chishti who came to India around 1192 The Chishtirsquos are considered to be the most influential of the groups of Sufis who migrated to India in the late twelfth century They adapted successfully to the local environment and adopted several features of Indian devotional

traditions

The historical dargah of Sufi mystic Khwaja Moinuddin Chishti in Ajmer is all set to get a facelift This 13 th century dargah has been included among the Swachh Iconic Places a clean-up initiative focused on iconic

heritage spiritual and cultural places

Page 47: GENERAL STUDIES (PAPER I) · Test is part of Rau’s IAS Test series for Preliminary Exam 2019 FOUNDATION + CURRENT AFFAIRS GENERAL STUDIES (PAPER –I) FOUNDATION TEST –III TOPIC:

RAUSIAS-FC19E1003 47

Brij Kishore Rajendra Prasad Mahadev

Desai and Narhari Parikh accompanied

Gandhi ji during the Champaran

Satyagraha

Q58) Ans (b)

The Satvahanas started the practice of granting tax-free villages to brahmanas and Buddhist monks

Q59) Ans c)

The objectives of the Programme are

listed as under

- Developing basic tourism

infrastructure

- Promoting cultural and heritage

value of the country to generate

livelihoods in the identified regions

- Enhancing the tourist attractiveness

in a sustainable manner by

developing world-class

infrastructure at the heritage

monument sites

- Creating employment through active

involvement of local communities

- Harnessing tourism potential for its

effects on employment generation

and economic development

- Developing sustainable tourism

infrastructure and ensuring proper

Operations and maintenance

therein

Q60) Ans (b)

The Tribal Cooperative Marketing

Development Federation of India

(TRIFED) came into existence in 1987

It is a national-level apex organization

functioning under the administrative

control of Ministry of Tribal Affairs

Govt of India

TRIFED has its registered and Head

Office located in New Delhi

Q61) Ans (c)

Premchandrsquos novels include

Premashram Rangabhumi Ghaban

Karmabhumi and Godan

Gora is a novel written by Rabindranath

Tagore

138th birth anniversary of Munshi

Premchand was celebrated across the

country

Q62) Ans (b)

Giddha is a traditional pastoral dance

performed by the women of the Punjab

India and Pakistan at festival times

and at the sowing and reaping of the

harvest

By this dance the Punjabi women

reveal their joy expel their suppressed

feelings in a male dominated society

through the performance of Giddha

Since this dance has nothing to do with

men only women can participate in it

During the Teej celebrations Giddha

dance is celebrated in Punjab every

year Teej is a generic name for a

number of festivals that are celebrated

by women in some parts of India

Q63) Ans (a)

Dara Shukoh wrote the remarkable

work called ldquoMajma-ul-Bahrainrdquo or the

ldquoThe confluence of two seasrdquo

The Vice President of India Shri M

Venkaiah Naidu has said that Prince

Dara Shukohrsquos writings can come as a

refreshing source for infusing peace and

harmony He was addressing the

gathering after visiting the exhibition

that showcases the forgotten Prince of

yesteryears Dara Shukoh organized by

Mr Francois Gautier at Indira Gandhi

National Centre for the Arts in New

Delhi

Q64) Ans (c)

The statue Gommateshwara is

dedicated to the Jain God Bahubali

It is a monolithic statue

President Ram Nath Kovind

inaugurated the grand anointing

ceremony mdash Mahamastakabhisheka mdash

held once in 12 years at

Shravanabelagola (Karnataka)

Q65) Ans (c)

Prachi Valley had come up around the

Prachi river Prachi Valley gradually

disappeared

RAUSIAS-FC19E1003 48

The Prachi river originates from

Bhubaneswar

It is a tributary of the Mahanadi and

flows through the districts of Puri

Khurda Cuttack and Jagatsinghpur

and the entire region of the river is

termed as the Prachi Valley

It falls into the Bay of Bengal

Archaeological evidence shows that the

Prachi Valley Civilisation predates both

Harappa and Mohenjo-Daro

The Prachi river originates from

Bhubaneswar

Q66) Ans (d)

These monuments are located in

Chhatarpur district Madhya Pradesh

within Vindhya mountain range

Q67) Ans (a)

The book lsquoThoughts on Pakistanrsquo was

written by Dr BR Ambedkar

On the occasion of the birth anniversary

of Dr BR Ambedkar the president of

India pays homage to this icon of India

In 1924 he founded the Depressed

Classes Institute (Bahishkrit Hitkarini

Sabha) and in 1927 the Samaj Samata

Sangh

Another area of attention for Ambedkar

was education For its spread among

the low classes he set up a network of

colleges by the name of Peoples

Education Society and founded hostels

Q68) Ans(b)

Mehrgarh is a famous Neolithic

settlement in the Indian subcontinent

which is situated in Baluchistan

province Pakistan

A pre-historic rock art site is discovered

in the vast expanse of limestone blocks

on the eastern banks of Naguleru river

near Dachepalli (Andhra Pradesh) It

has thrown light on the Neolithic

civilisation that flourished in Guntur

(Andhra Pradesh) during 1500-2000

BC

Q69) Ans (c)

The 12th and the 13th centuries saw

the emergence of the Kakatiyas They

were at first the feudatories of the

Western Chalukyas of Kalyana Initially

they ruled over a small territory near

Warangal (Telangana)

They introduced Nayakships which was

later adopted and developed by the

Rayas of Vijayanagara

Q70) Ans (a)

The fast had effect of putting pressure

on mill owners who finally agreed to

give the workers a 35 per cent increase

in wages

Google celebrated with a doodle the

132nd birth anniversary of Anasuya

Sarabhai who played a pioneering role

in Indiarsquos labour movement

Q71) Ans (d)

The UNESCOrsquos list of the representative

list of the intangible cultural heritage of

humanity from India are

- Koodiyattam Sanskrit Theatre of

Kerala

- Mudiyettu ritual theatre and dance

drama of Kerala

- Tradition of Vedic Chanting

- Kalbelia folk songs and dances of

Rajasthan

- Ramlila Traditional Performance of

the Ramayana

- Sankirtana ritual singing

drumming and dancing of Manipur

- Ramman religious festival and

ritual theatre of the Garhwal

Himalayas India

- Traditional brass and copper craft of

utensil making among the Thatheras

of Jandiala Guru Punjab India

- Chhau dance classical Indian dance

originated in the eastern Indian

states

- Buddhist chanting of Ladakh

recitation of sacred Buddhist texts

in the trans-Himalayan Ladakh

region Jammu and Kashmir India

- Yoga

- Nouroz

- Kumbh Mela

RAUSIAS-FC19E1003 49

Q72) Ans(b)

The President of India Shri Ram Nath Kovind inaugurated the Hornbill Festival and State Formation Day celebrations of Nagaland in Kisama

The festival is named after the Indian hornbill the large and colourful forest bird which is displayed in the folklore of most of the states tribes

The major recognized tribes of Nagaland are Angami Ao Chakhesang Chang

Kuki Rengma and Zeling etc

Onge Jarawa and Sentinelese are the

tribes of Andman amp Nicobar Islands

Q73) Ans (c)

The Rashtrakutas rule in the Deccan lasted for almost two hundred years till the end of the tenth century The Rashtrakutas rulers were tolerant in their religious views and patronized not only Shaivism and Vaishnavism but

Jainism as well

The famous rock-cut temple of Shiva at Ellora was built by one of the Rashtrakutas kings Krishna I in the ninth century His successor Amoghavarsha was a Jain but he also

patronized other faiths

The Rashtrakutas allowed Muslims traders to settle and permitted Islam to

be preached in their dominions

Recently increasing defacement at the prehistoric rock paintings of Pandavulagutta Telangana has created a cause for grave concern It can spoil

the prehistoric rock

Pandavulagutta is home to

- Painted rock shelters dating to

10000 BC-8000 BC

- An 8th century inscription of the

Rashtrakuta period and

- Painted frescoes from the 12th century Kakatiya empire

Q74) Ans (b)

In 1828 Raja Ram Mohan Roy founded a new religious society the Brahma Sabha later known as the Brahmo

Samaj

Debendranath Tagore headed the Tattvabodhini Sabha which was

engaged in search of spiritual truth

Its purpose was to purify Hinduism and to preach monotheism or belief in one God

The new society was to be based on the twin pillars of reason and the Vedas and

Upanishads

Recently Sadharan Brahmo Samaj (SBS) has entered into a legal battle with the West Bengal government due

to some legal issue

Q75) Ans (c)

The Chishti order was established in India by Khwaja Moinuddin Chishti who came to India around 1192 The Chishtirsquos are considered to be the most influential of the groups of Sufis who migrated to India in the late twelfth century They adapted successfully to the local environment and adopted several features of Indian devotional

traditions

The historical dargah of Sufi mystic Khwaja Moinuddin Chishti in Ajmer is all set to get a facelift This 13 th century dargah has been included among the Swachh Iconic Places a clean-up initiative focused on iconic

heritage spiritual and cultural places

Page 48: GENERAL STUDIES (PAPER I) · Test is part of Rau’s IAS Test series for Preliminary Exam 2019 FOUNDATION + CURRENT AFFAIRS GENERAL STUDIES (PAPER –I) FOUNDATION TEST –III TOPIC:

RAUSIAS-FC19E1003 48

The Prachi river originates from

Bhubaneswar

It is a tributary of the Mahanadi and

flows through the districts of Puri

Khurda Cuttack and Jagatsinghpur

and the entire region of the river is

termed as the Prachi Valley

It falls into the Bay of Bengal

Archaeological evidence shows that the

Prachi Valley Civilisation predates both

Harappa and Mohenjo-Daro

The Prachi river originates from

Bhubaneswar

Q66) Ans (d)

These monuments are located in

Chhatarpur district Madhya Pradesh

within Vindhya mountain range

Q67) Ans (a)

The book lsquoThoughts on Pakistanrsquo was

written by Dr BR Ambedkar

On the occasion of the birth anniversary

of Dr BR Ambedkar the president of

India pays homage to this icon of India

In 1924 he founded the Depressed

Classes Institute (Bahishkrit Hitkarini

Sabha) and in 1927 the Samaj Samata

Sangh

Another area of attention for Ambedkar

was education For its spread among

the low classes he set up a network of

colleges by the name of Peoples

Education Society and founded hostels

Q68) Ans(b)

Mehrgarh is a famous Neolithic

settlement in the Indian subcontinent

which is situated in Baluchistan

province Pakistan

A pre-historic rock art site is discovered

in the vast expanse of limestone blocks

on the eastern banks of Naguleru river

near Dachepalli (Andhra Pradesh) It

has thrown light on the Neolithic

civilisation that flourished in Guntur

(Andhra Pradesh) during 1500-2000

BC

Q69) Ans (c)

The 12th and the 13th centuries saw

the emergence of the Kakatiyas They

were at first the feudatories of the

Western Chalukyas of Kalyana Initially

they ruled over a small territory near

Warangal (Telangana)

They introduced Nayakships which was

later adopted and developed by the

Rayas of Vijayanagara

Q70) Ans (a)

The fast had effect of putting pressure

on mill owners who finally agreed to

give the workers a 35 per cent increase

in wages

Google celebrated with a doodle the

132nd birth anniversary of Anasuya

Sarabhai who played a pioneering role

in Indiarsquos labour movement

Q71) Ans (d)

The UNESCOrsquos list of the representative

list of the intangible cultural heritage of

humanity from India are

- Koodiyattam Sanskrit Theatre of

Kerala

- Mudiyettu ritual theatre and dance

drama of Kerala

- Tradition of Vedic Chanting

- Kalbelia folk songs and dances of

Rajasthan

- Ramlila Traditional Performance of

the Ramayana

- Sankirtana ritual singing

drumming and dancing of Manipur

- Ramman religious festival and

ritual theatre of the Garhwal

Himalayas India

- Traditional brass and copper craft of

utensil making among the Thatheras

of Jandiala Guru Punjab India

- Chhau dance classical Indian dance

originated in the eastern Indian

states

- Buddhist chanting of Ladakh

recitation of sacred Buddhist texts

in the trans-Himalayan Ladakh

region Jammu and Kashmir India

- Yoga

- Nouroz

- Kumbh Mela

RAUSIAS-FC19E1003 49

Q72) Ans(b)

The President of India Shri Ram Nath Kovind inaugurated the Hornbill Festival and State Formation Day celebrations of Nagaland in Kisama

The festival is named after the Indian hornbill the large and colourful forest bird which is displayed in the folklore of most of the states tribes

The major recognized tribes of Nagaland are Angami Ao Chakhesang Chang

Kuki Rengma and Zeling etc

Onge Jarawa and Sentinelese are the

tribes of Andman amp Nicobar Islands

Q73) Ans (c)

The Rashtrakutas rule in the Deccan lasted for almost two hundred years till the end of the tenth century The Rashtrakutas rulers were tolerant in their religious views and patronized not only Shaivism and Vaishnavism but

Jainism as well

The famous rock-cut temple of Shiva at Ellora was built by one of the Rashtrakutas kings Krishna I in the ninth century His successor Amoghavarsha was a Jain but he also

patronized other faiths

The Rashtrakutas allowed Muslims traders to settle and permitted Islam to

be preached in their dominions

Recently increasing defacement at the prehistoric rock paintings of Pandavulagutta Telangana has created a cause for grave concern It can spoil

the prehistoric rock

Pandavulagutta is home to

- Painted rock shelters dating to

10000 BC-8000 BC

- An 8th century inscription of the

Rashtrakuta period and

- Painted frescoes from the 12th century Kakatiya empire

Q74) Ans (b)

In 1828 Raja Ram Mohan Roy founded a new religious society the Brahma Sabha later known as the Brahmo

Samaj

Debendranath Tagore headed the Tattvabodhini Sabha which was

engaged in search of spiritual truth

Its purpose was to purify Hinduism and to preach monotheism or belief in one God

The new society was to be based on the twin pillars of reason and the Vedas and

Upanishads

Recently Sadharan Brahmo Samaj (SBS) has entered into a legal battle with the West Bengal government due

to some legal issue

Q75) Ans (c)

The Chishti order was established in India by Khwaja Moinuddin Chishti who came to India around 1192 The Chishtirsquos are considered to be the most influential of the groups of Sufis who migrated to India in the late twelfth century They adapted successfully to the local environment and adopted several features of Indian devotional

traditions

The historical dargah of Sufi mystic Khwaja Moinuddin Chishti in Ajmer is all set to get a facelift This 13 th century dargah has been included among the Swachh Iconic Places a clean-up initiative focused on iconic

heritage spiritual and cultural places

Page 49: GENERAL STUDIES (PAPER I) · Test is part of Rau’s IAS Test series for Preliminary Exam 2019 FOUNDATION + CURRENT AFFAIRS GENERAL STUDIES (PAPER –I) FOUNDATION TEST –III TOPIC:

RAUSIAS-FC19E1003 49

Q72) Ans(b)

The President of India Shri Ram Nath Kovind inaugurated the Hornbill Festival and State Formation Day celebrations of Nagaland in Kisama

The festival is named after the Indian hornbill the large and colourful forest bird which is displayed in the folklore of most of the states tribes

The major recognized tribes of Nagaland are Angami Ao Chakhesang Chang

Kuki Rengma and Zeling etc

Onge Jarawa and Sentinelese are the

tribes of Andman amp Nicobar Islands

Q73) Ans (c)

The Rashtrakutas rule in the Deccan lasted for almost two hundred years till the end of the tenth century The Rashtrakutas rulers were tolerant in their religious views and patronized not only Shaivism and Vaishnavism but

Jainism as well

The famous rock-cut temple of Shiva at Ellora was built by one of the Rashtrakutas kings Krishna I in the ninth century His successor Amoghavarsha was a Jain but he also

patronized other faiths

The Rashtrakutas allowed Muslims traders to settle and permitted Islam to

be preached in their dominions

Recently increasing defacement at the prehistoric rock paintings of Pandavulagutta Telangana has created a cause for grave concern It can spoil

the prehistoric rock

Pandavulagutta is home to

- Painted rock shelters dating to

10000 BC-8000 BC

- An 8th century inscription of the

Rashtrakuta period and

- Painted frescoes from the 12th century Kakatiya empire

Q74) Ans (b)

In 1828 Raja Ram Mohan Roy founded a new religious society the Brahma Sabha later known as the Brahmo

Samaj

Debendranath Tagore headed the Tattvabodhini Sabha which was

engaged in search of spiritual truth

Its purpose was to purify Hinduism and to preach monotheism or belief in one God

The new society was to be based on the twin pillars of reason and the Vedas and

Upanishads

Recently Sadharan Brahmo Samaj (SBS) has entered into a legal battle with the West Bengal government due

to some legal issue

Q75) Ans (c)

The Chishti order was established in India by Khwaja Moinuddin Chishti who came to India around 1192 The Chishtirsquos are considered to be the most influential of the groups of Sufis who migrated to India in the late twelfth century They adapted successfully to the local environment and adopted several features of Indian devotional

traditions

The historical dargah of Sufi mystic Khwaja Moinuddin Chishti in Ajmer is all set to get a facelift This 13 th century dargah has been included among the Swachh Iconic Places a clean-up initiative focused on iconic

heritage spiritual and cultural places